You are on page 1of 2425

EUREKA!

1-37
Matemtica Olmpica Compilao Omegaleph da

Revista Eureka de 1998 a 2013


03 de Janeiro de 2014

CONTEDO
APRESENTAO A NOVA OLIMPADA BRASILEIRA DE MATEMTICA Introduo OLIMPADA BRASILEIRA DE MATEMTICA Problemas de treinamento OLIMPADA BRASILEIRA DE MATEMTICA Problemas das provas das primeiras fases Jnior e Snior 1997 A OLIMPADA DE MAIO Introduo III OLIMPADA DE MAIO Primeiro nvel III OLIMPADA DE MAIO Segundo nvel 9a. OLIMPADA DO CONE SUL Introduo 8a. OLIMPADA DO CONE SUL Problemas 2 5 7 12 22 23 29 35 36

ARTIGOS

NMEROS MGICOS E CONTAS DE DIVIDIR Carlos Gustavo Tamm de Arajo Moreira COMO PERDER AMIGOS E ENGANAR PESSOAS Nicolau C. Saldanha DOIS PROBLEMAS SOBRE GRAFOS Paulo Cezar Pinto Carvalho

38 41 51 58 60 61

PROBLEMAS PROPOSTOS AGENDA OLMPICA COORDENADORES REGIONAIS

Sociedade Brasileira de Matemtica

APRESENTAO
EUREKA!, a revista da Olimpada Brasileira de Matemtica faz parte de um grande projeto que tem como objetivo principal contribuir decisivamente para a melhoria de ensino de Matemtica em nosso pas. O que planejamos realizar descrito (de forma resumida), nesta apresentao. DOS OBJETIVOS O ensino de Matemtica hoje no Brasil difere pouco do ensino praticado h 20 anos. A cada ano, livros novos so editados repetindo quase sempre o mesmo estilo e os mesmos contedos dos anteriores. Existem hoje no Brasil bons livros de Matemtica dedicados aos alunos tanto do ensino fundamental quanto do ensino mdio. Entretanto, o que lhes falta um ingrediente que, no mundo de hoje, fundamental: o estmulo criatividade. Entendemos que no suficiente para a formao do futuro cidado um aprendizado burocrtico da Matemtica e percebemos a importncia de estimular os alunos desde tenra idade a resolver problemas novos e desafiantes, propiciando o desenvolvimento da imaginao e da criatividade. O programa de Olimpadas de Matemtica reconhecido em todos os pases do mundo desenvolvido como o mais eficiente instrumento para atingir esse objetivo. Aproveitando o natural gosto dos jovens pelas competies, as Olimpadas de Matemtica tm conseguido estimular alunos a estudar contedos alm do currculo escolar e, tambm, por outro lado, aumentar e desenvolver a competncia dos professores. DO PROJETO O programa de Olimpadas de Matemtica existe no pas h 19 anos. Sempre foi pequeno e dedicado a encontrar jovens talentos para a Matemtica ou para cincias afins e, neste aspecto, cumpriu sua finalidade. Temos hoje brilhantes matemticos e cientistas de renome mundial que tiveram origem nas Olimpadas de Matemtica. Entretanto, reconhecemos que, com esta atividade, pode-se fazer muito mais. Com parceria do IMPA
EUREKA! N 1, 1998

Sociedade Brasileira de Matemtica

(Instituto de Matemtica Pura e Aplicada) e com a SBM (Sociedade Brasileira de Matemtica), foi submetido ao CNPq um projeto que pretende contribuir para a melhoria do ensino de Matemtica no Brasil utilizando as Olimpadas de Matemtica como mecanismo propagador. Este projeto teve boa acolhida e neste momento estamos iniciando um trabalho de grandes dimenses que, para ter seus objetivos cumpridos, necessitar tambm (e principalmente) do apoio e da ajuda de diversos segmentos da sociedade: alunos, professores, escolas, universidades, secretarias de educao etc. Nossa atividade estar centrada na resoluo de problemas e atingir alunos desde a 5a. srie do ensino fundamental at a 3a. srie do ensino mdio e, naturalmente, seus professores. Para a divulgao deste material, utilizaremos esta revista, cartazes mensais com diversas informaes sobre atividades olmpicas e um site na Internet. Para movimentar os jovens realizar-se- anualmente uma nova Olimpada Brasileira de Matemtica, que estar dividida em nveis de acordo com a escolaridade do aluno. Alm disso, estaremos apoiando a realizao de competies de Matemtica em nvel regional. Para os professores, esto sendo planejados cursos de aperfeioamento em diversas regies do pas, tambm colocaremos disposio, atravs do site da Internet, um vasto banco de problemas e uma biblioteca especializada localizados na nossa sede no IMPA. DA REVISTA EUREKA!, a revista da Olimpada de Matemtica uma publicao dedicada principalmente aos alunos e professores da escola secundria a qual ser editada quatro vezes ao ano e ter basicamente a seguinte estrutura: a) Seo de problemas de treinamento com solues, dividida, em trs nveis: para os alunos de 5a. e 6a. sries, para os alunos de 7a. e 8a. sries e para os alunos de ensino mdio. Esta seo pretende fornecer aos alunos material para estudo e pesquisa dirigidos Olimpada Brasileira, que ser realizada nesses mesmos trs nveis.

EUREKA! N 1, 1998

Sociedade Brasileira de Matemtica

b)

Seo de artigos de Matemtica elementar, tratando de assuntos que complementem o currculo escolar e que tambm abordem novos contedos. Estes artigos estaro classificados em iniciante, intermedirio ou avanado, de acordo com o estgio de desenvolvimento dos leitores aos quais se destinem os artigos. Seo de Problemas de diversos nveis, sem soluo, para que os leitores possam pesquisar e enviar suas solues para a revista, sendo as melhores publicadas nos nmeros seguintes. Seo de Cartas dos Leitores, em que alunos e professores tero possibilidade de fazer quaisquer perguntas. Todas as cartas sero respondidas e as mais relevantes sero publicadas. Agenda, para informarmos todas as atividades ligadas s Olimpadas de Matemtica no Brasil e no exterior.

c)

d)

e)

DOS CARTAZES Para que nossa atividade permanea viva durante o ano, enviaremos todos os meses para as escolas cadastradas um cartaz da Olimpada Brasileira de Matemtica. Esse cartaz conter todas as informaes sobre as atividades olmpicas e tambm o Problema do Ms, em cada um dos trs nveis. Contamos com que muitos alunos fiquem interessados nesse desafio e que nos enviem solues. Todos os colgios cadastrados recebero gratuitamente a revista EUREKA! e os cartazes mensais. Para cadastrar um colgio, basta entrar em contato conosco dando o nome do colgio, endereo completo e o nome de um professor responsvel para receber a correspondncia.

Rio de Janeiro, abril de 1998

EUREKA! N 1, 1998

Sociedade Brasileira de Matemtica

A NOVA OLIMPADA BRASILEIRA DE MATEMTICA


A Olimpada Brasileira de Matemtica ser realizada a partir deste ano de 1998 de forma bastante diferente da que vinha sendo praticada nos ltimos anos. Isto porque agora passa a atingir os alunos desde a 5a. srie do ensino fundamental. Antes, a Olimpada Brasileira de Matemtica era principalmente um instrumento para detectar talentos e desenvolv-los, mas, agora, tem tambm por objetivo promover em mbito nacional a melhoria do ensino de Matemtica nas escolas, com o desenvolvimento conjunto de alunos e professores. A Olimpada Brasileira de Matemtica, a partir deste ano, no ser apenas uma competio. Para a preparao dos alunos e para o aperfeioamento dos professores, a OBM distribuir aos colgios revistas e cartazes contendo farto material para estudo e pesquisa, dedicados a cada faixa de escolaridade e desenvolvimento dos alunos. A realizao das provas uma finalizao (sempre parcial) dessa atividade. A Olimpada Brasileira de Matemtica ser realizada em trs fases e em trs nveis. So eles: Nvel 1 - para alunos da 5. e 6. sries do ensino fundamental. Nvel 2 - para alunos da 7. e 8. sries do ensino fundamental. Nvel 3 - para alunos do ensino mdio (antigo 2. grau). Para cada um dos nveis, a OBM ter trs fases. Na primeira, qualquer aluno interessado poder participar. Para participar das outras, existir um critrio de promoo. A prova da primeira fase ser de mltipla escolha, contendo de 20 a 25 questes sobre contedo adequado a cada um dos nveis de escolaridade. Nestas questes, sero includas algumas que dependam de alguma criatividade, porm no fugindo dos contedos tradicionais das escolas. A prova da segunda fase ser discursiva e constar de 6 problemas, exigindo uma maior dose de iniciativa e criatividade. A prova da terceira fase ser tambm discursiva. As provas da primeira e segunda fases da OBM sero realizadas nas escolas que desejarem participar dessa atividade. A correo

EUREKA! N 1, 1998

Sociedade Brasileira de Matemtica

das provas tambm ser realizada nas escolas, com o salutar envolvimento de seus professores, de acordo com critrios determinados pela organizao. Os coordenadores oferecero locais alternativos aos alunos que desejarem participar da Olimpada, caso o colgio onde realizam seus estudos no venha a organizar a atividade. A prova da terceira fase ser realizada em um local central designado pelo coordenador local e corrigida pelo comit organizador da OBM.

Para tornar vivel a realizao de uma competio de Matemtica em mbito nacional, foi criada uma estrutura operativa. As atividades de elaborao das provas, edio da revista, publicao dos cartazes etc. sero centralizadas na Secretaria da Olimpada Brasileira de Matemtica, localizada no IMPA (Rio de Janeiro). Para apoiar as atividades no pas, existem hoje cerca de 30 coordenadores regionais que daro assistncia s escolas de sua rea de atuao. Cada colgio participante da OBM ficar, portanto, ligado ao coordenador regional mais prximo, que fornecer toda a assistncia necessria. Em 1998, a Olimpada Brasileira de Matemtica ser realizada nas seguintes datas: Primeira fase: Segunda fase: Terceira fase: Sbado, 6 de junho Sbado, 12 de setembro Sbado, 24 de outubro (nveis 1, 2 e 3) e Domingo, 25 de outubro (nvel 3)

A Olimpada Brasileira de Matemtica no de forma alguma uma competio entre colgios. Ela pretende essencialmente despertar nos alunos o gosto pelo estudo da Matemtica atravs da resoluo de problemas novos, estimulando o desenvolvimento da imaginao e da criatividade. O aspecto da competio naturalmente existe, mas jamais estar ligado a grupos, equipes, colgios, cidades ou regies. Desejamos deixar bem claro que uma medalha oferecida pela Olimpada Brasileira de Matemtica um reconhecimento ao esforo individual do aluno premiado, mas representa tambm o coroamento de um trabalho em que centenas ou milhares de annimos alunos tambm se desenvolveram. E isto, no fundo, o que importa.

EUREKA! N 1, 1998

Sociedade Brasileira de Matemtica

OLIMPADA BRASILEIRA DE MATEMTICA


Problemas de treinamento para a Primeira fase Primeiro nvel 1) Num quadrado formado por 9 quadrados menores e do mesmo tamanho, queremos escrever um X e um O, de forma que eles no fiquem vizinhos, isto , os quadrados em que se encontram no podem ter um lado ou um vrtice comum. O desenho abaixo mostra uma dessas possibilidades:

De quantas maneiras podemos localizar os dois sinais, respeitadas as condies apresentadas? a) 32 2) b) 20 c) 64 d) 18 e) 12

Jacira consegue datilografar 20 pginas de um manuscrito em 4 horas e Joana o faz em 5 horas. Ainda restam 900 pginas do manuscrito para datilografar. Se as duas comearem a datilografar no mesmo instante essas pginas, quantas pginas dever pegar a mais lenta, de forma que ambas terminem juntas? a) 225 b) 500 c) 400 d) 450 e) 180

3)

O professor Epaminondas, no primeiro dia de aula, apostou que, entre os alunos daquela classe, pelo menos dois fariam aniversrio no mesmo dia do ms. O professor tinha certeza de que ganharia a aposta, pois naquela classe o nmero de alunos era maior ou igual a: a) 15 b) 32 c) 28 d) 31 e) 30

EUREKA! N 1, 1998

Sociedade Brasileira de Matemtica

4)

Seu Pedro possui trs lotes quadrados: um deles tem lado de 10 metros, e os outros dois tm lados de 20 metros cada. Seu Pedro quer trocar os trs lotes por um outro lote quadrado, cuja rea seja a soma das reas daqueles trs lotes. O novo lote dever ter lado de medida: a) impossvel de obter b) 24 metros d) 40 metros e) 30 metros c) 25 metros

5)

Um jogo consiste em partir da casa 1 casa 36 numa trilha com casas numeradas de 1 a 36. Os dois jogadores comeam na casa 1 e o avano de casas depende do lanamento de dois dados cbicos comuns. Se a soma dos pontos for par, o jogador avana 3 casas. Se a soma dos pontos for mpar, o jogador avana 1 casa. Se o jogador ultrapassar a ltima casa, retorna casa 1. A ordem com que os jogadores iniciam suas jogadas definida por alguma forma de sorteio. Ganha quem parar primeiro na casa 36. O menor nmero de jogadas que algum pode fazer e ganhar a) 37 b) 13 c) 12 d) 14 e) 17

Segundo nvel 1) A equao do 2. grau ax2 + bx 3 = 0 tem 1 como uma de suas razes. Sabendo que os coeficientes a e b so nmeros primos positivos, podemos afirmar que a2 + b2 igual a: a) 29 b) 89 c) 17 d) 13 e) 53

2)

Voc j conhece o quadrado mgico de ordem 3: a soma dos nmeros das linhas, das colunas e das diagonais 15. A figura a se-

EUREKA! N 1, 1998

Sociedade Brasileira de Matemtica

guir mostra uma das oito possibilidades de escrever os nmeros no quadrado:

O nico nmero que no pode mudar de posio em todos esses quadrados mgicos : a) 1 3) b) 3 c) 5 d) 7 e) 9

No modo SP, o aparelho de videocassete grava exatamente duas horas e, no modo EP, grava quatro horas de filme, com menor qualidade. Carlinhos quer gravar um filme de 136 minutos, com a melhor qualidade possvel. Ele decidiu comear no modo EP e terminar no modo SP. Aps quantos minutos de gravao no modo EP ele deve passar ao modo SP ? a) 20 b) 16 c) 8 d) 32 e) 68

4)

Os pontos A, B e C so vrtices de um tringulo cujos lados medem 3, 4 e 5 cm e pertencem ao interior de uma circunferncia, da qual esto a uma distncia de 1 cm. O raio da circunferncia, em centmetros, : a) 5 b) 7 c) 2,5 d) 4,2 e) 3,5

5)

Um nmero de dois algarismos no nulos igual ao dobro do produto desses algarismos. Esse nmero pertence ao conjunto: a){11,12,..., 30} c){51,52,..., 70} e){91,92,..., 99} b){31,32,..., 50} d){71,72,..., 90}

EUREKA! N 1, 1998

Sociedade Brasileira de Matemtica

Terceiro nvel 1) Considere trs circunferncias concntricas ( mesmo centro T ) de raios 1, 2 e 3, respectivamente. Considere um tringulo cujos vrtices pertencem, um a cada uma das circunferncias. Sabendo que o tringulo tem rea mxima sob essas condices, podemos afirmar que, para este tringulo, o ponto T o: a) baricentro b) incentro e) ex-incentro 2) c) circuncentro d) ortocentro

Dada a funo f: Z Z ( Z o conjunto dos nmeros inteiros) definida por x 1 se x mpar e x + 1 se x par, podemos afirmar que o nmero de solues da equao f(x) = f(2x) : a) 1 b) 2 c) 3 d) 4 e) 0

f(x) =

3)

Na seqncia de inteiros positivos a1, a2,..., ak,, para 1 i k,o termo ai o i-simo mpar positivo; para i > k, o termo ai a mdia aritmtica dos termos anteriores. Podemos concluir que a2k igual a: a) k2 b)k c)2k d) 0 e)

4)

Os vrtices de um tringulo tm coordenadas (0,0), (3,1) e (1,7), respectivamente. As retas que passam pelos vrtices e por um ponto T no interior do tringulo dividem-no em 6 tringulos de mesma rea. Ento: a) T = (3,6) d) T = (2,8) b) T = (2,4) e) T = (2/3, 4/3) c) T = (4/3, 8/3)

EUREKA! N 1, 1998

10

Sociedade Brasileira de Matemtica

5)

Para quantos valores reais de p a equao x3 px2 + px 1 = 0 tem todas as razes reais e inteiras ? a) 1 b) 2 c) 3 d) 4 e) 5 ou mais

6)

Considere o conjunto P dos pontos (x,y) do R2 tais que x e y sejam inteiros. Por exemplo, (1,1) P. Tome agora uma circunferncia de dimetro igual a 5, de forma que em seu interior haja o maior nmero possvel de pontos de P. Esse nmero : a) 10 b) 16 c) 20 d) 14 e) 21

Nota: Veja as respostas na pgina 21.

Voc sabia que os antigos egpcios no usavam fraes com numerador maior que 1 nem somavam fraes iguais de numerador 1 ? Assim por exemplo, eles se referiam ao nmero 2/5 como 1/3 + 1/15.
Veja o problema 9 na pgina 60.


EUREKA! N 1, 1998

11

Sociedade Brasileira de Matemtica

OLIMPADA BRASILEIRA DE MATEMTICA


Provas Jnior e Snior 1997 At o ano passado a Olimpada Brasileira de Matemtica era realizada em apenas dois nveis. PRIMEIRA FASE JNIOR
PROBLEMA 1

O nmero N tem trs algarismos. O produto dos algarismos de N 126 e a soma dos dois ltimos algarismos de N 11. O algarismo das centenas de N : a)2 b) 3 c)6 d)7 e)9

PROBLEMA 2

A fortuna de Joo foi dividida da seguinte forma: um quinto para seu irmo mais velho, um sexto do restante para seu irmo mais novo e partes iguais do restante para cada um de seus 12 filhos. Que frao da fortuna cada filho recebeu? a)

1 20

b)

1 18

c)

1 16

d)

1 15

e)

1 14

PROBLEMA 3

No alvo abaixo, uma certa pontuao dada para a flecha que cai na regio A e outra para a flecha que cai na regio B. Alberto lanou 3 flechas: uma caiu em B e duas em A, e obteve 17 pontos. Carlos tambm lanou 3 flechas: uma caiu em A e duas em B, e obteve 22 pontos. Quantos pontos so atribudos para uma flecha que cai na regio A? a) 2 b) 3 c) 4 d) 5
A B

e) 6

EUREKA! N 1, 1998

12

Sociedade Brasileira de Matemtica

PROBLEMA 4

Seja f uma funo definida para todo x real, satisfazendo as condies:

f(3) = 2 f(x + 3) = f(x) f(3)


Ento, f(3) vale: a)6 b)0 c)

1 2

d)2

e)1

PROBLEMA 5

Quatro carros, de cores amarela, verde, azul e preta, esto em fila. Sabe-se que o carro que est imediatamente antes do carro azul menor do que o que est imediatamente depois do carro azul; que o carro verde o menor de todos; que o carro verde est depois do carro azul; e que o carro amarelo est depois do preto. O primeiro carro da fila: a) amarelo. b) azul. c) preto. d) verde. e) no pode ser determinado apenas com esses dados. OBS: O primeiro da fila o que vem antes de todos os outros.

PROBLEMA 6

64 jogadores de habilidades diferentes disputam um torneio de tnis. Na primeira rodada, so feitos 32 jogos (os emparelhamentos so por sorteio), e os perdedores so eliminados. Na segunda rodada, so feitos 16 jogos, os perdedores so eliminados, e assim por diante. Se os emparelhamentos so feitos por sorteio e no h surpresas ( se A melhor que B, A vence B), qual o nmero mximo de jogos que o dcimo melhor jogador consegue jogar? a)2 b)3 c)4 d)5 e)6

EUREKA! N 1, 1998

13

Sociedade Brasileira de Matemtica

PROBLEMA 7

O nmero de pares (x, y) de reais que satisfazem o sistema de equaes

2 2 x xy y + 1 = 0 3 2 2 x x y xy + x y + 2 = 0
a)0 b)1 c)2 d)3 e)4

igual a:

PROBLEMA 8

Seja y = x + 2 + x 1 + x 3 . Se 1 x < 2, ento y igual a: a)x + 4 b)3x 2 c)x 4 d)3x + 2 e)x 2

PROBLEMA 9

Um gramado tem a forma de um quadrado com 10m de lado. Uma corda tem um dos extremos fixado em um dos vrtices, e no outro extremo est amarrado um bode. Se o bode consegue comer metade da grama, ento o comprimento da corda de aproximadamente: a)8m b)7,5m c)7m d)6,5m e)6m

PROBLEMA 10

Se p e q so inteiros positivos tais que pode ter : a)6 b)7 c)25 d)30 e)60

7 10

<

p q

<

11 15

, o menor valor que q

EUREKA! N 1, 1998

14

Sociedade Brasileira de Matemtica

PROBLEMA 11

A equao

x + 10 2 x + 3 = 1 3 x :

a) no tem soluo. b) tem uma nica soluo positiva. c) tem uma nica soluo negativa. d) tem duas solues, uma positiva e outra negativa. e) tem duas solues, ambas negativas.

PROBLEMA 12

Como o mdico me recomendou caminhadas, todo dia de manh dou uma volta (com velocidade constante) na quadra em que resido. Minha mulher aproveita para correr (com velocidade constante) em volta do quarteiro. Samos juntos e chegamos juntos. Ela percorre a quadra no mesmo sentido que eu e me ultrapassa duas vezes durante o percurso. Se ela corresse no sentido contrrio ao meu, quantas vezes ela cruzaria comigo? a)2 b)3 c)4 d)5 e)6

PROBLEMA 13

Em uma urna h 28 bolas azuis, 20 bolas verdes, 12 bolas amarelas, 10 bolas pretas e 8 bolas brancas. Qual o nmero mnimo de bolas que devemos sacar dessa urna para termos certeza de que sacaremos pelo menos 15 bolas da mesma cor? a)58 b)59 c)60 d)71 e)72

PROBLEMA 14

Um ladrilho, em forma de polgono regular, foi retirado do lugar que ocupava em um painel. Observou-se, ento, que esse ladrilho, se sofresse uma rotao de 40o ou de 600 em torno de seu centro, poderia ser encaixado perEUREKA! N 1, 1998

15

Sociedade Brasileira de Matemtica

feitamente no lugar que ficou vago no painel. O menor nmero de lados que pode ter esse ladrilho : a)6 b)9 c)12 d)15 e)18

PROBLEMA 15

No tringulo retngulo ABC da figura abaixo, est inscrito um quadrado. Se AB = 20 e AC = 5, que porcentagem a rea do quadrado representa da rea do tringulo ABC? a) 25% b) 30% c) 32% d) 36% e) 40%
PROBLEMA 16 C

Em certo pas, a unidade monetria o pau. H notas de 1 pau e moedas de meio pau, um tero de pau, um quarto de pau e um quinto de pau. Qual a maior quantia, em paus, que um cidado pode ter em moedas sem que possa juntar algumas delas para formar exatamente um pau? a)

11 12

b) 1

5 12

c) 2

7 15

d) 2

13 60

e) 2

43 60

PROBLEMA 17

Joo e Pedro so vendedores e ganham R$ 1000,00 de salrio e comisso de 8% sobre as vendas. Em setembro, Joo ganhou R$ 2000,00 e Pedro ganhou R$ 2 500,00. Nesse ms, as vendas de Pedro superaram as de Joo em: a) 20% b) 25% c) 30% d) 40% e) 50%

EUREKA! N 1, 1998

16

Sociedade Brasileira de Matemtica

PROBLEMA 18

Um tringulo ABC, de lados AB = c, AC = b e BC = a, tem permetro 2p. Uma circunferncia tangencia o lado BC e os prolongamentos dos lados AB e AC nos pontos P, Q e R, respectivamente. O comprimento AR igual a: a)p a
PROBLEMA 19

b)p b

c)p c

d)p

e) 2p

P um ponto interior a um quadrado ABCD. As distncias de P aos vrtices A e D e ao lado BC so iguais a 10cm. O lado do quadrado mede: a)10cm
PROBLEMA 20

b)12cm

c)14cm

d)16cm

e)18cm

A figura ao lado mostra trs dados iguais. O nmero da face que a base inferior da coluna de dados: a) 1. b) 2. c) 4. d) 6. e) pode ser 1 ou 4. PRIMEIRA FASE SNIOR (*) (*) Na prova da primeira fase snior apareceram os problemas 5, 6, 7, 10, 12, 13, 14, 16, 17, 18, 19 e 20 da primeira fase jnior.
PROBLEMA 1

Quantos so os pares no-ordenados de inteiros positivos tais que, em cada par, a soma do produto dos nmeros do par com a soma dos nmeros do par com o mdulo da diferena dos nmeros do par seja igual a 20? a)1 b)2 c)3 d)4 e)5

EUREKA! N 1, 1998

17

Sociedade Brasileira de Matemtica

PROBLEMA 2

O nmero de pares (x, y) de inteiros que satisfazem a equao x + y + xy = 120 : a)1 b)2 c)3 d)4 e)6

PROBLEMA 3

O conjunto-soluo da inequao

a) dos reais diferentes de 0 e de 1. b) dos reais positivos diferentes de 1. c) dos reais diferentes de zero e menores que 1. d) dos reais entre 0 e 1. e) vazio.
PROBLEMA 4

1 1 o conjunto: > x x1

O nmero de valores inteiros de m para os quais as razes da equao x2 (m + m2)x + m3 1 = 0 so inteiras igual a: a)0 b)1 c)2 d)3 e)4

PROBLEMA 5

Os vrtices de um decgono regular convexo ABC...J devem ser coloridos usando-se apenas as cores verde, amarela e azul. De quantos modos isso pode ser feito se vrtices adjacentes no podem receber a mesma cor? a)1022
PROBLEMA 6

b)1024

c)1026

d)1524

e)1536

Uma das solues inteiras e positivas da equao 19x + 97y = 1997 , evidentemente, (x0, y0) = (100,1). Alm desse, h apenas mais um par de nmeros inteiros e positivos, (x1, y1), satisfazendo a equao. O valor de x1+y1 : a)23 b)52 c)54 d)101 e)1997

EUREKA! N 1, 1998

18

Sociedade Brasileira de Matemtica

PROBLEMA 7

Selecionam-se 3 vrtices de um cubo. Qual a probabilidade de eles pertencerem a uma mesma face? 1 1 1 2 3 b) c) d) e) a) 5 6 7 7 7
PROBLEMA 8

Sendo k inteiro, o nmero de valores distintos de sen a)5 b)8 c)9 d)10 e)18

igual a:

PROBLEMA 9

Para cobrir um terrao em forma de um retngulo ABCD, usa-se uma placa plana de alumnio apoiada em quatro estacas verticais fixadas nos vrtices do retngulo. A placa fica inclinada em relao ao cho para escoar a gua das chuvas. Se as estacas que partem dos vrtices A, B e C tm comprimentos respectivamente iguais a 3, 4 e 5 metros, o comprimento da que parte de D : a)3m
PROBLEMA 10

b)4m

c)5m

d)6m

e)8m

Se seu salrio sobe 26% e os preos sobem 20%, de quanto aumenta o seu poder aquisitivo? a)5% b)6% c)7% d)8% e)9%
PROBLEMA 11

O reservatrio de um caminho-tanque tem a forma de um cilindro de re3 voluo com eixo horizontal e est cheio at da altura. A frao da ca4 pacidade total do resevatrio que est ocupada de aproximadamente: a)80% b)75% c)68% d)60% e)56%

EUREKA! N 1, 1998

19

Sociedade Brasileira de Matemtica

PROBLEMA 12

O preo de um estacionamento formado por um valor fixo para as duas primeiras horas e um adicional por cada hora subseqente. Se o estacionamento por 3 horas custa R$ 5,00 e por 5 horas custa R$ 6,00, quanto custa o estacionamento por 8 horas? a)R$ 7,00 e)R$ 13,33
PROBLEMA 13

b)R$ 7,50

c)R$ 9,60

d)R$ 12,00

O nmero de solues reais da equao x2 = 2x : a)0 b)1 c)2 d)3 e)4

Voc sabia que aproximadamente:

3,14159265358979323846264338327950288419716939937510582097494459230781 640628620899862803482534211706798214808651328230664709384460955058223 17253594081284811174502841027019385211055596446229489549303819644288109 756659334461284756482337867831652712019091456485669234603486104543266 482133936072602491412737245870066?

EUREKA! N 1, 1998

20

Sociedade Brasileira de Matemtica

Respostas dos problemas de treinamento Primeira fase da Olimpada Brasileira de Matemtica.


Primeiro nvel

1) a
Segundo nvel

2) c 2) c 2) a 3) b

3) b 3) d 4) c

4) e 4) e 5) b

5) b 5) b 6) e

1) a
Terceiro nvel

1) d

RESPOSTAS DA PRIMEIRA FASE -OBM JNIOR- 1997 1) d 2) b 3) c 4) c 5) c 6) e 7) c 8) a 9) a 10) b 11) e 12) c 13) b 14) e 15) c 16) d 17) e 18) d 19) d 20) c

RESPOSTAS DA PRIMEIRA FASE -OBM SNIOR- 1997 1) b 2) e 3) d 4) c 5) c 6) a 7) e 8) c 9) b 10) a 11) a 12) b 13) d

de por 355/113 menor que 310-7?

Voc sabia que o erro da aproximao

EUREKA! N 1, 1998

21

Sociedade Brasileira de Matemtica

A OLIMPADA DE MAIO
Introduo A Federao Iberoamericana de Competies de Matemtica organizou pela primeira vez a Olimpada de Maio no ano de 1995. A competio est dividida em dois nveis: estudantes menores de 13 anos e estudantes menores de 15 anos. O concurso se realiza por correspondncia e est baseado no modelo que segue a Olimpada de Matemtica do Pacfico (APMO), concurso de longa distncia com grande tradio. En maio deste ano se realizar a IV Olimpada de maio, seguindo o calendrio seguinte: Limite para o envio dos problemas. Envio dos enunciados das provas aos delegados de cada pas: Prova: Limite da chegada dos listados e provas para cada pas: Envio dos resultados e diplomas de honra aos delegados de cada pas: 31 de janeiro 11 de abril 09 de maio, 14h 13 de junho 27 de junho

A seguir apresentamos as provas da III Olimpada de Maio, realizada em maio de 1997, com as respectivas solues.

EUREKA! N 1, 1998

22

Sociedade Brasileira de Matemtica

III OLIMPADA DE MAIO


Primeiro nvel
Durao da prova: 3 horas. Cada problema vale 10 pontos. No se pode usar mquina de calcular. No se pode consultar livros nem notas.

1)

Num tabuleiro quadrado de 9 casas (de trs por trs), deve-se colocar nove elementos do conjunto S = {0, 1, 2, 3, 4, 5, 6, 7,8,9 }, distintos um do outro, de modo que cada um deles fique numa casa e se verifiquem as seguintes condies: As somas dos nmeros da segunda e terceira fileira sejam, respectivamente, o dobro e o triplo da soma dos nmeros da primeira fileira. As somas dos nmeros da segunda e terceira coluna sejam, respectivamente, o dobro e o triplo da soma dos nmeros da primeira coluna. Mostre todas as formas possveis de colocar elementos de S no tabuleiro,cumprindo com as condies indicadas.

2)
A M B

No retngulo ABCD, M, N, P e Q so os pontos mdios dos lados. Se a rea do tringulo sombreado 1, calcular a rea do retngulo ABCD.

EUREKA! N 1, 1998

23

Sociedade Brasileira de Matemtica

3)

Num tabuleiro de 8 por 8, colocam-se 10 fichas que ocupam, cada uma, uma casa. Em cada casa sem ficha est escrito um nmero entre 0 e 8, que igual quantidade de fichas colocadas nas casas vizinhas. Casas vizinhas so as que tm um lado ou um vrtice em comum.Mostre uma distribuio das fichas que faa que a soma dos nmeros escritos no tabuleiro seja a maior possvel. Joaqun e seu irmo Andrs vo todos os dias para a aula no nibus da linha 62. Joaqun paga sempre as passagens. Cada passagem tem impresso um nmero de 5 dgitos. Um dia, Joaqun observa que os nmeros das passagens, alm de consecutivos, so tais que a soma dos dez dgitos precisamente 62.

4)

Andrs pergunta para ele se a soma dos dgitos de algum dos boletos 35 e, ao saber a resposta, pde dizer corretamente o nmero de cada boleto. Quais so estes nmeros?

5)

Quando Pablo fez 15 anos, fez uma festa convidando 43 amigos. Ele tem uma torta com forma de polgono regular de 15 lados e sobre ela coloca 15 velas.

As velas so colocadas de modo que entre velas e vrtices nunca h trs alinhados (trs velas quaisquer no esto alinhadas, nem duas velas quaisquer com um vrtice do polgono, nem dois vrtices quaisquer do polgono com uma vela). Logo depois, Pablo divide a torta em pedaos triangulares, mediante cortes que unem velas entre si ou velas e vrtices, mas nunca se cruzam com outros j realizados. Por que, ao fazer isto, Pablo consegue distribuir um pedao para cada um de seus amigos mas ele fica sem comer?

EUREKA! N 1, 1998

24

Sociedade Brasileira de Matemtica

SOLUES 1)

Nas condies do problema, a soma de todos os elementos do quadrado deve ser um mltiplo de 6. Como 0 + 1 +2 +3 + 4 + 5 + 6 + 7 + 8 + 9 = 45, que deixa resto 3 quando dividido por 6, as nicas possibilidades para o conjunto dos nmeros que aparecem no quadrado so 0, 1, 2, 3, 4, 5, 6, 7, 8 e 0, 1, 2, 4, 5, 6, 7, 8, 9 (note que retiramos respectivamente 3 e 9 que so os elementos que deixam resto 3 quando divididos por 6). No primeiro caso a soma dos elementos da primeira linha (e da primeira coluna) deve ser :

6=

45 9 6

E no segundo: 7 =

45 3 6

No primeiro caso, as possibilidades para o conjunto resp. C1 elementos da primeira linha (resp. da primeira coluna) so:{0,1,5}, {0,2,4} e {1,2,3} 1.a) Se L1 = {0,1,5} e C1 = {0,2,4}, temos a nica soluo

015 024 236 , e, por simetria, se L1 = {0,2,4} e C1 = {0,1,5}, temos 138 487 567
a) Se L1 {0,1,5} e C1 = {1,2,3} ou L1 = {1,2,3} e C1 = {0,1,5}, temos

105 123 246 ou 048 387 567


b) Se L1 = {0,2,4} e C1 = {1,2,3} ou L1 = {1,2,3} e C1 = {0,1,5}, temos

EUREKA! N 1, 1998

25

Sociedade Brasileira de Matemtica

204 213 156 ou 057 378 468


No segundo caso, as possibilidades para L1 (ou C1) so {0,1,6}, {0,2,5} e {1,2,4} (no pode aparecer o 3). Para cada escolha de L1 e C1 temos uma nica possibilidade de soluo, e as solues so:

016 025 106 124 205 214 248 , 149 , 257 , 059 , 167 e 068 597 687 498 678 489 579
2)

O P

Sejam O o centro do retngulo e T a interseo de ON com BP. Os tringulos OTP e OTB so de reas iguais, pois tm a mesma base e igual altura (OP = NB). Como T o ponto mdio, os tringulos OTP e NTB so iguais, ambos so de rea 1.Ento, a rea do OTP 2 e, como a metade da rea de ONB, a rea de ABCD 16.

3)

Cada ficha soma 1 em cada uma das casas vizinhas que esto livres de ficha. Uma casa tem como mximo 8 vizinhas ( perde vizinhas se est numa borda do tabuleiro). Vejamos que impossvel colocar as 10 fichas em 10 casas isoladas, tais que nenhuma fique na borda do tabuleiro. Podemos pensar que temos um tabuleiro de 6 por 6 pois as casas das bordas no
EUREKA! N 1, 1998

26

Sociedade Brasileira de Matemtica

interessam ou dividimos em 9 setores 2 por 2, mediante paralelas aos lados. Se queremos selecionar casas isoladas, em cada setor podemos escolher ao mximo 1. So, em total, no mximo 9 casas isoladas. Se uma casa fica na borda do tabuleiro, ter como mximo 5 vizinhas. Ou seja ao colocar uma ficha ali, somar no mximo 5. Por outro lado, podemse colocar 8 fichas isoladas mais 2 nas quais as casas se toquem num vrtice; neste caso se perde s uma casa vizinha por cada uma delas. A soma total : 8 8 + 2 7 = 78. Duas possveis distribuies so as seguintes:
1 1 2 1 2 1 1 0 0 0 1 1 1 1 1 1 4) 1 * 2 * 2 * 1 0 1 1 2 * 2 2 * 1 2 2 4 2 4 2 2 0 1 * 3 2 3 * 2 1 1 * 2 * 2 * 1 0 2 2 3 * 3 2 2 0 2 2 4 2 4 2 2 0 1 * 3 2 3 * 1 0 1 * 2 * 2 * 2 1 2 2 3 * 3 2 2 0 1 1 2 1 2 2 * 1 1 * 2 1 2 * 1 0 0 0 0 0 0 1 1 1 1 1 1 0 1 1 1 0

Se o nmero menor abcde, e deve ser 9, pois caso contrrio o maior seria abcd (e + 1), e a soma dos dez dgitos 2 ( a + b + c + d + e ) + 1, que mpar e no poderia ser nunca 62. Alm disso, se o nmero menor acaba num nmero par de noves (99 ou 9999), a soma dos dez dgitos tambm seria um nmero mpar. Assim, o nmero menor abcd9 (d no 9) ou ab999 (b no 9).

No primeiro caso, o outro nmero ser abc(d + 1)0, e a soma dos dez dgitos 2( a + b + c + d) + 10 = 62, ou seja, a + b + c + d = 26, e os dgitos
EUREKA! N 1, 1998

27

Sociedade Brasileira de Matemtica

do nmero menor somam a + b + c + d +9 = 35. Haveria mais de um nmero de cinco dgitos nessas condies 85859, 77669, etc.,) pelo que a resposta que deu Joaqun pergunta do seu irmo foi "no". Assim, os nmeros sero:

ab999 e a(b + 1)000, a soma dos dez dgitos 2(a + b) + 28 = 62; assim: a + b = 17, e como b no 9, a = 9 e b = 8.
Os nmeros dos boletos so: 98999 e 99000
5)

Seja n o nmero de tringulos em que se pode dividir a torta com as condies dadas. Somaremos os ngulos interiores destes n tringulos de duas formas:

(1) (2)

180 n 360 15 + 180 (15 2)

Cada ponto interior (vela) contribui com 360, e a soma dos ngulos interiores de um polgono convexo de L lados, 180 (L 2). Portanto: 180 n = 360 15 + 180 13, onde n = 43. Tambm pode utilizar-se a relao de Euler de um mapa plano: R + V = L + 1 ( R = regies, V = vrtices e L = lados). Ento V = 30, 3R = 2L 15 ( Todos os lados so comuns a duas regies, exceto os 15 lados do contorno do polgono). Assim, pois: 2 L 15 + 30 = L + 1 Portanto: L = 72 e R = 43 3

que a pgina web da Olimpada Brasileira de Matemtica

Voc sabi@

http://www.obm.org.br

EUREKA! N 1, 1998

28

Sociedade Brasileira de Matemtica

III OLIMPADA DE MAIO


Segundo nvel
Durao da prova: 3 horas. Cada problema vale 10 pontos. No se pode usar mquina de calcular. No se pode consultar livros nem notas.

1)

Quantos so os nmeros de sete algarismos que so mltiplos de 388 e terminam em 388?

2) A B P M

Em um quadrado ABCD de lado k, colocam-se os pontos P e Q sobre os lados BC e CD , respectivamente, de forma que PC = 3PB e QD = 2QC. Sendo M o ponto de interseo de AQ e PD, determine a rea do tringulo QMD em funo de k.

3)

Temos 10000 fichas iguais com a forma de um tringulo equiltero. Com esses pequenos tringulos se podem formar hexgonos regulares sem superposies de fichas ou vazios. Considere agora o hexgono regular que desperdia a menor quantidade possvel de tringulos. Quantos tringulos sobram?

4)

Nas figuras, assinalam-se os vrtices com um crculo. Chamam-se caminhos aos segmentos que unem os vrtices. Distribuem-se nmeros inteiros no negativos nos vrtices, e nos caminhos se assinalam as diferenas entre os nmeros de seus extremos.

Diremos que uma distribuio elegante se aparecem nos caminhos todos os nmeros de 1 a n, em que n o nmero de caminhos.
EUREKA! N 1, 1998

29

Sociedade Brasileira de Matemtica

Veja um exemplo de distribuio elegante:

11 11 0

4 9 2 12

7 5 2 10 12

6 1 7 3

1 8 9

Dar se possvel uma distribuio elegante para as seguintes figuras. Em caso de no ser possvel, mostrar por qu.

5)

Quais so as possveis reas de um hexgono com todos os ngulos iguais e cujos lados medem 1,2,3,4,5 e 6 em alguma ordem?

SOLUES 1) Soluo A
O nmero se expressa como: n. 103 + 388, em que n um nmero de quatro cifras.

n. 103 + 388 = k. 388. n. 103 = (k 1).388. Mas 388 = 22.97, ento o nmero n de quatro cifras deve ser mltiplo de 97.
EUREKA! N 1, 1998

30

Sociedade Brasileira de Matemtica

N = t.97, com 11 t 103. So 93 nmeros.

Soluo B
Para que um nmero multiplicado por 388 termine em 388, as ltimas cifras devem ser 001, 501, 251 ou 751. O menor mltiplo de 388 que tem sete cifras 388 2578, e o maior 388 25773. Entre 2578 e 25773 temos: 23 nmeros terminados en 001, desde 3001 at 25001 23 nmeros terminados en 501, desde 3501 at 25501 23 nmeros terminados en 251, desde 3251 at 25251 24 nmeros terminados en 751, desde 2751 at 25751 So em total: 23 + 23 + 23 + 24 = 93

Voc sabia que o maior nmero primo conhecido 23021377-1,


que tem 909529 dgitos e foi descoberto com a ajuda de um computador pessoal?


EUREKA! N 1, 1998

Consulte na Internet a pgina

http://www.mersenne.org/prime.htm

31

Sociedade Brasileira de Matemtica

Sejam D = (0,0), C = (k,0), B = (k,k) e A = (0,k), temos 3k 2k P = k , e Q = ,0 4 3 3 3 A equao de PD y = x, e de AQ y = k x . Se M= (x0,y0), temos 4 2 3 3 9 4k 3 k x =k x x =kx = y = x = , que a altu0 0 0 0 0 0 2 4 9 4 3 4 ra de M em relao a BQ, donde a rea do QMD

2)

DQ (k / 3) (2k / 3)(k / 3) k 2 = = 2 2 9

3)

Um hexgono a unio de 6 tringulos equilteros iguais. Cada um destes tringulos, se tem lado n, decompe-se em n2 tringulos pequenos. Lado 1 Lado 2 Lado 3 Lado n 1 tringulo pequeno 4 tringulos pequenos 9 tringulos pequenos n2 tringulos pequenos

O hexgono de lado n contem 6n2 destes tringulos pequenos. Busca-se o maior n tais que 6n2 10000 n =

100 = 40 6

Usam-se 6 402 tringulos pequenos. Perdem-se 400 = (1000 6 402) tringulos.

EUREKA! N 1, 1998

32

Sociedade Brasileira de Matemtica

4) 0 6 6 1 4 2 5 1 3 4

No segundo caso, no possvel, pois devem aparecer nas arestas os nmeros 1,2,3,,10 (5 pares e 5 mpares). Se quatro ou cinco vrtices recebem nmeros com a mesma paridade, temos pelo menos 6 arestas pares, portanto a numerao no ser elegante. Nos outros casos, teramos seis arestas mpares.
5)

Sejam x, y, z, u, v, w os lados consecutivos do hexgono. Prolongamos os lados y, u e w e obtemos um tringulo equiltero. A rea igual rea deste tringulo equiltero menos as reas de trs tringulos equilteros de lados x, z e v.

rea do hexgono:

3 ( x + y + z )2 x 2 v 2 z 2 4

Vejamos quais so os possveis valores de x, y, z, u, v, w. Seja x = 1, equiltero) Donde temos temos w + x + v = y + x + z w + x + v = v + u + z (pois o tringulo de fora

w+v = y+z w+x = u+z


E temos v + x = y u. No pode ser v x = 5, porque os nicos dois nmeros que tm diferena 5 so 1 e 6.

EUREKA! N 1, 1998

33

Sociedade Brasileira de Matemtica

Se v x = 4, temos v = 5, y = 6, u = 2. De w + 6 = z + 7, resulta, alm disso, w = 4, z = 3. Se v x = 3, ento v = 4. Pode ser y = 5, u = 2 ou y = 6, u = 3. O primeiro caso impossvel, porque no quedam valores de w, z tais que w + 5 = z + 7. O segundo tambm impossvel, pois no restam valores de w, z tais que w + 4 = z +6 Se v x = 2, temos v = 3 e pode ser y = 6, u = 4 ou y = 4, u = 2 No primeiro caso, w + 4 = z + 7, donde w = 5, z = 2. Se v x = 1, temos v = 2 e pode ser y = 4, u = 3 ou y = 5, u = 4 ou y = 6, u = 5. O primeiro e terceiro casos so impossveis. No segundo caso, w + 3 = 6 + z, onde w = 6, z = 3. Os possveis valores da rea so:

3 4

(100 1 25 9) = 65

3 4

3 4

(81 1 4 9) = 67

3 4

Os hexgonos so:
3 6 1 4 5 2 6 3 2 4 5 1

EUREKA! N 1, 1998

34

Sociedade Brasileira de Matemtica

9a. OLIMPADA DE MATEMTICA DO CONE SUL


Salvador - BA, 13 a 21 de junho de 1998
A 9. Olimpada de Matemtica dos pases do Cone Sul ser realizada em Salvador, BA, no perodo de 13 a 21 de junho de 1998. Esta Olimpada ser realizada pela segunda vez no pas (a primeira foi em 1993, em Petrpolis, RJ). Dela participam alunos de at 15 anos dos seguintes pases: Argentina, Brasil, Bolvia, Chile, Paraguai, Peru e Uruguai. A organizao da Olimpada est a cargo da Professora Luzinalva Amorim, da Universidade Federal da Bahia. A equipe brasileira ser selecionada atravs de provas realizadas em maro e maio deste ano e ser liderada pelos professores Paulo Cezar Pinto Carvalho, do IMPA, e Florncio Ferreira Guimares, da UFES. A competio consta de duas provas, realizadas em dois dias, cada uma com trs problemas, valendo 10 pontos cada. Veja abaixo as provas da ltima Olimpada de Matemtica do Cone Sul, realizada em Assuno (Paraguai), em 1997, e os resultados obtidos pela equipe brasileira.

Voc sabi@ que a Olimpada Brasileira de Matemtica


j tem pgina web??

Visite-nos no endereo eletrnico

http://www.obm.org.br

EUREKA! N 1, 1998

35

Sociedade Brasileira de Matemtica

8a. OLIMPADA DO CONE SUL


21 a 25 de Abril de 1997. Assuno, Paraguai. Primeiro dia. Tempo: trs horas.
PROBLEMA 1

De cada nmero inteiro positivo n, n 99, subtramos a soma dos quadrados de seus algarismos. Para que valores de n esta diferena a maior possvel?
PROBLEMA 2

Seja C uma circunferncia de centro O, AB um dimetro dela e R um ponto qualquer em C distinto de A e de B. Seja P a interseo da perpendicular traada por O a AR. Sobre a reta OP se marca o ponto Q, de maneira que QP a metade de PO e Q no pertence ao segmento OP. Por Q traamos a paralela a AB que corta a reta AR em T. Chamamos de H o ponto de interseo das retas AQ e OT. Provar que H, R e B so colineares.
PROBLEMA 3

Demonstrar que existem infinitos ternos (a, b, c), com a, b, c nmeros naturais, que satisfazem a relao: 2a2 + 3b2 5c2 = 1997.

Segundo dia. Tempo: trs horas.


PROBLEMA 4

Considere um tabuleiro de n linhas e 4 colunas. Na 1a. linha so escritos 4 zeros (um em cada casa). A seguir, cada linha obtida a partir da linha anterior realizando a seguinte operao: uma das casas, a escolher, mantida como na linha anterior; as outras trs so troEUREKA! N 1, 1998

36

Sociedade Brasileira de Matemtica

cadas: se na linha anterior havia um 0, coloca-se 1; se havia 1, coloca-se 2; e se havia 2, coloca-se 0. Construa o maior tabuleiro possvel com todas as suas linhas distintas e demonstre que impossvel construir um maior.
PROBLEMA 5

Seja n um nmero natural, n > 3. Demonstrar que entre os mltiplos de 9 menores que 10n h mais nmeros com a soma de seus dgitos igual a 9(n-2) que nmeros com a soma de seus dgitos igual a 9(n-1).
PROBLEMA 6

Considere un tringulo acutngulo ABC, e seja X um ponto do plano do tringulo. Sejam M, N e P as projees ortogonais de X sobre as retas que contm as alturas do tringulo ABC. Determinar para que posies de X o tringulo MNP congruente a ABC.

Nota: a projeo ortogonal de um ponto X sobre uma reta l a interseo de l com a perpendicular a ela que passa por X.

RESULTADOS OBTIDOS PELA EQUIPE BRASILEIRA

BRA 1 BRA 2 BRA 3 BRA 4

Murali Srinivasan Vajapeyam Rui Lopes Viana Filho Christian Iveson Daniele Vras de Andrade

OURO OURO BRONZE BRONZE

EUREKA! N 1, 1998

37

Sociedade Brasileira de Matemtica

NMEROS MGICOS E CONTAS DE DIVIDIR


Carlos Gustavo Tamm de Arajo Moreira Nvel Iniciante.
Temas muito inocentes de aritmtica bsica, como contas de multiplicar, podem gerar resultados bastante interessantes e surprendentes, como ao multiplicar o nmero 142857 por 2, 3, 4, 5, 6 e 7: 142857 2 142857 3 142857 4 142857 5 142857 6 = = = = = 285714 428571 571428 714285 857142

Por que razo acontece essa repetio dos dgitos de 142857 ao multipliclo por 2, 3, 4, 5 e 6, sempre com a mesma ordem circular? Ser mera coincidncia? Ser possvel obter outros exemplos desse tipo? A resposta tem a ver com o resultado de 142857 7, que 999999. Isso quer dizer que o perodo da representao decimal de 1/7 exatamente 142857. Vamos examinar com cuidado a conta de diviso de 1 por 7: 10 30 20 60 40 50 1 7 0,142857

repetindo o resto 1, o que quer dizer que todo o processo se repete e o resultado da diviso 1/7 = 0,142857142857142857 Podemos reescrever o processo assim: 1 10 30 20 60 40 =07+1 =17+3 =47+2 =27+6 =87+4 =57+5

EUREKA! N 1, 1998

38

Sociedade Brasileira de Matemtica

50 = 7 7 + 1. Da temos: 10 7 1 = 3, e portanto 100-7 10 = 30, e como 30 7 4 = 2 temos: 100 7 (10 + 4) = 2, e analogamente obtemos: 1000 7 (100 + 40 + 2) = 6 10000 7 (1000 + 400 + 20 +8) = 4 100000 7 (10000 + 4000 + 200 + 80 + 5) = 5 1000000 7 (100000 + 40000 + 2000 + 800 + 50 + 7 ) = 1 ( A ltima igualdade diz que 142857 7 = 999999) Desta forma, os restos sucessivos que aparecem na diviso de 1 por 7, que so 3, 2, 6, 4, 5, 1 so, respectivamente, os restos na diviso por 7 de 10, 100, 1000, 10000, 100000 e 1000000. Estes restos assumem todos os valores possveis entre 1 e 6 e isso equivale ao fato de o perodo de 1/7 ter 6 casas. Desta forma, temos: 2 0,142857142857142857 = 2/7 = 100/714 = 100 0, 14285714 2857142857 14 = 0,285714285714285714, e, portanto, temos 2 142857 = 285714 Da mesma maneira temos que 3/7 = 10/7 1 implica 3 142857 = 428571, e as outras igualdades seguem de modo anlogo. Notemos agora que sempre que o perodo da representao decimal de 1/n tiver n 1 casas decimais (que o mximo possvel), o perodo (que ser igual a (10n-1 1) / n ) ter as mesmas propiedades de 142857. O primeiro valor de n maior que 7 para o qual isso acontece 17, e o perodo de 1/17 0588235294117647. Multiplique esse nmero por 2, 3, 4, 5, 6, 7, 8, 9, 10, 11, 12, 13, 14, 15, 16 e 17 para conferir. Observe que, para que isso acontea, n deve ser um nmero primo, pois se n = p b, com b maior que 1 e p um nmero primo diferente de 2 e 5, ento p nunca aparecer como resto na diviso de 1 por n, pois em geral um fator primo comum de n e de um resto que aparece na diviso de 1 por n s pode ser 2 ou 5 ( de fato, um resto que aparece na diviso de 1 por n resto da diviso de alguma potncia de 10 por n ). Por outro lado, se os nicos fatores primos de n so 2 e 5, ento 1/n tem representao decimal finita.

EUREKA! N 1, 1998

39

Sociedade Brasileira de Matemtica

Concluso: Se o perodo de 1/n tiver n1 casas decimais, ele ter propiedades anlogas s de 142857: os dgitos de seus produtos por 1, 2, 3, 4, , n1 sero sempre os mesmos, na mesma ordem circular. Para que isso acontea, n deve ser primo e a menor potncia de 10 que deixa resto 1 quando dividida por n deve ser 10n1. Dizemos que, nesse caso, 10 raiz primitiva mdulo n. No se sabe se existem infinitos primos n com essa propriedade. Isso seguiria de uma famosa conjectura de teoria dos nmeros, a conjectura de Artin (vide [V]).
Os nmeros primos n menores que 100 tais que o perodo de 1/n na base 10 tem n 1 casas so 7, 17, 19, 23, 29, 47, 59, 61 e 97. Por outro lado, para todo nmero primo n existem nmeros naturais B entre 2 e n 1 tais que o perodo de 1/n na base B tem exatamente n 1 casas (nesses casos B raiz primitiva mdulo n). Se um nmero B tem essa propriedade, todas as bases da forma kn + B com k natural tambm tm. Nesses casos, o perodo de 1/n na base B ( ou seja, o nmero (Bn-11)/n ), quando multiplicado por 1, 2, 3, , n 1 ter representaes na base B que sero permutaes uma da outra com a mesma ordem circular. Por exemplo, com n = 5 e B = 8, temos que a representao de 1/5 na base 8 0,146314631463 Na base 8 temos: 2 (1463)8 =(3146)8 , 3 (1463)8 = (4631)8 , 4 (1463)8 = (6314)8 , 5 (1463)8 = (7777)8

Referncias: [L] Lima, Elon L., Meu Professor de Matemtica e outras histrias, pp. 158-170 SBM, 1991. [T] Tahan, Malba, O homen que calculava, Ed. Record. [V] Voloch, Jos Felipe, Raizes Primitivas e a Conjectura de Artin, Revista Matemtica Universitria N9/10, dezembro de 1989, pp. 153-158.

EUREKA! N 1, 1998

40

Sociedade Brasileira de Matemtica

COMO PERDER AMIGOS E ENGANAR PESSOAS


Nicolau C. Saldanha Nvel Avanado.
Neste artigo apresentaremos quatro situaes simples em que probabilidades enganam. Em alguns casos a probabilidade de certos eventos tem um valor diferente daquele que a maioria das pessoas parece julgar razovel, pelo menos de incio; em um exemplo mostraremos como facil chegar a concluses absurdas. Para que o leitor possa pensar sozinho, apresentaremos primeiro quatro "enunciados", em que lanamos cada situao, e depois quatro "desenvolvimentos" em que voltamos a discutir as quatro situaes na mesma ordem. Qualquer um pode usar estes exemplos para divertir-se s custas de seus amigos, mas em nenhum caso o autor tem responsabilidade pela integridade fsica daqueles que usarem a Matemtica para o mal.

ENUNCIADOS 1. Em um programa de auditrio, o convidado deve escolher uma dentre trs portas. Atrs de uma das portas h um carro e atrs de cada uma das outras duas h um bode. O convidado ganhar o que estiver atrs da porta; devemos supor neste problema que o convidado prefere ganhar o carro. O procedimento para escolha da porta o seguinte: o convidado escolhe inicialmente, em carter provisrio, uma das trs portas. O apresentador do programa, que sabe o que h atrs de cada porta, abre neste momento uma das outras duas portas, sempre revelando um dos dois bodes. O convidado agora tem a opo de ficar com a primeira porta que ele escolheu ou trocar pela outra porta fechada. Que estratgia deve o convidado adotar? Com uma boa estratgia, que probabilidade tem o convidado de ganhar o carro? 2. Um mvel tem trs gavetas iguais. Em uma gaveta h duas bolas brancas, em outra h duas bolas pretas, e na terceira h uma bola branca e outra preta. Abrimos uma gaveta ao acaso e tiramos uma bola ao acaso sem olhar a segunda bola que est na gaveta. A bola que tiramos branca. Qual a probabilidade de que a segunda bola que ficou sozinha na gaveta seja tambm branca?
EUREKA! N 1, 1998

41

Sociedade Brasileira de Matemtica

3. Dois amigos querem decidir quem pagar a conta do restaurante com

uma aposta. Cada um deles escolhe uma seqncia de trs caras ou coroas, e eles jogam uma moeda at que saia uma das duas seqncias: aquele que tiver escolhido a primeira seqncia a sair ganhou a aposta. Por exemplo, Andr (por ser o primeiro em ordem alfabtica) o primeiro a escolher e fica com a seqncia ckc (em que c representa cara e k coroa) enquanto Bernardo responde com cck. Eles jogam a moeda obtendo kckkckkkkccck, e neste momento Bernardo declara-se o vencedor. Esta aposta justa? Andr leva vantagem ou desvantagem por ser o primeiro a escolher? Quais so as probabilidades de vitria de cada um?

4. Aqui novamente devemos nos imaginar em um programa de auditrio. Eugnio foi sorteado e tem direito a um prmio, mas ele deve escolher entre dois envelopes lacrados aparentemente iguais. O apresentador informa que cada envelope tem um cheque e que o valor de um cheque o dobro do outro, mas no diz nada sobre o valor dos cheques, nem indica qual envelope contm o cheque de maior valor. Eugnio escolhe e abre um envelope que contm um cheque de, digamos, R$ 100. Neste momento, o apresentador sempre faz uma proposta ao convidado: ele pode trocar de envelope mediante uma multa de 5% do valor do cheque que ele tem em mos, no caso, R$ 5. Assim, se Eugnio aceitar, ele pode ganhar R$ 45 (se o cheque no segundo envelope for de R$ 50) ou R$ 195 (se o outro cheque for de R$ 200). Suponhamos que Eugnio (que fez um curso de Introduo Probabilidade no perodo anterior) queira maximizar o valor esperado de seu prmio. Deve ele aceitar a troca? E se o valor do primeiro cheque tivesse sido outro, de que forma deveria isto influenciar a deciso de Eugnio? Se Eugnio trocar de envelope independentemente do valor do cheque, no vale mais a pena para ele trocar de envelope antes de abrir, evitando, assim, a multa? DESENVOLVIMENTOS

1. A resposta correta que, trocando de porta, a probabilidade de ganhar o carro 2/3, enquanto no trocando a probabilidade apenas 1/3. Uma forma simples de ver isto a seguinte: trocando de porta, o convidado ganha, desde que a primeira porta que ele escolher esconda um dos dois bodes, como se pode facilmente perceber. A melhor estratgia para o convidado ,
EUREKA! N 1, 1998

42

Sociedade Brasileira de Matemtica

portanto, trocar sempre, e assim sua probabilidade de ganhar fica sendo 2/3. O erro comum aqui achar que, aps a eliminao de uma porta (que foi aberta pelo apresentador, revelando um bode), h uma simetria entre as duas outras portas e a probabilidade de cada uma esconder o carro 1/2. No existe, entretanto, tal simetria, pois a porta escolhida pelo convidado no poderia, pelas regras, ser trocada pelo apresentador, enquanto a outra poderia ter sido aberta, mas no foi. Este processo de fato era seguido em um programa nos Estados Unidos. Uma longa e spera discusso ocorreu na imprensa quanto a qual era o valor correto da probabilidade, e pessoas que deveriam ser capazes de resolver um problema trivial como este passaram pela vergonha de publicar solues erradas. Julgamos melhor esquecer os detalhes deste episdio deprimente.

2. A resposta correta 2/3 (e no 1/2). As seis bolas seriam de incio igualmente provveis, mas sabemos que a primeira bola escolhida foi branca: assim, as trs bolas brancas tm igual probabilidade. Estamos interessados em saber a cor da companheira de gaveta de cada bola branca: em dois casos branca, em um caso preta. Assim, a probabilidade de que a segunda bola seja branca 2/3, como j afirmamos.
Um raciocnio comum, mas errado, dizer: as gavetas so igualmente provveis, mas obviamente no escolhemos a gaveta que contm duas bolas pretas. Portanto, teramos probabilidade 1/2 de termos escolhido a gaveta com duas bolas brancas e 1/2 de termos escolhido a gaveta com uma bola de cada cor; no primeiro caso, a segunda bola branca e, no segundo caso, a bola preta. Assim, a resposta seria 1/2. O que h de errado neste raciocnio? O erro est em dizer que as duas gavetas possveis so igualmente provveis. Inicialmente a probabilidade de cada gaveta de fato a mesma (inclusive para a gaveta com duas bolas pretas), mas, ao tirarmos uma bola e constatarmos que ela branca, isto deixa de ser verdade. Isto bem bvio para a gaveta com duas bolas pretas: passou a ser impossvel termos escolhido esta gaveta. Entre as duas outras gavetas, entretanto, h uma diferena que est sendo ignorada no raciocnio do pargrafo anterior. Se pr-escolhermos a gaveta com duas bolas branEUREKA! N 1, 1998

43

Sociedade Brasileira de Matemtica

cas, temos certeza de passar no teste: uma bola escolhida ao acaso nesta gaveta ser sempre branca. Por outro lado, se pr-escolhermos a gaveta com uma bola de cada cor, ainda temos probabilidade 1/2 de sacarmos uma bola preta, o que estaria em contradio com o enunciado. Assim, a probabilidade de termos escolhido cada uma destas duas gavetas 2/3 e 1/3, respectivamente. Podemos, a partir deste ponto facilmente deduzir a resposta correta de 2/3. fato emprico desencorajador que muitas pessoas teimam em dizer que a probabilidade 1/2 mesmo aps esta explicao. O seguinte exemplo serve como exerccio para aqueles que entenderam a explicao e uma espcie de reduo ao absurdo do raciocnio "rival". Temos novamente trs gavetas, uma com vinte bolas brancas, uma com vinte bolas pretas e a terceira com dez bolas de cada cor. Abrimos uma gaveta e, sem olhar, retiramos ao acaso dez bolas: elas so todas brancas. Qual a probabilidade de que as dez bolas restantes sejam tambm brancas?

3. No nosso exemplo, Bernardo tinha probabilidade 2/3 de ganhar. Em geral, o segundo a jogar leva uma vantagem considervel e, se escolher bem sua resposta, pode garantir uma probabilidade de vitria de pelo menos 2/3, mas s vezes at 7/8, dependendo da primeira jogada. A Tabela 1 d a probabilidade de vitria de Bernardo para cada par de jogadas (a coluna a jogada de Andr e a linha a de Bernardo).
ccc ccc ccc ckc ckk kcc kck kkc kkk _ 1/2 3/5 3/5 7/8 7/12 7/10 1/2 cck 1/2 _ 1/3 1/3 3/4 3/8 1/2 3/10 ckc 2/5 2/3 _ 1/2 1/2 1/2 5/8 5/12 ckk 2/5 2/3 1/2 _ 1/2 1/2 1/4 1/8 kcc 1/8 1/4 1/2 1/2 _ 1/2 2/3 2/5 kck 5/12 5/8 1/2 1/2 1/2 _ 2/3 2/5 kkc 3/10 1/2 3/8 3/4 1/3 1/3 _ 1/2 kkk 1/2 7/10 7/12 7/8 3/5 3/5 1/2 _
Tabela 1

No reconstruiremos aqui toda a tabela: apresentaremos apenas como exemplo a situao descrita no enunciado. O leitor que estiver interessado em aprender mais sobre este problema pode consultar nosso Precisa-se de algum para ganhar muito dinheiro, a ser publicado na Revista do ProfesEUREKA! N 1, 1998

44

Sociedade Brasileira de Matemtica

sor de Matemtica do Chile, mas j disponvel na home page do autor: http://www.mat.puc-rio.br/~nicolau/. O Diagrama 2 descreve bem a situao. Os seis vrtices indicam as seis situaes possveis durante o processo de jogar a moeda. O ponto indica que nenhum jogador tem como esperar fazer uso das jogadas j feitas, ou seja, ou nenhum lance ainda foi feito, ou foi lanado apenas um k, ou os dois ltimos lances foram kk; como o jogo sempre comea nesta situao, chamaremos este vrtice de inicial. O c indica que o ltimo lance foi um c mas o anterior ou no existiu ou foi um k. Os vrtices cc e ck indicam que estes foram os dois ltimos lances. Finalmente, os vrtices cck e ckc indicam que o jogo terminou; chamaremos estes vrtices de finais.

cc c ck

cck ckc

Diagrama 2
As duas setas partindo de cada vrtice (exceto os finais) indicam como a situao se modifica a cada lance de moeda: elas correspondem s possibilidades de tirar c ou k em um dado momento. Queremos agora calcular a probabilidade de vitria de Bernardo, dado que o jogo chegou a uma certa situao. Temos, assim, quatro probabilidades a serem calculadas: p., pc, pcc e pck; consideramos naturalmente pcck = 1 e pckc = 0. Como a partir de cada vrtice no final as probabilidades associadas s duas setas so iguais, temos as seguintes equaes:

p. = 1/2 (p. + pc) pc = 1/2 (pcc + pck) pcc= 1/2 (pcc + 1) pck= 1/2 p. .
Resolvendo o sistema, temos p. = 2/3, conforme afirmamos. O erro mais natural aqui achar que todas as seqncias so igualmente boas: isto no verdade, pois os dois ltimos lances em geral serviram,
EUREKA! N 1, 1998

45

Sociedade Brasileira de Matemtica

sem sucesso, para tentar finalizar as seqncias e serviro agora para tentar inici-las. Mais surpreendente ainda o fato de que o segundo jogador sempre tem uma boa resposta: este jogo um pouco como jogar par-oumpar ou pedra-papel-tesoura com um dos jogadores tendo o direito de escolher sua jogada s depois de ver a jogada do adversrio.

4. Antes de mais nada gostaramos de lembrar que Eugnio deseja, por hiptese, maximizar o valor esperado do prmio. Este critrio razovel em algumas situaes e em outras no. Outro convidado poderia precisar desesperadamente de uma certa quantia, talvez R$ 100, e gostaria, portanto, de maximizar a probabilidade de ganhar pelo menos este valor crtico. Ainda outro convidado pode ser to curioso que deseja saber quanto h em cada envelope mais do maximizar seu prmio. O leitor, se fosse o convidado, talvez julgasse interessante considerar ainda outros aspectos. Podemos imaginar inmeros critrios diferentes e em princpio cada critrio gera um novo problema. Ns nos propomos aqui a estudar o problema na forma em que foi proposto e no a discutir se Eugnio, com sua opo pelo valor esperado, um homem verdadeiramente sbio.
Neste problema, ao contrrio dos outros, apresentaremos inicialmente um raciocnio falho e vamos segui-lo at chegarmos a um absurdo deixando a anlise dos erros deste raciocnio para o final. Para tornar a discusso toda mais viva, acompanharemos o pensamento de Eugnio. Ao receber a proposta de troca, Eugnio pensa: Se ficar com este cheque, meu prmio ser de R$ 100. Se trocar de cheque, tenho probabilidade 1/2 de ganhar R$ 45 e probabilidade 1/2 de ganhar R$ 195: o valor esperado de (1/2) 45 + (1/2) 195 = 120 reais. Como 120 maior que 100, a troca vantajosa. Eugnio troca de cheque e fica felicssimo ao ver que o outro cheque de R$ 200: ele ganhou R$ 195! Ao voltar para seu lugar no auditrio, Eugnio continua pensando: Na verdade vale a pena trocar qualquer que seja o valor do primeiro cheque. Se chamarmos este valor de x, temos por um lado a opo de ficar com x e por outro lado a opo de arriscar, com probabilidade 1/2 de ganhar 0.45x e probabilidade 1/2 de ganhar 1.95x. No primeiro caso, o valor esperado x e, no segundo caso, o valor esperado 1.2x. Assim, como x > 0,
EUREKA! N 1, 1998

46

Sociedade Brasileira de Matemtica

vale sempre a pena trocar. Eugnio fica feliz com sua concluso e pensa como seu curso de Probabilidade foi til.
Mas um pouco mais tarde Eugnio comea a ter dvidas quanto a suas concluses: Se vale a pena trocar de envelope sempre, ento no necessrio abrir o envelope e ler o valor do cheque para tomar a deciso de trocar. Neste caso, eu poderia ter trocado de envelope um minuto antes e ter evitado a multa. Eugnio fica irritado, pensando que poderia ter ganhado 5 reais a mais se apenas tivesse pensado mais rpido. Mas ele continua pensando: Ei, espere, h algo errado! Um minuto antes os dois envelopes estavam lacrados e pareciam iguais para mim: trocar significaria apenas escolher o outro. Mas, ento, cada vez que eu penso em um envelope tenho que trocar e nunca posso escolher nada! Assim, ao invs de aproveitar seu prmio, Eugnio passa a noite angustiado com seu paradoxo. Na manh seguinte, Eugnio procura seus colegas do curso de Probabilidade com a pergunta: o que exatamente h de errado com este raciocnio? O erro de Eugnio est logo no incio, quando aceita, sem alis sequer questionar, que a probabilidade do segundo cheque ser maior 1/2. O leitor deve estar muito surpreso: quase como se de repente dissssemos que cara e coroa tm probabilidades diferentes. Por isso daremos uma explicao relativamente longa para tentar convencer. Comearemos fazendo algumas digresses considerando o que um outro convidado, o Joo, que nunca estudou probabilidade, mas que tem bom senso, faria em algumas situaes extremas. Joo no acompanha todos os sorteios, mas mesmo assim ele certamente tem alguma noo, por vaga que seja, de qual a faixa dos prmios. Assim, se o valor do primeiro cheque fosse muito baixo, Joo certamente pensaria: No possvel, ou pelo menos no provvel, que o segundo cheque seja ainda menor. Assim, quase certamente eu peguei o envelope com o cheque de menor valor (alm de ter tido o azar de vir em um dia em que os prmios foram baixos) e aposto que o outro cheque maior: vou trocar. Por outro lado, se o valor do primeiro cheque fosse muito alto, seu pensamento seria: Que sorte, hoje os prmios esto timos! E muito improvvel que o segundo cheque seja ainda maior! Vou ficar com este cheque mesmo! Assim, Joo no atribui probabilidades iguais s duas possibilidades (o segundo cheque ser maior ou meEUREKA! N 1, 1998

47

Sociedade Brasileira de Matemtica

nor), e as probabilidades que ele atribui (inconscientemente) a estes dois eventos dependem do valor do primeiro cheque. Bem, este era o Joo e no o Eugnio: ao consider-lo, desviamo-nos temporariamente do problema original e do contexto que nos impusemos no primeiro pargrafo deste desenvolvimento, pois Joo nem sabe o que o valor esperado e seus critrios no so os de Eugnio. Joo atribuiu subjetivamente probabilidades diferentes aos dois eventos; Eugnio (que alis no se defrontou com situaes extremas) atribuiu probabilidades iguais. Ser que em algum sentido errado atribuir sempre probabilidades iguais? Sim, atribuir probabilidades sempre iguais no apenas errado, mas contraditrio com a Teoria da Probabilidade que Eugnio tenta usar. Para entender isto, vamos representar cada configurao inicial de envelopes por um par ordenado (x1, x2) de nmeros reais positivos: x1 o valor do cheque no primeiro envelope escolhido pelo convidado, e x2 o valor do segundo cheque. Assim, o espao amostral R2 a unio de duas semi-retas abertas partindo da origem, como mostrado na Figura 3. A histria que contamos envolvendo Eugnio corresponde ao ponto (100, 200), tambm indicado. Ao abrir o primeiro envelope, definimos o valor de x1 e ficamos restritos interseo de com uma reta vertical, ou seja, aos dois pontos (x1, x2 = 2x1) e (x1, x2 = x1/2). Eugnio implicitamente aceita que a probabilidade condicional a um valor qualquer fixo para x1 destes dois pontos 1/2. Assim, ele deve aceitar que:

P({(t, 2t); t [T, 2T)}) = P({(t, t/2); t [T, 2T)})


para qualquer nmero positivo T, em que P(C), C , denota a probabilidade de que (x1, x2) esteja em C. Por outro lado, a simetria inicial entre os envelopes diz que

P({(t, 2t); t [T, 2T)}) = P({(2t, t); t [T, 2T)}).


Sejam

An = {(t, 2t); t [2n, 2n+1)}, Bn = {(2t, t); t [2n ,2n+1)},

EUREKA! N 1, 1998

48

Sociedade Brasileira de Matemtica

em que n um inteiro qualquer; as identidades acima nos do P(An) = P(Bn-1) e P(An) = P(Bn), respectivamente. Por induo, P(An) = P(Bn) = P(A0) para todo n. Observemos desde j que esta concluso no mnimo estranha: ela diz que a probabilidade de o valor de menor cheque estar entre 64 e 128 igual probabilidade de o menor cheque estar entre 264 e 265, ou entre 24199021 e 24199022; no prximo pargrafo veremos que esta concluso no apenas estranha, mas realmente absurda, mesmo ignorando o fato de que um prmio de R$ 24199021 uma impossibilidade prtica.

X2

300

200

(100,200)

100 (100,50)

100

200

300

X1 Figura 3

Observemos que os conjuntos An e Bn so dois a dois disjuntos e sua unio . Se P(A0)>0, podemos tomar N N tal que N P(A0)>1 e temos

An > 1, 0 n < N

o que absurdo. Por outro lado, se P(A0) = 0 temos

EUREKA! N 1, 1998

49

Sociedade Brasileira de Matemtica

( An N < n < N

Bn )

=0

para todo N, o que tambm um absurdo, pois, quando N cresce, este conjunto tambm cresce, tendendo no limite para , donde teramos P()= 0, contradizendo P() = 1. Assim, em qualquer caso, temos um absurdo. Esta explicao um pouco tcnica, mas coincide perfeitamente com o "bom senso" de Joo: no podemos ignorar o primeiro cheque. Se seu valor for muito baixo, a probabilidade de que o segundo cheque seja maior deve em geral ser muito maior do que 1/2, pois P(An) deve tender a zero quando n tende a . Por outro lado, se o seu valor for muito alto, a probabilidade de que o segundo cheque seja ainda maior deve ser muito menor do que 1/2, pois P(An) tambm deve tender a zero quando n tende a +. E Eugnio, afinal de contas, precisa fazer uma avaliao sutil, dependendo de que valores so plausveis como prmio: at um certo valor-limite vale a pena trocar, acima deste valor no.

Nicolau C. Saldanha Departamento de Matemtica, PUC-RIO Gvea, Rio de Janeiro, RJ 22453-900, BRASIL nicolau@mat.puc-rio.br, http://www.mat.puc-rio.br/~nicolau/

EUREKA! N 1, 1998

50

Sociedade Brasileira de Matemtica

DOIS PROBLEMAS SOBRE GRAFOS


Paulo Cezar Pinto Carvalho IMPA Nvel Intermediario. INTRODUO
A figura abaixo mostra um mapa rodovirio de um pas fictcio. Neste artigo vamos examinar dois problemas relativos a este mapa: 1. Um funcionrio, encarregado de verificar, periodicamente, o estado das estradas, deseja planejar a sua rota de inspeo. Idealmente, esta rota deveria se iniciar na capital e percorrer cada estrada exatamente uma vez, voltando, ento, ao ponto de partida. Existe tal rota? 2. Um representante de vendas de uma companhia deseja planejar uma rota na qual ele visite cada cidade exatamente uma vez, voltando ao ponto de partida. Existe tal rota?

Fig. 1 - Mapa rodovirio de um pas fictcio


H vrios pontos em comum entre os dois problemas. Por exemplo: em ambos se deseja verificar a existncia de um circuito (ou ciclo) no grafo determinado pelo mapa (um grafo um par (V, A), em que V o conjunto de vrtices do grafo, e A um conjunto de pares de vrtices os arcos do grafo). No primeiro problema, este circuito deve incluir exatamente uma vez cada arco do grafo. No segundo problema, o circuito deve incluir exatamente uma vez cada vrtice do grafo. Embora os dois problemas sejam aparentemente semelhantes, h algumas diferenas fundamentais entre eles. Convidamos os leitores a refletir um pouco sobre cada um deles antes de prosseguir.

EUREKA! N 1, 1998

51

Sociedade Brasileira de Matemtica

CIRCUITOS EULERIANOS
O primeiro problema o do inspetor de estradas foi estudado pela primeira vez por Euler (1707-1783). Por esta razo, um circuito que percorre cada arco de um grafo exatamente uma vez chamado de circuito euleriano e um grafo que possui um tal circuito chamado de grafo euleriano. A situao estudada por Euler ficou imortalizada como o Problema das Pontes de Knisberg, ilustrado na figura abaixo, e que possivelmente j conhecido por muitos dos leitores. O objetivo percorrer exatamente uma vez todas as sete pontes da cidade (hoje Kaliningrado), que conectam as duas ilhas entre si e com as margens do rio, voltando ao ponto de partida.

Fig. 2 O Problema das Pontes de Knisberg

Em linguagem de grafos, trata-se de encontrar um circuito euleriano no grafo da figura acima, no qual os vrtices representam as ilhas e as margens e os arcos so as pontes1. Euler mostrou a no-existncia de tal circuito atravs de um argumento extremamente simples. Consideremos, por exemplo, a ilha da direita. Um circuito qualquer deve chegar ilha e sair dela o mesmo nmero de vezes. Logo, para que exista um circuito euleriano, deve haver um nmero par de pontes com extremidade nesta ilha. Como existem trs pontes nessas condies, conclumos que no possvel encontrar um circuito euleriano. De modo mais geral, temos o seguinte:

Teorema: Existe um circuito euleriano em um grafo se e somente se o grafo conexo (isto , existe um caminho ligando qualquer par de vrtices) e cada vrtice tem grau par (ou seja, o nmero de arcos que nele incidem par).

A rigor, neste caso temos um multi-grafo, j que certos pares de vrtices so ligados por mais de um arco.

EUREKA! N 1, 1998

52

Sociedade Brasileira de Matemtica

O argumento acima mostra a necessidade de se ter grau em cada vrtice para existir um circuito euleriano. tambm bvio que o grafo precisa ser conexo. A prova de que essas duas condies implicam na existncia de um circuito euleriano pode ser feita por induo finita no nmero de arcos do grafo e deixada como um exerccio para o leitor. [Sugesto: suponha a propriedade verdadeira para grafos com menos de n arcos e considere um grafo com n arcos, satisfazendo s duas condies. Comeando em um vrtice qualquer, percorra arcos do grafo, at voltar a um vrtice j visitado (o caminho gerado possui, ento, um ciclo). Retirando do grafo os arcos desse ciclo, obtm-se um ou mais grafos satisfazendo as duas condies e com menor nmero de arcos (portanto, com circuitos eulerianos, de acordo com a hiptese de induo). Basta explicar como costurar esses circuitos eulerianos ao ciclo descrito acima]. Podemos aplicar este teorema ao nosso problema de inspeo de estradas. Da mesma forma como no Problema das Pontes de Knisberg, no existe qualquer circuito euleriano no grafo determinado pelo mapa rodovirio, j que o vrtice correspondente capital tem grau 3. Assim, se o nosso inspetor de estradas recebesse de seu chefe a incumbncia de elaborar um trajeto nas condies do problema 1, ele poderia facilmente convenc-lo da impossibilidade de faz-lo. Como veremos a seguir, a situao do seu colega representante de vendas bem pior...

CIRCUITOS HAMILTONIANOS
Um circuito passando exatamente uma vez por cada vrtice de um grafo chamado de circuito hamiltoniano, em homenagem ao matemtico irlands William Rowan Hamilton (1805-1865), que estudou este problema no grafo determinado pelas arestas de um dodecaedro regular (existe ou no um circuito hamiltoniano neste caso?). Um grafo que possui um circuito hamiltoniano chamado de grafo hamiltoniano. A situao do problema de verificar se um grafo hamiltoniano bem diferente da do problema anterior. Apesar de terem sido estudados por vrios sculos, no h uma boa caracterizao dos grafos hamiltonianos. H diversas famlias de grafos para os quais existe um circuito hamiltoniano (um exemplo trivial um grafo completo, em que cada vrtice
EUREKA! N 1, 1998

53

Sociedade Brasileira de Matemtica

ligado a todos os outros); tambm possvel estabelecer certas condies que implicam na no-existncia de um circuito. Mas uma caracterizao geral no foi encontrada e, luz de certos avanos em teoria da computao das ltimas dcadas, parece improvvel que ela seja encontrada algum dia. O problema de decidir se um grafo hamiltoniano est na companhia de diversos problemas ilustres, com as seguintes caractersticas em comum:

O problema possui uma assimetria fundamental: muito fcil convencer algum da existncia de um circuito hamiltoniano em um grafo: basta exibir tal caminho. No entanto, difcil, em geral, convencer algum da no-existncia de um tal circuito. Por exemplo, o grafo da figura abaixo (o leitor capaz de reconhec-lo?) tem um circuito hamiltoniano, de cuja existncia o leitor fica imediatamente convencido pela figura. J o grafo dado no incio do artigo no tem circuito hamiltoniano, mas no existe um argumento simples e geral para demonstrar esse fato (assim, nosso amigo representante de vendas certamente ter mais trabalho para convencer seu chefe da impossibilidade de elaborar uma rota nas condies do problema 2 do que seu colega inspetor de estradas).

Fig. 3 Um grafo hamiltoniano


No se conhece um algoritmo eficiente para verificar se um grafo hamiltoniano (por eficiente, entendemos aqui um algoritmo em que o nmero de passos seja limitado por um polinmio no nmero de vrtices do grafo). Alm disso, parece improvvel que um tal algoritmo possa algum dia ser encontrado, porque sua existncia implicaria na existncia de algoritmos eficientes para um grande nmero de outros

EUREKA! N 1, 1998

54

Sociedade Brasileira de Matemtica

problemas, para os quais tambm no se conhecem algoritmos eficientes. Estes problemas (incluindo o de verificar a existncia de circuito hamiltoniano) formam uma classe de problemas chamados de NPcompletos. Um outro problema famoso da classe o de determinar o nmero mnimo de cores que podem ser usadas para colorir os vrtices de um grafo de modo que vrtices de mesma cor no sejam ligados por um arco. O leitor poder estar pensando assim: mas ser que esta histria de algoritmos eficientes tem relevncia, numa era de computadores cada vez mais velozes? Afinal de contas, existe um algoritmo extremamente simples para verificar se um grafo possui um circuito hamiltoniano. Se existir um tal circuito, ele corresponder a uma permutao (circular) dos vrtices com a propriedade de que vrtices consecutivos sejam ligados por um arco do grafo. Ora, para verificar a existncia de circuito hamiltoniano basta gerar todas as permutaes circulares dos vrtices e testar se uma delas corresponde a um percurso no grafo. claro que este algoritmo funciona para grafos de tamanho moderado (ele poderia ser o recurso usado pelo nosso vendedor: como so apenas 9 cidades, ele teria que testar apenas 8! = 40.320 caminhos, o que seria feito com rapidez em um computador). Mas o que ocorre com grafos maiores? Vejamos, por exemplo, uma situao em que o nmero de cidades cresce para 50 (o que representaria um tamanho ainda bastante razovel para uma situao real). Neste caso, o computador deveria examinar 49! circuitos potenciais. Tentemos estimar a magnitude deste nmero. A forma mais simples usar a frmula de Stirling, que fornece a estimativa

n n! 2n . Mas, neste caso, podemos usar estimativas mais elemene


tares. Por exemplo, podemos usar apenas potncias de 2. Temos: 49! = 1 2 3 4 5 6 7 8 ... 15 16 ... 31 32 49 > 1 2 2 4 4 4 4 8 ... 8 16 ... 16 32 32 = 22 x 44 x 88 x 1616 x 3218 = 22+8+64+90 = 2164. Mas 210 = 1024 >103. Logo 49! > 16. 1048. Ora, um computador moderno pode realizar cerca de 200 milhes de operaes por segundo. Se em cada operao ele conseguir testar um circuito, ele ainda assim precisar de mais de 16. 1048 / 2. 106 = 8 1042 seEUREKA! N 1, 1998

55

Sociedade Brasileira de Matemtica

gundos, o que corresponde a aproximadamente a 2 1035 anos. Assim, trata-se claramente de uma misso impossvel para o algoritmo de fora bruta baseado na anlise de cada permutao de vrtices.

PROBLEMAS DIFCEIS QUE TAMBM SO TEIS


O resultado da discusso acima pode parecer bastante desanimador: no parece haver bons mtodos para verificar a existncia de um circuito hamiltoniano e algoritmos de fora bruta s funcionam para problemas com pequeno nmero de vrtices ( bom que se diga que existe um meio termo: h estratgias que permitem resolver o problema acima para valores razoveis de n, reduzindo substancialmente o nmero de possibilidades a serem examinadas; mesmo estes algoritmos, no entanto, tornam-se imprticos a partir de um certo ponto). O mesmo ocorre com todos os chamados problemas NP-completos. No entanto, ao invs de ficarmos deprimidos com esta caracterstica desses problemas, podemos explor-la para uma importante finalidade em criptografia, que a parte da Matemtica que estuda mtodos para criar e decifrar cdigos. Para tal, tambm muito importante a assimetria apontada acima (e que ocorre em todos os problemas NP-completos): difcil encontrar um circuito hamiltoniano (ou mostrar que no existe um), mas fcil testar se uma seqncia de vrtices forma um circuito hamiltoniano. Suponhamos que voc seja cliente de um banco. Para ter acesso aos servios, voc usa o nmero de sua conta (que pblico) e uma senha, que em princpio deve ser conhecida apenas por voc. O procedimento mais simples seria ter a sua senha armazenada no sistema do banco. Mas a voc correria o risco de que ela fosse descoberta, por exemplo, por um funcionrio desonesto. Em lugar disto, o sistema do banco armazena uma verso codificada da senha, que no precisa ficar em segredo. Esta codificao deve ser feita de tal forma que seja simples verificar se sua senha est correta (para que voc seja autorizado a retirar dinheiro do caixa eletrnico), mas seja praticamente impossvel recuperar a senha a partir da verso codificada. Problemas NP-completos servem como uma luva para esta tarefa. Se quisssemos usar o problema do circuito hamiltoniano, poderamos agir mais ou menos da formadescrita a seguir. O cliente poderia escolher uma permutao dos nmeros de 1 a 50, conhecida apenas por ele. A partir desEUREKA! N 1, 1998

56

Sociedade Brasileira de Matemtica

sa informao, seria gerado um grafo, contendo necessariamente os arcos correspondentes ao circuito (os demais poderiam, por exemplo, ser gerados por um mtodo aleatrio, em que cada um dos possveis arcos teria uma certa probabilidade de sere escolhido). Este grafo seria armazenado no sistema. A figura a seguir mostra uma representao de uma permutao dos nmeros de 1 a 50 e um grafo, gerado aleatoriamente, que possui um ciclo hamiltoniano dado por esta permutao.

Fig. 4 Um ciclo hamiltoniano e um grafo gerado a partir dele


Quando o cliente fosse utilizar sua conta, o sistema simplesmente verificaria se a permutao apresentada corresponde a um caminho no grafo. Como improvvel que um tal ciclo pudesse ser encontrado para um grafo deste tamanho, dificilmente um impostor conseguiria se fazer passar pelo cliente, ainda que conhecesse o grafo-problema. Na prtica, so utilizados outros problemas NP-completos para se fazer codificao de senhas, mas a idia exatamente a mesma acima.

PALAVRAS FINAIS
Grafos so uma fonte inesgotvel de problemas com enunciado simples mas que escondem, muitas vezes, uma sofisticada estrutura matemtica. Neste artigo abordamos apenas alguns aspectos de dois desses problemas. Certamente voltaremos a falar em grafos em outros artigos desta revista. Para o leitor que deseja saber mais sobre o assunto, recomendamos os livros a seguir:

Jaime Luiz Szwarcfiter. Grafos e Algoritmos Computacionais. Editora Campus. Oynstein Ore. Graphs and Their Uses. The Mathematical Association of America.

EUREKA! N 1, 1998

57

Sociedade Brasileira de Matemtica

PROBLEMAS PROPOSTOS
Convidamos o leitor a enviar solues dos problemas propostos e sugestes de novos problemas para os prximos nmeros.
1) 2)
Mostre que, dado um conjunto de n pessoas, existem duas que possuem o mesmo nmero de amigos entre as pessoas do conjunto. Em uma pista circular h postos de gasolina, e o total de gasolina que h nos postos exatamente o suficiente para um carro dar uma volta. Prove que existe um posto de onde um carro com o tanque inicialmente vazio pode partir e conseguir dar uma volta completa na pista (parando para reabastecer nos postos). Prove que existe n N tal que os ltimos 1000 dgitos de n1998 so iguais a 1. Escreva 1998 como soma de (um nmero arbitrrio de ) parcelas de modo que o produto das parcelas seja o maior possvel. Sejam a > 0 e P1P2P3P4P5 uma poligonal aberta contida em um dos semi-planos determinados pela reta P1 P5 . Prove que possvel escolher pontos P6 e P7 no plano com P5 P6 = a de modo que possvel ladrilhar o plano com infinitos ladrilhos congruentes ao heptgono P1P2P3P4P5P6P7.
6)

3)

4) 5)

Mostre que toda seqncia com n2 + 1 elementos possui uma subseqncia crescente com n + 1 elementos ou uma subseqncia decrescente com n + 1 elementos. Prove que

7) 8)

1 + 2 + 3 + ... + 1998 < 2

Considere um torneio de xadrez envolvendo brasileiros e argentinos em que cada jogador joga contra todos os outros exatamente uma vez. Ao final do torneio, cada jogador obteve metade dos pon-

EUREKA! N 1, 1998

58

Sociedade Brasileira de Matemtica

tos que conquistou jogando contra brasileiros e metade jogando contra argentinos. Prove que o nmero total de jogadores do torneio um quadrado perfeito (obs: cada vitria vale 1 ponto, empate 1/2 ponto e derrota 0 ponto).

9)

Prove que todo nmero racional positivo pode ser escrito como soma de um certo nmero de fraes distintas de numerador 1.

Voc sabia que


2 5 5 1 4

cos


16

cos

2 17

1 + 17 + 34 2 17 + 2 17 + 3 17 34 2 17 2 34 + 2 17

mas no possvel escrever cos

2 7

e cos

2 9

usando radicais reais

EUREKA! N 1, 1998

59

Sociedade Brasileira de Matemtica

AGENDA OLMPICA
IV OLIMPADA DE MAIO 09 de maio, 14 h OLIMPADA DO CONE SUL 13 a 21 de junho de 1998 Salvador BA. OLIMPADA BRASILEIRA DE MATEMTICA Primeira Fase Sbado, 6 de junho Segunda Fase Sbado, 12 de setembro Terceira Fase Sbado, 24 de outubro (nveis 1,2 e 3) Domingo, 25 de outubro (nvel 3). 39 a. OLIMPADA INTERNACIONAL DE MATEMTICA 10 a 21 de julho Taiwan. OLIMPADA IBEROAMERICANA DE MATEMTICA 13 a 20 de setembro de 1998 Repblica Dominicana.

EUREKA! N 1, 1998

60

Sociedade Brasileira de Matemtica

COORDENADORES REGIONAIS
Alberto Hassen Raad Antnio C. Rodrigues Monteiro Amarsio da Silva Arajo Angela Camargo Antnio C. do Patrocnio Benedito T. Vasconcelos Freire Carlos A. Bandeira Braga Claudio Arconcher lio Mega Florncio F. Guimares F. Francisco Dutenhefner Gisele de A. Prateado G. Joo B. de Melo Neto Jos Carlos Pinto Leivas Jos Paulo Carneiro Jos Vieira Alves Leonardo Matteo D'orio Luzinalva M. de Amorim Marco Polo Marcondes Cavalcante Frana Mario Jorge Dias Carneiro Ma-To-F Pablo Rodrigo Ganassim Paulo H. Cruz Neiva de L. Jr. Reinaldo Gen Ichiro Arakaki Ricardo Amorim Sergio Claudio Ramos (UFJF) (UFPE) (UFV) (Esc. Tec. Hermann Hering) (IMECC/UNICAMP) (UFRGDN) (UFPB) (Col. Leonardo da Vinci) (Col. ETAPA) (UFES) (UFMG ) (UFGO) (UFPI) (URG) (USU) (UFPB) (Parque de Material Aeronutico de Belm) (UFBA) (Colgio Singular) (UF Cear) (UFMG) (UEM) (L. Albert Einstein) (Esc. Tec.Everardo Passos) (INPE) (Centro Educ. Logos) (IM-UFRGS) Juiz de Fora-MG Recife-PE Viosa-MG Blumenau-SC Campinas-SP Natal-RN Joo Pessoa-PB Jundia-SP So Paulo-SP Vitria-ES BH-MG Goinia-GO Teresina-PI Rio Grande-RS Rio de Janeiro-RJ Campina Grande-PB Belm-PA L. de Freitas-BA Santo Andr-SP Fortaleza-CE BH-MG Maring-PR Rio das Pedras-SP Piracicaba-SP S.J.Campos-SP Nova Iguau-RJ Porto Alegre-RS

EUREKA! N 1, 1998

61

CONTEDO
AOS LEITORES OLIMPADA BRASILEIRA DE MATEMTICA Problemas de treinamento para a Segunda Fase XIX OLIMPADA BRASILEIRA DE MATEMTICA Problemas Jnior Segunda Fase e Solues IV OLIMPADA DE MAIO Resultados IV OLIMPADA DE MAIO Prova 9a. OLIMPADA DE MATEMTICA DO CONE SUL 9a. OLIMPADA DE MATEMTICA DO CONE SUL Problemas e solues 39a. OLIMPADA INTERNACIONAL DE MATEMTICA Resultados e problemas 2 4 10 16 17 21 22 30

ARTIGOS
PARIDADE Eduardo Wagner OS PROBLEMAS DO VISITANTE MATEMTICO 32 39

DIVISIBILIDADE, CONGRUNCIAS E ARITMTICA MDULO n 41 Carlos Gustavo Moreira SOLUES DE PROBLEMAS PROPOSTOS EUREKA N1 PROBLEMAS PROPOSTOS AGENDA OLMPICA COORDENADORES REGIONAIS 53 59 61 62

Sociedade Brasileira de Matemtica

AOS LEITORES
Iniciamos este segundo nmero da revista EUREKA! transmitindo aos leitores nossa satisfao pela acolhida do primeiro nmero por alunos e professores. A comunidade estudantil e os professores das escolas passam a ter, de forma que esperamos permanente, uma publicao especfica que, alm de fornecer material para tornar as aulas mais ricas e interessantes, um veculo de contato entre todos para expor experincias, dirimir dvidas e nos aproximarmos cada vez mais. J estamos recebendo correspondncia de muitos alunos e alguns professores com respeito s solues dos problemas propostos. Isto muito nos alegra e temos a certeza de que nos prximos nmeros essa correspondncia s tender a crescer. Entretanto, gostaramos de pedir aos professores que nos enviem tambm colaboraes para os nmeros seguintes da revista: problemas interessantes com solues, pequenos artigos, experincias em sala de aula, olimpadas ou torneios regionais, enfim, material que seja adequado aos alunos da 5 srie do ensino fundamental ltima srie do ensino mdio. Estas colaboraes sero fundamentais para que nossa revista permanea viva e seja sobretudo til a toda a comunidade.

A Olimpada Brasileira de Matemtica de 1998 Realizamos a primeira fase da Olimpada Brasileira de Matemtica em mais de mil colgios do nosso pas. Em nosso projeto pretendamos atingir, nesta primeira etapa dessa nova atividade, cerca de 20 000 alunos mas, para nossa surpresa, esse nmero j superou o dobro do pretendido. Atravs dos relatrios enviados pelas escolas aos Coordenadores Regionais, estabelecemos as notas de corte para a promoo dos alunos segunda fase que se realizar em setembro. A terceira fase, j mais centralizada, ser feita em outubro e esperamos que no final de novembro possamos divulgar a lista dos alunos premiados. Como em toda competio, natural que o nmero de premiados seja relativamente pequeno em relao ao nmero inicial de participantes. Porm, aqui no h perdedores. Todos so de alguma forma ganhadores: de
EUREKA! N 2, 1998

Sociedade Brasileira de Matemtica

uma experincia nova, de um estmulo para estudar mais e crescer, ou da possibilidade de ver que objetivos que pareciam longnquos realmente podem ser atingidos. Devemos ainda relatar que alguns colgios no participaram da Olimpada Brasileira de Matemtica com receio de que, sem uma preparao adequada, seus alunos no tivessem um resultado satisfatrio. Especialmente para estes colgios enviamos nossa mensagem final: A Olimpada Brasileira de Matemtica no uma competio entre colgios. A OBM tem como objetivo principal estimular o estudo de Matemtica entre os jovens, desenvolver professores e propiciar uma melhoria do ensino e do aprendizado desta matria nas escolas brasileiras.

Comit Editorial.

EUREKA! N 2, 1998

Sociedade Brasileira de Matemtica

OLIMPADA BRASILEIRA DE MATEMTICA


Problemas de treinamento para a Segunda Fase

Primeiro Nvel
1)

Determine o menor inteiro cuja representao decimal consiste somente de 1's e que divisvel pelo nmero 333333 formado por 100 algarismos iguais a 3. (Problema proposto por Antonio Luiz Santos.) Numa gaveta h 6 meias pretas e 6 meias brancas. Qual o nmero mnimo de meias a se retirar (no escuro) para garantir que: As meias retiradas contenham um par da mesma cor? As meias retiradas contenham um par de cor branca? Quando se escrevem os nmeros 1, 2, 3, 4, 5, 6, 7, 8, 9, 10, 11, 12,1998, qual o dgito que ocupa o lugar 1998?

2)

a) b)
3)

Segundo Nvel
1)

Determine com quantos zeros consecutivos termina representao decimal do nmero 1 2 3 1998.

2)

Suponha que desejamos saber de qual janela de um prdio de 36 andares seguro jogarmos ovos para baixo, de modo que os ovos no se quebrem ao atingirem o cho. Para tal, admitimos que: Um ovo que sobrevive a uma queda pode ser usado novamente. Um ovo quebrado deve ser descartado. O efeito da queda o mesmo para todos os ovos. Se um ovo se quebra quando jogado de uma certa janela ento ele quebrar se jogado de uma altura superior. Se um ovo sobrevive a uma queda ento ele sobreviver a uma queda menor.

EUREKA! N 2, 1998

Sociedade Brasileira de Matemtica

No se sabe se da janela do primeiro andar os ovos quebram, e tambm no se sabe se da janela do ltimo andar os ovos quebram.

Se temos apenas 1 ovo e queremos ter certeza de obter um resultado correto, o experimento deve ser guiado apenas por um nico caminho: jogue o ovo pela janela do primeiro andar; se no se quebrar, jogue o ovo pela janela do segundo andar. Continue at que o ovo se quebre. Na pior das hipteses, este mtodo necessitar de 36 lanamentos para ser concludo. Suponha que 2 ovos esto disponveis. Qual o menor nmero de lanamentos de ovos necessrios para garantir todos os casos?
3)

Considere cinco pontos quaisquer P1, P2, , P5 no interior de um quadrado de lado 1. Mostre que pelo menos uma das distncias dij entre Pi e Pj menor que
2/2.

Terceiro Nvel
1)

Determine quantos nmeros naturais menores que 1998 tm um nmero mpar de divisores positivos. Mostre que, dados 5 pontos do plano em posio geral (isto trs pontos quaisquer nunca esto em linha reta), h 4 que formam um quadriltero convexo. Dois discos A e B so divididos em 2n setores iguais. No disco A, n setores so pintados de azul e n de vermelho. No disco B, os setores so pintados de azul ou vermelho de forma completamente arbitrria. Mostre que A e B podem ser superpostos de modo que pelo menos n setores tenham cores coincidentes.

2)

3)

EUREKA! N 2, 1998

Sociedade Brasileira de Matemtica

Solues do Primeiro Nvel


1)
claro que d = 333...333 = 3 111...111 = 3n . Portanto, o nmero
100 trs

100 uns

procurado N = 111...111 deve ser divisvel por n e por 3 (n no divisvel


k uns

por 3 porque a soma dos seus algarismos igual a 100 que no divisvel por 3). Se k um nmero da forma k = 100q + r onde 0 r < 100 ento obviamente N = 111...111 000...00+ 111...11 = M + R . Como M
100 q uns r zeros r uns

divisvel por n ento, N divisvel por n se, e somente se, R = 0 ou seja, se r = 0 e conseqentemente se, e somente se, k for divisvel por 100. Se k = 100q ento a soma dos algarismos de N igual a 100q e esta soma ser divisvel por 3 (e consequentemente tambm o nmero N)se, e somente se, q for divisivel por 3. Portanto, o menor nmero N = 111...111 divisvel
k uns

por d consiste em 300 uns.

2)
a) 3 meias (necessariamente teremos 2 meias brancas ou 2 meias pretas; se tirarmos apenas duas pode ser que uma seja branca e a outra preta). 8 meias (se tirassemos apenas 7 meias poderiam ser 6 pretas e apenas uma branca).

b)

3)
Quando se escrevem os nmeros do 1 ao 99, usam-se 9 + 2 (99 9) = 189 dgitos. Ficam por preencher 1809 (1998 189) lugares. Para cada uma das centenas que seguem usam-se 300 dgitos. Como 1809 = 300 6 + 9, ao terminar de escrever os 1998 dgitos se escrevem todos os nmeros desde o nmero 1 at completar 7 centenas (do nmero 1 at 699) e 9 dgitos mais: 700, 701 e 702. Portanto o dgito que ocupa o lugar 1998 o nmero 2.

EUREKA! N 2, 1998

Sociedade Brasileira de Matemtica

Solues do Segundo Nvel


1)
A resposta 496. Se a decomposio de 1 2 1998 em fatores primos 2a 3b 5c, temos necessariamente c < a, pois para todo r natural h mais mltiplos de 2r que de 5r entre 1 e 1998. Assim, o nmero de zeros do final de 1 2 1998 igual a c. Para determinar c, observamos que entre 1 e 1998 h 399 mltiplos de 5 (pois 399 5 < 1998 < 400 5), 79 mltiplos de 25, 15 mltiplos de 125, 3 mltiplos de 625 mas nenhum mltiplo de 3125, e portanto temos c = 399 + 79 + 15 + 3 = 496. (De fato, ao contar os mltiplos de 5, que so 399, j contamos os mltiplos de 25, mas estes devem ser contados pelo menos em dobro para calcular o exponente de 5, por isso somamos 79, mas preciso contar os mltiplos de 125 pelo menos 3 vezes e s foram contados 2 vezes, por isso somamos 15. E assim por diante.)

2)
8 lanamentos. Jogamos o primeiro ovo do oitavo andar. Se quebrar, basta testar os 7 primeiros com o segundo ovo. Se no quebrar, o jogamos do 15., depois do 21., depois do 26., depois do 30., depois do 33., depois do 35. e finalmente do 36.. Se ele quebrar por exemplo quando jogado do 26. andar, basta testar o segundo ovo nos andares 22, 23, 24 e 25, para o que gastamos 4 + 4 = 8 lanamentos. A escolha dos andares se devem a 8 + 7 = 15, 8 + 7 + 6 = 21, 8 + 7 + 6 + 5 = 26, 8 + 7 + 6 + 5 + 4 = 30, 8 + 7 + 6 + 5 + 4 + 3 + 2 + 1 = 36. O resultado no pode ser melhorado, pois se o primeiro ovo quebra no n-simo lanamento, devemos testar com o ovo restante todos os andares entre os usados nos (n 1)-simo e n-simo lanamentos, no pior caso. Tente generalizar este problema fazendo variar o nmero de ovos disponveis e o nmero de andares do prdio.

EUREKA! N 2, 1998

Sociedade Brasileira de Matemtica

3)
Dividimos o quadrado em 4 quadrados de lado 1/2. Necessariamente dois desses pontos, digamos Pi e Pj, estaro num mesmo quadradinho, e sua distncia dij ser menor que a diagonal do quadradinho (que a maior 2 distncia possvel entre dois de seus pontos), ou seja . 2

Solues do Terceiro Nvel


1)
1 2 Se n = p1 p 2 ... p k k a fatorao em primos de n, os divisores positivos 2 de n so todos os nmeros da forma p11 p 2 ... p k k com 0 1 1, 0 2

2, , 0 k k, i N, i. Assim , o nmero de divisores positivos de n (1 + 1) (1 + 2)(1 + k ). Para que este nmero seja mpar necessrio e suficiente que todos os i sejam pares, ou seja, que n seja quadrado perfeito. Como 442 = 1936 < 1998 < 2025 = 452, h 44 quadrados perfeitos entre 1 e 1998, portanto h 44 naturais menores que 1998 com um nmero mpar de divisores positivos. 2)
Se o menor polgono convexo que contm os 5 pontos tiver mais de 3 lados o problema trivial. Caso contrrio, dois dentre os 5 pontos (digamos D e E), esto dentro do tringulo cujos vrtices so os outros 3. Ao prolongar a reta que une esses dois pontos cortamos dois dos lados do tringulo, digamos AB e AC. Nesse caso, fcil ver que o quadriltero BDEC convexo.

EUREKA! N 2, 1998

Sociedade Brasileira de Matemtica

3)
Sejam S1, S2, , S2n os setores do disco B. Tentamos colocar o disco A sobre o disco B nas 2n posies possveis (com os setores coincidindo). Para cada i com 1 i 2n , em exatamente n das posies do disco A o setor Si ter cor coincidente com o setor do disco A que est sobre ele. Assim, o nmero mdio de setores com cores coincidentes nos dois discos para as 2n posies do disco A 2n n/2n = n, e necessariamente h posies do disco A para as quais h pelo menos n setores com cores coincidentes.

2fisdufiows uhf

Voc sabia

que h tantos nmeros racionais quanto

nmeros naturais, mas h estritamente mais nmeros reais que racionais (isto , existe uma bijeo f : N Q mas no existe nenhuma bijeo g : Q R) ? E que impossvel decidir se existe algum conjunto com estritamente mais elementos que os naturais mas estritamente menos elementos que os reais ??

EUREKA! N 2, 1998

Sociedade Brasileira de Matemtica

XIX OLIMPIADA BRASILEIRA DE MATEMTICA


Problemas Jnior Segunda Fase e Solues
A Olimpada Brasileira Jnior correspondia aproximadamente aos atuais nveis 1 e 2 da OBM. Estamos publicando a prova da segunda fase jnior do ano passado com solues, a qual acreditamos ser bom material de treinamento tanto para a segunda fase da OBM quanto para a terceira nos nveis 1 e 2. No prximo nmero da EUREKA! publicaremos a segunda fase da Olimpada Brasileira snior do ano passado.
PROBLEMA 1

No edifcio mais alto de Terra Brasilis moram Eduardo e Augusto. O nmero do andar do apartamento de Eduardo coincide com o nmero do apartamento de Augusto. A soma dos nmeros dos apartamentos dos dois 2164. Calcule o nmero do apartamento de Eduardo sabendo que h 12 apartamentos por andar. (Por exemplo, no primeiro andar esto os apartamentos de 1 a 12, no segundo, de 13 a 24, e assim por diante.)
PROBLEMA 2

A professora de Matemtica props o seguinte problema para seus alunos: "Marquem 6 pontos sobre uma circunferncia. Eu quero que vocs pintem o maior nmero de cordas determinadas por estes pontos, de modo que no existam quatro dos pontos sobre a circunferncia determinando um quadriltero com todos os lados e diagonais coloridos." a) Edmilson encontrou uma soluo correta colorindo 12 cordas. Exiba uma maneira de como fazer isto. b) Gustavo afirmou ter encontrado uma soluo na qual pintara 13 cordas. Mostre que a soluo de Gustavo no est correta.

EUREKA! N 2, 1998

10

Sociedade Brasileira de Matemtica

PROBLEMA 3

Sejam ABCD um quadrado, M o ponto mdio de AD e E um ponto sobre o lado AB. P a interseo de EC e MB. Mostre que a reta DP divide o segmento EB em dois segmentos de mesma medida.
PROBLEMA 4

Mostre que existem infinitos inteiros positivos n satisfazendo simultaneamente as seguintes condies: n mpar; i. n possui exatamente 1200 divisores positivos; ii. existem exatamente 1997 tringulos retngulos, dois a dois no iii. congruentes, de lados inteiros e n como medida de um dos catetos.
PROBLEMA 5

Seja n 1 um inteiro. Temos n lmpadas alinhadas e numeradas, da esquerda para a direita, de 1 a n. Cada lmpada pode estar acesa ou apagada. A cada segundo, determina-se a lmpada apagada de maior nmero e inverte-se o estado desta (de acesa para apagada ou de apagada para acesa) e das lmpadas posteriores (as lmpadas de maior nmero). a) b) Mostre que em algum momento todas as lmpadas estaro acesas (e o processo se encerrar). Suponha que inicialmente todas as lmpadas estejam apagadas. Determine depois de quantos segundos todas as lmpadas estaro acesas. Suponha agora n = 11 e que no incio somente as lmpadas de nmeros 6, 7 e 10 estejam acesas. Mostre que aps exatamente 1997 segundos todas as lmpadas estaro acesas.

c)

EUREKA! N 2, 1998

11

Sociedade Brasileira de Matemtica

SOLUES 1)

Seja a o andar do apartamento de Eduardo. Ento o nmero de seu apartamento 12 (a 1) + b, com 1 b 12. Da,

a + 12 ( a 1 ) + b = 2164, b = 2176 13a 1 2176 13a 12 a = 167, b = 5


Portanto, o nmero do apartamento de Eduardo : 12 (a 1) + b = 12 166 + 5 = 1997.
2)

a) Uma maneira mostrada abaixo:

b) Suponha que a soluo de Gustavo esteja correta. Sejam A, B, C, D, E, F os pontos. Ento, como os 6 pontos determinam 15 cordas, somente dois segmentos no foram coloridos. Estes dois segmentos incidem em 3 ou 4 vrtices. i.) ii.) Se A vrtice comum de dois segmentos no coloridos, AB e AF, ento caso existem 6 quadrilteros totalmente coloridos: ACDE, BCDE, BCDF, BCEF, BDEF e CDEF. Se os segmentos AB e EF no foram coloridos ento existem 4 quadrilteros coloridos: CDAE, CDAF, CDBE, CDBF.

EUREKA! N 2, 1998

12

Sociedade Brasileira de Matemtica

3)

E P

Prolongue BM at encontrar o prolongamento do lado CD no ponto N. Claramente, AMB DMN , donde segue que AB = DN . Portanto, D o ponto mdio de CN. O resultado segue observando que os tringulos CPN e EPB so semelhantes e, como PD mediana do tringulo CPN, conclui-se que o prolongamento de DP encontra EB em seu ponto mdio.
4)

Seja n um nmero natural mpar. Vamos calcular o nmero de tringulos retngulos de lados inteiros nos quais n medida de um dos catetos. Para isso, devemos ter

n2 + x2 = y2 , n 2 = ( y x)( y + x),
com x e y inteiros positivos, x < y. Observe que (y x) < (y + x). Se fizermos (y x) = d, com d um divisor de n2, d ser menor que n e (y + x) = n2/d ser maior que n. Para qualquer d satisfazendo estas condies, podemos encontrar uma soluo:

EUREKA! N 2, 1998

13

Sociedade Brasileira de Matemtica

1 n2 x = d 2 d y x = d n2 y + x = 1 n2 d = +d y 2 d
Estas solues so inteiras e positivas, pois n mpar (logo d tambm), e d < n . Portanto, o nmero de tringulos retngulos o nmero de divisores de n2 menor que n. Mas para cada divisor de n2 menor que n, corresponde um divisor maior que n. Lembrando que n tambm um divisor, conclumos que o nmero procurado 1/2 (d(n2) 1), onde d(n) o nmero de divisores positivos de n. Portanto, necessrio e suficiente que n2 seja um nmero mpar com d(n2) = 2 1997 + 1 = 3995 divisores. Uma das vrias possibilidades para n2 ter 3995 divisores ser da forma p4q798, com p e q primos distintos. Neste caso, n = p2q399, possui d(n) = (2 +1) (399 +1) = 1200 divisores.
5)

Vamos representar por 1 uma lmpada acesa, e por 0 uma lmpada apagada e interpretar o nmero obtido na base 2. Veja que se, em algum passo, o ltimo dgito for 0, ele ser o nico dgito alterado no prximo passo. Isto significa que o nmero aumentar 1 unidade. Caso contrrio, o nmero terminar com um bloco de 1's antecipado por um 0:0111. No prximo passo, o nmero ser 1000. Mas observe que (0111) + 1 = 1000. Portanto, em qualquer caso, o nmero k sucedido pelo nmero k + 1. a) Dada qualquer disposio inicial das lmpadas, ou seja, qualquer nmero binrio de no mximo n dgitos, em algum momento todos os dgitos sero iguais a 1, pois este o maior nmero de n dgitos na base 2.
EUREKA! N 2, 1998

14

Sociedade Brasileira de Matemtica

b) Existem 2n nmeros de no mximo n dgitos na base 2. Comeando com 0, devemos chegar a 2n1, passando por todos os naturais intermedirios. So necessrios, ento, 2n1 segundos. c) Observe que a configurao inicial representa o nmero 25 + 24 + 2 = 50. Para n = 11, todas as lmpadas estaro acesas depois de (2111) 50 = 1997 segundos.

Voc sabia

Que um polgono regular com

um nmero mpar de lados s pode ser construdo exatamente com rgua e compasso se o nmero de lados for um produto de primos distintos da forma 2 + 1 (esses primos so chamados primos de Fermat) ? E que s so conhecidos 5 primos de Fermat: 3, 5, 17, 257 e 65537, apesar de Fermat ter conjecturado que todo nmero da forma (isso j falso para k = 5 : 2
32

2k

+ 1 divisvel por 641.) ??

22 + 1

primo

EUREKA! N 2, 1998

15

Sociedade Brasileira de Matemtica

IV OLIMPADA DE MAIO
Resultados Primeiro nvel
Fabio Dias Moreira Davi M. Alexandrino Nogueira Lyussei Abe Cibele Norie Sakai Uyhara Pedro Davoli Ometto Kelly Correa de Paula Marcelo Kenji Honda Rafael Martins Gomes Nascimento Priscila Carrara Thiago Pimentel Nykiel Rodrigo Evangelista Delgado Luiz Eduardo de Godoi Ouro Prata Prata Prata Bronze Bronze Bronze Bronze Meno Meno Meno Meno Coord. Est. Militar Etapa Integrado Koelle M.Schledorn Pioneiro S. Dumont Cass. Ricardo Militar Militar Cass. Ricardo Rio de Janeiro-RJ Fortaleza-CE So Paulo-SP Itatiba-SP Rio Claro-SP Jundia-SP So Paulo-SP Fortaleza-CE S. J. Campos-SP Juiz de Fora-MG Juiz de Fora-MG S. J. Campos-SP

Segundo nvel
Hugo Pinto Iwata Ulisses Medeiros de Albuquerque Afonso de Paula P. Rocha Artur D. Nelmi Luiz Fernando Mendes Correa Andre de Almeira Bosso Fabricio Siqueira Benevides Luiz Brizeno Firmeza Neto Daniel Nobuo Uno Juliana Regina C. Zucare Ouro Prata Prata Bronze Bronze Bronze Bronze Meno Meno Meno SETA Militar S. Dumont Bandeirantes Militar Progresso 7 de setembro Evolutivo Etapa Bandeirantes S.J.Rio Preto-SP Fortaleza-CE Fortaleza-CE So Paulo-SP Juiz de Fora-MG Araraquara-SP Fortaleza-CE Fortaleza-CE So Paulo-SP So Paulo-SP

Os alunos Fabio Dias Moreira (Rio de Janeiro, RJ) e Hugo Pinto Iwata (So Jos do Rio Preto, SP) receberam medalha de ouro na Olimpada e com isso, ganharam uma viagem de uma semana para a Argentina onde se reuniro com os ouros dos outros pases para diversas atividades tursticas e culturais. Esta viagem ser realizada em outubro, em data que ainda ser marcada. A seguir, publicamos a prova da IV Olimpada de maio, com as respostas dos problemas.

EUREKA! N 2, 1998

16

Sociedade Brasileira de Matemtica

IV OLIMPADA DE MAIO
Primeiro nvel
Durao da prova: 3 horas. Cada problema vale 10 pontos. No se pode usar mquina de calcular. No se pode consultar livros nem notas.

PROBLEMA 1

Com seis varetas se construiu uma pea como a da figura. As trs varetas exteriores so iguais entre si. As trs varetas interiores so iguais entre si. Desejase pintar cada vareta de uma cor s de modo que, em cada ponto de unio, as trs varetas que chegam tenham cores diferentes.As varetas s podem ser pintadas de azul, branco, vermelho ou verde. De quantas maneiras pode-se pintar a pea?
PROBLEMA 2

Tm-se 1998 peas retangulares de 2cm de altura e 3cm de comprimento e com elas se armam quadrados (sem superposies nem buracos). Qual a maior quantidade de quadrados diferentes que se pode ter ao mesmo tempo?
PROBLEMA 3

Existem quatro botes numa margem de um rio; seus nomes so Oito, Quatro, Dois e Um, porque essas so as quantidades de horas que cada um deles demora para cruzar o rio. Pode-se atar um bote a outro, porm no mais de um, e ento o tempo que demoram em cruzar igual ao do mais lento dos botes. Um s marinheiro deve levar todos os botes at outra margem do rio. Qual o menor tempo necessrio para completar o translado?

EUREKA! N 2, 1998

17

Sociedade Brasileira de Matemtica

PROBLEMA 4

ABCD um quadrado de centro O. Sobre os lados DC e AD foram construidos os tringulos equilteros DAF e DCE. Decida se a rea do tringulo EDF maior do que, menor do que ou igual rea do tringulo DOC.

D F A

O B

PROBLEMA 5

Escolha um nmero de quatro dgitos ( nenhum deles zero) e comeando com ele construa uma lista de 21 nmeros distintos, de quatro dgitos cada um, que satisfaa a seguinte regra: depois de escrever cada novo nmero da lista devem-se calcular todas as mdias entre dois dgitos desse nmero, descartando-se as mdias que no do um nmero inteiro, e com os que restam se forma um nmero de quatro dgitos que ocupar o lugar seguinte na lista. Por exemplo, se na lista se escreveu o nmero 2946, o seguinte pode ser 3333 ou 3434 ou 5345 ou qualquer outro nmero armado com os dgitos 3, 4 ou 5.

Segundo nvel
PROBLEMA 1

Ins escolheu quatro dgitos distintos do conjunto {1,2,3,4,5,6,7,8,9}. Formou com eles todos os possveis nmeros de quatro dgitos distintos e somou todos eses nmeros de quatro dgitos. O resultado 193314. Encontre os quatro dgitos que Ins escolheu.

EUREKA! N 2, 1998

18

Sociedade Brasileira de Matemtica

PROBLEMA 2

ABC um tringulo equiltero. N um ponto do lado AC tal que


AC = 7.AN , M um ponto do lado AB tal que MN paralelo a BC e P

um ponto do lado BC tal que MP paralelo a AC. Encontre a frao

rea( MNP) . rea( ABC )


PROBLEMA 3

Dado um tabuleiro quadriculado de 4 4, com cada casa pintada de uma cor distinta, deseja-se cort-lo em dois pedaos de igual rea mediante um s corte, que siga os lados das casas do tabuleiro. De quantas maneiras se pode fazer isto? Obs. Os pedaos em que se divide o tabuleiro devem ser peas inteiras; no devem ser desconectados pelo corte.
PROBLEMA 4

O cho do ptio tem desenhado um octgono regular. Emiliano escreve nos vrtices deste os nmeros de 1 a 8 em qualquer ordem. Deixa uma pedra no ponto 1. Caminha em direo ao ponto 2 e, havendo percorrido 1/2 do caminho, se detm e deixa a segunda pedra. Da caminha em direo ao ponto 3 e, havendo percorrido 1/3 do caminho, se detm e deixa a terceira pedra. Da caminha em direo ao ponto 4 e, havendo percorrido 1/4 do caminho, se detm e deixa a quarta pedra. Deste modo segue at que, depois de deixar a stima pedra, caminha em direo ao ponto 8, e havendo percorrido 1/8 do caminho, deixa a oitava pedra. A quantidade de pedras que ficarem no centro do octgono depende da ordem em que ele escreveu os nmeros nos vrtices. Qual a maior quantidade de pedras que podem ficar no centro?

EUREKA! N 2, 1998

19

Sociedade Brasileira de Matemtica

PROBLEMA 5

O planeta X31 tem s dois tipos de notas, mas o sistema no to mau j que s h quinze preos inteiros para os quais o pagamento no pode ser feito de forma exata (nesses casos deve-se pagar a mais e receber o troco). Se 18 um dos preos para os quais no se pode fazer pagamento exato, encontre o valor de cada tipo de nota.

RESPOSTAS IV OLIMPADA DE MAIO Primeiro nvel 1998


PROBLEMA 1: 16 formas. PROBLEMA 2 : 9 PROBLEMA 3: 15h PROBLEMA 4 : As reas so iguais. PROBLEMA 5 : H muitas solues.

IV OLIMPADA DE MAIO Segundo nvel 1998


PROBLEMA 1 : 5, 7, 8, e 9 PROBLEMA 2 : 6/49 PROBLEMA 3: 70 maneiras. PROBLEMA 4 : 4 pedras. PROBLEMA 5: 4 e 11

Voc sabia

Que a todo momento h dois

pontos antpodas na terra com a mesma temperatura e a mesma presso (admitindo que temperatura e presso dependem continuamente do ponto)??

EUREKA! N 2, 1998

20

Sociedade Brasileira de Matemtica

9a. OLIMPADA DE MATEMTICA DO CONE SUL


A 9. Olimpada de Matemtica do Cone Sul foi realizada em Salvador, BA, no perodo de 13 a 21 de junho de 1998. Esta Olimpada foi realizada pela segunda vez no pas (a primeira foi em 1993, em Petrpolis, RJ). Dela participaram alunos de at 15 anos dos seguintes pases: Argentina, Brasil, Bolvia, Chile, Paraguai, Peru e Uruguai. A organizao da Olimpada esteve a cargo da Professora Luzinalva Amorim, da Universidade Federal da Bahia. A equipe brasileira foi selecionada atravs de provas realizadas em maro e maio deste ano e foi liderada pelos professores Paulo Cezar Pinto Carvalho, do IMPA, e Florncio Ferreira Guimares, da UFES. A competio constou de duas provas, realizadas em dois dias, cada uma com trs problemas, valendo 10 pontos cada. Veja a seguir os resultados obtidos pela equipe brasileira e as provas da 9a. Olimpada de Matemtica do Cone Sul.

RESULTADOS DA EQUIPE BRASILEIRA BRA 1 BRA 2 BRA 3 BRA 4 Mila Lopes Viana Pedro Paulo Gouveia Fabricio Siqueira Benevides Jnathas Digenes Castelo Branco Bronze Prata Prata Bronze

Voc sabi@

que a

Olimpada Brasileira de Matemtica tem pgina web?? Visite-nos no endereo eletrnico

http://www.obm.org.br

EUREKA! N 2, 1998

21

Sociedade Brasileira de Matemtica

9a. OLIMPADA DE MATEMTICA DO CONE SUL


Problemas e solues Primeiro dia. Tempo: 4 horas 30 min.
PROBLEMA 1

So dados 98 cartes. Em cada um deles est escrito um dos nmeros 1, 2, 3, , 98 (no existem nmeros repetidos). Pode-se ordenar os 98 cartes de tal modo que ao considerar dois cartes consecutivos a diferena entre o nmero maior e o nmero menor escritos neles seja sempre maior que 48. Indicar como e de quantas formas possvel efetuar a ordenao.
PROBLEMA 2

Sejam H o ortocentro (interseo das alturas) do tringulo acutngulo ABC e M o ponto mdio do lado BC. Seja X o ponto em que a reta HM intersecta o arco BC (que no contm A) da circunsferncia circunscrita a ABC. Seja Y o ponto de interseo da reta BH com a circunsferncia, distinto de B. Demonstre que XY = BC.
PROBLEMA 3

Prove que, pelo menos para 30% dos naturais n entre 1 e 1.000.000, o primeiro dgito de 2n 1.

Segundo dia. Tempo: 4 horas 30 minutos.


PROBLEMA 4

Determine todas as funes f tais que

f ( x 2 ) f ( y 2 ) + 2 x + 1 = f ( x + y) f ( x y)
quaisquer que sejam os nmeros reais x, y.

EUREKA! N 2, 1998

22

Sociedade Brasileira de Matemtica

PROBLEMA 5

Em Terra Brasilis existem n casas onde vivem n duendes, cada um em uma casa. Existem estradas de mo nica de tal modo que:

cada estrada liga duas casas; em cada casa comea exatamente uma estrada; em cada casa termina exatamente uma estrada.

Todos os dias, a partir do dia 1, cada duende sai da casa onde est e chega casa vizinha. Uma lenda de Terra Brasilis diz que, quando todos os duendes regressarem posio original, o mundo acabar. (a) (b) Demonstre que o mundo acabar. Se n = 98, demonstre que possvel que os duendes construam e orientem as estradas de modo que o mundo no se acabe antes de 300.000 anos.

PROBLEMA 6

O Prefeito de uma cidade deseja estabelecer um sistema de transportes com pelo menos uma linha de nibus, no qual: (i) (ii) (iii) cada linha passe exatamente por trs paradas; cada duas linhas distintas tenham exatamente uma parada em comum; para cada duas paradas de nibus distintas exista exatamente uma linha que passe por ambas.

Determine o nmero de paradas de nibus da cidade.

EUREKA! N 2, 1998

23

Sociedade Brasileira de Matemtica

SOLUES 1)

Vamos provar uma verso um pouco mais geral do problema: Seja k um nmero natural. Encontrar todas as permutaes a1, a2, a2k dos nmeros 1, 2, , 2k que verificam ai ai +1 k para todo i = 1, 2, , 2k 1.

Soluo Em primeiro lugar observamos que, se dois nmeros entre 1, 2, , 2k diferem de pelo menos k, ento o maior dos nmeros est entre k + 1, k + 2, 2k e o menor, entre 1, 2, , k . Chamemos simplesmente os nmeros destes dois conjuntos de "grandes" e "pequenos", respectivamente. Suponhamos que a1, a2, a2k uma permutao com a propriedade em questo. Pelo que dissemos acima, seus termos com ndice mpar (par) devem ser todos grandes ou todos pequenos. Sejam, por exemplo, a1, a3, a2k 1 pequenos e a2, a4, a2k grandes. Consideremos a soma

S = a1 a 2 + a 2 + a3 + ... + a 2 k 2 a 2 k 1 + a 2 k 1 a 2 k
Como cada termo de ndice par maior do que seus vizinhos,

S = (a 2 a1 ) + (a 2 a3 ) + ... + (a 2 k 2 a 2 k 1) + (a 2 k a 2 k 1 )
= 2(a 2 + a 4 + ... + a 2 k ) 2(a1 + a3 + ...a 2 k 1 ) a 2 k + a1 = 2((k + 1) + (k + 2) + ... + 2k ) 2(1 + 2 + ... + k ) a 2 k + a1

= 2k 2 (a 2 k a1 )
Notemos que a condio ai ai +1 k determina a escolha de a1 e a2k . Os nicos vizinhos possveis de k e k +1 so 2k e 1, respectivamente. Logo k e k+1 devem ser o primeiro e o ltimo termos da permutao. E como escolhemos comear com a1 pequeno, a1 = k, a2k = k +1. Ento a2 = 2k, a2k 1 = 1.
EUREKA! N 2, 1998

24

Sociedade Brasileira de Matemtica

Regressando soma S, vemos que ela igual a 2k2 (( k +1) k ) = 2k2 1. Por outro lado, cada dois somandos da forma ai 1 ai + ai ai +1 contribui com pelo menos k + ( k + 1) = 2k + 1. Isto se deve ao fato de ser impossvel que

ai 1 ai = ai ai +1 = k , pois

teramos, neste caso, ai 1 = ai + 1. Assim, temos

S (2k + 1) + (2k + 1) + ... + (2k + 1) + k = 2k 2 1


k 1

Ento ai 1 a i + a i a i +1 = 2k + 1 para todo i = 1, 3, 2k 1. Isto , as diferenas consecutivas so k, k + 1, k, k + 1, , k. Comeando com a1 = k, a2 = 2k (que diferem em k ), podemos determinar todos os ai da seqncia:

a 3 = 2k ( k + 1) = k 1 a 4 = ( k 1) + k = 2k 1 a 5 = ( 2k 1) ( k + 1) = k 2


Portanto a1, a2 , , a2k

k, 2k, k 1, 2k 1, k 2, , 2, k + 2, 1, k + 1
Por simetria, existe exatamente uma soluo alm desta: a que obtemos tomando a soluo acima na ordem inversa.

EUREKA! N 2, 1998

25

Sociedade Brasileira de Matemtica

2)

Seja {L} = PM AH. Mostraremos que L coincide com H. Inicialmente, observe que NPA = 90 (pois AM dimetro). Prolonguemos PM at encontrar a circunsferncia circunscrita no ponto N, diametralmente oposto ao vrtice A (pois NPA = 90.) Logo o circuncentro O o ponto mdio de AN e, como OM AL, segue que M o ponto mdio de LN; como m ponto mdio de BC , segue que LBNC um paralelogramo, de modo que BL NC . Mas NCA = 90 (pois AN dimetro), ou seja, NC AC. Da segue que BL AC e, como AL BC, concluimos que L H.
3)

Vamos provar que para cada inteiro positivo k existe uma potncia de 2 com exatamente k dgitos (na base 10) e cujo primeiro dgito 1. De fato, se considerarmos a menor potncia de 2 maior que 10k + 1, devemos ter 2 n < 10 k + 1 2n + 1, ou 10 k + 1 2n + 1 < 2 10 k + 1 . Portanto basta calcular quantos dgitos possui 210 . Mas, de 103 < 210, obtemos 10 310 < 210 , donde segue que 210 tem mais de 300.000 algarismos e segue que no mnimo 300.000/1.000.000 = 30% de tais potncias comeam com o algarismo 1.
5 6
6

EUREKA! N 2, 1998

26

Sociedade Brasileira de Matemtica

Observaes:
1. Utilizamos somente que existe uma potncia de 2 que comea com o dgito 1 e possui exatamente k dgitos. Como verifica-se imediatamente, existe exatamente uma potncia de 2 com k dgitos que comea com 1. 2.

aplicando logaritmos, vem t < 106 log 2 < t + 1, donde t + 1 = 301.030.

210 possui exatamente 301.030 algarismos, pois se 10t < 210 < 10t+1,

3. Utilizando as idias de 1 e 2, possvel mostrar que a probabilidade de uma potncia de 2 comear com o algarismo 1 log 2. Mais precisamente, se f (n) o nmero de inteiros k (1 k n ) tais que 2k que iniciam com o algarismo 1, ento

f ( n) lim = log 2 0,301029995664. n n


4)

Fazendo x = y temos

f (2x) f (0)= 2x + 1
Logo, para x = 0, (f (0))2 = 1 f (0) = 1 Assim, f (2x) = (2x +1) e, portanto, f (x) = x + 1 ou f (x) = (x + 1). Substituindo as funes encontradas na equao funcional original, verificamos que apenas f (x) = x + 1 satisfaz as condies do problema.
5)

(a)

Numere os duendes de 1 a n e seja f(i) o vizinho do duende nmero i. A funo f claramente uma bijeo. Em algum momento cada duende retornar a sua casa pois a seqncia f (i), f ( f (i) ), f ( f ( f (i))),assume um nmero finito de valores, donde existiro inteiros positivos r < s tais que f s (i) = f r(i), portanto f s r (i) = i (pois f bijetora). Seja g(i) o menor inteiro positivo tal que o duende i retorna sua casa depois de g(i) dias. Depois de

EUREKA! N 2, 1998

27

Sociedade Brasileira de Matemtica

mmc(g(1), g (2),,g(n)) dias, todos os duendes retornaro posio original e o mundo acabar.
(b)

Divida os 98 duendes em 8 ciclos de tamanhos 3, 5, 7, 11, 13, 17, 19, 23 (98 = 3 + 5 + 7 + 11 + 13 + 17 + 19 + 23). Os duendes retornaro posio inicial depois de 3 5 7 11 13 17 19 23 = 111546435 > 366 300.000. Alternativamente, podemos dividir os duendes em ciclos de tamanhos 3, 8, 9, 5, 7, 11, 13, 19 e 23, e eles retornaro posio original em mmc (3, 8, 9, 5, 7, 11, 13, 19, 23) = 8 9 5 7 11 13 19 23 = 157.477.320

6)

Um exemplo de tal sistema aquele que tem uma s linha com exatamente 3 pontos. Para o que segue, suponhamos que haja pelo menos 4 pontos 1, 2, 3, 4 e que uma das linhas R1 = 123 (aqui, e no que segue, R = abc significa que a linha R passa pelos pontos a, b, c, no importando a ordem. Assim, por exemplo, R = bca a mesma linha.) Por (iii), devem existir linhas R2, R3 e R4 que passam pelos pares de pontos {1, 4}, {2, 4} e {3, 4}, respectivamente. Notemos que R2 , R3 e R4 devem ser distintas. De fato, se, digamos, os pares {2, 4} e {3, 4} esto na mesma linha R2 , ento R2 = 234, logo R1 e R2 tm duas paradas em comum e isto impossvel por (ii). Novamente por (ii), cada uma das linhas R2 , R3 e R4 tem exatamente um ponto em comum com R1 = 123. Como no podem haver dois pontos entre 1, 2, 3, que esto em R2 , R3 e R4 (novamente por (ii)), devemos ter R2 =14a, R3 = 24b, R4 = 34c para pontos distintos a, b, c que so por sua vez distintos de 1, 2, 3, 4. Para manter uma notao consistente, sejam a = 5, b = 6 e c = 7. Logo R2 = 145, R3 = 246 e R4 = 347. Com isso, provamos que h pelo menos 7 pontos. Agora, suponhamos que exista pelo menos um ponto a mais, digamos 8. Por (iii), existe uma linha S que passa por 1 e 8. Como S tem uma parada em comum com R3 = 246, conclumos que S = 128, S = 148 ou S = 168. Nenhuma destas possvel, pois 1. 2. as linhas 128 e 148 tm dois pontos em comum com R1 = 123 e R2 = 145, respectivamente. 168 no tem ponto em comum com R4 = 347.

EUREKA! N 2, 1998

28

Sociedade Brasileira de Matemtica

Esta contradio devida a termos suposto que existem mais de 7 pontos. Completamos a construo do sistema de transportes com 7 pontos de nibus. Devem haver linhas R5, R6, R7 por {1, 6}, {2, 5}, {3, 5}, respectivamente (pois as linhas no esto entre as j existentes R1 , R2 , R2 , R3 e R4). Pode-se verificar que a escolha R5 = 167, R6 = 257 e R7 = 356 funciona. As 7 linhas 123, 145, 246, 347, 167, 257, 356 formam um exemplo de tal sistema. Conclumos, ento, que a cidade pode ter exatamente 3 ou exatamente 7 pontos de nibus.

Observao:
Este problema equivalente a particionar as arestas de um grafo completo Kn em tringulos de modo que quaisquer dois tringulos tenham exatamente um vrtice em comum. Observando que, satisfeitas as condies do problema, cada vrtice de um tringulo comum a (n 3)/2 outros tringulos, o total de tringulos em Kn n 3 1 n( n 1) , o que s se verifica quando n = 3 ou n = 7. 1+ 3 = 2 3 2 Para concluir a resoluo, basta obter as parties nestes casos.

EUREKA! N 2, 1998

29

Sociedade Brasileira de Matemtica

39a. OLIMPADA INTERNACIONAL DE MATEMTICA


Resultados e problemas

No ms de tantas expectativas dos brasileiros, o Brasil consegue uma medalha de ouro na 39a. Olimpada Internacional de Matemtica realizada com a presena de 76 pases em Taiwan nos dias 10 a 21 de julho ltimo. O estudante Rui Lopes Viana Filho (SP) foi ganhador de uma medalha de ouro. Tambm foram premiados os estudantes Emanuel Carneiro (CE) medalha de bronze, Murali Vajapeyam (PB) meno honrosa e Mauricio Carrari (SP) meno honrosa. Trata-se de feito muito importante, visto que pases como Alemanha, Inglaterra, Israel, Sucia, Australia e muitos outros no conquistaram medalhas de ouro. Merece tambm elogios o fato da equipe brasileira ter sido uma das que tiveram melhor desempenho na questo 5 da prova, superando por exemplo, as equipes dos EUA e da Rssia. Veja a seguir as questes da 39a.Olimpada Internacional de Matemtica.

Primeiro dia Durao da Prova: 4 horas 30 min.


PROBLEMA 1

No quadriltero convexo ABCD, as diagonais AC e BD so perpendiculares e os lados opostos AB e DC no so paralelos. Sabemos que o ponto P, onde se intersectam as mediatrizes de AB e DC, est no interior de ABCD. Prove que ABCD um quadriltero inscritvel se, e somente se, os tringulos ABP e CDP tm reas iguais.
PROBLEMA 2

Numa competio, existem a concorrentes e b juzes, onde b 3 um inteiro mpar. Cada juiz avalia cada um dos concorrentes, classificando-o como "aprovado" ou "reprovado". Suponha que k um nmero tal que as
EUREKA! N 2, 1998

30

Sociedade Brasileira de Matemtica

classificaes dadas por dois juzes quaisquer coincidem no mximo para k k b 1 . concorrentes. Prove que a 2b
PROBLEMA 3

Para qualquer inteiro positivo n, seja d(n) o nmero de divisores positivos de n (incluindo 1 e n). Determine todos os inteiros positivos k tais que

d (n 2 ) = k para algum n. d ( n)

Segundo dia Durao da Prova: 4 horas 30 min.


PROBLEMA 4

Determine todos os pares (a, b) de inteiros positivos tais que ab2 + b + 7 divide a2b + a + b.
PROBLEMA 5

Seja I o incentro do tringulo ABC. A circunferncia inscrita no tringulo ABC tangente aos lados BC, CA e AB nos pontos K, L e M, respectivamente. A reta que passa por B, paralela ao segmento MK, intersecta as retas LM e LK nos pontos R e S, respectivamente. Prove que o ngulo RIS agudo.
PROBLEMA 6

Considere todas as funes f definidas no conjunto N dos inteiros positivos, com valores no mesmo conjunto, que satisfazem f (t 2 f ( s )) = s ( f (t )) 2 , para todos s e t em N. Determine o menor valor possvel de f(1998)

EUREKA! N 2, 1998

31

Sociedade Brasileira de Matemtica

PARIDADE
Eduardo Wagner Nvel Iniciante. Todo nmero natural par ou mpar.
Elementar, no? A afirmao acima, que uma das mais simples e bvias da Matemtica, tambm uma ferramenta de grande utilidade na resoluo de muitos problemas envolvendo nmeros naturais. Vamos comentar neste artigo alguns deles, em graus diferentes de dificuldade, mas inicialmente precisamos recordar trs importantes propriedades: a) a soma de dois nmeros pares par. b) a soma de dois nmeros mpares par. c) a soma de um nmero par com um nmero mpar mpar. Dizemos que dois nmeros inteiros tm mesma paridade, quando so ambos pares ou ambos mpares. Assim, podemos dizer que a soma de dois nmeros inteiros par se, e somente se, eles tm mesma paridade. Vamos aos problemas.
PROBLEMA 1

Em um quartel existem 100 soldados e, todas as noites, trs deles so escolhidos para trabalhar de sentinela. possvel que aps certo tempo um dos soldados tenha trabalhado com cada um dos outros exatamente uma vez?
RESPOSTA : No.

Escolha um soldado. Em cada noite em que trabalha, ele est em companhia de dois outros. Como 99 um nmero mpar, no podemos formar pares de soldados sempre diferentes para trabalhar com o escolhido.
PROBLEMA 2

Um jogo consiste de 9 botes luminosos (de cor verde ou vermelha) dispostos da seguinte forma:

EUREKA! N 2, 1998

32

Sociedade Brasileira de Matemtica

1 4 7

2 5 8

3 6 9

Apertando um boto do bordo do retngulo, trocam de cor ele e seus vizinhos (do lado ou em diagonal). Apertando o boto do centro, trocam de cor todos os seus 8 vizinhos porm ele no. Exemplos: Apertando 1, trocam de cor 1, 2, 4 e 5. Apertando 2, trocam de cor 1, 2, 3, 4, 5 e 6. Apertando 5, trocam de cor 1, 2, 3, 4, 6, 7, 8 e 9. Inicialmente todos os botes esto verdes. possvel, apertando sucessivamente alguns botes, torn-los todos vermelhos?
RESPOSTA : No possvel.

Observe que apertando um boto do vrtice do retngulo, trocam de cor 4 botes. Apertando um boto do meio de um lado, trocam de cor 6 botes e apertando um boto do centro trocam de cor 8 botes. Assim, cada vez que apertamos um boto trocam de cor um nmero par de botes. Como existem 9 botes, no possvel que todos troquem de cor.
PROBLEMA 3

Escrevemos abaixo os nmeros naturais de 1 a 10. 1 2 3 4 5 6 7 8 9 10.

Antes de cada um deles, coloque sinais + ou de forma que a soma de todos seja zero.
SOLUO: No possvel fazer isto. Imaginando que fosse possvel, deveramos separar os nmeros dados em dois grupos com a mesma soma. Ento colocaramos sinais negativos nos
EUREKA! N 2, 1998

33

Sociedade Brasileira de Matemtica

nmeros de um dos grupos e sinais positivos nos nmeros do outro. Teramos ento uma soma igual a zero. Acontece que a soma dos nmeros naturais de 1 a 10 igual a 55. Como este nmero mpar, no podemos separar os nmeros dados em dois grupos que tenham a mesma soma. Como o leitor deve estar percebendo, os argumentos utilizados permitiram concluir que as respostas dos trs problemas propostos foram iguais: no possvel fazer tal coisa. Na maioria das vezes, um argumento de paridade serve exatamente para isto. Mostrar que um determinado fato no pode ocorrer e isto no desanimador, muito pelo contrrio. Serve para nos convencer que no adianta ficar gastando tempo demais fazendo tentativas inteis. As experincias so valiosas no sentido de nos abrir os olhos para a possibilidade do problema no ter soluo e, a partir da, buscar um argumento que resolva definitivamente a questo. muito importante tambm explorar um problema, ou seja, imaginar pequenas modificaes no enunciado e verificar o que ocorre com sua resposta. Por exemplo, o problema 3 no tem soluo porque a soma dos naturais de 1 at 10 55 (mpar). O que ocorreria se a soma fosse par? Este um novo e atrativo problema. Vamos enunci-lo:
PROBLEMA 3A: Escrevemos abaixo os nmeros naturais de 1 a 11.

10

11

Antes de cada um deles, coloque sinais + ou de forma que a soma de todos seja zero.
SOLUO: A soma dos nmeros naturais de 1 a 11 66. Como podemos separ-los em dois grupos de soma 33? Comeando pelos maiores observe que 11 + 10 + 9 = 30. Logo, 11 + 10 + 9 + 3 = 33. O problema 3A tem como uma soluo possvel:

+1 + 2 3 + 4 + 5 + 6 + 7 + 8 9 10 11 = 0

EUREKA! N 2, 1998

34

Sociedade Brasileira de Matemtica

Fica ao encargo do leitor mostrar que sempre que a soma dos naturais de 1 at n par ento podemos separ-los em dois grupos de igual soma. Voc pode utilizar o caminho que utilizamos acima, ou buscar uma outra forma.
Para saber mais e intrigar seus colegas

Voc pode propor aos seus amigos os problemas 3 ou 3A com uma lista grande de nmeros naturais consecutivos. O problema ter ou no soluo caso a soma desses nmeros seja par ou mpar, respectivamente. Entretanto, possvel encontrar o resultado desta soma rapidamente, sem precisar somar todas as parcelas. A soma de todos os naturais de 1 at n (1 + n) n . Por exemplo, a soma de todos os naturais de 1 at 10 igual a 2 (1 + 10)10 11 10 = = 55 . Procure demonstrar este fato e, se no conseguir, 2 2 pergunte ao seu professor ou escreva para a EUREKA!

PROBLEMA 4

Mostre que se a, b e c so inteiros mpares, a equao ax + bx + c = 0 no tem raiz racional. Comentrios: 1) Um nmero raiz de uma equao dada se quando for substitudo no 2 raiz (ou lugar do x a igualdade ficar correta. Por exemplo, x = 3 2 soluo) da equao 3 x 2 = 0 porque 3 2 = 0 . Ainda, x = 2 3 4 3 soluo da equao x x + x 10 = 0 porque 2 4 2 3 + 2 10 = 0 . Freqentemente no sabemos como resolver uma equao mas, em geral, podemos verificar se um certo valor de x ou no uma de suas razes.

EUREKA! N 2, 1998

35

Sociedade Brasileira de Matemtica

2) Um nmero racional quando puder ser escrito como uma frao de 2 4 e so exemplos de numerador e denominador inteiros. Por exemplo, 7 1 nmeros racionais. 3) Quando desejamos demonstrar que certo fato impossvel utilizamos freqentemente o mtodo da reduo ao absurdo. Este mtodo consiste em imaginar o contrrio, ou seja, que tal fato seja possvel. A partir da procuramos chegar a uma contradio, a um absurdo. Conseguindo isso, teremos mostrado que nossa hiptese (a do contrrio) falsa e conseqentemente, que a afirmao inicial verdadeira. Vamos ver tudo isso na soluo do problema. No se preocupe se voc ainda no sabe resolver uma equao do segundo grau. Isto no ser necessrio. Tudo o que precisamos verificar se um nmero racional pode ser uma raiz.

Soluo do problema 4
Imaginemos que o nmero racional

p q

seja raiz da equao

ax 2 + bx + c = 0 onde a, b e c so inteiros mpares. Logo, fazendo a


substituio, devemos ter,

p p a q + b q +c =0
a p2 p +b +c = 0 2 q q

ap 2 + bpq + cq 2 = 0
Vamos acrescentar agora uma hiptese importante para facilitar nosso p trabalho. Vamos supor que a nossa frao seja irredutvel, ou seja, que q
EUREKA! N 2, 1998

36

Sociedade Brasileira de Matemtica

ela j foi simplificada ao mximo. Por exemplo, no lugar de considerando

4 6

estaremos

o que a mesma coisa. Consideramos ento, para a 3 soluo do problema, que p e q no so ambos pares.
2 2

Observe agora a equao ap + bpq + cq = 0 nos seguintes casos: a) p e q so mpares: neste caso, ap mpar, bpq mpar e cq mpar. Como a soma de trs nmeros mpares mpar, o resultado no pode ser zero. b) p par e q mpar: neste caso, ap par, bpq par e cq mpar. Como a soma de dois nmeros pares e um mpar mpar, o resultado no pode ser zero. c) p mpar e q par: vale o mesmo argumento do caso b). Demonstramos ento que nenhuma frao de numerador e denominador inteiros pode ser raiz da equao ax 2 + bx + c = 0 onde a, b e c so inteiros mpares.
PROBLEMA 5
2 2
2 2

Um tabuleiro 6 6 est coberto com domins 2 1. Mostre que existe uma reta que separa as peas do tabuleiro sem cortar nenhum domin.
SOLUO: Cada domin formado por dois quadrados e portanto, se o tabuleiro est inteiramente coberto, 18 domins foram utilizados. Imagine agora uma reta (horizontal, por exemplo) que separe o tabuleiro em duas partes. Se ela no corta nenhum domin, est resolvido o problema. Suponha ento que ela corte ao meio um domin. Neste caso, acima desta reta teremos n domins inteiros mais meio domin, ou seja, teremos acima desta reta 2n + 1 quadrados, que um nmero mpar. Mas isto impossvel porque se o tabuleiro tem 6 unidades de largura, qualquer reta o dividir em partes que contm nmeros pares de quadrados acima e abaixo dela. Assim, se uma reta corta um domin, dever cortar um outro domin. Para a diviso do
EUREKA! N 2, 1998

37

Sociedade Brasileira de Matemtica

tabuleiro, existem 10 retas possveis e, se cada uma delas cortar dois domins, deveramos ter 20 domins no tabuleiro. Como eles so apenas 18 ento existe uma reta (pelo menos) que no corta nenhum domin.

Problemas para pesquisa


PROBLEMA 6

Os nmeros naturais de 1 at 1998 so escritos em um imenso quadro negro. Em seguida, um aluno apaga dois quaisquer colocando no lugar sua diferena (no negativa). Depois de muitas operaes, um nico nmero ficar escrito no quadro. possvel que esse nmero seja zero?

PROBLEMA 7

Em uma ilha plana existem 11 cidades numeradas de 1 a 11. Estradas retas ligam 1 a 2, 2 a 3, 3 a 4, ..., 10 a 11 e 11 a 1. possvel que uma reta corte todas as estradas?

EUREKA! N 2, 1998

38

Sociedade Brasileira de Matemtica

OS PROBLEMAS DO VISITANTE MATEMTICO


The Mathematical Visitor foi um peridico que existiu nos Estados Unidos entre 1877 e 1896. Era uma revista destinada aos amantes da arte de resolver problemas de Matemtica. Publicava problemas propostos pelo seu abnegado editor ou leitores e, em nmeros subseqentes, trazia as melhores solues apresentadas. Procurava fortalecer entre os norteamericanos, na poca em que sua nao lutava para se inserir no rol dos pases mais desenvolvidos, uma tradio h muito existente na Europa: a prtica das saudveis competies matemticas pblicas, institudas por revistas como a famosa Ladies Diary, da Inglaterra. Os problemas do The Mathematical Visitor eram, em sua grande maioria, de nvel elementar, embora alguns deles exigissem o uso de integrais em sua resoluo. Quanto criatividade e elegncia das questes propostas, a qualidade variava bastante. Num perodo em que faltavam calculadoras eletrnicas e sobrava lazer para as pessoas, eram muito freqentes problemas cuja soluo requeria muito mais pacincia e tempo disponvel de que engenhosidade e talento. Um exemplo de problemas desse tipo o seguinte, que foi proposto em 1887: 1) Considere a seqncia dos tringulos pitagricos (tringulos retngulos de lados inteiros) nos quais os catetos so inteiros consecutivos. Ache a expresso geral para os lados n-simo tringulo e calcule explicitamente os lados do centsimo. (A resposta da segunda parte envolve nmeros com 76 algarismos.) Outros problemas computacionais so: 2) 3) Calcular 4 4 . Obter a raiz cbica de 2 com 100 algarismos decimais!
4

Mas no se pense que The Mathematical Visitor s trazia perguntas sem graa. Alguns problemas bem elementares l propostos ainda guardam interesse a so apresentados aqui como desafio aos nossos leitores.
EUREKA! N 2, 1998

39

Sociedade Brasileira de Matemtica

4)

Com apenas dois cortes retilneos e recolagem, transforme um retngulo num quadrado de mesma rea, supondo que a razo entre o maior e o menor lado do retngulo menor do que ou igual a 4. Comprei na feira um queijo que pesou 9 quilos. Desconfiei da pesagem e o vendedor props, como compensao, vender-me um queijo igual, desta vez pesado no outro prato da balana. O peso foi de 4 quilos. Ganhei ou perdi na transao? Qual o verdadeiro peso do queijo? Ache trs nmeros inteiros cuja soma um cubo e a soma de dois quaisquer deles tambm um cubo. O doutor A mata 3 pacientes em cada 7 que trata; o doutor B mata 4 em cada 13 e o doutor C mata 5 em cada 19. Qual a probabilidade de um doente sobreviver se for tratado por esses 3 mdicos ao mesmo tempo? Ache quatro inteiros que so quadrados e a soma de dois quaisquer deles ainda um quadrado. (Observao: nem a redao da revista nem o autor do problema sabiam como resolv-lo.) Um dos problemas mais interessantes, propostos em 1881, foi o seguinte:

5)

6) 7)

8)

9)

Um vaso de vinho est suspenso sobre outro, de igual capacidade (digamos 1 litro), cheio de gua. Por um orifcio no fundo de cada, o vinho escorre sobre o vaso de gua e a mistura se esvai na mesma velocidade. Quando o vaso de vinho estiver vazio, qual o volume de gua no vaso inferior?

A coleo de problemas do The Mathematical Visitor foi reeditada em 1996 pela Math Pro Press, Westford, Mass., sob o ttulo "Problems and Solutions from The Mathematical Visitor". Nossa revista aguarda respostas de nossos leitores para os problemas acima propostos, especialmente os de nmeros 4, 5 e 9.

EUREKA! N 2, 1998

40

Sociedade Brasileira de Matemtica

DIVISIBILIDADE, CONGRUNCIAS E ARITMTICA MDULO n


Carlos Gustavo Moreira Nvel Avanado
INTRODUO

Este artigo se prope a ser uma referncia sobre os temas citados no ttulo, que aparecem naturalmente em diversos problemas de Matemtica elementar, alguns dos quais sero explicitamente tratados aqui. O estilo mais conciso do que a maioria dos outros artigos desta revista, o que pode tornar a leitura mais difcil, mas no desanime! Procure entender os enunciados das proposies e os problemas resolvidos e buscar sua propria soluo para eles, alm de pensar nos problemas propostos e enviar-nos suas solues. Em caso de qualquer dvida no deixe de escrever-nos.

Seo 1: Diviso euclidiana e o teorema fundamental da aritmtica Os resultados que seguem tm como base o seguinte fato sobre os inteiros: Dados a Z, b N* existem q, r Z com 0 r < b e a = bq + r. Tais q e r esto unicamente determinados. De fato, q = [a/b] e r = a bq (aqui [x] denota o nico inteiro k tal que k x < k + 1). Como conseqncia temos a Proposio 0 (Diviso Euclidiana): Dados a Z, b Z* existem q, r Z
unicamente determinados tais que 0 r < be a = bq + r

Definio: Dados dois inteiros a e b , com a 0 dizemos que a divide b (denotamos ab) se existe c inteiro tal que b = ac. Proposio 1: Dados a, b Z no ambos nulos existe d N* tal que da, db e, para todo c N*, ca, cb cd. Alm disso existem x, y Z com d = ax + by. (Esse d chamado o mximo divisor comum entre a e b : d = mdc (a, b). )

EUREKA! N 2, 1998

41

Sociedade Brasileira de Matemtica

Demonstrao: Seja A = {k > 0 x, y Z tais que k = ax + by} e seja d = ax0 + by0 o menor elemento de A. Mostraremos que da. Como d N*, existem q, r Z com a = dq + r e 0 r < d. Queremos mostrar que r = 0. De fato, se r > 0, r = a dq = a (1 qx0) + b( qy0) A, contradizendo o fato de d ser o menor elemento de A. Portanto, r = 0 e a = dq da. Do mesmo modo prova-se que db. Suponha agora que ca e cb. Ento cax0 + by0 = d, como queramos provar Lema: Se mdc (q, n) = 1 e nqk ento nk. Prova do Lema: Como mdc(q, n) = 1, existem x, y Z com qx + ny = 1,
logo qkx + nky = k, portanto nk (pois qkx e nnky)

Corolrio: Sejam p um nmero primo e a, b Z. Se pab ento pa ou pb

Teorema fundamental da aritmtica: Todo nmero natural n 2 possui uma nica fatorao (a menos da ordem dos fatores), como produto de primos. Demonstrao: n = 2 primo. Vamos mostrar a existencia da fatorao por primos por induo: Se n primo no h o que provar. Se n composto, n = ab, a, b N, a < n, b < n e, por hiptese de induo, a e b se decompem como produto de primos, portanto n se decompe como produto de primos. Vamos agora mostrar a unicidade, tambm por induo: Suponha que n admita duas fatoraes n = p1p2pr e n = q1q2qs como produto de primos. O Corolrio acima mostra que, como p1q1q2qr, p1 deve dividir algum qi e portanto p1 = qi (pois so ambos nmeros primos) e, como n/p1 = n/qi < n admite uma nica fatorao prima, por hiptese de induo, conclumos que a fatorao de n nica Proposio 2: O conjunto dos nmeros primos infinito.

EUREKA! N 2, 1998

42

Sociedade Brasileira de Matemtica

Demonstrao: Suponha que o conjunto dos nmeros primos seja finito, digamos { p1, p2,, pn}. Nesse caso, o nmero N = p1p2pn +1 seria maior que todos os primos, mas no divisvel por nenhum deles, pois pi( p1p2pn + 1) pi 1, absurdo. Teramos ento um natural N > 2 que no seria mltiplo de nenhum primo, contradizendo o teorema fundamental
da aritmtica Obs.: As idias desta seo podem ser utilizadas em situaes mais gerais, como no estudo de polinmios (por exemplo com coeficientes racionais), onde existe um algoritmo de diviso, a partir do qual pode-se provar de modo anlogo resultados correspondentes aos aqui apresentados sobre mximo divisor comum, existncia e unicidade de fatorao.

Seo 2: Congruncias Definio: Sejam a, b, n Z, n > 0. Dizemos que a congruente a b (mdulo n) (denota-se a b (mdulo n)) se n(b a) Obs: a a (mdulo n), a b (mdulo n) b a (mdulo n), a b (mdulo n), b c (mdulo n) a c (mdulo n), ou seja, congruncia (mdulo n) uma relao de equivalncia. Proposio: Se a b (mdulo n) e c d (mdulo n) ento a + c b + d (mdulo n) e ac bd (mdulo n). Demonstrao: n(b a), n (d c) n (b + d) (a + c) (a + c) (b + d) (mdulo n), e bd ac = b(d c) + ((b a) n(bd ac) bd ac (mdulo n) Definio: Dados n, a Z n > 0, definimos a =a (mdulo n) =
= {k Zk a (mdulo n)}. Dados a, b Z definimos a +b = a + b ea b = ab ( estas operaes de soma e produto esto bem definidas pela proposio anterior). Definimos ainda Z/nZ = {a (mdulo n), a Z}={0, 1, 2, n 1 }. Cada a chamada uma classe de congruncia mdulo n.

EUREKA! N 2, 1998

43

Sociedade Brasileira de Matemtica

Definio: Sejam n, a Z, n > 0. Dizemos que a invertvel mdulo n se existe b Z com ab 1(mdulo n) (ou seja, tal que a b = 1). Dizemos que b o inverso de a em Z/nZ. Definio: (Z/nZ)* ={a a Z e a invertvel (mdulo n)}. Obs. a invertvel (mdulo n) mdc (a, n) =1. De fato, mdc (a, n) = 1 x, y Z tais que ax + ny = 1a x = 1 (mdulo n). Notao: Dado um conjunto finito X, escrevemos # X para significar o nmero de elementos de X. Definio: A funo de Euler, : N N definida por (n) = # (Z/nZ)* = # {k Z 0 k < n e mdc (k, n) = 1}.
Notemos que se p um nmero primo e k N ento (pk) = pk pk1 =

pk (11/p). De fato, mdc (r, pk ) = 1 se e s se p no divide r. Logo (pk) = #{r Z0 r < pk e mdc (r, pk ) =1} = # {r Z 0 r < pk } # {r Z 0 r < pk e pr} = pk pk 1. Definio: n nmeros inteiros a1, a2,an formam um sistema completo de resduos (s.c.r.) mdulo n se {a1 , a2,, an}= Z/nZ isto , se os a representam todas as classes de congruncia mdulo n ( por exemplo, 0,1,2,n 1 formam um s.c.r. (mdulo n)). (n) nmeros inteiros b1, b2,b(n) formam um sistema completo de invertveis (s.c.i.) mdulo n se {b1, b2,b(n)} = (Z/nZ)*, isto , se os bi representam todas as classes de congruncias invertveis mdulo n. Proposio: Sejam q, r, n Z, n > 0, q invertvel mdulo n, a1, a2,,an um s.c.r. (mdulo n) e b1, b2,,b(n) um s.c.i. (mdulo n). Ento qa1 + r, qa2 + r,, qan + r formam um s.c.r. (mdulo n) e qb1, qb2,qb(n) formam um s.c.i. (mdulo n).

EUREKA! N 2, 1998

44

Sociedade Brasileira de Matemtica

Demonstrao: Vamos provar que se a1, an formam um s.c.r. (mdulo n) ento qa1 + r, qan + r formam um s.c.r. (mdulo n). Basta provar que qai + r qaj + r (mdulo n) i = j, pois nesse caso teremos n classes de congruncias distintas mdulo n, que devem ser todas as classes de Z/nZ.
Seja y Z tal que qy 1 (mdulo n). Temos qai = qai + r r qaj + r r = qaj (mdulo n) qyai qyaj (mdulo n) ai aj (mdulo n) i = j. Seja agora b1, b2,b(n) um s.c.r. (mdulo n). Temos que qbi invertvel mdulo n. para todo i,1 i (n), pois se xi tal que bi xi 1 (mdulo n). ento (qbi) (xiy) = (qy) (bixi) 1 (mdulo n). Por outro lado, se qbi qbj (mdulo n) ento bi yqbi yqbj bj (mdulo n) i = j, e portanto qb1, qb2,qb(n) um s.c.i. (mdulo n)

Teorema (Euler): Sejam a, n Z, n > 0, tais que mdc (a, n) = 1. Ento a(n) 1 (mdulo n). Demonstrao: Seja b1, b2,b(n) um s.c.i. (mdulo n) Pela proposio anterior, (ab1), (ab2),,(ab(n)) formam um s.c.i. (mdulo n), e temos
{b1, b2,,b(n)} = { ab1 , ab 2 , ab (n)} = (Z/nZ)*b1 b2.b(n) = ab 1 ab 2 .ab(n) =a(n) b1b2 b(n) b1 b2 b(n) (a(n) 1) = 0 a(n) =1 pois b1, b2,b(n) so invertveis (mdulo n) a(n) 1 (mdulo n)

Corolrio: (Pequeno Teorema de Fermat): Se a Z e p primo ento ap a (mdulo p). Prova : Se pa, ento ap a 0 (mdulo p). Se p no divide a, ento mdc (a, p) =1 a (mdulo p)

p1

1 (mdulo p) ap a

Exerccio resolvido: Exiba n N tal que 2n tenha mais de duas mil casas decimais e tenha entre suas 2000 ltimas casas decimais 1000 zeros consecutivos. Soluo: 2 ( 5
EUREKA! N 2, 1998
2000

1 (mdulo 52000), pelo teorema de Euler. Portanto,

45

Sociedade Brasileira de Matemtica

existe b N com 2 ( 5

2000

= 52000 b + 1, e teremos 2 2000 + ( 5


2000

2000

= 102000 b +

22000, e portanto os 2000 ltimos dgitos de 2 2000+ (5 ) coincidem com a representao decimal de 22000, que tem no mximo 667 dgitos, pois tem pelo menos 2000 667 = 1333 zeros consecutivos dentre as 2000 ltimas casas decimais, de modo que n = 451999 + 2000 satisfaz as condies do enunciado (pois (52000) = 45 1999)
23 < 10 22000 < 23.667 < 10667. Desta forma , 2
2000 + ( 52000 )

Teorema Chins dos restos: Se mdc (m, n) = 1 ento f: Z/mnZ Z/mZ Z/nZ, f (a (mdulo n)) = (a (mdulo m), a (mdulo n)) uma bijeo. Demonstrao: f est bem definida, pois se a = b (mdulo mn) ento a b (mdulo m) e a b (mdulo n). Como Z/mnZ e Z/mZ Z/nZ tm mn elementos cada, suficiente verificar que f injetiva. E, de fato, se a b (mdulo m) e a b (mdulo n) ento m (b a) e n(b a) b a = = mk, nmk nk, pois mdc (m, n) = 1 mn(b a) a b (mdulo mn) Corolrio: Se m1, m2,, mr 1 so inteiros, e mdc (mi, mj) = 1 para i j ento f: Z/m1 m2,mrZ Z/m1Z Z/m2Z Z/mrZ, f (a (mdulo m1 m2 . . mr)) = (a (mdulo m1), , a (mdulo mr)) uma bijeo
Notemos que este Corolrio mostra que, dados inteiros a1, a2,ar, existe um inteiro n com n = a1 (mdulo m1), n a2 (mdulo m2), , n ar (mdulo mr).

Proposio: Temos f ((Z/mnZ)*) = (Z/mZ)* (Z/nZ)* para a funo f definida acima. Demonstrao: Isto segue do fato de que a primo com mn se e s se a primo com m e a primo com n

EUREKA! N 2, 1998

46

Sociedade Brasileira de Matemtica

Corolrio: mdc (m, n) = 1 (mn) = (m) (n) Como conseqncia, se n = p1 1 , p 2 2 ... p k k onde p1, p2, pk so primos
distintos, 1, 2, k N* ento (n) = n (11/p1) (11/p2)(11/pk). Em particular, se n 3 ento (n) par Vamos mostrar um problema cuja soluo usa de modo no trivial o teorema chins dos restos:

Problema: Prove que dado n N existe um conjunto de n elementos


xB

A N tal que para todo B A, B , x uma potncia no trivial

(isto , um nmero da forma mk, onde m, k so inteiros maiores ou iguais a 2), ou seja, A = {x1, x2, xn} tal que x1, x2,xn, x1 + x2, x1 +x3,, x n 1 + x n , ,x1 + x2 +xn so todos potncias no triviais.

Soluo: A = {4} soluo para n = 1, A = {9,16} soluo para n = 2. Vamos provar a existencia de um tal conjunto por induo em n. Suponha que A={x1,, xn} um conjunto com n elementos e para todo B A, B kB , x = mB Vamos mostrar que existe c N tal que o conjunto

xB

={cx1, cx2, , cxn, c} satisfaz o enunciado. Seja = mmc {kB, B A, B } o mnimo mltiplo comum de todos os exponentes kB. Para cada B A, B associamos um nmero primo pB > , de forma que B1 B2 p B1 p B2 , e associamos um natural r com rB 0 (mdulo px ), X B, rB + 1 0 (mdulo pB) (tal rB existe pelo teorema chins dos restos), e tomamos
k B rB c = (1 + mB ) B A B

Como c uma potncia -sima, c uma potncia kB-sima para todo B A, B , portanto, para B {cx1, cx2,,cxn}, B, teremos B= {cxx B} para algum B A, B . Logo x ser uma potncia
xB '

kB-sima.

EUREKA! N 2, 1998

47

Sociedade Brasileira de Matemtica

Alm disso,

X B 'U {c}

x = c(1 + m

KB B

K B rB +1 K X rX , ) = (1 + m X ) (1 + m B ) X A X , B

que uma potncia pB-sima, pois rX mltiplo de pB para X B e rB + 1 mltiplo de pB

Seo 3: Ordens e razes primitivas. Dados n N* e a Z com mcd (a, n) = 1, definimos a ordem de a mdulo n, ordn a: = min {t N*at 1(mdulo n)}. Dado a (Z/nZ)* definidos orda = ord n a. Proposio: {t N*a t 1(mdulo n)}={k.ord n a, k N*}. Demonstrao: Como a
ord n a

1 (mdulo n), para todo k N tem-se

k .ord n a

= (a

ord n a k

) 1 = 1 (mdulo n). Por outro lado, se t n, at 1


k

(mdulo n), existe k N com

t = k ord n a + r ,0 r < ord n a a t = a k .ord n a a r 1.a r a r (mdulo n) a r 1 (mdulo n), portanto r = 0 ( pois 0 < r < ord n a contradiria a
minimalidade de ord n a ), e t = k. ord n a

Corolrio: ord n a (n) Definio: Se ord n a = (n), dizemos que a raiz primitiva mdulo n. Exemplos: 2 raiz primitiva mdulo 5, pois 21 = 2, 22 = 4, 23 = 8, 24 = 16, que a primeira potncia de 2 congruente a 1 mdulo 5 e 4 = (5). 1 raiz primitiva mdulo 2, pois ord2 1 =1 = (2). 3 raiz primitiva mdulo 4, pois ord4 3 = 2 = (4). Proposio 3.1: a raiz primitiva mdulo n {a t, t N} = (Z/nZ)*.

EUREKA! N 2, 1998

48

Sociedade Brasileira de Matemtica

Demonstrao: Para todo a Z com mdc (a, n) = 1 temos {a t, t N} (Z/nZ)*. Se a raiz primitiva mdulo n ento os nmeros 1, a, a2,a(n)1 so distintos (mdulo n) pois ai = aj (mdulo n), com 0 i < j < (n) aji 1 (mdulo n) com 0 < j i < (n), absurdo {at, t N} = (Z/nZ)*.
Por outro lado, #{a t, t N} ord n a (o argumento acima mostra que de fato vale a igualdade), e portanto {a t, t N} = (Z/nZ)* ord n a = (n)

Corolrio 1: Se m divide n e a a raiz primitiva mdulo n ento a raiz primitiva mdulo m Corolrio 2: Se k 3, ento no existe nenhuma raiz primitiva mdulo 2k. Prova: Pelo corolrio anterior, basta provar que no existe raiz primitiva mdulo 8, e isso segue do fato que se a mpar, a = 2r + 1, r Z a2 = 4r ( r + 1) + 1 1 (mdulo 8) Proposio 3.2: Sejam p um nmero primo, e a Z raiz primitiva mdulo p. Ento a ou a + p raiz primitiva mdulo p2. Demonstrao: Por hiptese, ord p a = ordp(a+ p) = (p) = p 1. Portanto p 1 ord p 2 a (pois at 1 (mdulo p2) at 1(mdulo p)), e, como

ord p 2 a (p2) = p( p 1), devemos ter ord p 2 a = p 1 ou ord p 2 a = p ( p 1) = (p2). Do mesmo modo, ord p 2 (a + p) = p 1 ou

ord p 2 (a + p) = p(p 1) = (p2).


Basta provar, portanto, que ord p 2 a p 1 ou ord p 2 ( a + p) p 1. Suponha que ord p 2 a = p 1. Portanto, a p1 1 (mdulo p2), e ento (a + p)
p1

=a

p1

+ (p 1) pa

p2

2 p3 + Cp . p2 + 1 + (p 1) pa 1 a

p2

(mdulo p2), portanto (a + p) p1 no congruente a 1(mdulo p2), pois p2 no divide (p 1) pa p 2, donde ord p 2 (a + p) p 1
EUREKA! N 2, 1998

49

Sociedade Brasileira de Matemtica

Proposio 3.3: Se p um nmero primo mpar e a raiz primitiva mdulo p2 ento a raiz primitiva mdulo pk para todo k N. Demonstrao: temos ap1 1 (mdulo p), mas ap1 no congruente a 1 (mdulo p2), portanto ap1 = 1 + b1 p, onde p no divide b1. Vamos mostrar
por induo que a p
k 1

( p 1)

= 1 + bk p k , onde p no divide bk, para todo


k 1

k 1: Temos a p
k

( p 1)

= (ap

( p 1)

) p = (1 + bk p k 1 ) p =
bk (mdulo p). Segue-se que p no

2 2 2k = 1 + bk p k +1 + C p bk p + ... 1 + bk p k +1 (mdulo pk + 2). Logo

ap

( p 1)

= 1 + bk p k +1 , com bk

+1

divide bk +1. Vamos agora mostrar por induo que a raiz primitiva mdulo pk para todo k 2. Suponha que a seja raiz primitiva mdulo pk. Ento temos pk 1( p 1) = (pk) = ord p k a ord p k +1 a (pk +1) = pk( p 1). Portanto,

ord p k +1 a = pk 1(p 1) ou ord p k +1 a = pk (p 1) = (pk+1), mas o primeiro


= 1 + bk p k ,que no congruente a 1 mdulo pk+1, pois p no divide bk. Portanto ord p k +1 a = (pk+1) e a raiz
caso impossvel, pois a p primitiva mdulo p k+1

k 1

( p 1)

Exemplo: 2 raiz primitiva mdulo 5k, k N. De fato, 2 raiz primitiva mdulo 5, e, como 24 = 16 1 (mdulo 25), 2 raiz primitiva mdulo 25 = 52 (como na proposio 3.2). Portanto, pela proposio 3.3, 2 raiz primitiva mdulo 5k, k N. Exerccio resolvido: Mostre que existe n natural tal que os mil ltimos dgitos de 2n pertencem a {1, 2}.

Soluo: Observamos inicialmente que para todo k N existe um nmero mk de k algarismos, todos 1 ou 2, divisvel por 2k. De fato, m1 = 2 e m2 = 12 satisfazem o enunciado. Seja mk = 2 k rk , rk N. Se rk par, tome mk+1 = 210k + mk = 2k+1 (5k + rk /2), e se rk mpar, tome mk+1 = 10k + mk=2k+1(5k + rk)/2.
EUREKA! N 2, 1998

50

Sociedade Brasileira de Matemtica

Como m1000 2 (mdulo 10), 5 no divide r1000 = m1000/21000. Como 2 raiz primitiva mdulo 51000, existe k N com 2k r1000 (mdulo 51000). Logo 2k = b 51000 + r1000, para algum b N. Portanto, 2k+1000 = b 101000 + 21000 r1000 = b 101000 + m1000, e as 1000 ltimas casas de 2k+1000 so as 1000 casas de m1000, que pertencem todas a {1, 2}

Observao: Se p primo mpar, k N e a um inteiro mpar tal que a raiz primitiva mdulo pk ento a raiz primitiva mdulo 2pk, pois (pk)= ord p k a ord 2 p k a (2pk) = (pk) ord 2 p k a = (2pk). Isso implica que
se a raiz primitiva mdulo pk ento a ou a + pk raiz primitiva mdulo 2pk ( pois a e a + pk so razes primitivas mdulo pk e um deles mpar.)

Proposio 3.4: Se n = ab, com a 3 e b 3 inteiros tais que mdc(a, b) = 1, ento no existe raiz primitiva mdulo n. Demonstrao: Temos (n) = (a) (b) e, como a 3 e b 3 , (a) e (b) so pares. Se mdc (k, n) = 1 ento temos k(n)/2 = (k(b)/2)(a) 1 (mdulo a), e k(n)/2 = (k(a)/2)(b) 1 (mdulo b). Assim, k(n)/2 = 1(mdulo n), e portanto ord n k (n)/2 < (n) Teorema: Existe alguma raiz primitiva mdulo n se, e s se, n = 2, n = 4, n = pk ou n = 2pk onde p primo mpar. Prova: Pelos resultados anteriores, basta provar que se p primo mpar
ento existe raiz primitiva mdulo p, ou seja, existe a (Z/pZ)* com ordp a = p 1. Para cada a (Z/pZ)*, tem-se ordp a( p 1). Seja d um divisor de p 1. Definimos N(d) = # {a (Z/pZ)*ordp a = d}. Temos portanto p 1 = seguintes:
d p 1

N(d). O resultado seguir dos dois lemas

Lema 1: N(d) (d) para todo d divisor de p 1.

EUREKA! N 2, 1998

51

Sociedade Brasileira de Matemtica

Prova: Se N(d) > 0 ento existe a (Z/pZ)* com ordp a. Se ordp a = d, entoad = 1 e, para 0 k < d, as classes de ak so todas distintas mdulo p, e (ak)d =1. Como a equao xd 1 = 0 tem no mximo d razes distintas em Z/pZ (pois Z/pZ um corpo), suas razes so exatamente ak, 0 k < d. Por outro lado, ordp ak = d mcd(k, d) = 1, pois se r > 1 tal que rk e rd ento (ak)d/r = (ad)k/r 1(mdulo p), logo ord p(ak) d/r < d. Desta forma, {b (Z/pZ)* ord pb = d} {ak, 0 k < d e mcd (k,d)
= 1}, portanto N(d) (d) Lema 2: (d) = n, para todo n N.

d n

Prova do Lema 2: Considere os n nmeros racionais 1/n, 2/n, ,n/n. Ao simplific-los, aparecem exatamente (d) deles com denominador d, para cada divisor d de n. Portanto, (d) = n

d n

Fim da prova do teorema: Do Lema 2 segue que (d) = p 1 e, como p 1=


d p 1

d p 1

N(d) e N(d)

(d) para todo d, devemos ter N(d) = (d) para todo d. Em particular, N(p 1) = (p 1) > 0 existem razes primitivas mdulo p
PROBLEMAS

1)

2) 3) 4) 5)

Prove que existem infinitos nmeros primos congruentes a 3 mdulo 4. Determine todos os n naturais tais que (2n 1)/n inteiro. Determine todos os n naturais tais que (2n + 1)/n2 inteiro. Prove que se a e b so naturais e (a2 + b2) / (ab + 1) inteiro ento (a2 + b2) / (ab + 1) quadrado perfeito. Sejam a, n N*. Considere a sequncia (xn) definida por x1 = a,

xk+1 = a k , k N. Mostre que existe N N tal que xk+1 xk (mdulo n), para todo k N.

Obs.: Os problemas 3 e 4 foram propostos na 31a. e na 29a. Olimpada Internacional de Matemtica (1990 e 1988) respectivamente.
EUREKA! N 2, 1998

52

Sociedade Brasileira de Matemtica

SOLUES DE PROBLEMAS PROPOSTOS EUREKA! N1


Publicamos aqui algumas das respostas enviadas por nossos leitores. 1)

Mostre que, dado um conjunto de n pessoas, existem duas que possuem o mesmo nmero de amigos entre as pessoas do conjunto.

SOLUO

Primeira Hiptese: h apenas uma nica pessoa sem amigos; logo entre as n 1 pessoas restantes, cada pessoa amiga de no mnimo uma pessoa e no mximo n 2 pessoas. Seja f : P Q onde P = conjunto das pessoas restantes e Q = conjunto dos possveis nmeros de amigos de uma determinada pessoa em P, ou seja: P = { p1, p2, , pn1} Q = { 1, 2, 3, , n 2} Observe que h n 2 valores no conjunto Q para n 1 valores em P ; isto quer dizer que n1, n2 P tais que f ( n1 ) = f ( n2 ). Segunda Hiptese: Suponha que todas as n pessoas tenham amigos entre si, ou seja: P = { p1, p2, , pn} e Q = { 1, 2, 3, , n 1} Observe que agora o conjunto Q possui n 1 valores, pois cada pessoa de P possui no mnimo 1 amigo e no mximo (n 1) amigos entre as (n 1) pessoas restantes. Pelo mesmo motivo da primeira hiptese n1, n2 P tais que f ( n1 ) = f ( n2 ). Concluso: h pelo menos duas pessoas com a mesma quantidade de amigos.
2)

Em uma pista circular h postos de gasolina, e o total de gasolina que h nos postos exatamente o suficiente para um carro dar uma volta. Prove que existe um posto de onde um carro com o tanque inicialmente vazio pode partir e conseguir dar uma volta completa na pista (parando para reabastecer nos postos). O Professor Carlos Alberto da Silva Victor observou que o problema 3 estava com o enunciado errado (de fato, n1998 um quadrado

3)

EUREKA! N 2, 1998

53

Sociedade Brasileira de Matemtica

perfeito e portanto deve ser congruente a 0 ou a 1 mdulo 4, no podendo pois terminar por 11 na representao decimal.) O enunciado correto : Prove que existe n N tal que os 1000 primeiros dgitos de n1998 so iguais a 1.
4)

Escreva 1998 como soma de (um nmero arbitrrio de ) parcelas de modo que o produto das parcelas seja o maior possvel.

Observe inicialmente que, dado n N, n n (i) se n (n > 4) par, temos > n 2 2 n 1 n +1 (ii) se n (n > 3) mpar, temos >n 2 2 Sejam 1998 = n1 + n2 + n3 + nk e P = n1 n2 n3 nk Com as observaes (i) e (ii) devemos ter ni { 1, 2, 3, 4} e como 4 = 2 2 podemos substituir 4 por "2 + 2" e teremos ni { 1, 2, 3}; logo P = 1 2 3 . evidente que = 0; pois se = 1, "1 + 2" pode ser substitudo por um 3 e "1 + 3" pode ser substitudo por "2 + 2". Tambm 2, pois "2 + 2 + 2" pode ser substitudo por "3 +3" ( 3 3 > 2 2 2) e conseqentemente P = 2 3 com ( = 1 ou 2 ). Como 1998 = 3 666 + 0, P = 3666 e S = 3 + 3 + 3+ ... + 3
666 vezes

SOLUO

5)

Sejam a > 0 e P1P2P3P4P5 uma poligonal aberta contida em um dos semiplanos determinados pela reta P1 P5 . Prove que existem pontos P6 e P7 no plano, com P5 P6 = a, de modo que possvel ladrilhar o plano com infinitos ladrilhos congruentes ao heptgono P1P2P3P4P5P6P7.

6)

Mostre que toda seqncia com n2 + 1 elementos possui uma subseqncia crescente com n + 1 elementos ou uma subseqncia decrescente com n +1 elementos.

EUREKA! N 2, 1998

54

Sociedade Brasileira de Matemtica

7)

Prove que 1 + 2 + 3 + ... + 1998 < 2

SOLUO

Definamos a funo : N {0} Z tal que (1) = 2 (n + 1) = (n)2 n, n 1 Temos que 1< (1) = 2 2< (2) = (1)2 1 = 22 1 = 3

Mostraremos que agora por induo que n < (n) para todo n 3

(3) = (2)2 2 = 32 2 = 7. Logo, 3 < (3) (Hiptese de induo) suponhamos que n < (n)

como 0 < n < (n), segue que n2 < (n)2 isto , n2 < (n +1) + n. Dai, n2 n < (n +1) Mas n + 1 < n2 n se e somente se 0 < n2 2n 1 se e somente se 0 < n2 2n + 1 2 se e somente se 0 < (n 1)2 2. Esta ltima desigualdade verdadeira se n 3 Portanto, se n 3, n +1 < n2 n < (n + 1) e dai n +1 < (n + 1). Pelo princpio de induo, segue que n < (n) para todo n 3 como para todo n N {0} e da, Portanto,

n n

n (n) para todo n N {0}

1998 < (1998) 1998 < (1997)2 1997


1997 +

1998 < (1997)2

1997 + 1998 < (1997) pois 0 < 1997 < (1997).


Prosseguindo desta maneira, chegaremos a

1 + 2 + 3 + ... + 1998 < (1) = 2


8)

Considere um torneio de xadrez envolvendo brasileiros e argentinos em que cada jogador joga contra todos os outros

EUREKA! N 2, 1998

55

Sociedade Brasileira de Matemtica

exatamente uma vez. Ao final do torneio, cada jogador obteve metade dos pontos que conquistou jogando contra brasileiros e metade jogando contra argentinos. Prove que o nmero total de jogadores do torneio um quadrado perfeito (obs: cada vitria vale 1 ponto, empate 1/2 ponto e derrota 0 ponto).
SOLUO

Sejam k o nmero de brasileiros e n o nmero de argentinos no torneio. Cada jogador brasileiro jogou k 1 partidas contra brasileiros. Observe que o nmero de vitrias, o nmero de empates e o nmero de derrotas (de cada jogador brasileiro contra jogadores brasileiros) somadas deve ser igual a k 1. (i) Seja s o nmero total de vitrias ocorridas entre brasileiros e E o nmero de empates, logo: 2s + E = k ( k 1); pois o nmero de vitrias igual ao nmero de derrotas. Usando a mesma idia do item (i) para os argentinos, temos: 2s' + E' = n ( n 1); onde s' o nmero total de vitorias entre argentinos e E' o nmero total de empates entre argentinos. , os totais de pontos obtidos nos 2 2 itens (i) e (ii) entre brasileiros e entre argentinos, respectivamente. Suponha agora que os jogos entre brasileiros e argentinos; logo cada brasileiro joga n partidas com os argentinos e cada argentino jogou k partidas com os brasileiros. Seja p o total de vitrias que os brasileiros obtiveram com os argentinos e q o total de empates que os brasileiros obtiveram com os argentinos, logo 2p + q = nk. Como o total de pontos de cada brasileiro, metade foi contra q brasileiros e outra metade entre argentinos, temos P = p + 2 Sejam p' o total de vitrias que os argentinos obtiveram contra os brasileiros e q' o total de empates que os argentinos obtiveram contra os brasileiros, logo: Sejam P = s +

(ii)

(iii)

e P' = s +

E'

(iv)

EUREKA! N 2, 1998

56

Sociedade Brasileira de Matemtica

2p' + q' = n k e tambm P' = p' + De (i), (ii), (iii) e (iv) temos:

q'

E q P = s + = p + 2s + E = 2 p + q 2 2 P ' = s '+ E ' = p'+ q ' 2s '+ E = 2 p '+ q ' 2 2


Somando (v) teremos:

(v)

k(k1) + n(n 1) = nk

2 s + E+2 s '+ E ' = 2 p + q + 2 p'+ q'


+

nk

n + k = (n k)2 ou seja o total de jogadores um quadrado perfeito: Nota: Para cada n, k com n + k = (n k)2 possvel construir torneios com k brasileiros e n argentinos satisfazendo as condies do enunciado. Note

t2 + t t2 t tambm que se n + k = t ento n = e k= . 2 2


2

9)

Prove que todo nmero racional positivo pode ser escrito como soma de um certo nmero de fraes distintas de numerador 1.

SOLUO

(i)

Seja inicialmente a frao

p q

< 1 , logo n N tal que


p q

1
n

p q

<

1
n 1

, observe que para n 2, temos:

1
n

np q nq

at nq encontrarmos a frao inicial como uma soma de fraes com Ns podemos repetir o processo inicial para a frao
EUREKA! N 2, 1998

np q

57

Sociedade Brasileira de Matemtica

numeradores iguais a 1; observe tambm que np q < p, ou seja o numerador da frao

np q menor do que o numerador da nq

frao original e j que os numeradores dessas fraes no podem decrescer indefinidamente, este procedimento dever terminar com um nmero finito de fraes com numeradores iguais a 1. Resta ento mostrar que essas fraes so todas distintas; se no vejamos:

np q p 1 1 1 1 1 = < = (n 2); nq q n n 1 n n(n 1) n


escrita como uma soma de fraes de nq numeradores iguais a 1, todos os denominadores dessas fraes so maiores do que n, mostrando portanto que essas fraes so todas distintas. ento quando (ii) Seja

np q

p > 1 , ento n N tal que: q


+

1+

1 2

1 3

+ ... +

1
n

p q

< 1+

1 2

1 3 1

+ ... +

1
n +1

logo:

1 , com < < 1, 2 3 4 n +1 q n usando o tem (i) podemos expandir como uma soma finita de
p = 1+

+ ... +

fraes unitrias cujos denominadores so maiores que "n + 1".


Solues dos problemas 1, 4, 8 e 9 enviadas por Carlos Alberto da Silva Victor, Nilpolis, Rio de Janeiro-RJ. Soluo do problema 7 enviada por Manuel Joo de Jesus Almeida, Rio de Janeiro-RJ. Agradecemos tambm a participao de Carlos Eduardo Cardoso Borges, Wayne L. Silva de Paula, Marco Rogrio Vieira e Vicente Wilson Moura Gaeta. Continuamos esperando as solues dos problema 2, 3, 5 e 6.

EUREKA! N 2, 1998

58

Sociedade Brasileira de Matemtica

PROBLEMAS PROPOSTOS
Convidamos o leitor a enviar solues dos problemas propostos e sugestes de novos problemas para os prximos nmeros.

10)

Suponha que temos k moedas, todas iguais exceto por uma que tem peso ligeiramente diferente das anteriores (no se sabe se maior ou menor), e uma balana de dois pratos. possvel determinar com n pesagens 2 qual a moeda diferente, e se ela mais leve ou mais pesada que as outras. 3n 1 Mostre que se k = possvel determinar com n pesagens 2 qual a moeda diferente, mas nem sempre possvel dizer se ela mais leve ou mais pesada que as outras. Mostre que se k > a moeda diferente. Mostre que se k

a)

3n 3

b)

c)

3n 1 2

no sempre possvel determinar qual

11) 12)

Determine todas as solues de xy = yx com x e y racionais positivos.

a) Prove que se n N e 2n + 1 um nmero primo ento n uma potncia de 2. b) Prove que se a, n N, n 2 e an 1 primo, ento a = 2 e n primo.
Dado n N determine determine o maior k N tal que existam conjuntos A1, A2,, Ak contidos em {1, 2, , n} de forma que Ai Aj para todo i j.

13)

EUREKA! N 2, 1998

59

Sociedade Brasileira de Matemtica

14)

(Problema proposto por Antonio Luiz Santos): Determine o nmero 1 1 1 com x e y inteiros positivos. de solues de + = x y 1998 Considere uma seqncia de tringulos retngulos AnBnCn no plano cuja hipotenusa seja BnCn, com as seguintes condies:

15)

i) ii) iii)

A1 B1 = A1C1 = 1
Bn + 1 = Bn e An + 1 = Cn para todo n N. An+1 Cn+1 congruente altura de An em relao a BnCn. Mostre que qualquer ponto do plano pertence a infinitos tringulos AnBnCn .

Voc sabia
dimenses mas apenas 3 em 5 ou mais dimenses ?? Em dimenso n h o simplexo, com n + 1 "faces" (que so simplexos) de dimenso n 1, o hipercubo, com 2n "faces" ( que so hipercubos) de dimenso n 1 e o hiperoctaedro, dual do hipercubo, com 2n "faces" (que so simplexos) de dimenso n 1. Em dimenso 4, alm desses h o C24, que tem 24 "faces" octadricas, o C120, que tem 120 "faces"dodecadricas e o C600, que tem 600 "faces" tetradricas. Lembre-se que em 3 dimenses h 5 poliedros regulares: o tetraedro (caso particular do simplexo), o cubo (caso particular do hipercubo), o octaedro (caso particular do hiperoctaedro), o dodecaedro, que tem 12 faces pentagonais, e o icosaedro, que tem 20 faces triangulares. Que h 6 poliedros regulares no espao euclidiano de 4

EUREKA! N 2, 1998

60

Sociedade Brasileira de Matemtica

AGENDA OLMPICA

OLIMPADA BRASILEIRA DE MATEMTICA Primeira Fase Sbado, 6 de junho Segunda Fase Sbado, 12 de setembro Terceira Fase Sbado, 24 de outubro (nveis 1, 2 e 3) Domingo, 25 de outubro (nvel 3). OLIMPADA IBEROAMERICANA DE MATEMTICA
13 a 20 de setembro de 1998 Repblica Dominicana.

Voc sabia Que possvel pentear um toro (superfcie em forma de pneu) cabeludo mas
no uma esfera cabeluda sem deixar rodamoinhos??

EUREKA! N 2, 1998

61

Sociedade Brasileira de Matemtica

COORDENADORES REGIONAIS
Alberto Hassen Raad Antnio C. Rodrigues Monteiro Amarsio da Silva Arajo Angela Camargo Antnio C. do Patrocnio Ariosto de Oliveira Lima Benedito T. Vasconcelos Freire Carlos A. Bandeira Braga Claudio Arconcher Egnilson Miranda de Moura lio Mega Florncio F. Guimares F. Francisco Dutenhefner Gisele de A. Prateado G. Ivanilde H. Fernandes Saad Joo B. de Melo Neto Joo F. Melo Libonati Jos Carlos Pinto Leivas Jos Luis Rosas Pinho Jos Paulo Carneiro Jos Vieira Alves Leonardo Matteo D'orio Licio Hernandes Bezerra Luzinalva M. de Amorim Marco Polo Marcondes Cavalcante Frana Mario Jorge Dias Carneiro Ma-To-F Pablo Rodrigo Ganassim Paulo H. Cruz Neiva de L. Jr. Reinaldo Gen Ichiro Arakaki Ricardo Amorim Sergio Claudio Ramos Tadeu Ferreira Gomes Wagner Pereira Lopes (UFJF) (UFPE) (UFV) (Centro de Educao de Adultos CEA) (IMECC/UNICAMP) (UFPI) (UFRGDN) (UFPB) (Col. Leonardo da Vinci) (Col. Agrcola do Bom Jesus) (Col. ETAPA) (UFES) (UFMG ) (UFGO) (U. Catlica Dom Bosco) (UFPI) (Grupo Educ. IDEAL) (URG) (UFSC) (USU) (UFPB) (Parque de Material Aeronutico de Belm) (UFSC) (UFBA) (Colgio Singular) (UF Cear) (UFMG) (UEM) (L. Albert Einstein) (Esc. Tec.Everardo Passos) (INPE) (Centro Educ. Logos) (IM-UFRGS) (U. do Estado da Bahia) (Esc. Tec. Fed. de Gois) Juiz de Fora-MG Recife-PE Viosa-MG Blumenau-SC Campinas-SP Parnaba-PI Natal-RN Joo Pessoa-PB Jundia-SP Bom Jesus-PI So Paulo-SP Vitria-ES BH-MG Goinia-GO Campo Grande-MS Teresina-PI Belm-PA Rio Grande-RS Florianpolis-SC Rio de Janeiro-RJ Campina Grande-PB Belm-PA Florianpolis-SC L. de Freitas-BA Santo Andr-SP Fortaleza-CE BH-MG Maring-PR Rio das Pedras-SP Piracicaba-SP S.J.Campos-SP Nova Iguau-RJ Porto Alegre-RS Juazeiro-BA Jata-GO

EUREKA! N 2, 1998

62

CONTEDO
AOS LEITORES OLIMPADA BRASILEIRA DE MATEMTICA Problemas de treinamento para a terceira fase XIX OLIMPADA BRASILEIRA DE MATEMTICA (1997) Problemas e solues da segunda fase snior 13a. OLIMPADA IBEROAMERICANA DE MATEMTICA Repblica Dominicana 1998 - Problemas e resultados 39a. OLIMPADA INTERNACIONAL DE MATEMTICA Solues 2 3 8 18 20

ARTIGOS
O PRINCPIO DA INDUO Elon Lages Lima FRAES CONTNUAS, REPRESENTAES DE NMEROS E APROXIMAES Carlos Gustavo Moreira SOLUES DE PROBLEMAS PROPOSTOS EUREKA Nos. 1 e 2 PROBLEMAS PROPOSTOS COORDENADORES REGIONAIS 26 44 56 61 62

Sociedade Brasileira de Matemtica

AOS LEITORES
1998 tem sido um bom ano para o programa brasileiro de Olimpadas de Matemtica. Tivemos em torno de 40.000 participantes na primeira fase, ganhamos mais uma medalha de ouro na Olimpada Internacional de Matemtica e fomos o pas com maior soma de pontos na Olimpada Iberoamericana de Matemtica. Esperamos conclu-lo com uma terceira fase da Olimpada Brasileira de Matemtica que faa jus aos resultados at agora obtidos, estimulando ainda mais a imaginao criativa dos jovens competidores, propiciando a descoberta de novos talentos para a matemtica e, em particular, criando as bases para as equipes brasileiras que participaro nas olimpadas internacionais do ano que vem. Esperamos que os nmeros da revista Eureka! que apresentamos este ano sejam teis para aumentar o nmero de participantes da Olimpada Brasileira de Matemtica e que permitam a todos os classificados chegarem bem preparados terceira fase, alm de contribuir para o enriquecimento da cultura matemtica de nossa comunidade acadmica e escolar. Esta Eureka! 3 est mais difcil que as anteriores, entre outras razes, por ter boa parte de seu material dedicado preparao para a terceira fase do terceiro nvel. Grande parte do material das Eureka! 1 e 2 adequada preparao para a terceira fase dos primeiros dois nveis, mas no terceiro nvel a prova (como mostra a segunda fase snior da Olimpada Brasileira de Matemtica do ano passado, aqui resolvida) costuma ser mais tcnica, de modo que resolvemos usar a Eureka! 3 para oferecer aos participantes da terceira fase uma preparao adequada, com problemas mais difceis e bem diferentes dos que usualmente se estudam nas escolas. A terceira fase ser realizada nas seguintes datas. Sbado 24 de outubro 1o. nvel 2o. nvel 3o. nvel (primeira prova). 3o. nvel (segunda prova).

Sbado 14 de novembro

Comit Editorial.
EUREKA! N 3, 1998

Sociedade Brasileira de Matemtica

OLIMPADA BRASILEIRA DE MATEMTICA


Problemas de treinamento para a terceira fase

1)

Sejam trs pontos A, B e C pertencentes a uma circunferncia de centro O tais que AOB < BOC. Seja D o ponto mdio do arco AC que contm o ponto B. Seja K o p da perpendicular a BC por D. Prove que AB + BK = KC.

2)

Prove que existe uma seqncia a0, a1, , ak, , onde cada ai um algarismo (ou seja, ai {0, 1, 2, 3, 4, 5, 6, 7, 8, 9}) com a0 = 6, tal que para cada inteiro positivo n o nmero xn = a0 + 10a1 + 100a2 + + 10n1 an1 (cuja representao decimal an1 an2 a1a0) tal 2 que x n x n divisvel por 10n. Seja A = {x1 < x2 << xn} um conjunto de nmeros inteiros positivos tal que se x e y so dois nmeros naturais que no pertencem a A ento x + y no pertence a A. Provar que xi 2i 1 para i = 1, 2,, n. Considere a seqncia (xn) todo n N, x n + 2
nN

3)

4)

definida por x1 = 19, x2 = 98 e, para

1 , se x n +1 0 xn x n +1 = 0, se x = 0. n +1

Prove que existe n N tal que xn = 0 e encontre o menor n com essa propriedade. 5) Sejam ABC um tringulo, M o p da bissetriz interna do ngulo A e N o p da bissetriz interna do ngulo B. Suponha que MN seja bissetriz do ngulo AMC. Calcule o ngulo A. Ache todas as solues reais de [x ] +

6)

( [ y ] denota o nico inteiro tal que

[ y ] y < [ y ] + 1) .

[ 1998x ] = 1998

EUREKA! N 3, 1998

Sociedade Brasileira de Matemtica

7)

Mostre

que

produto

de

todos

os

nmeros

da

forma

1 2 3 ... 100 o quadrado de um nmero inteiro.

Solues 1)

Sejam AB = x, BD = y; marcamos D' tal que D'C = y. Ento D'D = x por ser D ponto mdio de AC e resulta DD' // BC. Se K' o p da perpendicular a BC por D', ento temos AB = DD' = KK' e BK = K'C AB + BK = KK'+ K'C = KC.

2) O primeiro termo a0 = 6; ento x1 = 6 e x12 x1 = 36 6 = 30 , que divisvel por 101. Seja n 1. Suponhamos que existem a0, a1,,an1 tais que x n = a 0 + 10a1 + 10 2 a 2 + ... + 10 n 1 a n 1
2 2 verifica que x n x n divisvel por 10n (ou seja x n x n = 10 n r , com r N) Temos que encontrar an tal que x n +1 = a 0 + 10a1 + 10 2 a 2 + ... + 10 n 1 a n 1 + 10 n a n = x n + 10 n a n
2 n +1 . seja tal que x n +1 x n +1 divisvel por 10

EUREKA! N 3, 1998

Sociedade Brasileira de Matemtica


2 n 2 n 2 n 2n 2 n xn +1 x n +1 = ( x n + 10 a n ) ( x n + 10 a n ) = x n + 2 x n 10 a n + 10 a n x n 10 a n = 2 2 2 = ( xn xn ) + 10n (2xn an an ) + 102n an = 10n r +10n (2xn an an ) +102n an = 2 = 10n (r + 2xn an an ) + 102n an . Assim, 2 n +1 xn r + 2xn an an divisvel por 10 +1 x n +1 e divisvel por 10

r + (2xn 1)an divisvel por 10.


Dado que a0 = 6, temos que xn = 10t + 6 com t N; ento 2xn = 10h + 2. (com h = 2t + 1). Logo r + (2xn 1)an divisvel por 10 r + (10h + 1)an divisvel por 10 r + an divisvel por 10. Sempre existe um nico inteiro an entre 0 e 9 de modo tal que isto se verifique.

Obs: A seqncia (an) comea por 6, 7, 3, 9, 0, 1, 7, 8, 7, 1, 8, 0, 0, 4, 7, 3 Assim, por exemplo, x10 = 1787109376. Problema extra: Prove que a seqncia (an) obtida no peridica nem pr-peridica. 3) Suponhamos que o enunciado falso, ou seja que existe k tal que
xk > 2k 1, 1 k n. Ento os conjuntos B1 = {1, xk 1}, B2 = {2, xk 2}, , Bk = {k, xk k} so disjuntos dois a dois e seus elementos so menores que xk. Alm disso, para cada j, 1 j k, j A ou xk j A, pois no caso contrario, ou seja, se j A e xk j A, teramos que xk = j + (xk j) A. Portanto, para cada j, 1 j k, A Bj , donde A tem pelo menos k elementos menores que xk , absurdo.

4) Se xn+1 0, temos xn+2 xn+1 = xn+1 xn 1. Definindo yn = xn xn+1 temos yn+1 = yn1 para todo n tal que xn+1 0. Como y 1 = x1 x2 = 19 98 = 1862, temos yk = 1863 k enquanto yk 1 for diferente de 0, e portanto y1862 = 1 e
EUREKA! N 3, 1998

Sociedade Brasileira de Matemtica

y1863 = 0 x1862 x1863 = 1 e x1863 x1864 = 0. Assim, x1863 0 e x1864 = 0, donde 1864 o menor n tal que xn = 0.

5)

, e como MC d b+c C devemos ter MA = AN = c , donde MA = bc MN bissetriz de AM MC CN a b+c ab pela lei dos senos aplicada aos tringulos ABC e ABM (pois MC = b +1
Pelo teorema das bissetrizes,

AN

NC

BM

MC =

ab

ac

temos senA = a = b + c = BM = sen ( A / 2 ) , e portanto sen (A/2) = sen A


senB b bc MA senB b+c

= 2 sen (A/2) cos (A/2) cos

2 A A 1 . = = A= 2 3 3 2 2

6) [x] + 1998x sempre inteiro. Seja x0 a soluo de x + 1998x = 1998, ou

,8... seja x0 = 999 (3 5) = 763,1... e 1998x0 = 1998 x0 = 999( 5 1) = 1234


Temos x +
0

[ ] [ 1998x ] = 1997. A funo


0

f ( x ) = [x ] +

[ 1998x ] aumenta

de uma unidade quando x ou

1998 x torna-se inteiro. Os prximos valores

de x maiores que x0 para os quais x e 1998 x so inteiros so respectivamente 764 e 12352 / 1998 < 764. Assim, f (12352 /1998) = 763 + 1235 = 1998 e f (764) = 764 + 1235 = 1999 (de fato 1998 764 < 1236). Como f (x) no-decrescente, o conjunto das solues o intervalo 1235 2 ,764 = [763 ,3758758758 ..., 764 ). 1998

EUREKA! N 3, 1998

Sociedade Brasileira de Matemtica

7) O nmero referido no enunciado o quadrado do produto de todos os 299


nmeros da forma 1 2 3 ... 100 (no produto do enunciado cada um desses nmeros aparece uma vez, assim como seu simtrico). Neste ltimo produto, obtemos uma soma de termos do tipo

( , a , )(
1

a )...(
2

a ), com m 2 99 , a , a ,...a {2,3,...,100} e


m 1 2 m

j {-1,1}, j.
todos os termos como acima que tm exatamente k valores de a j = k , Fixamos 2 , 3 ,..., 100 N com 2 + 3 + ... + 100 2 99 , e consideramos

para 2 k 100 . Se todos os j so pares esses termos so todos inteiros. Se algum deles (digamos r ) mpar, podemos associar de forma bijetiva a cada termo desses o termo obtido trocando os sinais de todos os j para os quais a j = r. Assim, a cada termo associamos o seu simtrico, e portanto, nesse caso a soma dos termos considerados 0. Assim, o produto de todos os nmeros da forma 1 2 3 ... 100 um inteiro, e portanto o produto do enunciado um quadrado perfeito.

Voc sabia

que so conhecidos 51539600000

casas decimais de

, calculadas por Y. Kamada e D. ?

Takahashi, da Universidade de Tokio em 1997? E que em 21/8/1998 foi calculada pelo projeto Pihex a 5000000000000a. casa binria de

Consulte a home-page http://www.cecm.sfu.ca/pi

EUREKA! N 3, 1998

Sociedade Brasileira de Matemtica

XIX OLIMPIADA BRASILEIRA DE MATEMTICA (1997)


Problemas e solues da segunda fase snior
PROBLEMA 1

Duas circunferncias de raios R e r e centros O e O', respectivamente, intersectam-se nos pontos P e P'. Seja l a reta que passa por P e P'. Determine em funo de R e r, o menor valor que pode assumir a soma das distncias de l a O e O'.
PROBLEMA 2

Dizemos que um conjunto A N satisfaz a propriedade P(n) se A tem n elementos e A + A = {x + y tal que x A e y A} tem
n( n + 1) 2

elementos.

Dado A N finito definimos o dimetro de A como sendo a diferena entre o maior e o menor elemento de A. Seja f (n) o menor dimetro que um conjunto A satisfazendo P(n) pode ter. Mostre que
n
2

f ( n ) < n para todo

n 2.
(Se o seu tempo de prova no estiver esgotado, tente melhorar esta estimativa. Por exemplo, tente mostrar que f (p) < 2p2, para todo nmero primo p.)
PROBLEMA 3

a)

Prove que no existem funes f : R R e g : R satisfazendo g ( f (x) ) = x3 e f ( g(x) ) = x2 para todo x R. Exiba funes f : ( 1, ) ( 1, ) e g :( 1, ) (1, ) tais que

b)

g ( f(x) ) = x3 e f ( g(x) ) = x2 , para todo x (1, ).

EUREKA! N 3, 1998

Sociedade Brasileira de Matemtica

PROBLEMA 4

Seja Fn definido por F1 = 1, F2 = 1 e Fn+2 = Fn+1 + Fn , para todo n 1. Seja

Vn = Fn2 + Fn2+ 2 , n 1. Mostre que, para todo n inteiro positivo, Vn ,


Vn+1 e Vn+2 so lados de um tringulo de rea 1/2.
PROBLEMA 5

Sejam c Q, f (x) = x2 + c. Definimos f 0 (x) = x , f n+1 (x) = f ( f n (x)), n N . Dizemos que x R pr-peridico se { f n (x), n N } finito. Mostre que { x Q | x pr-peridico } finito.
PROBLEMA 6

Seja f uma funo do plano no plano que satisfaz d (P,Q) = 1 d (f (P), f(Q)) = 1 para todos os pontos P e Q do plano. Mostre que d (f (P), f(Q)) = d (P, Q) para todos os pontos P e Q do plano. (d (X,Y) denota a distncia entre X e Y).
SOLUES 1)

Considere a circunferncia de raio R fixa, cujo centro O est sobre uma reta s. O problema se resume a determinar a posio de O' em s que minimiza a soma d das distncias de O e O' a . Claramente, perpendicular a s. Seja I o ponto de interseco de s com . Temos dois casos a considerar: (i) (ii)

OO' OI. Neste caso, d = OO' e d mnimo quando I = O'. OO' OI. Neste caso, considere O'' O' em s tal que O'I = IO'' ( O'' simtrico de O' em relao ). Assim, temos que d = OO'' OI (primeiro caso) e d mnimo quando O' = I = O''. Em ambos casos, temos que d mnimo quando O' = I . Por Pitgoras, este

mnimo igual a

R2 r 2 .

EUREKA! N 3, 1998

Sociedade Brasileira de Matemtica

2)

Dado um conjunto finito A N , denotaremos por d(A) o dimetro de A. Temos duas desigualdades a provar:

(i)

, para todo n 2. 4 Vamos supor, por absurdo, que exista um conjunto A = {a1, a2, ,an}, n 2,

f (n)

n2

tal que A satisfaz P(n), a1 < a2 < < an e d(A) <

n2 . Como A satisfaz P(n), 4

elementos. Como 2 a1 + a1 < a1 + a2 < < an + an, temos que (an + an) (a1 + a1) + 1 n( n + 1) n2 n 2 n2 n2 an a1 + d ( A) , o que uma 2 4 4 4 4 contradio. Isto demonstra (i). (ii)

A + A = {a1 + a2, a1 + a2, ,an + an} tem

n( n + 1)

f (n) < n 3 , para todo n 2.

Como {0, 1} satisfaz P(2), temos que f(2) 1 < 23. Agora, vamos supor que f (n) < n 3 para algum n 2. Seja An = {a1, a2, , an} N tal que An satizfaz P(n) e d(An) = f(n) < n3. Sem perda de generalidade, podemos supor que 0 = a1 < a2 < < an = d(An), bastando para isto subtrair de cada elemento de An o menor de seus elementos. Agora, queremos achar an + 1 N An tal que An + 1 = {a1, a2, , an + 1} satisfaa P(n +1) e

d (An + 1) < (n + 1)3. Como An + An tem

n( n + 1) 2

elementos e temos que

An +1 + An +1 = ( An + An ) {ai + a n +1 1 i n + 1},

P (n + 1) se e somente se, ai + a j a n +1 P = {ai + a j a k 1 i, j , k n} { 1 i, j n}. Como 2 n(n + 1) n( n + 1) , temos que a n +1 n 3 + , pois basta escolher P n3 + 2 2
a n +1 N An
e

An+1

satisfaz

an+1

como

menor

natural

que
10

no

est

em

P.

Assim,

EUREKA! N 3, 1998

Sociedade Brasileira de Matemtica

f (n + 1) d ( An +1 ) < (n + 1) 3 . Por induo finita em n, temos que (ii)


verdade, o que completa nossa demonstrao. Vamos ainda, verificar que, para p primo mpar, f ( p ) < 2 p 2 . Para isto, A = {k + 2 pg ( k ),0 k p 1}, onde construmos o conjunto

g ( k ) = k 2 (mod p), 0 g(k) p 1.


Temos d ( A) p 1 + 2 p ( p 1) = 2 p 2 p 1 < 2 p 2 e se tivssemos i + 2 pg (i ) + j + 2 pg (i ) = r + 2 pg ( r ) + s + 2 pg ( s ), ento i + j + 2 p ( g (i ) + g (i )) = r + s + 2 p ( g (r ) + g ( s )) i + j =

= r + s , g (i ) + g ( j ) = g ( r ) + g ( s ) Assim,

i r = s j e i 2 + j 2 r 2 + s 2 (mod p), logo


(i r )(i + r ) ( s j )( s + j )(mod p ) i r s j 0(mod p ) ou i + r s + j (mod p ) . Portanto i = r e s = j ou i = s e r = j. Com um pouco de teoria dos Corpos, possvel provar, utilizando um elegante argumento devido a Bose-Chowla, que, de fato, temos f ( p) < p2 para p primo. Seja K = Z/pZ o corpo com p elementos e L K um corpo com p2 elementos. Seja um gerador do grupo (cclico) multiplicativo de L, ou seja, tal que { k , k Z} = L {0}. Para cada m K, + m L {0, 1},
e, portanto, existe am Z, 0 < am < p2 1 tal que am = + m. O conjunto

A = {a m ,0 m p 1}
i j r s

tem

dimetro

no

mximo

p2 3 < p2 e

a + a = a + a ( + i)( + j ) = ( + r )( + s) (i + j r s) + (ij rs) = 0.

Como K, temos i + j = r + s e ij = rs {i, j} = {r , s}.

3)
a) Vamos supor, por absurdo, que existam funes f, g : R R satisfazendo, para todo x R,

( I ) g ( f ( x)) = x 3 e
( II ) f ( g ( x)) = x 2

EUREKA! N 3, 1998

11

Sociedade Brasileira de Matemtica

Agora Logo

x, y R , f ( x ) = f ( y ) g ( f ( x )) = g ( f ( y )) x 3 = y 3 x = y. f injetora. Ainda, de (I) e (II), temos

( f ( x) 2 = f ( g ( f ( x)) = f ( x 3 ) f (0) 2 = f (0), f (1) 2 = f (1) e f (1) 2 = f (1), logo { f ( 0 ), f ( 1 ), f ( 1 )} { 0 ,1} o que um contradio (pois f injetora e, portanto, { f (0), f (1), f (1)} tem 3 elementos).

(b) Vamos supor, por enquanto, que existam funes 3 2 f , g : (1,+) (1,+) tais que g ( f ( x)) = x e f ( g ( x )) = x , para todo x (1, ). Agora, considere as funes

( x) = log 2 (log 2 g (2 2 )) ( x) = log 2 (log 2 f (2 2 ))


Temos
x

( x) = log 2 (log 2 g (2 2
x

log 2 (log 2 f ( 2 2

))

)) = log 2 (log 2 g ( f (2 2 ))) =

log 2 (log 2 (2 2 ) 3 ) = log 2 3 2 x = x + log 2 3


log 2 (log 2 g ( 2 2 x

( x) = log 2 (log 2 f (2 2
x

))

)) = log 2 (log 2 f ( g (2 2 ))) =

log 2 (log 2 (2 2 ) 2 ) = log 2 2 x +1 = x + 1


Supondo que ( x) = ax + b todo x R, e ( x) = cx + d , devemos ter, para

( x) = acx + ad + b = x + log 2 3 ( x) = acx + d + bc = x + 1


Podemos escolher, por exemplo, a = log 2 3, b = 0, c = log 3 2 e d = 1. (ou seja, ( x) = x log 2 3 e ( x ) = x log 3 2 + 1 De (A), temos

g (2 2 ) = 2 2

( x)

g ( x) = 2 2

(log2 (log2 x ))

= 22

log 2 3. log 2 log 2 x

= 2 log 2 x

log 2 3

EUREKA! N 3, 1998

12

Sociedade Brasileira de Matemtica

e de (B)

f (2 2 ) = 2 2

(x)

f ( x) = 2 2

(log2 (log2 x ))

= 22

1+ log2 log2 x . log3 2

= 2 2 log 2 x

log3 2

fcil verificar que as funes acima esto definidas em (1, ) e satisfazem as condies do enunciado. Elas fornecem, portanto, uma possvel soluo para o item b).

4)
Primeiramente, notemos que, para n 0, Fn Fn + 2 Fn2+1 = (1) n +1 . De fato,

F1 F3 F22 = 1 2 12 = (1) 2 e por induo supondo que


Fn Fn + 2 Fn2+1 = ( 1) n +1 temos que

Fn +1 Fn + 3 Fn2+ 2 = Fn +1 ( Fn + 2 + Fn +1 ) Fn2+ 2 = Fn2+1 Fn + 2 ( Fn + 2 Fn +1 ) = ( Fn Fn + 2 Fn2+1 ) = (1) n +1 = (1) n + 2 .


Dividimos o problema em dois casos; indicados pelas seguintes figuras: (i)

Fn Fn+2 Fn+1

Vn Vn+2 Vn+1 Fn+2 Fn+4 Fn+3

Se A a rea do tringulo sombreado, de lados Vn, Vn+1 e Vn+2, temos

A=

1 2

Fn + 2 Fn + 4
2

1 2

Fn Fn + 2
2

+ Fn +1 Fn + 2 +

Fn +1 Fn +3
2

Fn+2 Fn+4 = 1+ Fn+2 (Fn + 2Fn+1 ) + Fn+1Fn+3 = 1+ Fn+2 (Fn+2 + Fn+1 ) + Fn+1Fn+3 =
EUREKA! N 3, 1998

13

Sociedade Brasileira de Matemtica

= 1 + Fn + 2 Fn +3 + Fn +1 Fn + 3 = 1 + Fn2+3 Fn + 2 Fn + 4 Fn2 +3 = 1, o que ocorre


sempre que n mpar. (ii)
Fn Vn

Fn+2 Fn+1 Fn+2

Vn+1 Vn+2

Fn+4

Fn+3

Se A a rea do tringulo sombreado, de lados Vn, Vn+1 e V n+2, temos analogamente que

F F F F 1 1 FF n + 2 n + 4 = + n n + 2 + Fn +1 Fn + 2 + n +1 n +3 2 2 2 2 2 Fn + 2 Fn + 4 Fn2+3 = 1, o que ocorre sempre que n mpar. A=


Em qualquer dos casos, temos que a rea do tringulo de lados Vn, Vn+1 e 1 Vn+2 . 2

5)
Se x > c + 1, ento

x 2 x = x ( x 1) > c + c c x 2 + c x 2 c > x e, portanto,

n +1

( x) > f n ( x) > c + 1 para todo n 0. Logo, se x pr-perodico,


r
p q

ento x c + 1 (*).

s com p, q, r , s Z e q, s > 0. Temos


EUREKA! N 3, 1998

Agora, sejam c =

, onde ( r , s ) = 1, e x =

, onde ( p, q ) = 1,

14

Sociedade Brasileira de Matemtica

s( x 2 + c) = Se x 2 + c =

sp 2 q2

+r

u , u , v Z, v 0, ento v

q2 su sp 2 = 2 + r svp 2 = q 2 (su rv) q 2 svp2 q 2 sv sv q 2 v . v s q


Se q > s, o denominador v de x 2 + c maior ou igual a denominador de x isto , o denominador de

> q, que o s f n +1 ( x ) maior que o

q2

denominador de f n ( x ), n 0, e, portanto, se x pr-peridico, ento seu denominador no mximo s (**). De (*) e (**), segue que h apenas um nmero finito de pontos prperidicos racionais.

6)
Em primeiro lugar, observe que as imagens dos vrtices de um tringulo equiltero de lado 1 formam tambm um tringulo eqiltero de lado 1. Assim, dados dois tringulos eqilteros de lado 1 com um lado em comum, os vrtices opostos ao lado comum podem ter mesma imagem ou imagens diferentes distando AA' =

3. Em outras palavras, se A e A' so pontos tais que

3. ento d ( f ( A), f ( A' )) {0, 3}. Vamos mostrar que, de fato,

d ( f ( A), f ( A' )) = 3. Se f ( A) = f ( A' ), ento tomando B com AB = 1 e A'


B = 3 , teramos d ( f ( A), f ( B)) = 1 d ( f ( A' ), f ( B)) = 1, o que seria absurdo. Assim, d(A,A' ) = d( f (A), f (A' )) = 3 d( f (A), f (A' )) = 3. Desta forma qualquer reticulado triangular formado por vrtices de tringulos eqilteros de lado 1 de interiores disjuntos e cobrindo o plano preservado por f, no seguinte sentido: a imagem deste reticulado tambm ser outro reticulado do mesmo tipo. Em particular, pontos a distncia n so levados em pontos tambm distncia n, n N. Este ltimo fato mostra que tringulos de lados 1,
n 2 + n 1 que tm rea

n 2 n +1

3 / 4 so preservados pela funo f , j que seus vrtices esto em reticulado triangular de lado 1.

EUREKA! N 3, 1998

15

Sociedade Brasileira de Matemtica

B
1

1 1

A n
AB = n 2 n + 1 AC = n 2 + n + 1

Utilizando um procedimento anlogo ao anterior, vamos agora considerar a imagem dos vrtices de dois tringulos deste tipo com o lado de medida

n 2 + n + 1 em comum. Sendo X e Y os vrtices destes tringulos opostos ao lado comum, temos novamente que XY = n d ( f ( X ), f (Y )) = 0 ou

d ( f ( X ), f (Y )) = XY = n , onde
n =
3 n2 + n +1

o dobro da altura dos tringulos considerados em relao ao lado comum. Vamos demonstrar que os pontos distncia n tm, de fato, imagens distintas. Seja kn tal que k n n < 1 ( k n + 1) n . Sendo

d ( A0 , A1 ) = n , considere pontos para d ( Ai , Ai +1 ) = n 0 i kn e

Ai , 2 i k n + 1

tais

que

d ( A0 , Akn +1 ) = 1

Temos

d ( f ( A0 , f ( Akn +1 )) = 1 e, portanto,
1 d ( f ( Ai ), f ( Ai +1 )) d ( f ( A0 ), f ( A1 )) + k n n ,
i =0 kn

Se d ( f ( A0 ), f ( A1 )) fosse 0, ento 1 k n n < 1, o que seria absurdo assim,

XY = n d ( f ( X ), f (Y )) = n .

Como

antes,

temos

que

XY

k n d ( f ( X ), f (Y )) = k n para k N.

EUREKA! N 3, 1998

16

Sociedade Brasileira de Matemtica

Agora, suponha que existam X e Y tais que d ( f ( X ), f (Y )) d ( X , Y ). Sejam n N tal que 4 n < d ( f ( X ), f (Y )) d ( X , Y ) e P R 2 com

d ( P, X )

N, d ( P, Y ) < 2 n .

Tome Q R 2 com d ( P, Q ) = d (Y , Q ) = n d ( f ( P ), f (Q )) =

d ( f (Y ), f (Q )) = n d ( f ( P ), f (Y )) 2 n e como d ( P, X ) = d ( f ( P), f ( X )), temos d ( f ( X ), f (Y )) d ( X , Y ) d ( f ( X ), f (Y )) d ( f ( X ), f ( P)) +

d ( X , P) d ( X , Y ) d ( f (Y ), f ( P)) + d ( P, Y ) < 4 n , absurdo.


Obs: As funes f : R2 R2 que satisfazem as condies do enunciado so chamadas isometrias, e so composies de translaes com rotaes e / ou reflexes.

Voc sabia

Que o nmero de pessoas que j

apertaram a mo de outras pessoas um nmero mpar de vezes par

??

EUREKA! N 3, 1998

17

Sociedade Brasileira de Matemtica

13a. OLIMPADA IBEROAMERICANA DE MATEMTICA


Repblica Dominicana 1998 - Problemas e resultados
Primeiro dia Durao da Prova: 4 h e 30 minutos.
PROBLEMA 1

So dados 98 pontos sobre uma circunferncia. Maria e Jos jogam alternadamente da seguinte maneira: cada um deles traa um segmento unindo dois dos pontos dados que no tenham sido unidos entre si anteriormente. O jogo termina quando os 98 pontos tenham sido usados como extremos de um segmento pelo menos uma vez. O vencedor a pessoa que faz o ltimo trao. Se o Jos comea o jogo, quem pode garantir a sua prpria vitria?
PROBLEMA 2

A circunferncia inscrita no tringulo ABC tangente aos lados BC, CA e AB nos pontos D, E e F, respectivamente. AD corta a circunferncia num segundo ponto Q. Demonstrar que a reta EQ passa pelo ponto mdio de AF se e somente se AC = BC .
PROBLEMA 3

Encontrar o menor nmero natural n com a seguinte propriedade: entre quaisquer n nmeros distintos do conjunto {1, 2, , 999} pode-se escolher quatro nmeros diferentes a, b, c, d, tais que a + 2b + 3c = d. Segundo dia Durao da Prova: 4 h e 30 minutos.
PROBLEMA 4

Em volta de uma mesa redonda esto sentados representantes de n pases (n 2), satisfazendo a seguinte condio: se duas pessoas so do mesmo pas, ento, seus respectivos vizinhos da direita no podem ser de um mesmo pas. Determinar, para cada n, o nmero mximo de pessoas que pode haver em volta da mesa.
EUREKA! N 3, 1998

18

Sociedade Brasileira de Matemtica

PROBLEMA 5

Encontrar o maior valor possvel n para que existam pontos distintos P1, P2, P3, , Pn no plano, e nmeros reais r1, r2, , rn de modo que a distncia entre quaisquer dois pontos diferentes Pi e Pj seja ri + rj.
PROBLEMA 6

Seja a raiz positiva da equao t2 1998t 1 = 0. Define-se a sucesso x0, x1, x2, , xn , por: xo = 1 x n +1 = [x n ], para n = 0,1,2,... Encontrar o resto da diviso de x1998 por 1998.

Nota: [x] indica a parte inteira de x, ou seja, [x] o nico inteiro k tal que k x < k + 1. RESULTADOS
A equipe Brasileira teve uma excelente participao na 13a. Olmpada Iberoamericana de Matemtica realizada em Repblica Dominicana de 18 a 27 de setembro na qual participaram 18 pases. Os pases que obtiveram maior soma de pontos foram:

BRASIL CHILE ARGENTINA PERU MXICO ESPANHA

132 pontos 127 pontos 120 pontos 117 pontos 115 pontos 112 pontos

O Resultado da Equipe Brasileira BRA 1 BRA 2 BRA 3 BRA 4 Murali Srinivasam Vajapeyam Emanuel Augusto de Souza Carneiro Fabricio Shigueu Catae Mauricio Pereira Carrari Prata - 32 pontos Ouro - 37 pontos Ouro - 35 pontos Bronze - 28 pontos

Cada um dos seis problemas da prova vale 7 pontos.


EUREKA! N 3, 1998

19

Sociedade Brasileira de Matemtica

39a. OLIMPADA INTERNACIONAL DE MATEMTICA


Solues
PROBLEMA 1

No quadriltero convexo ABCD, as diagonais AC e BD so perpendiculares e os lados opostos AB e DC no so paralelos. Sabemos que o ponto P, onde se intersectam as mediatrizes de AB e DC, est no interior de ABCD. Prove que ABCD um quadriltero inscritvel se, e somente se, os tringulos ABP e CDP tm reas iguais.

SOLUO

Suponha primeiro que ABCD


seja inscritivel. Como AC BD temos

AB + CD = . Claramente o centro O do crculo circunscrito pertence s mediatrizes de AB e DC, logo P = O, e como rea de

OAB =

1 1 2 r sen AB = r 2 sen CD = rea de OCD (onde r o raio do 2 2

crculo), vale a primeira implicao. Suponha agora que ABCD no seja inscritvel. Suponha sem perda de generalidade que PC < PA. Seja Q o ponto de interseo de AC e BD. Prolongamos QC e QD at intersectarmos o crculo de centro p e raio PA = PB em novos pontos C e D . Como AC e BD so perpendiculares, pela primeira implicao sabemos que rea de PAB = rea de PCD, mas CD > CD ( CD hipotenusa do tringulo retngulo QCD, de catetos maiores que o tringulo retngulo QCD, do qual CD hipotenusa), e d(P, CD) > d (P, CD) (de fato, C e D esto no mesmo semiplano determinado pela reta CD , distinto do semiplano ao qual pertence P, e d (P, CD) = d(P, M), onde M o ponto mdio de CD, e portanto pertence ao mesmo semiplano que C e D , logo d(P, CD) < d(P, M) = = d (P, CD )). Portanto rea de PCD > rea de PCD, absurdo, pois estamos supondo que rea de PAB = rea de PCD.

PROBLEMA 2

Numa competio, existem a concorrentes e b juzes, onde b 3 um inteiro mpar. Cada juiz avalia cada um dos concorrentes, classificando-o como "aprovado" ou "reprovado". Suponha que k um nmero tal que as
EUREKA! N 3, 1998

20

Sociedade Brasileira de Matemtica

classificaes dadas por dois juzes quaisquer coincidem no mximo para k k b 1 . concorrentes. Prove que a 2b
SOLUO

Para cada um dos candidatos, se j o nmero de juizes que o aprovam, o nmero de pares de juzes que tem julgamentos coincidentes em relao a
2 2 2 ele C 2 j + C b j C b +1 + C b 1 = 2 2

(b 1) 2 , de modo que o nmero total de 4

a(b 1) 2 , que, por outro 4 b(b 1) 2 =k . Assim, lado, por hiptese, deve ser no mximo k C b 2 (b 1) a(b 1) 2 k b 1 . devemos ter kb 2 4 2b a
pares de julgamentos coincidentes no mximo
PROBLEMA 3

Para qualquer inteiro positivo n, seja d(n) o nmero de divisores positivos de n (incluindo 1 e n). Determine todos os inteiros positivos k tais que
SOLUO
1 2 Obsevemos inicialmente que se n = p1 p 2 ... p k k ( pi primos distintos)

d (n 2 ) = k para algum n. d ( n)

ento d (n) = (1 + 1 )(1 + 2 )...(1 + k ).

. Como o numerador (1 + 1 )(1 + 2 )...(1 + k ) mpar, se o resultado for inteiro deve ser mpar (e todos os i devem ser pares). Vamos mostrar que qualquer nmero natural mpar da forma desejada. Para isso, devemos mostrar que todo nmero mpar pode ser escrito como 2r + 1 produto de fraes da forma , r N, no necessariamente distintas. r +1 Faremos isso por induo. Seja m um nmero mpar, e seja 2s a maior
EUREKA! N 3, 1998

Assim, d ( n 2 ) / d ( n) =

(1 + 2 1 )(1 + 2 2 )...(1 + 2 k )

21

Sociedade Brasileira de Matemtica

potncia de 2 que divide m + 1. Temos portanto m = 2 s +1 q + 2 s 1 para algum q N, donde m(2 s 1) 2 2 s (2q + 1) 2 s +1 (q + 1) + 1 2 2 s (2q + 1) 2 s +1 (q + 1) + 1 m= = = 2 s 1 2s 1 2s 1 2 (2q + 1) 2 s (q + 1) + 1

2 2 s 1 (2q + 1) 2 s (q + 1) + 1 2 2 s +1 (2q + 1) 4(q + 1) + 1 (2q + 1). 2 2 s 2 (2q + 1) 2 s 1 (q + 1) + 1 2 s (2q + 1) (2q + 1) Como 2q + 1 < 2s + 1 q + 2s 1 = m, por hiptese de induo, 2q + 1 se 2r + 1 , e portanto m tambm. escreve como produto de fraes da forma r +1
Determine todos os pares (a, b) de inteiros positivos tais que ab2 + b + 7 divide a2b + a + b.
SOLUO PROBLEMA 4

a 2b + a + b inteiro ento ab 2 + b + 7 b(a 2 b + a + b) a (ab 2 + b + 7) b 2 7a = inteiro. Como ab 2 + b + 7 ab 2 + b + 7 b 2 7a b 2 7a Se b 2 7 a < b 2 < ab 2 + b + 7 temos que < 1 . =0 ab 2 + b + 7 ab 2 + b + 7 teremos b2 = 7a, donde b mltiplo de 7 (digamos b = 7t ), e (7t)2 = 7a nos d a = 7t2. fcil ver que (a, b) = (7t2, 7t) satisfaz as condies do enunciado para todo t inteiro positivo (temos nesse caso a 2b + a + b = t ). ab 2 + b + 7 b 2 7a b 2 7a 2 Se < 0 devemos ter b < 7 a e 1 (pois inteiro), e ab 2 + b + 7 ab 2 + b + 7 portanto 7a > 7a b2 ab2 + b + 7 7a > ab2 b2 < 7 b = 1 ou b = 2. b 2 7a 1 7a 57 = = 7 + Se b = 1, 2 , e devemos ter que a + 8 divide a+8 ab + b + 7 a + 8 57, com a inteiro positivo a + 8 = 19 ou a + 8 = 57 a = 11 ou
Se

EUREKA! N 3, 1998

22

Sociedade Brasileira de Matemtica

a = 49. Para a = 11 e b = 1 temos b = 1 temos

a 2 b + a + b 133 = = 7, e para a = 49 e ab 2 + b + 7 19

a 2 b + a + b 2451 = = 43. 57 ab 2 + b + 7

4 7a b 2 7a Se b = 2, = . Como 4 7a > 18 8a = 2 (4a + 9), se 2 ab + b + 7 4a + 9 4 7a 4a + 9


4 7a 4a + 9

inteiro negativo, devemos ter


= 1 4 7 a = 4 a 9 a = 13 3 N.

Assim, as solues so dadas por (a, b) = (7t 2 ,7t ), t N; (a, b) = ( 11,1 ) e (a, b) = (49,1).

PROBLEMA 5

Seja I o incentro do tringulo ABC. A circunferncia inscrita no tringulo ABC tangente aos lados BC, CA e AB nos pontos K, L e M, respectivamente. A reta que passa por B, paralela ao segmento MK, intersecta as retas LM e LK nos pontos R e S, respectivamente. Prove que o ngulo RIS agudo.

SOLUO

__ __ __ K temos BI MK , e portanto Como BM = BK e BI bissetriz de MB

S agudo, o que equivalente a BI RS . Queremos mostrar que RI 2 2 2 RI + SI > RS , o que equivale a BR + BI + BS + BI


2 2 2 2

__

__

> ( BR + BS ) 2 = BR + 2 BR BS + BS ,
BI
2

e portanto devemos provar que

> BR BS .

= BA C , B = AB A, temos KB C e C = BC S = MB R = B , Se A 2
EUREKA! N 3, 1998

23

Sociedade Brasileira de Matemtica

B = KS

A 2

(e portanto

B = SK

C 2

B = ) e MR

C 2

(e portanto

R = A ). Assim, os tringulos MBR e SBK so semelhantes e BM 2


BR BM = BK BS

, donde

BR BS = BM BK = BM

< BI

(pois

BI

hipotenusa do tringulo retngulo BMI ).

PROBLEMA 6

Considere todas as funes f definidas no conjunto N dos inteiros positivos, com valores no mesmo conjunto, que satisfazem f (t 2 f ( s )) = s ( f (t )) 2 , para todos s e t em N. Determine o menor valor possvel de f(1998)

SOLUO

Dizemos que h : N N estritamente multiplicativa se h(xy) = h(x) h(y), para quaisquer x, y N, e dizemos que h uma involuo se h(h(x)) = x para todo x N. facil ver que se f satisfaz a involuo estritamente multiplicativa ento f satisfaz a condio do enunciado: f (t2 f (s)) = (f (t)2 f (f (s)) = s (f (t))2. Podemos definir f : N N estritamente multiplicativa por
1 2 k 1 k f ( p1 p 2 ... p ( pi primos distintos), onde f (2) = 3, k ) = f ( p1 ) ... f ( p k ) f (3) = 2, f (37) = 5, f (5) = 37 e f (p) = p, para todo p primo no pertencente a {2, 3, 5, 37}, e teremos f (1998) = f ( 2 33 37 ) = f (2) f (3)3 f (37) = 3 23 5 = 120. Vamos provar que 120 menor valor possvel para f (1998).

Fazendo t = 1 temos f (f (s )) = s f (1)2, s N. Em particular, f injetiva, pois f (s) = f (u) f (f (s)) = f ( f (u)) s f (1)2 = u f (1)2 s = u. Temos ainda f (t 2 f (1)) = f (t)2 para todo t N ( fazendo s = 1), e portanto temos f (t2 f (s) 2) = f (t2 f (s2 f (1))) = s2 f (1) f (t)2, e fazendo s = f (u) temos f (t2 ( f ( f (u))2)= f (u)2 f (1) f (t)2. Assim, provamos que f (t 2 u2 f (1)4) = = ( f (u) f (t))2 f (1), para quaisquer u, t N. Portanto, se ut = xy, f (t 2 u2 f (1)4) = f (x2 y2 f (1)4), logo ( f (u) f (t))2 f (1) = ( f (x) f (y))2 f (1) f (u) f (t) = f (x) f (y). Como x2 1 = x x, f (x2) f (1) =
EUREKA! N 3, 1998

24

Sociedade Brasileira de Matemtica

f (x)2, x N. Se pk uma potncia de primo que divide f (1), e p r a maior potncia de p que divide f (x) para todo x N, temos que f (x)2 mltiplo de pr pk f (x) mltiplo de p
r +k 2

onde denota o menor inteiro que

maior ou igual a , para todo x N, o que absurdo se r < k (pois teramos

r + k > r ). Logo pk divide f (x) para todo x N, e portanto f (1) divide 2 f (x), para todo x N. Como xy 1 = x y , f (xy) f (1) = f (x) f (y)
f ( xy ) f ( x ) f ( y ) f ( x) temos que g = . Definindo g : N N, g (x) = f (1) f (1) f (1) f (1) estritamente multiplicativa, g injetiva, g (1) = 1 e g (x) f (x) para todo x N. Temos g(1998) = g(2 33 37) = g(2) g(3)3 g(37). Observemos agora que g(2), g(3) e g(37) devem ser naturais distintos maiores que 1, e no podemos ter {2, 4} {g(2), g(3), g(37)}, pois se g( p) = 2 e g(q) = 4 com {p, q} {2, 3, 37} teramos g(p2) = g(p)2 = g(q) p2 = q, absurdo. Assim g(1998) = g(2) g(3)3 g(37) = g(2) g(3) g(37) g(3)2 2 3 5 g(3)2 2 3 5 22 = 120, logo f (1998) 120, como afirmamos

Que o matemtico Harald Bohr (irmo do fsico Niels Bohr), famoso por sua teoria das funes quase-peridicas, era um consagrado jogador de futebol? Ele jogou no meio-de-campo da seleo da Dinamarca, que ganhou a medalha de prata nos jogos Olmpicos de Londres, em 1908, quando seu time derrotou a Frana pela contagem de 17 a 1

Voc sabia

(!).

EUREKA! N 3, 1998

25

Sociedade Brasileira de Matemtica

O PRINCPIO DA INDUO
Elon Lages Lima
Nvel Avanado.
INTRODUO

O Princpio da Induo um eficiente instrumento para a demonstrao de fatos referentes aos nmeros naturais. Por isso deve-se adquirir prtica em sua utilizao. Por outro lado, importante tambm conhecer seu significado e sua posio dentro do arcabouo da Matemtica. Entender o Princpio da Induo praticamente o mesmo que entender os nmeros naturais. Apresentamos abaixo uma breve exposio sobre os nmeros naturais, onde o Princpio da Induo se insere adequadamente e mostra sua fora terica antes de ser utilizado na lista de exerccios propostos ao final.
1. A SEQNCIA DOS NMEROS NATURAIS

Os nmeros naturais constituem um modelo matemtico, uma escala padro, que nos permite a operao de contagem. A seqncia desses nmeros uma livre e antiga criao do esprito humano. Comparar conjuntos de objetos com essa escala abstrata ideal o processo que torna mais precisa a noo de quantidade; esse processo (a contagem) pressupe portanto o conhecimento da seqncia numrica. Sabemos que os nmeros naturais so 1, 2, 3, 4, 5, A totalidade desses nmeros constitui um conjunto, que indicaremos com o smbolo N e que chamaremos de conjunto dos naturais. Portanto N = {1, 2, 3, 4, 5,}. Evidentemente, o que acabamos de dizer s faz sentido quando j se sabe o que um nmero natural. Faamos de conta que esse conceito nos desconhecido e procuremos investigar o que h de essencial na seqncia 1, 2, 3, 4, 5 . Deve-se a Giussepe Peano (1858-1932) a constatao de que se pode elaborar toda a teoria dos nmeros naturais a partir de quatro fatos bsicos, conhecidos atualmente como os axiomas de Peano. Noutras palavras, o conjunto N dos nmeros naturais possui quatro propriedades fundamentais, das quais resultam, como conseqncias lgicas, todas as afirmaes verdadeiras que se podem fazer sobre esses nmeros. Comearemos com o enunciado e a apreciao do significado dessas quatro proposies fundamentais a respeito dos nmeros naturais.
EUREKA! N 3, 1998

26

Sociedade Brasileira de Matemtica

2. OS AXIOMAS DE PEANO

Um matemtico profissional, em sua linguagem direta e objetiva, diria que o conjunto N dos nmeros naturais caracterizado pelas seguintes propriedades:

A. B. C. D.

Existe uma funo s : N N, que associa a cada n N um elemento s(n) N, chamado o sucessor de n. A funo s : N N injetiva. Existe um nico elemento 1 no conjunto N, tal que 1 s(n) para todo n N. Se um subconjunto X N tal que 1 N e s(X) X (isto , n X s(n) X), ento X = N.

Observe que, como estamos chamando de N o conjunto dos nmeros naturais, a notao n N significa que n um nmero natural. As afirmaes A, B, C e D so os axiomas de Peano. A notao s(n) provisria. Depois de definirmos adio, escreveremos n + 1 em vez de s(n). Como concesso fraqueza humana, nosso matemtico nos faria a gentileza de reformular os axiomas de Peano em linguagem corrente, livre de notao matemtica. E nos diria ento que as afirmaes acima significam exatamente o mesmo que estas outras:

A'. B'. C'. D'.

Todo nmero natural possui um nico sucessor, que tambm um nmero natural. Nmeros naturais diferentes possuem sucessores diferentes. (Ou ainda: nmeros que tm o mesmo sucessor so iguais.) Existe um nico nmero natural que no sucessor de nenhum outro. Este nmero representado pelo smbolo 1 e chamado de "nmero um". Se um conjunto de nmeros naturais contm o nmero 1 e, alm disso, contm o sucessor de cada um de seus elementos, ento esse conjunto coincide com N, isto , contm todos os nmeros naturais.

A partir da, retomamos a palavra para dizer que o sucessor de 1 chama-se "dois", o sucessor de dois chama-se "trs", etc. Nossa civilizao progrediu ao ponto em que temos um sistema de numerao, o qual nos
EUREKA! N 3, 1998

27

Sociedade Brasileira de Matemtica

permite representar, mediante o uso apropriado dos smbolos 0, 1, 2, 3, 4, 5, 6, 7, 8 e 9, todos os nmeros naturais. Alm disso, nossa linguagem tambm fornece nomes para os primeiros termos da seqncia dos nmeros naturais. (Nmeros muito grandes no tm nomes especficos, ao contrrio dos menores como "mil novecentos e noventa e oito". Quem sabe, por exemplo, o nome do nmero de tomos do universo?) Voltando a usar a notao s(n) para o sucessor do nmero natural n, teremos ento 2 = s(1), 3 = s(2), 4 = s(3), 5 = s(4), etc. Assim, por exemplo, a igualdade 2 = s(1) significa apenas que estamos usando o smbolo 2 para representar o sucessor de 1. A seqncia dos nmeros naturais pode ser indicada assim: s s s s s 1 2 3 4 5 As flechas ligam cada nmero ao seu sucessor. Nenhuma flecha aponta para 1, pois este nmero no sucessor de nenhum outro. O diagrama acima diz muito sobre a estrutura do conjunto N dos nmeros naturais.
3. O AXIOMA DA INDUO

Um dos axiomas de Peano, o ltimo, possui claramente uma natureza mais elaborada do que os demais. Ele conhecido como o axioma da induo. Faremos dele uma anlise detida, acompanhada de comentrios. O significado informal do axioma D que todo nmero natural pode ser obtido a partir de 1 por meio de repetidas aplicaes da operao de tomar o sucessor. Assim, por exemplo, 2 o sucessor de 1, 3 o sucessor do sucessor de 1, etc. Para se entender melhor o axioma da induo util examinar o exemplo, no qual N = {1, 2, 3,} mas a funo s : N N modificada, pondo-se s(n) = n + 2. Ento, se comearmos com 1 e a este nmero aplicarmos repetidamente a operao de tomar o "sucessor" (nesta nova acepo) obteremos s(1) = 3, s(3) = 5, s(5) = 7, etc., e nunca chegaremos a qualquer nmero par. Portanto, o diagrama
s s s s s s 1 3 5 2 4 6

exibe uma funo injetiva s : N N para a qual no verdade que todo nmero natural n pode ser obtido, a partir de 1, mediante repetidas aplicaes da operao de passar de k para s(k).
EUREKA! N 3, 1998

28

Sociedade Brasileira de Matemtica

Dentro de um ponto de vista estritamente matemtico, podemos reformular o axioma da induo do seguinte modo: Um subconjunto X N chama-se indutivo quando s(X) X, ou seja, quando n X s(n) X, ou ainda, quando o sucessor de qualquer elemento de X tambm pertence a X. Dito isto, o axioma da induo afirma que o nico subconjunto indutivo de N que contm o nmero 1 o proprio N. No exemplo acima, os nmeros mpares 1, 3, 5, formam um conjunto indutivo que contm o elemento 1 mas no igual a N. O papel fundamental do axioma da induo na teoria dos nmeros naturais e, mais geralmente, em toda a Matemtica, resulta do fato de que ele pode ser visto como um mtodo de demonstrao, chamado o Mtodo de Induo Matemtica, ou Princpio da Induo Finita, ou Princpio da Induo, conforme explicaremos agora. Seja P uma propriedade que se refere a nmeros naturais. Um dado nmero natural pode gozar ou no da propriedade P. Por exemplo, seja P a propriedade de um nmero natural n ser sucessor de outro nmero natural. Ento 1 no goza da propriedade P, mas todos os demais nmeros gozam de P. O Princpio da Induo diz o seguinte:

Princpio da Induo: Seja P uma propriedade referente a nmeros naturais. Se 1 goza de P e se, alm disso, o fato de o nmero natural n gozar de P implica que seu sucessor s(n) tambm goza, ento todos os nmeros naturais gozam da propriedade P. Para ver que o Princpio da Induo verdadeiro (uma vez admitidos os axiomas de Peano) basta observar que, dada a propriedade P cumprindo as condies estipuladas no enunciado do Princpio, o conjunto X dos nmeros naturais que gozam da propriedade P contm o nmero 1 e indutivo. Logo X = N, isto , todo nmero natural goza da propriedade P. As propriedades bsicas dos nmeros naturais so demonstradas por induo. Comecemos com um exemplo bem simples. Exemplo 1. Entre afirmao de que provaremos agora. nmero natural n, afirmao n s(n).
EUREKA! N 3, 1998

os axiomas de Peano no consta explicitamente a todo nmero diferente do seu sucessor, a qual Seja P esta propriedade. Mais precisamente, dado o escrevamos P(n) para significar, abreviadamente, a Ento P(1) verdadeira, pois 1 s(1), j que 1 no
29

Sociedade Brasileira de Matemtica

sucessor de nmero algum; em particular, 1 no sucessor de si prprio. Alm disso, se supusermos P(n) verdadeira, isto , se admitimos que n s(n), ento s(n) s(s(n)), pois a funo s : N N injetiva. Mas a afirmao s(n) s(s(n) significa que P(s(n)) verdadeira. Assim, a verdade de P(n) acarreta a verdade de P(s(n)). Pelo Princpio da Induo, todos os nmeros naturais gozam da propriedade P, ou seja, so diferentes de seus sucessores. Nas demonstraes por induo, a hiptese de que a propriedade P vlida para o nmero natural n (da qual deve decorrer que P vale tambm para s(n)) chama-se hiptese de induo. O Princpio da Induo no utilizado somente como mtodo de demonstrao. Ele serve tambm para definir funes f: N Y que tm como dominio o conjunto N dos nmeros naturais. Para se definir uma funo f : X Y exige-se em geral que seja dada uma regra bem determinada, a qual mostre como se deve associar a cada elemento x X um nico elemento y = f(x) Y. Entretanto, no caso particular em que o domnio da funo o conjunto N dos nmeros naturais, a fim de definir uma funo f : N Y no necessrio dizer, de uma s vez, qual a receita que d o valor f(n) para todo n N. Basta que se tenha conhecimento dos seguintes dados: (1) O valor f (1); (2) Uma regra que permita calcular f (s(n)) quando se conhece f (n). Esses dois dados permitem que se conhea f (n) para todo nmero natural n. (Diz-se ento que a funo f foi definida por recorrncia.) Com efeito, se chamarmos de X o conjunto dos nmeros naturais n para os quais se pode determinar f (n), o dado (1) acima diz que 1 X e o dado (2) assegura que n X s(n) X. Logo, pelo axioma da induo, tem-se X = N. Obs. : Uma funo f : N Y cujo domnio o conjunto dos nmeros naturais chama-se uma seqncia ou sucesso de elementos de Y. A notao usada para uma tal seqncia (y1, y2,,yn,), onde se usa yn em vez de f(n) para indicar o valor da funo f no nmero n. O elemento yn .
4. ADIO E MULTIPLICAO DE NMEROS NATURAIS

A adio e a multiplicao de nmeros naturais so exemplos de funes definidas por recorrncia.


EUREKA! N 3, 1998

30

Sociedade Brasileira de Matemtica

Para definir a adio, fixaremos um nmero natural arbitrrio k e definiremos a soma k + n para todo n N. Fixado k, a correspondncia n k + n ser uma funo f: N N, f(n) = k + n, chamada "somar k". Ela se define por recorrncia, a partir dos seguintes dados: (S1) k + 1 = s(k) (S2) k + s(n) = s(k + n). Portanto, k + 1 , por definio, o sucessor de k. E, se conhecermos k + n, saberemos o valor de k + s(n): por definio, tem-se k + s(n) = s(k + n). Isto nos permite conhecer k + n para todo n N (e todo k N). Usando as notaes definitivas n + 1 em vez de s(n) e (k + n) + 1 em vez de s(k + n), a igualdade (S2) se escreve assim: (S2') k + (n + 1) = (k + n) +1. Assim, as igualdades (S1) e (S2) ou, equivalentemente, (S1) e (S2') definem por recorrncia a soma k + n de dois nmeros naturais quaisquer k e n. A multiplicao de nmeros naturais se define de modo anlogo adio. Fixado arbitrariamente um nmero natural k, a multiplicao por k associa a todo nmero mnatural n o produto n k, definido por induo da seguinte maneira: (P1) 1 k = k. (P2) (n + 1) k = nk + k. O produto nk escreve-se tambm nk e l-se "n vezes k". A definio acima diz portanto que uma vez k igual a k e n + 1 vezes k igual a n vezes k mais (uma vez) k . Assim, por definio, 2 k = k + k, 3 k = k + k + k, etc. Usa-se induo para provar as propriedades bsicas da adio e da multiplicao de nmeros naturais. Entre elas, destacam-se as seguintes, vlidas para quaisquer k, n, p N: Associatividade: k + ( n + p ) = ( k + n ) + p e k ( n p ) = ( k n ) p Comutatividade: k+n=n+k e kn=nk Lei do Corte: k+n=k+pn=p e kn=kpn=p Distributividade: k ( n + p) = k n + k p. Omitiremos as demonstraes destes fatos. O leitor pode considerlas como exerccios sobre o mtodo da induo.
5. ORDEM

A adio de nmeros naturais permite introduzir uma relao de ordem em N. Dados os nmeros naturais m, n diremos que m menor do que
EUREKA! N 3, 1998

31

Sociedade Brasileira de Matemtica

n, e escreveremos m < n, para significar que existe p N tal que n = m + p. Neste caso, diz-se tambm que n maior do que m e escreve-se n > m para exprimir que se tem m < n. A notao m n significa que m < n ou m = n. Por definio, tem-se portanto m < m + p para quaisquer m, p N. Em particular, m < m + 1. Segue-se tambm da definio que 1 < n para todo nmero natural n 1. Com efeito, pelo axioma C, n 1 implica que n sucessor de algum nmero natural m, ou seja, n = m + 1 = 1 + m, logo n > 1. Assim, 1 o menor dos nmeros naturais. Provaremos a seguir as propriedades bsicas da relao de ordem m < n que definimos. A primeira delas a transitividade.

Teorema 1. (Transitividade.) Se m < n e n < p, ento m < p. Demonstrao: Se m < n, n < p ento n = m + k, p = n + r, logo p = (m + k) + r = m + (k + r), portanto m < p. Outra importante propriedade de relao de ordem que, dados dois nmeros naturais diferentes m, n, ou se tem m < n ou ento n < m. Esta propriedade pode ser reformulada de outra maneira, como segue. Diremos que os nmeros naturais m, n so comparveis quando se tem m = n, m < n ou n < m. Podemos ento enunciar o seguinte teorema. Teorema 2. (Comparabilidade.) Todo nmero natural n comparvel com qualquer nmero natural m. Demonstrao: Isto se prova por induo. O nmero 1 comparvel com qualquer outro nmero natural pois j sabemos que 1 < m para todo m 1. Suponhamos agora que o nmero n seja comparvel com todos os nmeros naturais. Mostremos, a partir da, que n + 1 tambm tem essa propriedade. Com efeito, seja m N tomado arbitrariamente. Sabemos que se tem m < n, m = n ou n < m. Examinemos cada uma dessas possibilidades: Se for m < n ento m < n + 1 por transitividade, pois sabemos que n < n + 1. Se for m = n, ento m < n + 1. Se for n < m ento m = n + p. Neste caso, h duas possibilidades. Ou se tem p = 1, donde m = n + 1, ou ento p > 1, logo p = 1 + p', e da m = (n + 1) + p' e conclumos que n + 1 < m. Em qualquer hiptese, vemos que n + 1 comparvel com qualquer nmero natural m. Por induo, fica provada a comparabilidade de quaisquer nmeros naturais m, n. A comparabilidade dos nmeros naturais complementada pela proposio abaixo.

EUREKA! N 3, 1998

32

Sociedade Brasileira de Matemtica

Teorema 3. (Tricotomia.) Dados m, n N, qualquer das afirmaes m < n, m = n, n < m exclui as outras duas. Demonstrao: Se tivssemos m < n e m = n, ento seria m = m + p, donde m + 1 = m + p + 1 e, cortando m, concluiramos que 1 = p + 1, um absurdo, pois 1 no sucessor de p. Portanto m < n (e analogamente, n < m) incompatvel com m = n. Do mesmo modo, se tivssemos m < n e n < m, ento teramos n = m + p e m = n + k, do que resultaria n = n + k + p, logo n + 1 = n + k + p + 1 e, cortando n, concluiramos que 1 = k + p + 1, um absurdo. O teorema seguinte mostra que n e n + 1 so nmeros consecutivos. Teorema 4. No existem nmeros naturais entre n e n + 1. Demonstrao: Se fosse possvel ter n < p < n + 1, teramos p = n + k e n + 1 = p + r, logo n + 1 = n + k + r. Cortando n, obteramos 1 = k + r. Por definio, isto significaria k < 1, o que absurdo, pois j vimos que k 1 k > 1. A conexo entre a relao de ordem e as operaes de adio e multiplicao dada pelo seguinte teorema: Teorema 5. (Monotonicidade.) Se m < n, ento m + p < n + p e mp < np. Demonstrao: Usando a definio de <, temos que m < n n = m + k n + p = (m + k) + p m + p < n + p. Analogamente, m < n n = m + k np = mp + kp np >mp. A recproca da monotonicidade a Lei do Corte para desigualdades: m + p < n + p m < n e mp < np m < n. O leitor poder prov-la por absurdo, usando a tricotomia e a prpria monotonicidade.
6. BOA ORDENAO

Dado o subconjunto A N, diz-se que o nmero natural a o menor (ou primeiro) elemento de a quando a A e, alm disso, a x, para todos os elementos x A. Por exemplo, 1 o menor elemento de N. De agora em diante, dado n N, indicaremos com In o conjunto dos nmeros naturais p tais que 1 p n. Assim, I1 = {1}, I2 = {1, 2}, I3 = {1, 2, 3} etc. As propriedades da relao de ordem m < n, demonstradas na seo anterior para os nmeros naturais (exceto o Teorema 4 que vale apenas para
EUREKA! N 3, 1998

33

Sociedade Brasileira de Matemtica

nmeros inteiros), so igualmente vlidas para os nmeros inteiros, racionais e, mais geralmente, para nmeros reais quaisquer. Existe, porm, uma propriedade de suma importncia que vlida para a ordem entre os nmeros naturais, mas sem equivalente para nmeros inteiros, racionais ou reais.

Teorema 6. (Princpio da Boa Ordenao.) Todo subconjunto no-vazio A N possui um menor elemento. Demonstrao: Sem perda de generalidade, podemos admitir que 1 A, pois caso contrrio 1 seria evidentemente o menor elemento de A. O menor elemento de A, cuja existncia queremos provar, dever ser da forma n + 1. Devemos pois encontrar um nmero natural n tal que n +1 A e, alm disso, todos os elementos de A so maiores do que n, logo maiores do que 1, 2, , n. Noutras palavras, procuramos um nmero natural n tal que In N A e n + 1 A. Com esse objetivo, consideramos o conjunto X = {n N; In N A}. Portanto, X o conjunto dos nmeros naturais n tais que todos os elementos de A so maiores do que n. Como estamos supondo que 1 A, sabemos que 1 X. Por outro lado, como A no vazio, nem todos os nmeros naturais pertencem a X, ou seja, temos X N. Pelo axioma D, vemos que o conjunto X no indutivo, isto , deve existir algum n X tal que n + 1 X Isto significa que todos os elementos de A so maiores do que n mas nem todos so maiores do que n + 1. Como no h nmeros naturais entre n e n + 1, conclumos que n + 1 pertence a A e o menor elemento de A.

O Princpio da Boa Ordenao pode muitas vezes ser usado em demonstraes, substituindo o Princpio da Induo. Vejamos um exemplo. Dissemos anteriormente que um subconjunto X N chama-se indutivo quando n X n + 1 X, ou seja, quando X contm o sucessor de cada um dos seus elementos. O Princpio da Induo afirma que se um conjunto indutivo X contm o nmero 1 ento X contm todos os nmeros naturais. Vamos usar o Princpio da Boa Ordenao para provar que se um conjunto indutivo X contm o nmero a, ento X contm todos os nmeros naturais maiores do que a. A prova desta afirmao se faz por absurdo, como ocorre em geral quando se usa a boa ordenao. Suponhamos ento que existam nmeros
EUREKA! N 3, 1998

34

Sociedade Brasileira de Matemtica

naturais, maiores do que a, no pertencentes ao conjunto indutivo X. Seja b o menor desses nmeros. Como b > a, podemos escrever b = c + 1, onde, pela definio de b, tem-se necessariamente c X. Mas, como X indutivo, isto obriga que b = c + 1 X, uma contradio. A proposio qua acabamos de demonstrar pode ser enunciada da seguinte forma:

Teorema 7: (Princpio da Induo Generalizado.) Seja P uma propriedade referente a nmeros naturais, cumprindo as seguintes condies: (1) O nmero natural a goza da propriedade P; (2) Se um nmero natural n goza da propriedade P ento seu sucessor n + 1 tambm goza de P. Ento todos os nmeros naturais maiores do que ou iguais a a gozam da propriedade P. Exemplo 2. Vejamos uma situao simples onde se emprega o Princpio da Induo Generalizado. Trata-se de provar que 2n + 1 < 2n, para todo n 3. Esta afirmao, (que falsa para n = 1 ou n = 2), vale quando n = 3. Supondo-a vlida para um certo n 3, mostremos que da decorre sua validez para n + 1. Com efeito, 2(n + 1) + 1 = (2n + 1) + 2 < 2n + 2 < 2n + 2n = 2n + 1. (Na primeira desigualdade, usamos a hiptese de induo.) Exemplo 3. Usando a desigualdade 2n + 1 < 2n, qua acabamos de provar para n 3, podemos demonstrar que n2 < 2n para todo n 5, empregando novamente o Princpio da Induo Generalizado. Com efeito, vale 52 < 25 pois 25 < 32. Supondo vlida a desigualdade n2 < 2n para um certo valor de n 5, da segue-se que (n + 1)2 = n2 + 2n + 1 < 2n + 2n + 1 (pela hiptese de induo) < 2n + 2n (pelo exemplo anterior) = 2n + 1. Portanto P(n) P(n + 1). Pelo Princpio de Induo Generalizado, segue-se que P(n) vale para todo n 5. Evidentemente, a desigualdade n2 < 2n falsa para n = 1, 2, 3, 4. O teorema abaixo contm outra aplicao do Princpio da Boa Ordenao. Teorema 8. Toda funo montona no-crescente f: N N constante a partir de um certo ponto. ( Isto , existe n0 N tal que f(n) = f(n0), para todo n n0.) Demonstrao: Seja n0 o menor elemento do conjunto X = {f(1), f(2), , f(n),}. Ento n > n0 f(n) f(n0) (porque a funo f no-crescente) o que acarreta que f(n) = f(n0) (porque f(n0) o menor elemento de X).

EUREKA! N 3, 1998

35

Sociedade Brasileira de Matemtica

Corolrio: Toda seqncia decrescente n1 > n2 > de nmeros naturais finita. Com efeito, do contrrio, pondo f(k) = nk, obteramos uma funo estritamente decrescente f : N N.
7. SEGUNDO PRINCPIO DA INDUO

Em algumas situaes, ao tentarmos fazer uma demonstrao por induo, na passagem de n para n + 1, sentimos necessidade de admitir que a proposio valha no apenas para n e sim para todos os nmeros naturais menores do que ou iguais a n. A justificativa de um raciocnio desse tipo se encontra no Teorema 9: (Segundo Princpio da Induo.) Seja X N um conjunto com a seguinte propriedade: Dado n N, se todos os nmeros naturais menores do que n (I) pertencem a X, ento n X. O segundo Princpio da Induo afirma que um conjunto X N com a propriedade (I) coincide com N. Demonstrao: Com efeito, supondo, por absurdo, que X N, isto , que N X , seja n o menor elemento do conjunto N X, ou seja, o menor nmero natural que no pertence a X. Isto quer dizer que todos os nmeros naturais menores do que n pertencem a X. Mas ento, pela propriedade (I), n pertence a X, uma contradio. Segue-se que N X = e X = N.

Obs. : Se um conjunto X N goza da propriedade (I), para que um nmero natural n no pertencesse a X seria necessrio que existisse algum nmero natural r < n tal que r X. Em particular, se n = 1, como no existe nmero natural menor do que 1, a hiptese 1 X no pode ser cumprida. Noutras palavras, (I) j contm implicitamente a afirmao de que 1 X. Assim, ao utilizar o Segundo Princpio da Induo, no preciso estipular que X contm o nmero 1.
Toda propriedade P que se refira a nmeros naturais define um subconjunto X N, a saber, o conjunto dos nmeros naturais que gozam da propriedade P. (E reciprocamente, todo conjunto X N define uma propriedade referente a nmeros naturais, a saber, a propriedade de pertencer a X.) Deste modo, "propriedade" e "conjunto" so noes equivalentes. Por isso, natural que o Segundo Princpio da Induo possua a formulao seguinte, onde ele aparece como o
EUREKA! N 3, 1998

36

Sociedade Brasileira de Matemtica

Teorema 10: (Segundo mtodo de demonstrao por induo.) Seja P uma propriedade referente a nmeros naturais. Dado n N, se a validade de P para todo nmero natural menor do que n implicar que P verdadeira para n, ento P verdadeira para todos os nmeros naturais. Demonstrao: Com efeito, nas condies do enunciado, o conjunto X dos nmeros naturais que gozam da propriedade P satisfaz a condio (I) do Segundo Princpio da Induo, logo X = N e P vale para todos os nmeros naturais. Aplicaremos agora o Segundo Princpio da Induo para demonstrar um fato geomtrico. No exemplo a seguir, usamos os nmeros naturais como instrumento de contagem, isto , como nmeros cardinais, pois empregamos expresses do tipo um polgono de n lados". (Vide seo 6.) Sabe-se que, traando diagonais internas que no se cortam, pode-se decompor qualquer polgono em tringulos justapostos. Isto evidente quando o polgono convexo: basta fixar um vrtice e traar as diagonais a partir dele. Se o polgono no convexo, a prova requer mais cuidados. (Vide "Meu Professor de Matemtica", pag. 109.) O leitor pode experimentar com um polgono no-convexo e verificar qua h muitas maneiras diferentes de decomp-lo em tringulos justapostos mediante diagonais internas. Mas vale o resultado seguinte, no qual usaremos o Segundo Princpio da Induo. Exemplo 4. Qualquer que seja a maneira de decompor um polgono P, de n lados, em tringulos justapostos por meio de diagonais internas que no se intersectam, o nmero de diagonais utilizadas sempre n 3.
Com efeito, dado n, suponhamos que a proposio acima seja verdadeira para todo polgono com menos de n lados. Seja ento dada uma decomposio do polgono P, de n lados, em tringulos justapostos, mediante diagonais internas. Fixemos uma dessas diagonais. Ela decompe P como reunio de dois polgonos justapostos P1, de n1 lados, e P2, de n2 lados, onde n1 < n e n2 < n, logo a proposio vale para os polgonos P1 e P2. Evidentemente, n1 + n2 = n + 2.

EUREKA! N 3, 1998

37

Sociedade Brasileira de Matemtica

P1

P2

As d diagonais que efetuam a decomposio de P se agrupam assim: n1 3 delas decompem P1, n2 3 decompem P2 e uma foi usada para separar P1 de P2. Portanto d = n1 3 + n2 3 + 1 = n1 + n2 5. Como n1 + n2 = n + 2, resulta que d = n 3. Isto completa a demonstrao.

Observaes:
1. Para habituar-se com o mtodo de demonstrao por induo preciso pratic-lo muitas vezes, a fim de perder aquela vaga sensao de desonestidade que o principiante tem quando admite que o fato a ser provado verdadeiro para n, antes de demonstr-lo para n + 1. Pratique tambm (com moderao) o exerccio de descobrir o erro em paradoxos que resultam do uso inadequado do mtodo de induo. Vejamos dois desses sofismas:

2.

Exemplo 5. Todo nmero natural pequeno. Ora, 1 certamente pequeno. E se n pequeno, n + 1 no vai subitamente tornar-se grande, logo tambm pequeno. (O erro aqui consiste em que a noo "nmero pequeno" no bem definida.) Exemplo 6. Toda funo f : X Y, cujo domnio um conjunto finito X, constante. Isto obviamente verdadeiro se X tem apenas 1 elemento. Supondo a afirmao verdadeira para todos os conjuntos com n elementos, seja f : X Y definida num conjunto X com n + 1 elementos. Considere um elemento a X. Como X' = X {a} tem n elementos, f assume o mesmo valor c Y em todos os elementos de X'. Agora troque a por um outro elemento b X'. Obtm-se X'' = X {b} um conjunto com n elementos
EUREKA! N 3, 1998

38

Sociedade Brasileira de Matemtica

(entre os quais a). Novamente pela hiptese de induo, f constante e igual a c em X''. Logo f (a) = c e da f : X Y constante. (Aqui o erro reside no uso inadequado da hiptese de induo. O raciocnio empregado supe implicitamente que X tem pelo menos 3 elementos. Na realidade, no vale a implicao P(1) P(2).) O perigo de fazer generalizaes apressadas relativamente a asseres sobre nmeros naturais fica evidenciado com o seguinte exemplo:

Exemplo 7. Considere o polinmio p(n) = n2 n + 41 e a afirmao "o valor de p(n) sempre um primo para n = 0, 1, 2, 3, ". Embora isso seja verdadeiro para n = 0, 1, 2, , 40, temos p(41) = 412 41 + 41 = 412 no primo, logo a afirmao no verdadeira.
Semelhantemente, a expresso q(n) = n2 79n + 1601 fornece primos para n = 1, 2, , 79, mas q(80) = 802 79 80 + 1601 = 1681 no primo, pois divisvel por 41. A moral da histria : S aceite que uma afirmao sobre os nmeros naturais realmente verdadeira para todos os naturais se isso houver de fato sido demonstrado!
8. NMEROS CARDINAIS

Vamos agora mostrar como se usam os nmeros naturais para contar os elementos de um conjunto finito. O Princpio da Induo ser essencial. Lembremos que, dado n N, escrevemos In = {p N; p n}, portanto In = {1, 2, , n}. Uma contagem dos elementos de um conjunto no-vazio X uma bijeo f : In X. Podemos pr x1 = f(1), x2 = f(2),, xn = f(n) e escrever X = {x1, x2,xn}. Diz-se ento que X possui n elementos. O conjunto X chama-se um conjunto finito quando existe n N tal que X possui n elementos. Um exemplo bvio de conjunto finito In. Evidentemente, a funo identidade f: In In uma contagem dos elementos de In. Um exemplo de conjunto infinito o proprio conjunto N dos nmeros naturais, pois nenhuma funo f : In N pode ser sobrejetiva, no importa qual n se tome. De fato, dada f, tomamos k = f(1) + f(2) ++ f(n) e vemos que k > f(x) para todo x In, logo k f(In), e f no sobrejetiva. A fim de que no haja ambigidade quando se falar do nmero de elementos de um conjunto finito X, necessrio provar que todas as
EUREKA! N 3, 1998

39

Sociedade Brasileira de Matemtica

contagens de X fornecem o mesmo resultado. Noutras palavras, dado o conjunto X, os nmeros naturais m, n e as bijees f : Im X, g : In X, devemos mostrar que se tem m = n. Comeamos observando que se f e g so bijees, ento = g1 f : Im In tambm uma bijeo. Basta portanto provar o seguinte:

Teorema 11. Dados m, n N, se : Im In uma bijeo, ento m = n. Demonstrao. Com efeito, chamemos de X o conjunto dos nmeros naturais n que tm a seguinte propriedade: s existe uma bijeo : Im In quando m = n. Evidentemente, 1 X. Suponhamos agora que n X. Dada uma bijeo : Im+1 In+1, duas coisas podem acontecer. Primeira: (m + 1) = n + 1. Neste caso, a restrio |Im : Im In uma bijeo, logo m = n, donde m + 1 = n + 1. Segunda: (m + 1) = b, com b < n + 1. Neste caso, consideramos a = 1(n + 1) e definimos uma nova bijeo : Im + 1 In + 1, pondo (m + 1) = n + 1, (a) = b e (x) = (x) para os demais elementos x Im + 1. Ento recamos no caso anterior e novamente conclumos que m + 1 = n + 1. Isto mostra que n X n + 1 X, logo X = N e a unicidade do nmero cardinal de um conjunto finito fica demonstrada.
Agora os nmeros naturais no so apenas elementos do conjuntopadro N, mas servem tambm para responder perguntas do tipo "quantos elementos tem o conjunto X?,"ou seja, podem ser usados tambm como nmeros cardinais. A adio de nmeros naturais se relaciona com a cardinalidade dos conjuntos por meio da seguinte proposio.

Teorema 12: Sejam X, Y conjuntos finitos disjuntos. Se X tem m elementos e Y tem n elementos, ento X Y tem m + n elementos. Demonstrao: Com efeito, se f : Im X e g : In Y so bijees, definimos uma bijeo h : Im+n X Y por h (x) = f (x) se 1 x m e h(x) = g(x) + m se m + 1 x m + n, o que conclui a demonstrao. Prova-se, por induo, que todo subconjunto de um conjunto finito X tambm finito e seu nmero de elementos menor do que ou igual ao de X (Veja E.L.Lima, "Anlise Real", vol 1, pag. 5.)
E conveniente incluir, por definio, o conjunto vazio entre os conjuntos finitos e dizer que o seu nmero de elementos zero. Embora zero
EUREKA! N 3, 1998

40

Sociedade Brasileira de Matemtica

no seja um nmero natural, ele passa a ser o nmero cardinal do conjunto vazio. Seguem-se algumas proposies que devem ser demonstradas por induo ou boa ordenao. Os dez ltimos exerccios foram sugeridos pelo Professor A. C. Morgado.

Exerccios:
1. Construa um esquema de setas comeando com os nmeros mpares, seguidos dos nmeros pares divisveis por 4 em ordem decrescente e, por fim, os pares no divisveis por 4 em ordem crescente. Noutras palavras, tome X = N e defina s : X X pondo s(n) = n + 2 se n no divisvel por 4, s(n) = n 2 se n for mltiplo de 4. Mostre que s : X X cumpre os axiomas A, B, C mas no D. 2. Defina, por recorrncia, uma funo f : N N estipulando que f (1) = 3 e f (n + 1) = 5. f (n) + 1. D uma formula explcita para f (n). 3. D uma frmula explcita para f : N N sabendo que f(1) = 1, f(2) = 5 e f (n + 2) = 3f (n + 1) 2f (n). 4. Seja X N um conjunto indutivo no-vazio. Mostre que existe a N tal que X = {n N; n a}. n(n + 1)(2n + 1) 5. Prove, por induo, que 12 + 2 2 + ... + n 2 = . 6 6. Num polgono com n 6 lados, o nmero de diagonais maior do que n. 7. Prove, por induo que [(n + 1)/n]n < n, para todo n 3. (Sugesto: Observe que (n + 2)/(n + 1) < ( n + 1)/n e eleve ambos os membros desta desigualdade potncia n + 1.) Conclua da que a seqncia 1, 2 , 3 3 , 4 4 , 5 5 ,... decrescente a partir do terceiro termo. 8. Prove, por induo a desigualdade de Bernoulli: (1 + a)n > 1 + na quando 1 + a > 0.

(n + 1) 2 9. Para todo n N, ponha x n = e prove, por induo que se n ( n + 2) n+2 tem x n < . Conclua, a partir da, que a seqncia de termo geral n +1 n n + 1 crescente. n

EUREKA! N 3, 1998

41

Sociedade Brasileira de Matemtica

n n + 2 Sugesto: observe que x n +1 = xn . n + n +3 1


10. Use a distributividade de duas maneiras diferentes para calcular (m + n )(1 + 1) e aplique em seguida a Lei do Corte para obter uma nova prova de que m + n = n + m. 11. Um conjunto S N, no-vazio, limitado superiormente, se existe um natural k tal que para todo natural x S, ento x k. Mostre que S possui um maior elemento. (Isto , existe m S tal que x m, para todo x S.) 12. Demonstre que a soma dos n primeiros nmeros mpares n2, ou seja, que 1 + 3 + 5 ++ (2n 1) = n2. 13. Prove que 2n 1 mltiplo de 3, para todo nmero natural n par. 14. Demonstre que, para todo nmero natural n, vale 1 1 1 1 1 + 1 + 1 + ...1 + n + 1. 1 2 3 n 15. Demonstre que 1 1 1 1 1 1 1 1 1 + + .. + = + + ... + . 200 199 200 101 102 2 3 4

1 2 16. Determine An se A = 2 4
17. Demonstre, usando o Princpio da Induo Finita, que p p + 1 p + n p + n + 1 p + p + ... + p = . p Este resultado comumente conhecido por Teorema das Colunas. (Por qu?). p 1 3 7 18. Considere a seqncia , , ,..., n ,..., onde 1 2 5 qn p n +1 = p n + 2q n e q n +1 = p n + q n . Demonstre que a) m.d.c (pn, qn) = 1;

EUREKA! N 3, 1998

42

Sociedade Brasileira de Matemtica

b)

pn o inteiro mais prximo de

(1 + 2 ) n e qn o inteiro mais prximo 2

de

2 (1 + 2 ) n . 4

19. [A Torre de Hani.] So dados trs suportes A, B e C. No suporte A esto encaixados n discos cujos dimetros, de baixo para cima, esto em ordem estritamente decrescente. Mostre que possvel, com 2n 1 movimentos, transferir todos os discos para o suporte B, usando o suporte C como auxiliar, de modo que jamais, durante a operao, um disco maior fique sobre um disco menor. 20. Demonstre que 2n < n!, para n 4. 21. Demonstre que 2n3 > 3n2 + 3n + 1 para n 3. 22. Considere n retas em um plano. Mostre que o "mapa" determinado por elas pode ser colorido com apenas duas cores sem que duas regies vizinhas tenham a mesma cor.

EUREKA! N 3, 1998

43

Sociedade Brasileira de Matemtica

FRAES CONTNUAS, REPRESENTAES DE NMEROS E APROXIMAES


Carlos Gustavo Moreira Nvel Avanado.
INTRODUO

A teoria de fraes contnuas um dos mais belos temas da matemtica elementar, sendo ainda hoje assunto de pesquisa recente (incluindo a do autor destas linhas). O objetivo deste artigo servir como referncia didtica em portugus a nvel secundrio sobre o assunto. Nas incluses N Z Q R a passagem de Q para R sem dvida a mais complicada conceitualmente, e a representao de um nmero real est diretamente ligada propria noo de nmero real. De fato, o conceito de nmero natural quase um conceito primitivo no ensino secundrio. J um nmero inteiro um nmero natural com um sinal que pode ser + ou , e um nmero racional a razo entre um nmero inteiro e um natural no nulo. Por outro lado, dizer o que um nmero real tarefa bem mais complicada, mas h coisas que podemos dizer sobre eles. Uma propriedade essencial de R que todo nmero real pode ser bem aproximado por nmeros racionais. Efetivamente, dado x R, existe k Z (k = [x]) tal que 0 x k < 1. Podemos escrever a representao decimal de x k = 0, a1a2an, ai {0, 1, , 9}, o que significa que se
rn = an + 10.an1 + 100.an2 ++ 10n1 . a1,

ento

rn 10
n

xk<

rn + 1 10
n

, e portanto k +

rn 10 n rn
n

uma boa aproximao racional de

1 , que um 10 10 n nmero bem pequeno se n for grande. A representao decimal de um nmero real fornece pois uma seqncia de aproximaes por racionais cujos denominadores so potncias de 10.
x, no sentido que o erro x (k +

menor que

EUREKA! N 3, 1998

44

Sociedade Brasileira de Matemtica

Dado qualquer x R e q natural no nulo existe p Z tal que , e portanto x

p p +1 x< q q

p p +1 1 1 < e x . Em particular h aproximaes q q q q 1

. A q representao decimal de x equivale a dar essas aproximaes para os denominadores q que so potncias de 10, e tem mritos como sua praticidade para efetuar clculos que a fazem a mais popular das representaes dos nmeros reais. Por outro lado, envolve a escolha arbitrria da base 10, e oculta freqentemente aproximaes racionais de x muito mais eficientes do que as que exibe. Por exemplo, 22 1 314 355 1 3141592 < < < < e 7 700 100 113 3000000 1000000 22 355 mostram que e so melhores aproximaes de que aproximaes 7 113 decimais com denominadores muito maiores, e de fato so aproximaes muito mais espectaculares do que se podia esperar. O objetivo deste artigo apresentar uma outra maneira de representar nmeros reais, que sempre fornece aproximaes racionais surpreendentemente boas, e de fato fornece todas essas aproximaes excepcionalmente boas, alm de ser natural e conceitualmente simples: a representao por fraes contnuas. Dado x R definimos [x] como o nico inteiro tal que [x] x < [x] + 1). Definimos recursivamente 1 , para todo n N . 0 = x, a n = [ n ], e, se n Z , n +1 = n an Se, para algum n, n = an temos

de x por racionais com denominador q com erro menor que

x = 0 = a0 +

1 a1 + 1 a 2 + ... + 1 an

=: [ a ; a1 , a 2 ..., a n ].
0

Se no denotamos
EUREKA! N 3, 1998

45

Sociedade Brasileira de Matemtica

x = a0 +

1 1 a1 + a 2 + ...

=: [ a 0 ; a1 , a 2 ...].

O sentido dessa ltima notao ficar claro mais tarde. A representao acima se chama a representao por fraes contnuas de x.

Curiosidade: O denominador da n-sima aproximao em base B de um nmero real Bn. J o denominador qn da n-sima aproximao por frao contnua de x depende de x. Apesar disso, para quase todo real x,
n

q n converge a e
x pn qn

/ 12 ln 2

= 3,27582291872... (meu nmero real preferido!) e


2

converge a e

/ 6 ln 2

= 0,093187822954...

Observao: Os n (como funes de x) so funes distintas do tipo ax + b com a, b, c, d inteiros. Se a frao contnua de x peridica, ou seja, cx + d se n + k = n, n N, k N*, ento x ser raiz de uma equao do segundo grau com coeficientes inteiros, ou seja, ser um irracional da forma
r + s , r , s Q. A recproca verdadeira (de fato j foi enunciada no artigo de Jos Paulo Carneiro na RPM, ver referncias), mas sua prova mais difcil, e ser apresentada no Apndice. Se x Q, sua representao ser finita, e seus coeficientes an vm do algoritmo de Euclides:
x= p q , q>0 p = a 0 q + r0 q = a1 r0 + r1 r0 = a 2 r1 + r2 rn 2 = a n rn 1 0 r0 < q 0 r1 < r0 0 r2 < r1

EUREKA! N 3, 1998

46

Sociedade Brasileira de Matemtica

Isso j uma vantagem da representao por fraes contnuas (alm de no depender de escolhas artificiais de base), pois o reconhecimento de racionais mais simples que na representao decimal.

Seo 1: Reduzidas e boas aproximaes.


Seja x = [a0; a1, a2, ]. Sejam pn Z, qn N* primos entre si tais que pn = [a0; a1, a2, , an], n 0. O seguinte resultado ser fundamental no qn que seguir.

Proposio: (pn) e (qn) satisfazem a recorrncia pn+2 = an+2 pn+1+ pn e qn+2 = an+2 qn+1 +qn, para todo n 0. Temos ainda p0 = a0, p1 = a0a1 +1, q0 = 1,q1 = a1. Alm disso, pn+1 qn pnqn+1 = (1)n, n 0. Prova: Por induo em n, provaremos que se tk > 0, para k > 1 ento
[t0; t1, t2, , tk] =

xk onde as seqncias (xm) e (ym) so definidas por yk

x0 = t0, y0 = 1, x1 = t0t1 + 1, y1 = t0, xn+2 = tn+2 xn+1 + xn, yn+2 = tn+2 yn+1 + yn, n. Suponha que a afirmao seja vlida para k = n. Para k = n+1 temos

[t0; t1, t2, , tn, tn+1] = [t0; t1, t2, , tn +


1

1
t n +1

]=

) x n 1 + x n 2 t n +1 t (t x + x ) + x n 1 t n +1 x n + x n 1 . = n +1 n n 1 n 2 = 1 t n +1 (t n y n 1 + y n 2 ) + y n 1 t n +1 y n + y n 1 (t n + ) y n 1 + y n 2 t n +1 Por outro lado as igualdades p1q0 p0q1 = (a0a1 +1) a0a1 = 1 pn+2 qn+1 pn+1 qn+2 = (an+2 pn+1 + pn) qn+1 (an+2 qn+1 + qn) pn+1 = = ( pn+1 qn pnqn+1)
EUREKA! N 3, 1998

(t n +

47

Sociedade Brasileira de Matemtica

mostram que pn+1 qn pnqn+1 = (1)n, n N, o que implica em particular que os pn, qn dados pelas recorrncias acima so primos entre si.
n p n 1 + p n 2 n q n 1 + q n 2
p n2 q

Corolrio: x =

n =

n2

q n 1 p n 1

, n N.

Prova: A primeira igualdade conseqncia direta da prova, e a segunda conseqncia direta da primeira pois x = [a0; a1, a2, , an1, n]. Note que as reduzidas de ordem par so menores e as de ordem mpar maiores que x = [a0; a1,].
pn qn 1 1 < 2 , n N. q n q n +1 q n

Teorema 1: x
Alm disso, x

pn qn

<

p 1 ou x n +1 2 q n +1 2q n

<

1 , n N. 2 2q n +1
p n +1 pn e cujo qn q n +1

Prova: x sempre pertence ao segmento de extremos


comprimento :

p n +1 p n q n +1 q n
Alm disso, se

( 1) n q n q n +1

1
q n q n +1

pn 1 1 < 2. qn q n q n +1 q n

x
1

pn p p p 1 1 1 2 e x n+1 2 ento = x n + x n+1 qn qn+1 qn qn+1 2qn 2qn+1 qnqn+1


+ 1
2 2q n +1

2 2q n

q n +1 = q n , absurdo

Observao: De fato
EUREKA! N 3, 1998

pn qn

<

1 q n q n +1

<

1
2 a n +1 q n

. Quanto

maior for

48

Sociedade Brasileira de Matemtica

an+1 melhor ser a aproximao

pn de x. O prximo resultado nos d qn


pn qn

explicitamente o erro da aproximao de x por

Proposio:
x

q pn ( 1) n , onde n +1 = n 1 = [0; a n , a n 1 , a n 2 ,..., a1 ]. = 2 qn q n ( n +1 + n +1 )q n p n 1 q n 1 x . Portanto, qn x pn

Demonstrao: Temos n +1 =

n +1 + n +1 =

p pn 1 qn 1x qn 1 pn 1qn pn qn 1 (1)n + = = x n = qn x pn qn qn (qn x pn ) qn (qn x pn ) qn

q n (q n x p n )
2 qn

(1) n 2 ( n +1 + n +1 )q n

Como aplicao podemos provar o seguinte.

Teorema (Hurwitz, Markov): Para todo irracional, n 1 p pn1 pn pn+1 p 1 , , para pelo menos um racional < q qn1 qn qn+1 q 5q 2
particular p 1 tem infinitas solues racionais p/q. < q 5q 2

temos . Em

Demonstrao: Suponha que o teorema seja falso. Ento existe irracional, n 1 com n + n 5, n +1 + n +1 5 e n + 2 + n + 2 5. Devemos portanto ter an = an+1 = an+2 = 1 (todos so claramente no mximo 2, e se algum ak igual a 2 com k {n, n + 1, n + 2}, teramos 1 k + k 2 + > 5 , absurdo.) 3 Seja x = 1/n+2 e y = n+1. As desigualdades acima se traduzem em 1 1 1 1 + 5, 1 + x + y 5 e + 5 . Temos x 1+ y 1+ x y

EUREKA! N 3, 1998

49

Sociedade Brasileira de Matemtica

1+ x + y 5 1+ x 5 y
portanto

1 1 1 1 5 + + = 1+ x y 5 y y y( 5 y),
5 1 . 2
Por outro lado

e temos e ter

y( 5 y) 1 y

x 5 1 y

1 1 1 1 5 + + = x 1+ 4 5 1 y 1 + 4 (1 + y )( 5 1 y ) 5 1 , 2
e portanto devemos

portanto (1 + 4)( 5 1 y ) 1 y

y=

q 5 1 , o que absurdo pois y = n +1 = n 1 Q. qn 2


1 5q
2

Obs: em particular provamos que p <


q

tem infinitas solues

p , para todo irracional. q propriedade, De fato, se


racionais > 0, =

5 o maior nmero com essa

p 1+ 5 1 e < , temos 2 q ( 5 + )q 2
1 5 2 p q

1+ 5 1+ 5 1 p q 1 5 p < p < q q 2 2 2 ( 5 + )q

5 +

ou seja , p 2 pq q 2 < 1 + 5 p 5
2 q

( 5 + ). Se q grande, 1/q

pequeno,

1+ 2

p q

muito

prximo

de

0,

donde

1+ 5 p 5 2 q

( 5 + ) muito prximo de

5 5 +

< 1, absurdo, pois

p 2 pq q 2 1 (de fato p2 pq q2 um inteiro no nulo, pois se


EUREKA! N 3, 1998

50

Sociedade Brasileira de Matemtica

1 + 5 1 5 p p p p pq q = 0 teramos , , q q 1= 0 q 2 2 p absurdo, pois Q .) q


2 2

Outra maneira de ver que, para todo > 0, apenas um nmero finito de solues

1+ 2

p 1 < q ( 5 + )q 2

tem

p Q observar que as melhores q p 1+ 5 aproximaes racionais de so as reduzidas n de sua frao 2 q


n

contnua [1, 1, 1, 1, ] (ver seo 2 e exemplos), para as quais temos


p 1+ 5 1 n = , com n + 1 + n + 1 se aproximando cada 2 q 2 ( n + 1 + n +1 ) q n n

vez mais de

[1;1,1,1...] + [0;1,1,1,...] =
Exemplos:

1+ 5 5 1 + = 5. 2 2

= [3; 7, 15, 1, 292, 1, 1, 1, 2, 1, 3, 1, 14, 2, 1,], portanto p0 p 22 p 2 333 p 3 355 = 3, 1 = , = , = ,... q0 q1 7 q 2 106 q 3 113 e = [2; 1, 2, 1, 1, 4, 1, 1, 6, 1, 1, 8,, 1, 1, 2n, ], (isso no fcil de provar.) 2 = [1;2,2,2,...] pois
2 =1+ 1 2 +1 =1+ 2+ 1 1 2 +1 =1+ 2+ 2+ 1 1 1 2 +1 = ...

EUREKA! N 3, 1998

51

Sociedade Brasileira de Matemtica

1+ 2

= [1;1,1,1,...] pois 1 +

5 2

=1+

1 1+ 2 5

=1+ 1+

1 1 1+ 2 5

= ...

Isso prova em particular que contnua infinita.

2 e 1 + 5 so irracionais, pois sua frao


2

Seo 2: Boas aproximaes so reduzidas.


O prximo teorema (e seu Corolrio 2) caracteriza as reduzidas em termo do erro reduzido da aproximao de x por p/q, o qual , por definio, a razo entre x p / q e o erro mximo da aproximao por falta com denominador q, que 1/q. Assim, o erro reduzido da aproximao de x por p/q qx p .
p pn . q qn

Teorema 2: q n x p n < qx p , p, q Z,0 < q q n ,

Alm disso, q n x p n qx p , p, q Z,0 < q < q n +1 .

Prova:

p pn 1 1 se q < qn+1, e assim > q qn qq n q n q n +1

p est fora do q

p n p n +1 intervalo q ,q . Portanto, n +1 n
p p p p p min n , n +1 q q q n q q n +1 1 1 qx p qn x pn . q n +1 qq n + 1

p n +1 , donde an+1 2, e qn+1 > 2qn, q n +1 pois numa frao contnua finita, como no algaritmo de Euclides, o ltimo coeficiente an sempre maior que 1. Nesse caso, se q qn , teremos
Alm disso, se vale a igualdade, ento x =
EUREKA! N 3, 1998

52

Sociedade Brasileira de Matemtica

1 1 1 1 p p p q q p x n n+1 n = n+1 > qx p > qn x pn . qn qn+1 qn qqn qnqn+1 qqnqn+1 qqn+1 qn+1 q

Corolrio 1: x

pn p < x , q < q n . qn q

Corolrio 2: Se qx p < q ' x p ' , q ' q , p q ' ento p/q uma reduzida
q q'

da frao contnua de x.

Prova: Tome n tal que q n q < q n +1 .


Teremos q n x p n qx p , e portanto p/q = p n /q n

p Teorema 3: Se x p < 1 ento uma reduzida da frao contnua de x. 2


q 2q

Prova: Seja n tal que qn < q qn+1. Suponha que


duas possibilidades: q p 1 1 a) . q n +1 x 2 q qq n +1 2q 2 b)
q< =

p p n +1 . Ento, temos q q n +1

1 1 qn +1 p p p p p qn +1 > 2qn x n nm n = 2 q qn q qnm qn qqn qn qn +1

1 1 qn +1 q > > 2. qqn qn +1 2qqn 2q

Apndice: Fraes contnuas peridicas


Nesta seo provaremos que os nmeros reais com frao contnua peridica so exatamente as razes de equaes do segundo grau com coeficientes inteiros. Lembramos que na representao de x por frao contnua, an, n so definidos por recurso por
EUREKA! N 3, 1998

53

Sociedade Brasileira de Matemtica

0 = x, a n = [ n ], n +1 =

1 . n an

e temos p q n2 x n = n2 , n N . q n 1 x p n 1 Isso d uma prova explcita do fato de que se a frao contnua de x peridica, ento x raiz de uma equao do segundo grau com coeficientes inteiros. De fato, se n + k = n , n N, k N* ento p n 2 q n 2 x p n + k 2 q n + k 2 x = qn1 x pn1 qn+k 1 x pn+k 1

(qn1qn+k 2 qn2 qn+k 1 ) x 2 + ( pn+k 1qn2 + pn2 qn+k 1 pn+k 2 qn1 pn1qn+k 2 ) x + pn1 pn+k 2 pn2 pn+k 1 = 0. Note que o coeficiente de x2 no-nulo, pois
q n 1 uma frao irredutvel q n2

(de fato p n 1 q n 2 p n 2 q n 1 = (1) n ) de denominador qn2 e

q n + k 1 uma q n+k 2

frao irredutvel de denominador qn+k2 > qn2 , donde q qn 1 n + k 1 q n 1 q n + k 2 q n 2 q n + k 1 0. qn 2 q n + k 2 Vamos provar agora um resultado devido a Lagrange segundo o qual se x uma irracionalidade quadrtica, isto , se x um irracional do tipo r + s, r , s Q, s > 0 ento a frao contnua de x peridica, i. e, existem n N, k N* com n + k = n . Neste caso, existem a, b, c inteiros tais que

ax 2 + bx + c = 0 , com b 2 4ac > 0 e b 2 4ac irracional. Como vimos na seo 1, p + p n2 x = n 1 n , e portanto q n 1 n + q n 2


p n 1 n + p n 2 ax 2 + bx + c = 0 a q +q n2 n 1 n
2 An n + B n n + C n = 0,

p n 1 n + p n 2 + b q + q n2 n 1 n

+c= 0

onde
EUREKA! N 3, 1998

54

Sociedade Brasileira de Matemtica

2 2 An = ap n 1 + bp n 1 q n 1 + cq n 1

Bn = 2ap n 1 p n 2 + b( p n 1 q n 2 + p n 2 q n 1 ) + 2cq n 1 q n 2
2 2 C n = ap n 2 + bp n 2 q n 2 + cq n 2.

Note que Cn = An1. Vamos provar que existe M > 0 tal que 0 < A n M para todo n N, e portanto 0 < C n M , n N :

p n 1 2 2 2 An = ap n 1 + bp n 1 q n 1 + cq n 1 = aq n 1 x q n 1

p n 1 x q , n 1

onde x e x so as razes de a, X2 + bX + c = 0, mas

a x x + 1 =: M .
2 Notemos agora que Bn 4 An C n = b 2 4ac, n N. De fato, 2 Bn 4 An C n = ( p n 1 q n 2 p n 2 q n 1 ) 2 (b 2 4ac) = b 2 4ac. Portanto,
2 Bn 4 AnCn + b2 4ac = 4M 2 + b2 4ac Bn M ' = 4M 2 + b2 4ac,n N. Provamos assim que An, Bn e Cn esto uniformemente limitados, donde h apenas um nmero finito de possveis equaes An X2 + BnX + Cn = 0, e portanto de possveis valores de n. Assim, necessariamente n+k = n para alguma escolha de n N, k N*.

pn1 pn1 p p 1 2 < 2 1 An = aqn x n1 a x x + x n1 1 x qn1 qn1 qn1 qn1 qn1

Referncias: N. Beskin - Fraes contnuas - Iniciao Matemtica - Editora Mir. Jos Paulo Q. Carneiro - Um processo finito para a raiz quadrada Revista do Professor de Matemtica 34, 1997, pp. 36-44. C.D. Olds - Continued Fractions - New Mathematical Library - Random House. A. M. Rockett, P. Szsz - Continued Fractions - World Scientific.

EUREKA! N 3, 1998

55

Sociedade Brasileira de Matemtica

SOLUES DE PROBLEMAS PROPOSTOS EUREKA! Nos. 1 e 2


Publicamos aqui algumas das respostas enviadas por nossos leitores.

2)

Em uma pista circular h postos de gasolina, e o total de gasolina que h nos postos exatamente o suficiente para um carro dar uma volta. Prove que existe um posto de onde um carro com o tanque inicialmente vazio pode partir e conseguir dar uma volta completa na pista (parando para reabastecer nos postos).

Soluo
Sejam P1, P2,,Pn os postos de gasolina, li a quantidade de gasolina no posto Pi e ci a quantidade de gasolina necessria para ir de Pi a Pi+1, para i = 1, 2,, n (conveno: para 1 k n, Pn+k : = Pk ). Por hiptese,

l = c .
i i i =1 i =1 k0

Suponha que exista k com 1 k n e

l < c
i i =1 i =1

(se no

existe tal k podemos dar a volta comeando em P1). Tome k0 com 1 k0 n tal que

(l
i =1

c i ) seja o menor possvel. Afirmamos que podemos dar a

volta comeando em Pk0 +1 . De fato, se no for assim, existe r com 1 r n e


k0 + r i = k 0 +1 k0 + r

(l i ci ) < 0, mas ento teramos

k0 + r

i =1

(l i ci ) <

(l
i =1

k0

ci ), o que um

absurdo (se k0+r > n temos

i =1

(l i ci ) =

ko + r n

i =1

(l i ci ), pois

(l
i =1

ci ) = 0).

3)

Prove que existe n N tal que os 1000 primeiros dgitos de n1998 so iguais a 1.

Soluo
Seja n N tal que

EUREKA! N 3, 1998

56

Sociedade Brasileira de Matemtica

n1998 =

111 .. 11 0 1 2 ... p ; onde i tal que 0 i 9;


1000 algarismos

i = 1, 2, 3, , p. Seja tambm k = 11111, da: k .10s n1998 k 9999 ... 9, logo k . 10s n1998 < (k + 1) . 10s,
s algarismos

Precisamos garantir que h algum n N que satisfaa a desigualdade acima; seja ento s = 1998 . p : k . 101998 . p n1998 < (k + 1) . 101998 . p

n < 1998 k + 1. 10 p observe que se tomarmos n = 10 p .1998 k + 1; onde z = maior inteiro menor ou igual a z, e p suficientemente grande satisfaremos a condio do enunciado.
1998

k .10 p n < 1998 k + 1.10 p

1998

Concluso: n N tal que n1998 escrito como no enunciado. 5)


Sejam a > 0 e P1P2P3P4P5 uma poligonal aberta contida em um dos semiplanos determinados pela reta P1 P5 . Prove que existem pontos P6 e P7 no plano, com P5 P6 = a, de modo que possvel ladrilhar o plano com infinitos ladrilhos congruentes ao heptgono P1P2P3P4P5P6P7.

Soluo
Trae a paralela a P3P2 passando por P1. O ponto P7 pertencer a essa reta e teremos P1 P7 = P3 P2 . O ponto P6 pertencer paralela a P3P4 passando por P5 e satisfar P5 P6 = a, ou seja, P5 P6 =

a P3 P4 . P3 P4

Rodando o heptgono H = P1P2P3P4P5P6P7 de 180 em torno do ponto mdio de P1P2 obtemos o heptgono H' = P1'P2'P3'P4'P5'P6'P7' com P1' = P2 , P2' = P1 , P3' = P7 , P7' = P3. Transladando infinitas vezes os heptgonos H e H' por k . P3 P6 , k Z, cobrimos uma faixa dentada, que, transladada infinitas vezes por m . P4 ' P5 , m Z, nos permite cobrir o plano.
EUREKA! N 3, 1998

57

Sociedade Brasileira de Matemtica

Desta

H + K . P3 P6 + m . P4 ' P5 e H' + k . P3 P6 + m . P4 ' P5 , k Z, de interiores disjuntos e todos congruentes a H.


6)
Mostre que toda seqncia com n2+1 elementos possui uma subseqncia crescente com n+1 elementos ou uma subseqncia decrescente com n+1 elementos.

forma,

cobrimos

plano

com

os

heptgonos

Soluo
Dada uma seqncia a1, a2,, a n 2 +1 de nmeros reais, definimos para 1 i

n2+1 o nmero f (i) como sendo o nmero mximo de termos de uma subseqncia decrescente de a1, a2,, a n 2 +1 comeando em ai. Suponha que no exista nenhuma subseqncia decrescente de n +1 elementos. Ento f (i) n para todo i, e portanto f (i) s pode assumir os n valores 1, 2, , n. Assim, existem 1 i1 < i2 << in+1 com f (i1) = f (i2) = = f (in+1), mas nesse caso devemos ter a i1 a i2 ... a n 2 +1 , com n + 1 termos.

Obs. 1: Mostra-se com um argumento anlogo que toda seqncia com mn+1 elementos possui uma subseqncia crescente de m+1 elementos ou uma subseqncia decrecente de n+1 elementos (de fato que existe uma seqncia crescente de m+1 elementos ou uma seqncia estritamente decrescente de n+1 elementos.) Obs. 2: O resultado (e sua generalizao na obs. 1) o melhor possvel. De fato, dados m, n N, a seqncia de mn termos n, n1, n2, , 1, 2n, 2n1, 2n2, , n+1, 3n, , 2n+1, , mn, mn1, , (m1) n+1 no contm nenhuma seqncia crescente de mais de m elementos nem nenhuma seqncia decrescente de mais de n elementos.

12)

a) Prove que se n N e 2n + 1 um nmero primo ento n uma potncia de 2. b) Prove que se a, n N, n 2 e an 1 primo, ento a = 2 e n primo.

EUREKA! N 3, 1998

58

Sociedade Brasileira de Matemtica

Soluo a) Sabemos que n N pode ser escrito da seguinte forma: n = 2 k p onde k N e p mpar.
Seja n = 2n + 1, logo n = 2 2 p + 1 = 2 2 + 1, fazendo = 2 2 x = p + 1. Se p im mpar maior do que 1, teremos: x = p + 1 = ( + 1)( p 1 p 2 + p 3 ... + 1) e, como x primo, ele no poder ser fatorvel em um produto de fatores diferentes de 1. Basta ento observar que o segundo fator da multiplicao acima no igual a 1 com p mpar maior do que 1, mas isso segue de p > p + 1 + 1. Logo devemos ter necessariamente x = 2 2 + 1, ou seja n = 2k.
k
k

( )
k

b) Seja y = an 1 = (a 1)(an1 + an2 ++ a +1) primo:


i) ii) Vamos verificar inicialmente que a deve ser igual a 2. De fato a 1 = 1, j que o segundo fator no pode ser igual a 1 (a 1). Suponha que n no seja primo, n = k1 . k2 com k1 2 e k2 2, Logo
( k 2 1)

y = 2 k1 k 2 1 = (2 k1 ) k 2 1 = ( 2 k1 1) (2 k1

+ 2 k1 ( k 2 2) + ... + 2 k1 + 1), obser-

ve que 2 k1 1 3 e 2 k1 ( k 2 1) + ... + 2 k1 + 1 > 3 e conseqentemente no teremos y primo, logo n no pode ser escrito como acima; donde n primo.

14)

Determine o nmero de solues de positivos.

1 1998

com x e y inteiros

Soluo
Temos 1998x + 1998y = xy . Somando 19982 dos dois lados temos xy 1998x 1998y + 19982 = 19982, logo x(y 1998) 1998(y 1998) = 19982, donde (x 1998) (y 1998) = 19982.

EUREKA! N 3, 1998

59

Sociedade Brasileira de Matemtica

Desta forma o nmero de solues o mesmo que a quantidade de sistemas da forma abaixo que possamos obter:

x 1998 = a y 1998 = b 2 ab = 1998


com a observao de que os pares (x, y) soluo devam ser inteiros e positivos, devemos ter

a + 1998 > 0 a > 1998 b + 1998 > 0 b > 1998 logo, s servem a e b positivos, j que se 1998< a < 0 e 1998 < b < 0 implica ab < 19982. O nmero de solues , portanto, o nmero de divisores positivos de 19982 = 22. 36. 372, que dado por (2 + 1) (6 + 1) (2 + 1) = 63.

Solues dos problemas 2, 5 e 6 enviadas por Zoroastro Azambuja Neto. Solues dos problemas 3 e 12 enviadas por Carlos Alberto da Silva Victor. Soluo do problema 14 enviada por Vicente Wilson Moura Gaete e Andr Luiz Arruda Marques. Continuamos esperando as solues dos problemas 10, 11, 12, 13 e 15.

EUREKA! N 3, 1998

60

Sociedade Brasileira de Matemtica

PROBLEMAS PROPOSTOS
Convidamos o leitor a enviar solues dos problemas propostos e sugestes de novos problemas para os prximos nmeros.

16)

Seja l a reta {( x, y ) R 2 y = 0}, C1 o crculo centrado em

i) ii)

1 1 1 1 (0, ) de raio e C 2 o crculo centrado em (1, ) de raio . 2 2 2 2 Seja F o conjunto de crculos em R2 com as seguintes propriedades: {C1, C2} F Se C e C pertencem a F, so tangentes entre si e tangentes a l
ento todo crculo C tangente aos dois crculos C e C e reta l pertence a F.
~

iii)

Se F um conjunto de crculos satisfazendo as propriedades i) e ii) ento F F . Determine o conjunto dos pontos de tangncia dos crculos C F com a reta l.
~

17)

Dado n N, uma partio de n uma lista ordenada = (a , a ,...,a ) , r, a , a ,..., a N* com a a ... a e a + a + ...+ a = n.
1 2 r 1 2 r 1 2 r 1 2 r

Seja Pn o conjunto das parties de n. Para Pn , definimos A() como o nmero de termos iguais a 1 em ( ou seja , A( ) =

# {i {1,2,..., r} ai = 1}), e B() como o nmero de termos distintos


na partio (ou seja, B () = # {a1, a2, , ar}). A( ) = B ( ) para todo n N. Prove que
Pn

Pn

18)

Seja a maior raiz real da equao x3 3x2 + 1 = 0. Prove que [2004] divisvel por 17. Obs: [y] o nico inteiro tal que [y] y < [y] + 1. a) Determine o nmero mximo de regies em que n retas podem dividir o plano. b) Determine o nmero mximo de regies em que n planos podem dividir o espao.

19)

EUREKA! N 3, 1998

61

Sociedade Brasileira de Matemtica

COORDENADORES REGIONAIS
Alberto Hassen Raad Antnio C. Rodrigues Monteiro Amarsio da Silva Arajo Angela Camargo Antnio C. do Patrocnio Ariosto de Oliveira Lima Benedito T. Vasconcelos Freire Carlos A. Bandeira Braga Claudio Arconcher Egnilson Miranda de Moura lio Mega Florncio F. Guimares F. Francisco Dutenhefner Gisele de A. Prateado G. Ivanilde H. Fernandes Saad Joo B. de Melo Neto Joo F. Melo Libonati Jorge Ferreira Jos Carlos Pinto Leivas Jos Luis Rosas Pinho Jos Paulo Carneiro Jos Vieira Alves Leonardo Matteo D'orio Licio Hernandes Bezerra Luzinalva M. de Amorim Marco Polo Marcondes Cavalcante Frana Mario Jorge Dias Carneiro Pablo Rodrigo Ganassim Paulo H. Cruz Neiva de L. Jr. Reinaldo Gen Ichiro Arakaki Ricardo Amorim Sergio Claudio Ramos Tadeu Ferreira Gomes Valdenberg Arajo da Silva Wagner Pereira Lopes
(UFJF) Juiz de Fora-MG (UFPE) Recife-PE (UFV) Viosa-MG (Centro de Educao de Adultos CEA) Blumenau-SC (IMECC/UNICAMP) Campinas-SP (UFPI) Parnaba-PI (UFRN) Natal-RN (UFPB) Joo Pessoa-PB (Col. Leonardo da Vinci) Jundia-SP (Col. Agrcola do Bom Jesus) Bom Jesus-PI (Col. ETAPA) So Paulo-SP (UFES) Vitria-ES (UFMG ) BH-MG (UFGO) Goinia-GO (U. Catlica Dom Bosco) Campo Grande-MS (UFPI) Teresina-PI (Grupo Educ. IDEAL) Belm-PA (UEM) Maring-PR (URG) Rio Grande-RS (UFSC) Florianpolis-SC (USU) Rio de Janeiro-RJ (UFPB) Campina Grande-PB (Parque de Material Belm-PA Aeronutico de Belm) (UFSC) Florianpolis-SC (UFBA) L. de Freitas-BA (Colgio Singular) Santo Andr-SP (UF Cear) Fortaleza-CE (UFMG) BH-MG (L. Albert Einstein) Piracicaba-SP (Esc. Tec.Everardo Passos) S. J.Campos-SP (INPE) S.J.Campos-SP (Centro Educ. Logos) Nova Iguau-RJ (IM-UFRGS) Porto Alegre-RS (U. do Estado da Bahia) Juazeiro-BA (U. Federal de Sergipe) So Cristovo-SE (Esc. Tec. Fed. de Gois) Jata-GO

EUREKA! N 3, 1998

62



     !#"$ %& '&   ( %)!* !#+ , ./103254 637*8396*9:054 ;3<:=5639>58?./1@ 7#65@ /A8*B859:6   !#"$ %& '&   ( %)!* !#+ , ./103254 637*8396*9:054 ;3<:=5639>58?E65F3;3G5>38*B859:6   !#"$ %& '&   ( %)!* !#+ , ./103254 637*8396*7*634 H305/A6599:054 ;3<:=5639>38
IJ63/1K:63@ /18#B 839:6   !#"$ %& '&   ( %)!* !#+ , L639:;54 M 83>5039 NOP&QARTS
U "'V!* &
W X Z>5;385/A>30T[#85F3G365/ \X "\%&  !*"$ &% ]'& &   & %
!? !*+ ,& .85;34 0*^63_`83/V.@ G5M 0*^83/bac834 H50  def)%)"' !# &)""3 "'V!* &
"&"3 &  ,   &
%
  & ,!#\ & W& ] d&kj l XW&% $ !*"(, ,%&W& %&k
kX W    C5D 5 -5D -5Y D5 D5Y gh 5 gY 5 gi 5 gm 5 hn 5 hC 5

Sociedade Brasileira de Matemtica

o prqsTt()uTp
Iniciamos este segundo ano da revista vVwyxvVz&{*| transmitindo nossa satisfao pela acolhida no primeiro ano de vida da revista, de toda a comunidade estudantil e dos diretores e professores dos colgios envolvidos. Agradecemos a todos os que tm apoiado esta iniciativa e esperamos continuar apoiando, atravs desta publicao, o trabalho dos professores. Nesta nova edio da revista vVwyxvVz&{*| aproveitamos para registrar a XX Olimpada Brasileira de Matemtica, da qual publicamos as provas da primeira, segunda e terceira fases com solues (que, esperamos, sero teis para a preparao para a XXI OBM), bem como as listas de premiados nos trs nveis. Nesta edio, tambm publicamos artigos de dificuldade intermediria e material enviado por numerosos professores e alunos. Esperamos seguir recebendo colaboraes dos nossos leitores: solues dos problemas propostos, pequenos artigos e curiosidades matemticas. Devido ao interesse manifestado por diversas pessoas, criamos recentemente as assinaturas individuais da revista vVwyxvVz&{*| . Para maiores informaes, veja pgina 59 desta edio. Por outro lado, estamos planejando criar, nos prximos nmeros, um pequeno espao publicitrio ligado ao ensino da matemtica, para o qual aguardamos propostas de leitores, editoras e instituies de ensino. Desta forma, estaremos gerando recursos que ajudaro a manter a publicao da revista. Aproveitamos, por fim, para registrar que foi realizada em janeiro de 1999 a segunda Semana Olmpica, que reuniu premiados na XX Olimpada Brasileira de Matemtica nos 3 nveis e professores de vrios estados. A atividade foi realizada em Maracana, Cear, no centro de treinamento do Colgio 7 de Setembro, ao qual gostaramos de agradecer pelo apoio.

Comit Editorial.

EUREKA! N4, 1999

Sociedade Brasileira de Matemtica

}T} ~cs(s~scs3ck V T 35y(


01. Qual dos nmeros a seguir o maior? A) 345 B) 920 C) 2714 D) 2439 E) 8112

02. Um menino joga trs dados e soma os nmeros que aparecem nas faces voltadas para cima. O nmero dos diferentes resultados dessa adio : A)12 B) 18 C) 216 D) 16 E) 15 03. Renata digitou um nmero em sua calculadora, multiplicou-o por 3, somou 12, dividiu o resultado por 7 e obteve o nmero 15. O nmero digitado foi: A) 31 B) 7 C) 39 D) 279 E) 27 04. Numa competio de ciclismo, Carlinhos d uma volta completa na pista em 30 segundos, enquanto que Paulinho leva 32 segundos para completar uma volta. Quando Carlinhos completar a volta nmero 80, Paulinho estar completando a volta nmero: A) 79 B) 78 C) 76 D) 77 E) 75 05. Elevei um nmero positivo ao quadrado, subtrai do resultado o mesmo nmero e o que restou dividi ainda pelo mesmo nmero. O resultado que achei foi igual: A) Ao prprio nmero B) Ao dobro do nmero C) Ao nmero mais 1 D) raiz quadrada do nmero E) Ao nmero menos 1 06. Quantos nmeros de 3 algarismos existem cuja soma dos algarismos 25 ? A) 2 B) 4 C) 6 D) 8 E) 10 07. Joo mais velho que Pedro, que mais novo que Carlos; Antnio mais velho do que Carlos, que mais novo do que Joo. Antnio no mais novo do que Joo e todos os quatro meninos tm idades diferentes. O mais jovem deles : A) Joo B) Antnio C) Pedro D) Carlos E) impossvel de ser identificado a partir dos dados apresentados

EUREKA! N4, 1999

Sociedade Brasileira de Matemtica

08. Escreva um nmero em cada crculo da fila abaixo, de modo que a soma de trs nmeros quaisquer vizinhos (consecutivos) seja 12.
3 5

No ltimo crculo direita deve estar escrito o nmero: A) 3 B) 2 C) 1 D) 4

E) 7

09. Dezesseis cubos de 1cm de lado so colocados juntos, formando o paraleleppedo representado abaixo.

A superfcie do mesmo foi pintada de verde e, em seguida, os cubos foram separados. O nmero de cubos com exatamente duas faces verdes : A) 2 B) 6 C) 4 D) 8 E) 10 10. Uma fazenda retangular que possui 10 km de largura por 20 km de comprimento foi desapropriada para reforma agrria. Se a fazenda deve ser dividida para 200 famlias de modo que todas as famlias recebam a mesma rea, ento cada famlia deve receber: A) 1.000.000 m2 B) 100.000 m2 C) 5.000 m2 D) 1.000 m2 2 E) 10.000 m 11. Um estacionamento para carros cobra 1 real pela primeira hora e 75 centavos a cada hora ou frao de hora seguinte. Andr estacionou seu carro s 11h 20min e saiu s 15h 40min. Quantos reais ele deve pagar pelo estacionamento? A) 2,50 B) 4,00 C) 5,00 D) 4,75 E) 3,75 12. Para fazer 12 bolinhos, preciso exatamente de 100g de acar, 50g de manteiga, meio litro de leite e 400g de farinha. A maior quantidade desses bolinhos que serei capaz de fazer com 500g de acar, 300g de manteiga, 4 litros de leite e 5 quilogramas de farinha : A) 48 B) 60 C) 72 D) 54 E) 42

EUREKA! N4, 1999

Sociedade Brasileira de Matemtica

13. Joozinho brinca de formar quadrados com palitos de fsforo como na figura a seguir.

A quantidade de palitos necessria para fazer 100 quadrados : A) 296 B) 293 C) 297 D) 301

E) 28

14. A soma de todos os nmeros mpares de dois algarismos menos a soma de todos os nmeros pares de dois algarismos : A) 50 B) 46 C) 45 D) 49 E) 48 15. O nmero que devemos somar ao numerador e subtrair do denominador 1478 da frao para transform-la na sua inversa : 5394 A) 3.916 B) 3.913 C) 3.915 D) 3.912 E) 3.917 16. O alfabeto usado no planeta X tem somente duas letras: X e x. O sobrenome (nome de famlia) de cada um de seus habitantes uma seqncia formada por 4 letras. Por exemplo, xXxx um possvel sobrenome utilizado nesse planeta. O maior nmero de sobrenomes diferentes que podem ser dados no planeta X : A) 12 B) 14 C) 15 D) 16 E) 18 17. Joo quer desfazer-se de sua coleo de 1.000 bolinhas. Para tanto escolhe dez garotos da rua onde mora. D ao primeiro garoto x bolinhas, ao segundo x + 1 bolinhas. Assim faz at chegar ao dcimo garoto. Sempre d uma bolinha a mais para o prximo garoto. No final, Joo ainda fica com um resto de bolinhas. Sendo x o nmero que deixa Joo com o menor resto possvel, x igual a: A) 94 B) 95 C) 96 D) 97 E) 98 18. No planeta Z todos os habitantes possuem 3 pernas e cada carro possui 5 rodas. Em uma pequena cidade desse planeta, existem ao todo 97 pernas e rodas. Ento podemos afirmar: A) possvel que existam 19 carros nessa cidade B) Existem no mximo 16 carros nessa cidade
EUREKA! N4, 1999

Sociedade Brasileira de Matemtica

C) Essa cidade tem 9 habitantes e 14 carros D) Essa cidade possui no mximo 17 carros E) Nessa cidade existem mais carros do que pessoas 19. So dados um tabuleiro e uma pea, como mostra a figura.

De quantas maneiras diferentes podemos colocar a pea no tabuleiro, de modo que cubra completamente 3 casas? A) 16 B) 24 C) 36 D) 48 E) 60 20. Pedro e Maria formam um estranho casal. Pedro mente s quartas, quintas e sextas-feiras, dizendo a verdade no resto da semana. Maria mente aos domingos, segundas e teras-feiras, dizendo a verdade no resto da semana. Certo dia, ambos dizem: ''Amanh dia de mentir''. O dia em que foi feita essa afirmao era: A) segunda-feira B) tera-feira C) sexta-feira D) sbado E) domingo

s k3

01. Quantos so os nmeros inteiros x que satisfazem inequao 3< x < 7? A) 13 B) 26 C) 38 D) 39 E) 40 02. Hoje sbado. Que dia da semana ser daqui a 99 dias? A) segunda-feira B) sbado C) domingo D) sexta-feira E) quinta feira 03. Anulada. 04. Um pai tem 33 anos e seu filho, 7 anos. Depois de quantos anos a idade do pai ser o triplo da idade do filho? A) 3 B) 7 C) 6 D) 9 E) 13

EUREKA! N4, 1999

Sociedade Brasileira de Matemtica

05. O quadriltero ABCD um quadrado de rea 4m2. Os pontos M e N esto no meio dos lados a que pertencem. Podemos afirmar que a rea do tringulo em destaque , em m2,

A) 2

B) 1,5

C) 2,5

D) 3

E) 3,5

06. Qual o dgito das unidades do nmero 31998? A) 1 B) 3 C) 5 D) 7

E) 9

07. Num cdigo secreto, as 10 primeiras letras do nosso alfabeto representam os algarismos de 0 a 9, sendo que a cada letra corresponde um nico algarismo e vice-versa. Sabe-se que d + d = f, d . d = f, c + c = d, c + d = a e a a = b. Podemos concluir que a + b + c + d igual a: A) 0 B) 2 C) 4 D) 6 E) 8 08. O nmero 1234a6 divisvel por 7. O algarismo a vale: A) 0 B) 2 C) 5 D) 6

E) 8

09. No trapzio abaixo, tm-se: AB paralelo a CD, AD = 10 cm e CD = 15 cm. O ngulo C mede 75 e o ngulo D, 30 . Quanto mede o lado AB, em centmetros?

A) 5

B) 7,5

C) 10

D) 12,5

E) 5 3

10. No quadrado mgico abaixo, a soma dos nmeros em cada linha, coluna e diagonal sempre a mesma. Por isso, no lugar do X devemos colocar o nmero:

EUREKA! N4, 1999

Sociedade Brasileira de Matemtica

A)30

B) 20

C) 35

D) 45

E) 40

11. Passarinhos brincam em volta de uma velha rvore. Se dois passarinhos pousam em cada galho, um passarinho fica voando. Se todos os passarinhos pousam, com trs em um mesmo galho, um galho fica vazio. Quantos so os passarinhos? A) 6 B) 9 C) 10 D) 12 E) 15 12. Pelo menos quantos metros de barbante so necessrios para amarrar 15 pacotes, conforme a figura, sabendo que cada pacote mede 10cm 20cm 40cm, sendo reservados 20cm para o lao?

A) 39

B) 36

C) 48

D) 56

E) 42

13. Para assistir ao filme Central do Brasil, cada um dos x alunos de uma turma deveria pagar y reais pelo frete do nibus. Como faltaram 3 alunos, cada um dos alunos presentes teve que pagar 2 reais a mais para cobrir o preo do frete. Qual foi esse preo? A) (x + 3)(y 2) B) (x 3) y + 2 C) x(y + 2) 3 D) xy 6 E) (x 3)(y + 2) 14. Seu Horcio resolveu incrementar a venda de CDs em sua loja e anunciou uma liquidao para um certo dia, com descontos de 30% sobre o preo das etiquetas. Acontece que, no dia anterior liquidao, seu Horrio aumentou o preo marcado nas etiquetas, de forma que o desconto verdadeiro fosse de apenas 9%. De quanto foi o aumento aplicado por seu Horcio? A) 30% B) 39% C) 21% D) 40% E) 31% 15. Um fabricante de brinquedos embala bolas de pingue-pongue em dois tipos de caixas. Num dos tipos ele coloca 10 bolas e no outro coloca 24
EUREKA! N4, 1999

Sociedade Brasileira de Matemtica

bolas. Num certo dia foram embaladas 198 bolas e usadas mais de 10 caixas. Quantas caixas foram feitas nesse dia? A)14 B) 16 C) 15 D) 17 E) 11 16. Coloque em cada quadradinho, no desenho a seguir, os algarismos 1, 2, 3, 4 ou 5, de forma que cada um deles aparea pelo menos uma vez e que o nmero formado seja o maior possvel e mltiplo de 9.

No nmero que voc construiu, o algarismo mais repetido apareceu: A) 6 vezes B) 5 vezes C) 4 vezes D) 3 vezes E) 2 vezes 17. Observe as igualdades a seguir:
32 + 42 = 52 52 + 122 = 132 7 2 + 242 = 252 92 + 402 = 412 ...

Considere a igualdade 17 + x = y 2 com base nos exemplos anteriores, procure determinar os nmeros naturais x e y. Podemos concluir que x + y igual a: A) 289 B) 121 C) 81 D) 144 E) 196
2 2

18. Voc vai pintar a bandeira abaixo utilizando 4 cores: azul, verde, amarelo e vermelho, uma em cada regio.

Se o vermelho e o amarelo no podem ficar juntos, de quantas maneiras pode ser pintada a bandeira? A) 12 B) 4 C) 18 D) 20 E) 16 19. Um crime cometido por uma pessoa e h quatro suspeitos: Andr, Eduardo, Rafael e Joo. Interrogados, eles fazem as seguintes declaraes:

EUREKA! N4, 1999

Sociedade Brasileira de Matemtica

Andr: Eduardo o culpado. Rafael: Eu no sou culpado.

Eduardo: Joo o culpado. Joo: Eduardo mente quando diz que eu sou culpado. Sabendo que apenas um dos quatro disse a verdade, quem o culpado? A) Andr. B) Eduardo. C) Rafael. D) Joo. E) No se pode saber. 20. Anulada.

s k3
01. Veja Problema 1 Nvel 2. 03. Veja Problema 5 do Nvel 2. 05. Veja Problema 15 do Nvel 2. 06.- 0,4444... = A) 0,2222 B) 0,3333 02. Veja Problema 2 do Nvel 2. 04. Veja Problema 6 do Nvel 2.

C) 0,4444

D) 0,5555

E) 0,6666

07.- Veja Problema 8 do Nvel 2. 08.- Todos os ngulos internos de um polgono convexo so menores que (no podendo ser iguais a) 160. O nmero de lados desse polgono , no mximo, igual a: A) 12 B) 14 C) 15 D) 17 E) 18 09.- A mdia aritmtica de seis nmeros 4. Quando acrescentamos um stimo nmero, a nova mdia 5. O nmero que foi acrescentado : A) 5 B) 6 C) 8 D) 10 E) 11 10. Veja Problema 19 do Nvel 2. 11.- Em uma calculadora, a tecla A transforma o nmero x que est no visor em

1 e a tecla B multiplica por 2 o nmero que est no visor. Se o nmero x

2 est no visor e digitamos a seqncia ABABABAB...AB (total de digitaes: 998), obteremos no visor um nmero que igual a: A) 1 B) 2-498 C) 2-500 D) 2499 E) 2500

EUREKA! N4, 1999

10

Sociedade Brasileira de Matemtica

12.- Um nmero inteiro n bom quando 4n + 1 um mltiplo de 5. Quantos nmeros bons h entre 500 e 1.000? A) 50 B) 51 C)100 D) 101 E) 102 13.- Em um conjunto de pontos do espao, a distncia entre dois pontos diferentes quaisquer igual a 1. O nmero mximo de pontos que pode haver nesse conjunto : A) 2 B) 3 C) 4 D) 6 E) 8 14.- Se x homens fazem x embrulhos em x segundos, em quantos segundos y homens faro y embrulhos? A) y B) x

x2 C) y

D)

y2 x

E)

y x

15.- Voc entra em um restaurante para comer pizza e espera pagar uma quantia proporcional quantidade de comida pedida. Se uma pizza com 20 cm de dimetro custa R$ 3,60, quanto voc espera pagar por uma outra do mesmo sabor com 30cm de dimetro? A) R$ 5,40 B) R$ 5,80 C) R$ 6,60 D) R$ 7,50 E) R$ 8,10 16.- A funo f associa a cada real x o menor elemento do conjunto 15 x x + 1, . O valor mximo de f(x) : 2 A) 4 B) 5 C) 11/2 D) 16/3 E) 19/4 17.- Vendi dois rdios por preos iguais. Em um deles tive lucro de 25% sobre o preo de compra e no outro tive prejuzo de 25%. Em relao ao capital investido: A) no tive lucro nem prejuzo B) lucrei 6,25% C) lucrei 16% D) tive prejuzo de 6,25% E) tive prejuzo de 16% 18.- A respeito da resposta de um problema, Maurcio, Paulo, Eduardo e Carlos fizeram as seguintes afirmaes: Maurcio: maior que 5. Paulo: menor que 10. Eduardo: um nmero primo. Carlos: maior que 12. Entre as afirmaes acima, quantas, no mximo, podem ser verdadeiras? A) 0 B) 1 C) 2 D) 3 E) 4

EUREKA! N4, 1999

11

Sociedade Brasileira de Matemtica

19.- Os valores reais de x que satisfazem a inequao A) 1 x 1 E) x 2 B) x = 1 C) x 1

x+

1 2 so: x D) x 1

20.- De quantos modos se pode colocar na tabela abaixo duas letras A, duas letras B e duas letras C, uma em cada casa, de modo que no haja duas letras iguais na mesma coluna?

A) 12

B) 24

C) 36

D) 48

E) 64

21.- Um viajante deveria caminhar durante uma hora num sentido entre o norte e o leste, fazendo 300 com o norte. Atrapalhou-se e caminhou uma hora num sentido entre o norte e o oeste, formando 300 com o norte. Para chegar ao seu destino, ele deve agora tomar um rumo que faa com o norte um ngulo de: A) 00 B) 300 C) 450 D) 600 E) 900 22.- Barcas vo do Rio a Niteri em 25 minutos e lanchas fazem a viagem em 15 minutos. A que horas a barca que partiu do Rio s 10h 01min alcanada pela lancha que saiu do Rio s 10h 07min? A) 10h 15min B) 10h 16min C) 10h 17min D) 10h 18min E)10h 20min 23.- Veja Problema 17 do Nvel 2.
3 2 24.- A soma das razes reais de x + 3 x + 3 x 1 = 0 :

A) 3

B) 1 3 2

C) 1

D)

2 1

E) 3

25.- Dado um cubo, considere o conjunto de 27 pontos formado pelos vrtices desse cubo, pelos pontos mdios de suas arestas, pelos centros de suas faces e pelo centro do cubo. Quantas so as retas que passam por trs desses pontos? A) 49 B) 54 C) 63 D) 81 E) 108

EUREKA! N4, 1999

12

Sociedade Brasileira de Matemtica

Respostas Nvel 1: 01.- E 06.- C 02.- D 07.- C 03.- A 08.- A 04.- E 09.- D 05.- E 10.- A

11.- B 12.- B 13.- D 14.- C 15.- A

16.- D 17.- B 18.- D 19.- C 20.- B

Respostas Nvel 2: 01.- D 06.- E 02.- C 07.- D 03.- Anulada 08.- D 04.- C 09.- A 05.- B 10.- B

11.- B 12.- B 13.- E 14.- A 15.- D

16.- B 17.- A 18.- A 19.- C 20.- Anulada

Respostas Nvel 3: 01.- D 06.- E 02.- C 07.- D 03.- B 08.- D 04.- E 09.- E 05.- D 10.- C

11.- A 12.- C 13.- C 14.- B 15.- E

16.- D 17.- D 18.- D 19.- B 20.- D

21.- E 22.- B 23.- A 24.- D 25.- A

#A ) 5
?33V5 k J V)3:*3 5:3#( 3:] J 5 3555 V:V *5V
:V#5 VJ:: 55: 5::
5:T       

!"$#%!'&()!"+*

EUREKA! N4, 1999

13

Sociedade Brasileira de Matemtica

u o pTtq?
tu o r o
0/ )tc o }
} qst ,.- o o 1 3 254637 3 "'V!* 98
Joo comprou um livro e reparou que ele tinha 200 pginas. Seu irmo mais novo arrancou ao acaso 25 folhas e somou os nmeros das 50 pginas. Explique porque o resultado desta soma no pode ser igual a 1998. Ateno: cada folha tem duas pginas. A primeira folha tem as pginas 1 e 2, a segunda folha tem as pginas 3 e 4, e assim por diante.

;:< =>?:

Como cada folha contm duas pginas tais que a soma dos seus respectivos nmeros mpar, ao adicionarmos todos esses 25 nmeros, obteremos necessariamente uma soma mpar que, portanto, no pode ser igual a 1998.

"'V!* 9@

Que fraes devem ser retiradas da soma a soma das restantes seja igual a 1?

1 2

1 4

1 6

1 8

1 10

1 12

para que

;:< =>?:
1 2 + 1 4

1 6

1 8

1 10

1 12

60 120

30 120

20 120

15 120

12 120

10 120

(*)

Uma vez que 60 + 30 + 20 + 10 = 120, claro que podemos remover


15 120 = 1 8

12 120

1 10

. (alm disso, v-se claramente no lado direito da 27 120

igualdade (*) que no existem outros termos cuja soma seja igual a Assim, devemos remover
1 8

1 10

"'V!* 9A

Encontre dois nmeros de trs algarismos cada um, usando cada um dos dgitos 1, 2, 3, 4, 5, 6 exatamente uma vez, de forma que a diferena entre eles (o maior menos o menor) seja a menor possvel.

EUREKA! N4, 1999

14

Sociedade Brasileira de Matemtica

;:< =>?:
Para que a diferena seja a menor possvel, os nmeros devem ser os mais prximos possveis. Assim, os algarismos das centenas devem ser consecutivos. A melhor escolha aquela em que as dezenas formadas pelos algarismos restantes tenham a maior diferena possvel, o que ocorre para as dezenas 65 e 12. Assim, os algarismos das centenas devem ser 3 e 4. O menor nmero comeado por 4 412 e o maior comeado por 3 365, cuja diferena 47.

"'V!* 9B

Existem casas em volta de uma praa. Joo e Pedro do uma volta na praa, caminhando no mesmo sentido e contando as casas. Como no comearam a contar da mesma casa, a 5. casa de Joo a 12. de Pedro e a 5. casa de Pedro a 30. de Joo. Quantas casas existem em volta da praa?

;:< =>?:
Sejam Jn e Pn respectivamente as n-simas casas de Joo e Pedro. De J5 a J30 exclusive, existem 30 5 1 = 24 casas. De P5 a P12 exclusive existem 12 5 1 = 6. Logo, no total existem 24 + 6 + 2 = 32 casas.

"'V!* 9C

Existem 20 balas sobre uma mesa e duas crianas comeam a com-las, uma criana de cada vez. Em cada vez, cada criana deve comer pelo menos uma bala e est proibida de comer mais que a metade das balas que existem sobre a mesa. Nesta brincadeira, ganha a criana que deixar apenas uma bala sobre a mesa. Qual das duas crianas pode sempre ganhar na brincadeira: a primeira ou a segunda a jogar? Como deve fazer para ganhar?

;:< =>?:

Ganha a primeira criana. No incio ele deve comer 5 balas, deixando 15 balas sobre a mesa. A segunda criana deve comer no mnimo uma e no mximo 7 balas, sobrando entre 8 e 14 balas sobre a mesa. Em qualquer caso a primeira criana pode comer algumas balas, deixando exatamente 7 sobre a mesa. A segunda criana agora deve comer entre uma e trs balas, deixando de 4 a 6 balas sobre a mesa. A primeira criana agora come algumas delas, deixando exatamente 3 balas, forando a segunda criana a comer uma. Comendo mais uma aps isso, a primeira criana acaba deixando apenas uma bala no final e ganhando o jogo. De um modo mais geral, a estratgia ganhadora consiste em deixar o adversrio com 2k 1 balas, para algum k
D

. O adversrio obrigado a comer de 1 a ( 2 k 1 1) balas, deixando sobre a

EUREKA! N4, 1999

15

Sociedade Brasileira de Matemtica

mesa um nmero de balas que est sempre entre 2k1 e 2k 2. O primeiro jogador pode, ento, jogar novamente de modo a deixar o adversrio com 1 1 2 k 1 balas. O processo prossegue at o adversrio ser reduzido a 2 1 = 1 bala.

"'V!* 9E

Pintam-se de preto todas as faces de um cubo de madeira cujas arestas medem 10 centmetros. Por cortes paralelos s faces, o cubo dividido em 1.000 cubos pequenos, cada um com arestas medindo 1 centmetro. Determine: a) O nmero de cubos que no possuem nenhuma face pintada de preto. b) O nmero de cubos que possuem uma nica face pintada de preto. c) O nmero de cubos que possuem exatamente duas faces pintadas de preto. d) O nmero de cubos que possuem trs faces pintadas de preto.

;:< =>?:

Esto sem nenhuma face pintada, os cubos interiores ao cubo maior. Portanto devem ser retiradas uma fila de cima e uma fila de baixo, uma da frente e outra de trs, e uma de cada lado, ficando assim com um cubo de aresta 8 que contm 83 = 512 cubos pequenos. a) Esto com uma face pintada aqueles que pertencem a uma face mas no possuem lado comum com a aresta do cubo maior, isto , 82 = 64 em cada face. Como so seis faces, temos 6 64 = 384 cubos pequenos. Esto com duas faces pintadas aqueles que esto ao longo de uma aresta mas no no vrtice do cubo maior, isto , 8 cubos em cada aresta. Como so 12 arestas, temos 8 12 = 96 cubos pequenos. Esto com 3 faces pintadas aqueles que esto nos vrtices do cubo maior, ou seja, 8 cubos pequenos.

b)

c)

"'V!* 98

s k3
1 +

1 1 1 1 1 1 + + + + + para 2 3 4 6 8 10 12 que a soma das restantes seja igual a 1? D todas as solues. Que fraes devem ser retiradas da soma

EUREKA! N4, 1999

16

Sociedade Brasileira de Matemtica

;:< =>?:
1 2 + 1 60 30 20 15 12 10 1 1 1 1 1 40 + + + + + = + + + + + + (*) 3 4 6 8 10 12 120 120 120 120 120 120 120

Devemos escrever 120 como soma de algumas parcelas 60, 40, 30, 20, 15, 12, 10. As solues possveis so 60 + 40 + 20 = 120 60 + 30 + 20 + 10 = 120. Assim, podemos remover
1 1 1 1 1 1 1 , , e ou , e . 4 8 10 12 3 8 10

Evidentemente 15 e 12 no podem aparecer, pois a soma no seria mltipla de 10 nesse caso.

"'V!* 9@ "'V!* 9A

Veja Problema 3 do Nvel 1. Cinco cartes numerados com 3, 4, 5, 6 e 7, respectivamente, so colocados em uma caixa. Os cartes so retirados da caixa, um de cada vez e colocados sobre a mesa. Se o nmero de um carto retirado menor do que o nmero do carto imediatamente anterior, ento este carto imediatamente anterior colocado de volta na caixa. O procedimento continua at que todos os cartes estejam sobre a mesa. Qual o nmero mximo de vezes que retiramos cartes da caixa?

;:< =>?:

O nmero mximo de vezes que retiramos cartes da caixa 15, o que corresponde seqncia de cartes retirados 7, 6, 5, 4, 3, 7, 6, 5, 4, 7, 6, 5, 7, 6, 7. De fato, dentre os primeiros 5 cartes h necessariamente um que menor que o carto seguinte, e que portanto no voltar mais para a caixa, o mesmo acontecendo para pelo menos um carto dentre os 4 seguintes, depois para pelo menos um dentre os 3 seguintes, depois para pelo menos um dentre os dois seguintes, sobrando no mximo um carto, que ser o ltimo a ser retirado da caixa.

"'V!* 9B

Em um tringulo acutngulo ABC o ngulo interno de vrtice A mede 300. Os pontos B1 e C1 so os ps das alturas traadas por B e C, respectivamente e os pontos B2 e C2 so mdios dos lados AC e AB, respectivamente. Mostre que os segmentos B1C2 e B2C1 so perpendiculares.

EUREKA! N4, 1999

17

Sociedade Brasileira de Matemtica

;:< =>?:
O segmento B1 C2 uma mediana do tringulo AB1 B e portanto AC2 = B1 C2 e C = 30. A =BA C2 B 1
B =C B Da BC 2 1 2 1 A + BAC = 60.
B = 30. Finalmente Analogamente, AC
1 2

A
30

C2 C1

B2

C = 180 BC B AC B = 90 C1O 2 2 1 1 2

B1

"'V!* 9C
Veja Problema 5 do Nvel 1.

"'V!* 9E

Pintam-se de preto todas as faces de um cubo de madeira cujas arestas medem n centmetros onde n 3. Por cortes paralelos s faces, o cubo dividido em n3 cubos pequenos, cada um com arestas medindo 1 centmetro. Sabendo que o nmero total de cubos pequenos com exatamente uma face pintada de preto igual ao nmero de cubos pequenos apresentando todas as faces sem pintura, determine o valor de n.

;:< =>?:

Um cubo pequeno que no possui qualquer face pintada provm do interior do cubo grande. Isto significa que esse cubo pequeno parte de um cubo de lado n 2, obtido quando retiramos uma unidade de cada face do cubo original. Assim, existem (n 2)3 cubos pequenos no pintados. Por outro lado, um cubo pequeno com uma face pintada provm da face do cubo original, mas no tendo qualquer parte da aresta deste cubo. Assim, existem 6(n 2)2 cubos pequenos com face pintada. Portanto, (n 2)3 = 6(n 2)2, com n > 2. Logo, n 2 = 6, ou seja, n = 8.

s k3

"'V!* 98
Veja Problema 3 do Nvel 2.

"'V!* 9@

Veja problema 5 do Nvel 1.


EUREKA! N4, 1999

18

Sociedade Brasileira de Matemtica

"'V!* 9A
Uma reta que passa pelos pontos mdios de dois lados opostos de um quadriltero convexo forma ngulos iguais com ambas as diagonais. Mostre que as duas diagonais tm o mesmo comprimento.

;:< =>?:

Sejam ABCD o quadriltero, M,N,P e Q os pontos mdios dos lados AB, BC, CD e DA, respectivamente. MN e PQ so paralelos diagonal AC e medem a metade de seu comprimento, enquanto NP e QM so paralelos diagonal BD e medem a metade de seu comprimento. Assim, MNPQ um paralelogramo. As condies do problema dizem que a reta que passa pelos pontos mdios de dois lados opostos de ABCD (digamos MP , sem perda de generalidade) formam ngulos iguais com AC e BD , portanto com PQ e
Q . Logo MNPQ deve ser um losango, NP , donde MP bissetriz de NP

donde MN = NP , e portanto AC = BD (pois MN = AC / 2 e NP = BD / 2 ).

"'V!* 9B
Sobre os lados AB e AC de um tringulo acutngulo ABC so construdos, exteriormente ao tringulo, semicrculos tendo estes lados como dimetros. As retas contendo as alturas relativas aos lados AB e AC cortam esses semicrculos nos pontos P e Q. Prove que AP = AQ.

;:< =>?:
2

Sejam M o p da altura relativa ao lado AB. Como o tringulo APB retngulo em P, e PM a altura de P em relao a AB temos
C. AP = AB AM = AB AC cos BA
2

Analogamente

mostra-se

que

"'V!* 9C

C . Portanto, AP = AQ. AQ = AC AB cos BA

Seja f : F G uma funo tal que f(1) = 999 e f(1) + f(2) + ... + f(n) = n2f(n) para todo n inteiro positivo. Determine o valor de f(1998).

EUREKA! N4, 1999

19

Sociedade Brasileira de Matemtica

;:< =>?:
Calculemos alguns valores de f(n):
2 f (1) = 999; f (1) + f ( 2) = 2 f ( 2) 3 f ( 2) = 999 f ( 2) = 333
333 2 f (1) + f ( 2) + f (3) = 3 f (3) 8 f (3) = 999 + 333 f (3) = 2 333 999 2 f ( 4) = f (1) + f ( 2) + f (3) + f ( 4) = 4 f ( 4) 15 f ( 4) = 999 + 333 + 2 10 999 999 999 999 , f ( 2) = , f (3) = , f ( 4) = , e razovel Assim, temos f (1) = 1 3 6 10 999 1998 = conjecturar que f ( n) = Para todo n H . Vamos 1 + 2 + ... + n n ( n + 1)

provar esse fato: Para n 2 temos


f (1) + f ( 2) + ... + f ( n ) = n 2 f ( n ) ( n 2 1) f ( n) = f (1) + f ( 2) + ... + f ( n 1) f (n) = f (1) + f ( 2) + ... + f ( n 1) . n2 1 1998 k ( k + 1) = 1998 k 1998 k +1 1998 1 , para k = 1, 2, , n 1, e portanto 1998 + 1998 1998 + ... + 1998 1998 = 1998

Por hiptese de induo,


f (k ) =

2 2 3 1 n 1 n 1998 1998( n 1) 1998(n 1) 1998 2 = f (n) = = pois n 1 = ( n 1)(n + 1)), 2 n n n ( n + 1) n ( n 1)

f (1) + f ( 2) + ... f (n 1) =

como queramos demonstrar. Fazendo n = 1998 temos f(1998) =


1998 1998 1999 = 1 1999 .

"'V!* 9E ;:< =>?:

O menor mltiplo de 1998 que possui apenas os algarismos 0 e 9 9990. Qual o menor mltiplo de 1998 que possui apenas os algarismos 0 e 3?

1998 = 2 999 = 2 33 37. Um nmero formado apenas pelos algarismos 0 e 3 mltiplo de 33 se e somente se o nmero de algarismos 3 mltiplo de 9 (pois ao dividi-lo por 3 obtemos um nmero que possui apenas os
EUREKA! N4, 1999

20

Sociedade Brasileira de Matemtica

algarismos 0 e 1 que deve ser mltiplo de 9, o que ocorre se e s se o nmero de algarismos 1 mltiplo de 9 ). Assim, o nmero desejado deve ter pelo menos 9 algarismos 3, e deve terminar por 0, por ser par. O menor nmero com essas propriedades 3333333330, que mltiplo de 1998 pois par, mltiplo de 33 e mltiplo de 37 por ser mltiplo de 111 ( igual a 111 30030030)).

IKJMLONQPSRTVUWRYX Z\[]_^a`bccdfe]hgjilk]bmn oprq]sk]3tKu5n en u5n m[rqpavb;gfn swp'u]yx[u]zs[rq{s|rq]3mb }3~ ~ (33 ~ f;~ 3r; !3z3r rr 3 rr }+3 3~ rY3 Y ';rSrr

KMOQSVWY \rV. V +V Vh .;f

(k + 2){1 ( [wz + h + j q]2 + [ (gk + 2g + k + 1) (h + j) + h z]2 + [16(k + 1)3 (k + 2) (n + 1)2 + 1 f 2]2 + [2n + p + q + z e]2 + [e3 (e + 2) (a + 1)2 + 1 o2]2 + [(a2 1) y2 + 1 x2]2 + [16r2y4 (a2 1) + 1 u2]2 + [((a + u2 (u2 a))2 1) (n + 4dy)2 + 1 (x + cu)2]2 + [(a2 1) 2 + 1 m2]2 +[ai + k + 1 i]2 + [n + + v y]2 + [p + (a n 1) + b (2an +2a n2 2n 2) m]2 + [q + y (a p 1) + s (2ap + 2a p2 2p 2) x]2 + [z +p (a p) + t (2ap p2 1) pm]2)} rV+r  3
   

  

 

EUREKA! N4, 1999

21

Sociedade Brasileira de Matemtica

u o T p tq?
tu o r o
0/ )tc o }
} qst ,
t o o V" !#  ? T $! s ! W4%& 7 ( ' *)3 3 3 "'V!* 98
,+

s k.-

Considere a tabela 3 3 abaixo, onde todas as casas, inicialmente, contm zeros:


0 0 0

Para alterar os nmeros da tabela, permitida a seguinte operao: escolher uma sub-tabela 2 2 formada por casas adjacentes, e somar 1 a todos os seus nmeros. a) Diga se possvel, aps uma seqncia de operaes permitidas, chegar tabela abaixo:
7 9 2

15

25

12

18

10

b)

Complete o quadro abaixo, sabendo que foi obtido por uma seqncia de operaes permitidas:
14

19

36

14

/10325436780 93:;=<8>3?@0BAC?ED8FHG03IJ:?5IJDK
Antes de resolver o problema, preciso notar que existem quatro quadrados 2 2 no quadrado 3 3. Analisando-os, percebemos que os quadrados do canto so afetados por apenas um deles. Com isso, deduzimos que cada nmero nos quadradinhos do canto indica o nmero de vezes que a operao permitida foi utilizada com o quadrado 2 2 que continha o quadradinho do canto. Os quadradinhos do lado so afetados por dois quadrados diferentes, assim como no diagrama abaixo.
EUREKA! N4, 1999

22

Sociedade Brasileira de Matemtica

1
1 1 ,3 B

3
3 3 3 ,4 C

1
1 ,2 A

2
2 2 ,4 D 4

Como a cada utilizao da operao permitida, se for utilizada uma das subtabelas escritas em um dos quadrados laterais, o quadradinho lateral aumentar em um, conclumos que o valor do quadradinho lateral igual soma dos dois quadradinhos da ponta adjacentes. O quadradinho central, como afetado por todos os quatro quadrados, fcil deduzir que ele nada mais que o nmero de operaes aplicados na tabela. a) Como os nmeros dos quadradinhos do canto representam quantas vezes a sub-tabela que contm aquele quadradinho foi utilizada para fazer uma operao e o quadradinho central o nmero de operaes feitas, podemos concluir que de acordo com o diagrama, A + B + C + D = E*. Mas no nosso caso, temos 7 + 2 + 8 + 10 = 25, ou 27 = 25. Absurdo. Conclumos ento que impossvel obter esta tabela. Outra prova que a cada operao, aumentamos em um o valor de quatro casos, mantendo o resultado divisvel por quatro. No nosso caso, a soma dos nmeros 106, que no divisvel por 4.
A G C F E H B I D

b) No diagrama acima, com os raciocnios antes do problema, temos: G = A + C, F = A + B, I = B + D e H = C + D. Portanto, no problema, 19 = 14 + C, onde C = 5. Com isso, 14 = 5 + D, onde D = 9. Prosseguindo com base em *, 36 = 5 + 9 + 14 + B, onde B = 8. F = 8 + 14 F = 22. Finalizando, 1 = 8 + 9 = 17. A tabela
14 22 8

19

36

17

14

EUREKA! N4, 1999

23

Sociedade Brasileira de Matemtica

"'V!* 9@
Encontre uma maneira de se escrever os algarismos de 1 a 9 em seqncia, de forma que os nmeros determinados por quaisquer dois algarismos consecutivos sejam divisveis ou por 7 ou por 13.

L1M3N5O3PQ8MBR3STVU3R=WXSY8YZ [V\5Y8YS8]"]"\^1Z8\5_a`
Mltiplos de 7 : 7, 14, 21, 28, 35, 42, 49, 56, 63, 70, 77, 84, 91, 98. Mltiplos de 13 : 13, 26, 39, 52, 65, 78, 91. Como nenhum dos mltiplos de 7 e 13, a no ser 77, terminava em 7, na seqncia, 7 deveria ser o 1. nmero, ento, o nmero formado foi 784913526.
5* 3 6 2 784 13 9135 9 1* 5* 5 6* 9* 26 6*

Nota do editor: * significa que no d para continuar sem repetir um nmero j usado.

"'V!* 9A
Em um jogo existem 20 buracos vazios em fila e o jogador deve colocar um pino em cada buraco de acordo com as seguintes regras: a) b) c) Se colocar um pino em um buraco e se os dois buracos vizinhos estiverem vazios, o pino permanece. Se colocar um pino em um buraco e se um dos buracos vizinhos estiver ocupado, o pino deste buraco vizinho deve ser retirado. Se colocar um pino em um buraco e se os dois buracos vizinhos estiverem ocupados, ento um dos pinos vizinhos deve ser retirado.

Determine qual o nmero mximo de pinos que podem ser colocados.

EUREKA! N4, 1999

24

Sociedade Brasileira de Matemtica

L1M3N5O3PQ8MBR3SbVZ\5MacHZ8d3U3MBbVZ8Y8]"WJM3`
O nmero mximo 19. Veja a explicao abaixo: Comeo colocando um pino no primeiro buraco da esquerda, pulo um buraco e coloco outro pino. Depois eu coloco um pino no buraco que est entre os dois e retiro o da direita. Depois pulo uma casa do segundo pino e coloco um terceiro pino. Depois coloco um pino entre o terceiro e o segundo pino e retiro o terceiro pino, o da direita. E fao essa operao sucessivamente at chegar ao ltimo pino.

"'V!* 9B
Sete nmeros naturais so escritos em crculo. Sabe-se que, em cada par de nmeros vizinhos, um deles divide o outro. Mostre que h dois nmeros no vizinhos com a mesma propriedade (isto : um deles divide o outro).

L1M3N5O3PQ8MBR3Sce8WJf8\EMhg8O=UhiV\5Y8Z_aM3]"M3`
b c d f e a g

Em cada dois nmeros adjacentes pelo menos um mltiplo do outro; desse modo impossvel fechar o crculo sem que algum nmero divida um outro nmero que no seja adjacente a ele, pois se a for mltiplo de b e b for mltiplo de c, ento c divide a e j haver dois nmeros no vizinhos com a propriedade. Se a for mltiplo de b e c for mltiplo de b e d, e for mltiplo de d e q e o g no existisse, poderia no haver dois nmeros no vizinhos com a mesma propriedade mas como o g existe ele ter de ser mltiplo de f e divisor de a. Desse modo f ter de ser divisor de a, sendo que com isso haver a propriedade.

"'V!* 98

kj

kj

? 

Prove que em qualquer pentgono convexo existem dois ngulos internos consecutivos cuja soma maior ou igual a 216.

L1M3N5O3PQ8MBR3Sl=m3\5Zd3MBnVZ8WJWJM3Y[VM3R=WX\Ed3O3S8YHbVM3Y8]"Z`
Considere A, B, C, D e E os vrtices do pentgono.
EUREKA! N4, 1999

25

Sociedade Brasileira de Matemtica

Suponha que no existam dois ngulos consecutivos cuja soma seja maior ou igual a 216. Assim: A+ B < 216 B + C < 216 C + D < 216 D + E < 216 E + A < 216 Somando membro a membro: A+ B + B + C + C + D + D + E + A < 5216 2 ( A + B + C + D + E ) < 1080 A + B + C + D + E < 540 , mas a soma dos ngulos internos de um pentgono 540*. Logo A+ B + C + D + E = 540 absurdo. Ento pelo menos 2 ngulos consecutivos tem soma maior ou igual a 216. *Lema: A soma dos ngulos de um pentgono 540. Prova: Seja P um ponto interior a ABCDE, logo P vai formar 5 tringulos no pentgono, a soma de todos os ngulos P dos tringulos 360. E a soma dos ngulos que restam justamente a soma dos ngulos de ABCDE. A soma dos ngulos do pentgono 5180 360 = 540.

C D

"'V!* 9@

No tringulo ABC, D o ponto mdio de AB e E o ponto do lado BC tal que BE = 2 EC. Dado que os ngulos
r oqps

t1or L1M3N5O3P.Q8MBR3SvVZU3\5S8N8^1\5U3w3S\5WJMhL1M3x3WJS\5WJZ8`
B x y D y A P

u t1o

so iguais, encontre o ngulo

W W 90

O 2

E x W 90 C

EUREKA! N4, 1999

26

Sociedade Brasileira de Matemtica

Chamarei de P o ponto mdio de BE . E chamarei BP = PE = EC = X. e BD = DA = y. A interseo de AE com DC , chamarei de O. O tringulo DOA issceles, portanto AO = DO . Chamarei AO de W. O segmento DP a base mdia do tringulo ABE, pois D o ponto mdio de AB e P o ponto mdio de BE , ento DP // AE. Conseqentemente os tringulos OCE e DCP so semelhantes, na razo de 1/2. Ento temos CO = 1 / 2 = D , 2 CO = CO + W = D , W = CO. CO + W Chamarei o ngulo OD de , ADO, tambm ser , e o ADC, ser 2, pois externo ao tringulo DOA. Como AO = W e OC = W o tringulo ADC issceles e o ngulo da base 90 . O ngulo BAC = + 90 , BC = 90.

"'V!* 9A
Veja problema 3 do Nvel 1.

"'V!* 9B
So dados 15 nmeros naturais maiores que 1 e menores que 1998 tais que dois quaisquer so primos entre si. Mostre que pelo menos um desses 15 nmeros primo.

L1M3N5O3PQ8MBR3SiVO3_Bx3S8WJ]"MBL1\5N5yZHzVZ8y8SY8`
Teorema: Dado um nmero n, composto, ento ele possui um fator (1) menor ou igual raiz quadrada deste nmero. Prova: Se n = a b, podemos ter ou a < 1.2.3.a= a< a> n n n b=
n n < < n a n

n,a=

n ou a >

n:

Em qualquer caso, temos um fator menor ou igual a

n e diferente de 1.

EUREKA! N4, 1999

27

Sociedade Brasileira de Matemtica

Resposta: Dado 1 < n < 1998, se ele no for primo, ele tem que ter um fator primo menor que 1998 , ou seja, um fator primo, menor que 45. Como s existem 14 primos menores que 45, e so 15 nmeros, ento um desses no ter fator primo menor que 45, logo ser primo. (Pelo Corolrio do teorema anterior.)

|{ .qq.
Veja Problema 4 do Nvel 2.

|{

|{

}~ 3

Primeira Prova.

s
No tringulo ABC, D o ponto mdio de AB e E o ponto do lado BC tal que o p r BE = 2 . EC. Dado que os ngulos e so iguais, encontre o ngulo

t or .

Veja a soluo do Problema 2 do Nvel 2.

L1M3N5O3PQ8MBZ8N5]"SWXU=Z8]"\5yZHR3SH3Z8x=WX\Ef\5MhL1\E3O3S8\EWXZHVS8U3S8y8\5R3SY8`

A y D y A

F 2x E x C

Seja F = AE CD. Denotamos por [XYZ] a rea do tringulo XYZ. Veja que [ABE] = 2 [ACE] pois possuem a mesma altura relativa a BE e EC respectivamente e DE = 2 EC. Analogamente [BFE] = 2 [FEC]. Temos: [ABE] = 2 [ACE] (I) [BFE] = 2 [FEC] (II)
EUREKA! N4, 1999

28

Sociedade Brasileira de Matemtica

I II [ABE] [BFE] = 2 ([ACE] [FEC]) [ABF] = 2 [AFC]. S que: [ABF] = 2 [ADF] (mesma altura relativa a BA e DA respectivamente e BA = 2 DA) Donde 2 [ADF] = 2 [AFC] [ADF] = [AFC]. Como esses possuem a mesma altura relativa a DF e FC respectivamente, temos: DF = FC F A C = A C F = Mas = A D C = B A E DF = AF . Da, DF = AF = FC e CDA

retngulo em A . Prova: ADC : + + + + = 180 + = 90 DC = 90 Logo BC = 90 o que queramos achar.

.qq.
Duas pessoas disputam um jogo da maneira descrita a seguir. Inicialmente escolhem dois nmeros naturais: n 2 (o nmero de rodadas) e t 1 (o incremento mximo). Na primeira rodada o jogador A escolhe um natural m1 > 0 e, posteriormente, o jogador B escolhe um natural positivo n1 m1. Para 2 k n, na rodada k o jogador A escolhe um natural mk com mk 1 < mk mk 1 + t e posteriormente o jogador B escolhe um natural nk com nk 1 < nk nk 1 + t. Aps essas escolhas, nessa k-sima rodada, o jogador A ganha mdc (mk , nk 1) pontos e o jogador B ganha mdc (mk , nk) pontos. Ganha o jogo o jogador com maior pontuao total ao fim das n rodadas. Em caso de pontuaes totais iguais, o jogador A considerado vencedor. Para cada escolha de n e t, determine qual dos jogadores possui estratgia vencedora.

13538B3V33338153H3@533
Nota: A soluo usa a notao abreviada (m, n) = mcd(m,n). Seja S um super-nmero. Esse super-nmero divisvel por todos os naturais. Para B ganhar sempre, basta fazer nk = S + mk. Pois, na rodada k, B ganha (nk, mk) = mk pontos e A ganha (S + mk1, mk) pontos. Assim A ganha no mximo mk pontos, temos que mkS + mk1 mkmk1, mas mk > mk1. Assim A nunca ganhar mk pontos. Ganhar sempre menos que mk. Portanto
EUREKA! N4, 1999

29

Sociedade Brasileira de Matemtica

B no total ficar com mais pontos que A, j que ganha mais em todas as rodadas. Obs. claro que no existe um super-nmero, mais para cada n, t e m1, B pode criar um nmero, que no chega a ser super, mas tambm serve. Basta fazer: S = ( 235711p)M, com e M suficientemente grandes!, Assim B sempre ganha! Explicando melhor: Aps A escolher m1, B sabe que o maior nmero que A pode escolher m1 + (n 1) t B s precisa fazer S=
m1 + ( n 1) t j = m1

j e nk = S + mk

Justificativa: m1 < mi m1 + (n 1) t miS Na rodada k: B ganha (mk, nk) = mk, j que mkS + mk A ganha (mk, nk 1) = (mk, S + mk 1)

Como m k > m k 1 m k no divide m k 1 m k no divide ( mk 1 + S ) m k > (m k , n k 1 ) m S m S k k Portanto, a cada rodada B ganha mais pontos que A, e portanto B ganha o jogo. Obs. mk 1 < mk mk 1 + t S + mk 1 < S + mk S + mk 1 + t nk 1 < nk nk 1 + t Resposta: B sempre ganha (se for esperto)!!!

.qq.

}~ 3

Segunda Prova.

Dois meninos jogam o seguinte jogo. O primeiro escolhe dois nmeros inteiros diferentes de zero e o segundo monta uma equao do segundo grau usando como coeficientes os dois nmeros escolhidos pelo primeiro jogador
EUREKA! N4, 1999

30

Sociedade Brasileira de Matemtica

e 1998, na ordem que quiser (ou seja, se o primeiro jogador escolhe a e b o segundo jogador pode montar a equao 1998x2 + ax + b = 0, ou bx2 + 1998x + a = 0, etc.) O primeiro jogador considerado vencedor se a equao tiver duas razes racionais diferentes. Mostre que o primeiro jogador pode ganhar sempre.

13538B3=83J5Eh153=85JHC3538
Inicialmente veja que, se num polinmio p(x) = anxn + + a1x + a0, tivermos an + a1 + a0 = 0, teremos p (1) = 0 e 1 raiz de p. Dessa forma se o primeiro jogador escolhe b = (1998 + a), 1 ser raiz da equao do segundo grau que o seu oponente ir montar. Se um polinmio tem coeficientes inteiros (na verdade vale para coeficientes racionais) e possui uma raiz irracional do tipo a + b r (r no quadrado perfeito), ento a b r tambm raiz. Ou seja, as raizes irracionais vm aos pares. No caso de uma equao de segundo grau, e coeficientes inteiros, ambas as raizes so irracionais, ou ambas so racionais. No nosso caso, como 1 j raiz, a outra raiz ser racional. Basta ver ento, apenas se 1 no raiz multipla (pois queremos que as raizes sejam distintas). Para isso basta escolher a adequadamente. Se o primeiro jogador escolher os nmeros a = n1998, e b = (n + 1) 1998, n 2 , ele ganha. (1 no ser raiz mltipla e a equao ter duas raizes racionais distintas.) Obs. possivel obter solues com a + b + 1998 0, por exemplo com {a, b} = {2040, 5478} (soluo obtida com o auxlio de um computador.)

.qq.
Determine todas as funes f : que satisfazem f (2 f ( x)) = x + 1998 para todo x = {0,1,2,...}.

13538B31B83.8V33=8"h=13.HCJ385J3
1 Passo. f injetiva. Prova: Suponha que exista k tal que f (x) = f (y) = k para x, y e x y. Da f(x) = f(y) 2f(x) = 2 f(y) f(2f(x)) = f (2f(y)) x + 1998 = y + 1998 x = y. Absurdo! Logo f injetiva.
EUREKA! N4, 1999

31

Sociedade Brasileira de Matemtica

2 Passo. f e sobrejetiva no contradomnio{k k 1998}, isto , para todo k , k 1998 tal que f() = k. Prova: Se k 1998 faa k = x + 1998 para x 0 e x , logo f(2f(x)) = x + 1998 = k Tome ento 2f(x) = . Logo f sobrejetiva no contradomnio acima. 3 Passo. Ento para todo k A = {k k 1998} existe um nico natural x (que chamaremos f ' (k) = x) tal que f(x) = k. Obviamente existe um natural i mpar tal que f (i) = w 1998 pois no podemos ter 0 f(i) 1997 para todos os naturais mpares porque a funo f injetiva e de em (princpio de casa de pombos). Para esse w teremos f '(w) = i, pois f (i) = w. Por outro lado faa w = a + 1998 com a Pela definio: f (2f(a)) = a + 1998 = w. Logo f '(w) = 2f (a). Como f '(w) nico, pela concluso 1 i = 2f(a) como f(a) i par, absurdo! Concluso 2. No existe funo desse tipo.

.qq.
Dois matemticos, perdidos em Berlim, chegam esquina da rua Barbarossa com a rua Martin Luther, e precisam chegar esquina da rua Meininger com a rua Martin Luther. Infelizmente eles no sabem para que lado fica a rua Meininger, nem a que distncia ela est, logo so obrigados a ir e voltar ao longo da rua Martin Luther at chegaram esquina desejada. Qual o menor valor para o nmero positivo K tal que eles podem ter certeza de que se h N quarteires (ou quadras) entre as ruas Barbarossa e Meininger ento eles conseguem chegar ao destino andando no mximo KN quarteires (ou quadras)?

13538B"33Hh8*h8"E8338J35=38V8J5Eh
Este problema baseado numa situao real, ocorrida com os matemticos Nicolau Saldanha e Carlos Gustavo Moreira, que se encontravam em Berlim por ocasio do Congresso Internacional de Matemtica de 1998. Era de noite, no havia ningum na Martin-Luther-Strasse a quem pedir informaes e eles queriam chegar rpido ao destino. A idia do problema era que eles andassem juntos. Apareceram solues em que os matemticos se separavam (o que no havia sido previsto), as quais foram avaliadas caso a caso. Como os matemticos no sabiam para que lado nem a que distncia estava a Meiningerstrasse, deviam adotar uma estratgia do seguinte tipo:
EUREKA! N4, 1999

32

Sociedade Brasileira de Matemtica

andar a1 quarteires para um lado (digamos o direito), depois voltar ao ponto inicial e andar a2 quarteires para a esquerda, depois a3 para a direita, depois a4 para a esquerda e assim sucessivamente, onde a1, a2, a7so nmeros inteiros positivos com a1 < a3 < a5 < e a2 < a4 < a6 < at encontrar a Meiningerstrasse. Os piores casos so quando a Meininger est a a2k + 1 + 1 quarteires direita ou a2k + 1 esquerda da Barbarossastrasse, com k natural (convencionamos a0 = 0). Nesses casos, temos que entre o ponto inicial e o destino h an + 1 quarteires, e os matemticos andam no total 2a1 + 2a2 + + 2an + 2an + 1 + an + 1 quarteires at chegarem ao destino (com n = 2k+1 ou n = 2k). Assim, devemos ter 2a1 + 2a2 + + 2an + 2an + 1 + an + 1 k (an + 1), ou seja, k 1 S n +1 (a n + 1 ) para todo n , onde Sn = a1 + a2 + + an.
2

Para k = 9 existem estratgias que satisfazem as condies do problema, por exemplo tomando am = 2m para todo m . De fato, teremos Sn + 1 = 21 + 22 k 1 + + 2n + 1 = 2n + 2 2 < 4 (2 n + 1) = (a n + 1) para todo n . 2 k 1 Mostraremos que 9 o menor k possvel. Seja k < 9. Ento c = 2 menor que 4. Se k satisfaz as condies do problema, deve haver uma sequncia (an) como acima com Sn + 1 c (an + 1) para todo n . Como an = Sn Sn 1 teremos Sn + 1 c (Sn Sn 1 + 1) para todo n . Definimos Un = Sn c, temos Un + 1 c (Un Un 1) para todo n . Como c < 4, Un > 0 para todo n 3, e, definindo Vn = Un + 1 /Un para todo n 3, teremos Vn c (1 1/Vn 1) para todo n 4, onde Vn > 0 para todo n 3. Entretanto, Vn c (1 1/Vn 1) implica Vn Vn 1 c ( 1 1/Vn 1) Vn 1 =
cV n 1 c V n V n 1 todo n
2

c ( c 4) c(c 4) para todo n 4, donde Vn Vn 1 Vn Vn 1 4V n 1 4V3 (n 3) c(c 4) para todo n 4, absurdo, pois o lado direito negativo V n V3 + 4V3
2

c2 c c V n 1 1 4 2 c (c 4) < 0 V < V = n n 1 4V n 1 V n 1 4. Por outro lado, para todo n 4

para temos

4V3 para n > 3 + c(4 c) K


EUREKA! N4, 1999

33

Sociedade Brasileira de Matemtica

@ HBa @@CC" = 5


Nome Mrcio Jun Hisamoto Fbio Dias Moreira Andressa Rissetti Paim Henry Wei Cheng Hsu Natlia Argene Lovate Pereira Daniel Cherobini Luis Eduardo de Godoi Milton Eiji Kato Fabrcio Henrique de Faria Davi Mximo Alexandrino Nogueira Caio Magno Castro de Paula Bruno Moreira de Souza Das Patrcia Akemi Komura Marcelo Li Koga Alberto Hikaru Shintani Diego Gomes Gripp Renato Mendes Coutinho Antnio Monteiro Guimares Jr. Leonardo Luis Desideri Freitas Helder Seiji Kato Aline Galvo Rodrigo Miyashiro Nunes dos Santos Joo Marcos da Cunha Silva Thiago Mizuta Oliveiro Ribeiro Barbosa Jr. Jorge Peixoto Andria Lcio de Castro Lo Jaime Zandonai Pedro Junqueira de Barros Lucas Ikeda Frana Paulo Roberto Sampaio Santiago Rafael Marini Silva Eduardo Souza Cruz Breno Igncio da Silva Diogo dos Santos Suyama Flavio Schiavini Abe Cidade - Estado So Paulo - SP R. de Janeiro - RJ Santa Maria - RS So Paulo - SP Jundia - SP Santa Maria - RS SJ dos Campos - SP So Paulo - SP So Paulo - SP Fortaleza - CE Fortaleza - CE SJ dos Campos - SP So Paulo - SP So Paulo - SP So Paulo - SP Vitria - ES Americana - SP Campina G - PB Vitria - ES So Paulo - SP So Paulo - SP So Paulo - SP Fortaleza - CE So Paulo - SP Teresina - PI Goinia - GO Goinia - GO BentoGonalves - RS So Paulo - SP So Paulo - SP Salvador - BA Vila Velha - ES Vitria - ES Sertozinho - SP Belo H. - MG Vitria - ES Prmio Ouro Ouro Prata Prata Prata Prata Prata Prata Prata Prata Prata Prata Bronze Bronze Bronze Bronze Bronze Bronze Bronze Bronze Bronze Bronze Bronze Bronze Bronze Meno Honrosa Meno Honrosa Meno Honrosa Meno Honrosa Meno Honrosa Meno Honrosa Meno Honrosa Meno Honrosa Meno Honrosa Meno Honrosa Meno Honrosa

EUREKA! N4, 1999

34

@  @CV
= C 5
Nome Thiago Barros Rodrigues Costa Humberto Silva Naves Daniel Pinheiro Sobreira Afonso de Paula Pinheiro Rocha Hugo Pinto Iwata Thiago da Silva Sobral Joo Alfredo Castellani Fajardo Freire Artur Duarte Nehmi Einstein do Nascimento Jr. Daniel Pessa Martins Cunha Gustavo Alonso Daud Patavino Rafael de Holanda Barroso Eduardo Kunio Kuroda Abe Renata Loureno Delamanha Ricardo de Castro Palcio Helen Wei Ling Hsu Frederico Pinto Eduardo Famini Silva Victor Marchesini Ferreira Thalita Basso Eduardo Suaiden Klein Mauricio Massao Soares Matsumoto Claudia Giacomin Bof Guilherme Silveira Barrozo Netto Thiago Arajo Fiorio Pedro Fernando Almeida Di Donato Gustavo Modenesi Jaquellyne Gurgel Penaforte Jordan Guimares Lombardi Tiago Monteiro Fernandes Marcelo Loula Novais de Paula Cidade - Estado Fortaleza - CE Goinia - GO Fortaleza - CE Fortaleza - CE SJ de Rio Preto - SP Fortaleza - CE Salvador - BA So Paulo - SP Fortaleza - CE Fortaleza - CE Santos - SP Fortaleza - CE So Paulo - SP Jundia - SP Fortaleza - CE So Paulo - SP So Paulo - SP Salvador - BA Salvador - BA Jundia - SP R. de Janeiro - RJ So Paulo - SP Aracruz - ES R. de Janeiro - RJ Fortaleza - CE SJ dos Campos - SP So Paulo - SP Fortaleza - CE SJ dos Campos - SP Piracicaba - SP Salvador - BA Prmio Ouro Ouro Ouro Prata Prata Prata Prata Prata Prata Prata Prata Bronze Bronze Bronze Bronze Bronze Bronze Bronze Bronze Bronze Meno Honrosa Meno Honrosa Meno Honrosa Meno Honrosa Meno Honrosa Meno Honrosa Meno Honrosa Meno Honrosa Meno Honrosa Meno Honrosa Meno Honrosa

Sociedade Brasileira de Matemtica

EUREKA! N4, 1999

35

C@  = 5CC 


Nome Emanuel Augusto de Souza Carneiro Mauricio Pereira Carrari Rui Lopes Viana Filho Fernando Paz Cardoso Fabricio Siqueira Benevides Tony Calleri Frana Jnathas Digenes Castello Branco Glauf Sidney Duarte Moreira Jr. Sergio Alvarez Arajo Correia Srgio Tadao Martins Lucas Heitzmann Gabrielli Christian Iveson Bruno Gurgel Fernandes Tvora Mila Lopes Viana Daniel Massaki Yamamoto Daniele Vras de Andrade Mauricio Masayuki Honda Leonardo Cardoso Souza Daniel Nobuo Uno Daniel Mouro Martins Joo Paulo de Tarso Ferreira Evandro Makiyama de Melo Christian Lyoiti Watanabe Fred Olavo A. Carneiro Bruno Da Cunha Raymundo Pedro Paulo de Simoni Gouveia Carlos Alexandre Rolim Fernandes Mrcio Afonso Assad Cohen Matheus de Lima Faheina Rodrigo M. Gorgoll Vtor Menezes Santana Alexandre Ferreira Terezan Ricardo Sallai Viciana Remo H. de M. Furtado Wayne Leonardo Silva de Paula Thiago Steiner Alfeu Mrcio Reis Lopes Seong Ho Lee Cidade - Estado Fortaleza - CE So Paulo - SP So Paulo - SP So Paulo - SP Fortaleza - CE Fortaleza - CE Fortaleza - CE Fortaleza - CE Fortaleza - CE So Paulo - SP So Paulo - SP So Paulo - SP Fortaleza - CE So Paulo - SP So Paulo - SP Fortaleza - CE So Paulo - SP Angra dos Reis - RJ So Paulo - SP Fortaleza - CE Angra dos Reis - RJ So Paulo - SP Angra dos Reis - RJ Fortaleza - CE Rio de Janeiro - RJ Fortaleza - CE Fortaleza - CE Rio de Janeiro - RJ Fortaleza - CE So Paulo - SP Goinia - GO Rio de Janeiro - RJ So Paulo - SP Fortaleza - CE Belm - PA Fortaleza - CE Salvador - BA Santo Andr-SP Prmio Ouro Prmio especial Ouro Ouro Ouro Ouro Prata Prata Prata Prata Prata Bronze Bronze Bronze Bronze Bronze Bronze Bronze Bronze Bronze Bronze Bronze Bronze Bronze Meno Honrosa Meno Honrosa Meno Honrosa Meno Honrosa Meno Honrosa Meno Honrosa Meno Honrosa Meno Honrosa Meno Honrosa Meno Honrosa Meno Honrosa Meno Honrosa Meno Honrosa Meno Honrosa Meno Honrosa

Sociedade Brasileira de Matemtica

EUREKA! N4, 1999

36

Sociedade Brasileira de Matemtica

 H a 
Eduardo Wagner

   !"$#%&' ("$' )+*

Voc sabe quando foi realizada a primeira Olimpada de Matemtica? Foi no ano de 1894, na Hungria. Neste ano, a Sociedade de Matemtica e Fsica da Hungria promoveu uma competio de Matemtica, envolvendo todos os alunos dos ltimos anos das escolas, para homenagear seu presidente Lornd Etvs, eleito ministro da educao do pas. O evento foi um sucesso, e passou a ser realizado todos os anos. Vamos mostrar neste artigo alguns problemas dessas competies com suas solues resumidas. Os problemas escolhidos no so muito difceis, mas so bastante interessantes. Recomendo aos leitores pensar um pouco em cada um deles antes de ver a soluo. As ferramentas exigidas so elementares (apenas no problema 2 a noo de congruncia adequada) mas as solues necessitam de uma certa dose de criatividade. Aproveitem! Olimpada de 1894 Prove que as expresses 2x + 3y e 9x + 5y so divisveis por 17 para os mesmos pares de valores dos inteiros x e y. Olimpada de 1898 Determine todos os valores do natural n, para os quais 2 n + 1 mltiplo de 3. Olimpada de 1905 Na figura a seguir, AM, BN e CP so paralelos.
N M P

.qq. .qq.-, .qq.

Prove que

EUREKA! N4, 1999

37

Sociedade Brasileira de Matemtica

Olimpada de 1906 A seqncia a1, a2, a3, , an representa uma arrumao arbitrria dos nmeros 1, 2, 3, , n. Prove que se n um nmero mpar o produto (a1 1)(a2 2)(a3 3) (an n) um nmero par. Olimpada de 1910 Se a, b, c so nmeros reais tais que a 2 + b 2 + c 2 = 1 , prove que 1 ab + bc + ca 1 2

.qq.-.

1 1 1 + = AM BN CP

.qq.-/

Olimpada de 1913 2 Prove que para todo natural n > 2, tem-se (1 2 3 n ) > n n . Olimpada de 1916 No tringulo ABC, AD a bissetriz do ngulo A. Prove que AD < AB AC . Olimpada de 1916 Divida os nmeros 1, 2, 3, 4, 5 em dois conjuntos quaisquer. Prove que um dos conjuntos contm dois nmeros e sua diferena.

.qq.-0 .qq.-1 .qq.-2

 354%6  .qq.
Observe que 4(2x + 3y) + (9x + 5y) = 17(x + y). Portanto, se 2x + 3y for mltiplo de 17, ento 9x + 5y tambm ser, e vice versa. OBS: Esta aparente mgica no a nica forma de resolver este problema. Os leitores que conseguirem outra soluo (para este ou para qualquer problema deste artigo) podem enviar suas descobertas para publicao nos prximos nmeros da

798;:<79=?>@

EUREKA! N4, 1999

38

.qq.-,

Sociedade Brasileira de Matemtica

A soluo mais natural para este problema utiliza congruncias. Observe que 2 (1) md.3. Logo, 2n (1)n md.3 e, portanto, 2n + 1 (1)n + 1 md.3. Concluimos ento que , 2n + 1 0 md.3 se, e somente se, n mpar. OBS: O leitor familiarizado com induo pode conseguir outra soluo.

.qq.
N M P

Utilizando semelhana de tringulos na figura acima temos: CP CB = AM AB CP AC = BN AB Somando temos: CP CP AC + CB + = =1 AM BN AB Da, 1 1 1 + = AM BN CP

.qq.-.

O produto (a1 1)(a2 2)(a3 3) (an n) possui um nmero mpar de termos porque n mpar. Mas, a soma desses termos zero, que par. Como a soma de uma quantidade mpar de nmeros mpares no pode ser par, conclumos que um dos termos par e, conseqentemente, o produto um nmero par.

EUREKA! N4, 1999

39

.qq.-/
Primeira parte:

Sociedade Brasileira de Matemtica

(a + b + c )2 0
a 2 + b 2 + c 2 + 2ab + 2bc + 2ac 0

1 + 2(ab + bc + ca ) 0
ab + bc + ca 1 2

Segunda parte:

(a b )2 + (b c )2 + (c a )2 0 2(a 2 + b 2 + c 2 ) 2(ab + bc + ca ) 0 1 (ab + bc + ca ) 0


ab + bc + ca 1

.qBA.-0

A expresso do lado esquerdo da desigualdade pode ser escrita assim: 1.n.2.(n 1).3.(n 2). .(n 2).3.(n 1).2.n.1 Considere agora separadamente os produtos: 1.n, 2.(n 1), 3.(n 2), , (n 2).3, (n 1).2, n.1 O primeiro e o ltimo so iguais a n, mas afirmamos que qualquer um dos outros maior que n. De fato, os produtos do meio so da forma (k + 1)(n k) onde k assume os valores: 0, 1, 2, , n 1. Como para eles, n k maior que 1, temos que (k + 1)(n k) = k(n k) + (n k) > k.1 + (n k) = n Logo, como n maior que 2, o produto do lado esquerdo maior que n.n.n. .n = nn.

.qBA.-1
Considere a circunferncia circunscrita ao tringulo ABC.

EUREKA! N4, 1999

40

Sociedade Brasileira de Matemtica

D B E C

A bissetriz AD encontra a circunferncia em E, ponto mdio do arco BC. Como os ngulos ABC e AEC so iguais (cada um deles vale a metade do arco AC) e como os ngulos BAE e EAC so tambm iguais (porque AD uma bissetriz), concluimos que os tringulos ABD e AEC so semelhantes. Da, AB AD = AE AC ou seja, AD AE = AB AC Como AD menor que AE temos que AD AD < AB AC ou seja, AD < AB AC

.qBA.-2
Vamos tentar dividir 1, 2, 3, 4, 5, em dois conjuntos tais que nenhum deles contm a diferena de dois de seus elementos. O 2 no pode estar no mesmo conjunto que o 1 ou o 4 porque 2 1 = 1 e 4 2 = 2. Portanto, vamos colocar o 2 em um conjunto e o 1 e o 4 no outro. O 3 no pode ficar no segundo conjunto porque 4 3 = 1. Logo, o 3 deve ficar no primeiro conjunto, junto com o 2. Agora, o 5 no pode ficar no primeiro conjunto porque 5 3 = 2, e nem pode ficar no segundo porque 5 4 = 1. A diviso proposta portanto impossvel.

EUREKA! N4, 1999

41

C ha H@Ha

Sociedade Brasileira de Matemtica

   !"$#%&' ("$' )+*

Paulo Cezar Pinto Carvalho IMPA

Voc j prestou ateno ao logotipo da Olimpada Brasileira de Matemtica, presente na capa da e (em sua verso animada) na pgina da Internet da OBM? Os crculos coloridos so uma referncia ao smbolo dos Jogos Olmpicos, que formado por 5 anis entrelaados representando os continentes. No logotipo da OBM, porm, estes anis esto dispostos de um modo tal que conhecimentos matemticos so essenciais para sua construo. O que existe de difcil em dispor cinco anis de modo que cada um seja tangente a dois outros e, alm disso, tangente a dois crculos adicionais, um interior e outro exterior? Vejamos.

798;:<79=?>@

Tomemos dois crculos arbitrrios, um contido no outro e posicionemos um novo crculo, de modo a ser tangente a ambos. A partir da, os demais crculos esto definidos e a Fig. 1 mostra o que ocorre no caso geral: quando tentamos colocar o ltimo crculo, vemos que a figura no fecha, ou seja, no possvel colocar um quinto crculo tangente a dois dos quatro crculos j colocados e aos dois iniciais.

DE FHGJILK M NJOPE QJR$SUTWV XYTZOP[ SUQP\JS^]Z__JQ`TZaPE bZa


Ser que possvel colocar o primeiro crculo colorido em outra posio, de modo a fazer com que a figura se feche exatamente? Pode-se ter uma idia da resposta a esta pergunta observando a verso animada do logotipo. Observe que os crculos interno e externo so fixos, mas os coloridos assumem tamanhos e posies variveis e parecem girar em torno deles (veja a Fig. 2 a seguir). Ou seja, a animao sugere que o fechamento da figura no depende da posio ou tamanho do primeiro crculo colorido, dependendo somente do tamanho e posio relativas dos crculos interno e externo!
EUREKA! N4, 1999

42

Sociedade Brasileira de Matemtica

DcE FHGedUK M]TZE QJTSUTWV X!TZOP[ S`]]f__PQJTfa`E bfaPgihJaPXjaNPOPaJ[ N`OJ_PXhJS`]ZE k\JSlJS^hJXjE gU_`E XjS
A explicao para estes fatos est em uma transformao geomtrica dos pontos do plano chamada de inverso e definida do seguinte modo. Definio: Seja O um ponto do plano e k um nmero real positivo. A inverso de centro O e constante k associa a cada ponto P do plano, distinto de O, o ponto P (chamado de inverso de P) sobre a semi-reta OP tal que OP. OP = k. A Fig. 3 a seguir ilustra o resultado de se aplicar uma transformao de inverso a um conjunto de pontos do plano. Como o produto OP. OP deve ser constante, quanto mais prximo um ponto estiver de O, mais distante o seu inverso estar.
Q' Q P O P' R R'

DcE FBGemUKn Q`oZ_PXj]f\PS

O logotipo da OBM construdo com o auxlio deste tipo de transformao, explorando dois fatos fundamentais. a) Inverses so tranformaes injetivas (isto , pontos distintos possuem inversos distintos). Para verificar este fato, basta observar que o ponto P cujo inverso um certo ponto P est univocamente determinado e justamente o inverso de P (ou seja, a transformao inversa de uma inverso ela mesma).

EUREKA! N4, 1999

43

Sociedade Brasileira de Matemtica

b) O inverso de um crculo que no passa pelo centro de inverso um outro crculo. Consideremos uma inverso de centro O e constante k e tomemos um crculo C que no passa por O. Seja P um ponto de C, P o seu inverso e Q o outro ponto em que a reta OP corta C.
P'

Q O

DcE FHGqpK M E Q`of_JXj]fSlP_UO`grTWV XjTOP[ S


Uma propriedade fundamental do crculo que o produto OP. OQ igual a uma constante p (a potncia de O em relao a C) para qualquer posio de P. Assim, OP ' OP.OP ' k = = OQ OP.OQ p Portanto, o inverso de C pode ser obtido assim: para cada ponto Q de C, tomamos o ponto P' sobre a semi-reta OQ tal que OP' = (k/p) OQ. Este tipo de transformao chamado de homotetia e sempre transforma uma figura em outra semelhante (ela faz uma ampliao ou reduo da figura, conforme k/p seja maior ou menor que 1). Em particular, o transformado de um crculo por homotetia sempre um outro crculo. Em resumo: o inverso de um crculo (que no passa pelo centro de inverso O) um outro crculo, obtido atravs de uma homotetia de centro O (para voc pensar: como ser o inverso de um crculo que passa por O?). Agora estamos em condies de entender como construdo o logotipo da OBM. O ponto de partida a figura abaixo: dois crculos concntricos, com cinco crculos de raios iguais encaixados entre eles.

DcE FBGesUt^M hJS`QPR$SUlP_hPaJXjR$E l`aBueTZE Q`TZS^T V XjTZO`[ SJ]E FPOJa`E ]_PQJR$XY_T V XYTZOP[ SJ]TZS`QJTfvJQPR$XYE TZS`]
EUREKA! N4, 1999

44

Sociedade Brasileira de Matemtica

S possvel encaixar estes 5 crculos para um determinado valor da razo R/r entre os raios dos crculos externo e interno. De modo mais geral, vejamos qual deve ser esta razo para que n crculos possam ser encaixados entre os dois crculos concntricos. O dimetro de cada um dos crculos iguais a diferena R r entre os raios dos crculos concntricos. Por outro lado, seus centros formam um polgono regular de n lados, inscrito em um R+r crculo de raio concntrico aos dois crculos iniciais, como mostra a 2 figura abaixo.

DE FHGqwK xOJaPQPlPSyUhPS`]Z] V of_J[f_`QJTfaPE bZaPX z TWV XjTfOP[ SP]{_PQPR|Xj_^T V XjTfOP[ SP]TfSPQPTfvPQJR$XjE TZSP]Z}
No tringulo retngulo OAM , a hipotenusa OA mede cateto AM mede Rr e oposto a um ngulo igual a 180/n. Assim: 2 Rr R+r = sen(180 / n) 2 2 ou, desenvolvendo: R 1 + sen(180/ n) = r 1 sen(180 / n) No nosso caso, em que n = 5, devemos ter R 1 + sen(36 ) = r 1 sen(36 ) Quando os raios R e r esto nessa proporo, possvel encaixar cinco crculos iguais entre os dois crculos concntricos.
EUREKA! N4, 1999

R+r eo 2

45

Sociedade Brasileira de Matemtica

Para terminar de formar o logotipo, tomamos o conjunto formado pelos dois crculos concntricos e pelos cinco crculos de raios iguais encaixados entre eles e aplicamos uma transformao de inverso.

~c HeUt^ `PJ$ hPlJa^?5qP`| lPhPJ `oZ_PjZ`


A menos que o centro de inverso seja o seu centro comum, os inversos dos crculos concntricos no so concntricos. Alm disso, os inversos dos cinco crculos iguais no tem mais raios iguais, dando o aspecto irregular do logotipo. Os raios dos crculos tornam-se mais desiguais quanto mais o centro de inverso se afasta do centro dos crculos concntricos. Note, no entanto, que as propriedades de tangncia so preservadas, em virtude da injetividade da inverso, que faz com que o nmero de pontos de interseo de figuras seja preservado atravs da transformao. Para produzir a animao do logotipo, basta girar o conjunto de cinco crculos de raios iguais na figura original: seus transformados por inverso mudaro de tamanho e posio medida que isto ocorre. Se voc quiser, pode experimentar com as propriedades dessa transformao visitando a pgina da OBM na Internet. Basta clicar sobre o smbolo animado da OBM, ou ir diretamente ao seguinte endereo: http: //www.obm.org.br/logotipo.htm. Voc encontrar uma pgina interativa que permite variar as propores do smbolo atravs da escolha do centro de inverso. Voc at poder criar smbolos diferentes mudando o nmero de anis! Na verdade, os "designers" que criaram o logotipo da OBM utilizaram um programa parecido, para ajustar o tamanho e posio relativa dos anis de modo a produzir uma figura agradvel do ponto de vista visual. Este um bom exemplo do emprego da Matemtica em artes visuais. H casos notveis de artistas, como Escher, que usaram a Matemtica como ferramenta essencial em seu processo criativo. Em futuros nmeros da voltaremos a falar de inverso, estudando suas propriedades em mais detalhe e mostrando outras aplicaes. Aguardem!

9;<9?

EUREKA! N4, 1999

46

5%5--U%U Zf ZZZZfZ fZZZZ!f f ZZ ZZfWZPZZZ c ZZ


11)

Sociedade Brasileira de Matemtica

?$JP% <J|<$Pj?$$J?$Jjq
i) ii)
a

Determine todas as solues de xy = yx com x e y racionais positivos.

evidente que x = y

satisfaz a equao.
x = a (a y

Suponha x y, seja ento


1 a a 1

+),

logo:

ay = y y =

e, fazendo a 1 =

p p+q ,a = , temos : q q

q p+qp y = q q+ p p+q p x = q

J que p e q so primos entre si, p + q e q tambm so primos entre si, e portanto devemos garantir que p p + q e p q sejam inteiros. Necessariamente devemos ter p = 1; se no vejamos: suponha que p 2 e que q = sp para algum inteiro positivo s; da s p < p + q < ( s + 1) p (pois (s + 1)p sp > p . sp 1 p) e no teremos que nos obriga fazer p = 1.
p

p + q sendo um inteiro, o

<P|PJ

1 + q q +1 x = q Para x y, teremos: q 1 + q = y q
EUREKA! N4, 1999

47

Sociedade Brasileira de Matemtica

onde q um inteiro maior do que ou igual a 1. fcil verificar que tais x e y so racionais e so solues do problema, para todo q 1 natural. 13) Dado n determine determine o maior k tal que existam conjuntos A1, A2,, Ak contidos em {1, 2, , n} de forma que Ai Aj para todo i j.

?$JP%PJj%<P%$U<Pj
k = Observemos inicialmente que C n

n! [n / 2 ] Cn para todo k com k! (n k )!

0 k n (onde [n/2] o nico inteiro tal que [n/2] n/2 [n/2] + 1). De fato, k +1 Cn nk = 1 pois n > 2k e portanto se k < n / 2, k k +1 Cn
k [n / 2 ] Cn e se n 2k + 1, ou seja n k k + 1. Portanto, se k < n / 2, C n k nk [n / 2 ] = Cn Cn , pois n k < n / 2. Seja A {1, 2,, n} e k > n / 2, C n P = (a1, a2, , an) uma permutao de {1, 2, , n}. Dizemos que P passa por A se existe m n com A = {a1, a2, , am}. Se A tem m elementos existem exatamente m! (n m)! permutaes que passam por A. Como n! n! [n / 2 ] m = Cn Cn = , donde [n / 2]!(n [n / 2])! m!(n m)!

m! (n m)! [n / 2]!(n [n / 2])! para todo m n.

Note agora que se A1, A2, , Ak {1, 2, , n} so tais que, para todo i j Ai Aj , ento, se i j, nenhuma permutao passa por Ai e Aj ao mesmo tempo. Se mi o nmero de elementos do conjunto Ai (1 i k ), podemos concluir que

m !(n m)! n! k.[n / 2]! (n [n / 2])! n! k C [


i i =1

n / 2] . n

[n / 2 ] Por outro lado, h C n subconjuntos de {1, 2, , n} com exatamente [n / 2] elementos e, obviamente, se A e B so dois subconjuntos distintos de [n / 2] elementos de {1, 2,, n} ento A B, de modo que o [n / 2 ] maior k que satisfaz as condies do enunciado C n .

EUREKA! N4, 1999

48

Sociedade Brasileira de Matemtica

15)

Considere uma seqncia de tringulos retngulos AnBnCn no plano cuja hipotenusa seja BnCn, com as seguintes condies: A1B1 = A1C1 = 1; Bn+1 = Bn e An+1 = Cn para todo n ; An+1Cn+1 congruente altura de An em relao a BnCn.

i) ii) iii)

Mostre que qualquer ponto do plano pertence a infinitos tringulos AnBnCn.

<PjJU^J$JP P U
+J U  P

Sejam an, bn e cn os comprimentos dos lados BnCn, AnCn e AnBn do tringulo C . AnBnCn e n o ngulo An B n n suficiente mostrarmos que as hipotenusas dos tringulos crescem infinitamente e que os tringulos do infinitas voltas em torno do ponto B1. Em outras palavras, devemos mostrar que a seqncia (an) ilimitada e que Sn =

k =1

tambm cresce ilimitadamente.

Seja hn a altura relativa hipotenusa BnCn. imediato que anhn = bncn. Alm disso, pelas condies (ii) e (iii), segue que an = cn+1 e bn+1 = hn, donde conclumos que bn+1cn+1 = bncn = ... = b1c1 = 1. Da, anhn = 1, para todo natural n, e como an+12 = bn+12 + cn+12, vem 1 2 2 2 an +1 = a n + 2 , a1 = 2. an Seja pn = an2. Ento, para mostrar que an ilimitada, suficiente mostrarmos que (pn) ilimitada. Note que an+1 > an, para todo natural n, de modo que (pn) crescente. Temos 1 , p1 = 2. p n +1 = p n + pn Vamos mostrar por induo que sempre temos Pn n + 1. De fato, 1 1 n+2 n +1 + = n + 2. P 1 = 2 2, e P n n +1 implica P n+1 = P n+ P n +1 n +1 n Assim, Pn (e portanto an) ilimitada. Vamos agora provar que (Sn)
EUREKA! N4, 1999

49

Sociedade Brasileira de Matemtica

ilimitada. Temos n > sen n =

bn 1 2 = bn bn +1 > bn decresce +1 (pois bn = an cn

quando n cresce). Como 2 2 2 2 2 2 2 2 2 bn +1 = a n +1 a n , 1 + 2 + ... + n > ( a 2 a1 ) + ( a 3 a 2 ) + ... + ( a n +1 a n ) =


2 2 an +1 a1 n + 2 2, e portanto S n =

k =1

k ilimitado.

18)

Seja a maior raiz real da equao x3 3x2 + 1 = 0. Prove que [2004] divisvel por 17. Obs: [y] o nico inteiro tal que [y] y < [y] + 1.

!#"%$&
$'$( ")$+*-,

Considere , e as razes de f(x) = x3 3x2 + 1, a maior delas, e S(n) = n + n + n . (Obs. fcil de se ver que todas as razes so reais. De fato, f (1) < 0 < f (0), e f (1) < 0 < f (3)). Com estas consideraes e as relaes de Girard para f(x)= 0, tem-se que: (I) e S(0) = 0 + 0 + 0 = 3, S(1) = 1 + 1 + 1 = 3 S(2) = 2 + 2 + 2 = (1 + 1 + 1)2 2 ( + + ) = 9 0 = 9 f(x) = x2 . ( x 3 ) +1 > 0 para todo x 3. Como f contnua e f( 2,87 ) 0,07 < 0, segue-se que 2,87 < < 3. Por outro lado, de (I) tem-se 2 + 2 + 2 = 9 . Assim, 2 + 2 < 9 (2,87)2 < 1. Sendo f(0) = 1, conclui-se que e so no nulas . Logo, 0 < 2 + 2 < 1 e conseqentemente , 0 < 2 < 1 e 0 < 2 < 1. (III) P(x) = xn 3 . f(x) = x n 3x n 1 + x n 3 ( n 3) um polinmio, tal que:

(II)

EUREKA! N4, 1999

50

Sociedade Brasileira de Matemtica

P ( ) = n 3 n 1 + n 3= 0 n n 1 + n 3 = 0 P ( ) = 3 n n 1 + n 3 = 0 P ( ) = 3 Adicionando-se membro a membro , obtm-se para n 3, a seguinte relao de recorrncia:


S( n ) 3. S ( n 1 ) + S ( n 3 ) = 0 , ou, melhor ainda

S(n) = 3. S (n 1) S (n 3) (IV)

(n 3)

Sendo S(0), S(1) e S(2) inteiros por (I), podemos concluir, atravs da relao de recorrncia acima e induo sobre n, que S(n) ser um inteiro para qualquer natural n. Assim, S(2004) = 2004 + 2004 + 2004 um inteiro. Como 0 < 2 < 1 e 0 < 2 < 1 ....... por (II), segue-se que 0 < 2004 + 2004 = ( 2) 1002 + (2 )1002 < 2 + 2 < 1. Logo, S(2004) 1 < 2004 < S(2004), ou seja,
2004 = S(2004) 1.

(V)

A relao de recorrncia obtida em (III) implica em particular que S (n) = 3S (n 1) S (n 3) (mod 17) para todo n 3, o que permite construir a tabela seguinte de S(n) (mod 17): S(0) 3 (mod 17), S(1) 3 (mod 17), S(2) 9 (mod 17), S(3) 7 (mod 17), S(4) 1 (mod 17), S(5) 11(mod 17), S(6) 9 (mod 17), S(7) 9 (mod 17), S(8) 16 (mod 17), S(9) 5 (mod 17), S(10) 6 (mod 17), S(11) 2 (mod 17), S(12) 1 (mod 17), S(13) 14 (mod 17), S(14) 6 (mod 17), S(15) 0 (mod 17), S(16) 3 (mod 17), S(17) 3 (mod 17), S(18) 9 (mod 17). Note que S(16) S(0) (mod 17), S(17) S(1) (mod 17) e S(18) S(2) (mod 17). Isso permite mostrar que S(n +16) S(n) (mod 17) para todo n . . De fato, por induo, S(n + 16) = 3S(n + 15) S (n + 13) 3S(n 1) S(n 3) (mod 17) = S(n)( mod 17), se n 3 (usamos como base de induo os casos

(VI)

EUREKA! N4, 1999

51

Sociedade Brasileira de Matemtica

n = 0, n = 1 e n = 2). Como conseqncia, concluimos que S(n) S(n + 16p) (mod 17), para todo p / . (VII) Como 2004 = 4 + 16.p , com p = 125, temos de (IV), (V) e (VI): S(2004) S(4) 1 (mod 17). Portanto, 2004 = S(2004) 1 divisvel por 17.

Nota: A demonstrao acima foi baseada na resoluo de um problema de enunciado similar a este, proposto pela Frana e no utilizado na IMO de 1988. 19) a) Determine o nmero mximo de regies em que n retas podem dividir o plano. b) Determine o nmero mximo de regies em que n planos podem dividir o espao.

 0)1234*")526&
71819(&
2:,

a) i) ii)

Observe inicialmente os casos abaixo:

Uma reta divide o plano em duas regies: 1 + 1. Uma segunda reta dividida pela anterior no mximo em duas partes e mais duas regies so acrescentadas, ou seja: com 2 retas temos: ( 1 + 1 + 2) regies (no mximo.) iii) Uma terceira reta dividida pelas duas retas anteriores no mximo em trs partes e acrescentando ento mais trs regies, ou seja: com 3 retas temos: (1 +1 +2 +3) regies (no mximo.) iv) Suponha agora que tenhamos n retas; a n-sima reta dividida pelas (n 1) outras retas no mximo em n partes e evidentemente acrescentando n regies, o que nos dar: (1 + 1 + 2 + 3 ++ n) n2 + n regies ou + 1 regies. 2 Obs: Se as retas esto em posio geral todas as desigualdades acima so igualdades. b) Observe que:

EUREKA! N4, 1999

52

Sociedade Brasileira de Matemtica

i)

ii) iii)

Quando temos dois planos, o segundo plano intersecta o primeiro plano no mximo atravs de uma reta e o segundo plano dividido em duas partes. Um terceiro plano intersecta os planos anteriores em no mximo duas retas e o terceiro plano dividido em 4 partes. Um quarto plano intersecta os planos anteriores em no mximo 3 retas e o quarto plano dividido em 7 partes.

Em geral, o k-simo plano intersecta os anteriores em no mximo k 1 retas, (k 1)2 + (k 1) + 1 = k 2 k + 2 regies que o dividem em no mximo 2 2 (pelo tem a), ou seja, ao ser acrescentado o k-simo plano so criadas no k2 k + 2 mximo novas regies do espao. 2 Como um plano divide o espao em duas regies temos no mximo n k2 k + 2 regies em que k planos dividem o espao. 2+ 2 k =2 n(n + 1)(2n + 1) Sabendo que 12 + 2 2 + 3 2 + ... + n 2 = , temos um total de 6 n 3 + 5n + 6 regies (no mximo). 6 Obs: Se os planos esto em posio geral todas as desigualdades acima so igualdades.
;=<>-? @>-ACBD7<EGFEHFI6BJ>-KC<LBEME<NOAQPRFESKT<EMHI6<UNFDBJEWV:XZY[V]\^FWV]_`KQBE`FKT@PRFE B>-? FI6@<I6FEa bdcOegf)h ikjmlnobpcOegqe)nr3e)nos=eutdlvc3r6lw x)yz {$4(|$&


Maria convidou 9 garotos e 8 garotas para sua festa de aniversrio. Ela preparou camisetas com os nmeros de 1 a 18 e ficou com a de nmero 1 e distribuiu as demais para seus convidados. Em determinado momento, em que todos estavam danando, a soma dos nmeros de cada casal era um quadrado perfeito. Quais pares estavam danando?
-,

A maior soma possvel dos nmeros de um casal 18 + 17, que menor que 62. Assim, os valores das somas dos nmeros de cada casal devem valer 4, 9,
EUREKA! N4, 1999

53

Sociedade Brasileira de Matemtica

16 ou 25, de modo que os pares de 18, 17 e 16 devem ser 7, 8 e 9, respectivamente. Portanto o par de 2 deve ser 14 (pois no pode ser o prprio 2 e o 7 j par do 18), o do 11 deve ser 5 (pois 14 j par de 2), o de 4 deve deve ser 12 (pois 5 j par de 11), o de 13 deve ser 3 (pois 12 j par de 4), o de 1 deve ser 15 (pois 3 j par de 13 e 8 j par de 17) e o de 10 deve ser 6 (pois 15 j par de 1). Assim, os pares so 18 e 7, 17 e 8, 16 e 9, 15 e 1, 14 e 2, 13 e 3, 12 e 4, 11 e 5, 10 e 6.
 &
|, "%$|~} |6|)#+7|6*~=|34$|~~}d z |~|S*(!|S*&
|| x '6
**6|*$ z |6|*+* (36&
|J**&
 6!7| x)yz {$&
676

AD a bissetriz interna do ngulo A do tringulo ABC, com D sobre o lado BC. As bissetrizes dos ngulos ADB e ACB concorrem em E, com E sobre o lado AB. Determine a medida do ngulo BAC. O problema equivalente ao problema 5 da pgina 3 da resolvido na pgina 6 da mesma.
x)yz " z |$|

9;<9?

No.3,

So dadas 13 moedas, das quais 12 tm o mesmo peso. No se sabe se a dcima terceira moeda mais leve ou mais pesada que as demais. Mostre que possvel determinar a moeda diferente empregando trs pesagens em uma balana de braos. Isto ainda seria possvel com 14 moedas?
-,

Das 13 moedas selecionaremos dois grupos de 4 moedas A1, A2, A3, A4 e B1, B2, B3, B4 e as pesamos. Sobram 5 moedas, C1, C2, C3, C4 e C5. Temos duas possibilidades: i) A balana fica equilibrada. Neste caso a moeda diferente est entre as 5 restantes. Pesamos agora A1, A2, A3 e C1, C2, C3 (A1, A2 e A3 so padro). Temos mais duas possibilidades Equilbrio. A moeda diferente C4 ou C5. Pesamos A1 e C4. Se der diferente a moeda diferente C4, e se houver equilbrio C5. Desequilbrio. Vamos supor sem perda de generalidade que o grupo C1C2C3 mais pesado que A1A2A3. Nesse caso a moeda diferente C1, C2 ou C3 e mais pesada que as outras. Pesamos C1 e C2. Se houver desequilbrio a mais pesada a diferente. Se houver equilbrio a C3.

i.1) i.2)

EUREKA! N4, 1999

54

Sociedade Brasileira de Matemtica

ii)

ii.1)

ii.2)

ii.3)

Desequilbrio. Vamos supor sem perda de generalidade que o grupo A1A2A3A4 mais pesado que B1B2B3B4. Pesamos agora A1A2B1 e A3B2C1. Temos trs possibilidades: Equilbrio. Nesse caso a moeda diferente A4, B3 ou B4. Se for A4 mais pesada e se for B3 ou B4 mais leve. Pesamos B3 e B4. Se houver equilbrio a diferente A4. Se no a mais leve das duas a diferente. O grupo A1A2B1 mais pesado. Nesse caso a moeda diferente A1, A2 ou B2. Pesamos A1 e A2. Se houver equilbrio a diferente B2, se no a mais pesada das duas. O grupo A1A2B1 mais leve. Nesse caso a moeda diferente B1 ou A3. Pesamos B1 e C1. Se houver equilbrio a moeda diferente A3, se no B1.

Se tivermos 14 moedas no possvel determinar sempre a moeda diferente. Se na primeira pesagem pesamos dois grupos de 5 ou mais moedas e no houver equilbrio, a moeda diferente pode ser qualquer uma das pelo menos 10 envolvidas na pesagem. Como cada pesagem tem apenas 3 resultados possveis, as duas ltimas pesagens do no total no mximo 9 resultados diferentes, que no permitem distinguir todas as (pelo menos 10) possibilidades de moeda diferente. Se pesarmos dois grupos de 4 ou menos moedas na primeira pesagem e houver equilbrio sobram pelo menos 6 moedas para anlise. Se na segunda pesagem usarmos 4 ou mais dessas moedas de situao desconhecida e no houver equilbrio, qualquer uma delas pode ser a diferente e a ltima pesagem (que s tem 3 resultados possveis) no pode determin-la com segurana. Se na pesagem usamos 3 ou menos das moedas de situao desconhecida sobram pelo menos 3 em situao desconhecida. Se na terceira pesagem usamos duas delas ou todas as 3 e houver desequilibrio qualquer uma dessas moedas pode ser a diferente, e no conclumos nossa tarefa. Se usamos uma ou nemhuma e houver equilbrio sobram pelo menos duas de situao desconhecida, e em qualquer caso no sempre possvel determinar a moeda diferente. Nota: O problema 10, proposto na pgina 59 da No. 2 generaliza este problema. Tente resolv-lo agora, adaptando para a situao geral os argumentos desta soluo!

9;<9?

EUREKA! N4, 1999

55

-5U U-U
+O TT% T T+O ] T]Q%T+T +T] %]TT] TQ-TT+T T TT+T^J %TT .

Sociedade Brasileira de Matemtica

20) 21)

Diga se existe uma funo polinomial de o intervalo (0,+) = {x x > 0}.

em cuja imagem seja a + b = c.


3

a) Encontre todas as solues inteiras da equao b) Encontre todas as solues inteiras da equao

a + 3 b = 3 c.

22)

Sejam , , , os ngulos de um quadriltero, nessa ordem. Prove que esse quadriltero inscritvel se, e somente se, a relao + + + = 2 ocorre. Seja ABC um tringulo qualquer de ortocentro H e sejam ha, hb, hc os comprimentos das alturas relativas a A, B, C respectivamente. Prove ____ ____ ____ 1 que ha . AH + hb . BH + hc CH = (a 2 + b 2 + c 2 ). 2 Na loteria de Truchilndia, cada bilhete tem um nmero de trs algarismos que usa somente os algarismos 1, 2, 3, 4 ( permitido repetir os dgitos). Um bilhete ganhador se coincide em pelo menos duas posies com o nmero sorteado. Um apostador quer compar vrios bilhetes, de maneira que um deles ganhe com certeza, mas gastando o mnimo possvel. Determinar quantos bilhetes deve comprar e quais bilhetes deve comprar. Obs. Se o bilhete sorteado for o 423 ento 123 um bilhete ganhador, mas 243 no .

23)

24)

25)

Durante o ano de 1998, uma pequena livraria, que abria nos sete dias da semana, vendeu no mnimo um livro por dia e um total de 600 livros no ano todo. Diga, justificando, se existiu, obrigatriamente, um perodo de dias consecutivos onde foram vendidos exatamente 129 livros.
Os problemas 21, 22, 23 e 24 foram propostos na 1a. lista de preparao para a X Olimpada de Matemtica do Cone Sul. O problema 25 foi proposto na IX Olimpada de Matemtica do Rio Grande do Norte.

Nota:

EUREKA! N4, 1999

56

Sociedade Brasileira de Matemtica

55{

A soluo do problema 2 da 9a. Olimpada de Matemtica do Cone Sul, (cujo enunciado est na pgina 22 da No.2) est errada. Na verdade publicamos a soluo de outro problema do banco, cujo enunciado era:

9;<9?

Sejam H o ortocentro do tringulo ABC , no retngulo, e M o ponto mdio do lado BC. A circunferncia circunscrita em P. Mostre que P, H, M so colineares. A soluo correta do problema 2 da 9a. Olimpada de Matemtica do Cone Sul como segue: : Sejam H o ortocentro (interseo das alturas) do tringulo acutngulo ABC e M o ponto mdio do lado BC. Seja X o ponto em que a reta HM intersecta o arco BC (que no contm A) da circunsferncia circunscrita a ABC. Seja Y o ponto de interseo da reta BH com a circunsferncia, distinto de B. Demonstre que XY = BC. Soluo:
+-+

Seja X' o simtrico de H em relao ao ponto M. Vamos mostrar que X X'. O quadriltero HBX'C um paralelogramo, pois os pontos mdios de suas diagonais coincidem. Ento BX'C = BHC = 180 BAC. Segue que X' deve pertencer ao arco BC que no contm A, donde X X'. Observe agora que o quadriltero BYCX um trapzio inscritvel (pois BY X'C = XC ), donde BC = XY.

EUREKA! N4, 1999

57

 -C[ 5

Sociedade Brasileira de Matemtica

+T ]%TTQ T%] %TO TQTT]]T- T ] +%+ +

Eureka! Lembrei-me de Arquimedes quando tive a luz para a idia que mexeu comigo e um punhado de alunos que at ento estavam inertes face necessidade de aprender Matemtica. Descobri! Diria Arquimedes, radiante de felicidade, no bom portugus. E tal foi a emoo que vivi quando atinei para a soluo que revolucionaria o ensino da Matemtica numa to carente escola pblica. Tal luz, que me veio mente, iluminou alguns alunos que poderiam hoje estar mergulhados, como muitos, nas trevas do descaso e do abandono do ensino pblico. Descobri! Que o saber dos nmeros independe das classes sociais. democrtico! E nasce no espirito daqueles a quem seja dada a fagulha e uma palavra de confiana, de incentivo. A funo da Matemtica profunda, humanstica, pois o bem estar da coletividade o fim de tudo. Cabe, Matemtica a criao de espritos disciplinados, mentes sadias e aprimoradas. E esse o nosso escopo: estimulando, propiciando e melhorando o ensino dessa matria nas Escolas Brasileiras.

Prof. Paulo Araripe Fortaleza-CE

EUREKA! N4, 1999

58

Sociedade Brasileira de Matemtica

[O^ [55[^

Se voc fantico por Matemtica e deseja receber na sua casa a revista g) , faa o seu pedido escrevendo para: Secretaria da Olimpada Brasileira de Matemtica, Estrada Dona Castorina, 110 Jardim Botnico - Rio de Janeiro, RJ - CEP: 22460-320. O custo de cada exemplar avulso ou atrasado de R$4,00. Voc pode fazer uma assinatura anual o que dar direito a receber as publicaes do referido ano (mnimo 3 exemplares) por um valor promocional de R$10,00. Para isso, faa um depsito no Banco do Brasil - Agncia 0598-3 - Conta N52208-2 em nome do professor Eduardo Wagner. Envie-nos a fotocopia do depsito e faa referncia aos nmeros desejados. No esquea de colocar seu nome e endereo completos e ns remeteremos a(s) revista(s) pelo correio. Pedidos podem ser feitos tambm por e-mail e comprovantes de depsito podero ser enviados pelo fax. Se tiver qualquer dvida entre em contato conosco. Telefone: 021-5295077 / Fax: 021-5295023 e-mail: obm@impa.br Home-Page: http//www.obm.org.br/

kC   Z Qo dC!Q CT ^


:
T Q




 !"#$%&'(#)
*,+ '+ #-.%/01'232'+ 4# 57698 :<;>=9?>@?BAC.?D8E6 FG8 C2?H@FBIB?>J2F:<K>J8 L? M NPOQ.R2STSUWVX
OQYQXTVX,OXZ>VX,[

EUREKA! N4, 1999

59

]\

Sociedade Brasileira de Matemtica

-^`_

<

a b c#{} '0d"fe%"ge)G} "){9c

8 de maio (sbado)
h,i b ,c#{} '0d"fe)"gejclkfc x G0m)

17 a 24 de maio Argentina
n,h b ,c#{} '0d"fe)"Lfo%"){{9o)"ge%M}"fp}rqfp{9k)"

Primeira Fase 12 de junho (sbado) Segunda Fase 28 de agosto (sbado) Terceira Fase 23 de outubro (sbado) e 24 de outubro (domingo)
s,i b ,c#{} '0d"fe)"L{ x p0o x "fk){9c x ")te)G} "jp}Tqfp{9k)"

10 a 22 de julho Bucharest, Romnia.


h,s b ,c#{4}'0d"fe)"L{)0ofc#"%} 0o){9k)" x "ge)G} "fp}rqjp{9k)"

setembro Cuba.
uwvxy z{}|~{0 &
T9 <,$ G, $$ f.&> .
9 .4
%2 $&. &&, .r>
$7} $72>,E 7
> 2 2T$&<, H.2>&,T
, 2 2$ 2 , .H0&, $$9  $
<
    !"#%$'&)(+* ,.-+/ 0123.457698;:=<+>?)?.8;<5@-<9> ?)?)A)A ? BCEDGFHJILKMFNFII BO@PBQSRUT VQ

EUREKA! N4, 1999

60

Sociedade Brasileira de Matemtica

LXWZY@[[]\^YXWZ[[_WZ[r\L L]\LO
Amarisio da Silva Arajo Alberto Hassen Raad Antnio C. Rodrigues Monteiro Angela Camargo Ariosto de Oliveira Lima Benedito T. Vasconcelos Freire Claudio Arconcher Egnilson Miranda de Moura lio Mega Florncio F. Guimares F. Francisco Dutenhefner Gisele de A. Prateado G. Ivanilde H. Fernandes Saad Joo B. de Melo Neto Joo F. Melo Libonati Jorge Ferreira Jos Carlos Pinto Leivas Jos Luis Rosas Pinho Jos Paulo Carneiro Jos Vieira Alves Leonardo Matteo D'orio Licio Hernandes Bezerra Luzinalva M. de Amorim Marco Polo Marcondes Cavalcante Frana Mario Jorge Dias Carneiro Pablo Rodrigo Ganassim Paulo H. Cruz Neiva de L. Jr. Reinaldo Gen Ichiro Arakaki Ricardo Amorim Roberto Vizeu Barros Sergio Claudio Ramos Seme Gebara Neto Tadeu Ferreira Gomes Valdenberg Arajo da Silva Wagner Pereira Lopes Waldemar M. Canalli (UFV) Viosa - MG (UFJF) Juiz de Fora - MG (UFPE) Recife - PE (Centro de Educao de Adultos CEA) Blumenau - SC (UFPI) Parnaba - PI (UFRN) Natal - RN (Col. Leonardo da Vinci) Jundia - SP (Col. Agrcola do Bom Jesus) Bom Jesus - PI (Col. ETAPA) So Paulo - SP (UFES) Vitria - ES (UFMG) Belo Horizonte - MG (UFGO) Goinia - GO (U. Catlica Dom Bosco) Campo Grande - MS (UFPI) Teresina - PI (Grupo Educ. IDEAL) Belm - PA (UEM) Maring - PR (URG) Rio Grande - RS (UFSC) Florianpolis - SC (USU) Rio de Janeiro - RJ (UFPB) Campina Grande - PB (Parque de Material Aeronutico de Belm) Belm - PA (UFSC) Florianpolis - SC (UFBA) Salvador - BA (Colgio Singular) Santo Andr - SP (UF Cear) Fortaleza - CE (UFMG) Belo Horizonte - MG (L. Albert Einstein) Piracicaba - SP (Esc. Tec.Everardo Passos) SJ dos Campos - SP (INPE) SJ dos Campos - SP (Centro Educ. Logos) Nova Iguau - RJ (Colgio ACAE) Volta Redonda - RJ (IM-UFRGS) Porto Alegre - RS (UFMG) Belo Horizonte - MG (U. do Estado da Bahia) Juazeiro - BA (U. Federal de Sergipe) So Cristovo - SE (Esc. Tec. Fed. de Gois) Jata - GO (P.M. S. Joo de Meriti) S. Joo de Meriti - RJ

EUREKA! N4, 1999

61

Sociedade Brasileira de Matemtica

EUREKA! N4, 1999

62

CONTEDO
AOS LEITORES 5a. OLIMPADA DE MAIO Problemas 5a. OLIMPADA DE MAIO Resultados 10a. OLIMPADA DE MATEMTICA DO CONE SUL Problemas e Solues 40a. OLIMPADA INTERNACIONAL DE MATEMTICA Problemas e Resultados 2 3 5 7 15

ARTIGOS
ADEDANHA OU "DE COMO OS DEUSES MATEMTICOS TROUXERAM A PAZ AO MUNDO" Pablo Emanuel QUADRILTEROS E TRINGULOS Marcelo Mendes O PRINCPIO DAS GAVETAS Paulo Cezar Pinto Carvalho DESIGUALDADES ELEMENTARES Antonio Caminha Muniz Neto 40a. OLIMPADA INTERNACIONAL E 14a. OLIMPADA IBEROAMERICANA DE MATEMTICA Primeiro teste de Seleo SOLUES DE PROBLEMAS PROPOSTOS PROBLEMAS PROPOSTOS COMO ASSINAR A EUREKA! AGENDA OLMPICA COORDENADORES REGIONAIS 17 23 27 34 50

51 57 59 60 61

Sociedade Brasileira de Matemtica

AOS LEITORES
Realizamos a primeira fase da XXI Olimpada Brasileira de Matemtica em mais de 3.000 colgios do nosso pas. Atravs dos relatrios enviados pelas escolas aos Coordenadores Regionais, estabelecemos as notas de corte para a promoo dos alunos segunda fase que se realizar em agosto. Notas de corte para promoo a segunda fase: Primeiro nvel (5a. e 6a. sries) : Segundo nvel (7a. e 8a. sries) : Terceiro nvel (Ensino mdio) : 09 acertos ou mais. 10 acertos ou mais. 12 acertos ou mais.

A prova da segunda fase ser discursiva e ainda ser aplicada nos colgios. Os enunciados, as solues e os criterios de pontuao sero enviados a todas as escolas participantes. A Comisso de Olimpadas agradece a participao de alunos e professores na primeira fase e deseja sucesso a todos nas fases seguintes. Agradecemos s pessoas que colaboraram com este nmero propondo problemas e enviando solues de problemas propostos. Aproveitamos para continuar estimulando nossos leitores a contribuir para nossa revista com artigos, problemas e solues. Lembramos da existncia da lista de discusso de problemas de matemtica da OBM, cujo endereo eletrnico mudou para: obm-l@mat.puc-rio.br . Para maiores informaes escreva para obm@impa.br ou para o Prof. Nicolau Saldanha, administrador desta lista em nicolau@mat.pucrio.br .

Comit Editorial.

EUREKA! N5, 1999

Sociedade Brasileira de Matemtica

5a. OLIMPADA DE MAIO


Primeiro Nvel
PROBLEMA 1

So escolhidos 2 nmeros inteiros entre 1 e 100 inclusive, tais que a diferena 7 e o produto mltiplo de 5. De quantas maneiras pode ser feita a escolha?
PROBLEMA 2

Num paralelogramo ABCD, BD a diagonal maior. Ao fazer coincidir B com D, mediante uma dobra, se forma um pentgono regular. Calcular as medidas dos ngulos que a diagonal BD forma com cada um dos lados do paralelogramo.
PROBLEMA 3

Em cada um dos 10 degraus de uma escada existe uma r. Cada r pode, de um pulo, colocar-se em outro degrau, mas quando uma r faz isso, ao mesmo tempo, uma outra r pula a mesma quantidade de degraus em sentido contrrio: uma sobe e outra desce. Conseguiro as rs colocar-se todas juntas num mesmo degrau?
PROBLEMA 4
2 1

Dez cartes quadrados de 3 centmetros de lado so cortados por uma lnha, como mostra a figura. Depois dos cortes tem-se 20 peas: 10 tringulos e 10 trapzios. Forme um quadrado que utilize as 20 peas sem superposies nem espaos.

PROBLEMA 5

Ana, Beatriz, Carlos, Diego e Emilia jogam um torneio de xadrez. Cada jogador enfrenta uma vez s cada um dos outros quatro jogadores. Cada jogador consegue 2 pontos se ganha a partida, 1 ponto se empata e 0 pontos se perde a partida. Ao finalizar o torneio, as pontuaes dos 5 jogadores so todas diferentes. Encontre o mximo nmero de empates que pode ter tido o torneio e justifique por que no pode ter havido um nmero maior de empates.

EUREKA! N5, 1999

Sociedade Brasileira de Matemtica

SEGUNDO NVEL
PROBLEMA 1

Um nmero natural de trs algarismos chamado de tricbico se igual soma dos cubos dos seus dgitos. Encontre todos os pares de nmeros consecutivos tais que ambos sejam tricbicos.
PROBLEMA 2

A figura representa a quarta parte de um crculo de raio 1. No arco AB, se consideram pontos P e Q de forma tal que a reta PQ seja paralela reta AB. Sejam X e Y os pontos de interseo da reta PQ com as retas OA e OB respectivamente. Calcular PX + PY .
PROBLEMA 3
_____2 ____ 2

A primeira fileira da tabela ao lado preenchida com os nmeros de 1 a 10, em ordem crescente. A segunda fileira preenchida com os nmeros de 1 a 10, em qualquer ordem. Em cada casa da terceira fileira se escreve a soma dos dois nmeros escritos nas casas acima. Existe alguma maneira de preencher a segunda fileira de modo que os algarismos das unidades dos nmeros da terceira fileira sejam todos distintos?
PROBLEMA 4

Seja ABC um tringulo equiltero. M o ponto mdio do segmento AB e N o ponto mdio do segmento BC. Seja P o ponto exterior a ABC tal que o tringulo ACP issceles e retngulo em P. PM e AN cortam-se em I. Prove que CI a bissetriz do ngulo MCA.
PROBLEMA 5

So dados 12 pontos que so os vrtices de um polgono regular de 12 lados. Rafael deve traar segmentos que tenham seus dois extremos em dois dos pontos desenhados. permitido que cada ponto seja extremo de mais de um segmento e que os segmentos se cruzem, mas proibido traar trs segmentos que sejam os trs lados de um tringulo em que cada vrtice um dos 12 pontos iniciais. Encontre o nmero mximo de segmentos que pode traar Rafael e justifique por que no possivel traar um nmero maior de segmentos.

EUREKA! N5, 1999

Sociedade Brasileira de Matemtica

5a. OLIMPADA DE MAIO


Resultado Brasileiro Primeiro Nvel
Fbio Dias Moreira Andr de Carvalho Amaro Diego Costa de Almeida Daniel Haanwinckel Junqueira Ral M. Alexandrino Nogueira Bruna Griguol Felipe Oliveira de Sousa Guilherme Finkelfarb Lichand Leonardo Luis Desideri Freitas Zilma K. Barbosa Bezerra Thiago Augusto Caldas Bello Mateus Gomes Filgueiras Ouro Prata Prata Bronze Bronze Bronze Bronze Meno Honrosa Meno Honrosa Meno Honrosa Meno Honrosa Meno Honrosa Rio de Janeiro - RJ So Paulo - SP Fortaleza - CE Salvador - BA Fortaleza - CE Cafelndia - PR Fortaleza - CE So Paulo - SP Vitria - ES Fortaleza - CE Salvador - BA Fortaleza - CE

Segundo Nvel
Joo Alfredo Castellani F. Freire Arthur Duarte Nehmi Davi M. Alexandrino Nogueira Luiz Brizeno Firmeza Neto Luciana Andretta do Nascimento Thiago Barros Rodrigues Costa Daniel Pinheiro Sobreira Maurcio Masso Soares Matsumoto Fabio S. Toniolo Einstein do Nascimento Jr. Hugo Pinto Iwata Rodrigo Roque Dias Ouro Prata Prata Bronze Bronze Bronze Bronze Meno Honrosa Meno Honrosa Meno Honrosa Meno Honrosa Meno Honrosa Salvador - BA So Paulo - SP Fortaleza - CE Fortaleza - CE Cafelndia - PR Fortaleza - CE Fortaleza - CE So Paulo - SP So Paulo - SP Fortaleza - CE S J do Rio Preto - SP So Paulo - SP

O Brasil teve uma excelente participao na 5a. Olimpada de Maio na qual participaram 13 paises, sendo o pas com maior pontuao nos dois nveis em que realizada esta competio.

EUREKA! N5, 1999

Sociedade Brasileira de Matemtica

5a. OLIMPADA DE MAIO


Clasificao por paises

PRIMEIRO NVEL
Pas Brasil Argentina Espanha Mxico Peru Colombia Cuba Costa Rica Uruguai Chile Bolvia Venezuela Equador Pontuao 286 272 206 168 157 154 152 143 142 121 118 95 55

SEGUNDO NVEL
Pas Brasil Peru Argentina Cuba Colombia Mxico Espanha Uruguai Chile Bolvia Costa Rica Venezuela Equador Pontuao 405 341 264 219 188 175 136 125 123 108 57 57 22

!!!!

Voc sabia
6972593

Que o maior nmero primo

1, que tem 2.098.960 dgitos e conhecido 2 foi descoberto em 1/6/99 por Nayan Hafratwala, um participante do GIMPS, um projeto cooperativo para procurar primos de mersenne? Consulte na Internet a pgina Http://www.mersenne.org/prime.htm

EUREKA! N5, 1999

Sociedade Brasileira de Matemtica

10a. OLIMPADA DE MATEMTICA DO CONE SUL


Problemas e Solues A 10a. Olimpada de Matemtica do Cone Sul foi realizada na cidade de Crdoba, Argentina no perodo de 17 a 24 de maio de 1999. Dela participaram alunos de at 15 anos dos seguintes pases: Argentina, Brasil, Bolvia, Chile, Paraguai, Peru e Uruguai. A equipe brasileira foi selecionada atravs de provas realizadas em maro e maio deste ano e foi liderada pelos professores Florncio Ferreira Guimares Filho da UFES, e Antnio Caminha Muniz Neto, da UFCE. O Resultado da Equipe Brasileira BRA 1 BRA 2 BRA 3 BRA 4 Daniel Massaki Yamamoto Daniel Pinheiro Sobreira Fabrcio Siqueira Benevides Humberto Silva Naves BRONZE BRONZE PRATA PRATA

Primeiro Dia Durao da prova: 4 horas


PROBLEMA 1

Achar o menor inteiro positivo n tal que as 73 fraes

19 20 21 91 , , ,........., n + 21 n + 22 n + 23 n + 93
sejam todas irredutveis.
SOLUO

a a irredutvel se e s se irredutvel ( se a e b tem um fator b ba comum, ento a e b a tm um fator comum, e reciprocamente). O problema se transforma em achar o menor valor de n tais que as fraes 19 20 91 ,..., , n+2 n+2 n+2 sejam todas irredutveis.

A frao

EUREKA! N5, 1999

Sociedade Brasileira de Matemtica

Se n + 2 primo, maior que 91, todas as fraes so irredutveis. Assim, um valor possvel de n 95. Verifiquemos que o menor possvel. 20 Se n + 2 < 97 e n + 2 par (n par) h fraes redutveis, por exemplo . n+2 Se 19 n + 2 91, obviamente h uma frao redutvel. Se n + 2 < 19, ento n + 2 tem um mltiplo entre 19 e 91, e portanto, h uma frao redutvel. 31 redutvel. Se n + 2 = 93 = 3 . 31, ento n+2 19 Se n + 2 = 95 = 5 . 19, ento redutvel. n+2 Ento, o valor mnimo de n + 2 97, que corresponde a n = 95.
PROBLEMA 2

Seja ABC um tringulo retngulo em A. Construir o ponto P sobre a hipotenusa BC, tal que se Q for o p da perpendicular traada desde P ao cateto AC, ento a rea do quadrado de lado PQ igual rea do retngulo de lados iguais a PB e PC. Mostrar os passos da construo.
SOLUO
B H P

P BC satisfaz as condies do enunciado se e s se PQ = PB PC (*). Como


PQC BAC , vem que
PC PB
____ ____

____

____ 2

____ ____

PQ PC
____

____

. Da, PQ = PC ____ , e segue de (*) que BC BC


____
____

AB

___

____

____

AB

___

BC

____ 2 ____

. Sendo H o p da altura relativa a BC , temos BH BC = AB 2 , donde

____ ____

____

AB 2

EUREKA! N5, 1999

Sociedade Brasileira de Matemtica


____ ____ ____

PC PB

BC

BH

____

Temos ento a seguinte construo:


B' B H'

P H

A r

i) ii) iii) iv) v)

Trae, por C, a reta r tal que r r AC . Marque em r o ponto B' tal que B' C = BC e B, B' estejam num mesmo semi-plano dos determinados por BC . Trace a altura AH relativa hipotenusa BC. Marque H' AB tal que BH' = BH. BC B'H' = {P}

____ ____

PROBLEMA 3

H 1999 bolinhas em uma reta; algumas so vermelhas e as demais azuis (poderiam ser todas vermelhas ou todas azuis). Debaixo de cada bolinha escrevemos o nmero igual soma da quantidade de bolinhas vermelhas direita dela mais a quantidade de bolinhas azuis esquerda dela. Se, na sequncia de nmeros assim obtida, houver exatamente trs nmeros que aparecem uma quantidade mpar de vezes, quais podem ser estes trs nmeros?
SOLUO

Se as 1999 bolinhas so de uma mesma cor, a sucesso de nmeros crescente ou decrescente. Cada nmero aparece uma vez s e h 1999 (portanto, no h
EUREKA! N5, 1999

Sociedade Brasileira de Matemtica

exatamente 3 nmeros que se repetem um nmero mpar de vezes (1 mpar). Logo, h bolinhas das duas cores. Dada uma distribuio das bolinhas que tem em certa posio uma bolinha azul A e na posio seguinte uma bolinha vermelha R, se h a bolinhas azuis esquerda de A e r bolinhas vermelhas sua direita, ento h a + 1 bolinhas azuis esquerda de R e r 1 bolinhas vermelhas sua direita. O nmero escrito embaixo de A n = a + r e o nmero escrito embaixo de R a + 1 + r 1 = n. Se trocamos de lugar A e R, e no mexemos em nenhuma outra bolinha, na nova distribuio h a bolinhas azuis esquerda de R e r 1 bolinhas vermelhas sua direita, enquanto que esquerda de A h a bolinhas azuis e, sua direita, r 1 bolinhas vermelhas. Os nmeros escritos embaixo de R e A so a + r 1= n 1 e a + r 1 = n 1. Os nmeros escritos embaixo das outras bolinhas no mudam. Ento, depois da troca, o nmero n se repete duas vezes menos e o nmero n 1 se repete duas vezes mais. Os nmeros que se repetem uma quantidade mpar de vezes sero os mesmos em ambas configuraes. Portanto, basta estudar a configurao na qual todas as bolinhas vermelhas so consecutivas, a partir da primeira, e todas as azuis so consecutivas, a partir da ltima vermelha. Sejam , , as quantidades de bolinhas vermelhas e azuis, respectivamente; ento + = 1999. Embaixo da primeira bolinha ( vermelha) est o nmero 1, na seguinte, 2, depois 3, e assim por diante, at ter 0 na ltima bolinha vermelha (na posio ). Ento, embaixo da primeira bolinha azul h 0, na segunda 1 e assim por diante, at a ltima, que tem 1 embaixo. Se < , os nmeros 0, 1, 2, , 1 aparecem duas vezes (quantidade par) e os nmeros , + 1, + 2, , 1 aparecem uma vez (quantidade mpar). Se h exatamente 3 nmeros que aparecem uma quantidade mpar de vezes, estes so , + 1 e + 2 = 1. Portanto, + = 2 + 3, donde = 998, e os trs nmeros que se repetem uma quantidade mpar de vezes so 998, 999 e 1000. Se > , os trs nmeros que aparecem uma quantidade mpar de vezes so , +1 e + 2 = 1, donde + = 2 + 3 e os tres nmeros so, novamente, 998, 999 e 1000.

Segundo Dia Durao da prova 4 horas.


PROBLEMA 4

Seja A um nmero de seis algarismos, trs dos quais esto coloridos e so iguais a 1, 2 e 4.
EUREKA! N5, 1999

10

Sociedade Brasileira de Matemtica

Demonstrar que sempre possvel obter um nmero que mltiplo de 7, efetuando uma s das seguintes operaes: ou suprimir os trs algarismos coloridos, ou escrever todos os algarismos de A em alguma ordem.
SOLUO

Provaremos o seguinte resultado mais geral: Seja A um nmero de mais de 3 algarismos, trs dos quais so 1, 2, 4. Prove que sempre possvel permutarmos os algarismos de A de modo que o nmero resultante seja um mltiplo de 7. Prova: Seja B = (aka1)10, k 1, o nmero obtido a partir de A ao suprimirmos uma ocorrncia de cada um dos algarismos 1, 2, 4 e C o nmero que queremos obter a partir de A. i)
ii)

B = 7 : tome C = 2471 B= 7 ...... 7 : tome C = 7 ... 72471 . Analogamente tratamos o caso em que
k >1 k 1

iii)

iv)

s h algarismos 0 e 7 em B. Suponhamos, de agora em diante, que nem todos os algarismos de B sejam iguais a 7 ou zero. B no equivalente a 0 (mod 7): Como {0, 124, 142, 214, 241, 412, 421} um sistema completo de restos, mdulo 7, (isto , esses nmeros, quando divididos por 7 deixam todos os restos possveis: 0, 1, 2, 3, 4, 5 e 6) obtemos C justapondo, direita de B, uma permutao conveniente de 124. B 0 (mod. 7): Se a1 7,0 ento B' = (aka2 0 a1)10 no mltiplo de 7, pois 10B B' = 9a1. Como {0, 1024, 1042, 2014, 2041, 4012, 4021} tambm um sistema completo de restos, mdulo 7, obtemos C como em (iii) (isto , somando 100B' a um dos seis nmeros 1024,,4021).

PROBLEMA 5

dado um quadrado de lado 1. Demonstrar que, para cada conjunto finito de pontos no bordo do quadrado, possvel achar um vrtice do quadrado com a seguinte propriedade: a mdia aritmtica dos quadrados das distncias de tal 3 . vrtice aos pontos do conjunto maior ou igual a 4

EUREKA! N5, 1999

11

Sociedade Brasileira de Matemtica

SOLUO

Sejam A1A2A3A4 o quadrado e X1, X2, , Xn os pontos do permetro. Devemos provar que uma das quatro somas Ai X 1 + Ai X 2 + ... + Ai X n , i = 1, 2, 3, 4, maior 3n ou igual que . 4 Somamos entre si estas quatro somas
_____ 2 _____ 2 _____ 2

( A1 X 1 + A1 X 2 + ... + A1 X n ) + ( A2 X 1 + A2 X 2 + ... + A2 X n ) + ( A3 X 1 + A3 X 2 + ... + A3 X n ) + ( A4 X 1 + A4 X 2 + ... + A4 X n ) e reagrupamos os somandos em n grupos, um por cada ponto Xi ( A1 X 1 + A2 X 1 + A3 X 1 + A4 X 1 ) + ( A1 X 2 + A2 X 2 + A3 X 2 + A4 X 2 ) + ... + ( A1 X n + A2 X n + A3 X n + A4 X n ). Demostraremos que, se X um ponto do permetro do quadrado, ento
______ 2 ______ 2 ______ 2 _____ 2 _____ 2 _____ 2 _____ 2 _____ 2 _____ 2 ______ 2 _____ 2 _____ 2 ______ 2 ______ 2 ______ 2 _____ 2 _____ 2 _____ 2

_____ 2

_____ 2

_____ 2

_____ 2

_____ 2

_____ 2

( A1 X + A2 X + A3 X + A4 X ) 3.
Sejam x e 1 x as distncias X aos extremos do lado ao que pertencem. Ento as distncias de X aos outros dois vrtices do quadrado so, em alguma ordem,

_____ 2

_____ 2

_____ 2

_____ 2

1 + x 2 e 1 + (1 x) 2 , e temos
Devemos provar que, para todo x [0,1] ,

A1 X + A2 X + A3 X + A4 X

_____2

_____2

______2

______2

= x 2 + (1 x) 2 + (1 + x 2 ) + (1 + (1 x) 2 ) = 4( x 2 x +1).
1 3 3 ( x ) 2 + , que claramente 2 4 4
_____ 2 _____ 2

4( x 2 x + 1) 3, mas isto equivale a


verdadeira. Temos assim que

( A1 X 1 + A1 X 2 + ... + A1 X n ) + ( A2 X 1 + A2 X 2 + ... + A2 X n ) + + ( A3 X 1 + A3 X 2 + ... + A3 X n ) + ( A4 X 1 + A4 X 2 + ... + A4 X n ) 3n, 3n portanto, um dos quatro somandos maior ou igual que , c.q.d. 4
_____ 2 ______ 2 _____ 2 ____ 2 _____ 2 _____ 2

_____ 2

_____ 2

_____ 2

_____ 2

EUREKA! N5, 1999

12

Sociedade Brasileira de Matemtica

PROBLEMA 6

Uma formiga caminha pelo piso de um ptio circular de raio r e avana em linha reta, mas s vezes se detm. Cada vez que se detm, antes de continuar a caminhar, gira 60o alternando o sentido (se da ltima vez ela girou 60o para a direita da prxima vez gira 60o para a esquerda, e vice-versa). Achar o maior comprimento possvel do caminho percorrido pela formiga. Demonstrar que o comprimento assim obtido efetivamente, o mximo possvel.

60o

Giro de 60o direita.


SOLUO

Podemos supor que a formiga s se detm uma vez, pois, caso se detenha mais vezes podemos substituir seu caminho por outro de mesmo comprimento onde ela s se detm uma vez, como na figura abaixo

Sejam A e B respectivamente os pontos inicial e final do caminho percorrido pela formiga. Se AB C' no dimetro, traamos o dimetro A'B' // AB. Se C' tal que A'C'B' ACB, temos C' dentro do B crculo (pois A'C'B' = 120) e A'C' + C'B' = B' A O A' B' A' ( AC + CB ) > AC + CB. AB Logo, para o caminho ser mximo AB deve ser dimetro, ou seja, AB = 2r Temos agora que maximizar AC + CB sobre todos os tringulos ACB. tais que
C

que ACB = 120 e AB = 2r. Seguem duas demonstraes de que AC + CB mxima quando AC = CB:

EUREKA! N5, 1999

13

Sociedade Brasileira de Matemtica

1a. demonstrao

a 2 + b 2 + ab = 4r 2 (a + b) 2 ab = 4r 2
C
120

a B

( a + b) 2 a+b 4r 3

b A 2r

( a + b) 2 (a + b) 2 ab = 4r 2 4 , com igualdade se e s se a = b.

2a. demonstrao
C''

Seja C ' ' sobre AC tal que C ' ' C = CB e C ' ' A. Temos AC ' ' = AC + CB e AC ' ' B = 60.

C
120

C'' C O'

Ento devemos determinar a maior corda AC'', com C'' sobre o arco capaz do ngulo 60 sobre AB. Se O' o centro da circunferncia do arco capaz, a corda o dimetro por O', Da, C O' e AC = CC'' = CB

Assim, a resposta do problema

4r 3

EUREKA! N5, 1999

14

Sociedade Brasileira de Matemtica

40a. OLIMPADA INTERNACIONAL DE MATEMTICA


Problemas e Resultados
A 40a. Olimpada Internacional de Matemtica foi realizada na cidade de Bucharest, Romnia, no perodo de 10 a 22 de julho de 1999. A equipe brasileira foi liderada pelos professores Nicolau Coro Saldanha, da PUC - Rio, e Carlos Gustavo T. de A. Moreira, do IMPA.

O Resultado da Equipe Brasileira BRA 1 BRA 2 BRA 3 BRA 4 BRA 5 BRA 6 Fabrcio Siqueira Benevides Pedro Paulo de Simoni Gouveia Daniel Massaki Yamamoto Srgio Tadao Martins Daniel Nobuo Uno Humberto Silva Naves 13 pontos 12 pontos 08 pontos 14 pontos 11 pontos 17 pontos (Medalha de Bronze) (Medalha de Bronze) (Medalha de Bronze) (Medalha de Bronze)

A prova deste ano foi considerada difcil. Pela primeira vez em muitos anos nenhum participante obteve a pontuao mxima (42 pontos). A maior pontuao obtida este ano foi de 39 pontos.

Primeiro Dia Durao da prova: 4 horas e 30 minutos


PROBLEMA 1

Determine todos os conjuntos finitos S de pontos do plano com pelo menos trs elementos que satisfazem a seguinte condio: Para quaisquer dois pontos distintos A e B de S, a mediatriz do segmento AB um eixo de simetria de S.
PROBLEMA 2

Seja n 2 um inteiro fixo. a) Determinar a menor constante C para a qual a desigualdade

1 i < j n

x i x j ( xi2 + x 2 j ) C(

1i n

x )
i

b)

vlida para quaisquer nmeros reais x1, , xn 0. Para esta constante C, determine quando ocorre a igualdade.

EUREKA! N5, 1999

15

Sociedade Brasileira de Matemtica

Considere um tabuleiro quadrado n n, onde n um inteiro positivo par fixo. O tabuleiro est dividido em n2 quadrados unitrios. Dizemos que dois quadrados distintos do tabuleiro so adjacentes se eles tm um lado comum. Marcam-se N quadrados unitrios do tabuleiro de tal forma que qualquer quadrado (marcado ou no) adjacente a pelo menos um quadrado marcado. Determine o menor valor possvel para N.

PROBLEMA 3

Segundo Dia Durao da prova: 4 horas e 30 minutos


PROBLEMA 4

Determine todos os pares (n, p) de inteiros estritamente positivos tais que

p primo, n 2p, e (p 1)n + 1 divisvel por np1.


PROBLEMA 5

Duas circunferncias 1 e 2 esto contidas no interior de uma circunferncia e so tangentes a em pontos distintos M e N, respectivamente. A circunferncia 1 passa pelo centro de 2 . A reta que passa pelos dois pontos de interseo de 1 e 2 intersecta em A e B. As retas MA e MB intersectam 1 respectivamente em C e D. Prove que CD tangente a 2 .
PROBLEMA 6

Determine todas as funes f : R R tais que

f ( x f ( y )) = f ( f ( y )) + x f ( y ) + f ( x) 1
para quaisquer x, y R.

EUREKA! N5, 1999

16

Sociedade Brasileira de Matemtica

ADEDANHA OU DE COMO OS DEUSES MATEMTICOS TROUXERAM A PAZ AO MUNDO Nvel Iniciante


Pablo Emanuel - IMPA

Diz a lenda que, h muitos milnios, o mundo vivia em guerra constante, pois as pessoas no sabiam como resolver as suas discordncias, a no ser pela fora bruta. Um dia, os deuses (que so exmios matemticos), para resolver esta situao, enviaram um mensageiro Terra, com a misso de ensinar os homens a resolverem as suas disputas. O anjo se dirigiu ento aos homens, dizendo: - Quando dois entre vs precisarem chegar a um acordo, que se faa como vos digo: que um escolha par e o outro escolha mpar, ento que ambos mostrem ao mesmo tempo a mo exibindo uma certa quantidade de dedos. Sero ento somadas estas quantidades. Se a soma for um nmero par declara-se vencedor o jogador que escolheu par e, caso contrrio, declara-se vencedor aquele que escolheu mpar. Os homens ficaram maravilhados com a sabedoria dos deuses e, deste dia em diante, houve um grande perodo de paz, pois todas as questes eram resolvidas com o jogo que eles haviam aprendido dos deuses. Um dia, porm, esta paz foi abalada. Trs reis disputavam um pedao de terra, que ficava exatamente na divisa entre os trs pases. Eles estavam prontos a utilizar o jogo divino do par-ou-mpar, mas o rei que sabia mais matemtica entre os trs se levantou e disse: - Caros colegas, ns todos sabemos que um nmero s pode ser par ou mpar, no existindo uma terceira opo. Como somos trs, algum de ns no vai ter opo alguma. Este era realmente um problema muito srio. Para resolv-lo, foi chamado o melhor matemtico da Terra na poca, chamado Zerinhoum. Ele pensou durante vrias semanas em como resolver o problema dos reis, e finalmente chegou a uma soluo: - Majestades, encontrei a soluo para o vosso problema. Ao mesmo tempo, vs estendereis vossas mos, mantendo-as ou com a palma para cima ou com a palma para baixo. Aquele dentre vs que tiver a mo em posio diferente dos demais ganha a disputa.

EUREKA! N5, 1999

17

Sociedade Brasileira de Matemtica

- E se todos ns tivermos as palmas das mos viradas para o mesmo lado? -indagaram os reis. - Neste caso, majestades, vs jogareis novamente, at que algum entre vs vena. Como a disputa era muito urgente, os reis aceitaram a sugesto do eminente matemtico. Houve mais um perodo de paz, desta vez muito mais curto. Em pouco tempo, as pessoas perceberam que o jogo de Zerinhoum podia se alongar indefinidamente, e que era possvel se fazer alianas para prejudicar adversrios polticos. Ento as pessoas rezaram aos deuses, pedindo um novo jogo, que trouxesse de novo a paz Terra. Os deuses ento enviaram novamente um mensageiro. Quando ele chegou, os homens lhe cercaram dizendo: - Mensageiro dos deuses, atendeste as nossas preces. Vivamos em guerra, e os deuses nos enviaram o sagrado jogo do par-ou-mpar, que nos trouxe a paz. Mas este jogo s podia ser jogado por dois jogadores, e as trevas se abateram de novo sobre ns. Ento um grande homem nos ensinou um novo jogo, que chamamos Zerinhoum em sua homenagem. Mas este jogo tinha problemas, e a guerra voltou a nos assolar. Por favor, grande sbio, que vem em nome dos deuses, ensina-nos um novo jogo, que possa nos trazer de volta nossa paz. E o anjo assim respondeu: - Eu vos ensinarei um novo jogo. Zerinhoum era um grande matemtico, mas no conhecia os segredos dos deuses. Eu vos revelarei estes segredos. Para isto, o melhor comear pelo antigo jogo do par-ou-mpar. Como se decide se um nmero par ou mpar? Basta dividi-lo por 2. Se o resto for igual a 0, o nmero ser par, se for igual a 1, o nmero ser mpar. Estas so as nicas duas opes, porque o resto sempre menor do que o dividendo (2). Reparai que se dividirmos o nmero por 3, passam a existir 3 opes para o resto, pois ele pode ser 0, 1 ou 2. Na diviso por 4, existem 4 restos possveis ( 0, 1, 2 e 3). Em geral, quando dividimos um nmero por n , existem n restos possveis ( 0, 1, 2, , n 2 e n 1 ). E o que isto tem a ver com o jogo? Tudo, eu vos digo. Se n pessoas estiverem em uma disputa, vs fareis como eu vos digo: As pessoas escolhero, cada uma, um nmero entre 0 e n 1 diferente. Depois, ao mesmo tempo, elas mostraro as mos, exibindo uma quantidade qualquer de dedos. As quantidades sero somadas, e o nmero resultante ser dividido por n. A pessoa que escolheu o resto desta diviso ser a vencedora.

EUREKA! N5, 1999

18

Sociedade Brasileira de Matemtica

Esta a forma que os deuses jogam. Mas vs da Terra sois muito desorganizados para poder escolher tantos nmeros de forma tranqila. Portanto, eu vos ensinarei uma forma alternativa de jogar este jogo. Vs vos arrumareis em um crculo. Uma pessoa ser designada a contar. Ento vs gritareis a palavra mgica Adedanha e todos mostraro as mos. Os resultados sero somados, e aquele que havia sido designado far o seguinte procedimento: Em primeiro lugar falar Um, e apontar para o cu, para que nunca vos esqueais de que foram os deuses que vos ensinaram este jogo. Ento apontar para si mesmo e falar Dois. Depois apontar para o jogador sua esquerda e falar Trs, e depois seguir apontando para o jogador esquerda deste e assim por diante, sempre acrescentando um ao nmero que havia falado anteriormente, at chegar soma que havia sido calculada. O jogador que estiver sendo apontado neste momento ser o vencedor. Se a soma for 1, o jogador que estiver direita do que estiver contando ser declarado vencedor. Se for 0, ser o que estiver direita deste. Os homens entenderam as determinaes do mensageiro, mas ainda no entendiam porque o segundo jogo era equivalente ao primeiro. O anjo ento lhes explicou: - A pessoa que est contando vai apontar para si mesma quando estiver falando 2. Depois vai dar uma volta completa no crculo e vai apontar para si mesma novamente quando estiver no 2 + n, e novamente no 2 + 2n. Ou seja, ela vai estar apontando para si mesma se e somente se estiver falando um nmero cujo resto na diviso por n seja 2. Da mesma forma, vai estar apontando para o jogador sua esquerda se e somente se estiver falando um nmero que deixa resto 3 ao ser dividido por n. E assim por diante, de forma que cada jogador ter associado a si um nmero entre 0 e n 1 tal que ele o vencedor se e somente se o resultado da soma deixa aquele resto quando dividido por n. Os homens estavam maravilhados com a explicao do mensageiro, mas um sbio ancio levantou uma questo: - , mensageiro divino, sem dvida s sbio e sagaz. No entanto, uma dvida me corri o esprito. Tendo cada jogador 10 dedos, esta soma pode atingir nmeros muito elevados, fazendo com que o responsvel pela contagem passe um tempo enorme falando e apontando at que se descubra o vencedor. - Tens toda a razo, sbio homem. Mas em verdade vos digo que tolice que um jogador exiba uma quantidade de dedos maior ou igual quantidade de jogadores. Com efeito, suponde que um jogador coloque um nmero maior ou
EUREKA! N5, 1999

19

Sociedade Brasileira de Matemtica

igual a n. Os primeiros n dedos s vo ter o efeito de fazer com que a contagem d uma volta completa no crculo, sem alterar em nada quem ser o vencedor. Portanto, ele pode subtrair n da sua quantidade sem que isto altere o resultado. Se o nmero persistir maior ou igual a n, basta voltar a subtrair, at que o nmero fique entre 0 e n 1. - Isto de fato diminui sobremaneira o esforo requerido- replicou o ancio. Mas ainda assim o resultado pode chegar a n(n 1), que ainda bastante grande. - s de fato perspicaz, meu nobre homem. Mas no penseis que a sabedoria dos deuses possui limite. O mesmo processo que foi aplicado a cada nmero individualmente pode ser aplicado soma. Por exemplo, considerai um jogo com 4 jogadores. Suponde que um dos jogadores exibe 3 dedos e outro exibe 2 dedos. Por que considerar a sua soma como sendo 5, se o efeito de somar 4 apenas fazer com que o responsvel pela contagem d uma volta a mais? Em vez disto, muito mais sensato considerar a sua soma como sendo 5 4 = 1. Mais geralmente, considere um jogo com n jogadores. Em primeiro lugar diminui-se n dos valores jogados por cada um, de forma que todos eles estejam entre 0 e n 1 (se todos os jogadores dessem ouvidos s palavras dos deuses, no jogariam alm destes limites). Depois procede-se a soma, da seguinte forma. Soma-se o primeiro valor com o segundo. Caso esta soma seja um valor maior ou igual a n, subtrai-se n do resultado ( Sede espertos e sabereis que fazendo isto sempre obtereis um nmero entre 0 e n 1 ). Depois, a este resultado, soma-se o terceiro valor, tomando-se o cuidado de se subtrair n caso a soma exceda n 1. Prossegue-se desta forma at que todos os valores tenham sido somados. Se seguistes o meu raciocnio at este ponto, no deveria ser-vos surpresa o fato que o resultado de uma tal operao est sempre entre 0 e n 1, e portanto o jogador responsvel pela contagem nunca precisar dar mais de uma volta. E ento todos os habitantes se ajoelharam aos ps do anjo, reconhecendo a sua suprema sabedoria, e o mundo conheceu enfim a paz. At hoje os homens jogam os jogos de par-ou-mpar e adedanha da forma como foram ensinados pelos deuses, embora, infelizmente, a maioria tenha se esquecido da lio final e continue se extenuando em uma interminvel contagem que d voltas e mais voltas.

EUREKA! N5, 1999

20

Sociedade Brasileira de Matemtica

E foi assim que a lenda me foi contada pela minha av, que ouviu de sua av, que ouviu de sua prpria av, e assim por diante, at o princpio dos tempos. Voc deve estar achando meio esquisita a maneira de somar que foi ensinada pelos deuses. No entanto, eles a usaram em vrias outras coisas que nos so muito familiares. Se voc no acredita, responda rpido a estas perguntas: a) Se uma coisa comea em uma segunda-feira e dura 7 dias, em que dia ela termina? E se durar 14 dias? E se durar 701 dias? b) Se uma coisa comea s 8 horas da manh e dura 24 horas, a que horas ela acaba? E se durar 48 horas? E se durar 4804 horas? c) Se o ponteiro dos minutos de um relgio est apontando 23 minutos, para onde ele estar apontando daqui a 60 minutos? e daqui a 120 minutos? e daqui a 66681 minutos? Garanto que, se voc respondeu terceira pergunta dos 3 tens, no contou de um em um (ou ento j estamos no terceiro milnio ). Voc percebeu que os dias da semana se repetem de 7 em 7 dias, que as horas do dia se repetem de 24 em 24 horas e que o ponteiro do relgio volta a apontar para o mesmo ponto de 60 em 60 minutos. Garanto tambm que voc, sem se dar conta, j pensou vrias vezes coisas como 5 horas depois das 21 horas so 2 horas da manh, ou seja, fez a conta 21 + 5 = 2 ! E, por incrvel que parea, esta conta est certa!!! Est certa, porque voc est pensando a menos de mltiplos de 24 (ou, como preferem os matemticos, mdulo 24) , ou seja: 21 + 5 = 2 ( + um mltiplo de 24 ) , ou, como preferem os matemticos, 21 + 5 = 2 (mod 24) . Desta forma, a terceira pergunta do item c) pode ser reescrita como Quanto 23 + 66681 (mod 60) . Se voc foi esperto(a) o suficiente para responder quela pergunta, voc j deve ter percebido que 66681 = 21 (mod 60), e que 23 + 66681 = 23 + 21 (mod 60), ou seja, 23 + 66681 = 44 (mod 60), logo o ponteiro estar apontando para o minuto 44. S para ver se voc entendeu at agora, preencha estas lacunas:
EUREKA! N5, 1999

21

Sociedade Brasileira de Matemtica

2 + 2 = 1 (mod __ ) 2 +__ = 0 (mod 17) 26 = 3 (mod __ ) No se esquea que a expresso mod n s uma forma abreviada de + um mltiplo de n. Lembrando-se disto, veja quantas coisas voc sabia, mas no sabia que sabia: 3 3 = 1 (mod 4) 1 = 1 (mod 2) 2 2 2 2 = 1 (mod 5) 3 3 3 3 = 1 (mod 5) (esta talvez voc no saiba, mas n n n n = 1 (mod 5), sempre que n no mltiplo de 5. Voc pode ver isto e muito mais no artigo do professor Carlos Gustavo Moreira, na EUREKA! No. 2. Pergunta: se n mltiplo de 5, quanto n n n n (mod 5)? ) Agora que voc j sabe o segredo dos deuses matemticos, j pode jogar adedanha da forma original, como os deuses a conceberam, e manter a paz no mundo sem fazer esforo.

EUREKA! N5, 1999

22

Sociedade Brasileira de Matemtica

QUADRILTEROS E TRINGULOS
Marcelo Mendes

Nvel Intermedirio
Apresentamos a seguir alguns resultados que servem de ferramenta para resoluo de problemas de geometria elementar envolvendo quadrilteros e tringulos, bastante freqentes em problemas de olimpada.

QUADRILTEROS INSCRITVEIS

Os ngulos opostos de um quadriltero inscritvel so suplementares. Reciprocamente, se os ngulos opostos de um quadriltero so suplementares, ento esse quadriltero inscritvel (cclico).
D C O

Alm disso, se ocorrer uma situao onde dois ngulos iguais olham para um mesmo segmento, ento os extremos desse segmento e os vrtices dos dois ngulos formam um quadriltero inscritvel.
D C

Exemplo: Seja AB o dimetro de um semicrculo. Um ponto M marcado no semicrculo e um ponto K marcado sobre AB. Um crculo com o centro P passa
EUREKA! N5, 1999

23

Sociedade Brasileira de Matemtica

por A, M, K e um crculo com centro Q passa por M, K, B. Prove que M, K, P, Q pertencem a um mesmo crculo.

Soluo: No crculo circunscrito de AMK, MPK = 2MAK; e no crculo circunscrito de BMK, MQK = 2MBK. Como AB dimetro do semicrculo, AMB = 90o e MAK+MBK = 90o. Da, MPK+MQK = 180o e MPKQ inscritvel.
TEOREMA DE PTOLOMEU

Se ABCD um quadriltero inscritvel de diagonais AC e BD, ento: AB CD + AD BC = AC BD.


A a E B b C d D c

Prova: Seja x = BD e y = AC e a, b, c, d, os comprimentos dos lados. Construa CDE = ABD, E AC. Da, CDE ADB e ADE BCD, dando, respectivamente, ECx = ac e AEx = bd. Somando essas duas ltimas equaes, temos xy = ac + bd, como queramos provar
H tambm uma extenso para esse teorema que vale para quadrilteros no inscritveis: AB CD + AD BC > AC BD, isto , numa situao geral vale AB CD + AD BC AC BD.

Exemplo: Prove que, se ABCDEFG um heptgono regular convexo, ento:


1 1 1 . = + AB AC AD

Aplicando o Teorema de Ptolomeu no quadriltero inscritvel ACDE, onde


EUREKA! N5, 1999

24

Sociedade Brasileira de Matemtica

CD = DE = a = AB, AC = CE = b e AD = AE = c, temos bc = ac + ab. Dividindo essa ltima equao por abc, segue o resultado.
A RELAO ENTRE A DISTNCIA DO ORTOCENTRO A UM VRTICE E DO CIRCUNCENTRO AO LADO OPOSTO

Sejam H e O respectivamente o ortocentro e o circuncentro, do ABC e M, o ponto mdio do lado BC. Ento AH = 2OM.

A Y
H

O M

N C

Prova: Sejam AX e BY alturas e N, o ponto mdio de AC. Como MN base mdia, MN // AB e MN = AB. Da, ABH OMN pois tm lados paralelos entre si (e razo 2:1). Portanto, AH = 2OM Exemplo: Prove que o ortocentro, o baricentro e o circuncentro de um tringulo qualquer so colineares. (Reta de Euler)
Seja G a interseo de AM e HO (na figura acima). Ento, AHG GOM na razo 2:1. Da, AG = 2GM. Portanto, G o baricentro e pertence reta HO.
PROBLEMAS

1. 2. 3.

Seja P um ponto sobre o menor arco AC da circunferncia circunscrita a um tringulo equiltero ABC. Calcule a medida do ngulo APC. Prove que um trapzio inscritvel se, e somente se, ele for issceles (lados no paralelos iguais). Sejam AX e BY alturas de um tringulo issceles ABC (AC = BC) de ortocentro H. Prove que 2HXXC = XYHC.

EUREKA! N5, 1999

25

Sociedade Brasileira de Matemtica

4. 5. 6.

Seja ABCD um losango inscritvel de lado 1 e P, um ponto sobre o menor arco CD. Prove que PD2 + PCPA = 1. Seja P um ponto sobre o menor arco AC da circunferncia circunscrita a um tringulo equiltero ABC. Prove que PB = PA + PC. Seja H o ortocentro de um tringulo ABC e P, o ponto diametralmente oposto a B na circunferncia circunscrita a ABC. Prove que AHCP um paralelogramo. ABCD um paralelogramo. H o ortocentro do ABC e O, o circuncentro do ACD. Prove que H, O, D so colineares. 1 1 1 Seja A1A2An um polgono regular de n lados. Se , = + A1 A2 A1 A3 A1 A4 calcule n. Sejam M, N, P os pontos mdios dos lados de um ABC acutngulo de circuncentro O. Prolongue MO, NO, PO, a partir de O, at X, Y, Z, respectivamente, tais que MX = 2 OM , NY = 2 ON , PZ = 2 OP. Prove que XYZ semelhante ao ABC . Sejam M, N, P os pontos mdios dos lados de um ABC acutngulo de circuncentro O. Prolongue MO, NO, PO, a partir de O, at X, Y, Z, respectivamente, tais que MX, NY, PZ tenham comprimentos respectivamente iguais s metades das alturas do tringulo a partir dos vrtices A, B, C. Prove que XYZ semelhante ao tringulo rtico de ABC (tringulo formado pelos ps das alturas do ABC).

7. 8.

9.

10.

EUREKA! N5, 1999

26

Sociedade Brasileira de Matemtica

O PRINCPIO DAS GAVETAS Nvel Intermedirio


Paulo Cezar Pinto Carvalho - IMPA

Muitos problemas atraentes de matemtica elementar exploram relaes entre conjuntos finitos, expressas em linguagem coloquial. Parte de sua atrao vem justamente do fato de que podem ser formulados e, muitas vezes, resolvidos sem recorrer a frmulas ou a tcnicas complicadas. Vejamos um exemplo simples.

Exemplo 1. Qual o nmero mnimo de pessoas que devemos reunir para que tenhamos certeza de que entre elas h duas que fazem aniversrio no mesmo ms? Soluo: A resposta 13. Se houvesse apenas 12 pessoas, seria possvel que cada uma delas fizesse aniversrio em um ms diferente. Com 13 pessoas, h , obrigatoriamente, pelo menos um ms com mais de um aniversrio (se houvesse, no mximo, um aniversrio por ms, o nmero de pessoas presentes seria, no mximo, 12).
O argumento empregado acima conhecido como Princpio das Gavetas de Dirichlet ou Princpio das Casas do Pombos. Um possvel enunciado para este princpio o seguinte:

Se n objetos forem colocados em, no mximo, n 1 gavetas, ento pelo menos uma delas conter pelo menos dois objetos.
(Uma maneira um pouco mais formal de dizer o mesmo : se o nmero de elementos de um conjunto finito A maior do que o nmero de elementos de um outro conjunto B, ento uma funo de A em B no pode ser injetiva.) Embora trate-se de um fato extremamente elementar, ele til para resolver problemas que, pelo menos primeira vista, no so imediatos. Para aplic-lo, devemos identificar, na situao dada, quem faz o papel dos objetos e quem faz o papel das gavetas.

Exemplo 2. Uma prova de concurso possui 10 questes de mltipla escolha, com cinco alternativas cada. Qual o menor nmero de candidatos para o qual podemos garantir que pelo menos dois deles deram exatamente as mesmas respostas para todas as questes?
EUREKA! N5, 1999

27

Sociedade Brasileira de Matemtica

Soluo: Neste caso, os objetos so os alunos e as gavetas so as possveis seqncias de respostas . Como cada questo pode ser respondida de 5 modos, a prova pode ser preenchida de 5 5 5 5 = 510 = 9 765 625 modos. Logo, s se pode ter a certeza de que dois candidatos fornecem exatamente as mesmas respostas se houver pelo menos 9 765 626 candidatos. Exemplo 3. Em uma reuniao h n pessoas. Mostre que existem duas pessoas que conhecem exatamente o mesmo nmero de outros participantes (admitimos que conhecerseja uma relao simtrica, ou seja, se a conhece b, ento b conhece a). Soluo: Os objetos so as pessoas. As gavetas, naturalmente, so as quantidades de pessoas conhecidas. Temos, no entanto, uma dificuldade: as possveis quantidades de conhecidos so 0, 1, 2, , n 1. Assim, primeira vista, temos n gavetas para n objetos, o que nos impede de usar o princpio das gavetas. Note, porm, que as gavetas 0 e n 1 no podem ser usadas simultaneamente: se existir uma pessoa que no conhece nenhum participante, ento no pode existir um participante que conhea todos! Assim, uma das gavetas 0 ou n 1 permanece desocupada e os n objetos devem ser, portanto, distribudos em n 1 gavetas. Portanto, uma delas ser ocupada por pelo menos dois objetos, o que mostra que h duas pessoas que conhecem exatamente o mesmo nmero de participantes.
Nos casos anteriores, foi bastante simples identificar as gavetas. Nem sempre assim. Os exemplos a seguir ilustram situaes em que necessrio construir as gavetas a serem usadas.

Exemplo 4: Escolhem-se 5 pontos ao acaso sobre a superfcie de um quadrado de lado 2. Mostre que pelo menos um dos segmentos que eles determinam tem comprimento menor ou igual a 2 . Soluo: Neste caso, est claro que os objetos so os 5 pontos. O ponto chave da resoluo est na identificao das gavetas. Devemos subdividir o quadrado dado em 4 partes de modo tal que a distncia entre dois pontos situados em uma destas partes nunca seja maior que 2 . A Fig. 1 mostra como faz-lo: basta dividi-lo nos quatro quadrados determinados pelas retas que unem os pontos mdios dos lados opostos. Em cada uma destas quatro gavetas, a distncia mxima entre dois pontos igual sua diagonal, que mede 2 . Portanto, dados 5 pontos, pelo menos 2 estaro em uma mesma gaveta e, assim, determinam um segmento de comprimento menor ou igual a 2 .
EUREKA! N5, 1999

28

Sociedade Brasileira de Matemtica


1 1 1 1

Figura 1

Exemplo 5. Escolha 101 nmeros dentre os elementos do conjunto {1, 2, 3, , 200}. Mostre que, dentre os nmeros escolhidos, h sempre dois nmeros tais que um divide o outro. Soluo: Antes de mais nada, observe que podemos escolher 100 nmeros do conjunto sem que exista um par onde um nmero divide o outro: basta tomar os nmeros 101, 102, , 200. claro que se acrescentamos mais um nmero p (obrigatoriamente menor ou igual a 100) a essa coleo, um mltiplo seu j estar l. Na verdade, podemos garantir que esse mltiplo da forma 2rp (basta tomar p e multiplic-lo sucessivamente por 2 at que ele se torne maior do que 100). Mostraremos que isso ocorre para qualquer conjunto de 101 elementos. Ou seja, todo subconjunto com 101 dos nmeros de 1 a 200 sempre contm um nmero e um mltiplo seu obtido atravs de multiplicao por uma potncia de 2. Note que esta afirmativa mais forte do que a dada do enunciado, mas, como veremos, nos permite estruturar uma demonstrao. Isto ocorre com frequncia nos problemas envolvendo o Princpio das Gavetas: parte do sucesso nas solues depende da habilidade em perceber o que deve ser demonstrado. Voltando soluo, observemos que todo inteiro n se escreve, de modo nico, na forma n = 2rb, onde r um inteiro no negativo e b um nmero mpar. Por exemplo, 18 = 21.9 , 36 = 22.9 e 125 = 20.125. Para os nmeros de 1 a 200, os valores possveis de b so os mpares de 1 a 199, que so 100. Aqui esto nossas gavetas! J que h 100 valores possveis de b, qualquer coleo de 101 nmeros de 1 a 200 possui dois nmeros x = 2rb e y = 2sb com o mesmo b (isto , temos dois objetos que sero colocados na mesma gaveta). Se r < s, ento x divide y; seno, y divide x, o que conclui a demonstrao.
O ltimo exemplo requer um argumento um pouco mais sofisticado.

Exemplo 6: Em uma reunio, h 6 pessoas. Mostre que necessariamente existem 3 pessoas que se conhecem mutuamente ou 3 pessoas que no se conhecem mutuamente (como no exemplo 3 admitimos que, se a conhece b, ento b conhece a).
EUREKA! N5, 1999

29

Sociedade Brasileira de Matemtica

Soluo: Usaremos o diagrama da Fig. 2 abaixo para ilustrar a situo. Cada pessoa representada por um vrtice do hexgono. Quando duas pessoas se conhecem, ligamos os vrtices correspondentes por um segmento contnuo; seno, usamos um segmento tracejado. O que devemos mostrar que, nesta figura, necessariamente existe um tringulo formado por linhas contnuas ou um tringulo formado por linhas tracejadas.

Figura 2

Consideremos os segmentos que incidem em um dos vrtices p1. Como eles so 5, h pelo menos 3 deles que so contnuos ou pelo menos 3 que so tracejados. Admitamos que haja 3 contnuos (o argumento seria anlogo no outro caso). Denotemos por p2, p3 e p4 vrtices ligados a p1 por segmentos contnuos (veja a Fig. 3). Se algum dos segmentos p2p3, p2p4 ou p3p4 contnuo, este segmento, juntamente com os que ligam seus extremos a p1, formam um tringulo contnuo. Por outro lado, se nenhum deles contnuo, eles formam um tringulo tracejado, o que completa a demonstrao.
p2

p1

p3

p4

Figura 3

Este exemplo um caso particular de um teorema mais geral, chamado de Teorema de Ramsey. Dado qualquer inteiro k 3, existe um inteiro R(k) tal que, em uma reunio de R(k) pessoas, sempre existem k que se conhecem
EUREKA! N5, 1999

30

Sociedade Brasileira de Matemtica

mutuamente ou k que no se conhecem mutuamente. Este resultado normalmente expresso usando a linguagem de grafos: ao se colorir, com duas cores, as arestas de um grafo completo com R(k) vrtices, h sempre um subgrafo completo com k vrtices onde todas as arestas tm a mesma cor. (Na realidade, o Teorema de Ramsey aborda situaes mais gerais; veja, por exemplo os problemas 8 e 9 abaixo). Aproveitamos para mencionar a seguinte generalizao do princpio das gavetas: Se n objetos so colocados em m gavetas e n > mk (onde m, n e k so nmeros naturais) ento alguma gaveta conter pelo menos k + 1 objetos. Terminamos com uma lista de problemas que podem ser resolvidos com as tcnicas aqui ilustradas. As solues sero publicadas nos prximos nmeros da EUREKA!.
PROBLEMAS

1) Numa gaveta h 6 meias pretas e 6 meias brancas. Qual o nmero mnimo de meias a se retirar (no escuro) para garantir que:
a) b) As meias retiradas contenham um par da mesma cor? As meias retiradas contenham um para de cor branca?

2) Sejam n um natural mpar e A uma matriz simtrica em que cada linha e coluna seja uma permutao dos inteiros 1, 2,, n. Mostre que cada um destes nmeros aparece uma vez na diagonal de A. 3) Mostre que se um subconjunto com n + 1 elementos escolhido do conjunto {1, 2, 3,, 2n} ento este subconjunto necessariamente contm um par de nmeros primos entre si. 4) Considere 9 pontos de coordenadas inteiras no R3. Mostre que o ponto mdio de um dos segmentos de reta definidos por estes pontos tambm tem coordenadas inteiras. 5) Mostre que se n mpar e a1, a2,,an uma permutao de 1, 2,,n, ento o produto (a1 1) (a2 2)(an n) par. 6) Mostre que em qualquer coleo de n inteiros h um subconjunto cuja soma dos elementos divisvel por n.
EUREKA! N5, 1999

31

Sociedade Brasileira de Matemtica

7) Mostre que em qualquer coleo de n inteiros existe um par cuja soma ou diferena divisvel por n. 8) Mostre que em toda reunio com 10 pessoas existem 3 que se conhecem mutuamente ou 4 que se desconhecem mutuamente. Mostre que, na realidade, o resultado vale mesmo que na reunio s existam 9 pessoas. 9) Dados inteiros a, b 2, seja N (a, b) o menor nmero para o qual, dado um conjunto com N (a, b) pessoas, sempre existam a que se conheam mutuamente ou b que se desconheam mutuamente (se existir tal nmero). Os problemas anteriores implicam que N (3, 3) 6 e N (3, 4) 9. Mostre que:
a) b) c)

N(a, 2) = a; N(a, b) = N (b, a); N(a, b) N (a 1, b) + N (a, b 1); observe que, em consequncia, N(a, b) existe para todo par (a, b).

10) Dois discos A e B so divididos em 2n setores iguais. No disco A, n setores so pintados de azul e n de vermelho. No disco B, os setores so pintados de azul ou vermelho de forma completamente arbitrria. Mostre que A e B podem ser superpostos de modo que pelo menos n setores tenham cores coincidentes. 11) Sejam A1, A2,, A100 subconjuntos distintos de um conjunto X satisfazendo a propriedade de que cada Ai possua mais da metade dos elementos de X. Mostre que existem 6 elementos x1, x2,x6 de X tais que cada Ai contenha pelo menos um destes 6 elementos. 12) Considere um conjunto A com n elementos. Seja F uma famlia de subconjuntos de A tal que:
Quaisquer dois elementos de F tm interseo no vazia. Nenhum outro subconjunto de A intersecta todos os elementos de F.

a) D exemplo de uma famlia F satisfazendo a estas condies. b) Mostre que F possui 2n 1 elementos.

13) Uma fbrica produz pelo menos uma unidade de um produto X por dia e no mximo 10 unidades deste produto por semana. Mostre que dado qualquer
EUREKA! N5, 1999

32

Sociedade Brasileira de Matemtica

inteiro positivo n existe um conjunto de dias consecutivos em que a produo total igual a n [ Sugesto: mostre que existe um nmero k (dependente de n) suficientemente grande para o qual os conjuntos {S1, S2,Sk} e {S1 + n, S2 + n, , Sk + n} tem pelo menos um elemento comum, onde Si a soma das produes nos dias 1, 2, , i.].

14) Mostre que toda sequncia com n2 + 1 elementos possui uma subsequncia crescente com n + 1 elementos ou uma subsequncia decrescente com n + 1 elementos. 15) Sejam mn + 1 elementos tais que a1 < a2 < < amn + 1. Mostre que ou existem m + 1 destes nmeros tais que nenhum divisor de um outro ou existem n + 1 deles tais que cada um divisor do seguinte. 16) Prove que se o conjunto {1, 2, 3, , 1978} partido em 6 subconjuntos, em algum destes subconjuntos existe um elemento que igual soma de dois elementos, no necessariamente distintos, do mesmo subconjunto. 17) Considere um conjunto com 2n pontos.
a) Mostre que possvel conectar estes pontos com n2 segmentos de reta sem que um tringulo de vrtices nos pontos dados seja formado. b) Mostre que se os pontos so conectados por n2 + 1 segmentos de reta, ento pelo menos um tringulo formado.

18) Considere um conjunto de n pontos 1, 2, , n. Para cada par de pontos escolhida uma orientao para o segmento de reta que os une. Se o segmento ij orientado de i para j dizemos que i j. Mostre que existe uma permutao a1, a2, an de 1, 2, , n tais que a1 a2 an. 19) So dados n pontos azuis e n pontos vermelhos no plano. Mostre que possvel formar n pares de pontos (um azul e um vermelho em cada par) de modo que os n segmentos de reta definidos por estes pares no se cruzem. 20) Mostre que dados 5 pontos do plano em posio geral h 4 que formam um quadriltero convexo.

EUREKA! N5, 1999

33

Sociedade Brasileira de Matemtica

DESIGUALDADES ELEMENTARES
Antonio Caminha Muniz Neto
Nvel Avanado.
Pretendemos neste artigo desenvolver ferramentas bsicas a fim de que o leitor se torne apto a resolver uma vasta gama de problemas de competies matemticas que envolvam desigualdades. Tentamos tornar nossa exposio a mais auto-suficiente possvel. Em certas passagens, contudo, algum conhecimento de clculo til, ainda que no imprescindvel. Em tais ocasies indicamos ao leitor a referncia [3] como bibliografia auxiliar. Antes de discutirmos qualquer desigualdade em especial, consideremos um exemplo preliminar.

Exemplo
1 2

1:
1 3

Para
1 4

todo
1 n 1 2

1 + + + +... +

(log

inteiro
2

n +1 .

positivo

n,

prove

que

Soluo : Veja que, para todo inteiro k > 1,


1 2 k 1 +1

1 2 k 1 + 2

+ ... +

1 2k

>

1 1 1 k + k + ...+ k 2 2 2 2 k 1 vezes

1 2

Portanto, sendo 2 k a maior potncia de 2 menor ou igual a n, temos

1+ 1 2

j =3

1 j

1+ 1 2+

k 1 1 1 1 1j = 1 + 1 2 + ( 2 +1 + ... + 2 ) >1 + 2 + 2 = 1 + 2 2k k k j =3 j =2
j 1 j

j =2

Mas 2 k n < 2 k +1 k log 2 n < k + 1 1 + k > 2 do enunciado imediata.

1 2

(log

n + 1 , e a desigualdade

O exemplo acima foi colocado de propsito. Ele chama ateno para o fato de que nem sempre precisamos de algo mais que raciocnio para resolver problemas envolvendo desigualdades. A proposio abaixo mostra um pouco mais sobre como podemos derivar desigualdades interessantes com muito pouca matemtica.

EUREKA! N5, 1999

34

Sociedade Brasileira de Matemtica

Proposio 1 (Desigualdade do Rearranjo): Sejam a1 < a 2 < ... < a n reais positivos dados, e considere a expresso S = a1b1 + a 2 b2 + ...+ a n bn , onde ( b1 , b2 ,..., bn ) uma reordenao de ( a1 , a2 ,..., a n ) . Ento
2 2 2 a1a n + a 2 a n 1 + ...+ a n a1 S a1 + a2 + ...+ a n

Prova : Vamos primeiro tornar S a maior possvel. Como s h um nmero finito (n fatorial) de possveis reordenaes ( b1 , b2 ,..., bn ) , h uma delas que torna S mxima. Suponha ento que estamos com a reordenao ( b1 , b2 ,..., bn ) que torna S mxima.
Queremos mostrar que essa reordenao exatamente ( a1 , a2 ,..., a n ) . Para isso, basta mostrarmos que deve ser b1 < b2 < ... < bn . Suponha o contrrio, i.e., que existam ndices i < j tais que bi > b j . Trocando as posies de bi e b j (i.e.,

bi ao lado de a j e b j ao lado de ai ), S varia de a i b j + a j bi ( a i bi + a j b j ) = ( a i a j )( b j bi ) > 0 , quer dizer, S aumenta. Mas isso contraria o fato de ser a reordenao ( b1 , b2 ,..., bn ) aquela que torna S mxima. Logo, b1 < b2 < ... < bn e da bi = ai para todo i, donde o maior valor
pondo
2 2 2 . possvel de S a1 + a2 + ...+ a n O raciocnio para minimizar S anlogo.

Passemos agora a nosso principal objetivo, o estudo de desigualdades especiais. A mais importante destas a dada pela proposio 2 abaixo. Antes, uma definio.

Definio 1 : Dados n > 1 reais positivos a1 , a2 ,..., a n , definimos

i. A mdia aritmtica de a1 , a2 ,..., a n como o nmero ii. A mdia geomtrica de a1 , a2 ,..., a n como o nmero

a1 + a 2 + ...+ a n n n

a1a2 ... a n .

Proposio 2 (Desigualdade Entre as Mdias Aritmtica e Geomtrica) : Dados n > 1 reais positivos a1 , a2 ,..., a n , sua mdia aritmtica sempre maior ou igual que a mdia geomtrica, ocorrendo a igualdade se e s se a1 , a2 ,..., a n forem todos iguais. Em smbolos:
EUREKA! N5, 1999

35

Sociedade Brasileira de Matemtica a1 + a 2 + ...+ a n n

n a1a 2 ... a n

Prova : Faamos a prova em dois passos:

i. A desigualdade verdadeira quando n for uma potncia de 2, ocorrendo a


igualdade se e s se todos os nmeros forem iguais.

ii. A desigualdade verdadeira em geral, e a igualdade ocorre se e s se os


nmeros forem todos iguais.

i. Faamos induo sobre k 1, sendo n = 2 k : Para k = 1, temos


a1 + a 2 2

a1a2 a1 2 a1a2 + a2 0 ( a1 a2 ) 2 0 , o que


a1 + a 2 + ...+ a n n k

verdade. H igualdade se e s se ( a1 a 2 )2 = 0 , i.e., se e s se a1 = a2 . Se j provamos que

n a1a 2 ... a n , com igualdade se e s se

a1 = ... = a n para n = 2 , ento


( a1 +...+ a n ) + ( a n +1 +...+ a 2 n ) 2n

1 a1 +...+ a n 2 n

a n +1 +...+ a 2 n n

a1 ...a n +

a n +1 ...a 2 n

a1... a n

a n +1... a2 n = 2 n a1... a n a n +1... a2 n

Para haver igualdade, devemos ter igualdade em todas as passagens. Ento, deve ser
a1 +...+ a n n
n

= n a1... a n ,
n

a n +1 +...+ a 2 n n

= n a n +1... a2 n e

a1 ...a n +

a n +1 ...a 2 n

= 2 n a1... a n a n +1... a2 n

Para as duas primeiras igualdades, segue da hipteses de induo que deve ser a1 = ... = a n e a n +1 = ... = a2 n A ltima igualdade ocorre se e s se n a1... a n = n a n +1... a 2 n . Estas duas condies juntas implicam que devemos ter a1 = ... = a n = a n +1 = ... = a 2 n . tambm evidente que se os nmeros forem todos iguais a igualdade ocorre.
EUREKA! N5, 1999

36

Sociedade Brasileira de Matemtica

ii. Seja agora n > 1 um natural qualquer e a1 ,a2 ,...,an reais positivos. Tome k
natural tal que 2 k > n. Usando a desigualdade entre as mdias para os

2 k nmeros a1 , a2 ,..., a n e 2 k n cpias de a = n a1a 2 ... a n , obtemos


a1 + ...+ a n + a + ...+ a 2
k

2 a1... a n a 2

2k

a na 2

2k

a2 = a ,

e da a1 + ...+ a n + ( 2 k n )a 2 k a , ou ainda era a desigualdade desejada.

a1 + ...+ a n n

a = n a1 ... a n , que

Para haver igualdade, segue do item i que deve ser a1 = ... = a n = a = ... = a . Em particular, todos os nmeros a1 , a2 ,..., a n devem ser iguais. fcil ver que se esses nmeros forem todos iguais ento h igualdade.

Corolrio 2.1 : Dados n > 1 reais positivos a1 , a2 ,..., a n , temos

( a1 + a2 + ...+ a n )

1 a1

1 a2

+ ...+ a1 n 2 ,
n

com igualdade se e s se a1 , a2 ,..., a n forem todos iguais.

Prova : Basta aplicarmos duas vezes a proposio 2 e multiplicarmos os resultados:

a1 + a2 + ...+ a n n n a1a 2 ... a n e

1 a1

1 a2

+ ...+ a1 n n
n

1 a1a 2 ...a n

Exemplo 2 : (Olimpada Israelense) Sejam k e n inteiros positivos, n > 1. Prove que


1 kn

1 kn +1

+ ...+ kn +1n 1 > n

n k +1 k

Prova : Basta ver que

n 1 1 + n = kn + j j=0

n 1 j =0

1 kn + j

+1 =

n 1

j=0

kn + j +1 kn + j

> nn

n 1 j =0

kn + j +1 kn + j

=nn

k +1 k

EUREKA! N5, 1999

37

Sociedade Brasileira de Matemtica

onde aplicamos a desigualdade entre as mdias aritmtica e geomtrica uma vez. Note que, como os nmeros razo do sinal > acima. Dentre todas as desigualdades especiais que temos oportunidade de usar em problemas de competies matemticas, a desigualdade a seguir se constitui, juntamente com a desigualdade entre as mdias aritmtica e geomtrica, num dos dois mais importantes resultados a serem guardados.
kn + j +1 kn + j

so dois a dois distintos, no h igualdade,

Proposio 3 (Desigualdade de Cauchy): Sejam a1 , ..., a n , b1 , ..., bn reais dados, no todos nulos (n > 1). Ento
2 2 | a1b1 + ... + a n bn | a1 +... + a n 2 2 b1 +... + bn

Alm disso, teremos a igualdade se e s se os ai e os bi forem proporcionais, i.e., se e s se existir um real positivo tal que bi = ai para todo i.

Prova : Considere o seguinte trinmio do segundo grau

f ( x ) = ( a1 x b1 )2 + ( a2 x b2 )2 + ...+( a n x bn ) 2
Desenvolvendo os parnteses, chegamos a
2 2 2 2 2 2 2 f ( x ) = (a1 + a2 + ...+an )x 2(a1b1 + a2b2 + ...+anbn )x + (b1 + b2 + ...+bn ) Por ser uma soma de quadrados, temos f ( x ) 0 para todo real x, e da deve ser 0 , i.e., 2 2 2 2 2 2 4( a1b1 + a2 b2 + ...+ a n bn )2 4( a1 + a2 + ...+ a n )( b1 + b2 + ...+ bn )

Cancelando o fator 4 e extraindo a raiz quadrada de ambos os membros, chegamos na desigualdade de Cauchy. Examinemos agora a igualdade. Se houver igualdade, quer dizer, se for = 0 , ento o trinmio tem uma raiz real : Mas a todos os parnteses devem ser nulos, i.e., bi = ai para todo i. Ento, havendo igualdade os a i e bi devem ser proporcionais. evidente que se eles forem proporcionais a igualdade ocorre. Temos a seguir alguns corolrios importantes.
EUREKA! N5, 1999

( a1 b1 )2 + ( a2 b2 )2 + ...+( a n bn )2 = 0

38

Sociedade Brasileira de Matemtica

Corolrio 3.1 (Desigualdade entre as Mdias Quadrtica e Aritmtica) : Dados


reais positivos a1 , ..., a n , temos se e s se a1 = ... = a n . Prova Fazendo b1 = b2 = ... = bn = 1 na desigualdade de Cauchy, obtemos
2 2 a1 + a2 + ... + a n a1 +... + a n
2 2 2 a1 + a2 + ...+ a n n

a1 + a 2 +..+ a n n

, com igualdade

n,

com igualdade se e s se existir um real positivo tal que ai = para todo i, quer dizer, se e s se os ai forem todos iguais. Para obter a desigualdade do enunciado, basta dividir ambos os membros da desigualdade acima por n.

Corolrio 3.2 : Se n > 1 inteiro e a1 , ..., a n , b1 , ..., bn so reais positivos, ento

a1 b1

+ ...+ bn (a1b1 +... + a n bn ) (a1 +... + a n ) ,


a 2
n

com igualdade se e s se b1 = ... = bn .

Prova : Faa x i =

ai bi

, yi = a i bi e aplique a desigualdade de Cauchy para os

nmeros x1 ,..., x n , y1 ,..., y n . Exemplo 3 : (Teste de Seleo da Romnia para IMO) Sejam x1 , x 2 ,..., x n +1 reais positivos tais que x1 + x 2 + ...+ x n = x n +1 . Prove que

x1( xn+1 x1) + ...+ xn ( xn+1 xn ) xn+1( xn+1 x1 ) + ...+xn+1( xn+1 xn ) Prova : Para 1 j n , seja y j = x n +1 x j . Pela desigualdade de Cauchy
temos

x1 y1 + ...+ x n y n =

x1 + ...+ x n

y1 + ...+ y n =

x n +1 ( x n +1 x1 ) + ...+ ( x n +1 x n )

EUREKA! N5, 1999

39

Sociedade Brasileira de Matemtica

Temos mais duas desigualdades importantes.

Proposio

(Chebychev): Sejam a1 , ..., a n , b1 ,..., bn a1 a 2 ... a n e b1 b2 ... bn . Ento

reais,

com

a1 + a 2 + ...+ a n n

)(

b1 + b2 + ...+ b n n

a1 b1 + a 2 b2 + ...+ a n b n , n

com igualdade se e s se a1 = a2 = ... = a n ou b1 = b2 = ... = bn .

Prova :
a1 b1 + a 2 b2 + ...+ a n b n n

a1 + a 2 + ...+ a n n

)(

b1 + b2 + ...+ bn n

)=

1 n2

[ n( a1b1 + a2b2 + ...+an bn ) ( a1 + a2 + ...+an )( b1 + b2 + ...+bn )] =


= 12
n i , j =1

(ai a j )(bi b j ) 0 ,

j que os ai , bi so igualmente ordenados. Note que a condio do enunciado suficiente para haver igualdade. Por outro lado, suponha que tenhamos a igualdade. Como ( a i a j )( bi b j ) 0 para todos i, j, devemos ter ( a i a j )( bi b j ) = 0 para todos os i, j. Suponha que existisse um ndice k com bk < bk +1 . Ento b1 ... bk < bk +1 ... bn , e de ( a i a k +1 )( bi bk +1 ) = 0 segue que a i = a k +1 para i k. Portanto a1 = a2 = ... = a k = a k +1 . De ( a i a k )( bi bk ) = 0 e i > k conclumos que

a k +1 = ... = a n . Logo, todos os ai devem ser iguais.

Corolrio 4.1 : Sejam a1 , a2 ,..., a n reais positivos e k um natural. Ento


k k k a1 + a2 + ...+ a n n

a1 + a 2 + ...+ a n k n

),

com igualdade se e s se todos os ai forem iguais ou k {0, 1}.


EUREKA! N5, 1999

40

Sociedade Brasileira de Matemtica

Prova : Por induo, o resultado acima trivialmente verdadeiro para k = 1. Suponha k > 1 e o resultado vlido para k - 1. Como ambos os membros da desigualdade acima so invariantes por permutaes dos ndices 1, 2, ..., n, podemos supor sem perda de generalidade que a1 a2 ... a n . Da,
k 1 k 1 k 1 a1 a2 ... a n ,
k a1 k + a2 k + ...+ a n

a1 + a 2 + ...+ a n n

da

desigualdade
n

de

Chebychev

obtemos

k 1 k 1 k 1 a1 + a2 + ... + a n

Pela hiptese de induo, vem que

k 1 k 1 k 1 a1 + a2 + ... + a n n

a1 + a 2 + ... + a n k 1 . n

Combinando as duas desigualdades acima segue o resultado. A condio de igualdade bvia a partir da desigualdade de Chebychev. Por fim, vejamos algo um pouco mais sofisticado.

Definio 2 : Seja I um intervalo da reta e f : I R uma funo. A funo f dita

( ) ii. Cncava se f ( )
i. Convexa se f
x+ y 2

f ( x )+ f ( y ) 2

para todos os x, y em I. para todos os x, y em I.

x+ y 2

f ( x )+ f ( y ) 2

Nas aplicaes, quase sempre lidamos com funes contnuas (se voc no sabe o que vem a ser uma funo contnua, pense na mesma como uma funo cujo grfico no sofre interrupes ou saltos ao longo de seu domnio). Se f for contnua a proposio a seguir geometricamente evidente. A partir de agora, sempre que nos referirmos a uma funo estaremos sempre supondo ser seu domnio um intervalo da reta e a funo contnua nesse intervalo.

Proposio 5 : Sejam f : I R uma funo. Ento:

i. f convexa se e s se, para todos x, y em I e todo t [0, 1] tivermos

f ((1 t ) x + ty ) (1 t ) f ( x ) + tf ( y ) ii. f convexa se e s se, para todos x, y em I e todo t [0,1] tivermos


f ((1 t ) x + ty ) (1 t ) f ( x ) + tf ( y )
Faamos o caso em que f convexa. O outro caso anlogo. Observe que

(1 t ) x + ty [ x, y ] I , e que, no trapzio abaixo, (1 t ) f ( x ) + tf ( y )


41

EUREKA! N5, 1999

Sociedade Brasileira de Matemtica

o comprimento da paralela s bases pelo ponto (1 t ) x + ty

f (z)

z = ( 1 t ) x + ty

Prova : Suponha primeiro que f satisfaz a condio do item i. Tomando t = 1 2 conclumos que f convexa. Reciprocamente, suponha que f seja convexa. Dados x, y em I, temos

( )
x +3y 4

y x+ 2 +y = f 2

( )+ f ( y)
x+ y 2

f ( x )+ f ( y ) + 2

f ( y)

1 4

f ( x) +

3 4

f ( y ) Tro

cando x por y e raciocinando como acima segue que, para


1 3 t 0, 1 4 , 2 , 4 ,1 ,

f ((1 t ) x + ty ) (1 t ) f ( x ) + tf ( y ) (*)
Por induo sobre k inteiro positivo podemos concluir de maneira anloga que (*)
k inteiro. Como todo continua vlida para todo t da forma m k , onde 0 m 2 2

real em [0,1] limite de uma seqncia de nmeros dessa forma, segue que (*) continua vlida para todo t em [0, 1]. As afirmaes a seguir so agora bastante evidentes, e vo ser nosso principal guia quando quisermos decidir se uma dada funo ou no convexa ou cncava.

EUREKA! N5, 1999

42

Sociedade Brasileira de Matemtica

i. Se para todos a < b em I o grfico de f entre as retas x = a e x = b estiver abaixo da reta que passa por ( a , f ( a )), ( b, f ( b )) , ento f convexa, e
reciprocamente.

ii. Se para todos a < b em I o grfico de f entre as retas x = a e x = b estiver acima da reta que passa por ( a , f ( a )), ( b, f ( b)) , ento f cncava, e
reciprocamente. A figura abaixo para se convencer da validez desse resultado no caso em que f convexa.

y = f(x)

(c, e)

(c, d)

f ( a ) + f ( b) a+b b e e= . Da, Nele, c = a + 2 2 2 . evidente que d = f ( c ) = f

f ( a ) + f ( b) b e f convexa. ( a+ 2 ) 2

( )

Para ns, a importncia dessa discusso sobre funes cncavas e convexas reside na seguinte:

Proposio 6 (Desigualdade de Jensen): Sejam I um intervalo da reta e f : I R uma funo. Se x1 ,..., x n I e t1 ,..., t n [0,1] , com t1 + ... + t n = 1 , ento t1 x1 + ... + t n x n I e

EUREKA! N5, 1999

43

Sociedade Brasileira de Matemtica

i. f convexa f t1 x1 + ... + t n x n t1 f x1 + ... + t n f x n

( )

( )

ii. f cncava f ( t1 x1 + ... + t n x n ) t1 f ( x1 ) + ... + t n f ( x n )


Prova : Faamos a prova, por induo sobre n > 1, para o caso em que f convexa, sendo o outro caso anlogo. O caso n = 2 nossa hiptese. Suponha agora que para um certo n > 1 e todos x1 ,..., x n I e t1 ,..., t n [0,1] , com t1 + ... + t n = 1 , tenhamos

t1 x1 +...+ t n x n I e f t1 x1 + ... + t n x n t1 f x1 + ... + t n f x n Considere agora x1 ,..., x n +1 I e t1 ,..., t n +1 [0,1] , com t1 + ... + t n +1 = 1 . Se t n +1 = 1 ento t1 = ... = t n = 0 e nada h a fazer. Seno, defina

( )

( )

y=
tj

t1 x1 + ... + t n x n 1 t n +1

= s1 x1 + ... + sn x n ,

onde s j = 1 t . Como s1 + ...+ sn = 1 , segue da hiptese de induo que n +1 y I . Da,

(1 t n +1 ) f (t1 x1 + ... + t n +1 x n +1 ) = f

t1 x1 + ... + t n x n 1 t n +1

)+t

n +1 x n +1

= f (1 t n +1 ) y + t n +1 x n +1 (1 t n +1 ) f ( y ) + t n +1 f ( x n +1 ) ,
j que f convexa. Aplicando a outra metade da hiptese de induo, obtemos

f ( y) = f ( s1x1 +... + sn xn ) s1 f ( x1) + ...+sn f ( xn ) = 1t1


t

n+1

f ( x1) + ...+ 1tn


t

n+1

f ( xn )

Juntando essas duas desigualdades, obtemos a desigualdade de Jensen. Vejamos agora um exemplo de como aplicar a desigualdade de Jensen.

Exemplo 5: (Olimpada Balcnica) Sejam n > 1 e a1 ,..., a n reais positivos com


soma
a1 1+ b1

1.
a

Para
a

cada
n 2 n 1 .

i,

seja

bi =

j =1, j 1

aj .

Prove

que

n + 1+ 2 b + ...+ 1+ b 2 n

EUREKA! N5, 1999

44

Sociedade Brasileira de Matemtica

Prova : Veja que b j = 1 + 1 a j = 2 a j , e ento temos de provar que


a1 2 a1

a2 2 a2

+ ...+ 2 n a
n

n 2 n 1

x Afirmamos que a funo f : ( ,2 ) R dada por f ( x ) = 2 x convexa. Para


2 ver isso, basta escrever f ( x ) = 2 x 1 , e esboar o grfico de f, como abaixo.

Portanto, temos pela desigualdade de Jensen que

f (a j )
j =1

1 n f n

1 a j = nf ( n ) = j =1

n 2 n 1

Exemplo 6 : Utilizando a funo logaritmo natural e a desigualdade de Jensen, vamos dar outra prova da desigualdade entre as mdias aritmtica e geomtrica. Prova : Sejam a1 ,..., a n reais positivos. Existem reais x1 ,..., x n tais que

a j = ln x j para todo j. Como f ( x ) = ln x uma funo cncava, vem que


f ( x1 ) + ... + f ( x n ) n

x1 + ... + x n n

),

ou seja,

ln n x1 ... x n ln
EUREKA! N5, 1999

x1 + ...+ x n n

45

Sociedade Brasileira de Matemtica

Como f crescente, chegamos ao resultado desejado. Vale notar, para quem tem familiaridade com derivadas, que possvel provar que, se f ' ' existe, ento f convexa se e s se f ' ' ( x ) 0 para todo x em I e f cncava se e s se f ' ' ( x ) 0 para todo x em I. Finalizamos este artigo com alguns problemas onde procuramos oferecer oportunidade de exercitar o que foi aprendido no texto, alm de desenvolver um pouco mais a teoria. bom salientar que em alguns deles mais de uma desigualdade pode ser usada.

Problemas onde no precisamos das desigualdades acima, mas de criatividade 1. (Olimpada Americana): Prove que, para todos a, b, c reais positivos, temos
1 a 3 + b 3 + abc 1 b 3 + c 3 + abc 1 c 3 + a 3 + abc 1 abc

2. (Desigualdade de Abel): Sejam a1 , ..., a n , b1 ,..., bn reais dados (n > 1), com a1 a 2 ... a n 0 . Se M e m so respectivamente o mximo e o mnimo do conjunto {b1 , b1 + b2 ,..., b1 + ... + bn } , prove que
com igualdade se e s se a1 = a 2 = ... = a n .

ma1 a1b1 + a2 b2 + ...+ a n bn Ma1 ,

3. (Teste de Seleo de Singapura para IMO): Prove que, quaisquer que sejam os reais positivos a, b e c, tem-se

c a 2 ab + b 2 + a b 2 bc + c 2 b a 2 + ac + c 2 .
4. (Banco IMO): Sejam n > 1 um inteiro dado. Determine o maior valor
possvel da soma
1 i < j n

x i x j x i + x j sobre todas as n-uplas x1 ,..., x n de reais

no negativos cuja soma 1.

EUREKA! N5, 1999

46

Sociedade Brasileira de Matemtica

Desigualdade entre as mdias aritmtica e geomtrica 5. (Torneio das Cidades) Sejam a, b e c reais positivos dados. Prove que
a3 a + ab + b 2
2

b3 b + bc + c 2
2

c3 c + ca + a 2
2

a + b+ c 3

6. Dados os reais positivos a1 , a2 , a3 , b1 , b2 , b3 , prove que


3

( a1 + b1 )( a2 + b2 )( a3 + b3 )

a1a2 a3 + 3 b1b2 b3

Desigualdades de Chebychev, Jensen e Cauchy

7. (Olimpada Turca) Sejam n > 1 inteiro e x1 ,..., x n reais positivos tais


que

xi2 = 1 .
i =1

Determine

o .

valor

mnimo

de

5 x1 x 2 + x 3 + ...+ x n

5 x2 x1 + x 3 + ...+ x n

+ ...+ x

5 xn 1 + x 2 + ...+ x n 1

8. (Olimpada Romena): Seja h a altura de um tetraedro regular e h1 , h2 , h3 , h4 as distncias desde um ponto P em seu interior s faces do
tetraedro. Prove que
h h1 h + h1

h h2 h + h2

h h3 h + h3

h h4 h + h4

12 5

9. (Banco IMO) : Sejam a, b, c, d reais no negativos tais que ab + bc + cd + da = 1 . Prove que


a3 b+c + d

b3 c+d +a

c3 d +a+b

d3 a + b+ c

1 3

10. Sejam n > 1 e x1 , x 2 ,..., x n reais positivos cuja soma 1. Prove que
x1 1 x1

+ ...+

xn 1 x n

n n 1

x1 + ...+ x n n 1

11. Sejam x1 , x 2 ,..., x n reais pertencentes ao intervalo [0, 1] e tais que x1 + x 2 + ... + x n = a , com 0 < a < 1 . Prove que
EUREKA! N5, 1999

47

Sociedade Brasileira de Matemtica

1 a 1+ a

1 x1 1 x 2 1+ x1 1+ x 2

...

1 x n 1+ x n

na n (n +a )

Outras Desigualdades

12. (Desigualdade de Bernoulli): Sejam n um inteiro positivo e x 1 um n real. Prove que (1 + x ) 1 + nx . 13. (Desigualdade entre as Mdias de Potncias): Sejam < reais positivos. Ento, para todos a1 , a2 ,..., a n reais positivos, vale
a1 + a 2 + ... + a n + ... + a n a1 + a 2 , n n com igualdade se e s se a1 , a2 ,..., a n forem todos iguais. 1 1

e considere a funo f :I 1 ...I n R de n variveis, convexa separadamente em relao a cada varivel. Ento, se I j = [a j , b j ] , f atinge seu valor mximo em um dos 2 n pontos da forma ( c1 ,..., cn ) , com ci = a i ou bi para cada i. Prove isto e enuncie um resultado anlogo desigualdade de Giroux para uma funo de n variveis f, cncava separadamente em relao a cada varivel.

14. (Desigualdade de Giroux): Sejam I1 ,..., I n intervalos fechados da reta

15. (Olimpada Blgara): Sejam n 2 um inteiro e 0 x i 1 para 1 i n. Prove que

( x1 + x 2 +...+ x n ) ( x1 x 2 + x 2 x3 +...+ x n 1 x n + x n x1 ) n 2
16. Os trs itens a seguir visam derivar uma desigualdade difcil.
1 i. (Desigualdade de Young): Sejam p e q reais positivos tais que 1 p + q = 1. Prove que

xy

xp p

yq q

, x, y 0

ii. (Desigualdade de Holder): Sejam a1 , a2 ,..., a n , b1 , b2 ,..., bn reais


EUREKA! N5, 1999

48

Sociedade Brasileira de Matemtica 1 positivos e p, q reais positivos tais que 1 . Prove que p + q =1

n p a i bi ai i =1 i =1
n

1/ p

n q bi i =1

1/ q

iii. (Desigualdade de Minkowsky): Sejam a1 , a2 ,..., a n , b1 , b2 ,..., bn reais


positivos e p um real maior que 1. Prove que
p

(a1 + b1 ) p +...+(a n + bn ) p

a1p +...+ a np + p b1p +...+ bnp

Sugesto: Faa ci = (a i + bi ) p 1 e use o tem anterior para (ai) e (ci) e para (bi) e (ci).

Referncias:
[1] Shklarsky, D. O., Chentzov, N. N. e Yaglom, I. M. The USSR Olympiad Problem Book. Dover. Toronto, 1993. [2] Rousseau, C. e Lozansky, E. Winning Solutions. Springer-Verlag, 1996. [3] Lima, Elon L., Anlise Real, vol. 1. IMPA, 1995.

EUREKA! N5, 1999

49

Sociedade Brasileira de Matemtica

40a. OLIMPADA INTERNACIONAL E 14a. OLIMPADA IBEROAMERICANA DE MATEMTICA


Primeiro teste de Seleo
PROBLEMA 1

Determine todos os inteiros positivos n > 1 para os quais existem um inteiro positivo k inteiros x1, x2, xn dois a dois distintos tais que o conjunto {xi + x j ; 1 i < j n} Seja um conjunto de potncias distintas de k. Observao: x1, x2, xn no so necessariamente positivos.
PROBLEMA 2

Sejam a, b, c, d nmeros reais tais que

a = 4 5 a , b = 4 + 5 b , c = 4 5 + c e d = 4 + 5 + d . Calcule abcd.
PROBLEMA 3

Considere um tringulo ABC e BD e CE as bissetrizes dos ngulos B e C, respectivamente ( D AC e E AB). A circunferncia circunscrita a ABC tem centro O e a circunferncia ex-inscrita tangente ao lado BC tem centro Ia. Estas duas circunferncias intersectam-se nos pontos P e Q. Mostre que PQ paralelo a DE. (i) Prove que IaO perpendicular a DE. (ii)
PROBLEMA 4

Sejam Q+ e Z o conjunto dos racionais extritamente positivos e o conjunto dos inteiros. Determine todas as funes f : Q+ Z satisfazendo as seguintes condies: (1) f (1999) = 1 (2) f(ab) = f (a) + f(b), para quaisquer a, b Q+. (3) f(a + b) min{f(a), f(b)}, para quaisquer a, b Q+. A notao min{x, y} denota o menor dentre os inteiros x e y. Por exemplo, min{3, 4} = 4 e min{3, 3} = 3.
PROBLEMA 5

(i) (ii)

Se m, n so inteiros positivos tais que 2n 1 divide m2 + 9, prove que n uma potncia de 2. Se n uma potncia de 2, prove que existe um inteiro positivo m tal que 2n 1 divide m2 + 9.

EUREKA! N5, 1999

50

Sociedade Brasileira de Matemtica

SOLUES DE PROBLEMAS PROPOSTOS


Publicamos aqui algumas das respostas enviadas por nossos leitores.

21)

a) Encontre todas as solues inteiras da equao b) Encontre todas as solues inteiras da equao
3

a + b = c.
a + 3 b = 3 c.

Soluo de Carlos Alberto da Silva Victor.


a) Vamos separar em 2 etapas: i) Tomando a = k2 ; b = t2 teremos c = (k + t)2 consequentemente (k2, t2, (k + t)2) soluo da equao para todo k, t N. Suponha que a no seja um quadrado perfeito. Neste caso, a pode ser escrito na forma a = k2s, onde s um produto de primos distintos. Como c = a + b + 2 ab , devemos ter b = t2 . s (t N) j que ab dever ser um quadrado perfeito (observe que se tomarmos s com exponente mpar diferente de 1, podemos inclu-lo em t2); Da: c = a + b + 2 ab = k 2 s + t 2 s + 2kts

ii)

c = s (k 2 + t 2 + 2kt ) c = s. (k + t ) 2 e neste caso ( k 2 s, t 2 s, s (k + t ) 2 ) soluo da equao onde k , s, t , N.


Nota: observe que o tem (ii) representa a situao genrica e inclui as solues de (i), fazendo s = 1. b) i) a + 3 b = 3 c , logo c = a + b + 3(3 a 2 b + 3 ab 2 ). Vamos separar tambm em 3 etapas: Tomando a = k3; b = t3 teremos c = (k + t)3 com k, t N e
3

consequentemente (k 3 , t 3 , (k + t ) 3 ) soluo da equao. ii) Suponha que a no seja um cubo perfeito. Observe que a pode ser escrito na forma: a = t 3 . . 2 onde e so produtos de primos distintos. J que c = a + b + 3 (3 a 2 b + 3 ab2 ), devemos tomar
EUREKA! N5, 1999

51

Sociedade Brasileira de Matemtica

b = t 3 . . 2 pois a 2 = k 6 . 2 4 (de fato,

a 2 b + 3 ab 2 racional, e,

elevando ao quadrado, obtemos que a 3 ab 2 + 2ab + b3 a 2 b (e logo

a 3 ab 2 + b3 a 2 b ) tambm racional, donde, assumindo a b,


3

ab 2 e

a 2 b so racionais e inteiros). da:

c = k 3 . . 2 + t 3 . 2 + 3( k 2 t . 2 + k . t 2 . . 2 )

c = . 2 ( k 3 + t 3 + 3k 2 t + 3kt 2 )
Logo (k 3 . . 2 , t 3 . 2 , . 2 (k + t ) 3 ) soluo da equao dada. (Novamente, as solues em (i) so obtidas de (ii) com = = 1.

22)

Sejam , , , os ngulos de um quadriltero, nessa ordem. Prove que esse quadriltero inscritvel se, e somente se, a relao + + + = 2 ocorre.

Soluo de Andr Luiz Arruda Marques.


A

1a. parte:

Hiptese: O quadriltero inscritvel. Tese: vlida a relao + + + = 2

BCD BAD + = 360 Sabe-se que : BCD = 2 2 + 2 = 360 = 2 + = (1) BAD = 2

EUREKA! N5, 1999

52

Sociedade Brasileira de Matemtica


ABC ADC + = 360 ABC = 2 2 + 2 = 360 = 2 + = ( 2) ADC = 2

de (1) e (2) ( + )( + ) = 2 + + + = 2 (c.q.d)

2a. parte:

Hiptese: vlida a relao + + + = 2 Tese: O quadriltero inscritvel

Tm-se a relao: + + + = 2 Fatorando vem: ( + )( + ) = 2 = Sabe-se que: + + + = 360 = 2

xy = 2 x = y = Seja: + = x e + = y x + y = 2 + = Logo: o quadriltero inscritvel (c.q.d.). + =


23)
Seja ABC um tringulo qualquer de ortocentro H e sejam ha, hb, hc os comprimentos das alturas relativas a A, B, C respectivamente. Prove que ____ ____ ____ 1 ha . AH + hb . BH + hc CH = (a 2 + b 2 + c 2 ). 2
A z O N M C
EUREKA! N5, 1999

Soluo de Maria Ivete Caetano Rodrigues.

H B

53

Sociedade Brasileira de Matemtica


____ ____ ____ ____

ha = AM
Notaes: hb = BN
____ ____

CM = x BO = y
____ ____

AB = c

BC = a
____

hc = CO

AN = z

AC = b
___ ____

Temos que : CMH retngulo (CH ) 2 = (ha AH ) 2 + x 2

I II

ACM retngulo b =

2 ha

+ x x =b

2 ha ____

2 2 substituindo II em I, temos (CH ) 2 = ha 2ha AH + ( AH ) 2 + b 2 ha

___

____

( CH ) 2 = b 2 + ( AH ) 2 2ha AH
____ ____

___

____

____

(1)
____

Analogamente temos: ( BH ) 2 = ( CH ) 2 + a 2 - 2hc CH

(2) e

( AH ) 2 = ( BH ) 2 + c 2 - 2hb BH

____

____

____

(3)

somando (1), (2) e (3), obtemos:

2ha AH + 2hc CH + 2hb BH = a 2 + b 2 + c 2 2(ha AH + 2hb BH + hc CH ) = a 2 + b 2 + c 2


___ ____ ____

___

____

____

ha AH + hb BH + hc CH =
25)

___

____

____

a2 + b2 + c2 (c.q.d.) 2

Durante o ano de 1998, uma pequena livraria, que abria nos sete dias da semana, vendeu no mnimo um livro por dia e um total de 600 livros no ano todo. Diga, justificando, se existiu, obrigatoriamente, um perodo de dias consecutivos onde foram vendidos exatamente 129 livros.

Soluo de Marcelo Rufino de Oliveira.


Seja ai o total acumulado de livros vendidos at o final do i-simo dia. Por exemplo: a5 = total vendidos at o quinto dia a35 a31 = total de livros vendidos entre os dias 35 e 32 (inclusive). Ento: a1 < a2 < a3 < < a364 < a365 = 600 (1)
EUREKA! N5, 1999

54

Sociedade Brasileira de Matemtica

Resta agora analisar se existem ai, aj (i, j N menores que 365, j > i) tais que aj ai = 129 Somemos agora a cada termo de (1) o valor 129: a1 + 129 < a2 + 129 < a3 + 129 < < a364 + 129 < a365 + 129 = 729 Chamemos estes termos de bi (bi = ai + 129): b1 < b2 < b3 < < b364 < b365 = 729 Temos ento 730 termos, 365 termos ai e 365 termos bi, com ai aj (i j) e bp bk (p k). Notemos que estes 730 termos naturais esto entre 1 e 729, ou seja, existem dois valores iguais (princpios das casa dos pombos ou Princpio das Gavetas de Dirichet) entre estes 730 termos. Como cada ai distinto e cada bi distinto, ento existe um am que igual a um bn: am = bn => am = an + 129 => am an = 129 Que prova que existe um perodo de dias consecutivos (m n dias) onde foram vendidos exatamente 129 livros.
PROBLEMA No. 9 DO VISITANTE MATEMTICO (Revista EUREKA! No. 2)

Um vaso de vinho est suspenso sobre outro, de igual capacidade (digamos 1 litro), cheio de gua. Por um orifcio no fundo de cada vaso, o vinho escorre sobre o vaso de gua e a mistura se esvai na mesma velocidade. Quando o vaso de vinho estiver vazio, qual o volume de gua no vaso inferior?

Soluo de Carlos Frederico Borges Palmeira.


Aqui est uma soluo, que usa apenas a noo de limite e o fato que 1 1 lim(1 ) n = . n e Vamos discretizar o problema, supondo que primeiro deixamos passar uma frao 1 de um vaso para o outro, a gua e o vinho se misturam, e depois retiramos n 1 do novo volume do segundo vaso. Aps n passos o primeiro vaso estar vazio. n Seja v(k + 1) a quantidade de vinho no segundo vaso aps o k-simo passo.

1 1 Temos v( k + 1) = (1 ) (v(k ) + ), sendo v (0) = 0. n n


EUREKA! N5, 1999

55

Sociedade Brasileira de Matemtica

Agora,

equao
n

de

diferenas

v(k + 1) = av( k ) + b

tem

soluo

1 a ) b (nada alm de progresses geomtricas aqui). 1 a 1 (1 (1 ) n 1 n n ) 1 (1 1 ), e como v (0) = 0, Segue que v( n) = (1 ) v(0) + ( 1 n n n n 1 1 temos apenas o segundo termo, que fica (1 ) (1 (1 ) n ) , e passando ao n n 1 limite quando n tende ao infinito, obtemos 1 ( ). Esta a frao de vinho que e resta no segundo vaso quando o primeiro fica vazio. v( k ) = a n v(0) + (

Voc sabia

que todo poliedro convexo (com

as faces rgidas) rgido (isto , no pode ser deformado) mas existem poliedros no - convexos flexveis?

Errata: No Voc sabia da EUREKA! No. 4, pgina 21, sobre um polinmio que assume apenas valores negativos ou primos, h um erro tipogrfico na terceira linha da expresso do polinmio: onde est "[ e3 . (e + 2) . (a +1)2 + " leia-se "[e3 . (e + 2) . (a + 1) 2 + ". Agradecemos tambm o envio das solues da EUREKA! No.4 a: Osvaldo Ribeiro da Silva Jnior, Robrio Bacelar da Silva, Manuel Joo de Jesus Almeida, Vicente Wilson Moura Gaeta, Rubens Henriques, Raul Rabello Mesquita, Otvio Braga, Antonio Caminha Neto, Alexandre Celestino Leite de Almeida, Luciana Rocha Pedro, Francisco Dutenhefner, Seme Gebara Neto. Continuamos esperando as solues dos problemas 10, 16, 17, 20 e 24.

EUREKA! N5, 1999

56

Sociedade Brasileira de Matemtica

PROBLEMAS PROPOSTOS
Convidamos o leitor a enviar solues dos problemas propostos e sugestes de novos problemas para os prximos nmeros.

26) 27)

Sejam as funes fo (x) = xn e fi (x) = fi 1(x + 1) fi 1(x) onde x, n e i so inteiros positivos. Prove que, para todo x, fn(x) = n! O tringulo equiltero ABC possui um ponto interno P tal que em P chegam trs segmentos de reta (PA, PB, PC) onde PA = 6, PB = 8 e PC = 7. Com esses dados descubra qual a rea do tringulo.
A

P B C

28)

Seja n 2 um nmero inteiro. Prove que n e n + 2 so ambos primos se e somente se


4((n 1)! + 1) + n inteiro. n ( n + 2)

29)

Seja n > 1 um nmero inteiro. Existem n lmpadas L0, L1, ... , Ln1 colocadas em um crculo. Cada lmpada est ACESA ou APAGADA. Uma seqncia de passos S0, S1, ... , Si, ... executada. O passo Sj afeta apenas o estado da lmpada Lj (deixando o estado de todas as outras inalterado) da seguinte forma: Se Lj1 est ACESA, Sj muda o estado de Lj de ACESA para APAGADA, ou de APAGADA para ACESA; Se Lj1 est APAGADA, Sj deixa o estado de Lj inalterado. As lmpadas so rotuladas mod n, ou seja,

EUREKA! N5, 1999

57

Sociedade Brasileira de Matemtica

L 1 = Ln 1, L0 = Ln, L1 = Ln + 1, etc. Inicialmente todas as lmpadas esto ACESAS. Mostre que: a. b. c. Existe um inteiro positivo M(n) tal que depois de M(n) passos todas as lmpadas esto ACESAS de novo; Se n da forma 2k ento todas as lmpadas esto ACESAS depois de n2 1 passos; Se n tem a forma 2k + 1 ento todas as lmpadas esto ACESAS depois de n2 n + 1 passos.

O problema 6 da Olimpada Internacional de Matemtica de 1994 pedia que fosse mostrada a existncia de um conjunto A de inteiros positivos com a seguinte propriedade: para todo conjunto infinito de nmeros primos S existem um inteiro positivo k 2 e dois inteiros positivos m A e n A, cada um dos quais um produto de k elementos distintos de S. Os leitores que enviarem solues corretas (at o dia 15 de novembro de 1999) para o seguinte problema concorrero a um exemplar do livro "10 Olimpadas Iberoamericanas de Matemtica".

PROBLEMA "CUTICO":
Prove que no problema acima k no pode ser escolhido independentemente do conjunto S. Mais precisamente: Prove que para qualquer conjunto de inteiros positivos A e para todo inteiro positivo k existe um conjunto infinito de nmeros primos S tal que o produto de k elementos distintos de S est sempre em A ou o produto de k elementos distintos de S nunca pertence a A.

Nota: Os problemas 26 e 27 foram propostos por Christian Lyoiti Watanabe e Roberto Gomides respectivamente.
EUREKA! N5, 1999

58

Sociedade Brasileira de Matemtica

COMO ASSINAR A EUREKA!


Se voc fantico por Matemtica e deseja receber na sua casa a revista EUREKA!, faa o seu pedido escrevendo para: Secretaria da Olimpada Brasileira de Matemtica, Estrada Dona Castorina, 110 Jardim Botnico - Rio de Janeiro, RJ - CEP: 22460-320. O custo de cada exemplar avulso ou atrasado de R$4,00. Voc pode fazer uma assinatura anual o que dar direito a receber as publicaes do ano (mnimo 3 exemplares) por um valor promocional de R$10,00. Para isso, faa um depsito no Banco do Brasil - Agncia 0598-3 Conta N52208-2 em nome do professor Eduardo Wagner. Envie-nos a fotocopia do depsito e faa referncia aos nmeros desejados. No esquea de colocar seu nome e endereo completos e ns remeteremos a(s) revista(s) pelo correio. Pedidos podem ser feitos tambm por e-mail e comprovantes de depsito podero ser enviados pelo fax.

Se tiver qualquer dvida entre em contato conosco. Telefone: 021-5295077 / Fax: 021-5295023 e-mail: obm@impa.br Home-Page: http//www.obm.org.br/

Revista do Professor de Matemtica


A RPM uma publicao da SBM destinada aos professores de Matemtica do ensino mdio, que pretende ser um veculo de circulao e intercmbio de idias atravs de seus artigos e sees.
Revista do Professor de Matemtica Caixa Postal 66281 CEP 05315-970 So Paulo SP e-mail: rpm@ime.usp.br Fone/Fax: (11) 818-6124

Estamos comemorando o nmero 40 da RPM e 17 anos de publicao sem interrupo, com capa nova e com renovado estmulo para continuar auxiliando o professor de Matemtica.

EUREKA! N5, 1999

59

Sociedade Brasileira de Matemtica

AGENDA OLMPICA

21a. OLIMPADA BRASILEIRA DE MATEMTICA Primeira Fase 12 de junho (sbado) Segunda Fase 28 de agosto (sbado) Terceira Fase 23 de outubro (sbado) e 24 de outubro (domingo)

14a. OLIMPADA IBEROAMERICANA DE MATEMTICA 12 a 19 de setembro La Habana, Cuba.

2a. OLIMPADA IBEROAMERICANA DE MATEMTICA UNIVERSITRIA 16 de setembro

Que se an + 1 primo (com a e n naturais) ento n = 2k para algum k natural? Tente provar isso!, (primos desta forma so conhecidos como primos de Fermat generalizados.) E que se a = 2 s se conhecem 5 primos desta forma (3, 5, 17, 257 e 65537), os chamados primos de Fermat? E que, por outro lado, se conhecem muitos primos de Fermat generalizados grandes (como 10183016384 + 1 e 6723416384 + 1, respectivamente o 16o e 18o. maiores primos conhecidos em 27/7/99)?

Voc sabia

EUREKA! N5, 1999

60

Sociedade Brasileira de Matemtica

COORDENADORES REGIONAIS
Amarisio da Silva Arajo Alberto Hassen Raad Antnio C. Rodrigues Monteiro Angela Camargo Benedito T. Vasconcelos Freire Claudio Arconcher lio Mega Florncio F. Guimares F. Francisco Dutenhefner Gisele de A. Prateado G. Ivanilde H. Fernandes Saad Joo B. de Melo Neto Joo F. Melo Libonati Jorge Ferreira Jos Carlos Pinto Leivas Jos Clovis Saraiba Jos Luis Rosas Pinho Jos Paulo Carneiro Jos Vieira Alves Leonardo Matteo D'orio Licio Hernandes Bezerra Luzinalva M. de Amorim Marco Polo Marcondes Cavalcante Frana Pablo Rodrigo Ganassim Paulo H. Cruz Neiva de L. Jr. Reinaldo Gen Ichiro Arakaki Ricardo Amorim Roberto Vizeu Barros Sergio Claudio Ramos Seme Gebara Neto Tadeu Ferreira Gomes Toms Menndez Rodrigues Valdenberg Arajo da Silva Wagner Pereira Lopes Waldemar M. Canalli (UFV) Viosa - MG (UFJF) Juiz de Fora - MG (UFPE) Recife - PE (Centro de Educao de Adultos - CEA) Blumenau - SC (UFRN) Natal - RN (Col. Leonardo da Vinci) Jundia - SP (Col. ETAPA) So Paulo - SP (UFES) Vitria - ES (UFMG) Belo Horizonte - MG (UFGO) Goinia - GO (U. Catlica Dom Bosco) Campo Grande - MS (UFPI) Teresina - PI (Grupo Educ. IDEAL) Belm - PA (UEM) Maring - PR (UFRG) Rio Grande - RS (UFMA) So Luis - MA (UFSC) Florianpolis - SC (USU) Rio de Janeiro - RJ (UFPB) Campina Grande - PB (Sistema Titular de Ensino)Belm - PA (UFSC) Florianpolis - SC (UFBA) Salvador - BA (Colgio Singular) Santo Andr - SP (UF Cear) Fortaleza - CE (L. Albert Einstein) Piracicaba - SP (Esc. Tec.Everardo Passos) SJ dos Campos - SP (INPE) SJ dos Campos - SP (Centro Educ. Logos) Nova Iguau - RJ (Colgio ACAE) Volta Redonda - RJ (IM-UFRGS) Porto Alegre - RS (UFMG) Belo Horizonte - MG (U. do Estado da Bahia) Juazeiro - BA (U. Federal de Rondonia) Porto Velho - RO (U. Federal de Sergipe) So Cristovo - SE (Esc. Tec. Fed. de Gois) Jata - GO (P.M. S. Joo de Meriti) S. Joo de Meriti - RJ

EUREKA! N5, 1999

61

CONTEDO
40a. OLIMPADA INTERNACIONAL DE MATEMTICA Problemas e Solues 14a. OLIMPADA IBEROAMERICANA DE MATEMTICA Prova 2 13

ARTIGOS
CONTAR DE DUAS MANEIRAS, PARA GENERALIZAR Jos Paulo Q. Carneiro - Universidade Santa rsula A MATEMTICA DAS ABELHAS Jos Cloves Saraiva - Universidade Federal de Maranho O TEOREMA DE RAMSEY Carlos Gustavo T. de A. Moreira - IMPA APLICAES DOS NMEROS COMPLEXOS GEOMETRIA Edmilson Motta - Colgio Etapa 40a. OLIMPADA INTERNACIONAL E 14a. OLIMPADA IBEROAMERICANA DE MATEMTICA Segundo teste de Seleo 10a. OLIMPADA DE MATEMTICA DO CONE SUL Primeiro teste de Seleo 10a. OLIMPADA DE MATEMTICA DO CONE SUL Segundo teste de Seleo SOLUES DE PROBLEMAS PROPOSTOS PROBLEMAS PROPOSTOS ASSINATURA DA REVISTA EUREKA! 2000 FICHA DE CADASTRAMENTO ASSINATURA DA REVISTA EUREKA! COORDENADORES REGIONAIS 15 18 23 30 39

40 41 42 57 59 60 61

Sociedade Brasileira de Matemtica

40a. OLIMPADA INTERNACIONAL DE MATEMTICA


Problemas e Solues Primeiro Dia Durao da prova: 4 horas e 30 minutos
PROBLEMA 1

Determine todos os conjuntos finitos S de pontos do plano com pelo menos trs elementos que satisfazem a seguinte condio: Para quaisquer dois pontos distintos A e B de S, a mediatriz do segmento AB um eixo de simetria de S.

Soluo de Fabrcio Siqueira Benevides (Fortaleza - CE): Seja o fecho convexo dos pontos de S, isto conjunto de pontos da periferia de S que formam um polgono convexo que contm todos os pontos de S. Sejam P1 , P2 ,..., Pn os vrtices do polgono formado pelo fecho convexo.

P3'

Pn

Pn'

P3

P1

P2
______

Afirmao 1) P3 o simtrico de Pn em relao mediatriz de P1 P2 que denotaremos por md( P1 P2 ). Prova: Suponha o contrrio. Seja Pn' o simtrico de Pn a md( P1 P2 ). Pn' deve estar no interior do polgono P1 , P2 ,..., Pn , pois este o fecho convexo.
EUREKA! N6, 1999
______ ______

Sociedade Brasileira de Matemtica


______

Da, P1 P2 Pn ' < P1 P2 P3 , e se P3 ' o simtrico de P3 md( P1 P2 ) teremos


P1 P2 P3 ' > P2 P1 Pn (pela simetria), a P3 ' pertenceria a S e estaria fora do fecho convexo. Absurdo. Se Pn ' est sobre o segmento P2P3, teremos Pn P1 P2 = P1 P2 P3 P3 ' P1 Pn e P1 P3 ' > P1 Pn P3 ' est fora do fecho convexo (Absurdo). Se Pn' est sobre o prolongamento de P2P3, j temos um absurdo pois Pn' estaria fora do fecho convexo. Logo, P3 realmente o simtrico de Pn. Assim
______

______

______

P3
_____

simtrico

de

Pn

em

relao

md( P1 P2 ),

______

donde

P1 Pn = P 2 P 3 ( e Pn P1 P2 = P1 P2 P3 ).
______

Analogamente, se olharmos para a mediatriz de


______

P2 P3

concluimos que

P1 P2 = P3 P4 ( e P1 P2 P3 = P2 P3 P4 ).

______

Se continuarmos a olhar para as mediatrizes de Pi Pi +1 , i = 1,..., n 1, facil concluir que: todos os ngulos internos de P1 , P2 ,..., Pn so iguais e:
______

P1 P2 = P 3 P 4 = P5 P6 = ... = P2 k +1 P2 ( k +1)

_____

_____

______

P2 P3 = P 4 P 5 = P6 P7 = ... = P2 q P2 q +1

_____

_____

(ver indices mdulo n.) Temos agora 2 casos: i) n mpar n = 2k + 1, teremos:

P1 P2 = P 3 P 4 = ... = P2 k +1 P1 = P2 P3 = P 4 P5 = ... = P2 k P2 k +1 ; ou fecho convexo um polgono regular


ii) n par: n = 2k.
______

______

_____

________

______

_____

________

seja

P1 P2 = P 3 P 4 = P2 k +1 P2 k

_____

_________

______

P2 P3 = P 4 P 5 ... = P2 k 2 P2 k 1

_____

____________

mas md (P1P3) tambm eixo de simetria e fcil ver que o simtrico de P2 est sobre a mediatriz de P1 P3 (e seu simtrico ser ele mesmo).
EUREKA! N6, 1999
______

Sociedade Brasileira de Matemtica


______ ______

Nesse caso P1 P2 = P2 P3 , donde novamente o fecho convexo forma um polgono regular. Portanto, o fecho convexo um polgono regular. Para finalizar, provaremos que no pode haver qualquer ponto em S alm de P1 , P2 ,..., Pn . J vimos que P1 P2 ...Pn regular. Seja O o centro da circunferncia circunscrita a P1 P2 ...Pn . (pode ser O S ou O S). Seja P um ponto de S diferente de P1 ...Pn . (P esta sobre algum lado ou no interior de P1 ...Pn ). Seja Pi o vrtice do nosso polgono cuja distncia a P seja mnima. E seja

r = md ( PPi ). claro que r no passa por O, (que o centro de simetria de P1 ...Pn . ), assim se d1 a maior distncia de um vrtice Pj que est no semi-plano esquerdo de r a r; e d2 a maior distncia de um vrtice Pk que est no semiplano direito de r a r; ento d1 d2. Suponha d1 > d2. Ento o simtrico de Pj deveria pertencer a S mas est fora do fecho convexo. Absurdo. Na verdade, quero dizer que se r no um eixo de simetria de P1 ...Pn , tomamos P1 ' ,..., Pn ' os simtricos de P1 ,..., Pn em relao a r, um dos Pi' estar fora do polgono P1 ...Pn .
Concluso: Um conjunto finito de pontos S satisfaz o problema somente se e s se o conjunto dos vrtices de um dado polgono regular.
Seja n 2 um inteiro fixo. Determinar a menor constante C para a qual a desigualdade a)
PROBLEMA 2

_____

1 i < j n

x i x j ( xi2 + x 2 j ) C(

1i n

x )
i

b)

vlida para quaisquer nmeros reais x1, , xn 0. Para esta constante C, determine quando ocorre a igualdade.

Soluo: Consideremos inicialmente o caso n = 2. Queremos achar a menor constante C tal 2 que x1 x 2 ( x12 + x 2 ) C ( x1 + x 2 ) 4 para quaisquer x1 , x 2 0.
EUREKA! N6, 1999

Sociedade Brasileira de Matemtica

1 2 Para x1 = x 2 = 1 temos x1 x 2 ( x12 + x 2 ) = ( x1 + x 2 ) 4 , donde necessariamente 8 1 1 C . Vamos mostrar que C = . Para isso, basta mostrar que 8 8

1 2 x1 x 2 ( x12 + x 2 ) ( x1 + x 2 ) 4 para x1 , x 2 0. 8
2 2 4 Como ( x1 + x 2 ) 4 = x14 + 4 x1 x 2 ( x12 + x 2 ) + 6 x12 x 2 + x2 , isso equivale a mostrar

que

mas isso segue diretamente de 1 ( x1 x 2 ) 4 0. Para n > 2 temos ainda C , pois fazendo x1 = x 2 = 1 e x k = 0 8 1 para k 3 vale a igualdade com C = . 8

4 2 2 x14 + x 2 + 6 x12 x 2 4 x1 x 2 ( x12 + x 2 ),

1 Mostraremos que ainda vale a desigualdade do enunciado para C = . Para isso, 8 mostraremos o seguinte:
Lema: Se x1 ,..., x n 0 e o nmero de termos xi > 0 com 1 i n m 3 ento podemos substituir x1 ,..., x n por x1 , x 2 ,..., x n tais que
~ ~ ~

1 i n ~

x = x ,
i i 1 i n

xi x j ( xi2 + x 2 j)<

1 i j n

1 i j n

xi x j ( xi2 + x 2 j ) e o nmero de termos

xi > 0 com 1 i n m 1. Prova do Lema: Sejam x r e x s , r s os dois menores termos no nulos dentre

x1 , x 2 ..., x n .

Definimos
i
1 i n

xr = xr + xs , xs = 0
~

xk = xk
~

para

k {r , s}.

Obviamente temos

x = x
1 i n

e h m 1 termos xi = 0. Note agora que

1 i j n

y y
i

2 j ( yi

2 2 3 3 + y2 j ) =y r y s ( y r + y s ) + ( y r + y s )

k{r , s}

yk + ( yr + ys )

k{r , s}

3 k,

EUREKA! N6, 1999

Sociedade Brasileira de Matemtica

donde
1i j n

xi x j (xi2 + x 2 j )

1i < j n

x x (x
i j k

2 i

2 2 3 3 3 + x2 j ) = xr xs ( xr + xs ) + ((xr + xs ) ( xr + xs ))

k {r ,s}

= x r x s (3( x r + x s )

k{r , s}

2 ( x r2 + x s )), mas como xr e xs so os dois menores

termos no nulos dentre os xi (e h pelo menos 3), temos x + xs 3 xk r 3( x r + x s ) x k ( x r + x s ) 2 > x r2 + x s2 . 2 2 k{r , s} k{r , s}

Para concluir observamos que: i) Se no mximo um termo dentre os xi no nulo ento xi x j ( x12 + x 2 j ) = 0, e vale a igualdade se e s se todos os termos so
1i < j n

ii)

nulos. Se exatamente dois termos, digamos xr e xs, so no nulos, camos no caso n = 2, ou seja, temos igualdade se e s se xr = xs.

1 xi x j ( x12 + x 2 xi , valendo a j) 8 1i n 1i < j n

iii)

Finalmente, se h m termos no nulos com m 3, aplicamos o lema m 2 vezes e obtemos x1 ' ,..., x n ' onde exatamente dois dos xi ' so no nulos e vale

1i < j n

2 xi x j ( x1

x2 j)

xi ' x j ' ( xi'2 1i < j n

1 x 'j2 )

1 = xi ' xi , onde 8 1in 8 1in

a segunda desigualdade segue de ii). 1 Assim , temos sempre C = , ocorrendo a igualdade se e s se h exatamente 8 dois termos xr e xs no nulos e xr = xs.
PROBLEMA 3

Considere um tabuleiro quadrado n n, onde n um inteiro positivo par fixo. O tabuleiro est dividido em n2 quadrados unitrios. Dizemos que dois quadrados distintos do tabuleiro so adjacentes se eles tm um lado comum.
EUREKA! N6, 1999

Sociedade Brasileira de Matemtica

Marcam-se N quadrados unitrios do tabuleiro de tal forma que qualquer quadrado (marcado ou no) adjacente a pelo menos um quadrado marcado. Determine o menor valor possvel para N.

Adaptao da soluo de Humberto Silva Naves (Goinia - GO):


Vamos criar um algortmo para preencher estes quadrados n n . Quando pintarmos um dos quadradinhos temos que pintar pelo menos outro quadradinho adjacente.

Procedemos da seguinte maneira: Pintamos os dois quadradinhos do canto superior esquerdo (1,1) e (1,2). "Caminhamos" no sentido horrio no bordo do quadrado, de modo que pintamos 2 quadradinhos e pulamos outros 2. Isso possvel pois o nmero de quadradinhos no bordo mltiplo de quatro. Agora formamos outro quadrado menor de canto superior (3,3) e continuamos o mesmo procedimento para este quadrado,que tambm possui lado de medida par e assim sucessivamente. O nmero de quadrados pintados vai ser: N(n n) = 2n 2 + N((n 4) (n 4)) Este algortmo determina o menor N, pois cada quadradinho s possui um, e somente um "vizinho" pintado. Essa construo mostra por induo que possvel marcar obter uma soluo.
EUREKA! N6, 1999

n2 n + quadrados e 4 2

Sociedade Brasileira de Matemtica

Para provar que esse nmero de quadrados marcados o menor possvel associamos a cada quadrado coordenadas (x, y) com 1 x n e 1 y n. Se considerarmos os quadrados {(x, y) | (x e y so mpares e x + y 2 (mod 4)) ou (x mpar, y par e x + y n + 1 (mod 4))} verificamos que nenhuma pea adjacente a mais de um dos quadrados deste conjunto. Como este conjunto tem n2 n + elementos, necessitamos no mnimo esta quantidade de peas 4 2

Segundo Dia Durao da prova: 4 horas e 30 minutos


PROBLEMA 4

Determine todos os pares (n, p) de inteiros estritamente positivos tais que p primo, n 2p, e (p 1)n + 1 divisvel por np1.

Soluo de Humberto Silva Naves (Goinia - GO):


Analiando os casos: p = 2: temos que as nicas solues so para n = 1 e n = 2. p = 3: temos que as nicas solues so para n = 1 e n = 3. p >3: se n < p 1, temos: (Caso trivial n = 1 p divisvel por 1). n mpar, pois, (p 1)n + 1 mpar, logo n no pode ser par. Lema: Seja a > b 3, ento: ab < ba b a , mas isto obvio para a, < ln b ln a ln x 1 x crescente a partir de e. (t'(x) = 0 para b 3, pois a funo t ( x) = ln x (ln x) 2 Prova: Basta provar que: b ln a < a ln b x e.) Afirmao: Se p 1 > n 3 ( pois n mpar), ento:

( p 1) n < n p 1 ( p 1) n + 1 = n p 1 ou ( p 1) n + 1 < n p 1 . Como


EUREKA! N6, 1999

Sociedade Brasileira de Matemtica

( p 1) n + 1 n p 1 (pois ( p 1) n + 1 mltiplo de p e n
( p 1) + 1 < n . Portanto n no pode ser menor que p 1. Afirmao: Se n > p, ento n no primo:
n p 1

p 1

no , temos

Prova: se n for primo temos: ( p 1) n + 1 0(mod n) [a n a(mod n)] ( p 1) + 1 0(mod n) p 0(mod n) (um absurdo!). Logo, n composto: n = a b (a o menor fator primo de n). Ento: ( p 1) ab + 1 0(mod(ab) p1 ) ( p 1) ab + 1 0(moda p1 ) ( p 1) ab 1(moda p1 ) Seja K o menor natural tal que ( p 1) K 1(mod a p 1 ). Devemos ter K 2ab ( pois 2 ab = K q + r ( p 1)r 1 (mod ap1) r = 0). Alm disso devemos ter K (ap1) K ap2(a 1). Ento K = 2 ac, onde {0, 1} (pois se K tivesse um outro fator primo alm do 2 que dividisse (a 1), esse fator tambm teria que dividir 2ab que implica que esse fator dividiria ab, mas como o menor fator primo de n = ab o a, teriamos absurdo). K par, pois se no fosse teriamos K ab (p 1)ab 1 1 (mod ap1), um absurdo pois mpar. Logo K = 2 ac. Temos

( p 1) 2a 1(mod a p 1 ) (( p 1) a ) 2 1(mod a p 1 )
( p 1) a 1(mod a p 1 ) ( p 1) a 1(mod a ) p 1 1(mod a ), pois b a b(mod a )) p 0(mod a ), um absurdo pois p a, porque p < n < 2p). Logo no podemos ter n > p. As nicas possibilidades que restam n = p 1 ou n = p. n = p 1 falsa pois p 1 par. Se n = p temos: ( p 1) p + 1 0(mod p p 1 ) e como p > 3, temos: p p [ p p + ( ip ) p p 1 (1)1 + ... + ( 3 ) p 3 (1) p 3 + ( 2 ) p 2 (1) p 2 + (1p ) p1 (1) p 1 + (1) p ] +
c c

+ 1 0(mod p 3 ) p 2 0(mod p 3 ). (Um absurdo pois p3 no divide p2). Logo as solues so: S = {(1, 2); (2, 2); (1, 3); (3, 3)} { (1, p ) p primo}

EUREKA! N6, 1999

Sociedade Brasileira de Matemtica

PROBLEMA 5

Duas circunferncias 1 e 2 esto contidas no interior de uma circunferncia e so tangentes a em pontos distintos M e N, respectivamente. A

circunferncia 1 passa pelo centro de 2 . A reta que passa pelos dois pontos de interseo de 1 e 2 intersecta em A e B. As retas MA e MB intersectam 1 respectivamente em C e D. Prove que CD tangente a 2 .

Soluo de Pedro Paulo de Simoni Gouveia (Fortaleza - CE):


Considere a figura relativa ao problema:
1 M O 2 W Z N'' O2 N X 2 N' A

2 C

Sejam O e O2 os centros de 1 e 2 respectivamente: Tome uma inverso = inverso de centro em A e razo igual potncia do ponto A em relao a 1 e 2 (que a mesma pois A ao eixo radical de 1 e 2 ). Assim ( M ) = C e ( N ) = N '. Ento a imagem de pela inverso ser uma reta passando por C e N ', como tangente 1 e 2 CN ' tangente tambm

1 e 2 . Tome agora uma inverso com centro em B e razo igual potencia de B em relao 1 e 2 (B eixo radical).

( M ) = D e ( N ) = N ". Essa inverso leva na reta DN " que ser tangente


comum 1 e 2 .
EUREKA! N6, 1999

10

Sociedade Brasileira de Matemtica

Temos a seguinte situao.


Temos DW = ZC, e como DWC = DZC que DCZW um trapzio issceles AB // CD CD OO2 . Basta-nos provar agora que O2 D

______

W N'' O2

bissetriz de N " D C pois a, O2 ser o incentro do tringulo formado por D, C


N'

e o encontro de DN " e CN ' .

Prova:

N " D O2 = D O 1 O2 = 2 C O 1 O2 = 2 C D O2 = .
Logo N " O O2 = C D O2 = DO2 bissetriz 2 o incrculo 2 tangente a CD
PROBLEMA 6

Determine todas as funes f : R R tais que

f ( x f ( y )) = f ( f ( y )) + x f ( y ) + f ( x) 1 para quaisquer x, y R.
Soluo: Fazendo x = f ( z ), obtemos f ( f ( z ) f ( y )) = f ( f ( z )) + f ( f ( y )) + f ( z ) f ( y ) 1.
Fazendo x = f ( y ), obtemos f (0) = 2 f ( f ( y )) + f ( y ) 2 1, donde

f ( f ( y )) =

1 + f (0) f ( y ) 2 . Portanto, 2
1+ f (0) f (z) 2 1+ f (0) f ( y) 2 + + 2 2

f ( f (z) f ( y)) = f ( f (z)) + f ( f ( y)) + f (z) f ( y) 1 =

EUREKA! N6, 1999

11

Sociedade Brasileira de Matemtica

( f (z) f ( y))2 w2 . Provamos assim que f (w) = f (0) 2 2 sempre que w puder ser escrito como f ( z ) f ( y ). Vamos ver que qualquer w R pode ser escrito desta forma: f ( x f (0)) f ( x) = f ( f (0)) + xf (0) 1. Se tivssemos f (0) = 0 o lado esquerdo seria sempre 0 e o lado direito seria igual a 1, absurdo, portanto f (0) 0 e f ( f (0)) + xf (0) 1 pode assumir qualquer 1+ w f ( f (0)) valor real w (basta tomar x = ) , mas ento w = f ( z ) f ( y ) f (0) com z = x f (0) e y = x. + f (z) f ( y) 1 = f (0)
w2 para todo w R. 2 Em particular f ( f ( y )) = f (0) f ( y ) 2 / 2. Por outro lado

Mostramos assim que f (w) = f (0)

1 + f (0) f ( y)2 w2 1 + f (0) para , donde = f (0) f (0) = 1, e f (w) = 1 2 2 2 todo w R. fcil verificar que f satisfaz a condio do enunciado f ( f ( y)) =

Voc sabia

que o livro "Problemas de

las Olimpadas Matemticas del Cono Sur (I a IV)" j est a venda na Secretaria da Olimpada Brasileira de Matemtica? Informaes: Tel: 21-5295077 e-mail:obm@impa.br

EUREKA! N6, 1999

12

Sociedade Brasileira de Matemtica

14a. OLIMPADA IBEROAMERICANA DE MATEMTICA


12 a 19 de setembro, La Havana, Cuba O Resultado da equipe Brasileira
BRA1 BRA2 BRA3 BRA4 Christian Iveson Fabrcio Siqueira Benevides Fernando Paz Cardoso Rui Lopes Viana Filho PRATA BRONZE PRATA OURO

Primeiro Dia
PROBLEMA 1:

Encontre todos os inteiros positivos que so menores que 1000 e cumprem a seguinte condio: o cubo da soma dos seus dgitos igual ao quadrado do referido inteiro.
PROBLEMA 2:

Dadas duas circunferncias M e N, dizemos que M bissecta N se a corda comum um dimetro de N. Considere duas circunferncias fixas C1 e C2 no-concntricas. a) Prove que existem infinitas circunferncias B tais que B bissecta C1 e B bissecta C2. b) Determine o lugar geomtrico dos centros das circunferncias B. Sejam P1, P2,, Pn n pontos distintos sobre uma reta do plano (n 2). Consideram-se as circunferncias de dimetro Pi Pj (1 i < j n) e colorimos cada circunferncia com uma cor escolhida entre k cores dadas. Chamamos (n, k)-nuvem a esta configurao. Para cada inteiro positivo k, determine todos os n para os quais se verifica que qualquer (n,k)-nuvem contm duas circunferncias tangentes exteriormente da mesma cor. Nota: Para evitar ambiguidades, os pontos que pertencem a mais de uma circunferncia no so coloridos.
EUREKA! N6, 1999
_____

PROBLEMA 3:

13

Sociedade Brasileira de Matemtica

Segundo Dia
PROBLEMA 4:

Seja B um inteiro maior que 10 tal que cada um dos seus dgitos pertence ao conjunto {1, 3, 7, 9}. Demonstre que B tem fator primo maior ou igual a 11.
PROBLEMA 5:

Um tringulo acutngulo ABC est inscrito numa circunferncia de centro O. As alturas do tringulo so AD, BE, e CF. A reta EF intersecta a circunferncia em P e Q. a) Prove que OA perpendicular a PQ. b) Se M o ponto mdio de BC, prove que AP = 2 . AD. OM .
PROBLEMA 6:
2

Sejam A e B pontos do plano e C um ponto da mediatriz de AB.Constri-se uma sucesso C1, C2,, Cn ,., da seguinte maneira: C1 = C e, para n 1, se Cn no pertence ao segmento AB, Cn+1 o circuncentro do tringulo ABCn. Determine todos os pontos C tais que a sucesso C1, C2,, Cn,est definida para todo n e peridica a partir de um certo ponto. Nota: Uma sucesso C1, C2, , Cn, peridica a partir de um certo ponto se existem inteiros positivos k e p tais que C n + p = C n para todo
n k.

EUREKA! N6, 1999

14

Sociedade Brasileira de Matemtica

CONTAR DE DUAS MANEIRAS, PARA GENERALIZAR


Jos Paulo Q. Carneiro - Universidade Santa rsula

Nvel Intermedirio
1. O seguinte problema muito conhecido: quantas partidas h em um campeonato de futebol de um s turno com n times?
Uma maneira de raciocinar : cada time vai jogar n 1 partidas (com os outros n 1 times). Portanto, o nmero de partidas parece ser n(n 1) . Mas nessa contagem, cada partida foi contada duas vezes (por exemplo, o jogo Vasco Flamengo foi contado entre os jogos do Vasco e tambm entre os jogos do n(n 1) . Flamengo). Logo, o nmero correto : 2 Por outro lado, pode-se raciocinar tambm de outra maneira: O time T1 vai jogar

n 1 partidas. Colocando essas partidas de fora, o time T2 vai jogar n 2 partidas contra os clubes restantes (agora j estamos evitando a repetio desde o incio!). Alm das que j jogou nessa enumerao, o time T3 vai jogar n 3 partidas, e assim por diante. Ento o nmero total de partidas : (n 1) + (n 2) + + 1 .
(n 1)n , 2 que a clebre frmula da soma dos n 1 primeiros inteiros positivos. O interessante que, ao resolver um problema de contagem por dois mtodos diferentes, chegamos a uma frmula de carter geral. Comparando as duas contagens, conclui-se que: 1 + 2 + + ( n 1) = Deve ser notado tambm que esta frmula bsica para deduzir a frmula geral da soma dos termos de uma progresso aritmtica, pois: a1 + (a1 + r ) + ( a1 + 2r ) + + (a1 + (n 1)r ) = na1 + (1 + 2 + + (n 1))r =

n( a1 + a1 + (n 1)r ) n(a1 + a n ) (n 1) n r == = . 2 2 2 2. Vamos agora pensar no nmero de partidas de um torneio usual de tnis, onde todos os jogos so eliminatrios. Observe primeiramente o torneio do fim para o incio. A final jogada pelos 2 vencedores das 2 semifinais (jogadas por 4 jogadores), que vieram de 4 quartas de final (jogadas por 8 jogadores), e assim por diante. De um modo geral, um torneio desse tipo comea com 2 n jogadores, = na1 +
EUREKA! N6, 1999

15

Sociedade Brasileira de Matemtica

que jogam 2 n 1 partidas entre si de forma eliminatria. Na fase seguinte, sobram 2 n 1 jogadores, que jogam 2 n 2 partidas, e assim sucessivamente, at a final. O nmero total de partidas , portanto: 2 n 1 + 2 n 2 + + 2 + 1 . Vamos agora contar o nmero de partidas de outra forma. Cada partida tem um perdedor (no h empate em tnis). Por outro lado, como o torneio eliminatrio, todos os 2 n jogadores iniciais, exceto o vencedor final, perdem exatamente uma partida. Logo, o nmero total de partidas do torneio 2 n 1 . Comparando os resultados, conclui-se que 1 + 2 + + 2 n 2 + 2 n 1 = 2 n 1 .

A ltima frmula um caso particular da frmula da soma dos termos de a (1 q n ) . uma progresso geomtrica: a + aq + aq 2 + + aq n 1 = 1 q Ser que esta frmula tambm pode ser deduzida por um raciocnio anlogo aos anteriores? Para isto, imaginemos um torneio de tnis tambm eliminatrio, mas com q n jogadores iniciais (onde q um natural > 1 ), e onde, a partir de um ranqueamento prvio, os jogadores so grupados em q n 1 grupos (ou chaves) de q jogadores. Dentro de cada grupo, os jogadores previamente ordenados J 1 , , J q jogam entre si da seguinte forma: J 1 contra J 2 ; o vencedor desta partida joga contra J 3 ; e assim por diante, at J q . Portanto, dentro de cada grupo, h q 1 partidas, e como so q n 1 grupos, h, nesta primeira fase,

3.

(q 1) q n1

partidas. De cada grupo sai um vencedor, e os q n 1 vencedores so

novamente grupados em q n 2 grupos de q jogadores. O processo se repete, at que se atinge um nico grupo de q jogadores, e desse grupo sai o vencedor do torneio. O nmero total de partidas (q 1) q n1 + (q 1) q n 2 + + (q 1) . Por outro lado, por ser o torneio eliminatrio, pelo mesmo princpio anterior, o nmero total de partidas tambm igual ao nmero inicial de jogadores menos um, isto : q n 1. Comparando as duas contagens, conclui-se que: (q 1)(1 + q + q 2 + + q n 1 ) = q n 1 , que a frmula procurada.

EUREKA! N6, 1999

16

Sociedade Brasileira de Matemtica

Deve-se notar que esta deduo s vale para q natural > 1 . Mas a frmula obtida serve tambm como uma sugesto para qualquer q real (ou complexo!). Para comprovar a sugesto, basta multiplicar os dois fatores do lado esquerdo e simplificar, obtendo o lado direito (ou observer que dois polinmios cujos valores coincidem em infinitos pontos so necessariamente iguais).

4. curioso que se obtenham as frmulas de soma dos termos de progresses aritmticas e geomtricas atravs de contagens por dois mtodos diferentes, porm mais importante talvez o prprio mtodo, que muito fecundo e muito usado, por exemplo, em Anlise Combinatria. n S para dar um exemplo concreto, o smbolo k representa o nmero de subconjuntos de k elementos que se pode extrair de um conjunto com n elementos (tambm se usa C kn ), e pode ser calculado, como conhecido, por:

n n(n 1) 1 k = 1 2 k .
Suponha que se deseje calcular quantas comisses de p pessoas se podem formar a partir de um certo conjunto que consiste de n homens e n mulheres (supe-se 2n p n ). claro que este nmero igual a p . Por outro lado, se contarmos separadamente as comisses que so formadas por p homens, mais as que so formadas por p 1 homens e 1 mulher, mais as que so formadas por p 2 homens e 2 mulheres, etc., conclumos que este nmero tambm igual a n n n n n n p 1 ++ p . Da se deduz, portanto, a interessante 0 p + 1 0 frmula:

k = . p k p
k =0

n n

2n

Para dar um exemplo concreto, tomemos n = 6 e p = 3 , obtendo:


6 6 6 6 6 6 6 6 12 0 3 + 1 2 + 2 1 + 3 0 = 1 20 + 6 15 + 6 15 + 1 20 = 220 = 3 . Espero que o leitor tenha ficado estimulado a criar outros exemplos anlogos.

EUREKA! N6, 1999

17

Sociedade Brasileira de Matemtica

A MATEMTICA DAS ABELHAS


Jos Cloves Saraiva - UFMA

Nvel Avanado
As abelhas constrem colmias para reservar o mel e para o desenvolvimento e reproduo da sua prpria espcie. Cada alvolo tem a forma de um prisma reto hexagonal fechado numa das extremidades por uma cobertura de trs losngos de forma que o volume mximo enquanto a rea lateral total (incluindo a cobertura) fixada. Com isso as abelhas fazem o menor gasto de cera na construo das paredes peliculares que o constituem. Nesta construo econmica cada parede serve para dois alvolos contguos, o que motiva a escolha de prismas retos com seo transversal poligonal regular, como nas figuras abaixo:

fcil ver que as formas prismticas com seo transversal um polgono regular de n lados que podem ser usadas para cobrir o espa determinadas pela 360 condio de que um nmero inteiro positivo, onde In o ngulo interno In de um polgono regular.

360 360 2n 2n 4 + 4 4 =2+ = = = que In ( n 2) ( n 2) (n 2 )180 n 2 n um inteiro positivo se e s se n 2 um divisor de 4, logo devemos ter n = 3, n = 4 ou n = 6 como mostra a figura acima.
Podemos escrever

Por um clculo volumtrico simples das trs posibilidades acima observando o mesmo volume, ou escolhendo a forma mais convexa, das trs acima indicadas a escolha das abelhas pela forma hexagonal de fato a mais
EUREKA! N6, 1999

18

Sociedade Brasileira de Matemtica

econmica (dentre um tringulo, um quadrado e um hexgono regulares de mesmo permetro o hexgono tem a maior rea.)

Eis como as abelhas colocam os seus alvolos hexagonais. Esses alvolos, para maior economia de material , so fechados por trs losangos iguais. O valor constante do melhor ngulo agudo de um losango de fechamento causou srio debate entre telogos, naturalistas e matemticos.

O problema que as abelhas resolvem pode ser abordado, sem grande dificuldade, com os recursos da Matemtica elementar. Como mostra a figura acima, as abelhas fecham uma das bases do prisma com trs losangos congruentes cujo ngulo agudo do losango, , mede aproximadamente 7032', o que d origem ao problema do ngulo de fechamento da cobertura rmbica, que o seguinte:

De todas as coberturas rmbicas de um prisma reto hexagonal, a que faz volume mximo para rea lateral total fixada formada por trs losangos congruentes com ngulo agudo 7032'.
Para uma soluo deste problema observe primeiro a comparao do prisma hexagonal com a cobertura rmbica:
EUREKA! N6, 1999

19

Sociedade Brasileira de Matemtica

E D A C B

E r r A

BB ' = h
B'

____

B r r

D C

A figura tambm ajuda a obtermos as relaes mtricas necessrias para deduzir o volume V e a rea lateral total S em funo do raio r da base hexagonal e do ngulo , com vrtice no ponto D, e os itens seguintes so facilmente comprovados: 1) O trapzio ACDE retngulo com bases CD e AE perpendiculares ao plano que contm o tringulo equiltero ABC, de lado AB medindo r . 2) 3) 4)
____
____
____

____

EA = 2 DC
EA 4 ____ 2 ____ 2 BE ___ 2 EA 2 2 2 = BD sen = r + sen 4 2 4 2 BD = DE = r +
2 ____ 2 ____ 2

____

____ 2

5) 6)

____ 2

BE = EA + r (4r + EA ) sen = EA + r EA = r 3tg 2 1 2 2


___ 2 2 2 ___ 2 2 ___ 2 2 ___ 2

1/ 2

A rea lateral total S do prisma dada por:


1/ 2 3 r2 2 1 + rh S = 6 r 2 tg + 3tg 2 4 2 4

EUREKA! N6, 1999

20

Sociedade Brasileira de Matemtica

Usamos aqui que a rea de BDE


___ 2

BD DE sen = 2

EA 3 3 1 = (r 2 + )sen = r 2 (1 + tg 2 )sen = r 2 tg (pois sen = 4 2 8 2 2 4


7) O volume V do prisma dado por:

2tg 1 + tg

2 ).
2

1/ 2 3 3 2 r 2 1 + h r 3tg 2 2 2 Fixe V, e obtemos a seguinte expresso para S em funo de .

V=

1/ 2 3 2 2 4V 1 + S = r 3tg 3tg 2 2 2 3r

Observando a expresso acima, vemos que ela torna-se mnima se 1/ 2 2 1 for mnimo para valores de com a condio 3tg 3tg 2 2 3 tg . A seguinte desigualdade trigonomtrica, que provaremos a seguir, 2 3 garante o resultado do problema do ngulo (agudo) de fechamento da cobertura do prisma:

3 ento 3tg 3tg 2 1 2 3 2 2 2 s se tg = . 2 2


Se S = tg

1/ 2

2 , valendo a igualdade se e

Demonstrao: Para todo vale:

3 2 tg 1 0 6 tg 2 6 2tg + 3 0 2 2 2
Somando-se 3tg 2

1 aos dois membros da desigualdade obtemos:

EUREKA! N6, 1999

21

Sociedade Brasileira de Matemtica

9tg 2

6 2 tg

+ 2 3tg 2
1/ 2

1 3tg 2 3tg 2 1 2 2 2

3tg 2 3tg 2 1 2 2
3tg

, e como tg

3 , 3

2 > 0, logo:
1/ 2 1/ 2

2 3tg 2 1 3tg 3tg 2 1 2 2 2 2 2 Da, claro que, o menor valor de S determinado pela igualdade:
3tg
3tg tg

3tg 2 1 2 2
=

1/ 2

= 2 2 tg 1 = 0 2

2 2 = 2arc tg 7032' 2 2

Finalizando estas notas, sugerimos que o leitor faa a prova de que, o ngulo o mesmo ngulo agudo entre diagonais internas de um cubo.

Referncia:
Malba Tahan, As Maravilhas da Matemtica, Bloch Editores.

EUREKA! N6, 1999

22

Sociedade Brasileira de Matemtica

O TEOREMA DE RAMSEY
Carlos Gustavo T. de A. Moreira - IMPA

Nvel Avanado
1. O teorema de Ramsey para grafos. Vamos comear este artigo lembrando do exemplo 6 do artigo "O Princpio das Gavetas", de Paulo Cezar Pinto Carvalho, publicado na EUREKA! No. 5: se h 6 pessoas numa reunio ento h necessariamente 3 pessoas que se conhecem mutuamente ou 3 pessoas que no se conhecem mutuamente na reunio (onde admitimos que, se a conhece b, ento b conhece a). Este exemplo equivale ao seguinte: se tomamos 6 pontos, e pintamos cada segmento que une dois desses pontos de preto ou vermelho ento necessariamente existe um tringulo cujos vrtices so trs desses pontos e cujos 3 lados so da mesma cor.
Nos exerccios 8 e 9 do mesmo artigo proposta uma generalizao:

Proposio 0: Dados os inteiros a, b 2 sempre existe um nmero N inteiro positivo tal que, em qualquer conjunto de N pessoas, sempre existem a pessoas que se conhecem mutuamente ou b pessoas que se desconhecem mutuamente. Em homenagem a F. P. Ramsey, que provou este e outros resultados deste artigo, chamamos o menor nmero N com esta propriedade de R(a, b).
Vamos provar este resultado seguindo os passos propostos do problema 9 do artigo do Prof. Paulo Cezar: i) R(a, 2) = a para todo a 2. De fato, num conjunto de a pessoas, ou todas se conhecem ou existem duas que se desconhecem. R(a, b) = R(b, a). De fato, conhecer e desconhecer desempenham um papel simtrico no enunciado.

ii)

iii)

R( a, b) R (a 1, b) + R (a, b 1), para a, b 3.

Para provar isto, fixemos uma pessoa P. Se temos R( a 1, b) + R( a, b 1) ou mais pessoas na reunio, h pelo menos R (a 1, b) + R( a, b 1) 1 outras pessoas, e um dos seguintes casos se verificar:
EUREKA! N6, 1999

23

Sociedade Brasileira de Matemtica

1)

P conhece pelo menos R( a 1, b) pessoas. Neste caso, por definio de R (a 1, b) , dentre essas R (a 1, b) pessoas h b que se desconhecem mutuamente (e no temos mais o que provar), ou a 1 que se conhecem mutuamente, e, juntando P a essas a 1 pessoas, obtemos a pessoas que se conhecem mutuamente. P desconhece pelo menos R(a, b 1) pessoas. A anlise deste caso anloga do caso anterior, trocando os papeis de conhecer e desconhecer no argumento

2)

J mencionamos que podemos enunciar resultados deste tipo em termos de coloraes de segmentos (ou mais tecnicamente, de arestas de grafos completos) usando duas cores. O resultado acima pode ser generalizado aumentando o nmero de cores:

Proposio 1: Dados inteiros k 1, a1 , a 2 ,...a k 2 existe n N tal que dados n pontos, se pintarmos cada segmento que une dois desses pontos de uma dentre k cores possveis ento haver a1 pontos tais que todo segmento que une dois desses pontos da primeira cor, ou a2 pontos tais que todos os segmentos que unem dois desses pontos so da segunda cor, ou ou ak pontos tais que todos os segmentos que unem dois desses pontos so da k-sima cor. Chamamos de R( a1 , a 2 ,...a k ) o menor nmero com essa propriedade. Demonstrao: Esta proposio pode ser provada de modo anlogo anterior. A existncia dos nmeros R (a1 ,...a k ) segue dos seguintes fatos, cuja prova deixamos como exerccio:
i) ii)

R( a1 ,...a k 1 , 2) = R(a1 ,...a k 1 ) para a1 ,...a k 1 2. R (a (1) , a ( 2) ,...a ( k ) ) = R( a1 , a 2 ,...a k ) para a1 ,...a k 2
permutao de {1, 2, , k}.

e qualquer

iii)

R( a1 ,...a k ) R (a1 1, a 2 ,..., a k ) + R( a1 , a 2 1, a 3 ,..., a k ) + ... + + R (a1 , a 2 ,...a k 1 , a k 1) k + 2.

EUREKA! N6, 1999

24

Sociedade Brasileira de Matemtica

2. Estimativas de nmeros de Ramsey. As demonstraes dos resultados anteriores fornecem estimativas superiores para os nmeros de Ramsey R( a, b) e R( a1 ,...a k ) , por exemplo:
i)

( a + b 2)! , para a, b 2 (a 1)!(b 1)! De fato vale a igualdade para b = 2 (temos os dois lados iguais a a), e se a, b > 2 temos, por induo,
a 1 R ( a, b ) C a +b 2 = a2 a 1 a 1 R( a, b) R (a 1, b) + R (a, b 1) C a + b 3 + C a + b 3 = C a + b 2

ii)

(a1 + ... + ak 2k + 2)! a1 1,a2 1,a3 2,...ak 2 =: Ca 1 +...+ak 2k +2 (a1 1)!(a2 1)!(a3 2)!(a4 2)!...(ak 2)! De fato, isso vale quando todos os ai exceto dois so iguais a 2, e o caso geral segue, por induo da identidade R(a1 ,...,ak )
b1 ,...,bk b1 1,b2 ,...,bk b1 ,b2 1,...,bk , k Cb = Cb + Cb + ... + C b11+...+kbk1 1 1 + b2 +...+ bk 1 +...+ bk 1 1 +...+ bk 1 b ,...,b b 1

menos trivial, entretanto, dar boas estimativas inferiores para nmeros de Ramsey. Para provar, por exemplo, que R(3,3) = 6, necessrio mostrar que h exemplos de grafos completos bicoloridos de 5 vrtices sem tringulos monocromticos, o que pode ser feito explicitamente:

Se o nmero de pontos, cresce, entretanto, bastante difcil construir exemplos explcitos. As melhores estimativas conhecidas para R(k, k) se devem ao mtodo probabilistico introduzido pelo grande matemtico hngaro Pl Erds, que se tornou uma das tcnicas mais poderosas da teoria dos grafos:

Proposio 2: R(k, k) > 2k/2 para todo k N. Demonstrao: Dados n pontos, pintamos aleatoriamente as arestas que ligam dois desses pontos de vermelho ou preto, com probabilidade 1/2. Dado um k (k 1) arestas subconjunto de k desses pontos, a probabilidade de que todas as 2
EUREKA! N6, 1999

25

Sociedade Brasileira de Matemtica

1 que unem dois desses pontos sejam da mesma cor 2 . Como h 2 n! k Cn subconjuntos de k pontos do conjunto inicial de n pontos, a = k!(n k )! probabilidade de que em algum deles todas as arestas sejam da mesma cor no 2 nk 1 k 1 mximo 2C n que menor ou igual a se 2 k ! 2 k ! 2 n 2 k / 2 , mas 2 k / 2+1 < k! para k 4, donde h probabilidade positiva de que em nenhum subconjunto de k pontos todas as arestas sejam da mesma cor, e em particular h exemplos desta situao, donde R(k, k) necessariamente maior que 2k/2 para k 4.
Para k = 3, R(k, k) = 6 > 23/2, e para k =2, R(k, k) = 2 > 21/2, o que conclui a demonstrao Vamos discutir agora como estimar a funo f (n) = R(3,3,...,3), com n termos iguais a 3, ou seja, o menor nmero de vrtices de um grafo completo tal que ao pintarmos suas arestas usando n cores necessariamente obtemos um tringulo monocromtico. A demonstrao da proposio 1 nos fornece f (n) nf ( n 1) n + 2, o que implica, por exemplo, f (n) 3n! para todo n 2. (lembremos que f (3) = R(3,3) = 6). Suponha agora que sejam dados 2,
k ( k 1) / 2 k ( k 1) / 2
k / 2 +1

k ( k 1) / 2

3n + 1 pontos, aos quais atribumos ndices 0, 1, 2

3n 1 . Vamos descrever uma forma de, a cada par desses pontos, atribuir 2 um nmero entre 0 e n 1 (o que equivale a colorir as arestas do grafo completo 3n + 1 vrtices usando n cores), sem que haja 3 pontos tais que a cada par de 2 desses pontos atribuido o mesmo nmero (ou seja, sem que haja tringulos monocromticos).
Ao par de pontos de ndices i e j atribumos um nmero da seguinte maneira: escrevemos i j como

r =0

n 1

3 r onde r {1,0,1} para todo r, e atribuimos a

{i, j} o menor k com k = 1 (sempre existe um tal k pois i j > 0).


EUREKA! N6, 1999

26

Sociedade Brasileira de Matemtica

Deixamos como exerccio para o leitor verificar que no h tringulos monocromticos nesta configurao (ver problema proposto No. 32 pg 55).

3n + 1 . 2 Determinar exatamente os valores de nmeros de Ramsey clssicos R(a, b) com 3 a b que so conhecidos so: R(3, 3) = 6, R(3, 4) = 9, R(3, 5) = 14, R (3, 6) = 18, R(3, 7) = 23, R(3, 8) = 28, R (3, 9) = 36, R(4, 4) = 18, e R(4, 5) = 25. O nico nmero Ramsey com mais de duas cores cujo valor conhecido R(3, 3, 3) = 17.
Obtemos assim a estimativa f (n) >

3. O Teorema de Ramsey para multigrafos: At agora estivemos falando sobre coloraes de arestas de grafos, ou seja, a cada conjunto de dois vrtices associamos uma cor. Uma maneira de generalizar este resultado associar cores no a pares de vrtices, mas a conjuntos de m vrtices, onde m um inteiro positivo fixo (que pode ser maior que 2). Como as configuraes que aparecem so um pouco mais complicadas, vamos introduzir notaes um pouco mais formais:
Dado um conjunto A e um inteiro positivo k denotamos por [A]m o conjunto dos subconjuntos de m elementos de A, ou seja [ A] m = {B A # B = k} . Dado j inteiro positivo, definimos Ij = {1, 2, , j}. A verso m-dimensional (ou para m-hipergrafos) do teorema de Ramsey dada pelo seguinte teorema (do qual os resultados da seo 1 so casos particulares):

Teorema (Ramsey): Sejam m, k inteiros positivos. Dados a1, a2,, ak inteiros positivos existe um inteiro positivo, que denotaremos por Rm (a1 ,..., a k ) tal que
para todo n Rm (a1 ,..., a k ) e para qualquer funo f : [ I n ] m I k existem j I k e A I n com # A = a j tal que f ([ A] m ) := { f ( x), x [ A] m } { j}.

Demonstrao: Para m = 1 o resultado uma aplicao simples, do princpio das gavetas: basta tomar R1 (a1 ,..., a k ) = a1 + ... + a k k + 1. Vamos provar o resultado geral por induo em m. De fato, provaremos que podemos tomar Rm (a1 ,..., ak ) 1 + Rm1 ( Rm (a1 1,..., a k ), Rm (a1 , a2 1..., ak ),..., Rm (a1 , a2 ,...ak 1)) (e se algum dos ai menor que m podemos tomar Rm (a1 ,..., a k ) = min{a1 , a 2 ,..., a k }). Note que isto fornece exatamente a recurso de proposio 1 da seo 1 no caso m = 2.
EUREKA! N6, 1999

27

Sociedade Brasileira de Matemtica

Se n 1 + Rm 1 ( Rm (a1 1, a 2 ,..., a k ),..., Rm ( a1 , a 2 ,...a k 1)), dada uma funo

f : [ I n ] m I k , definimos uma funo f : [ I n 1 ] m 1 I k da seguinte forma:


dado A [ I n 1 ] m 1 , definimos f ( A) := f ( A {n}). Como n 1 Rm 1 ( Rm (a1 1,..., a k ),..., Rm (a1 ,...a k 1)), existem j I k e B I n 1 com # B = Rm (a1 ,..., a j 1,..., a k ) tal que f ([ B] m 1 ) = { j}. Agora, por definio de
~

Rm ( a1 ,..., a j 1,..., a k )
m

existe

i Ik

com

i j

A B com # A = a i e

f ([ A] ) = {i}, caso em que j conseguimos o que

queramos, ou existe A B com # A = a j 1 e f ([ A] m ) = { j}. Nesse caso, teremos # ( A {n}) = a j e conseguimos nosso objetivo .

f ([ A {n}] m ) = { j}, pois A B , e novamente

Obs.: O nico nmero de Ramsey cujo valor conhecido com m > 2 (e a j > m, j ) R3 (4,4) = 13 .
O exerccio 20 de [PC] pede para mostrar que dados 5 pontos no plano em posio geral h 4 que formam um quadriltero convexo. O leitor poderia perguntar: o que faz um problema geomtrico como este num artigo de combinatria? A resposta est ligada a uma generalizao deste resultado, descoberta por Erds e Szekeres: Dado um inteiro positivo n 4 existe um inteiro positivo f(n) tal que dados f(n) pontos no plano em posio geral h n deles que so vrtices de um n-gono convexo. Para provar isso, mostraremos que podemos tomar f (n) = R4 (n,5). Para isso, dados R4 (n,5) pontos no plano, e um conjunto de 4 desses pontos, associamos a esse conjunto o nmero 1 se eles formam um quadriltero convexo, e 2 em caso contrrio.No possvel que haja 5 pontos tais que a cada 4 deles associado o nmero 2, pelo resultado do exerccio 20 de [PC], donde, por definio de R4 (n,5) , necessariamente h n desses pontos tais que cada 4 desses pontos formam um quadriltero convexo, mas isso implica que esses n pontos so vrtices de um n-gono convexo.

5. O teorema de Ramsey infinito: Teorema: Sejam m, k inteiros positivos e A um conjunto infinito. Para qualquer funo f: [ A] m I k existem j I k e um conjunto infinito
EUREKA! N6, 1999

28

Sociedade Brasileira de Matemtica

B A tal que f ([ B ] m ) = { f ( x) x [ B ] m } = { j}

Demonstrao: Vamos provar o resultado por induo em m. Para m = 1 o resultado segue do fato de que se X infinito e C finito ento para toda funo f := X C existe C C tal que f 1 (c) = {x X f ( x) = c} infinito.
Seja agora m 2 e f : [ A] m I k , onde A infinito.Fixamos xo A, e definimos

Ao = A \ {xo } e g : [ Ao ] m 1 I k por g (C ) := f (C {xo }), onde C um subconjunto de m 1 elementos de Ao . Pela hiptese de induo existe um conjunto infinito Bo Ao e j o I k tal que g o ([ Bo ] m 1 ) = { j o }. A partir da repetimos o processo recursivamente: dado n 0 fixamos xn +1 Bn e definimos An +1 = Bn \ {xn +1} e g n +1 = [ An+1 ] m 1 I k por g n +1(C ) = f (C {x n +1 }) para C An +1 com m 1 elementos. Pela hiptese de induo existe Bn +1 An +1
infinito e j n +1 I k tal que g ([ Bn+1 ] m 1 ) = { j n+1 }. Podemos agora tomar D = {x0, x1, x2,}, que um conjunto infinito e definir h : D I k por h( x r ) = j r . Como Ik finito, existe jIk tal que

h 1 ( j ) = {x D h( x) = j} infinito. Afirmamos que B = h 1 ( j ) satisfaz a


condio do enunciado. De fato, dado um subconjunto X = {xi1 , xi2 ,.., xim } de B com m elementos, temos f ( X ) = g i1 ({xi2 ,..., xim }) = j i1 = h( xi1 ) = j )

Sugesto: Tente usar os resultados deste artigo para resolver o problema Cuticoda Eureka! No5. Pg. 58. (ver tambm pg. 56 desta EUREKA!).

Referncias:
Ramsey, F.P., On a Problem of Formal Logic, Proc. London Math. Soc. 30 (1930) P P. 264-286. -[PC]- Paulo Cezar Pinto Carvalho, Princpio das Gavetas Eureka! 5 PP.27-33. -[GRS]- R.L. Graham, , B.L. Rothscild, e J.H. Spencer, Ramsey Theory. Wiley. Interscience, 1990. -[ES]- Erds e G. Szekeres A Combinatorial Problem in Geometry Compositio Math.2 (1935), PP-464-470. -[Rad]- Stanislaw P. Radziszowski Smal Ramsey Numbers Dinamic SurveysElectronic Journal of Combinatorics. http://www.combinatorics.org -[Er]- Pl Erds, Some Remarks on The Theory Of Graphs, Bull. Amer. Math. Soc. 53 (1947), PP.292-294.
EUREKA! N6, 1999

-[R]-

29

Sociedade Brasileira de Matemtica

APLICAES DOS NMEROS COMPLEXOS GEOMETRIA


Edmilson Motta - Colgio Etapa

Nvel Avanado
importante ter em mente que os nmeros complexos no so apenas vetores; eles podem ser multiplicados. Nas aplicaes Geometria, ns faremos uso extensivo desta propriedade. Nmeros Complexos so particularmente eficientes para certos tipos de problemas, mas podem gerar dificuldades artificiais em problemas que admitem solues mais diretas utilizando outros mtodos. Na Geometria Elementar, os tringulos so as peas bsicas e a congruncia e a semelhana de tringulos, os conceitos fundamentais. Ns comearemos caraterizando a semelhana de tringulos em termos de nmeros complexos. Inicialmente, vamos estabelecer algumas convenes. Sejam z1, z2, z3, w1, w2, w3 nmeros complexos. Ns dizemos que z1z2z3 e w1w2w3 so semelhantes, e escrevemos z1 z2 z3 ~ w1w2w3, se e somente se, o ngulo em zk igual ao ngulo em wk , k = 1, 2, 3, e tm mesma orientao, isto , ambos anti-horrios ou ambos horrios (Veja figura a seguir.)
z1 w3 w2

z3 w1 z2

Se os tringulos tem orientaes distintas (um horrio, o outro antihorrio), ento escrevemos

z1 z2 z3 ~ w1w2w3 (reverso)


Como para complexos , e distintos,

EUREKA! N6, 1999

30

Sociedade Brasileira de Matemtica

arg

= arg ( ) arg ( ) = medida do ngulo orientado entre

e , ento , , so colineares a medida do ngulo orientado entre


e mltipla de

. = = R __ __
Exerccio 1:

___________

__

__

Mostre que imaginrio puro = 0. + __ __ Generalizando, para quatro pontos distintos , , , C,


__

__

, || = R __ __

__

__

alm disso, e tem a mesma direo se, e somente se,


um real
__

positivo; imaginrio puro = 0. + __ __


Exerccio 2:
Mostre que, se 0, ento + = +

__

> 0.

z1 w1 1 z 2 z1 w2 w1 Teorema: z1 z2 z3 ~ w1w2w3 = = z 2 w2 1 = 0. z 3 z1 w3 w1 z 3 w2 1 Demonstrao: Dois tringulos so semelhantes se, e somente se, (caso LAL) as razes entre as medidas de dois pares de lados correspondentes so iguais e os ngulos entre estes lados so iguais (incluindo a orientao). Assim, z z1 w2 w1 z z1 w w1 z1 z2 z3 ~ w1w2w3 2 = e arg 2 = arg 2 z 3 z1 w3 w1 z 3 z1 w3 w1
EUREKA! N6, 1999

31

Sociedade Brasileira de Matemtica

z1 w1 1 z 2 z1 w2 w1 = z 2 w2 1 = 0. z 3 z1 w3 w1 z 3 w3 1
Corolrio: z1 z2 z3 ~ w1w2w3 (reverso) z 2 z 1 = w 2 w1 z __ 2 w 2 1 = 0. __ __
z 3 z1
__
__ __

z1 w 1 1
__

__

w 3 w1

z3 w 3 1

Demonstrao: Temos que w1 w2 w3 ~ w1 w2 w3 (reverso). Logo z1 z2 z3 ~ w1w2w3 (reverso) z1 z2 z3 ~ w1 w2 w3 Exerccio 3:
Complete a demonstrao acima, verificando que w1 w2 w3 ~ w1w2w3 (reverso). Vejamos agora alguns exemplos de aplicaes mais especficas. CARACTERIZAES DOS TRINGULOS EQUILTEROS. Sendo 2 + + 1 = 0,
__ __ __ __ __ __

__ __

z1 z 3 1
z1 z2 z3 equiltero z1 z2 z3 ~ z3 z1 z2 z 2 z1 1 = 0

z3 z 2 1 z12 +
2 z2

2 z3

z 2 z 3 z 3 z1 z1 z 2 = 0

( z1 + z 2 + 2 z 3 ) ( z1 + 2 z 2 + z 3 ) = 0 ( z1 + z 2 + 2 z 3 ) = 0 ou ( z1 + 2 z 2 + z 3 ) = 0 z1 1 1 z1 1 1 z 2 1 = 0 ou z 2 2 1 = 0 z3 2 1 z3 1
2

z1 z2 z3 ~ 1 ou z1 z2 z3 ~12


EUREKA! N6, 1999

32

Sociedade Brasileira de Matemtica

(geometricamente, esta ltima caracterizao bastante intuitiva).

x 0 1

O TEOREMA DE NAPOLEO. Sobre cada lado de um tringulo arbitrrio, desenhe um tringulo equiltero (no exterior). Temos ento que os baricentros desses trs tringulos equilteros so os vrtices de um quarto tringulo equiltero.
w3 3 z1 2 w2

z3 z2 1

Demonstrao: Sejam z1z2z3 o tringulo dado; w1z3z2, z3w2z1, z2z1w3 tringulos equilteros com a mesma orientao que 12, digamos; e 1, 2, 3, os baricentros desses tringulos. Ento w1 + z 3 + 2 z 2 = 0

w1

z 3 + w 2 + 2 z1 = 0 z 2 + z1 + 2 w3 = 0
Para provarmos que 123 equiltero, calculamos
EUREKA! N6, 1999

33

Sociedade Brasileira de Matemtica

1 + 2 + 2 3 = ( w1 + z 3 + z 2 ) +
=

1 3

( z 3 + w 2 + z1 ) +

2
3

( z 2 + z1 + w3 )

1 ((w1 + z 3 + 2 z 2 ) + ( z 3 + w2 + 2 z1 ) + ( z 2 + z1 + 2 w3 )) = 0. 3 Portanto123 um tringulo equiltero.


Exerccio 4: Sendo , , , nmeros complexos, temos que ( ).( ) + ( ) ( ) = ( ) ( ) (a) A partir da identidade acima, mostre que + . (b) Demonstre o teorema de Ptolomeu-Euler:
Para quaisquer pontos A, B, C, D no plano, AB CD + BC DA AC BD, com igualdade se, e somente se, estes quatro pontos so concclicos ou colineares.

Exerccio 5: (a) Sejam a, b, c, d, e, f nmeros complexos. Prove que

a b c d e f bc a e f d . =1 = 1. c b e d a f a c f e bd (b) Utilizando o item anterior, resolva o seguinte problema do banco da IMO 98: AB CD EF = 1. Seja ABCDEF um hexgono convexo tal que B + D + F = 360 e BC DE FA BC AE FD = 1. Prove que CA EF DB
MAIS EXEMPLOS PROBLEMA (BANCO / IMO 98)

Seja ABC um tringulo, H o seu ortocentro, O o seu circuncentro e R o seu circunraio. Seja D o simtrico de A com relao a BC, E o simtrico de B com relao a AC e F o simtrico de C com relao a AB. Prove que D, E e F so colineares se, e somente se, OH = 2R.

EUREKA! N6, 1999

34

Sociedade Brasileira de Matemtica

RESOLUO:

Sejam a, b, c, h e 0 as coordenadas complexas do A, B, C, H e O, respectivamente. Conseqentemente, a a = b b = c c = R2 e h = a + b + c. Como D o simtrico de A com relao a BC, d satisfaz
__ __ __ __ __ d b a b ( ) b c d b c a + = b c b c = 0. c b c b
_________

__

__

__

(1)

Temos que
__ __

b c =

__ __ R2(b c) R2(b2 c2) e b c b c = , bc bc

substituindo em (1), obtemos

bc+ ca+ ab k 2bc = , a a R2(a + b + c) R2(h 2a) d= = , bc bc d=


onde k = bc + ca + ab. Analogamente k 2ca __ R2(h 2b) k 2ab __ R2(h 2c) e= , e = ,f = e f= . b ca c ab Como

(b a)(k 2ab) R2(a b)(h 2c) __ e d e d ab abc = e e 1 = = __ __ 2 (c a)(k 2ca) R (a c)(h 2b) __ f d f d f f 1 ca abc
__ __

d d1

__

=
=
__

) (h 2c) R2(c-a )(a b) (ck-2abc 2 2 2 (bk 2abc ) (h 2b) abc

R 2 (b c)(c a )(a b)(hk 4abc) a 2b 2c 2

, segue que D, E e F so colineares e h = R2k / abc

EUREKA! N6, 1999

35

Sociedade Brasileira de Matemtica

=0 hk 4abc= 0 h h = 4R2 OH = 2R.


PROBLEMA (Olimpada Chinesa 98)
__

Seja D um ponto no interior de um tringulo acutngulo ABC, com DA DB AB + DB DC BC + DC DA CA = AB BC CA. Determine quais so as possveis posies que D pode ocupar.
RESOLUO:

Sejam a, b, c, e 0 as coordenadas complexas de A, B, C e D, respectivamente. Temos, ento que DA DB AB + DB DC BC + DC DA CA = AB BC CA a b (b a) + b c (c b) + c a (a c) = (b a)(c b)(a c) (*) Como ab(b a ) + bc(c b) + ca(a c) = (b a)(c b)(a c) , sendo w1 = ab(b a ),

w2 = bc(c b), w3 = ca( a c), (*) w1 + w2 + w3 = w1 + w2 + w3 e portanto, w1, w2, w3 esto alinhados. Assim, existem reais positivos e tais que w1 = w2 ab(b a ) = bc(c b) a (b a ) = c(c b) bc a , = ac b w1 = w3 ab(b a ) = ca( a c) b(b a) = c(a c)
isto , A C B = 180 A DB e, analogamente, A B C = 180 A DC e

B A C = 180 B DC. O nico ponto D no interior de um tringulo acutngulo que satisfaz essas condies o ortocentro.
PROBLEMA (Olimpada Universitria Hngara 1995)

So dados n pontos na circunferncia unitria de modo que o produto das distncias de qualquer ponto da circunferncia a estes pontos menor ou igual a 2. Prove que os pontos so vrtices de um ngono regular.
RESOLUO:

Considere a circunferncia centrada na origem e sejam z1, z2, , zn os nmeros complexos que representam os pontos. Podemos assumir que (1) n z1 z 2 ...z n = 1 (verifique! ).
EUREKA! N6, 1999

36

Sociedade Brasileira de Matemtica

Considere ainda o seguinte polinmio p( w) = ( w z1 )( w z 2 )...(w z n ) = wn + a1 wn1 + ... + an1w + 1 = wn + Q (w) + 1 Ento |p(z)| o produto das distncias do ponto representado pelo nmero complexo z aos pontos dados . Logo, se z um nmero complexo de mdulo 1, ento | p(z)| 2. Sejam w1, w2, wn as razes n-simas da unidade. k k Sabe-se que w1k + w2 + ... + wn = 0 para todo k = 1, 2,,n 1. Portanto Q( w1 ) + Q( w2 ) + ... + Q( wn ) = 0. Se Q(w) no identicamente nulo, ento, para algum j, Q (wj) diferente de zero e tem parte real no negativa, pois Q(0) = 0 e Q tem no mximo n 1 razes. Consequentemente, p ( w j ) = 2 + Q ( w j ) > 2 , uma contradio. Desta forma o polinmio Q identicamente nulo e p(z) = zn + 1. As razes z1, z2, , zn do polinmio p(z) formam um n-gono regular.

Mais alguns exerccios relacionados.


PROBLEMA 1 (IMO 63)

Todos os ngulos internos de um n-gono so iguais e seus lados satisfazem a relao a1 a 2 ... a n . Prove que a1 = a 2 = ... = a n .
PROBLEMA 2 (Treinamento para IMO 92)

Prove que para todo inteiro positivo n, (2 + i ) n (2 i ) n . Conclua que os ngulos agudos do tringulo de lados 3, 4 e 5 so irracionais quando expressos em graus.
PROBLEMA 3 (IMO 75)

Determine se existem ou no 1975 pontos sobre a circunferncia unitria tais que a distncia entre quaisquer dois um nmero racional.
PROBLEMA 4 (Olimpada Romnia 92)

Sejam p, q C, q 0. Se as razes da equao x 2 + px + q = 0 tm o mesmo p um nmero real. mdulo, mostre que q

EUREKA! N6, 1999

37

Sociedade Brasileira de Matemtica

A rea do polgono A1 A2 ... An S. So dados um ngulo e um ponto Q. Rodemos Q de um ngulo no sentido anti-horrio ao redor de Ai para encontrar um ponto Pi. Ache a rea de P1 P2 ...Pn .
PROBLEMA 6 (Putnam 67)

PROBLEMA 5 (Seleo para IMO 92)

Seja ABCDEF um hexgono inscrito em uma circunferncia de raio r. Mostre que se AB = CD = EF = r, ento os pontos mdios de BC , DE , e FA so os vrtices de um tringulo equiltero. So dados um tringulo A1 A2 A3 e um ponto P0 no plano. Definimos As = As 3 para todo s 4 e construmos uma seqncia de pontos P0 , P1 , P2 ,... tais que Pk +1 a imagem de Pk sob a rotao com centro Ak +1 e ngulo de 120 (sentido horrio). Prove que se P1986 = P0 , ento o tringulo A1 A2 A3 equiltero.
PROBLEMA 8 (Olimpada Hungria 1899) PROBLEMA 7 (IMO 86)
____ ____ ___

A0 , A1 , A2 , A3 , A4 dividem a circunferncia unitria em cinco partes iguais. Prove


que ( A0 A1 A3 A2 ) 2 = 5.
PROBLEMA 9 (Putnam 55)

A1 , A2 ,... An um polgono regular inscrito em uma circunferncia de raio r e


centro O. P um ponto sobre OA1 . Mostre que
PROBLEMA 10

PA
k =1

= OP n r n .

Dados um ponto P sobre uma circunferncia unitria e os vrtices A1 , A2 ,... An de um n-gono regular inscrito, prove que 2 2 2 4 4 4 PA n e PA n 1 + PA 2 + ... + PA 1 + PA 2 + ... + PA so constantes.

Bibliografia:
Complex numbers & Geometry, The Mathematical Association of America Liang - Shin Hahn, 1994.

EUREKA! N6, 1999

38

Sociedade Brasileira de Matemtica

40a. OLIMPADA INTERNACIONAL E 14a. OLIMPADA IBEROAMERICANA DE MATEMTICA


Segundo teste de Seleo
PROBLEMA 1

Encontre todos os inteiros positivos l, m, n, primos dois a dois, tais que (l + m + n)(1/l + 1/m + 1/n) um inteiro positivo.
PROBLEMA 2

Considere ABC inscrito em uma circunferncia fixa. Sejam D, E e F pontos arbitrrios distintos dos vrtices, pertencentes aos lados BC, CA e AB , respectivamente. Sejam P, Q e R os pontos de interseco das retas AD, BE e CF com a circunferncia, respectivamente. Mostre que AD BE CF + + 9. PD QE RF Determine tambm as condies sobre o ABC e os pontos D, E e F para que a igualdade ocorra. Para n natural, seja (n) a quantidade de nmeros naturais que so primos com n e (n) o nmero de fatores primos de n. Mostre que se (n) um divisor de n 1 e (n) 3, ento n primo.
PROBLEMA 4 PROBLEMA 3
____ ____ ___

Para reais positivos satisfazendo a + b + c = abc, mostre que 1 1 1 3 + + , e determine quando a igualdade ocorre. 1+ a2 1+ b2 1+ c2 2 Seja I o incentro de ABC, O1 uma circunferncia passando por B e tangente reta CI e O2 uma circunferncia passando por C e tangente reta BI. Mostre que a circunferncia circunscrita ao ABC e as circunferncias O1 e O2 interceptam-se em um nico ponto.
PROBLEMA 6 PROBLEMA 5

Seja Fn o conjunto de todas as bijees f de {1, , n} em {1, , n} satisfazendo f (k) k +1 para k = 1, 2, n f (k) k para k = 2, , n. (i) e (ii) Determine a probabilidade de que f (1) 1 para um f arbitrrio em Fn
EUREKA! N6, 1999

39

Sociedade Brasileira de Matemtica

10a. OLIMPADA DE MATEMTICA DO CONE SUL


Primeiro teste de Seleo
PROBLEMA 1

Em um tabuleiro 1999 1999 encontra-se um certo nmero de torres (torre uma pea que se move horizontalmente ou verticalmente). Prove que possvel colorir as torres utilizando trs cores de modo que nenhuma torre ataque outra de mesma cor (uma torre ataca outra quando ambas esto na mesma linha ou coluna sem peas intermedirias).
PROBLEMA 2

Encontre todas as solues reais de x [x [x [x]]]= 88, onde [x] o inteiro satisfazendo [x] x < x +1 (por exemplo, [3, 7] = 3, [4] = 4 e [] = 4.
PROBLEMA 3

A bissetriz do ngulo B em um tringulo ABC intercepta o lado AC no ponto D. Seja E um ponto sobre o lado BC tal que 3C AE = 2B AE. Os segmentos BD e AE interceptam-se no ponto P. Se ED = AD = AP, determine os ngulos do tringulo.
PROBLEMA 4

Mostre que h infinitos naturais n tais que n2+1 divide n!, onde n!= n (n 1)...21 por exemplo, 4!= 4 3 21 = 24).
PROBLEMA 5

Considere um polgono convexo com 2000 lados no plano. Prove que possvel escolher 1998 pontos no plano tais que qualquer tringulo formado por vrtices do polgono tenha exatamente um dos pontos escolhidos em seu interior.

EUREKA! N6, 1999

40

Sociedade Brasileira de Matemtica

10a. OLIMPADA DE MATEMTICA DO CONE SUL


Segundo teste de Seleo
PROBLEMA 1

Sejam p, q, r, s inteiros no negativos tais que ( p + q)2 + q = (r + s)2 + s Prove que p = r e q = s.


PROBLEMA 2

As mn casas de um tabuleiro m n (m, n > 1 inteiros) so pintadas alternadamente de branco e preto, como em um tabuleiro de xadrez. Em seguida, colocamos em cada casa um nmero inteiro, de modo que a soma dos nmeros de cada linha e a soma dos nmeros de cada coluna sejam pares. Prove que a soma dos nmeros escritos nas casas pretas par.
PROBLEMA 3

Seja ABCD um paralelogramo, H o ortocentro do tringulo ABD e O o circuncentro do tringulo BCD. Prove que os pontos H, O e C so colineares.
PROBLEMA 4

Determine todos os ternos (a, b, c) de inteiros positivos tais que a e b so pares e ab + ba = 2c.

Voc sabia
Que o record de maior primo de Fermat generalizado (i. e., da forma a 2 + 1 ) conhecido foi batido em 2/11/1999? Este primo 11402416384 + 1, tem 82854 dgitos e foi descoberto por Kimmo Herranen, usando um programa desenvolvido por Yves Gallot
(veja http://perso.wanadoo.fr/yves.gallot/primes/gfn.html)
n

EUREKA! N6, 1999

41

Sociedade Brasileira de Matemtica

SOLUES DE PROBLEMAS PROPOSTOS Publicamos aqui algumas das respostas enviadas por nossos leitores.
10) Suponha que temos k moedas, todas iguais exceto por uma que tem peso ligeiramente diferente das anteriores (no se sabe se maior ou menor), e uma balana de dois pratos.
possvel determinar com n pesagens qual 2 a moeda diferente, e se ela mais leve ou mais pesada que as outras. 3n 1 possvel determinar com n pesagens qual b) Mostre que se k = 2 a moeda diferente, mas nem sempre possvel dizer se ela mais leve ou mais pesada que as outras. 3n 1 no sempre possvel determinar qual a c) Mostre que se k > 2 moeda diferente. a) Mostre que se k

3n 3

Soluo do Editor:
Fato 1: Se tivermos 3n ou menos moedas com apenas uma diferente, e que seja sabido se a moeda diferente mais leve ou mais pesada ento possvel determinar a moeda diferente com apenas n pesagens. Prova : Se n = 0 a afirmao bvia. Vamos provar o fato por induo. Se tivermos 3 n +1 ou menos moedas de modo que uma delas , digamos, mais pesada que as outras, dividimos as moedas em trs grupos, A, B e C, de 3 n ou menos moedas cada, de modo que A e B tenham o mesmo nmero de moedas. Pesamos o grupo A com o grupo B. Se os pesos forem diferentes, o grupo mais pesado tem 3 n ou menos moedas e contm a moeda diferente, e portanto podemos , com mais n pesagens (por hiptese de induo), determin-la. Se os pesos forem iguais a moeda mais pesada estar no grupo C, e , como antes, podemos determin-la em mais n pesagens.

EUREKA! N6, 1999

42

Sociedade Brasileira de Matemtica

Corolrio 1:

3n 1 moedas das quais uma diferente, e tivermos um 2 2k + 1 n 1 grupo de pelo menos moedas padro ento possvel 3 3 determinar a moeda diferente e se ela mais leve ou mais pesada com n pesagens.
Se tivermos k

3n 1 2k + 1 n 1 moedas dentre as , e Prova: Separamos um grupo A de 3 2 3 2k + 1 n 1 moedas padro. Se der pesamos com um grupo de 3 3 diferente saberemos que a moeda diferente est no grupo A e se ela mais leve ou mais pesada, e, usando o Fato 1, podemos determin-la com mais n 1 pesagens. Se der igual a moeda diferente est no grupo das n 3 n 1 2k + 1 3 1 n 1 moedas restantes, o que prova o 3 k = 2 2 3 corolrio por induo, dado que temos uma quantidade suficiente de moedas padro e, para n = 1, se tivermos uma moeda padro s pes-la com a moeda diferente para descobrir se esta mais leve ou mais pesada.
Corolrio 2: Se tivermos k

3n + 1 moedas das quais uma diferente e tivermos pelo 2

2k 1 n 1 menos 3 moedas padro ento possvel determinar com n 3 pesagens a moeda diferente.
Prova: Para n = 1 temos 2 moedas. Pesamos uma delas com uma moeda padro. Se der diferente ela a diferente, se der igual a outra. O resto da prova (por induo) igual do corolrio 1. Fato 2: Se tivermos dois grupos A e B de k

3n 1 moedas cada uma das quais 2 diferente tais que o peso total de A maior que o peso total de B e pelo
43

EUREKA! N6, 1999

Sociedade Brasileira de Matemtica

2k + 1 n 2 moedas padro ento possvel determinar menos 2 23 9 com n pesagens a moeda diferente e se ela mais leve ou mais pesada.

2k + 1 n2 Prova: Seja r = 3 . Pesamos de um lado 2r moedas do grupo A, r 9 do grupo B e do outro r moedas do grupo A, r do grupo B e 2r moedas padro. Se o primeiro lado tiver peso maior dividimos suas 2r moedas do grupo A em dois grupos de r e os pesamos. Se der igual a moeda diferente ser mais leve e estar entre as r 3 n 2 moedas do grupo B que estavam do outro lado. Se der diferente a moeda diferente ser mais pesada e estar no lado mais pesado nessa ltima pesagem. Nesses casos o Fato 1 implica o resultado pois com mais n 2 pesagens descobrimos a moeda diferente. Se o primeiro lado tiver peso menor a anlise anloga, trocando os papeis dos grupos A e B. Finalmente, se der igual a moeda diferente estar nos grupos A' A ou n 1 2k + 1 3 1 moedas que no foram pesadas, e o B ' B de k 3 2 9
resultado segue por induo ( O caso n = 1 trivial) Vamos agora resolver os tens originais do problema: a) Se

3n 3 moedas pesamos dois grupos de 2 n k 1 3 1 moedas cada. Se der diferente usamos o Fato 2, e se m= 2 3
temos

der igual usamos o Corolrio 1 para as restantes. b) Se temos k =

k 2m

3 n 1 1 moedas 2

3n 1 3 n 1 1 pesamos dois grupos de moedas cada. Se 2 2 der diferente usamos o Fato 2, e se der igual usamos o Corolrio 2 para as 3 n 1 + 1 moedas restantes. 2
44

EUREKA! N6, 1999

Sociedade Brasileira de Matemtica

Para ver que no possvel descobrir a moeda diferente e se ela mais leve ou mais pesada com n pesagens, observamos que se na primeira 3 n 1 1 moedas em cada prato e der pesagem pesamos mais que 2 3n 1 +1 = 3n 1 +1 > 3n 1 diferente, teremos no mnimo 2 2 possibilidades para a moeda diferente, e no podemos distingui-las com n 1 pesagens (que do no mximo 3 n 1 possveis resultados finais). 3 n 1 1 moedas em cada prato na Assim, devemos ter no mximo 2 3 n 1 + 1 moedas. O primeira pesagem, e se der igual sobram pelo menos 2 resultado seguir do 3k + 1 moedas das quais sabemos apenas que uma diferente, e Lema: Se temos 2 algumas moedas padro no podemos distinguir a moeda diferente e se ela mais leve ou mais pesada com k pesagens. Prova: Pelo argumento acima, no podemos usar mais de 3 k 1 moedas desconhecidas na primeira pesagem, e se esta der equilbrio sobram pelo 3 k 1 + 1 moedas, e o resultado segue por induo (o caso k = 0 menos 2 trivial). 3n 1 moedas em cada prato na c) Como antes no podemos usar mais que 2 3 n 1 + 3 primeira pesagem, e se esta der igual sobraro pelo menos 2 moedas, caso em que no possvel determinar a moeda diferente com mais n 1 pesagens, o que pode ser provado do mesmo modo que o lema anterior

EUREKA! N6, 1999

45

Sociedade Brasileira de Matemtica

16) Seja

reta

{( x, y ) R 2 y = 0}, C1

crculo

centrado

em

1 1 1 1 (0, ) de raio e C 2 o crculo centrado em (1, ) de raio . 2 2 2 2 Seja F o conjunto de crculos em R2 com as seguintes propriedades:
i) ii) {C1, C2} F Se C e C pertencem a F, so tangentes entre si e tangentes a l ento todo crculo C tangente aos dois crculos C e C e reta l pertence a F. iii) Se F um conjunto de crculos satisfazendo as propriedades i) e ii) ento F F . Determine o conjunto dos pontos de tangncia dos crculos C F com a reta l.
~ ~
~

Soluo de Zoroastro Azambuja Neto (Rio de Janeiro - RJ):


O conjunto dos pontos de tangncia ser o conjunto {( x,0), x Q [0,1]}. Cada p racional x [0,1] ser representado por onde p inteiro, q inteiro positivo e q mdc( p, q) = 1. Para provar isso mostraremos os seguintes fatos por induo: p 1 O crculo tangente em ( , 0) ter raio . i) q 2q 2 p r e Se os crculos tangentes em so tangentes entre si ii) q s ento ps qr = 1 Para isso, notemos que se dois crculos centrados em (x, r1) e (y, r2) so tangentes reta l e tangentes entre si ento (r1 r2 ) 2 + d 2 = (r1 + r2 ) 2 d 2 = 4r1r2 d = 2 r1 r2 , onde d = x y .

EUREKA! N6, 1999

46

Sociedade Brasileira de Matemtica


r1 + r2 r1 r2

As afirmaes i) e ii) so verdadeiras para os crculos iniciais C1 e C2. Se o p 1 crculo C tangente a l e tem centro q , 2q 2 , o crculo C ' tangente a l e tem ~ r 1 centro , 2 e qr ps = 1 ento, se o crculo C tangente a C e C ' e reta l s 2s p p r r tem centro (x, y) com < x < ento, se d ' = x e d ' ' = x, devemos q s q s ter

d'=

2 q

y 2

d" =

2 s

y , 2

d '+ d " =

r p 1 = , s q qs

donde

2( q + s ) qs
=

y 1 p p(q + s) + 1 1 1 = y= e d'= x = + d'= = 2 q(q + s) q q(q + s) 2 qs 2( q + s )

~ p+r p+r (pois ps = qr 1). Assim, C tangente em e tem q + s ,0 q+s 1 raio . 2(q + s ) 2

Como q ( p + r ) p (q + s ) = qr ps = 1 e (q + s )r ( p + r ) s = qr ps = 1 vemos que C satisfaz i) e ii). Esses fatos implicam que todos os crculos criados tero centro em pontos p p racionais. Basta provar agora que para todo racional [0,1], o ponto q , 0 q ponto de tangncia de algum dos crculos. Faremos isto por induo em q (para q = 1 o resultado bvio): basta mostrar se mdc(p, q) = 1 e q 2 que e possvel escrever p = p' + p" e q = q' + q" com p', p", q', q" inteiros, q', p" 0 e q'p" p'q" = 1. Estas equaes podem ser escritas como q ' ( p p' ) p ' (q q ' ) = 1, ou seja, q ' p p' q = 1, onde 0 < q ' < q e 0 < p' < p. Como mdc(p, q) = 1 existem x, y Z
EUREKA! N6, 1999

47

Sociedade Brasileira de Matemtica

com px + qy = 1, e teremos para todo k Z, p( x + kq) + q ( y kp) = 1. Certamente podemos escolher k de modo que 0 < x + kq < q (note que x no mltiplo de q, seno 1 = px + qy tambm seria), e ento tomamos q ' = x kq e p' = kp y pq '1 , mas 1 q' < q, donde 0 p ' < p). (temos p' = q

17) Dado n N, uma partio de n uma lista ordenada = (a , a ,...,a ) , r, a , a ,..., a N* com a a ... a e a + a + ...+ a = n.
1 2
r

Seja Pn o conjunto das parties de n. Para Pn , definimos A() como o nmero de termos iguais a 1 em ( ou seja , A( ) = # {i {1,2,..., r} a i = 1}), e B() como o nmero de termos distintos na partio (ou seja, B () = # {a1, a2, , ar}). Prove que
Pn

A( ) = B( ) para todo n N.
Pn

Soluo de Zoroastro Azambuja Neto (Rio de Janeiro - RJ):


Provaremos o resultado por induo em n. Para n = 1 o resultado bvio, pois a nica partio de 1 (1). As parties = (a ,..., a ) de n + 1 com a = 1 (isto ,
1
n

com A( ) > 0) so obtidas das parties = (a ,..., a ) de n acrescentando um 1


2
r

na primeira posio. Esta operao acrescenta 1 ao A() e acrescenta 1 ao B() caso A() = 0, e 0 caso contrrio (ou seja, caso A( ) 2). Admitindo o resultado para n, o resultado para n + 1 segue ento de
Pn +1
~

A( ) B( ) =#{ partio de n + 1 | A( ) 2} B( ).
Pn +1
~

Pn +1
A ( ) = 0
~

O lado direito igual a 0, pois, considerando f = { Pn +1 | A( ) 2} { Pn +1 | A( ) = 0}, tal que se = (a1 , a 2 ,...a r )
EUREKA! N6, 1999

48

Sociedade Brasileira de Matemtica

ento os elementos da partio f ( ) so A( ), a


~ ~

A( ) +1

,..., a r (somamos todos os

A( ) 2 termos de iguais a 1), fcil ver que f sobrejetiva, e , dado

Pn +1 com A( ) = 0, # { ' Pn +1 | A( ) 2 e f ( ' ) = } = B( ),


(escolheremos um elemento de e o decompomos como soma de uns) donde

# { Pn +1 | A( ) 2} =
~

B( ).
~

Pn +1
A( ) = 0

20) Diga se existe uma funo polinomial de R2 em R cuja imagem seja


o intervalo (0,+) = {x R x > 0}.

Soluo de Zoroastro Azambuja Neto (Rio de Janeiro - RJ):


Considere a funo f ( x, y ) = x 2 + ( xy 1) 2 fcil ver que f ( x, y ) 0, ( x, y ) R 2 , e se f ( x) = 0 ento x = 0 e xy 1 = 0, o 1 que impossvel. Dado a > 0, f = a, donde Im (f) = (0, + ). a, a

24) Na loteria de Truchilndia, cada bilhete tem um nmero de trs algarismos que usa somente os algarismos 1, 2, 3, 4 ( permitido repetir os dgitos). Um bilhete ganhador se coincide em pelo menos duas posies com o nmero sorteado. Um apostador quer compar vrios bilhetes, de maneira que um deles ganhe com certeza, mas gastando o mnimo possvel. Determinar quantos bilhetes deve comprar e quais bilhetes deve comprar. Obs. Se o bilhete sorteado for o 423 ento 123 um bilhete ganhador, mas 243 no . Soluo de Flvio Wagner Rodrigues (So Paulo - SP):
1. Vamos observar inicialmente que o sorteio dessa loteria admite 64 resultados possveis.
EUREKA! N6, 1999

49

Sociedade Brasileira de Matemtica

2. Com um nico bilhete o apostador tem 10 resultados favorveis. De fato, para cada par de posies 1 e 2, 1 e 3 e 2 e 3 existem quatro bilhetes favorveis o que nos d um total de 12 mas, o nmero de seu bilhete contado 3 vezes e portanto so apenas 10 resultados favorveis distintos. 3. Segue-se de 1 e 2 que 6 bilhetes no so suficientes para garantir a vitria do apostador. .Vamos mostrar agora que 7 bilhetes tambm no so suficientes. Como so quatro algarismos e 7 jogos existe pelo menos um algarismo que aparece uma nica vez na primeira posio. Vamos supor que existe um nico jogo que tenha o 1 na primeira posio. Esse jogo garante a vitria do jogador em 7 jogos comeados com 1. Como existem 16 jogos comeados por 1, a vitria nos outros 9 deve ser garantida pelos elementos que aparecem nas posies 2 e 3 dos demais jogos. Como existem apenas 6 outros jogos existiro 3 jogos comeados por 1 que se forem sorteados no daro vitria ao jogador.

4. Existem vrios conjuntos de oito jogos que resolvem o problema. Vamos dar dois exemplos e mostrar em seguida como esses conjuntos podem ser construdos. 5. Exemplo 1 : 124, 131, 212, 313, 342, 243, 421, 434 Exemplo 2 : 123, 111, 232, 334, 342, 244, 413, 421 Exemplo 3 : 111, 122, 212, 221, 333, 344, 434, 443 Roteiro para construir um conjunto de oito jogos que resolve o problema. Inicialmente vamos observar que o conjunto dever satisfazer duas condies importantes: 1- Cada algarismo deve aparecer em cada posio exatamente duas vezes. 2- necessrio que existam jogos que contenham elementos repetidos como 11, 22, 33, 44 no importando em que posies. 1- Escolha dois jogos que tenham o 1 na primeira posio e algarismos distintos nas outras posies. Por exemplo 1 3 4 e 1 4 2. No difcil mostrar que dos 16 jogos que comeam por 1 esses dois jogos garantem a vitria do apostador em 12. Restam 4 jogos para os quais ser necessrio
EUREKA! N6, 1999

50

Sociedade Brasileira de Matemtica

que a vitria seja garantida pelas posies 2 e 3 dos demais jogos. No nosso exemplo, vamos necessitar de jogos terminando em 11, 13, 21 e 23 2- A primeira observao acima e a convenincia de no repetir ligaes nos permitem dizer que os dois jogos que faltam devem terminar por 32 e 44. A distribuio dos algarismos que devem ocupar as primeiras posies deve ser feita levando em conta as condies iniciais evitando sempre que possvel a repetio de ligaes. Uma possibilidade seria : 134, 142, 211, 223, 313, 321, 432, 444

26) Sejam as funes fo (x) = xn e fi (x) = fi 1(x + 1) fi inteiros positivos. Prove que, para todo x, fn(x) = n!

1(x)

onde x, n e i so

Soluo de Bruno Fernandes Cerqueira Leite (So Paulo - SP):


Antes de comear, importante deduzir (pelo Binmio de Newton) que: a a2 ( x + 1) a x a = a x a 1 + + ... (isto ser muito usado na demonstrao). x 2 Agora temos f 0 ( x) = x n e f i +1 ( x) = f i ( x + 1) f i ( x)

n n2 f ( x) + ... 1 = f 1 ( x) = f 0 ( x + 1) f 0 ( x) = ( x + 1) n x n = n x n 1 + 2 x n x n 1 + a1 x n 2 + a 2 x n 3 + ..., onde a1, a2, , etc so coeficientes racionais (que no nos interessam).

f 2 ( x) f1 ( x + 1) f1 ( x) = = ( x + 1) n1 + a1 ( x + 1) n2 + ... n n n f ( x) x n1 + a1 x n2 + ... = ( x + 1) n1 x n1 + a1 ( x +1) n2 x n2 + ... 2 = n n2 n 1 n3 n3 n 2 n 4 x + ... + a1 (n 2) x + x + ... + ... (n 1) x + 2 2 f 2 ( x) = f1 ( x + 1) f1 ( x)

] [

] [

f 2 ( x) = x n2 + b1 x n3 + b2 x n4 + ..., onde b1, b2, , etc, so coeficientes n(n 1) racionais (cuja determinao no nos interessa).
EUREKA! N6, 1999

51

Sociedade Brasileira de Matemtica

Continuando com esses procedimentos, podemos chegar a expresses anlogas para f3(x), f4(x), etc. A semelhana entre essas expresses nos leva a "desconfiar"que

f b ( x) = x nb + 1 x nb1 + 2 x nb2 + ..., onde An,b


B j , j = 1,2, etc; so coeficientes (no nos interssam) n! An,b = n(n 1)(n 2)... = (n b)! b fatores
Vamos provar isto por induo. Suponha que a frmula valha para f1(x), , fb1(x). Teremos:

f b ( x) = f b1 ( x +1) f b1 ( x)

f b ( x) f b1 ( x +1) f b1 ( x) = e por hiptese de An,b1 An,b1 An,b1

induo, f b ( x) = ( x + 1) nb+1 + 1 ( x + 1) nb + ... x nb+1 + 1 x nb + ... = ( x + 1) nb+1 x nb+1 + An,b1

][

] [

1[(x +1)nb + xnb] +...= (nb+1)xnb +

+...

nb+1 nb1 nb+1 n b nb+2 x +... x +... +1(nb)x + 2 2

fb(x) fb(x) = (n b +1) xnb + 1xnb1 + 2xnb2 +... = An,b1 An,b1(n b +1)

fb(x) = n! (n b +1) (n b +1)!

fb (x) f (x) = xnb + 1xnb1 + 2xnb2 +... b = xnb + 1xnb1 + 2xnb2 +..., n! An,b (n b)! confirmando o que foi "desconfiado". f (x) Agora vamos usar a frmula para n = xnn = 1 fn (x) = An,n = n! An,n =

EUREKA! N6, 1999

52

Sociedade Brasileira de Matemtica

27) O tringulo equiltero ABC possui um ponto interno P tal que em P chegam trs segmentos de reta (PA, PB, PC) onde PA = 6, PB = 8 e PC = 7. Com esses dados descubra qual a rea do tringulo.

P B C

Soluo de Ricardo Klein Hoffmann (Porto Alegre - RS):


A

6 P 8 B 8 D 6

7 C

1. Seja BCD o tringulo congruente ao tringulo APB onde: ___ ___ PB = BD = 8 ___ ___ AP = CD = 6 ___ ___ AB = BC = x 2. Temos que A B C = P B D = 60. Ento, o PBD equiltero com PD = 8. 3. Seja C P D = 4. ( PDC ) Lei dos cossenos

___

EUREKA! N6, 1999

53

Sociedade Brasileira de Matemtica

6 2 = 7 2 + 8 2 2 7 8 cos cos =
5.

( BPC ) Lei dos cossenos

135 11 daqui, sen = 16 16 149 + 63 5 2

x 2 = 8 2 + 7 2 2 8 7 cos( + 60) x 2 =
6. rea ( ABC ) =

x2 3 =S 4 3 149 + 63 5 Ento, S = 4 2

Portanto, a rea do ( ABC ) =

3 (149 + 63 5 ). 8

28) Seja n 2 um nmero inteiro. Prove que n e n + 2 so ambos primos se e


somente se

4((n 1)! + 1) + n inteiro. n(n + 2)

Soluo de Marcelo Rufino de Oliveira (So Jos dos Campos - SP):


Antes de iniciar a resoluo, vou enunciar e provar os seguintes Teoremas, que sero de grande utilidade:

1o Teorema (Teorema de Wilson): Se p um nmero primo, ento o nmero (p 1)! + 1 divisvel por p. Prova: Consideremos a congruncia linear: ax 1 (mod p) onde a qualquer dos p 1 inteiros positivos da seqncia 1, 2, 3, , p 1 (1) Ento, o mdc (a, p) = 1 e sabido que esta congruncia admite uma nica soluo mdulo p, isto , existe um nico inteiro a1, com 1 a1 p 1, tal que aa1 1 (mod p) Se p primo, temos que a = a1 se e somente se a = 1 ou a = p 1. De fato, a congruncia quadrtica a2 1 (mod p) equivalente seguinte (a 1)(a + 1) 0 (mod p) de modo que p | (a 1) ou p | (a + 1), o que implica que: a 1 0 (mod p) e a = 1 ou a + 1 0 (mod p) e a = p 1
EUREKA! N6, 1999

54

Sociedade Brasileira de Matemtica

Ento para cada a distinto da seqncia (1) existe somente um a1 correspondente na seqncia (1) que satisfaz aa1 1 (mod p). Omitindo os pares 1 e p 1, com os p 3 inteiros restantes: 2, 3, , p 3, podemos formar (p 3)/2 pares a, a1, com a a1, e tais que aa1 1 (mod p). Multiplicando todas essas (p 3)/2 congruncias, obtemos: 2.3.4(p 3)(p 2) 1 (mod p) => (p 2)! 1 (mod p) Multiplicando por p 1: (p 1)! p 1 (mod p) => (p 1)! 1 (mod p)

2o Teorema: Uma condio necessria e suficiente para que um nmero natural p > 1 seja primo que o nmero (p 1)! + 1 seja divisvel por p. Prova: Suponhamos que p no seja primo. Ento existe um divisor q de p tal que 1 < q < p. O nmero (p 1)! + 1 sendo divisvel por p, tambm deve ser divisvel por q. Desde que q < p, ento q p 1, implicando que q divide algum inteiro positivo entre 2 e p 1. Assim q divide (p 1)!. Como q divide (p 1)! + 1 ento q tambm divide 1, que uma contradio, pois 1 < q < p. 1a parte (ida) I) Suponhamos que n e n + 2 so nmeros primos. De acordo com o Teorema de Wilson ns temos que (n 1)! + 1 divisvel por n e (n + 1)! + 1 divisvel por n + 2. Como n divide 4((n 1)! + 1) e, evidentemente, n tambm divide n, ento divide a sua soma, ou seja, 4((n 1)! + 1) + n divisvel por n. Basta analisar agora se 4((n 1)! + 1) + n tambm divisvel por n + 2. x = 4((n 1)! + 1) + n = 4((n 1)! + 1) + n + 2.(n + 1)! 2.(n + 1)! x = 2.(n + 1)! + n + 4 2[(n + 1)! 2.(n 1)!] x = 2[(n + 1)! + 1] + (n + 2) + [(n 1)!n(n + 1) 2.(n 1)!] x = 2[(n + 1)! + 1] + (n + 2) + (n 1)![n2 + n 2] x = 2[(n + 1)! + 1] + (n + 2) + (n 1)!(n + 2)(n 1) Como n + 2 divide tanto 2[(n + 1)! + 1], como (n + 2) e tambm (n 1)!(n + 2)(n 1), ento divide a sua soma, que x = 4((n 1)! + 1) + n. Como n e n + 2 so ambos primos e dividem 4((n 1)! + 1) + n, ento o seu produto tambm divide 4((n 1)! + 1) + n. 2a parte (volta) II) Suponhamos agora que para um nmero natural n > 1, o valor 4((n 1)! + 1) + n divisvel pelo produto n(n + 2). Se n for par, isto , se n = 2k, onde k um nmero natural, ento n 1 k.
EUREKA! N6, 1999

55

Sociedade Brasileira de Matemtica

Isto implica que (n 1)! divisvel por k e tambm que 4.(n 1)! divisvel por 2 k. Consequentemente 4.(n 1)! divisvel por n, e como 4((n 1)! + 1) + n = 4.(n 1)! + n + 4 divisvel por n, ento 4 divisvel por n. Como n par ento n = 2 ou n = 4. Entretanto fcil verificar que 4((n 1)! + 1) + n no divisvel por n(n + 2) caso n = 2 ou n = 4. Assim, o fato de que 4((n 1)! + 1) + n divisvel por n implica que (n 1)! + 1 divisvel por n. Pelo 2o Teorema temos que n um nmero primo. Fazendo as mesmas operaes da 1a parte da soluo: 4((n 1)! + 1) + n = 2[(n + 1)! + 1] + (n + 2) + (n 1)!(n + 2)(n 1) Como 4((n 1)! + 1) + n, n + 2 e (n 1)!(n + 2)(n 1) so todos termos divisveis por n + 2, ento (n + 1)! + 1 tambm divisvel por n + 2. Pelo 2o Teorema temos que n + 2 tambm um nmero primo.

Agradecemos tambm o envio das solues a: Carlos Alberto da Silva Victor (Rio de Janeiro - RJ), Tertuliano Franco (Salvador - BA), Manuel Joo de Jesus Almeida Rio de Janeiro - RJ) e Daniel Pessa Martins Cunha (Fortaleza - CE). Seguimos esperando a soluo do problema 29. O prazo para recebimento das solues do problema "Cutico" publicado na revista EUREKA! No. 5 (que concorre a prmio!) foi prorrogado at dia 15 de fevereiro. Problema "Cutico" Prove que para qualquer conjunto de inteiros positivos A e para todo inteiro positivo k existe um conjunto infinito de nmeros primos S tal que o produto de k elementos distintos de S est sempre em A ou o produto de k elementos distintos de S nunca pertence a A. Errata: No primeiro teste de seleo para a 40a. IMO e a 14a. OIM, publicado na EUREKA! No. 5, pg 50, no problema 4, onde diz: "Por exemplo, min{3, 4} = 4" deve dizer "Por exemplo, min {3, 4} = 3"

EUREKA! N6, 1999

56

Sociedade Brasileira de Matemtica

PROBLEMAS PROPOSTOS
Convidamos o leitor a enviar solues dos problemas propostos e sugestes de novos problemas para os prximos nmeros.
30) Determine todas as funes f : R R que satisfazem as condies: 1 f ( x) (i) f( x) = f(x), (ii) f(x + 1) = f(x) + 1, (iii) f = 2 para x 0. x x 31) Seja x1, x2, x3, uma seqncia de nmeros reais no negativos satisfazendo x n 2 x n 1 xn = para n = 3, 4, 5, Estabelea condies necessrias e 2 x n 2 x n 1 suficientes em x1 e x2 para xn ser inteiro para infinitos valores de n. 32) a) Prove que todo nmero inteiro no nulo m admite uma nica representao
da forma m =
n 1

k =0

3 k , onde n um inteiro positivo e k {1,0,1} para

todo k, com n 1 0.
b) Dado um conjunto de

3n + 1 pontos V = {P0 , P1 ,..., P3n 1 } , escrevemos em 2


2

cada aresta que une dois desses pontos Pi e Pj (i j) um nmero pertencente a {0, 1, , n 1} da seguinte forma: escreveremos

i j =

k =0

n 1

3 k , com k {1,0,1}, k , e associamos aresta Pi Pj o

nmero m = min{k 0 k = 1}. Prove que no existe nenhum tringulo cujos vrtices pertenam a V com o mesmo nmero escrito em seus trs lados.

33) Na parede interna de um vaso cilndrico de cristal existe uma gota de mel num ponto B situado a trs centmetros do seu bordo superior. Na parede externa, num ponto A diametralmente oposto ao da gota, est uma formiga. Sabendo que a altura do vaso de 20cm e o seu dimetro 10 cm. Indicar o caminho mais curto para que a formiga atinja a gota de mel. 34) ABC um tringulo, tal que AB = c, AC = b e BC = a. Por um ponto interior P deste tringulo, so traadas paralelas aos seus lados.
EUREKA! N6, 1999

57

Sociedade Brasileira de Matemtica

Sabe-se que as interseces, da paralela ao lado de medida a, com os lados deste tringulo, determinam um segmento de medida a'. Analogamente, as paralelas aos lados de medidas b e c, determinam com os lados do tringulo, segmentos de medidas b' e c' respectivamente. a ' b' c' Nestas condies demonstre que + + = 2. a b c

35) Sabendo que num tringulo ABC a altura relativa ao vrtice A mede 12cm. e a altura relativa ao vrtice B mede 20cm., determine todos os valores possveis para a altura relativa ao vrtice C.

Problemas 30 e 31 propostos por Marcelo Rufino de Oliveira (So Jos dos Campos - SP), problema 33 proposto por Gleydson Chaves Ricarte (Fortaleza - CE), problema 34 proposto por Paulo Jacob (Lista de discusso de problemas), problema 35 proposto por Carlos Alexandre Gomes da Silva (Natal - RN).

Que ningum sabe se existem infinitos primos p tais que p + 2 tambm seja primo (mas todo mundo acredita nisso)? Um para de primos p, p + 2 chamado um par de primos gmeos. Os maiores primos gmeos conhecidos so 361700055 239020 1, que tm 11755 dgitos e foram descobertos este ano por Henri Lifchitz. Tambm no se sabe se todo nmero par maior ou igual a 4 pode ser escrito como soma de dois primos (esta a conjectura de Golbach).

Voc sabia

EUREKA! N6, 1999

58

Sociedade Brasileira de Matemtica

ASSINATURA DA REVISTA EUREKA! 2000


Se voc deseja seguir recebendo na sua casa a revista EUREKA!, faa o seu pedido escrevendo para: Secretaria da Olimpada Brasileira de Matemtica, Estrada Dona Castorina, 110 Jardim Botnico - Rio de Janeiro, RJ CEP: 22460-320. Voc pode fazer uma assinatura anual o que dar direito a receber as publicaes do ano 2000 (mnimo 3 exemplares) por um valor de R$10,00. Para isso, faa um depsito no Banco do Brasil - Agncia 0598-3 Conta N52208-2 em nome do professor Eduardo Wagner. Envie-nos a fotocopia da ficha do verso preenchida e uma cpia do recibo do depsito. Ns remeteremos a(s) revista(s) pelo correio. Pedidos podem ser feitos tambm por email e comprovantes de depsito podero ser enviados pelo fax. Se tiver qualquer dvida entre em contato conosco. Telefone: 21-5295077 / Fax: 21-5295023 e-mail: obm@impa.br Home-Page: http//www.obm.org.br/

LISTA DE DISCUSSO DE PROBLEMAS DE MATEMTICA OLMPICA


Est aberta uma lista de discusso de problemas de Matemtica Olmpica. A lista aberta todos. O endereo da lista : Obm-l@mat.puc-rio.br Para assinar, envie uma mensagem para: Majordomo@mat.puc-rio.br Com texto Subscribe obm-l Quem assim proceder deve receber um pedido de confirmao auto-explicativo (automaticamente gerado pelo majordomo em ingls). Em caso de problemas, escreva para o Professor Nicolau Saldanha (administrador desta lista): e-mail: nicolau@mat.puc-rio.br Desta lista esto participando muitos alunos e professores e nela so discutidos problemas e aspectos tericos de matemtica. Trata-se de um meio informal e eficiente de preparao para as olimpadas.

EUREKA! N6, 1999

59

Sociedade Brasileira de Matemtica

ASSINATURA 2000 DA REVISTA EUREKA!


Ateno! Leia atentamente as instrues da pgina anterior. Por favor preencha esta ficha com letra de forma.

Nome: Endereo: Bairro: Cidade: Cep: Telefone: ( Fax: ( e-mail:

Estado: ) )

Nmeros atrasados podem ser comprados utilizando a mesma forma de pagamento da assinatura, sendo os preos:

Nmeros 1, 2 e 3 Nmeros 4, 5 e 6 Nmeros por separado

R$10,00 R$10,00 R$4,00 (cada)

Voc sabia

que ningum sabe se para

todo n N existe k N tal que a k ( n) = 1 , onde a1 ( n) = n e

n / 2, se n par a k +1 (n) = 3n + 1, se n mpar

(Isto conhecido como o problema 3 x + 1, e a resposta afirmativa para todo n 2 40 )

EUREKA! N6, 1999

60

Sociedade Brasileira de Matemtica

COORDENADORES REGIONAIS
Amarisio da Silva Arajo Alberto Hassen Raad Antnio C. Rodrigues Monteiro Angela Camargo Benedito T. Vasconcelos Freire Claudio Arconcher Clauss Haetinger Crescncio das Neves lio Mega Enzo Marcom Takara Flvia Jernimo Barbosa Florncio F. Guimares Filho Francisco Dutenhefner Gisele de A. Prateado Gusmo Ivanilde H. Fernandes Saad Joo Bencio de Melo Neto Joo F. Melo Libonati Jorge Ferreira Jos Carlos Pinto Leivas Jos Cloves Saraiva Jos Gaspar Ruas Filho Jos Luis Rosas Pinho Jos Paulo Carneiro Jos Vieira Alves Leonardo Matteo D'orio Licio Hernandes Bezerra Luzinalva M. de Amorim Marcondes Cavalcante Frana Pablo Rodrigo Ganassim Paulo H. Cruz Neiva de L. Jr. Reinaldo Gen Ichiro Arakaki Ricardo Amorim Roberto Vizeu Barros Sergio Claudio Ramos Seme Gebara Neto Tadeu Ferreira Gomes Toms Menndez Rodrigues Valdenberg Arajo da Silva Wagner Pereira Lopes Waldemar M. Canalli
EUREKA! N6, 1999

(UFV) Viosa - MG (UFJF) Juiz de Fora - MG (UFPE) Recife - PE (Centro de Educao de Adultos - CEA) Blumenau - SC (UFRN) Natal - RN (Col. Leonardo da Vinci) Jundia - SP (UNIVATES) Lajeado - RS (UFAM) Manaus-AM (Col. ETAPA) So Paulo - SP (Col. Singular) Santo Andr - SP (UFPB Campus I) Joo Pessoa - PB (UFES) Vitria - ES (UFMG) Belo Horizonte - MG (UFGO) Goinia - GO (U. Catlica Dom Bosco) Campo Grande - MS (UFPI) Teresina - PI (Grupo Educ. IDEAL) Belm - PA (UEM) Maring - PR (UFRG) Rio Grande - RS (UFMA) So Luis - MA (ICMC-USP) So Carlos - SP (UFSC) Florianpolis - SC (Univ. Santa rsula) Rio de Janeiro - RJ (UFPB) Campina Grande - PB (Sistema Titular de Ensino)Belm - PA (UFSC) Florianpolis - SC (UFBA) Salvador - BA (UF Cear) Fortaleza - CE (L. Albert Einstein) Piracicaba - SP (Esc. Tec.Everardo Passos) SJ dos Campos - SP (INPE) SJ dos Campos - SP (Centro Educ. Logos) Nova Iguau - RJ (Colgio ACAE) Volta Redonda - RJ (IM-UFRGS) Porto Alegre - RS (UFMG) Belo Horizonte - MG (U. do Estado da Bahia) Juazeiro - BA (U. Federal de Rondonia) Porto Velho - RO (U. Federal de Sergipe) So Cristovo - SE (Esc. Tec. Fed. de Gois) Jata - GO (P.M. S. Joo de Meriti) S. Joo de Meriti - RJ

61

CONTEDO

AOS LEITORES XXI OLIMPADA BRASILEIRA DE MATEMTICA Problemas e solues da Primeira Fase XXI OLIMPADA BRASILEIRA DE MATEMTICA Problemas e solues da Segunda Fase XXI OLIMPADA BRASILEIRA DE MATEMTICA Problemas e solues da Terceira Fase XXI OLIMPADA BRASILEIRA DE MATEMTICA Resultados

2 3

13

21

36

ARTIGOS
EQUAES DIOFANTINAS Antonio Caminha Muniz Neto SOLUES DE PROBLEMAS PROPOSTOS PROBLEMAS PROPOSTOS AGENDA OLMPICA COORDENADORES REGIONAIS 39 49 59 61 62

Sociedade Brasileira de Matemtica

AOS LEITORES
Realizamos durante 1999 a XXI Olimpada Brasileira de Matemtica em mais de 2.500 colgios de nosso pas, atingindo na realizao da primeira etapa cerca de 60.000 alunos. Este ano a Olimpada se realizar nas seguintes datas: Primeira Fase Sbado, 10 de junho Segunda Fase Sbado, 02 de setembro Terceira Fase Sbado, 21 de outubro (nveis 1,2 e 3) Domingo, 22 de outubro (nvel 3 - segundo dia). A Comisso Nacional de Olimpadas entende que todo aluno que desejar participar da OBM deve poder faz-lo sem restries. A comisso oferece inclusive a alunos de escolas que no participam da OBM a possibilidade de fazer as provas sob superviso direta do Coordenador Regional. As escolas podem naturalmente aconselhar seus alunos a participar ou no da Olimpada de acordo com seus prprios critrios, mas a escola nunca deve impedir um aluno de participar se este for o seu desejo. Lembramos que a Olimpada Brasileira de Matemtica uma competio entre alunos e no entre colgios. A OBM divulga apenas os nomes e pontuaes dos alunos premiados; a OBM nunca divulgou nem divulgar comparaes entre colgios. Nosso objetivo estimular o estudo de Matemtica entre os jovens, contribuir para o aprimoramento dos professores e propiciar uma melhoria do ensino e do aprendizado desta matria nas escolas brasileiras e no comparar desempenhos de escolas. O Regulamento da OBM foi atualizado. Leia o novo regulamento no site:
http://www.obm.org.br/regulamento.htm

Finalmente, aproveitamos, para registrar a realizao da Semana Olmpica 2000, atividade que vem sendo realizada desde 1998. Nesta oportunidade o evento teve lugar na Universidade Metodista de Piracicaba (UNIMEP) entre os dias 21 a 27 de janeiro de 2000. Durante a Semana Olmpica 2000, reunimos alunos ganhadores da XXI Olmpiada Brasileira de Matemtica nos seus trs nveis de competio. Estes alunos participaram de um treinamento intensivo com professores de diversas partes do pas como preparao para a futura formao das equipes que representaro o Brasil em Olimpadas Internacionais. Alm disso eles tiveram a oportunidade de conquistar novas amizades, iniciando um relacionamento extremamente proveitoso com outros jovens da mesma faixa de idade e com interesses semelhantes.
EUREKA! N7, 2000

Sociedade Brasileira de Matemtica

XXI OLIMPADA BRASILEIRA DE MATEMTICA


Primeira Fase - Nvel 1
01. Um pequeno caminho pode carregar 50 sacos de areia ou 400 tijolos. Se foram colocados no caminho 32 sacos de areia, quantos tijolos pode ainda ele carregar? A) 132 B) 144 C) 146 D) 148 E) 152 02. A calculadora de Juliana bem diferente. Ela tem uma tecla D, que duplica o nmero escrito no visor e a tecla T, que apaga o algarismo das unidades do nmero escrito no visor.Assim, por exemplo, se estiver escrito 123 no visor e apertarmos D, teremos 246; depois, apertando T, teremos 24. Suponha que esteja escrito 1999. Se apertamos D depois T, em seguida D, depois T, teremos o nmero: A) 96 B) 98 C) 123 D) 79 E) 99 03. O grfico abaixo mostra o valor aproximado do dlar em reais no dia 15 dos ltimos 6 meses.

2,0 1,5 1,0 D J F M A M J

Marcelo comprou um carro usando um sistema de financiamento chamado leasing corrigido pela variao do dlar e suas prestaes vencem exatamente no dia 15 de cada ms. Em dezembro, Marcelo pagou R$ 600,00 de prestao. Com base na tabela, podemos dizer que em maio a prestao foi de: A) R$ 700,00 B) R$ 850,00 C) R$ 650,00 D) R$ 900,00 E) R$ 800,00 04. Numa certa cidade, o metr tem todas suas 12 estaes em linha reta. A distncia entre duas estaes vizinhas sempre a mesma. Sabe-se que a distncia entre a terceira e a sexta estaes igual a 3 300 metros. Qual o comprimento dessa linha? A) 8,4 km B) 12,1 km C) 9,9 km D) 13,2 km E) 9,075 km

EUREKA! N7, 2000

Sociedade Brasileira de Matemtica

05. A metade do nmero 211 + 4 8 igual a: B) 2 5 + 2 8 C) 110 + 2 8 A) 2 5 + 4 4

D) 215 + 4 5

E) 2 9 + 4 7

06. Quantos nmeros de dois algarismos so primos e tm como antecessor um quadrado perfeito ? A) 2 B) nenhum C) 1 D) 3 E) 6 07. Quantas vezes num dia (24 horas) os ponteiros de um relgio formam ngulo reto ? A) 48 B) 44 C) 24 D) 22 E) 23 08. Dona Zizi comprou 2 balas para cada aluno de uma 5a srie. Mas como os meninos andavam meio barulhentos, ela resolveu redistribuir essas balas, dando 5 para cada menina e apenas 1 para cada menino. Podemos concluir que na 5a srie A) 20% so meninos B) 30% so meninas C) 75% so meninos D) 50% so meninas E) 66,6...% so meninos 09. Vrios caixotes cbicos de plstico Azul ficaram armazenados ao ar livre, na posio indicada na figura ao lado, na qual apenas um dos caixotes no visvel. Com o tempo, o plstico exposto ao ar perdeu sua cor, tornando-se cinza. Ao desfazer a pilha, verificaremos que o nmero de caixotes com trs faces azuis e trs cinzentas ser: A) 4 B) 5 C) 3 D) 2 E) 1 10. Ronaldo, sempre que pode, guarda moedas de 50 centavos ou 1 real. Atualmente, ele tem 100 moedas, num total de 76 reais. Quantas moedas de um valor ele tem a mais do que a de outro valor ? A) 48 B) 4 C) 8 D) 52 E) 96 11. Juntando trs quadrados congruentes e fazendo coincidir lados comuns, sem superposio, podemos formar duas figuras diferentes, como mostra a figura ao lado. Observe que uma figura obtida de outra por rotao, deslocamento ou reflexo, congruente mesma figura (muda apenas a posio). Por exemplo, temos abaixo figura iguais em 4 posies diferentes:
EUREKA! N7, 2000

Sociedade Brasileira de Matemtica

120 60
o

Vamos agora pegar trs losangos congruentes, um deles representado ao lado. Quantas figuras diferentes podemos formar com os trs losangos, fazendo coincidir lados comuns?

A) 1

B) 2

C) 3

D) 5

E) 9

12. Renata digitou um trabalho de 100 pginas numerados de 1 a 100 e o imprimiu. Ao folhear o trabalho, percebeu que sua impressora estava com defeito, pois trocava o zero pelo um e o um pelo zero na numerao das pginas. Depois de consertar a impressora, quantas pginas teve que reimprimir, no mnimo ? A) 18 B) 20 C) 22 D) 30 E) 28 13. Letcia vendeu todos seus CDs de videogames para trs amigos, que lhe pagaram, respectivamente, R$ 240,00, R$ 180,00 e R$ 320,00. Todos os CDs tinham o mesmo preo. Quantos CDs tinha Letcia no mnimo? A) 20 B) 37 C) 28 D) 21 E) 25 14. 6 cartes com nmeros somente numa das faces so colocados sobre uma mesa conforme a ilustrao. Os cartes X e Y esto com a face numerada voltada para baixo. A mdia aritmtica dos nmeros de todos os cartes 5. A mdia aritmtica dos nmeros do carto Y e seus trs vizinhos 3. Qual o nmero escrito no carto X ? A) 4 B) 12 C) 0 D) 15 15. Rafael tem

8 X

2 6

4 Y

E) 10

2 da idade de Roberto e 2 anos mais jovem que Reinaldo. A 3 4 da idade de Reinaldo. Em anos, a soma das idades idade de Roberto representa 3 dos trs : A) 48 B) 72 C) 58 D) 60 E) 34

EUREKA! N7, 2000

Sociedade Brasileira de Matemtica

16. Marcos quer pesar trs maas numa balana de Dois pratos, mas ele dispe de apenas um bloco de 200 gramas. Observando o equilbrio na balana, ele observa que a ma maior tem o mesmo peso que as outras duas mas juntas; o bloco e a ma menor pesam tanto quanto as outras duas mas juntas; a ma maior junto com a menor pesam tanto quanto bloco. O peso total das trs mas : A) 250 g B) 300 g C) 350 g D) 400 g 17. No desenho ao lado esto representados Quatro tringulos retngulos e um retngulo, bem como suas medidas. Juntando todas essas figuras, podemos construir um quadrado. O lado desse quadrado ir medir: A) 88 cm B) 100 cm C) 60 cm D) 96 cm E) 80 cm
100

E) 450 g
20 12 80 16

60 16 88 20 12 16 80 100 60

18. Numa certa cidade, foi adotado o seguinte sistema de rodzio de carros: duas vezes por semana, de segunda a sexta, cada carro fica proibido de circular, de acordo com o final de sua placa (algarismo das unidades). O nmero mdio de finais de placa proibidos diferentes para cada dia de proibio : A) 4 B) 1 C) 3 D) 2 E) indefinido 19. Alexandre, consultando a programao de filmes, decidiu gravar Contato, cuja durao de 150 minutos. Para gravar numa nica fita, ele comeou com velocidade menor (modo EP, que permite gravar 6 horas) e, num dado momento, mudou para a velocidade maior (modo SP, que permite gravar 2 horas), de forma que a fita acabou exatamente no fim do filme. Do incio do filme at o momento da mudana do modo de gravao, quantos minutos se passaram? A) 60 B) 30 C) 15 D) 45 E) 105 20. Voc sabe que existem 9 nmeros de um algarismo, 90 nmeros de dois algarismos, 900 nmeros de trs algarismos, etc. Considere agora cada nmero cujo ltimo algarismo direita representa o nmero de algarismos desse nmero. Por exemplo, o nmero 9 074 um deles, pois 4 o nmero de seus algarismos. Quantos nmeros desse tipo existem ? A) 99 999 999 B) 99 999 992 C) 100 000 000 D) 10 000 000 E) 1 000 000 000
EUREKA! N7, 2000

Sociedade Brasileira de Matemtica

XXI OLIMPADA BRASILEIRA DE MATEMTICA


Primeira Fase - Nvel 2
01. Veja problema 01 do Nvel 1. 02. Em um hotel h 100 pessoas. 30 comem porco, 60 comem galinha e 80 comem alface. Qual o maior nmero possvel de pessoas que no comem nenhum desses dois tipos de carne? A) 10 B) 20 C) 30 D) 40 E) 50 03. Uma folha quadrada foi dobrada duas vezes ao longo de suas diagonais conforme ilustrao abaixo, obtendo-se um tringulo issceles. Foi feito um corte na folha dobrada, paralelo base desse tringulo, pelos pontos mdios dos outros lados. A rea do buraco na folha corresponde a que frao da rea da folha original ?

A)

1 2

B)

1 6

C)

3 8

D)

3 4

E)

1 4

04. Veja problema 9 do Nvel 1. 05. Veja problema 17 do Nvel 1. 06. Contando-se os alunos de uma classe de 4 em 4 sobram 2 e contando-se de 5 em 5 sobra 1. Sabendo-se que 15 alunos so meninas e que nesta classe o nmero de meninas maior que o nmero de meninos, o nmero de meninos nesta classe igual a : A) 7 B) 8 C) 9 D) 10 E) 11 07. O quociente de 5050 por 2525 igual a : B) 1025 C) 10025 A) 2525
D) 225 E) 2 2525

08. Qual o 1999o algarismo aps a vrgula na representao decimal de


A) 0 B) 1 C) 2 D) 7 E) 8

4 ? 37

EUREKA! N7, 2000

Sociedade Brasileira de Matemtica

09. Um retngulo ABCD est dividido em quatro retngulos menores. As reas de trs deles esto na figura abaixo. Qual a rea do retngulo ABCD?
A 16 27 C D

12 B

A) 80

B) 84

C) 86

D) 88

E) 91

10. Em um aqurio h peixes amarelos e vermelhos: 90% so amarelos e 10% so vermelhos. Uma misteriosa doena matou muitos peixes amarelos, mas nenhum vermelho. Depois que a doena foi controlada verificou-se que no aqurio, 75% dos peixes vivos eram amarelos. Aproximadamente, que porcentagem dos peixes amarelos morreram? A) 15% B) 37% C) 50% D) 67% E) 84% 11. Pedro saiu de casa e fez compras em quatro lojas, cada uma num bairro diferente. Em cada uma gastou a metade do que possua e a seguir, ainda pagou R$ 2,00 de estacionamento. Se no final ainda tinha R$ 8,00, que quantia tinha Pedro ao sair de casa? A) R$ 220,00 B) R$ 204,00 C) R$ 196,00 D) R$ 188,00 E) R$ 180,00 x + 99 12. Quantos so os possveis valores inteiros de x para que seja um x + 19 nmero inteiro? A) 5 B) 10 C) 20 D) 30 E) 40 13. A diferena entre a maior raiz e a menor raiz da equao (2 x 45)2 (x 21)2 = 0 : A) 2 B) 3 C) 4 D) 5 E) 6 14. Uma bola de futebol feita com 32 peas de couro. 12 delas so pentgonos regulares e as outras 20 so hexgonos tambm regulares. Os lados dos pentgonos so iguais aos dos hexgonos de forma que possam ser costurados. Cada costura une dois lados de duas dessas peas. Quantas so as costuras feitas na fabricao de uma bola de futebol? A) 60 B) 64 C) 90 D) 120 E) 180
EUREKA! N7, 2000

Sociedade Brasileira de Matemtica

15. Hoje, 12/6/1999, Pedro e Maria fazem aniversrio. No mesmo dia em 1996, a idade de Pedro era 3/4 da idade de Maria. No mesmo dia em 2002, a idade de Pedro ser igual de Maria quando ele tinha 20 anos. Quantos anos Maria est fazendo hoje? A) 30 B) 31 C) 32 D) 33 E) 34 16. Uma caixa contm 100 bolas de cores distintas. Destas, 30 so vermelhas, 30 so verdes, 30 so azuis e entre as 10 restantes, algumas so brancas e outras so pretas. O menor nmero de bolas que devemos tirar da caixa, sem lhes ver a cor, para termos a certeza de haver pelo menos 10 bolas da mesma cor : A) 31 B) 33 C) 35 D) 37 E) 38 17. Quantos so os tringulos que possuem medidas dos seus lados expressas por nmeros inteiros e tais que a medida do maior lado seja igual a 11 ? A) 10 B) 11 C) 12 D) 24 E) 36 18. Os pontos S, T e U so os pontos de tangncia do crculo inscrito no tringulo PQR sobre os lados RQ, RP e PQ respectivamente. Sabendo que os comprimentos dos arcos TU, ST e US esto na razo TU : ST : US = 5 : 8 : 11, a razo TPU : SRT : UQS igual a : A) 7 : 4 : 1 B) 8 : 5 : 2 C) 7 : 3 : 2 D) 11 : 8 : 5 E) 9 : 5 : 1 19. Aos vrtices de um cubo so atribudos os nmeros de 1 a 8 de modo que os conjuntos dos nmeros correspondentes aos vrtices das seis faces so {1, 2, 6, 7}, {1, 4, 6, 8}, {1, 2, 5, 8}, {2, 3, 5, 7}, {3, 4, 6, 7} e {3, 4, 5, 8}. O vrtice atribudo ao nmero 6 est mais longe do vrtice de nmero A) 1 B) 3 C) 4 D) 5 E) 7 20. Com os 5 nmeros mpares entre 5 e 4 e com os 5 nmeros pares entre 5 e 4 so formados 5 pares de nmeros. Se N a soma dos produtos, obtidos em cada par de nmeros, o valor mnimo possvel de N igual a : A) 41 B) 40 C) 28 D) 10 E) 0

EUREKA! N7, 2000

Sociedade Brasileira de Matemtica

XXI OLIMPADA BRASILEIRA DE MATEMTICA


Primeira Fase - Nvel 3
01. Veja problema 01 do Nvel 1. 02. Veja problema 02 do Nvel 2. 03. Um gafanhoto pula exatamente 1 metro. Ele est em um ponto A de uma reta, s pula sobre ela, e deseja atingir um ponto B dessa mesma reta que est a 5 metros de distncia de A com exatamente 9 pulos. De quantas maneiras ele pode fazer isso? A) 16 B) 18 C) 24 D) 36 E) 48 04. Sendo a b e b 0, sabe-se que as razes da equao x 2 + ax + b = 0 so exatamente a e b. Ento, a b igual a: A) 0 B) 1 C) 2 D) 3 E) 4 05. Veja problema 09 do Nvel 2. 06. Veja problema 14 do Nvel 2. 07. A diferena entre a maior raiz e a menor raiz da equao (2 x 45)2 (x 21)2 = 0 : A) 2 B) 3 C) 4 D) 5 E) 6 08. Veja problema 12 do Nvel 2. 09. Se 00 < x < 900 e cos x =
A) 00 e 300 1 ento x est entre: 4 B) 300 e 450 C) 450 e 600 D) 600 e 750

E) 750 e 900

10. Veja problema 11 do Nvel 2.

n se n par, 2 f (n ) = 3n + 1 se n mpar. O nmero de solues da equao f ( f ( f (n ))) = 16 : A) 2 B) 3 C) 4 D) 5 E) 6


11. Para todo n natural definimos a funo f por:

f (n ) =

12. O nmero N = 11111 . . . 11 possui 1999 dgitos, todos iguais a 1. O resto da diviso de N por 7 : A) 1 B) 2 C) 4 D) 5 E) 6
EUREKA! N7, 2000

10

Sociedade Brasileira de Matemtica

13. Um quadrado ABCD possui lado 40cm. Uma circunferncia contm os vrtices A e B e tangente ao lado CD. O raio desta circunferncia : A) 20cm B) 22cm C) 24cm D) 25cm E) 28cm 14. Veja problema 18 do Nvel 2. 15. Para quantos valores inteiros de x existe um tringulo acutngulo de lados 12, 10 e x? A) 9 B) 10 C) 12 D) 16 E) 18 16. A circunferncia abaixo tem raio 1, o arco AB mede 700 e o arco BC mede 400. A rea da regio limitada pelas cordas AB e AC e pelo arco BC mede:

A B

A) /8

B) /9

C) /10

D) /12

E) /14

17. A reta r contm os pontos (0, 4) e (7, 7). Dos pontos abaixo, qual o mais prximo da reta r? A) (1999, 858) B) (1999, 859) C) (1999, 860) D) (1999, 861) E) (1999, 862) 18. Quantos so os pares (x, y) de inteiros positivos que satisfazem a equao 2x +3y = 101 ? A) 13 B) 14 C) 15 D) 16 E) 17 19. Quantos nmeros inteiros entre 10 e 1000 possuem seus dgitos em ordem estritamente crescente? (Por exemplo, 47 e 126 so nmeros deste tipo; 52 e 566 no). A) 90 B) 98 C) 112 D) 118 E) 120 20. Veja problema 10 do Nvel 2. 21. Veja problema 15 do Nvel 2. 22. No quadrado ABCD o ponto E mdio de BC e o ponto F do lado CD tal que o ngulo AEF reto. Aproximadamente, que porcentagem a rea do tringulo AEF representa da rea do quadrado? A) 28% B) 31% C) 34% D) 36% E) 39%
EUREKA! N7, 2000

11

Sociedade Brasileira de Matemtica

23. Dois irmos herdaram o terreno ABC com a forma de um tringulo retngulo em A, e com o cateto AB de 84m de comprimento. Eles resolveram dividir o terreno em duas partes de mesma rea, por um muro MN paralelo a AC como mostra a figura abaixo. Assinale a opo que contm o valor mais aproximado do segmento BM.
B

M A

N C

A) 55m

B) 57m

C) 59m

D) 61m

E) 63m

24. As representaes decimais dos nmeros 21999 e 51999 so escritas lado a lado. O nmero de algarismos escritos igual a : A) 1999 B) 2000 C) 2001 D) 3998 E) 3999 25. Veja problema 16 do Nvel 2. GABARITO
Primeiro Nvel (5a. e 6a. sries)
1) B 2) D 3) B 4) B 5) D 6) A 7) B 8) C 9) A 10) B 11) E 12) E 13) B 14) E 15) C 16) B 17) E 18) A 19) D 20) C

Segundo Nvel (7a. e 8a. sries)


1) B 2) D 3) E 4) A 5) E 6) E 7) C 8) B 9) E 10) D 11) D 12) C 13) A 14) C 15) B 16) E 17) E 18) A 19) D 20) B

Terceiro Nvel (Ensino Mdio)


1) B 2) D 3) D 4) D 5) E 6) C 7) A 8) C 9) E 10) D 11) C 12) A 13) D 14) A 15) A 16) B 17) D 18) E 19) E 20) D 21) B 22) B 23) C 24) B 25) E

EUREKA! N7, 2000

12

Sociedade Brasileira de Matemtica

XXI OLIMPADA BRASILEIRA DE MATEMTICA


Segunda Fase - Nvel 1
PROBLEMA 1

Corte 10 algarismos do nmero 1234512345123451234512345, para que o nmero restante seja o maior possvel.
PROBLEMA 2

Sabe-se que trs meses consecutivos de um determinado ano, no bissexto, possuem cada um exatamente quatro domingos. Estes meses podem ser janeiro, fevereiro e maro? a) b) Podem ser agosto, setembro e outubro?
PROBLEMA 3

Na figura, os tringulos ABC e EGF so equilteros. O permetro do tringulo ABC 132cm e, alm disso, B AE = EC BD = DC EF = FC DG = GE D a) Qual o permetro da rea sombreada? G b) Que frao da rea do tringulo ABC representa a rea sombreada?
A E F C

PROBLEMA 4

Pedro distribuiu 127 moedas de 1 real em sete caixas e colocou em cada uma delas uma etiqueta dizendo o nmero de moedas da caixa. Essa distribuio foi feita de forma que qualquer quantia de R$1,00 a R$127,00 pudesse ser paga entregando-se apenas caixas fechadas. De que maneira Pedro fez essa distribuio?
PROBLEMA 5

Um edifcio muito alto possui 1000 andares, excluindo-se o trreo. Do andar trreo partem 5 elevadores: O elevador A pra em todos os andares. O elevador B pra nos andares mltiplos de 5, isto , 0, 5, 10, 15, O elevador C pra nos andares mltiplos de 7, isto , 0, 7, 14, 21, O elevador D pra nos andares mltiplos de 17, isto , 0, 17, 34, 51, O elevador E pra nos andares mltiplos de 23, isto , 0, 23, 46, 69,
EUREKA! N7, 2000

13

Sociedade Brasileira de Matemtica

a) Mostre que, excetuando-se o andar trreo, no existe nenhum andar onde param os 5 elevadores. b) Determine todos os andares onde param 4 elevadores.
PROBLEMA 6

Encontre o menor tabuleiro quadrado que pode ser ladrilhado usando peas com o seguinte formato:

Obs: Ladrilhado significa completamente coberto, sem superposio de peas, e de modo que nenhum ponto fora do tabuleiro seja coberto por alguma pea.

SOLUES SEGUNDA FASE - NVEL 1


SOLUO PROBLEMA 1

O maior nmero restante 553451234512345. Para ver isto, podemos supor que os cortes so feitos da esquerda para a direita. Se deixarmos de cortar todos os quatro primeiros algarismos, o nmero que resta comear por 1, 2, 3 ou 4. Logo, menor que o nmero acima. Feito isto, se deixarmos de cortar a segunda seqncia 1234, o nmero que resta ter na primeira ou segunda casa, da esquerda para a direita, 1, 2, 3 ou 4. Ainda menor que o nmero acima. Os dois primeiros 5 devem permanecer, pois retirando-se um deles, completamos 9 retiradas e a algum algarismo da terceira seqncia 1234 aparecer na 1a ou na 2a casa. Finalmente devemos cortar a seqncia 12, que ocupa a 11a e 12a posio.
SOLUO PROBLEMA 2

Se o dia primeiro de janeiro for Segunda-feira, e o ano no for bissexto, ento os meses de janeiro, fevereiro e maro tero 4 domingos cada.
SOLUO PROBLEMA 3 (Soluo resumida) a)

Permetro = 2 (44 ) + 3 = 121 .

b) S ' =

13 3 1 1 S+ . S= S 16 4 4 4

SOLUO PROBLEMA 4

Basta distribuir as moedas em 7 caixas contendo respectivamente 1, 2, 4, 8, 16, 32 e 64 moedas. Para outros pagamentos Pedro pode fazer 3 = 1 + 2, 5 = 1 + 4, 6 = 2 + 4, 7 = 1 + 2 + 4. Assim j pode pagar as quantias de 1 a 7 reais com o contedo das caixas. Somando-se a parcela de 8 a estas somas chega-se nas somas de 9 at 15. Somando-se a parcela de 16 s 15 somas assim formadas
EUREKA! N7, 2000

14

Sociedade Brasileira de Matemtica

obtm-se somas de 17 a 31. A estas acrescenta-se a parcela de 32. E finalmente a parcela de 64, obtendo-se assim todas as somas de 1 a 127 = 1+ 2 + 4 + 8 + 16 + 32 + 64.
SOLUO PROBLEMA 5

a)

O elevador B pra nos mltiplos de 5. O elevador C pra nos mltiplos de 7. O elevador D pra nos mltiplos de 17. O elevador E pra nos mltiplos de 23.

Como 5, 7, 17 e 23 so nmeros primos, para que todos parem num mesmo andar, este tem que ser mltiplo de 5 7 17 23 = 13685 e o prdio s tem 1000 andares. b) Para que num andar parem exatamente quatro elevadores, devem parar A, que pra em todos, e trs dos restantes. B, C e D param nos mltiplos de 5 7 17 = 595 B, C e E param nos mltiplos de 5 7 23 = 805 B, D e E param nos mltiplos de 5 17 23 = 1955 C, D e E param nos mltiplos de 7 17 23 = 2737 Logo, os andares onde param 4 elevadores so o 595 e o 805.

O menor tabuleiro do tipo 10 10 coberto com 20 peas, como mostrado, por exemplo, pela figura abaixo, esquerda. Com efeito, o nmero de casas do tabuleiro um quadrado perfeito mltiplo de 5. Logo 25, 100, 225 ou ... etc. Mas um tabuleiro 5 5 no pode ser coberto com peas deste tipo, pois ao tentarmos completar uma lateral do tabuleiro, seremos conduzidos a uma das duas figuras direita, as quais no se deixam completar pelas peas para formar todo o tabuleiro.

SOLUO PROBLEMA 6

EUREKA! N7, 2000

15

Sociedade Brasileira de Matemtica

XXI OLIMPADA BRASILEIRA DE MATEMTICA


Segunda Fase - Nvel 2
PROBLEMA 1

Trs meses consecutivos de um determinado ano, no bissexto, possuem exatamente quatro domingos cada um. Prove que um destes meses fevereiro.
PROBLEMA 2

Num quadro-negro so escritos trs inteiros. Comea-se, ento, uma sequncia de movimentos onde, em cada passo, apaga-se um deles e escreve-se em seu lugar a soma dos outros dois diminuda de uma unidade. Aps vrios movimentos, esto escritos no quadro os nmeros 17, 75 e 91. possvel que no incio estejam escritos no quadro : a) 2, 2, 2 ? b) 3, 3, 3 ?
PROBLEMA 3

Seja ABCD um quadrado. Escolhemos pontos M, N, P, Q respectivamente sobre AB, BC, CD e DA, de modo que as circunferncias circunscritas aos tringulos MBN e PDQ sejam tangentes exteriormente. Mostre que MN +PQ AC.
PROBLEMA 4

Determine o maior natural n para o qual existe uma reordenao (a, b, c, d) de (3, 6, 9, 12) (isto , {a, b, c, d} = {3, 6, 9, 12}) tal que o nmero n 3a 6 b9 c12 d seja inteiro. Justifique sua resposta.
PROBLEMA 5

Um professor de matemtica passou aos seus alunos a adio

A C + onde A, B, B D C e D so inteiros positivos, as fraes esto simplificadas ao mximo e os denominadores so nmeros primos entre si. Os alunos adicionaram as fraes tirando o mnimo mltiplo comum dos denominadores das parcelas e escrevendo este como o denominador do resultado. Mostre que a frao que os alunos encontraram como resultado est simplificada.
PROBLEMA 6

Determine todos os inteiros positivos n para os quais possvel montarmos um retngulo 9 10 usando peas 1 n.
EUREKA! N7, 2000

16

Sociedade Brasileira de Matemtica

SOLUES SEGUNDA FASE - NVEL 2


SOLUO PROBLEMA 1

Se nenhum destes meses for fevereiro, o nmero total de dias no pode ser menor que 91 = 7. 13 e portanto o nmero total de domingos no poderia ser menor do que 13.
SOLUO PROBLEMA 2

a) Esto escritos inicialmente 3 nmeros pares. Quando um deles apagado, escrito em seu lugar um nmero mpar. Aps o 1 movimento ficam ento dois nmeros pares e um nmero mpar. Se apagarmos agora o nmero mpar, surgir em seu lugar outro nmro mpar e se apagarmos um nmero par aparecer em seu lugar outro nmero par. Deste modo, aps qualquer nmero de movimentos restaro dois nmeros pares e um nmero mpar e portanto, no possvel termos no final os trs nmeros mpares 17, 75 e 91. b) Sim, uma possvel sequncia de movimentos : 3, 3, 3 5, 3, 3 5, 3, 7 5, 11, 7 17, 11, 7 17 , 11, 27 17, 43, 27 17, 43, 59 17, 75, 59 17, 75, 91.
SOLUO PROBLEMA 3

A figura abaixo representa a situao, onde X e Y so os pontos mdios dos segmentos MN e PQ e Z o ponto de tangncia das circunferncias. Ento, como MBN = PDQ = 90 , segue que BX = MX = NX = XZ e DY = QY = YP = YZ. Assim, MN + PQ = BX + XZ + ZY + YD BD = AC .
A Q D

M X

SOLUO PROBLEMA 4

Temos 3 a 6 b 9 c 12 d = 2 b + 2 d 3 a +b + 2 c + d . Para (a, b, c, d) dados, o maior n possvel mdc{b + 2d , a + b + 2c + d } b + 2d . Note que b + 2d mximo (com b e d elementos distintos de {3, 6, 9, 12}) quando d = 12 e b = 9. Neste caso, b + 2d = 33, e a + b + 2c + d = 21 + a + 2c. Tomando a = 6 e c = 3, temos tambm a + b + 2c + d = 33, que obviamente o maior valor possvel para n, obtido para (a, b, c, d) = (6, 9, 3, 12).
EUREKA! N7, 2000

17

Sociedade Brasileira de Matemtica

SOLUO PROBLEMA 5

Como os denominadores das fraes so primos entre si, seu MMC BD e assim, a frao resultante

AD + CB . Suponhamos que esta frao no seja irredutvel BD

isto , que exista algum nmero primo p que divida o numerador e o denominador desta frao. Como o produto BD divisvel por p, um dos seus termos, digamos B sem perda de generalidade o seja. Entretanto, uma das parcelas da soma AD + CB divisvel por p e como a soma, por hiptese, divisvel por p a parcela AD tambm divisvel por p. Portanto A ou D divisvel por p. No primeiro caso A temos uma contradio com o fato da frao ser irredutvel, no outro casos a B contradio est no fato de que os denominadores das fraes iniciais sempre so primos entre si.
SOLUO PROBLEMA 6

claro que n deve ser no mximo 10 e dividir 90. Assim, restam para n as possibilidades 1, 2, 3, 5, 6, 9, 10. Fora n = 6, imediato que n pode assumir qualquer um dos outros valores acima. Comeando a tentar montar o retngulo com peas 1 6 a partir de um canto, conclumos prontamente que a tarefa no possvel.

EUREKA! N7, 2000

18

Sociedade Brasileira de Matemtica

XXI OLIMPADA BRASILEIRA DE MATEMTICA


Segunda Fase - Nvel 3
PROBLEMA 1

Nos extremos de um dimetro de um crculo, escreve-se o nmero 1 (primeiro passo) . A seguir, cada semicrculo dividido ao meio e em cada um dos seus pontos mdios escreve-se a soma dos nmeros que esto nos extremos do semicrculo (segundo passo) . A seguir, cada quarto de crculo dividido ao meio e em cada um dos seus pontos mdios coloca-se a soma dos nmeros que esto nos extremos de cada arco (terceiro passo). Procede-se, assim, sucessivamente: sempre cada arco dividido ao meio e em seu ponto mdio escrita a soma dos nmeros que esto em seus extremos. Determinar a soma de todos os nmeros escritos aps 1999 passos.
PROBLEMA 2 Veja problema 3 do nvel 2. PROBLEMA 3 Veja problema 4 do nvel 2. PROBLEMA 4

Determine todos os inteiros positivos n para os quais possvel montarmos um retngulo 9 10 usando peas 1 n.
PROBLEMA 5

Jos tem trs pares de culos, um magenta, um amarelo e um ciano. Todo dia de manh ele escolhe um ao acaso, tendo apenas o cuidado de nunca usar o mesmo que usou no dia anterior. Se dia primeiro de agosto ele usou o magenta, qual a probabilidade de que dia 31 de agosto ele volte a usar o magenta?
PROBLEMA 6

Encontre as solues inteiras de x 3 y 3 = 999 .

SOLUES SEGUNDA FASE - NVEL 3


SOLUO PROBLEMA 1

Seja S(n) a soma dos termos em cada passo em um dos semicrculos. Observemos que S(1) = 2, S(2) = 4, e S(3) = 10. Deste modo, nos parece razovel conjecturar que S(n) = 3n 1 +1. Claramente, S(1) = 31 1 + 1. Os novos termos adicionados para formar Ln +1 representam somas de dois termos consecutivos de Ln e cada
EUREKA! N7, 2000

19

Sociedade Brasileira de Matemtica

termo de Ln, excetuando-se o primeiro e o ltimo, aparece em exatamente duas destas somas. Da, S(n +1) = S(n) + 2(S(n) 1) = 3S(n) 2 = 3(3n 1 + 1) 2 = 3(n + 1) 1 + 1. Levando em considerao o outro semicrculo, temos que a soma aps os 1999 passos igual a 2.(31999 1 + 1) 2 = 2. 31998
SOLUO PROBLEMA 2 Veja soluo do problema 3 do nvel 2. SOLUO PROBLEMA 3 Veja soluo do problema 4 do nvel 2. SOLUO PROBLEMA 4

claro que n deve ser no mximo 10 e dividir 90. Assim, restam para n as possibilidades 1, 2, 3, 5, 6, 9, 10. Fora n = 6, imediato que n pode assumir qualquer um dos outros valores acima. Comeando a tentar montar o retngulo com peas 1 6 a partir de um canto, conclumos prontamente que a tarefa no possvel.
SOLUO PROBLEMA 5

Sejam mn , an e cn as probabilidades de que no dia n ele use culos magenta, amarelo e ciano, respectivamente. Temos m1 = 1, a1 = c1 = 0 e mn +1 =
a n +1 =

1 mn mn + c n m + an , e c n +1 = n Como an + cn + mn = 1, temos m n +1 = . 2 2 2

an + cn , 2

1 ( 2) 2 n , e em 31 de agosto a probabilidade de que ele volte a 3 1 + 2 29 usar o magenta m31 = . 3


Assim, m n =
SOLUO PROBLEMA 6

Temos ( x y ) ( x 2 + xy + y 2 ) = 33 37 . Suponhamos x > y. Assim, os possveis valores de a = x y so 1, 3, 9, 27, 37, 3 37, 9 37, 27 37 e cada valor permite fazer y = x a e precisamos apenas verificar se as razes de 999 so inteiras. Na verdade, alguns destes valores so x 2 + x( x a) + ( x a) 2 = a obviamente inapropriados: a = x y x 3 y 3 0 (mod 3) , donde os valores 1 e 37 podem ser descartados. Por outro lado, se x y 3b temos

( x 3 y 3 ) 3b 3 , donde podemos descartar a 27. Os dois valores restantes, 3 e 9, so de fato possveis e do as quatro solues: (10,1), ( 1,10), (12,9) e (9,12).
EUREKA! N7, 2000

20

Sociedade Brasileira de Matemtica

XXI OLIMPADA BRASILEIRA DE MATEMTICA


Terceira Fase - Nvel 1
PROBLEMA 1

Diga como dividir um cubo em 1999 cubinhos. A figura mostra uma maneira de dividir um cubo em 15 cubinhos.

PROBLEMA 2

Emanuela, Marta e Isabel so trs nadadoras que gostam de competir e por isso resolveram organizar um desafio de natao entre elas. Ficou combinado o total de pontos para o primeiro, o segundo e o terceiro lugares em cada prova. A pontuao para primeiro lugar maior que a para o segundo e esta maior que a pontuao para o terceiro. As pontuaes so nmeros inteiros positivos. O desafio consistiu de vrias provas e ao final observou-se que Emanuela fez 20 pontos, Marta 9 pontos e Isabel 10. A primeira prova foi vencida por Isabel. (a) Quantas provas foram disputadas? (b) Determine o total de pontos para o primeiro, segundo e terceiro lugares.
PROBLEMA 3

Um reino formado por dez cidades. Um cidado muito chato foi exilado da cidade A para cidade B, que a cidade do reino mais longe de A. Aps um tempo, ele foi expulso da cidade B para a cidade C do reino mais longe de B. Sabe-se que a cidade C no a mesma cidade A. Se ele continuar sendo exilado dessa maneira, possvel que ele retorne cidade A? Nota: as distncias entre as cidades so todas diferentes.
PROBLEMA 4

Adriano, Bruno e Carlos disputaram uma srie de partidas de tnis de mesa. Cada vez que um jogador perdia, era substitudo pelo que estava a esperar. A primeira partida foi disputada por Adriano e Bruno. Sabe-se que Adriano venceu 12 partidas e Bruno 21. Quantas vezes Adriano e Bruno se enfrentaram?
EUREKA! N7, 2000

21

Sociedade Brasileira de Matemtica

SOLUES TERCEIRA FASE - NVEL 1


PROBLEMA 1 SOLUO DE MARIANA DE MORAES SILVEIRA (Belo Horizonte - MG)

O cubo deve ser dividido em 1000 cubinhos, ou seja 10 10 10, depois, devese pegar um deles e divid-lo novamente em 1000 cubinhos para que obtenhamos 1999 cubinhos. Assim teremos 1000 1 (que ser dividido) + 1000 = 1999 cubinhos.
PROBLEMA 2 SOLUO DE DIOGO DOS SANTOS SUYAMA (Belo Horizonte - MG)

a) Foram disputadas 3 provas. Como 20 + 10 + 9 = 39, o nmero de pontos distribuidos por prova s poderia ser 3 ou 13, pois estes so os nicos divisores de 39, a no ser o mesmo e o 1. Em consequncias, o nmero de provas tambm ser um desses nmeros. Porm, se forem disputadas 13 provas, s h uma maneira de se distribuir os pontos: 2 para o primeiro, 1 para o segundo e 0 para o terceiro. Entretanto, 0 no positivo, sendo assim descartada essa hiptese. b) J sabendo que so 3 provas, impossvel que a vencedora ganhe menos que 8 pontos, pois assim, Emanuela s conseguiria os 20 pontos fazendo 7, 7 e 6 pontos em cada prova. Para isso, seria preciso que a vencedora fizesse 7 pontos, a segunda colocada 6 e a ltima 0, mas como vimos, 0 no positivo. impossvel, tambm que a vencedora faa mais de 10 pontos, pois no seria possvel que a segunda fizesse mais pontos que a ltima, ou que esta no fizesse 0 pontos. Ento, as nicas possibilidades so: 1a. 10, 2a. 2, 3a. 1; 1a. 9, 2a. 3, 3a. 1; 1a. 8, 2a. 4, 3a. 1; e 1a. 8, 2a. 3, 3a. 2. A primeira opo incorreta, pois Isabel, que venceu uma das provas, no poderia ter feito pontos nas outras. A segunda opo tambm no correta, pois Isabel teria que marcar apenas um ponto em duas provas. A ltima opo incorreta tambm, pois Isabel teria que marcar 2 pontos em duas provas. Terceira opo: 1a. 8, 2a. 4, 3a. 1 a correta. Veja o quadro abaixo:

Emanuela Marta Isabel

1a. Prova 4 1 8

2a. Prova 8 4 1

3a. Prova 8 4 1

Total 20 9 10

PROBLEMA 3 Veja soluo do problema 2 do nvel 2. PROBLEMA 4 Veja soluo do problema 3 do nvel 2
EUREKA! N7, 2000

22

Sociedade Brasileira de Matemtica

XXI OLIMPADA BRASILEIRA DE MATEMTICA


Terceira Fase - Nvel 2
PROBLEMA 1

Seja ABCDE um pentgono regular tal que a estrela ACEBD tem rea 1. Sejam P interseo entre AC e BE e Q a interseo entre BD e CE. Determine a rea de APQD.
D Q

P A B

PROBLEMA 2

Um reino formado por dez cidades. Um cidado muito chato foi exilado da cidade A para a cidade B, que a cidade do reino mais longe de A. Aps um tempo, ele foi expulso da cidade B para a cidade C do reino mais longe de B. Sabe-se que a cidade C no a mesma cidade A. Se ele continuar sendo exilado dessa maneira, possvel que ele retorne cidade A?

Nota: as distncias entre as cidades so todas diferentes.


PROBLEMA 3

Adriano, Bruno e Carlos disputaram uma srie de partidas de tnis de mesa. Cada vez que um jogador perdia, era substitudo pelo que estava a esperar. A primeira partida foi disputada por Adriano e Bruno. Sabe-se que Adriano venceu 12 partidas e Bruno 21. Quantas vezes Adriano e Bruno se enfrentaram?
PROBLEMA 4

Prove que h pelo menos um algarismo diferente de zero entre a 1.000.000a. e a 3.000.000a. casa decimal de 2 aps a vrgula.

EUREKA! N7, 2000

23

Sociedade Brasileira de Matemtica

SOLUES TERCEIRA FASE - NVEL 2


PROBLEMA 1 Veja soluo do problema 1 do nvel 3.

PROBLEMA 2 SOLUO DE EINSTEIN DO NASCIMENTO JNIOR ( Fortaleza - CE )

H dez cidades A, B, C, D, E, F, G, H, I, J. Um chato da cidade A foi exilado para a cidade mais longe de A, a cidade B. Como B a cidade mais longe de A, pode-se dizer que se tomarmos A como sendo o centro de uma circunferncia de raio AB, todas as cidades estaro dentro dos limites da circunferncia, exceto a cidade B que estar em cima dela.
C

A E D

Como as distncias entre as cidades no so iguais e o chato foi exilado para a cidade C que a mais longe de B ento BC > AB. Da cidade C ele ser exilado para a cidade D que a mais longe de C e assim sucessivamente at chegar na cidade J onde teremos a seguinte verdade: AB < BC < CD < . . . < HI < IJ. Ao chegar nesse ponto vemos que A com certeza no a cidade mais longe de J pois AB = raio AJ < raio AJ < AB AB < IJ AJ < IJ Logo ele ir para uma cidade diferente de A, e nunca retornar cidade A.
PROBLEMA 3 SOLUO DE FBIO DIAS MOREIRA (Rio de Janeiro - RJ)

Quando comea a srie, j ocorre um encontro entre Adriano (A) e Bruno (B). Vamos chamar de VA, VB e VC o nmero de vitrias de Adriano, Bruno e Carlos, respectivamente. Ento ao final da srie VA + VB = 33 e depois do 1o. jogo VA + VB
EUREKA! N7, 2000

24

Sociedade Brasileira de Matemtica

= 1. Suponhamos que o segundo jogo seja x C. Chamemos de E o nmero de jogos A B. Ento no 2o. jogo E = 1. Enquanto C ganhar, VA + VB e E permanecem constantes. Quando C perder, VA + VB aumenta uma unidade. O prximo jogo ser A B, aumentando VA + VB e E em uma unidade. Aps este jogo, o prximo ser x C. Ou seja, para que E aumente uma unidade, VA + VB aumenta duas, e o aumento de um em E. Como no 2o. jogo E = 1 e falta que VA + VB aumente 32 unidades, ocorrem 1 + 16 = 17 jogos A B.
PROBLEMA 4 SOLUO DE HENRIQUE CHOCIAY (Pinhais - PR)

Para comear a desenvolver 2 , utilizei o processo de extrao que no utiliza tentativas (processo prtico por aproximao).

2
1 1.00 96 4.00 281 1.1900 1.1296 0060400

1,414 1 2 = 24 4 100 96 14 2 = 281 1 400 141 2 = 2824 4 11900 11296

Deste lado, o nmero de casas sempre aumenta em 1 casa, nunca mais. (mesmo se houvesse um caso de 99999 9 = 899991 (s aumenta 1 casa) (entre 1.000.000 e 3.000.000) Quando estivermos no nmero 1.000.000 de casas no multiplicador, teremos 999.999 casas decimais. Supondo que haja s 1 casa no resto nesta situao, depois de 1.000.000 de operaes, teremos 1.999.999 casas decimais (1 milho de zeros), 2.000.000 no multiplicador e 2.000.001 no resto, podendo obter nmero diferente de zero. Em geral o fato de, no podendo haver diviso, com o aumento das casas divisoras em 1 e do resto em 2 e as casas decimais serem menores que as divisoras em 1 torna impossvel a obteno desta seqncia de zeros entre as casas de 1.000.000 e 3.000.000.

EUREKA! N7, 2000

25

Sociedade Brasileira de Matemtica

XXI OLIMPADA BRASILEIRA DE MATEMTICA


Terceira Fase - Nvel 3
PRIMEIRO DIA PROBLEMA 1 Veja problema 1 do nvel 2. PROBLEMA 2 Veja problema 4 do nvel 2. PROBLEMA 3

Temos um tabuleiro quadrado 10 10. Desejamos colocar n peas em casas do tabuleiro de tal forma que no existam 4 peas formando em retngulo de lados paralelos aos lados do tabuleiro. Determine o maior valor de n para o qual possvel fazer esta construo.
SEGUNDO DIA PROBLEMA 4

O planeta Zork esfrico e tem vrias cidades. Dada qualquer cidade existe uma cidade antpoda (i.e., simtrica em relao ao centro do planeta). Existem em Zork estradas ligando pares de cidades. Se existe uma estrada ligando as cidades P e Q ento tambm existe uma estrada ligando as cidades P' e Q', onde P' a antpoda de P e Q' a antpoda de Q. Alm disso, estradas no se cruzam e dadas duas cidades P e Q sempre possvel viajar de P a Q usando alguma seqncia de estradas. O preo da Kriptonita em Urghs (a moeda planetria) em duas cidades ligadas por uma estrada difere por no mximo 100 Urghs. Prove que existem duas cidades antpodas onde o preo da Kriptonita difere por no mximo 100 Urghs.
PROBLEMA 5

Em Tumblia existem n times de futebol . Deseja-se organizar um campeonato em que cada time joga exatamente uma vez com cada um dos outros. Todos os jogos ocorrem aos domingos, e um time no pode jogar mais de uma vez no mesmo dia. Determine o menor inteiro positivo m para o qual possvel realizar um tal campeonato em m domingos.

EUREKA! N7, 2000

26

Sociedade Brasileira de Matemtica

PROBLEMA 6

Dado tringulo ABC mostre como construir com rgua e compasso um tringulo ABCde rea mnima com C ' AC , A' AB e B' BC tal que

B ' A' C ' = B A C e A' C ' B ' = A C B.

C'

B'

A'

SOLUES TERCEIRA FASE - NVEL 3


PROBLEMA 1 SOLUO DE HUGO PINTO IWATA (So Jos do Rio Preto - SP)
D Q

E S

P A B

Como o pentgono e a estrela so regulares, o quadriltero APQD um trapzio. A rea do trapzio APQD igual rea do tringulo APD somada do tringulo PQD. Como BDRP tambm um trapzio, RP// QD, ento a rea de PQD
EUREKA! N7, 2000

____

27

Sociedade Brasileira de Matemtica

igual de RQD. Como a estrela regular, a rea de RQD igual de ERS, ento, a rea de PQD igual de ERS. Assim a rea do trapzio APQD igual soma das reas dos tringulos APD e ERS, que igual figura APDRES, que exatamente metade da estrela toda. Resposta: A rea de APQD 0,5.
PROBLEMA 2 SOLUO DE HUMBERTO SILVA NAVES (Goinia - GO)

Suponhamos, por absurdo, que todos os algarismos das casas decimais entre a 1.000.000a. casa decimal e a 3.000.000a. casa decimal de 2 fossem zero, ento:

10 210 1010
6

10210 K = 10310 2 (onde K = 1010 2 ) 10210 K 10310 2 < 10210 K + 1,


mas como 10 210 K 10 310 um absurdo, pois
6 6

2 = 10 310
6

2 (onde x Z e x x < x + 1)

2 , (pois se no fosse teramos


6

2 = K / 1010 ,
1

2 irracional!) ento:
6

10 210 K < 10 310

2 < 10 210 K + 1

K 10
10 6

< 2<

K 10
6

K2 10210
6

<2<

K2 10 210
6

+
6

2K 10410
6

1 10610
6

K 2 < 2 10210

6 10 310 2K 1 < K 2 + 2106 + 4106 , 10 10


10 6

mas como K = 1010

2 Z, temos (pela definio de x ):

K 1010
logo:

2 < K +1

K 10
10 6

K 10
210 6

2 10
10 6

<

1 2K 1 2106 , 4 2 10

K 2 < 2 10 210 < K 2 + K < 2 10


2 210 6 2

2K
210 6

10 10 6 < K + 1 0 < 2 10 210 K 2 < 1, um absurdo, pois no


6

1
410 6

< K2 +

1 1 + 4106 < K 2 + 1 2 10

existe nenhum inteiro maior que 0 e menor que 1, disto conclumos que h um algarismo diferente de 0 nestas casas decimais. (Poderamos ter uma aproximao melhor pois 2K bem menor que 10 210 ). Obs: x denota a funo do "maior inteiro": o nico inteiro tal que

x x < x + 1.
EUREKA! N7, 2000

28

Sociedade Brasileira de Matemtica

PROBLEMA 3 SOLUO DA BANCA

O problema equivalente a encontrar subconjuntos A1, A2, , A10 do conjunto {1, 2, 3, , 10} cuja soma do nmero de elementos seja a maior possvel tais que a interseo de dois quaisquer deles tenha no mximo um elemento (Ai o conjunto das posies das peas na i-sima linha do tabuleiro). Se Ai tem ki

k i (k i 1) subconjuntos de 2 elementos no pode 2 2 = 45 subconjuntos pertencer a dois dos conjuntos Ai, e h no total C10
elementos ento h C k2i = de 2 elementos de {1, 2,,10}. Assim, devemos ter

i =1

10

k i (k i 1) 45. 2
i, j com kj > ki + 1, temos

k (k +1) (k j 1)(k j 2) ki (ki +1) k j (k j 1) Ck2i +1 + Ck2j 1 = i i + = + + ki +1 k j < Ck2i + Ck2j . 2 2 2 2 10 10 k i (k i 1) Assim para minimizar mantendo k i fixo devemos ter 2 i =1 i =1
2 k i k j 1 para todo i, j. Se observamos que 5C 4 + 5C 32 = 5 6 + 5 3 = 45,

Por

outro

lado,

se

existem

conclumos que se

i =1

10

k i (k i 1) 45 ento 2

k
i =1

10

5 4 + 5 3 = 35, valendo a igualdade se e s

se 5 dos ki so iguais a 4 e os outros 5 iguais a 3. Para que a contruo seja possvel nesse caso precisamos de que cada par de elementos aparea em exatamente um dos conjuntos Ai . Nesse caso, cada elemento de {1, 2, 3, 10} deve aparecer em 3 conjuntos com 4 elementos ou em um conjunto com 4 elementos e 3 conjuntos com 3 elementos (pois cada um dos outros 9 elementos aparece exatamente uma vez junto com ele). Como haveria 5 conjuntos com 4 elementos, o nmero mdio de conjuntos com 4 elementos aos quais cada elemento pertence 2, donde h elementos que pertencem a 3 conjuntos com 4 elementos (pois um elemento no pode pertencer a exatamente 2 conjuntos com 4 elemetos). Assim, podemos supor sem perda de generalidade que A1 = {1, 2, 3, 4}, A2 = {1, 5, 6, 7} e A3 = { 1, 8, 9, 10}, mais ento qualquer outro conjunto de 4 elementos deve estar contido em {2, 3, , 10}, e portanto deve intersectar um dos conjunto A1, A2, A3, A4, em pelo menos 2 elementos. Portanto, no possvel
EUREKA! N7, 2000

29

Sociedade Brasileira de Matemtica

que

k
i =1 i

10

seja igual a 35. Por outro lado possvel construir exemplos com

k
i =1

10

= 34, como abaixo:

A1 = {1, 2, 3, 4}, A2 = {1, 5, 6, 7}, A3 = { 2, 5, 8, 9}, A4 = {3, 6, 8, 10}, A5 = {1, 9, 10}, A6 = { 2, 7, 10}, A7 = {3, 7, 9}, A8 = {4, 5, 10}, A9 = { 4, 6, 9} e A10 = {4, 7, 8}.

PROBLEMA 4 SOLUO DE GILBERTO SANTOS DO NASCIMENTO (So Paulo - SP)

Seja C1' o antpoda de C1 . Vamos ligar C1 a C1' e vice-versa, formando uma linha fechada. Abaixo

C 'j o antpoda de C j para todo j.


C3 C2 C'1 C1 C'n

Cn

C'3

C'2

Agora, supondo que a diferena da Kriptonita de C1' para C1 seja maior que 100. Ento, vamos supor que (C2 C1) + (C3 C2) ++ (C1' Cn) > 100. Como ao
EUREKA! N7, 2000

30

Sociedade Brasileira de Matemtica

percorrer o caminho, temos de ter uma diferena zero ao chegarmos em C1 novamente, somando (C2' C1') + (C3' C2') ++ (C1 Cn') < 100. Agora, supondo que o Superman trace uma linha de C1 a C1' (esta linha no poderia ser uma estrada, pois | C1' C1| > 100) a soma das diferenas na parte de cima da linha deve ser maior que 100 e embaixo menor que 100. > 100
Parte de cima (p.c.)
C1

Parte de baixo (p.b.)

C'1

Agora, supondo que esta linha percorra a figura, ligando todas as cidades antpodas, na parte de cima, a soma deve continuar sendo maior que 100 e embaixo menor que 100. Em p.c. (parte de cima), a soma no pode passar bruscamente de > 100 para < 100, pois so somadas apenas duas diferenas de cada vez (menores que 200 no total!). Assim, para que p.c. fique negativo < 100 e p.b. fique positivo > 100, teramos de ter duas cidades antpodas com diferena > 100 em mdulo.
> 100

Ck

C1'
C1 < 100 p.b.

p.c. p.b.

p.c. > 100 C1'

< 100

C2'

Ck'

Continuando o percurso, ao chegarmos em C1', teremos de lig-lo a C1. No entanto, p.c. estar em baixo e a soma das diferenas na direo de C1' para C1 ter de ser positivo > 100. Mas essa soma era negativa e < 100 quando comeamos () Contradio. O mesmo ocorre analogamente com p.b. Logo, em algum par da cidades (uma cidade e sua antpoda), a diferena do preo da Kriptonita dever ser menor ou igual a 100. Viva o Superman!.

EUREKA! N7, 2000

31

Sociedade Brasileira de Matemtica

PROBLEMA 4 SOLUO DE HUMBERTO SILVA NAVES (Goinia - GO)

Suponhamos, por absurdo, que os preos diferem por mais de 100 Urghs em todas as cidades antpodas, ento: | x0 y0| > 100 M0 m0 > 100 (onde xn e yn so antpodas e representam o preo da Kriptonita). | x1 y1| > 100 M1 m1 > 100 | xn yn| > 100 Mn mn > 100 (Onde Mn = mx (xn, yn) e mn = min (xn, yn)) Como sabemos que existe um caminho de estradas que leva de M0 at m0, ento deve existir uma estrada que liga (para certo i, j N; i, j n) Mi mj. Como existe uma estrada ligando Mi mj, tambm existe uma estrada ligando mj Mi (antpodas). Pode acontecer i = j, caso em que se conclui facilmente que Mi mi > 100, um absurdo pois mi e Mi so "vizinhas", logo o preo da Kriptonita difere por no mximo 100 Urghs. Se i j, ento: | Mj mi | 100 (so "vizinhas") | Mi mj | 100, mas como Mi mi > 100 e Mj mj > 100, ento: Mi + Mj mi mj > 200 Mi mj + Mj mi > 200 | Mi mj + Mj mi| > 200 | Mi mj| + | Mj mi| > 200 200 | Mi mj| + | Mj mi| > 200, um absurdo, logo existem cidades antpodas cujo preo difere no mximo em 100 Urghs.
PROBLEMA 5 SOLUO DE FABRCIO SIQUEIRA BENEVIDES (Fortaleza - CE)

Faamos 2 casos, n par e n mpar. i) n par.

Cada time tem que jogar com cada um dos outros. Se os times so: T1, T2, , Tn; temos que um time Ti tem que jogar (n 1) vezes e para isso precisar de pelo menos (n 1) domingos. (pois s pode jogar 1 vez por domingo). Mostraremos que possvel realizar o campeonato em (n 1) domingos. Para isso basta que o jogo entre Ti e Tj (i j) ocorra no seguinte domingo.

EUREKA! N7, 2000

32

Sociedade Brasileira de Matemtica

1) 2)

dij i + j (mod n 1), 1 dij n 1 para i n, j n din 2i (mod n 1), 1 din n 1 para todo i n, j n (se um dos times for Tn).

Podemos observar isso numa tabela que indique o dia entre Ti e Tj Exemplo: para n = 6

dij T1 T2 T3 T4 T5 T6

T1 3 4 5 1 2

T2 3 5 1 2 3

T3 4 5 2 3 4

T4 5 1 2 4 5

T5 1 2 3 4 1

T6 2 4 1 3 5

O campeonato organizado assim satisfaz o problema pois: fcil ver que um time i joga com cada um dos outros times (no domingo dij, j i). E cada time s joga uma vez num mesmo dia, caso contrrio teramos: um time Ti que joga contra Tj e Tk no mesmo domingo, ou seja dij = dki 1) Se i = n: djn = dkn 2j 2k (mod n 1) como (2, n 1) = 1 teriamos j k (mod n 1), {j, k} {1, 2, , n 1} j = k, uma contradio. 2) Se i n. 2.1)
2.2)

j e k n : dik = dij i + k i + j (mod n 1) j k (mod n 1) e k = j. uma contradio. j = n, k n, sem perda de generalidade: din = dik i + i i + k (mod n 1) i k (mod n 1), {i, j}{1, 2, , n 1} i = j, uma contradio.

Agora se n for mpar, como cada time tem que jogar com todos os outros seria necessrio pelo menos (n 1) domingos. S que (n 1) domingos no so suficientes pois em cada dia h um time que fica sem jogar. Assim, se no primeiro dia Ti foi o time que no jogou, ele ainda precisar de mais ( n 1) domingos para jogar contra os outros. De modo que so necessrios pelo menos n domingos.
EUREKA! N7, 2000

33

Sociedade Brasileira de Matemtica

Para ver que n domingos so suficientes, basta que o campeonato se organize assim: Sejam T1, T2, , Tn os times. Criamos um time virtual chamado Tn + 1 onde jogar contra Tn + 1 um certo dia, significa no jogar naquele dia. Temos ento n + 1 = x times, organizamos ento como no caso anterior o campeonato. Como x par isso pode ser feito em x 1 = n dias. Obs: O exemplo para (2k 1) times obtido do de (2k) times esquecendo-se um dos times.

Resposta:

Se n par m = n 1. Se n mpar m = n.

PROBLEMA 6 SOLUO DA BANCA


C

c B'

C'

a A a A'

Sejam A, B, C os ngulos internos do tringulo ABC, sejam A', B', C' os ngulos internos do tringulo A'B'C' e consideremos A' = A e C' = C. Seja D o ponto de interseo das circunferncias circuscritas aos tringulos AA'C' e CC'B'. Nos quadrilteros inscritveis AA'DC' e CC'DB' temos A'DC' = A e C'DB' = C. Logo, A'DB' = 2 ( A) ( C) = B, e portanto, a circunferncia circunscrita ao tringulo BB'A' passa por D. No quadriltero inscritvel AA'DC', DAA' = DC'A' = e DA'C' = DAC' = a. Como A = A' concluimos que DA'B' = . Logo, no quadriltero inscritvel BB'DA' temos que DBB' = . No quadriltero inscritvel CC'DB' temos que DCB' = DC'B' = c, e como C = C' conclumos que DCC' = .
EUREKA! N7, 2000

34

Sociedade Brasileira de Matemtica

O ponto D est ento associado ao tringulo ABC pela propriedade: DAB = DBC = CDA e portanto no depende da posio de A', B' e C'. O ponto D fixo e sua construo ser mostrada no final da soluo. Como os ngulos A'DB', B'DC' e C'DA' so constantes, a menor rea possvel do tringulo A'B'C' obtida quando os segmentos DA', DB' e DC' forem os menores possveis. Logo, DA', DB' e DC' so respectivamente perpendiculares aos lados AB, BC e CA.

Construo do ponto D
Seja E a interseo da mediatriz de AB com a perpendicular a BC traada por B. A circunferncia de centro E e raio EA = EB tangente em B reta BC. Logo, para qualquer ponto X do menor arco AB tem-se que XAB = XBC. Seja F a interseo da mediatriz de BC com a perpendicular a CA traada por C. A circunferncia de centro F e raio FB = FC tangente em C reta CA. Logo, para qualquer ponto X do menor arco BC tem-se que XBC = XCA. O ponto D, interseo desses dois arcos tal que DAB = DBC = DCA. (Note que qualquer ponto D com esta propriedade deve pertencer a cada um dos lugares geomtricos descritos acima, o que nos d a unicidade).

EUREKA! N7, 2000

35

Sociedade Brasileira de Matemtica

XXI OLIMPADA BRASILEIRA DE MATEMTICA


Resultado - Primeiro Nvel (5a. e 6a. sries)
NOME
Henry Wei Cheng Hsu Diogo dos Santos Suyama Sergio Santos do Nascimento Gustavo Eufrsio Farias Luciano Lacerda Silveira Emanuel Augusto Varussa Padovan Fabrcio Henrique de Faria Thiago Jorge Marinho Vieira Paulo Roberto Sampaio Santiago Mariana de Moraes Silveira Gabriel Vieira Lana Joo Cludio Telles Vianna Rafael Daigo Hirama Ana Cludia de Franco Suzuki Luiza de Almeida Aoki Bruno Leonardo Schneider Paulo Rebello Bortolini Victor Mesquita Barbosa Thiago Augusto Caldas Bello Sinuhe Djin Maschio Shin Raul Mximo Alexandrino Nogueira Bruno Fiorio Pedro H. Milet Pinheiro Pereira Bernardo Melo Sobreira Mrio Luiz Aranha da Silva Conrado F. Paulo da Costa Rodrigo Aguiar Pinheiro Daniel Medeiros de Albuqerque Gabriela Duarte Costa Constantino Tiago Porto Barbosa Vitor Henrique Gonalves Gabriel Tom de Lima Gustavo Pinheiro Melo Tlio Ivo Cordeiro Fullio Leonardo Lucas Rentz Daniela Satie Kondo Rafael Santos Correia de Araujo Felipe Paupitz Schlichting linson Santos Xavier Antonia Taline de Souza Mendona Gustavo Hbner Leonardo Deeke Boguszewski Paola Valente Giorgini Roberta Pieroni Visconti Alan Hideki Uchida Cincinato Furtado Leite Neto Marcus Edson Barreto Brito Thiago de S Jorge Vento Inte Nunes Vieira

CIDADE ESTADO
So Paulo SP Belo Horizonte MG So Paulo SP Fortaleza CE Campo Grande MS Rio Claro SP So Paulo SP Fortaleza CE Salvador BA Belo Horizonte MG Belo Horizonte MG Rio de Janeiro RJ Campinas SP So Paulo SP S. J. dos Campos SP So Jos SC Jundia SP Fortaleza CE Salvador BA So Paulo SP Fortaleza CE Fortaleza CE Rio de Janeiro RJ Fortaleza CE Salvador BA Rio de Janeiro RJ Fortaleza CE Fortaleza CE Timteo MG Fortaleza CE So Carlos SP Mogi das Cruzes SP Fortaleza CE Campina Grande PB Macei AL So Paulo SP Salvador BA Florianpolis SC Fortaleza CE Fortaleza CE Campina Grande PB Curitiba PR Rio de Janeiro RJ So Paulo SP So Paulo SP Fortaleza CE Fortaleza CE Curitiba PR Curitiba PR

PRMIO
Ouro Ouro Ouro Ouro Prata Prata Prata Prata Prata Prata Bronze Bronze Bronze Bronze Bronze Bronze Bronze Bronze Bronze Bronze Bronze Bronze Bronze Bronze Bronze Bronze Bronze Bronze Bronze Meno Honrosa Meno Honrosa Meno Honrosa Meno Honrosa Meno Honrosa Meno Honrosa Meno Honrosa Meno Honrosa Meno Honrosa Meno Honrosa Meno Honrosa Meno Honrosa Meno Honrosa Meno Honrosa Meno Honrosa Meno Honrosa Meno Honrosa Meno Honrosa Meno Honrosa Meno Honrosa

EUREKA! N7, 2000

36

Sociedade Brasileira de Matemtica

Resultado - Segundo Nvel (7a. e 8a. sries)


NOME
Henrique Chociay Davi Mximo Alexandrino Nogueira Maurcio Massao Soares Matsumoto Fbio Dias Moreira Eduardo Kunio Kuroda Abe Larissa de Lima Einstein do Nascimento Jnior Diego Cortez Gutierrez Bernardo Freitas Paulo da Costa Bruno Koga Rafael Tajra Fonteles Andr Luis Hirschfeld Danila Rodrigo Barbosa dos Santos Stein Daniel Pessa Martins Cunha Jaquelyne Gurgel Penaforte Thiago Braga Cavalcante Henrique Cortada Barbieri Vincius Piovesan de Toledo Lucas Gabriel Maltoni Romano Danilo Vieira Castejon Thiago da Silva Sobral Guilherme Oliveira Campos Eduardo Barbosa Arajo Tatyana Zabanova Rafael Montorfano Franco Otaclio Torres Vilas Boas Henrique Fernandes Macedo Vinicius de Aguiar Furvie Renato R. Sinohara da S. Souza Daniel Teixeira Jefferson Ho Yun Lee Kiyoshi Horie Filho Fbio Eiji Arimura Toni Chenson Wang Guilherme Tosi Yuri Gomes Lima Thiago Mizuta Lucas Sber Rocha Daniel Nascimento Duplat Tiago Monteiro Fernandes Caio Ribeiro de Souza Adalberto Studart Neto Renato Arajo Barbosa Cibele Ferreira de Souza Marina Lima Medeiros Carolina Nunes Nery Germanna Oliveira Queiroz Luciana Akemi Nishimaru Fabiano Siggelkow Linhares SandraTie Nishibe Minamoto Daniel Haanwinckel Junqueira Solleon Natus Tavares de Menezes Anna Laura Sfredo Bruno Gomes Coelho Daniel Brscia dos Reis Andr Bastos Veras LincolnYoshyiti Hamaji Joo Paulo Aguiar Santos Bruno Bozon Furlan Mrcio Antonio Ferreira Belo Filho Joo Felipe Almeida Destri Caio Bria de Oliveira Patrick Gonalves Larissa Goulart Rodrigues Eduardo Horai

CIDADE ESTADO
Pinhais PR Fortaleza CE So Paulo SP Rio de Janeiro RJ So Paulo SP Fortaleza CE Fortaleza CE S. J. dos Campos SP Rio de Janeiro RJ Fortaleza CE Teresina PI So Paulo SP Vitria ES Fortaleza CE Fortaleza CE Fortaleza CE So Paulo SP Jundia SP Jundia SP Goinia GO Fortaleza CE Bauru SP Fortaleza CE Campinas SP Maring PR Salvador BA Juiz de Fora MG So Paulo SP S. J. dos Campos SP Braslia DF So Paulo SP Ourinho SP So Paulo SP So Paulo SP Nova Vencia ES Fortaleza CE So Paulo SP Maca RJ Salvador BA Rio Claro SP Rio de Janeiro RJ Fortaleza CE Sete Lagoas MG Mineiros GO Fortaleza CE Belo Horizonte MG Fortaleza CE So Paulo SP So Paulo SP Mogi das Cruzes SP Salvador BA Fortaleza CE So Paulo SP So Paulo SP Belo Horizonte MG Teresina PI So Paulo SP Juiz de Fora MG So Paulo SP Goinia GO Florianpolis SC S. J. dos Campos SP Jaguar ES Goinia GO So Paulo SP

PRMIO
Ouro Ouro Ouro Ouro Ouro Prata Prata Prata Prata Prata Prata Prata Prata Bronze Bronze Bronze Bronze Bronze Bronze Bronze Bronze Bronze Bronze Bronze Bronze Bronze Meno Honrosa Meno Honrosa Meno Honrosa Meno Honrosa Meno Honrosa Meno Honrosa Meno Honrosa Meno Honrosa Meno Honrosa Meno Honrosa Meno Honrosa Meno Honrosa Meno Honrosa Meno Honrosa Meno Honrosa Meno Honrosa Meno Honrosa Meno Honrosa Meno Honrosa Meno Honrosa Meno Honrosa Meno Honrosa Meno Honrosa Meno Honrosa Meno Honrosa Meno Honrosa Meno Honrosa Meno Honrosa Meno Honrosa Meno Honrosa Meno Honrosa Meno Honrosa Meno Honrosa Meno Honrosa Meno Honrosa Meno Honrosa Meno Honrosa Meno Honrosa Meno Honrosa

EUREKA! N7, 2000

37

Sociedade Brasileira de Matemtica

Resultado - Terceiro Nvel (Ensino Mdio)


NOME
Daniel Massaki Yamamoto Daniel Nobuo Uno Ulisses Medeiros Albuquerque Humberto Silva Naves Carlos Stein Naves de Brito Lucas Heitzmann Gabrielli Fabrcio Siqueira Benevides Giuliano Boava Jnathas Digenes Castello Branco Ronaldo Ikaro Farias Arajo Carlos Emanuel Rodrigues Nogueira Daniel Mouro Martins Gilberto Santos do Nascimento Rogrio Uhlmann Yamauti Fernando Silva Barros Leandro dos Santos de Jesus Hugo Pinto Iwata Leandro de Mattos Ferreira Bruno Fernandes Cerqueira Leite Adenilson Pereira Bonfim Mnica Mitiko Soares Matsumoto Leonardo da Costa Linhares Tertuliano Franco Santos Franco Arthur Duarte Nehmi Paulo Csar de Melo Hanaoka Joo Alfredo Castellani F. Freire Eduardo Famini Silva Lvia Camargos Rodrigues Oliveira Roberto Tiburcio Canito Frota Rui Facundo Vigelis Carlos Yuji Hatae Daniel Pinheiro Sobreira Pedro Paulo de Simoni Gouvia Thiago Barros Rodrigues Costa Mauricio Masayuki Honda Christian Lyoiti Watanabe Guilherme Goettems Schneider Danilo Castello Branco A. Bessa Bruno Woltzenlogel Paleo Camila Shirota Miriam Ou Digo Veloso Ucha Pedro Ferreira Celio Hira Gustavo Maltez Lengler Fernando Duarte Menezes Fernando Prado Rocha Paulo Henrique Jacob Silva Zhang He Renato Takamatsu Ulisses Duarte Nehmi Eduardo Moraes de Morais Humberto Vinhais Ilan Felts Almog Pietro Kreitlon Carolino Ivo Almino Gondim
EUREKA! N7, 2000

CIDADE ESTADO
So Paulo SP So Paulo SP Fortaleza CE Goinia GO Goinia GO So Paulo SP Fortaleza CE Cricima SC Fortaleza CE Fortaleza CE Fortaleza CE Fortaleza CE So Paulo SP So Paulo SP C. Lafaiete MG Rio de Janeiro RJ S. Jos do Rio Preto SP Rio de Janeiro RJ So Paulo SP Belm PA So Paulo SP Rio de Janeiro RJ Salvador BA So Paulo SP Campo Grande MS Salvador BA Salvador BA Belo Horizonte MG Fortaleza CE Fortaleza CE So Paulo SP Fortaleza CE Fortaleza CE Fortaleza CE So Paulo SP Itagua RJ So Leopoldo RS So Paulo SP Piracicaba SP Piracicaba SP So Paulo SP Teresina PI Fortaleza CE So Paulo SP Rio de Janeiro RJ Fortaleza CE Goinia GO So Paulo SP So Paulo SP So Paulo SP So Paulo SP So Paulo SP So Paulo SP So Paulo SP Salvador BA Fortaleza CE

PRMIO
Ouro Ouro Ouro Ouro Prata Prata Prata Prata Prata Prata Bronze Bronze Bronze Bronze Bronze Bronze Bronze Bronze Bronze Bronze Bronze Bronze Bronze Bronze Bronze Meno Honrosa Meno Honrosa Meno Honrosa Meno Honrosa Meno Honrosa Meno Honrosa Meno Honrosa Meno Honrosa Meno Honrosa Meno Honrosa Meno Honrosa Meno Honrosa Meno Honrosa Meno Honrosa Meno Honrosa Meno Honrosa Meno Honrosa Meno Honrosa Meno Honrosa Meno Honrosa Meno Honrosa Meno Honrosa Meno Honrosa Meno Honrosa Meno Honrosa Meno Honrosa Meno Honrosa Meno Honrosa Meno Honrosa Meno Honrosa Meno Honrosa

38

Sociedade Brasileira de Matemtica

EQUAES DIOFANTINAS
Antonio Caminha Muniz Neto Nvel Intermedirio
Denominaremos equao diofantina (em homenagem ao matemtico grego Diofanto de Alexandria) uma equao em nmeros inteiros. Nosso objetivo ser estudar dois tipos particulares de equaes diofantinas, a equao de Pitgoras e a de Pell, e determinar suas solues. Tambm estudaremos o mtodo da descida, que nos permitir mostrar que algumas equaes diofantinas no possuem solues no triviais, num sentido a ser precisado.

Ternos Pitagricos
Queremos estudar as solues (x, y, z) da equao x 2 + y 2 = z 2 , com x, y, z inteiros no nulos. Aps determinar tais solues, vamos ver como podemos utilizar as informaes obtidas para resolver outras equaes em nmeros inteiros. O resultado fundamental o seguinte

Teorema 1: As solues (x, y, z) da equao x 2 + y 2 = z 2 , com x, y, z inteiros


no nulos, so dadas por: ( x , y , z ) = ( 2uvd , ( u 2 v 2 )d , ( u 2 + v 2 )d ) ou ( x, y, z ) = ((u 2 v 2 )d ,2uvd ,(u 2 + v 2 )d ) onde d, u, v so inteiros no nulos, com u v, mdc(u, v) = 1 e u e v de paridades distintas.

Prova: Sejam x, y, z inteiros positivos quaisquer satisfazendo a equao acima (os demais casos so anlogos), e d o mdc de x e y. Ento d 2 divide z 2 , e da d divide z. Existem portanto inteiros no nulos a, b, c, com mdc(a, b) = 1, tais que (x, y, z) = (da, db, dc). Ademais, como

x 2 + y 2 = z 2 a 2 + b2 = c 2 ,
basta determinarmos as solues (a, b, c) da equao, sujeitas condio mdc(a, b) = 1 (que por sua vez implica mdc(a, c) = 1 e mdc(b, c) = 1). Note agora que, dado um inteiro qualquer t, temos que t 2 deixa resto 0 ou 1 na diviso por 4, quando t for respectivamente par ou mpar. Assim, se fossem a e b mpares, teramos a 2 e b 2 deixando resto 1 na diviso por 4, e da c 2 = a 2 + b 2 deixaria resto 2 quando dividido por 4, o que um absurdo. Como a e b so primos entre si, no podem ser ambos pares. H ento dois casos: a mpar e b par, a par e b mpar. Analisemos o primeiro caso (o segundo anlogo).
EUREKA! N7, 2000

39

Sociedade Brasileira de Matemtica

Se a for mpar e b par, ento c tambm mpar. De a 2 + b 2 = c 2 obtemos


c a c + a ento escrever ( b 2 ) =( 2 )( 2 ) . Note que 2

a (c ) e ( c+2a ) so primos entre si. 2 a c+a ) um quadrado Mas se o produto de dois naturais primos entre si ( c 2 e 2

b 2 = ( c a )( c + a ) , e no difcil concluir que mdc(c a, c + a) = 2. Podemos

perfeito, ento cada um deles deve ser um quadrado perfeito. Existem ento inteiros positivos primos entre si u e v, tais que c a = 2v 2 , c + a = 2u 2 , e da

( a, b, c ) = ( u 2 v 2 , 2uv , u 2 + v 2 ) .
Note ainda que, como u 2 + v 2 = c mpar, u e v devem ter paridades distintas. Por substituio na equao original, conclumos que os ternos acima so realmente solues da equao, de modo que nada mais h a fazer. Vemos ento que h uma quantidade infinita de ternos (x, y, z) satisfazendo a equao acima. Por exemplo, fazendo d = v = 1 e u = 2n, n inteiro positivo, obtemos o terno ( x , y , z ) = ( 4n, 4n 2 1, 4n 2 + 1) Um terno de inteiros positivos (x, y, z) tais que x 2 + y 2 = z 2 denominado um terno Pitagrico, em aluso ao matemtico grego Pitgoras e seu famoso teorema sobre tringulos retngulos. De fato, um tal terno (x, y, z) determina um tringulo retngulo de catetos x e y e hipotenusa z inteiros.

x y

Vejamos em que a equao acima pode ajudar na soluo de outros problemas. Consideremos a tarefa de determinar as solues inteiras no nulas da equao

x 2 + y 2 = 2 z 2 , com x y. Em uma qualquer dessas solues, devemos ter x e y


com a mesma paridade, pois caso contrrio x 2 + y 2 seria um nmero mpar. Assim, existem inteiros a e b tais que

x = a + b, y = a b

Basta tomarmos a = ( x + y ) e b = 1 2 ( x y ) , notando que x + y e x y so nmeros pares. Substituindo as expresses acima para x e y na equao original, conclumos que
1 2
EUREKA! N7, 2000

40

Sociedade Brasileira de Matemtica

x 2 + y 2 = 2 z 2 a 2 + b2 = z 2
Mas essa ltima equao a nossa j conhecida equao de Pitgoras. Ento, de acordo com o teorema acima, podemos escrever

(a, b, z ) = (2uvd,(u2 v 2 )d,(u2 + v 2 )d ) ou (a, b, z ) = ((u2 v 2 )d, 2uvd,(u2 + v 2 )d )


onde d, u, v so inteiros no nulos, com u v, mdc(u, v) = 1 e u e v de paridades distintas. Segue da que as solues (x, y, z) de nossa equao so de um dos tipos abaixo, onde d, u, v satisfazem as mesmas condies do teorema acima.

(x, y, z) = (2uvd +(u2 v2 )d, 2uvd (u2 v2 )d,(u2 + v2 )d)


ou

(x, y, z) = ((u 2 v2 )d + 2uvd,(u 2 v2 )d 2uvd,(u 2 + v2 )d )


Descida de Fermat e Equaes sem Solues As equaes analisadas acima so, em um certo sentido, privilegiadas, pois possuem uma infinidade de solues. Nosso prximo exemplo ser o de uma equao que s admite a soluo inteira x = y = z = 0. Ela ilustra um mtodo que pode ser estendido a outras equaes, a fim de provar que elas no possuem solues inteiras no nulas. Exemplo 1: A equao 3x 2 + y 2 = 2 z 2 no possui solues inteiras no nulas. Prova: Suponha o contrrio. Ento a equao possui uma soluo (x, y, z) em inteiros positivos. Ento, dentre todas as solues (x, y, z), com x, y e z inteiros positivos, existe uma (x, y, z) = (a, b, c) para a qual z = c o menor possvel. Trabalhemos tal soluo.
Vamos usar o seguinte fato, que voc pode provar facilmente: se um inteiro u no for mltiplo de 3, ento u 2 deixa resto 1, quando dividido por 3. Ento, se b no for mltiplo de 3, teremos de 3a 2 + b 2 = 2c 2 que c tambm no ser mltiplo de 3. Olhando os restos de cada termo da equao por 3, teremos que 3a 2 + b 2 deixa resto 1 e 2c 2 deixa resto 2.1 = 2. Logo, no poderia ser 3a 2 + b 2 = 2c 2 . Assim, b deve ser mltiplo de 3, digamos b = 3b1 . Da vem que

3a 2 + 9b12 = 2c 2 , e c tambm mltiplo de 3, digamos c = 3c1 . Substituindo na


2 equao, chegamos a 3b12 + a 2 = 6c1 .
EUREKA! N7, 2000

41

Sociedade Brasileira de Matemtica

Ento, a tambm mltiplo de 3. Sendo a = 3a1 , a equao acima nos d


2 2 , e ( b1 , a1 , c1 ) uma outra soluo de nossa equao original, b12 + 3a1 = 2c1 c com c1 = 3 < c . Mas isso uma contradio, pois partimos de uma soluo na

qual o valor de z era c, mnimo possvel. Logo, nossa equao no possui solues no nulas. Esquematicamente, o mtodo da descida (devido ao matemtico francs Pierre Simon de Fermat) consiste ento no seguinte:

i. Supor que uma dada equao possui uma soluo em inteiros no nulos. ii. Concluir da que ela possui uma soluo em inteiros positivos que seja, em
algum sentido, mnima.

iii. Deduzir a existncia de uma soluo positiva menor que a mnima, chegando
a uma contradio. J que determinamos acima as solues da equao de Pitgoras, nada mais natural que tentar estudar a equao mais geral abaixo, denominada equao de Fermat. Aqui, n > 2 um inteiro fixado.

xn + yn = zn ,
Por cerca de trs sculos os matemticos defrontaram-se com o problema de decidir sobre a existncia de solues no nulas (x, y, z) dessa equao, problema que somente foi resolvido na dcada de noventa, utilizando mtodos muitssimo complexos. Vamos aproveitar o mtodo da descida para analisar um caso simples dessa equao, aquele em que n um mltiplo de 4. O leitor interessado em saber mais sobre essa equao pode consultar uma das referncias [2] ou [3] da bibliografia, onde o caso n = 3 discutido.

Teorema 2: Se n for mltiplo de 4 ento no existem inteiros no nulos x, y, z tais


que x n + y n = z n .

Prova:

Seja

4 k,

natural.

Se

x n + y n = z n , ento teremos

( x k ) 4 + ( y k ) 4 = ( z 2 k )2 , ou seja, ( x k , y k , z 2 k ) ser uma soluo da equao

a 4 + b 4 = c 2 . Assim, basta mostrarmos que essa ltima equao no admite


solues no nulas. Por absurdo, suponhamos que existam inteiros positivos a, b, c tais que a 4 + b 4 = c 2 . Podemos tambm supor que a, b e c foram escolhidos
EUREKA! N7, 2000

42

Sociedade Brasileira de Matemtica

de tal modo que no h outra soluo positiva a ' , b' , c' com c ' < c (aqui vamos usar o mtodo da descida). Ento a e b so primos entre si, e o teorema 1 garante a existncia de inteiros positivos primos entre si u e v tais que

a 2 = u 2 v 2 , b 2 = 2uv , c = u 2 + v 2 . Como a 2 + v 2 = u 2 , segue novamente


do teorema 1 a existncia de inteiros positivos primos entre si p e q tais que

a = p 2 q 2 , v = 2 pq, u = p 2 + q 2 . Mas a b 2 = 2uv = 4 pq( p 2 + q 2 )


Como p e q so primos entre si, temos que ambos so tambm primos com

p 2 + q 2 . Portanto, sendo 4 pq( p 2 + q 2 ) um quadrado devemos ter p, q e p 2 + q 2 quadrados, digamos p = 2 , q = 2 , p 2 + q 2 = 2 , com , ,


positivos. Por fim, segue que 4 + 4 = 2 , com c =u2 +v2 >u = p2 +q2 = 2 , contrariando a minimalidade de c. Logo, no h solues no nulas de

x n + y n = z n quando n for mltiplo de 4.


A Equao de Pell Nem sempre fcil, ou mesmo possvel, determinar todas as solues em inteiros
de uma dada equao. Por exemplo, para a equao x 2 2 y 2 = 1 , bem mais fcil mostrar que ela possui uma infinidade de solues do que determinar todas elas. Podemos gerar infinitas solues dessa equao a partir de uma s soluo no nula. Uma vez que a 2 2b 2 = 1 , teremos ( a + b 2 )( a b 2 ) = 1 , e da

( a + b 2 )2 ( a b 2 )2 = 1
Desenvolvendo os binmios, chegamos a

(a 2 + 2b 2 + 2ab 2 )(a 2 + 2b 2 2ab 2 ) = 1 , e da a a 2 + 2b 2 2(2ab) = 1


2 2

Portanto, se (a, b) for uma soluo, a 2 + 2b 2 ,2ab ser outra soluo. Sendo a e b positivos, temos a < a + 2b , e desse modo determinamos uma infinidade de solues da equao (contanto que tenhamos uma soluo no nula). Veja que (3, 2) uma soluo no nula de nossa equao. fcil ver que o mtodo acima utilizado tambm garante que, quando d for um
2 2

inteiro tal que d irracional, a equao x 2 dy 2 = 1 admite infinitas solues no nulas, desde que admita uma soluo no nula. Tambm, com poucas modificaes podemos tratar a equao x 2 dy 2 = 1 (veja o exerccio 6).
EUREKA! N7, 2000

43

Sociedade Brasileira de Matemtica

Observe que, apesar de determinarmos facilmente infinitas solues da equao acima, no sabemos se h outras. Vamos agora comear a responder essa pergunta, para uma classe mais ampla de equaes.

Definio 1 (Equao de Pell): Seja d um inteiro positivo que no seja um


quadrado. Nesse caso, sabemos que
2 2

d irracional. Chamamos equao de Pell

equao x dy = m , onde m um inteiro qualquer. claro que no caso m = 0 a equao no admite solues alm da trivial x = y = 0, pois se esse fosse o caso teramos x e y no nulos, e da racional. Lema 1: Seja um irracional qualquer. Existem infinitos racionais inteiros no nulos primos entre si, tais que
x y
x y

d =

x y

, um

, com x e y

<

1 y2

Prova: Seja n > 1 um natural qualquer, e considere os nmeros j , com j = 0, 1, ..., n. Seja { j } = j [ j ] [0,1). Como
2 n 1 [0,1) = [0, 1n ) [ 1 n , n ) ... [ n , 1) ,

segue do princpio de Dirichlet que existem 0 k < j n tais que { j} e {k} pertencem a um mesmo intervalo dos que aparecem no lado direito da igualdade acima. Ento | { j} - {k} | < que

j k
j k

)<

1 n

Da,

( j k ) ( j k)

<1 n , e segue

1 ( j k )n

1 ( j k )2

Existe ento um par (x, y) de inteiros, x = j k , y = j k n , tais que


x y

<

1 y2

. Se x = dx1 , y = dy1 , com d > 1, ento

x1 y1

<

1 y2

<

1 2 y1

, de

modo que podemos supor que x e y so primos entre si. Para garantirmos a existncia de infinitos tais pares, suponha que achamos x e y primos entre si e tais que
1 n x y

<

1 y2

. Escolha agora um natural n tal que

<

x y

. Repetindo o argumento acima, chegamos a um par de inteiros


x1 y1
1 < ny e y1 n . Portanto, 1

primos entre si x1 , y1 , com


x1 y1
1 < ny 1 1 2 y1

x1 y1

1 < ny <x y e 1

, donde ( x1 , y1 ) ( x , y ) satisfaz o lema.

EUREKA! N7, 2000

44

Sociedade Brasileira de Matemtica

Lema 2: Seja d um inteiro positivo que no seja um quadrado. Existe um inteiro


m para o qual a equao x 2 dy 2 = m admite infinitas solues inteiras.

Prova: Sabemos que

d irracional. Assim, o conjunto S dos pares (x, y) de


x y

inteiros primos entre si tais que

d <

1 y2

infinito. Mas se x e y forem

inteiros satisfazendo essa desigualdade, ento

x 2 dy 2 = x d y x + d y < 1 d y +2 d y < 1 y x y

) (

1 y

+ 2 d y <2 d + 1

Segue que algum inteiro no nulo m entre ( 2 d + 1) e 2 d + 1 se repete um nmero infinito de vezes entre os valores de x 2 dy 2 , com (x, y) em S. Mas isto o mesmo que dizer que a equao x 2 dy 2 = m admite infinitas solues.

Teorema 3 (Solues da Equao de Pell): Seja d um inteiro positivo que no


seja um quadrado. A equao x 2 dy 2 = 1 admite infinitas solues em inteiros positivos x, y. Ademais, existe uma soluo em inteiros positivos x1 , y1 tal que todas as demais solues dessa equao so da forma

x n + y n d = ( x1 + y1 d ) n , onde n um nmero natural.


Prova: Admitamos por enquanto que nossa equao tenha uma soluo em inteiros positivos x, y. Dentre todas essas solues, escolha aquela x1 , y1 tal que

= x1 + y1 d seja o menor possvel.


Dado um natural qualquer n, sabemos que existem inteiros positivos x n , y n tais que ( x1 + y1 d ) n = x n + y n d . Da, sabemos que

( x1 y1 d ) n = x n y n d , e assim
2 2 n 1 = ( x1 dy1 ) = ( x1 + y1 d ) n ( x1 y1 d ) n = 2 2 = ( x n + y n d )( x n y n d ) = x n dy n

Ento todos os pares ( x n , y n ) so solues da equao.


EUREKA! N7, 2000

45

Sociedade Brasileira de Matemtica

Seja agora (x, y) uma soluo qualquer em inteiros positivos. Para terminar, basta mostrarmos que existe um natural n tal que x + y d = n . Suponha o contrrio. Ento existe um natural n tal que n < x + y d < n +1 . Da, vem que 1 < n ( x + y d ) < . Mas

n ( x + y d ) = ( x1 + y1 d ) n ( x + y d ) = ( x n + y n d ) 1 ( x + y d ) = = ( x n y n d )( x + y d ) = ( xx n dyy n ) + ( x n y y n x ) d
e ocorre que
2 2 2 2 2 ( xxn dyyn )2 d ( xn y yn x )2 = xn ( x dy 2 ) + yn ( dy 2 x 2 ) = xn dyn = 1,

de modo que n ( x + y d ) = ( xx n dyy n , x n y y n x ) tambm soluo. Como 1 < n ( x + y d ) < , basta mostrarmos que xxn dyyn , xn y yn x > 0 para chegarmos numa contradio. Sejam a = xx n dyy n , b = x n y y n x . Temos

a + b d > 0 e a 2 db 2 = 1 , donde a b d = ( a + b d ) 1 > 0 .


Ento, 2a = ( a + b d ) + ( a b d ) > 0 . Por outro lado, a + b d > 1 implica

a b d = ( a + b d ) 1 < 1 , e da b d > a 1 0 . Logo, b > 0.


Para terminar, basta mostrarmos que a equao x 2 dy 2 = 1 admite uma soluo. Tome, de acordo com o lema 2, um inteiro (no nulo) m tal que

x 2 dy 2 = m admita uma infinidade de solues. Podemos escolher duas dessas solues, ( x1 , y1 ), ( x 2 , y 2 ) digamos, tais que | x1 | | x 2 | mas x1 x 2 e y1 y2 , mdulo m. Ento

( x1 + y1 d )( x 2 y 2 d ) = ( x1 x 2 dy1 y 2 ) + ( x 2 y1 x1 y 2 ) d (*)
2 2 Mas x1 x 2 dy1 y 2 x1 dy1 0 (mod m) e x 2 y1 x1 y 2 (mod m) , donde existem inteiros u e v tais que x1 x 2 dy1 y 2 = mu, x 2 y1 x1 y 2 = mv

Segue de (*) que ( x1 + y1 d )( x 2 y2 d ) = m( u + v d ) , e da

( x1 y1 d )( x 2 + y 2 d ) = m( u v d ) .
Multiplicando ordenadamente essas duas igualdades, chegamos a
2 2 2 2 m2 = ( x1 dy1 )( x 2 dy 2 ) = m2 ( u 2 dv 2 ) ,

ou seja, u 2 dv 2 = 1 . Resta mostrarmos que u e v so no nulos. Se u = 0 teramos dv 2 = 1 , um absurdo. Se v = 0, viria u = 1 ou 1. De (*) seguiria que
EUREKA! N7, 2000

46

Sociedade Brasileira de Matemtica

( x1 + y1 d )( x 2 y 2 d ) = m , e assim ( x1 + y1 d ) = ( x 2 + y 2 d ) , donde por fim | x1 | = | x 2 | , o que um absurdo.


Exemplo 2: Agora podemos determinar todas as solues inteiras no nulas da
equao x 2 2 y 2 = 1 . O teorema 3 ensina que as solues positivas dessa equao so da forma ( x n , y n ) , onde x n e y n so os nicos inteiros para os quais x n + y n 2 = ( x1 + y1 2 ) n , sendo ( x1 , y1 ) a soluo positiva para a qual

x1 + y1 2 o menor possvel. Como os pares (x, y) = (1, 1), (1, 2), (2, 1), (2, 2), (2, 3) no so solues da equao e (3, 2) , fcil nos convencermos de que ( x1 , y1 ) = (3, 2). Desse modo, temos os pares ( x n , y n ) dados pela
igualdade x n + y n 2 = (3 + 2 2 ) n Determine agora as demais solues no nulas da equao acima. O exerccio 7 discute mais alguns aspectos dessa equao.

Exerccios: 1. Seguindo os passos da prova do teorema 1, mostre que as solues em


inteiros no nulos da equao

x2 + 2 y2 = z2

so

da

forma

x = ( u 2 2v 2 )d , y = 2uvd , z = ( u 2 + 2v 2 )d , onde d, u, v so inteiros


no nulos, com u e 2v primos entre si. 2. Mostre que as equaes a seguir no possuem solues inteiras no nulas:

i. x 4 + 4 y 4 = z 2 ii. x 4 + 2 y 4 = z 2 iii. x 2 + y 2 = 3z 2
O item i do exerccio a seguir tem a ver com o exemplo 1 do texto.

3. i. Mostre que no existem racionais x e y tais que x 2 + xy + y 2 = 2 .

ii. Determine todas as solues racionais da equao x 2 + xy + y 2 = 1 .


Para resolver os prximos dois exerccios utilizamos o teorema 1. Eles so mais difceis que os anteriores, e no primeiro deles voc pode achar til o seguinte resultado, conhecido como Teorema de Ptolomeu: dado um quadriltero convexo inscritvel ABCD, tem-se

AB. CD + AD. BC = AC. BD

EUREKA! N7, 2000

47

Sociedade Brasileira de Matemtica

D B C

Para uma prova do Teorema de Ptolomeu, voc pode consultar a referncia [4]. 4. Temos no plano uma circunferncia de raio 1. Mostre que podemos escolher em tal circunferncia 2000 pontos A1 , A2 ,..., A2000 tais que Ai A j racional, quaisquer que sejam 1 i < j 2000 . 5. Seja r um inteiro positivo dado. Queremos determinar o nmero de tringulos ABC, dois a dois no congruentes, satisfazendo as seguintes condies: i. O raio da circunferncia inscrita em ABC mede r. ii. Os comprimentos dos lados de ABC so nmeros inteiros, primos entre si. Mostre que o nmero de tais tringulos 2 k , onde k o nmero de fatores primos distintos de r.

6. Prove, sem apelar para o teorema 2, que a equao x 2 2 y 2 = 1


admite uma infinidade de solues inteiras.

7. Prove que as solues positivas ( x n , y n ) da equao do exemplo 2 so dadas pelas seqncias ( x1 , y1 ) = (3, 2 ) e x n+1 = 3x n + 4 y n , y n+1 = 2 x n + 3 y n
8. Prove que h infinitos inteiros n tais que n 2 + ( n + 1) 2 seja quadrado.

Bibliografia
[1] Introduo Teoria dos Nmeros. Plnio O. dos Santos. Coleo Matemtica Universitria. IMPA. 1999. [2] An Introduction to the Theory of Numbers. I. Niven, H. Zuckermann. John Wiley & Sons. New York. 1980. [3] A Classical Introduction to Modern Number Theory. K. Ireland & M. Rosen. Springer-Verlag. New York. 1990. [4] Quadrilteros e Tringulos. M. Mendes. Eureka! No5. OBM 1999

EUREKA! N7, 2000

48

Sociedade Brasileira de Matemtica

SOLUES DE PROBLEMAS PROPOSTOS Publicamos aqui algumas das respostas enviadas por nossos leitores.
29)
Seja n > 1 um nmero inteiro. Existem n lmpadas L0, L1, ... , Ln1 colocadas em um crculo. Cada lmpada est ACESA ou APAGADA. Uma seqncia de passos S0, S1, ... , Si, ... executada. O passo Sj afeta apenas o estado da lmpada Lj (deixando o estado de todas as outras inalterado) da seguinte forma:

Se Lj1 est ACESA, Sj muda o estado de Lj de ACESA para APAGADA, ou de APAGADA para ACESA; Se Lj1 est APAGADA, Sj deixa o estado de Lj inalterado. As lmpadas so rotuladas mod n, ou seja, L 1 = Ln 1, L0 = Ln, L1 = Ln + 1, etc. Inicialmente todas as lmpadas esto ACESAS. Mostre que:

a) b)

Existe um inteiro positivo M(n) tal que depois de M(n) passos todas as lmpadas esto ACESAS de novo; Se n da forma 2k ento todas as lmpadas esto ACESAS depois de n2 1 passos; Se n tem a forma 2k + 1 ento todas as lmpadas esto ACESAS depois de n2 n + 1 passos.

c)

Soluo de Frank Castro (So Paulo - SP):


a) Vamos inicialmente representar o estado das lmpadas L0, L1, L2, ..., Ln1 por uma n-upla u = (u0, u1, u2, ..., un1), onde ui = 0 se Li est apagada e ui = 1 se Li est acesa. Evidentemente o estado inicial das lmpadas dado pela n-upla e = (1, 1, 1, ...,1). Nessas condies a operao Sj tranforma a n-upla (u0, u1, u2, ..., un1) na nova n-upla (u0, u1,..., uj1, uj1 + uj,..., uj+1,..., un1), onde a soma uj1 + uj tomada mdulo 2 ( e j tomada mdulo n). Assim sendo, nosso problema consiste em determinar um valor natural r, tal que:

S r ( S r 1 (...( S1 ( S 0 (e)))...)) = e
EUREKA! N7, 2000

49

Sociedade Brasileira de Matemtica

Para tanto, denotemos por Rj a operao que transforma a n-upla (u0, u1,..., uj1, uj, uj+1,..., un1) na n-upla (u j mod n , u ( j +1) mod n ,...u ( j + n 1) mod n ). Observe que Rj a operao inversa de Rj e R0 deixa a n-upla inalterada. Nesses termos temos para uma n-upla qualquer S j = R j ( S 0 ( R j )) . Agora para uma n-upla qualquer u podemos escrever:
__
__ __

S r (S r 1 ...(S1 (S 0 ( u )))...) = S r S r 1 ...S1 S 0 ( u ) = ( Rr S 0 Rr )(Rr +1 S 0 Rr 1 )... ...(R1 S 0 R1 )S 0 ( u ) = Rr S 0 ( RS0 ) r ( u ) =R r 1 ( RS0 ) r +1 ( u ) onde R = R1 . Consequentemente, S r S r 1 ...S1 S 0 (e) = e ( RS 0 ) r +1 (e) = Rr +1 (e) = e como
existe apenas um nmero finito de estados das lmpadas (2n precisamente) que equivale ao nmero total de n-uplas, em algum estgio a seqncia e, ( RS 0 )1 (e), ( RS 0 ) 2 (e),... deve repetir algum de seus elementos. Nessas condies para algum m e n ( m < n ) teremos : ( RS 0 ) m (e) = ( RS 0 ) n (e). Sendo
__ __ __

RS 0 uma bijeo (verifique!) temos ( RS 0 ) n m (e) = e, o que conclui o tem a.


b) Primeiramente associaremos n-upla u = (u0, u1, u2, ..., un1) o polinmio P(x) da forma : P ( x) = u n 2 + u n 3 x + u n 4 x 2 + ... + u 0 x n 2 + u n 1 x n 1 , onde os coeficientes sero olhados mdulo 2 (isto , P(x) Z / 2Z [ x]). Chamemos tal polinmio de polinmio de posio. Observe agora que para a n-upla ( RS 0 )1 (u ) temos:

( RS0 )1 (u) = R1 (S 0 (u)) = R1 (u n1 + u 0 , u1 , u 2 ,..., u n1 ) = (u1 , u 2 ,..., u n1 , u n1 + u 0 )


e seu polinmio de posio dado por

Q( x) = u n 1 + u n 2 x + u n 3 x 2 + ... + u1 x n 2 + (u n1 + u 0 ) x n 1 No se esquea que


a adio u n 1 + u 0 tomada mdulo 2. Assim, Q(x) x P(x) mod( x n x n1 1). Assim sendo, queremos encontrar r tal que x r 1 mod( x n x n 1 1). Note que quando x r 1 mod( x n x n 1 1). o polinmio de posio associado a ( RS 0 ) r ser congruente a P(x) que representar a n-upla e = (1, 1, 1,...1) no estado inicial das lmpadas. Suponha agora n = 2k. Ento x n ( x n ) n ( x n1 + 1) n x n( n 1) + 1 mod(x n x n 1 1) pois se n uma potncia de 2, todos os coeficientes, exceto o primeiro e o ltimo da expanso binomial ( x n 1 + 1) n so pares, logo congruentes a zero mdulo 2.
EUREKA! N7, 2000
2

50

Sociedade Brasileira de Matemtica

Finalmente temos: 2 2 2 x n x n ( n 1) 1 mod( x n x n 1 1) x n x n n 1 mod( x n x n 1 1)


xn
2

( x n 1) 1 mod( x n x n 1 1) x n

1 mod( x n x n 1 1)

( pois x n 1 x n 1 (mod x n x n 1 1)). Assim, aps ( n 2 1) etapas todas as lmpadas estaro acesas novamente. c) Suponha n = 2 k + 1 Assim x n
2

( x n+1 ) n1 ( x n + x) n1 x n( n1) + x n1 onde todas as congruncias

foram tomadas mod(x n x n1 1). Como no tem anterior n 1 potncia de 2, logo todos os coeficientes, exceto o primeiro e o ltimo da expanso binmial ( x n + x) n1 so pares, consequentemente congruentes a zero mdulo 2. Finalmente temos: 2 2 x n 1 x n( n 1) x n 1 mod(x n x n 1 1) x n n ( x n 1 1) x n 1 mod( x n x n 1 1)

(*) x n

( x n ) x n 1 mod( x n x n 1 1) x n x n 1 mod( x n x n 1 1)

x n n +1 1 mod( x n x n 1 1). Observe que, como estamos trabalhando mdulo 2, x n x n 1 + 1 x n 1 1 mod( x n x n 1 1), e isso justifica a
congruncia (*). Assim sendo, aps n 2 n + 1 etapas, todas as lmpadas estaro acesas novamente.

30) Determine todas as funes f : R R que satisfazem as condies: 1 f ( x) (i) f( x) = f(x), (ii) f(x + 1) = f(x) + 1, (iii) f = 2 para x 0. x x Soluo de Carlos Alberto da Silva Victor (Rio de Janeiro - RJ):
Do tem (ii) : f(x + 1) = f(x) temos que f(x) = 1 + f( x 1) e usando (i) f(x) + f (1 x) = 1. Sejam x 0 e x 1, logo: f + f 1

1 x

1 = 1 e usando x

1 1 1 1 x f (iii) 2 f (x) + f =1 e usando novamente (iii) 2 f (x)+ =1 2 x x x x1 x x1 x1

EUREKA! N7, 2000

51

Sociedade Brasileira de Matemtica

1 (x 1)2 1 1 (x 1)2 1 f ( x ) + f 1 + = 1 f ( x ) 1 + + f ( x 1 ) =1 2 2 2 2 2 x 1 x x x x (x 1)

1 (x 1)2 1 f x ( ) + + 2 f (x 1) = 1 x2 x2 x f (x) + (x 1)2 + f (x 1) = x2 f (x) + f (x 1) = 2x 1 f (x) + f (1 x) = 1 donde: f (x) = x f (x) f (1 x) = 2x 1 para x = 0 f (1) = f (0) +1 x = 1 f (0) = f (1) +1 = f (1) +1 f (1) f (0) = 1 f (1) =1 e f (0) = 0. f (0) + f (1) =1
Concluso: x R, teremos f(x) = x como sendo a nica soluo.

31) Seja x1, x2, x3, uma seqncia de nmeros reais no negativos satisfazendo x n 2 x n 1 xn = para n = 3, 4, 5, Estabelea condies necessrias e 2 x n 2 x n 1 suficientes em x1 e x2 para xn ser inteiro para infinitos valores de n. Soluo de Davi Mximo Alexandrino Nogueira (Fortaleza - CE):
Afirmao: x1 = x 2 , x1 e x 2 inteiros. Prova: Se, x n =

xn (2 x n 2

x n 2 x n 1 teremos, 2 x n 2 x n 1 1 2 x n 2 x n 1 1 2 1 x n 1 ) = x n 2 x n 1 = = xn x n 2 x n 1 x n x n 1 x n 2

1 1 1 1 1 tome y n = logo, a seqncia y1, y2, , uma = x n x n 1 x n 1 x n 2 xn x x2 P.A., de razo r = 1 desse modo, yn = y1 + (n 1)r x1 = xn + (n 1)rx1xn x1 x 2
EUREKA! N7, 2000

52

Sociedade Brasileira de Matemtica

x1 . Suponha r 0. 1+ (n 1)x1r x1 x 2 x x2 Como r = 1 , temos x n = fazendo x1 x2 = a, teremos x1 x 2 x 2 + (n 1)( x1 x 2 ) x1 = xn (1+ (n 1)rx1) = x1 xn =

xn =

x1 x 2 . Porm, para algum k, tal que x1 x 2 < x 2 + (k 1) a x 2 + (n 1)a

teremos x n < 1 , para todo n k. Logo, devemos ter r = 0, o que conclui a demonstrao.

32) a) Prove que todo nmero inteiro no nulo m admite uma nica representao
da forma m = com n 1 0.

k =0

n 1

3 k , onde n um inteiro positivo e k {1,0,1} para todo k,

Dado um conjunto de

3n + 1 pontos V = {P0 , P1 ,..., P3n 1 } , escrevemos em cada 2


2

aresta que une dois desses pontos Pi e Pj (i j) um nmero pertencente a {0, 1, , n 1} da seguinte forma: escreveremos i j =

k =0

n 1

3 k , com
nmero

k {1,0,1}, k ,

associamos

aresta

Pi

Pj

m = min{k 0 k = 1}.
Prove que no existe nenhum tringulo cujos vrtices pertenam a V com o mesmo nmero escrito em seus trs lados.

Soluo de Carlos Alberto da Silva Victor (Rio de Janeiro - RJ):


a) Sabendo que m 0 (mod 3) ; m 1 (mod 3) ou m 1 (mod 3), teremos: m = 3k0 + r0 k0 = 3k1 +r1 k1 = 3k2 + r2

kn2 = 3kn1 + rn1


EUREKA! N7, 2000

53

Sociedade Brasileira de Matemtica

com kn1 = 0 e rn1 0, onde ri {1, 0, 1} (note que os ri esto unicamente determinados, e existe n com kn1 = 0 pois m > k0 > k1 > ... > ki enquanto tivermos k i 1 0. ) Substituindo na primeira igualdade k0 pela sua igualdade, obteremos m como funo de k1, r0. Tomando novamente k1 = 3 k2 + r2 e fazendo as substituies sucesivas de k2, k3, ..., kn-2 ; obtemos: m = 3 n k n 1 + rn 1 3 n 1 + rn 2 3 n 2 + ... + r2 3 2 + r1 31 + r0 3 0 e j que

k n 1 = 0 , teremos m = rk 3 k com rn 1 0.
k =0

n 1

b) Sejam Pi , Pj e Ps trs pontos quaisquer de V e por hiptese i < j < s; portanto podemos
k ,
~ p

escrever:

j i = k 3k
k =0
~ t- 1

n 1

s i = p 3 p ,
p =0

t 1 ~

com

{ 1, 0 ,1}; n 1 = 1 e

= 1.
~

Vamos tambm fazer a hiptese de que para as arestas Pi Pj e Pi Ps , tenhamos o nmero = min{ k 0 / k = 1} = min{ p 0 /
p

= 1} onde

0 < n 1 e 0 < t 1. Podemos ento escrever: 1 n 1 k j i . 3 k . 3 k + 1 . 3 (1) = + k k =0 k = +1 1 t 1 s i = p . 3 p + pk . 3 p + 1 . 3 ( 2 ) p=0 p = +1 1 ~ t 1 ~ n 1 1 p p k k De (2) (1) obtemos s j = p .3 + p .3 k .3 + k .3 p =0 p = +1 k = +1 k =0
Para 0 k 1, k e k pertencem a {1, 0}, donde k k {1,0,1}, e os
~ ~

somatrios com p +1 e k +1 so mltiplos de 3 +1 , e portanto, ao escrever s j =

3 , com
' 0

' {1,0,1}, no aparece o termo 1 3 , o que

garante que o nmero associado aresta Pj Ps, que min{ 0 / ' = 1} em

( s j ) no ser igual a .
Concluso: no existe tringulo cujos vrtices pertenam a V com o mesmo nmero escrito em seus trs lados.
EUREKA! N7, 2000

54

Sociedade Brasileira de Matemtica

33) Na parede interna de um vaso cilndrico de cristal existe uma gota de mel num ponto B situado a trs centmetros do seu bordo superior. Na parede externa, num ponto A diametralmente oposto ao da gota, est uma formiga. Sabendo que a altura do vaso de 20cm e o seu dimetro 10cm. Indicar o caminho mais curto para que a formiga atinja a gota de mel. Soluo de Daniel Pessa Martins Cunha (Fortaleza - CE):
Cobrindo o vaso com papel por dentro e por fora, e marcando nele a localizao da formiga, da gota de mel e da borda, poderemos ver que ao desamassar o papel ficaro as seguintes impresses, com as seguintes medidas:
B Gota Parte de dentro 3 cm r = 5 Borda A' 5/2 3 cm 5/2 B' Parte de fora

A Formiga

Como a menor distncia entre 2 pontos a medida do segmento que os une, o menor caminho o segmento AB de medida : 36 + 25 2 (Teorema de Pitgoras). Ao colocar o papel de volta ao vaso veremos o menor caminho a ser percorrido pela formiga. Que subir em diagonal at o ponto mdio do arco A' B ' , determinado pelo dimetro na borda. Depois descer em diagonal at a gota de mel.
____

EUREKA! N7, 2000

55

Sociedade Brasileira de Matemtica

34) ABC um tringulo, tal que AB = c, AC = b e BC = a. Por um ponto interior P deste tringulo, so traadas paralelas aos seus lados. Sabe-se que as interseces, da paralela ao lado de medida a, com os lados deste tringulo, determinam um segmento de medida a'. Analogamente, as paralelas aos lados de medidas b e c, determinam com os lados do tringulo, segmentos de medidas b' e c' respectivamente. a ' b' c' Nestas condies demonstre que + + = 2. a b c Soluo de Francisco Antonio Martins de Paiva (Fortaleza - CE):
De acordo com o problema temos:
A t s s I y w y D E C P x z z G t F

Como as retas traadas so paralelas aos lados ento os quadrilteros PFCE, PIBD, PHAG so paralelogramos, e com isso conclumos que seus lados opostos so congruentes. Da temos: BC = a, AC = b, AB = c, IF = x + y = a' , GD = t + w = c', HE = s + z = b' Os tringulos ABC e AIF so semelhantes pois IF // BC , de onde temos:
___ ____

x+ y cw x+ y bz = e = a c a b

a' w = 1 a c

a' z = 1 a b

EUREKA! N7, 2000

56

Sociedade Brasileira de Matemtica


___ ____

Os tringulos ACB e GCD so semelhantes pois AB// GD , de donde temos:

t+w bs t+w a y e = = c b c a c' s c' y = 1 = 1 c b c a


Os tringulos ACB e HEB so semelhantes pois AC// HE , de onde temos:
___ ____

s+z ax s+ z ct e = = b a b c

b' x = 1 b a
Da temos que:

b' t = 1 b c

x y z s t w a ' b' c ' 2 + + = 6 a a b b c c a b c x+ y z+s t+w a ' b' c ' 2 + + = 6 a b c a b c a ' b' c' a ' b' c' 2 + + = 6 a b c a b c a ' b' c ' 3 + + = 6 a b c a ' b' c ' + + = 2. a b c
35) Sabendo que num tringulo ABC a altura relativa ao vrtice A mede 12cm. e a altura relativa ao vrtice B mede 20cm, determine todos os valores possveis para a altura relativa ao vrtice C.

EUREKA! N7, 2000

57

Sociedade Brasileira de Matemtica

Soluo de Frank Castro (So Paulo - SP):


Temos: ha = 12, hb = 20. Sendo a, b e c os lados do tringulo e S sua rea, valem as seguintes relaes:

c > b a (I)

a=

2S 2S 2S e c= Substituindo as trs ltimas igualdades em (I) , b= ha hb hc

Vem que:

1 1 1 1 1 1 > = = , assim hc = 30. Agora, sabemos que: a + b > c hc hb ha 20 12 30 a= 2S 2S 2S e c= . Substituindo temos: ,b= ha hb hc 1 1 1 1 1 1 + > + > hc > 7,5 ha hb hc 12 20 hc

Resposta: 7,5cm < hc < 30 cm. Agradecemos tambm o envio das solues a: Ricardo Klein Hoffmann (Porto Alegre - RS), Geraldo Perlino Jnior (So Paulo - SP), Jos Heleno Faro (Cachoeiro de Itapemirim - ES).

Voc sabia
Que h novos records de primos grandes descobertos em 2000? Maior par de primos gmeos conhecido: 24091107798452600001. Esses primos tm 18075 dgitos, e foram descobertos por Wassing, Jrai e Indlekofer. Maior primo de Fermat generalizado conhecido: 16717632768 + 1, que tem 171153 dgitos e foi descoberto por Yves Gallot (este o oitavo maior primo conhecido atualmente, e maior primo conhecido que no de Mersenne).

EUREKA! N7, 2000

58

Sociedade Brasileira de Matemtica

PROBLEMAS PROPOSTOS
Convidamos o leitor a enviar solues dos problemas propostos e sugestes de novos problemas para os prximos nmeros.
36) Na figura abaixo o tringulo DEF tem rea de medida S. Sabendo-se que o tringulo DEF est inscrito num tringulo arbitrrio ABC, mostre que as medidas Si ( i = 1, 2, 3) das reas dos outros tringulos formados satisfazem a
desigualdade S

3 e que a igualdade ocorre se e s se os 1 1 1 + + S1 S 2 S 3


A

pontos DEF so os pontos mdios dos lados do tringulo, ABC.

F S2 B

S1 S S3 D C E

37) Cinco quadrados so dispostos conforme ilustra o diagrama abaixo. Mostre que a medida da rea do quadrado S igual a medida da rea do tringulo T.

38) Os lados e diagonais de um polgono regular de n lados so coloridos em k cores tais que: i) para cada cor a e dois vrtices A e B do polgono, o segmento AB colorido de a ou existe um vrtice C tal que AC e BC so coloridos de a.

EUREKA! N7, 2000

59

Sociedade Brasileira de Matemtica

ii) os lados de qualquer tringulo com vrtices entre os vrtices do polgono so coloridos usando no mximo 2 cores. Prove que k 2.

39) Sejam x, y

e z os ngulos de um tringulo de lados opostos a, b e c

respectivamente. Prove que, a y + z + b z + x + c + 2 + + . x y x y z 40) a) Calcular a soma dos divisores positivos de um nmero natural em termos de sua fatorao prima. b) Dizemos que n 1 abundante se a soma de seus divisores maior que 2n. Prove que se n abundante ento kn abundante para todo inteiro k 1. c) Prove que existe n0 N tal que todo inteiro n n0 pode ser escrito como soma de dois nmeros abundantes.

1 1

1 1

1 1

a b c

Problemas 36 e 37 propostos por Carlos Alexandre Gomes da Silva (Natal - RN), problema 38 proposto na Olimpada Blgara - 1998, problema 39 proposto por Aldo Trajano Louredo, problema 40 proposto por Gleydson Chaves Ricarte (Fortaleza CE) e Zoroastro Azambuja Neto (Rio de Janeiro - RJ).

Errata:
Eureka!No. 6, pg 40: O enunciado do problema 4 deve dizer: Problema 4: Mostre que h infinitos naturais n tais que n2 + 1 divide n!, onde n! = n (n1)..21 (por exemplo, 4! = 4 3 2 1 = 24). Eureka! No. 6, pg 27: o segundo pargrafo est truncado. A verso correta : Determinar exatamente os valores de nmeros de Ramsey clssicos R(a, b) , em geral, um problema computacionalmente muito difcil. Os nicos valores de R(a, b) com 3 a b que so conhecidos so: R(3, 3) = 6, R(3, 4) = 9, R(3, 5) = 14, R (3, 6) = 18, R(3, 7) = 23, R(3, 8) = 28, R (3, 9) = 36, R(4, 4) = 18, e R(4, 5) = 25. O nico nmero Ramsey com mais de duas cores cujo valor conhecido R(3, 3, 3) = 17.

EUREKA! N7, 2000

60

Sociedade Brasileira de Matemtica

AGENDA OLMPICA
XI OLIMPADA DO CONE SUL 14 a 19 de abril de 2000 Montevideu Uruguai

VI OLIMPADA DE MAIO 13 de maio de 2000

XXII OLIMPADA BRASILEIRA DE MATEMTICA Primeira Fase Sbado, 10 de junho Segunda Fase Sbado, 02 de setembro Terceira Fase Sbado, 21 de outubro (nveis 1,2 e 3) Domingo, 22 de outubro (nvel 3 - segundo dia).

XLI OLIMPADA INTERNACIONAL DE MATEMTICA 13 a 25 de julho Taejon, Coreia do Sul.

XV OLIMPADA IBEROAMERICANA DE MATEMTICA 16 a 24 de setembro de 1998 Caracas, Venezuela

III OLIMPADA IBEROAMERICANA DE MATEMTICA UNIVERSITRIA outubro de 2000

EUREKA! N7, 2000

61

Sociedade Brasileira de Matemtica

COORDENADORES REGIONAIS
Amarisio da Silva Arajo (UFV) Viosa - MG Alberto Hassen Raad (UFJF) Juiz de Fora - MG Angela Camargo (Centro de Educ.de Adultos - CEA) Blumenau - SC Benedito T. Vasconcelos Freire (UFRN) Natal - RN Claudio Arconcher (Col. Leonardo da Vinci) Jundia - SP Claus Haetinger (UNIVATES) Lajeado - RS Crescncio das Neves (UFAM) Manaus-AM lio Mega (Col. ETAPA) So Paulo - SP Enzo Marcom Takara (Col. Singular) Santo Andr - SP Flvia Jernimo Barbosa (UFPB Campus I) Joo Pessoa - PB Florncio F. Guimares Filho (UFES) Vitria - ES Francisco Dutenhefner (UFMG) Belo Horizonte - MG Gisele de A. Prateado Gusmo (UFGO) Goinia - GO Ivanilde H. Fernandes Saad (U. Catlica Dom Bosco) Campo Grande - MS Joo Bencio de Melo Neto (UFPI) Teresina - PI Joo F. Melo Libonati (Grupo Educ. IDEAL) Belm - PA Jorge Ferreira (UEM) Maring - PR Jos Carlos Pinto Leivas (UFRG) Rio Grande - RS Jos Cloves Saraiva (UFMA) So Luis - MA Jos Gaspar Ruas Filho (ICMC-USP) So Carlos - SP (UFSC) Florianpolis - SC Jos Luis Rosas Pinho Jos Paulo Carneiro (Univ. Santa rsula) Rio de Janeiro - RJ Jos Vieira Alves (UFPB) Campina Grande - PB Leonardo Matteo D'orio (Sistema Titular de Ensino)Belm - PA Licio Hernandes Bezerra (UFSC) Florianpolis - SC Luzinalva M. de Amorim (UFBA) Salvador - BA (UF Cear) Fortaleza - CE Marcondes Cavalcante Frana Pablo Rodrigo Ganassim (L. Albert Einstein) Piracicaba - SP Paulo H. Cruz Neiva de L. Jr. (Esc. Tec.Everardo Passos) SJ dos Campos - SP Reinaldo Gen Ichiro Arakaki (INPE) SJ dos Campos - SP Ricardo Amorim (Centro Educ. Logos) Nova Iguau - RJ Roberto Vizeu Barros (Colgio ACAE) Volta Redonda - RJ Sergio Claudio Ramos (IM-UFRGS) Porto Alegre - RS Seme Gebara Neto (UFMG) Belo Horizonte - MG Silvio de Barros Melo (UFPE) Recife - PE (U. do Estado da Bahia) Juazeiro - BA Tadeu Ferreira Gomes Toms Menndez Rodrigues (U. Federal de Rondonia) Porto Velho - RO Valdenberg Arajo da Silva (U. Federal de Sergipe) So Cristovo - SE Wagner Pereira Lopes (Esc. Tec. Fed. de Gois) Jata - GO Waldemar M. Canalli (P.M. S. Joo de Meriti) S. Joo de Meriti - RJ

EUREKA! N7, 2000

62

CONTEDO

AOS LEITORES XI OLIMPADA DE MATEMTICA DO CONE SUL Problemas e solues VI OLIMPADA DE MAIO Problemas VI OLIMPADA DE MAIO Resultados XLI OLIMPADA INTERNACIONAL DE MATEMTICA Problemas e Resultados

2 3

11 13

14

ARTIGOS
INTRODUO GEOMETRIA PROJETIVA Luciano G. M. Castro CONTAR DUAS VEZES PARA GENERALIZAR (O RETORNO) Jos Paulo Carneiro, Universidade Santa rsula O PRINCPIO DO ELEMENTO EXTREMO Jos Rosales Ortega, Escola de Matemtica - Instituto Tecnolgico de Costa Rica FUNES MULTIPLICATIVAS E A FUNO DE MBIUS Carlos Gustavo T. de A. Moreira, IMPA & Nicolau Coro Saldanha, PUC-Rio OLIMPADAS AO REDOR DO MUNDO SOLUES DE PROBLEMAS PROPOSTOS PROBLEMAS PROPOSTOS AGENDA OLMPICA COORDENADORES REGIONAIS 16 28 33 43 47 51 60 61 62

Sociedade Brasileira de Matemtica

AOS LEITORES

Este nmero da Eureka! contm as provas das competies internacionais de que participamos na primeira parte do ano 2000: a Olimpada de Maio, a Olimpada do Cone Sul e a Olimpada Internacional de Matemtica. Estas provas fornecem material que pode (e deve) ser usado na preparao para a Terceira Fase da Olimpada Brasileira de Matemtica. Na seo de artigos, com prazer que publicamos artigos de novos colaboradores da Eureka!. Destacamos o artigo do Prof. Jos Rosales Ortega, da Costa Rica, que esperamos d incio a uma colaborao intensa com professores de outros pases, igualmente dedicados disseminao da matemtica entre os jovens. Neste nmero inauguramos uma nova seo, Olimpadas ao Redor do Mundo, organizada pelo Prof. Antnio Luiz Santos, que trar problemas de Olimpadas realizadas em outros pases. Esta seo se junta de problemas propostos no objetivo de fornecer ainda mais material para treinamento e desenvolvimento individual. Aproveitamos para registrar, com satisfao, um grau cada vez maior de participao de nossos leitores. Temos recebido um nmero crescente de solues para os problemas propostos, alm de sugestes de novos problemas. Obrigado a todos que tm colaborado!

Comit Editorial

EUREKA! N8, 2000

Sociedade Brasileira de Matemtica

XI OLIMPADA DE MATEMTICA DO CONE SUL


14 a 19 de abril, Montevidu - Uruguai
A XI Olimpada de Matemtica do Cone Sul foi realizada em Montevidu, Uruguai, no perodo de 14 a 19 de abril de 2000. A equipe brasileira foi liderada pelos professores Paulo Jos Bonfim Gomes Rodrigues e Marcelo Mendes, ambos de Fortaleza - CE. Nesta oportunidade a equipe brasileira obteve a maior pontuao entre os pases participantes e a nica medalha de ouro da competio. RESULTADOS DA EQUIPE BRASILEIRA BRA1 BRA2 BRA3 BRA4 Carlos Stein Naves de Brito Davi Mximo Alexandrino Nogueira Humberto Silva Naves Larissa Cavalcante Queiroz de Lima Prata Bronze Ouro Prata

PROBLEMA 1

Dizemos que um nmero descendente se cada um de seus dgitos menor do que ou igual ao dgito anterior, da esquerda para a direita. Por exemplo, 4221 e 751 so nmeros descendentes, enquanto 476 e 455 no so descendentes. Determine se existem inteiros positivos n para os quais 16n descendente.
SOLUO DE CARLOS STEIN NAVES DE BRITO (GOINIA - GO)

Sabemos que 16 n 6(mod 10), pois 6 n 6(mod 10). Assim o dgito das unidades ser sempre 6. Temos ento: 2 4 n 6(mod 10)

2 4 n 2 4 k (mod 10.000) pois (10.000,2 4 n ) = 2 4.


Temos que 2 4 k 6(mod 10) k = 5q + 1.

2 4 n 2 4 (5q + 1)(mod 10.000) 2 4 n 10(8q + 1) + 6(mod 10.000)

EUREKA! N8, 2000

Sociedade Brasileira de Matemtica

Temos que 8q + 1 deve ter dgitos maiores ou iguais a 6. Em particular, 8q + 1 termina por 7 ou 9. Temos ento as seguintes possibilidades para os seus ltimos 3 dgitos: 999, 997, 987, 977, 887, 877, 777. Os nicos que so da forma 8q + 1 so 977 e 777. Como 25 divide 7776, 16n no termina em 77776 nem em 97776. 16 n 87776(mod 10 5 ) 16 n 987776(mod 10 6 ). Como 27 divide 987776, 16n no termina em 9987776. Como 26 divide 99776, 16n no termina em 999776 16n tem no mximo 6 dgitos, e basta verificar os casos. Como para nenhum caso haver soluo, 16 n nunca descendente.
PROBLEMA 2

Em um tabuleiro 8 8 distribumos os inteiros de 1 a 64, um em cada casa. A seguir, colocam-se sobre o tabuleiro fichas quadradas 22, que cobrem exatamente quatro casas (sem superposio) e de modo que os quatro nmeros cobertos por cada ficha determinem uma soma menor que 100. Mostrar uma distribuio desses inteiros que permita colocar o maior nmero de fichas, e demonstrar que no possvel obter uma distribuio que permita colocar mais fichas.
SOLUO DE CARLOS STEIN NAVES DE BRITO (GOINIA - GO)

Sabemos que o somatrio dos nmeros sobre os quais colocamos fichas dividido pelo nmero de fichas deve ser menor que 100. Logo se preenchessemos todo o tabuleiro (com 16 fichas):

1 + 2 + 3 + ... + 64 100 16

(64 + 1)64 32 65 2 100 100 130 100. Absurdo! 16 16

Ento a cada ficha a menos que colocamos devemos tirar o maior somatrio de nmeros sem estar preenchidos, pois assim a razo anterior vai ser mnima. A cada ficha que retiramos tiraremos 64, 63, 62, 61, depois 60, 59, 58, 57... at a razo do somatrio dos nmeros preenchidos dividido pelo nmero de fichas ser menor que 100. Disso temos: Dado: o somatrio inicial 2080 e o nmero inicial de fichas 16 e sendo n o nmero de fichas retiradas que deve ser mnimo
EUREKA! N8, 2000

Sociedade Brasileira de Matemtica

2080 [64 + ( 64 1) + ( 64 2 ) + ... + ( 64 ( 4 n 1 )) ] 100 16 n / n (4 n 1 + 0)4 2080 4 n 64 / 2 100 16 n 2080 256 n + 8 n 2 2 n 1600 100 n 4 n 2 79 n + 240 0
79 49 n1 =16 2 < n = 3 , 75 4 n 79 n + 240 = 0 n = 8 2

3,75

16

Como se quer o n mnimo, que satisfaa a desigualdade, n 4 e teremos 12 fichas no mximo. Para n = 3, com 13 fichas: Podemos colocar 12 fichas, do seguinte modo: Vamos ter os nmeros de 1 at 48. agrupamos eles de 4 em 4 para a soma ser menor que 100. Esses grupos so {1, 24, 25, 48}, {2, 23, 26, 47},...,{12, 13, 36, 37}. Da forma {1+ n, 24 n, 25 + n, 48 n} com n {0, 1, ..., 11}. Ento colocaremos esses nmeros em espaos 2 2: 1 25 24 45 2 26 23 47 3 27 22 46 4 28 21 45

Faremos isso com todos os grupos, sobrando ainda um espao 2 8, que no tero ficha, onde colocaremos aleatriamente os nmeros {49, 50,...,64}. Sendo essa uma soluo com cada ficha sob um grupo daqueles citados.

EUREKA! N8, 2000

Sociedade Brasileira de Matemtica

Exemplo completo:
1 25 5 29 9 33 49 57 24 48 20 44 16 40 50 58 2 26 6 30 10 34 51 59 23 47 19 43 15 39 52 60 3 27 7 31 11 35 53 61 22 46 18 42 14 38 54 62 4 28 8 32 12 36 55 63 21 45 17 41 13 37 56 64

PROBLEMA 3

Um quadrado de lado 2 dividido em retngulos mediante vrias retas paralelas aos lados (algumas horizontais e outras verticais). Os retngulos so coloridos alternadamente de preto e branco, como se fosse um tabuleiro de xadrez. Se deste modo a rea branca resultou igual a rea preta, demonstrar que ao recortar os retngulos pretos ao longo de seus bordos, possvel formar com estes (sem superposio) um retngulo preto 1 2.
SOLUO DE HUMBERTO SILVA NAVES (SO PAULO - SP)

Seja x 1 ; x 2 ; x 3 ;...; x n as distncias entre as retas verticais ( x i distncia entre a i-sima reta e a (i 1)-sima reta) e y 1 ; y 2 ;...; y p as distncias entre as retas horizontais: ( y i a distncia entre a i-sima reta vertical e a (i 1)-sima reta). Por simetria, podemos considerar: rea sombreada =

i e j de mesma paridade

xi y

i e j pares

xi y j +

xi i e j mpares

Logo, rea sombreada =


2 =
i e j pares

xi y j +

ie j mpares

xi y

= xi i par

y i + i par

x j mpar

yj mpar

e denotamos: A = xi e B =
" i " par

" i "par

yi

EUREKA! N8, 2000

Sociedade Brasileira de Matemtica

mas x i =

" i " par

xi +
" j " mpar

" j " mpar

xj = 2

" j " mpar

x j = 2 A , e de mesmo modo

conclumos que:

= 2 B.

Logo: rea sombreada = 2 = A B + ( 2 A )( 2 B ) 2 = 2 AB + 4 2 ( A + B ) 2 ( A + B ) 2 AB = 2 A + B AB = 1 A (1 B ) = 1 B ( A 1)( 1 B ) = 0


A 1 = 0 Logo devemos ter ou 1 B = 0

A = 1 ou B = 1 Agora o problema fica fcil, pois se A = 1 (por simetria), temos: x i = x j , logo basta juntar os "quadradinhos" de cada linha, a
i par j mpar

vai formar um retngulo de base 1, e se juntarmos todos esses retngulos de base 1, vamos formar outro retngulo, cujos lados medem: 2 e 1.
PROBLEMA 4

Sejam ABCD um quadrado (sentido horrio) e P um ponto qualquer pertencente ao interior do segmento BC. Constri-se o quadrado APRS (sentido horrio). Demonstrar que a reta CR tangente a circunferncia circunscrita ao tringulo ABC.
SOLUO DE LARISSA CAVALCANTE QUEIROZ DE LIMA (FORTALEZA-CE)
A


45

M 45+ S D 45 R

O ABC retngulo, portanto o centro da circunferncia circunscrita est no ponto mdio de sua hipotenusa: AC centro da circunferncia o ponto M *ABCD um quadrado as diagonais se cortam ao meio, e as diagonais so iguais AM = BM = MC = MD

EUREKA! N8, 2000

Sociedade Brasileira de Matemtica

P = e BP A = ; + = 90 ( retngulo ABP); + + 90 = 180 Seja BA C = 180 90 RP C = Note que RP

C = 45( ABC retngulo issceles) * BA C = BA C BA P = 45 PA


AP = PR R = PR A = 45 * APR issceles e retngulo PA APR = 90 C = PA R PA C = 45 (45 ) = 45 45 + = RA

C = RP C = APCR um quadriltero inscritvel RA R = 90 R = AC AP


CR perpendicular a AC e que o dimetro da circunferncia circunscrita a ABC CR tangente.
PROBLEMA 5

No plano cartesiano, considere os pontos de coordenadas inteiras. Uma operao consiste em: Escolher um destes pontos e realizar uma rotao de 90o. no sentido anti-horrio, com centro neste ponto. possvel, atravs de uma seqncia dessas operaes, levar o tringulo de vrtices (0, 0), (1, 0), e (0, 1) no tringulo de vrtices (0, 0), (1, 0) e (1, 1)?
SOLUO ADAPTADA DA SOLUO DE DAVI MXIMO ALEXANDRINO NOGUEIRA (FORTALEZA - CE)

Considere a figura relativa a demonstrao:


Y B A B A B A B B A A B A B A B A B A B A B A X A

A (0,0) B

Considere duas cores A e B. Pinte o ponto (0,0) de A. A partir da, pinte todos os outros pontos (coordenadas inteiras) do plano com as cores A e B,
EUREKA! N8, 2000

Sociedade Brasileira de Matemtica

alternadamente. Isto , pintamos (a, b) de A se a + b par, e de B se a + b mpar. Vamos provar que um ponto e sua imagem possuem a mesma cor. De fato, se P = (x, y), a imagem de (a, b) pela rotao de 90o no sentido antihorrio com centro em P e (x + y b, x + y + a), cuja soma das coordenadas 2x + 2y + a b a + b (mod 2). Como o primero tringulo tem um ponto da cor A e dois da cor B e o segundo tem dois pontos da cor A e um da cor B no possvel tal coisa.
PROBLEMA 6

Existe um inteiro positivo divisvel pelo produto de seus algarismos e tal que esse produto maior que 102000?
SOLUO DE HUMBERTO SILVA NAVES (SO PAULO - SP)

Primeiramente vamos provar que 10 raiz primitiva no mdulo 7n. *Sabemos que quando n = 1 ou n = 2, isto verdadeiro. ** Suponhamos que 10 seja uma raiz primitiva no mdulo 7 n ( n 2) Seja "a" uma raiz primitiva no mdulo 7 n +1 (ela existe pois 7 n +1 uma potncia de um primo), isto : a j percorre todas as classes de congruncia que so primas com 7, no mdulo 7 n +1 , consequentemente "a" tambm raiz primitiva no mdulo 7 n . Pela definio de "a", existe um x N e um y N , tais que:

a x 10(mod 7 n ) a y 10(mod 7 n +1 )
Temos que mdc ( x; (7 n )) = 1, pois 10 tambm raiz primitiva no mdulo 7 n . Se mdc ( y; (7 n +1 )) = d 1, teramos:
y n a 10(mod 7 ) a y 10(mod 7 n +1 ) x a x a y (mod 7 n ) x y (mod (7 n )) n a 10(mod 7 )

y primo com (7 n ) (pois x tambm )


Chegamos a uma contradio, pois mdc ( y; (7 n +1 )) = d e

mdc ( y; (7 n )) = 1, isto quer dizer: mdc ( y;6 7 n ) 1 e mdc ( y;6 7 n 1 ) = 1 (com n 2 ), que um absurdo.
Da conclumos que mdc ( y; (7 n +1 )) = 1.
EUREKA! N8, 2000

Sociedade Brasileira de Matemtica

Logo 10 tambm uma raiz primitiva no mdulo 7 n +1 , e por induo conclumos que: n N; 10 raiz primitiva no mdulo 7 n . Agora vamos achar um exemplo: Considere a, tal que: 7 a > 10 2000 E como 10 raiz primitiva no mdulo 7 a , considere b > a, tal que: 10 b 7 6 10 a (mod 7 a ), temos que:

10 a 1 10 b 10 a x= 0 (mod 7 a ) x 0(mod 7 a ) 9 7 + 9 mas: b a dgitos a dgitos x = 1111111 ... 1 000 ... 0 + 777 ... 7 = 1111111 ... 1 777...7
total de: "b a" "1"'s "a" 0 ' s "a" 7's

Ou seja x divisvel pelo produto de seus dgitos.

Voc sabia
Que foi novamente batido o record de maior primo de Fermat generalizado conhecido? o nmero 4859465536 + 1 descoberto este ano por Scott e Gallot, que o 6o. maior primo conhecido (e o nico primo conhecido com mais de um milho de bits que no de Mersenne). Com isso, os 9 maiores primos conhecidos so de Mersenne ou de Fermat generalizados. So eles: 26972593 1, 23021377 1, 22976221 1, 21398269 1, 21257787 1, 4859465536 + 1, 2859433 1, 2756839 1 e 16717632768 + 1, os quais tm, respectivamente, 2098960, 909526, 895932, 420921, 378632, 307140, 258716, 227832 e 171153 dgitos.

EUREKA! N8, 2000

10

Sociedade Brasileira de Matemtica

VI OLIMPADA DE MAIO
13 de maio de 2000
PRIMEIRO NVEL
Durao da prova: 3 horas PROBLEMA 1

Encontre todos os nmeros naturais de quatro algarismos formados por dois dgitos pares e dois dgitos mpares tais que, ao multiplic-los por 2, se obtm nmeros de quatro algarismos com todos os seus dgitos pares e, ao divid-los por 2, se obtm nmeros naturais de quatro algarismos com todos os seus dgitos mpares.
PROBLEMA 2

Seja ABC um tringulo retngulo em A, cujo cateto AC mede 1cm. A bissetriz do

C corta a hipotenusa em R; a perpendicular a AR traada por R corta o ngulo BA lado AB em seu ponto mdio. Encontre a medida do lado AB.
PROBLEMA 3

Para escrever todos os nmeros naturais consecutivos desde 1ab at ab2 inclusive foram utilizados 1ab1 algarismos. Determine quantos algarismos a mais precisam-se para escrever os nmeros naturais at o aab inclusive. Diga todas as possibilidades. (a e b representam dgitos).
PROBLEMA 4

Temos peas com forma de tringulo equiltero de lados 1; 2; 3; 4; 5 e 6 (50 peas de cada medida). Precisa-se armar um tringulo equiltero de lado 7 utilizando algumas destas peas, sem buracos nem superposies. Qual o menor nmero de peas necessrias?
PROBLEMA 5

Numa fileira temos 12 cartas que podem ser de trs tipos: com as duas faces brancas, com as duas faces pretas ou com uma face branca e a outra preta. Inicialmente temos 9 cartas com a face preta voltada para cima. Viram-se as seis primeiras cartas da esquerda e ficam 9 cartas com a face preta voltada para cima. Continuando, viram-se as seis cartas centrais, ficando 8 cartas com a face preta voltada para cima.
EUREKA! N8, 2000

11

Sociedade Brasileira de Matemtica

Finalmente, viram-se seis cartas: as trs primeiras da esquerda e as trs ltimas da direita, ficando 3 cartas com a face preta voltada para cima. Diga se com esta informao se pode saber com certeza quantas cartas de cada tipo existem na fileira.

SEGUNDO NVEL
PROBLEMA 1

O conjunto {1, 2, 3, 4} pode ser dividido em dois subconjuntos A = {1, 4} e B = {3, 2} sem elementos comuns e tais que a soma dos elementos de A seja igual a soma dos elementos de B. Essa diviso impossvel para o conjunto {1, 2, 3, 4, 5} e tambm para o conjunto {1, 2, 3, 4, 5, 6}. Determine todos os valores de n para os quais o conjunto dos primeiros n nmeros naturais pode ser dividido em dois subconjuntos sem elementos comuns tais que a soma dos elementos de cada subconjunto seja a mesma.
PROBLEMA 2

Num paralelogramo de rea 1 so traadas retas que unem cada vrtice com o ponto mdio de cada lado no adjacente a ele. As oito retas traadas determinam um octgono no interior do paralelogramo. Calcule a rea do octgono.
PROBLEMA 3

Sejam S uma circunferncia de raio 2; S1 uma circunferncia de raio 1 tangente interiormente a S em B e S2 uma circunferncia de raio 1 tangente a S1 no ponto A, mas que no tangente a S. Se K o ponto de interseo da reta AB com a circunferncia S, demonstre que K pertence a circunferncia S2.
PROBLEMA 4

Temos um cubo de 3 3 3 formado pela unio de 27 cubinhos 1 1 1. Retiramos alguns cubinhos de tal modo que os que permanecem seguem formando um slido constitudo por cubinhos que esto unidos pelo menos por uma face ao resto do slido. Quando um cubinho retirado, os que permanecem ficam no mesmo lugar em que estavam inicialmente. Qual o mximo nmero de cubinhos que podem ser retirados de modo que a rea do slido que resulte seja igual rea do cubo original?
PROBLEMA 5

Um retngulo pode ser dividido em n quadrados iguais e tambm pode ser dividido em n + 98 quadrados iguais. Se a rea do retngulo n, com n inteiro, encontre os lados do retngulo. Diga todas as possibilidades.
EUREKA! N8, 2000

12

Sociedade Brasileira de Matemtica

VI OLIMPADA DE MAIO
Resultados PRIMEIRO NVEL Fabio Dias Moreira Guilherme Salermo Santos Raul M. Alexandrino Nogueira Alex Correa Abreu Iuri Lima Ribeiro Antnia Taline de Souza Mendona Cincinato Furtado Leite Neto Alan Hideki Uchida Rodrigo Aguiar Pinheiro Luty Rodrigues Ribeiro SEGUNDO NVEL Marcio Antonio F. Belo Henrique Chociay Davi M. Alexandrino Nogueira Larissa Goulart Rodrigues Andreia Lucio dos Santos Thiago da Silva Sobral Luis Gustavo Bastos Pinho Samuel Barbosa Feitosa Adriano Arantes Paterlini Germanna de Oliveira Queiroz Medalha de Ouro Medalha de Prata Medalha de Prata Medalha de Bronze Medalha de Bronze Medalha de Bronze Medalha de Bronze Meno Honrosa Meno Honrosa Meno Honrosa Goinia - GO Curitiba - PR Fortaleza - CE Goinia - GO Goinia - GO Fortaleza - CE Fortaleza - CE Fortaleza - CE Tatu - SP Fortaleza - CE Medalha de Ouro Medalha de Prata Medalha de Prata Medalha de Bronze Medalha de Bronze Medalha de Bronze Medalha de Bronze Meno Honrosa Meno Honrosa Meno Honrosa Rio de Janeiro - RJ Goinia - GO Fortaleza - CE Niteroi - RJ Fortaleza - CE Fortaleza - CE Fortaleza - CE So Paulo - SP Fortaleza - CE Fortaleza - CE

EUREKA! N8, 2000

13

Sociedade Brasileira de Matemtica

XLI OLIMPADA INTERNACIONAL DE MATEMTICA


13 a 25 de julho, Taejon - Coria do Sul
A XLI Olimpada Internacional de Matemtica foi realizada em Taejon, Coria do Sul, no perodo de 13 a 25 de julho de 2000. A equipe brasileira foi liderada pelos professores lio Mega e Edmilson Motta, ambos de So Paulo SP.

RESULTADOS DA EQUIPE BRASILEIRA


BRA1 BRA2 BRA3 BRA4 BRA5 BRA6 Daniel Nobuo Uno Daniel Massaki Yamamoto Fabrcio Siqueira Benevides Humberto Silva Naves Sergio Tadao Martins Ulisses Medeiros de Albuquerque Bronze Bronze Bronze ------------------Meno Honrosa

Duas circunferncias 1 e 2 intersectam-se em M e N. Seja l a tangente comum a 1 e 2 que est mais prxima de M do que de N. A reta l tangente a 1 em A e a 2 em B. A reta paralela a l que passa por M intersecta novamente a circunferncia 1 em C e novamente a circunferncia 2 em D. As retas CA e DB intersectam-se em E; as retas AN e CD intersectam-se em P; as retas BN e CD intersectam-se em Q. Mostre que EP = EQ.
PROBLEMA 2

PROBLEMA 1

Sejam a, b, c nmeros reais positivos tais que abc = 1. Prove que

a 1+

1 1 1 b 1 + c 1 + 1. b c a

EUREKA! N8, 2000

14

Sociedade Brasileira de Matemtica

Seja n 2 um nmero inteiro positivo. No incio existem n pulgas numa reta horizontal, nem todas no mesmo ponto. Para um nmero real positivo , define-se um salto da seguinte maneira: Escolhem-se duas pulgas quaisquer nos pontos A e B com o ponto A esquerda do ponto B; A pulga que est em A salta at o ponto C da reta, direita de B, tal que BC = . AB Determine todos os valores de para os quais, para qualquer ponto M na reta e quaisquer posies iniciais das n pulgas, existe uma sucesso finita de saltos que levam todas as pulgas para pontos direita de M.
PROBLEMA 4

PROBLEMA 3

Um mgico tem cem cartes numerados de 1 a 100. Coloca-os em trs caixas, uma vermelha, uma branca e uma azul, de modo que cada caixa contm pelo menos um carto. Uma pessoa da platia escolhe duas das trs caixas, seleciona um carto de cada caixa e anuncia a soma dos nmeros dos dois cartes que escolheu. Ao saber esta soma, o mgico identifica a caixa da qual no se retirou nenhum carto. De quantas maneiras podem ser colocados todos os cartes nas caixas de modo de que este truque sempre funcione? (Duas maneiras consideram-se diferentes se pelo menos um carto colocado numa caixa diferente).
PROBLEMA 5

Verifique se existe um inteiro positivo n tal que n divisvel por exatamente 2000 nmeros primos diferentes e 2 n + 1 divisvel por n.
PROBLEMA 6

Sejam AH 1 , BH 2 , CH 3 as alturas de um tringulo acutngulo ABC. A circunferncia inscrita no tringulo ABC tangente aos lados BC, CA, AB em T1 , T2 , T3, respectivamente. Seja l1 a reta simtrica da reta H 2 H 3 relativamente
reta T2T3 , l 2 a reta simtrica da reta H 3 H 1 relativamente reta T3T1 e l 3 a reta simtrica da reta H 1 H 2 relativamente reta T1T2 . Prove que l1 , l 2 , l 3 determinam um tringulo cujos vrtices pertencem circunferncia inscrita no tringulo ABC.

EUREKA! N8, 2000

15

Sociedade Brasileira de Matemtica

INTRODUO GEOMETRIA PROJETIVA


Luciano G. M. Castro Nvel Avanado
Artigo baseado em aula ministrada na III Semana Olmpica Piracicaba - SP
Comeamos com um problema de Geometria Euclidiana:
Problema Inicial:

As tangentes a uma circunferncia de centro O, traadas por um ponto exterior C, tocam a circunferncia nos pontos A e B. Seja S um ponto qualquer da circunferncia. As retas SA, SB e SC cortam o dimetro perpendicular a OS nos pontos A', B' e C ', respectivamente. Prove que C' o ponto mdio de A'B'. Encorajamos o leitor a resolver este problema utilizando mtodos da Geometria Euclidiana, antes de prosseguir. Nossa principal meta desenvolver ferramentas da Geometria Projetiva que nos permitam resolver este e outros problemas similares de forma direta e natural.
1. POLARIDADE

Dada uma circunferncia , de centro O e raio R, vamos criar uma associao entre pontos e retas do plano, da seguinte maneira: Para cada ponto A distinto de O, seja A' o ponto da semi-reta OA tal que OA OA' = R 2 . (A' chamado inverso de A em relao a . A transformao A A' a inverso relativa a ). Seja a a reta perpendicular a OA passando por A'. Dizemos que a a reta polar de A em relao a , e que A o plo de a em relao a .

EUREKA! N8, 2000

16

Sociedade Brasileira de Matemtica

A'

A transformao do plano que leva cada ponto em sua polar e cada reta em seu plo chamada de polaridade. Para simplificar a notao, usaremos a mesma letra para designar um ponto (maiscula) e sua polar (minscula).
Teorema 1: Sejam A e B dois pontos do plano, a e b suas respectivas polares. Se B a, ento A b. Neste caso, dizemos que A e B so conjugados.
a b

Considere um ponto B a. Seja B' OB tal que AB' OB. Os tringulos OAB' e OBA' so retngulos e tm um ngulo comum ( AB' BA' ), logo so semelhantes. Assim,

A O B'

A'

OA OB' = OB OB' = OA OA' = R 2 . OB OA' Logo B' o inverso de B, de onde AB' = b e A b.

Assim, se imaginarmos o ponto B variando ao longo da reta a, sua polar, b, variar ao longo do feixe de retas que passam pelo ponto A. Diremos que um ponto e uma reta so incidentes quando o ponto pertence reta, o que o mesmo que dizer que a reta passa pelo ponto. A polaridade, portanto, uma transformao que preserva incidncias.

EUREKA! N8, 2000

17

Sociedade Brasileira de Matemtica

Exerccio 1: Se um ponto conjugado a si mesmo, ento ele pertence circunferncia e sua polar a tangente circunferncia por ele.

Este resultado nos permite desenvolver a seguinte construo para a reta polar de um ponto A exterior circunferncia:
Exerccio 2: Se A exterior circunferncia, sejam B e C os pontos de contato das

duas tangentes circunferncia traadas por A. A reta BC a polar de A.

Soluo:
B

Como A pertence s polares de B e C, ento B e C pertencem polar de A. Logo a = BC

C a

2. O PLANO PROJETIVO

A polaridade definida anteriormente sugere que pontos e retas tm comportamentos parecidos em relao incidncia. H algumas falhas, porm. A transformao no est definida para o ponto O, centro da circunferncia, nem tampouco para as retas que passam por O. Podemos resolver este problema ampliando o plano euclidiano, acrescentando-lhe uma nova reta que chamaremos de "reta do infinito", que representaremos por o. Esta nova reta ser a polar do ponto O. Formalmente, os pontos da nova reta do infinito esto em correspondncia biunvoca com os feixes de retas paralelas no plano euclidiano.

EUREKA! N8, 2000

18

Sociedade Brasileira de Matemtica

Vejamos como a polaridade nos leva naturalmente a esta definio para os pontos do infinito. Por exemplo, vamos identificar o plo de uma reta r que passa por O. Sejam A e B os pontos de contato de r com a circunferncia. Como A e B esto sobre a reta r, suas retas polares a e b passam pelo plo R. Logo R o ponto de encontro das duas retas a e b, que no plano Euclidiano seriam paralelas. De fato, a reta polar de qualquer ponto de r ser perpendicular a r no plano euclidiano. Estas retas passam a ser, no plano projetivo, um feixe de retas concorrentes (no ponto do infinito R). Esta a maneira de trabalhar com a reta do infinito: cada um de seus pontos corresponde a um nico feixe de retas paralelas no plano euclidiano. E vice-versa: a cada feixe de retas paralelas no plano euclidiano corresponde um nico ponto da reta do infinito.
A a ...

...

... B b

3. O PRINCPIO DA DUALIDADE

Os pontos e retas do plano projetivo tm exatamente o mesmo comportamento em relao a incidncia. Assim, qualquer propriedade envolvendo pontos, retas e incidncia permenece vlida ao trocarmos pontos por retas e retas por pontos. A nova propriedade assim obtida denominada "dual" da primeira. Em outras palavras, para todo teorema da Geometria Projetiva recebemos outro grtis, oferecido pelo Princpio da Dualidade. Basta trocar a palavra "ponto" pela palavra "reta" e vice versa.
EUREKA! N8, 2000

19

Sociedade Brasileira de Matemtica

Exemplos: Propriedade Dual

Dada uma reta, sempre existe um Dado um ponto, sempre existe uma reta ponto no incidente a ela. no incidente a ele. Cada reta incidente a pelo menos Cada ponto incidente a pelo menos trs trs pontos distintos. retas distintas. Dois pontos distintos determinam Duas retas distintas determinam um nico uma nica reta a eles incidente. ponto a elas incidente.
Observao:

Apesar de termos definido o plano projetivo como uma extenso do plano euclidiano, isto no necessrio. O plano projetivo existe de forma independente, podendo ser caracterizado a partir de um conjunto de axiomas, entre os quais esto as propriedades duais citadas anteriormente.
4. QUDRUPLAS HARMNICAS

No plano euclidiano, se quatro pontos A, B, C e D de uma reta so tais que: AC AD = , BC BD dizemos que C e D "dividem harmonicamente" o segmento AB . Observe que, de acordo com a definio, isto tambm implica que A e B dividem harmonicamente o segmento CD . Representaremos esta situao com o smbolo H ( AB, CD). Tambm diremos que A, B, C e D , nesta ordem, formam uma "qudrupla harmnica". Dados os pontos A, B e C sobre uma reta, o ponto D tal que H ( AB; CD) chamado "conjugado harmnico" de C em relao a AB . Surpreendentemente, apesar da definio utilizar a noo de distncia (que no faz sentido no plano projetivo), o conceito de qudruplas harmnicas faz sentido no Plano Projetivo, por meio da seguinte construo para o conjugado harmnico:

EUREKA! N8, 2000

20

Sociedade Brasileira de Matemtica

E F G a1 H c a2 A C b2 b1 Figura 1

Dados os pontos A, B e C sobre uma reta r, traamos duas retas quaisquer a1 e a2 passando por A e uma reta c passando por C. Unindo a B os pontos de incidncia de c com a1 e a2 , respectivamente, obtemos as retas b1 e b2. Fica ento formado um quadriltero (EFHG, na figura) tal que os lados opostos concorrem em A e B, e tal que uma de suas diagonais passa por C. Seja D o ponto de encontro de r com a outra diagonal do quadriltero. Ento D o conjugado harmnico de C em relao a AB . Esta construo a definio de qudruplas harmnicas no plano projetivo. Vejamos que ela coincide, no plano Euclidiano, com a definio usual. Sejam os pontos E, F, G como na figura 1. Aplicando o Teorema de Menelaus* no ABE , secante DGF, temos:

AD BG EF = 1. BD EG AF

(1)

No ABE , aplicamos o Teorema de Ceva* para as cevianas concorrentes EC , BF e AG : AC BG EF = 1. (2) BC EG AF

De (1) e (2) temos

AC AD = . BC BD

*Ver apndice.
EUREKA! N8, 2000

21

Sociedade Brasileira de Matemtica

5. PONTO MDIO E CONJUGAO HARMNICA

O principal indcio de que qudruplas harmnicas so uma noo projetiva o fato de, no plano euclidiano, o ponto mdio de um segmento no possuir conjugado harmnico. Porm, no plano projetivo, sejam A e B pontos sobre a reta r e C o ponto mdio de AB . Ao realizarmos a construo da figura 1, verificamos que FC paralelo a r. No plano projetivo, o conjugado harmnico D o ponto do infinito correspondente ao feixe de retas paralelas a r.
6. FEIXES HARMNICOS

Vamos agora dualizar a definio de qudrupla harmnica. Dadas 3 retas a, b e c concorrentes em um ponto R, podemos dualizar, passo a passo, a construo do conjugado harmnico: Sobre a reta a tomamos dois pontos distintos A1 e A2 e sobre a reta c tomamos um ponto C. Sejam B1 e B2 os pontos de interseco da reta b com as retas CA1 e CA2 , respectivamente. Seja d a reta determinada pelos pontos R e

A1 B2 A2 B1 . Chamamos d de conjugado harmnico de c em relao a a e b.


Dizemos que as quatro retas concorrentes a, b, c, e d formam um "feixe harmnico". Representamos esta situao com o smbolo H (ab, cd ).
R

A2 B1 C d Figura 2

A1 a c

B2 b

Teorema 2: Uma reta qualquer do plano corta um feixe harmnico em quatro

pontos que formam uma qudrupla harmnica.


EUREKA! N8, 2000

22

Sociedade Brasileira de Matemtica

Se voc percebeu a semelhana entre as figuras 1 e 2 deve ter desconfiado deste fato. A demonstrao imediata. Na construo da figura 2, os pontos A1 e B2 podem ser escolhidos sobre uma reta s arbitrria (que no passe por R), e o ponto C fora de s. As retas

A1C e B2 C determinam os pontos B1 e A2. Sendo c s = C ' e d s = D , vemos que o quadriltero RA2 CB1 possui dois lados opostos concorrendo em A1 e B2, com suas diagonais passando por C' e D . Portanto H ( A1 B2 ; C ' D ) , como
queramos demonstrar.
Exerccio 3:

Escreva o dual doTeorema anterior.


7. O TEOREMA DE PASCAL

Sem dvida, um dos mais belos teoremas da Geometria Projetiva. vlido para qualquer cnica, apesar de que aqui s veremos a demonstrao para a circunferncia, no plano euclidiano. importante mencionar, no entanto, que no Plano Projetivo no h qualquer diferena entre uma circunferncia e qualquer outra cnica no-degenerada.
Teorema 3: Os pontos de encontro entre os 3 pares de lados opostos de um hexgono ABCDEF (convexo ou no) inscrito em uma circunferncia so colineares. Consideremos o tringulo XYZ indicado na X figura. Aplicamos o Teorema de Menelaus trs vezes: XYZ , secante PDE: PX DY EZ E =1 PY DZ EX A
P R D Y Z Q F C

XYZ , secante QBC:


QZ BX CY =1 QX BY CZ

XYZ , secante RAF:


B

RY AX FZ =1 RZ AY FX

EUREKA! N8, 2000

23

Sociedade Brasileira de Matemtica

Multiplicando essas trs ltimas equaes e lembrando que XA XB = XE XF, YA YB = YC YD e ZC ZD = ZE ZF (potncia dos pontos x, y, z em relao circunferncia), obtemos
PX QZ RY = 1. PY QX RZ

Logo, pelo recproco do Teorema de Menelaus no tringulo XYZ, secante PQR, temos que P, Q e R so colineares. Fazendo coincidir certos pares de pontos no hexgono ABCDEF, podemos deduzir teoremas anlogos ao de Pascal para pentgonos, quadrilteros e at tringulos inscritos na circunferncia. Por exemplo, fazendo coincidir A com B e D com E, as retas AB e DE tornam-se tangentes circunferncia,
e obtemos a seguinte configurao:
F AB Q P A DE Figura 3 R

Exerccio 4:

Na figura anterior, verifique que o ponto comum s tangentes em C e F tambm pertence reta PQR.
8. MAIS POLARIDADES

Agora estamos prontos para retomar nosso estudo das polaridade. Aproveitando tudo o que vimos at aqui, vamos deduzir algumas propriedades mais avanadas.

EUREKA! N8, 2000

24

Sociedade Brasileira de Matemtica

Teorema 4: (Construo da reta polar usando apenas rgua) Seja uma circunferncia e A um ponto exterior a ela. Consideremos duas retas distintas passando por A e cortando nos pontos B, C, D e E (figura). Ento a reta polar de A em relao a a reta que une os pontos

BD EC e BE CD .
P

Demonstrao:

R C Q A

As polares de B, C, D e E so as retas b, c, d, e e tangentes a em seus respectivos plos. Sendo R = b c e S = d e, temos que as polares de R e S so as retas

r = BC e s = DE. Como A = r s, sua polar a reta a = RS .


Sendo P = BE CD e Q = BD EC , um dos corolrios do Teorema de Pascal garante que P, Q, R e S so colineares,

logo a = PQ, demonstrar.

como

queramos

Teorema 5: (Relao entre reta polar e qudruplas harmnicas) Dados uma circunferncia e um ponto exterior A, qualquer reta secante circunferncia passando por A corta a polar a no conjugado harmnico do ponto A em relao ao segmento com extremos nos dois pontos de corte da secante com a circunferncia. Demonstrao: Exerccio 5

(Dica: na figura anterior, use o quadriltero PBQC para encontrar o conjugado harmnico de A em relao a ED ).

EUREKA! N8, 2000

25

Sociedade Brasileira de Matemtica

9. RESOLUO DO PROBLEMA INICIAL:

Podemos agora apresentar uma soluo simples e elegante para o problema proposto no incio deste artigo.

A' O

S s

C' B c B' d D

Seja d o dimetro perpendicular a OS .


Seja D o ponto do infinito correspondente ao feixe de retas paralelas a d. Queremos provar que H ( A' B ' , C ' D ) . Para isto, basta provar que as retas SA ,

SB , SC e SD formam um feixe harmnico. Parece natural tentar verificar que


a reta AB corta o feixe em uma qudrupla harmnica. Mas isso equivale a provar que SC a reta polar do ponto SD AB. Isto simples:

C a interseco das polares de A e B, logo sua polar c = AB. SD tangente circunferncia no ponto S, logo a polar de S ( SD = s ) .
Assim, SD AB = s c, e sua polar , portanto, SC , como queramos demonstrar.

EUREKA! N8, 2000

26

Sociedade Brasileira de Matemtica

APNDICE: TEOREMA DE CEVA:


A

P Q

Suponha que as cevianas AM, BN e CP de um tringulo ABC se encontrem em um ponto Q. Ento AN CM BP = 1. CN BM AP

Suponha que Q = t1 A + t 2 B + t 3 C com t1 + t 2 + t 3 = 1. t B + t 3C t A + t 3C t A + t2 B , N= 1 e P= 1 . Ento teremos M = 2 t 2 + t3 t1 + t 3 t1 + t 2 AN CM BP t 3 t 2 t1 Assim, =1 CN BM AP t1 t 3 t 2


TEOREMA DE MENELAUS:
A

Prova:

Suponha que X AB, Y BC e Z AC sejam colineares. Ento


X

Prova:
B C Y

AX BY CZ = 1. BX CY AZ

Suponha que X = tA + (1 t ) B e Y = sB + (1 s )C. Ento Z = uX + (1 u )Y , onde u tal que (1 t )u + s (1 u ) = 0, ou seja, (1 s )(1 t ) st Z= C. Assim, A s + t 1 s + t 1 AX BY CZ 1 t 1 s st = =1 BX CY AZ t s (1 s)(1 t )

EUREKA! N8, 2000

27

Sociedade Brasileira de Matemtica

CONTAR DUAS VEZES PARA GENERALIZAR (O RETORNO)


Jos Paulo Q. Carneiro, Universidade Santa rsula

Nvel Avanado
( 2) 1. A frmula que d diretamente a soma dos quadrados S n = 12 + 22 + + n 2 dos n primeiros inteiros positivos pode ser deduzida de vrias maneiras (por exemplo, [3]). Uma das mais comuns partir da identidade:

(k + 1)3 k 3 = 3k 2 + 3k + 1 , escrev-la para k variando de 1 at n:

23 13 = 3 12 + 3 1 + 1 33 23 = 3 22 + 3 2 + 1
.........................................

(n + 1)3 n3 = 3n2 + 3n + 1
(n + 1)3 13 = 3Sn( 2) + 3Sn(1) + n
n(n + 1) , como bem conhecido (ver [1]). 2

e somar termo a termo estas n igualdades, obtendo:

(1) onde S n = 1+ 2 ++ n =

Substituindo este valor e fazendo as contas, chega-se a :


( 2) Sn = 12 + 22 + + n 2 =

n(n + 1)(2n + 1) 6

Esta deduo bastante eficiente e rpida, mas, quando apresentada pela primeira vez a um estudante, costuma deixar aquela sensao de coelho tirado da cartola, devido ao aparecimento sbito de uma identidade cuja motivao no se sabe de onde veio. Este tipo de sensao desperta admirao em uns, mas em outros inspira uma frustrao, proveniente da reflexo: eu nunca vou conseguir bolar um artifcio destes!. Coloca-se, portanto, a questo: h algum problema onde a soma dos quadrados aparea naturalmente? E, para este problema, h alguma outra maneira de resolv-lo, por meio da qual possamos deduzir a frmula da soma dos quadrados?

2. Tradicionalmente, em problemas de contagem, o smbolo Cnp ( combinao de n, p a p) representa o nmero de subconjuntos de p elementos contidos em 2 o um conjunto de n elementos. Se, por exemplo, fizermos p = 2 , ento Cn nmero de pares (no ordenados) que se pode extrair de um conjunto com n
EUREKA! N8, 2000

28

Sociedade Brasileira de Matemtica

elementos. Exemplos: o nmero de apertos de mo dados por n pessoas quando cada uma cumprimenta todas as outras somente uma vez, ou ainda o nmero de partidas de futebol em um campeonato com um s turno e n equipes. Em [1], um artigo com o mesmo ttulo que o presente aproveitava justamente o ltimo exemplo citado para mostrar como, resolvendo um mesmo problema de contagem por dois mtodos diferentes, era possvel deduzir que:
2 Cn = 1 + 2 + + (n 1) =

(n 1)n . 2

2 3. Os pitagricos (sec.VI a.C.) chamavam os nmeros Cn de nmeros triangulares. O motivo que eles podem ser vistos como tringulos nas figuras:

T1 = 1 T2 = 1 + 2 = 3 T3 = 1 + 2 + 3 = 6 T4 = 1 + 2 + 3 + 4 = 10
2 Deste modo: Cn = Tn 1 , para n > 1 . Alm dos nmeros triangulares, os pitagricos consideravam tambm os nmeros quadrados Q1 = 12 = 1 , Q2 = 22 = 4 , etc., que podem ser visualizados como quadrados (da seu nome).

Estas figuras pitagricas sugerem tambm uma relao interessante entre os nmeros triangulares e os nmeros quadrados. Se voc partir o quadrado usando a diagonal sudoeste-nordeste, e incluindo esta diagonal na parte de baixo, voc poder olhar cada nmero quadrado como a soma de dois nmeros triangulares consecutivos; mais especificamente: Qn = Tn 1 + Tn .

22 = 1 + 3
EUREKA! N8, 2000

32 = 3 + 6

42 = 6 + 10

29

Sociedade Brasileira de Matemtica

Esta relao pode, claro, ser confirmada algebricamente, j que:

Tn 1 + Tn =

(n 1)n n(n + 1) + = n 2 = Qn . 2 2

4. A observao precedente pode ser usada para calcular a soma dos quadrados dos n primeiros nmeros naturais. De fato:

Q1 = T1 Q2 = T1 + T2 Q3 = T2 + T3
...................

Qn = Tn 1 + Tn
( 2) Somando termo a termo, temos: S n = Q1 + + Qn = 2(T1 + + Tn 1 ) + Tn . S

resta agora calcular T1 + + Tn 1 , isto , a soma dos n 1 primeiros nmeros triangulares. Para isto, lembremos que esta soma o mesmo que 2 2 C2 + C32 + Cn , a qual vamos calcular pelo artifcio de resolver um mesmo problema por duas contagens diferentes (ver [1]). O nmero de subconjuntos de 3 elementos contidos em um conjunto A de n + 1 3 Vamos contar estes elementos representado, como j se sabe, por C n +1 . subconjuntos. Para formar um subconjunto de A com 3 elementos, primeiramente escolhemos um elemento a A . Para isto, temos n + 1 escolhas. Uma vez escolhido a, temos n escolhas possveis para tomar um segundo elemento b; e para cada escolha de a e b, temos n 1 escolhas possveis para selecionar o terceiro elemento c. Isto d ento um total de (n + 1)n(n 1) escolhas. Mas claro que esta contagem inclui repeties. Para cada a, b, c escolhidos, houve 6 repeties, correspondentes s 6 permutaes destes elementos, a saber: a, b, c ; a, c, b ; (n + 1)n(n 1) 3 b, a, c ; b, c, a ; c, a, b ; c, b, a . Portanto: C n . +1 = 6 Por outro lado, se quisermos evitar desde o incio as repeties, podemos contar do seguinte modo. Primeiramente, fixamos o elemento a; o nmero de subconjuntos de A com 3 elementos e que possuem a o mesmo que o de 2 subconjuntos de A {a} com 2 elementos, isto : Cn . Tomemos agora um
EUREKA! N8, 2000

30

Sociedade Brasileira de Matemtica

segundo elemento b a . O nmero subconjuntos de A com 3 elementos, que possuem b mas no a, o mesmo que o de subconjuntos de A {a; b} com 2
2 elementos, isto : Cn 1 . Analogamente, o nmero subconjuntos de A com 3 elementos, que contm c, mas no intersectam {a, b} , o mesmo que o de 2 subconjuntos de A {a; b; c} com 2 elementos, isto : Cn 2 . E assim por diante, at que cheguemos ao antepenltimo elemento, quando j teremos contado todos 3 2 2 2 os subconjuntos A com 3 elementos. Logo: Cn +1 = Cn + Cn 1 + + C2 .

Deste modo, conclumos que:


2 2 3 + C32 + Cn = Cn T1 + + Tn 1 = C2 +1 =

portanto, calcular a soma dos n 1 primeiros nmeros triangulares. Da conclumos que:


( 2) Sn = Q1 + + Qn = 2(T1 + + Tn 1 ) + Tn =

(n + 1)n(n 1) . 6

Conseguimos,

n(n + 1)(2n + 1) . 6

(n + 1)n(n 1) (n + 1)n + 3 2

Podemos generalizar as frmulas acima, calculando de duas maneiras diferentes o nmero de subconjuntos de k + 1 elementos contidos em um conjunto A de n + 1 k +1 elementos, que representado por C n +1 .
k +1 A primeira expresso para C n +1 clssica e pode ser provada do mesmo modo que foi feito para k + 1 = 3: temos
k +1 Cn +1 =

( n + 1)n( n 1)(n 2)...(n k + 1) ( n + 1)! = (k + 1)! ( k + 1)!(n k )!

(lembremos que m! = 1 . 2 . ... . m). Seja agora A = {a1 , a 2 ,..., a n +1 }. O nmero de subconjuntos de k + 1 elementos de
k A que contm a1 C n (escolhemos os k elementos de A diferentes de a1 ). O nmero de subconjuntos de k + 1 elementos de A que contm a 2 mas no contm k a1 C n 1 , e assim sucessivamente, o que mostra a igualdade k +1 k k k Cn +1 = C n + C n 1 + ... + C k .

EUREKA! N8, 2000

31

Sociedade Brasileira de Matemtica


k Se Pk (n) = C n + k 1 =

( n + k 1)(n + k 2)...(n + 1) n o "polinmio triangular k! generalizado de dimenso k", temos que Pk (n) um polinmio em n de grau k, e, pela frmula acima, temos k k +1 Pk (1) + Pk ( 2) + ... + Pk ( m) = C kk + C kk+1 + ... + C m + k 1 = C m + k . Podemos, como antes, escrever n k como uma combinao linear dos polinmios Pj (n),0 j k , e usar a frmula acima para obter uma frmula para
(k ) Sn = 1k + 2 k + ... + n k (essa frmula ser a combinao correspondente dos

+1 termos C nj+ j , com 0 j k ).

Tal frmula tambm pode ser obtida recursivamente como no incio do artigo, somando as identidades ( j + 1) k +1 j k +1 = C kr +1 j r , desde j = 1 at j = n,
r =0 k

ficando o lado esquerdo igual a


(k ) (k ) (r ) = (k + 1) S n + C kr +1 S n , o que d S n
r =0 k 1

(n + 1) k +1 1

e o direito igual a

k 1 1 (r ) k +1 ( n 1 ) 1 C kr +1 S n + . k +1 r =0

Referncias Bibliogrficas: [1] Carneiro, J.P., Contar duas vezes para generalizar, Eureka!, n6, pp.15-17, 1999. [2] Eves, H., Introduo Histria da Matemtica, Editora da UNICAMP, 1995 [3] Valadares, E.C., e Wagner, E., Usando geometria para somar, Revista do Professor de Matemtica, n39, pp.1-8, 1999.
EUREKA! N8, 2000

32

Sociedade Brasileira de Matemtica

O PRINCPIO DO ELEMENTO EXTREMO


Jos Rosales Ortega Escola de Matemtica, Instituto Tecnolgico de Costa Rica

Nvel Avanado
Resumo
O artigo expe um princpio de natureza heurstica chamado o "princpio do extremo", que permite resolver problemas matemticos de nvel olmpico de maneira simples.

1.- Introduo. Muitos matemticos profissionais desejam contribuir para tornar a Matemtica mais atrativa aos estudantes com talento. Uma forma de seguir este objetivo criar problemas que requeiram uma grande dose de sentido comum, imaginao, e muitas vezes, uma estratgia especfica de resoluo de problemas. Este artigo introduz uma dessas estratgias, o "princpio do elemento extremo". Ainda que este nome no seja amplamente usado, este princpio pode lhe ajudar a resolver problemas matemticos que aparecem freqentemente em olimpadas. O material baseado na experincia pessoal ganha ao trabalhar com estudantes com talento em matemtica e na minha participao como organizador de vrias competies olmpicas. 2.- A idia do princpio. Considere uma fileira de estudantes ordenada em forma decrescente segundo a altura. A maioria deles tem dois vizinhos. Dois "elementos extremos", o mais alto e o mais baixo, tem somente um vizinho, porm estes dois elementos extremos possuem outras propriedades muito teis. Por exemplo, quando contamos os estudantes na fileira, a melhor maneira comear com um destes elementos extremos. Em matemtica, algumas vezes trabalhamos com conjuntos cujos elementos parecem ser equivalentes e cujas propriedades conhecidas so poucas. Uma estratgia poderosssima em tais casos considerar o elemento, ou os elementos, que de alguma forma so elementos extremos. Por exemplo, quando consideramos um conjunto infinito de nmeros naturais, o elemento extremo seu elemento menor. Para um conjunto finito de nmeros reais os elementos extremos so o mximo e o mnimo do conjunto.
EUREKA! N8, 2000

33

Sociedade Brasileira de Matemtica

Em muitos casos o elemento extremo atrativo devido a que suas propriedades adicionais nos permitem obter concluir sobre o mesmo elemento, ou sobre o do conjunto como um todo. Por exemplo, em um tringulo o lado maior se ope ao ngulo maior e vice-versa. Na continuao apresentamos mais exemplos.

Exemplo 1. Sejam os ngulos de um tringulo. Como o ngulo maior, ento / 3, j que, caso contrrio, teramos < / 3, < / 3 e < / 3 , o que condradiz o fato de que + + = . Da mesma forma podemos concluir que / 3. Tambm no difcil obter que, se o menor ngulo de um polgono convexo com n lados (n > 3), ento (n 2) / n. Para provar isto assuma o contrrio, e use o resultado que estabelece que a soma dos ngulos no polgono igual a (n 2) . Exemplo 2. Considere trs raios com origem comum num mesmo plano, formando trs ngulos a b c, tal que a + b + c = 2 . fcil ver que c 2 / 3 e a 2 / 3. Expresses similares podem ser encontradas se, em lugar de trs raios considerarmos n raios com um origem comum.

a c

Exemplo 3. Os exemplos anteriores podem ser generalizados se considerarmos uma sucesso a1 a 2 ... a n de nmeros reais tais que s = a1 + a 2 + ... + a n .
Ento a1 s / n e a n s / n. Estes exemplos so elementares, mas eles preparam o caminho para resolver o primeiro exemplo no trivial.

Exemplo 4. Seis pontos em um plano so tais que quaisquer trs deles no so colineares. Prove que trs desses seis pontos formam um tringulo que possui um 2 . ngulo interno maior ou igual a 3
EUREKA! N8, 2000

34

Sociedade Brasileira de Matemtica

Soluo: Denote os pontos por A1 . A2 ,..., A6 , e seja M seu fecho convexo. Podem ocorrer dois casos:

M possui seis vrtices. Aplicando o resultado da segunda parte do exemplo 1, para n = 6, vemos que o maior dos ngulos de M satisfaz a desigualdade 2 / 3. Se denotarmos por Ai o vrtice de , e por A j e

Ak os vrtices adjacentes a , ento o tringulo Ai A j Ak tem a


propriedade requerida.

M possui menos de seis vrtices. Neste caso existem trs vrtices Ai , A j e Ak de M, e um ponto Al dentro do tringulo Ai A j Ak .
Ai

Al

Aj Ak

Aplicando o resultado do exemplo 2 aos raios AlAi, AlAj, e AlAk, segue-se que o maior dos ngulos satisfaz a desigualdade 2 / 3. Ento o tringulo Al A j Ak possui a propriedade requerida.

3.- Aplicaes Geomtricas. As aplicaes nesta seo esto relacionadas com objetos geomtricos. Em cada caso o problema resolvido ao encontrar a maior ou a menor distncia, ngulo ou rea. Problema 1.- Em certo pas existem 100 cidades. As distncias entre cada par de cidades esto especificadas, e todas so diferentes. Uma estrada conecta duas cidades A e B se, e somente se, B a cidade mais prxima de A ou A a cidade mais prxima de B.

Prove que existem no mximo 5 estradas que saem de cada cidade. possvel que algumas das estradas formem um polgono?

EUREKA! N8, 2000

35

Sociedade Brasileira de Matemtica

Soluo: Provemos a primeira parte. Considere uma cidade X e duas estradas XA e XB que ligam X a A e a B, respectivamente.
X

Segue-se que AB o maior lado do tringulo ABX . Isto verdade, pois, se (por exemplo) AX for o maior lado do tringulo ABX , ento nem A a cidade mais prxima para X, nem X a cidade mais prxima para A e portanto a estrada X no B o maior ngulo no tringulo ABX . deveria existir. Portanto, o ngulo AX B 60 porque ABX escaleno. Segue-se (exemplo 1) que AX Suponha que exista uma cidade X e que seis estradas vo desde X at outras cidades. Ento a soma dos seis ngulos em volta de X deveria ser maior que 6 60 = 360 , o que impossvel. Mostremos a segunda parte. Para isto suponhamos que existam estradas que formam um polgono. A estrada AB foi construda por um dos seguintes motivos:
A B

D C

B a cidade mais prxima de A, ou A a cidade mais prxima de B.

Considere ainda sem perda de generalidade, que AB o maior lado do polgono. Ento CA < AB e BD < AB. Portanto, B no a cidade mais prxima de A e A no a cidade mais prxima de B. Logo, a estrada AB no deveria existir. Segue-se que tal polgono no existe.

Problema 2.- Os comprimentos das bissetrizes de um tringulo ABC so menores


ou iguais a 1. Prove que a rea do tringulo menor ou igual a
EUREKA! N8, 2000

3 / 3.

36

Sociedade Brasileira de Matemtica

Soluo: Seja = BAC o menor ngulo do tringulo, e seja AD a sua bissetriz. AB e AC no podem ser ambos maiores que AD / cos( / 2). Para demonstrar isso, veja a figura: AD Ax = Ay = y cos( / 2)
B D

/2

Suponhamos que AB AD / cos( / 2). . Como 60 , ento

AB

AD cos

AD 2 . cos 30 3

2 Denotemos por hc e l c a altura e a bissetriz do vrtice C do tringulo ABC, respectivamente. Ento, a rea do tringulo : 1 1 1 ( ABC ) = hc AB l c AB . 2 2 3
Problema 3.- Sejam n(n > 3) pontos num plano tais que a rea de qualquer tringulo com trs desses pontos como vrtices no seja maior que 1. Prove que todos os pontos esto contidos num tringulo, cuja rea menor ou igual a 4. Soluo: Este problema tem o aspecto de ser muito difcil. A idia para resolv-lo est baseada no seguinte: se voc tem um tringulo de rea 4, como poderia relacion-lo com um tringulo de rea 1? Uma boa idia conectar os pontos mdios dos lados do tringulo de rea 4. A rea do tringulo obtido 1. Raciocinando inversamente, se temos um tringulo de rea 1 e se traarmos paralelas m, n, p aos lados AB, BC e CA (de modo que C m, A n e B p ), respectivamente, obteremos um tringulo de rea 4. Agora no difcil completar a soluo. Considere todos os tringulos cujos vrtices so trs quaisquer dos n pontos dados. Seja ABC o tringulo de maior rea. Trace as retas m, n, p como foi descrito anteriormente. Se o ponto A e o ponto X onde X um dos n pontos dados esto em diferentes lados de m, ento ( ABX ) > ( ABC ). Os outros casos so anlogos. Segue-se que nenhum dos pontos
EUREKA! N8, 2000

37

Sociedade Brasileira de Matemtica

dados est fora do tringulo MNP tringulo formado pelas interseces de m, n, p. Como ( ABC ) 1, ento o tringulo MNP contm os pontos, e ( MNP) 4.

4.- Aplicaes Algbricas. Problema 4.- Em cada quadrado de um tabuleiro com infinitas fileiras e colunas, se escreve um nmero natural. O nmero escrito em cada quadrado igual mdia dos nmeros escritos em todos seus quadrados vizinhos (Dois quadrados so vizinhos se eles compartilham um lado em comum.) Prove que todos os nmeros escritos so iguais. Soluo: Aqui aplicaremos o famoso resultado sobre conjuntos no vazios de nmeros naturais, o qual estabelece que sempre h um elemento mnimo. Seja f o menor dos nmeros naturais escritos no tabuleiro, e sejam a, b, c, d os nmeros escritos nos quatro quadrados vizinhos de f. Ento

f =

quer dizer a + b + c + d = 4 f . Como f o elemento mnimo, segue-se que a f , b f , c f e d f . Se uma destas quatro desigualdades no uma igualdade, ento teramos a + b + c + d > 4 f , o que um absurdo. Portanto, se x um nmero escrito em uma casa da mesma coluna da casa na qual est escrito o nmero f, ento x = f. O mesmo resultado vlido para as linhas. Logo, todos os nmeros escritos so iguais.

a+b+c+d , 4

Problema 5.- Em cada quadrado de um tabuleiro de m fileiras por n colunas, se escreve um nmero real. O nmero em cada quadrado igual a mdia dos nmeros escritos em todos seus quadrados vizinhos. (Dois quadrados so vizinhos se eles compartilham um lado comum.) Prove que todos os nmeros escritos so iguais. A soluo deste exerccio um pouco diferente da do exerccio prvio. Soluo: H duas coisas diferentes. Primeiro, alguns quadrados tem menos que quatro quadrados vizinhos. O leitor pode adaptar facilmente a situao ao raciocnio da soluo do exemplo 4. Segundo, a existncia do nmero menor se baseia em uma razo diferente: cada conjunto finito de nmeros possui um elemento menor. Este um assunto importante. Se estamos tratando com elementos extremos, devemos estar certos de que existem, qualquer que seja a razo.

EUREKA! N8, 2000

38

Sociedade Brasileira de Matemtica

Problema 6.- Prove que no existem inteiros positivos x, y, z e t tais que

x 2 + y 2 = 3( z 2 + t 2 ).
Soluo: Algumas vezes no fcil imaginar como introduzir um elemento extremo. Uma boa ideia nestes casos assumir a negao da proposio, e ver onde se pode encontrar uma contradio. Assuma que existem inteiros positivos x, y, z e t tais que x 2 + y 2 = 3( z 2 + t 2 ). J
que x 2 + y 2 divisvel por 3, ento x, y tambm so divisveis por 3 (Prove). Portanto, x = 3m e y = 3n, onde m, n so inteiros positivos. Depois de substituir 3m por x e 3n por y na equao, e dividindo por 3, obtemos que

z 2 + t 2 = 3(m 2 + n 2 ).
Pela mesma razo que antes se conclui que z = 3p e t =3q, onde p, q so inteiros positivos. Logo, a equao original equivalente a

m 2 + n 2 = 3( p 2 + q 2 ).
Portanto, obtivemos inteiros positivos m, n, p e q, que satisfazem a equao, e tais que m < x, n < y, p < z e q < t. O argumento anterior pode ser usado indefinidamente para obter sucesses decrescentes de nmeros inteiros positivos, o que impossvel. Logo, a idia considerar o menor elemento, em algum sentido. Sejam x, y, z e t inteiros positivos tais que x 2 + y 2 = 3( z 2 + t 2 ) e a soma

( x 2 + y 2 ) a menor entre todas as solues da equao. Seguindo o raciocnio de


antes obteremos os nmeros m, n, p e q, que satisfazem a equao, com m < x e n < y. Portanto,

m2 + n2 < x2 + y 2.
Esta uma contradio.

5.- Aplicaes Variadas.


A pergunta "Como comear a soluo?" parece ser a principal pergunta nas solues dos problemas deste artigo. Espera-se que, quanto maior a quantidade de exemplos que o leitor vir, maior ser a experiencia ganha. Portanto exporemos mais exemplos que ajudem a exemplificar o princpio do extremo.

Problema 7.- Existir uma funo f : N* N*; onde N* o conjunto dos inteiros positivos tais que se cumpra a seguinte igualdade para cada nmero natural n > 1:

f (n) = f ( f (n 1)) + f ( f (n + 1)) ?

Soluo: A resposta NO. Para ver isto observe que entre os valores
EUREKA! N8, 2000

39

Sociedade Brasileira de Matemtica

f (2), f (3),..., f (n),...,


deve haver um elemento mnimo, digamos que seja f(n0), onde n0 > 1. Observe que

f (n0 + 1) f (n0 ) = f ( f (n0 1)) + f ( f (n0 + 1)) 1 + 1 > 1.


Como

f (n 0 + 1) > 1, ento f ( f ( n0 + 1)) { f (2), f (3),...}


Portanto, f ( f ( n0 + 1)) f ( n0 ), o que implica que

f (n0 ) = f ( f (n0 1)) + f ( f (n0 + 1)) 1 + f (n0 ),


o que impossvel.

Problema 8.- Cada quadrado de um tabuleiro de dimenses 8 8 contm ou um 0 ou um 1. Para cada quadrado A que contm um 0, a soma dos nmeros na mesma fileira de A e os nmeros na mesma coluna de A maior ou igual a 8. Prove que a soma de todos os nmeros no tabuleiro maior ou igual a 32. Soluo: Considere a soma dos nmeros em cada fileira e em cada coluna. Escolha a menor destas somas. Suponha que tal soma corresponda fileira L. Denote por k o nmero de nmeros 1 que aparecem em L. Podem ocorrer os seguintes casos:

k 4. Ento cada fileira contm ao menos quatro nmeros 1. Portanto, a soma de todos os nmeros no tabuleiro maior ou igual a 4 8 = 32. k < 4. Ento existem 8 k zeros em L. Cada coluna que cruza L em um quadrado com um 0 contm no menos que 8 k uns. Portanto, a soma de todos os nmeros no tabuleiro maior ou igual a

(8 k ) 2 + k 2 = 2(32 8k + k 2 ) = 2((k 4) 2 + 16) 2 16 = 32.


Uma extenso do princpio do extremo a seguinte regra: "ordene os elementos segundo o seu tamanho (valor)". Esta regra usada na soluo do seguinte problema.

Problema 9.- A soma de 17 inteiros positivos distintos igual a 1000. Prove que podem ser escolhidos 8 destes inteiros de tal forma que a sua soma maior ou igual a 500.
EUREKA! N8, 2000

40

Sociedade Brasileira de Matemtica

Soluo: Ordene os inteiros em uma fileira

a1 < a 2 < ... < a17 . Considere o

nmero a9 que a metade da fileira, e o valor mdio da soma, parte inteira de 1000/17, que 58. Podem ocorrer os seguintes casos:

a9 58. Ento a10 59, a11 60,..., a17 66. Portanto, a10 + a11 +...+ a17 59 + 60 +...+ 66 = 500. a9 < 58. Ento a9 57, a8 56,..., a1 49. Portanto, a1 + a2 +...+ a9 49 + 50 +...+ 57 = 477.

Segue que a10 + a11 +...+ a17 1000 477 > 500.

Problema 10.- Encontre todas as solues positivas do sistema


2 2 2 2 x1 + x 2 = x3 , x 2 + x3 = x 4 , x3 + x 4 = x5 , x 4 + x5 = x12 , x5 + x1 = x 2

Soluo: Sejam x e y o maior e o menor dos nmeros x1 ,..., x5 , respectivamente.


Observe que temos que x 2 2 x e y 2 2 y. Como x > 0 e y > 0 segue-se que x 2 e que y 2, logo se conclui que

2 y x 2.
Portanto, segue-se que a nica soluo do sistema dada por

x1 = x 2 = ... = x5 = 2.
6.- Exerccios.
Nesta seo voce encontrar alguns problemas que so resolvidos por meio do princpio do extremo, estando claro que pode haver outras solues que no usem este princpio. Mas pede-se ao leitor que faa todo o esforo possvel para resolver os seguintes exerccios usando unicamente o princpio do extremo. 1. Os nmeros positivos x, y e z so tais que

x=
Prove que x = y = z.

2y 2z 2x ,y = ,z = . 1+ y 1+ z 1+ x

2. Seis crculos iguais num mesmo plano possuem um ponto em comum. Prove que um dos crculos contm o centro de outro dos crculos.
EUREKA! N8, 2000

41

Sociedade Brasileira de Matemtica

3. Oito pontos so escolhidos dentro de um crculo de raio um. Prove que existem dois pontos cuja distncia menor que 1. 4. A soma de vrios nmeros reais no negativos 3, e a soma de seus quadrados estritamente maior que um. Prove que podem ser escolhidos trs destes nmeros cuja soma estritamente maior que um.

Referncias
[1] [2] [3] [4] [5] [6] Mara Falk de Losada, Problemas y Soluciones 1987-1991, Nivel Superior, Universidad Antonio Nario, Colombia, 1994. Eduardo Wagner, Carlos Gustavo T. de A. Moreira et al, 10 Olimpadas Iberoamericanas de Matemtica, OEI, Madrid, 1996. Loren Larson, Problem -Solving Through Problems, Springer - Verlag, New York, 1983. G. Polya, How to solve it, Princeton University Press, USA, 1965. D. O. Shklarsky, N.N Chentzov e I.M. Yaglom, The USSR Olympiad Problem Book, Dover Publications, New-York, 1993. Ravi Vakil, A Mathematical Mosaic: patterns and problem-solving, Bredan Kelly Publishing, Burlington, Ontario, 1996.

EUREKA! N8, 2000

42

Sociedade Brasileira de Matemtica

Carlos Gustavo. T. de A. Moreira, IMPA & Nicolau Saldanha, PUC-Rio

FUNES MULTIPLICATIVAS E A FUNO DE MBIUS*

Nvel Avanado
Recordamos inicialmente uma propriedade da funo de Euler, provada em [2] (Lema 2, pgina 52). Lembremos que, para n inteiro positivo, ( n) := # {a Z / nZ | a invertvel mdulo n } = #{k Z | 0 k < n e mdc ( m, n) = 1}.

Teorema 1: Para todo natural n,

(d ) = n.
d |n

Prova: Considere as n fraes 0 1 n 1 , ,..., n n n


e simplifique cada uma delas: obtemos assim, para cada d|n, (d) fraes com denominador d, donde segue a identidade do enunciado. Mais formalmente, dado a Z / nZ , sejam d = n/(n, a) e a' = a/(n, a). Claramente a ' (Z / dZ)*, e definimos assim uma funo de Z /( n) para a unio disjunta dos conjuntos (Z / dZ) * , onde d varia sobre os divisores de n. A inversa desta funo leva a ' (Z / dZ) * em a , com a = na ' / d , donde a funo uma bijeo O processo de construir g a partir de f como g ( n) = f ( d )
d |n

bastante comum em teoria dos nmeros Um fato interessante sobre este tipo de construo ligado noo de funes multiplicativas. Dizemos que f : N C multiplicativa se mdc(m, n) = 1 f ( mn) = f ( m) f (n). A funo de Euler, por exemplo, multiplicativa (ver o corolrio da pgina 47 de [2]). Se f uma funo 1 1 k multiplicativa e n = p1 p 2 ... p k a fatorao prima de n, ento
* Adaptado do livro Primos de Mersenne (e outros primos muito grandes), dos mesmos autores([1]).
EUREKA! N8, 2000

43

Sociedade Brasileira de Matemtica

f (n) =

f ( p ). Alm disso, vale a seguinte


i =1 i
i

Proposio: Se f : N C multiplicativa ento g : N C , g (n) = f ( d )


d |n

multiplicativa.

Prova: Se mdc(m, n) = 1,g (mn) =

d |mn

f (d ) = f (d1 d 2 ) = f (d1 ) f (d 2 ) =
d1 |m d 2 |n d1 |m d 2 |n

f ( d 1 ) f ( d 2 ) = g (m) g (n) d |m d |n 1 2
Note que esta proposio fornece uma nova prova do Teorema 1: pela multiplicidade de (d ) , basta provar que ( d ) = n se n potncia de
d |n d |n

primo, mas se p primo


d| pk

(d ) = ( p j ) = 1 + ( p j ) = 1 + ( p j p j 1 ) = p k .
j =0 j =1 j =1 d |n

Seria interessante poder inverter em geral identidades do tipo g (n) = f ( d ) para escrever f a partir de g. O teorema anterior nos mostra que se fazemos f = na equao acima temos g(n) = n; invertendo esta identidade teramos uma frmula para . Vamos mostrar como fazer este tipo de inverso. Definimos a funo de Mbius : N Z por

( n) =

(1) m , se n = p1 p 2 ... p m , com p1 , p 2 ,..., p m primos distintos, 0, se n tem algum fator primo repetido em sua fatorao.

Assim, (1) = (6) = (10) = 1, (2) = (3) = (5) = (7) = 1 e (4) = (8) = (9) = 0. Note que uma funo multiplicativa.

EUREKA! N8, 2000

44

Sociedade Brasileira de Matemtica

Lema: Para todo inteiro positivo n temos 1, se n = 1, (d ) = 0, se n > 1. d |n Dem: Como multiplicativa, h( n) = (d ) multiplicativa.
d |n

Temos h(1) = 1 e, para cada p primo e k 1 inteiro, h( p k ) = ( p j ) = 1 + (1) = 0,


j =0

donde, se n > 1, n = p1 ... p k h( n) = h( p1 )h( p 2 )h( p k ) = 0

Teorema 2: (Frmula de inverso de Mbius) Se para todo n > 0 temos g ( n) = f ( d )


d |n

ento

g ( n) = (n / d ) g (d ).
d |n

Dem: Basta provar que

f (n) = (n / d ) f (d ' ) . d |n d '|d


Mas, escrevendo d'' = n/d e m = n/d' temos

(n / d ) f ( d ' ) = ( d ' ' ) f ( n / m) = f ( n )


d |n

d '|d

m|n d ''|m

Corolrio: Para todo natural n, ( n) = ( n / d )d = n


d |n d |n

. d Teorema 1.22: (Segunda frmula de inverso de Mbius) Sejam f e g funes reais com domnio (0, +) tais que f(t) = g(t) = 0 para todo t < 1. Se x x x g ( x) = f = f k =1 k k =1 k
para todo x ento, para todo x,
EUREKA! N8, 2000

(d )

45

Sociedade Brasileira de Matemtica


x x x f ( x) = (k ) g = (k ) g . k k =1 k k =1 Prova: Basta provar que x f ( x) = (k ) , f kr k =1 r =1 mas, tomando m = kr a ltima soma igual a x (k ) f , m =1 k |m m que pelo lema igual a f(x)

Apesar de no estar relacionada com o resto da nossa discusso, no podemos deixar de mencionar a seguinte conjectura.

Conjectura (Hiptese de Riemann): Se > 1 / 2 ento 1 n lim ( m) = 0. n n m =1


Esta uma das formulaes da famosa hiptese de Riemann, um dos problemas em aberto mais importantes da matemtica. Podemos reenunciar esta conjectura assim: seja f : (0,+) R definida por f (t ) = 0 se t < 1 e

f (t / k ) = 1, se t 1.
Ento, para todo > 1 / 2,
k =1

f (t ) = 0. n t De fato, pela segunda frmula de inverso de Mbius temos lim f (t ) = ( m).


m =1

[1] Carlos Gustavo T. de A. Moreira e Nicolau Saldanha, Primos de Mersenne (e outros primos muito grandes), 22o. Colquio Brasilerio de Matemtica IMPA, 1999. [2] Carlos Gustavo T. de A. Moreira, Divisibilidade, congruncias e aritmtica mdulo n, Eureka! No. 2, pp. 41-52.

EUREKA! N8, 2000

46

Sociedade Brasileira de Matemtica

OLIMPADAS AO REDOR DO MUNDO


A partir deste nmero da EUREKA! apresentaremos esta nova seo cujo objetivo contemplar os leitores que no tm facilidade de acesso a problemas de competies de outras naes com alguns exemplos de problemas, de tais competies.
Naes distintas possuem culturas matemticas distintas, portanto o leitor pode achar alguns problemas extremamente fceis e outros extremamente difceis. Tentaremos apresentar uma grande variedade de problemas principalmente daqueles pases que costumam ter um bom desempenho na Olimpada Internacional de Matemtica. Divirtam-se e mandem suas solues.

Antonio Luiz Santos PROBLEMAS: 1. (Bulgria-1998) Seja f (x ) = x 3 3 x + 1 . Determine o nmero de solues reais distintas da equao f ( f (x )) = 0 . 2. (Repblicas Tcheca e Eslovaca-1998) Determine todos os nmeros reais x tais que x x x x = 88 . 3. (ustria/Polnia-1998) Considere todos os pares ordenados (a, b ) de nmeros
naturais tais que o produto a a b b , escrito na base 10, termina com exatamente 98 zeros. Determine o par (a, b ) para o qual o produto ab o menor possvel. (Reino Unido-1998) Em um tringulo ABC, D o ponto mdio de AB e E um ponto do lado BC tal que BE = 2EC. Sabendo que ADC = BAE determine a medida do ngulo BAC. (Turquia-1998) Seja (a n ) uma seqncia de nmeros reais definida por a1 = t e a n +1 = 4a n (1 a n ) para n 1 . Para quantos valores distintos de t teremos a1998 = 0 ? (Rssia-1998) Um nmero de 10 algarismos dito interessante se todos os seus algarismos so distintos e ele um mltiplo de 11111 . Quantos nmeros interessantes existem? (Rssia-1998) Existem nmeros de 5 algarismos M e N onde todos os algarismos de M sejam pares, todos os algarismos de N sejam mpares, cada um dos algarismos de 0 a 9 ocorrendo exatamente uma vez entre M e N e tais que M divide N?

4.

5.

6. 7.

EUREKA! N8, 2000

47

Sociedade Brasileira de Matemtica

8. (Romnia-1998) O volume de um paraleleppedo 216cm 3 e a sua rea total 216cm 2 . Mostre que o paraleleppedo um cubo. 9. (Irlanda-1998) Um tringulo ABC possui medidas dos lados expressas por nmeros inteiros, A = 2B e C > 90. Determine o valor mnimo do permetro deste tringulo. 10. (Canad-1998) Em um tringulo ABC tem-se que BAC = 40 e ABC = 60. Sejam D e E pontos sobre os lados AC e AB respectivamente tais que CBD = 40 e BCE = 70. Mostre que a reta que contm AF perpendicular que contm BC. 11. (China-1999) A base de uma pirmide um polgono convexo de 9 lados. Pintase cada uma das diagonais da base e cada uma de suas arestas laterais de preto ou de branco (observe que os lados da base no esto coloridos). Mostre que existem trs segmentos coloridos com a mesma cor que formam um tringulo. 12. (Irlanda-1999) Trs nmeros a < b < c esto em progresso aritmtica se c b = b a . Definamos a seqncia (u n ) , n = 0,1,2,3,... da seguinte maneira : u 0 = 0 , u1 = 1 e para cada n 1 , u n +1 o menor inteiro positivo tal que u n +1 > u n e {u 0 , u1 ,..., u n , u n +1 } no possui trs elementos em progresso aritmtica. Determine u100 . 13. (Irlanda-1999) Uma funo f : N N satisfaz s condies : f (ab ) = f (a ) f (b ) se o mximo divisor comum de a e b 1, f ( p + q ) = f ( p ) + f (q ) para todos os nmeros primos p e q. Mostre que f (2 ) = 2, f (3) = 3 e f (1999 ) = 1999 . 14. (Sua-1999) Determine todas as funes f : R\ {0} R satisfazendo

1 f = x para todo x R\ {0} . x 15. (Sua-1999) Dois crculos intersectam-se em dois pontos M e N. Um ponto A qualquer do primeiro crculo, distinto de M e N, unido aos pontos M e N de modo que as retas AM e AN intersectam novamente o segundo crculo nos pontos B e C. Mostre que a tangente ao primeiro crculo em A paralela a BC. 1 f ( x ) + x
16. (Estnia-1999) Mostre que o segmento que une o ortocentro e o baricentro de um tringulo acutngulo ABC paralelo ao lado AB se, e somente se, tgA tgB = 3 .

EUREKA! N8, 2000

48

Sociedade Brasileira de Matemtica

17. (Ucrnia-1999) Mostre que o nmero 9999999 + 1999000 composto. 18. (Armnia-1999) Resolva a equao
1
x
2

(4

1 3x

=1

19. (Litunia-1999) Duas cordas AB e CD de um crculo intersectam-se no ponto K. O ponto A divide o arco CAD em duas partes iguais. Se AK = a, KB = b, determine a medida da corda AD. 20. (Espanha-1999) Mostre que existe uma seqncia de inteiros positivos (a1 , a 2 ,..., a n ,...) tal que a12 + a 22 + ... + a n2 um quadrado perfeito para todo inteiro positivo n. 21. (Estnia-1999) Determine o valor da expresso

1 2 1999 2000 2000 2000 f + f + + f + f + f + + f 2000 2000 2000 2000 1999 1


supondo que f ( x ) =

x2 . 1 + x2 1 1 1 1 so escritos , , ..., , 2 3 1998 1999

22. (Eslovnia-1999) Inicialmente os nmeros 1,


em um quadro negro.

Em cada passo, escolhemos dois destes nmeros, digamos a e b, e os substitumos pelo nmero a + b + ab . Continuamos desta maneira at que reste um nico nmero no quadro negro. possvel que este nmero seja 2000? Justifique sua resposta.

23. (Rssia-1999) A soma dos algarismos de um inteiro positivo n escrito no sistema de numerao decimal igual a 100 e a soma dos algarismos do nmero 44n 800. Determine a soma dos algarismos do nmero 3n . 24. (Rssia-1999) Um crculo que passa pelos vrtices A e B de um tringulo ABC tangente ao lado BC, e o crculo que passa pelos vrtices B e C e tangente ao lado AB intersecta o primeiro crculo no ponto K, K B . Se O o centro
do crculo circunscrito ao tringulo ABC, mostre que BKO =

25. (Espanha-2000) Determine o maior nmero inteiro N que satisfaz as seguintes condies :
EUREKA! N8, 2000

49

Sociedade Brasileira de Matemtica

N (a) possui seus trs algarismos iguais. 3


N (b) igual soma de n nmeros naturais consecutivos a partir de 1. 3 A figura mostra um plano com ruas que 26. (Espanha-2000) delimitam 12 quadras quadradas. Uma A pessoa P caminha de A at B e outra Q caminha de B at A. Ambas partem ao mesmo tempo seguindo caminhos de comprimento mnimo com a mesma velocidade constante. Em cada ponto com duas possveis direes a tomar, ambas possuem a B mesma probabilidade. Determine a probabilidade de que P e Q se cruzem.

27. (Polonia-2000) Prove ou disprove a seguinte afirmativa :


Todo nmero racional positivo pode ser escrito sob a forma b, c e d so inteiros positivos. a 2 + b3 onde a, c5 + d 7

28. (Polonia-2000) Seja I o incentro de um tringulo ABC com AB AC. As retas suportes dos segmentos BI e CI intersectam os lados AC e AB nos pontos D e E respectivamente. Determine todos os ngulos BAC para os quais a igualdade DI = EI pode ser satisfeita. 29. (ustria/Polonia-1999) Determine todos os pares de inteiros positivos ( x, y ) tais
que x x + y = y y x . 30. (Polonia-1998) Determine todos os pares de inteiros positivos ( x, y ) tais que y x = x 50 .

31. (Baltic Way-1999) As bissetrizes dos ngulos A e B do tringulo ABC intersectam os lados BC e CA nos pontos D e E respectivamente. Supondo que AE + BD = AB , determine a medida do ngulo C.

EUREKA! N8, 2000

50

Sociedade Brasileira de Matemtica

SOLUES DE PROBLEMAS PROPOSTOS Publicamos aqui algumas das respostas enviadas por nossos leitores.
36) Na figura abaixo o tringulo DEF tem rea de medida S. Sabendo-se que o tringulo DEF est inscrito num tringulo arbitrrio ABC, mostre que as medidas Si ( i = 1, 2, 3) das reas dos outros tringulos formados satisfazem a
desigualdade S

3 e que a igualdade ocorre se e s se os 1 1 1 + + S1 S 2 S 3


A

pontos DEF so os pontos mdios dos lados do tringulo, ABC.

F S3 B

S2 S S1 D C E

Soluo de Carlos Alberto da Silva Victor (Rio de Janeiro - RJ):


A z F cz S3 B S2 E S S1 C ay D y x bx

AC = b AB = c BC = a

Provar que S

3 S S S idntico a mostrar que + + 3. 1 1 1 S1 S 2 S 3 + + S1 S 2 S 3

EUREKA! N8, 2000

51

Sociedade Brasileira de Matemtica

Se k a rea de ABC, temos ento: S1 = k x y k z (b x) k (c z ) ( a y ) ; S2 = ; S3 = e S = k S1 S 2 S 3 . ab bc ac

Faamos tambm: y x z e m3 = e evidentemente teremos 0 < m1 < 1 ; 0 < m 2 < 1 ; m1 = ; m 2 = b c a 0 < m3 < 1 ; e consequentemente: S1 = k m1 m3 ; S 2 = km2 (1 m1 ) e

1 1 1 S 3 = k (1 m 2 ) (1 m3 ). Seja = (k S1 S 2 S 3 ) S + S + S . 2 3 1
1 m3 (1 m2 ) m 2 m3 1 m 2 1 m 2 + m1 (m 2 m3 ) 1 m3 Portanto = + + + + 3 m3 m 2 m1 (1 m 2 )(1 m3 ) 1 m1

ou seja:

1 m3 1 m1 m 2 m3 1 m 2 m1 + + + + + 3 1 m 2 1 m1 1 m3 m3 m 2 m1


1 2 3

como a soma de qualquer nmero positivo x com o seu inverso sempre maior do que 2 ou igual a 2, valendo a igualdade se e s se x = 1 ( de fato,

1 1 x+ 2= x 0), teremos: 2 + 2 + 2 3 = 3. observe tambm x x que s temos 1 = 2 quando m 2 = m3 ; 2 = 2 quando m1 = 1 m 2 e que 3 = 2 1 quando 1 m3 = 1 m1 ; ou seja m1 = m 2 = m3 = , o que garante que os pontos 2 D, E e F so mdios dos lados correspondentes e como consequncia teremos o mnimo de = 3. Concluso: 3 e a igualdade ocorre para os pontos mdios.

EUREKA! N8, 2000

52

Sociedade Brasileira de Matemtica

37) Cinco quadrados so dispostos conforme ilustra o diagrama abaixo. Mostre que a medida da rea do quadrado S igual a medida da rea do tringulo T.
T

Soluo de Geraldo Perlino (Itapecerica da Serra - SP):

h S2 d e

d a b


c b


c a

ba S1

a b

a b

Prove : S1 = S2 S 1 = c2 e a 2 + b 2 = c 2 (Pitgoras) d dh h = e sen( + ) S 2 = e sen( + ) S2 = 2 2 d S 2 = e ( sen cos + sen cos ) ; d 2 = b 2 + 4a 2 e e 2 = a 2 + 4b 2 . 2

EUREKA! N8, 2000

53

Sociedade Brasileira de Matemtica

a b sen = ; sen = 2b 2 2a 2 d e d = S e 2 de + de 2 cos = 2a ; cos = 2b d e


S2 = a2 + b2 = c2.

38) Os lados e diagonais de um polgono regular de n lados so coloridos em k cores tais que: i) para cada cor a e dois vrtices A e B do polgono, o segmento AB colorido de a ou existe um vrtice C tal que AC e BC so coloridos de a.
ii) os lados de qualquer tringulo com vrtices entre os vrtices do polgono so coloridos usando no mximo 2 cores. Prove que k 2.

Soluo: Suponha que haja pelo menos 3 cores a, b e c. Vamos construir um subconjunto infinito de vrtices de X, o que uma contradio. Fixemos um vrtice Z X. Existe um vrtice A1 tal que A1 Z tem a cor a, e um vrtice B1 tal que a cor de B1 A1 e de B1 Z b. Existe um vrtice C tal que as cores de C1 Z e C1B1 so C. Considerando os tringulos C1 A1 Z e C1 A1 B1, e usando a condio ii), conclumos que a cor de C1A1 tem que ser C. Vamos construir por induo vrtices An , Bn , C n para cada inteiro positivo n, todos distintos tais que, para todo i < n, as cores de ZAn , Ai An , Bi An e C i An so a, as cores de ZB n , Ai Bn , Bi Bn , C i Bn e An Bn so b e as cores de ZC n , Ai C n , Bi C n , C i C n , An C n e Bn C n so c. Suponhamos contrudos A j , B j , C j para 1 j n.
Por i), existe An +1 tal que as cores de An +1 Bn e An +1C n so a. Considerando os tringulos An +1 Bn P e An +1C n P, (e usando a condio ii), conclumos que a cor de An +1 Bn P tem que ser a, para cada ponto P criado anteriormente. Do mesmo modo, existe Bn +1 tal que as cores de Bn +1 An +1 e Bn +1C n so b. Considerando os tringulos Bn +1 An +1 P e Bn +1C n P, para cada ponto P criado anteriormente, conclumos que a cor de Bn +1 P tem que ser b. Por fim, existe C n +1 tal que as cores de C n +1 Bn +1 e C n +1 An +1 so c, e, considerando os tringulos C n +1 Bn +1 P e C n +1 An +1 P, para cada ponto P criado anteriormente, conclumos que a cor de
EUREKA! N8, 2000

54

Sociedade Brasileira de Matemtica

C n +1 P tem que ser c. fcil ver que os pontos criados so todos distintos. Por exemplo: como a cor de An Z a, temos An Bk e An C k para todo k. Como a cor de An Bn 1 a, An Z , e como a cor de An C n 1 a, An A j , para todo j < n.

39) Sejam x, y e z os ngulos de um tringulo de lados opostos a, b e c 1 1 1 1 1 1 a b c respectivamente. Prove que a y + z + b z + x + c + 2 + + . x y x y z Soluo:
x

Suponha sem parda de generalidade que a b c. 1 1 1 Teremos por tanto x y z , logo . x y z

1 1 1 1 1 1 Temos ento (a b) y x 0, (b c) z y 0 e (c a) x z 0. Somando essas 3 desigualdades obtemos a desigualdade do enunciado.


40) a) Calcular a soma dos divisores positivos de um nmero natural em termos de sua fatorao prima. b) Dizemos que n 1 abundante se a soma de seus divisores maior que 2n. Prove que se n abundante ento kn abundante para todo inteiro k 1. c) Prove que existe n0 N tal que todo inteiro n n0 pode ser escrito como soma de dois nmeros abundantes.

EUREKA! N8, 2000

55

Sociedade Brasileira de Matemtica

Soluo de Marcio Afonso Assad Cohen (Rio de Janeiro - RJ): 3 1 n 2 a) Seja k = p1 p 2 p 3 ... p n
a2 an com 0 ai i , e reciprocamente. Todo divisor de k da forma p1a1 p 2 ... p n Portanto a soma de todos os divisores :

S (k ) =
(pois

a1 = 0 a2 = 0

... ( p1a1 p 2a2 ... p nan ) =


an = 0

a1 = 0

...

n 1

an 1 = 0

an n 1 ( p1a1 p 2a2 ... p na 1 ) pn = an =0

p1a1

...

an 1 pn 1

constante para o somatrio em an).

n +1 p 1 an 1 a1 a2 n (soma da P.G.) ... p p ... p 1 2 n 1 a1 = 0 an 1 pn 1 n 1 n +1 n +1 p p 1 1 1 an 1 n n ... p1a1 ... p n (pois constante em relao s . 1 p 1 a =0 pn 1 an 1 n 1 variveis a1 , a2 ,..., an 1 ). Procedendo de maneira anloga, agora para o termo p n 1 , e assim por diante obtemos: 2 +1 p 1 +1 1 p 2 pn 1 1 ... n , que o que queramos. S (k ) = 1 p 1 p 1 p 1 n 1 2 S (k ) 1 n para k = p1 b) Vamos analisar a razo ... p n k Temos que 1 +1 1 2 n +1 n p 1 (1 + p1 + p1 (1 + pn + ... + p + ... + p1 ) p1 1 S (k ) 1 n n ) ... = = ... = 1 p 1 n k k p p1 n n p1 1

n 1

1 1 1 1 1 + ... + . = 1 + + 2 + ... + ... 1 + 1 p1 p 2 pn pn n p1 k2 Agora se multiplicarmos k por m = q1k1 q 2 ... q sk s , duas coisas podem acontecer: i) Para cada primo p i que aparece na fatorao de k e de m , o fator S (kn) 1 1 referente a ele no produtrio aumenta, pois, +1 > 0,..., + r > 0 i kn pi pi i
1 1 1 1 e portanto, 1 + + ... + < 1 + + ... + r i pi pi pi pi i .

EUREKA! N8, 2000

56

Sociedade Brasileira de Matemtica

ii)

Para cada primo q j que s aparece na fatorao de k, vemos que ao


S (km) , aparecer um novo fator km
1 + 1 + ... + 1 k qj qjj > 1, de modo que

calcularmos

S (km) S (k ) . > k km

Em qualquer caso portanto, vale abundante


S (k ) S (km) >2 > 2. k km

S (km) S (k ) , m N * . Em particular, k km k

c)
m 2.

Note que na letra b), vale a desigualdade estrita

S (km) S (k ) para todo > km k

S ( 6) 1 1 3 4 = 1 + 1 + = = 2, 6 2 3 2 3 vemos que todo mltiplo de 6, maior do que 6 abundante. (pois S (n) S (6t ) S (6) = > = 2) n = 6t ; t > 1 6t 6 n Logo, se para um natural N, existe N1 abundante tal que N N1 mod 6, e N N1 > 6, ento N N1 = 6t ; t N N = N1 + 6t;t > 1 N pode ser escrito como soma de dois nmeros abundantes.

Em particular, como

Nosso problema se resume ento a descobrir 6 nmeros abundantes, dois a dois distintos mdulo 6: mas para isso suficiente achar N abundante tal que N 1 mod 6 (pois nesse caso 2 N 2 mod 6 e 2N abundante pela letra b); e analogamente, 3 N 3 mod 6,4 N 4;5 N 5 e 6 N 0 mod 6 so todos abundantes e distintos mdulo 6). Tambm seria suficiente achar algum T abundante tal que T 5 mod 6, pois nesse caso, 5T congruente a 1 mod 6 e recamos no caso anterior. Note agora, que todo nmero da forma 1 N 5 7 2 11 3 . (1) 1 1 2 (1) 3 ... mod 6 N 1 mod 6 ou N 1 5 mod 6. Obs: todo primo p > 3 congruente a 1 ou 1 mod 6, pois do contrrio teramos: p 2,4,0(mod 6) p par ou p 3(mod 6) p mltiplo de 3.

EUREKA! N8, 2000

57

Sociedade Brasileira de Matemtica

O problema ento fica sendo o de encontrar um nmero da forma 5 N 5 7 7 ... p p abundante. Isso possvel mesmo se nos restringirmos apenas a nmeros em que 5 = 7 = ... = p = 1. Basta ver que nesse caso,
S(N ) 1 1 1 1 1 = 1 + 1 + 1 + 1 + ... 1+ N p 5 7 11 13 S ( N ) 6 8 12 14 p +1 = ... . N 5 7 11 13 p 6 8 12 14 18 20 24 30 32 Em particular, S (5 7 1113 17 19 23 29 31) = = 5 7 11 13 17 19 23 29 31 6 8 12 14 18 20 24 30 32 145152 460800 = > 1,7059 1,1729 > 2,0008 > 2. = 5 7 11 13 17 19 23 29 31 85085 392863

Logo, N = 5 7 11 13 17 19 23 29 31 abundante. (temos N (1) 1 (1) 1 (1) 1 (1) (1) 1 5 mod 6) Tomando portanto N 0 = 5 N , temos N 0 ,2 N 0 ,3N 0 ,4 N 0 ,5 N 0 ,6 N 0 so abundantes distintos mdulo 6. Fazendo ento n0 = 6 N 0 + 6, vemos pelas observaes anteriores que n > n0 tem-se que n pode ser escrito como soma de dois nmeros abundantes!.

PROBLEMA "CUTICO" (Publicado na Eureka! No. 5): Prove que para qualquer conjunto de inteiros positivos A e para todo inteiro positivo k existe um conjunto infinito de nmeros primos S tal que o produto de k elementos distintos de S est sempre em A ou o produto de k elementos distintos de S nunca pertence a A. Soluo de Daniel Massaki Yamamoto (So Paulo - SP): Considere o Conjunto P formado por todos os primos. Para todo subconjunto de P com k elementos, pinte-o de azul se o produto destes pertencer a A e de vermelho caso contrrio. Pelo Teorema de Ramsey Infinito, existe um subconjunto infinito de P tal que todos os seus subconjuntos de k elementos so da mesma cor, ou seja os produtos de seus elementos sempre pertencem ou nunca a A. Chamando-o de S, acabamos o problema.

EUREKA! N8, 2000

58

Sociedade Brasileira de Matemtica

Agradecemos tambm o envio das solues e a colaborao de:


Alex Correa Abreu Carlos A. Gomes Diego Alvarez Arajo Correia Estillac Lins Maciel Borges Filho Fabrcio Siqueira Benevides Fernando Carvalho Ramos Geraldo Perlino Jnior Jos Clovis Ado Macedo Jos Guilherme Moreira Pinto Luciano Marinho Filho Luiz Fernando Athayde Jnior Marcelo Rufino de Oliveira Nijair Arajo Pinto Osvaldo Mello Sponquiado Paulo de Sousa Sobrinho (Niteroi - RJ) (Natal - RN) (Fortaleza - CE) (Belm - PA) (Fortaleza - CE) (Santa Maria - RS) (So Paulo - SP) (Mato - SP) (Juiz de Fora - MG) (Recife - PE) (Rio de Janeiro - RJ) (Belm - PA) (Fortaleza - CE) (Olmpia - SP) (Natal - RN)

Errata: O problema No. 4 (Olimpada Romnia 92) publicado Na Eureka! No. 6, pg 37, deveria dizer: Sejam p, q C, q 0. Se as razes da equao x 2 + px + q = 0 tm
o mesmo mdulo, mostre que

p2 um nmero real. q O problema No. 8 (Olimpada Hungria 1899) publicado Na Eureka! No. 6, pg 38, deveria dizer: A0 , A1 , A2 , A3 , A4 dividem a circunferncia unitria em cinco partes
iguais. Prove que ( A0 A1 A2 A4 ) 2 = 5.

Voc sabia
Que foram recentemente batidos os recordes de maior par de primos gmeos (p, p +2) conhecido? So eles 4648619711505 260000 1, descobertos este ano por Wassing, Jarai e Indlekofer, e tm 18075 dgitos cada. Tambm tem 18075 dgitos o maior primo conhecido p tal que 2p + 1 tambm primo (tais primos so conhecidos como primos de Sophie Germain). o nmero 3714089895285 260000 1, descoberto pelos mesmos Wassing, Jarai e Indlekofer. Este o maior primo conhecido p tal que o nmero de Mersenne 2p 1 composto (de fato divisvel por 2p +1; veja o problema 43 proposto na pgina 60).

EUREKA! N8, 2000

59

Sociedade Brasileira de Matemtica

PROBLEMAS PROPOSTOS
Convidamos o leitor a enviar solues dos problemas propostos e sugestes de novos problemas para os prximos nmeros.
41) Se a e b so nmeros reais positivos, ento a b + b a > 1. 42) Suponha que a, b e c so as medidas dos lados de um tringulo ABC, com semi-permetro p e rea S, verifique que 1 1 1 3 p + + a b c 2 s e mais ainda: verifique que a igualdade acima ocorre apenas se o tringulo for equiltero. 43) Prove que se p um primo da forma 4k + 3, ento 2p + 1 tambm primo se e somente se 2p + 1 divide 2p 1. 44) O produto de dois inteiros positivos consecutivos pode ser igual ao produto de dois inteiros positivos consecutivos pares? 45) Existe uma seqncia infinita de: a) Nmeros reais b) Nmeros inteiros Tais que a soma de quaisquer dez termos consecutivos positiva, enquanto que para todo n a soma dos primeiros 10n + 1 termos consecutivos negativa?
46) (Baltic Way, 1997)

i) Prove a existncia de dois conjuntos infinitos A e B, no necessariamente disjuntos, de inteiros no negativos tais que cada inteiro no negativo pode ser representado de uma nica forma como a + b, com a A e b B. ii) Prove que em cada tal par (A, B), ou A ou B contm apenas mltiplos de algum inteiro k > 1.
Problemas 41 e 42 propostos por Carlos Alexandre Gomes da Silva (Natal - RN), problemas 44 e 45 obtidos do 21o. Torneio das Cidades - Primaveira 2000.

EUREKA! N8, 2000

60

Sociedade Brasileira de Matemtica

AGENDA

OLMPICA

XI OLIMPADA DO CONE SUL 14 a 19 de abril de 2000 Montevidu Uruguai

VI OLIMPADA DE MAIO 13 de maio de 2000

XXII OLIMPADA BRASILEIRA DE MATEMTICA - 2000 Primeira Fase Sbado, 10 de junho Segunda Fase Sbado, 02 de setembro Terceira Fase Sbado, 21 de outubro (nveis 1,2 e 3) Domingo, 22 de outubro (nvel 3 - segundo dia).

XLI OLIMPADA INTERNACIONAL DE MATEMTICA 13 a 25 de julho de 2000 Taejon, Coria do Sul.

XV OLIMPADA IBEROAMERICANA DE MATEMTICA 16 a 24 de setembro de 2000 Caracas, Venezuela

III OLIMPADA IBEROAMERICANA DE MATEMTICA UNIVERSITRIA 7 de outubro de 2000

EUREKA! N8, 2000

61

Sociedade Brasileira de Matemtica

COORDENADORES REGIONAIS
Amarisio da Silva Arajo (UFV) Viosa - MG Alberto Hassen Raad (UFJF) Juiz de Fora - MG Angela Camargo (Centro de Educ.de Adultos - CEA) Blumenau - SC Benedito T. Vasconcelos Freire (UFRN) Natal - RN Claudio Arconcher (Col. Leonardo da Vinci) Jundia - SP Claus Haetinger (UNIVATES) Lajeado - RS Crescncio das Neves (UFAM) Manaus-AM lio Mega (Col. ETAPA) So Paulo - SP Enzo Marcom Takara (Col. Singular) Santo Andr - SP Florncio F. Guimares Filho (UFES) Vitria - ES Francisco Dutenhefner (UFMG) Belo Horizonte - MG Gisele de A. Prateado Gusmo (UFGO) Goinia - GO Ivanilde H. Fernandes Saad (U. Catlica Dom Bosco) Campo Grande - MS Jacqueline F. Rojas Arancibia (UFPB) Joo Pessoa - PB Joo Bencio de Melo Neto (UFPI) Teresina - PI Joo F. Melo Libonati (Grupo Educ. IDEAL) Belm - PA Irene Nakaoka (UEM) Maring - PR Jos Carlos Pinto Leivas (UFRG) Rio Grande - RS Jos Cloves Saraiva (UFMA) So Luis - MA Jos Gaspar Ruas Filho (ICMC-USP) So Carlos - SP Jos Luis Rosas Pinho (UFSC) Florianpolis - SC Jos Paulo Carneiro (Univ. Santa rsula) Rio de Janeiro - RJ Jos Vieira Alves (UFPB) Campina Grande - PB Leonardo Matteo D'orio (Sistema Titular de Ensino)Belm - PA Licio Hernandes Bezerra (UFSC) Florianpolis - SC Luzinalva M. de Amorim (UFBA) Salvador - BA Marcondes Cavalcante Frana (UF Cear) Fortaleza - CE Pablo Rodrigo Ganassim (L. Albert Einstein) Piracicaba - SP Paulo H. Cruz Neiva de L. Jr. (Esc. Tec.Everardo Passos) SJ dos Campos - SP Reinaldo Gen Ichiro Arakaki (INPE) SJ dos Campos - SP Ricardo Amorim (Centro Educ. Logos) Nova Iguau - RJ Roberto Vizeu Barros (Colgio ACAE) Volta Redonda - RJ Sergio Claudio Ramos (IM-UFRGS) Porto Alegre - RS Seme Gebara Neto (UFMG) Belo Horizonte - MG Silvio de Barros Melo (UFPE) Recife - PE Tadeu Ferreira Gomes (U. do Estado da Bahia) Juazeiro - BA Toms Menndez Rodrigues (U. Federal de Rondonia) Porto Velho - RO Valdenberg Arajo da Silva (U. Federal de Sergipe) So Cristovo - SE Wagner Pereira Lopes (Esc. Tec. Fed. de Gois) Jata - GO Waldemar M. Canalli (P.M. S. Joo de Meriti) S. Joo de Meriti - RJ

EUREKA! N8, 2000

62

CONTEDO

AOS LEITORES XV OLIMPADA IBEROAMERICANA DE MATEMTICA Problemas e Solues XLI OLIMPADA INTERNACIONAL DE MATEMTICA Problemas e Solues

2 3 16

ARTIGOS
BRAHMAGUPTA PARA TODOS Jos Cloves Verde Saraiva EQUAES DE RECORRNCIA Hctor Soza Pollman EQUAES FUNCIONAIS Eduardo Tengan OLIMPADAS AO REDOR DO MUNDO SOLUES DE PROBLEMAS PROPOSTOS PROBLEMAS PROPOSTOS COORDENADORES REGIONAIS RECADASTRAMENTO PARA COLGIOS 2001 28 33 41 45 55 59 60 61

Sociedade Brasileira de Matemtica

AOS LEITORES
Pases de grande tradio nas olimpadas de Matemtica tm o seu excelente rendimento em muito baseado na existncia de revistas de divulgao para estudantes pr-universitrios. Apenas citando alguns exemplos, a hngara KMaL, a russa (agora tambm americana) Kvant (Quantum), a romena Gazeta Matematica e a canadense (lida por alguns, poucos, brasileiros) Crux Mathematicorum desenvolvem em seus pases o ambiente ideal para o aparecimento e desenvolvimento de jovens talentosos. Ns queremos que a Eureka! tenha o mesmo sucesso que essas revistas. Para isso estamos tentando trazer artigos cada vez mais interessantes e elucidativos, e problemas cada vez mais bonitos e desafiantes para todos os leitores da Eureka! (inclusive os dos Nveis 1 e 2), porm tal sucesso no depende exclusivamente de ns. fundamental a participao (muito) ativa de todos os leitores. Ficamos bastante contentes com a repercusso da seo "Olimpadas ao Redor do Mundo", com vrios leitores enviando resolues dos problemas propostos. Mas queremos que todas as outras sees e artigos tenham a mesma repercusso. Por exemplo, vrios artigos trazem problemas propostos porm raramente recebemos resoluo desses problemas. Ficaremos contentes em receb-las e poderemos saber se o pblico-alvo os entendeu. Assim, faa-nos saber todo o estudo(!) e a diverso(!!) que cada uma das Eureka! proporcionou (no se esquea dos nmeros antigos). INSTRUES PARA AUTORES Sero publicados na revistas Eureka! artigos relevantes na preparao dos estudantes para a Olimpada Brasileira de Matemtica em seus diversos nveis e para vrias olimpadas de carter internacional das quais o Brasil participa. Como para a grande maioria dos tpicos e tcnicas explorados nas olimpadas no existem publicaes expositrias adequadas em lngua portuguesa, nosso objetivo inicial abord-los todos em artigos auto-suficientes. Assim, daremos preferncia queles que tratem de assuntos ainda no explorados nos nmeros anteriores da Eureka!. Como a deficincia em artigos adequados para estudantes do Ensino Fundamental (Nveis 1 e 2 da OBM) ainda mais grave, estes tero primazia na sua publicao. Vale a pena observar que, quando um tema importante para os estudantes de diversos nveis, ele deve aparecer em artigos adequados para cada um desses nveis, separadamente. recomendvel que os artigos tragam alguns problemas resolvidos detalhadamente e referncias que o complementem ou aprofundem. Os Editores.
EUREKA! N9, 2000

Sociedade Brasileira de Matemtica

XV OLIMPADA IBEROAMERICANA DE MATEMTICA


16 a 24 de setembro, Caracas - Venezuela
A XV Olimpada Iberoamericana de Matemtica foi realizada em Caracas, Venezuela no perodo de 16 a 24 de setembro de 2000. A equipe brasileira foi liderada pelos professores Ralph Costa Teixeira (Rio de Janeiro RJ) e Eduardo Tengan (So Paulo - SP). Nesta oportunidade a equipe brasileira obteve a maior pontuao entre os pases participantes alm da Copa Puerto Rico, prmio entregue ao pas com maior progresso nos ltimos dois anos na competio. RESULTADOS DA EQUIPE BRASILEIRA BRA1 Daniel Nobuo Uno BRA2 Daniel Massaki Yamamoto BRA3 Fabrcio Siqueira Benevides BRA4 Humberto Silva Naves Prata Ouro Ouro Ouro

PRIMEIRO DIA
Constri-se um polgono regular de n lados ( n 3 ) e enumeram-se os vrtices de 1 a n. Traam-se todas as diagonais do polgono. Demonstrar que se n mpar, possvel associar a cada lado e a cada diagonal um nmero inteiro de 1 a n, tal que se verifiquem simultaneamente as seguintes condies: 1. O nmero associado a cada lado ou diagonal seja diferente dos nmeros dos seus vrtices. 2. Para cada vrtice, todos os lados e diagonais que nele se intersectem tenham nmeros diferentes.
SOLUO DE HUMBERTO SILVA NAVES (SO PAULO - SP) PROBLEMA 1

Existe um jeito simples de pintar os lados e as diagonais do polgono regular. Escolhe-se um ponto do polgono regular com o nmero "i" associado este ponto, que chamaremos de Pi. Seja O o centro da circunferncia circunscrita ao polgono regular.

EUREKA! N9, 2000

Sociedade Brasileira de Matemtica

i Pi x1 i x1 '

O x2

x2 '

Fig. 1

Sabemos que a reta Pi O eixo de simetria dos pontos do polgono regular e nenhum ponto do polgono est sobre a reta Pi O (pois n mpar). Digamos esses pares de pontos simtricos e numeramos com o nmero "i", ou seja pegamos um outro ponto qualquer (diferente de Pi) do polgono regular, ligamos com o seu respectivo simtrico e numeramos com o nmero "i" (fazemos isso para todos os outros pontos). Vide fig. 1. Este um exemplo do processo para o ponto Pi. Repetimos esse processo para todos os pontos do polgono regular. Desta forma pintamos todos os lados e todas as diagonais, pois: De uma forma geral (e mais simples): Se quisermos saber o nmero de um segmento Pi Pj do polgono regular, basta traarmos a mediatriz desse segmento, que vai certamente encontrar um outro vrtice X do polgono regular (pois n mpar) e o segmento Pi Pj vai ter o mesmo nmero de X. Esta forma de numerao claramente satisfaz as condies do enunciado pois: 1) No existe nenhum segmento com o mesmo nmero de um de seus vrtices, pois a mediatriz desse segmento no passa pelos vrtices do segmento. Para cada vrtice, todos os lados e todas as diagonais que incidem neste vrtice tem nmeros diferentes, pois se existissem dois segmentos de mesmo nmero, teramos:

2)

EUREKA! N9, 2000

Sociedade Brasileira de Matemtica

Pj

O ponto Pw (que deve estar na mediatriz de Pi Pj e na mediatriz de


w Pw= O w Pi

Pi Pt ) o prprio O, um absurdo.
Logo vale a propriedade 2 e isso acaba o problema.

Pt

PROBLEMA 2

Sejam S1 e S2 duas circunferncias de centros O1 e O2, respectivamente, secantes em M e N. A reta t a tangente comum a S1 e S2, mais prxima de M. Os pontos A e B so os pontos de tangncia de t com S1 e S2, respectivamente, C o ponto diametralmente oposto a B e D o ponto de interseo da reta O1O2 com a reta perpendicular reta AM que passa por B. Demonstrar que M, D e C so colineares.
SOLUO DE DANIEL MASSAKI YAMAMOTO (SO PAULO - SP)

C = 90 BC dimetro BM D = BM C = 90 Para provar [ MDC ], basta provar que BM


B

A 1

90 2

O1 D O2

EUREKA! N9, 2000

Sociedade Brasileira de Matemtica

Geometria Analtica: Dados: M = (0, 1) A = (m, 0) O1 = (m, y1) B = (n, 0) O2 = (n, y2) Quero determinar D:

C (n, 22)

N O2 2 M n B = (n, 0)

D O1 1

1 A m0 = (m, 0)

1 ( x m) m BD : AM AM : y =
BD : y = m( x n) x = O1O2 : y y 2 = y +n m

y 2 y1 ( x n) m+n BD O1O2 = {D}

y y2 =

y 2 y1 y m+ n m y 2 y1 y 1 + ( m + n ) m = y2 m( m + n) yD = y2 m( m + n) + y y 2 1 y 2 ( m + n) +n m( m + n) + y 2 y 1

x0 =

y 2 m(m + n) m( m + n) y 2 + y1 m( m + n) + y 2 y1 yD 1 = coef. angular de DM : xD mn(m + n) + n( y 2 y1 ) y 2 (m + n) m(m + n) y 2 y1


EUREKA! N9, 2000

Sociedade Brasileira de Matemtica

n2 + 1 n2 + 1 m2 + 1 m(m + n ) m 2 mn 2 2 + 2 = 2 2 2 n m n + 1 mn(m + n ) + n 2 (m + n) 2

n2 2 m 2 mn n2 m2 2 + + m + mn + m mn 2 2 2 2 2 = = 2 n( n m) n 1 (m + n ) mn + 2 2 2 n [mn(m + n) (m + n)] n(m + n)(mn 1) 2 = = =n ( m + n)(mn 1) mn 1 (m + n) 2


coef. ang. de BM :
1 n = 1 , logo n

m n n (m 2 + mn) 2 2 mn 1 ( m + n) 2

n 2

1 n

BM DM .
PROBLEMA 3

Encontrar todas as solues da equao

(x + 1) y x z = 1
para x, y, z inteiros maiores que 1. Considerando a equao mdulo x + 1 , obtemos ( 1) z 1(mod x + 1), o que implica que z mpar. Logo y (1) (x + 1) x z = 1 ( x + 1) y = x z + 1 ( x + 1) y = ( x + 1)(x z 1 x z 2 + ... x + 1)
y 1

SOLUO

(2) (x + 1) = x z 1 x z 2 + ... x + 1 e, ento, x par pois, caso contrrio, os dois lados de (2) seriam de paridade oposta.

EUREKA! N9, 2000

Sociedade Brasileira de Matemtica

Analogamente, escrevendo (1) na forma (x + 1) + ( x + 1) y 2 + ... + ( x + 1) + 1 = x z 1 , ns vemos que y par tambm. Sejam, portanto, x = 2x1 e y = 2 y1 Ento por (1),
y 1

(3) ((x + 1) y1 1)((x + 1) y1 + 1) = x z . Temos ainda que x | ( x + 1) y1 1 e, como x par, mdc ((x + 1) y1 1, ( x + 1) y + 1) = 2. Assim de (3), ( x + 1) y1 1 = 2x1z (4). ( x + 1) y1 + 1 = 2 z 1 Conseqentemente, 2 z 1 > 2x1z , ou seja, x1 = 1 x = 2 e, de (4), y = 2 e z = 3. P.S. gratificante saber que tal equao estudada por grandes matemticos desse sculo como W.J. Leveque e A. Schinzel, pode ser resolvida por brilhantes estudantes pr-universitrios em um tempo to limitado quando o de uma prova de olimpada.

SEGUNDO DIA
PROBLEMA 4

De uma progresso aritmtica infinita 1, a1 , a 2 , de nmeros reais, eliminam-se termos, obtendo-se uma progresso geomtrica infinita 1, a n1 , a n2 , de razo q. Determinar os possveis valores de q.
SOLUO DE DANIEL NOBUO UNO (SO PAULO - SP)

PG = 1, 1 + m1 r ;1 + m 2 r ;1 + m3 r... com mi Z + ; apenas 1 + mi +1 r 1 + m1 r 1 + m 2 r m0 = 0 q = = = ... = . 1 1 + m1 r 1 + mi r (1 + m 2 r ) 1 = (1 + m1 r ) 2 1 + m 2 r = 1 + m12 r 2 + 2m1 r. Supondo

m2 2 m m1 2 2 r 0; m 2 = m1 r + 2m1 m 2 = m1 ( m1 r + 2) = m1 r + 2 =r m1 m1

EUREKA! N9, 2000

Sociedade Brasileira de Matemtica

Como m1 ; m2 Z + , ento r racional; Seja r = primos entre si. Da: q = 1 + m1 r .

a b

; onde a; b Z ; b 0, a e b

m1 r Q q Q.
Seja q =

; c; d Z; d 0 c e d primos entre si. d Da teremos a ki a + b PA : 1 + k i = b b

cj c = j PG: d d Anlise do denominador:


PA: PG:

ki a + b b
cj d
j

; k i a + b Z o mximo do denominador b.

como mdc(c; d) = 1 o denominador d j .


j

Podemos pegar um j suficientemente grande tal que d Logo, a PG no pode ser infinita para d > 1.

>b

Logo; como d Z d = 1 ou d = 1. Se d = 1 q = c razo inteira, e para isso, s pegar r = 1 e tirar os termos desnecessrios. A PA ser sempre crescente ou sempre decrescente ( p r 0) se d = 1.
PROBLEMA 5

Dois jogadores, alternadamente, retiram pedras de um conjunto de 2000 pedras, de acordo com as seguintes regras:

1. Em cada jogada pode-se retirar 1, 2, 3, 4 ou 5 pedras. 2. Em cada jogada, um jogador no pode retirar o mesmo nmero de pedras que o seu adversrio retirou na jogada anterior.
EUREKA! N9, 2000

Sociedade Brasileira de Matemtica

O jogador que, na sua vez, no puder jogar de maneira vlida, perde. Determinar que jogador tem uma estratgia que lhe garanta a vitria e encontrar essa estratgia.
SOLUO DE DANIEL MASSAKI YAMAMOTO (SO PAULO - SP)
n\ m 0 1 2 3 4 5 6 7 8 9 10 11 12 13 14 15 16 17 18 19 20 21 22 23 1 0 0 1 1 1 1 1 0 1 1 1 1 1 0 0 1 1 1 1 1 0 0 1 1 2 0 1 1 1 1 1 1 0 1 1 1 1 1 0 1 1 1 1 1 1 0 1 1 1 3 0 1 1 0 1 1 0 0 1 1 1 1 1 0 1 1 0 1 1 0 0 1 1 1 4 0 1 1 1 0 1 1 0 1 1 1 0 1 0 1 1 1 1 1 1 0 1 1 1 5 0 1 1 1 1 0 1 0 1 1 1 1 0 0 1 1 1 1 0 1 0 1 1 1 n\ m 24 25 26 27 28 29 30 31 32 33 34 35 36 37 38 39 40 41 42 43 44 45 46 1 1 1 0 0 1 1 1 1 1 0 0 1 1 1 1 0 0 1 1 1 1 1 0 2 1 1 0 1 1 1 1 1 1 0 1 1 1 1 1 0 1 1 1 1 1 1 0 3 1 1 0 1 1 0 1 1 0 0 1 1 1 1 1 0 1 1 0 1 1 0 0 4 0 1 0 1 1 1 1 1 1 0 1 1 1 0 1 0 1 1 1 1 1 1 0 5 1 0 0 1 1 1 1 0 1 0 1 1 1 1 0 0 1 1 1 1 0 1 0

0 se quem vai jogar agora perde (com ambos jogando o melhor possvel). G ( n, m ) = 1 se ganha. n = atual m = movimento anterior

G (1, 1) = 0 A frmula recursiva


0 se para todo 1 i 5, n i, i m, G ( n i, i ) = 1 (isto , se o outro ganha G (n, m) = para qualquer movimento) 1 caso contrrio.
EUREKA! N9, 2000

10

Sociedade Brasileira de Matemtica

Para 1 n 5, G (n, m) = 1 se m n Podemos definir G (0, m) = 0 Visualizando: Como preencher a linha n: veja os quadrados com (*) (*) (*) (*) (*) (*) n Se foram todos 1's, a linha n ter s zeros. Se tiver dois ou mais zeros, a linha n ter todos 1's. Se for zero, todos sero 1's, a menos da que est na mesma coluna do zero. Sempre perde: 7, 13, 20, 26, 33, 39, 46 Parece que para n = 7 ou 13 (md 13) sempre perde (G(n, m) = 0) Basta ver que h um bloco 5 5 da tabela que se repete, (tirando as 6 primeiras linhas). Agora vamos provar a frmula recursiva. Primeiras 5 linhas: - G (n, m) = 1 se m n, pois ele pode tirar n pedras e ganhar. - G (1, 1) = 0, pois no pode jogar. - G (2, 2) = 1, pois tirando uma pedra, forar o outro a ficar com G (1, 1),... - G (i, i) = 0, i = 3, 4 ou 5, pois no importa como jogue o outro pode ganhar. Agora, a frmula nos outros casos: Tirando i pedras, se voc est com G(n, m), o adversrio vai ficar com G(n i, i). Se algum deles for 0, o adversrio vai perder e voc ganha. Se todos forem 1, o adversrio sempre ganha. Note que voc no pode tirar m pedras, ento no pode deixar o adversrio com G(n m, m).
EUREKA! N9, 2000

11

Sociedade Brasileira de Matemtica

Na tabela, o bloco que se repete 13 n 17, igual a 26 n 30.


Ento o primeiro jogador ganha, pois se ele tirar 4 pedras na primeira jogada, vai deixar o outro com 1996 pedras. 1996 7(mod 13) o segundo perde.
PROBLEMA 6

Um hexgono convexo bonito se tem quatro diagonais de comprimento 1 cujos extremos contm todos os vrtices do hexgono. (a) Para cada nmero k maior que 0 e menor ou igual a 1, encontrar um hexgono bonito de rea k. 3 (b) Demonstrar que a rea de um hexgono bonito qualquer menor que . 2
SOLUO

Consideremos todos os hexgonos bonitos ABCDEF com lados opostos paralelos tais que os quatro segmentos dados de comprimento 1 so AC , CE , BF e FD. Seja = BFD = ACE (figura (a)).
A F E (a) D B C A' F E' E (b) A B C' D C

Translademos os segmentos AC e CE na direo de AB para obter A' C ' e C ' E ' respectivamente de forma que A' , F , E ' sejam colineares do mesmo modo que B, C ' , D. A rea do hexgono ABCDEF igual a rea do retngulo A'BDE' que por sua vez igual a sen (figura (b)). Dado que quaisquer k (0,1] igual a sen para algum com 0 < 90 ento temos um hexgono bonito que cumpre o pedido. b) Para demonstrar o resultado pedido, enunciamos os seguintes lemas sem demonstrao:
EUREKA! N9, 2000

12

Sociedade Brasileira de Matemtica

1- A soma dos comprimentos de um par de lados opostos de um quadriltero convexo menor que a soma dos comprimentos das diagonais. 2- A rea de um quadriltero convexo menor ou igual que a metade do produto das suas diagonais. Utilizaremos por simplicidade a seguinte terminologa: Sabemos que no hexgono h quatro diagonais de comprimento 1 cujos extremos contm todos os vrtices do mesmo. Chamaremos diagonais principais as diagonais em questo. Definamos o grau de um vrtice de um hexgono bonito como o nmero de diagonais principais que passam por ele. Alm disso denotaremos por diagonal que divida ao hexgono nos quadrilteros. Dado que a soma dos graus dos vrtices 8 e o grau de cada um pelo menos 1, temos dois casos: a) Um vrtice tem grau 3 e os restantes tm grau 1. b) Dois vrtices tm grau 2 e os restantes tm grau 1. No primeiro caso, dado que h exatamente trs diagonais que saem de um mesmo vrtice num hexgono, s h um possvel tipo de configurao (salvo "issomorfismo" de grafos): 1 Pelo lema 2 a rea do hexgono menor ou igual que ( AB + AC ) e isto 2 1 menor que (2 AX + BC ) pela desigualdade triangular. Como AX 1, BC = 1, a 2 3 rea menor que . 2
A M X O N

Para o segundo caso, sejam P e Q os vrtices de grau mximo. Consideremos as possibilidades: c) Os vrtices P, Q esto conectados por uma diagonal principal. Nesse caso se a diagonal que os conecta no central, as outras diagonais principais que saem destes dois vrtices devem estar num mesmo lado de PQ . Alm disso, alguma dessas diagonais deve ser central, pois caso contrrio, chegariam a um mesmo ponto e nesse caso se formaria um tringulo equiltero com diagonais principais como lados e sabemos que isso no pode ocorrer pois haveria trs vrtices de
EUREKA! N9, 2000

13

Sociedade Brasileira de Matemtica

grau 2. Portanto, dependendo se ambas diagonais so centrais ou s uma , se apresentam as seguintes configuraes: No desenho direita, AB 1 pelo lema 1. Logo a rea do hexgono menor ou igual que 1 1 ( AB CQ + PQ AD) < (CQ + AD) 2 2 e isto menor que 1 pelo lema 1.
C A B

C A B

P D

No desenho esquerda, AB < 2 pelo lema 1. Logo, a rea do hexgono menor ou igual que
Q

P D

1 1 3 ( AB PC + PQ BD) < ( 2 + 1) = 2 2 2

Se a diagonal que os conecta central, as outras diagonais principais que saem de P e Q devem estar em lados distintos de PQ , pois caso contrrio, se utilizaro j trs diagonais sem haver unido dois vrtices consecutivos que ficam a um lado de PQ e estes no podem ser conectados pela diagonal que falta. Ento, s h uma configurao possvel: Aqui a rea menor ou igual que 1 1 ( PX + QY ) < (2) = 1, pelos lemas 2 e 2 2 1 respectivamente.
P Y X

d) Os vrtices P e Q no esto conectados por uma diagonal principal. Nesse caso s h trs configuraes possveis: Pelo lema 2, neste caso a rea menor ou 1 igual que (1 1 + 1 1) = 1. 2
P

EUREKA! N9, 2000

14

Sociedade Brasileira de Matemtica

Neste caso a rea menor ou igual que 3 1 1 ( XY + 1) < (2 + 1) < , onde o ltimo 2 2 2 se da por desigualdade triangular.

Pelo lema 1, algum dos segmentos PX ou QY mede menos que 1, logo esta configurao pode ser reduzida configurao de um vrtice de grau trs, que j foi estudada (se bem no a mesma, o raciocnio idntico). Esgotadas as possibilidades, se resolve completamente o problema.

X Y

Voc sabia
Que existe um algoritmo que, dada uma soma envolvendo uma funo hipergeomtrica (por exemplo: binomiais, exponenciais, logartmicas, trigonomtricas), expressa-a como uma soma telescpica. Com isso, podemos encontrar uma recurso satisfeita por tal soma ou mesmo, quando possvel, obter uma frmula fechada. O mais impressionante que o algoritmo s no obtm uma frmula fechada se esta no existir (mais precisamente, quando a soma no puder ser escrita como uma funo hipergeomtrica). Os autores deste algoritmo - Marko Petkovsek, Herbert Wilf e Doron Zeilberger - publicaram um livro descrevendo sua descoberta, com o inusitado ttulo "A =B ". Este livro encontra-se disponvel na rede. Veja : http://www.cis.upenn.edu/~wilf/AeqB.html

EUREKA! N9, 2000

15

Sociedade Brasileira de Matemtica

XLI OLIMPADA INTERNACIONAL DE MATEMTICA


Problemas e Solues
PROBLEMA 1

Duas circunferncias 1 e 2 intersectam-se em M e N. Seja l a tangente comum a 1 e 2 que est mais prxima de M do que de N. A reta l tangente a 1 em A e a 2 em B. A reta paralela a l que passa por M intersecta novamente a circunferncia 1 em C e novamente a circunferncia 2 em D. As retas CA e DB intersectam-se em E; as retas AN e CD intersectam-se em P; as retas BN e CD intersectam-se em Q. Mostre que EP = EQ.
SOLUO DE ONOFRE CAMPOS DA SILVA FARIAS (FORTALEZA - CE) e DANIEL MASSAKI YAMAMOTO (SO PAULO - SP) E l B D A M Q P C

Inicialmente, vamos mostrar que AE = AM e que BE = BM. De fato, como AB || CD e AB tangente a 1, segue-se que EAB =ACM = BAM = e EBA = BDM = ABM = Da, imediato que os tringulos EAB e MAB so congruentes (caso ALA), de modo que AE = AM e BE = BM.
EUREKA! N9, 2000

16

Sociedade Brasileira de Matemtica

Alm disso, segue da congruncia que EM AB, e portanto, EM CD. Agora, basta mostrarmos que PM = MQ, porque dessa forma EM ser mediatriz de PQ, tal que EP = EQ. Para isto, note que o prolongamento de MN passa pelo ponto mdio de AB, pois MN o eixo radical das duas circunferncias e AB tangente comum. Assim, como PQ || AB, conclumos que MN tambm passa pelo ponto mdio de PQ. Logo, EM PQ e EM passa pelo ponto mdio de PQ, de modo que EM mediatriz de PQ e EP = EQ.
SOLUO DE DANIEL NOBUO UNO (SO PAULO - SP)

e 2

X C 1

A P

[ XABY ] N = CM N D = NM D . Como N + NM D = e Temos CA NB CM N + EB N + NB N = CA D = ; temos que AEBN cclico. EA B = EC D = AN C = CN M = ; XA A = . Seja EA M = BN M = YB D = DN B = A = , analogamente, BD Seja EB B = EN A = EN A = B = ; EB Como AEBN cclico, EA
Q = EN C = EN Q ECNQ cclico EQ C = + EC D = EN D = + . P = EN P EDNP cclico EP ED Logo, EPQ issceles.

EUREKA! N9, 2000

17

Sociedade Brasileira de Matemtica

PROBLEMA 2

Sejam a, b, c nmeros reais positivos tais que abc = 1. Prove que

1 1 1 a 1 + b 1 + c 1 + 1. b c a
SOLUO

Fazendo a =

y x z ,b = ,c = y z x y = 1, x = a, z =

1 = ac) , a desigualdade b equivalente a ( x + z y )( x + y z )( y + z x) xyz. Fazendo u = x + z y, v = x + y z e w = y + z x, temos u + v = 2 x, u + w = 2 z e v + w = 2 y, donde a desigualdade equivalente a 8uvw (u + v)(u + w)(v + w). Como u + v, u + w e v + w so todos positivos, no mximo um dentre os nmeros u, v e w no positivo. Se houver um tal nmero, 8uvw ser negativo ou nulo, enquanto (u + v)(u + w)(v + w) positivo, e a desigualdade acima trivial. Se u , v e w so positivos, a desigualdade acima conseqncia direta da desigualdade entre as mdias aritmtica e geomtrica, pois (u + v)(u + w)(v + w) = u 2 v + uv 2 + u 2 w +
(podemos tomar, por exemplo,

+ uw 2 + v 2 w + vw 2 + 2uvw , donde a desigualdade u 2 v + uv 2 + u 2 w + uw 2 + v 2 w + vw 2 6uvw, que segue


u 2 v + vw 2 2uvw, uv 2 + uw 2 2uvw e u 2 w + v 2 w 2uvw.

equivalente a das desigualdades

Seja n 2 um nmero inteiro positivo. No incio existem n pulgas numa reta horizontal, nem todas no mesmo ponto. Para um nmero real positivo , define-se um salto da seguinte maneira: Escolhem-se duas pulgas quaisquer nos pontos A e B com o ponto A esquerda do ponto B; A pulga que est em A salta at o ponto C da reta, direita de B, tal que BC = . AB Determine todos os valores de para os quais, para qualquer ponto M na reta e quaisquer posies iniciais das n pulgas, existe uma sucesso finita de saltos que levam todas as pulgas para pontos direita de M.
EUREKA! N9, 2000

PROBLEMA 3

18

Sociedade Brasileira de Matemtica

SOLUO:

1 (n 1) Devemos demonstrar duas coisas: 1 , existe uma seqncia infinita de movimentos que vai a) que, para l (n 1) levando as pulgas cada vez mais para a direita, ultrapassando qualquer ponto prefixado M; A resposta para l

1 e para qualquer posio inicial dada as pulgas, existe um (n 1) ponto M tal que as pulgas em um nmero finito de movimentos jamais alcanam ou ultrapassam M.
b) que, para l < Comearemos pelo item b). Sejam x1 , x 2 , ..., x n as posies iniciais das pulgas, com x1 x 2 ... x n , de tal forma que x n a posio da pulga mais direita.

1 Seja P = (1 (n 1) l ) ( x n l x1 l x 2 ... l x n 1 ).
O ponto P claramente est direita de todas as pulgas. Afirmamos que se aps alguns movimentos as novas posies so x'1 ,..., x' n e definimos 1 P' = (1 (n 1) l ) ( x' n l x'1 l x' 2 ... l x' n 1 ). Se P ' P, isto conclui a demonstrao. Basta considerar o que ocorre aps um movimento. Se a pulga que estava em xi pula sobre a pulga que estava em x n ento x' n x n = l ( x n xi ) e x' n l x n = x n l xi e P' = P. Qualquer outro caso ainda mais favorvel. De fato, se a pulga que pulou continua atrs de x n , temos x n ' = x n e x1 '+... + x n 1 ' > x1 + ... + x n 1 , donde P' < P. Se ela passa de x n , teremos x n ' = x j + ( x j xi ) x n 'x n < x n 'x j = x j x i < x n x i .

EUREKA! N9, 2000

19

Sociedade Brasileira de Matemtica

Item a) Se P = x n ( x1 + x 2 + ... + x n 1 ) se, em cada movimento, a pulga mais esquerda pula sobre a pulga mais direita, temos x n ' = x n + ( x n x1 ) x n 'x n = x n x1 e P ' = P, donde P uma constante positiva (escolhendo a origem, por exemplo, em x n ) . Temos ento

1 n 1 1 ( xn x j ) = xn n 1 ( x1 + ... + x n1 ) x n ( x1 + ... + xn1 ) = P n 1 j =1 x n x1 1 n 1 P ( x n x j ) = P x n ' x n = ( x n x1 ) , donde o ponto n 1 j =1 n 1

P para a direita a cada passo, logo tende a n 1 infinito. Como o ponto mais a direita, aps n 1 passos ser o ponto mais esquerda, todos os pontos tendem a infinito (para a direita). mais direita caminha pelo menos

Nota: Na estratgia descrita na soluo do item a), o ponto mais esquerda se torna sempre o mais direita, donde podemos definir x n +1 = x n ' = x n + ( x n x1 ), e teriamos simplesmente x j ' = x j +1 , j. Reduzimos
ento a anlise dessa estratgia ao estudo da recorrncia linear x n +1 = (1 + ) x n x1 , cujo polinmio caracterstico P( x) = x n +1 (1 + ) x n + , P( x) do qual 1 raiz, donde, como = x n ( x n 1 + x n 2 + ... + x + 1), a expresso x 1 y m = x m ( x m 1 + x n 2 + ... + x m n + 2 + x m n +1 ) um invariante da recorrncia, isto , y m +1 = y m m, donde y m constante. Da vem nossa frmula para P. Veja o artigo sobre equaes de recorrncia nesta Eureka.
PROBLEMA 4

Um mgico tem cem cartes numerados de 1 a 100. Coloca-os em trs caixas, uma vermelha, uma branca e uma azul, de modo que cada caixa contm pelo menos um carto. Uma pessoa da platia escolhe duas das trs caixas, seleciona um carto de cada caixa e anuncia a soma dos nmeros dos dois cartes que escolheu. Ao saber esta soma, o mgico identifica a caixa da qual no se retirou nenhum carto. De quantas maneiras podem ser colocados todos os cartes nas caixas de modo de que este truque sempre funcione? (Duas maneiras consideram-se diferentes se pelo menos um carto colocado numa caixa diferente).

EUREKA! N9, 2000

20

Sociedade Brasileira de Matemtica

SOLUO DE FABRCIO SIQUEIRA BENEVIDES (FORTALEZA - CE)

Seja f (n) o nmero de maneiras de se colocarem os cartes de 1 a n , mas sem contar a ordem das caixas. No final, basta multiplicar f (100) por 6 = 3!. Achamos facilmente que f (4) = 2, onde as duas nicas maneiras so: 1, 4 2 3 1 2, 3 4

Vamos provar por induo que f (n + 1) = f ( n) = 2 para n 4, e que as nicas maneiras so das formas: A) 1, 4, 7,... 2, 5,... 3, 6,... e B) 1 2, 3, ..., n 1 n

H duas possibilidades para o carto n + 1:

1) Ele aparece junto com outros cartes. Nesse caso, os cartes de 1 a n estaro na configurao A ou B. Se for A): n 2, n 1 e n esto em caixas diferentes. n + 1 no pode estar na mesma caixa de n ou n 1, pois (n 2) + ( n + 1) = ( n 1) + ( n) Se colocarmos n + 1 na mesma caixa de n 2, a mgica funcionar. Basta ver a soma mdulo 3. No caso B): n + 1 no pode entrar em nenhuma caixa, pois 1 + (n + 1) = 2 + (n) e (n) + 3 = ( n + 1) + 2 2) (n + 1) est em uma caixa isolada. Como (n + 1) + (1) = (2) + (n), 2 e n devem estar na mesma caixa. Temos a seguinte configurao:

n +1 2, n, k k +1

Se k est junto com 2 e k + 1 est na outra caixa, (n + 1) + k = n + (k + 1) . Ento k e k + 1 esto na mesma caixa.

Fazendo isso para k = 2, 3,..., n 2, temos que os cartes 2, 3,..., n esto na mesma caixa . Como a outra caixa no pode ficar vazia, 1 est nela.
EUREKA! N9, 2000

21

Sociedade Brasileira de Matemtica

Para ver que esta configurao funciona, basta ver os intervalos das possveis somas para cada par de caixas. o nmero total 6 f (100) = 12.

PROBLEMA 5

Verifique se existe um inteiro positivo n tal que n divisvel por exatamente 2000 nmeros primos diferentes e 2 n + 1 divisvel por n.
SOLUO DE HUMBERTO SILVA NAVES (SO PAULO - SP)

Vamos, primeiramente, provar o seguinte "super-lema": Lema: 3 n | 2 3 + 1 e 2 3 + 1 tem pelo menos "n" fatores primos distintos: Esse lema ser provado por induo: Quando n = 1 ou n = 2, o lema verdadeiro pois 23 + 1 = 9 = 32 e 2 3 + 1 = 513 = 3 3 19 Suponha que seja vlido, tambm, para um certo "p" ( p 2), Vamos provar que tambm verdadeiro para " p + 1":
2

* 3 p | 2 3 + 1 k Z | 2 3 + 1 = k 3 p 2 3 = 1 + k 3 p (2 3 ) 3 = = (1 + k 3 p ) 3 2 3 * *2 3
p +1 p p +1

= 1 + k 3 p +1 k 2 3 2 p +1 + k 3 3 3 p 2 3
p +1

p +1

+1=

= (k k 2 3 p + k 3 3 2 p 1 ) 3 p +1 3 p +1 | 2 3
p p

+ 1.
p

+ 1 = ( 2 3 ) 3 + 13 = (2 3 + 1)[(2 3 ) 2 (2 3 ) + 1], pois


p p p p p p p

a, b R temos : a 3 + b 3 = ( a + b)(a 2 ab + b 2 ) S que 2 23 23 + 1 = (23 + 1) 2 3(23 + 1) + 3 mdc (23 + 1;2 23 23 + 1) = = mdc(23 + 1;3) = 3 pois 3 p | 23 + 1 3 | 23 + 1. Logo deve existir um fator primo
"j" tal que j | ( 2 23 2 3 + 1) e j / | (2 3 + 1), logo 23
p
p p P
p +1

+ 1 = (23 + 1)(223 23 + 1)
p +1

tem pelo menos 1 fator primo a mais que 2 3 + 1, logo 2 3 + 1 tem pelo menos "p + 1" fatores primos em sua decomposio. Logo o lema vlido para "p + 1", tambm. Pelo princpio da induo finita, provamos que o lema verdadeiro para todo natural "n".
EUREKA! N9, 2000

22

Sociedade Brasileira de Matemtica

Pronto, o "super-lema" veio te salvar nessa hora de sufoco! 2000 Sabemos que mpar 2 3 + 1 tem pelo menos 2000 fatores primos e pegamos 1999 fatores primos que so diferentes de 3 e os chamamos de p1 ; p 2 ; p 3 ;...; p1999 . Temos que

3 2000 | 2 3
Basta escolhermos o inteiro mpar n = 3

2000

+1 e
2000

p1 p 2 p 3 ... p1999 | 2 3
2000

+1

p1 ... p1999 , pois:

2000

|n2

32000

+ 1 | 2 + 1 e como n | 23
2000

+ 1 n | 2n + 1

Logo existe um "n" de 2000 fatores primos, tal que n | 2 n + 1 .


PROBLEMA 6

Sejam AH 1 , BH 2 , CH 3 as alturas de um tringulo acutngulo ABC. A circunferncia inscrita no tringulo ABC tangente aos lados BC, CA, AB em T1 , T2 , T3, respectivamente. Seja l1 a reta simtrica da reta H 2 H 3 relativamente reta T2T3 , l 2 a reta simtrica da reta H 3 H 1 relativamente reta T3T1 e l 3 a reta simtrica da reta H 1 H 2 relativamente reta T1T2 . Prove que l1 , l 2 , l 3 determinam um tringulo cujos vrtices pertencem circunferncia inscrita no tringulo ABC.
SOLUO DE ONOFRE CAMPOS DA SILVA FARIAS (FORTALEZA - CE) A H2

H3 T3 P L M

T2 N

l1

T1

fig. 01
EUREKA! N9, 2000

23

Sociedade Brasileira de Matemtica

Inicialmente, vamos mostrar que as retas l1, l2 e l3 so paralelas aos lados BC, CA e AB, respectivamente. Sejam P H 2 H 3 T2T3 , M e N os pontos de interseo da reta l1 com a circunferncia inscrita. Supondo AB < AC, temos a fig. 01 acima. Veja que PH3T3 = AH3H2 = C , B+C , AT3T2 = AT2T3 = 2 de modo que BC H 3 PT3 = e H3PM = 2.H3PT3 = B C, 2 e, portanto, H3LM = H3PM + LH3P = (B C) + C = B. Logo, l1 || BC e, analogamente, conclumos que l2 || AC e l3 || BA. Se AB = AC, ento, l1 || BC por simetria. Neste caso, teremos H2H3 || T2T3 || l1 || BC. Agora, vamos calcular a distncia entre as retas l1 e BC. Como PT2 a bissetriz do ngulo H2PN, ento Mas, d(T2, PH2) = d(T2, l1) e d(l1, BC) = d(T2, BC) d(T2, PH2) (1) d(T2, BC) = (p c).sen C e como AH2H3 = B, segue que d(T2, PH2) = H2T2.sen B. (2) Tambm, Em (2), obtemos: H2T2 = CH2 CT2 = a.cos C (p c). d(T2, PH2) = (a.cos C (p c))sen B. (3) Usando que a. cos C =

a2 + b2 c2 b c , obtemos: , sen B = , sen C = 2b 2R 2R

EUREKA! N9, 2000

24

Sociedade Brasileira de Matemtica

d (l1 , BC ) = ( p c).senC + ( p c).senB senB.(a. cos C ) c b b a2 + b2 c2 + ) . 2R 2R 2R 2b 2 2 (a + b c).(b + c) a + b c 2 = 4R 4R 2 ab + ac a = 4R a b+ca = 2R 2 = ( p a).senA. Dessa forma, acabamos de mostrar que = ( p c)(
d(l1, BC) = d(T2, AB) = d(T3, AC) = (p a).sen A. (*) Vamos mostra agora que BT2 = BN e, analogamente, mostraremos que CT3 = CM. Sejam U e V as projees ortogonais de T2 e N sobre os lados AB e BC respectivamente. Ento, NV = T2U = (p a).sen A. que Sejam I o incentro e E a projeo ortogonal de I sobre l1 (fig. 02). Veja

A U

EN = T1V e EI = |(p a).sen A r|.

T2 T3 M

E I r

l1

T1

V
fig. 02

EUREKA! N9, 2000

25

Sociedade Brasileira de Matemtica

i ) T1V 2 = EN 2 = IN 2 EI 2 = r 2 (( p a ). sen A r ) 2 = 2r.( p a ) sen A ( p a) 2 . sen 2 A

ii ) T3U 2 = T2T3 2 T2U 2 = T2T3 2 ( p a ) 2 . sen 2 A


Agora, o quadriltero AT3IT2 inscritvel na circunferncia de dimetro AI. Logo, temos duas relaes: I) II) T2T3 = AI.sen A; T2T3.AI = 2r.AT2 = 2r.(p a) (pelo teorema de Ptolomeu).

De (I) e (II), obtemos T2T32 = 2r.(p a).sen A. Portanto, em (i) e (ii) ficamos com T1V 2 = T3U 2 T1V = T3U , e como BT1 = BT3, por ltimo segue que BVN BUT2 (**) de modo que BT2 = BN, como queramos demonstrar.

A T2 T3 r I r

B
fig. 03

De (**), ainda podemos concluir que ABT2 = CBN, e como ABI = CBI, temos T2BI = NBI.

EUREKA! N9, 2000

26

Sociedade Brasileira de Matemtica

Finalmente, como T2B = NB, podemos concluir que N o simtrico de T2 em relao bissetriz do ngulo B. Analogamente, M o simtrico de T3 em relao bissetriz interna do ngulo C. Dessa forma, podemos redefinir as retas l1, l2 e l3 da seguinte forma: Sejam S1, S2 e S3, respectivamente, os simtricos de T1, T2 e T3 em relao s bissetrizes dos ngulos A, B e C, respectivamente. L1 a reta que passa por S2 e S3, l2 a reta que passa por S1 e S3 e l3 a reta que passa por S1 e S2. Claramente, l1, l2 e l3 determinam um tringulo inscrito na circunferncia inscrita no tringulo ABC.

EUREKA! N9, 2000

27

Sociedade Brasileira de Matemtica

BRAHMAGUPTA PARA TODOS


Jos Cloves Verde Saraiva - UEMA
Nvel Intermedirio
Nestas notas apresentaremos as frmulas da geometria plana para o clculo da rea de um tringulo e de um quadriltero cclico (inscrito numa circunferncia) em funo do comprimento de seus lados e de seu semipermetro. Para o tringulo esta conhecida como a frmula de Heron de Alexandria, embora escritores rabes afirmem que esta foi descoberta por Arquimedes. Para os quadrilteros cclicos h uma generalizao natural da frmula de Heron, to importante, que considerada como a mais notvel descoberta da geometria hindu, feita por BRAHMAGUPTA. Originalmente a prova de Brahmagupta faz uso do conhecido Teorema de Ptolomeu para quadrilteros cclicos. Aqui, estas duas frmulas so deduzidas, elementarmente, da Lei dos co-senos para um tringulo.
1. A FRMULA DE HERON

Seja ABC um tringulo cujos lados medem a, b e c indicados na figura abaixo, ento a medida da rea deste tringulo dada pela frmula:

S=
onde p =

p ( p a )( p b)( p c)

1 ( a + b + c) o semi-perimetro do ABC . 2
B

________________________________________________________________________________ O autor agradece o apoio pesquisa da Universidade Estadual de Maranho (UEMA) e o saudvel ambiente de trabalho do DEMATI-CECEN
EUREKA! N9, 2000

28

Sociedade Brasileira de Matemtica

Prova: Todos sabem que a rea de um tringulo calculada pela frmula 1 S = bh. Nas condies aqui tratadas sabemos que a altura relativa ao lado CA 2 , donde podemos escrever para a rea: dada por h = a senC

S=

1 (*) ab senC 2

tambm muito conhecida a Lei dos co-senos para um tringulo. E, pela figura acima, esta pode ser escrita como: (**) c 2 = a 2 + b 2 2ab cos C Com efeito, elevando ao quadrado S na expresso (*) temos que:

= a 2 b 2 (1 cos 2 C ) = ab(1 cos C ) ab(1 + cos C ) = 4 S 2 = a 2 b 2 sen 2 C )(ab + ab cos C ) = (ab ab cos C
Multiplicando este ltimo resultado por 4, obtemos: )(2ab + 2ab cos C ) 16 S 2 = ( 2ab 2ab cos C no qual podemos completar quadrados e adequar os fatores para o uso da Lei dos co-senos (**) temos:
][a 2 + b 2 + 2ab a 2 b 2 + 2ab cosC ] = 16S 2 = [a 2 b 2 + 2ab + a 2 + b 2 2ab cosC = [(a b) 2 + c 2 ][(a + b) 2 c 2 ] = [c 2 (a b) 2 ][(a + b) 2 c 2 ]

fatorando as diferenas de quadrados, podemos escrever: 16S 2 = (c + a b)(c a + b)(a + b + c)(a + b c)

= (a + b + c)( a + b + c)(a b + c)(a + b c)


Aonde, dividindo por 16: 1 1 1 1 S 2 = (a + b + c) (a + b + c 2a ) ( a + b + c 2b) (a + b + c 2c) 2 2 2 2

EUREKA! N9, 2000

29

Sociedade Brasileira de Matemtica

Portanto: 1 1 1 1 S 2 = (a + b + c) ( a + b + c) a (a + b + c) b (a + b + c) c 2 2 2 2 1 substituindo o semi-permetro p = ( a + b + c), conclumos que: 2 S = p ( p a )( p b)( p c) , a conhecida frmula de Heron. De uma certa analogia da frmula acima demonstrada deduzimos, a seguir, o resultado principal destas notas.
2. A FRMULA DE BRAHMAGUPTA

A medida da rea de um quadriltero cclico de lados a, b, c, d cujo semipermetro denotado por p a seguinte: K = ( p a)( p b)( p c)( p d )
D

onde p =
c x

d A a b B

1 (a + b + c + d ) 2 1 ( p a) = ( a + b + c + d ) 2 1 ( p b) = (a b + c + d ) 2 1 ( p c) = (a + b c + d ) 2 1 ( p d ) = (a + b + c d ) 2

Os seguintes fatos elementares so considerados na prova: I. Seja um quadriltero tal que seus ngulos opostos internos sejam +D = 180 (esta relao vale para os suplementares: na figura temos B quadrilteros cclicos). As reas dos tringulos S1 e S 2 so dadas pelas relaes: 1 e S = 1 cd senD S1 = ab senB 2 2 2

II.

EUREKA! N9, 2000

30

Sociedade Brasileira de Matemtica

III.

A diagonal x, indicada na figura anterior, pela Lei dos co-senos aplicada aos tringulos S1 e S 2 verifica a relao: = x 2 = c 2 + d 2 2cd cos D a 2 + b 2 2ab cos B

+D ) = 1, que implica a igualdade Prova: De I. obtemos que cos( B senD = 1 + cos B cos D (*). Com efeito, seja K a medida da rea do senB quadriltero cclico dada por: K = S1 + S 2 , Substituindo II. Obtemos a igualdade + cd senD . Elevando ao quadrado obtemos: 2 K = ab senB 2 2 2 + 2 abcd senB senD + c 2 d 2 sen 2 D 4 K = a b sen 2 B substituindo (*) no produto de senos temos: ) + 2abcd (1 + cos B cos D ) + c 2 d 2 (1 cos 2 D ) 4 K 2 = a 2 b 2 (1 cos 2 B ou melhor,
4K 2 2 2 2 cos D ) + abcd (1 + cos B cos D ) + c 2 d 2 (1 cos 2 D ) = a b (1 cos B ) + abcd (1 + cos B

)(1 + cos B ) + ab(1 + cos D )cd (1 + cos B ) + c 2 d 2 (1 cos D )(1 + cos D ) 4 K 2 = a 2 b 2 (1 cos B

Multiplicando, adequadamente, por 4 os dois membros,

) + cd (1 + cos D )] 2 [ab (1 + cos B ) + cd (1 cos D )] 16K 2 = 2[ab (1 cos B 2ab 2cd cos D 2cd][2ab cos B + 2ab + 2cd cos D 2cd cos D ] 16K 2 = [2ab cos B
Substituindo a relao III. obtemos:

16 K 2 = [ a 2 + b 2 2ab (c 2 + d 2 ) 2cd ][a 2 + b 2 + 2ab (c 2 + d 2 ) + 2cd ] 16 K 2 = [(a b) 2 (c + d ) 2 ][(a + b) 2 (c d ) 2 ]


16 K 2 = [(a b) (c + d )][(a b) + (c + d )][(a + b) (c d )][(a + b) + (c d )] por tanto temos que: 1 1 1 1 K 2 = (a + b + c + d ) (a b + c + d ) (a + b c + d ) (a + b + c d ) 2 2 2 2 da, imediato ver que:
EUREKA! N9, 2000

31

Sociedade Brasileira de Matemtica

K 2 = ( p a)( p b)( p c)( p d ) permitindo concluir que, K = ( p a)( p b)( p c)( p d ) concluindo a prova.
Como observao final, a frmula acima demonstrada no pode ser mais geral do que foi provado por BRAHMAGUPTA. De fato, a frmula vale exatamente para os quadrilteros cclicos. Em geral, se o quadriltero no for cclico, sua rea estritamente menor que ( p a)( p b)( p c)( p d ) , pois nesse caso senD < 1 + cos B cosD , o que pode ser usado como na prova acima para senB provar nossa afirmao.

REFERNCIA E.W. Hobson, A Treatise on Plane Trigonometry (NY); Macmillan Company, 4a ed., (1902), pg. 204.
EUREKA! N9, 2000

32

Sociedade Brasileira de Matemtica

EQUAES DE RECORRNCIA
Hctor Soza Pollman - Universidade Catlica do Norte - Antofagasta, Chile
Nvel Avanado
Frenqentemente em teoria da Computao (ver exemplo [2]), ao analisar o tempo de execuo de um algoritmo (ou o espao ocupado na memria pelos dados), obtemos uma (ou mais) equaes discretas, chamadas de Equaes de Recorrncia, cuja incgnita uma funo inteira f(n), que geralmente uma funo do tamanho n do problema (por exemplo: a quantidade de dados a ordenar se um algoritmo de ordenamento). Esta equao resulta ser uma relao entre f(n) e seus valores prvios, como so f(n 1), f(n / 2), ou outro. Alm disso se conhecemos o algoritmo analisado com detalhe, podemos estabelecer um valor de bordo num ponto dado (como f(0) por exemplo). Neste artigo so apresentados alguns dos mtodos desenvolvidos para resolver este tipo de equaes, as quais aparecem em ordem de dificuldade. Os tipos de equaes de recorrncia a serem consideradas so as seguintes, em que a incgnita a sucesso x n com n 0 :
1. EQUAO LINEAR DE PRIMEIRA ORDEM COM COEFICIENTES DE VALOR INTEIRO 1.

O tipo mais simples de equao de recorrncia de primeira ordem : x n +1 = x n + bn , n 0 em que x 0 e a sucesso bn so dados do problema. Sua resoluo faz uso da propriedade telescpica da soma obtendo:

x n = x 0 + bi , n 1
i =0

n 1

Exemplo: Para a equao:

x n +1 = x n + 2 n , n 0,

com

x 0 = 1,

obtemos

x n = 2 , n 0.
2. EQUAO LINEAR DE PRIMEIRA ORDEM COM COEFICIENTES CONSTANTES.

A equao linear de primeira ordem com coeficientes constantes : a n x n = bn x n 1 + c n , n 0


EUREKA! N9, 2000

33

Sociedade Brasileira de Matemtica

em que x 0 e as sucesses numricas a n , bn e c n so dados do problema (as sucesses a n e bn no devem ser nulas). Para resolver esta equao ela deve ser multiplicada pelo fator S n (chamado fator somante): S n a n x n = S n bn x n 1 + S n c n Impe-se a condio: S n bn = S n 1 a n 1 (1) com o qual obtemos: S n a n x n = S n 1 a n 1 x n 1 + S n c n Observa-se que a equao anterior se reduz a uma de primeiro tipo, e que sua soluo : n 1 xn = ( S 0 a 0 x 0 + S i ci ), n 0 S n an i =1 O fator somante obtido a partir da condio (1) e considerando que S 0 = 1 : a a ...a a S n = n 1 n 2 1 0 , n 1 bn bn 1 ...b2 b1
EXEMPLO: AS TORRES DE HANOI.

Dadas trs varetas e n discos de distintos tamanhos colocados na primeira vareta em ordem de tamanho (do menor ao maior), mover estes n discos desde a vareta inicial at a terceira usando a segunda como auxiliar, sem colocar um disco de tamanho maior sobre um de tamanho menor (para maiores explicaes ver [4]). Se x n a quantidade de movimentos para levar os n discos da primeira a terceira vareta, podemos provar, ao analisar como so distribudos os movimentos, que, se x n a quantidade de movimentos para mover os n discos desde a primeira terceira vareta (com n 0), ento: x n = 2 x n 1 + 1, n 1 com x 0 = 0. De fato, dada uma soluo do problema de Hanoi com n 1 discos em x n 1 movimentos, podemos mover os n 1 primeiros discos para a segunda vareta, depois mover o ltimo disco para a terceira vareta e por fim mover os n 1 primeiros discos para a terceira vareta, gastando x n 1 + 1 + x n 1 = 2 x n 1 + 1 movimentos. Neste caso temos que o fator somante 1 resulta ser: S n = n . Logo, a soluo da equao das torres de Hanoi : 2
EUREKA! N9, 2000

34

Sociedade Brasileira de Matemtica

x n = 2 n 1, n 0 Observamos, por exemplo, que para n = 3 devem ser realizados 7 movimentos. Deixamos como exerccio para o leitor provar que impossvel resolver este problema usando uma quantidade menor de movimentos.
3. EQUAES HOMOGNEAS DE PRIMEIRA ORDEM COM COEFICIENTES CONSTANTES.

Considere a equao: a k x n + k + a k 1 x n + k 1 + ... + a 0 x n = 0, n 0 (2) em que a 0 ,..., a k so sucesses independentes de n, e os valores de xi so conhecidos para i = 0, ..., k 1 (correspondem aos valores de bordo). Supondo que a equao (2) admite uma soluo do tipo: x n = n , em que um parmetro inteiro, e substituindo em (2) temos: a k n + k + a k 1 n + k 1 + ... + a 0 n = 0 Se 0 ento obtemos a equao caracterstica associada a equao (2): a k k + a k 1k 1 + ... + a 0 0 = 0 Vamos mostrar que se esta equao tem as razes complexas 1 ,..., r com multiplicidades 1 , 2 ,..., r N , respectivamente, ento as solues de (2) so
n n exatamente as seqncias ( x n ) da forma xn = Q1(n)1 + Q2 (n)n 2 + ... + Qr (n)r , onde Q1 ,..., Qr so polinmios com grau (Qi ) < i , 1 i r (em particular, se i uma raz simples ento Qi constante).

Seja P( x) = ak x k + ak 1 x k 1 + ... + a0 um polinmio. Dizemos que uma seqncia ( x n ) nN satisfaz a propriedade Rec ( P( x)) se ak xn+k + ak 1 xn+k 1 + ... + a0 xn = 0, n N . No difcil verificar os seguintes fatos:

i) Se ( X n ) e (Yn ) satisfazem Rec ( P ( x)) e c C ento ( Z n ) = X n + cYn satisfaz Rec ( P( x)) . ii) Se Q( x) = br X r + br 1 X r 1 + ... + b0 e ( X n ) satisfaz Rec ( P ( x)) ento ( X n ) satisfaz Rec ( P( x)Q( x))
(isso segue de

b j (a k X n+ j + k + a k 1 X n+ j + k 1 + ... + a0 X n+ j ) = 0, n N )
j =0

EUREKA! N9, 2000

35

Sociedade Brasileira de Matemtica

iii)

(X n )

satisfaz

Rec ( P ( x))

se

e
k

s
j =0

se

(Yn ) = X n n

satisfaz

Rec ( P (X )) (substitua X n + j = n + j Yn + j em
n

a j X n+ j = 0).

iv) Se S n = x k ento ( x n ) satisfaz Rec ( P( x)) se e s se ( S n ) satisfaz


k =0

Rec (( x 1) P ( x)) (escreva x n + j +1 = S n + j +1 S n + j e substitua em

a j x n+ j +1 = 0).
j =0

Por iii), para ver que, para todo polinmio Q( x) de grau menor que m,
X n = Q ( n) n

satisfaz Rec (( x ) m ), basta ver que (Yn ) = (Q(n) ) satisfaz

Rec (( x 1) m ), o que faremos por induo. Isso claro se m = 1, e em geral, se ~ Z n = Yn +1 Yn = Q(n + 1) Q(n), como Q ( x) = Q ( x + 1) Q ( x) tem grau menor que m 1, ( Z n ) satisfaz Rec (( x 1) m 1 ) (por hiptese de induo), e logo, por (iv), (Yn ) satisfaz Rec (( x 1) m ). Essa observao, combinada com ii), mostra que se ( P ( x) = ( x 1 )1 ( x 2 ) 2 ...( x r ) r , e grau (Qi ) < i para 1 i r ento x n = Qi (n) n i satisfaz Rec ( P ( x )) .
i =1 r

Para ver que se ( x n ) satisfaz Rec ( P( x)) ento x n da forma acima, usaremos induo novamente.
n , Z n = Yn +1 Yn (com Z 0 = Y0 ). Supomos 1 0 e tomamos Yn = X n 1

Por iii) e iv), Z n satisfaz Rec ( P (1 x) ( x 1)) e, portanto por hiptese de induo, ~ ~ ~ ~ Z n = Q1 ( x) + Q2 ( x)( 2 1 ) n + ... + Qr ( x)( r 1 ) n , onde grau Qi < i para ~ 2 i r e grau Q1 < 1 1. Para terminar a prova, vamos mostrar que se existem polinmios P1 , P2 ,..., Pk tais
n que Yn +1 Yn = P1 (n) + P2 (n) 2 + ... + Pk ( n) kn (onde 1, 2 ,..., k so complexos ~ ~ ~ n distintos e Pi 0, i 2 ) ento Yn = P1 (n) + P2 ( n) 2 + ... + Pk ( n) kn , onde ~ ~ ~ P1 ,..., Pk so polinmios com grau Pi = grau Pi para i 2 e grau ~ P1 = grau P1 + 1, por induo na soma dos graus dos polinmios Pi , onde convencionamos que o grau do polinmio nulo 1.

EUREKA! N9, 2000

36

Sociedade Brasileira de Matemtica


n (no nosso caso temos i = i 1 , e como X n = 1 Yn o resultado segue imediatamente).

Para provar essa afirmao observamos inicialmente que, se a soma dos graus de Pi 1, ento Yn +1 Yn = 0, n, e logo, Yn constante e, em geral, consideramos 2 casos:

a)

P1 ( x) = c m x m + c m 1 x m 1 + ... + c 0 , c m 0.
m +1

Nesse

caso

definimos

c n ~ ~ ~ Yn = Yn m , e temos Yn +1 Yn = Q1 (n) + P2 ( n) 1n + ... + Pk (n) kn , com m +1 ~ grau Q < m. Por hiptese de induo, Yn (e logo Yn ) da forma desejada.
b)
~ Yn = Yn

P2 ( x) = d s x s + d s 1 x s 1 + ... + d 0 , d s 0.
dsns n 2

Nesse

caso,

definimos

~ ~ n + P3 (n) 3n + ... + Pk (n) kn , , e temos Yn +1 Yn = P1 (n) + Q(n) 2 2 1 ~ com grau Q < s. Por hiptese de induo, Yn (e logo Yn ) da forma desejada.
Exemplo: x n = sen(n ) satisfaz uma recorrncia linear. De fato, x n +1 = sen( n + ) = sen( n ) cos + cos(n ) sen x n + 2 = sen(n + 2 ) = sen(n ) cos 2 + cos(n ) sen2 sen2 sen2 xn+2 cos ) x n , ou seja, x n +1 = (cos 2 sen sen x n + 2 = 2 cos X n +1 X n . Note que x n no parece ser da forma geral descrita nesta seo, mas de fato,
xn = 1 e in e in 1 i n 1 i n 1 = (e ) (e ) = (cos + isen ) n (cos isen ) n 2i 2i 2i 2i 2i

Obs. Se ( x n ) satisfaz Rec (( x 1) P ( x)) , onde P ( x) = a n x k + a k 1 x k 1 + ... + a 0 , ento, se definirmos Yn = a k x n + k + a k 1 x n + k 1 + ... + a 0 x n , teremos Yn +1 = Yn , n N , ou seja, Yn constante. Assim, a k x n + k + ... + a 0 x n um invariante da seqncia x n , o que uma observao til para muitos problemas olmpicos. Veja o problema 3 da IMO.
EUREKA! N9, 2000

37

Sociedade Brasileira de Matemtica

EXEMPLO: OS NMEROS DE FIBONACCI.

A sucesso que lhes d origem : f n + 2 = f n +1 + f n , n 0 em que f 0 e f 1 so dados. Ao aplicar o mtodo analisado, considerando f 0 = 0 e

f 1 = 1, obtemos o polinmio caracterstico 2 1 = 0, cujas solues so:


1+ 5 1 5 , 2 = 2 2 Considerando as condies do bordo a soluo geral da equao de Fibonacci (ver [3]): 1 n fn = (1 n 2 ), n 0 5

1 =

Observa-se que os valores associados a esta sucesso so todos inteiros. Por exemplo: f 3 = 2, f 4 = 3, etc. Podemos comprovar que, se n converge a infinito
n ento 2 converge a zero, portanto, f n da ordem de 1 , e a frao f n +1 f n converge a 1 .

4. EQUAES NO HOMOGNEAS DE PRIMEIRA ORDEM COM COEFICIENTES CONSTANTES.

A equao mencionada do tipo: a k x n + k + a k 1 + ... + a 0 x n = y n , onde a 0 , a1 ,..., a k so constantes e y n satisfaz uma equao homognea de primeira ordem com coeficientes constantes. Supondo que y n satisfaa b y n + + b 1 y n + 1 + ... + b0 y n = 0, onde b0 , b1 ,..., b so constantes, observamos que b (a k x m + + k + ... + a 0 x m + ) + b 1 ( a k x m + 1+ k + ... + a 0 x m + 1 ) + ... b0 (a k x m + k + ... + a 0 x m ) = 0, ou seja, temos uma equao homognea de primeira ordem com coeficientes constantes. Pode-se demonstrar que a equao caracterstica da recorrncia : (a k x k + a k 1 x k 1 + ... + a 0 ) (b x + b 1 x 1 + ... + b0 ) = 0.
EUREKA! N9, 2000

38

Sociedade Brasileira de Matemtica

Exemplo 1: Considere a seguinte equao de recorrncia 1, se n multiplo de 4, n 1; x n x n 1 = 0, caso contrrio x 0 = 0.

y n+4

1, se n multiplo de 4, satisfaz a recorrncia. yn = 0, caso contrrio = y n y n + 4 y n = 0, n 0. Assim, ( x n + 4 x n +3 ) ( x n x n 1 ) = 0 x n + 4 x n + 3 x n + x n 1 = 0 e x 0 = 0; x1 x 0 = 0 x1 = 0; x 2 x1 = 0 x 2 = 0; x3 x 2 = 0 x3 = 0; x 4 x 3 = 1 x 4 = 1.

A equao caracterstica x 5 x 4 x + 1 = 0 ( x 4 1) ( x 1) = 0 a qual possui as razes 1 (raiz dupla); 1; i ; i. Ou seja, x n = ( An + B ) 1n + C ( 1) n + D i n + E ( 1) n , A, B, C, D, E constantes. De fato, considerando as condies de bordo, n 3 (1) n (1 + i ) n (1 i ) xn = + + i + ( i ) n 4 8 8 8 8 n i n ((1 + i ) + (1) n (1 i )) + (1) n 3 xn = + 4 8 n ( interessante notar que, na verdade, x n = .) 4

Exemplo 2: Seja a seguinte equao de recorrncia, que considera logaritmos em base 2:

f (n) = 2 f ( n ) + log 2 log 2 n, n 3 f (2) = 1 Neste caso aplicamos uma troca de varivel para ir desta equao a uma equao linear, e poder resolv-la, o qual significa que haver soluo s para os valores de n que tome com este cambio. Este :
EUREKA! N9, 2000

39

Sociedade Brasileira de Matemtica

n = 22 ,

n = 22

k 1

, log 2 log 2 n = k

Ao efetuar essas substituies na equao obtemos: x k 2 x k 1 = k (3) onde:

x k = f ( n) = f ( 2 2 ) x0 = f (2 2 ) = 1
A equao (3) uma equao no homognea. Procedendo como acima obtemos: x k + 2 4 x k +1 + 5 x k 2 x k 1 = 0 cuja soluo considerando a condio do bordo : x k = 2 k + 3 2 k , k 0 Logo, voltando a varivel n original, a soluo final :
0

f (n) = 3 log 2 n log 2 log 2 n 2, n = 2 2 , k 0


A soluo s tem resultados inteiros para os valores de n mencionados. Por exemplo: f ( 4) = 3, f (16) = 8, etc. Deixamos a prova deste fato como exerccio para o leitor.

EUREKA! N9, 2000

40

Sociedade Brasileira de Matemtica

EQUAES FUNCIONAIS
Eduardo Tengan - Colgio Etapa
Nvel Avanado
Uma das tcnicas bsicas para a resoluo de equaes com funes perceber quando ela injetora, isto , quando f (a ) = f (b) a = b. Isto particularmente freqente em problemas em que temos equaes do tipo f ( f ( x)) = kx, k 0. De fato, f (a) = f (b) f ( f (a)) = f ( f (b)) ka = kb a = b. "Sabendo que f injetora, podemos provar novas relaes aplicando f dos dois lados da equao". Por exemplo, considere o seguinte problema: (IMO) Seja Q + o conjunto dos racionais positivos. Construa uma funo

f : Q + Q + tal que f ( xf ( y )) = f ( x) y para todo x, y Q + . Para x = 1, temos f ( f ( y )) = f (1) y e da temos que f injetora: f (a ) = f (b) f ( f (a )) = f ( f (b)) f (1) a = f (1) b a = b. ( f (1) Q + , logo f (1) 0). Agora, vamos provar que a funo multiplicativa, isto , que f (ab) = f (a ) f (b). Aplicamos f a cada membro da equao, f (1) f ( f (ab)) = ab f ( f (a )) f (1) f ( f ( a ) f (b)) = = b ab Como os resultados so iguais e f injetora, conclumos que f (ab) = f ( a) f (b). Da temos: f (1 1) = f (1) f (1) f (1) = 1 1 1 1 1 1 f a = f (a) f 1 = f (a) f f = a a a a f (a) a 1 f (a) f = f a = b b f (b) Assim, basta construir a funo para os inteiros positivos. Mais ainda, basta defini-la para os primos. Devemos ter f ( f ( p)) = f (1) p = 1 p . Pensando um pouco, sendo p1 , p 2 , p 3 ,... todos os primos, podemos tomar

EUREKA! N9, 2000

41

Sociedade Brasileira de Matemtica

p n 1 se n par f ( pn ) = 1 p n +1 se n mpar e verificar que a condio inicial satisfeita.


EXERCCIO 1

(IMO) Determine o menor valor possvel de f(1998), onde f uma funo do conjunto N dos inteiros positivos nele mesmo, tal que, para todo m, n N : f (n 2 f (m)) = m( f (n)) 2 . Para funes de domnio real, podemos utilizar desigualdades para obter igualdades. Por exemplo, considere o seguinte problema. Determine todas as funes f : R 2 R tais que f (a; a) = a para todo a R e a + b < c + d f ( a, b) < f (c, d ) para quaisquer a, b, c, d R . Observe, em primeiro lugar, que, para > 0, a+b a+b a+b a+b , f ( a, b) f + , + f 2 2 2 2 a+b a+b f ( a, b) + (*) 2 2 Logo razovel que f (a, b) = ( a + b) 2 . Suponha que existam a 0 e b0 tais que f (a0 , b0 ) (a0 + b0 ) 2. Se f (a0,b0 ) > (a0 + b0 ) 2, ento f (a0,b0) = (a0 + b0) 2 + p, p > 0 . Mas f (a 0 , b0 ) (a 0 + b0 ) 2 + p 2 por (*), ou seja, (a 0 + b0 ) 2 + p (a 0 + b0 ) 2 + p 2 p 0, absurdo. Analogamente f (a 0 , b0 ) < (a 0 + b0 ) 2 impossvel. Logo f (a, b) = (a + b) 2 para todo a, b R. Podemos utilizar um raciocnio semelhante em diversos problemas que envolvem funes crescentes. s vezes, necessrio obter a desigualdade a partir das 2 condies do problema, muitas vezes, utilizamos relaes como f x 2 = ( f ( x ))

( )

para concluir que x 0 f ( x) 0 (basta substituir x no lugar de x na relao anterior). Observe o exerccio a seguir. Seja f uma funo de R em R tal que f(1) = 1, f (a + b) = f(a) +f(b) para todo a, b e f ( x) f (1 x) = 1 para todo x 0. Prove que f ( x) = x para todo nmero real.

EUREKA! N9, 2000

42

Sociedade Brasileira de Matemtica

fcil ver que f (n) = n para todo n inteiro positivo e de f (0) = f (0) + f (0) f (0) = 0 e f (0) = f (1) + f (1) f (1) = 1, que f ( x) = x para todo x Z . Para verificar este resultado para x Q, basta utilizar f (1 x) = 1 f ( x). Observamos ainda que f injetora: temos f ( y ) + f ( x y ) = f ( x) f ( x y ) = f ( x) f ( y ). Como f ( x) f (1 x) = 1 f ( x) 0, ento f ( x) = 0 x = 0 e f ( x) = f ( y ) f ( x) f ( y ) = 0 f ( x y ) = 0 x y = 0 x = y. Para estender o resultado para R, precisamos obter uma desigualdade (na verdade, s desta forma que poderemos distinguir o conjunto dos racionais do conjunto dos reais. No jargo matemtico, dizemos que R um corpo ordenado completo). Utilizando a observao que precede o exerccio, vamos tentar calcular f ( x 2 ). Se a a 2 , f (a a 2 ) 0 (pois f injetora), logo

1 1 1 1 1 = = f = f + 2 2 2 f (a) f (a ) f (a a ) a 1 a aa 1 1 1 1 + f (a 2 ) = ( f ( a)) 2 , = f + f = a 1 a f ( a ) f ( 1 a )
que vale tambm quando a = a 2 a = 0 ou a = 1. a > b a b > 0 f (a b) > 0 f (a ) > f (b), Agora, observando que conclumos verificando que, por exemplo, se f ( x 0 ) > x 0 para algum x 0 R, que se f ( x 0 ) < x 0 ento existe um q Q tal que f ( x 0 ) > q > x 0 . Porm q > x 0 f ( q) > f ( x 0 ) q > f ( x 0 ), o que absurdo. O caso f ( x 0 ) < x 0 anlogo, o que termina o problema.
EXERCCIO 2

(IMO) Encontre todas as funes f : R R tais que f ( x 2 + f ( y )) = y + f ( x) 2 . Dica: Prove que f ( x 2 ) = ( f ( x)) 2 e que f ( x + y ) = f ( x) + f ( y ), para x 0 e y R, ento conclua. Se voce no conseguir concluir, puxa!! Voc passou muito perto da resoluo.
EXERCCIO 3

(IMO) Encontre todas as funes f, definidas no conjunto dos reais no negativos e assumindo valores reais no negativos, tais que: i) f ( xf ( y )) f ( y ) = f ( x + y ) para todo x, y 0
EUREKA! N9, 2000

43

Sociedade Brasileira de Matemtica

ii) f ( 2) = 0 iii) f ( x) 0 para 0 x < 2 Dica: x 2 y x 2 f ( y ). Incrvel, no?


PONTO FIXO

Muitas vezes, til considerarmos os pontos fixos de uma funo, isto , pontos x tais que f ( x) = x. Para mostrar que esta simples considerao leva, muitas vezes, soluo do problema, observe abaixo o seguinte exemplo: (IMO) Seja S o conjunto dos reais maiores que 1. Encontre todas as funes f : S S satisfazendo as condies f ( x + f ( y ) + xf ( y )) = y + f ( x) + yf ( x), x, y S i) ii) f ( x) x estritamente crescente para 1 < x < 0 e x > 0. Para x = 0, temos f ( f ( y )) = y (1 + f (0)) + f (0), donde conclumos que f injetora. De f ( f (0)) = f (0) e da injetividade de f, conclumos que f (0) = 0. Seja x 0 um ponto fixo de f. Sabemos da condio ii) que h no mximo um ponto em cada um dos intervalos (1; 0) e (0; + ). Substituindo x = y = x 0 em i),
2 2 encontramos f x 0 + 2 x0 = x0 + 2 x0 . 2 Se + 2 x 0 (1;0), logo x 0 + 2 x 0 = x 0 , absurdo. Analogamente, no h pontos fixos em (0;+). Assim, 0 o nico ponto fixo de f. Substituindo x = y em i), temos f ( x + f ( x) + xf ( x)) = x + f ( x) + xf ( x), ou seja xf ( x) ponto fixo e, portanto, igual a 0, logo f ( x) = x (1 + x), que satisfaz i) e ii). 2 x 0 ( 1;0), x 0

EXERCCIO 4

(IMO) Encontre todas as funes f definidas no conjunto dos reais positivos e assumindo valores neste conjunto e que satisfaz as condies: f ( xf ( y )) = yf ( x) para todo x, y R * i) +; f ( x) 0 quando x +. ii)
EXERCCIO 5

(Torneio das Cidades) Mostre que no existem funes f : R R tais

f ( f ( x)) = x 2 1996. Dica: utilize pontos fixos, mas utilize mesmo!


EXERCCIO 6

(IMO) Seja N 0 o conjunto dos inteiros no negativos. Encontre todas as funes f : N 0 N 0 tais que f (m + f (n)) = f ( f (m)) + f ( n), m, n N 0 . Dica: considere o menor ponto fixo da funo.
EUREKA! N9, 2000

44

Sociedade Brasileira de Matemtica

OLIMPADAS AO REDOR DO MUNDO


O comit editorial da EUREKA! sente-se gratificado pela acolhida desta nova seo por parte dos seus leitores. Aproveitamos a oportunidade para agradecer aqueles que nos enviaram sugestes, opinies, crticas e principalmente solues para os problemas. Cumpre informar, aos leitores, que por uma questo de espao fsico as solues de todos os problemas propostos, em um exemplar de EUREKA!, no podero ser apresentadas no nmero posterior ao daquele em que foram publicados visto que a revista possui outras sees de grande interesse do pblico em geral. Entretanto, as mesmas sero divulgadas nos nmeros posteriores medida que os leitores as enviarem. Se houver interesse mais urgente na soluo de algum problema especfico, solicitamos contactar a OBM, seo OLIMPADAS AO REDOR DO MUNDO, atravs de carta ou e-mail.

Antonio Luiz Santos



Primeiramente vamos aos problemas propostos deste nmero

32. (Moldvia-1998) A seqncia

(a n ) ,

n N* verifica as relaes a1 =

1 e 2

an =

a n 1 para todo nmero natural n > 1 . Calcule a1 + a 2 + + a1998 . 2na n 1 + 1

33. (Moldvia-1999) Seja n um nmero natural tal que 2n 2 possui 28 divisores distintos e o nmero 3n 2 possui 30 divisores distintos . Qual o nmero de divisores do nmero 6n 2 ? 34. (Ucrnia-1996) A seqncia (a n ) , n 0 tal que a 0 = 1 , a100 = 0 , e para todo n 1 , tem-se que a n +1 = 2a1 a n a n 1 .
a) Mostre que a1 1 . b) Determine a1996

35. (Ucrnia-1997) Seja d (n ) o maior divisor mpar de um nmero natural n .

2n Definamos uma funo f : N N tal que f (2n 1) = 2 n e f (2n ) = n + d (n )


EUREKA! N9, 2000

45

Sociedade Brasileira de Matemtica

para todo n N. Determine todos os valores de k tais que f ( f (... f (1)...)) = 1997 onde f iterada k vezes.

36. (China-1999) Seja PQRS um quadriltero inscrito num crculo e cuja medida do ngulo PSR seja igual a 90 o . Se H e K so os ps das perpendiculares baixadas de Q sobre PR e PS respectivamente (convenientemente prolongados se necessrio). Mostre que HK divide QS ao meio. 37. (Rssia-1999) Os algarismos de um inteiro positivo A em sua representao no sistema de numerao decimal crescem da esquerda para a direita. Determine a soma dos algarismos do nmero 9 A . 38. (Japo-1999) Para um hexgono convexo ABCDEF cujos lados possuem todos medidas iguais a 1 , determine o valor mximo M e o valor mnimo m das diagonais AD , BE e CF e seus possveis conjunto de valores. 39. (Irlanda-1999) Determine todos os inteiros positivos m tais que a quarta potncia do nmero de seus divisores positivos igual a m . 40. (Irlanda-1999) Mostre que existe um nmero inteiro positivo na seqncia de Fibonacci que divisvel por 1000 . 41. (Taiwan-1999) Seja P * o conjunto de todos os nmeros primos mpares menores do que 10000 . Determine todos os nmeros primos p P * tal que para cada subconjunto S de P * , digamos, S = {p1 , p 2 , ... , p k } , com k 2 , sempre que p S , existe algum q em P * , mas no em S tal que q + 1 um divisor de ( p1 + 1)( p 2 + 1) ( p k + 1). 42. (Taiwan-1999) As alturas de um tringulo acutngulo intersectam os lados BC , AC e BC nos pontos D , E e EF intersecta BC no ponto P e a reta que passa por intersecta AC e AB em Q e R respectivamente, seja N BC tal que NQP + NRP < 180o . Prove que BN > CN . 43.
2

ABC onde AB > AC F respectivamente. Se D e paralela a EF um ponto sobre o lado

(Bulgria-1999)

Seja

um

parmetro

real

tal

que

equao

x 3 px p = 0 possui duas razes reais distintas x1 e x 2 .


EUREKA! N9, 2000

46

Sociedade Brasileira de Matemtica


2 a) Prove que 3 px1 + x 2 p > 0.

b) Determine o menor valor possvel de A = Quando ocorre a igualdade ?

p2
2 + 3p 3px1 + x2

2 + 3p 3px2 + x1

p2

44. (Bulgria-1999) Determine o menor nmero natural n tal que a soma dos 2 quadrados de seus divisores (incluindo 1 e n ) igual a (n + 3) . 45. (Bulgria-1999) Seja M o ponto mdio do lado BC de um tringulo ABC no qual CAB = 45 o e ABC = 30 o . a) Determine AMC AB BC b) Prove que AM = 2 AC 46. (Bulgria-1999) Sejam M um ponto do interior de um quadrado ABCD e A1 , B1 , C1 e D1 os pontos de interseo de AM , BM , CM e DM respectivamente com o crculo circunscrito ao quadrado ABCD . Mostre que A1 B1 C1 D1 = A1 D1 B1C1 . 47. (Ir-1999) Determine todas as funes

f ( f ( x ) + y ) = f x y + 4 f ( x ) y para todos os nmeros reais x e y .


2

f : R R

que satisfazem a

48. (Ir-1999) Em um tringulo ABC a bissetriz do ngulo BAC intersecta o lado BC no ponto D . Seja um crculo tangente a BC no ponto D e que passa pelo ponto A . Se M o segundo ponto de interseo de AC com e se BM intersecta o crculo em P , mostre que AP uma mediana do tringulo ABD . 49. (Repblicas Tcheca e Eslovaca-1999) Determine o menor nmero natural que pode ser obtido colocando-se parntesis na expresso 15 : 14 : 13 : 12 : 11 : 10 : 9 : 8 : 7 : 6 : 5 : 4 : 3 : 2 50. (Repblicas Tcheca e Eslovaca-1999) A mdia aritmtica de uma quantidade de nmeros primos distintos igual a 27 . Determine o maior nmero primo que aparece entre eles.
EUREKA! N9, 2000

47

Sociedade Brasileira de Matemtica

51. (Repblicas Tcheca e Eslovaca-1999) Mostre que para todo nmero natural n o produto 2 2 2 2 4 4 4 4 1 2 3 n um inteiro. 52. (Espanha-1998) As tangentes dos ngulos de um tringulo so inteiros positivos. Determine estes nmeros. 53. (Espanha-1998) Determine todas as funes estritamente crescentes f : N* N* tais que f (n + f (n )) = 2 f (n ) 54. (Eslovnia-1999) Seja O o centro do crculo circunscrito ao tringulo ABC . Se P e Q so os pontos mdios de AO e BC respectivamente, determine a medida do ngulo OPQ se CBA = 4 OPQ e ACB = 6 OPQ . 55. (Eslovnia-1999) Determine todos os inteiros x e y que satisfazem equao
x 3 + 9 xy + 127 = y 3 .

56. (Estnia-1999) Determine todos os valores de a tais que o valor absoluto de uma das razes da equao x 2 + (a 2 )x 2a 2 + 5a 3 = 0 seja igual a duas vezes o valor absoluto da outra raiz. 57. (Estnia-1999) Sejam O1 e O2 os centros de dois crculos que no se intersectam e de mesmo raio. Se s a reta que passa pelos seus centros e t sua tangente comum externa, considere um crculo tangente aos dois crculos nos pontos K e L e tambm tangente s retas s e t nos pontos M e P respectivamente. Determine a medida de O1O2 . Mostre ainda que os pontos M , K e N esto alinhados onde N o ponto de tangncia da reta t com o primeiro crculo. 58. (St.Petersburg-1999) 150 bolas de encher (bexigas) vermelhas , 150 azuis e 150 verdes flutuam sob o teto de um circo. Existem exatamente 13 bolas verdes dentro de cada bola azul e exatamente 5 bolas azuis e 19 bolas verdes dentro de cada bola vermelha. Mostre que algumas bolas verdes no esto contidas no interior de nenhuma das outras 449 bolas.

EUREKA! N9, 2000

48

Sociedade Brasileira de Matemtica

59. (St.Petersburg-1999) Todos os nmeros inteiros positivos no superiores a 100 so escritos em ambos os lados de 50 cartas (cada nmero escrito exatamente uma vez). Estas cartas so postas sobre uma mesa de modo que somente os nmeros que estejam virados para cima podem ser vistos. Gustavo pode escolher vrias cartas, vir-las e ento calcular a soma de todos os 50 nmeros que aparecem agora. Qual o valor mximo da soma S tal que Gustavo pode com certeza obter uma soma no inferior a S ? 60. (St.Petersburg-1999) Trs mgicos apresentam um truque entregando a uma pessoa da platia um mao de cartas numeradas com 1,2,...,2n + 1(n > 6). O espectador fica com uma das cartas e aleatoriamente distribui as restantes entre o primeiro e o segundo mgicos (cada um deles fica com n cartas) . Estes olham suas cartas (sem se comunicar um ao outro) e cada um escolhe duas cartas formando um mao (ordenado) com estas cartas e as entrega ao terceiro mgico. O terceiro mgico olha estas quatro cartas e anuncia a carta que ficou com o espectador. Explique como este truque pode funcionar.
Agora vamos aos comentrios e solues dos leitores para alguns dos problemas apresentados no nmero anterior de EUREKA!. O critrio por ns adotado para este nmero foi apresentar as solues dos problemas que foram, at o presente momento, resolvidos pelo maior nmero de leitores.

4. (Reino Unido-1998) Em um tringulo ABC , D o ponto mdio de AB e E um ponto do lado BC tal que BE = 2 EC . Sabendo que ADC = BAE determine a medida do ngulo BAC .
Enviaram solues Einstein do Nascimento Jnior (Fortaleza-CE), Geraldo Perlino Jnior (SP) e Diego Alvarez Arajo Correia (Fortaleza-CE).

Soluo de Einstein do Nascimento Jnior: Sejam = ADC = BAE e P = AE CD e tracemos pelo ponto D uma reta paralela a AE e que intersecta o lado BC no ponto Q . Como EC = EQ e DQ // PE ento AP = PD . Da, PD = PA = PC e seja PCA = PAC =
logo, PCA + PAC + PAB + PDA = 180 o 2 + 2 = 180 o + = 90 o .

7. (Rssia-1998) Existem nmeros de n algarismos M e N onde todos os algarismos de M sejam pares, todos os algarismos de N sejam mpares, cada um dos
EUREKA! N9, 2000

49

Sociedade Brasileira de Matemtica

algarismos de 0 a 9 ocorrendo exatamente uma vez entre M e N e tais que M divide N ? Enviaram solues com comentrios sobre um possvel erro no enunciado: Diego Alvarez Arajo Correia (Fortaleza-CE) e Marclio Miranda de Carvalho (Teresina PI).

Soluo de Marclio Miranda de Carvalho:

M |NM K= N absurdo!
par mpar

Concluso : No existem M e N que satisfaam s condies do problema Marclio tambm observou que como a Rssia tem grande tradio em IMOs deveria haver um erro no enunciado e que o mesmo possivelmente deva ser : ... e tais que N divide M apresentando a seguinte soluo para o novo enunciado :

M 0 + 2 + 4 + 6 + 8 2(mod 9 ) N 1 + 3 + 5 + 7 + 9 7(mod 9 ) N | M N K = M 7 K 2(mod 9) K 8(mod 9) K 8.


Mas N 13579 N K > 105 M possui mais de 5 algarismos.

Concluso : No existem M e N que satisfaam s condies do problema. 8. (Romnia-1998) O volume de um paraleleppedo 216cm 3 e a sua rea total 216cm 2 . Mostre que o paraleleppedo um cubo.
Enviaram solues Einstein do Nascimento Jnior (Fortaleza-CE), Geraldo Perlino Jnior (SP), Jos Guilherme Moreira Pinto (Juiz de Fora - MG) e Diego Alvarez Arajo Correia (Fortaleza-CE).

Soluo de Diego Alvarez Arajo Correia: Sejam a , b e c as medidas das dimenses do paraleleppedo. Pelo enunciado tem-se : abc = 216 e ab + ac + bc = 108 .
EUREKA! N9, 2000

50

Sociedade Brasileira de Matemtica

Como MA MG ,

ab + bc + ac 3 ab bc ac ento 3

108 3 2 2 (abc ) = 3 (216 ) = 36 36 36 . Como ocorre a igualdade, temos que 3 ab = bc = ca a = b = c .


13. (Irlanda-1999) Uma funo f : N N satisfaz s condies :

f (ab ) = f (a ) f (b ) se o mximo divisor comum de a e b 1, f ( p + q ) = f ( p ) + f (q ) para todos os nmeros primos p e q. Mostre que f (2 ) = 2, f (3) = 3 e f (1999 ) = 1999 .
Enviaram solues Marclio Miranda de Carvalho (Teresina PI) e Geraldo Perlino Jnior (SP).

Soluo de Marclio Miranda de Carvalho: Seja p um nmero primo mpar, ento f (2 p ) = f (2 ) f ( p ) . Como, f (2 p ) = f ( p ) + f ( p ) = 2 f ( p ) f (2) = 2 Alm disso, f (4 ) = f (2 ) + f (2 ) = 4 f (12 ) = 4 f (3) .
Por outro lado
f (12 ) = f (7 ) + f (5) f (12 ) = 2 f (2 ) + f (3) + f (2 ) + f (3) = 6 + 2 f (3) f (3) = 3 .

Finalmente,

f (5) = f (2) + f (3) = 5 f (15) = 15 f (13) = 13 f (26 ) = 26 f (23) = 23. Mas, f (13) = 13 f (11) = 11 f (33) = 33 f (31) = 31 f (29) = 29 . Logo, f (2001) = f (3) f (23) f (29) = 2001 f (1999) = 1999
14. (Sua-1999) Determine todas as funes f : R\ {0} R
satisfazendo a

1 1 f ( x ) + f = x para todos x R\ {0} . x x


Enviaram solues Marclio Miranda de Carvalho (Teresina PI) e Geraldo Perlino Jnior (SP).
EUREKA! N9, 2000

51

Sociedade Brasileira de Matemtica

Soluo de Geraldo Perlino Jnior:


Seja a R com a 0 . Fazendo x = a e x = 1 na equao dada temos a

1 1 1 1 f (a ) + f = a e a f + f (a ) = . Resolvendo-se o sistema a a a a 1 1 formado por estas duas equaes chegamos a f (a ) = a 2 + e portanto, 2 a 1 1 f (x ) = x 2 + . 2 x


15. (Sua-1999) Dois crculos intersectam-se em dois pontos M e N . Um ponto A qualquer do primeiro crculo, distinto de M e N , unido aos pontos M e N de modo que as retas AM e AN intersectam novamente o segundo crculo nos pontos B e C . Mostre que a tangente ao primeiro crculo em A paralela a BC .
Enviaram solues Einstein do Nascimento Jnior (Fortaleza-CE), Geraldo Perlino Jnior (SP) e Diego Alvarez Arajo Correia (Fortaleza-CE) .

Soluo de Einstein do Nascimento Jnior: Seja MNA = e P um ponto sobre a tangente ao primeiro crculo em A . Temos ento que MAP = = MNA e como o quadriltero MNBC inscritvel ento MNB = 180 o e da MCB = resultando em ACB = CAP AP // BC . 17. (Ucrnia-1999) Mostre que o nmero 9999999 + 1999000 composto. Soluo de Marclio Miranda de Carvalho:

9999999+1999000= 107 1+ 2 103 1 103 =


3

= 9 106 +106 1+ 2 106 103 = 3103 3103 1 + 3106 103 + 3103 1 = = 310 310 1 + 310 10 +1 10 +1 = 3103 3103 1 + 3103 1 103 +1 = = 3103 1 3103 +103 +1 = 2999 4001

)(

)(

)(

EUREKA! N9, 2000

52

Sociedade Brasileira de Matemtica

19. (Litunia-1999) Duas cordas AB e CD de um crculo intersectam-se no ponto K . O ponto A divide o arco CAD em duas partes iguais. Se AK = a e KB = b , determine a medida da corda AD .
Enviaram solues Marclio Miranda de Carvalho (Teresina PI), Einstein do Nascimento Jnior (Fortaleza-CE), e Geraldo Perlino (SP) e Geraldo Perlino Jnior (SP).

Soluo de Marclio Miranda de Carvalho:


Seja ABD = ADK = ento tem-se que os tringulos ADK e ABD so semelhantes logo, AD a + b 2 = ( AD ) = a(a + b ) AD = a (a + b ) a AD

21. (Estnia-1999) Determine o valor da expresso

1 2 1999 2000 2000 2000 f + + f + f + f + + f + f 2000 2000 2000 2000 1999 1


supondo que f (x ) =

x2 . 1 + x2

Enviaram solues Marclio Miranda de Carvalho (Teresina PI), Einstein do Nascimento Jnior (Fortaleza-CE), Geraldo Perlino Jnior (SP), Diego Alvarez Arajo Correia (Fortaleza-CE) e Gibran Medeiros de Souza (Natal-RN).

Soluo de Marclio Miranda de Carvalho: 2 1 1 2 2 2 1 x x x = x 2 = x + 1 = 1 . Logo o valor da f (x ) + f = + + 2 2 1 + x2 x2 + 1 x2 + 1 x 1+ x 1 1+ x2 x expresso 3999 2000 1999 + f = 1999 + f (1) = 2000 2

EUREKA! N9, 2000

53

Sociedade Brasileira de Matemtica

1 1 1 1 22. (Eslovnia-1999) Inicialmente os nmeros 1, , , ..., so escritos , 2 3 1998 1999 em um quadro negro. Em cada passo, escolhemos dois destes nmeros, digamos a e b, e os substitumos pelo nmero a + b + ab . Continuamos desta maneira at que reste um nico nmero no quadro negro. possvel que este nmero seja 2000 ? Justifique sua resposta. Enviaram solues Einstein do Nascimento Jnior (Fortaleza-CE) e Geraldo Perlino Jnior (SP).

Resumo da soluo de ambos com adaptaes:


Seja a b = a + b + ab . fcil ver que a b = b a e a (b c ) = (a b ) c e portanto podemos escolher os nmeros em qualquer ordem. Calculemos os resultados ento na ordem dada : 1 1 1 1 = 2, 2 = 3,...,1998 = 1999 2 3 1999 Deste modo vemos que ns sempre alcanaremos 1999 no podendo ento alcanar 2000 .

EUREKA! N9, 2000

54

Sociedade Brasileira de Matemtica

SOLUES DE PROBLEMAS PROPOSTOS Publicamos aqui algumas das respostas enviadas por nossos leitores.
42) Suponha que a, b e c so as medidas dos lados de um tringulo ABC, com semi-permetro p e rea S, verifique que 1 1 1 3 p + + a b c 2 s e mais ainda: verifique que a igualdade acima ocorre apenas se o tringulo for equiltero. Soluo de Marcelo Rufino de Oliveira (Belm-PA):
Sejam x = a + c b y = a + b c z = b + c a Pela Desigualdade Triangular temos que x > 0, y > 0 e z > 0. Assim, isoloando a, b e c: a = x + y b = y + z c = z + x Pela Desigualdade entre as Mdias Aritmtica e Geomtrica temos: x + y 2 xy , y + z 2 yz e z + x 2 zx (1)

1 1 1 Vamos desenvolver agora o valor de + + em funo de x, y e z, usando a b c para isso o resultado (1):
2 2 1 1 1 1 1 1 1 + 1 + 1 = 1 1 + 1 + 1 + + + + = 4 a b c x+ y y+ z z+x yz zx 2 xy 2 yz 2 zx xy 2 2

Pela Desigualdade de Cauchy podemos afirmar que 2 2 (a1 + a 2 + a 3 ) 2 3( a12 + a 2 + a3 ). 1 1 1 , a2 = e a3 = Fazendo ento a1 = conclumos que: zx xy yz

3 1 1 1 1 1 1 + + xy + yz + zx (2) a b c 4 p Calculando em funo de x, y e z obtemos: s p x+ y+z x+ y+z = = = s xyz ( x + y + z ) xyz


EUREKA! N9, 2000

1 1 1 + + xy yz zx

55

Sociedade Brasileira de Matemtica

Assim, usando o resultado (2):

3 1 1 1 3 p2 1 1 1 = + + + + 2 4 xy yz zx a b c 4 s

1 1 1 3 p + + a b c 2 s

(3)

Como nas duas desigualdade usadas a igualdade vale se e s se os termos so iguais, ento temos a igualdade na desigualdade (3) s e s se a = b = c.

43) Prove que se p um primo da forma 4k + 3, ento 2p + 1 tambm primo se e somente se 2p + 1 divide 2p 1. Soluo de Alex Corra Abreu (Niteri - RJ): Primeira parte:
Se 2p + 1 tambm primo, temos:

2 ( 2 p +1) 1(mod 2 p + 1) 2 2 p 1(mod 2 p + 1) 2 p 1(mod 2 p + 1), s que 2 p + 1 da forma 8k + 7, logo 2 p 1(mod 2 p + 1) 2 p + 1 2 p 1


Segunda parte:

Se 2 p 1(mod 2 p + 1), como p primo ento : p = ord 2 p +1 2, s que (2,2 p + 1 ) = 1

(2 p + 1) = kp, com k 2.
No podemos ter k = 1, pois ( n) par para todo n 3. Assim, (2 p + 1) = 2 p 2 p + 1 primo.

44) O produto de dois inteiros positivos consecutivos pode ser igual ao produto de dois inteiros positivos consecutivos pares?

EUREKA! N9, 2000

56

Sociedade Brasileira de Matemtica

Soluo de Daniel Pessa Martins Cunha (Fortaleza - CE):


Seja A o produto de dois nmeros inteiros positivos consecutivos. Isto implica que A = x( x + 1) onde x Z . Observe que: - Caso x seja par temos: ( x 2) x < x( x + 1) < x( x + 2) ( x 2) x < A < x( x + 2) (Esta desigualdade fcil de ser vista) - Caso x seja mpar temos: (Logo x + 1 par) ( x 1)( x + 1) < x( x + 1) < ( x + 1)( x + 3) ( x 1)( x + 1) < A < ( x + 1)( x + 3) (desigualdade fcil de ser vista) Analizando os casos vemos que A est entre dois produtos consecutivos de dois inteiros pares consecutivos positivos. Logo conclui-se que no possvel que o produto de dois inteiros positivos seja igual ao produto de dois inteiros positivos consecutivos pares.
46) (Baltic Way, 1997)

i) Prove a existncia de dois conjuntos infinitos A e B, no necessariamente disjuntos, de inteiros no negativos tais que cada inteiro no negativo pode ser representado de uma nica forma como a + b, com a A e b B. ii) Prove que em cada tal par (A, B), ou A ou B contm apenas mltiplos de algum inteiro k > 1.

Soluo de Humberto Silva Naves (So Paulo - SP):


i) Todo natural se escreve da maneira nica como soma de potncias de 2 distintas, donde os conjuntos A = {n N | n se escreve como soma de potncias de 2 distintas com expoente mpar} e B = {n N | n se escreve como soma de potncias de 2 distintas com expoente par} satisfazem as condies do enunciado (note que 0 A e 0 B). ii) 0 deve pertencer aos dois conjuntos, e 1 a exatamente um deles (seno 1 = 0 + 1 = 1 + 0 seria representado de 2 formas distintas), digamos ao conjunto B. Seja k o menor elemento positivo do conjunto A. fcil ver que {0, 1, ..., k 1} B. Vamos provar que o mdc dos elementos de A k.
EUREKA! N9, 2000

57

Sociedade Brasileira de Matemtica

Para isso, vamos mostrar por induo que para cada inteiro positivo m, existem inteiros r1 ,..., rs tais que {x B | x < mk } = U {ri k , ri k + 1,..., ri k + k 1}, e todos os
i =1

elementos de A menores que mk so mltiplos de k. Consideremos o inteiro mk. Ele deve ser escrito de maneira nica como soma de um elemento de A com um podemos escrever os elementos elemento de B. Se mk A, mk , mk + 1,..., mk + k 1 Como soma de um elemento de A(mk) com um elemento de B (pertencente a {0,1,..., k 1}. Se mk A, podemos escrever mk = m + m(k ), com m A e m(k ) B. < k. Se por hiptese de induo, Devemos ter 1, m(k ), m(k )+,..., m(k ) + k 1 pertencem a B , donde, para O j k 1, mk + j = ml + (m( k ) + j ) soma de elementos de A e de B menores que mk, donde nenhum dos mk + j , 0 j k 1 pertencem a A nem a B (pela unicidade da representao, seno poderamos escrev-los como 0 + (mk + j ) ou (mk + j ) + 0), o que prova a afirmao para m + 1. Se = 0, mk B. Queremos mostrar que para 0 j k 1, mk + j pertence a B (e logo no pertence a A), provando a afirmao para m + 1. Suponha o contrrio, e considere o menor j com 0 j k 1 tal que mk + j no pertence a B. Devemos ter mk + j = x + y, com x A \ {0} e y B. Se x < mk , x = rk e e y = (m r )k + j , donde por hiptese de induo, (m r )k B mk = 0 + mk = rk + (m r )k , contradizendo a unicidade. Se x mk , como mk , mk + 1,..., mk + j 1 pertencem a B (e portanto no pertencem a A), devemos e mas nesse caso teramos ter x = mk + j y = 0, (m + 1)k = k + mk = (mk + j ) + ( k j ), contradizendo novamente a unicidade.

Agradecemos tambm o envio das solues e a colaborao de:


Jos Guilherme Moreira Pinto Osvaldo Melo Sponquiado Digo Veloso Uchoa Nijair Arajo Pinto Gibran M. de Souza Carlos Alberto da Silva Victor Einstein do Nascimento Jnior Samuel Barbosa Feitosa Geraldo Perlino Jr. (Juiz de Fora - MG) (Olmpia - SP) (Teresina - PI) (Fortaleza - CE) (Natal - RN) (Nilpolis - RJ) (Fortaleza - CE) (Fortaleza - CE) (So Paulo - SP)

EUREKA! N9, 2000

58

Sociedade Brasileira de Matemtica

PROBLEMAS PROPOSTOS
Convidamos o leitor a enviar solues dos problemas propostos e sugestes de novos problemas para os prximos nmeros.
47) Dada uma circunferncia , trace as tangentes a ela por um ponto exterior, A, tocando-a em M e N. Trace a reta r passando por A e tocando em B e C. Se D o ponto mdio de MN , prove que MN a bissetriz de BDC. 48) Doze pintores vivem em doze casas construdas ao longo de uma rua circular e so pintadas ou de branco ou de azul. Cada ms um dos pintores, pegando consigo bastante tinta branca e azul, deixa sua casa e caminha ao longo da rua no sentido anti-horrio. Desta forma, ele repinta cada casa (iniciando na sua) com a cor oposta. Finaliza o trabalho to longo repinte alguma casa branca de azul. Em um ano, cada casa estar pintada com a sua cor original sabendo que, no comeo do ano, ao menos uma casa estava pintada de azul. 49) Dado um polgono regular de n lados. Assinale aleatoriamente, no seu interior, um ponto M. Sendo x1 , x 2 ,..., x n as distncias de M a cada um dos lados, verifique que: 1 1 1 2 + + ... > , onde a a medida do lado do poligono. x1 x 2 xn a 50) Calcule o determinante: MDC (1, 1) MDC (1,2) MDC (1, n)

MDC ( 2,1) MDC (2,2) MDC ( 2, n) MDC ( n,1) MDC ( n,2) MDC ( n, n) Onde MDC (a, b) o mximo divisor comum de a e b.
51) Trs feirantes foram vender melancias. Um levou 10; outro 16; o terceiro, 26. Todos venderam algumas melancias pelo mesmo preo at o meio dia. Depois disso, os trs baixaram o preo, mas continuaram vendendo por preos iguais. Quando voltaram para casa, aps venderem todas as melancias, cada um tinha a mesma quantia de dinheiro; 35 mil cruzeiros. Por quanto foi vendida cada melancia antes e aps o meio-dia?
Problema 47 proposto por Carlos Lucas de Melo Pontes e Silva (Fortaleza - CE), problemas 48 e 51 propostos por Jorge Luis Rodrigues Costa (Fortaleza - CE) e problemas 49 e 50 propostos por Carlos A. Gomes (Natal - RN).
EUREKA! N9, 2000

59

Sociedade Brasileira de Matemtica

COORDENADORES REGIONAIS
Amarsio da Silva Arajo (UFV) Viosa - MG Alberto Hassen Raad (UFJF) Juiz de Fora - MG Angela Camargo (Centro de Educ.de Adultos - CEA) Blumenau - SC Benedito T. Vasconcelos Freire (UFRN) Natal - RN Claudio Arconcher (Col. Leonardo da Vinci) Jundia - SP Claus Haetinger (UNIVATES) Lajeado - RS Cleonor Crescncio das Neves (UTAM) Manaus-AM lio Mega (Col. ETAPA) So Paulo - SP Ktia Gonalves de Faria (Col. Singular) Santo Andr - SP Florncio F. Guimares Filho (UFES) Vitria - ES Francisco Dutenhefner (UFMG) Belo Horizonte - MG Gisele de A. Prateado Gusmo (UFGO) Goinia - GO Ivanilde H. Fernandes Saad (U. Catlica Dom Bosco) Campo Grande - MS Jacqueline F. Rojas Arancibia (UFPB) Joo Pessoa - PB Joo Bencio de Melo Neto (UFPI) Teresina - PI Joo F. Melo Libonati (Grupo Educ. IDEAL) Belm - PA Irene Nakaoka (UEM) Maring - PR Jos Carlos Pinto Leivas (UFRG) Rio Grande - RS Jos Cloves Saraiva (UFMA) So Luis - MA Jos Gaspar Ruas Filho (ICMC-USP) So Carlos - SP Jos Luis Rosas Pinho (UFSC) Florianpolis - SC Jos Paulo Carneiro (Univ. Santa rsula) Rio de Janeiro - RJ Jos Vieira Alves (UFPB) Campina Grande - PB Marcelo Rufino de Oliveira (Sistema Titular de Ensino)Belm - PA Licio Hernandes Bezerra (UFSC) Florianpolis - SC Luzinalva M. de Amorim (UFBA) Salvador - BA Marcondes Cavalcante Frana (UF Cear) Fortaleza - CE Pablo Rodrigo Ganassim (L. Albert Einstein) Piracicaba - SP Paulo H. Cruz Neiva de L. Jr. (Esc. Tec.Everardo Passos) SJ dos Campos - SP Reinaldo Gen Ichiro Arakaki (INPE) SJ dos Campos - SP Ricardo Amorim (Centro Educ. Logos) Nova Iguau - RJ Roberto Vizeu Barros (Colgio ACAE) Volta Redonda - RJ Srgio Cludio Ramos (IM-UFRGS) Porto Alegre - RS Seme Gebara Neto (UFMG) Belo Horizonte -MG Silvio de Barros Melo (UFPE) Recife - PE Tadeu Ferreira Gomes (U. do Estado da Bahia) Juazeiro - BA Toms Menndez Rodrigues (U. Federal de Rondonia) Porto Velho - RO Valdenberg Arajo da Silva (U. Federal de Sergipe) So Cristovo - SE Wagner Pereira Lopes (Esc. Tec. Fed. de Gois) Jata - GO Waldemar M. Canalli (P.M. S. Joo de Meriti) S. Joo de Meriti - RJ

EUREKA! N9, 2000

60

Sociedade Brasileira de Matemtica

CADASTRAMENTO 2001
Colgios (Preencher com letra de forma)

Instituio: Pblica Diretor: Endereo: Bairro: Cidade: Cep: Telefone: ( Fax: ( e-mail: Privada

Estado: ) )

Professor Responsvel: Endereo: Bairro: Cidade: Cep: Telefone: ( Fax: ( e-mail:

Estado: ) )

Para seguir participando Olimpada Brasileira de Matemtica, uma cpia desta ficha deve ser preenchida e enviada para a Secretaria da Olimpada Brasileira de Matemtica pelos colgios ainda no recadastrados.

EUREKA! N9, 2000

61

     "!# $% &% ' ()' $%* (%*+(' , ./103234 568795*9:034 ;<:=359>7@?A/CB 6*53B /C7*D 79:5   "!# $% &% ' ()' $%* (%*+(' , ./103234 568795*9:034 ;<:=359>7"9:5F3;3G>37*D 739H5   "!# $% &% ' ()' $%* (%*+(' , ./103234 568795*9:034 ;<:=359>78I 5/CJ:53B /C7*D 739:5   "!# $% &% ' ()' $%* (%*+(' , L59:;34 I 73>309 MNO%PCQRS TU8V %' , VT' ) X5/C6*73G08Y/CB > ( *!# $' % Z'[$'$\*]T%$ (]^'_V`$`!V&'(* %`!V!3V !&b* %!%!V3 dT%$ (# 8!), ,$%T% '$%d T ' 

 E33E -3K

-3W KW a  ac  ef  eE 

Sociedade Brasileira de Matemtica

g hjikl mh

Realizamos durante o ano 2000 a XXII Olimpada Brasileira de Matemtica, atingindo na realizao da Primeira Fase cerca de 80.000 alunos. Este ano a Olimpada Brasileira de Matemtica ter importantes modificaes em relao aplicao das provas da segunda e terceira fases (para maiores esclarecimentos leia com ateno o nosso novo regulamento publicado no site: http://www.obm.org.br/). Alm disso a competio contar pela primeira vez com a participao de alunos de ensino superior, para os quais foi criado o nvel Universitrio. Assim, a partir deste ano, a OBM passa a ser realizada em 4 nveis de competio. O calendrio para este ano o seguinte:

nVoqp%r%sqtvu(wx
yz

{%|~}y}|~
(y Sbado, 09 de junho t y(
(y % 01 de setembro y(|y}|~
(y Sbado,Sbado, 20 de outubro (nveis 1, 2 e 3)
Domingo, 21 de outubro (Nveis 2 e 3 Segundo dia de prova).

nVoqp%r%VnVsqp%r%Vt%sqVsq

t%y([(y

{%|}qy(}q|(y

Sbado, 01 de setembro Sbado, 20 de outubro e Domingo, 21 de outubro

Gostaramos de registrar a realizao da IV Semana Olmpica. Neste ano o evento teve lugar no Colgio Militar de Salvador (Salvador BA) entre os dias 19 a 26 de janeiro. Aproveitamos a oportunidade para expressar o nosso agradecimento pela calorosa acolhida. Durante a IV Semana Olmpica, reunimos os alunos premiados na XXII OBM nos trs nveis de competio. Um arquivo com as aulas ministradas durante o evento pode ser consultado no seguinte endereo eletrnico: http://www.obm.org.br/semana.htm Por fim, queremos agradecer aos alunos que tm nos ajudado com a reviso da revista bb%*

ky(}q|y(
EUREKA! N10, 2001

 llikl g g m g h
lik
lm g j g Zl g     [ q 1
1. Observe as multiplicaes a seguir:
12 345 679 18 = 222 222 222 12 345 679 27 = 333 333 333 12 345 679 54 = 666 666 666

Sociedade Brasileira de Matemtica

Para obter 999 999 999 devemos multiplicar 12 345 679 por: A) 29 B) 99 C) 72 D) 41

E) 81

2. Outro dia ganhei 250 reais, incluindo o pagamento de horas extras. O salrio (sem horas extras) excede em 200 reais o que recebi pelas horas extras. Qual o meu salrio sem horas extras? A) 200 reais B) 150 reais C) 225 reais D) 175 reais E) 180 reais 3. Num relgio digital, que marca de 0:00 at 23:59, quantas vezes por dia o mostrador apresenta todos os algarismos iguais?

A) 10

B) 8

C) 6

D) 7

E) 9

4. A prefeitura de uma certa cidade fez uma campanha que permite trocar 4 garrafas de 1 litro vazias por uma garrafa de 1 litro cheia de leite. At quantos litros de leite pode obter uma pessoa que possua 43 dessas garrafas vazias? A) 11 B) 12 C) 13 D) 14 E) 15 5. Numa caixa havia vrias bolas, sendo 5 azuis, 4 amarelas, 3 vermelhas, 2 brancas e 1 preta. Renato retirou 3 bolas da caixa. Sabendo que nenhuma delas era azul, nem amarela, nem preta, podemos afirmar a respeito dessas 3 bolas que: A) so da mesma cor. B) so vermelhas. C) uma vermelha e duas so brancas. D) uma branca e duas so vermelhas. E) pelo menos uma vermelha. 6. Se a rea do retngulo dado 12, qual a rea da figura sombreada?

A) 3

B) 4

C) 5

D) 6

E) 8

EUREKA! N10, 2001

Sociedade Brasileira de Matemtica

7. O nmero 10 pode ser escrito de duas formas como soma de dois nmeros primos: 10 = 5 + 5 e 10 = 7 + 3. De quantas maneiras podemos expressar o nmero 25 como uma soma de dois nmeros primos? A) 4 B) 1 C) 2 D) 3 E) nenhuma 8. 1 litro de lcool custa R$0,75. O carro de Henrique percorre 25 km com 3 litros de lcool. Quantos reais sero gastos em lcool para percorrer 600 km? A) 54 B) 72 C) 50 D) 52 E) 45 9. Um certo nmero N de dois algarismos o quadrado de um nmero natural. Invertendo-se a ordem dos algarismos desse nmero, obtm-se um nmero mpar. A diferena entre os dois nmeros o cubo de um nmero natural. Podemos afirmar que a soma dos algarismos de N : A) 7 B) 10 C) 13 D) 9 E) 11 10. Juliano colou uma bandeirinha cinza em cada engrenagem, como mostra a figura abaixo:

As engrenagens so iguais e quando a engrenagem da esquerda girou um pouco, a sua bandeirinha ficou na posio indicada com a bandeirinha branca pontilhada. Nesta condio, podemos afirmar que a posio da bandeirinha na engrenagem da direita :
a)
A)

b)
B)

c)
C)

d)
D)

e)
E)

11. Uma fbrica embala 8 latas de palmito em caixas de papelo cbicas de 20 cm de lado. Para que possam ser melhor transportadas, essas caixas so colocadas, da melhor maneira possvel, em caixotes de madeira de 80 cm de largura por 120 cm de comprimento por 60 cm de altura. O nmero de latas de palmito em cada caixote A) 576 B) 4.608 C) 2.304 D) 720 E) 144 12. H 18 anos Hlio tinha precisamente trs vezes a idade de seu filho. Agora tem o dobro da idade desse filho. Quantos anos tm Hlio e seu filho?
EUREKA! N10, 2001

Sociedade Brasileira de Matemtica

A) 72 anos e 36 anos. D) 50 anos e 25 anos.

B) 36 anos e 18 anos. E) 38 anos e 19 anos.

C) 40 anos e 20 anos.

13. Se os nmeros naturais so colocados em colunas, como se mostra abaixo, debaixo de que letra aparecer o nmero 2000?
A 1 10 18 19 20 B 9 11 17 21 C 2 D 8 12 16 ... E 3 F 7 13 15 ... G 4 H 6 14 I 5

A) F

B) B

C) C

D) I

E) A

14. O emir Abdel Azir ficou famoso por vrios motivos. Ele teve mais de 39 filhos, incluindo muitos gmeos. De fato, o historiador Ahmed Aab afirma num dos seus escritos que todos os filhos do emir eram gmeos duplos, exceto 39; todos eram gmeos triplos, exceto 39; todos eram gmeos qudruplos, exceto 39. O numero de filhos do emir : A) 111 B) 48 C) 51 D) 78 E) 75 15. Quatro amigos vo visitar um museu e um deles resolve entrar sem pagar. Aparece um fiscal que quer saber qual deles entrou sem pagar. Eu no fui, diz o Benjamim. Foi o Pedro, diz o Carlos. Foi o Carlos, diz o Mrio. O Mrio no tem razo, diz o Pedro.

S um deles mentiu. Quem no pagou a entrada do museu? A) Mrio B) Pedro C) Benjamim E) no possvel saber, pois faltam dados D) Carlos

16. Em um jogo de duas pessoas, os jogadores tiram, alternadamente, 1, 2, 3, 4 ou 5 palitos de uma pilha que inicialmente tem 1000 palitos. Ganha o jogador que tirar o ltimo palito da pilha. Quantos palitos o jogador que comea deve tirar na sua jogada inicial de modo a assegurar sua vitria? A) 1 B) 2 C) 3 D) 4 E) 5 17. Quantos nmeros inteiros e positivos menores do que 1.000.000 existem cujos cubos terminam em 1? A) 1.000 B) 10.000 C) 50.000 D) 100.000 E) 500.000
EUREKA! N10, 2001

Sociedade Brasileira de Matemtica

18. Os 61 aprovados em um concurso, cujas notas foram todas distintas, foram distribudos em duas turmas, de acordo com a nota obtida no concurso: os 31 primeiros foram colocados na turma A e os 30 seguintes na turma B. As mdias das duas turmas no concurso foram calculadas. Depois, no entanto, decidiu-se passar o ltimo colocado da turma A para a turma B. Com isso: A) A mdia da turma A melhorou, mas a da B piorou. B) A mdia da turma A piorou, mas a da B melhorou. C) As mdias de ambas as turmas melhoraram. D) As mdias de ambas as turmas pioraram. E) As mdias das turmas podem melhorar ou piorar, dependendo das notas dos candidatos. 19. Escrevem-se, em ordem crescente, os nmeros inteiros e positivos que sejam mltiplos de 7 ou de 8 (ou de ambos), obtendo-se 7, 8, 14, 16, ... . O 100o nmero escrito : A) 406 B) 376 C) 392 D) 384 E) 400 20. A figura abaixo foi desenhada em cartolina e dobrada de modo a formar um cubo.

Qual das alternativas mostra o cubo assim formado?

A)

B)

C)

D)

E)

\ T# 
1) E 2) C 3) B 4) D 5) E
EUREKA! N10, 2001

6) D 7) B 8) A 9) D 10) A

11) A 12) A 13) C 14) C 15) B

16) D 17) D 18) C 19) E 20) B

 llikl g g m g h
lik
lm g j g Zl g     [ q 1:
1. 2. 3. 4. 5. Veja o problema 17 do nvel 1. Veja o problema 11 do nvel 1. Veja o problema 10 do nvel 1. Veja o problema 15 do nvel 1. Veja o problema 18 do nvel 1.
B A

Sociedade Brasileira de Matemtica

6. No tringulo ABC representado ao lado, a 60 e a bissetriz do Medida do ngulo C ngulo B forma 70 com a altura relativa ao : vrtice A. A medida do ngulo A A) 50 B) 30 C) 40 D) 80 E) 70

7. Veja o problema 6 do nvel 1. 8. Alberto, Beatriz e Carlos correm numa pista circular. Todos saem ao mesmo tempo e do mesmo lugar, cada um desenvolvendo velocidade constante. Alberto e Beatriz correm no mesmo sentido. Correndo no sentido oposto, Carlos encontra Alberto, pela primeira vez, exatamente 90 segundos aps o incio da corrida e encontra Beatriz exatamente 15 segundos depois. Quantos segundos so necessrios para que Alberto ultrapasse Beatriz pela primeira vez? A) 105 B) 630 C) 900 D) 1.050 E) no pode ser determinado 9. DEFG um quadrado no exterior do pentgono regular ABCDE. Quanto mede o ngulo EF? A) 9o B) 12o C) 15o D) 18o E) 21o 10. Quantos so os nmeros inteiros de 2 algarismos que so iguais ao dobro do produto de seus algarismos? A) 0 B) 1 C) 2 D) 3 E) 4 11. Veja o problema 19 do nvel 1. 12. Uma caixa contm 900 cartes, numerados de 100 a 999. Retiram-se
EUREKA! N10, 2001

Sociedade Brasileira de Matemtica

ao acaso (sem reposio) cartes da caixa e anotamos a soma dos seus algarismos. Qual a menor quantidade de cartes que devem ser retirados da caixa, para garantirmos que pelo menos trs destas somas sejam iguais? A) 51 B) 52 C) 53 D) 54 E) 55 13. Se x e y so nmeros reais positivos, qual dos nmeros a seguir o maior? A) xy B) x + y
2 2

C) (x + y)

x3 + y3 D) x + y(x + y) E) x+ y
2

14. Na figura, as distncias entre dois pontos horizontais consecutivos e as distncias entre dois pontos verticais consecutivos so iguais a 1. A regio comum ao tringulo e ao quadrado tem rea:

14 15 a a +1 15. Sejam a e b nmeros reais positivos tais que < 1. Ento b b +1 a a a A) igual a + 1. B) igual a . C) menor que . b b b a D) maior que mas menor que 1. E) pode ser maior que 1. b
A) B) C) D) E) 16. Veja o problema 16 do nvel 1. 17. Quantos so os retngulos que tm os pontos A e B como vrtices, e cujos vrtices esto entre os pontos de interseo das 9 retas horizontais com as 9 retas verticais da figura abaixo?

9 10

15 16

8 9

11 12

A) 3
EUREKA! N10, 2001

B) 4

C) 7

D) 2

E) 5

Sociedade Brasileira de Matemtica

18. Veja o problema 14 do nvel 1. 19. De Itacimirim a Salvador, pela estrada do Coco, so 60 km. s 11 horas, a 15 km de Salvador, d-se um acidente que provoca um engarrafamento, que cresce velocidade de 4 km/h, no sentido de Itacimirim. A que horas, aproximadamente, devemos sair de Itacimirim para chegar a Salvador ao meiodia, sabendo que viajamos a 60 km/h, exceto na zona de engarrafamento, onde a velocidade 6 km/h? A) 10h43min B) 10h17min C) 10h48min D) 10h53min E) 11h01min 20. Colocamos em ordem crescente os nmeros escritos nas casas brancas do tabuleiro a seguir (estamos mostrando apenas as suas quatro primeiras linhas). Assim, por exemplo, o nono nmero da nossa lista 14. Qual o 2000o nmero da nossa lista? 1 3 7 13

A) 3931

10

5 11

2 6 12

4 8 14

9 15

16

E) 3939

B) 3933

C) 3935

D) 3937

\ T# 
1) D 2) A 3) A 4) B 5) C
EUREKA! N10, 2001

6) D 7) D 8) B 9) A 10) B

11) E 12) C 13) C 14) D 15) D

16) D 17) E 18) C 19) A 20) D

 llikl g g m g h
lik
lm g j g Zl g     [ q 1
1. Veja o problema 13 do nvel 2. 2. Veja o problema 9 do nvel 2. 3. Veja o problema 14 do nvel 2. 4. Escrevemos uma lista com todos os nmeros inteiros de 1 a 30, inclusive. Em seguida, eliminamos alguns destes nmeros de forma que no sobrem dois nmeros tais que um seja o dobro do outro. Qual a quantidade mxima de inteiros que podem permanecer na lista? A) 15 B) 18 C) 19 D) 20 E) 21 5. Veja o problema 15 do nvel 2. 6. Seja f uma funo real que tem as seguintes propriedades: i) Para todos x, y reais, f(x + y) = x + f(y); ii) f(0) = 2. Quanto vale f(2000)? A) 0 B) 2 C) 1998 D) 2000

Sociedade Brasileira de Matemtica

E) 2002

7. H trs cartas viradas sobre uma mesa. Sabe-se que em cada uma delas est escrito um nmero inteiro positivo. So dadas a Carlos, Samuel e Toms as seguintes informaes: i) todos os nmeros escritos nas cartas so diferentes; ii) a soma dos nmeros 13; iii) os nmeros esto em ordem crescente, da esquerda para a direita. Primeiro, Carlos olha o nmero na carta da esquerda e diz: No tenho informaes suficientes para determinar os outros dois nmeros. Em seguida, Toms olha o nmero na carta da direita e diz: No tenho informaes suficientes para determinar os outros dois nmeros. Por fim, Samuel olha o nmero na carta do meio e diz: No tenho informaes suficientes para determinar os outros dois nmeros. Sabendo que cada um deles sabe que os outros dois so inteligentes e escuta os comentrios dos outros, qual o nmero da carta do meio? A) 2 B) 3 C) 4 D) 5 E) No h informaes suficientes para determinar o nmero.
EUREKA! N10, 2001

10

Sociedade Brasileira de Matemtica

8. Veja o problema 16 do nvel 2. 9. Veja o problema 12 do nvel 2. 10. A notao x significa o maior inteiro que no supera x. Por exemplo, 3,5 = 3 e 5 = 5. O nmero de inteiros positivos x para os quais A) 11

x + x = 10 :
1 2 1 3

B) 12

C) 13

D) 14

E) 15

11. Veja o problema 20 do nvel 2. 12. Veja o problema 18 do nvel 1. 13. A figura abaixo mostra o logotipo de uma empresa, formado por dois crculos concntricos e por quatro crculos de mesmo raio, cada um deles tangente a dois dos outros e aos dois crculos concntricos. O raio do crculo interno mede 1 cm. Ento o raio do crculo externo dever medir, em cm:

A) 2 2 + 3

B)

2 +2

C) 4 2 + 1

D) 3 2

E)

2 +1

14. Veja o problema 8 do nvel 2. 15. Veja o problema 10 do nvel 2. 16. Dois nadadores, inicialmente em lados opostos de uma piscina, comeam simultaneamente a nadar um em direo ao outro. Um deles vai de um lado a outro da piscina em 45 segundos e o outro em 30 segundos. Eles nadam de um lado para outro por 12 minutos, sem perder qualquer tempo nas viradas. Quantas vezes eles passam um pelo outro (indo no mesmo sentido ou em sentidos opostos) durante este tempo, contando as vezes em que se encontram nos extremos da piscina. A) 10 B) 12 C) 15 D) 18 E) 20 17. A soma de dois nmeros naturais 29. O mnimo valor para a soma de seus quadrados : A) 785 B) 733 C) 647 D) 421 E) 334 18. Veja o problema 1 do nvel 2. 19. Veja o problema 17 do nvel 2.
EUREKA! N10, 2001

11

Sociedade Brasileira de Matemtica

20. Veja o problema 10 do nvel 1. 21. Na figura temos que os tringulos ABC e ABC so equilteros e a regio destacada um hexgono regular. A razo entre a rea da regio destacada e a rea do tringulo ABC igual a:
A C B O B C A

A) 1

2 B) 3

4 C) 5

D)

2 2

E)

3 2

22. Veja o problema 14 do nvel 1. 23. Veja o problema 19 do nvel 2. 24. Seja P(x) = a2000x2000 + a1999x1999 + a1998x1998 + + a1x + a0. Ento a2000 + a1998 + a1996 + + a0 igual a A)

D) P(0) P(1)

P(1) P(1) 2

B)

P(1) + P(1) 2

C) P(2000) + P(1998) + + P(0)

E) P(1) P(1)

25. Quantos nmeros de trs algarismos (que no comeam com 0) possuem um algarismo que a mdia aritmtica dos outros dois? A) 121 B) 117 C) 112 D) 115 E) 105

\ T# 
1) C 2) A 3) D 4) D 5) D
EUREKA! N10, 2001

6) E 7) C 8) D 9) C 10) E

11) D 12) C 13) A 14) B 15) B

16) E 17) D 18) D 19) E 20) A

21) B 22) C 23) A 24) B 25) A

12

 llikl g g m g h
lik
lm g j g Zl g   333 [ 1[ !&b* 
De quantas maneiras diferentes podemos construir um paraleleppedo usando exatamente 24 blocos cbicos de medidas 1 1 1? Obs: Blocos de dimenses 2 3 4 e 2 4 3 devem ser considerados iguais.

Sociedade Brasileira de Matemtica

!&b* 

O retngulo ao lado est dividido em 9 quadrados, A, B, C, D, E, F, G, H e I. O quadrado A tem lado 1 e o quadrado B tem lado 9.
C

G F A H E

Qual o lado do quadrado I?

!&b* 

!&b* 

Pintamos de vermelho ou azul 100 pontos em uma reta. Se dois pontos vizinhos so vermelhos, pintamos o segmento que os une de vermelho. Se dois pontos vizinhos so azuis, pintamos o segmento de azul. Finalmente, se dois pontos vizinhos tm cores distintas, pintamos o segmento de verde. Feito isto, existem exatamente 20 segmentos verdes. O ponto na ponta esquerda vermelho. possvel determinar com estes dados a cor do ponto na ponta direita? Em caso afirmativo, qual a cor deste ponto?

Desejamos escrever os inteiros de 1 a 10 nas casas do desenho ao lado de tal forma que quaisquer quatro nmeros alinhados aparecem em ordem crescente ou decrescente. a) Mostre uma maneira de dispor os nmeros respeitando estas condies. b) Quais nmeros podem aparecer nas pontas da estrela? c) Quais nmeros podem aparecer nas outras cinco posies?

!&b* 
Qual o menor inteiro positivo que o dobro de um cubo e o quntuplo de um quadrado?

EUREKA! N10, 2001

13

!&b* 

Sociedade Brasileira de Matemtica

Qual o maior inteiro positivo n tal que os restos das divises de 154, 238 e 334 por n so iguais?

SRv%"v'SMSQv"kMbMkS\*1

Sejam a b c as dimenses do paraleleppedo. Temos que a, b, c * e


3

(]^'%$!V&* 3

abc = 24. Como abc a.a.a a 24, temos a 2, ou seja a = 1 ou a = 2 . Se a = 1, bc = 24. As possibilidades para b e c so b = 1 e c = 24; b = 2 e c = 12; b = 3 e c = 8; b = 4 e c = 6. Se a = 2, bc = 12. As possibilidades para b e c com b 2 so b = 2 e c = 6; b = 3 e c = 4. Assim, h 6 maneiras de construirmos o paraleleppedo.

(]^'%$!V&* 3

O quadrado A medida de lado 1cm enquanto que o quadrado B tem medida de lado 9cm. Da que as longitudes dos lados dos quadrados restantes so: C = 10cm E = 8cm. F = 7cm G = 4cm. D = 14cm. I = 18cm.

(]^'%$!V&* 3

Temos que os segmentos verdes dividem os pontos da reta em conjuntos de pontos com cores iguais, sendo que o primeiro conjunto esquerda contm pontos vermelhos, o segundo conjunto contm pontos azuis, o terceiro conjunto contm pontos vermelhos, e assim por diante. Como h 20 segmentos verdes, temos 21 conjuntos de pontos. Assim, como o 21 conjunto contm pontos vermelhos, o ponto na ponta direita vermelho.

(]^'%$!V&* 3
6 1 4 7

10

10

3 5

3 6

1) 1 e 2 ocupam pontas vizinhas. fcil ver que colocando o 2 no meio ou em uma ponta "oposta"a 1 o problema no tem soluo.
EUREKA! N10, 2001

14

Sociedade Brasileira de Matemtica

2) 9 e 10 ocupam pontas vizinhas. Pelo mesmo raciocinio anterior. 3) Uma vez que 1 e 2 esto colocados o 3 est no meio, entre o 1 e o 2. Observe que colocar o 3 em qualquer outra posio leva a um absurdo. 4) Uma vez que 1, 2 e 3 esto colocados, fica claro que o 4 vizinho ao 3. 5) Se 1, 2, 3 e 4 j esto colocados, 5 pode estar no meio ou em uma ponta, e o mesmo ocorre com o 6. (ver figuras) Quando um deles est numa ponta, o outro est no meio. 6) O 7 est no meio. Respostas: a) Ver figuras b) 1, 2, 9 e 10 obrigatrios mais 5 ou 6. c) 3, 4, 7, 8 obrigatrios mais 5 ou 6.

(]^'%$!V&* 3

Decomponha N em primos = 2 a2 3 a3 ... Dobro de um cubo quer dizer que todos os ai so mltiplos de 3 exceto a2 que deixa resto 1 na diviso por 3. Quntuplo de um quadrado quer dizer que todos so pares exceto a5. Os menores expoentes possveis so ento a2 = 4; a5 = 3 e os outros a3 = a7 =...= 0. Resposta: N = 24 53 = 2000.

(]^'%$!V&* 3

Dois nmeros deixam o mesmo resto quando divididos por n se e s se sua diferena mltipla de n. Logo, as diferenas 238 154 = 84 e 334 238 = 96 so ambas mltiplas de n. Como n o maior possvel, conclumos que n deve ser o maior divisor comum de 84 e 96, que 12.

EUREKA! N10, 2001

15

 llikl g g m g h
lik
lm g j g Zl g   333 [ 1H !&b* 
Qual o menor inteiro positivo que o dobro de um cubo e o quntuplo de um quadrado?

Sociedade Brasileira de Matemtica

!&b* 

De quantas maneiras diferentes podemos construir um paraleleppedo usando exatamente 216 blocos cbicos de medidas 1 1 1? Obs: Blocos de dimenses 2 3 36 e 2 36 3 devem ser considerados iguais.

!&b* 

C G E

No retngulo ABCD, E o ponto mdio do lado BC e F o ponto mdio do lado CD. A interseo

de DE com FB G. O ngulo EAF mede 20 . Quanto vale o ngulo EGB ?


A B

!&b* 

O retngulo ao lado est dividido em 9 quadrados, A, B, C, D, E, F, G, H e I. O quadrado A tem lado 1. Qual o lado do quadrado I?
C

G F

!&b* 
nmeros restantes
134 . Determine n e m. 11

A B E

Listamos os inteiros de 1 a n. Desta lista apagamos o inteiro m. A mdia dos n 1

!&b* 

O campeonato Venusiano de futebol disputado por 10 times, em dois turnos. Em cada turno cada equipe joga uma vez contra cada uma das outras. Suponha que o Vulcano FC vena todas as partidas do 1o. turno. Caso no vena o 2o. turno, o Vulcano FC jogar uma final contra o vencedor do 2o. turno, na qual ter vantagem caso faa mais pontos que o adversrio durante todo o campeonato (vitria vale 3 pontos, empate vale 1 ponto e derrota 0 pontos).
EUREKA! N10, 2001

16

Sociedade Brasileira de Matemtica

a) Determine o menor n tal que, se o Vulcano FC fizer exatamente n pontos no segundo turno, garantir pelo menos a vantagem na final (independente de contra quem e com que placares conquiste os n pontos). b) Determine o menor n tal que, se o Vulcano FC fizer pelo menos n pontos no segundo turno, garantir pelo menos a vantagem na final (independente de contra quem e com que placares conquiste os n pontos).

SRv%"v'SMSQv"kMbMkS\*1 (]^'%$!V&* 3 (]^'%$!V&* 3


Veja a soluo do problema 5 do nvel 1.

Sejam a b c as medidas do paraleleppedo. Temos ento que a, b e c so inteiros positivos e abc = 216 . Como a b c a a a a 6 e a | 216, temos a = 1, a = 2, a = 3, a = 4 ou a = 6. Se a = 1, temos b c = 216. As possibilidades neste caso so b = 1 e c = 216; b = 2 e c = 108; b = 3 e c = 72; b = 4 e c = 54; temos a = 2, b = 6 e c = 36; b = 8 e c = 27; b = 9 e c = 24; b = 12 e c = 18. Se Temos ento as possibilidades b c = 108, com b 2. b = 2 e c = 54; b = 3 e c = 36; b = 4 e c = 27; b = 6 e c = 18; b = 9 e c = 12. Se a = 3, temos b c = 72, com b 3. Temos ento as possibilidades b = 3 e c = 24; b = 4 e c = 18; b = 6 e c = 12; b = 8 e c = 9. Se a = 6, a nica soluo b = c = 6. Temos, assim, 19 maneiras de construirmos o paraleleppedo. Observao: pode-se verificar que o nmero de solues de b.c = r, com
b c naturais, d (n ) , onde [x ] denota o menor nmero inteiro maior ou igual
2

d (216 ) a x e d (n ) o nmero de divisores de n. Assim, b c = 216 tem =8

solues; b c = 108 com b 2 tem

d (108 ) solues 2 1= 5

(descontamos
d (72 ) 2 2=4

aqui a soluo b = 1 e c = 108 ); b c = 72 com b 3 tem

solues (eliminamos b = 5 e c = 72 e b = 2 e c = 36 ); b c = 54 com b 4 tem d (54 ) 3 = 1 soluo (eliminamos b = 1, b = 2 e b = 3 ) e b c = 36


d (36 ) com b 6 tem soluo (elimina-se b = 1, 2 ,3 ou 4). 2 4 =1
2

EUREKA! N10, 2001

17

(]^'%$!V&* 3
D y F G

Sociedade Brasileira de Matemtica

1) F A D = F B C = x x + y = 70 2) E A B = E D C = y
^ ^
^

E x B

x
20

D E C = 90 y = + x
o

90 ( x + y ) = = 20

(]^'%$!V&* 3
Seja x o lado de B. O lado de C = x 1, D = x + 5, E = x 1, F = x 2, G = 4, H = 2x 3, I = x + 9 (=D + G) mas tambm 3x 9 (=F + H G). Assim x + 9 = 3x 9 e x = 9. Assim, o lado de I 18.

(]^'%$!V&* 3

A mdia aritmtica dos inteiros de 1 a n (n + 1)/2. Quando se apaga um destes nmeros, a menor mdia possvel a dos nmeros de 1 a (n 1), que n / 2, e a maior a dos nmeros de 2 a n, que n/2 +1. Logo, deve-se ter
2 n n 4 4 e, portanto, n igual a 23 ou < 12 < + 1 o que fornece 22 n 24 11 11 2 11 2

24. Mas a mdia dos nmeros restantes uma frao de denominador 11. Logo, a quantidade de nmeros que restam no quadro deve ser mltipla de 11. Portanto, n s pode ser igual a 23. Finalmente, a soma dos nmeros que restam 22 x 12 2/11 = 268.A soma dos nmeros de 1 a 23 23 x 12 = 276. Logo, o nmero apagado foi m = 276 268 = 8.

(]^'%$!V&* 3
No pior caso, o 2o. colocado do 1o. turno faz 24 pontos no 1o. turno. Se o Vulcano FC fizer 23 pontos no 2o. turno, ele ganhar 7 jogos e empatar 2, e o 2o. colocado no 1o. turno chegar a um mximo de 25 pontos (pois no mximo empatar com o Vulcano FC) no segundo turno. Assim, o Vulcano FC ter vantagem na deciso, nesse caso. Note que se o Vulcano FC fizer 24 pontos no 2o. turno perdendo para o 2o. colocado do 1o. turno, este pode fazer 27 pontos no 2o. turno e ganhar a vantagem para a deciso. Se o Vulcano FC fizer 22 pontos ou menos e o Klingon FC tiver feito 24 pontos no 1o. turno poder fazer 27 pontos no 2o. turno, somando 51 pontos, mais que os 49 (ou menos) pontos do Vulcano FC. Assim, a resposta da segunda pergunta n = 25, enquanto a resposta da 1a. pergunta n = 23.

EUREKA! N10, 2001

18

 llikl g g m g h
lik
lm g j g Zl g   333 [ 1 !&b*  !&b*  !&b*  !&b*  !&b* 
O nmero Veja o problema 1 do nvel 2. Veja o problema 4 do nvel 2.

Sociedade Brasileira de Matemtica

O trapzio ABCD tem bases AB e CD. O lado DA mede x e o lado BC mede 2x. A soma dos ngulos DAB e ABC 120o. Determine o ngulo DAB . Veja o problema 6 do nvel 2.

1 1 1 1 1 1 + 2 + 1 + 2 + 2 + ... + 1 + + racional; 2 2 1 2 2 3 2000 20012 p escreva-o na forma , p e q inteiros. q 1+

!&b* 

Para efetuar um sorteio entre os n alunos de uma escola (n > 1) se adota o seguinte procedimento. Os alunos so colocados em roda e inicia-se uma contagem da forma "um, DOIS, um, DOIS,...". Cada vez que se diz DOIS o aluno correspondente eliminado e sai da roda. A contagem prossegue at que sobre um nico aluno, que o escolhido. a) Para que valores de n o aluno escolhido aquele por quem comeou o sorteio? b) Se h 192 alunos na roda inicial, qual a posio na roda do aluno escolhido?

SRv%"v'SMSQv"kMbMkS\*1 (]^'%$!V&* 3 Veja a soluo do problema 1 do nvel 2. (]^'%$!V&* 3 Veja a soluo do problema 4 do nvel 2. (]^'%$!V&* 3
Tracemos DM // BC (vide figura abaixo). Como AMD = ABC e DAM + AMD = DAM + ABC = 120 tem-se que ADM = 60. Como AD = x e BC = 2x, sendo P o ponto mdio de DM, ento, AD = DP = x e ADP um tringulo equiltero, isto , AP = x. Portanto APM um tringulo
EUREKA! N10, 2001

19

Sociedade Brasileira de Matemtica

issceles com PAM = AMP e como DPA um ngulo externo do tringulo APM temos 60 = DPA = PAM + AMP = 2. AMP = 2. ABC. Portanto, ABC =30 e DAB =120 ABC = 90.
D C

(]^'%$!V&* 3 (]^'%$!V&* 3
S= =
2000

Veja a soluo do problema 6 do nvel 2.

a =1

1+

a =1

2000 1 1 a 4 + 2 a 3 + 3a 2 + 2 a + 1 + = 2 a 2 (a + 1)2 a 2 (a + 1) a =1

2000

1 a 2 + a + 1 2000 = 1 + 2 +a a2 + a a a =1
2000 a =1

= 2000 +

= 2000 + 1 + + ... + 2 2 3 a a + 1 2000 2001 1 2000 = 2000 + 2001 2001

= 2000 + 1

('%V* 3

a) Para que o primeiro da fila seja o escolhido preciso, antes de mais nada, que haja um nmero par de alunos (caso contrrio, ele ser eliminado quando comear a segunda rodada). Mais precisamente, o primeiro da fila o escolhido se e s se, a cada rodada, a fila tem um nmero par de alunos. Portanto, o primeiro da fila escolhido se e s se o nmero de alunos uma potncia de 2. b) Como 192 = 26 . 3, nas primeiras 6 rodadas a fila tem um nmero par de alunos. Aps estas 6 rodadas, a fila se reduz a trs alunos e fcil verificar que o escolhido o terceiro deles. Resta, portanto, determinar quem so os alunos que restam aps as primeiras 6 rodadas. Na primeira rodada, sobrevivem 1, 3, 5, 7, ..., 191. De um modo geral, sobrevivem rodada de ordem n (n = 1, 2, ..., 6) os nmeros da forma 2n . k + 1. Portanto, aps 6 rodadas os sobreviventes so 1, 65 e 129 e o aluno escolhido o de nmero 129.

EUREKA! N10, 2001

20

vk



j  

Sociedade Brasileira de Matemtica

 "!$#%#&$ '$()$*#%+*!,-./0$1 !2 !$#& 3547698$;:

b*5<
Paulo tem trs dados comuns idnticos nos quais a soma dos nmeros em duas faces opostas sempre igual a 7. Ele cola os dados, de modo que cada par de faces coladas tenha o mesmo nmero, e depois os coloca sobre uma mesa no transparente, conforme indica a figura. A soma dos nmeros em todas as onze faces visveis 36. Qual a soma dos nmeros das trs faces que esto em contato com a mesa?

b*5=
Um tringulo equiltero pode ser recortado em tringulos equilteros menores. A figura abaixo mostra como recortar um tringulo equiltero em 7 tringulos equilteros. Mostre como recortar um tringulo equiltero em 20 tringulos equilteros menores.

b*5>
Isabel tem dois baralhos, cada um com 50 cartas. Em cada um dos baralhos esto escritos os nmeros de 1 a 100 (em cada carta esto escritos dois nmeros, um em cada face da carta). Por um defeito de fabricao, a distribuio dos nmeros nas cartas no a mesma nos dois baralhos (por exemplo, em um dos baralhos o 1 aparece na mesma carta do 2; no outro, o 1 aparece com o 76). Mostre como Isabel deve fazer para que, ao colocar as 100 cartas sobre uma mesa, as faces voltadas para cima mostrem todos os nmeros de 1 a 100.

b*5?

Considere a seguinte tabela 5 5, preenchida com os nmeros de 1 a 25.


1 6 11 16 21 2 7 12 17 22 3 8 13 18 23 4 9 14 19 24 5 10 15 20 25

EUREKA! N10, 2001

21

Sociedade Brasileira de Matemtica

Em cada fileira horizontal e em cada fileira vertical, trocamos o sinal de 2 nmeros, de forma que, em cada fileira horizontal e em cada fileira vertical, haja 3 nmeros positivos e 2 nmeros negativos. Somamos, ento, todos os nmeros da tabela. Calcule os possveis valores dessa soma.

@ACBEDFG%H;@JIKMLNHOPQH;RSOKUTVKW@ HCXY[Z]\^HB`_

b*5< Veja a soluo do problema 1 do nvel 2.

b*5=(('Z`;a%%"a%`%;bc5dCfehgi(kj
Esta uma forma de dividir um tringulo equiltero em 20 tringulos equilteros menores. Dividindo o maior tringulo em 4 partes (tringulos equilteros menores), dois de esses quatro menores tringulos em nove outros menores tringulos equilteros, obteremos 20 tringulos equilteros: 2 maiores e 18 menores.

b*5> Veja a soluo do problema 2 do nvel 2. b*5?(('ZEefl%


0+1 5+1 10 + 1 15 + 1 20 + 1

Escrevemos os nmeros da tabela na seguinte forma:


0+2 5+2 10 + 2 15 + 2 20 + 2 0+3 5+3 10 + 3 15 + 3 20 + 3 0+4 5+4 10 + 4 15 + 4 20 + 4 0+5 5+5 10 + 5 15 + 5 20 + 5

Cada nmero da forma 5a + b, com 0 a 4 e 1 b 5. Depois de trocar de sinal temos que em cada linha h dois nmeros negativos, se em cada linha fazemos a soma s das partes 5a temos que a soma dessa linha 5a (j que h trs 5a e dois 5a ) e a soma de todas as linhas considerando somente os nmeros 0 +5 +10 + 15 + 20 = 50. Agora consideremos os nmeros b . Em cada coluna h dois nmeros b que trocaram de sinal e trs que no, portanto a soma dos nmeros dessa coluna b e a soma das colunas considerando somente as partes b , : 1 + 2 + 3 + 4 + 5 = 15. Logo a soma total sempre 65.
EUREKA! N10, 2001

22

vk



j  

Sociedade Brasileira de Matemtica

 "!$#%#&$ '$()$*#%+*!,-./0$1 !2 !$#& 3547698$nm

b*5< b*5=

Veja o problema 1 do nvel 1.

Isabel tem dois baralhos, cada um com 50 cartas. Em cada um dos baralhos esto escritos os nmeros de 1 a 100 (em cada carta esto escritos dois nmeros, um em cada face da carta). Por um defeito de fabricao, a distribuio dos nmeros nas cartas no a mesma nos dois baralhos (por exemplo, em um dos baralhos o 1 aparece na mesma carta do 2; no outro, o 1 aparece com o 76). Mostre como Isabel deve fazer para que, ao colocar as 100 cartas sobre uma mesa, as faces voltadas para cima mostrem todos os nmeros de 1 a 100.

b*5>

Em uma folha de papel a reta r passa pelo canto A da folha e forma um ngulo com a borda horizontal, como na figura 1. Para dividir este ngulo em trs partes iguais, executaremos as seguintes construes: a) inicialmente, marcamos dois pontos B e C sobre a borda vertical de modo que AB = BC; pelo ponto B traamos a reta s paralela borda (figura 2); b) a seguir, dobramos o papel, ajustando-o de modo que o ponto C coincida com um ponto C sobre a reta r e o ponto A coincida com um ponto A sobre a reta s (figura 3); chamamos de B o ponto com o qual B coincide. Mostre que as retas AA e AB dividem o ngulo em trs partes iguais.
r r C C B A A A s C B B A

b*5?

Figura 1

Figura 2

Figura 3

possvel encontrar duas potncias de 2, distintas e com o mesmo nmero de algarismos, tais que uma possa ser obtida atravs de uma reordenao dos dgitos da outra?
EUREKA! N10, 2001

23

@ACBEDFG%H;@JIKMLNHOPQH;RSOKUTVKW@ HCXY[Z]\^HBQo b*5< %Z'bqpr' l E s '%5d9efb % g ;t7bq i fuvj


Sejam a e b os nmeros das faces coladas. Como a o nmero da face oposta face de b, no dado central, temos que a + b = 7. A soma dos nmeros das faces de cada dado 21, ento a soma dos nmeros das faces de todos os dados 63, mas a soma das faces coladas 2 (a + b) = 14, e a soma das faces visveis 36, temos, ento, que a soma de nmeros das faces em contato com a mesa : 63 36 14 = 13. Resposta: a soma 13.

Sociedade Brasileira de Matemtica

b*5= %wckb g'"x%fb)^ykb e%d18x7b%wlfz(i[{j

Podemos fazer isso pegando qualquer carta de qualquer baralho, colocando sobre a mesa e vendo seu verso. Depois disso procuramos a carta de mesmo nmero do verso (procurando no outro baralho, j que foi usada no primeiro baralho). Fazemos com esta carta o mesmo que foi feito com a primeira carta. Continua-se a fazer isso at fechar um ciclo (um mesmo nmero que j saiu em um baralho sair no outro). Exemplo:
baralho: 1 x verso: y 2 y z 1 z w 2 w x Ciclo fechado.

Quando um ciclo for fechado pega-se outra carta e comea um novo ciclo. Fazendo isso at o final das cartas as faces voltadas para cima mostraro todos os nmeros de 1 a 100.

b*5> Veja soluo do problema 1 do nvel 3. b*5 ?(('Z`'8%%'V5 |k b g'V5 d9|(}g'bkz5ihlkj

Sejam A = 2 m e A' = 2 n onde A' uma reordenao dos dgitos de A, suponha sem perda de generalidade que A > A' , da A um mltiplo de A' pois possui os mesmos fatores primos. Ento temos que A = A'k , onde k Z , k > 1 (pois caso no fosse A' = A , o que seria um absurdo, pois A e A' so distintos.) e k uma potncia de dois, pois A s possui fatores primos iguais a 2, da k = 2,4,8 A = 2 A' ou A = 4 A' ou A = 8 A'. Como a soma de seus dgitos a mesma, A e A' deixam o mesmo resto (mod 9) e sua diferena divisvel por 9, mas A A' s pode se: A', 3A', 7A', onde nenhuma desas diferenas divisvel por 9. Da no existem tais nmeros. Nenhum nmero divisvel por 9, pois cada um desses nmeros no possui pelo menos dois fatores primos iguais a 3.
EUREKA! N10, 2001

24

vk



j  

Sociedade Brasileira de Matemtica

~V}h h{0vh]nv}hWh% hqh % hq {;-0q

b*5< b*5=

Veja o problema 3 do nvel 2.

Seja (n) a soma de todos os divisores positivos de n, onde n um inteiro positivo (por exemplo, (6) = 12 e (11) = 12). Dizemos que n quase perfeito se (n) = 2n 1 (por exemplo, 4 quase perfeito, pois ( 4) = 7). Sejam n mod k o resto da diviso de n por k e s( n) = n mod k (por exemplo: s(6) = 0 + 0 + 0 +
k =1 n

2 + 1 + 0 = 3 e s(11) = 0 + 1 + 2 + 3 + 1 + 5 + 4 + 3 + 2 + 1 + 0 = 22). Prove que n quase perfeito se, e somente se, s( n) = s (n 1).

b*5>

Seja f uma funo definida nos inteiros positivos da seguinte forma: Dado n, escrevemos n = 2 a (2b + 1), com a e b inteiros e definimos f (n) = a 2 + a + 1. Determine o menor inteiro positivo n tal que f (1) + f (2) + ... + f (n) 123456.

b*5?

A avenida Providncia tem infinitos semforos igualmente espaados e sincronizados. A distncia entre dois semforos consecutivos de 1.500m. Os semforos ficam abertos por 1 min 30s, depois fechados por 1 min, depois abertos por 1 min 30s e assim sucessivamente. Suponha que um carro trafegue com velocidade constante igual a v, em m/s, pela avenida Providncia. Para quais valores de v possvel que o carro passe por uma quantidade arbitrariamente grande de semforos sem parar em qualquer um deles?

b*5

Seja X o conjunto de todas as seqncias a = (a1 , a 2 ,..., a 2000 ) tais que a i {0,1,2} se 1 i 1000 e a i {0,1} se 1001 i 2000. Dados a e b em X, definimos a distncia d (a, b) entre a e b como sendo o nmero de valores de i, 1 i 2000, tais que a i bi . Determine o nmero de funes f : X X que
EUREKA! N10, 2001

25

Sociedade Brasileira de Matemtica

preservam distncia, isto , tais que d ( f (a ), f (b)) = d (a, b), para quaisquer a e b em X.

b* 
Seja C um cubo de madeira. Para cada um dos 28 pares de vrtices de C cortamos o cubo C pelo plano mediador dos dois vrtices do par. Em quantos pedaos fica dividido o cubo? Nota: Dados dois pontos A e B no espao, o plano mediador de A e B o conjunto dos pontos do espao cujas distncias a A e B so iguais. Em outras palavras: o plano perpendicular ao segmento AB passando pelo ponto mdio de AB.

@ACBEDFG%H;@JIKMLNHOPQH;RSOKUTVKW@ HCXY[Z]\^HBQ

b*5<(('Z
8%}g;k%ffuZ
8V'%d]|}g'(kz("i}lkj

r C B B A P K X A

A' = ento Veja que AP = A' P , ento AA' P issceles. Seja PA ' B = ( BA' // AP). AA Note que APX A' PX , Da: P = XA ' P XA + XAA' = 2 A' = XA Agora observe que: X = 90 2 BX A = 2 BA
EUREKA! N10, 2001

26

Sociedade Brasileira de Matemtica

X = 90 2 B ' X A' = 2 BA Donde segue que os pontos A, X, B' so colineares. Como CB = C ' B ' , AB = A' B ' e CB = AB, temos que C ' B ' = A' B ' . Ento AB' mediana e altura do C ' AA' , sendo, conseqentemente, bissetriz do B' = B' A A' = PA A' = . C ' AA'. Da: C ' A

b*5=(('Z{|(%; lfb)\kb Vkb)%ke%kckb)wd]|hgkz5ihl%kj


n n

Fixe n. Seja a i = n mod i e bi = (n 1) mod i Temos s (n) = a i e s( n 1) = bi


i =1 i =1

Veja

que

se

d n,

por

definio,

a d = 0,

que

n 0(mod d ) n 1 1(mod d ) bd = d 1 (j que 0 bd d 1) (inclusive se d = 1) Alm disso se t / \ n, a t > 0 e fcil ver que bt = at 1. Sendo assim:

s(n) = s(n 1) ai = bi ad + at + an = bd + bt (bd ad ) =


i =1 i =1 d |n d n /n t\ d|n d n /n t\ d |n d n

n1

= (at bt ) (d 1) = 1
|n t/ d |n d n |n t/

Seja f(n) o nmero de divisores de n. Temos:


d|n d n

(d 1) = 1 d 1 = 1 ( (n) n) ( f (n) 1) = n f (n) (n) n f (n) +1 =


|n t/ d|n d n d|n d n |n t/

= n f ( n ) ( n ) = 2 n 1. De modo que s (n) = s (n 1) (n) = 2n 1.

b*5>(('Z`'}b *kb %`'%f%fd]|hg(kz("i}l'kj

Considere as representaes binrias dos nmeros, ex: 17 = (10001); 24 = (11000) e 5 = (101) Seja n na base 2 igual a (...a i ...a 3 a 2 a1 a 0 ), onde a i = 0 ou a i = 1, i Z + se 2 j > n a j = 0.

EUREKA! N10, 2001

27

Sociedade Brasileira de Matemtica

n = 2 a (2b + 1) a a quantidade de zeros direita na sua representao binria. Exemplo: a para o 24 3, j a = 0 para o 17 e o 5. Isto vem exatamente do que significa a representao de um nmero em uma dada base. (*) Seja S k = f (1) + f (2) + f (3) + ... + f (2 k ) Como a s depende da quantidade de zeros no final (*), temos que se 2 j > n, n 1 ento f (2 j + n) = f (n), pois tero a mesma quantidade de zeros direita na base 2. Assim, S k = f (1) + f (2) + ... f (2 k 1 1) + f (2 k 1 ) + f (2 k 1 + 1) + f (2 k 1 + 2) + ... + f (2 k ) S k = ( f (1) + f (2) + ... + f (2 k 1 1) + f (2 k 1 )) + ( f (1) + f (2) + ... + f (2 k 1 1)) + f (2 k )

S k = ( S k 1 ) + ( S k 1 f (2 k 1 )) + f (2 k )

S k = S k 1 + S k 1 + (k 1) 2 (k 1) 1 + k 2 + k + 1 S k = 2 S k 1 + 2 k S k = 2 ( S k 1 + k ). Primeiros Sk's: S0 = 1, S1 = 4, S2 = 12, S3 = 30, S4 = 68, S5 = 146, S6 = 304, S7 = 622, S8 = 1260, S9 = 2538, S10 = 5096, S11 = 10214, S12 = 20452, S13 = 40930, S14 = 81888, S15 = 163806 Seja g (n) = f (1) + f (2) + ... + f (n), provaremos que g ( n) = a i S i , onde
i =1 +

] [

n = (...a j ...a 5 a 4 a 3 a 2 a1 a 0 ). Seja j o maior possvel tal que a j = 1. n = 2 j + a j 1 2 j 1 + ... + a1 2 + a 0 2 0

g(n) = ( f (1) + f (2) + ...+ f (2 j )) + ( f (2 j +1) + f (2 j + 2) + ...+ f (2 j + a j1 2 j1 + ...a0 ))


g (n) = ( S j ) + f (1) + f (2) + ... + f (a j 1 2 j 1 + ... + a 0 )) De modo anlogo, tomamos o maior j 0 , tal que j > j 0 e a j0 = 1.

g(n) = S j + ( f (1) + f (2) + f (3) + ... + f (2 j0 )) + ( f (2 j0 +1) + ...+ f (2 j0 + a j0 1 2 j0 1 + ... + a0 ))


g ( n) = S j + ( S j0 ) + ( f (1) + f ( 2) + ... + f (a j0 1 2 j0 1 + ... + a 0 ))

De maneira anloga, fazemos (vamos baixando) para todos os ai's = 1.

EUREKA! N10, 2001

28

Sociedade Brasileira de Matemtica

Como a i = 1 ou a i = 0, podemos escrever g (n) = a i S i


i =1

Para termos o menor n, tal que g (n) 123456 Temos que conseguir uma soma de S k 's 123456, com os menores k's possveis, pois isto se refletir em (...a i ...a 2 a1 a 0 ) com os menores i's possveis. Mas isto uma tarefa fcil se tomarmos os Sk's calculados na pgina anterior e tambm sabendo que:
S k > 2 S k 1 > S k 1 + 2S k 2 ... > S k 1 + S k 2 + S k 3 + ... + S 0

Da, temos que a soma procurada :


S14 + S13 + S 7 + S 2 + S1 = 81888 + 40930 + 622 + 12 + 4 = 1234456

Assim, o menor n tal que g ( n) 123456 (110000010000110)2


n = 214 + 213 + 2 7 + 2 2 + 21 = 16384 + 8192 + 128 + 4 + 2

n = 24710 O menor inteiro positivo tal que f (1) + ... + f (n) 123456 24710.

kff *5?} }f;f Eefl%


Suponha que no tempo 0 os sinais se abram e que o carro passe pela primeira vez por um sinal no tempo t 0 0 (mediremos o tempo sempre em segundos). Os sinais estaro abertos entre os tempos 150k e 150k + 90 e fechados entre os tempos 150k + 90 e 150(k + 1), para todo inteiro k. O carro passar pelos sinais 1500 nos tempos t 0 + r , para todo inteiro no negativo r . Assim, a condio v necessria e suficiente para que o carro encontre sempre o sinal aberto que t0 10 3 + r seja igual a um inteiro mais um nmero entre 0 e para todo r 150 v 5 10 inteiro. Isso claramente possvel se inteiro (com qualquer t 0 entre 0 e 90) v 10 e se a metade de um inteiro mpar (com qualquer t 0 entre 0 e 15). v Vamos mostrar que esses so os nicos casos possveis.
EUREKA! N10, 2001

29

Sociedade Brasileira de Matemtica

Primeiro mostraremos que

10r igual a um inteiro mais um nmero pertencente v

10 2 = j + , com j inteiro e [0,1). Se a 0, para algum r0: seja v 5 2 0 < < , tomamos r0 = 1. 5 1 3 1 Se < < ,2 = 1 + , com 0 < < , e tomamos r0 = 2. 2 5 5 3 Se < < 1, tomamos = 1 e k inteiro tal que k < 1 < ( k + 1) . Como 5 2 3 < , temos k > , e podemos tomar r0 = k . 5 5 2 1 3 4 Se < , temos < 2 < 1, e podemos proceder como no caso anterior, 5 2 5 5 tomando = 1 2 e r0 = 2k . Para finalizar, vamos mostrar que, nesses casos, existe k inteiro positivo tal que t0 10 3 + k igual a um inteiro mais um elemento de ,1. 150 v 5 10 2 De fato, existe m inteiro tal que r0 = m + , com 0 < < , e existem e j 5 v t0 t0 inteiros com + < j + ( + 1) , donde 150 150 t0 t 10 3 r0 = lm + 0 + = ( m + j 1) + , onde < 1 < < 1. + 150 v 150 5 20 Assim as possveis velocidades so v = m / s, para cada inteiro positivo k. k

kff *5} }f;f w g kb c5efc$d] fr fh ih j

Vamos observar um caso particular primeiro: Sabemos que: d ( f (0,0,0,...,0), f (1,0,0,...,0)) = 1 e d ( f (1,0,0,...,0), f (2,0,0,0,...,0)) = 1 e d ( f (2,0,0,...,0), f (0,0,0,...,0)) = 1 Seja A = f (0,0,...,0), B = f (1,0,0,...,0)
EUREKA! N10, 2001

30

Sociedade Brasileira de Matemtica

e C = f (2,0,0,...,0) A = ( a1 , a 2 ,..., a 2000 ) e B = (b1 , b2 ,..., b2000 ) e C = (c1 , c 2 ,..., c 2000 ) Deve existir um nico i1 e 1 i1 2000, tal que: a i1 bi1 , vamos provar que i1 1000. Deve existir um nico i1 ' tal que bi1 ' ci1 ' e se fosse i1 i1 ' teramos que d ( A, C ) = 2, um absurdo, logo i1 = i1 ' Logo temos: A = (..., a i1 ,...) B = (..., bi1 ,...) e como a i1 bi1 ci1 a i1 logo i1 1000. Vamos provar que se: x 0 = f (0, a 2 ' , a 3 ' ,..., a 2000 ' ) = ( x1 , x 2 ,..., x 2000 ) ento xi1 = a i1 . Suponhamos por absurdo que xi1 a i1 (por simetria, consideramos xi1 = bi1 ) Se

d ( A; x0 ) = m,

ento

d ( B; x 0 ) = m + 1,

pois

B = f (1,0,0,0,0,...,0) e

A = f (0,0,0,...,0)

x0 = f (0, a 2 ' , a 3 ' ,..., a 2000 ' ) mas d ( B, x 0 ) = m 1 (pois

xi1 = bi1 ) que um absurdo, logo xi1 = a i1 Analogamente verificamos que se

x1 = f (1, b2 ' , b3 ' ,..., b2000 ' ) = ( y1 , y 2 ,..., y 2000 ) ento y i1 = bi1
Vamos generalizar o argumento (ns s fizemos para o 1o. termo): Teorema 1: Seja

At = f (0,0,...,0,0,...) = (a1 , a 2 ,..., a 2000 ) Bt = f (0,0,...,1,0,...) = (b1 , b2 ,..., b2000 ) C t = f (0,0,...,2,0,...,0) = (c1 , c 2 ,..., c 2000 ) onde t 1000 : e alteramos apenas o t-simo termo no domnio. Ento se x' = f ( x1 ,..., x 2000 ) = ( y1 , y 2 ,..., y 2000 )
y it = a it se xt = 0 (onde it posio que muda de At para Bt) y it = bit se xt = 1 y it = cit se xt = 2 Demonstrao: Anloga anterior (basta trocar algumas variveis e copiar a demonstrao acima).
EUREKA! N10, 2001

31

Sociedade Brasileira de Matemtica

claro que i1 , i 2 ,..., i1000 so todos distintos. Na verdade (i1 ,..., i1000 ) uma permutao de (1,2,...,1000). Consideramos agora as seguintes 2000-uplas. A j = f (0,0,...,0) = (a1 , a 2 ,..., a 2000 ) B j = f (0,0,...,1,0,0,0,...,0) = (b1 ,..., b2000 ) onde j > 1000 e colocamos o 1 na j-sima posio. Sabemos que d ( A j , B j ) = 1 t

tal que:

a t bt e esse t nico! claro que t > 1000 (pois se fosse t < 1000, existiria w 1000 tal que i w = t , um absurdo, pois o valor de posio wi da imagem determinado exclusivamente pelo valor da posio w da 2000-upla do domnio da funo t (devido ao teorema 1).

Vamos chamar esse t de i j , assim como fizemos anteriormente. Seja x' = f ( x1 , x j ,..., x 2000 ) = ( y1 ,..., y j ,..., y 2000 ) De forma semelhante anterior, mostramos que: y i j = a i j se x j = 0
y i j = bi j se x j = 1

Para contar o nmero de funes f : X X , basta contar o nmero de permutaes de {1,2,...,1000} vezes o nmero de permutaes de {1001,...,2000} (3!)1000 (2!)1000 que 1000! 1000! 121000 pois para determinarmos uma funo f : X X basta escolher: (i1 ,..., i 2000 ) que uma permutao de (1,2,...,1000) e (i1001 ,..., i 2000 ) que uma permutao de (1001,...,2000) e escolher os valores apropriados de (a it , bit , c it ), para 1 t 1000 (1000 permutaes de {0,1,2}) e de (a it , bit ), para
1001 t 2000 (1000 permutaes de {0, 1}).

EUREKA! N10, 2001

32

kffVk } }f;ff l; bqg;b fevhgfe% E d]b g'fx fqi uvj


Plano mediador de dois vrtices adjacentes (PMVA). Existem 12 arestas, logo so 12 pares de vrtices adjacentes, mas 4 pares possuem o mesmo plano mediador. Portanto so 12 : 4 = 3 planos.

Sociedade Brasileira de Matemtica

Plano mediador de dois vrtices opostos de uma face (PMVOF).

Plano mediador de dois vrtices opostos (PMVO).

Repare que todos os planos mediadores juntos determina em cada face a seguinte figura: Como o centro do cubo interseo de todos os PMs e todas as intersees entre retas da figura ao lado so extremidades das intersees entre PMs, ao ligarmos as intersees entre PMs, teremos vrias pirmides cujo vrtice comum o centro do cubo e as bases so os tringulos da face. Como so 16 6 = 96 tringulos no total, o cubo fica dividido em 96 pirmides.

PMVO

PMVA PMVOF

EUREKA! N10, 2001

33

EEh${E7E.E7r$E{$ fqv} Sqhh kv <Nd] qq v j


NOME Maricy Miki Hisamoto Yuriy Thallickson Bincovsky Guilherme Rohden Echelmeier Gabriel Tavares Bujokas Heitor Silva Lima Lacerda Camila Santos Costa Thoms Yoiti Sasaki Hoshina Jlia Ribeiro Lamardo Augusto Ossamu Shitani Thiago de Paula Garcia Caixeta Vitor Rezende Faria Vitor Humia Fontoura Adriano Jorge Braun Vieira Neto Mariana de Camargo Penteado Luiz Mller Floris Uyttenhove Raul Mximo Alexandrino Nogueira Rud Moreira de Lima e Silva Fabio Eigi Imada Blandina Lavor Barbosa Bezerra Felipe Sanches Varroni Nicoli Gavassa Rafael Fonseca de Campos Bruna Aguilar Trotta Daniel Folador Rossi Max Douglas Peixoto da Silva Tiago Abreu Tavares de Sousa Paulo Andr Carvalho de Melo Eduardo Martins de Figueiredo Joo Marcos Nobuo Umetsu Hansen Gabriel Salvagno Adalberto Delgado Neto Gilberto Marques Arsiolli Felipe Leon Peres Camargo Shalders Martin Alexander Barrios Gundelach Louise Nagashima Omi dipo Martins Spoli Andr Ikeda Canto Paulo Henrique Macera Lucas de Barros Navarro Ana Paula Seno Pinheiro Eduardo Fischer Germano Bezerra de Menezes Pinho Wagner Silveira Aniceto Gabryel Melo Lutz Diego Frade Bernardes tila Pereira Ricarte Frederico de Souza Frydman Pedro Thiago Ezequiel de Andrade Hugo Siqueira Robert Pinto Brbara Gomes Arabe Saraiva Domingos Gomes de Aguiar Neto Fernanda Mary Sonoki Carolina Lisboa Borgo Raphael Rodrigues Mata Gil Henriques Gustavo Schmidt Joau e Silva Anderson Cipriano de Lima Rafael Santos Correia de Arajo Paulo Henrique Gonalves dos Santos Hanna Kirihara e Silva Luciana Salomo Vilar Thas Viveiro Raffaello Couto Caser CIDADE ESTADO So Paulo SP Curitiba PR Itaja SC So Paulo SP Fortaleza CE Salvador BA Rio de Janeiro RJ So Paulo SP So Paulo SP Colatina ES Goinia GO Salvador BA Fortaleza CE So Paulo SP Vitria ES Vitria ES Fortaleza CE Una MG S.J. dos Campos SP Vila Velha ES So Paulo SP So Paulo SP Atibaia SP Belo Horizonte MG So Mateus ES Fortaleza CE Campina Grande PB Rio de Janeiro RJ Vitria ES Jundia SP Jundia SP Fortaleza CE Trs Lagoas MS Vitria ES Rio de Janeiro RJ So Paulo SP Curitiba PR Curitiba PR S.J. dos Campos SP Salvador BA Ourinhos SP Encantado RS Fortaleza CE Campo Grande MS Goinia GO Fortaleza CE Fortaleza CE Salvador BA Fortaleza CE Fortaleza CE Santos SP Fortaleza CE So Paulo SP Montanha ES Salvador BA Vassouras RJ Juiz de Fora MG Jaboato dos G. PE Salvador BA So Paulo SP Florianpolis SC So Carlos SP So Paulo SP Vitria ES PRMIO Ouro Ouro Ouro Ouro Ouro Prata Prata Prata Prata Prata Prata Prata Prata Prata Prata Bronze Bronze Bronze Bronze Bronze Bronze Bronze Bronze Bronze Bronze Bronze Bronze Bronze Bronze Bronze Bronze Bronze Bronze Bronze Meno Honrosa Meno Honrosa Meno Honrosa Meno Honrosa Meno Honrosa Meno Honrosa Meno Honrosa Meno Honrosa Meno Honrosa Meno Honrosa Meno Honrosa Meno Honrosa Meno Honrosa Meno Honrosa Meno Honrosa Meno Honrosa Meno Honrosa Meno Honrosa Meno Honrosa Meno Honrosa Meno Honrosa Meno Honrosa Meno Honrosa Meno Honrosa Meno Honrosa Meno Honrosa Meno Honrosa Meno Honrosa Meno Honrosa Meno Honrosa

Sociedade Brasileira de Matemtica

EUREKA! N10, 2001

34

EE-0${E7E$EC{$E$-7 Ok$N $EX N $7qo5 $$* $.


NOME Alex Corra Abreu Fabio Dias Moreira Larissa Cavalcante Queiroz de Lima Rafael Daigo Hirama Samuel Barbosa Feitosa Jorge Peixoto de Morais Neto Israel Franklim Dourado Carrah Davi Mximo Alexandrino Nogueira Vitor Gabriel Kleine Juliana Gomes Varela Ayran Ayres Barbosa Loriato Daniel Haanwickel Junqueira Raquel Lamboglia Guimares Paulo Roberto Sampaio Santiago Telmo Luis Correa Jr. Adalberto Studart Neto Joo Marcos da Cunha Silva Thiago Costa Leite Santos Kiyoshi Horie Filho Antnio Monteiro Guimares Jr. Adriano Brasileiro Silva Renato Mendes Coutinho Diogo dos Santos Suyama Gustavo Ferruzzi Martucci Henry Wei Cheng Hsu Thiago Pinheiro Faury Otaclio Torres Vilas Boas Vitor Sarmento Mesquita Felipe Netto de Santana Patricia Akemi Komura Regiane Cristina Yamanari Guilherme Honda Saito Milton Eiji Kato Vinicius Antonio Batagello Thiago Mizuta Daniel Costa Garcia Henrique Castro Noronha Raphael Henrique Ribas Dafne de Albuquerque Simo Michel Renato Manzolli Ballestero Fernanda Ramos Correia Fernando Santos Simes Ferreira Lucas Lolli Saui Luciano Lacerda Silveira Samara Anny Maia Fava Gustavo Eufrsio Farias Vinicius Figueiredo de Castro Lus Eduardo de Godoi Helder Seiji Kato Mrcio Jun Hisamoto Vinicius Augusto Paccola Erika Famini Silva CIDADE ESTADO Niteri RJ Rio de Janeiro RJ Fortaleza CE Campinas SP Fortaleza CE Goinia GO Fortaleza CE Fortaleza CE Mogi das Cruzes SP Fortaleza CE Vitria ES Salvador BA Fortaleza CE Salvador BA Santo Andr SP Fortaleza CE Fortaleza CE So Paulo SP Ourinhos SP Campina Grande PB Fortaleza CE Americana SP Belo Horizonte MG Piracicaba SP So Paulo SP So Paulo SP Salvador BA Fortaleza CE Rio de Janeiro RJ So Paulo SP Guararapes SP So Paulo SP So Paulo SP Araatuba SP So Paulo SP Goinia GO Valinhos SP Curitiba PR Fortaleza CE Araraquara SP Salvador BA Vitria ES Florianpolis SC Campo Grande MS Fortaleza CE Fortaleza CE Rio de Janeiro RJ S.J. dos Campos SP So Paulo SP So Paulo SP Mato SP Salvador BA PRMIO Ouro Ouro Ouro Ouro Prata Prata Prata Prata Prata Prata Prata Prata Prata Prata Prata Bronze Bronze Bronze Bronze Bronze Bronze Bronze Bronze Bronze Bronze Bronze Bronze Bronze Bronze Meno Honrosa Meno Honrosa Meno Honrosa Meno Honrosa Meno Honrosa Meno Honrosa Meno Honrosa Meno Honrosa Meno Honrosa Meno Honrosa Meno Honrosa Meno Honrosa Meno Honrosa Meno Honrosa Meno Honrosa Meno Honrosa Meno Honrosa Meno Honrosa Meno Honrosa Meno Honrosa Meno Honrosa Meno Honrosa Meno Honrosa

Sociedade Brasileira de Matemtica

EUREKA! N10, 2001

35

EE-0${E7E$EC{$E$-7 k$N $E N $7q;f E`$ E


NOME Ulisses Medeiros de Albuquerque Srgio Tadao Martins Daniel Massaki Yamamoto Humberto Silva Naves Carlos Stein Naves de Brito Lucas Heitzmann Gabrielli Daniel Mouro Martins Leonardo Augusto Zo Fabrcio Siqueira Benevides Christian Lyoiti Watanabe Rodrigo Roque Dias Rui Facundo Vigelis Thiago Barros Rodrigues Costa Ronaldo Ikaro Farias Arajo Daniel Nobuo Uno Daniel Pinheiro Sobreira Joo Alfredo Castellani Fajardo Freire Daniel Pessoa Martins Cunha Guilherme Fujiwara Gilberto Kirk Rodrigues Artur Duarte Nehmi Rodrigo Villard Milet Eduardo Moraes de Morais Carlos Sartori Ferreira Filho Thiago da Silva Sobral Silvano Jos Gomes Flumignan Mateus Ymanaka Barretto Hugo Pinto Iwata Paulo Ribeiro de Almeida Neto Digo Veloso Ucha Artur Radoman de Oliveira Yuri Gomes Lima Rafael Tajra Fonteles Maurcio de Carvalho Paiva Bernardo Freitas Paulo da Costa Eduardo Barbosa Arajo Antnio Davi Macdo de Castro Eduardo Famini Silva Arnaldo Joo do Nascimento Jnior Augusto Quadros Teixeira Einstein do Nascimento Jr. Marcos Soares de Souza Caio Augusto P. del Bianco Licciardi Luiz Antonio Felinto Cruz Diego Alvarez Araujo Correia Afonso de Paula P. Rocha Tibrio Bittencourt de Oliveira CIDADE ESTADO Fortaleza CE So Paulo SP So Paulo SP So Paulo SP Goinia GO So Paulo SP Fortaleza CE Nilpolis RJ Fortaleza CE Itagua RJ So Paulo SP Fortaleza CE Fortaleza CE Fortaleza CE So Paulo SP Fortaleza CE Salvador BA Fortaleza CE So Paulo SP Rio de Janeiro RJ So Paulo SP Rio de Janeiro RJ So Paulo SP Rio de Janeiro RJ Fortaleza CE P. Prudente SP So Paulo SP S.J. do Rio Preto SP Ananindeua PA Teresina PI Rio de Janeiro RJ Fortaleza CE Teresina PI Belm PA Rio de Janeiro RJ Fortaleza CE Fortaleza CE Salvador BA Duque de Caxias RJ Belo Horizonte MG Fortaleza CE Rio de Janeiro RJ Atibaia SP Fortaleza CE Fortaleza CE Fortaleza CE Goinia - GO PRMIO Ouro Ouro Ouro Ouro Ouro Prata Prata Prata Prata Prata Prata Prata Prata Prata Prata Prata Bronze Bronze Bronze Bronze Bronze Bronze Bronze Bronze Bronze Bronze Bronze Bronze Bronze Bronze Bronze Meno Honrosa Meno Honrosa Meno Honrosa Meno Honrosa Meno Honrosa Meno Honrosa Meno Honrosa Meno Honrosa Meno Honrosa Meno Honrosa Meno Honrosa Meno Honrosa Meno Honrosa Meno Honrosa Meno Honrosa Menao Honrosa

Sociedade Brasileira de Matemtica

EUREKA! N10, 2001

36


    
   !"#$% &  ' () !*$
+-, .0/ 12 34/ 576/89 :59 ;< No texto a seguir fazemos uma breve introduo ao conceito de nmero irracional. Na sua maior parte o texto ser acessvel a alunos da ltima srie do primeiro grau. As duas ltimas sees talvez requeiram um pouco mais de maturidade embora no exijam nenhum conhecimento prvio adicional. Para simplificar a exposio nos restringiremos a nmeros positivos. A extenso dos fatos abordados ao contexto geral de nmeros positivos, negativos e 0 no requer nenhuma dificuldade adicional. Pode-se imaginar que a idia de nmero inteiro positivo tenha surgido num estgio primrio da civilizao, juntamente com a necessidade da prtica da contagem. Por exemplo, era necessrio a um pastor saber contar de algum modo o nmero de animais no seu rebanho. A maneira de representar o resultado dessa contagem era no incio bastante diferente da que usamos agora e provvel que no comeo cada pessoa tivesse sua maneira prpria de faz-lo. Contar significa estabelecer um modo de comparar quantidades de elementos de conjuntos distintos. Por exemplo, a quantidade de pedrinhas em um saco com a quantidade de animais num rebanho, ou a quantidade de alimentos conseguidos em uma caada ou em colheita com a quantidade de membros da tribo. Tambm no difcil imaginar que a ideia de frao tenha surgido na evoluo da civilizao humana, primeiramente e de forma mais elementar, com a ocorrncia usual da necessidade de um determinado grupo de pessoas partilhar um ou mais bens de propriedade comum entre seus membros. E num estgio mais avanado, dentre outras motivaes possveis, com a necessidade de as pessoas trocarem entre si bens de tipos distintos. Por exemplo, um pastor deseja trocar com um agricultor peles de carneiro por sacos de milho numa razo de 3 peles de carneiro para cada grupo de 7 sacos de milho. Por outro lado, a idia de um nmero que no seja nem inteiro nem frao , em princpio, muito menos natural que a daqueles e surge num estgio muito mais avanado da civilizao com a necessidade da prtica da medio. Por exemplo, medir as dimenses ou a rea de um terreno, comparar as distncias entre pares de pontos distintos, etc. Procuraremos, a seguir, mostrar as propriedades bsicas destes nmeros estranhos em contraste com as propriedades, na maior parte j bem conhecidas, daqueles mais intuitivos, os inteiros e as fraes.

5  5 

Sociedade Brasileira de Matemtica

EUREKA! N10, 2001

37

Sociedade Brasileira de Matemtica

=>'?@A)BDC"BFEG HI@J KCL MG HN@"A


Os nmeros reais positivos podem ser representados no sistema decimal por uma seqncia de algarismos elementos do conjunto {0, 1, 2, 3, 4, 5, 6, 7, 8, 9} Separados por uma vrgula. Assim, se a N , a N 1 ,..., a 0 , a 1 , a 2 , a 3 ,..., so algarismos quaisquer, um nmero real positivo representado no sistema decimal tem a forma a N a N 1 a N 2 ...a1 a 0 , a 1 a 2 a 3 ..., (1) onde a N > 0. Nessa representao, esquerda da vrgula temos sempre um nmero finito de algarismos, porm direita podemos ter uma infinidade de algarismos. Por exemplo, 783,5231 representa o nmero obtido como resultado da expresso (2) 7 10 2 + 8 10 1 + 3 10 0 + 5 10 1 + 2 10 2 + 3 10 3 + 1 10 4 . Por outro lado, a frao 154 tem representao decimal 0, 1545454 com uma
999

infinidade de algarismos direita. Essa representao se traduz como resultado de uma expresso com infinitas parcelas (3) 1 10 1 + 5 10 2 + 4 10 3 + 5 10 4 + 4 10 5 + 5 10 6 + ... 154 Essa expresso significa exatamente que se quisermos aproximar no sistema
999

decimal com preciso de 8 casas decimais, por exemplo, devemos tomar como aproximao o nmero 0,15454545 que resultado da expresso (4) Claro, o nmero 0, 1545454 o que chamamos de uma dzima peridica e por isso pode ser obtido como uma frao 154 .
999

1 10 1 + 5 10 2 + 4 10 3 + 5 10 4 + 4 10 5 + 5 10 6 + 4 10 7 + 5 10 8.

OQPSR BT@E OSU V BFE"B UO E"@A O CB R HI@WCL MG HN@ UXOZY7[ BF\G^]SCG E@_
Neste caso, assim como no peridico, temos uma infinidade de algarismos direita da vrgula e assim s nos possvel escrever a representao decimal at uma certa casa decimal, porm, diferentemente do que acontece no caso peridico, no h repetio indefinidamente de um determinado grupo de algarismos e, assim, o nmero em questo no pode ser obtido como uma frao p com e e q diferente de 0. Os nmeros que podem ser obtidos como fraes so
q

chamados racionais; os que no podem ser obtidos como fraes so chamados irracionais.
EUREKA! N10, 2001

38

Sociedade Brasileira de Matemtica

`> [FO \ PR B [ \"BFEG^AF@H O AC O A Ua HbBF\ O AG \"\@EG OU @G^AF_


Responderemos esta pregunta atravs de um exemplo. Euclides provou que o nmero positivo cujo quadrado 2, isto , o nmero positivo x que satisfaz a x 2 = 2, (5) equao no racional. Euclides argumentou da seguinte forma: Suponhamos que o nmero x satisfazendo (5) seja racional. Ento existem inteiros positivos p e q, 2 2 2 primos entre si, tais que p 2 = 2. ou seja p = 2q . (6) q
2 Portanto p par e p tambm par; p pode ser escrito na forma p = 2k. Assim,

(2k ) 2 = 2q 2 2k 2 = q 2

(7)

Pela mesma razo que acabamos de expor, conclumos que q tambm deve ser par. Mas isto nos leva a uma contradio pois p e q so primos entre si por hiptese! Assim, a suposio de que x = p nos leva a uma contradio e, q portanto, deve ser descartada, considerada falsa. Chegamos concluso que 2 , que como representamos o nmero positivo cujo quadrado 2, um nmero irracional!!

c>'E O H OQO ? V B)\d@ [ \ Ofe G HI@ghB)AD\@"EG OSU @G^A [ @\"@

Podemos obter aproximaes cada vez melhores de 2 (o nmero x que satisfaz (5)) atravs do seguinte procedimento que um caso particular de um esquema inventado por Newton conhecido como mtodo de Newton. (Com base nesse mtodo podemos programar as mquinas de calcular para produzirem aproximaes de 2 to precisas quanto o avano da eletrnica nos permitir). primeiro chutamos um nmero x0 como uma primeira aproximao de x que nos parea razovel; por exemplo, x0 = 1. Em seguida observamos que
2 x 2 x0 = ( x + x0 )( x x0 ) 2 x0 ( x x0 ),

onde o smbolo significa aproximadamente igual a. Assim,


2 x 2 x0 2 x0 ( x x0 ), EUREKA! N10, 2001

39

Sociedade Brasileira de Matemtica

e, portanto, dividindo a equao aproximada por 2 x 0 e arranjando os termos, obtemos


2 x 2 x0 x + x0 . 2 x0 2 substituindo x = 2 e x0 = 1 em (8), obtemos x

(8)

2 1 3 +1= . 2 2

3 Assim temos uma segunda aproximao x1 = . Encontramos tambm x 2 : 2


x2 2 9 4 + 3 x 1 1 + 3 x 1 + 3 x 2 17 . Da mesma 2 2 3 2 4 3 2 12 2 12

forma, podemos obter uma quarta aproximao x3 , fazendo


x3 =
2 x 2 x2 2 (17 / 12) 2 17 288 289 17 288 289 + 2 289 577 = + = = . + x2 = + 2 12 17 408 2 x2 17 / 6 12 2 12 17 12

Assim, x3 = 577 seria a aproximao seguinte: Sua representao decimal a


408

dzima peridica x3 = 1,4142156862745 m n n n p n 098039 m k k l i i ... j 9.... n n o 21568627


perodo

Agora se voc pegar uma mquina de calcular e pedir (atravs dos devidos comandos) que ela calcule 2 , voc obter, se sua mquina puder exibir 33 dgitos (incluindo a vrgula ou ponto), a expresso decimal 1,4142135623730950488016887242097. Horrvel, no ? Voc obter uma expresso ainda maior se sua mquina puder exibir mais dgitos. Repare como nossas aproximaes x1 , x 2 e x3 esto cada vez mais prximas desse nmero!

q> O A Ua HbBF\ O AD\"@EG OSU @G A [FO C"BFHrAFBF\dB U"R HIB)\@C O A
Isto significa que podemos dispor os nmeros racionais numa sucesso da forma r1 , r2 , r3 ,..., com uma infinidade de elementos. Podemos interpretar este fato como significando que a quantidade de nmeros racionais, embora sendo infinita, de uma ordem de infinitude equivalente a dos nmeros naturais 1, 2, 3. O argumento para a demonstrao desse fato devido a Georg Cantor.
EUREKA! N10, 2001

40

Sociedade Brasileira de Matemtica

Como todo racional tem uma representao nica como frao

p q

com p e q

inteiros positivos primos entre si, basta que saibamos enumerar os pares ordenados (p, q) de naturais primos entre si. A forma de obter essa enumerao est descrita pela figura abaixo:

A enumerao obtida seguindo-se o caminho indicado pelas flechas, iniciando a partir de (1,1), tendo o cuidado de descartar os pares de naturais que no so primos entre si, como, por exemplo, (2,2), (4,2), (3,3) etc.. Com isso, teramos
r1 =

3 1 1 1 2 = 1, r2 = , r3 = = 2, r4 = = 3, r5 = , etc. 1 2 1 3 1

s >'\B [ \B)A)B U V @g XO C"BFEG HN@J-C O AD\"@EG OU @G A


H pouco dissemos que no era possvel pr uma dzima no peridica em forma de frao p com p e q naturais primos entre si. Vamos dar uma explicao para
q

este fato. Fixemos um natural q. Quando dividimos um nmero qualquer N > q pelo nmero q. Obtemos como resto da diviso um elemento do conjunto (finito) {0, 1, 2,, q 1}. Tomemos como exemplo q = 7 e N = 17; nesse caso os restos possveis pertencem ao conjunto {0, 1, 2, 3, 4, 5, 6}. Agora vamos recordar o algoritmo da diviso com esse exemplo especfico:

EUREKA! N10, 2001

41

Sociedade Brasileira de Matemtica

t u u t v w xyvSw-zZ{ZuZtfv|w}zZ{Zut#~ f w}z wZ tfv f {S v { {F v t) u D


O que acontece que os restos possveis so elementos do conjunto finito de q elementos {0, 1,, q 1}(no exemplo acima q = 7). Assim, em no mximo q iteraes do algoritmo ou acabamos repetindo um elemento do conjunto de restos possveis (no exemplo acima o primeiro a se repetir foi o 3), ou o 0 ocorre como resto e o processo termina. No primeiro caso, a partir da passamos a repetir os restos ocorridos anteriormente na mesma ordem (3, 2, 6, 4, 5, 1, no exemplo acima). As casas decimais no quociente por sua vez tambm se repetem o obtemos ento uma dzima peridica. No segundo caso, obtemos simplesmente um nmero finito de casas decimais.

>'\B [ \B)A)B U V @g XO C"BFEG HN@J-C O ADG \"\@EG OSU @G^A
Todo nmero irracional positivo possui uma representao decimal nica por meio de uma dzima no peridica. Para simplificar vamos nos restringir aos nmeros entre 0 e 1. J sabemos que um nmero cuja representao decimal possui uma quantidade finita de casas decimais pertence ao conjunto dos racionais. Da mesma forma aprendemos que um nmero cuja representao decimal uma dzima peridica tambm um nmero racional. Por outro lado, vimos no item anterior que as representaes decimais de um racional so necessariamente de um dos dois tipos: ou possuem uma quantidade finita de casas decimais, ou terminam em uma dzima peridica. Logo, uma representao decimal para um nmero irracional tem necessariamente que ser uma dzima noperidica. Afirmamos que essa representao nica. Repare que isso no ocorre em geral com os racionais. Por exemplo, 0, 21 e 0, 20999 representam o mesmo racional 21 . Suponhamos que um irracional x entre 0 e 1 possua duas
100

representaes decimais distintas:


EUREKA! N10, 2001

42

Sociedade Brasileira de Matemtica

x = 0, a 1 a 2 a 3 ..., x = 0, b1b 2 b3 ...,


Se essas representaes so distintas certamente existe um p ento que a p b p + 1 e por (10) e (11)

(10) (11) tal que

a k = b k , para k = 0,..., p 1, e a p b p . Para fixar idias vamos assumir x 0, a 1 a 2 ...a p , x 0, a 1 a 2 ...b p 999... = 0, a 1 a 2 ...(b p + 1),
e (13) implicam que a p = b p + 1 e x = 0, a 1 a 2 ...a p . Porm nesse caso x racional e chegamos a uma contradio! Chegaramos a uma contradio semelhante tambm se tivssemos assumido b p > a p , argumentando da mesma forma apenas trocando os papis dos a k e b k . A contridio tem origem no fato de termos suposto que havia duas representaes decimais distintas para o mesmo irracional x. Logo essa possibilidade tem que ser descartada, considerada falsa, e assim conclumos que todo irracional possui uma representao decima nica como dzima no-peridica. (12) (13)

j que b k = a k se k = 0,..., p 1 e b k sempre menor ou igual a 9. Mas (12)

U R I A G \\"@EG OU @G A UXO[FO CB)HA)BF\ B " H BF\"@C O A >O


Isto significa que no podemos dispor os nmeros irracionais numa sucesso s1 , s 2 , s3 ,..., mesmo admitindo uma infinidade de elementos. Quer dizer, diferentemente dos racionais, a ordem de infinitude da quantidade dos nmeros irracionais maior que a dos nmeros naturais. Conclumos da que existem muito mais nmeros irracionais do que racionais! Vamos tentar justificar nossa afirmao sobre a no-enumerabilidade dos irracionais. O argumento uma adaptao de uma idia tambm devida a G. Cantor. Suponhamos que fosse possvel dispor os irracionais numa sucesso s1 , s 2 , s3 ,..., . Basta considerarmos apenas os irracionais entre 0 e 1. Criamos um nmero irracional x, tambm entre 0 e 1, atravs de uma representao decimal (portanto, uma dzima no peridica) da seguinte forma. O nmero x tem representao decimal dada por x = 0, x 1 x 2 x 3 ... onde x p escolhido dentro do conjunto {0, 1, , 9} de modo que x p diferente de ( s p ) p onde este ltimo o algarismo que aparece na casa decimal de ordem p do irracional s p (p-sima
EUREKA! N10, 2001

43

Sociedade Brasileira de Matemtica

elemento da sucesso s1 , s 2 ,...s p ,...). A escolha de cada x p tambm deve atender a condio de no permitir que nenhum grupo de algarismos dentre os j escolhidos x 1 , x 2 ,..., x ( p 1) possa se tornar o gerador de uma dzima peridica. Desta forma obtemos uma dzima no peridica representando um nico irracional que, no entanto, no pode constar na lista s1 , s 2 , s3 ,..., . De fato, se

x = s r , para algum r , ento como x r ( s r ) r teramos um absurdo (uma


contradio)!.

>'BFA VR C O A R[ J BFHIB UV @"\ O G \\"@EG OU @J

O nmero definido como sendo a rea limitada por um crculo de raio 1. Ele certamente o irracional transcendente mais conhecido. A expresso transcendente significa, neste contexto, um nmero irracional que no raiz de nenhuma equao polinomial com coeficientes inteiros. Por exemplo, os irracionais 2 ,1 + 3 no so transcendentes pois so razes das equaes polinomiais

x 2 = 2, x 2 2 x 2 = 0, respectivamente. Neste ltimo caso dizemos que os


nmeros so algbricos. A demonstrao de que um nmero irracional, apesar de no ser trivial, pode ser feita usando-se apenas o clculo diferencial elementar que ensinado no primeiro perodo dos cursos de cincias exatas. A primeira demonstrao de que irracional s foi obtida em 1766 por J. H. Lambert, de forma no completamente rigorosa, tendo sido finalmente (re)obtida de modo rigoroso pelo famoso matemtico A. M. Legendre e publicada em 1855. A prova de que transcendente muito mais complexa e s foi obtida em 1882 por F. Lindermann. O fabuloso matemtica grego Arquimedes foi o primeiro a obter uma aproximao razovel de por numeros racionais. Ele provou que 10 1 3+ < < 3+ , 71 7 usando dois polgonos regulares de 96 lados, um inscrito e outro circunscrito a um crculo de raio 1. Podemos obter aproximaes cada vez melhores de , com o auxlio de uma mquina de calcular bastante rudimentar, capaz apenas de fazer as operaes bsicas (+, , ) e mais a operao de extrair raiz quadrada, da seguinte forma. A idia aproximarmos o crculo de raio 1 por polgonos regulares de 2 n lados inscritos neste crculo. Primeiramente, fcil verificar que para a rea e o permetro do polgono regular de 2 n lados inscritos num crculo de raio 1 temos
EUREKA! N10, 2001

44

Sociedade Brasileira de Matemtica

1 Permetro 4 l 2 , 4 onde l o comprimento do lado do polgono. Como l se aproxima mais e mais de 0 a medida que n cresce, vemos que para o crculo de raio 1 devemos ter (fazendo l = 0 na frmula acima) rea = 1 Permetro
rea = Assim, podemos tambm definir como sendo a metade do permetro do crculo de raio 1. Por outro lado, usando o teorema de Pitgoras que diz que em um tringulo retngulo o quadrado da hipotenusa a soma dos quadrados dos catetos, se l n denota o comprimento do lado do polgono regular de 2 n lados, fcil mostrar que
l n +1 = 2
2 4 ln .

(14)

Para n = 2 temos o polgono regular de 4 lados, quadrado, inscrito no crculo de raio 1, cujo lado, facilmente obtido usando-se o teorema de Pitgoras, l2 = 2. Por meio de (14) obtemos sucessivamente
l3 = 2 2 ,
l4 = 2 2 + 2 ,
l5 = 2 2 + 2 + 2 ,
l6 = 2 2 + 2 + 2 + 2 ,
l7 = 2 2+ 2+ 2+ 2+ 2 ,

l8 =

2 2+ 2+ 2+ 2+ 2+ 2 ,

Para obter uma boa aproximao de calculemos, por exemplo, o valor da 8 metade do permetro do polgono de 2 = 256 lados, inscrito no crculo de raio 1, cujo lado tem comprimento igual a l8 . Podemos obter um valor aproximado para l8 executando a seguinte seqncia de operaes numa calculadora
EUREKA! N10, 2001

45

Sociedade Brasileira de Matemtica

2 sqrt + 2 = sqrt + 2 = sqrt + 2 = sqrt + 2 = sqrt + 2 = sqrt + / + 2 = sqrt, e obtemos

l8 = 0.0245430765714398521588165239020064. Agora, multiplicaremos o resultado obtido para l8 por 256, que o nmero de
lados da polgono em questo, e em seguida dividimos por 2 o que nos d

~ 3.14151380114430107632851505945682
o que fornece uma aproximao com erro menor que 0, 0001 j que sabido que 3, 1415 < < 3, 1416. : Exceto pelas duas ltimas sees, o texto acima foi elaborado a partir de um pedido de minha filha, Marina, atualmente na 8a. srie do primeiro grau, urgida por um trabalho de casa em grupo passado por sua professora. O referido trabalho, felizmente, resultou bastante diferente do que foi exposto acima, que acabou servindo apenas como uma entre vrias referncias usadas pelo grupo. No entanto, as 7 primeiras sees foram bem compreendidas por ela e seu grupo; as duas ltimas foram escritas depois que o prazo para a entrega do trabalho havia esgotado e, portanto, no chegaram a ser testadas. Para concluir gostaria de deixar aqui meus agradecimentos ao estimado professor e colega Elon Lages Lima pelas sugestes sobre uma verso preliminar destas notas.

))) ))

B e BF\EL^EG O AF
1) Usando o mesmo argumento de Euclides descrito em 2. prove que

3, 5 e 7 so irracionais.
2) Usando o mtodo de Newton, descrito em 3, obtenha aproximaes correspondentes ao x3 do texto para os irracionais 3 , 5 , 7 e compare com o resultado fornecido pela mquina de calcular. 3) Pesquise sobre a vida e a obra dos grandes matemticos mencionados no texto: Arquimedes, Pitgoras, Euclides, Isaac Newton e Georg Cantor. 4) Prove a frmula (14).
EUREKA! N10, 2001

46

D D d
O comit editorial de *-S*" ! agradece aos inmeros elogios recebidos pela criao desta seo bem como ao crescente nmero de leitores que nos enviam solues para os problemas da mesma. A realizao de inmeras olimpadas nesta poca do ano torna mais importante a preparao para as mesmas e por isto resolvemos apresentar neste nmero somente novos problemas acompanhados dos nomes dos leitores que nos enviaram solues de problemas anteriores. No prximo nmero de *}*" ! voltaremos ao normal apresentando problemas e solues. Continuamos salientando que estamos disposio na OBM para aqueles que estiverem interessados na soluo de algum problema particular. Para tanto, basta contactar a OBM, seo b)"  , atravs de carta ou e-mail.

Sociedade Brasileira de Matemtica

\ ' Y `''

f))

 f)

Primeiramente vamos aos problemas propostos deste nmero


61.

Sejam a, b, c nmeros reais tais que as equaes x 2 + ax + 1 = 0 e x 2 + bx + c = 0 possuem exatamente uma raiz real comum e as equaes x 2 + x + a = 0 e x 2 + cx + b = 0 tambm possuem exatamente uma raiz comum. Determine a soma a + b + c .

62.

\ ' Y `''

2 1000 1 2 22 23 Determine a soma + + + + + 3 3 3 3 3

onde, como usual, x o maior inteiro que no supera x .


63.

\ ' Y `''

Sejam a e b nmeros reais no nulos que satisfazem equao

a 2b 2 a 2b 2 + 4 = 2 a 6 + b 6
Mostre que pelo menos um deles no racional.
64.

) (

Seja M o conjunto que consiste dos 2000 nmeros 11, 101, 1001, ... . Mostre que pelo menos 99% dos elementos de M no so primos.

\ ' Y `''

EUREKA! N10, 2001

47

Sociedade Brasileira de Matemtica

65.

Sejam ABCD um paralelogramo com A = 60 e O o circuncentro do tringulo ABD . A reta AO intersecta a bissetriz externa do ngulo C do paralelogramo em K . Determine AO OK . Sejam ABC um tringulo acutngulo com ACB = 60 , AD e BE duas de suas alturas que se intersectam no ponto H . Mostre que o circuncentro do tringulo ABC pertence bissetriz dos ngulos AHE e BHD . Sobre o lado AC do tringulo ABC , toma-se um ponto D distinto de A e C . Sejam O1 e O2 os centros dos crculos circunscritos aos tringulos ABD e CBD respectivamente. Mostre que os tringulos O1 DO2 e ABC so semelhantes.

\ ' Y `''

66.

BF Y `'''

67.

BF Y `'''

68.

Determine todos os inteiros n para os quais o valor da expresso abaixo inteira.


25 + 2 625 n + 4 25 2 625 n 4

BF' #F Y `''

69.

BF' #F Y `'' ?'  Y `''

f ( f (n )) = n e f ( f (n + 2 ) + 2 ) = n para todo inteiro n e f (0 ) = 1 .

Determine todas as funes f :

tais que

70.

Os tringulos equilteros ABF e CAG so construdos externamente sobre a hipotenusa AB e sobre o cateto CA do tringulo retngulo ABC . Se M o ponto mdio de BC , determine BC se MF = 11 e MG = 7 . Determine todos os pares de inteiros positivos satisfazem equao

71.

?'  Y `'' ?'  Y `''

(m, n )

que

72.

Quantos pares (n, q ) satisfazem a igualdade

(m n )2 (n 2
2

m = 4m 2 n

com n inteiro positivo e q um nmero racional no inteiro tal que

n! {q }= 2000

0 < q < 2000 , onde {x} = x x ?

EUREKA! N10, 2001

48

Sociedade Brasileira de Matemtica

73.

O ortocentro H de um tringulo ABC no pertence a nenhum de seus lados. Sabendo que a medida de AH igual ao raio do crculo circunscrito ao tringulo ABC , determine a medida do ngulo A .

HI F Y `''

74.

HI F Y `'' HI F Y `''

Resolva em

(x

3 x 2) 3(x 2 3 x 2) 2 x = 0
2

a equao

75.

Dado o nmero 2000 , calcule a soma das dcimas potncias dos algarismos deste nmero e continue fazendo o mesmo com o nmero obtido e assim sucessivamente. Mostre que entre os nmeros obtidos existem pelo menos dois nmeros iguais.

76.

a, b, c satisfazem relao a + b + c = 0 . Mostre que o nmero 2a 4 + 2b 4 + 2c 4 um quadrado


Os nmeros inteiros

HI F Y `''

perfeito.
77.

Se A = 1000 + 1000 2 + 1 , determine o 2000simo algarismo aps a vrgula da sua representao decimal.

 

Y `'' Y `''
3

1000

78.

todos os inteiros n n 4n + 14n 20n + 10 um quadrado perfeito. Determine


2 4

para

os

quais

79.

tal que f : k f (n ) = 1 se n mpar, e f (n ) = k para todo inteiro par n = 2 l , onde k um nmero natural e l um nmero mpar . Determine o maior nmero natural n para o qual f (1) + f (2 ) + + f (n ) 123456 Uma funo

\' ' V  'BF #F' Y `'''

80.

\' ' V ''BF #F' Y `'''


43 + 3 4
2n 2n

Se n nmero natural, mostre que

divisvel por 13 se, e somente se, n par.

81.

Dois crculos se intersectam nos pontos A e B . Uma reta l que passa por A intersecta estes crculos nos pontos C e D . Se M e N so os pontos mdios dos arcos BC e BD que no contm A e K o ponto mdio de CD , mostre que MKN = 90 .

G Y `''

EUREKA! N10, 2001

49

Sociedade Brasileira de Matemtica

82.

G ' Y `''
2 2

que x y x + y 2 2 .
2

Sejam x 0 , y 0 nmeros reais tais que x + y = 2 . Mostre

83.

Em um tubo de ensaio h exatamente uma ameba. A cada segundo algumas das amebas dividem-se em sete novas amebas ou morre exatamente uma das amebas. Determine o perodo mnimo de tempo aps o qual o nmero de amebas no tubo de ensaio ser igual a 2000 . Mostre que todo nmero racional positivo pode ser representado sob a forma

' F 7''

84.

' F 7''

r=

a3 + b3 c3 + d 3

onde a, b, c e d so inteiros positivos.


85.

Os lados AC e BC de um tringulo ABC possuem a mesma medida. Sejam P um ponto do interior do tringulo tal que PAB = PBC e M o ponto mdio de AB Mostre que APM + BPC = 180 . Dois crculos C1 e C 2 se intersectam nos pontos M e N e possuem uma tangente comum sendo P e Q respectivamente os pontos de tangncia com os crculos. Se N o ponto mais prximo de PQ e a reta determinada por PN intersecta C 2 novamente em R , mostre que MQ a bissetriz do ngulo PMR .

''''

86.

'''''

87.

a0 = 1 e a n = kn + ( 1) a n1 para n 1 . Determine todos os valores de k para os quais 2000 um termo da seqncia.


n

'''''

Para cada inteiro positivo k , definamos a seqncia (a n ) por

88.

Sejam x, y, z nmeros reais positivos tais que xyz = 32 . Determine o valor mnimo de que possuem a

'''''

x 2 + 4 xy + 4 y 2 + 2 z 2 ' ' '7'' 89. Determine todas as funes f :


propriedade :

EUREKA! N10, 2001

50

Sociedade Brasileira de Matemtica

f (xf (x ) + f ( y )) = ( f (x )) + y para todos os nmeros reais x e y .


2

90.

"

7''

Determine a razo A : C .

) Seja ABC um tringulo tal que BC AB + BC = AB BC AC

Enviaram solues de problemas anteriores os seguintes leitores da !



  
!"#$!%  & % (
" 4( +,/+ ( +  %&:"; < .=*
(&<"+, > ? ( +  %  (?.=*
:&+ ("7D=*+ EF, +,+ (  %  K?.LM(
N O"4P 8":"+,; T& U". 5+, ? X(+ 
"(+,
 .)+,0 ) 
 XD.("*L+ (
 Z O2"!(; (
#6(" Z O" +[ (#6( ; +,O$(
' /( +\(! 7 ; +,O&^
 ;* + !(!+ :"
4 ; +,O& ;* + !(!+ :&
4 ; +,  Z  .!; (( +, ' +, ?'_ (!%  " ' +, ?.LD

+,;*
(&# Q 4 ?!M
:"!M/+ 
` 

O2(' +, ? ("!(!a 
!;* +  b +\"X..(" Z  . cedgfihkjmlonqp2lsr tvuWwyxMz|{zy}~z }uD z|{zW{z wuz {zzw*moY*o& *m*!D*Y W WD  .W W M WoW MW |W.6WoM .  "~  " *&,| !& D &i '  (*)"( +,.-!') 5+ #6
("7*-! 8 5+ #6
("7*-! 8 5+ #6
("7*-! 8 Q(+ (" *-!R "V" 
.-! = #$ / *-!W "V" 
.-! 5+ #6
("7*-! 8 5+ #6
("7*-! 8 '  (*)"( +,.-!') '  (*)"( +,.]') Q(+ (" *-!R Q(+ (" *-!R 5+ #6
("7*-! 8 "V" 
.-! '  (*)"( +,.-!') 5+ #6
("7*-! 8 Q(+ (" *-!R 5+ #6
("7*-! 8 +,/021&3 +,/021&9 +,/029@"A2B"CA$B@ +,/029G"A21"9A$1"3A2B1A$B@"AIHJ"A$H"9 +,/0"J&A29"3A29&SA21J"AI9@ +,/021&HA21"SA$B"HA2B3A$B"CAIH"9A$H"3 +,/021&9A2B"1A$B"HA2H3 +,/021&9Y.H"S +,/021&CA2B"1A$B"H +,/021&9A21"HA$1"3A2B1A$B"HAIB"CA$HJA2H"9A$HB +,/021&9A2B"1A$B"BA2BCA$HJ"AIH"9 +,/021&9A2B"B +,/0"@&A21"CA2B&HA2HJ +,/029&3A29"CA$B"BA2BSA$HG +,/02B&C +,/029G"A2B"1 +,/021&9A21"3A$B"1A2BHA$HJ"AIH"1A$H"BMA2H"3A$HC +,/021&9Y.H"3 +,/021&9A21"1A$B"SA2HMJ"A$H"9AIH"3A$H"S +,/021&9Y.H"3

EUREKA! N10, 2001

51

Sociedade Brasileira de Matemtica

v
o

gv v

   !#"$%&  '!#$%($%)+*$ ,"$.-/$%*0213 ($4,")50"$6$ "$% *7 -/")8*$ 9

41) Se a e b so nmeros reais positivos, ento a b + b a > 1.

:<;>=@?>ACB5;ED>;>FHG5=@?I>;JFD>;EKML<NO<PCQSR>T5UVTWFCX@I>GHYO<P@Z
Observe que se a ou b for maior ou igual a 1 a desigualdade imediata. Ento analisaremos o caso em que 0 < a, b < 1.

1 1 ,b= , u > 0 e v > 0. (1 + u ) (1 + v) 1 1 1 >* = = Ento a b = b 1+ u b 1+ u b (1 + u )


Seja a = Analogamente b a =

*Agora o problema se reduz a mostrarmos que (1 + u ) b < 1 + ub para 0 < b < 1. Para isso basta termos uma noo de clculo diferencial

1+ u e portanto: 1+ u + v 1+ v 1+ u 1 ab + ba > + = 1+ > 1. 1+ u + v 1+ u + v 1+ u + v

1 1+ v = e 1 1+ u + v 1+ u 1+ v

1 f (u ) = 1 + bu (1 + u ) b f ' (u ) = b b(1 + u ) b 1 = b 1 > 0. 1 b (1 + u ) Como f crescente no intervalo (0,1) f (u ) > f (0) = 0 e portanto segue
imediatamente a demonstrao. 45) Existe uma seqncia infinita de: a) Nmeros reais b) Nmeros inteiros Tais que a soma de quaisquer dez termos consecutivos positiva, enquanto que para todo n a soma dos primeiros 10n + 1 termos consecutivos negativa?

[<\>]@^>_C`5\Ea>bcJ\>dV\>e5fCgSdh\EiMjke5lEm>^>n@eHoMb5gS\EpSq4rs\ta>bHu5e5vJbWr@dV\xwHqMu5y
a) Sim. Tome a10 k = 1 + mltiplo de 10.

1 1 para todo inteiro positivo k e a n = se n no k 9 10

EUREKA! N10, 2001

52

Sociedade Brasileira de Matemtica

1 1 1 1 1 + + ... + n = < 0 e, se 10 100 9 10 9 10 n 1 10k < n 10( k + 1), a n + a n +1 + ... + a n + 9 = k +1 > 0. 10 b) No. Se n > a1 , a1 + a2 + ... + a10n+1 = a1 + (a2 + ... + a11 ) + (a12 + ... + a21 ) +
Temos a1 + ... + a10 n +1 =

+ (a10n g + ... + a10n+1 ) > a1 + 1 + 1 + ... + 1 > a1 + n > 0.


47) Dada uma circunferncia , trace as tangentes a ela por um ponto exterior, A, tocando-a em M e N. Trace a reta r passando por A e tocando em B e C. Se D o ponto mdio de MN , prove que MN a bissetriz de BDC.

[<\>]@^>_C`5\Ea>bzM]@{C^>a>r@\EiMdh|C\>v|5}Jb5d<pSu5^Jv>a>r@eC~Ww[<<y
L r*

O N P A B D O M C r

Consideremos a inverso na circunferncia de centro O. Notemos que o ponto D, pela sua definio, o inverso do ponto A em relao a , B e C coincidem com seus inversos em relao a . Assim a figura inversa da reta r a circunferncia r* que contm O. Assim sendo as circunferncias e r* compartilham a corda BC . A reta perpendicular a BC pelo ponto mdio P passa por O e O (centro de r*). Por esse motivo os arcos RL e CL indicados na
___

figura acima so congruentes, ( OL um dimetro de r*).

O implica que a reta pelos Notemos agora que o fato de ser reto o ngulo ND pontos N e D passa por L. Isso completa nossa demonstrao: os ngulos inscritos L enxergam arcos congruentes do mesmo crculo, melhor, L e CD BD
circunferncia, r*, so congruentes por esse motivo. 49) dado um polgono regular de n lados.
EUREKA! N10, 2001

53

Sociedade Brasileira de Matemtica

Assinale aleatoriamente, no seu interior, um ponto M. Sendo x1 , x 2 ,..., x n as distncias de M a cada um dos lados, verifique que: 1 1 1 2 + + ... > , onde a a medida do lado do poligono. x1 x 2 xn a

[<\>]@^>_C`5\Ea>biMv>gS\Jv>r@\xzMeClxrv>}>eHoMb5gS\EpS>\>dVgSe5]@b5jeHwzM<y
Sejam O o centro e A1 , A2 , ...., An os vrtices do polgono. Podemos supor, sem perda de generalidade, que xi a distncia de M ao lado Ai Ai +1 (aqui,

An +1 = A1 ). Ento A( A1 A2 ... An ) = A( Ai MAi +1 ) =


i =1 i =1 n n n

axi a = ( x1 + x 2 + .... + x n ) (*) 2 2


n

Por outro lado, sendo r o raio do crculo inscrito no polgono, temos tambm

ar ar (**) = n 2 i =1 i =1 2 Comparando (*) e (**), conclumos que x1 + x 2 + .... + x n = nr . Agora, note que o permetro de menor que o do polgono, o que nos d 2 r < na . Ento n2a x1 + x 2 + .... + x n = nr < 2 A( A1 A2 ... An ) = A( Ai OAi +1 ) =
Finalmente, sabemos (ver artigo Desigualdades Elementares, A. Caminha, x1 , x 2 , ...., x n Eureka 5) que para os reais positivos tem-se

1 1 1 n2 + + .... + , e da x1 x 2 x n x1 + x 2 + .... + x n

1 1 1 n2 n2 2 + + .... + > 2 = . x1 x 2 x n x1 + x 2 + .... + x n n /(n 2 a / 2 ) a


50) Calcule o determinante: MDC (1, 1) MDC (1,2) MDC (1, n) MDC (2,1) MDC (2,2) MDC (2, n)

MDC (n,1) Onde MDC (a, b) o mximo divisor comum de a e b.

MDC (n,2) MDC ( n, n)

EUREKA! N10, 2001

54

Sociedade Brasileira de Matemtica

[<\>]@^>_C`5\Ea>b{5dV|5rs\xiM5iMf5fCe5aEz4\J}>b5vtbH>{Cm>r@\xabH]@r5bCr@dVezM\>fCgSepSqMr@\EabuCe5v>bCr@dV\xwHq%uCy
Conveno: det (n) igual ao determinante desejado quando este tem n linhas e colunas. Inicialmente escreveremos o determinante com uma quantidade de linhas suficiente para percebermos como o problema se comporta ao mudarmos o valor de n.
1 1 1 1 1 1 1 1 1 2 1 2 1 2 1 2 1 1 3 1 1 3 1 1 1 2 1 4 1 2 1 4 1 1 1 1 5 1 1 1 1 2 3 2 1 6 1 2 1 1 1 1 1 1 7 1 1 2 1 4 1 2 1 8

Determinante quando n = 8

Pode-se perceber que em cada linha k (1 k n ) do determinante, os elementos dessa linha considerada repetem-se com perodo k, pois mdc(x,k) = mdc(x + k, k) Como mdc(1,n) = 1 n , a primeira linha 1 1 1 ... 1 Alm disso, para todo p primo, mdc(n, p) = 1 se n < p, logo a p-sima linha da forma 1 1 1 ... 1 p, e a p-sima coluna idem quando tomamos n = p Logo, fazendo Lp = Lp L1 vemos que det(p) = (p-1) det(p-1), sendo det(p) o determinante desejado quando o nmero de linhas primo. Obs: Fazer Lp = Lp L1 significa trocar a linha p do determinante pela combinao linear (linha p linha 1). Essa notao ser usada outras vezes no decorrer dessa resoluo. Calculemos ento o determinante para alguns valores pequenos de n: det(2) = 1.det(1) = 1; det(3) = 2.det(2) = 2; Para calcular det(4), note que podemos fazer L4 = L4 L2 para concluir que det(4) = 2.det(3) = 4, e portanto det(5) = 4.det(4) = 16; det(6) um pouco mais difcil de calcular, mas notando a similaridade da L6 com L2 e L3, pode-se perceber que fazendo L6 = L6 L3 + L1 L2 reduzimos a ordem do determinante de forma que det(6) = 2.det(5) ( pois a6,6 vira 6 3 + 1 2 = 2 e para i < 6, a6,i = 0). Da, det(7) = 6.det(6) tambm fica determinado. Para det(8) fcil ver que a combinao linear a ser utilizada L8 = L8 L4 e para det(9), a combinao linear L9 = L9 L3. razovel supor que sempre possvel encontrar uma combinao linear que reduza de 1 a ordem do determinante. Note que os nmeros das linhas utilizadas nas combinaes lineares sempre foram divisores do nmero de linhas do determinante correspondente ( por exemplo, no caso n = 6 usamos as linhas 1, 2, 3 e 6 ).
EUREKA! N10, 2001

55

Sociedade Brasileira de Matemtica

Mais do que isso, os sinais que antecedem as linhas nas combinaes lineares parecem estar ligados funo de Mbius! De fato, no caso do 6, temos: L6 = (6) L6 + (3)L3 + (2)L2 + (1)L1. Entretanto, essa combinao linear falha para L8. No caso, entre os divisores de 8, apenas 1 e 2 tm no nulo. Mas estamos usando exatamente L4 e L8. Uma maneira de satisfazer esse caso e o do 6 fazer: L8 = L8 + L4 = L8 L4. Note que para n = 6 temos: L6 = L6 + L3 + L2 + L1 = L6 L3 L2 + L1, como desejvamos. Tentaremos ento provar que fazer a substituio Ln = para reduzir a ordem ( note que para d = n isso d Ln). Temos ento que as entradas da ltima linha zeram, com exceo daquele que tambm est na ltima coluna do determinante. De fato, consideramos a coluna x, x n. ( que na ltima linha equivale entrada mdc(x, n)) Quero calcular

8 8

8 4

6 6

6 3

6 2

6 1

L d
d |n

suficiente

.mdc( x,d ). d
d |n

Vamos tentar usar a frmula da inverso de Mbius (no caso, g(d) = mdc(x, d)). f (d )n > 0 , o Se acharmos uma funo f tal que g ( n ) =

d |n

problema termina porque a f ( n ) =

.g (d ) . d
d |n

Tomando ento f : Euler temos:

f (d )= 0 se d no divide x , , sendo a funo de f (d )= (d ) se d divide x


(***)

f (d ) = (d ) = (d )
d |n d |n d|x d |mdc ( x ,n ) EUREKA! N10, 2001

= mdc(x, n) = g(n) n .

56

Sociedade Brasileira de Matemtica

E se x < n, temos que n no divide x, e portanto

f (n )= 0 . Logo,

.mdc( x , d ) = f (n )= 0 d
d |n

para x < n. Se, entretanto, x = n, ento

f (n )= (n ) ( pois n divide x = n ) e

ltima linha passa a ser da forma 0 0 0 0 ... (n ) e ento


n i =1

.mdc( n , d ) = (n ) portanto, a d
d |n

det (n )= (n ). det (n 1) Logo, det (n )= (i )= (1).(2 ).(3)...(n ) .


(***) pois temos (d )= n .

d |n

como conseqncia da frmula da inverso de Mbius,

51) Trs feirantes foram vender melancias. Um levou 10; outro 16; o terceiro, 26. Todos venderam algumas melancias pelo mesmo preo at o meio dia. Depois disso, os trs baixaram o preo, mas continuaram vendendo por preos iguais. Quando voltaram para casa, aps venderem todas as melancias, cada um tinha a mesma quantia de dinheiro; 35 mil cruzeiros. Por quanto foi vendida cada melancia antes e aps o meio-dia?

[<\>]@^>_C`5\Ea>bMr@C>\x<bC]\>f5\EM|5}>>eHpSb5dVbCf5r@v>eHwH<@y
Sejam A1 , A2 e A3 os vendedores que venderam 10, 16 e 26 melancias respectivamente, xi a quantidade de melancias que o vendedor Ai vendeu antes do meio dia e y o preo antes do meio dia e y1 o preo aps o meio dia. Portanto, teremos:

I) y x1 + (10 x1 ) y1 = 35000 II) y x 2 + (16 x 2 ) y1 = 35000 III) y x + (26 x ) y = 35000 3 3 1


De I) e II)

y ( x1 ) + (10 x1 ) y1 = y x 2 + (16 x 2 ) y1 y ( x1 x 2 ) = (6 + x1 x 2 ) y1 Analogamente de II), III) e I), III) teremos y ( x1 x3 ) = (16 + x1 x3 ) y1 e y ( x 2 x3 ) = (10 + x 2 x3 ) y1

EUREKA! N10, 2001

57

Sociedade Brasileira de Matemtica

x1 x 2 = w1 x 2 x3 = w3

y w1 = (6 + w1 ) y1 y w3 = (10 + w3 ) y1

Faa x1 x3 = w2 y w2 = (16 + w2 ) y1

(6 + w1 ) y1 16 + w2 6 + w1 y w1 16 w2 6 w1 = = + + y w2 (16 + w2 ) y1 w2 w1 w2 w2 w1 w1 16 6 8 3 ; Imagine, agora, essas duas fraes nas suas formas = = w2 w1 w2 w1


irredutveis e como os seus numeradores devero ser iguais e o nmero 8 no tem o fator 3 ento w1 mltiplo de "3" w1 = 3k ( com k ) se

k 1 w1 6 w2 16 (Absurdo! pois w2 = x1 x 2 < 10, pois x1 < 10) logo k = 1 e w1 = 3 w2 = 8 e w3 = 5 . Como todos os vendedores venderam algumas melancias antes e aps o meio dia temos que 1 x1 9 e 1 x3 25 mas w3 = x1 x3 = 8 x3 = 1 e x1 = 9 x 2 = 6. Lembrando que y w1 = (6 + w1 ) y1 y = 3 y1 e assim substituindo em I) 3 y1 x1 + (10 9) y1 = 35000 28 y1 = 35000 y1 = 1250 cruzeiros e y = 3750 cruzeiros.

[<b5>^Jr@lx\>fHe5J^>e5dVa>eCva>\xeHf5\>]s^_5`5\Ea>\xJdh\Jm>]b5lEeC>^>mJ]@r@|5eCa>\tv>eH<MqM#Ji7o 5 iM>dVe5a>bC|5b5lE\>fHgSe5lxmCl\Eb5v>Cr\xa>eCff5\>]@^>_C>b5fbeH|5\>]eCm>\>dVe5_C`5\Eab / C# %C   / C # 5  # F C # -  / C#5C C5 F   4 <  / 5./  5 5  ' kC /  5 C    5   5 C/ %/ 5/ 5 %/ 5/ 55C C % 5 55CC 5 5 5C/5CV # 5C#C/ M /  5# 55C5C W C 55  55 C5 5 5 5 5C5   < C#C5/%
EUREKA! N10, 2001

+  + C5C H 5    5 / MH + C   + C   F # N < F C MH

+ V5/ #5/<> +  C MH + 55 /MH 5V C <H  +C MH +#55H
58

Sociedade Brasileira de Matemtica

. E      E+ + x6 7. + #>8t+W 4#+ .


52) Quatro retas se interceptam formando quatro tringulos conforme a figura abaixo.
A



gv

D E F B

a) Prove que as circunferncias circunscritas aos quatro tringulos possuem um ponto em comum. b) Prove que os centros dessas quatro circunferncias so concclicos (i.e. existe uma circunferncia que passa por todos eles). 53) Prove que num crculo convexo dado e para o mesmo nmero de lados. O polgono regular inscrito aquele cuja superfcie mxima.
x 54) Sejam ( x n ) a seqncia definida por x1 = 2, x n +1 = 2 n , n 1, e ( y n ) a

y1 = 2001, y n +1 = 2001( yn ) , n 1. Prove que existe c natural tal que y n x n + c para todo n e determine o menor c
seqncia definida por com essa propriedade. 55) Seja S o conjunto de pontos interiores de uma esfera de raio 1 e C o conjunto de pontos interiores de um crculo tambm de raio 1. Existe alguma funo f : S C tal que d ( A, B) d ( f ( A), f ( B)) para quaisquer pontos A, B S? (d (A, B) denota a distncia euclidiana entre A e B).

2001

/5
5/
5 
C5   C%  !#" $&%('")+*,
C/5 
5/
5
5C ./ 10  321 3!#0C5 4 567%89)
EUREKA! N10, 2001

59

Sociedade Brasileira de Matemtica

;:8<

>=/ 8
?A@

B7B

s@5>C>@H7D@iMMiFEMq4iM[<GCJ<@qMi4iM><IHM>@zMi

Sbado, 20 de outubro (nveis 1, 2 e 3) Domingo, 21 de outubro (nveis 2 e 3 - segundo dia de prova).

o D@<<@[LKMNHbPO J  dVr@lxbCr@dVeJe5f5b Q < [<b5>^>vJa>eJe5f5b Q Sbado, 9 de junho Sbado, 1 de setembro >bCdh|Cb5r@dVeH>e5fCb Q o D@<7 J CxMoM@<<qM[<@RH qM@  dVr@lEb5r@dVeH>e5fCb Q < Sbado, 1 de setembro [<b5J^>v>a>eH>e5fCb Q <@C>C>@H7D@iM4iMHiM@
Sbado, 12 de maio
B C>@5>C>s7D@iMMi@oM><qMoMiMz4s

Sbado, 20 e Domingo, 21 de outubro

oMiJCxMHiM><IH4J@zMi

1 a 14 de julho Washington, Estados Unidos

@SCJ@7Dsi4MiGEM<qMiM<qM@zMiMoMi4iM><IHM>@zMi oM@<<qM[<@RHMqMsi
Sbado, 6 de outubro

O leitor Diego Alvarez Arajo Correia, de Fortaleza CE, notou um erro tipogrfico na pgina 45 da revista Eureka No. 8. O enunciado correto do Teorema 2 (Frmula de inverso de Mbius) o seguinte: (n / d ) g (d ). Se para todo n > 0 temos g (n) = f (d ) ento, para todo n, f (n) =
d |n

8TUT4V,WV

d |n

EUREKA! N10, 2001

60

Sociedade Brasileira de Matemtica

@ v 8< P:F? F<;?$

Y[Z]\^G_ `a bcd\[efa g h \Y[^ \ikj b Y[g {|^ } b~d\ `4` |Ud\4\ c Y[6|g \R\Z\^ b |Uk|4cpa } b\4c6|U\ `44b]A |g b6`6^ |a ^ | p\^Gg b`6^ | c|^Ga 4bb]^ |4`,$\g Z|a ^ \ [g \Ukcpa bY[^ 4b]AUA|^ [g \Uk`~d\4| }na k|^ [g |4b]b]^$[^ |4`4UAa bc\ `d|4h |4` g a b| \ }a \bpk4\g h4|4`Rc|\^Ga \ 6g bp^ Ua bA|^G^ |a ^ \R6a Z\^ 4| `6a g kb a `|g |Rcd|[Y[^ \Uikj bJ^ \ } |\4cb7`Z b h \U6a g c6|A|^A\Ukc|4`Re\ \4c \4 p|g a A|k\U{6a b]g \dbj \4`[Y[^ \Ua {a \ b bJ|U6_ a bc||g bd| } b b bJ6^ \Uka ` 4b|g b6a {b]A\ }a ^ |k|\Un\4bpn\ b` p\^Gg b`Ua k} bJk|a h \4` b` [g bdh|4`Re\^ \a h\ b` \4`\^Uk\4`6a g Ab b` 6a db`4\4`,Ua Ab b` fa |a ^ \Y[g h4| ` \^ 4|g bJpka kbc6|Rg a h |a ^ \ 6a a bJ~|^A\Ukc|4`d| A|^G^ \ 6A6a \g h4\Ra ^ \Ukc\Rcd|[YZb]^Ga Z \^ 4bpkcd|`p\4h \g \Uk} |6^ \Uk4\ 4\U{6g bdbdc]^Ga 6b\Uk\4`4`a Z 4\U6g b~d|U^a pA|R[^k|a h\c|6a Z\^G |a k\g cb|U a ^ b&Y[^ \U\U a pa 4\^ c6bY[Z]b]^Ga Z b{k|^ } bfa k|&\^G^ b` eU^ pa b[g Acpa b\UZbd` efa g ha bc6|d\^G^ b`|g b 6\4c|UA|^G^ |a ^ \Rb]Z|4` 6bZ4`|AAcd|bcp^a k|4` U\g c6|U6{|^ &Y[^ \UiAj bc\Refa g h4\ \4]A|^4|^ |a ^ \Abk|4` \g c|Z\^R+\A\g g a

lnmpokqr lnmpok6oAr l p4 uR4,$ kw t u 4 uvxRR$r lnmpo6r lnmp+s u r l pu 4 s un4u4 w u wqUs As r lnmpp qUUp4Ur lnm6y[r l pu 4 s u4 w4 w r l pu 4 s uUs 4 w r lnmpo6$r lnmpokzr lnm[4du $u vAku r lnmpo6 r lnmpo6 r l z7 4u$ w As u 4w 6 4 w r lnmp yRr lnmpo6dzfr lnmpokyr ln py[pnmd r lnmpokpr lnmpo6 r l s vk Rw6s w4 vks 4u r lnmpokpr lnmpo6$r lnmpokpr lns ARd w v4u,$ 44 4u r ln vAku wR k s kw4 w 4u$wvAvAuvAr ln p4r l p4 u4 AwAs u 4w+u 4u vkr l pu 4 s uUAw4r l y[nmpozr lnmpo6 r lnmp$Ur lnmp4ok4  w k4u44 4 s w r lnmp4ok4  w k4RU s 4r ln vAku wR k s kwo4 w k zfu s vkr ln  s wRy s As 4w A4R+Au 4u[4Ry s s r

qUs tAu vkwRxyRz A s  ,ou wxRyRz 4 4 w4Rx w w Ax, s uR4[Aw4 s ux, A  s w+xRf nw w4 uxd yRw4 w4 vxRy U4u4w uRx[ Uw4 u[4 Rxf qUs s wRx4 zus 4 s w[xRz pw4R4uRz7 w444xRy Au u4 vkvAu4wRx 6 vks wx,U 4 x4 yRw s 4 x, s uRz7 w4 xd U4u+s vx[yR U4uRw uvxR ok u s w4 44u s vxRU pw4R s wRz w 44RxU 4 x4 ok u s w4 44u s vxRU Uw Aw4 ux oku w Aw[x[ 4s w ks kw 4wRxR U u vw u vx U u vw u vx u Aw, 4w tARx qUu w44u 4 wx 4u uU 4 Rx As RxU A wA s ux4 4u uqU ux U4uR[ s vk u A4u[xR Aw w+xRz U Ry s skx

EUREKA! N10, 2001

61

    !" $# #$


%   '()*(*+-, .*/*01 & 2 G J*K J*G *a &*a ao KJ  Ks  2o  2J    !" $# #$
%   3415)*6 7 .*/*0!89:.15, 6 4*, 9;0 <=  !" $# #$
% >?A@ B? #$CBD$EF '()*(*+-, .*/*014%34*15)6 7 .*/0%89;.*1H, 6 4, 9:0 <= *> %" ?#$  D>?D $B D I#! >%@>? B '()*(*+-, .*/*014%34*15)6 7 .*/0%89;.*1H, 6 4, 9:0 LM=N$O:PQR SE#ET ?D$U V .9:6 01XW*)*Y-0AZ[*, (4 ? \D%* E#?] \F$ # #$^`_! $#$ !" $# $ 3.b .*46dcH.feg9:.!h0*(*7 4*6 4*1 ijk?% j '/)*.*9;/*0
cl4(*m.*( > !" #? $ n?]#?#C!FD$# >Fp?q`E#E`_?>! $E`*` _r! $$`* \D$# >" %jB B#$D$ ?#$\ D ? 

Sociedade Brasileira de Matemtica

t uwvxy zu

com grande satisfao que informamos a excelente participao do Brasil na IMO. Pela primeira vez, os seis alunos da equipe ganharam medalhas, sendo 4 de prata e 2 de bronze. Na soma dos pontos, o Brasil ficou frente de mais de 80% das naes representadas, incluindo muitos pases de grande tradio olmpica. Veja a prova nesta edio e mais detalhes sobre a IMO no site
http://imo.wolfram.com/

Isso s nos estimula ainda mais a fazer uma Eureka cada vez melhor e que atinja um pblico cada vez maior. Gostaramos de agradecer mais uma vez o envio de grande nmero de problemas e solues pelos leitores, o que importante para a Eureka!, e nos anima a continuar o trabalho de proporcionar diverso e material de treinamento nossa comunidade olmpica. Gostaramos ainda de dizer que continuamos contando com o envio de artigos pelos leitores. Artigos para iniciantes so prioritrios. So tambm prioritrios artigos sobre temas olmpicos que at hoje no apareceram na Eureka!, como Contagem, Grafos (nvel avanado), Funes (nvel intermedirio), Trigonometria aplicada Geometria (pode ser uma compilao de problemas). Tradues (devidamente autorizadas, claro) de artigos de boas revistas (com o mesmo perfil, claro) sero muito bem recebidas. Finalmente agradecemos a valiosa ajuda dos professores Eduardo Tengan e Carlos Shine e dos estudantes Alex Cardoso Lopes, Guilherme I. C. Fujiwara e Rodrigo K. Yamashita na reviso da revista.

{x|}>~}>|

EUREKA! N11, 2001

yyvAy
t t w t y $g$ {` $ _r!

Sociedade Brasileira de Matemtica

Sara escreveu no quadro negro um nmero inteiro de menos de trinta algarismos e que termina em 2. Clia apaga o 2 do fim e escreve-o no incio. O nmero que fica igual ao dobro do nmero que tinha escrito Sara. Qual o nmero que Sara escreveu?

_r!
Vamos pegar um retngulo ABCD de papel; o lado AB mede 5 cm e o lado BC mede 9 cm. Fazemos trs dobras: 1- Levamos o lado AB sobre o lado BC e chamamos de P o ponto do lado BC que coincide com A. Forma-se ento um trapzio retngulo BCDQ. 2- Dobramos de forma que B e Q coincidam. Forma-se um polgono de 5 lados RPCDQ. 3- Dobramos de novo fazendo coincidir D com C e Q com P. Forma-se um novo trapzio retngulo RPCS. Aps fazer estas dobras, fazemos um corte perpendicular a SC pelo seu ponto mdio T, obtendo o trapzio retngulo RUTS. Calcule a rea da figura que aparece ao desdobrarmos o ltimo trapzio RUTS.

_r!

Temos trs caixas, uma azul, uma branca e uma vermelha, e 8 bolinhas. Cada bolinha tem um nmero de 1 a 8, sem repeties. Distribumos as 8 bolinhas nas caixas, de maneira que h pelo menos duas bolinhas em cada caixa. Logo, em cada caixa, somam-se todos os nmeros escritos nas bolinhas contidas na caixa. Os trs resultados denominam-se soma azul, soma branca e soma vermelha, segundo a cor da caixa correspondente. Encontre todas as possveis distribuies das bolinhas tais que a soma vermelha seja igual ao dobro da soma azul, e a soma vermelha menos a soma branca seja igual soma branca menos a soma azul.

_r!

Utilizando exclusivamente nmeros primos forma-se um conjunto com as seguintes condies: 1Qualquer nmero primo de um algarismo pode estar no conjunto. 2Para que um nmero primo de mais de um algarismo esteja no conjunto, devem estar no conjunto o nmero que se obtm ao suprimir-lhe s o
EUREKA! N11, 2001

Sociedade Brasileira de Matemtica

primeiro algarismo e tambm o nmero que se obtm ao suprimir-lhe s o ltimo algarismo. Determine, entre conjuntos que cumpram estas condies, aquele que tem maior quantidade de elementos. Justifique por que no pode haver um com mais elementos. Lembre-se de que o nmero 1 no primo.

_r!
Num tabuleiro de 8 casas, como na figura abaixo, h inicialmente uma ficha em cada casa. Uma jogada consiste em escolher duas fichas e mover uma delas uma casa direita e a outra, uma casa esquerda. Se depois de 4 jogadas as 8 fichas esto distribudas somente em 2 casas, determine quais podem ser estas casas e quantas fichas h em cada uma delas.

_r!

$x ``{` $

Na minha calculadora, uma das teclas de 1 a 9 est com defeito: ao pression-la aparece na tela um dgito entre 1 e 9 que no o correspondente. Quando tentei escrever o nmero 987654321, apareceu na tela um nmero divisvel por 11 e que deixa resto 3 ao ser dividido por 9. Qual a tecla defeituosa? Qual o nmero que apareceu na tela?

_r!
No trapzio ABCD, o lado DA perpendicular s bases AB e CD. A base AB mede 45, a base CD mede 20 e o lado BC mede 65. Seja P no lado BC tal que BP mede 45 e seja M o ponto mdio de DA. Calcule a medida do segmento PM.

_r!
Num tabuleiro de 3 fileiras e 555 colunas, pintam-se de vermelho 3 casas, uma em cada uma das 3 fileiras. Se escrevemos nas casas, ordenadamente por fileiras, da esquerda para a direita, os nmeros de 1 a 1665 (na primeira fileira de 1 a 555, na segunda de 556 a 1110
EUREKA! N11, 2001

Sociedade Brasileira de Matemtica

e na terceira de 1111 a 1665) h 3 nmeros que ficam escritos nas casas vermelhas. Escrevemos nas casas, ordenadamente por colunas, de cima para baixo, os nmeros de 1 a 1665 (na primeira coluna de 1 a 3, na segunda de 4 a 6, na terceira de 7 a 9,, e na ltima de 1663 a 1665) h 3 nmeros que ficam escritos nas casas vermelhas. Chamamos nmeros vermelhos aos que em alguma das duas distribuies ficam escritos nas casas vermelhas. Diga quais so as 3 casas que devemos pintar de vermelho para que existam s 3 nmeros vermelhos. Mostre todas as possibilidades.

_r!
Em volta de um crculo situam-se dez moedas de 1 cm de raio como indicado na figura abaixo. Cada moeda tangente ao crculo e s duas moedas vizinhas. Demonstre que a soma das reas das dez moedas o dobro da rea do crculo.

_r!
No quadro negro esto escritos os nmeros naturais desde 1 at 2001, inclusive. Temos que apagar alguns nmeros de modo que entre os que ficam sem apagar seja impossvel escolher dois nmeros distintos tais que o resultado de sua multiplicao seja igual a algum dos nmeros que ficam sem apagar. Qual a quantidade mnima de nmeros que devem ser apagados? Para esta quantidade, apresente um exemplo que mostre quais nmeros so apagados. Justifique por que no obtemos a propriedade desejada se apagarmos menos nmeros.

EUREKA! N11, 2001

yyvAy
t t t y ]>l: >d
$* > $`$g {` $ -*:%%?*5**%*5  *; *!**- %%; ?* *!*!** !:5 ! f *!%: :!*!* !%! d ?* !%* *;:* !5* * ***-*: **% ?*% d5*H5*-55 * * %f`!%5*;*** * d* d5* f * %**-* d=55 $x ``{` $ = X::=: *: -5%?fd 5*%*% ! * *:!%?**; ** *;% *5 ?*;:* *`* -:*5:*5 *:* * *:*!*!* * % !*;* : =*: %*5 !*!: * 5**%5 *!*$= ***:*% % %** %%*; ** %%;f % %** %%;f !** *!*!:*5 !** *!*!:*5 !** *!*!:*5 !** *!*!:*5 %*5%*;*5 !**-*!*:*- !**-*!*:*- %** %%*; ** %%;f % %** %%;f !** *!*!:*5 !** *!*!:*5 !** *!*!:*5 !** *!*!:*5 %*5%?**;*5 !**-*!*:*- %*5%*;*5

Sociedade Brasileira de Matemtica

* * !> * * !> ?* f *!> ?**>! f$=*:!>  d**> ? %%5* :!> * * !> %-*5!*`*!* ?*!* *!;**!>

? *;*5 * * 5!> * * !>  d**> * * 5!> %-*5!*`*!* ? %%5* :!> * * !> %* !r * * 5!>

EUREKA! N11, 2001

yyv%y t t w t E
nEy t uv
Enunciados e Resultado Brasileiro A XII Olimpada de Matemtica do Cone Sul foi realizada na cidade de Santiago do Chile no perodo de 1 a 6 de julho de 2001. Dela participaram alunos de at 15 anos dos seguintes pases: Argentina, Brasil, Chile, Equador, Peru e Uruguai. A equipe brasileira foi selecionada atravs de provas realizadas em maro e maio deste ano e foi liderada pelos professores lio Mega e Carlos Yuzo Shine de So Paulo - SP.

Sociedade Brasileira de Matemtica

{ }|]~~ $ }
`>|g}d  |>}~`|}  
}> } jx  ` > d> } A}> x~}   |>|  }>`  `   $`$g { ?r! " # & *:!% * _r!

 

$` ] $` ] {  {

Em cada casa de um tabuleiro quadriculado 2000 2000 deve-se escrever um dos trs nmeros: 1, 0 ou 1. Se, em seguida, somam-se os nmeros escritos em cada linha e cada coluna, obtm-se 4000 resultados. Mostre que possvel preencher o tabuleiro de modo que os 4000 resultados assim obtidos sejam todos distintos.

_r!

Tem-se uma sucesso a1, a2, a3, ..., an, ... de nmeros inteiros positivos, com as seguintes propriedades: i) Todo nmero inteiro positivo aparece uma ou mais vezes na sucesso. ii) a1 = 1 iii) a3n+1 = 2an + 1 iv) an+1 an v) a2001 = 200 Calcule o valor de a1000. Obs. O enunciado deste problema est incorreto, pois na verdade no existe tal sequncia (tente demonstrar isto!). Entretanto, se suprimirmos a condio i) possvel resolver o problema (tente fazer isto tambm!).
EUREKA! N11, 2001

Sociedade Brasileira de Matemtica

_r!
Trs tringulos acutngulos esto inscritos em uma mesma circunferncia, de modo que seus vrtices so nove pontos distintos. Demonstre que se pode escolher um vrtice de cada tringulo de maneira que os trs pontos escolhidos determinem um tringulo cujos ngulos sejam menores que ou iguais a 90o.

$x `{ r!"$# & *:*%!*  _r!


Um polgono de rea S est contido no interior de um quadrado de lado a. Demonstre que h pelo menos dois pontos do polgono que esto separados por uma distncia maior que ou igual a S . a

_r!
Ache todos os nmeros inteiros positivos m tais que m + 2001S(m) = 2m onde S(m) representa a soma dos algarismos de m.

_r! &%
Seja g uma funo definida para todo inteiro positivo n, que satisfaz i) ii) iii) iv) g(1) = 1 g(n + 1) = g(n) + 1 ou g(n + 1) = g(n) 1 para todo n 1 g(3n) = g(n) para todo n 1 g(k) = 2001 para algum inteiro positivo k.

Ache o menor valor possvel de k entre todas as funes g que cumprem as condies anteriores e demonstre que o menor.

EUREKA! N11, 2001

>Fp('$ # `_!

Sociedade Brasileira de Matemtica

Antes de procurarmos uma maneira de preencher um tabuleiro 2000 2000, vamos preencher um tabuleiro menor, digamos, 2 2. Uma maneira de preench-lo :
1
1 1 soma: 0 1 soma: 1

soma: soma: 2 1

Vamos tentar preencher agora um tabuleiro 4 4. Para isso, aproveitamos o tabuleiro 2 2. Para que as somas obtidos continuem iguais, colocamos 1's e 1's desta forma:
1 1 1 1
1

1 1

soma: 0

0 1 1

1 soma: 1

soma: soma: 2 1

Para completar o trabalho, basta preencher o subtabuleiro 2 2 do canto inferior direito:


1 1 1 1
1

1 1 1 1

soma: 0

0 1 1

1 soma: 1

0
1

soma: 3 soma: 2

soma: soma: soma: soma: 2 1 4 3

EUREKA! N11, 2001

Sociedade Brasileira de Matemtica

Observe que podemos preencher um tabuleiro (2k + 2) (2k + 2) a partir de um tabuleiro 2k 2k. Digamos que no tabuleiro 2k 2k as somas sejam iguais a (2k 1), (2k 2), 2k. Para preencher um tabuleiro (2k + 2) (2k + 2), colocamos o tabuleiro 2k 2k no canto superior esquerdo, preenchemos as 2k primerias casas da 2k + 1-sima linha e da 2k + 1-sima coluna con 1's e as 2k primeiras casas da 2k +-sima linha e da 2k + 2-sima coluna com 1's. No canto interior direito preenchemos da mesma forma que anteriormente:
1 1 2k 2k 1 1

)
1

)
1

(A)

Nas linhas e colunas em (A) e (B) temos as somas de (2k 1) a 2k

(B)

1 1

0 soma:2k + 1

1 1

1 1 soma: 2k

soma soma 2k + 2 (2k + 1)

Assim, temos todas as somas de (2k + 1) a 2k + 2. Logo possvel preencher um tabuleiro 2n 2n para todo n natural no nulo; em particular, o tabuleiro 2000 2000.

>Fp('$ # `_!

O enunciado do problema est incorreto: Vamos mostrar que no existe uma tal sucesso. Temos que a n +1 = a n ou a n +1 = a n + 1 (observe que, se a n +1 a n + 2, o nmero inteiro positivo a n + 1 no apareceria na sucesso). O nmero da forma 3n + 1 mais prximo de 2001 2002 = 3 667 + 1. Alm disso, a 2002 = a 2001 = 200 ou a 2002 = a 2001 + 1 = 201. Sendo a 2002 = 2a 667 + 1, temos que a2002 mpar, logo a2002 = 201. Assim, 2a667 + 1 = 201 a667 = 100. Mas 667 = 3 222 +1, e temos 2a222 + 1 = 100 a222 = 99 , que no inteiro. Contradio. 2 Trs alunos da delegao brasileira deram uma soluo equivalente anterior. Alguns alunos argentinos resolveram o problema ignorando a condio (i). Mostremos que possvel calcular a2000 sem utilizar a condio (i).
EUREKA! N11, 2001

10

Sociedade Brasileira de Matemtica

De (iv), indutivamente mostra-se que a m a n para m n. Aplicando (iii) temos:


a 4 = a 31+1 = 2a1 + 1 = 2 1 + 1 = 3 a13 = a 34 +1 = 2a 3 + 1 = 2 3 + 1 = 7 a 40 = a 313+1 = 2a13 + 1 = 2 7 + 1 = 15 a121 = a 340 +1 = 2a 40 + 1 = 2 15 + 1 = 31 a 364 = a 3121+1 = 2a121 + 1 = 2 31 + 1 = 63 a1093 = a 3364 +1 = 2a 364 + 1 = 2 63 + 1 = 127

Logo a1000 a1093 a1000 127

(I)

Agora estudemos a3. Seja a 3 = k . Temos: a10 = a 33+1 = 2a 3 + 1 = 2k + 1 a 31 = a 310 +1 = 2a10 + 1 = 2(2k + 1) + 1 = 4k + 3 a 94 = a 331+1 = 2a 31 + 1 = 2(4k + 3) + 1 = 8k + 7 a 203 = a 394 +1 = 2a 94 + 1 = 2(9k + 7) + 1 = 16k + 15 a 650 = a 3283+1 = 2a 283 + 1 = 2(26k + 15) + 1 = 32k + 31 a 2561 = a 3850 +1 = 2a850 + 1 = 2(32 k + 31) + 1 = 64k + 63 Como a 2551 a 2001 , temos 64k + 63 200 k 3. Mas a 3 a 4 k 3. Logo k = 3 e, portanto, a850 = 32 3 + 31 = 127. Desta forma a1000 a850 a100 127(II). De (I) e (II), temos a1000 = 127.

>Fp('$ # `_!


Sejam A1 A2 A3, B1 B2 B3 e C1 C2 C3 os tringulos. Tome o ponto A1 e trace o dimetro AA1 que passa por A1. Podemos supor, sem perda de generalidade, que, dentre os pontos B1, B2, B3, C1, C2 e C3, o mais prximo de A B1 e que, dentre os

pontos C1, C2 e C3, o mais prximo de A, contido no arco AA1 que contm B1 C1.

EUREKA! N11, 2001

11

Sociedade Brasileira de Matemtica

C1 B1

A1

O dimetro C1 C que passa por C1 divide a circunferncia em dois arcos. Se C1, C2 e C3 estivessem no arco que contm A1, teramos que o maior ngulo do 180 = 90 , o que no possvel. Logo tringulo C1C2C3 seria maior ou igual a 2 existe um ponto, Ci no arco que contm B1. Sendo C1 o mais prximo de A, e

como AC1 que contm B1, temos que Ci pertence ao arco CA . Temos ento que o tringulo A1B1Ci satisfaz as condies do enunciado, pois
m(Ci A1 ) m( A1 A) m (B A ) m ( AA1 ) = 90, m( A1 B1 Ci ) = m A1 C i B1 = a 1 1 < a = 90 2 2 2 2

m(C i B1 ) m(CC1 ) < = 90. Observe que o tringulo retngulo e m(C i A1 B1 ) = 2 2 se, e somente se, Ci = A. Obs. Como o nmero total de pontos 9, que mpar, podemos escolher um deles (que ser o A1) cujo antpoda no nenhum dos outros pontos. Nossa soluo mostra ento que possvel obter um tringulo acutngulo AiBjCk.

>Fp('$ # `_!


Suponha, por absurdo, que quaisquer dois pontos do polgono estejam separados por uma distncia menor que S/a.

EUREKA! N11, 2001

12

Sociedade Brasileira de Matemtica

y a x

Consideramos o ponto mais esquerda e mais direita x e y do nosso polgono. Se b a diferena de suas abscissas, o polgono est contido num retngulo de lados b e a. Temos claramente S ab, donde b S/a. Como claramente xy b, temos um absurdo e o resultado est provado.

>Fp('$ # `_!


Temos m + 2001 S(m) = 2m 2001 S(m) = m. Assim, m divisvel por 3 e conseqentemente S(m) tambm o . Logo S(m) = 3k, para algum k inteiro, e m = 2001 3k = 9 667k divisvel por 9. Desta forma, 9 divide S(m). Seja n o nmero de algarismos de m. temos que S(m) 9n (cada algarismo menor ou igual a 9), assim 2001S(m) 18009n m 18009n. n n Mas m 100 , + ...0 = 10 , logo 10 18009n. Esta ltima desigualdade s vlida para n 6. Assim, S(m) 9 6. Como S(m) divisvel por 9, temos S(m) = 9, 18, 27, 36, 45 ou 54. Temos S(m) = 9 m = 92001 = 18009, o que no possvel pois 1+8+0+0+9 = 18. S(m) = 18 n = 182001 = 36018. S(m) = 27 m = 272001 = 54027, o que no possvel pois 5+ 4+0+2+7 = 18 S(m) = 36 m = 362001 = 72036, o que no possvel pois 7+2+0+3+6=18 S(m) = 45 m = 452001 = 90045, o que no possvel pois 9+0+0+0+4+5=18 S(m) = 54 m = 542001 = 108054, o que no possvel pois 1+0+8+0+5+4=18 Logo a nica soluo n = 36018.
n 1 zeros *

EUREKA! N11, 2001

13

>Fp('$ # `_! &%

Sociedade Brasileira de Matemtica

Queremos encontrar o menor valor de k tal que g(k) = 2001. Assim, seja ak o menor valor tal que g(ak) = n. Desta forma, devemos calcular a2001. Temos g(1) = 1, logo a1 = 1. Podemos tomar g(2) = 2, logo a2 = 2. Observe que nesse caso g(3) = g(1) = 1 e g(6) = g(2) = 2. Podemos tomar g(4) = 2 e g(5) = 3, logo a3 = 5. Considere g(k) e g(k + 1). Temos g(3k) = g(k) e g(3k + 3 ) = g(k + 1). Assim, g(3k + 3) = g(3k) + 1 ou g(3k + 3) = g(3k) 1. Desta forma, g(3k +1) e g(3k + 2) so no mximo iguais a g(k) + 1 ou g(k + 1) + 1. Por exemplo, se g(4) = 2 e g(5) = 3, temos g(12) = 2 e g(15) = 3. Tomando g(13) = 3 e g(14) = 4, temos que a4 = 14. Note que, como g(k) 2 para k 4, temos g(k) 3 para k 12. Assim, podemos encontrar an + 1 em funo de an. Temos que g(an 1) = n 1 pois g(an 1 ) < n (se g(an 1) > n, existiria k < an tal que g(k) = n, o que contradiz a hiptese de an ser mnimo. Assim, g(3(an 1) = n 1 e g(3an) = n. Para k an 1, temos g(k) n 1, logo g(k) n para k 3(an 1). Podemos tomar g(3an 2) = n e g(3an 1) = n + 1. Logo an + 1 = 3an 1. Note que temos "quase" uma progresso geomtrica. Se somarmos um nmero x x 1 de cada lado da igual dado, temos a n +1 + x = 3a n 1 + x = 3 a n + . Se 3 1 x 1 1 1 1 x = , temos a n +1 = 3 a n . Sendo bn = a n , fizermos x = 3 2 2 2 2 temos bn+1 = 3bn, ou seja , bn uma progresso geomtrica de razo 3. Logo bn = 1 1 b1 3n 1. Como b1 = a1 = , temos 2 2 n 1 n1 3 2000 + 1 3 1 3 3n1 + 1 an = an = . . Para n = 2001, temos a n = bn = 2 2 2 2 2 Obs. A funo g que construmos tal que g(k) o maior possvel, para todo k, e obtida da seguinte forma: Primeiro observamos que todo natural n pode ser escrito de maneira nica como n = 3 k + 1 3 k 1 + 2 3 k 2 + ... + k 1 3 + k , com

j {1,0,1} para 0 j < k (onde k tal que


g (n) = 1 + acima.

3k + 1 3 k +1 + 1 n< ) . Temos ento 2 2

j =0

k 1

,ou seja, g(n) o nmero de termos no nulos na representao

EUREKA! N11, 2001

14

vxyy`vAy
t t yz t y t v w t En y t
Enunciados e Resultado Brasileiro A XLII Olimpada Internacional de Matemtica foi realizada na cidade de Washington DC, USA no perodo de 1 a 14 de julho de 2001 e teve a participao de 85 pases. A equipe brasileira foi selecionada atravs de provas realizadas em maro e maio deste ano e foi liderada pelos professores Nicolau C. Saldanha (Rio de Janeiro RJ) e Antonio Caminha Muniz Neto (Fortaleza - CE).

Sociedade Brasileira de Matemtica

{ }|]~~ $ }
`>|g}d        2  4

}- `/.> 10 }> `|


$}`}>|~]}` `>| ~ ]} | ]| 
 ` 0 }>$ >` }| `   } ->~]g` ]} 3 -  `}> 5
}$ 0 }

  `] }   `] }

$`$g { ?r! "# a * :!% * _r!


Seja ABC um tringulo acutngulo com circuncentro O. Seja PA uma altura do tringulo com P no lado BC. A AB C + 30 . Considere que BC B + CO P < 90 . Prove que CA

_r!
Prove que a a + 8bc
2

b b + 8ca
2

c c + 8ab
2

para quaisquer nmeros reais positivos a, b, e c.

EUREKA! N11, 2001

15

_r!

Sociedade Brasileira de Matemtica

Vinte e uma meninas e vinte e um meninos participaram numa competio matemtica. Cada participante resolveu no mximo seis problemas. Para cada menina e cada menino, existe pelo menos um problema que foi resolvido por ambos. Prove que existe um problema que foi resolvido por pelo menos trs meninas e pelo menos trs meninos.

$x `{ r!"$# a *:*%!* 

_r!
Seja n um inteiro mpar maior do que 1 e sejam k1 , k 2 ,..., k n inteiros dados. Para cada uma das n! permutaes a = (a1 , a 2 ,..., a n ) de {1,2,..., n}, defina S (a) =

k a .
i i i =1

Prove que existem duas permutaes b e c, b c, tais que n! um divisor de S (b) S (c).

_r!
C com P no lado BC, e seja BQ a Num tringulo ABC, seja AP a bissectriz de BA C com Q no lado CA. bissectriz de AB C = 60 e que AB + BP = AQ + QB. Sabemos que BA Quais so os possveis valores dos ngulos do tringulo ABC?

_r! &%
Sejam a, b, c, d inteiros com a > b > c > d > 0. Considere que ac + bd = (b + d + a c)(b + d a + c). Prove que ab + cd um nmero primo.

EUREKA! N11, 2001

16

Sociedade Brasileira de Matemtica

9 H

t Ezz 76 t 8y > >;:$=< ?>A@ B?C 9 > DE ?F$ G>

$f E>I>d*>IJ 5 JI B  >KB>ML/NO>rJ
PB?Q=SR J Gd T*U(>rSVfd>$CWH
XZY []\^`_ ab cdb ePaf\(g A Torre de Hani um dos quebra-cabeas matemticos mais populares. Ele foi inventado por Edouard Lucas em 1883.

}h>|

As peas so n discos de tamanhos diferentes e todos com um furo em seu centro e trs pinos onde so colocados os discos. Certamente podem ser encontrados em qualquer loja de brinquedos.

}|
}
0 }>|
~g

Inicialmente os discos formam uma torre onde todos so colocados em um dos pinos em ordem decrescente de tamanho.

Devemos transferir toda a torre para um dos outros pinos de modo que cada movimento feito somente com um disco, nunca havendo um disco maior sobre um disco menor.


} ] ]}|>} 3 > d>~

Queremos saber qual o menor nmero de movimentos necessrios para resolver uma torre de Hani com n discos. H uma histria (imaginada pelo prprio Edouard Lucas) sobre a torre de Hani: No comeo dos tempos, Deus criou a Torre de Brahma, que contm trs pinos de diamante e colocou no primeiro pino 64 discos de ouro macio. Deus ento chamou seus saserdotes e ordenou-lhes que transferissem todos os discos para o terceiro pino, seguindo as regras acima. Os sacerdotes ento obedeceram e comearam o seu trabalho, dia e noite. Quando eles terminarem, a Torre de Brahma ir ruir e o mundo acabar.
EUREKA! N11, 2001

17

|~]~ 0 }

Sociedade Brasileira de Matemtica

Para resolver um problema (no s este, mas vrios outros problemas na matemtica) que envolve n coisas, ajuda ver o que acontece para valores pequenos de n. Vejamos alguns casos. n = 1. Fazemos

1 movimento foi suficiente. n = 2. Fazemos

3 movimentos deram. n = 3. Fazemos

7 movimentos deram. Mas claro que no podemos fazer s isso. No podemos ficar observando o que acontece para todos os valores de n! Ento temos que comear a tirar algumas concluses.

EUREKA! N11, 2001

18

`  }>|}$ 0 }  

Sociedade Brasileira de Matemtica

~||j

Vamos olhar o caso n = 3 mais perto. Observe os trs primeiros movimentos:

Note que o que fizemos foi mesmo para resolver o caso n = 2. O prximo movimento foi

Isto , passamos o disco maior para o pino sem discos. Agora, veja os trs ltimos movimentos:

Novamente fizemos o mesmo que foi feito para o caso n = 2, s que transferindo agora a "subtorre" para o pino onde estava o disco maior. Agora, imaginemos uma torre com n discos. Imagine tambm que sabemos resolver o problema com n 1 discos.
k

n discos

Podemos transferir os n 1 discos de cima para um pino vazio:


vrios movimentos n1 discos

EUREKA! N11, 2001

19

Sociedade Brasileira de Matemtica

Depois passamos o disco maior para o outro pino vazio:

Por fim, colocamos os n 1 discos menores sobre o disco maior:


vrios movimentos n1 discos

Assim, podemos resolver o problema com n discos. Por exemplo, para resolver o problema com 4 discos, transferimos os 4 1 = 3 discos de cima para um pino vazio (j sabemos fazer isso!), depois passamos o disco maior para o outro pino vazio e por fim colocamos os 3 discos sobre o disco maior. Para resolver o problema com 5 discos, transferimos os 5 1 = 4 discos de cima para um pino vazio (acabamos de aprender a fazer isso!), e assim por diante.
4

`~}>| 0 ] |

Voltemos pergunta que ser calada: queremos saber o nmero mnimo de movimentos necessrios para resolver uma torre de Hani com n discos. Vamos dar um nome para este nmero, digamos Tn. Assim, o nmero mnimo de movimentos necessrios para resolver um problema com 1 disco T1, com 2 discos T2, com 2001 discos T2001, com discos T, e, em especial, com n 1 discos Tn 1.
n

$>~> 0 }
vrios movimentos n1 discos

J vimos que podemos resolver o problema da seguinte forma:

EUREKA! N11, 2001

20

Sociedade Brasileira de Matemtica

vrios movimentos n1 discos

Vamos ver quantos movimentos so necessrios neste modo de resolver o problema. Precisamos de Tn 1 movimentos para movimentar os n 1 primeiros discos, mais um para movimentar o disco maior e mais Tn 1 para colocar os n 1 discos sobre o disco maior. Assim, precisamos de Tn 1 + 1 + Tn 1 = 2Tn 1 + 1 movimentos. Mas no sabemos se este modo de resolver o problema usa o menor nmero de movimentos; poderia haver outro modo que use menos movimentos. Como o menor nmero de movimentos Tn, temos: (I) Tn 2Tn 1 + 1 Provemos que na verdade Tn = 2Tn 1 + 1. Para isso, mostraremos que Tn 2Tn 1 + 1 (lembre-se de que se a b e a b ento a = b). Esta aparentemente estranha maneira de se demonstrar que uma coisa igual a outra na verdade bem comum em vrios problemas. Muitas igualdades podem ser obtidas a partir de desigualdades. Considere agora, ento, o disco maior. Ele vai ter que sair da torre inicial uma hora. Mas para ele sair, preciso que os outros n 1 discos saiam de cima dele! E mais, se quisermos mud-lo de lugar ele vai ter que ir para um pino vazio, pois ele no pode ficar sobre nenhum dos outros discos por ser o maior (que trabalho esse disco d!)! Logo precisamos transferir os n 1 discos para um pino s, o que requer no mnimo Tn 1 movimentos. Para mudarmos ele de lugar, precisamos, claro, de mais um movimento. E depois, para colocarmos os n 1 discos sobre o disco maior precisamos no mnimo mais Tn 1 movimentos. Assim, para resolver o problema precisamos na verdade de no mnimo Tn 1 + 1 + Tn 1 = 2Tn 1 + 1 movimentos. Logo

EUREKA! N11, 2001

21

Sociedade Brasileira de Matemtica

Tn 2Tn 1 + 1

(II)

Assim, de (I) e (II), (*) Tn = 2Tn 1 + 1 Assim, como T1 = 1 ( s ver o caso n = 1), podemos, fazendo n = 2, concluir que T2 = 2T1 + 1 = 2 1 + 1 = 3 (exatamente como achamos antes!!) e, fazendo n = 3, descobriramos que T3 = 2T2 + 1 = 2 3 + 1 = 7 (que coisa!). Para n = 4, acharamos T4 = 2T3 + 1 = 2 7 + 1 = 15. Se quisssemos ento Tn para um valor qualquer de n, devemos ter todos os valores de Tk para k = 1, 2, , n 1, mas com certeza possvel calcular. Uma seqncia deste tipo (isto , tal que para calcular um dos valores usamos os valores anteriores) chamada recorrente e a equao que relaciona os termos da seqncia chamada de relao de recorrncia (no caso, temos que (*) uma equao de recorrncia).1 Poderamos parar por aqui (pois j sabemos como calcular os valores de Tn ), mas encontraremos uma frmula para Tn que no depende de seus valores anteriores (tal frmula costumeiramente chamada frmula fechada). Nem sempre se pode (e quando se pode, pode ser bem difcil) fazer isso com uma relao de recorrncia, mas com esta em particular pode ser feita. Observe que temos "quase" Tn = 2Tn 1 . Vamos ver se podemos acertar isso. Se somarmos um nmero x aos dois lados da equao (*), temos 1+ x Tn + x = 2Tn 1 + 1 + x Tn + x = 2 Tn 1 + 2 Se fizermos x = (1 + x) / 2 x = 1 e sendo An = Tn + 1, temos An = 2 An 1 = 2 2 An 2 = 2 2 An 2 = 2 2 2 An 3 = 2 3 An 3 = ... = 2 n 1 A1 Como A1 = T1 + 1 = 1 + 1 = 2, temos An = 2 n . Assim, An = Tn + 1 2 n = Tn + 1 Tn = 2 n 1 Assim, precisamos de 2 n 1 movimentos para resolver o problema da torre de Hani com n discos. Ou seja, os sacerdotes precisaro de 2 64 1 movimentos. Mesmo se eles fizessem um movimento por segundo, eles precisariam de mais de 500 bilhes de anos!! Podemos ficar tranqilos por enquanto.

Para outros comentrios e resultados sobre recorrncia veja o artigo "Equaes de Recorrncia", de Hctor Soza Pollman, publicado na revista Eureka! No. 9

EUREKA! N11, 2001

22

| }>/>hrd]} { 0

Sociedade Brasileira de Matemtica

Os alunos mais observadores devem ter notado de antemo que Tn = 2 n 1 bem antes, quando calculamos Tn para valores pequenos de n. Ter essa percepo bom, mas s perceber que Tn = 2 n 1 no suficiente. preciso provar que esta relao realmente verdadeira. As aparncias podem enganar!! Por exemplo, considere a seqncia n(n 1)(n 2)...(n 2000) +n 2001! (lembre-se : 2001! = 1 2 3 2001) Temos a1 = 1, a 2 = 2,..., a 2000 = 2000. Isto poderia nos levar a crer que a n = n, no? Pois veja quanto vale a 2001 e voc ter uma bela surpresa! an =
- }>/sg| 

01. Encontre uma frmula fechada para cada uma das relaes de recorrncia a seguir: a) a n = 3a n 1 + 4, a1 = 0 b) bn = 2bn 1 + 3 , b1 = 5 02. (Prova de Seleo para a IMO e Olimpada Iberoamericana 2001, adaptada) Seja f uma funo de t em t tal que, para todos x, y, z reais, f ( x + y ) + f ( y + z ) + f ( z + x) 3 f ( x + 2 y + 3 z ) a) Mostre que f (a) f (0) para todo a real. uv Mostre que f (a) f (0) para todo a real e conclua que as funes f onde f (a) = f (0) so as nicas solues do problema.

{ 0 | }>/>hrd$w
A grosso modo, uma funo f de um conjunto A em um outro conjunto B, uma relao que toma cada elemento x de A e o transforma em um elemento f(x) de B. As equaes de recorrncia que acabamos de estudar so exemplos de funes de N em R. 03. Na torre de Hani, suponha que em vez de transferir a torre para um dos pinos, voc tenha que transferir a torre para cada um dos outros pinos uma vez. Encontre o nmero mnimo de movimentos para resolver esse problema.

EUREKA! N11, 2001

23

zyyxn!zy t w uyzx t vx t u  z { vx
t u wnvxy  t t u ( | >~ } MB?  
XZY []\^`_ af\S \b b (g O que desejamos mostrar com esse texto o potencial significativo da trigonometria para resolver problemas de olimpadas de matemtica, principalmente quando combinada com algumas desigualdades. Esse texto, aps a resoluo de cada exemplo, apresenta um esquema da mesma, um Guia de Resoluo, o qual busca facilitar o entendimento geral do que foi feito. Caso o leitor queira tentar resolver tais exemplos antes de conhecer a resoluo descrita aqui, poder recorrer a este guia, como instrumento auxiliar. Vamos aos exemplos. (Seleo para IMO 99 Brasil ) Para reais positivos satisfazendo 1 1 1 3 + + , e determine a + b + c = abc, mostre que 2 2 2 2 1+ a 1+ b 1+ c quando a igualdade ocorre.

Sociedade Brasileira de Matemtica

*<!>C :

>Fp('$
A idia bsica para resolver esse problema fazer uso da transformao de um nmero real em tangente de outro. Isso vem do simples fato de que qualquer nmero real a pode ser representado pela tangente de outro nmero real pertencente ao intervalo (/2, /2), sendo tal nico isso explicado pelo fato da funo tangente, nesse intervalo, ser bijetora e ter como imagem todo o conjunto dos nmeros reais. E sendo ainda a um real positivo, podemos fazer a = tg , (0, /2). Agora, podemos perguntar: por que essa transformao nos seria til? Isso respondido se percebermos que a partir da conhecida identidade trigonomtrica 1+ tg = sec, obtemos o seguinte resultado: 1 / 1 + tg 2 = cos , ,

+ k , com o qual podemos simplificar a desigualdade a ser provada. 2 claro que se o estudante no tem o devido costume com essas frmulas, ele, provavelmente, no as reconheceria e nem pensaria em utilizar a transformao
EUREKA! N11, 2001

24

Sociedade Brasileira de Matemtica

para tangente. Mas a que entra a relevncia da trigonometria . Agora podemos prosseguir com a resoluo. Faamos a = tg , b = tg e c = tg , onde , , (0, /2). Temos ento que tg + tg + tg = tg .tg .tg (1) . Como 1 / 1 + tg 2 = cos , ,

3/2 para quaisquer , , (0, /2) que satisfaam a condio (1). Mas de (1) vem que tg ( + + ) = 0 (verifique! Dica: use a frmula da tangente da soma de trs termos). Logo, como , , (0, /2), temos que + + = . Para finalizarmos a demonstrao, usaremos a seguinte forma especial da desigualdade de Jensen (ver [4]): se uma funo f estritamente cncava (ver observao abaixo) num dado intervalo (a,b), ento
a1 + ... + a n f n f (a1 ) + ... + f (a n ) , n

+ k , ento o que devemos mostrar agora que cos + cos + cos 2

para quaisquer ai (a, b), ocorrendo a igualdade se e somente se os ai ' s forem todos iguais. E caso a funo seja estritamente convexa (ver observao abaixo) em um determinado intervalo a desigualdade muda de sinal. Continuando, como a funo cosseno estritamente cncava no intervalo (0,/2), temos que: cos + cos + cos 3 + + 1 cos = cos = cos + cos + cos , 3 3 2 3 2 para quaisquer , , (0,/2), ocorrendo a igualdade se e somente se = = = /3 a = b = c = tg /3 = 3 , concluindo a demonstrao.

\F #$
$F$p'
A transformao de um nmero real em tangente de outro e o uso da frmula 1+ tg =sec . Uso da propriedade dada no enunciado, para encontrar outra de melhor proveito. Uso de uma forma especial da desigualdade de Jensen.

EUREKA! N11, 2001

25

Sociedade Brasileira de Matemtica

{ 0 | z

Formalmente, uma funo f : I , onde I

um intervalo,

x + y f ( x) + f ( y ) estritamente cncava se f , para quaisquer x, y distintos > 2 2 x + y f ( x) + f ( y ) em I. E estritamente convexa se f , para quaisquer x, y < 2 2 distintos em I. Uma maneira geomtrica de identificar funes estritamente cncavas ou convexas observar a forma do grfico das mesmas: se o grfico for uma curva com concavidade voltada para baixo, a funo estritamente cncava, e se a concavidade for voltada para cima, estritamente convexa. Como exemplos de funes estritamente cncavas, temos a funo f(x) = cos x no domnio (0,/2) e as funes logartmicas cujas bases so maiores do que 1. E de funes estritamente convexas temos a funo f(x) = tg x no domnio (0,/2) e a funo f(x)=1/sen x, com x em (0,). (Como exerccio, classifique outras funes conhecidas em estritamente convexas ou cncavas). importante o leitor ver as referncias [2] e [4], onde encontram-se definies e resultados mais precisos e genricos, alm das demonstraes.

O prximo exemplo, da IMO de 1996, tido por alguns matemticos interessados em olimpadas (ver [1]), como o problema mais difcil j proposto em IMOs. um problema de geometria associado a desigualdade (algo bastante explorado em olimpadas).

*<!> : (IMO 96) Seja ABCDEF um hexgono convexo tal que AB paralelo
a DE, BC paralelo a EF e CD paralelo a FA. Sejam RA, RC, RE os raios das circunferncias circunscritas aos tringulos FAB, BCD, DEF respectivamente, e seja p o permetro do hexgono. Prove que p R A + RC + R E . 2 >Fp('$ Algo nesse problema j nos insinua a usar a trigonometria, voc percebe? O fato dele relacionar raio de circunferncia circunscrita com lado (que tem a ver com o permetro) faz-nos lembrar da conhecida lei dos senos, que afirma: dado um BC AC AB = = = 2 R , onde R o raio da tringulo ABC, temos que senA senB senC circunferncia circunscrita ao tringulo dado (como exerccio, prove-a). Da, portanto, podemos agora no mais trabalhar com os raios dos tringulos citados, mas sim com algumas diagonais do hexgono. Isso porque, pela lei dos senos,
EUREKA! N11, 2001

26

Sociedade Brasileira de Matemtica


obtemos que BF = 2RA.sen F A B , BD = 2RC.sen B C D , FD = 2RE.sen D E F (ou seja, encontramos uma relao entre os raios citados no problema e algumas diagonais do hexgono, o que facilitar o nosso trabalho). A prxima parte da resoluo do problema a que exige uma maior dose de criatividade por parte do estudante. Vejamos. Prolonguemos os lados paralelos BC e EF do hexgono (vide figura 1). Por A e D tracemos perpendiculares aos lados prolongados, obtendo o retngulo de vrtices M, N, P e Q, ilustrados na figura 1. Como MN e PQ so as menores distncias entre pontos das retas paralelas BC e EF (pois esses segmentos so perpendiculares s mesmas), temos que BF MN e BF PQ 2BF MN + PQ 2BF AM + NA + DP + DQ 2BF AB.senB + AF.senF + CD.senC + DE.senE (1), onde sen X denota o seno do ngulo interno de vrtice X do hexgono, o qual igual ao seno do respectivo ngulo externo, pois os mesmos so suplementares.
M B C P

figura 1 Pela lei dos senos, ns j sabemos que BF/senA = 2RA . Ento dividindo ambos os lados da desigualdade (1) por senA, obtemos: 4 R A AB. senB senF senC senE + AF . + CD. + DE. (I) senA senA senA senA

E de forma anloga, seguindo os mesmos passos com as diagonais BD e DF do hexgono, obtemos:

EUREKA! N11, 2001

27

Sociedade Brasileira de Matemtica

4 RC BC.

senB senD senA senE + CD. + AF . + EF . (ii) senC senC senC senC senA senC senD senF + BC. + DE. + EF . (iii) senE senE senE senE

4 R E AB.

E agora, somando (I), (ii) e (iii), obtemos senA senB senC senB senC senD + + + 4( R A + RC + R E ) AB. + BC. + CD. + senE senA senE senC senA senC senE senD senE senF senA senF + DE. + + + + EF . + FA. . senA senE senC senE senC senA Agora observe que como os lados opostos do hexgono convexo so paralelos, ns temos que os ngulos opostos do mesmo so congruentes. Assim, ns obtemos: senA = senD; senB = senE; senC = senF. Por conseguinte, ns temos que os fatores que esto multiplicando os lados do hexgono na ltima desigualdade acima so da forma (z + 1/z), sendo z positivo (pois o seno de um ngulo maior que 0 e menor que 180 sempre positivo). E fcil verificar que z + 1/z 2, para todo z positivo. Assim ns obtemos:
4( RA + RC + RE ) 2( AB + BC + CD + DE + EF + FA) 4( RA + RC + RE ) 2 p p , concluindo a demonstrao. $ &W W~~W 2 RA + RC + RE

Uso da Lei dos Senos. Uso das construes: prolongamento de dois lados opostos e traado de perpendiculares pelos dois vrtices restantes. Congruncia dos ngulos opostos do hexgono convexo. Uso da desigualdade: z + 1/z 2, z > 0.

(IMO 91) Seja ABC um tringulo e X um ponto interior do mesmo. Prove que pelo menos um dos ngulos XAB, XBC, XCA menor ou igual a 30.

d$~~

EUREKA! N11, 2001

28

Sociedade Brasileira de Matemtica

~$(W

Geralmente, em questes que envolvem um ponto num interior de um tringulo, til traarmos perpendiculares a partir desse ponto aos lados do tringulo. Vamos utilizar isso. Sejam P, Q, R os ps das perpendiculares traadas por X aos lados BC, CA e AB, respectivamente. Para facilitar, denotaremos por , , os ngulos do tringulo (BAC, CBA, ACB) e por , , os ngulos XAB, XBC, XCA.

A R Q

X B P
figura 2 Ns temos que PX = BX.sen = CX.sen( ); QX = CX.sen = AX.sen( ); RX = AX.sen = BX.sen( ). Multiplicando essas trs igualdades, ns obtemos: sen( ' ).sen( ' ).sen( ' ) = sen '.sen '.sen '
sen( ' ) sen( ' ) sen( ' ) = 1. sen ' sen sen '

sen( A x) = senA. cot x cos A senx estritamente decrescente no intervalo (0, ), visto que a funo cotangente estritamente decrescente nesse intervalo. Assim, se , , forem todos maiores que 30, teremos que: Agora observe que a funo f ( x) = 1= sen( ' ) sen( ' ) sen( ' ) sen( 30 ) sen( 30 ) sen( 30 ) < sen ' sen sen ' sen30 sen30 sen30 1 ( 1) sen( 30 ) sen( 30 ) sen( 30 ) > sen30 sen30 sen30 = 8

EUREKA! N11, 2001

29

Sociedade Brasileira de Matemtica

Mas, ns temos que: sen( 30 ).sen( 30 ) = = 1 (1 sen( 30 )) 2 1 (cos( ) cos( + 60 )) 1 (1 cos( + 60 )) = 2 2

Observe que essa ltima igualdade decorre do fato de ( 30) ser complementar a ( + 60). Continuando, ns temos que: sen( 30 ) sen( 30 ) sen( 30 ) 11 1 = sen( 30 ) 24 2
2

1 (1 sen( 30 )).sen( 30 ) = 2

1 . 8

Mas esta ltima desigualdade obtida contradiz (1), logo , , no podem ser todos maiores do que 30, o que encerra a nossa demonstrao.
$ &W W~~W

Construo das perpendiculares a partir de X aos lados do tringulo e obteno da igualdade: sen( ' ).sen( ' ).sen( ' ) = sen '.sen '.sen ' . Esses ngulos so identificados no incio da resoluo. sen( A x) Observar que a funo f ( x) = estritamente decrescente. senx Supor, por absurdo, que todos os trs ngulos so maiores do que 30, e chegar a uma contradio.

Prove que, dentre quaisquer cinco reais y1 , y2 , y3 , y4 , y5 , existem dois, que satisfazem: yi y j 1. 0 1 + yi y j

d$~~d

EUREKA! N11, 2001

30

Sociedade Brasileira de Matemtica

~$(W

Olhando para o termo do meio da desigualdade acima, o que ele nos faz lembrar? Sem muita dificuldade, associamo-lo logo formula da tangente da diferena. Ento mais uma vez faamos uso da transformao para tangente. Isto , faamos yi = tg xi, i = 1, 2, 3, 4, 5 , xi (/2, /2). Como tg 0 = 0 e tg /4 = 1, devemos ter agora: tg 0 tgx i tgx j 1 + tgx i .tgx j tg

tg 0 tg (x i x j ) tg . 4 4

E ainda, como no intervalo (/2, /2) a funo tangente sempre crescente, obtemos 0 xi x j . Agora o que temos que provar que existem dois 4 dentre os cinco xis que satisfazem esta ltima desigualdade. Para isso, usamos o conhecido Princpio da Casa dos Pombos. Dividamos o intervalo (/2, /2) de tamanho em outros quatro intervalos de tamanho /4. Assim, pelo princpio citado, dois dentro os cinco xis estaro no mesmo intervalo, os quais vo satisfazer a desigualdade pedida.
$ &W W~~W

Uso da transformao para tangente. Aplicao da tangente da diferena. Uso do Princpio da Casa dos Pombos.

Finalizamos esse texto com alguns problemas para o leitor exercitar o que foi mostrado. lgico que existe mais de uma soluo para cada problema, mas pede-se que o leitor tente resolv-los utilizando a trigonometria e as desigualdades mostradas ou outras conhecidas.
d$~( ~

. (IMO 61) Prove que, para qualquer tringulo de lados a, b, c e rea A, temos que: a 2 + b 2 + c 2 4 3 A .

EUREKA! N11, 2001

31

Sociedade Brasileira de Matemtica

Prove que, dentre 13 nmeros reais, existem dois, x e y, tais que:


x y 2 3 .1 + xy .

(OBM 85) Um quadriltero convexo est inscrito em uma circunferncia de raio unitrio. Demonstre que a diferena entre seu permetro e a soma de suas diagonais maior do que zero e menor do que 2. (Ibero-Americana 88) As medidas dos ngulos de um tringulo esto em progresso aritmtica e as medidas das alturas do mesmo tambm. Prove que o tringulo equiltero. (Putnam 78) Encontre a rea de um octgono convexo que est inscrito em uma circunferncia e que tem que quatro lados consecutivos medindo 3 unidades e os lados restantes medindo 2 unidades. D a resposta na forma r + s t , com r, s e t inteiros positivos.
d d

(Gr-Bretanha 84) O quadriltero ABCD tem uma circunferncia inscrita.


Para o lado AB ns associamos a expresso f(AB) = p1.(sen D A B ) + p2. (sen A B C ), onde p1 e p2 so as medidas das perpendiculares traadas de A e B, respectivamente, at o lado oposto CD. Definimos f(BC), f(CD) e f(DA) similarmente, usando para cada um as perpendiculares ao lado oposto. Mostre que f(AB) = f(BC) = f(CD) = f(DA). (IMO 91) Em um tringulo ABC, as bissetrizes AD, BE, CF encontram-se no ponto I. Mostre que: 1 8 IA IB IC . . . 4 AD BE CF 27 Mostre que se um quadriltero de lados a, b, c, e d inscritvel e circunscritvel ento sua rea abcd .
d d

EUREKA! N11, 2001

32

Sociedade Brasileira de Matemtica

Uma funo d(x, y) de dois reais x, y chamada distncia se d(x, y) = d(y, x); d(x, x) = 0 ; e d(x, y) + d(y, z) d(x, z), para quaisquer reais x, y, z. Prove que a seguinte funo uma distncia: d ( x, y ) =
d

x y 1+ x2 1+ y2

Sejam x, y, z reais positivos tais que xy + yz + zx = 1. Prove que: 2x 1 x 2 2y 1 y2 2z 1 z 2 x y z + + + + 2 2 2 2 2 2 2 2 (1 + x ) (1 + y ) (1 + z ) 1+ x 1+ y 1+ z2

Rafael Tajra Fonteles cursa a 2. Srie do Ensino Mdio no Instituto Dom Barreto de Teresina PI. O Prof. Jos Nazareno Cardeal Fonteles da Universidade Federal do Piau e coordenador de matemtica do Instituto Dom Barreto colaborou com o artigo, fazendo a reviso do mesmo.
(
[1] ENGEL, Arthur. Problem-Solving Strategies. Springer-Verlag, New York, 1998. [2] LARSON, Loren C. Problem-Solving Through Problems. Springer-Verlag, New York, 1983. [3] MEGA, lio & WATANABE, Renate. Olimpadas Brasileiras de Matemtica 1 a 8. Comisso de Olimpadas da SBM. Ncleo, So Paulo, 1988. [4] MUNIZ NETO, Antnio Caminha. Desigualdade Elementares. Eureka! N 5. OBM, 1999. [5] Pgina da Web mantida por Jonh Scholes: www.kalva.demon.co.uk/

EUREKA! N11, 2001

33

Sociedade Brasileira de Matemtica

OO&zOMy A(=MPM55  ?W 

Z ]WdPP( Um polinmio uma expresso da forma p( x) = a 0 + a1 x + a 2 x 2 + ... + a n x n . Uma srie formal uma expresso ainda mais simples - basta apagar o ltimo termo: p( x) = a 0 + a1 x + a 2 x 2 + ... Somas e produtos so definidos de maneira anloga s operaes correspondentes com polinmios. Assim, por exemplo, (1 3x + 3 2 x 2 3 3 x 3 + ...)(1 + 3x) = 1 3x + 3 2 x 2 33 x 3 + ...
+ 3 x 3 2 x 2 + 3 3 x 3 ...

=1 de modo que podemos escrever 1 3 x + 3 2 x 2 33 x 3 + ... = 1 /(1 + 3 x). De maneira geral, podemos "compactar" uma srie formal 1 + ax + a2x2 + na forma 1/(1 ax), que certamente ocupa bem menos espao que uma srie infinita Vejamos uma primeira aplicao das sries formais. Vamos determinar uma "formula fechada" para a seqncia definida por a 0 = 0 , a1 = 1 e a n + 2 = 5a n +1 6a n para n 0. A idia considerar a srie formal f ( x) = a 0 + a1 x + a 2 x 2 + ... e tentar "compact-la" e depois "descompact-la". Para alcanar o primeiro objetivo, observe que f ( x) = a 0 + a1 x + a 2 x 2 + a 3 x 3 + ... 5 xf ( x) = 5a 0 x 5a1 x 2 5a 2 x 3 + ... 6 x 2 f ( x) = 6a 0 x 2 + 6a1 x 3 + ... Somando as equaes acima, os coeficientes de x n , n 2, anulam-se e ficamos com x (1 5 x + 6 x 2 ) f ( x) = a 0 + ( a1 5a 0 ) x f ( x) = 1 5x + 6 x 2 Agora, como descompactar f(x)? O truque aqui "quebr-lo" em pedaos que sabemos como descompactar. Observe que 1 5 x + 6 x 2 = (1 2 x)(1 3x) e que razovel procurar constantes a e b tais que
EUREKA! N11, 2001

34

Sociedade Brasileira de Matemtica

(a + b) (3a + 2b) x a b x x + = = 2 1 2 x 1 3x 1 5 x + 6 x (1 2 x)(1 3 x) 1 5x + 6 x 2

a + b = 0 a = 1 e b = 1 3a + 2b = 1 Logo f (x) = 1 1 x = = (1 + 3x + 32 x2 + 33 x3 + ...) (1 + 2x + 22 x2 + 23 x3 + ...) 2 1 5x + 6x 1 3x 1 2x = (3 0 2 0 ) + (31 21 ) x + (3 2 2 2 ) x 2 + (3 3 2 3 ) x 3 + ... Assim, o coeficiente de x n em f(x) (denotado por x n f ( x)) 3 n 2 n . Mas

[x ]f ( x) = a
n

[ ]

, pela definio de f(x), logo a n = 3 2 .


n n

~ A~

Utilizando sries formais, encontre "formulas fechadas" para as seguintes seqncias:


F0 = 0, F1 = 1, Fn + 2 = Fn +1 + Fn para n 0 (esta a famosa seqncia de Fibonacci) b) t 0 = t1 = 1, t n + 2 = 2t n +1 4t n para n 0 c) p 0 = p1 = 1, p 2 = 0, p n + 3 = 7 p n +1 6 p n para todo n 0

a)

(As outras seqncias no tem nome. Alguma sugesto?)


~ A~

Calcule F0 10
0

F1 10
1

F2 10
2

F3 10 3

+ ...

em que Fn denota a seqncia de Fibonacci.

Outra aplicao das sries formais ajudar a contar. Neste contexto, as sries formais recebem o nome de funes geratrizes. O esquema geral o seguinte: o exponente de x quantifica alguma propriedade em que estamos interessados,
EUREKA! N11, 2001

35

Sociedade Brasileira de Matemtica

como o comprimento de uma seqncia, o nmero de conjuntos em uma partio, a quantidade de duendes verdes em um jardim, etc. Se para cada objeto associarmos tal potncia de x e somarmos estas potncias, o coeficiente de x n ser, respectivamente, o nmero de objetos com comprimento n, o nmero de parties com n conjuntos, o nmero de jardins com n duendes verdes, etc. Por exemplo, considere o problema de determinar o nmero de maneiras de se escrever n como soma de termos 1, 2, 3, sem levar em conta a ordem dos termos. A idia aqui no tentar obter este nmero para um valor particular de n. Somos mais ousados: vamos obter todos estes nmeros de uma s vez. Para isto, escrevemos a funo geratriz f(x) que a soma das potncias xs para cada soma s: f ( x) = x 0 + x 1 + x 1+1 + x 2 + x 1+1+1 + x 1+ 2 + x 3 + ... = 1 + x + 2 x 2 + 3x 3 + ... ( x 0 corresponde soma sem nenhum termo), de modo que, por exemplo, o coeficiente de x 3 o nmero de maneiras de escrever 3 como soma no ordenada de termos 1, 2, 3. Aparentemente, obter f ( x) uma tarefa mais difcil do que a inicial. Mas observe que cada termo de f ( x) o produto de um termo da forma xsomas de 1's , um termo da forma xsomas de 2's e um termo da forma xsomas de 3's, logo f ( x) = ( x0 + x1 + x1+1 + x1+1+1 + ...)(x0 + x2 + x2+2 + x2+2+2 + ...)(x0 + x3 + x3+3 + x3+3+3 + ...) f ( x ) = (1 + x + x 2 + x 3 + ...)( 1 + x 2 + x 4 + x 6 + ...)( 1 + x 3 + x 6 + x 9 + ...) 1 1 1 f ( x) = 2 1 x 1 x 1 x3 O problema agora encontrar x n f ( x ), o que pode ser feito utilizando-se as tcnicas j vistas.
~ A~&

[ ]

Mostre que o nmero de parties no ordenadas de n com exatamente k termos distintos 1 + x j ( y 1) xn yk 1 x j j 1

EUREKA! N11, 2001

36

Sociedade Brasileira de Matemtica

~ A~&

a) Encontre constantes a, b, c, d, e, f tais que 1 1 1 a b c d e f = + + + + + f ( x) = 2 3 3 2 1 x 1 x 1 x 1 x 1 + x 1 x 1 2 x (1 x ) (1 x ) em que = ( 1 + i 3 ) / 2 . b) Mostre que x n f ( x) = (n + 3) 2 7 (1) n n + 2 n (n + 3) 2 1 + + = + 12 72 8 9 2 12 em que x denota o maior inteiro menor ou igual a x.

[ ]

~ A~

a) Determine a funo geratriz do nmero de solues da equao x1 + x 2 + ... + x k = n , em que x i so inteiros positivos, 1 x i k . b) Determine a funo geratriz do nmero de parties ordenadas de n. (por exemplo, 4 = 1 + 3 = 3 + 1 = 2 + 2 = 2 + 1 + 1 = 1 + 2 + 1 = 1 + 1 + 2 = 1 + 1 + 1 + 1, de modo que h 8 parties ordenadas de 4) c) Determine o nmero de parties ordenadas de n. Os prximos problemas mostram uma tcnica muito importante chamada convoluo. Ela se baseia no seguinte fato: se f ( x ) = f 0 + f 1 x + f 2 x 2 + f 3 x 3 + ... g ( x ) = g 0 + g 1 x + g 2 x 2 + g 3 x 3 + ... ento h ( x ) = f ( x ) g ( x ) a srie formal associada seqncia h 0 = f 0 g 0 ,
h1 = f 0 g 1 + f 1 g 0 , h 2 = f 0 g 2 + f 1 g 1 + f 2 g 0 , ..., h k =
~ A~
i+ j=k

fg
i

,...

a) Mostre que se f(x) a funo geratriz da seqncia ( a n ) n 0 , ento f ( x ) /(1 x ) a funo geratriz da seqncia b n = a 0 + a1 + ... + a n . b) Prove que F0 + F1 + ... + Fn = F n + 2 1, em que Fn denota a seqncia de Fibonacci. (Este resultado pode tambm ser provado facilmente sem o uso de sries formais. Tente!)
EUREKA! N11, 2001

37

Sociedade Brasileira de Matemtica

~ A~

Prove que n n n n n F0 + n 1 F1 + n 2 F 2 + ... + 0 Fn = F2 n em que, adivinhe, Fn denota a seqncia de Fibonacci!


~ A~

a) Mostre que o nmero de triangulaes Tn (por diagonais que no se interceptam fora dos vrtices) de um polgono convexo de n vrtices satisfaz T n +1 = T 2 T n + T3 T n 1 + ... + T n T 2 em que T2 = 1. b) Prove que T ( x ) = T 2 + T3 x + T 4 x 2 + ... = c) Mostre que T n = 1 2n 2 . n 1 n 1 1 (1 4 x ) 1 / 2 2x

(T n +1 o assim chamado n-simo nmero de Catalan). Para isto, lembre-se da frmula do binmio de Newton generalizada:

(1 + x ) r =

k x
k 0

r r ( r 1)( r 2 )...( r k + 1) ,k k = k!

e r

Muitas vezes, as funes geratrizes so utilizadas no para calcular o nmero exato de maneiras de se fazer isto ou aquilo, mas para mostrar que duas quantidade so iguais. Vamos mostrar que o nmero de parties (no ordenadas) de n em naturais distintos igual ao nmero de parties (tambm no ordenadas) de n em naturais mpares. Por exemplo, 7 = 5 + 1 + 1 = 3 + 3 + 1 = 3 +1 + 1 + 1 + 1 = 1 + 1 + 1 + 1 + 1 + 1 + 1 e 7 = 6 + 1 = 5 + 2 = 4 + 3 = 4 + 2 + 1, de modo que em ambos os casos o nmero de parties em naturais distintos : (1 + x )(1 + x 2 )(1 + x 3 )(1 + x 4 )... enquanto que o nmero de parties em naturais mpares dado por

EUREKA! N11, 2001

38

Sociedade Brasileira de Matemtica

(1 + x + x1+1 + x1+1+1 + ...)(1 + x 3 + x 3+3 + x 3+3+3 + ...)...(1 + x 5 + x 5+5 + x 5+ 5+5 + ...)... = (1 + x + x 2 + x 3 + ...)(1 + x 3 + x 6 + x 9 + ...)...(1 + x 5 + x 10 + x 15 + ...)... 1 1 1 = ... 3 1 x 1 x 1 x5 Observe que as expresses acima so iguais! De fato, para se convencer disto, basta multiplicar as igualdades
1 x2 1 x4 1 x6 2 = 1 + x, = + = 1 + x 3 ,... 1 , x 2 3 1 x 1 x 1 x

Isto completa a demonstrao.


~ A~

a) Escreva a funo geratriz do nmero de maneiras de escrever um nmero n como soma de potncias distintas de 2. b) Verifique que a funo acima igual a 1/(1 x). c) Utilizando os resultados acima, mostre que todo nmero pode ser escrito de maneira nica em base 2.
~ A~

Prove que o nmero de parties de n em que apenas as partes mpares podem ser repetidas igual ao nmero de parties de n em que nenhuma parte aparece mais do que trs vezes.
~ A~

Prove que o nmero de parties de n com uma nica parte menor (ela ocorre uma nica vez) e parte maior no mximo duas vezes a parte menor igual ao nmero de parties de n em que a maior parte mpar e a menor parte maior do que metade da parte maior.
~ A~

Mostre que o nmero total de 1's nas parties de n igual soma dos nmeros de partes distintas em cada partio de n.

EUREKA! N11, 2001

39

Sociedade Brasileira de Matemtica

OMOMMM&M

O comit editorial de AZA ! agradece aos admiradores da seo Olimpadas ao redor do Mundo o envio de crticas, solues e possveis erros nos enunciados dos problemas. Diversos leitores apontaram um possvel erro no enunciado de:
` d ddd Seja n um nmero natural tal que 2n 2 possui 28 divisores

distintos e o nmero 3n 2 possui 30 divisores distintos. Qual o nmero de divisores do nmero 6n 2 ? Na realidade o problema est errado mesmo, apesar de corresponder verso publicada em : http://www.olsedim.com/olympiad/99.html De fato, se 2n2 tem 28 divisores, 3n2 s pode ter 24, 42 ou 54 divisores (prove!). O problema parecido com a 29. questo da P7
onde os nmeros que possuam 28 e 30 divisores eram respectivamente 2n e 3n, e pedese o nmero de divisores de 6n (Esse tem soluo!). Continuamos salientando que estamos disposio na OBM para aqueles que estiverem interessados na soluo de algum problema particular. Para tanto, basta contactar a OBM, atravs de carta ou e-mail.


d

5 5

Primeiramente vamos aos problemas propostos deste nmero Os coeficientes a e b da equao x 2 + ax + b = 0 e suas razes so quatro nmeros distintos. possvel determinar a equao usando estes quatro nmeros? Um inteiro positivo n chamado perfeito se a soma de todos os seus divisores, excluindo n, igual a n. Prove que se um nmero perfeito maior do que 28 divisvel por 7 ento ele divisvel por 49. Prove a existncia de nmeros reais distintos a1 , a 2 , ... , a10 tais que a equao: (x a1 ) (x a 2 ) ... (x a10 ) = (x + a1 ) (x + a 2 ) ... (x + a10 ) possua exatamente 5 razes reais distintas.

!#"
$%$

dd

!#"
$%$

dd

!#"
$%$

dd

EUREKA! N11, 2001

40

Sociedade Brasileira de Matemtica

O crculo inscrito no tringulo ABC possui centro O e tangencia o lado AC no ponto K. Um segundo crculo S com centro no mesmo ponto O intersecta todos os lados do tringulo ABC. Sejam E e F os pontos de interseo de S com os lados AB e BC que esto mais prximos do vrtice B; B1 e B2 so os pontos de interseo de S com o lado AC com B1 mais prximo de A. Se P o ponto de interseo dos segmentos B 2 E e B1 F , mostre que os pontos B, K e P so colineares. condio: + + + = (a1 + a 2 + + a n ) para todo n , 1 n 2000 . Mostre que todo elemento da seqncia um nmero inteiro.
3 a1 3 a2 3 an 2

!#"
$%$'& (%)

dd%*

+!,#-
.%. & (%)

dd%*

A seqncia de nmeros reais

(a1 , a 2 ,..., a 2000 )

satisfaz a

Cada um dos nmeros 1, 2, 3, ..., N preto ou branco. permitido mudar simultaneamente as cores de quaisquer trs dos nmeros se um deles a mdia aritmtica dos outros dois. Determine os valores de N para os quais possvel fazer com que todos os nmeros fiquem brancos. Determine todos os restos possveis da diviso do quadrado de um nmero primo com 120 por 120. Em um inteiro positivo M, de trs algarismos, o algarismo das centenas menor do que o algarismo das dezenas e o algarismo das dezenas menor que o algarismo da unidades simples. A mdia aritmtica de M com todos os nmeros de trs algarismos obtidos pela reordenao dos algarismos de M termina em 5. Determine tais nmeros M. Gustavo convidou um nmero mpar de pessoas para a festa de seu aniversrio e os disps em torno de uma mesa circular de modo que os vizinhos de cada menina fossem meninos, os vizinhos de cada menino, exceto Gustavo cujos vizinhos eram ambas meninas, fossem uma menina e um menino. Mostre que : a) o nmero de meninos convidados para a festa divisvel por 4. b) na direo diametralmente oposta a Gustavo est sentada uma menina . Seja H o ortocentro de um tringulo acutngulo ABC com AC BC . A reta que passa pelos pontos mdios de AB e HC intersecta a bissetriz do ngulo ACB no ponto D. A reta HD passa pelo circuncentro do tringulo ABC. Determine a medida do ngulo ACB .

+!,#-
.%. & (%)

dd%*

d d

+!/0.2143 5 & (') +!/0.2143 5 & (')

dd'* dd'*

+!/0.2143 5 & (')

dd'*

dd

+!/0.'6 7980:%5 & (%)

dd%*

EUREKA! N11, 2001

41

Sociedade Brasileira de Matemtica

dd

Sobre uma mesa existem pilhas de moedas. Um movimento consiste em escolher uma das pilhas com pelo menos trs moedas, pegar uma moeda desta pilha, retir-la da mesa e finalmente dividir as moedas restantes desta pilha em duas outras pilhas (no necessariamente do mesmo tamanho). possvel obtermos somente pilhas com trs moedas aps vrios movimentos se comearmos com uma nica pilha com 2000 moedas? Determine todas as funes f : ; ; condio f (x f ( y )) = 1 x y para todos x, y ; .

+!/0.'6 7980:%5 & (%)

dd%*

dd

+!/0.'6 7980:%5 & (%) +!/0.'6 7980:%5 & (%)

dd%*

que satisfazem a

Sobre uma mesa esto trs caixas com pelo menos uma bola em cada uma delas. Um movimento consiste em duplicar o nmero de bolas de uma das caixas de modo que o nmero de bolas necessrio para tal seja retirado de uma das outras duas caixas. possvel que aps vrios movimentos uma das caixas esteja vazia? Determine todos os pares de nmeros inteiros (x, y) que satisfazem a equao: y x 2 + 36 + x y 2 36 + y 2 ( y 12 ) = 0

dd%*

+ & < 6 7
=>- .'. & (') + & < 6 7
=>- .'. & (')

d'*

) (

dd

Seja M o ponto de interseo das diagonais AC e BD de um quadriltero convexo ABCD. Seja K o ponto de interseo do prolongamento do lado AB, no sentido de B para A, com a bissetriz do ngulo ACD . Sabendo que MA MC + MA CD = MB MD , prove que BKC = CDB . Para cada subconjunto no vazio X do conjunto 1,2,...,2000}, seja a X a soma do menor com o maior elemento de X. M ={ Determine a mdia aritmtica de todos tais nmeros a X assim obtidos. Determine o valor mximo do produto xy se os nmeros reais x 2 e y satisfazem a relao: y (1 + x 2 ) = x 1 4 y 1 . Dois crculos se intersectam nos pontos M e N. A reta que passa por M intersecta os crculos nos pontos A e B de modo que M (AB ) . Os pontos C e D so os pontos mdios dos arcos AN e BN respectivamente e que no contm o ponto M e os pontos K e L so os pontos mdios dos segmentos AB e CD respectivamente. Prove que CL = KL.

d'*

dd

+ 7 6 ? @28 & (')

d'*

dd

+ 7 6 ? @28 & (')

d'*

dd

+ 7 6 ? @28 & (')

d'*

EUREKA! N11, 2001

42

Sociedade Brasileira de Matemtica

dd

+!A0B
C D (')

Seja q(n) a soma dos algarismos de n. Qual o valor de ? q q q 2000

(((

dd'*

2000

)))

dd

+!EF>G'H & (')

Determine o nmero primo p para o qual o nmero 1 + p + p + p 3 + p 4 um quadrado perfeito.


2

dd'*

dd

Sejam P (x ) e Q(x ) dois trinmios quadrticos tais que trs das razes da equao P (Q (x )) = 0 so os nmeros 22 , 7 e 13 . Determine a quarta raiz desta equao.

+!,#I'J -
K 6 & H ( .MLNH'O I'H ( IP/0.%6 7Q8 ( H (')

d'*

dd

Sabendo que a , b e c so nmeros reais positivos, resolva o sistema no conjunto dos nmeros reais positivos : xy + xz x = a yz + yx y = b zx + zy z = c dd'* ( 7
6 3 5 & (') Uma seqncia ( p1 , p 2 ,..., p n ) de nmeros primos satisfaz seguinte condio: para n 3 , p n o maior divisor primo de p n 1 + p n 2 + 2000. Mostre que a seqncia ( p n ) limitada.

ABC um tringulo do Plano Cartesiano cujos vrtices so pontos de coordenadas inteiras. As medidas de dois dos lados AB, BC e CA pertencem ao conjunto 17 , 1999 , 2000 . Qual o valor mximo possvel da rea de ABC ?

+!,#I'J -
K 6 & H ( .RLNH'O I'H ( IS/0.%6 7Q8 ( H (')

d'*

+!T .'F ( I%6 )

d'*

dd

Sejam P e Q os pontos de tangncia da tangente comum a dois crculos C1 e C 2 que se intersectam nos pontos M e N sendo N mais prximo de PQ do que M. Mostre que os tringulos MNP e MNQ possuem reas iguais.

+!T 5
U 6 ( 14I'F>F (')

dd'*

dd

+!T 5
U 6 ( 14I'F>F (') +!V#6 I'W ( 5
O (')

dd'*

Quais os inteiros positivos a e b tais que

( a+
3

b 1 = 49 + 203 6 .

dd

Determine os nmeros reais x tais que: x 2 x 1 3 5 7 9 11 13 = x 2 2 x 48

dd'*

EUREKA! N11, 2001

43

Sociedade Brasileira de Matemtica

dd

Um quadriltero convexo ABCD est inscrito em um semicrculo de dimetro AB. Sejam S o ponto de interseo de AC e BD e T o p da perpendicular baixada de S a AB. Mostre que ST divide o ngulo CTD ao meio. As medidas dos ngulos B e C de um tringulo issceles ABC so iguais a 50 . Sejam D e E pontos sobre os lados BC e AC respectivamente de modo que BAD = 50 e ABE = 30 . Determine a medida do ngulo BED . o conjunto de todos os inteiros positivos. Prove ou ) Seja X disprove a seguinte afirmao : Existe uma funo f : X X tal que a igualdade f ( f (n )) = 2n verdadeira para todo n Y .

+!V#6 I'W ( 5
O (')

dd'*

dd

+!T 5
U 6 ( 14I'F>F (')

dd'*

dd

+ 7
6 3 5 & (')

dd

Z[Z\Z
Agora vamos aos comentrios e solues dos leitores para alguns dos problemas apresentados em nossa seo nos nmeros anteriores da ]^_]a`cbed f

Seja f (x ) = x 3 3 x + 1 . Determine o nmero de solues reais e distintas da equao f ( f (x )) = 0 .

+ B
6 U
g%F & (')

d'*

hijkl
m inoqpo rts
jnivuo
rtjwxiy{zxwicr|hm ivus
kji}hu~2
A resposta 7. Utilizando clculo, a derivada de f (x ) 3x 2 3 , e ento seus pontos crticos ocorrem em ( 1,3) e (1,1) , respectivamente mximo e mnimo. Mas f ( 2 ) = 1 , f (0) = 1 e f (2 ) = 3 assim, as razes de f pertencem aos intervalos ( 2,1) , (0,1) e (1,2) . Se contarmos o nmero de vezes que f atravessa completamente cada intervalo obteremos a nossa resposta. Com efeito, f atravessa ( 2,1) uma vez (quando x < 2 ), atravessa o intervalo (0,1) trs vezes (quando 2 < x < 1 , 0 < x < 1 e 1 < x < 2 ); atravessa o intervalo (1,2) trs vezes tambm e portanto a equao f ( f (x )) = 0 possui 7 razes.

EUREKA! N11, 2001

44

Sociedade Brasileira de Matemtica

+!,#I'J -
K 6 & H ( .L#H%O
I%H ( IR/0.%6 798 ( H (')
x x x x = 88 .

dd

) Determine todos os nmeros reais x tais que


 t {
t 
ct 
tt
'2
22 . Com efeito, 7 88 88 3 x x x = x da forma com k x . x k 1) Se x 0 ento x 3 pois, se x < 3 teramos A resposta

x x x x < 3 4 = 81
Como 88 1 = 29 tem-se que 29 x x x 27 e da : 3 3

x x x = 27 x = 27 (no serve)
88 22 = 28 7 88 x x x = 29 x = (no serve) 29 2) Se x 0 ento

88

x x x = 28 x =

x < 3 x 4 x x x x 3 3 4 > 88
por outro lado,

x 3 x 3 x x x x 3 4 = 81
e portanto, no h solues negativas.
d

+!07
6 3 5 & (')

dd

) Prove ou disprove a seguinte afirmativa :

Todo nmero racional positivo pode ser escrito sob a forma

a 2 + b3 onde a , b , c5 + d 7

c e d so inteiros positivos.


 t vt 0
t v

c2
A afirmativa verdadeira. Fazendo a = x 3 y 2 , b = x 5 y 2 , c = xy e d = x 2 y para quaisquer inteiros positivos x e y , temos:
EUREKA! N11, 2001

45

Sociedade Brasileira de Matemtica

a 2 + b3 x3 y 2 + x5 y 2 = 7 c5 + d 7 (xy )5 + x 2 y

) ( ) ( )
2

x 6 y 4 + x 15 y 6 x = x 5 y 5 + x 14 y 7 y

Como todo nmero racional pode ser obtido pelo quociente de dois nmeros inteiros positivos, a afirmao est provada.

) Um nmero de 10 algarismos dito interessante se todos os seus algarismos so distintos e ele um mltiplo de 11111. Quantos nmeros interessantes existem ?

!#
%' %%'


 t vt{c t
vc t
qc2
A resposta 3456 . Seja I um nmero interessante ento I 0 + 1 + 2 + + 9 0(mod 9 ) Logo, I = 99999 N = 10 5 1 N para algum nmero natural N de 5 algarismos. Digamos, N = a1 a 2 a 3 a 4 a 5 logo, I = 10 9 a1 + + 10 5 a5 10 4 a1 10a 4 a5 I = 10 9 a1 + + 10 6 a 4 + 10 5 (a 5 1) + 10 4 (9 a1 ) + + 10(9 a 4 ) + 10 a 5 . Sejam d1 , d 2 ,..., d 9 , d10 os dgitos de I, nesta ordem, ento d1 + d 6 = 9 , d 2 + d 7 = 9 , d 3 + d 8 = 9 , d 4 + d 9 = 9 e d 5 + d10 = 9 . Como os nicos pares de dgitos cuja soma 9 so (0,9), (1,8), (2,7), (3,6) e (4,5) o nmero de possibilidades para d1 , d 2 ,..., d 9 , d10 9 8 6 4 2 1 1 1 1 1 = 3456

'

!#>' '!''%' Resolva a equao

1 1 + 2 x 4 3x

=1


q t
v
t v#
vc2
As solues da equao so 2 3 (1 + 2 cos 80 ) . 3 2 3 2 3 (1 2 cos 20 ) , 2 3 (1 + 2 cos 40 ) e , 3 3 3

EUREKA! N11, 2001

46

Sociedade Brasileira de Matemtica

Com efeito fazendo-se 4 3 x = y + 2 temos que x =


x 4 3

2 y 3

para x 0 e

. Fazendo as devidas substituies e simplificando chegamos a

2 3 e y 3 12 y 8 = 0 . 3 Aplicando-se a frmula da equao do terceiro grau nesta ltima temos: y y 3 12 y 8 = 0 o que implica em y = 0 x =
8 8 8 12 8 12 + + +3 + 2 2 3 2 2 3
2 3 2 3

y=3

ou
3 3 3 y = 3 4 1 + 3i + 1 3i =

onde = 2k + outras razes.

(1 + 6k ) . Da, y = 4 cos para k = 0,1,2 e assim obtemos as 3 9

3 3 4 2cis 3 + 2cis 3

Obs: Tambm possvel chegar s solues fazendo

1 1 3 1 = cos , = sen + =4 x cos sen 4 3x


3 1 sen + cos = 2 sen cos sen( + 30) = sen(2 ) . Podemos fazer 2 2 as mesmas = 30, = 50 , = 170 ou = 290 , e obtemos

solues

(ainda que escritas numa outra forma:

2 3 1 2 3 1 1 = = , (1 + 2 cos 40) = , 3 cos 30 3 cos 290 cos 70 2 3 1 2 3 1 1 e ). = (1 2 cos 20) = (1 + 2 cos 80) = 3 cos 50 3 cos170 cos10

EUREKA! N11, 2001

47

Sociedade Brasileira de Matemtica

%'

!0' '
%

'%'

(a 1 , a 2 ,..., a n ,...) tal que

Mostre que existe uma seqncia de inteiros positivos

2 2 + ... + a n um quadrado perfeito para todo a12 + a 2

inteiro positivo n .


 t vt{c t
vc t
qc2
Consideremos a seguinte seqncia:
a1 = 3 e a n =

(a )
k k =1

n 1

, n {2,3,4,...}

Desta forma teremos :

(a )
k k =1

n 1 2 2 2 2 = (a k ) + (a n ) = (2a n + 1) + (a n ) = (a n + 1) k =1

o que conclui a demonstrao. (note que an sempre par, e tambm igual a a n 1 (a n 1 + 2 ) , para todo n 3). 2

%'

!0' '
%

'%'

Determine o maior nmero inteiro N que satisfaz as seguintes

condies :
N (a) possui seus trs algarismos iguais. 3 N (b) igual soma de n nmeros naturais consecutivos a partir de 1. 3


 
ct
q cqt c0

c2
De acordo com as condies (a) e (b), tem-se que

N n(n + 1) = 111 k , k ; 1 k 9 3= 2
ou seja, n= 1 + 1 + 8111k 2

EUREKA! N11, 2001

48

Sociedade Brasileira de Matemtica

Como n natural, o radicando deve ser um quadrado perfeito o que ocorre somente para k = 6 que substitudo na expresso anterior nos fornece n = 36 e N N da, = 111 6 = 666 667 > 666 2001 > N 1998 3 3 e portanto o maior N que satisfaz s condies dadas 2000.
Atendendo a um pedido especial do leitor Jos Renato Carneiro e Carneiro antecipamos a soluo de:

%'

!0' 90%
%%''% Determine todos os inteiros x e y que satisfazem equao
x 3 + 9 xy + 127 = y 3 .



 
tvt
tc
{'c2
As solues da equao so (3,7 ) e ( 7,3) . Fazendo-se y = x + a , substituindo-se e simplificando a equao proposta chegamos a: (9 3a )x 2 + 9a 3a 2 x + 127 a 3 = 0 (*) Esta equao deve possuir solues reais para possuir razes inteiras. Seu discriminante :
D = (9 3a ) 4 (9 3a ) 127 a 3 = (9 3a ) a 3 + 9 a 2 508
2

Se a 6 ento a 3 + 9a 2 540 e D < 0 . Para 2 a 2 temos 9 3a > 0 e para a 9, a 2 (a + 9) 0 < 508. Assim, D 0 somente para a { 3,4,5}. Para a 3 + 9a 2 = a 2 (a + 9) 4 11 < 508 . Para 8 a 3, a 2 (a + 9) 64 6 < 508, e

a = 3 obtemos uma contradio em (*) . Para a = 4 , a equao x 2 + 4 x 21 = 0 satisfeita com x = 3 e x = 7 . Para a = 5 a equao 3x 2 + 15 x 1 = 0 no tem nenhuma soluo inteira. As nicas solues da equao proposta so portanto (3,7 ) e ( 7,3) .



EUREKA! N11, 2001

49

Sociedade Brasileira de Matemtica

Enviaram solues de problemas os seguintes leitores da  !

c% 4#
c> $ # / 0 ># %
1  5   % 4 4%  71  94    4 4%: ; <= ? > A @ 5 % A   5 % A   N= D { 5  E @ %
1 F  HG %
I 1 @ J  { K4= D L5E M @'{ N 
@ %   %
1 @ % 
O N P ;  @ % > A @ %>  % ;' M @6R%  & @   % ;4% M @ = & < 0 4% 6
1 @ E SQ  {  c>{ 4 @ %$V {7  P$JU % & 4= #c 4 c = D
W % 6
1 36 %=   M  { 3N X  % % 6 H 
Y % JZ  @4@
%

t 4 % '  # 4   < % 1 2 9 < % 1 2 9 t '%  % 4& t '%  % 4&  ' 6 % % ;4{  / % % ;4{  / 1 <  @ 5  # 4   / Q  #   =I #   =I  # 4   @  TE G P > EV 2%  #   =I < % 1 2 9 < % 1 2 9 #   =I

      & '%  2


3    6

      !"  ()*+,*+-*.+  (.*+"*+,*."*.*(.+  (.*+*+.*.)  (*,*+*.)*.8*(!"  ()**+*+"*+,*(.*.8*.*!"  (..  BC.*+"    =   (!"  ()*+ (,*-*!*+"*+*(+.*."*.=*.)*.8*..  (8)*)*+=J*+-*+*(+.  (-  (*+,  (*)*,*-*+8*(+*.)*.-*..*.!  ()*!*+*+!*."*(.)  ()*-*.*+)  (*+ ()*,  BC)*)8*,)*-,*)*(.*+"*+8*.)*.,  (,,*-*-+*8=*8)*(8  (*+*.*+"*+,*(.)*.  ()*.*+ (*)*,*-*.*(!*+)*+*+.*."*.)*.+

! !

' 0

[&\$]_^a` bEcZd bfe gih$jlkmEn m_k o:p_qEmlrsout vxw_jy m$vzjr : L_:__$  L _::L {4|Z|&}~jLp_k$m:p$yo4rm$voEn 4h$ro_v

ffffQZ_fffZ: $E : ffffZf&Qf_fZfffB: $$u $ ffffZff&f_ZffffZJ: $$u $ ffffZffffQZ_fffZffffE2J_ $'i$u :

u $:L_ $L $ : _E__$_ fL$L:_ $:4_ $$:_ L _

EUREKA! N11, 2001

50

Sociedade Brasileira de Matemtica


iiiiiQi L
      H  

48) Doze pintores vivem em doze casas construdas ao longo de uma rua circular e so pintadas ou de branco ou de azul. Cada ms um dos pintores, pegando consigo bastante tinta branca e azul, deixa sua casa e caminha ao longo da rua no sentido anti-horrio. Desta forma, ele repinta cada casa (iniciando na sua) com a cor oposta. Finaliza o trabalho to longo repinte alguma casa branca de azul. Em um ano, cada casa estar pintada com a sua cor original sabendo que, no comeo do ano, ao menos uma casa estava pintada de azul.


 c  t
"! #%$'&  '( c{tv)(
2
Dizemos que uma pintura de uma casa induzida se no feita pelo dono da casa. Isso s acontece quando o pintor acabou de mudar a cor da casa imediatamente anterior de azul para branca. Vamos mostrar que cada casa muda de cor exatamente duas vezes, sendo uma induzida e outra no, o que claramente implica o resultado. Primeiro vamos ver que uma casa no pode mudar de cor trs vezes. Para isso, supomos por absurdo que alguma casa muda de cor pelo menos 3 vezes, e consideramos o primeiro momento em que uma casa muda de cor pela terceira vez. Nesse caso, pelo menos duas das 3 pinturas so induzidas, e portanto a casa imediatamente anterior muda duas vezes de azul para branca antes disso, e portanto muda de cor pelo menos 3 vezes antes desse momento, o que uma contradio. Para terminar, vamos agora ver que cada casa muda de cor duas vezes, o que equivale a dizer que muda (pelo menos) uma vez de forma induzida. Para isso, basta ver que toda casa muda alguma vez de azul para branca (pois uma casa muda de forma induzida quando a casa anterior muda de azul para branca ela muda de forma induzida). Isso obvio para casas azuis. Vamos provar isso por induo no nmero de casas imediatamente anteriores que so inicialmente brancas. Para a casa imediatamente anterior esse nmero menor (lembre que h pelo menos uma casa azul no incio), e portanto em algum momento ela muda de azul para branca, e nossa casa muda de forma induzida, donde muda de cor pelo menos duas vezes, e portanto em algum momento muda de azul para branca J

EUREKA! N11, 2001

51

Sociedade Brasileira de Matemtica

Obs. Note que essa soluo no depende da ordem em que os pintores executam seu trabalho (depois de um pintor acabar de pintar, o prximo no precisa ser seu vizinho, mas importante que cada pintor faa o processo uma vez). 52) Quatro retas se interceptam formando quatro tringulos conforme a figura abaixo.
A D

a) Prove que as circunferncias circunscritas aos quatro tringulos possuem um ponto em comum. b) Prove que os centros dessas quatro circunferncias so concclicos (i.e. existe uma circunferncia que passa por todos eles).


 * t
+(
0 +&
-, /.00
 )( 2
A P

Seja P o outro ponto de interseo dos circuncrculos aos tringulos AED e FEB. Temos que #FBEP inscritvel (# denota quadriltero nesta soluo), logo B = BP E . Da mesma forma EP D A EF
EUREKA! N11, 2001

52

Sociedade Brasileira de Matemtica

D AE D FE B , mas no tringulo FBE temos (#ADEP inscritvel) e AP +C e FE +C + EF +C ), B 180 ( A B ) FE B + EF B 180 ( A E A FB 180 A C +A 180 C , A AP D + EP D + BP E FE B + EF B + A mas como BP portanto #APBC inscritvel o que implica que o circuncrculo ao tringulo ABC B FE B (# FBEP inscritvel), logo passa por P. Temos ainda que FP , o que nos d que o #FPDC tambm inscritvel, com D BP A 180 C FP isso os circuncrculos aos tringulos ABC, FDC, FEB e AED possuem o ponto P comum. b)
A P O2 O1 I2 E I1 O3 D

B O4

Sejam: O1: Centro do circuncrculo a BEF ; O2: centro do circuncrculo a ADE ; O3: centro do circuncrculo a ABC ; O4: centro do circuncrculo a CDF ; Lema: O segmento determinado pelos pontos de interseco de dois crculos perpendicular ao segmento determinado pelos centros dos crculos:

C1 N

C2

EUREKA! N11, 2001

53

Sociedade Brasileira de Matemtica

C NC C o que significa que C C C1 MC 2 C1 NC 2 (L.L.L), logo MC 2 1 2 1 1 2 bissetriz de MC 2 N , mas como este tringulo issceles ento C1C 2 tambm altura e C1C 2 MN (c.q.d). Do lema acima temos PB O1O3 e PE O1O2 , e portanto B P E = O2 O1 O3 e E BF E (subentendem o mesmo arco BE no crculo de centro O1). BP O , mas DP C O O C DF C (subtendem o mesmo arco Analogamente DP
2 4 3

C BF E temos DC no crculo de centro O4). Como DF C BF E BP E O O DP 2 1 O3 O2 O4 O3 O2 O1O3 . Com isso provamos que #O1O2O3O4 inscritvel.

53) Prove que num crculo convexo dado e para o mesmo nmero de lados, o polgono regular inscrito aquele cuja superfcie mxima.



 * c tv
1 
325456%7/8%4)9 2
a) Observe inicialmente o polgono convexo A1 A2 A3An no crculo de centro O.
B2 A2 B1 A1 A3 Bn O An
: ;

B3

A4

Tracemos os segmentos OA1 , OA 2 , OA 3 ,..., OA n que encontram a circunferncia nos pontos B1, B2, B3,,Bn; respectivamente. Observe que o polgono B1B2B3Bn tem superfcie maior que o polgono A1A2An; isto nos garante que o polgono de rea mxima deve ser inscrito na circunferncia.

EUREKA! N11, 2001

54

Sociedade Brasileira de Matemtica

b) Considere agora, o polgono convexo inscrito de n lados como na figura abaixo:


B2 B1 B3

1 n
Bn O

2 3
B4

<

A rea S do polgono dado por S =


n

R2 (sen 1 + sen 2 + ... + sen n ) 2

onde

i = 2 . S ser mximo quando


i =1

sen
i =1

for mximo.

Para este problema, vamos utilizar a "desigualdade de Jensen" que diz: suponha que f com concavidade para baixo (cncava) no intervalo I; ento para todo x1 , x 2 ,..., x n I teremos: f ( x1 ) + f ( x 2 ) + ... + f ( x n ) x + x 2 + ... + x n f 1 n n e a igualdade ocorrer se e somente se x1 = x 2 = ... = x n . Para o nosso problema temos f ( x) = senx e I = ]0, [ onde 1 , 2 ,..., n I ; portanto sen 1 + sen 2 + ... + sen n + 2 + ... + n 2 sen 1 = sen n n n 2 e para o caso mximo: 1 = 2 = 3 = ... = n = ; ou seja, o polgono regular n inscrito.

EUREKA! N11, 2001

55

Sociedade Brasileira de Matemtica


x 54) Sejam ( x n ) a seqncia definida por x1 = 2, x n +1 = 2 n , n 1, e ( y n ) a

seqncia definida por y1 = 2001, y n +1 = 2001 existe c natural tal que y n x n + c para todo n > com essa propriedade.
?@8%45A%BDC8"E%F *)45FD0)8%GHGHIJ'*%GHFA"K32L3FGH6%LMc9DNPO

2001 ( yn )

, n 1. Prove que

e determine o menor c

2001 Provaremos no que x n + c y n mas sim x n + c 44022 y n = 22 2001 y n pois fcil ver que s com o fato xn + c yn no se pode fazer induo. 2001

Escolheremos c de tal modo que x1+ c 22 2001 y12001 . Suponha vlida a desigualdade para um certo k ento
2001 xk +c 22 2001 yk xk +1+c = 2 xk +c > 2 222001 yk ,
2001

2 2001 2001 2001 mas 211 2001 xk +1+ c (2001y k ) 2 2001 = yk = yk +1 +1 yk +1 ,


2001

2001 2001 mas y1 = 2001 y k 20012001 > 22 2001 x k +1+ c 22 2001 y k +1

o que completa a induo. Vemos que c = 4 satisfaz a condio pois x5 = 265536 > 22 20012002 mas
x5 = 2 65536 < y2 = 20012001 j que 65536 < 20012001 < 2 < 2001, portanto c > 3 e para c = 4 funciona c = 4 o menor valor.
2001

55) Seja S o conjunto de pontos interiores de uma esfera de raio 1 e C o conjunto de pontos interiores de um crculo tambm de raio 1. Existe alguma funo f : S C tal que d ( A, B) d ( f ( A), f ( B)) para quaisquer pontos A, B S? (d (A, B) denota a distncia euclidiana entre A e B).
?@8%45A%BDC8"E%FRQSA%L54UT%FGH,+F'V%AXW5LUYZJDGHJ K3?@CD8\[@JA%4U8+M%?@[@NPO

Seja O o centro da esfera e C o da circunferncia. Tome A, B S tal que A, B e O sejam colineares,


EUREKA! N11, 2001

56

Sociedade Brasileira de Matemtica

d ( A; O) = d ( B; O), d ( A; B ) = 2 , 0, > 0. Tome tambm P, Q S tal que P, Q e O sejam colineares, d ( P; O) = d (Q; O), PQ AB e d ( P; Q) = 2 . Agora note que d ( f ( A); f ( B)) 2 f ( A) C f ( B ) 0, analogamente d ( f ( P ); f (Q)) 2 f ( P ) C f (Q) 0.
Como d ( A; P ) = d ( B; P) = 2 1 ento 2
d ( f ( A); f ( P)), d ( f ( B ); f ( P )) 2 1 f ( P) C f ( A) 90. 2

Agora vamos tomar P' e Q' do mesmo modo que tomamos P e Q, porm em um plano perpendicular ao plano de A, B e P. Temos que d ( P ' ; P) = d ( P' ; Q) = 2 1 . 2 Mas como vimos antes, f ( P ' ) C f (Q' ) 0 e f ( P ' ) C f ( A) 90, logo f ( P ' ) f ( P ) ou f ( P ' ) f (Q) d ( f ( P ' ); f ( P )) 0 ou d ( f ( P' ); f (Q)) 0, o que um absurdo, pois
d ( f ( P ' ); f ( P )), d ( f ( P ' ); f (Q)) 2 1 2 J 2
*)]%^H_`%aDba,"8%c'd3_,+eXfD,g8"ah%iDj-8+`%_Dc c8%45k%lDm%ac a _'b8%4-_De%8%^H_lDn8"`Xapo qr s tuv wyx zD{|}z|  uvu vu s z ~y{Dv{

r s Dzys Xu}zR
zvr Dz r s s v

z } r u|wz

u{zDtzw{Xy

z Xy

r zDux |Du r vz

|w vr z r }Pw r zDux |Du r vz

x x

EUREKA! N11, 2001

57

Sociedade Brasileira de Matemtica


iiiQi

y " X " X P +  X  %X \   .

56) Para cada nmero n, seja f(n) a quantidade de maneiras que se pode expressar n como a soma de nmeros iguais a 1, 3 ou 4. Por exemplo, f(4) = 4, pois todas as maneiras possveis so 4 = 1 + 1 + 1 + 1, 4 = 1 + 3, 4 = 3 + 1, 4 = 4. Demonstrar que se n par, f(n) um quadrado perfeito. 57) Dado n nmeros reais x1, x2, , xn satisfazendo as condies 2 = 1 , prove que existem i e j tais que x1 + ... + x n = 0 e x12 + ... + x n 1 xi x j . n 2 p 1 1 o quadrado 58) Determine todos os primos p para os quais o nmero p de um inteiro. 59) Um pedestal de altura a sustenta uma coluna de altura b (b > a). A que distncia do monumento se deve colocar um observador para ver o pedestal e a coluna sob ngulos iguais? 60) Se num tringulo ABC , A = 2B, provar que a 2 = b(b + c). Obs.: a, b e c so, respectivamente, os lados opostos aos ngulos A, B e C. 61) Na figura abaixo um quadrado EFGH foi colocado no interior do quadrado ABCD, determinando 4 quadrilteros. Se a, b, c, e d denotam os medidas das reas dos quadrilteros, mostre que a + b = c + d .

a c d b

EUREKA! N11, 2001

58

Sociedade Brasileira de Matemtica

62) Se ABCD um quadriltero convexo tal que os lados AB, BC , CD e DA medem respectivamente a, b, c e d e que , , e so as medidas dos seus ngulos internos, mostre que a medida da rea desse quadriltero, denotada por (ABCD), dada por:
D

d1

(ABCD) = p=

( p a )( p b)( p c)( p d ) abcd cos 2 onde:

a+b+c+d 2 + + = (a frmula tambm vale se fizermos = ) 2 2

v ztDs u }\\u\\vzzD}wz}+zv~y u vzp{r z s {D u}+zv-w s u ~yXvzDts u  vzDzD}wzDzv vus z r |DzDu"s r Xur v u s y )DvztDs u } u DvzDz}wz}Dzv r zs r X DvztDs u }u DvzDz}PwHzD}zv'~ vs zD} }P s z us s z Dz|D qzDu} w s y

EUREKA! N11, 2001

59

Sociedade Brasileira de Matemtica


i S

@@U5S%5 [@5*))*)*) 5/@ )* *) @' 5 * 2)51 @ D'a [@^Hj5,+aDj5^H_ V/_ca '@DpRDDD %a^HbaDj5^H_ V/_ca @DD'RD5D 3
   @aD]%k%h%`/_ V%_Dca @DD D5'3

  DpyDRD53  R / DD  3 


2)51 @ 2 1 ) 5

[@^Hj5,+aDj5^H_ V/_ca '@DpDD 5R3 @a]/k%h%`%_'V%_cDa '@Dp'   yp'D53

1 5DS%5 [@5*)* * 5 @Dy R

@5DZ/5 [@U*)* *) @ U * @D p)  @%5S%U [@5*))*

) 2

' 

)

2) )2)* U 2)* *) @' 5 *

' !   "  5Dp# 5D$) D 1 S%5 [@5*))* * ' 5 *2)* *) @ U *

'   - 3 % D $&3D


1 Z/5 [@U*)* * 5 *)2)* *) @' 5 * @Dy' D53

 2 1 ) *

EUREKA! N11, 2001

60

Sociedade Brasileira de Matemtica


iiii 7

')(*,+- .0/ 123*)45/ 6 78*9')+ *0:<; 1 ')6 OQP,+ R 1TS3*8.U.8PEVW3*U*82 ')VQ_P06 *L`a*0(*0+ _b1 pbPEV<PU2A/ R 1
qb*U2PErs*8.UtU1VHt8P06 1.vu+ P0/ + P `A*0+6 1b.yu+ P82bP0+/ tU1Tpb1+ _bPU.]zh*06 (P0/ + * `)6 *Er<2A/ 1')+ tU1VHtE~HP,+ `)6 *Er<.yS3*UP8R?/ V<_bP0+ `)6 PU1VQ1+h`)+ PU.tUEVHt,/ 12b*8.y3PU78PU. 6 / 1T,P8_b* Wb13.EVQ_P06 *L401ar<H* u6 1A+ EVQt0/ 1TuHP0++ P,/ + *L
r/ (*0+ UP8.vu/ 6 ~<1 / .P06 PL23P)')+ *E:<; 1&z0+ *8R P*U2b1
rQ.0(81 78*EV/ 6 2PvuHP,+VH*EV<2bPU.L4*8*U2 8*Ut8ArQP,6 / VHPvu<*EO/ 16 *yW31b; *U.)')+ *EVQt,/ Ob/ * 81b81&pbPEV- t0/ 12bPy,P06 1T3P8R 1 81b81&u+ *EV<t0/ .8tU1T,P06 1/ OQ1VH*8R/ + P0V<Pvb*E?*U1A?* 81b.89`A*0+6 1b.yzE/ V<R 1&<P0/ 78*U. 81b.89`)6 137PU.L4*0+ *,/ 7* 81b.89*U.,Q*0+EW,r<*U.yu/ 6 ~H1 81b.8vrb/ W31b.U*U.]zE/ Vb~H1 81b.89s5/ P0/ + *9')6 7UP8. a*0+ tUP,6 1&WAr</ V<1
2PL6 / 78P,/ + * / t0/ 1&SbP,+VH*EV<2bPU.yp3P8HP,++ * rH/ VQ*,6 7U*yL/ + *EV<2b*L23P)'9(1+/ ( a*0+ tU1AV<23P.9`A*U78*06 t8*EV<R Pvu+ *EV<U* zU*EO6 1TW3132+/ _1
*EV<*U.U.0/ ( zU*Er6 1TS3PEVb+f/ ArHPL`)+r<
bP,/ 7*92bPv/ (*90+ WbP0/ V<*06 2b1PEV
t,~b/ + 1')+ *E*E8/ WA/ tU*0+ 21
')(1+/ ( Wb1aO<P0+ R 1
s5/ <P0rpb*,++ 1b. 4E0+ _A/ 1`)6 0rH2A/ 1Wb*E(13. 45/ 6 70/ 1
2Pyp3*0++ 1b.y,P06 1 q*U2bPEruHP0++ P,/ + *L1(PU. q1a(U.y,P0VH0VH23P
Wb1b2A+f/ _ar<PU. sE*06 2PEVOQP,+ _')+ *E:H; 12b*L45/ 6 7U* v*U_VHP0+zUP,+ P0/ + *yH1a<PU. v*06 2bP0(*0+L}`a*0VH*06 6 /

=?>A@<B0C =?>A@<X@HC = cA\Udef^ GL[U\]gh[8Y8i<J8FQj8G9[8\9k0[UY8l e GIKLcLghk0C =?>A@waxbC =?{0>bcA|}wbD G8C = cAGl U8D Gv?\UGUd8J0^ [8G9[8J9BED d8iHD C =?>AxA BEkEAgUoEC =?>bkM)C = cAGl U8D GvgUe JU8J8C = cAGl U8D G9oED dUY8l J0^ C =?>A@gho0C =?>A@<NC =?>bc)U3G8nmhG8IHi<G8C =?>A@{m8C =?>A@{0 C = N^ Y8UGygh[8YiQJ8iHD G8dUJ8l [U\8J8l C =?>Ag8MLC =?>A@w3N5C =?>A@<M9k0C =? cAM)cA|?>3o{8C =?>A@<ocAC =?>A@{m8C = o0D I<e \8nLJ9D e Y8l J0^[\vgUd8I<D dUG8C =?>A@<ocAC =?>A@mhk0C =?>A@<cAC =?D iH\YL3\,^ ^ J8I[UG]ghd8U\Ud8UG8C =?g8IHi<G8l JLb8i<d8D i<JygUQ\,^ J0^ [UGy{hJIHIHGIHC =? xA{0gUC = cA\Udef^ GygU[8Y8iHJiHD G8dUJlb}G8UG8I<C = cAGl U8D G9kEiHJU\C =? M)|?>A@bwbNo0C =?>A@{g8C =?>Ag0mhkEC =?>AU@<\ [8\,^ J8l<[U\ywbGUdU[8Ud8D J8C =?>AU@<\U[8\,^ J8l<[U\LoE\0^ 8D U\C =?g8IHi<G8l JLb8i<d8D i<Jy@Q\U[\0^ J8l<[8\9N5G8D 8I<C =?{0^ \ \8D e Y0^ JLM9Y8d8D iHD UJ8lH[U\Lo}XHG8jUG)[U\LM9\0^ D e D C

BED FHG8I<JLK9MLN XHY8D Z[8\]@QG0^ J9KLMLN m l Y8n9\Ud8JUYLK9o0c xbJ8e J8lHK]w,x wbD GL[U\)XHJUd8\8D ^ G9K]wbX XHY8d[8D J0}KLo5{ ?J8 \JU[8G9Kyw3o MLJUd8JUY8IKLkM oEjUGy{UJ8Y8l GLK)o{ oEJUd8e G)kdU[0^ LK9o5{ BED e 0^ D JLKvgUo NGD Ud8D J)KLN cAJUnLUGLN^ JUdU[\UKLM9o XHG8jGv{U\8I<IHGUJLKv{0m \0^ \8I<D d8J9K]{E mU\8l 8nKv{Uk MLJ0^ D dU89K]{w wbD GLN^ JdU[8\9Kyw3o oEjUGy}YD IK)MLk oEjUGLcaJ0^ l GIKLo{ @<l G0^ D JUd8UUGl D IKLoEc cAJ nL8D dJLN^ J8dU[U\LKy{Em mU\8l 8nKv{Uk @<l G0^ D JUd8UUGl D IKLoEc oEJ8l HJU[8G,^Kym8k @<G0^ e Jl \8ZHJ)K9c)g {UD ^ J8i<D i<J8UJLKLo{ oEX[8G8IcaJ8n9G8IK9o{ oEX[8G8IcaJ8n9G8IK9o{ xbG8HJ] YUJ8FHYLKvwbX BEG8l e Jvwb\U[UG8dU[8J9KywbX {UG0^ e G9kEl \U0^ \LKvwbo wb\8iHD \LKv{Eg XHY8JZH\8D ^ G9KymUk {UG0^ e G9BE\l 8G9Kywb oEjUGLc)^ D I<e G8HjUG)KLog XHJ8e J,}KLN oEX[8\LM9\0^ D e D<KywbX

EUREKA! N11, 2001

61

CONTEDO

AOS LEITORES XLII OLIMPADA INTERNACIONAL DE MATEMTICA Problemas e Solues XVI OLIMPADA IBEROAMERICANA DE MATEMTICA Problemas e Solues

2 3

13

ARTIGOS
COMO FERMAT E BZOUT PODEM SALVAR O DIA Antonio Caminha Muniz Neto GRAFOS E CONTAGEM DUPLA Carlos Yuzo Shine OLIMPADAS AO REDOR DO MUNDO SOLUES DE PROBLEMAS PROPOSTOS PROBLEMAS PROPOSTOS AGENDA OLMPICA COORDENADORES REGIONAIS 25

31

40 49 59 60 61

Sociedade Brasileira de Matemtica

AOS LEITORES
Nesta ltima edio de 2001 publicamos solues dos problemas da Olimpada Internacional e da Olimpada Iberoamericana deste ano, nas quais o Brasil obteve timos resultados, que deixaram toda a nossa comunidade olmpica muito contente. Por falar na comunidade olmpica, lembramos que neste ano foi criado o nvel universitrio da OBM, cujas provas e solues sero publicadas no prximo nmero da Eureka!. Aproveitamos para saudar os novos (e antigos) olmpicos universitrios. Mais uma vez agradecemos o grande nmero de solues e problemas propostos que so enviados pelos leitores, e que nos ajudam a fazer a Eureka!. Como um estmulo adicional a nossos colaboradores, vamos dar um reconhecimento especial aos leitores que resolverem mais problemas propostos,ou aqueles problemas propostos que considerarmos mais difceis, ou que estiverem h bastante tempo sem soluo. Aguardem e continuem colaborando! Finalmente, agradecemos a colaborao de Carlos Yuzo Shine, Eduardo Tengan e Pablo Ganassim, que ajudaram na reviso deste nmero, e desejamos um timo 2002 a todos.

Os editores.

EUREKA! N12, 2001

Sociedade Brasileira de Matemtica

XLII OLIMPADA INTERNACIONAL DE MATEMTICA


Problemas e Solues
PROBLEMA 1

Seja ABC um tringulo acutngulo com circuncentro O. Seja PA uma altura do tringulo com P no lado BC. A AB C + 30 . Considere que BC B + CO P < 90 . Prove que CA
SOLUO DE ALEX CORRA ABREU (NITERI RJ):
A

90 C

90 B

O
A

B = C , AB O = A e CO P = CP = b cos C . Seja M o C = B , CA Sejam AC ponto mdio de BC. M = A P = PO M = A tg A = CM = a Como CO e CO e OM 2OM ) = PM onde BC = a , AC = b , AB = c e R = OA = OB = OC tg ( A OM CM PM CP OM OM A = tg A tg A = = = tg = tg A CM PM OM 2 + CM PM 1 + tgA tg A 1+ 2 OM R cos A b cos C , j que OM = R cos A = CP = b cos C a 2 ab 2 2 R cos A + cos C 4 2

( (

))

EUREKA! N12, 2001

Sociedade Brasileira de Matemtica

a a e CM = a e, pela lei dos senos no tringulo CP = b cos C 2 2 2 ABC b = 2 RsenB e a = 2Rsen A 2 cos A cos A cos C 2R senB cos C 2senB tg = 2 = , mas + R 2 sen 2 A 2 R 2 senA senB 1 2senA senB cos C cos C R cos 2 A senB > 0 tg tgA = 2senB cos C cos A senA = cos C como tg > 0 1 2senA senB cos A cos C 1 2senA cos C senA 2 senB 1 sen A <1 = e como 2x 2 x 1 < 0, x ,1 2 sen 2 A 2 1 2 senBsenA cos C 1 (0,90), mas sen C B B 30 e a funo seno A pois 90 > C 2 sen C B senA (0,90) senA crescente no intervalo 2 2 senA 2 senAsen C B 2 senA 2senAsen C B 2 sen A senA < 1 PM =

( ) (senA sen(C B (sen(B ) + sen(B ))< 1 ))< 1 2 senA +C C 2 senA


senB <1 cos C 4 senA

senB cos C 2 senA pois senB cos C 1 2 senA senB > 0 tgA tg < 1 tg A + = tg + tgA >0 A + < 90. cos C 1 2 senA

PROBLEMA 2

Prove que a a 2 + 8bc + b b 2 + 8ca + c c 2 + 8ab 1

para quaisquer nmeros reais positivos a, b, e c.


SOLUO DE ALEX CORRA ABREU (NITERI RJ):

a a 2 + 8bc

b b 2 + 8ac

c c 2 + 8ba

1 a 2 (b 2 + 8ca)(c 2 + 8ba) + b 2 (a 2 + 8cb)(c 2 + 8ba) +

c 2 (b 2 + 8ca)(a 2 + 8bc) 513a 2b 2 c 2 + 8(a 3b 3 + a 3c 3 + c 3b 3 ) + 64abc(a 3 + b 3 + c 3 ) Elevando ao quadrado, obtemos


EUREKA! N12, 2001

Sociedade Brasileira de Matemtica

(I)
2 513a 2 b 2 c 2 + 8(a 3 b 3 + a 3 c 3 + c 3 b 3 ) + 64abc(a 3 + b 3 + c 3 ) (ab c 2 + 8ab + ac b 2 + 8ac + bc a 2 + 8bc ) 510a 2 b 2 c 2 8(a 3 b 3 + a 3 c 3 + b 3 c 3 ) mas pela desigualdade das mdias vale:

(II) 8(a 3b3 + a 3c 3 + c 3b3 ) 24a 2b 2c 2 e donde:

64 abc ( a 3 + b 3 + c 3 ) 192 a 2 b 2 c 2

(III) 2 513a 2 b 2 c 2 + 8( a 3 b 3 + a 3 c 3 + c 3 b 3 ) + 64abc(a 3 + b 3 + c 3 ) 27abc portanto basta mostrar que: (IV) (ab c 2 + 8ab + ac b 2 + 8ac + bc a 2 + 8bc ) 9abc . fcil ver que (II), (III) e (IV) implicam (I).
ac bc Dividindo (IV) por abc, obtemos 8 ab + 1 + 8 2 + 1 + 8 2 + 1 9 , e fazendo 2 c b a

ab ac bc = u , 2 = v, 2 = w temos que provar que: 8u + 1 + 8v + 1 + 8 w + 1 9 , 2 c b a dado que uvw = 1. Usando vrias vezes a desigualdade entre as mdias aritmtica e geomtrica, obtemos 8u + 1 + 8v + 1 + 8w + 1 3 6 512uvw + 64(uv + uw + vw) + 8(u + v + w) + 1

3 6 512 uvw + 192 3 u 2 v 2 w 2 + 24 3 uvw + 1 = 3 6 729 = 9 , o que prova (IV), e de (II), (III) e (IV) obtemos (I), c.q.d..
PROBLEMA 3

Vinte e uma meninas e vinte e um meninos participaram numa competio matemtica. Cada participante resolveu no mximo seis problemas. Para cada menina e cada menino, existe pelo menos um problema que foi resolvido por ambos.

Prove que existe um problema que foi resolvido por pelo menos trs meninas e pelo menos trs meninos.

EUREKA! N12, 2001

Sociedade Brasileira de Matemtica

SOLUO DE CARLOS STEIN NAVES DE BRITO (GOINIA GO):

Considere a tabela:
O A A1 A2 A3 . . . A 21 X 21,1 X21,2 X21,3 O1 X1,1 X2,1 X3,1 O2 X1,2 X2,2 X3,2 O3 X1,3 X2,3 X3,3

...

O21 X1,21 X2,21 X3,21 . . .

...

X 21,21

Cada interseco Xi,j a questo que a menina Ai e o menino Oj fizeram em comum (se fizeram mais de uma em comum, escolha uma delas qualquer)
Lema: em cada linha i da tabela temos pelo menos 11 Xi,k's com a propriedade (I):

esse Xi,k aparece pelo menos 3 vezes nesta linha (contando ele mesmo).
Prova: Se por absurdo h menos de 11, ento h pelo menos 11 que no tm essa

propriedade (so 21 no total). Ento cada um desses se refere a uma questo que aparece no mximo 2 vezes na linha. Mas se h pelo menos 11 Xi,ks e para cada questo no mximo dois Xi,ks se referem a ela, ento h no mnimo 6 questes referidas nesse grupo. Mas uma linha s tem no mximo 6 questes referidas, pois a menina i fez no mximo 6 questes. Assim todas 6 questes feitas pela menina aparecem no mximo 2 vezes. Absurdo, pois 6 2 = 12, mas na linha aparecem 21 (21 > 12) questes. Assim o lema verdadeiro. H o lema anlogo de que em cada coluna j h pelo menos 11 Xkjs com a propriedade (II): cada um se refere a uma questo que aparece pelo menos 3 vezes na coluna. A prova anloga. Contando os Xi,js com a propriedade (I) temos pelo menos 11 21 (21 linhas). Analogamente pelo menos 11 21 com a propriedade (II). Se no houvesse nenhum Xi,j que tem ambas propriedades, teramos pelo menos 11 21 com propriedade (I), outros 11 21 com propriedade (II), somando 22 21 Xi,js, absurdo, pois h s 21 21 Xi,js. Assim h um Xi,j que tem ambas propriedades. Assim ele se refere a uma questo y que aparece 3 vezes em sua coluna e 3 vezes em sua linha. Como aparece 3 vezes em sua coluna (em 3 linhas distintas), as 3 meninas relativas a essas 3 linhas fizeram a questo y (pois tem ela em comum).
EUREKA! N12, 2001

Sociedade Brasileira de Matemtica

Analogamente 3 meninos fizeram a questo y, analisando as 3 colunas distintas. Assim 3 meninos e 3 meninas fizeram a questo y, o que sempre ocorre para certa questo y. SEGUNDO DIA
DURAO: 4 horas e meia. PROBLEMA 4

Seja n um inteiro mpar maior do que 1 e sejam k1 , k 2 ,..., k n inteiros dados. Para cada uma das n! permutaes a = (a1 , a 2 ,..., a n ) de {1,2,..., n}, defina S (a) =

k a .
i i i =1

Prove que existem duas permutaes b e c, b c, tais que n! um divisor de S (b) S (c).
SOLUO DE CARLOS STEIN NAVES DE BRITO (GOINIA GO):

Se no h S(b) S(c) tal que n! S (b) S (c) , e h n! permutaes, cada permutao de uma classe mdulo n!. Se no,existem b e c tais que S (b) S (c)(mod n) S (b) S (c) 0(mod n). Absurdo.

S (a ), o somatrio de todas n! permutaes possveis. Clculo de S ( a ) :


Seja
i i

Cada ki aparece em (n 1)! permutaes multiplicando cada coeficiente (de 1 a n). Logo no total temos (n 1)!(1 + 2 + ... + n ) de cada ki. Assim,
*

S (a ) = (n 1)!
i

(n + 1)n
2

(n + 1) n!
2

0(mod n!)

(pois n mpar).Temos tambm que cada permutao de uma classe, logo

S (a ) 1 + 2 + ... + n!
i

n!(n! + 1) (mod n!). 2

n!(n!+1) n! n! n! n! + (mod n!) . 2 2 2 2 n! De * e ** temos que 0 (mod n!) , absurdo. 2

Como n > 1, 2 n! , e

**

S (a )
i

EUREKA! N12, 2001

Sociedade Brasileira de Matemtica

Logo h S(b) e S(c) tais que S(b) S(c) (mod n!) n! S (b) S (c).
PROBLEMA 5

C com P no lado BC, e seja BQ a Num tringulo ABC, seja AP a bissectriz de BA C com Q no lado CA. bissectriz de AB C = 60 e que AB + BP = AQ + QB. Sabemos que BA Quais so os possveis valores dos ngulos do tringulo ABC?
SOLUO DE THIAGO BARROS RODRIGUES COSTA (FORTALEZA CE):
A

Q c b

P a

Nomenclatura: a = BC b = AC c = AB Q = CB Q = AB 2ac cos Lema 1: BQ = (a + c) Prova: note que a rea do tringulo ABC igual a soma das reas dos tringulos ABQ e BQC. Logo a c sen2 c BQ sen a BQ sen = + 2 2 2 a c 2 sen cos = (a + c )BQ sen 2ac cos BQ = (a + c)

EUREKA! N12, 2001

Sociedade Brasileira de Matemtica

Lema 2: AQ =

bc (a + c)

Prova: Pelo teorema das bissetrizes internas temos que:

(a + c) CQ a CQ + AQ (a + c) b bc . = = = AQ = (a + c) AQ c AQ c AQ c
Lema 3: BP =

ac (b + c )

Prova: anlogo ao lema 2.

Do problema, temos que AB + BP = AQ + QB. Ento, pelos lemas: 2ac cos ac bc a + b + c b + 2a cos +c= + = b+c a+c a+c b+c a+c (a + c )(a + b + c ) = (b + c )(b + 2a cos ) a 2 + c 2 b 2 + 2ac + ab = 2a(b + c )cos . (I) Agora, pela lei dos cossenos no ABC : a 2 + c 2 2ac cos 2 = b 2 a 2 + c 2 b 2 = 2ac cos 2 Assim, de (I): 2ac cos 2 + 2ac + ab = 2a(b + c )cos 2c(cos 2 + 1) + b = 2(b + c )cos a 2 + c 2 + 2ac + ab + bc = b 2 + bc + 2a (b + c )cos

4c cos 2(b + c )cos + b = 0. Resolvendo a equao: 1 b cos = ou cos = . 2 2c 1 Se cos = , o nico valor que ele poderia assumir era 60, nesse caso 2 C , absurdo. ABC = 2 = 120 = 180 60 = 180 = BA b Ento cos = . Pela lei dos senos no tringulo 2c
2

2c cos 2 sen 2 + cos 2 + sen 2 + b = 2(b + c )cos

EUREKA! N12, 2001

Sociedade Brasileira de Matemtica

2sen cos b sen2 sen2 = cos = = 2 sen(120 2 ) 2 sen(120 2 ) c sen(120 2 ) sen(120 2 ) = sen (note que cos 0) Veja que deve pertencer ao intervalo (0, 60) para que o tringulo possa existir. Assim, o nico valor possvel para seria fazendo = 120 2 = 40 ABC = 80 BCA = 40 Logo os ngulos do tringulo so: A = 40 e BA C = 60 . C = 80, BC AB ABC :

PROBLEMA 6

Sejam a, b, c, d inteiros com a > b > c > d > 0. Considere que ac + bd = (b + d + a c)(b + d a + c). Prove que ab + cd um nmero primo.
SOLUO DE ANTONIO CAMINHA MUNIZ NETO (FORTALEZA CE):

Simplificando a condio do enunciado obtemos a2 + c2 ac = b2 + d2 + bd, ou ainda (2a c)2 + 3c2 = (b + 2d)2 + 3b2 (*). Suponha, por contradio, que ab + cd = p, p primo. A condio a > b > c > d > 0 garante que p = ab + cd 43 + 21 = 14, de modo que p 17. Por outro lado, 2p = 2ab + 2cd = (2a c)b + (b + 2d)c, de modo que mdc(2a c, b + 2d) divide 2p. Para mdc(2a c, b + 2d) ser par, deveramos ter b e c pares, donde p = ab + cd seria par, o que um absurdo. Logo, mdc(2a c, b + 2d) = 1 ou p Afirmao: mdc(2a c, b + 2d) = 1: Suponha que mdc(2a c, b + 2d) = p. Ento (*) nos daria que p23(b2 c ), e da p2(b2 c2), uma vez que p 3. Porm, p = ab + cd > b, de modo que
2

0 < b2 c2 < p2, um absurdo.

EUREKA! N12, 2001

10

Sociedade Brasileira de Matemtica

Agora, sejam x = 2a c, y = b + 2d. Ento mdc(x, y) = 1 e segue de (*) que x2 y2 = 3(b2 c2), ou ainda (x y)(x + y) = 3(b c)(b + c) Consideraremos dois casos separadamente: i. b / c (mod 2): o fato de ser p = ab + cd garante que mdc(b, c) = 1. Como b + c e b c so mpares, isto implica em mdc(b + c, b c) = 1. Um argumento anlogo implica em mdc(x + y, x y) = 1 tambm. Se 3 (x + y) ento x y = mdc(x y, (b + c)(b c)) = mdc(x y, b + c) mdc(x y, b c) e x + y = 3mdc(x + y, (b + c)(b c)) = 3mdc(x + y, b + c) mdc(x + y, b c) Escrevendo = mdc(x y, b c), = mdc(x y, b + c), = mdc(x + y, b c) e finalmente = mdc(x + y, b + c), temos x y = e x + y = 3. Por outro lado, b c = mdc(b c, x y)mdc(b c, x + y) = e b + c = , analogamente. Resolvendo para a, b, c e d obtemos 4a = + + 3 , 2b = + , 2c = + e 4d = + 3 . Da 4p = 4(ab + cd) = (2 + 32) Mas b / c (mod 2) implica em b + c e b c mpares, de modo que , , e so mpares. Assim, temos os seguintes casos: a) = p: nesse caso, 2 + 3 2 = 4 e da = = 1. Portanto, a = b, um absurdo. b) = 1: neste caso, = = 1 e da c = d, um novo absurdo. ii. b c (mod 2): Nesse caso b e c devem ser mpares, pois do contrrio 2 dividiria ab + cd = p. Assim, nas notaes acima, temos x e y tambm mpares. Segue que mdc(b + c, b c) = mdc(x + y, x y) = 2

EUREKA! N12, 2001

11

Sociedade Brasileira de Matemtica

(pois j temos mdc(x, y) = 1). Se 3(x + y) (o outro caso novamente anlogo), escrevendo
x y x + y b c b + c = 3 2 2 2 2

e pondo = mdc = mdc


x y bc x y b+c , , , = mdc , 2 2 2 2

x+ y bc x+ y b+c , , , = mdc 2 2 2 2

chegamos, como acima, a 2a = + + 3 , b = + , c = + e 2d = + 3 . Da, p = ab + cd = ( 2 + 3 2) Nem nem so iguais a p, pois do contrrio teramos b > p, o que contradiz ab + cd = p. Logo = = 1 e da c = d, um novo absurdo.

EUREKA! N12, 2001

12

Sociedade Brasileira de Matemtica

XVI OLIMPADA IBEROAMERICANA DE MATEMTICA


Enunciados e Resultado Brasileiro A XVI Olimpada Iberoamericana de Matemtica foi realizada na cidade de Minas, Uruguai no perodo de 24 a 29 de setembro de 2001. A equipe brasileira foi liderada pelos professores Carlos Gustavo Tamm de Araujo Moreira, do Rio de Janeiro RJ e Pablo Rodrigo Ganassim, de So Paulo SP. O Resultado da Equipe Brasileira BRA 1 BRA 2 BRA 3 BRA 4 Carlos Stein Naves de Brito Daniel Massaki Yamamoto Daniel Pinheiro Sobreira Thiago Barros Rodrigues Costa Ouro Prata Ouro Prata

PRIMEIRO DIA DURAO: 4 horas e meia. PROBLEMA 1

Dizemos que um nmero natural n "charrua" se satisfaz simultaneamente as seguintes condies: - Todos os algarismos de n so maiores que 1 - Sempre que se multiplicam quatro algarismos de n, obtm-se um divisor de n. Demonstrar que para cada nmero natural k existe um nmero "charrua" com mais de k algarismos.
SOLUO DE DANIEL MASSAKI YAMAMOTO (SO PAULO SP):

Vou construir nmeros charruas (o que, afinal, significa isso?), utilizando os algarismos 2 e 3. Multiplicando-se 4 algarismos, obtemos um nmero da forma d = 3 . 2 , onde 0 ^ ^  Vou fixar o final do nmero em 3232 e completar o resto apenas com 2s. Ento 0 ^ ^  2 (I) 3 n Para que d/n, basta garantirmos que: 4 (II) 2 n

EUREKA! N12, 2001

13

Sociedade Brasileira de Matemtica

fcil ver que 2 4 3232 2 4 M 10 4 + 3232, M  2 4 n , logo (II) est satisfeito. Para garantirmos a divisibilidade por 9 , basta que a soma dos algarismos seja divisvel por 9. Pegue n0 = 22223232 . Note que 9/ Podemos colocar blocos de nove 2s esquerda de n0 que no afetar a divisibilidade por 9. k k Adicionando blocos, o nmero ter mais de 9 + 8 > k dgitos. 9 9
PROBLEMA 2

A circunferncia inscrita no tringulo ABC tem o centro O e tangente aos lados BC, AC e AB nos pontos X, Y e Z, respectivamente. As rectas BO e CO intersectam a recta YZ nos pontos P e Q, respectivamente. Demonstrar que se os segmentos XP e XQ tm o mesmo comprimento, ento o tringulo ABC issceles.
SOLUO DE THIAGO BARROS RODRIGUES COSTA (FORTALEZA CE):
A

Q
O .

Pelo teorema do bico, o tringulo BXZ issceles. Como OB bissetriz do ngulo B , OB mediatriz do segmento XZ . Considere o tringulo ZXP. O ponto P pertence a mediatriz de XZ . Assim, a altura relativa ao vrtice P e a mediana relativa ao mesmo so iguais XZP issceles XP = PZ . (*) De maneira anloga, XQ = QY .(**) . Suponha agora que XP = XQ.
EUREKA! N12, 2001

14

Sociedade Brasileira de Matemtica

Ento, por (*) e (**), QY = PZ (* * *) . Y = AY Z (O AZY issceles, pelo teorema do bico). Seja = AZ O = 90 . Como O incentro OZ AZ YZ Z = 90 Da mesma forma, OY AY OY Z = YZ O . (1) OY Note que OZ e OY so raios OZ = OY . (2) Por (***), (1) e (2) temos que os tringulos OZP e OYQ so congruentes Y = OP Z . (3) OQ Y , temos que (no ZPB ) Z e = OC Se = OB Y mas, por (3), OQ Y = OP O = = + OQ Z + ZP B ABC isosceles. C = 2 = 2 = AC = , mas AB
PROBLEMA 3

Sejam S um conjunto de n elementos e S1, S2,,Sk subconjuntos de S (k 2), cada um deles com pelo menos r elementos. Demonstrar que existem i e j, com 1 i < j k , tais que o nmero de elementos nk comuns a Si e Sj maior ou igual a r . 4(k 1)
SOLUO DE CARLOS STEIN NAVES DE BRITO (GOINIA GO):

Chamaremos os n elementos de S de b1, b2, ...,bn .Se o nmero de subconjuntos Si ai que contm {bi} ai, o nmero de pares de subconjuntos que 2 compartilham bi. Temos que

a
i =1

kr , pois a soma de quantos elementos h em cada um dos

subconjuntos Si. x + k x x + k 1 x + 1 Temos que, para k 1, 2 , pois: + + 2 2 2 x 2 + 2xk + k 2 k x + x 2 x x 2 + k 2 +1+ 2kx 2x 2k x k +1 + x 2 + x k 1.

)(

) (

)(

EUREKA! N12, 2001

15

Sociedade Brasileira de Matemtica

Logo em

2 voc pode ir reduzindo a diferena entre os a s que a soma s


i i =1

ai

diminui at que voc tenha nmero com diferena mxima, dois a dois, de um. Seja kr = yn + q, 0 q n 1. Assim teremos no fim q ais sendo (y + 1) e (n q) ais sendo y ( para minimizar Temos ento:

2 ).
i =1

ai

2 q

ai

y + 1 y y + (n q ) = qy + n 2 . 2 2

Vamos mostrar que q kr (yn + q ) kr 1 y + 1 2 y n n 2qy + ny ny qy + n 2 2 2 2 2qy + ny 2 ny ny 2 + Logo temos: kr kr 1 n . 2 q2 q2 , o que segue de n q. + 2 yq ny q q n n

ai i =1 2
n

Note agora que

2 conta com multiplicidade o nmero de subconjuntos S

ai

k k (k 1) com algum elemento em comum. Dividindo por , que o nmero = 2 2 de pares de subconjuntos Si , obtemos o tamanho mdio das intersees de dois kr kr 1 n 2 1 1 . desses subconjuntos, que portanto maior ou igual a 2 k k 1 Basta mostrar que esse mnimo para esse certo subconjunto maior ou igual a nk e acaba o problema.Para isso,note que r 4(k 1) kr r 1 n r nk 4(k 1) k 1

EUREKA! N12, 2001

16

Sociedade Brasileira de Matemtica

kr r 1 n 4r (k 1) nk 4(k 1) k 1 4r 2 k 4n 4nrk 4rn n 2 k 4r 2 k 4nrk n 2 k 4r 2 4nr n 2 n 2 4nr + 4r 2 0 (n 2r ) 0 (sempre verdade).


2

SEGUNDO DIA DURAO: 4 horas e meia. PROBLEMA 4

Determinar o nmero mximo de progresses aritmticas crescente de trs termos que pode ter uma sucesso a1 < a 2 < ... < a n de n 3 nmeros reais. Nota: trs termos a i , a j , a k de uma sucesso de nmeros reais formam uma progresso aritmtica crescente se a i < a j < a k e a j a i = a k a j .
SOLUO DE DANIEL PINHEIRO SOBREIRA (FORTALEZA CE):

Primeiro vejamos que cada P.A. tem um termo central, pois so formadas de trs termos. Vamos estimar o nmero mximo de posibilidades para o termo central. Lema: Se em uma seqncia temos um termo ak, e temos x nmeros menores que ak e y nmeros maiores que ak , vamos ver que ak termo central de no mximo min{x, y} P.A.s. Para cada P.A. onde ak o termo central, um dos outros termos maior e o outro menor, logo devo ter a mesma quantidade de termos maiores e menores que ak que participam das P.A.s de que ak termo central. Ento se ak for termo central de j > min{x, y} P.A.s teramos que ter no mnimo j nmeros menores que ak e no mnimo j nmeros maiores que ak, ou seja, x _ j e y _ j, logo min{x, y}_ j. Ento o nmero de P.A.s com ak como termo central no mximo min{x, y}. Agora vamos dividir em 2 casos: Caso 1: n mpar. Temos: a1 < a 2 < a 3 < ... < a n +1 < a n + 3 < ... < a n 1 < a n
2 2

lgico que a1 e a n no podem ser termos centrais.


EUREKA! N12, 2001

17

Sociedade Brasileira de Matemtica

Se pegarmos o nmero a j , temos ( j 1) nmeros menores que ele (a1 , a 2 ,..., a j 1 ) e (n j ) nmeros maiores que ele (a j +1 , a j + 2 ,...a n ). Logo a j termo central de no mximo min{ j 1, n j} P.A.s. Ento temos que o nmero de P.A.s menor ou igual a

min{ j 1, n j}.
j =2

n 1

n +1 temos j 1 < n j 2 j < n + 1. Mas para 2 j < 2 n +1 n +1 < j n 1 temos j 1 > n j. temos j 1 = n j e para Para j = 2 2 Portanto, o nmero de P.A.s menor ou igual a

n 1 ( j 1) + + 2 j =2
n 3 2 i =1

n 1 2

(n j ) =
j= n +3 2

n 1

= i +

n 1 n 3 n 1 + i = 21 + 2 + ... + = + 2 2 2 i =1

n 3 2

n 3 n 3 1 + 2 2 n 1 2 + n 3 n 3 n 1 = 2 = + + 2 = 2 2 2 2

n 1 n 3 n 1 n 1 n 3 n 1 n 1 n 1 + 1 = = + = = . 2 2 2 2 2 2 2 2
2

fcil ver que a seqncia (1, 2, 3,..., n) tem

n 1 P.A.s, pois acontece 2

justamente o que queremos: 2 termo central de uma P.A. o 3 de duas P.A.s e assim exatamente como ocorre a igualdade. Caso 2: n par. Temos: a1 < a 2 < a3 < ... < a n < a n
2 2

+1

< ... < a n 1 < a n

Da mesma forma a1 e a n no podem ser termos centrais.

EUREKA! N12, 2001

18

Sociedade Brasileira de Matemtica

Se pegarmos o nmero a j , temos ( j 1) nmeros menores que ele (a1 , a 2 ,..., a j 1 ) e (n j ) nmeros maiores que ele (a j +1 , a j + 2 ,...a n ). Logo a j termo central de no mximo min{ j 1, n j} P.A.s. Mas para n n + 1 j n 1, temos , temos j 1 < n j e para 2 2 j 1 > n j , logo o nmero de P.A.s menor ou igual a 2 j
n 2

min{ j 1, n j} = ( j 1) +
j =2 j =2 n 1 2 n 1 2

n 1

n j = +1 2

(n j ) =

n 1

n = i + (i ) = 2 = 1 + 2 + ... + 2 1 i =1 i =1 n n 1 + 1 1 2 2 = n n 1 = n(n 2) . = 2 2 4 2 2
Mas fcil ver que a seqncia (1, 2, ..., n) tem satisfaz todas as igualdades. Logo o nmero mximo de P.A.s : n 1 Para n mpar: 2 n(n 2) . Para n par: 4
PROBLEMA 5
2

n(n 2 ) P.A.s, porque ela 4

Num tabuleiro de 2000 2001 as casas tm coordenadas (x, y) com x, y inteiros, 0 x 1999 e 0 y 2000. Uma nave no tabuleiro move-se da seguinte maneira: antes de cada movimento, a nave est numa posio (x, y) e tem uma velocidade (h, v) onde h e v so inteiros. A nave escolhe uma nova velocidade (h', v') de forma que h' h seja igual a 1, 0 ou 1 e v' v seja igual a 1, 0 ou 1. A nova posio da nave ser (x', y') onde x' o resto da diviso de x + h' por 2000 e y' o resto da diviso de y + v' por 2001.
EUREKA! N12, 2001

19

Sociedade Brasileira de Matemtica

H duas naves no tabuleiro: a marciana e a terrestre que quer capturar a marciana. Inicialmente cada nave est numa casa do tabuleiro e tem velocidade (0, 0). Move-se primeiro a nave terrestre e continuam movendo-se alternadamente. Existe uma estratgia que permita sempre nave terrestre capturar a nave marciana, quaisquer que sejam as posies inicias? Nota: a nave terrestre, que sempre v a marciana, captura a marciana se depois de um movimento seu cai na mesma posio da marciana.
SOLUO DE CARLOS STEIN NAVES DE BRITO (GOINIA GO):

A estratgia existe! A nave terrestre deve fazer uma estratgia para ficar na mesma coordenada x, independente do movimento da nave marciana e tambm na mesma coordenada y, at que isso ocorra simultaneamente e a terrestre pegue a nave marciana. i i Para ficar no mesmo x : x m , ym a coordenada depois de i rodadas da nave marciana e xti , y ti a coordenada da terrestre.
0 0 Seja 0 p 1999, p x m xt (mod 2000).

Se o primeiro h escolhido for 1, xt0 aumenta 1 (mod 2000) .


' A partir da se a nave marciana fizer sua h tal que hm hm = a, ' terrestre far o mesmo: ht ht = a.

a 1, a nave

A partir da temos ht hm = 1 ( ht e hm so as velocidades momentneas da nave terrestre e marciana, respectivamente), isso porque a terrestre j tem ht = 1 por causa da primeira jogada. Esse ht hm = 1 , se manter sempre depois da jogada
' nm = a e terrestre, pois se nm ' ' ht' ht = a; ht hm = 1 (ht' a) (hm a) = 1 ht' hm = 1, logo a induo bvia. Assim a cada rodada a nave marciana anda hm e a terrestre hm + 1. Logo

a cada rodada da nave terrestre temos (tudo mod 2000):

0 0 1 ' 0 x x = p; x m + hm xt + hm + 1 x m x10 1 p 1; x1 x1 m t
0 m 0 t

) (

2 ' 2 2 ' xm xt2 x m + nm xt' + n m + 1 xm xt' 1 ( p 1) 1 p 2,

) (
k m

Assim por induo finita:

x x x
k m k t

k 1 m

+h

( ) (x

))

k 1 se x m k 1 k t m

xtk 1 p (k 1)

+ h +1 x

) (
20

k 1 m

k 1 t

) ) 1 p k.

EUREKA! N12, 2001

Sociedade Brasileira de Matemtica

Logo depois k jogadas terrestres a diferena entre as coordenadas x m e xt p k (mod 2000), assim para k x = 2000a + p, para qualquer inteiro a 0, p k 0(mod 2000), assim estaro na mesma coordenada x. O estudo para as coordenadas y m e y t anlogo, como a nave terrquea tendo
v1 = 1 , e depois alternando sua velocidade igual marciana, mas agora tudo 0 0 mdulo 2001, logo se a diferena inicial x m y t = q temos que depois de k

jogadas a diferena ser. q k ' (mod 2001), logo, para k y = 2001b + q, estaro no mesmo y. Ento a terrestre capturar a marciana quando est no mesmo x e no mesmo y, o que acontece quando o nmero de rodadas k satisfaz k x e k y :

k = k x = k y 2000a + p = 2001b + q, a, b  2001b 2000a = p q.


Achar soluo para isso fcil: s adotar a = b = p q, que obviamente soluo da equao e se p q < 0, s pegar b = p q + 2000 0 e a = p q + 2001 0, que tambm so solues e agora positivas. Logo na jogada terrestre de nmero k = 2001 p 2000q + 2000 2001, ela ter o mesmo x e y da marciana, capturando-a.
PROBLEMA 6

Demonstrar que impossvel cobrir um quadrado de lado 1 com cinco quadrados 1 iguais de lado menor que . 2
SOLUO OFICIAL:
D C

v A u B

EUREKA! N12, 2001

21

Sociedade Brasileira de Matemtica

Seja ABCD um quadrado unitrio e suponhamos que possvel cobri-lo 1 utilizando cinco quadrados iguais de lado a < . Ento, como o dimetro de cada 2 2 um dos quadrados pequenos a 2 < , ou seja, menor que a metade da 2 diagonal de ABCD, os vrtices A, B, C, D e o centro O de ABCD devem pertencer a quadrados distintos. Chamaremos de QA, QB, QC, QD e QO estes quadrados. Para obter uma contradio, demonstraremos primeiro que a parte do permetro que cobre cada um dos quadrados QA, QB, QC, QD tem comprimento menor ou igual a 2a. Consideremos, por exemplo, QA; como contm a A, o quadrado intersecta ABCD tal como mostra a figura. Sejam u, v as pores dos lados AB, AD, contidas em QA. Ento u + v 2(u 2 + v 2 ) e
u 2 + v 2 a hipotenusa de um tringulo retngulo contido

em QA. Logo, u 2 + v 2 a 2 e, em conseqncia, u + v 2 .a 2 = 2a. Segue que QA, QB, QC, QD em conjunto cobrem uma poro do permetro de ABCD de comprimento total menor ou igual a 8a < 4. Ento, o quinto quadrado, QO, deve ter interseo no vazia com o permetro de ABCD. Digamos, por exemplo, que intersecte AB.
D C

Vamos ver que os cinco quadrados em conjunto no podem cobrir simultaneamente os segmentos AB, CD, e EF, onde E e F so os pontos mdios de AD e BC. A demonstrao se apoia no lema a seguir: 1 1 Dadas duas retas paralelas l e m a distncia e um quadrado de lado a < 2 2 que tem interseo no vazia com cada uma das retas l e m, a soma dos
EUREKA! N12, 2001

22

Sociedade Brasileira de Matemtica

comprimentos dos segmentos da interseo do quadrado com as retas menor que a. Antes de demonstrar o lema, vejamos que suficiente para completar a soluo. De fato, do lema se conclui que ao menos k 2 dos quadrados QA, QB, QC, QD devem ter interseo com EF (um destes quadrados mais QO cobriro em conjunto uma poro menor que 2a < 1 do comprimento do segmento EF). Ento, novamente pelo lema, esses k quadrados e QO cobrem uma poro do comprimento menor que (k + 1)a dos segmentos AB, CD, e EF. Os restantes (4 k) quadrados intersectam exatamente um dos segmentos AB e CD, e cada um deles pode cobrir uma poro de comprimento no mximo a 2 do segmento correspondente. Assim, a poro de AB, CD e EF coberta pelos cinco quadrados menor que: (k + 1)a + (4 k )a 2 = ka(1 2 ) + a (1 + 4 2 ). 1 Como 1 2 < 0, k 2 e a < , observamos que 2 3+ 2 2 <3 ka 1 2 + a 1 + 4 2 2a 1 2 + a 1 + 4 2 = a 3 + 2 2 < 2 o que contradiz que AB, CD e EF esto cobertos.

) (

) (

) (
l

M h1 N

L m

h2

Falta demonstrar o lema. Se o quadrado KLMN de lado a < paralelas l e m que esto a distncia

1 intersecta as retas 2

1 , ento dois vrtices de KLMN devem estar 2 em lados distintos da banda determinada por l e m (tambm podem estar em l e 1 implica que esses dois vrtices de KLMN so opostos, m). A condio a < 2 digamos K e M. Ento, pela mesma condio, os restantes dois vrtices do quadrado devem estar contidos na faixa comprendida entre l e m. Sejam h1 e h2 as alturas
EUREKA! N12, 2001

23

Sociedade Brasileira de Matemtica

correspondentes s hipotenusas dos tringulos retngulos semelhantes que so as duas pores do quadrado que esto fora da faixa comprendida entre l e m, e denotamos ao menor dos ngulos agudos dos tringulos. Ento, o ngulo comprendido entre KM e qualquer reta perpendicular a l e m 45 e a projeo de KLMN sobre uma reta mede a 2 cos(45 ). Logo, 1 1 h1 + h2 = a 2 cos(45 ) = a (sen + cos ) . 2 2 A soma das hipotenusas consideradas :

h1 + h2 S = (h1tg + h1 cot ) + (h2 tg + h2 cot ) = = sen cos

a (sen + cos ) sen cos

1 2.

Como sen + cos < 1 + sen cos equivale desigualdade evidente (1 sen )(1 cos ) > 0 e a < 1 , obtemos 2 1 1 1 a(sen + cos ) a(1 + sen cos ) a 2< 2 =a+ 2 < a. S= sen cos sen cos sen cos A demonstrao est completa.

EUREKA! N12, 2001

24

Sociedade Brasileira de Matemtica

COMO FERMAT E BZOUT PODEM SALVAR O DIA


Antonio Caminha Muniz Neto, Fortaleza - CE Nvel Avanado. Certamente voc, leitor, tem alguma familiaridade com os fatos mais bsicos da teoria elementar dos nmeros. Portanto, no objetivo desenvolv-los aqui de modo sistemtico. Para isso voc pode consultar [1], [2] ou [3] nas referncias. O que vou fazer mostrar como dois resultados particulares, os teoremas de Bzout e Fermat, podem ser usados para abordar com sucesso alguns problemas interessantes. Para facilitar a leitura, vamos relembrar alguns conceitos e provar os resultados mais centrais para ns.
Definio 1: Dados dois inteiros no nulos a e b, definimos o mximo divisor

comum (mdc) como o maior inteiro d que divide ambos a e b. A definio acima certamente faz sentido, uma vez que a e b tm divisores comuns (1, por exemplo) e qualquer inteiro que divida a e b deve ser, em particular, menor ou igual a a. Assim, realmente existe um maior inteiro que divide a e b. Caso esse maior inteiro seja igual a 1, dizemos que a e b so primos entre si, ou ainda relativamente primos. Para nossos propsitos, o seguinte resultado sobre o mdc de dois inteiros ser suficiente:
Teorema 1 (Bzout): Sejam a e b inteiros no nulos dados e d seu mdc. Ento existem inteiros x e y tais que d = ax0 + by0. Mais ainda, se a e b so positivos, podemos escolher x0 > 0 e y0 < 0, ou vice-versa. Prova: Seja S o conjunto dos inteiros da forma ax + by, com x e y inteiros.

Escrevendo a= a(1) + b0, conclumos quea S, e portanto S contm inteiros positivos. Podemos ento escolher o menor inteiro positivo pertencente a S, o qual vamos denotar por d. Afirmamos que tal menor elemento positivo d o mdc de a e b.

Desde que d est em S, devem existir inteiros x0 e y0 tais que d = ax0 + by0. Para provar que d divide a, dividamos a por d: a = dq + r, com 0 r < d. r = a dq = a (ax0 + by0)q = a(1 x0q) + b( y0q), que por definio est em S. O fato de ser d o menor inteiro positivo em S, juntamente com 0 r < d, implica que r = 0, e assim d divide a. Analogamente mostramos que d divide b. Por outro lado, se d for qualquer outro divisor comum de a e b, segue que d divide ax + by, quaisquer que sejam os inteiros x e y. Em
EUREKA! N12, 2001

25

Sociedade Brasileira de Matemtica

particular, d divide d = ax0 + by0, de modo que d d. Isso prova ser d o mdc de a e b. Para o que falta, analisemos somente o caso em que a, b > 0 (a anlise dos demais casos totalmente anloga). Como d = ax0 + by0 = a(x0 + tb) + b(y0 ta), escolhendo t >
x0 y 0 , obtemos x0 + tb > 0 > y0 ta. b a

Corolrio 1.1: Dois inteiros no nulos a e b so primos entre si se e s se existirem inteiros x e y tais que ax + by = 1.

O outro resultado que usaremos nos problemas a seguir o pequeno teorema de Fermat. Recordemos seu enunciado e prova:
Teorema 2 (Fermat): Dados inteiros a > 1 e p primo, tem-se a a (mod p).
p

Prova: Se a for mltiplo de p nada h a fazer. Seno, como p primo temos que a

e p so primos entre si. Considere agora os nmeros a, 2a, 3a, ...., (p 1)a. Dados 1 i < j p 1, como ja ia = (j i)a um produto de dois nmeros no divisveis por p, temos que ja ia no divisvel por p. Em linguagem de congruncias isso o mesmo que ja / ia (mod p ) . Tambm, nenhum dos nmeros ja mltiplo de p, e portanto os restos dos nmeros a, 2a, 3a, ...., (p 1)a na diviso por p formam uma permutao de 1, 2, 3, ..., p 1. Voltando s congruncias, isso implica que a2a3a .... (p 1)a 123 .... (p 1) (mod p), ou ainda (p 1)!ap 1 (p 1)! (mod p) Mas (p 1)! relativamente primo com p, e portanto pode ser cancelado em ambos os membros da ltima congruncia acima, dando o resultado desejado. Aps essa breve reviso(?), vamos aos problemas!
PROBLEMAS RESOLVIDOS Problema 1: Sejam a e b inteiros positivos primos entre si. Ento todo inteiro c maior ou igual que o nmero (a 1)(b 1) pode ser escrito da forma c = ar + bs, com r, s 0. Mais ainda, o menor inteiro com essa propriedade (a 1)(b 1). Soluo: Dado c inteiro, o fato de serem a e b primos entre si garante que existem

inteiros x e y tais que c = ax + by (voc entendeu por qu?). Seja agora y = da +


EUREKA! N12, 2001

26

Sociedade Brasileira de Matemtica

s, onde 0 s < a. Temos c = ax + b(da + s) = a(x + bd) + bs. Seja r = x + bd. Se c (a 1)(b 1) ento (a 1)(b 1) c = ar + bs ar + b(a 1), de modo que ar (a 1) , e portanto r 0. Resta mostrarmos que (a 1)(b 1) 1 = ab a b no pode ser escrito da forma ar + bs, com r, s 0. Supondo o contrrio, seja ab a b = ar + bs, onde r, s 0. Ento temos a (b 1 r) = b(s + 1). Como a e b so primos entre si, segue que a divide s + 1 e b divide b 1 r. Como b 1 r < b, deve ser b 1 r 0, ou ainda r b 1. Tambm, como s + 1 > 0 e a divide s + 1, deve ser s + 1 a, ou s a 1. Mas a, ar + bs a(b 1) + b(a 1) = 2ab a b > ab a b = (a 1)(b 1) 1, uma contradio.
Problema 2: (Seleo da Romnia para IMO) Prove que no existe um inteiro n >

1 tal que n divida 3n 2n. 0 (mod n). Obviamente 2 e 3 no dividem n. Seja agora p o menor fator primo de n e n = pm (aqui que usamos ser n > 1, para garantir que n tem fator primo). Nossa hiptese, juntamente com o pequeno teorema de Fermat, nos do: 3n 2n (mod n) 3mp 2mp (mod p) 3m 2m (mod p) (*) Se d = mdc(m, p 1), temos em particular que d divide n. Portanto, o fato de ser p o menor divisor primo de n implica que d = 1. Tome ento inteiros positivos x e y satisfazendo mx = (p 1)y + 1. O pequeno teorema de Fermat de novo, juntamente com (*), nos do 3 3(p 1)y + 1 = 3mx 2mx = 2(p 1)y + 1 2 (mod p), o que um absurdo.
Problema 3: Sejam m e n inteiros positivos. Determine o polinmio mnico p, de maior grau possvel, que divide simultaneamente os polinmios xm 1 e xn 1 . Soluo: Primeiro, no difcil vermos que, sendo d o mdc de m e n, ento x 1
d

Prova: Suponha o contrrio, isto , que para algum inteiro n > 1 tenhamos 3 2

divide ambos x 1 e x 1. De fato, seja por exemplo m = dk, com k > 0 inteiro. Ento xm 1 = (xd)k 1 = (xd 1)(xd(k 1) + xd(k 2) + .... + xd + 1) Mostrar que xd 1 divide xn 1 anlogo. A parte mais difcil mostrar que xd 1 o polinmio mnico p de maior grau que divide ambos xm 1 e xn 1, e para isso que precisamos de Bzout. Seja p um polinmio mnico que divide ambos xm 1 e xn 1 e z uma raiz complexa de p. Como p divide xm 1 e xn 1, temos que z raiz de ambos esses polinmios. Em outras palavras, zm = zn = 1. Mas o
EUREKA! N12, 2001

27

Sociedade Brasileira de Matemtica

teorema de Bzout garante que existem inteiros u e v tais que mu + nv = 1. Isso nos d zd = zmu + nv = (zm)u(zn)v = 1u1v = 1, e assim z raiz de xd 1. Como toda raiz de p tambm raiz de xd 1 e como xd 1 s tem razes simples, segue que p divide xd 1. Portanto, xd 1 o polinmio mnico de maior grau que divide ambos xm 1 e xn 1.
Problema 4: (The William Lowell Putnam Competition) Sejam m e n inteiros mdc (m, n) m positivos, com m n. Prove que n inteiro. m Prova: Para esse problema usamos o teorema de Bzout de um modo bastante x m elegante. Seja S o conjunto dos inteiros x tais que seja inteiro. Veja que m m n

est em S j que nmeros binomiais so inteiros. Tambm n est em S, pois


m 1 n m n m! = = n m m (m n)! n! n 1

Por outro lado note que, se x e y estiverem em S, ento ux + vy tambm estar em S, quaisquer que sejam u e v inteiros. De fato,
ux + vy m x m y m =u +v , m n mn m n

que um inteiro. Como o mdc de m e n pode ser escrito da forma mu + nv, para algum par de inteiros u e v, segue que mdc(m, n) est em S.
Problema 5: (Seleo do Brasil para a IMO) Determine todas as funes

f : "+
*

satisfazendo as seguintes condies:

i. f (1999) = 1. ii. f (ab) = f (a) + f (b), para todos os racionais positivos a, b. iii. f (a + b) min {f (a), f (b)}, para todos os racionais positivos a, b.
Soluo: Fazendo a = b = 1 em ii. Vem que f (1) = 2f (1), donde f (1) = 0. Da,

dados inteiros positivos m e n, temos


m 1 f = f (m) + f n n

1 f + f (n) = f (1) = 0, n

donde
EUREKA! N12, 2001

28

Sociedade Brasileira de Matemtica

m f = f (m) f (n) n

Assim, basta calcularmos os valores de f (n), com n inteiro positivo. Seja n = a a p1 1 .... p k k a decomposio de n em fatores primos. Usando ii. vrias vezes, vem que f (n) = a1 f (p1) + .... + ak f (pk), de modo que basta calcularmos f (p), com p primo. Afirmamos que f (n) 0 para todo inteiro positivo n. Para provar esse fato, faamos induo sobre n. J vimos que f (1) = 0. Suponhamos agora que f (n) 0 para algum inteiro positivo n. Ento f (n + 1) min {f (n), f (1)} 0, e isso termina nossa induo. Afirmamos agora que f (2) = 0 ou f (3) = 0. De fato, como 1999 = 39 + 4934, temos que 1 = f (1999) min {f (3) + f (9), f (493) + f (4)} min {3f (3), 2f (2)}, e da segue o afirmado. Suponhamos que f (2) = 0 ( f (3) = 0 anlogo), e seja p um primo diferente de 2 e de 1999. Tomando k inteiro suficientemente grande, o problema 1 garante a existncia de inteiros positivos x e y tais que px + 1999y = 2k. Ento 0 = f (2k) min { f (p) + f (x), f (1999) + f (y)} Como f (1999) + f (y) 1, segue que f (p) = 0. Ento, sendo n = 1999am, com 1999 e m primos entre si, segue que f (n) = a. Mas imediato verificar que tal funo satisfaz as condies impostas no enunciado.

PROBLEMAS PROPOSTOS Problema 6: Generalizando a teoria desenvolvida acima e o problema 1, sejam a1,

a2, ...., an (n 2) inteiros positivos tais que mdc(a1, a2, ...., an) = d (a definio de mdc que demos no incio se aplica nesse caso ipsis literis). Prove que:

i. Existem inteiros x1, x2, ...., xn tais que a1x1 + a2x2 + .... + anxn = d. ii. Se d = 1, mostre que existe um inteiro positivo m0 tal que todo inteiro m m0 pode ser escrito da forma a1y1 + a2y2 + .... + anyn, com y1, y2, ...., yn 0. (sugesto: para o item i. imite a prova do texto acima. Para ii., que tal usar induo sobre n > 1? Para uma cota mais precisa para m0, veja [2]).
Problema 7: Considere duas progresses aritmticas infinitas e no constantes de

inteiros positivos. Prove que existem infinitos naturais termos de ambas as


EUREKA! N12, 2001

29

Sociedade Brasileira de Matemtica

seqncias se e s se o mdc de suas razes dividir a diferena entre seus termos iniciais. (sugesto: use Bzout. fcil!).
Problema 8: (The William Lowell Putnam Competition) Prove que no existe inteiro n > 1 tal que n divida 2n 1.

(sugesto: use as idias que apareceram na prova do problema 2).


Problema 9: (Olimpada Blgara) Determine todos os primos p, q tais que pq

divida o nmero (5p 2p)(5q 2q) (sugesto: se q dividir 5p 2p e p q, ento mdc(p, q 1) = 1. Como antes, use Bzout e Fermat).
Problema 10: (Seleo da Romnia para IMO) Sejam p, q nmeros primos. Se q

dividir 2p + 3p, prove que q > p ou q = 5. (sugesto: se p > 3 e q p, ento q 1 < p, donde q 1 e p so primos entre si. Mais uma vez use Bzout e Fermat).

REFERNCIAS [1] Introduo Teoria dos Nmeros. Antnio Plnio dos Santos. IMPA. Rio de Janeiro, 1998. [2] Olimpadas de Matemtica, um Curso de Introduo, vol. 2. Antonio Caminha. Editora Ipiranga. Fortaleza, 2001 (a ser publicado). [3] Divisibilidade,Congrncias e Aritmtica mdulo n. Carlos Gustavo Moreira, Eureka! no.2 (1998),pp. 41-52.
EUREKA! N12, 2001

30

Sociedade Brasileira de Matemtica

GRAFOS E CONTAGEM DUPLA


Carlos Yuzo Shine, Colgio Etapa Nvel Intermedirio.
1. GRAFOS 1.1 O que so e para que servem grafos?

Define-se grafo como o par (V, A) onde V = {v1, v2,...,vn} um conjunto de vrtices e A {{vi, vj}t.q. vi, vj V, i j} um conjunto de arestas (na verdade, uma aresta um par no-orientado de vrtices). A representao mais comum de grafos associar os vrtices a pontos e as arestas a linhas que ligam os pares de vrtices que as formam. Mas o mais importante que os grafos podem representar inmeras situaes. Por exemplo, quando voc brinca de ligar os pontos, no fundo voc est traando arestas em um grafo onde os vrtices so dados (em vez de ligue os pontos, poderamos escrever areste o grafo...). Embora paream simples, os grafos tm muito mais utilidades, como veremos. Na verdade, a Teoria dos Grafos uma das partes mais importantes da Matemtica, e muito utilizada principalmente em computao.
Exemplo 1.1

Podemos construir um grafo que represente pessoas apertando mos. Os vrtices seriam as pessoas. Ligamos dois vrtices (formando assim uma aresta) se duas pessoas se cumprimentaram.
Edmilson Gustavo

Carlos

Eduardo

Onofre Emanuel

Paulo

EUREKA! N12, 2001

31

Sociedade Brasileira de Matemtica

Exemplo 1.2

possvel que os cavalos do tabuleiro (I) fiquem na posio do tabuleiro (II) ?

(II) (I) Observao: um cavalo, no xadrez, se movimenta da seguinte forma: ele se move duas casas na vertical ou horizontal e depois se move uma casa na direo perpendicular direo em que havia se movimentado antes.

Resoluo

Vamos numerar as casas do tabuleiro da seguinte forma:


1 2 3

Vamos construir um grafo onde os vrtices so as casas do tabuleiro. Ligaremos dois vrtices i e j se possvel um cavalo ir da casa i casa j. Temos ento o seguinte grafo (verifique!).
EUREKA! N12, 2001

32

Sociedade Brasileira de Matemtica


1 6 8

2 9

Coloquemos agora os cavalos nas situaes inicial e final, respectivamente:


1 6 8 6 1 8

2 9

2 9

Inicial

Final

Observe que no podemos ter dois cavalos na mesma casa, assim os cavalos devem sempre estar na mesma ordem no ciclo. Logo no possvel chegar na posio final.
Exerccios

01. (IMO) Considere um inteiro positivo r e um retngulo de dimenses AB = 20, BC = 12. O retngulo dividido em uma grade de 20 12 quadrados unitarios. Uma moeda pode ser movida de um quadrado a outro se, e somente se, a distncia entre os centros dos quadrados r . A tarefa encontrar uma seqncia de movimentos que levem uma moeda do quadrado que tem A como vrtice ao quadrado que tem B como vrtice. a) Mostre que a tarefa no pode ser feita se r divisvel por 2 ou 3. b) Prove que a tarefa pode ser feita se r = 73. c) Pode a tarefa ser feita quando r = 97? Dicas: Para o item a), use o fato de que um quadrado perfeiro pode deixar somente os restos 0 ou 1 quando divididos por 3 ou 4. Para os itens b) e c),
EUREKA! N12, 2001

33

Sociedade Brasileira de Matemtica

construa dois grafos: um que considera a posio da moeda na horizontal e outro na vertical.
1.2. Grau de vrtice

Definimos grau de vrtice vi como o nmero de arestas que contm vi e denotamos d(vi). No ltimo exemplo, o grau de um vrtice seria o nmero de apertos de mo que a pessoa correspondente deu.
Exemplo 1.3.

Na cidade de Micrpolis, h 7 telefones. Um candidato a prefeito prometeu que ampliaria a rede de telefonia de modo que cada um dos 7 telefones esteja conectado diretamente a exatamente 5 outros telefones. possvel que ele cumpra sua promessa?
Resoluo

Se imaginarmos um grafo onde os vrtices so os telefones e as arestas, as conexes, teramos que o grau de cada vrtice seria 5. Vamos contar o nmero de conexes entre dois telefones (ou seja, o nmero de arestas do grafo). Como de cada telefone sairiam 5 conexes, teramos a princpio 5.7 = 35 conexes; mas contamos cada conexo duas vezes, uma vez em cada um dos dois telefones a que ele est conectado. Assim, deveramos ter na verdade 35/2 conexes, o que seria um absurdo. Assim, o candidato a prefeito no pode cumprir sua promessa (no votem nele!!). Este exemplo mostra
1.3. Um teorema importante Teorema. Em um grafo, a soma dos graus de todos os vrtices igual ao dobro do

nmero de arestas. Em smbolos: no grafo (V, A), d (v i ) = 2 A


vi V

( X denota o nmero de elementos do conjunto X.)


Demonstrao

De cada vrtice v saem d(v) arestas. Assim, se somarmos os graus de todos os vrtices, obtemos o nmero de arestas multiplicado por dois, pois contamos cada aresta duas vezes (lembre-se de que cada aresta est associada a dois vrtices).

EUREKA! N12, 2001

34

Sociedade Brasileira de Matemtica

2. Contagem Dupla

O que acabamos de fazer foi contar algo de duas maneiras diferentes, no caso o nmero de arestas (na verdade, o seu dobro). Esta idia de contar duas vezes s vezes muito til para demonstrar algumas relaes.
Exemplo 2.1.

(Combinaes) De quantos modos podemos escolher k elementos dentre n disponveis?


n l-se n escolhe k ou Importante: Tal nmero representado por k

combinao de n k a k.
Resoluo

Vamos contar de duas maneiras o nmero de filas com os k elementos escolhidos. Podemos (i) primeiro escolher os k elementos e coloc-los em filas ou (ii) escolher diretamente os elementos e irmos colocando na fila.
n maneiras de escolhermos os elementos; Fazendo como em (i), temos k

podemos escolher o primeiro da fila de k maneiras, o segundo de k 1 maneiras, e assim por diante. Assim temos n n k! k k k (k 1) ... 1 = maneiras de formar a fila. (lembrete: k (k 1) ... 1 = k! l-se k fatorial). Por outro lado, fazendo como em (ii), temos n maneiras de escolher o primeiro da fila, n 1 maneiras de escolher o segundo e assim por diante, at o ltimo, que pode ser escolhido de n k + 1 maneiras. Assim, temos n (n 1) ... (n k + 1) maneiras de formar a fila. Logo, de (i) e (ii), conclumos que n k! = nmero de filas = n (n 1) ... (n k + 1) k n n (n 1) ... (n k + 1) (n k )! n! = = k (n k )! k!(n k )! k!

Exemplo 2.2.

(Lema de Sperner) Dividimos um tringulo grande 123 em tringulos menores de modo que quaisquer dois dentre os tringulos menores ou no tm ponto em comum, ou tm um vrtice em comum ou tem um lado (completo) em comum.
EUREKA! N12, 2001

35

Sociedade Brasileira de Matemtica

Os vrtices dos tringulos so numerados: 1, 2 ou 3. A numerao arbitrria, exceto que os vrtices sobre os vrtices do tringulo maior oposto ao vrtice i no podem receber o nmero i. Mostre que entre os tringulos menores existe um com os vrtices 1, 2, 3.
1 1 1 1 1 2 2 2 2 2 3 3 1 3 1 1

Resoluo

Contaremos o nmero de segmentos 12 (com algumas repeties). Eles aparecem nos tringulos
1

3 2

Digamos que h x tringulos 123, y tringulos 122 e z tringulos 112. Observe que os segmentos 12 internos ao tringulo grande so contados duas vezes (eles so comuns a dois tringulos) e os segmentos do lado do tringulo grande, somente uma vez. Notemos tambm que os segmentos 12 aparecem duas vezes nos tringulos 122 e 112 e uma vez nos tringulos 123. Assim, 2 segmentos interiores + segmentos nos lados = nmero de segmentos = x + 2 y + 2 z Mostraremos um fato mais forte que o lema: provaremos que x mpar e portanto no pode ser zero. Observando a equao acima, vemos que basta provarmos que o nmero de segmentos 12 sobre os lados do tringulo grande mpar. Como no podemos ter pontos 1 no lado 23 nem pontos 2 no lado 13 , todos os segmentos 12 esto sobre o lado 12 do tringulo grande. Provemos que o nmero de segmentos sobre o lado mpar. Para isso, vamos colocar vrtices 1 ou 2 no lado 12 . Assim, no comeo, temos somente o lado 12 :
EUREKA! N12, 2001

36

Sociedade Brasileira de Matemtica


1 2

Na hora de colocar vrtices, considere o menor segmento em cujo interior colocaremos o vrtice. Poderemos estar em uma das seguintes situaes: Este segmento do tipo 11 :
1 1 ou 2 1 2

Se colocarmos 1, o nmero de segmentos 12 no muda; se colocarmos 2, aumenta de 2. De qualquer forma, a paridade do nmero de segmentos 12 no muda. Este segmento do tipo 22 :
1 2 1 ou 2 2

Se colocarmos 1, o nmero de segmentos 12 aumenta de 2; se colocarmos 2, no muda. De qualquer forma a paridade do nmero de segmentos 12 no muda. Este segmento do tipo 12 :
1 1 1 ou 2 2

Se colocarmos 1 ou 2, o nmero de segmentos 12 no muda e claro que a paridade desse nmero no muda tambm. Logo a paridade do nmero de segmentos 12 nunca muda (ou seja, invariante). Como no comeo temos um segmento 12 (o prprio lado 12 ), temos que o nmero de segmentos 12 no lado do tringulo grande sempre mpar, o que completa nossa demonstrao. Contar algo de duas maneiras tambm nos ajuda a demonstrar desigualdades.
Exemplo 2.3.

Na terra de Oz h n castelos e vrias estradas, sendo que cada uma liga dois castelos e no h mais do que uma estrada ligando diretamente dois castelos. Diz a lenda que se houver quatro castelos ligados em ciclo (ou seja, se existirem quatro castelos A, B, C e D tais que A e B, B e C, C e D e D e A esto ligados), um drago aparecer do centro dos castelos e destruir a Terra de Oz. Mostre que para esta desgraa no acontecer o nmero de estradas deve ser menor ou igual a 1 + 4n 3 n / 4.

EUREKA! N12, 2001

37

Sociedade Brasileira de Matemtica

Resoluo

Considere um castelo ligado a outros dois.

d (v ) Para cada castelo v do conjunto V dos castelos temos pares de estradas. 2 Para a desgraa no ocorrer, observemos que devemos Ter no mximo um par de estradas asociado a um mesmo par de castelos. Assim, a quantidade de pares de estradas menor ou igual quantidade de pares de castelos. Logo

d (v ) n = pares de estradas pares de castelos = 2 vV 2

((d (v ))
vV vV

d (v ) n 2 n

(d (v )) d (v ) n 2 n
2 vV

(*)

Sabemos que a soma

vV

d (v) igual ao dobro do nmero de estradas

2 A . Alm disso, pode-se mostrar (usando a desigualdade entre as mdias quadrtica e aritmtica, ou mesmo Cauchy-Schwarz) que

(d (v))
vV

(
2

vV

d (v)

4A n

Assim n Resolvendo (**) em A , obtemos (*) 4A


2

2 A n 2 n 4 A 2n A n n 2 n 0 (**)

n n n 4n 3 n + n 4n 3 A A 1 + 4n 3 . 4 4 4

EUREKA! N12, 2001

38

Sociedade Brasileira de Matemtica

Exerccios

02. Dizemos que dois poliedros P e Q so equidecomponveis se possvel cortar o poliedro P em vrios poliedros menores e montar, sem deixar espaos vazios e sem sobrar poliedros, o poliedro Q. Sejam 1 , 2 ,..., m os ngulos didricos (ngulos entre faces adjacentes) de P e 1 , 2 ,..., m os ngulos didricos de Q. Mostre que se P e Q so equidecomponveis ento existem nmeros inteiros positivos a1 , a 2 ,..., a m , b1 , b2 ,..., bn e um nmero inteiro k tais que a1 1 + a 2 2 + ... + a m m (b1 1 + b2 2 + ... + bn m ) = k A partir desta relao podemos mostrar (isto um pouco mais difcil!!) que um cubo e um tetraedro de mesmo volume no so equidecomponveis. 03. Dado n inteiro, seja d(n) o nmero de divisores de n. Seja d (n) o nmero mdio de divisores dos nmeros entre 1 e n, ou seja, 1 n d (n) = d ( j) n j =1

Mostre que

1 d ( n) i=2 i

i
i =1

Esta desigualdade nos mostra que d (n) ln n, e que a diferena d (n) ln n no mximo 1. 04. (IMO) Num concurso, h m candidatos e n juzes, onde n 3 mpar. Cada candidato avaliado por cada juiz, podendo ser aprovado ou reprovado. Sabe-se que os julgamentos de cada par de juzes coincidem em no mximo k candidatos. Prove que k n 1 2n m

Referncias Bibliogrficas A parte de grafos foi baseada em um dos captulos do livro Mathematical Circles A Russian Experience, que aborda o treinamento russo para as olimpadas. Muitos dos exerccios de contagem dupla foram extrados do livro Proofs From The Book, que contm as demonstraes consideradas pelos autores (e tambm por muitos leitores!) as mais elegantes.

EUREKA! N12, 2001

39

Sociedade Brasileira de Matemtica

OLIMPADAS AO REDOR DO MUNDO


O comit editorial de EUREKA! agradece aos leitores o crescente envio de solues e aos inmeros elogios para a seo OLIMPADAS AO REDOR DO MUNDO. Continuamos salientando que estamos disposio na OBM para aqueles que estiverem interessados na soluo de algum problema particular. Para tanto, basta contactar a OBM, atravs de carta ou e-mail.
Antonio Luiz Santos

Primeiramente vamos aos problemas propostos deste nmero


121. (Rssia-2001) Os valores da funo quadrtica f (x ) = x + ax + b para dois
2

inteiros consecutivos so os quadrados de dois inteiros tambm consecutivos. Mostre que os valores da funo quadrtica so quadrados perfeitos para todos os inteiros.
122. (Rssia-2001) Determine todos os nmeros inteiros positivos que podem ser

representados de maneira nica sob a forma inteiros positivos.


123. (Rssia-2001) Os

x2 + y onde x e y so xy + 1

senos dos ngulos de um tringulo so nmeros racionais.

Mostre que os seus cossenos so tambem racionais.


124. (Rssia-2001) Dois crculos

s1 e s 2 de centros O1 e O2 intersectam-se nos pontos A e B . Seja M um ponto qualquer do crculo s1 tal que MA intersecta s 2 no ponto P e MB intersecta s 2 no ponto Q . Mostre que se o quadriltero AO1 BO2 cclico (inscritvel) ento AQ e BP intersectamse em s1 .
o

125. (Rssia-2001) Eliminando-se o 2000 . algarismo da expanso decimal da

frao

1 (onde p um nmero primo maior que 5) obtemos a frao p a irredutvel . Mostre que b divisvel por p . b

EUREKA! N12, 2001

40

Sociedade Brasileira de Matemtica

126. (Rssia-2001) Cinco nmeros, um dos quais 2000, so escritos em um

quadro negro. permitido apagar qualquer um destes nmeros e o substituirmos pelo nmero a + b c , onde a , b e c so trs quaisquer dos nmeros restantes. possvel com estas operaes obter cinco nmeros iguais a 2000?
127. (Estnia-2001) Quantos nmeros inteiros positivos menores que 20002001

no contm outros algarismos distintos de 0 e 2?


128. (Estnia-2001) Em um tringulo ABC , as medidas dos seus lados so inteiros

consecutivos e a mediana relativa ao lado BC perpendicular bissetriz interna do ngulo ABC . Determine as medidas dos lados do tringulo ABC .
129. (Estnia-2001) Considere todos os produtos por 2, 4, 6,...,2000 dos elementos

1 1 1 1 1 do conjunto A = , , ,..., , . Determine a soma de todos 2000 2001 2 3 4 estes produtos.


130. (Eslovnia-2001) Para os inteiros positivos
2 2

x e y verdadeira a igualdade

3x + x = 4 y + y . Mostre que x y um quadrado perfeito.


131. (Eslovnia-2001) Determine todos os nmeros primos da forma 101010...101 . 132. (Eslovnia-2001) Gustavo tentou escrever alguns nmeros utilizando somente

o algarismo 1 e o sinal de mais. Ele percebeu , por exemplo, que existem apenas dois inteiros positivos n (13 e 4) para os quais o nmero 13 pode ser escrito utilizando n 1s e o sinal de mais. De fato, o nmero 13 pode ser escrito como a soma de 13 1s ou 11 + 1 + 1 onde 4 1s foram utilizados. Determine quntos so os inteiros positivos n tais que o nmero 125 seja escrito utilizando-se n 1s e o sinal de mais.
133. (Eslovnia-2001) Seja ABC um tringulo retngulo de hipotenusa AC .

Sabendo que sobre o lado BC existem pontos D e E tais que BAD = DAE = EAC e EC = 2 BD . Determine os ngulos do tringulo.

EUREKA! N12, 2001

41

Sociedade Brasileira de Matemtica

134. (Crocia-2001)

Determine todas as funes satisfazem equao : x 3 3+ x f =x + f x +1 1 x

f : #\{-1, 1} # que

135. (Crocia-2001) Se x + y + z = 0 , simplifique

Sugesto : calcule (x + y ) e (x + y )
4

x7 + y7 + z 7 xyz x 4 + y 4 + z 4

136. (Crocia-2001). Dado o nmero n = p1 p 2 p 3 p 4 onde p1 , p 2 , p 3 e p 4 so

quatro nmeros primos distintos. Sejam

d1 = 1 < d 2 < d 3 < < d15 < d16 = n os divisores positivos de n . Determine todos os n < 2001 tais que
d 9 d 8 = 22 .

137. (Albnia-2001) Mostre as igualdades:

3 2 6 1 2 = sen sen 2 (i) cos 2 sen 11 11 11 11 4 3 2 + 4 sen = 11 (ii) tg 11 11


138. (Bielorssia-2001) Em um tringulo issceles ABC , no qual AB = AC e

BAC = 30 , marcam-se os pontos Q e P sobre o lado AB e sobre a mediana AD respectivamente, de modo que PC = PQ ( Q B ). Determine a medida do ngulo PQC .

139. (Bielorssia-2001) Eduardo escreveu todos os produtos, todas as somas e todos

os valores absolutos das diferenas dos inteiros positivos a1 , a 2 ,..., a100 tomados dois a dois. Qual o maior nmero de inteiros mpares obtidos por Eduardo?
140. (Bielorssia-2001). Um inteiro positivo k chamado bom se existe um inteiro

positivo N com k algarismos em sua representao decimal e tal que o i simo algarismo da esquerda para a direita do nmero 3N igual ao
EUREKA! N12, 2001

42

Sociedade Brasileira de Matemtica

algarismo da direita para a esquerda do nmero 2N, 1 i k . Determine a soma de todos os nmeros bons existentes entre 1 e 100.
141. (Bielorssia-2001) Mostre que

(k i + 1) simo

1 1 2 n2 a + 2 n a a para todo inteiro positivo n e todo real positivo a . an +


142. (Finlndia-2001) Determine n N tais que n + 2 divida 2 + 2001n .
2

143. (Itlia-2001) Em um hexgono equingulo, as medidas de quatro lados

consecutivos so, nesta ordem, 5, 3, 6 e 7. Determine as medidas dos outros dois lados.
144. (Itlia-2001) Dada a equao x

= yx , (i) determine todos os pares de solues (x, y) tais que x seja um nmero primo e y um inteiro positivo. (ii) determine todos os pares de solues (x, y) tais que x e y so inteiros positivos.
2001

145. (Israel-2001) As medidas dos lados de um tringulo ABC so 4, 5 e 6. Por um

ponto D qualquer de um de seus lados, traamos as perpendiculares DP e DQ sobre os outros lados. Determine o valor mnimo de PQ.
146. (Israel-2001) Dados 2001 nmeros reais x1 , x 2 ,..., x 2001 tais que 0 x n 1

para cada n = 1,2,...,2001 determine o valor mximo de 1 2001 2 1 2001 xn xn 2001 2001 n =1 n =1 e onde este mximo atingido.

147. (Grcia-2000) Seja

f (2m ) + f (2n ) 2 para todos os inteiros no negativos m e n com m n . Determine a expresso de f(m). f (m + n ) + f (m n ) m + n 1 =

f :  # uma funo tal que f (1) = 3 e

EUREKA! N12, 2001

43

Sociedade Brasileira de Matemtica

148. (Grcia-2001) Fatore a expresso

A = x 4 + y 4 + z 4 2x 2 y 2 2 y 2 z 2 2z 2 x 2 e mostre que a equao A = 2000 no possui soluo no conjunto dos nmeros inteiros.
149. (Hungria-2000) O produto de 2001 inteiros positivos distintos possui

exatamente 2000 divisores primos distintos. Mostre que podemos escolher alguns destes 2001 nmeros de modo que seu produto seja um quadrado perfeito.
150. (Hungria-2000) Seja S o nmero de subconjuntos com 77 elementos de H = {1,

2, 3,...2000, 2001} para os quais a soma dos elementos de cada subconjunto seja par e seja N o nmero de subconjuntos com 77 elementos de H para os quais a soma dos elementos de cada subconjunto seja mpar. Determine qual dos dois S ou N maior e o quanto? Agora vamos aos comentrios e solues dos leitores para alguns dos problemas apresentados em nossa seo nos nmeros anteriores de EUREKA!.
9. (Irlanda-1998) Um tringulo ABC possui medidas dos lados expressas por nmeros inteiros, A = 2 B e C > 90 . Determine o valor mnimo do permetro deste tringulo.

A resposta 77. De fato, fazendo-se B = x temos A = 2 x , sejam a, b e c as medidas dos lados opostos aos ngulos A , B e C respectivamente. Prolongando-se o lado CA no sentido de C para A at o ponto D de um comprimento igual a c, vemos que o tringulo BCD semelhante ao tringulo ABC logo, b a ou a 2 = b(b + c ) = a b+c Como estamos procurando um tringulo com o menor permetro possvel, podemos supor que a, b e c no possuem fatores primos comuns pois, de outra forma um outro exemplo menor haveria. De fato, b e c devem ser primos entre si porque qualquer fator primo comum a b e c seria tambm um fator de a. Alem disso, como o produto b(b + c) um quadrado perfeito que igual ao produto de dois nmeros primos entre si tanto b quanto b + c devem eles mesmos serem
EUREKA! N12, 2001

SOLUO DE EINSTEIN DO NASCIMENTO JNIOR (FORTALEZA CE) (com adaptaes):

44

Sociedade Brasileira de Matemtica

quadrados perfeitos. Deste modo, para m e n inteiros positivos quaisquer primos entre si temos b = m2, b + c = n2, a = mn e da n a = = 2 cos B (pela lei dos senos) m b o que implica em O ngulo C = 3 B obtuso assim, 0 < B < 6 n 3 < cos B < 1 e portanto, 3 < < 2 . facil ver que esta desigualdade no 2 m possui solues inteiras com m = 1, 2 ou 3 da m 4 , n 7 e a + b + c = mn + n 2 4 7 + 7 2 = 77 De fato, o par (m, n) = (4, 7) gera o tringulo (a, b, c) = (28, 16, 33) que satisfaz todas as condies do problema.
10.

Em um tringulo ABC tem-se que BAC = 40 e ABC = 60 . Sejam D e E pontos sobre os lados AC e AB respectivamente tais que CBD = 40 e BCE = 70 . Seja F o ponto de interseo de BD e CE. Mostre que a reta que que contm AF perpendicular que contm BC.
(Canad-1998)

SOLUO DE LUCAS DE MELO PONTES E SILVA (FORTALEZA CE):

Sejam X o ponto de interseo de AF com BC e H o p da altura que parte de A. Ento pela lei dos senos temos nos tringulos CBD e BDA : CD BD DA BD = = e sen40 sen80 sen20 sen40 CD sen40sen40 = ou . DA sen80sen20 EA sen60sen10 = . Pelo Teorema de Ceva, se AX, BD e CE Analogamente EB sen70sen40 so cevianas concorrentes no tringulo ABC tem-se que:
CD EA BX sen40sen40sen60sen10 BX =1 = 1 ou DA EB XC sen20sen80sen70sen40 XC
XC 2 sen 20 cos 20sen60 cos 80 sen60 tg 60 = = = 1 tg 80 tg 80 BX sen 20sen80 cos 20 2 Como AH temos nos tringulos AHB e AHC respectivamente:

EUREKA! N12, 2001

45

Sociedade Brasileira de Matemtica

tg 60 = Logo

AH AH tg 60 HC e tg 80 = logo = HB HC tg 80 HB

HC XC HC + HB XC + XB = = e como BAC agudo ento H est HB XB HB XB entre B e C e portanto, BC BC = XB = HB X coincide com H. HB XB


13. (Irlanda-1999). Uma funo

(i) f (ab ) = f (a ) f (b ) se o mximo divisor comum de a e b 1.

f :   satisfaz s seguintes condies:

Mostre que f (2 ) = 2, f (3) = 3 e f (1999 ) = 1999 .

(ii) f ( p + q ) = f ( p ) + f (q ) para todos os nmeros primos p e q.

SOLUO DE MARCLIO MIRANDA DE CARVALHO (TERESINA PI):

Seja p um nmero primo mpar. Ento, f (2 p ) = f ( p + p ) = f ( p ) + f ( p ) = 2 f ( p ) enquanto que f (2 p ) = f (2) f ( p ) e da, f (2 ) = 2. Agora, Por f (4 ) = f (2) + f (2 ) = 4 f (12 ) = 4 f (3) . f (12 ) = f (7 ) + f (5) f (12 ) = 2 f (2) + f (3) + f (2 ) + f (3)
f (3) = 3.

outro

lado,

Finalmente,

f (26) = 26 f (23) = 23.

f (5) = f (2 ) + f (3) = 5 f (15) = 15 f (13) = 13

Por outro lado,

f (29) = 29.

f (13) = 13 f (11) = 11 f (33) = 33 f (31) = 31

Logo,

f (2001) = f (3) f (23) f (29 ) = 2001 f (1999 ) = 1999

EUREKA! N12, 2001

46

Sociedade Brasileira de Matemtica

16.

(Estnia-1999) Mostre que o segmento que une o ortocentro e o baricentro de um tringulo acutngulo ABC paralelo ao lado AB se, e somente se, tgA tgB = 3 .

SOLUO DE GERALDO PERLINO JNIOR (SO PAULO SP) (com adaptaes): Sejam D e E os ps das alturas traadas dos vrtices A e C respectivamente. Se H o ortocentro do tringulo, observe que H pertence uma reta paralela a AB e CE = 3 . Deste modo, basta mostrar que passando pelo baricentro se, e s se EH CE = tgA tgB . AHE = CHD Como temos tambm que EH EAH = BCE , isto os tringulos retngulos CEB e AEH so semelhantes e CE AE CE = . Observando que tgA = , obtemos a relao necessria. tgB = EB EH AE 23. (Rssia-1999) A soma dos algarismos de um inteiro positivo n escrito no sistema de numerao decimal igual a 100 e a soma dos algarismos do nmero 44n 800. Determine a soma dos algarismos do nmero 3n.

SOLUO DE MARCLIO MIRANDA DE CARVALHO (TERESINA PI):

Sejam a1 , a2 , a3 ,..., ak os algarismos de n e S (n ) a soma de seus algarismos. Ento, 44n = 40(a1 a 2 ...a k ) + 4(a1 a 2 ...a k ) = 4(a1 a 2 ...a k ) (10 + 1) Logo, se os algarismos de 44 n forem : 4a1 , (4 a2 + 4 a1 ) , (4a 3 + 4a 2 ) , ... , (4a k + 4a k 1 ) e 4a k teramos S (44n ) = 8 (a1 + a 2 + + a k ) = 800 . Assim, se houvesse algum vaium, a soma dos algarismos de 44n cairia . Deste modo, todos os algarismos de n so menores ou iguais a 2 donde os algarismos de 3n so 3a1 ,3a 2 ,3a3 ,...,3a k . Assim, S (3n) = 3a1 + 3a 2 + 3a3 + ... + 3a k = 3.S (n) = 300 .

EUREKA! N12, 2001

47

Sociedade Brasileira de Matemtica

Acusamos o recebimento de solues de problemas anteriores dos seguintes leitores de EUREKA!:


Anderson Torres Geraldo Perlino Ioziel Matos Corra Jnior Jorge Silva Jnior Marcelo R. De Souza Marcelo Rufino de Oliveira Mauro Flix de Souza Osvaldo Mello Sponquiado Paulo Alexandre Arajo Sousa Renato Francisco Lopes Mello Wallace Rodrigues de Holanda Miranda Wilson Carlos de Silva Ramos So Paulo SP Itapec. da Serra SP Rio de Janeiro RJ Cachoeiro do Itapemirim ES Rio de Janeiro RJ Belm PA Cordovil RJ Olmpia SP Teresina PI Jaboato dos Guararapes PE Teresina PI Belm PA Prob.16, 52, 67, 69, 70, 76, 102, 105, 118, 120. Prob. 115. Prob. 52, 62, 71, 74, 76, 78. Prob. 96, 98, 99, 101, 103, 110. Prob. 95, 115. Prob. 91, 92, 93, 95, 97, 98, 99, 101, 102, 104, 107, 109, 110, 111, 112, 114, 115, 118. Prob. 98. Prob. 29, 30, 38, 56, 57, 59, 63, 66, 67, 69, 74, 98, 107, 114, 118. Prob. 62, 78, 82, 88, 76, 70. Prob. 98, 99, 101, 109, 117. Prob. 97, 98, 99, 101, 102, 104, 106, 109, 112, 114, 116, 117, 120. Prob. 43, 68, 74, 82.

'4.H8,- ,

":0 14 /08.4-0794 :2 34;4 57 24 /0 0780330 02


 6:0 902    /J 948 0      A 4 3 2074 
 

F ,4 ,/4 /0  2, 8 /0 :2 2 /0  ,348

 

 :2 /48 /4 8 57 248 .43 0. /48 .42 /08.4-7 /47  . ,0 ,20743 /4 !$ :2 574 094 5,79 . 5,390 3,

4 /0 /J 948 F :2

.44507,9 ;4 5,7, 574.:7,7 57 248 /0 0780330 438: 90 3, 3907309 , 5E 995

20780330 47 57 20 92

EUREKA! N12, 2001

48

Sociedade Brasileira de Matemtica

 Publicamos aqui algumas das respostas enviadas por nossos leitores.

SOLUES DE PROBLEMAS PROPOSTOS

56) Para cada nmero n, seja f(n) a quantidade de maneiras que se pode expressar n como a soma de nmeros iguais a 1, 3 ou 4. Por exemplo, f(4) = 4, pois todas as maneiras possveis so 4 = 1 + 1 + 1 + 1, 4 = 1 + 3, 4 = 3 + 1, 4 = 4. Demonstrar que se n par, f(n) um quadrado perfeito.
SOLUO DE MRCIO ASSAD COHEN (RIO DE JANEIRO RJ):

O coeficiente de xn em (x + x3 + x4)k conta exatamente uma vez cada modo de representar o nmero n na forma + 3 + 4 com + + = k , ou seja, conta os modos de representar n utilizando 1s, 3s e 4s com um total de k nmeros. Variando k, segue que f(n) o coeficiente de xn na srie formal g ( x) =

(x + x
k =0

+ x4 )k .

Somando a PG, fatorando, e decompondo em fraes parciais tem-se: 1 g ( x) = 1 x x3 x4


g ( x) = 1 1 x+2 1 x+3 + = 2 2 5 1 x x2 (1 + x )(1 x x ) 5 1 + x
2

Utilizando respectivamente soma de PG e o mtodo desenvolvido para sries formais no artigo da Revista Eureka! No. 11 tem-se: 1 = 1 x 2 + x 4 x 6 + ... 1+ x2 1 = F0 + F1 x + F2 x 2 + F3 x 3 + ... 2 1 x x onde Fn o n-simo nmero de Fibonacci, com F0 = F1 = 1, Fn+2 = Fn+1 + Fn. Se n par, n = 2t, o coeficiente de xn no desenvolvimento de g(x) ser ento:

[x ]g ( x ) = 2 ( 1)
n EUREKA! N12, 2001

+ F2 t 1 + 3 F2 t 5

49

Sociedade Brasileira de Matemtica

Utilizando que Fn =

1 5
t

n +1

n +1 , onde =

1+ 5 1 5 , = : 2 2

2 (1) + F2t 1 + 3F2t = 2 (1) t + [( F2t 1 + F2t ) + F2t ] + F2t = 2 (1) t + F2t + 2 + F2t = 2 (1) t + 2 (1) t + Notando que = 1 : 1 5 1 5

2t + 3

2t +3 +
1 2t + 2

1 5

2 t +1

2t +1 =

[( + )

+ 1 2t + 2

+ 1 = + 1
Juntando tudo:

1+ 5 1 5 = 5 2 2 = 1 = 5

[x ]g ( x ) = 1 [2 ( 1) 5
n

+ 2t + 2 + 2t + 2 = = Ft 2
2

2 t + 2 + 2 t + 2 2 ( ) t +1 t +1 t +1 = 5 5
E, em particular, f(n) quadrado perfeito sempre que n par.

58) Determine todos os primos p para os quais o nmero de um inteiro.


SOLUO DE RODRIGO VILLARD MILET (RIO DE JANEIRO RJ):

2 p 1 1 o quadrado p

Resposta: p = 3 ou p = 7.
EUREKA! N12, 2001

50

Sociedade Brasileira de Matemtica

Primeiramente, note que

2 p 1 1 inteiro, para p > 2 (pelo pequeno Teorema de p


2p1 1 quadrado, p

Fermat), e p = 2 no serve. Da, como p mpar, 2 p 1 1 0(mod 3). Se p = 3, 2 31 1 = 1 que quadrado. Se p = 3, como 3

9 2 p 1 1 2 p 1 1(mod 9) ord 9 2 = 6 p 1 p 1(mod 6).

2 p 1 1 2 6 k 1 ( 2 3 k 1)( 2 3 k + 1) k ; p = 1 + 6k = = p p p
mdc{2 3 k 1, 2 3 k + 1} = mdc{2, 2 3 k + 1} = 1 como p primo ( p > 3), p 2 3 k 1

ou p 2 3 k + 1. (i) p 2 3 k 1. Da, como 2 3 k 1 e 2 3 k + 1 no tem fatores em comum, 2 3 k + 1 quadrado q N ;2 3 k + 1 = q 2 2 3 k = ( q 1)( q + 1) q = 3 e k = 1 (de fato, q-1 e q+1 devem ser potncias de 2,o que s possvel se forem 2 e 4). Nesse caso, 2 7 1 1 = 9 que quadrado. 7 (ii) p 2 3 k + 1. (Assuma k > 1, pois k = 1 j soluo)

Da, 2 3 k 1 quadrado m N ;2 3 k 1 = m 2 2 k 1 2 2 k + 2 k + 1 = m 2 . Seja


d = mdc{2 k 1, 2 2 k + 2 k + 1} . d 2 k 1 d ( 2 k 1) = ( 2
2 2k

)(

2 2 k + 1),
divide

mas

d 2 2 k + 2 k + 1,

logo

d 3.2 k .

Como

d 2 k 1, d no
No entanto,
2
2k k

2 k d | 3 d = 1 ou 3 . d = 1 2 2k + 2 k + 1
(2 ) < 2 quadrado.
k 2 2k k

+ 2 +1< 2 +1 ,
k

quadrado.

temos ser

ento no possvel

+ 2 +1

EUREKA! N12, 2001

51

Sociedade Brasileira de Matemtica

2k 1 quadrado. d =3 3
Como
2k 1 mpar, 3

2k 1 1(mod 8) 2 k 4 mod 8 k = 2 p = 13 , que no soluo. 3


59) Um pedestal de altura a sustenta uma coluna de altura b (b > a). A que distncia do monumento se deve colocar um observador para ver o pedestal e a coluna sob ngulos iguais?
SOLUO DE CARLOS ALBERTO DA SILVA VCTOR (NILPOLIS RJ):

Supondo a altura do observador desprezvel, teremos:

(b > a)

a a+b tg = ; tg 2 = x x 2tg 2.a/x a + b a+b = = 2 x x 1 tg a2 1 2 x 2 2a.x a+b = 2ax 2 = (a + b )x 2 a 2 (a + b) (b a )x 2 = a 2 (a + b ), 2 2 x x(a a )


logo
x = a. a+b . ba

EUREKA! N12, 2001

52

Sociedade Brasileira de Matemtica

60) Se num tringulo ABC , A = 2B, provar que a 2 = b(b + c). Obs.: a, b e c so, respectivamente, os lados opostos aos ngulos A, B e C.
SOLUO DE MARCELO RIBEIRO DE SOUZA (RIO DE JANEIRO RJ):

A 2x c b

x B a C

Pela lei dos senos, temos: I) senx sen2 x = b a senx 2 senx cos x = , obviamente x k , k b a a = 2b cos x (1)

II) Usando lei dos cossenos: b 2 = a 2 + c 2 2ac cos x 2ac cos x = a 2 + c 2 b 2 a2 + c2 b2 (2) cos x = 2ac a2 + c2 b2 III) Substituindo (2) em (1): a = 2b ac 2

a 2 c = a 2 b + bc 2 b 3 a 2 (c b ) = b ( c 2 b 2 ) a 2 (c b) = b(c b)(c + b).


Se b c, temos a 2 = b(b + c). Se b=c, temos b=c=x, donde 4 x = x = / 4 e

a = b 2 a 2 = 2b 2 = b(b + c).

EUREKA! N12, 2001

53

Sociedade Brasileira de Matemtica

61) Na figura abaixo um quadrado EFGH foi colocado no interior do quadrado ABCD, determinando 4 quadrilteros. Se a, b, c, e d denotam os medidas das reas dos quadrilteros, mostre que a + b = c + d .
a c d b

SOLUO DE MRCIO ASSAD COHEN (RIO DE JANEIRO RJ): Lema: A rea de um tringulo cujos vrtices no plano complexo so z1, z2, z3

1 Im ( z 2 z1 )( z 3 z1 ) . 2 Prova: Se z 2 z1 = a cis 1 ; z 3 z1 = b cis 2 : (sentido horrio) dada por S =

1 1 Im ( z 2 z 1 ) ( z 3 z 1 ) = Im { a cis 1 b cis ( 2 )}= 2 2 ab absen Im { cis ( 1 2 )}= =S 2 2


Problema:

i
w+iv w+v w wv wiv

-1

-i

EUREKA! N12, 2001

54

Sociedade Brasileira de Matemtica

Coloque os eixos de modo que os vrtices do quadrado maior sejam 1, i, 1, i (basta adotar uma unidade de medida tal que 2u.m = diagonal). Seja w o centro do quadrado menor, e w + v um de seus vrtices. Ento, os outros vrtices do quadrado menor sero w + iv, w i, w iv (pois rodar 90o multiplicar por i). Dividindo cada quadriltero em tringulos como mostra a figura temos: 1 a = Im ( w + iv 1)(i 1) + (1 w iv)v(1 i ) 2 1 c = Im ( w iv 1)(i + 1) + (1 w + iv)v(i 1) 2 1 a + c = Im (1 + i) ( w iv + 1) + v(1 w iv) + ( w iv + 1) + v(1 + w iv) 2

]}

Note que w cancelado e a + c independe do centro do quadrado menor: a + c = Im (1 + i)(1 iv)(1 + v) Da mesma forma, obtemos:

1 b + d = Im (w + v + i)(1 + i) + (i w v)v(i 1) + (w v i)(1 i) + (1 w + v)v(1 + i) = 2 1 Im (1 i) (w + v + i) + (i + w + v)v + (w + v + i) + (w + v + i)v = 2 Im (1 i)(i + v)(v + 1)

]}

Fatorando, obtemos b + d = Im i(i + 1)(i + v)(v + 1) = Im (i + 1)(1 iv)(v + 1) = a + c . 62) Se ABCD um quadriltero convexo tal que os lados AB, BC , CD e DA medem respectivamente a, b, c e d e que , , e so as medidas dos seus ngulos internos, mostre que a medida da rea desse quadriltero, denotada por (ABCD), dada por:

EUREKA! N12, 2001

55

Sociedade Brasileira de Matemtica


D

d1

(ABCD) = p=

( p a )( p b)( p c)( p d ) abcd cos 2 onde:

a+b+c+d 2 + + = (a frmula tambm vale se fizermos = ). 2 2


SOLUO DE WALLACE RODRIGUES DE HOLANDA MIRANDA (TERESINA PI):

Antes da resoluo do problema precisa-se ressaltar algumas noes fundamentais: 1) cos( + ) = cos cos sen sen cos(2 ) = cos cos sen sen

cos 2 sen 2 = cos cos sen sen


cos 2 1 cos 2 = cos cos sen sen
sen sen = cos cos 2 cos 2 + 1

2)

2 2 3) Pela lei dos cossenos nos tringulos ABD e BCD:

(ABD ) = 1 a d sen

(BCD ) = 1 b c sen

a 2 + d 2 2ad cos = d 12 = b 2 + c 2 2bc cos


a 2 + d 2 b 2 c 2 = 2ad cos 2bc cos

Agora vamos resoluo do problema: (ABCD) = (ABD) + (BCD)


EUREKA! N12, 2001

56

Sociedade Brasileira de Matemtica

(ABCD ) = 1 a d sen + 1 b c sen


(2(ABCD ))2 = (a d sen + b c sen )2 2 4(ABCD ) = a 2 d 2 sen 2 + 2abcd sen sen + b 2 c 2 sen 2 Substituindo (sen 2 ) por ( 1 cos 2 ) e (sen sen ) por
2(ABCD ) = a d sen + b c sen

(cos cos 2 cos


2 2 2

+ 1 de 1) fica:

4(ABCD) = a d 1 cos 2 + 2abcd 1 + cos cos 2 cos 2 +

4(ABCD ) = a d 2 1 cos 2 + abcd (2 + 2 cos cos ) +


2 2

+ b 2 c 2 1 cos 2
2 2

+ b c 1 cos 4abcd cos 2


2 2 2 2

4(ABCD ) = a d 1 cos 2 + abcd (1 cos cos + cos cos + 1 +


2 2 2

4( ABCD) = a d 1 cos + abcd [(1 cos )(1 cos ) + (1 + cos )(1 + cos )] +
2

+ cos + cos + cos cos ) + b 2 c 2 1 cos 2 4 abcd cos 2

b 2 c 2 1 cos 2 4abcd cos 2


4( ABCD) 2 = ad (1 + cos )ad (1 cos ) + ad (1 cos )bc(1 cos ) + 4( ABCD) 2 = ad (1 + cos )[ad (1 cos ) + bc(1 + cos )] + bc(1 cos )[ad (1 cos ) + 4( ABCD) 2 = [ad (1 cos ) + bc(1 + cos )][ad (1 + cos ) + bc(1 cos )] 4abcd cos2 16( ABCD) 2 = 2(ad ad cos + bc + bc cos ) 2 (ad + ad cos + bc bc cos )
16abcd cos 2 16( ABCD) 2 = (2ad cos 2bc cos 2ad 2bc)(2ad cos 2bc cos +

+ ad (1 + cos )bc(1 + cos ) + bc(1 + cos )bc(1 cos ) 4abcd cos 2

+ bc(1 + cos )] 4abcd cos2

+ 2ad + 2bc) 16abcd cos 2 Substituindo (2ad cos 2bc cos ) por (a 2 + d 2 b 2 c 2 ) de 3) fica: 16( ABCD) 2 = (a 2 2ad + d 2 b 2 2bc c 2 )(a 2 + 2ad + d 2 b 2 + 2bc c 2 ) 16abcd cos 2 16 ( ABCD ) 2 = ( a d ) 2 (b + c ) 2 ( a + d ) (b c ) 2 16 abcd cos 2

][

EUREKA! N12, 2001

57

Sociedade Brasileira de Matemtica

16( ABCD ) 2 = (a d + b + c)(a d b c)(a + d b + c)(a + d + b c) 16abcd cos2 16( ABCD ) 2 = (a + b + c d )(a + b + c + d )(a b + c + d )(a + b c + d ) 16abcd cos2 16( ABCD ) 2 = (a + b + c + d )(a b + c + d )(a + b c + d )(a + b + c d ) 16abcd cos2
( ABCD )2 = (a + b + c + d) (a b + c + d) (a + b c + d ) (a + b + c d) abcd cos2 2 2 2 2 ( ABCD) 2 = ( p a )( p b)( p c)( p d ) abcd cos 2
( ABCD) = ( p a )( p b)( p c)( p d ) abcd cos 2 .
Agradecemos tambm o envio das solues e a colaborao de: Bruno de Souza Ramos Everaldo de Melo Bonotto Fernando Carvalho Ramos Geraldo Perlino Jorge Silva Junior Marcelo Ribeiro Marcelo Rufino de Oliveira Marclio Miranda de Carvalho Mateus Queiroz Guilherme de Oliveira Oswaldo Melo Sponquiado Realengo RJ Guararapes SP Santa Mara RS Itapecerica da Serra SP Cachoeiro de Itapemerim ES Rio de Janeiro RJ Belm PA Teresina PI Fortaleza CE Olmpia SP

Seguimos aguardando o envio de solues do problema proposto No. 57 publicado na revista Eureka! No. 11.

EUREKA! N12, 2001

58

Sociedade Brasileira de Matemtica

PROBLEMAS PROPOSTOS
Convidamos o leitor a enviar solues dos problemas propostos e sugestes de novos problemas para os prximos nmeros.

63) Prove que existem infinitos nmeros naturais mltiplos de 51000 sem nenhum 0 na representao decimal. 64) Iniciando de um certo inteiro positivo, permitido fazer apenas uma operao: o dgito das unidades separado e multiplicado por 4, e ento este valor somado ao restante do nmero. Por exemplo, o nmero 1997 transformado para 7.4 + 199 = 227. A operao feita repetidamente. Prove que se a seqncia de nmeros obtida contm 1001, ento nenhum dos nmeros na seqncia pode ser um nmero primo. 65) Determine todos os inteiros N tais que, em base 10, os dgitos de 9N so os mesmos dgitos de N na ordem inversa, e N possui no mximo um dgito igual a 0. com n 66) Prove que, dados um inteiro n 1 e um conjunto A / n 2 B /n elementos existe com n elementos tal que 2 A + B = x + y x A, y B / n tem mais de n 2 / 2 elementos.
2

67) Seja ABCD um quadriltero tal que os crculos circunscritos aos tringulos ABC e BCD so ortogonais.Prove que os crculos circunscritos aos tringulos BCD e DAB tambm so ortogonais.

Problema 63 proposto por Wallace Rodrigues de Holanda Miranda (Teresina PI); Problemas 64 e 65 propostos por Marcelo Rufino de Oliveira (Belm PA); Problema 67 proposto por Luciano Castro (Rio de Janeiro RJ).

EUREKA! N12, 2001

59

Sociedade Brasileira de Matemtica

AGENDA OLMPICA
XXIV OLIMPADA BRASILEIRA DE MATEMTICA NVEIS 1, 2 e 3 Primeira Fase Sbado, 8 de junho de 2002 Segunda Fase Sbado, 14 de setembro de 2002 Terceira Fase Sbado, 19 de outubro de 2002 (nveis 1, 2 e 3) Domingo, 20 de outubro de 2002 (nveis 2 e 3 - segundo dia de prova). NVEL UNIVERSITRIO Primeira Fase Sbado, 14 de setembro de 2002 Segunda Fase Sbado, 19 e Domingo, 20 de outubro de 2002

VIII OLIMPADA DE MAIO maio de 2002

XIII OLIMPADA DE MATEMTICA DO CONE SUL junho de 2002 Fortaleza CE, Brasil

XLIII OLIMPADA INTERNACIONAL DE MATEMTICA julho de 2002 Glasgow, Reino Unido

XVII OLIMPADA IBEROAMERICANA DE MATEMTICA setembro de 2002 El Salvador

V OLIMPADA IBEROAMERICANA DE MATEMTICA UNIVERSITRIA outubro de 2002

EUREKA! N12, 2001

60

Sociedade Brasileira de Matemtica

COORDENADORES REGIONAIS
Amarsio da Silva Arajo Alberto Hassen Raad Angela Camargo Benedito Tadeu Vasconcelos Freire Carlos Frederico Borges Palmeira Claudio Arconcher Claus Haetinger Cleonor Crescncio das Neves lio Mega Rosngela Souza Florncio Ferreira Guimares Filho Gisele de Arajo Prateado Gusmo Ivanilde Fernandes Saad Jacqueline Fabiola Rojas Arancibia Joo Bencio de Melo Neto Joo Francisco Melo Libonati Irene Nakaoka Jos Carlos Pinto Leivas Jos Cloves Saraiva Jos Gaspar Ruas Filho Jos Luiz Rosas Pinho Jos Vieira Alves Marcelo Rufino de Oliveira Licio Hernandes Bezerra Luzinalva Miranda de Amorim Marcondes Cavalcante Frana Pablo Rodrigo Ganassim Paulo Henrique Cruz Neiva de Lima Jr. Reinaldo Gen Ichiro Arakaki Ricardo Amorim Roberto Vizeu Barros Srgio Cludio Ramos Silvio de Barros Melo Tadeu Ferreira Gomes Toms Menndez Rodrigues Valdenberg Arajo da Silva Wagner Pereira Lopes (UFV) (UFJF) (Centro de Educao de Adultos CEA) (UFRN) (PUC-Rio) (Colgio Leonardo da Vinci) (UNIVATES) (UTAM) (Colgio Etapa) (Colgio Singular) (UFES) (UFGO) (UC. Dom Bosco) (UFPB) (UFPI) (Grupo Educacional Ideal) (UEM) (UFRG) (UFMA) (ICMC-USP) (UFSC) (UFPB) (Sistema Titular de Ensino) (UFSC) (UFBA) (UFC) (Liceu Terras do Engenho) (Escola Tcnica Everardo Passos) (INPE) (Centro Educacional Logos) (Colgio Acae) (IM-UFRGS) (UFPE) (UEBA) (U. Federal de Rondnia) (U. Federal de Sergipe) (Escola Tcnica Federal de Gois) Viosa MG Juiz de Fora MG Blumenau SC Natal RN Rio de Janeiro RJ Jundia SP Lajeado RS Manaus AM So Paulo SP Santo Andr SP Vitria ES Goinia GO Campo Grande MS Joo Pessoa PB Teresina PI Belm PA Maring PR Rio Grande RS So Luis MA So Carlos SP Florianpolis SC Campina Grande PB Belm PA Florianpolis SC Salvador BA Fortaleza CE Piracicaba SP SJ dos Campos SP SJ dos Campos SP Nova Iguau RJ Volta Redonda RJ Porto Alegre RS Recife PE Juazeiro BA Porto Velho RO So Cristovo SE Jata GO

EUREKA! N12, 2001

61

CONTEDO

XXIII OLIMPADA BRASILEIRA DE MATEMTICA Problemas e Solues da Primeira Fase XXIII OLIMPADA BRASILEIRA DE MATEMTICA Problemas e Solues da Segunda Fase XXIII OLIMPADA BRASILEIRA DE MATEMTICA Problemas e Solues da Terceira Fase XXIII OLIMPADA BRASILEIRA DE MATEMTICA Problemas e Solues da Primeira Fase - Nvel Universitrio XXIII OLIMPADA BRASILEIRA DE MATEMTICA Problemas e Solues da Segunda Fase - Nvel Universitrio XXIII OLIMPADA BRASILEIRA DE MATEMTICA Premiados AGENDA OLMPICA COORDENADORES REGIONAIS

11

21

41

46

57

61 62

Sociedade Brasileira de Matemtica

XXIII OLIMPADA BRASILEIRA DE MATEMTICA


Problemas e Solues da Primeira Fase PROBLEMAS NVEL 1
1. Considere dois nmeros naturais, cada um deles com trs algarismos diferentes. O maior deles s tem algarismos pares e o menor s tem algarismos mpares. O menor valor possvel para a diferena entre eles : A) 111 B) 49 C) 29 D) 69 E) 5 2. Na figura abaixo, temos 4 circunferncias e alguns pontos destacados no interior dessas circunferncias. Escolhendo exatamente um desses pontos dentro de cada uma das circunferncias, e unindo-os por segmentos de reta que no se cruzam, formamos um quadriltero. Quantos quadrilteros diferentes seremos capazes de desenhar nessas condies?

A) 4

B) 14

C) 60

D) 120

E) 24

3. Joana escreve a seqncia de nmeros naturais 1, 6, 11,..., onde cada nmero, com exceo do primeiro, igual ao anterior mais cinco. Joana pra quando encontra o primeiro nmero de trs algarismos. Esse nmero : A) 100 B) 104 C) 101 D) 103 E) 102 4. Quantos nmeros de dois algarismos no so primos nem mltiplos de 2, 3 ou 5? A) 1 B) 3 C) 2 D) 4 E) mais de 4 5. No conjunto {101, 1 001, 10 001,..., 1 000 000 000 001} cada elemento um nmero formado pelo algarismo 1 nas extremidades e por algarismos 0 entre eles. Alguns desses elementos so nmeros primos e outros so compostos. Sobre a quantidade de nmeros compostos podemos afirmar que: A) igual 11 B) igual a 4 C) menor do que 3 D) maior do que 4 e menor do que 11 E) 3
EUREKA! N13, 2002

Sociedade Brasileira de Matemtica

6. Uma pra tem cerca de 90% de gua e 10% de matria slida. Um produtor coloca 100 quilogramas de pra para desidratar at o ponto em que a gua represente 60% da massa total. Quantos litros de gua sero evaporados? (lembre-se: 1 litro de gua tem massa de 1 quilograma). A) 15 litros B) 45 litros C) 75 litros D) 80 litros E) 30 litros 7. O tringulo equiltero T direita tem lado 1. Juntando tringulos congruentes a esse, podemos formar outros tringulos equilteros maiores, conforme indicado no desenho abaixo.

Qual o lado do tringulo equiltero formado por 49 dos tringulos T? A) 7 B) 49 C) 13 D) 21 E) impossvel formar um tringulo equiltero com esse nmero de tringulos T 8. Os nmeros inteiros positivos de 1 a 1000 so escritos lado a lado, em ordem crescente, formando a seqncia 123456789101112131415... 9991000. Nesta seqncia, quantas vezes aparece o grupo 89 ? A) 98 B) 32 C) 22 D) 89 E) 21 9. Um serralheiro tem 10 pedaos de 3 elos de ferro cada um, mostrados abaixo.

Ele quer fazer uma nica corrente de 30 elos. Para abrir e depois soldar um elo o serralheiro leva 5 minutos. Quantos minutos no mnimo ele levar para fazer a corrente? A) 30 B) 35 C) 40 D) 45 E) 50 10. Escrevem-se os nmeros naturais numa faixa decorativa, da seguinte maneira:
1 3 5 7

Assinale a figura correta:

EUREKA! N13, 2002

Sociedade Brasileira de Matemtica


A) a)
2 00 1

B) b)

2 00 1

C) c)

2 00 0

2 00 0

2 00 0

2 00 1

d)

D)

2 00 1

E)

e)

2 00 1

2 00 0

2 00 0

11. 2 melancias custam o mesmo que 9 laranjas mais 6 bananas; alm disso, meia dzia de bananas custa a metade de uma melancia. Portanto, o preo pago por uma dzia de laranjas e uma dzia de bananas igual ao preo de: A) 3 melancias B) 4 melancias C) 6 melancias D) 5 melancias E) 2 melancias 12. Qual o ltimo algarismo da soma de 70 nmeros inteiros positivos consecutivos? A) 4 B) 0 C) 7 D) 5 E) Faltam dados 13. Em Tumblia, um quilograma de moedas de 50 centavos equivale em dinheiro a dois quilogramas de moedas de 20 centavos. Sendo 8 gramas o peso de uma moeda de 20 centavos, uma moeda de 50 centavos pesar: A) 15 gramas B) 10 gramas C) 12 gramas D) 20 gramas E) 22 gramas 14. As medidas dos lados de um retngulo so nmeros inteiros distintos. O permetro e a rea do retngulo se exprimem pelo mesmo nmero. Determine esse nmero. A) 18 B) 12 C) 24 D) 9 E) 36 15. O nmero N de trs algarismos multiplicado por 7 deu como resultado um nmero que termina em 171. A soma dos algarismos de N : A) 10 B) 11 C) 12 D) 13 E) 14 16. Em um tabuleiro retangular com 6 linhas e 9 colunas, 32 casas esto ocupadas. Podemos afirmar que: A) B) C) Todas as colunas tm pelo menos 3 casas ocupadas. Nenhuma coluna tem mais de 3 casas ocupadas. Alguma coluna no tem casas ocupadas.

EUREKA! N13, 2002

Sociedade Brasileira de Matemtica

D) E)

Alguma linha tem pelo menos 6 casas ocupadas. Todas as linhas tm pelo menos 4 casas ocupadas.

17. Contando-se os alunos de uma classe de 4 em 4 sobram 2, e contando-se de 5 em 5 sobra 1. Sabendo-se que 15 alunos so meninas e que nesta classe o nmero de meninas maior que o nmero de meninos, o nmero de meninos nesta classe : A) 7 B) 8 C) 9 D) 10 E) 11 18. So escritos todos os nmeros de 1 a 999 nos quais o algarismo 1 aparece exatamente 2 vezes (tais como, 11, 121, 411, etc). A soma de todos estes nmeros : A) 6882 B) 5994 C) 4668 D) 7224 E) 3448 19. Cinco animais A, B, C, D, e E, so ces ou so lobos. Ces sempre contam a verdade e lobos sempre mentem. A diz que B um co. B diz que C um lobo. C diz que D um lobo. D diz que B e E so animais de espcies diferentes. E diz que A um co. Quantos lobos h entre os cinco animais? A) 1 B) 2 C) 3 D) 4 E) 5 20. Com azulejos quadrados brancos e pretos todos do mesmo tamanho, construmos os seguintes mosaicos.

A regra para se construir estes mosaicos a seguinte: inicialmente formamos um quadrado com 1 azulejo branco cercado por azulejos pretos; e em seguida, outro quadrado, este com 4 azulejos brancos, tambm cercado por azulejos pretos; e assim sucessivamente. Com 80 azulejos pretos, quantos azulejos brancos sero necessrios para se fazer uma seqncia de mosaicos como esta?
A) 55 B) 65 C) 75 D) 85 E) 100

PROBLEMAS NVEL 2
1. Veja o problema 4 do Nvel 1.

EUREKA! N13, 2002

Sociedade Brasileira de Matemtica

2. O tringulo CDE pode ser obtido pela rotao do tringulo ABC de 90o no sentido anti-horrio ao redor de C, conforme mostrado no desenho abaixo. Podemos afirmar que igual a:
B D 60
O

40

A) 75o 3. 5. 7. 9.

B) 65o

C) 70o

D) 45o

E) 55o

Veja o problema 5 do Nvel 1. Veja o problema 8 do Nvel 1. Veja o problema 11 do Nvel 1. Veja o problema 14 do Nvel 1.

4. Veja o problema 6 do Nvel 1. 6. Veja o problema 9 do Nvel 1. 8. Veja o problema 12 do Nvel 1. 10. Veja o problema 15 do Nvel 1.

11. Os pontos P1, P2, P3, esto nesta ordem sobre uma circunferncia e so tais que o arco que une cada ponto ao seguinte mede 35. O menor valor de n > 1 tal que Pn coincide com P1 : A) 37 B) 73 C) 109 D) 141 E) 361 12. Veja o problema 16 do Nvel 1. 13. ABCDE um pentgono regular e ABF um tringulo equiltero interior. O ngulo FCD mede: A) 38 B) 40 C) 42 D) 44 E) 46 14. Veja o problema 19 do Nvel 1. 15. Um crculo dividido, por 2n + 1 raios, em 2n + 1 setores congruentes. Qual o nmero mximo de regies do crculo determinadas por estes raios e por uma reta? A) 3n B) 3n + 1 C) 3n + 2 D) 3n + 3 E) 4n 16. Paulo e Cezar tm algum dinheiro. Paulo d a Cezar R$5,00 e, em seguida, Cezar d a Paulo
1 do que possui. Assim, ambos ficam com R$18,00. A 3

diferena entre as quantias que cada um tinha inicialmente : A) R$7,00 B) R$8,00 C) R$9,00 D) R$10,00
EUREKA! N13, 2002

E) R$11,00

Sociedade Brasileira de Matemtica

17. Um fazendeiro tinha 24 vacas e rao para aliment-las por 60 dias. Entretanto, 10 dias depois, ele comprou mais 6 vacas e 10 dias depois dessa compra ele vendeu 20 vacas. Por mais quantos dias aps esta ltima compra ele pode alimentar o gado com a rao restante? A) 50 B) 60 C) 70 D) 80 E) 90 18. Veja o problema 18 do Nvel 1. 19. Uma mesa retangular, cujos ps tm rodas, deve ser empurrada por um corredor de largura constante, que forma um ngulo reto.
b a

Se as dimenses da mesa so a e b (com 2a < b), qual deve ser a largura mnima do corredor para que a mesa possa ser empurrada atravs dele? A) a + b B) (a + b)
2 2

C) (a + b)

2 4

D) (2a + b)

2 4

E) (a + 2b)

2 4

20. Somente uma das figuras a seguir representa a planificao de um cubo na qual est destacada a sua interseo com um plano. Qual? A) B) C) D) E)

21. Quantos dgitos tem o menor quadrado perfeito cujos quatro ltimos dgitos so 2001? A) 9 B) 5 C) 6 D) 7 E) 8 22. Papa-Lguas participou de uma corrida (junto com o Ligeirinho e o Flash), que consistia em dar 100 voltas em um circuito. Como sempre, o Coiote queria pegar o Papa-Lguas e colocou um monte de alpiste no meio da pista. claro que o Coiote no conseguiu pegar o Papa-Lguas, mas ele fez com que a velocidade mdia dele na primeira volta fosse de apenas 200 km/h. Sabendo disso, a velocidade mdia do Papa-Lguas na corrida: A) No ultrapassa 200 km/h. B) No ultrapassa 250 km/h, mas pode ultrapassar 200km/h.
EUREKA! N13, 2002

Sociedade Brasileira de Matemtica

C) No ultrapassa 2000 km/h, mas pode ultrapassar 250km/h. D) No ultrapassa 20000 km/h, mas pode ultrapassar os 2000km/h. E) Pode ultrapassar 20000 km/h. 23. Veja o problema 20 do Nvel 1. 24. Veja o problema 19 do Nvel 1. 25. O hexgono ABCDEF circunscritvel. Se AB = 1, BC = 2, CD = 3, DE = 4 e EF = 5, quanto mede FA?
C 3 D 4 E 5 2 B 1 A ? F

A) 1

B) 3

C) 15/8

D) 6

E) 9

PROBLEMAS NVEL 3
1. 3. 5. 7. 9. 11.
Veja o problema 4 do Nvel 1. Veja o problema 5 do Nvel 1. Veja o problema 8 do Nvel 1. Veja o problema 15 do Nvel 1. Veja o problema 13 do Nvel 2. Veja o problema 22 do Nvel 2.

2. Veja o problema 2 do Nvel 2. 4. Veja o problema 6 do Nvel 1. 6. Veja o problema 9 do Nvel 1. 8. Veja o problema 11 do Nvel 2. 10. Veja o problema 15 do Nvel 2.
2

12. O nmero de solues inteiras distintas da equao (6 x 2 + 12x 2) x 2 x+2 = 4 : A) 0 B) 1 C) 2 D) 3 E) 4 13. Uma rifa foi organizada entre os 30 alunos da turma do Pedro. Para tal, 30 bolinhas numeradas de 1 a 30 foram colocadas em uma urna. Uma delas foi, ento, retirada da urna. No entanto, a bola caiu no cho e se perdeu e uma segunda bola teve que ser sorteada entre as 29 restantes. Qual a probabilidade de que o nmero de Pedro tenha sido o sorteado desta segunda vez? A) 1/29 B) 1/30 C) 1/31 D) 1/60 E) 2/31 14. Cinco animais A, B, C, D, e E, so ces ou so lobos. Ces sempre contam a verdade e lobos sempre mentem. A diz que B um co. B diz que C um lobo. C
EUREKA! N13, 2002

Sociedade Brasileira de Matemtica

diz que D um lobo. D diz que B e E so animais de espcies diferentes. E diz que A um co. Quantos lobos h entre os cinco animais? A) 1 B) 2 C) 3 D) 4 E) 5 15. So escritos todos os nmeros de 1 a 999 nos quais o algarismo 1 aparece exatamente 2 vezes (tais como, 11, 121, 411, etc). A soma de todos estes nmeros : A) 6882 B) 5994 C) 4668 D) 7224 E) 3448 16. Veja o problema 19 do Nvel 2. 17. Veja o problema 20 do Nvel 2. 18. Seja f(x) = x2 3x + 4. Quantas solues reais tem a equao f(f(f(...f(x)))) = 2 (onde f aplicada 2001 vezes)? A) 0 B) 1 C) 2 D) 2001 E) 22001 19. Veja o problema 21 do Nvel 2.

e B .MeN 20. Seja ABCD um trapzio retngulo cujos nicos ngulos retos so A so os pontos mdios de AB e CD, respectivamente. A respeito dos B e = CM D , podemos dizer que: ngulos = AN A) < B) > C) = D) pode ocorrer qualquer uma das situaes das alternativas A), B) e C). E) o ngulo reto
21. A soma dos valores reais de x tais que x2 + x + 1 = 156/(x2 + x) : A) 13 B) 6 C) 1 D) 2 E) 6

22. Para cada ponto pertencente ao interior e aos lados de um tringulo acutngulo ABC, considere a soma de suas distncias aos trs lados do tringulo. O valor mximo desta soma igual A) mdia aritmtica das 3 alturas do tringulo. B) ao maior lado do tringulo. C) maior altura do tringulo D) ao triplo do raio do crculo inscrito no tringulo. E) ao dimetro do crculo circunscrito ao tringulo.

EUREKA! N13, 2002

Sociedade Brasileira de Matemtica

23. Seja f uma funo de Z em Z definida como f(x) = x/10 se x divisvel por 10 e f(x) = x + 1 caso contrrio. Se a0 = 2001 e an+1 = f(an), qual o menor valor de n para o qual an = 1? A) 20 B) 38 C) 93 D) 2000 E) an nunca igual a 1 24. Veja o problema 25 do Nvel 2. 25. No tringulo ABC, AB = 5 e BC = 6. Qual a rea do tringulo ABC, sabendo

tem a maior medida possvel? que o ngulo C


A) 15 E) 5 11 / 2 B) 5 7 C) 7 7 / 2 D) 3 11

GABARITO
NVEL 1 (5a. e 6a. sries)
1) E 2) D 3) C 4) B 5) D 6) C 7) A 8) B 9) B 10) D 11) A 12) D 13) B 14) A 15) C 16) D 17) E 18) A 19) D 20) A

NVEL 2 (7a. e 8a. sries)


1) B 2) E 3) D 4) C 5) B 6) B 7) A 8) D 9) Anulada 10) C 11) B 12) D 13) C 14) E 15) D 16) B 17) E 18) A 19) D 20) B 21) B 22) D 23) A 24) D 25) B

NVEL 3 (Ensino Mdio)


1) B 2) E 3) D 4) C 5) B 6) B 7) C 8) B 9) C 10) D 11) D 12) D 13) B 14) D 15) A 16) D 17) B 18) C 19) B 20) A 21) C 22) C 23) B 24) B 25) E

EUREKA! N13, 2002

10

Sociedade Brasileira de Matemtica

XXIII OLIMPADA BRASILEIRA DE MATEMTICA


Problemas e Solues da Segunda Fase PROBLEMAS NVEL 1
PROBLEMA 1

O jogo de domin formado por 28 peas retangulares distintas, cada uma com duas partes, com cada parte contendo de 0 a 6 pontinhos. Por exemplo, veja trs dessas peas:

Qual o nmero total de pontinhos de todas as peas?


PROBLEMA 2

As peas de um jogo chamado Tangram so construdas cortando-se um quadrado em sete partes, como mostra o desenho: dois tringulos retngulos grandes, um tringulo retngulo mdio, dois tringulos retngulos pequenos, um quadrado e um paralelogramo. Se a rea do quadrado grande 1, qual a rea do paralelogramo?

PROBLEMA 3

Carlinhos faz um furo numa folha de papel retangular. Dobra a folha ao meio e fura o papel dobrado; em seguida, dobra e fura novamente o papel dobrado. Ele pode repetir esse procedimento quantas vezes quiser, evitando furar onde j havia furos. Ao desdobrar a folha, ele conta o nmero total de furos feitos. No mnimo, quantas dobras dever fazer para obter mais de 100 furos na folha?
PROBLEMA 4

Os pontos da rede quadriculada abaixo so numerados a partir do vrtice inferior esquerdo seguindo o caminho poligonal sugerido no desenho. Considere o ponto correspondente ao nmero 2001. Quais so os nmeros dos pontos situados imediatamente abaixo e imediatamente esquerda dele?

EUREKA! N13, 2002

11

Sociedade Brasileira de Matemtica

13 5 4 6 3 7 8 12 11

10

PROBLEMA 5

Apresente todos os nmeros inteiros positivos menores do que 1000 que tm exatamente trs divisores positivos. Por exemplo: o nmero 4 tem exatamente trs divisores positivos: 1, 2 e 4.
PROBLEMA 6

Seja N o nmero inteiro positivo dado por N = 12 + 22 + 32 + 42 ++ (196883)2 . Qual o algarismo das unidades de N ?

PROBLEMAS NVEL 2
PROBLEMA 1: Veja o problema 2 do Nvel 1. PROBLEMA 2: Veja o problema 4 do Nvel 1. PROBLEMA 3

Se a n-sima OBM realizada em um ano que divisvel por n, dizemos que esse ano super-olmpico. Por exemplo, o ano 2001, em que est sendo realizada a 23a OBM, super-olmpico pois 2001 = 87 23 divisvel por 23. Determine todos os anos super-olmpicos, sabendo que a OBM nunca deixou de ser realizada desde sua primeira edio, em 1979, e supondo que continuar sendo realizada todo ano.
PROBLEMA 4

<B . < 90 , < C As medidas dos ngulos do tringulo ABC so tais que A

As

cortam os prolongamentos dos lados e C bissetrizes externas dos ngulos A


opostos BC e AB nos pontos P e Q, respectivamente. AP = CQ = AC , determine os ngulos de ABC.
PROBLEMA 5

Sabendo que

Dizemos que um conjunto A formado por 4 algarismos distintos e no nulos intercambivel se podemos formar dois pares de nmeros, cada um com 2
EUREKA! N13, 2002

12

Sociedade Brasileira de Matemtica

algarismos de A, de modo que o produto dos nmeros de cada par seja o mesmo e que, em cada par, todos os dgitos de A sejam utilizados. Por exemplo, o conjunto {1;2;3;6} intercambivel pois 21 36 = 12 63. Determine todos os conjuntos intercambiveis.
PROBLEMA 6

O matemtico excntrico Jones, especialista em Teoria dos Ns, tem uma bota com 5 pares de furos pelos quais o cadaro deve passar. Para no se aborrecer, ele gosta de diversificar as maneiras de passar o cadaro pelos furos, obedecendo sempre s seguintes regras: o cadaro deve formar um padro simtrico em relao ao eixo vertical; o cadaro deve passar exatamente uma vez por cada furo, sendo indiferente se ele o faz por cima ou por baixo; o cadaro deve comear e terminar nos dois furos superiores e deve ligar diretamente (isto , sem passar por outros furos) os dois furos inferiores. Representamos a seguir algumas possibilidades.

Qual o nmero total de possibilidades que o matemtico tem para amarrar seu cadaro, obedecendo s regras acima? Observao: Maneiras como as exibidas a seguir devem ser consideradas iguais (isto , deve ser levada em conta apenas a ordem na qual o cadaro passa pelos furos).

PROBLEMAS NVEL 3
PROBLEMA 1: Veja o problema 3 do Nvel 2. PROBLEMA 2

No tringulo ABC, a mediana e a altura relativas ao vrtice A dividem o ngulo BC em trs ngulos de mesma medida. Determine as medidas dos ngulos do tringulo ABC.
EUREKA! N13, 2002

13

Sociedade Brasileira de Matemtica

PROBLEMA 3

Determine todas as funes f: # # tais que f(x) = f(x) e f(x + y) = f(x) + f(y) + 8xy + 115 para todos os reais x e y.
PROBLEMA 4: Veja o problema 5 do Nvel 2. PROBLEMA 5

O matemtico excntrico Jones, especialista em Teoria dos Ns, tem uma bota com n pares de furos pelos quais o cadaro deve passar. Para no se aborrecer, ele gosta de diversificar as maneiras de passar o cadaro pelos furos, obedecendo sempre s seguintes regras: o cadaro deve formar um padro simtrico em relao ao eixo vertical; o cadaro deve passar exatamente uma vez por cada furo, sendo indiferente se ele o faz por cima ou por baixo; o cadaro deve comear e terminar nos dois furos superiores e deve ligar diretamente (isto , sem passar por outros furos) os dois furos inferiores. Por exemplo, para n = 4, representamos a seguir algumas possibilidades.

Determine, em funo de n 2, o nmero total de maneiras de passar o cadaro pelos furos obedecendo s regras acima. Observao: Maneiras como as exibidas a seguir devem ser consideradas iguais.

PROBLEMA 6

Seja f(x) =

x2 . Calcule 1+ x2 1 2 3 n f + f + f + ... + f 1 1 1 1 1 2 3 n + f + f + f + ... + f 2 2 2 2

EUREKA! N13, 2002

14

Sociedade Brasileira de Matemtica

+ f + f + f + ... + f + ... + f + f + f + ... + f , sendo n inteiro positivo.

1 3

2 3

3 3

n 3

1 n

2 n

3 n

n n

SOLUES NVEL 1
SOLUO DO PROBLEMA 1:

Cada tipo de pontuao aparece 8 vezes dentre as 28 peas do domin. Portanto o nmero total de pontos : 8.(0 + 1 + 2 + 3 + 4 + 5 + 6) = 168.
SOLUO DO PROBLEMA 2:

Traando a menor diagonal do paralelogramo, observamos que metade do mesmo equivale a um tringulo retngulo pequeno, cuja rea da rea do tringulo retngulo grande, que, por sua vez, da rea do quadrado. Logo a rea do paralelogramo igual a 2 1/16 = 1/8.
SOLUO DO PROBLEMA 3:

Ao furar aps a primeira dobra, Carlinhos faz 2 furos; aps a segunda dobra, faz 4 furos, aps a terceira dobra, faz 8 furos, etc. Assim, ao desdobrar a folha, ele ir furos. Notando que: contar 1 + 2 + 4 + 8 + (aps a primeira dobra) 1 + 2 = 22 1 3 (aps a segunda dobra) 1+2+4=2 1 (aps a terceira dobra), etc 1 + 2 + 4 + 8 = 24 1

Basta encontrar o menor k tal que 2k 1 maior ou igual a 100 2 k 1 100 k 7 Assim, o menor k vale 7. Isso corresponde a 6 dobras.
SOLUO DO PROBLEMA 4:

Os pontos correspondentes aos quadrados perfeitos pares e mpares esto sobre os lados vertical e horizontal do quadriculado, respectivamente. Os quadrados perfeitos mais prximos de 2001 so 1936 = 442 e 2025 = 452. Como 2001 est mais prximo de 2025, o ponto correspondente est no segmento vertical descendente que termina em 2025. Logo o ponto imediatamente abaixo dele corresponde ao nmero 2002. Para achar o nmero do ponto imediatamente esquerda, consideramos o
EUREKA! N13, 2002

15

Sociedade Brasileira de Matemtica

quadrado perfeito mpar anterior, que 432 = 1849. O ponto desejado est no segmento ascendente que comea em 1850 e situado mesma distncia que o ponto 2001 est de 2025. Logo o nmero correspondente : 1850 + (20252001) = 1850 + 24 = 1874.

1874 24

2001 24

1849

1850

2025

SOLUO DO PROBLEMA 5:

Sabemos que todos os nmeros inteiros maiores do que 1 admitem pelo menos um divisor (ou fator) primo. Dessa forma, se n tem dois divisores primos p e q ento 1, p, q e pq so divisores de n; logo n tem mais que trs divisores; se n primo, ento tem somente dois divisores: 1 e n; se n uma potncia de um primo p, ou seja, da forma ps, ento 1, p, p2, ..., ps so os divisores positivos de n. Para que n tenha trs divisores s dever ser igual a 2, isto , n = p2. Assim, os inteiros menores que 1000 com trs divisores so: 4, 9, 25, 49, 121, 169, 289, 361, 529, 841, 961.
SOLUO DO PROBLEMA 6:

Os algarismos das unidades dos quadrados dos nmeros de 1 a 10 so, respectivamente, 1, 4, 9, 6, 5, 6, 9, 4, 1 e 0. Ora, a soma dos nmeros formados por esses algarismos 45. Portanto, a soma 12 + 22 + 32 + 42 + + 102 tem como algarismo das unidades o nmero 5. De 11 a 20, os algarismos das unidades dos nmeros se repetem na mesma ordem; portanto, o algarismo das unidades da soma de seus quadrados tambm 5. Conseqentemente, a soma dos quadrados dos nmeros de 1 a 20 tem 0 como algarismo das unidades. Logo a soma 12 + 22 + 32 + 42 + ... + n2 tem zero como algarismo das unidades se N mltiplo de 20. Como N = 12 + 22 + 32 + 42 + ... + 1968832 = 12 + 22 + 32 + 42 + + 1968802 + 1968812 + 1968822 + 1968832, conclumos que o algarismo das unidades de N o mesmo do nmero 0 + 1 + 4 + 9 = 14, ou seja, 4.

EUREKA! N13, 2002

16

Sociedade Brasileira de Matemtica

SOLUES NVEL 2
SOLUO DO PROBLEMA 1: Veja a soluo do problema 2 do Nvel 1. SOLUO DO PROBLEMA 2: Veja a soluo do problema 4 do Nvel 1. SOLUO DO PROBLEMA 3:

Observando que no ano n realizada a (n 1978)-sima OBM, temos que o ano n super-olmpico se, e somente se, n 1978 divide n. Assim, n 1978 divide n (n 1978) = 1978. Como os divisores positivos de 1978 so 1, 2, 23, 43, 46, 86, 989 e 1978, os anos super-olmpicos so 1979, 1980, 2001, 2021, 2024, 2064, 2967 e 3956.
SOLUO DO PROBLEMA 4:

C = A Q = AQ Os tringulos ACQ e PAC so issceles. No tringulo ACQ, temos: CA Q= C + (180 C )/2 = 90 + C /2 AC /2) = 180 + (90 + C Logo 2A No tringulo PAC, temos: P = (180 A )/2 CA P = AP C = 180 C AC ) = 180 )/2 + 2(180 C Logo (180 A (1)

(2) = 132; Resolvendo o sistema formado pelas equaes (1) e (2), obtemos A = 12 e C = 180 12 132 = 36. da, B
Q B

90 , +
A 180 , A 180 , C 2

C 2
C

EUREKA! N13, 2002

17

Sociedade Brasileira de Matemtica

SOLUO DO PROBLEMA 5:

Seja A = {x ;y ;t ;z} um conjunto intercambivel. Ento podemos supor, sem perda de generalidade, que (10x + y)(10t + z) = (10y + x)(10z + t) xt = yz (1) Por (1), temos que 5 e 7 no podem aparecer em A. Se o maior dos elementos de A fosse menor ou igual a 4, teramos A = {1;2;3;4}, que no intercambivel. Logo A possui pelo menos um dos dgitos 6, 8 ou 9. Se o maior elemento de A 9, temos por (1) que 3 e 6 tambm pertencem a A. Neste caso temos o conjunto intercambivel A = {2;3;6;9}. Se o maior elemento de A 8, temos que 4 e outro algarismo par esto em A. Assim, temos A = {1;2;4;8} ou A = {3;4;6;8}. Se o maior elemento de A 6, temos que 3 e outro algarismo par esto em A. Desta forma, A = {1;2;3;6} ou A = {2;3;4;6}. Assim, temos no total 5 conjuntos intercambiveis: {2;3;6;9}, {1;2;4;8}, {3;4;6;8}, {1;2;3;6} e {2;3;4;6}. Obs. O enunciado no deixaria claro que as outras possibilidades, por exemplo: (10 x + y ) (10t + z ) = (10 x + z ) (10 y + t ) , no deveriam ser consideradas. A anlise dessas possibilidades torna o problema bem mais complicado, porm no acrescenta novos conjuntos intercambiveis aos listados acima.
SOLUO DO PROBLEMA 6:

Como o padro deve ser simtrico, basta decidir os primeiros 5 furos pelos quais o cadaro deve passar. A partir da, os furos ficam determinados pela simetria. Por exemplo, o 7 furo deve ser o outro furo da mesma linha visitada no 4 furo. Note, ainda, que a simetria implica em que as linhas visitadas nos 5 primeiros furos so todas distintas. Alm disso, a primeira destas linhas obrigatoriamente a de cima e a 5 obrigatoriamente a de baixo, j que os furos da linha de baixo so visitados consecutivamente. Assim, para obter um padro para o cadaro, podemos iniciar pelo furo da esquerda da linha superior e devemos decidir: em que ordem as 3 linhas intermedirias so visitadas de que lado queremos passar nestas 3 linhas e na linha de baixo. Para escolher a ordem das 3 linhas, observamos que a primeira pode ser escolhida de 3 modos; a seguir, a segunda pode ser escolhida de 2 modos, ficando a terceira determinada. Logo h 6 possibilidades de escolha para a ordem das linhas. Para escolher o lado por onde passar nas 4 linhas, temos duas opes para cada uma delas, para um total de 2 2 2 2 = 16 possibilidades. Logo o nmero total de modos de amarrar o cadaro 6 16 = 96.

EUREKA! N13, 2002

18

Sociedade Brasileira de Matemtica

Outra soluo: Comeando do lado esquerdo da linha superior, o segundo furo pode ser escolhido de 6 modos (qualquer um das linhas intermedirias); o terceiro de 4 modos (nas duas intermedirias restantes) e o quarto e quinto de 2 modos cada (suas linhas esto determinadas, bastando escolher o lado). Logo h um total de 6 4 2 2 = 96 possibilidades. SOLUES NVEL 3
SOLUO DO PROBLEMA 1: Veja a soluo do problema 3 do nvel 2. SOLUO DO PROBLEMA 2:
A

Seja M o ponto mdio de BC e H o p da altura relativa a A. Temos que AH comum B AH M (retos) e HM HB, logo, pelo caso aos tringulos AHM e AHB, AH ALA, os tringulos AHM e AHB so congruentes. Assim, BH = HM = MC/2, pois MC = MB. Como AM bissetriz de HC, pelo teorema das bissetrizes AH/AC = HM/MC AH/AC = 1/2 cos2 = 1/2. Como 0 < 2  o, 2 = 60o = 30o. C ) = 90o  Portanto os ngulos do tringulo ABC so m(BC) = 3 = 90o, m( AB

B ) = 90o 2   o. 60o e m( AC
SOLUO DO PROBLEMA 3:

Fazendo y = x, temos f(x + (x)) = f(x) + f(x) + 8x(x) + 115 f(0) = 2f(x) 8x2 + 115 f(x) = 4x2 + (f(0) 115)/2. Fazendo x = 0 nesta ltima igualdade, temos f(0) = 4 02 + (f(0) 115)/2 f(0) = 115. Logo f(x) = 4x2 + (f(0) 115)/2 f(x) = 4x2 115 e verificamos de fato que esta funo satisfaz as condies do enunciado: f(x) = 4(x)2 115 = 4x2 115 = f(x) e f(x) + f(y) + 8xy + 115 = 4x2 115 + 4y2 115 + 8xy + 115 = 4(x + y)2 115 = f(x + y). Assim, f(x) = 4x2 115 a nica funo que satisfaz todas as condies do enunciado.
SOLUO DO PROBLEMA 4: Veja a soluo do problema 5 do Nvel 2. SOLUO DO PROBLEMA 5:

Numere os furos superiores com o nmero 1, os furos imediatamente abaixo com o nmero 2 e assim por diante, at os furos inferiores, que recebem o nmero n.
EUREKA! N13, 2002

19

Sociedade Brasileira de Matemtica

Observe que basta estabelecermos os primeiros n furos onde o cadaro ir passar (o padro simtrico). Uma maneira pode ser definida por uma seqncia indicando os nmeros dos primeiros n furos onde o lao passa (observe que tal seqncia tem todos os nmeros de 1 a n, comea com 1 e termina com n) e por uma outra seqncia de comprimento n 1 cujo k-simo termo indica se o cadaro muda de lado ao passarmos do k-simo para o (k + 1)-simo termo da primeira seqncia. Por exemplo, (1, 3, 2, 4) e (muda, no muda, muda) representa
1 2 3 4 1 2 3 4

Assim, como h (n 2)! seqncias com os nmeros de 1 a n comeando com 1 e terminando com n e 2n 1 seqncias indicando se o cadaro muda de lado ou no, h (n 2)! 2n 1 maneiras.
SOLUO DO PROBLEMA 6:

Seja S a soma pedida. Como f(x) + f(1/x) = escrever

(1 / x) 2 x2 + = 1, podemos 1 + x 2 1 + (1 / x) 2
n  + f 1 n  + f 2

1 2 3 2S = f + f + f + 1 1 1 1 2 3 + f + f + f + 2 2 2 +

1 2 3 n + f + f + f + + f n n n n 1 2 3 n + f + f + f + + f 1 1 1 1 1 2 3 n + f + f + f + + f 2 2 2 2 + 1 2 3 n + f + f + f + + f n n n n 1 1 1 2 3 2S = f 1 + f 1 f + f 1 f + + + 2 1 + n (n  + f n + f n n n2 2
2

  

1 f 3

pares de parcelas)

2S = n 2 S=

EUREKA! N13, 2002

20

Sociedade Brasileira de Matemtica

XXIII OLIMPADA BRASILEIRA DE MATEMTICA


Problemas e Solues da Terceira Fase PROBLEMAS NVEL 1
PROBLEMA 1:

Numa famosa joalheria esto armazenadas vrias pedras preciosas dos seguintes tipos: esmeraldas; safiras e rubis. Todas as pedras do mesmo tipo tm o mesmo valor. Alm disso, 24 esmeraldas valem tanto quanto 12 rubis e tambm valem tanto quanto 8 safiras. Com R$350.000,00 um prncipe comprou um conjunto com 4 esmeraldas, 6 rubis e 4 safiras. Quanto custa cada tipo de pedra?
PROBLEMA 2:

Um cubinho foi colocado no canto de uma sala, conforme a Figura 1. Empilharam-se outros cubinhos iguais ao primeiro, de forma a cobrir as faces visveis do mesmo, usando-se o menor nmero possvel de peas. Como se pode ver na Figura 2, aps a colocao dos novos cubinhos, restam 9 faces visveis desses cubinhos.

Figura 1

Figura 2

a) Quantos cubinhos iguais a esses, no mnimo, seria necessrio empilhar, de forma


a cobrir aquelas 9 faces visveis?

b) Continua-se a fazer essa pilha, repetindo-se o procedimento descrito. Quando a


pilha tiver um total de 56 cubinhos, quantas faces podero ser vistas?
PROBLEMA 3:

No tringulo ABC tem-se que M o ponto mdio do lado AB (isto , os segmentos AM e MB tm o mesmo comprimento). N o ponto mdio de MC e R o ponto mdio de NA. O tringulo ABC tem rea 2000. Determine a rea do tringulo AMR.
C

N R A M B

EUREKA! N13, 2002

21

Sociedade Brasileira de Matemtica

PROBLEMA 4:

Dizemos que um nmero natural legal quando for soma de dois naturais consecutivos e tambm for soma de trs naturais consecutivos. a) Mostre que 2001 legal, mas 1999 e 2002 no so legais. b) Mostre que 20012001 legal.
PROBLEMA 5:

As 42 crianas de uma escola infantil deram as mos formando uma fila e cada uma delas recebeu um nmero da seguinte maneira: a primeira delas ficou com o nmero 1, a segunda ficou com o nmero 2 e, assim sucessivamente, at a ltima, que ficou com o nmero 42. Continuando de mos dadas, foram para um ptio, onde cada uma delas ficou sobre uma lajota quadrada; duas crianas com nmeros consecutivos ficaram em lajotas vizinhas com um lado comum (ou seja, do lado esquerdo, do lado direito, na frente ou atrs, mas nunca em diagonal). Ao relatar esse fato para a diretora, a inspetora Maria fez o desenho esquerda, mostrando a posio de trs crianas sobre o retngulo formado pelas 42 lajotas, sobre as quais estavam as crianas. Num outro comunicado, a inspetora Clia fez outro desenho, mostrado direita, com a posio das mesmas crianas sobre o mesmo retngulo. Ao receber os dois desenhos a diretora disse a uma das inspetoras: "O seu desenho est errado". i) ii) Com qual das duas inspetoras a diretora falou? Qual foi o raciocnio da diretora? Complete o desenho correto satisfazendo as condies do enunciado.
11 20 11 20

31 31

(Desenho de Maria)

(Desenho de Clia)

EUREKA! N13, 2002

22

Sociedade Brasileira de Matemtica

PROBLEMAS NVEL 2
PROBLEMA 1:

Uma folha de papel retangular ABCD, de rea 1, dobrada em sua diagonal AC e, em seguida, desdobrada; depois dobrada de forma que o vrtice A coincida com o vrtice C e, em seguida, desdobrada, deixando o vinco MN, conforme desenho abaixo.
A M B

a) Mostre que o quadriltero AMCN um losango. b) Se a diagonal AC o dobro da largura AD, qual a rea do losango AMCN?
PROBLEMA 2: Veja o problema 5 do Nvel 2. PROBLEMA 3:

Dado um inteiro positivo h demonstre que existe um nmero finito de tringulos de lados inteiros a, b, c e altura relativa ao lado c igual a h .
a h . c b

PROBLEMA 4:

Mostre que no existem dois nmeros inteiros a e b tais que (a + b) (a2 + b2) = 2001.
PROBLEMA 5:

Sejam a, b e c nmeros reais no nulos tais que a + b + c = 0. (a 3 + b 3 + c 3 ) 2 (a 4 + b 4 + c 4 ) . Calcule os possveis valores de (a 5 + b 5 + c 5 ) 2


PROBLEMA 6:

Em um quadriltero convexo, a altura em relao a um lado definida como a perpendicular a esse lado passando pelo ponto mdio do lado oposto. Prove que as quatro alturas tm um ponto comum se e somente se o quadriltero inscritvel, isto , se e somente se existe uma circunferncia que contm seus quatro vrtices.

EUREKA! N13, 2002

23

Sociedade Brasileira de Matemtica

PROBLEMAS NVEL 3
PROBLEMA 1:

Prove que (a + b)(a + c) 2 abc (a + b + c) para quaisquer nmeros reais positivos a, b e c.


PROBLEMA 2:

Dado um inteiro a 0 > 1 definimos uma seqncia (a n ) n 0 da seguinte forma; para cada k 0 , a k +1 o menor inteiro a k +1 > a k tal que mdc (a k +1 , a 0 a1 ... a k ) = 1. Diga para quais valores de a 0 temos que todos os termos ak da seqncia so primos ou potncias de primos.
PROBLEMA 3:

E e F so pontos do lado AB, do tringulo ABC, tais que AE = EF = FB. D ponto da reta BC tal que BC perpendicular a ED. AD perpendicular a CF. Os ngulos BDF e CFA medem x e 3x, respectivamente. Calcule a razo (DB) / (DC).
PROBLEMA 4:

Uma calculadora tem o nmero 1 na tela. Devemos efetuar 2001 operaes, cada uma das quais consistindo em pressionar a tecla sen ou a tecla cos. Essas operaes calculam respectivamente o seno e o cosseno com argumentos em radianos. Qual o maior resultado possvel depois das 2001 operaes?
PROBLEMA 5: Veja o problema 6 do Nvel 2. PROBLEMA 6:

Temos uma fileira longa de copos e n pedras no copo central (copo 0). Os seguintes movimentos so permitidos: Movimento tipo A

i1

i+1

i+2

i1 i i+1 i+2

Se h pelo menos uma pedra no copo i e pelo menos uma no copo i + 1 podemos fazer uma pedra que est no copo i + 1 pular para o copo i 1 eliminando uma pedra do copo i. Movimento tipo B.

EUREKA! N13, 2002

24

Sociedade Brasileira de Matemtica

i1

i+1

i+2

i1

i+1

i+2

Se h pelo menos duas pedras no copo i podemos pular uma para o copo i + 2 e uma outra para o copo i 1. Demonstre o seguinte fato: fazendo os movimentos tipo A ou B durante um tempo suficientemente longo sempre chegaremos a uma configurao a partir da qual no mais possvel fazer nenhum desses dois tipos de movimento. Alm disso essa configurao final no depende da escolha de movimentos durante o processo. SOLUES NVEL 1
PROBLEMA 1: SOLUO DE RAPHAEL RODRIGUES MATA (SALVADOR BA)

Se 24 esmeraldas equivalem a 12 rubis, significa que 1 rubi equivale a duas esmeraldas, e se 24 esmeraldas equivalem a 8 safiras, uma safira equivale a 3 esmeraldas. Assim, se o prncipe comprar 6 rubis, o mesmo que ele comprar 12 esmeraldas, e se ele comprar 4 safiras, o mesmo que ele comprar 12 esmeraldas. Assim, o conjunto comprado pelo prncipe tem o mesmo valor de 28 esmeraldas (4 + 12 + 12 = 28). 350000 Para se descobrir o valor de cada esmeralda, basta efetuar = 12500. 28 Sabemos que o rubi vale o dobro da esmeralda, assim, temos 12500 2 = 25000. Por fim, sendo a safira o triplo do valor da esmeralda, temos 12500 3 = 37500. Finalmente, descobrimos que a esmeralda custa R$12500,00; o rubi custa R$25000,00; e cada safira tem o valor de R$37500,00.
PROBLEMA 2: SOLUO DE EDUARDO FISCHER (ENCANTADO - RS)

a) So 6 cubos; chego a esta soluo apenas olhando. Na fileira de baixo se acrescentam 3 cubos, na do meio 2 e na de cima 1 (a de cima antes estava vazia). Note que primeiro foi botado um cubo, depois 3, que um mais dois, agora bota 6, que 1 + 2 + 3. Depois acrescentarei 10 (1 + 2 + 3 + 4) e depois 15 (1 + 2 + 3 + 4 + 5). Isso se deve ao fato que, ao quadricular o cho, na figura 1 se acrescenta um cubo ao nada. Depois se acrescenta 2 cubos (no cho) e 1 em cima. Aps termos 4 cubos, se acrescenta 3 (no cho), 2 para cobrir os 2 que antes esto no cho (os mais distantes da parede) e 1 para cobrir l em cima. E assim segue. b) Para chegar a 56, vou somando: 1, 3 = 1 + 2, 6 = 1 + 2 + 3, 10 = 1 + 2 + 3 + 4, 15 = 1 + 2 + 3 + 4 + 5 e 21 = 1 + 2 + 3 + 4 + 5 + 6. A parede ter a altura de 6 cubos, quando isso acontecer. Vamos listar as faces e cubos mostra:
EUREKA! N13, 2002

25

Sociedade Brasileira de Matemtica

No andar de cima h 1 cubo e 3 faces. No segundo andar h 6 faces e 2 cubos. No terceiro andar h 9 faces e 3 cubos. No quarto andar h 12 faces e 4 cubos. No quinto andar h 15 faces e 5 cubos. No andar de baixo h 18 faces e 6 cubos. Para cada cubo mostra, h 3 faces vistas. So 21 cubos mostra, 63 faces no total.
PROBLEMA 3: SOLUO DE FELIPE GONALVES ASSIS (CAMPINA GRANDE PB)

O tringulo ABC tem rea 2000. Ao ser cortado pelo segmento de reta MC, divide-se em outros 2 tringulos menores, de mesma rea, ACM e CMB. Pode-se perceber que eles tm reas iguais pois: A base de ambos tem o mesmo comprimento, pois AM = MB, j que M o ponto mdio de AB. A altura dos dois tambm a mesma. Duas medidas que determinam a rea de um tringulo so, justamente, base e altura. Assim descobrimos uma propriedade dos tringulos: Se um tringulo for cortado por um segmento de reta que parte do ponto mdio de um dos segmentos que o compem at o vrtice formado pelas outras duas retas, obter-se-o 2 novos tringulos, de mesma rea, correspondente a metade da rea do primeiro tringulo. Isto o que ocorre com o tringulo ABC que forma os tringulos ACM e CMB, cada um com rea 1000 (= 2000:2). Ocorre isto tambm com ACM, cortado pelo segmento AN, ele forma AMN e ACN, ambos com rea 500 (= 1000:2). Acontece o mesmo com AMN que cortado por RM, originando AMR e RMN, cada qual com rea 250(= 500:2). Resposta: A rea de AMR de 250.
PROBLEMA 4: SOLUO DE PAULO ANDR CARVALHO DE MELO (RIO DE JANEIRO RJ)

Para um nmero ser a soma de 2 naturais consecutivos ele tem que ser: x + x + 1 = 2x + 1. Ou seja, mpar. Para um nmero ser a soma de 3 naturais consecutivos ele tem que ser: x + x + 1 + x + 2 = 3x + 3. Ou seja, mltiplo de 3. Portanto um nmero legal aquele que mltiplo de 3 e mpar a) O 2001 mltiplo de 3 e mpar, mas o 2002 e o 1999 no so.
EUREKA! N13, 2002

26

Sociedade Brasileira de Matemtica

b) J que 2001 mltiplo de 3, 20012001 ter 32001 como um de seus fatores primos e ser mpar pois um nmero mpar multiplicado por outro nmero mpar igual a nmero mpar. Portanto 20012001 legal, j que respeita as condies para um nmero ser legal.
PROBLEMA 5: Veja a soluo do Problema 2 do Nvel 2.

SOLUES NVEL 2
PROBLEMA 1: a) SOLUO DE ELTON GOMES CORIOLANO (FORTALEZA CE)

Se fizermos a segunda dobradura, teremos a seguinte figura:


B C A M

Logo percebemos que AN = CN e que AM = CM . Temos, ento, dois tringulos N e AC N so issceles: o tringulo ANC e o tringulo AMC. Ento os ngulos CA M e AC M so congruentes tambm. Sabendo que AM congruentes e os ngulos CA M e AC N so congruentes, pois estes paralelo a NC, pode-se dizer que os ngulos CA N = AC N = CA M = AC M . Portanto, os so ngulos alternos internos. Assim, CA N e MC N so congruentes. Logo, AC N congruente a AM C . Assim, ngulos MA AMCN paralelogramo, pois seus ngulos opostos so congruentes. Por este motivo, os lados opostos tambm sero iguais, ou seja, AM = CN e AN = CM. Dessa forma, AM = CN = AN = CM. Logo, o quadriltero AMCN um paralelogramo com todos os lados iguais, ou seja, AMCN um losango.
A M B

b) SOLUO DE THIAGO COSTA LEITE SANTOS (SO PAULO SP)

Seja T o centro do retngulo. Como AT metade de AC , AT = AD , os tringulos ATN e ADN so retngulos, logo ATN ADN , pelo caso especial catetoEUREKA! N13, 2002

27

Sociedade Brasileira de Matemtica

hipotenusa (a hipotenusa comum aos dois tringulos). Analogamente, temos CBM CTM e portanto o retngulo ABCD est dividido em 6 tringulos congruentes. Portanto a rea de cada tringulo igual a
( ABCD ) 1 e como o = 6 6 1 2 losango AMCN possui 4 dos 6 tringulos, sua rea ser igual a 4 = . 6 3 PROBLEMA 2: SOLUO DE RAFAEL DAIGO HIRAMA (CAMPINAS SP)

i) Vamos pintar o retngulo igual a um tabuleiro de damas. Pelo enunciado o nmero anterior (e o posterior) de um nmero est acima, abaixo, a esquerda ou a direita. Portanto, um nmero tem sua cor diferente de seus dois vizinhos. Ento, se o primeiro no pintado, o segundo , o terceiro no , o quarto , etc. Ou seja, os nmeros pares tem uma cor e os impares outra. Pintando os dois tabuleiros percebemos que no de Clia o 20 e o 31 esto da mesma cor. Logo o desenho de Clia est errado.
11 20
11 20

31

31

Desenho da Maria ii) Para completar o desenho, vejamos que o 10 e o 12 devem estar no quadrado esquerda e no abaixo do 11 pois este no tem mais vizinhos. Igualmente o 19 e o 21 esto direita ou abaixo do 20. O nmero 12 deve estar abaixo do 11 pois, se no o 9 teria que ficar no lugar marcado com um crculo, e no haveria lugar para o 8 (pois o nico modo de conectar o 12 com o 20 seria como na figura).
12 13 14 15 11 10 9 16 20 19 18 17

Logo o 12 est abaixo do 11.

Vamos provar que o 19 est abaixo de 20. Se o 19 estivesse a direita de 20 travaria a sada para o 21 formando uma barreira sem sada. Logo o 19 est abaixo do 20. Com isso, podemos montar algo obrigatrio (tudo por falta de outras opes).

EUREKA! N13, 2002

28

Sociedade Brasileira de Matemtica


10 9 8 7 31 11 20 21

12 19 13

1) 2) 3) 4)

O 9 tem que estar abaixo do 10. O 13 abaixo do 12. O 8 abaixo do 9. O 7 abaixo do 8.

Temos que o 21 deve alcanar o 31 em 9 espaos, o 13 alcanar o 19 em 5 espaos e temos mais 6 espaos para fazer do 6 ao 1. Se colocarmos o 6 direita do 7 teramos problemas, pois o caminho entre o 21 e o 31 (o 30 no pode ficar embaixo pois seno isolaria o 31 de uma parte em branco pois o cordo 29 a 31 impediria e nenhuma parte do cordo pois seno tambm faria uma rea sem alcance).
10 9 8 7 11 20 21 22 23 24 25 26 30 29 28 27 12 19 13 6 31

Daria pouco espao para 11 espaos (14 18 e 1 6) (O 30 deve ficar na direita do 31 neste caso pois abaixo e esquerda ocorreria o espao sem alcance), 8 no mximo.

Ento o 6 abaixo do 7, o 5 abaixo do 6, o 4 direita do 5, o 3 direita do 4 (estes 3 por falta de opo) e o 2 direita do 3 (pois esquerda do 31 tem o 6 e abaixo tem o 4 sobrando os outros dois (acima e direita para o 30 e o 32). Logo, teremos:
10 9 8 7 6 5 31 4 11 20 21 22 27 26 29 2 22 23 24 25 28 23 24 25 26 27 12 19 13 28 29 30 3

10 9 8 7 6 5

11

20

21 18 15 28 33 2

22 17 16 27 34 1

23 24 25 26 35

12 19 13 30 31 4 14 29 32 3

Agora, o cordo 21 31 deve ter o 30 acima do 31 pois seno forma a mesma rea sem alcance como os exemplos. Ento o 32 fica direita do 31 e o 29 direita do 30, o 14 direita do 13, o 28 direita do 29, o 33 direita do 32, o 1 direita do 2, o 34 direita do 33, o 15 direita do 14, o 18 direita do 19, o 22, 17, 16, 27 direita do 21, 18, 15 e 28 respectivamente. O 23, 26 e 35 devem ficar direita do 22, 27 e 34, respectivamente. Como o 23 deve chegar ao 26 em 2 espaos, o 24 fica embaixo do 23 e o 25, embaixo do 24. Ento s completar com o 35 a 42 do nico modo possvel.

EUREKA! N13, 2002

29

Sociedade Brasileira de Matemtica

Respostas: i) A diretora falou com Clia. A direitora percebeu que o 20 e o 31 no poderiam estar na mesma cor se o tabuleiro fosse pintado como o de damas. ii)
10 9 8 7 6 5 11 20 21 18 15 22 17 16 23 24 25 26 35 36 42 41 40 39 38 37 12 19 13 14 30 29 31 32 4 3

28 27 33 34 2 1

Obs. Estendendo-se esse raciocnio possvel demonstrar que esta a nica maneira de se completar o desenho.
PROBLEMA 3: SOLUO DE THIAGO COSTA LEITE SANTOS (SO PAULO SP)

. c

Temos

a2 = h2 + m2 b2 = h2 + n2

Para uma equao Pitagrica: x2 = y2 + z2, resolvida em inteiros positivos, temos: z 2 2 y + 1 (pois seno y 2 < x 2 < ( y + 1) 2 ) y 2 2 z + 1 (anloga a de cima)
y2 1 z2 2y +1 y2 1 y 2 12 2 2 z z 2 2 z2 2y + 1 z2 2y +1
2

Mas se substituirmos y por h e z por m ou n, teremos que estes estaro limitados a certos valores, logo a, b, e c esto limitados a certos valores e, portanto acabou!!! Mas supomos que m e n so inteiros positivos e eles poderiam ser irracionais e a soma de dois irracionais dar um inteiro. Para isto podemos aplicar a lei dos cossenos.

EUREKA! N13, 2002

30

Sociedade Brasileira de Matemtica

cos =

n ; n = (b 2 h 2 ) b

a 2 = b 2 + c 2 2bc cos a 2 = b 2 + c 2 2c b 2 h 2 . Assim, se n irracional, a2 tambm ser, absurdo. Portanto m e n so inteiros.

Obs.: m e n no poderiam ser fracionrios, observando as equaes iniciais, que mostram que m2 e n2 so inteiros. Note que se n = p/q um racional no inteiro, ento n2 = p2/q2 tambm no ser inteiro, pois se q no divide p ento q2 no divide p2.
PROBLEMA 4: SOLUO DE HENRY WEI CHENG HSU (SO PAULO SP)

2001 = 3. 23 . 29 Temos que a2 + b2 sempre 0, ento, para o produto (a + b) (a2 + b2) valer 2001, a + b no pode ser negativo. Como a e b so inteiros, a2 a e b2 b, assim a2 +b2 a + b (a2 < a somente quando 0 < a < 1) Os valores possveis para (a + b) e (a2 + b2) so: 1) a + b = 1 e a2 + b2 = 2001 2) a + b = 3 e a2 + b2 = 667 3) a + b = 23 e a2 + b2 = 87 4) a + b = 29 e a2 + b2 = 69 Vamos analisar os casos 2) e 3). Em ambos temos a + b 3(mod 4) e a2 + b2 3 (mod 4) Para a + b 3 (mod 4), podemos ter os seguintes casos: a 0 e b 3 a 2 0 e b2 1 a 1 e b 2 a 2 1 e b2 0 a 2 e b 1 a 2 0 e b2 1 a 3 e b 0 a 2 1 e b2 0 Ento, quando a + b 3 (mod 4), a2 + b2 ser congruente a 1 (mod 4). Portanto os casos 2) e 3) no podem existir. Vamos analisar o caso 1) Para a = 1 e b = 0 e a = 0 e b = 1, a2 + b2 sempre valer 1. Quando um dos dois for negativo: a = n + 1 e b = n ou vice-versa a+b=n+1n=1 a 2 + b 2 = n 2 + 2 n + 1 + n 2 = 2 n 2 + 2n + 1 a2 + b2 = 2001 2n2 + 2n + 1 = 2001 2n2 + 2n 2000 = 0 n2 + n 1000 = 0 n2 + n 1000 = 0 = 1 + 4000 = 4001 como 4001 no quadrado perfeito, n no inteiro. Outra maneira: a + b = 1 a = 1 b a2 + b2 = b2 2b + 1 + b2 = 2b2 2b + 1
EUREKA! N13, 2002

31

Sociedade Brasileira de Matemtica

2b2 2b + 1 = 2001 2b2 2b 2000 = 0 b2 b 1000 = 0 = 1 + 4000 = 4001 (no quadrado perfeito). Assim, o caso 1) no existe. Vamos analisar o caso 4). a + b = 29 b = 29 a a2 + b2 = a2 + 841 58a + a2 = 2a2 58a + 841 a2 + b2 = 69 2a2 58a + 772 = 0 a2 29a + 386 = 0 = 841 1544 < 0 Como < 0, a no inteiro. Assim, o caso 4) no pode ocorrer. Como nenhum caso pode ocorrer, no existem dois nmeros a e b tais que (a + b) (a2 + b2) = 2001.
PROBLEMA 5: SOLUO DE MARCELA SOBRINHO PEREIRA (FORTALEZA CE)

Pelo enunciado, temos: a + b + c = 0 a = (b + c) a3 = b3 c3 3b2c 3bc2 = b3 c3 3bc(b + c), a4 = b4 + c4 + 4b3c + 4c3b + 6c2b2 e a5 = b5 c5 5b4c 5c4b 10b3c2 10c3b2. Substituindo na expresso, temos: ( 3bc(b + c))2 2b 4 + 2c 4 + 4b 3 c + 4bc 3 + 6c 2 b 2 = 2 5bc b 3 + c 3 + 2b 2 c + 2bc 2

( ( (
2

))

9b c (b + c) (2b + 2c + 4b c + 4bc 3 + 6c 2 b 2 ) = 25b 2 c 2 (b 3 + c 3 + 2b 2 c + 2bc 2 ) 2


2 2 4 4 3

9 2(b 6 + c 6 + 8b 2 c 4 + 4b 5 c + 10b 3 c 3 + 8b 4 c 2 + 4bc 5 ) 18 = . 25 b 6 + c 6 + 8b 2 c 4 + 4b 5 c + 10b 3 c 3 + 8b 4 c 2 + 4bc 5 25

PROBLEMA 6: Veja a soluo do problema 5 do nvel 3.

SOLUES NVEL 3
PROBLEMA 1: SOLUO DE CARLOS STEIN NAVES DE BRITO (GOINIA - GO)

Seja a + b = x, a + c = y e b + c = z. (x, y, z > 0) Temos um sistema linear de variveis a, b e c e x+ yz a + b = x a = 2 a + b = x a + b = x x+z y Resolvendo a + c = y b + c = y x b + c = y x b = 2 b + c = z b + c = z z+ yx c = 2


EUREKA! N13, 2002

32

Sociedade Brasileira de Matemtica

Chamando x + y + z = 2p. Temos a = p z, b = p y e c = p x. Logo a desigualdade vira: (a + b)(a + c) abc(a + b + c) p z + p y p z + p x 2 ( p z )( p y )( p x)( p x + p y + p z ) x y z

x y 2 ( p z )( p y )( p x) p . Basta provar isso. Temos que: ( a + b) + ( a + c ) > b + c (a + c) + (b + c) > ( a + b) 2a > 0 2c > 0 x+ y>z ;y+z>x a>0 c>0 (a + b) + (b + c) > (a + c) 2b > 0 e x+z> y . b>0

Assim x, y e z podem ser lados de um tringulo, sendo p o semipermetro. Assim seja o ngulo desse tringulo entre x e y. x ysen x y 1 x ysen ; como sen 1, temos Logo a rea A dele : A = =A 2 2 2 Mas A em funo dos lados logo ( p z )( p y )( p x) p , xy A = ( p z )( p y )( p x) p s xy 2 ( p z )( p y )( p x) p , cqd. 2
PROBLEMA 2: SOLUO DE DAVI MXIMO ALEXANDRINO NOGUEIRA (FORTALEZA - CE)

Notao: mdc (x, y) = (x, y). J que a0 faz parte de (an)n 0, o prprio deve ser potncia de primo. Suponha a0 = pm (p primo). Considere primeiro p = 2: a0 = 2m. a1 o menor inteiro que satisfaz a1 > a0 e (a1, a0) = 1. Portanto, temos a1 = a0 + 1 = 2m + 1. Se m for mpar, a1 = 2m + 1 0 (mod 3) 3|a1 a1 = 3n, pois todo termo de (an)n 0 deve ser potncia de primo. Logo 2m + 1 = 3n. Se m = 1, 3 = 3n n = 1, a1 = 3 e a0 = 2 (I) Se m 2, segue que 4|2m, e 2m +1 3n (mod 4) 1 (1)n (mod 4) n par, digamos n = 2n0. Logo, ficamos com:

EUREKA! N13, 2002

33

Sociedade Brasileira de Matemtica


n 3 0 1 = 2 2 = 2 2 2 = 2 (2 1) = 1 2m +1 = 32n0 2m = (3n0 1)(3n0 +1) n 0 3 +1 = 2

e = + 1 = 2 m = 3 e n = 2 a 0 = 8 a1 = 9 (II) Agora observemos: (I) a 0 = 2 a1 = 3 Ento, a 2 = 5 , a 3 = 7 , a 4 = 11. Provaremos por induo: ai = (i + 1)-simo primo Supondo a hiptese vlida para j, olhemos o passo indutivo j j + 1: a j +1 > a j ; (a j +1 , a 0 a1 ...a j ) = 1 . Seja p o (j + 2)-simo primo. Por hiptese, ai = (i + 1)-simo primo. Seja x {a j + 1, a j + 2,..., p 1} x > a j porm, ( x, a 0 a1 ...a j ) 1 pois os fatores primos de x pertencem ao produto a 0 a1 ...a j logo, a j +1 = p pois p o menor inteiro tal que p > aj e (p, a0aj) = 1 e o resultado segue. (II): a0 = 8 a1 = 9 a2 = 11, a3 = 13, a4 = 17, a5 = 19, a6 = 23, a7 = 25 = 52, a8 = 29, a9 = 31, a10 = 37, a11 = 41, a12 = 43, a13 = 47, a14 = 49 = 72, a15 = 53. Prova analogamente por induo ("mesmo" passo indutivo anterior) que a partir de i 15, ai = (i + 1)-simo primo (induo feita no anexo). Sendo assim, suponha a0 = 2m com m par (m > 0). Temos: a0 = 2m a1 = 2m + 1 a2 = 2m + 3 (pois, (a2, a1) = (a2, a0) = 1) a3 = 2m + 5. Porm, como m par, a3 2m + 5 (1)m + 5 0 (mod 3) 2m + 5 = 3n Resolvamos ento 2m + 5 = 3n. Como 2m = a0 > 1 m > 0. Mas como estamos supondo m par m 2 4 | 2 m . Logo, 2m + 5 = 3n 2m + 5 3n (mod 4) 3n 1 (mod 4) n par, digamos n = 2n0. Logo, 22m0 + 5 = 32n0 32n0 22m0 = 5 (3n0 2 m0 )(3n0 + 2 m0 ) = 5
n m 3 0 2 0 = 1 n (pois 3n0 2m0 < 3n0 + 2m0 ) m0 0 3 + 2 = 5

2 m0 +1 = 4 m0 = 1 m = n = 2 a 0 = 4
a1 = 5, a 2 = 7, a 3 = 9

Prova-se novamente por induo (mesmo passo indutivo da anterior) que para todo i 4, ai = (i + 1)-simo primo.
EUREKA! N13, 2002

34

Sociedade Brasileira de Matemtica

Agora, suponha a 0 = p m , p 2. a1 = p m + 1 = 2 n pois 2 | p m + 1. Se n for par, digamos n = 2n0, veja (mod 3): p m + 1 2 2 n0 (mod 3) p m + 1 1(mod 3) p = 3 sendo assim; resolvemos 3 m + 1 = 2 2 n0 n 2 0 + 1 = 3 3 m = (2 n0 1)(2 n0 + 1) n 0 2 1 = 3 3 3 = 2 2 = 3 (3 1) = 0, = 1 m = 1 e n = 2 a 0 = 3, a1 = 4 a 2 = 5 e para i 2 a i = (i + 1) -simo primo (pelo mesmo argumento indutivo). Sendo assim, suponha n mpar: a 0 = p m , a1 = p m + 1 = 2 n e a 2 = p m + 2 = 2 n + 1 (j que (pm, pm + 2) = (pm + 2, pm + 1) = 1) 3 | a 2 (j que n mpar) a 2 = 3 t logo,
3 t 2 n = 1 (*) mas j sabido nosso que as solues de (*) se do para: t = 1 e n = 1 a2 = 3 e a1 = 2 a0 = 1 (Absurdo!) t = 2 e n = 3 a2 = 9 e a1 = 8 a0 = 7 (III) (III): a0 = 7, a1 = 8, a2 = 9, a3 = 11, a4 = 13, a5 = 17, a6 = 19, a7 = 23, a8 = 25 = 52, a9 = 29 e, para i 9, ai = (i + 1)-simo primo (como antes). Resposta: a0 pode ser 2, 3, 4, 7 ou 8.

Anexo: Para um j > 1, suponha a0, a1, , aj definidos como disse e a hiptese valendo. Basta ver que no conjunto {a0, a1,,aj} aparecem todos os j + 1 primeiros fatores primos. Chame p = (j + 2)-simo primo. Os candidatos a aj + 1 antes de p seriam aj + 1, aj + 2,, p 1. Porm, os fatores primos de qualquer um desses caras aparecem no produto a0 a1,, aj se x { aj + 1,, p 1}, mdc (x, a0 a1,, aj) 1. Logo, x = p (veja que p >aj e mdc (p, a0 a1, ,aj) = 1). Os outros passos de induo so totalmente anlogos.

EUREKA! N13, 2002

35

Sociedade Brasileira de Matemtica

PROBLEMA 3: SOLUO DE LARISSA CAVALCANTE QUEIROZ DE LIMA (FORTALEZA - CE) E F A B x 2x (m + n ) x m x D G m n C x n x M

ED BC EDB um tringulo retngulo. Como EF = FB, F ponto mdio de EB EF = FB = DF B = x , temos FB D = x DF E = FD B + DB F = x + x = 2 x < 3 x = CF A Se FD portanto D no est dentro do segmento BC H, p da altura relativa ao lado BC est fora do segmento BC (pois AH // ED e H, D, B esto na reta nessa ordem H B um ngulo obtuso. DB e como D CB , temos H CB ) AC D = CF A DF A = 3 x 2 x = x CF D = FD C CFD issceles com CD = * CF CF = m. Seja M o ponto mdio de DB MF base mdia de BDE MF // ED F = 90 pois AD CF e G = AD CF) B = ED B = 90 ( DG FM M = x e DF M = FG G = DM G = x DGMF um quadriltero inscritvel FD M = CM G = x CMG issceles com CM = CG = n CG DM = MB MB = DC + CM = m + n e DB = DM + MB = 2(m + n) GF = GC + CF = m + n Menelaus FCB e reta ADG 2 2(m + n) AF DB GC n =1 = 1 4n = 3m 3 AB DC GF m m+n 3m 4m + 3m 7 2m + m DB 2(m + n) 2m + 2n 2 = 2 2=7 = = = = 2 DC m m m m m DB 7 = . DC 2 no obtuso H CB e portanto Obs. C D = AF C + CF D = 3x + y = 2 x y = x < 0 AF D CB contradio. CFD 0.
EUREKA! N13, 2002

36

Sociedade Brasileira de Matemtica

PROBLEMA 4: SOLUO DE ALEX CORRA ABREU (NITERI - RJ)

Definimos as seqncias (xk), (yk) como sendo respectivamente o mximo e o mnimo depois da operao feita k vezes x k +1 = senx k ou cos y k pois a funo seno crescente e a co-seno decrescente no intervalo considerado e y k +1 = seny k ou

cos x k , mas senx > cos y senx > sen y x + y > 2 2 se x k + y k > , temos x k +1 = senx k e y k +1 = cos x k . Analogamente, se 2 2 2 + yk = 1, k 1. mas x k + y k < , temos x k +1 = cos y k e y k +1 = seny k x k 2 2 2 2 2 + yk + 2xk y k ) 2( xk + yk ( xk + y k ) 2 = ( xk ) = 2 xk + y k 2 < 2 xk +1 = cos y k e y k +1 = senyk . Assim, temos x 2001 = cos y 2000 = cos seny1999 = cos sensen...seny1 = cos sensen...sen cos 1 j que

1999 vezes

cos 1 < sen 1, pois

<1< . 4 2

PROBLEMA 5: SOLUO DE EINSTEIN DO NASCIMENTO JUNIOR (FORTALEZA - CE)

Lema 1: Sabemos que um quadriltero inscritvel se e somente se as mediatrizes dos lados desse quadriltero so concorrentes. Lema 2: Os pontos mdios dos lados de um quadriltero qualquer, convexo ou no, formam um paralelogramo. Tome agora um quadriltero convexo ABCD, com lados opostos no paralelos.
N C

O'1 O1

Sejam M, N, P, Q os pontos mdios de AB, BC, CD, DA. Trace agora as mediatrizes de BC e AD, que se encontram em O1. Chame NQ MP = E. Provaremos que as alturas relativas a BC e a AD se encontram no ponto simtrico a O1 em relao a E. Chame O'1 o simtrico de O1 em relao a E.

EUREKA! N13, 2002

37

Sociedade Brasileira de Matemtica

Pelo Lema 2, E ponto mdio de NQ e E ponto mdio de MP e alm disso por definio de O'1, E ponto mdio de O'1O1. Ento temos que NO1QO'1 um paralelogramo! Da: NO'1 // O1Q NO'1 AD NO'1 a altura em relao a AD. QO'1 // NO1 QO'1 BC QO'1 a altura em relao a BC. Logo O'1 o encontro das alturas relativas a BC e AD. Fazendo o mesmo para os lados AB e CD podemos concluir que: O simtrico em relao a E do encontro de mediatrizes de lados opostos igual interseo das alturas destes lados opostos. Chame O2 o encontro das mediatrizes de AB e CD. O'2 ser o simtrico em relao a E de O2 e consequentemente o encontro das alturas relativas a AB e CD. Note que: O'1 O'2 O1 O2. Ento: O'1 O'2 O1 O2 ABCD inscritvel. Segue que O'1 O'2 ABCD inscritvel. Logo as 4 alturas tem um ponto em comum se e somente se o quadrado for inscritvel.
PROBLEMA 6: SOLUO DE HUMBERTO SILVA NAVES (SO PAULO - SP)

Vamos introduzir o conceito de energia para as pedras: 3 , onde pos(x) a posio de x. x 4 O que acontece se realizarmos um movimento? Vamos mostrar que a energia sempre diminui a cada momento: Movimento tipo A: E=

pos ( x )

p1

p+1

p1
p +1

p+1
p 1

3 3 E' = E 4 4 Movimento tipo B:

3 + 4

5 3 = E < E 12 4

p1

p+1

p+2

p1

p+1 p+2

EUREKA! N13, 2002

38

Sociedade Brasileira de Matemtica

3 3 3 3 5 =E < E E ' = E 2 + + 4 4 4 4 48 3 Obs. O nmero no foi escolhido ao acaso, foi escolhido um nmero q tal que: 4 1 < q + q 2 e 1 + q 3 < 2q Considere um copo de posio p, onde p tal que 3 > E 0 , onde E0 a energia inicial do sistema. 4 Como a energia, a cada movimento, sempre diminui, qualquer que sejam os movimentos que se faa, nenhuma pedra ficar numa posio menor ou igual a p. Ou seja, existe uma "barreira" esquerda para as pedras. Estamos agora capazes de resolver o problema (a primeira parte): * Dada uma configurao inicial das "n" pedras, impossvel realizar uma seqncia infinita de movimentos. Demonstrao: Vamos demonstrar (*) por induo: Base: Para n = 1 verdadeiro! Passo indutivo: Suponha, por absurdo, que seja possvel realizar uma seqncia infinita de movimentos. Sabemos que a posio da pedra mais esquerda no aumenta a cada movimento e como existe uma barreira esquerda, ento a partir de um certo ponto a pedra mais a esquerda no mais ser movimentada, e s com os restantes (o nmero de pedras restantes no mximo n 1) impossvel realizar uma seqncia infinita de movimentos, o que um absurdo! Logo por induo, (*) verdadeiro para todo n *. Vamos resolver a segunda parte do problema: Sabemos que dada uma configurao inicial, independente das escolhas dos movimentos sempre chegamos a uma configurao onde impossvel mover (configurao parada). Suponha por absurdo que a partir de uma configurao inicial se chegue a duas configuraes paradas distintas A e B. Seja k' a posio da pedra mais direita das configuraes A e B e k = k' + 2. Considere o seguinte invariante: (no varia a cada movimento) I= Fk pos ( x ) , onde Fn o n-simo nmero de Fibonacci.
p

p 1

p+2

Lembramos que F1 = 1, F2 = 1 e Fn + 2 = Fn + 1 + Fn, para todo n 1) Sabemos que I A = I B , pois I invariante, isto , permanece o mesmo depois de cada movimento. De fato, Fk (p 1) = Fk p + 1 = Fk p + Fk p 1 = Fk p + Fk (p + 1), donde I no muda aps um movimento do tipo A, e Fk ( p 1) + Fk ( p + 2) = Fk p + Fk p 1 + Fk p 2 =
EUREKA! N13, 2002

39

Sociedade Brasileira de Matemtica

= 2 Fk p , donde I no muda aps um movimento do tipo B. Algoritmo: 1- Seja x a pedra mais esquerda de A e y a pedra mais esquerda de B. Devemos ter pos(x) = pos(y), pois se fosse pos(x) pos(y) (assumimos sem perda de generalidade que pos(x) > pos(y)), teramos: IA = Fk pos (t ) Fk pos ( x ) + Fk pos ( x ) 2 + Fk pos ( x ) 4 + ... + F2

t A

se k pos ( x) for par e IA =

F
t A

k pos ( t )

Fk pos ( x ) + Fk pos ( x ) 2 + ... + F3 caso contrrio:

Mas F2 + ... + F2 k = F2 k +1 1 < F2 k +1 e F3 + ... + F2 k +1 = F2 k + 2 1 < F2 k + 2 (como se prova facilmente por induo). logo I A Fk pos ( x ) +1 1 < Fk pos ( x ) +1 Fk pos ( y ) I B , um absurdo! 2- Seja A: = A {x} e B: = {y}. 3- V para o 1. Pronto! Demonstramos que A e B so a mesma configurao, o que um absurdo! A configurao final independe da escolha dos movimentos.

EUREKA! N13, 2002

40

Sociedade Brasileira de Matemtica

XXIII OLIMPADA BRASILEIRA DE MATEMTICA


Problemas e Solues da Primeira Fase Nvel Universitrio
PROBLEMA 1

Seja f ( x) = e x sen x. Calcule f (2001)(0). (Denotamos por f (n)(x) a derivada de ordem n no ponto x; assim, f (2)(x) = f '' (x).)
PROBLEMA 2

Seja s(n) a soma dos algarismos de n. Assim, por exemplo, s(77) = 14 e s(2001) = 3. Diga se existe um inteiro positivo n com s(n) = 10 e s(n2) = 100. Se no existir, demonstre este fato. Se existir, d um exemplo.
PROBLEMA 3

O centro de massa de uma lata cilndrica de refrigerante tem a mesma posio quando a lata est vazia ou cheia. Se a massa da lata vazia m e a massa do refrigerante dentro da lata cheia M, determine a frao de refrigerante que deve ser deixado na lata para que seu centro de massa fique o mais baixo possvel.
PROBLEMA 4

Um ratinho ocupa inicialmente a gaiola A e treinado para mudar da gaiola atravessando um tnel sempre que soa um alarme. Cada vez que soa o alarme o ratinho escolhe qualquer um dos tneis incidentes a sua gaiola com igual probabilidade e sem ser afetado por escolhas anteriores. Qual a probabilidade de que aps o alarme soar 23 vezes o ratinho ocupe a gaiola B?
A B C

PROBLEMA 5

Seja A uma matriz n n com a1, j = ai, 1 = 1 (para quaisquer i e j, 1 i, j n) e a i +1, j +1 = a i , j + a i +1, j + a i , j +1 (para quaisquer i e j, 1 i, j < n). Assim,
1 1 A = 1 1

  . Calcule det(A).      
1 1 1 3 5 7 5 13 25 7 25 63

PROBLEMA 6
2 Seja xn uma seqncia de nmeros reais definida por x n +1 = x n

xn , n 0. 2

Para quais valores de x0 a seqncia converge? Para que valor?


EUREKA! N13, 2002

41

Sociedade Brasileira de Matemtica

SOLUES NVEL UNIVERSITRIO


SOLUO DO PROBLEMA 1

f ' ( x) = e x ( senx cos x) = 2e x sen x 4 Em geral, se u = x + a, onde a uma constante, a derivada de e x sen u igual a 2e x sen u . 4 Logo f ' ' ( x) = ( 2 ) 2 e x sen x = 2e x sen x . 4 4 2 k Se f ( k ) ( x) = ( 2 ) k e x sen x , para k , teremos 4 (k + 1) k k +1 x x f ( k +1) = ( 2 ) k . 2e sen x 4 4 = ( 2 ) e sen x 4 n Logo, por induo, f ( n ) ( x) = ( 2 ) n e x sen x ( n ) e 4 2001 1000 x ( x) = ( 2 ) 2001 e x sen x = 2 2 e sen x 4 4 1000 x = 2 e (cos x sen x).
f
( 2001)

SOLUO DO PROBLEMA 2

Existem muitos inteiros n com as propriedades pedidas. A menor soluo 1101111211. Algumas outras so 10111111111, 11011111111, 200220000202 e

n=

10
j =0

2j

A nica condio necessria que, ao calcular n2 pelo algoritmo usual no deve ocorrer nenhum 'vai um'. Mais precisamente, se a expanso decimal de n n = p (10), p ( x) = a k x k + ... + a 0 com 0 a i < 10 ento temos p(1) = s (n) = 10. Temos q ( x) = ( p( x)) 2 = b2 k x 2 k + ... + b0 com b j = a j a 0 + a j 1 a1 + ... + a 0 a j , q (10) = n 2 e q (1) = 100. Para que s (n 2 ) = q (1) precisamos apenas que cada bj seja menor do que 10. Exemplos de solues podem ser facilmente obtidos tomando os algarismos de n pequenos e espalhados.
EUREKA! N13, 2002

42

Sociedade Brasileira de Matemtica

Se h frao de refrigerante na lata, [0,1] , a massa total de refrigerante ser M e o centro de massa do refrigerante (sem contar a lata) tem altura h , onde h 2 h a altura total da lata. Como o centro de massa da lata vazia tem altura , a altura do 2 h h + m / (M + m ). centro de massa f ( ) = M 2 2 Temos h ((M + m) 2M ( 2 M + m) M ) f ' ( ) = 2 2(M + m) h = ( 2 M 2 + 2Mm mM ). 2 2(M + m) As razes de 2 M 2 + 2Mm mM so 1 = Mm M 2 m 2 + M 3 m <0 e M2

SOLUO DO PROBLEMA 3

2 =

Mm + M 2 m 2 + M 3 m m 2 + Mm m m = = < 1. Assim, 2 M M m + m 2 + Mm f ' ( ) < 0 para 0 < 2 e f ' ( ) > 0 para 2 < 1, e portanto f ( ) mnimo m 2 + Mm m . M

para = 2 =

SOLUO DO PROBLEMA 4

Seja an a probabilidade de que aps n apitos o ratinho esteja na coluna central (B ou E). Temos a0 = 0 (o ratinho no comea na coluna central). Claramente aps o apito soar um nmero par de vezes o ratinho estar em A, C ou E e aps um nmero mpar de vezes em B, D ou F. Assim, queremos calcular a23. Se, antes de soar o alarme, o ratinho est na coluna central ele tem 1/3 de probabilidade de permanecer l (independentemente da gaiola onde o ratinho estava ser B ou E). Por outro lado, se ele no est na coluna central ele tem probabilidade 1/2 de ir para l (novamente independentemente da gaiola onde o ratinho comeou). Assim, 1 1 1 1 3 1 3 a n +1 = a n + (1 a n ) = a n ou a n +1 = (a n ). 3 2 2 6 7 6 7

EUREKA! N13, 2002

43

Sociedade Brasileira de Matemtica

A seqncia bn = a n

3 1 portanto uma progresso geomtrica de razo com 7 6 23 3 3 3 3 3(6 + 1) = e a 23 = + b0 = . Assim b23 = . 23 23 7 7 76 76 7 6 23

SOLUO DO PROBLEMA 5

Fazendo operaes em linhas (subtraindo a primeira linha da segunda, a segunda da terceira e assim por diante) temos
1 0 det( A) = det 0 0

  2 8 18  2 12 38      
1 1 1 2 4 6

Fazendo agora operaes em colunas (subtraindo a primeira coluna da segunda, a segunda da terceira e assim por diante) temos
1 0 det( A) = det 0 0

 2 2 2  2 6 10  = 2 2 10 26      
0 0 0

1 1 1 1 3 5 n 1 det 1 5 13

  .      

Esta ltima matriz a verso (n 1) (n 1) de A. De fato, sua entrada (i, j) 1 ((a i +1, j +1 a i +1, j ) (a i , j +1 a i , j )) = a i , j . 2 Chamando o valor de det(A) para matrizes n n de bn temos portanto
b1 = 1,
SOLUO DO PROBLEMA 6

bn +1 = 2 bn donde bn =
n

n ( n 1) 2 2 .

Seja

x 3 3 f ( x) = x 2 . Temos f ( x) = x x = 0 ou x = . Para x > temos 2 2 2 3 3 f ( x) > x. Assim, se x 0 > temos x n x 0 > para todo n e, por induo, 2 2 x n +1 = f ( x n ) > x n para todo n. Assim, nesse caso, (xn) crescente, e portanto lim n x n = L para algum L # ou lim n x n = +. No primeiro caso, teramos

EUREKA! N13, 2002

44

Sociedade Brasileira de Matemtica

3 L = lim n x n +1 = lim n f ( x n ) = f (lim n x n ) = f ( L), mas L = lim x n x 0 > , 2

donde f ( L) > L , absurdo. Assim, se x 0 > , lim n x n = +. Se x 0 < 1, x1 = f ( x 0 ) > Se 3 , donde tambm temos lim n x n = +. 2

3 2

1 3 < x < , com x 0, temos f (x ) < x . Por outro lado, para todo 2 2 1 1 1 1 n 1, x n = f ( x n 1 ) = ( x n 1 ) 2 > . Assim, se 4 16 16 2 3 1 < x 0 < , x n +1 = f ( x n ) x n para todo Portanto, existe n 1. 2 3 c = lim n x n < . Temos portanto 2 0 c = lim n x n +1 = lim n f ( x n ) { f (c) , f (c) }. Temos que f (c) = c, com

c > 0, implica c =

1 1 1 , e, como x n para todo n 1, se lim n x n = teramos 2 16 2 1 1 1 lim n x n = , donde = lim n x n +1 = lim n f ( x n ) = f = 0, absurdo. Como 2 2 2 3 f (c) < c se 0 < c < , temos necessariamente c = 0, o portanto lim n x n = 0. 2 3 3 3 Se x 0 = 1 ou x 0 = temos x n = para todo n 1, donde lim n x n = . 2 2 2 3 3 Assim, xn converge se 1 < x 0 < , quando lim n x n = 0 e se x 0 1, , 2 2 3 quando lim n x n = . Em qualquer outro caso, lim n x n = +. 2

EUREKA! N13, 2002

45

Sociedade Brasileira de Matemtica

XXIII OLIMPADA BRASILEIRA DE MATEMTICA


Problemas e Solues da Segunda Fase Nvel Universitrio
PROBLEMA 1:

So dados um ponto O e uma reta r no plano. Para cada ponto P de r, seja rp a reta perpendicular a OP passando por P. Prove que o conjunto {r p P r} o conjunto de todas as retas tangentes a uma parbola.
PROBLEMA 2:

Seja um nmero real positivo arbitrrio. Com centro em todos os pontos do plano com coordenadas inteiras, traa-se um crculo de raio . Prove que toda reta passando pela origem intercepta uma infinidade desses crculos.
PROBLEMA 3:

Definimos SL(2, ) como o conjunto das matrizes 2 2 com coeficientes inteiros e determinante 1. Seja A SL(2, ) uma matriz tal que existe n > 0 inteiro com An = I. Prove que existe X SL(2, ) tal que X1AX igual a uma das matrizes: 1 0 0 1 0 1 0 1 ; ; ; 0 1 1 1 1 0 1 1 .
PROBLEMA 4:

Caminhando sobre os segmentos unitrios da figura abaixo, determine quantos percursos distintos existem de A at B sem passar duas vezes por um mesmo ponto.

PROBLEMA 5:

Para todo real u, seja I (u ) = ln (1 2u cos x + u 2 )dx.


0

a) Prove que I (u ) = I (u ) =
PROBLEMA 6:

1 I (u 2 ). 2

b) Calcule I(u) para todo u #. p 1. Seja f : D D uma funo

Seja D o conjunto dos pontos p em

#2 com

sobrejetora satisfazendo f ( p ) f (q ) p q para quaisquer p, q D. Prove que f uma isometria, isto , que f ( p ) f (q ) = p q para quaisquer p, q D. (Observao: ( x, y ) denota
EUREKA! N13, 2002

x 2 + y 2 .)

46

Sociedade Brasileira de Matemtica

SOLUES NVEL UNIVERSITRIO


PROBLEMA 1: SOLUO DE EMANUEL DE SOUZA CARNEIRO (FORTALEZA - CE)

Suponha, sem perda de generalidade, que o ponto O seja (0,1) e a reta r seja o eixo x. Seja P um ponto sobre r, P(p, 0), logo: (x, y) rp (x p, y) (p, 1) = 0 xp p2 y = 0 (Equao da reta rp). Buscamos agora uma parbola do tipo y = ax2 + bx + c, de modo que estas retas rp sejam tangentes a ela. Derivando, obtemos a equao da reta tangente a essa parbola no ponto (x0, y0). y y0 = 2ax 0 + b x x0
2 + bx 0 + c) = ( x x 0 )(2ax0 + b) ou seja: y (ax 0 2 x(2ax 0 + b) ax 0 +c y=0

1 1 2 tome a = 1/4, b = c = 0, da teremos: x x 0 x 0 y = 0.

Esta ser uma idia: em {rp | P r} basta tomar p = x0 /2. Reciprocamente, cada rp {rp | P r} tangente parbola y = 1 2 x no ponto 4 (2p, p2), pois a equao da reta tangente a essa parbola nesse ponto : y p2 = f ' (2 p ) = p xp p 2 y = 0 , que a equao da reta rp. x 2p
4

2 Logo o conjunto{rp | P r} o conjunto das retas tangentes parbola y = x .

PROBLEMA 2: SOLUO DE EMANUEL DE SOUZA CARNEIRO (FORTALEZA - CE)

Para provar a questo, mostraremos uma lema equivalente ao lema de Kronecker.


Lema: Seja um nmero irracional. Dado > 0 arbitrrio, existe a

* tal que

{a } < , onde {x}= x [x] indica a parte fracionria de x. tal que n < 1 (n + 1) . Divida o intervalo [0, 1) em (n + 1) Prova: Seja n intervalos do tipo [i , (i + 1) ), 0 i n; o ltimo intervalo [n, 1).
0

(n + 1)

Observe agora os nmeros , 2, 3,;(n + 2) . Como a funo parte fracionria vai dos reais em [0,1), pelo princpio da casa dos pombos, dois dentre os (n + 2) nmeros { }, {2 }, {3 },,{(n +2) }, cairo num mesmo intervalo [i, (i + 1)).
EUREKA! N13, 2002

47

Sociedade Brasileira de Matemtica

Sejam j, ) os dois nmeros tais que: {j }, { ) } [i , (i + 1) ). Suponha, sem perda de generalidade, que {j} { ) } Assim: j = [ j ] + { j } ( j ) = [ j ] [ ] + ({ j } { }) = [ ]+ { }

Obs: No podemos ter {k} = 0, k *, pois isso implicaria k = n ", absurdo! Vamos agora resolver o problema. Seja dado > 0. Uma reta passando pela origem tem a forma y = mx

)  ) ) Logo {( j )) } = { j } {) } < (i + 1) i = e j ) o inteiro procurado ) ) )


inteiro

= n/k

Primeiro caso: Se m ", nesse caso a reta passa por infinitos "Lattice points" (pontos

de coordenadas inteiras), pois se m = p/q os pontos da forma (x, y) = (kq, kq . p/q) = (kq, kp), k = 1, 2, 3 esto todos na reta, ento ela interceptar todos os crculos que tm centros nesses pontos.
Segundo caso: Se m for irracional. Os pontos da reta so da forma (x, mx), logo, pela observao feita aps o lema, esta reta no contm nenhum ponto de coordenadas inteiras.

Suponha que ela intersecte somente uma quantidade finita de crculos, digamos C1, C2,, Cn de centros P1, P2, , Pn Observe que para qualquer outro ponto no "Lattice" P (fora os centros) teremos: d(P, r) > . Seja d a menor das distncias, d1(P1, r), d2(P2, r),, dn(Pn , r) onde di(Pi, r) a distncia do centro Pi reta r. (Note que di (Pi, r) > 0 d > 0). Logo teramos dentro todos os pontos de coordenadas inteiras no plano, um deles Pi que minimizaria a distncia reta r, com d(Pi, r) = d > 0. Isso contradiz o lema, vejamos: Pelo lema existe um inteiro a, de modo que {am} < d/2. Seja b = [am] , o ponto Q = (a, am) r e sua distncia a (a, b) menor ou igual que a distncia de (a, b) a r.

EUREKA! N13, 2002

48

Sociedade Brasileira de Matemtica


r y (a, am) (a, b) d'

Logo d' d((a, b), (a; am)) = |am b| = {am} < d/2. Absurdo, pois d era a distncia mnima a reta corta uma infinidade de crculos.

PROBLEMA 3: SOLUO DE CARLOS YUZO SHINE (SO PAULO SP)

Por induo, pode-se mostrar que Ak = kA (k 1)I. Desta forma, Am = I mA (m 1)I = I A = I. Se a + d = 2, temos, analogamente, que Ak = (1)k + 1(kA + (k 1)I) e portanto Am = I (1)m + 1(mA + (m 1)I) = I A = I e m par (na verdade, m = 2). Se 1 2, ento a matriz A diagonalizvel em , ou seja, existe uma matriz P tal 1 0 k k 1 que A = PDP1, sendo D = . Neste caso, temos que A = PD P , com 0 2
k 1 0 Dk = 0 k . 2 Assim, Am = I PDmP1 = I Dm = I 1m = 2m = 1, ou seja, 1 e 2 so razes m-simas da unidade. Assim, como a + d = 1 + 2, 1 a + d 1. Como a + d inteiro, temos os seguintes casos:

a b n Seja A = uma matriz de SL(2, ) tal que A = I para algum inteiro positivo n. c d Seja m o menor inteiro positivo tal que Am = I. Sejam 1 e 2 os autovalores de A, ou seja, as razes da equao det(A xI) = 0 x2 (a + d)x + 1 = 0. (I) Se 1 = 2, temos que o discriminante da equao (I) nulo, logo a + d = 2 ou a + d = 2. Se a + d = 2, temos a 2 + bc b(a + d ) 2a 1 2b a b 2 = = 2A I = A A= c d c(a + d ) d 2 + bc 2d 1 2c

(i) (ii)

a + d = 1. Neste caso, (I) x2 + x + 1 = 0 e portanto 1 e 2 so razes cbicas da unidade. Portanto m = 3. a + d = 0. Neste caso, (I) x2 + 1 = 0 e portanto 1 e 2 so razes quartas da unidade. Portanto m = 4.

EUREKA! N13, 2002

49

Sociedade Brasileira de Matemtica

(iii)

a + d = 1. Neste caso, (I) x2 x + 1 = 0 e portanto 1 e 2 so razes sextas da unidade. Portanto m = 6. Observando as matrizes dadas, temos
3 4 6

0 1 0 1 0 1 =I 1 1 1 0 1 1 = I; = I; Assim, basta provarmos que existem inteiros x, y, z e w com xw yz = 1 e x y w y A = , z w W z x sendo W uma das trs matrizes acima. Note que w y x y x y I z x z w z w = det yw + xz + yz x 2 y 2 xy 0 1 Caso (i) Tomamos W = = . Assim A 1 1 z 2 + w 2 + zw yw xz xw . Veja que a + d = 1 equivalente a xw yz = 1. A condio det A = 1 equivalente a ad bc = 1 a(1 a) bc = 1 bc = (a2 + a + 1). Assim, devemos ter a = ( yw + xz + yz ) b = ( x 2 y 2 xy ) (II) c = ( z 2 + w 2 + zw) Isso pode ser verificado fatorando a2 + a + 1 = (a )(a ) em inteiros de Eisenstein. Considere um fator x y de a e seja z w o fator de a tal que (x y)(z w) = a . Note que (x y )(z w ) = a . As normas dos fatores so x2 + y2 + xy e z2 + w2 + zw e portanto sempre existem x, y, z e w tais que |b| = x2 + y2 + xy e |c| = z2 + w2 + zw. Substituindo o valor de a de (II) na equao original verifica-se que de fato bc = (a2 + a + 1). Reciprocamente, a outra raiz desta ltima equao 1 a = d = xz yw xw. Mas para resolver isso basta trocar x e y por z e w, respectivamente, e trocar o sinal. Obs. Usamos implicitamente a existncia e unicidade da fatorao para inteiros de Eisenstein para obter x, y, z, w como acima.

EUREKA! N13, 2002

50

Sociedade Brasileira de Matemtica

yw + xz x 2 y 2 0 1 Caso (ii) Agora tomamos W = = , com A 1 0 z 2 + w 2 yw xz . Novamente, bvio que a condio a + d = 0 satisfeita. A condio det A = 1 equivalente a ad bc = 1 bc = (a2 + 1). O sistema correspondente agora a = ( yw + xz ) b = ( x 2 y 2 ) (III) c = ( z 2 + w 2 ) Agora usamos inteiros de Gauss! Considere a fatorao a2 + 1 = (a + i)(a i) e sejam x + yi e z + wi fatores de a + i com (x + yi)(z + wi) = a + i. Note que (x yi)(z wi) = a i. Ento existem sempre x, y, z e w tais que |b| = x2 + y2 e |c| = z2 + w2. Novamente, substituindo o valor de a de (III) na equao original vemos que de fato bc = (a2 + 1). A outra raiz oposta ao valor de a de (III), mas s trocar o sinal. Obs. Aqui usamos implicitamente a existncia e unicidade de fatorao para inteiros de Gauss para obter x, y, z, w como acima. 0 1 Caso (iii) Tomamos dessa vez W = . Temos nesse caso 1 1 yw + xz yz x 2 y 2 + xy A = z 2 + w 2 zw yw xz + xw . Mas esse caso anlogo ao caso (i)! Tome y = y e w = w e obtemos a mesma matriz do caso (i). Logo existe sempre uma matriz X tal que A = XWX1, onde W uma das matrizes dadas no enunciado.
PROBLEMA 4: SOLUO DE JULIANA ABRANTES FREIRE (RIO DE JANEIRO RJ)

Vamos numerar os pontos assim:


1 3 5 A (6) 0=A 2 4 B (6) 6 7 C (6) 8 B = 10 9

F(n) = O nmero de percursos que chegam em n, sem passar por n + 2. A(n) = O nmero de percursos que chegam em n vindos de n 1. B(n) = O nmero de percursos que chegam em n vindos de n 2. C(n) = O nmero de percursos que chegam em n vindos de n + 1. A(6), B(6) e C(6) esto ilustrados na figura acima.
EUREKA! N13, 2002

51

Sociedade Brasileira de Matemtica

Note que no h mais caminhos relevantes para o problema alm destes: Qualquer caminho indo a 6 vindo de 8, passou obrigatoriamente por 6 ou 7 ou ambos. Se passou por 6, este caminho no pode porque passaria por 6 duas vezes. Mesmo se no passou por 6, este caminho no pode: 7 e 8 j foram usados, ento este caminho no pode chegar at B. Ento F(n) = A(n) + B(n) + C(n). B(n) = F (n 2), porque nenhum caminho chegando a n 2 passou por n, como no exemplo descrito anteriormente. Ento B(n) = A(n 2) + B(n 2) + C(n 2). A(n) = B(n 1) + A(n 1):
B (n 1) C (n 1)

A (n 1) A(n) n

Porque todos os C(n 1) passaram por n (mais que isto, eles so os caminhos que vieram para n 1 diretamente de n) e nenhum A(n 1) ou B(n 1) passou por n porque eles no teriam como passar por n e voltar para n 2 e

n 3, respectivamente, sem repetir pontos e mantendo a possibilidade do caminho chegar a B. C(n) = B (n 1) + A(n 1), porque estes so os caminhos que chegam a n + 1 sem passar por n: eles vo direto de n 1 para n + 1. Para chegar ao ponto 1: B(n) = 0, no existe caminho. B (1) 1 3 A(1) = 1 (caminho direto ) C(1) A (1) ). C(1) = 1 (caminho
0 2

Para chegar ao ponto 2:


1 3 C(2) 2 4

Os caminhos possveis so:

A (2) 0 B(2)

Ou seja A(2) = B(2) = C(2) = 1. A(3) = A(2) + B(2) = 2 B(3) = A(1) + B(1) + C(1) = 2 C(3) = A(2) + B(2) = 2 A(4) = A(3) + B(3) = 4 B(4) = A(2) + B(2) + C(2) = 3 C(4) = A(3) + B(3) = 4
EUREKA! N13, 2002

A(5) = A(4) + B(4) = 7 B(5) = A(3) + B(3) + C(3) = 6 C(5) = A(4) + B(4) = 7 A(6) = A(5) + B(5) = 13 B(6) = A(4) + B(4) + C(4) = 11 C(6) = A(5) + B(5) = 13

52

Sociedade Brasileira de Matemtica

A(7) = A(6) + B(6) = 24 B(7) = A(5) + B(5) + C(5) = 20 C(7) = A(6) + B(6) = 24 A(9) = A(8) + B(8) = 81 B(9) = A(7) + B(7) + C(7) = 68 C(9) = A(8) + B(8) = 81

A(8) = A(7) + B(7) = 44 B(8) = A(6) + B(6) + C(6) = 37 C(8) = A(7) + B(7) = 44

C(10) = 0 porque no existe ponto 11. Mas ainda vale A(10) = A(9) + B(9) = 149 B(10) = A(8) + B(8) + C(8) = 125 F(10) = A(10) + B(10) + C(10) = 149 + 125 + 0 = 274.
PROBLEMA 5: SOLUO DE MARCIO ASSAD COHEN (RIO DE JANEIRO RJ)

I (u ) =

a) fazendo t = x ; dt = dx:

ln(1 2u cos x + u 2 )dx

1 2u cos( I (u) = ln( t) + u2 )dt =

ln(1+ 2ucos t + u )dt = ln(1 2(u)cosx + (u) )dx =


2 2 0 0

I( u). Logo, I(u) = I(u). Note agora que, como cosx = 2cos2x/2 1: 1 2u 2 cos x + u 4 = 1 + 2u 2 + u 4 4u 2 cos 2 x / 2 = (1 + u 2 ) 2 4u 2 cos 2 x / 2 = (1 + 2u cos x / 2 + u 2 )(1 2u cos x / 2 + u 2 ) Logo, I (u 2 ) = ln (1 + 2u cos x / 2 + u 2 )dx +
0

ln(1 2u cos x / 2 + u 2 ) dx

Fazendo t = x/2 nessas ltimas integrais vem: /2 /2 2 ln( 1 2 cos ) ln(1 2u cos t + u 2 )dt . I (u 2 ) = 2 u t u dt + + + 0 0

(II)

Agora, note que


t = x

/2

ln(1 + 2u cos t + u 2 )dt =

x = t-

-/2

ln(1 2u cos x + u 2 )dx = (III)

/2

ln(1 2u cos x + u 2 )dx =

/2

ln(1 2u cos x + u 2 )dx

De (II) e (III) vem /2 I (u 2 ) = 2 ln(1 + 2u cos x + u 2 )dx + 0 1 Logo, I(u) = I(u2). 2

/2

ln(1 + 2u cos x + u 2 )dx = 2 I (u )

EUREKA! N13, 2002

53

Sociedade Brasileira de Matemtica

b) Fazendo u = 1, vem I(1) = 2I(1) I(1) = 0. Em geral, I(u) = I(u) =

k 1 I (u 2 ), x : k 2

1 1 1 1 1 I (u 2 ) = I (u 4 ) = I (u 8 )... ( uma induo simples). 2 2 2 4 2 Supondo 0 u 1 inicialmente: cos x 1 I (u ) =

ln(1 2u cos x + u 2 )dx


k

ln(1 + u ) 2 dx = 2 ln(1 + u )

k k 1 1 I (u 2 ) k 2 ln(1 + u 2 ) . k 2 2 2 2 ln(1 + u t ) = k Fazendo k e substituindo t = 2 , deve-se ter I (u ) lim 0. t t Logo, I(u) = 0 se u [ 1, 1] (lembrando j que I(u) = I( u)). 1 Se |u| > 1, fao a substituio u = : Ento ( < 1) : 2 2 cos x + 1 1 1 2 cos x dx = + 2 dx = ln I = ln1 0 2 0 1 I = ln(1 2 cos x + 2 )dx ln( 2 )dx = I ( ) 2 ln = 2 ln . 0 0 1 Logo, se u > 1, temos: I(u) = 2 ln I (u ) = 2 ln u u

Logo, I (u )

2 ln(1 + u 2 )
k

, x pois

zero, se u 1 Concluindo: I (u ) = 2 ln u , se u > 1.


PROBLEMA 6: SOLUO DE BRUNO FERNANDES CERQUEIRA LEITE (SO PAULO SP) Fato 1: Seja 0 = (0, 0). Ento f(0) = 0. Prova: x D , temos x 0 1. Logo devemos ter f ( x) f (0) 1, x D! Como

a funo sobrejetora,

y D

x 0 D com

f ( x 0 ) = y.

Logo,

y D ,

y f (0) 1, ou seja, f(0) dista no mximo 1 de qualquer ponto do disco D. Logo f(0) = 0.
Fato 2: Sejam Br = ( x, y ) x + y < r , Br = ( x, y ) x + y r . Ento, se p Br,
2 2 2 2

f ( p ) B r . Se p Br , f ( p ) Br .
EUREKA! N13, 2002

54

Sociedade Brasileira de Matemtica

Prova: p Br Br p 0 < r [ r ] . Mas f ( p) f (0) = f ( p) 0 p 0 < r[ r ].

[ ]

Logo f ( p ) B r ( B r ). Fato 3: Se f(p) est no bordo de D ( | f (p) 0| = 1) ento p est no bordo de D. Prova: f(p) e f(p) so diametralmente opostos, logo f ( p) ( f ( p )) = 2. Se p no estivesse no bordo de D, q D, p q < 2, absurdo pois deveramos ter f ( p ) f (q ) p q , p, q D. Fato 4: Se f(p) e f(q) so diametralmente opostos, ento p e q tambm so. Prova: Se p e q no fossem diametralmente opostos, p q < 2 = f ( p) f (q ) , absurdo. Fato 5: Se p e q so opostos tais que f(p) e f(q) so diametralmente opostos, e se x est entre p e q ento f(x) est entre f(p) e f(q). Alm disso, nesse caso f ( x) f ( q) = xq, f ( x) f ( p ) = xp e f ( x) f (0) = x0 Se A, B, C so pontos distintos, dizemos que B est entre A e C (e denotamos A B C) se AB + BC = AC ) ( onde AB = A B ). Prova: Sabemos, pelo fato 4, que p e q so diametralmente opostos. Se x = 0, f(x) = 0 e o lema fica trivial. Suponhamos, sem perda de generalidade, x 0 e p x 0. Ento Como e px + x0 = p 0. f ( p ) f ( x) px f ( x) f (0) x0, f ( p ) f ( x) + f ( x) f (0) px + x0 = 1 , por outro lado, pela desigualdade triangular, f ( p ) f ( x) + f ( x) f (0) f ( p ) f (0) = 1. Logo f(x) est entre f(p) e 0 e portanto est entre f(p) e f(q), e temos f ( p ) f ( x) = px, f (0) f ( x) = 0 x e f (q) f ( x) = qx. Fato 6: Se p est no bordo de D ( |p 0| = 1) ento f(p) est no bordo de D (o bordo , s para facilitar a notao). Alm disso, a restrio de f a B (que tem de D B ) uma composio de rotao com espelhamento. imagem B . A imagem inversa de Prova: Seja p D tal que f(p) = (1, 0). Pelo fato 3, p B (1, 0), pelo fato 4, p, isto , f( p) = (1, 0). ( p B ). Sejam q e q as imagens inversas de (0, 1) e ( 1, 0). e bem fcil ver que p, q, p e q formam um quadrado (ou no Ento q e q B teramos f (m) f (n) m n , m, n D). ~ Seja f : D D a composio de rotao com espelhamento que coincide com f nos ~ . pontos p, q, p e q. Vamos mostrar que f ( x) = f ( x), x D ~ ~ D tambm uma composio de Note que f uma bijeo, e que f 1 : D rotao com espelhamento.
EUREKA! N13, 2002

55

Sociedade Brasileira de Matemtica

. Existem n, n B tais que f(n) = m e f( n) = m. Devemos Sejam m, m D ter m (1,0) = f (n) f ( p) n p , m (1,0) = f (n) f ( p ) n + p . Isso j ~ ~ | x p | | m (1,0) | e implica que n = f 1 (m) ou n = f 1 (m) , pois {x D ~ ~ ~ ~ | x + p | | m (1,0) |} = { f 1 (m), f 1 (m)} (de fato | f (m) p | = | m f ( p) | = | m (1,0) | ~ ~ e Como, alm disso, | f 1 (m) + p | = | m f ( p) = | m (1,0) | . | m (0,1) | = | f (n) f (q ) | | n q | e | m (0,1) | = | f (n) f ( q) | | n + q | , sobra ~ ~ existe apenas a possibilidade n = f 1 (m). Como f 1 sobrejetiva, dado x D ~ ~ tal que x = f 1 (m) , e portanto teremos f ( x) = m = f ( x). mD Agora estamos em condies de terminar a prova: de fato segue dos fatos 5 e 6 que ~ f = f e logo uma composio de rotao com espelhamento, e portanto preserva ~ ~ em distncias. Com efeito, f = f em D , e f leva dimetros pq com q = p D ~ ~ ~ ~ ), e tanto f quanto ~ dimetros f ( p ) f (q) (e f ( p ), f (q) D f restritos ao dimetro pq preservam as distncias aos extremos, e portanto preservam distncias, logo coincidem.

EUREKA! N13, 2002

56

Sociedade Brasileira de Matemtica

XXIII OLIMPADA BRASILEIRA DE MATEMTICA


Resultado Nvel 1 (5a. e 6a. Sries)
NOME
Eduardo Fischer Raphael Rodrigues Mata Guilherme R. Nogueira de Souza Andr Linhares Rodrigues Andr Martins Costa Aranha Luiz Mller Rafael Bandeira Lages Felipe Gonalves Assis Renato Rebouas de Medeiros Thas Viveiro Adriano Jorge Braun Vieira Neto Jaques Deivinson da Silva Castello Enzo Haruo Hiraoka Moriyama Priscilla Yu Chen Kashiwakura Jefferson Quesado Neto William Vasconcelos de Morais Sophia Cherem Lopes Arthur Rodrigues de Oliveira Sobral Pedro Paulo Gondim Cardoso Regina Reis da Costa Alves Weslen Costa Timteo Andr Rodrigues Salerno Caroline Goulart Campos Edson Augusto Bezerra Lopes Luiz Felipe Bruzzi Curi Bernardo de Oliveira Veiga Felipe Alves Tom Luiza Cristina Maia e Silva Igor Ribeiro Azevedo Mariana Nasser Brolezzi Paulo Alexandre Pavoni Paulo Andr Carvalho de Melo Guilherme Pereira Barbosa Gustavo Sampaio Sousa Dennis G. de Macedo Bragagnolo Pedro Nogueira Machado Yuriy Thallickson Bincovski Gil Henriques Cssio Kendi Takamori Iuri Lima Ribeiro Franco Veronez Ribeiro Mateos Kruchelski Tsch Kleber Varela dos Santos Nbia Martins Domingues Eduardo Tadafumi Sato Marco Aurlio Buono Carone Thalles Melo de Oliveira Lopes Michel Ricardo Nigri Raquel Pereira Martins

CIDADE ESTADO
Encantado RS Salvador BA So Paulo SP Fortaleza CE Rio de Janeiro RJ Vitria ES Teresina PI Campina Grande PB Fortaleza CE So Paulo SP Fortaleza CE Serra ES So Paulo SP So Paulo SP Fortaleza CE Fortaleza CE Belo Horizonte MG S. J. dos Campos SP Salvador BA Rio de Janeiro RJ Paulista PE Goinia GO Rio de Janeiro RJ Fortaleza CE Belo Horizonte MG Rio de Janeiro RJ Fortaleza CE Recife PE Belo Horizonte MG Santo Andr SP Curitiba PR Rio de Janeiro RJ Belo Horizonte MG Fortaleza CE Curitiba PR Rio de Janeiro RJ Curitiba PR Vassouras RJ S. J. dos Campos SP Fortaleza CE Vitria ES Curitiba PR Jaboato dos Guararapes PE Belo Horizonte MG Mogi das Cruzes SP Belo Horizonte MG Goinia GO Rio de Janeiro RJ Rio de Janeiro RJ

PRMIO
Ouro Ouro Ouro Ouro Ouro Ouro Prata Prata Prata Prata Prata Prata Prata Prata Prata Prata Bronze Bronze Bronze Bronze Bronze Bronze Bronze Bronze Bronze Bronze Bronze Bronze Bronze Meno Honrosa Meno Honrosa Meno Honrosa Meno Honrosa Meno Honrosa Meno Honrosa Meno Honrosa Meno Honrosa Meno Honrosa Meno Honrosa Meno Honrosa Meno Honrosa Meno Honrosa Meno Honrosa Meno Honrosa Meno Honrosa Meno Honrosa Meno Honrosa Meno Honrosa Meno Honrosa

EUREKA! N13, 2002

57

Sociedade Brasileira de Matemtica

Resultado Nvel 2 (7a. e 8a. Sries)


NOME
Thiago Costa Leite Santos Henry Wei Cheng Hsu Rafael Daigo Hirama Rodrigo Aguiar Pinheiro Marcus Edson Barreto Brito Daniela Satie Kondo Telmo Luis Correa Junior Alan Hideki Uchida Felipe Rodrigues Nogueira de Souza Diogo dos Santos Suyama Ricardo Mizoguchi Gorgoll Karoline Matias Morais Paulo Roberto Sampaio Santiago Marcela Sobrinho Pereira Thoms Yoiti Sasaki Hoshina Guilherme Rodrigues Salerno Hector Kenzo Horiuti Kitahara Mauro Cardoso Lopes Andr Lucas Ribeiro dos Santos Renata Sayuri Takehara Henrique Castro Noronha Matheus Migliolo Coelho Lucas de Freitas Frenay Rafael Marini Silva Andr Slepetys Luiza Fontana Barbosa Jefferson Fonlin Tsai Deborah Regina Fujisawa Okuno Felipe Paupitz Schlichting Elton Gomes Coriolano lison Santos Xavier Marcus Vincius Martins da Costa Rodrigo Viana Soares Lucas M. Pereira Castello Branco Larissa Rodrigues Ribeiro Thiago Jorge Marinho Vieira Cincinato Furtado Leite Neto Rafael Kitayama Shiraiwa Anderson Hoshiko Aiziro Guilherme Alonso Daud Patavino Vitor Humia Fontoura Andr Schultz Gabriel Tavares Bujokas Francisco Bruno de Lima Holanda Gustavo Eufrsio Farias Katja Stephanie Ried Raul Mximo Alexandrino Nogueira Marcos Vainer Loeff Antonia Taline de Souza Mendona Jlio Vitrio dos Santos Ferreira

CIDADE ESTADO
So Paulo SP So Paulo SP Campinas SP Fortaleza CE Fortaleza CE So Paulo SP Santo Andr SP So Paulo SP So Paulo SP Belo Horizonte MG So Paulo SP Fortaleza CE Salvador BA Fortaleza CE Rio de Janeiro RJ Goinia GO So Paulo SP So Paulo SP Pindamonhangaba SP S. J. dos Campos SP Valinhos SP Limeira SP Santo Andr SP Vila Velha ES So Paulo SP Curitiba PR So Paulo SP So Paulo SP Florianpolis SC Fortaleza CE Fortaleza CE Belo Horizonte MG Fortaleza CE Fortaleza CE Fortaleza CE Fortaleza CE Fortaleza CE So Paulo SP So Paulo SP Santos SP Salvador BA Santa Brbara D'Oeste SP So Paulo SP Fortaleza CE Fortaleza CE Valinhos SP Fortaleza CE So Paulo SP Fortaleza CE Rio de Janeiro RJ

PRMIO
Ouro Ouro Ouro Ouro Prata Prata Prata Prata Prata Prata Prata Prata Prata Prata Prata Bronze Bronze Bronze Bronze Bronze Bronze Bronze Bronze Bronze Bronze Bronze Bronze Bronze Bronze Bronze Meno Honrosa Meno Honrosa Meno Honrosa Meno Honrosa Meno Honrosa Meno Honrosa Meno Honrosa Meno Honrosa Meno Honrosa Meno Honrosa Meno Honrosa Meno Honrosa Meno Honrosa Meno Honrosa Meno Honrosa Meno Honrosa Meno Honrosa Meno Honrosa Meno Honrosa Meno Honrosa

EUREKA! N13, 2002

58

Sociedade Brasileira de Matemtica

Resultado Nvel 3 (Ensino Mdio)


NOME
Humberto Silva Naves Davi Mximo Alexandrino Nogueira Larissa Cavalcante Queiroz de Lima Carlos Stein Naves de Brito Alex Corra Abreu Daniel Pinheiro Sobreira Einstein do Nascimento Jnior Guilherme Fujiwara Thiago Barros Rodrigues Costa Rafael Tajra Fonteles Eduardo Famini Silva Rodrigo Roque Dias Fbio Dias Moreira Daniel Pessa Martins Cunha Yuri Gomes Lima Paulo Ribeiro de Almeida Neto Thiago da Silva Sobral Germanna de Oliveira Queiroz Bernardo Freitas Paulo da Costa Samuel Barbosa Feitosa Isaac Newton Ferreira Santa Rita Jos Luiz Gomes Junior Ayran Ayres Barbosa Loriato Fernanda Maria de Oliveira Nicacio Henrique Chociay Joo Alfredo Castellani Fajardo Freire Rafael da Silva Faria Israel Franklim Dourado Carrah Artur Duarte Nehmi Lucas de Melo Pontes e Silva Diogo Luiz Duarte Estillac B. Filho Alex Cardoso Lopes Arthur M. Rocha de Azevedo Scalercio Diego Silva Dias Martha Priscilla Arajo de Moraes Ricardo Monteiro da Silva Lanna Fernando Souza Martins Maurcio Richartz Lo Tsukui Vitor Gabriel Kleine

CIDADE ESTADO
So Jos dos Campos SP Fortaleza CE Fortaleza CE Goinia GO Niteri RJ Fortaleza CE Fortaleza CE So Paulo SP Fortaleza CE Teresina PI Salvador BA So Paulo SP Rio de Janeiro RJ Fortaleza CE Fortaleza CE Ananindeua PA Fortaleza CE Fortaleza CE Rio de Janeiro RJ Fortaleza CE Nova Iguau RJ Belm PA Vitria ES Fortaleza CE Pinhais PR Salvador BA Rio de Janeiro RJ Fortaleza CE So Paulo SP Fortaleza CE Rio de Janeiro RJ Belm PA So Paulo SP Belm PA Belm PA Fortaleza CE Belo Horizonte MG S. J. dos Campos SP Curitiba PR Belm PA Mogi das Cruzes SP

PRMIO
Ouro Ouro Ouro Ouro Ouro Prata Prata Prata Prata Prata Prata Prata Prata Bronze Bronze Bronze Bronze Bronze Bronze Bronze Bronze Bronze Bronze Bronze Bronze Bronze Bronze Meno Honrosa Meno Honrosa Meno Honrosa Meno Honrosa Meno Honrosa Meno Honrosa Meno Honrosa Meno Honrosa Meno Honrosa Meno Honrosa Meno Honrosa Meno Honrosa Meno Honrosa Meno Honrosa

EUREKA! N13, 2002

59

Sociedade Brasileira de Matemtica

Resultado Nvel Universitrio


NOME
Emanuel Augusto de Souza Carneiro Carlos Yuzo Shine Daniel Massaki Yamamoto Fabrcio Siqueira Benevides Digo Veloso Ucha Frederico Vale Giro Bruno Fernandes Cerqueira Leite Marcio Afonso Assad Cohen Lucas Heitzmann Gabrielli Christian Iveson Daniel Nobuo Uno Giuliano Boava Vincius Jos Fortuna Leonardo Augusto Zo Leandro de Mattos Ferreira Rodrigo Villard Milet Tertuliano Franco Santos Franco Aleksander Medella Campos da Silva Arnaldo Joo do Nascimento Junior Artur Radoman de Oliveira Bruno Germano Borics Thiago Afonso de Andr Juliana Abrantes Freire Felipe Duarte Cardozo de Pina Fernando Prado Rocha Camilo Marcantonio Junior Dlio Matos Leite de Carvalho e Silva Diogo Diniz Pereira da Silva e Silva Ilan Lobel Rafael Pellizzer Soares Anderson Rodrigues Ferreira Rafael de Freitas Lemos Bruno Martins Reboredo Daniele Vras de Andrade

CIDADE ESTADO
Fortaleza CE So Paulo SP So Paulo SP Fortaleza CE Teresina PI Fortaleza CE So Paulo SP Rio de Janeiro RJ So Paulo SP So Paulo SP So Paulo SP Florianpolis SC Campinas SP Nilpolis RJ Rio de Janeiro RJ Rio de Janeiro RJ Salvador BA Rio de Janeiro RJ Duque de Caxias RJ Rio de Janeiro RJ Rio de Janeiro RJ So Paulo SP Rio de Janeiro RJ Rio de Janeiro RJ Goinia GO Rio de Janeiro RJ Rio de Janeiro RJ Campina Grande PB Rio de Janeiro RJ Jundia SP Rio de Janeiro RJ S. J. dos Campos SP Rio de Janeiro RJ Rio de Janeiro RJ

PRMIO
Ouro Ouro Ouro Prata Prata Prata Prata Prata Prata Bronze Bronze Bronze Bronze Bronze Bronze Bronze Bronze Bronze Bronze Bronze Bronze Bronze Bronze Bronze Meno Honrosa Meno Honrosa Meno Honrosa Meno Honrosa Meno Honrosa Meno Honrosa Meno Honrosa Meno Honrosa Meno Honrosa Meno Honrosa

EUREKA! N13, 2002

60

Sociedade Brasileira de Matemtica

AGENDA OLMPICA
XXIV OLIMPADA BRASILEIRA DE MATEMTICA NVEIS 1, 2 e 3 Primeira Fase Sbado, 8 de junho de 2002 Segunda Fase Sbado, 14 de setembro de 2002 Terceira Fase Sbado, 19 de outubro de 2002 (nveis 1, 2 e 3) Domingo, 20 de outubro de 2002 (nveis 2 e 3 - segundo dia de prova). NVEL UNIVERSITRIO Primeira Fase Sbado, 14 de setembro de 2002 Segunda Fase Sbado, 19 e Domingo, 20 de outubro de 2002

VIII OLIMPADA DE MAIO maio de 2002

XIII OLIMPADA DE MATEMTICA DO CONE SUL 22 a 28 de junho de 2002 Fortaleza CE, Brasil

XLIII OLIMPADA INTERNACIONAL DE MATEMTICA 18 a 31 de julho de 2002 Glasgow, Reino Unido

XVII OLIMPADA IBEROAMERICANA DE MATEMTICA 30 de setembro a 5 de outubro de 2002 El Salvador

V OLIMPADA IBEROAMERICANA DE MATEMTICA UNIVERSITRIA outubro de 2002

EUREKA! N13, 2002

61

Sociedade Brasileira de Matemtica

COORDENADORES REGIONAIS
Amarsio da Silva Arajo Alberto Hassen Raad Benedito Tadeu Vasconcelos Freire Carlos Frederico Borges Palmeira Claudio Arconcher Claus Haetinger Cleonor Crescncio das Neves lio Mega Rosngela Souza Florncio Ferreira Guimares Filho Gisele de Arajo Prateado Gusmo Ivanilde Fernandes Saad Jacqueline Fabiola Rojas Arancibia Joo Bencio de Melo Neto Joo Francisco Melo Libonati Irene Nakaoka Jos Carlos Pinto Leivas Jos Cloves Saraiva Jos Gaspar Ruas Filho Jos Luiz Rosas Pinho Jos Vieira Alves Marcelo Rufino de Oliveira Licio Hernandes Bezerra Luzinalva Miranda de Amorim Marcondes Cavalcante Frana Pablo Rodrigo Ganassim Reinaldo Gen Ichiro Arakaki Ricardo Amorim Roberto Vizeu Barros Srgio Cludio Ramos Silvio de Barros Melo Tadeu Ferreira Gomes Toms Menndez Rodrigues Valdenberg Arajo da Silva Wagner Pereira Lopes (UFV) (UFJF) (UFRN) (PUC-Rio) (Colgio Leonardo da Vinci) (UNIVATES) (UTAM) (Colgio Etapa) (Colgio Singular) (UFES) (UFGO) (UC. Dom Bosco) (UFPB) (UFPI) (Grupo Educacional Ideal) (UEM) (UFRG) (UFMA) (ICMC-USP) (UFSC) (UFPB) (Grupo Educacional Ideal) (UFSC) (UFBA) (UFC) (Liceu Terras do Engenho) (INPE) (Centro Educacional Logos) (Colgio Acae) (IM-UFRGS) (UFPE) (UEBA) (U. Federal de Rondnia) (U. Federal de Sergipe) (Escola Tcnica Federal de Gois) Viosa MG Juiz de Fora MG Natal RN Rio de Janeiro RJ Jundia SP Lajeado RS Manaus AM So Paulo SP Santo Andr SP Vitria ES Goinia GO Campo Grande MS Joo Pessoa PB Teresina PI Belm PA Maring PR Rio Grande RS So Luis MA So Carlos SP Florianpolis SC Campina Grande PB Belm PA Florianpolis SC Salvador BA Fortaleza CE Piracicaba SP SJ dos Campos SP Nova Iguau RJ Volta Redonda RJ Porto Alegre RS Recife PE Juazeiro BA Porto Velho RO So Cristovo SE Jata GO

EUREKA! N13, 2002

62

CONTEDO

AOS LEITORES VIII OLIMPADA DE MAIO Enunciados e Resultado Brasileiro XIII OLIMPADA DE MATEMTICA DO CONE SUL Enunciados, Solues e Resultado Brasileiro XLIII OLIMPADA INTERNACIONAL DE MATEMTICA Enunciados e Resultado Brasileiro

2 3

17

ARTIGOS
MUROS, PRDIOS E ESCADAS Ccero de Oliveira Holmer INTEIROS DE GAUSS E INTEIROS DE EISENSTEIN Guilherme Fujiwara SEQNCIAS ARITMTICO-GEOMTRICAS Jos Paulo Carneiro & Carlos Gustavo Moreira O PRINCPIO DA INVARINCIA Marcelo Rufino de Oliveira TORNEIO DAS CIDADES Provas OLIMPADAS AO REDOR DO MUNDO SOLUES DE PROBLEMAS PROPOSTOS PROBLEMAS PROPOSTOS AGENDA OLMPICA COORDENADORES REGIONAIS 19

23

32

35

43

48 53 59 61 62

Sociedade Brasileira de Matemtica

AOS LEITORES
com grande alegria que anunciamos que, mais uma vez, os 6 estudantes da equipe brasileira obtiveram medalhas na IMO. Isto mostra a nossa grande evoluo com a nova OBM e as atividades extras que apareceram com ela: a Eureka, a Semana Olmpica, as semanas de treinamento antes das competies internacionais. Mas o que nos traz maior satisfao saber que esses 6 jovens so a ponta de um iceberg. Basta observar o nmero de pessoas que resolvem os vrios problemas que trazemos a cada nova edio da Eureka. So estudantes, professores, profissionais liberais, enfim, amantes da Matemtica que provam que no nosso pas h muitas pessoas de boa vontade e de grande competncia. Falando mais um pouco sobre problemas, nesse nmero publicamos algumas provas do Torneio das Cidades, uma competio que se caracteriza pela originalidade de suas questes, algumas das quais j entraram para o folclore matemtico. Ela possui duas modalidades, Snior (2a e 3a sries EM) e Jnior (8a srie EF e 1a srie EM); nveis O (iniciante) e A (avanado). Para encerrar, os agradecimentos. Mais uma vez, o professor Carlos Shine e os estudantes Alex Lopes, Felipe de Souza, Henry Hsu, Rodrigo Yamashita e Guilherme Fujiwara fizeram uma leitura cuidadosa das verses prvias desta edio.

Os editores.

EUREKA! N14, 2002

Sociedade Brasileira de Matemtica

VIII OLIMPADA DE MAIO


Enunciados e Resultado Brasileiro
PRIMEIRO NVEL PROBLEMA 1

Um grupo de homens, alguns dos quais acompanhados pelas esposas, gastaram 1000 dlares num hotel. Cada homem gastou 19 dlares e cada mulher, 13 dlares. Determine quantas mulheres e quantos homens estavam no hotel.
PROBLEMA 2

Uma folha de papel retangular (branca de um lado e cinza do outro) foi dobrada trs vezes, como mostra a figura abaixo: 2 3 1 O retngulo 1, que ficou da cor branca aps a primeira dobra, tem 20cm a mais de permetro que o retngulo 2, que ficou branco aps a segunda dobra, e este por sua vez tem 16cm a mais de permetro que o retngulo 3, que ficou branco aps a terceira dobra. Determine a rea da folha.
PROBLEMA 3

Mustaf comprou um tapete. O vendedor mediu o tapete com uma rgua que supostamente media um metro. Como o resultado foi que o tapete tinha 30 metros de largura e 20 metros de comprimento, o vendedor cobrou 120000 rupias. Quando Mustaf chegou a sua casa mediu novamente o tapete e percebeu que o vendedor tinha cobrado 9408 rupias a mais. Quantos centmetros mede a rgua usada pelo vendedor?
PROBLEMA 4

Num banco s o diretor conhece o segredo do cofre, que um nmero de cinco dgitos. Para proteger este segredo so dados a cada um dos dez empregados do banco um nmero de cinco dgitos. Cada um destes nmeros tem numa das cinco posies o mesmo dgito que o segredo e nas outras quatro posies um dgito diferente do que tem o segredo nesse lugar. Os nmeros de proteo so: 07344, 14098, 27356, 36429, 45374, 52207, 63822, 70558, 85237, 97665. Qual o segredo do cofre?
EUREKA! N14, 2002

Sociedade Brasileira de Matemtica

Encontre o mximo nmero de caixinhas de 3 5 7 que podem ser colocadas dentro de uma caixa de 11 35 39. Para o nmero encontrado, indique como colocar essa quantidade de caixinhas dentro da caixa.
SEGUNDO NVEL PROBLEMA 1

PROBLEMA 5

Utilizando cubinhos brancos de lado 1 foi montado um prisma (sem buracos). As faces do prisma foram pintadas de preto. Sabe-se que os cubinhos que ficaram com exatamente 4 faces brancas so 20 no total. Determine quais podem ser as dimenses do prisma. Encontre todas as possibilidades.
PROBLEMA 2

Seja k um nmero inteiro positivo fixo, k 10. Dada uma lista de dez nmeros, a operao permitida : escolher k nmeros da lista, e somar 1 a cada um deles. Obtm-se assim uma nova lista de dez nmeros. Se inicialmente temos a lista 1, 2, 3, 4, 5, 6, 7, 8, 9, 10, determine os valores de k para os quais possvel, mediante uma seqncia de operaes permitidas, obter uma lista que tenha os dez nmeros iguais. Indique a seqncia para cada caso. Num tringulo ABC, retngulo em A e issceles, seja D um ponto do lado AC (D A e D C) e seja E o ponto do prolongamento do lado BA tal que o tringulo ADE issceles. Se P o ponto mdio do segmento BD, R o ponto mdio do segmento CE e Q o ponto onde se cortam as retas ED e BC, demonstre que o quadriltero ARQP um quadrado.
PROBLEMA 4 PROBLEMA 3

Os vrtices de um polgono regular de 2002 lados esto numerados de 1 a 2002, no sentido horrio. Dado um inteiro n, 1 n 2002, pinta-se de azul o vrtice n, logo, seguindo o sentido horrio, contam-se n vrtices comeando no seguinte de n, e pinta-se de azul o nmero n. E assim sucessivamente, a partir do vrtice que segue ao ltimo vrtice que h sido pintado, contam-se n vrtices, pintados ou sem pintar, e o nmero n pintado de azul. Quando o vrtice que tem que ser pintado j azul, o processo pra. Denotamos P(n) ao conjunto de vrtices azuis que se obtm com este procedimento quando se comea pelo vrtice n. Por exemplo, P(364) est formado pelos vrtices 364, 728, 1092, 1456, 1820, 182, 546, 910, 1274, 1638 e 2002. Determine todos os inteiros n, 1 n 2002, tais que P(n) tem exatamente 14 vrtices.
EUREKA! N14, 2002

Sociedade Brasileira de Matemtica

PROBLEMA 5

Dados x e y inteiros positivos, consideramos um quadriculado de x y, que tem pintados de vermelho os (x + 1) (y + 1) pontos que so vrtices de quadradinhos. Inicialmente h uma formiga em cada um dos pontos vermelhos. Num instante dado, todas as formigas comeam a caminhar pelas linhas do quadriculado, todas com a mesma velocidade. Cada vez que chegam num ponto vermelho, giram 90 em alguma direo. Determine todos os valores de x e y para os quais possvel que as formigas continuem movendo-se indefinidamente de maneira que em nenhum momento h duas ou mais formigas num mesmo ponto vermelho. (No interessam as possveis coincidncias em pontos das linhas do quadriculado que no so vermelhos.)
RESULTADO BRASILEIRO PRIMEIRO NVEL (AT 13 Anos)
Eduardo Fischer Pedro Nogueira Machado Andr Mrcio de Lima Curvello Katja Stephanie Ried Enzo Haruo Hiraoka Moriyama Anderson Gleryston Silva Sousa Mariana Nasser Brolezzi Arthur Rodrigues de Oliveira Sobral Cssio Kendi Takamori Diogo Bonfim Moraes Morant de Holanda Encantado RS Rio de Janeiro RJ Goinia GO Valinhos SP So Paulo SP Campina Grande PB Santo Andr SP S. Jos dos Campos SP S. Jos dos Campos SP Rio de Janeiro RJ Medalha de Ouro Medalha de Prata Medalha de Prata Medalha de Bronze Medalha de Bronze Medalha de Bronze Medalha de Bronze Meno Honrosa Meno Honrosa Meno Honrosa

SEGUNDO NVEL (AT 15 Anos)


Fbio Dias Moreira Guilherme Rodrigues Salerno Telmo Luis Correa Jnior Thiago Costa Leite Santos Andr Rodrigues Salerno Henry Wei Cheng Hsu Rodrigo Aguiar Pinheiro Rafael Marini Silva Larissa Rodrigues Ribeiro Douglas Bokliang Ang Cunha Rio de Janeiro RJ Goinia GO Santo Andr SP So Paulo SP Goinia GO So Paulo SP Fortaleza CE Vila Velha ES Fortaleza CE S. Jos dos Campos SP Medalha de Ouro Medalha de Prata Medalha de Prata Medalha de Bronze Medalha de Bronze Medalha de Bronze Medalha de Bronze Meno Honrosa Meno Honrosa Meno Honrosa

EUREKA! N14, 2002

Sociedade Brasileira de Matemtica

XIII OLIMPADA DE MATEMTICA DO CONE SUL


Enunciados, Solues e Resultado Brasileiro A XIII Olimpada de Matemtica do Cone Sul foi realizada na cidade de Fortaleza, Cear no perodo de 22 a 28 de junho de 2002. A equipe brasileira foi liderada pelos professores Yoshiharu Kohayakawa (So Paulo SP) e Luciano Guimares Castro (Rio de Janeiro RJ). O Resultado da Equipe Brasileira BRA 1 BRA 2 BRA 3 BRA 4 Alex Corra Abreu Israel Dourado Carrah Larissa Cavalcante Queiroz de Lima Rafael Daigo Hirama Ouro Bronze Ouro Ouro

PRIMEIRO DIA DURAO: 4 horas e meia. PROBLEMA 1:

Os alunos da turma de Pedro praticam a soma e a multiplicao de nmeros inteiros. A professora escreve os nmeros de 1 a 9 em nove fichas, uma para cada nmero, e as coloca em uma urna. Pedro retira trs fichas e deve calcular a soma e o produto dos trs nmeros correspondentes. Ana e Julin fazem o mesmo, esvaziando a urna. Pedro informa professora que retirou trs nmeros consecutivos cujo produto 5 vezes a soma. Ana informa que no tem nenhum nmero primo, mas sim dois consecutivos e que o produto desses trs nmeros 4 vezes a soma dos mesmos. Quais nmeros retirou Julin?
SOLUO DE ISRAEL FRANKLIM DOURADO CARRAH (FORTALEZA CE):

Diremos que Pedro escolheu os nmeros P1, P1 + 1 e P1 + 2, Ana retirou os nmeros A1, A2 e A3. Logo, temos que: P1 ( P1 + 1) ( P1 + 2) = 5 ( P1 + P1 + 1 + P1 + 2) = 5 (3P1 + 3) = 3 5 ( P1 + 1) e como (P1 + 1) um nmero positivo P1 ( P1 + 2) = 3 5 = 15 P1 e ( P1 + 2) so divisores de 15 (obviamente P1 + 2 > P1). Assim, temos duas possibilidades: P1 = 1 1a . Absurdo! P1 + 2 = 15 P1 = 13 = 1 P1 = 3 2 a . Ok! P1 + 2 = 5 P1 = 3 Portanto, Pedro escolheu os nmeros 3, 4 e 5.
EUREKA! N14, 2002

Sociedade Brasileira de Matemtica

Como Ana escolheu 3 nmeros que no so primos os possveis nmeros retirados por Ana so 1, 6, 8 e 9. (pois 2 e 7 so nmeros primos e 3, 4 e 5 so nmeros que j foram retirados por Pedro.) Mas temos tambm que Ana retirou dois nmeros consecutivos e dentre (1, 6, 8 e 9) os nicos dois nmeros consecutivos so 8 e 9 Ana escolheu os nmeros 8 e 9. Assim: A2 = 8, A3 = 9 e A1 8 9 = 4 (A1 + 8 + 9) A1 18 = A1 + 17 17A1 = 17 A1 = 1. Logo, Ana escolheu os nmeros 1, 8 e 9. Portanto, Julin retirou os nmeros restantes: 2, 6 e 7.
PROBLEMA 2:

De um tringulo ABC, retngulo em A, conhecemos: o ponto T de tangncia da circunferncia inscrita em ABC com a hipotenusa BC, o ponto D de interseo da com o lado AC e o ponto E de interseo da bissetriz bissetriz interna do ngulo B com o lado AB. Descreva uma construo com rgua e interna do ngulo C compasso para obter os pontos A, B e C. Justifique.
SOLUO DE LARISSA CAVALCANTE QUEIROZ DE LIMA (FORTALEZA CE):
A . D N E

BC = a, AC = b, AB = c; p =

. W

CT = p c; BT = p b sejam Y tal que EY BC e W tal que DW BC


a +b+c 2

. Y

EC comum Y (CE bissetriz) * EYC AEC AC E = EC CAE = EYC = 90


AE = EY AC = CY = b

BD comum D = 90 * DWB ADB BW D = BA ABD = DBW ( DB bissetriz) DW = AD AB = WB = c


EUREKA! N14, 2002

Sociedade Brasileira de Matemtica

* WT = WB TB = c (p b) = (c + b + a) a p = 2p p a = p a * YT = YC CT = b (p c) = c + b p = p a Y = WY E = 90 . T ponto mdio de WY. *DEYW um trapzio retngulo DW

Construo do ABC. Dados D e E, fcil obter com rgua e compasso o ponto N, ponto mdio de DE. Sabemos que os pontos D, E, Y e W do ABC formam um trapzio retngulo, sendo T o ponto mdio de YW NT ser base mdia e NT//EY//DW Y = 90 NTBC Conhecemos j a reta NT, com compasso, marcamos NT N ' NT ; N ' T = TN . A mediatriz de NN' perpendicular a NN' em T, portanto coincide com a reta BC conhecemos agora a reta BC . T = 90 para encontrar Y, encontramos M1, ponto mdio de ET e contrumos * EY com compasso a circunferncia 1 de centro M1 passando por E e T. Onde 1 encontrar a reta BC ser o ponto Y (note que ser no segundo ponto de encontro com BC , o primeiro T). Analogamente, encontramos o ponto W no encontro da circunfrncia 2 de dimetro DT e da reta BC . Dessa maneira, encontramos EY e DW . Com o compasso, encontramos 3 de raio EY e centro E, e 4 de raio DW e centro D. 3 4 = { A, A' } A ser o ponto que est "acima"de DE ( supondo o ponto T "abaixo"de DE). Encontramos ento as retas AE e AD . B = AE BC e C = AD BC Encontramos ento ABC obs: Se Y coincidir com T, temos que W coincidir com T e p a = 0 ou seja, a+b+c a = 0 a + b + c 2a = 0 b + c = a Absurdo! 2 Y T e W T.
PROBLEMA 3:

Arnaldo e Bernardo jogam uma Super Batalha Naval. Cada um tem um tabuleiro n x n. Arnaldo coloca barcos em seu tabuleiro (pelo menos um mas no se sabe quantos). Cada barco ocupa as n casas de uma linha ou de uma coluna e os barcos no podem se superpor nem ter um lado comum. Bernardo marca m casas (representando tiros) em seu tabuleiro. Depois que Bernardo marcou as m casas, Arnaldo diz quais dentre elas correspondem a posies ocupadas por barcos. Bernardo ganha se, a seguir, descobre quais so as posies de todos os barcos de
EUREKA! N14, 2002

Sociedade Brasileira de Matemtica

Arnaldo. Determine o menor valor de m para o qual Bernardo pode garantir sua vitria.
SOLUO DE ALEX CORRA ABREU (NITERI RJ):

Suponha n 4 (I) Primeiro vemos que obviamente tem que marcar casas em todas as linhas e colunas, pois se uma linha no tiver nenhuma casa marcada obviamente podemos ter um barco em uma linha no adjacente portanto Bernardo no saber se ali tem ou no um barco. De modo anlogo para as colunas. (II) Agora suponha que uma casa est marcada, se na linha e na coluna dessa casa no tiver mais nenhuma outra casa marcada, pode vir a calhar de arnaldo ter colocado apenas um barco no tabuleiro e ser exatamente nessa linha, portanto Bernardo no sabe se o barco est na vertical ou na horizontal pois s vai ter uma casa onde sabe que o barco est. Ento para cada casa marcada existe outra na linha ou na coluna. Considere agora que Arnaldo marcou um barco na linha i se uma casa marcada na linha i, para identificar se o barco est na horizontal ou na vertical precisamos de mais uma casa adjacente. Se esta estiver pintada o barco est na vertical (baseado no desenho) e na horizontal caso contrrio. O problema consiste em pintar blocos 1 k tais que cada linha e cada coluna tem interseo com um deles, agora se tivermos um 1 k se k > 3 podemos dividir e de fato apenas melhoramos as coisas ento os blocos so 1 2 e 1 3.

k1 k k2

M(n) o mnimo procurado para um tabuleiro n n.

EUREKA! N14, 2002

Sociedade Brasileira de Matemtica


3 1 A

Se n 3 retire as 3 primeiras linas e as 3 primeiras colunas ento m(n) = m(3) + m (n 3) pois precisamos de m(n 3) pois se no tiver nenhum barco nas 6 fileiras que saram e m(3) se no tiver nas que ficaram pois se considerarmos s a regio A. temos que colocar 3 em cada no mnimo, o que d mais 6 contra 4 de m(3) (tambm porque podemos diminuir as interseces)

m(3k ) = km(3) = 4k m(3k + 1) = (k 1)m(3) + m(4) = 4k 4 + 6 = 4k + 2 m(3k + 2) = 4(k 1) + m(5) = 4k + 3 fcil ver que m(3) = 4 pois 3 obviamente no e m(4) = 6, m (5) = 7 como ao lado.
SEGUNDO DIA DURAO: 4 horas e meia. PROBLEMA 4:

Seja ABCD um quadriltero convexo tal que suas diagonais AC e BD so perpendiculares. Seja P a interseo de AC e BD e seja M o ponto mdio de AB. Mostre que o quadriltero ABCD inscritvel se, e somente se, as retas PM e CD so perpendiculares.
SOLUO DE ISRAEL FRANKLIM DOURADO CARRAH (FORTALEZA CE):

Primeiramente vejamos quando PM e CD so perpendiculares


B A 90 2 M

90 P

90

.. K

EUREKA! N14, 2002

10

Sociedade Brasileira de Matemtica

Seja MP CD = {K }. Como no ABP , retngulo em P, M o ponto mdio da hipotenusa AB PM = MA = MB. Assim, seja

D = MP B = AM P = 2 AB
A = 90 CP K = AP M = 90 e como MP D = . Logo, AB D = C = 90 PC D = AC PK inscritvel! Vejamos agora se ABCD inscritvel:

O quadriltero ABCD

B M A 90
90 P

90

Do mesmo modo como M o ponto mdio da hipotenusa AB do tringulo retngulo APB PM = MA = MB. P = MP D = BA A = 90 CP K = 90 e como ABCD Logo, se AB D = AB D = PK C = 180 (90 + ) = 90 MPCD . inscritvel AC Portanto, ABCD inscritvel PMCD.
PROBLEMA 5:

Considere o conjunto A = {1, 2,, n}. Para cada inteiro k, seja rk a maior quantidade de elementos distintos de A que podemos escolher de maneira que a diferena entre dois nmeros escolhidos seja sempre diferente de k. Determine o maior valor n possvel de rk, onde 1 k . 2
EUREKA! N14, 2002

11

Sociedade Brasileira de Matemtica

SOLUO DE RAFAEL DAIGO HIRAMA (CAMPINAS SP):

Vamos analizar casos pequenos: 1, 2, 3, 4, 5, 6, 7, 8 k = 1 rk = 4 k = 2 rk = 4 k = 3 rk = 5 k = 4 rk = 4 1, 2, 3, 4, 5, 6, 7, 8, 9, 10 k = 1 rk = 5 k = 2 rk = 6 k = 3 rk = 6 k = 4 rk = 6 k = 5 rk = 5 Isso me deu idia para um lema!

1, 2, 3, 4, 5, 6, 7, 8, 9 k = 1 k=2 k=3 k=4

rk = 5 rk = 5 rk = 6 rk = 5

m + 1 Lema: Para n = m k (m inteiro positivo maior que 1) temos que rk = k 2 Prova: Podemos dividir os nmeros em casas de congruncias mdulo k. Por exemplo o 0. Seus componentes sero k, 2k, 3k,...,mk. Como a diferena entre dois deles deve ser diferente de k, no podemos escolher dois nmeros consecutivos nessa seqncia. Sempre deve haver um "ausente" ou mais entre dois "presentes". Para m par temos m m + 1 escolhidos teremos escolhidos pois caso tenhamos que s poderemos ter 2 2 m 1 no escolhidos, mas para separar os escolhidos (para no serem consecutivos) 2 m deveramos ter pelo menos no escolhidos. Absurdo. 2 m +1 m +1 1 no escolhidos que podem ser espaados pelos Para m mpar teremos 2 2 m +1 m +1 + 1 deveramos ter escolhidos. Do mesmo modo, para espaos mas s 2 2 m +1 m 3 + 1 = teriam sobrado m no escolhidos que no so suficientes. 2 2
EUREKA! N14, 2002

12

Sociedade Brasileira de Matemtica

Como temos k casas de congruncia com m nmeros cada e pelo fato de m +1 se m mpar 2 m + 1 m + 1 temos que rk = k . 2 = m 2 se m par 2 Agora precisamos ver como transformar o lema em algo que seja mais verstil ao nosso problema, ou seja, no devemos ter de usar o fato n = m k. 2n Analizando mais casos pequenos estou conjecturando que o rk mximo . 3 2n Vamos provar que rk para todo k. 3 2n Suponha o contrrio, que h rk > . Logo vamos provar primeiro que em uma casa 3 2 de congruncia mdulo k com j termos o aproveitamento mximo de termos do 3 total j de termos. Temos a seguinte regra: se x foram escolhidos ento pelo menos x x 2 1 no podem ter sido. Logo devemos provar que 2 j 3 x. Sabemos que j 3 Para x 2 j est provado. Mas se escolhermos x = 1 necessariamente j = 2 pois se j = 1, ou seja, se h somente um nmero entre 1 e n com congruncia mdulo k n significa que 2k > n, ento k > o que contradiz o enunciado. 2 1 2 Nesse caso o aproveitamento que menor que . 2 3 Chamando o aproveitamento para a casa de congruncia i de ai e o nmero de termos nessa casa de congruncia de bi temos 2 2 rk = a1b1 + a 2 b2 + ... + a k bk (b1 + b2 + b3 + ... + bk ) = n 3 3

j x + x 1, portanto vale 2 j 3x 2[x + ( x 1)] 3x 4x 2 3x x 2.

( pois b1 + b2 + ... + bk = n) portanto rk

2 n 3

2 como rk inteiro rk mximo n . 3 Falta provar a existncia de tal rk. Se faz assim:

EUREKA! N14, 2002

13

Sociedade Brasileira de Matemtica

n Divide-se n por 3 e arredonda-o para cima. Esse o nosso k. k = . obvio que 3 n para n 2, k . ( se n =1 k no existe) 2 2 Se n = 3k ento rk = n e mximo 3 Se n = 3k 1 ento teramos, em relao ao caso acima a perda de um termo, a n + 1 = 3k 2(n + 1) 2 1 2 1 = n = n escolher. O 3k rk mximo rk = 3 3 3 3 Se n = 3k 2 do mesmo modo perdemos 2 em relao ao primeiro caso n + 2 = 3k 2(n + 2) 2 2 2 2 = n = n . rk = 3 3 3 3 Portanto ao montar o caso n = 3k escolhe-se: 2 1, 2, 3,..., k, 2k, 2k + 1,..., 3k totalizando 2k termos n 3 O caso n = 3k 1 e n = 3k 2 retira-se o 3k; e o 3k e o 3k 1 respectivamente. 1 n = 1 no podemos ter k 1 k 2 Resposta: 2n n 1 rk mximo 3 Obs: x maior inteiro menor ou igual a x x 1 < x x x inteiro x menor inteiro maior ou igual a x x x < x + 1 x inteiro.
PROBLEMA 6:

Dizemos que um inteiro n, n, > 1, ensolarado se ele divisvel pela soma dos seus fatores primos. Por exemplo, 90 ensolarado pois 90 = 2 3 2 5 e 2 + 3 + 5 = 10 divide 90. Mostre que existe um nmero ensolarado com pelo menos 10 2002 fatores primos distintos.

EUREKA! N14, 2002

14

Sociedade Brasileira de Matemtica

SOLUO DE RAFAEL DAIGO HIRAMA (CAMPINAS SP):

Vamos ver casos pequenos: 2 3 5 = 30 e ensolarado pois 2 + 3 + 5 = 10|30 Olha s que interessante: se escolhemos alguns nmeros primos e a soma deles puder ser escrita como um produto qualquer deles o produto de todos esses primos vezes a soma deles um nmero ensolarado, alis o mmc ensolarado. Vamos ver at onde isso vai: 2 + 3 + 5 + 7 = 17 droga! 17 primo, vamos adaptar 2 + 3 + 5 + 7 + 17 = 34 e 34 = 2 17 mmc (34, 2 3 5 7 17) = 2 3 5 7 17 que ensolarado 2 + 3 + 5 + 7 + 11 = 28 e 28 = 22 7 mmc ( 28, 2 3 5 7 11) = 22 3 5 7 11 que ensolarado 2 + 3 + 5 + 7 + 11 + 13 = 41 2 + 3 + 5 + 7 + 11 + 13 + 41 = 82 e 82 = 2 41 2 3 5 7 11 13 41 ensolarado. 1 2 n Prova geral: p1 + p2 + p3 + p4 + ... + pn = x, x = p1 (ou seja, no tem fatores p 2 ... p n primos alm dos p1 a pn, mas i pode ser 0)
2 n = y. Como x | y e y s tem fatores p1, p2, ..., pn, mmc(p1p2p3...pn), x) = p11 p 2 ... p n y ensolarado. Percebendo os meus testes podemos ver um modo de adaptar se tivermos a soma desses primos um nmero primo diferente dos anteriores. E se tivermos mais? 1 2 3 4 n n +1 n + 2 n + 3 p1 < p 2 < p3 < ... < p n e p1 + p 2 + ... + p n = p1 p 2 p3 p 4 ... p n p n +1 p n + 2 p n + 3 ... 1 2 3 n n +1 1 n + 2 p1 + p 2 + ... + p n + p n +1 = p n +1 ( p1 p 2 p3 ... p n p n +1 p n + 2 ... + 1)

este nmero com certeza menor que a soma inicial logo ter um limite para seus fatores primos. Fazendo: 2 3 n +1 n + 2 n + 3 p1 + p2 + ... + pn + pn +1 = p11 p2 p3 ... pn +1 pn + 2 pn + 3 ...
2 3 4 n +1 n + 2 1 n +3 p1 + p2 + ... + pn + pn +1 + pn + 2 = pn + 2 ( p1 1 p2 p3 p4 ... pn pn + 3 ... + 1) + 1 pn + 2 Calma, podemos evitar tudo isso se escolhermos os n primeiros primos: 2 + 3 + 5 +...+ pn = x n pn. Se tivesem dois primos pi e pj em x tal que i, j > n teremos pipj x n pn. Mas perceba que pk > k para todo k  (isso acontece porque a seqncia dos k percorre todos os naturais enquanto a dos pk "pula" vrios naturais). Continuando Vamos provar que n pn < pi pj, no o contrrio. pi p j > p n p n

EUREKA! N14, 2002

15

Sociedade Brasileira de Matemtica

pi p j > n p n Logo em x s pode haver um fator primo diferente dos 2, 3, 5,..., pn. Usaremos pi e do mesmo modo vemos que x no divisvel duas vezes por pi ( s fazer j = i) 2 + 3 + 5 +...+pn = m pi n pn n pn m pi Olhe s, como

pn < 1 porque pn < pi porque n < i, m < n s que n < pn m < pn pi

Como m e pn so inteiros m + 1 pn Agora pronto: 2 + 3 + 5 +...+ pn = m pi 2 + 3 + 5 +...+ pn + pi = (m + 1) pi mas como m + 1 pn , m + 1 pode ser escrito como produto dos primos 2, 3, 5,..., pn, n ou seja 2 + 3 + 5 + ... + p n + pi = 21 3 2 ... p n pi .
Como 2 + 3 + 5 + ... + p n + pi = 2 1 3 2 ... p n n pi
n n pi alis, mmc (21 3 2 ... p n pi ,2 3 5 ... pn pi ) = 2 max(1 ,1) 3max( 2 ,1)... p n como m + 1 pn n= 0 ou 1, ou seja max(n , 1) = 1 Como queremos 102002 primos distintos, se 2 + 3 + ... + p10 2002 no for fatorvel nos
max( ,1)

primos 2,3,..., p102002 ele ser da forma m pi , i > 10 m < p102002.

os 10 2002 primeiros primos 2002

Com isso 2 + 3 + ... + p10 2002 + pi = (m + 1) pi , que fatorvel em 2,3,5,..., p102002 , pi , j que m + 1 p102002 . pi , com 2 1 3 2 ... p1010 Logo pelo menos um entre 21 3 2 ... p1010 2002 ou 2002
2002 2002

pi 2+3+...+ 10 2002 , com i e i suficientemente grandes so ensolarados, ou seja, h um nmero ensolarado com 102002 ou 102002 + 1(ou ambos) fatores primos distintos Alias, esse mtodo prova que para todo t inteiro positivo existe pelo menos um nmero ensolarado com t fatores primos ou pelo menos um nmero ensolarado com t + 1 fatores primos (ou ambos). x, se x y Obs. max( x, y ) = y, se y x
p

EUREKA! N14, 2002

16

Sociedade Brasileira de Matemtica

XLIII OLIMPADA INTERNACIONAL DE MATEMTICA


Enunciados e Resultado Brasileiro A LXIII Olimpada Internacional de Matemtica foi realizada na cidade de Glasgow, Reino Unido no perodo de 18 a 31 de julho de 2002. A equipe brasileira foi liderada pelos professores Edmilson Motta (So Paulo SP) e Ralph Costa Teixeira (Niteri RJ). O Resultado da Equipe Brasileira BRA 1 BRA 2 BRA 3 BRA 4 BRA 5 BRA 6 Alex Corra Abreu Larissa Cavalcante Queiroz de Lima Guilherme Issao Camarinha Fujiwara Yuri Gomes Lima Davi Mximo Alexandrino Nogueira Thiago da Silva Sobral Bronze Prata Bronze Bronze Bronze Bronze

PRIMEIRO DIA DURAO: 4 horas e meia.

PROBLEMA 1

Seja n um inteiro positivo. Seja T o conjunto de pontos (x; y) no plano onde x e y so inteiros no negativos e x + y < n. Cada ponto de T pintado de vermelho ou azul. Se um ponto (x; y) vermelho, ento todos os pontos (x'; y') com x' x e y' y tambm so. Um conjunto X um conjunto de n pontos azuis com abcissas todas distintas, e um conjunto Y um conjunto de n pontos azuis com ordenadas todas distintas. Prove que o nmero de conjuntos X igual ao nmero de conjuntos Y. Seja BC um dimetro do crculo de centro O. Seja A um ponto de tal que 0 0 < AOB < 120 0 . Seja D o ponto mdio do arco AB que no contm C. A reta que passa por O e paralela a DA encontra a reta AC em J. A mediatriz de OA corta em E e F. Prove que J o incentro do tringulo CEF.
PROBLEMA 3 PROBLEMA 2

Encontre todos os pares de inteiros m, n 3 tais que h infinitos inteiros positivos a am + a 1 inteiro. para os quais n a + a2 1

EUREKA! N14, 2002

17

Sociedade Brasileira de Matemtica

SEGUNDO DIA DURAO: 4 horas e meia.

PROBLEMA 4

Seja n inteiro maior que 1. Os divisores positivos de n so d1, d2,,dk , onde


1 = d 1 < d 2 < ... < d k = n

Seja D = d1d2 + d2d3 ++dk 1 dk. (a) Prove que D < n2. (b) Encontre todos os valores de n para os quais D um divisor de n2.
PROBLEMA 5

Encontre todas as funes f de # em # tais que ( f ( x) + f ( z ))( f ( y ) + f (t )) = f ( xy zt ) + f ( xt + yz ) para todo x, y, z, t #.


PROBLEMA 6

Sejam 1 , 2 ,..., n crculos de raio 1 no plano, onde n 3. Seus centros so O1, O2,,On, respectivamente. Suponha que no exista reta que intercepte mais que dois dos crculos. Prove que (n 1) 1 . 4 1i < j n Oi O j

EUREKA! N14, 2002

18

Sociedade Brasileira de Matemtica

MUROS, PRDIOS E ESCADAS


Ccero de Oliveira Holmer, So Paulo SP Nvel Avanado. H um clssico problema de mximos e mnimos cujo enunciado envolve um prdio (to alto quanto se queira) e um muro de altura h, uma distncia d deste prdio. Pretende-se colocar uma escada, apoiada no muro, a partir do solo e alcanando o prdio, conforme o esquema:

h d

Pergunta-se ento o seguinte: Qual o comprimento mnimo da escada? Vamos montar um modelo, considerando um tringulo retngulo ABC e um retngulo APQR inscrito neste tringulo:
C

B P A

C ) = , temos: Sendo PQ = a, QR = b e m( AB a b a. cos b sen BC = BQ + QC = . Assim, BC = f ( ) e f ' ( ) = + + sen cos sen 2 cos 2 a cos b sen Para termos BC mnimo, preciso que f ' ( ) = 0, isto , + =0 sen 2 cos 2 b sen a cos sen 3 a a (I). = = tg = 3 2 2 3 b sen cos cos b Pelo teorema de Pitgoras, BC 2 = BA2 + AC 2 BC 2 = (BP + PA) 2 + ( AR + RC) 2 =

EUREKA! N14, 2002

19

Sociedade Brasileira de Matemtica

a 2 = + (a + b tg ) . Como BC deve ser mnimo, de (I), temos: tg + b


2 2 2 a a 3 2 BC = a2 b + b a + 3 a b2 + b + a + b 3 = + = b a 3 b
2

= a 3 a b2 + 2 b 3 a2 b + b2 + a2 + 2 a 3 a b2 + b 3 a2 b =
3 3 2 3 3 2 2 2 = a2 + 3 a 3 a b2 + 3 b 3 a2 b + b2 = BC = a + b a + b . 3 3

Portanto, o comprimento mnimo da escada deve ser

3 3 h2 + d 2 . Vamos agora considerar uma situao com valores numricos (talvez voc possa aproveitar melhor o que vem a seguir tendo em mos papel, caneta e, se possvel, uma boa calculadora). A partir de um tringulo retngulo bem conhecido, de lados 3, 4 e 5, e outro tringulo semelhante, por exemplo o de lados 9, 12 e 15, podemos montar a figura:

10 m

Q 5m 3m B

9m

4m

8m

Formulamos, ento, o seguinte problema: Se o muro tem 3 metros de altura, a distncia do muro ao prdio igual a 8 metros e a escada tem 15 metros de comprimento, poderamos afirmar que a distncia do p da escada ao muro igual a 4 metros?

EUREKA! N14, 2002

20

Sociedade Brasileira de Matemtica

Vejamos: O menor comprimento possvel da escada de metros.


3 9 + 4 < 15, e isto quer dizer que h duas maneiras Pode-se verificar que distintas de posicionarmos a escada e, portanto, existem duas distncias possveis do p da escada ao muro. Vamos ento, novamente, montar um modelo:

32 + 3 82 =

( 9 + 4)
3

15

3 B

Uma soluo possvel, claro, x = 4 metros. Busquemos a outra soluo: PBQ ~ ABC , logo No BPQ temos

15 x BP BA x x+8 = = BQ = 15 BQ BC BQ x+8
2

15 x 4 3 2 x = BQ 3 x = 9 x + 16 x 152 x + 144 x + 576 = 0 x+8 Aplicando-se o algoritmo de Briot-Ruffini, obtemos:


2 2 2 2

1 1

16 20

152 72

144 144

576 0

Assim, a outra soluo raiz de x 3 + 20 x 2 72 x 144 = 0. possvel mostrar que essa equao tem duas razes reais negativas e uma raiz real positiva, que aproximadamente 4,3274534 e pode ser escrita como
EUREKA! N14, 2002

21

Sociedade Brasileira de Matemtica

1567 arccos 4 154 154 (ver por exemplo [3] para um mtodo de 5 154 cos 3 3 resoluo de equaes do terceiro e quarto graus). Assim, as possveis distncias do p da escada ao muro, so de 4 metros e de 1567 arccos 4 154 154 = 154 . cos 5 4,3274534... metros. 3 3

Referncias Bibliogrficas [1] Piskunov N., Clculo Diferencial e Integral, Tomos I e II, Ed. Mir 1977 [2] Demidovitch B., Problemas e Exerccios de Anlise Matemtica, Ed. Mir 1978 [3] Moreira, C.G., Uma soluo das equaes do terceiro e quarto graus, RPM 25, pp. 23-28.

EUREKA! N14, 2002

22

Sociedade Brasileira de Matemtica

INTEIROS DE GAUSS E INTEIROS DE EISENSTEIN


Guilherme Fujiwara, So Paulo SP Nvel Avanado. Vamos abordar nesse artigo a aritmtica de dois conjuntos de inteiros algbricos: os Inteiros de Gauss e os Inteiros de Eisenstein.
1. INTEIROS DE GAUSS

Definimos o conjunto [i] dos inteiros de Gauss como [i] = {a + bi | a, b }, onde (i2 = 1). A seguir veremos as duas coisas mais importantes de sua aritmtica, o teorema da fatorao nica e os primos. 1.1 Norma Vamos definir uma funo N: [i] chamada norma, tal que z [i], N(z) = z

z sendo z o conjugado complexo de z. Observe que como ab = a b, ento

N (a ) N (b) = a a bb = a b a b = ab ab = N (ab) , ou seja, a norma


multiplicativa. 1.2. Unidades As unidades em [i], analogamente a , so todos os elementos z [i] que possuem inverso, ou seja, que z ' [i ] tal que z z ' = 1. Segue que se z = a + bi uma unidade, ento 1 = N ( z z ' ) = N ( z ) N ( z ' ) N ( z ) = 1 a 2 + b 2 = 1 a = 1, b = 0 ou a = 0, b = 1 = 1 ou = i , e como esses quatro tem inverso, todas as unidades so 1 e i. Observe ento que x [i] unidade N ( x) = 1. 1.3. Divisibilidade Dizemos que para a, b [i], a|b (l-se a divide b) se c [i ] tal que b = ac. 1.4. Diviso Euclidiana Vamos ver como funciona a diviso euclidiana. A diviso Euclidiana a existncia de q, r [i], a, b [i], b 0 tal que a = bq + r, sendo 0 N (r ) < N (b). Para demonstr-la, basta dividir: a = x + yi, b = z + wi, onde x, y, z, w .

EUREKA! N14, 2002

23

Sociedade Brasileira de Matemtica

a x + yi x + yi z wi xz xwi + yzi ywi 2 xz + yw yz xw = = = 2 + i b z + wi z + wi z wi z 2 + w2 z + w2 z 2 + w2

Tomamos m e n como os inteiros mais prximos de respectivamente. Note que m


xz + yw
2 2

xz + yw yz xw e 2 , 2 2 z +w z + w2

z +w z +w a yz xw yz xw m+ 2 n r = a bq = b q = b 2 2 2 b z +w z +w 2 2 1 1 N (b) < N (b) N (r ) N (b) + = 2 2 2


2 2

,n

yz xw

1 Se q = (m + ni), ento: 2

1.5. Lema de Euclides A partir da diviso euclidiana podemos demonstrar o lema de Euclides, ou seja, se p um primo de Gauss (ou seja, no pode ser escrito como o produto de dois inteiros de Gauss cujas normas so maiores que 1), ento sendo a, b [i], p|ab p|a ou p|b. Para demonstr-lo, vamos fazer sucessivas divises euclidianas, sendo a0 = a e a1 = p. Seja ak + 2 o resto da diviso euclidiana de ak por ak+1. Temos ento as divises: a 0 = q1 a1 + a 2 a1 = q 2 a 2 + a 3 a 2 = q3 a3 + a 4 a n 2 = q n 1 a n 1 + a n a n 1 = q n a n + a n +1 Observe que como a k 0 N (a k +1 ) < N (a k ) , podemos tomar n tal que N(an +1) = 0, ou seja, an + 1 = 0. Logo an|an 1. Observe que a n | a k +1 e a n | a k a n | a k 1 . Logo a n | a n e a n | a n 1 , ento indutivamente,

a n | a k , k 0 k n,

particularmente

an | a0 = a

a n | a1 = p. Tomando as j + 1 primeiras equaes e realizando substituies adequadas, temos que aj = xj a1 + yj a0 = xj p +yj a; particularmente a n = x n p + y n a.
EUREKA! N14, 2002

24

Sociedade Brasileira de Matemtica

Voltando ao lema, veja que se p|a ento o lema est certo. Se p no divide a, ento, como a n | p, an|a e an = xnp + yna, ento an {1; 1; i; i} e temos:
1 ( px n b + aby n ) p | b, pois an = xn p + y n a b = an demonstrao.

p | ab, o que conclui a

1.6. Fatorao nica A fatorao nica uma das propriedades mais usadas em problemas envolvendo nmeros inteiros. Vamos prov-la para os inteiros de Gauss. Primeiramente provaremos que todo inteiro z de Gauss com norma maior que 1 pode ser escrito como o produto de um ou mais primos de Gauss. Se N(z) = 2, como 2 primo e a norma multiplicativa, ento z primo, portanto est provado. Considere N(z) > 2. Se z primo a fatorao imediata. Se z no primo, ento z = a b N(z) = N(a) N(b), onde N(a), N(b) > 1, portanto N(a), N(b) < N(z). Podemos supor, por induo, que se N(x) < N(z), ento x fatorvel. Logo a e b so fatorveis, e portanto z. Para provar que esta fatorao unica, basta considerar as duas fatoraes p1p2pn e q1q2qm . Suponha, por induo, que p1p2pn = q1q2qm, sendo uma unidade, implica que a seqncia (pi) uma permutao (a menos que sejam multiplicaes por unidades) da (qi). Se max{n; m} = 1, ento o resultado imediato. Supondo que ele vale se max{n'; m'}< max{n; m}, pelo lema de Euclides, vemos que para algum i, pn|qi. Sem perda de generalidade, i = m. Como pn e qm so primos, ento qm = 'pn, onde ' uma unidade. Logo p1p2pn = q1q2qm p1p2pn 1 = ' q1 q 2 ...q m 1 . Por induo, p1, p2,...,pn-1 uma permutao (a menos que sejam multiplicaes por unidades) de q1, q2, , qm, portanto a fatorao nica est provada. 1.7. Nmeros primos Vamos agora ver quem so os nmeros primos em Z[i]. Observe que se N() primo em , ento um primo de Gauss (pois se fatora ento N() fatora). Observe que todo primo divide N(), portanto ele deve dividir ao menos um fator primo em de N(). Se dividir ao menos dois nmeros distintos (absolutamente) x e y primos em , como sempre possvel tomar a, b tal que ax + by = 1, teramos |1, um absurdo. Logo todo primo de Gauss divide exatamente um primo inteiro positivo (e seu oposto negativo) em . Seja esse primo inteiro positivo p. Temos trs casos: Se p par, ento p = 2. Sendo = a + bi, ento a2 + b2 = 2 = 1 i , e obtemos os quatro primos 1 + i, 1 i, 1 + i e 1 i. Observe que eles so dois a dois um a multiplicao por uma unidade do outro.
EUREKA! N14, 2002

25

Sociedade Brasileira de Matemtica

x2 0 ou 1 (md. 4), ento, se existisse = c + Se p 3 (md. 4), como x di, c, d Z, 1 < N() < p2 tal que p = , facil ver que, como p um primo inteiro = c di , logo p = c2 + d2 0, 1 ou 2 (mod.4), absurdo, pois p = 4k + 3. Logo p um primo de Gauss. Se p 1 (md. 4), ento, sendo x = 1 2 ( p 1)/2, ento: ( p 1) ( p 1) x 2 1 2 ... 1 2 ... 2 2 ( p 1) ( p + 1) 1 2 ... ... ( p 2) ( p 1) 1 ( p 1) 2 2 1(md . p ) Logo p | x 2 + 1 = ( x + i)( x i). Como um primo de Gauss que divide p, ento Z, |x + i ou |x i |1, absurdo. Portanto [i] tal que p = . Seja = a + bi e = c + di, a, b, c, d Z. Como p primo em z, ento mdc(a; b) = mdc(c;d) = 1. Temos p = (a + bi)(c + di) = ac bd + (bc + ad)i. Como p Z, ento bc = ad (a = c e b = d) ou (a = c e b = d) = . Como p > 0, ento = N ( ) = p, logo primo (e e seu conjugado so nicos primos de Gauss que dividem p). Portanto vimos que os nmeros primos em [i] so: (1) O primo 1 + i e seus produtos pelas unidades. (2) Os primos p em tal que p 3 (mod. 4) e seus produtos pelas unidades. (3) Para cada primo p em + tal que p 1 (mod. 4), os primos a + bi, a bi e seus produtos pelas unidades, sendo a2 + b2 = p. 1.8. Ternas pitgoricas Agora que j vimos a aritmtica bsica dos inteiros de Gauss, vamos comear com um resultado simples e interessante. Vamos achar as solues da equao a2 + b2 = c2, sendo a, b, c . Seja m = mdc(a; b), a' = a/m e b' = b/m. Temos ento m2 (a2 + b2) = c2 m|c. Seja ento c' = c/m, temos a'2 + b'2 = c'2, mdc(a';b';c') = 1. Note que a'2 + b'2 = c'2 (a'+ b'i)(a' b'i) = c'2. Observe que se d = mdc(a' + b'i; a' b'i), ento d|2a' e d|2b' d|2. Se d no divide 1, ento d|a'2 + b'2 a' e b' so mpares, o que um absurdo, basta ver congruncia mdulo 4. Portanto d|1 a' + b'i e a' + b'i so primos entre si, logo ambos so quadrados perfeitos. Observe tambm que quaisquer a'e b' primos entre si tais que a' + b'i e a' b'i so quadrados perfeitos so solues da equao. Portanto a'e b' formam uma soluo se e somente se existem x, y, z, w tal que:

EUREKA! N14, 2002

26

Sociedade Brasileira de Matemtica

a '+b' i = ( x + yi) 2 a 'b' i = ( z + wi ) 2

a '+b' i = ( x + yi) 2 a 'b' i = ( x yi) 2

a '+b' i = ( x + yi) 2

a' = x 2 y 2 b' = 2 xy

Veja ento que a' e b' so primos entre si se e s se x e y so primos entre si. Logo as solues so a = (x2 y2) d, b = 2xy d, ou vice-versa, e conseqentemente c = (x2 + y2) d, para x, y, d , sendo x e y primos entre si. 1.9. O nmero de representaes de um inteiro como a soma de dois quadrados Provaremos agora o seguinte Teorema. Dado n  , o nmero de pares a, b tais que n = a2 + b2 igual a quatro vezes a diferena entre o nmero de divisores da forma 4k + 1 de n e o nmero de divisores da forma 4k + 3 de n. Podemos expressar n da forma: 1 1 m m k ... p n = 2 p1 k q1 ( q i ) ...q m ( q m ) Sendo pi primos de Gauss inteiros (da forma 4k + 3) e os pares de conjugados qi q i primos de Gauss (N(qi) da forma 4k + 1) e esses primos diferem dois a dois por mais do que uma multiplicao por uma unidade. Sendo n = a2 + b2 = (a + bi)(a bi), ento, pelo teorema da fatorao nica e a multiplicidade do conjugado, temos:
1 m (q 1 ) 1 1 ...q m (q m ) m m , onde 0 i i e a + bi = (1 + i ) p12 ... p k2 q1 uma unidade. Portanto o nmero de representaes de m como a soma de dois quadrados ser 0 se algum i for mpar e ser 4(1 + 1)( m + 1) se todos i forem pares, sendo o fator 4 pois h 4 escolhas possveis para a unidade. Observe agora que a fatorao de n em primos inteiros ser: k 1 N (q1 ) 1 ...N (q m ) m ... p k n = 2 p1 Onde pi sero primos da forma 4k + 3 e N(qi) sero primos da forma 4k + 1. Observe agora que um divisor mpar de n ser da forma: ak N (q1 ) b1 ...N (q m ) bm , onde a1 1 ,..., a k k , b1 1 ,..., bm m d = p1a1 ... p k Note que se a1 + + ak par, ento d da forma 4k + 1, se for mpar da forma 4k + 3. Portanto, conseguimos verificar que se algum i for mpar, o nmero de ds da forma 4k + 3 ser igual ao nmero de ds da forma 4k + 1, e se todos os i forem pares, a diferena entre esses nmeros ser ( 1 + 1)( m + 1), o que termina a demonstrao do teorema. 1 k

EUREKA! N14, 2002

27

Sociedade Brasileira de Matemtica

1.10. Problemas Problema 1. Determine todos os pares x, y tal que y3 = x2 + 1

Problema 2. Sejam x, y, z  tais que xy = z2 + 1. Prove que existem inteiros a, b, c, d tais que x = a2 + b2, y = c2 + d2 e z = ac + bd. Problema 3. Prove que existem duas seqncias inteiras (an) e (bn) infinitas e estritamente crescentes tais que ak(ak + 1) divide bk2 + 1 para todo natural k. 2. INTEIROS DE EISENSTEIN Vamos agora ver os Inteiros de Eisenstein. Definimos o conjunto [] dos inteiros 1 i 3 + , donde 2 + 2 2 + 1 = 0. Para = a + b [] definiremos a norma como N() = = a2 ab + b2. Observe que essa norma segue as mesmas propriedades da norma dos inteiros de Gauss ( inteira no negativa e multiplicativa). de Eisenstein como [] = {a + b| a, b }, sendo = 2.1. Unidades As unidades em [] so definidas como os seus elementos que possuem inverso, ou seja u, tal que u 1 tal que u u 1 = 1 N(u) = 1 u = 1, , (1 + ), e verificamos que esses quatro nmeros tem inverso, portanto u unidade se, e s se N(u) = 1. 2 Obs. Note que (1 + ) = . 2.2. Diviso Euclidiana Para provar a existncia de diviso Euclidiana entre a, b [], b 0. Sejam e tais que: a = + b Tomando q = c + d, tais que c e d so respectivamente os inteiros mais prximos de e . Portanto: r = a bq = b( + q ) N (r ) = 1 1 = N (b)(( c) 2 ( c)( d ) + ( d ) 2 ) N (b) + + 4 4 Portanto existe a diviso Euclidiana.
EUREKA! N14, 2002

1 < N (b) 4

28

Sociedade Brasileira de Matemtica

2.3. Teorema da fatorao nica Note que, para os inteiros de Gauss, provamos o lema de Euclides e a fatorao nica, usando somente o fato de que existe diviso Euclidiana, portanto, seguindo os mesmos passos para provar o lema de Euclides e a fatorao nica, provaremos a fatorao nica para os inteiros de Eisenstein. 2.4. Primos Tudo muito parecido com os inteiros de Gauss: N() primo em primo em []; todo primo em [] divide exatamente um primo inteiro positivo. A demonstrao desses dois fatos exatamente igual que foi dada na seo de inteiros de Gauss. Seja p o inteiro positivo primo que o primo, em Z[] divide. Temos trs casos: Se p da forma 3k, ento p = 3, e obtemos = (1 ) ou (2 + ). Se p da forma 3k + 2, como a2 ab + b2 s da forma 3k ou 3k + 1 (verifique voc mesmo), ento p um primo de em Z[] tal que N() = p2. Se p da forma 3k + 1, pela lei da reprocidade quadrtica*: p 3 = (1) 3 p
p 1 31 2 2

3 =1 =1 p

Portanto existe x inteiro tal que p | ( x 1) 2 + 3 = x 2 2 x + 2 2 = ( x 2 )(x 2 2 ) , e como p no divide 2, ento p no primo em Z[] e existem e tal que = p. Como p um primo inteiro, ento = , logo e = so primos em [] e

= p . Portanto os primos em [] so: (1) O primo 1 e suas multiplicaes por unidades. (2) Os primos inteiros da forma 3k + 2 e seus produtos pelas unidades, que tambm so primos em [].
(3) Para todo primo inteiro p da forma 3k + 1, os primos e tal que = p e seus produtos pelas unidades so primos em []. *A lei de reciprocidade quadrtica de Gauss diz o seguinte: dados a e p primo que no divide
a 1, se a resduo quadrtico mod.p. Para p, q a, definimos = p 0, caso contrrio.
p q vale sempre = ( 1) q p p 1 q 1 2 2

primos mpares com p > 0

EUREKA! N14, 2002

29

Sociedade Brasileira de Matemtica

2.5. Exemplo Ache todos os a, b, c


* +

lados de um tringulo com um ngulo de 60o.

Vamos supor, sem perda de generalidade, que o ngulo de 60o entre os lados de medidas a e b. Pela lei dos co-senos, temos: c 2 = a 2 + b 2 2ab cos(60) = a 2 + b 2 ab = (a + b )(a b ) Observe a semelhana deste problema com o das ternas pitagricas. Seja m = mdc(a; b), a = a'm, b = b'm. Segue que m 2 c 2 m c , e teremos c = c'm. Logo c 2 = a 2 + b 2 ab c' 2 = a' 2 +b' 2 a' b ', e temos mdc(a'; b'; c') = 1. Seja d tal que d a '+b' e d a 'b' . Segue que d 2a ' e d 2b' , logo d 2 . Se d no divide 1, ento 2 d 2 a' 2 a ' b'+b' 2 2 a ' e 2 b' , absurdo, logo d 1 , e portanto: a '+b' = ( x + y ) 2 = x 2 y 2 + (2 xy y 2 ) a' = x 2 y 2 b' = 2 xy y 2

Portanto as solues so a = ( x 2 y 2 )m, b = (2 xy y 2 )m e c = ( x 2 xy + y 2 )m , para todo x, y, m + com x > y, e as permutaes de a, b e c. Outro bom exemplo de aplicao dos inteiros de Eisenstein o problema 6 da IMO de 2001: Sejam a, b, c, d inteiros com a > b > c > d > 0. Considere que ac + bd = (b + d + a c)(b + d a + c) Prove que ab + cd no primo. Primeiramente vamos mostrar por que usar inteiros de Eisenstein: ac + bd = (b + d + a c)(b + d a + c) ac + bd = b 2 + bd ab + bc + bd + d 2 ad + cd + ab + ad a 2 + ac bc cd + ac c 2 b 2 + bd + d 2 = a 2 ac + c 2 A vemos por que usar inteiros de Eisenstein.

(b d )(b d 2 ) = (a + c )(a + c 2 )

Observe que como a, b, c > 1, ento mdc(b; d) > 1 ab + cd no primo, logo podemos supor que b e d so primos entre si. Analogamente supomos que mdc(a; c) = mdc(a; d) = mdc(b; c) =1.
EUREKA! N14, 2002

30

Sociedade Brasileira de Matemtica

Seja um primo em

[ ] tal que b d . Vamos provar que no divide

a + c no divide a + c 2 , e segue que b d a + c . Suponha ento que

a + c . Veja que N ( ) (b d )(a + c ), e temos (b d )(a + c ) = ab + cd + (bc + cd ad )


Como N ( ) , e N ( ) (b d )(a + c ) , ento N ( ) ab + cd e, supondo ab + cd primo, teramos

ab + cd = N ( ) .
k

Mas
k

nesse

caso

segue

que

b d = k b d 2 = a + c = ' , sendo e ' unidades.


Se = ' / , ento b + d + d = b d 2 = (a + c ). Considerando o fato de mdc(b; d) = mdc(a; c) = mdc(b; c) = 1 e que a > b > c > d > 0, temos que isto um absurdo (a verificao fica para o leitor, basta considerar as 6 possibilidades para ) . Logo b d a + c , e analogamente a + c b d . Portanto b d = (a + c ) , onde uma unidade. Novamente, basta verificar todas as possibilidades para e verificar que isto um absurdo. Portanto ab + cd no primo. 2.6. Problemas Deixamos aqui mais alguns problemas para o leitor: Problema 1. (a) Prove que, para cada inteiro n, o nmero de solues inteiras de x2 xy + y2 = n finito e divisvel por 6. (b) Determine todas as solues inteiras de x2 xy + y2 = 727. Problema 2. Mostre que a equao diofantina x3 + y3 + z3 = 0 s tem solues triviais, ou seja, tais que xyz = 0. Problema 3. Prove que se n um inteiro positivo tal que a equao x3 3xy2 + y3 = n tem solues em inteiros (x; y), ento ela tem pelo menos trs solues.

EUREKA! N14, 2002

31

Sociedade Brasileira de Matemtica

SEQNCIAS ARITMTICO-GEOMTRICAS
Jos Paulo Carneiro & Carlos Gustavo Moreira Nvel Intermedirio. A) Seqncias Aritmtico-Geomtricas ( la Z Paulo) Uma progresso aritmtica (PA) uma seqncia tal que cada termo igual ao anterior adicionado de uma constante (a razo), isto , seu termo geral (an) satisfaz relao de recorrncia a n = a n 1 + r. Da se deduz, como conhecido, que a n = a1 + (n 1)r . Analogamente, uma progresso geomtrica (PG) uma seqncia tal que cada termo igual ao anterior multiplicado por uma constante (a razo), isto , seu termo geral (an) satisfaz relao de recorrncia a n = q a n 1 . Da se deduz, como conhecido, que a n = a1 q n 1 . Existe um outro tipo de seqncia que aparece freqentemente, que uma espcie de mistura de uma PA e uma PG. aquela cujo termo geral (an) satisfaz a relao de recorrncia a n = q a n 1 + r , e que vamos chamar de seqncia aritmticogeomtrica ( la Z Paulo) (SAG), de "razo geomtrica" q e "razo aritmtica" r. (s vamos considerar os casos em que r 0 e q 1, para no recair numa PA ou numa PG.) Observemos que, uma vez conhecido o primeiro termo a1 e a relao de recorrncia a n = q a n 1 + r , conhecem-se sucessivamente a2, a3, etc., e, em princpio, todos os an. Mas permanece o interesse em determinar uma expresso explcita para o termo geral de uma SAG em funo de n, uma vez conhecidos r, q e a1. Um exemplo de SAG aparece na soluo do clebre problema da Torre de Hani (ver [2]), onde so dados trs pinos A, B e C, e se quer determinar o nmero mnimo de movimentos necessrios para se mover do pino A para um dos dois outros pinos uma pilha de n discos de tamanhos desiguais, de modo que nunca um disco maior fique em cima de um disco menor. Se an for o nmero procurado, podemos raciocinar que, inicialmente, vo ser necessrios an 1 movimentos para mover os n 1 discos superiores para o pino B, digamos; em seguida, um movimento para mover o maior de todos os discos para o pino C; e finalmente, mais an 1 movimentos para mover a pilha restante para C e completar a operao. Portanto, an satisfaz relao de recorrncia an = an 1 + 1 + an 1 = 2an 1 + 1, alm da condio inicial a1 = 1. A partir da, podem ser determinados sucessivamente: a2 = 2 1 + 1 = 3, a3 = 2 3 + 1 = 7, e assim por diante. Uma seqncia constante pode ser considerada uma PA de razo 0 ou uma PG de razo 1.
EUREKA! N14, 2002

32

Sociedade Brasileira de Matemtica

Uma pergunta interessante : uma SAG pode ser uma seqncia constante? claro que, se c for este valor constante, isto ocorrer se e s se: c = qc + r, ou seja: r . c= 1 q Vamos agora determinar uma expresso explcita para o termo geral de uma SAG em funo de n. Para isto, consideremos duas SAGs quaisquer, de termos gerais, respectivamente, an e bn, que tenham a mesma razo aritmtica r e a mesma razo geomtrica q, e consideremos a sua diferena dn = an bn . Como an = q an 1 + r e bn = q bn 1 + r, ento dn = an bn = q (an 1 bn 1) = q dn 1. Mas isto mostra que (dn) uma PG de razo q e, portanto: dn = d1 qn 1, ou seja: an bn = (a1 b1)qn 1 , ou ainda: an = bn + (a1 b1)qn 1. Como an e bn eram SAGs quaisquer, esta frmula indica como qualquer SAG pode ser obtida de outra que tenha a mesma razo aritmtica e a mesma razo geomtrica q, podemos tomar bn constante e igual a r r n 1 r q , + a1 , obtendo: a n = 1 q 1 q 1 q que a expresso que se procurava para an. Por exemplo, no caso da Torre de Hani, an = 2an 1 + 1, com a1 = 1. Portanto:
1 1 n 1 n + 1 2 = 2 1. 1 2 1 2 Vale a pena acrescentar que, se a SAG tiver uma infinidade de termos e |q| < 1, ento r qn 1 tende a zero, quando n tende a infinito. Portanto, an tende a . 1 q

Por exemplo: a seqncia infinita definida por: a n =

2 + a n 1 , com a1 = 1 tem 5

3 13 63 termos: 1; ; ; ;..., ou, em decimais: 1,000; 0,600; 0,520; 0,504;. 5 25 125 1 2 2 1 Como a n = a n 1 + , esta seqncia uma SAG com r = e q = , de modo 5 5 5 5 1 1 1 1 que seu termo geral a n = + , e o seu limite = 0,5. 2 2 5 n 1 2 B) Seqncias Aritmtico-Geomtricas ( la Gugu ) Na seo anterior definimos seqncias aritmtico-geomtricas ( la Z Paulo) generalizando as recorrncias de PA's e PG's. Vamos adotar agora um ponto de vista alternativo, generalizando a frmula do termo geral de PA's e PG's.
EUREKA! N14, 2002

33

Sociedade Brasileira de Matemtica

Lembremos que em uma PA de termo inicial a0 = a e razo r temos an = a + nr, n  e numa PG de termo inicial a0 = a e razo q, temos an = aqn. Fazemos, inspirados nessas frmulas, a seguinte definio: Definio: Uma progresso Aritmtico-Geomtrica (PAG) ( la Gugu) uma seqncia (an) cujo termo geral satisfaz a frmula an = (a + nr)qn, n  . Note que se r = 0 nossa PAG uma PG, e se q = 1 nossa PAG uma PA. interessante obter uma frmula para a soma dos n primeiros termos de uma PAG ( sn =

a
j =0

n 1

), a qual generalizaria as frmulas para a soma dos n primeiros termos de

PG's (a princpio generalizaria tambm de PA's, mas suporemos q 1. Se fizermos q tender a 1 a frmula tender frmula da soma dos n primeiros termos de uma PA). n 2 n 1 s a n1 Temos sn = (a + jr )q j . Ento n = (a + jr)q j 1 = (a + ( + 1)r )q  = q j =1  =0 j =0 =

 =0

n2

rq  +


n 2
 =0

(a + r )q  = r

( q n 1 1) + s n (a + (n 1)r )q n 1 e, portanto, q 1

q 1 q q n1 1 r a r n1 a ( ) a n r q r sn ( 1 ) (a + (n 1)r )q n1 + = + sn = = q q q 1 q 1 q 1 q q 1 q qr n 1 (r a )q + a q + . a + nr = q 1 q 1 (q 1) 2 aq n a (1 q ) a (q n 1) + = , que a q 1 (q 1) 2 q 1 frmula da soma dos n primeiros termos de uma PG. Notemos finalmente que SAG's e PAG's satisfazem recorrncias lineares homogneas (ver [1]): Para SAG's, temos a n + 2 qa n +1 = r = a n +1 qa n , logo Note que se r = 0 a expresso acima se reduz a
a n + 2 = (q + 1) a n +1 qa n , e para PAG's, notamos que se bn = a n / q n , temos bn =

a + nr, e bn + 2 bn +1 = r = bn +1 bn bn + 2 = 2bn +1 bn = 2qa n +1 q 2 a n . Os polinmios caractersticos

a n+ 2 q
n+2

2a n 1 q
n +1

an qn

a n+ 2 =

dessas

recorrncias

so

respectivamente x 2 (q + 1) x + q = ( x q )( x 1) e x2 2qx + q2 = (x q)2. Referncia: [1] Hctor Soza Pollman, Equaes de Recorrncia, Eureka! No. 9, pp. 33-40. [2] Carlos Yuzo Shine, A torre de Hani, Eureka! No. 11, pp. 17-23.
EUREKA! N14, 2002

34

Sociedade Brasileira de Matemtica

O PRINCPIO DA INVARINCIA
Marcelo Rufino de Oliveira, Belm PA Artigo baseado em aula ministrada no II Teorema, Fortaleza CE Nvel Intermedirio. Uma das principais estratgias para resoluo de problemas de olimpadas a procura por invariantes. O fundamento do Princpio da Invarincia simples: busca pelo que se mantm constante quando uma operao permitida realizada. Entre as principais formas de invariantes destacam-se trs, que sero apresentadas a seguir atravs de exemplos resolvidos.
1. Expresses ou Valores Numricos Invariantes Exemplo 1.1: Comeando com o conjunto {3, 4, 12}, permitido apagar dois nmeros

a e b e escrever em seus lugares 0,6.a 0,8.b e 0,8.a + 0,6.b. possvel chegar ao conjunto {4, 6, 12}?
Resoluo:

Repare que (0,6.a 0,8.b)2 + (0,8.a + 0,6.b)2 = a2 + b2, implicando que a soma dos quadrados dos nmeros dos conjuntos obtidos invariante. Como 32 + 42 + 122 = 132 e 42 + 62 + 122 = 142 ento no possvel chegar ao conjunto {4, 6, 12}.
Exemplo 1.2: (Maio-99) Em cada um dos 10 degraus de uma escada existe uma r.

Cada r pode, de um pulo, colocar-se em outro degrau, mas quando uma r faz isso, ao mesmo tempo, uma outra r pular a mesma quantidade de degraus em sentido contrrio: uma sobe e outra desce. Conseguiro as rs colocar-se todas juntas num mesmo degrau?
Resoluo:

Numeremos os degraus de 1 a 10 e associemos a cada r o nmero do degrau que ocupam. O somatrio inicial destes valores S = 1 + 2 + + 10 = 55. Perceba agora que este somatrio invariante, pois quando uma r sobe uma certa quantidade x de degraus, temos outra r que desce x, fazendo com que a soma das numeraes destas duas rs no se altere. Caso todas as rs ocupem um mesmo degrau (digamos y), ento todas as suas numeraes so iguais a deste degrau, ou seja, teremos 10y = 55, que no possui soluo inteira. Deste modo, impossvel que todas as rs ocupem um mesmo degrau.

EUREKA! N14, 2002

35

Sociedade Brasileira de Matemtica

Exemplo 1.3: As seguintes operaes so permitidas com a equao quadrtica ax2 + bx + c: a) trocar a e c; b) trocar x por x + t, onde t um nmero real. Repetindo estas transformaes possvel transformar x2 x 2 em x2 x 1? Resoluo:

Mostraremos que invariante o valor do discriminante de todas as equaes obtidas pela aplicao das operaes permitidas. Inicialmente temos 0 = b2 4ac. Aplicando a primeira operao: ax2 + bx + c cx2 + bx + a (1) Para esta equao temos 1 = b2 4ca = 0 Aplicando a segunda operao: ax2 + bx + c a(x + t)2 + b(x + t) + c = ax2 + (b + 2at)x + at2 + bt + c (2) 2 = (b + 2at)2 4a(at2 + bt + c) = b2 + 4abt + 4a2t2 4a2t2 4abt 4ac 2 = b2 4ac = 0 = 1 Como o discriminante de x2 x 2 9 e o discriminante de x2 x 1 5, a transformao impossvel.
2. Restos de Divises Invariantes Exemplo 2.1: (Torneio das Cidades-85) Todo membro de uma seqncia, iniciando do segundo, igual a soma entre o termo precedente e a soma dos seus dgitos. O primeiro nmero 1. possvel que 123456 pertena seqncia? Resoluo:

Perceba que: a2 = 2 = 3.0 + 2 a3 = 4 = 3.1 + 1 a4 = 8 = 3.2 + 2 a5 = 16 = 3.5 + 1 a6 = 23 = 3.7 + 2 a7 = 28 = 3.9 + 1 ... Aparentemente os restos da diviso por 3 dos termos so alternadamente 1 e 2. Vamos demonstrar isto. Seja S(n) a soma dos dgitos de n. Sabemos que n e S(n) deixam o mesmo resto na diviso por 3: i) Se an = 3k1 + 1 an + 1 = an + S(an) = 3k1 + 1 + 3k2 + 1 = 3k3 + 2. ii) Se an = 3k1 + 2 an + 1 = an + S(an) = 3k1 + 2 + 3k2 + 2 = 3k3 + 1. Deste modo, se n par ento an = 3k + 2 e se n mpar ento an = 3k + 1 (invariante). Como 123456 divisvel por 3 ento no pertence seqncia.
Exemplo 2.2: (Leningrado-85) Trs cangurus esto alinhados em uma estrada. A cada

segundo um dos cangurus salta. permitido que um canguru salte por cima de um outro canguru, mas no de dois cangurus de uma s vez. Prove que depois de 1985 segundos, os cangurus no podem voltar a ocupar a posio relativa inicial.

EUREKA! N14, 2002

36

Sociedade Brasileira de Matemtica

Resoluo:

Existem seis posies para os cangurus: 123 132 312 321 231 213 Note que as posies sublinhadas somente podem ser alcanadas atravs de um posio anterior em negrito e vice-versa (invariante). Assim, depois de um nmero mpar de pulos somente as posies sublinhadas (132, 321 e 213) podem ser alcanadas, fazendo com que depois de 1985 pulos no seja possvel que os cangurus voltem a ocupar a posio inicial (123).
Exemplo 2.3: (Par-2001) Um tabuleiro 4x4 possui, inicialmente, todas as casas pintadas de branco. Uma operao permitida escolher um retngulo consistindo de 3 casas e pintar cada uma das casas da seguinte forma: se a casa branca ento pinta-se de preto; se a casa preta ento pinta-se de branco. Prove que, aplicando vrias vezes a operao permitida, impossvel conseguirmos que todo o tabuleiro fique pintado de preto. Resoluo:

Distribuimos as letras a, b e c no tabuleiro da seguinte forma:

a b c c a b b c a a b c

a c b a

Note que as letras esto alternadas tanto nas linhas quanto nas colunas. Esta alternncia faz com que toda vez que um retngulo com 3 casas seja selecionado, ento exatamente uma letra a, uma letra b e uma letra c so selecionadas. Sejam: A a quantidade de casas brancas com a letra a, B a quantidade de casas brancas com a letra b e C a quantidade de casas brancas com a letra c. No incio temos: A = 6 B = 5 C = 5. Toda vez que selecionamos um retngulo formado de trs casas, estamos somando a cada valor de A, B e C os valores + 1 ou 1. Perceba que se todas casas ficarem pretas, ento teremos A = 0, B = 0 e C = 0. Entretanto, note que iniciando de A = 6, B = 5 e C = 5, e alterando simultaneamente por + 1 ou 1 estes valores, sempre teremos entre os valores de A, B e C dois nmeros mpares e um par ou dois pares e um mpar (invariante), fazendo com que a situao A = 0, B = 0 e C = 0 seja impossvel.
Exemplo 2.4: (OBM Jr.-95) Temos um tabuleiro 1995 1995. A cada uma de suas

19952 casas associamos um dos nmeros + 1 ou 1. Em seguida, associamos a cada

EUREKA! N14, 2002

37

Sociedade Brasileira de Matemtica

linha o produto dos nmeros das casas desta linha, e a cada coluna o produto dos nmeros das casas de cada coluna. i) Se T a soma dos nmeros associados s linhas, colunas e casas, prove que T diferente de 0. ii) Se S a soma dos nmeros associados s linhas e s colunas, prove que S diferente de 0.
Resoluo:

i) Como temos 1995 colunas e 1995 linhas, ento as somas dos nmeros associados s linhas e colunas so nmeros mpares, pois so soma ou subtrao de 1995 nmeros mpares. Em relao s casas temos o mesmo raciocnio, pois so ao todo 19952 casas, cujo valor de cada casa pode ser igual a 1 ou 1. Somando todos estes valores teremos tambm um valor mpar. Assim, T a soma de trs valores mpares, sendo tambm mpar, nunca podendo assumir o valor zero. ii) Inicialmente notemos que qualquer disposio no tabuleiro pode ser alcanada partindo de uma configurao inicial na qual todas as casas possuem valor 1 e alterando-se os sinais desejados. Com o tabuleiro contendo somente 1's, temos que as somas das linhas e colunas 1995, fazendo uma soma total de 3990, que obviamente o maior valor possvel. Quando trocamos um 1 por um 1, notamos que as somas das linhas e colunas passam a ser 1993, fazendo S = 3986. Assim, fica evidente que uma alterao de um sinal em uma casa do tabuleiro altera o valor da soma da linha e da coluna a qual pertence esta casa em 2, e por conseqncia altera a soma total em 0 ou 4 (invariante). Deste modo, a soma total pode ser escrita da forma S = 3990 4k. Caso S = 0, teramos 4k = 3990, que no possui soluo inteira, absurdo.
Exemplo 2.5: (Hong Kong-97) Cinco nmeros 1, 2, 3, 4, 5 esto escritos em um quadro negro. Um estudante pode apagar dois dos nmeros a e b no quadro e escrever os nmeros a + b e ab nos seus lugares. Se esta operao repetida indefinidamente, podem os nmeros 21, 27, 64, 180, 540 aparecer no quadro negro ao mesmo tempo? Resoluo:

No possvel. Note que no incio existe somente um nmero divisvel por 3 e no final existem quatro nmeros divisveis por 3. Observemos o que acontece com os restos da diviso por 3 dos nmeros no quadro quando fazemos uma operao: i) se a = 3x e b = 3y a + b = 3z e ab = 3w ii) se a = 3x + 1 e b = 3y a + b = 3z + 1 e ab = 3w iii) se a = 3x + 2 e b = 3y a + b = 3z + 2 e ab = 3w iv) se a = 3x + 1 e b = 3y + 1 a + b = 3z + 2 e ab = 3w + 1
EUREKA! N14, 2002

38

Sociedade Brasileira de Matemtica

v) se a = 3x + 1 e b = 3y + 2 a + b = 3z e ab = 3w + 2 vi) se a = 3x + 2 e b = 3y + 2 a + b = 3z + 1 e ab = 3w + 1 Portanto, a nica forma de aumentar os divisveis por 3 escolher a = 3x + 1 e b = 3y + 2. Consequentemente tambm acrescentamos um nmero da forma 3k + 2. Por outro lado, na situao final o nico nmero que no divisvel por 3 64, que da forma 3k + 1, contradio, pois este nmero deveria ser da forma 3k + 2 (O caso iii) mostra que sempre haver um nmero da forma 3k + 2 aps termos 4 deles da forma 3k).
3. Tendncias de Crescimento ou Decrescimento Invariantes Exemplo 3.1: Um total de 2000 pessoas esto divididas entre os 115 quartos de uma

manso. A cada minuto, at que todas no estejam em um mesmo quarto, uma pessoa anda para um quarto com um nmero igual ou maior de pessoas do que o quarto que ocupava. Prove que eventualmente todas as pessoas vo estar em um mesmo quarto. Seja ai a quantidade de pessoas no quarto i, 1 i 115. 2 2 2 + a2 + ... + a115 Considere a expresso I = a1 . Digamos que uma pessoa sai de um quarto que possui n pessoas e vai para um quarto que possui m pessoas (m n). A variao de I : I = ((m + 1)2 + (n 1)2) (m2 + n2) = 2(m n + 1) > 0 Assim, toda vez que uma pessoa troca de quarto o valor de I cresce (tendncia de crescimento invariante). Entretanto note que o valor de I no pode crescer indefinidamente, uma vez que o nmero total de pessoas finito, implicando que uma hora todas as pessoas estaro em um mesmo quarto.
Exemplo 3.2: (2 Lista de Preparao para a Cone Sul-2001) Existem inicialmente n
a

Resoluo:

nmeros 1 em um quadro negro. Em cada passo permitido apagar quaisquer dois nmeros a e b e escrever o nmero ab 2 . Esta operao feita n 1 vezes. Prove a+b que o ltimo nmero no menor que 1 . n
Resoluo:

Suponha que depois de k operaes temos os seguintes nmeros escritos no quadro: 1 . a1, a2, ..., an k. Considere a expresso Ik = 12 + 12 + ... + 2 a1 a2 an k
EUREKA! N14, 2002

39

Sociedade Brasileira de Matemtica

Depois de uma operao a variao de I vale:


2 a+b 1 1 = 1 1 + 1 1 1 I = 1 1 1 I = ab 2a 2 2b 2 a 2 b2 2 a b a2 b2 2 ab Pela desigualdade entre as mdias aritmtica e geomtrica: 1 + 1 a 2 b2 1 1 = 1 1 1 1 0 2 ab 2a 2 2b 2 a 2 b 2 ab Desta forma I 0, ou seja, I nunca cresce. Como I0 = n e In 1 = 12 ento In 1 I0 12 n a1 1 . n a1 a1 2

Exemplo 3.3: (St. Petersburg-96) Vrios inteiros positivos distintos esto escritos em

um quadro negro. Uma operao permitida apagar dois inteiros distintos e escrever em seus lugares o mximo divisor comum e o mnimo mltiplo comum destes nmeros. Prove que, depois da aplicao de operaes permitidas vrias vezes, os nmeros eventualmente vo parar de mudar.
Resoluo:

Suponha que a e b (a > b) so dois nmeros escritos no quadro. Sejam D = mdc (a, b) e L = mmc (a, b). Consequentemente: D < a, D b, L a, L > b. Como ab = DL ab + b2 = DL + b2 (1) Como (L b)(D b) 0 DL Lb bD + b2 0 DL + b2 bL + bD (2) Aplicando (2) em (1): ab + b2 bL + bD a + b D + L (3) Assim, a soma S dos nmeros escritos no quadro nunca decresce. Repare tambm que a igualdade em (3) implica (L b)(D b) = 0, ou seja, D = b L = a. Por outro lado, como mdc (x1, x2, ..., xm) mmc (x1, x2, ..., xm) x1x2...xm, ento, depois de um nmero qualquer de operaes realizadas, cada nmero no quadro menor ou igual que o produto de todos os nmeros inicialmente escritos. Portanto, S menor ou igual a n vezes o produto dos nmeros inicialmente escritos. Como podese fazer um nmero infinito de operaes e S possui ao mesmo tempo as propriedades de ser limitada e nunca decrescer, ento a partir de um certo momento S fica constante, fazendo com que D = b e L = a, implicando que eventualmente os nmeros vo parar de mudar.

EUREKA! N14, 2002

40

Sociedade Brasileira de Matemtica

Exerccios: 01. Um crculo dividido em seis setores. Os nmeros 1, 0, 1, 0, 0, 0 so escritos em

sentido horrio. permitido aumentar em 1 dois nmeros vizinhos. possvel que em algum momento todos os nmeros sejam iguais?
02. Divide-se um crculo em 10 setores e coloca-se uma ficha em cada setor. Um movimento consiste em selecionar duas fichas e mover cada uma para um setor adjacente. Prove que, depois de uma seqncia arbitrria de movimentos, impossvel que todas as fichas localizem-se em um mesmo setor. 03. (Rio Grande do Norte-99) A professora desafia Andr e Thiago com o seguinte

jogo, em que eles jogam alternadamente. Ela escreve no quadro-negro os inteiros de 1 a 50. Uma jogada consiste em escolher dois dos nmeros escritos, apagar esses nmeros, substituindo-os pela soma (Por exemplo, se Andr escolheu 8 e 23, apagaos e escreve 31). Depois de algum tempo, vai restar no quadro negro um nico nmero. Se esse nmero par, o ganhador Andr, caso contrrio, o ganhador Thiago. Quem vence o jogo: Andr ou Thiago?
04. (Torneio das Cidades-93) Trs pilhas de caroos so dadas sobre uma mesa.

permitido adicionar ou remover de uma pilha um nmero de caroos que igual a soma do nmero de caroos das outras duas pilhas. Por exemplo [12, 3, 5] pode tornar-se [12, 20, 5] pela adio de 17 = 12 + 5 para a pilha de 3 ou tornar-se [4, 3, 5] pela remoo de 8 = 3 + 5 caroos da pilha com 12. possvel, iniciando com pilhas possuindo 1993, 199 e 19 caroos, conseguir uma pilha vazia depois de uma seqncia de operaes permitidas?
05. (Torneio das Cidades-85) Na ilha de Camelot vivem 13 camalees roxos, 15

verdes e 17 amarelos. Quando dois de cores distintas se encontram, mudam simultaneamente para a terceira cor. Poderia dar-se a situao na qual todos tenham a mesma cor?
06. (Rssia-78) So dadas 3 mquinas que produzem cartes com pares de nmeros

naturais. A primeira, sendo dado o carto com (a, b), produz um novo carto com (a + 1, b + 1); a segunda, sendo dado o carto com (a, b), produz novo carto com (a/2, b/2), se ambos a e b so pares e nada no caso oposto e a terceira mquina, sendo dados os cartes (a, b) e (b, c), produz um novo carto com (a, c). Todas as mquinas retornam tambm os cartes iniciais. Suponha que foi dado o carto inicial (5, 19). possvel obter: a) (1, 50)?
EUREKA! N14, 2002

41

Sociedade Brasileira de Matemtica

b) (1, 100)? c) Suponha que foi dado o carto inicial (a, b) (a < b). Ns queremos obter o carto (1, n). Para quais n isto possvel?
07. Em um quadro negro esto escritos n nmeros. A cada minuto apaga-se dois

nmeros a e b e escreve-se o nmero (a + b)/4. Repetindo esta operao n 1 vezes, existir somente um nmero no final. Prove que se inicialmente existirem n 1's no quadro, ento o ltimo nmero no menor que 1/n.
08. (Leningrado-89) Vrios (mas no menos que 2) nmeros no nulos so escritos

em um quadro negro. possvel apagar dois dos nmeros, A e B, e ento escrever nos seus lugares os nmeros A + B/2 e B A/2. Prove que o conjunto de nmeros no quadro negro, depois de um nmero qualquer de operaes, no pode coincidir com o conjunto inicial.
09. (2 Lista de Preparao para a Cone Sul-96) 119 anes vivem em uma aldeia com
a

120 pequenas casas. Uma casa dita super-habitada se 15 anes ou mais vivem l. Todo dia, os anes de uma casa super-habitada tm uma discusso e se mudam para outras (distintas) casas da aldeia. Algum dia, necessariamente, esse processo se encerrar?

Bibliografia:

[1] A. Engel, Problem-Solving Strategies, Springer-Verlag, 1998. [2] P. J. Taylor, Tournament of the Towns 1984-1989, Australian International Centre for Mathematics Enrichment, 1992.
[3] D. Fomin, A. Kirichenko, Leningrad Mathematical Olympiads 1987-1991, MathPro Press, 1994.

EUREKA! N14, 2002

42

Sociedade Brasileira de Matemtica

XXII TORNEIO DAS CIDADES


Primavera 2001 Nvel O Jnior
PROBLEMA 1

O nmero natural n pode ser trocado por ab se a + b = n, onde a e b tambm so nmeros naturais. O nmero 2001 pode ser obtido a partir de 22 atravs destas trocas?
PROBLEMA 2

Um dos segmentos que liga os pontos mdios dos lados de um tringulo maior que uma das medianas do tringulo. Prove que o tringulo obtusngulo.
PROBLEMA 3

Vinte quilogramas de queijo esto venda em uma mercearia e os fregueses esto em fila para comprar esse queijo. Aps algum tempo, tendo acabado de atender um dos fregueses, a vendedora calcula (corretamente) a quantidade total de queijo j vendida e anuncia o nmero de fregueses para os quais h queijo, na quantia exata, se cada fregus comprar uma poro cuja massa exatamente igual a quantidade mdia comprada pelos anteriores. Pode ocorrer de a vendedora poder declarar, aps cada um dos 10 primeiros fregueses ter feito sua compra, que h queijo na quantia exata para os prximos 10? Se isso puder ocorrer, quanto queijo haver ainda na mercearia aps os primeiros 10 fregueses terem feito suas compras? (A quantidade mdia de uma seqncia de compras a massa total de queijo vendida dividida pelo nmero de vendas.)
PROBLEMA 4

a) H 5 tringulos de papel idnticos sobre uma mesa. Cada um pode ser transladado em qualquer direo. verdade, ento, que qualquer um deles pode ser coberto pelos outros 4? b) H 5 tringulos equilteros de papel idnticos sobre uma mesa. Cada um pode ser transladado em qualquer direo. Prove que, ento, qualquer um deles pode ser coberto pelos outros 4.
PROBLEMA 5

Sobre um tabuleiro 15 15 so colocados quinze cavalos de modo que em cada fileira (linha ou coluna) do tabuleiro haja exatamente um cavalo. Ento, simultaneamente, cada cavalo faz um movimento segundo as regras do xadrez. Prove que aps os movimentos haver dois cavalos em uma mesma fileira do tabuleiro.
EUREKA! N14, 2002

43

Sociedade Brasileira de Matemtica

Primavera 2001 Nvel O Snior


PROBLEMA 1

Um nibus que percorre um trajeto de 100 km equipado com um computador, que prev quanto tempo falta para chegar ao destino final. Esta previso feita assumindo que a velocidade mdia do nibus na parte restante do trajeto ser a mesma da parte j percorrida. Quarenta minutos aps a partida do nibus, o computador previu que o tempo restante de viagem seria de uma hora. E este tempo previsto permaneceu inalterado pelas prximas 5 horas. Isso pode de fato ocorrer? Caso possa, quantos quilmetros o nibus percorreu nestas 5 horas? (A velocidade mdia do nibus o nmero de quilmetros percorridos dividido pelo tempo gasto para percorr-los.)
PROBLEMA 2

A representao decimal do nmero natural a consiste de n dgitos, enquanto que a representao decimal de a3 consiste de m dgitos. Pode n + m ser igual a 2001?
PROBLEMA 3

No tringulo ABC o ponto X est sobre o lado AB, enquanto o ponto Y est sobre o lado BC. Os segmentos AY e CX interceptam-se no ponto Z. Sabe-se que AY = YC e AB = ZC. Prove que os pontos B, X, Z e Y esto sobre uma circunferncia. Duas pessoas jogam sobre um tabuleiro 3 100. Elas jogam alternadamente: a primeira coloca domins 1 2 sobre o tabuleiro, a segunda coloca domins 2 1. A perdedora aquela que, na sua vez, no puder colocar domin. Qual dos dois jogadores tm estratgia vencedora? Descreva-a.
PROBLEMA 5 PROBLEMA 4

Nove pontos so desenhados sobre a superfcie de um tetraedro regular de aresta 1 cm. Prove que entre estes pontos existem dois cuja distncia (no espao) menor ou igual a 0.5 cm.

EUREKA! N14, 2002

44

Sociedade Brasileira de Matemtica

XXIII TORNEIO DAS CIDADES


Outono 2002 Nvel O Jnior
PROBLEMA 1

dado um nmero suficientemente grande de cartes retangulares a cm b cm, onde a e b so inteiros positivos e a menor do que b. Sabe-se que com esses cartes podemos montar (sem sobrepor cartes e sem buracos) um retngulo 49 cm 51 cm e um 99 cm 101 cm. Os valores de a e b esto determinados unicamente a partir desses dados?
PROBLEMA 2

Dado um tringulo qualquer, possvel cort-lo em quatro conjuntos convexos: um tringulo, um quadriltero, um pentgono e um hexgono?
PROBLEMA 3

O nmero x 2 + xy + y 2 , no qual x e y so inteiros positivos, escrito na notao decimal termina em zero. Prove que ele termina em pelo menos dois zeros.
PROBLEMA 4

Os lados AB, BC, CD e DA do quadriltero ABCD so tangentes a uma circunferncia nos pontos K, L, M e N, respectivamente; S o ponto de interseco dos segmentos KM e LN. Sabe-se que o quadriltero SKLB inscritvel. Prove que o quadriltero SNDM tambm inscritvel.
PROBLEMA 5

a) So dadas 128 moedas de duas massas distintas, 64 de cada tipo. Como obter duas moedas de massas distintas fazendo no mais de 7 pesagens um uma balana com dois braos (e sem pesos auxiliares)? b) So dadas oito moedas de duas massas distintas, 4 de cada tipo. Como obter duas moedas de massas distintas fazendo no mais de duas pesagens em uma balana com dois braos (e sem pesos auxiliares)?

EUREKA! N14, 2002

45

Sociedade Brasileira de Matemtica

Outono 2002 Nvel A Jnior


PROBLEMA 1

Sejam a, b, e c as medidas dos lados de um tringulo. Prove a desigualdade a 3 + b 3 + 3abc >c 3


PROBLEMA 2

Quatro peas (duas brancas e duas pretas) so colocadas em um tabuleiro quadriculado 23 23: as peas brancas so colocadas no canto superior esquerdo e no canto inferior direito; as peas pretas so colocadas no canto inferior esquerdo e no canto superior direito. As brancas e pretas se movem alternadamente, sendo que as brancas comeam. Em cada movimento, uma pea movida para qualquer casa vizinha (isto , que tem uma lado em comum) que no tenha pea. O objetivo das peas brancas ocupar duas casas vizinhas. As pretas podem evitar que isso acontea?
PROBLEMA 3

Sejam E e F os pontos mdios dos lados BC e CD, respectivamente, do quadriltero convexo ABCD. Os segmentos AE, AF e EF dividem o quadriltero em 4 tringulos cujas reas (em alguma ordem) so inteiros positivos consecutivos. Qual a maior rea que o tringulo ABC pode ter?
PROBLEMA 4

N lmpadas esto enfileiradas. Inicialmente, algumas delas esto acesas. A cada minuto, todas as lmpadas acesas so apagadas e acendemos cada lmpada apagada que for vizinha a exatamente uma lmpada que estava acesa. Para que valores de n possvel escolher uma configurao inicial de lmpadas de modo que em qualquer momento pelo menos uma lmpada esteja acessa?
PROBLEMA 5

Um tringulo acutngulo cortado por uma reta em dois pedaos (no necessariamente triangulares). Em seguida, um dos pedaos cortado por uma reta em dois pedaos, e assim por diante: a cada passo um dos pedaos obtido em qualquer passo anterior escolhido e cortado por uma reta em dois novos pedaos. Aps um certo nmero de passos, todos os pedaos so triangulares. possvel que todos tenham um ngulo obtuso?

EUREKA! N14, 2002

46

Sociedade Brasileira de Matemtica

PROBLEMA 6

Em uma seqncia crescente de inteiros positivos, cada termo, a partir do 2002simo, divide a soma de todos os termos anteriores. Prove que cada termo da seqncia, a partir de um certo ponto, igual soma de todos os anteriores.
PROBLEMA 7

Dada uma cadeia de domins, montada de acordo com as regras usuais, permitida a seguinte operao: escolha uma subcadeia contida nela cujas extremidades so iguais (tm o mesmo nmero de pontos marcados), retire-a, inverta sua ordem e recoloque. Mostre que, dadas duas cadeias montadas a partir de um mesmo conjunto de domins e com extremidades respectivamente iguais (isto , os incios so iguais entre si, bem como os finais), possvel transformar, atravs de uma seqncia de operaes como a descrita, uma das cadeias na outra.

EUREKA! N14, 2002

47

Sociedade Brasileira de Matemtica

OLIMPADAS AO REDOR DO MUNDO


Como sempre acontece no perodo em que realizada a maior parte das competies internacionais o comit editorial da revista EUREKA! se preocupa em mostrar as resolues das competies nacionais dos anos anteriores para o treinamento dos nossos atuais e futuros competidores. Por isto, estivemos ausentes da edio anterior. Continuamos a disposio na OBM para aqueles que estiverem interessados na soluo de algum problema particular. Para tanto, basta contactar a OBM, atravs de carta ou e-mail. Antonio Luiz Santos

151. (Irlanda-2001) Seja ABC um tringulo de lados BC , CA, AB cujas medidas so

respectivamente iguais a a, b, c. Se D e E so os pontos mdios de AC e AB respectivamente, mostre que a mediana BD perpendicular a CE se, e somente se, b 2 + c 2 = 5a 2 .
152. (Irlanda-2001) Mostre que se um nmero primo mpar p pode ser colocado sob a

forma x 5 y 5 para alguns inteiros x e y ento


4 p + 1 v2 + 1 = 5 2

para algum inteiro mpar v.


153. (Irlanda-2001) Determine os nmeros reais x no negativos para os quais
3

13 + x + 3 13 x

um nmero inteiro.
154. (Irlanda-2001) Determine todas as funes f :   que satisfazem

f (x + f ( y )) = f (x ) + y, x, y N.

155. (Rssia-2001) Os nmeros de 1 a 999 999 so divididos em dois grupos: no

primeiro, cada nmero nele colocado tal que o quadrado perfeito mais prximo dele o quadrado de um nmero mpar. No segundo, os nmeros esto mais prximos de quadrados perfeitos de nmeros pares. Determine em qual dos grupos a soma dos nmeros a ele pertencentes maior.
EUREKA! N14, 2002

48

Sociedade Brasileira de Matemtica

156. (Rssia-2001) Sobre o maior lado AC de um tringulo ABC , toma-se um ponto

N de modo que as mediatrizes dos segmentos AN e NC intersectam os lados AB e BC nos pontos K e M, respectivamente. Prove que o circuncentro O do tringulo ABC pertence ao crculo circunscrito ao tringulo KBM.
157. (Rssia-2001) Dois crculos so tangentes internamente no ponto N . Uma

tangente traada de um ponto K do crculo interno intersecta o crculo externo nos pontos A e B . Se M o ponto mdio do arco AB que no contm o ponto N , mostre que o raio do crculo circunscrito ao tringulo BMK no depende da escolha do ponto K do crculo interior.
158. (Rssia-2001) A seqncia (xn ) tal que x1 = 1 , xn +1 = n sen xn + 1 . Mostre que

esta seqncia no peridica.


159. (Eslovnia-2001) Sejam a1 , a 2 , a3 , a 4 e a5 nmeros reais distintos. Denotando por

m o nmero de valores distintos das somas ai + a j , onde 1 i j 5 ,

determine o menor valor possvel de m .


160. (Eslovnia-2001) Sejam a, b, c, d , e e

f nmeros reais positivos tais que a seqncia (a, b, c, d ) seja aritmtica e a seqncia (a, e, f , d ) seja geomtrica. Mostre que bc ef . Sabendo que a bissetriz interna do ngulo C intersecta o lado oposto no ponto E e que CEA = , determine a medida do ngulo EDB . 4

161. (Eslovnia-2001) Seja D o p da altura relativa ao lado BC do tringulo ABC .

162. (Eslovnia-2001) (a) Mostre que a desigualdade

2 n verdadeira para todo inteiro positivo n .

n +1 n <

< n n 1

(b) Mostre que a parte inteira da expresso 1 1 1 1 1+ + + + + 2 2 3 m 1 m2 onde m um inteiro positivo igual a 2m 2 ou 2m 1 .

EUREKA! N14, 2002

49

Sociedade Brasileira de Matemtica

163. (Crocia-2001) Resolva a inequao

x1+ log a x > a 2 x, a > 0, a 1


164. (Crocia-2001) A partir dos pontos mdios dos lados de um tringulo acutngulo

so traadas perpendiculares aos outros dois lados. Mostre que a rea do hexgono definido por esses segmentos igual metade da rea do tringulo.
165. (Crocia-2001) Determine todos os pares ordenados de nmeros reais x, y que

satisfazem equao 2 1 1 = log 2 2 cos 2 (xy ) + 1 y 2 2 cos 2 (xy )


166. (Crocia-2001) Quatro esferas de raios iguais a R so mutuamente tangentes entre

si. Determine o raio r da maior esfera que pode ser inserida entre elas.
167. (Bielorssia-2001) No grfico da parbola y = x no plano cartesiano marcamos os
2

pontos A, B e C (com A entre B e C ). No segmento BC marca-se o ponto N de modo que AN seja paralelo ao eixo das ordenadas. Se S1 e S 2 so as reas dos tringulos ABN e ACN , respectivamente, determine a medida do segmento AN.
168. (Bielorssia-2001) A comisso organizadora da OBM preparou algumas variantes

de uma competio com 4 problemas cada uma. Duas variantes distintas podem conter um mesmo problema mas no mais do que um. Qual o menor nmero de problemas necessrios para que a comisso organizadora prepare 10 variantes da competio?
169. (Bielorssia-2001) O quadriltero ABCD est inscrito num crculo S1 ; um outro

crculo S 2 passa pelo ponto D , pelo ponto O , de interseo das diagonais do quadriltero, e intersecta AD e CD em M e N, respectivamente. Sabendo que OM e AB intersectam-se em R enquanto que ON e BC intersectam-se em T ( R , T e A pertencem ao mesmo semiplano em relao a BC ), mostre que O, R, T e B pertencem ao mesmo crculo.

EUREKA! N14, 2002

50

Sociedade Brasileira de Matemtica

170. (Bielorssia-2001) Determine o resto da diviso de

1! 5 + 2! 11 + + k! k 2 + 3k + 1 + + 200! 40601 por 2004 .


171. (Inglaterra-2001) Determine todos os nmeros naturais N de dois algarismos para

os quais a soma dos algarismos de 10 N N divisvel por 170 .


172. (Inglaterra-2001) Um crculo S interior a um crculo T e o tangencia no ponto

A . De um ponto P A sobre T , traam-se as cordas PQ e PT de T que tangenciam S em X e Y, respectivamente. Mostre que QAR = 2XAY .

173. (Inglaterra-2001) Uma seqncia

(an )

tal que a n = n +

n , onde n um

inteiro positivo e x o inteiro mais prximo a x , sendo as metades arredondadas para cima se for necessrio. Determine o menor inteiro k para os quais os termos ak , ak +1 ,..., ak + 2000 formam uma seqncia de 2001 inteiros consecutivos.
174. (Inglaterra-2001) As medidas dos lados de um tringulo so a, b, c e a medida do

raio do crculo circunscrito ao tringulo R . Mostre que o tringulo retngulo se, e somente se, a 2 + b 2 + c 2 = 8 R 2 .
175. (Torneio das Cidades-2001) Diga se existem inteiros positivos a1 < a2 < ... < a100 tais

que para todo k , 2 k < 100 , o mnimo mltiplo comum de ak 1 e ak maior que o mnimo mltiplo comum de ak e ak +1 .
176. (Torneio das Cidades-2001) Os vrtices de um tringulo tm coordenadas (x1 , y1 ) ,

(x2 , y2 ) e (x3 , y3 ). Para quaisquer inteiros h e k , o tringulo cujos vrtices tm coordenadas (x1 + h, y1 + k ) , (x2 + h, y2 + k ) e (x3 + h, y3 + k ) disjunto do
1 ? 2

tringulo original. (i) possvel que a rea deste tringulo seja maior que (ii) Qual a rea mxima deste tringulo?

EUREKA! N14, 2002

51

Sociedade Brasileira de Matemtica

177. (Torneio das Cidades-2001) Sejam a e d inteiros positivos tais que, para qualquer

inteiro positivo n , a expanso decimal de a + nd contm um bloco de algarismos consecutivos igual expanso decimal de n. Prove que d uma potncia de 10.

178. (ustria-Polnia-2001) Determine o nmero de inteiros positivos a para os quais

existem inteiros no negativos x0 , x1 ,..., x2001 que satisfazem a a x0 =


179. (Estnia-2001) Os ngulos de um
2001 k =1

xk

n gono convexo so , 2, ... , n . Determine todos os valores possveis de n e os valores correspondentes de .

180. (Estnia-2000) Mostre que para todo inteiro a > 1 , existe um nmero primo p tal

que

1 + a + a 2 + + a p 1 composto.

Acusamos o recebimento de solues de problemas anteriores dos seguintes leitores de EUREKA!:. No prximo nmero publicaremos algumas delas.
Alessandra A. da Gama Gomes de A. Anderson Torres Bruno Borges de Souza Lima Bruno de Souza Ramos Carlos Jos Amorim da Silva Diego Alonso Teixeira Evandro Makiyama de Melo Filipe Rodrigues de Souza Moreira Geraldo Perlino Jr. Gibran Medeiros de Souza Helainy Igncio de Almeida Torres Helder Oliveira de Castro Karla Detagne Santos Leonardo Freitas de Lima Luiz Srgio Carvalho de Mello Marcelo Ribeiro de Souza Marcelo Rufino de Oliveira Marclio Miranda de Carvalho Mauro Flix de Souza Okakamo Matsubachi Raquel Teresa de Souza Gomes Renato Francisco Lopes Mello Wallace Alves Martins Rio de Janeiro RJ So Paulo SP Goinia GO Rio de Janeiro RJ Rio de Janeiro RJ Santos SP So Paulo SP Rio de Janeiro RJ Cotia SP Natal RN Rio de Janeiro RJ Mogi das Cruzes SP Rio de Janeiro RJ Rio de Janeiro RJ Rio de Janeiro RJ Rio de Janeiro RJ Belm PA Teresina PI Rio de Janeiro RJ So Paulo SP Rio de Janeiro RJ J. dos Guararapes PE Rio de Janeiro RJ Prob. 119. Prob. 4, 16, 28, 31, 36, 46, 47, 53, 57, 59, 65,66, 68, 78, 80, 85, 88, 89, 95, 104, 109, 121, 123, 130,135,137, 145. Prob. 102, 107, 147. Prob. 127, 128, 131, 132, 134, 138, 139, 143, 144. Prob. 133, 138, 143. Prob. 121. Prob. 51, 121, 123, 128, 131, 133, 135, 142, 147. Prob. 64, 121, 128, 131, 133, 135, 138, 145, 147. Prob. 121, 122, 125 a 139, 141 a 144, 146 a 148. Prob. 138. Prob. 135, 137, 147. Prob. 32, 57, 68, 69, 76, 87, 98, 121,127, 132, 138, 139. Prob. 128, 138. Prob. 138. Prob. 8, 47, 69, 127, 128, 132, 143. Prob. 121, 122, 124, 126, 127, 128, 130, 132, 133, 134, 138, 143, 147. Prob. 121 a 150. Prob. 66, 87, 106. Prob. 121, 128, 134. Prob. 130. Prob. 128, 133, 143. Prob. 127, 132, 143. Prob. 121, 128, 130, 133, 138.

EUREKA! N14, 2002

52

Sociedade Brasileira de Matemtica

SOLUES DE PROBLEMAS PROPOSTOS  Publicamos aqui algumas das respostas enviadas por nossos leitores.
57) Dado n nmeros reais x1, x2 , xn satisfazendo as condies x1 ++ xn = 0 e 1 2 x12 + ... + x n = 1 , prove que existem i e j tais que xi x j . n
SOLUO DE ZOROASTRO AZAMBUJA NETO (RIO DE JANEIRO RJ):

Podemos supor que x1 ,..., x k 0, x k +1 ,..., x n < 0. Temos

i =1 k i =1

xi2 + xi2

j = k +1

2 j

=1=

nk k + . n n

Assim,

nk ou n

j = k +1

2 j

k . Supomos sem perda de generalidade que n

x
i =1 k

2 i

nk (o outro caso anlogo). Assim, se xi para 1 i k , temos n nk , n


j

i =1 n

xi

pois

i =1

xi2

i =1

xi .

Como

j = k +1

( x j ) = x j0

x ,
i i =1

temos Tomando 1 ,e n

j = k +1

( x
1 i k

nk , n

donde

existe

k + 1 j 0 n com

1 . n

= max x i , temos que existe i0 k com xi0 = , donde x i0 x j0 = ( x j0 )


1 logo xi0 x j0 . n

63) Prove que existem infinitos nmeros naturais mltiplos de 51000 sem nenhum 0 na representao decimal.
SOLUO DE EDUARDO CASAGRANDE STABEL (PORTO ALEGRE RS):

Provaremos por induo que existe um nmero de k algarismos, todos diferentes de 0 (zero), divisvel por 5k. Para k = 1, tome a1 = 5; para k = 2, tome a2a1 = 25. Agora suponhamos que seja n = ak 1ak 2a2a1 divisvel por 5k 1.
EUREKA! N14, 2002

53

Sociedade Brasileira de Matemtica

Temos a k 2 k 1 +

a k a k 1 ...a 2 a1 5
k 1

a k 10 k 1 + a k 1 ...a 2 a1 5
k 1

= a k 2 k 1 +

n 5
k 1

preciso

que

n 5
k 1

0 mdulo 5 para que ak ak 1a2a1 seja mltiplo de 5k. A equao n


k 1

equivalente a a k a k 2 k 13 k 1

5 Podemos escolher um tal ak no conjunto {1, 2, 3, 4, 5}, pois a temos um conjunto de restos da diviso por 5. Para concluir, basta tomar k = 1000, 1001, 1002, 64) Iniciando de um certo inteiro positivo, permitido fazer apenas uma operao: o dgito das unidades separado e multiplicado por 4, e ento este valor somado ao restante do nmero. Por exemplo, o nmero 1997 transformado para 7.4 + 199 = 227. A operao feita repetidamente. Prove que se a seqncia de nmeros obtida contm 1001, ento nenhum dos nmeros na seqncia pode ser um nmero primo.

3 k 1 mdulo 5.

ADAPTAO DA SOLUO DE MARCELO RIBEIRO DE SOUZA (RIO DE JANEIRO RJ):

Vamos ver que todos os termos da seqncia so mltiplos de 13, caso a seqncia contenha 1001 (que mltiplo de 13). De fato, se bn = 10kn + rn , com 0 rn 9, o prximo termo ser bn + 1 = kn + 4rn, ou seja, bn = 10(bn +1 4rn ) + rn = 10bn +1 39rn . Assim, bn mltiplo de 13 bn + 1 mltiplo de 13, ou nenhum deles . Como os termos antes de 1001 so maiores que 1001 e, partindo de 1001, obtemos a seqncia 1001 104 26 26 , todos os termos da seqncia so mltiplos de 13, mas nenhum igual a 13, donde nenhum primo. 65) Determine todos os inteiros N tais que, em base 10, os dgitos de 9N so os mesmos dgitos de N na ordem inversa, e N possui no mximo um dgito igual a 0.
SOLUO DE DANIEL DE SOUZA RAMOS (PIRASSUNUNGA SP):

Seja N um nmero de n dgitos na base 10, com no mximo um dgito igual a 0. Sabemos pelo enunciado definio do problema que 9N tem n dgitos. Da temos que ao multiplicarmos N por 9, 9N s ter n dgitos se o primeiro dgito de N for 1. Isto implica que o ltimo dgito de 9N 1. Mas tal fato s ser possvel se o ltimo dgito de N for 9. Assim N = 19 e 9N = 91. Imaginemos a multiplicao por 9. Para que o primeiro algarismo de 9N seja 9 necessrio que o dgito anterior ao dgito 1 de N seja 0 ou 1, uma vez que qualquer dgito maior do que 1, em tal posio, far com que 9N tenha um nmero de dgitos diferente do de N.
EUREKA! N14, 2002

54

Sociedade Brasileira de Matemtica

Analisemos ento os 2 casos: 1o. Caso) N = 119 (9N = 911); 0 k 9, k  9k + 8 1(mod 10) 11... k 9 k 7(mod 10) k = 7 9 9k...11 Da: N = 1179(9N = 9711), 0 k', k'' 9, k', k'' 
k '' k' 8

9 1 + k ' 7(mod 10) 1 1 ... .7 9 k ' 2 8(mod 10) k ' = 8 k ' ' = 1 9 9 7....1 1

Porm com k'' = 1 no ser possvel que 9N seja da forma 9711. Portanto o caso N = 119 no satisfaz. 2o. Caso) N = 109 (9N = 901); 0 k 9, k  9k + 8 0(mod 10) 10... k 9 k 8(mod 10) k = 8 9 9k...01 Da: N = 1089(9N = 9801), 0 k', k'' 9, k', k''  (4)
8

Para que esta situao acontea, k = 8 1 0 k ....k ' 8 9 ou k = 9 (pois apenas 9 8 pode, somado a 8 chegar a 80, 9 e 9 9 = 81). 9 8 k '....k 0 1
Para o caso k = 8 [k = 8 k' = 0 pois 9k' + 8 8 (mod 10) k' 0(mod 10)]
8 8 8 8

8*

Neste caso o nmero N possuiria mais de um dgito 1 0 8 ... .0 8 9 0, no condizendo com a situao proposta. 9 9 8 0....8 0 1 Para o caso k = 9:
EUREKA! N14, 2002

55

Sociedade Brasileira de Matemtica

9k '+8 9(mod 10) 1 0 9... .k ' 8 9 k ' 9(mod 10) k ' = 9 9 9 8 k '....9 0 1 Temos agora que N = 1 0 99 8 9. Desenvolvendo:
8 8 8 8

8 8

1 0 9 k ... k ' 9 8 9 Para que esta situao acontea 9 k = 8 ou k = 9 9 8 0k '....k 9 0 1 Para o caso k = 8.
8 8 8

1 0 9 8... k ' 9 8 9 9k' + 8 8(mod 10) 9 k ' 0(mod 10) k ' = 0 9 8 9 k '....8 9 0 1 No satisfaz o problema pois possui mais de um dgito 0. Para o caso k = 9.
8 8 8 8 8

1 0 9 9... k ' 9 8 9 9k' + 8 9(mod 10) 9 k ' 9(mod 10) k ' = 9 9 8 9 k '....9 9 0 1 Temos agora que N = 1 0 9 9 9 9 8 9. Desenvolvendo:
8 8 8 8 8 8 8

1 0 9 9 k ... k ' 9 9 8 9 9 9 9 8 9 k '....k 9 9 0 1 Procedendo de forma anloga ao momento em que sabia-se que N = 1 0 9 9 8 9, teremos N = 1 0 9 9 9 9 9 9 8 9. Conclui-se ento que todos os inteiros N que satisfazem a condio do problema so da forma: N = 10 9...9 89, n  n 4 .
( n 4 ) dgitos

EUREKA! N14, 2002

56

Sociedade Brasileira de Matemtica

67)

Este problema saiu com o enunciado errado, devido a um erro tipogrfico, como observaram Rodrigo Villard Milet, Anderson Torres e Carlos da Silva Victor, que enviaram verses corrigidas, com solues. Pedimos desculpas aos leitores pelo erro e publicamos as verses corrigidas a seguir:

VERSO DE RODRIGO VILLARD MILET (RIO DE JANEIRO RJ) e ANDERSON TORRES (SO PAULO SP):

Seja ABCD um quadriltero tal que os crculos circunscritos aos tringulos ABC e BCD so ortogonais. Prove que os crculos circunscritos aos tringulos ACD e ABD tambm so ortogonais. Vamos denotar por (XYZ) o crculo que passa por X, Y e Z e por (w1, w2) e ngulo entre as curvas w1 e w2. Considere uma inverso com centro em B e de razo k. Como (ABC) e (BCD) passam por B, ((ABC)) = A' C' e ((BCD)) = C'D'. Como ((ABC), (BCD)) = 90, segue que A'C'D' = 90. Temos (( ACD)) = ( A' C ' D' ). Da, A'D' dimetro de (A'C'D'), pois A'C'D' = 90. E assim, ((A'C'D'), A'D') = 90, logo ( ((A'C'D')), (A'D')) = ((ACD), (ABD)) = 90, pois (A'D') = (ABD)(A'D' no passa por B). Ento os crculos (ACD) e (ABD) so ortogonais.
VERSO DE CARLOS ALBERTO DA SILVA VICTOR (NILPOLIS RJ): SOLUO:

Seja ABCD um quadriltero tal que os crculos circunscritos aos tringulos ABC e ACD so ortogonais. Prove que os crculos circunscritos aos tringulos BCD e DAB tambm so ortogonais.
SOLUO:

Sejam O1, O2, O3 e O4 os centros dos crculos circunscritos aos tringulos ABC, ADC, ADB e BDC respectivamente. Sejam: O1CB = a ; O2 CD = b ; O1CA = ; DBC = e ; DBA = d ; BDA = c e BDC = f . Note que, pela hiptese do problema,

O2 AC = O2 CA = 90 . Podemos concluir que = O CB + O CA + O CA + O CD = a + + 90 b = a + b + 90 . C 1 1 2 2 1 Alm disso, fcil ver que O1 AB = 90 a e O2 AD = b, portanto O1 AB + O1 AC + O2 AC + O2 AD = (90 a ) + + (90 ) + b = = 180 a b. Veja tambm que DO4 O3 = e + f e DO3 O 4 = c + d . Observe
agora no tringulo DO3 O4 em que e + f + c + d = 180 O3 DO4 e, pelo
EUREKA! N14, 2002

57

Sociedade Brasileira de Matemtica

+B +D +C = 360 , ou seja e + f + c + d = 360 quadriltero ABCD temos A +A ) = 360 [90 + a + b + 180 a b] e + f + c + d = 360 270 = 90. (C

Concluso: O3 DO4 = 180 DO4 O3 DO3O4 = 180 (e + f + c + d ) = 90 e os crculos circunscritos aos tringulos BCD e DAB tambm so ortogonais.
Agradecemos tambm o envio das solues e a colaborao de:
Fbio Nunes Ribeiro Maia Anderson Torres Fbio Dias Moreira Marcelo Rufino de Oliveira Helder Oliveira de Castro Oswaldo Mello Sponquiado Renato Francisco Lopes Mello Marclio Miranda de Carvalho Evandro Makiyama de Melo Gibran Medeiros de Souza Carlos A. Gomes Jorge Silva Jnior Rio de Janeiro RJ So Paulo SP Rio de Janeiro RJ Belm PA Mogi das Cruzes SP Olimpia SP Jaboato dos Guararapes PE Teresina PI So Paulo SP Natal RN Natal RN Cachoeiro de Itapemirim ES

Seguimos aguardando o envio de solues do problema proposto N. 66 publicado na revista Eureka! N. 12

'4.H8,- ,

438 /070 :2 - 4 /0 3 20748 / 89 3948 08.7 948 .,/, :2 02 :2 /0 :2 - 4 /0 5,50 3 48 , , 5,50  02 :2 . ,5F: '4.H /0;0 709 7,7 :2 5,50 /0 .,/, ;0 '4.H /0;0 / 07 6:0 ;4.H 03.43974: 4 2, 47 /0 94/48 48 3 20748 4 4 ,5O8 709 7E
4  4 ;, 0 / 07 6:0 :2 4:974 3 2074 6:0 ;4.H E 9 3 , 709 7,/4 ,3908 F 4 2, 47  574-,- /,/0 /0 ;4.H ,.079,7 8:, ,1 72,9 ;, 5,70.0 2: 94 506:03, 3 4 '4.H 8,- , 6:0 ;4.H 54/0 ,/49,7 :2, 0897,9F , /0 24/4 6:0 , 574-,- /,/0 /0 ,.079,7 80 , 2, 47 6:0   '4.H /0;0 /08.,79,7 48 57 20 748 8 3 20748 43/0 8 F ,574 2,/,20390 3 0 0     F , .4389,390 /0 : 07 0 02 80 : /, 08.4 07 4 57O 24 3 2074 6:0 147 2, 47 6:0 94/48 48 ,3907 4708 '4.H 902 574-,- /,/0 2: 94 57O 2, /0  0 /0 ,.079,7

&2 574- 02, 8020 ,390 14 57454894 3, 89, /0 / 8.:88 4 /,  &2 3850947 8,-0 6:0 4 . 010 /0  -,3/ /48 F 4 2, 8 -, 3 4 /0 94/48 0 6:0 94/,8 ,8 , 9:7,8 8 4 / 10703908 $,-0
80 9,2-F2 6:0 0 08 089,7 4 570803908 3:2, 70:3 4 02 :2 0/ 1J. 4 3:2 3907;, 4 /0  2 3:948 424 4 3850947 3 4 8,-0 6:, /48 -,3/ /48 F 4 2, 8 -, 4 /0. /0 /0 ,7 8, 7 48 /4 8 57 20 748 -,3/ /48 0 5703/07 4 57 20 74 /48 80 : 3908 6:0 80 , 2, 8 -, 4 /0 94/48 6:0 8,J702 ":, , 574-,- /,/0 /0 4 3850947 5703/07 , 50884, .079,
 574-,- /,/0 /0 ,.079,7 F , 9, 2, 47 6:0   , .: 0
,
EUREKA! N14, 2002

58

Sociedade Brasileira de Matemtica

PROBLEMAS PROPOSTOS
Convidamos o leitor a enviar solues dos problemas propostos e sugestes de novos problemas para os prximos nmeros.

68)

Seja ABC um tringulo de lados inteiros e rea racional. Prove que existem pontos X, Y, Z com coordenadas inteiras no plano #2 tais que o tringulo XYZ congruente ao tringulo ABC. Sejam a e b inteiros positivos tais que an 1 divide bn 1 para todo inteiro positivo n. Prove que existe k  tal que b = ak.

69)

70)
P1

X5 P5 Q5 Q4 X4 Q3 Q1 Q2

X1

P2

X2

X3 P4 P3

Na figura acima, para 1 j 5, X j o ponto de interseo dos crculos circunscritos aos tringulos Qj 1 PjQj e Qj Pj + 1 Qj + 1 distintos de Qj (os ndices so tomados mdulo 5). Prove que o pentgono X1X2X3 X4X5 inscritvel. Obs: O pentgono P1P2P3P4P5 no necessariamente regular.

EUREKA! N14, 2002

59

Sociedade Brasileira de Matemtica

71)

Considere trs circunferncias, tangentes duas a duas. Prove que h apenas duas circunferncias tangentes s trs simultaneamente, e mostre como constru-las. Ache todas as funes f : x, y #.

72)

# # tais que

f ( x 4 + y ) = x 3 f ( x) + f ( f ( y )),

73)

Prove que, dado um inteiro positivo n, existe uma progresso aritmtica crescente formada por n inteiros positivos cujas somas dos dgitos tambm formam uma progresso aritmtica crescente, mas no existe uma progresso aritmtica infinita de inteiros positivos cujas somas dos dgitos formem uma progresso aritmtica crescente.

Problema 70 proposto por Jiang Zemin, presidente da China, a membros da direo da Unio Internacional de Matemtica, durante uma reunio preparatria do Congresso Internacional de Matemtica (ICM), realizado em agosto de 2002, em Beijing, China; Problema 71 proposto por Marcelo Ribeiro de Souza (Rio de Janeiro RJ); Problema 72 adaptado da 14a. Asian Pacific Mathematical Olympiad; Problema 73 adaptado do 17o.Torneio das Cidades, 1995.
EUREKA! N14, 2002

60

Sociedade Brasileira de Matemtica

AGENDA OLMPICA
XXIV OLIMPADA BRASILEIRA DE MATEMTICA NVEIS 1, 2 e 3 Primeira Fase Sbado, 8 de junho de 2002 Segunda Fase Sbado, 14 de setembro de 2002 Terceira Fase Sbado, 19 de outubro de 2002 (nveis 1, 2 e 3) Domingo, 20 de outubro de 2002 (nveis 2 e 3 - segundo dia de prova). NVEL UNIVERSITRIO Primeira Fase Sbado, 14 de setembro de 2002 Segunda Fase Sbado, 19 e Domingo, 20 de outubro de 2002

VIII OLIMPADA DE MAIO 11 de maio de 2002

XIII OLIMPADA DE MATEMTICA DO CONE SUL 22 a 28 de junho de 2002 Fortaleza CE, Brasil

XLIII OLIMPADA INTERNACIONAL DE MATEMTICA 18 a 31 de julho de 2002 Glasgow, Reino Unido

XVII OLIMPADA IBEROAMERICANA DE MATEMTICA 30 de setembro a 5 de outubro de 2002 El Salvador

V OLIMPADA IBEROAMERICANA DE MATEMTICA UNIVERSITRIA 9 de novembro de 2002

EUREKA! N14, 2002

61

Sociedade Brasileira de Matemtica

COORDENADORES REGIONAIS
Alberto Hassen Raad Amarsio da Silva Arajo Benedito Tadeu Vasconcelos Freire Carlos Frederico Borges Palmeira Claudio Arconcher Claus Haetinger Cleonor Crescncio das Neves lio Mega Florncio Ferreira Guimares Filho Gisele de Arajo Prateado Gusmo Irene Nakaoka Ivanilde Fernandes Saad Jacqueline Fabiola Rojas Arancibia Joo Bencio de Melo Neto Joo Francisco Melo Libonati Jos Carlos Pinto Leivas Jos Cloves Saraiva Jos Gaspar Ruas Filho Jos Luiz Rosas Pinho Jos Vieira Alves Licio Hernandes Bezerra Luzinalva Miranda de Amorim Marcelo Rufino de Oliveira Marcondes Cavalcante Frana Pablo Rodrigo Ganassim Ramn Mendoza Reinaldo Gen Ichiro Arakaki Ricardo Amorim Roberto Vizeu Barros Rosngela Souza Srgio Cludio Ramos Tadeu Ferreira Gomes Toms Menndez Rodrigues Valdenberg Arajo da Silva Wagner Pereira Lopes
EUREKA! N14, 2002

(UFJF) (UFV) (UFRN) (PUC-Rio) (Colgio Leonardo da Vinci) (UNIVATES) (UTAM) (Colgio Etapa) (UFES) (UFGO) (UEM) (UC. Dom Bosco) (UFPB) (UFPI) (Grupo Educacional Ideal) (UFRG) (UFMA) (ICMC-USP) (UFSC) (UFPB) (UFSC) (UFBA) (Grupo Educacional Ideal) (UFC) (Liceu Terras do Engenho) (UFPE) (INPE) (Centro Educacional Logos) (Colgio Acae) (Colgio Singular) (IM-UFRGS) (UEBA) (U. Federal de Rondnia) (U. Federal de Sergipe) (CEFET GO)

Juiz de Fora MG Viosa MG Natal RN Rio de Janeiro RJ Jundia SP Lajeado RS Manaus AM So Paulo SP Vitria ES Goinia GO Maring PR Campo Grande MS Joo Pessoa PB Teresina PI Belm PA Rio Grande RS So Luis MA So Carlos SP Florianpolis SC Campina Grande PB Florianpolis SC Salvador BA Belm PA Fortaleza CE Piracicaba SP Recife PE SJ dos Campos SP Nova Iguau RJ Volta Redonda RJ Santo Andr SP Porto Alegre RS Juazeiro BA Porto Velho RO So Cristovo SE Jata GO

62

CONTEDO

AOS LEITORES XLIII OLIMPADA INTERNACIONAL DE MATEMTICA Enunciados, Solues e Resultado Brasileiro XVII OLIMPADA IBEROAMERICANA DE MATEMTICA Enunciados, Solues e Resultado Brasileiro

2 3

15

ARTIGOS
A FRMULA DE CARDANO ALM DAS CBICAS Jos Cloves Verde Saraiva, So Luis - MA RECIPROCIDADE QUADRTICA Carlos Gustavo T. de A. Moreira & Nicolau Coro Saldanha, Rio de Janeiro - RJ APLICAES DE PLANOS PROJETIVOS EM TEORIA DOS NMEROS E COMBINATRIA Carlos Yuzo Shine, So Paulo - SP OLIMPADAS AO REDOR DO MUNDO SOLUES DE PROBLEMAS PROPOSTOS PROBLEMAS PROPOSTOS COORDENADORES REGIONAIS 24

27

31 43 54 59 61

Sociedade Brasileira de Matemtica

AOS LEITORES
Chegamos a esta ltima edio do ano 2002 muito contentes com o desempenho olmpico do Brasil: Pelo segundo ano consecutivo todos os integrantes da equipe brasileira ganharam medalha na Olimpada Internacional de Matemtica, e alm disso tivemos excelentes resultados na Olimpada de Matemtica do Cone Sul e na Olimpada Iberoamericana de Matemtica, onde ganhamos a maioria das medalhas de ouro em disputa. Publicamos aqui as solues dos problemas da IMO e da Ibero, sendo a maioria delas dos membros das equipes brasileiras. Agradecemos mais uma vez a crescente colaborao dos leitores, enviando problemas propostos e solues, e pedindo que publiquemos solues de problemas de vrias fontes, como das Olimpadas ao Redor do Mundo. Esse intercmbio fundamental para ns, e ajuda a manter a revista Eureka! viva e interessante.

Abraos e feliz 2003 para todos!

Os editores.

EUREKA! N15, 2002

Sociedade Brasileira de Matemtica

XLIII OLIMPADA INTERNACIONAL DE MATEMTICA


Enunciados, Solues e Resultado Brasileiro A XLIII Olimpada Internacional de Matemtica foi realizada na cidade de Glasgow, Reino Unido no perodo de 18 a 31 de julho de 2002. A equipe brasileira foi liderada pelos professores Edmilson Motta (So Paulo SP) e Ralph Costa Teixeira (Niteri RJ). O Resultado da Equipe Brasileira BRA 1 BRA 2 BRA 3 BRA 4 BRA 5 BRA 6 Alex Corra Abreu Larissa Cavalcante Queiroz de Lima Guilherme Issao Camarinha Fujiwara Yuri Gomes Lima Davi Mximo Alexandrino Nogueira Thiago da Silva Sobral Bronze Prata Bronze Bronze Bronze Bronze

PRIMEIRO DIA DURAO: 4 horas e meia. PROBLEMA 1

Seja n um inteiro positivo. Seja T o conjunto de pontos (x; y) no plano onde x e y so inteiros no negativos e x + y < n. Cada ponto de T pintado de vermelho ou azul. Se um ponto (x; y) vermelho, ento todos os pontos (x'; y') com x' x e y' y tambm so. Um conjunto X um conjunto de n pontos azuis com abcissas todas distintas, e um conjunto Y um conjunto de n pontos azuis com ordenadas todas distintas. Prove que o nmero de conjuntos X igual ao nmero de conjuntos Y.
SOLUO DE GUILHERME FUJIWARA (SO PAULO SP)

Primeiramente, seja x(i) o nmero de pontos azuis cuja ordenada i, e y(i) o nmero de pontos azuis de abscissa i. Veja que o nmero de X-conjuntos

i=0

n 1

y (i) , e o nmero de Y-conjuntos

x(i) .
i=0

n 1

Para provar que o nmero de X-conjuntos igual ao nmero de Y-conjuntos, suficiente provar que os nmeros x(0), x(1), x(2),...,x(n 1) so uma permutao dos nmeros y(0), y(1), y(2),..., y(n 1). Provaremos este lema por induo no n. Se n = 1, temos que x(0) = y(0) = 1 ou 0 dependendo se (0; 0) azul ou no. Suponhamos que o lema verdadeiro para n < k, provaremos para n = k. Vamos olhar para a ltima diagonal de T (reta x + y = k 1): Se nela no houver pontos vermelhos, ento tome T' como os conjuntos de pontos de
EUREKA! N15, 2002

Sociedade Brasileira de Matemtica

T que no esto na ltima diagonal. Temos que o lema vale para os x'(i) e y'(i) de T', e como x(i) = x'(i) + 1 e y(i) = y'(i) + 1, ento x'(a) = y'(b) x(a) = y(b), e alm disso x(k 1) = y(k 1) = 1, portanto o lema vale para T (vide fig. 1).
Y (ordenada)
Fig. 1

X (abscissa) S pontos azuis na reta y + x = k 1

Se nela houver algum ponto vermelho, digamos (a; k 1 a), ento aplicamos a hiptese de induo nos dois conjuntos T' formados acima e direita de T, que so menores que T, e assim demonstramos o lema para T (vide fig. 2). Provamos ento o nosso lema e, como j foi visto anteriormente, segue o que pedido no enunciado.
Y (ordenada)
Fig. 2

2 conjuntos T' menores que T, no qual aplicamos a hiptese de induo.

X (abscissa)

PROBLEMA 2
EUREKA! N15, 2002

Sociedade Brasileira de Matemtica

Seja BC um dimetro do crculo de centro O. Seja A um ponto de tal que 0 0 < AOB < 120 0 . Seja D o ponto mdio do arco AB que no contm C. A reta que passa por O e paralela a DA encontra a reta AC em J. A mediatriz de OA corta em E e F. Prove que J o incentro do tringulo CEF.
SOLUO DE YURI GOMES LIMA (FORTALEZA CE)

Como EF perpendicular ao raio OA, os arcos EA e AF so iguais, donde ECA = ACF ECJ = JCF. Portanto, J j pertence bissetriz de ECF. Se mostrarmos ento que AJ = AE, acabou (pois sabemos que o mdio do arco EF eqidista de E, de F e do incentro de CEF). Mas, se l a paralela a DA por O, temos que l // DA e DO // AC ( pois DOB = AOB/2 = ACB ) DAJO paralelogramo AJ = DO. Tambm, sendo EF mediatriz de OA, segue que EA = EO. Mas EO = raio de = DO , donde AJ = EA = AF. OBS: necessrio AOB < 120, pois caso contrrio teramos AOC 60 e BOD 60, o que implicaria J = l AC fora de , no podendo ento ser este o incentro de CEF. Encontre todos os pares de inteiros m, n 3 tais que h infinitos inteiros positivos a am + a 1 para os quais n inteiro. a + a2 1
SOLUO DE ALEX CORRA ABREU (NITERI RJ) PROBLEMA 3

Seja P = P ( x ) = x m + x 1 e Q( x) = x n + x 2 1 T , R [ x] tq P( x) = T ( x) Q( x) + R( x), P (a ) R (a ) R (a ) P (a ) com deg R < deg Q ou R 0 = T ( a) + = T ( a) inteiro Q (a ) Q(a ) Q (a ) Q (a ) para infinitos a's mas se R 0 n tq x > n 0 < deg R < deg Q
R(a) Q( a)

R( x) < 1 pois Q( x)

R( a) < 1 se a > n existe apenas um nmero finito de a's tq Q( a)

inteiro R 0 P( x) = T ( x) Q( x) Q( x) P( x) Q( x) P( x) ( x + 1) Q( x) =

= x m +1 + x m + x 2 1 x n x 2 + 1 = x n ( x m n +1 + x m n 1) ; note que temos, m n


EUREKA! N15, 2002

Sociedade Brasileira de Matemtica

(pois Q ( x ) | P( x)) mas mdc (Q( x), x n ) = mdc( x 2 1, x n ) = 1 Q( x) x mn +1 + x mn 1 por raciocnio anlogo ao anterior, m n + 1 n 2n m + 1( I ). Agora, como Q (0) = 1 , Q (1) = 1 0 < < 1 tq Q ( ) = 0 P ( ) = T ( )Q ( ) = 0 n + 2 1 = 0 = m + 1( II ) de (I), 2 n m+1 ( III ) por (II), (1 2 ) 2 m = (1 ) (1 ) 2 (1 + ) 2 (1 ) (1 )(1 + ) 2 pois < 1

3 + 2 1 0 3 1 2 = n n 3 mas n 3 n = 3 e a desigualdade s no estrita em (III) se m + 1 = 2n m = 5 logo se existir uma soluo ser (5, 3), que de fato uma soluo, pois obviamente x 5 + x 1 = x 2 x + 1 x 3 + x 2 1 P ( x ) = ( x 2 x + 1) Q ( x ).

)(

SEGUNDO DIA DURAO: 4 horas e meia. PROBLEMA 4

Seja n inteiro maior que 1. Os divisores positivos de n so d1, d2,,dk , onde 1 = d1 < d 2 < ... < d k = n Seja D = d1d2 + d2d3 ++dk 1 dk. (a) Prove que D < n2. (b) Encontre todos os valores de n para os quais D um divisor de n2.
SOLUO DE THIAGO DA SILVA SOBRAL (FORTALEZA CE)

Lema: d i =

n d k +1i

Prova: Observando que se d divisor de n ento n/d tambm o , podemos agrupar os divisores aos pares, donde conclumos que d i d k +1i = n , 0 i k . Esse fato tambm vale para n sendo quadrado perfeito, pois d k +1 = d
2 k +1 k +1 2

= n

Corolrio: d i

n k +1 i

EUREKA! N15, 2002

Sociedade Brasileira de Matemtica

Prova: De fato, sendo d k +1i o (k + 1 i)-simo divisor de n, temos d k +1i k + 1 i , e segue o resultado pelo lema. Pelo corolrio, k 1 1 1 n n n n nn D = d1 d 2 + d 2 d 3 + + d k 1 d k + + = n2 = 21 k k 1 k 1 k 2 j =1 j j + 1

=n

j j + 1 =n
j =1

k 1

1 2 1 < n k

b) Veja que se n primo, D = 1 n , e assim temos que D | n 2 . Suponha n composto, e seja p o seu menor fator primo. Pelo lema, n n2 D = d1 d 2 + d 2 d 3 + + d k 1 d k = 1 p + + n > p p

n2 n2 o maior divisor de n 2 menor que n 2 , < D < n 2 , e como p p 2 conclumos que D / |n Por fim, conclumos que D | n 2 n primo.
Veja ento que
PROBLEMA 5

Encontre todas as funes f de # em # tais que ( f ( x ) + f ( z ))( f ( y ) + f (t )) = f ( xy zt ) + f ( xt + yz ) para todo x, y, z, t #.


SOLUO DE LARISSA CAVALCANTE QUEIROZ DE LIMA (FORTALEZA CE)

( f ( x ) + f ( z ))( f ( y ) + f (t )) = f ( xy zt ) + f ( xt + yz )

x, z, y , t 0 : (2 f (0)) ( 2 f (0)) = 2 f (0) 2 f (0) 2 = f (0) 1 f (0) 0 2 f (0) = 1 f (0) = 2 z, y , t 0 : ( f ( x ) + f (0))( 2 f (0)) = 2 f (0) 1 1 1 se f (0) 0 (ou seja f (0) = ) , f ( x ) + f (0) = 1 f ( x) = 1 = x # 2 2 2
EUREKA! N15, 2002

Sociedade Brasileira de Matemtica

Se f (0) 0, ento f (0) = Suponha ento f (0) = 0

1 1 e f ( x) = , x # (o que uma soluo). 2 2

f (a ) = f ( x 2 + y 2 ) = [ f ( x ) + f ( y ) ]2 0 , contradio! Assim, f (a ) 0, a 0. z z ( f ( x ) + f ( z ))( f ( y ) + f (t )) = f ( xy + zt ) + f ( xt zy ) = ( f ( x) + f ( z ))( f (t ) + f ( y )) f ( x ) + f ( z ) = f ( x ) + f ( z ) f ( z ) = f ( z ) f ( z ) = f ( 1) f ( z ) f ( 1) = 1 = f (1). z, y , t 1 : ( f ( x ) + 1) 2 = f ( x 1) + f ( x + 1) 2 f ( x ) + 2 = f ( x 1) + f ( x + 1) . * 4 f ( xy ) = f ( 2 xy ) = f ( 2) f ( xy ) f (2) = 4. * conjectura: f ( x) = x 2 ( x 2 + z 2 )( y 2 + t 2 ) = ( xy zt ) 2 + ( xt + yz ) 2

[ f ( x ) + f ( y )]2 = f ( x 2 + y 2 ) = f ( x) 2 + 2 f ( x ) f ( y ) + f ( y ) 2 z x; t y ( 2 f ( x))( 2 f ( y )) = f ( xy xy ) + f ( xy + xy ) 4 f ( x ) f ( y ) = f ( 2 xy ) t , z 0 ( f ( x) + f (0))( f ( y ) + f (0)) = f ( xy 0) + f (0 + 0) f multiplicativa. f ( x ) f ( y ) = f ( xy ) 4 f ( xy ) = f (2 xy ) y t; t y ( f ( x ) + f ( z ))( f (t ) + f ( y )) = f ( xt zy ) + f ( xy + tz ) f ( xt zy ) + f ( xy + tz ) = f ( xt + zy ) + f ( xy zt ) Suponhamos que f no seja identicamente nula (note que f(x) 0 uma soluo). Suponha f (a ) < 0 e que x, y : x 2 + y 2 = a

2 z y; t x [ f ( x ) + f ( y ) ] = f ( xy xy ) + f ( x 2 + y 2 )

x 2 y 2 + x 2 t 2 + z 2 y 2 + z 2 t 2 = x 2 y 2 + z 2 t 2 2 xyzt + x 2 t 2 + 2 xtyz + y 2 z 2 Ok!! A funo f ( x) = x 2 funciona!!! 2 f ( x ) + 2 = f ( x 1) + f ( x + 1) ( f (1) = 1; f (2) = 4) f (m) = m 2 m n, m . 2 f (n) + 2 = f (n 1) + f (n + 1) f (n + 1) = 2 n 2 + 2 (n 1) 2 = n 2 + 2n + 1 = ( n + 1) 2


EUREKA! N15, 2002

Sociedade Brasileira de Matemtica

f (n) = n 2 , n  . como f ( n) = f (n); temos f ( z ) = z 2 , z . p Tome r " ; r = , mdc( p, q) = 1; p, q , q 0 q p f (r ) = f q = f ( p) 1 f (1) f ( p) p 2 f f ( p ) = = = 2 = (r) 2 q f (q ) f (q ) q

f (r ) = r 2 , r " .

( f ( x ) + f ( r ))2 f ( r ) = f ( x r + r 2 ) + f ( x r r 2 ) 2 f ( x ) f ( r ) + 2 f ( r ) 2 = f ( x r + r 2 ) + f ( xr r 2 ) r " 2 f ( y ) + 2r 4 = f ( y + r 2 ) + f ( y r 2 ), y #, r ". Suponha que y , r tais que y 0 , r > 0 e 2 f ( xr ) + 2r 4 = f ( xr + r 2 ) + f ( xr r 2 ) x # ,

[ ( y ) + f (r ) ] < f ( y ) f ( y ) + 2 f ( y ) f ( r ) + f ( r )
f
2 2 2 0 >0 >0 2

f ( y + r 2 ) < f ( y)

<f

( y ) , Absurdo!!!
2

f ( y + r ) > f ( y ) y # f crescente em #+. Vamos mostrar que f contnua em #+: note que se y, r 0, f(y +r2) f(y) = se r " . Assim, dado > 0 , para r0 " suficientemente pequeno, temos f ( y + r02 ) f ( y ) < . e se r < r0 , temos f ( y + r 2 ) f ( y ) < f ( y + r02 ) f ( y ) < se x y < r02 temos f ( x ) f ( y ) < ( x > y ) . * f ( y ) f ( y r02 ) = f ( y + r02 ) f ( y ) 2r04 < 2r04 < Se r < r0 ento y r 2 > y r02 f ( y r 2 ) > f ( y r02 ) f ( y ) f ( y r 2 ) < f ( y ) f ( y r02 ) < para x y < r02 temos f ( x ) f ( y ) < portanto f contnua. Suponha f ( w) = w 2 + , 0
EUREKA! N15, 2002

f ( y ) 2 + 2 f ( y ) f (r ) + f (r ) 2 f ( y ) = f ( r )(2 f ( y ) + f ( r )) = r 2 ( 2 f ( y ) + r 2 ),

Sociedade Brasileira de Matemtica

1) > 0 Temos que r0 > 0 tal que x w < r02 f ( x ) f ( w) < ou seja x 2 w 2 < caso x " mas se x < w temos x 2 < w 2 e portanto w 2 x 2 > 0 x 2 w 2 = w 2 + x 2 = w 2 x 2 + > , contradio!! (existe x " tal que 0 < w x < r02 ). 2) < 0 f ( w) = w 2 temos que r0 > 0 tal que x w < r02 f ( x) f ( w) < ou seja x 2 w 2 + < caso x " Note que x " tal que 0 < x w < r02 x > w x 2 > w 2 x 2 w 2 + = x 2 w 2 + > contradio!! f ( x ) = x 2 , x # . As nicas funes so: 1 f ( x ) ; f ( x) 0 ou f ( x) = x 2 , x # . 2
PROBLEMA 6

Sejam 1 , 2 ,..., n crculos de raio 1 no plano, onde n 3. Seus centros so O1, O2,,On, respectivamente. Suponha que no exista reta que intercepte mais que dois dos crculos. Prove que 1 (n 1) . 4 1 i < j n Oi O j

SOLUO DE LUCIANO GUIMARES CASTRO (RIO DE JANEIRO RJ)

Seja ij a medida, em radianos, do ngulo agudo formado pela reta OiOj com uma reta tangente a j passando por Oi. As circunferncias tm raio 1, e sen ij ij , i, j {1, 2,

, n} . Assim,

1 sen ij = ij . 1 i < j n Oi O j 1i < j n 1 i < j n


EUREKA! N15, 2002

10

Sociedade Brasileira de Matemtica

suficiente, portanto, provar que que o enunciado original.

1 i < j n

ij

(n 1) , o que parece menos assustador 4

Vejamos de que maneira a condio de nenhuma reta cortar mais de duas circunferncias limita a soma dos ij. Fixemos i. Para cada j, a unio de todas as retas que passam por Oi e cortam a circunferncia j forma dois ngulos opostos pelo vrtice Oi, cada um medindo 2ij. Como essas retas no cortam outra circunferncia alm de i e j, variando j obtemos ngulos disjuntos com vrtice Oi, de forma que a soma de suas medidas no pode ultrapassar 2, ou seja,
1 j n j i

ij

2 .
2

(1)

2 i2 2 i1

2 ij j

Oi

Agora, somando estas desigualdades para i {1, 2,

ij = ji , obtemos
1i n 1 j n j i EUREKA! N15, 2002

, n}

e observando que n . 4

ij

1i n

1 i < j n

ij

2 n

1i < j n

ij

11

Sociedade Brasileira de Matemtica

Hummmm... quase! De fato, este resultado j assintoticamente equivalente ao desejado. Apesar de que conseguir um n 1 no lugar daquele n a parte difcil deste problema, voc vai perceber que se trata apenas de ir adaptando esta primeira idia. Intuitivamente, o que acabamos de fazer foi girar uma reta 180o em torno de cada ponto Oi, sabendo que neste percurso ela cortar todas as outras circunferncias, mas nunca duas ao mesmo tempo. Para melhorar a estimativa, precisamos encontrar uma forma de girar menos que 180o e, ainda assim, encontrar todas as demais circunferncias. Isto no possvel para todos os Oi, mas podemos faz-lo com os mais afastados. Mais precisamente, nossa idia trabalhar com o fecho convexo do conjunto {O1, O2, ..., On}, ou seja, o menor conjunto convexo que contm {O1, O2, ..., On}. Sem perda de generalidade, podemos supor que esse fecho convexo o polgono O1O2O3... Om (m n) . Desta forma, os pontos Om+1, Om+2, ..., On so interiores ao polgono. Vamos separar a soma L= D= T= I=
1i < j n

ij

em quatro partes:

1 i m 1

i ,i +1

+ 1m (soma dos ij tais que OiOj lado do polgono);

1i < j m

ij

ij

L (soma dos ij tais que OiOj diagonal do polgono); (soma dos ij tais que Oi vrtice do polgono e Oj interior); (soma dos ij tais que Oi e Oj so interiores ao polgono).

1 i m m< j n

m<i< j n

ij

Observe que L + D + T + I =

1 i < j n

ij

Vamos usar os ngulos externos do polgono para limitar L. Sejam ai e ei, respectivamente, as medidas dos ngulos interno e externo do polgono no vrtice Oi. Para simplificar a notao, trabalharemos com o vrtice O2. Seja t a tangente comum exterior a 1 e 3, mais prxima de 2. Ento 2 est totalmente contida no semiplano determinado por t que no contm 1 e 3 (caso contrrio existiria uma reta cortando 1, 2 e 3). Sejam r a paralela a t por O2 e P1 o ponto de r tal que P1O2O1 agudo e sua bissetriz perpendicular reta O1P1. Ento a distncia de O1 OP a essa bissetriz igual a 1 1 . Esta distncia maior que 1, pois P1 pertence a r e a 2
EUREKA! N15, 2002

12

Sociedade Brasileira de Matemtica

distncia de O1 a r maior que 2. Isto significa que a bissetriz de P1O2O1 exterior m(P 1O2O1 ) . Procedendo de forma anloga com a 1, do que conclumos que 12 < 2 a circunferncia 3 e somando as duas desigualdades, temos a 2 e2 . = 12 + 23 < 2 2

P1 O2 12

O1

O3

Fazendo o mesmo para os demais vrtices do polgono e somando as desigualdades obtidas, conclumos que ei 2L < (2) 2L < . 1 i m 2 Agora utilizaremos os ngulos internos do polgono em um procedimento parecido ao que fizemos para descobrir a desigualdade (1). Salvo 1, 2 e 3, todas as demais circunferncias esto completamente contidas no ngulo interno 2 = O1O2O3. Para cada j 2, considere o conjunto unio das semi-retas com origem O2, interiores a 2, que cortam a circunferncia j. Este conjunto forma um ngulo de medida 2j, para j = 1 e j = 3, e um ngulo de medida 22j para os demais valores de j. Como cada semi-reta pode cortar apenas uma circunferncia alm de 2, os conjuntos correspondentes a distintos valores de j so disjuntos. Assim, 12 + 23 + 2 2 j a 2 .

4 i n

Procedendo de forma anloga com os demais vrtices do polgono e somando as desigualdades obtidas conclumos que 2 L + 4 D + 2T ai = ( m 2) . (3)
1 i m

EUREKA! N15, 2002

13

Sociedade Brasileira de Matemtica

Para concluir, seja Oi um ponto interior ao polgono, ou seja, i > m. J provamos a 4 ij 2 . desigualdade (1):
1 j n j i

Somando essas desigualdades para todos os pontos Oi interiores temos 4 ij 2 4T + 8I ( n m) 2 2T + 4 I ( n m) .


m < i n 1 j n j i

m<i n

(4)

Agora basta somar as desigualdades (2), (3) e (4), obtendo 2 L + 2 L + 4 D + 2T + 2T + 4 I + ( m 2) + (n m) ( n 1) 4 L + 4 D + 4T + 4 I (n 1) L + D + T + I . 4 Portanto 1 (n 1) = , sen ij ij = L + D + T + I 4 1i < j n Oi O j 1 i < j n 1 i < j n

como queramos demonstrar.

EUREKA! N15, 2002

14

Sociedade Brasileira de Matemtica

XVII OLIMPADA IBEROAMERICANA DE MATEMTICA


Enunciados, Solues e Resultado Brasileiro A XVII Olimpada Iberoamericana de Matemtica foi realizada na cidade de So Salvador, El Salvador no perodo de 30 de setembro a 5 de outubro de 2002. A equipe brasileira foi liderada pelos professores Eduardo Wagner (Rio de Janeiro RJ) e Onofre da Silva Farias (Fortaleza CE). O Resultado da Equipe Brasileira BRA 1 BRA 2 BRA 3 BRA 4 Guilherme Camarinha Fujiwara Humberto Silva Naves Larissa Cavalcante Queiroz de Lima Yuri Gomes Lima Ouro Ouro Ouro Prata

PRIMEIRO DIA DURAO: 4 horas e meia. PROBLEMA 1

Os nmeros inteiros desde 1 at 2002, ambos includos, escrevem-se num quadro por ordem crescente 1, 2,..., 2001, 2002. Em seguida apagam-se os que ocupam o primeiro lugar, quarto lugar, stimo lugar, etc, ou seja, os que ocupam os lugares da forma 3k + 1. Na nova lista apagam-se os nmeros que esto nos lugares da forma 3k + 1. Repetese este processo at que se apagam todos os nmeros da lista. Qual foi o ltimo nmero que se apagou?
SOLUO DE GUILHERME CAMARINHA FUJIWARA (SO PAULO - SP)

Considere uma sequncia infinita ao invs de uma sequncia at 2002. Seja ento R(k) o primeiro nmero a ser apagado na k-sima seo de apagamento. Temos ento R(1) = 1, R(2) = 2, R(3) = 3, R(4) = 5, R(5) = 8, R(6) = 12. Queremos ento achar o maior R(k) menor ou igual 2002. 3 Vamos provar que R(k + 1) = R( k ) (onde k o menor inteiro maior ou igual 2 a x). Para tal, basta ver que se considerarmos a seqncia que sobra aps a primeira srie de apagamentos, teremos que o R(k)-simo termo ser o primeiro a ser apagado na k-sima prxima srie de apagamento. Considerando tambm o primeiro apagamento, temos que o R(k)-simo nmero dessa lista sr o primeiro a ser apagado na (k + 1)-sima srie de apagamento, logo ser o R(k + 1).
EUREKA! N15, 2002

15

Sociedade Brasileira de Matemtica

fcil ver que o n-simo termo da sequncia que sobra aps a primeira srie de 3 apagamentos ser o n (basta ver os dois casos de paridade de n), logo temos 2 3 que R(k +1) = R( k ) . 2 Fazendo as contas, temos ento R(6) = 12, R(7) = 18, R(8) = 27, R(9) = 41, R(10) = 62, R(11) = 93, R(12) = 140, R(13) = 210, R(14) = 315, R(15) = 473, R(16) = 710, R(17) = 1065, R(18) = 1598 e finalmente R(19) = 2397. Como R(18) = 1598 e 2002 < R(19) = 2397, ento o ltimo nmero apagado foi 1598.
PROBLEMA 2

Dado qualquer conjunto de 9 pontos no plano entre os quais no existem trs colineares, demonstre que para cada ponto P do conjunto, o nmero de tringulos que tm como vrtices trs dos oito pontos restantes e P no seu interior par.
SOLUO DE HUMBERTO SILVA NAVES (SO JOS DOS CAMPOS - SP)

Seja S o conjunto dos 9 pontos. Se um ponto P est no interior do tringulo ABC com A, B, C e P S, temos: A

B'

C' P

A' B

Sejam A = AP BC ; B = BP C e C = CP B. As semi-retas PA, PB e PC dividem o plano em trs regies: , e (A , e C ). Vamos construir um grafo G cujos vrtices representam os tringulos (com vrtices em S) com o ponto P em seu interior. Ligaremos 2 vrtices deste grafo se e somente se os tringulos correspondentes tiverem um lado em comum. Vamos agora provar que o grau de cada vrtice 5. Seja Q S {A ; B ; C ; P}. Temos 3 possibilidades:
EUREKA! N15, 2002

16

Sociedade Brasileira de Matemtica

1) Se Q : Temos que o ponto P est no tringulo QBC, logo QBC V(G). Como os tringulos BC e QBC tem um lado em comum, eles esto ligados por uma aresta em G. Obs.: Claro que P no est no QAB e nem no QAC. 2) Se Q : QAC V(G) e est ligado ABC em G. 3) Se Q : QAB V(G) e est ligado ABC em G. Logo deg (ABC) = #(S {A ; B ; C ; P}) = 5. E como deg() = 2 #E(G) #V(G) par. Portanto o nmero de tringulos com P em seu interior par.
PROBLEMA 3

Um ponto P interior ao tringulo equiltero ABC e tal que APC = 120 . Sejam M a interseco de CP com AB e N a interseco de AP com BC. Encontrar o lugar geomtrico do circuncentro do tringulo MBN quando P varia.
SOLUO DE YURI GOMES LIMA (FORTALEZA CE)
B X O1 M P O N

Vamos mostrar que tal L.G. est contido na reta  mediatriz do raio BO , onde O o C = 120 , centro de ABC. Para isso, seja O1 o circuncentro de BMN. Da, como AP
EUREKA! N15, 2002

17

Sociedade Brasileira de Matemtica

M = 120 BNPM inscritvel. Para O1 pertencer a  , devemos ter temos NP BO1 = OO1 , ou seja, O tambm deve pertencer circunferncia circunscrita em BMPN . Vamos mostrar que O o mdio do arco MN . N = 60 NA C = AC M . C = 120 , temos ento que BA Agora, veja que, como AP Assim, ABN CAM . Logo, a rotao com centro em O e ngulo 120 que leva B em A e A em C tambm leva N em M. M = 120 e NO = MO . Mas sendo NO M = 120 , segue que BMON Assim, NO inscritvel. Assim, provamos o que queramos e O1 . N < 90 , pois O BN Mas o LG no a reta toda. De fato, devemos ter O B
1 1

issceles, da, se X e Y so os pontos de interseo de  com a circunferncia circunscrita a ABC, teremos que O1 pertence ao INTERIOR do segmento XY , C = YB A = 90 (para ver isto, observe que XBYO losango com pois XB B = 60 X o mdio do arco AB). XO = BO XO Agora, dado O1 pertencente ao interior do segmento XY , trace a circunferncia de centro O1 e raio O1 B (que passa por O). Ela determinar dois pontos M e N sobre AB , BC tais que N = 120 . Da, a mesma rotao considerada N = OB N = OB M = ON M e MO OM antes levar N em M, levando ento o ABN no CAM AN = CM e N = AC M 60 BA N = 60 AC M PA C + AC P = 60 , onde P = AN CM , BA C = 120. donde AP Logo, o L.G. procurado o interior do segmento XY .

SEGUNDO DIA DURAO: 4 horas e meia. PROBLEMA 4

Num tringulo escaleno ABC traa-se a bissectriz interna BD, com D sobre AC. Sejam E e F, respectivamente, os ps das perpendiculares traadas desde A e C at recta BD, e seja M o ponto sobre o lado BC tal que DM perpendicular a BC. Demonstre que EMD = DMF .

EUREKA! N15, 2002

18

Sociedade Brasileira de Matemtica

SOLUO DE YURI GOMES LIMA (FORTALEZA CE)

M A' E A D F C

Seja A' a interseo de AE com BC . Ento, como BE bissetriz, segue que AE = EA' . Mas ento os tringulos ADE e A'DE so congruentes, donde E = DA ' E (I). Como AE e CF so perpendiculares a BD , ento DA D = DC F (II) AE // CF EA Tambm: F = DM C = DM C = 90 o quadriltero MCFD inscritvel DC F. DF ' D. D = A' E D = 90 o quadriltero A'MED inscritvel EM D = EA A' M ' D = DC F Mas, por (I) e (II), temos que EA D = DM F . EM Obs. Por ABC ser escaleno, temos que BD no perpendicular a AC , i.e, E D F E.
PROBLEMA 5

A sucesso de nmeros reais a1 , a 2 ,... define-se como: 1 para cada inteiro n 1. a1 = 56 e a n +1 = a n an Demonstre que existe um inteiro k , 1 k 2002, tal que a k < 0.

EUREKA! N15, 2002

19

Sociedade Brasileira de Matemtica

SOLUO DE LARISSA CAVALCANTE QUEIROZ DE LIMA (FORTALEZA CE)

Lema: a k < m; m, ak > 0 1 a k +1 < m m 1 1 1 1 Prova: a k < m < < m ak ak m 1 1 1 ak < m a k +1 < m ak m m 1 1 1 * a k +1 = a k a k +1 2 = a k 2 2a k + 2 ak ak ak a k +1 2 = a k 2 + 1 ak
2

Soma telescpica 1 * a 2 2 = a1 2 + 2 2 a1 1 2 2 a3 = a 2 + 2 2 a2 1 (+) a k +1 2 = a k 2 + 2 2 ak 1 1 1 a k +1 2 = a1 2 + 2 + 2 + ... + 2 2 k a1 a2 ak 1) Suponha que i 1999 tal que ai < 2( ai > 0) 1 3 3 2 94 5 Isso implica: ai +1 < 2 = . Caso a i + 1 > 0 , ai + 2 < = = < 1. 2 2 2 3 6 6 1 Se ai + 2 > 0 ai +3 < 1 = 0; i + 3 1999 + 3 = 2002 1 k ,1 k 2002 tal que a k < 0. 2) Suponha que / i 1999 tal que ai < 2 1 1 1 1 ai 2 1 i 1999 2 2 ai 4 ai

EUREKA! N15, 2002

20

Sociedade Brasileira de Matemtica

a 2000 2 = 56 2 +

1 a1
2

+ ... +

1 a1999
2

2(1999) 56 2 + 1999

1 2 1999 4

1 7 7 a 2000 2 56 2 1999 2 = 3136 (2000 1) = 3136 3500 + < 0 4 4 4 Absurdo! Portanto i 1999 tal que ai < 2 temos que ai , ai +1 , a i + 2 ou ai + 3 menor que zero (por 1) k ,1 k 2002 tal que a k < 0.
PROBLEMA 6

Um polcia tenta capturar um ladro num tabuleiro de 2001 2001. Eles jogam alternadamente. Cada jogador, na sua vez, deve mover-se uma casa num dos trs seguintes sentidos:

(abaixo);

(direita);

(diagonal superior esquerda).

Se o polcia se encontra na casa da esquina inferior direita, pode usar a sua jogada para passar directamente para a casa da esquina superior esquerda (o ladro no pode fazer esta jogada). Inicialmente o polcia est na casa central e o ladro est na casa vizinha diagonal superior direita do polcia. O polcia comea o jogo. Demonstre que: a) O ladro consegue mover-se pelo menos 10000 vezes sem ser capturado. b) O polcia possui uma estratgia para capturar o ladro. Nota: O polcia captura o ladro quando entra na casa em que est o ladro. Se o ladro entra na casa do polcia, no h captura.
SOLUO OFICIAL: Pinte o tabuleiro com 3 cores da seguinte forma:

Figura1:
1 a b c a 2 b c a b 3 c a b c 4  1001 b c a


Movimentos
c c a c a b


a b 2001 c a b 1 2 3

b 2001

EUREKA! N15, 2002

21

Sociedade Brasileira de Matemtica

Observe que os movimentos nos do o seguinte ciclo: Quer dizer: de uma casa a s se vai para b, de uma b a b s se vai para c e de uma c s se vai para a.

c
Inicialmente o polcia comea numa casa c e o ladro tambm. (casas hachuradas na figura) Assim teremos as seqncias Polcia: c a b c O polcia nunca poderia entrar numa casa de mesma cor do ladro. Ladro: c a b c Para sua sorte existe o tnel. Se pensarmos um pouco, veremos que o polcia deve atravessar o tnel 2 vezes para poder tornar compatvel seu ciclo com o do ladro, ou seja, jogar e cair numa casa de mesma cor do ladro (podendo peg-lo). Seja X = casa superior esquerda Y = casa inferior direita. Logo o polcia precisa de 2000 movimentos para chegar at Y, cruza o tnel (1 movimento), mais 4000 para chegar de novo at Y, cruza o tnel (1 movimento). Neste momento o ladro deve estar prximo de Y e o polcia precisar de mais 3999 movimentos (pelo menos) para capturar o ladro (que ficar rodando no quadrado 2 2 inferior esquerdo, totalizando 2000 + 4000 + 3999 + 2 = 10001 movimentos do polcia, ou seja, 10000 movimentos do ladro. b) Vejamos agora uma estratgia para que o polcia prenda o ladro. Suponha que ele j tenha passado 2 vezes pelo tnel. Numere as linhas do tabuleiro de 1 a 2001, de cima para baixo e as colunas de 1 a 2001, da esquerda para a direita. Aps sair do tnel, o polcia se encontra na casa da linha 1 e coluna 1. A estratgia a seguinte: i) ii) O polcia deve se mover para a direita at que o ladro fique na mesma diagonal (inferior direita) do polcia ou uma casa direita. Em seguida, o polcia deve fazer o seguinte movimento:

EUREKA! N15, 2002

22

Sociedade Brasileira de Matemtica

Ladro Joga

Polcia Joga

iii)

O polcia deve repetir os passos i) e ii) Note que a diferena entre os nmeros das linhas do polcia e do ladro sempre diminui e o ladro sempre fica na regio direita da diagonal inferior direita do polcia. Assim, aps repetirmos i) e ii) um nmero finito de vezes, chegamos na situao (na vez do ladro) Polcia Ladro (note que no pode ocorrer outra situao na qual a diferena entre linhas 1, pois, na vez do ladro, os dois devem estar em casas do mesmo tipo). Seguindo o que feito em ii), o ladro, em algum momento, se move para a casa direita do polcia. Assim, na prxima jogada, o polcia prende o ladro.

EUREKA! N15, 2002

23

Sociedade Brasileira de Matemtica

A FRMULA DE CARDANO ALM DAS CBICAS


Jos Cloves Verde Saraiva, So Luis MA Nvel Avanado.
INTRODUO:

Motivado pela leitura do trabalho Equao do Terceiro Grau do Professor Alberto de Azevedo [1], ocorreu-me a curiosidade de saber as frmulas das razes calculadas por radicais de uma equao polinomial do 5 grau, solvel, que no fosse a trivial x 5 + a = 0 , que todos conhecem. Da ento, seguindo os mesmos passos da deduo da frmula de Cardano para as equaes polinomiais cbicas foi possvel provar que 1 a x 5 px 3 + p 2 x r = 0 , j estudada por DE MOIVRE, tem razes dadas por uma 5 frmula anloga a de Cardano. Alm desta, outras frmulas semelhantes so possveis deduzir para graus maiores que o quinto. Deixamos para o leitor essa generalizao!.
A FRMULA DE CARDANO:

fascinante toda a histria da resoluo das equaes polinomiais do 3 grau. Em resumo a referncia [2] apresenta o seguinte: "O descobridor do mtodo foi Scipione del Ferro (1465 - 1562), matemtico italiano, que antes de morrer o revelou aos discpulos Antnio Maria Fior e Annibale Della Nave". "Houve uma disputa matemtica entre Fior contra Niccolo Fontana (1500 1557), conhecido pelo apelido de Tartaglia (gago, em italiano). A vitria deste ltimo, muito divulgada, foi do conhecimento do mdico e professor Girolano Cardano (1501 1576) que conseguiu lhe atrair para ensinar a regra de resoluo sob o juramento de jamais public-la. Cardano procurou a demonstrao da regra - e achou - e ainda motivou seu discpulo Ludovico Ferrari (1522 1565) a descobrir soluo para as equaes do quarto grau." "Cardano, numa visita a Della Nave, soube do manuscrito de Del Ferro contendo a regra de Tartaglia que j existia h 30 anos. Motivo que o levou quebrar o juramento. Publicou os mtodos no seu famoso livro Ars Magna, em 1545, onde no deixou de fazer referncia aos descobridores, embora a contragosto de Tartaglia que se considerou trado." Podemos representar a equao geral do 3 grau na forma x 3 + a1 x 2 + a 2 x + a3 = 0 e a por uma mudana de varivel x = y 1 a equao fica mais simples na forma 3
EUREKA! N15, 2002

24

Sociedade Brasileira de Matemtica

y 3 py q = 0. Calculando o cubo de um binmio (u + v) 3 = u 3 + 3u 2 v + 3uv2 + v 3 , e pondo em evidncia 3uv, temos: (*) (u + v ) 3 = 3uv(u + v ) + (u 3 + v 3 ) ou melhor, (u + v ) 3 3uv (u + v) (u 3 + v 3 ) = 0 isto , y = u + v uma raiz para valores de p = 3uv e q = u3 + v3, onde podemos p3 elevar ao cubo a primeira e ter = u 3 v 3 e q = u 3 + v 3 e cair num problema onde 27 u3 e v3 so as razes de uma equao do 2 grau conhecendo a soma e o produto das razes, cuja soluo conhecida: q q2 p3 q q 2 p3 e v3 = + 2 4 27 2 4 27 donde obtemos a famosa frmula de Cardano: u3 = y =u+v= 3 q q2 p3 3 q q2 p3 + + 2 4 27 2 4 27

A FRMULA DE CARDANO ALM DAS CBICAS:

O resultado principal destas notas foi motivado por uma analogia da deduo na 1 frmula de Cardano. Vamos provar que: a equao x 5 px 3 + p 2 x r = 0 tem 5 uma de suas razes dada pela frmula: x=u +v=5
DEMONSTRAO:

r + 2

r2 p5 r r2 p5 + +5 + 4 3125 2 4 3125

Considere x 5 px 3 qx r = 0 . Calculemos polinmios do binmio: (u + v ) 5 = u 5 + 5u 4 v + 10u 3 v 2 + 10u 2 v 3 + 5uv 4 + v 5 pondo em evidncia obtemos: (u + v ) 5 = 5uv(u 3 + v 3 ) + 10u 2 v 2 (u + v) + (u 5 + v 5 ) da igualdade (*) obtemos que: (u 3 + v 3 ) = (u + v) 3 3uv(u + v) para substituir no desenvolvimento donde obtemos que: (u + v ) 5 = 5uv(u + v ) 3 15u 2 v 2 (u + v ) + 10u 2 v 2 (u + v) + (u 5 + v 5 ) , isto ,
EUREKA! N15, 2002

25

Sociedade Brasileira de Matemtica

(u + v ) 5 = 5uv(u + v ) 3 5u 2 v 2 (u + v ) + (u 5 + v 5 ) o que permite obter as igualdades: p = 5uv; q = 5u2v2 e r = u5 + v5. p2 faz com que a equao seja Estabelecemos p 2 = 25u 2 v 2 , logo temos que q = 5 1 da forma x 5 px 3 + p 2 x r = 0, se x = u + v for uma raiz. 5 r = u 5 + v 5 nos leva ao j estudado na deduo da frmula de Verificar as relaes p = 5uv p Cardano r = u 5 + v 5 e = u 5 v 5 , da mesma forma as razes so: 5 r r 2 p5 r r2 p5 + 5 e v5 = 2 4 5 2 4 55 de onde conclumos que a raiz x = u + v dada pela frmula: u5 =
x =
5

r + 2

r2 p5 + 4 3125

r 2

r2 p5 4 3125

OBSERVAES FINAIS:

Esta frmula torna mais fcil a determinao das razes do que a indicada por De Moivre estudada na referncia [3], onde uma anlise completa das razes e o estudo dos Grupos de Galois nos diversos casos feito. Como exerccio estude a stima x 7 + px 5 qx 3 + rx + s = 0 e generalize. Finalizando, seria interessante o leitor paciente calcular todas as razes da equao abaixo estudada por Adriaan van Roomen (1561 1615) por polinmios trigonomtricos. x 45 45 x 43 + 945 x 41 12300 x 39 + 111150 x 37 740259 x 35 + 3764565x 33 14945040 x 31 + 46955700 x 29 117679100 x 27 + 236030652 x 25 378658800 x 23 + + 483841800 x 21 488494125 x19 + 384942375 x17 232676280 x15 + 105306075 x13 34512075x 11 + 7811375 x 9 1138500 x 7 + 95634 x 5 3795 x 3 + 45 x = 2 + 2 (Ver referncia [4], pp 154).
REFERNCIAS:
[1] Alberto de Azevedo, Equao do 3 grau, Depto. Matemtica, UNB, 2002. [2] Csar Polcino Milies, A Resoluo das equaes de terceiro e quarto graus, Notas de aula, IMEUSP, 2000. [3] R.L. Borger, On De Moivre' s Quintic, American Math. Monthly, pp. 171 - 174, vol. 15, 1908. [4] Paulo A. Martin, Introduo Teoria dos Grupos e a Teoria de Galois, IME-USP, 1996.
EUREKA! N15, 2002

26

Sociedade Brasileira de Matemtica

RECIPROCIDADE QUADRTICA
Carlos Gustavo T. de A. Moreira & Nicolau C. Saldanha, Rio de Janeiro - RJ Nvel Avanado. A lei de Gauss de reciprocidade quadrtica afirma que se p e q so primos h uma relao direta entre p ser quadrado mdulo q e q ser quadrado mdulo p. Este teorema fornece um rpido algoritmo para determinar se a quadrado mdulo p onde a um inteiro e p um nmero primo. Lembramos que a quadrado mdulo n se existe x com x 2 a(mod n). Este artigo foi adaptado de [3]. a Definio: Seja p um primo e a um inteiro. Definimos o smbolo de Lagrange p por 0 se p divide a a p = 1 se a no quadrado mdulo p 1 se p no divide a e a quadrado mdulo p. Proposio: Seja p um primo mpar e a
p 1 2 (mod

tal que p no divide a.

a p). Ento p a Demonstrao: Sabemos que se p no divide a ento a p 1 1(mod p) , ou seja, x p 1 1 tem como razes 1, 2,, p 1 em
p 1 2

/p . Por outro lado,

x p 1 1 = x a
p 1 2

1 x

p 1 2

+ 1. Se existe b
p 1 2

tal que a b 2 (mod p) ento

a b p 1 1(mod p ); ou seja, =1 a p

(mod p ). p 1 quadrados em 2 1 em /p , donde os

Como x 2 y 2 (mod p) x y (mod p) , h pelo menos


p 1

( /p )*, logo os quadrados so exatamente as razes de x


p 1

no quadrados so exatamente as razes de x


p 1 2

b + 1 , ou seja, se p = 1 ento

1(mod p ).

EUREKA! N15, 2002

27

Sociedade Brasileira de Matemtica

1 2 Corolrio: Se p primo mpar ento = ( 1) . p Vamos agora reinterpretar a proposio. Seja j = 1, 2,...,

p 1

a( / p

)* .

Para

cada

p 1 p 1 escrevemos a j como j m j com j {1,1} e m j 1,2,..., . 2 2 Se mi = m j temos a i = a j ou a i = a j; a primeira possibilidade implica i = j e a segunda impossvel. Assim, se i j p 1 m1 ; m2 ;...; m p 1 = 1, 2,..., . Assim, 2 2 p 1 (a 1)(a 2)... a p 1 a 2 2 = a p p 1 1 2 ... 2 1 2 ... p 1 m1 m 2 ... m p 1 p 1 1 2 ... 2
2 2

temos mi m j donde

= 1 2 ... p 1 (mod p)
2

a a m donde = 1 2 ... p 1 , pois ambos pertencem a { 1, 1}. Assim, = ( 1) , p p 2 p 1 onde m o nmero de elementos j de 1, 2,..., tais que j = 1. Como 2 primeira conseqncia deste fato temos o seguinte resultado. Proposio: Se p um primo mpar ento p 2 1 2 1, se p 1 (mod 8), 1 = = ( ) 8 p 1, se p 3 (mod 8). Demonstrao: Se p 1(mod 4) , digamos p = 4k + 1, temos 1 2 j p 1 = 2k . Como 2

p 1 p 1 para j k e < 2 j p 1 para k + 1 j 2k , temos 2 2 a 1, se p 1 (mod 8), k p = ( 1) = 1, se p 5 (mod 8).

EUREKA! N15, 2002

28

Sociedade Brasileira de Matemtica

Se p 3(mod 4) , digamos p = 4k + 3, temos

p 1 = 2k + 1. Para 1 j k temos 2 p 1 p 1 e para k + 1 j 2k + 1 temos 1 2 j < 2 j p 1 , donde 2 2 a 1, se p 3 (mod 8), k +1 p = ( 1) = 1, se p 7 (mod 8).

Teorema: (Lei de reciprocidade quadrtica) Sejam p e q primos mpares. p ( p 1)( q 1) / 4 q Ento q p . = ( 1) Demonstrao: Na notao acima, com a = q, para cada j P, onde P = {1,2,..., ( p 1) / 2}, temos que j = 1 se e s se existe y tal que ( p 1) / 2 qj py < 0. Tal y deve pertencer a Q, onde Q = {1,2,..., (q 1) / 2}. q m Assim, temos que = ( 1) onde m = X e p

( x, y ) P Q ( p 1) / 2 qx py < 0}; X ={

p n note que qx py nunca assume o valor 0. Analogamente, = ( 1) , onde n = Y q e Y ={ ( x, y ) P Q 0 < qx py (q 1) / 2} . p q k Da segue que = (1) onde k = m + n = Z onde q p

( x, y ) P Q ( p 1) / 2 qx py ( q 1) / 2} Z ={ ( x, y ) C qx py < ( p 1) / 2} , A={ ( x, y ) C qx py > (q 1) / 2} . B ={

pois qx py nunca assume o valor 0. Temos k = C A B onde C = P Q,

Como C = ( p 1)(q 1) / 4 , basta mostrar que A = B . Mas f : C C definida por f ( x, y ) = ((( p + 1) / 2) x, (( q + 1) / 2) y ) define uma bijeo entre A e B [

EUREKA! N15, 2002

29

Sociedade Brasileira de Matemtica

Exemplo: Se n 1 e p = 2 2 + 1 primo, ento 3 no quadrado mdulo p (e logo 3 raiz primitiva mdulo p; ver [1]). 3 p 2n De fato, como p 1(mod 4), p = 3 , mas 2 1(mod 3) , como pode ser
n n p 2 facilmente mostrado por induo, donde p = 2 2 + 1 2(mod 3) , e = = 1. 3 3

REFERNCIAS:
[1] Carlos Gustavo T. de A. Moreira, Divisibilidade, Congruncias e Aritmtica mdulo n, Eureka! No. 2, pp. 41-52, 1998. [2] Guilherme Camarinha Fujiwara, Inteiros de Gauss e Inteiros de Eisenstein, Eureka! No. 14, pp. 2331, 2002. [3] Carlos Gustavo T. de A. Moreira e Nicolau C. Saldanha, Primos de Mersenne (e outros primos muito grandes), 22o. Colquio Brasileiro de Matemtica, IMPA, 1999.

EUREKA! N15, 2002

30

Sociedade Brasileira de Matemtica

APLICAES DE PLANOS PROJETIVOS EM TEORIA DOS NMEROS E COMBINATRIA


Carlos Yuzo Shine, So Paulo - SP Nvel Avanado.
1. Definio de plano projetivo

A definio dada aqui a mais geral. Dizemos que um conjunto S um plano projetivo se existem subconjuntos  1 ,  2 ,... de S que satisfazem as seguintes propriedades: (i) Se P e Q pertencem a S, um e somente um dos subconjuntos  i contm P e Q. (ii) A interseo de  i e  j consiste sempre de um nico elemento, para todo i j. Existem pelo menos quatro elementos de S tais que, entre eles no haja trs contidos em um dos conjuntos  i . Os elementos de S so normalmente chamados pontos e os subconjuntos  i , retas. Note que a propriedade (i) pode ser entendida como "por dois pontos passa uma nica reta" e a propriedade (iii) nos diz que "existem quatro pontos, trs a trs no colineares". (iii)
Exerccios

01. Um torneio de tnis disputado entre duas equipes. Cada membro de uma equipe joga com um ou mais membros da outra equipe, de modo que (i) Dois membros de uma mesma equipe tm exatamente um oponente em comum; (ii) No existem dois membros de uma equipe que enfrentam, juntos, todos os membros da outra equipe. Prove que cada jogador deve jogar um mesmo nmero de partidas. 02. (1) (2) (3) (4) (5) (6) Mostre que as seguintes proposies so equivalentes em planos projetivos: Existe uma reta que passa por exatamente n + 1 pontos; Existe um ponto que est contido em exatamente n + 1 retas; Todas as retas passam por exatamente n + 1 pontos; Todos os pontos esto contidos em exatamente n + 1 retas; H exatamente n2 + n + 1 retas; O plano projetivo tem exatamente n2 + n + 1 pontos (diz-se nesse caso que o plano projetivo tem ordem n).

EUREKA! N15, 2002

31

Sociedade Brasileira de Matemtica

03. (Cone Sul 1998) O prefeito de uma cidade deseja estabelecer um sistema de transportes com pelo menos uma linha de nibus, no qual: (i) cada linha passe exatamente por trs paradas; (ii) cada duas linhas distintas tenham exatamente uma parada em comum; (iii) para cada duas paradas de nibus distintas exista exatamente uma linha que passe por ambas. Determine o nmero de paradas de nibus da cidade. Dica: com o que parecem as condies (ii) e (iii)? Para encontrar os possveis valores, faa uma contagem dupla.
2. Construo de planos projetivos baseados em corpos

Seja K um corpo (ou seja, um conjunto onde esto definidas duas operaes, + e , tal que todo elemento admite oposto e todo elemento no nulo admite inverso). Ento o conjunto P2(K) de ternas (x, y, z) (0, 0, 0), x, y, z K, onde ternas da forma (x, y, z) e (kx, ky, kz) devem ser consideradas iguais, um plano projetivo na qual a reta correspondente ao ponto (a, b, c), reta dual de (abc), o conjunto de pontos (x, y, z) que satisfazem ax + by + cz = 0
Demonstrao

Basta mostrar que o conjunto dado tem as propriedades (i), (ii) e (iii). (i) Sejam (x1, y1, z1) e (x2, y2, z2) pontos distintos, ou seja, tais que no existe K tal que ( x1 , y1 , z1 ) = ( x2 , y2 , z2 ) . Temos que mostrar que existe somente uma reta que contm ambos os pontos, ou seja, que existe uma nica terna (a, b, c) (que so os coeficientes da reta) tal que ax1 + by1 + cz1 = 0 (*) ax 2 + by 2 + cz 2 = 0 Vejamos (*) como um sistema em a, b e c. Como ( x1 , y1 , z1 ) ( x 2 , y 2 , z 2 ) para todo K , a matriz completa x1 C = x 2 y1 y2 z1 do sistema (*) tem posto 2. Suponhamos, sem perda de z2

x1 y1 0. Logo, resolvendo (*) em a e b, obtemos (verifique!) x2 y2 a = mc e b = nc, onde m e n so constantes de K. Logo, as solues de (*) so da forma (mc, nc, c) = c(m, n, 1). Note que c 0 pois c = 0 implicaria a = b = c = 0. Assim, no plano projetivo P2(K), todas as solues so equivalentes, ou seja, nica. Logo existe somente uma reta que passa por esses dois pontos. generalidade, que
EUREKA! N15, 2002

32

Sociedade Brasileira de Matemtica

(ii) (iii)

Anlogo ao item (i). Tente voc fazer! Observe que (1, 1, 1), (1, 0, 0), (0, 1, 0) e (0, 0, 1) satisfazem essas condies. Aqui, 0 e 1 so os elementos neutros da soma e produto em K, respectivamente.

3. Dualidade

Observe que se r e s so retas duais dos pontos R = (a, b, c) e S = (d, e, f) de P2(K), respectivamente, ento R s da + eb + fc = 0 ad + be + cf = 0 S r A propriedade R s S r a chave do princpio da dualidade. Por isso, pontos e retas no plano projetivo se comportam de maneira semelhante quando se fala de incidncia. Quando K = #, temos o plano projetivo P2(#) visto na referncia [2]. Vamos entender a semelhana entre o plano definido aqui e o plano estudado em [2] (se voc no leu essa referncia, voc pode pular essa seo; mas leia o artigo, ele realmente vale muito a pena!!). Primeiro, vamos explicar a primeira "estranheza" do plano projetivo P2(#). Por que ele tem trs coordenadas, e no duas? A coordenada adicional que faz aparecerem pontos e retas no infinito. Podemos fazer a seguinte correspondncia: o ponto (x, y) do plano #2 corresponde ao ponto (x, y, 1) de P2(#). Sobraram os pontos do tipo (x, y, 0) de P2(#), que so os pontos do infinito. A reta ax + by + c = 0 de #2 pode ser transformada na reta ax + by + cz = 0 de P2(#) substituindo-se x e y na reta de #2 por x/z e y/z. Aqui, ns homogeneizamos a equao ax + by + c = 0. Voc pode visualizar a correspondncia que fizemos da seguinte forma: se imaginarmos as ternas (x, y, z) como pontos no espao, percebemos que os pontos de P2(#) correspondem a retas que passam pela origem. Se tomarmos o plano : z = 1 de #3, por exemplo, fazemos corresponder um ponto de P2(#) com o trao da reta correspondente nesse plano.
4. O caso K = #

EUREKA! N15, 2002

33

Sociedade Brasileira de Matemtica


z

A(a, b, 1)

x y

( a, b,1)

A reta dual de (a, b, c) corresponderia ao trao do plano ax + by + cz = 0 em . Com alguns clculos verifica-se que se R = (a, b, c) e r: ax + by + cz = 0, ento, sendo O = (0, 0, 1), R' e r'os respectivos correspondentes ao ponto R e reta r em , ento OR' r'.
4.1. O plano projetivo e curvas elpticas

Considere a curva cbica de R2 (**) ax 3 + bx 2 y + cxy 2 + dy 3 + ex 2 + fxy + gy 2 + hx + iy + j = 0, onde todas as letras de a a j so nmeros racionais. Gigantesco, no? Estamos interessados em saber sobre os seus pontos racionais (isto , pontos cujas coordenadas so ambas racionais). Para isso, feita a seguinte operao: tomamos um ponto racional da curva, denominado O. Dados dois pontos P e Q racionais da curva, encontre o terceiro ponto P * Q de interseo de PQ com a curva. Defina P + Q (isso mesmo, estamos somando pontos!) como o terceiro ponto de interseo da reta que passa por O e P * Q com a curva (isto , P + Q = O * (P * Q). Pode-se provar que P + Q tambm um ponto racional.

O Q P*Q

P+Q

S que a gigantesca equao (**) pode ser simplificada para a forma y2 = x3 + ax2 + bx + c, ()
EUREKA! N15, 2002

34

Sociedade Brasileira de Matemtica

que bem mais simples. Esta a forma de Weierstrass. Para isso, usamos a geometria projetiva. Vamos fazer um caso particular. Considere a curva u3 + 2v3 = 2uv + 1. Um de seus pontos racionais (1; 1). Primeiro devemos homogeneizar a curva. Fazendo u = U/W, obtemos U3 + 2V3 = 2UVW + W3. Note que agora vamos trabalhar no plano projetivo P2(#). Depois, escolhemos um ponto racional O na curva. Uma escolha O = (1; 1; 1). Agora, ns vamos mudar as coordenadas. A tangente curva pelo ponto O o nosso eixo de equao Z = 0. Se a tangente encontra a curva novamente em P (isso sempre ocorre se O no for ponto de inflexo), o eixo de equao X = 0 a tangente curva que passa por P. O eixo de equao Y = 0 pode ser qualquer reta que passe por O. Se O for ponto de inflexo, podemos escolher qualquer reta que no passa por O como o eixo X = 0. Observe que como P2(#) tem trs coordenadas, precisamos de trs eixos.

Y= 0

X=0 Z=0
Fazendo algumas contas (use um pouquinho de clculo; para trabalhar s com duas variveis, "desomogenize" - essa palavra existe? - a equao), obtemos que a tangente a O = (1; 1; 1) a reta U + 4V 5W = 0. Esta reta corta novamente a curva em (3; 2; 1). A tangente por esse ponto (use clculo novamente) 23U + 30V + 9W = 0. Por fim, tomamos a reta U V = 0 como o eixo Y = 0. Assim, queremos que
U + 4V 5W 62U 62V 23U + 30V + 9W 5 X + 3Y + 9Z 310 =Z 5 X 2Y + 9Z =Y V = 310 =X 5 X Y 53Z W = 310 U =

(usamos Y = 62U 62V para simplificar um pouco as contas). Substituindo U, V e W (veja que o 310 dos denominadores vai cancelar), temos

EUREKA! N15, 2002

35

Sociedade Brasileira de Matemtica

(5 X + 3Y + 9 Z ) 3 + 2(5 X 2Y + 9 Z ) 3 = 2(5 X + 3Y + 9Z ) (5 X 2Y + 9 Z ) (5 X Y 53Z ) + (5 X Y 53Z ) 3 A equao parece mais gigantesca que antes, mas aps "desomogeneizar" (basta substituir Z = 1) e abrir tudo (eu colocaria as contas, mas a margem neste artigo muito pequena para isso) obtemos xy 2 ( 4 x 30) y = 25x 2 + 96 x 515 Em geral, quando fazemos essa mudana de eixos (a transformao que fizemos ao resolver o sistema uma transformao projetiva) a equao se reduz forma xy 2 + ( ax + b) y = cx 2 + dx + e

Multiplicando por x nos dois membros, temos (xy )2 + ( ax + b) xy = cx 3 + dx 2 + ex Agora substitua z = xy: z 2 + ( ax + b) z = cx 3 + dx 2 + ex Completando quadrado no primeiro membro e substituindo z +

1 ( ax + b) por t 2

chegamos finalmente em t 2 = cbica em x Para a cbica ser mnica (ou seja, ter coeficiente dominante unitrio e no ser dentua), como o coeficiente dominante c, basta multiplicar ambos os membros por c2 e fazer as substituies (so as ltimas!) t = m/c e x = n/c. Vamos fazer isso no nosso exemplo. Multiplicando por x, fazendo z = xy e completando quadrado: z 2 ( 4 x 30) z + ( 2 x 15) 2 = 25 x 3 + 96 x 2 515x + (2 x 15) 2 ( z 2 x + 15) 2 = 25 x 3 + 100 x 2 575 x + 225 Agora, t = z 2x + 15: t 2 = 25 x 3 + 100 x 2 575 x + 225 m n e chegamos forma de Weierstrass: Por fim, t = e x= 25 25 m 2 = n 3 + 100n 2 + 14375n + 140625 Se voc escrever m e n em funo de u e v (e vice-versa), vai obter funes racionais (isto , o quociente de dois polinmios) com todos os coeficientes racionais. Logo pontos racionais so transformados em pontos racionais, de modo que achar os pontos racionais da curva original a mesma coisa que achar os pontos racionais da curva na forma de Weierstrass (fora, claro, os pontos que anulam os denominadores das funes racionais, que no so muitos e so fceis de achar). Qual a vantagem da forma de Weierstrass? A vantagem que podemos normalizar a curva para a forma (), alm da soma de pontos. O ponto O pode ser um ponto infinito, por exemplo. Homogeneizando (), obtemos Y 2 Z = X 3 + aX 2 Z + bXZ2 + cZ3 ,
EUREKA! N15, 2002

36

Sociedade Brasileira de Matemtica

e no difcil ver que o ponto do infinito (0; 1; 0) pertence curva (e alm disso, o nico!). Ento podemos fazer O = (0; 1; 0). Isso facilita um pouco as contas para adio de pontos. Se P * Q = (a; b)((a; b; 1) no plano projetivo), a reta que passa por O e P * Q Y = bZ, ou y = b. Conseqentemente, como a curva simtrica em relao ao eixo Ox, temos P + Q = (a; b).
y

P Q P*Q P+Q x

Exerccios

04. Prove que se P e Q so pontos racionais numa cbica ento P + Q tambm racional. 05. Transforme a cbica u3 + v3 = na forma de Weierstrass (um ponto racional da curva homogeneizada (1; 1; 0) e a conta no to terrvel assim). 06. (OBM 2002, nvel U) Considere a curva C = {( x; y ) R 2 | y 2 = x 3 43x + 166}. (a) Seja Q = (a; b) um ponto de C. Suponha que a reta tangente a C no ponto Q intersecte C num nico outro ponto, Q'. Determine as coordenadas de Q'. (b) Seja P0 = (3; 8). Para cada inteiro no negativo n, definimos Pn + 1 = P'n, o ponto de interseo de C com a reta tangente a C em Pn. Determine P2002. 07. A adio de pontos definida somente quando a cbica no segundo membro de () tem trs razes distintas (no necessariamente todas reais). Os outros casos so mais fceis! (a) Encontre todos os pontos racionais da curva y2 = x2 (x 1). (b) Encontre todos os pontos racionais da curva y2 = x3.

EUREKA! N15, 2002

37

Sociedade Brasileira de Matemtica

5. O caso K = /p

Sendo K qualquer corpo, podemos tomar K = /p (os inteiros vistos mod p), em que p um primo. Nesse caso, sendo /p um corpo finito (com p elementos) o plano projetivo P2( /p ) finito. Por exemplo, fazendo p = 2, obtemos o plano de Fano (rimou!), como pode ser visto na figura.
(0, 0, 1)

(0, 1, 1)

(1, 1, 1) (1, 0, 1)

(0, 1, 0)

(1, 0, 0)

(1, 1, 0)

Vamos fazer algumas continhas.


6. Algumas contagens 6.1. Nmero de elementos de P2( /p )

Temos p3 1 ternas (a, b, c) (0, 0, 0). Como (a, b, c) equivalente a (a, b, c) e pode assumir p 1 valores (1 a p 1), cada ponto est sendo contado p 1 vezes. p3 1 Logo P2( /p ) tem = p 2 + p + 1 pontos. p 1 Observe que, pelo princpio da dualidade, h tambm p2 + p + 1 retas.
6.2. Nmero de pontos em cada reta

Fixados a, b, e c, no todos nulos, queremos contar o nmero de solues no equivalentes (x, y, z) (0, 0, 0) da congruncia (***) ax + by + cz 0 (mod p) Suponhamos, sem perda de generalidade, que c 0 (mod p). Temos ento (***)
EUREKA! N15, 2002

z ac 1 x bc 1 y (mod p)

38

Sociedade Brasileira de Matemtica

Podemos escolher x e y de p2 maneiras. Porm, no podemos escolher x = y = 0, pois isso implicaria z = 0. Logo, considerando que cada elemento de P2( /p ) tem p 1 equivalentes, temos que cada reta tem p 2 1 = p + 1 pontos. p 1

6.3. Nmero de retas que passam por um ponto

Pelo princpio da dualidade, h p + 1 retas que passam por um ponto dado.


7. Um problema

A seguir, o problema 3 da Olimpada Iberoamericana de 1996, realizada na Costa Rica. Temos um tabuleiro quadriculado de k 2 k + 1 linhas e k 2 k + 1 colunas, onde k = p + 1 e p um nmero primo. Para cada primo p, d um mtodo para distribuir nmeros 0 e 1, um nmero em cada casa do tabuleiro, de modo que em cada linha haja exatamente k nmeros 0, em cada coluna haja exatamente k nmeros 0 e, alm disso, no haja nenhum retngulo, de lados paralelos aos lados do tabuleiro, com nmeros 0 em seus quatro vrtices.
Resoluo

Para k = p + 1, k 2 k + 1 = ( p + 1) 2 ( p + 1) + 1 = p 2 + p + 1 (coincidncia? destino? ou puramente sorte?). Assim, devemos preencher um tabuleiro (p2 + p + 1) (p2 + p + 1) com 0 e 1 de modo que haja p + 1 (que coisa) 0 em cada linha e coluna, e sem retngulos com 0 vrtices. Considere o plano projetivo P2( /p ) (por que ser?) e a cada linha associe um ponto e a cada coluna associe uma reta. Coloque 0 na casa (i, j) se, e somente se, o ponto i pertence reta j. Nas demais casas, coloque 1. H claramente p + 1 zeros em cada coluna. Pelo princpio da dualidade, tambm h p + 1 zeros em cada linha. Agora, suponha que exista um retngulo de vrtices (a, c), (a, d), (b, c) e (b, d), a b e c d, todos com 0. Logo, pela nossa construo, os pontos a e b pertencem a ambas as retas c e d. Absurdo, pois a interseo de duas retas exatamente um ponto. claro que na prova voc teria que demonstrar todas as propriedades que demonstramos antes.
8. Outros fatos sobre planos projetivos finitos

A construo baseada em corpos vale para qualquer corpo. Mas infelizmente, o nmero de elementos de um corpo finito deve ser potncia de primo (um corpo com
EUREKA! N15, 2002

39

Sociedade Brasileira de Matemtica

pn elementos, p primo, n inteiro positivo, o conjunto dos polinmios com coeficientes em /p , vistos mdulo um polinmio P(x) de grau n e irredutvel em /p [x]). Assim, tal construo s nos permite construir planos projetivos de ordem potncia de primo. Ser que existem planos projetivos com outra ordem? Conjectura-se que no, porm esse problema continua em aberto. O teorema de Bruck-Ryser-Chowla ajuda um pouquinho, dizendo que se n 1 (mod 4) ou n 2 (mod 4) e n ordem de um plano projetivo ento n deve ser soma de dois quadrados. Pensando nos casos pequenos, esse teorema elimina o caso n = 6. A demonstrao de que no existe plano projetivo de ordem n = 10 foi obtida em 1989 por C. W. H. Lam (com o auxlio de um computador!) e a histria da prova est disponvel em [7]. O caso n = 12 est em aberto. Planos projetivos tambm servem para construir alguns block designs. Um block design consiste num sistema de incidncia (v, k, , r, b) na qual um conjunto X de v pontos particionado numa famlia A de b subconjuntos (chamados blocks) de modo que dois pontos quaisquer determinam blocks com k pontos em cada block, e cada ponto est contido em r blocks. Na verdade, os cinco parmetros no so independentes. Fica como exerccio para voc mostrar que vr = bk e (v 1) = r (k 1) (so duas contagens duplas). Assim, pode-se representar o block design simplesmente como (v, k, ). Se b = v (e, conseqentemente, r = k) o block design dito simtrico. Note que um plano projetivo de ordem n block design (n2 + n + 1, n, 1). Outros exemplos e mais informaes podem ser encontrados no site: http://mathworld.wolfram.com/BlockDesign.html.
Exerccios

08. (OPM 2001) No condado Heptaprojetivo havia 7 castelos, batizados segundo grandes personagens: Arnold (A), Borcherds(B), Conway (C), Dilbert (D), Erds (E), Faltings (F), Gowers (G). Havia tambm 7 ruas, cada uma com 3 castelos, como mostra o mapa a seguir (uma rua circular):
A

B E

EUREKA! N15, 2002

40

Sociedade Brasileira de Matemtica

Um belo dia, o conde Steiner decidiu retirar as placas que identificavam os castelos para fazer uma limpeza. Na hora de recoloc-las, ningum se lembrava do lugar correto de cada uma, nem mesmo os moradores dos castelos! Os arquivos do condado s indicavam os castelos que ficavam numa mesma rua, mas no a ordem em que eles estavam. Assim, o conde sabia que havia uma rua com os castelos {A, B, D}, outra com {B, C, E}, outra com {B, F, G}, etc. Frente aos fatos, Steiner resolveu determinar todas as maneiras de recolocar as placas respeitando os arquivos do condado, isto , todas as maneiras nas quais placas que estavam juntas em uma mesma rua continuassem juntas em uma rua, possivelmente em outra ordem. Duas destas maneiras esto representadas a seguir:
A B

F B

G E

Chamaremos essas maneiras de vlidas. (a) Prove que o total de maneiras vlidas 7 vezes o nmero de maneiras vlidas nas quais a placa A colocada no castelo A. (b) Prove que o total de maneiras vlidas nas quais a placa A colocada no castelo A 6 vezes o nmero de maneiras vlidas nas quais a placa A colocada no castelo A e a placa B colocada no castelo B. (c) Determine o nmero de maneiras vlidas. 09. No reino da Alndia h n cidades, assim como no reino da Belndia. Foram construdas m estradas, sendo que cada estrada passa por exatamente duas cidades, uma de cada reino. Mostre que se no existem quatro cidades ligadas n por um ciclo de estradas, ento m (1 + 4n 3 ) 2 Mostre que a igualdade pode ocorrer para infinitos valores de n. Dicas: a igualdade pode ser demonstrada com uma injeo e contagem dupla - veja o artigo Grafos e Contagem Dupla, na Eureka! 12; para mostrar que a igualdade pode ocorrer para infinitos valores de n, encontre primeiro os valores de n para os quais 4n 3 um quadrado perfeito - voc vai se surpreender!
EUREKA! N15, 2002

41

Sociedade Brasileira de Matemtica

10. (Extenso do exemplo do artigo Grafos e Contagem Dupla) Na Terra de Oz h n castelos e vrias estradas, sendo que cada uma liga dois castelos. Diz a lenda que se houver quatro castelos ligados em ciclo (ou seja, se existirem quatro castelos A, B, C, e D tais que A e B, B e C, C e D e D e A esto ligados), um drago aparecer do centro dos castelos e destruir a Terra de Oz. Mostre que para esta desgraa no acontecer o nmero de estradas deve ser menor ou igual a n mostre que para infinitos n possvel construir 1 + 4n 3 4 n 1 1 + 4n 3 estradas. (para mostrar isto voc vai precisar saber um pouco 4 de lgebra Linear).

( (

) )

REFERNCIAS:
[1] D. Pedoe, Geometry: A Comprehensive Course. Este livro tem um bom texto no somente introdutrio sobre geometria projetiva e contm mais aplicaes geomtricas. O problema 1 e os axiomas de planos projetivos foram retirados deste livro, assim como a construo de planos projetivos com coordenadas. [2] L. Castro, Introduo Geometria Projetiva, Revista Eureka! 8. A melhor referncia que conheo para comear a estudar geometria projetiva aplicada a problemas de geometria. [3] O problema 9 foi adaptado de um exemplo retirado do livro Graph Theory: An Introductory Course, de B. Bollobs. Ele um caso particular do problema de Zaranckiewsky: qual o nmero mximo de arestas de um grafo bipartido com m vrtices em uma classe V1 e n vrtices na outra classe V2, de modo que no haja nenhum subgrafo bipartido completo com r vrtices de V1 e s vrtices de V2? Tal nmero normalmente representado por z(m; n; r; s). [4] O problema 10 uma extenso de um exemplo de meu artigo Grafos e Contagem Dupla, que est na Revista Eureka! 12. [5] A construo do exemplo do problema 10 pode ser encontrada no livro Proofs From The Book, de Martin Aigner e Gnter M. Ziegler. Mas tente voc mesmo faz-la antes! [6] Curvas elpticas e a reduo para a forma de Weierstrass podem ser encontradas em Rational Points on Elliptic Curves, de Joseph H. Silverman e Jonh Tate. [7] A histria da demonstrao de que no existe plano projetivo de ordem 10 est em http://www.cecm.sfu.ca/organics/papers/lam/paper/html/paper.html [8] Um glosrio de termos da Matemtica e principais fatos relacionados est em http://mathworld.wolfram.com/ [9] Uma discusso muito interessante sobre o assunto est em
http://www.mathematik.uni-bielefeld.de/~sillke/PUZZLES/projective_plane

(o grande matemtico John Conway inclusive participa desta discusso).

EUREKA! N15, 2002

42

Sociedade Brasileira de Matemtica

OLIMPADAS AO REDOR DO MUNDO


Chegamos ao ltimo nmero de 2002. Apresentamos, como sempre, questes que no so encontradas facilmente na Internet. Feliz 2003, divirtam-se e enviem as suas solues. Continuamos disposio na OBM para aqueles que estiverem interessados na soluo de algum problema particular. Para tanto, basta contactar a OBM, atravs de carta ou e-mail.
Antonio Luiz Santos

Primeiramente vamos aos problemas propostos deste nmero


181.(Rssia-2002) Um nmero de quatro algarismos escrito em um quadro negro.

permitido adicionar 1 a quaisquer dois algarismos adjacentes se nenhum deles for igual a 9 ou subtrair 1 de quaisquer dois algarismos adjacentes se nenhum deles for igual a 0. Partindo de 1234 possvel obtermos 2002 aps efetuarmos estas operaes diversas vezes ?
182.(Rssia-2002) Seja ABC um tringulo cujas medidas dos lados so distintas duas a

duas. Exteriormente ao tringulo, so construdos sobre os seus lados os tringulos eqilteros ABC1, BCA1 e CAB1. Mostre que o tringulo A1B1C1 no equiltero.
183.(Rssia-2002) Um polinmio quadrtico de coeficientes inteiros e coeficiente do

segundo grau igual a 1 assume valores primos em trs valores inteiros e consecutivos. Mostre que ele assume um valor primo em pelo menos mais um valor inteiro.
184.(Rssia-2002) Seja O o circuncentro de um tringulo issceles ABC(AB = BC). Um ponto M pertence ao segmento BO e o ponto M' o simtrico de M em relao ao ponto mdio de AB. Sejam ainda K o ponto de interseo de M'O com AB e L um ponto sobre o lado BC tal que CLO = BLM . Mostre que os pontos O, K, B, L pertencem a um mesmo crculo. 185.(Rssia-2002) Qual o maior nmero de termos possvel de uma progresso

aritmtica de nmeros inteiros positivos (a1, a2, a3,,an) de razo igual a 2 tal que o 2 nmero a k + 1 seja primo para todo k = 1, 2, 3,,n?
186.(Rssia-2002) Sobre o eixo x so escolhidos pontos x1, x2,.., xn com n 3 distintos

dois a dois. Traam-se ento todas as parbolas de coeficientes do segundo grau igual a 1 e que cortam o eixo dos x somente nos pontos escolhidos. Sejam
EUREKA! N15, 2002

43

Sociedade Brasileira de Matemtica

y = f1 , ... , y = f m as equaes destas parbolas. Mostre que a parbola y = f1 + + f m corta o eixo x em exatamente dois pontos.
187.(Rssia -2002) Uma seqncia de nmeros (an) tal que a0 = 0 e a n +1 a n + 1 .

Mostre a desigualdade :

k =1

3 ak

n ak = k 1

n n n 188.(Rssia-2002) No intervalo (22 , 32 ) so escolhidos 2 2 1 + 1 nmeros mpares.

Mostre que podemos encontrar entre estes nmeros dois nmeros tais que o quadrado de cada um deles no divisvel pelo outro.
189.(Albnia-2002) Sendo f : # # uma funo definida por

1 f (x ) = 2 3 Determine o valor da soma :

1+ x x

2x

1 2 2002 2002 2002 2002 f + f + + f +2f +2f + + 2 f 2002 2002 2002 2001 2000 1

190.(Albnia-2002) Mostre como dividir um tringulo qualquer em :

(i) 2002 tringulos semelhantes a ele. (ii) 2003 tringulos semelhantes a ele.
191.(Irlanda-2002) Em um tringulo ABC tem-se que AB = 20, AC = 21 e BC = 29. Se D e E so pontos do segmento BC tais que BD = 8 e EC = 9, determine a medida do ngulo DAE . 192.(Irlanda-2002) Suponha que n seja o produto de quatro nmeros primos distintos a,

b, c, d tais que : (i) a + c = d (ii) a(a + b + c + d) = c (d b) (iii) 1 + bc + d = bd Determine n.

EUREKA! N15, 2002

44

Sociedade Brasileira de Matemtica

193.(Irlanda-2002) Seja (an) uma seqncia definida por a1 = 1, a2 = 1, a3 = 1

a n +1 a n 2 a n an 1 n 1.

e = 2 para todo n 3 . Prove que an um inteiro positivo para todo

194.(Eslovnia-2002) 38 o menor inteiro positivo tal que o seu quadrado termina com trs quatros (382 = 1444). Qual o prximo inteiro positivo com esta propriedade ? 195.(Eslovnia-2002) Sejam M o ponto mdio da base AB do trapzio ABCD; E um

ponto interior ao segmento AC tal que BC e ME intersectam-se em F; G o ponto de interseo de FD e AB; H o ponto de interseo de DE e AB. Mostre que M o ponto mdio do segmento GH.
196.(Eslovnia-2002) Sejam k um crculo do plano euclidiano, k1 e k2 dois crculos

disjuntos interiores a k e que o tangenciam nos pontos A e B respectivamente. Seja ainda (t) a tangente comum aos crculos k1 e k2 nos pontos C e D respectivamente tal que k1 e k2 estejam situados num mesmo semiplano determinado pela reta (t) enquanto que o centro do crculo k pertence ao outro semiplano. Se E o ponto de interseo de AC e BD, mostre que o ponto E pertence ao crculo k.
197.(Eslovnia-2002) Determine o menor inteiro positivo que pode ser escrito como uma soma de 9, 10 e 11 inteiros positivos consecutivos. 198.(Estnia-2002) Em um tringulo ABC tem-se que B = 2 C e a bissetriz do ngulo A intersecta o lado BC no ponto D tal que AB = CD. Determine a medida do ngulo A . 199.(Estnia-2002) Um nmero natural de 10 algarismos distintos dito mgico se ele for mltiplo de 99999. Quantos nmeros mgicos no comeados por zero existem ? 200. (Grcia-2002) Determine todos os inteiros positivos N tais que :

(ii) d 5 = (d 2 + d 4 )d 6 .

(i) N possui exatamente16 divisores 1 = d1 < d 2 < ... < d16 = N

201. (Inglaterra-2002) Um quadriltero ABCD est inscrito em um crculo. As diagonais

AC e BD intersectam-se no ponto Q. Os prolongamentos dos lados DA e CB, a partir de A e B respectivamente, intersectam-se em P. Sabendo que CD = CP = DQ, mostre que CAD = 60 .

EUREKA! N15, 2002

45

Sociedade Brasileira de Matemtica

202.(Inglaterra-2002) Doze pessoas esto sentadas em torno de uma mesa circular. De

quantos modos seis pares de pessoas podem trocar apertos de mo de modo que no haja cruzamentos de braos?
(No permitido que uma pessoa troque apertos de mo com mais de uma pessoa de cada vez)

203.(Inglaterra-2002) Seja f uma funo de  em , onde  o conjunto dos nmeros inteiros no negativos, que possui as seguintes propriedades : (i) f (n + 1) > f (n ), para cada n  . (ii) f (n + f (m )) = f (n ) + m + 1 , para todos m, n  . Determine todos os valores possveis de f(2001). 204.(Inglaterra-2002) A altura traada de um dos vrtices de um tringulo ABC

intersecta o lado oposto no ponto D. A partir do ponto D so traadas perpendiculares aos outros dois lados intersectando-os nos pontos E e F. Mostre que a medida do segmento EF independe do vrtice do qual a altura traada.
205.(ustria-2002) Determine o menor inteiro positivo x para o qual todas as fraes :

3 x + 9 3x + 10 3x + 11 3 x + 49 , , , ... , 8 9 10 48 so irredutveis.
206.(ustria-2002) Determine o maior nmero real C tal que

((x + y )

verdadeira para todos os nmeros reais x e y (x y ) com xy = 2. Para quais pares ordenados (x, y) a igualdade verdadeira ?
207.(ustria-2002) Seja f (x ) =

(x y )

6 (x y ) + 8
2 2

)(

) C

9x 9x + 3

k Determine a soma de todas as expresses da forma f onde k um nmero 2002 k seja irredutvel. inteiro entre 0 e 2002 tal que a frao 2002 208. (Bielorssia-2002) Determine o maior nmero possvel de grupos que podem ser extrados do conjunto {1, 2, 3,,19, 20}, de modo que o produto dos nmeros de cada um dos grupos seja um quadrado perfeito .
(O grupo pode conter apenas um nmero e, neste caso, o produto igual a este nmero; alm disso, cada nmero deve fazer parte de apenas um grupo)
EUREKA! N15, 2002

46

Sociedade Brasileira de Matemtica

209. (Bielorssia-2002) A altura CH de um tringulo retngulo ABC, com C = 90

intersecta as bissetrizes AM e BN nos pontos P e Q respectivamente. Prove que a reta que passa pelos pontos mdios de QN e PM paralela hipotenusa de ABC.
210. (Bielorssia-2002) Um conjunto de nmeros naturais de trs algarismos formados

com 1, 2, 3, 4, 5, 6 chamado agradvel se ele satisfaz seguinte condio: para quaisquer dois algarismos distintos de 1, 2, 3, 4, 5, 6, existe um nmero deste conjunto que contm ambos. Se, para qualquer conjunto agradvel calcularmos a soma de seus elementos, determine o menor valor possvel destas somas.

Agora vamos aos comentrios e solues dos leitores para alguns dos problemas apresentados em nmeros anteriores de EUREKA!. Como sempre, o critrio por ns adotado para este nmero foi apresentar as solues dos problemas que foram, at o presente momento, resolvidos pelo maior nmero de leitores.
32. (Moldvia-1998) A seqncia

(a n ) ,

n * verifica as relaes a1 =

1 e 2

an =

a n 1 para todo nmero natural n > 1 . Calcule a1 + a 2 + + a1998 . 2na n 1 + 1

Resumo das solues de diversos leitores: Escrevamos a lei de recorrncia de uma forma ligeiramente diferente: 1 1 1 1 = 2n + = 2n an a n 1 a n a n 1 Logo, n 1 1 n = (2k ) a k 1 k =2 a k k =2

O primeiro membro desta ltima igualdade uma soma telescpica e igual a 1 1 enquanto que o segundo membro igual a a n a1 (4 + 2n )(n 1) . Da, 4 + 6 + 8 + + 2n = 2

EUREKA! N15, 2002

47

Sociedade Brasileira de Matemtica

1 1 1 1 1 2 = (n + 2)(n 1) = n(n + 1) an = = an an n(n + 1) n n + 1 Finalmente, 1998 1998 1 1 1998 1 = ai = =1 1999 1999 n + 1 i =1 n =1 n

36.(China-1999) Seja PQRS um quadriltero inscrito num crculo e cuja medida do

ngulo PSR seja igual a 90 o . Se H e K so os ps das perpendiculares baixadas de Q sobre PR e PS respectivamente (convenientemente prolongados se necessrio). Mostre que HK divide QS ao meio. Resumo das solues de diversos leitores: Seja G o ponto de interseo de KH e SR. Como P, Q, R e S so cclicos (pertencem a um mesmo crculo), assim como K, Q, H e P tem-se que QKG = QKH = QPR = QSG . Da K, Q, G e S so cclicos com KSG = meio.
39. (Irlanda-1999) Determine todos os inteiros positivos m tais que a quarta potncia do

= SKQ . Desta forma, KQGS um retngulo e KH divide QS ao

nmero de seus divisores positivos igual a m. Resumo das solues de diversos leitores: Observemos em primeiro lugar que m = 1 possui a referida propriedade. Suponhamos agora que m 2. Como m uma quarta potncia ele pode ser escrito univocamente como 4 2 4 n m = p14 1 p 2 pn onde p1 < p 2 < < p n so nmeros primos e i > 0, i = 1, 2, ... , n . O nmero de divisores de m dado por (4  1 + 1)(4  2 + 1) (4  n + 1) A requerida propriedade 2 n pn (4 1 + 1)(4  2 + 1) (4  n + 1) = p11 p2 Como o lado esquerdo mpar ento p1 3 . Observe que 3 > 4  + 1 se 3 ; tambm, 5 > 4  + 1 se 2 ; finalmente p > 4  + 1 se 1 e p 7 . Alm disso, 5 4  + 1 se 1 , assim, se p < 4 + 1 para algum primo mpar p e algum 1 ento p = 3 . Deste modo, devemos ter p1 = 3 ou p1 = 5 .
EUREKA! N15, 2002

48

Sociedade Brasileira de Matemtica

1 Caso : p1 = 5 Neste caso, 1 = 1 e assim


2 n 5(4  2 + 1) (4  n + 1) = 5 p 2 pn

Pela terceira desigualdade acima no podem existir outros fatores primos em m distintos de 5 e portanto a nica soluo m = 5 4 . 2 Caso : p1 = 3 . Claramente 1 = 1 ou 1 = 2 uma vez que 33 > 4(3) + 1 .
2 n . Assim p 2 = 5 . Se Se 1 = 1 , ento 5(4  2 + 1) (4  n + 1) = 3 p 2 pn 5 2 > 2 ento 52 > (4  2 + 1) o que nos leva a uma contradio. Deste modo, 3 2 = 1 ou 2 = 2 . 3 n o que uma contradio Se 2 = 1 ento 25(4  3 + 1) (4  n + 1) = 15 p3 pn j que 25 no divide o lado direito. 3 n Se 2 = 2 , ento 45(4  3 + 1) (4  n + 1) = 75 p3 pn contradio j que 9 no divide o lado direito. 2 n Se 1 = 2 ento 9(4  2 + 1) (4  n + 1) = 9 p 2 . pn

o que uma

No podemos ter p 2 7 logo, n = 1 ou ( n = 2 e p 2 = 5 e 2 = 1 ). O primeiro caso nos d m = 38 ; o ltimo caso nos d m = 38 5 4 . Concluindo existem quatro solues a saber, 1, 5 4 , 38 e 38 5 4
40. (Irlanda-1999) Mostre que existe um nmero inteiro positivo na seqncia de

Fibonacci que divisvel por 1000 . Resumo das solues de diversos leitores: Considere o conjunto de 106 + 1 pares tomados mod 10 3 . Como o conjunto

( )

{(F , F
i

i +1

)|

i = 1,2, ... ,10 6

{(a, b ) |

0 a, b 999} com a e b inteiros

possui apenas 10 6 elementos, o Princpio da Casa dos Pombos nos diz que existem inteiros i > j tais que Fi + 1 F j + 1 e Fi F j so ambos divisveis por 1000. Mas Fi 1 F j 1 = Fi +1 F j +1 + F j Fi
EUREKA! N15, 2002

) (

49

Sociedade Brasileira de Matemtica

Logo, Fi 1 F j 1 tambm divisvel por 1000. Argumentando desta maneira de frente para trs vemos que Fi j + 1 F1 e Fi j F0 so divisveis por 1000. Mas, F0 = 0 da, Fi j divisvel por 1000.
60. (St.Petersburg-1999) Trs mgicos apresentam um truque entregando a uma pessoa da platia um mao de cartas numeradas com 1, 2,...,2 n + 1 (n > 6). O espectador fica com uma das cartas e aleatoriamente distribui as restantes entre o primeiro e o segundo mgicos (cada um deles fica com n cartas) . Estes olham suas cartas (sem se comunicar um ao outro) e cada um escolhe duas cartas formando um mao (ordenado) com estas cartas e as entrega ao terceiro mgico. O terceiro mgico olha estas quatro cartas e anuncia a carta que ficou com o espectador. Explique como este truque pode funcionar. SOLUO DE JEAN-CHRISTOPHE YOCCOZ (PARIS - FRANA):

A ideia que cada mgico transmita de algum jeito a soma de suas n cartas mdulo 2n + 1 (e ento o outro mgico descobre a carta que foi retirada, que o simtrico da soma dos dois resultados mdulo 2n + 1). Assim, basta definir uma funo f : que, dado c em ( 2n + 1) (2n + 1) (2n + 1) tal

e um subconjunto arbitrrio A de com n (2n + 1) (2n + 1) elementos eles satisfazem a propriedade P(A, c): existem elementos distintos x e y de A com f(x, y) = c. A idia mostrar a existncia de uma funo f(x, y) como acima que seja equivariante, isto , que satisfaa f(x + c, y + c) = f(x, y) + c, para quaisquer x, y e c. fcil ver que para uma tal funo f vale P(A, c) se e somente se vale P(A c, 0), onde A c = {x c | x A}. Assim, basta ver que todo A com n elementos satisfaz P(A, 0). Agora, em vez de construir f, vamos construir a pr-imagem de 0 pela f (ou um subconjunto conveniente dela). Pela equivarincia, se (x, y) e (u, v) so pares distintos com f(x, y) = f(u, v) = 0 ento y x tem que ser diferente de v u, seno, para c = u x, temos (u, v) = (x+ c, y + c), donde f(u, v) = c. Por outro lado, dado um conjunto de pares{(x1, y1),...,(xk, yk)} com yj xj distinto de yi xi para todo i distinto de j, existe uma funo equivariante f com f(xi, yi) = 0 para todo i. Nosso problema agora se reduz a encontrar um conjunto de pares {(x1, y1),...,(xk, yk)} com os yi xi todos distintos (e distintos de 0) tal que para todo A com n elementos exista i tal que xi e yi estejam em A. Fazemos com que esse conjunto de pares contenha os pares (0, 1), (2, 1), (0, 2), (3, 6), (9, 6), (3, 9), (4, 8), (12, 8) e (4, 12). Note que as diferenas yi xi so 1, 1, 2, 3, 3, 6, 4, 4 e 8, e que um A ruim s pode conter um elemento de cada uma das triplas {0, 1, 2}, {3, 6, 9} e
EUREKA! N15, 2002

50

Sociedade Brasileira de Matemtica

{4, 8, 12}. Vamos completar nosso conjunto de pares com n 4 pares que formaro \{0, 1, 2, 3, 6, 9, 4, 8, 12}. Um conjunto ruim s pode (2n + 1) ter um elemento de cada par, tendo pois no mximo n 4 + 3 = n 1 elementos, como queramos. Para isso considere alguns casos: I) n par, n 12: Escrevemos n = 2k. Os pares sero ( k + 6, k + 6), ( k + 7, k + 5),...,(1, 13) (diferenas n, n 2 ,...,14); (k + 7, 3k + 6), (k + 8, 3k + 5),...,(2k + 4, 2k + 9) (diferenas n 1, n 3,...,5); e (2k + 5, 5), (2k + 6, 7), (2k + 7, 11) e (2k + 8, 10) (diferenas n, 1 n, 4 n e 2 n). n mpar, m 13: Escrevemos n = 2k + 1. Os pares sero (1, 13),..., ( k + 6, k + 6) (diferenas 14 ,..., n 1); (k + 7, 3k + 8),...,(2k + 5, 2k + 10) (diferenas n ,..., 5); e (2k + 6, 5), (2k + 7, 7), (2k + 8, 11) e (2k + 9, 10) (diferenas n, 1 n, 4 n e 2 n). uma partio de

II)

Casos particulares anteriores: n = 7: Pares (5, 10), (13, 11) e (7, 14) (diferenas 5, 2 e 7). n = 8: Pares (5, 1), (7, 14), (15, 10) e (13, 11) (diferenas 6, 7, 5 e 2). n = 9: Pares (5, 14), (16, 7), (10, 17), (18, 11) e (15, 13) (diferenas 9, 9, 7, 7 e 2). n = 10: Pares (20, 10),(19, 11),(18, 13),(5, 14),(16, 7) e (17, 15) (diferenas 10, 8, 5, 9, 9 e 2). n = 11: Pares (16, 5), (17, 7), (20, 11), (18, 10), (19, 13), (21, 14) e (15, 22) (diferenas 11, 10 , 9, 8, 6, 7 e 7).

Publicamos a seguir, a pedidos, uma soluo do problema 12, proposto na Eureka! No. 8.
12. (Irlanda-1999) Trs nmeros a < b < c esto em progresso aritmtica se c b = b

a. Definamos a seqncia (un), n = 0, 1, 2, 3, da seguinte maneira: u0 = 0, u1 = 1 e para cada n 1, un + 1 o menor inteiro positivo tal que u n +1 > u n e {u 0 , u1 ,..., u n , u n +1 } no possui trs elementos em progresso aritmtica. Determine u100.
EUREKA! N15, 2002

51

Sociedade Brasileira de Matemtica

Vamos mostrar por induo que se

n = ( ak ak 1 ...a0 ) 2 , i.e, n =

a
j =0

2 j , com

a j {0,1}, j , ento u n = (a k a k 1 ...a0 ) 3 =


k

a
j =0

3 j . Suponha que isso seja vlido

para 0 n 2 1 (isso verdade para k = 1). Como {u 0 , u1 ,..., u 2 k 1 } no possui trs elementos em progresso aritmtica e u 2 k 1 =

j =0

k 1

1 3 j =

3k 1 , ento u 0 , u1 ,..., u 2 k 1 ,3k + u 0 ,3 k + u1 ,...,3 k + u 2 k 1 tambm 2

no possui trs elementos em progresso aritmtica, pois se dois termos de uma 3k 1 progresso aritmtica esto entre 0 e , ento o terceiro termo menor ou igual 2 a 3 k 1 < 3k , e se dois termos de uma progresso aritmtica esto entre 3k e 3k + 3k 1 3k 1 3 k + 1 3 k 1 ento o termo anterior maior ou igual a 3 k . = > 2 2 2 2 Vamos ver que, de fato, u 2 k = 3 k , donde u 2 k + j 3 k + u j para 0 j 2 k 1 , e logo, pelo fato acima, u 2 k + j = 3k + u j , para 0 j 2 k 1. Para isso, basta ver que se 3k 1 < u < 3 k ento {u 0 , u1 ,..., u 2 k 1 , u} contm uma progresso aritmtica de trs 2 termos. Seja u = v2 =

b
j =0 k 1 j =0

k 1

3 j , com b j {0,1,2}, j. Ento u = 2v 2 v1 , onde 3 j , e, se b j = 0 , ento (c j , d j ) = (0,0); se b j = 1 ento

d
j =0

k 1

3 j , v1 =

(c j , d j ) = (1,1) e, se bj = 2 ento (c j , d j ) = (0,1). Em particular, {c j , d j } {0,1}, j , donde v1 e v2 pertencem a {u 0 , u1 ,..., u 2 k 1 } , e u v 2 = v 2 v1 , logo v1 < v 2 < u esto em progresso aritmtica. Finalmente, 100 = 26 + 25 + 22, donde u100 = 36 + 35 + 32 = 981.

EUREKA! N15, 2002

52

Sociedade Brasileira de Matemtica

Acusamos o recebimento de solues de problemas anteriores dos seguintes leitores de EUREKA!:


Anderson Torres Carlos A. Gomes Erasmo de Souza Dias Fernando Carvalho Ramos Francisco B. de Lima Holanda Geraldo Perlino Geraldo Perlino Jnior Gibran Medeiros de Souza Helder Oliveira de Castro Luiz Srgio Carvalho de Mello Marcelo Ribeiro de Souza Marcelo Rufino de Oliveira Okakama Matsubaxi Renato F. L. Mello Tertuliano C. de Souza Neto So Paulo - SP Natal - RN Goinia - GO Santa Maria - RS Fortaleza - CE Itapecerica da Serra - SP Cotia - SP Natal - RN Mogi das Cruzes - SP Rio de Janeiro - RJ Rio de Janeiro - RJ Belm - PA So Paulo - Sp J. dos Guararapes - PE Salvador - BA Prob. 24, 32, 35, 37, 40, 43, 45, 51, 54, 56, 63, 88, 103, 143, 151, 154, 158, 160, 164, 168, 169, 170, 171, 172. Prob. 151, 162, 174. Prob. 151. Prob. 151, 160, 163, 174. Prob. 36, 45, 47, 69, 87, 88, 102, 104, 109, 169, 170. Prob. 151, 160, 161, 164, 167. Prob. 151, 152, 153, 154, 158, 159, 160, 161, 162, 163, 167, 168, 170, 171, 172, 173, 174, 178, 179, Prob. 163, 166. Prob. 43, 45, 46, 54, 66, 63, 67, 70, 74, 86, 90, 99, 101, 115, 128, 133, 143. Prob. 138, 151, 166. Prob. 151, 153, 154, 160, 162. Prob. 151, 152, 153, 154, 158, 160, 162, 163, 164, 165, 171, 172, 173, 174, 179, 180. Prob. 130. Prob. 154, 160, 162, 166, 173, 179. Prob. 61, 62, 68, 69, 70, 72, 74, 76, 82, 83, 87, 88, 90. 114, 124,

165, 166,

102, 105,

166, 170,

EUREKA! N15, 2002

53

Sociedade Brasileira de Matemtica

SOLUES DE PROBLEMAS PROPOSTOS


Publicamos aqui algumas das respostas enviadas por nossos leitores.

66) Prove que, dados um inteiro n 1 e um conjunto A

elementos existe B / n2 A + B = {x + y | x A, y B} / n 2

/ n 2 com n com elementos tal que n 2 tem mais de n /2 elementos.

SOLUO DE ZOROASTRO AZAMBUJA NETO (RIO DE JANEIRO - RJ):

Vamos mostrar que, dado um conjunto X / n 2 , existe t / n 2 tal que ( A + {t}) X tem pelo menos X / n elementos, onde X o nmero de elementos de X. De fato,
t / n 2

(A + {t}) X = {t
aA x X

/ n 2 a + t = x} = A X = n X ,

pois, para cada a A e x X , existe um nico t / n 2 tal que a + t = x. Assim, 1 ( A + {t}) X = X / n , donde o nmero mdio de elementos de n 2 t / n 2

(A + {t}) X

X / n , o que claramente implica a nossa afirmao. k 0 existem t1 , t 2 ,..., t k / n 2 tais que

Agora,

dado

1 n 2 A + {t1 , t 2 ,..., t k } n 2 (1 ) k . De fato, por induo, dados tais t1 , t 2 ,..., t k , n pela afirmao acima existe t k +1 / n 2 com (A + {t k +1}) ( / n 2 ) \ ( A + {t1 , t 2 ,..., t k }) 1 / n 2 \ ( A + {t1 , t 2 ,...,t k }) , n C donde ( A + {t1 , t 2 ,..., t k +1 }) = (( A + {t1 , t 2 ,..., t k }) (A + {t k +1 }) ) C

1 1 1 2 1 C 2 1 . ( A + {t1 , t 2 ,..., t k }) 1 n 1 = n 1 n n n n denota

k +1

(aqui

XC

/ n2

)\ X ; temos

X C = n2 X ).
n

1 Assim, existe B = {t1 , t 2 ,..., t n } tal que n 2 A + B n 2 1 < n 2 / 2 , donde n A + B > n 2 / 2.

EUREKA! N15, 2002

54

Sociedade Brasileira de Matemtica

Obs.

Para
n

1 n n 1, 1 n

n 1 n 1 = > 1 + 1 + 1 = 2, n 1 n n
n n

donde

1 n < . 2 n 1

70)

P1

X5 P5 Q5 Q4 X4 Q3 Q1 Q2

X1

P2

X2

X3 P4 P3

Na figura acima, para 1 j 5, X j o ponto de interseo dos crculos circunscritos aos tringulos Qj 1 PjQj e Qj Pj + 1 Qj + 1 distintos de Qj (os ndices so tomados mdulo 5). Prove que o pentgono X1X2X3 X4X5 inscritvel. Obs: O pentgono P1P2P3P4P5 no necessariamente regular.

EUREKA! N15, 2002

55

Sociedade Brasileira de Matemtica

SOLUO DE ALEX CORRA ABREU (NITERI RJ):

Y1

Y2 . F Y3 Y

. .
C

Y4

Lema: As circunferencias circunscritas a BDF, CEF, CDA, BEA, tem um ponto em comum. Prova: Seja X a interseco das circunferencias circunscritas a BDF, CEF. Y1, Y2, Y3, Y4, o p das perpendiculares por Y a BD, BE , CD, CE respectivamente Y1 , Y2 , Y3 so colineares pela reta de Simson j que Y pertence a circunferncia circunscrita a BDF e analogamente Y2 , Y3 , Y4 tambm so logo Y1 , Y2 , Y3 , Y4 so colineares. Portanto pela recproca da reta de Simson, Y pertence circunferncia circunscrita a ABE, ( Y1 , Y2 , Y4 colineares) e circunferncia circunscrita a ADC (Y1 , Y3 , Y4 colineares).
P1

X5 P5 Q5 Q4 X4 Q3 Q1 Q2

X1

P2

X2

X3 P4 P3

EUREKA! N15, 2002

56

Sociedade Brasileira de Matemtica

Q =Q P X pois Agora X 2 X 1 2 2 2 X 2 pois X 1 X 2 P 2Q2 inscritvel e Q2 X 1 X 3 = Q2 P 4 3 X 1 circunferncia circunscrita a P 1Q2 P 4 pelo lema e X 3 tambm. X =X P Q X X Q =Q X X e Analogamente Q X X = Q P X e Q X
3 4 3 3 4 3 3 4 2 2 2 2 2 1 2 3 4 2

X =Q X Q2 X 1 3 3 4 X 3 X 2 X1 X 3 = X 2 X 4 X 3 X1 X 2 X 3 X 4 inscritvel; analogamente X 2 X 3 X 4 X 5 tambm , logo X 1 X 2 X 3 X 4 X 5 inscritvel.


72) Ache todas as funes f :

x, y #.

tais que f ( x 4 + y ) = x 3 f ( x ) + f ( f ( y )),

SOLUO DE RODRIGO VILLARD MILET (RIO DE JANEIRO RJ):

f ( x 4 + y ) = x 3 f ( x) + f ( f ( y )) (I) Em (I) , x = 0 f ( y ) = f ( f ( y )), logo f ( x 4 + y ) = x 3 f ( x) + f ( y ) (I'). Agora fazendo x = 1 e y = 0, segue que f(0) = 0. Portanto, fazendo y = 0, temos f ( x 4 ) = x 3 f ( x ) (II) (Em particular, f mpar). Em (I'), f ( x 4 + y ) = f ( x 4 ) + f ( y ) , donde f ( x + y ) = f ( x ) + f ( y ) , para todo x > 0. Mas como f mpar, fcil ver que vale f ( x + y ) = f ( x ) + f ( y ) , para todos x, y reais. Lema: Todo x real pode ser escrito (de forma nica) como x = u + v, onde f(u) = u e f(v) = 0. Prova: x = f ( x) + ( x f ( x)) . Tome u = f ( x ) e v = x f ( x) . Para a unicidade, suponha que x = u + v = u '+v ' . Da u u ' = v 'v e aplicando f, segue que u = u' e assim, v = v'. Agora, em (II), escreva x = u + v, como no lema. Ento: f ((u + v) 4 ) = (u + v) 3 f (u + v) 4 f (u 3 v) + 6 f (u 2 v 2 ) + 4 f (uv3 ) = 3u 2 v(u + v) (III) Podemos trocar v por v, pois f(v) = 0, logo: 4 f (u 3 v) + 6 f (u 2 v 2 ) 4 f (uv 3 ) = 3u 2 v(u v) (IV) u 2v2 . Pelo lema, u 2 v 2 = u '+v ' , com f(u') = u' e f(v') 2 u '+ v' = 0. Da segue que u ' = f (u '+v ' ) = u ' = v ' . Aplicando f, segue que u' = v' = 0, 2 ou seja u 0 ou v 0 , o que nos d respectivamente as solues f ( x) 0 e f ( x ) x , que claramente satisfazem a equao (I). Fazendo (III) + (IV): f (u 2 v 2 ) =

EUREKA! N15, 2002

57

Sociedade Brasileira de Matemtica

Agradecemos tambm o envio das solues e a colaborao de:


Rildo Alves do Nascimento Leno Silva Rocha Minh Perez Carlos Alberto da Silva Victor Santa Maria da Boa Vista PE Goinia GO Rehoboth, EUA Nilpolis RJ

Seguimos aguardando o envio de solues dos problemas propostos N. 68, 69, 71 e 73 publicado na revista Eureka! N. 14

'4.H 8,- ,

6:0 5,7, .425 09,7 :2 E -:2 .4254894 547 n 1 :7 3 ,8 8:543/4 6:0 ;4.H .42570  1 :7 3 , 547 / , 4 90254 08507,/4 2F/ 4 F n ln n / ,8 438 /07,3/4 :2, 8 9:, 4 3, 6:, ;4.H 902 :2 E -:2 .42  1 :7 3 ,8 0 .42570 :2 5,.490 /0  1 :7 3 ,8 547 / , ;4.H /0;0 /0247,7 :2 ,34 5,7, .425 09,7 4 E -:2

'4.H 8,- ,

6:0 14 /08.4-0794 547 ,3 3/7,  7, ,  9 3 $, 03, 0 007, , , :2 , 47 924 /09072 3J89 .4 6:0 . 0., , 57 2, /,/0 /0 3 20748 3,9:7, 8 n 02 90254 54 342 , 02 4 n !,7, 2, 8 31472, 08 ;0 , http://www.cse.iitk.ac.in/news/primality.html 4:
http://www.utm.edu/research/primes/prove/prove4_3.html

EUREKA! N15, 2002

58

Sociedade Brasileira de Matemtica

PROBLEMAS PROPOSTOS
Convidamos o leitor a enviar solues dos problemas propostos e sugestes de novos problemas para os prximos nmeros. 74) Ache todas as funes f : # # tais que: f ( x + y ) + f ( x y ) = 2 f ( x) cos y,

x, y #.

75) Seja Tn um tringulo retngulo cujos lados medem (4 n 2 ,4 n 4 1,4 n 4 + 1) ,

onde n um nmero inteiro positivo. Seja n a medida do ngulo oposto ao lado de medida 4 n 2 . Mostre que, se n varia dentro dos inteiros positivos, 1 + 2 + 3 + ... = 90 0 .
76) Mostre que um polgono qualquer pode ser recortado e os recortes

reorganizados, sem superposio, de tal jeito que formem um quadrado.


77) Prove que as distncias entre um ponto sobre uma circunferncia e os quatro

vrtices de um quadrado nesta inscrita no podem ser todos nmeros racionais.


78) Seja ABCD um quadriltero convexo no trapzio, de diagonais AC e BD iguais.

Tomamos sobre os lados AB e CD, respectivamente, pontos P e Q tais que: AP DQ AD = = PB QC BC Mostre que os pontos P e Q so colinares com o ponto de interseo das mediatrizes dos lados AD e BC.
79) Temos uma fileira infinita de copos, cada um deles associado a um inteiro k, e

um nmero finito de pedras distribudas de alguma maneira por esses copos. Se h pelo menos duas pedras no copo k podemos pular uma pedra para o copo k 1 e outra para o copo k + 1. Prove que fazendo movimentos desse tipo um nmero suficientemente grande de vezes, chega-se necessariamente a uma situao onde no possvel fazer nenhum movimento desse tipo (i.e., onde h no mximo uma pedra em cada copo), e que a configurao final no depende da escolha dos movimentos durante o processo.

EUREKA! N15, 2002

59

Sociedade Brasileira de Matemtica

80) Sejam =

1+ 5 , A = {n , n  *} e B = n 2 , n  * . Prove que 2 A B = e A B = *.

Obs. x o inteiro tal que x x < x + 1.

Problema 74 proposto por Gibran M. de Souza (Natal - RN); Problema 75 proposto por Carlos A. Gomes (Natal - RN); Problema 76 proposto por Eduardo Casagrande Stabel (Porto Alegre - RS); Problemas 77 e 78 propostos por Evandro Makiyama de Melo (So Paulo - SP).

EUREKA! N15, 2002

60

Sociedade Brasileira de Matemtica

COORDENADORES REGIONAIS
Alberto Hassen Raad Amarsio da Silva Arajo Benedito Tadeu Vasconcelos Freire Carlos Frederico Borges Palmeira Claudio Arconcher Claus Haetinger Cleonor Crescncio das Neves lio Mega Florncio Ferreira Guimares Filho Ronaldo Alves Garcia Alexandra de Oliveira Abdala Cousin Ivanilde Fernandes Saad Jacqueline Fabiola Rojas Arancibia Joo Bencio de Melo Neto Joo Francisco Melo Libonati Jos Carlos Pinto Leivas Jos Cloves Saraiva Jos Gaspar Ruas Filho Jos Luiz Rosas Pinho Jos Vieira Alves Licio Hernandes Bezerra Luzinalva Miranda de Amorim Marcelo Rufino de Oliveira Marcondes Cavalcante Frana Pablo Rodrigo Ganassim Ramn Mendoza Reinaldo Gen Ichiro Arakaki Ricardo Amorim Roberto Vizeu Barros Rosngela Souza Srgio Cludio Ramos Tadeu Ferreira Gomes Toms Menndez Rodrigues Valdenberg Arajo da Silva Wagner Pereira Lopes (UFJF) (UFV) (UFRN) (PUC-Rio) (Colgio Leonardo da Vinci) (UNIVATES) (UTAM) (Colgio Etapa) (UFES) (UFGO) (UEM) (UC. Dom Bosco) (UFPB) (UFPI) (Grupo Educacional Ideal) (UFRG) (UFMA) (ICMC-USP) (UFSC) (UFPB) (UFSC) (UFBA) (Grupo Educacional Ideal) (UFC) (Liceu Terras do Engenho) (UFPE) (INPE) (Centro Educacional Logos) (Colgio Acae) (Colgio Singular) (IM-UFRGS) (UEBA) (U. Federal de Rondnia) (U. Federal de Sergipe) (CEFET GO) Juiz de Fora MG Viosa MG Natal RN Rio de Janeiro RJ Jundia SP Lajeado RS Manaus AM So Paulo SP Vitria ES Goinia GO Maring PR Campo Grande MS Joo Pessoa PB Teresina PI Belm PA Rio Grande RS So Luis MA So Carlos SP Florianpolis SC Campina Grande PB Florianpolis SC Salvador BA Belm PA Fortaleza CE Piracicaba SP Recife PE SJ dos Campos SP Nova Iguau RJ Volta Redonda RJ Santo Andr SP Porto Alegre RS Juazeiro BA Porto Velho RO So Cristovo SE Jata GO

EUREKA! N15, 2002

61

CONTEDO

XIV OLIMPADA BRASILEIRA DE MATEMTICA Problemas e Solues da Primeira Fase XIV OLIMPADA BRASILEIRA DE MATEMTICA Problemas e Solues da Segunda Fase XIV OLIMPADA BRASILEIRA DE MATEMTICA Problemas e Solues da Terceira Fase XIV OLIMPADA BRASILEIRA DE MATEMTICA Problemas e Solues da Primeira Fase - Nvel Universitrio XIV OLIMPADA BRASILEIRA DE MATEMTICA Problemas e Solues da Segunda Fase - Nvel Universitrio XIV OLIMPADA BRASILEIRA DE MATEMTICA Premiados AGENDA OLMPICA

2 14

25

44

49

58

62

Sociedade Brasileira de Matemtica

XXIV OLIMPADA BRASILEIRA DE MATEMTICA


Problemas e Solues da Primeira Fase
PROBLEMAS NVEL 1
4 8 1. A razo (2 ) igual a:

(48 ) 2

A)

1 4

B)

1 2

C) 1

D) 2

E) 8

2. Num armazm foram empilhadas embalagens cbicas conforme mostra a figura a seguir. Se cada caixa pesa 25 kg, quanto pesa toda a pilha?

A) 300 kg

B) 325 kg

C) 350 kg

D) 375 kg

E) 400 kg

3. Na balana a seguir temos pesadas bolas de chumbo, todas iguais, e leves saquinhos de plstico, todos com a mesma quantidade de bolinhas, iguais s que esto fora dos mesmos. Quantas bolinhas h em cada saquinho?

a a a

a a

a a

A) 1

B) 2

C) 3

D) 5

E) 6

4. Escreva os nmeros inteiros de 1 a 9 nos nove quadradinhos, de forma que as somas dos quatro nmeros em cada uma das ps da hlice sejam iguais e de maior valor possvel. Esse valor :

A) 23
EUREKA! N16, 2003

B) 22

C) 21

D) 20

E) 19

Sociedade Brasileira de Matemtica

5. Qual a quantidade total de letras de todas as respostas incorretas desta questo? A) Quarenta e oito. B) Quarenta e nove. C) Cinqenta. D) Cinqenta e um. E) Cinqenta e quatro. 6. Toda a produo mensal de latas de refrigerante de uma certa fbrica foi vendida a trs lojas. Para a loja A, foi vendida metade da produo; para a loja B, foram vendidos 2 da produo e para a loja C, foram vendidas 2500 unidades. Qual
5

foi a produo mensal dessa fbrica? A) 4166 latas B) 10000 latas C) 20000 latas E) 30000 latas

D) 25000 latas

7. Um quadrado de rea 1 foi dividido em 4 retngulos congruentes, conforme indicado no desenho esquerda. Em seguida, os quatro retngulos foram reagrupados de maneira a formar um quadrado, com um buraco quadrado no centro, conforme indica o desenho direita.

A rea do buraco igual a: A) 1 B) 9


2 16

C) 16
25

D) 3
4

E) 1

8. A linha poligonal AB desenhada mantendo-se sempre o mesmo padro mostrado na figura. Seu comprimento total igual a:
2 1

A
1 2 3

B
4 5 6 7 8 9 30 31

A) 31

B) 88

C) 90

D) 97

E) 105

9. A diferena entre os quadrados de dois nmeros inteiros positivos consecutivos sempre: A) um nmero primo. B) um mltiplo de 3. C) igual soma desses nmeros. D) um nmero par. E) um quadrado perfeito.
EUREKA! N16, 2003

Sociedade Brasileira de Matemtica

10. Marcelo leva exatamente 20 minutos para ir de sua casa at a escola. Uma certa vez, durante o caminho, percebeu que esquecera em casa a revista Eureka! que ia mostrar para a classe; ele sabia que se continuasse a andar, chegaria escola 8 minutos antes do sinal, mas se voltasse para pegar a revista, no mesmo passo, chegaria atrasado 10 minutos. Que frao do caminho j tinha percorrido neste ponto? A) 2 B) 9 C) 1 D) 2 E) 9
5

20

10

11. O grfico abaixo mostra o faturamento mensal das empresas A e B no segundo semestre de 2001.
m ilhes de rea is
2 00 1 80 1 60 1 40 1 20 1 00

 
ago nov jul set out dez

Com base nesse grfico, podemos afirmar que: A) houve um ms em que o faturamento da empresa A foi o dobro do faturamento da empresa B. B) no ms de julho, a diferena de faturamentos foi maior que nos demais meses. C) a empresa B foi a que sofreu a maior queda de faturamento entre dois meses consecutivos. D) no semestre, o faturamento total de A foi maior que o de B. E) a diferena entre os faturamentos totais do semestre excedeu os 20 milhes de reais. 12. Patrcia mora em So Paulo e quer visitar o Rio de Janeiro num feriado prolongado. A viagem de ida e volta, de nibus, custa 80 reais, mas Patrcia est querendo ir com seu carro, que faz, em mdia, 12 quilmetros com um litro de gasolina. O litro da gasolina custa, em mdia, R$1,60 e Patrcia calcula que ter de rodar cerca de 900 quilmetros com seu carro e pagar 48 reais de pedgio. Ela ir de carro e para reduzir suas despesas, chama duas amigas, que iro repartir com ela todos os gastos. Dessa forma, no levando em conta o desgaste do carro e outras despesas inesperadas, Patrcia ir: A) economizar R$20,00. B) gastar apenas R$2,00 a mais.
EUREKA! N16, 2003

Sociedade Brasileira de Matemtica

C) economizar R$24,00. D) gastar o mesmo que se fosse de nibus. E) gastar R$14,00 a mais. 13. Uma escola vai organizar um passeio ao zoolgico. H duas opes de transporte. A primeira opo alugar "vans": cada van pode levar at 6 crianas e seu aluguel custa R$60,00. A segunda opo contratar uma empresa para fazer o servio: a empresa usa nibus com capacidade para 48 crianas e cobra R$237,00, mais R$120,00 por nibus utilizado. A escola deve preferir a empresa de nibus se forem ao passeio pelo menos N crianas. O valor de N : A) 28 B) 31 C) 32 D) 33 E) 36 14. O produto de um milho de nmeros naturais, no necessariamente distintos, igual a um milho. Qual o maior valor possvel para a soma desses nmeros? A) 1 000 000 E) 13 999 432 B) 1 250 002 C) 1 501 999 D) 1 999 999

15. Se voc tiver uma mesa de bilhar retangular cuja razo entre a largura e o comprimento seja 5/7 e bater em uma bola que est em um canto, de modo que ela saia na direo da bissetriz do ngulo desse canto, quantas vezes ela bater nos lados antes de bater em um dos cantos? A) 10 vezes B) 12 vezes C) 13 vezes D) 14 vezes E) 15 vezes 16. Na malha quadriculada a seguir, todas as circunferncias tm centro em M. Ento pode-se concluir que a rea preta :

A) B) C) D) E)

dois quintos da rea do crculo maior. trs stimos da rea do crculo maior. metade da rea do crculo maior. quatro stimos da rea do crculo maior. trs quintos da rea do crculo maior.

EUREKA! N16, 2003

Sociedade Brasileira de Matemtica

17. As figuras a seguir so construdas com palitos pretos e brancos. Para construir as figuras, os palitos pretos foram colocados apenas nas bordas e os brancos apenas no interior. A figura de nmero n corresponde a um retngulo 3 por n. Continuando esse procedimento, quantos palitos brancos teremos na figura 2002?
1 2 3

A) 2001

B) 4004

C) 12006

D) 10007

E) 10010

18. Um produtor de leite engarrafa diariamente toda a produo de leite de sua fazenda. Depois de tirado, o leite segue para um tanque de forma cilndrica e ento engarrafado, conforme vemos na figura a seguir. Na tabela vemos a quantidade de garrafas que foram enchidas e o nvel do leite dentro do tanque. Depois de quantas garrafas serem enchidas o tanque ficar vazio? Quantidade de garrafas enchidas Nvel do tanque (cm)

0 210

200 170

400 130

600 90

A) 1000

B) 1050

C) 1100

D) 1150

E) 1200

19. Escrevendo todos os nmeros inteiros de 100 a 999, quantas vezes escrevemos o algarismo 5? A) 250 B) 270 C) 271 D) 280 E) 292 20. Uma usina comprou 2000 litros de leite puro e ento retirou certo volume V desse leite para produo de iogurte e substituiu esse volume por gua. Em seguida, retirou novamente o mesmo volume V da mistura e novamente substituiu por gua. Na mistura final existem 1125 litros de leite. O volume V : A) 500 litros B) 600 litros C) 700 litros D) 800 litros E) 900 litros

EUREKA! N16, 2003

Sociedade Brasileira de Matemtica

PROBLEMAS NVEL 2

1. Um comerciante comprou dois carros por um total de R$ 27.000,00. Vendeu o primeiro com lucro de 10% e o segundo com prejuzo de 5%. No total ganhou R$ 750,00. Os preos de compra foram, respectivamente, A) R$ 10.000,00 e R$ 17.000,00 B) R$ 13.000,00 e R$ 14.000,00 C) R$ 14.000,00 e R$ 13.000,00 D) R$ 15.000,00 e R$ 12.000,00 E) R$ 18.000,00 e R$ 9.000,00 2. Veja o problema No. 15 do Nvel 1. 3. Dizer que uma tela de televiso tem 20 polegadas significa que a diagonal da tela mede 20 polegadas. Quantas telas de televiso de 20 polegadas cabem numa de 60 polegadas? A) 9 B) 10 C) 18 D) 20 E) 30 4. Veja o problema No. 20 do Nvel 1. 5. Dois irmos, Pedro e Joo, decidiram brincar de pega-pega. Como Pedro mais velho, enquanto Joo d 6 passos, Pedro d apenas 5. No entanto, 2 passos de Pedro equivalem distncia que Joo percorre com 3 passos. Para comear a brincadeira, Joo d 60 passos antes de Pedro comear a persegui-lo. Depois de quantos passos Pedro alcana Joo? A) 90 passos B) 120 passos C) 150 passos D) 180 passos E) 200 passos 6. 7. 8. 9. Veja o problema No. 9 do Nvel 1. Veja o problema No. 10 do Nvel 1. Veja o problema No. 4 do Nvel 1. Veja o problema No. 12 do Nvel 1.

10. Traando segmentos, podemos dividir um quadrado em dois quadradinhos congruentes, quatro trapzios congruentes e dois tringulos congruentes, conforme indica o desenho abaixo, esquerda. Eliminando algumas dessas partes, podemos montar o octgono representado direita. Que frao da rea do quadrado foi eliminada?

EUREKA! N16, 2003

Sociedade Brasileira de Matemtica

A) 1
9

B) 2
9

C) 1

D) 1

E) 3
8

11. Veja o problema No. 11 do Nvel 1. 12. Veja o problema No. 14 do Nvel 1. 13. O lava-rpido "Lave Bem" faz uma promoo: Lavagem simples R$5,00 Lavagem completa R$7,00 No dia da promoo, o faturamento do lava-rpido foi de R$176,00. Nesse dia, qual o menor nmero possvel de clientes que foram atendidos? A) 23 B) 24 C) 26 D) 28 E) 30 14. Veja o problema No. 7 do Nvel 1. 15. Quantos nmeros inteiros positivos menores que 900 so mltiplos de 7 e terminam em 7? A) 10 B) 11 C) 12 D) 13 E) 14 = 80 e C = 40 , a medida do ngulo agudo 16. Dado um tringulo ABC onde A e B : formado pelas bissetrizes dos ngulos A o A) 40 B) 60 C) 70 D) 80 E) 110o 17. Na malha quadrada abaixo, h 6 quadrados de lado 30 cm. A rea do tringulo ABC :

C A B

B) 100 cm2 C) 75 cm2 A) 150 cm2 o 18. Veja o problema N . 8 do Nvel 1. 19. Veja o problema No. 19 do Nvel 1.
EUREKA! N16, 2003

D) 50 cm2

E) 25 cm2

Sociedade Brasileira de Matemtica


2 2 20. Se xy = 2 e x2 + y2 = 5, ento x 2 + y 2 + 2 vale:

A) 5
2

B) 25
4

C) 5
4

D) 1

E) 1

21. Veja o problema No. 13 do Nvel 1. 22. Durante sua viagem ao pas das Maravilhas a altura de Alice sofreu quatro mudanas sucessivas da seguinte forma: primeiro ela tomou um gole de um lquido que estava numa garrafa em cujo rtulo se lia: "beba-me e fique 25% mais alta". A seguir, comeu um pedao de uma torta onde estava escrito: "proveme e fique 10% mais baixa"; logo aps tomou um gole do lquido de outra garrafa cujo rtulo estampava a mensagem: "beba-me e fique 10% mais alta". Finalmente, comeu um pedao de outra torta na qual estava escrito:"prove-me e fique 20% mais baixa". Aps a viagem de Alice, podemos afirmar que ela: A) ficou 1% mais baixa B) ficou 1% mais alta C) ficou 5% mais baixa D) ficou 5% mais alta E) ficou 10% mais alta 23. Vamos provar que 4 maior que 4. Sejam a e b dois nmeros tais que a > 4 e a = b. 1) Vamos subtrair 4 dos dois termos desta equao: a=b a4=b4 2) Colocamos 1 em evidncia no segundo membro da equao: a 4 = 1 ( b + 4) a 4 = 1 (4 b) 3) Elevamos ambos os termos da equao ao quadrado: (a 4) 2 = [ 1 ( 4 b)] 2 (a 4) 2 = (1) 2 (4 b) 2 (a 4) 2 = 1 (4 b) 2 (a 4) 2 = ( 4 b) 2 4) Extramos a raiz quadrada dos dois membros da equao: ( a 4) 2 = ( 4 b) 2 a4=4b 5) Como a = b, substitumos b por a a4=4a
EUREKA! N16, 2003

Sociedade Brasileira de Matemtica

6) Resolvemos a equao: a4=4a 2a = 8 a=4 Como escolhemos a tal que a > 4, chegamos inacreditvel concluso de que 4 > 4. Onde est o erro no argumento acima? A) Na passagem 2. B) Na passagem 3. C) Na passagem 4. D) Na passagem 5. E) Na passagem 6. 24. Veja o problema No. 5 do Nvel 1. 25. O resto da diviso por 9 de 1111111111 22222 : A) 0 B) 1 C) 3 D) 6
PROBLEMAS NVEL 3

E) 8

1. Veja o problema No. 11 do Nvel 1. 6,888... p o valor de p + q igual a: a frao irredutvel equivalente a 2. Se 2,444... q A) 38 B) 39 C) 40 D) 41 E) 42 3. Veja o problema No. 1 do Nvel 2. 4. A seguir vemos quatro vasos, os quais Angela vai encher com gua, numa torneira cuja vazo constante.

Os grficos A e B a seguir representam o nvel da gua (eixo vertical), em dois dos vasos, de acordo com o tempo (eixo horizontal).

Qual dos vasos corresponde ao grfico A e qual ao grfico B, respectivamente? A) 3 e 4 B) 2 e 4 C) 1 e 3 D) 2 e 3 E) 1 e 4


EUREKA! N16, 2003

10

Sociedade Brasileira de Matemtica

5. 6. 7. 8. 9. 10.

Veja o problema No. 13 do Nvel 1. Veja o problema No. 22 do Nvel 2. Veja o problema No. 10 do Nvel 1. Veja o problema No. 8 do Nvel 1. Veja o problema No. 10 do Nvel 2. Veja o problema No. 20 do Nvel 2.

11. A mdia aritmtica das idades de um grupo de mdicos e advogados 40 anos. A mdia aritmtica das idades dos mdicos 35 anos e a dos advogados 50 anos. Pode-se, ento, afirmar que: A) O nmero de advogados o dobro do nmero de mdicos no grupo. B) O nmero de mdicos o dobro do nmero de advogados no grupo. C) H um mdico a mais no grupo. D) H um advogado a mais no grupo. E) Existem as mesmas quantidades de mdicos e advogados no grupo. 12. Os valores de x, y e z que satisfazem s equaes x + 1 = 2 so tais que x + 3 y + 2 z igual a: x A) 5 B) 6 C) 7 z+ 13. Veja o problema No. 23 do Nvel 2. 14. Veja o problema No. 5 do Nvel 1. 15. Sejam x, y, z nmeros inteiros tais que x + y + z = 0. Sobre x 3 + y 3 + z 3 so feitas as seguintes afirmativas: i) necessariamente mltiplo de 2. ii) necessariamente mltiplo de 3. iii) necessariamente mltiplo de 5. Podemos afirmar que: A) B) C) D) E) somente i) correta. somente ii) correta. somente i) e ii) so corretas. somente i) e iii) so corretas. i), ii) e iii) so corretas. 1 1 = 5 , y + =1 e y z

D) 8

E) 9

EUREKA! N16, 2003

11

Sociedade Brasileira de Matemtica

16. Seja f uma funo real de varivel real que satisfaz a condio: 2002 f ( x) + 2 f = 3x x para x > 0. O valor de f(2) igual a: A) 1000 B) 2000 C) 3000 D) 4000 E) 6000 17. Veja o problema No. 25 do Nvel 2. 18. Na circunferncia abaixo, temos que: AB = 4, BC = 2, AC dimetro e os D e CB D so iguais. Qual o valor de BD? ngulos AB
A

D B C

A) 2 3 + 1

B)

9 5

C) 3 2

D) 2 + 5

E) 4

19. Seja a maior raiz de x2 + x 1 = 0. O valor de 5 5 : A) 1 B) 2 C) 3 D) 1


77 ... 7

E) 2

, onde aparecem 2002 setes? 20. Qual o dgito das unidades de 7 7 A) 7 B) 9 C) 3 D) 1 E) 5. 21. Em um trapzio ABCD de rea 1, a base BC mede a metade da base AD. Seja K o ponto mdio da diagonal AC. A reta DK corta o lado AB no ponto L. A rea do quadriltero BCKL igual a: 3 2 1 2 1 A) C) D) E) B) 4 3 3 9 9 22. N = 539984 um nmero inteiro positivo com oito algarismos, sendo o primeiro e o ltimo desconhecidos. Sabendo que N um mltiplo de 198, encontre o algarismo das unidades de N / 198. E) 9 A) 5 B) 6 C) 7 D) 8
EUREKA! N16, 2003

12

Sociedade Brasileira de Matemtica

23. No trimin marciano, as peas tm 3 nmeros cada (diferente do domin da terra, onde cada pea tem apenas 2 nmeros). Os nmeros no trimin marciano tambm variam de 0 a 6, e para cada escolha de 3 nmeros (no necessariamente distintos) existe uma e somente uma pea que contm esses 3 nmeros. Qual a soma dos nmeros de todas as peas do trimin marciano? A) 756 B) 1512 C) 84 D) 315 E) 900 mede 60 e o ngulo B mede 50. Sejam M o 24. No tringulo ABC, o ngulo A ponto mdio do lado AB e P o ponto sobre o lado BC tal que AC + CP = BP. Qual a medida do ngulo MPC? A) 120 B) 125 C) 130 D) 135 E) 145 25. Duas pessoas vo disputar uma partida de par ou mpar. Elas no gostam do zero e, assim, cada uma coloca 1, 2, 3, 4 ou 5 dedos com igual probabilidade. A probabilidade de que a pessoa que escolheu par ganhe : A) 1/2 B) 2/5 C) 3/5 D) 12/25 E) 13/25

GABARITO
NVEL 1 (5a. e 6a. Sries) 1) C 6) D 2) C 7) B 3) B 8) D 4) B 9) C 5) D 10) B NVEL 2 (7a. e 8a. Sries) 1) C 6) C 2) A 7) B 3) A 8) B 4) A 9) C 5) E 10) B NVEL 3 (Ensino Mdio) 1) D 6) A 2) E 7) B 3) C 8) D 4) C 9) B 5) B 10) B
EUREKA! N16, 2003

11) D 12) C 13) B 14) D 15) A 11) D 12) D 13) C 14) B 15) D

16) C 17) D 18) B 19) D 20) A 16) C 17) C 18) D 19) D 20) B 21) B 22) A 23) C 24) D 25) D

11) B 12) B 13) C 14) D 15) C

16) B 17) D 18) C 19) C 20) C

21) D 22) C 23) A 24) E 25) E

13

Sociedade Brasileira de Matemtica

XXIV OLIMPADA BRASILEIRA DE MATEMTICA


Problemas e Solues da Segunda Fase
PROBLEMAS NVEL 1 PROBLEMA 1

O ano 2002 palndromo, ou seja, continua o mesmo se lido da direita para a esquerda. a) b) Depois de 2002, quais sero os prximos quatro anos palndromos? O ltimo ano palndromo, 1991, era mpar. Quando ser o prximo ano palndromo mpar?
A E G

PROBLEMA 2

Um fazendeiro resolveu repartir sua fazenda para seus cinco filhos. O desenho ao lado (fora de escala) representa a fazenda e as partes dos herdeiros, que so da forma triangular, de BC AC DC , AE = modo que BD = , DF = e 4 3 2 EG = GC. O filho mais novo recebeu o terreno representado pelo tringulo escuro, de 40 alqueires. Quantos alqueires tinha a propriedade original?
PROBLEMA 3

C D F

Dado um nmero, pode-se escrever o seu dobro ou suprimir o seu algarismo das unidades. Apresente uma seqncia que comea com 2002 e termina com 13, usando somente essas duas operaes.
PROBLEMA 4

Trs amigas foram para uma festa com vestidos azul, preto e branco, respectivamente. Seus pares de sapato apresentavam essas mesmas trs cores, mas somente Ana usava vestido e sapatos de mesma cor. Nem o vestido nem os sapatos de Jlia eram brancos. Marisa usava sapatos azuis. Descreva a cor do vestido de cada uma das moas.

EUREKA! N16, 2003

14

Sociedade Brasileira de Matemtica

PROBLEMA 5

No jogo pega-varetas, as varetas verdes valem 5 pontos cada uma, as azuis valem 10 pontos, as amarelas valem 15, as vermelhas, 20 e a preta, 50. Existem 5 varetas verdes, 5 azuis, 10 amarelas, 10 vermelhas e 1 preta. Carlinhos conseguiu fazer 40 pontos numa jogada. Levando em conta apenas a quantidade de varetas e suas cores, de quantas maneiras diferentes ele poderia ter conseguido essa pontuao, supondo que em cada caso fosse possvel pegar as varetas necessrias?
PROBLEMA 6

Nas casas de um tabuleiro 8 8 foram escritos nmeros inteiros positivos de forma que a diferena entre nmeros escritos em casas vizinhas (quadrados com um lado comum) 1. Sabe-se que numa das casas est escrito 17 e, em outra, est escrito 3. Desenhe um tabuleiro 8 8, preencha-o segundo essas regras e calcule a soma dos nmeros escritos nas duas diagonais do tabuleiro.

PROBLEMAS NVEL 2 PROBLEMA 1

Geraldinho e Magro saram de suas casas no mesmo instante com a inteno de um visitar o outro, caminhando pelo mesmo percurso. Geraldinho ia pensando num problema de olimpada e Magro ia refletindo sobre questes filosficas e nem perceberam quando se cruzaram. Dez minutos depois, Geraldinho chegava casa de Magro e meia hora mais tarde, Magro chegava casa de Geraldinho. Quanto tempo cada um deles andou? Cada um velocidade constante.
Observao: PROBLEMA 2
;07/0

deles

anda

com

, - 7 ,2,70 4
EUREKA! N16, 2003

4 3.

, :

Um grande painel na forma de um quarto de crculo foi composto com 4 cores, conforme indicado na figura ao lado, onde o segmento divide o setor em duas partes iguais e o arco interno uma semicircunferncia. Qual a cor que cobre a maior rea?

15

Sociedade Brasileira de Matemtica

PROBLEMA 3

Nas casas de um tabuleiro 8 8 foram escritos nmeros inteiros positivos de forma que a diferena entre nmeros escritos em casas vizinhas (quadrados com um lado comum) 1. Sabe-se que numa das casas est escrito 17 e, em outra, est escrito 3. Calcule a soma dos nmeros escritos nas duas diagonais do tabuleiro.
PROBLEMA 4
C B

O professor Pardal est estudando o comportamento familiar de uma espcie de pssaro. Os pontos A, B, C e D da figura ao lado, representam a disposio de quatro ninhos desses pssaros. O professor construiu um posto de observao equidistante dos quatro ninhos. Todos os ninhos e o posto de observao esto em um mesmo nvel de altura a partir do solo, a distncia de B D = 45 . Determine a distncia a D de 16 metros e BA que o posto guarda de cada ninho.

PROBLEMA 5

O primeiro nmero de uma seqncia 7. O prximo obtido da seguinte maneira: Calculamos o quadrado do nmero anterior 72 = 49 e a seguir efetuamos a soma de seus algarismos e adicionamos 1, isto , o segundo nmero 4 + 9 + 1 = 14. Repetimos este processo, obtendo 142 = 196 e o terceiro nmero da seqncia 1 + 9 + 6 + 1 = 17 e assim sucessivamente. Qual o 2002o elemento desta seqncia?
PROBLEMA 6

O ano 2002 palndromo, ou seja, continua o mesmo se lido da direita para a esquerda. a) Depois de 2002, quais sero os prximos quatro anos palndromos? b) O ltimo ano palndromo, 1991, era mpar. Quando ser o prximo ano palndromo mpar? c) O ltimo ano palndromo primo aconteceu h mais de 1000 anos, em 929. Determine qual ser o prximo ano palndromo primo.
PROBLEMAS NVEL 3 PROBLEMA 1

Veja o problema No. 5 do Nvel 2.


PROBLEMA 2

Para quais inteiros positivos n existe um polgono no regular de n lados, inscrito em uma circunferncia, e com todos os ngulos internos de mesma medida?
EUREKA! N16, 2003

16

Sociedade Brasileira de Matemtica

PROBLEMA 3

Determine o maior natural k para o qual existe um inteiro n tal que 3k divide n3 3n2 + 22.
PROBLEMA 4

Quantos dados devem ser lanados ao mesmo tempo para maximizar a probabilidade de se obter exatamente um 2?
PROBLEMA 5

Em um quadriltero convexo ABCD, os lados opostos AD e BC so congruentes e os pontos mdios das diagonais AC e BD so distintos. Prove que a reta determinada pelos pontos mdios das diagonais forma ngulos iguais com AD e BC.
PROBLEMA 6

Colocamos vrios palitos sobre uma mesa de modo a formar um retngulo m n, como mostra a figura. Devemos pintar cada palito de azul, vermelho ou preto de modo que cada um dos quadradinhos da figura seja delimitado por exatamente dois palitos de uma cor e dois de outra cor. De quantas formas podemos realizar esta pintura?

... ...
m


...
n

EUREKA! N16, 2003

17

Sociedade Brasileira de Matemtica

SOLUES NVEL 1 SOLUO DO PROBLEMA 1

a) Os palndromos entre 2000 e 3000 so da forma 2aa2, onde a um algarismo. Logo os prximos quatro sero 2112, 2222, 2332 e 2442. b) Como o primeiro algarismo igual ao ltimo, um palndromo mpar maior que 2002 deve comear e terminar por um nmero mpar maior ou igual a 3. Logo o prximo ser 3003.
SOLUO DO PROBLEMA 2

Seja S a rea do tringulo ABC. BC Se BD = , ento ( ABD) = S . 4 4


S AC 4 = 4 =S. Se AE = , ento ( AED ) = ( ADC ) = 3 3 3 3 4 S S S + 4 4 S. = 2 4 S 3 S EFC ( ) 4 = S. Se EG = EC, ento (GFC ) = = 2 2 8 S Como (GFC) = 40 temos = 40 S = 320 alqueires. 8 S 3S

DC Se DF = , ento ( DEF ) = ( DEC ) = 2 2

SOLUO DO PROBLEMA 3

Uma possvel soluo : 2002, 200, 20, 2, 4, 8, 16, 32, 64, 128, 256, 512, 51, 102, 204, 408, 816, 1632, 163, 326, 652, 1304, 130, 13.
SOLUO DO PROBLEMA 4

Como os sapatos de Marisa eram azuis, e nem o vestido nem os sapatos de Jlia eram brancos, conclui-se que os sapatos de Jlia eram pretos e portanto os sapatos de Ana eram brancos. O vestido de Ana era branco, pois era a nica que usava vestido e sapatos da mesma cor; conseqentemente, o vestido de Jlia era azul e o de Marisa era preto.
SOLUO DO PROBLEMA 5

A soma dos pontos 40. Segundo as regras do jogo, as possibilidades so:


EUREKA! N16, 2003

18

Sociedade Brasileira de Matemtica

20 20 15 5 10 10 55 5555 15 10 55 10 5 5 55

20 + 20 20 + 15 + 5 20 + 10 + 10

(1) (2) (3)

20 + 10 + 5 + 5 (4) 20 + 5 + 5 + 5 + 5 (5) 15 + 15 + 10 (6) 15 + 15 + 5 + 5 (7) 15 + 10 + 10 + 5 (8) 15 + 10 + 5 + 5 + 5 (9) 15 + 5 + 5 + 5 + 5 + 5 (10)

15

10

55555

10 10 10 5555 55

10 + 10 + 10 + 10 (11) 10 + 10 + 10 + 5 + 5 (12) 10 + 10 + 5 + 5 + 5 + 5 (13)

10 5 5 5 5 5 5 no d, pois h apenas 5 varetas verdes.

A resposta portanto: de 13 maneiras diferentes.


SOLUO DO PROBLEMA 6

Como a diferena entre o 17 e o 3 14, esses nmeros devem estar em posies afastadas de 14 casas, contadas na horizontal ou vertical. Portanto 17 e 3 devem ocupar as extremidades de uma das diagonais do tabuleiro. A partir disso, o preenchimento das diagonais feito de maneira nica. E uma maneira de se preencher o tabuleiro a seguinte:
17 16 15 14 13 12 11 10 16 15 14 13 12 11 10 9 15 14 13 12 11 10 9 8 14 13 12 11 10 9 8 7 13 12 11 10 9 8 7 6 12 11 10 9 8 7 6 5 11 10 9 8 7 6 5 4 10 9 8 7 6 5 4 3

a soma dos nmeros escritos nas diagonais : 8 10 + (3 + 5 +...+ 17) = 160.


EUREKA! N16, 2003

19

Sociedade Brasileira de Matemtica

SOLUES NVEL 2 SOLUO DO PROBLEMA 1

Seja t > 0 o tempo, em minutos, decorrido desde a sada de Geraldinho e Magro at o instante do encontro. Sejam g e m as distncias entre o ponto de encontro e as casas de Geraldinho e Magro, respectivamente. Como Geraldino percorre a distncia g em t minutos e a distncia m em 10 minutos, temos g = t . Analogamente,
g 40 = . m t

Logo

m 10 t 40 = t 2 = 400 t = 20 10 t

(pois t > 0). Logo

Geraldinho andou 10 + 20 = 30 minutos e Magro andou 40 + 20 = 60 minutos.


SOLUO DO PROBLEMA 2

Sejam x, y, z e w as reas das regies branca, amarela, azul e verde, respectivamente. Seja R o raio do semicrculo. Temos x + y = R
2

w x y

R 2 1 e y + z = x + w = (2 R ) 2 = 8 2

Assim, x + y = y + z = x + w, logo x = z e y = w. Como se x a rea de um segmento circular de ngulo R 2 R 2 2 2 90 e raio R, e x= = R


4 2 4
+ 2 2 Assim x = z < y = w. y = R . 4

SOLUO DO PROBLEMA 3

Como a diferena entre o 17 e o 3 14, esses nmeros devem estar em posies afastadas de 14 casas, contadas na horizontal ou vertical. Portanto 17 e 3 devem ocupar as extremidades de uma das diagonais do tabuleiro. A partir disso, o preenchimento das diagonais feito de maneira nica. E uma maneira de se preencher o tabuleiro a seguinte:
17 16 15 14 13 12 11 10 16 15 14 13 12 11 10 9 15 14 13 12 11 10 9 8 14 13 12 11 10 9 8 7 13 12 11 10 9 8 7 6 12 11 10 9 8 7 6 5 11 10 9 8 7 6 5 4 10 9 8 7 6 5 4 3

EUREKA! N16, 2003

20

Sociedade Brasileira de Matemtica

a soma dos nmeros escritos nas diagonais : 8 10 + (3 + 5 +...+ 17) = 160.


SOLUO DO PROBLEMA 4

Observe que o posto do observador coincide com o centro do crculo circunscrito ao quadriltero ABCD. Como BD = 16 , sendo O o centro do crculo circunscrito, temos = 2 BAD = 90 e BO = OD = r , donde 16 2 = r 2 + r 2 , pelo teorema de BOD Pitgoras, e logo r = 128 = 8 2 . Assim, a distncia do posto (que deve ficar em O) aos ninhos ser de 8 2 metros.
SOLUO DO PROBLEMA 5

Os primeiros nmeros da seqncia so (7, 14, 17, 20, 5, 8, 11, 5...) donde vemos que, exceto pelos 4 primeiros termos, a seqncia peridica com perodo 3. Como 2002 deixa resto 1 quando dividido por 3, o nmero procurado coincide com aquele que ocupa o 7o. lugar na seqncia, a saber, 11.
Observao:

Para qualquer termo inicial, a seqncia construda de acordo com mtodo descrito no enunciado do problema ser eventualmente peridica, (isto teremos an + k = ak para todo k m, para certos valores positivos de m e n).
SOLUO DO PROBLEMA 6

a) Os palndromos entre 2000 e 3000 so da forma 2aa2, onde a um algarismo. Logo os prximos quatro sero 2112, 2222, 2332 e 2442. b) Como o primeiro algarismo igual ao ltimo, um palndromo mpar maior que 2002 deve comear e terminar por um nmero mpar maior ou igual a 3. Logo o prximo ser 3003. c) Um palndromo de quatro algarismos da forma abba = a + 10b + 100b + 1000a = 1001a + 110b, que mltiplo de 11, j que 110 e 1001 so mltiplos de 11. Logo o prximo ano palndromo primo tem no mnimo 5 algarismos. Os menores palndromos de 5 algarismos so 10001, que mltiplo de 73 e 10101, que mltiplo de 3. O prximo 10201 = 1012, divisvel por 101. O seguinte, 10301, primo, pois no divisvel por qualquer primo menor que 10301 < 102.

EUREKA! N16, 2003

21

Sociedade Brasileira de Matemtica

SOLUES NVEL 3 SOLUO DO PROBLEMA 1

Veja a soluo do problema No. 5 do Nvel 2.


SOLUO DO PROBLEMA 2

Seja C a circunferncia de centro O circunscrita ao polgono A1A2...An. Os tringulos A . AiAi + 1 O (com An + 1 = A1) so issceles. Seja i = OA i i +1 Ento (1) 1 + 2 = 2 + 3 = 3 + 4 = ... = n + 1 . Portanto. 1 = 3 = 5 = ..., 2 = 4 = 6 = ... = 2 n
A2

A1

2 2 3 3
O A3

n n

A sero iguais Se n for mpar, ento 1 = 2 = ... = n , logo todos os ngulos Ai O i +1 e o polgono ser regular. Para n par, no necessrio que todos os ngulos sejam iguais. 180(n 2) Escolhendo x y de modo que x + y = ngulo interno = e fazendo n x = 1 = 3 = ... = n 1 , y = 2 = 4 = ... = n , obtemos um polgono inscritvel no regular com todos os ngulos de mesma medida. Portanto, para n par 4, existe um polgono de n lados satisfazendo as condies do problema.
SOLUO DO PROBLEMA 3

Se n = 3r, ento n 3 3n 2 + 22 = (3r ) 3 3 (3r ) 2 + 22 a soma de um mltiplo de 3 com 22, logo no mltiplo de 3. Se n = 3r + 1, ento
EUREKA! N16, 2003

22

Sociedade Brasileira de Matemtica

n3 3n2 + 22= (3r +1)3 3(3r +1)2 + 22= (3r)3 + 3 (3r)2 + 3 (3r) +1 3 (3r)2
3 2 (3r ) 3 + 22 = (3r )3 3 (3r ) + 20 , que tambm no mltiplo de 3.

Finalmente, se n = 3r 1, ento n 3 3n 2 + 22 = (3r 1) 3 3(3r 1) 2 + 22 = = (3r) 3 3 (3r) 2 + 3 (3r) 1 3 (3r) 2 + 3 2 (3r) 3 + 22 = (3r) 3 6 (3r) 2 + 9 3r + 18, que a soma de um mltiplo de 27 com 18, e portanto mltiplo de 9 mas no de 27, logo a maior potncia de 3 que divide um nmero da forma n 3 3n 2 + 22 32 = 9. Assim, k no mximo 2.
SOLUO DO PROBLEMA 4

Suponha que os dados esto numerados de 1 a n. A probabilidade de que somente o dado No. 1 resulte em 2 : 1 5 5 5 5 n 1 ... = . 6 6 6 6 6n Analogamente, a probabilidade de que somente o dado k, (1 k n) resulte em 2 5 5 5 1 5 5 5 n 1 ... ... = n . 6 6 6 6 6 6 6 Portanto, a probabilidade de obter exatamente um 2 5 n 1 5 n 1 5 n 1 5 n 1 Pn = n + n + ... + n = n n . 6 6 6 6 n 1 n 5 5 Agora observe que Pn Pn +1 n n (n + 1) n +1 6n 5( n + 1) n 5. 6 6 Para n = 5, ocorre a igualdade (P5 = P6), P5 = P6 > P7 > P8 > P9 >... e P1 < P2 < P3 < P4 < P5 = P6 E a probabilidade mxima para n = 5 ou n = 6.
SOLUO DO PROBLEMA 5

Sejam M e N os pontos mdios de AC e BD e P o ponto mdio do lado AB. Ento PM base mdia do ABC e PN base mdia do ABD. Segue que
PM = BC AD = = PN . 2 2

Sendo X e Y as intersees da reta MN com BC e AD, temos ento M = PM N = PN M = AY N ou BX M = PM N = PN M = AY N . BX

EUREKA! N16, 2003

23

Sociedade Brasileira de Matemtica


B

P A

X Y N M C D

SOLUO ALTERNATIVA:

Provaremos que se, M =

A+C B+D e N= ento o vetor MN faz ngulos iguais 2 2

com AD e BC . Para isso, como AD = BC , basta ver que os produtos internos MN AD e MN BC tm o mesmo mdulo. Temos
(C B ) ( D A ) D A A+C BD MN AD = ( N M ) ( D A ) = (D A ) = 2 2 = (C B ) ( D A ) C B 2
2
2

( D + B A C ) (C B ) = ( M N ) ( C B ) = MN BC 2

SOLUO DO PROBLEMA 6

H 3n maneiras de colorir a fileira horizontal superior de palitos. O palito vertical mais esquerda da primeira linha tambm pode ser colorido de 3 maneiras.
n

... . . .  . . .  . . . . . .  . . .
Uma vez definidas as cores dos palitos superior e mais esquerda de um quadradinho, h duas maneiras de complet-lo segundo as condies do enunciado: se ambos tm mesma cor, h duas escolhas para a cor dos dois palitos restantes; se ambos tm cores diferentes, h duas maneiras de colorir os dois palitos restantes com estas cores. Assim, para completar a primeira linha de quadrados h 3n 3 2n maneiras Da mesma forma, a cor do palito vertical mais esquerda da segunda linha de quadrados pode ser escolhido de 3 maneiras, e h 2n maneiras de colorir os demais palitos desta linha. Assim, para m = 2, h 3n 3 2n 3 2n coloraes possveis. Analogamente, no caso geral, h 3 n (3 2 n ) m = 3 n + m 2 nm maneiras de realizar a pintura pedida.
EUREKA! N16, 2003

24

Sociedade Brasileira de Matemtica

XXIV OLIMPADA BRASILEIRA DE MATEMTICA


Problemas e Solues da Terceira Fase
PROBLEMAS NVEL 1 PROBLEMA 1

No quadriculado ao lado esto escritos todos os inteiros de 1 a 25. Considere todos os conjuntos formados por cinco desses nmeros, de modo que, para cada conjunto, no existem dois nmeros que esto na mesma linha ou na mesma coluna. a) Apresente um conjunto cujo maior elemento o 23. b) Apresente um conjunto cujo maior elemento o menor possvel.
PROBLEMA 2

2 15 14 3

13 16 1 9

11 23 7 10 8 4 17

12 21 24 25 22 18 6 5

20 19

No desenho ao lado, a reta t perpendicular ao segmento AB e passa pelo seu ponto mdio M. Dizemos que A o simtrico de B em relao reta t (ou em relao ao segmento PQ ). Seja XYZ um tringulo retngulo de rea 1m2. Considere o tringulo X'Y'Z' tal que X' o simtrico de X em relao ao lado YZ , Y' o simtrico de Y em relao ao lado XZ e Z' o simtrico de Z em relao ao lado XY . Calcule a rea do tringulo X'Y'Z'.

A P M

B t

PROBLEMA 3

Um parque tem a forma de um quadriltero e possui oito portes de entrada: um em cada vrtice do quadriltero e um no meio de cada lado. Os portes foram numerados de 1 a 8, de forma que a soma T dos nmeros em cada lado a mesma para os quatro lados. Apresente um exemplo de numerao dos pontos para cada um dos possveis valores de T.

EUREKA! N16, 2003

25

Sociedade Brasileira de Matemtica

PROBLEMA 4

Sete moedas esto dispostas em crculo, com a coroa visvel. a) Mostre que possvel, virando-se cinco moedas consecutivas de cada vez, fazer com que todas fiquem com a cara visvel. b) Mostre que no possvel, virando-se quatro moedas consecutivas de cada vez, fazer com que todas fiquem com a cara visvel.
PROBLEMA 5

So dados um tabuleiro de xadrez (8 8) e palitinhos do tamanho dos lados das casas. Dois jogadores jogam alternadamente e, em cada jogada, um dos jogadores coloca um palitinho sobre um lado de uma casa do tabuleiro, sendo proibido sobrepor palitinhos. Vence o jogador que conseguir completar primeiro um quadrado 1 1 de palitinhos. Supondo que nenhum jogador cometa erros, qual dos dois jogadores tem a estratgia vencedora, ou seja, consegue vencer independentemente de como jogue seu adversrio?
PROBLEMAS NVEL 2 PROBLEMA 1

Veja o problema No. 2 do Nvel 1.


PROBLEMA 2

Mostre que, entre dezoito inteiros consecutivos de trs algarismos, sempre existe algum que divisvel pela soma de seus algarismos.
PROBLEMA 3

So dados um tabuleiro quadriculado m n e palitinhos do tamanho dos lados das casas. Dois jogadores jogam alternadamente e, em cada jogada, um dos jogadores coloca um palitinho sobre um lado de uma casa do tabuleiro, sendo proibido sobrepor palitinhos. Vence o jogador que conseguir completar primeiro um quadrado 1 1 de palitinhos. Supondo que nenhum jogador cometa erros, qual dos dois jogadores tem a estratgia vencedora, ou seja, consegue vencer independentemente de como jogue seu adversrio?

EUREKA! N16, 2003

26

Sociedade Brasileira de Matemtica

PROBLEMA 4

Uma mistura possui os componentes A e B na razo 3 : 5, uma segunda mistura possui os componentes B e C na razo 1 : 2 e uma terceira mistura possui os componentes A e C na razo 2 : 3. Em que razo devemos combinar a 1a, 2a e 3a misturas para que os componentes A, B e C apaream na razo 3 : 5 : 2?
PROBLEMA 5

Seja ABC um tringulo inscrito em uma circunferncia de centro O e P um ponto sobre o arco AB que no contm C. A perpendicular traada por P reta BO intersecta AB em S e BC em T. A perpendicular traada por P a AO intersecta AB em Q e AC em R. Prove as duas afirmaes a seguir: a) PQS um tringulo issceles b) PQ 2 = QR ST
PROBLEMA 6

1 1 1 Seja n um inteiro positivo. Definimos ( n) = ... 1 p 1 p 1 p n, k 1 2 onde p1 , p 2 ,..., p k so os fatores primos distintos de n. Prove que para todo m 1, existe n tal que ( n) = m! . Obs: m! = 1 2 ... m .
PROBLEMAS NVEL 3 PROBLEMA 1

Mostre que existe um conjunto A formado por inteiros positivos tendo as seguintes propriedades: a) A tem 2002 elementos. b) A soma de qualquer quantidade de elementos distintos de A (pelo menos um) nunca uma potncia perfeita. Obs: Uma potncia perfeita um nmero da forma ab, onde a e b so inteiros positivos e b 2.

EUREKA! N16, 2003

27

Sociedade Brasileira de Matemtica

PROBLEMA 2

ABCD um quadriltero convexo e inscritvel e M um ponto sobre o lado CD, tal que o tringulo ADM e o quadriltero ABCM tm a mesma rea e o mesmo permetro. Prove que ABCD tem dois lados de comprimentos iguais.
PROBLEMA 3

Numeramos as casas de um tabuleiro quadriculado m n, onde m, n 2, com os inteiros 1, 2, 3,...,mn de modo que, para todo i mn 1, as casas i e i + 1 tenham um lado em comum. Prove que existe i mn 3 tal que as casas i e i + 3 tm um lado em comum.
PROBLEMA 4

Definimos o dimetro de um subconjunto no vazio de {1, 2,..., n} como a diferena entre seu maior elemento e seu menor elemento (em mdulo). Calcule a soma dos dimetros de todos os subconjuntos no vazios de {1, 2,..., n}.
PROBLEMA 5

Temos um nmero finito de quadrados, de rea total 4. Prove que possvel arranjlos de modo a cobrir um quadrado de lado 1. Obs: permitido sobrepor quadrados e parte deles pode ultrapassar os limites do quadrado a ser coberto.
PROBLEMA 6

Arnaldo e Beatriz se comunicam durante um acampamento usando sinais de fumaa, s vezes usando uma nuvem grande, s vezes uma pequena. No tempo disponvel antes do caf da manh, Arnaldo consegue enviar uma seqncia de 24 nuvens. Como Beatriz nem sempre consegue distinguir uma nuvem pequena de uma grande, ela e Arnaldo fizeram um dicionrio antes de ir para o acampamento. No dicionrio aparecem N seqncias de 24 tamanhos de nuvem (como por exemplo a seqncia PGPGPGPGPGPGGPGPGPGPGPGP, onde G significa nuvem grande e P significa nuvem pequena). Para cada uma das N seqncias, o dicionrio indica seu significado. Para evitar interpretaes erradas, Arnaldo e Beatriz evitaram incluir no dicionrio seqncias parecidas. Mais precisamente, duas seqncias no dicionrio sempre diferem em pelo menos 8 das 24 posies. Demonstre que N 4096 .

EUREKA! N16, 2003

28

Sociedade Brasileira de Matemtica

SOLUES NVEL 1 PROBLEMA 1: SOLUO DE MRCIO H. MORAES FERNANDES (RIO DE JANEIRO - RJ)

Das informaes dadas pelo problema conclui-se a seguinte propriedade: Propriedade: Como dois nmeros no podem ficar na mesma coluna ou na mesma linha, sendo que para formar um conjunto so precisos 5 nmeros e sabendo que o quadriculado possui 5 linhas e 5 colunas, cada nmero do conjunto tem que ocupar uma linha e uma coluna e conseqentemente, cada linha e cada coluna estaro ocupadas por um nmero do conjunto a ser formado. A) A resoluo mais simples para que dois nmeros no se encontrem na mesma linha ou na mesma coluna so as diagonais. Na diagonal do nmero 23, apenas o nmero 25 maior que este. Assim peguei todos os nmeros da diagonal menos o 25 que tive que substituir pelo 3 (que estava numa coluna que ainda no usara) e assim no pude utilizar o 20 se no repetiria a coluna, dessa forma o ltimo nmero foi o 19 que estava em linha e coluna que no utilizei. Conjunto A = {23, 7, 21, 3, 19}. O conjunto A a soluo para o item a). B) O menor nmero que pode ser maior no conjunto de 5 nmeros o 5. Assim, fui eliminando os nmeros na seqncia. O nmero 5 pode ser descartado porque o 4 e o 3 esto na mesma linha e para fazer o conjunto sendo 5 o maior, os dois teriam que ser utilizados. O nmero 6 pode ser descartado porque na 4a. coluna da esquerda para a direita, o nico nmero menor que 6 est na mesma linha que ele. Se o nmero 7 for usado, o nico nmero menor que ele na segunda coluna, estar na mesma linha dele. Com o 8, na quarta coluna poder ser escolhido o 7 ou 5, escolhendo qualquer um, outros nmeros no podero ser utilizados: O 1 (na segunda coluna) ou o 6 (na terceira coluna). Com o 9 e o 10, se forem escolhidos, o nmero 1 no poder ser utilizado por estar na mesma linha, no restando outro nmero na segunda coluna a ser utilizado. E com o 11 pode se fazer um conjunto obedecendo a propriedade. Conjunto B = {1, 3, 6, 8, 11} sendo 11 o menor nmero possvel.
PROBLEMA 2

Veja a soluo do problema No. 1 do Nvel 2.


PROBLEMA 3: SOLUO DE LUCIO ASSAOKA HOSSAKA (CURITIBA PR)

Os possveis valores de T so 12, 13, 14 e 15 pois 8 < T < 16 = 1 + 8 + 7 (note que 4 + 5 + 6 < 16 a maior soma possvel de nmeros fora de {1, 7, 8}). 9 no o valor de T pois 8 deve existir e, usando 3 nmeros, impossvel fazer com que a soma de seu lado seja 9. O mesmo acontece com 10, j que no se pode usar 1
EUREKA! N16, 2003

29

Sociedade Brasileira de Matemtica

e 1. No pode, igualmente, ser 11 o valor de T pois 7 e 8 devem existir. O nico jeito de 7 chegar a 11 com mais dois nmeros, 7, 1, 3, pois no se pode repetir nmeros. O nico jeito de somar dois nmeros a 8 com o resultado 11 8, 1 e 2. Fazemos ento uma ilustrao:
1 7 3 8 2

Restam para colocar, os nmeros 4, 5 e 6. impossvel somar 2 desses nmeros com 3 resultando 11. Os nicos valores para T, so: 12, 13, 14 e 15.
3 12 8 1
7 14 3 4 2

2 4
13

5 7 1

6 3

5
6

6
1 5 8
15

7 5 3

6 1

PROBLEMA 4: SOLUO DE DEBORAH DE S PEREIRA BELFORT (RECIFE PE)

a) Para conseguir desviar as sete moedas, foi preciso desvirar as cinco primeiras moedas, e depois desvira-se as prximas cinco, e algumas voltaro a estar viradas no lado Coroa. Continuo com este ciclo at chegar o resultado:
= coroa = cara

EUREKA! N16, 2003

30

Sociedade Brasileira de Matemtica

b) De 4 em 4, que um nmero par, no se consegue as sete moedas viradas. Virando as moedas de 4 em 4, a quantidade de caras vai ser sempre nmero par; e 7 mpar.
PROBLEMA 5

Veja a soluo do problema No. 3 do Nvel 2.


SOLUES NVEL 2 PROBLEMA 1: SOLUO DE ANDR L. RIBEIRO DOS SANTOS (PINDAMONHANGABA - SP)
X Y' B

.A

Z'

..

A2

X'

' X ' = YZX = XYZ X ' YZ ( LAL) YZ XZ = X ' Z '


Logo XZ // X ' Z ' (olhe os ngulos formados pela transversal ZZ ' ). Marque os pontos B e C no segmento XZ , como mostra a figura. Seja Y ' A1 a altura do BY ' C , em relao a Y ' . Prolongue A1 at encontrar o segmento X ' Z ' , formando 90 em A2.
EUREKA! N16, 2003

31

Sociedade Brasileira de Matemtica

Agora, note que Y ' A2 a altura do

Z ' Y ' X ' , em relao a Y ' .

Chame a medida de XZ de y med( XZ ) = y = med( X ' Z ') . Chame a medida de YA1 de h med(YA1 ) = h

ZYX , em relao a Y; portanto h = YA1 = YA2 que a altura correspondente no Z ' YX '. Como Y simtrico a Y' em relao a XZ , ento YA1 = Y ' A1 = h YA1 a altura do
Assim Y ' A1 = YA1 = YA2 = h, rea do Logo
XYZ = XZ YA1 b h yh = = 2 2 2

yh = 1. 2
X 'Y ' Z ' =

b h X ' Z ' Y ' A2 = = 2 2 X Z Y ' A1 + Y A1 + Y A 2 3 yh y (h + h + h) = = = 2 2 2 3 yh yh De =1 =3 2 2

rea do

rea do

X ' Y ' Z ' = 3m 2 .

PROBLEMA 2: SOLUO DE EDUARDO FISCHER (ENCANTADO - RS)

Um nmero divisvel por 18 cumpre a condio. Um nmero assim possui a soma dos algarismos igual a 9 ou a 18 (27 s com o 999, que no par). Qualquer nmero divisvel por 18 divisvel por 9 e 18. Como em cada 18 nmeros inteiros consecutivos um divisvel por 18 o problema est resolvido. Resp. Entre quaisquer 18 inteiros consecutivos, um divisvel por 18. A soma dos algarismos de um mltiplo de 18 (com 3 algarismos) 18 ou 9. Em qualquer caso, o nmero divisvel pela soma dos algarismos.
PROBLEMA 3: SOLUO DE ANDR L. RIBEIRO DOS SANTOS (PINDAMONHANGABA - SP)

Para preencher todos os quadrados do tabuleiro, precisamos de um nmero mpar de palitos, se as paridades de m e n forem diferentes; ou de um nmero par de palitos, se as paridades forem iguais: i) m e n so de paridades diferentes: o primeiro jogador coloca o primeiro palito na posio central do tabuleiro e imita espelhadamente *(em relao ao palito) as
EUREKA! N16, 2003

32

Sociedade Brasileira de Matemtica

jogadas do adversrio. Haver uma hora em que todos os quadrados sero ocupados com 2 palitos e ser a vez do segundo jogador. Este por sua vez preenche um dos quadradinhos com o terceiro palito e o primeiro jogador o completa em seguida, vencendo o jogo. ii) m e n so de paridades iguais: o segundo jogador copia as jogadas do primeiro, espelhadamente*, quando sobram todos os quadrados preenchidos com 2 palitos a vez do primeiro jogador, este preenche um quadrado com o terceiro palito, e o segundo jogador o completa ganhando o jogo. *espelhadamente: como se estivesse olhando para um espelho, tem a mesma profundidade mas invertido lateralmente. Exemplos:
par x par mpar x mpar
mpar x par

A C B D C A
central

A C
central

Em todos os casos A est espelhando a B e C est espelhando D. Se m e n tem a mesma paridade o segundo jogador ganha, se tem paridades diferentes o primeiro ganha.
PROBLEMA 4: SOLUO DE THOMS YOITI SASAKI HOSHINA (RIO DE JANEIRO RJ)

Temos na mistura:

3 5 A e B 8 8 1 2 II B e C 3 3 2 3 III A e C 5 5
I Queremos que na mistura IV

3 1 1 A, B e C 10 2 5

Se pegarmos x da I, y da II e z da III teremos:

EUREKA! N16, 2003

33

Sociedade Brasileira de Matemtica

3 2 3 A x+ z = 8 5 10 15 x + 16 z = 12 1 5 1 B y + x = 15 x + 8 y = 12 3 8 2 10 y + 9 z = 3 2 3 1 + = C y z 3 5 5 y= 6 3 z= 29 29

y = 2z

Teriamos que y =

6 3 20 e z= , logo x = 29 29 29 x : y : z = 20 : 6 : 3.
A

PROBLEMA 5: SOLUO DE THOMS YOITI SASAKI HOSHINA (RIO DE JANEIRO RJ)


P

90

MR

B 90 N

T C X

de , logo QAM = 90 e sendo ABO issceles a) Chamemos PQS = . = 90 , ento PSQ ABO Logo PSQ issceles. = e OBC = teremos ento que, como b) Agora chamemos PAQ =, = BXP = PAB BPS
EUREKA! N16, 2003

APQ PSB , logo

AQ PQ = PQ PS = AQ BS PS BS

34

Sociedade Brasileira de Matemtica

Como queremos provar que PQ = QR ST , e PQ = PS , Basta apenas provar AQ BS = QR ST ou Pelo Pelo

AQ ST = QR BS

AQM , sen = AQR,

AM AM AQ = sen AQ

QR AR AR cos = QR = sen(90 ) sen sen AQ AM Logo = QR AR cos cos( ) =


Pelo BST ,

AM AR

ST BT BT cos( ) = ST = sen(90 + ) sen sen BN BN Pelo BSN , sen = BS = sen BS ST BT cos( ) BN Logo = ; cos = BS BN BT AM BT cos( ) AM BT cos = = AR cos BN AR cos( ) BN cos( ) cos = 1 = 1. cos( ) cos
PROBLEMA 6: SOLUO ADAPTADA DE GABRIEL BUJOKAS (SO PAULO - SP)

Seja pi o i-simo primo positivo. (isso vem diretamente da frmula). Ento basta escrever M! da forma ao lado direito da igualdade. Para M pequeno fcil.
e2 en e2 n 1 en 1 ( p1e1 p2 ... pn pn ) = p1e1 1 p2 ( p1 1)...( pn 1); com n, e1 + *

1! = 20 (2 1) = (2) 2! = 2 (2 1) = (4) 3! = 20 31 (2 1)(3 1) = (18)


EUREKA! N16, 2003

35

Sociedade Brasileira de Matemtica

4! = 22 31 (2 1)(3 1) = (72)
Agora utilizarei induo. Seja pn 5 o n-simo primo. Suponha que para todo

k < pn , k ! possa ser escrito na forma acima utilizando apenas primos menores
que

pn

na

fatorao.

Ento

e2 en1 ... pn ( pn 2)! = ( p1e1 p2 1 )

implica

e2 en1 2 2 ) ( pn 2)! = ( p1e1 p2 ... pn pn ! = pn ( pn 1)( pn 2)! = ( pn 1 pn ). Para os m com pn < m < pn +1 , m um produto de primos menores ou iguais a pn , donde m ! = m (m 1)! tambm da forma acima. Concluso: Para todo M existe um N tal que M ! = ( N ).

SOLUES NVEL 3 PROBLEMA 1: SOLUO DE THIAGO MORELLO PERES (RIO DE JANEIRO - RJ)

Por absurdo, suponhamos a inexistncia da seqncia satisfazendo o item b. Seja p um nmero primo maior que 2005003. Seja uma seqncia a progresso aritmtica de primeiro termo p e a razo p:

A = { p, 2 p,3 p..., 2002 p}

Assim qualquer soma do tipo n p com n < p at mesmo para a soma total: (1 + 2002) 2002 = p 2005003 p 2 Garante-se assim, que a soma no potncia perfeita, quaisquer que sejam as parcelas desta. Como este exemplo no confere com a suposio, esta um absurdo e, portanto existem seqncias satisfazendo os itens a e b simultaneamente. cqd.
PROBLEMA 2: SOLUO DE ELDER RODRIGO B. CAMPOS (RIO DE JANEIRO - RJ)
C e M d

k a c

B b A

EUREKA! N16, 2003

36

Sociedade Brasileira de Matemtica

2 p (ADM ) = 2 p( ABCM ) a + b + e + k = c + d + k c + d = a + b + e (I)

(d + e) c sen ab sen + . 2 2 cd sen S ( ADM ) = ora, se S (ADM ) = S ( ABCM ) e 2 S (ADM ) + S ( ABCM ) = S ( ABCD ) S ( ABCD ) = 2S (ADM ) (d + e)c sen + ab sen = 2cd sen ec + ab = cd
S ( ABCD ) =

(II) ab = c(d e). De (I): b + a c = d e. (I) em (II) ab = c(a + b c) ab = ac + bc c 2

a(b c) = c(b c) b = c ou a = c Logo, ABCD tem dois lados de mesmo comprimento. cqd.
PROBLEMA 3: SOLUO DE HENRIQUE CHOCIAY (PINHAIS - PR)

A numerao da tabela pode ser comparada com o preenchimento de uma malha de pontos, observe: Ex.: Tabela 3 4

11 10 9
Malha 3 4

12 1 8

3 2 7

4 5 6

Fim

Incio

O preenchimento da tabela anlogo tarefa de passar por todos os pontos da malha com uma linha nica (sem "quebras" ou bifurcaes). A ocorrncia de i ao lado de (i + 3), por sua vez, anloga s figuras:

EUREKA! N16, 2003

37

Sociedade Brasileira de Matemtica

i+1

i+3

i+2

na malha. O problema torna-se, ento, provar que impossvel preencher a tabela sem realizar uma dessas figuras. A malha formada por (m 1)(n 1) quadrados de 4 pontos prximos, os quais tero alguns de seus lados preenchidos ao fim do preenchimento. Se houver quadrado com 3 lados pintados, haver

i+3

Ou i ao lado de i + 3. O total de lados dos quadrados (com multiplicidade) 4(m 1) (n 1) = t) Para fazer a linha, efetuamos (mn 1) riscos, que podem preencher lados de 1 ou de 2 quadrados. Se o risco for feito na lateral da malha, preencher apenas 1 lado de quadrado. Exemplo:

Se o risco for feito no "miolo" da malha, preencher dois lados de quadrado. Exemplo:

Supondo a distribuio mais homognea de lados preenchidos, cada quadrado tem o mesmo nmero de seus lados preenchidos. Se o nmero de lados preenchidos por riscos for maior que a metade do total de lados de quadrados, haver com certeza um quadrado com 3 lados riscados.
EUREKA! N16, 2003

38

Sociedade Brasileira de Matemtica

No >

t) 2

O nmero de lados preenchidos por p riscos laterais ( p 1) = p O nmero de lados preenchidos por (mn 1) p riscos no meio 2(mn 1) 2p O nmero total de riscos : [2(mn 1) 2p] + (p) = 2mn p 2 = No. O nmero mximo de riscos laterais :
n pontos

2(m 1) + 2(n 1) 1 Pmax = 2m + 2n 5

n1 riscos/lados

m 1 riscos

O nmero mnimo de lados preenchidos 2mn Pmax 2 = 2mn 2m 2n + 3 = = No min. Se N o min >

t) , fica provado que h 2 2mn 2m 2n + 3 >

(ou similar) e i ao lado de i + 3

4(m 1)(n 1) 2 2mn 2m 2n + 3 > 2mn 2m 2n + 2 3>2


O nmero de lados preenchido maior que a metade do total de lados. H e portanto h i ao lado de i + 3 para qualquer tabela.

PROBLEMA 4: SOLUO ADAPTADA DE RODRIGO KENDY YAMASHITA (SO PAULO - SP)

Sejam m e M as somas dos elementos mnimos e mximos dos subconjuntos. Como o dimetro de um conjunto definido como a diferena entre seu mximo e seu mnimo, a soma desejada igual a M m. Note que podemos incluir os subconjuntos unitrios, j que seus mximos e mnimos coincidem.

EUREKA! N16, 2003

39

Sociedade Brasileira de Matemtica

O nmero k , 1 k n, elemento mnimo dos subconjuntos da forma {k} A, sendo A um subconjunto de {k + 1; k + 2;...; n}. Logo k elemento mnimo de 2n
k

subconjuntos.

Conseqentemente, m =

k 2
k =1

n k

= ( n k ) 2k .
k =0

n 1

Contemos o nmero de subconjuntos de dimetro k. Seja a o mnimo de um desses subconjuntos. O seu mximo , ento, a + k. Assim, a + k n a n k . Logo podemos escolher a de n k maneiras. Como h k 1 nmeros entre a e a + k, podemos escolher os demais elementos do subconjunto de 2k 1 maneiras. Logo h (n k ) 2k 1 subconjuntos de dimetro k. Como h, no total, 2n n 1 subconjuntos no vazios e no unitrios,

(n k ) 2
k =1

n 1

k 1

= 2 n 1 2 (n k ) 2
n k =1 n 1

n 1

k 1

=2

n +1

2n 2 (n k ) 2k =
k =1

n 1

= 2n +1 2n 2 (n k ) 2k = 2n +1 2n 2 + n = 2n +1 n 2
k =0

Logo m = 2 n 2. Para calcular M, basta observar que podemos associar cada conjunto A = {a1 ; a2 ;...; an } ao conjunto f ( A) = {n + 1 a1; n + 1 a2 ;...; n + 1 an }, de modo que se a = mn A ento n + 1 a = max f ( A). A funo f claramente uma bijeo. subconjuntos no vazios, 2n 1 n n M = (n + 1) (2 1) m M m = (n + 1) 2 n 1 2m M m = Logo, como h

n +1

= (n + 1) 2n n 1 2 (2n +1 n 2) = (n 3) 2n + n + 3.
PROBLEMA 5: SOLUO DE RAFAEL DAIGO HIRAMA (CAMPINAS SP)

Podemos supor ento que os quadrados tm lado menor que 1, caso contrrio s posicionar o quadrado de 1 ou mais sobre o quadrado a se cobrir. Vamos classificar os quadrados como do tipo Qk tal que o lado do quadrado seja menor que

1 2
k 1

e maior ou igual a

1 . 2k

Se tivermos um Q0 acabou, pois ele ter lado maior ou igual a 1 e pronto. Caso contrrio vamos dividir o tabuleiro em 4 partes iguais. Cada uma tem lado 1/2, ou seja, satisfatoriamente coberto por um Q1 cada um. Ento se posiciona todos os disponveis. Se tiver pelo menos 4 Q1 acabou.
EUREKA! N16, 2003

40

Sociedade Brasileira de Matemtica

Caso contrrio os que sobraram devem ser divididos em 4 e preenchidos por quantos Q2 tiverem. E assim sucessivamente at preencher o tabuleiro. Exemplo:
Q2 Q1 Q2 Q2 Q2

Q3 Q3 Q2 Q2 Q2 Q3 Q3

Q1

Agora, para provar que isso sempre possvel basta provar que a rea total dos quadrados usados menor que 4. Assim, j que o modo de preenchimento pede "use tantos do Qk quanto existirem", se sobrar buraco ou esqueceu-se de usar um quadrado em um passo anterior ou falta usar os quadrados menores. Para isso vamos ver o desperdcio de cada quadrado, ou seja, quanto do quadrado no usamos para preencher a rea de interesse (por exemplo, o desperdcio de um quadrado Q3 ao ser colocado sobre um tabuleiro de lado 1/8 o quanto do quadrado ficar de fora desse tabuleiro, mesmo que esse resto esteja sobre outra parte do tabuleiro total ele vai ser contado como desperdcio). Vejamos, como sempre usamos Qk para cobrir um tabuleiro de lado Qk de

1 , a rea do 2k

1 2
2k 2

no mximo e

1 1 no mnimo e a do tabuleiro 2 k , logo o desperdcio 2k 2 2

1 2
2k 2

1 3 = 2 k no mximo, isso prova que o desperdcio no passa de 2k 2 2

3 2 2 k = 3 vezes da rea preenchida, ou seja, desperdiado no total no mximo 3/4 1 2 2k


da rea dos quadrados utilizados, ou seja, 1/4 pelo menos utilizado. Como o total da rea dos quadrados 4, a rea utilizada pelo menos 1, o que termina o problema.

EUREKA! N16, 2003

41

Sociedade Brasileira de Matemtica

PROBLEMA 6: SOLUO DE FBIO DIAS MOREIRA (RIO DE JANEIRO - RJ) Definio: A distncia entre duas palavras p e q o nmero de posies em que duas

palavras diferem (smbolo: d(p, q)).


Teorema 1: d(p, q) + d(q, r) d(p, r). Prova: Seja o conjunto das posies em que e diferem. Ento o teorema

equivale a #( pq ) + #( qr ) #( pr ). Mas pr = ( pq qr ) ( pq qr ) , pois s h dois tipos de nuvens, logo p e r so iguais nas posies onde ambos diferem de q. Mas pq qr pq qr , logo #( pr ) = #( pq qr ) #( pq qr ) =

= #( pq ) + #(qr ) 2#( pq qr ) , e nossa afirmao equivale a #( pq qr ) 0,


obviamente verdadeiro.
Definio: A palavra real mais prxima a uma palavra q a palavra p que:

i)

Pertence ao dicionrio. Minimiza a distncia entre p e q. (se existir mais de uma palavra que atende i) e ii) todas elas so mais prximas).
ii) Definio: A vizinhana de uma palavra p pertencente ao dicionrio o conjunto de todas as palavras mais prximas a p (smbolo: p ). Teorema 2: Toda vizinhana de uma palavra p contm todas as palavras cujas

distncias at p sejam menores ou iguais a 4. Prova: Seja q tal que d ( p, q) 4 mas q p . Ento q r para r pertencente ao dicionrio. Isso implica d(q, r) < d( p, q) 4, logo d( p, r) d( p, q) + d(q, r) < 4+ 4 = 8 , absurdo, pois p e r no poderiam estar simultaneamente no dicionrio. Teorema 3: Toda palavra p pertence a no mximo seis vizinhanas. Suponha que Como Prova: p q1 q2 q3 q4 q5 q6 q7 .

d (qi , q j ) d ( p, qi ) + d ( p, q j ), d ( qi , q j ) 8. Como d (qi , q j ) 8, d ( qi , q j ) = 8.


Como d ( p, qi ) = d ( p, q j ), d ( p, qi ) = 4, i {1,..., 7}. Como cada palavra s tem 24 nuvens, existem duas palavras (q1 e q2, sem perda de generalidade) tais que Mas ento, pelos argumentos do teorema 1, pq1 pq2 .

d (q1, q2 ) = d ( p, q1) + d ( p, q2 ) 2#(pq1 pq2 ) d (q1, q2 ) = 8 2#(pq1 pq2 ) 8 2 1 = 6


absurdo, pois q1 e q2 no poderiam pertencer simultaneamente ao dicionrio. N mximo quando todas as palavras distam no mximo quatro da palavra do dicionrio mais prxima a ela e todas as palavras que distam exatamente quatro da
EUREKA! N16, 2003

42

Sociedade Brasileira de Matemtica

palavra do dicionrio mais prxima pertencem a seis vizinhanas, j que isso caracteriza a formao mais densa possvel, devido ao seguinte teorema:
Teorema 4: p q r , q r d ( p, q) = d ( p, r ) = 4. Prova: Suponha que d ( p, q) = d ( p, r ) < 4 (a igualdade obrigatria pela definio

de vizinhana). Ento d (q, r ) d ( p, q) + d ( q, r ) < 4 + 4 = 8 , absurdo, pois as duas palavras no poderiam pertencer simultaneamente ao dicionrio. Porqu isso valida nossa afirmao acima? Porque nenhum ponto que dista trs ou menos ao ponto mais prximo pertence a mais de uma vizinhana. Assim, o arranjo descrito acima o mais denso, pois todas as palavras que no pertencem ao dicionrio esto sempre cercadas por palavras do dicionrio (pertencem sempre ao nmero mximo de vizinhanas). Nas circunstancias acima descritas #( p ) = #( q ) para todo p e q (pois
0 1 2 3 4 constante e igual a #( p ) por (1)). Alm disso, + C24 + C24 + C24 + C24 C24

#(
pD

) , onde D o dicionrio, seria 224 , mas no, pois as palavras que distam

quatro de uma palavra no dicionrio so contadas seis vezes. Vamos achar ento n'p , com um fator de correo apropriado:
0 1 2 3 4 + C24 + C24 + C24 + C24 n 'p = C24

d =4

6 d=0 (prpria palavra) d =1 d =2 d =3 Vamos contar s uma vez!

n'p = 1 + 24 + 276 + 2024 +

10626 = 4096 (que coincidncia!) 6

Mas at agora consideramos o melhor caso h algum desperdcio de palavras envolvido. Logo algumas vizinhanas so maiores do que so no caso ideal. Por isso, n'p 4096 em geral. Assim 224 4096 N = 212 N N 212 = 4096.

EUREKA! N16, 2003

43

Sociedade Brasileira de Matemtica

XXIV OLIMPADA BRASILEIRA DE MATEMTICA


Problemas e Solues da Primeira Fase Nvel Universitrio
PROBLEMA 1

A funo f : (1,+) # contnua e derivvel. Sabe-se que f(0) = 0, f '(0) = a e que f(x + 1) = e f(x) para todo x > 1. Calcule f '(3).
PROBLEMA 2

Seja A a matriz real n n

x ... x x + y x + y ... x x A= . x x x y +

   

Diga para que valores de x e y a matriz A inversvel e calcule A 1.


PROBLEMA 3

Calcule

x2 +1 + x 1 x2 + 1 + x + 1

dx.

PROBLEMA 4

Determine todos os valores inteiros positivos de m para os quais o polinmio (x + 1)m + xm + 1 divisvel por (x2 + x + 1)2.
PROBLEMA 5

Jogamos 10 dados comuns (com 6 faces equiprovveis numeradas de 1 a 6). Calcule a probabilidade de que a soma dos 10 resultados seja igual a 20.
PROBLEMA 6

Considere a curva C = {( x, y ) # 2 y 2 = x 3 43x + 166}. a) Seja Q = (a, b) um ponto de C. Suponha que a reta tangente a C no ponto Q intersecte C num nico outro ponto, Q'. Determine as coordenadas de Q'. Seja P0 = (3, 8). Para cada inteiro no negativo n, definimos Pn +1 = P' n , o ponto de interseo de C com a reta tangente a C em Pn. Determine P2002.

b)

EUREKA! N16, 2003

44

Sociedade Brasileira de Matemtica

SOLUO DO PROBLEMA 1

Derivando a equao f ( x + 1) = e f ( x ) temos f ' ( x + 1) = f ' ( x) e f ( x ) . Assim f (1) = e 0 = 1, f ' (1) = f ' (0) e f (0 ) = a f (2) = e1 = e, f ' (2) = f ' (1) e f (1) = ae

f '(3) = f '(2) e f (2) = ae e+1 .


SOLUO DO PROBLEMA 2

Se y = 0 todas as linhas so iguais e a matriz no inversvel. Se nx + y = 0 a soma das n linhas 0 e portanto a matriz novamente no inversvel. Vamos mostrar que se nenhuma destas duas condies ocorre a matriz inversvel.
1 1 Se B =  1 1 1


... ...
 

1 1 temos B2 = nB e A = yI + xB.  1

Tome C =

1 x B. I y y ( nx + y )

1 x AC = (yI + xB ) y I y (nx + y ) B =I
Comentrio (no faz parte da soluo)

Encontramos C conjecturando que A 1 = uI + vB. E resolvendo um sistema para encontrar u e v. Pode-se demonstrar antes de tentar resolver o sistema que A1, se existir, deve ter a forma acima: A 1 uma funo analtica de A, logo um polinmio em A, logo um polinmio em B. Como observamos que B2 um mltiplo escalar de B segue que todo polinmio em B da forma uI + vB.
SOLUO ALTERNATIVA

Vamos resolver o sistema


( x + y )a1 + x a 2 + ... + x a n = b1 x a ( x y ) a ... x a b + n= 2 1+ + 2 +  x a1 + x a 2 + ... + ( x + y ) a n = bn
EUREKA! N16, 2003

45

Sociedade Brasileira de Matemtica

Somando todas as equaes, obtemos ( nx + y )(a1 + ... + a n ) = (b1 + ... + bn ), donde


x(a1 + ... + an ) = x (b1 + ... + bn ) , caso nx + y 0. nx + y

Diminuindo
y a j = bj

essa

igualdade

da

j-sima

equao,

obtemos

x (b1 + ... + bn ) e, caso nx + y x (n 1) x + y x y 0, a j = b1 ...+ bj ... bn. y(nx + y) y(nx + y) y(nx + y)


(n 1) x + y y (nx + y ) x = y (nx + y ) x y (nx + y ) x y (nx + y ) ( n 1) x + y y (nx + y ) x y (nx + y ) ... ... . (n 1) x + y y (nx + y ) x y( nx + y ) x y( nx + y )

Assim,

A 1

Note que, se nx + y = 0, o sistema no tem soluo se b1 +...+ bn 0, e, se y = 0, o x sistema no tem soluo se b j (b1 + ... + bn ) 0 para algum j . Em nenhum nx + y desses casos A invertvel.
SOLUO DO PROBLEMA 3
2 Seja f ( x) = x + 1 + x 1 . Racionalizando, temos

x2 +1 + x +1

2 2 2 x2 ( x + 1 + x 1) x + 1 x 1 x + 1 1 = f ( x) = , logo f ( x ) = f ( x), para 2 2 x 2 ( x + 1) ( x + 1)

todo x, e portanto,

f ( x) dx = 0.

SOLUO ALTERNATIVA

Vamos achar uma primitiva de f: Em sec2 d , e, como

sec + tan + 1 sec


EUREKA! N16, 2003

sec + tan 1

dx fazemos x = tan , dx = x 2 +1 + x +1 tan 2 + 1 = sec (para < < ), obtemos 2 2


d =

x 2 +1 + x 1

46

Sociedade Brasileira de Matemtica

cos

1 + sen cos
2

(1 + sen + cos )

d .

Fazendo tan

2dz 2z 1 z 2 , = z, d = , obtemos , cos = = sen 2 1+ z 2 1+ z 2 1+ z 2


2

2 z 2 + 2 z 1 + z 2 2dz 2 z (1 + z 2 )dz = . 2 2 2 + 2z 1 z 1+ z (1 z 2 ) 2

Buscando A, B, C, D tais que

2 z (1 + z 2 ) A B C D , obtemos (A + + + = 1 + z (1 + z) 2 1 z (1 z ) 2 (1 z 2 ) 2

+ B + Az)(1 z)2 + (C + D Cz)(1+z)2 = 2z(1 + z2), donde A C = 2, B A + D C = 0, A 2B + C + 2D = 2, A + B + C + D = 0. Assim, D = B, C = A, logo A = 1, C = 1, D = 1, B = 1. Assim,

(1 z

2 z (1 + z 2 )
2 2

dz = ln(1 + z) +

1 1 2 . + ln(1 z ) + = ln(1 z 2 ) + 1+ z 1 z 1 z 2

Quando x varia entre 1 e 1, varia entre


e , donde z varia entre tan 4 4 8

sen 2 4 = e tan . Temos tan = = 2 1, donde z varia entre 1 2 e 8 8 1 + cos 2+ 2 4

2 1. Assim, a integral ln(1 z 2 ) + SOLUO DO PROBLEMA 4

2
2

2 1

1 z 1 2

= 0,

pois ( 2 1) 2 = (1 2 ) 2 .

1 i 3 + uma raiz de x2 + x + 1. Para que (x2 + x + 1)2 divida 2 2 (x + 1)m + xm + 1 = P(x), devemos ter P() = 0 e P'() = 0. Assim, ( + 1) m + m = 1 e m(( + 1) m1 + m 1 ) = 0. Seja = 1 i 3 + tal que ( + 1) 2 = e ( + 1) 3 = 1 . Como e + 1 2 2 so razes sextas da unidade, o comportamento se repetir com perodo 6. Assim, ( + 1) m 1 + m 1 = 0 equivale a ( + 1) m 1 = m 1 = ( + 1) 3+ 2 ( m 1) , ou seja Temos que + 1 = ( + 1) m + 2 = 1, o que equivale a m 2 (mod 6). Nesse caso, temos ( + 1) m + m = ( + 1) 4 + = 2 + = 1, donde as duas condies so satisfeitas. Assim, os nmeros que satisfazem o enunciado so os inteiros positivos da forma 6k 2.
EUREKA! N16, 2003

47

Sociedade Brasileira de Matemtica

SOLUO DO PROBLEMA 5

Devemos encontrar o nmero de solues de a1 + a 2 + ... + a10 = 20, 1 a i 6. O nmero de solues de a1 + a 2 + ... + a10 = 20, ai 1 claramente 9 . Devemos agora descontar as solues para as quais apenas um dentre os ai 7 pois caso contrrio tal soma seria 7 + 7 + 8 1 = 22. Assim, basta descontar 10 vezes o nmero de solues de a1 + a 2 + ... + a10 = 20, a1 7 , ai 1 , ~ + a + ... + a = 14, a 19 13 e a  , a 1, que 13 . Assim ou de a
1 2 10 1 i

19

N = 10 9 9

19 13 9 9 10 . probabilidade pedida p = 10 6

SOLUO DO PROBLEMA 6
2 dy = 3x 2 43, donde dy = 3 x 43 . dx dx 2y A equao da reta tangente a C passando por (a, b)

a) De y 2 = x 3 43x + 166, temos 2 y

3a 2 43 3a 2 43 2 3 Substituindo em y = x 43 x + 166 temos y = 2b x + b 2b a .

3a 2 43 2b 2 3a 3 + 43a , que ter uma raiz dupla em x = a, x + x 43 x + 166 = 2b 2b


3

2 e cuja soma das razes 3a 43 . Assim, o outro ponto ter primeira coordenada 2b igual a 3a 43 2a , e, substituindo na equao da reta, segunda coordenada
2 2

2b

2 3 3 2 2 2 igual a 3a 43 2a 3a 43 + 2b 3a + 43a = 3a 43 + 2b 9a + 129a . 2b 2b b b b 2 2 2 2

b) Usando a frmula acima, obtemos P1 = (5, 16), P2 = (11, 32), P3 = (3, 8), P4 = (5, 16), P5 = (11, 32) e P6 = (3, 8). Assim, a seqncia (Pn) peridica de perodo 6, logo P2002 = P4 = ( 5, 16) . Observao: No item b), o fato de P3 diferir de P0 apenas por uma troca de sinal da segunda coordenada j suficiente para concluir que a seqncia peridica de perodo 6.
EUREKA! N16, 2003

48

Sociedade Brasileira de Matemtica

XXIV OLIMPADA BRASILEIRA DE MATEMTICA


Problemas e Solues da Segunda Fase Nvel Universitrio
Seja y = P(x) um polinmio de grau 4. Mostre que se existe uma reta (em #2) que corta o grfico de P em 4 pontos ento existe uma reta que corta o grfico em 4 pontos igualmente espaados.
PROBLEMA 2 PROBLEMA 1

A = (a ij ) uma matriz real simtrica n n tal que aii = 1 e i {1,2,..., n} . Prove que 0 < det A 1.
PROBLEMA 3

a
j =1

ij

< 2 , para todo

Sejam A1 , A2 ,..., Ak {1, 2,..., n} conjuntos com A i todo i, j com i j . Prove que
PROBLEMA 4
k i =1

n n e Ai A j para 2 4

Ai

k n. k +1

Determine todas as solues reais da equao x = 2 + 2 2 + x.


PROBLEMA 5

Dado x #, definimos ln 0 ( x) = x e, para cada k , se ln k ( x) > 0 , definimos ln k +1 ( x) = ln(ln k ( x)), onde ln o logaritmo natural. Dado n inteiro positivo, definimos k(n) como o maior k tal que ln k ( n) 1 , e an como

ln
j =0

k (n)

(n) = n ln(n) ln ln(n) ... ln k ( n) (n).

Diga se a srie
PROBLEMA 6

a
n =1

1
n

converge ou diverge.

Considere duas elipses no plano #2 que se intersectam em 4 pontos. Nestes 4 pontos trace as retas tangentes s duas elipses, obtendo assim 8 retas. Prove que existe uma elipse (ou circunferncia) tangente a estas 8 retas.
EUREKA! N16, 2003

49

Sociedade Brasileira de Matemtica

SOLUES NVEL UNIVERSITRIO PROBLEMA 1: SOLUO DE FABRCIO SIQUEIRA BENEVIDES (FORTALEZA CE)

Seja P ( x ) = ax 4 + bx 3 + cx 2 + dx + e e seja )( x) = kx + q a reta que o intersecta

em 4 pontos. Ou seja, Q( x) = P( x) )( x) tem quatro raizes. Queremos mostrar que existe r ( x) = tx + s tal que S ( x ) = P( x ) r ( x) tem quatro raizes igualmente espaadas. S ( x ) = ax 4 + bx 3 + cx 2 + (d t ) x + (e s ) = ax 4 + bx 3 + cx 2 + d ' x + e '. Note que nosso problema equivalente a dados a, b, c achar d', e' tais que S(x) acima tenha 4 raizes igualmente espaadas. Primeiro, mostraremos que possvel escolher d' de modo que S(x) seja b b b simtrico em relao reta x = , isto , S k = S + k , k . 4a 4a 4a b Para escrever menos seja u = . 4a S (u k ) = S (u + k ) a (u k ) 4 + b(u k )3 + c(u k ) 2 + d '(u k ) + e ' = = a(u + k )4 + b(u + k )3 + c(u + k )2 + d '(u + k ) + e ' a(u4 4u3k + 6u2k 2 4uk3 + k 4 ) + b(u3 3u2k + 3uk 2 k3 ) + c(u2 2uk + k 2 ) dk = a(u4 + 4u3k + 6u2k 2 + 4uk 3 + k 4 ) + b(u3 + 3u2k + 3uk 2 + k 3 ) + c(u2 + 2uk + k 2 ) + d ' k = 8au 3 k + 8auk 3 + 6bu 2 k + 2bk 3 + 4cuk + 2d ' k = 0 b 8auk 3 = 2bk 3 u = 4a 3 2 (4au + 3bu + 2cu + d ') k = 0 Basta ento tomar d ' = 4au 3 3bu 2 2cu O fato de j existir uma reta que intersecta o P(x) inicial em 4 pontos, nos diz que a, b, c nos foram dados de modo que S(x) tenha 3 pontos de mx/mn locais. (Seno o grfico de S(x) seria convexo ou cncavo, e qualquer reta o intersectaria em no mximo 2 pontos).
Logo, o grfico de S(x) algo do tipo:

EUREKA! N16, 2003

50

Sociedade Brasileira de Matemtica

ou

x=

b 4a

Finalmente mudar e' significa transladar o grfico de S para cima ou para baixo. Claramente podemos escolher um e tal que S tenha 4 razes x1e , x2e , x3e , x4e . Para cada e' desses considere as funes

f (e) = x2e x1e


s f

g (e) = x3e x2 e , h(e) = x4 e x3 e

para r e s (ver grfico)

f (r ) = 0, g (r ) > 0 f ( s ) > 0, g (s ) = 0 Pelo T.V.I. existe e ' (r , s ) tal que f (e ') = g (e '). Neste caso, f (e ') = g (e ') = h(e ') , e este o nosso to procurado e'.
PROBLEMA 2: SOLUO DE HUMBERTO SILVA NAVES (S.J. DOS CAMPOS - SP)

Pela escolha de d', S simtrico e f(e) = h (e).

Temos que a matriz A diagonalizvel, pois simtrica, ou seja: A = H D H 1 onde H T = H 1 e D uma matriz diagonal formada pelos autovalores de A. Obs. As matrizes D e H so reais. Claramente det. A = det. D. Primeiramente vamos provar que todos os auto-valores de A so positivos: Seja X um auto-vetor de A, i.e., X uma matriz n 1, no nula, tal que

EUREKA! N16, 2003

51

Sociedade Brasileira de Matemtica

AX = X

x1 X =  xn

Seja i, tal que 0 < xi = max{ x1 ; x 2 ;...; xn } . Se fosse 0 teramos: xi = ai1 x1 + ai 2 x2 + ... + ain xn

( 1) xi = ai1 x1 + ai 2 x2 + ... + aii 1 xi 1 + aii +1 xi +1 + ... + ain xn


Temos

( 1) xi

xi e

j i 1 j n

aij < 1

j i 1 j n

aij x j < xi , pois

xi = max{ x1 ;...; xn }, logo xi ( 1) xi =

j i 1 j n

ax
ij

j i 1 j n

aij x j < xi um

absurdo! Logo det D > 0 det A > 0. Claramente um auto-valor de A uma raiz de P(x) = det (A xI). O coeficiente de x n 1 de P(x) a soma da diagonal principal de A multiplicada por

( 1)

n 1

, ou seja, ( 1)

n 1

n . Logo a soma das razes de P(x) (com suas respectivas


n 1

(1) n multiplicidades) = n. (1) n


Temos: 1 + 2 + ... + n = n onde os i's so os auto-valores de A, com i 0 i ; 1 i n. Pela desigualdade das mdias, temos:

1 + 2 + ... + n n 12 ...n 12 3 ...n 1 n Mas det D = 12 ...n 1 det A 1.


1=
PROBLEMA 3: SOLUO DE CARLOS STEIN NAVES DE BRITO (S.J. DOS CAMPOS - SP)

Seja

k i =1

Ai = U
k

Seja S = Ai k
i =1

n (I) 2
k n (II) k +1

Por absurdo, suponha que U <


EUREKA! N16, 2003

52

Sociedade Brasileira de Matemtica

Seja ai quantas vezes os elementos de Ai aparecem nos outros A j ( j i ) , por exemplo, se A1 = {1, 2,3} , A2 = {1, 2, 5} e A3 = {1,3, 6} , temos que a1 = 4. Temos

ai = Ai A j
j =1 j i

Para cada t Lema:

k i =1

Ai , o seja bt o nmero de Ai's que contm t.

a
i =1

j
k

b j (b j 1).

Ai
i =1

Prova: Temos que ai quantas vezes aparece cada t Ai em outros A j ( j i ) , logo cada t aparece em (bt 1) outros Aj, pois bt conjuntos contm t, tirando o Ai, logo temos (bt 1) outros que contm t. Assim ai = Para cada t
k i =1

tAi

(b 1) a = ( (b 1)).
t i =1 i i =1 tAi t

Ai , existem bt conjuntos que contm t, logo cada parcela bt 1

aparece bt vezes em

(b 1) , logo a
k

i =1

t Ai

i =1

b (b 1). cqd.
t t t
k

Ai
i =1

Logo

ai =
i =1

t
k

bt2 bt =

t
k

bt2

t
k

bt (note que

Ai
i =1

Ai
i =1

Ai
i =1

t
i=1

Ai

k n bt = Ai k , pois 2 i =1

estamos contando quantas vezes aparece cada elemento). Por Cauchy-Schwarz:

2 bt k k k 2 n2 k 2 n2 t Ai Ai S 2 (k +1)k n (bt 2 ) i=k1 = i=1 = 4 > 4 = k k U U U 4 n t Ai Ai k +1 i =1 i =1

ai = bt2 bt >
i =1
EUREKA! N16, 2003

(k +1)kn kn kn k n(k 1) = (k +1 2) = 4 2 4 4

53

Sociedade Brasileira de Matemtica

Pelo princpio das casas dos pombos, a j , tal que a j >

kn(k 1) , absurdo). 4

k n (k 1) 4 k

(seno

n(k 1) Aj Ai , . De novo pela casa dos pombos, como aj = Logo a j , a j > 4 i=1

i j

n(k 1) n n 4 existe um p tal que A j Ap > = A j Ap > , absurdo. 4 4 k 1 k 1 aj


Logo U

k n. k +1

PROBLEMA 4: SOLUO DE MRCIO AFONSO ASSAD COHEN (RIO DE JANEIRO - RJ)

, t difcilvamos tentar uma idia: Seja x = 2cos , com (0, / 3) (Ok, pois j sei que 1 < 2 x 2 + 2 < 2). Obs: cos 2 = 2 cos 2 1 = 1 2 sen 2 2 + x = 2(1 + cos ) = 4 cos 2 ( / 2) 2 + x = 2 cos( / 2) (pois / 2 (0, / 6) logo cos( / 2) > 0 ).

2 + x = 2 cos( / 2) 2 2 + x = 2 2 cos( / 2) = 2(1 cos( / 2)) = = 2 2 sen 2 ( / 4) 2 2 + x = 2 sen( / 4) (pois sen( / 4) > 0 ).
Logo, x = 2 + 2sen( / 4) Obs.: sen( / 4) = cos( / 2 / 4) Portanto, tirando raiz: x = 2cos( / 4 / 8), i.e. 2 cos = 2 cos( / 4 / 8) cos = cos( / 4 / 8)
2 2 + 2sen( / 4) = 2 + 2cos( / 2 / 4) = 2 [1+ cos( / 2 / 4)] = 2 2 cos ( / 4 /8)

= / 4 / 8 9 / 8 = / 4 = 2 / 9 Logo, x = 2cos 2 / 9 a nica soluo real da equao.

EUREKA! N16, 2003

54

Sociedade Brasileira de Matemtica

PROBLEMA 5: SOLUO DA BANCA

Sejam b0 = 1, bk +1 = e bk para todo k . Para bk n < bk +1 temos k(n) = k. Como a derivada de ln k +1 ( x)


ln k +1 ( n + 1) ln k +1 ( n) =
n +1 n

ln
j =0

1 =: g k ( x), temos, para cada n, ( ) j x

g k ( x) dx g k ( n ), para todo n bk, pois gk decrescente.

Assim, Como

1 = g k (n ) ln k +1 ( bk +1 ) ln k +1 ( bk ). bk n < bk +1 an bk n < bk +1

ln k +1 ( bk ) < ln k +1 (bk + 1) < ln k +1 (bk ) +

1 1 = 2 2

para todo k 1 e

ln k +1 ( bk +1 ) ln k +1 (bk +1 ) = 1 , temos
1 a = diverge. n n =1 k = 0 bk n < bk +1 an

1 1 para todo k, donde > 2 bk n < bk +1 a n

PROBLEMA 6: SOLUO DA BANCA

Vamos utilizar coordenadas projetivas (Ref.: "Aplicaes de planos projetivos em Teoria dos Nmeros e Combinatria" de Carlos Yuzo Shine - Eureka! No. 15).

. O fato de serem elipses significa que essas cnicas no cortam a "reta do infinito" z = 0. Lema: Se, A, B, C e D so tais que 3 quaisquer no so colineares, existe um sistema de coordenadas projetivas no qual A = [1, 0, 0], B = [0, 1, 0], C = [0, 0, 1] e D = [1, 1, 1]
Consideremos as duas cnicas do problema inseridas no plano projetivo

Demonstrao: Sejam a , b , c , d vetores de 3 representando as classes de equivalncia de A, B, C e D respectivamente. Como A, B e C no so / / / colineares, a , b e c so l.i., logo existem reais k, l, m tais que d = ka + lb + mc . Como d no pertence s retas AB, BC ou AC, k, l, m so / / / diferentes de zero. Assim, se considerarmos a base ka, lb , mc de 3 teremos A = [1, 0, 0], B = [0, 1, 0], C = [0, 0, 1], D = [1, 1, 1], o que prova o lema.
EUREKA! N16, 2003

/ // /

55

Sociedade Brasileira de Matemtica

Sejam A, B, C e D os pontos de interseo entre as duas elipses. Usando o lema, podemos realizar uma mudana de coordenadas que leve os pontos [1, 1, 1], [1, 1, 1], [1, 1, 1] e [1, 1, 1] em [1, 0, 0], [0, 1, 0], [0, 0, 1] e [1, 1, 1] como toda mudana de coordenadas invertvel, usando o lema podemos realizar uma mudana de coordenadas que leve os pontos A, B, C e D em [1, 1, 1], [1, 1, 1], [1, 1, 1], [1, 1, 1]. Podemos escolher esse novo sistema de coordenadas de modo que as duas cnicas continuem sendo elipses, em relao reta do infinito z = 0. De fato a famlia de cnicas no plano que passam por (1, 1), (1, 1), (1, 1) e (1, 1) dada pelas equaes tx 2 + (1 t ) y 2 = 1 , t ( se t {0,1} a cnica se degenera num par de retas). No novo sistema de coordenadas temos duas cnicas dessa famlia. Se uma delas uma hiprbole, digamos tx 2 + (1 t ) y 2 = 1 , com t > 1, podemos aplicar a mudana de coordenadas y 1 projetivas que leva [X, Y, Z] em [Y, Z, X] (e, no plano, leva (x, y) em , ): x x a imagem de Q = {(1, 1), (1, 1), (1, 1), (1, 1)} ainda Q e a imagem da (t 1) x 2 y 2 + = 1 . hiperbole ( tx 2 + (1 t ) y 2 = 1 ) a elipse t t Assim, temos agora duas cnicas que passam pelos pontos de Q tais que, se uma delas uma hiperbole ento a outra uma elipse. Assim, ou essas cnicas so duas elipses ou qualquer reta no plano intersecta uma dessas cnicas. O segundo caso no possvel, pois nesse caso as cnicas no poderiam ser imagem de duas elipses por uma mudana de coordenadas projetivas, dado que a imagem da reta do infinito, que continua sendo uma reta, sempre intersecta uma dessas cnicas. Agora, temos duas elipses que passam pelos pontos de Q. Suponhamos que suas tangentes no ponto (1, 1) sejam as retas (ax + by = 1) e (cx + dy = 1), com a, b, c, d > 0, a + b = c + d = 1. Aps aplicarmos uma mudana de coordenadas afim do tipo T(x, y) = ( x / , y ) , com > 0, obtemos duas outras elipses cujas retas tangentes em T (1, 1) = (1/ ,1) so (a x + by = 1) e (c x + dy = 1). As distncias dessas retas origem so, 1 1 respectivamente, . e a 2 2 + b 2 c2 2 + d 2
EUREKA! N16, 2003

56

Sociedade Brasileira de Matemtica

d 2 b2 Temos a + b = c + d = 2 2 , a c a + b = c + d ( a c > 0 d b > 0).


2 2 2 2 2 2 2

que

positivo,

pois

Assim, tomando =

d 2 b2 , e aplicando T ( x, y ) = ( x / , y ) s nossas duas a2 c2 elipses, obtemos duas elipses que se intersectam em quatro pontos de modo que todas as 8 retas tangentes s duas elipses nesses pontos esto a uma mesma distncia da origem (por simetria), e logo existe uma crculo tangente a todas elas, o qual est contido na unio dos interiores dessas elipses, e portanto no intersecta a imagem da reta do infinito pela mudana de coordenadas projetivas que leva as elipses originais nestas, e logo imagem de uma elipse por essa mudana de coordenadas. Essa elipse tangente s 8 retas do enunciado. Isso resolve o problema.

Nota: Os enunciados dos problemas 3 e 5 da segunda fase do Nvel Universitrio


n

saram com alguns erros na prova: no problema 3, aparecia


i =1 k

Ai em vez de
(x)

Ai , e , no problema 5, aparecia
i =1
k ( n) j =0

ln (x) = x ln(x) lnln(x) ... ln


j =0 j

k (n)

k (n)

em vez de

ln (n) = n ln(n) lnln(n) ... ln


j

k ( n)

(n).

Agradecemos a Okakamo Kokobongo Matsubashi pela reviso deste nmero. Errata: No artigo "Reciprocidade Quadrtica", de Carlos Gustavo Moreira e Nicolau Saldanha (publicado na Eureka! No. 15), onde est "smbolo de Lagrange" deveria ser "smbolo de Legendre".

EUREKA! N16, 2003

57

Sociedade Brasileira de Matemtica

XXIV OLIMPADA BRASILEIRA DE MATEMTICA Resultado Nvel 1 (5a. e 6a. Sries)


Henrique Ponde de Oliveira Pinto Camila Alves Pereira Cssio Kendi Takamori Jssica Guerra Caldato Vincius Marques Regitano Mrio Henrique Mendona Castilho Bernardo de Oliveira Veiga Rafael Tupynamb Dutra Gabrielle Collato Marcelino Maria Fernanda Petri Beto Guilherme Philippe Figueiredo Cristiano Peres Guimares Gustavo Henrique dos Santos Figueiredo Larissa Lais de S Rafael Augusto da Silva Gonalves Diogo Bonfim Moraes Morant de Holanda Fernanda S Leal de Moura Lusa Dias Barbosa Alves David Francisco dos Santos Marcos Coppa Gomes Filho Anderson Vasconcelos Maciel Samuel Carvalho Lima Holanda Rodolfo de Andrade Marinho Silva Franz Biondi Siemon Rafael Sampaio de Rezende Andr Vasconcelos Barros Guilherme Silva Moura
Daniel Luna de Menezes Las Moutinho Medeiros Rafael Moura e Sucupira Vinicius de Souza Lima e Oliveira Weslen Costa Timoteo Filipe Alves Tom Lays Cardoso Tatagiba Marlon Vieira de Lima Jnior Lukas Carmona Macedo de Souza Alessandro Wagner Palmeira Nathlia Pereira Gonalves Renan Magri Rafael Alcoforado Domingues Letcia Duarte Ferrari Lucio Eiji Assaoka Hossaka Flavia Contartesi Odair Dutra Santana Jnior Mayara Franco Rodrigues Artur de Almeida Losnak Thasa Giorno Dantas Rabaneda Lopes Cssio dos Santos Arajo Amanda Yumi Iseri Tiago Madeira Matheus Mello Asuno Lusa Castro Noronha Rafael Ellis Reuben

MEDALHA DE OURO Salvador - BA Gloria do Goit - PE So Jos dos Campos - SP Santo Andr - SP Piracicaba - SP MEDALHA DE PRATA So Joo da Boa Vista - SP Rio de Janeiro - RJ Belo Horizonte - MG Santo Andr - SP So Paulo - SP Fortaleza - CE Mendona- SP Santo Andr - SP So Paulo - SP Salvador - BA MEDALHA DE BRONZE Rio de Janeiro - RJ Teresina - PI Recife - PE Serra - ES Natal - RN Fortaleza - CE Fortaleza - CE Campina Grande - PB Vitria - ES Fortaleza - CE Natal - RN Jequi - BA MENO HONROSA
Joo Pessoa - PB Recife - PE Fortaleza - CE Rio de Janeiro - RJ Paulista - PE Fortaleza - CE Itaperuna - RJ Fortaleza - CE So Paulo - SP Guarulhos - SP Rio de Janeiro - RJ Itapor - PR Joo Pessoa - PB Rio de Janeiro - RJ Curitiba - PR So Carlos - SP Botuporanga - SP Araraquara - SP So Paulo - SP Atibaia - SP Recife - PE Uberaba - MG Itaja - SC Belm - PA Valinhos - SP So Paulo - SP

EUREKA! N16, 2003

58

Sociedade Brasileira de Matemtica

Resultado Nvel 2 (7a. e 8a. Sries)


Thoms Yoiti Sasaki Hoshina Andr Lucas Ribeiro dos Santos Vitor Humia Fontoura Gabriel Tavares Bujokas Guilherme Rodrigues Nogueira de Souza Douglas Bokliang Ang Cunha Hector Kenzo Horiuti Kitahara Guilherme Rohden Echelmeier Enzo Haruo Hiraoka Moriyama Luty Rodrigues Ribeiro Eduardo Fischer Rafael Kitayama Shiraiwa Thas Viveiro Caio dos Santos Pereira Gazzola Rodrigo Augusto Santana Rodrigo Viana Soares Andr Linhares Rodrigues Fbio Eigi Imada Rafael Montezuma Pinheiro Cabral Pedro Paulo Gondim Cardoso Rhamon Barroso de Sousa Lucas Magalhes Pereira Castello Branco Max Douglas Peixoto da Silva Renata Mayer Gukovas Milena Pinheiro Martins Anderson Hoshiko Aiziro Daniel Yoshio Futenma da Silva Landerson Bezerra Santiago Jos Armando Barbosa Filho Danilo Eiki Yokoyama Fernando Mizoguchi Gorgoll Pedro Thiago Ezequiel de Andrade Jos Robrio Xavier dos Santos Jnior Erick Vizolli Camila Vasconcelos de Oliveira Raphael Rodrigues Mata Adriano Csar Braga Borges Gustavo Eidji Camarinha Fujiwara Henrique Kenji Formagio Noguchi Andr Ikeda Canto Paulo Andr Carvalho de Melo Fbio Queiroz Vasconcelos Cunha Flaviano Ramos Pereira Junior Mauro Cardoso Lopes Luiz Mller Tiago Nery Vasconcelos Thiago de Azevedo Pinheiro Hoshino MEDALHA DE OURO Rio de Janeiro - RJ Pindamonhangaba - SP Salvador - BA So Paulo - SP MEDALHA DE PRATA So Paulo - SP So Jos dos Campos - SP So Paulo - SP Itaja - SC So Paulo - SP Fortaleza - CE Encantado - RS So Paulo - SP So Paulo - SP MEDALHA DE BRONZE Belo Horizonte - MG Belm - PA Fortaleza - CE Fortaleza - CE So Jos dos Campos - SP Fortaleza - CE Salvador - BA Fortaleza - CE Fortaleza - CE Fortaleza - CE So Paulo - SP Teresina - PI So Carlos - SP MENO HONROSA So Paulo - SP Maracana - CE Fortaleza - CE So Paulo - SP So Paulo - SP Fortaleza - CE Fortaleza - CE Curitiba - PR Fortaleza - CE Salvador - BA Contagem - MG So Paulo - SP So Paulo - SP Curitiba - PR Rio de Janeiro - RJ Salvador - BA Belm - PA So Paulo - SP Vitria - ES So Paulo - SP So Paulo - SP

EUREKA! N16, 2003

59

Sociedade Brasileira de Matemtica

Resultado Nvel 3 (Ensino Mdio)


MEDALHA DE OURO Guilherme Issao Camarinha Fujiwara Fbio Dias Moreira Rafael Daigo Hirama Yuri Gomes Lima Thiago da Silva Sobral Alex Corra Abreu Henrique Chociay Antonio Carlos Maldonado Silveira A. Munhoz Henry Wei Cheng Hsu Samuel Barbosa Feitosa Larissa Cavalcante Queiroz de Lima Bernardo Freitas Paulo da Costa Davi Mximo Alexandrino Nogueira Thiago Costa Leite Santos Einstein do Nascimento Jnior Eduardo de Moraes Rodrigues Poo Rafael Tajra Fonteles Felipe Rodrigues Nogueira de Souza Murilo Vasconcelos Andrade Thiago Braga Cavalcante Paulo Ribeiro de Almeida Neto Germanna de Oliveira Queiroz Juliana Gomes Varela Rodrigo Aguiar Pinheiro Israel Franklim Dourado Carrah Daniel Pessoa Martins Cunha Renato Seiji Tavares Carlos Augusto David Ribeiro Letcia Rosa dos Santos Rafael Marini Silva Telmo Luis Correa Junior Diego Alvarez Araujo Correia Vitor Gabriel Kleine Francisco Bruno de Lima Holanda Diogo dos Santos Suyama Anderson Torres Larissa Rodrigues Ribeiro Marina Lima Medeiros Antonia Taline de Souza Mendona Rodrigo Angelo Muniz Eduardo Paiva Costa Eduardo Monteiro Nicodemos Thiago Morello Peres Elder Rodrigo Barbosa Campos Thiago Lus Viana de Santana Filipe Rodrigues de Souza Moreira Rodrigo Kendy Yamashita Joo Marcos da Cunha Silva Lyussei Abe EUREKA! N16, 2003 So Paulo - SP Rio de Janeiro - RJ Campinas - SP MEDALHA DE PRATA Fortaleza - CE Fortaleza - CE Niteri - RJ Curitiba - PR Rio de Janeiro - RJ So Paulo - SP Fortaleza - CE Fortaleza - CE Rio de Janeiro - RJ Fortaleza - CE So Paulo - SP MEDALHA DE BRONZE Fortaleza - CE So Paulo - SP Teresina - PI So Paulo - SP Macei - AL Fortaleza - CE Ananindeua - PA Fortaleza - CE Fortaleza - CE Fortaleza - CE Fortaleza - CE Fortaleza - CE So Paulo - SP Fortaleza - CE Rio de Janeiro - RJ MENO HONROSA Vila Velha - ES So Paulo - SP Fortaleza - CE Mogi das Cruzes - SP Fortaleza - CE Belo Horizonte - MG So Paulo - SP Fortaleza - CE Fortaleza - CE Fortaleza - CE Cariacica - ES Teresina - PI Rio de Janeiro - RJ Rio de Janeiro - RJ Rio de Janeiro - RJ Rio de Janeiro - RJ Rio de Janeiro - RJ So Paulo - SP Fortaleza - CE So Paulo - SP

60

Sociedade Brasileira de Matemtica

Resultado Nvel Universitrio


MEDALHA DE OURO Carlos Yuzo Shine Humberto Silva Naves Marcio Afonso Assad Cohen Thiago Barros Rodrigues Costa Carlos Stein Naves de Brito Rodrigo Villard Milet Daniel Massaki Yamamoto Giuliano Boava Fabrcio Siqueira Benevides Eduardo Famini Silva Tertuliano Franco Santos Franco Rodrigo Roque Dias Lucas de Melo Pontes e Silva Thiago Afonso de Andr Sergio Alvarez Araujo Correia Daniel Nobuo Uno Evandro Makiyama de Melo Leonardo Augusto Zo Bruno Fernandes Cerqueira Leite Daniel Mouro Martins Daniele Vras de Andrade Lucas Heitzmann Gabrielli Diogo Diniz P.S. Silva Digo Veloso Ucha Marcelo Handro Maia Gilberto Kirk Rodrigues Diogo Luiz Duarte Camilo Marcantonio Junior Marcio Miranda de Carvalho Marcio Paiva Reis Arnaldo Joo do Nascimento Junior So Paulo - SP So Jos dos Campos - SP Rio de Janeiro - RJ MEDALHA DE PRATA Fortaleza - CE So Jos dos Campos - SP Rio de Janeiro - RJ So Paulo - SP Florianpolis - SC Fortaleza - CE Rio de Janeiro - RJ MEDALHA DE BRONZE Salvador - BA So Paulo - SP So Paulo - SP So Paulo - SP Fortaleza - CE So Paulo - SP So Paulo - SP Nilpolis - RJ So Paulo - SP Rio de Janeiro - RJ Rio de Janeiro - RJ So Paulo - SP Campina Grande - PB Teresina - PI So Jos dos Campos - SP MENO HONROSA Rio de Janeiro - RJ Rio de Janeiro - RJ Rio de Janeiro - RJ Teresina - PI Vitria - ES Duque de Caxias - RJ

'0 , , 89, /0 .447/03,/4708 70 43, 8 3, 3488, 5E 3, 3, 3907309 4-2 47 -7 .447/70 92

EUREKA! N16, 2003

61

Sociedade Brasileira de Matemtica

AGENDA OLMPICA
XXV OLIMPADA BRASILEIRA DE MATEMTICA NVEIS 1, 2 e 3 Primeira Fase Sbado, 7 de junho de 2003 Segunda Fase Sbado, 13 de setembro de 2003 Terceira Fase Sbado, 18 de outubro de 2003 (nveis 1, 2 e 3) Domingo, 19 de outubro de 2003 (nveis 2 e 3 - segundo dia de prova). NVEL UNIVERSITRIO Primeira Fase Sbado, 13 de setembro de 2003 Segunda Fase Sbado, 18 e Domingo, 19 de outubro de 2003

IX OLIMPADA DE MAIO 10 de maio de 2003

XIV OLIMPADA DE MATEMTICA DO CONE SUL 23 a 30 de maio de 2003 Ica Peru

XLIV OLIMPADA INTERNACIONAL DE MATEMTICA 07 a 19 de julho de 2003 Tquio Japo

X OLIMPADA INTERNACIONAL DE MATEMTICA UNIVERSITRIA 25 a 31 de julho de 2003 Universidade Babes-Bolyai, Cluj-Napoca, Romnia

XVIII OLIMPADA IBEROAMERICANA DE MATEMTICA 13 a 20 de setembro de 2003 Argentina

VI OLIMPADA IBEROAMERICANA DE MATEMTICA UNIVERSITRIA 8 de novembro de 2003

EUREKA! N16, 2003

62

CONTEDO

AOS LEITORES IX OLIMPADA DE MAIO Enunciados e Resultado Brasileiro XIV OLIMPADA DE MATEMTICA DO CONE SUL Enunciados e Resultado Brasileiro XLIV OLIMPADA INTERNACIONAL DE MATEMTICA Enunciados e Resultado Brasileiro

2 3 6 9

X OLIMPADA INTERNACIONAL DE MATEMTICA PARA ESTUDANTES UNIVERSITRIOS 11 Enunciados e Resultado Brasileiro XVIII OLIMPADA IBERO-AMERICANA DE MATEMTICA Enunciados e Resultado Brasileiro GEOMETRIA COM CONTAS Carlos Yuzo Shine A ENUMERABILIDADE DE  x  E O CHO TRIANGULAR Jos Paulo Carneiro COMO QUE FAZ? SOLUES DE PROBLEMAS PROPOSTOS PROBLEMAS PROPOSTOS AGENDA OLMPICA COORDENADORES REGIONAIS 14

17

36

41 45 59 61 62

Sociedade Brasileira de Matemtica

AOS LEITORES
Caros leitores,

Neste nmero a Eureka! publica vrias provas de 2003 de competies internacionais nas quais o Brasil participa: Olimpada de Maio, Olimpada do Cone Sul, Olimpada Internacional, Olimpada Ibero-americana e Olimpada Internacional Uhniversitria (IMC). Este foi o primeiro ano em que uma delegao brasileira participa desta ltima competio, resultado de uma parceria da OBM com as Universidades em que estudam os membros da equipe brasileira. Comeamos bem: obtivemos trs segundos prmios, trs terceiros prmios e duas menes honrosas. Neste nmero excepcionalmente no publicamos a seo "Olimpadas ao redor do mundo", que volta no prximo. Por outro lado, criamos a seo "Como que faz?", onde vamos resolver problemas sugeridos pelos leitores. A idia que os leitores enviem problemas que no conseguem resolver, que tenham aparecido em alguma Eureka! (por exemplo em algum artigo ou prova publicada) ou alguma competio (ou de qualquer outra origem), que tentaremos resolver e publicar os mais interessantes na nossa seo.

Os editores

EUREKA! N17, 2003

Sociedade Brasileira de Matemtica

IX OLIMPADA DE MAIO
Enunciados e Resultado Brasileiro
PRIMEIRO NVEL
Durao da prova: 3 horas PROBLEMA 1

Pedro escreve todos os nmeros de quatro algarismos diferentes que podem ser armados com dgitos a, b, c, d que cumprem as seguintes condies: a 0 ; b = a + 2; c = b + 2 ; d = c + 2. Calcule a soma de todos os nmeros que Pedro escreveu.
PROBLEMA 2

O tringulo ABC retngulo em A e R o ponto mdio da hipotenusa BC. Sobre o cateto maior AB se marca o ponto P tal que CP = BP e sobre o segmento BP se marca o ponto Q tal que o tringulo PQR equiltero. Se a rea do tringulo ABC 27, calcule a rea do tringulo PQR.
PROBLEMA 3

Determine o menor nmero inteiro positivo que termina em 56, mltiplo de 56 e tem a soma de seus dgitos igual a 56.
PROBLEMA 4

Clia escolhe um nmero n e escreve a lista dos nmeros naturais de 1 at n: 1, 2, 3, 4, , n 1, n. Em cada passo, troca a lista: copia o primeiro nmero ao final e apaga os dois primeiros. Depois de n 1 passos ficar escrito um nico nmero. Por exemplo, para n = 6 os cinco passos so: 1, 2, 3, 4, 5, 6 3, 4, 5, 6, 1 5, 6, 1, 3 1, 3, 5 5, 1 5 e ficar escrito o nmero 5. Clia escolheu um nmero n entre 1000 e 3000 e depois de n 1 passos ficou o nmero 1.

EUREKA! N17, 2003

Sociedade Brasileira de Matemtica

Determine todos os valores de n que Clia pode ter escolhido. Justifique porque estes valores servem e os demais no.
PROBLEMA 5

Temos um tabuleiro quadriculado 4 4. Definimos a separao entre duas casas como o menor nmero de movimentos que deve empregar um cavalo de xadrez para ir de uma casa a outra (utilizando movimentos do cavalo). Trs casas A, B, C formam um trio bom se as trs separaes entre A e B, entre A e C e entre B e C so iguais. Determine um nmero de trios bons que se formam no tabuleiro.
OBSERVAO:

Em cada movimento o cavalo se desloca 2 casas em direo horizontal mais uma casa em direo vertical ou se desloca 2 casas em direo vertical mais uma casa em direo horizontal. SEGUNDO NVEL
Durao da prova: 3 horas

PROBLEMA 1

So escolhidos quatro dgitos a, b, c, d diferentes entre si e diferentes de zero e se escreve a lista de todos os nmeros de quatro algarismos que se obtm trocando de lugar os dgitos a, b, c, d. Que dgitos deve-se escolher para que a lista tenha a maior quantidade possvel de nmeros de quatro algarismos que sejam mltiplos de 36?
PROBLEMA 2

Seja ABCD um retngulo de lados AB = 4 e BC =3. A perpendicular diagonal BD traada por A corta BD no ponto H. Chamamos de M o ponto mdio de BH e de N o ponto mdio de CD. Calcule a medida do segmento MN.
PROBLEMA 3

Encontre todos os pares de nmeros inteiros positivos (a, b) tais que 8b + 1 mltiplo de a e 8a + 1 mltiplo de b.

EUREKA! N17, 2003

Sociedade Brasileira de Matemtica

PROBLEMA 4

Beto marcou 2003 pontos verdes no plano, de maneira que todos os tringulos com seus trs vrtices verdes tm rea menor que 1. Demonstre que os 2003 pontos verdes esto contidos num tringulo T de rea menor que 4.
PROBLEMA 5

Uma formiga, que est numa aresta de um cubo de lado 8, deve realizar um percurso pela superfcie do cubo e regressar ao ponto de partida. Seu caminho deve conter pontos interiores das seis faces do cubo e deve visitar s uma vez cada face do cubo. Encontre o comprimento do caminho mais curto que a formiga pode realizar e justifique porque o caminho mais curto.

RESULTADOS
PRIMEIRO NVEL
Lucio Eiji Assaoka Hossaka Marlon Vieira de Lima Junior Henrique Pond de Oliveira Pinto Guilherme Philippe Figueiredo Fernanda S Leal de Moura Rgis Prado Barbosa Tiago Madeira Mateus Faitanin Yin Diogo Bonfim Moraes Morant de Holanda Ronaldo Rozenbaum Paiva Medalha de Ouro Medalha de Prata Medalha de Prata Medalha de Bronze Medalha de Bronze Medalha de Bronze Medalha de Bronze Meno Honrosa Meno Honrosa Meno Honrosa Curitiba - PR Fortaleza - CE Salvador - BA Fortaleza - CE Teresina - PI Fortaleza - CE Itaja - SC Vitria - ES Rio de Janeiro - RJ Rio de Janeiro - RJ

SEGUNDO NVEL
Telmo Luis Correa Junior Luty Rodrigues Ribeiro Pedro Thiago Ezequiel de Andrade Gabriel Tavares Bujokas Juliana G. Cavalcante Leandro Farias Lima Eduardo Fischer Andr Linhares Rodrigues Rodrigo Viana Soares Raphael Rodrigues Viana
EUREKA! N17, 2003

Medalha de Ouro Medalha de Prata Medalha de Prata Medalha de Bronze Medalha de Bronze Medalha de Bronze Medalha de Bronze Meno Honrosa Meno Honrosa Meno Honrosa

So Paulo - SP Fortaleza - CE Fortaleza - CE So Paulo - SP Fortaleza - CE Fortaleza - CE Encantado - RS Fortaleza - CE Fortaleza - CE Salvador - BA

Sociedade Brasileira de Matemtica

XIV OLIMPADA DE MATEMTICA DO CONE SUL


Enunciados e Resultado Brasileiro
A XIV Olimpada de Matemtica do Cone Sul foi realizada na cidade de Ica, Peru, no perodo de 23 a 30 de maio de 2003. A equipe brasileira foi liderada pelos professores Paulo Jos Bonfim Gomes Rodrigues e Emanuel de Souza Carneiro, ambos de Fortaleza CE. Novamente a equipe brasileira obteve a maior pontuao entre os pases participantes. RESULTADOS DA EQUIPE BRASILEIRA
BRA1 BRA2 BRA3 BRA4 Fbio Dias Moreira Henry Wei Cheng Hsu Thiago Costa Leite Santos Rodrigo Aguiar Pinheiro Prata Prata Prata Ouro

PROBLEMA 1

Em um torneio de futebol entre quatro equipes, A, B, C e D, cada equipe joga com cada uma das outras exatamente uma vez. a) Decidir se possvel que, ao finalizar o torneio, as quantidades de gols marcados e sofridos pelas equipes sejam: A 1 4 B 3 4 C 6 4 D 7 5

Gols marcados Gols sofridos

Se a resposta afirmativa, d um exemplo com os resultados das seis partidas; em caso contrrio, justifique. b) Decidir se possvel que, ao finalizar o torneio, as quantidades de gols marcados e sofridos pelas equipes sejam: A 1 4 B 3 4 C 6 4 D 13 11

Gols marcados Gols sofridos

EUREKA! N17, 2003

Sociedade Brasileira de Matemtica

Se a resposta afirmativa, d um exemplo com os resultados das seis partidas; em caso contrrio, justifique.
PROBLEMA 2

Considere a seqncia {an} definida da seguinte maneira: a1 = 1 a2 = 3 an+2 = 2an+1 an + 1, para todo inteiro n 1. Provar que a mxima potncia de 2 que divide a4006 a4005 22003.
PROBLEMA 3

Seja ABC um tringulo acutngulo tal que o ngulo B mede 60o . A circunferncia de dimetro AC intersecta as bissetrizes internas de A e C nos pontos M e N respectivamente (M A, N C). A bissetriz interna do ngulo B intersecta MN e AC nos pontos R e S, respectivamente. Demonstrar que BR RS.
PROBLEMA 4

No tringulo acutngulo ABC, os pontos H, G e M encontram-se sobre o lado BC, de modo que AH, AG e AM so altura, bissetriz e mediana do tringulo, respectivamente. Sabe-se que HG = GM, AB = 10 e AC = 14. Determinar a rea do tringulo ABC.
PROBLEMA 5

Seja n = 3k + 1 , onde k um inteiro, k 1 . Constri-se um arranjo triangular de lado n formado por crculos de mesmo raio como o mostrado na figura para n = 7.

EUREKA! N17, 2003

Sociedade Brasileira de Matemtica

Determinar, para cada k, o maior nmero de crculos que podem ser coloridos de vermelho de tal modo que no existam dois crculos vermelhos tangentes entre si.

PROBLEMA 6

Demonstrar que existe uma seqncia de inteiros positivos x1 , x2 ,..., xn ,... que satisfaz as duas condies seguintes: i) contm exatamente uma vez cada um dos inteiros positivos, ii)
n para cada n = 1,2,... a soma parcial x1 + x2 + ... + xn divisvel por n .

EUREKA! N17, 2003

Sociedade Brasileira de Matemtica

XLIV OLIMPADA INTERNACIONAL DE MATEMTICA


07 a 19 de julho, Tquio - Japo
A XLIV Olimpada Internacional de Matemtica foi realizada em Tquio, Japo, no perodo de 07 a 19 de julho de 2003. A equipe brasileira foi liderada pelos professores Nicolau Saldanha, do Rio de Janeiro RJ e lio Mega, de So Paulo SP. RESULTADOS DA EQUIPE BRASILEIRA
BRA1 BRA2 BRA3 BRA4 BRA5 BRA6 Alex Corra Abreu Samuel Barbosa Feitosa Rafael Daigo Hirama Larissa Cavalcante Queiroz de Lima Fbio Dias Moreira Davi Mximo Alexandrino Nogueira Medalha de Bronze Medalha de Bronze Meno Honrosa Meno Honrosa Medalha de Prata Medalha de Bronze

PROBLEMA 1 Seja A um subconjunto do conjunto S = {1,2,

,1000000} com exatamente 101

elementos. Demonstre que existem nmeros t1 , t 2 , , t100 em S tais que os conjuntos para A j = {x + t j | x A} , j = 1, 2, ,100,

so disjuntos dois a dois.


PROBLEMA 2

Determine todos os pares de inteiros positivos (a, b) tais que


a2 2ab b3 + 1
2

um inteiro positivo.
PROBLEMA 3

Considere um hexgono convexo tal que para cada quaisquer dois lados opostos verifica-se a seguinte propriedade: a distncia entre os seus pontos mdios igual a 3 2 vezes a soma dos seus comprimentos. Demonstre que todos os ngulos do hexgono so iguais.

EUREKA! N17, 2003

Sociedade Brasileira de Matemtica

(Um hexgono convexo ABCDEF tem trs pares de lados opostos: AB e DE , BC e EF , CD e FA ).


PROBLEMA 4 Seja ABCD

um quadriltero convexo cujos vrtices esto sobre uma circunferncia. Sejam P , Q e R os ps das perpendiculares s retas BC , CA e AB , respectivamente, passando por D . Demonstre que PQ = QR se e s se as bissetrizes dos ngulos ABC e ADC se intersectam sobre a reta AC .

PROBLEMA 5

Sejam n um inteiro positivo e x1 x2 xn .

x1 , x2 ,

, x

nmeros reais tais que

(a) Demonstre que

n n 2(n 2 1) n n ( xi x j ) 2 . x x i j 3 i =1 j =1 i =1 j =1
(b) Demonstre que a igualdade vlida se e s se x1 , x2 , , xn formam uma progresso aritmtica.
PROBLEMA 6 Seja p um nmero primo. Demonstre que existe um nmero primo q tal que,

para todo inteiro n, o nmero n p no divisvel por q.

EUREKA! N17, 2003

10

Sociedade Brasileira de Matemtica

X OLIMPADA INTERNACIONAL DE MATEMTICA PARA ESTUDANTES UNIVERSITRIOS


25 a 31 de Julho, Cluj - Napoca, Romnia
A X Olimpada Internacional de Matemtica para estudantes universitrios foi realizada na cidade de Cluj-Napoca, Romnia, no perodo de 25 a 31 de Julho de 2003. A equipe brasileira foi liderada pelo professor Luciano Castro, do Rio de Janeiro RJ. RESULTADOS DA EQUIPE BRASILEIRA
Mrcio Afonso Assad Cohen Humberto Silva Naves Rodrigo Villard Milet Carlos Stein Naves de Brito Daniel Yamamoto Giuliano Boava Eduardo Famini Silva Thiago Barros Rodrigues Costa IME ITA UFRJ ITA ITA UFSC IME UNICAMP Medalha de Prata Medalha de Prata Medalha de Prata Medalha de Bronze Medalha de Bronze Medalha de Bronze Meno Honrosa Meno Honrosa

PRIMEIRO DIA
PROBLEMA 1

a) Seja a1 , a2 ,

, a ,... uma seqncia de nmeros reais tais que


n

a1 = 1 e

3 an+1 > an , n. 2
Prove que a seqncia an 3 2
n 1

tem um limite finito ou tende a infinito.

b) Prove que para todo

> 1 existe uma seqncia a1 , a2 ,


an 3 2
n 1

, a ,... com as
n

mesmas propriedades, tal que lim n

= .

EUREKA! N17, 2003

11

Sociedade Brasileira de Matemtica

PROBLEMA 2

Sejam a1 , a2 , , a51 elementos no nulos de um corpo. Simultaneamente trocamos cada elemento pela soma dos outros 50. Desta forma a nova seqncia b1 , b2 , , b51 uma permutao da anterior. Quais so os possveis valores da caracterstica do corpo?

PROBLEMA 3

Seja A uma matriz quadrada n n tal que 3A3 = A2 + A + I. Prove que (Ak )k  converge a uma matriz idempotente B (i.e., a uma matriz B tal que B2 = B).
PROBLEMA 4

Determine o conjunto de todos os pares (a, b) de inteiros positivos para os quais o conjunto dos inteiros positivos pode ser decomposto em dois conjuntos A e B tais que a A = b B.
PROBLEMA 5

Sejam g :[0,1] # uma funo contnua e f n : (0,1] # a seqncia de funes definida por f 0 ( x) = g ( x) e f n+1 ( x) = Determine lim f n ( x) para todo x (0,1] .
n

1 x f n (t )dt, x (0,1], n 0. x 0

PROBLEMA 6

Seja f ( z ) = an z + an 1 z + ... + a1 z + a0 um polinmio com coeficientes reais. Prove que se as razes de f esto no semi-plano esquerdo
n

n 1

{z  | Re( z ) < 0} ento ak ak + 3 < ak +1ak + 2 para todo k = 0, 1,, n 3.

EUREKA! N17, 2003

12

Sociedade Brasileira de Matemtica

SEGUNDO DIA
PROBLEMA 1

Sejam A e B matrizes reais n n tais que AB + A + B = 0. Prove que AB = BA.


PROBLEMA 2

Calcule o seguinte limite: lim + x x


0

2x

sen m t dt (m, n naturais dados). tn

PROBLEMA 3

a b . #n tais que existe exatamente um ponto a0 em A tal que a0 b = inf a A Prove que B denso em #n.
PROBLEMA 4

Seja A um subconjunto fechado de #n e seja B o conjunto de todos os pontos b de

Encontre todos os inteiros positivos n para os quais existe uma famlia F de subconjuntos de trs elementos de S ={1, 2, ,n} que satisfaz as seguintes condies: (i) para quaisquer elementos distintos a, b S existe exatamente um A F tal que a, b A. (ii) Se a, b, c, x, y, z so tais que {a, b, x}, {a, c, y}, {b, c, z} F ento {x, y, z} F. a) Mostre que para toda funo f : " " # existe uma funo g : " # tal que f ( x, y ) g ( x) + g ( y), x, y . para a qual no existe g : # # tal b) Encontre uma funo f : que f(x, y) g(x) + g(y), x, y #.
PROBLEMA 5

" # # #

PROBLEMA 6

Seja a0 , a1 ,..., an ,... a seqncia definida por a0 = 1 , an+1 = Calcule

ak 1 n . n + 1 k =0 n k + 2

2
k =0

ak
k

(se existir).

EUREKA! N17, 2003

13

Sociedade Brasileira de Matemtica

XVIII OLIMPADA IBEROAMERICANA DE MATEMTICA


13 a 20 de setembro, Mar del Plata - Argentina
A XVIII Olimpada Iberoamericana de Matemtica foi realizada na cidade de Mar del Plata, Argentina, no perodo de 13 a 20 de setembro de 2003. A equipe brasileira foi liderada pelos professores Augusto C. de Oliveira Morgado, do Rio de Janeiro RJ e Luzinalva Miranda de Amorim, de Salvador BA. RESULTADOS DA EQUIPE BRASILEIRA
BRA1 BRA2 BRA3 BRA4 Alex Corra Abreu Davi Mximo Alexandrino Nogueira Fbio Dias Moreira Samuel Barbosa Feitosa Medalha de Ouro Medalha de Prata Medalha de Ouro Medalha de Bronze

PRIMEIRO DIA
PROBLEMA 1

a) Tm-se duas sucesses, cada uma de 2003 inteiros consecutivos, e um tabuleiro de 2 linhas e 2003 colunas

Decida se sempre possvel distribuir os nmeros da primeira sucesso na primeira linha e os da segunda sucesso na segunda linha, de modo que os resultados obtidos ao somar os dois nmeros de cada coluna formem uma nova sucesso de 2003 nmeros consecutivos. b) E se trocssemos 2003 por 2004? Tanto em a) como em b), se a resposta for afirmativa, explique como distribuiria os nmeros, e se for negativa, justifique o porqu.

EUREKA! N17, 2003

14

Sociedade Brasileira de Matemtica

PROBLEMA 2

Sejam C e D dois pontos da semicircunferncia de dimetro AB tais que B e C esto em semiplanos distintos em relao reta AD. Denotemos por M, N e P os pontos mdios de AC, DB e CD, respectivamente. Sejam OA e OB os circuncentros dos tringulos ACP e BDP. Demonstre que as retas OA OB e MN so paralelas.
PROBLEMA 3

Pablo copia o seguinte problema: Considere todas as sucesses de 2004 nmeros reais ( x0 , x1 , x2 ,..., x2003 ), tais que

x0 = 1, 0 x1 2 x0 , 0 x2 2 x1 ,


0 x2003 2 x2002 .
Entre todas estas sucesses, determine aquela para a qual a expresso seguinte assume o seu maior valor: S = . Quando Pablo ia copiar a expresso S, apagaram o quadro. S conseguia lembrarse de que S era da forma

S = x1 x2 ... x2002 + x2003 ,


onde o ltimo termo, x2003 , tinha coeficiente +1, e os anteriores tinham coeficiente +1 ou 1. Demonstre que Pablo, apesar de no ter o enunciado completo, pode determinar com certeza a soluo do problema.

SEGUNDO DIA
PROBLEMA 4

Seja M ={1, 2,,49} o conjunto dos primeiros 49 inteiros positivos. Determine o maior inteiro k tal que o conjunto M tenha um subconjunto de k elementos em que no haja 6 nmeros consecutivos. Para esse valor mximo de k, encontre a quantidade de subconjuntos de m, de k elementos, que tenham a propriedade mencionada.

EUREKA! N17, 2003

15

Sociedade Brasileira de Matemtica

PROBLEMA 5

No quadrado ABCD, sejam P e Q pontos pertencentes aos lados BC e CD respectivemante, distintos dos extremos, tais que BP = CQ. Consideram-se pontos X e Y, X Y, pertencentes aos segmentos AP e AQ respectivamente. Demonstre que, quaisquer que sejam X e Y, existe um tringulo cujos lados tm os comprimentos dos segmentos BX, XY e DY.
PROBLEMA 6

Definen-se as sucesses (an )n 0 ,(bn ) n 0 por:

a0 = 1, b0 = 4 e 2001 2001 + bn , bn+1 = bn + a n para n 0. an+1 = an


Demonstre que 2003 no divide nenhum dos termos destas sucesses.

EUREKA! N17, 2003

16

Sociedade Brasileira de Matemtica

GEOMETRIA COM CONTAS


Carlos Yuzo Shine, Colgio Etapa Nvel Avanado
s vezes precisamos de mais elementos para resolver problemas de geometria. Pode-se traar novos elementos na figura que possam ajudar ou fazer algumas contas. Mostraremos algumas tcnicas para fazer algumas contas que ajudam (e at resolvem!). Em geral, pode-se pensar em problemas de geometria seguindo esses passos: (i) Faa a figura do problema (praticamente nenhum problema vem com figura), bem grande e com certa preciso (ou seja, use a rgua e o compasso, mas no necessrio muito rigor). Mexa um pouco com os elementos da figura. Algo que sempre til fixar um certo nmero de ngulos (de preferncia, o menor nmero possvel, de modo que os ngulos marcados determinem a figura - a no ser, claro, que acrescentar algum outro ngulo adicione alguma simetria algbrica til) e calcular todos os outros ngulos possveis (se os ngulos que voc escolheu determinam a figura, possvel calcular todos os outros, de um jeito ou de outro). Procure quadrilteros inscritveis para ajudar. Se necessrio, faa conjecturas ( para isso que voc fez um desenho bem feito!). Alguns problemas de geometria j so resolvidos nesse passo! Se o problema ainda no foi resolvido, hora de elaborar uma estratgia para resolver o problema, ou seja, determinar quais clculos devem ser feitos. Nada de fazer clculos sem planej-los! Execute sua estratgia. Lembre-se sempre de ter uma meta em mente (algo do tipo "precisamos calcular tal ngulo") e, se voc estiver numa prova, de controlar seu tempo e o tamanho da conta (no deixe a conta crescer muito; a falta de controle um fermento muito poderoso para contas.)

(ii)

(iii)

(iv)

claro que esses passos no so precisos e que, para domin-los, preciso muito treino e, por que no, aprender algumas tcnicas.

EUREKA! N17, 2003

17

Sociedade Brasileira de Matemtica

TRIGONOMETRIA

Muitos problemas de geometria podem ser resolvidos com o auxlio da trigonometria. As frmulas que voc deve saber so basicamente essas quatro:

sen(a + b) = sena cos b + senb cos a sen(a b) = sena cos b senb cos a cos(a + b) = cos a cos b senasena cos(a b) = cos a cos b + senasenb
A partir dessas voc pode deduzir essas outras, que na verdade so as mais teis para ns e que tornam a trigonometria to poderosa. Transformando produtos em somas Transformando somas em produtos

1 x+ y x y senasenb = (cos(a b) cos(a + b)) senx + seny = 2sen cos 2 2 2 1 x y x+ y cos a cos b = (cos(a b) + cos(a + b)) senx seny = 2sen cos 2 2 2 1 x+ y x y sena cos b = (sen(a b) + sen(a + b)) cos x + cos y = 2 cos cos 2 2 2
x+ y x y cos x cos y = 2sen sen 2 2 Por fim, relembramos a lei dos senos e a lei dos co-senos. No tringulo ABC, seja AB = c, AC = b, BC = a, A = , B = e C = . O circunraio de ABC R. a b c = = = 2R sen sen sen a 2 = b 2 + c 2 2bc cos b 2 = a 2 + c 2 2 ac cos c 2 = a 2 + b 2 2ab cos A lei dos senos, por envolver propores (que so mais simples) e elementos adicionais do tringulo (o circunraio), particularmente til. Vamos resolver alguns problemas e mostrar algumas tcnicas de clculo.

EUREKA! N17, 2003

18

Sociedade Brasileira de Matemtica

CONVENO

Sempre que houver um tringulo ABC, , e so as medidas dos ngulos BAC, ABC e ACB, respectivamente.
UM COMEO E O TRUQUE DA CO-TANGENTE

Exemplo (Prova de Seleo para a IMO) Seja uma circunferncia de centro O tangente aos lados AB e AC do tringulo ABC nos pontos E e F. A reta perpendicular ao lado BC por O intercepta EF no ponto D. Mostre que A, D e M (ponto mdio de BC) so colineares. Resoluo Primeiro, um bom desenho, com todos os ngulos que pudermos marcar (a tcnica do arrasto bastante til - por isso que voc deve fazer um desenho grande!!). Note que os ngulos do tringulo ABC j determinam os ngulos toda a figura (para perceber isso, note que se construir ABC todos os outros ngulos da figura j esto determinados).
A

2 2 E 2 2 O D 2

2 F

B P M

EUREKA! N17, 2003

19

Sociedade Brasileira de Matemtica

sempre bom justificar os clculos. Seja P a interseo de BC e da reta perpendicular a BC por O. Como BEO e BPO so retos, o quadriltero BPOE inscritvel, de modo que DOE = EBM = . Analogamente, A reta AO bissetriz de e AOEF inscritvel, logo OEF = OFE = / 2. Mas, como provar que A, D e M esto alinhados? Uma maneira provar que BAD = BAM , por exemplo. Para isso, s calcular os dois ngulos. Como calcularemos = BAD ? Veja o tringulo ADE. Sendo r o raio de , com uma lei dos senos calculamos DE. AE pode ser facilmente calculado. Como j conhecemos AED (viu como bom fazer o arrasto?), temos elementos suficientes para calcular . Para calcular = BAM , usaremos o tringulo BAM, da qual conhecemos BM, AB, e ABM . J temos uma estratgia. Vamos executar o plano!

DOF = .

2
E r
No tringulo ODE,

D O

sen + 2 (note que ODE = ( + / 2) - utilizamos o fato de que senx = sen( x) para todo x real; utilizaremos bastante esse fato e o fato sen( / 2 x) = cos x) Sendo o tringulo AEO retngulo em E, obtemos AE = rcotg( / 2).
EUREKA! N17, 2003

DE = sen

DE =

rsen sen + 2

20

Sociedade Brasileira de Matemtica

No tringulo ADE,

DE = sen

AE cos + 2

(*)

Quando temos uma equao do tipo

a b = , senx sen ( x + )
e queremos determinar x, utilizamos o truque da co-tangente:

sen( x + ) b senx cos + sen cos x a b = = = senx sen( x + ) senx senx a b b = cos + sen cotgx = a a
e podemos isolar cotg x. Voltemos a (*). Substituindo DE e AE e utilizando o truque da co-tangente, temos

cotg sen + 2 2 cos cotg + sen = sen 2 2 cos sen + sen 2 sen 2 2 2 cotg = sen sen cos 2 2 2sen + cos 2sen2 sen 2 2 2 cotg = sensen sen( + ) + sen 2sen2 sen 2 cotg = sensen sen( + ) + sen 1 2sen 2 2 cotg = sensen

EUREKA! N17, 2003

21

Sociedade Brasileira de Matemtica

cotg =

sen( + ) + sen cos sensen

Calculemos . Uma prtica normal em trigonometria adotar o circunraio de algum tringulo igual a 1/2, de modo que, pela lei dos senos, seus lados sejam iguais aos senos dos seus respectivos ngulos opostos. Podemos fazer isso porque estamos s fixando o tamanho da figura. claro que s podemos fazer isso uma vez s em cada problema.
A

sen

B sen 2 M

Nesse caso, faamos isso com ABC. Temos

AB = sen = sen( + ). No tringulo ABM, 2sen( + ) BM AB = sen cotg + cos = cotg = sen sen( + ) sen 2sen( + ) sen cos = sensen Puxa, os resultados de cotg e cotg so diferentes! Na verdade, no so. Nunca
perca a f!
EUREKA! N17, 2003

1 BM = BC / 2 = sen e 2

22

Sociedade Brasileira de Matemtica

cotg = cotg sen( + ) + sen cos = 2sen( + ) sen cos sen( + ) = sen cos + sen cos ,
que sempre verdade. ALGUMAS IDENTIDADES Suponha que o circunraio do tringulo ABC R = 1/2. Ento, c = AB = sen, b = AC = sen e a = BC = sen. Alm disso, por exemplo,

O permetro do tringulo 2 p = 4cos cos cos ; 2 2 2 A rea do tringulo S = sen sen sen / 2; O inraio do tringulo r = 2sen sen sen ; 2 2 2

cos + cos + cos = 1 + r / R;

p a = 2cos sen sen . 2 2 2

Exerccio: Prove todas as identidades acima. Exemplo: (IMO) Sejam AH1, BH2 e CH3 as alturas de um tringulo acutngulo ABC. A circunferncia inscrita no tringulo ABC tangente aos lados BC, CA, AB em T1, T2 e T3, respectivamente. Considere a reta simtrica da reta H1H2 relativamente reta T1T2, a reta simtrica da reta H2H3 relativamente reta T2T3, a reta simtrica da reta H1H3 relativamente reta T1T3. Prove que estas retas simtricas determinam um tringulo cujos vrtices pertencem circunferncia inscrita no tringulo ABC. Resoluo: Esse o problema 6 da IMO de 2001. Primeiro, uma boa, e bem grande, figura. Vamos s desenhar a reta simtrica relacionada a T2T3. H o ortocentro de ABC.

EUREKA! N17, 2003

23

Sociedade Brasileira de Matemtica

A
2

2 2

H2
2 2

H3 T3 X3

T2
)1

2 2

B H1

Faamos o arrasto: veja que AH 2 HH 3 inscritvel, logo AH3 H2 = . Seja P a interseo de T2T3 e H2 H3 (s no podemos escolher duas retas TT i j e Hi H j concorrentes quando o tringulo ABC equiltero; tal caso trivial). Como AT2 = AT3 , os ngulos AT2T3 e AT3T2 medem ambos / 2 / 2. Assim,

H3 PT3 = AT3T2 PH3T3 = / 2 e, sendo l1 a reta simtrica da reta H2 H3 relativamente reta T2T3 , o ngulo entre l1 e T2T3 igual tambm a / 2 . Logo o ngulo entre l1 e AB 2( / 2 / 2 ) + = = , ou seja, l1 e BC so paralelos.
Definindo analogamente l2 e l3 , temos l2 // AC e l3 // AB. Com isso, j sabemos que o tringulo determinado por l1 , l2 e l3 semelhante a ABC, e com lados homlogos paralelos. Temos, ento, dois candidatos a tal tringulo:

EUREKA! N17, 2003

24

Sociedade Brasileira de Matemtica


A

A'

C'

B'

B'

C'

A'

Estudando um caso particular (o tringulo equiltero, por exemplo), vemos que o candidato mais indicado o da direita. Podemos, ento calcular a distncia entre lados homlogos nessa situao e compararmos com a distncia entre BC e l1. Assuma que o circunraio de ABC 1/2, para termos BC = sen , CA = sen e

BC = sen.

Vamos calcular a distncia entre BC e l1 . Seja X 3 a interseo de l1 e AB. A distncia de A a l1 AX 3sen . E a distncia desejada AH1 AX 3sen . Bom,

AH1 fcil de calcular: AH1 = ABsen = sen sen . E AX 3 ? AH3 fcil de calcular, AT3 tambm. Podemos calcular H3T3 = AT3 AH3 e usar a lei dos senos no tringulo PH3 X 3 , com a ceviana PT3 . Mos obra!! Para comear, AH3 = AC cos = sen cos e AT3 = p sen , sendo p o semipermetro de ABC. Portanto H3T3 = p sen sen cos . Pela lei dos senos no tringulo PH3T3 , PT3 H 3T3 = sen senH3 PT3 No tringulo PT3 X 3 , PT3 X 3T3 = sen senX 3PT3 Dividindo as duas ltimas equaes e tendo em vista que H3 PT3 = X 3 PT3 , sen psen sen sen sen sen cos obtemos X 3T3 = H3T3 = sen sen
EUREKA! N17, 2003

25

Sociedade Brasileira de Matemtica

Da lei dos co-senos no ABC (ela tambm til de vez em quando!),

sen 2 + sen 2 sen2 2 sen + sen + sen Logo, substituindo p = , 2 sensen + sen sen + sen 2 sen 2 sen 2 + sen 2 X 3T3 = 2sen sensen + sen sen sen 2 + sen 2 = 2sen Enfim, podemos calcular AX 3 = AT3 + X 3T3 . Veja que sen + sen + sen . AT3 = p sen = 2 sen (sen + sen + sen ) sensen + sen sen sen 2 + sen 2 AX 3 = 2sen sensen + 2sen sen sensen + sen 2 = 2sen Enfim, a distncia entre l1 e BC sen sen cos = sensen + 2sen sen sensen + sen 2 AH1 AX 3sen = sen sen 2 sen (sen + sen + sen ) = 2
Na seo de identidades, voc deve provar que

p a = 2cos sen sen 2 2 2 Logo a distncia entre l1 e BC (ufa!) d = 2cos sen sen sen 2 2 2
Agora calculemos a distncia entre os lados homlogos dos tringulos ABC e o de lados respectivamente paralelos aos lados de ABC.

EUREKA! N17, 2003

26

Sociedade Brasileira de Matemtica

C'

B'

I r

B A' C

Seja I o incentro do tringulo ABC. A distncia de I a BC igual ao inraio r e a distncia de I a B'C' r cos. Assim, a distncia entre BC e B'C' :

d ' = r + r cos = r (1 + cos ) = 2r cos 2 2


Voc tem outra identidade para provar:

r = 2sen sen sen 2 2 2


Logo

d ' = 2 2sen sen sen cos 2 2 2 2 2 = 2 2sen cos sen sen cos 2 2 2 2 2 = 2cos sen sen sen = d 2 2 2
EUREKA! N17, 2003

27

Sociedade Brasileira de Matemtica

Conseqentemente, l1 contm B' C'. Analogamente (ou voc acha que eu faria todas as contas de novo?), l2 contm A' C' e l3 contm A'B'. s vezes traar novos elementos na figura tambm ajuda. Exemplo (IMO) Seja P um ponto interior ao tringulo ABC tal que

APC ABC = APB ACB


Sejam D e E os incentros dos tringulos APB e APC, respectivamente. Prove que as retas BD, CE e AP passam por um ponto em comum. Resoluo Seja = APC ABC = APB ACB.
A

1 2

+
P

Veja que podemos "separar" de e . Note que se ficar "para baixo" obtemos um quadriltero inscritvel, ento faremos isso.

EUREKA! N17, 2003

28

Sociedade Brasileira de Matemtica

1 +

1 +

O quadriltero AFPG inscritvel, logo AFG = , ou seja, FG // BC. O problema pede, na verdade, para provarmos que as bissetrizes de ACP e ABP se encontram sobre AP. Sejam Q e R as intersees de BD e CE com AP. Devemos ter Q = R. Do teorema das bissetrizes,

AQ AB AR AC e = = QP BP RP CP
Como

Q = R AQ = AR
suficiente demonstrarmos que

AQ AR AQ AR = = , AP AQ AP AR QP RP AB AC = BP CP

Vamos, ento, calcular BP e CP. Sendo FG paralela a BC, temos FB = k AB. Aplicando a lei dos senos ao tringulo BFP, temos

sen BP FB k ABsen(1 + ) AB = BP = = sen (1 + ) sen sen BP ksen (1 + )

EUREKA! N17, 2003

29

Sociedade Brasileira de Matemtica

Analogamente,

sen AC = CP ksen (2 + )
Como 1 + e 2 + somam , o resultado est demonstrado. GEOMETRIA ANALTICA Quando aparecem problemas com muitos ngulos retos e que envolvam s retas, geometria analtica s vezes indicada. Exemplo (IMO) No quadriltero convexo ABCD, as diagonais AC e BD so perpendiculares e os lados opostos AB e CD no so paralelos. Sabemos que o ponto P, onde se intersectam as mediatrizes de AB e CD, est no interior de ABCD. Prove que ABCD um quadriltero cclico se, e somente se, os tringulos ABP e CDP tm reas iguais. Resoluo Esse problema perfeito para se resolver com geometria analtica: muito fcil colocar as coisas nos eixos (tome como eixos as diagonais); tudo muito fcil de calcular analiticamente (mediatrizes e reas); e , por fim, a nica condio que poderia complicar, que saber quando ABCD cclico, pode ser facilmente transformada na potncia da interseo das diagonais em relao ao seu circuncrculo.
y (0 ; b) B

C (c ; 0)

O P

A (a ; 0) x

(0 ; d) D
EUREKA! N17, 2003

30

Sociedade Brasileira de Matemtica

Sejam, ento, A = (a; 0), B = (0, b), C = (c, 0) e D = (0; d). O quadriltero ABCD inscritvel se, e somente se, OA OC = OB OD ac = bd. Fcil no? Seja P = (x; y). Como P pertence s mediatrizes de AB e CD, temos PA = PB e PC = PD. PA = PB (x a)2 + (y 0)2 = (x 0)2 + (y b)2 2ax a2 = 2by b2 Analogamente, PC = PD 2cx c2 = 2by b2. Resolvendo o sistema obtido, temos

2ax 2by = a 2 b2 2cx 2dy = c2 d 2

x= y=

(a2 b2 )d (c2 d 2 )b 2(ad bc) ( a 2 b2 ) c ( c 2 d 2 ) a 2(ad bc)

Tudo bem com os denominadores pois, como AB e CD no so paralelos, OA / OB OC / OD a / b c / d ad bc 0 (nunca se esquea de verificar quando os denominadores so nulos; essa verificao s vezes faz voc perceber que tem que estudar alguns casos em separado). A rea do tringulo PAB igual a |D|/2 em que

x y 1 D = a 0 1 = ay bx + ab 0 b 1
Da mesma forma, a rea do tringulo PCD igual a D ' / 2, em que

x y 1 D ' = c 0 1 = cy dx + cd 0 d 1
Assim, devemos ter

EUREKA! N17, 2003

31

Sociedade Brasileira de Matemtica

ay bx + ab = cy dx + cd
Seria muito bom nos livrarmos do mdulo. O sinal de D depende da ordem em que colocamos as coordenadas no determinante. Se os pontos correspondentes esto dispostos no sentido anti-horrio, D positivo; se esto no sentido horrio, negativo. Como P pertence ao interior de ABCD, PAB e PCD tm a mesma orientao, de modo que realmente podemos nos livrar do mdulo. Logo, tirando o mdulo e substituindo x e y, temos que as reas de PAB e PCD so iguais se, e somente se,

(a c)

(a2 b2 )c (c2 d 2 )a (a2 b2 )d (c2 d 2 )b + (b d ) = ab cd (**) 2(ad bc) 2(ad bc)

Nada de abrir tudo com pressa! Queremos ac = bd, e isso significa que provavelmente em algum momento fatoraremos a equao com ac bd como um dos fatores. (**)

(a2 b2 )(ac + bd c2 d 2 ) + (c2 d 2 )(ac + bd a2 b2 ) = 2(ab cd )(ad bc) (ac + bd )(a2 + c2 b2 d 2 ) a2c 2 a2 d 2 + b2c2 + b2 d 2 a 2c2 b2c2 + a2 d 2 + b2 d 2 = 2(a2bd ab2c acd 2 + bc2d ) ac(a2 + c2 ) bd (b2 + d 2 ) acb2 acd 2 + bda2 + bdc2 2(a2c2 b2d 2 ) = 2(a2bd ab2c acd 2 + bc2d ) ac(a2 + c2 ) bd (b2 + d 2 ) (acb2 acd 2 + bda2 + bdc2 ) 2(a2c2 b2 d 2 ) = 0 ac(a2 + c2 ) bd (b2 + d 2 ) + ac(b2 + d 2 ) bd (a2 + c2 ) 2(ac bd )(ac + bd ) = 0 (ac bd )((a c)2 + (b d )2 ) = 0 ac = bd ou (a = c e b = d )
No possvel termos a = c e b = d pois j vimos que ad bc . Logo as reas de PAB e PCD so iguais se, e somente se, ac = bd.
EUREKA! N17, 2003

32

Sociedade Brasileira de Matemtica

A geometria analtica tem uma pequena desvantagem: no passa de aplicaes extensivas do teorema de Pitgoras. Apesar de Pitgoras resolver problemas como o que acabamos de ver, mesclar um pouco as contas com trigonometria e nmeros complexos pode vir a calhar. Agora, alguns problemas para voc pensar.
PROBLEMAS

01. Seja ABC um tringulo acutngulo, M o ponto mdio do segmento BC, P o ponto sobre o segmento AM tal PM = BM, H o p da perpendicular de P a BC, Q o ponto de interseo entre o segmento AB e a reta que passa atravs de H e perpendicular a PB e, finalmente, R o ponto de interseo entre o segmento AC e a reta que passa atravs de H e perpendicular a PC. Mostre que o circuncrculo do tringulo QHR tangente a BC no ponto H. 02. No tringulo ABC, AB = AC. D um ponto sobre o lado BC tal que BD = 2 CD. Se P o ponto de AD tal que ABP = PAC, prove que

2DPC = BAC.
03. Um quadriltero convexo est inscrito em uma circunferncia de raio unitrio. Demonstre que a diferena entre seu permetro e a soma das diagonais maior do que zero e menor do que 2. 04. (IMO) O prolongamento da bissetriz AL do tringulo acutngulo ABC intercepta a circunferncia circunscrita no ponto N. A partir do ponto L traam-se perpendiculares LK e LM aos lados AB e AC, respectivamente. Prove que a rea do tringulo ABC igual a rea do quadriltero AKNM. 05. (Ibero) A circunferncia inscrita no tringulo ABC tangente aos lados BC, CA e AB nos pontos D, E e F, respectivamente. AD corta a circunferncia num segundo ponto Q. Demonstrar que a reta EQ passa pelo ponto mdio de AF se, e somente se, AC = BC. 06. (IMO) Seja I o incentro do tringulo ABC. A circunferncia inscrita no tringulo ABC tangente aos lados BC, CA e AB nos pontos K, L e M, respectivamente. A reta que passa por B, paralela ao segmento MK, intercepta as retas LM e LK nos pontos R e S, respectivamente. Prove que o ngulo RIS agudo.
EUREKA! N17, 2003

33

Sociedade Brasileira de Matemtica

07. (Vietn) Seja ABC um tringulo e A', B', C' pontos mdios dos arcos BC, AC e AB do circuncrculo de ABC, respectivamente. As retas A'B' e A'C' interceptam o lado BC em M e N, respectivamente. Defina os pares de pontos P, Q e R, S analogamente. Prove que MN = PQ = RS se, e somente se, ABC equiltero. 08. (IMO) Seja ABC um tringulo acutngulo com circuncentro O. Seja PA uma altura do tringulo com P no lado BC. Considere que BCA ABC + 30. Prove que CAB + COP < 90. 09. (IMO) Num tringulo ABC, seja AP a bissetriz de BAC com P no lado BC, e seja BQ a bissetriz de ABC com Q no lado CA. Sabemos que BAC = 60 e que AB + BP = AQ + QB. Quais so os possveis valores dos ngulos do tringulo ABC? 10. (Coria) Sejam R e r o circunraio e o inraio, respectivamente, do tringulo ABC, e R' e r' o circunraio e o inraio, respectivamente, do tringulo A'B'C'. Prove que se C = C ' e Rr ' = R ' r ento os tringulos so semelhantes. 11. (Turquia) Sejam AC e PC a rea e o permetro, respectivamente, do quadriltero cclico C. Se a rea e o permetro do quadriltero cujos lados so tangentes ao circuncrculo de C so AT e PT , respectivamente, prove que
AC P C AT P T
2

12. (EUA) Seja ABCD um trapzio issceles com AB // CD. O incrculo do tringulo BCD toca CD em E. Seja F um ponto da bissetriz de DAC tal que EF CD. O circuncrculo do tringulo ACF corta a reta CD em C e G. Mostre que o tringulo AFG issceles.

EUREKA! N17, 2003

34

Sociedade Brasileira de Matemtica

13. (Balcnica, adaptado) Seja ABC um tringulo acutngulo e M, N e P as projees ortogonais do baricentro de ABC sobre seus lados. Prove que
2 [MNP] 1 < 9 [ ABC ] 4

([XYZ] a rea do tringulo XYZ) 14. (Ibero) Dados dois crculos 1 e 2 , dizemos que 1 bissecta 2 quando se intersectam e a corda comum um dimetro de 2 . Se 1 e 2 so idnticas, dizemos que 1 e 2 bissectam-se mutuamente. Considere dois crculos fixos e no concntricos 1 e 2 . (a) Mostre que h infinitos crculos que bissectam tanto 1 como 2 . (b) Encontre o lugar geomtrico do centro de . 15. (Ibero) Seja ABC um tringulo acutngulo com circuncrculo centrado em O. Seja AD, BE e CF as alturas de ABC. A reta EF corta em P e Q. (a) Prove que AO PQ. (b) Se M o ponto mdio de BC, prove que AP2 = 2 AD OM 16. (So Petersburgo) Seja AL uma bissetriz interna do tringulo ABC, com L sobre BC. As retas paralelas l1 e l2 passam por B e C, respectivamente, e so equidistantes de A. Os postos M e N pertencem a l1 e l2 , respectivamente, e so tais que os pontos mdios de LM e LN pertencem a AB e AC, respectivamente. Prove que LM = LN. 17. (IMO) No plano, considere uma circunferncia C, uma reta L tangente circunferncia e M um ponto da reta L. Encontre o lugar geomtrico dos pontos P com a seguinte propriedade: existem dois pontos Q, R da reta L tais que M o ponto mdio de QR e C a circunferncia inscrita no tringulo PQR.

EUREKA! N17, 2003

35

Sociedade Brasileira de Matemtica

A ENUMERABILIDADE DE
Nvel Intermedirio

 

E O CHO TRIANGULAR

Jos Paulo Carneiro

Uma das maneiras mais conhecidas de mostrar que o conjunto enumervel, isto , que existe uma bijeo entre e , (onde = {0; 1; 2; } o conjunto dos nmeros naturais) exibir uma bijeo de sobre , inspirada na figura:

<< <

< <<

<< <

(0; 0) (1; 0) (2; 0) (3; 0) a qual sugere a enumerao:

(0; 1) (1; 1) (2; 1) (3; 1)

(0; 2) (1; 2) (2; 2) (3; 2)

(0; 3) (1; 3) (2; 3) (3; 3)

(0; 0); (1; 0); (0; 1); (2; 0); (1; 1); (0; 2); (3; 0); (2; 1); (1; 2); (0; 3); Ou seja, colocamos, sucessivamente, os pares (a; b) tais que a soma a + b assuma os valores 0; 1; 2; 3; , e dentro da cada grupamento que tenha a + b constante (correspondente, na figura, a uma das diagonais indicadas), ordenamos os pares pela ordem natural de sua segunda componente. Obtm-se ento a bijeo: f:

(0; 0) 0 (1; 0) 1 (0; 1) 2 (2; 0) 3 (1; 1) 4 (0; 2) 5 .

<< <

Onde f(x; y) o lugar que ocupa (x; y) nesta enumerao. (Como estamos incluindo 0 em N, preciso comear a contar a partir do 0-simo lugar). Uma questo interessante produzir uma frmula explcita para esta funo e utilizar esta frmula para provar que f realmente uma bijeo.
EUREKA! N17, 2003

36

Sociedade Brasileira de Matemtica

Para isto, seja (x; y) . Observando a figura, v-se que se (x; y) for tal que x + y = s > 0, ento o par (x; y) precedido, pelo menos, por todos os pares (u; v) tais que u + v = 0; 1; 2;;s 1. Existe um par que tem soma 0, dois que tm soma 1, e assim por diante, at s pares que tm soma s 1, de modo que esses pares so em nmero de 1 + ... + s = precedido por y pares. Portanto,

<<

s ( s + 1) . Alm disto, j na sua diagonal, o par (x; y) 2


Finalmente,

f ( x; y ) =

( x + y )( x + y + 1) ( x + y )2 + x + 3 y . +y= 2 2

constata-se diretamente que esta frmula tambm vlida se (x; y) = (0; 0). Podemos ento concluir que f dada pela frmula:

f: f ( x; y ) =

<< <

( x + y) 2 + x + 3 y 2

Pode ser til ao leitor testar esta frmula para pares especficos. Imaginemos agora que seja apresentado ao leitor, sem nenhuma meno a sua origem, o seguinte problema: provar que a funo definida por

( x + y) 2 + x + 3 y uma bijeo de f ( x; y ) = 2

< < sobre < .

Naturalmente, o leitor iniciar tentando provar que f injetiva, mostrando que f(a; b) = f(c; d) implica (a; b) = (c; d). Mas da equao (a + b)2 + a + 3b (c + d )2 + c + 3d no imediato concluir que (a; b) = (c; d), como = 2 2 o leitor pode experimentar. No entanto, se lembrarmos a maneira pela qual criamos a frmula definidora de f, podemos raciocinar do seguinte modo. Suponhamos primeiro que a + b = c + d (ou seja, (a; b) e (c; d) esto na mesma diagonal). Neste caso, f(a; b) = f (c; d) (a + b)2 + a + 3b = (c + d)2 + c + 3d a + b + 2b = c + d + 2d 2b = 2d b = d e, portanto, a = c. Por outro lado; se a + b c + d, podemos supor, sem perda de generalidade, que a + b > c + d. Mas ento (a + b)2 > (c + d )2 e, como f (a; b) = f(c; d), obtm-se (a + b)2 (c + d)2 = 3d + c (3b + a) > 0, mas (a + b)2 (c + d )2 =
EUREKA! N17, 2003

37

Sociedade Brasileira de Matemtica

(a + b c d)(a + b + c + d) a + b + c + d, e logo 3d + c(3b +a) a + b + c +d, donde 2d 2a + 4b, e portanto c + d d a + 2b a + b, absurdo. Logo, no pode ocorrer o caso a + b > c + d, e a concluso final que f injetiva. Provar a sobrejetividade de f, sem apelar para a forma pela qual criamos f, no parece fcil. O que queremos provar que, dado n , existe um par (x; y) tal que f ( x; y ) =

( x + y) + x + 3 y = n. aconselhvel que o leitor primeiro tente 2


2

<

faz-lo, para sentir a dificuldade de calcular x e y em funo de n. Para comear por um exemplo concreto, se n = 50, quem (x; y) = f 1(50)? Ou seja, calcule x e y naturais (e j sabemos, pela injetividade, que sero nicos!) tais que

( x + y )2 + x + 3 y = 50. Se apelarmos para o esquema das diagonais, vamos 2

iniciar perguntando em que diagonal est o 50o. par. Para isto, vamos descobrir primeiro quais nmeros da forma 1; 1 + 2 = 3; 1 + 2 + 3 = 6, etc., precedem 50. Calculando "no brao", vemos que 50 precedido por 1; 3; 6; 10; 15; 21; 28; 36; 45 = 1 + 2 + 3 + 4 + 5 + 6 + 7 + 8 + 9 (o prximo j seria 55). Isto significa que f 1(50) precedido pelos 45 pares (u; v) que tm somas u + v = 0; 1; 2;;8, sendo o ltimo deles igual a (0; 8) = f 1(44) (no esquea de contar a partir do 0). A diagonal seguinte comea com o par (9; 0) = f 1(45). Para chegar ao 50, precisamos andar mais cinco pares sobre esta diagonal: (8; 1); (7; 2); (6; 3); (5; 4); (4; 5), chegando at o (4; 5) = f 1(50). Este par o 50o. da seqncia (nunca esquecendo de contar a partir do 0) e, portanto, o nico que satisfaz a f(x; y) = 50. Para abordar o caso geral, vamos primeiro lembrar que os nmeros T1 = 1; T2 = 1 + 2 = 3, T3 = 1 + 2 + 3,, Tk = 1 + ... + k =

triangulares (ver [1]). Dado n (vamos considerar somente n > 0, j que f 1(0) = (0; 0) imediato), para determinar (x; y) = f 1(n) pelo mtodo usado acima, necessrio primeiro determinar o maior nmero triangular Tk tal que Tk n , que vamos chamar de cho triangular de n. Por exemplo, como vimos, o cho triangular de 50 45. A existncia do cho triangular de n, no caso geral, decorre do seguinte: a seqncia T1; ; Tm ; ilimitada superiormente; logo, algum Tm maior do que n; pelo Princpio da Boa Ordenao dos nmeros naturais (ver [2]), seja Tj o menor dos Tm que so maiores do que n; ento Tj1 o cho triangular de n; de fato, T j 1 n < T j .
EUREKA! N17, 2003

k (k + 1) , so chamados nmeros 2

38

Sociedade Brasileira de Matemtica

Para mostrar a unicidade do cho triangular de n, suponha (por absurdo) que Tp n < Tp +1 e Tq n < Tq +1 com p < q (sem perda de generalidade). Como a seqncia (Tm) estritamente crescente, n < Tp +1 Tq n, absurdo. Uma vez ento determinado Tk, o cho triangular de n, o par (x; y) = f 1(n) estar na diagonal de soma k, formada pelos pares (k; 0); (k 1; 1);;(0; k). Nesta diagonal, f(k; 0) = Tk; f(k 1; 1) = Tk + 1;; f(0; k) = Tk + k = Tk + 1 1, ou seja, de um modo geral: f(k j; j) = Tk + j. Quando j = n Tk, termos f (k n + Tk ; n Tk ) = n . Logo: (x; y) = f 1 (n) = (k n + Tk ; n Tk ). Por exemplo:
1 f (50) = (9 50 + 45;50 45) = (4;5).

Como conseqncia deste raciocnio, fica claro que f injetiva e sobrejetiva e que se pode determinar de modo nico f 1(n) para cada n pela frmula:

0; f 1 (n) = (k n + Tk ; n Tk );
onde Tk o cho triangular de n.

se se

n=0 n>0

Mas permanece um problema mais sutil; existe alguma frmula fechada que calcule k tal que Tk seja o cho triangular de um dado n? Para responder a isto, observe que a soluo positiva de

x(x +1) =n 2

8n +1 1 x(x +1) e que a funo f (x) = crescente em [0; +[. Sendo, pois 2 2 k = x0 (onde x0 simboliza o cho inteiro de x0 , ou a parte inteira de x0 ), (k +1)(k + 2) k (k +1) x0 (x0 +1) temos k x0 < k +1 e, portanto: Tk = =n< = Tk +1. 2 2 2 x0 =

EUREKA! N17, 2003

39

Sociedade Brasileira de Matemtica

Concluso: uma expresso explcita para k tal que Tk seja o cho triangular de n :

8n +1 1 k = . 2
Por exemplo, para n = 50, temos: k =

401 1 9,51... = 9. = 2

Referncias:
[1] Carneiro, J.P.; Contar de duas maneiras, para generalizar; Eureka! No. 6; pp. 15-17. [2] Carneiro, J.P; O Princpio da Descida Infinita de Fermat; Revista do Professor de Matemtica, No. 32; 3o. quadrimestre de 1996; pp. 39-44.

EUREKA! N17, 2003

40

Sociedade Brasileira de Matemtica

COMO QUE FAZ?


PROBLEMA 1 PROPOSTO POR ASDRUBAL SANTOS (BOTUCATU SP)

Encontre todas as funes f : i) ii)

tais que:

f ( x) 0, x

+ +.

f ( f (x) x) = 2x, para todo x

SOLUO DE GUILHERME RODRIGUES NOGUEIRA DE SOUZA (SO PAULO - SP)

Seja a funo g : Logo:

+ tal

que g( x) = f ( x) x, para todo x

f (g(x)) = 2x g(x) + g(g(x)) = 2x g(g(x)) = g(x) + 2x (I)


n n

Seja: g 0 ( x) = x, g +1 ( x) = g ( g ( x)), para todo n . Logo, por (I):


( n) ( n) ( n) g( g( g ( x))) = g( g ( x)) + 2g ( x)

<

g (n+2) ( x) = g (n+1) ( x) + 2g (n) ( x)


Seja an = g(n) (x), com x fixo. Ento chegamos na seguinte recurso:

an+2 = an+1 + 2an


Resolvendo essa recurso chegamos em:
n g(x) + 2x n g(x) x an = 1 ( 2) 3 3 n g(x) + 2x g(x) x g(n) (x) = ( 2) 3 3

Para n suficientemente grande temos

g(x) + 2x n g(x) x <2 , desde que g( x) x. 3 3 Suponha, por absurdo que g(x) > x para algum x
EUREKA! N17, 2003

41

Sociedade Brasileira de Matemtica

Tome n par. Temos que g (n) ( x) negativo se n suficientemente grande, o que absurdo, pois g de + + . Suponha agora que g( x) < x. Tambm por absurdo, tomando n mpar, conclumos que g (n) ( x) negativo, e como j vimos antes isso um absurdo. Portanto, para todo x positivo, g(x) > x e g(x) < x so falsos. Logo g(x) = x, para todo x positivo. Ento f(x) x = x f(x) = 2x Vamos testar f(x) = 2x. Temos: f ( f (x) x) = 2x f (2x x) = 2x f (x) = 2x 2x = 2x, ento a nica funo que satisfaz o enunciado f (x) = 2x.
PROBLEMA 2 PROPOSTO POR ASDRUBAL SANTOS (BOTUCATU SP)

Mostre que no existem funes f :

tais que f ( f ( x)) = x2 2, x .

SOLUO DE TELMO LUIS CORREA JNIOR (SO PAULO SP)

Vamos determinar os possveis valores de x para que f ( f (x)) = x: x = x2 2 x = 2 ou x = 1. Agora, os valores tais que f ( f ( f ( f (x)))) = x :

f ( f ( x2 2)) = x ( x2 2)2 2 = x

x4 4x2 x + 2 = 0
Mas 2 e 1 devem ser razes dessa equao, pois f ( f (x)) = x f ( f ( f ( f (x)))) = x . Fatorando, obtemos:

( x 2)( x +1)( x2 + x 1) = 0

1+ 5 1 5 x x (x 2)(x +1) 2 2 =0
Logo os nicos valores tais que f ( f ( f ( f (x)))) = x so 2, 1, Considere os valores x, f (x), f ( f (x)) e f ( f ( f (x))).

1+ 5 1 5 e . 2 2

EUREKA! N17, 2003

42

Sociedade Brasileira de Matemtica

f(x) f f

Para x =

1+ 5 , f ( f ( f ( f (x)))) = x. 2

Considere os possveis valores de f(x): como f ( f ( f ( f ( f (x))))) = f (x), f (x) pode ser

f(f (x)) f f f(f (f (x))

1 5 1+ 5 , . 2 2 1+ 5 De fato, se f (x) = , ento f (x) = x , 2 logo f ( f (x)) = f (x). Absurdo, pois temos
apenas 2, 1,

1+ 5 1+ 5 1+ 5 = 2 . ff 2 2 2
Se f(x) = 2, como f (f (2)) = 2, ento f( f ( f (x))) = 2,
2 f
1 + 5 2

logo f(2) =

1+ 5 , absurdo. 2

f (2) f 2 f

Se f(x) = 1, de modo anlogo ao caso f(x) = 2 chegamos a um absurdo.

1 5 , os possveis valores para f( f(x)) 2 1+ 5 1 5 so , 2 ou 1. , 2 2


Se f(x) = Se f (f (x)) =

f
1 + 5 2

1 5 2

f (f (x)) f f (f (f (x))) f

1 + 5 , ento x = f (f (x)), absurdo. 2 1 5 1 5 1 5 Se f (f (x)) = , ento f 2 = 2 , 2

EUREKA! N17, 2003

43

Sociedade Brasileira de Matemtica

Como em qualquer caso f(f(x)) = x2 2 um absurdo, no existe funo f nessas condies.


PROBLEMA 3 PROPOSTO POR DAVI MXIMO ALEXANDRINO NOGUEIRA (FORTALEZA - CE)

1 5 1 5 , absurdo. 2 )) = 2 1 + 5 Se f ( f(x)) = 2, f ( f (f ( f (x)))) = 2 2 = , absurdo. 2 1 + 5 Se f ( f(x)) = 1, f ( f ( f ( f (x)))) = 1 1 = , absurdo. 2


Logo

1 + 5 = f(f(f 2

Prove que possvel decompor o conjunto {1, 2, 3,,2n} em dois subconjuntos A e B no contendo nenhuma progresso aritmtica de tamanho 2n.
SOLUO:

Vamos escolher o conjunto A aleatoriamente: Cada elemento de U = {1, 2,,2n} colocado em A (ou em B = U \ A) com probabilidade 1/2, de forma independente. Agora, se fixarmos uma PA com 2n termos, a probabilidade de ela estar contida em A ou em B 2

1 1 = 2n1 . Por 2n 2 2

outro lado, podemos estimar o nmero de progresses aritmticas de tamanho 2n

2n possibilidades para a razo e 2n 1 22 n n no mximo 2 possibilidades para o termo inicial, e logo temos no mximo 2n 1
em U da seguinte forma: temos no mximo tais PA's. Como, para cada uma dessas PA's, a probabilidade de ela estar contida em A ou em B

1 2
2 n 1

, a probabilidade de alguma dessas PA's estar contida em A ou em B

, no mximo,

(de fato a grande maioria, se n grande) de decomposies U = A B onde nem A nem B contm progresses aritmticas de tamanho 2n (para n = 1 podemos tomar o exemplo A ={1} e B ={2}).

22 n 1 2 2n 1 = < 1 , para todo n 2, donde h exemplos 2n 1 2 2n 1

EUREKA! N17, 2003

44

Sociedade Brasileira de Matemtica

SOLUES DE PROBLEMAS PROPOSTOS


Publicamos aqui algumas das respostas enviadas por nossos leitores.

68. Seja ABC um tringulo de lados inteiros e rea racional. Prove que existem pontos X, Y, Z com coordenadas inteiras no plano #2 tais que o tringulo XYZ congruente ao tringulo ABC.
SOLUO DE ZOROASTRO AZAMBUJA NETO (RIO DE JANEIRO RJ)

Sejam a, b, c as medidas dos lados do tringulo, respectivamente opostos aos vrtices A, B e C. Podemos supor que A = (0,0) , B = (c ,0) e C = ( m, h) . Temos que c inteiro e m2 + h2 = b2 ,( m c)2 + h2 = a 2 , com a e b inteiros. Da obtemos b2 a2 + c2 4b 2 c 2 (b 2 a 2 + c 2 ) 2 ch e logo h2 = . Como a rea de ABC 2 2c 4c 2 racional, segue que h racional, e logo 4b 2 c 2 (b 2 a 2 + c 2 ) 2 um quadrado m= perfeito. Isso implica que b 2 a 2 + c 2 par (seno 4b 2 c 2 (b 2 a 2 + c 2 ) 2 seria congruente a 3 mdulo 4, absurdo). r s Assim, m = e h= com r, s inteiros, e portanto, r 2 + s 2 = b2 c 2 (pois c c m 2 + h 2 = b 2 ). Vamos agora mostrar que existem x, y , u , v tais que x 2 + y 2 = c 2 , u 2 +v 2 = b 2 e ( x + yi)(u + vi) = r + si. Isso resolve o problema, pois o nmero complexo x y = i tem mdulo 1, e portanto a multiplicao por uma rotao em 2 c c ( x yi) ( r + si) = que leva A, B, e C em X = (0,0), Y = ( x, y ) e Z = (m + hi) = c c

( x yi)( x + yi )(u + vi ) = u + vi = (u , v ). Assim, XYZ congruente a ABC e c2

seus vrtices tm coordenadas inteiras. Vamos mostrar a existncia dos x, y, u, v: queremos mostrar que se N (r + si) = r 2 + s 2 = b2 c 2 , com b, c, r, s ento existem x, y, u, v com

( x + yi )(u + vi ) = r + si, N ( x + yi) = x 2 + y 2 = c 2 e N (u + vi ) = u 2 + v 2 = b 2 . Vamos provar isso por induo em N (r + si) (se N (r + si ) = 1 o resultado bvio), usando existncia e unicidade de fatorao em [i ] = {a + bi , a , b }
(vejam o artigo de Guilherme Fujiwara na Eureka! No. 14).
EUREKA! N17, 2003

45

Sociedade Brasileira de Matemtica

Se c = 1, tomamos x = 1, y = 0, u = r e v = s. Se c > 1, tome um fator irredutvel de c. Temos que b 2 c 2 = (r + si )(r si ), e logo r + si ou r si (e, nesse caso r + si ). Suponha ento, sem perda de generalidade, que r + si . Temos ento r + si r + si ( r + si ) ( r si ) [i] e N . Se N ( ) um = quadrado perfeito (o que nesse caso equivale a ser o produto de um elemento de , y ,u,v por um elemento de {1, i}), existem, por hiptese de induo, x
2 r + si 2 + y  2 = c ,u2 + v2 = b2 , e basta tomar (x + yi) = (x  + yi )  + yi  )(u +vi), x = (x N() para concluir. Caso contrrio, como | c, | c, e logo | c (pois irredutvel e  [i]). Assim, 2 2 | c 2 | b 2 c 2 = (r + si )(r si ), e portanto | r + si ou c2 r + si N ( r + si ) 2 = = , e podemos b 2 | r + si. No primeiro caso, N 2 N ( ) N ( ) 2 r + si 2 + y  2 = c 2 , u 2 + v2 = b2 , e basta  + yi  )(u + vi) com x = (x N ( )  + yi  ). x + yi = ( x O segundo caso anlogo:

com

escrever tomar

c2 r + si N ( r + si) 2 , N 2 = b = 2 N ( 2 ) N ( )

podemos

escrever

c2 r + si 2 2     + = = + + ( )( ) , u 2 + v 2 = b 2 , e basta tomar x y x yi u vi com N ( ) 2 2


 + yi  ). x + yi = 2 ( x

69. Sejam a e b inteiros positivos tais que a n 1 divide b n 1 para todo inteiro positivo n. Prove que existe k  tal que b = a k .
SOLUO DE ZOROASTRO AZAMBUJA NETO (RIO DE JANEIRO RJ)

Suponha por absurdo que b no seja uma potncia de a. Ento existe k  tal que a k < b < a k +1 . Consideremos a seqncia bn 1 1 1 1 1 xn = n = + + = Como ... , temos , n 1. n n jn 2n a 1 a a a 1 j =1 a
EUREKA! N17, 2003

46

Sociedade Brasileira de Matemtica

xn =

1 b b 1 bn bn b = + + + n . ... 2 k jn jn kn n ( 1) a 1 a a a a a j =1 a j =1 a n

Note que

b a k +1 ) ( bn = como kn n 1 a n a (a 1)
n n

e
n

1 tendem a 0 quando n cresce, se definimos a 1


n n

k 1 bn b b b b n yn = + 2 + ... + k = j , temos que xn yn = kn n a ( a 1) a 1 a a a j =1 a tende a 0 quando n tende a infinito. Por outro lado, como yn uma soma de k

progresses geomtricas de razes b a j , 1 j k , yn satisfaz a equao de recorrncia C 0 y n + k + C 1 y n + k 1 + ... + C k y n = 0, n 0 , onde b b b C0 x k + C1 x k 1 + ... + Ck 1 x + Ck = a k ( k +1) / 2 x x 2 ... x k a a a (vejam o artigo "Equaes de recorrncia" na Eureka! No. 9). so inteiros. Note tambm que Note que todos os Ci + + + = + + + C0 xn + k C1 xn + k 1 ... Ck xn C0 ( xn + k yn + k ) C1 ( xn + k 1 yn + k 1 ) ... Ck ( xn yn ) tende a 0 quando n tende a infinito, pois xn + j yn + j tende a 0 para todo j com 0 j k (e k est fixo). Como os Ci e os xn so todos inteiros, isso mostra que C0 xn + k + C1 xn + k 1 + ... + Ck xn = 0 para todo n grande. 1 bn b n , temos Agora, como x n = y n + k +1 + ( k +1) n n ( a 1) a 1 a a b n+k j + z n + k j , onde C 0 xn + k + C1 x n + k 1 + ... + C k xn = C j k +1 j=0 a 1 bm m . zm = (k +1)m m ( a 1) a 1 a
k n

Note que

b C k k +1 a j=0
k

n+k j

b b = P k + 1 k + 1 , onde a a

b b b P( x) = C0 x k + C1 x k 1 + ... + Ck 1 x + Ck = a k ( k +1) / 2 x x 2 ... x k , donde a a a b P k +1 0. Por outro lado, para todo j com a

EUREKA! N17, 2003

47

Sociedade Brasileira de Matemtica


n ( b a k +1 ) 1 b , que tende a 0 = k +1 n+ k j k j n k n 1 a a a a b a ( )( ) k j

0 j k , z n+ k j

quando n tende a

infinito,

donde

k wn = C j xn + k j j =0

b k +1 tende a a

b P k +1 0 , o que um absurdo, pois, como vimos antes, wn igual a 0 para a todo n grande. 71. Considere trs circunferncias, tangentes duas a duas. Prove que h apenas duas circunferncias tangentes s trs simultaneamente, e mostre como construlas.
SOLUO DE ANTONIO CAMINHA MUNIZ NETO (FORTALEZA CE)

Prova Analisemos o caso em que as trs circunferncias, , , digamos, so tangentes exteriormente duas a duas (a anlise dos demais casos completamente anloga).

Seja A o ponto de tangncia das circunferncias e . Aplicando configurao mostrada na figura acima a inverso I de plo A e mdulo igual potncia de A em relao a , a circunferncia permanece fixa, ao passo que as circunferncias e so respectivamente transformadas em retas distintas r e s, perpendiculares reta que une seus centros. Como inverses preservam tangncia de curvas, segue que r e s so ainda tangentes a , de modo que a situao exatamente a da figura abaixo (observe que r e s podem ser construdas com rgua
EUREKA! N17, 2003

48

Sociedade Brasileira de Matemtica

e compasso: elas so as tangentes a que so perpendiculares reta dos centros de e ):

Seja uma circunferncia qualquer que tangncia , e . claro que no contm A, donde conclumos que sua inversa ainda uma circunferncia, ademais tangente s inversas de , e , quer dizer, tangente a , r e s. Portanto, segue imediatamente da figura acima que s h duas possibilidades para , digamos 1 e 2. Tais circunferncias podem ser facilmente construdas com rgua e compasso, basta para tanto observar que, sendo R o raio e O o centro de , os centros de 1 e 2 so os pontos situados sobre a paralela a r por O, distando 2R de O. Portanto, h exatamente duas circunferncias tangentes simultaneamente , e e tais circunferncias so as inversas 1 e 2 de 1 e 2 por I (as quais tambm podem ser facilmente construdas). 73. Prove que, dado um inteiro positivo n, existe uma progresso aritmtica crescente formada por n inteiros positivos cujas somas dos dgitos tambm formam uma progresso aritmtica crescente, mas no existe uma progresso aritmtica infinita de inteiros positivos cujas somas dos dgitos formem uma progresso aritmtica crescente.
SOLUO DE JOS DE ALMEIDA PANTERA (RIO DE JANEIRO RJ)

Vamos mostrar, por induo em k, que para quaisquer inteiros positivos k 2 e N e todo > 0 existe uma progresso aritmtica crescente de inteiros positivos ( x j ) N j k + N tal que se s ( x j ) a soma dos dgitos de x j ento ( s ( x j )) N j 1 constante, ( s ( x j ))1 j k uma progresso aritmtica crescente de razo r e
EUREKA! N17, 2003

49

Sociedade Brasileira de Matemtica

s ( xk +i ) s ( xk +i 1 ) < r para 1 i N . De fato, para k = 2, podemos tomar x1 = 102 m 10m , x2 x1 = 10m 1, onde m grande o suficiente para que m N (e 1 < . logo 10 m > N ) e m De fato, s ( xi ) = 9m , para 2 10 m i 1 [pois, para 1 a 10 m , a (10 m 1) =

(a 1) 10m +10m 1 (a 1), donde s (a (10m 1)) = s (a 1) + 9m s (a 1) = 9m) , s (a (10m 1)) = s ( x2 ) = 18m e, para j > 2, digamos j = 2 + a, x j = a 10m + 102 m 1 a = 10 2 m + ( a 1) 10m + 10m 1 a , e, como
1 + a N < 10 m , s( x j +1 ) s( x j ) = 1 + s(a) + 9m s(a + 1) (1 + s(a 1) + 9m s(a)) = = 2s(a) s(a 1) s(a + 1) < 9 m < (9m). Vamos agora conseguir uma tal progresso aritmtica com k = 1 elementos. Para isso, seja ( x j ) N 1 j k + N +1 uma progresso como acima associada a k , N + 1, . 3 1 < . Seja ) um inteiro tal que 10 ) > x k + N + 1 . Podemos supor que N k e N 2 y j = x j + 10 ) t j k + 1 , Considere a seqncia ( y j ) N j k + N +1 , onde

t s = 10 2 m 1 + s (10 m 1)

r ( s ( xk +1 ) s ( xk )) (1 ) N , onde 9 r = s( x2 ) s( x1 ) (note que m inteiro pois s ( x j ) x j (mod 9) para todo j). e m= s ( y j ) = s ( x j ) + s (t j k 1 ). Temos ento ( s ( y j )) N j 1 constante e s ( y j +1 ) s ( y j ) = 9m , para 1 j k . De fato, para 1 j k 1, isso segue do fato

Assim,

anlogo para ( x j )1 j k , e de ( s (t j k 1 ))1 j k ser constante, e para j = k, s (t j k ) s (t j k 1 ) = 9m = r ( xk +1 xk ) , donde s ( yk +1 ) s ( yk ) = r = s ( x2 ) s ( x1 ). O fato de termos s ( yk +1+i ) s ( yk +i ) < r para 1 i N segue dos fatos anlogos para s ( xk +i +1 ) e s ( yi ). Suponha agora que exista uma progresso aritmtica infinita de inteiros positivos ( x j ) j 1 cujas somas dos dgitos ( s ( x j ) j 1 ) formam uma progresso aritmtica crescente. Tome um inteiro positivo k tal que 10 k > x2 > x1 , e note que s ( x10k +1 ) = s ( x1 + 10k ( x2 x1 )) = s ( x1 ) + s ( x2 x1 ) = s ( x1 + 10 k +1 ( x2 x1 )) = s ( x10k +1 +1 ) , absurdo.

EUREKA! N17, 2003

50

Sociedade Brasileira de Matemtica

74. Ache todas as funes f : # # tais que: f ( x + y ) + f ( x y ) = 2 f ( x) cos y, x, y #.


SOLUO DE MARCLIO MIRANDA DE CARVALHO (TERESINA PI)

e x = + t f ( + t ) + f (t ) = 0 (I) 2 2 Faa x = 0 e y = t f(t) + f(t) = 2a cos t, onde a = f(0). (II) (III) Faa x = e y = + t f ( + t ) + f (t ) = 2b cos + t = 2bsent , 2 2 2 onde b = f . 2 Somando (I) com (II) e subtraindo de (III) chegamos a f(t) = a cost + b sent, t #. fcil verificar, apenas substituindo, que todas as funes f(t) desta forma funcionam.
Faa y = 75. Seja Tn um tringulo retngulo cujos lados medem (4 n 2 , 4 n 4 1, 4 n 4 + 1) , onde n um nmero inteiro positivo. Seja n a medida do ngulo oposto ao lado de medida 4 n 2 . Mostre que, se n varia dentro dos inteiros positivos, 1 + 2 + 3 + ... = 90 0 .
SOLUO DE ANTONIO CAMINHA MUNIZ NETO (FORTALEZA CE)

Seja Tn um tringulo retngulo cujos lados medem 4n2, 4n4 1 e 4n4 + 1, onde n um nmero inteiro positivo. Seja n a medida, em graus, do ngulo oposto ao lado de medida 4n2. Mostre que 1 + 2 + 3 + .... = 90.

4n 2 . Portanto, sendo n = 2n temos Soluo: Sabemos que tg n = 4n 4 1 2tg n 4n 2 . = 1 tg 2 n 4n 4 1 1 para todo n 1. Resolvendo a equao acima para tg n obtemos tg n = 2n 2

Assim, basta provar que 1 + 2 + 3 + .... = 45. Sendo bn = tg (1 + 2 + ... + n) para n 1, temos ento de provar que bn 1 quando n + .

EUREKA! N17, 2003

51

Sociedade Brasileira de Matemtica

Temos b1 = tg 1 =

1 e, pela frmula para a tangente da soma, 2


1 bn + tg n +1 bn + 2(n +1)2 2(n + 1) 2 bn + 1 . = = 2 n 1 bn tg n +1 1 2( nb+ 2( 1) + n b 2 n 1)

bn +1 =

Provemos, por induo sobre k 1, que (*) bk =

k , o que terminar a k +1

demonstrao. A relao (*) trivialmente verdadeira para k = 1. Suponha que j provamos que ela verdadeira para 1 k n, onde n 1 um inteiro. Ento

bn+1 = =

2(n + 1) 2 (nn +1 ) + 1 2(n + 1) (


2 n n +1

[2(n + 1)n + 1](n + 1) 2(n + 1) 3 n

(2n 2 + 2n + 1)(n + 1) n + 1 . = (2n 2 + 2n + 1)(n + 2) n + 2

76. Mostre que um polgono qualquer pode ser recortado e os recortes reorganizados, sem superposio, de tal jeito que formem um quadrado.
SOLUO DE ANDERSON TORRES (SO PAULO SP)

Este problema to legal que d para generalizar! Vamos demonstrar que se dois polgonos tm mesma rea podemos fatiar um deles e reorganizar as fatias de modo a produzir um polgono congruente ao segundo. Lema 1: Um retngulo pode ser convertido em um novo retngulo do qual conhecemos um dos lados (por exemplo, qualquer retngulo de rea x pode ser decomposto para formar um retngulo em que um dos lados mede 1 (e o outro medir x neste caso). Prova: Basta exibir uma maneira de fazer isto. Veja este exemplo:
D 1 I K 3 C H N 3' 2' J M 1' A
EUREKA! N17, 2003

52

Sociedade Brasileira de Matemtica

1) Faa com que as bases dos retngulos estejam alinhadas. Prolongue o lado HG at cortar o outro retngulo no ponto I. 2) Trace DJ//IB. Seja KI //AB e K DJ. Agora s deslizar o tringulo DCJ at DC se alinhar com IK. E depois encaixa DIK no espao abaixo, em MLF. E acabou! Esta construo s vale se 2.HE DI. Caso no seja possvel, basta fatiar da maneira mais primitiva: fatia em partes iguais at cair no caso anterior: Neste caso nem precisou!

Agora s escrever. Dado um polgono qualquer, recortamos o desdito em tringulos quaisquer. Cada um deles pode ser fatiado para virar um paralelogramo (parte pela base mdia) e cada paralelogramo pode ser fatiado nun retngulo, s cortar em uma altura. Podemos transformar cada um destes retngulos em fatias de lado 1, empilh-los pelo lado comum e transformar isto tudo num quadrado (oras, um quadrado um retngulo equiltero) E fim! 77. Prove que as distncias entre um ponto sobre uma circunferncia e os quatro vrtices de um quadrado nesta inscrita no podem ser todos nmeros racionais.
SOLUO DE CLAUDIO BUFFARA (SO PAULO SP)

Seja o quadrado inscrito ABCD de lado L e suponhamos, sem perda de generalidade, que o ponto P se encontra no arco AB. Se P coincide com B, ento m(PA) = m(PC) = L e m(PD) = L 2 . Assim, se L racional, ento m(PD) irracional e se L irracional, ento m(PA) = m(PC) irracional. De forma anloga pode-se provar que se P coincide com A, ento m(PB), m(PD) ou m(PD) ser irracional. Suponhamos agora que P no coincide com A nem com B e que m(PA), m(PB), m(PC) e m(PD) sejam todos racionais. O quadriltero PBCD inscritvel. Assim, pelo Teorema de Ptolomeu, teremos: m(PD) m(BC) + m(PB) m(CD) = m(BD) m(PC).
EUREKA! N17, 2003

53

Sociedade Brasileira de Matemtica

Mas m(BC) = m(CD) = L e m(BD) = L 2 . Portanto: m(PD) + m(PB) = m(PC) 2 . Mas: m(PD) e m(PB) so racionais m(PD) + m(PB) racional e m(PC) racional m(PC) 2 irracional

Ou seja, um nmero racional igual a um nmero irracional. Temos, portanto, uma contradio, a qual ocorreu em virtude da hiptese feita inicialmente de serem m(PA), m(PB), m(PC), m(PD) todos racionais. Concluso: pelo menos um destes quatro segmentos tem de ter comprimento irracional. Observao: se tomarmos o quadriltero inscritvel PADB, o Teorema de Ptolomeu nos dar: m(PA) m(BD) + m(PB) m(AD) = m(PD) m(AB), ou seja: m(PA) 2 + m(PB) = m(PD), e mais uma vez cairemos na contradio de ter um nmero racional igual a um nmero irracional.

78. Seja ABCD um quadriltero convexo no trapzio, de diagonais AC e BD iguais. Tomamos sobre os lados AB e CD, respectivamente, pontos P e Q tais que: AP DQ AD = = PB QC BC Mostre que os pontos P e Q so colinares com o ponto de interseo das mediatrizes dos lados AD e BC.
SOLUO DE MARCOS FRANCISCO FERREIRA MARTINELLI (RIO DE JANEIRO RJ)

Considere o quadriltero ABCD AC = BD, onde O o ponto de encontro das diagonais e O' o ponto de encontro das mediatrizes de AD e BC.

EUREKA! N17, 2003

54

Sociedade Brasileira de Matemtica


B P' A

O'
D

Provarei que os quadrilteros AOO'D e BOO'C so inscritveis (i). ' = ' . Como DO' = O'A DAO Seja = ADO Observe que os ACO ' e BDO ' so congruentes (L.L.L), mais uma vez porque O ' B = O ' C e, do enunciado, AC = BD. = ODO ' i ) est provado. ' e DBO ' = ACO O ' AO ' = DAO ' = Como o quadriltero AOO'D inscritvel, DOO = 180 O ' OA = 180 (180 ) = E ainda, O ' OC = O ' BC (ii) = = O ' CB Como o quadriltero BOO'C inscritvel , O ' OC De ii), observe que o AO ' D
O ' BC AO ' BO ' AD AO ' (iii) = = AD BC BC BO ' DQ AD Seja Q CD, , bem como P' a interseo de QO' com AB. Se = QC BC AP ' AD , teremos P ' P. provarmos que = P ' B BC DQ AD DQ AO ' DO ' Como, por hiptese, , e conclui-se e de (iii), temos = = = QC BO ' CO ' QC BC que O'Q bissetriz do DO ' C.

EUREKA! N17, 2003

55

Sociedade Brasileira de Matemtica

'Q = Q 'O ' C = AO ' P ' = 180 ( + 180 2 ) = 2 e Seja DO P ' O ' B = 180 ( + 180 2 ) = 2 AP ' AO ' AD = = est provado que P ' P. O ' P ' bissetriz do AO ' B P ' B O ' B BC Portanto, P, O'e Q esto sobre uma mesma reta. (c.q.d.) 79. Temos uma fileira infinita de copos, cada um deles associado a um inteiro k, e um nmero finito de pedras distribudas de alguma maneira por esses copos. Se h pelo menos duas pedras no copo k podemos pular uma pedra para o copo k 1 e outra para o copo k + 1. Prove que fazendo movimentos desse tipo um nmero suficientemente grande de vezes, chega-se necessariamente a uma situao onde no possvel fazer nenhum movimento desse tipo (i.e., onde h no mximo uma pedra em cada copo), e que a configurao final no depende da escolha dos movimentos durante o processo.
SOLUO:

Sejam x1 ,..., xn as posies das n pedras. O nmero de pedras sempre n, e em um movimento trocamos duas pedras na posio k (digamos com xi = x j = k ) por xi = k 1 e x j = k + 1. Temos ento que

x
i =1

permanece constante e

x
i =1

2 i

aumenta a cada movimento, pois

( k + 1)2 + ( k 1)2 = 2k 2 + 2 > 2k 2 . Seja

agora m o maior nmero de copos vazios entre dois copos ocupados. Ento, se m 0 , m no aumenta em nenhum movimento e, se m = 0, aps um movimento m passa a ser no mximo 1. Assim, a distncia entre dois copos ocupados 1 n consecutivos fica limitada, e como o centro de gravidade xi das pedras n i =1 constante, a "energia"

x
i =1

2 i

tambm fica limitada, e como sempre aumenta, em

algum momento no ser mais possvel fazer nenhum movimento. O nmero de movimentos limitado por f ( x1 , ..., x n ) =
2i n

)2

j 2 , onde

j =1

) = m ax{2, r }

e,

se x1 x 2 ... x n , r = max ( x i x i 1 ) + 1. Para cada

x 1 , . . . , x n , seja

g ( x1 ,..., xn ) f ( x1 ,..., xn ) o nmero mximo possvel de movimentos a partir da posio inicial x1 , x2 ,..., xn .
EUREKA! N17, 2003

56

Sociedade Brasileira de Matemtica

aps esses dois lances, em qualquer ordem, em xi = k 1, x j = k + 1, xr = ) 1 e xs = ) + 1 ) , donde as posies finais so iguais, absurdo. 1+ 5 , A ={ n , n  *} e B = 2 A B = e A B = *.

Suponha agora que exista uma posio inicial a partir da qual seja possvel chegar a duas posies finais distintas. Seja x1 ,..., xn uma tal posio com g ( x1 ,..., xn ) mnimo. Esses dois jeitos de chegar em posies finais diferentes no podem comear com o mesmo movimento, pois seno, aps esse movimento, o valor de g diminui, e a posio final passa a ser nica pela minimalidade de g ( x1 ,..., xn ) , absurdo. Agora, se os movimentos iniciais das duas seqncias de movimentos que levam a posies finais diferentes so feitos nas posies k e ) , aps cada um desses movimentos o valor de g diminui e as posies finais ficam determinadas. Por outro lado, se nos dois primeiros lances mexemos primeiro no copo k e depois no copo ) chegamos mesma configurao que se primeiro mexermos no copo ) e depois no copo k (de fato, se inicialmente xi = x j = k e xr = xs = ) , chegaremos

80. Sejam =

{ n

, n  * . Prove que

Observao: x o inteiro tal que x x < x + 1.


SOLUO DE RODRIGO VILLARD MILET (RIO DE JANEIRO RJ)

Temos que = + 1. 1parte: A B = Suponha o contrrio, ou seja, que existem m e n naturais tais que [m] = [ 2 n] = k . Da temos que k < m < k + 1 e k < n < k + 1 (a desigualdade estrita, pois irracional), portanto :

1 m+n 1 m+n < + 2 =1< k < m + n < k +1 k +1 k


o que uma contradio, j que k, m e n so naturais.

EUREKA! N17, 2003

57

Sociedade Brasileira de Matemtica

2parte: A B = * Suponha que exista um natural h que no est nem em A nem em B. Ento existem naturais m e n tais que m < h < h + 1 < (m + 1) e n < h < h + 1< (n + 1). Logo :

1 m+n 1 m+n+2 < + 2 =1< m + n < h < h +1< m + n + 2 h h +1


o que uma contradio, j que m + n + 1 o nico natural entre m + n e m + n + 2. Enviaram solues de problemas anteriores os seguintes leitores da EUREKA!
Carlos Alberto da Silva Victor Digo Veloso Ucha Diogo Diniz Pereira da Silva e Silva Helder Oliveira de Castro Joo Fernandes de Moura Leno Silva Rocha Murilo Rebouas Fernandes de Lima Nilpolis RJ Teresina PI Enviado via correio eletrnico Mogi das Cruzes SP Niteri RJ Goinia GO Goinia GO

EUREKA! N17, 2003

58

Sociedade Brasileira de Matemtica

PROBLEMAS PROPOSTOS
Convidamos o leitor a enviar solues dos problemas propostos e sugestes de novos problemas para os prximos nmeros.

81) Num tringulo issceles ABC com AB = BC, temos AC = BH, onde BH a altura relativa ao lado AC. Traamos uma reta BD que corta o prolongamento da reta AC em D de tal forma que os raios dos crculos inscritos nos tringulos . ABC e CBD so iguais. Determine o ngulo ABD 82) a) Demonstre a identidade

cos() cos(2) cos(4)...cos(2n) = cos(2 j ) =


j =0

sen(2n+1) 2n+1 sen()

b) Prove que

1 1 1 1 1 1 1 1 1 2 + + + ... = cos j+2 = . 2 2 2 2 2 2 2 2 2 2 j =0

83) Seja

Determine quantas funes f :   satisfazem f(2003) = 2003, f(n) 2003 para todo n 2003 e f(m + f(n)) = f(f(m)) + f(n), para todo m, n . 84) Prove que se

 = {0,1,2,3,...}.

ento A = * . n A 4n A e n A

A * = {1,2,3,...} um conjunto no-vazio tal que

Obs. x o nico inteiro tal que x 1 < x x 85) Mostre que todo tringulo pode ser dividido em 9 pentgonos convexos de reas iguais. 86) Encontre todas as triplas de inteiros positivos (a, m, n) tais que am +1 divide

(a +1)n .

EUREKA! N17, 2003

59

Sociedade Brasileira de Matemtica

87) Seja a(1) = 1 e, para cada inteiro n 2, a(n) igual ao menor inteiro positivo que no pertence a {a( j), j < n} tal que

a( j) seja mltiplo de n. Prove que


j =1

a(a(n)) = n para todo inteiro positivo n.


88) Prove que se r e cos(r ) ento cos(r ) 1, ,0, ,1.

1 2

1 2

'4.H 8,- , 7,3/0 043,7/ : 07 ,1 724: 6:0 3 4 E 84 : 08 390 7,8 548 9 ;,8 5,7, , 06:, 4      :7,390  ,348 3 3 :F2 .4380 : : /0243897,7 894 !,70. , 807 :2, ,1 72, 4 ;07/,/0 7, :2, ;0 6:0 9,2-F2 3 3 :F2 54/0 574;,7 6:0 07, 1, 8, 39709,394 4,2  08 /, &3 ;078 /,/0 /0 ,7;,7/ 97,-, ,3/4 .42 :2 5490390 .425:9,/47 03.43974:
   


      

 ,1 72, 4 /0 : 07 F 1, 8,
Problema 81 proposto por Geraldo Perlino (Itapecerica da Serra SP); Problema 82 proposto por Clodoaldo Lessa (Mogi das Cruzes SP); Problema 83 adaptado de um problema proposto por Gibran M. de Souza (Natal RN); Problema 84 proposto por Anderson Torres (So Paulo SP); Problema 85 proposto por Gibran M. de Souza (Natal RN); Problema 88 proposto por C.G. Moreira e Jos Paulo Carneiro (Rio de Janeiro RJ).

EUREKA! N17, 2003

60

Sociedade Brasileira de Matemtica

AGENDA OLMPICA
XXV OLIMPADA BRASILEIRA DE MATEMTICA NVEIS 1, 2 e 3 Primeira Fase Sbado, 7 de junho de 2003 Segunda Fase Sbado, 13 de setembro de 2003 Terceira Fase Sbado, 18 de outubro de 2003 (nveis 1, 2 e 3) Domingo, 19 de outubro de 2003 (nveis 2 e 3 - segundo dia de prova). NVEL UNIVERSITRIO Primeira Fase Sbado, 13 de setembro de 2003 Segunda Fase Sbado, 18 e Domingo, 19 de outubro de 2003

IX OLIMPADA DE MAIO 10 de maio de 2003

XIV OLIMPADA DE MATEMTICA DO CONE SUL 23 a 30 de maio de 2003 Ica Peru

XLIV OLIMPADA INTERNACIONAL DE MATEMTICA 07 a 19 de julho de 2003 Tquio Japo

X OLIMPADA INTERNACIONAL DE MATEMTICA UNIVERSITRIA 25 a 31 de julho de 2003 Universidade Babes-Bolyai, Cluj-Napoca, Romnia

XVIII OLIMPADA IBEROAMERICANA DE MATEMTICA 13 a 20 de setembro de 2003 Argentina

VI OLIMPADA IBEROAMERICANA DE MATEMTICA UNIVERSITRIA 8 de novembro de 2003

EUREKA! N17, 2003

61

Sociedade Brasileira de Matemtica

COORDENADORES REGIONAIS
Alberto Hassen Raad Amarsio da Silva Arajo Ana Paula Bernardi da Silva Benedito Tadeu Vasconcelos Freire Carlos Frederico Borges Palmeira Claus Haetinger Cleonor Crescncio das Neves lio Mega Florncio Ferreira Guimares Filho Gil Cunha Gomes Filho Ronaldo Alves Garcia Reginaldo de Lima Pereira Ivanilde Fernandes Saad Jacqueline Fabiola Rojas Arancibia Janice T. Reichert Joo Bencio de Melo Neto Joo Francisco Melo Libonati Jos Carlos dos Santos Rodrigues Jos Cloves Saraiva Jos Gaspar Ruas Filho Jos Luiz Rosas Pinho Jos Vieira Alves Licio Hernandes Bezerra Luzinalva Miranda de Amorim Mrio Rocha Retamoso Marcelo Rufino de Oliveira Marcelo Mendes Pablo Rodrigo Ganassim Ramn Mendoza Ral Cintra de Negreiros Ribeiro Reinaldo Gen Ichiro Arakaki Ricardo Amorim Srgio Cludio Ramos Tadeu Ferreira Gomes Toms Menndez Rodrigues Valdenberg Arajo da Silva Valdeni Soliani Franco Vnia Cristina Silva Rodrigues Wagner Pereira Lopes (UFJF) (UFV) (Universidade Catlica de Braslia) (UFRN) (PUC-Rio) (UNIVATES) (UTAM) (Colgio Etapa) (UFES) (Colgio ACAE) (UFGO) (Escola Tcnica Federal de Roraima) (UC. Dom Bosco) (UFPB) (UNOCHAPEC) (UFPI) (Grupo Educacional Ideal) (Unespar) (UFMA) (ICMC-USP) (UFSC) (UFPB) (UFSC) (UFBA) (UFRG) (Grupo Educacional Ideal) (Colgio Farias Brito, Pr-vestibular) (Liceu Terras do Engenho) (UFPE) (Colgio Anglo) (INPE) (Centro Educacional Logos) (IM-UFRGS) (UEBA) (U. Federal de Rondnia) (U. Federal de Sergipe) (U. Estadual de Maring) (U. Metodista de SP) (CEFET GO) Juiz de Fora MG Viosa MG Braslia DF Natal RN Rio de Janeiro RJ Lajeado RS Manaus AM So Paulo SP Vitria ES Volta Redonda RJ Goinia GO Boa Vista RR Campo Grande MS Joo Pessoa PB Chapec SC Teresina PI Belm PA Campo Mouro PR So Luis MA So Carlos SP Florianpolis SC Campina Grande PB Florianpolis SC Salvador BA Rio Grande RS Belm PA Fortaleza CE Piracicaba SP Recife PE Atibaia SP SJ dos Campos SP Nova Iguau RJ Porto Alegre RS Juazeiro BA Porto Velho RO So Cristovo SE Maring PR S.B. do Campo SP Jata GO

EUREKA! N17, 2003

62

CONTEDO

AOS LEITORES XIV OLIMPADA DE MATEMTICA DO CONE SUL Enunciados e Solues XLIV OLIMPADA INTERNACIONAL DE MATEMTICA Enunciados e Solues

2 3 13

X OLIMPADA INTERNACIONAL DE MATEMTICA PARA ESTUDANTES UNIVERSITRIOS 23 Enunciados e Solues XVIII OLIMPADA IBERO-AMERICANA DE MATEMTICA Enunciados e Solues 34

ARTIGOS
A DESIGUALDADE DE ERDS - MORDELL Anderson Torres COMO QUE FAZ? SOLUES DE PROBLEMAS PROPOSTOS PROBLEMAS PROPOSTOS COORDENADORES REGIONAIS 42 53 54 58 62

Sociedade Brasileira de Matemtica

AOS LEITORES
Chegamos ltima edio de 2003 publicando solues de diversas olimpadas internacionais: a Olimpada de Matemtica do Cone Sul, a Olimpada Internacional de Matemtica, a Olimpada Ibero-americana e a Olimpada Internacional de Matemtica para Estudantes Universitrios, da qual participamos pela primeira vez. Publicamos tambm um artigo sobre a desigualdade de Erds Mordell, que ajuda a resolver um dos problemas mais difceis que j caram na IMO. Estamos propondo sete problemas bacanas na seo de problemas propostos, que se somam aos 6 problemas da ltima edio dos quais ainda no recebemos soluo. Haver portanto bastante diverso para o comeo de 2004 Em 2004 a Olimpada Internacional de Matemtica ser realizada na Grcia, assim como as Olimpadas. Vamos torcer para o Brasil trazer belas medalhas nos dois casos Abraos e feliz ano novo a todos,

Os editores

EUREKA! N18, 2003

Sociedade Brasileira de Matemtica

XIV OLIMPADA DE MATEMTICA DO CONE SUL


Enunciados e Solues
PROBLEMA 1

Em um torneio de futebol entre quatro equipes, A, B, C e D, cada equipe joga com cada uma das outras exatamente uma vez. a) Decidir se possvel que, ao finalizar o torneio, as quantidades de gols marcados e sofridos pelas equipes sejam: A 1 4 B 3 4 C 6 4 D 7 5

Gols marcados Gols sofridos

Se a resposta afirmativa, d um exemplo com os resultados das seis partidas; em caso contrrio, justifique. b) Decidir se possvel que, ao finalizar o torneio, as quantidades de gols marcados e sofridos pelas equipes sejam: A 1 4 B 3 4 C 6 4 D 13 11

Gols marcados Gols sofridos

Se a resposta afirmativa, d um exemplo com os resultados das seis partidas; em caso contrrio, justifique.
SOLUO DE FBIO DIAS MOREIRA (RIO DE JANEIRO RJ)

a) A configurao apresentada possvel. Por exemplo, AB AC AD BC BD CD


EUREKA! N18, 2003

11 00 03 03 20 34

Sociedade Brasileira de Matemtica

b) Como D no joga contra si mesmo, todos os gols que marca so contra A, B e C. Logo todo gol marcado por D aumenta em um a contagem de gols marcados por D e aumenta em um a soma do nmero de gols sofridos por A, B e C. Mas como D marcou treze gols, A, B e C deveriam ter sofrido pelo menos treze gols no total, mas sofreram apenas 12, absurdo! Logo a situao apresentada impossvel.
PROBLEMA 2

Considere a seqncia {an} definida da seguinte maneira: a1 = 1 a2 = 3 an+2 = 2an+1 an + 1, para todo inteiro n 1. Provar que a mxima potncia de 2 que divide a4006 a4005 22003. Seja An = an an 1 ; n Induo: n
*

SOLUO DE RODRIGO AGUIAR PINHEIRO (FORTALEZA CE)

e n2

n , temos 2 sendo a maior potncia de 2 que divide A2 n e

2n +1 A2 n +1
Base de Induo: n = 1: a3 = 2a2 a1 + 1 = 2 3 1 + 1 = 7 A21 = a2 a1 = 2 e

A21+1 = a3 a2 = 4
1 1+1 2 Temos que 2 A21 mas 2 no divide A21 e 2 A21+1. OK!!

n = 2 : a4 = 2 a3 a2 + 1 = 2 7 3 + 1 = 43 a5 = 2 a4 a3 + 1 = 2 43 7 + 1 = 603 A4 = 43 7 = 36 e A5 = 603 43 = 560 36 = 22 9, logo 22 A22 , mas 23 no divide A22 560 = 24 7 5, logo 22 +1 A22 +1 .
Passo Indutivo: Supondo que para todos os naturais menores ou iguais a k, valha a sentena da induo, provaremos que vale para k + 1.

A2 k + 2 = a2 k + 2 a2 k +1 = (2 a2 k +1 a2 k + 1) (2 a2 k a2 k 1 + 1) = 2a2 k ( a2 k +1 a2 k 1 ) = 2a2 k ( a2 k +1 a2 k + a2 k a2 k 1 ) A2 k + 2 = 2a2 k ( A2 k +1 + A2 k )


EUREKA! N18, 2003

Sociedade Brasileira de Matemtica


k k +1 Seja A2 k = 2 x2 k , x2 k mpar (hiptese de induo) e A2 k +1 = 2 x2 k +1 .

2k k k +1 Temos: A2 k + 2 = 2 a2 k (2 x2 k +1 + 2 x2 k ) = 2 a2 k (2 x2 k +1 + x2 k ) mpar mpar


k +1 k +2

Portanto 2

A2 k + 2 , mas 2

no divide A2 k + 2

A2 k + 3 = 2a2 k +1 ( A2 k + 2 + A2 k +1 ) = 2 a2 k +1 (2k +1 x2 k + 2 + 2k +1 x2 k +1 )
= 2k + 2 a2 k +1 ( x2 k + 2 + x2 k +1 ) , portanto 2k +1+1 A2( k +1) +1
2003 A4006 = a4006 a4005 , mas 22004 no divide A4006 . Para k = 2003, temos que 2

Seja ABC um tringulo acutngulo tal que o ngulo B mede 60o . A circunferncia de dimetro AC intersecta as bissetrizes internas de A e C nos pontos M e N respectivamente (M A, N C). A bissetriz interna do ngulo B intersecta MN e AC nos pontos R e S, respectivamente. Demonstrar que BR RS .
SOLUO DE THIAGO COSTA LEITE SANTOS (SO PAULO SP)

PROBLEMA 3

B 30 30 W 30 30 N

30


T S C

EUREKA! N18, 2003

Sociedade Brasileira de Matemtica

Digamos que BAC = 2 BCA = 2 , Sem perda de generalidade 2 + 2 = 120 + = 60 . N est dentro do tringulo, pois como

ANC =AMC = 90 NAC = 90 NAM = 90 = 30 e NAB = 30 .

AC

dimetro

De modo anlogo M est fora do tringulo. Assim, MCA = 90 MCB = 90 2 = 30 Como KMI =NCA , pois ANMC inscritvel, KMI = =KCI IKMC inscritvel IKC = IMC = 90. Como KMI = e

KCM = 30 , CKM = 60 = NKB e KWB = 60. Como I o incentro,

KI 1 = BI = 2KI = 2r BI 2 (r o inraio). O tringulo KWB equiltero, pois provamos que BWK =BKW =WBK = 60. Como BR bissetriz de WBK, BR WK . IK = IT = r sen30 =
No tringulo RKI:

RI 1 KI r = RI = = . 2 2 KI 2 r 3r BR = BI RI = 2r = 2 2 SI IT = r, pois ITS = 90 r r 3r RS = RI + SI = + SI + r = = BR RS BR . 2 2 2
sen30 =

PROBLEMA 4

No tringulo acutngulo ABC, os pontos H, G e M encontram-se sobre o lado BC, de modo que AH, AG e AM so altura, bissetriz e mediana do tringulo, respectivamente. Sabe-se que HG = GM, AB = 10 e AC = 14. Determinar a rea do tringulo ABC.

EUREKA! N18, 2003

Sociedade Brasileira de Matemtica

SOLUO DE RODRIGO AGUIAR PINHEIRO (FORTALEZA CE)

10

14

z y C

H k G

Seja BH = x, HG = GM = k, MG = y e AH = z. Como AM mediana, BM = MC BH + HG + GM = MC x + 2k = y (I) Apliquemos o teorema da bissetriz interna, considerando a bissetriz AG:

AB BG 10 x + k = = 5( y + k) = 7(x + k) AC GC 14 y + k 5y = 7x + 2k (II) 3 (I) em (II): 5y = 6x + x + 2k = 6x + y y = x (III) 2 3 1 (III) em (I): x + 2k = y = x x = 2k x = 4k 2 2 3 3 (III): y = x = (4k) y = 6k. 2 2 Apliquemos o teorema de Pitgoras no AHC : AC2 = AH2 + HC2 z2 + (8k)2 =142 Teorema de Pitgoras no AHB : AB2 = AH2 + HB2 z2 + (4k)2 =102
Temos:

z2 + 64k2 =196 z2 +16k2 =100

z2 + 64k2 (z2 +16k2 ) =196 100

48k2 = 96 k2 = 2 k = 2 z2 +16k2 =100 z2 +16 2 =100 z2 = 68 z = 68; rea =


68 12 2 =12 34 . 2

EUREKA! N18, 2003

Sociedade Brasileira de Matemtica

PROBLEMA 5

Seja n = 3k + 1 , onde k um inteiro, k 1 . Constri-se um arranjo triangular de lado n formado por crculos de mesmo raio como o mostrado na figura para n = 7.

Determinar, para cada k, o maior nmero de crculos que podem ser coloridos de vermelho de tal modo que no existam dois crculos vermelhos tangentes entre si.
ADAPTAO DA SOLUO DE THIAGO COSTA LEITE SANTOS (SO PAULO SP)

Obviamente n = 6k0 + 1 ou n = 6k0 + 4 , quando k = 2k0 ou k = 2k0 + 1 , respectivamente. Observe que se tivermos seis crculos do seguinte formato: ns s podemos pintar no mximo 2 bolas; caso pintemos 3, haver duas bolas pintadas tangentes, o que absurdo!

Primeiro caso: k = 2k0 n = 6k0 + 1 Denote por ak o maior nmero de crculos que podemos pintar. No tringulo n n pegue as 3 ltimas linhas:
Primeira linha Segunda linha


3k +1 3k +2 3k +3 3(k+1)+1

3k +2 3k +3 3k + 4

EUREKA! N18, 2003

Sociedade Brasileira de Matemtica

Dividimos aqueles crculos em peas de 6 crculos e uma pea de 3 crculos da seguinte forma:
3k +2 3k +3 3k + 4

3k + 2 2

Como cada pea peas (note que

tem no mximo 2 crculos pintados e ns temos

3k + 2 pois k 0(mod 2)) e na pea 2 um crculo ak +1 ak + 3(k + 1) nesse caso.


Segundo caso: k = 2k0 + 1 n = 6k0 + 4 As trs ltimas linhas so assim:

3k + 2 2

podemos pintar

3k +2 3k +3 3k + 4

3k + 1 ak +1 ak + 2 . + r onde r o nmero de crculos pintados da pea 2 Temos que r 3 .(*) Se r = 2 temos de novo ak +1 ak + 3(k + 1) e um jeito de montarmos o seguinte:

EUREKA! N18, 2003

Sociedade Brasileira de Matemtica

1 2 3 4 5 6 7 8 9

Pegue a pea destacada e coloque assim:

3k + 1= n 3k + 2 = n + 1 3k + 3 = n + 2 3k + 4 = n + 3

A partir da fazemos por induo:

Nos dois casos (k par ou k mpar), as trs ltimas linhas podem ser pintadas assim: Suponha pintado at a linha n e pinte as linhas n + 1, n + 2, n + 3 da seguinte forma. Na linha n + i = 3k + 1 + i, 1 i 3 faa o seguinte: enumere de 1 a n + i. Se i = 1: na linha n + 1, se o crculo tiver um nmero da forma 3k, pinte-o de vermelho. Se i = 2: na linha n + 2, se o crculo tiver um nmero da forma 3k + 2, pinte-o. Se i = 3: na linha n + 3, se o crculo tiver um nmero da forma 3k + 1, pinte-o.

3k (k + 1) , k 1. 2 3k + 1 Vamos ver o que acontece se em (*) tivermos r = 3 e 2 + r crculos 2


Assim, teremos ak +1 ak + 3(k + 1), a1 = 4 ak 1 + pintados nas 3 ltimas linhas. A pea ser pintada assim As 3 ltimas linhas sero pintadas assim:

EUREKA! N18, 2003

10

Sociedade Brasileira de Matemtica

3k 1 3k 3k + 1 3k + 2 3k + 3 3k + 4

Na linha i coloque os nmeros assim:


1 2 3 4 5 6 i

Na linha 3k + 2 e 3k + 4 os nmeros impares sero pintados e na linha 3k + 3 s tem no mximo 2 crculos pintados, ningum. Como cada pea da forma nas linhas 3k + 2 e 3k + 4 todos os nmeros mpares sero pintados, porque se no ak +1 ak + 3(k + 1). Assim, nas 3 linhas anteriores (3k 1, 3k e 3k + 1) temos no mximo 2k crculos pintados (pois na linha 3k + 1 no podemos ter nenhum), donde ak +1 ak 1 + 2k + 3k + 4 ak 1 + 3k + 3( k + 1) e, de qualquer jeito, temos

ak 1 + 3 j = 1 +
j =1

3k (k + 1) , 2

por

induo.

resposta,

portanto,

ak = 1 +

3k (k + 1) . 2

PROBLEMA 6

Demonstrar que existe uma seqncia de inteiros positivos x1 , x2 ,..., xn ,... que satisfaz as duas condies seguintes: i) contm exatamente uma vez cada um dos inteiros positivos, ii) para cada n = 1, 2,..., a soma parcial x1 + x2 + ... + xn divisvel por n .
n

SOLUO DE FBIO DIAS MOREIRA (RIO DE JANEIRO RJ)

Definiremos indutivamente uma tal seqncia. Seja x1 = 1, x2 = 3, e suponha a seqncia definida at x2n. Seja k o menor inteiro positivo que ainda no apareceu entre x1 ,..., x2 n . Pelo Teorema Chins dos Restos, existem infinitos x2 n +1 tais que

EUREKA! N18, 2003

11

Sociedade Brasileira de Matemtica

x2 n +1 x1 x2 ... x2 n (mod(2n + 1) 2 n +1 ) , e x2 n +1 x1 x2 ... x2 n k (mod(2n + 2) 2 n + 2 ) mdc(2n + 1, 2n + 2) = 1 mdc((2n + 1)2 n +1 ,(2n + 2) 2 n + 2 ) = 1. Em particular, existe um tal x2 n +1 {x1 , x2 ,..., x2 n , k}, j que este ltimo conjunto
j que
2 n +1 x1 + ... + x2 n +1 e finito. Tomamos x2 n +1 = x2 n +1 e x2 n + 2 = k . Note que (2n + 1)

(2n + 2) 2 n + 2 x1 + ... + x2 n + 2 , logo a extenso da seqncia respeita (ii). Como,


para 1 i 2n, x2 n +1 xi , x2 n +1 k por construo e x2 n + 2 xi pela definio de k, a extenso respeita que cada inteiro aparece no mximo uma vez. Mas depois de x extenses, todos os nmeros 1, 2,,x j apareceram, pois so os x menores inteiros positivos. Logo todo inteiro positivo x aparece at o (2x + 2)-simo termo, logo todo inteiro aparece pelo menos uma vez. Logo ( xi )i respeita (i), e portanto ( xi )i satisfaz as condies do enunciado.

EUREKA! N18, 2003

12

Sociedade Brasileira de Matemtica

XLIV OLIMPADA INTERNACIONAL DE MATEMTICA


07 a 19 de julho, Tquio - Japo
PROBLEMA 1 Seja A um subconjunto do conjunto S = {1,2, ,1000000} com exatamente 101

elementos. Demonstre que existem nmeros t1 , t 2 , , t100 em S tais que os conjuntos A j = {x + t j | x A} , para j = 1, 2, ,100, so disjuntos dois a dois.
SOLUO DE FBIO DIAS MOREIRA (RIO DE JANEIRO RJ)

101 Seja A = {x y | x, y A, x > y}. Certamente A S ,| A | = 5050, j que 2 101 2 h pares de inteiros ( x, y ) A tais que x > y. Afirmamos que se 2 T = {t1 ,..., t100 } tal que t j ti A, i, j {1,...,100}, ento t1 ,..., t100 satisfazem o
enunciado. De fato, se houver x, y A, x > y, tais que x + ti = y + t j , ento
x y = t j ti . Mas o lado esquerdo da igualdade est em Aj , o direito no,

absurdo! Ento construmos T da seguinte forma: i) 1 T Para cada elemento ti adicionado, proibimos todos os inteiros da forma ii) ti + x, x A. Escolhemos o menor inteiro que no foi proibido, nem escolhido iii) anteriormente. iv) Repetimos ii) e iii) 99 vezes.
Se existissem ti , t j , t j > ti em T, tais que t j ti A, ento, como t j > ti ,

t j = ti + ,

* +

, logo A, absurdo, pois t j teria sido proibido, pois da

forma ti + x, x A. Logo basta verificar que T S . Mas como min T = 1 , basta verificar max T 106. Mas se T = {1 = t1 < t2 < ... < t100 } , ento t2 5052 , pois h no mximo 5050 inteiros proibidos entre ele e t1 , alm de 1 j escolhido. Da mesma
EUREKA! N18, 2003

13

Sociedade Brasileira de Matemtica

maneira, t3 10103,..., t100 99 5050 + 100 = 500050 < 10 6 , demonstrao.


PROBLEMA 2

concluindo

Determine todos os pares de inteiros positivos (a, b) tais que


a2 2ab b 3 + 1
2

um inteiro positivo.
SOLUO DE ALEX CORRA ABREU (NITERI RJ)

Primeiro caso: b = 1 a2 inteiro a par temos as solues (2k, 1) k N*. 2a Segundo caso: b > 1 como queremos inteiro positivo 2ab 2 b3 + 1 > 0 b 2 (2a b) + 1 > 0 e como b > 1, devemos ter b 2a agora: 2')se 2a b a, b 2 (2a b) + 1 a 2 a 2 (2a b) + 1 a 2 1 a 2 (b 2a + 1)

b = 2a . Temos ento as solues (a, 2a ) a N* . 2'') se b < a e (a, b) uma soluo, suponha que a2 = k a2 2kb2a + (b3 1)k = 0 ento a equao x 2 2kb 2 x + (b3 1) k = 0 2ab2 b3 +1 tem uma soluo a e a outra 2kb 2 a, que positiva pois o produto das solues (b3 1)k > 0. Ento vamos mostrar que 2a 2b2 ab3 a b 2kb2 a b a b 2ab3 b4 + b ab3 a 2ab2 b3 + 1 2ab2 b3 + 1 ab3 + b b4 + a 0 b3 (a b) + a + b 0 o que verdade, pois a > b. Assim, pelo
caso 2'), b = 2(2kb2 a) =

4a2b2 2ab3 2a 2 a = 2ab3 b4 + b = 2ab3 2a 2ab2 b3 +1 2ab2 b3 +1 b4 b b2 , com b par, pois k = . Temos ento as solues 2a = b4 b a = 2 4 b4 b , b , b > 1 par. 2

EUREKA! N18, 2003

14

Sociedade Brasileira de Matemtica

PROBLEMA 3

Considere um hexgono convexo tal que para cada quaisquer dois lados opostos verifica-se a seguinte propriedade: a distncia entre os seus pontos mdios igual a 3 2 vezes a soma dos seus comprimentos. Demonstre que todos os ngulos do hexgono so iguais. (Um hexgono convexo ABCDEF tem trs pares de lados opostos: AB e DE , BC e EF , CD e FA ).
SOLUO OFICIAL

Primeira Soluo: Lema: Considere o tringulo PQR com QPR 60. Seja L o ponto mdio de QR. Logo PL 3 QR 2 , com igualdade se e somente se o tringulo PQR equiltero. Prova:

S P

Seja S um ponto tal que o tringulo QRS equiltero, e os pontos P e S encontram-se no mesmo semiplano limitado pela linha QR. Logo o ponto P pertence ao circuncrculo do tringulo QRS, que se encontra dentro do crculo de centro L e raio 3 QR 2 . Isto completa a prova do lema.

EUREKA! N18, 2003

15

Sociedade Brasileira de Matemtica

A F

As diagonais principais do hexgono convexo formam um tringulo embora o tringulo possa ser degenerado. Assim podemos escolher duas das trs diagonais que formam um ngulo maior ou igual a 60. Sem perda de generalidade, podemos assumir que as diagonais AD e BE do hexgono dado ABCDEF satisfazem APB 60 , onde P a interseo dessas diagonais. Logo, usando o lema, temos: 3 MN = ( AB + DE ) PM + PN MN , 2 onde M e N so pontos mdios de AB e DE, respectivamente. Assim pelo lema, os tringulos ABP e DEP so equilteros. Consequentemente a diagonal CF forma um ngulo maior ou igual a 60 com uma das diagonais AD e BE. Sem perda de generalidade, assumimos que AQF 60, onde Q a interseo de AD e CF. Da mesma maneira acima, deduzimos que os tringulos AQF e CQD so equilteros. Isto implica que BRC = 60, onde R a interseo de BE e CF. Usando o mesmo argumento acima pela terceira vez, obtemos que os tringulos BCR e EFR so equilteros. Isto completa a soluo.

EUREKA! N18, 2003

16

Sociedade Brasileira de Matemtica

Segunda Soluo: Seja ABCDEF o hexgono dado e seja a = AB, b = BC ,..., f = FA.

B b C c D

A f F e

Sejam M e N os pontos mdios dos lados AB e DE, respectivamente. Temos 1 1 1 1 MN = a + b + c + d e MN = a f e d . 2 2 2 2 Assim temos 1 (1) MN = (b + c e f ). 2 Da propriedade dada temos

MN =

3 3 a + d ) ad . ( 2 2

(2)

x = a d, y = c f, z = e b. desde (1) e (2) obtemos:

yz 3 x.
Similarmente vemos que

(3)

zx 3 y , x y 3 z .
Note que (3) y 2 y z + z 3 x ,
2 2 2

(4) (5)

EUREKA! N18, 2003

17

Sociedade Brasileira de Matemtica

(4) z 2 z x + x 3 y , (5) x 2 x y + y 3 z . Adicionando as ltimas trs inequaes, obtemos


2 2 2

x y z 2 y z 2 z x 2 x y 0,
ou x + y + z 0. Assim x + y + z = 0 e valem as igualdades em todas as inequaes acima. Da, conclumos que: x + y + z = 0,
2

y z = 3 x , a // d // x, z x = 3 y , c // f // y, x y = 3 z , e // b // z.
Supondo que PQR o tringulo tal que PQ = x, QR = y, RP = z. Podemos assumir QRP 60 , sem perda de generalidade. Seja L o ponto mdio de QR, logo PL = z x 2 = 3 y 2 = 3Q R 2. Segue do lema na soluo 1 que o tringulo PQR equiltero. Assim temos ABC = BCD = ... = FAB = 120.

Nota: Obtemos a caracterizao completa dos hexgonos satisfazendo a propriedade dada. So obtidos a partir do tringulo equiltero cortando seus cantos na mesma altura.
PROBLEMA 4 Seja ABCD

um quadriltero convexo cujos vrtices esto sobre uma circunferncia. Sejam P , Q e R os ps das perpendiculares s retas BC , CA e AB , respectivamente, passando por D . Demonstre que PQ = QR se e s se as bissetrizes dos ngulos ABC e ADC se intersectam sobre a reta AC .

EUREKA! N18, 2003

18

Sociedade Brasileira de Matemtica

SOLUO DE ALEX CORRA ABREU (NITERI RJ)


B

A R

C P

PQ AB AD sen AD = DC pois DQCP inscritvel = = .Mas BC DC sen CD sen QCP de dimetro DC PQ = DC sen e, analogamente, AD sen QR = AD sen PQ = QR DC sen = AD sen = DC sen
como queramos. Observe que a nossa demonstrao independe do fato de ABCD ser um quadriltero inscritvel. De fato, o quadriltero mostrado no desenho no o : A est no segmento BR e C est no segmento BP. Todos os argumentos utilizados continuam vlidos se modificarmos as posies de A ou C.
PROBLEMA 5 Sejam n um inteiro positivo e

Seja CAB = e BCA = . Primeiro temos que as bisetrizes se intersectam sobreAC

x1 , x2 , , xn nmeros reais tais que

x1 x2

xn .

(a) Demonstre que

n n 2(n 2 1) n n ( xi x j ) 2 . xi x j 3 i =1 j =1 i =1 j =1
EUREKA! N18, 2003

19

Sociedade Brasileira de Matemtica

(b) Demonstre que a igualdade vlida se e s se x1 , x2 , , xn formam uma progresso aritmtica.


SOLUO

Vamos provar (a) por induo em n. Para n = 1 os dois lados valem 0 e, para n = 2, valem 4( x2 x1 ) . Faamos o passo da induo: Temos
2

x1 x2 xn xn +1 . Assim, sendo An = xi x j e
i =1 j =1

Bn = ( xi x j ) , temos An +1 = xi x j = 2 ( xn +1 x j ) + An
2 i =1 j =1 i =1 j =1 j =1

n +1 n +1

e Bn +1 = 2

( x
j =1

n +1

x j ) + Bn . Queremos ento provar que


2

2 n 2 ( n + 1) 1 A = 2n xn +1 2 x j + An Bn +1 = 3 j =1 n n 2n ( n + 2 ) 2 = + 2 nx 4 x x 2 x2 n +1 n +1 j j + Bn , ou equivalentemente, que 3 j =1 j =1 2 2 n +1

4n2 ( n 1) 3

n n 2(n 1) n 2n(n + 2) 2 x 4n x A x B 2 x A 2 + + + n j n xj 0. j n n+1 3 j =1 j =1 3 j=1 2 n+1

Olhando o lado esquerdo como uma funo quadrtica de xn +1 , conclumos que ela atinge o seu mnimo para xn +1 =

2(n 1) x j + 3 An
j =1

2n(n 1)

, quando temos
2

n 2( n 1) x j + 3 An 2 n n j =1 2 2 x j + An = An +1 = 2nxn +1 2 x j + An = n 1 j =1 j =1

n+2 2 2 An , enquanto Bn +1 = 2nxn +1 4 xn +1 x j + 2 x j + Bn = n 1 j =1 j =1


EUREKA! N18, 2003

20

Sociedade Brasileira de Matemtica


n n = 2 xn +1 nxn +1 2 x j + 2 x 2 j + Bn = j =1 j =1

2( n 1) x j + 3 An 3 An 2(n 1) x j n j =1 j =1 = + 2 x 2 j + Bn = n( n 1) 2( n 1) j =1

n 2 xj 2 2 2 n n n 9An 9An 1 j =1 2 2 2 2 = + + + = + + x B B n x x , j j n n j 2n(n 1)2 2n(n 1)2 n n j=1 j =1 j =1


mas Bn = ( x j xi )
i =1 j =1 n n 2 2 n n 2n(n + 2) 2 2 = 2 n x j x j , donde An Bn +1 +1 j =1 3 j =1

equivalente a

( n + 2) 2 ( n 1)

2 2 An

2 9 An 2n( n + 2) n + 1 Bn + 2 3 2n(n 1) n

2 3(n + 2) An2 2( n + 1)(n 1) 2 Bn + 9 An

2(n 2 1) Bn , que a hiptese de 3 induo. Assim, o valor mnimo de nossa funo quadrtica maior ou igual a 0, o que implica o resultado. Para provar b), note que, se valem as igualdades nas desigualdades acima, 2( n 2 1) 2 = Bn donde, por hiptese de induo, devemos ter em particular An 3 x j = x1 + ( j 1)r para um certo r 0 , j n; alm disso, devemos ter
2 2 3(n 1) An 2( n + 1)(n 1) 2 Bn An

xn +1 =
=

2(n 1) x j + 3 An
j =1

2n(n 1)

2( n 1) x j + 6
j =1

i i < j n

( j i )r =

2n(n 1)

n 1 1 n 3 x + j r + ( ( 1) ) 1 (n k )kr = n j =1 n(n 1) k =1

(n 1) 3 n( n 1)(n + 1) r r+ = x1 + nr , 2 6 n( n 1) progresso aritmtica, como queramos provar. x1 +


EUREKA! N18, 2003

donde

( xi )1 i n +1 uma

21

Sociedade Brasileira de Matemtica

PROBLEMA 6 Seja p um nmero primo. Demonstre que existe um nmero primo q tal que,
p para todo inteiro n, o nmero n p no divisvel por q.

evidente que p q pois se p = q basta tomarmos n mltiplo de q para termos um absurdo. Se encontrarmos um primo q tal que p q 1 e p
p
p q 1 p q 1 p

SOLUO DE SAMUEL BARBOSA FEITOSA (FORTALEZA - CE)

q 1 p

1(mod q) (1)

o problema ter acabado pois se n p (mod q ) , como p q ento mdc (n, q) = 1, donde n

(mod q ) mas pelo pequeno teorema de Fermat temos

q 1

1(mod q ) . Isso produz um absurdo por (1), logo devemos ter

n p (mod q ) .
p

Para encontrarmos o nosso tal primo q que satisfaz (1) consideremos o nmero

X = p p 1 + p p 2 + ... + p + 1 . claro que X p p 1 (2).


Seja q um divisor primo de X e

k = ord q p . De (2) temos que

k p k = 1 ou k = p . Se k = 1 temos

p 1(mod q ) p j 1(mod q ) ; da

X 1 + 1... + 1 = p (mod q ) mas q X e q p , absurdo, logo k = p, mas


p q 1 1(mod q ) p = k q 1 q = py + 1 para algum y inteiro. Existe algum
fator primo q de X tal que y no seja divisvel por p, pois se para todo fator primo q de X tivermos q 1(mod p 2 ) , teramos X 1(mod p 2 ) , mas

X p + 1 1(mod p 2 ) logo existe um primo q divisor de X tal que p q 1 e q 1(mod p 2 ) . Mostremos que tal primo q satisfaz (1). Como k = ord q p = p e

q 1 q 1(mod p ) k = p no divide p p
2

q 1 p

1(mod q) , donde o primo

q satisfaz (1).

EUREKA! N18, 2003

22

Sociedade Brasileira de Matemtica

X OLIMPADA INTERNACIONAL DE MATEMTICA PARA ESTUDANTES UNIVERSITRIOS


25 a 31 de Julho, Cluj - Napoca, Romnia
PROBLEMA 1

a) Seja a1 , a2 , , an ,... uma seqncia de nmeros reais tais que a1 = 1 e 3 an +1 > an , n. 2 an Prove que a seqncia tem um limite finito ou tende a infinito. n 1 3 2
b) Prove que para todo

> 1 existe uma seqncia a1 , a2 , , an ,... com as


an 3 2
.
n 1

mesmas propriedades, tal que lim n


SOLUO

= .

(a) Considere a seqncia bn =

an 3 2
n 1

bn +1 a = n +1 > 1 , logo (bn) uma seqncia crescente. bn 3an 2 Mas toda seqncia montona limitada convergente. Logo, h duas opes possveis: ou (bn) ilimitada e, como crescente, tende a infinito ou ento, se ela for limitada, ela tem um limite finito (pois crescente).
Temos bn > 0, n e (b) Tome q = 1

> 1 0 < q < 1 e portanto (bn) uma seqncia convergente, com


lim bn =
n

e considere a seqncia

bn = 1 + q + q 2 + ... + q n 1 .

1 = . 1 q

EUREKA! N18, 2003

23

Sociedade Brasileira de Matemtica

3 Portanto, a seqncia a n = 2

n 1

bn satisfaz as condies do enunciado.

Observao: A soluo da letra (b) bem mais natural do que parece. A letra (a) induz voc a pensar apenas na seqncia (bn). E na tentativa de encontrar uma seqncia com determinadas condies, nada mais natural do que tentar uma seqncia fcil como uma PG (e a descobrir o valor de q necessrio).
PROBLEMA 2

Sejam a1 , a2 , , a51 elementos no nulos de um corpo. Simultaneamente trocamos cada elemento pela soma dos outros 50. Desta forma a nova seqncia b1 , b2 , , b51 uma permutao da anterior. Quais so os possveis valores da caracterstica do corpo?
SOLUO

Seja p a caracterstica do corpo.

S = bk = ( S a k ) = 51S a k = 51S S , logo 49S = 0.


Se S 0 , temos que 49 = 0, logo p | 49 e como p primo, p = 7. Um exemplo de corpo que satisfaz essa propriedade
k =1 k =1 k =1

51

51

51

a k = 1 , para k = 1,2,...,51. 7 Se S = 0, cada a k igual a bk = a j , para algum j, donde a permutao tal


(pela ao da ) e 51 mpar. Portanto existe um n tal que a n = a ( n ) = a n , logo 2a n = 0 e como a n 0 , temos que 2 = 0 e o corpo

) , com
*

que a k = a ( k ) possui um ponto fixo, pois os nmeros 1,2,...,51 esto divididos em pares

possui caracterstica 2. Um exemplo de corpo que satisfaz essa propriedade GF(22), isto , o corpo cujos elementos so polinmios, tomados mdulo x2 + x + 1, com coeficientes em 2 . Basta tomar ( a1 , a 2 ,..., a 51 ) = (1, x,1 + x,1, x,1 + x,...,1, x,1 + x ) .
PROBLEMA 3

Seja A uma matriz quadrada n n tal que 3A3 = A2 + A + I. Prove que (Ak )k N converge a uma matriz idempotente B (i.e., a uma matriz B tal que B2 = B).

EUREKA! N18, 2003

24

Sociedade Brasileira de Matemtica

SOLUO

Seja m(x) o polinmio minimal de A. Como 3 A 3 A 2 A I = 0 , m(x) deve ser divisor de 3 x 3 x 2 x 1 = ( x 1)( x 1 )( x 2 ) , com 1, 2 =

1 2 . 3

Isso implica que m(x) tem razes distintas, e portanto A diagonalizvel, isto , existe uma matriz P inversvel tal que A = P 1 Diag (1,...,1, 1,..., 1, 2 ,..., 2 ) P , onde Diag(a,b,c,...) representa a matriz diagonal de entradas a,b,c,...
k k k Segue que Ak = P 1 Diag (1,...,1, k 1 ,..., 1 , 2 ,..., 2 ) P .

Como 1, 2 < 1 , segue que B = lim k Ak = P 1 Diag (1,...,1,0,...,0) P , que claramente idempotente.
SOLUO ALTERNATIVA

Seja Ak = Ak . Ento 3 Ak +3 = Ak + 2 + Ak +1 + Ak . Resolvendo a recorrncia (veja o artigo "Equaes de recorrncia", na Eureka! 9), obtemos
Ak = C0 + C1 1k + C2 2k para
k

todo k, sendo C0 , C1 , C2 matrizes quadradas de ordem n. Como | 1,2 | 1, lim Ak = C0 .


Como lim Ak = lim A2 k C02 = C0 , C0 idempotente.
k k

PROBLEMA 4

Determine o conjunto de todos os pares (a, b) de inteiros positivos para os quais o conjunto dos inteiros positivos pode ser decomposto em dois conjuntos A e B tais que a A = b B.
SOLUO

Note que o par (a, b) funciona se e somente se o par de coprimos a b , funciona. Vamos ento analisar os casos com mdc(a, b)=1. mdc (a, b) mdc (a, b) Suponha 1 A (o outro caso anlogo). Ento, a A = b B a b B , e portanto a mltiplo de b (pois os elementos de B so todos inteiros), e mdc(a, b) = 1 implica b = 1. Reciprocamente, dado qualquer par da forma (a, 1), com a > 1, possvel construir conjuntos A e B satisfazendo o enunciado. De fato, dado n N , seja kn a maior potncia de a que divide n. Tomando A = {n | kn par}, B = {n | kn mpar} temos A B = N , A B = , a A = 1 B .
EUREKA! N18, 2003

25

Sociedade Brasileira de Matemtica

Portanto, os pares possveis so os pares (a, b) com a b tais que a mltiplo de b ou b mltiplo de a.

PROBLEMA 5

Sejam g :[0,1] R uma funo contnua e f n : (0,1] R a seqncia de funes definida por f 0 ( x ) = g ( x) e fn+1 ( x) = Determine lim f n ( x ) para todo x (0,1] .
n

1 x fn (t )dt, x (0,1], n 0. x 0

SOLUO

Veja inicialmente que se g polinmio, o problema fcil, pois se g ( x) =

a
k =0

k f n ( x) = k x , temos claramente que

ak x k , portanto n k = 0 ( k + 1)
N

todos os coeficientes, com exceo do independente, tendem a zero quando n + , e lim f n ( x) = a 0 = g (0) . A idia tentar mostrar que isso tambm vale no caso de g no ser polinmio. Para isso, vamos usar o teorema da aproximao de Stone-Weierstrass: Dado

> 0 , existe um polinmio P tal que P( x) g ( x) <


x

, para todo x em [0,1]

(pois g contnua e [0,1] compacto). Agora olhe para a seqncia Pn tal que

1 ~ ~ ~ Pn +1 ( x) = Pn (t )dt e P0 = P . x0
Como P polinmio,
n +

~ lim Pn ( x) = P (0) ,

donde

para

grande,

~ Pn ( x) P(0) < . 3
Alm disso, temos que se Pn ( x ) f n ( x) <
EUREKA! N18, 2003

, ento:

26

Sociedade Brasileira de Matemtica

1 ~ Pn +1 ( x) f n +1 ( x) = x

[
x 0

1 ~ 1 ~ Pn (t ) f n (t ) dt Pn (t ) f n (t ) dt < dt = 3 x0 x03
x x

Segue por induo que Pn ( x ) f n ( x) < j que P0 ( x ) f 0 ( x) <

, para todo n natural e todo x em (0,1]

3 ~ ~ f n ( x) g (0) f n ( x) Pn ( x) + Pn ( x) P(0) + P(0) g (0) < + + = , 3 3 3 para n grande, o que prova que lim f n ( x) = g (0) .
n +

. Agora fica fcil:

SOLUO ALTERNATIVA

Vamos provar que lim f n ( x ) = g (0), x (0,1]. Para isso, vamos mostrar que,
n

para todo x (0,1] e

> 0, limsup f n ( x) g (0) .


n

Como g contnua, existe

> 0 tal que x < g ( x) g (0) < Se

0 < x < , para todo n 1, 1 x 1 x f n ( x) g (0) = ( f n 1 (t ) g (0)) dt f n 1 (t ) g (0) dt x 0 x 0


por induo,

donde,
temos

f n ( x) g (0) < , n .

Se

x ,

f n +1 ( x) g (0) =

x 1 f n (t ) g (0) dt + f n (t ) g (0) dt , donde, se 0 x M n = max{ f n ( x) g (0) , x (0,1]}, temos

1 x 1 x f n (t ) g (0) ) dt f n (t ) g (0) dt = ( x 0 x 0

M n +1 max{ , max
n

1 ( + ( x ) M n )} = max{ , + (1 ) M n }. Assim, se < x 1 x L = lim sup M n (que existe, pois M n m ax{ , M 0 }, n ) , temos

L max{ , + (1 ) L}, donde L ou L + (1 ) L L L , o que encerra a prova.


EUREKA! N18, 2003

27

Sociedade Brasileira de Matemtica

PROBLEMA 6

Seja f ( z ) = an z + an 1 z + ... + a1 z + a0 um polinmio com coeficientes reais. Prove que se as razes de f esto no semi-plano esquerdo {z C | Re( z ) < 0} ento ak ak +3 < ak +1ak + 2 para todo k = 0, 1,, n 3.
n

n 1

SOLUO

Podemos supor sem perda de generalidade que an = 1. Nesse caso, f(z) pode ser fatorado como produto de monmios da forma z + a ou z + a + bi com a > 0 (no segundo caso, se b 0, deve aparecer tambm o fator z + a bi ). Como ( z + a + bi )( z + a bi ) = z + 2az + a + b e
2 2 2

( z + a) + ( z + a) = z 2 + (a + a) z + aa, temos que f(z) pode ser obtido a partir


de um polinmio de grau 0 ou 1 com todos os coeficientes positivos por meio de sucessivas multiplicaes por polinmios da forma z + Az + B , com A, B > 0. Em particular, todos os seus coeficientes so positivos. Vamos agora proceder por induo: para n = 0 ou n = 1 o resultado vale por vacuidade. Seja agora f(z) um polinmio de grau n + 2, da forma
2

f ( z ) = (an z n + ... + a0 )( z 2 + Az + B), com an = 1 e


f ( z ) = an z n + ... + a0 satisfazendo a hiptese de induo. Convencionando ak = 0 se k < 0 ou k > n , e escrevendo f ( z ) = an + 2 z n + 2 + ... + a0 , temos, para 0 k n + 2, ak = ak 2 + Aak 1 + Bak . Queremos ento provar que (ak+1 + Aak+2 + Bak+3 )(ak2 + Aak1 + Bak ) < (ak + Aak+1 + Bak+2 )(ak1 + Aak + Bak+1) , mas (ak + Aak+1 + Bak+2 )(ak1 + Aak + Bak+1) (ak+1 + Aak+2 + Bak+3)(ak2 + Aak1 + Bak )

= (ak ak1 ak+1ak2 ) + B2 (ak+2ak+1 ak+3ak ) + A2 (ak+1ak ak+2ak1) +


2 2 +B(ak+2ak1 ak+3ak2 ) + A(ak ak+2ak2 ) + AB(ak +1 ak +3ak 1). Temos, por hiptese de induo, ak ak1 ak+1ak2 , ak+2ak+1 ak+3ak , ak+1ak ak+2ak1 ak+3ak ak+2ak+1 ak+1ak2 ak1ak , e, alm disso, e donde ak ak+1ak+3ak2 ak ak+1ak+2ak1, e logo ak+3ak2 ak+2ak1 (de fato temos ak ak+1 > 0 , a menos que k = n, quando ak +3ak2 = ak+2ak1 = 0) ; ak +2ak1 ak+1ak e
EUREKA! N18, 2003

28

Sociedade Brasileira de Matemtica


2 2 ak+1ak2 ak ak1 , donde ak1ak+1ak+2ak2 ak1ak+1ak , e logo ak+2ak2 ak , e 2 analogamente ak+3ak1 ak+1, sendo que pelo menos uma dessas duas ltimas

desigualdades estrita, o que conclui a prova.

SEGUNDO DIA
PROBLEMA 1 SOLUO

Sejam A e B matrizes reais n n tais que AB + A + B = 0. Prove que AB = BA.

AB + A + B = 0 AB + A + B + I = I A( B + I ) + ( B + I ) = I ( A + I )( B + I ) = I Logo, A + I e B + I so inversas uma da outra, donde ( A + I )( B + I ) = ( B + I )( A + I ) = I . Expandindo a ltima desigualdade vem BA + B + A + I = I e subtraindo esta da igualdade dada no enunciado obtm-se AB = BA .
PROBLEMA 2

Calcule o seguinte limite: lim +


x 0

2x

sen m t dt (m, n naturais dados). tn

SOLUO

Como a funo

sen t decrescente em (0, ) , e tende a 1 quando t tende a 0+, t sen(2 x) sen t temos que, para x (0, ) e x < t < 2x: < < 1 e portanto: t 2 2x
m m 2 x sen m t 2x t m sen(2 x) 2 x t dt < n dt < n dt . n 2x x t x t x t

sen(2 x) Como lim x 0 = 1 , a desigualdade acima mostra que o limite 2x procurado igual a

EUREKA! N18, 2003

29

Sociedade Brasileira de Matemtica

t m n +1 2 x 0, se m n + 1 > 0 x 0 m n +1 x m n +1 2 x 2x t mn , se m n + 1 < 0 lim x 0 t dt = x 0 x m n +1 x 2x ln 2, se m n + 1 = 0 ln t x x 0


PROBLEMA 3

Seja A um subconjunto fechado de Rn e seja B o conjunto de todos os pontos b de

Rn tais que existe exatamente um ponto a0 em A tal que a0 b = inf a b .


a A

Prove que B denso em Rn.


SOLUO

Vamos mostrar que dado > 0 e x n , existe y B( x, ) B . Se x A , ento basta tomar y = x. Caso contrrio, seja = inf aA | a x | . Como A fechado, existe um ponto

a A que realiza essa distncia (basta observar por exemplo que B ( x, 2 ) A compacto). Se esse ponto a no for nico, considere um ponto y = x + t ( a x), t (0,1) do segmento (x, a). Seja a um outro ponto de A. Se a estiver no prolongamento desse segmento de reta, ento claramente | a ' y | > | a y | . Caso contrrio, temos a desigualdade triangular estrita | a ' x | < | a ' y | + | y x | e portanto: | a ' y | > | a' x | | x y | | a x | | x y |=| a y | , a onde a 2 desigualdade usa que a um ponto de A tal que |a x| mnimo e a igualdade final usa o fato que y est no segmento de reta (x, a). Ou seja, todo ponto y escolhido dessa forma est em B. Escolhendo t por exemplo) obtemos um ponto suficientemente pequeno ( t = 2 a x y B( x, ) B como desejado.
PROBLEMA 4
EUREKA! N18, 2003

30

Sociedade Brasileira de Matemtica

Encontre todos os inteiros positivos n para os quais existe uma famlia F de subconjuntos de trs elementos de S ={1, 2, , n} que satisfaz as seguintes condies: para quaisquer elementos distintos a, b S existe exatamente um A F (i) tal que a, b A. Se a, b, c, x, y, z so tais que {a, b, x}, {a, c, y}, {b, c, z} F ento (ii) {x, y, z} F.
SOLUO

Vamos mostrar que uma tal famlia F existe se e somente se n = 2 k 1 para algum inteiro positivo k. k De fato, se G = ( 2 ) = 2 2 ... 2 , onde est definida a adio

( x1 , x2 ,..., xn ) + ( y1 , y2 ,..., yn ) = ( x1 + y1 (mod 2),

x2 + y2 (mod 2),..., xn + yn (mod 2)) ,

podemos definir S = G \ {(0,0,...,0)} e F = {{u , v, u + v}, u , v S , u v}. A propriedade (i) segue com A = {a, b, a + b} e a propriedade ii) segue de x = a + b , y = a + c x + y = a + a + b + c = b + c = z. Sejam agora S e F como no enunciado. Consideramos um conjunto G = S {0} , onde 0 um elemento de G que no pertence a S (um "zero artificial") e uma operao + definida em G por x se a, b S , a b e {a, b, x} F 0 se a = b a+b= a se b = 0 b se a = 0 No difcil ver que com essa operao G um grupo abeliano ( a + b = b + a para quaisquer a, b em G, (a + b) + c = a + (b + c), para quaisquer a, b, c G e para todo a G existe b em G com a+b=0. Segue que G isomorfo a k ( 2 ) para algum inteiro positivo k * , donde n = # S = # G 1 = 2k 1 , o que conclui a soluo. * De fato, se a G , a 0, H = {0, a} um subgrupo de G isomorfo a 2 , e, se definimos em G a relao de equivalncia x ~ y y x H , obtemos um quociente, G/H, o conjunto das classes de equivalncia x = { y G | y ~ x}, que um grupo com a operao x + y : = x + y , o qual tm as mesmas propriedades que G. Alm disso, G naturalmente isomorfo a ( G H ) H , e, por induo em # G ,

G H isomorfo a
EUREKA! N18, 2003

para algum r

, donde G isomorfo a

r +1

31

Sociedade Brasileira de Matemtica

Obs.: O conjunto S um espao projetivo finito (a generalizao k-dimensional do plano projetivo) sobre o corpo 2 (ou seja, de ordem 2), cuja famlia das retas F. De fato, a propriedade (i) equivalente a "por dois pontos passa uma nica reta" e a propriedade (ii) equivalente ao axioma de Veblen-Young: "dado um tringulo ABC, se uma reta r corta dois lados, ento corta o terceiro lado tambm" (no caso, os pontos do tringulo so a,b,c e a reta r={x,y,z}). As propriedades (i) e (ii) so, ento, equivalentes aos axiomas que definem um espao projetivo; veja o artigo "Aplicaes de planos projetivos finitos em Teoria dos Nmeros e Combinatria", de Carlos Shine, na Eureka! 15. PROBLEMA 5

a) Mostre que para toda funo f : Q Q R existe uma funo g : Q R tal que f ( x, y ) g ( x) + g ( y ), x, y Q . b) Encontre uma funo f : R R R para a qual no existe g : R R tal que f(x, y) g(x) + g(y), x, y R.
SOLUO

a)

Q enumervel, digamos Q = {r1 , r2 , r3 ,...}.

Assim, podemos definir g : Q R por g ( rn ) = max{ f ( ri , rj ) ,1 i , j n}. Assim, f ( ri , rj ) f ( ri , rj ) g ( rmax( i , j ) ) g ( ri ) + g ( rj ), i, j.

0, se x = y b) Podemos definir f ( x, y ) = 1 x y , se x y.
Se existisse g :

tal que f ( x, y ) g ( x ) + g ( y ), x, y

, se definirmos,

para cada inteiro positivo n, X n = { x

| g ( x ) n 2}, teremos

X
n =1

donde, como no-enumervel, algum dos X n deve ser no enumervel, e portanto tem pontos de acumulao, isto , existe uma seqncia de termos distintos ( y k ) k com yk X n para todo k tal que ( yk ) converge a um certo 1 x . Em particular, lim yk +1 yk = x x = 0 , e logo f ( yk , yk +1 ) = k yk +1 yk

tende a + , mas devemos ter f ( yk , yk +1 ) g ( yk ) + g ( yk +1 ) k , pois { yk , yk +1} X n , donde ( f ( y k , yk +1 )) k

n n + = n para todo 2 2 limitada, absurdo.

EUREKA! N18, 2003

32

Sociedade Brasileira de Matemtica

PROBLEMA 6

Seja a0 , a1 ,..., an ,... a seqncia definida por a0 = 1 , an+1 = Calcule

ak 1 n . n + 1 k =0 n k + 2

2
k =0

ak
k

(se existir).

SOLUO

Os an so positivos e, por induo, tem-se an 1, n (de fato, supondo vlido at n tem-se 1 n 1 1 n an +1 1 = 1 ). n + 1 k =0 n k + 2 n + 1 k =0 Considere ento a funo geratriz f ( x) = an x n (ele convergente para 0 < x <
n =0

1 pela observao acima). Derivando e usando a expresso dada obtemos: n n ak f ' ( x) = n an x n 1 = (n + 1) an +1 x n = x nk +2 n =0 k =0 n =1 n =0

Trocando a ordem do somatrio obtm-se: xm xnk ak x n k = f ' ( x) = a x = f ( x ) k k =0 n = k n k + 2 k =0 m=0 m + 2 n=k n k + 2 Portanto, x f ' x m +1 x m +1 x m +1 ln( f ( x)) ln( f (0)) = = = ( m + 1) (m + 2) f ( m 1 )( m 2 ) + + = = m 0 m 0 0 Como f (0) = 1:

ln( f ( x)) =

xm+1 1 xm+1 1 1 1 xm+1 1 xm+2 = = ln x ln m=0 m + 1 x m=0 m + 2 m=0 m + 1 x m=1 m + 1 1 x x 1 x

Colocando x =

1 obtemos 2

1 ln f ( ) = ln 2 2 ln 2 + 1 = 1 ln 2 , de modo que 2

2
n =0

an
n

1 e = f ( ) = e e ln 2 = . 2 2

Estas solues da International Mathematical Competition - 2003 foram redigidas por Mrcio Assad Cohen, Rodrigo Villard Milet e Carlos Gustavo Moreira do Rio de Janeiro RJ.
EUREKA! N18, 2003

33

Sociedade Brasileira de Matemtica

XVIII OLIMPADA IBEROAMERICANA DE MATEMTICA


13 a 20 de setembro, Mar del Plata - Argentina PRIMEIRO DIA
PROBLEMA 1

a) Tm-se duas sucesses, cada uma de 2003 inteiros consecutivos, e um tabuleiro de 2 linhas e 2003 colunas Decida se sempre possvel distribuir os nmeros da primeira sucesso na primeira linha e os da segunda sucesso na segunda linha, de modo que os resultados obtidos ao somar os dois nmeros de cada coluna formem uma nova sucesso de 2003 nmeros consecutivos. b) E se trocssemos 2003 por 2004? Tanto em a) como em b), se a resposta for afirmativa, explique como distribuiria os nmeros, e se for negativa, justifique o porqu.
SOLUO

Note que somar ou subtrair uma constante de uma sucesso de nmeros consecutivos a transforma em uma sucesso de nmeros consecutivos. Note tambm que somar ou subtrair uma constante de uma linha do tabuleiro soma ou subtrai a mesma constante da sucesso formada pela soma das colunas, logo esta operao no altera a "consecutividade" das linhas do tabuleiro. Portanto, sem perda de generalidade, as sucesses escritas nas duas primeiras linhas do tabuleiro so 1, 2, ,n onde n {2003, 2004} a) Sim. Escreva na primeira linha 1, 2,,2003 e na segunda linha 1002, 1003,,2003, 1, 2, , 1000, 1001, ou seja, o i-simo termo i + 1001 se i 1002. A seqncia final 2i + 1001 se i 1002 e 2i 1002 se i 1003 , que obviamente permutao de 1003, 1004, 1005,,3003, 3004, 3005. b) No. Uma seqncia de 2004 nmeros inteiros consecutivos tem forma k + 1,..., k + 2004, k . Sua soma vale
EUREKA! N18, 2003

34

Sociedade Brasileira de Matemtica

2004k +

2004 2005 = 2004k + 2004 1002 + 1002 1002(mod 2004). 2

A soma dos nmeros da primeira e da segunda linhas vale 1002 (mod 2004). Como a terceira linha (formada pela soma das colunas) formada pela soma das duas primeiras linhas, a soma das nmeros da terceira linha 1002 + 1002 0 (mod 2004). Mas se a terceira linha fosse composta por uma sucesso de nmeros consecutivos em alguma ordem, sua soma seria 1002 (mod 2004), absurdo! Logo a soma das colunas no pode formar uma sucesso de nmeros consecutivos.
PROBLEMA 2

Sejam C e D dois pontos da semicircunferncia de dimetro AB tais que B e C esto em semiplanos distintos em relao reta AD. Denotemos por M, N e P os pontos mdios de AC, DB e CD, respectivamente. Sejam OA e OB os circuncentros dos tringulos ACP e BDP. Demonstre que as retas OAOB e MN so paralelas.
SOLUO

2 T

C 2 M.

D
+

.Q R.
O

.N

O2 B

Sejam 2 , 2 e 2 as medidas de CD, BD e AC , respectivamente. Seja O o centro da semicircunferncia ACDB. Sejam R e Q os ps das perpendiculares a

OP que passam por O2 e O1 , respectivamente.


Sejam ainda S e CP , T e PD pontos mdios de CP e PD, respectivamente.

EUREKA! N18, 2003

35

Sociedade Brasileira de Matemtica

Note que SCM = +

pois ngulo inscrito. Note ainda que MOA AC e

SOA CP , pois so mediatrizes de AC e CP . Logo MCSOA inscritvel e MOA S =

Como SOA OAQ (pois SOA CP PO , logo PO // SOA ), QOAO = . Logo

OAQ SP CD CD = = = cos OAO = . OAO OAO 4OAO 4cos CD . Olhando para o tringulo OMA, MOA = Analogamente, OB O = 4cos
AC , logo MOA = AC 2 ), logo
OM OM = = cos OM = R cos . Analogamente, ON = R cos . Mas OA R OM R cos 4cos R cos ON = = = . Logo existe uma homotetia de CD OA O CD OB O 4cos centro O que leva M em OA e N em OB . Como homotetias preservam

(pois M ponto mdio da corda

paralelismo, MN // OAOB .
PROBLEMA 3

Pablo copia o seguinte problema: Considere todas as sucesses de 2004 nmeros reais ( x0 , x1 , x2 ,..., x2003 ), tais que

x0 = 1, 0 x1 2 x0 , 0 x2 2 x1 ,

0 x2003 2 x2002 .
Entre todas estas sucesses, determine aquela para a qual a expresso seguinte assume o seu maior valor: S = .
EUREKA! N18, 2003

36

Sociedade Brasileira de Matemtica

Quando Pablo ia copiar a expresso S, apagaram o quadro. S conseguia lembrarse de que S era da forma

S = x1 x2 ... x2002 + x2003 ,


onde o ltimo termo, x2003 , tinha coeficiente +1, e os anteriores tinham coeficiente +1 ou 1. Demonstre que Pablo, apesar de no ter o enunciado completo, pode determinar com certeza a soluo do problema.
SOLUO

Seja ci {1,1} o coeficiente associado ao termo xi na expresso de S. Em particular, c2003 = 1 . Dizemos que um termo xi positivo se ci = 1 , ou que negativo se ci = 1 . Dizemos que xi est maximizado se xi = 2 xi 1 .

Lema: Se xi +1 ,..., x j esto maximizados, ento:

ci xi + ci +1 xi +1 + ... + c j x j = xi (ci 20 + ci +1 21 + ... + c j 2 j i )


Prova: fcil ver que xk = 2
k i

xi , i k j. A prova segue trivialmente.


0 j i

Corolrio: A soma acima tem o mesmo sinal de c j . Prova: Como xi 0 , basta analisar o sinal de ci 2 + ... + c j 2
j j 1

. Suponha c j

k i k i j i j i j i positivo (i.e. = 1). Ento ck 2 2 + 2 = 2 + 1 + 2 = 1. O k =i k =i

caso c j = 1 anlogo. obvio que maximizar os termos positivos aumenta a soma (j que x y [0, 2 x] [0, 2 y ] , podemos aumentar um termo sem alterar nenhum dos seguintes). Logo, sem perda de generalidade, na seqncia que maximiza S todos os termos positivos so mximos. Se houver algum termo negativo no maximizado, escolha o ltimo deles, digamos xi e maximize tanto ele quanto todos os termos que o seguem. Sejam a e b os valores antigo e novo de Seja ainda C = 2 ci + 2 ci +1 + ... + 2
0 1
EUREKA! N18, 2003

xi e A e B os valores da soma
2003 i

ci +1 xi +1 + ... + c2003 x2003 antes e depois da mudana na seqncia. c2003 . Pelo lema, A = aC e B = bC.

37

Sociedade Brasileira de Matemtica

Logo B A = C(b a) > 0, pois, pelo corolrio, C > 0, e como b > a, b a > 0. Logo, na seqncia de S mximo, todos os termos so mximos ou poderamos aumentar S maximizando algum termo. Logo

x2 = 2 x1 = 2 2 ,..., xi = 2 xi 1 = 21 ,..., x2003 = 22003 , e 0 1 2003 ). portanto a seqncia que maximiza S (2 , 2 ,..., 2
SEGUNDO DIA
PROBLEMA 4

x1 = 2 x0 = 2,

Seja M ={1, 2,,49} o conjunto dos primeiros 49 inteiros positivos. Determine o maior inteiro k tal que o conjunto M tenha um subconjunto de k elementos em que no haja 6 nmeros consecutivos. Para esse valor mximo de k, encontre a quantidade de subconjuntos de m, de k elementos, que tenham a propriedade mencionada.

Definio: Um conjunto A M feliz se no contm seis inteiros consecutivos. Seja N M feliz, | N | 42. Ento se P = M N ,| P | 7. Logo a interseo de P com algum dos conjuntos

SOLUO

A1 = {1, 2,3, 4,5,6} , A2 = {7,8,9,10,11,12} , A3 = {13,14,15,16,17,18} A7 = {37,38,39, 40, 41, 42} , A8 = {43, 44, 45, 46, 47, 48, 49}
vazia. Chame de i algum inteiro tal que Ai P = . Ento Ai N , absurdo, pois ento N teria uma seqncia de seis inteiros consecutivos. Por outro lado, bvio que N = M {6,12,18, 24,30,36, 42, 48} feliz. Assim, o maior k tal que existe N M feliz 41 (como se | N | 42 ento N no feliz, | N | 41; e acabamos de exibir um exemplo para 41). Seja N M feliz, | N |= 41, P = M N = {n1 < n2 < ... < n8 }. Os oito elementos de P separam naturalmente N em nove conjuntos

N1 ,..., N9 , Ni N j = ,

= N , N i 5 e cada N i composto apenas

de nmeros consecutivos. fcil ver que o nmero de possveis conjuntos N o nmero de solues de | N1 | + | N 2 | +...+ | N 9 |= 41 (pois estamos escolhendo o
EUREKA! N18, 2003

38

Sociedade Brasileira de Matemtica

tamanho dos N i ' s ) onde | N i | 5 e exigimos que os Ni estejam ordenados pelos seus menores elementos. Seja ai =| N i | . Seja bi = 5 ai 5 bi = ai . Como 0 ai 5,0 bi 5. Substituindo na equao, Mas 5 b1 + 5 bi + ... + 5 b9 = 41 45 (b1 + ... + b9 ) = 41 b1 + ... + b9 = 4. ento a restrio bi 5 redundante, logo o nmero de N M ,| N |= 41, N feliz, o nmero de solues de b1 + ... + b9 = 4 nos inteiros no negativos, que

9 + 4 1 12 12 11 10 9 = 495. = = 24 4 4
PROBLEMA 5

No quadrado ABCD, sejam P e Q pontos pertencentes aos lados BC e CD respectivamente, distintos dos extremos, tais que BP = CQ. Consideram-se pontos X e Y, X Y, pertencentes aos segmentos AP e AQ respectivamente. Demonstre que, quaisquer que sejam X e Y, existe um tringulo cujos lados tm os comprimentos dos segmentos BX, XY e DY.
SOLUO

1 a X

B P

y 1

c D 1

Y Q C

Seja AB = BC = CD = DA = 1, BP = CQ = , BX = a, XY = b, YD = c.

EUREKA! N18, 2003

39

Sociedade Brasileira de Matemtica

Como

a + b y = BY , basta provar que

y > c para demonstrar que

a + b > c. Mas a mediatriz de BD a reta AC , que divide o plano em dois semiplanos B e D . exceo do ponto A, todo o segmento AQ est contido em , logo Y y > c (o caso Y A trivial). Logo a + b > c e, analogamente, c + b > a. Basta provar que a + c > b
Seja

A = (0,0); B = (1,0); C = (1,1); P = (1, ); Q = ( ,1); X = ( x, x); D = (0,1); y = ( y , y ) , onde + = 1 (para que BP = CQ ).

Ento a =

( x 1) 2 + 2 x 2 , c = 2 y 2 + ( y 1) 2 ,

b = ( x y ) 2 + ( x y ) 2 = x 2 2 xy + y 2 2 + 2 x 2 2 xy + y 2 = b = x 2 2 + y 2 2 + x 2 + y 2 2 xy ( + ) = x 2 2 + y 2 2 + ( x y ) 2 .
Mas b < a + c b < a + 2ac + c
2 2
2 2 2

(j que a, b, c so positivos). Basta

demonstrar que a + c b , j que 2ac > 0. Logo basta provar que

( x 1) 2 + 2 x 2 + ( y 1) 2 + 2 y 2 x 2 2 + y 2 2 + x 2 2 xy + y 2 x 2 2 x + 1 + y 2 2 y + 1 x 2 + 2 xy y 2 0 (1 x)(1 y ) 0. Mas x AP , logo X [0,1] 1 x [0,1]. Analogamente, 1 y [0,1] e o


resultado segue trivialmente.
PROBLEMA 6

Definem-se as sucesses ( an ) n 0 ,(bn ) n 0 por:

an +1 = a
SOLUO

2001 n

a0 = 1, b0 = 4 e 2001 + bn , bn +1 = bn + an para n 0.

Demonstre que 2003 no divide nenhum dos termos destas sucesses. Observe que 2003 primo; logo, se
2002 x 1(mod 2003) x 2001 x 1 (mod 2003). / 0 (mod 2003), x

EUREKA! N18, 2003

40

Sociedade Brasileira de Matemtica

Suponha

que

exista

tal

que

2003 | an +1bn +1

(note

que

a0b0 = 4 / 0(mod 2003) ); escolha o menor deles. Suponha que 2003 | an +1 (o


outro caso anlogo). Temos 0 an +1 an
1
2001

1 + bn an + bn (mod 2003),

logo an bn (mod 2003). Como nem an nem bn so zero (mod 2003), podemos inverter os dois lados, obtendo
1 an bn1 bn an (mod 2003) (usamos o fato de que (1) 1 1). Mas bn +1 bn1 + an an + an 0(mod 2003), logo 2003 | bn +1 2003 | an +1.

fcil ver que, nesse caso, an bn an ( an ) 1(mod 2003). Seja c = an 1 e

d = bn 1 (para n = 0, temos a1 = 12001 + 4 = 5 / 0(mod 2003) , logo podemos 1 1 supor n 1). Ento an c + d e bn c + d (mod 2003), logo

an bn c 1c + c 1d 1 + dc + dd 1 2 + cd + (cd ) 1 (mod 2003). (Como n 1 o menor possvel, 2003 no divide ai bi para todo i n , logo existe (cd ) ).
Seja x = cd.

1 2 + x + x 1 x 2 + 3x + 1 0(mod 2003). Esta equao tem uma raiz 2 (que cd), logo seu discriminante = 3 4 1 1 = 5 resduo quadrtico (mod
2003) : de fato,

(2 x + 3)2 = 4( x 2 + 3x + 1) + 5 5(mod 2003). Mas pela lei de Reciprocidade


20031 51 5 2003 2 2 Quadrtica, = (1) 2003 5 5 3 5 5 2002 = (1) (1) =1 =1, logo 5 no 2003 5 2003 2003 quadrtico mdulo 2003, absurdo! Assim, no possvel que 2003 | an +1bn +1 , logo {2003 x | x } ({ak }k {bk }k ) = .

resduo

Estas solues da Olimpada Ibero-americana de Matemtica - 2003 foram redigidas por Fbio Dias Moreira de Rio de Janeiro RJ.

EUREKA! N18, 2003

41

Sociedade Brasileira de Matemtica

A DESIGUALDADE DE ERDS-MORDELL
Anderson Torres, So Paulo - SP
Nvel Avanado
Neste artigo demonstraremos (vrias vezes) a desigualdade de Erds-Mordell e mostraremos uma bela aplicao na resoluo do problema 5 da IMO de 1996, realizada em Mumbai, ndia.

1- Uma histria do teorema Considere um tringulo ABC e um ponto P do mesmo plano. Sejam PA , PB , PC as projees ortogonais de P nos lados BC, CA, AB respectivamente. Vale ento a desigualdade:

2( PPA + PPB + PPC ) AP + BP + CP


com igualdade se e somente se P for o circuncentro de um tringulo ABC eqiltero. Este o enunciado da famosa Desigualdade de Erds-Mordell. Ela foi inicialmente conjecturada pelo matemtico hngaro Paul Erds e demonstrada no mesmo ano por Louis Mordell, na revista American Mathematical Monthly (problema n 3740). Logo aps surgiram vrias solues e alguns artigos sobre a desigualdade, cada uma usando variadas tcnicas: trigonometria (Louis J. Mordell e P.F. Barrow), desigualdades angulares e semelhanas (Leon Bankoff), teorema de Ptolomeu (Andr Avez e Hojoo Lee), reas de polgonos (V. Komornik). Mostraremos algumas delas, acrescidas de um pequeno comentrio:

Lema Importante: AP BC AB PPB + AC PPC ,


com igualdade se, e somente se,

PB PC // BC .

A esmagadora maioria das demonstraes difere apenas na demonstrao desta pequena desigualdade. Veja que esta desigualdade equivale a estas desigualdades (as outras duas seguem por permutao cclica das variveis):

EUREKA! N18, 2003

42

Sociedade Brasileira de Matemtica

AB AC PPB + PPC CB BC CB BA PPC + PPA BP AC CA CA CB PPA + PPB CP BA AB AP


Ao som-las, obtemos: CA BA CB AB AC BC AB + BP + CP + + + C PPA + PPB + PP BA CA AB CB BC AC e lembrando que a soma de um real positivo com seu inverso no pode ser menor que 2, e esse valor s se iguala a dois se o nmero em questo for 1 (isto conseqncia da desigualdade das mdias), a desigualdade segue, com igualdade se e apenas se AB = BC = CA. Alm disso, devemos ter PA PB // AB , PA P C // AC e PB PC ABC.

// BC , o que implica facilmente que P o circuncentro do tringulo

Vamos ento demonstrar este lema!

Demonstrao 1: (trigonometria)
O quadriltero APB PPC cclico, pois os ngulos retos so opostos e somam 180. Assim, pela Sagrada Lei dos Senos Generalizada, PB PC PB PC = AP = sen PB PPC sen BAC

AP BC = 2R PB PC
em que R o circunraio do tringulo ABC.

EUREKA! N18, 2003

43

Sociedade Brasileira de Matemtica

PB PC P B C

PA

Pela Sagrada Lei dos Cossenos, PB PC = PPB + PPC 2 PPB PPC cos (PB PPC )

= PPB + PPC 2 PPB PPC cos (PA PPC + PA PPB ) = ( PPB sen PB PPA + PPC sen PC PPA )

+( PPB cosPB PPA PPC cos PC PPA ) onde usamos o fato (bastante conhecido ): sen x + cos x = 1 para fatorar. Assim, jogando um dos parnteses fora, obtemos:
PB PC PPB sen PB PPA + PPC sen PC PPA PB PC PPB sen ACB + PPC sen ABC AP BC = 2 R PB PC PPB AB + PPC AC
A igualdade ocorre se, e somente se,
PPB cos PB PPA PPC cos PC PPA = 0 2 sen PAB cos ABC = 2 sen PAC cos ACB sen(PAB + ABC ) + sen(PAB ABC ) = sen(PAC + ACB ) + sen(PAC ACB );

mas PAB + ABC + PAC + ACB = 180 ,donde sen(PAB + ABC ) = sen(PAC + ACB), e logo sen(PAC ACB) = sen(PAB ABC ) PAB ABC = PAC ACB PAB PAC = ABC ACB
EUREKA! N18, 2003

44

Sociedade Brasileira de Matemtica

e (fica como exerccio mostrar que) isto equivale, de fato, a PB PC // BC .

Demonstrao 2: (reas de paralelogramos)


Escolha dois pontos B1 AC , C1 AB e construa os paralelogramos APC ' C1 e

APB ' B1 .

PB' , PC ' cortam BC em X,Y e B1C1 em X 1 , Y1 respectivamente, caso P seja


interno ao tringulo (mas isto no afeta muito a demonstrao). Veja que B1 B ' C ' C1 um paralelogramo.
A

PC P

PB

B C1 Y1

X X1

C B1

C'

B'

Por congruncias, [ AB1C1 ] = [ PB ' C ' ] , em que [algo] significa rea de algo. Agora, veja: [ AB1C1 ] [PX1Y1 ] + [C ' C1Y ] + [B ' B1 X1 ] = [ APC ' C1 ] + [ APB ' B1 ]

[PB ' C '] [ PX1Y1 ] + [C ' C1Y1 ] + [B ' B1 X1 ] = [ APC ' C1 ] + [ APB ' B1 ] [C 'Y1 X1B '] + [C ' C1Y1 ] + [B ' B1 X1 ] = [ APC ' C1 ] + [ APB ' B1 ]

[ APC ' C1 ] + [ APB ' B1 ] = [B1B ' C ' C1 ] Com isto vemos que AC1 PPC + AB1 PPB B1C1 C ' C1 = AP B1C1 , com igualdade se, e apenas se,

C ' C1 B1C1 , ou AP B1C1 , ou seja, AP contm o circuncentro do tringulo


EUREKA! N18, 2003

45

Sociedade Brasileira de Matemtica

AB1C1 . Fazendo AC1 = AC, AB1 = AB , teremos por congruncias (para variar...)

BC = C1 B1 , e (por paralelismo mesmo!) PB PC // BC , e pronto! Fim!


Observao: Veja que possvel modificar esta demonstrao apenas usando uma reflexo pela bissetriz do ngulo ABC para obter os pontos

AC1 = AC, AB1 = AB , B1 AC, C1 AB . Esta observao ser til mais tarde.
Demonstrao 3: (teorema de Ptolomeu)
Sejam B' , C ' pontos da reta ver que

PB PC tais que BB ' // CC ' PB PC . Ento fcil


,

BC B ' C ' = B ' PC + PC PB + PB C ' AP BC AP B ' PC + AP PC PB + AP C ' PB

com igualdade se, e somente se, PB PC // BC .


A B'

PC PB C' P

PA

Vamos calcular cada uma das parcelas em relao ao ponto P. Veja que APPC ~ CPB C ' pois os ngulos correspondentes so iguais. De fato, temos os ngulos retos, e APPC = APB PC = CPB C ' (quadriltero cclico e ngulos opostos pelo vrtice). Assim, obtemos as relaes:

EUREKA! N18, 2003

46

Sociedade Brasileira de Matemtica

C ' PB PC P = AP C ' PB = PC P PB C . PB C AP
Analogamente,

B ' PC PB P = AP B ' PC = PB P PC B PC B PA
Pelo Teorema de Ptolomeu-Euler,

P BP C AP = AP B PP C + AP C PP B
Adicionando as igualdades, obtemos:

AP BC AP ( B ' PC + PC PB + PB C ') = PC P PB C + PC P APB + PB P APC + PB P PC B = PPC AC + PPB AB


2- Problema 5, IMO 1996 (Mumbai, ndia)
Seja ABCDEF um hexgono convexo tal que AB paralelo a DE, BC paralelo a EF, e CD paralelo a FA. Sejam R A , RC , RE os circunraios dos tringulos FAB, BCD, DEF respectivamente, e seja P o permetro do hexgono. Prove que: R A + RC + RE P / 2 . Este foi um dos problemas mais difceis (e considerado o mais difcil por muitos problemistas) j propostos na histria da IMO. Para se ter uma idia, apenas seis participantes (dois romenos e quatro armnios) fecharam este problema, enquanto os seis estudantes da equipe chinesa zeraram-no! Mostraremos neste artigo duas solues. A primeira um esboo de como foi criado o problema, segundo a Banca Examinadora da IMO de 1996 (o problema foi proposto pela Armnia), segundo a referncia [Nairi M. Sedrakian, The History of a Creation of a 1996 IMO Problem, Mathematics Competitions, n 2 vol.9], e se assemelha muito com a soluo oficial, presente na Eureka! N 11. A segunda (com algumas modificaes), totalmente sinttica, considerada a mais bela das solues, de autoria de Ciprian Manolescu, da equipe da Romnia, o nico Perfect Score (tambm conhecido como Ouro-42) da IMO 1996.

EUREKA! N18, 2003

47

Sociedade Brasileira de Matemtica

Soluo 1: usaremos o seguinte lema (demonstre-o!): Considere um tringulo de circunraio R, lados a e b, e o ngulo entre eles.
Ento, para quaisquer , tais que < , > 0, + + = 2 , vlida a sen sen +b desigualdade: 2 R a sen sen

(Sugesto: note que sen = cos , sen = cos , e considere uma 2 2


com o lado b; calcule a medida da projeo 2 ortogonal do lado do tringulo oposto ao ngulo nessa reta). Usando este lema, podemos estimar os raios. Para tal sejam = FAB, = BCD, = DEF , FA = a, AB = b, BC = c, CD = d , DE = e, EF = f . Com isto, sen sen sen sen sen sen 2 RA b ; 2 RC c ; 2 RE e + a +d + f sen sen sen sen sen sen Vamos tentar obter outra estimativa para 2 RA , desta vez em relao aos lados d e e. Pela Sagrada Lei dos Senos, BF = 2 RA sen . Podemos ento escrever BF b sen + a sen . Veja que BF no pode ser menor que a distncia entre as retas BC e EF. Olhando este fato, vamos projetar o ponto A nas retas BC, EF obtendo os respectivos pontos ABC , AEF . Analogamente para o ponto D, obtemos o retngulo ABC AEF DEF DBC .
A BC B c C D BC d D

reta fazendo um ngulo

e a

A EF F f E D EF

EUREKA! N18, 2003

48

Sociedade Brasileira de Matemtica

Com isto, podemos escrever:

BF b sen + a sen = AABC + AAEF = ABC AEF = DBC DEF = DDBC + DDEF = d sen + e sen
Conclumos as seguintes desigualdades:
2 RA d sen sen sen sen sen sen + e + f + b ; 2 RC a ;2 RE c sen sen sen sen sen sen

Somando tudo:

4 ( RA + RC + RE ) sen sen (a + d ) + sen sen sen sen + (b + e) + sen sen sen sen + (c + f ) + sen sen
E o problema segue, aplicando a Desigualdade das Mdias aos parnteses e dividindo tudo por 4. E fim!

Soluo 2: Este problema, por si s, j incita o uso de Erds-Mordell ou de alguma generalizao conveniente (muito provavelmente at s ultimas conseqncias ()). Para tal, devemos de algum modo produzir a configurao deste teorema. Aproveitando o paralelismo, desenhe os paralelogramos MDEF, NFAB, PBCD. Com isto j temos algo dentro do hexgono (mesmo que no seja um ponto, como em Erds-Mordell, mas j alguma coisa... s vezes necessrio um pouco de coragem para no desistir de algumas idias, mesmo que paream no dar certo. Muitos problemas de IMO e vrios problemas difceis em geral so, na verdade, aplicaes de fatos simples at s ltimas conseqncias).

EUREKA! N18, 2003

49

Sociedade Brasileira de Matemtica


Y A B F M N P E X D Z C

O problema agora tentar achar um modo de identificar os raios. Lembrando que raios e dimetros tm tudo a ver com perpendicularidade, desenhe o tringulo XYZ, com XFY FN , YBZ BP, ZDX DM . Assim, o quadriltero FMDX inscritvel de dimetro MX. Mas os tringulos FED e FMD so congruentes, logo XM = 2 RA . Com isso o problema demonstrar a seguinte desigualdade: XM + YN + ZP BN + BP + DP + DM + FM + FN . Vamos dividir em dois casos: 1- M = N = P. E este caso a prpria Desigualdade de Erds-Mordell. 2- O tringulo MNP existe (no degenerado). A partir daqui vamos adaptar a demonstrao de Erds-Mordell. Estimaremos XM primeiro. Sejam Y e Z as reflexes dos pontos Y e Z em relao bissetriz de YXZ . Sejam G e H as projees de M, X em YZ respectivamente. Como [XYZ] = [YXZ] = [ZMY] + [XMZ] + [YMX], temos: YZ XH = YZ MG + ZX FM + XY DM . Mas, usando a desigualdade triangular no tringulo XMG e a desigualdade cateto < hipotenusa no tringulo XHG (ou mesmo distncia de X reta YZ), obtemos:

XM + MG XG XH XM XH MG

Substituindo na igualdade recm-descoberta, XY XZ XM DM + FM YZ YZ


EUREKA! N18, 2003

50

Sociedade Brasileira de Matemtica

Analogamente,

XY XZ DM + FM YZ YZ YX YZ YN BN + FN XZ XZ ZX ZY ZP BP + DP XY XY XM
Somando tudo:
XM + YN + ZP XY XZ DM + FM YZ YZ YZ YX + FN + BN XZ XZ ZX ZY + BP + DP XY XY

Agora falta pouco...Basta arranjar um modo de sumir com as fraes. Agora vamos usar a Desigualdade das Mdias para concluir. Para tal, outra estimativa. Primeiramente, veja que os tringulos XYZ e MNP so semelhantes, o que nos FM FN BN BP DP DM permite definir k = . Com isto, podemos = = YZ XY ZX escrever:
ZX YX BP + BN XY XZ XZ XY BN + BP = + + 2 XY XZ XZ XY BN BP 2 XY XZ XZ YZ XY YZ ( BP + BN ) k XZ XY

Analogamente,

EUREKA! N18, 2003

51

Sociedade Brasileira de Matemtica

XZ XY XZ YZ XY YZ BP + BN ( BP + BN ) k XY XZ XZ XY YX ZY XY XZ XZ YZ DM + DP ( DM + DP ) k ZY YX XY YZ XZ YZ YZ XY XZ XY FM + FN ( FM + FN ) k YZ XZ YZ XZ

Agora, basta somar estas desigualdades e acabamos o problema!!

REFERNCIAS :
[1]A demonstrao de Hojoo Lee pode ser encontrada na famosa revista Forum Geometricorum, a qual voc pode ler no site http://forumgeom.fau.edu .Neste artigo voc encontra as referncias de toda a histria deste problema enquanto ele se passava na American Mathematical Monthly. [2]Na lista de discusso de problemas da OBM (obm-l@mat.puc-rio.br; ver tambm www.obm.org.br/lista.htm) foi deixada, h algum tempo atrs, a demonstrao de

Ciprian Manolescu.

[3]Na Internet tem uns livros do Kiran Kedlaya.V ao site abaixo e faa o download: http://www.unl.edu/amc/a-activities/a4-for-students/problemtext/ Um deles trata sobre desigualdades, e outro sobre geometria euclidiana plana. Ainda tem uns dois livros com provas de algumas olimpadas de matemtica de vrias partes do mundo. [4]Aps uma longa caa achei este artigo,que trata de uma generalizao interessante: A weighted Erds-Mordell Inequality for Polygons. Este livro pode ser encontrado no endereo:

www.math.technion.ac.il/~shafrir/pub_ps/m18.ps.gz
[5]Um site de divulgao cientifica: http.://mathworld.wolfram.com [6]Rafael Tajra Fonteles - Trigonometria e desigualdades em problemas de olimpadas, Eureka! 11, p. 24-33. [7]A segunda demonstrao da desigualdade de Erds-Mordell tambm foi objeto de uma questo da fase final da OPM-2001. Confira no livro da OPM-2001 ou no site http://www.opm.mat.br/

EUREKA! N18, 2003

52

Sociedade Brasileira de Matemtica

COMO QUE FAZ?


PROBLEMA 3 PROPOSTO POR DAVI MXIMO ALEXANDRINO NOGUEIRA (FORTALEZA CE)

possvel escolher 102 subconjuntos com 17 elementos cada do conjunto {1,2,3...102} tais que a interseo de quaisquer 2 deles tem no mximo 3 elementos?
SOLUO

A ideia olhar para o plano projetivo P sobre Z/17Z, que o quociente de pela relao de equivalncia x ~ y y = a x, (Z/17Z)3\{(0,0,0)} 2 a 17 \ {0}. P tem 17 + 17 + 1 = 307 pontos e as retas em P tm 18 pontos cada (e duas delas sempre se intersectam num ponto). Vamos fazer uma espcie de quociente de P. Para isso, considere o isomorfismo linear T de (Z/17Z)3 dado por T(x, y, z) = (y, z, x). Note que T3 = Id. As retas em P so dadas por vetores no nulos w de (Z/17Z)3 (de fato por elementos de P): uma reta Rw o conjunto dos v tais que < v, w > = 0. Note que T tem um nico ponto fixo em P: o elemento v0 = [1:1:1], correspondente ao vetor (1,1,1). Jogamos v0 fora e dividimos os outros 306 pontos de P em 102 classes de equivalncia de 3 elementos (as orbitas de T): a classe de equivalncia de u {u, Tu, T(Tu)}. Temos 307 retas em P. Jogamos fora a Rv0 . O quociente de cada uma das outras retas tem 17 elementos. De fato, se Rw contm pontos da forma u e Tu, devemos ter <u, w> = 0 e < u, T*w> = <Tu, w> = 0. Como w no v0 ento T*w = T1(w) no mltiplo de w, donde h apenas um elemento de P satisfazendo essas duas igualdades, isto , apenas dois pontos em Rw so identificados pela nossa equivalncia, donde as retas (projetadas pelo quociente por essa relao de equivalncia) tm agora 17 pontos cada. Para cada reta na projeo, existem exatamente 3 retas (em P) que se projetam sobre ela: Rw, RTw e RT (Tw) . Dadas duas retas na projeo, elas se intersectam em (no mximo) 3 pontos: se elas so as projees de Rw e Rv, suas intersees sero as projees das intersees de Rw com Rv, de RTw com Rv e de RT(Tw) com Rv. Assim, na projeo (ou, se voc preferir, no quociente), temos 102 pontos e 102 retas, cada uma com 17 elementos, sendo que duas delas se intersectam em (no mximo) 3 pontos.

EUREKA! N18, 2003

53

Sociedade Brasileira de Matemtica

SOLUES DE PROBLEMAS PROPOSTOS


Publicamos aqui algumas das respostas enviadas por nossos leitores.

81) Num tringulo issceles ABC com AB = BC, temos AC = BH, onde BH a altura relativa ao lado AC. Traamos uma reta BD que corta o prolongamento da reta AC em D de tal forma que os raios dos crculos inscritos nos tringulos . ABC e CBD so iguais. Determine o ngulo ABD
SOLUO DE FRANCISCO JARDEL ALMEIDA MOREIRA (FORTALEZA CE)
B

O1

O2

Sejam, O1 o incentro do ABC , O2 o incentro do BCD , X = BC OO 1 2 ,

E = BO2 AD e finalmente, F o p da perpendicular de O2 at AD .


Note que OO 1 2 FH um retngulo, pois O1 H F = O2 FH = 90 e O1 H = O2 F , da O1O2 // AD. Agora observe que:

HCO1 + BCO1 + O2 CB + DCO2 = 180 ademais, HCO1 = BCO1 e O2 CB = DCO2


portanto, O1 CO2 = 90 . Do fato de O1 CO2 = 90 e O1O2 // AD, X mdio de

O1O2

(de fato,

X O1C = H CO1 = X CO1 , donde XC = XO1 e logo CX

mediana do tringulo retngulo O1CO2 ) mdio de HE. Logo, HB = AC HE = AC = HB.


EUREKA! N18, 2003

54

Sociedade Brasileira de Matemtica

Como BH E = 45 e assim ABD = 2 H BE = 90.

82) a) Demonstre a identidade

cos() cos(2) cos(4 )...cos(2n ) = cos(2 j ) =


j =0

sen(2n+1) 2n+1 sen()

b) Prove que

1 1 1 1 1 1 1 1 1 2 + + + ... = cos j +2 = . 2 2 2 2 2 2 2 2 2 2 j =0

SOLUO DE WALLACE ALVES MARTINS (RIO DE JANEIRO RJ)

a)

Demonstraremos a identidade utilizando o princpio da induo.

Para n = 0 temos cos e


k

sen(20+1) sen2 2sen cos = = = cos. . Assim que a 20+1sen 2sen 2sen

propriedade vlida para n = k, temos:

cos(2 j) =
j =0

sen(2k +1) . Multiplicando-se ambos os membros da identidade por 2k +1sen

cos(2k +1) temos:


sen(2 2k +1 ) k +1 sen(2k +1 )cos(2k +1 ) sen(2( k +1)+1 ) j 2 cos(2 ) = = = . 2k +1 sen 2k +1 sen 2( k +1)+1 sen j =0
Portanto a propriedade tambm vlida para n = k + 1. Logo, pelo princpio da induo segue-se que a identidade vlida n , n 0. b) Demonstraremos primeiramente a identidade entre o primeiro membro e o membro central da identidade acima; por induo. Para n = 0, temos que cos verdadeira para n = 0.

1 = cos = . Assim a propriedade 0+2 2 2 4


Vamos mostrar que para Para todo k temos vamos

k 1 1 1 1 1 1 1 1 1 + ... + + ... + = cos j +2 . 2 2 2 2 2 2 2 2 2 j =0 2 ( k +1) Radicais

isso,

EUREKA! N18, 2003

55

Sociedade Brasileira de Matemtica

mostrar que para todo k temos

1 1 1 1 1 + + ... + = cos k + 2 . Considere 2 2 2 2 2 2


( k +1) Radicais

x=

1 1 1 1 1 1 1 + + ... + temos que x2 = cos k +2 cos k +2 = 2x2 1. 2 2 2 2 2 2 2 2 2


( k +2) Radicais

Como sabemos

cos(2 ) = 2 cos 1. Logo


2

k +2 x = cos 2 2

= cos k +3 2

satisfaz a equao. Logo x =

1 1 1 1 1 + + ... + = cos (k +1)+2 . Portanto a 2 2 2 2 2 2


[( k +1)+1] Radicais

identidade tambm vlida para n = k + 1. Logo, pelo Princpio da Induo seguese que a identidade vlida n , n 0. Em particular, quando n temos a identidade conforme acima. Agora, sabemos que

cos 2
j =0

= cos cos cos ... cos n+2 = j +2 4 8 16 2

n = cos n+2 cos n+1 ... cos cos = cos(2 j ), onde = n + 2 . 2 2 2 8 4 j =0

sen 2 n +1 , n + 2 1 2 Utilizando (a) temos: cos(2 j ) = = . n +1 n +1 j =0 2 sen n + 2 2 sen n + 2 2 2 senx Sabemos que, quando n ento 2n+1 sen n+2 . (Pois lim = 1) . x 0 x 2 2
n

2 1 = . Logo: lim cos j +2 = lim n n 2n+1 sen j =0 2 n+ 2 2


n

EUREKA! N18, 2003

56

Sociedade Brasileira de Matemtica

83) Seja N = {0,1,2,3,...}. Determine quantas funes f : N N satisfazem f(2003) = 2003, f(n) 2003 para todo n 2003 e f(m + f(n)) = f(f(m)) + f(n), para todo m, n N. 84) Prove que se

A N* = {1,2,3,...} um conjunto no-vazio tal que

n A 4n A e n A ento A = N* . Obs. x x x o nico inteiro tal que x 1 <


85) Mostre que todo tringulo pode ser dividido em 9 pentgonos convexos de reas iguais. 86) Encontre todas as triplas de inteiros positivos (a, m, n) tais que am +1 divide

(a +1)n .
87) Seja a(1) = 1 e, para cada inteiro n 2, a(n) igual ao menor inteiro positivo
que no pertence a {a( j), j < n} tal que

a( j) seja mltiplo de n. Prove que


j =1

a(a(n)) = n para todo inteiro positivo n.


88) Prove que se r
e cos(r ) ento cos(r ) 1, ,0, ,1.

1 2

1 2

Seguimos aguardando as solues dos problemas: 83, 84, 85, 86, 87 e 88

EUREKA! N18, 2003

57

Sociedade Brasileira de Matemtica

PROBLEMAS PROPOSTOS
Convidamos o leitor a enviar solues dos problemas propostos e sugestes de novos problemas para os prximos nmeros.

89) Uma prova de mltipla escolha com n questes feita por k alunos. Uma resposta correta na i-sima questo vale pi pontos, onde pi um inteiro positivo, para 1 i n. A nota de cada aluno a soma dos pontos correspondentes s questes que ele acertou. Aps a realizao a prova, foi observado que, mudando os pesos pi, as notas dos alunos podem estar em qualquer uma das k! possveis ordens (em que no h duas notas iguais). Dado n, qual o maior valor possvel de k? 90) Prove que, para todo inteiro positivo n e para todo inteiro no nulo a, o polinmio xn + axn1 + axn2 + ... + ax 1 irredutvel, i.e., no pode ser escrito como o produto de dois polinmios no constantes com coeficientes inteiros. 91) Um jardinero deve construir um canteiro com a forma de setor circular. Ele dispe de 100 metros de fio para cerc-lo. Figura:
r

Qual deve ser o valor do raio do crculo para que o canteiro tenha rea mxima? Qual a rea mxima?

92) Seja (Fn)nN a seqncia de Fibonacci, definida por F1 = 1, F2 = 1 e

Fn+2 = Fn+1 + Fn , n . Prove que mdc (Fm , Fn ) = Fmdc(m,n) para quaisquer


inteiros positivos m e n.

EUREKA! N18, 2003

58

Sociedade Brasileira de Matemtica

93) Um inteiro positivo n dito perfeito se n igual soma dos divisores positivos de n que so menores que n. Prove que um nmero par n perfeito se e somente se existe um nmero primo p 2 tal que 2 p 1 primo e

n = 2p1 (2 p 1) .
94) A ilha das amazonas habitada por amazonas e homens. As amazonas mandam em tudo, so inteligentssimas, ciumentssimas e muito fofoqueiras. O que uma amazona mais gosta de fazer trair outra amazona com o marido desta. Consumada a traio, ela conta o seu feito a todas as amazonas da ilha menos amazona trada. As outras amazonas tambm no contam nada vtima da traio. Mas se uma amazona descobre que est sendo trada ela mata o seu marido na prxima meia noite. A rainha das amazonas, que viva, v esta situao com desagrado. Ela v que h traio na ilha mas, como nunca ningum descobre nada, nenhum marido morre. No dia 1 de janeiro de 3333, ento, contrariando a tradio, ela chama todas as amazonas para a praa central e faz uma proclamao solene: "H traio nesta ilha." Nenhuma amazona sonha em duvidar da palavra da rainha e todas as amazonas sabem disso. Como j foi dito, todas so inteligentes e ciumentas: estes e os outros fatos mencionados neste enunciado at aqui so conhecimento comum entre as amazonas. Supondo que haja 1000 amazonas na ilha e que 365 delas tenham sido tradas, o que acontecer? 95) "Resta-Um" um jogo de tabuleiro na qual as peas ocupam um tabuleiro formando parte de um reticulado retangular (na verdade, existem variaes em tabuleiros de reticulado triangular). O nico movimento permitido consiste em tomar duas peas em casas adjacentes vizinhas a uma casa vazia, e fazer a pea mais distante da casa vazia pular sobre a outra pea, ocupando a casa vazia. A pea pulada retirada.

(esse movimento pode ser feito para a direita, para a esquerda, para cima ou para baixo). Agora imagine um tabuleiro que um reticulado retangular infinito e uma reta que contm uma linha do reticulado, dividindo-o em dois lados. Todas as casas de um dos lados da linha esto vazias e cada casa do outro lado da linha pode ou no ter uma pea.
EUREKA! N18, 2003

59

Sociedade Brasileira de Matemtica

Quantas peas, no mnimo, precisamos para chegar a uma casa do lado vazio do tabuleiro, a uma distncia n da linha ? Abaixo indicamos uma casa a distncia n, para n = 1,2,3,4,5.

5 4 3 2 1

6320430 dgitos e foi descoberto por Michael Shafer, um participante do GIMPS (um projeto distribudo para procurar primos de Mersenne. Veja: http://www.mersenne.org para mais informaes). Agora so conhecidos 40 expoentes p para os quais 2 1 primo (e portanto 2
p

Voc sabia 20996011 Que 2 1 primo? Este o maior primo conhecido, tem

p 1

(2 p 1) perfeito - veja o

problema proposto 93): 2, 3, 5, 7, 13, 17, 19, 31, 61, 89, 107, 127, 521, 607, 1279, 2203, 2281, 3217, 4253, 4423, 9689, 9941, 11213, 19937, 21701, 23209, 44497, 86243, 110503, 132049, 216091, 756839, 859433, 1257787, 1398269, 2976221, 3021377, 6972593, 13466217 e 20996011.

Problema 89 proposto na 27 Olimpada Russa de Matemtica em 2001, problema 90 proposto na Olimpada Romena de Matemtica de 1992, problema 91 proposto por Osvaldo Mello Sponquiado (Ilha Solteira SP).

EUREKA! N18, 2003

60

Sociedade Brasileira de Matemtica

Voc sabia
Que existem infinitos inteiros positivos mpares k
n

tais que

k 2 + 1 composto para todo n ? Tais inteiros k so chamados nmeros de Sierpinski. Em 1962, John Selfridge provou que 78557 um nmero de Sierpinski, e conjectura-se que seja o menor deles. Atualmente h 11 nmeros menores que 78557 sobre os quais no se sabe se so nmeros de Sierpinski ou no: 4847, 10223, 19249, 21181, 22699, 24737, 27653, 28433, 33661, 55459 e 67607. O nmero 5359 fazia parte dessa lista at 6/12/2003, quando Randy Sundquist ( um participante do Seventeen or Bust, um projeto distribudo para atacar o problema de Sierpinski) encontrou o primo

5359 25054502 + 1, que tem 1521561 dgitos e o quarto maior


primo conhecido, e maior primo conhecido que no de Merssenne. Veja: http://www.seventeenorbust.com para mais informaes. Exerccio: Prove que 78557 um nmero de Sierpinski, e que existem infinitos nmeros de Sierpinski a partir das congruncias

78557 2 0 + 1 0 (mod 3) 78557 21 + 1 0 (mod 5) 78557 2 7 + 1 0 (mod 7) 78557 211 + 1 0 (mod 13)
78557 23 + 1 78557 239 + 1 0 (mod 73)

78557 215 + 1 0 (mod 19) 78557 2 27 + 1 0 (mod 37).

EUREKA! N18, 2003

61

Sociedade Brasileira de Matemtica

COORDENADORES REGIONAIS
Alberto Hassen Raad Amarsio da Silva Arajo Ana Paula Bernardi da Silva Benedito Tadeu Vasconcelos Freire Carlos Frederico Borges Palmeira Claus Haetinger Cleonor Crescncio das Neves lio Mega Florncio Ferreira Guimares Filho Gil Cunha Gomes Filho Ronaldo Alves Garcia Reginaldo de Lima Pereira Ivanilde Fernandes Saad Jacqueline Fabiola Rojas Arancibia Janice T. Reichert Joo Bencio de Melo Neto Joo Francisco Melo Libonati Jos Carlos dos Santos Rodrigues Jos Cloves Saraiva Jos Gaspar Ruas Filho Jos Luiz Rosas Pinho Jos Vieira Alves Licio Hernandes Bezerra Luzinalva Miranda de Amorim Mrio Rocha Retamoso Marcelo Rufino de Oliveira Marcelo Mendes Pablo Rodrigo Ganassim Ramn Mendoza Ral Cintra de Negreiros Ribeiro Reinaldo Gen Ichiro Arakaki Ricardo Amorim Srgio Cludio Ramos Tadeu Ferreira Gomes Toms Menndez Rodrigues Valdenberg Arajo da Silva Valdeni Soliani Franco Vnia Cristina Silva Rodrigues Wagner Pereira Lopes (UFJF) (UFV) (Universidade Catlica de Braslia) (UFRN) (PUC-Rio) (UNIVATES) (UTAM) (Colgio Etapa) (UFES) (Colgio ACAE) (UFGO) (Escola Tcnica Federal de Roraima) (UC. Dom Bosco) (UFPB) (UNOCHAPEC) (UFPI) (Grupo Educacional Ideal) (Unespar) (UFMA) (ICMC-USP) (UFSC) (UFPB) (UFSC) (UFBA) (UFRG) (Grupo Educacional Ideal) (Colgio Farias Brito, Pr-vestibular) (Liceu Terras do Engenho) (UFPE) (Colgio Anglo) (INPE) (Centro Educacional Logos) (IM-UFRGS) (UEBA) (U. Federal de Rondnia) (U. Federal de Sergipe) (U. Estadual de Maring) (U. Metodista de SP) (CEFET GO) Juiz de Fora MG Viosa MG Braslia DF Natal RN Rio de Janeiro RJ Lajeado RS Manaus AM So Paulo SP Vitria ES Volta Redonda RJ Goinia GO Boa Vista RR Campo Grande MS Joo Pessoa PB Chapec SC Teresina PI Belm PA Campo Mouro PR So Luis MA So Carlos SP Florianpolis SC Campina Grande PB Florianpolis SC Salvador BA Rio Grande RS Belm PA Fortaleza CE Piracicaba SP Recife PE Atibaia SP SJ dos Campos SP Nova Iguau RJ Porto Alegre RS Juazeiro BA Porto Velho RO So Cristovo SE Maring PR S.B. do Campo SP Jata GO

EUREKA! N18, 2003

62

CONTEDO

AOS LEITORES XXV OLIMPADA BRASILEIRA DE MATEMTICA Problemas e Solues da Primeira Fase XXV OLIMPADA BRASILEIRA DE MATEMTICA Problemas e Solues da Segunda Fase XXV OLIMPADA BRASILEIRA DE MATEMTICA Problemas e Solues da Terceira Fase XXV OLIMPADA BRASILEIRA DE MATEMTICA Problemas e Solues da Primeira Fase Nvel Universitrio XXV OLIMPADA BRASILEIRA DE MATEMTICA Problemas e Solues da Segunda Fase Nvel Universitrio XXV OLIMPADA BRASILEIRA DE MATEMTICA Premiados AGENDA OLMPICA COORDENADORES REGIONAIS

2 4

17 32

49

54

63

67 68

Sociedade Brasileira de Matemtica

AOS LEITORES
Neste nmero apresentamos os problemas e solues da XXV Olimpada Brasileira de Matemtica, realizada durante o ano passado. A seguir o discurso do Prof. Jacob Palis Jr. na premiao da XXV Olimpada Brasileira de Matemtica, realizada na VII Semana Olmpica, na cidade de Belo Horizonte - MG em janeiro de 2004. Os editores. Antes de apresentar breves, e certamente apaixonadas palavras sobre as Olimpadas Brasileiras de Matemtica, quero registrar a minha admirao por um dos seus grandes e talvez o maior de seus precursores: o professor Shigeo Watanabe. Embora fsico devido a ele uma pioneira e ampla atividade de Olimpadas de Matemtica no Estado de So Paulo, com o apoio da Academia de Cincias de So Paulo e da Secretaria Estadual de Educao de So Paulo. Seu exemplo, mais do que qualquer outro, inspirou a todos ns. Tambm congratulo-me com todos os alunos premiados, seus professores e familiares. A Olimpada Brasileira de Matemtica OBM existe desde 1979. Segundo o prprio relatrio de atividades 1997 2003 do Programa Nacional de Olimpadas de Matemtica, que gentilmente foi preparado a meu pedido por Nelly Carvajal e Sonia de Souza Silva de Melo, a OBM at ento, era relativamente limitada devido escassez de recursos e sua influncia na melhoria do ensino resultava consideravelmente menor que o almejado. J desenvolvia, no entanto, um extraordinrio trabalho na busca de jovens talentos para a Matemtica ou cincias afins. Basta citar a impressionante lista, certamente incompleta, de excelentes matemticos que da resultaram: Edson de Faria (USP), Nicolau Saldanha (PUCRio), Pedro Paulo Schimer (USP), Eduardo Esteves (IMPA), Ralph Costa Teixeira (FGV-Rio), Carlos Moreira, (Gugu) (IMPA), Eduardo Laber (PUC-Rio), Daniel Tausk (USP), Artur Avila (CNRS, Frana). A OBM caracterizou-se sempre pela extrema dedicao de seus dirigentes, aliada competncia, bom gosto e f inquebrantvel quanto aos seus benefcios, no s para a comunidade matemtica, mas para a sociedade em geral. Infelizmente, ao lado do idealismo dos olmpicos, nem sempre foi possvel participar da Olimpada Internacional de Matemtica, com sua equipe completa, por falta de recursos. Houve ocasies em que alguns de ns, matemticos j estabelecidos, cotizamos a passagem de um ou mais brasileiros, de excepcional qualificao, para possibilitar a prticipao do Brasil na Olimpada Internacional. Em 1997, sonhei, j h anos totalmente convencido da importncia das Olimpadas, ser possvel modificar radicalmente a situao. Conversei bastante
EUREKA! N19, 2004

Sociedade Brasileira de Matemtica

com Gugu, Nicolau, Elon Lima, Eduardo Wagner, Augusto Morgado, Paulo Czar Pinto Carvalho, dentre outros. Da, com minha convico e paixo em nveis elevados, parti para o convencimento da Diretoria do CNPq, sob a Presidncia de Jos Galizia Tundisi. A receptividade quanto importncia de um novo Programa Nacional de Olimpadas de Matemtica foi excepcionalmente entusistica. Nasceu a uma nova etapa da OBM, agora sim ampla e permanente de to importante atividade. Os recursos multiplicaram-se consideravelmente, indo de muito pouco a cerca de R$200.000 nesta transio e a R$400.000 agora. Com o entusiasmo renovado e at ampliado de seus dirigentes, ouso dizer que o Programa Nacional de Olimpadas de Matemtica tornou-se eternamente robusto. No mais possvel pensar seno em crescer, fortalecer-se tcnica e administrativamente e contribuir decisivamente para o formao de uma ampla e slida competncia nacional em matemtica, passando por uma almejada incluso cientfica. A OBM hoje uma atividade da Sociedade Brasileira de Matemtica, compartilhada com o Instituto Nacional de Matemtica Pura e Aplicada IMPA e, a partir de 2001, com o Instituto do Milnio Avano Global e Integrado da Matemtica Brasileira, (IM-AGIMB). Nesta nova etapa, a participao do Brasil em Olimpadas Internacionais cresceu exponencialmente incluindo alm da Olimpada Internacional, a Ibero-americana, a Olimpada de Maio, e a do Cone Sul, alm de Olimpadas Regionais. Foi criada em 1998 a Eureka!, excelente publicao dedicada principalmente aos alunos e professores da escola secundria e editada trs vezes ao ano. Multiplicou-se o incentivo realizao de Olimpadas Regionais e ao fortalecimento das coordenaes regionais. O treinamento de alunos e professores em diversos nveis passou a ser atividade permanente. A melhoria do ensino de matemtica nas escolas tornou-se um objetivo exequvel e contnuo. Criou-se um Banco de Questes e Biblioteca o um site interativo, assim como uma secretaria permanente no IMPA. Estabeleceu-se a Semana Olmpica, como atividade anual, ocasio em que h um intenso treinamento dos alunos premiados com medalhas de Ouro, Prata, Bronze e Menes Honrosas. Aps cerca de sete anos, deixo a Presidncia da Comisso de Olimpadas da SBM, muito feliz pelas conquistas que vocs obtiveram e com a certeza absoluta de que muito mais ser alcanado, de forma permanente. Lugares como Ribeiro Preto, Uberaba e Uberlndia e tantos outros de Norte a Sul e de Leste a Oeste do pas devem fazer parte do mapa da OBM. Sonhem muito e partam para sua realizao. Estarei sempre com vocs Jacob Palis Jnior

EUREKA! N19, 2004

Sociedade Brasileira de Matemtica

XXV OLIMPADA BRASILEIRA DE MATEMTICA


Problemas e Solues da Primeira Fase
PROBLEMAS NVEL 1 1. Onze cubinhos, todos de mesma aresta, foram colados conforme a figura a seguir.

O menor nmero de cubinhos, iguais aos j utilizados, que devem ser agregados ao slido formado pelos onze cubinhos para obtermos um cubo macio igual a: A) 48 B) 49 C) 52 D) 53 E) 56 2. Na tabela a seguir vemos o consumo mensal de gua de uma famlia durante os 5 primeiros meses de 2003. Meses Janeiro Fevereiro Maro Abril Maio Consumo (m3) 12,5 13,8 13,7 11,4 12,1 E) 317,5 m3

O consumo mensal mdio dessa famlia durante os 5 meses foi: A) 11,3 m3 B) 11,7 m3 C) 12,7 m3 D) 63,5 m3

3. Voc possui muitos palitos com 6 cm e 7 cm de comprimento. Para fazer uma fila de palitos com comprimento total de 2 metros, o nmero mnimo de palitos que voc precisa utilizar : A) 29 B) 30 C) 31 D) 32 E) 33

EUREKA! N19, 2004

Sociedade Brasileira de Matemtica

4. Em um quadrado mgico, a soma dos nmeros de cada linha, coluna ou diagonal sempre a mesma. No quadrado mgico a seguir, o valor de x :

1 26 A) 20 B) 22

14

x 13

C) 23

D) 25

E) 27

5. Considere um nmero inteiro x e faa com ele as seguintes operaes sucessivas: multiplique por 2, some 1, multiplique por 3 e subtraia 5. Se o resultado for 220, o valor de x : A) um nmero primo. B) um nmero par. C) um nmero entre 40 e 50. D) um nmero mltiplo de 3. E) um nmero cuja soma dos algarismos 9. 6. Escreva os nmeros de 0 a 9 nos crculos ao lado, de forma que eles cresam no sentido antihorrio. Em seguida, subtraia 1 dos nmeros mpares e some 1 aos nmeros pares. Escolhendo trs crculos consecutivos, qual a maior soma que se pode obter? A) 19 B) 21 C) 23 D) 24 E) 25 7. O retngulo da figura a seguir est dividido em 7 quadrados. Se a rea do menor quadrado igual a 1, a rea do retngulo igual a:

A) 42

B) 44

C) 45

D) 48

E) 49

EUREKA! N19, 2004

Sociedade Brasileira de Matemtica

8. Considere a seqncia oscilante: 1, 2, 3, 4, 5, 4, 3, 2, 1, 2, 3, 4, 5, 4, 3, 2, 1, 2, 3, 4, O 2003o termo desta seqncia : A) 1 B) 2 C) 3 D) 4 E) 5 9. Joo disse para Maria: Se eu lhe der um quarto do que tenho, voc ficar com metade do que vai me sobrar. Maria acrescentou: E eu lhe daria 5 reais, se lhe desse a metade do que tenho. Juntos, os dois possuem: A) 80 reais B) 90 reais C) 100 reais D) 120 reais E)130 reais 10. Uma escola precisa comprar mesas e cadeiras novas para seu refeitrio, cada mesa com 4 cadeiras, que sero distribudas nos 3 setores do refeitrio. Em cada setor do refeitrio cabem 8 fileiras de mesas e, em cada fileira, cabem 14 mesas. Quantas mesas e cadeiras devero ser compradas? A) 112 mesas e 448 cadeiras B) 112 mesas e 1344 cadeiras C) 336 mesas e 448 cadeiras D) 336 mesas e 896 cadeiras E) 336 mesas e 1344 cadeiras 11. As 4 coloraes a seguir so consideradas iguais por coincidirem por rotao.

De quantos modos diferentes possvel colorir as casas de um tabuleiro 2 2 de branco ou preto de modo que no existam dois tabuleiros que coincidam por rotao? A) 4 B) 5 C) 6 D) 7 E) 8 12. Numa festa tpica, cada prato de arroz foi servido para duas pessoas, cada prato de maionese para trs pessoas, cada prato de carne servia quatro pessoas e cada prato de doces dava exatamente para cinco pessoas. Foram utilizados 77 pratos e todas as pessoas se serviram de todos os pratos oferecidos. Quantas pessoas havia na festa? A) 20 B) 30 C) 45 D) 60 E) 75

EUREKA! N19, 2004

Sociedade Brasileira de Matemtica

13. Na organizao retangular de pontos da figura abaixo, a distncia entre pontos vizinhos em uma mesma linha ou coluna igual a 1 cm.
C D

E A

A rea do pentgono ABCDE , em cm2, igual a: A) 9 B)

19 2

C) 10

D)

21 2

E) 11

14. Um quadrado de rea 1 foi cortado em cinco filas de 5 quadradinhos cada. Todos os quadradinhos so congruentes. Marcam-se os quadradinhos de uma linha qualquer, de uma diagonal qualquer e de uma coluna qualquer, e, em seguida, retiram-se os quadrados assinalados. A rea coberta pelos quadradinhos restantes vale, no mnimo,

2 5 13 D) 25
A)

11 25 3 E) 5
B)

C)

12 25

15. Um trofu formado por cinco recipientes cbicos foi construdo da seguinte maneira: sob o cubo de lado 10 cm foi soldado o cubo de lado 20 cm, sob este foi soldado o cubo de lado 30 cm, e assim por diante. Toda a superfcie externa desse trofu dever ser coberta com um certo tipo de revestimento. Quantos metros quadrados desse revestimento sero necessrios? A) 1,5 B) 2,5 C) 2,7 D) 2,75 E) 3
EUREKA! N19, 2004

Sociedade Brasileira de Matemtica

16. Num certo aeroporto, Nelly caminhava calmamente razo de um metro por segundo; ao tomar uma esteira rolante de 210 metros, Nelly continuou andando no mesmo passo e notou ter levado um minuto para chegar ao fim da esteira. Se Gugu ficar parado nesta esteira, quanto tempo levar para ser transportado? A) 1min20s B) 1min24s C) 1min30s D) 1min40s E) 2min 17. Uma certa mquina tem um visor, onde aparece um nmero inteiro x, e duas teclas A e B. Quando se aperta a tecla A o nmero do visor substitudo por 2x + 1. Quando se aperta a tecla B o nmero do visor substitudo por 3x 1. Se no visor est o nmero 5, apertando alguma seqncia das teclas A e B, o maior nmero de dois algarismos que se pode obter : A) 85 B) 87 C) 92 D) 95 E) 96 18. A seqncia 22 descreve a si mesma, pois ela formada por exatamente dois 2. Analogamente, a seqncia 31 12 33 15 descreve a si mesma, pois formada por exatamente trs 1, um 2, trs 3 e um 5. Qual das seguintes seqncias no descreve a si mesma? A) 21 32 23 16 B) 31 12 33 18 C) 31 22 33 17 19 D) 21 32 33 24 15 E) 41 32 23 24 15 16 18 19. Camila e Lara esto disputando o seguinte jogo num tabuleiro 4 4: Camila marca algumas casas do tabuleiro e informa Lara o nmero de casas marcadas na vizinhana de cada casa do tabuleiro. Neste jogo, duas casas distintas so consideradas vizinhas se possuem um lado ou um canto (vrtice) em comum. Lara deve descobrir quais casas foram marcadas por Camila. Aps marcar algumas casas, Camila passou para Lara o seguinte tabuleiro:
1 0 2 1 2 2 3 0 1 1 3 2 1 2 1 1

O nmero de casas marcadas foi: A) 3 B) 4

C) 5

D) 6

E) 7

EUREKA! N19, 2004

Sociedade Brasileira de Matemtica

20. Imagine uma pilha com cem milhes de folhas de papel sulfite, cada uma com 0,1 milmetro de espessura. Assinale a alternativa mais prxima da altura da pilha. A) a sua altura. B) o comprimento do maior animal do mundo, a baleia azul, que cerca de 29 metros. C) a altura do edifcio mais alto do mundo, o Petronas Tower, que tem 88 andares. D) a altura do pico mais alto do mundo, o Monte Everest, que 8848 metros. E) a distncia do planeta Terra Lua, que muito maior que todas as alternativas anteriores.

PROBLEMAS NVEL 2 1. 2. Veja o problema No. 7 do Nvel 1. Veja o problema No. 3 do Nvel 1.

3. A maior raiz da equao (x 37)2 169 = 0 : A) 39 B) 43 C) 47 4. Veja o problema No. 17 do Nvel 1. 5. Veja o problema No. 4 do Nvel 1.

D) 50

E) 53

6. Seja n = 9867. Se voc calculasse n3 n2 voc encontraria um nmero cujo algarismo das unidades : A) 0 B) 2 C) 4 D) 6 E) 8 7. Na figura, o nmero 8 foi obtido somando-se os dois nmeros diretamente abaixo de sua casinha. Os outros nmeros nas trs linhas superiores so obtidos da mesma forma. Qual o valor de x? 42

8 3 A) 7
EUREKA! N19, 2004

x C) 5

6 D) 4 E) 6

B) 3

Sociedade Brasileira de Matemtica

8. Veja o problema No. 5 do Nvel 1. 9. Os nmeros a, b, e c so naturais consecutivos em ordem crescente. Ento, o 2 valor de c ab igual a: A) 0 B) 1 C) 2a + b D) 2a + c E) 2b + c 10. Veja o problema No. 8 do Nvel 1. 11. Considere as seguintes definies: A mdia aritmtica de dois nmeros reais positivos a metade da sua soma. A mdia harmnica de dois nmeros reais positivos o inverso da mdia aritmtica dos inversos desses nmeros. A diferena entre a mdia aritmtica e a mdia harmnica dos nmeros 4 e 6 : A) 0,1 B) 0,2 C) 0,3 D) 0,4 E) 0,5 12. Veja o problema No. 18 do Nvel 1. 13. O domin mais conhecido tem como maior pea o duplo 6. Neste domin so empregadas 28 peas diferentes. Quantas peas tem o domin cuja maior pea o duplo 8? A) 34 B) 36 C) 42 D) 55 E) 45

14. Os quadrados dos nmeros naturais maiores do que 2, subtrados de seus sucessores, formam a seqncia 5, 11, 19, ... . O primeiro elemento dessa seqncia que no um nmero primo o: A) quarto B) dcimo C) sexto D) nono E) stimo 15. Voc est em um pas estrangeiro, a LUCINIA, e no conhece o idioma, o LUCIANS, mas sabe que as palavras BAK e KAB significam sim e no, porm no sabe qual qual. Voc encontra uma pessoa que entende portugus e pergunta: "KAB significa sim?" A pessoa responde KAB. Podese deduzir que: A) KAB significa sim. B) KAB significa no. C) A pessoa que respondeu mentiu. D) A pessoa que respondeu disse a verdade. E) No possvel determinar sem um dicionrio LUCIANS-PORTUGUS.

EUREKA! N19, 2004

10

Sociedade Brasileira de Matemtica

16. Veja o problema No. 13 do Nvel 1. 17. Veja o problema No. 11 do Nvel 1. 18. O valor da soma A)

1 3

2 2003 91001 2 2002 91001 : + 41001 32003 41001 32003 2 B) C) 1 3

D)

4 3

E) 2

19. Considere os nmeros X = 2 700 , Y = 11200 e Z = 5 300 . Assinale a alternativa correta: A) X< Z< Y B) Y<X<Z C) Y<Z<X D) Z<X<Y E) Z<Y<X 20. Beatriz, Isabele e Nicole esto disputando um jogo fazendo lanamentos sucessivos com uma moeda. Beatriz ganha se, em dois lanamentos consecutivos, o primeiro resultar cara e o segundo coroa. Isabele ganha se forem obtidas duas coroas em dois lanamentos consecutivos, e Nicole ganha se forem obtidas duas caras em dois lanamentos consecutivos. Elas fazem os lanamentos at que uma das jogadoras seja vencedora. Qual(is) jogadora(s) possui(em) menos chances de ganhar o jogo? A) Beatriz B) Isabele C) Nicole D) Beatriz e Nicole E) As trs tm a mesma chance. 21. Veja o problema No. 19 do Nvel 1. 22. Divida os nmeros 2, 3, 5, 7, 11, 13 e 17 em dois grupos x e y com produtos A e B, respectivamente, de modo que A B = 1. A soma dos algarismos de A : A) 10 B) 11 C) 13 D) 14 E) 15 23. A figura a seguir mostra um quadrado ABCD e um tringulo eqiltero BEF, ambos com lado de medida 1cm . Os pontos A, B e E so colineares, assim como os pontos A, G e F.
D G C F

EUREKA! N19, 2004

11

Sociedade Brasileira de Matemtica

A rea do tringulo BFG , em cm 2 : A)

1 4

B)

1 3

C)

3 4

D)

3 12

E)

3 10

24. Carlinhos pensa num nmero mpar positivo menor do que 100. Pedrinho se dispe a descobrir que nmero esse fazendo a seguinte pergunta, quantas vezes forem necessrias: O nmero que voc pensou maior, menor ou igual a x ? . Note que x um nmero que Pedrinho escolhe. Quantas perguntas desse tipo Pedrinho poder ter que fazer at descobrir o nmero pensado por Carlinhos? A) 5 B) 7 C) 15 D) 25 E) 45 25. No tringulo ABC, AB = 20, AC = 21 e BC = 29. Os pontos D e E sobre o lado BC so tais que BD = 8 e EC = 9. A medida do ngulo DE, em graus, igual a: A) 30 B) 40 C) 45 D) 60 E) 75

PROBLEMAS NVEL 3 1. O nmero 19AB, onde A e B so dgitos, um quadrado perfeito. O valor de

AB da raiz quadrada do nmero cuja representao decimal AB :


A) 5 B) 6 C) 7 D) 8 E) 9 2. Veja o problema No. 8 do Nvel 1. 3. Veja o problema No. 19 do Nvel 1. 4. Cinco amigos, Arnaldo, Bernaldo, Cernaldo, Dernaldo e Ernaldo, devem formar uma fila com outras 30 pessoas. De quantas maneiras podemos formar esta fila de modo que Arnaldo fique na frente de seus 4 amigos? (Obs.: Os amigos no precisam ficar em posies consecutivas.) 35! 35! 163 35 A) 35! B) C) D) E) e 5! 5! 5 5 5. A Revoluo Francesa, em 1789, trouxe muitas mudanas na humanidade. Em 1791, aps a Revoluo Francesa, a Academia Francesa de Cincias props um novo sistema de medidas. Esse sistema era baseado numa
EUREKA! N19, 2004

12

Sociedade Brasileira de Matemtica

medida natural de comprimento, chamada metro, que foi definida como um dcimo de milionsimo da distncia do Plo Norte ao Equador, medida em torno da circunferncia do meridiano que passa por Paris. Tal sistema foi efetivamente adotado em 1795. A definio atual do metro diferente mas o valor aproximadamente o mesmo. Considerando os fatos acima, qual a ordem de grandeza do volume do planeta Terra, em metros cbicos? Obs.: Nesta questo voc pode querer utilizar a frmula do volume V da esfera, V = A) 1016

4 R3 , onde R o raio da esfera. 3


B) 1021 C) 1026 D) 1031 E) 1036

6. Na seqncia de Fibonacci 1, 1, 2, 3, 5, 8, 13, 21, 34, 55, cada termo, a partir do terceiro, igual soma dos dois termos anteriores. Quanto vale a soma infinita
1 1 2 3 5 8 13 21 34 55 + + + + + + + + + + 2 4 8 16 32 64 128 256 512 1024

,

onde o n-simo termo o n-simo termo da seqncia de Fibonacci dividido por 2n? A) 3/2 B) 2 C) 5/2 D) 3 E) 1 + 5
2

7. O grfico de y = x2 5 x + 9 rodado 180o em torno da origem. Qual a equao da nova curva obtida? A) y = x2 + 5 x + 9 D) y = x2 5 x + 9 B) y = x2 5 x 9 E) y = x2 5 x 9 C) y = x2 + 5 x 9

8. Um clube de tnis tem n jogadores canhotos e 2n jogadores destros e, ao todo, h menos do que 20 jogadores. No ltimo campeonato interno, no qual cada jogador enfrentou cada um dos outros jogadores do clube exatamente uma vez, a razo entre o nmero de jogos vencidos por jogadores canhotos e o nmero de jogos vencidos por jogadores destros foi 3 : 4. Qual o valor de n? A) 3 B) 4 C) 5 D) 6 E) So necessrias mais informaes.

EUREKA! N19, 2004

13

Sociedade Brasileira de Matemtica

9. A figura abaixo mostra duas retas paralelas r e s. A reta r tangente s circunferncias C1 e C3, a reta s tangente s circunferncias C2 e C3 e as circunferncias tocam-se como tambm mostra a figura.
r s

C1 C2 C3

As circunferncias C1 e C2 tm raios a e b, respectivamente. Qual o raio da circunferncia C3? A) 2 a 2 + b 2 E) 2b a 10. Veja o problema No. 18 do Nvel 1. 11. A funo f definida para todos os pares ordenados (x; y) de inteiros positivos e tem as seguintes propriedades: (x + y)f(x; y) = (2x + y)f(x; x + y). f(x; x) = x, f(x; y) = f(y; x), Qual o valor de f(21; 12)? A) 12. 13. 14. 15. 16. 17. 18.
7 4

B) a + b

C) 2 ab

D)

4ab a +b

B)

4 7

C)

11 6

D)

6 11

E)

1 2003

Veja o problema No. 14 do Nvel 2. Veja o problema No. 15 do Nvel 2. Veja o problema No. 20 do Nvel 2. Veja o problema No. 22 do Nvel 2. Veja o problema No. 23 do Nvel 2. Veja o problema No. 12 do Nvel 1. Veja o problema No. 24 do Nvel 2.

EUREKA! N19, 2004

14

Sociedade Brasileira de Matemtica

19. Dois amigos, Augusto e Eduardo, atravessavam uma ponte onde passava uma linha frrea. Quando tinham percorrido dois quintos da ponte, ouviram o barulho de um trem que se aproximava por trs deles. Apavorados, comearam a correr, cada um para o seu lado. Tiveram sorte: Augusto, que tinha voltado, conseguiu sair da ponte no exato instante em que o trem nela ia entrar. Por sua vez, Eduardo, que continuou para a frente, conseguiu sair da ponte no instante em que o trem tambm ia faz-lo. Refeitos do susto, quando se encontraram, comentaram que isto s foi possvel porque correram a 15 km/h e o trem estava a x km/h. O valor de x : A) 30 B) 45 C) 60 D) 75 E) 90 20. Seja N o menor inteiro positivo que pode ser escrito como a soma de 9, 10 e 11 inteiros positivos consecutivos. A soma dos algarismos de N igual a: A) 9 B) 18 C) 22 D) 27 E) 30 21. O maior inteiro que no supera A) 4 B) 6

32003 + 22003 igual a: 32001 + 22001


C) 7 D) 8 E) 9

22. Seja T = (a, b, c) tal que existe um tringulo ABC cujas medidas dos lados sejam BC = a, CA = b e AB = c satisfazendo c b a > 0 e a + b > c . Definimos T 2 = (a 2 , b 2 , c 2 ) e T = ( a, b, c) respectivamente, o quadrado e a raiz quadrada do "tringulo" T. Considere ento as afirmativas: 1) O quadrado de um tringulo equiltero equiltero. 2) O quadrado de um tringulo retngulo no um tringulo. 3) T 2 um tringulo se, e somente se, T acutngulo. 4) T sempre um tringulo para todo T. 5) Todos os ngulos de T so agudos. O nmero de afirmativas verdadeiras : A) 1 B) 2 C) 3 como sendo,

D) 4

E) 5

23. Em um quadro negro escreve-se o nmero 1. As nicas alteraes permitidas so substitu-lo pelo seu dobro ou pelo seu quadrado. Qual o maior nmero que pode ser obtido aps efetuarmos 2003 alteraes? A) 22003
EUREKA! N19, 2004

B) 42002

C) 2(2

4006

D) 2

(2 )
2003

E) 2

(2 )
2002

15

Sociedade Brasileira de Matemtica

24. Se f : uma funo tal que, para todo x , f ( x )( f ( x) x ) = 0 , ento A) f a funo nula. B) f a funo identidade, ou seja, f(x) = x para todo x real C) f a funo nula ou a funo identidade D) H 4 possveis funes f E) H infinitas funes f 25. Veja o problema No. 25 do Nvel 2.

GABARITO
NVEL 1 (5a. e 6a. sries) 1) D 6) C 2) C 7) C 3) A 8) C 4) E 9) B 5) A 10) E NVEL 2 (7a. e 8a. sries) 11) C 12) D 13) B 14) C 15) C 16) B 17) D 18) D 19) B 20) D

1) C 2) A 3) D 4) D 5) E

6) C 7) E 8) A 9) E 10) C

11) B 12) D 13) E 14) C 15) D

16) B 17) C 18) C 19) C 20) B

21) B 22) C 23) D 24) A 25) C

NVEL 3 (Ensino Mdio) 1) B 6) B 2) C 7) E 3) B 8) C 4) C 9) C 5) B 10) D

11) D 12) C 13) D 14) B 15) C

16) D 17) D 18) A 19) D 20) B

21) D 22) E 23) E 24) E 25) C

EUREKA! N19, 2004

16

Sociedade Brasileira de Matemtica

XXV OLIMPADA BRASILEIRA DE MATEMTICA


Problemas e Solues da Segunda Fase
PROBLEMAS NVEL 1 PARTE A (Cada problema vale 3 pontos)
01. Quantas vezes aparece o algarismo 9 no resultado da operao 10100 2003? 02. Quantos nmeros inteiros maiores do que 20032 e menores do que 20042 so

mltiplos de 100?
03. Quantos tringulos existem cujos lados

esto sobre alguns dos segmentos traados na figura ao lado?

04. Um estudante, com muito tempo livre e muita curiosidade, resolveu fazer o

seguinte: a cada minuto, ao mudar o horrio em seu relgio digital, marcava em seu caderno um X para cada algarismo que aparecia no visor. Assim, se seu relgio mostrava  ele marcava X e quando seu relgio mostrou  ele marcou XX. Comeou a fazer isso quando seu relgio mostrava  e parou quase doze horas depois, quando o relgio mostrava   . Calcule a metade da quantidade de X que ele marcou em seu caderno.

05. A grande atrao do OBM Parque uma roda gigante (a figura mostra uma roda gigante similar, porm com um nmero menor de cabines). As cabines so numeradas com 1, 2, 3,, no sentido horrio. Quando a cabine 25 est na posio mais baixa da roda-gigante, a de nmero 8 est na posio mais alta. Quantas cabines tem a roda-gigante?

EUREKA! N19, 2004

17

Sociedade Brasileira de Matemtica

06. Anos bissextos so mltiplos de 4, exceto aqueles que so mltiplos de 100

mas no de 400. Quantos anos bissextos houve desde a Proclamao da Repblica, em 1889, at hoje?
07. Em um dado comum a soma dos pontos sobre faces opostas sempre 7. Beatriz

construiu uma torre com 4 dados comuns iguais, colando as faces como mostrado na figura. Qual o menor nmero de pontos que Beatriz pode obter somando todos os pontos das dezoito faces da superfcie da torre?

08. Na multiplicao a seguir a, b, c e d so algarismos.

Calcule b + c + d.

45 a3 3bcd

09. A mdia de cinco inteiros positivos diferentes 11. Determine o maior valor possvel para o maior dos cinco inteiros.

EUREKA! N19, 2004

18

Sociedade Brasileira de Matemtica

10. Nove peas diferentes de domin esto sobre uma mesa, parcialmente cobertos

por um pedao de papel. Os domins se tocam de modo que 1 ponto vizinho a 1 ponto, 2 pontos so vizinhos a 2 pontos, etc. Qual o total de pontos escondidos pelo papel?

PROBLEMAS NVEL 1 PARTE B (Cada problema vale 10 pontos)


PROBLEMA 1

Quais nmeros inteiros positivos menores que 120 podem ser escritos como soma de duas ou mais potncias distintas de base 3 e exponente positivo? Por exemplo, 12 = 32 +31 um nmero deste tipo mas 18 = 32 + 32 no .
PROBLEMA 2

No desenho ao lado, o quadrado ABCD tem rea de 64 cm2 e o quadrado FHIJ tem rea de 36 cm2. Os vrtices A, D, E, H e I dos trs quadrados pertencem a uma mesma reta. Calcule a rea do quadrado BEFG.

EUREKA! N19, 2004

19

Sociedade Brasileira de Matemtica

PROBLEMA 3

Considere o produto de todos os divisores positivos de um nmero inteiro positivo, diferentes desse nmero. Dizemos que o nmero poderoso se o produto desses divisores for igual ao quadrado do nmero. Por exemplo, o nmero 12 poderoso, pois seus divisores positivos menores do que ele so 1, 2, 3, 4 e 6 e 1 2 3 4 6 = 144 = 122 . Apresente todos os nmeros poderosos menores do que 100.

PROBLEMAS NVEL 2
PROBLEMA 1

No desenho ao lado, o quadrado ABCD tem rea de 30 cm2 e o quadrado FHIJ tem rea de 20 cm2. Os vrtices A, D, E, H e I dos trs quadrados pertencem a uma mesma reta. Calcule a rea do quadrado BEFG.

PROBLEMA 2

Dados os nmeros inteiros de 1 a 26, escolha 13 dentre eles de forma que: 1) O nmero 4 est entre os nmeros escolhidos. 2) Nenhum nmero escolhido divisor de outro nmero escolhido.

PROBLEMA 3

Uma folha retangular ABCD de rea 1000 cm2 foi dobrada ao meio e em seguida desdobrada (segmento MN); foi dobrada e desdobrada novamente (segmento MC) e finalmente, dobrada e desdobrada segundo a diagonal BD. Calcule a rea do pedao de papel limitado pelos trs vincos (regio escura no desenho).

EUREKA! N19, 2004

20

Sociedade Brasileira de Matemtica

PROBLEMA 4

Veja o problema No. 3 do Nvel 1 Parte B.


* x y quaisquer Seja f : #* + , + # + , uma funo tal que f ( x ) f ( y ) f ( xy ) = y x que sejam os reais no nulos x e y . (a) Calcule f(1) (b) Encontre uma frmula para f(x)

PROBLEMA 5

PROBLEMA 6

Dizemos que um nmero N de quatro algarismos biquadrado quando igual soma dos quadrados de dois nmeros: um formado pelos dois primeiros algarismos de N, na ordem em que aparecem em N e o outro, pelos dois ltimos algarismos de N, tambm na ordem em que aparecem em N. Por exemplo, 1233 biquadrado pois 1233 = 122 + 332. Encontre um outro nmero biquadrado. Observao: Lembre-se de que um nmero de quatro algarismos no pode comear com zero.

PROBLEMAS NVEL 3
PROBLEMA 1

No tringulo ABC, M o ponto mdio do lado AC, D um ponto sobre o lado BC tal que AD bissetriz do ngulo BC e P o ponto de interseo de AD e BM. Sabendo que a rea de ABC 100, AB = 10 e AC = 30, calcule a rea do tringulo APB.
PROBLEMA 2

Veja o problema No. 6 do Nvel 2


PROBLEMA 3

Entre 15 nmeros reais distintos, o menor deles igual a 1, no h trs que podem ser lados de um tringulo. Quais valores o maior dos 15 nmeros pode assumir?
PROBLEMA 4

O tringulo ABC retngulo em A. Dentre os pontos P pertencentes ao permetro do tringulo, encontre aquele que minimiza a soma AP + BP + CP.
EUREKA! N19, 2004

21

Sociedade Brasileira de Matemtica

PROBLEMA 5

Um quadrado de lado 3 dividido em 9 quadrados de lado unitrio, formando um quadriculado. Cada quadrado unitrio pintado de azul ou vermelho. Cada cor tem probabilidade

1 de ser escolhida e a cor de cada quadrado escolhida 2

independentemente das demais. Qual a probabilidade de obtermos, aps colorirmos todos os quadrados unitrios, um quadrado de lado 2 pintado inteiramente de uma mesma cor?

PROBLEMA 6

Calcule a soma

3
k =0

2k +1
2
k

+1

21 22 23 24 + + + + 31 + 1 32 + 1 34 + 1 38 + 1

 + 32 + 1
n +1 2
n

Solues Nvel 1 Segunda Fase Parte A


Problema Resposta 01 98 02 40 03 17 04 66 05 34 06 27 07 58 08 15 09 45 10 22

Solues Nvel 1 Segunda Fase Parte B


SOLUO DO PROBLEMA 1:

Temos 31 = 3, 32 = 9, 33 = 27, 34 = 81 mas 35 = 243 (no serve) . Assim, nmeros obtidos de acordo com as condies do problema so: 3 + 9 =12, 3 + 27 = 30, 3 + 81 = 84, 9 + 27 = 36, 9 + 81 = 90, 27 + 81 = 108, 3 + 9 + 27 = 39, 3 + 9 + 81 = 93, 3 + 27 + 81 = 111, 9 + 27 + 81 = 117. Note que o nmero 3 + 9 + 27 + 81 = 120 no serve.
SOLUO DO PROBLEMA 2:

os

Os tringulos ABE e EHF so retngulos em A e H, respectivamente; a medida do ngulo BF de 90o; se a medida do ngulo HF x, ento a medida dos ngulos e AB 90o x e, conseqentemente, a medida do ngulo ABE x; como EFH BE = EF (so lados do mesmo quadrado), ento os tringulos mencionados so congruentes (pelo caso ALA de congruncia de tringulos). Utilizando o teorema de Pitgoras, podemos escrever BE 2 = AB 2 + AE 2 , o que mostra que a rea do quadrado BEFG a soma das reas dos quadrados ABCD e FHIJ, ou seja, 64 + 36 = 100 cm2.
EUREKA! N19, 2004

22

Sociedade Brasileira de Matemtica

SOLUO DO PROBLEMA 3:

Os divisores positivos de um nmero inteiro N so d 1 , d 2 , d 3 , , d k , tais que dk = N e podemos observar que 1 = d1 d 2 d 3

1 N = d 2 d k 1 = d 3 d k 2 etc. Por exemplo, os divisores positivos de 12 so


1, 2, 3, 4, 6 e 12, de forma que 112 = 26 = 34. Note que ao excluir os divisores 1 e 12, restam 2, 3, 4 e 6, cujo produto 2346 = (26)(34) = 1212 = 122. Assim, conclumos que o produto dos divisores positivos de um inteiro, excluindo 1 e o prprio nmero, igual ao quadrado do nmero se, e somente se, o nmero
2 tem 6 divisores. Portanto, o nmero da forma p5 ou p q , onde p e q so nmeros primos positivos, distintos. Se o nmero positivo menor do que 100, temos as 16 seguintes possibilidades:

2 5 = 32 2 2 3 = 12 2 2 5 = 20 2 2 7 = 28 2 11 = 44
2

3 2 2 = 18 3 2 5 = 45 3 2 7 = 63 3 2 11 = 99
52 2 = 50 52 3 = 75
7 2 2 = 98

2 2 13 = 52 2 2 17 = 68 2 2 19 = 76 2 2 23 = 92

Solues Nvel 2 Segunda Fase


SOLUO DO PROBLEMA 1:

Os tringulos ABE e EHF so retngulos em A e H, respectivamente; a medida do de 90; se a medida do ngulo BEF ngulo HEF x, ento a medida dos ngulos EFH e AEB 90 x e, conseqentemente, a medida do ngulo x; como BE = EF (so lados do ABE mesmo quadrado), ento os tringulos mencionados so congruentes (pelo caso ALA de congruncia de tringulos).
EUREKA! N19, 2004

B F J

23

Sociedade Brasileira de Matemtica

Utilizando o teorema de Pitgoras, podemos escrever BE 2 = AB 2 + AE 2 , o que mostra que a rea do quadrado BEFG a soma das reas dos quadrados ABCD e FHIJ, ou seja, 30 + 20 = 50cm2.
SOLUO DO PROBLEMA 2:

Todo nmero inteiro positivo n pode ser escrito na forma 2a b, a 0, b > 0 e b mpar (chamamos b de parte mpar de n). Considere dois nmeros com a mesma parte mpar: n1 = 2a1 b e n2 = 2a2 b . Supondo, sem perda da generalidade, que se a1 < a2 , ento teremos que n1 divisor de n2 . Assim, como de 1 a 26 temos 13 partes mpares possveis, a saber: 1, 3, 5, 7, 9, 11, 13, 15, 17, 19, 21, 23 e 25, cada um dos nmeros deve ter uma parte mpar diferente. Mais ainda, considerando que 1 divide todos os nmeros inteiros, o nmero com parte mpar 1 o que deve ter maior a. Porm 4 = 22 1 e est entre os nmeros escolhidos, logo para os demais nmeros escolhidos devemos ter a = 0 ou a = 1. E podemos determinar todas as escolhas possveis: 3 divisor de 9; 15 e 21. Logo 2 3 = 6,9,15 e 21 devem estar na nossa escolha. 5 divisor de 15 e 25. Logo 2 5 = 10 e 25 devem estar na nossa escolha. 7 divisor de 21. Logo 2 7 = 14 deve estar na nossa escolha. Com parte mpar 11 podemos escolher 11 ou 22 e com parte mpar 13, 13 ou 26. As demais escolhas so 17, 19 e 23. Portanto as escolhas possveis so (ordenadas segundo a parte mpar): 4; 6; 10; 14; 9; 11 ou 22; 13 ou 26; 15; 17; 19; 21; 23; 25.
SOLUO DO PROBLEMA 3:

Vamos usar a notao [X] para denotar a rea do polgono X.

M E P F

Sejam E e F os pontos de interseo como mostrados na figura. Sejam AB = 2a e BC = 2b. Ento AM = MB = DN = NC = a e ME = EN = b. Trace AN e seja P o
EUREKA! N19, 2004

24

Sociedade Brasileira de Matemtica

ponto de interseo dos segmentos AN e BD. Os segmentos AN e MC so paralelos (pois AM = NC e AM || NC). Como M ponto mdio de AB e MF || AP, temos que F o ponto mdio do segmento PB. Analogamente P o ponto mdio do segmento DF. Segue ento que DP = PF = FB. Por simetria verificamos que PE = EF e ento EF/FB = 1/2. Portanto, podemos escrever: [ MEF ] = 1/ 2. [ MBF ] 1 1 125 2 cm e Mas, por outro lado, [ MBE ] = [ ABD] = 125 , donde [ MEF ] = 125 = 3 3 4 2 250 2 [ MBF ] = 125 = cm . 3 3
SOLUO DO PROBLEMA 4:

Veja a soluo do problema No. 3 do Nvel 1 Parte B.


SOLUO DO PROBLEMA 5:

(a) Fazendo x = y = 1, obtemos [ f (1)]2 f (1) = 2, donde, resolvendo a equao, obtemos f(1) = 2 ou f(1) = 1. Este ltimo valor no serve, pois o contra-domnio da funo o conjunto dos nmeros reais estritamente positivos. Portanto, f(1) = 2. Fazendo 1 na identidade do problema obtemos 1 f ( x ) f (1) f ( x) = x + . Substituindo o valor de f(1), obtemos a frmula para x 1 f(x): f ( x ) = x + . x y
SOLUO DO PROBLEMA 6:

Vamos separar o nmero de quatro dgitos em duas partes: os dois primeiros dgitos, da esquerda para a direita, formam o nmero x e os dois restantes formam o nmero y. Ento a propriedade significa que 100 x + y = x 2 + y 2 . Esta igualdade pode ser considerada uma equao do segundo grau em x: x2 100x + y2 y = 0. Resolvendo encontramos x = 50 2500 ( y 2 y ).
EUREKA! N19, 2004

(3) (4)

25

Sociedade Brasileira de Matemtica

Com o exemplo do enunciado, y = 33 resulta em x = 12 com o sinal () na expresso: x = 50 1444 = 50 38 = 12. Naturalmente outra soluo aparece quando colocamos o sinal (+) na mesma expresso: x1 = 50 + 1444 = 50 + 38 = 88. Ento outro nmero com a mesma propriedade 8833 = 882 + 332.

Solues Nvel 3 Segunda Fase


SOLUO DO PROBLEMA 1:
A

P B D C

As alturas que passam por B dos tringulos ABC e ABM so iguais a distncia d de B reta AC, logo AM d rea ABM 1 1 AM 1 2 = = = rea ABM = rea ABC = 100 = 50. AC d rea ABC 2 2 AC 2 2 rea ABP BP Analogamente, . Pelo Teorema das Bissetrizes, = rea ABM BM 3 BP AB 10 2 = = = PM = BP 2 PM AM 15 3 Logo
BP BP BP BP rea ABP 2 2 2 = = = = = rea ABP = rea ABM = 50 = 20. rea ABM BM BP + PM BP + 3 BP 5 BP 5 5 5 2 2
EUREKA! N19, 2004

26

Sociedade Brasileira de Matemtica

SOLUO DO PROBLEMA 2:

Veja a soluo do problema No. 6 do Nvel 2.


SOLUO DO PROBLEMA 3:

Sejam a, b, c reais positivos tais que a b c . Esses nmeros so medidas dos lados de um tringulo se, e somente se, c < a + b . Ou seja, no so se, e somente se, c a + b. Assim, sendo 1 = x1 < x2 < x3 < x4 < ... < x15 os nmeros dados, devemos ter: x3 x2 + x1 x4 x3 + x2 x15 x14 + x13 De fato, esse sistema de desigualdades equivale a no haver trs que podem ser lado de um tringulo. Observe que se, i < j < k , xk < x j + xi , ento xk < xk 1 + xk 2 . Considere a seqncia de Fibonacci ( F0 = 0, F1 = 1 e Fn +2 = Fn +1 + Fn , n 0), x3 x2 + x1 ; x4 x3 + x2 x2 + x1 + x2 = 2 x2 + x1 ; x5 x4 + x3 2 x2 + x1 + x2 + x1 = 3 x2 + 2 x1 ; x6 x5 + x4 3x2 + 2 x1 + 2 x2 + x1 = 5 x2 + 3x1 ; parece que xn Fn 1 x2 + Fn 2 x1 e, com efeito, xk + 2 xk +1 + xk Fk x2 + Fk 1 x1 + Fk 1 x2 + Fk 2 x1 = Fk +1 x2 + Fk x1 Portanto, sendo x2 = 1 + , > 0, x15 F14 x2 + F13 x1 = 377 (1 + ) + 233 1 = 610 + 377 . Como podemos tornar to pequeno quanto queiramos, o maior dos 15 nmeros pode assumir qualquer valor real maior do que 610.

EUREKA! N19, 2004

27

Sociedade Brasileira de Matemtica

SOLUO DO PROBLEMA 4:

Sejam a, b, c as medidas dos segmentos BC, AC e AB, respectivamente. Consideraremos separadamente os casos em que P est em AC, em AB e em BC. Se P est em AC, ento AP + CP = b. Ento, minimizar AP + BP + CP reduz-se a minimizar BP. Isso ocorre quando P coincide com A, pois a menor distncia entre um ponto e uma reta determinada pelo p da perpendicular traada a partir desse ponto. Nesse caso o valor mnimo de AP + BP + CP b + c. O caso em que P est em AB inteiramente anlogo. Suponha, agora, que P est em BC. Eto BP + CP = a, ou seja, minimizar AP + BP + CP reduz-se a minimizar AP. Isso ocorre quando AP perpendicular a BC. Essa medida est representada por d no diagrama ao lado. Nesse caso, o mnimo de AP + BP + CP a + d. Assim, para completar a resoluo da questo, basta comparar a + d e b + c. Temos, ento, vrias maneiras de concluir a resoluo. Uma maneira: bc ad Observe que = bc = ad e a 2 = b2 + c 2 . 2 2 Logo (a + d )2 = a 2 + 2ad + d 2 = b2 + c2 + 2bc + d 2 = (b + c)2 + d 2 e, como d 2 > 0,( a + d ) 2 > (b + c ) 2 a + d > b + c. Outra maneira: d = c sen ; b = a sen .

a b

a b d

A c B

Logo (a + d ) (b + c ) = a + c sen a sen c = (a c )(1 sen ) > 0, isto , a + d > b + c. Resposta: O ponto que minimiza AP + BP + CP P = A (nesse caso AP + BP + CP = b + c).

EUREKA! N19, 2004

28

Sociedade Brasileira de Matemtica

SOLUO DO PROBLEMA 5:

O quadrado de lado 2 pode ser

ou

ou

ou

A probabilidade de cada um desses quadrados de lado 2 ser inteiramente de uma mesma cor 1 2 . Observe que todos os quatro quadrados unitrios devem ser da mesma 2 cor azul ou vermelho. Os demais quadradinhos podem ser de qualquer cor. Algumas configuraes so consideradas pelo menos 2 vezes:
Probabilidade 2
4

ou

1 2

ou

ou

1 Probabilidade 2 2

ou

Algumas configuraes so consideradas pelo menos 3 vezes:

EUREKA! N19, 2004

29

Sociedade Brasileira de Matemtica

ou

ou
Probabilidade 2

1 2

ou

E as configuraes com todos azuis ou todos vermelhos so contadas 4 vezes 1 (probabilidade: ). 2 Pelo Princpio da Incluso-Excluso, a probabilidade pedida : 95 1 1 1 1 1 4 2 2 2 4 2 + 4 2 2 = . 2 2 2 2 2 256
SOLUO DO PROBLEMA 6:
4 7 6 8 9 9

Analisando casos pequenos: 21 2 2 21 1 1 = = = 31 + 1 4 4 31 + 1 1 2 2 2 36 4 22 + = = = 1 1 31 + 1 32 + 1 40 40 (31 + 1) (32 + 1) 21 22 23 3272 8 23 1 1 + + = = = 31 + 1 32 + 1 34 + 1 3280 3280 (31 + 1) (32 + 1) (34 + 1) (Observe que no compensaria simplificar as fraes. Isso comum quando queremos descobrir um padro.) Parece ento, que podemos conjecturar que n 2k +1 2n+1 1 = n 2k +1 (31 + 1) (32 + 1)(34 + 1)...(32 + 1) k =0 3 Simplificando um pouco essa expresso antes de tentar demonstr-la.

EUREKA! N19, 2004

30

Sociedade Brasileira de Matemtica

(31 1)(31 + 1)(32 + 1)(34 + 1)...(32 + 1) = 31 1 n n n+1 (32 1)(32 + 1)(34 + 1)...(32 + 1) (34 1)(34 + 1)...(32 + 1) 32 1 . = = ... = 2 2 2 n 2k +1 2n+1 2 n+ 2 Ou seja, 2 k = 1 2n+1 = 1 2n+1 +1 3 1 3 1 k =0 3 2 Podemos agora demonstrar nossa conjuectura pelo uso direto do Princpio da Induo Finita ou considerando que, se descobrirmos f(k) tal que 2k +1 = f (k + 1) f (k ), k 32 + 1 n n 2k +1 = [ f (k + 1) f (k )] = 2k + 1 k =0 k =0 3
n

(31 + 1)(32 + 1)(34 + 1)...(32 + 1) =


n

f (1) f (0) + f (2) f (1) + ... + f (n + 1) f (n) = f (n + 1) f (0) .) k 32 + 1 Levando em conta novamente nossa conjectura, podemos inferir que 2k +1 e, de fato, f (k ) = 2 k 3 1 f (k + 1) f (k ) = Portanto n 2k +1 2k +2
k +1

(f a "integral discreta" de

2k +1

32 1 32 1
k

2k +1

2k +2 + 2k +1 (32 + 1)
k

(32 + 1)(32 1)
k k

2k +1 (32 + 1 2)
k

(32 + 1)(32 1)
k k

2k +1 32 + 1
k

3
k =0

+1

= f (n + 1) f (0) =

2n + 2 32
n+1

n 21 2k +1 2n +2 . 20 =1 2n+1 2k 3 1 1 3 1 +1 k =0 3

EUREKA! N19, 2004

31

Sociedade Brasileira de Matemtica

XXV OLIMPADA BRASILEIRA DE MATEMTICA


Problemas e Solues da Terceira Fase
PROBLEMAS NVEL 1
PROBLEMA 1:

Quantos inteiros positivos menores que 1000 tm a soma de seus algarismos igual a 7?
PROBLEMA 2:

Considere as seqncias de inteiros positivos tais que cada termo mais a soma dos seus algarismos igual ao termo seguinte. Por exemplo: 6, 12, 15, 21, 24, 30, 33, 39 uma seqncia nessas condies. Escreva a maior seqncia cujo ltimo termo 103 e que satisfaz tais condies. Observao: maior seqncia aquela com o maior nmero de termos.
PROBLEMA 3:

Os nmeros 1, 2, 4, 8, 16, 32, 64, so potncias de 2. Deseja-se dividir um quadrado de lado 2003 em outros quadrados cujos lados so potncias de 2. Mostre uma maneira de se fazer a diviso e obter 6364 quadrados cujos lados so potncias de 2.
PROBLEMA 4:

a) Dois quadrados esto posicionados de modo que o centro do primeiro vrtice do segundo, como mostra a figura abaixo.

Se o lado do primeiro quadrado mede 12cm, quanto mede a rea comum aos dois quadrados? b) Na figura a seguir, o paralelogramo tem lados de medida 12cm e 4cm e rea 40cm2. Sejam P, Q, R e S os centros dos quadrados construdos externamente
EUREKA! N19, 2004

32

Sociedade Brasileira de Matemtica

sobre os quatro lados desse paralelogramo. Sabendo que o quadriltero PQRS um quadrado, calcule a sua rea.

PROBLEMA 5:

Queremos construir o permetro de um retngulo utilizando 2003 varetas cujas medidas so inteiros positivos. Para isso s vezes teremos de quebrar algumas delas, mas todas as varetas e pedaos de varetas devem ser utilizados na construo do retngulo. a) Mostre que com uma nica quebra nem sempre possvel construir o retngulo. b) Mostre que com duas quebras sempre possvel construir o retngulo.

PROBLEMAS NVEL 2
PROBLEMA 1:

Num tabuleiro 2 2, como o mostrado a seguir, escreveremos nmeros inteiros de 1 a 9 obedecendo seguinte regra: A > B, C > D, A > C e B > D.

A C

B D

a) Quantos tabuleiros diferentes existem tais que B = C? b) Quantos tabuleiros diferentes existem no total?
PROBLEMA 2:

Determine o menor nmero primo positivo que divide x2 + 5x + 23 para algum inteiro x.

EUREKA! N19, 2004

33

Sociedade Brasileira de Matemtica

PROBLEMA 3:

O tringulo ABC est inscrito na circunferncia S e AB < AC. A reta que contm A e perpendicular a BC encontra S em P (P A). O ponto X situa-se sobre o segmento AC e a reta BX intersecta S em Q (Q B). Mostre que BX = CX se, e somente se, PQ um dimetro de S.
PROBLEMA 4:

Mostre que x 2 + 4 y 2 4 xy + 2 x 4 y + 2 > 0, quaisquer que sejam os reais x e y.


PROBLEMA 5:

So dados: uma circunferncia K e um ponto A interior, fixo, distinto do centro. Determine os pontos B, C e D sobre a circunferncia de forma que a rea do quadriltero ABCD seja a maior possvel.
PROBLEMA 6:

H N cidades na Tumblia. Cada duas cidades desse pas so ligadas por uma rodovia ou uma ferrovia, no existindo nenhum par de cidades ligadas por ambos os meios. Um turista deseja viajar por toda a Tumblia, visitando cada cidade exatamente uma vez, e retornar a cidade onde ele comeou sua jornada. Prove que possvel escolher a ordem na qual as cidades sero visitadas de modo que o turista mude o meio de transporte no mximo uma vez.

PROBLEMAS NVEL 3
PROBLEMA 1: Veja o problema 2 do Nvel 2. PROBLEMA 2:

Seja S um conjunto de n elementos. Determine o menor inteiro positivo k com a seguinte propriedade: dados quaisquer k subconjuntos distintos A1 , A2 ,..., Ak de S, existe uma escolha adequada dos sinais + e de modo que + S = A1 A2 ... Ak , onde Ai = Ai e Ai = S Ai o complementar de Ai em relao a S.
PROBLEMA 3:

Seja ABCD um losango. Sejam E, F, G e H pontos sobre os lados AB, BC, CD e DA, respectivamente, e tais que as retas EF e GH so tangentes circunferncia inscrita no losango. Prove que as retas EH e FG so paralelas.
EUREKA! N19, 2004

34

Sociedade Brasileira de Matemtica

PROBLEMA 4: Veja o problema 5 do Nvel 2. PROBLEMA 5:

Suponha que f : (0, +) # satisfaz: i) ii) x < y f ( x) < f ( y )

2 xy f ( x ) + f ( y ) , para todo x, y (0, +). f 2 x+ y Prove que existe x0 (0, +) tal que f ( x0 ) < 0.
PROBLEMA 6:

Um grafo cujo conjunto de vrtices V tem n elementos bacana se existir um 2 conjunto D  e uma funo injetiva f : V 1, n 4  tal que os vrtices p e q so ligados por uma aresta se e somente se f ( p ) f ( q) D. Mostre que existe n0  tal que para todo n n0 existem grafos com n vrtices que no so bacanas.

Observao: Um grafo com conjunto de vrtices V um par (V, E) onde E um conjunto de subconjuntos de V, todos com exatamente dois elementos. Um conjunto {p, q} chamado de aresta se pertencer a E e neste caso dizemos que esta aresta liga os vrtices p e q.

SOLUES NVEL 1
PROBLEMA 1: SOLUO DE VINCIUS H. CAMPOS SENRA (BELO HORIZONTE - MG):

700 o ltimo nmero possvel at 1000 tal que a soma de seus algarismos seja igual a 7: nmeros menores que 700 tm soma dos algarismos maior que 7. O primeiro nmero 7 mesmo. De 1 a 100, existem 8 nmeros que a soma de seus algarismos igual a 7: 07, 16, 25,, 61, 70. A medida que vai aumentando a ordem das centenas, diminui um nmero que possvel fazer isto, ou seja: De 101 a 200, existem 7 nmeros: 106, 115,, 160. De 201 a 300, existem 6; De 301 a 400, 5; De 401 a 500, 4; De 501 a 600, 3.
EUREKA! N19, 2004

35

Sociedade Brasileira de Matemtica

Apenas de 601 a 700 que no ocorre isso, pois fica includo o 700 tambm, sendo portanto 3 nmeros. (601, 610 e 700). Resposta: Somando todos esses resultados 36 nmeros at 1000 tm a soma de seus algarismos igual a 7.
PROBLEMA 2: SOLUO DE VITOR MORI (SO PAULO SP)

Para a seqncia terminar em 103, devemos comear pelo fim. Utilizando o diagrama da rvore, teremos:

92 103

82 91

68 77

61 70

53 62 49 38 28 23 16 8 4 2 1

101 100 86

Logo a maior seqncia : 1, 2, 4, 8, 16, 23, 28, 38, 49, 62, 70, 77, 91, 101, 103.
PROBLEMA 3: SOLUO DO EDITOR

Tomamos um quadrado de lado 1, dobramos o seu lado e colocamos 5 quadrados de lado 1 esquerda e em cima para completar um quadrado de lado 3: Dobramos de novo a figura e colocamos 13 quadrados de lado 1 para completar um quadrado de lado 7. Dobramos a figura e colocamos 29 quadrados de lado 1 para completar um quadrado de lado 15. Dobramos a figura e colocamos 61 quadrados de lado 1 para completar um quadrado de lado 31. Quadruplicamos a figura e colocamos 249 quadrados de lado 1 para completar um quadrado de lado 125. Octuplicamos a figura e colocamos 2001 quadrados de lado 1 para completar um quadrado de lado 1001. Finalmente, dobramos a figura e colocamos mais 4005 quadrados de lado 1 para completar um quadrado de lado 2003. Gastamos assim, no total, 4005 + 2001 + 249 + 61 + 29 + 13 + 5 + 1 = 6364 quadrados cujos lados so potncias de 2 para cobrir o quadrado de lado 2003.
PROBLEMA 4: SOLUO DE HENRIQUE POND DE OLIVEIRA PINTO (SALVADOR - BA)

a) Se prolongarmos os lados do segundo quadrado temos:

EUREKA! N19, 2004

36

Sociedade Brasileira de Matemtica

m s

As retas m e s dividem o primeiro quadrado em quatro partes iguais, e ento a 1 parte escura representa do primeiro quadrado (assim como as outras trs 4 partes). Como a rea do primeiro quadrado 12. 12 = 144 ento a rea escura 1 144 = 36cm2. 4 Observao: Toda reta que passa pelo centro de um quadrado divide ele em duas partes iguais. Como a reta r no exemplo a seguir:
A B

C S

Pois se a reta S divide o quadrado em duas partes iguais, basta o tringulo ABO e o tringulo CDO serem iguais. Como os ngulos OCD e OBA so iguais, e AOB e COD so iguais, O AB e ODC tambm so iguais. Como OC = OB ento os tringulos ABO e CDO, por possurem 3 ngulos iguais (COD = AOB, OCD = OBA e ODC = O AB ) e o lado igual (OC = OB ) so tringulos iguais.

EUREKA! N19, 2004

37

Sociedade Brasileira de Matemtica

b) As reas rabiscadas so conhecidas (cada uma tem qual est, diferente de PQRS)
T X P G M Z S DE R U F V Y H I B O J K L A C Q N

1 da rea do quadrado na 4

W Q

O tringulo ABC igual ao DEF e o tringulo GHI igual ao JKL. Se B AC = x e GI H = 360 90 90 (180 x ) ento GI H = 180 (180 x ) que igual a x ento GIH = B AC. Podemos dizer que AC = HI pois AC = 12cm CJ , HI = 12cm HM e CJ = MH (as retas W, N e Z dividem os dois grandes quadrados de forma idntica e em partes iguais). CJ = DI e DI = HM pois as retas dividem o quadrado em 4 partes idnticas, logo CJ = MH . Pelo mesmo raciocnio podemos deduzir que AB = IG . Sabendo que GI H = B AC e que AB = IG e AC = HI ento deduzimos que o tringulo ABC = GHI. Como o tringulo ABC igual ao DEF, GHI igual ao JKL e o ABC igual ao GHI, logo os tringulos ABC, DEF, GHI e JKL so iguais. A rea do quadrado PQRS igual a rea riscada mais a rea de BCJEDI mais a rea de GHI mais a rea de JKL. Como a rea de GHI mais a rea de JKL igual a rea de ABC mais DEF ento a rea total do quadrado igual a: a rea rabiscada mais a rea de BCJEDI mais a rea de ABC mais a rea de DEF. Isso tudo igual a: 1 1 1 1 4 cm 2 + 12 cm 2 + 4 cm 2 + 12 cm 2 + 40 cm 2 = 4 4 4 4 2 2 2 2 = 4 cm + 36 cm + 4 cm + 36 cm + 40 cm 2 = 120 cm 2 .

EUREKA! N19, 2004

38

Sociedade Brasileira de Matemtica

PROBLEMA 5: SOLUO DA BANCA

a) Considere uma seqncia na qual cada termo maior que a soma dos termos anteriores, como por exemplo 1, 3, 3 2 , 33 ,... , 3 2003 . Obrigatoriamente temos que quebrar a vareta de comprimento 3 2003 e colocar os pedaos em dois lados opostos, pois 3 2003 maior que o dobro da soma de todas as restantes. Agora, a maior das varetas usadas nos dois lados restantes, 3n , maior que a soma das varetas 1, 3, 32 , 33 ,... , 3n +1 , o que torna impossvel a construo do retngulo. b) Quebrando inicialmente uma vareta qualquer ao meio, construmos dois lados opostos. Em seguida, dividimos as varetas restantes em dois conjuntos A e B. Se as somas dos comprimentos das varetas dos dois conjuntos forem iguais, no necessrio fazer mais quebras. Caso contrrio, passamos quantas varetas forem necessrias de um para o outro at que a desigualdade das somas se inverta; agora basta mais uma nica quebra na ltima vareta que mudou de lado para que as somas se igualem. SOLUES NVEL 2
PROBLEMA 1: SOLUO DE FELIPE GONALVES ASSIS (CAMPINA GRANDE - PB)

a) Temos que A > B e B > D, logo A > D. Tambm sabemos que A > C e C > D, ento podemos afirmar com certeza que "D" o menor nmero, pois B > D; C > D; A > D e "A" o maior nmero pois A > B; A > C; A > D. Se "B" for igual a "C", teremos trs nmeros dispostos de tal forma que o menor deles ser "D", o maior ser igual "A" e o outro ser tanto "B" como "C". De quantas formas ento eu posso escolher 3 inteiros diferentes entre 1 e 9? Vamos pensar da seguinte maneira: para escolher o primeiro nmero eu tenho 9 possibilidades: 1; 2; 3; 4; 5; 6; 7; 8; 9, j para cada escolha eu tenho outros 8 nmeros para escolher, os de 1 a 9 exceto o primeiro escolhido. Finalmente, para cada possibilidade eu tenho outros 7 inteiros para escolher, os 9 exceo dos j escolhidos, assim, eu tirei 9 8 7 possibilidades, ou seja: 504. Todavia, a ordem dos nmeros no importa, isto , o menor deve ser "D", o maior "A" e o do meio "B" e "C", e escolhendo da forma supracitada os mesmos nmeros so escolhidos 6 vezes (3!). Observe: "x"; "y" e "z" s podem ser usados juntos uma vez mas nas 504 possibilidades aparecem 6 vezes: x y z ; x z y; y x z; y z x; z x y; z y x 504 = 84 Logo, devemos dividir 504 por 6, assim 6
EUREKA! N19, 2004

39

Sociedade Brasileira de Matemtica

Finalmente conclumos que existem 84 = quais A > B; C > D; A > C; B > D; B = C.

9! tabuleiros diferentes nos (9 3)! 3!

a) Como no podemos definir relao entre B e C, vamos analisar trs casos: 1. B > C A > B > C > D 2. B < C A > C > B > D 3. B = C A > B = C > D Para o caso 1, temos que escolher 4 nmeros distintos de 1 a 9 e p-los em ordem j descrita A o maior, o segundo maior B e D o menor), seguindo o raciocnio do quesito "a" temos 9 8 7 6 possibilidades de escolha sendo que os mesmos 4 nmeros se repetem em 24 escolhas ou 4!. Ns s utilizamos a; b; c; d uma vez, mas eles ocorrem 24 vezes, apenas alterando a ordem, logo as possibilidades se reduzem: 9 8 7 6 3024 9! = = 126 = 24 24 (9 4)! 4! O caso 2 ter tantas possibilidades quanto o caso 1, apenas trocando B por C e o caso 3 j foi estudado no quesito "a". Assim, o total de tabuleiros : 126 + 126 + 84 = 336.
PROBLEMA 2: SOLUO DE GUILHERME R. NOGUEIRA DE SOUZA (SO PAULO SP)

Vamos analizar x 2 + 5 x + 23 mdulo 2. Temos: x 2 + x + 1 0 para ser divisvel por 2. Se x 1 1 + 1 + 1 1 ; se x 0 0 + 0 + 1 1 Logo 2 no o menor primo, que vamos chamar de p. Vamos analisar x 2 + 5 x + 23 mdulo 3. Temos: x 2 + 2 x + 2 0 para ser divisvel por 3. Se x 0 0 + 0 + 2 2 ; se x 1 1 + 2 + 2 2 ; se x 2 1 + 1 + 2 1 . Logo p no 3. Vamos analisar x 2 + 5 x + 23 mdulo 5. Temos: x 2 + 3 0 para ser divisvel por 5. se x 1 1 + 3 4 ; se x 2 4 + 3 7 ; se x 3 4 + 3 2 ; se x 4 1 + 3 4 ; se x 0 0 + 3 3 . Logo p no 5. Vamos analisar x 2 + 5 x + 23 mdulo 7. Temos: x 2 2 x + 2 0 para ser divisvel por 7.
EUREKA! N19, 2004

40

Sociedade Brasileira de Matemtica

se x 1 1 2 + 2 1 ; se x 2 4 4 + 2 2 ; se x 0 0 0 + 2 2 ; se x 3 2 6 + 2 5 ; se x 4 2 1 + 2 3 ; se x 5 4 3 + 2 3 ; se x 6 1 5 + 2 5 . Logo p no 7. Vamos analisar x 2 + 5 x + 23 mdulo 11. Temos: x 2 + 5 x + 1 0 para ser divisvel por 11. se x 1 1 + 5 + 1 7 ; se x 3 9 + 4 + 1 3 ; se x 5 3 + 3 + 1 7 ; se x 2 4 + 10 + 1 4 ; se x 4 5 + 9 + 1 4 ; se x 6 3 + 8 + 1 11 ; se x 7 5 + 2 + 1 8 ; se x 9 4 + 1 + 1 6 ; se x 0 0 + 0 + 1 1 ; se x 8 9 + 7 + 1 6 ; se x 10 1 + 6 + 1 8 . Logo p no 11. Vamos analisar x 2 + 5 x + 23 mdulo 13. Temos: x 2 + 5 x 3 0 para ser divisvel por 13. se x 1 1 + 5 3 3 ; se x 5 12 + 12 3 8 ; se x 9 3 + 6 3 6 ; se x 2 4 + 10 3 11 ; se x 6 10 + 4 3 11 ; se x 10 9 + 11 3 4 ; se x 3 9 + 2 3 8 ; se x 7 10 + 9 3 3 ; se x 11 4 + 3 3 4 ; se x 4 3 + 7 3 7 ; se x 8 12 + 1 3 10 ; se x 12 1 + 8 3 6 , se x 0 0 + 0 3 10 Logo p no 13. Chegamos at agora que p no 2,3,5,7,11,13. Ento, p no mnimo 17, e 17 divide x 2 + 5 x + 23 quando x 2 : ( 2) 2 + 5( 2) + 23 = 17. Logo o menor primo que divide x 2 + 5 x + 23 para algum x inteiro 17.
PROBLEMA 3: SOLUO DE ANDR LINHARES RODRIGUES (FORTALEZA CE)

Vamos dividir o problema em duas partes: a) BX = CX PQ um dimetro de S. Seja ACB = . Assim, temos que QBC = (j que BX = CX) e P AC = 180 90 = 90 . Observe que os ngulos PC e PBC esto "olhando" para o mesmo arco. Assim, vemos que PBC = P AC = 90 PBQ = 90 + = 90 PQ dimetro.

EUREKA! N19, 2004

41

Sociedade Brasileira de Matemtica


A Q 90 X

90

b) PQ um dimetro de S BX = CX . Se Mas ACB = , P AC = PBC = 90 .

PQ

dimetro,

donde

PBQ = 90 90 + QBC = 90 QBC = BXC issceles BX = XC. A figura poderia ser um pouco diferente:
Q

180 2
2 X

P 180 B

180 2

a) Chamando ABQ de e C BQ = ACB = , teramos P AB = 90 + + , B AC = 180 2 e C AQ = , e o ngulo P AQ seria 90 + + + 180 2 + = 90 . b) Chamando QBC de , temos que C AQ = . Mas PQ dimetro
P AC + = 90 P AC + = 90 P AC = 90 ACB = 180 90 (90 ) =

EUREKA! N19, 2004

42

Sociedade Brasileira de Matemtica

= BCX issceles BX = CX . A figura pode ainda ficar da seguinte forma:

P Q A

180

T B

=
C

a) Seja X BC = e X BA = . Seja T a interseo entre as retas BC e AP. Temos que b) Seja


T AB = + 90 , QBC = , B AC = 180 2

C AQ = CBQ = .

Ento, que

P AQ = 180 (180 2 ) ( + 90) = 90 PQ dimetro de S. ABQ =

ACB = .

Ento

teremos

B AC = 180 , QAC = e B AT = + 90. Como PQ dimetro, P AQ = 90 QAT = 180 90 = 90 + 90 + 180 + = 90 = BXC issceles BX = CX .

PROBLEMA 4: SOLUO DE MARICY MIKI HISAMOTO (SO PAULO SP)

x + 4 y 4xy + 2 x 4 y + 2 > 0, 
2 2 x2 2x2 y +(2 y )2

( x 2 y ) 2 + (2 x 4 y ) + 1 + 1 = ( x 2 y ) 2 + 2( x 2 y ) + 2 = [( x 2 y ) + 1]2 + 1 Como qualquer real elevado ao quadrado resulta em um nmero positivo, [( x 2 y ) + 1]2 positivo para quaisquer x e y reais. Assim, [( x 2 y ) + 1]2 + 1 tambm ser positivo, logo x 2 + 4 y 2 4 xy + 2 x 4 y + 2 maior que 0.
PROBLEMA 5: VEJA A SOLUO DO PROBLEMA 4 DO NVEL 3
EUREKA! N19, 2004

43

Sociedade Brasileira de Matemtica

PROBLEMA 6: SOLUO DE EDUARDO FISCHER (LAJEADO RS)

Vamos meter uma induo: Se N = 1, 2, 3 verdadeiro: qualquer caminho serve. Se N > 3, separamos o ponto PN e consideramos (usando a hiptese de induo) um caminho fechado passando pelas outras cidades, digamos P 1P 2 ...P 1, N 1 P mudando o meio de transporte no mximo uma vez. Temos duas possibilidades: a) O caminho usa s um meio de transporte. Nesse caso, o caminho fechado P 1P 2 ...P 1 ou o P 1...P NP NP N 1 muda de meio de transporte no mximo uma vez. b) O caminho muda de meio de transporte exatamente uma vez, digamos de PN 1 para P 1 (quando muda de rodovia para ferrovia, sem perda de generalidade). Temos ento os seguintes caminhos fechados, em cada caso abaixo: b.1) PN 1 PN e PN P 1 so ferrovias: P 1P 2 ...P 1 N 1 P NP b.2) PN 1 PN e PN P 1 so rodovias: P 1P 2 ...P N 1 P NP N 1 b.3) PN 1 PN rodovia e PN P 1 ferrovia: P 1P 2 ...P 1 N 1 P NP b.4) PN 1 PN ferrovia e PN P 1 rodovia: P 1P 2 ...P NP N 1 P N Pelo princpio indutivo conclumos que para qualquer nmero possvel de cidades em Tumblia, nosso turista poder visit-las mudando o meio de transporte no mximo uma vez.

SOLUES NVEL 3
PROBLEMA 1: VEJA A SOLUO DO PROBLEMA 2 DO NVEL 2. PROBLEMA 2: ADAPTADA DA SOLUO DE MURILO VASCONCELOS DE ANDRADE (MACEI - AL)
k k +1 Vamos mostrar que kn = ento kn = k + 1 , log 2 n + 1 , ou seja, se 2 n < 2 por induo em k. Primeiro vamos ver que kn k + 1. De fato, k1 = 1 e, dado n.

n n Sejam dados conjuntos A1 , A2 ,..., Ak +1 S . Como # Ak++1 ou # Ak+1 , 2 2 n n pois Ak+1 Ak++1 = S , segue que # ( S \ Ak++1 ) < 2k ou # ( S \ Ak+1 ) < 2k . 2 2 + k Supondo sem perda de generalidade que # ( S \ Ak +1 ) < 2 , por hiptese de induo, A j := A j ( S \ Ak++1 ) , para 1 j k , temos trocando Aj por E

EUREKA! N19, 2004

44

Sociedade Brasileira de Matemtica


E2 Ek ... A , para alguma escolha dos sinais + e , donde S \ Ak +1 = E A1 A

... A2 Ak++1 , para a mesma escolha dos k primeiros sinais. S = A1 A2

Por outro lado, se n 2k e, para 1 i k , k 1 Ai = j 2 j j {0,1}, j < k e i 1 = 1 {0,1,..., n 1} o conjunto dos j =0 k naturais menores que 2 cujo (i 1)-simo algarismo binrio igual a 1, * ... Ak* no vazio para qualquer escolha dos sinais, donde A1* A2
A1 A2 ... Ak S = {0,1,..., n 1} , para qualquer escolha dos sinais. Isso mostra que kn > k , e portanto kn > k .

PROBLEMA 3: SOLUO DE RAFAEL MARINI SILVA (VILA VELHA ES)

No difcil ver que o centro O da circunferncia inscrita ao losango ABCD AH AE = encontro das diagonais. Se B AD = 2 = BCD , vamos mostrar que , ou CF CG ainda, AH CG = CF AE = K constante; o que implica o resultado, pois os tringulos AEH e CGF so semelhantes, e logo EH paralelo a FG. Sendo I, J, K, L, P, P' os pontos de encontro A da circunferncia inscrita com AD, DC, CB, AB, EF e GH respectivamente temos: I L H E I OP I OE = EOP = (pois EI e EP so O P' 2 P B D tangentes comuns circunferncia) J OP F G J OF = FOP = J K 2 I OJ = J OP + I OP = 2( POE + F OP) =
C

2( EOF ) = 180 I DF = D AB = 2 EOF = , analogamente temos: H OG = . Destacando o tringulo ABC temos:

EUREKA! N19, 2004

45

Sociedade Brasileira de Matemtica

Fazendo BOG = X , temos: AHO = 90 X AOH = 90 + X OGC = 90 + X = AOH COG = 90 X = AHO OCG = O AH
B

d 2

90 X 90 + X

90 X

X
X

d 2

90 + X

Logo AOH ~ CGO e AH AO d2 2 = AH CG = AO CO = ( AO ) = . CO CG 4 Analogamente, temos d2 AC AE CF = , constante pois d = 4 2 constante.

PROBLEMA 4: SOLUO DE FBIO DIAS MOREIRA (RIO DE JANEIRO - RJ)

Seja a circunferncia do problema. Suponha que B, C , D so os pontos que maximizam a rea de ABCD (obviamente, ABCD convexo). Ento C ponto mdio de BD : caso no fosse, considere M, ponto mdio de BD . Ento S ( ABMD ) = S ( ABD ) + S ( BMD ) > S ( ABD ) + S ( BCD ) = S ( ABCD ), pois, como M mdio de BD , d ( M , BD) > d (C , BD) . Como BMD e BCD tm a mesma base, S ( BMD ) > S ( BCD ). Considere agora { A '} = ( AC )\{C}. Pelo mesmo argumento, B e D so pontos mdios dos seus respectivos arcos A ' C. Em particular, isso implica que BD a mediatriz de A ' C O BD (em F particular, O ponto mdio de BD ). Alm disso, como 0 / C mdio de BD , e O

0/

0/

0/

0/

0 /

0/

mdio de BD , CO BD. Mas A ' C BD , pois BD mediatriz de A ' C . Em particular, AC BD . Mas OC BD O, A, C so colineares. Construo: Trace a semi-reta AO : ela intersecta em C (pois o polgono deve ser convexo com A, O, C colineares, logo esta a nica possvel posio de C). Como BD ortogonal a AC em O, a perpendicular a AC por O determina B e D - estes B e D so os que maximizam a rea do quadriltero #ABCD.

EUREKA! N19, 2004

46

Sociedade Brasileira de Matemtica

PROBLEMA 5: SOLUO DE ALEX CORRA ABREU (NITERI - RJ)

1 f + 2 1 1 x Em (iii) troque x por e y por f 2 x y x+ y 1 g ( x) = f x i) ii) x < y g ( x) > g ( y ) x + y g ( x) + g ( y) g 2 2 g (2 z y ) + g ( y ) x+ y g ( z) 2 2

1 f y defina agora

Fazendo z =

2 mas 2 z > 2 z 1 g (2 z 1) g (2 z ) g (2 z ) 2 g ( z ) g (1). Vamos agora provar por induo que g (2n z ) 2n g ( z) (2 n 1) g (1) = 2n ( g ( z ) g (1)) + g (1) : se g (2k z) 2 k g ( z ) (2 k 1) g (1) g (2k +1 z ) 2 g (2k z) g (1) 2k +1 g ( z ) (2k +1 2) g (1) g (1) = 2k +1 g ( z ) (2k +1 1) g (1) . Se tomarmos z > 1, g ( z ) g (1) < 0 n tal que 2n ( g ( z ) g (1)) + g (1) < 0 x tal que g ( x) < 0 y = e Rafael Daigo Hirama A idia contar o nmero de pares (f; D), sendo f e D funo e conjunto correspondentes a um grafo bacana e comparar com o nmero de grafos com n vrtices. Cada par (f; D) gera no mximo um grafo: de fato, dada f, ligamos os vrtices p e q se, e somente se, |f(p) f(q)| D. Caso ocorra alguma contradio entre os pares de vrtices, o par no gera um grafo. Assim, basta provar que a quantidade de grafos de n vrtices maior que a quantidade de pares (f; D), pois isso demonstraria a existncia de um grafo que no gerado por nenhum par e, portanto, no bacana.
EUREKA! N19, 2004

Fazendo agora y = 1 2g ( z ) g (2z 1) + g (1) g (2z 1) 2g ( z) g (1) z > 1

1 tal que f ( y ) < 0. x

PROBLEMA 6: Soluo adaptada das solues de Davi Mximo Alexandrino Nogueira

47

Sociedade Brasileira de Matemtica

No que se segue, x denota o maior inteiro menor que ou igual a x. H

N e, como podemos tomar D [1, n2/4]


2

n 2 ! n 4 n2 funes injetoras de V, com n elementos, para [1, n2/4] 4 n 2 n ! 4

,

(seno, trocamos D por sua

interseo com 1, n 4 ) temos no mximo 2

n2 4

n2 24 2

conjuntos D. Logo o

n2 nmero de pares (f; D) menor que ou igual a 4


n 2 n 2 n 2 2

n 2 4 .

grafos com n vrtices dados. Assim, basta mostrar Alm disso, h 2 = que para todo n suficientemente grande,
n2 n 2 2

n2 > 4

n2 4

n2 n 24 2

>2

2 n log 2

n 2

n2 n > log 2 2 8 2

n2 8

>

n , 2

o que verdade para n > 58 (sendo n > 58,


n2 2 8

n2 (1 + 1) 8

n2 > 1+ + 8

n2 8

2 (n 1) 8

>

n ). Na verdade, poderamos 2

simplesmente notar que uma funo exponencial cresce muito mais que qualquer funo polinomial, que por sua vez, cresce muito mais que qualquer funo logartmica. De fato, quando n fica muito grande quase nenhum grafo bacana. Isto quer dizer que, embora existam grafos bacanas com um nmero qualquer de vrtices (como, por exemplo, o grafo completo e o grafo vazio), a probabilidade de um grafo ser bacana fica muito prxima de zero quando n muito grande.

EUREKA! N19, 2004

48

Sociedade Brasileira de Matemtica

XXV OLIMPADA BRASILEIRA DE MATEMTICA Problemas e Solues da Primeira Fase Nvel Universitrio
PROBLEMA 1

Seja X # 3 o poliedro convexo cujos vrtices so todos os pontos ( x, y , z) com x 2 + y 2 + z 2 = 2. Calcule o volume de X.
PROBLEMA 2

O tenista Berrando Gemigemi tem 30 dias para preparar-se para um torneio. Se ele treina 3 dias seguidos ele tem fadiga muscular. Ele, ento, decide que, durante esses 30 dias, ir treinar 20 dias, sem nunca treinar 3 dias seguidos, e descansar nos outros 10 dias. De quantas maneiras diferentes ele pode escolher os 10 dias de descanso?
PROBLEMA 3

Sejam A e B matrizes reais n n inversveis. Mostre que se vale a condio ( AB) k = Ak B k para trs valores inteiros consecutivos de k ento AB = BA.
PROBLEMA 4

1 1 2 ... . Defina + + = 2 22 32 6 k >0 1 1 1 f (n) = 2 = 1 + 2 + ... + 2 . 2 k n 0< k n Prove que existe um nmero real a > 0 tal que existe o limite: 2 a 2 lim f (n ) + n . n 6 n Calcule a e este limite. Sabemos que

=1+

PROBLEMA 5

Sejam a e n inteiros, n > 1, mdc(a, n) = 1. Prove que o polinmio (1 n ) (( X + a ) n X n a ) tem todos os coeficientes inteiros se e somente se n primo.

EUREKA! N19, 2004

49

Sociedade Brasileira de Matemtica

PROBLEMA 6
2 2. Defina a1 = 3, an +1 = an log log an = log 2 e calcule lim ( log log an n log 2). Prove que lim n n n (Observao: os logaritmos esto todos na base e).

SOLUES NVEL UNIVERSITRIO


Os vrtices de X so os doze pontos (1, 1,0),(1,0, 1),(0, 1, 1), que so os pontos mdios das arestas do cubo (1, 1, 1) , donde X obtido a partir do cubo tirando fora uma pirmide (ou tetraedro) em cada vrtice.
SOLUO DO PROBLEMA 1:

O volume do cubo 23 = 8. Cada pirmide tem base de rea 1/2 e altura 1 logo tem volume igual a 1/6. 8 20 Assim o volume do slido igual a 8 = 6 3
SOLUO DO PROBLEMA 2:

Podemos dividir os 30 dias em 10 blocos de trs dias. claro que ele dever descansar em exatamente um dia por bloco. Se alguma vez ele descansa no dia central de um bloco depois disso ele no poder descansar no ltimo dia de um bloco; analogamente, se alguma vez ele descansa no primeiro dia de um bloco ele dever descansar no primeiro dia de todos os blocos que vierem depois. Assim, ele deve descansar no ltimo dia nos primeiros
EUREKA! N19, 2004

50

Sociedade Brasileira de Matemtica

x blocos, depois descansar no dia central durante y blocos e finalmente descansar no primeiro dia nos ltimos z blocos, onde x + y + z = 10, x, y, z 0, x, y, z . 12 O nmero de solues = 66. 2
SOLUO DO PROBLEMA 3:

Suponha que ( AB) k = Ak B k , ( AB) k +1 = Ak +1 B k +1 e ( AB) k +2 = Ak +2 B k +2 . Temos ento Ak +1 B k +1 = ( AB) k +1 = ( AB )( AB) k = AB Ak B k , e logo (multiplicando esquerda por A1 e direita por B k ) obtemos Ak B = BAk . Analogamente, usando a segunda e a terceira igualdades, obtemos Ak +1 B = BAk +1 Assim temos BAk +1 = Ak +1 B = A Ak B = A BAk , donde, multiplicando direita por Ak , obtemos BA = AB.
SOLUO DO PROBLEMA 4:

1 1 1 2 f (n ) = + + + ... 2 2 6 (n + 1) (n + 2) (n + 3) 2 Podemos obter uma boa estimativa para esta soma estimando a rea sob o grfico 1 de y = 2 , x n , pela regra dos trapzios: x Temos
y

n+1

n+2

n+3

EUREKA! N19, 2004

51

Sociedade Brasileira de Matemtica

1 1 dt = . 2 t n A rea obtida pela aproximao, que ligeiramente maior, 1 1 1 1 1 1 2+ + + ... = + 2 2 2 ( n + 1) (n + 2)2 2 n + 1 n ( ) 1 1 1 1 1 1 2 = + + + + = + f (n ) ... 2 n 2 ( n + 1)2 ( n + 2 )2 ( n + 3)2 2n 2 6 A rea exata

2 1 2 1 Donde f ( n) + n < 6 n 2 Assim a = 1 e, se acreditarmos que esta aproximao boa, teremos que o limite igual a 1/2. Para demonstrarmos que o erro realmente pequeno, devemos estimar a diferena entre as reas: 1 1 1 n +1 1 1 1 1 1 1 1 2 + = 2+ = 2 < 4 2 2 2 2 n 2 n (n + 1) 2 n ( n + 1) n(n + 1) 2n ( n + 1) 2n t Assim 2 1 1 1 1 1 f (n ) + 2 < 4 + + ... 0< 4 6 2n ( n + 1) n 2n 1 1 1 1 < 4 dt = 1 n 2 6 (n 1)3 t e com isso 1 1 2 1 2 1 n2 < + n < , ( ) f n 2 6 ( n 1) 3 6 n 2 o que confirma que o limite igual a 1/2.
SOLUO DO PROBLEMA 5:
n n Temos ( X + a)n = ank X k . Assim k =0 k n 1 n ( X + a)n X n a = a n k X k + (a n a). k =1 k n n(n 1)...(n k + 1) Se n primo e 1 k n 1, = mltiplo de n, pois o k! k numerador mltiplo de n mas o denominador no, e, pelo pequeno Teorema de

EUREKA! N19, 2004

52

Sociedade Brasileira de Matemtica

Fermat, a n a mltiplo de n. Assim, nesse caso, o polinmio 1 n n (( X + a) X a ) tem todos os coeficientes inteiros. n Se n composto, seja q um fator primo de n. Temos ento que n n(n 1)...(n q + 1) no mltiplo de n. De fato, se q k a maior potncia de = q ! q n q que divide n, a maior potncia de q que divide q k 1 , pois o nico fator q mltiplo de q no numerador n e o nico fator mltiplo de q em q! = q(q 1) . 1 2 . 1 q. Assim, nesse caso, o coeficiente de X q em (( X + a)n X n a ) no n inteiro.
SOLUO DO PROBLEMA 6:

A recurso satisfeita por an = 2 + 2 para qualquer nmero real , de fato,


n n

2 por induo an +1 = an 2 = 2 + 2
n

) 2
2

= 2 + 2 + 2 2 = 2 + 2 .
1+ 5 . 2

n+1

n+1

n+1

n+1

Para a seqncia do problema, basta resolver 2 + 2 = 3 que tem raiz = 1 + 5 1 + 5 Assim an = Assim, + . 2 2
2n 2n

1+ 5 1+ 5 onde lim ( n ) = 0 donde log an = 2 n log ( ), + an = n 2 + 1 (n ), n 2 1+ 5 lim 1 ( n ) = 0 e log log a n = n log 2 + log log lim (n ) = 0 2 + 2 ( n ), n 2 n 1+ 5 loglog an Assim lim = log2 e lim (log log an n log 2 ) = log log . n n n 2

2n

EUREKA! N19, 2004

53

Sociedade Brasileira de Matemtica

XXV OLIMPADA BRASILEIRA DE MATEMTICA Problemas e Solues da Segunda Fase Nvel Universitrio
PROBLEMA 1:

So dados uma parbola e um ponto A fora dela. Para cada ponto P da parbola, seja t a tangente parbola por P e r a reta paralela ao eixo da parbola por P. A reta perpendicular a t por A corta r em Q. Prove que, ao variar P, o ponto Q percorre uma hiprbole equiltera.
PROBLEMA 2:

Suponha que existam 4 vetores (a, b) 2 , dois a dois linearmente independentes (sobre  ), tais que ap + bq o quadrado de um polinmio em [ x] . Prove que p e q so constantes. b) Prove que no existem polinmios no constantes r, s, t, u [ x ] tais que r t f = , g = e f 2 = g ( g 1)( g a ), onde a  , a 0 , a 1. s u
PROBLEMA 3:

a) Sejam p e q [ x ] polinmios primos entre si com coeficientes complexos.

Seja p > 2 um nmero primo. Seja Xp o conjunto de todas as matrizes quadradas A com coeficientes em de ordem 4 para as quais A2 = I : X p = A ( Calcule o nmero de elementos de Xp.

( p) e

( p) )

4 4

A2 = I

Observao: ( p ) = {0,1, 2,..., p 1} o corpo finito com p elementos. A soma e o produto so definidos mdulo p; assim, por exemplo, em (7) , 4 + 5 = 2 e 4 5 = 6.
PROBLEMA 4:

Temos um dado de 6 faces, no necessariamente honesto. Jogamos o dado trs vezes e obtemos resultados a, b e c. Prove que P( a = c | a = b) P( a = c | a b) e que vale a igualdade se e somente se o dado honesto. Observao: P( a = c | a = b) a probabilidade condicional P ( a = b = c) . P( a = c | a = b) = P( a = b)
EUREKA! N19, 2004

54

Sociedade Brasileira de Matemtica

Um dado honesto se a probabilidade de cada face


PROBLEMA 5:

1 . 6

Uma funo f : (1,1) # de classe C bacana se existem um inteiro positivo n e polinmios Pj #[ t ], 0 j n, com Pn no identicamente nulo tais que

P (t ) f
j j =0

( j)

( t ) = 0, para todo t ( 1,1). Prove que se f e g so bacanas ento

f + g e f g tambm so bacanas. Observao: Definimos f (0) = f e, para cada inteiro m 0, f ( m +1) = ( f ( m) ) '.

PROBLEMA 6:

Seja

A = (aij )1i , j n

uma matriz tal que

aij {0,1}, para quaisquer i, j


k

{(i, j ) | aij = 1}

99 2 n n . Prove que tr ( Ak ) , para todo k 2. 100 2


n i =1

Observao: Se B = (bij )1i, j n uma matriz quadrada ento tr ( B) = bii denota o trao de B.

SOLUES - NVEL UNIVERSITRIO


PROBLEMA 1: SOLUO DE LEONARDO AUGUSTO ZO (NILPOLIS RJ)

Dada a parbola , fixamos os eixos coordenados de forma que tenhamos a equao da parbola dada por : y = ax 2 . Supomos que A = (m, n) e P = ( x p , y p ).
2 2 ). Como p y p = ax p P = ( x p , ax p

Pelo enunciado, a reta t tem coeficiente angular 2ax p , e a reta perpendicular a t 1 . 2ax p Assim, a reta perpendicular a t que passa por A dada pela equao: 1 ( x m). (*) t1 = ( y n) = 2ax p tem coeficiente angular
EUREKA! N19, 2004

55

Sociedade Brasileira de Matemtica

A reta r dada por r : x = x p Assim: Q = t1 r = ( x p , 1 1 m ( x p m) + n) = ( x p , + n) 2ax p 2ax p 2a 1 1 m m + n. Se =q e n = k , ento: 2ax 2a 2a 2a

Em Q, variando x p , teremos: y = y=

q q + k ( y k) = ( y k) x = q . x x Assim, se deslocarmos o eixo y para y k, teremos a hiprbole equiltera: m yx = q, onde q = . 2a


PROBLEMA 2: SOLUO DE MRCIO AFONSO ASSAD COHEN (RIO DE JANEIRO RJ)

a) Escrevendo a i p + bi q = v i2 , i = 1, 2,3,4 , temos que toda raiz a de vi dupla de ai p + bi q,:

a i p(a ) + bi q (a) = 0 xp(a ) + yq(a ) = 0 . Portanto o sistema , a i p ' (a ) + bi q ' (a ) = 0 xp' (a ) + yq' (a ) = 0

894 F p(a )q' (a) q (a) p' (a) = 0 e as solues podem ser parametrizadas de modo que (a i , bi ) = t (q(a), p (a)) .
indeterminado (j que (ai, bi) Como os (ai, bi) so 2 a 2 LI, os vis so 2 a 2 co-primos (porque se a fosse raiz de vi e de vj, ento (ai, bi) e (aj, bj) seriam ambos mltiplos de (q(a), p(a)). Alm disso, toda raiz de vi tambm raiz de h = pq' qp' . Mais geralmente, se a tem multiplicidade k como raiz de vi ento a raiz de h com multiplicidade 2k 1 k. (pois, para cada n, a n-sima derivada de n uma soma de termos do tipo p ( j ) q ( n j ) p ( n j ) q ( j ) , que se anulam em a se n 2k 1). Vou mostrar agora que h identicamente nulo, de modo que a derivada de p /q zero. Seja m = grau (p), n = grau (q): 1o caso: m > n: Cada vi tem grau m/2, exceto possivelmente um que pode ter grau n/2 (se existir algum ai nulo) e portanto os vis contribuem com pelo menos 3m/2 + n/2 razes de h.
EUREKA! N19, 2004

56

Sociedade Brasileira de Matemtica

Por outro lado, grau (h) = grau ( pq ' qp ' ) m + n 1 < m + n < ltima igualdade decorre de m > n. Logo, h )

3m + n , onde a 2

2o caso: m = n: Aqui pode ocorrer de algum polinmio aip + biq ter grau menor que n (como os pares so 2 a 2 LI, isso ocorre no mximo uma vez). Mas se ai p( x) + bi q ( x ) = Tl ( x ) , onde Tl de grau l < n, ento ai 0 1 1 h = pq ' p ' q = [( bi q + Tl ) q ' ( bi q '+ T 'l ) q ] = (Tl q ' T 'l q) ai ai tem grau ^ l + n 1. Por outro lado, os polinmios ai p + bi q esto contribuindo com h. Como grau (h) l + n 1 < l + n nulo.
'

l n + 3 razes de 2 2

l + 3n (j que l n), h identicamente 2

p h p( x) Portanto, = q 2 0 q( x) = cte . Como p e q so primos entre si, ambos q


so constantes. b) Substituindo f = r /s e g = t / u, veja que mdc(t, u) = 1 implica que os polinmios t, u, t u, t au so 2 a 2 primos entre si (se dois deles tivessem uma raiz comum , fcil ver que seria raiz de t e de u). Substituindo em f2 = g(g 1)(g a) temos: r 2 u 3 = s 2 t (t u )(t au ) . Como os polinmios t, t u, t au so primos com u, eles devem ser divisores de r2. Como eles so 2 a 2 primos entre si, cada um deve ser um quadrado perfeito (pois todas as razes de r2 e s2 tem multiplicidade par e s primo com r). Analogamente, como u3 primo com t, t u e t au, ele deve ser divisor de s2. Logo, u s tem razes de multiplicidade par (se a fosse raiz com multiplicidade mpar de u, tambm o seria de u3 e portanto de s2) e portanto quadrado de um polinmio complexo. Ou seja, escrevemos u, t, t u e t au como quadrados de polinmios, o que contradiz a letra (a), pois os vetores (1, 0), (0, 1), (1, 1), ( a, 1) so 2 a 2 LI (a 0ea 

EUREKA! N19, 2004

57

Sociedade Brasileira de Matemtica

PROBLEMA 3: SOLUO DE EDUARDO CASAGRANDE STABEL (PORTO ALEGRE RS)

Seja p um primo fixado ( p > 2) e V = (

p )4 um espao vetorial.
p

Proposio 1: Se A uma matriz 4 4 com coeficientes em

e A2 = I uma

involuo ento V = E F onde E = {u V : Au = u} e F = {u V : Au = u} Demonstrao: Suponha que A involuo. claro que os espaos E e F do enunciado s possuem o vetor nulo em comum, pois se Au = u e Au = u ento u = u e u = (0, 0, 0, 0). Mostremos que E + F = V. Seja v V qualquer. Ento v = e + f ( Av + v ) ( Av + v ) 1 1 e f = . Temos Ae = ( A2 v + Av ) = (v + Av ) = e e onde e = 2 2 2 2 1 1 1 2 Af = ( A v + Av ) = ( v + Av ) = ( Av + v ) = f , logo e E e f F . 2 2 2 Suponha que V = E F . Definiremos uma aplicao L : 4p 4p tal que L (v ) = e f onde v = e + f a decomposio nica com e E e f F . v1 = e1 + f1 e o2 = e2 + f 2 ento Afirmo que L linear. Se L(o1 + o2 ) = L(e1 + e2 + f1 + f 2 ) = (e1 + e2 ) ( f1 + f 2 ) = (e1 f1 ) + (e2 f 2 ) = L(v1 ) + L(v2 ). Caso p e v V ento L ( v ) = L ( e + f ) = e f = L (v ). Mais ainda, L
2

uma

involuo.

De

fato,

se

v = e + f V

ento

L (v) = L ( Lv ) = L (e f ) = e ( f ) = e + f = v. Logo existe uma matriz A tal que L (v ) = Av para todo v sendo A involuo. Deste modo, para cada matriz existe uma aplicao e vice-versa. Portanto o nmero de matrizes A igual ao nmero de decomposies V = E F . Proposio 2: Se E subespao de V = 4p e dim E = k ento # E = p k . Demonstrao: Dois subespaos de mesma dimenso so isomorfos, basta contar um subespao especfico de dimenso k. Se ei = (0,...,1,...,0) o vetor cannico e E = gerado (e1 ,..., ek ) fcil ver que # E = p k . Proposio 3: Seja M um espao vetorial sobre Caso 1 n m, existem ( p 1)( p subespaos de M de dimenso n.
m m 1 p

cuja dimenso dim M = m.

1)...( p

m n +1

1) /( p n 1)( p n 1 1)...( p 1)

Demonstrao: Faremos escolhas de n retas (subespaos de M de dimenso um) ordenadas (r1 , r2 ,..., rn ) tal que cada ri +1 no esteja contida no subespao gerado
EUREKA! N19, 2004

58

Sociedade Brasileira de Matemtica

por {r1 , r2 ,..., ri }. Para a primeira reta existem ( p n 1) ( p 1) escolhas possveis, pois duas retas quaisquer tem p elementos e s o vetor zero em comum. Ou seja, retirando-se o zero (d p n 1 elementos), repartimos este conjunto em ( p n 1) ( p 1) cada um dos quais com os elementos de uma reta, sem o vetor nulo. Com um argumento similar, chegamos a ( p n p ) ( p 1) possibilidades para a segunda reta, ( p n p 2 ) ( p 1) para a terceira e assim por diante. So ( p m 1)( p m p)( p m p 2 )...( p m p n 1 ) possibilidades. ( p 1) n Cada escolha (r1 , r2 ,..., rn ) gera um subespao N = gerado {r1 ,..., rn } de M de dimenso n. Dado um N M subespao de dimenso n, por um argumento ( p n 1)( p n p )( p n p 2 )...( p n p n 1 ) escolhas similar, gerado por ( p 1) n (r1 , r2 ,..., rn ) em N tal que ri +1 no est contido no espao gerado {r1 , r2 ,..., ri }. Logo existem ( pm 1)( pm p)...( pm pn1) ( pn 1)( pn p)...( pn pn1 ) ( pm 1)( pm1 1)...( pmn+1 1) = ( p 1)n ( p 1)n ( pn 1)( pn1 1)...( p 1) subespaos N M de dimenso n. A estratgia ser contar o nmero de decomposies do espao V em somas diretas V = E F . Caso (dim E = 0 e dim F = 4) ou (dim E = 4 e dim F = 0). Contam-se as seguintes possibilidades: E = {0} e F = V ou E = V e F = {0}. Assim, nesse caso, o nmero de possibilidades 2 (o leitor pode verificar que essas decomposies correspondem a A = I e A = I ) (1) Caso (dim E = 1 e dim F = 3) ou (dim E = 3 e dim F = 1) : Faamos s o caso dim E = 3 e dim F = 1 , o outro anlogo. Pela proposio 3, existem ( p 4 1)( p3 1)( p 2 1) ( p 4 1) = = 1 + p + p2 + p3 3 2 ( p 1)( p 1)( p 1) p 1 escolhas para o espao E. Em V E, restam ( p 4 p 3 ) elementos e ( p 4 p3 ) ( p 1)

retas, portanto so ao todo ( p 4 1)( p 4 p3 ) ( p 1) 2 escolhas para E e F. Contando tambm o caso dim E = 1 e dim F = 3 , temos:

EUREKA! N19, 2004

59

Sociedade Brasileira de Matemtica

( p 4 1)( p 4 p 3 ) = 2(1 + p + p 2 + p 3 ) p 3 = 2 p 6 + 2 p 5 + 2 p 4 + 2 p3 ( p 1) 2

(2)

Caso (dim E = 2 = dim F ). Novamente pela proposio 3, existem

( p 4 1)( p3 1) ( p 2 1)( p 1) escolhas para E. Escolhido E, fazemos um procedimento anlogo ao da demonstrao da proposio 3, escolhendo duas retas (r1 , r2 ) fora de E tal que r2 no est no subespao gerado por r1 e E, para contar as possibilidades de F. Resultam ( p 4 p 2 )( p 4 p3 ) escolhas para (r1 , r2 ) de onde resultam para E e F: ( p 1) 2

( p4 1)( p3 1) ( p4 p2 )( p4 p3 ) ( p2 1)( p2 p) ( p4 1)( p3 1) p2 ( p2 1) p3 ( p 1) = 2 ( p2 1)( p 1) ( p 1)2 ( p 1)2 ( p 1)( p 1) ( p2 1) p( p 1) p4 ( p2 1)( p2 +1)( p 1)( p2 + p +1) 4 4 3 = = p ( p + p + 2 p2 + p +1) = p8 + p7 + 2 p6 + p5 + p4 (3) ( p2 1)( p 1) Por fim, somando (1) + (2) + (3): 2 + (2 p6 + 2 p5 + 2 p4 + 2 p3 ) + ( p8 + p7 + 2 p6 + p5 + p4 ) = p8 + p7 + 4 p6 + 3p5 + 3p4 + 2 p3 + 2.
PROBLEMA 4: SOLUO DE YURI GOMES LIMA (FORTALEZA CE)

Sejam x1 , x2 , x3 , x4 , x5 , x6 as probabilidades. Temos ento que:

P(a = b) = x1 x1 + x2 x2 + ... + x6 x6 = xi2


i =1

P(a b) = 1 P(a = b) = 1 xi2 P ( a = c e a b) = P ( a = c) P ( a = b = c ) = P ( a = b) P ( a = b = c) P(a = c e a b) = xi2 xi3


Logo queremos mostrar que:

P(a = b = c) = x1 x1 x1 + ... + x6 x6 x6 = xi3

i =1

P ( a = b = c) P ( a = c e a b) P ( a = b) P ( a b) P ( a = b = c) P ( a b) P ( a = c e a b) P ( a = b) P ( a = c | a = b) P ( a = c | a b) ( xi3 )(1 xi2 ) ( xi2 xi3 )( xi2 ) xi3 ( xi2 ) .


2

EUREKA! N19, 2004

60

Sociedade Brasileira de Matemtica

Vamos mostrar ento que se x1 ,..., xn so reais no-negativos tais que ento

= 1,

x ( x ) .
3 i 2 2 i

(*)

De fato, temos
3 i

x = ( x ) ( x ) = ( x
3 i i 4 i 3 i

3 1

3 + ... + xn )( x1 + ... + xn ) = xi4 + xi3 x j i j 4 i 2 3 i

(*) x + x x j x + 2 ( xi x j ) x x j 2 ( xi x j )2
i< j i j i< j

mas observe que por MA MG temos


3 3 2 3 3 3 2 2 xi3 xj + xi x3 j 2 xi x j xi x j = 2( xi x j ) xi x j = ( xi x j + xi x j ) 2( xi x j ) = 2(xi x j ) , i j i< j i< j

como queramos. Para haver igualdade,

devemos

ter

j, xi3 x j = xi x3 j, i

ou

seja,

xi = x j , i, j, pois eles so positivos. Mas isso equivale a dizermos que xi2 = x 2 j P( a = c | a = b) = P( a = c | a b) o dado honesto!
PROBLEMA 5: SOLUO ADAPTADA DE CARLOS STEIN NAVES DE BRITO (S.J. DOS CAMPOS SP)

Suponha que

P (t ) f
j =0 j n 1 j =0

( j)

(t ) = 0 e

Q (t ) g
j =0 j m 1 j =0

( j)

(t ) = 0, para todo t ( 1,1).

Ento f ( n ) (t ) = rj (t ) f ( j ) (t ), e g ( m ) (t ) = s j (t ) g ( j ) (t ), onde rj (t ) = Pj (t ) Pn (t ) e S j (t ) = Q j (t ) Qm (t ) so funes racionais. Como (a (t )b (t ))' = a '(t )b(t ) + a (t )b '(t ) e (a (t ) + b(t )) ' = a '(t ) + b '(t ), segue, por induo, que, para todo k , ( f (t ) + g (t )) ( k ) pode ser escrito como combinao linear dos f ( j ) (t ),0 j n 1 e g ( j ) (t ), 0 j m 1, com coeficientes funes racionais de t gerado pelos f ( j ) (t ),0 j n 1 e pelos g ( j ) (t ), 0 j m 1.
m+n

<

Assim, deve haver uma combinao linear nula

h (t ) ( f + g )
j j =0

( j)

(t ) das

m + n +1> m + n funes ( f + g ) ( j ) (t ),0 j m + n , com coeficientes h j (t ) (t ). Multiplicando pelo m.d.c. dos denominadores dos h j (t ), concluimos que f + g bacana.
EUREKA! N19, 2004

61

Sociedade Brasileira de Matemtica

Provamos que f g bacana de modo anlogo, observando que, para todo k ( f g) pelas
(k )

pertence ao espao vetorial de dimenso m n sobre f


(i )

<,

(t ) gerado existe uma

funes

(t ) g (t ),0 i n 1,0 j m 1,
( j)

donde

combinao linear nula

l (t )( f g )
j j =0

m n

(f)

(t ), com l j (t ) (t ), j mn.

PROBLEMA 6: SOLUO DA BANCA

9n 9n tal que, para todo i x , { j n | aij = 1} . 10 10 n n linhas na matriz, cada uma delas com pelo menos De fato, se h mais de 10 10 n2 , entradas nulas, o nmero de entradas nulas da matriz ser maior que 100 absurdo. 9n Analogamente, existe y {1,2,..., n} com tal que, para todo y 10 9n j y , {i n | aij = 1 > . 10 9n 9n 4n Seja Z = X Y . Temos Z + n= . 10 10 5 9n 9n 4n Se i, j Z ,( A2 )ij + n = . Vamos mostrar, por induo, que, se 10 10 5 k 2 4 3 De fato, para todo nk 1 , i, j Z , k 2. ( Ak )ij 5 5 Existe X {1, 2,..., n} com x 4n n 4 3 4 3 nk 1 > nk , ( A )ij = ( A )ir arj ( A )ir arj 5 5 5 10 5 5 r =1 rZ n 4n n pois, de fato, {r Z | arj = 1} Z . 10 5 10 k 2 k k 4n 4 3 3 k n k k k 1 Assim, para todo k 2, tr ( A ) ( A )ii n > n > . 5 5 5 5 2 iZ
k +1 n k k k 2 k 1

EUREKA! N19, 2004

62

Sociedade Brasileira de Matemtica

XXV OLIMPADA BRASILEIRA DE MATEMTICA Resultado Nvel 1 (5a. e 6a. Sries)


NOME Henrique Pond de Oliveira Pinto Guilherme Philippe Figueiredo Rodrigo Clemente de Brito Pereira Henrique Hiroshi Motoyama Watanabe Rafael Tupinamb Dutra Rafael Rabelo de Carvalho Carolina Yumi Yida Alice Duarte Scarpa Bruna da Silveira Afonso Daniella Alves Rebouas Fbio Mallaco Moreira Carlos Renato de Andrade Figueiredo Rodrigo Bartels Luiz Gustavo Antunes Magalhes Vitor Mori Illan Feiman Halpern Vinicius Colho Machado Pollyanna Stfani Borges Freitas Danilo Takeshi Abe Jaune Gabriel Moreira Francisco Nicolas Mansur Beleski Anna Clara Leite Pestana Rafael Pacheco Gomes Lus Otvio Valente Barcellos Hannah Drummond Davico de Barros James Jun Hong Thiago da Silva Pinheiro Bernardo Duque Guimares Saraiva Guilherme Rodrigues Carvalho de Souza Tiago Paula e Silva de Holanda Cavalcanti Mateus Sampaio de Mendona Eduardo Barthel Monteiro Thales de Oliveira Gonalves Lucas Matsumoto Tominaga Nathana Alcntara Lima Lucas Cordeiro Gonalves de Carvalho Bruna Melo Coelho Loureiro Rafael Issamu Isuyama Bruna de Oliveira Neuenschwander Gabriel Somavilla Nunes Vincius Henrique Campos Senra Mariana Silva de Oliveira Hannah Menezes Lira Alessandro Wagner Palmeira Gustavo Henrique dos Santos Figueiredo Andr Bain Luara Prado Louvison Kamila Satomi Haida Dan Zylberglejd Lucas Souza Carmo Carvalho Igor Dantas Rocha Felipe Ferreira Torres CIDADE ESTADO Salvador - BA Fortaleza - CE Joo Pessoa - PB So Paulo - SP Belo Horizonte - MG Braslia - DF So Paulo - SP Goinia - GO Salvador - BA Vila Velha - ES SJ dos Campos - SP Recife - PE Porto Alegre - RS Muria - MG So Paulo - SP Itatiaia - RJ Fortaleza - CE Fortaleza - CE So Paulo - SP Santo Andr - SP Curitiba - PR Recife - PE Fortaleza - CE Sta. Rita do Passa Quatro - SP Rio de Janeiro - RJ So Paulo - SP So Paulo - SP Rio de Janeiro - RJ Rio de Janeiro - RJ Recife - PE Belo Horizonte - MG SJ dos Campos - SP Vitria - ES Diadema - SP Fortaleza - CE Salvador - BA Lauro de Freitas - BA So Paulo - SP Belo Horizonte - MG Videira - SC Belo Horizonte - MG Salvador - BA Salvador - BA Guarulhos - SP Santo Andr - SP So Paulo - SP So Paulo - SP Lucas do Rio Verde - MT Rio de Janeiro - RJ Belo Horizonte - MG Campina Grande - PB Fortaleza - CE PRMIO Ouro Ouro Ouro Ouro Ouro Prata Prata Prata Prata Prata Prata Prata Prata Prata Prata Bronze Bronze Bronze Bronze Bronze Bronze Bronze Bronze Bronze Bronze Bronze Bronze Bronze Bronze Meno Honrosa Meno Honrosa Meno Honrosa Meno Honrosa Meno Honrosa Meno Honrosa Meno Honrosa Meno Honrosa Meno Honrosa Meno Honrosa Meno Honrosa Meno Honrosa Meno Honrosa Meno Honrosa Meno Honrosa Meno Honrosa Meno Honrosa Meno Honrosa Meno Honrosa Meno Honrosa Meno Honrosa Meno Honrosa Meno Honrosa

EUREKA! N19, 2004

63

Sociedade Brasileira de Matemtica

Resultado Nvel 2 (7a. e 8a. Sries)


NOME Eduardo Fischer Guilherme Rodrigues Nogueira de Souza Edson Augusto Bezerra Lopes Enzo Haruo Hiraoka Moriyama Andr Linhares Rodrigues Leandro Farias Maia Lucio Eiji Assaoka Hossaka Jos Marcos Andrade Ferraro Gustavo Sampaio Sousa Maricy Miki Hisamoto Rafael Bandeira Lages Paulo Andr Carvalho de Melo Leonardo Esmeraldo de Aquino Felipe Gonalves Assis Fernando Nascimento Coelho Rgis Prado Barbosa Cesar Ryudi Kawakami Marlon Vieira de Lima Jnior Srgio Ricardo Furtado Sampaio Filho Felipe Ferreira Villar Coelho Iuri Lima Ribeiro Iris Chyun Mian Tseng Filipe Alves Tom Danilo Eiki Yokoyama Joo Jos Ribeiro e Silva Caio Carvalho Torres Rafael Morioka Oda Adriano Jorge Braun Vieira Neto Renato Rebouas de Medeiros Dante Mattos de Salles Soares Rafael da Silva Holanda Theodoro Ribeiro Gonalves Neto Gabriel Caser Brito Regina Reis da Costa Alves Raphael Rodrigues Mata Felipe Diz Diz Louise Rodrigues Martins Dandas Daniel de Almeida Renato Pinto Oliveira Pedro Paulo Albuquerque Goes Leonardo Simes Freire Roberto Cmara Gentil Porto Pedro Paulo Gondim Cardoso David Mosiah Terceiro Batista Vincius Marques Regitano Guilherme Hiroshigue Motomura Hashimoto Kaique Knothe de Andrade Ramon Moreira Nunes Martin Alexander Barrios Gundelach Plicia Maciel Carvalho Tiago Pellegrini Travassos Vieira CIDADE ESTADO Encantado - RS So Paulo - SP Fortaleza - CE So Paulo - SP Fortaleza - CE Fortaleza - CE Curitiba - PR So Paulo - SP Fortaleza - CE So Paulo - SP Teresina - PI Rio de Janeiro - RJ Fortaleza - CE Campina Grande - PB Fortaleza - CE Fortaleza - CE So Paulo - SP Fortaleza - CE Fortaleza - CE Serra - ES Fortaleza - CE So Paulo - SP Fortaleza - CE So Paulo - SP Salvador - BA Teresina - PI So Paulo - SP Fortaleza - CE Fortaleza - CE Rio de Janeiro - RJ Fortaleza - CE Teresina - PI Rio de Janeiro - RJ Rio de Janeiro - RJ Salvador - BA So Paulo - SP Fortaleza - CE Piracicaba - SP Rio de Janeiro - RJ Fortaleza - CE So Paulo - SP Fortaleza - CE Salvador - BA Fortaleza - CE Piracicaba - SP So Paulo - SP Rio Claro - SP Fortaleza - CE Rio de Janeiro - RJ Redeno - PA So Paulo - SP PRMIO Ouro Ouro Ouro Ouro Ouro Prata Prata Prata Prata Prata Prata Prata Prata Prata Prata Bronze Bronze Bronze Bronze Bronze Bronze Bronze Bronze Bronze Bronze Bronze Bronze Bronze Bronze Meno Honrosa Meno Honrosa Meno Honrosa Meno Honrosa Meno Honrosa Meno Honrosa Meno Honrosa Meno Honrosa Meno Honrosa Meno Honrosa Meno Honrosa Meno Honrosa Meno Honrosa Meno Honrosa Meno Honrosa Meno Honrosa Meno Honrosa Meno Honrosa Meno Honrosa Meno Honrosa Meno Honrosa Meno Honrosa

EUREKA! N19, 2004

64

Sociedade Brasileira de Matemtica

Resultado Nvel 3 (Ensino Mdio)


NOME Fbio Dias Moreira Davi Mximo Alexandrino Nogueira Rafael Daigo Hirama Henry Wei Cheng Hsu Alex Corra Abreu Felipe Rodrigues Nogueira de Souza Thiago Costa Leite Santos Rafael Marini Silva Larissa Cavalcante Queiroz de Lima Rodrigo Aguiar Pinheiro Telmo Luis Correa Junior Samuel Barbosa Feitosa Alex Cardoso Lopes Israel Franklim Dourado Carrah Gabriel Tavares Bujokas Renato Francisco Lopes Mello Murilo Vasconcelos Andrade Francisco Bruno de Lima Holanda Davi Valle Ferreira Raphael Constant da Costa Ricardo Mizoguchi Gorgoll Raul Celistrino Teixeira Elton Gomes Coriolano Thoms Yoiti Sasaki Hoshina Rodrigo Kendy Yamashita Eduardo de Moraes Rodrigues Poo Carlos Augusto David Ribeiro Germanna de Oliveira Queiroz Helder Oliveira de Castro Diogo dos Santos Suyama Guilherme Rodrigues Salerno Larissa Rodrigues Ribeiro Domingos Afono de Moura Junior Lucas de Arago Bittencourt Eduardo Martins Spina Hugo Francisco Lisboa Santos Rafael Assato Ando Andre Belem Ferreira da Silva Renato Mendes Coutinho Ayran Ayres Barbosa Loriato Vitor Gabriel Kleine Victor de Andrade Lazarte Daniel Ponciano dos Santos Barbosa Rodrigo Viana Soares Thiago Costas Casal Monteiro Silva Eduardo Vieira de Oliveira Aguiar Glauco Rocha Machado Juliana Gomes Varela Luty Rodrigues Ribeiro CIDADE ESTADO Rio de Janeiro - RJ Fortaleza - CE Campinas - SP So Paulo - SP Niteri - RJ So Paulo - SP So Paulo - SP Vila Velha - ES Fortaleza - CE Fortaleza - CE Santo Andr - SP Fortaleza - CE So Paulo - SP Fortaleza - CE So Paulo - SP Jaboato dos Guararapes - PE Macei - AL Fortaleza - CE Belo Horizonte - MG Rio de Janeiro - RJ So Paulo - SP Adamantina - SP Fortaleza - CE Rio de Janeiro - RJ Osasco - SP So Paulo - SP Fortaleza - CE Fortaleza - CE Mogi das Cruzes - SP Belo Horizonte - MG Goinia - GO Fortaleza - CE Rio de Janeiro - RJ Rio de Janeiro - RJ Jundia - SP Rio de Janeiro - RJ Campinas - SP Rio de Janeiro - RJ Americana - SP Vitria - ES Mogi das Cruzes - SP So Paulo - SP Rio de Janeiro - RJ Fortaleza - CE Rio de Janeiro - RJ Rio de Janeiro - RJ Rio de Janeiro - RJ Fortaleza - CE Fortaleza - CE PRMIO Ouro Ouro Ouro Ouro Ouro Prata Prata Prata Prata Prata Prata Prata Prata Prata Bronze Bronze Bronze Bronze Bronze Bronze Bronze Bronze Bronze Bronze Bronze Bronze Bronze Bronze Bronze Bronze Meno Honrosa Meno Honrosa Meno Honrosa Meno Honrosa Meno Honrosa Meno Honrosa Meno Honrosa Meno Honrosa Meno Honrosa Meno Honrosa Meno Honrosa Meno Honrosa Meno Honrosa Meno Honrosa Meno Honrosa Meno Honrosa Meno Honrosa Meno Honrosa Meno Honrosa

EUREKA! N19, 2004

65

Sociedade Brasileira de Matemtica

Resultado Nvel Universitrio


NOME Humberto Silva Naves Mrcio Afonso Assad Cohen Yuri Gomes Lima Carlos Stein Naves de Brito Daniel Massaki Yamamoto Rodrigo Villard Milet Eduardo Casagrande Stabel Rafael Tajra Fonteles Digo Veloso Ucha Fabrcio Siqueira Benevides Thiago da Silva Sobral Rodrigo Roque Dias Leonardo Augusto Zo Einstein do Nascimento Jnior Eduardo Famini Silva Bernardo Freitas Paulo da Costa Daniel Nobuo Uno Lucas de Melo Pontes e Silva Antonio Carlos Maldonado S. Alonso Munhoz Thiago Barros Rodrigues Costa Joo Alfredo Castellani Fajardo Freire Daniel Pinheiro Sobreira Eduardo Paiva Costa Giuliano Boava Rodrigo Angelo Muniz Tertuliano Franco Santos Franco Diego Sadao Saito Mrcio Rodrigo da Rocha Pinheiro talo Gervsio Cavalcante Estillac Lins Maciel Borges Filho Daniele Vras de Andrade Eduardo Bertoldi Domingos Dellamonica Jnior Andr Luis Hirschfeld Danila Jorge Peixoto de Morais Neto Bruno Germano Borics Rodrigo Jos Gondim Neves Gustavo Gomes Arajo Eduardo Monteiro Nicodemos CIDADE - ESTADO So Jos dos Campos - SP Rio de Janeiro - RJ Fortaleza - CE Goinia - GO So Paulo - SP Rio de Janeiro - RJ Porto Alegre - RS Teresina - PI Teresina - PI Fortaleza - CE Fortaleza - CE So Paulo - SP Nilpolis - RJ Fortaleza - CE Salvador - BA Rio de Janeiro - RJ So Paulo - SP Fortaleza - CE Rio de Janeiro - RJ Fortaleza - CE Salvador - BA Fortaleza - CE Teresina - PI Florianpolis - SC Cariacica - ES Salvador - BA Tup - SP Ananindeua - PA Teresina PI Belm - PA Rio de Janeiro - RJ SJ dos Campos - SP So Paulo - SP So Paulo - SP Goinia - GO Rio de Janeiro - RJ Jaboato dos Guararapes - PE Ribeiro Preto - SP Rio de Janeiro - RJ PRMIO Ouro Ouro Ouro Ouro Ouro Ouro Prata Prata Prata Prata Prata Prata Prata Prata Bronze Bronze Bronze Bronze Bronze Bronze Bronze Bronze Bronze Bronze Bronze Bronze Bronze Bronze Meno Honrosa Meno Honrosa Meno Honrosa Meno Honrosa Meno Honrosa Meno Honrosa Meno Honrosa Meno Honrosa Meno Honrosa Meno Honrosa Meno Honrosa

EUREKA! N19, 2004

66

Sociedade Brasileira de Matemtica

AGENDA OLMPICA
XXVI OLIMPADA BRASILEIRA DE MATEMTICA NVEIS 1, 2 e 3 Primeira Fase Sbado, 5 de junho de 2004 Segunda Fase Sbado, 11 de setembro de 2004 Terceira Fase Sbado, 16 de outubro de 2004 (nveis 1, 2 e 3) Domingo, 17 de outubro de 2004 (nveis 2 e 3 - segundo dia de prova). NVEL UNIVERSITRIO Primeira Fase Sbado, 11 de setembro de 2004 Segunda Fase Sbado, 16 e Domingo, 17 de outubro de 2004

X OLIMPADA DE MAIO 8 de maio de 2004

XV OLIMPADA DE MATEMTICA DO CONE SUL 14 a 22 de maio de 2004 Assuno, Paraguai

XLV OLIMPADA INTERNACIONAL DE MATEMTICA 06 a 18 de julho de 2004 Atenas, Grcia

X OLIMPADA INTERNACIONAL DE MATEMTICA UNIVERSITRIA 23 a 29 de julho de 2004 Skopje, Macednia

XIX OLIMPADA IBEROAMERICANA DE MATEMTICA 17 a 26 de setembro de 2004 Castelln, Espanha

VI OLIMPADA IBEROAMERICANA DE MATEMTICA UNIVERSITRIA 6 de novembro de 2004

EUREKA! N19, 2004

67

Sociedade Brasileira de Matemtica

COORDENADORES REGIONAIS
Alberto Hassen Raad Amarsio da Silva Arajo Ana Paula Bernardi da Silva Benedito Tadeu Vasconcelos Freire Carlos Frederico Borges Palmeira Claus Haetinger Cleonor Crescncio das Neves lio Mega Florncio Ferreira Guimares Filho Gil Cunha Gomes Filho Ronaldo Alves Garcia Reginaldo de Lima Pereira Ivanilde Fernandes Saad Jacqueline Fabiola Rojas Arancibia Janice T. Reichert Joo Bencio de Melo Neto Joo Francisco Melo Libonati Jos Carlos dos Santos Rodrigues Jos Cloves Saraiva Jos Gaspar Ruas Filho Jos Luiz Rosas Pinho Jos Vieira Alves Licio Hernandes Bezerra Luzinalva Miranda de Amorim Mrio Rocha Retamoso Marcelo Rufino de Oliveira Marcelo Mendes Pablo Rodrigo Ganassim Ramn Mendoza Ral Cintra de Negreiros Ribeiro Reinaldo Gen Ichiro Arakaki Ricardo Amorim Srgio Cludio Ramos Tadeu Ferreira Gomes Toms Menndez Rodrigues Valdenberg Arajo da Silva Valdeni Soliani Franco Vnia Cristina Silva Rodrigues Wagner Pereira Lopes (UFJF) (UFV) (Universidade Catlica de Braslia) (UFRN) (PUC-Rio) (UNIVATES) (UTAM) (Colgio Etapa) (UFES) (Colgio ACAE) (UFGO) (Escola Tcnica Federal de Roraima) (UC. Dom Bosco) (UFPB) (UNOCHAPEC) (UFPI) (Grupo Educacional Ideal) (Unespar) (UFMA) (ICMC-USP) (UFSC) (UFPB) (UFSC) (UFBA) (UFRG) (Grupo Educacional Ideal) (Colgio Farias Brito, Pr-vestibular) (Liceu Terras do Engenho) (UFPE) (Colgio Anglo) (INPE) (Centro Educacional Logos) (IM-UFRGS) (UEBA) (U. Federal de Rondnia) (U. Federal de Sergipe) (U. Estadual de Maring) (U. Metodista de SP) (CEFET GO) Juiz de Fora MG Viosa MG Braslia DF Natal RN Rio de Janeiro RJ Lajeado RS Manaus AM So Paulo SP Vitria ES Volta Redonda RJ Goinia GO Boa Vista RR Campo Grande MS Joo Pessoa PB Chapec SC Teresina PI Belm PA Campo Mouro PR So Luis MA So Carlos SP Florianpolis SC Campina Grande PB Florianpolis SC Salvador BA Rio Grande RS Belm PA Fortaleza CE Piracicaba SP Recife PE Atibaia SP SJ dos Campos SP Nova Iguau RJ Porto Alegre RS Juazeiro BA Porto Velho RO So Cristovo SE Maring PR S.B. do Campo SP Jata GO

EUREKA! N19, 2004

68

CONTEDO

AOS LEITORES X OLIMPADA DE MAIO Enunciados e Resultado Brasileiro XV OLIMPADA DE MATEMTICA DO CONE SUL Enunciados e Resultado Brasileiro XLV OLIMPADA INTERNACIONAL DE MATEMTICA Enunciados e Resultado Brasileiro XIX OLIMPADA IBERO-AMERICANA DE MATEMTICA Enunciados e Resultado Brasileiro XI OLIMPADA INTERNACIONAL DE MATEMTICA UNIVERSITRIA Enunciados e Resultado Brasileiro

2 3

7 9

11

13

ARTIGOS
O TRINGULO E SUAS PRINCIPAIS CIRCUNFERNCIAS Eduardo Wagner DOIS PROBLEMAS CHINESES SOBRE GEOMETRIA PROJETIVA Helder Oliveira de Castro RETA DE EULER E NMEROS COMPLEXOS Jos Paulo Carneiro COMO QUE FAZ? SOLUES DE PROBLEMAS PROPOSTOS PROBLEMAS PROPOSTOS COORDENADORES REGIONAIS 17

26 31

37 40 60 62

Sociedade Brasileira de Matemtica

AOS LEITORES
Neste nmero apresentamos os resultados das equipes brasileiras e os problemas propostos na X Olimpada de maio, na XV Olimpada do Cone Sul, na XLV Olimpada Internacional (IMO), na XI Olimpada Internacional para Estudantes Universitrios (IMC) e na XIX Olimpada Ibero-americana. Realmente temos muito a comemorar: o primeiro colocado na Cone Sul, a maior nota do Ocidente no IMC (lembrem-se de que boa parte da Europa e os Estados Unidos ficam no Ocidente!), mais uma vez todos os integrantes de nossa equipe conquistaram medalhas na IMO, colocando o Brasil frente de diversos pases de grande tradio matemtica, como a Frana e a Alemanha e fomos o primeiro pas a conquistar 4 medalhas de ouro na Ibero. Voc ainda poder ler trs excelentes artigos de Geometria, com os quais certamente voc aprender muito. No se esquea de que, caso no consiga entender algum agora (ou mesmo todos, no h problema), vale a pena retornar a eles depois. Agradecemos as solues de problemas propostos e os novos problemas enviados pelos nossos leitores, que continuamos estimulando a colaborar com a Eureka!. Agradecemos finalmente a Ccero Thiago Magalhes de Fortaleza CE e a Wilberson Ivo Della Nina de So Jos dos Campos SP que colaboraram com a reviso deste nmero.

Os editores

EUREKA! N20, 2004

Sociedade Brasileira de Matemtica

X OLIMPADA DE MAIO
Enunciados e Resultado Brasileiro PRIMEIRO NVEL
Durao da Prova: 3 horas PROBLEMA 1

Xavier multiplica quatro dgitos, no necessariamente distintos, e obtm um nmero terminado em 7. Determine quanto pode valer a soma dos quatros dgitos multiplicados por Xavier. D todas as possibilidades.
PROBLEMA 2

No interior de um quadrado 11 11, Pablo desenhou um retngulo e prolongando seus lados dividiu o quadrado em 5 retngulos, como mostra a figura. Sofia fez o mesmo, conseguindo, alm disso, que os comprimentos dos lados dos 5 retngulos fossem nmeros inteiros entre 1 e 10, todos distintos. Mostre uma figura como a que Sofia fez.
PROBLEMA 3

Em cada casa de um tabuleiro 5 5 est escrito 1 ou 1. Em cada passo troca-se o nmero de cada uma das 25 casas pelo resultado da multiplicao dos nmeros de todas as suas casas vizinhas. Inicialmente se tem o tabuleiro da figura. Mostre como fica o tabuleiro ao final de 2004 passos. Observao: Duas casas so vizinhas se tiverem um lado em comum.
PROBLEMA 4

1 1 1 1 1

1 1 1 1 1

1 1 1 1 1

1 1 1 1 1

1 1 1 1 1

Em um quadrado ABCD de diagonais AC e BD, chamamos de O o centro do quadrado. Constri-se um quadrado PQRS de lados paralelos aos de ABCD com P no segmento AO, Q no segmento BO, R no segmento CO, S no segmento DO. Se rea (ABCD) = 2rea(PQRS) e M o ponto mdio do lado AB, calcule a medida do ngulo AM P . (No vale medir.)
EUREKA! N20, 2004

Sociedade Brasileira de Matemtica

PROBLEMA 5

Tem-se 90 cartes e em cada um esto escritos dois dgitos distintos: 01, 02, 03, 04, 05, 06, 07, 08, 09, 10, 12, e assim sucessivamente at 98. Um conjunto de cartes correto se no contm nenhum carto que tenha o primeiro dgito igual ao segundo dgito de outro carto do conjunto. Chamamos valor de um conjunto de cartes a soma dos nmeros escritos em cada carto. Por exemplo, os quatros cartes 04, 35, 78 e 98 formam um conjunto correto e seu valor 215, pois 04 + 35 + 78 + 98 = 215. Encontre um conjunto correto que tenha o maior valor possvel. Explique por que impossvel obter um conjunto correto de maior valor.

SEGUNDO NVEL
Durao da Prova: 3 horas PROBLEMA 1

Juliano escreveu cinco nmeros inteiros positivos, no necessariamente distintos, tais que seu produto seja igual sua soma. Quais podem ser os nmeros que Juliano escreveu?
PROBLEMA 2

A me de Zezinho quer preparar n pacotes de 3 balas para dar de presente na festa de aniversrio, e para isto comprar balas sortidas de 3 sabores diferentes. Ela pode comprar qualquer nmero de balas, mas no pode escolher quantas so de cada sabor. Ela quer colocar em cada pacote uma bala de cada sabor, e se isto no for possvel usar somente balas de um sabor e todos os pacotes tero 3 balas desse sabor. Determine o menor nmero de balas que ela deve comprar para poder preparar os n pacotes. Explique por que se ela compra menos balas no ter a certeza de poder preparar os pacotes como ela quer.
PROBLEMA 3

Temos uma mesa de bilhar de 8 metros de comprimento e 2 metros de largura, com uma nica bola no centro. Lanamos a bola em linha reta e, depois de percorrer 29 metros, ela pra numa esquina da mesa. Quantas vezes a bola rebateu nas bordas da mesa? Nota: Quando a bola rebate na borda da mesa, os dois ngulos que formam sua trajetria com a borda da mesa so iguais.

EUREKA! N20, 2004

Sociedade Brasileira de Matemtica

PROBLEMA 4

Ache todos os nmeros naturais x, y, z que verificam simultaneamente x y z = 4104 x + y + z = 77


PROBLEMA 5

Sobre um tabuleiro 9 9, dividido em casas 1 1, se colocam sem superposies e sem sair do tabuleiro, peas da forma

Cada pea cobre exatamente 3 casas. a) A partir do tabuleiro vazio, qual a mxima quantidade de peas que se pode colocar? b) A partir do tabuleiro com 3 peas e colocadas como mostra o diagrama seguinte,

qual a mxima quantidade de peas que se pode colocar?

EUREKA! N20, 2004

Sociedade Brasileira de Matemtica

RESULTADOS
PRIMEIRO NVEL (At 13 anos)
Gustavo Henrique dos Santos Figueiredo Vincius Henrique Campos Senra Rafael Pacheco Gomes Danilo Takeshi Abe Jaune Ilan Feiman Halpern Emanuelle Meneses Barros Dayana Basilio Batista Guilherme Albuquerque Pinto Rebello Bernardo Duque Guimares Saraiva Amanda Maria Barradas M. de Santana Medalha de Ouro Medalha de Prata Medalha de Prata Medalha de Bronze Medalha de Bronze Medalha de Bronze Medalha de Bronze Meno Honrosa Meno Honrosa Meno Honrosa Santo Andr - SP Belo Horizonte - MG Fortaleza - CE So Paulo - SP Itatiaia - RJ Fortaleza - CE Campo Grande - MS Rio de Janeiro - RJ Rio de Janeiro - RJ Teresina - PI

SEGUNDO NVEL (At 15 anos)


Eduardo Fischer Lucio Eiji Assaoka Hossaka Guilherme Nogueira de Souza Jos Marcos Andrade Ferraro Paulo Andr Carvalho de Melo Rodrigo Clemente de Brito Pereira Henrique Pond de Oliveira Pinto Rafael Tupinamb Dutra Amanda Freitas Santos Edson Augusto Bezerra Lopes Medalha de Ouro Medalha de Prata Medalha de Prata Medalha de Bronze Medalha de Bronze Medalha de Bronze Medalha de Bronze Meno Honrosa Meno Honrosa Meno Honrosa Encantado - RS Curitiba - PR So Paulo - SP So Paulo - SP Rio de Janeiro - RJ Joo Pessoa - PB Salvador - BA Belo Horizonte - MG Rio de Janeiro - RJ Fortaleza - CE

EUREKA! N20, 2004

Sociedade Brasileira de Matemtica

XV OLIMPADA DE MATEMTICA DO CONE SUL


Enunciados e Resultado Brasileiro
A XV Olimpada de Matemtica do Cone Sul foi realizada na cidade de Caaguaz, Paraguai no perodo de 14 a 23 de Maio de 2004. A equipe brasileira foi liderada pelos professores Pablo Rodrigo Ganassim (So Paulo SP) e Mrcio Cohen (Rio de Janeiro RJ).
RESULTADOS DA EQUIPE BRASILEIRA BRA1 BRA2 BRA3 BRA4 Gabriel Tavares Bujokas Leandro Farias Maia Andr Linhares Rodrigues Telmo Luis Correa Jnior Medalha de Ouro Medalha de Prata Medalha de Bronze Medalha de Bronze

PROBLEMA 1

Maxi escolheu 3 dgitos e, fazendo todas as permutaes possveis, obteve 6 nmeros distintos, cada um com 3 dgitos. Se exatamente um dos nmeros que Maxi obteve um quadrado perfeito e exatamente trs so primos, encontrar os 3 dgitos que Maxi escolheu. D todas as possibilidades para os 3 dgitos.
PROBLEMA 2

Dada uma circunferncia C e um ponto P exterior a ela, traam-se por P as duas tangentes circunferncia, sendo A e B os pontos de tangncia. Toma-se um ponto Q sobre o menor arco AB de C. Seja M a interseo da reta AQ com a perpendicular a AQ traada por P, e seja N a interseo da reta BQ com a perpendicular a BQ traada por P. Demonstre que, ao variar Q no arco AB, todas as retas MN passam por um mesmo ponto.
PROBLEMA 3

Seja n um inteiro positivo. Chamamos Cn a quantidade de inteiros positivos x, menores que 10n, tais que a soma dos dgitos de 2x menor que a soma dos dgitos de x. 4 Demonstre que Cn _ (10n 1) . 9

EUREKA! N20, 2004

Sociedade Brasileira de Matemtica

PROBLEMA 4

Arnaldo escolhe um inteiro a, a _ 0 073, /4 08.4 0 :2 390 74 b, b _ Ambos dizem, em segredo, o nmero que escolheram a Cernaldo, e este escreve em um quadro os nmeros 5, 8 e 15, sendo um desses a soma a + b. Cernaldo toca uma campainha e Arnaldo e Bernaldo, individualmente, escrevem em papis distintos se sabem ou no qual dos nmeros no quadro a soma de a e b, e entregam seus papis para Cernaldo. Se em ambos os papis est escrito NO, Cernaldo toca novamente a campainha, e o procedimento se repete. Sabe-se que Arnaldo e Bernaldo so sinceros e inteligentes. Qual o nmero mximo de vezes que a campainha pode ser tocada at que um deles escreva que sabe o valor da soma?
PROBLEMA 5

Utilizando triangulinhos eqilteros de papel, de lado 1, forma-se um tringulo eqiltero de lado 2 2004 . Desse tringulo retira-se o triangulinho de lado 1 cujo centro coincide com o centro do tringulo maior. Determine se possvel cobrir totalmente a superfcie restante, sem superposies nem buracos, dispondo-se somente de fichas em forma de trapzio issceles, cada uma formada por trs triangulinhos eqilteros de lado 1.
PROBLEMA 6

Sejam m, n inteiros positivos. Em um tabuleiro m n, quadriculado em quadradinhos de lado 1, considere todos os caminhos que vo do vrtice superior direito ao inferior esquerdo, percorrendo as linhas do quadriculado exclusivamente nas direes e . Define-se a rea de um caminho como sendo a quantidade de quadradinhos do tabuleiro que h abaixo desse caminho. Seja p um primo tal que rp(m) + rp(n) p, onde rp(m) representa o resto da diviso de m por p e rp(n) representa o resto da diviso de n por p. Em quantos caminhos a rea um mltiplo de p?

EUREKA! N20, 2004

Sociedade Brasileira de Matemtica

XLV OLIMPADA INTERNACIONAL DE MATEMTICA


Enunciados e Resultado Brasileiro
A XLV Olimpada Internacional de Matemtica foi realizada na cidade de Atenas, Grcia no perodo de 06 a 18 de julho de 2004. A equipe brasileira foi liderada pelos professores Carlos Gustavo Moreira (Rio de Janeiro RJ) e Carlos Yuzo Shine (So Paulo SP).
RESULTADOS DA EQUIPE BRASILEIRA BRA1 BRA2 BRA3 BRA4 BRA5 BRA6 Fbio Dias Moreira Gabriel Tavares Bujokas Henry Wei Cheng Hsu Rafael Daigo Hirama Rafael Marini Silva Thiago Costa Leite Santos Medalha de Bronze Medalha de Prata Medalha de Bronze Medalha de Prata Medalha de Bronze Medalha de Bronze

PRIMEIRO DIA
PROBLEMA 1

Seja ABC um tringulo acutngulo com AB AC. A circunferncia de dimetro BC intersecta os lados AB e AC nos pontos M e N, respectivamente. Seja O o ponto mdio do lado BC. As bissetrizes dos ngulos BAC e MON intersectam-se em R. Prove que as circunferncias circunscritas aos tringulos BMR e CNR tm um ponto em comum que pertence ao lado BC.

PROBLEMA 2

Determine todos os polinmios P(x) de coeficientes reais que satisfazem a igualdade


P( a b) + P(b c) + P(c a ) = 2 P( a + b + c )

para quaisquer nmeros reais a, b, c, tais que ab + bc + ca = 0.

EUREKA! N20, 2004

Sociedade Brasileira de Matemtica

PROBLEMA 3

Um gancho uma figura formada por seis quadrados unitrios como no seguinte diagrama

ou qualquer uma das figuras obtidas desta aplicando rotaes ou reflexes. Determine todos os retngulos m n que podem ser cobertos com ganchos de modo que: i) O retngulo coberto sem buracos e sem sobreposies; ii) Nenhuma parte de nenhum gancho pode cobrir regies fora do retngulo. SEGUNDO DIA
PROBLEMA 4 Seja n 3 um inteiro. Sejam t1 , t2 ,..., tn nmeros reais positivos tais que

1 1 1 n 2 + 1 > (t1 + t2 + ... + tn ) + + ... + . tn t1 t2

Mostre que ti , t j e tk so as medidas dos lados de um tringulo para quaisquer i, j, k com 1 i < j < k n.
PROBLEMA 5

Num quadriltero convexo ABCD a diagonal BD no bissetriz do ngulo ABC nem do ngulo CDA . Um ponto P no interior de ABCD satisfaz PBC = DBA e PDC = BDA. Prove que os vrtices do quadriltero ABCD pertencem a uma mesma circunferncia se e s se AP = CP.
PROBLEMA 6

Um inteiro positivo dito alternante se, na sua representao decimal, quaisquer dois dgitos consecutivos tm paridade diferente. Determine todos os inteiros positivos n tais que n tem um mltiplo que alternante.

EUREKA! N20, 2004

10

Sociedade Brasileira de Matemtica

XIX OLIMPADA IBERO-AMERICANA DE MATEMTICA


Enunciados e Resultado Brasileiro
A XIX Olimpada Ibero-americana de Matemtica foi realizada na cidade de Castelln, Espanha no perodo de 17 a 26 de setembro de 2004. A equipe brasileira foi liderada pelos professores Eduardo Wagner e Luciano Guimares Monteiro de Castro, ambos do Rio de Janeiro RJ.
RESULTADOS DA EQUIPE BRASILEIRA BRA1 BRA2 BRA3 BRA4 Alex Corra Abreu Fbio Dias Moreira Gabriel Tavares Bujokas Rafael Daigo Hirama Medalha de Ouro Medalha de Ouro Medalha de Ouro Medalha de Ouro

PRIMEIRO DIA
PROBLEMA 1

Deve-se colorir as casas de um tabuleiro 1001 1001 de acordo com as seguintes regras: Se duas casas tm um lado comum, ento pelo menos uma delas deve ser colorida. De cada seis casas consecutivas de uma linha ou de uma coluna, devem colorir-se sempre pelo menos duas delas que sejam adjacentes. Determinar o nmero mnimo de casas que devem ser coloridas.

PROBLEMA 2

Considera-se no plano uma circunferncia de centro O e raio r, e um ponto A exterior a ela. Seja M um ponto da circunferncia e N o ponto diametralmente oposto a M. Determinar o lugar geomtrico dos centros das circunferncias que passam por A, M e N quando M varia.
PROBLEMA 3

Sejam n e k nmeros inteiros positivos tais que n mpar ou n e k so pares. Provar que existem inteiros a e b tais que mdc (a, n) = mdc(b, n) = 1 e k = a + b.
EUREKA! N20, 2004

11

Sociedade Brasileira de Matemtica

PROBLEMA 4

Determinar todos os pares (a, b), onde a e b so nmeros inteiros positivos de dois dgitos cada um, tais que 100a + b e 201a + b so quadrados perfeitos de quatro dgitos.

PROBLEMA 5

Dado um tringulo escaleno ABC, designam-se por A', B', C' os pontos de interseo das bissetrizes interiores dos ngulos A, B e C com os lados opostos, respectivamente. Sejam: A'' a interseo de BC com a mediatriz de AA', B'' a interseo de AC com a mediatriz de BB' e C'' a interseo de AB com a mediatriz de CC'. Provar que A'', B'' e C'' so colineares.
PROBLEMA 6

Para um conjunto H de pontos no plano, diz-se que um ponto P do plano um ponto de corte de H, se existem quatro pontos distintos A, B, C e D em H tais que as retas AB e CD so distintas e se cortam em P. Dado um conjunto finito A0 de pontos no plano, constri-se uma sucesso de conjuntos A1 , A2 , A3 ,... da seguinte forma: para qualquer j 0, Aj +1 a unio de Aj com o conjunto de todos os pontos de corte de Aj . Demonstrar que se a unio de todos os conjuntos da sucesso um conjunto finito ento, para qualquer j 1, tem-se Aj = A1 .

EUREKA! N20, 2004

12

Sociedade Brasileira de Matemtica

XI OLIMPADA INTERNACIONAL DE MATEMTICA PARA ESTUDANTES UNIVERSITRIOS


Enunciados e Resultado Brasileiro
A XI Olimpada Internacional de Matemtica para estudantes universitrios foi realizada na cidade de Skopje, Macednia no perodo de 23 a 29 de julho de 2004. A equipe brasileira foi liderada pelo professor Fernando Pimentel, da cidade de Fortaleza CE.
RESULTADOS DA EQUIPE BRASILEIRA
Yuri Gomes Lima Humberto Silva Naves Carlos Stein Naves de Brito Alex Corra Abreu Eduardo Casagrande Stabel Murilo Vasconcelos de Andrade Rafael Tajra Fonteles Thiago Barros Rodrigues Costa Digo Veloso Ucha Eduardo Famini Silva Tertuliano Franco Santos Franco UFC ITA ITA UFRJ UFRGS IME UFPI Unicamp IME IME UFBA

Medalha de Ouro
Medalha de Prata Medalha de Prata Medalha de Prata Medalha de Bronze Medalha de Bronze Medalha de Bronze Meno Honrosa Meno Honrosa Meno Honrosa Meno Honrosa

PRIMEIRO DIA
PROBLEMA 1

Seja S um conjunto infinito de nmeros reais tal que x1 + x2 + ... + xn 1 para todo subconjunto finito {x1 , x2 ,..., xn } S . Demonstre que S enumervel.

PROBLEMA 2

Seja f1 ( x) = x 2 1, e para cada inteiro positivo n 2 defina f n ( x ) = f n 1 ( f1 ( x )). Quantas razes reais distintas tem o polinmio f 2004 ?

EUREKA! N20, 2004

13

Sociedade Brasileira de Matemtica

PROBLEMA 3

Seja An o conjunto de todas as somas

arcsin x ,
k k =1

onde n 2, xk [0,1], e

x
k =1

= 1.

i) Prove que An um intervalo.

lim an . ii) Seja an o comprimento do intervalo An . Calcule n


PROBLEMA 4

, de maneira Suponha n 4 e seja S um conjunto finito de pontos no espao que quaisquer quatro de seus pontos no sejam coplanares. Suponha que todos os pontos de S podem ser coloridos de vermelho e azul de modo que qualquer esfera que intersecte S em ao menos 4 pontos tenha a propriedade de que exatamente a metade dos pontos na interseo de S com a esfera azul. Prove que todos os pontos de S encontram-se numa esfera.
3

PROBLEMA 5

Seja S um conjunto de

Prove que onde existe uma seqncia montona {ai }1i n + 2 S tal que

2n + 1 nmeros reais, onde n um inteiro positivo. n xi +1 x1 2 xi x1 ,

para todo i = 2, 3,, n.


PROBLEMA 6

Para cada nmero complexo z diferente de 0 e 1 definimos a seguinte funo:

f ( z) =

1 log 4 z

onde a soma sobre todos os ramos do logaritmo complexo. i) Prove que h dois polinmios P e Q tais que f ( z ) =

P( z ) para todo Q( z )

{0,1}.

EUREKA! N20, 2004

14

Sociedade Brasileira de Matemtica

ii) Prove que para todo z

{0,1} temos f (z) = z3 + 4z 2 + z . 6( z 1)4

SEGUNDO DIA
PROBLEMA 7

Seja A uma matriz real 4 2 e B uma matriz real 2 4 tal que


1 0 1 0 0 1 0 1 . AB = 1 0 1 0 0 1 0 1

Encontre BA.
PROBLEMA 8

Sejam f , g :[a, b] [0, ) duas funes continuas no decrescentes tais que para cada x [a, b] temos

Prove que

f (t ) dt

g (t ) dt e

b a

f (t ) dt = 1 + g (t ) dt.

g (t ) dt.

PROBLEMA 9

1 + f (t ) dt

Seja D um disco unitrio fechado, e sejam z1 , z2 ,..., z n pontos fixados em D. Prove que existe um ponto z em D tal que a soma das distancias desde z a cada um dos n pontos maior ou igual que n.

EUREKA! N20, 2004

15

Sociedade Brasileira de Matemtica

PROBLEMA 10

Para n 1 seja M uma matriz complexa n n com autovalores 1 , 2 ,..., k , distintos com respectivas multiplicidades m1 , m2 ,..., mk . Considere o operador
T linear LM definido por LM X = MX + XM , para qualquer X matriz complexa

n n . Encontre os autovalores de LM e suas multiplicidades.


PROBLEMA 11

Prove que

PROBLEMA 12

1 1

dx dy 1 + log y 1 x

0 0

1.

Para n 0 defina as matrizes An e Bn como segue: A0 = B0 = (1), e, para cada n > 0,

An An = 1 An 1

An 1 An 1 e Bn = Bn 1 An 1

An 1 . 0

Denote por S(M) a soma de todos os elementos da matriz M. Prove que


k 1 S ( An ) = S ( Akn1 ) , para quaisquer n, k 2.

EUREKA! N20, 2004

16

Sociedade Brasileira de Matemtica

O TRINGULO E SUAS PRINCIPAIS CIRCUNFERNCIAS


Eduardo Wagner, Rio de Janeiro - RJ Nvel Iniciante
Vamos tratar neste artigo das circunferncias inscrita, circunscrita e exinscritas de um tringulo. Mostraremos diversas propriedades, relaes interessantes e alguns problemas. Em todo o artigo, o tringulo ABC possui lados AB = c, BC = a e CA = b. O seu semipermetro p e sua rea S. Ser necessrio que o leitor conhea a frmula de Heron para rea do tringulo em funo de seus lados: S = p( p a)( p b)( p c ) .
A circunferncia inscrita

A circunferncia inscrita tem centro I, incentro do tringulo, que o ponto de interseo das bissetrizes internas. A

b c I r B a C

A rea do tringulo ABC a soma das reas dos tringulos AIB, BIC e CIA, que possuem altura igual a r, raio da circunferncia inscrita. Portanto,

S=

cr ar br a + b + c + + = r = pr 2 2 2 2
S = pr

A nossa primeira relao :

EUREKA! N20, 2004

17

Sociedade Brasileira de Matemtica

que permite calcular o raio da circunferncia inscrita em um tringulo em funo de seus lados. A circunferncia circunscrita Considere agora o tringulo ABC inscrito em uma circunferncia de raio R. Seja AH = h uma altura e seja AD um dimetro dessa circunferncia.
A

c h 2R B H

Os tringulos AHB e ACD so semelhantes uma vez que os ngulos AHB e ACD so retos e os ngulos ABC e ADC so iguais pois subtendem o mesmo arco. Logo,

AB AH = AD AC c h = 2R b
ou seja, bc = 2Rh. Multiplicando pelo comprimento do lado BC os dois lados, temos abc = 2Rah. Mas ah o dobro da rea do tringulo ABC e assim encontramos a nossa segunda relao : abc = 4RS Ela permite calcular o raio da circunferncia circunscrita a um tringulo em funo dos seus lados.

EUREKA! N20, 2004

18

Sociedade Brasileira de Matemtica

As circunferncias exinscritas

A circunferncia exinscrita relativa ao vrtice A do tringulo ABC tangente ao lado BC e s retas AB e AC. Seu raio ser designado por ra e seu centro por IA , chamado de exincentro (ou excentro) relativo ao vrtice A do tringulo ABC. O ponto IA a interseo da bissetriz interna de A e das bissetrizes externas de B e C. As outras duas circunferncias exinscritas e os dois outros exincentros so definidas de forma anloga.

C IA b a ra

A rea do tringulo ABC igual a rea de ABIA mais a rea de ACIA menos a rea de BCIA. Assim,

S=

cra bra ara b + c a + = ra . 2 2 2 2 S = ra ( p a ) S = rb ( p b ) S = rc ( p c)

Observe que b + c a = a + b + c 2a = 2p 2a = 2(p a). Logo, a nossa nova relao : e, analogamente, temos

que permitem calcular os raios das circunferncias exinscritas em funo dos lados do tringulo ABC. Para fixar o que apresentamos at aqui, resolva o problema a seguir.

EUREKA! N20, 2004

19

Sociedade Brasileira de Matemtica

Problema 1: Em um tringulo de lados 5, 7, e 8, calcule os raios das circunferncias

inscrita, circunscrita e exinscritas. 7 3 10 3 5 3 , 2 3, , . Respostas: 3, 3 7 4


Duas relaes

Primeira: S = r ra rb rc Esta fcil de demonstrar. Multiplicando as relaes da circunferncia inscrita e das exinscritas obtemos:

S = r ra rb rc p( p a)( p b)( p c ) = r ra rb rc S
o que demonstra a relao. Segunda:

1 1 1 1 = + + r ra rb rc

Observe que

S S S S + + = p a + p b + p c = 3p (a + b + c) = p = ra rb rc r
que demonstra a relao.

Problema: 2 Existe um tringulo cujas circunferncias exinscritas tenham raios 1cm, 2cm e 6cm? 4 5 7 5 9 5 , , centmetros. Resposta: sim. os lados medem 5 5 5 Os pontos de tangncia

Vamos agora localizar os pontos de tangncia das circunferncias inscrita e exinscrita em relao da cada um dos lados. Consideremos inicialmente a circunferncia inscrita tangenciando os lados AB, BC e CA nos pontos L, M e N, respectivamente.

EUREKA! N20, 2004

20

Sociedade Brasileira de Matemtica

A N

Sejam AL = AN = x, BL = BM = y, CM = CN = z. Temos ento o sistema: x+y=c y+z=a z+x=b que resolvido d AL = AN = p a, BL = BM = p b, CM = CN = p c. Considerando uma das circunferncias exinscritas como mostra a figura a seguir temos:
R A

Q B C P

o permetro do tringulo ABC 2p = BA + AC + BC = BA + AQ + CQ + BC = BA + AR + CP + BC = BR + BP = 2BP. Logo, BP = p, o semipermetro do tringulo. Uma desigualdade interessante Em todo tringulo ABC, r ra

a2 . 4

Esta desigualdade, alm de interessante pelo seu aspecto, vai ser til para a resoluo de outros problemas. Observe a figura a seguir.

EUREKA! N20, 2004

21

Sociedade Brasileira de Matemtica

B J

I r A D C a

ra

Na figura acima, I o incentro de ABC e J o exincentro relativo ao vrtice A. Sabemos pelo tem anterior que CD = p c e que AE = p. Logo, CE = p b e portanto, DE = p c + p c = 2p (b + c) = a. No tringulo retngulo IJF temos IJ r + ra , valendo a igualdade se, e somente se AB = AC. Portanto,

(r + ra ) 2 IJ 2 = a 2 + (ra r)2 2 r ra = a2 2r ra a2 r ra 4
como queramos demonstrar. Repare que a igualdade ocorre se, e somente se, o tringulo ABC issceles com vrtice A.
A desigualdade entre os raios das circunferncias inscrita e circunscrita

Em qualquer tringulo,

r 1 . R 2

Esta linda desigualdade intrigante, pois afirma que o raio da circunferncia circunscrita no menor que o dobro do raio da circunferncia inscrita. H diversas demonstraes desta desigualdade; todas muito engenhosas. Mas, seguindo o que estamos desenvolvendo neste artigo, vamos apresentar a demonstrao seguinte. Considerando a desigualdade que acabamos de demonstrar, temos:
EUREKA! N20, 2004

22

Sociedade Brasileira de Matemtica

a2 4 b2 r rb 4 c2 r rc 4 r ra
Multiplicando estas trs relaes temos:

r 2 r ra rb rc

a2 b 2 c 2 64 2 (4 ) RS r 2S 2 64 r 1 R 2

A pergunta natural que devemos fazer quando vale a igualdade. Repare que na demonstrao da desigualdade r ra

a2 , a igualdade vale se, e somente se, AB = 4 r 1 = ocorre se, e somente se o tringulo R 2

AC, quando as circunferncias inscrita e exinscrita relativa ao vrtice A so tangentes no ponto mdio do lado BC. Utilizando o mesmo argumento para as outras desigualdades, conclumos que ABC equiltero.
Problema 3

Sabendo que em um tringulo ABC, sin demonstrar mais tarde), mostre que sin
A relao dos cinco raios

( p b)( p c ) A = (isto voc poder 2 bc

A B C 1 sin sin . 2 2 2 8

Os raios das circunferncias inscrita, circunscrita e exinscritas esto ligados pela relao:
EUREKA! N20, 2004

23

Sociedade Brasileira de Matemtica

ra + rb + rc r = 4 R
Para demonstrar isto, necessitamos apenas de resultados anteriores e de alguma manipulao algbrica.

rb + rc =

S S aS + = p b p c ( p b)( p c ) ra r = S S pa p

Somando,

1 1 + ra + rb + rc r = aS ( p b)( p c) p( p a)
= aS

p( p a) + ( p b)( p c) p( p a)( p b)( p c ) 2 p 2 p(a + b + c ) + bc S2

= aS

abc = 4R S

O assunto no tem fim. H muitssimas outras relaes entre os elementos de um tringulo e suas principais circunferncias; algumas legais e outras desinteressantes. Mas, nosso objetivo foi fornecer um material bsico para que os alunos iniciantes possam se desenvolver e, por isso, paramos aqui. Para fixar as idias, voc poder curtir uns probleminhas bacanas na lista abaixo.
Problemas suplementares Problema 4

Em um tringulo ABC com incentro I, a bissetriz interna do ngulo A encontra a circunferncia circunscrita em E. Prove que EB = EC = EI.

EUREKA! N20, 2004

24

Sociedade Brasileira de Matemtica

Problema 5

Dados um ngulo agudo XOY, um ponto P exterior e um nmero positivo k (como sugerido na figura abaixo), mostre como se pode construir uma reta que passe por P e que corte os lados do ngulo dado formando um tringulo de permetro k.
Y

O P Problema 6

Em um tringulo acutngulo, mostre que o simtrico do ortocentro em relao a um lado pertence a circunferncia circunscrita ao tringulo .
Problema 7 (de uma olimpada internacional)

O tringulo acutngulo ABC est inscrito em uma circunferncia. Sejam M, N e P os pontos mdios dos arcos AB, BC e CA, respectivamente. Prove que a rea do hexgono AMBNCP maior ou igual que o dobro da rea do tringulo ABC.
Problema 8

Em um quadriltero convexo ABCD, as bissetrizes dos ngulos A e B cortam-se em M, as bissetrizes dos ngulos C e D cortam-se em N e as retas AD e BC cortam-se em P. Mostre que os pontos M, N e P so colineares.
Problema 9

Em um tringulo ABC com incentro I, e exincentros J, K, L, mostre que I o ortocentro do tringulo JKL.
Problema 10

Em um tringulo, mostre que a distncia do ortocentro a um vrtice o dobro da distncia do circuncentro ao lado oposto. Mostre a seguir que o ortocentro, o baricentro e o circuncentro so colineares.
Problema 11 (este difcil)

Em um tringulo ABC, AX uma bissetriz (X BC), N o ponto mdio de AX, e M o ponto mdio de BC. Sendo I o incentro do tringulo, mostre que M, I e N so colineares.
EUREKA! N20, 2004

25

Sociedade Brasileira de Matemtica

DOIS PROBLEMAS CHINESES SOBRE GEOMETRIA PROJETIVA


Helder Oliveira de Castro, So Paulo - SP Nvel Avanado
INTRODUO:

Para aqueles que nunca tiveram uma aula sobre esse assunto, depois de consultarem [1] ou at alguns problemas em [2], podem ficar meio em dvida sobre como desenvolver essa poderosa ferramenta e apelarem para outros mtodos. muito interessante quando, depois de horas e mais horas fazendo centenas de clculos intrincados de trigonometria e geometria analtica e gastando algumas dzias de folhas de almao, desistimos de um problema de geometria sem ter chegado a lugar algum. Motivado (na verdade irritado) por isso comecei a estudar Geometria Projetiva, e com esse intuito que gostaria de expor dois problemas nos quais so exploradas tcnicas de soluo por polaridade, fornecendo bases para que o leitor possa aplic-las em outras situaes. Mas para comear necessrio retomar algumas definies de [1], que servem de alicerce para a soluo dos dois problemas que nos interessam.
PLO E RETA POLAR

Dados uma circunferncia S, de centro O e raio R, e um ponto A, distinto de O, definimos A tal que OA.OA = R2, e esta transformao chamada de inverso. A reta a que perpendicular OA, passando por A, chamada de reta polar de A em relao a S, e o ponto A chamado de plo de a em relao a S.
Teorema 1: Dados uma circunferncia S no plano e pontos A e B, sejam a e b suas

respectivas polares em relao a S. Temos ento que A b B a.


b a

O B'

A'

EUREKA! N20, 2004

26

Sociedade Brasileira de Matemtica

Prova: Tome B a, e seja B OB tal que ABOB. Temos que OAB OBA

pelo critrio AA, e logo OA/OB = OB/OA OB.OB = OA.OA = R2 B o inverso de B em relao a S, e como ABOB temos que A b.

Corolrio 1: Para um ponto pertencente prpria circunferncia, sua reta polar tangente circunferncia por ele. Corolrio 2: Se A exterior circunferncia, sejam B e C os pontos de contato das tangentes traadas circunferncia por A. Ento a reta polar de A passa por B e C. Prova: Temos que A pertence s polares de B e C, e logo B e C devem pertencer

polar de A. Bem, finalmente vamos aos problemas chineses:


(CHINA-1997) O quadriltero ABCD est inscrito num crculo S. Seja X o ponto de

interseco entre os lados AB e CD e W o ponto de interseco entre os lados AD e BC. As tangentes traadas por X intersectam S em Y e Z. Prove que W, Y e Z so colineares.
Resoluo: Antes de mais nada vamos ter que enunciar e provar um lema, que

tambm encontra-se em [1], mas que certamente no dia da prova voc teria de demonstrar. assim:
LEMA: Se por um ponto M exterior a um crculo S traarmos secantes que intersectam-no nos pontos A, B, C e D (vide figura), e se tomarmos {P} = AB CD e {Q} = AC BD, ento a polar de M em relao a S ser a reta PQ.
P

EUREKA! N20, 2004

27

Sociedade Brasileira de Matemtica

H quem ache essa parte um pouco mais salgada, pois justamente a parte mais difcil do assunto o qual vamos tratar. Tome as retas polares de A, B, C e D como a, b, c e d que, como vimos, so tangentes S nos seus respectivos plos. Defina {R} = b c e {T} = a d. A reta polar de R ser BC e a reta polar de T ser AD, pelo Corolrio 2, e pelo Teorema 1 teremos que a polar de M ser a reta RT. Basta provar ento que RT passa por P e Q, ou melhor, que R, P, Q e T so colineares. Considere o hexgono ABBCCD, no qual B B e C C (vamos usar aqui a estratgia proposta em [1]: fazer vrtices de um hexgono coincidirem para obtermos novas relaes). Pelo Teorema de Pascal, P, R e Q so colineares (os 3 pontos de encontro dos 3 pares de lados opostos do hexgono devem ser colineares). Analogamente no hexgono AABCDD, com A A e D D, teremos que P, Q e T so colineares. Segue que R, P, Q e T so colineares, como queramos demonstrar. Bem, fim de Lema. O problema agora fica fcil: suponha que, no dia da prova, voc j conhea todas estas propriedades. A voc as demonstra bem rpido na folha de respostas, e para dar o Gran Finale, bem... vejamos:
X

C B Z Y W

A S D

Note que podemos fazer uma certa analogia entre o problema e o Lema. O ponto W corresponde ao ponto M do Lema, e o ponto X ao ponto P. Temos ento que a reta polar de W passa por X a reta polar de X passa por W. Mas a reta polar de X
EUREKA! N20, 2004

28

Sociedade Brasileira de Matemtica

passa por Y e Z, pelo Corolrio 2 W, Y e Z so colineares, finalizando o problema.


(CHINA-1996) Seja H o ortocentro do tringulo acutngulo ABC. As tangentes

traadas por A ao crculo de dimetro BC intersectam o crculo em P e Q. Prove que P, Q e H so colineares.


Resoluo:

A S

P H Q

B O

A idia aqui relativamente simples. Tome S TH tal que TS AS. Sabemos que HTO HSA (AA) HS/AH = HO/TH HS = (AH.HO)/HT HS.HT = AH.HO. Como visto na figura, vamos usar Geometria Analtica:
COORDENADAS:

A (0, a) B ( b, 0) C (c, 0) T((c b)/2, 0) T ponto mdio de BC.


EUREKA! N20, 2004

29

Sociedade Brasileira de Matemtica

Temos que BH AC m(BH).m(AC) = 1 (h/b).(a/( c)) = 1 h = bc/a.


MEDIDAS:

r = ( b + c)/2 raio do crculo por B e C AH = a h = (a2 bc)/a HO = h = bc/a TH2 = h2 + ((c b)/2 )2 = (bc/a)2 + (b c)2/4. Assim vem que TH.TS = TH.(TH + HS) = TH2 + TH.HS = TH2 + AH.HO = (bc/a)2 + (b c)2/4 + bc/a. (a2 bc)/a = (bc/a)2 + (b c)2/4 + bc (bc/a)2 = ((b c)2 + 4bc)/4 = ((b + c)/2)2 = r2 de fato S H, onde H o inverso de H A polar de H H polar de A H PQ.

Referncias:
[1] Luciano G. M. Castro, Introduo Geometria Projetiva, Eureka! N. 8, pp. 16 27. [2] http://www.kalva.demon.co.uk/. Site muito bom com um verdadeiro arsenal de problemas.

EUREKA! N20, 2004

30

Sociedade Brasileira de Matemtica

RETA DE EULER E NMEROS COMPLEXOS


Jos Paulo Carneiro, Rio de Janeiro - RJ Nvel Intermedirio
INTRODUO:

O fato de os nmeros complexos terem nascido no contexto da resoluo de equaes algbricas fez com que muitas vezes sua utilidade em Geometria no seja suficientemente explorada (uma notvel exceo a esta tendncia pode ser encontrada em Eureka, Vol 6, no artigo Aplicaes dos Nmeros Complexos Geometria, do Prof. Edmilson Motta). Aqui, vamos usar a lgebra dos nmeros complexos para mostrar um belo resultado de Geometria, o fato de que, em qualquer tringulo, o circuncentro K, o baricentro G e o ortocentro H so colineares. A reta que contm estes trs pontos chamada Reta de Euler, j que foi Euler o primeiro a chamar a ateno para este fato. Mais que isto, vamos provar que, vetorialmente: KH = 3KG , o que, alm de implicar que os trs pontos esto alinhados, acarreta que a distncia KH o triplo da distncia KG e que G e H esto na mesma semi-reta de origem K (ver Figura 1).
B

H A

C Figura 1

Para usar nmeros complexos, de agora em diante estar fixado no plano um sistema de coordenadas cartesianas ortogonais e as letras maisculas A, B,..., designaro pontos do plano ou nmeros complexos, de modo que cada ponto ( x; y ) esteja identificado com o nmero complexo mais usualmente representado por x + yi . Ser fundamental a igualdade AB = B A , a qual traduz que a translao definida pelo vetor AB a mesma que leva a origem no complexo B A (Figura 2).
EUREKA! N20, 2004

31

Sociedade Brasileira de Matemtica

B A BA

O
Figura 2

Deve ser observado que, mais usualmente, o smbolo AB designa o comprimento do segmento AB . Porm aqui, como estamos identificando pontos do plano com nmeros complexos, o smbolo AB no ser usado para o comprimento do segmento AB , e sim para o produto dos complexos A e B.
Baricentro

bem sabido que AG = 2GM , onde M o ponto mdio de BC e G o baricentro do tringulo ABC, isto , o ponto de encontro das medianas do tringulo (Figura 3)

G B M
Figura 3

Logo: G A = 2(M G ) , ou seja: G = A + 2 de onde se conclui que G =


EUREKA! N20, 2004

A+ B+C . 3
32

B+C G = A + B + C 2G , 2

Sociedade Brasileira de Matemtica

At a, os complexos parecem no estar presentes. que ainda no figura o produto de complexos, que a sua mais forte caracterstica. Para efeito de soma e de multiplicao por nmero real, os complexos funcionam apenas como vetores do plano.
Um caso particular

Comecemos com um caso particular, a saber: vamos supor que os trs vrtices do tringulo ABC estejam na circunferncia unitria do plano, isto , a circunferncia de centro na origem e raio 1, que o conjunto dos complexos de mdulo 1. Ento, o circuncentro de ABC coincide com a origem de coordenadas e A = B = C = 1 . Mas para qualquer complexo z de mdulo 1, temos:

zz =| z |2 = 1 (onde z o conjugado de z). Conseqentemente, A = 1/ A , B = 1/ B , C = 1/ C . / / Para usar agora a condio AH BC (onde H o ortocentro de ABC), vamos
observar que o complexo v perpendicular ao complexo w (considerados como vetores no nulos) se e s se forem colineares com a origem os complexos v e iw , ou seja, se e s se v iw for real (ver Figura 4).
y w

v x iw
Figura 4

Por outro lado, um complexo real se e s se for igual ao seu conjugado e, portanto:

vw

v v v = = iw iw iw

vw + vw = 0

(o leitor

pode

verificar, colocando isto em coordenadas, que esta condio equivale nulidade do produto escalar dos dois vetores). Temos, pois:

EUREKA! N20, 2004

33

Sociedade Brasileira de Matemtica

( H A)(C B ) + ( H A)(C B) = 0 1 1 1 ( H A) + H (C B) = 0 A C B (B C ) 1 ( H A) + H (C B ) = 0 BC A A( H A) + BC (1 AH ) = 0 AH = A2 + ABCH BC

AH BC

Ateno! lembre que AH no o comprimento do segmento do segmento AH , e sim o produto dos complexos A e H! O mesmo vale para BC, etc. Analogamente:

BH CA
Subtraindo:

BH = B 2 + ABCH CA

( A B ) H = A2 B 2 + C ( A B ) ( A B) H = ( A B)( A + B) + C ( A B) H = A+ B +C
Este resultado significa que, dados trs complexos de mdulo 1, sua soma o ortocentro do tringulo por eles formado.
Primeira generalizao

Suponha agora que os trs vrtices do tringulo ABC estejam em uma circunferncia , de centro na origem e raio r qualquer, ou seja, o circuncentro de ABC coincide com a origem de coordenadas e A = B = C = r > 0 . Neste caso, como ilustra a Figura 5, os complexos (ou pontos) A ' =

A B , B'= e r r

C'=

C estaro na circunferncia , de centro na origem e raio 1. De fato: r A A = = 1 , etc. r r

EUREKA! N20, 2004

34

Sociedade Brasileira de Matemtica

y B A A' O C' B'

C
Figura 5

Na verdade, a circunferncia e o tringulo A ' B ' C ' so os transformados da circunferncia e do tringulo ABC, pela homotetia de centro O e razo 1 r , a qual preserva ngulos, de modo que o ortocentro H ' do tringulo A ' B ' C ' a imagem do ortocentro H do tringulo ABC, isto : H ' = resultado do caso particular estudado, temos:

H . Mas ento, pelo r

H '=

H A B C = A '+ B '+ C ' = + + , donde se conclui que: H = A + B + C . r r r r

Portanto: dados trs complexos de mesmo mdulo, sua soma o ortocentro do tringulo por eles formado (o que, em si mesmo, um resultado curioso).
Caso geral

Passemos agora ao caso geral: dado um tringulo ABC qualquer, sejam K o seu circuncentro e r o raio do seu crculo circunscrito. Transladando os pontos A, B e C pelo vetor KO (onde O a origem do plano complexo), obtemos os complexos (ou pontos) A K , B K e C K , que pertencem a uma circunferncia de centro na origem e raio r, como ilustra a Figura 6.

EUREKA! N20, 2004

35

Sociedade Brasileira de Matemtica

y A H BK AK H K O CK
Figura 6

K C x

Sendo a translao uma isometria (que preserva distncias e ngulos), o ponto H K ser o ortocentro do tringulo de vrtices A K , B K e C K . Mas ento, pelo resultado anterior, temos: H K = A K + B K + C K = A + B + C 3K . Como A + B + C = 3G , onde G o baricentro de ABC, temos: H K = 3G 3K = 3(G K ) , ou

KH = 3KG , como se queria provar.

EUREKA! N20, 2004

36

Sociedade Brasileira de Matemtica

COMO QUE FAZ?


PROBLEMA 4 PROPOSTO POR SAMUEL BARBOSA FEITOSA (FORTALEZA - CE)

a) Prove que, para todo inteiro positivo m, todo inteiro A. b) Defina a seqncia An por todo inteiro positivo n.
SOLUO:

(d) A
d |m

md

divisvel por m, para

A
d |n

= 2n. Prove que An divisvel por n, para

a) Podemos supor A > 0, pois a afirmao s depende da classe de congruncia de A mdulo m. Seja g(m) = (d) Am d .

d |n

Temos

g(k) = (d) A
k| n k| n d | k

kd

= Ar (d) = An .
r|n d|n / r

Dada uma seqncia ( xn )n1 . Definimos seu perodo como o menor inteiro positivo t tal que xn+t = xn , n 1 (se existir). Veremos que g(n) o nmero de seqncias de nmeros inteiros ( xk )k1 de perodo n tais que 1 xk A, k 1. De fato, o nmero de seqncias ( xk )k1 com xk +n = xk , k 1 e xk {1,2,..., A}, k 1 A n (uma tal seqncia determinada pela escolha de x1, x2 ,..., xn {1,2,..., A}). Essas seqncias so exatamente as seqncias cujo perodo um divisor d de n. Assim, se f(d) o nmero de tais seqncias de perodo d, f (d ) = An , n 1,

d |n

donde f (k) = g(k ), k 1. Finalmente, o nmero g(n) de tais seqncias com de perodo n equivalentes se existe t < com xk +t = xk , k 1, as classes de equivalncia de seqncias de perodo n tm exatamente n elementos: a classe de equivalncia de ( xk )k1 {( xk +t )k1 ,0 t n 1} . b) Temos, para A = 2, An = g(n) , na notao do item a), donde n | An , n 1, pelo item a).
EUREKA! N20, 2004

perodo n mltiplo de n, pois, se declararmos duas seqncias ( xk )k1 e ( yk )k1

37

Sociedade Brasileira de Matemtica

PROBLEMA 5 PROPOSTO POR WILSON CARLOS DA SILVA RAMOS (BELM - PA) (de uma olimpada chinesa)

Seja ABC um tringulo acutngulo de incentro I, ortocentro H e tal que AB AC . B1 e C1 so os pontos mdios de AC e AB. B2 o ponto de interseo de IB1 com AB. C2 definido analogamente. Sejam tambm k a interseo de B2C2 com BC e O o circuncentro de BHC. Se a rea dos tringulos BKB2 e CKC2 igual, mostre que A, I e O so colineares.
SOLUO DE YURI GOMES LIMA (FORTALEZA - CE):

Lema: Seja ABC um tringulo de incentro I. Se M o ponto mdio de BC e X o ponto de tangncia ao ex-incrculo relativo a BC com esse lado, ento MI // AX . Prova: Seja D como na figura e P a outra interseo de DI com o incrculo. A homotetia de centro A que leva o incrculo no ex-incrculo leva P em X, e portanto A, P, X so colineares. Sabemos que BD = XC = p b.
A P I B D M X C

Da, M mdio de DX e assim MI base mdia do tringulo PDX MI // AX


A

Z C1 B2 B K C2 B1 I Y C

EUREKA! N20, 2004

38

Sociedade Brasileira de Matemtica

Note que

[BKB2 ] = [CKC2 ] [ ABC] = [ AB2C2 ] AB AC = AB2 AC2

AB ACsen AB2 AC2 sen = 2 2


(1)

AB AC2 = BC2 // B2C. AB2 AC

Sejam Y, Z os pontos de tangncia dos ex-incrculos relativos a AC e AB com esses lados, respectivamente. Ento, pelo lema temos B1I // BY (2) e C1I // CZ (3). Da, as relaes (2), (1) e (3) implicam, nessa ordem, em:

AY AB AC2 AC1 = = = AB1 AB2 AC AZ AY AC1 = AY AZ = AB1 AC1 AB1 AZ b c ( p c)( p b) = (a + b c)(a b + c) = bc 2 2 2 a ab + ac + ab b2 + bc ac + bc c2 = bc a2 = b2 + c2 bc 1 cos = = 60 . 2  = 120 B, H, I ,C so concclicos. Isso garante que BHC = BIC
A H B O

I C

IA

Para concluir, o centro I A do ex-incrculo relativo a BC est em AI e satisfaz

I BI A = ICIA = 90 I IA dimetro do circuncrculo de BHC, de modo que A, I e


O so colineares.
EUREKA! N20, 2004

39

Sociedade Brasileira de Matemtica

SOLUES DE PROBLEMAS PROPOSTOS


Publicamos aqui algumas das respostas enviadas por nossos leitores.

83) Seja  = {0,1,2,3,...}. Determine quantas funes f : < < satisfazem f (2003) = 2003, f (n) 2003 para todo n 2003 e f (m + f (n)) = f ( f (m)) + f (n) , para todo m, n < .
SOLUO DE ZOROASTRO AZAMBUJA NETO (RIO DE JANEIRO RJ)

Fazendo m = n = 0, obtemos f ( f (0)) = f ( f (0)) + f (0), donde f (0) = 0, e logo f ( f (0)) = f (0) = 0. Assim, fazendo m = 0, obtemos f ( f (n)) = f (n) para todo n < , donde f ( y) = y, y Im( f ). Seja t = min{ f (n); n <, f (n) > 0}. Note que Temos t = f (n0 ) para um certo n0 < , donde f (t ) = f ( f (n0 )) = f (n0 ) = t. Temos, por induo, f (kt) = kt, k <. De fato,

t 2003.

f ((k +1)t ) = f (kt + t) = f (kt + f (t)) = f ( f (kt )) + f (t ) = f (kt) + t = kt + t = (k + 1)t. Alm disso, para todo m < e k <, f (m + kt) = f (m + f (kt)) = f ( f (m)) + f (kt ) = f (m) + kt. Afirmamos que Im( f ) = { f (n), n <} = {kt, k <}. De fato, j vimos que kt = f (kt)Im( f ), k <, e, se kt f (n) < (k +1)t, temos f ( f (n)) = f (n) = kt +( f (n) kt), donde f (n) = f (( f (n) kt) + kt) = f (( f (n) kt) + f (kt)) = f ( f ( f (n) kt)) + f (kt) = f ( f ( f (n) kt)) + kt, donde f ( f ( f (n) kt)) = f (n) kt, e logo f (n) kt Im( f ), mas 0 f (n) kt < t, donde f (n) kt = 0, para no contradizermos a definio de t. No nosso caso, como f (2003) = 2003, temos t | 2003, donde, como 2003 primo, t = 1 ou t = 2003. Se t = 1, f (n) = n, n<, o que claramente satisfaz as condies do enunciado. Se t = 2003, 2003| f (n), n<. Se 1 n 2002, devemos ter f (n) 2003, donde f (n) {0,2003}, e podemos escolher f(n) arbitrariamente em {0, 2003} para 1 n 2002 (para o que temos, portanto, 22002 escolhas) estendendo f a < de modo que f (n + k 2003) = f (n) + k 2003, n < 2003, k < (lembremos que f(0) = 0). De fato, para quaisquer m, n <, temos f (n) = 2003k, para algum k < , e, escrevendo m = r + 2003 s, com 0 r 2002, s <, temos

EUREKA! N20, 2004

40

Sociedade Brasileira de Matemtica

f (m + f (n)) = f (r + 2003(s + k)) = f (r) + 2003(s + k) = = ( f (r) + 2003s) + 2003k = f (m) + f (n) = f ( f (m)) + f (n).
Assim, temos 1+ 22002 funes f que satisfazem as condies do enunciado.

A * = {1,2,3,...} um conjunto no-vazio tal que ento A = * . n A 4n A e n A Obs: x o nico inteiro tal que x 1 < x x.
84) Prove que se
SOLUO DE RODRIGO VILLARD MILET (RIO DE JANEIRO RJ)

(i) n A 4n A (ii) n A n A Seja u um elemento de A (existe, pois A no vazio).Veja que 1 A , pois, se u>1, pela propriedade (ii), temos um elemento menor que u em A, logo, repetindo esse argumento um nmero finito de vezes, temos que 1 A . Isso mostra que todas as potncias de 4 esto em A (por (i)). Agora vou fazer o seguinte: dado m natural, mostrarei que existe alguma potncia k k , para algum k natural (da, como a potncia de de 4 no intervalo m 2 , (m + 1)2 4 est em A, usando a propriedade (ii) k vezes, temos que m est em A; note que

x = x , x 1 ). Suponha ento que para todo k tenhamos um t tal que


4 t < m 2 < (m + 1)2 < 4 t +1 . Da, segue que ( m + 1) < 4t +1 = 4.4t < 4m 2 , logo
k k
2k
k

1 1 + m

2k

< 4 , para todo k natural, o que uma contradio, pois

1 2k k m > 4 , para k = m + 2, pois 2 = 2 + 2 = 4 2m > 4m. Logo, m A , 1 + 1 + m m

2k

para todo m natural. 85) Mostre que todo tringulo pode ser dividido em 9 pentgonos convexos de reas iguais.
SOLUO DE JOS DE ALMEIDA PANTERA (RIO DE JANEIRO RJ)

Em primeiro lugar, aplicando uma transformao afim (que preserva relaes entre reas e leva pentgonos convexos em pentgonos convexos) ao tringulo,
EUREKA! N20, 2004

41

Sociedade Brasileira de Matemtica

podemos supor, sem perda de generalidade, que o tringulo equiltero de lado 1 (digamos com vrtices (0, 0), (1, 0) e 2, 2 ). Fazemos ento a seguinte construo, simtrica em relao rotao de 120 em torno do centro O do tringulo (e tambm simtrica em relao s bissetrizes internas do tringulo):
1 3

X3 Z3

Y3 W3

T3 Y1 W1

O T1

T2 Z2 X2

Z1 A
1 3

W2 Y2 B

X1

Temos AX 1 = AY1 = a = , (onde > 0 pequeno) (assim, por exemplo, X1 = (a, 0) e Z1 = (a, b)) , Z1 X 1 A = W1 Y 1 A = 90, X 1 Z1 = YW 1 1 = b onde b escolhido de modo que a rea
AX 1 Z1W1Y1 seja igual a b= 1 3 a2 3 b 3b do pentgono a + 2 4 2

1 3 da rea do tringulo, ou seja, temos , que 36 9

1 8 + 2 . Note que, se > 0 pequeno ento b > 0 pequeno. 3 3 27 9 h a Escolhemos T1 = , b + h de modo que a rea (1 2a ) b + do pentgono 2 2 X 1Y2W2T1 Z1
EUREKA! N20, 2004

seja

tambm

igual

3 . 36

Ou

seja,

temos

42

Sociedade Brasileira de Matemtica

h=

3 2b = 18(1 2a)

3 1 18 + 2 3

2b <

3 6

(e h prximo a

3 36

se

>0

pequeno). Como os pentgonos AX 1 Z1W1Y1 , BX 2 Z 2W2Y2 e CX 3 Z 3W3Y3 so congruentes, so congruentes, W1 Z1T1OT3 , W2 Z 2T2 OT1 e X 1Y2W2T1 Z1 , X 2Y3W3T2 Z 2 e X 3YW 1 1T3 Z 3
W3 Z 3T3 OT2 so congruentes e os 6 primeiros tm 1 da rea do tringulo. 9 1 da rea do tringulo, todos tm 9

86) Encontre todas as triplas de inteiros positivos (a, m, n) tais que am +1 divide

(a +1)n .
SOLUO DE ANDERSON TORRES (SO PAULO SP)

fcil ver que (1; m; n), (a; 1; n) so solues. Vamos ento supor m 2, a 2. Lema 1: m mpar. Demonstrao: vamos dividir em casos: m Caso 1: 4 divide ( a + 1). Assim, a 1(mod 4). um fato muito conhecido que ( 1) no resduo quadrtico mdulo 4, pois todo quadrado perfeito mpar congruente a 1 mdulo 4. m Logo a no quadrado perfeito, e portanto m no pode ser par.
m m Caso 2: Existe um primo 2 tal que ( a + 1). m n Ento ( a + 1) ( a + 1) , e, como primo, a + 1. m a 1(mod ) a m + 1 ( 1) m + 1(mod ), e j que ( a + 1), temos

( 1) m ( 1)(mod ) m mpar.
Caso 3: O complementar dos casos 1 e 2. 2 m Assim sendo, a m + 1 = 2 a m = 1. Mas a 2 = 4 > 1, absurdo e fim do lema.
EUREKA! N20, 2004

43

Sociedade Brasileira de Matemtica

Seja p m um primo (mpar, como j sabemos).


p m n Ento ( a + 1) ( a + 1) ( a + 1) ( a + 1)

ap +1 ( a + 1) n 1 . a +1

xp +1 = x p 1 x p 2 + x p 3 ... x + 1. x +1 Temos f ( 1) = p, logo f ( x) = ( x + 1) g ( x) + p g [ x]. Em particular, f ( a ) = ( a + 1) g ( a ) + p.


Seja f ( x) = Seja q um fator primo de f(a). Ento q f ( a ) ( a + 1) Ento 0 f ( a ) p(mod q ).
n 1

para

algum

polinmio

q ( a + 1).

Como p e q so primos, p = q. Logo p ( a + 1) e f(a) potncia de p. Note que f ( a) > 1. Vamos fazer mais. Lema 2: f(a) = p. Demonstrao: Em dois casos:
2 Caso 1: p divide a + 1. 2 2 Ento f ( a) = ( a + 1) g ( a ) + p p 0(mod p ) p no divide f(a)

2 Caso 2: p no divide (a + 1).

Ento a + 1 = hp , onde p no divide h. Assim a = hp 1,

a p + 1 ( hp 1) p + 1 = a +1 hp

p p p p p p p p p p p ( hp 1) = ( hp) ( 1) + hp (1) 1 + h 2 p 2 ( 1) 2 + ... + h p , 0 1 2 p p p hp h2 p2 + ... + h p p p p p p (hp 1) +1 2 = = p hp + h2 p 2 + ... + h p 2 p p 1 , hp hp 2 3 e assim, p (hp 1) p + 1 p 1 2 p hp p p hp p 0 (mod p ) . hp 2 2

EUREKA! N20, 2004

44

Sociedade Brasileira de Matemtica

Em todo caso, p no divide f ( a ) mas p f ( a ). Logo, como f ( a ) potncia de p, f ( a ) = p.

ap +1 = p. Assim, a +1
Lema 3: p = 3 Demonstrao: vamos por absurdo. Suponha p 5.

ap +1 Ento crescente. De fato, a +1


1 ap +1 a p a a p 1 =1+ = 1 + a ( a 1) 2 a +1 a +1 a 1

ap +1 = 1 + a (a 1)( a p 3 + a p 5 + ... + 1), a +1


crescentes. Assim, p =

uma

composio

de

funes

a p +1 2 p +1 p p (pois a 2 ) 3 p 2 + 1 > 2 + 1 3 a

p Mas fcil ver que p 5 3 p < 2 ( uma induo simples).

E isto absurdo! Logo p < 5 e p primo mpar, logo p = 3.


3 a +1 = 3 a 3 3a 2 = 0. fcil ver que ( 1) raiz disto. Assim, a +1 3 a 3a 2 = a 2 a 2 = 0 a {2, 1}. a +1

Como a > 0, temos a = 2.


k p k n Se m = pk, e 2m + 1 divide 3n, (2 ) + 1 divide (2 + 1) , pois k mpar, donde

2 k + 1 mltiplo de 3. Assim, (2 k ; p; n) soluo, e logo 2 k = 2, donde k = 1 e m = p = 3. Assim, todas as outras solues so da forma (2; 3; n), com n 2.

EUREKA! N20, 2004

45

Sociedade Brasileira de Matemtica

87) Seja a(1) = 1 e, para cada inteiro n 2, a(n) igual ao menor inteiro positivo que no pertence a {a( j), j < n} tal que

a( j) seja mltiplo de n. Prove que


j =1

a(a(n)) = n para todo inteiro positivo n.


SOLUO DE SAMUEL BARBOSA FEITOSA (FORTALEZA CE)

Sejam F2 = F1 = 1 e, para n 2, Fn = Fn 1 + Fn 2 os nmeros de Fibonacci: Pelo Teorema de Zeckendorff sabemos que todo nmero natural pode ser escrito de maneira nica como soma de nmeros de Fibonacci com ndices maiores que 1 e no consecutivos. (isso pode ser provado por induo: temos 1 = F2 e, se Fk o maior nmero de Fibonacci que menor ou igual a n, devemos ter n Fk < Fk 1

n Fk Fk 1 , e logo n Fk + Fk 1 = Fk +1 , absurdo; escrevemos ento n = Fk + ( n Fk ) e aplicamos e resultado para n Fk ) .


pois, caso contrrio, Vamos criar uma pequena variao desta representao; chamemos esta nova representao de representao F. Suponha que m = Fi1 + Fi2 + ... + F in (na representao descrita no Teorema anterior) Com i1 > i2 > ... > in > 1. Se in 2 a representao de m na representao F ser a mesma. Se in = 2 m ser representando na F como:

Fi1 + Fi2 + ... + Fi"n1 + F2 se in 1 mpar. Fi1 + Fi2 + ... + Fin1 + F1 se in 1 par. (1 ser F2 ).
Veja que todo nmero pode ser escrito na "F" de maneira nica. Seja S k =

A . Afirmamos que:
i i =1

Se K = Fi1 + Fi2 + ... + Fin com i1 > i2 > ... > in a representao de K na F. A) in par Ak = Fi1 1 + Fi2 1 + ... + Fin 1 Sk = (Fi1 + Fi2 + ... + Fin )(Fi1 1 + Fi2 1 + ... + Fin 1) B) in mpar Ak = Fi11 + Fi2 1 + ... + Fin 1 Sk = Fi1+1 + Fi2 +1 + ... + Fin +1 Sk =

= ( Fi1 + Fi2 + ... + Fin )( Fi1 1 + Fi2 1 + ... + Fin 1 + 1)


Vamos provar a afirmao acima por induo: Casos iniciais: A1 = 1, A2 = 3, A3 = 2, S1 = 1, S 2 = 4, S3 = 6 (verifica-se facilmente que eles satisfazem as condies.
EUREKA! N20, 2004

46

Sociedade Brasileira de Matemtica

Supondo a afirmao acima verdadeira todo r k . Provemos que ela tambm verdadeira para k + 1. Suponhamos in > 3 (o caso em que in = 1, 2, 3 totalmente anlogo ao que faremos agora, a nica diferena consiste na utilizao das seguintes relaes:

F1 + F3 + ... + F2 r +1 = F2 r + 2 , F2 + F4 + F6 + ... + F2 r = F2 r +1 1)
Vamos dividir agora em dois casos: i) in par na "F" k + 1 = Fi1 + Fi2 + ... + Fin + F1. Sabemos que

Ak +1 + S k 0(mod k + 1) = (mod Fi1 + Fi2 + ... + Fin + F1 ). Mas


Sk = ( Fi1 + Fi2 + ... + Fin )( Fi1 1 + Fi2 1 + ... + Fin 1 ) ( Fi1 1 + Fi2 1 + ... + Fin 1 )(mod k + 1) Ak +1 Fi1 1 + Fi2 1 + ... + Fin 1 (mod k +1). Fi1 1 + Fi2 1 + ... + Fin 1 = Ak
j est Veja na que o nmero Logo seqncia.

Ak +1 Fi1 1 + Fi2 1 + ... + Fin 1 + (k +1) = Fi1 +1 + Fi2 +1 + ... + Fin +1 + F2 (trocamos F1 por

F2 para termos uma representao F)


mas pela nossa hiptese de induo o nmero Fi1 +1 + Fi2 +1 + ... + F2 s pode ter aparecido na seqncia oriundo do nmero Fi1 + Fi2 + ... + F1 = k + 1 > k logo ele ainda no est na seqncia a(k + 1) = Fi1 +1 + Fi2 +1 + ... + F2 . Vejamos que

S k +1 = a (k + 1) + S k = Fi1 +1 + ... + F2 + ( Fi1 + Fi2 + ... + Fn )( Fi1 1 + ... + Fin 1 ) =


= Fi1 + Fi2 + ... + Fin + F2 + Fi1 1 + Fi2 1 + ... + Fin 1 + (Fi1 + Fi2 + ... + Fin )( Fi1 1 + ... + Fin 1 ) = = (Fi1 1 + Fi2 1 + ... + Fin 1 )(1+ Fi1 + Fi2 + ... + Fin ) + (Fi1 + Fi2 + ... + Fin + F2 ) = = (1 + Fi1 + Fi2 + ... + Fin )( Fi1 1 + Fi2 1 + ... + Fin 1 + 1) = = (Fi1 + Fi2 + ... + Fin + F1 )(Fi1 1 + Fi2 1 + ... + Fin 1 + 1) (que coincide exatamente com a
nossa afirmao para k + 1). ii)

in mpar. Este caso anlogo ao anterior.

Com a afirmao verdadeira fcil concluir que a(a(n)) = n . Na F, a(a(n)) somar 1 a todos os ndices e depois subtrair, ou o contrrio, da os ndices ficam os mesmos.

EUREKA! N20, 2004

47

Sociedade Brasileira de Matemtica

88) Prove que se r

e cos(r )

ento cos(r ) 1, ,0, ,1.

1 2

1 2

SOLUO DE ANDRS SNCHEZ PREZ (LA HABANA, CUBA) f p r r = onde p, q , com mdc ( p; q ) = 1 . cos(r ) Q cos(r ) = q g

donde f , g Z , tambm com mdc ( f ; g ) = 1 . Aplicando Moivre, se z = cis = cos + isen ento
n n n n j j z n = cis ( n ) = cos ( n ) + isen ( n ) = (cos + isen ) = (cos ) (isen ) j j =0 2 n j n n 2 j = ( 1) (cos ) ( sen )2 j + i ( 1) j ( cos )n 2 j1 ( sen )2 j+1 j =0 j =0 2 j 2 j + 1 n 2 n 1

igualando parte real com parte real: cos(n ) = n (sen ) ( 1) 2 j (cos )


j n 2 j j =0
n 2

n 2

2j

j j n n2 j 1 (cos )2 = cos ( n ) = ( 1) (cos ) j =0 2j n n 2 j 2 n k n n2 j k j j n2 j 2k = ( 1) (cos ) ( 1) (cos ) = ( 1) (cos ) , j =0 k = j 2 k j 2 j k k =0 j =0 n 2 j

k lembrando que k

k j = . j

n 2 n k j n 2 j ( 1) Logo temos que cos(n ) = . Agora, veja que (cos ) 2k j j =0 k = j

n 2

sempre

que

cos = cos( ) ). Se a

cos ,cos ( n ) ,

para

todo

a mpar ento cos ( a + ) = cos . .

e a par ento cos(a + ) = cos , e se a

(pois

tambm e

EUREKA! N20, 2004

48

Sociedade Brasileira de Matemtica

Logo, se cos ,cos ( a + b ) , a, b . Fazendo = ( aq + bp ) p p cos a + b = cos q q q inteiros tais que aq + bp = 1 . Por . Como se

mdc ( p; q ) = 1, a, b

conseguinte,

com q = 2 h (2c + 1) , onde h <, c , supondo sem perda de cos q generalidade que cos = cos ( ) ), temos (pois 2c + 1 > 0 cos 2c + 1 x, y
2c +1 2

x tais que cos = , com mdc ( x; y ) = 1 . 2c + 1 y


2c +1 2 2 c +1 2 j 2c + 1 k j 1 cos = ( ) k j 2 c 2 1 + = k j

cos ( 2c + 1) = c 2 +1
c j

j =0

2c +1 2 j c c c 2c + 1 k 2c + 1 k x 2c +1 2 j j = ( 1) = ( 1) cos 2 c +12 j 2c + 1 j =0 j =0 k = j 2k j k = j 2k j y c c 2c + 1 k x 2 c +12 j j 1 = (1) 2 c +12 j j =0 k = j 2k j y c c 2c + 1 k 2 c +12 j 2 j j y 2 c +1 = ( 1) y x j =0 k = j 2k j c c 2c + 1 k 2 c 2 j 2 j j y 2 c +1 = x ( 1) y x j =0 k = j 2k j

x y 2 c +1 , mas como (x; y ) = 1 x = 1, x = 0 ou x = 1

Por outro lado


c c c 2c + 1 k 2c + 1 k 2 c +12 j 2 j 1 j y 2 c +1 = x y + y ( 1) x k = 0 2 k 0 j =1 k = j 2k j c c 2c + 1 k 2c + 1 y y k = 0 2 k 0 k =0 2k

EUREKA! N20, 2004

49

Sociedade Brasileira de Matemtica

Do Binmio de Newton (1 1)2c+1 = ( 1)k


k =0 2 c +1

(1 + 1)2c +1 =

2 c +1

2c + 1 c 2c + 1 c 2c + 1 e = + k k =0 k =0 2k k = 0 2 k + 1

2c + 1 c 2c + 1 c 2c + 1 ; somando e dividindo por = k k =0 2k k =0 2k + 1

2,

2c + 1 2c t t = 2 , logo y = 2 ou y = 2 com t <, t 2c . 2 k k =0


c

2c 1 cos . = sen = sen 2c + 1 2 2c + 1 2(2c + 1) Se e > 0 , e 2c 1 e e 2 (2c + 1), e 2 (2c 1) 2 ( 2c + 1) e 4 porm como e mpar ( e 2c 1 ) ento e = 1 mdc ( 2c 1; 4c + 2 ) = 1 . Volvendo a Moivre, e igualando parte imaginria a parte imaginria, com n mpar: sen(n ) =
n 1 2 j =0

( 1) 2 j + 1 (cos )
( 1) j
j =0

n 2 j 1

(sen )2 j +1

n 1 2

n 1 n j 2 2 j +1 cos ( ) 2 (sen ) 2 j + 1

n 1 2 j =0

( 1) j

n 2 1 (sen ) 2 j + 1

n 1 j 2

(sen )2 j +1
2k j ( ) ( )2 j +1 sen sen

n 1 2

j =0

n 1 j n 1 n 2 k ( 1) j ( ) 1 2 2 j + 1 k =0 k

n 1 2 j =0

( 1) j
j

n 1 n k 2 j +1 2 (sen ) k =0 2k + 1 j k e n mpar, sen ( n ) u e u . Se u e u mpar ento

Agora, veja que sempre que sen par ento


EUREKA! N20, 2004

sen(u + ) = sen , e se

50

Sociedade Brasileira de Matemtica

sen(u + ) = sen , logo, se sen v mpar. Fazendo = 2c 1 , 2 ( 2c + 1)

sen (u + v ) , u , v

com

(2u ( 2c + 1) + v ( 2c 1)) 2c 1 + = sen u v sen . 2 (2c + 1) 2 2 1 + c ( ) Como mdc ( 2c 1;4c + 2 ) = 1, u, v inteiros tal que u (4c + 2) + v(2c 1) = 1 . do Note que v claramente mpar. Por conseguinte, sen 2 ( 2c + 1) cos = 1 2 sen 2 ( 2c + 1) 2c + 1
2

2 Se cos que uma contradio. = 0, sen = 2 2c + 1 2 ( 2c + 1) 2t 1 1 Se cos = = , . Como ambos (numerador e sen t 2 ( 2c + 1) 2t +1 2c + 1 2 denominador) so primos relativos (para t 0 ), ento so quadrados perfeitos, logo t mpar. Porm se t 3, 4 2t 2t 1 1( mod 4 ) , o que absurdo pois um quadrado. Com t = 0, sen = 0 , mas 2k . Ento, nesse 2 ( 2c + 1) 2c + 1 1 caso, cos = . 2c + 1 2 1 2t + 1 Se cos = t , sen = . Como ambos (numerador e 2 ( 2c + 1) 2 2t +1 2c + 1 denominador) so primos relativos (para t 0 ), ento so quadrados perfeitos, e logo t mpar. Temos 2 t + 1 = d 2 2 t = (d + 1)(d 1) d 1 = 2 , d + 1 = 2 + 2 = 2 . Dividindo por 2 conclumos que = 1, = 2 1 t = 3 cos = , porm, se c = 0, cos = 1 , e para c 1 8 2c + 1 2c + 1 e

EUREKA! N20, 2004

51

Sociedade Brasileira de Matemtica

0<

< cos > 0 . Para t = 0, sen = 1 . Ento, nesse 2c + 1 2 2c + 1 2 ( 2c + 1)

caso, cos = 1 . 2c + 1 1 cos Finalmente 1,3} h , , 1 2c + 1 { 2 1 2 q 2 3 2h c+ h < . Se para absurdo. Se 2 h , h 2 , cos h 2h 2 = cos = q 2 2 4 2

com

, o que

3 , h 1 , , cos 2h 1 = cos = , o que h h q 3 2 32 6 2 p tambm absurdo. Assim, , , , p , p , p e q 2 3 q 2 3 p 1 1 cos q 1, 2 ,0, 2 ,1 .


SEGUNDA SOLUO DE CARLOS GUSTAVO TAMM DE ARAUJO MOREIRA e JOS PAULO CARNEIRO (RIO DE JANEIRO - RJ)

Note que se cos( x) =

q2 p2 p p , ento z := + i uma raiz da unidade (isto , q q q

z n = 1 para algum inteiro positivo n) se e s se x um mltiplo racional de . Assim, se x = r , com r ento z n = 1 para um certo inteiro positivo n (e logo tambm temos z 2 n = 1 ). Vamos mostrar que q 2 , o que resolve o problema. Para isso, vamos supor por absurdo que q > 2 (e p primo com q). Temos ento dois casos: a) q mpar. Nesse caso, para cada m < , seja xm = (( p + i q 2 p 2 ) m ( p i q 2 p 2 ) m ) / 2i q 2 p2 . Temos x0 = 0, x1 = 1 e, como p + i q 2 p 2 e p i q 2 p 2 so razes da equao x 2 2 px + q 2 = 0 , ( xm ) satisfaz a recorrncia xm +2 = 2 pxm +1 q 2 xm , m <. xm + 2 2 pxm +1 (mod q),
m 1

Assim,

m <,

donde, por induo, para todo

(mod q). Em particular, como mdc(2 p, q) = 1, temos m 1, xm (2 p) para todo e logo mas mdc( xm , q) = 1, xm 0 , m 1, m 1,
EUREKA! N20, 2004

52

Sociedade Brasileira de Matemtica

2n

p + i q2 p2 =1 q

= 1 p + i q 2 p2

2n

) =(pi
n

q2 p2

(pois

p + i p2 q2 = q p i q2 p2 p + i q2 p2 b) q par. Nesse caso, se p + i q2 p2 ym = 2 e, como

) , e logo xn = 0 , absurdo.

p i q2 p2 2

i q 2 p 2 , temos y0 = 0, y1 = 1

p + i q2 p2 p i q2 p2 q2 = 0, e so razes da equao x 2 px + 2 2 4 q2 temos ym+ 2 = pym+1 y , m <. 4 m Assim, donde, por induo, para todo ym +2 pym +1 (mod q 2) , m 1, ym p m 1 (mod q 2), e logo mdc ( ym , q 2 ) = 1, para todo m 1, pois mdc ( p, q 2 ) = 1. Em particular, ym 0, m 1 (note que q > 2, logo q 2 2, z
2n

pois q par). Por outro lado, p + i q2 p2 2 p i q2 p 2 = 2


n n

p + i q2 p2 =1 q

= 1,

2n

donde

, como antes, e logo yn = 0, absurdo.

91) Um jardinero deve construir um canteiro com a forma de setor circular. Ele dispe de 100 metros de fio para cerc-lo. Figura:
r

EUREKA! N20, 2004

53

Sociedade Brasileira de Matemtica

Qual deve ser o valor do raio do crculo para que o canteiro tenha rea mxima? Qual a rea mxima?
SOLUO DE GLAUBER MORENO BARBOSA (RIO DE JANEIRO - RJ)

Primeiramente nomeia-se os elementos do setor circular: (a)  o ngulo do setor circular, (b) S, rea mxima, (c) 2p, o permetro do setor circular, (d) k, comprimento do arco do setor circular. Comprimento da circunferncia = 2r Fazendo uma regra de trs:

k 360k 180k = = = 2r 360 2r r

(1)

Conforme o enunciado, o jardineiro dispe de 100 metros de fio para cercar o setor circular. Somando-se os segmentos referentes aos raios e ao arco do setor circular, tem-se:

2 p = 2r + k = 100 k = 100 2r
Substituindo-se a relao (1) em (2) tem-se:

(2) (3)

180k 180 (100 2r ) = r r

Com (3) determina-se a rea do setor circular com uma relao entre a rea do setor circular e a rea da circunferncia:

Substituindo-se o  em (3) na relao (4), tem-se uma nova relao para a rea do setor circular 180 (100 2 r ) 2 r 2 180 (100 2r ) r (100 2 r )r r r = = = r 2 + 50r = S= 360 360 360 2 S ( r ) = r 2 + 50r (5) Observando-se a funo para a rea S em (5), que uma funo do 2 grau, com a < 0, e logo tem-se um mximo para a funo. Assim S(r) mximo em (5) para:

S r 2 S = = 360 r 2 360

(4)

r =

b 50 = 25 2a 2

(6)

Para o valor de r em (6), tem a rea mxima da funo S(r) em (6):

(25) 2 + 50(25) = 625 + 1250 S = 625


Assim, se o raio para se ter rea mxima 25 metros e a rea mxima correspondente 625 m2.

EUREKA! N20, 2004

54

Sociedade Brasileira de Matemtica

92) Seja (Fn)n a seqncia de Fibonacci, definida por F1 = 1, F2 = 1 e Fn+2 = Fn+1 + Fn , n <. Prove que mdc (Fm , Fn ) = Fmdc(m, n) para quaisquer inteiros positivos m e n.
SOLUO DE GIBRAN MEDEIROS DE SOUZA (NATAL - RN)

Para provarmos o que queremos temos que antes mostrar trs premissas, que so: I) Se m 1 e n > 1, ento Fn+m = Fn1 Fm + Fn Fm+1 Prova: (por induo sobre m)

m = 1: Fn+1 = Fn1 F n F n1 + F n (verdadeira) 1 +F 2 =F m = 2: Fn+2 = Fn1 F2 + Fn F3 = Fn1 + 2Fn = ( Fn1 + Fn ) + Fn = Fn + Fn+1 (verdadeira) Seja r > 2 e suponhamos a propriedade verdadeira para todo k, 2 k < r, e para todo n > 1.
Esta suposio, mais o fato de que a propriedade vale tambm para k = 1, nos garante que: Fn+(r2) = Fn1 Fr 2 + Fn Fr 1 e Fn+(r1) = Fn1 Fr1 + Fn Fr Somando membro a membro essas igualdades e levando em conta a frmula recursiva que define (Fn ):

Fn+r = Fn1 Fr + Fn Fr+1


Ou seja, a frmula vale tambm para r, sempre que n > 1 . O segundo princpio de induo nos garante ento que vale para todo m 1 e qualquer n > 1. II) Dois nmeros de Fibonacci consecutivos ( Fn e Fn+1 ) so primos entre si.

Prova: Seja d = mdc(Fn Fn+1 ). Como Fn e Fn+1 so maiores que zero, o mesmo ocorre com d. O fato de d ser divisor de Fn e Fn+1 implica que d | Fn1 pois

Fn1 = Fn+1 Fn .
Dividindo Fn e Fn1 , ento d divide Fn2 . Prosseguindo nesse raciocnio chegaremos a concluso que d | F2. Ento d = 1, pois F2 = 1 III) Se m | n , ento Fm | Fn .

EUREKA! N20, 2004

55

Sociedade Brasileira de Matemtica

Prova: Por hiptese n = mr, para algum r < . Procedemos por induo sobre r. Se r = 1, ento m = n e imediato que Fm | Fn . Seja r 1 e admitamos que Fm | Fmr . Ento levado em conta a relao fornecida por I:

Fm(r+1) = Fmr+m = Fmr1 Fm + Fmr Fm+1


Como Fm | Fmr 1 Fm e Fm | Fmr Fm+1 (pois, pela hiptese de induo, Fm divide

Fmr ), ento Fm divide a soma desses dois produtos. Ou seja: Fm | Fm(r+1)


Com as trs premissas em mo vamos questo: Se d = mdc(m, n) , ento prove que mdc( Fm , Fn ) = Fd . Prova: Mostraremos primeiro que se m = nq + r, ento mdc( Fm , Fn ) = mdc( Fn , Fr ). Observando as premissas feitas e levando em conta I:

mdc(Fm , Fn ) = mdc(Fnq+r , Fn ) = mdc(Fnq1 Fr + Fnq Fr +1, Fn )


Considerando porm que mdc(a, b) = mdc(a + c, b), sempre que b|c, e ainda que

Fn | Fnq (premissa III), chegamos a: mdc(Fm , Fn ) = mdc(Fnq1 Fr , Fn )


Mostremos que Fnq1 e Fn so primos entre si. De fato, se d um divisor comum a esses dois nmeros, ento d | Fnq1 e d | Fnq (devido a premissa III). Da a premissa II nos assegura que d = 1. Ora, se mdc(Fnq1, Fn ) = 1, ento mdc(Fr , Fn ) = mdc(Fnq1 Fr , Fn ), donde:

mdc( Fm , Fn ) = mdc( Fn , Fr ).
Assim, supondo m > n, e aplicando o processo das divises sucessivas para chegar a d = mdc(m, n) :

m = nq1 + r1 (r1 < n) n = rq 1 2 +r 2 (r 2 <r 1) r1 = r2q3 + r3 (r3 < r2 ) .......................................... rn2 = rn1qn + rr (rn < rn1) rn1 = rnqn+1 (onde rn = d )
EUREKA! N20, 2004

56

Sociedade Brasileira de Matemtica

o uso repetido do resultado anterior a cada uma das igualdades anteriores nos levar a concluir que:

mdc( Fm , Fn ) = mdc( Frn1 , Fd )


Como d | rn 1 , em virtude da premissa III, Fd | Frn1 , logo: mdc( Fm , Fn ) = Fd , cqd.

93) Um inteiro positivo n dito perfeito se n igual soma dos divisores positivos de n que so menores que n. Prove que um nmero par n perfeito se e somente se existe um nmero primo p 2 tal que 2p 1 primo e

n = 2 p1 (2 p 1) .
SOLUO DE CARLOS ALBERTO DA SILVA VICTOR (NILPOLIS - RJ)

(1) Lema: Se 2k 1 um nmero primo ento k tambm primo. Prova: Suponha que k no seja primo, logo podemos escrever k = x1 x2 onde x1 e

x2 so maiores que 1. 2k 1 = 2x1x2 1 = (2x1 1)(2x1 (x2 1) + 2x1 (x2 2) + ... + 2x1 +1) , ou seja, 2k 1 no
primo. Logo k dever ser necessariamente primo. (2) Suponha n = 2 p 1 (2 p 1) com p primo ( p 2). Seja S(n) a soma dos divisores de n menores que n:

S (n) = (1 + 2 + 22 + ... + 2 p 1 ) (1 + 2 p 1) 2 p 1 (2 p 1) S (n) = (2 p 1) 2 p 2 p1 (2 p 1) = 2 p1 (2 p+1 2 2 p + 1) S (n) = 2 p 1 (2 p 1) = n.


(3) Suponha S (n) = n (n par). Vamos mostrar que existe um primo (2 p 1) tal que n = 2 p 1 (2 p 1). Prova: J que n par podemos escrever na forma n = 2t m , onde m um nmero inteiro positivo mpar. J que n perfeiro, temos que 2n = 1 + 2 + 22 + ... + 2t

ou seja, m = (2t +1 1) s onde s um inteiro.


EUREKA! N20, 2004

(m) a soma dos divisores do mpar m; (2 1) (m) = 2n = 2 m . Como 2


t +1 t +1

) ( (m ) ) ,

onde

t +1

1 mpar temos que (2t +1 1)m,

57

Sociedade Brasileira de Matemtica

Suponha que s > 1, ento 1, s e (2t +1 1) s so divisores de m, e logo:

o que uma contradio, donde s = 1 e m = 2t +1 1 . Observe que t +1 (m ) = 2t +1 = 2  1 + 1 , ou seja, a soma dos divisores de m a soma do

(m) 1 + s + (2 1) s > 2 (2 1) (m) > (2 1) 2


t +1 t +1 t +1

t +1

s e

t +1

s = 2t +1 m = 2n ,

prprio m com a unidade, da m = 2t +1 1 primo, ou seja n = 2 p 1 (2 p 1) para t + 1 = p e, pelo lema (1), conclumos que existe p primo como nas condies do enunciado. 94) A ilha das amazonas habitada por amazonas e homens. As amazonas mandam em tudo, so inteligentssimas, ciumentssimas e muito fofoqueiras. O que uma amazona mais gosta de fazer trair outra amazona com o marido desta. Consumada a traio, ela conta o seu feito a todas as amazonas da ilha menos amazona trada. As outras amazonas tambm no contam nada vtima da traio. Mas se uma amazona descobre que est sendo trada ela mata o seu marido na prxima meia noite. A rainha das amazonas, que viva, v esta situao com desagrado. Ela v que h traio na ilha mas, como nunca ningum descobre nada, nenhum marido morre. No dia 1 de janeiro de 3333, ento, contrariando a tradio, ela chama todas as amazonas para a praa central e faz uma proclamao solene: "H traio nesta ilha." Nenhuma amazona sonha em duvidar da palavra da rainha e todas as amazonas sabem disso. Como j foi dito, todas so inteligentes e ciumentas: estes e os outros fatos mencionados neste enunciado at aqui so conhecimento comum entre as amazonas. Supondo que haja 1000 amazonas na ilha e que 365 delas tenham sido tradas, o que acontecer?
SOLUO DE EDUARDO FISCHER (ENCANTADO - RS)

Se houvesse somente uma traio, a trada no saberia de nada, e como havia pelo menos uma traio que ela no soubesse, mataria o seu marido na primeira noite. Como na primeira noite ningum morreu, uma mulher que soubesse de uma nica traio mataria seu marido na segunda noite, pois, como no houve morte na primeira noite, havia algo que ela no sabia. Assim, se houver exatamente duas traies, as tradas mataro seus maridos na seguinte noite. ...
EUREKA! N20, 2004

58

Sociedade Brasileira de Matemtica

Supondo que na (n 1)-sima noite ningum morreu, uma mulher que soubesse de apenas (n 1) traies mataria seu marido na n-sima noite, pois, como no houve morte na (n 1)-sima noite, havia algo que ela no sabia. Mostramos assim, por induo em n, que, se houver exatamente n traies (i.e., n maridos traidores) as tradas mataro seus maridos na n-sima noite. Lembrando que cada trada sabe de 364 traies, cada uma mataria o seu marido depois de uma ano, no 365 dia (isto , no reveillon de 3334).
Enviaram solues de problemas os seguintes leitores da EUREKA!
Besaleel Ferreira de Assuno Jnior Carlos Augusto David Ribeiro Georges Cobiniano Sousa de Melo Glauber Moreno Barbosa Guilherme Marques dos Santos Silva Jos Renato Carneiro e Carneiro Marcos Francisco Ferreira Martinelli Raphael Rodrigues Mata Wallace Alves Martins Wellington Klezewsky Pires Zzimo Pereira Teresina PI Fortaleza CE Joo Pessoa PB Rio de Janeiro RJ Enviado via e-mail Ribeiro Preto SP Rio de Janeiro RJ Salvador BA Rio de Janeiro RJ Aquidauana MS Campina Grande PB

Continuamos aguardando solues dos problemas 89, 90 e 95, propostos na Eureka! No. 18.

EUREKA! N20, 2004

59

Sociedade Brasileira de Matemtica

PROBLEMAS PROPOSTOS
Convidamos o leitor a enviar solues dos problemas propostos e sugestes de novos problemas para os prximos nmeros.

96) No quadriltero ABCD os ngulos A, C e D medem 100 e o ngulo ACB mede 40. Demonstre que BC DA = ( BC + AB DA)2 . 97) Seja p um primo mpar. Encontre todas as funes f : as seguintes condies: i)
ii)

que satisfazem

Se m n(mod p) ento f (m) = f (n). f (mn) = f (m) f (n) para quaisquer m, n

2 2, n <. 98) Seja (an ) n< uma seqncia tal que a1 > 2 e an +1 = an

Mostre que

a1 a12 4 1 = . 2 n =1 a1 a2 ... an

99) Num tringulo, a razo entre os raios das circunferncias circunscrita e inscrita

5 . Os lados do tringulo esto em progresso aritmtica e sua rea 2

numericamente igual ao seu permetro. Determine os lados do tringulo. 100) a) Um conjunto X < dito impressionante se existe m < tal que, para todo k < , existem elementos de X, a1 < a2 < ... < ak , tais que

a j +1 a j m, j < k .
Determine se possvel particionar nenhum deles impressionante.

<

em um nmero finito de conjuntos,

b) Determine se possvel particionar < em dois conjuntos A e B de modo que nem A nem B contm progresses aritmticas infinitas mas, para cada q <, A e B contm progresses aritmticas de q termos.

EUREKA! N20, 2004

60

Sociedade Brasileira de Matemtica

101)

a) Sejam ai , bi , ci reais positivos, para 1 i 3.

3 3 3 3 3 3 3 3 Prove que (a1 + a2 + a3 + b3 + c2 + c3 )(b13 + b2 )(c1 ) (a1b1c1 + a2b2c2 + a3b3c3 )3 .

b) Sejam a, b, c, x, y, z reais positivos. Prove que

a 3 b 3 c 3 ( a + b + c )3 . + + x 2 y 2 z 2 ( x + y + z)2

Problema 96 proposto por Miguel Cruz (Holguin, Cuba), problema 97 (Coria 2001) proposto por Samuel Barbosa Feitosa (Fortaleza - CE), problema 98 proposto por Gleydson Chaves Ricarte (Fortaleza - CE), problema 99 proposto por Geraldo Perlino (Itapecerica da Serra SP), problema 100 proposto por Anderson Torres (So Paulo - SP), problema 101 proposto por Okakamo Matsubashi (So Paulo - SP).
EUREKA! N20, 2004

61

Sociedade Brasileira de Matemtica

COORDENADORES REGIONAIS
Alberto Hassen Raad Amarsio da Silva Arajo Ana Paula Bernardi da Silva Antonio Carlos Nogueira Benedito Tadeu Vasconcelos Freire Carlos Frederico Borges Palmeira Claus Haetinger Cleonor Crescncio das Neves lio Mega Florncio Ferreira Guimares Filho Ronaldo Alves Garcia Reginaldo de Lima Pereira Ivanilde Fernandes Saad Jacqueline Fabiola Rojas Arancibia Janice T. Reichert Joo Bencio de Melo Neto Joo Francisco Melo Libonati Jos Carlos dos Santos Rodrigues Jos Cloves Saraiva Jos Gaspar Ruas Filho Jos Luiz Rosas Pinho Jos Vieira Alves Krerley Oliveira Licio Hernandes Bezerra Luzinalva Miranda de Amorim Mrio Rocha Retamoso Marcelo Rufino de Oliveira Marcelo Mendes Pablo Rodrigo Ganassim Ramn Mendoza Ral Cintra de Negreiros Ribeiro Reinaldo Gen Ichiro Arakaki Ricardo Amorim Srgio Cludio Ramos Tadeu Ferreira Gomes Toms Menndez Rodrigues Valdenberg Arajo da Silva Valdeni Soliani Franco Vnia Cristina Silva Rodrigues Wagner Pereira Lopes (UFJF) (UFV) (Universidade Catlica de Braslia) (UFU) (UFRN) (PUC-Rio) (UNIVATES) (UTAM) (Colgio Etapa) (UFES) (UFGO) (Escola Tcnica Federal de Roraima) (UC. Dom Bosco) (UFPB) (UNOCHAPEC) (UFPI) (Grupo Educacional Ideal) (Unespar) (UFMA) (ICMC-USP) (UFSC) (UFPB) (UFAL) (UFSC) (UFBA) (UFRG) (Grupo Educacional Ideal) (Colgio Farias Brito, Pr-vestibular) (Liceu Terras do Engenho) (UFPE) (Colgio Anglo) (INPE) (Centro Educacional Logos) (IM-UFRGS) (UEBA) (U. Federal de Rondnia) (U. Federal de Sergipe) (U. Estadual de Maring) (U. Metodista de SP) (CEFET GO) Juiz de Fora MG Viosa MG Braslia DF Uberlndia MG Natal RN Rio de Janeiro RJ Lajeado RS Manaus AM So Paulo SP Vitria ES Goinia GO Boa Vista RR Campo Grande MS Joo Pessoa PB Chapec SC Teresina PI Belm PA Campo Mouro PR So Luis MA So Carlos SP Florianpolis SC Campina Grande PB Macei AL Florianpolis SC Salvador BA Rio Grande RS Belm PA Fortaleza CE Piracicaba SP Recife PE Atibaia SP SJ dos Campos SP Nova Iguau RJ Porto Alegre RS Juazeiro BA Porto Velho RO So Cristovo SE Maring PR S.B. do Campo SP Jata GO

EUREKA! N20, 2004

62

CONTEDO

AOS LEITORES XV OLIMPADA DE MATEMTICA DO CONE SUL Enunciados e Solues XLV OLIMPADA INTERNACIONAL DE MATEMTICA Enunciados e Solues

2 3 13

ARTIGOS
CENTRO DE MASSA E APLICAES GEOMETRIA Emanuel Carneiro & Frederico Giro SEQNCIA DE FIBONACCI Ccero Thiago B. Magalhes COMO QUE FAZ? SOLUES DE PROBLEMAS PROPOSTOS PROBLEMAS PROPOSTOS COORDENADORES REGIONAIS 29

38 43 46 60 62

Sociedade Brasileira de Matemtica

AOS LEITORES
Chegamos a este nmero 21 com dois artigos: um de geometria e outro sobre a seqncia de Fibonacci. So as primeiras publicaes dos respectivos autores na Eureka!. Esperamos que venham outras e que cada vez mais autores contribuam com a revista. Apresentamos as solues da Olimpada do Cone Sul e da Olimpada Internacional de 2004, competies nas quais as equipes brasileiras tiveram muito bom desempenho. Agradecemos mais uma vez as contribuies dos leitores para a seo "Como que faz?" e para a seo dos problemas propostos, com solues e novos problemas, que fazem da Eureka! cada vez mais uma obra de criao coletiva. Aproveitamos para registrar que foi criada em 2004 a Associao Olimpada Brasileira de Matemtica AOBM, uma pessoa jurdica destinada a ajudar as Olimpadas de Matemtica no Brasil a crescerem e se consolidarem. Esperamos que a AOBM sirva como instrumento para maior integrao e organizao da comunidade olmpica. Estimulamos nossos leitores a se associarem AOBM, cujos scios recebem gratuitamente a revista Eureka!. As informaes sobre como se associar AOBM podem ser encontradas na nossa pgina na internet: www.obm.org.br

Os editores

EUREKA! N21, 2005

Sociedade Brasileira de Matemtica

XV OLIMPADA DE MATEMTICA DO CONE SUL


Enunciados e Solues
PROBLEMA 1

Maxi escolheu 3 dgitos e, fazendo todas as permutaes possveis, obteve 6 nmeros distintos, cada um com 3 dgitos. Se exatamente um dos nmeros que Maxi obteve um quadrado perfeito e exatamente trs so primos, encontrar os 3 dgitos que Maxi escolheu. D todas as possibilidades para os 3 dgitos.
SOLUO DE LEANDRO FARIAS MAIA (FORTALEZA CE)

Sejam x1 , x2 , x3 os dgitos escolhidos por Maxi. Note que x1 x2 , x1 x3 , x2 x3 pois as 6 reordenaes: x1 x2 x3 ,..., x3 x2 x1 so distintas. Agora faremos a lista de todos os nmeros de 3 dgitos quadrados perfeitos: 10 2 = 100 202 = 400 302 = 900 2 2 11 = 121 21 = 441 312 = 961 2 2 12 = 144 22 = 484 132 = 169 14 2 = 196 152 = 225 16 2 = 256 17 2 = 289 182 = 324 232 = 529 242 = 576 252 = 625 262 = 676 27 2 = 729 282 = 784

19 2 = 361 292 = 841 Perceba que: Os nmeros que tm algum zero no satisfazem o enunciado: 10 2 , 20 2 ,30 2. Reordenando 1, 6, 9 podemos obter 132 ,14 2 ,312. Assim, 132 ,14 2 ,312 no satisfazem o enunciado. O nmero dever apresentar no mnimo 2 dgitos mpares, pois se tiver no mximo um, teremos (se tivermos) no mximo 2 nmeros primos. Assim; 16 2 ,17 2 ,182 , 252 , 28 2 , 29 2 no satisfazem o enunciado. Os dgitos so distintos. Assim, 112 ,12 2 ,152 , 212 , 22 2 , 26 2 no satisfazem o enunciado.
EUREKA! N21, 2005

Sociedade Brasileira de Matemtica

Nos restam os nmeros: 19 2 , 232 , 24 2 , 27 2. Reordenando: 19 2 = 361;136,163,316,361,613, 631. 232 = 529; 259, 295,529,592,925,952. 24 2 = 576;567,576, 657,675,756,765. 27 2 = 729; 279, 297,729, 792,927,972. Perceba que os dgitos: 1, 3, 6 satisfazem o enunciado, pois, 163, 613, 631 (apenas) so primos e 361 (apenas) quadrado perfeito. 2, 5, 9 no satisfazem o enunciado, pois, 295, 592, 925, 952 so compostos e 529 (apenas) quadrado perfeito, assim teremos no mximo um primo. 5, 6, 7 no satisfazem o enunciado pelo mesmo raciocnio acima: 675, 765, 756 e 576 so compostos. 2, 7, 9 no satisfazem o enunciado pois todas as suas reordenaes so mltiplos de 9. Portanto, os dgitos que Maxi escolheu foram 1, 3, 6.
PROBLEMA 2

Dada uma circunferncia C e um ponto P exterior a ela, traam-se por P as duas tangentes circunferncia, sendo A e B os pontos de tangncia. Toma-se um ponto Q sobre o menor arco AB de C. Seja M a interseo da reta AQ com a perpendicular a AQ traada por P, e seja N a interseo da reta BQ com a perpendicular a BQ traada por P. Demonstre que, ao variar Q no arco AB, todas as retas MN passam por um mesmo ponto.
SOLUO DE LEANDRO FARIAS MAIA (FORTALEZA CE)

Como AP tangente a "C", ento: QBA = QAP = . Analogamente: PBQ = . Agora veja que: i) PNDB inscritvel, pois PNB = 90 = PDB. Assim; NPD = NBD = e NDP = NBP = .

Sejam = QBA, = QAB e "D" um ponto sobre AB de modo que PD AB.

EUREKA! N21, 2005

Sociedade Brasileira de Matemtica

M X

N A

Q D

ii) PMDA inscritvel, pois PMA = 90 = PDA. Assim; PDM = PAM = e MPD = MAD = . Portanto, de i) e ii) temos que NPD = MDP = NP // MD. NDP = MPD = PM // ND. Logo, PMDN paralelogramo, e ento NM e PD se cruzam no ponto mdio (*), assim para qualquer "Q" e AB (menor), MN passar por um ponto fixo que o ponto mdio da altura PD. Prova de (*):
A B

M D

AB // DC ACD = CAB, BC // AD ADB = CBD.


EUREKA! N21, 2005

ABD = BDC.

Sociedade Brasileira de Matemtica

Assim, ACD ACB( LAAO ) DC = AB e AD = BC. MDC ABM ( ALA) AM = MC e BM = MD.


PROBLEMA 3

Seja n um inteiro positivo. Chamamos Cn a quantidade de inteiros positivos x, menores que 10n, tais que a soma dos dgitos de 2x menor que a soma dos dgitos de x. Demonstre que Cn _ 4 n (10 1) . 9

SOLUO:
j Se m = a j 10 , com j=0 k j a j {0,1, 2,...,9}, j k , temos 2m = (2a j ) 10 . Note j =0 k

que, se a {0,1, 2,3, 4}, 2a {0, 2, 4,6,8} tem apenas um dgito, e, se a {5,6,7,8,9}, 2a {10,12,14,16,18} tem dois dgitos, sendo o primeiro deles igual a 1. Assim, na soma
j

(2a ) 10
j j =0

, no h "vai um", pois, se (2ai ) 10i e

(2a j ) 10 tm dgitos no nulos na k-sima, posio, com i < j, ento k = j = i + 1, sendo o dgito de (2ai ) 10i igual a 1, nessa k-sima posio, donde sua soma menor que 10 (pois o dgito corresponde de (2a j ) 10 j no mximo 8). Portanto, se s(m) denota a soma dos dgitos de m, s (m) = a j e s (2m) = s (2a j ). Agora, para
j =0 j =0 k k

a {0,1, 2,3, 4}, s (2a ) = 2a, e, para a {5, 6, 7,8,9} , s (2a ) = 2a 9. Portanto, para 0 a 9, s (2a ) + s (2 (9 a )) = 2a + 2(9 a ) 9 = 9 = a + (9 a ). Assim, se x = a j 10 j , (com 0 a j 9, j ) um inteiro positivo menor que 10 n , temos
j=0 n 1

(10n 1) x = (9 a j ) 10 j , e logo, s (2 x ) + s (2(10n 1 x )) =


j =0

n 1

= s (2a j ) + s (2 (9 a j )) = ( s (2a j ) + s (2 (9 a j )) = (a j + (9 a j )) =
j =0 j =0 j =0 j =0

n 1

n 1

n 1

n 1

EUREKA! N21, 2005

Sociedade Brasileira de Matemtica

= a j + (9 a j ) = s ( x ) + s ((10n 1) x ). Assim, s (2 x ) < s ( x ) se e somente se


j =0 j =0

n 1

n 1

s (2(10 1 x )) > s (10n 1 x ), e s (2 x ) = s ( x ) se e somente se


n

10n 1 A , onde 2 A = #{1 x < 10n | s(2 x) = s( x)}. Note agora que s ( x ) x (mod 9) e s (2 x ) 2 x (mod 9) , donde, se s (2 x ) = s ( x ), ento 2 x x (mod 9) , e logo s (2(10n 1 x )) = s (10n 1 x ). Portanto, Cn = x 0(mod 9). Cn 4 n (10 1). 9 Assim, A #{1 x < 10 n | x 0(mod 9)} = 10n 1 , 9 donde

Arnaldo escolhe um inteiro a, a _ 0 073, /4 08.4 0 :2 390 74 b, b _ 2-48 dizem, em segredo, o nmero que escolheram a Cernaldo, e este escreve em um quadro os nmeros 5, 8 e 15, sendo um desses a soma a + b. Cernaldo toca uma campainha e Arnaldo e Bernaldo, individualmente, escrevem em papis distintos se sabem ou no qual dos nmeros no quadro a soma de a e b, e entregam seus papis para Cernaldo. Se em ambos os papis est escrito NO, Cernaldo toca novamente a campainha, e o procedimento se repete. Sabe-se que Arnaldo e Bernaldo so sinceros e inteligentes. Qual o nmero mximo de vezes que a campainha pode ser tocada at que um deles escreva que sabe o valor da soma?
SOLUO DE TELMO LUIS CORREA JNIOR (SANTO ANDR SP)

PROBLEMA 4

Se a 9, a campainha toca apenas 1 vez: Arnaldo sabe que b 0, logo a nica soma possvel 15. O mesmo ocorre se b 9, com Bernaldo. Caso contrrio, a campainha toca 2 vezes: com a, b 8 no possvel decidir entre as somas dadas na primeira vez. Se a = 6, Arnaldo agora sabe que b 8, ento a soma no pode ser 5 ou 15, apenas 8. O mesmo ocorre se b = 6. Se a {0,1, 2,3,4,5,7,8}, ainda no possvel decidir entre 5, 8 e 15. O mesmo ocorre para Bernaldo. Caso a campainha toque pela terceira vez a, b {0,1,2,3,4,5,7,8} .

EUREKA! N21, 2005

Sociedade Brasileira de Matemtica

Se a = 2, Arnaldo sabe que b 6 e b 8, logo a nica soma possvel 5. O mesmo ocorre se b = 2. Caso a 2 e b 2 , ningum escreve o SIM. De fato: Se a {0,1,3, 4,5}, Arnaldo no pode decidir entre as somas 5 e 8. Se a {7,8}, Arnaldo no pode decidir entre as somas 8 e 15. O mesmo ocorre para Bernaldo. A campainha toca pela quarta vez, e ambos sabem que a, b {0,1,3, 4,5,7,8}. Se a = 3, Arnaldo sabe que b 2, logo a soma deve ser 8. Se a {0,1, 4,5} no possvel decidir entre 5 e 8, se a {7,8} , no possvel decidir entre 8 e 15. O mesmo ocorre com Bernaldo. A campainha toca pela quinta vez: ambos sabem que a, b {0,1,4,5,7,8} . Se a = 5, Arnaldo sabe que b 3 , logo a soma deve ser 5. Se a {0,1, 4} no possvel decidir entre 5 e 8, se a {7,8} no possvel decidir entre 8 e 15. O mesmo para Bernaldo. A campainha toca: sexta vez - ambos sabem que a, b {0,1, 4,7,8}. Se a = 0, Arnaldo sabe que b 5 , logo a soma deve ser 8. Se a {1, 4}, no possvel decidir entre 5 e 8, se a {7,8}, entre 8 e 15. O mesmo para Bernaldo. Stima vez: ambos sabem que a, b {1,4,7,8}. Se a = 8, Arnaldo sabe que a soma deve ser 15, se a {1, 4}, no pode decidir entre 5 e 8, se a = 7, no pode decidir entre 8 e 15. O mesmo com Bernaldo. Oitava vez: ambos sabem que a, b {1,4,7}. Se a = 7, Arnaldo sabe que a soma deve ser 8, se a {1, 4}, no pode decidir entre 5 e 8. O mesmo para Bernaldo. Nona vez: ambos sabem que a, b {1,4}. Arnaldo sabe que nica soma possvel se a = 1 5; se a = 4, a soma pode ser 5 ou 8. O mesmo para Bernaldo. A campainha toca pela dcima vez: agora ambos sabem que a = b = 4, e determinam a soma com segurana. Logo, a campainha pode ser tocada no mximo dez vezes.
PROBLEMA 5

Utilizando triangulinhos eqilteros de papel, de lado 1, forma-se um tringulo eqiltero de lado 2 2004 . Desse tringulo retira-se o triangulinho de lado 1 cujo centro coincide com o centro do tringulo maior. Determine se possvel cobrir totalmente a superfcie restante, sem superposies nem buracos, dispondo-se somente de fichas em forma de trapzio issceles, cada uma formada por trs triangulinhos eqilteros de lado 1.
EUREKA! N21, 2005

Sociedade Brasileira de Matemtica

SOLUO DE GABRIEL TAVARES BUJOKAS (SO PAULO SP)

Vamos mostrar por induo que qualquer n 1 (mod 9) possvel cobrir o tringulo dividido em n partes cada lado. Como 22004 (+1) (mod 9) j que 26 n 1 e 6 | 2004, 26 n 1 (mod 9) e 6 | 2004 para n = 22004 possvel. Vamos mostrar que se para n 1 (mod 9) possvel, ento para n + 9 tambm . Temos que n + 9 1 (mod 9). Cobriremos os 3 lados do tringulo maior assim: Comece por um vrtice A e cubra um lado a que este pertena com peas at restarem 2 espaos (como n 1 (mod 3), isto possvel). Agora preencha da mesma forma a segunda linha, at restarem 4 espaos. Na terceira linha, deixe um espao esquerda e preencha at onde foi a segunda linha (como indicado na figura acima). E, acima das primeiras duas peas da terceira linha, ponha mais uma (como na figura). Faa isso para todos os vrtices. Observe que no h nem espaos nem superposies. Assim criamos uma "barba" e o meio um tringulo de lado (n + 9) 9 (muito fcil de ver, j que a borda tem "largura" 3). O centro do tringulo grande coincide com o do de lado n. Assim, por hiptese de induo, este tringulo central pode ser preenchido, logo o tringulo grande tambm!. A base da induo n =1, caso em que no h nada a fazer. Assim, nossa afirmao est provada.

EUREKA! N21, 2005

Sociedade Brasileira de Matemtica

PROBLEMA 6

Sejam m, n inteiros positivos. Em um tabuleiro m n, quadriculado em quadradinhos de lado 1, considere todos os caminhos que vo do vrtice superior direito ao inferior esquerdo, percorrendo as linhas do quadriculado exclusivamente nas direes e . Define-se a rea de um caminho como sendo a quantidade de quadradinhos do tabuleiro que h abaixo desse caminho. Seja p um primo tal que rp(m) + rp(n) p, onde rp(m) representa o resto da diviso de m por p e rp(n) representa o resto da diviso de n por p. Em quantos caminhos a rea um mltiplo de p?
SOLUO:

Para resolver este problema, usaremos tcnicas de funes geratrizes descritas no artigo "Sries formais", de Eduardo Tengan, da Eureka! No. 11, pp. 34 39. Representaremos o tabuleiro por {1, 2,,m} {1, 2,,n}. Desta forma, os quadradinhos so representados por pares (i, j), com 1 i m,1 j n. Considere agora um caminho como no enunciado. Para 1 i m, seja bi o nmero de quadradinhos da forma (i, y) que esto abaixo do caminho. Como o caminho s desce ou anda para a esquerda, temos 0 b1 b2 b3 ... bm (e, reciprocamente, tais bi determinam um caminho). A rea do caminho yi = bi bi 1 . k m + 1,
m

b .
i =1 i

Definamos bm +1 = n e b0 = 0. Para 1 i m + 1, seja

Temos
i

y
i =1 m+1 i =1

m +1

= bn +1 b0 = n , Portanto,
m

e, rea

em do

geral, caminho

para

y
i =1 m i =1

= bk b0 = bk .

A = bi = (m + 1 i) yi = (m + 1 i) yi = j ym+1 j .
i =1 j =0

Assim, fazendo z j = ym +1 j , temos

zj = n e
j =0

jz
j =0

= A.
m

Assim, o problema equivalente a achar o nmero de solues de z0 + z1 + ... + zm = n, com zi 0 para 0 i m + 1 e isso, considere a funo F ( x, y ) =
EUREKA! N21, 2005
m

jz
j =0

mltiplo de p. Para

1 1 1 1 . Como = ... k 1 y 1 xy 1 xm y k =0 1 x y

10

Sociedade Brasileira de Matemtica

1 = 1 + x k y + ( x k y ) 2 + ..., F ( x, y ) = ar , s x r y s , onde ar , s o nmero de k 1 x y r , s< solues de z0 + z1 + ... + zm = s com zi 0, i 0 e zi + 2 z2 + 3 z3 + ... = jz j = r.


j =0 m

Em particular, ar ,n o nmero de caminhos como no enunciado com rea r. Queremos ento calcular

a
p|r

r ,n

, i.e., a soma dos ar ,n para os r mltiplos de p.

Para isso, consideremos a soma unidade (e logo

F (
k =0

p 1

, y ), onde = e 2 i

uma raiz p-sima da

k =0

p 1

kr

= 0, sempre que r no mltiplo de p). Temos kr y s =

F (
k =0

p 1

, y) =
k =0

p 1

r , s

< a

r ,s

r , s

< a

r ,s

p 1 kr y s = p ar ,s y s (pois, se r , s< k =0
p|r

p | r,

kr = p ) . Em particular, o coeficiente de y n em
k =0

p 1

F (
k =0

p 1

, y ) p vezes o

nmero de caminhos como no enunciado cuja rea um mltiplo de p. 1 = (1 + y + y 2 + ...)m +1 , e o coeficiente de y n m +1 (1 y ) em F (1, y ) o nmero de solues de k1 + k2 + ... + km +1 = n , ki 0, i , que
m m+n 1 1 m 1 k = = . Por outro lado, para 1 k p 1, F , y . ( ) kr m+1 1 y r =0 y kr r =0 1 y m Escrevendo m = qp + rp ( m) = ( q + 1) p ( p rp (m)), com 0 rp (m) < p, temos ( q +1) p 1

Para k = 0, F ( k , y ) = F (1, y ) =

1 r = m +1 ( q+ F ( , y ) = 1) p 1 y (q + 1) p
k

(1 y )
kr

( q +1) p 1

r =0

(y
1 kr

kr

1 y (q +1) p

r =m +1 p

(y

(1 kr y ) 1) q +1 =

( q +1) p 1 r =m +1

(1 y )
kr

(1 y )

+ p q 1

( q +1) p 1

Como o produto

r = m+1

(1 y )

tem p rp (m) 1 termos, ao desenvolver esse

produto, obtemos termos da forma a y s , com 0 s p rp ( m) 1. Por outro lado, todas as potncias de y em
EUREKA! N21, 2005
q +1 1 = (1 + y p + y 2 p + ...) tm expoente mltiplo p q +1 (1 y )

11

Sociedade Brasileira de Matemtica

de p, donde os termos no-nulos em F ( k , y ) tm o expoente de y congruente a s mdulo p, para algum s com 0 s p rp ( m) 1. Entretanto, rp ( m) + rp ( n) p, donde rp ( n) > p rp ( m) 1 , e, como n congruente a rp ( n) mdulo p, nenhum termo no nulo em F ( k , y ) tem expoente n. Assim, o nmero procurado de caminhos cuja rea um mltiplo de p 1 m + n . p m

":0 225964951 1 F 57 24 890 3 2074 /0   /J 948 F 4 2, 47 57 24 .43 0. /4 0 F :2 57 24 /0 0780330 ,88 2 .424 4 80 :3/4 4 907.0 74 0 4 6:,794 2, 4708 57 248 .43 0. /48 6:0 8 4 2 24036583 1, 220996011 1 0 213466917 1. /08.4-7 /47 /0890 34;4 57 24 4  57 24 /0 0780330 .43 0. /4 7 ,79 3 4 , F :2 419, 24 4 89, , 02 4 6:0 5,79 . 5, /4 !$ :2 574 094 .44507,9 ;4 5,7, 574.:7,7 57 248 /0 0780330 7085438E;0 50 , /08.4-079, /48 4 94 2, 4708 57 248 /0 0780330 ,9:, 20390 .43 0. /48 '0 , 995 20780330 47 5,7, 2, 4708 31472, 08
4

'4.H 8,- ,

EUREKA! N21, 2005

12

Sociedade Brasileira de Matemtica

XLV OLIMPADA INTERNACIONAL DE MATEMTICA


Enunciados e Solues
Caros leitores: nossa equipe da IMO-2004 tinha seis alunos muito diferentes, mas com uma caracterstica em comum: garra. Todos eles adotaram a ttica de escrever tudo o que passava pelas suas cabeas, de modo a maximizar as suas pontuaes. Assim, no podemos transcrever as suas solues ipsis literis. Ao contrrio, vamos colocar somente partes das solues deles, com alguns comentrios sobre as suas idias. Carlos Yuzo Shine, vice-lder do Brasil na IMO-2004 PRIMEIRO DIA
PROBLEMA 1

Seja ABC um tringulo acutngulo com AB AC. A circunferncia de dimetro BC intersecta os lados AB e AC nos pontos M e N, respectivamente. Seja O o ponto mdio do lado BC. As bissetrizes dos ngulos BAC e MON intersectam-se em R. Prove que as circunferncias circunscritas aos tringulos BMR e CNR tm um ponto em comum que pertence ao lado BC.
SOLUO DE THIAGO COSTA LEITE SANTOS (SO PAULO - SP)
A

/2 /2
M B

/ 2 / 2

2
K O

EUREKA! N21, 2005

13

Sociedade Brasileira de Matemtica

Seja a circunferncia com dimetro BC, B o circuncrculo de BMR e C o circuncrculo de CNR. Suponhamos sem perda de generalidade que o circunraio de ABC mede 1/2, de modo que BC = sen , AC = sen e AB = sen . Aqui, Thiago nota (e prova) que o ponto comum s duas circunferncias e o lado BC s pode ser o p da bissetriz: Temos pot A = AM AB = AN AC. Mas pot B A = AM AB e potC A = AN AC, logo A pertence ao eixo radical de B e C. Como R tambm pertence ao eixo radical, a reta AR o eixo radical de B e C. Logo o ponto comum a B e C e BC s pode ser a interseo de AR e BC, ou seja, a interseo da bissetriz de BAC e BC. Mais algumas consideraes geomtricas: como OM = ON (raios de ), NOR = MOR (OR bissetriz de MON) e OR comum, os tringulos MOR e NOR so congruentes (caso LAL), portanto RNO = RMO. Nesse momento, Thiago percebe como poderia terminar o problema: seja K a interseo da bissetriz de BAC e BC. A idia dele provar que K pertence a BC, B e C. Que pertence a BC bvio. Para provar que pertence a B, basta provar que B, M, R e K so concclicos; e, de modo anlogo, para provar que pertence a C, basta provar que C, N, R e K so concclicos. Aqui est a formalizao dele: se provarmos que RNO = /2, teramos RNC = + /2 e RNC + RKC = + /2 + + /2 = , portanto K pertence a C. Analogamente (aqui voc deve verificar que Thiago usa o fato que ele provou antes de que RNO = RMO), vamos ter K pertencente a B, portanto B e C tm um ponto comum em BC. Deste modo, s falta provar que RNO = /2. Isso um problema que costuma ser resolvido de duas maneiras: (1) com arrasto (isto , calculando todos os ngulos na figura) ou (2) arrasto seguido de contas (isto , com trigonometria, geometria analtica ou complexos). Thiago resolveu adotar a opo (2), com trigonometria (quando o coordenador viu as contas, ele comentou, estava indo to bem at aqui, por que ele teve que fazer essas contas??)

EUREKA! N21, 2005

14

Sociedade Brasileira de Matemtica

O tringulo NBO issceles, logo ONC = OCN = e NOC = 2. ( 2) ( 2 ) = Analogamente, MOB = 2 e = MOR = NOR = 2 . 2 Seja RNO = . A idia calcular alguma funo trigonomtrica de (geralmente acaba sendo a cotangente, por causa do truque da cotangente veja o artigo Geometria com Contas, Eureka! 17) e compar-la com a mesma funo trigonomtrica de /2 para provar que = /2. Mas por onde comear? Primeiro devemos procurar por tringulos que envolvam . Um deles vem da prpria definio de : RNO. Outro bem interessante o ARN, cujos ngulos envolvem tanto como /2, alm de ter RN em comum (um segmento a menos para calcular, como voc vai ver depois!). Ento parece ser uma boa estratgia utilizar esses dois tringulos. Lei dos senos em RNO: RN NO = sen ( 2 ) sen ( 2 + ) Lei dos senos em ARN: RN AN = sen 2 sen ( + 2) (1)

(2)

Veja que se dividirmos (1) por (2), RN se cancela, ento no precisamos calcular BC sen . Alm disso, = RN. S precisamos de NO e AN. NO fcil: o raio 2 2 AN = AC NC = sen NC. S falta NC! Pela lei dos senos em NOC, NC OC sensen2 sen 2sen cos = NC = = = sen cos 2 sen 2 sen sen2 sen logo AN = sen sen cos = sen( + ) sen cos = sen cos + sen cos sen cos = sen cos . Vamos usar repetidas vezes o fato de que sen(/2 x) = cos x e as frmulas de Prostafrese (que os gregos tiveram que deduzir na coordenao! procure-as na
EUREKA! N21, 2005

15

Sociedade Brasileira de Matemtica

Eureka! 17). Agora sim, dividindo (1) por (2) e substituindo NO e AN, obtemos a nossa conta final:
sen sen ( + sen sen 2 sen ( + 2 ) 2) 2 = = cos sen cos cos( ) cos( ) cos 2

cos (*) sen cos( ) = sen ( + 2) 2 Parece loucura, mas Thiago abriu os senos e co-senos. Mas isso tinha um motivo: ele queria achar cotg (lembre-se: o segredo do sucesso nas contas sempre ter uma meta em mente!). cos cos sen ( cos (*) cos cos sen + sensen sen = sen cos ( 2) 2 + 2) 2 cos cos = sen cos ( sensen cos cos sen sen ( 2) 2 2) 2
cos ( sen( + ) + sen( ) sen sen( )) =

sen(cos + cos( ) cos( ) + cos( + ))


+ cos cos 2 sen ( 2 ) sen ( 2 ) sen ( 2 ) = = = cotg cotg = + 2 sen sen 2sen ( 2 ) cos ( 2 ) cos ( 2 )

Como cotg = cotg que precisvamos.

e e pertencem ao intervalo ]0; [, temos = ,o 2 2 2

PROBLEMA 2

Determine todos os polinmios P(x) de coeficientes reais que satisfazem a igualdade


P( a b) + P(b c) + P(c a ) = 2 P( a + b + c )

para quaisquer nmeros reais a, b, c, tais que ab + bc + ca = 0.


SOLUO DE FBIO DIAS MOREIRA (RIO DE JANEIRO - RJ)

Note inicialmente que P(a b) + P(b c) + P(c a) = 2P(a + b + c) P(b a) + P(a c) + P(c b) = 2P(b + a + c) (*) P(b a) + P(c b) + P(a c) = 2P( a b c) (**)
EUREKA! N21, 2005

16

Sociedade Brasileira de Matemtica

Desde que ab + bc + ca = 0 (a primeira equao uma aplicao direta; a segunda segue se permutarmos a e b; a terceira se levarmos (a; b; c) em ( a; b; c), o que permitido pois ( a)( b) + ( b)( c) + ( c)( a) = ab + bc + ca = 0). Igualando (*) e (**), P(a + b + c) = P( a b c) desde que ab + bc + ca = 0. Mas dado S real, se tomarmos a = S, b = 0 e c = 0, ento a + b + c = S e ab + bc + ca = 0. Logo P(x) = P(x) para todo x real, ou seja, P par. Em particular, isto implica que o grau de P, gr(P), par. Agora vem a parte da soluo que usa anlise. Ele provou um lema geral bem til para o problema. Note inicialmente que se P(x) = k para todo x real ento 3k = 2k k = 0 que, de fato, uma soluo. Suponha agora que P no constante, de grau n. Seja Qx() = P(x) (aqui, Fbio usa uma idia relacionada a homogeneidade). Fixado x, Qx um polinmio em de grau n (de fato, de grau n se x 0 e de grau 0 se x = 0). Lema. lim Qx () = Cn x n , sendo Cn o coeficiente lder de P(x). n

Demonstrao: Note que Qx(), Qx(), , Qx(n 1)(), n, n 1, , so todos polinmios, logo so derivveis, e que para um desses polinmios qualquer, digamos R, temos lim | R() | = + , pois todos eles tm grau maior ou igual a 1. Ento, aplicando o

Qx () Qx( n ) () lim . Mas se P(x) = + n + n! = Cnxn + + C1x + C0, Qx() = Cnxnn + + C1x + C0, logo Qx(n)() = n!Cnxn, logo o limite desejado mesmo Cnxn. teorema de LHpital repetidas vezes, temos lim Agora, Fbio aproveita-se da homogeneidade da condio ab + bc + ca = 0: como ab + bc + ca = 0 (a)(b) + (b)(c) + (c)(a) = 0 para todo real, logo P((a b)) + P((b c)) + P((c a)) = 2P((a + b + c)) Qab() + Qbc() + Qca() = 2Qa+b+c() para todos , a, b, c reais, ab + bc + ca = 0. Q () Como cada um dos limites lim x n , x {a b; b c; c a}, n = gr(P) existe, + existe o limite da soma, que a soma dos limites. Logo

EUREKA! N21, 2005

17

Sociedade Brasileira de Matemtica

( ) Qa b () + Qb c ( ) + Qc a () Q = 2 lim a +b +nc n n n n Cn (a b) + Cn (b c ) + Cn (c a ) = 2Cn (a + b + c )n sendo a, b, c reais tais que ab + bc + ca = 0 e n = gr(P). Como Cn, coeficiente lder de P, no-nulo por hiptese, (a b)n + (b c)n + (c a)n = 2(a + b + c)n (***) Tome a = 2, b = 3 e c = 6 (note que ab + bc + ca = 0). Ento, substituindo em (***), (5)n + (3)n + 8n = 2 7n Como n par, obtemos 5n + 3n + 8n = 2 7n. Se n 7,

lim

1 8 1 8n = 7 n 7 n 1 + > 7 n 1 + 7 = 2 7 n , absurdo. 7 7 7 Logo n 6. Mas n = 6 implica 0 2 76 36 + 56 + 86 1 + 1 + 1 3 (mod. 7), absurdo. Logo P(x) = Ax2 + B ou P(x) = Ax4 + Bx2 + C. Fbio no precisava estudar o caso P(x) = Ax2 + B, pois isso um caso particular do caso P(x) = Ax4 + Bx2 + C, mas ele estudou. Na hora de estudar o caso P(x) = Ax4 + Bx2 + C, ele viu que a conta seria grande. Mas nesse momento ele lembrou as palavras que o prof. Luciano sempre diz: tem que ter garra para ser campeo! e no fraquejou, fazendo as contas: Substitua a = b = c = 0 na equao do enunciado. Segue imediatamente que C = 0. Logo P(x) = Ax4 + Bx2. Substituindo (isso mesmo!) e tendo em mente que ab + bc + ca = 0 e que hay que hacer las cuentas, pero sin perder la simetra jams, (a b)2[A(a b)2 + B] + (b c)2[A(b c)2 + B] + (c a)2[A(c a)2 + B] = = 2(a + b + c)2 [A(a + b + c)2 + B] A[(a b)4 + (b c)4 + (c a)4] + B[(a b)2 + (b c)2 + (c a)2] = 2A(a + b + c)4 + 2B(a + b + c)2 A[a4 4a3b + 6a2b2 4ab3 + b4 + b4 4b3c + 6b2c2 4bc3 + c4 + c4 4c3a + 6c2a2 4ca3 + a4] + B[a2 2ab + b2 + b2 2bc + c2 + c2 2ca + a2] = 2A(a2 + b2 + c2 + 2ab + 2bc + 2ca)2 + 2B(a2 + b2 + c2 + 2ab + 2bc + 2ca) 2A[a4 + b4 + c4 2ab(a2 + b2) 2bc(b2 + c2) 2ca(c2 + a2) + 3a2b2 + 3b2c2 + 3c2a2] + 2B[a2 + b2 + c2] = 2A[a4 + b4 + c4 + 2a2b2 + 2b2c2 + 2c2a2] + 2B[a2 + b2 + c2] A[a2b2 + b2c2 + c2a2 2ab(a2 + b2) 2bc(b2 + c2) 2ca(c2 + a2)] = 0 A[a2b2 + b2c2 + c2a2 2a2(ab + bc + ca) + 2a2bc 2b2(ab + bc + ca) + 2ab2c 2c2(ab + bc + ca) + 2abc2] = 0 A[(ab + bc + ca)2 2abc(a + b + c) + 2abc(a + b + c)] = 0 0 = 0.
EUREKA! N21, 2005

18

Sociedade Brasileira de Matemtica

Como todas as passagens so equivalncias, no h restrio sobre A e B. Portanto os nicos polinmios que satisfazem as condies do enunciado so os da forma P(x) = Ax4 + Bx2, A, B reais.
PROBLEMA 3

Um gancho uma figura formada por seis quadrados unitrios como no seguinte diagrama

ou qualquer uma das figuras obtidas desta aplicando rotaes ou reflexes. Determine todos os retngulos m n que podem ser cobertos com ganchos de modo que: i) O retngulo coberto sem buracos e sem sobreposies; ii) Nenhuma parte de nenhum gancho pode cobrir regies fora do retngulo.

SOLUO OFICIAL

Considere um preenchimento do tabuleiro m n. Para cada gancho A, existe um nico gancho B cobrindo o quadradinho interno de A com uma de suas extremidades. Alm disso, o quadradinho interno de B deve ser coberto por uma das extremidades de A. Assim, num recobrimento, todos os ganchos formam pares. H apenas duas maneiras de formar tais pares. Em um caso, A e B formam um retngulo 3 4; no outro, a sua unio um octgono, com lados 3, 2, 1, 2, 3, 2, 1, 2. Ento um tabuleiro m n pode ser coberto por ganchos se e somente se pode ser coberto com os pares de 12 quadradinhos acima. Suponha que tal recobrimento exista. Ento mn divisvel por 12. Provaremos que m ou n divisvel por 4. Assuma por absurdo que isso no acontece; ento m e n so ambos pares, pois mn divisvel por 4. Imagine o tabuleiro dividido em quadradinhos unitrios, com linhas e colunas rotuladas 1, 2, , m e 1, 2, , n, respectivamente. Escreva 1 no quadradinho (i, j) se exatamente um entre os nmeros i e j divisvel por 4, e 2, se i e j so ambos divisveis por 4. Como o nmero de quadrados em cada linha e coluna par, a soma de todos os nmeros escritos par. Mas no difcil verificar que um retngulo 3 4 sempre cobre nmeros com soma 3 ou 7; e o outro tipo de par sempre cobre nmeros com soma 5 ou 7. Conseqentemente, o nmero de pares de peas par. Assim, mn divisvel por 24 e, portanto, por 8, absurdo, pois supusemos que nem m nem n mltiplo de 4.

EUREKA! N21, 2005

19

Sociedade Brasileira de Matemtica

Note tambm que nem m nem n pode ser 1, 2 ou 5 (qualquer tentativa de colocar as peas nesses casos falha). Conclumos ento que se um recobrimento possvel ento m ou n divisvel por 3, m ou n divisvel por 4 e m, n {1; 2; 5}. Reciprocamente, se essas condies acima so verificadas, o recobrimento possvel (utilizando somente retngulos 3 4). Isso imediato quando 3 divide m e 4 divide n (ou vice-versa). Seja m divisvel por 12 e n {1; 2; 5} (ou vice-versa). Ento n pode ser representado como a soma de vrios 3s e vrios 4s. Ento o tabuleiro pode ser particionado em retngulos m 3 e m 4, que so fceis de cobrir, utilizando novamente retngulos 3 4. SEGUNDO DIA
PROBLEMA 4 Seja n 3 um inteiro. Sejam t1 , t2 ,..., tn nmeros reais positivos tais que

1 1 1 n 2 + 1 > (t1 + t2 + ... + tn ) + + ... + . tn t1 t2

Mostre que ti , t j e tk so as medidas dos lados de um tringulo para quaisquer i, j, k com 1 i < j < k n.
SOLUO DE HENRY WEI CHENG HSU (SO PAULO - SP)

Suponha tn tn 1 t2 t1 e seja tn = t1 + t2 + k. Temos 1 1 1 1 1 t1 + t2 + + tn1 1 + tn + +  + n2 + 1 > (t1 + t2 + + tn1 ) + + + + +1 tn1 tn tn1 t1 t2 t1 t2

Por Chebyshev (na verdade, poderamos usar Cauchy ou MA-MH), 1 1 1 + + + tn 1 n 1 (t1 + t2 +  + tn 1 ) t1 t2 n 1 n 1 n 1 1 1 1 2 (t1 + t2 +  + tn 1 ) + +  + (n 1) t t t 1 2 n 1 Assim,
EUREKA! N21, 2005

20

Sociedade Brasileira de Matemtica

1 1 1 t1 + t2 +  + tn 1 + tn + +  + tn tn 1 t1 t2 t +t t t t +  + tn 1 tn t 2n 1 > 1 2 + n + n + 3 + ++ n tn t1 t2 tn t3 tn 1 t t Como k + n 2 , temos tn tk t +t t t t +t t t 2n 1 > 2n 6 + 1 2 + n + n 5 > 1 2 + n + n tn t1 t2 tn t1 t2 Sendo tn = t1 + t2 + k, k t +t t + t + k t1 + t2 + k k t k t + 5 > 1 +1+ 2 + + 1 +1+ 5> 1 2 + 1 2 + + + + t1 t2 k t1 t2 t1 t2 k t1 t2 t2 t1 n2 + 1 > (n 1)2 + 1 + t1 t2 k k k + + + t2 t1 t2 t1 t1 + t2 + k t t Mas 1 + 2 2 . Deste modo, t2 t1 2> 1 1 1 1 1 1 0> k + 0 > k + t1 t2 t1 + t2 + k t1 t2 tn 1 1 1 Veja que + > pois t1 t2 tn. Logo k < 0 e tn < t1 + t2. t1 t2 tn O resto mais fcil: sejam 1 i < j n. Temos ti + tj t1 + t2 > tn tk para 1 k n e, portanto, ti, tj e tk so sempre lados de um tringulo.
PROBLEMA 5

Num quadriltero convexo ABCD a diagonal BD no bissetriz do ngulo ABC nem do ngulo CDA . Um ponto P no interior de ABCD satisfaz PBC = DBA e PDC = BDA. Prove que os vrtices do quadriltero ABCD pertencem a uma mesma circunferncia se e s se AP = CP.

EUREKA! N21, 2005

21

Sociedade Brasileira de Matemtica

SOLUO DE RAFAEL DAIGO HIRAMA (CAMPINAS - SP)

1 1 2

A P

2 1
D

Primeiro ato: consideraes gerais. Vamos determinar a figura a partir de quatro ngulos: sejam ABD = 1, BDA = 1, CBD = 2, CDB = 2. Ento DBP = 2 1 e BDP = 2 1. Veja que esses ngulos no dependem de P estar dentro do tringulo ABD (o que implica ABD > DBC e ADB > BDC) ou P estar dentro do tringulo BCD (o que implica ABD < DBC e ADB < BDC) pois se ABD = PBC ento ABD DBP = PBC DBP (+ se P est dentro de BDC e se P est dentro de ABD) ABP = DBC = 2. Para os ngulos com vrtice D anlogo. Usando lei dos senos nos tringulos ABP, BCP, ADP e DCP, temos AP AB CP BC = = (1) (2) sen 2 senAPB sen1 senBPC AP AD CP DC = = (3) (4) sen2 senAPD sen1 senDPC De (1) e (4), temos AP AB sen 2 senDPC = (I) CP DC sen1 senAPB

EUREKA! N21, 2005

22

Sociedade Brasileira de Matemtica

De (2) e (3), temos AP AD sen2 senBPC (II) = CP BC sen1 senAPD As equaes (I) e (II) so fundamentais para a soluo do problema, pois envolvem AP e CP, lados do quadriltero ABCD e, alm disso, observando a figura e essas equaes, notamos algumas simetrias interessantes. Rafael comea a us-las a partir de agora. Vamos l: senAPD = sen(APD + BPC BPC) = sen(APD + BPC) cosBPC cos(APD + BPC) senBPC. Seja = APD + BPC = 360 (APB + CPD). Logo senAPD = sen cosBPC cos senBPC (III) e senAPB = sen(APB + CPD CPD) = sen(APB + CPD) cosCPD sen(APB + CPD) cosCPD = sen cosCPD cos senCPD (IV) (lembre-se de que sen(360 x) = sen x e cos(360 x) = cos x). Segundo ato: ABCD cclico AP = CP. Supondo que ABCD cclico (esta a primeira vez que vamos usar este fato), AB CD AD BC temos, pela lei dos senos estendida, que . e = = sen1 sen2 sen1 sen 2 Substituindo essas duas ltimas igualdades e (III) e (IV) em (I) e (II), respectivamente, temos PC senAPD sen (VI): = = cos AP senBPC tg BPC PC senAPB sen (V): = = cos AP senCPD tg CPD Logo tgBPC = tg CPD ou sen = 0. A primeira possibilidade implica BPC + CPD = 180, mas a B, P e C seriam colineares e ento BD bissectaria os ngulos ABC e/ou ADC. Portanto sen = 0 e ento = 180, pois = APD + BPC est entre 0 e 360. Ento PC sen180, = cos180, = 1, ou seja, AP = PC. AP tg BPC

EUREKA! N21, 2005

23

Sociedade Brasileira de Matemtica

Terceiro ato: AP = CP ABCD cclico. AD sen2 senAPD CD sen1 senCPD e . = = AB sen 2 senAPB BC sen1 senBPC AD sen2 CD sen1 senAPD senCPD Multiplicando, temos . = BC sen1 AB sen 2 senBPC senAPB De (II) e (I), temos Aplicando a lei dos senos em ABD e BCD temos Assim, senAPD senAPB = senBPC senCPD AD AB CD BC e . = = sen1 sen1 sen 2 sen 2

Mas, fazendo as mesmas contas que fizemos em (V) e em (VI) (veja que as igualdades direita em (V) e (VI) no dependem de ABCD ser cclico), vemos que senAPD senAPB AD sen2 AD BC = 1. E ento . Sejam R1 = = =1 BC sen1 sen1 sen 2 senBPC senCPD e R2 os circunraios de ABD e BCD. Pela lei dos senos estendida, AD BC 2 R1 = = = 2 R2 R1 = R2. sen1 sen2 Logo os tringulos ABD e BCD tm o mesmo circunraio. S temos duas possibilidades para um circuncrculo de raio fixado em um segmento fixado: uma das possibilidades A, B, C e D serem concclicos, que o que queremos; a outra a circunferncia que passa por A, B e D ser a simtrica da que passa por B, C e D em relao a BD.
B

1
D
EUREKA! N21, 2005

24

Sociedade Brasileira de Matemtica

Neste ltimo caso vamos ter BAD = BCD. Como 1 + 1 + BAD = 180 e 2 + 2 + BCD = 180, 1 + 1 = 2 + 2. Mas veja que (1 > 2 e 1 > 2) ou (1 < 2 e 1 < 2) pois caso contrrio, como P est no interior do quadriltero ABCD, P deveria estar no interior de ABD e BCD ao mesmo tempo, o que impossvel. Logo no possvel que 1 + 1 = 2 + 2, e o problema acabou. Enfim, uma soluo muito parecida com uma trilogia grega, porm sem nenhuma tragdia.
PROBLEMA 6

Um inteiro positivo dito alternante se, na sua representao decimal, quaisquer dois dgitos consecutivos tm paridade diferente. Determine todos os inteiros positivos n tais que n tem um mltiplo que alternante.
SOLUO DE RAFAEL MARINI SILVA (RIO DE JANEIRO - RJ)

O principal diferencial da soluo do Marini que, em vez de procurar exemplos particulares para mltiplos alternantes de n, como a maioria dos estudantes fez, ele deu uma caracterizao bastante interessante dos nmeros alternantes. Vamos soluo: Seja T o conjunto dos nmeros cuja representao decimal consiste em algarismos todos menores do que 5. Veja que todo nmero alternante a pode ser escrito em uma das duas seguintes formas: se a mpar, a = 101010...101 + 2k = 
2 n 1 dgitos

102 n 1 + 2k, k T, k com no mximo 2n 99

dgitos; (I) + 2k = se a par, a = 101010...1010 


2 n dgitos

102 n 1 10 + 2k, k T, k com no mximo 99

2n + 1 dgitos; (II) Agora, Marini divide o problema em casos:


Caso 1: mdc(10; p) = 1.

Aqui, Marini toma sim um nmero particular. No muito difcil encontrar um nmero alternante nesse caso, se voc conhece o teorema de Euler-Fermat.

EUREKA! N21, 2005

25

Sociedade Brasileira de Matemtica

Tome k = 0 na forma (I) e n = (99p). Pelo teorema de Euler-Fermat, 10 (99p) 1 102 (99 p ) 1 (mod 99p) (veja que mdc(10; 99p) = 1) e portanto o nmero alternante 99 mltiplo de p.
Caso 2: p par, mas no mltiplo de 5.

Assim, p = 2e+1u, sendo u mpar e no mltiplo de 5. O nmero alternante que 102 n 1 10 + 2k, devemos encontrar da forma (II). Assim, 2e+1u| 99 102 n 1 5 + k, k T, k com no mximo 2n + 1 algarismos. 2eu| 99 102 n -1 5 + k. Para A nova dificuldade nesse caso a potncia de 2, ou seja, que 2e| 99 102 n 1 5 + k 0 (mod 2e) n suficientemente grande (maior que e/2), temos 99 k 5 991 (mod 2e). Marini resolveu, ento, provar um lema um pouco mais geral, mas que faz muito sentido: o conjunto T contm uma boa parte dos nmeros inteiros, ento
m Lema. Fixados a e m naturais e q {2,5} , existe t T tal que t a (mod q ).

Demonstrao. Induo sobre m.

Base de induo: m = 1: imediato, pois T > {0, 1, 2, 3, 4} > {0, 1}. Passo de induo: suponha o fato vlido para m = k. Vamos provar para m = k + 1. Suponha que 0 t < 10k . Seja Xt = {t; t + 10k; ; t + (q 1) 10k}. Note que esse conjunto est contido em T (pois q 5) e que Xt contm todos os possveis restos congruentes a t mdulo qk+1 (qk+1 no divide 10k e q divide 10). Da hiptese de induo, dado x natural existe t T com t T com t < 10k (se t 10k , trocamos t pelo nmero formado pelos seus k ltimos dgitos que congruente a t mdulo 10k, e logo mdulo qk) tal que x t (mod qk). Vendo mdulo qk+1 temos que x congruente a um elemento de Xt. Considerando a unio de todos os conjuntos Xt, com t variando entre 0 e qk 1, vemos que cobrimos todos os possveis restos (de fato, como Xt tem q elementos, a unio tem qk+1 elementos) e portanto o lema est demonstrado.
EUREKA! N21, 2005

26

Sociedade Brasileira de Matemtica

Assim, aplicando o lema congruncia k 5 991 (mod 2e), vemos que podemos escolher k0 T que satisfaz essa congruncia. Seja m o nmero de dgitos de k0. Tome s cpias de k0 (com alguns zeros esquerda para que fique com m + e + 1 (10m +e +1 )s 1 s 1 ( m + e +1) j . Veja que dgitos) e concatene-as, isto , seja k = k0 10 = k0 10m +e +1 1 j =0 k ainda um elemento de T e que k 5 991 (mod 2e), pois 10m+e+1 0 (mod 2e). Vamos fazer com que k seja mltiplo de u. Para isso, basta tomar s = ((10m+e+1 1) u) que o resto segue de Euler-Fermat. 102 n 1 10 + 2k. J 99 encontramos k. Agora, basta escolher um n adequado par acertar o nmero de algarismos de a. Tome n = (99u)v maior que a quantidade de dgitos de k. Pronto! Temos 102 n 1 102 n 1 10 + 2k = 2( 5 + k) 0 (mod 2e+1) (graas ao lema!) a= 99 99 102 n 1 10 + 2k 0 + 0 0 (mod u) a= 99 Logo a mltiplo de p. Retomando: o nmero alternante que procuramos a =
Caso 3: p mltiplo de 5, mas mpar. Seja p = 5eu, u mpar. Agora vamos procurar um nmero alternante da forma (I), ou 102 n 1 + 2k, k T. Para n suficientemente grande, seja, da forma 99 102 n 1 + 2k 0 (mod 5e) k 1981 (mod. 5e) 99

Novamente pelo lema, existe k0 T que satisfaz essa congruncia. Assim como no (10m +e +1 )s 1 , com = k0 caso anterior, tomemos k = k0 10 m + e +1 10 1 j =0 m+e+1 1) u) e n = (99u)v maior que a quantidade de dgitos de k. De s = ((10 modo anlogo ao caso anterior, conseguimos um alternante mltiplo de p.
s 1 m + e +1

Caso 4: p par e mltiplo de 5, ou seja, p mltiplo de 10.


EUREKA! N21, 2005

27

Sociedade Brasileira de Matemtica

Neste caso, devemos tomar um alternante da forma (II), ou seja, a = 2k, k T. Seja p = 10u. Temos que 10u|

102 n 1 10 + 99

102 n 1 10 + 2k 5|k. Como k T, seu 99 ltimo dgito s pode ser zero, de modo que 10|k. Seja k = 10w, w T. Temos ento 102 n 1 102 n 1 + 2w, w T. Como + 2w mpar, u deve ser mpar, e que u| 99 99 camos no caso 1 ou 3. Logo existe um mltiplo alternante de u s quando u mpar, ou seja, quando p no mltiplo de 20. Logo os inteiros positivos que no possuem mltiplo alternante so exatamente os mltiplos de 20.

'4.H 8,- ,

":0 0 8902 574 7088 08 ,7 92F9 .,8 ,7- 97,7 ,20390 43 ,8 1472,/,8 ,503,8 547 3 20748 57 248 894 14 574;,/4 70.039020390 547 03 7003 0 %0703.0 %,4 /4 8 0
4 J25 .48 0 07, ,9F 039 4 :2, .43 0.9:7, -,89,390 ,39 ,
4 9 24 574 70884 25479,390 02 8:, / 70 4 ,; , 8 /4 4 708: 9,/4 /0 ',3 /07 475:9 6:0 574;4: 02   6:0 0 8902 31 3 9,8 574 7088 08 ,7 92F9 .,8 /0 97H8 907248 1472,/,8 547 57 248 '0 , 4 ,79 4 :2 57057 39 , 3/, 3 4 5:- .,/4 /0 7003 0 %,4 02 995 ,7 ; 47 ,-8 2,9 %   0 4 ,79 4 0 548 9O7 4 /0 7 3, 7, 84-70 4 ,88:394 02 995 2,9 3479 089073 0/: = 7, 5,5078 57 208 5/1

EUREKA! N21, 2005

28

Sociedade Brasileira de Matemtica

CENTRO DE MASSA E APLICAES GEOMETRIA


Emanuel Carneiro & Frederico Giro UFC Nvel Avanado
1. INTRODUO

Chamaremos de sistema de massas um conjunto de n pontos P n no plano, 1, P 2 ,..., P sendo que ao ponto Pk = ( xk , yk ) est associada uma massa mk , de modo que

m1 + m2 + ... + mn 0. Definiremos o centro de massa desse sistema como sendo o ponto ( x, y) tal que: m1 x1 + m2 x2 + ... + mn xn m y + m2 y2 + ... + mn yn ; y= 1 1 , M M Onde M = m1 + m2 + ... + mn a massa associada a ele. x=
Notao:

Quando ao ponto (x, y) estiver associada uma massa m escreveremos (x, y)[m].
Observaes:

(i) Podemos interpretar fisicamente o centro de massa de um sistema como sendo o ponto onde ele concentra toda sua massa. Em termos prticos, isso nos ajuda a simplificar, por exemplo, problemas de Dinmica onde h aplicaes de foras sobre o sistema. (ii) Podemos considerar os pontos em massa de um sistema anlogo. (iii) Claramente o centro de massa nico.
n

. Neste caso, o clculo do centro de

2. PROPRIEDADES BSICAS Proposio 1.

Seja (x, y)[m] o centro de massa do sistema

S1 = {( x1 , y1 )[m1 ], ( x2 , y 2 )[m2 ],..., ( xk , y k )[m k ]}, e seja (a, b)[N] o centro de massa do sistema S 2 = {( a1 , b1 )[ n1 ], ( a2 , b2 )[n2 ],..., ( al , bl )[ nl ]}. Ento, se
EUREKA! N21, 2005

29

Sociedade Brasileira de Matemtica

M + N 0, o centro de massa do sistema S = S1 S 2 o centro de massa do sistema {( x , y )[ M ], (a , b )[ N ]}.


Demonstrao:

Por definio o centro de massa do sistema

S = S1 S 2 M x + Na M +N

o ponto

( X , Y )[ M + N ], onde:

X =

k i =1

mi xi + lj =1 n j a j M +N

que justamente a primeira coordenada do centro de massa do sistema {( x , y )[ M ], (a , b )[ N ]}. Para a segunda coordenada anlogo. A proposio acima nos d um algoritmo para calcular o centro de massa de um sistema com n pontos. Para isso tomamos dois pontos ( x1 , y1 )[m1 ] e

( x2 , y 2 )[ m2 ] quaisquer desse sistema e os substitumos pelo seu centro de massa com a massa m1 + m2 . Recamos assim num sistema com n 1 pontos e
continuamos o processo. Assim o clculo de centros de massa resume-se apenas ao caso n = 2, que estudamos a seguir:
Centro de massa de um sistema com duas massas

O centro de massa ( x , y )[ M ] de um sistema {( x1 , y1 )[m1 ], ( x 2 , y 2 )[m 2 ]} colinear com os pontos ( x1 , y1 ) e ( x2 , y 2 ) pois

x1 x2 x

y1 y2 y

1 1 = x1 y 2 + xy1 + x2 y xy 2 x2 y1 x1 y 1 m x + m2 x2 m1 y1 + m2 y2 = x1 y2 + 1 1 y1 + m1 + m2 m1 + m2 m x + m2 x2 m1 y1 + m2 y2 1 1 y 2 x2 y1 m1 + m2 m1 + m2 =0 x2 x1

E alm disso se chamamos ( x1 , y1 ) = A, ( x2 , y 2 ) = B e ( x , y ) = G vale que:


EUREKA! N21, 2005

30

Sociedade Brasileira de Matemtica

m1 AG + m2 BG = 0
tal fato deixado como exerccio para o leitor.
Observao:

Pela equao acima distinguimos alguns casos: As duas massas tm o mesmo sinal. Nesse caso o ponto G est entre A e B e vale que m1 AG = m2 BG . As duas massas tm sinais contrrios. Nesse caso G est fora do segmento AB e vale que m1 AG = m2 BG .
3. APLICAES GEOMETRIA Exemplo 1: Vamos tomar um tringulo ABC qualquer e pr massas iguais em seus

trs vrtices, ou seja consideraremos o sistema A[ p ], B[ p ], C [ p ]. Chamaremos de G o centro de massa desse sistema. Como encontrar o ponto G? (hummm) denotaremos C.M. = centro de massa. A[p]

P G

B[p]

M[2p]

C[p]

Vamos usar a proposio da seo anterior. O.C.M. de B[ p ] e C [ p ] o seu ponto mdio M. Podemos ento trocar B[ p ] e C [ p ] por M [2 p ]. Logo o ponto G ser o C.M. de A[ p ] e M [2 p ], que est sobre AM e divide AM na razo

AG 2 = . GM 1
EUREKA! N21, 2005

31

Sociedade Brasileira de Matemtica

Sejam N e P os pontos mdios de AC e AB. De modo anlogo poderamos ter provado que G BN e que G CP. Esta uma demonstrao diferente que as trs medianas concorrem em G, que portanto o baricentro do tringulo. Alm disso, segue do exposto acima que:

AG BG OG 2 = = = GM GN GP 1
Exemplo 2: Denote por a, b, c, os lados do tringulo ABC da maneira usual.

Vamos pr agora massas nos vrtices do tringulo proporcionais aos lados opostos, ou seja, considere o sistema A[ a ], B[b], C[ c ]. Seja I o C.M. desse sistema. Voc merece um prmio se descobrir quem I A[a]

K L I

B[b]
A[a]

J[b+c] a

C[c]

O raciocnio igual ao do exemplo anterior. O C.M. de B[b ] e C [c ] um ponto J

JB c = , ou seja, J o p da bissetriz interna. Logo I ser o JC b AI b + c = . C.M. de A[ a ] e J [b + c ]. Tiramos da que I AJ e que IJ a


no lado BC tal que Sejam BL e CK bissetrizes internas. De modo anlogo poderamos ter provado que I BL e que I CK , o que mostra que I o incentro. As razes saem de graa:

BI a + c CI a + b = = ; IL b IK c
Exemplo 3: Seja p o semipermetro do tringulo. Agora uma novidade: o sistema de massas ser A[ p a ], B[ p b], C[ p c]. Seja N o C.M. desse sistema. Voc realmente merece um prmio se descobrir quem o N.
EUREKA! N21, 2005

32

Sociedade Brasileira de Matemtica

A[ p a ]

pb Z

pc

Y N pa pa

B [p b]

pc

X[a]

pb

C[p c]

O C.M. de B[ p b ] e C [ p c ] um ponto X sobre o lado BC tal que

BX p c = , donde conclumos que BX = p c e que CX = p b. Este CX pb


ponto X onde o exincrculo relativo ao lado a toca este lado (como referncia sobre este fato podemos indicar [1]). Logo N ser o C.M. de A[ p a ] e

X [ p c + p b] = X [ a ].
Portanto N AX e

AN a = . Se considerarmos os pontos Y e Z onde os NX pa

exincrculos relativos aos lados b e c tocam estes lados, respectivamente, podemos mostrar que N BY e N CZ . Concluso: AX, BY e CZ so concorrentes em N que chamado Ponto de Nagel do ABC . Ora, ora, poderamos saber disso usando o teorema de Ceva (veja por exemplo [3]). Calma, o melhor ainda est por vir. As razes aqui so cortesias para ns:

BN b CN c = = ; NY p b NZ p c
O prximo resultado foi o que nos motivou a escrever este artigo. Ele mostra toda a beleza desta teoria, enquanto outros mtodos so ineficazes. Para uma demonstrao
EUREKA! N21, 2005

33

Sociedade Brasileira de Matemtica

completa (e bastante extensa) do prximo resultado usando a geometria plana clssica, veja [2].
Teorema 3.1. No ABC considere os pontos I, G e N como definidos acima.

Vale ento que I, G e N so colineares e ainda:

NG 2 = 1 GI
Prova: Seja p o semipermetro do tringulo. Considere um sistema de massas

A[ p ], B[ p ], C [ p ]. J sabemos que o C.M. desse sistema o baricentro G.


Fazendo uso da proposio 1, podemos dividir esse sistema em dois subsistemas S1 = A[ a ], B[b ], C [c ] e S 2 = A[ p a ], B[ p b ], C [ p c ]. O C.M. de S1 o incentro I com massa [a + b + c ] = [2 p ], enquanto o C.M. de S 2 o ponto da Nagel N com massa [ p a + p b + p c ] = [ p ]. Logo G ser o C.M. de

I [2 p ], N [ p ] o que implica I, N, G colineares (com G entre I e N) e ainda pela


equao do momento linear:

NG 2 = 1 GI
A[p] =[a + (p a)]

I[p] x
2y

O
2x

N[2p]

B[p] =[b + (p b)]

C[p] =[c + (p c )]

Corolrio 3.1.1. Em um tringulo qualquer ABC, sejam I,G,N como acima, O o circuncentro e I o ortocentro. Ento os pontos I, O, N, H formam um trapzio.

EUREKA! N21, 2005

34

Sociedade Brasileira de Matemtica

Prova: Sabemos que H, G, O so colineares (reta de Euler) e que:

HG 2 = GO 1
Segue ento do teorema anterior que IO paralelo a NH, logo I, O, N, H formam um trapzio, cujo encontro das diagonais G. Podemos aplicar estes mtodos do centro de massa em problemas que envolvem o ortocentro, o baricentro e os exincentros, para saber que massas devem estar nos vrtices, veja o problema 1. Divirta-se resolvendo estes problemas. Vale usar tudo, mas experimente a sua mais nova arma.
4. PROBLEMAS RELACIONADOS Problema 1

,B ,C (a) Verifique que o sistema A tem como C.M. o cos A cos B cos C ortocentro do tringulo.
(b) Verifique que o sistema A[ sen 2 A], B[ sen 2 B ], C [ sen 2C ] tem como C.M. o circuncentro. (c) Prove que o C.M. do sistema A[ a ], B[b], C [c ] o exincentro relativo ao lado a. Verifique os anlogos para os outros exincentros.
Problema 2

Sejam A, B, C, D pontos concclicos. Sejam G A , GB , GC , G D os baricentros dos tringulos BCD, ACD, ABD e ABC. Prove que G A , GB , GC , G D so concclicos.
Problema 3

Sejam ABCD um quadriltero no espao de forma que AB, BC, CD e DA sejam tangentes a uma esfera nos pontos X, Y, Z, W. Prove que estes pontos so coplanares.
Problema 4

Sejam X, Y e Z os pontos onde o incrculo do tringulo ABC toca os lados BC, AC e AB, respectivamente. Mostre que o incentro de ABC est sobre a reta que passa pelos pontos mdios de BC e AX. (veja uma soluo em [5]).
EUREKA! N21, 2005

35

Sociedade Brasileira de Matemtica

Problema 5

Considere 6 pontos em uma dada circunferncia. Tomamos trs destes pontos e marcamos seu baricentro G1 . Em seguida, marcamos o ortocentro H 2 dos outros trs pontos e traamos o segmento G1 H 2 . Mostre que todos os segmentos G1 H 2 passam por um ponto fixo.
Problema 6
6 = 20 3

possveis

Seja ABCD um quadriltero convexo inscritvel com os lados opostos AD e BC se encontrando em P, e AB e CD em Q. Prove que o quadriltero EFGH, determinado em ABCD pelas bissetrizes de DPC e CQB , um losango.
Problema 7

Seja PABC um tetraedro e sejam A1 , B1 , C1 os pontos mdios das arestas BC, AC e AB, respectivamente. Seja um plano paralelo face ABC que intercepta as arestas PA, PB, PC nos pontos A2 , B2 , C 2 respectivamente. (a) Prove que A1 A2 , B1 B2 , C1C 2 concorrem em um ponto D. (b) Determine o lugar geomtrico dos pontos D quando varia.
Problema 8

(a) Considere 4 pontos que formam um sistema ortocntrico (cada um o ortocentro do tringulo formado pelos outros trs). Ponha massas iguais nesses 4 pontos. Prove que o centro de massa o centro do crculo dos nove pontos de cada um desses 4 tringulos (veja [1] e o problema proposto No. 107 na pgina 61). (b) (Beltrami) Prove que o C.M. do sistema formado pelo incentro e pelos trs exincentros com massas iguais o circuncentro.
Problema 9

Seja ABCD um quadriltero convexo inscritvel com os lados opostos AD e BC se encontrando em P, e AB e CD em Q. Prove que as bissetrizes dos ngulos DPC e

CQB e a reta que une os pontos mdios das diagonais do quadriltero ABCD (diagonal de Euler) concorrem.

EUREKA! N21, 2005

36

Sociedade Brasileira de Matemtica

Problema 10

(Banco IMO/97) No ABC acutngulo, sejam AD, BE alturas e AP, BQ bissetrizes internas. Sejam I e O o incentro e o circuncentro do tringulo ABC, respectivamente. Prove que os pontos D, E e I so colineares se e somente se P, Q e O so colineares.
Agradecimentos: A nosso amigo Carlos Shine pela primeira verso digitada deste

material, na Semana Olmpica de 2001, em Salvador - BA.

REFERNCIAS: [1] Coxeter, H.S.M.; Greitzer, S.L., Geometry Revisited, MAA, 1967. [2] Johnson, R.A., Advanced Euclidean Geometry, Dover Publications, 1960. [3] Castro, L.G.M., Introduo Geometria Projetiva, Eureka!, vol 8, pp 16-27, 2000. [4] Honsberger, R., Mathematical Morsels, MAA, 1978. [5] Moreira, C.G.T., Wagner, E., 10 Olimpadas Iberoamericanas de Matemtica, OEI, 1996.

+ 1 F 57 24 ":0 28433 2 890 3 2074 /0   /J 948 F 4 6: 394 2, 47 57 24 .43 0. /4 0 14 /08.4-0794 02     894 24897, 6:0   3 4 F :2 3 2074 /0 $ 075 38 894 F :2 3,9:7, J25,7 9, 6:0 k 2 + 1 F .4254894 5,7, 94/4 n < ;0 , , :70 ,  5E  70/: 3/4 5,7,  4 3 2074 /0 3,9:7, 8 2034708 6:0  6:0 F 4 20347 3 2074 /0 $ 075 38 .43 0. /4 84-70 48 6:, 8 3 4 80 8,-0 80 8 4 3 20748 /0 $ 075 38 4: 3 4 .43 0.9:7,
80 6:0 303 :2 /0 08 80 ,                0   '0 , 995 80;03900347-:89 .42 5,7, 2, 8 31472, 08 3. :8 ;0 84-70 .424 , :/,7 , 574;,7 088, .43 0.9:7,
7830457 n EUREKA! N21, 2005

'4.H 8,- ,

37

Sociedade Brasileira de Matemtica

SEQNCIA DE FIBONACCI
Ccero Thiago B. Magalhes Nvel Avanado
INTRODUO

O nome seqncia de Fibonacci, foi dado pelo matemtico francs Edouard Lucas no sculo XIX. Porm, a seqncia surgiu de um problema que estava proposto na obra "Liber Abaci" de Leonardo de Pisa (1180 1250), conhecido como Fibonacci. O problema era o seguinte: "Um homem pe um casal de coelhos dentro de um cercado. Quantos pares de coelhos sero produzidos em um ano, se a natureza desses coelhos tal que a cada ms um casal gera um novo casal, que se torna frtil a partir do segundo ms?" Depois de sculos de trabalho, possvel hoje citar uma quantidade enorme de propriedades da seqncia do nmero de coelhos existentes aps n meses. O objetivo deste trabalho apresentar algumas propriedades bsicas desta seqncia.
Definio

A seqncia de Fibonacci definida da seguinte maneira: f1 = f 2 = 1 e f n = f n 1 + f n 2 , n > 2. Por convenincia, algumas vezes usaremos f 0 = 0.
Propriedades bsicas

(I) Para todo n 1 : f1 + f 2 + ... + f n = f n + 2 1 Prova: n = 1: f1 = f 3 1 Vamos supor q 1 e f1 + f 2 + ... + f q = f q + 2 1

n = q + 1 : f1 + f 2 + ... + f q + f q +1 = f q + 2 1 + f q +1 = f q + 3 1
(II) Se m 1 e n > 1, ento f n + m = f n 1 f m + f n f m +1 Prova: Vamos fazer induo sobre m: m = 1: f n +1 = f n 1 f1 + f n f 2 = f n 1 + f n (verdadeira)

m = 2 : f n + 2 = f n 1 f 2 + f n f3 = f n 1 + 2 f n = ( f n 1 + f n ) + f n = f n + f n +1
(verdadeira) Seja q > 2 e suponhamos a propriedade vlida para todo k , 2 k < q , e para todo n > 1. Esta suposio, mais o fato de que a propriedade vale tambm para k = 1, nos garante:
EUREKA! N21, 2005

38

Sociedade Brasileira de Matemtica

f n + ( q 2) = f n 1 f q 2 + f n f q 1 f n + ( q 1) = f n 1 f q 1 + f n f q
Somando membro a membro essas igualdades e levando em considerao a frmula recursiva que define ( f n ) :

f n + q = f n 1 f q + f n f q +1
Ou seja, a frmula vale tambm para q, sempre que n > 1. O princpio da induo nos garante ento que vale para todo m 1 e qualquer n > 1. (III) Dois nmeros de Fibonacci consecutivos f n e f n +1 so primos entre si. A prova fica como exerccio. (IV) Se m | n, ento f m | f n . Prova: Por hiptese n = mq, para algum q <. Usaremos induo sobre r. Se q = 1, ento m = n e fcil ver que f m | f n . Seja q 1 e admitamos que

fm | f

mq

Ento, usando a propriedade (II):

f m ( q +1) = f mq + m = f mq 1 f m + f mq f m +1
Como f m | f mq 1 f m e f m | f mq f m +1 (pois, pela hiptese de induo, fm divide

f mq ), ento f m divide a soma desses dois produtos. Ou seja: f m | f m ( q +1) .


(V) Seja d = mdc(m, n), ento mdc ( f m , f n ) = f d . Prova: Induo em m + n. Se m = 1, mdc(m, n) = 1 e mdc(fm, fn) = mdc(1, fn) = 1 = f1. Se m + n = 2 trivial. Se m = n no h o que provar. Se 2 m < n, f n = f m + ( n m ) = f m 1 f n m + f m f n m + 1 m dc ( f m , f n ) =

mdc ( f m , f m 1 f n m ) = m dc ( f m , f n m ), que igual, por hiptese de induo a f mdc ( m , n m ) = f mdc ( m , n ) .


Veja tambm a soluo do problema proposto No. 92 na Eureka! 20, pp 55 57.
2 (VI) Seja x = x + 1, ento, para n = 2, 3, ns temos que

( III )

x n = f n x + f n 1 .
Prova: trivial o caso n = 2. E se x = f n x + f n 1 para algum n 2, ento
n

x n +1 = x n x = ( f n x + f n 1 ) x = f n x 2 + f n 1 x
EUREKA! N21, 2005

39

Sociedade Brasileira de Matemtica

= f n ( x + 1) + f n 1 x = ( f n + f n 1 ) x + f n = f n +1 x + f n ,
como desejado!
2 Vejamos que as razes de x = x + 1 so os nmeros =

1+ 5 1 5 . e= 2 2

Ento, para todo n = 2, 3, ns temos

n = f n + f n 1
e

n = f n + f n 1 .
Subtraindo as duas ltimas equaes temos que = f n ( ), e notando
n n

que =

5 , ns encontramos a frmula de Binet n n . fn =

Problema 1

Seja =
Soluo:

1+ 5 . Determine todos os n < tais que n n 2 seja inteiro. 2


2

Note que raiz do polinmio p ( x ) = x x 1, com isso e usando a expresso da propriedade (V) temos que:

n n 2 = f n + f n 1 n 2 = ( f n n 2 ) + f n 1 .
Uma vez que irracional, segue da igualdade acima que n s ser
n 2

inteiro quando f n n = 0, e nosso problema equivale a determinar todos os


2 2 n < tais que f n = n . Para tanto, observe que: f1 = 1, f 2 = 1, f 3 = 2, f 4 = 3, f 5 = 5, f 6 = 8, f 7 = 13, f 8 = 21, f 9 = 34, f10 = 55, f11 = 89, f12 = 144, f13 = 233, f14 = 377.

Assim,

f12 = 12 2 e f13 > 13 2 , f14 > 14 2. Por

outro lado, se

f n > n2 e

f n +1 > (n + 1) 2 , ento f n + 2 = f n +1 + f n > n 2 + (n + 1) 2 > (n + 2) 2 , desde que n > 3,


EUREKA! N21, 2005

40

Sociedade Brasileira de Matemtica


2 donde segue por induo que f n > n para n > 13. Assim, as nicas solues so n = 1 e n = 12.

Problema 2:

Sejam n e k dois inteiros positivos quaisquer. Ento entre duas potncias k k +1 consecutivas n e n no podemos ter mais que n nmeros de Finonacci. Sugesto: use a propriedade (I)
Problema 3:

Seja f n a seqncia de Fibonacci ( f1 = 1, f 2 = 1, f n +1 = f n + f n 1 ). Calcule a srie

Problema 4:

n=2

fn . f n 1 f n +1

2 Ache a, se a e b so nmeros inteiros tais que x x 1 um fator de

ax17 + bx16 + 1.
Fortemente ligada seqncia de Fibonacci, e to interessante quanto, a seqncia de Lucas que definida da seguinte maneira:

Ln = f n 1 + f n +1 L1 = 1, L2 = 3, L3 = 4,...
Obs: fcil perceber que de acordo com a definio da seqncia de Lucas temos que L0 = 2. Usando a frmula de Binet temos que:

Ln = f n 1 + f n +1 =
Como =

n 1 n 1 n +1 n +1 = +

1 n1 n1 + + .
1 + = 5. Portanto,

1 1+ 5 1 5 + = 5 e , ento e = 2 2

Ln = n + n .
Problema 5:

Prove que f 2 n = f n Ln .

EUREKA! N21, 2005

41

Sociedade Brasileira de Matemtica

Problema 6:

Sejam L0 = 2, L1 = 1, e Ln + 2 = Ln +1 + Ln , para n 0, a seqncia de Lucas. Prove que, para todo m 1,

L
k =1

2k

= f 2m +1 ,

onde f n a seqncia de Fibonacci.


Problema 7:
3 Achar o termo geral pn se p0 = 1 e pn +1 = 5 pn 3 pn , para n 0.

Problema 8:

Todo natural pode ser unicamente escrito como soma de nmeros de Fibonacci distintos, de ndices no consecutivos e maiores que 1. (Teorema de Zeckendorff).
Problema 9:
93 19 93 Sejam a e b inteiros positivos tais que b divisvel por a e a divisvel por f f b19 . Prove que ( a 4 + b 8 ) n+1 divisvel por ( ab ) n para todo n > 1.

Problema 10:

Prove que nenhum nmero de Fibonacci potncia de 7.


Problema 11:

Sejam ( f n )

a seqncia de Fibonacci e, para todo inteiro positivo n,

Vn =
rea 1 .
2

f n2 + f n2+ 2 .

Prove que, para todo inteiro positivo n, Vn , Vn +1 , Vn + 2 so lados de um tringulo de

REFERNCIAS: [1] Ross Honsberger, Mathematical Gems III, MAA, 1985. [2] Hygino H. Domingues, Fundamentos da Aritmtica, Atual, 1991.

EUREKA! N21, 2005

42

Sociedade Brasileira de Matemtica

COMO QUE FAZ?


PROBLEMA 6 PROPOSTO POR RAFAEL ALVES DA SILVA (TERESINA - PI) da Olimpada Ibero-americana de Matemtica de 1999.

Seja n um inteiro maior que 10 tal que cada um dos seus dgitos pertence ao conjunto S = {1, 3, 7, 9}. Prove que n tem algum divisor primo maior ou igual a 11. Soluo: Como o ltimo dgito de n mpar, n mpar. Como o ltimo dgito de n no 0 nem 5, n no mltiplo de 5. Assim, se n no tivesse nenhum fator primo maior ou igual a 11, os nicos possveis fatores primos de n seriam 3 e 7. Mostraremos por induo que todo nmero natural cujos nicos fatores primos so 3 e 7 da forma n = 20k + r, onde k natural e r {1, 3, 7, 9} , e portanto o dgito das dezenas de n deve ser par, e logo no pode pertencer a S = {1, 3, 7, 9} (e assim, se todos os dgitos de n pertencem a S, devemos ter n < 10). Para isso, note que 1 = 20 0 + 1, e ainda n = 20k + 1 3n = 20 (3k) + 3 e 7n = 20 (7k) + 7 n = 20k + 3 3n = 20 (3k) + 9 e 7n = 20 (7k + 1) + 1 n = 20k + 7 3n = 20 (3k + 1) + 1 e 7n = 20 (7k + 2) + 9 n = 20k + 9 3n = 20 (3k + 1) + 7 e 7n = 20 (7k + 3) + 3, e nossa afirmao est provada.
PROBLEMA 7 PROPOSTO POR RAFAEL ALVES DA SILVA (TERESINA - PI) da Olimpada da Bielorrsia de 2000.

Seja M = {1, 2, 3,..., 40}. Ache o menor inteiro n para o qual possvel particionar M em M subconjuntos disjuntos tais que, sempre que a, b e c (no necessariamente distintos) pertencem ao mesmo subconjunto, ento a b + c. Soluo: Mostraremos que n = 4. Vamos ver como construir um exemplo de uma partio de n em 4 conjuntos de acordo com o enunciado, a partir de exemplos menores: temos a partio {1} = {1}, com n = 1, para M = {1}; a partio {1, 2, 3, 4} = {1, 4} {2, 3}, com n = 2, para M = {1, 2, 3, 4}; a partio {1, 2, 3, 4, 5, 6, 7, 8, 9, 10, 11, 12, 13} = {1, 4, 10, 13} {2, 3, 11, 12} {5, 6, 7, 8, 9} com n = 3, para M = {1, 2,,13}; a idia , dada uma partio {1, 2,, k} =
EUREKA! N21, 2005

43

Sociedade Brasileira de Matemtica

A n , obter uma partio {1, 2,..., 3k + 1} = B1 B2 ... Bk Bk +1 , 1 A 2 ... A tomando Bk+1 = {k +1, k + 2,...,2k +1} e, para i k, Bi = Ai (Ai + (2k +1)). Assim, obtemos, para M = {1,2,...,40}, M = {1,4,10,13,28,31,37,40}{2,3,11,12,29,30,38,39} {5,6,7,8,9,32,33,34,35,36}{14,15,16,17,18,19,20,21,22,23,24,25,26,27}, e fcil
verificar que esta partio satisfaz a condio do enunciado. Para mostrar que no possvel obter uma tal partio de M em apenas 3 subconjuntos, suponha que M = A B C. Como M tem 40 elementos, algum dos conjuntos A, B ou C tem pelo menos 14 elementos. Digamos que A contenha os 14 elementos x1 < x2 < ... < x14 . Os 13 elementos x14 x1 , x14 x2 ,..., x14 x13 devem pertencer a B C, pois, se x14 xj = y A, ento x14 = xj + y, com y , x j , x14 A, contradio. Assim, B ou C contm pelo menos 7 desses 13 elementos, digamos y1 < y2 < ... < y7 , onde yi = x14 xji , onde j7 < j6 < ... < j1. Suponhamos que esses elementos pertencem a B. Temos ento e, ( x14 x j3 ) ( x14 x ji ) como

y3 y2 C e y3 y1 C, pois, se y3 y j = z B, para algum j {1, 2}, y3 = y j + z , com y3 , y j , z B , absurdo,

y3 yi = ( x14 x j3 ) ( x14 x ji ) = x ji x j3 , se

y3 yi = z A, para algum i {1, 2}, teramos x ji = x j3 + z , com xji , xj3 , z A ,


absurdo. Vamos agora considerar o elemento y2 y1 > 0. Como y1 e y2 pertencem a B, y2 y1 no pode pertencem a B. Como y2 y1 = ( y3 y1 ) ( y3 y2 ), com

( y3 y2 ), ( y3 y1 ) C, y 2 y1 no pode pertencer a C. Finalmente, como y2 y1 = (x14 x j2 ) (x14 x j1 ) = x j1 x j2 , com x j1 A e xj2 A, y2 y1 no pode


pertencer a A, absurdo, pois y2 y1 M = A B C.
Publicamos a seguir as solues dos problemas 121 e 195 da seo "Olimpadas ao redor do mundo", por sugesto de Bruno, da Espanha. 121. (Rssia-2001) Os valores da funo quadrtica f (x ) = x 2 + ax + b para dois

inteiros consecutivos so os quadrados de dois inteiros tambm consecutivos. Mostre que os valores da funo quadrtica so quadrados perfeitos para todos os inteiros coincide com o conjunto dos valores de g para os inteiros. Soluo: Suponha f (m) = k 2 e f (m + 1) = (k + 1) 2 , com m e k inteiros. Seja g ( x) = f ( x + m). O conjunto dos valores de f para os inteiros.
EUREKA! N21, 2005

44

Sociedade Brasileira de Matemtica

Temos g ( x) = x 2 + cx + d para certos valores de c e d. Temos d = g (0) = f ( m) = k 2 e 1 + c + d = g (1) = f (m + 1) = ( k + 1) 2 , donde d = k 2 e c = (k + 1) 2 1 k 2 = 2k , ou seja, g ( x) = x 2 + 2kx + k 2 = ( x + k ) 2 , e logo g(x) um quadrado perfeito para todo x inteiro.
195.(Eslovnia-2002) Sejam M o ponto mdio da base AB do trapzio ABCD; E um

ponto interior ao segmento AC tal que BC e ME intersectam-se em F; G o ponto de interseo de FD e AB; H o ponto de interseo de DE e AB. Mostre que M o ponto mdio do segmento GH. Soluo: Podemos supor sem perda de generalidade, aplicando uma transformao afim, que A = (0, 0), B = (1, 0), C = (1, 1), e logo D = (a, 1), para algum a < 1. Ento 1 M = ,0 e E = (t, t), para algum t (0,1). Como F pertence a BC, F = (1, y) para 2 1 algum y, e, supondo t , como F pertence a ME, y 1 = t t 1 , donde 2 2 2 1 t (caso t = , BC e ME so paralelas e no se intersectam, logo no y= 2t 1 2 existiria o ponto F). Como H pertence a AB, H = (x, 0) para um certo x. Como H 1 t t (1 a ) , donde (1 t ) x = t (1 a ), ou seja, x = . pertence a DE, = 1 t tx ax Finalmente, como G pertence a AB, G = (z, 0) para um certo z, e como G pertence a 1 (2t 1) at t (2t 1) t 1 at (2t 1) , donde , ou seja, z = . = z = FD, 1 z 2t 1 2t 1 1 t az x + z 1 1 t 1 G+H Assim, , ou seja, M o ponto mdio do = = , e logo M = 2 2 1 t 2 2 segmento GH.

( ) (

EUREKA! N21, 2005

45

Sociedade Brasileira de Matemtica

SOLUES DE PROBLEMAS PROPOSTOS


Publicamos aqui algumas das respostas enviadas por nossos leitores.

90) Prove que, para todo inteiro positivo n e para todo inteiro no nulo a, o polinmio xn + axn1 + axn2 + ... + ax 1 irredutvel, i.e., no pode ser escrito como o produto de dois polinmios no constantes com coeficientes inteiros.
SOLUO (baseada em um argumento de Artur Avila):

1 n n1 n P + axn2 + ... + ax 1. Como P P a ( x) a ( x) = x + ax a , a ( x) = x P x irredutvel se e somente se P a ( x) irredutvel. Assim, podemos supor sem perda de e generalidade que a um inteiro positivo. Nesse caso, P a (0) = 1 P a (1) = (n 1)a > 0, n 2. claro que para n = 1 o polinmio P a ( x) = x 1 irredutvel.
Seja Para n 2 , deve existir ( 0,1) com P a ( ) = 0.

Qa (x) = Pa (x) (x 1) = (x 1)(xn 1) + ax(xn1 1) = xn+1 + (a 1)xn (a +1)x +1. Como raiz de P a ( ) = 0.
Seja Seja

Qa (x) = Pa (x) (x 1) = (x 1)(xn 1) + ax(xn1 1) = xn+1 + (a 1)xn (a +1)x +1. Como raiz de P tambm raiz de Qa ( x). Seja ento a ( x), Q (x) 1 R(x) = a = xn + (a + 1)xn1 +(a + 1)xn2 +2 (a + 1)xn3 +... +n2 (a + 1)x . x Temos R(1) = Q (1) (1 ) = 0, e, se raiz de P raiz de e de Qa ( x) a ( x), R(x). Mostraremos que > 1 para todo , tal que raiz de Pa (x).
Como raiz de R(x), temos n + ( a + 1) n 1 + ( a + 1) n 2 + ... +

1 . Se 1, n + ( a + 1) n 1 + ( a + 1) n 2 + ... + n 2 ( a + 1) + n 2 (a + 1) = + ( a + 1)
n n 1

+ ... + n 2 ( a + 1) 1n + ( a + 1)1n 1 + ... +

EUREKA! N21, 2005

46

Sociedade Brasileira de Matemtica

1 1 1 = = , valendo a igualdade se, e somente se, = = 1, mas Pa (1) 0, donde 1, absurdo (na verdade, mostramos que todas as raizes de R ( x ) tm mdulo maior ou igual a 1; em particular, raiz simples de Pa ( x ), caso contrrio (0,1) seria raiz de R ( x)). Suponha agora que Pa ( x ) = f ( x ) g ( x), onde f e g so polinmios no constantes com coeficientes inteiros. Como o coeficiente constante de Pa ( x ) 1, os + n 2 (a + 1) 1 = R(1) +
coeficientes constantes de f(x) e de g(x) pertencem a {1, 1} (e podemos supor sem perda de generalidade que seus coeficientes lderes so iguais a 1). Assim, o produto das razes de g(x) (e de f(x)) pertencem a {1, 1}. Por outro lado, podemos supor sem perda de generalidade que raiz de f ( x ). Como todas as razes de g(x) so razes de Pa ( x ) distintas de , elas tm todas mdulo maior que 1, e logo seu produto no pode pertencer a {1, 1}, absurdo. 95) "Resta-Um" um jogo de tabuleiro na qual as peas ocupam um tabuleiro formando parte de um reticulado retangular (na verdade, existem variaes em tabuleiros de reticulado triangular). O nico movimento permitido consiste em tomar duas peas em casas adjacentes vizinhas a uma casa vazia, e fazer a pea mais distante da casa vazia pular sobre a outra pea, ocupando a casa vazia. A pea pulada retirada.

(esse movimento pode ser feito para a direita, para a esquerda, para cima ou para baixo). Agora imagine um tabuleiro que um reticulado retangular infinito e uma reta que contm uma linha do reticulado, dividindo-o em dois lados. Todas as casas de um dos lados da linha esto vazias e cada casa do outro lado da linha pode ou no ter uma pea. Quantas peas, no mnimo, precisamos para chegar a uma casa do lado vazio do tabuleiro, a uma distncia n da linha ? Abaixo indicamos uma casa a distncia n, para n = 1,2,3,4,5.

EUREKA! N21, 2005

47

Sociedade Brasileira de Matemtica

5 4 3 2 1

SOLUO DE BERNARDO FREITAS PAULO DA COSTA (RIO DE JANEIRO RJ)

Vamos botar eixos no quadriculado, de acordo com a posio: Y

"Fronteira" X

Assim, a casa marcada com ' (0, 0), sua vizinha esquerda (1, 0) e acima (0, 1). Supomos, sem perda de generalidade, que a pea final termina na casa (0, n) para n = 1, 2, 3, 4, 5. Usando isto, vemos que o movimento deveria "tender para o centro e para cima". Com esta idia, vamos associar energias s peas nos quadrados do tabuleiro, atravs da seguinte funo:
2 = + 1, > 1 E (a, b) = onde 2 + = 1, 0 < < 1 b a

EUREKA! N21, 2005

48

Sociedade Brasileira de Matemtica

Como as equaes que os definem so simtricas, temos = 1

S para conferir:

1 5 2

1+ 5 >1 1+ 5 2 = 2 1 5 <0 2 5 1 (0,1) 5 1 2 = 2 1 5 <0 2

1 5 2

Assim, se fizermos um movimento

a 2

a 1

a 2

,a0

ou um movimento

,a0 a +2 a+ na direo do centro, estaremos trocando uma energia b + b 1 por

a +1

a +2

a +2

b a , que so iguais por construo.


J se fizermos

a +1

a +2

a +2

,a>0 ou o contrrio (com a < 0) trocaremos energia com perdas, pois a a + 2 (para a > 0; se a < 0 a a 2 ). Para movimentos verticais, vemos que

EUREKA! N21, 2005

49

Sociedade Brasileira de Matemtica

b+2

b+1

conserva energia b + b +1 = b + 2
a a
b+2

) e que

b+1

perde (pois b + 2 > b e b +1 > 0 ). Assim, a energia das peas no tabuleiro no aumenta. Vejamos a energia mxima do tabuleiro: ET = E (coluna central) + E (quadrantes negativos) = E(coluna central) + 2E(1 quadrante negativo) =

= =

b = 0

b 0 + 2 E (coluna em a)
a =1

0 a + 2 b b = a =1 b = + a = b + 2 b b =0 a =1 b =0 1 1 1 1 1 1 a Mas 2 = + 1 1 = + 2 1 = 2 = + 2 1 1 a =1 1 1

1 1 + 2 a 1 2 1 2 a =1

2 = 2 1 + 2 a = 2 1 + 1 a =1

EUREKA! N21, 2005

50

Sociedade Brasileira de Matemtica

Como 2 + = 1, temos 1 = 2 , e assim

2 ET = 2 1 + 2 = 2 1 + 2 = 2 (1 + 2 ) = 2 (1 + + ) 2 2 = ( + ) = 2 3 = 5
Ora, como a energia total do tabuleiro 5 , se tivermos um nmero finito de movimentos (para poder chegar em (0, 5) de verdade) teremos usado um conjunto finito de peas e portanto de energia estritamente menor do que 5 . Desta forma, impossvel chegar a esta casa com movimentos "resta-um". Decomponhamos n para n = 1, 2, 3, 4 para sabermos o nmero de peas

a + 2 = a +1 + a necessrias. Vale lembrar as relaes: a = a +1 + a +2 1 = 1 + = 1 + 2 ( = +1) = 1 1 = 1 + 2 peas:


"Fronteira" OK

Altura 2: 2 = + 1 = 1 + (1 + )

s temos 1 pea com energia 1 s temos 3 peas com energia

= 1 + ( + 2 + )

4 peas:

s temos 5 peas com energia 2 (em geral, para h 0, s temos 2h + 1 peas com energia h )
"Fronteira"

EUREKA! N21, 2005

51

Sociedade Brasileira de Matemtica

"Fronteira"

"Fronteira"

Altura 3: 3 = 2 + = 2 + 1 = 3 + 2 = 1 + 2 + (2)

= 1 + 2 + (2 + 2 2 ) = 1 + 3 + 2 2 + () = 1 + 3 + 3 2 + 3 8 peas:
vai (0, 2)

"Fronteira"

"Fronteira"

OK

"Fronteira"

EUREKA! N21, 2005

52

Sociedade Brasileira de Matemtica

Vamos ao ltimo caso (altura 4):

4 = 3 + 2 = 2 2 + = 3 + 2 = 5 + 3
= 1 + 3 + (4) = 1 + 3 + (4 + 4 2 ) = 1 + 3 + 4 2 + (4 2 + 43 ) = 1 + 3 + 5 2 + 4 3 + (3 2 ) = 1 + 3 + 5 2 + 7 3 + 3 4
Teoricamente, isto daria 1 + 3 + 5 +7 + 3 = 19 peas para chegar a altura 4. Entretanto, isto impossvel. Vejamos: Pintemos o tabuleiro com 3 cores em diagonal
B C A B C A B C A B C C A B C A B C B C A B C A B C A B C A B C A B C

A B C A B C A

A B C A B C

A "Fronteira" B A C B A C B A

Toda vez que realizarmos um movimento, tiramos uma pea de duas cores adjacentes e a terceira cor recebe uma pea a mais. Assim, invertemos a paridade dos 3 simultaneamente. Portanto, a paridade relativa se mantm. As casas dentro da regio indicada estaro todas ocupadas (suas energias somam 1 + 3 + 5 2 + 7 3 ). A soma das cores 5A + 7B + 4C Se a soma final 1A + 0B + 0C, temos que a paridade de B e C a mesma, e oposta paridade de A, logo das 3 4 que restam, temos uma quantidade par da cor A, uma quantidade mpar de cor B e uma quantidade para da cor C. Assim, temos as seguintes possibilidades: 2A + 1B + 0C 0A + 3B + 0C 0A + 1B + 2C Mas s h 1 casa B valendo 4 logo a possibilidade OA + 3B + 0C est excluda. Assim, conclumos que h 1 pea numa casa B, a nica do nvel 4 . Por simetria (ou seja, pintando o tabuleiro na outra direo : /// ),
EUREKA! N21, 2005

53

Sociedade Brasileira de Matemtica

temos que deve haver tambm uma pea na casa simtrica a essa casa B do nvel 4 em relao coluna central, a qual uma casa C, e logo a soluo OA + 1B + 2C. Assim, temos a seguinte configurao:

"Fronteira"

X ? X ? X ?

As casas marcadas com X so A ou simtricas a casas A, e logo esto proibidas. As casas com esto forosamente ocupadas, e das casas com ?, uma, e apenas uma, ser ocupada (para fazer 19). Isolemos a coluna central; sobra (pelo menos) um bloco consistindo apenas das seguintes peas: (ou do outro lado, por simetria igual).

Ora, a pea , para ser aproveitada, deve tomar uma pea que est a sua direita e ir para a coluna 1. Entretanto, para terminarmos com peas apenas na coluna zero, as duas indicadas pelas setas devem tomar as peas que esto direita (pois seno no conseguiramos mandar todas as peas desse bloco para a coluna central usando apenas essas peas; se usssemos outras peas para isso perderamos energia). Nesse caso, as peas que sobram do lado esquerdo ficam todas na altura 0, e a pea que estava originalmente na coluna 0 e na coluna 1 no conseguira chegar na coluna central sem que haja perda de energia. Assim, no conseguimos (agora por impedimento de movimento, no de energia) chegar em (0, 4) com 19 peas. Mas possvel com 20 (fao 3 = 4 + 5 ):

EUREKA! N21, 2005

54

Sociedade Brasileira de Matemtica

"Fronteira"

"Fronteira"

"Fronteira"

"Fronteira"

"Fronteira" "Fronteira"

"Fronteira"

OK!!

96) No quadriltero ABCD os ngulos A, C e D medem 100 e o ngulo ACB mede 40. Demonstre que BC DA = ( BC + AB DA)2 .

EUREKA! N21, 2005

55

Sociedade Brasileira de Matemtica

SOLUO DE HEYTOR BRUNO E MARLON JNIOR (FORTALEZA CE)

A figura ilustra o problema:


A 20 80

60 D 100 60 C

40

AC sen100 = DA sen60 BC sen80 ACB (Lei dos Senos): = AC sen 60 AC BC = AC 2 = BC DA DA AC ACD (Lei dos Senos):
Assim,

BC DA = ( BC + AB DA)2 AC 2 = ( BC + AB DA)2 . Basta provar ento


que AC = BC + AB DA.

ACD (Lei dos Senos): DA = ACB (Lei dos Senos): BC =


Da, AC + DA = BC + AB

AC sen60 sen100 AC sen80 AC sen 40 e AB = sen60 sen 60

AC +

AC sen60 AC sen80 AC sen 40 = + sen60 sen60 sen100

EUREKA! N21, 2005

56

Sociedade Brasileira de Matemtica

sen100 + sen60 sen80 + sen 40 = . sen60 sen100

Usando o fato de que

p+q pq sen p + sen q = 2 sen cos , 2 2


A equao acima verdadeira se e somente se

2 sen80 cos 20 2 sen60 cos 20 = , o que obviamente verdade. sen100 sen 60


99) Num tringulo, a razo entre os raios das circunferncias circunscrita e inscrita

5 . Os lados do tringulo esto em progresso aritmtica e sua rea 2


numericamente igual ao seu permetro. Determine os lados do tringulo.
SOLUO DE KELLEM CORRA SANTOS (RIO DE JANEIRO RJ)
A

R 5 = 2 R = 5r r 2
R a

a+q

r B

aq

Perimetro de ABC 2 p = a + (a + q) + (a q) = 3a p = pr = A( ABC ) =

3a 2

a( a + q)(a q) 4R

EUREKA! N21, 2005

57

Sociedade Brasileira de Matemtica

3a a (a 2 q 2 ) a2 q2 a2 q2 3r = = 15r 2 = a 2 q 2 r= 2 4R 2R 5r
Como a rea numericamente igual ao permetro, temos:

(1)

pr = 2 p r = 2
Por outro lado,

A( ABC ) = =

p( p a )( p a q)( p a + q ) =

3a a a a q + q = 2 2 2 2

a2 1 a 3 q2 2 4

De (1), temos:

15r 2 = a 2 q 2 15 2 2 = a 2 q 2 60 = a 2 q 2 q 2 = a 2 60
Logo A( ABC ) =

a 3a 2 a a2 3 a 2 + 60 = 3 60 2 4 4 2 3a 2 = 3a 2

Tambm podemos escrever:

A( ABC ) = pr =
Igualando:

3a =

3a 2 a 3 60 2 4 3a 2 36 3a 2 3a 2 = = 62 = 3 60 60 12 60 4 3 4 4 3a 2 a 2 = 16 4 a = 8, pois a > 0. 4

48 =
De (1):

15r 2 = a 2 q 2 60 = 64 q 2 q = 2
Logo, os lados do tringulo ABC so 6, 8 e 10. 101) a) Sejam ai , bi , ci reais positivos, para 1 i 3.

3 3 3 3 3 3 3 3 Prove que (a1 + a2 + a3 + b3 + c2 + c3 )(b13 + b2 )(c1 ) (a1b1c1 + a2b2c2 + a3b3c3 )3 .

b) Sejam a, b, c, x, y, z reais positivos. Prove que

EUREKA! N21, 2005

58

Sociedade Brasileira de Matemtica

a 3 b 3 c 3 ( a + b + c )3 + + . x 2 y 2 z 2 ( x + y + z)2
SOLUO DE MARCELO RIBEIRO DE SOUZA e WALLACE MARTINS (RIO DE JANEIRO RJ)

a) Como esta uma desigualdade homognea, podemos supor


3 3 3 + a2 + a3 (a1 ) =1 3 3 + b3 (b13 + b2 ) =1 3 3 + c3 (c13 + c2 ) =1

Com isto nossa desigualdade passa a ser: 1 (a1b1c1 + a2b2 c2 + a3b3c3 )3 . Isto imediato de MA MG:

(a13 + b13 + c13 ) a1b1c1 3 3 3 3 + b2 + c2 (a2 ) a2b2c2 3 3 3 3 + b3 + c3 (a3 ) a3b3c3 3


Somando tudo e elevando ao cubo ficamos com:

1 ( a1b1c1 + a2b2 c2 + a3b3c3 ) cqd.


3

b) Este item decorre diretamente do anterior, tendo em vista que


3 3 3 1 3 1 3 1 3 1 3 1 3 1 3 a b c 3 3 3 3 3 3 2 + 2 + 2 x + y + z x + y + z 3 3 x 3 y z

1 1 1 1 1 1 a x3 x3 + b y3 y3 + c z3 z 3 2 2 2 x3 3 y z3
Portanto segue que

a 3 b3 c 3 ( a + b + c ) . 2 + 2 + 2 y z ( x + y + z )2 x
3

EUREKA! N21, 2005

59

Sociedade Brasileira de Matemtica

PROBLEMAS PROPOSTOS
Convidamos o leitor a enviar solues dos problemas propostos e sugestes de novos problemas para os prximos nmeros.

102) Voc recebe x metros de arame para cercar um terreno na forma de um tringulo pitagrico (os lados so nmeros inteiros), com a condio de que a medida do cateto menor seja 24 metros. Qual dever ser a medida do cateto maior e o comprimento do arame, a fim de que a rea seja: a) mxima? b) mnima? 103) Sejam A e B matrizes 2 2 com elementos inteiros. Sabendo que A, A + B, A + 2B, A + 3B e A + 4B so invertveis e que os elementos das respectivas inversas tambm so todos inteiros, mostre que A + 5B tambm invertvel e que os elementos da sua inversa tambm so inteiros. 104) ABC um tringulo. Mostre que existe um nico ponto P de modo que:
2 2 2

( PA ) + ( PB ) + ( AB )

= ( PB ) + ( PC ) + ( BC ) = ( PC ) + ( PA ) + (CA)
2 2 2 2 2

105) O baricentro do tringulo ABC G. Denotamos por g a , gb , gc as distncias desde G aos lados a, b e c respectivamente. Seja r o rdio da circunferncia inscrita. Prove que:

2r 2r 2r , gb , g c 3 3 3 g + gb + gc b) a 3 r
a) g a 106) Os polinmios P0 ( x, y, z ), P 1 ( x, y, z ), P 2 ( x, y, z ),... so definidos por

P0 ( x, y, z ) = 1 e Pm+1 ( x, y , z ) = ( x + z )( y + z ) Pm ( x, y , z + 1) z 2 Pm ( x, y, z ), m 0. Mostre que os polinmios Pm ( x, y , z ), m < so simtricos em x, y, z, i.e., P m ( x, y, z) = P m ( x, z, y) = P m ( y, x, z) = P m ( y, z, x) = P m ( z, x, y) = P m ( z, y, x),


para quaisquer x, y, z.

EUREKA! N21, 2005

60

Sociedade Brasileira de Matemtica

107) a) Dado um tringulo qualquer, prove que existe um crculo que passa pelos pontos mdios dos seus lados, pelos ps das suas alturas e pelos pontos mdios dos segmentos que unem o ortocentro aos vrtices do tringulo (o chamado "crculo dos nove pontos"). b) Prove que, se X o centro do crculo dos nove pontos de um tringulo, H o seu ortocentro, O seu circuncentro e G seu baricentro, ento

/ 3 / 1 / OX = OG = OH . 2 2

Problema 102 proposto por Sebastio Vieira do Nascimento (Campina Grande PB); Problema 103 proposto por Carlos A. Gomes (Natal RN); Problemas 104 e 106 propostos por Wlson Carlos da Silva Ramos (Belm PA); Problema 105 selecionado da XXXV Olimpada Matemtica Espanhola, fase nacional, 1999 e enviado por Bruno Salgueiro Fanego (Espanha). Agradecemos tambm o envio das solues e a colaborao de: Carlos Alberto da Silva Victor Daniel Lopes Alves de Medeiros Gabriel Ponce Georges Cobiniano Sousa de Melo Glauber Moreno Barbosa Jnatas de Souza Jnior Rafael Silva Raphael Constant da Costa Nilpolis RJ Fortaleza CE Por e-mail Joo Pessoa PB Rio de Janeiro RJ Recife PE Teresina PI Rio de Janeiro RJ

Seguimos aguardando o envio de solues dos problemas propostos Nos. 89, 97, 98 e 100.

EUREKA! N21, 2005

61

Sociedade Brasileira de Matemtica

COORDENADORES REGIONAIS
Alberto Hassen Raad Amrico Lpez Glvez Amarsio da Silva Arajo Ana Paula Bernardi da Silva Antonio Carlos Nogueira Ali Tahzibi Benedito Tadeu Vasconcelos Freire Carlos Frederico Borges Palmeira Claus Haetinger Cleonor Crescncio das Neves Cludio de Lima Vidal Edson Roberto Abe lio Mega der Luiz Pereira de Andrade Eudes Antonio da Costa Florncio Ferreira Guimares Filho Ivanilde Fernandes Saad Jacqueline Fabiola Rojas Arancibia Janice T. Reichert Joo Bencio de Melo Neto Joo Francisco Melo Libonati Jos Carlos dos Santos Rodrigues Jos Cloves Saraiva Jos Gaspar Ruas Filho Jos Luiz Rosas Pinho Jos Vieira Alves Krerley Oliveira Licio Hernandes Bezerra Luzinalva Miranda de Amorim Mrio Rocha Retamoso Marcelo Rufino de Oliveira Marcelo Mendes Marilane de Fraga Sant'Ana Pablo Rodrigo Ganassim Ramn Mendoza Ral Cintra de Negreiros Ribeiro Ronaldo Alves Garcia Reginaldo de Lima Pereira Reinaldo Gen Ichiro Arakaki Ricardo Amorim Srgio Cludio Ramos Seme Guevara Neto Tadeu Ferreira Gomes Toms Menndez Rodrigues Valdenberg Arajo da Silva Valdeni Soliani Franco Vnia Cristina Silva Rodrigues Wagner Pereira Lopes (UFJF) (USP) (UFV) (Universidade Catlica de Braslia) (UFU) (USP) (UFRN) (PUC-Rio) (UNIVATES) (UTAM) (UNESP) (Colgio Objetivo de Campinas) (Colgio Etapa) (UNESPAR/FECILCAM) (Univ. do Tocantins) (UFES) (UC. Dom Bosco) (UFPB) (UNOCHAPEC) (UFPI) (Grupo Educacional Ideal) (Unespar) (UFMA) (ICMC-USP) (UFSC) (UFPB) (UFAL) (UFSC) (UFBA) (UFRG) (Grupo Educacional Ideal) (Colgio Farias Brito, Pr-vestibular) FACOS (Liceu Terras do Engenho) (UFPE) (Colgio Anglo) (UFGO) (Escola Tcnica Federal de Roraima) (UNIVAP) (Centro Educacional Logos) (IM-UFRGS) (UFMG) (UEBA) (U. Federal de Rondnia) (U. Federal de Sergipe) (U. Estadual de Maring) (U. Metodista de SP) (CEFET GO) Juiz de Fora MG Ribeiro Preto SP Viosa MG Braslia DF Uberlndia MG So Carlos SP Natal RN Rio de Janeiro RJ Lajeado RS Manaus AM S.J. do Rio Preto SP Campinas SP So Paulo SP Campo Mouro PR Arraias TO Vitria ES Campo Grande MS Joo Pessoa PB Chapec SC Teresina PI Belm PA Campo Mouro PR So Luis MA So Carlos SP Florianpolis SC Campina Grande PB Macei AL Florianpolis SC Salvador BA Rio Grande RS Belm PA Fortaleza CE Osrio RS Piracicaba SP Recife PE Atibaia SP Goinia GO Boa Vista RR SJ dos Campos SP Nova Iguau RJ Porto Alegre RS Belo Horizonte MG Juazeiro BA Porto Velho RO So Cristovo SE Maring PR S.B. do Campo SP Jata GO

EUREKA! N21, 2005

62

CONTEDO

AOS LEITORES XXVI OLIMPADA BRASILEIRA DE MATEMTICA Problemas e Solues da Primeira Fase XXVI OLIMPADA BRASILEIRA DE MATEMTICA Problemas e Solues da Segunda Fase XXVI OLIMPADA BRASILEIRA DE MATEMTICA Problemas e Solues da Terceira Fase XXVI OLIMPADA BRASILEIRA DE MATEMTICA Problemas e Solues da Primeira Fase - Nvel Universitrio XXVI OLIMPADA BRASILEIRA DE MATEMTICA Problemas e Solues da Segunda Fase - Nvel Universitrio XXVI OLIMPADA BRASILEIRA DE MATEMTICA Premiados AGENDA OLMPICA COORDENADORES REGIONAIS

2 3

17 29

49

55

64

68 69

Sociedade Brasileira de Matemtica

AOS LEITORES
O Programa Nacional de Olimpadas de Matemtica tem crescido muito nos ltimos anos, contando, atualmente, com a adeso ao Programa de mais de 5.000 escolas pblicas e privadas de todo o Brasil, o que implica em uma participao na Olimpada Brasileira de Matemtica de cerca de 250.000 jovens estudantes e seus professores. Alm disso, o Programa Nacional de Olimpadas de Matemtica conta com a colaborao de professores do ensino bsico de todo o Brasil e de professores universitrios de mais de 80 instituies de ensino superior. Eles participam de todas as atividades da Olimpada Brasileira de Matemtica, em atividades de coordenao, divulgao, treinamento de alunos, aperfeioamento de professores e aplicao das distintas fases da Olimpada Brasileira de Matemtica. Em relao promoo do ensino da Matemtica em nvel regional, temos alcanado resultados extremamente positivos: atravs do apoio a 22 Olimpadas Regionais conseguimos atingir um universo de cerca de 150.000 estudantes e seus professores, os quais so desafiados resoluo de problemas que estimulam o raciocnio e a criatividade. No que se refere participao em competies internacionais, o Programa Nacional de Olimpadas tambm tem muito a comemorar. Em 2005 os resultados so excepcionais: Excelente resultado na Olimpada de Matemtica do Cone Sul (2 medalhas de Ouro, 2 medalhas de Prata); o primeiro estudante Latino-Americano premiado com Medalha de Ouro Especial (Grand First Prize) e duas outras medalhas de Ouro na Olimpada Internacional para Estudantes Universitrios (IMC), mais uma Medalha de Ouro na Olimpada Internacional de Matemtica (IMO) e novamente quatro medalhas de ouro na Olimpada Ibero-americana de Matemtica. Todos estes resultados nacionais e internacionais demonstram que, alm de influenciar positivamente o ensino da Matemtica em instituies de ensino fundamental, mdio e superior de todo o pas, conseguimos detectar jovens muito talentosos que so estimulados a seguir uma carreira cientfica, o que fundamental para o crescimento da Cincia e Tecnologia no Brasil.

Os editores

EUREKA! N22, 2005

Sociedade Brasileira de Matemtica

XXVI OLIMPADA BRASILEIRA DE MATEMTICA


Problemas e Solues da Primeira Fase PROBLEMAS NVEL 1
1. Calcule o valor de 1997 + 2004 + 2996 + 4003. A) 10000 B) 11000 C) 10900 2. Qual dos nmeros a seguir mpar? A) 7 8 B) 37 23 C) 9 36 3. Quanto 26 + 26 + 26 + 26 44? A) 0 B) 2 4. O 20% de 40 igual a A) 5 B) 8 5. Simplificando a frao A) 2004 C) 4

D) 12000

E) 13000 E) 17 61 E) 44

D) 144 : 36 D) 42

C) 10

D) 12

E) 20

2004 + 2004 , obtemos: 2004 + 2004 + 2004 113 1 2 B) C) D) 355 2004 3

E)

2 7

6. Os alunos de uma escola participaram de uma excurso, para a qual dois nibus foram contratados. Quando os nibus chegaram, 57 alunos entraram no primeiro nibus e apenas 31 no segundo. Quantos alunos devem passar do primeiro para o segundo nibus para que a mesma quantidade de alunos seja transportada nos dois nibus? A) 8 B) 13 C) 16 D) 26 E) 31 7. Uma professora tem 237 balas para dar a seus 31 alunos. Qual o nmero mnimo de balas a mais que ela precisa conseguir para que todos os alunos recebam a mesma quantidade de balas, sem sobrar nenhuma para ela? A) 11 B) 20 C) 21 D) 31 E) 41

EUREKA! N22, 2005

Sociedade Brasileira de Matemtica

8. Dezoito quadrados iguais so construdos e sombreados como mostra a figura. Qual frao da rea total sombreada?

A)

7 18

B)

4 9

C)

1 3

D)

5 9

E)

1 2

9. O preo de uma corrida de txi igual a R$2,50 ("bandeirada"), mais R$0,10 por cada 100 metros rodados. Tenho apenas R$10,00 no bolso. Logo tenho dinheiro para uma corrida de at: A) 2,5 km B) 5,0 km C) 7,5 km D) 10,0 km E) 12,5 km 10. Um arquiteto apresenta ao seu cliente cinco plantas diferentes para o projeto de ajardinamento de um terreno retangular, onde as linhas cheias representam a cerca que deve ser construda para proteger as flores. As regies claras so todas retangulares e o tipo de cerca o mesmo em todos os casos. Em qual dos projetos o custo da construo da cerca ser maior?

A)

B)

C)

D)

E)

11. 108 crianas da 5 e 6 sries vo fazer um passeio numa caverna. So formados grupos iguais com mais de 5 porm menos de 20 alunos. Com relao ao nmero de estudantes por grupo, de quantas formas diferentes eles podem ser feitos? A) 2 B) 8 C) 5 D) 4 E) 3 12. desenho ao lado mostra o mapa de um pas (imaginrio) constitudo por cinco estados. Desejase colorir esse mapa com as cores verde, azul e amarela, de modo que dois estados vizinhos no possuam a mesma cor. De quantas maneiras diferentes o mapa pode ser pintado? A) 12 B) 6 C) 10 D) 24 E) 120

EUREKA! N22, 2005

Sociedade Brasileira de Matemtica

13. Um arteso comea a trabalhar s 8h e produz 6 braceletes a cada vinte minutos; seu auxiliar comea a trabalhar uma hora depois e produz 8 braceletes do mesmo tipo a cada meia hora. O arteso pra de trabalhar s 12h mas avisa ao seu auxiliar que este dever continuar trabalhando at produzir o mesmo que ele. A que horas o auxiliar ir parar? A) 12h B) 12h30min C) 13h D) 13h30min E) 14h30min 14. algarismo das unidades do nmero 1 3 5 97 99

A) 1

B) 3

C) 5

D) 7

E) 9

15. Dois quadrados, cada um com rea 25cm2, so colocados lado a lado para formar um retngulo. Qual o permetro do retngulo? A) 30 cm B) 25 cm C) 50 cm D) 20 cm E) 15 cm 16. Se girarmos o pentgono regular, ao lado, de um ngulo de 252, em torno do seu centro, no sentido horrio, qual figura ser obtida?

A)

B)

C)

D)

E)

17. Os resultados de uma pesquisa das cores de cabelo de 1200 pessoas so mostrados no grfico abaixo.

castanho 30%

preto 24%

ruivo 16% loiro

Quantas dessas pessoas possuem o cabelo loiro? A) 60 B) 320 C) 360

D) 400

B E) 840

EUREKA! N22, 2005

Sociedade Brasileira de Matemtica

18. Um cubo pode ser construdo, a partir dos dois pedaos de papelo apresentados em uma das alternativas a seguir, bastando apenas dobrar nas linhas tracejadas e unir nas linhas contnuas. Esses dois pedaos so:

A)

B)

C)

D)

E)
19. Ao somar cinco nmeros consecutivos em sua calculadora, Esmeralda encontrou um nmero de 4 algarismos: 2 0 0 *. O ltimo algarismo no est ntido, pois o visor da calculadora est arranhado, mas ela sabe que ele no zero. Este algarismo s pode ser: A) 5 B) 4 C) 3 D) 2 E) 9 20. Sobre uma mesa esto trs caixas e trs objetos, cada um em uma caixa diferente: uma moeda, um grampo e uma borracha. Sabe-se que A caixa verde est esquerda da caixa azul; A moeda est esquerda da borracha; A caixa vermelha est direita do grampo; A borracha est direita da caixa vermelha. Em que caixa est a moeda? A) Na caixa vermelha. B) Na caixa verde. C) Na caixa azul. D) As informaes fornecidas so insuficientes para se dar uma resposta. E) As informaes fornecidas so contraditrias.

EUREKA! N22, 2005

Sociedade Brasileira de Matemtica

21. Um feirante vende batatas e, para pesar, utiliza uma balana de dois pratos, um peso de 1 kg, um peso de 3 kg e um peso de 10 kg. Considere a seguinte afirmao: Este feirante consegue pesar (com uma pesagem) n quilogramas de batatas. Quantos valores positivos de n tornam essa afirmao verdadeira, supondo que ele pode colocar pesos nos dois pratos? A) 7 B) 10 C) 12 D)13 E)14 22. O mapa ao lado mostra um conjunto residencial onde as casas, numeradas, so interligadas por 23 ruelas. O vendedor Z Ruela, que mora na casa 8, planeja passar por todas as outras casas e retornar sua, percorrendo o menor nmero possvel de ruelas. Ele deixar de caminhar por quantas ruelas? A) 15 B) 10 C) 13 D)12 11 E)
1 2 3 4

5 6 7 8

9 11 12

10

23. O arranjo a seguir, composto por 32 hexgonos, foi montado com varetas, todas com comprimento igual ao lado do hexgono. Quantas varetas, no mnimo, so necessrias para montar o arranjo?


A) 113 24. Observe a figura: B) 123 C) 122 D) 132 E) 152

Duas das figuras abaixo representam o objeto acima colocado em outras posies. II) I)

EUREKA! N22, 2005

Sociedade Brasileira de Matemtica

III)

IV)

Elas so: A) I e II

B) I e IV

C) II e IV

D) I e III

E) II e III

25. Entre 1986 e 1989, poca em que vocs ainda no tinham nascido, a moeda do pas era o cruzado (Cz$). Com a imensa inflao que tivemos, a moeda foi mudada algumas vezes: tivemos o cruzado novo, o cruzeiro, o cruzeiro real e, finalmente, o real. A converso entre o cruzado e o real : 1 real = 2.750.000.000 cruzados Imagine que a moeda no tivesse mudado e que Joo, que ganha hoje 640 reais por ms, tivesse que receber seu salrio em notas novas de 1 cruzado. Se uma pilha de 100 notas novas tem 1,5 cm de altura, o salrio em cruzados de Joo faria uma pilha de altura: A) 26,4 km B) 264 km C) 26 400 km D) 264 000 km E) 2 640 000 km

PROBLEMAS NVEL 2
1. Veja o problema No. 3 do Nvel 1 2. Se m e n so inteiros no negativos com m < n, definimos m n como a soma dos inteiros entre m e n, incluindo m e n. Por exemplo, 5 8 = 5 + 6 + 7 + 8 = 26. O valor numrico de A) 4

22 26 : 46
C) 8 D) 10 E) 12

B) 6

3. Entre 1986 e 1989, poca em que vocs ainda no tinham nascido, a moeda do pas era o cruzado (Cz$). Com a imensa inflao que tivemos, a moeda foi mudada algumas vezes: tivemos o cruzado novo, o cruzeiro, o cruzeiro real e, finalmente, o real. A converso entre o cruzado e o real :

EUREKA! N22, 2005

Sociedade Brasileira de Matemtica

1 real = 2.750.000.000 cruzados Imagine que a moeda no tivesse mudado e que Joo, que ganha hoje 640 reais por ms, tivesse que receber seu salrio em notas novas de 1 cruzado. Se uma pilha de 100 notas novas tem 1,5 cm de altura, o salrio em cruzados de Joo faria uma pilha de altura: A) 26,4km B) 264km C) 26400km D) 264000km E) 2640000km 4. Veja o problema No. 23 do Nvel 1. 5. Veja o problema No. 14 do Nvel 1. 6. Veja o problema No. 16 do Nvel 1. 7. H 1002 balas de banana e 1002 balas de ma numa caixa. Lara tira, sem olhar o sabor, duas balas da caixa. Seja p a probabilidade de as duas balas serem do mesmo sabor e seja q a probabilidade de as duas balas serem de sabores diferentes. Quanto vale a diferena entre p e q? A) 0 B)

1 2004

C)

1 2003

D)

2 2003

E)

1 1001 x2 2

8. O permetro de um retngulo 100 e a diagonal mede x. Qual a rea do retngulo? A) 625 x2 D) 250

x2 2

x2 2 x2 E) 2500 2
B) 625

C) 1250

9. Veja o problema No. 19 do Nvel 1. 10. Para quantos inteiros positivos m o nmero A) um B) dois E) mais do que quatro C) trs

2004 um inteiro positivo? m2 2


D) quatro

11. Se x + y = 8 e xy = 15, qual o valor de x2 + 6xy + y2? A) 64 B) 109 C) 120 D) 124


EUREKA! N22, 2005

E) 154

Sociedade Brasileira de Matemtica

12. Dois espelhos formam um ngulo de 30o no ponto V. Um raio de luz, vindo de uma fonte S, emitido paralelamente a um dos espelhos e refletido pelo outro espelho no ponto A, como mostra a figura. Depois de uma certa quantidade de reflexes, o raio retorna a S. Se AS e AV tm 1 metro de comprimento, a distncia percorrida pelo raio de luz, em metros,
S A

30 V

A) 2 E) 5 3

B) 2 + 3

C) 1 + 2 + 3

D)

2 1+ 3

13. Na figura, quanto vale x? A) 6 B) 12 D) 20 E) 24

C) 18

5x

3x 6x 4x

2x

14. Se 2(22x) = 4x + 64, ento x igual a: A) 2 B) 1 C) 1

D) 2

E) 3

15. Qual o maior valor da soma dos algarismos da soma dos algarismos de um nmero de trs algarismos? A) 7 B) 8 C) 9 D) 10 E) 11 16. Veja o problema No. 10 do Nvel 1. 17. Um ponto P pertence ao interior de um quadrado com 10 cm de lado. No mximo, quantos pontos da borda do quadrado podem estar a uma distncia de 6 cm do ponto P? A) 1 B) 2 C) 4 D) 6 E) 8
EUREKA! N22, 2005

10

Sociedade Brasileira de Matemtica

18. Veja o problema No. 18 do Nvel 1. 19. No tringulo PQR, a altura PF divide o lado QR em dois segmentos de medidas QF = 9 e RF = 5. Se PR = 13, qual a medida de PQ? A) 5 B) 10 C) 15 D) 20 E) 25 20. Veja o problema No. 20 do Nvel 1. 21. No desenho ao lado, o quadriltero ABCD um quadrado de lado 3 cm e os tringulos ABF e AED so ambos equilteros. Qual a rea da regio destacada? A) 2 cm2 B) 1,5 cm2 A) 3 cm2 D) 4,5 cm2 E) 2,5 cm2
A E B

F C

22. Uma folha quadrada foi cortada em 42 quadrados menores, dos quais um tem rea maior do que 1 cm2 e os demais tm rea de 1 cm2. Qual a medida do lado da folha? A) 6 cm B) 12 cm C) 21 cm D) 19 cm E) 20 cm 23. Eu planejava fazer um curral quadrado, com uma certa rea, usando uma certa quantidade de cerca de arame farpado. Descobri, porm, que tenho 10% a menos de cerca do que esperava. Por esta razo, a rea cercada ser: A) 5% menor B) 10% menor C) 19% menor D) 20% menor E) 25% menor 24. Veja o problema No. 13 do Nvel 1. 20. Esmeralda, a digitadora, tentou digitar um nmero de seis algarismos, mas os dois algarismos 1 no apareceram (a tecla devia estar com defeito). O que apareceu foi 2004. Quantos so os nmeros de seis algarismos que ela pode ter tentado digitar? A) 4 B) 8 C) 10 D) 15 E) 20

EUREKA! N22, 2005

11

Sociedade Brasileira de Matemtica

PROBLEMAS NVEL 3
1. A funo f dada pela tabela a seguir. 1 2 3 4 5 f(x) 4 1 3 5 2 Por exemplo, f(2) = 1. Quanto vale f ( f (...( f ( f (4))...)) ?
2004 vezes


C) 3

A) 1

B) 2

D) 4

E) 5

2. Seja AB um segmento de comprimento 26, e sejam C e D pontos sobre o segmento AB tais que AC = 1 e AD = 8. Sejam E e F pontos sobre uma semicircunferncia de dimetro AB, sendo EC e FD perpendiculares a AB. Quanto mede o segmento EF? A) 5 B) 5 2 C) 7 D) 7 2 E) 12

3. As alturas de um tringulo medem 12, 15 e 20. O maior ngulo interno do tringulo mede A) 72o B) 75o C) 90o D) 108o E) 120o 4. Esmeralda, a digitadora, tentou digitar um nmero de seis algarismos, mas os dois algarismos 1 no apareceram (a tecla devia estar com defeito). O que apareceu foi 2004. Quantos so os nmeros de seis algarismos que ela pode ter tentado digitar? A) 4 B) 8 C) 10 D) 15 E) 20 5. O produto dos nmeros que aparecem nas alternativas incorretas dessa questo um cubo perfeito. Assinale a alternativa correta. A) 4 B) 8 C) 18 D) 54 E) 192 6. Qual o menor inteiro positivo n para o qual qualquer subconjunto de n elementos de {1,2,3,,20} contm dois nmeros cuja diferena 8? A) 2 B) 8 C) 12 D) 13 E) 15

EUREKA! N22, 2005

12

Sociedade Brasileira de Matemtica

7. Sejam

a=
e

12 2 2 3 2 + + + 1 3 5 12 2 2 3 2 + + + 3 5 7

 + 1001 2001
2

b=

.  + 1001 2003
2

Qual o inteiro mais prximo de a b? A) 500 B) 501 C) 999

D) 1000

E) 1001

8. Uma ampulheta formada por dois cones idnticos. Inicialmente, o cone superior est cheio de areia e o cone inferior est vazio. A areia flui do cone superior para o inferior com vazo constante. O cone superior se esvazia em exatamente uma hora e meia. Quanto tempo demora at que a altura da areia no cone inferior seja metade da altura da areia no cone superior? A) 30min B) 10h C) 1h03min20s D) 1h10min12s E) 1h14min30s 9. A funo real f, definida nos inteiros, satisfaz f(n) (n + 1)f(2 n) = (n + 3)2, para todo n inteiro. Quanto vale f(0)?
A) 17 B) 0 C) 1 D) 2 E) 9

10. Com trs algarismos distintos a, b e c, possvel formar 6 nmeros de dois algarismos distintos. Quantos conjuntos {a, b, c} so tais que a soma dos 6 nmeros formados 484? A) Um B) Dois C) Trs D) Quatro E) Mais que quatro 11. Dois cubos tm faces pintadas de ocre ou magenta. O primeiro cubo tem cinco faces ocres e uma face magenta. Quando os dois cubos so lanados, a probabilidade de as faces viradas para cima dos dois cubos serem da mesma cor (sim, ocre e magenta so cores!) 1/2. Quantas faces ocres tem o segundo cubo? A) 1 B) 2 C) 3 D) 4 E) 5 12. Veja o problema No. 10 do Nvel 2. 13. Veja o problema No. 12 do Nvel 2.
EUREKA! N22, 2005

13

Sociedade Brasileira de Matemtica

14. Para n inteiro positivo, definimos n! (l-se n fatorial) o produto de todos os inteiros positivos menores ou iguais a n. Por exemplo, 6! = 1 2 3 4 5 6. Se n! = 215 36 53 72 11 13, ento n igual a A) 13 B) 14 C) 15 D) 16 E) 17 15. Constri-se o quadrado ABXY sobre o lado AB do heptgono regular ABCDEFG, , em exteriormente ao heptgono. Determine a medida do ngulo BXC

radianos. A)
7

B)

3 7

C)

14

D)

3 14

E)

3 28

16. O conjunto das razes reais da equao x + 2 x 1 + x 2 x 1 = 2 A) {1} B) {1, 2} C) [1, 2] D) ]1, 2[ E) {2} 17. No desenho ao lado, os segmentos AB e CD so perpendiculares ao segmento BC . Sabendo que o ponto M pertence ao segmento AD e que o tringulo BMC retngulo no issceles, qual a rea do tringulo ABM ? 6 7 8 9 A) 1 B) C) D) E) 5 5 5 5 18. Veja o problema No. 3 do Nvel 2 19. O dono de uma loja empilhou vrios blocos medindo 0,8m x 0,8m x 0,8m no canto da loja e encostados numa parede de vidro que d para a rua, conforme mostra a figura abaixo. Quantos blocos no mximo, uma pessoa de 1,80m de altura que est do lado de fora da loja pode enxergar? Obs. Consideramos que uma pessoa pode enxergar uma caixa se consegue ver uma pequena regio de rea positiva de sua superfcie.
D

M A 2 B 6 C 4

EUREKA! N22, 2005

14

Sociedade Brasileira de Matemtica

A) 13

B) 14

C) 15

D) 16

E) 17

20. Veja o problema No. 4 do Nvel 2. 21. Numa prova para uma sala com 30 alunos, a mdia aritmtica das 10 piores notas 3 e a mdia aritmtica das 10 melhores notas 9. O menor valor possvel e o maior valor possvel para a mdia da sala so, respectivamente: A) 6 e 7 B) 5 e 7 C) 4 e 6 D) 3 e 9 E) 4 e 8 22. Veja o problema No. 20 do Nvel 1. 23. Veja o Problema No. 25 do Nvel 2. 24. Esmeralda escreveu (corretamente!) todos os nmeros de 1 a 999, um atrs do outro: 12345678910111213 997998999. Quantas vezes aparece o agrupamento 21, nesta ordem? A) 11 B) 21 C) 31 D) 41 E) 51 25. Um feirante vende batatas e, para pesar, utiliza uma balana de dois pratos, um peso de 1 kg, um peso de 3 kg e um peso de 10 kg. Considere a seguinte afirmao: Este feirante consegue pesar (com uma pesagem) n quilogramas de batatas. Quantos valores positivos de n tornam essa afirmao verdadeira, supondo que ele pode colocar pesos nos dois pratos? A) 7 B) 10 C) 12 D)13 E)14

EUREKA! N22, 2005

15

Sociedade Brasileira de Matemtica

GABARITO
NVEL 1 (5a. e 6a. sries) 1) B 2) E 3) A 4) B 5) D 6) B 7) A 8) B 9) C 10) C 11) D 12) B 13) D 14) C 15) A 16) B 17) C 18) E 19) A 20) A 21) D 22) E 23) B 24) C 25) D

NVEL 2 (7a. e 8a. sries) 1) A 2) C 3) D 4) B 5) C 6) B 7) C 8) C 9) A 10) B 11) D 12) B 13) C 14) E 15) D 16) C 17) E 18) E 19) C 20) A 21) D 22) C 23) C 24) D 25) D

NVEL 3 (Ensino Mdio) 1) D 2) D 3) C 4) D 5) D 6) D 7) B 8) C 9) A 10) B 11) C 12) B 13) B 14) D 15) E 16) C 17) B 18) E 19) B 20) B 21) B 22) A 23) D 24) C 25) D

EUREKA! N22, 2005

16

Sociedade Brasileira de Matemtica

XXVI OLIMPADA BRASILEIRA DE MATEMTICA


Problemas e Solues da Segunda Fase
PROBLEMAS NVEL 1 PARTE A (Cada problema vale 3 pontos)
01. O nmero 100002 tem 20 zeros. Qual a soma dos algarismos do nmero que obtemos como quociente quando dividimos esse nmero por 3? 02. A soma de dois nmeros primos a e b 34 e a soma dos primos a e c 33. Quanto vale a + b + c? 03. No desenho, os quadrilteros ABCD, EFAG e IAJH so retngulos e H ponto mdio de AE. Calcule a razo entre a rea do retngulo ABCD e o tringulo AHI. A I H F B

J G

04. Dizemos que um nmero natural teimoso se, ao ser elevado a qualquer expoente inteiro positivo, termina com o mesmo algarismo. Por exemplo, 10 teimoso, pois 102 ,103 ,104 ,..., so nmeros que tambm terminam em zero. Quantos nmeros naturais teimosos de trs algarismos existem? 05. Qual o maior nmero natural menor que 100 cuja soma dos divisores positivos

mpar?
06. Na multiplicao a seguir, a, b e c so algarismos:
1 a b b 3 * * * * * * 1c c 0 1

Calcule a + b + c.

EUREKA! N22, 2005

17

Sociedade Brasileira de Matemtica

07. Esmeralda, de olhos vendados, retira cartes de uma urna contendo inicialmente

100 cartes numerados de 1 a 100, cada um com um nmero diferente. Qual o nmero mnimo de cartes que Esmeralda deve retirar para ter certeza de que o nmero do carto seja um mltiplo de 4?
08. De quantos modos podemos sombrear quatro casas do tabuleiro 4 4 abaixo de

modo que em cada linha e em cada coluna exista uma nica casa sombreada?

09. Juntando cubinhos de mesmo volume mas feitos de materiais diferentes - cada

cubo branco pesando 1 grama e cada cubo cinza pesando 2 gramas - formou-se um bloco retangular, conforme mostrado na figura abaixo. Qual a massa, em gramas, desse bloco?

10. Na populao de uma espcie rara de 1000 aves da floresta amaznica, 98%

tinham cauda de cor verde. Aps uma misteriosa epidemia que matou parte das aves com cauda verde, esta porcentagem caiu para 95%. Quantas aves foram eliminadas com a epidemia?

EUREKA! N22, 2005

18

Sociedade Brasileira de Matemtica

PROBLEMAS NVEL 1 PARTE B (Cada problema vale 10 pontos)


PROBLEMA 1:

No desenho abaixo, o tringulo ABC retngulo e os lados do polgono (regio escura) so paralelos ou coincidem com algum dos catetos do tringulo.
5 A x 10

2 B C

Calcule x de modo que a rea do polgono seja igual do tringulo.


PROBLEMA 2:

Esmeralda, a digitadora, construiu uma tabela com 100 linhas e 100 colunas, preenchendo uma casa com 1, se o nmero da linha da casa divide o nmero da coluna e com 0, caso contrrio. Assim, por exemplo, a casa da linha 2 e da coluna 4 foi preenchida com 1, porque 2 divide 4 e a casa na linha 3 e da coluna 7 foi preenchida com 0.
1 1 2 3 4 0 0 0 1 1 0 0 2 1 1 0 3 1 0 1 4 1 1 0 5 1 0 0 6 1 1 1 99 1 0 1 100 1 1 0

100

a) Qual a soma dos nmeros escritos na linha 5? b) Qual a soma dos nmeros da coluna 60?
PROBLEMA 3:

a) possvel dividir o conjunto {12, 22,,72} em dois grupos A e B de modo que a soma dos elementos de A seja igual soma dos elementos de B? Justifique. b) possvel dividir o conjunto {12, 22, 32,,92} em dois grupos C e D de modo que a soma dos elementos de C seja igual soma dos elementos de D? Justifique.

EUREKA! N22, 2005

19

Sociedade Brasileira de Matemtica

PROBLEMAS NVEL 2 PARTE A (Cada problema vale 3 pontos)


01. Veja o problema No. 6 do Nvel 1. 02. Veja o problema No. 8 do Nvel 1. 03. Qual a soma dos algarismos do nmero

2004 2002 1998 1996 + 36 ?

04. Veja o problema No. 1 da Parte B do Nvel 1. 05. Um polgono com 20 lados chamado icosgono. Unindo-se trs dos vrtices de

um icosgono regular obtemos tringulos. Quantos so tringulos retngulos?

PROBLEMAS NVEL 2 PARTE B


(Cada problema vale 10 pontos)
PROBLEMA 1:

(a) possvel dividir o conjunto {12, 22,,72} em dois grupos A e B de modo que a soma dos elementos de A seja igual soma dos elementos de B? Justifique. (b) possvel dividir o conjunto {12, 22, 32,,92} em dois grupos C e D de modo que a soma dos elementos de C seja igual soma dos elementos de D? Justifique.
PROBLEMA 2:

(a) Simplifique a expresso

(b) Certa calculadora tem duas teclas especiais: A e B. A tecla A transforma o nmero x que est no visor em 1 . A tecla B transforma o nmero x que est no visor em 1 x . Pedro tem um nmero no visor e aperta sucessivamente, de forma alternada, as duas teclas: A, B, A, B, . Aps 1000 operaes, o visor mostrava o nmero 2004. Que nmero Pedro tinha inicialmente no visor?
x

EUREKA! N22, 2005

20

Sociedade Brasileira de Matemtica

PROBLEMA 3:

Uma folha de papel retangular ABCD foi dobrada de modo que o vrtice B foi levado no ponto B sobre o lado AD. A dobra EF, com E sobre AB e F sobre CD. Sabe-se que AE = 8, BE = 17 e C F = 3. (a) Calcule a medida do segmento AB. (b) Calcule a medida do lado AD.

PROBLEMA 4:

Um nmero de 4 algarismos a b c d chamado de legal quando a soma dos nmeros formados pelos dois primeiros e pelos dois ltimos algarismos igual ao nmero formado pelos algarismos centrais (ou seja, ab + cd = bc). Por exemplo, 2307 um nmero legal pois 23 + 07 = 30. (a) Qual o menor nmero legal maior do que 2307? (b) Quantos so os nmeros legais de 4 algarismos?

PROBLEMAS NVEL 3
PROBLEMA 1:

Cada um dos nmeros x1 , x 2 , ..., x 20 04 pode ser igual a Quantos valores inteiros distintos a soma
2004

2 1 ou a

2 +1.

k =1

x 2 k 1 x 2 k = x1 x 2 + x 3 x 4 + x 5 x 6 + ... + x 2003 x 2 004 pode assumir?

PROBLEMA 2

Seja ABCD um trapzio retngulo de bases AB e CD, com ngulos retos em A e D. Dado que a diagonal menor BD perpendicular ao lado BC, determine o menor valor possvel para a razo C D .
AD

EUREKA! N22, 2005

21

Sociedade Brasileira de Matemtica

PROBLEMA 3:

Os doze alunos de uma turma de olimpada saam para jogar futebol todos os dias aps a aula de matemtica, formando dois times de 6 jogadores cada e jogando entre si. A cada dia eles formavam dois times diferentes dos times formados em dias anteriores. Ao final do ano, eles verificaram que cada 5 alunos haviam jogado juntos num mesmo time exatamente uma vez. Quantos times diferentes foram formados ao longo do ano?
PROBLEMA 4:

Determine todas as solues da equao n 2 n 1 + 1 = m 2 , com n e m naturais.


PROBLEMA 5:

Dizemos que um nmero inteiro positivo sinistro quando a soma de seus fatores primos igual soma dos expoentes de sua decomposio em fatores primos. Encontre todos os nmeros sinistros de quatro algarismos.
PROBLEMA 6:

Sejam H, I e O o ortocentro, o incentro e o circuncentro do tringulo ABC, respectivamente. A reta CI corta o circuncrculo de ABC no ponto L, distinto de C. Sabe-se que AB = IL e AH = OH. Determine os ngulos do tringulo ABC.

EUREKA! N22, 2005

22

Sociedade Brasileira de Matemtica

SOLUES SEGUNDA FASE NVEL 1 PARTE A


Problema Resposta 01 064 02 036 03 032 04 360 05 098 06 010 07 076 08 024 09 262 10 600

SOLUES SEGUNDA FASE NVEL 1 PARTE B


SOLUO DO PROBLEMA 1:

O polgono consiste na reunio de dois retngulos: um deles tem largura 10 e altura 2 e o outro tem largura 5 e altura x + 2 ; o tringulo tem catetos de medidas 15 e x + 2 . Como a rea do polgono igual rea do tringulo, temos

10 2 + 5( x + 2) =

15( x + 2) 40 + 10 x + 20 = 15 x + 30 5 x = 30 x = 6 2

SOLUO DO PROBLEMA 2:

a) Cada linha apresenta 1 nas colunas cujos nmeros so mltiplos do nmero da linha. Assim, a linha 5 tem 1 nas colunas 5, 10, 15, etc. At 100, existem 20 mltiplos de 5, logo a soma dos nmeros na linha 5 igual a 20. b) Cada coluna apresenta 1 no cruzamento com as linhas cujos nmeros so divisores do nmero da coluna. Assim, a soma dos nmeros da coluna 60 igual ao nmero de divisores de 60. Como 60 = 2 2 3 5 , conclumos que 60 tem 3.2.2 = 12 divisores. Logo, a soma dos nmeros da coluna 60 12.
SOLUO DO PROBLEMA 3:

a) A soma total dos elementos 12 + 22 + 32 + 42 + 52 + 62 + 72 = 1 + 4 + 9 + 16 + 25 + 36 + 49 = 140 . Logo, cada um dos grupos deve conter elementos que somem 70. Examinando as parcelas, vemos que 49 + 1 + 4 +16 = 70. Assim podemos escrever, por exemplo, A = {12, 22, 42, 72} e B = {32, 52, 62}. b) Como 12 + 22 + 32 + 42 + 52 + 62 + 72 + 82 + 92 = 140 + 64 + 81 = 285 mpar, impossvel dividir em dois grupos de mesma soma.

SOLUES SEGUNDA FASE NVEL 2 PARTE A


Problema Resposta 01 010 02 024 03 048 04 006 05 180

EUREKA! N22, 2005

23

Sociedade Brasileira de Matemtica

SOLUES SEGUNDA FASE NVEL 2 PARTE B


SOLUO DO PROBLEMA 1:Ver o problema 3 Parte B do Nvel 1 SOLUO DO PROBLEMA 2:

a) 1

1 1 1 1

= 1 1 x

1 1 1 x 1 x

= 1

1 x 1 x 1

= 1

1 x 1 = 1 =1+ x 1 = x x 1 x 1 x 1

2003 x 1 x 1 = 2004 = 2004 2004 x 2004 = 1 x = 2004 x 1 x 1

SOLUO DO PROBLEMA 3:

a) A partir da dobra da folha podemos ver que B'E = BE = 17, e como AE = 8, aplicando o teorema de Pitgoras temos AB= BE 2 AE 2 = 172 82 = 15 .
SOLUO DO PROBLEMA 4:

Como ab + cd = bc 10a + b + 10c + d = 10b + c 10a + d = 9(b c ) , ou seja, 10a + d o nmero de dois algarismo a d , e um mltiplo de 9. a) Mantendo a = 2, temos d = 7. Alm disso, 10 2 + 7 = 9(b c) b c = 3 . O menor valor de b que podemos escolher, aps 3, 4, e nesse caso, c = 1. O nmero procurado , ento, 2417. b) Uma vez que escolhemos b c , a e d esto determinados: a o algarismo das dezenas de 9(b c) , e d, o das unidades. Alm disso, 9(b c) 10 b c 2 . Se b c = 2 , (b, c ) {( 2,0 ); (3,1); ( 4, 2 );...; (9,0 )} , um total de 8 possibilidades. Da total de 7 possibilidades. Para b c = 4 , mesma forma, vemos que se b c = 3 , (b, c ) {(3,0 ); ( 4,1); (5, 2 );...; (9,6 )} , h um

(b, c ) {( 4,0 ); (5,1); (6, 2 );...; (9,5 )} ,

H, portanto, um total de 8 + 7 + 6 + 5 + 4 + 3 + 2 + 1 = 36 nmeros legais.


EUREKA! N22, 2005

(b, c ) {(6,0 ); ( 7,1); (8,2 ); (9,3)} , 4 possibilidades, b c = 7 , (b, c ) {(7,0 ); (8,1); (9,2 )} , 3 possibilidades, b c = 8 , (b, c ) {(8,0 ); (9,1)} , 2 possibilidades e, finalmente, para b c = 9 , (b, c ) = (9,0 ) , 1 possibilidade.

possibilidades, b c = 5 , (b, c ) {(5,0 ); (6,1); (7, 2 );...; (9, 4 )} , 5 possibilidades, b c = 6 ,

24

Sociedade Brasileira de Matemtica

SOLUES SEGUNDA FASE NVEL 3


SOLUO DO PROBLEMA 1:

Os

possveis

produtos

x2 k 1 x2 k

so

( 2 1)( 2 1) = 3 2 2,

( 2 + 1)( 2 + 1) = 3 + 2 2 e ( 2 1)( 2 + 1) = 1. Suponha que a produtos so iguais a 3 2 2, b produtos so iguais a 3 + 2 2 e 1002 a b produtos so iguais a 1. A soma igual a a(3 2 2) + b(3 + 2 2) + 1002 a b = 1002 + 2a + 2b + 2(b a) 2. Assim, para que a soma seja inteira, devemos ter a = b. Logo a soma igual a 1002 + 4a. Como a varia de 0 a 501 (pois a + b no pode ser maior que 1002), a soma pode assumir 502 valores inteiros.
SOLUO DO PROBLEMA 2:

Seja ABD = BDC = . Ento DC =

BD cos

e AD = BD sen , donde

BD 1 2 DC = cos = = 2. AD BD sen sen cos sen2


A igualdade ocorre quando sen 2 = 1, ou seja, quando = 45.

EUREKA! N22, 2005

25

Sociedade Brasileira de Matemtica

SEGUNDA SOLUO DO PROBLEMA 2:

Sejam H a projeo de B sobre DC, DH = m, HC = n e BH = h. ABDH ento um retngulo, donde AD = BH = h.

DC DC m + n m + n = = = . AD BH h mn 2 Mas sabemos que ( m n ) 0, donde m + n 2 mn . A igualdade ocorre


quando m = n. Segue que

2 Como o tringulo CBD retngulo, temos h = mn. Logo,

DC m + n = 2. AD mn
SOLUO DO PROBLEMA 3:

Para cada grupo de 5 alunos, existe um nico time formado que os contm. Logo, 12 12 11 10 9 8 = 792 times para cada 5 alunos escolhidos. Por contamos = 5 5!

6 outro lado, em cada time de 6 jogadores, temos = 6 modos de escolhermos 5 cinco jogadores, ou seja, existem 6 grupos de 5 jogadores que geram o mesmo time
na nossa primeira contagem. Logo, o total de times formados igual a
SEGUNDA SOLUO DO PROBLEMA 3:

792 = 132 . 6

12 H 6 maneiras de escolher 6 dentre 12 alunos. Alm disso, fixados 5 alunos, h 7 maneiras de montar um time com esses 5 alunos mais outro aluno. Assim, considerando que cada 5 alunos jogaram juntos num mesmo time exatamente uma vez, o total de maneiras de escolher 6 dentre 12 alunos igual a 7 vezes o nmero de
EUREKA! N22, 2005

26

Sociedade Brasileira de Matemtica

maneiras de formar os times ao longo do ano. Logo o nmero de maneiras de formar 1 12 12 11 10 9 8 7 os times ao longo do ano = 132 . = 7 6 7 6 5 4 3 2 1
SOLUO DO PROBLEMA 4:

A equao equivalente a n 2n 1 = ( m 1)( m + 1). Suponha n > 3. Temos m mpar, digamos m = 2k +1. A equao fica ento n 2n 3 = k ( k + 1). Portanto, 2n 3 divide k ou k + 1, pois k ou k + 1 (o que for mpar) divide n. Assim, k + 1 2n 3 e k n, donde n + 1 2 n 3 . Mostremos, por induo, que n + 1 < 2 n 3 para n > 5. Para n = 6 (base de induo), temos 6 + 1 = 7 e 26 3 = 8. Supondo que a desigualdade vlida para n = k, provemos que a mesma vlida para n = k + 1 (passo indutivo). De fato, temos k + 1 < 2k 3 2(k + 1) < 2k 2. Como k + 2 < 2(k + 1), temos k + 2 < 2k 2, completando a demonstrao. Assim, basta testar 0 n 5 . Portanto as solues so (m; n) = (1;0) e (m; n) = (9;5).
SOLUO DO PROBLEMA 5:
1 2 k Seja n = p1 , (com p1 < p2 < ... < pk primos) um nmero sinistro. Como p2 ... pk pi 2 para todo i, n 21 +2 +...+k .

Como n tem 4 algarismos, n < 10000, donde 21 +2 +...+k n < 10000 , e logo 1 + 2 + ... + k 13. Se um dos fatores primos fosse maior ou igual a 11, a soma dos fatores primos seria 11 , donde 1 + 2 + ... + k 11 e n 210 11 > 10000 , absurdo. Assim, os nicos fatores primos possveis so 2, 3, 5 e 7. Como 35 < 1000, se a soma dos expoentes for 5, o nmero deve ser 55 = 3125 . A soma pode ser tambm igual a 7, donde o nmero pode ser 24 53 = 2000, ou 23 54 = 5000 (note que 77 > 10000 ). No pode ser igual a 8 = 3 + 5 , pois 37 5 > 10000. Pode ser igual a 9 = 7 + 2, podendo o nmero ser igual a 28 7 = 1792 ou 27 72 = 6272.
EUREKA! N22, 2005

27

Sociedade Brasileira de Matemtica

Pode ser igual a 10 = 2 + 3 + 5, podendo o nmero ser igual a 28 3 5 = 3840, 27 32 5 = 5760, 27 3 52 = 9600 ou 26 33 5 = 8640 (note que os fatores primos no podem ser 3 e 7, pois 39 7 > 10000). A soma no pode ser 11, nem 12 (pois 210 3 7 e 511 7 so maiores que 10000) nem 13. Assim os nmeros sinistros de quatro algarismos so 55 = 3125 , 24 53 = 2000, 23 54 = 5000, 28 7 = 1792, 27 72 = 6272, 28 3 5 = 3840, 27 32 5 = 5760, 27 3 52 = 9600 e 26 33 5 = 8640.
SOLUO DO PROBLEMA 6:

Sendo , e as medidas dos ngulos internos nos vrtices A, B e C, respectivamente, temos m(IBL) = m(BIL) =

= L e IL = AB, conclumos que o tringulo ABL equiltero, logo o arco AB mede 60o e, portanto, m(ACB) = 120o. O quadriltero CXHY inscritvel, onde X e Y so os ps das alturas traadas de A e B. Logo AHB mede 180o 120o = 60o. Como m(AOB) = 120o, conclumos que o quadriltero OAHB inscritvel. (Isto tambm pode ser provado, por exemplo, utilizando-se a propriedade de que o simtrico de H em relao a AB pertence ao circuncrculo de ABC). Isto implica que m(AHO) = m(ABO) = 30o, e como OH = AH, temos m(AOH) = m(OAH) = 75o. Finalmente, temos m(BAC) = m(OAH) m(OAB) m(XAH) = 75o 30o 30o = 15o; e m(ABC) = 180o 120o 15o = 45o.
L

+ . Logo BL = IL, e como BL 2

A C X Y H

EUREKA! N22, 2005

28

Sociedade Brasileira de Matemtica

XXVI OLIMPADA BRASILEIRA DE MATEMTICA


Problemas e Solues da Terceira Fase
PROBLEMAS NVEL 1
PROBLEMA 1:

Encontre todos os nmeros naturais n de trs algarismos que possuem todas as propriedades abaixo: n mpar; n um quadrado perfeito; A soma dos quadrados dos algarismos de n um quadrado perfeito.
PROBLEMA 2:

Com quatro tringulos eqilteros de lado 1 possvel formar uma pea, no formato de um tringulo eqiltero de lado 2, como mostra a figura ao lado. Imagine que voc tenha muitos tringulos eqilteros de lado 1 de trs tipos: brancos, pretos e cinzas para formar peas como no exemplo acima. Duas peas assim formadas so consideradas iguais quando podemos obter uma delas girando a outra, conforme ilustrado abaixo, esquerda.

Par de peas iguais Par de peas diferentes Quantas peas diferentes podem ser formadas nas condies apresentadas?
PROBLEMA 3:

Dizemos que um nmero natural composto quando pode ser escrito como produto de dois nmeros naturais maiores que 1. Assim, por exemplo, 91 composto porque podemos escrever 91 = 7 13. Mostre que o nmero

EUREKA! N22, 2005

29

Sociedade Brasileira de Matemtica

2004 + 2 2 +1 2

composto.
PROBLEMA 4:

Arnaldo e Bernaldo disputam um jogo num tabuleiro 2 n:

As peas do jogo so domins 2 1. Inicialmente Arnaldo coloca um domin cobrindo exatamente duas casas do tabuleiro, na horizontal ou na vertical. Os jogadores se revezam colocando uma pea no tabuleiro, na horizontal ou na vertical, sempre cobrindo exatamente duas casas do tabuleiro. No permitido colocar uma pea sobre outra j colocada anteriormente. Quem no conseguir colocar uma pea no tabuleiro perde.

Qual dos dois jogadores tem uma estratgia vencedora, ou seja, uma estratgia que o leva vitria quaisquer que sejam as jogadas de seu adversrio, para: (a) n = 2004? (b) n = 2005?
PROBLEMA 5:

Considere o polgono P de 6 lados.

EUREKA! N22, 2005

30

Sociedade Brasileira de Matemtica

Com cpias de P, podemos cobrir todo o plano, sem sobreposies, como mostrado a seguir.

Existe um polgono de 13 lados com o qual possvel cobrir todo o plano com suas cpias, sem sobreposies? Caso seja possvel, apresente um polgono. Caso no seja, diga o porqu. PROBLEMAS NVEL 2
PROBLEMA 1:

Na figura, ABC e DAE so tringulos issceles (AB = AC = AD = DE) e os ngulos BAC e ADE medem 36.

a) Utilizando propriedades geomtricas, calcule a medida do ngulo b) Sabendo que BC = 2, calcule a medida do segmento DC. c) Calcule a medida do segmento AC.
EUREKA! N22, 2005

C . ED

31

Sociedade Brasileira de Matemtica

PROBLEMA 2:

A seqncia de algarismos 1, 2, 3, 4, 0, 9, 6, 9, 4, 8, 7, construda da seguinte maneira: cada elemento, a partir do quinto, igual ao ltimo algarismo da soma dos quatro anteriores. a) Os algarismos 2, 0, 0, 4, juntos e nesta ordem, aparecem na seqncia? b) Os algarismos iniciais 1, 2, 3, 4, juntos e nesta ordem, aparecem novamente na seqncia?
PROBLEMA 3:

Esmeralda tem uma pilha com 100 pedras. Ela divide essa pilha em duas novas pilhas e em seguida multiplica as quantidades de pedras nessas duas novas pilhas e escreve o produto em um quadro. Ela ento escolhe uma pilha com mais de uma pedra e repete esse procedimento: a pilha dividida em duas, as quantidades de pedras nessas duas pilhas so multiplicadas e o produto escrito no quadro. Esta operao realizada at se obter apenas pilhas com 1 pedra cada. Quais so os possveis valores da soma de todos os produtos escritos no quadro?
PROBLEMA 4:

Em um jogo para dois participantes, Arnaldo e Bernaldo alternadamente escolhem um nmero inteiro positivo. A cada jogada, deve-se escolher um nmero maior que o ltimo nmero escolhido e menor que o dobro do ltimo nmero escolhido. Nesse jogo, vence o jogador que conseguir escolher o nmero 2004. Arnaldo joga primeiro e inicia com o nmero 2. Qual dos dois tem estratgia vencedora, ou seja, consegue escolher o nmero 2004 independentemente das jogadas do adversrio?
PROBLEMA 5:

Seja D o ponto mdio da hipotenusa AB de um tringulo retngulo ABC. Sejam O1 e O2 os circuncentros dos tringulos ADC e DBC, respectivamente. a) Mostre que O1 DO 2 reto. b) Mostre que AB tangente ao crculo de dimetro O1O2 . Considere todas as maneiras de colocarmos nas casas de um tabuleiro 10 10 exatamente dez vezes cada um dos algarismos 0, 1, 2, , 9. Encontre o maior inteiro n com a propriedade de que, em cada tabuleiro, alguma linha ou alguma coluna contenha pelo menos n algarismos diferentes.
PROBLEMA 6:

EUREKA! N22, 2005

32

Sociedade Brasileira de Matemtica

PROBLEMAS NVEL 3
PROBLEMA 1:

Seja ABCD um quadriltero convexo. Prove que os incrculos de ABC, BCD, CDA e DAB tm um ponto em comum se, e somente se, ABCD um losango.
PROBLEMA 2:

Determine todos os valores de n tais que possvel dividir um tringulo em n tringulos de modo que no haja trs vrtices alinhados e em cada vrtice incida o mesmo nmero de segmentos. Mostramos a seguir tal diviso para n = 7. Observe que em cada um dos seis vrtices incidem quatro segmentos.

PROBLEMA 3:

Seja x1 , x 2 , , x 2004 uma seqncia x k +3 = x k + 2 + x k +1 x k , 1 k 2001 .

de

nmeros

inteiros

satisfazendo

possvel que mais da metade de seus termos sejam negativos? Considere todas as maneiras de colocarmos nas casas de um tabuleiro 10 10 exatamente dez vezes cada um dos algarismos 0, 1, 2, , 9. Encontre o maior inteiro n com a propriedade de que, em cada tabuleiro, alguma linha ou alguma coluna contenha pelo menos n algarismos diferentes.
PROBLEMA 4:

EUREKA! N22, 2005

33

Sociedade Brasileira de Matemtica

PROBLEMA 5:
2 Considere a seqncia (a n ) nN com a0 = a1 = a2 = a3 = 1 e an a n 4 = a n 1a n3 + an 2 .

Mostre que todos os termos dessa seqncia so nmeros inteiros.


PROBLEMA 6:

Sejam a e b nmeros reais. Considere a funo f a ,b : R2 R2 definida por f a ,b ( x; y ) = (a by x 2 ; x) . Sendo


+1 (P) f ak,b

P = ( x; y ) R 2 , definimos

f a0,b ( P ) = P

= f a ,b (

f ak,b ( P)) ,

para k inteiro no negativo.

O conjunto per(a; b) dos pontos peridicos da funo f a ,b o conjuntos dos pontos P de R2 para os quais existe um inteiro positivo n tal que f an,b ( P) = P . Fixado o real b, prove que o conjunto Ab = {a R | per (a, b) 0 / } tem um menor elemento. Calcule esse menor elemento. SOLUES NVEL 1
PROBLEMA 1: SOLUO DE DIANA VAISMAN (RIO DE JANEIRO - RJ) Nmeros mpares que so quadrados perfeitos Soma dos quadrados dos algarismos

121 169 225 289 361 441 529 625 729 841 961 Resposta: 841

1+4+1=6 1 + 36 + 81 = 118 4 + 4 + 25 = 33 4 + 64 +81 = 149 9 + 36 + 1 = 46 16 + 16 + 1 = 33 25 + 4 + 81 = 110 36 + 4 + 25 = 65 49 + 4 + 81 = 134 64 + 16 + 1 = 81 81 + 36 + 1 = 118

EUREKA! N22, 2005

34

Sociedade Brasileira de Matemtica

PROBLEMA 2: SOLUO DE HUGO FONSECA ARAJO (JUIZ DE FORA - MG):

Posso pintar de 33 modos. Com 1 cor o desenho este:


z z z z

sendo z uma das trs cores. Com 2 cores tenho estes desenhos:
x x x z x x z z x x z x

Sendo x uma das trs cores e z tambm. Com 3 cores tenho estes desenhos
z y x y y z y x

Sendo x uma das trs cores e y e z tambm. Com 1 cor tenho 3 1 possibilidades. Com 2 cores tenho 3 2 3 = 18 possibilidades. Com 3 cores tenho 3 2 1 2 = 12 possibilidades. No total tenho 12 + 18 + 3 = 33 possibilidades.

PROBLEMA 3: SOLUO DE ILLAN FEIMAN HALPERN (ITATIAIA - RJ):

O nmero 2(2 + 2) + 1 equivalente a 4(2 +1) + 1. Como toda potncia de 2 par ento 22003 + 1 ser mpar. Como 4 elevado a um nmero mpar d um nmero cujo ltimo algarismo 4, ento 2003 4(2 +1) + 1 ter 5 como ltimo algarismo. Como todo nmero que termina com 5 mltiplo de 5 ento o nmero 2004 2(2 + 2) + 1 ser mltiplo de 5 e poder ser escrito como 5x , com x maior do que 1, o 2004 que prova que 2(2 + 2) + 1 composto.
2004 2003

EUREKA! N22, 2005

35

Sociedade Brasileira de Matemtica

PROBLEMA 4: SOLUO DE JAMES JUN HONG (SO PAULO SP):

Para n mpar, Arnaldo tem a estratgia vencedora. Para n par, Bernaldo tem a estratgia vencedora. Quando o nmero n for mpar, basta Arnaldo comear com um domin na vertical. Sejam quais forem as jogadas de Bernaldo, Arnaldo vencer. Se Bernaldo puser um domin na vertical, Arnaldo dever por na vertical tambm, em qualquer lugar do tabuleiro. Se Bernaldo puser na horizontal, Arnaldo dever por um exatamente acima ou abaixo da pea de Bernaldo. Seguindo as regras, Arnaldo vencer porque no sobrar nenhum espao e o nmero de jogadas ser mpar. Como Arnaldo comea, ele tambm termina e Bernaldo no poder colocar uma pea no tabuleiro cheio. Se o nmero n for par, Bernaldo vence. As regras para preenchimento so as mesmas j citadas: se Arnaldo puser um domin na vertical, Bernaldo dever fazer o mesmo. Se Arnaldo puser na horizontal, Bernaldo dever colocar na horizontal, acima ou abaixo da pea de Arnaldo. Como o nmero de jogadas, seguindo as regras, ser par, Bernaldo terminar de preencher o tabuleiro porque Arnaldo comea. Conseqentemente, Arnaldo no poder colocar nenhuma pea e perder. Lembramos que sendo o tabuleiro 2 n, n o nmero mximo de jogadas pois o domin ocupa 2 casas do tabuleiro. a) sendo 2004 par, Bernaldo vence. b) sendo 2005 mpar, Arnaldo vence.
PROBLEMA 5: SOLUES DE WALLACE J. INOCNCIO e CAROLINE RIGOLON VEIGA (RIO DE JANEIRO - RJ)

EUREKA! N22, 2005

36

Sociedade Brasileira de Matemtica

SOLUES NVEL 2
PROBLEMA 1: SOLUO DE VALTER BARBALHO LIMA FILHO :

D 36 2 2 36 2 C 72 72 2 72 E B

A
a)

F 72 36 36 72

b)

180 36 = 72 DAF = 36 2 180 36 II) AD = AC ADC = ACD = = 72 EDC = 36 2 I) ADE ACB( LAL) AE = 2. II) AFE = 72 AE = AF = 2 I) AD = DE DEA = DAE = III) ADF = F AD = 36 AF = FD = 2 IV) DFC = DCF = 72 DC = FD = 2

c)

I) Seja AC = AB = x. AE FE 2 FE 4 = = x FE = 4 FE = AB CB x 2 x 4 x2 4 II) DE = x DF = DE FE DF = x DF = x x 2 x 4 III) DF = AF = 2 = 2 x2 2 x 4 = 0 x = 1 + 5 x IV) AC = DE = x = 1 + 5. Temos AFE ~ ACD =

PROBLEMA 2: SOLUO DE RGIS PRADO BARBOSA (FORTALEZA - CE):

a) note que se um natural n par ou mpar, n 10k ter mesma paridade pois n par =2t 2t 10k = 2(t 5k ) par n mpar =2t + 1 2t + 1 10k = 2(t 5k ) + 1 par
EUREKA! N22, 2005

37

Sociedade Brasileira de Matemtica

logo se a soma de quatro nmeros da seqncia tiver certa paridade o quinto nmero ter a mesma. Seja agora: i = No. mpar p = No. par 1, 2, 3, 4, 0, 9, 6, 9, 4, 8, 7, 8, 7, 0, 2 i, p, i, p, p, i, p, i, p, p, i, p, i, p, p Note que as paridades se repetem de 5 em 5 nmeros. Provemos que de fato ela sempre assim. Por induo: 1, 2, 3, 4, 0, 9, 6, 9, 4, 8 Caso inicial: i, p, i, p, p , i, p, i, p, p ...

 
a1 a2

Hiptese: suponha que d certo at o ak simo perodo. Passo indutivo: ...i, p, i, p, p a, b, c, d, e


ak

a p + i + p + p i(mod10), logo a mpar. b i + p + p + a i + p + p + i p(mod10), logo b par. c p + p + a + b p + p + i + p i( mod10), logo c mpar. d p + a + b + c p + i + p + i p(mod10), logo d par. e a + b + c + d i + p + i + p p(mod10), logo e par. Assim, a, b, c, d, e = i, p, i, p, p , e logo nunca poderemos chegar a 2, 0, 0, 4, pois no


ak +1

poderemos ter 4 p's seguidos. b) Veja que, para esta seqncia, a seqncia dos grupos de 4 termos consecutivos dela no poder ter infinitos termos diferentes, pois no temos infinitas possibilidades para a, b, c, d: sero no mximo a , b , c , d = 104 possibilidades (aqui so contadas possibilidades que, assim
10 10 10 10

como vimos no item a), no podem aparecer, mas o que queremos com isso no achar um nmero exato mas sim um mximo e mostrar que as possibilidades so finitas). Assim, num certo ponto comearo a se repetir os nmeros formando um perodo, assim:

EUREKA! N22, 2005

38

Sociedade Brasileira de Matemtica

y 1, 2, 3, 4 x

a, b, c, d

Mas note que se: y o ltimo nmero antes de comear a repetio, x o ltimo nmero do perodo e a, b, c, d so os 4 primeiros nmeros do perodo, teremos y + a + b + c d (mod 10) e x + a + b + c d (mod 10) logo y d a b c x (mod 10), e x y (mod 10), 0 x 9 e 0 y 9 x = y. Se fizermos isso vrias vezes veremos que na verdade o perodo :
1, 2, 3, 4

Logo ele aparecer novamente.


PROBLEMA 3: SOLUO DE RGIS PRADO BARBOSA (FORTALEZA - CE):

Note que se temos uma pilha com a pedras e fazemos o processo dividindo-a em duas pilhas com b e e pedras o nmero escrito ser b e, mas se a foi dividido em b e, temos: b + e = a (b + e)2 = a2 b2 + 2be + e2 = a2 2be = a 2 b2 e2 be = Assim note que a soma ser: b1 b2 + b3 b4 ... + bi bi +1 = S ( bi = No. de pedras na pilha i)
2 2 2 b2 b32 b4 b 2 b2 bi +1 1002 b12 b2 ... + i 1 i + 1 =S 2 2 2

a2 b2 e2 . 2

EUREKA! N22, 2005

39

Sociedade Brasileira de Matemtica

veja s que se bt > 1 ele ainda ser dividido em mais pilhas, ou seja quando ele aparecer ser:
2 bt2 bz2 bw . Assim, os 2 2 bt2 tero soma 0, a no ser quando bt = 1 e, como a pilha inicial tem 100 pedras, no fim so 100 pilhas com 1 pedra cada, e teremos: 1002 12 12 ... 12 10000 100 12 9900 100 vezes . = = S= 2 2 2 9900 S = 4950 e esta a nica possibilidade. S= 2
2 2 bt2 by bx

mas se bt > 1 aparecer tambm

PROBLEMA 4: SOLUO DE HENRIQUE POND DE OLIVEIRA PINTO (SALVADOR - BA):

Seja uma Rodada definida como a jogada de cada jogador assim na 1 Rodada Arnaldo escolheu o nmero 2 e na 2 Rodada Bernaldo ser obrigado a escolher um nmero maior que 2 e menor que 2 2 = 4 logo na 2 Rodada ser escolhido o nmero 3. Sejam os dois jogadores do enunciado os jogadores X e Y. 1. Suponha que o jogador X escolhe 2004 e ganha na Rodada n. 2. Para isso Y teria que ter escolhido qualquer nmero entre 1003 e 2003 na Rodada n 1 e isso visto facilmente. 3. A jogada de X na Rodada n 2 teria que ser 1002, pois se fosse maior ou menor que 1002 Y no seria obrigado a escolher um nmero entre 1003 e 2003 na Rodada n 1. 4. Para X ter escolhido 1002 na Rodada n 2 Y teria que ter escolhido um nmero entre 502 e 1001 na Rodada n 3. 5. A jogada de X na Rodada n 4 teria que ser 501, pois se fosse maior ou menor que 501, Y no seria obrigado a escolher um nmero entre 502 e 1001 na Rodada n 3. 6. Para X ter escolhido 501 na Rodada n 4, Y teria que ter escolhido um nmero entre 251 e 500 na Rodada n 5. 7. A jogada de X na Rodada n 6 teria que ser 250, pois se fosse maior ou menor que 250, Y no seria obrigado a escolher um nmero entre 251 e 500 na Rodada n 5. Agora que o raciocnio da questo j foi monstrado podemos continuar sem escrever tanto. Rodada n 6: (X): 250 (Como 7. mostrou) Rodada n 7: (Y): Entre 126 e 249 Rodada n 8: (X): 125
EUREKA! N22, 2005

40

Sociedade Brasileira de Matemtica

Rodada n 9: (Y): Entre 63 e 124 Rodada n 10: (X): 62 Rodada n 11: (Y): Entre 32 e 61 Rodada n 12: (X): 31 Rodada n 13: (Y): Entre 16 e 30 Rodada n 14: (X): 15 Rodada n 15: (Y): Entre 8 e 14 Rodada n 16: (X): 7 Rodada n 17: (Y): Entre 4 e 6 Rodada n 18: (X): 3 Rodada n 19: (Y): 2 Logo X = Bernaldo e Y = Arnaldo, e como X ganha, Bernaldo ganha. Obs: Quando falo nmeros entre r e s, r e s esto includos neste intervalo. PS: Observe as jogadas de Bernaldo: 3, 7, 15, 31, 62, 125, 250, 501, 1002, 2004. Se ele escolhe x na Rodada a escolhe 2x ou 2x + 1 na Rodada a + 2. Isso nos leva a uma observao que se Bernaldo escolhe x na Rodada a, Arnaldo pode escolher entre x + 1 e 2x 1 na Rodada a + 1, o que implica que Bernaldo pode escolher 2x ou 2x + 1 na Rodada a + 2, SEMPRE. Isso nos leva a pensar no sistema binrio de numerao. Ou seja, se na Rodada a o Jogador escolheu o nmero AnAn 1...A0, na Rodada a + 2 pode escolher os nmeros AnAn1.....A00 ou AnAn1...A01. Logo ele pode ir adicionando algarismos direita da representao binria do nmero. Veja que os dois algarismos esquerda de um nmero no sistema binrio so 10 ou 11 e que o 1 nmero do 1 jogador 2 = (10)2 e o do 2 3 = (11)2. Logo se K o nmero a ser alcanado, se K comear com 10 no sistema binrio o 1 ganha e se K comear com 11, no sistema binrio o 2 ganha. Como a representao binria de 2004 11111010100 que comea com 11 temos que o 2 ganha.
PROBLEMA 5: SOLUO DE LCIO EIJI ASSAOKA HOSSAKA (CURITIBA - PR):

a) Primeiro, necessrio dizer que ABC pode ser inscrito em uma circunferncia de raio AD, onde AB o dimetro, pois ACB e reto e deve estar compreendendo um arco AB = 180 , que uma semi-circunferncia. Logo, por D ser centro dessa circunferncia, AD = BD = CD, e os tringulos ACD e BCD so issceles, tendo AC e BC como base, respectivamente. Logo, a mediatriz de AC contm O1 assim como D (pois D est mesma distncia de A e C, assim como todos os pontos da mediatriz). Da mesma maneira, a mediatriz
EUREKA! N22, 2005

41

Sociedade Brasileira de Matemtica

de CB contm O2 e D (lembrando que o circuncentro o encontro das mediatrizes dos lados do tringulo). A mediatriz de AC perpendicular a AC, e a mediatriz de CB perpendicular a CB. Logo, ambas so perpendiculares entre si, pois AC e BC tambm o so. Como ambas contm D e so retas, elas se interceptam em D, e alm disso cada uma contm um dos circuncentros, o que mostra que O1 DO2 reto. b) Seja E o ponto mdio de AD, e F o ponto mdio de BD. A mediatriz de AD contm E e O1 , e portanto O1 ED um ngulo reto. Analogamente, DFO2 reto tambm. Como E, D e F so colineares (AB contm os trs), ento EO1 e FO2 so segmentos paralelos entre si. Seja G o ponto mdio de O1O2 . Como O1O2 D um tringulo retngulo G o centro do crculo que deveremos provar que tangente a AB. Perceba que G o ponto mdio de O1O2 assim como D o ponto mdio de EF ( ED O1G . DG bvio que ED = DF, pois D o ponto mdio de AB), e que portanto = DF GO2 , portanto, paralelo a EO1 e FO2 , pelo teorema de Tales. Assim, DG perpendicular a AB, e como DG raio do crculo de dimetro O1O2 , AB tangencia esse crculo.
PROBLEMA 6: SOLUO DE LCIO EIJI ASSAOKA HOSSAKA (CURITIBA - PR):

Se so 10 linhas e 10 colunas, ento h espao para 20 "presenas" de alguns dos 10 algarismos. "Presena" significa que cada vez que um nmero aparece em alguma linha ou coluna, significa uma "presena". No caso das linhas, por exemplo. Se um algarismo estiver distribudo dez vezes em duas linhas, so duas "presenas". Assim como se algarismo aparecer apenas 3 vezes em 3 linhas, so 3 "presenas". n maior que 3. Como j disse, cada algarismo ocupa pelo menos 3 linhas ou 3 colunas. Se um algarismo ocupar, por exemplo, 3 linhas, ocupar 4 colunas, pois se fossem 3 linhas e 3 colunas, apareceria, no mximo, 9 vezes. E vice-versa. O melhor jeito de distribuir os algarismos fazer com que cada um tenha o mnimo de presenas. A soma das presenas de cada algarismo no mnimo 7 (3 linhas e 4 colunas, e vice-versa, e 2 linhas e 5 colunas, e vice-versa), sendo 7 10 = 70 presenas no total. Se so 60 disponveis (10 linhas e 10 colunas, cada uma podendo comportar 3 algarismos distintos), impossvel criar um tabuleiro com no mximo 3 algarismos diferentes em cada coluna e cada linha. portanto, n maior que 3. Assim, n 4, pois possvel distribuir os nmeros de forma e que cada linha e cada coluna tenha at 4 algarismos diferentes:
EUREKA! N22, 2005

42

Sociedade Brasileira de Matemtica

Assim, existe um arranjo que limita o valor de n, e esse valor 4. 9 9 6 6 6 6 1 1 1 1 0 0 6 6 6 6 2 1 1 1 9 9 5 5 5 5 2 2 1 1 0 0 5 5 5 5 2 2 2 1 9 9 5 5 6 6 2 2 2 2 0 0 7 7 8 8 3 3 3 3 9 9 7 7 7 7 3 3 3 4 0 0 7 7 7 7 3 3 4 4 9 9 8 8 8 8 3 4 4 4 0 0 8 8 8 8 4 4 4 4

SOLUES NVEL 3
PROBLEMA 1: SOLUO DE EDSON AUGUSTO BEZERRA LOPES (FORTALEZA - CE):

Parte 1) A O B

Se ABCD um losango, ento AB = BC = CD = DA, ou seja, os tringulos ABC, BCD, CDA, DAB so todos issceles. Veja tambm que AC BD. Seja AC BD = O . Como em um tringulo issceles a altura relativa base tambm bissetriz. AO, OB, OC, OD possam pelos incentros de DAB, ABC, BCD e CDA respectivamente. Como ao traarmos uma perpendicular ao lado pelo incentro obtemos o ponto de toque do incrculo a esse lado, O est contido nos 4 incirculos, j que o ponto de toque do incrculo na base dos 4 tringulos.

EUREKA! N22, 2005

43

Sociedade Brasileira de Matemtica

Parte 2)

A O

Suponhamos agora que os 4 crculos tm um ponto em comum. Temos ento que os incrculos dos tringulos ABC e ACD tocam AC no mesmo ponto, pois do contrrio no teriam nenhum ponto em comum. Seja O esse ponto. Veja que se O estiver no interior do tringulo ABC o incrculo do tringulo BDC no o conter, e seguindo o mesmo raciocnio vemos que O no est no interior do tringulo BDC. Logo O est sobre BD AC BD = O. Sejam O1 , O2 , O3 e O4 Os incentros dos tringulos DAB, ABC, BCD e CDA, respectivamente. Suponhamos que AC e BD no so perpendiculares. Suponhamos agora, sem perda de generalidade, que AOB e DOC so obtusos e BOC e AOD so agudos. Claramente O2 est no interior do tringulo AOB, pois j que O2O AC , temos O2 OC > BOC. Com o mesmo raciocnio encontramos que O1 e O2 esto no interior do tringulo AOB e que O3 e O4 esto no interior do tringulo COD. Seja G1 = AO1 BO2. Claramente G1 AB = DAB ABC DAB + ABC e G1 BA = = 180 AG1B 2 2 2 = 180 (OAG1 + OBG1 + AOB) < 180 AOB DAB + ABC < 2(180 AOB) < 180,

pois AOB > 90 . De modo anlogo temos BCD + CDA < 180 DAB + ABC + BCD + CDA < 360 360 < 360 . Absurdo! Assim, AC BD, donde O1 AO, O2 BO, O3 CO, O4 DO AO, BO, CO e DO so, alm de alturas, bissetrizes ABC , BCD, CDA, DAB so issceles AB = BC = CD = DA ABCD um losango.

EUREKA! N22, 2005

44

Sociedade Brasileira de Matemtica

PROBLEMA 2: SOLUO DE LARISSA CAVALCANTE QUEIROZ DE LIMA (FORTALEZA - CE):

Seja N tal que o tringulo est dividido em N tringulos de modo que no h 3 vrtices colineares e em cada vrtice incida exatamente k segmentos. Seja M o total de vrtices (incluindo os vrtices do tringulo dividido ABC). Contando a soma dos ngulos de todos os tringulos temos: 180 N = 360(M 3) + 180

pois h N 's

soma dos ngulos de cada vrtice do interior de ABC

soma dos ngulos A + B +C

N = 2( M 3) + 1 = 2M 6 + 1 = 2M 5 N +5 M = 2 Contando o total de segmentos: M k N 3 + 3 = h N tringulos e cada segmento lado de dois tringulos, exceto os trs segmentos: AB, BC, CA. 2 2
h m vrtices de grau k cada

M k = 3N + 3 k = k =

3N + 3 3N + 3 6N + 6 = = M ( N + 5) / 2 N + 5

6( N + 5) 24 24 . =6 N +5 N +5 N +5 como k inteiro, N + 5 deve ser um divisor de 24. Assim, N + 5 = 6,8,12 ou 24 (pois N inteiro positivo) N = 1,3,7 ou 19. De fato, para cada um desses valores, h uma diviso do tringulo:
N = 1:
N=3:

N = 7:

N = 19:

EUREKA! N22, 2005

45

Sociedade Brasileira de Matemtica

Observao: as duas contagens feitas pela Larissa, se generalizadas, levam a uma

demonstrao do Teorema de Euler: sendo V, A, e F o nmero de vrtices, arestas e faces, respectivamente, de um poliedro (ou se voc quiser, um grafo plano), ento V A + F = 2. No problema 2, F = n + 1, V = M e A o nmero de segmentos. Em contrapartida, tambm possvel resolver este problema usando o Teorema de Euler.
PROBLEMA 3: SOLUO DE FBIO DIAS MOREIRA (RIO DE JANEIRO - RJ):

Sim. Se tomarmos a suficientemente grande e ( x1 , x2 , x3 ) = (a, a, a), trivial ver que cada xi um polinmio em a. Em particular, como a seqncia possui um nmero finito de termos, podemos tomar a suficientemente grande de tal forma que cada termo xi da seqncia tenha o sinal do coeficiente lder do polinmio. Mas fcil ver que a seqncia dos termos lderes : (a, a, a, a 2 , 2a 2 , a3 ,2a 4 ,2a5 , 2a7 ,4a9 , 4a12 ,...) e fcil provar, por induo que dois termos lderes nunca se cancelam (basta notar que a partir de 4a12 , os exponentes dos dois termos anteriores so sempre maiores que metade do expoente c do termo atual: isso verdade para 4a12 , e se os expoentes so a, b e c, com a > 2 c a+b pois e b > , a < b < c (j que a seqncia dos expoentes crescente), ento b > 2 2 a +b pois c > a e c > b). Mas a seqncia dos sinais dos termos lderes b>ae c> 2 claramente peridica de perodo 7: (+, , +, , , , +, +, , +, , , , +, +, , +,...) Logo a seqncia definida com o a supracitado e x1 = a, x2 = a, x3 = a tem pelo 4 menos 2004 > 1002 termos negativos. 7
PROBLEMA 4:

Veja a soluo do problema No. 6 do nvel 2.

EUREKA! N22, 2005

46

Sociedade Brasileira de Matemtica

PROBLEMA 5: SOLUO DE FBIO DIAS MOREIRA (RIO DE JANEIRO - RJ):

Vamos demonstrar que an sempre inteiro por induo: isto evidentemente verdadeiro para n = 0, 1, 2, 3, 4, 5, 6, 7 (pois a4 = 2, a5 = 3, a6 = 7 e a7 = 23) . Suponha ento que n 8 , e a0 , a1 , a2 ,..., an 1 so todos inteiros. Lema: Se 0 < k < n, ento (a k , ak 1 ) = 1. Prova: O resultado evidentemente verdadeiro para k =1. Suponha-o verdadeiro para k 1. Ento a a + a2 2 2 (ak , ak 1 ) = k 1 k 3 k 2 , ak 1 (ak 1ak 3 + ak 2 , ak 1 ) = (ak 2 , ak 1 ) = 1, pois ak 4 (ak 1 , ak 2 ) = 1. Logo o resultado vlido para todo 0 < k < n. Corolrio: 1 < k < n (ak , ak 2 ) = 1 e 2 < k < n ( ak , ak 3 ) = 1. Prova: Basta notar que 2 (ak , ak 2 ) (ak 1ak 3 + ak 2 , ak 2 ) = (ak 1ak 3 , ak 2 ) (ak 1 , ak 2 ) (ak 3 , a k 2 ) = 1 1 = 1
2 2 2 e (ak , ak 3 ) (ak 1ak 3 + ak22 , ak 3 ) = (ak 2 , ak 3 ) (ak 2 , ak 3 ) = 1 = 1. 2 Para provar que an inteiro, basta provar que an 4 | an 1an 3 + an 2 . Em outras palavras, temos que demonstrar que 2 an 3an 1 + an 2 0 (mod an 4 )

a a + a2 + n 5 n 3 n 4 0 (mod an 4 ) an6 2 2 2 2 2 an 6 (an 6 an 4 + an 5 )(an 4 an 2 + an 3 ) + an 5 an 7 (an 5 an 3 + an 4 ) 0 (mod an 4 ) 2 an5 an 6 an7


2 2 an 6 an 4 + an 5 an 4 an 2 + an 3 an7 an5 2 Pelo Lema, podemos multiplicar por an 5 an 6 an 7 e manter a equivalncia, logo basta provar que 2 2 2 2 2 an 6 (an6 an4 + an5 )(an4 an2 + an3 ) + an5 an7 (an5 an3 + an 4 ) 0(mod an 4 ) 2 2 2 an 5 an3 (an6 + an7 an5 ) 0(mod an 4 ) 2 2 an 5 an3an 4an8 0(mod an 4 ) 0 0 (mod an 4 ).

PROBLEMA 6: SOLUO DE GABRIEL TAVARES BUJOKAS (SO PAULO - SP):

(b + 1)2 . 4 Seja k o tamanho do ciclo, P = ( x1 , x0 ), f ar,b ( P) = ( xr +1 , xr ), para todo r < . Resposta:

EUREKA! N22, 2005

47

Sociedade Brasileira de Matemtica


2 x0 = a bxk 2 xk 1 2 x1 = a bxk 1 x0

Logo x2 = a bx0 x12 xk 1 = a bxk 3 xk22

Sendo S1 = xi , S2 = xi2 , temos S1 (b + 1) + S 2 = ka

S2 xi S1 S2 S2 1 k k k k
MQ MA

S12 x2 (b +1)2 . + S1(b +1) + x(b +1) ka tem raiz = (b +1)2 + 4a 0 a k k 4 (b + 1)2 (b + 1) (b + 1) (b + 1) (b + 1) Para a = : f a ,b ; ; = per (a,b) . 4 2 2 2 2 Logo: ka

EUREKA! N22, 2005

48

Sociedade Brasileira de Matemtica

XXVI OLIMPADA BRASILEIRA DE MATEMTICA


Problemas e Solues da Primeira Fase Nvel Universitrio
PROBLEMA 1:

1 0 i Considere a matriz complexa A = 0 0 0 . Calcule A2004 . i 0 1


PROBLEMA 2:

Calcule a integral:

x 2004 1 1 + e x dx
1

PROBLEMA 3:

Determine a equao da reta que tangencia a curva de equao y = 3x 4 4 x 3 em dois pontos distintos.
PROBLEMA 4:

Quantas triplas ordenadas (A, B, C) de subconjuntos de {1, 2,..., n} existem para as quais A B C = ; A B ; A C ?

PROBLEMA 5:

Considere a matriz A n n definida por aij = n(i 1) + j , para todos 1 i, j n. As intersees de k linhas e k colunas quaisquer de A determinam uma submatriz de ordem k de A. Seja ( n) a soma dos determinantes de todas as submatrizes de A. (n ) a) Determine real de forma que lim exista e seja no nulo. n + n b) Determine o valor do limite acima para o valor de encontrado.
PROBLEMA 6:

Calcule

(3k + 1)(3k + 2)(3k + 3) .


k =0

EUREKA! N22, 2005

49

Sociedade Brasileira de Matemtica

SOLUES PRIMEIRA FASE NVEL UNIVERSITRIO


SOLUO DO PROBLEMA 1:

(4)k 0 0 4 0 0 Temos A 4= 0 0 0 , donde A 4k = 0 0 0 . 0 0 4 0 0 (4)k 21002 0 0 Em particular, A 2004= 0 0 0 . 0 0 21002 Solues cujo nico erro se refere aos sinais:
SEGUNDA SOLUO DO PROBLEMA 1:

1 0 1 1 + i 0 0 Diagonalizando A, temos A = XDX onde X = 0 1 0 , D = 0 0 0 1 0 1 0 0 1 i Assim, (1+ i)2004 0 21002 0 0 21002 0 0 0 0 0 X 1 = X 0 0 0 X 1 = 0 0 0 . A2004 = X 0 0 (1 i)2004 0 21002 0 21002 0 0 0
1

SOLUO DO PROBLEMA 2:

Por substituio,

2004 1 x x 2004 = dx 1 1 + e x 1 1 + e x dx . 1

Assim 2004 2004 1 x 1 x 1 1 1 1 x 2004 1 1 = + + dx dx dx = x 2004 1 x x x x 1 1 + e x 1 1 + e 2 1+ e 1+ e 1+ e 2 1 1 1 1 1 . = x 2004 dx = x 2004 dx = 1 0 2 2005


SOLUO DO PROBLEMA 3:

dx

Queremos encontrar a e b tais que, P = 3 x 4 4 x3 ax b tenha duas razes reais duplas; em particular, P deve ser um quadrado perfeito.

EUREKA! N22, 2005

50

Sociedade Brasileira de Matemtica

3 x 4 4 x3 ax b = ( cx 2 + dx + e) 2 = c 2 x 4 + 2cdx3 + (2ce + d 2 ) x 2 + 2 dex + e 2 8 4 c c implica em c = 3, d = 2 , e = 2 donde a = , b = . 3 9 9 27 8 4 Assim a reta tem equao y = x . 9 27


SOLUO DO PROBLEMA 4:

Vamos contar inicialmente o nmero de escolhas tais que A B C = .


A S1 B

S1

S2

S5

S6

S3

Cada elemento de {1,2,,n} pode ser colocado em um dos 7 subconjuntos indicados acima (S1, S2, ,S7). Logo h 7n tais escolhas. Fazendo um raciocnio similar, temos, dentre essas escolhas, 6n com A B = ; 6n com A C = e 5n com A B = e A C = . Portanto, pelo princpio de Incluso-Excluso, h 7 n 2 6n + 5n maneiras de escolher A, B, C.
SOLUO ALTERNATIVA DO PROBLEMA 4:

Fixe o nmero k de elementos de A B (1 ^ k ^ n), e j de A C (1 ^ j^ n k).


EUREKA! N22, 2005

51

Sociedade Brasileira de Matemtica

H k modos de escolher esses k elementos,

n k modos de escolher os j e 5 j

regies permitidas para cada um dos outros n k j elementos, de forma que a resposta :

R=

n n k n k j = j 5 k =1 j =1 k
n nk

n nk n k nk j j 5 k =1 k j =1
n

Utilizando o binmio de Newton, tem-se


R=

nk

n k nk j = (1 + 5 ) n k = 6 n k . 5 j j=0
n n nk n nk = 7 n 6 n (6 n 5 n ) = 7 n 2 6 n + 5 n . 6 k k 5 k =1 k =1 n

Portanto,

SOLUO DO PROBLEMA 5:

Se k 3 , qualquer submatriz de ordem k de A determinada por k linhas e k colunas tem determinante O. De fato, se j1 < j2 < j3 so ndices de 3 das k colunas da submatriz, denotando essas colunas por C j1 , C j2 e C j3 , temos que todas as entradas de C j2 C j1 so iguais a j2 j1 e todas as entradas de C j3 C j2 so iguais a j3 j2 , donde C j3 C j2 = j3 j2 j j j j (C j2 C j1 ), e logo C j3 = 3 1 C j2 3 2 C j1 . j2 j1 j2 j1 j2 j1

Assim, ( n) a soma dos determinantes das submatrizes de A de ordem 1 ou 2. A soma dos determinantes das submatrizes de A de ordem 1

m =
m =1

n2

n 2 (n 2 + 1) . 2

As submatrizes de ordem 2 so obtidas da seguintes forma: Dados 1 a < b n e 1 c < d n , associamos a seguinte submatriz de ordem 2: n(a 1) + c n(a 1) + d , cujo determinante n(b 1) + c n(b 1) + d n(( a 1) d + (b 1)c (b 1) d ( a 1) c) = n( a b)( d c). Assim, a soma dos determinantes das submatrizes de ordem 2

EUREKA! N22, 2005

52

Sociedade Brasileira de Matemtica

n j 1 n j ( j 1) n (a b) (d c) = n ( j i) = n r = n = 1a<bn 1c<d n 1i< j n j =1 r =1 j =1 2 g (n) n n3 = + g (n) , onde lim 3 = 0 (de fato, g (n) um polinmio de grau 2). n 4 3 n 2 2 1 n (n + 1) Assim, como lim 7 =0 e n n 2
2 1 1 g(n) 2 1 n n3 1 lim 7 + g (n) = lim + 3 = , n 4 3 n 36 n 4 3 n 2

Temos = 7 e lim

1 (n ) = . 7 n n 36

SOLUO DO PROBLEMA 6:

Decompondo em fraes parciais, procuramos constantes A, B e C tais que:

1 A B C = + + (1) (3k + 1)(3k + 2)(3k + 3) 3k + 1 3k + 2 3k + 3


Comparando os numeradores, verifica-se que (1) identidade para

A=

1 1 , B = 1, C = . 2 2

Sendo S a soma procurada: 2S =


1

3k + 1 3k + 2 + 3k + 3 .
k =0 1

Como

1 = x 3k dx , tem-se 2S = x 3k 2 x 3k +1 + x 3k + 2 dx . 3k + 1 0 k =0 0

Trocando a integral com o somatrio e somando a PG infinita:


1 1 (1 x ) 2 3k 3 k +1 3k + 2 2 3k +x 2S = x 2 x dx = (1 2 x + x ) x dx = dx 1 x3 k =0 0 k =0 0 0 1

EUREKA! N22, 2005

53

Sociedade Brasileira de Matemtica

2S =
0

1 x 1 (2 x + 1) + 3 dx = 2 dx 2 2 0 x + x +1 x + x +1
1

1 1 2x + 1 31 dx + dx 2 2 2 2 1 2 + + x x 0 0 1 3 x+ 2 + 2
1

1 1 3 2 2 = 1 ln 3 + 3 ( ) = ln( x 2 + x + 1) + arctan 2 2 3 2 3 6 3 2 0 x+ Logo, S =

3 3ln 3 12

EUREKA! N22, 2005

54

Sociedade Brasileira de Matemtica

XXVI OLIMPADA BRASILEIRA DE MATEMTICA


Problemas e Solues da Segunda Fase Nvel Universitrio
PROBLEMA 1:

A funo derivvel f : a) f (0) = 0 e f (2) = 2.

tem as seguintes propriedades:

b) Para todo a \ {0}, a reta tangente ao grfico de f no ponto P = (a, f (a )) corta o eixo x em um ponto A e o eixo y em um ponto B de tal forma que A o ponto mdio do segmento BP. Calcule f (3).
PROBLEMA 2:

Prove que no existe um conjunto A Para todo x Para todo y , {y , {x

tal que:

( x, y ) A} finito. ( x, y ) A} enumervel.

Obs: Um conjunto A dito enumervel se A = ou existe uma funo sobrejetiva f : A.

<

PROBLEMA 3:

Seja A uma matriz real inversvel de ordem n e At a sua transposta. Sejam 1 2 ... n > 0 os autovalores de At A. Definimos a norma de A por A = 1 e o fator de dilatao de A por d ( A) = 1 reais inversveis A e B, d ( AB)

. Prove que, para quaisquer matrizes

AB d ( B) . A B

PROBLEMA 4:

Seja

= {( x1 , x2 ,..., xn ); x1 , x2 ,..., xn }.
n

Seja p um primo, k um inteiro positivo e P 1, P 2 ,..., P k, Q Pj Q n. 1 j k, p


EUREKA! N22, 2005

tais que para todo j,

55

Sociedade Brasileira de Matemtica

Prove que existe um polinmio f ( x1 ,..., xn ) com coeficientes inteiros com f ( Pj ) = 0 para todo j, 1 j k , e
PROBLEMA 5:

f (Q) . p

Seja m 2 um inteiro. Arnaldo e Bernaldo disputam o seguinte jogo: cada jogador recebe, alternadamente, N um nmero N k e devolve para o outro jogador ou N k +1 = N k 1 ou N k +1 = k . m Arnaldo comea recebendo um nmero inteiro positivo N0 . Quem devolver zero vence o jogo. Seja An (resp. Bn ) o conjunto dos valores de N0 , N0 < n , tais que Arnaldo (resp. Bernaldo) tem estratgia vencedora. A Calcule lim n em funo de m. n B n
PROBLEMA 6:

Seja :[0,2 ]

uma funo derivvel, com derivada contnua, com

'(t ) = 1 para todo t e cuja imagem uma curva simples fechada, isto , (t0 ) = (t1 ), t0 < t1 t0 = 0, t1 = 2 .
Prove que existem 0 t0 < t1 < 2 tais que 2 (t0 ) (t1 ) min {t1 t0 ,2 + t0 t1} .

EUREKA! N22, 2005

56

Sociedade Brasileira de Matemtica

PROBLEMA 1: SOLUO DE LEONARDO AUGUSTO ZO (NILPOLIS RJ)

De acordo com o item (b) do enunciado, ao traarmos a reta tangente ao grfico no ponto (a, f(a)), com a 0, obtemos:
f (x) Reta tangente ao grfico de f em (a, f(a)) P = (a, f(a))

A = (xA, 0) x

B = (0, yB)

Como A ponto mdio de BP, ento: A = Assim, x A = a e yB = f (a ) . 2

B+P 2

Portanto, o coeficiente angular da reta dado por tg = Assim, f '( a) = 2 f (a ) , a *. a

( f (a ) yB ) 2 f (a ) = (a a / 2) a

Ento chegamos equao diferencial: f '( x) = Resolvendo, temos:

2 f ( x) ,x * x
* +

f '( x) 2 = [ln f ( x) ]' = [2ln x ]', x f ( x) x

Ento ln f ( x) = 2ln x + k ln f ( x ) = ln x 2 e k f ( x ) = e k x 2 , x 1 Para x = 2, f ( x) = 2. Ento f (2) = e k 22 ek = . 2 1 2 Assim, a funo f obedece f ( x ) = x , para x * + . 2 1 9 Ento, f (3) = 32 , e finalmente, f (3) = . 2 2
EUREKA! N22, 2005

* +

57

Sociedade Brasileira de Matemtica

PROBLEMA 2: SOLUO DE DAVI MXIMO ALEXANDRINO NOGUEIRA (FORTALEZA CE)

Sejam Ay = {x Bx = { y

| ( x, y ) A} | ( x , y ) A} e
i =1

Logo Bx finito, x

Ay enumervel, y

. . teramos

Agora sejam y1 < y2 < y3 < ..., uma seqncia crescente infinita em Logo,
i =1

Ayi =

( Ayi ) , Ayi

pois,

do
i =1

contrrio,

i =1

( Ayi ), mas cada


i =1

enumervel

( Ayi ) enumervel

enumervel, absurdo! Ayi . Temos { y1 , y2 ,...,} Bx , absurdo! (pois Bx finito). Logo A

Agora, tome x no existe.

PROBLEMA 3: SOLUO DE ALEX CORRA ABREU (NITERI RJ)

Sejam 1 2 ... n > 0; 1 2 ... n > 0; 1 2 ... n > 0 os autovalores de ( AB)t AB, B t B e At A respectivamente. A desigualdade pedida claramente equivalente a 2 1 2 . Como X X sempre simtrica e < X Xu, u >=< Xu, Xu > , temos pelo teorema do min-max (que provaremos no fim desta soluo) que: < ABx, ABx > < ABx, ABx > < Bx, Bx > 2 = min max = min max . dim S = n 1 < x, x > dim S = n 1 xS \{0} < Bx, Bx > < x, x > xS Por outro lado, como s h um subespao de dimenso n (e ele inclui todos os vetores), temos: < Bx, Bx > < Ax, Ax > 1 = max , e 2 = min max . dim S = n 1 xS \{0} < x, x > x n \{0} < x, x >
T T

Seja S o subespao no qual 2 atingido, i.e, 2 = max

< Bx, Bx > . xS \{0} < x, x >

Temos ento: < ABx, ABx > < Bx, Bx > < ABx, ABx > < Bx, Bx > 2 max max max , xS \{0} < x, x > xS \{0} < Bx, Bx > xS \{0} < x, x > < Bx, Bx >
EUREKA! N22, 2005

58

Sociedade Brasileira de Matemtica

(usamos respectivamente que o mnimo ^ 6:0 6:, 6:07 0 020394 /4 .43 :394 0 max(uv ) max u max v ) O segundo termo do produto exatamente 2 . Fazendo x' = Bx e notando que B( S ) n , tem-se: < ABx, ABx > < Ax ', Ax ' > max 1 , e juntando tudo obtemos 2 1 2 max n xS \{0} < Bx, Bx > x ' \{0} < x ', x ' > Lema: (Teorema do Min Max ): 1 = max < Ax, x >,
x =1

2 = min

dim S = n 1

max < Ax, x > , para toda A matriz simtrica, onde 1 2 ... n so
xS x =1

seus autovalores. Prova: Como A simtrica existe base ortonormal x1 ,..., xn tal que Axi = i xi logo,
2 2 2 + ... + n an 1 (a12 + ... + an ) = 1 se x = a1 x1 + ... + an xn , < Ax, x > = 1a12 + 2 a2 max < Ax, x > 1 , mas, tomando x = x1 , < Ax1 , x1 > = 1 . Para o 2 , seja L subespao gerado por x1 e x2 . Assim, dim S = n 1 max < Ax, x > max < Ax, x > , mas se y = a1 x1 + a2 x2 L S , xS x =1 xL S x =1 2 2 2 = 1 max < Ax, x > < Ay, y > = 1a12 + 2 a2 2 (a12 + a2 a12 + a2 ) = 2 xLS x =1

dim S = n 1

min

max < Ax, x > 2 . Agora, tomando S = ( x1 ) temos que o mnimo


xS

atingido.
PROBLEMA 4: SOLUO DE YURI GOMES LIMA (FORTALEZA - CE)

Seja Q = (b1 ,..., bn ). Sejam tambm Pj = ( a j1 ,..., a jn ), a j) , 1 j k , 1 n. Observe que que a ji j bi j Pj Q p


k

a b a j 1 b1 ,..., jn n p p

existe i j {1,2,..., n} tal

p Tome ento

p | (a ji j bi j )

f ( x1 ,..., xn ) = ( xi j a ji j )
j =1

Note que:
EUREKA! N22, 2005

59

Sociedade Brasileira de Matemtica

f ( x1 ,..., xn ) = ( xi j a ji j ) ( xi a ji ) = ( xi j a ji ) R j ( x1 ,..., xn )
)=1 ) j
) )

f ( Pj ) = f (a j ,..., a jn ) = (a ji j a ji j ) R j (a j1 ,..., a jn ) = 0 f ( Pj ) = 0 Ademais, f (Q) = f (b1 ,..., bn ) = (bij a ji j ). Como p | (bij a ji j ), j, e p primo
j =1 k

segue que p | f (Q)

f (Q) p

(c.q.d.)

PROBLEMA 5: SOLUO OFICIAL

Consideremos a seqncia a0 , a1 , a2 ,... com an {0,1}, n tal que an = 1 se Arnaldo tem estratgia vencedora recebendo N0 = n e an = 0, caso contrrio, i.e., caso Bernaldo tenha estratgia vencedora se Arnaldo recebe N0 = n . fcil ver que, km + r k para k 1, se ak = 0, akm+ r = 1 para 0 r m 1 (nesse caso = ), e, se m 1 ( 1)r 0, se r par ak = 1 caso akm 1 = 1, temos akm +r = para 0 r m 1, e, = 2 1, se r mpar 1 + ( 1) r 1, se r par . = 2 0, se r mpar Suponhamos inicialmente que m par. Assim, pela discusso acima, se m m ak = 1, para valores de r, com 0 r m 1, temos akm +r = 0, e para os demais 2 2 valores de r nesse intervalo temos akm +r = 1. Assim, se Bn = {0 k < n ak = 0} e caso akm 1 = 0, temos, para 0 r m 1, akm+ r = An = {0 k < n ak = 1}, temos B n + A n = n , Bmn = Amn = m Un + Bmn = m Vn 1, n 1, 2 donde, como m Vn + 1 e 2 An = n Bn , temos

<

Bmn 1 B 1 . Como a soluo de = 1 n + mn 2 n mn Bn 1 1 1 1 1 t = (1 t ) t = , fazemos . De = + n , e obtemos mn = n + n mn 2 3 3 2 Bmn Bmn+r Bmn + m 1 Bmn Bmn+r Bmn + (m 1) para 0 r m 1, obtemos , m(n + 1) mn + r mn m ( n Bn ) + 1, e logo 2

EUREKA! N22, 2005

60

Sociedade Brasileira de Matemtica

donde, para 0 r m 1, mn + r mn <

1 m , desde que n m 1. Assim, < n + 1 mn + r 2m 1 m , de onde fcil provar por se N m(m 1), n + m + N / m N / m N N 2 induo em k que, se k 2 e mk n < mk +1 , temos n k / 2k 2 , donde lim n = 0.
n

Portanto, se m par lim lim

Bn

A n Bn B 1 2 = , donde lim n = lim = 1 lim n = , e n n n 3 3 n n n n

An A n 23 = lim n = = 2. n Bn Bn n 1 3

Suponha agora m mpar. Para cada seqncia finita de 0's e 1's denotaremos por n a seqncia ... obtida pela concatenao de n cpias da seqncias .

n vezes

No que se segue, usaremos o termo bloco para denotar uma seqncia do tipo 1r 0 , com r 1. Veremos agora que (an )n m +1 obtida por uma concatenao de blocos dos tipos 10, 1m 0 e 1m +10. A seqncia am +1 am +2 ...am( m +1) (10)
m ( m 1) 1 2

1m+10. Vamos mostrar por induo que,

mk 1 m k +1 m m k +1 1 para todo k 2, a seqncia k = a j , j = 1 obtida m 1 m 1 m 1 m por uma concatenao de blocos dos tipos 10 e 1 0 , seguida por um nico bloco de m2 1 m3 m tipo 1m +10 (note que = m +1 e = m(m + 1), e logo j verificamos o m 1 m 1 caso k = 2). Se a j 1 = 0, a j +t = 1 para 0 t r 1 e a j +r = 0, temos am ( j +r ) +s = am ( j +1)+s = 1 para 0 s m 1, e, para 0 s mr 1, amj + s = 1 (1) s 0, se s par = . Assim, se 2 1, se s mpar

r = 1 ou r = m, (amj + s ) 0 s mr 1 comea e termina por 0, e, se r = m + 1, (amj + s ) 0 s mr 1 comea por 0 e termina por 1. Logo, blocos dos tipos 10 e 1m 0 geram apenas blocos desses mesmos tipos, e blocos do tipo 1m +1 s podem ser gerados por blocos do tipo 1m +10 (eles aparecem no fim da seqncia gerada por 1m +10 ). Essas observaes provam a nossa afirmao.
EUREKA! N22, 2005

61

Sociedade Brasileira de Matemtica

Como os nicos blocos diferentes desses que aparecem so os do tipo 1m +10 de m k +1 m m e tamanho total m + 2, os quais s aparecem entre as posies m 1 m k +1 m m k +1 m > m k + m, temos , k 2, e como m 1 m 1 log n ( m + 2) log n 1 n , donde 2un + (m + 1)vn n ( m + 1) (m + 2) log m log m 2u + (m + 1)vn = 1. Alm disso, pela discusso acima, seqncias dos tipos 10 e lim n n n 1m 0 umn geram seqncias 2 m 1 m 1 = vn + rn un + 2 2 desses e mesmos para tipos, n m + 1, donde onde

vmn = un + vn + sn ,

m 2 log n log n e sn os desvios rn e sn se devem a blocos gerados por log m log m log n blocos incompletos e pelos blocos do tipo 1m +10, que so no mximo em log m ( m + 2) log n (ak ) k n ). Como vimos acima, 2un + (m + 1)vn = n wn , onde wn . log m u v Sejam xn = n e yn = m n . Temos un + mvn An un + mvn + wn , donde n n A log n segue que e, de 2un + (m + 1)vn = n wn , xn + yn = n + O , n n m +1 log n yn = 1 O 2 xn + . m n rn

De Vmn = un + vn + sn e de vN v donde y N = x N +
m

N m m

1 segue que vN = u N + v N + O(log N ),


m m

1 1 log N 1 m + 1 log N y N + y N + O 1 y N + O = = m m m N 2 N m m m

EUREKA! N22, 2005

62

Sociedade Brasileira de Matemtica

1 m 1 m log N + (note que a soluo de y N + O . Escrevendo yn = 2 2m 3m 1 n m N 1 m 1 t 2 2m t= m m 1 log N N + O ) , temos N = , 3m 1 2m N m donde

t=

m 1 limsup n + 0, e logo limsup n = 0, ou 2m n n n n 1 m +1 m log n yn + O e, de xn = 1 seja, lim n = 0 , donde lim yn = , n n 3m 1 2 m n A 1 m + 1 m m 1 log n = segue que lim xn = 1 . De xn + yn = n + O , n 2 m 3m 1 3m 1 n n 0 limsup n 1 e limsup n An B A 2m 1 m = lim ( xn + yn ) = , e lim n = 1 lim n = . n n n n n 3m 1 3m 1 n n A A n (2m 1) (3m 1) 2m 1 = = . Portanto, se m mpar, lim n = lim n n B n B m (3m 1) m n n n segue que lim
PROBLEMA 6: SOLUO OFICIAL (do Prof. Alexandre Fernandes, comunicada pelo Prof. Lev Birbrair, de Fortaleza, CE)

Suponha por absurdo que exista um contra-exemplo . Defina :[0,2 ] (t ) = (t ) (t + ), para 0 t e (t ) = (t ) (t ) para t 2 .

por

Pela hiptese de absurdo temos (t ) > 2 para todo t. A funo contnua (pois (0) = (2 )) e derivvel exceto talvez para t = 0, . Temos ainda '(t ) '(t ) + '(t + ) = 2 para todo t, donde o comprimento da curva menor ou igual a 4 . Mas a imagem de uma curva fechada simtrica em relao origem (pois (t + ) = (t ), t [0, ]), que est no exterior da bola de raio 2, donde seu comprimento maior que 4 , absurdo.
(0)

() = (0)

EUREKA! N22, 2005

63

Sociedade Brasileira de Matemtica

XXVI OLIMPADA BRASILEIRA DE MATEMTICA Resultado Nvel 1 (5a. e 6a. Sries)


NOME Illan Feiman Halpern Leonardo Pereira Stdile James Jung Hong Victor Reis de Abreu Cavalcanti Wagner Carlos Morto Loyola Filho Rafael Alves da Silva Marcelo Tadeu de S Oliveira Sales Lereno Soares Netto Dan Zylberglejd Clarissa Maria Grosso Fabris Matheus Barros de Paula Renan Henrique Finder Leonardo Shinizu Yojo Ana Luiza Ferron Zanella Fernanda Daltro Costa Knoblauch Thiago Ribeiro Ramos Lucas Gouveia Omena Lopes Jos Rodolfo Bezerra Mesquita Arajo Dbora Kiame Paias Daniel Caueh Dunaiski Figueira Leal Alessandro D'Alessio de Souza Eduardo Kaiser Urubahy Nunes Fonseca Caroline Rigolon Veiga Erick Magno Costa Alonso Vinicius Tineli Paiva Marcelo Aires Moreira Rafael Fernandes Paixo Diana Vaisman Camila Miraglia Ribeiro Stefano Tonnasini Vitor Cruth Sturn Hugo Fonseca Arajo Brbara Seccato Ruis Chagas Rafael Sarto Mller Thiago Gonales Jos Nilo Alves de Souza Neto Vitor Costa Fabris Ana Lusa de Almeida Losnak Rebeca Meirelles de Arajo Assis Gabriela Barini de Oliveira talo Leite de Camargo Arthur Lerer Andr Saraiva Nobre dos Santos Jos Ailton Azevedo Arajo Filho Csar Ilharco Magalhes Mariana Hollos Fiorencio Fernando Fonseca Andrade Oliveira Beatriz Arruda Asfora Joo Paulo Sousa Lucas Tayrini da Cruz Beligoli Mnica Monteiro Gondim Izabela Karennina Travizani Maffra Mara Islena Tavares da Silva Caio Menezes Fac Andr Yoshio Oshiro Bastos Natasha Scaranello Cartolano Laura Andrade Santiago Guilherme Alencar Sorensen Colares Eduardo de Almeida Grande Breno Augusto Cardoso Barroso CIDADE ESTADO Itatiaia RJ So Paulo SP So Paulo SP Macei AL Vitria ES Teresina PI Barreiras BA Rio de Janeiro RJ Rio de Janeiro RJ So Paulo SP Taubat SP Joinville SC So Paulo SP Pato Branco PR Salvador BA Varginha MG Macei AL Joo Pessoa PB Atibaia SP Curitiba PR So Paulo SP Rezende RJ Rio de Janeiro RJ Uberaba MG Vitria ES Joo Pessoa PB Rio de Janeiro RJ Rio de Janeiro RJ Curitiba PR Itu SP So Paulo SP Juiz de Fora MG Vitria ES Vitria ES Piracicaba SP Fortaleza CE Cricima SC So Paulo SP Salvador BA So Jos dos Campos SP Assis SP Rio de Janeiro RJ Fortaleza CE Fortaleza CE Juiz de Fora MG Rio de Janeiro RJ Belo Horizonte MG Recife PE Fortaleza CE Juiz de Fora MG Fortaleza CE Belo Horizonte MG Belo Horizonte MG Fortaleza CE So Paulo SP Rio Claro SP Fortaleza CE Fortaleza CE Valinhos SP Fortaleza CE PRMIO Ouro Ouro Ouro Ouro Ouro Ouro Prata Prata Prata Prata Prata Prata Prata Prata Prata Prata Prata Prata Bronze Bronze Bronze Bronze Bronze Bronze Bronze Bronze Bronze Bronze Bronze Bronze Bronze Bronze Bronze Meno Honrosa Meno Honrosa Meno Honrosa Meno Honrosa Meno Honrosa Meno Honrosa Meno Honrosa Meno Honrosa Meno Honrosa Meno Honrosa Meno Honrosa Meno Honrosa Meno Honrosa Meno Honrosa Meno Honrosa Meno Honrosa Meno Honrosa Meno Honrosa Meno Honrosa Meno Honrosa Meno Honrosa Meno Honrosa Meno Honrosa Meno Honrosa Meno Honrosa Meno Honrosa Meno Honrosa

EUREKA! N22, 2005

64

Sociedade Brasileira de Matemtica

Resultado Nvel 2 (7a. e 8a. Sries)


NOME Lucio Eiji Assaoka Hosaka Regis Prado Barbosa Henrique Pond de Oliveira Pinto Rafael Sampaio de Rezende Wilson Camara Marriel Rafael Tupynamb Dutra Adenilson Arcanjo de Moura Jnior Jos Airton Colho Lima Filho Filipe Alves Tom Rafael Moura e Sucupira Ramon Moreira Nunes Rodrigo Clemente de Brito Pereira Srgio Ricardo Furtado Sampaio Filho Giuliano Pezzolo Giacaglia Daniel Ungaretti Borges Gabriel Marcos Pasmanik Eisencraft Anna Kelly Krislane de Vasconcelos Paulo Vitor de Souza Albuquerque Artur de Almeida Losnak Ricardo Turolla Bortolotti Henrique Hiroshi Motoyama Watanabe Guilherme Philippe Figueiredo Alfredo Roque de Oliveira Freire Filho Lucas Zanotto Portela Nathana Alcntara Lima Flvio Domingos de Azevedo Quadros Mateus Oliveira de Figueiredo Marilia Valeska Costa Medeiros Grazielly Muniz da Cunha Fbio Yoshiteru Nukui Ana Luiza Viegas de Almeida Vinicius Marques Regitano Andr Antonio Battagello Anderson Unlin Tsai Mateos Kruchelski Tsch Caio Gustavo Mesquita Angelo Guilherme Zatieff Topolski Chaves Marlen Lincoln da Silva Janailson Rodrigues Lima Gilson Mauro Costa Fernandes Filho Diogo Bonfim Moraes Morant de Holanda Paulo Wayner Carvalho dos Santos Douglas Tirre Carnevale Oliveira Ivan Hitoshi Oyama Marlon Vieira de Lima Jnior Cssio Kendi Takamori Jessica de Aguiar Frana Karin Naomi Harada de Oliveira Diogo Soares Drea da Silva Mauricio Henrique Bezerra Cardoso rik Fernando de Amorim Caio Prado Siqueira Campos Luan Leal Oliveira Diego Eloi Misquita Gomes Cintia Mayumi Sakurai Kimura Carolina Santos Andrade CIDADE ESTADO Curitiba PR Fortaleza CE Salvador BA Fortaleza CE Rio de Janeiro RJ Belo Horizonte MG Fortaleza CE Fortaleza CE Fortaleza CE Fortaleza CE Fortaleza CE Joo Pessoa PB Fortaleza CE Santo Andr SP Belo Horizonte MG So Paulo SP Fortaleza CE Rio de Janeiro RJ So Paulo SP Rio Claro SP So Paulo SP Fortaleza CE Salvador BA Curitiba PR Fortaleza CE Osasco SP Fortaleza CE Fortaleza CE Fortaleza CE So Paulo SP Marlia SP Piracicaba SP Araatuba SP So Paulo SP Curitiba PR Taguatinga DF So Paulo SP Fortaleza CE Fortaleza CE Joo Pessoa PB Rio de Janeiro RJ Rio de Janeiro RJ Rio de Janeiro RJ Osasco SP Fortaleza CE So Jos dos Campos SP Braslia DF So Paulo SP Feira de Santana BA Aracaj SE Araraquara SP Fortaleza CE Trememb SP Fortaleza CE So Paulo SP So Cristvo SE PRMIO Ouro Ouro Ouro Ouro Ouro Prata Prata Prata Prata Prata Prata Prata Prata Prata Bronze Bronze Bronze Bronze Bronze Bronze Bronze Bronze Bronze Bronze Bronze Bronze Bronze Bronze Meno Honrosa Meno Honrosa Meno Honrosa Meno Honrosa Meno Honrosa Meno Honrosa Meno Honrosa Meno Honrosa Meno Honrosa Meno Honrosa Meno Honrosa Meno Honrosa Meno Honrosa Meno Honrosa Meno Honrosa Meno Honrosa Meno Honrosa Meno Honrosa Meno Honrosa Meno Honrosa Meno Honrosa Meno Honrosa Meno Honrosa Meno Honrosa Meno Honrosa Meno Honrosa Meno Honrosa Meno Honrosa

EUREKA! N22, 2005

65

Sociedade Brasileira de Matemtica

Resultado Nvel 3 (Ensino Mdio)


NOME Fbio Dias Moreira Rafael Marini Silva Gabriel Tavares Bujokas Rafael Daigo Hirama Guilherme Rodrigues Nogueira de Souza Telmo Luis Correa Junior Thiago Costa Leite Santos Thoms Yoiti Sasaki Hoshina Larissa Cavalcante Queiroz de Lima Henry Wei Cheng Hsu Elton Gomes Coriolano Larissa Rodrigues Ribeiro Rodrigo Aguiar Pinheiro Felipe Rodrigues Nogueira de Souza Ricardo Mizoguchi Gorgoll Guilherme Rodrigues Salerno Edson Augusto Bezerra Lopes Luiz Adolfo Schiller Marcus Edson Barreto Brito Lucas Pavan Barros Diogo dos Santos Suyama Felipe Ferreira Villar Coelho Jos Marcos Andrade Ferraro Francisco Bruno de Lima Holanda Luty Rodrigues Ribeiro Israel Franklin Dourado Carrah Douglas Bokliang Ang Cunha Levi Mximo Viana Leandro Farias Maia Victor de Andrade Lazarte Rafael Mendes de Oliveira Raphael Rodrigues Mata Thiago Jorge Marinho Vieira Ana Cludia Onuchic Paulo Victor Teixeira Eufrsio Carlos Augusto David Ribeiro Eduardo Fischer Edson Leandro Finotti Bittar Leandro Farias Nogueira Sheyne Cristina Leal Hugo Caetano da Silva Junior Erasto Villa Branco Neto Antonia Taline de Souza Mendona Jos Gerardo Arruda Junior Jos Mrio da Silva Filho Bruno Cardoso Vieira Pedro Henrique Milet Pinheiro Pereira Mateus Gomes Filgueiras Vicente Matheus Moreira Zuffo CIDADE ESTADO Rio de Janeiro RJ Vila Velha ES So Paulo SP Campinas SP So Paulo SP Santo Andr SP So Paulo SP Rio de Janeiro RJ Fortaleza CE So Paulo SP Fortaleza CE Fortaleza CE Fortaleza CE So Paulo SP So Paulo SP Goinia GO Fortaleza CE Niteri RJ Fortaleza CE Vila Velha ES Belo Horizonte MG Serra ES So Paulo SP Fortaleza CE Fortaleza CE Fortaleza CE So Jos dos Campos SP Fortaleza CE Fortaleza CE So Paulo SP Mesquita RJ Salvador BA Fortaleza CE So Paulo SP Fortaleza CE Fortaleza CE Encantado RS So Paulo SP Recife PE Rio de Janeiro RJ Teresina PI Curitiba PR Fortaleza CE Fortaleza CE Fortaleza CE Rio de Janeiro RJ Rio de Janeiro RJ Fortaleza CE So Paulo SP PRMIO Ouro Ouro Ouro Ouro Ouro Ouro Prata Prata Prata Prata Prata Prata Prata Prata Prata Bronze Bronze Bronze Bronze Bronze Bronze Bronze Bronze Bronze Bronze Bronze Bronze Bronze Bronze Bronze Meno Honrosa Meno Honrosa Meno Honrosa Meno Honrosa Meno Honrosa Meno Honrosa Meno Honrosa Meno Honrosa Meno Honrosa Meno Honrosa Meno Honrosa Meno Honrosa Meno Honrosa Meno Honrosa Meno Honrosa Meno Honrosa Meno Honrosa Meno Honrosa Meno Honrosa

EUREKA! N22, 2005

66

Sociedade Brasileira de Matemtica

Resultado Nvel Universitrio


NOME Carlos Stein Naves de Brito Alex Corra Abreu Humberto Silva Naves Bernardo Freitas Paulo da Costa Daniel Massaki Yamamoto Murilo Vasconcelos Andrade Davi Mximo Alexandrino Nogueira Carlo Pietro Sousa da Silva Yuri Gomes Lima Tertuliano Franco Santos Franco Rodrigo Roque Dias Digo Veloso Ucha Pietro Kreitlon Carolino Thiago Barros Rodrigues Costa Thiago da Silva Sobral Leonardo Augusto Zo Samuel Barbosa Feitosa Eduardo Famini Silva Einstein do Nascimento Jnior Eduardo Bertoldi Rafael Tajra Fonteles Diogo Diniz Pereira da Silva e Silva Kellem Corra Santos Domingos Dellamonica Junior Luis Daniel Barbosa Coelho Jorge Peixoto de Morais Neto Anne Caroline Bronzi Evandro Makiyama de Melo Helder Oliveira de Castro Moyses Afonso Assad Cohen Rogrio de Assis Medeiros Rodrigo Angelo Muniz Fabrcio Siqueira Benevides Giuliano Boava Filipe Rodrigues de Souza Moreira Antonio Carlos Maldonado S. A. Munhoz Anderson Rodrigues Ferreira Eduardo Ferraz Castelo Branco Ferreira Arthur Omar de Andrade Lazarte Vitor Gabriel Kleine CIDADE - ESTADO So Jos dos Campos SP Niteri RJ So Jos dos Campos SP Rio de Janeiro RJ So Paulo SP Rio de Janeiro RJ Fortaleza CE So Cristvo SE Fortaleza CE Salvador BA So Paulo SP Teresina PI Campinas SP Fortaleza CE Fortaleza CE Nilpolis RJ Fortaleza CE Rio de Janeiro RJ Fortaleza CE Vila Velha ES Teresina PI Campina Grande PB Rio de Janeiro RJ So Paulo SP So Jos dos Campos SP So Jos dos Campos SP Ribeiro Preto SP So Paulo SP Mogi das Cruzes SP Rio de Janeiro RJ Franco da Rocha SP Cariacica ES Fortaleza CE Florianpolis SC So Jos dos Campos SP Rio de Janeiro RJ Rio de Janeiro RJ Rio de Janeiro RJ So Paulo SP Mogi das Cruzes SP PRMIO Ouro Ouro Ouro Ouro Ouro Ouro Prata Prata Prata Prata Prata Prata Prata Prata Prata Bronze Bronze Bronze Bronze Bronze Bronze Bronze Bronze Bronze Bronze Bronze Bronze Meno Honrosa Meno Honrosa Meno Honrosa Meno Honrosa Meno Honrosa Meno Honrosa Meno Honrosa Meno Honrosa Meno Honrosa Meno Honrosa Meno Honrosa Meno Honrosa Meno Honrosa

EUREKA! N22, 2005

67

Sociedade Brasileira de Matemtica

AGENDA OLMPICA
XXVII OLIMPADA BRASILEIRA DE MATEMTICA NVEIS 1, 2 e 3 Primeira Fase Sbado, 11 de junho de 2005 Segunda Fase Sbado, 03 de setembro de 2005 Terceira Fase Sbado, 22 de outubro de 2005 (nveis 1, 2 e 3) Domingo, 23 de outubro de 2005 (nveis 2 e 3 - segundo dia de prova). NVEL UNIVERSITRIO Primeira Fase Sbado, 03 de setembro de 2005 Segunda Fase Sbado, 22 e Domingo, 23 de outubro de 2005

XI OLIMPADA DE MAIO 14 de maio de 2005

XVI OLIMPADA DE MATEMTICA DO CONE SUL 23 a 28 de maio de 2005 Sucre, Bolvia

XLVI OLIMPADA INTERNACIONAL DE MATEMTICA 08 a 16 de julho de 2005 Yucatn, Mxico

XII OLIMPADA INTERNACIONAL DE MATEMTICA UNIVERSITRIA 22 a 28 de julho de 2005 Blagoevgrad, Bulgria

XX OLIMPADA IBEROAMERICANA DE MATEMTICA 24 a 30 de setembro de 2005 Cartagena de ndias, Colmbia

VIII OLIMPADA IBEROAMERICANA DE MATEMTICA UNIVERSITRIA 19 de novembro de 2005

EUREKA! N22, 2005

68

Sociedade Brasileira de Matemtica

COORDENADORES REGIONAIS
Alberto Hassen Raad Amrico Lpez Glvez Amarsio da Silva Arajo Ana Paula Bernardi da Silva Antonio Carlos Nogueira Ali Tahzibi Benedito Tadeu Vasconcelos Freire Carlos Frederico Borges Palmeira Claus Haetinger Cleonor Crescncio das Neves Cludio de Lima Vidal Edson Roberto Abe lio Mega der Luiz Pereira de Andrade Eudes Antonio da Costa Florncio Ferreira Guimares Filho Ivanilde Fernandes Saad Jacqueline Fabiola Rojas Arancibia Janice T. Reichert Joo Bencio de Melo Neto Joo Francisco Melo Libonati Jos Carlos dos Santos Rodrigues Jos Cloves Saraiva Jos Gaspar Ruas Filho Jos Luiz Rosas Pinho Jos Vieira Alves Krerley Oliveira Licio Hernandes Bezerra Luzinalva Miranda de Amorim Mrio Rocha Retamoso Marcelo Rufino de Oliveira Marcelo Mendes Marilane de Fraga Sant'Ana Pablo Rodrigo Ganassim Ramn Mendoza Ral Cintra de Negreiros Ribeiro Ronaldo Alves Garcia Reginaldo de Lima Pereira Reinaldo Gen Ichiro Arakaki Ricardo Amorim Srgio Cludio Ramos Seme Guevara Neto Tadeu Ferreira Gomes Toms Menndez Rodrigues Valdenberg Arajo da Silva Valdeni Soliani Franco Vnia Cristina Silva Rodrigues Wagner Pereira Lopes (UFJF) (USP) (UFV) (Universidade Catlica de Braslia) (UFU) (USP) (UFRN) (PUC-Rio) (UNIVATES) (UTAM) (UNESP) (Colgio Objetivo de Campinas) (Colgio Etapa) (UNESPAR/FECILCAM) (Univ. do Tocantins) (UFES) (UC. Dom Bosco) (UFPB) (UNOCHAPEC) (UFPI) (Grupo Educacional Ideal) (Unespar) (UFMA) (ICMC-USP) (UFSC) (UFPB) (UFAL) (UFSC) (UFBA) (UFRG) (Grupo Educacional Ideal) (Colgio Farias Brito, Pr-vestibular) FACOS (Liceu Terras do Engenho) (UFPE) (Colgio Anglo) (UFGO) (Escola Tcnica Federal de Roraima) (UNIVAP) (Centro Educacional Logos) (IM-UFRGS) (UFMG) (UEBA) (U. Federal de Rondnia) (U. Federal de Sergipe) (U. Estadual de Maring) (U. Metodista de SP) (CEFET GO) Juiz de Fora MG Ribeiro Preto SP Viosa MG Braslia DF Uberlndia MG So Carlos SP Natal RN Rio de Janeiro RJ Lajeado RS Manaus AM S.J. do Rio Preto SP Campinas SP So Paulo SP Campo Mouro PR Arraias TO Vitria ES Campo Grande MS Joo Pessoa PB Chapec SC Teresina PI Belm PA Campo Mouro PR So Luis MA So Carlos SP Florianpolis SC Campina Grande PB Macei AL Florianpolis SC Salvador BA Rio Grande RS Belm PA Fortaleza CE Osrio RS Piracicaba SP Recife PE Atibaia SP Goinia GO Boa Vista RR SJ dos Campos SP Nova Iguau RJ Porto Alegre RS Belo Horizonte MG Juazeiro BA Porto Velho RO So Cristovo SE Maring PR S.B. do Campo SP Jata GO

EUREKA! N22, 2005

69

CONTEDO

AOS LEITORES XIX OLIMPADA IBEROAMERICANA DE MATEMTICA Enunciados e Solues

2 3

ARTIGOS
PESSOAS CONFIVEIS E NO-CONFIVEIS Svetoslav Savchev O TEOREMA DE STOLZ Digo Veloso Ucha & Renato Purita Paes Leme CONTAS COM DESIGUALDADES Mrcio Afonso Assad Cohen & Rodrigo Villard Milet O PROBLEMA IMPOSSVEL Cssio Neri COMO QUE FAZ? SOLUES DE PROBLEMAS PROPOSTOS PROBLEMAS PROPOSTOS AGENDA OLMPICA COORDENADORES REGIONAIS 10

15 23

32 40 44 59 61 62

Sociedade Brasileira de Matemtica

AOS LEITORES
Neste nmero publicamos um artigo do Prof. Svetoslav Savchev, da Bulgria, um dos maiores especialistas internacionais em problemas de olimpada, alm de trs outros artigos de temas variados. Nos tem alegrado bastante a boa qualidade de artigos e problemas enviados por nossos colaboradores, que contribuem muito para a qualidade e a vitalidade de nossa revista. Publicamos tambm, alm das sees de Problemas propostos e Como que faz, solues da Olimpada Ibero-americana de 2004, na qual todos os quatro integrantes da equipe brasileira (Alex Corra Abreu, Fbio Dias Moreira, Gabriel Tavares Bujokas e Rafael Daigo Hirama) conquistaram medalha de ouro. Como a equipe de 2005 repetiu o feito, publicaremos no prximo nmero as suas solues. Aproveitamos tambm para registrar a realizao da IX Semana Olmpica, atividade que vem sendo realizada desde 1998. Nesta oportunidade o evento teve lugar na cidade de Juiz de Fora MG entre os dias 21 a 27 de janeiro de 2006. Durante a Semana Olmpica 2006, reunimos alunos ganhadores da XXVII Olimpada Brasileira de Matemtica nos seus quatro nveis de competio. Estes alunos participaram de um treinamento intensivo com professores de diversas partes do pas como preparao para a futura formao das equipes que representaro o Brasil em Olimpadas Internacionais.

Os editores

EUREKA! N23, 2006

Sociedade Brasileira de Matemtica

XIX OLIMPADA IBEROAMERICANA DE MATEMTICA


Enunciados e Solues
PRIMEIRO DIA
PROBLEMA 1

Deve-se colorir as casas de um tabuleiro 1001 1001 de acordo com as seguintes regras: Se duas casas tm um lado comum, ento pelo menos uma delas deve ser colorida. De cada seis casas consecutivas de uma linha ou de uma coluna, devem colorir-se sempre pelo menos duas delas que sejam adjacentes. Determinar o nmero mnimo de casas que devem ser coloridas.
SOLUO DE FBIO DIAS MOREIRA (RIO DE JANEIRO - RJ)

Lema: Todo retngulo 5 1 tem pelo menos trs quadrados pintados. Prova: Suponha que no, que o tabuleiro possui algum retngulo 5 1 (ou 1 5) com dois ou menos quadrados pintados. Ento, pela primeira condio, trivialmente impossvel que s 1 ou 0 quadrados estejam pintados e, se dois quadrados estiverem pintados, eles devem ser os da figura abaixo:
B

Mas ento tomamos um retngulo 6 1 que contenha o 5 1 e que esteja contido no tabuleiro (isto sempre possvel pois 5 < 1001). Independente do quadrado agregado ser ou no colorido, chegamos a um absurdo pois a segunda condio violada. Decompomos agora o tabuleiro em 200400 retngulos 5 1 e um quadrado 1 1 (200000 retngulos 5 1 na horizontal, contidos no quadrado 1000 1000 do canto superior esquerdo, 200 na horizontal, no retngulo 1000 1 na borda inferior, e 200 na vertical, no retngulo 1 1000 na borda direita e o quadrado 1 1 no canto inferior direito).
1000 1

1000

200000

200

200

EUREKA! N23, 2006

Sociedade Brasileira de Matemtica

Em cada um desses retngulos, pelo menos trs quadrados esto coloridos, logo pelo menos 601200 casas devem ser coloridos. E, de fato, este mnimo pode ser atingido: numere as colunas e as linhas do tabuleiro de 1 a 100. Pinte o quadrado (i, j) se somente se, i + j 1, 3 ou 4 (md. 5). Cada um dos retngulos 5 1 que criamos contm exatamente um quadrado de cada classe de congruncia, logo pintamos 601200 casas, pois o quadrado que no foi contado, (1001, 1001), no pintado (pois 1001 + 100 2 (md 5)). Alm disso fcil ver que qualquer retngulo 6 1 contm duas casas adjacentes pintadas.
PROBLEMA 2

Considera-se no plano uma circunferncia de centro O e raio r, e um ponto A exterior a ela. Seja M um ponto da circunferncia e N o ponto diametralmente oposto a M. Determinar o lugar geomtrico dos centros das circunferncias que passam por A, M e N quando M varia.
SOLUO DE ALEX CORRA ABREU (NITERI - RJ)
A

M P O

O'

Seja a circunferncia de centro O e raio r, ' a circunferncia qua passa por A, M, N; O' seu centro, B = AO ' e P AO tal que OPO ' = 90. Ento obviamente AP = PB. Agora como O est no eixo radical de e r2 r2 ' OA OB = OM ON = r 2 OB = . Chamando OA = d ,temos OB = , OA d 2 r d+ 2 2 d = d r logo P tal que OP = OA AP = d , e logo fixo e portanto 2 2d todos os centros O' esto na reta por P perpendicular a AO quando M varia.
EUREKA! N23, 2006

0 /

Sociedade Brasileira de Matemtica

Mas para todo O '' tq O '' PO = 90 se traarmos '' circunferncia com centro em O'' que passa por A ele tambm ir passar por B logo a potncia de O ser r 2 se {M '', N ''} = '' ento OM '' ON '' = r 2 e logo M'' e N'' so diametralmente opostos. Assim, o lugar geomtrico dos centros uma reta perpendicular a AO que passa pelo ponto P tal que OP =
PROBLEMA 3

OA2 r 2 na direo de A. (cqd). 2OA

Sejam n e k nmeros inteiros positivos tais que n mpar ou n e k so pares. Provar que existem inteiros a e b tais que mdc (a, n) = mdc(b, n) = 1 e k = a + b.
SOLUO DE RAFAEL DAIGO HIRAMA (CAMPINAS - SP)

Devemos achar ento, para n e k inteiros positivos, um a tal que mdc(a, n) = = mdc (k a, n) = 1 pois b = k a. mdc (k a, n) = mdc(a k, n), por definio. Ento devemos achar dois nmeros que distam k, primos com n. Deve ser til o Teorema Chins dos Restos. 2 ... ptt , com pi ' s primos e ,i s inteiros positivos e seja Fatorando: n = p11 p2 k i (mod pi ) com 0 i < pi Se trocarmos a k por c queremos que: c 0(mod pi ) e a = c + k 0 (mod pi) para todo i inteiro, 1 i t Se cada um dos pares de congruncia for obedecida, escolhendo as congruncias mdulo pi e usando o Teorema Chins dos Restos podemos achar tal c (pois cada par de pi ' s so primos entre si). Ento precisamos provar que existe possibilidade para a congruncia mdulo pi para o c. Queremos c 0(mod pi ) e c k (mod pi ) . Se pi for maior que 2, temos pelo menos 3 classes de congruncias distintas, e tirando essas duas: 0 e pi i , sobra pelo menos uma. Se pi 2, ento n par e pelo enunciado, k tambm e ento k 0(mod 2). Assim no so tirados duas classes de congruncia mdulo 2, e sim apenas a classe do 0. Podemos fazer ento c 1(mod 2). Ento o mtodo para construir a e b :
EUREKA! N23, 2006

Sociedade Brasileira de Matemtica

Fatora-se n e pega-se seus fatores primos p1 , p2 ,..., pt . Acha-se as classes de congruncia para cada pi , de forma que c 0(mod pi ) e c + k 0(mod pi ), que sempre existe como j provamos. Pode haver vrias, ento encolhe-se qualquer uma. Pelo Teorema Chins dos Restos, existe c que obedece tais congruncias, logo a = c e b = c + k , que obedecem a condio a + b = k e mdc(a, n) = mdc(b, n) = 1, pois pi | c pi | c = a e pi | c + k = b e, assim, a e n no tm fatores comuns, nem b e n os tm. Obs: Realmente, se n for par e k for mpar no d pois ou a ou b par e ento ou 2 | mdc (a, n) ou 2 | mdc (b, n).
PROBLEMA 4

Determinar todos os pares (a, b), onde a e b so nmeros inteiros positivos de dois dgitos cada um, tais que 100a + b e 201a + b so quadrados perfeitos de quatro dgitos.
SOLUO:

Devemos ter 100a + b = m 2 e 201a + b = n 2 , onde m e n so inteiros tais que 0 < m < n < 100. Da segue que n 2 m 2 = 101a. Como n 2 m 2 = (n m )(n + m ) mltiplo de 101, 0 < n m < 100 e n + m < 200, devemos ter n + m = 101, pois 101 primo, e 101 no divide n m. Da 101a 101 a . De = a, logo (101 m ) m = a, donde m = segue que n m = 2 n+m 101 a 2 100a + b = m 2 , segue que = 100a + b, e logo a 602a + 10201 = 4b. 2 2 ( ) = 602a + 10201 uma funo quadrtica cujo f a a 0 4 400. < b Como Temos
2

mnimo atingido para a =

602 = 301, f (a ) decrescente para 0 a 100. Como 2 f (16) = 825 > 400, f (17) = 256 e f (18) = 311 < 0, devemos ter a = 17, donde Com efeito,

101 17 256 = 64 . = 42, b= n = 101 m = 59 e 2 4 m 2 = 1764 = 100 17 + 64, e n 2 = 3481 = 201 17 + 64. m=
EUREKA! N23, 2006

Sociedade Brasileira de Matemtica

Assim, o nico par (a, b) que satisfaz as condies do enunciado (a, b) = (17, 64).
PROBLEMA 5

Dado um tringulo escaleno ABC, designam-se por A', B', C' os pontos de interseo das bissetrizes interiores dos ngulos A, B e C com os lados opostos, respectivamente. Sejam: A'' a interseo de BC com a mediatriz de AA', B'' a interseo de AC com a mediatriz de BB' e C'' a interseo de AB com a mediatriz de CC'. Provar que A'', B'' e C'' so colineares.
SOLUO:

Suponhamos a > b > c , sem perda de generalidade. A perpendicular a AA' passando E com a por A a bissetriz externa do ngulo A no tringulo ABC. Sua interseo A E A '+ A , pelo teorema de Tales. Por outro lado, se reta BC tal que A '' = 2 BC = a, AC = b e AB = c so os lados do tringulo ABC, pelo teorema das BA ' CA ' bB + cC , donde A ' = , e, pelo teorema das bissetrizes = bissetrizes temos c b b+c BE A CE A bB cC A= , donde E = externas, . bc c b 1 bB + cC bB cC b 2 B c 2 C + Assim, A '' = = 2 2 . 2 b+c b c b c
A

E A

A'' B

A'

Analogamente, B '' =

a 2 A c 2C a2 A b2 B '' . Assim, = C e a2 c2 a2 b2 b2 c2 a2 b2 2 2 2 2 2 2 '' '' = + B A C '' = tA''+ (1 t)C '', '' '' '', + = b c A a b C a c B donde ( ) ( ) ( ) a2 c2 a2 c2

EUREKA! N23, 2006

Sociedade Brasileira de Matemtica

com t =

b2 c2 , e logo B '' A '' C '', o que resolve o problema. a2 c2

PROBLEMA 6

Para um conjunto H de pontos no plano, diz-se que um ponto P do plano um ponto de corte de H, se existem quatro pontos distintos A, B, C e D em H tais que as retas AB e CD so distintas e se cortam em P. Dado um conjunto finito A0 de pontos no plano, constri-se uma sucesso de conjuntos A1 , A2 , A3 ,... da seguinte forma: para qualquer j 0, Aj +1 a unio de Aj com o conjunto de todos os pontos de corte de Aj . Demonstrar que se a unio de todos os conjuntos da sucesso um conjunto finito ento, para qualquer j 1, tem-se Aj = A1 .
SOLUO:
A

G D H B E

Observemos inicialmente que nenhum dos conjuntos Aj pode conter 4 pontos A, B, C, D tais que D interior ao tringulo ABC. De fato, se AD BC = {E}, BD AC = {F }, CD AB = {G} e BD EG = {H }, temos que ABE um tringulo estritamente contido em ABC e H interior a ABE. Repetindo indefinidamente esta construo, obtemos infinitos pontos na unio dos conjuntos An , absurdo.
D C F E B A

EUREKA! N23, 2006

Sociedade Brasileira de Matemtica

Suponha agora que algum dos conjuntos Aj contenha os vrtices de um quadriltero convexo ABCD. Se ABCD no um paralelogramo, podemos supor que AB no paralelo a CD (ver figura acima). Prolongando AB e CD obtemos um ponto de interseo E. As diagonais AC e BD se intersectam num ponto F interior ao tringulo ADE, absurdo pelo caso anterior. Considere agora o fecho convexo C de A0 . Se C um segmento de reta, temos An = A1 = A0 , n <. Se C um tringulo de vrtices A, B e C, estes pontos pertencem a A0 , que, como vimos, no pode conter nenhum ponto no interior de C. Se A0 contm pontos no interior de dois lados de C, digamos D em AB e E em AC, BE e CD se intersectam em F A1 , que interior a C, absurdo.
A

D F B

Assim, A0 s pode conter os vrtices A, B e C e, eventualmente, alguns pontos em um dos lados do tringulo, digamos AB. fcil ver que nesse caso ainda temos An = A1 = A0 , n <. Caso C seja um quadriltero, deve ser um paralelogramo ABCD. Se A0 contm algum ponto no interior de algum lado do paralelogramo, digamos E em AB, conter os vrtices do quadriltero convexo EBCD, que no um paralelogramo, absurdo. Se algum Aj contm algum ponto do interior de C, este no pode ser interior aos tringulos ABC e ACD, logo deve pertencer diagonal AC, e, analogamente, deve pertencer diagonal BD, e portanto deve ser o centro O de ABCD. Temos ento dois casos: i) Se A0 = { A, B, C , D} temos A1 = { A, B, C , D, O} e An = A1 , n 1. ii) Se A0 = { A, B, C , D, O}, temos An = A1 = A0 , n <. Finalmente, se C tem pelo menos 5 vrtices consecutivos A, B, C, D, E, devemos ter CD e DE paralelos a AB, pois ABCD e ABDE so quadrilteros convexos com vrtices em A0 , mas isso um absurdo, e o problema est resolvido.

EUREKA! N23, 2006

Sociedade Brasileira de Matemtica

PESSOAS CONFIVEIS E NO-CONFIVEIS


Svetoslav Savchev, Bulgria Traduo: Cesar Ryudi Kawakami e Rafael Morioka Oda So Paulo SP

Nvel Avanado
Em um grupo de n 3 pessoas, algumas so confiveis e as outras no. Uma pessoa que confivel sempre diz a verdade, enquanto uma no-confivel s vezes diz a verdade e s vezes mente. Sabemos que o nmero de pessoas no n confiveis no mximo k, onde k um inteiro que satisfaz 0 < k < . Um 2 observador quer identificar quem confivel e quem no . Ele pode fazer perguntar a qualquer pessoa do grupo sobre a confiabilidade de alguma outra. Qual o nmero mnimo de perguntas necessrias para descobrir o que deseja? Essa pergunta originou-se de uma questo da olimpada de Moscou de 1978, que perguntava sobre o nmero mximo e proposto pelo proeminente Sergei Konyagin. Depois do concurso, muitos matemticos, como Schlosman, Ruzsa e Galvin, acharam a idia to atraente que publicaram outras variantes do problema. Um possvel nmero suficiente foi encontrado e o necessrio, que era mais difcil, foi sendo gradualmente melhorado. Finalmente, o problema foi completamente estabelecido por Pavel Blecher, que j havia sido premiado na prpria olimpada de Moscou. Ns seguimos sua idia, numa verso um pouco diferente do problema. O menor nmero de questes n + k 1. Ns comeamos provando que esse nmero suficiente, usando induo em n 3. 3 Para n = 3 ns temos 0 < k < ento k = 1, ou seja, no mximo uma pessoa 2 no confivel em um grupo de 3. Numere as pessoas e pergunte para a pessoa 2 se a 1 confivel. Se a resposta for sim, ento 1 de fato confivel (caso contrrio 1 e 2 no seriam confiveis). Sabendo que 1 confivel, basta perguntar a ele sobre 2 e 3. Portanto temos que 3 = 3 + 1 1 = n + k 1 questes so suficientes. Se a pessoa 2 responder no, ento temos que ou 1 ou 2 no-confivel (seno a resposta teria sido sim). Portanto temos que 3 confivel, e basta perguntar a ele sobre 1 e 2. Novamente, trs perguntas bastam. Assuma que m + k 1 questes so suficientes para todo grupo de tamanho m , m 3 e menor que n, e considere um grupo de n pessoas que satisfaz as
EUREKA! N23, 2006

10

Sociedade Brasileira de Matemtica

condies dadas. Comece perguntando sucessivamente s pessoas 2,3...,n sobre a pessoa 1. Pare de perguntar assim que: i) O nmero de respostas sim se iguale a k. ii) O nmero de respostas no exceda o nmero de respostas sim. Note que (i) ou (ii) ir necessariamente ocorrer devido condio k < n / 2. No caso (i), a pessoa 1 confivel (ou ento ele mais as k pessoas que respoderam sim seriam no-confiveis, absurdo). Suponha que m pessoas tenham respondido no at aquele momento; todas elas so no confiveis. Agora basta perguntar a 1 sobre todas as pessoas no grupo, exceto ele mesmo e as m pessoas mencionadas. Isto leva n m 1 questes, ento (k + m) + (n m + 1) = n + k 1 questes so suficientes. No caso (ii), existe um nmero m tal que as pessoas 2, 3,..., 2m so perguntadas e temos m respostas no e m 1 respostas sim. Observe que no importa se 1 confivel ou no, pelo menos m pessoas do grupo G1 ={1,2,...,2m} so no confiveis. Se 1 confivel, ento temos que as m que responderam no so no-confiveis. Se 1 no confivel, ento ele e as m 1 pessoas que responderam sim so no-confiveis. Conclumos portanto que m k, j que existem apenas k pessoas no-confiveis, e assim temos que o grupo G 2 ao qual pertencem as n 2m pessoas restantes no vazio, uma vez que k < n / 2. Alm do mais, existem no mximo k m pessoas que no so confiveis em G 2 , o que menor que (n 2m)/2. Se m < k ento 1 k m < (n 2m)/2, n 2m n 2(k 1) 3, e logo a hiptese de induo se aplica ao grupo G 2 . Ento (n 2m) + (k m) 1 questes so suficientes para saber quem o que no grupo. A mesma concluso trivial para m = k, caso em que G 2 consiste de n 2k pessoas, todas confiveis, e portanto nenhuma pergunta necessria. Agora escolha uma pessoa confivel A em G 2 (existe sempre pelo menos uma) e pergunte-a sobre 1. Dependendo da resposta, pergunte a A sobre que disseram a verdade sobre 1. Para ser mais preciso, se 1 confivel (no-confivel), pergunte a aqueles que responderam sim (no). No mximo 1 + m questes so necessrias nessa ltima etapa. No h mais questes necessrias pois j sabemos que aqueles que mentiram sobre 1 no so confiveis. Um total de (2m 1) + (n 2m) + (k m) 1 + (1 + m) = n + k 1 questes so suficientes novamente. A cota superior est provada. A cota inferior consideravelmente mais difcil: mostrar que n + k 1 perguntas so tambm necessrias. Considere uma
EUREKA! N23, 2006

11

Sociedade Brasileira de Matemtica

seqncia arbitrria de n + k 2 questes ( p1 , q1 ),( p 2 , q 2 ),...,( p n + k 2 , q n + k 2 ); (1) onde a questo i pergunta pessoa pi sobre a pessoa qi . Arbitrria em particular quer dizer que cada questo de (1) pode depender das respostas s perguntas anteriores. Definimos tambm uma seqncia r1 , r2 ,, rn + k 2 de respostas, tal que, para cada i , a resposta ri depende apenas das questes ( p1 , q1 ), ( p 2 , q 2 ),...,( pi , qi ). Agora, sero apresentadas duas configuraes diferentes de pessoas confiveis e no-confiveis num grupo de n , e ambas consistentes no s com as respostas r1 , r2 ,, rn + k 2 mas tambm com o limite de k pessoas noconfiveis. Ento a seqncia de questes (1) falhar em diferenciar essas duas configuraes, e portanto no h n + k 2 questes que podem resolver o problema em geral. Ns definimos a resposta ri em dois estgios: o primeiro, quando as primeiras k 1 questes so feitas, e a segunda, envolvendo as n 1 questes restantes. Todas as respostas na primeira parte sero no. Depois que as k 1 perguntas acabam, represente como pontos todas as pessoas p1 ,, p k 1 ,

q1 ,, q k 1 envolvidas nelas e ligue pi a qi por uma aresta, i = 1,..., k 1. O grafo obtido ser usado a seguir. Chame de G1 ,..., Gm suas componentes conexas, com conjuntos de vrtices V1 ,..., Vm respectivamente. Ento cada questo ( pi , qi ) na
primeira parte (ou seja, i = 1,..., k 1) envolve duas diferentes pessoas do mesmo grupo V j (j = 1,..., m). Se e1 , e2 ,, em so os nmeros de arestas em G1 ,..., Gm , temos que e1 + e2 ++ em = k 1 (2)

Finalmente, denote por W o conjunto de pessoas no envolvidas no primeiro conjunto de perguntas, i.e. o complemento de V1 V2 ... Vm em relao ao conjunto de todas as n pessoas. As questes ( p k , q k ),...,( p n + k 2 , q n + k 2 ) so feitas na segunda etapa. De acordo com elas, ns escolhemos um representante de cada grupo V j (j = 1,..., m) mas podem haver dois diferentes tipos de representantes dependendo do grupo. Se forem feitas perguntas sobre cada pessoa em V j na segunda parte, dizemos que V j um grupo com um verdadeiro representante escolhido da seguinte
EUREKA! N23, 2006

12

Sociedade Brasileira de Matemtica

maneira: ache o menor i tal que V j est contido na seqncia q k ,..., qi ; ento qi o (verdadeiro) representante de V j . Se h pessoas em V j sobre as quais no foram feitas perguntas na segunda etapa, escolha uma qualquer dentre elas e chame-a de falsa representante. No momento em que a questo ( pi , qi ) feita, est claro se qi ou no um representante verdadeiro de algum V j . Conseqentemente as prximas questes na segunda fase so tais que a resposta ri depende somente das questes ( p1 , q1 ), ( p 2 , q 2 ),...,( pi , qi ): Para cada i = k,..., n + k 2, definimos a resposta ri como sim se qi pertence a W ou se qi um verdadeiro representante de um grupo V j ; caso contrrio defina ri como no. Agora considere uma configurao S de n pessoas onde a pessoa p : confivel se p pertence a W ou se p um representante de algum grupo V j (verdadeiro ou falso); no-confivel em qualquer outra hiptese. Vamos mostrar que S tem as seguintes propriedades: i) S consistente com todas as respostas r1 , r2 ,, rn + k 2 , i.e. todas as respostas dadas por pessoas confiveis em S so verdadeiras; ii) No h mais que k 1 pessoas no-confiveis em S. Para provar (i), observe que todas as respostas da segunda etapa rk ,, rn + k 2 so verdadeiras, pela sua definio e pela definio de pessoas confiveis e noconfiveis (usando o fato de que no perguntamos sobre nenhum representante falso na segunda parte). Ento ns precisamos apenas garantir que todas as pessoas confiveis deram respostas verdadeiras na primeira etapa. Escolha uma pessoa confivel p. Se p pertence a W ento no houve perguntas a p nem sobre p no primeiro estgio. Suponha que p representante de algum grupo V j . Ento as possveis perguntas que lhe podem ter sido feitas na primeira etapa so sobre as pessoas de seu prprio grupo que no ele prprio. Todas essas pessoas so noconfiveis, e todas as respostas da primeira parte foram no, portanto p disse a verdade. Para verificar (ii), lembre-se que, por construo, as pessoas no-confiveis esto todas nos grupos V1 ,..., Vm . Exatamente uma pessoa em cada grupo confivel seu representante. Uma vez que o nmero de pessoas no-confiveis em V j
EUREKA! N23, 2006

13

Sociedade Brasileira de Matemtica

V j 1, j = 1,..., m, e o nmero total u de pessoas no-confiveis igual. Agora ns usamos o fato de que V j o conjunto de vrtices de um grafo conexo: em tal grafo, o nmero de arestas pelo menos o nmero de vrtices menos um. Deste modo e j V j 1 para j = 1,..., m. De (2) temos que u =

( V
m j =1

1 e1 + e2 + ... + em = k 1 , e logo (ii) verdade.

Ter no mximo k 1 pessoas no-confiveis em S ser usado para obter uma outra configurao S consistente com as respostas r1 , r2 ,, rn + k 2 e tendo no mximo k pessoas no-confiveis. Descreveremos agora esta construo. Se algum grupo V j tem algum representante falso q, ele confivel em S. Defina q como um no-confivel em S e deixe todas as outras pessoas como eram em S. Como q no-confivel na nova configurao, suas respostas so irrelevantes para a consistncia com r1 , r2 ,, rn + k 2 . Portanto S consistente com as respostas se, e somente se, cada pessoa confivel de S disse a verdade quando perguntada sobre q. Mas no h pessoa confivel que foi perguntada sobre q. De fato, no perguntaram sobre sua confiabilidade na segunda etapa pela definio de falso representante. E se alguma pessoa foi perguntada sobre q na primeira fase, ela pertence ao mesmo grupo V j que q, e portanto no representante do grupo, o que significa que no confivel em ambas as configuraes. Assim, S satisfaz as condies. Se no h representantes falsos em S ento foram feitas perguntas sobre todas as pessoas em V1 ,..., Vm na segunda etapa. Porm, n 1 questes foram feitas no total durante ela, logo no houve perguntas sobre pelo menos uma pessoa q dentre as n. Claramente, q pertence a W. Defina q como no-confivel em S e deixe todos os restantes da mesma maneira que em S. Ningum perguntou sobre q (nem na primeira nem na segunda etapa), e portanto a nova configurao S satisfaz novamente todas as condies, e a prova est completa.

EUREKA! N23, 2006

14

Sociedade Brasileira de Matemtica

O TEOREMA DE STOLZ
Digo Veloso Ucha & Renato Purita Paes Leme Nvel Avanado
INTRODUO

Apresentaremos um pequeno resultado sobre limites com uma srie de conseqncias interessantes e, em seguida, alguns problemas que podem ser resolvidos usando-o. Usando a demontrao do teorema como pretexto, vamos mostrar como a intuio geomtrica pode nos dar boas indicaes sobre o caminho a seguir. Isso pode ser resumido na velha mxima: Pense geometricamente, prove algebricamente.
Teorema (Stolz)

Seja {xn} uma seqncia estritamente crescente com lim x n = + , e {yn} uma seqncia arbitrria. Se lim
Prova

y n +1 y n y = a ento: lim n = a . x n +1 x n xn

Podemos pensar em ( x n , y n ) como uma seqncia de pontos do plano pertencentes a uma curva da forma: y = f ( x ) , j que xn crescente. Assim,

y n +1 y n x n +1 x n

representa a razo incremental entre dois pontos consecutivos. medida que n aumenta, essa razo vai se tornando arbitrariamente prxima do valor constante a, ou seja, para x grande (xn com n grande) a curva f praticamente uma reta. Isto , se y = f ( x ) tiver uma assntota, ela deve ser uma reta da forma y = ax + b para algum b. Como para n grande y n ax n + b , quando n ,

y n ax n + b b =a+ a xn xn xn
Naturalmente, isso no constitui uma prova (ainda, veremos que nem sempre os pontos ( x n , y n ) se aproximam de uma reta quando n ) mas isso serve como uma boa orientao para uma prova formal. Vejamos:

EUREKA! N23, 2006

15

Sociedade Brasileira de Matemtica

(x7,y7) (x6,y6) (x5,y5) (x4,y4) (x3,y3) (x2,y2) (x1,y1)

Pela definio de limite, como lim

y n +1 y n = a: x n +1 x n y n +1 y n a < x n +1 x n

> 0, n0 tal que n > n0


ou seja, a razo incremental

y n +1 y n est entre a e a + . Geometricamente: x n +1 x n


y y n0 (a )( x xn )
0

y
(xn0,yn0)

y n0

(a

)( x

x n0 )

A geometria acima nos diz que para n > n0 , todo ( x n , y n ) est entre essas retas, ou seja:

y n0 + k y n0 x n0 + k x n0

a < , k > 0

EUREKA! N23, 2006

16

Sociedade Brasileira de Matemtica

Precisamos provar algebricamente o que a geometria nos mostra. Mas isso simples:

(a ) ( x n0 +1 x n0 ) < y n0 +1 y n0 < (a + ) ( x n0 +1 x n0 ) (a ) ( x n0 + 2 x n0 +1 ) < y n0 + 2 y n0 +1 < (a + ) ( x n0 + 2 x n0 +1 )


...

(a ) ( x n0 + k x n0 + k 1 ) < y n0 + k y n0 + k 1 < (a + ) ( x n0 + k x n0 + k 1 )
Somando as expresses acima, temos:

(a ) ( x n0 + k x n0 ) < y n0 + k y n0 < (a + ) ( x n0 + k x n0 )
Ou seja,

y n0 + k y n0 x n0 + k x n0

a < .

Dividindo tudo por x n0 + k e fazendo k tender ao infinito, temos:

y n0 + k x n0 + k 1

y n0 x n0 + k a < y n0 + k x n0 + k x n0 x n0 + k y n0 x n0 + k y n0 x n0 + k xn a1 0 xn +k 0 + a x n0 x n0 + k x < 1 n0 x n0 + k < 2

x n0 x n0 + k

y n0 + k x n0 + k

a < 1

para k suficientemente grande. Logo: lim

yn =a xn

A prova desse teorema composta de algumas poucas linhas. Demoramos mais, pois perdemos tempo discutindo a sua essncia geomtrica. Comentamos que nem sempre yn se aproxima de uma reta axn + b, ou seja, que nem sempre a seqncia bn = yn axn converge. Por exemplo: xn = n e yn = 2n + log(n)

EUREKA! N23, 2006

17

Sociedade Brasileira de Matemtica

No entanto, nas condies do teorema, bn uma seqncia cujas razes incrementais em relao a xn tendem a zero quando n tende ao infinito, isto : lim como o caso de bn = log(n).

bn +1 bn = 0, x n +1 x n

Corolrio

Dada uma seqncia {an} tal que lim a n = a , o limite da mdia artimtica dos n primeiros termos da seqncia tambm a. Ou seja:

lim
Prova

a1 + ... + a n = a. n

Basta fazer y n =

a
i =1

e x n = n . Como {xn} estritamente crescente e tende ao

infinito, podemos aplicar o Teorema de Stolz : lim


Problema Resolvido I

a1 + ... + a n a = lim n +1 = a [ 1 n

Mostre que se lim a n = a ento lim

1 n ak = a. ln n k =1 k

Soluo : Fazendo y n =
n

ak e x n = ln n e aplicando o Teorema de Stolz k =1 k


n +1 1 ln 1 + n +1

an +1 ak 1 n +1 = lim = lim lim + ln n k =1 k ln( n 1) ln n

an +1

=a.

Problema Resolvido II

Dado um nmero real p > 0, calcule lim

1 p + 2 p + ... + n p . n n p +1

Soluo: Faamos:
EUREKA! N23, 2006

y n = 1 p + 2 p + ... + n p e x n = n p +1 que estritamente

18

Sociedade Brasileira de Matemtica

crescente e tende ao infinito. Logo podemos aplicar o Teorema de Stolz:

y n +1 y n (n + 1) p = = x n +1 x n (n + 1) p +1 n p +1 1 p +1 1 + 1 n
Tomando x = 1 / n e, usando que = p + 1 (pois a derivada de x + (1 + x ) p 1 em x = 0 ) para calcular o limite quando x 0 , temos:
x 0

1 1 1 + n n

(1 + x ) lim

p +1

lim

x (1 + x ) p = x 0 (1 + x) p +1 1

1 lim
x 0

(1 + x )

p +1

1 p +1

Logo: lim

y n +1 y n y 1 1 , assim: lim n = [ = x n +1 x n 1 + p xn 1 + p

Problema Resolvido III

(Schweitzer Competition - alterada) Dada a seqncia definida recursivamente por: 0 < a1 < 1 e a n +1 = a n (1 a n ) , para n > 1 , prove que: (a) lim n a n = 1 (b) lim

n (1 na n ) =1 ln n n . Para isso, devemos 1 / an

Soluo: a) Queremos usar o Teorema de Stolz em lim

mostrar que 1 / a n estritamente crescente e tende ao infinito, o que equivale a mostrar que a n estritamente decrescente e tende a 0+. Por induo:

0 < a 2 = a1 (1 a1 ) < a1 < 1 pois 0 < a1 < 1


Assumindo como hiptese de induo que: 0 < ai +1 < ai < 1 , temos que:

0 < ai + 2 = ai +1 (1 ai +1 ) < ai +1 < 1 , pois 0 < ai +1 < 1


EUREKA! N23, 2006

19

Sociedade Brasileira de Matemtica

Como a n estritamente decrescente e limitada, ela converge. Logo a = lim a n e:

a = a (1 a ) a = 0 . Aplicando o Teorema de Stolz:


lim

a [a (1 an ) ] a a a 2 (1 an ) ( n + 1) n = lim n n +1 = lim n n = lim n = 2 1 1 (1 ) a a a a a a [ ] + 1 n n n n n n an +1 an

= lim(1 an ) = 1. b) Aplicando um truque algbrico:

1 1 1 n na n n na n n (1 na n ) n (1 na n ) an = lim = lim lim na n , ln n ln n ln n ln n 1 n an se lim existir. Mostremos que ele de fato existe e vale 1, por Stolz: ln n
1 1 1 1 1 1 n 1 n 1 1 n a an an +1 an = an +1 an = = n +1 +1 n ln(n + 1) ln n n +1 1 ln ln 1 + n n +1 a an +1 nan 1 n n a a 1 an = n +1 n n +1 = 1 quando n + [ n +1 1 1 ln 1 + ln 1 + n +1 n +1 A seguir apresentamos alguns problemas (em ordem crescente de dificuldade) que so resolvidos com a aplicao do Teorema de Stolz. 1 - Seja {xn} uma seqncia de termos positivos tais que x n = e lim ento lim y1 + ... + yn = a. x1 + ... + xn

yn =a xn

2 Seja {xn} uma seqncia de termos positivos tais que lim x n = a . Mostre que lim n x1 x2 ...xn = a. Sugesto: Use o corolrio aps o Teorema de Stolz.
EUREKA! N23, 2006

20

Sociedade Brasileira de Matemtica

3 Para uma seqncia de termos positivos {a n } mostre que se lim

a n +1 = a ento an

lim n a n = a .
4 Seja k um inteiro fixo arbitrrio maior que 1. Determine lim n n .
n j 5 Calcule lim n 1 + . n j =1

nk

6 Prove que se {an} uma seqncia que converge para a ento :

na1 + (n 1)a 2 + ... + 1a n a = . 2 n2 a a n 1 = 0. 7 Prove que se lim( a n a n 2 ) = 0 ento lim n n n 1 k k k , k natural fixo. 8 Calcule lim k (1 + 2 + ... + n ) k + 1 n lim
9 (OBM adaptado) Dada a seqncia {a n } definida recursivamente por a1 = 3 e a n +1 = a n 2 . Prove que lim
2

ln(ln a n ) = ln 2 . n

10 Para uma seqncia {a n } , considere a seqncia { An } , de mdias arirmticas, ou seja, An =

a1 + a 2 + ... + a n . Mostre que se lim An = A , ento: n 1 n ak lim = A. ln n k =1 k

Sugesto: Escreva a k em funo de alguns Ai . 11 Mostre que, se para a seqncia de termos positivos {a n } , o limite

EUREKA! N23, 2006

21

Sociedade Brasileira de Matemtica

a n +1 lim n 1 a n
existe ento o limite

lim

ln a n ln n

tambm existe e os dois so iguais. Ainda, se o primeiro limite infinito, o segundo tambm o . 12 Seja a1 = 1 e a n +1 = a n +

a
k =1

para n 1 , prove que: lim


k

an 2 ln n

=1

2 Sugesto: Estude a n .

REFERNCIAS

[ 1 ] KACZOR, W.J e NOWAK, M.T Problems in Mathematical Analysis I Real Numbers, Sequences and Series American Mathematical Society [ 2 ] LIMA, E. L. Curso de Anlise, Volume I Projeto Euclides IMPA [ 3 ] APOSTOL, T Calculus, Volume I Addison-Weasley
Digo Veloso Ucha e Renato Purita Paes Leme so alunos de Engenharia que gostam de matemtica. Cursam o 4 ano do INSTITUTO MILITAR DE ENGENHARIA e estudam no INSTITUTO DE MATEMTICA PURA E APLICADA. Os dois agradecem ao professor Carlos Gustavo T. A. Moreira pela reviso desse artigo.

EUREKA! N23, 2006

22

Sociedade Brasileira de Matemtica

CONTAS COM DESIGUALDADES


Mrcio Afonso Assad Cohen & Rodrigo Villard Milet Nvel Avanado
Vamos discutir aqui uma notao prtica para lidar com funes simtricas e estudar a utilizao de duas poderosas ferramentas, as desigualdades de Muirhead, mais conhecida como bunching e a de Schur. Por simplicidade de notao, os resultados sero mostrados para trs variveis. O leitor no ter dificuldade em generalizar esses resultados para n variveis (quando necessrio, sero fornecidas dicas para essa generalizao).
1. EXPRESSES SIMTRICAS

Uma funo f ( x, y , z ) simtrica quando se tem, para todo x, y, z:

f ( x, y , z ) = f ( x, z , y ) = f ( y , x, z ) = f ( y , z , x ) = f ( z , x, y ) = f ( z , y , x)
(isto , trocar uma varivel com outra no altera a expresso). Dada uma funo qualquer P ( x, y, z ) , definimos:
sym

P( x, y, z) = P( x, y, z) + P( x, z, y) + P( y, x, z) + P( y, z, x) + P(z, x, y) + P(z, y, x) . x
sym 2

Exemplo 1. De acordo com a definio, verifica-se que:

y = x2 y + x2 z + y2 x + y2 z + z 2 x + z 2 y ;

x
sym

= 2 (x3 + y3 + z 3 ) ;

xyz = 6 xyz
sym

(reflita um pouco para entender os coeficientes 2 e 6 dos exemplos anteriores!).


1.1. Propriedades 1.
sym

f uma funo simtrica, i.e, f ( x, y, z ) = f ( x, z, y) = ...


sym sym
sym sym

2. Se ( x, y , z ) uma funo simtrica, ento (x, y, z) f (x, y, z) =(x, y, z) f (x, y, z)

(porque ( x, y , z ) uma constante para esse somatrio).


Exemplo 2. (Quadrado e Cubo da soma) Sendo s = x + y + z , expresse 2s e 2s
2 3

como somas simtricas:


EUREKA! N23, 2006

23

Sociedade Brasileira de Matemtica

Soluo:

2s = x 2 s 2 = x ( x + y + z ) = ( x 2 + xy + xz ) = ( x 2 + 2 xy )
sym sym sym sym 3 2 2 3 2 2 sym sym

2s = 2s s = (x + 2xy) (x + y + z) = (x + x y + x z + 2x 2 y + 2xy2 + 2xyz) 2s = (x + 6x y + 2xyz)


3 3 2 sym

Exemplo 3. Sendo x, y, z > 0, mostre que:

9 x y z + + ( x + y )( x + z ) ( y + x)( y + z ) ( z + x)( z + y ) 4( x + y + z )
Soluo: Usando a notao de simetria, podemos reescrever o problema como:

1 9 x 2 sym ( x + y )( x + z ) 4( x + y + z )

(Se voc no entendeu o fator ,

volte ao exemplo 1). Multiplicando tudo pelo mmc ( x + y )( x + z )( y + z )( x + y + z ) :

2 x( y + z )( x + y + z ) 9( x + y )( x + z )( y + z )
sym

Agora observe que; chamando o lado esquerdo de LE e o direito de LD: (i) LD = 9 (x + y)(x + z)(y + z) = 9 (x2 y + x 2 z + ...+ xyz + xyz) = 9

1 x 2 y + xyz 3 sym sym

(para evitar erros, confira se a expresso est verdadeira para x = y = z = 1, 1 = 6 nesse caso). lembrando que

sym

(ii) LE= 2

(xy+ xz)(x + y + z) = 2 (x y + xy + xyz+ x z + xyz+ xz ) = (8x y + 4xyz)


2 2 2 2 2 sym sym sym

Portanto, LE LD

(8x
sym

y + 4 xyz) (9 x y + 3xyz) x 2 y xyz .


2 sym sym sym

A ltima inequao consequncia direta da desigualdade das mdias:

x 2 y + x 2 z + y 2 x + y 2 z + z 2 x + z 2 y 6 xyz .
2. MUIRHEAD (vulgo bunching)

Dadas duas seqncias no-crescentes (a1 , a2 , a3 ) e (b1 , b2 , b3 ) , diz-se que a majora b quando: a1 b1 ; a1 + a2 b1 + b2 ; e a1 + a2 + a3 = b1 + b2 + b3
EUREKA! N23, 2006

24

Sociedade Brasileira de Matemtica

Nesse caso, escreve-se [ a1 , a2 , a3 ] [b1 , b2 , b3 ] e se x, y, z so reais positivos temos:

x
sym

a1

y a2 z a3 xb1 y b2 z b3 .
sym

Demonstrao: Lema (smoothing): Se em

x
sym

u1

y u2 z u 3 substituirmos dois expoentes u i , u j (i < j)

por vi , v j com u i + u j = vi + v j e ui vi v j u j , o valor da expresso diminui (ou no muda). (isto , manter a soma constante e diminuir a distncia entre dois expoentes no aumenta a expresso)
Demonstrao do Lema:

(x
sym

S.p.g, suponha u1 = m + R , u 2 = m R, v1 = m + r , v 2 = m r :
u1 u u v v u y 2z 3 x1y 2z 3)=

= =

1 ( x m + R y m R z u3 x m + r y m r z u 3 + x m R y m + R z u 3 x m r y m + r z u 3 ) = 2 sym 1 z u3 x m y m ( x R y R + x R y R x r y r x r y r ) 2 sym

A ltima expresso no-negativa, pois para a positivo fixo (no caso a = x/y), a funo f (t ) = a t + a t crescente:

f (R) f (r) = a R ar +

1 1 1 r = (aR ar ) R r (a R ar ) = aRr (a R ar )(aR ar 1) R a a a a

Se a > 1 e R > r > 0, essa expresso obviamente positiva. Se 0 < a < 1, tambm (troque a por 1/a).
Demonstrao do teorema:

Como [a1 , a 2 , a 3 ] [b1 , b2 , b3 ] , se as seqncias no forem iguais, adotamos o seguinte procedimento: Seja i o maior ndice tal que ai > bi , e j o menor ndice maior do que i tal que a j < b j . (o qual existe, pois

a = b e a b , donde a > b
k k k <i k k <i k k i k k i

).

Se i < k < j , ak = bk . Seja r = min{ai bi , b j a j } > 0. Podemos fazer o troca de ai por ai r e de a j por a j + r, de modo que o nmero de ndices s tais que
EUREKA! N23, 2006

25

Sociedade Brasileira de Matemtica

as = bs aumenta de pelo menos uma unidade. A ordem dos ai preservada, pois ai r bi b j a j + r e, se i < k < j, ai r bi bk = ak . Como a soma simtrica

x
sym

a1

y a2 z a3

nunca aumenta (pelo lema), repetimos o procedimento at que

x
sym

as = bs para todo s, o que mostra que, no incio, tnhamos


a1

y a2 z a3 xb1 y b2 z b3 .
sym

Obs. A demonstrao do caso com n variveis anloga. (A demonstrao fornece uma idia intuitiva para o conceito de majorar. Uma seqncia na qual os expoentes esto prximos uns dos outros sempre majorada por uma com os termos mais espalhados!)
Exemplo 4. (Desigualdade das mdias) Dados a1 , a 2 ,..., a n reais positivos, mostre que

a1 + a 2 + ... + a n n a1 a 2 ...a n n
Soluo: Vamos usar a verso de n variveis de bunching:

1 1 1 1 [1,0,0,...,0] > , , ,... n n n n a1 a11 n a21 n ... an1 n


sym sym

(n 1)! (a1 + a2 + ... + an ) n !

a1a2 ...an MA MG.

Exemplo 5. (Vingana olmpica) Dados a, b, c, x reais positivos, prove que

a x+2 + 1 b x+2 + 1 c x+2 + 1 + + 3 a x bc + 1 b x ac + 1 c x ab + 1


Soluo: Na notao simtrica, o problema pode ser reescrito como:

1 a x+ 2 + 1 x 3 2 sym a bc + 1

(a
sym

x+2

+ 1)(b x ac + 1)(c x ab + 1) 6(a x bc + 1)(b x ac + 1)(c x ab + 1)

Desenvolvendo cada lado:

EUREKA! N23, 2006

26

Sociedade Brasileira de Matemtica

LE = (a x + 4 b x +1 c x +1 + 2a x + 3 b x c + a x + 2 + b x +1 c x +1 a 2 + 2a x bc + 1)
sym

LD = (a x + 2 b x + 2 c x + 2 + 3a x +1b x +1 c 2 + 3a x bc + 1)
sym

Temos LE LD j que, usando bunching trs vezes, podemos escrever:


sym sym sym

LE LD= (ax+4bx+1cx+1 ax+2bx+2cx+2 ) + 2 (ax+3bxc ax+1bx+1c2 ) + (ax+2 axbc) 0.

Exemplo 6. (Torneio das cidades e Japo) Mostre que sendo a, b, c positivos de

produto 1 tem-se

1 1 1 + + 1 a + b +1 b + c +1 a + c +1
Soluo: Inicialmente, vamos nos livrar da restrio, homogeneizando a

inequao:
1 1 1 1 + + 3 3 3 3 a + b + abc b + c + abc a + c + abc abc

Para evitar as razes, faa x = 3 a , y = 3 b , z = 3 c . xyz 2 3 3 sym x + y + xyz

xyz( y
sym

+ z 3 + xyz)(x3 + z 3 + xyz) 2(x 3 + y 3 + xyz)( y 3 + z 3 + xyz)(x 3 + z 3 + xyz)

Desenvolvendo,

(x3 + z 3 + xyz)(y3 + z 3 + xyz) = z 6 + x3 z 3 + y 3 z 3 + x3 y3 + x 4 yz + 2z 4 yx + y 4 xz + x2 y 2 z 2


(*) Logo, LE =

(x
sym

yz + 4 x 5 y 2 z 2 + 3x 4 y 4 z + x 3 y 3 z 3 )

Multiplicando (*) por x 3 + y 3 + xyz e escrevendo na notao simtrica:

LD = ( x 7 yz + 2 x 6 y 3 + 2 x 5 y 2 z 2 + 3x 4 y 4 z + x 3 y 3 z 3 )
sym

(No incio, difcil olhar direto para os coeficientes. Nesse caso, no perca tempo. Faa a conta com os 27 termos no LD e s depois escreva na notao de simetria!) Cancelando os termos comuns, a desigualdade portanto equivalente a x6 y3 x5 y 2 z 2 ,

xym

sym

EUREKA! N23, 2006

27

Sociedade Brasileira de Matemtica

que segue por bunching j que [6,3,0] [5,2,2] .


3. SCHUR (para trs variveis)

Se x, y, z, r so positivos, tem-se:

x ( x y )( x z ) 0 .
r sym

Demonstrao: Como a expresso simtrica, podemos supor, sem perda de generalidade, x y z , de forma que:

x r ( x y )( x z ) + y r ( y x)( y z ) = ( x y )( x r ( x z ) y r ( y z )) 0 ,
pois x z y z e x r y r . Para
r

completar

demonstrao,

basta

somar

essa

desigualdade

com

z ( z x)( z y ) 0 .
Quando r = 1, obtemos a principal variao da desigualdade de Schur:

(x
sym

+ xyz 2 x 2 y ) 0.

(Schur muito til quando o termo mais distribudo (ex: xyz) deve ser maior que algum outro termo).
Exemplo 7. (Ir) Dados x, y, z reais positivos, mostre que

1 1 1 ( xy + yz + xz ) ( x + y ) 2 + ( y + z ) 2 + ( x + z) 2
Soluo:

9 4.

Tirando mmc queremos provar que:

2 ( xy + yz + xz )( y + z ) 2 ( x + z ) 2 9 [( x + y )( x + z )( y + z )]
sym

Fazendo as contas, ( x + y )( x + z )( y + z ) = ( x + y )( z 2 + xy + xz + yz ) = x 2 y + x 2 z + y 2 x + y 2 z + z 2 x + z 2 y + 2 xyz = s

1 2 2 + 9 s xyz LD = 9 [( x + y )( x + z )( y + z )] = 9 s 2 = 9s x y sym 3 sym


EUREKA! N23, 2006

28

Sociedade Brasileira de Matemtica

LD = 9 ( x 4 y 2 + x 3 y 3 + x 4 yz + x 2 y 3 z + x 3 yz 2 + x 2 y 2 z 2 + 2 x 3 y 2 z ) +
sym

+3 (6 x 3 y 2 z + 2 x 2 y 2 z 2 )

LD = (9 x 4 y 2 + 9 x 4 yz + 9 x 3 y 3 + 54 x 3 y 2 z + 15 x 2 y 2 z 2 )
sym

sym

No lado esquerdo, pondo = xy + xz + yz :

LE = 2 [( y + z)(x + z)] = 2 (z 2 + )2 = 2 (6 3 + 2 2 x 2 + x 4 )
2 sym sym sym sym

Agora, usando o exemplo 2:

6 3 = 3 ( x 3 y 3 + 6 x 3 y 2 z + 2 x 2 y 2 z 2 )
sym

2 = ( x 2 y 2 + 2 x 2 yz );
2 sym 2

x = (x y
sym sym sym

2 2

+ 2x2 yz) 2 (x2 + y2 + z 2 ) = 2 (2x4 y2 + x2 y2 z 2 + 2x4 yz + 4x2 y3z)


sym

x 4 = x 4 ( xy + xz + yz ) = (2 x 5 y + x 4 yz )
sym sym

Juntando tudo,
sym

LE = 2 (2 x 5 y + 4 x 4 y 2 +5 x 4 yz + 3 x 3 y 3 + 26 x 3 y 2 z + 8 x 2 y 2 z 2 )

Usando duas vezes bunching e em seguida Schur temos:


sym

LE LD = (4 x 5 y x 4 y 2 + x 4 yz 3 x 3 y 3 2 x 3 y 2 z + x 2 y 2 z 2 ) =
sym sym sym

= ( x 5 y x 4 y 2 ) + 3 ( x 5 y x 3 y 3 ) + xyz ( x 3 + xyz 2 x 2 y ) 0
Exemplo 8. (IMO) Sejam x, y, z reais positivos com xyz = 1. Mostre que:

y5 y 2 x5 x2 z5 z 2 + + 0. x5 + y 2 + z 2 x2 + y5 + z 2 x2 + y 2 + z5
Soluo:

Escrevendo na notao de simetria, o problema equivalente a:

(x
sym

x 2 ) (x 2 + y 5 + z 2 ) (x 2 + y 2 + z 5 ) 0

EUREKA! N23, 2006

29

Sociedade Brasileira de Matemtica

Como (x2 + y 5 + z 2 ) (x2 + y 2 + z 5 ) = x 4 + ( y 5 + z 2 + y 2 + z 5 )x2 + ( y5 + z 2 )(y 2 + z 5 ) , basta que: x5 ( x 4 + 2 y 5 x 2 + 2 y 2 x2 + 2 y 7 + y 5 z 5 + z 2 y 2 )


sym

x2 (x4 + 2 y5 x2 + 2 y 2 x2 + 2 y7 + y5 z5 + z2 y 2 )

(x
sym

sym

+ 2x 7 y 5 + 2x 7 y 2 + 2x 5 y 7 + x 5 y 5 z 5 + x 5 y 2 z 2 ) + 2 x 4 y 5 + 2x 4 y 2 + 2 x 2 y 7 + x 5 y 5 z 2 + x 2 y 2 z 2 )

(x
sym

S bunching no vai resolver isso, pois o termo x5y5z5 fica sobrando do lado esquerdo. Usando respectivamente a desigualdade das mdias, bunching e xyz _  porm temos:

(x
sym sym

y 5 + x 7 y 5 + x 5 y 5 z 5 + x 5 y 2 z 2 ) 4 x 6 y 4, 25 z 1, 75
sym 6

(2 x (2 x
sym

y 4 z 2 + x5 y5 z 2 + x4 y 4 z 4 ) y 2 + x5 y 5 z 2 + x 2 y 2 z 2 )

O problema est resolvido somando-se essa desigualdade com:

(x
sym

+ 2 x 7 y 5 ) ( x 7 yz + 2 x 6 y 5 z ) ( x 6 + 2 x 5 y 4 ) ,
sym sym

que vale por bunching e xyz _  70850.9 ;,20390


4. EXERCCIOS PROPOSTOS:

(Assuma as variveis reais positivas nos exerccios abaixo)

a b c 3 + + . b+c a+c a+b 2 2 2 2 2 2 2 2 2 2 2 2 2 2. ( x + y z ) ( x y) + ( y + z x )( y z ) + ( z + x y )(z x) 0 . 7 3. (IMO) Se x + y + z = 1, mostre que 0 xy + yz + zx 2 xyz . 27


1. 4. (Banco IMO) Se xyz = 1, mostre que

y3 x3 z3 3 + + . (1 + y )(1 + z ) (1 + z )(1 + x) (1 + x )(1 + y ) 4

EUREKA! N23, 2006

30

Sociedade Brasileira de Matemtica

1 1 1 3 + 3 + 3 . a (b + c ) b (a + c) c (a + b) 2 1 1 1 6. (IMO) Se abc = 1, mostre que a 1 + b 1 + c 1 + 1 . b c a


5. (IMO) Sendo abc = 1, mostre que
3

7. (Banco IMO)

(a + b c) 2 (a b + c)2 (a + b + c)2 (a2 + b2 c 2 ) (a2 b2 + c 2 ) (a2 + b2 + c 2 ) . (b + c a ) 2 (c + a b ) 2 ( a + b c) 2 3 8. (Japo) + + . 2 2 2 2 2 2 5 (b + c) + a (c + a ) + b ( a + b) + c (2 x + y + z) 2 (2 y + x + z ) 2 ( 2 z + y + x) 2 9. (USA) + + 8. 2 x 2 + ( y + z ) 2 2 y 2 + ( x + z ) 2 2 z 2 + ( y + x) 2


10. (Bulgria) Se abc = 1, mostre que

1 1 1 1 1 1 + + + + . 1+ a + b 1+ b + c 1+ a + c 2 + a 2 + b 2 + c

EUREKA! N23, 2006

31

Sociedade Brasileira de Matemtica

O PROBLEMA IMPOSSVEL
Cssio Neri Instituto de Matemtica, Universidade Federal do Rio de Janeiro Nvel Avanado
INTRODUO:

O Problema Impossvel um lindo problema sobre nmeros inteiros. Sua forma original foi dada por Freudenthal [1] antes de ser popularizada por Martin Gardner1 [2]. As duas verses no so exatamente iguais. A verso de Gardner a seguinte:
Dois nmeros inteiros (no necessariamente diferentes) entre 2 e 20 so escolhidos. Apenas a soma dos dois nmeros dada ao matemtico Srgio. Apenas o produto dos dois nmeros dado ao matemtico Paulo. Por telefone, Srgio diz a Paulo: "No existem meios para que voc determine minha soma". Uma hora depois, Paulo telefona de volta para dizer: "Eu sei a sua soma". Mais tarde, Srgio telefona novamente para Paulo para anunciar: "Agora eu sei o seu produto". Quais so os nmeros?

No problema original, Freudenthal fixou uma cota superior de 100 (no para os nmeros, mas para a soma; no obstante, vamos considerar apenas variaes do problema com cotas para os nmeros, embora a verso de Freudenthal possa ser analisada de modo anlogo). Para simplificar o problema, Gardner preferiu usar a cota superior igual a 20. Fazendo isto, "o Problema Impossvel se tornou literalmente impossvel" como disse o prprio Gardner [3]. Neste texto vamos explicar este ponto. surpreendente que o problema original seja bem posto j que, como pode-se pensar, a pequena conversa telefnica no acrescenta nenhuma informao relevante sobre os nmeros. Mas, como veremos, esta conversa rica de informaes matemticas. O Problema Impossvel um problema 3 em 1. Os problemas propostos a Paulo e a Srgio so diferentes do nosso e entre si. Cada um dos personagens tem uma informao adicional (o produto para Paulo e a soma para Srgio) que ns no temos. Isto faz uma grande diferena.
1

O autor, alm de traduzir o enunciado, tomou a liberdade de chamar os personagens de Paulo e Srgio em vez de P e S como na verso em ingls.

EUREKA! N23, 2006

32

Sociedade Brasileira de Matemtica

Nas duas primeiras sees consideraremos o problema como nos foi proposto. Na ltima seo consideramos os problemas propostos a Paulo e a Srgio.
SOLUO DO PROBLEMA

Para resolver o problema vamos usar um programa de computador j que existem muitos casos a considerar. Um programa escrito em linguagem C est disponvel em [6]. Posteriormente veremos uma soluo sem a necessidade de ajuda computacional. Vejamos a estratgia da soluo. Aproximadamente falando, comearemos com dois conjuntos grandes: o dos produtos admissveis e o das somas admissveis. De cada frase do dilogo, extrairemos informao que nos permitir reduzir o tamanho destes conjuntos. Ao final, teremos apenas um produto e uma soma admissvel. Resolvendo uma equao de segundo grau, determinaremos os dois nmeros. Faremos a seguinte hiptese fundamental, caso contrrio, o problema no tem sentido matemtico.
Hiptese fundamental: Paulo e Srgio dizem a verdade. Esta hiptese merece um comentrio filosfico. Algum leitor pode julgar que tal hiptese seja falsa j que Srgio afirma que Paulo no pode determinar a soma e, uma hora depois, Paulo anuncia que o fez. Um deles est mentindo? No necessariamente. A hiptese continua sendo plausvel ao considerarmos o tempo. Inicialmente Paulo no podia resolver o problema. Por alguma razo, que veremos a seguir, Srgio sabia disto e anunciou o fato. Paulo pensou e descobriu qual era esta razo. S depois de descobri-la (portanto, no antes de Srgio se pronunciar) que Paulo foi capaz de determinar a soma.

Chamaremos de p ao produto e s soma dos dois nmeros. Seja N a cota superior dada para os nmeros. Nesta seo consideraremos N = 100. Sabemos, a priori, que p no primo j que ele um produto de dois inteiros maiores que 1. Alm disto, p no pode ter nenhum divisor primo maior que N (por exemplo, p no pode ser 2 101). H outras restries, por exemplo, p 11 11 11 e p 11 13 17. Veja que sabemos muita coisa sobre p. Por outro lado, no temos nenhuma restrio sobre s, exceto que 4 s 200. Para simplificar a apresentao, faremos as seguintes definies.

EUREKA! N23, 2006

33

Sociedade Brasileira de Matemtica

Definio: Dado um inteiro m 4, dizemos que o par (i, j) uma fatorao de m se m = ij com i e j inteiros entre 2 e N. Definio: Dado um inteiro m 4, dizemos que o par (i, j) uma decomposio de

m se m = i + j com i e j inteiros entre 2 e N. Repare que estas definies dependem de N. Assim, (2, 8) uma fatorao de 16 se, e somente se, N 8. Em se tratando de fatoraes e decomposies, consideramos (i, j) = (j, i). As restries sobre p, citadas anteriormente, so conseqncias do seguinte fato: existe pelo menos uma fatorao para p. Os conjuntos dos produtos e das somas admissveis so dados, respectivamente, por P0 = {m <; 4 m N 2 e m tem pelo menos uma fatorao}, S0 = {m <; 4 m 2 N }. O programa informa que P0 tem 2880 elementos. Informa tambm que S0 tem 197, entretanto isto trivialmente constatado sem necessidade de computador. Passemos anlise do dilogo. Inicialmente Srgio diz que Paulo no pode determinar a soma. Isto significa que p tem pelo menos duas fatoraes distintas. De fato, j sabemos que p tem fatorao. Se (i, j) fosse a nica fatorao de p, ento Paulo saberia que s = i + j. Desta forma, o conjunto dos produtos admissveis se reduz a P 1 = {m P 0 ; m tem pelo menos duas fatoraes}. Temos que p P 1 e, segundo o programa, P 1 tem 1087 elementos. Percebe-se facilmente que se i e j so primos, com i, j N , ento ij P0 \ P 1. Porm, nmeros desta forma constituem apenas uma pequena parte de P0 \ P 1 . Outros exemplos de
2 elementos de P0 \ P e N ( N 1). 1 so N Existe outra conseqncia da primeira frase de Srgio: qualquer que seja a decomposio (i, j) de s temos que ij P 1 . com efeito, suponhamos que s tenha decomposio (i, j) tal que ij P 1 . Vamos mostrar que, neste caso, existe uma circunstncia na qual Paulo poderia determinar a soma, de modo que Srgio no poderia fazer sua primeira declarao. Isto ocorre quando p = ij pois teramos p P 1 e, portanto, (i, j) seria a nica fatorao de p. Isto permitiria a Paulo concluir que s = i + j. Em vista disso, o conjunto de somas admissveis se reduz a

S1 = {m S 0 ; para toda decomposio (i, j) de m temos que ij P 1}.

EUREKA! N23, 2006

34

Sociedade Brasileira de Matemtica

Temos que s S1 e, de acordo com o programa, S1 = {11,17,23,27, 29,35,37,41,47,53} Em seguida, Paulo anuncia saber a soma. Segue da que existe uma nica fatorao (i, j) de p tal que i + j S1. De fato, s S1 , logo, p tem fatorao cuja soma est em S1 . Se existisse mais de uma fatorao nessa condio, ento Paulo no poderia saber qual delas seria correta. Assim, o conjunto dos produtos admissveis reduzido a P2 = {m P1 ; existe uma nica fatorao (i, j) de m tal que i + j S1}. Temos que p P2 . O programa diz que P2 tem 86 elementos. Finalmente, Srgio diz que tambm sabe o produto. Graas a um argumento anlogo ao do pargrafo anterior, conclumos que existe uma nica decomposio (i, j) de s tal que ij P2 . Portanto, o conjunto das somas admissveis se reduz a S 2 = {m S1 ; existe uma nica decomposio (i, j) de m tal que ij P2 }. Temos que s S2 . O programa informa que S 2 = {17}. Logo, s = 17. Neste momento, Srgio j anunciou ter encontrado o produto. Como est dizendo a verdade, ento ns tambm podemos encontr-lo. Com efeito, sabemos que a soma vale 17 e, portanto, podemos nos colocar no lugar de Srgio. Outra maneira de proceder reiterar o argumento que define P2 a partir de S1 e aquele que define S 2 a partir de P2 para, assim, construir Pn a partir de S n 1 e S n a partir de Pn . Repetimos isto at encontrar n < tal que Pn e S n sejam unitrios. Mais precisamente, para n = 3,, definimos Pn = {m Pn1 ; existe uma nica fatorao (i, j) de m tal que i + j S n 1}, S n = {m S n 1 ; existe uma nica decomposio (i, j) de m tal que ij Pn }. Obtemos assim, P3 = {52} e S3 = {17}, logo p = 52 e s = 17, de onde segue que os nmeros so 4 e 13.
O ERRO DO GARDNER

Conhecendo a resposta, 4 e 13, a intuio diria que a mesma soluo valeria se trocssemos, como fez Gardner, a cota superior N de 100 para 20.
EUREKA! N23, 2006

35

Sociedade Brasileira de Matemtica

Surpreendentemente, isto est errado! Veremos que para N 61 os nmeros 4 e 13 no resolvem o problema! O que acontece se N 61 ? Neste caso 17 S2 . De fato, o programa mostra que S 2 = , o que significa que o problema no tem soluo. Pode-se pensar que esta uma falha do mtodo e que talvez, por outro argumento encontrssemos a soluo, mas conclumos o seguinte. Se s e p so os nmeros fornecidos a Srgio e Paulo, respectivamente, e se ambos dizem a verdade, ento obtemos, sucessivamente, os seguintes fatos: 1. antes do dilogo: p P0 e s S0 ; 2. aps a primeira frase de Srgio: p P1 e s S1 ; 3. aps a primeira frase de Paulo: p P2 ; 4. aps a primeira frase de Srgio: s S2 . Logo, se os nmeros so 4 e 13, ento p = 52 P2 e s = 17 S2 . Suponhamos agora que N 61. Vamos mostrar que 52 P2 ou 17 S2 , de onde seguir que os nmeros no podem ser 4 e 13. Procedemos por absurdo supondo que 52 P2 e 17 S 2 . Afirmaremos que 19,37 S1 . De fato, como 34 tem apenas a fatorao (2, 17), temos que 34 P 1 e, portanto, 2 + 17 = 19 S1 . Observamos que 186 pode ser escrito como produto de dois nmeros naturais apenas das seguintes formas: 1 186, 2 93, 3 62 e 6 31. Como N 61, a nica fatorao de 186 (6, 31), logo, 186 P 1 . Segue da que 6 + 31 = 37 S1 . Agora, mostraremos que 70 P2 . Podemos escrever 70 como 1 70,2 35,5 14 e 7 10. Como j vimos, 37 = 2 + 35 e 19 = 5 + 14 no esto em S1 , porm, 17 S2 S1 . Logo, 70 P2 . Finalmente, entre todas as decomposies de 17 temos (4, 13) e (7, 10). Como 52,70 P2 temos que 17 S2 o que uma contradio. Salientamos os seguintes fatos: Mostramos apenas que (4, 13) no soluo para N 61. Talvez, outra soluo aparea quando N 61. Entretanto, o programa em [6] mostra que S 2 = e, portanto, o problema no tem soluo.

EUREKA! N23, 2006

36

Sociedade Brasileira de Matemtica

2. Mesmo os puristas, que no aceitam demonstraes assistidas por computador, devero aceitar que (4, 13) no soluo do problema quando N 61. 3. Quando dizemos que o problema no tem soluo, significa que o nosso problema no tem soluo. Por outro lado, o problema proposto a Paulo tem soluo desde que a ele seja fornecido um nmero adequado. A situao de Srgio no muito diferente da nossa. Abordaremos estas questes na prxima seo.
O PROBLEMA DE PAULO E O DE SRGIO

Nesta seo, consideraremos os outros problemas, especialmente, o dado a Paulo. Em [6] est disponvel um programa em linguagem C que poderia ser usado por Paulo (ou por ns em seu lugar) e outro para Srgio. Por hora, voltemos a considerar N = 100. Paulo sabe que p = 52 e que suas fatoraes so (2, 26) e (4, 13). Assim, ele sabe que s vale 28 ou 17. Aps Srgio falar pela primeira vez, Paulo procura as decomposies de 28: (2, 26), (3, 25), (4, 24), (5, 23), Ele pode parar por a e concluir que s 28 pois (5, 23) a nica fatorao de 115 (5 e 23 so primos) e, portanto, 115 P 1 e 28 S1 . Agora, ele pode procurar as decomposies de 17 (s para ter certeza que s = 17): (2, 15), (3, 14), (4, 13), (5, 12), (6, 11), (7, 10) e (8, 9). Estas decomposies so fatoraes, respectivamente, de 30, 42, 52, 60, 66, 70 e 72. Todos estes nmeros esto em P 1 j que cada um deles tem outra fatorao diferente daquela aqui mostrada. Paulo conclui que s = 17. Srgio deve trabalhar mais que Paulo. Porm, ele tambm pode achar o produto aps alguns minutos2. Consideremos, agora, outros valores para N. Para valores pequenos no podemos resolver o problema. Da mesma forma, para N grande, por exemplo N = 866, no possvel determinar os nmeros pois Pn = {52,244} e S n = {17,65} para todo N 3. Para Paulo, o efeito de trocar N quase nulo: sabendo que p = 52 ele encontra s = 17 para todo N 13 (fato em acordo com a intuio). Mesmo a ambigidade que aparece para ns quando N = 866 no atrapalha Paulo. Neste caso, se ele recebesse p = 244, ento acharia s = 65. Quando N 25,(2,26) no mais fatorao de 52. Assim, Paulo no teria dvida de que s = 17. Portanto, Srgio no diria que Paulo no podia determinar a soma. Paulo acharia s mas Srgio no acharia p (como veremos abaixo).
O autor se ps no lugar de Srgio e aps, aproximadamente, 15 minutos achou o produto. Nos seus clculos ele considerou em P0 \ P 1 apenas os nmeros que so produtos de dois primos.
EUREKA! N23, 2006
2

37

Sociedade Brasileira de Matemtica

Para N 61 , a situao de Srgio similar a nossa: ele no pode achar p. A razo a mesma de antes: (4, 13) e (7, 10) so duas decomposies de 17 com produtos em P2 . Outra maneira de ver este fato a seguinte. Tome N = 61 e coloquemo-nos no lugar de Paulo, primeiro com p = 52 e, depois, com p = 70. Em ambos os casos, aps a primeira frase de Srgio, encontraramos s = 17. Logo, Srgio no poderia decidir se p = 52 ou p = 70. Na verdade, existe ainda uma terceira possibilidade, p = 66, que aparece quando N = 61. Para N = 866, Srgio encontra p = 52, quando s = 17, ou p = 244, quando s = 65.
SOLUO SEM ASSISTNCIA COMPUTACIONAL

Vejamos agora uma soluo do problema original, com N = 100, que no necessita de assistncia computacional. Determinar P0 ou P 1 manualmente seria muito trabalhoso (lembre-se que, segundo o programa, esses conjuntos tm, respectivamente, 2880 e 1087 elementos). Vamos procurar S1 sem computador mas com pacincia. Para concluir que S S1 basta existir uma decomposio (i, j) de S que seja a nica fatorao de ij. Vejamos alguns casos (cuja verificao deixamos para o leitor). Para S = 200 temos a decomposio (100, 100). Para S = 199 e S = 198 temos (99, 100) e (99, 99), respectivamente. Se 99 S 197,(i, j ) = (97, S 97) resolve pois 2 S 97 100 e ij tem fator primo 97, logo, i, digamos, tem fator 97. Como 97 > 50, se i tivemos outro fator primo, ento teramos i > 100, que absurdo. Conclumos que i = 97 e j = S 97. Portanto, (S, S 97) a nica fatorao de ij. Analogamente, se 55 S 153, ento (53, S 53) a nica fatorao de 53(S 53). Resumindo, at aqui mostraremos que se S 55, ento S S1 . Agora, se S 54 e S par, ento pode-se verificar, caso a caso, que S tem decomposio (i, j) com i e j primos (o caso geral, isto , sem cota superior para S, um problema em aberto conhecido como Conjectura de Goldbach), logo, S S1 . Analogamente, os mpares 5, 7, 9, 13, 15, 19, 21, 25, 31, 33, 39, 43 e 45 so todos da forma 2 + P com P primo e, portanto, nenhum deles est em S1 . Finalmente, (17, 34) uma decomposio de 51 que a nica fatorao de 578 = 2 17 2 (pois 172 > 100), ou seja, 51 S1. E = {11,17,23,27, 29,35,37, 41, 47,53}. Quem quiser pode Conclumos que S1 S mostrar que vale a incluso contrria, mas isto no ser necessrio nossa demonstrao.
EUREKA! N23, 2006

38

Sociedade Brasileira de Matemtica

Determinar P2 tambm exigiria muito esforo. Por isto, passaremos diretamente para S 2 . Observaremos que se S S2 , ento existe uma nica decomposio (i, j) de S tal E. que ij P e esta a nica fatorao de ij com soma em S S

E \ S basta exibir duas decomposies distintas Assim, para concluir que S S 2 (i1 , j1 ) e (i2 , j2 ) de S tais que i1 j1 e i2 j2 admitam, no mximo, estas fatoraes com E. Por exemplo, 11 S . De fato, considere as decomposies (2, 9) e (3, soma em S
2

8). Elas tm produtos 18 e 24, respectivamente. Alm de (2, 9) e (3, 8), estes nmeros admitem as seguintes fatoraes: (3, 6), (2, 12) e (4, 6) cujas somas 9, 14 e E. 10 esto fora de S Analogamente, mostra-se que 23 S2 considerando as decomposies (4, 19) e (10, 13). Para 27, considere (4, 23) e (8, 19); para 29 considere (6, 23) e (10, 19). Finalmente, para S {35,37,41,47,53} considere (31, S 31) e (29, S 29) (mostre caso a caso). Finalmente, conclumos que S 2 = {17}, logo, a soma 17. As fatoraes de 17 so (2, 15), (3, 14), (4, 13), (5, 12), (6, 11), (7, 10) e (8, 9). Os produtos correspondentes so 30, 42, 52, 60, 66, 70 e 72. Ora, o produto procurando est em P2 , logo, ele tem uma nica fatorao cuja soma est em E . O nico dos nmeros anteriores com esta propriedade 52. Por exemplo, S S

E ). De modo E (alm disto, claro que 2 + 15 = 17 S 30 = 2 15 = 5 6 e 5 + 6 = 11 S anlogo consideraremos 42 = 3 14 = 2 21, 60 = 5 13 = 3 20,66 = 6 11 = 2 33,70 = 7 10 = 2 35 e 72 = 8 9 = 3 24. Logo, o produto 52 e os nmeros so 4 e 13.

Referncias [1] HANS FREUDENTHAL, Nicuw Archief Voor Wiskunde, Ser 3, 17 (1969) 152 [2] MARTIN GARDNER, Mathematical Games, Scientific American, 241 (Dec. 1979) 22. [3] MARTIN GARDNER, Mathematical Games, Scientific American, 242 (May 1980) [4] LEE SALLOWS, The Impossible Problem, The Mathematical Intelligencer, 17:1 (1995) 27. [5] ISAACS I. M. ISAACS, The Impossible Problem Revisited Again, Tha Mathematical Intelligencer, 17:4 (1195) 4. [6] URL: www.labma.ufrj.br/~cassio/f-impossivel.html

EUREKA! N23, 2006

39

Sociedade Brasileira de Matemtica

COMO QUE FAZ?


Resolveremos a seguir dois problemas da IV Olimpada Iberoamericana de Matemtica Universitria, por sugesto de Macelo da Silva Mendes, de Teresina PI. 1) Seja > 0 um nmero real. Sejam 0 < x1 < x2 < x3 < ... as solues reais da 1 converge. equao x sen( x ) = ln x. Encontre os valores de para os quais n =1 xn Suponhamos inicialmente que 1. Dado k <* , temos 1 1 (2k )1 sen ( 2k ) = 0 < ln ( 2k ) , enquanto 1 1 1 1 1 1 1 1 2 2 2 ln 2 , + + = + > + k sen k k k 2 2 2 2 * pois x > ln x, para todo x > 0. Assim, para todo k < , existe uma soluo
SOLUO:

1 1 1 x sen ( x ) = ln x . da equao yk ( 2 k ) , 2 k + 2 1 1 1 1 1 = +, caso 1. 1 1 2 k =1 k + 1 n =1 xn k =1 k =1 1 2 + k 2k + 2 2

Assim,

Consideremos

agora

o
1

caso

0 < < 1. Para

Seja

f ( x ) = x sen( x ). existe um

Temos nico temos

f '( x ) = sen( x ) + x cos( x ). wk ( 2k ) , (( 2k + 1) )


1

cada

k <, para

com

f '( wk ) = 0;
1

( 2k )

< x < wk

f '( x ) > 0, e, para wk < x < (( 2k + 1) ) , temos 1 0 < x 2k + 2


1

f '( x ) < 0. De fato, para

, temos que

f '( x ) > 0, f ' decrescente no intervalo

1 1 1 2k + , ((2k + 1) ) e f ' 2

(((2k + 1) ) ) < 0.
1

EUREKA! N23, 2006

40

Sociedade Brasileira de Matemtica


1 1 1 1 1 1 Como f 2k + = 2k + > ln 2k + , e 2 2 2

(((2k + 1)) ) = 0 < ln (((2k + 1) ) ),


1 1

existe uma soluo yk de f ( x ) = ln x

1 com 2k + 2 temos

< x < (( 2k + 1) ) . Para


1

((2k + 1) )

x (( 2k + 2 ) ) ,
1

f ( x ) 0 < ln x.

Para

1 x wk , (( 2k + 1) ) , temos

f '( x) 0

1 > 0, donde a equao f ( x ) = x tem no mximo uma soluo em x w , (( 2k + 1) )1 . k 1 1 1 1 5 5 Como f (( 2k + 2) ) = 0 < ln ((2k +1) ) e f 2k + > ln 2k + , 2 2 existe uma menor soluo de com uk f ( x ) = ln x

(ln x )' =

5 < uk < 2k + . Como f (uk ) = uk sen (uk ) = ln uk , donde 2 uk sen (uk . ) = uk sen (uk ) = ln uk , temos sen (uk ) = ln uk

( ( 2k + 2 ) )

ln uk 1 5 = > = ( ln ) ' (uk ) Temos, para uk < y 2k + , f '( y ) sen y sen (uk ) uk uk 2 (pois uk > ( 2 )
1

> 2 > e), donde f ( y ) > ln y , e logo uk a nica soluo de

1 1 1 5 1 > + + + f ( x ) ln x em (( 2k 2 ) ) , 2k . Se 2k y wk , 2 2 1 1 1 1 1 f ( y ) f 2k + = 2k + > ln (( 2k + 1) ) > ln wk ln y 2 2 1 1 1 1 e 1 (de fato, ln (( 2k + 1) ) = ln 2k + < ln (( 2k + 1) ) = e ln 2k + 2 2

EUREKA! N23, 2006

41

Sociedade Brasileira de Matemtica

1 1 8 2k + ; ln (( 2k +1) ) < eln 2k + segue de ln < ln < eln . 2 2 3 2 2 Assim, e solues de em yk uk so as nicas f ( x ) = ln x
1 1 1 5 2k + , 2k + , 2 2

para

todo

k < .

Finalmente,
1

se

x ( 0,1) , f ( x ) > 0 > ln x

e,

para

x x f ( x) x sen1 x sen = > ln x, donde no h soluo de f ( x) = ln x em 6 2 e 1 1 1 1 < 2 < < +, no caso 2 0, . Assim, 1 1 2 n=1 xn k =0 k =0 ( k + 1) 1 2k + 2 0 < < 1. A resposta , portanto, { 0 < < 1}. 2) Calcular n 3 ...cos . n n n=1 m < , temos SOLUO: Observemos inicialmente que, para todo sen(mx) = senx Qm (cos x), onde Q0 = 0 e, para m 1, Qm um polinmio de grau

1 x , 2

temos

cos n cos n

cos

m 1 e coeficiente lder 2m 1. Alm disso, o coeficiente constante de Qm 0 se m


par e igual a (1)k , se m = 2k + 1 , para todo k 0 . Isto pode ser provado a partir da identidade sen (( m + 1) x ) + sen (( m 1) x ) = 2sen ( mx ) cos x; temos ento Q0 ( y) = 0 , Q1 ( y) = 1 e Qm +1 ( y) = 2 yQm ( y) Qm 1 ( y), m 1. k Por outro lado, como sen(k ) = 0 e sen 0 para todo k inteiro com m k 1 k m 1, temos que cos raiz de Qm ( y ), para 1 k m 1. Como Qm tem m k grau m 1, tem coeficiente lder 2m 1 e os nmeros cos ,1 k m 1, so todos m m1 k distintos, temos Qm ( y) = 2m1 y cos , m 1. m k =1
EUREKA! N23, 2006

42

Sociedade Brasileira de Matemtica

Em particular, o coeficiente constante de Qm ( y ) ( 2 ) 0,se m par k Assim, cos = ( 1)r . m 2 r , se m = 2r + 1, r < k =1 2 n cos = cos = 1, a srie que Como n
m1

m 1

m 1 k cos . m k =1

queremos

calcular

vale

1 4 = . 1 5 r =0 2 1+ 4 _______________________________
2r

( 1)r

O leitor Joo Alexandre Jnior enviou uma soluo do problema No. 2 do artigo "Os 4 4 problemas do Visitante Matemtico", que pedia para calcular 44 . Como 44 = 4256 Joo Alexandre calculou manualmente, de forma sucessiva, 42 = 16, 44 = 162 = 256,
2

48 = 2562 = 65536, 416 = 655362 = 4294967296, 432 = ( 416 ) = 18446744073709551616,


464 = 432

( )

= 340282366920938463463374607431768211456, 4128 = 464

( )

=115792089237316195423570985008687907853269984665640564039457584007 913129639936 e, finalmente, 4256 = 4128

( )

= 1340780792994259709957402499820

5846127479365820592393377723561443721764030073546976801874298166903 427690031858186486050853753882811946569946433649006084096.

EUREKA! N23, 2006

43

Sociedade Brasileira de Matemtica

SOLUES DE PROBLEMAS PROPOSTOS


Publicamos aqui algumas das respostas enviadas por nossos leitores.

Uma prova de mltipla escolha com n questes feita por k alunos. Uma 89) resposta correta na i-sima questo vale pi pontos, onde pi um inteiro positivo, para 1 i n. A nota de cada aluno a soma dos pontos correspondentes s questes que ele acertou. Aps a realizao a prova, foi observado que, mudando os pesos pi, as notas dos alunos podem estar em qualquer uma das k! possveis ordens (em que no h duas notas iguais). Dado n, qual o maior valor possvel de k?
SOLUO DE ZOROASTRO AZAMBUJA NETO (RIO DE JANEIRO RJ)

Vamos mostrar que o maior valor possvel de k n. De fato, fcil ver k pode ser igual a n: dada uma bijeo = {1, 2,..., n} {1, 2,..., n}, e supondo que, para 1 i n, o i-simo aluno acertou a i-sima questo e errou todas as outras, se atribuirmos peso pi = n + 1 (i) questo i para 1 i n, temos que o aluno de nmero 1 (i ) obteve a i-sima melhor nota, para 1 i n. Suponha agora que k alunos tenham feito k provas como no enunciado. Para cada i k , denotamos por v(i) {0,1}n o resultado da i-sima prova:

v(i) = (vi1 , vi 2 ,..., vin ), onde vij = 1 se o i-simo aluno acertou a j-sima questo, e vij = 0 caso contrrio. Se k > n, os vetores v(i),1 i k , so linearmente
dependentes, ou seja existem constantes ci ,1 i k , no todas nulas, tais que

c v(i) = 0. Assim, passando os termos com


i =1 i

ci negativo para o lado direito da a1 , a2 ,..., ar , b1 , b2 ,..., bs


com

igualdade,

obtemos
r

constantes

positivas
s

r , s 1, r + s k e

ai v ( (i ) ) = b j v ( ( j ) ) , onde
i =1 j =1

(1), (2),..., (r ), (1), (2),..., (s ) so os ndices i tais que ci 0 os ( j ) so tais que c ( j ) > 0 e os ( j ) so tais que c ( j ) < 0 .
Agora, se o peso da i-sima questo pi > 0, a nota da i-sima prova

n(v(i)) = vij p j .
j =1

EUREKA! N23, 2006

44

Sociedade Brasileira de Matemtica

Sejam =

bj
j =1

 a , b
i =1 i

bi

b
j =1

 i = ai . e a r
j

a
i =1

Podemos supor sem perda de generalidade que 1 (seno trocamos os lados da igualdade). Como
s

b j v ( ( j) ) = ai v ( (i) ) , temos
j =1 i =1 r i i =1

 v ( j) =  v (i ) e, como n(v) depende linearmente de v, temos ( ) ( ) b a


j j =1 s

s  r  ( ) ( ) b j n (v ( j )) = n b j v ( j ) = n a i v ( (i ) ) j =1 i =1 j =1


=
i =1

i a

 i n v (i ) , n (v ( (i )) a ( ( ))
i =1

mas como

  j = a i = 1, temos que b
j =1 i =1

 j n ( v ( ( j ) )) e b
j =1

so respectivamente mdias ponderadas dos desigualdade

 n v (i ) ( ( )) a n (v ( ( j ) )) e dos n (v ( (i) )) , e a
i i =1

claramente implica que no podemos ter n v ( (i) ) > n v ( ( j ) ) para todo i r e j s , e portanto as notas no podem ficar em qualquer ordem. Nota: Se k > n ento quaisquer vetores v1 , v2 ,..., vk n = {( x1 , x2 ,..., xn ); xi ,

acima

i n} so linearmente dependentes, isto , existem constantes ci ,1 i k


no todas nulas tais que

c v
i =1

i i

= c1v1 + ... + ck vk = 0. Podemos provar isso por


no so

induo em n. O resultado claramente verdade se n = 1: se v1 , v2 ambos nulos, c1 = v2 e c2 = v1 so tais que c1v1 + c2 v2 = 0. identificar
m

Se m < n podemos

com{( x1, x2 ,..., xm ,0,0,...,0)


n +1

; xi , i m}. Se vi = 0 para

algum i, podemos tomar ci = 1 e c j = 0, j i. Se k > n + 1, e v1 , v2 ,..., vk so vetores em possibilidades: com


(i ) para cada i k , temos duas vi = ( x1(i ) ,..., xn +1 )

EUREKA! N23, 2006

45

Sociedade Brasileira de Matemtica


(i ) i) xn +1 = 0 para todo i k . Nesse caso vi imediatamente pela hiptese de induo. n

para todo i k e o resultado segue

(i ) ii) xn +1 0 para algum i. Podemos supor sem perda de generalidade que isso vale para i = k. ( j) xn +1 Nesse caso temos v i = v j ( k vk x n +)1

para todo j com 1 j k 1 e, como


k 1 j =1

(n + 1)-sima coordenada desses vetores se anula e k 1 > n, por hiptese de


 ,..., c  induo, existem c k 1 1

no todos nulos tais que


 v ( c
j j =1 k 1 j =1 k 1

 c

 v j = 0, mas

( j) (k )  v j = v j a j vk , onde a j = xn +1 xn +1 , e portanto k

a j vk ) = 0, donde

 para 1  , o que prova o j k 1 e ck = a j c j j c j v j = 0, onde c j = c


j =1

resultado. 97) Seja p um primo mpar. Encontre todas as funes f : seguintes condies: i)
ii)

que satisfazem as

Se m n(mod p) ento f (m) = f (n).

f (mn) = f (m) f (n) para quaisquer m, n .

SOLUO DE EDEL PREZ CASTILLO (PINAR DEL RIO CUBA)

Temos

f (1) = f (1) f (1) , o que implica f (1) {0,1}. Se f (1) = 0, ento f (m) = f (m 1) = f (m) f (1) = 0 , para todo m, o que claramente uma soluo. Suponhamos ento que f (1) = 1. De f (0) = f (0) f (0), temos f (0) {0,1}. Se f (0) = 1, ento f (0) = f (0 m) = f (0) f (m) implica f (m) = 1, para todo m, o

que tambm uma soluo. Suponhamos agora que f (0) = 0. Se m mltiplo de p, ou seja, m 0(mod p), temos tambm f (m) = 0. Se m no mltiplo de p ento existe um inteiro x tal que m x 1(mod p) , o que implica 1 = f (1) = f ( m) f ( x), e logo f (m) {1,1}, pois f (m) e f ( x ) so inteiros.
EUREKA! N23, 2006

46

Sociedade Brasileira de Matemtica

Se p / | m e m resduo quadrtico mdulo p, i.e., se existe b inteiro com

b 2 m(mod p ) ento f (m) = f (b) 2 = 1, pois f (b) {1,1}. Se m e n no so resduos quadrticos mdulo p ento m n resduo quadrtico mdulo p (veja [MS]), e logo f (mn) = f (m) f (n) = 1. Assim, f (m) = f (n), ou
seja, f toma o mesmo valor pertencente a {1, 1} em todos os inteiros que no so resduos quadrticos mdulo p. Assim, as funes f que satisfazem as condies do enunciado so: 1) f (n) = 0, n 2) f (n) = 1, n 3) f (n) = 0 se p| n e f (n) = 1 se p / |n 4) f (n) = 0 se p| n, f (n) = 1 se p / | n e n resduo quadrtico mdulo p e f (n) = 1 se n no resduo quadrtico mdulo p. Nota: A funo em 4) usualmente denotada pelo smbolo de Legendre

n f (n) = . Veja a referncia [MS]: Carlos G. Moreira e Nicolau Saldanha. p


Reciprocidade Quadrtica. Eureka! No. 15, pp. 27 30.
2 2, n <. 98) Seja (an ) n< uma seqncia tal que a1 > 2 e an +1 = an

a1 a12 4 1 Mostre que . = 2 n =1 a1 a2 ... an

SOLUO DE RODRIGO VILLARD MILET e MRCIO ASSAD COHEN (RIO DE JANEIRO RJ)

Faa a1 = x +

1 ( isso possvel pois a1 > 2 ). Podemos considerar, sem perda de x 1 generalidade, que x > 1 (se no for, troque x por ). Temos, por induo, que x
2n 1

2n 1 n 1 1 + n 1 , pois an = x + n 1 2 = x2 + n . x x2 x2 x2 Veja que fcil calcular o produto dos 1 n n 1 1 2 1 4 1 1 1 = x2 n . x 2 + n 1 x x + x + 2 x + 4 ... 2 x x x x x x2 1


EUREKA! N23, 2006

ai, Ento

pois temos

47

Sociedade Brasileira de Matemtica

1 = que n=1 a1 a 2 ...a n n =1

x x2
n

x2 O legal que chegamos em uma soma telescpica: n n +1 N +1 N x2 x2 x2 x2 = . Para < , a segunda parcela tende n x 2 1 x 2 N +1 1 2 n +1 n =1 x 2 1 1 x a 1 (pois x > 1), logo este ltimo somatrio igual a 1 1 1 1 =x 2 = . x x 1 x n =1 a1 a 2 ...a n 1 x2 1= 2 , portanto 2 x 1 x 1 o problema, pois

1 n n n+1 1 x2 1 x2 x2 x = . = x x n +1 2n 2n +1 1 x n=1 x 2 1 x n =1 1 x 1 x
n

Isso

finaliza

a1 = x + x=

2 a a1 4 1 x 2 a1 x + 1 = 0 x = 1 . 2 x 2 1 a1 a1 4 . . Isso d = 2 x

Como

>

1,

temos

2 a1 + a1 4

100) a) Um conjunto X < dito impressionante se existe m < tal que, para todo k < , existem elementos de X, a1 < a2 < ... < ak , tais que

a j +1 a j m, j < k .
Determine se possvel particionar < em um nmero finito de conjuntos, nenhum deles impressionante. b) Determine se possvel particionar < em dois conjuntos A e B de modo que nem A nem B contm progresses aritmticas infinitas mas, para cada q <, A e B contm progresses aritmticas de q termos.
SOLUO DE JOS DE ALMEIDA PANTERA (RIO DE JANEIRO RJ)

No.

r < = {1, 2,3,...} que, se < = A1 A2 ... An ento existe j com 1 j r tal que A j impressionante.
Vamos provar, por induo em

Para r = 1 isto bvio. Suponha agora que < = A1 A2 ... An An +1 , e que An +1 no impressionante. Ento existem intervalos arbitrariamente grandes contidos em < \ Ar +1 , isto , existe uma seqncia infinita de inteiros positivos ( a j ) j< tal que
EUREKA! N23, 2006

48

Sociedade Brasileira de Matemtica

a j +1 > a j + j, j <, e, para todo j < e 1 r j , a j + s Ar +1 . Para cada j <, seja f j :{1, 2,..., j} {1, 2,..., r} tal que a j + s Af j ( s ) , para 1 s j.
Construiremos recursivamente g (1), g (2), g (3),... {1, 2,..., r} tais que, para todo

E <, se j escolhemos g(m) com a propriedade acima para todo De fato, dado m E para esses infinitos j, podemos extrair h n possibilidades para o valor de f j m E tem sempre o mesmo valor, o um subconjunto infinito desses j para os quais f j m E , por definio. qual ser g m
Considere agora a decomposio

m <, existem infinitos j m tais que f j ( s ) = g ( s ), para 1 s m.

E , existem infinitos j m E tais que f j ( s) = g ( s), para todo s < m E . Como s m<m

( )

onde =E A1 E A2 ... E An , n E A j g ( n) = j. Por hiptese de induo, q r tal que E Aq impressionante. Afirmamos que o conjunto Aq tambm impressionante. De fato, seja m tal que para todo k <, existem elementos b < b < ... < b de E Aq com b b m, i < k .
<
1 2 k i +1 i

( )

( )

Para esse mesmo m, dado k <, escolhemos j bk tal que f j ( s ) = g ( s ), para

1 s bk . Como bi um elemento de E Aq para 1 i k , g (bi ) = q, para 1 i k . Assim, f j (bi ) = g (bi ) = q, para 1 i k , ou seja, a j + bi Aq , para
impressionante. b) Sim. Temos
2k

1 i k . Como ( a j + bi +1 ) ( a j + bi ) = bi +1 bi m, i < k , conclumos que Aq


<

= A B, onde

A = {n < 2 n < 22 k +1 , para algum inteiro k 0} e


+ + B = {n < 22k 1 n < 22k 2 , para algum inteiro k 0} .

Temos que [0, +) \ A e [0, +) \ B contm intervalos arbitrariamente grandes, donde nem A nem B podem conter progresses aritmticas infinitas. Por outro lado, para cada q <, se k < tal que 22 k q, A contm a progresso aritmtica da razo 1 e q termos {22 k , 2 2 k + 1,..., 2 2 k + q 1}, e B contm a progresso aritmtica da razo 1 e q termos {22 k +1 , 22 k +1 + 1,..., 22 k +1 + q 1}.
EUREKA! N23, 2006

49

Sociedade Brasileira de Matemtica

102) Voc recebe x metros de arame para cercar um terreno na forma de um tringulo pitagrico (os lados so nmeros inteiros), com a condio de que a medida do cateto menor seja 24 metros. Qual dever ser a medida do cateto maior e o comprimento do arame, a fim de que a rea seja: a) mxima? b) mnima?
SOLUO DE JOS FABRICIO LIMA (JOO PESSOA PB)

Considere o tringulo abaixo: Temos: a 2 = b 2 + 242

a 2 b2 = 242 (a + b) (a b) = 26 32

a b

24
Faamos 26 32 como sendo um produto de 2 fatores x y tal que a + b = x e

a b = y. Resolvendo o sistema temos que: a =

x+ y x y e b= . 2 2

Como x + y e x y so ambos divisveis por 2, temos que x, y so nmeros pares pois a decomposio de 26 32 para x e y nmeros mpares impossvel.

x y > 24, logo x y > 48. 2 Portanto as decomposies x y tais que x y > 48 so dadas por:
Sendo b > 24 temos que

x = 25 32 e y = 2 ( x y = 286 )

x = 23 32 e y = 23 ( x y = 64 )

x = 2 4 32 e y = 22 ( x y = 140 ) x = 25 3 e y = 2 3 ( x y = 90 )

EUREKA! N23, 2006

50

Sociedade Brasileira de Matemtica

Note que a rea do tringulo 12 b , sendo assim, para que a rea seja: a) Mxima: Basta que b seja o maior possvel, ou seja, a diferena x y seja a maior possvel. Devemos ter ento x = 25 32 = 288 e y = 2, donde a =

b=

x y = 143, e logo o comprimento da corda deve ser a + b + 24 = 312m. 2 x y 64 x + y 80 = = 32 e a = = = 40. 2 2 2 2

x+ y = 145 e 2

b) Mnima: Basta escolher o menor valor para x y, ou seja,

b=

Logo, o comprimento de corda :

x = a + b + 24 = 40 + 32 + 24 = 96m.
103) Sejam A e B matrizes 2 2 com elementos inteiros. Sabendo que A, A + B, A + 2B, A + 3B e A + 4B so invertveis e que os elementos das respectivas inversas tambm so todos inteiros, mostre que A + 5B tambm invertvel e que os elementos da sua inversa tambm so inteiros.
SOLUO DE ASDRBAL PAFNCIO SANTOS (BOTUCATU - SP)

Para cada t , seja f (t ) o determinante da matriz A + tB. Temos que f(t) uma funo polinomial de grau no mximo 2. Efetivamente, se

= ( a1 + ta2 )( d1 + td2 ) (b1 + tb2 )(c1 + tc2 ).

a A= 1 c1

b1 a2 e B= d1 c2

b2 a1 + ta2 , f (t ) = det + d2 c1 tc2

b1 + tb2 = d1 + td2

Se uma matriz C com elementos inteiros invertvel e os elementos de sua inversa so tambm inteiros, temos 1 = det I = det(C C 1 ) = det C det C 1 e, como

det C e det C 1 so inteiros, det C {1,1}.


Assim, f (t ) = det( A + tB ) {1,1}, para todo t {0,1, 2,3, 4}, e logo um dos valores 1 ou 1 igual a f(t) para pelo menos trs valores de t em {0, 1, 2, 3, 4}. Como f(t) um polinmio de grau no mximo 2, f(t) necessariamente constante igual a 1 ou 1, e logo, para todo t (e em particular para t = 5), A + tB tem
EUREKA! N23, 2006

51

Sociedade Brasileira de Matemtica

elementos inteiros e determinante pertencente a { 1, 1}, e logo tem uma inversa com todos os elementos inteiros. 104) ABC um tringulo. Mostre que existe um nico ponto P de modo que:

( PA )2 + ( PB )2 + ( AB )2 = ( PB )2 + ( PC )2 + ( BC )2 = ( PC )2 + ( PA )2 + (CA)2

SOLUO DE FRANK DE CASTRO (SO PAULO SP)

Considere o tringulo A ' B ' C ' cujos lados paralelos aos lados do tringulo ABC e passam por A, B e C. Considere tambm o ponto P, ortocentro do tringulo A'B'C' (figura abaixo)

A'

P C

B' A
Provaremos que:

C'

2 2 2 2 2 2 ( PA) + ( PB ) + ( AB) = ( PB ) + ( PC ) + ( BC ) P ortocentro de A ' B ' C ' 2 2 2 = ( PC ) + ( PA) + (CA)

(I) PA ' CB ( A ' P ) ( B C ) = 0 A ' B A ' C P B + P C = 0 Como o quadriltero A'BAC um paralelogramo temos A ' C = B A , ou A ' = C + B A (II). / / De (I) e (II) vem que PA ' CB (C + B A ) ( B C ) P B + P C = 0

Nesse caso o ponto procurado ser o ponto P e como o ortocentro do tringulo A'B'C' nico, segue diretamente a unicidade do ponto P. Utilizaremos vetores. Temos:
/ /

C B C 2 + B2 B C A B + A C P B + P C = 0 .
EUREKA! N23, 2006

52

Sociedade Brasileira de Matemtica

Sendo C B = B C , Vem que PA ' CB B2 P B A B = C 2 P C A C. Multiplicando a igualdade anterior por 2 e na seqncia somando P 2 + A2 a ambos os membros, segue que:
/ /

PA ' CB P2 + B2 2 P B + A2 2 A B + B2 = P 2 + C 2 2P C + A2 2 A C + C2 ( P B) + ( A B) = ( P C ) + ( A C ) .
2 2 2 2

Finalmente, somando conclumos que:


/ /
2 2 2

( P + A )2 a
2

ambos os membros da ltima igualdade obtida


2 2 2 2

PA ' CB ( P A) + ( P B ) + ( A B ) = ( P C ) + ( P A) + ( A C ) ( PA) + ( PB ) + ( AB ) = ( PC ) + ( PA) + (CA) .


2 2 2 2

racionando de forma anloga teremos:

PB ' CA ( PA) + ( PB) + ( AB) = ( PB) + ( PC) + ( BC ) , onde B ' = A + C B, e


2 2 2 2 2 2

PC ' BA ( PB) + ( PC ) + ( BC) = ( PC) + ( PA) + (CA) , onde C ' = B + A C. Nessas condies, o ortocentro do tringulo A ' B ' C ' o ponto P procurado.
2 2 2 2 2 2

Nota: Utilizamos:

( X Y ) ( Z W ) representando o produto escalar dos vetores YX e WZ . / / ( X Y ) ( Z W ) = 0 YX WZ. / / X Y = ( X O ) (Y O) = OX OY onde o ponto O a origem do nosso plano.
X 2 = ( X O) ( X O) = OX OX = OX = (OX ) = ( XO ) .
2 2

/ /

/2

( X Y ) = ( X Y ) ( X Y ) = YX YX = YX
2

/ /

/2

= XY .

/2

105) O baricentro do tringulo ABC G. Denotamos por g a , gb , gc as distncias desde G aos lados a, b e c respectivamente. Seja r o raio da circunferncia inscrita. Prove que:

2r 2r 2r , gb , g c 3 3 3 g + gb + gc b) a 3 r
a) g a
EUREKA! N23, 2006

53

Sociedade Brasileira de Matemtica

SOLUO DE CARLOS ALBERTO DA SILVA VICTOR (NILPOLIS RJ)


A

G ga B ha C

Observe que ha = 3 g a

1 a+b+c a ha ga = r. 2 3 a 1 a+b+c 1 a +b+c De forma anloga, temos: gb = r e gc = r. 3 3 b c a) como a < b + c a + b + c > 2a e 1 a +b+c 1 2a 2r ga = r g a > r = ; de forma anloga, temos: 3 3 a 3 a 2r 2r e gc > . gb > 3 3 2r Note que a igualdade g a = no ocorrer, pois caso contrrio, teramos: 3 1 a+b+c 2r a + b + c = 2a b + c = a e o tringulo no existiria. r = 3 3 a r a +b + c a +b + c a +b + c + + g a + gb + gc = 3 a b c r b a a c b c g a + gb + gc = 3 + + + + + + 3 a b c a c b 2 2 2 r logo g a + gb + gc (3 + 2 + 2 + 2 ) = 3r ou seja: 3 g a + gb + g c 3. r s = pr =
EUREKA! N23, 2006

54

Sociedade Brasileira de Matemtica

106) Os polinmios

so definidos por P0 ( x, y, z ), P 1 ( x, y, z ), P 2 ( x, y, z ),...

e P0 ( x, y, z ) = 1 Pm+1 ( x, y , z ) = ( x + z )( y + z ) Pm ( x, y , z + 1) z 2 Pm ( x, y, z ), m 0. Mostre que os polinmios Pm ( x, y , z ), m < so simtricos em x, y, z, i.e., P m ( x, y, z) = P m ( x, z, y) = P m ( y, x, z) = P m ( y, z, x) = P m ( z, x, y) = P m ( z, y, x), para quaisquer x, y, z.
SOLUO DE MARCOS FRANCISCO FERREIRA MARTINELLI (RIO DE JANEIRO RJ)

Provarei, por induo finita, o seguinte: I) Pn ( x, y , z ) = Pn ( x, z , y ) = Pn ( z , y , x ) = Pn ( y , x, z ) , n <.

II) ( z y ) Pn ( x, y , z ) + ( x + y ) Pn ( x, y + 1, z ) = ( x + z ) Pn ( x, y , z + 1) , n < i) P 1 ( x, y, z ) = xy + xz + yz = P 1 ( x, z , y ) = P 1 ( z , y, x ) = P 1 ( y, x, z ). E ainda ( z y ) P 1 ( x, y , z ) + ( x + y ) P 1 ( x, y + 1, z ) ( x + z ) P 1 ( x, y , z1 ) =

= ( z y) + ( x + y) ( xy + xz + yz) + ( x + y)( x + z) ( x + z)( xy + xz + yz) = ( x + z)( x + y) =

= ( z y)( xy + xz + yz) +( x + y) x( y +1) + xz +( y +1) z ( x + z) xy + x( z +1) + y ( z +1) =


= ( z + x ) + ( xy + xz + yz ) ( x + z )( xy + xz + yz ) = 0.

Logo est provado que

( z y) P 1 ( x, y, z ) + ( x + y) P 1 ( x, y +1, z ) = ( x + z ) P 1 ( x, y, z +1) .
ii) Suponhamos que para n = m ( m<) temos:

( z y) Pm ( x, y, z ) + ( x + y ) Pm ( x, y +1, z ) ( x + z ) Pm ( x, y, z +1) = 0. Logo ( z y ) P m+1 ( x, y, z ) + ( x + y ) P m+1 ( x, y + 1, z ) ( x + z ) P m+1 ( x, y, z + 1) = 2 = (z y ) ( x + z )( y + z ) Pm ( x , y , z + 1) z Pm ( x , y , z ) + 2 + (x + y ) ( x + z )( y + z + 1) Pm ( x , y + 1, z + 1) z Pm ( x , y + 1, z )


( x + z ) ( x + z +1)( y + z +1) P m ( x, y, z + 2) ( z +1) P m ( x, y, z + 1) = = z 2 (( z y ) Pm ( x, y, z + 1) + ( x + y ) Pm ( x, y + 1, z ) ( x + z ) Pm ( x, y, z + 1)) +
2

+( x + z)( y + z +1) (( z +1 y) P m ( x, y, z +1) +( x + y) P m ( x, y +1, z +1) ( x + z +1) P m ( x, y, z + 2)) = 0


(aqui usamos tambm a verso de (II) para n = m trocando z por z + 1). Isto prova (II) para n = m + 1.
EUREKA! N23, 2006

55

Sociedade Brasileira de Matemtica

Supondo ainda para n = m ( m < ) que

Pm ( x, y, z ) = Pm ( x, z, y ) = Pm ( z, y, x ) = Pm ( y, x, z ) , temos: Pm+1 ( x, y, z ) Pm+1 ( y, x, z ) = ( x + z )( y + z ) Pm ( x, y, z + 1) z Pm ( x, y, z )


2

2 ( y + z )( x + z ) Pm ( y, x, z + 1) z Pm ( y, x, z ) . Mas da hiptese de induo: Pm+1 ( x, y, z ) Pm+1 ( y, x, z ) = ( x + z )( y + z ) Pm ( x, y, z + 1) Pm ( y, x, z + 1)

z2 Pm ( x, y, z ) Pm ( y, x, z ) = 0 Pm+1 ( x, y, z ) = Pm+1 ( y, x, z )
2 ( x + z )( y + x ) Pm ( z, y, x + 1) x Pm ( z, y, x ) =

Pm+1 ( x, y, z ) Pm+1 ( z, y, x ) = ( x + z )( y + z ) Pm ( x, y, z + 1) z Pm ( x, y, z )
2

= ( x + z) ( y + z ) Pm ( x, y, z + 1) ( y + x ) Pm ( z, y, x + 1) ( z x ) Pm ( z, y, x ) .

E, por hiptese de induo e (II) temos:

( y + z ) Pm ( x, y, z + 1) ( y + x ) Pm ( x + 1, y, z ) ( z x ) Pm ( x, y, z ) = ( y + z ) Pm ( y, x, z + 1) ( y + x ) Pm ( y, x + 1, z ) ( z x ) Pm ( y, x, z ) = 0 Pm+1 ( x, y, z ) = Pm+1 ( z, y, x ).
P m+1 ( x, z, y) = P m+1 ( z, x, y) = P m+1 ( y, x, z ) = P m+1(x, y, z), c.q.d.

A partir da temos:

107) a) Dado um tringulo qualquer, prove que existe um crculo que passa pelos pontos mdios dos seus lados, pelos ps das suas alturas e pelos pontos mdios dos segmentos que unem o ortocentro aos vrtices do tringulo (o chamado "crculo dos nove pontos"). b) Prove que, se X o centro do crculo dos nove pontos de um tringulo, H o seu ortocentro, O seu circuncentro e G seu baricentro, ento OX =
/ / / 3 1 OG = OH . 2 2

EUREKA! N23, 2006

56

Sociedade Brasileira de Matemtica

SOLUO DE ANDERSON TORRES e CARLOS ALBERTO DA SILVA VITOR (SO PAULO SP, NILPOLIS RJ)
A

TA HC H O MA HA

HB

M' H'

a) Seja H A o p altura por A, M A o ponto mdio de BC e TA o ponto mdio de AM. Aplicando uma homotetia de centro H e razo 2, o ponto TA levado no ponto A. Assim sendo, o que queremos provar que H ' e M ' (os pontos em que H A e M A so levados na homotetia) esto no circuncrculo de ABC. Temos que BM A H CM A M ' (de fato, LAL : BM A = CM A , BM A H = CM AM ', M A H = M A M ' ). Analogamente

HM AC M ' M A B. Com isto, BHC CM ' B pelo caso LLL (isto sai das
congruncias). Portanto BHC = BM ' C . BC mediatriz de HH ' (de fato HH A = H A H ' e HH A B = HH AC = 900 ). Assim, BHC BH ' C e assim BH ' C = BHC = BM ' C. Para finalizar, basta demonstrar que BHC + BAC = 180. E isto simples: BHC = H B HH C , e o quadriltero AH C HH B cclico

(AH C H + AH B H = 90 + 90 = 180 ).

Assim H B HH C + BAC = 180 BHC + BAC = 180. c.q.d. b) Se X o centro deste crculo, a homotetia leva X em O.

EUREKA! N23, 2006

57

Sociedade Brasileira de Matemtica

OH Assim HX = XO, e assim OX = . Pelo Lema abaixo, 2 / / / / / OH 3 OG = , c.q.d. OH = 3 OG , e fim: OX = 2 2


Lema: O baricentro, o circuncentro e o ortocentro de qualquer tringulo esto alinhados. Prova: Considere a figura abaixo: A

c O S A M

T b

R B

HA

AB = R (raio do crculo circunscrito). c = 2 R (da lei dos senos). sen C


Seja O o circuncentro de ABC. A reta OH intersecta AM em S. Observe que OM // AH A e que:

AT AT c = = cos A = 2 R cos A cos(90 C ) sen C sen C AH AS Do OBM OM = R cos A = e S o baricentro G do , logo MS = 2 2 ABC. / / Assim, O, G e H esto alinhados, e OH = 3 OG. AT = c cos A AH =

EUREKA! N23, 2006

58

B
Sociedade Brasileira de Matemtica

x D

PROBLEMAS PROPOSTOS
Convidamos o leitor a enviar solues dos problemas propostos e sugestes de novos problemas para os prximos nmeros.

108)

Sejam
1 i1 < i2 <...< ik n

Sk =

A1 , A2 ,..., An

conjuntos

finitos.

Para

1 k n,

seja

Ai1 Ai2 ... Aik , a soma dos nmeros de elementos das

intersees de k dos conjuntos Ai . Prove que: a) O nmero de elementos que pertencem a exatamente r dos conjuntos Ai

( 1)
k =r

kr

k S k , para 1 r n. r k 1 S k , para 1 r n. r 1
Mostre que

b) O nmero de elementos que pertencem a pelo menos r dos conjuntos Ai

( 1)
k =r

k r

109) Na figura abaixo, AB = AC , B AC = 100 e AD = BC .

x = BCD racional quando expresso em graus.


A

110) Um conjunto finito de inteiros positivos chamado de Conjunto DS se cada elemento divide a soma dos elementos do conjunto. Prove que todo conjunto finito de inteiros positivos subconjunto de algum conjunto DS.

EUREKA! N23, 2006

59

Sociedade Brasileira de Matemtica

111) Prove que existem infinitos mltiplos de 7 na seqncia (an ) abaixo:


a 1 = 1 9 9 9 , a n = a n- 1 + p ( n ) , " n 2

, onde p(n) o menor primo que divide n.

112) a) Determine todos os inteiros positivos n tais que existe uma matriz n n com todas as entradas pertencentes a { 1, 0, 1} tal que os 2n nmeros obtidos como somas dos elementos de suas linhas e de suas colunas so todos distintos. b) Para os inteiros positivos n determinados no item anterior, encontre o nmero de matrizes n n com a propriedade do enunciado.
Problema 109 proposto por Jos do Nascimento Pantoja Jnior de Ananindeua PA, problemas 110 e 111 propostos por Anderson Torres de So Paulo SP. Agradecemos tambm o envio das solues e a colaborao de: Diego Andrs de Barros Dymitri Cardoso Leo Geraldo Perlino Glauber Moreno Barbosa Kellem Corra Santos Macelo da Silva Mendes Marcos Francisco Ferreira Martinelli Michel Faleiros Martins Raphael Constant da Costa Rodrigo Cardaretti dos Nascimento Rodrigo Peres Barcellos Recife PE Recife PE Itapecerica da Serra SP Rio de Janeiro RJ Rio de Janeiro RJ Teresina PI Rio de Janeiro RJ Campinas SP Rio de Janeiro RJ Curitiba PR Rio de Janeiro RJ

'4.H 8,- ,
":0   
50 4 !$

 F 57 24 890 3 2074 /0


4

  /J 948 F 4 31472, 08

2, 47 57 24 .43 0. /4 0 F 4 ;0 ,

!7 24 /0 0780330 /08.4-0794 5,7, 2, 8

20780330 47

3. :8 ;0 84-70 .424 , :/,7 , ,. ,7 4:9748 57 248 /0 0780330 89, /08.4-079, 14 10 9, 02    

EUREKA! N23, 2006

60

Sociedade Brasileira de Matemtica

AGENDA OLMPICA
XXVIII OLIMPADA BRASILEIRA DE MATEMTICA NVEIS 1, 2 e 3 Primeira Fase Sbado, 10 de junho de 2006 Segunda Fase Sbado, 2 de setembro de 2006 Terceira Fase Sbado, 28 de outubro de 2006 (nveis 1, 2 e 3) Domingo, 29 de outubro de 2006 (nveis 2 e 3 - segundo dia de prova). NVEL UNIVERSITRIO Primeira Fase Sbado, 2 de setembro de 2006 Segunda Fase Sbado, 28 e Domingo, 29 de outubro de 2006

XII OLIMPADA DE MAIO 13 de maio de 2006

XVII OLIMPADA DE MATEMTICA DO CONE SUL 5 a 11 de maio de 2006 Escobar, Argentina

XLVII OLIMPADA INTERNACIONAL DE MATEMTICA 8 a 19 de julho de 2006 Ljubljana - Eslovnia.

XIII OLIMPADA INTERNACIONAL DE MATEMTICA UNIVERSITRIA 20 a 26 de julho de 2006 Odessa, Ucrnia

XXI OLIMPADA IBEROAMERICANA DE MATEMTICA 22 de setembro a 01 de outubro de 2006 Equador

EUREKA! N23, 2006

61

Sociedade Brasileira de Matemtica

COORDENADORES REGIONAIS
Alberto Hassen Raad Amrico Lpez Glvez Amarsio da Silva Arajo Andreia Goldani Antonio Carlos Nogueira Ali Tahzibi Benedito Tadeu Vasconcelos Freire Carlos Alexandre Ribeiro Martins Carlos Frederico Borges Palmeira Claus Haetinger Cleonor Crescncio das Neves Cludio de Lima Vidal Edson Roberto Abe lio Mega der Luiz Pereira de Andrade Eudes Antonio da Costa Florncio Ferreira Guimares Filho Ivanilde Fernandes Saad Janice T. Reichert Joo Bencio de Melo Neto Joo Francisco Melo Libonati Jorge Costa Duarte Filho Jos Cloves Saraiva Jos Luiz Rosas Pinho Jos Vieira Alves Jos William Costa Krerley Oliveira Licio Hernandes Bezerra Luzinalva Miranda de Amorim Mrio Rocha Retamoso Marcelo Rufino de Oliveira Marcelo Mendes Newman Simes Ral Cintra de Negreiros Ribeiro Ronaldo Alves Garcia Rogrio da Silva Igncio Reginaldo de Lima Pereira Reinaldo Gen Ichiro Arakaki Ricardo Amorim Srgio Cludio Ramos Seme Guevara Neto Tadeu Ferreira Gomes Toms Menndez Rodrigues Turbio Jos Gomes dos Santos Valdenberg Arajo da Silva Valdeni Soliani Franco Vnia Cristina Silva Rodrigues Wagner Pereira Lopes (UFJF) (USP) (UFV) FACOS (UFU) (USP) (UFRN) (Univ. Tec. Fed. De Paran) (PUC-Rio) (UNIVATES) (UTAM) (UNESP) (Colgio Objetivo de Campinas) (Colgio Etapa) (UNESPAR/FECILCAM) (Univ. do Tocantins) (UFES) (UC. Dom Bosco) (UNOCHAPEC) (UFPI) (Grupo Educacional Ideal) (UFPB) (UFMA) (UFSC) (UFPB) (Instituto Pueri Domus) (UFAL) (UFSC) (UFBA) (UFRG) (Grupo Educacional Ideal) (Colgio Farias Brito, Pr-vestibular) (Cursinho CLQ Objetivo) (Colgio Anglo) (UFGO) (Col. Aplic. da UFPE) (Escola Tcnica Federal de Roraima) (LAC - Laboratrio Associado de Computao) (Centro Educacional Logos) (IM-UFRGS) (UFMG) (UEBA) (U. Federal de Rondnia) (UFPB) (U. Federal de Sergipe) (U. Estadual de Maring) (U. Metodista de SP) (CEFET GO) Juiz de Fora MG Ribeiro Preto SP Viosa MG Osrio RS Uberlndia MG So Carlos SP Natal RN pato Branco - PR Rio de Janeiro RJ Lajeado RS Manaus AM S.J. do Rio Preto SP Campinas SP So Paulo SP Campo Mouro PR Arraias TO Vitria ES Campo Grande MS Chapec SC Teresina PI Belm PA Joo Pessoa - PB So Luis MA Florianpolis SC Campina Grande PB Santo Andr SP Macei AL Florianpolis SC Salvador BA Rio Grande RS Belm PA Fortaleza CE Piracicaba SP Atibaia SP Goinia GO Recife PE Boa Vista RR SJ dos Campos SP Nova Iguau RJ Porto Alegre RS Belo Horizonte MG Juazeiro BA Porto Velho RO Joo Pessoa PB So Cristovo SE Maring PR S.B. do Campo SP Jata GO

EUREKA! N23, 2006

62

CONTEDO

XXVII OLIMPADA BRASILEIRA DE MATEMTICA Problemas e Solues da Primeira Fase XXVII OLIMPADA BRASILEIRA DE MATEMTICA Problemas e Solues da Segunda Fase XXVII OLIMPADA BRASILEIRA DE MATEMTICA Problemas e Solues da Terceira Fase XXVII OLIMPADA BRASILEIRA DE MATEMTICA Problemas e Solues da Primeira Fase Nvel Universitrio XXVII OLIMPADA BRASILEIRA DE MATEMTICA Problemas e Solues da Segunda Fase Nvel Universitrio XXVII OLIMPADA BRASILEIRA DE MATEMTICA Premiados AGENDA OLMPICA COORDENADORES REGIONAIS

2 14

36

59

65

73 77 78

Sociedade Brasileira de Matemtica

XXVII OLIMPADA BRASILEIRA DE MATEMTICA


Problemas e Solues da Primeira Fase
PROBLEMAS NVEL 1 1. Sabendo-se que 9 174 532 13 = 119 268 916 , pode-se concluir que divisvel por 13 o nmero: A) 119 268 903 B) 119 268 907 C) 119 268 911 D) 119 268 913 E) 119 268 923 2. Numa caixa havia 3 meias vermelhas, 2 brancas e 1 preta. Professor Piraldo retirou 3 meias da caixa. Sabendo-se que nenhuma delas era preta, podemos afirmar sobre as 3 meias retiradas que: A) so da mesma cor. B) so vermelhas. B) uma vermelha e duas so brancas. D) uma branca e duas so vermelhas. E) pelo menos uma vermelha. 3. Diamantino colocou em um recipiente trs litros de gua e um litro de suco composto de 20% de polpa e 80% de gua. Depois de misturar tudo, que porcentagem do volume final polpa? A) 5% B) 7% C) 8% D) 20% E) 60% 4. Perguntado, Arnaldo diz que 1 bilho o mesmo que um milho de milhes. Professor Piraldo o corrigiu e disse que 1 bilho o mesmo que mil milhes. Qual a diferena entre essas duas respostas? A) 1 000 B) 999 000 C) 1 000 000 D) 999 000 000 E) 999 000 000 000 5. Numa seqncia, cada termo, a partir do terceiro, a soma dos dois termos anteriores mais prximos. O segundo termo igual a 1 e o quinto termo vale 2005. Qual o sexto termo? A) 3 002 B) 3 008 C) 3 010 D) 4 002 E) 5 004

EUREKA! N24, 2006

Sociedade Brasileira de Matemtica

6. Um galo de mel fornece energia suficiente para uma abelha voar 7 milhes de quilmetros. Quantas abelhas iguais a ela conseguiriam voar mil quilmetros se houvesse 10 gales de mel para serem compartilhados entre elas? A) 7 000 B) 70 000 C) 700 000 D) 7 000 000 E) 70 000 000 7. Trs anos atrs, a populao de Pirajussara era igual populao que Tucupira tem hoje. De l para c, a populao de Pirajussara no mudou mas a populao de Tucupira cresceu 50%. Atualmente, as duas cidades somam 9000 habitantes. H trs anos, qual era a soma das duas populaes? A) 3 600 B) 4 500 C) 5 000 D) 6 000 E) 7 500 8. Um agricultor esperava receber cerca de 100 mil reais pela venda de sua safra. Entretanto, a falta de chuva provocou uma perda da safra avaliada entre 1 1 e do total previsto. Qual dos valores a seguir pode representar a perda do 5 4 agricultor? A) R$ 21.987,53 B) R$ 34.900,00 C) R$ 44.999,99 D) R$ 51.987,53 E) R$ 60.000,00 9. Devido a um defeito de impresso, um livro de 600 pginas apresenta em branco todas as pginas cujos nmeros so mltiplos de 3 ou de 4. Quantas pginas esto impressas? A) 100 B) 150 C) 250 D) 300 E) 430 10. Seis retngulos idnticos so reunidos para formar um retngulo maior conforme indicado na figura. Qual a rea deste retngulo maior? B) 280 cm2 A) 210 cm2 2 C) 430 cm D) 504 cm2 2 E) 588 cm

21 cm

11. O relgio do professor Piraldo, embora preciso, diferente, pois seus ponteiros se movem no sentido anti-horrio. Se voc olhar no espelho o relgio quando ele estiver marcando 2h23min, qual das seguintes imagens voc ver?

EUREKA! N24, 2006

Sociedade Brasileira de Matemtica

E
A) B)

E
C)

E
D)

E
E)

12. Uma placa decorativa consiste num quadrado de 4 metros de lado, pintada de forma simtrica com algumas faixas, conforme indicaes no desenho ao lado. Qual a frao da rea da placa que foi pintada?

1m 1m 1m 1m 1m

A)

1 2

B)

1 3

C)

3 8

D)

6 13

E)

7 11

1m

13. Pelculas de insulfilm so utilizadas em janelas de edifcios e vidros de veculos para reduzir a radiao solar. As pelculas so classificadas de acordo com seu grau de transparncia, ou seja, com o percentual da radiao solar que ela deixa passar. Colocando-se uma pelcula de 70% de transparncia sobre um vidro com 90% de transparncia, obtm-se uma reduo de radiao solar igual a : A) 3% B) 37% C) 40% D) 63% E) 160% 14. Na figura, os dois tringulos so eqilteros. Qual o valor do ngulo x?
x

75

65

A) 30o

B) 40o

C) 50o

D) 60o
10 10 5 5

E) 70o

15. Um serralheiro solda varetas de metal para produzir peas iguais que sero juntadas para formar o painel abaixo. O desenho ao lado apresenta as medidas, em centmetros, de uma dessas peas. O serralheiro usa exatamente 20
EUREKA! N24, 2006

5 10

Sociedade Brasileira de Matemtica

metros de vareta para fazer o seu trabalho.

Qual dos desenhos abaixo representa o final do painel?

A)

B)

C)

D)

E)

16. Dentre os nmeros 1, 2, 3, 4, 5, 6, 7, 8, 9 e 10, escolha alguns e coloque-os nos crculos brancos de tal forma que a soma dos nmeros em dois crculos vizinhos seja sempre um quadrado perfeito. Ateno: o 2 j foi colocado em um dos crculos e no permitido colocar nmeros repetidos; alm disso, crculos separados pelo retngulo preto no so vizinhos.
2

A soma dos nmeros colocados em todos os crculos brancos : A) 36 B) 46 C) 47 D) 49 E) 55 17. Figuras com mesma forma representam objetos de mesma massa. Quantos quadrados so necessrios para que a ltima balana fique em equilbrio?

A) 7
EUREKA! N24, 2006

B) 8

C) 9

D) 10

E) 12

Sociedade Brasileira de Matemtica

18. As 10 cadeiras de uma mesa circular foram numeradas com nmeros consecutivos de dois algarismos, entre os quais h dois que so quadrados perfeitos. Carlos sentou-se na cadeira com o maior nmero e Janana, sua namorada, sentou-se na cadeira com o menor nmero. Qual a soma dos nmeros dessas duas cadeiras? A) 29 B) 36 C) 37 D) 41 E) 64 19. Em um ano, no mximo quantos meses tm cinco domingos? A) 3 B) 4 C) 5 D) 6

E) 7

20. As nove casas de um tabuleiro 3 3 devem ser pintadas de foram que cada coluna, cada linha e cada uma das duas diagonais no tenham duas casas de mesma cor. Qual o menor nmero de cores necessrias para isso? A) 3 B) 4 C) 5 D) 6 E) 7

PROBLEMAS NVEL 2 1. Uma loja de sabonetes realiza uma promoo com o anncio "Compre um e leve outro pela metade do preo. Outra promoo que a loja poderia fazer oferecendo o mesmo desconto percentual A) "Leve dois e pague um B) "Leve trs e pague um C) "Leve trs e pague dois D) "Leve quatro e pague trs E) "Leve cinco e pague quatro 2. 3. 4. 5. Veja o problema No. 13 do Nvel 1. Veja o problema No. 10 do Nvel 1. Veja o problema No. 4 do Nvel 1. Veja o problema No. 9 do Nvel 1.

6. Platina um metal muito raro, mais raro at do que ouro. Sua densidade 21,45 g/cm3. Suponha que a produo mundial de platina foi de cerca de 110 toneladas em cada um dos ltimos 50 anos e desprezvel antes disso. Assinale a alternativa com o objeto cujo volume mais prximo do volume de platina produzido no mundo em toda a histria. A) uma caixa de sapatos B) uma piscina C) um edifcio de dez andares D) o monte Pascoal E) a Lua 7. Veja o problema No. 5 do Nvel 1. 8. Veja o problema No. 17 do Nvel 1.
EUREKA! N24, 2006

Sociedade Brasileira de Matemtica

9. Entre treze reais no nulos h mais nmeros positivos do que negativos. Dentre os
13 12 = 78 produtos de dois 2

dos treze nmeros, 22 so

negativos. Quantos nmeros dentre os treze nmeros dados so negativos? A) 2 B) 7 C) 8 D) 9 E) 10 10. O desenho ao lado mostra um pedao de papelo que ser dobrado e colado nas bordas para formar uma caixa retangular. Os ngulos nos cantos do papelo so todos retos. Qual ser o volume da caixa em cm3?
15 cm

40 cm

A) 1 500 E) 12 000

B) 3 000

C) 4 500

20 cm

D) 6 000

11. Sendo a, b e c nmeros reais, pela propriedade distributiva da multiplicao em relao adio, verdade que a (b + c) = (a b) + (a c). A distributiva da adio em relao multiplicao a + (b c) = (a + b) (a + c) no sempre verdadeira, mas ocorre se, e somente se, A) a = b = c = 1 ou a = 0
3

B) a = b = c D) a + b + c = 1 ou a = 0

C) A igualdade nunca ocorre E) a = b = c = 0

12. Em certa cidade, acontece um fato interessante. Dez por cento dos Baianos dizem que so Paulistas e dez por cento dos Paulistas dizem que so Baianos. Todos os outros Paulistas e Baianos assumem a sua verdadeira origem. Dentre os Paulistas e Baianos, 20% dizem que so Paulistas. Que percentual os realmente Paulistas representam dentre os Paulistas e Baianos? A) 12,5% B) 18% C) 20% D) 22% E) 22,5% 13. Veja o problema No. 14 do Nvel 1.
EUREKA! N24, 2006

Sociedade Brasileira de Matemtica

14. As letras O, B e M representam nmeros inteiros. Se O B M = 240, O B + M = 46 e O + B M = 64, quanto vale O + B + M? A) 19 B) 20 C) 21 D) 24 E) 36 15. Veja o problema No. 15 do Nvel 1. 16. Veja o problema No. 19 do Nvel 1. 17. Quantos nmeros entre 10 e 13000, quando lidos da esquerda para a direita, so formados por dgitos consecutivos e em ordem crescente? Exemplificando, 456 um desses nmeros, mas 7890 no : A) 10 B) 13 C) 18 D) 22 E) 25 18. Um piloto percorreu trs trechos de um rali, de extenses 240 km, 300 km e 400 km, respectivamente. As velocidades mdias nos trs trechos foram 40 km/h, 75 km/h e 80 km/h, mas no necessariamente nessa ordem. Podemos garantir que o tempo total em horas gasto pelo piloto nos trs trechos : A) menor ou igual a 13 horas B) maior ou igual a 13 horas e menor ou igual a 16 horas C) maior ou igual a 14 horas e menor ou igual a 17 horas D) maior ou igual a 15 horas e menor ou igual a 18 horas E) maior ou igual a 18 horas 19. Na figura, todas as circunferncias menores tm o mesmo raio r e os centros das circunferncias que tocam a circunferncia maior so vrtices de um a quadrado. Sejam a e b as reas cinzas indicadas na figura. Ento a razo b igual a:

b a

A) 1
2

B) 2
3

C) 1

D) 3
2

E) 2

EUREKA! N24, 2006

Sociedade Brasileira de Matemtica

20. Um professor de Ingls d aula particular para uma classe de 9 alunos, dos quais pelo menos um brasileiro. Se o professor escolher 4 alunos para fazer uma apresentao, ter no grupo pelo menos dois alunos de mesma nacionalidade; se escolher 5 alunos, ter no mximo trs alunos de mesma nacionalidade. Quantos brasileiros existem na classe? A) 1 B) 2 C) 3 D) 4 E) 5

21. Um relgio, com ponteiros de horas, minutos e segundos, faz plim toda vez que um ponteiro ultrapassa outro no mostrador. O nmero de plins registrados em um certo dia, no perodo entre as 12 horas e 1 segundo e as 23 horas, 59 minutos e 59 segundos : A) 732 B) 1438 C) 1440 D) 1446 E) 1452

22. Na figura, a reta PQ toca em N o crculo que passa por L, M e N. A reta LM corta a reta PQ em R. Se LM = LN e a medida do ngulo PNL , < 60o, quanto mede o ngulo LRP?
L

P N R Q

A)3 180o

B)180o 2

C) 180o

D) 90o /2

E)

23. Os inteiros positivos x e y satisfazem a equao

x+ 1 y x 1 y =1. 2 2
Qual das alternativas apresenta um possvel valor de y? A) 5 B) 6 C) 7 D) 8 E) 9

24. Veja o problema No. 16 do Nvel 1.

EUREKA! N24, 2006

Sociedade Brasileira de Matemtica

25. Um bloco de dimenses 1 2 3 colocado sobre um tabuleiro 8 8, como mostra a figura, com a face X, de dimenses 1 2, virada para baixo. Giramos o bloco em torno de uma de suas arestas de modo que a face Y fique virada para baixo. Em seguida, giramos novamente o bloco, mas desta vez de modo que a face Z fique virada para baixo. Giramos o bloco mais trs vezes, fazendo com que as faces X, Y e Z fiquem viradas para baixo, nessa ordem. Quantos quadradinhos diferentes do tabuleiro estiveram em contato com o bloco?

Z Y

A) 18 PROBLEMAS NVEL 3

B) 19

C) 20

D) 21

E)22

1. Veja o problema No. 17 do Nvel 2. 2. Os pontos L, M e N so pontos mdios de arestas do cubo, como mostra a figura. Quanto mede o ngulo LMN?
N

A) 90

B) 105

C) 120o

D) 135o

E) 150o

3. Veja o problema No. 22 do Nvel 2. 4. Veja o problema No. 14 do Nvel 2.


EUREKA! N24, 2006

10

Sociedade Brasileira de Matemtica

5. Esmeralda digitou corretamente um mltiplo de 7 muito grande, com 4010 algarismos. Da esquerda para a direita, os seus algarismos so 2004 algarismos 1, um algarismo n e 2005 algarismos 2. Qual o valor de n? A) 3 B) 4 C) 5 D) 6 E) 7 6. 7. 8. 9. Veja o problema No. 23 do Nvel 2. Veja o problema No. 25 do Nvel 2. Veja o problema No. 1 do Nvel 2. Veja o problema No. 6 do Nvel 2.

10. A figura mostra um cubo de aresta 1 no qual todas as doze diagonais de face foram desenhadas. Com isso, criou-se uma rede com 14 vrtices (os 8 vrtices do cubo e os 6 centros de faces) e 36 arestas (as 12 arestas do cubo e mais 4 sobre cada uma das 6 faces). Qual o comprimento do menor caminho que formado por arestas da rede e que passa por todos os 14 vrtices?

A) 1 + 6 2 E) 12 + 12 2

B) 4 + 2 2

C) 6

D) 8 + 6 2

11. Uma das faces de um poliedro um hexgono regular. Qual a quantidade mnima de arestas que esse poliedro pode ter? A) 7 B) 9 C) 12 D) 15 E) 18 12. Veja o problema No. 19 do Nvel 1. 13. O ponto D pertence ao lado BC do tringulo ABC. Sabendo que AB = AD = 2, ngulos BAD e CAD so congruentes, ento a medida do BD = 1 e os segmento CD : 3 4 5 6 7 A) B) C) D) E) 2 3 4 5 6

EUREKA! N24, 2006

11

Sociedade Brasileira de Matemtica

14. Esmeralda adora os nmeros triangulares (ou seja, os nmeros 1, 3, 6, 10, 15, 21, 28), tanto que mudou de lugar os nmeros 1, 2, 3, , 11 do relgio de parede do seu quarto de modo que a soma de cada par de nmeros vizinhos um nmero triangular. Ela deixou o 12 no seu lugar original. Que nmero ocupa o lugar que era do 6 no relgio original? A) 1 B) 4 C) 5 D) 10 E) 11 15. Os termos an de uma seqncia de inteiros positivos satisfazem a relao an+3 = an+2(an+1 + an) para n = 1, 2, 3 Se a5 = 35, quanto a4? A) 1 B) 3 C) 5 D) 7 E) 9 16. Veja o problema No. 11 do Nvel 2. 17. Veja o problema No. 19 do Nvel 2. 18. Entre treze no nulos h mais nmeros positivos do que 13 12 negativos. Dentre os = 78 produtos de dois dos treze nmeros, 22 2 so negativos. Quantos nmeros dentre os treze nmeros dados so negativos? A) 2 B) 7 C) 8 D) 9 E) 10 reais

19. Traando as quatro retas perpendiculares aos lados de um paralelogramo no retngulo pelos seus pontos mdios, obtm-se uma regio do plano limitada por essas quatro retas. Podemos afirmar que a rea dessa regio igual rea do paralelogramo se um dos ngulos do paralelogramo for igual a: A) 30o B) 45o C) 60o D) 75o E) 90o 20. O nmero (2 + 2)3 (3 2) 4 + (2 2)3 (3 + 2) 4 : A) inteiro mpar B) inteiro par C) racional no inteiro D) irracional positivo irracional negativo 21. Sejam A = 10(log10 2005) , B = 20053 e C = 2 A) A < B < C B) A < C < B C) B < A < C D) B < C < A
2

E)

2005

. Ento: E) C < A < B

22. Veja o problema No. 18 do Nvel 2.

EUREKA! N24, 2006

12

Sociedade Brasileira de Matemtica

23. Dois nmeros inteiros so chamados de primanos quando pertencem a uma progresso aritmtica de nmeros primos com pelo menos trs termos. Por exemplo, os nmeros 41 e 59 so primanos pois pertencem progresso aritmtica (41; 47; 53; 59) que contm somente nmeros primos. Assinale a alternativa com dois nmeros que no so primanos. A) 7 e 11 B) 13 e 53 C) 41 e 131 D) 31 e 43 E) 23 e 41 24. Um relgio, com ponteiros de horas, minutos e segundos, faz plim toda vez que um ponteiro ultrapassa outro no mostrador. O nmero de plins registrados em um certo dia no perodo entre as 12 horas e 1 segundo e as 23 horas, 59 minutos e 59 segundos : A) 732 B) 1438 C) 1440 D) 1446 E) 1452 25. Veja o problema No. 20 do Nvel 2.

GABARITO
NVEL 1 (5a. e 6a. sries) 1) A 6) B 2) E 7) E 3) A 8) A 4) E 9) D 5) B 10) E 11) A 12) C 13) B 14) B 15) B 16) B 17) D 18) D 19) C 20) C

NVEL 2 (7a. e 8a. sries) 1) D 6) B 2) B 7) B 3) E 8) D 4) E 9) A 5) D 10) B NVEL 3 (Ensino Mdio) 1) D 6) C 2) C 7) B 3) Anulada 8) D 4) B 9) B 5) B 10) A

11) D 12) A 13) B 14) B 15) B

16) C 17) D 18) Anulada 19) C 20) C

21) Anulada 22) Anulada 23) C 24) B 25) B

11) C 12) C 13) B 14) C 15) D

16) D 17) C 18) A 19) B 20) B

21) C 22) Anulada 23) B 24) Anulada 25) C

EUREKA! N24, 2006

13

Sociedade Brasileira de Matemtica

XXVII OLIMPADA BRASILEIRA DE MATEMTICA


Problemas e Solues da Segunda Fase PROBLEMAS Nvel 1 PARTE A (Cada problema vale 5 pontos)
01. O tanque do carro de Esmeralda, com capacidade de 60 litros, contm uma

mistura de 20% de lcool e 80% de gasolina ocupando metade de sua capacidade. Esmeralda pediu para colocar lcool no tanque at que a mistura ficasse com quantidades iguais de lcool e gasolina. Quantos litros de lcool devem ser colocados?
02. Na seqncia de nmeros 1, a, 2, b, c, d, ... dizemos que o primeiro termo 1, o

segundo termo a, o terceiro termo 2, o quarto termo b, e assim por diante. Sabe-se que esta seqncia tem 2005 termos e que cada termo, a partir do terceiro, a mdia aritmtica de todos os termos anteriores. Qual o ltimo termo dessa seqncia?
03. Natasha supersticiosa e, ao numerar as 200 pginas de seu dirio, comeou do 1 mas pulou todos os nmeros nos quais os algarismos 1 e 3 aparecem juntos, em qualquer ordem. Por exemplo, os nmeros 31 e 137 no aparecem no dirio, porm 103 aparece. Qual foi o nmero que Natasha escreveu na ltima pgina do seu dirio? 04. Juliana foi escrevendo os nmeros inteiros positivos em quadrados de papelo,

colados lado a lado por fitas adesivas representadas pelos retngulos escuros no desenho abaixo. Note que cada fila de quadrados tem um quadrado a mais que a fila de cima. Ela escreveu at o nmero 105 e parou. Quantos pedaos de fita adesiva ela usou?
1 2 4 7 3 5 8 6 9 10

05. Lara tem cubos iguais e quer pint-los de maneiras diferentes, utilizando as cores laranja ou azul para colorir cada uma de suas faces.
EUREKA! N24, 2006

14

Sociedade Brasileira de Matemtica

Para que dois cubos no se confundam, no deve ser possvel girar um deles de forma que fique idntico ao outro. Por exemplo, h uma nica maneira de pintar o cubo com uma face laranja e cinco azuis. Quantos cubos pintados de modos diferentes ela consegue obter?
06. Um carpinteiro fabrica caixas de madeira abertas na parte de cima, pregando duas placas retangulares de 600 cm2 cada uma, duas placas retangulares de 1200 cm2 cada uma e uma placa retangular de 800 cm2, conforme representado no desenho. Qual o volume, em litros, da caixa? Note que l litro = 1000 cm3.

PROBLEMAS Nvel 1 PARTE B (Cada problema vale 10 pontos)


PROBLEMA 1

Quatro peas iguais, em forma de tringulo retngulo, foram dispostas de dois modos diferentes, como mostram as figuras. H I D A L B K C E P J M N G

F Os quadrados ABCD e EFGH tm lados respectivamente iguais a 3 cm e 9 cm. Calcule as reas dos quadrados IJKL e MNOP.

EUREKA! N24, 2006

15

Sociedade Brasileira de Matemtica

PROBLEMA 2

Considere trs nmeros inteiros positivos consecutivos de trs algarismos tais que o menor mltiplo de 7, o seguinte mltiplo de 9 e o maior mltiplo de 11. Escreva todas as seqncias de nmeros que satisfazem essas propriedades.
PROBLEMA 3

Cada pea de um jogo de domin possui duas casas numeradas. Considere as 6 peas formadas apenas pelos nmeros 1, 2 e 3. (a) De quantos modos possvel colocar todas estas peas alinhadas em seqncia, de modo que o nmero da casa da direita de cada pea seja igual ao nmero da casa da esquerda da pea imediatamente direita? A seguir, mostramos dois exemplos:

(b) Explique por que no possvel fazer o mesmo com todas as 10 peas formadas apenas pelos nmeros 1, 2, 3 e 4. PROBLEMAS Nvel 2 PARTE A (Cada problema vale 4 pontos)
01. Veja o problema No. 3 do Nvel 1 Parte A. 02. Quatro peas iguais, em forma de tringulo retngulo, foram dispostas de dois modos diferentes, como mostram as figuras abaixo.

H I D A B L K E P F O C J M N G

EUREKA! N24, 2006

16

Sociedade Brasileira de Matemtica

Os quadrados ABCD e EFGH tm lados respectivamente iguais a 3 cm e 9 cm. Determine a medida do lado do quadrado IJKL.
03. Veja o problema No. 4 do Nvel 1 parte A. 04. Um terreno quadrangular foi dividido em quatro lotes menores por duas cercas retas unindo os pontos mdios dos lados do terreno. As reas de trs dos lotes esto indicadas em metros quadrados no mapa a seguir.

250

200

210

Qual a rea do quarto lote, representado pela regio escura no mapa?


05. Seja a um nmero inteiro positivo tal que a mltiplo de 5, a + 1 mltiplo de

7, a + 2 mltiplo de 9 e a + 3 mltiplo de 11. Determine o menor valor que a pode assumir. PROBLEMAS Nvel 2 PARTE B (Cada problema vale 10 pontos)
PROBLEMA 1

Gabriel resolveu uma prova de matemtica com questes de lgebra, geometria e lgica. Aps checar o resultado da prova Gabriel observou que respondeu corretamente 50% das questes de lgebra, 70% das questes de geometria e 80% das questes de lgica. Gabriel observou, tambm, que respondeu corretamente 62% das questes de lgebra e lgica e 74% das questes de geometria e lgica. Qual a porcentagem de questes corretas da prova de Gabriel?
PROBLEMA 2

O canto de um quadrado de cartolina foi cortado com uma tesoura. A soma dos comprimentos dos catetos do tringulo recortado igual ao comprimento do lado do quadrado. Qual o valor da soma dos ngulos e marcados na figura abaixo?

EUREKA! N24, 2006

17

Sociedade Brasileira de Matemtica

27

PROBLEMA 3

(a) Fatore a expresso x 2 9 xy + 8 y 2 . (b) Determine todos os pares de inteiros (x; y) tais que 9 xy x 2 8 y 2 = 2005 .
PROBLEMA 4

Veja o problema No. 3 do Nvel 1 Parte B.

PROBLEMAS Nvel 3 PARTE A (Cada problema vale 4 pontos)

01. Na figura, ABCDE um pentgono regular e AEF um tringulo eqiltero. Seja P um ponto sobre o segmento BF , no interior de ABCDE, e tal que o ngulo A mede 12, como mostra a figura abaixo. PE
F A P B E

Calcule a medida, em graus, do ngulo PC.

EUREKA! N24, 2006

18

Sociedade Brasileira de Matemtica

02. Seja a um nmero inteiro positivo tal que a mltiplo de 5, a + 1 mltiplo de

7, a + 2 mltiplo de 9 e a + 3 mltiplo de 11. Determine o menor valor que a pode assumir.


03. Veja o problema No. 4 do Nvel 2 parte A. 04. A funo f :

satisfaz f ( x + f ( y )) = x + f ( f ( y )) para todos os nmeros reais x e y. Sabendo que f (2) = 8 , calcule f(2005).

05. Voc tem que determinar o polinmio p(x) de coeficientes inteiros positivos

fazendo perguntas da forma Qual o valor numrico de p(k)?, sendo k um inteiro positivo sua escolha. Qual o menor nmero de perguntas suficiente para garantir que se descubra o polinmio? PROBLEMAS Nvel 3 PARTE B (Cada problema vale 10 pontos)
PROBLEMA 1

Determine todos os pares de inteiros (x; y) tais que 9 xy x 2 8 y 2 = 2005 .


PROBLEMA 2

Um prisma reto e tem como base um tringulo equiltero. Um plano corta o prisma mas no corta nenhuma de suas bases, determinando uma seco triangular de lados a, b e c. Calcule o lado da base do prisma em funo de a, b e c.
PROBLEMA 3

No campeonato tumboliano de futebol, cada vitria vale trs pontos, cada empate vale um ponto e cada derrota vale zero ponto. Um resultado uma vitria, empate ou derrota. Sabe-se que o Flameiras no sofreu nenhuma derrota e tem 20 pontos, mas no se sabe quantas partidas esse time jogou. Quantas seqncias ordenadas de resultados o Flameiras pode ter obtido? Representando vitria por V, empate por E e derrota por D, duas possibilidades, por exemplo, so (V, E, E, V, E, V, V, V, E, E) e (E, V, V, V, V, V, E, V).
PROBLEMA 4

Determine o menor valor possvel do maior termo de uma progresso aritmtica com todos os seus sete termos a1, a2, a3, a4, a5, a6, a7 primos positivos distintos.
EUREKA! N24, 2006

19

Sociedade Brasileira de Matemtica

Curiosidade: No ano passado, os ex-olmpicos Terence Tao (Austrlia, ouro na IMO 1988) e Ben Green (Reino Unido, prata na IMO 1994) provaram que existem progresses aritmticas arbitrariamente grandes com todos os termos primos positivos. Tal questo remonta ao sculo XVIII, aparecendo nas pesquisas de Lagrange e Waring. Solues Nvel 1 Segunda Fase Parte A Problema Resposta 01 18 02 2 03 214 04 182 05 10 06 24

01. O tanque contm uma mistura de 30 litros, sendo 0, 2 30 = 6 litros de lcool e 30 6 = 24 litros de gasolina. Portanto, para que as quantidades de gasolina e lcool fiquem iguais, devem ser colocados no tanque 24 6 = 18 litros de lcool. 02. Como 2 a mdia aritmtica de 1 e a, podemos escrever

1+ a = 4 a = 3; d=

portanto,

b=

1+ 3 + 2 + 2 + 2 = 2 . Esses exemplos sugerem que todos os termos, a 5

1+ 2 + 3 = 2; 3

1+ a = 2 , logo 2 1+ 3 + 2 + 2 = 2; c= 4

partir do terceiro, so iguais a 2. De fato, quando introduzimos em uma seqncia um termo igual mdia de todos os termos da seqncia, a mdia da nova seqncia a mesma que a da seqncia anterior. Assim, o ltimo termo da seqncia dada 2.
03. Natasha pulou os nmeros 13, 31, 113, 130,131, 132, ..., 139, num total de 13

nmeros. Portanto, na ltima pgina do seu dirio escreveu o nmero 200 + 13 +1 = 214. 04. Olhando para o ltimo nmero da fila n, vemos que ele a soma de todos os nmeros de 1 a n: por exemplo, na fila 4, o ltimo nmero da fila 1 + 2 + 3 + 4 = 10. Note que para obter a quantidade de nmeros at uma certa fila, basta somar o nmero da fila ao total de nmeros que havia antes dessa fila. Assim, temos, fila 5 : 15, fila 6: 21, fila 7: 28, fila 8: 36, fila 9: 45, fila 10: 55, fila 11: 66, fila 12: 78, fila 13: 91, fila 14: 105
EUREKA! N24, 2006

20

Sociedade Brasileira de Matemtica

O nmero de fitas adesivas horizontais entre uma fila n 1 e uma fila n igual a n 1 e o nmero de fitas adesivas verticais numa fila n igual n 1. Portanto, at a fila nmero 14, o nmero de fitas

(1 + 2 +  + 13) + (1 + 2 +  + 13 ) = 2

13 14 = 182. 2

05. Todas as faces azuis: uma maneira. Cinco faces azuis e uma amarela: uma maneira. Quatro faces azuis e duas amarelas: duas maneiras (duas faces amarelas opostas ou duas faces amarelas adjacentes). Trs faces azuis e trs faces amarelas: duas maneiras (trs azuis com um vrtice comum uma maneira ou trs azuis com uma aresta comum duas a duas uma maneira) Duas faces azuis e quatro amarelas: duas maneiras Uma face azul e cinco amarelas: uma maneira. Todas as faces amarelas: uma maneira. Portanto, o nmero de maneiras diferentes de pintar o cubo 10. 06. Sejam a, b e c as medidas da caixa, conforme indicado no desenho ao lado. Segundo o enunciado, podemos escrever ab = 600, ac = 1200 e bc = 800. Sabemos que o volume da caixa abc. Utilizando as propriedades das igualdades e de potncias, podemos escrever

( ab ) ( ac ) (bc ) = 600 1200 800 a 2 b2 c2 = 2 3 102 22 3 102 23 102 2 ( abc ) = 26 32 106 abc = 26 32 106 abc = 23 3 103 = 24 1000 cm3

Como 1 litro igual a 1000 cm3, conclumos que o volume da caixa de 24 litros. Solues Nvel 1 Segunda Fase Parte B
SOLUO DO PROBLEMA 1: 1 maneira: O quadrado IJKL e o quadrado MNOP tm como lados as hipotenusas

dos tringulos retngulos dados, logo tm a mesma rea s. Fazendo os dois quadrados coincidirem, conclumos que o dobro da soma t das reas dos quatro tringulos retngulos a diferena entre as reas dos quadrados IJKL e EFGH, ou seja, 2t 92 32 2t 72 t 36 . Assim, s = 9 + 36 = 81 36 = 45 cm2.

2 maneira: No quadrado IJKL, seja JC = x. Ento IC = ID + DC = JC + DC = x + 3. Ento, no quadrado EFGH, temos . Portanto, a rea do quadrado HN NG x 3 x 9 2 x 6 x 3

EUREKA! N24, 2006

21

Sociedade Brasileira de Matemtica

IJKL, igual soma das reas dos quatro tringulos retngulos com a rea do quadrado ABCD, vale 4

3 3 3 2

32

36

45 e a rea do quadrado

MNOP, igual diferena entre a rea do quadrado EFGH e a soma das reas dos quatro tringulos retngulos, vale 92
SOLUO DO PROBLEMA 2:

3 3 3 2

81 36

45 cm2.

Seja n = abc mltiplo de 11; ento n 1 deve ser mltiplo de 9 e n 2 deve ser mltiplo de 7. Seja c 0 : Como abc mltiplo de 11, podemos ter a b c 0 ou a b c 11 . Como abc 1 mltiplo de 9, podemos ter a b c 1 9 ou a b c 1 18 . No caso de a + b + c 1 = 0 , teramos n 1 = 99 n = 100 , que no mltiplo de 11. Assim, simultaneamente, somente podemos ter (i )

a + b + c = 10 2b = 10 b=5 a+c =b a+c=b a+c=5

ou

(ii )

a + b + c = 19 2b + 11 = 19 b=4 a + c = b + 11 a + c = b + 11 a + c = 15

No caso (i) existem as seguintes possibilidades para n: 154, 253, 352, 451, que so mltiplos de 11; para n 1 temos os nmeros 153, 252, 351, 450 e 549 so mltiplos de 9. Para os nmeros n 2 temos 152, 251, 350, 449 e 548, dos quais apenas 350 mltiplo de 7. No caso (ii) existem as seguintes possibilidades para n: 649, 748, 847 e 946, que so mltiplos de 11; para n 1 temos os nmeros 648, 747, 846 e 945 so mltiplos de 9. Para os nmeros n 2 temos 647, 746, 845 e 944, dos quais nenhum mltiplo de 7. Seja c = 0: Neste caso, n 1 tem os algarismos a, b 1 e 9. Assim, a + b 1 + 9 = 9 ou a + b 1 + 9 = 18 ou seja, a + b = 1 ou a + b = 10 . Como a b c a b 0 ou a b c a b 11 , conclumos que a = b. Assim, a = b = 5, o que fornece os nmeros n = 550, n 1 = 549 e n 2 = 548, que no divisvel por 7. Portanto, a nica seqncia de trs nmeros inteiros consecutivos nas condies dadas 350, 351 e 352.
EUREKA! N24, 2006

22

Sociedade Brasileira de Matemtica

SOLUO DO PROBLEMA 3: 1a maneira:

a) Podemos representar uma seqncia vlida como uma seqncia de pares ordenados. O primeiro exemplo a seqncia [(1,1),(1,2),(2,2),(2,3),(3,3),(3,1)] e, a partir dela, podemos criar outras seqncias vlidas movendo o par da esquerda para a direita (ou da direita para a esquerda). Assim, so vlidas as seqncias [(1,2),(2,2),(2,3),(3,3),(3,1),(1,1)], [(2,2),(2,3),(3,3),(3,1),(1,1), (1,2)],etc. num total de 6 seqncias diferentes. Mudando a posio dos nmeros dos pares ordenados, podemos criar outras 6 seqncias: [(2,1), (1,1), (1,3), (3,3),(3,2),(2,2)], [ (1,1), (1,3), (3,3),(3,2),(2,2), (2,1)], etc. Portanto, de acordo com as regras dadas h 12 modos de colocar as peas em seqncia.
2a maneira:

a) As pontas devem ter o mesmo nmero, pois eles aparecem um nmero par de vezes (se aparecer um nmero numa ponta e outro na outra, ento h pelo menos dois nmeros que aparecem um nmero mpar de vezes, o que no ocorre). Alguma pea com dois nmeros iguais deve aparecer em uma das pontas, pois do contrrio teramos trs das quatro peas centrais com duas iguais, vizinhas, o que impossvel). Sendo assim, a seqncia pode ser representada por XX-XY-YY-YZ-ZZ-ZX, onde para X temos trs possibilidades, para Y temos duas possibilidade e para Z, uma possibilidade, num total de 3.2.1 = 6 possibilidades para a seqncia que comea com uma dupla. Se a seqncia terminar com uma dupla, teremos novamente 6 possibilidades. Portanto, h 12 modos de colocar as seis peas em seqncia. b) Para cada nmero, existem 4 peas. Por exemplo, as peas com o nmero 1 esto desenhadas ao lado. O nmero de vezes em que aparece o nmero 1 mpar, logo a seqncia deveria comear com 1 e terminar com outro nmero ou comear com outro nmero e terminar com 1. Neste caso, os outros dois nmeros deveriam aparecer um nmero par de vezes, pois no estariam na ponta, mas isso no ocorre: todos os quatro nmeros aparecem um nmero mpar de vezes.

EUREKA! N24, 2006

23

Sociedade Brasileira de Matemtica

Solues Nvel 2 Segunda Fase Parte A Problema Resposta 01 214 02 -------03 182 04 240 05 1735

01. Natasha pulou os nmeros 13, 31, 113, 130,131, 132, ..., 139, num total de 13 nmeros. Portanto, na ltima pgina do seu dirio escreveu o nmero 200 + 13 +1 = 214. 02. Sejam x e y o maior e o menor catetos, respectivamente, do tringulo retngulo. Como o lado do quadrado ABCD mede 3 cm, temos x y = 3. Por outro lado, como o lado de EFGH mede 9 cm, temos x + y = 9. Resolvendo o sistema, encontramos x = 6 e y = 3. Logo, o lado do quadrado IJKL, que a hipotenusa do tringulo retngulo, mede

6 2 + 32 = 45 = 3 5 cm.

OUTRA SOLUO: O quadrado IJKL e o quadrado MNOP tm como lados as

hipotenusas dos tringulos retngulos dados, logo tm a mesma rea s. Fazendo os dois quadrados coincidirem, conclumos que o dobro da soma t das reas dos quatro tringulos retngulos a diferena entre as reas dos quadrados IJKL e EFGH, ou seja, 2t = 92 32 , o que fornece t = 36.. Assim, s = 9 + 36 = 81 36 = 45 cm2 e o lado do quadrado IJKL

45 = 3 5 cm.

03. Olhando para o ltimo nmero da fila n, vemos que ele a soma de todos os nmeros de 1 a n: por exemplo, na fila 4, o ltimo nmero da fila 1 + 2 + 3 + 4 = 10. Note que para obter a quantidade de nmeros at uma certa fila, basta somar o nmero da fila ao total de nmeros que havia antes dessa fila. Assim, temos, fila 5 : 15, fila 6: 21, fila 7: 28, fila 8: 36, fila 9: 45, fila 10: 55, fila 11: 66, fila 12: 78, fila 13: 91, fila 14: 105 O nmero de fitas adesivas horizontais entre uma fila n 1 e uma fila n igual a n 1 e o nmero de fitas adesivas verticais numa fila n igual n 1. Portanto, at a fila nmero 14, o nmero de fitas

(1 + 2 +  + 13) + (1 + 2 +  + 13) = 2

13 14 = 182. 2

04. Primeira Soluo: Unindo os pontos mdios de lados consecutivos do quadriltero, obtemos segmentos paralelos s suas diagonais e iguais metade delas. Portanto, o quadriltero assim obtido um paralelogramo. Os
EUREKA! N24, 2006

24

Sociedade Brasileira de Matemtica

segmentos traados dividem cada um dos quatro lotes em duas partes. Todas as partes internas tm a mesma rea s, igual a 1/4 da rea do paralelogramo. Cada uma das partes externas tem rea igual a 1/4 do tringulo determinado pela diagonal correspondente. Assim, a + c igual metade da rea do quadriltero, o mesmo ocorrendo com b + c. Da, a + s + c + s = b + s + d + s. Portanto, a rea S desconhecida satisfaz S + 210 = 200 + 250, ou seja, S = 240.

b a s s d s c s

Segunda Soluo: Ligando o ponto de interseo das retas que representam as duas cercas aos vrtices, obtemos:
M A B

O Q

Observemos que, como AQ = QD e as alturas de OAQ e OQD que passam por O so iguais, as reas de OAQ e OQD so iguais. Analogamente, as reas de OAM e OMB; OBN e ONC; OCP e OPD so iguais. Logo rea OAQ + rea OAM + rea OCP + rea ONC = rea OQD + rea OMB + rea OPD + rea OBN rea AMOQ + rea CNOP = rea DPOQ + rea BMON rea AMOQ = 200 + 250 210 = 240. 05. Como a + 3 mltiplo de 11, a + 3 = 11b, b Z. Sendo a mltiplo de 5, tambm , de modo que 3 = a 10b = b 3 b 5c b = 5c + 3 a = 11(5c + 3) 3 = 55c + 30, c +2 O nmero a + 2 mltiplo
EUREKA! N24, 2006

25

Sociedade Brasileira de Matemtica

de 9, assim como a + 2 54c 36 = c 4. Portanto c 4 = 9d c = 9d + 4 a = 55(9d + 4) + 30 = 495d + 250, d . Por fim, sendo a + 1 mltiplo de 7, ento a + 1 497d 245 = a + 1 7 (71d + 35) = 2d + 6 = 2(d 3) tambm , ou seja, d 3 = 7k d = 7 k + 3, k e a = 495(7 k + 3) + 250 = 3465t + 1735 Logo o menor valor de a 1735. Solues Nvel 2 Segunda Fase Parte B
SOLUO DO PROBLEMA 1:

Vamos representar por A, G e L a quantidade de questes de lgebra, Geometria e Lgica da Prova e por a, g e l as questes respondidas acertadamente em cada uma destas reas. As condies do problema fornecem as seguintes equaes: a g l a+l g+l = 0,5; = 0,7; = 0,8; = 0,62; = 0,74 A G L A+ L G+L Substituindo as relaes expressas pelas trs primeiras equaes nas outras duas, obtemos:

0,5 A + 0,8L 3L = 0,62 0,12 A = 0,18L A = A+L 2


0,7G + 0,8L 3L = 0,74 0,04G = 0,06 L G = G+L 2

A porcentagem de questes acertadas : a + g + l 0,5 A + 0,7G + 0,8 L = = A+G + L A+G + L


SOLUO DO PROBLEMA 2:

3 3 0,5. L + 0,7. L + 0,8L 2,6 2 2 = = 0,65 = 65% 3 3 4 L+ L+L 2 2

Vamos denotar por A, B, C e D os vrtices do quadrado e por MN o corte efetuado. Como CM + CN = BC = CD, resulta que BM = CN e DN = MC. Em conseqncia, os tringulos ADN e DCM so congruentes, o mesmo ocorrendo com ABM e BCN (em cada caso, os tringulos so retngulos e possuem catetos iguais). Logo, DN = CDM = e BM = CBN = . Assim, + + 27o = 90o e + = 63o.

EUREKA! N24, 2006

26

Sociedade Brasileira de Matemtica

A 27o

D N C

SOLUO DO PROBLEMA 3:

a) x2 9xy + 8y2 = x2 xy 8xy + 8y2 = x(x y) 8y (x y) = (x 8y)(x y). Alternativamente, as razes da equao do 2o grau x2 9xy + 8y2, de incgnita x, so y e 8y. Logo, x2 9xy + 8y2 fatora em (x 8y)(x y). b) A equao a ser resolvida (x y)(8y x) = 2005 (*) Observemos que a fatorao em primos de 2005 5 401. Alm disso, a soma dos fatores x y e 8y x 7y, que mltiplo de 7. A soma dos fatores 406, sendo que somente 406 mltiplo de 7. Assim, x y = 5 e 8 y x = 401 ou x y = 401 e 8 y x = 5 (*) ou x y = 5 e 8 y x = 401 ou x y = 401 e 8 y x = 5 x = 63 e y = 58 ou x = 459 e y = 58 ou x = 63 e y = 58 ou x = 459 e y = 58

As solues so, portanto, (63; 58), (459;58), (63; 58) e (459; 58).
OUTRA SOLUO:

Observando a equao dada como uma equao do segundo grau em x, obtemos x2 9yx + 8y2 + 2005 = 0 (*), cujo discriminante = (9y)2 4(8y2 + 2005) = 49y2 8020

EUREKA! N24, 2006

27

Sociedade Brasileira de Matemtica

Para que (*) admita solues inteiras, seu discriminante deve ser um quadrado perfeito; portanto 49y2 8020 = m2 (7y m)(7y + m) = 8020 = 22 5 401 (**) Podemos supor, sem perda de generalidade, que m 0, pois se (m; y) soluo de (**), ento ( m; y) tambm . Observando tambm que 7y m e 7y + m tm a mesma paridade e y m 7y + m, ento podemos dividir o problema em 4 casos: 7y m = 2 e 7y + m = 4010 m = 2004 e y = 2006/7, impossvel; 7y m = 10 e 7y + m = 802 m = 396 e y = 58; 7y m = 802 e 7y + m = 10 m = 396 e y = 58; 7y m = 4010 e 7y + m = 2 m = 2004 e y = 2006/7, impossvel.
9 y + m 9 58 + 396 = = 459 2 2

Se y = 58, as solues em x de (*) so


9 y m 9 58 396 = = 63 . 2 2

Se y = 58, as solues em x de (*) so e


9 y m 9 (58) 396 = = 459 . 2 2

9 y + m 9 (58) + 396 = = 63 2 2

Logo as solues so (63 ; 58), (459 ; 58), ( 63 ; 58) e ( 459 ; 58).


SOLUO DO PROBLEMA 4:

Veja a soluo do problema No. 3 do Nvel 1 parte B Solues Nvel 3 Segunda Fase Parte A Problema Resposta 01 12 02 1735 03 240 04 2011 05 2

01 . Primeiro observamos que os ngulos internos de um pentgono regular (5 2) 180 medem = 108 . 5
EUREKA! N24, 2006

28

Sociedade Brasileira de Matemtica

Como AF = AE = AB, o tringulo ABF issceles com


m( ABF ) = m( AFB) = 180 m( BAF ) 180 m( B AE ) m( E AF ) 180 108 60 = = = 6 . 2 2 2

No tringulo PEF, m( EFP) = m( AFE ) m( AFB) = 60 6 = 54 e m( EPF ) = 180 m( PEF ) m( E FP) = 180 60 12 54 = 54 , ou seja, o tringulo PEF issceles com PE = EF. Assim, como EF = AE, o tringulo PEA tambm issceles com m( P AE ) = m( E PA) = 180 m( PEA) 180 12 = = 84. 2 2 180 m( ABC ) 180 108 Alm disso, m(C AB) = = = 36 e 2 2 m(C AE ) = m( B AE ) m(C AB) = 108 36 = 72.

Logo, m( P AC ) = m( P AE ) m(C AE ) = 84 72 = 12. 02. PRIMEIRA SOLUO: Como a + 3 mltiplo de 11, a + 3 = 11b, b . Sendo a mltiplo de 5, a 10b = b 3 tambm , de modo que b 3 = 5c b = 5c + 3 a = 11(5c + 3) 3 = 55c + 30, c O nmero a + 2 mltiplo de 9, assim como a + 2 54c 36 = c 4. Portanto c 4 = 9d c = 9d + 4 a = 55(9d + 4) + 30 = 495d + 250, d . Por fim, sendo a + 1 mltiplo de 7, ento a + 1 497d 245 = a + 1 7 (71d + 35) = 2d + 6 = 2(d 3) tambm , ou seja, d 3 = 7k d = 7 k + 3, k e a = 495(7 k + 3) + 250 = 3465t + 1735. Logo o menor valor de a 1735.
SEGUNDA SOLUO:

As condies do problema equivalem a dizer que 2a 5 = 2(a + 1) 7 = 2(a + 2) 9 = 2(a + 3) 11 mltiplo de 5, 7, 9 e 11, donde mltiplo de 5 7 9 11 = 3465. Assim, o menor valor de a tal que 2a 5 = 3465 , ou seja, a = 1735. 03. Ligando o ponto de interseo das retas que representam as duas cercas aos vrtices, obtemos:

EUREKA! N24, 2006

29

Sociedade Brasileira de Matemtica

M A

O Q

Observemos que, como AQ = QD e as alturas de OAQ e OQD que passam por O so iguais, as reas de OAQ e OQD so iguais. Analogamente, as reas de OAM e OMB; OBN e ONC; OCP e OPD so iguais. Logo rea OAQ + rea OAM + rea OCP + rea ONC = rea OQD + rea OMB + rea OPD + rea OBN rea AMOQ + rea CNOP = rea DPOQ + rea BMON rea AMOQ = 200 + 250 210 = 240. 04. Substituindo y por 2 e x por a f(2) = a 8, obtemos f(a f(2) + f(2)) = a 8 + f ( f (2)) f(a) = a 8 + f(8). Substituindo a por 2 na ltima equao, obtemos f(2) = 2 8 + f(8) 8 = 2 8 + f(8) f(8) = 14. Assim f(a) = a 8 + 14 = a + 6 e f(2005) = 2005 + 6 = 2011. 05. A idia da soluo perguntar o valor numrico de p(k) para k suficientemente grande. Suponha que o polinmio seja: p(x) = an xn + an1 xn 1 + ... + a0, com an, an 1, ..., a0 inteiros positivos. Se k um inteiro, tal que: k > M = mx {an, an-1, ..., a0}, ento p(k) um inteiro, cujos dgitos na representao em base k so exatamente os coeficientes do polinmio p(x). Podemos ento tomar k igual a uma potncia de 10 suficientemente grande. Logo para resolver o problema, basta perguntarmos o valor de p(1), assim obtemos uma cota superior para M, e ento perguntamos o valor de p(x) para x igual a uma potncia de 10 maior do que p(1). Portanto, o nmero mnimo de perguntas que devemos fazer, para garantir que o polinmio p(x) seja determinado sem sombra de dvidas, 2. Por exemplo: Se p(1) = 29, perguntamos p(100), digamos que p(100) = 100613. Ento o nosso polinmio p(x) = 10x2 + 6x + 13.

EUREKA! N24, 2006

30

Sociedade Brasileira de Matemtica

Solues Nvel 3 Segunda Fase Parte B


SOLUO DO PROBLEMA 1:

Temos 9 xy x 2 8 y 2 = 2005 xy x 2 + 8 xy 8 y 2 = 2005 x( y x) + 8 y ( x y ) = 2005 ( x y )(8 y x) = 2005(*) Observemos que a fatorao em primos de 2005 5 401. Alm disso, a soma dos fatores x y e 8y x 7y, que mltiplo de 7. Devemos ento escrever 2005 como produto de dois fatores, cuja soma um mltiplo de 7. Para isso, os fatores devem ser 5 e 401. A soma dos fatores 406. x y = 5 e 8 y x = 401 ou x y = 401 e 8 y x = 5 (*) ou x y = 5 e 8 y x = 401 ou x y = 401 e 8 y x = 5 x = 63 e y = 58 ou x = 459 e y = 58 ou x = 63 e y = 58 ou x = 459 e y = 58

As solues so, portanto, (63; 58), (459;58), (63; 58) e (459; 58).
OUTRA SOLUO:

Observando a equao dada como uma equao do segundo grau em x, obtemos x2 9yx + 8y2 + 2005 = 0 (*), cujo discriminante = (9y)2 4(8y2 + 2005) = 49y2 8020 Para que (*) admita solues inteiras, seu discriminante deve ser um quadrado perfeito; portanto 49y2 8020 = m2 (7y m)(7y + m) = 8020 = 22 5 401 (**) Podemos supor, sem perda de generalidade, que m 0, pois se (m; y) soluo de (**), ento ( m; y) tambm . Observando tambm que 7y m e 7y + m tm a mesma paridade e 7y m 7y + m, podemos dividir o problema em 4 casos: 7y m = 2 e 7y + m = 4010 m = 2004 e y = 2006/7, impossvel;

EUREKA! N24, 2006

31

Sociedade Brasileira de Matemtica

7y m = 10 e 7y + m = 802 m = 396 e y = 58; 7y m = 802 e 7y + m = 10 m = 396 e y = 58; 7y m = 4010 e 7y + m = 2 m = 2004 e y = 2006/7, impossvel.
9 y + m 9 58 + 396 = = 459 2 2

Se y = 58, as solues em x de (*) so


9 y m 9 58 396 = = 63 . 2 2

Se y = 58, as solues em x de (*) so e


9 y m 9 (58) 396 = = 459 . 2 2

9 y + m 9 (58) + 396 = = 63 2 2

Logo as solues so (63 ; 58), (459 ; 58), ( 63 ; 58) e ( 459 ; 58).


SOLUO DO PROBLEMA 2:

c b 2 )2 b
) ) )

a2 ) 2

Podemos supor, sem perda de generalidade, a configurao acima e, portanto, pelo teorema de Pitgoras: 4(b2a2 b2)2 a2)2 + )4 ) = )4 + a4 + b4 + c4 2a2)2 2b2)2 + 2c2)2 + 2a2b2 2a2c2 2b2c2 3) 4 2 ( a 2 + b2 + c 2 ) ) 2 ( a 4 + b 4 + c 4 2 a 2b 2 2 a 2 c 2 2b 2 c 2 ) = 0 O discriminante da equao do segundo grau acima, em
2 2 2 2 4 4 4 2 2

)2 + (

b2 ) 2 a 2 ) 2

) =c
2

(b ) )(a ) ) = a
2 2 2 2

+ b2 c 2 ) 2

2 2 = 2( a + b + c ) + 4 3 ( a + b + c 2a b 2 a c 2b c ) = 2 2

EUREKA! N24, 2006

32

Sociedade Brasileira de Matemtica

16(a 4 + b4 + c 4 a 2 b 2 a 2 c 2 b 2 c 2 ). Logo

) 2 = 2(a
2

+ b2 + c 2 ) 16( a 4 + b4 + c 4 a 2 b2 a 2 c 2 b2 c 2 ) 2 3

+ b 2 + c 2 ) 2 a 4 + b 4 + c 4 a 2b 2 a 2 c 2 b 2 c 2 3 De fato, observando que ) menor ou igual a min {a, b, c}, temos 2 2 2 ) 2 a + b + c . Portanto 3

) 2 = (a

)=

( a 2 + b 2 + c 2 ) 2 a 4 + b 4 + c 4 a 2b 2 a 2c 2 b 2c 2 . 3

Observao: Outra maneira de obter as equaes trabalhar em R3, supondo, sem ) ) 3 , , com perda de generalidade, que C = (0, 0, 0), A = (),0, h) e B = , 2 2 z h, z 0 . Obteramos, ento, as equaes ) 2 + h 2 = a 2 , ) 2 + z 2 = b 2 e ) 2 + ( z h )2 = c 2 , que nos leva mesma equao da soluo acima. Curiosidade: Para o tringulo 3, 4, 5 a medida do lado da projeo que um tringulo equiltero aproximadamente e. O erro de apenas 0,1%.
SOLUO DO PROBLEMA 3:

Primeira Soluo: Seja an o nmero de ordenadas de resultados (sem derrotas), cujo total de pontos seja n. A pergunta do problema : quanto vale a20? Para responder a tal pergunta, iremos determinar uma relao recursiva entre os termos dessa seqncia. Pensando no ltimo resultado de uma ordenada de resultados totalizando n pontos, ele pode ser E ou V. Se for E, ento retirando o ltimo termo da ordenada, ela passa a totalizar n 1 pontos. Se for V, ento ao retiramos o ltimo resultado, a ordenada passa a totalizar n 3 pontos. Disto, conclumos que: an = an 1 + an 3. Calculando os valores da seqncia, temos: a1 = 1, a2 = 1, a3 = 2, a4 = 3, a5 = 4, a6 = 6, a7 = 9, a8 = 13, a9 = 19, a10 = 28, a11 = 41, a12 = 60, a13 = 88, a14 = 129, a15 = 189, a16 = 277, a17 = 406, a18 = 595, a19 = 872 e a20 = 1278.
EUREKA! N24, 2006

33

Sociedade Brasileira de Matemtica

Logo existem 1278 possveis seqncias ordenadas de resultados que o Flameiras pode ter obtido. Segunda Soluo: Sejam x e y o nmero de vitrias e empates do Flameiras, respectivamente. Temos que: x 0, y 0 x + y = 20. Dividindo em 7 possveis casos: 1 caso: x = 0 e y = 20: Temos exatamente uma seqncia ordenada de resultados. 2 caso: x = 1 e y = 17: Uma seqncia ordenada dever conter exatamente um V e 17 E, portanto o nmero de seqncias ordenadas exatamente o nmero de anagramas da palavra: VEEEEEEEEEEEEEEEEE, que : (17 + 1)! / (17! 1!) = 18. 3 caso: x = 2 e y = 14: Analogamente ao 2 caso, o nmero de seqncias ordenadas igual ao nmero de anagramas da palavra VVEEEEEEEEEEEEEE, que : (14 + 2)! / (14! 2!) = 120. 4 caso: x = 3 e y = 11: (11 + 3)! / (11! 3!) = 364 seqncias ordenadas. 5 caso: x = 4 e y = 8: (8 + 4)! / (8! 4!) = 495 seqncias ordenadas. 6 caso: x = 5 e y = 5: (5 + 5)! / (5! 5!) = 252 seqncias ordenadas. 7 caso: x = 6 e y = 2: (2 + 6)! / (2! 6!) = 28 seqncias ordenadas. Temos um total de 1 + 18 + 120 + 364 + 495 + 252 + 28 = 1278 seqncias ordenadas de resultados possveis.
SOLUO DO PROBLEMA 4:

Seja p, p + d, p + 2d, p + 3d, p + 4d, p + 5d, p + 6d a progresso aritmtica, que podemos supor crescente sem perda de generalidade. Ento: 1) p 2. De fato, se p = 2, p + 2d par e maior do que 2 e, portanto, no primo. 2) d mltiplo de 2. Caso contrrio, como p mpar, p + d seria par e maior do que 2. 3) p 3 Seno, teramos p + 3d mltiplo de 3, maior do que 3. 4) d mltiplo de 3 Caso contrrio, p + d ou p + 2d seria mltiplo de 3 e maior do que 3. 5) p 5 Seno teramos p + 5d mltiplo de 5, maior do que 5.
EUREKA! N24, 2006

34

Sociedade Brasileira de Matemtica

6) d mltiplo de 5. Caso contrrio, p + d , p + 2d, p + 3d ou p + 4d seria mltiplo de 5, maior do que 5. De 1), 2), 3), 4), 5) e 6), p 7 e d mltiplo de 30. Se p = 7, observando que 187 = 11 17, ento d 120. Para d = 120, a seqncia 7, 127, 247, 367, 487, 607, 727 a qual no serve, pois 247 = 13 19. Para d = 150, a seqncia 7, 157, 307, 457, 607, 757, 907 e satisfaz as condies do problema. Finalmente, se p 7, ento d mltiplo de 210 e o menor ltimo termo possvel para tais seqncias 11 + 6 210 = 1271. Portanto a resposta 907.

EUREKA! N24, 2006

35

Sociedade Brasileira de Matemtica

XXVII OLIMPADA BRASILEIRA DE MATEMTICA


Problemas e Solues da Terceira Fase
PROBLEMAS NVEL 1
PROBLEMA 1

Esmeraldinho tem alguns cubinhos de madeira de 2 cm de aresta. Ele quer construir um grande cubo de aresta 10 cm, mas como no tem cubinhos suficientes, ele cola os cubinhos de 2 cm de aresta de modo a formar apenas as faces do cubo, que fica oco. Qual o nmero de cubinhos de que ele precisar? Num tabuleiro quadrado 5 5 , sero colocados trs botes idnticos, cada um no centro de uma casa, determinando um tringulo. De quantas maneiras podemos colocar os botes formando um tringulo retngulo com catetos paralelos s bordas do tabuleiro? Observao: Tringulo retngulo todo tringulo que possui um ngulo de 90o. Os lados que formam esse ngulo so chamados de catetos.
PROBLEMA 3 PROBLEMA 2

A partir da casa localizada na linha 1 e na coluna 50 de um tabuleiro 100 100 , sero escritos os nmeros 1, 2, 3, 4, ..., n, como na figura a seguir, que apresenta uma parte do tabuleiro e mostra como os nmeros devero ser colocados. O nmero n ocupar a casa da linha 1, coluna 100.
Linha 100

  
46 45 44 43 42 41 40 39 38 37 47 29 28 27 26 25 24 23 22 48 30 16 15 14 13 12 11

  

Linha 10

Linha 1

... Coluna 1

49 31 17 7 6 5 4

50 32 51 18 33 8 19 2 9 1 3 Coluna 50

52 34 20 10

53 35 21

54 36

55

n Coluna 100 ...

EUREKA! N24, 2006

36

Sociedade Brasileira de Matemtica

a) Determine n. b) Em qual linha e coluna aparecer o nmero 2005?


PROBLEMA 4

No retngulo ABCD, com diagonais AC e BD, os lados AB e BC medem, respectivamente, 13 cm e 14 cm. Sendo M a interseco das diagonais, considere o tringulo BME, tal que ME = MB e BE = BA, sendo E A . a) Calcule a rea do tringulo BME. b) Mostre que o segmento BD paralelo ao segmento EC.
PROBLEMA 5

Um nmero inteiro positivo n tem a propriedade P se a soma de seus divisores positivos igual a 2n . Por exemplo: 6 tem a propriedade P, pois 1 + 2 + 3 + 6 = 2 6 , porm 10 no tem a propriedade P, pois 1 + 2 + 5 + 10 2 10 . Mostre que nenhum quadrado perfeito tem a propriedade P. Observao: Um nmero inteiro positivo um quadrado perfeito se igual ao quadrado de um inteiro. Por exemplo, 1 = 12 , 4 = 2 2 e 9 = 3 2 so quadrados perfeitos.

PROBLEMAS NVEL 2 Num tabuleiro quadrado 5 5 , sero colocados trs botes idnticos, cada um no centro de uma casa, determinando um tringulo. De quantas maneiras podemos colocar os botes formando um tringulo retngulo com catetos paralelos s bordas do tabuleiro?
PROBLEMA 2 PROBLEMA 1

No tringulo retngulo ABC, os catetos AB e BC medem, respectivamente, 3 cm e 4 cm. Seja M o ponto mdio da hipotenusa AC e seja D um ponto, distinto de A, tal que BM = MD e AB = BD. a) Prove que BM perpendicular a AD. b) Calcule a rea do quadriltero ABDC.

EUREKA! N24, 2006

37

Sociedade Brasileira de Matemtica

PROBLEMA 3

Dado que

(a b)(b c )(c a) 1 a b c + + ? = , qual o valor de a+b b+c c+a (a + b)(b + c)(c + a ) 11

PROBLEMA 4

Em seu treino dirio de natao, Esmeraldinho percorre vrias vezes, com um ritmo constante de braadas, o trajeto entre dois pontos A e B situados na mesma margem de um rio. O nado de A para B a favor da corrente e o nado em sentido contrrio contra a corrente. Um tronco arrastado pela corrente passa por A no exato instante em que Esmeraldinho sai de A. Esmeraldinho chega a B e imediatamente regressa a A. No trajeto de regresso, cruza com o tronco 6 minutos depois de sair de A. A seguir, Esmeraldinho chega a A e imediatamente sai em direo a B, alcanando o tronco 5 minutos depois da primeira vez que cruzou com ele ao ir de B para A. Quantos minutos o tronco leva para ir de A at B?
PROBLEMA 5

Prove que o nmero 12005 + 22005 + 32005 + ... + 20052005 mltiplo de 1 + 2 + 3 + ... + 2005.
PROBLEMA 6

A medida do ngulo B de um tringulo ABC 120. Sejam M um ponto sobre o lado AC e K um ponto sobre o prolongamento do lado AB, tais que BM a bissetriz interna do ngulo ABC e CK a bissetriz externa correspondente ao ngulo ACB. O segmento MK intersecta BC no ponto P. Prove que APM = 30. PROBLEMAS NVEL 3
PROBLEMA 1:

Um nmero natural palndromo quando se obtm o mesmo nmero ao escrevermos os seus dgitos na ordem inversa. Por exemplo, 481184, 131 e 2 so palndromos. Determine todos os pares de inteiros positivos (m, n) tais que 111...1  111...1 
m uns n uns

palndromo.
PROBLEMA 2:

Determine o menor nmero real C para o qual a desigualdade

EUREKA! N24, 2006

38

Sociedade Brasileira de Matemtica


2005 2005 2005 2005 125 + x3 + x4 + x5 x1 x 2 x3 x 4 x5 x1 + x 125 + x 125 + x 125 + x 125 C x12005 + x 2 2 3 4 5 vlida para todos os nmeros reais positivos x1, x2, x3, x4, x5.

16

PROBLEMA 3:

Dizemos que um quadrado est contido em um cubo quando todos os seus pontos esto nas faces ou no interior do cubo. Determine o maior ) > 0 tal que existe um quadrado de lado ) contido num cubo de aresta 1.
PROBLEMA 4:

Temos quatro baterias carregadas, quatro baterias descarregadas e um rdio que necessita de duas baterias carregadas para funcionar. Supondo que no sabemos quais baterias esto carregadas e quais esto descarregadas, determine o menor nmero de tentativas suficiente para garantirmos que o rdio funcione. Uma tentativa consiste em colocar duas das baterias no rdio e verificar se ele, ento, funciona.
PROBLEMA 5:

Sejam ABC um tringulo acutngulo e F o seu ponto de Fermat, isto , o ponto B , BF C e CF A medem 120 interior ao tringulo ABC tal que os trs ngulos AF graus. Para cada um dos tringulos ABF, ACF e BCF trace a sua reta de Euler, ou seja, a reta que liga o seu circuncentro e o seu baricentro. Prove que essas trs retas concorrem em um ponto.
PROBLEMA 6:

Dados a, c inteiros positivos e b inteiro, prove que existe x inteiro positivo tal que a x + x b (mod c ) , ou seja, existe x inteiro positivo tal que c um divisor de ax + x b. SOLUES NVEL 1
PROBLEMA 1: SOLUO DE DANIEL LUCAS FILGUEIRA (FORTALEZA - CE)

Como cada cubinho tem 2 cm de aresta e o cubo tem 10 cm de aresta, ento cabem 5 cubinhos no comprimento, na largura e na altura, ento em todo o cubo cabem 125 cubinhos. Se no lado do cubo coubessem n cubinhos, ento o No. de cubinhos da parte de dentro do cubo seria (n 2) (n 2) (n 2). Como no lado do cubo cabem 5 cubinhos, ento para sabermos o No. de cubinhos da parte de dentro, basta substituir o n pelo 5, e ficaria o seguinte: (5 2) (5 2) (5 2) = 3 3 3 = 27
EUREKA! N24, 2006

39

Sociedade Brasileira de Matemtica

Como em todo o cubo cabem 125 cubinhos, ento para deixar o cubo oco, basta tirar a parte de dentro, que tem 27 cubinhos. Logo, Esmeraldinho precisaria de 125 27 = 98 cubinhos para formar o cubo oco.
PROBLEMA 2: SOLUO DE RAFAEL SUSSUMU YAMAGUTI MIADA (CAMPINAS - SP)

Se o boto correspondente ao ngulo reto estiver em (1, 1) teremos mais 4 casas acima e 4 casas direita, portanto 4 4 = 16 possibilidades. Se ele estiver em (2, 1) teremos mais 4 casas acima, 3 casas direita e 1 casa esquerda o que d de novo 4 4 = 16 possibilidades. Do mesmo modo, vemos que, para cada casa escolhida para o boto correspondente ao ngulo reto temos 16 possibilidades, e como no campo existem 25 casas, teremos portanto 25 16 = 400 possibilidades. Ento teremos 400 possibilidades.
PROBLEMA 3: SOLUO DA BANCA

a) Quando for escrito o nmero n, todas as casas da diagonal que passa pela (linha 100; coluna 1) e (linha 1; coluna 100) e as que esto abaixo dela estaro preenchidas e, nesse caso, 100 + 99 + 98 +...+ 3 + 2 + 1 = 5050 nmeros tero sido escritos no tabuleiro. Como comeamos com o 1, o ltimo, n, ser 5050. b) A quantidade de termos nas camadas (1, 2, 3), (4, 5, 6, 7, 8, 9, 10), (11, 12, 13, 14 , 15, 16, 17, 18, 19, 20, 21) aumenta de 4 em 4. Ao final da 31a camada, que tem 3 + 30 4 = 123 nmeros, tero sido escritos 3 + 7 + 11 +...+ 123 = 1953 nmeros, ou seja, o ltimo nmero dessa camada 1953. O termo que ocupa a linha mais alta em cada camada aumenta de 2 em 2 (veja que a 1a camada sobe at alinha 2, a 2a camada at a linha 4, a 3a sobe at a linha 6, e assim por diante). Assim, o termo da 31a camada que ocupa a linha mais alta estar na linha 1 + (122 2) = 62. Por fim, a 32a camada iniciar na linha 1 e coluna 51 32 = 19, com o nmero 1954, e subir at a linha 62 + 2 = 64. Como 2005 = 1954 + 51, o nmero 2005 aparecer na linha 51 + 1 = 52 e coluna 19.
PROBLEMA 4: SOLUO MATHEUS BARROS DE PAULA (TAUBAT - SP)

a) Montando a figura, ela ficar assim:


E 13 B 13 M

A 14 D

EUREKA! N24, 2006

40

Sociedade Brasileira de Matemtica

Os tringulos BEM e BAM so congruentes pelo critrio LLL. Como a distncia de M ao lado AB metade do lado AD, o tringulo BAM possui uma base de 13 cm e 13 7 uma altura de 7cm, e sua rea de = 45,5 cm2. 2 b) O tringulo BEM congruente ao tringulo CMD pelo critrio LLL, logo a 0/ 0/ distncia de E reta BD idntica distncia de C reta BD , pois as alturas sero as mesmas. Assim, EC // BD.
PROBLEMA 5: BASEADA NA SOLUO DE GUSTAVO LISBA EMPINOTTI (FLORIANPOLIS - SC)

0/ 0/

Um quadrado perfeito sempre tem um nmero mpar de divisores, pois h pares de nmeros cujo produto o quadrado perfeito dado e mais um nmero, a sua raiz. Se o quadrado perfeito n for mpar, ento todos os seus divisores so mpares, e assim ser sua soma. Logo a soma no pode ser 2n, pois 2n par. Se o quadrado perfeito n for par, ento igual a uma potncia de 2 vezes o quadrado de um mpar. Os divisores mpares de n so divisores desse quadrado e, como j vimos, sua soma (de todos os divisores mpares de n) mpar e logo a soma de todos os divisores de n tambm mpar, no podendo ser igual a 2n, que par. Portanto nenhum quadrado perfeito tem a propriedade P.

SOLUES NVEL 2
PROBLEMA 1: SOLUO DE HENRIQUE POND DE OLIVEIRA PINTO (SALVADOR BA)

Ao invs de considerarmos um tabuleiro quadrado consideremos uma malha pontilhada onde os pontos so centros de cada quadradinho. Isto :

EUREKA! N24, 2006

41

Sociedade Brasileira de Matemtica

Observe que para cada tringulo do enunciado existe um nico conjunto dos 2 pontos extremos da hipotenusa. Ou seja, o conjunto de dois pontos extremos da hipotenusa no tringulo abaixo {8; 20}.
1 6 11 16 21 2 7 12 17 22 3 8 4 9 5 10 15 20 25

13 18 23

14 19 24

Ou seja os pontos 8 e 20 determinam a hipotenusa do tringulo abaixo. No entanto para cada dois pontos que determinam a hipotenusa existem outros dois pontos que podem ser o vrtice oposto hipotenusa. No exemplo acima os pontos 8 e 20 determinam a hipotenusa de dois tringulos retngulos; os tringulos 8; 18; 20 e 8; 10; 20. Basicamente, cada tringulo possui uma nica hipotenusa e cada hipotenusa comum a dois tringulos retngulos do enunciado. Para provar que cada hipotenusa pertence a dois tringulos retngulos distintos, vamos pegar uma hipotenusa genrica de extremos (5K + a) e (5 N + b) (no quadriculado acima) contando que ambos sejam menores que 25 e tanto a e b sejam maiores ou iguais a 1 e menores ou iguais a 5. Observe que K N e a b ou seja, ambos os pontos esto em linhas e colunas diferentes pois se coincidirem em linhas e ou colunas no h tringulos como definidos no enunciado com essa hipotenusa. Os dois e (5K + a; 5K + b; 5 N + b) tringulos com essa hipotenusa so: (5K + a; 5 N + a; 5N + b ) como para cada tringulo h uma nica hipotenusa e para cada hipotenusa dois tringulos, o nmero de tringulos o dobro do de hipotenusas. Vamos calcular o nmero de hipotenusas: O primeiro ponto pode ficar em 25 lugares (todos os pontos) j o segundo pode ficar em 16 (todos que no esto na mesma linha ou coluna do primeiro). Logo so 25 16 = 400 onde a ordem das escolhas importa, mas a ordem no importa. Logo, como so duas escolhas dividi-se por 2! = 2 e teremos 400/2 = 200 hipotenusas 400 tringulos. Logo a resposta 400 tringulos. Obs. Para um quadrado n n a quantidade de tringulos n 2 ( n 1) 2 2 = n (n 1) se generalizamos esse processo que foi utilizado. 2
2

EUREKA! N24, 2006

42

Sociedade Brasileira de Matemtica

PROBLEMA 2: SOLUO DE RAFAEL TUPYNAMB DUTRA (BELO HORIZONTE - MG)


B 90 2 D

A M

Chamemos BCA = . AB 3 Temos sen = = AC 5 BC 4 cos = = . AC 5 a) Como ABC retngulo em B, sabemos que B pertence circunferncia de dimetro AC. Desta forma, AM = MB = MD AM = MD. Isso significa que M pertence mediatriz de AD . Como AB = BD, B tambm pertence mediatriz de AD . 0 / Assim, a mediatriz de AD a reta BM e, assim, BM AD. c) D pertence circunferncia de centro M e raio BM , j que BM = MD. Assim, D pertence ao circuncrculo do ABC. Como AB = BD, temos AB = BD e,

F F

ABDC inscritvel, BDC = 180 (90 ) = 90 + . No tringulo BCD, temos C BD = 180 (90 + ) = 90 2. rea BCD = BD BC sen (90 2 ) 2 =

assim, BCD = BCA BCD = . No tringulo ABC , B AC = 90 . Logo, como

3cm 4cm cos 2 = 6cm 2 cos 2 e, como 2

3 4 sen = e cos = , 5 5

EUREKA! N24, 2006

43

Sociedade Brasileira de Matemtica

4 2 3 2 16 9 42 2 rea BCD = 6cm (cos sen ) = 6cm = 6cm2 = cm 5 5 25 25 Usamos aqui o fato de que cos 2 = cos 2 sen2 . AB BC 3cm 4cm = = 6cm 2 rea ABC = 2 2 42 192 2 rea ABDC = rea ABC + rea BCD = 6 + cm2 = cm . 25 25
2 2 2 2

PROBLEMA 3: SOLUO ADAPTADA DE MARCELO MATHEUS GAUY (SO JOS DO RIO PRETO - SP)

Inicialmente, podemos observar que a+b b+c c+a b c a b c a + + =3 + + = 3 + + , a+b b+c c+a a+b b+c c+a a +b b +c c +a a b c + + ou seja, obter o valor de = equivalente a obter o valor de a+b b+c c+a b c a = + + . a+b b+c c +a Como j sabemos que = 3 , basta agora conseguir outra relao entre e aproveitando a igualdade fornecida no enunciado a qual envolve =

( a b ) (b c ) ( c a ) ( a + b ) (b + c ) ( c + a )

Aps alguns testes, substituindo valores em a, b e c, somos levados a supor que ab bc ca ab bc ca + + = = , isto , . a+b b+c c+a a +b b +c c +a (Temos acima uma incrvel identidade, ela fornece infinitas triplas de reais cuja soma igual ao oposto do produto!). Vamos demonstrar tal identidade: a b b c c a ( a b)(b + c)(c + a) + (b c)( a + b)(c + a) + (c a)( a + b)(b + c) + + = (*) a+b b+ c c+ a ( a + b)(b + c)(c + a) Porm, 1) (a b)(b + c)(c + a ) = (a b )(b c + 2c)( a c + 2c ) =

(a b)(b c )(a c ) + 2c ( a b)(b c + a c + 2c) = (a b)(b c )(a c ) + 2c ( a b)(a + b)

EUREKA! N24, 2006

44

Sociedade Brasileira de Matemtica

2)

(b c)( a + b)(c + a) + (c a)( a + b)(b + c) = (a + b)((b c)(c + a ) + (c a )(b + c)) = (a + b)(bc + ba c2 ca + cb + c2 ab ac) = (a + b)( 2bc 2ac) = 2c (a b)( a + b)

Logo, de 1 e 2, ( a b)(b c)(c a) . (*) = ( a + b)(b + c)(c + a) Assim, = 1 1 16 (3 ) = = . 11 11 11

PROBLEMA 4: SOLUO DE HENRIQUE WATANABE (SO PAULO - SP)

Vamos supor que a velocidade da corrente do rio c e a velocidade de Esmeraldinho v (sem a corrente). Seja d o comprimento do rio. Em 6 minutos os dois juntos percorreram 2d. A velocidade no sentido A B (v + c ) e a velocidade no sentido B A

( v c) .

O primeiro encontro foi 6c do ponto A. d De A B ele leva minutos. v+c O segundo encontro ocorreu 11c do ponto A. 2dv d d + = Para ir e voltar Esmeraldinho leva: t = ( v + c v c v + c)(v c) e de A at 11c: 2dv + 11c (v c) 11c . Logo = 11 v+c (v + c)(v c)

2dv + 11cv 11c2 = 11v2 11c2 v (11v 11c 2d ) = 0 De B at o primeiro encontro em 6c t =

( d 6c)(v + c) + d (v c) d 6c . Logo 6 = v c (v + c)(v c )

6v2 6c2 = dv + dc 6vc 6c2 + dv dc 2v (3v + 3c d ) = 0 Como v 0: 11v 11c 2d = 0 3v + 3c d = 0 11v 11c = 2d 6v + 6c = 2d

EUREKA! N24, 2006

45

Sociedade Brasileira de Matemtica

17c 5 17c 51c + 15c 66c Ento: d = 3 + 3c = = 5 5 5 d 66c 1 66 minutos O tronco leva: t = = = 5 c 5 c 66 minutos, ou seja, 13 minutos e 12 segundos. O tronco vai de A para B em 5 11v 11c = 6v + 6c v =
PROBLEMA 5: SOLUO DE HENRIQUE POND DE OLIVEIRA PINTO (SALVADOR - BA)

Observe que 1 + 2 + 3 + ... + 2005 =

2005 ( 2005 + 1) 2

= 2005 1003

Seja E = 12005 + 22005 + ... + 20052005 Vamos provar que 2005| E Vendo E mdulo 2005 temos: E 12005 + 22006 +32005 +... +10012005 +10022005 + (1002)2005 + (1001)2005 +... +(2)2005 +(1)2005 + 02005
2005

como a2005 ( a )
2005

2005

a2005 + (a) 2005 0 Temos que o n-simo termo da


2005

expresso acima ir se anular com o (2005 n) e, portanto, E 0(mod2005) 2005| E . Vamos provar agora que 1003 divide E. Vendo E mdulo 1003 temos: E 12005 + 22005 + ... +10012005 +10022005 + 02005 + (1002)2005 + (1001)2005 + ... + (2)2005 + (1)2005
1003

como a2005 (a)2005 a2005 + (a)2005 0 cada n-simo termo ir se anular com o
1003 1003

(2006 n) termo e o 1003o. j mltiplo de 1003 pois igual a 10032005 temos que E 0(mod1003) 1003 | E com o 1003 | E e 2005 | E e (1003,2005 ) = 1 1003 2005 | E 1 + 2 + ... + 2005 |12005 + 22005 + ... + 20052005 c.q.d.

EUREKA! N24, 2006

46

Sociedade Brasileira de Matemtica

PROBLEMA 6: SOLUO DE MARLIA VALESKA COSTA MEDEIROS (FORTALEZA - CE)


A

60 B

Q 60 30 30

M 30 P

30

I L

Observe a figura acima: Vamos explicar como chegar at ela! Sejam: Q o ponto de interseco de BM e AP . I a interseco de CK e a bissetriz externa de ABC , que encontra o prolongamento de AC em L.

BCK KCL = (pelo enunciado)


Vamos provar que A, Q, P, I so colineares. Usando Menelaus no ABC, temos: M, P e K so colineares

AK BP CM = 1 (*) BK CP AM
S temos que: Pelo Teorema da bissetriz interna: (I) No ABC CM BC = AM AB Pelo Teorema da bissetriz externa:
EUREKA! N24, 2006

47

Sociedade Brasileira de Matemtica

(II) No ABC AK AC = BK BC Ento, substituindo em (*):

AC BP BC AC BP AC CP =1 =1 = BC CP AB AB CP AB BP
Com este resultado, observe que AP bissetriz do ngulo B AC . Como Q a interseco de AP e BM , Q pertence a bissetriz do ngulo B AC , o que implica, que no ABM , AQ bissetriz de B AM . Vamos provar que I pertence a bissetriz de BAM . Pelo teorema da bissetriz interna: (III) No BCK CI BC = IK BK Por (II), temos

AK BK IK IK CI = = = AC BC CI AK AC
Observe que isto nada mais nada menos do que a propriedade da bissetriz interna. Logo AI bissetriz do ngulo K AC = BAM . Assim, provamos que A, Q, P e I so colineares. Seja ACB = . No ABC podemos observar que 2 + = 60 S que:

BCL = 120 + 2 (teorema


BCL = 2 = 60 +

do

ngulo

externo).

Sabemos,

ento,

que

 K = 120 Logo, AI Olhando para o quadriltero BPIK, observe que este inscritvel, pois:  = 60 + 120 = 180 PBK + PIK
Assim, I BK I PK = 30 Agora, observe que APM I PK (o.p.v) Portanto, APM = 30

 K = + + = + + 60 + = 2 + + 60 Observe que o ngulo AI

EUREKA! N24, 2006

48

Sociedade Brasileira de Matemtica

SOLUES NVEL 3
PROBLEMA 1: SOLUO DE WILSON CAMARA MARRIEL (RIO DE JANEIRO - RJ)

Podemos escrever 11...1 como


k

10k 1 . 9

Ento:
m n

m m n n 1 n n n 1 10m (10n 1) 10n 1 10 1 10 1 11...100...0 11...1 11...1099...988...89 9 9 11...1 11...1 = = = = 9 9 9 9 9 m n

2 2 2

se supusermos m > n (o que no nos faz perder a generalidade!). Esse nmero tem m + n 1 algarismos. Ento vejamos: (fazendo n > 9)

11...1099...988...89 k

2
n 1

m n

n 1

9 1234567901
n 2

 x

onde x o ltimo termo do quociente antes de usar o 0 e k o resto de 11...1 por 9.


n 1 m n n 1

Isto , acima temos parte da diviso euclidiana de 11...1099...988...89 por 9. Dividimos nos casos para possveis valores de k. 1o. caso (k = 0) n = 9 + 1. Continuando a diviso, obtemos
mn n 1

2
n1

0099...988...89

9 1234567901
n 2

  x 011111  10987
mn

Contando da esquerda para a direita 8 o (m + 1)o. algarismo; como m + n 1 m +1> ento est na metade direita do resultado. Logo, j que o 8 no 2 aparece no lado esquerdo e aparece no direito, temos uma assimetria, ABSURDO! 2o. caso (k = 1): n = 9 +2
EUREKA! N24, 2006

49

Sociedade Brasileira de Matemtica


m n n 1

1099...988...89 19 19 18 088 mesmo caso anterior, ABSURDO! 3o. caso (k = 2): n = 9 +3


mn n 1

9 1234567901
n 2

 22098    x 122
mn

2099...988...89 29 29 28 18 088 mesmo caso anterior, ABSURDO!

9 1234567901
n 2

32098    x 233
mn

Ao tentarmos os 9 casos, vemos que um deles tem que ser 9. Basta conferir se verdade! Vejamos: n = 1 verdade! 11...11 n = 2 111...1 verdade! 122...21 11...11 111...1 verdade! Obs. fcil ver que sempre dar certo de 1 9. n=3 1111...1 1233...321

EUREKA! N24, 2006

50

Sociedade Brasileira de Matemtica

11...111111111 11111111...111 n = 9 verdade! Pois basta ir somando 1 aos 111111111...11 1111111111...1 1234567899...987654321 posteriores. Ento nossos pares so:

(1, k ), ( 2, k ), (3, k ), ( 4, k ), (5, k ), (6, k ), (7, k ), (8, k ) , (9, k ) ( m, n) = tal que k < *. (k ,1), (k ,2), (k ,3), (k ,4), (k ,5), (k ,6), (k ,7), (k ,8), (k ,9)
PROBLEMA 2: SOLUO DE ANDERSON HOSHIKO AIZIRO (SO PAULO SP)

Para x1 = x2 = x3 = x4 = x5 = 1 temos C 5 516 C 515. Por bunching (ou desigualdade de Muirhead), temos que

x
sym

1 2 3 4 5 1 2 3 4 5

1 2 3 4 5 x x x x x1 x2 x3 x4 x5 sym

no caso de termos uma desigualdade homognea e simtrica (o que o nosso caso). Alm disso, devemos ter 1 1 , 1 + 2 1 + 2 , 1 + 2 + 3 1 + 2 + 3 , 1 + 2 + 3 + 4 1 + 2 + 3 + 4 e 1 + 2 + 3 + 4 + 5 = 1 + 2 + 3 + 4 +5 . A notao

x
sym

1 2 3 4 5 1 2 3 4 5

x x x x

significa que estamos somando todos os 5! = 120

1 2 3 4 5 xi2 xi3 xi4 xi5 , sendo (i1 , i2 , i3 , i4 , i5 ) uma permutao de (1, 2, 3, termos da forma xi 1

4, 5). Aqui, impomos tambm 1 2 3 4 5 e o mesmo para os ' s. Observando x1 x2 x3 x4 x5 x

que +x
125 2

a +x
125 3

desigualdade +x
125 4

125 1

+x

125 16 5

dada

simtrica,

se

abrirmos

obtemos um somatrio simtrico no qual o

maior expoente de algum dos termos 125 16 + 1 < 2005. Podemos, ento, aplicar bunching. A desigualdade , ento, equivalente a C
EUREKA! N24, 2006

1 516 2005 x1 x1 1 x 2 2 x 3 3 x 4 4 x5 5 4! sym 5! sym

51

Sociedade Brasileira de Matemtica

Explicando:

x
sym

2005 1

possui 5! termos no desenvolvimento no qual temos 4!

2005 2005 2005 2005 , 4! x3 , 4! x4 e 4!x5 . x12005 , 4!x2

Alm

(x

disso,

temos

125 1

125 125 + x125 2 + x3 + x4 + x

125 16 5

516

termos

no

desenvolvimento

de

e, para cada conjunto (1 , 2 , 3 , 4 , 5 ) temos 5! 516 somatrios simtricos. 5!


2 3 4 5 x11 x2 x3 x4 x5 , se C sym

dos 516 termos e, portanto, h

Como por bunching desigualdade vlida.

x
sym

2005 1

1 516 = C = 515 4! 5!

Desse modo, conclumos que o menor nmero real C para o qual a desigualdade
2005 2005 2005 2005 125 125 125 125 + x3 + x4 + x5 x1 x2 x3 x4 x5 x1 + x125 C x12005 + x2 2 + x3 + x4 + x5

16

vlida

para todos os nmeros reais positivos x1 , x2 , x3 , x4 , x5 C = 515.

PROBLEMA 3: SOLUO DA BANCA

Primeiro, mostremos que podemos supor, sem perda de generalidade, que os centros do cubo (que doravante chamaremos C) e do quadrado coincidem. Suponha que os centros no coincidam. Considere os trs planos distintos, cada um deles paralelo a duas faces do cubo, que passam pelo centro do quadrado. Os trs planos determinam no cubo oito paraleleppedos; considere o de menores dimenses (ou seja, algum que tem todas as dimenses menores ou iguais a 1/2). Seja a a maior dimenso desse paraleleppedo. Ento construa um cubo C0 de lado 2a com centro no centro do quadrado e faces paralelas s faces do cubo do problema. O quadrado est contido nesse cubo, pois cada plano ou contm o quadrado ou o corta em dois polgonos congruentes e simtricos em relao ao centro do quadrado. Translade o cubo C0, incluindo o quadrado, que est em seu interior, de modo que o centro de C0 coincida com o centro do cubo. Agora os centros do quadrado e de C coincidem, e dado que 2a 1, C0 est contido em C, e o quadrado ainda est contido no cubo C. A figura a seguir mostra que
EUREKA! N24, 2006

) 3 42 .
52

Sociedade Brasileira de Matemtica

Note

que
2

3 2 3 1 1 , AD = BC = 2 AB = CD = + 12 + = 4 4 4 4

3 3 2 1 2. AC = BD = + 12 + 12 = = 2 2 4
1 4 3 4

3 4

D 1
4

1 B 4 3 4

3 4

1 4

Vamos provar que, na verdade,

) > 3 42 . Podemos supor que o centro do quadrado coincide com o centro do


cubo. Seja S uma esfera com centro no centro O de C e que passa pelos quatro 2 / 2 > 3/ 4. A figura a seguir mostra as vrtices do quadrado, ou seja, de raio seces de S no cubo C. Numeramos as oito regies contidas na superfcie da esfera e no interior do cubo com nmeros romanos

) = 3 42 . Suponha que exista um quadrado de lado )

EUREKA! N24, 2006

53

Sociedade Brasileira de Matemtica

IV

III

VIII

VI V VII II

Agora, vamos tentar localizar os vrtices do quadrado de lado

) > 3 42

em S.

Note que cada um dos quatro vrtices deve pertencer a uma das regies de I a VIII. Suponhamos, sem perda de generalidade, que dois vrtices opostos do quadrado esto contidos nas regies I e, conseqentemente, II, j que vrtices opostos do quadrado so diametralmente opostos em S. Considere o paraleleppedo de menores dimenses que contm as regies I e, digamos, III. Sejam x, x e 1 as suas dimenses. Vamos provar que dois pontos no 3 2 . interior desse paraleleppedo esto a uma distncia menor que 4
x x I 1

III

EUREKA! N24, 2006

54

Sociedade Brasileira de Matemtica

Primeiro, considere uma face do cubo e sua interseo com a esfera. A partir da figura a seguir, podemos concluir que o raio da esfera 1 1 1 y2 + + = y2 + . 2 2 2
2 2 2

Como

raio

da

esfera

maior

que

3 2 2 3 2 1 3 1 1 1 = , y + > y > . Conseqentemente, x = y < . 4 2 4 2 4 4 2 4


y y

1 2

1 3 2 e, portanto, +1 = 16 4 dois vrtices do quadrado no podem estar contidos em I e III. Como um dos vrtices pertence a I, no pode existir vrtice do quadrado em III e, analogamente, em IV e V. Da mesma forma, lembrando que um dos vrtices do quadrado est em II, no pode haver vrtices do quadrado em VI, VII e VIII. Mas ento no sobraram regies para os outros dois vrtices do quadrado, absurdo. 3 2 . Deste modo, o maior lado de um quadrado contido no cubo unitrio = 4 A diagonal do paraleleppedo mede x 2 + x 2 + 12 < 2

PROBLEMA 4: SOLUO DE GABRIEL TAVARES BUJOKAS (SO PAULO SP)

Separe as pilhas em 3 grupos, 2 de 3 pilhas e 1 de 2 pilhas, e faa todas as 3 3 2 possibilidades dentro dos grupos. Faremos + + = 7 tentativas. 2 2 2 Como so 3 grupos e 4 baterias boas, um grupo ter 2 baterias boas, e em algum momento sero testadas juntas.
EUREKA! N24, 2006

55

Sociedade Brasileira de Matemtica

Suponha que seja possvel com apenas 6 tentativas. Considere o grafo com 8 vrtices representando as 8 baterias e as arestas {i, j} representando que a bateria i 8 e a j no foram testadas juntas. O grafo tem 6 = 22 arestas. 2 Por Turn, ele tem um K4. De fato, o mximo de arestas sem K4 8 3 3 2 = 21 . 2 2 2 2 Se as 4 baterias carregadas forem as respectivas aos vrtices do K4, temos que nunca duas delas foram testadas juntas, logo o algoritmo com 6 tentativas falha nesta situao. Resposta: Com 7 tentativas possvel. Observao: o grafo completo de n vrtices (tambm conhecido como n-clique), notado por Kn, um grafo em que todo par de vrtices ligado. O teorema de Tran diz que, fixado o nmero de vrtices n, o grafo que no contm um Kr com a maior quantidade possvel de arestas o grafo (r 1)-partido completo com classes de vrtices as mais distribudas possveis. Esse grafo obtido da seguinte forma: divida os vrtices em r 1 conjuntos, de modo que a diferena entre as quantidades de vrtices nos conjuntos seja no mximo 1; em seguida, ligue todo par de vrtices que no esto no mesmo conjunto. Na aplicao do problema 4, Gabriel dividiu os 8 vrtices em trs grupos com 3, 3 8 3 3 2 e 2 vrtices. Da a quantidade de arestas mxima ser 2 = 21 . 2 2 2

Alis, a resoluo de Gabriel tambm resolve uma generalizao do problema, no qual h m baterias funcionando e n baterias descarregadas. Tente pensar nesse problema!

EUREKA! N24, 2006

56

Sociedade Brasileira de Matemtica

PROBLEMA 5: SOLUO DE LEANDRO FARIAS MAIA (FORTALEZA CE)

Construa um tringulo equiltero BXC, externo a ABC. O ponto O1 o circuncentro do BFC e tambm de BXC . G o baricentro do ABC . AG XO1 =2= O1G // XF. Mas: O3 A = O3 F e O2 A = O2 F Temos: GM O1M AF O2 O3 O1G O2 O3 . Analogamente temos: O2G O1O3 e O3G O1O2 G o ortocentro do O1O2O3 .
A O3 O2

G1 B C

Sendo G1 o baricentro do FBC temos:

FG1 AG =2= G1G // AF G1M GM G1G O2O3 como G o ortocentro de O1O2O3 , ento G1 est na altura relativa a O2O3 . Portanto, O1G1 , O2 G 2 e O3G3 so concorrentes em G (seu ortocentro).

Lema: Seja p primo e a, b, r, inteiros, a > 0, > 0. Ento existe x > 0 tal que x r (mod p 1) a x + x b (mod p ) Demonstrao: Induo em . Para = 1 (Base): x r (mod p 1) , que tem soluo pelo teorema chins Se p | a , ento obtemos x b (mod p ) dos restos. Se p / | a; x = ( p 1)(ar b + r + l c) + r; com l tal que x > 0. De fato, x r (mod p 1) e
EUREKA! N24, 2006

PROBLEMA 6: SOLUO DE GABRIEL TAVARES BUJOKAS (SO PAULO SP)

57

Sociedade Brasileira de Matemtica

a x + x ar ( a p1 )
p

(a b+r +l c)
r

+ ( p 1)( ar b + r + l c ) + r ar ar + b r l c + r b.
p p

Passo: Da hiptese, existe x0 tal que a

x0

+ x0 = b + t p

e x0 r

(mod p 1) .

Tomando x1 = x0 + ( p 1) p t : x1 x 0 r e a x1 + x1 a x0 a ( p 1) p +1
p

) +x
t

p 1

p 1

+ ( p 1) p t a x0 + x 0 p t = b +1
p

Isso termina a demonstrao do lema.


2 n ... pn ; p1 < p2 < ... < pn a fatorao em primos de c. Seja c = p11 p2

Vamos mostrar por induo em n que x tal que a x + x b (mod c n ) , onde


2 ... pii . ci = p11 p2

Base: n = 1: a x + x b mod p11 . Caso especial do lema. Passo: Se xi tal que a xi + xi b ( mod ci ) , pelo lema existe x tal que x x i (mod pi +1 1) a x + x b (mod p i +i1+1 ) Pelo teorema chins dos restos xi +1 tal que xi +1 x i (mod mmc( pi +1 1; ci ; (c i )) ) xi +1 x (mod p i +i1+1 )

i +1 Observe que mdc p i +1 , mmc( p i +1 1; ci ; (c i )) = 1 pois pi + 1 maior que todos os fatores primos de ci e, conseqentemente, de (c i ) . i +1 Logo xi +1 x i x (mod p i +1 1) e xi +1 x (mod p i +1 )

xi +1 x (mod ( p i +1 1) pi +i1+1 ) , ou seja, xi +1 = x + p i +i1+1 ( pi +1 1)l '


x+ p Assim, a xi+1 + xi +1 a i+1 + pi+i1 1
i +1

( pi+1 1)l '

+x ax + x be + +
pi+i1 1 pi+i1 1 ci

x + c l a xi+1 + xi +1 a i ( i ) + xi + ci k a xi + xi b onde (ci ) l = k ci = xi +1 xi . ci ci

b a xi+1 + xi +1 b. Portanto a xi+1 + xi +1 +


pi+i1 1 ci ci+1

Em especial, a xn + xn b a xn + xn b .
cn c

EUREKA! N24, 2006

58

Sociedade Brasileira de Matemtica

XXVII OLIMPADA BRASILEIRA DE MATEMTICA Primeira Fase Nvel Universitrio

PROBLEMA 1

Seja f : definida por f ( x) = x 3 + ax 2 + bx + c , sendo a, b e c inteiros. Sabe-se que f(1) = f(1) = 0. As retas tangentes ao grfico de f nos pontos A = (1; 0) e B = (1; 0) cortam-se em C. Calcule a rea do tringulo ABC, sabendo-se que tal rea inteira.
PROBLEMA 2
4

Calcule a integral:
PROBLEMA 3

ln(1 + tgx)dx

Determine o maior valor possvel para o volume de um tetraedro inscrito no x2 y2 z 2 elipside de equao + + =1. 9 16 25
PROBLEMA 4

Sejam A e B matrizes reais quadradas de mesma dimenso tais que, para todo inteiro positivo k, ( A + B ) k = A k + B k . Prove que se A invertvel ento B a matriz nula.
PROBLEMA 5

Determine todos os valores reais de para os quais a matriz A = (a ij ) nn definida por aij = cos((i 1) j), para 1 i, j n, tem determinante nulo.
PROBLEMA 6

Prove que existem pelo menos 2005 potncias 27-simas distintas (isto , nmeros da forma n27, com n inteiro positivo), todas com exatamente 2005 algarismos, tais que qualquer uma pode ser obtida de qualquer outra a partir de uma permutao de seus algarismos.

EUREKA! N24, 2006

59

Sociedade Brasileira de Matemtica

XXVII Olimpada Brasileira de Matemtica GABARITO Primeira Fase


Solues Nvel Universitrio
SOLUO DO PROBLEMA 1:

Pelo enunciado, temos f(x) = (x 1)(x + 1)(x c) = x3 cx2 x + c, f'(x) = 3x2 2cx 1, donde f '( 1) = 2(1 + c) e f '(1) = 2(1 c). Assim, as equaes das retas AC e BC so, respectivamente, y = 2(1 + c)(x + 1) e y = 2(1 c)(x 1). Igualando para obter as coordenadas de C, temos (1 + c)(x + 1) = (1 c)(x 1) x = 1/c y = 2(c + 1)(c 1)/c Assim a rea pedida S = |2(c + 1)(c 1)/c|, pois o tringulo ABC tem base AB = 2 e altura y = 2(c + 1)( c 1) / c . Como c e a rea S so inteiros, temos c | 2(c + 1)(c 1). Mas (c + 1) e (c 1) so primos com c, donde c | 2. Assim c = 1 ou c = 2. Os casos c = 1 do S = 0, um tringulo degenerado. Os casos c = 2 do S = 3. O valor da rea , portanto, igual a 3.
SOLUO DO PROBLEMA 2:

senx senx + cos x Temos ln(1 + tgx ) = ln 1 + = ln . cos x cos x 2 Entretanto, sen x + = senx cos + sen cos x = (senx + cos x), e logo 4 4 4 2 senx + cos x = 2 sen x + . 4 2 sen x + 4 ln 2 = Assim, ln(1 + tgx ) = ln + ln sen x + ln cos x, donde cos x 2 4
EUREKA! N24, 2006

60

Sociedade Brasileira de Matemtica

ln (1 + tgx ) dx =

ln 2 4 + ln sen x + dx 4 ln cos x dx. 0 0 8 4

Agora, sen x + = cos x + = cos x , donde 4 2 4 4


4 4 + = = ln sen ln cos ln cos y dy x dx x dx 0 0 0 4 4 ln 2 (fazendo a substituio y = x ), donde 4 ln (1 + tgx ) dx = . 0 4 8 4

SOLUO ALTERNATIVA DO PROBLEMA 2:

x, du = dx. 4 0 1 tgu 2 Ento I = ln 1 + tg u ( du ) = 4 ln 1 + du = 4 ln du 0 0 4 1 + tgu 1 + tgu 4 Seja I = 4 ln(1 + tgx )dx; faa u =
0

= 4 ln 2du 4 ln(1 + tgu ) du =


0 0

ln 2 ln 2 ln 2 I 2I = I = . 4 4 8

SOLUO DO PROBLEMA 3:

LEMA: O tetraedro de maior volume inscrito na esfera unitria x2 + y2 + z2 = 1 o tetraedro regular. Seus vrtices podem ser tomados como ( c, c, c) com um nmero par de sinais onde c = 3 3 . Sua aresta a = 2 6 3 e seu volume V = 8 3 27. O elipside do problema obtido a partir da esfera unitria aplicando a transformao linear 3 0 0 T = diag (3,4,5) = 0 4 0 . Tetraedros inscritos na esfera so levados em 0 0 5 tetraedros inscritos no elipside multiplicando o volume por |det (T)| = 60. Assim um tetraedro de volume mximo (3c, 4c, 5c), com um nmero par de sinais , de volume160 3 9. Demonstrao do LEMA: A nica parte no trivial a de provar que um tetraedro de volume mximo deve ser regular. Vamos provar que todas as faces de um tetraedro de volume mximo
EUREKA! N24, 2006

61

Sociedade Brasileira de Matemtica

so tringulos equilteros. Para isso vamos fixar o vrtice V0 e variar os vrtices V1, V2, V3 restritos ao crculo definido por estes pontos. Ora, com este tipo de mudana a altura do tetraedro no muda, donde maximizamos o volume maximizando a rea do tringulo V1, V2, V3. um fato sabido e de fcil demonstrao que o tringulo de rea mxima inscrito em um crculo dado o equiltero.
SOLUO DO PROBLEMA 4:

Temos, de A2 + B 2 = ( A + B) 2 = ( A + B)( A + B) = A2 + AB + BA + B 2 que AB + BA = 0. Agora, A3 + B3 = ( A + B)3 = ( A + B)( A + B) 2 = ( A + B)( A2 + B 2 ) = A3 + AB 2 + BA2 + B3 , donde AB 2 + BA2 = 0. Como BA = AB, 0 = AB 2 + BA2 = AB 2 ABA = A( B 2 BA) e, como A invertvel, B 2 BA = 0. Temos, tambm A3 + B 3 = ( A + B )3 = ( A + B ) 2 ( A + B) = ( A2 + B 2 )( A + B ) = A3 + A2 B + B 2 A + B 3 , donde A2 B + B 2 A = 0. Como B 2 = BA, segue que A2 B + BA2 = 0, e, como BA = AB, obtemos 0 = A2 B + BA2 = A2 B ABA = A( AB BA), donde AB = BA, pois A invertvel. Finalmente, de AB + BA = 0, segue que 2 AB = 0, donde, como A invertvel, devemos ter B = 0.
SOLUO DO PROBLEMA 5:

Sabemos que para todo k Pk (t ) = ck ,k t + ... + ck ,1t + ck ,0

natural

existe

um

polinmio

de grau k tal que cos(ka ) = Pk (cos a ) para todo a.


2 Por exemplo, P0 = 1, P 1 = t, P 2 = 2t 1 Temos portanto aij = Pi 1 (cos( j)) = ci 1, k (cos( j ))k 0 k <i

Podemos agora, para i > 1, subtrair ci 1,0 vezes a primeira linha da i-sima linha sem alterar o determinante obtendo assim que, para i > 1,  ij = a ci 1, k (cos( j))k .
0< k < i

EUREKA! N24, 2006

62

Sociedade Brasileira de Matemtica

Para i > 2, subtramos ci 1,1 vezes a segunda linha da i-sima linha, ainda sem alterar o determinante. Repetindo o processo, vemos que det(A) = det(B) onde bij = ci 1,i 1 (cos( j ))i 1 Assim, a menos dos fatores ci 1,i 1 , B uma matriz de Vandermonde, e seu determinante igual a ( j1 + j0 ) ci 1,i 1 (cos( j1 ) (cos( j0 )) =(2)n(n1)/ 2 ci 1,i 1 sen 2 1< j in j0 < j1 i n ( j1 j0 ) sen . 2 Assim det(A) = 0 se e somente se existem 1 j0 < j1 n tais que ( j + j ) ( j j ) sen 1 0 = 0 ou sen 1 0 . 2 2 Mas isto ocorre se e somente se ( j1 j0 ) = 2k , k inteiro. Ou seja, det(A) = 0 se e somente se = 2k /( j1 j0 ) para alguma escolha de 1 j0 < j1 n Falta verificar quais os valores possveis de j1 j0 . Para n 1 o problema trivial (det(A) = 1), donde no h nenhum com essa propriedade. Para n = 2, os nicos valores possveis de j1 j0 so 1 e 3, 2k , com k inteiro donde deve ser da forma 3 Para n > 2, j1 j0 assume todos os valores inteiros positivos m at 2n 1 , donde deve ser da forma 2k m , com m 2n 1 e k inteiro. Observao: Temos ainda ck ,k = 2k 1 para k > 1 donde

c
i n

i 1,i 1

= 2( n 1)( n 2) / 2 e
2

det( A) = (1) n ( n 1) / 2 2( n 1) .

sen
j1 < j0

( j1 + j0 ) ( j1 j0 ) sen 2 2

EUREKA! N24, 2006

63

Sociedade Brasileira de Matemtica

Demonstrao da afirmao cos(ka) = Pk(cos(a)) [No vale pontos extras]: Temos cos((k + 1)a ) + cos((k 1)a ) = 2cos(ka ) cos a, donde, assumindo que o resultado vale para k 1 e para k, cos((k + 1)a ) = 2cos a Pk (cos a) Pk 1 (cos a ), o que prova o resultado fazendo Pk +1 ( x) = 2 xPk ( x) Pk 1 ( x), para k 1, com P0 ( x) = 1 e P 1 ( x ) = x. Note que, sabendo que o coeficiente lder ck , k de P k ( x) 2k 1 , segue imediatamente que o coeficiente lder ck +1, k +1 de Pk +1 ( x) 2 2k 1 = 2k = 2( k +1)1. 06. Vamos estimar inicialmente a quantidade de tipos de nmeros de 2005 algarismos a menos de uma permutao de seus algarismos. Um tal tipo de nmeros est determinado pelas quantidades x0 , x1 ,..., x9 de algarismos iguais a 0, 1, , 9, respectivamente; devemos ter x0 + x1 + ... + x9 = 2005. Assim, a quantidade desses tipos de nmeros , no mximo, o nmero de solues de x0 + x1 + ... + x9 = 2005, com xi 0 para 0 i 9, que 2005 + 9 2014 9 4 9 36 = < 2014 < (10 ) = 10 . 9 9 Por outro lado n 27 tem 2005 algarismos
2004 27 2005 27

se,

somente
2005 27

se,
2004 27

102004 n 27 < 10 2005 10 n < 10 , donde h pelo menos 10 27 naturais n tais que n tem 2005 algarismos. Entretanto,

10

2005 2004 2004 2004 2004 1 ln10 ln10 ln10 10 27 10 27 =10 27 1027 1 =10 27 e 27 1 >10 27 >1074 >1072 20051036 , 27 27 donde, pelo princpio da casa dos pombos, h pelo menos 2005 naturais n tais que n 27 tem 2005 algarismos e esses nmeros n 27 so todos do mesmo tipo (seus algarismos so os mesmos a menos de uma permutao).

Nota: possvel estimar 10 27 1 sem usar a desigualdade e x 1 x. Por exemplo:


1 1 1 1 1 16 10 > 10 = 10 2 > 316 > (1,7 )8 > (1,3) 4 > (1,12) 2 > 1,05, donde 1 1 (que foi o que usamos). 10 27 1 > 0,05 > 100 1 27 1 32 1

EUREKA! N24, 2006

64

Sociedade Brasileira de Matemtica

XXVII OLIMPADA BRASILEIRA DE MATEMTICA SEGUNDA FASE NVEL UNIVERSITRIO PRIMEIRO DIA
PROBLEMA 1:

Determine, em funo de n, o nmero de possveis valores para o determinante de A, dado que A uma matriz real n n tal que A3 A2 3 A + 2I = 0 , onde I representa a matriz identidade n n, e 0 representa a matriz nula n n.
PROBLEMA 2:

Sejam f e g funes contnuas distintas de [0, 1] em (0, + ) tais que n +1 1 f ( x) 1 1 , seja Para 0 = n y ( ) ( ) . = f x dx g x dx n 0 g ( x)n dx. 0 0 Prove que ( yn )n 0 uma seqncia crescente e divergente.
PROBLEMA 3:

Sejam v1 , v2 ,..., vn vetores em

tais que vi 1 para 1 i n e

v
i =1

= 0. Prove

que existe uma permutao de {1, 2,, n} tal que k com 1 k n. Obs. Se v = ( x, y )
2

v
j =1

( j )

5 para qualquer

, v = x 2 + y 2 denota a norma euclidiana de v.

SEGUNDO DIA
PROBLEMA 4:

Considere a seqncia (an )n 1 dada por a1 = 1, an +1 = an + Prove que a srie

1 a
2005 n

, n 1.

na
n =1

converge.

EUREKA! N24, 2006

65

Sociedade Brasileira de Matemtica

PROBLEMA 5:

Prove que

n
n =1

1
n

= x x dx.
1 0

PROBLEMA 6:

Prove que para quaisquer naturais 0 i1 < i2 < ... < ik e 0 j1 < j2 < ... < jk , a matriz i + js (ir + js )! A = (ars )1 r ,s k dada por ars = r (1 r , s k ) invertvel. = ir ! js ! ir
SOLUES PROBLEMA 1: SOLUO DE MOYSES AFONSO ASSAD COHEN (RIO DE JANEIRO RJ)

Podemos Podemos

escrever

A3 A2 3 A + 2I = 0

como

( A 2 I ) ( A2 + A I ) = 0.

Se provarmos que no existem duas combinaes ( d1 , d2 , d3 ) e ( e1 , e2 , e3 ) tais que


d2 d3 e2 e3 3 = 2e1 2 3 2d1 2 , ento o nmero de possveis valores para o determinante, ser o nmero de maneiras de escolher d1 , d 2 , d3 satisfazendo di > 0 e d1 + d2 + d3 = n. E o nmero de escolher esses di ' s

concluir ento que os possveis autovalores de A so 1 5 1 + 5 1 = 2, 2 = e 3 = . Seja di a multiplicidade do autovalor i . 2 2 d2 d3 3 , onde d1 + d2 + d3 = n. Temos que o determinante de A da forma: 2d1 2

n! (n + 2)(n + 1) n 2 + 3n + 2 = . 2 n!2! n! 2 d2 e1 e2 e3 3 d se, e somente se, d1 = e1 , Vamos provar ento que 2d1 2 3 = 2 2 3 d 2 = e2 e d3 = e3 .
d2 d3 e2 3 3 = 2e1 2 e 2d1 2 3 , d1 + d 2 + d 3 = e1 + e2 + e3 = n d2 n d1 d2 e2 n e1 e2 2d1 2 1 = 2e1 2 3 , Podemos usar tambm que 2 3 = 1,

( n + 2 )! =

1 + 5 1 5 +1 + 5 5 5 4 = = 1.) (pois = 2 2 4 4

EUREKA! N24, 2006

66

Sociedade Brasileira de Matemtica

1 2 n d1 d2 1 2 n e1 e2 e1 2 3 = 2 3 3 3
d1
d1 e1 ) + 2 ( d 2 e2 ) n d1 2 d 2 n e1 2 e2 2d1 ( 1) 2 3 = 2e1 ( 1) 2 3 2d1 e1 = ( 1) e2 d2 ( ,e 3 d e

como no podemos escrever 1 + 5 , onde I um inteiro maior que 1, como uma potncia de 2, temos que a igualdade verdadeira se, e somente se, os expoentes so zero. Ou seja d1 e1 = 0 d1 = e1 e portanto d3 = e3 . e2 d 2 = 0 d 2 = e2
PROBLEMA 2: SOLUO DE HUMBERTO SILVA NAVES (S.J. DOS CAMPOS SP)

Sejam: un = yn +1 yn =
0 1

f ( x ) n +1 [ f ( x) g ( x)] dx g ( x) n +1 f ( x) n +1 2 f ( x) g ( x ) ) dx n+ 2 ( ( ) g x

e vn = un+1 un =
0 1

Sabemos que vn > 0, n , pois

f ( x ) n +1 2 f ( x ) g ( x ) ) > 0, n+ 2 ( g ( x) x f g Para [0;1] e como e so contnuas, temos: f ( x ) n +1 2 f ( x ) g ( x ) ) dx > 0, n , logo (un )n 0 crescente. n +2 ( g ( x) 0 Vamos agora provar que u0 > 0 : vn =
1

<

<

u0 = u0 ( f ( x ) g ( x ) ) dx =
0 0

1 f ( x) f x g x dx ( ) ( ) ( ) [ f ( x) g ( x )]dx = g ( x) 0

=
0

( f ( x) g ( x))
g ( x)

dx > 0 un uo > 0, n , pois (un )u >0 crescente.

<

Portanto claramente yn yo + n uo , n 0, e ( yn )n 0 crescente e divergente.

EUREKA! N24, 2006

67

Sociedade Brasileira de Matemtica

PROBLEMA 3: SOLUO DA BANCA

Vamos usar a soluo da verso em R do problema: se 1 , 2 ,..., m so nmeros reais com i 1, i m e

i =1

= 0 ento existe

uma permutao de {1, 2,,m} tal que ( i ) tenha sinal contrrio a (i. e., com (i ) ( j ) 0).
j <i

j <i

( j )

Seja I = {1, 2,,n}. Escolhemos um conjunto X I tal que

v
iX

seja o maior

possvel. Podemos supor (rodando os eixos coordenados, se necessrio) que

v
iX

um vetor da forma (0, y), com y > 0. Sejam 1 , 2 : 2 as projees na primeira e na segunda coordenadas, respectivamente. Usando a verso em do problema para reordenar os elementos de X e de I\X, podemos supor que X = {1, k +1 r 2,,k} para um certo k > n, 1 vi 1, r k e 1 v j 1, s n k . i =1 j = k +1 Note agora que 2 (vi ) 0, i X (e 2 ( vi ) 0, i I \ X ) , pois, se j X e 2 (v j ) < 0, teramos
iX \{ j}

vi >

iX

, absurdo. Podemos ento obter (como na

verso em do problema) uma permutao de I que intercala os ndices em X e em I\X , preservando a ordem dos ndices em X e em I\X, de modo que m 2 v (i ) 1, m n. Como os ndices em X e em I\X aparecem em ordem, i =1 m teremos 1 v (i ) 1 v( i ) + 1 v (i ) 1 + 1 = 2, m n, e logo i i m i =1 i i m (i )I \ X

i =1

(i )

22 + 12 = 5, m n.

EUREKA! N24, 2006

68

Sociedade Brasileira de Matemtica

PROBLEMA 4: SOLUO DE DIGO VELOSO UCHA (RIO DE JANEIRO RJ)

Primeiro recorde a expanso binomial de Newton: n n n ( a + b ) = a nk b k . k =0 k Agora usando isso verifique as seguintes contas: 1 an + (a + b ) = 2005 ( an ) (pois an sempre positivo) Logo: 2006 > a12006 + 2006 a2
2006 2006 2006 > a2 + 2006 a3 2006

= ( an )

2006

2006 2005 1 2006 + + ... > (an ) + 2006. an 2005 1 ( an )

( an +1 ) > ( an ) + 2006. 2006 2006 ( an+1 ) > ( a1 ) + ( 2006 ) n > (n + 1) 1/ 2006 1+ 1 an+1 > ( n + 1) ( n + 1) an+1 > ( n + 1) 2006
2006 2006
1 1 1 1 < < 1 1+ 1+ 1 2006 2006 ( n + 1) an+1 ( n + 1) n =1 n an n =1 ( n )

Como

1 com > 1 converge segue que n =1 n

na
n =1

tambm converge.
n

PROBLEMA 5: SOLUO DE LUS DANIEL BARBOSA COELHO (RIO DE JANEIRO - RJ)

=e

ln( x x )

=e

( x ) ln x

=
n=0

( x ) ( ln x )
n

n!

x dx =
1 x 0

n=0

( x ) ( ln x )
n

n!

dx

devido ao tipo de convergncia montona da srie de potncias e y =


n =0

yn , n!

podemos fazer troca da integral com o somatrio, obtendo:

x x dx =
n=0

( 1)
n!

x n ( ln x ) dx
n
1 n 0

( p) ( p) denotemos por I n a seguinte integral: I n = x p (ln x ) dx, para todo p inteiro

no negativo.
EUREKA! N24, 2006

69

Sociedade Brasileira de Matemtica

Integrando por partes: u = (ln x ) du =


n ( p) In =

n x p +1 n 1 ( ln x ) dx; dv = x p dx v = x p +1

1 (1) p +1 x p +1 n n n 1 n (ln x ) dx 1 x p (ln x ) dx = (ln1) 0 0 ( p + 1) p +1 p +1

lim +
x 0

x p +1 n n n n ( p) ( p) (ln x ) In In x p +1 (ln x ) = 0. 1 = 1 , pois lim + 0 x p +1 p +1 p +1

n ( p ) ( p) = In I n 1 p +1

1 ( p ) I1( p ) = I0 p +1 (1) n n! ( p ) (1)n n! 1 p (1) n n! ( p) = I0 = x dx = , tomando p = n temos: In n n 0 ( p + 1) ( p + 1) ( p + 1)n+1


1 0

x x dx =

(1)n n! n=0

(1)n n ! 1 1 . = = n +1 n +1 n n =1 n ( n + 1) n =0 ( n + 1)

PROBLEMA 6: SOLUO DE HUMBERTO SILVA NAVES (S.J. DOS CAMPOS SP) Considere os seguintes pontos no reticulado: An = ( in ;0) e Bn = ( 0; jn ) , onde

1 n k .
y

B4 B3 B2 B1 A4 A3 A2 A1 x

Um caminho ligando An com Bm um caminho no reticulado partindo de An e chegando em Bm que s pode ir para cima ou para a direita. Exemplos:

EUREKA! N24, 2006

70

Sociedade Brasileira de Matemtica


y
y

Bm 2
2

Bm

3 2 An

3 An

caminho!

No caminho!

i + j Um fato interessante que existem n m caminhos ligando An com Bm . in Uma rota uma coleo de k caminhos (de cada An parte exatamente um caminho e em cada Bm chega exatamente um caminho) e dizemos que uma rota bem feita se os caminhos no se cruzam em nenhum ponto do reticulado. Vamos provar que o nmero de rotas bem feitas igual a det(A). Pela definio de determinante, temos: k in + j ( n ) det( A) = (1) I ( ) . in S n =1
k

k i + j Mas (1) I ( ) n ( n ) exatamente o nmero de rotas ligando An com B( n) , in n =1 para 1 n k , multiplicando pela paridade I() da permutao . Vamos provar que as rotas mal feitas se cancelam neste somatrio:

y 4 X 3 2 Y 5 A4 4 A3 3 2 A2 A1
1

B4 B3 B2 B1 x

Considere uma rota mal feita R e seja Y o ponto de interseco com maior coordenada x (se existir mais de um, tome Y cuja coordenada y seja a menor possvel).

= {(4 ; 4) ; (3 ; 3) ; (2 ; 1) ; (2 ; 2)}

EUREKA! N24, 2006

71

Sociedade Brasileira de Matemtica

Vamos trocar os respectivos caminhos que se cruzam em Y (se existir mais de 2 caminhos que se cruzam em Y, troque os caminhos que comeam em An ' s de mais maior coordenada x). Assim obtemos uma nova rota mal y feita, s que com a paridade de permutao correspondente trocada 4 B4  X (1) I ( ) = (1) I ( ) 3 B3 Como a relao entre rotas mal 2 B2 feitas que acabamos de definir Y bijetora, ento provamos que as 1 B1 rotas mal feitas no contribuem para x 5 4 3 2 1 o somatrio, e como uma rota bem feita possui a identidade como A4 A3 A2 A1 permutao associada, provamos  = {(4 ; 4) ; (3 ; 3) ; (2 ; 1) ; (2 ; 2)} que: det (A) = nmero de rotas bem feitas. Como 0 i1 < i2 < ... < ik e 0 j1 < j2 < ... < jk , certamente o nmero de rotas bem feitas diferente de 0, pois uma rota bem feita.
y B4 B3 B2 Y B1 A1 x

A4

A3

A2

Errata: O item b) do problema No. 112 (Eureka! 23, p.60) foi proposto equivocadamente: ao contrrio do que pensvamos, parece no haver solues simples para ele. Gostaramos portanto de manter apenas o item a) do problema proposto No. 112. Pedimos desculpas pelos inconvenientes causados.

EUREKA! N24, 2006

72

Sociedade Brasileira de Matemtica

XXVII OLIMPADA BRASILEIRA DE MATEMTICA Resultado Nvel 1 (5a. e 6a. Sries)


NOME Matheus Barros de Paula Guilherme Vieira Melo Luis Musso Gualandi Rafael Dias da Fonsca Rodrigo Rolim Mendes de Alencar Gustavo Lisba Empinotti Iuri Rezende Souza Eduardo Cintra Simes Joo Mendes Vasconcelos Gabriel Lima Guimares Jonas Rocha Lima Amaro Bruno Cesar da Silva Guedes Kelve Torres Henrique Igor Rosiello Zenker Daniel Lucas Filgueira Cleiton Vilela Figueiredo da Silva Andreza Lais da Silva Nascimento Ivan Seki Hellmeister Matheus Henrique Botelho Cordeiro Breno Rocha Comin Henrique Lopes de Mello Leonardo Henrique Caldeira Pires Ferrari Leonardo Gonalves Fischer Francisco Vagner Dantas Leite Filho Elder Massahiro Yoshida Alex Lordello Magario Rafael Sussumu Yamaguti Miada Deborah Barbosa Alves Diogo Silva Freitas Matheus Barbosa Santos de Miranda Augusto dos Santos Morgan Andr Bandeira Pinheiro Fernando Fonseca Andrade Oliveira Eduardo F. Freire Neto Wellington Biing Jung Lee Mac'simus Alec'sander de Castro Duarte Alessandro Macdo de Arajo Camila Miraglia Ribeiro Douglas Barbosa da Fonsca Pedro Montebello Milani Gabriel Ricardo Loecsh Siebiger Tiago Yparraguirre Vigas Rafael de Melo Andrade Priscilla Lie Sato Yamaguti Joo Lucas Camelo S Franciely Juliani Chutti Frederico Nascimento Dutra Isaac Jernimo Moreira Anne Wang Rafael Farias Marinheiro Rafael Fernandes Paixo Filipe da Gama Martin Humberto Lopes Tabatinga Neto Gregory Cosac Daher Renata Aimi Fukuda Fabrcio Catani de Freitas Bruno Giordano Leite Victor Gonalves Elias Letcia Duchein Ferreira Larissa Firakawa Tamashiro Douglas Souza Alves Junior Lara Guimares Fernandes Peres CIDADE - ESTADO Taubat SP Fortaleza CE Vitria ES Arapiraca AL Fortaleza CE Florianpolis SC Mineiros GO Recife PE Fortaleza CE Vitria ES Fortaleza CE Recife PE Recife PE So Paulo SP Fortaleza CE Recife PE Recife PE So Paulo SP Curitiba PR Leme SP Rio de Janeiro RJ Rio de Janeiro RJ Fraiburgo SC Fortaleza CE So Paulo SP Salvador BA Campinas SP So Paulo SP Recife PE Joo Pessoa PB S. J. do Rio Pardo SP Fortaleza CE Belo Horizonte MG Salvador BA So Paulo SP Fortaleza CE Fortaleza CE Curitiba PR Arapiraca AL So Paulo SP Sobradinho DF Niteri RJ Boituba SP So Paulo SP Fortaleza CE Itajobi SP Porto Alegre RS Fortaleza CE So Paulo SP Recife PE Rio de Janeiro RJ Nanuque MG Teresina PI Rio de Janeiro RJ So Paulo SP Sorocaba SP Recife PE Joo Pessoa PB Londrina PR Jundia SP Vassouras RJ Rio de Janeiro RJ PRMIO Ouro Ouro Ouro Ouro Ouro Prata Prata Prata Prata Prata Prata Prata Prata Prata Prata Prata Prata Prata Bronze Bronze Bronze Bronze Bronze Bronze Bronze Bronze Bronze Bronze Bronze Bronze Bronze Bronze Bronze Meno Honrosa Meno Honrosa Meno Honrosa Meno Honrosa Meno Honrosa Meno Honrosa Meno Honrosa Meno Honrosa Meno Honrosa Meno Honrosa Meno Honrosa Meno Honrosa Meno Honrosa Meno Honrosa Meno Honrosa Meno Honrosa Meno Honrosa Meno Honrosa Meno Honrosa Meno Honrosa Meno Honrosa Meno Honrosa Meno Honrosa Meno Honrosa Meno Honrosa Meno Honrosa Meno Honrosa Meno Honrosa Meno Honrosa

EUREKA! N24, 2006

73

Sociedade Brasileira de Matemtica

Nvel 2 (7a. e 8a. Sries)


NOME Henrique Pond de Oliveira Pinto Marcelo Matheus Gary Rafael Tupynamb Dutra Pollyanna Stfani Borges Freitas Iuri Souza Ramos Barbosa Guilherme Philippe Figueiredo Marcelo Tadeu de Oliveira S Marlen Lincoln da Silva Henrique Watanabe Grazielly Muniz da Cunha James Jun Hong Pedro Pinheiro de Negreiros Bessa Marilia Valeska Costa Medeiros Camilla Matias Morais Mrcio Rabello de Freitas Alex Atsushi Takeda Renan Lima Novais Rafael Horimoto de Freitas Dielson de Britto Junior Hugo Fonseca Arajo Vitor Mori Cindy Yuchi Tsai Thiago Ribeiro Ramos Gabriel Moreira Francisco Tales Augusto Gonalves Alphonse Nathana Alcntara Lima Illan Feiman Halpern Thiago da Silva Pinheiro Jlio Czar Batista de Souza Thiago Ide Sousa Danilo Marcolongo Afonso Caio Jos Fonseca Santos Caio Srgio Parente Silva Isabella Amorim Gonalez Ana Lusa de Almeida Losnak Yuri Bastos Pereira Mateus Sampaio de Mendona Alisson de Brito Ninomia Alan Eduardo dos Santos Ges Jos Cabadas D. Neto Marcelo Rafael Silva Rempel Rafael Rabelo de Carvalho Davi Lopes Alves de Medeiros Gabriella Fonseca Ribeiro Christian Eduardo de Umeki e Saiki Marco Antonio Lopes Pedroso Catarina Yu Na Kim Rafael Alves da Silva Pedro Henrique Azevedo Damacena Renan Henrique Finder Ricardo Bioni Liberalquino Dalen Chen Kuang Izabela Karennina Travizani Maffra Jennifer Katherine Koshiba Yu Felipe Onrio da Silva Oliveira CIDADE ESTADO Salvador BA S. J. do Rio Preto SP Belo Horizonte MG Fortaleza CE Braslia DF Fortaleza CE Barreiras BA Fortaleza CE So Paulo SP Fortaleza CE So Paulo SP Fortaleza CE Fortaleza CE Fortaleza CE Mesquita RJ Londrina PR Niteri RJ So Paulo SP Rio de Janeiro RJ Juiz de Fora MG So Paulo SP So Paulo SP Varginha MG Santo Andr SP Paraguau Paulista SP Fortaleza CE Itatiaia RJ So Paulo SP Salvador BA Suzano SP S. B. do Campo SP Rio de Janeiro RJ Rio de Janeiro RJ Fortaleza CE So Paulo SP Rio de Janeiro RJ Belo Horizonte MG So Paulo SP Fortaleza CE Salvador BA Maring PR Braslia DF Fortaleza CE Betim MG So Paulo SP Santa Isabel SP So Paulo SP Teresina PI Fortaleza CE Joinville SC Macei AL Osasco SP Belo Horizonte MG So Paulo SP Botucatu SP PRMIO Ouro Ouro Ouro Ouro Ouro Ouro Prata Prata Prata Prata Prata Prata Prata Prata Prata Bronze Bronze Bronze Bronze Bronze Bronze Bronze Bronze Bronze Bronze Bronze Bronze Bronze Bronze Bronze Bronze Meno Honrosa Meno Honrosa Meno Honrosa Meno Honrosa Meno Honrosa Meno Honrosa Meno Honrosa Meno Honrosa Meno Honrosa Meno Honrosa Meno Honrosa Meno Honrosa Meno Honrosa Meno Honrosa Meno Honrosa Meno Honrosa Meno Honrosa Meno Honrosa Meno Honrosa Meno Honrosa Meno Honrosa Meno Honrosa Meno Honrosa Meno Honrosa

EUREKA! N24, 2006

74

Sociedade Brasileira de Matemtica

Nvel 3 (Ensino Mdio)


NOME Gabriel Tavares Bujokas Guilherme Rodrigues Nogueira de Souza Thoms Yoiti Sasaki Hoshina Regis Prado Barbosa Luty Rodrigues Ribeiro Rafael Mendes de Oliveira Cesar Ryudi Kawakami Jose Marcos Andrade Ferraro Jos Armando Barbosa Filho Anderson Hoshiko Aiziro Leandro Farias Maia Andr Linhares Rodrigues Levi Mximo Viana Leonardo Ribeiro de Castro Carvalho Wilson Camara Marriel Fabiano Edson Carlos Adenilson Arcanjo de Moura Junior Edson Augusto Bezerra Lopes Rodrigo Viana Soares Eduardo Fischer Rafael Sampaio de Rezende Rafael Montezuma Pinheiro Cabral Gustavo Sampaio Sousa Ramon Moreira Nunes Hector Kenzo Horiuti Kitahara Francisco Tarcsio Guedes Lima Verde Neto Alexandre Hideki Deguchi Martani Enzo Haruo Hiraoka Moriyama Rafael Morioka Oda Andr Lucas Ribeiro dos Santos Michel Faleiros Martins Antnio Felipe Cavalcante Carvalho Rafael Moura e Sucupira Artur de Almeida Losnak Tiago Porto Barbosa Willy George do Amaral Petrenko Douglas Bokliang Ang Cunha Breno Vieira de Aguiar Beatriz Laiate Vinicius Gripp Barros Ramos Lucio Eiji Assaoka Hossaka Mateus Oliveira de Figueiredo Marcus Edson Barreto Brito Flvio Henrique Moura Stakoviak Ricardo Turolla Bortolotti Pedro Henrique Silva Belisrio Filipe Alves Tom Frederico de Souza Frydman Heytor Bruno Nobre Pitombeira das Virgens Daniel Lopes Alves de Medeiros CIDADE ESTADO So Paulo SP So Paulo SP Rio de Janeiro RJ Fortaleza CE Fortaleza CE Rio de Janeiro RJ So Paulo SP So Paulo SP Fortaleza CE So Paulo SP Fortaleza CE Fortaleza CE Fortaleza CE So Paulo SP Rio de Janeiro RJ Fortaleza CE Fortaleza CE Fortaleza CE Fortaleza CE Encantado RS Fortaleza CE Fortaleza CE Fortaleza CE Fortaleza CE So Paulo SP Fortaleza CE So Paulo SP So Paulo SP So Paulo SP Pindamonhangaba SP Campinas SP Fortaleza CE Fortaleza CE So Paulo SP Fortaleza CE Rio de Janeiro RJ S. J. dos Campos SP Fortaleza CE Sorocaba SP Rio de Janeiro RJ Curitiba PR Fortaleza CE Fortaleza CE Belm PA Rio Claro SP Rio de Janeiro RJ Fortaleza CE Salvador BA Fortaleza CE Fortaleza CE PRMIO Ouro Ouro Ouro Ouro Ouro Ouro Prata Prata Prata Prata Prata Prata Prata Prata Prata Prata Bronze Bronze Bronze Bronze Bronze Bronze Bronze Bronze Bronze Bronze Bronze Bronze Bronze Bronze Bronze Bronze Meno Honrosa Meno Honrosa Meno Honrosa Meno Honrosa Meno Honrosa Meno Honrosa Meno Honrosa Meno Honrosa Meno Honrosa Meno Honrosa Meno Honrosa Meno Honrosa Meno Honrosa Meno Honrosa Meno Honrosa Meno Honrosa Meno Honrosa Meno Honrosa

EUREKA! N24, 2006

75

Sociedade Brasileira de Matemtica

Nvel Universitrio
NOME Humberto Silva Naves Bernardo Freitas Paulo da Costa Alex Corra Abreu Rafael Daigo Hirama Digo Veloso Ucha Fbio Dias Moreira Lus Daniel Barbosa Coelho Carlos Stein Naves de Brito Yuri Gomes Lima Rafael Marini Silva Murilo Vasconcelos Andrade Thiago Barros Rodrigues Costa Felipe Rodrigues Nogueira de Souza Leonardo Augusto Zo Vitor Gabriel Kleine Estillac Lins Maciel Borges Filho Rodrigo Roque Dias Eduardo de Moraes Rodrigues Poo Gustavo Gomes de Araujo Raphael Constant da Costa Davi Maximo Alexandrino Nogueira Jorge Peixoto de Morais Neto Eduardo Ferraz Castelo Branco Ferreira Eduardo Famini Silva Moyses Afonso Assad Cohen Kellem Corra Santos Evandro Makiyama Thiago da Silva Sobral Pedro Paiva Zhlke Dioliveira Helder Oliveira de Castro Thiago Costa Leite Santos Marcos Francisco Ferreira Martinelli Rogrio de Assis Medeiros Samuel Barbosa Feitosa Elder Rodrigo Barbosa Campos Francisco Bruno de Lima Holanda Giovana Siracusa Gouveia Henrique Roscoe de Oliveira CIDADE ESTADO S. J. dos Campos SP Rio de Janeiro RJ Niteri RJ Campinas SP Teresina PI Rio de Janeiro RJ Rio de Janeiro RJ S.J. dos Campos SP Fortaleza CE Vila Velha ES Macei AL Fortaleza CE So Paulo SP Nilpolis RJ Mogi das Cruzes SP Belm PA So Paulo SP So Paulo SP Ribeiro Preto SP Rio de Janeiro RJ Fortaleza CE Goinia GO Rio de Janeiro RJ Rio de Janeiro RJ Rio de Janeiro RJ Rio de Janeiro RJ So Paulo SP S.J. dos Campos SP Braslia DF Mogi das Cruzes SP So Paulo SP Rio de Janeiro RJ Franco da Rocha SP Fortaleza CE Rio de Janeiro RJ Fortaleza CE Recife PE Braslia DF PRMIO Ouro Ouro Ouro Ouro Ouro Ouro Prata Prata Prata Prata Prata Prata Prata Prata Prata Bronze Bronze Bronze Bronze Bronze Bronze Bronze Bronze Bronze Bronze Meno Honrosa Meno Honrosa Meno Honrosa Meno Honrosa Meno Honrosa Meno Honrosa Meno Honrosa Meno Honrosa Meno Honrosa Meno Honrosa Meno Honrosa Meno Honrosa Meno Honrosa

EUREKA! N24, 2006

76

Sociedade Brasileira de Matemtica

AGENDA OLMPICA
XXVIII OLIMPADA BRASILEIRA DE MATEMTICA NVEIS 1, 2 e 3 Primeira Fase Sbado, 10 de junho de 2006 Segunda Fase Sbado, 2 de setembro de 2006 Terceira Fase Sbado, 28 de outubro de 2006 (nveis 1, 2 e 3) Domingo, 29 de outubro de 2006 (nveis 2 e 3 - segundo dia de prova). NVEL UNIVERSITRIO Primeira Fase Sbado, 2 de setembro de 2006 Segunda Fase Sbado, 28 e Domingo, 29 de outubro de 2006

XII OLIMPADA DE MAIO 13 de maio de 2006

XVII OLIMPADA DE MATEMTICA DO CONE SUL 5 a 11 de maio de 2006 Escobar, Argentina

XLVII OLIMPADA INTERNACIONAL DE MATEMTICA 8 a 19 de julho de 2006 Ljubljana - Eslovnia.

XIII OLIMPADA INTERNACIONAL DE MATEMTICA UNIVERSITRIA 20 a 26 de julho de 2006 Odessa, Ucrnia

XXI OLIMPADA IBEROAMERICANA DE MATEMTICA 22 de setembro a 01 de outubro de 2006 Equador

IX OLIMPADA IBEROAMERICANA DE MATEMTICA UNIVERSITRIA 18 de novembro de 2006

EUREKA! N24, 2006

77

Sociedade Brasileira de Matemtica

COORDENADORES REGIONAIS
Alberto Hassen Raad Amrico Lpez Glvez Amarsio da Silva Arajo Andreia Goldani Antonio Carlos Nogueira Ali Tahzibi Benedito Tadeu Vasconcelos Freire Carlos Alexandre Ribeiro Martins Carlos Frederico Borges Palmeira Claus Haetinger Cleonor Crescncio das Neves Cludio de Lima Vidal Edson Roberto Abe lio Mega der Luiz Pereira de Andrade Eudes Antonio da Costa Florncio Ferreira Guimares Filho Ivanilde Fernandes Saad Janice T. Reichert Joo Bencio de Melo Neto Joo Francisco Melo Libonati Jorge Costa Duarte Filho Jos Cloves Saraiva Jos Luiz Rosas Pinho Jos Vieira Alves Jos William Costa Krerley Oliveira Licio Hernandes Bezerra Luzinalva Miranda de Amorim Mrio Rocha Retamoso Marcelo Rufino de Oliveira Marcelo Mendes Newman Simes Ral Cintra de Negreiros Ribeiro Ronaldo Alves Garcia Rogrio da Silva Igncio Reginaldo de Lima Pereira Reinaldo Gen Ichiro Arakaki Ricardo Amorim Srgio Cludio Ramos Seme Guevara Neto Tadeu Ferreira Gomes Toms Menndez Rodrigues Turbio Jos Gomes dos Santos Valdenberg Arajo da Silva Valdeni Soliani Franco Vnia Cristina Silva Rodrigues Wagner Pereira Lopes (UFJF) (USP) (UFV) FACOS (UFU) (USP) (UFRN) (Univ. Tec. Fed. De Paran) (PUC-Rio) (UNIVATES) (UTAM) (UNESP) (Colgio Objetivo de Campinas) (Colgio Etapa) (UNESPAR/FECILCAM) (Univ. do Tocantins) (UFES) (UC. Dom Bosco) (UNOCHAPEC) (UFPI) (Grupo Educacional Ideal) (UFPB) (UFMA) (UFSC) (UFPB) (Instituto Pueri Domus) (UFAL) (UFSC) (UFBA) (UFRG) (Grupo Educacional Ideal) (Colgio Farias Brito, Pr-vestibular) (Cursinho CLQ Objetivo) (Colgio Anglo) (UFGO) (Col. Aplic. da UFPE) (Escola Tcnica Federal de Roraima) (LAC - Laboratrio Associado de Computao) (Centro Educacional Logos) (IM-UFRGS) (UFMG) (UEBA) (U. Federal de Rondnia) (UFPB) (U. Federal de Sergipe) (U. Estadual de Maring) (U. Metodista de SP) (CEFET GO) Juiz de Fora MG Ribeiro Preto SP Viosa MG Osrio RS Uberlndia MG So Carlos SP Natal RN pato Branco - PR Rio de Janeiro RJ Lajeado RS Manaus AM S.J. do Rio Preto SP Campinas SP So Paulo SP Campo Mouro PR Arraias TO Vitria ES Campo Grande MS Chapec SC Teresina PI Belm PA Joo Pessoa - PB So Luis MA Florianpolis SC Campina Grande PB Santo Andr SP Macei AL Florianpolis SC Salvador BA Rio Grande RS Belm PA Fortaleza CE Piracicaba SP Atibaia SP Goinia GO Recife PE Boa Vista RR SJ dos Campos SP Nova Iguau RJ Porto Alegre RS Belo Horizonte MG Juazeiro BA Porto Velho RO Joo Pessoa PB So Cristovo SE Maring PR S.B. do Campo SP Jata GO

EUREKA! N24, 2006

78

CONTEDO
AOS LEITORES XI OLIMPADA DE MAIO Enunciados e Resultado Brasileiro XII OLIMPADA DE MAIO Enunciados e Resultado Brasileiro XVI OLIMPADA DE MATEMTICA DO CONE SUL Enunciados e Resultado Brasileiro XVII OLIMPADA DE MATEMTICA DO CONE SUL Enunciados e Resultado Brasileiro XLVI OLIMPADA INTERNACIONAL DE MATEMTICA Enunciados e Resultado Brasileiro XLVII OLIMPADA INTERNACIONAL DE MATEMTICA Enunciados e Resultado Brasileiro XX OLIMPADA IBEROAMERICANA DE MATEMTICA Enunciados e Resultado Brasileiro XXI OLIMPADA IBEROAMERICANA DE MATEMTICA Enunciados e Resultado Brasileiro 3 4 7

12

14 15

18 20

ARTIGOS
A FRMULA DE HERO Fabiano Alberton de Alencar Nogueira REAS PARA ACHAR RAZES DE SEGMENTOS Ccero Thiago e Marcelo Mendes PROBLEMAS SOBRE PONTOS Davi Mximo e Samuel Feitosa POLINMIOS SIMTRICOS Carlos A. Gomes 22 26

31 46

Sociedade Brasileira de Matemtica

OLIMPADAS AO REDOR DO MUNDO SOLUES DE PROBLEMAS PROPOSTOS PROBLEMAS PROPOSTOS AGNDA OLMPICA COORDENADORES REGIONAIS

53 57 60 61 62

EUREKA! N25, 2007

Sociedade Brasileira de Matemtica

AOS LEITORES
Chegamos ao nmero 25 da Eureka! apresentando as provas e os excelentes resultados brasileiros dos dois ltimos anos em diversas competies internacionais de que o Brasil participa. Temos tambm quatro belos artigos e, atendendo a muitos pedidos, a volta da seo Olimpadas ao redor do Mundo, agora com mais colaboradores. Agradecemos e continuamos estimulando a participao da comunidade olmpica na elaborao da Eureka! com problemas propostos, solues e artigos, que tm feito da Eureka! um instrumento vivo de difuso das olimpadas de Matemtica no Brasil, contribuindo para a preparao em alto nvel dos participantes da OBM em todo o pas.

Os editores

EUREKA! N25, 2007

Sociedade Brasileira de Matemtica

XI OLIMPADA DE MAIO
PRIMEIRO NVEL
PROBLEMA 1

Num quadro negro havia seis figuras: um crculo, um tringulo, um quadrado, um trapzio, um pentgono e um hexgono, pintados de seis cores: azul, branco, vermelho, amarelo, verde e marrom. Cada figura tinha somente uma cor e todas as figuras eram de cores diferentes. No dia seguinte perguntou-se qual era a cor de cada figura. Pablo respondeu: "O crculo era vermelho, o tringulo era azul, o quadrado era branco, o trapzio era verde, o pentgono era marrom e o hexgono era amarelo." Sofia respondeu: "O crculo era amarelo, o tringulo era verde, o quadrado era vermelho, o trapzio era azul, o pentgono era marrom e o hexgono era branco." Pablo errou trs vezes e Sofia duas vezes, e sabe-se que o pentgono era marrom. Determine se possvel saber com certeza qual era a cor de cada uma das figuras.
PROBLEMA 2

Um nmero inteiro chama-se autodivi se divisvel pelo nmero de dois algarismos formado por seus dois ltimos dgitos (dezenas e unidades). Por exemplo, 78013 autodivi pois divisvel por 13, 8517 autodivi pois divisvel por 17. Encontre 6 nmeros inteiros consecutivos que sejam autodivi e que tenham os dgitos das unidades, das dezenas e das centenas distintos de 0.
PROBLEMA 3

Um segmento AB de largura 100 est dividido em 100 segmentos menores de largura 1 mediante 99 pontos intermedirios. Ao extremo A designa-se o 0 e ao extremo B, o 1. Gustavo designa a cada um dos 99 pontos intermedirios um 0 ou um 1, a sua escolha, e logo pinta cada segmento de largura 1 de azul ou de vermelho, respeitando a seguinte regra: So vermelhos todos os segmentos que tm o mesmo nmero em seus extremos e so azuis os segmentos que tm nmeros diferentes em seus extremos. Determine se Gustavo pode designar os 0's e os 1's de modo a obter exatamente 30 segmentos azuis. E 35 segmentos azuis? (Em cada caso, se a resposta sim, mostre uma distribuio dos 0's e dos 1's, e se a resposta no, explique o porqu.)
PROBLEMA 4

H duas figuras de papel: um tringulo equiltero e um retngulo. A altura do retngulo igual altura do tringulo e a base do retngulo igual base do
EUREKA! N25, 2007

Sociedade Brasileira de Matemtica

tringulo. Divida o tringulo em trs partes e o retngulo em duas, mediante cortes retos, de modo que com os cinco pedaos possamos montar, sem buracos nem superposies, um tringulo equiltero. Para montar a figura, cada parte pode ser girada e/ou dar a volta. (Justifique que o tringulo montado equiltero.)
PROBLEMA 5

a) Em cada casa de um tabuleiro 7 7 se escreve um dos nmeros: 1, 2, 3, 4, 5, 6 ou 7 de forma que cada nmero esteja escrito em sete casas distintas. Ser possvel que em nenhuma fila e em nenhuma coluna fiquem escritos nmeros consecutivos? b) Em cada casa de um tabuleiro 5 5 se escreve um dos nmeros: 1, 2, 3, 4 ou 5 de forma que cada nmero esteja escrito em cinco casas distintas. Ser possvel que em nenhuma fila e nenhuma coluna fiquem escritos nmeros consecutivos?

SEGUNDO NVEL
PROBLEMA 1

Determine o menor nmero de trs dgitos que seja o produto de dois nmeros de dois dgitos, de forma que os sete dgitos destes trs nmeros sejam todos diferentes.
PROBLEMA 2

Gonalo escreve num quadro negro quatro nmeros escolhidos entre 0, 1, 2, 3 ou 4. Pode repetir nmeros. Nicols realiza repetidas vezes a seguinte operao: troca um dos nmeros, a sua escolha, pelo resto da diviso por 5 do produto de outros dois nmeros do quadro negro, a sua escolha. O objetivo de Nicols conseguir que os quatro nmeros sejam iguais. Determine se Gonalo pode escolher os nmeros iniciais de forma que seja impossvel a Nicols alcanar seu objetivo.
PROBLEMA 3

No tringulo issceles ABC, com AB = AC, seja M o ponto mdio de BC. O Ponto = 1 BAC . A reta perpendicular a AD por C corta a D no lado BC tal que BAD 6 AD em N de modo que DN = DM. Calcule os ngulos do tringulo ABC.
PROBLEMA 4

Num baile h 12 homens, numerados de 1 a 12 e 12 mulheres, numeradas de 1 a 12. A cada homem se designa um "amigo oculto" entre os outros 11. Todos danaram todas as msicas. Na primeira msica cada homem danou com a
EUREKA! N25, 2007

Sociedade Brasileira de Matemtica

mulher que tem seu mesmo nmero. A partir da, cada homem danou uma nova msica com uma mulher que havia danado a msica anterior com seu amigo oculto. Na terceira msica os casais foram: Homens 1 2 3 4 5 6 7 8 9 Mulheres 5 11 2 12 8 10 9 4 6 Encontre o nmero do amigo oculto de cada homem.
PROBLEMA 5

10 11 12 3 7 1

Sobre o tabuleiro 9 9 aterrissou a nave inimiga que cobre exatamente 5 casas do tabuleiro, assim:

A nave invisvel. Cada mssil defensivo cobre exatamente uma casa, e destri a nave se bater numa das 5 casas que esta ocupa. Determine o nmero mnimo de msseis que so necessrios para destruir com certeza a nave inimiga.
RESULTADOS BRASILEIROS PRIMEIRO NVEL Leonardo Pereira Stedile James Jun Hong Thiago Gonales Csar Ilharco Magalhes Fernando Fonseca Andrade Oliveira Erick Magno Costa Alonso Mara Islena T. da Silva Matheus Barros de Paula Wagner Carlos Morto Loyola Filho Andr Y. O. Bastos RESULTADOS BRASILEIROS SEGUNDO NVEL Henrique Pond de Oliveira Pinto Rafael Tupinamb Dutra Thiago Ribeiro Ramos Victor Reis de Abreu Cavalcante Lucas Zanotto Portela Lucio Eiji Assaoka Hossaka Tiago Madeira Hugo Musso Gualandi Giuliano Pezzolo Giacaglia Wilson Camara Marriel Illan Feiman Halpern So Paulo - SP So Paulo - SP Piracicaba - SP Juiz de Fora - MG Belo Horizonte - MG Uberaba - MG Belo Horizonte - MG Taubat - SP Vitria - ES So Paulo - SP Medalha de Ouro Medalha de Prata Medalha de Prata Medalha de Bronze Medalha de Bronze Medalha de Bronze Medalha de Bronze Meno Honrosa Meno Honrosa Meno Honrosa

Salvador - BA Belo Horizonte - MG Varginha - MG Macei - AL Curitiba - PR Curitiba - PR Itaja - SC Vitria - ES Santo Andr - SP Rio de Janeiro - RJ Itatiaia - RJ

Medalha de Ouro Medalha de Prata Medalha de Prata Medalha de Bronze Medalha de Bronze Medalha de Bronze Medalha de Bronze Meno Honrosa Meno Honrosa Meno Honrosa Meno Honrosa

EUREKA! N25, 2007

Sociedade Brasileira de Matemtica

XII OLIMPADA DE MAIO


PRIMEIRO NVEL
PROBLEMA 1

Um calendrio digital exibe a data: dia, ms e ano, com 2 dgitos para o dia, 2 dgitos para o ms e 2 dgitos para o ano. Por exemplo, 01-01-01 corresponde a primeiro de janeiro de 2001 e 25-05-23 corresponde a 25 de maio de 2023. Em frente ao calendrio h um espelho. Os dgitos do calendrio so como os da figura abaixo:



Se 0, 1, 2, 5 e 8 se refletem, respectivamente, em 0, 1, 5, 2 e 8, e os outros dgitos perdem sentido ao se refletirem, determine quantos dias do sculo, ao se refletirem no espelho, correspondem tambm a uma data.
PROBLEMA 2

Um retngulo de papel de 3cm 9cm dobrado ao longo de uma reta, fazendo coincidir dois vrtices opostos. Deste modo se forma um pentgono. Calcular sua rea.
PROBLEMA 3

H 20 pontos alinhados, separados por uma mesma distncia:


Miguel tem que pintar de vermelho trs ou mais destes pontos, de maneira que os pontos vermelhos estejam separados por uma mesma distncia e seja impossvel pintar de vermelho exatamente um ponto a mais sem desobedecer a condio anterior. Determinar de quantas maneiras Miguel poder fazer a tarefa.
PROBLEMA 4

Com 150 cubinhos brancos de 1 1 1 arma-se um paraleleppedo de 6 5 5, pintam-se as seis faces de azul e logo se desarma o paraleleppedo. Lucrecia deve armar um novo paraleleppedo, sem buracos, usando exclusivamente cubinhos que tenham ao menos uma face azul e de modo que as faces do paraleleppedo de Lucrcia sejam todas completamente azuis. Determinar as dimenses do paraleleppedo de maior volume que Lucrecia pode armar.
EUREKA! N25, 2007

Sociedade Brasileira de Matemtica

PROBLEMA 5

Em algumas casas de um tabuleiro 10 10 coloca-se uma ficha de maneira que se verifique a seguinte propriedade: Para cada casa que tem uma ficha, a quantidade de fichas colocadas em sua mesma linha deve ser maior ou igual que a quantidade de fichas colocadas em sua mesma coluna. Quantas fichas pode haver no tabuleiro? Diga todas as possibilidades.

SEGUNDO NVEL
PROBLEMA 1

Determinar todos os pares de nmeros naturais a e b tais que nmeros naturais.


PROBLEMA 2

a +1 b +1 e so b a

No quadro negro esto escritos vrios nmeros primos (alguns deles repetidos). Mauro somou os nmeros do quadro negro e Fernando multiplicou os nmeros do quadro negro. O resultado que obteve Fernando igual a 40 vezes o resultado que obteve Mauro. Determinar quais podem ser os nmeros do quadro negro. Diga todas as possibilidades.
PROBLEMA 3

Escrever um nmero inteiro positivo em cada casa de modo que: Os seis nmeros sejam distintos. A soma dos seis nmeros seja 100. Se cada nmero multiplicado pelo seu vizinho (no sentido dos ponteiros do relgio) e se somam os seis resultados das seis multiplicaes, obtmse o menor valor possvel. Explicar por que no possvel obter um valor menor.
PROBLEMA 4

Seja ABCD um trapzio de bases AB e CD. Seja O o ponto de interseo de suas diagonais AC e BD. Se a rea do tringulo ABC 150 e a rea do tringulo ACD 120, calcular a rea do tringulo BCO.

EUREKA! N25, 2007

Sociedade Brasileira de Matemtica

PROBLEMA 5

Com 28 pontos forma-se uma "grade triangular" de lados iguais, como se mostra na figura abaixo. Uma operao consiste em escolher trs pontos que sejam os vrtices de um tringulo equiltero e retirar estes trs pontos da grade. Se aps realizar vrias destas operaes resta somente um ponto, em quais posies pode ficar esse ponto? Determinar todas as possibilidades e indicar em cada caso as operaes realizadas. Justificar por que o ponto que restou no pode estar numa outra posio.

RESULTADOS BRASILEIROS PRIMEIRO NVEL


Matheus Barros de Paula Csar Ilharco Magalhes Henrique L. de Mello Iuri Rezende Souza Elder Massahiro Yoshida Deborah Barbosa Alves Victor Gonalves Elias Leonardo Gonalves Fischer Wagner Carlos Morto Loyola Filho Ivan Seiki Hellmeister Taubat - SP Juiz de Fora - MG Rio de Janeiro - RJ Mineiros - GO So Paulo - SP So Paulo - SP Joo Pessoa - PB Fraiburgo - SC Vitria - ES So Paulo - SP Medalha de Ouro Medalha de Prata Medalha de Prata Medalha de Bronze Medalha de Bronze Medalha de Bronze Medalha de Bronze Meno Honrosa Meno Honrosa Meno Honrosa

RESULTADOS BRASILEIROS SEGUNDO NVEL


Thiago Ribeiro Ramos Marcelo Tadeu de S O. Sales Rafael Horimoto de Freitas Renan Henrique Finder Illan Feiman Halpern Renan Lima Novais Rafael Rabelo de Carvalho Rafael Pacheco Gomes Caio Srgio Parente Silva Hugo Fonseca Arajo Varginha - MG Barreiras - BA So Paulo - SP Joinville - SC Itatiaia - RJ Niteri - RJ Braslia - DF Fortaleza - CE Rio de Janeiro - RJ Juiz de Fora - MG Medalha de Ouro Medalha de Prata Medalha de Prata Medalha de Bronze Medalha de Bronze Medalha de Bronze Medalha de Bronze Meno Honrosa Meno Honrosa Meno Honrosa

EUREKA! N25, 2007

Sociedade Brasileira de Matemtica

XVI OLIMPADA DE MATEMTICA DO CONE SUL


Enunciados e Resultado Brasileiro
A XVI Olimpada de Matemtica do Cone Sul foi realizada na cidade de Sucre, Bolvia no perodo de 14 a 23 de Maio de 2005. A equipe brasileira foi liderada pelos professores Emanuel Augusto de Souza Carneiro e Davi Alexandrino Nogueira, ambos da cidade de Fortaleza CE.
RESULTADOS DA EQUIPE BRASILEIRA BRA1 BRA2 BRA3 BRA4 Henrique Pond de Oliveira Pinto Guilherme R. Nogueira de Souza Edson Augusto Bezerra Lopes Rafael Tupynamb Dutra Medalha de Ouro Medalha de Ouro Medalha de Prata Medalha de Prata

PRIMEIRO DIA PROBLEMA 1

Considere a seguinte seqncia: a1 = ltimo dgito da soma dos dgitos do nmero 2005 a2 = ltimo dgito da soma dos dgitos do nmero 20052005 a3 = ltimo dgito da soma dos dgitos do nmero 200520052005 ... an = ltimo dgito da soma dos dgitos do nmero 20052005... 2005

n vezes 2005

Calcule: a1 + a2 + a3 + + a2005
PROBLEMA 2

Seja ABC um tringulo acutngulo e sejam AN, BM e CP as alturas relativas aos lados BC, CA e AB, respectivamente. Sejam R, S as projees de N sobre os lados AB, CA, respectivamente, e Q, W as projees de N sobre as alturas BM e CP, respectivamente. Mostre que R, Q, W, S so colineares; Mostre que MP = RS QW.

EUREKA! N25, 2007

10

Sociedade Brasileira de Matemtica

PROBLEMA 3

A unidade monetria de um certo pas se chama reo, e todas as moedas que circulam so de nmeros inteiros de reos. Em um grupo de trs pessoas, cada uma tem 60 reos em moedas (mas no se sabe que tipo de moedas cada uma tem). Cada uma das trs pessoas pode pagar a cada uma das outras qualquer valor inteiro entre 1 e 15 reos, inclusive, talvez com troco. Mostre que as trs pessoas em conjunto podem pagar exatamente (sem troco) qualquer valor inteiro entre 45 e 135 reos, inclusive.
SEGUNDO DIA PROBLEMA 4

Seja ABC um tringulo issceles, com AB = AC. Uma reta r que passa pelo incentro I de ABC intersecta os lados AB e AC nos pontos D e E, respectivamente. F e G so pontos sobre o lado BC tais que BF = CE e CG = BD. Mostre que o ngulo FIG constante ao variar r.
PROBLEMA 5

Diremos que um nmero de 20 dgitos especial se impossvel represent-lo como produto de um nmero de 10 dgitos por um nmero de 11 dgitos. Determine qual a mxima quantidade possvel de nmeros consecutivos que so especiais.
PROBLEMA 6

No plano cartesiano traamos circunferncias de raio 1/20 com centros em cada ponto de coordenadas inteiras. Mostre que qualquer circunferncia de raio 100 que se trace no plano intersecta pelo menos uma das circunferncias pequenas.

EUREKA! N25, 2007

11

Sociedade Brasileira de Matemtica

XVII OLIMPADA DE MATEMTICA DO CONE SUL


Enunciados e Resultado Brasileiro
A XVII Olimpada de Matemtica do Cone Sul foi realizada na cidade de Escobar, Argentina no perodo de 5 a 11 de Maio de 2006. A equipe brasileira foi liderada pelos professores Carlos Yuzo Shine (So Paulo SP) e Luzinalva Miranda de Amorim (Salvador BA).
RESULTADOS DA EQUIPE BRASILEIRA BRA1 BRA2 BRA3 BRA4 Henrique Pond de Oliveira Pinto Rafael Tupynamb Dutra Ramon Moreira Nunes Regis Prado Barbosa Medalha de Ouro Medalha de Prata Medalha de Prata Medalha de Prata

PRIMEIRO DIA PROBLEMA 1

No quadriltero convexo ABCD, sejam E e F os pontos mdios dos lados AD e BC, respectivamente. Os segmentos CE e DF cortam-se em O. Demonstrar que se as retas AO e BO dividem o lado CD em trs partes iguais ento ABCD um paralelogramo.
PROBLEMA 2

Duas pessoas, A e B, jogam o seguinte jogo: eles retiram moedas de uma pilha que contm, inicialmente, 2006 moedas. Os jogadores jogam alternadamente retirando, em cada jogada, 1 a 7 moedas; cada jogador guarda as moedas que retira. Se quiser, um jogador pode passar (no retirar moedas em sua vez), mas para isso deve pagar 7 moedas das que retirou da pilha em jogadas anteriores. Estas 7 moedas so colocadas em uma caixa separada e no interferem mais no jogo. Ganha quem retira a ltima moeda, e A comea o jogo. Determinar qual jogador pode assegurar a vitria, no importando como jogue o outro. Mostrar uma estratgia vencedora e explicar por que vencedora.
PROBLEMA 3

Seja n um nmero natural. A sucesso finita de inteiros positivos tem, entre seus termos, exatamente n nmeros distintos ( pode ter nmeros repetidos). Alm disso, se de um de seus termos qualquer subtramos 1, obtemos uma sucesso que tem, entre seus termos, pelo menos n nmeros positivos distintos. Qual o valor mnimo que pode ter a soma de todos os termos da sucesso ?

EUREKA! N25, 2007

12

Sociedade Brasileira de Matemtica

SEGUNDO DIA PROBLEMA 4

Daniel escreveu em uma lousa, de cima para baixo, uma lista de nmeros inteiros positivos menores ou iguais a 10. Ao lado de cada nmero da lista de Daniel, Martn anotou a quantidade de vezes que esse nmero aparece na lista de Daniel e obteve assim uma lista de mesmo tamanho. Se lemos a lista de Martn de baixo para cima obtemos a mesma lista de nmeros que Daniel escreveu de cima para baixo. Encontre o maior tamanho que a lista de Daniel pode ter. Encontrar todos os inteiros positivos n tais que n 2 divide n 4 e n + 2 divide n + 4 . ( [r ] denota a parte inteira de r, ou seja, o maior inteiro que menor ou igual a r. Por exemplo: [2, 5] = 2 ; 3 = 1 ; [5] = 5 .)
PROBLEMA 6 PROBLEMA 5

Dividimos o plano em casinhas quadradas de lado 1, traando retas paralelas aos eixos coordenados. Cada casinha pintada de branco ou preto. A cada segundo, recolorimos simultaneamente todas as casinhas, de acordo com a seguinte regra: cada casinha Q adota a cor que mais aparece na configurao de cinco casinhas indicadas na figura
Q

O processo de recolorao repetido indefinidamente. a) Determinar se existe uma colorao inicial com uma quantidade finita de casinhas pretas tal que sempre h pelo menos uma casinha preta, no importando quantos segundos se passaram desde o incio do processo. b) Determinar se existe uma colorao inicial com uma quantidade finita de casinhas pretas tal que o nmero de casinhas pretas, aps alguma quantidade de segundos, seja pelo menos 1010 vezes maior que o nmero inicial de casinhas pretas.

EUREKA! N25, 2007

13

Sociedade Brasileira de Matemtica

XLVI OLIMPADA INTERNACIONAL DE MATEMTICA


Enunciados e Resultado Brasileiro
A XLVI Olimpada Internacional de Matemtica foi realizada na cidade de Mrida Mxico no perodo de 08 a 19 de julho de 2005. A equipe brasileira foi liderada pelos professores Edmilson Luis Rodrigues Motta (So Paulo SP) e Onofre Campos da Silva Farias (Fortaleza CE).
RESULTADOS DA EQUIPE BRASILEIRA BRA1 BRA2 BRA3 BRA4 BRA5 BRA6 Gabriel Tavares Bujokas Thoms Yoiti Sasaki Hoshina Leandro Farias Maia Guilherme Rodrigues Nogueira de Souza Levi Mximo Viana Edson Augusto Bezerra Lopes Medalha de Ouro Medalha de Bronze Meno Honrosa Meno Honrosa **** ****

PRIMEIRO DIA PROBLEMA 1

So escolhidos seis pontos nos lados de um tringulo equiltero ABC: A1 e A2 em BC, B1 e B2 em CA, C1 e C2 em AB. Estes pontos so os vrtices de um hexgono convexo A1 A2 B1B2C1C 2 cujos lados so todos iguais. Demonstre que as retas A1 B2 , B1C 2 e C1 A2 so concorrentes.
PROBLEMA 2

Seja a1 , a2 ,... uma seqncia de inteiros que tem infinitos termos positivos e infinitos trmos negativos. Suponhamos que para cada inteiro positivo n, os nmeros a1 , a2 ,..., an tem n restos distintos ao ser divididos entre n. Demonstre que cada inteiro se encontra exatamente uma vez na sucesso.
PROBLEMA 3

Sejam x, y, z nmeros reais positivos tais que xy z 1 . Demonstre que

x5 x 2 y5 y 2 z5 z 2 + + 0. x5 + y 2 + z 2 y 5 + z 2 + x 2 z 5 + x 2 + y 2

EUREKA! N25, 2007

14

Sociedade Brasileira de Matemtica

SEGUNDO DIA PROBLEMA 4

Consideremos a seqncia infinita a 1 , a 2 , ... definida por

an = 2 n + 3n + 6 n 1

( n = 1, 2 , ...)

Determine todos os inteiros positivos que so relativamente primos relativos (coprimos) com todos os termos da seqncia.
PROBLEMA 5

Seja ABCD um quadriltero convexo que tem os lados BC e AD iguais e no paralelos. Sejam E e F pontos nos lados BC e AD, respectivamente, que so distintos dos vrtices e satisfazem BE = DF. As retas AC e BD se cortam em P e a reta EF corta AC e BD respectivamente em Q e R. Consideremos todos os tringulos PQR que se formam quando E e F variam. Demonstre que as circunferncias circunscritas a esses tringulos tm em comum outro ponto alm de P.
PROBLEMA 6

Numa competio de matemtica foram propostos 6 problemas aos estudantes. Cada par de problemas foi resolvido por mais de 2 5 dos estudantes. Ningum resolveu os 6 problemas. Demonstre que h pelo menos 2 estudantes tais que cada um tem exatamente 5 problemas resolvidos.

EUREKA! N25, 2007

15

Sociedade Brasileira de Matemtica

XLVII OLIMPADA INTERNACIONAL DE MATEMTICA


Enunciados e Resultado Brasileiro
A XLVII Olimpada Internacional de Matemtica foi realizada na cidade de Ljubljana Eslovnia no perodo de 08 a 19 de julho de 2006. A equipe brasileira foi liderada pelos professores Luciano Guimares Monteiro de Castro (Rio de Janeiro RJ) e Pablo Rodrigo Ganassim (So Paulo SP).
RESULTADOS DA EQUIPE BRASILEIRA BRA1 BRA2 BRA3 BRA4 BRA5 BRA6 Andr Linhares Rodrigues Guilherme Rodrigues Nogueira de Souza Leandro Farias Maia Leonardo Ribeiro de Castro Carvalho Rafael Mendes de Oliveira Rgis Prado Barbosa Medalha de Bronze Medalha de Bronze Medalha de Bronze Medalha de Bronze Medalha de Bronze Medalha de Bronze

PRIMEIRO DIA PROBLEMA 1

Seja ABC um tringulo com incentro I. Um ponto P no interior do tringulo verifica Prove que AP AI , com igualdade se, e somente se, P = I.
PROBLEMA 2

PBA + PCA = PBC + PCB.

Uma diagonal de um polgono regular P de 2006 lados um segmento bom se separa P em duas partes, cada uma tendo um nmero mpar de lados de P. Os lados de P tambm so segmentos bons. Divide-se P em tringulos, traando-se 2003 diagonais que, duas a duas, no se cortam no interior de P. Determine o maior nmero de tringulos issceles nos quais dois lados so segmentos bons que podem aparecer numa diviso como essa.
PROBLEMA 3

Determine o menor nmero real M tal que a desigualdade

ab ( a 2 b2 ) + bc (b2 c 2 ) + ca ( c2 a 2 ) M ( a 2 + b2 + c 2 )

verdadeira para todos os nmeros reais a, b, c.

EUREKA! N25, 2007

16

Sociedade Brasileira de Matemtica

SEGUNDO DIA PROBLEMA 4

Determine todos os pares de inteiros (x, y) tais que

1 + 2 x + 22 x +1 = y 2 .
PROBLEMA 5

Seja P(x) um polinmio de grau n > 1 com coeficientes inteiros e seja k um inteiro positivo. Considere o polinmio Q ( x ) = P ( P(...P( P ( x))...)), onde P aparece k vezes. Prove que existem no mximo n inteiros t tais que Q(t ) = t.
PROBLEMA 6

A cada lado b de um polgono convexo P associa-se a maior das reas dos tringulos contidos em P que tm b como um dos lados. Prove que a soma das reas associadas a todos os lados de P pelo menos o dobro da rea de P.

EUREKA! N25, 2007

17

Sociedade Brasileira de Matemtica

XX OLIMPADA IBEROAMERICANA DE MATEMTICA


Enunciados e Resultado Brasileiro
A XX Olimpada Iberoamericana de Matemtica foi realizada na cidade de Cartagena de ndias Colmbia no perodo de 22 de setembro a 1 de outubro de 2005. A equipe brasileira foi liderada pelos professores lio Mega (So Paulo SP) e Yuri Gomes Lima (Fortaleza CE).
RESULTADOS DA EQUIPE BRASILEIRA BRA1 BRA2 BRA3 BRA4 Rafael Marini Silva Thoms Yoiti Sasaki Hoshina Gabriel Tavares Bujokas Thiago Costa Leite Santos Medalha de Ouro Medalha de Ouro Medalha de Ouro Medalha de Ouro

PRIMEIRO DIA PROBLEMA 1

Determine todas as triplas de nmeros reais (x, y, z) que satisfazem o seguinte sistema de equaes:

xyz = 8, x 2 y + y 2 z + z 2 x = 73, x( y z ) 2 + y ( z x) 2 + z ( x y ) 2 = 98.

PROBLEMA 2

Uma pulga salta sobre os pontos inteiros de uma reta numrica. Em seu primeiro movimento salta desde o ponto 0 e cai no ponto 1. A partir da, se num movimento a pulga salta desde o ponto a e cai no ponto b, no seguinte movimento salta desde o ponto b e cai num dos pontos b + (b a) 1, b + (b a), b + (b a) + 1. Demonstre que se a pulga caiu duas vezes sobre o ponto n, para n inteiro positivo, ento deve ter feito ao menos t movimentos, onde t o menor inteiro maior ou igual a 2 n .
PROBLEMA 3

Seja p > 3 um nmero primo. Se

1 1 1 1 n + p + p + ... + = p p 1 2 3 ( p 1) m
onde o mximo divisor comum de n e m 1, demonstre que p 3 divide n.
EUREKA! N25, 2007

18

Sociedade Brasileira de Matemtica

SEGUNDO DIA PROBLEMA 4

Dados os inteiros positivos a e b, denota-se por ( a b ) o resto que obtido ao dividir a por b. Este resto um dos nmeros 0, 1,, b 1. Encontre todos os pares de nmeros (a, p) tais que p primo e vale:

( a p ) + ( a 2p ) + ( a 3p ) + ( a 4p ) = a + p.
PROBLEMA 5

Seja O o circuncentro de um tringulo acutngulo ABC e A1 um ponto no arco menor BC da circunferncia circunscrita ao tringulo ABC. Sejam A2 e A3 pontos nos lados AB e AC respectivamente, tais que

BA1 A2 = OAC e

CA1 A3 = OAB. Demonstre que a reta A2 A3 passa pelo ortocentro do


tringulo ABC.
PROBLEMA 6

Dado um inteiro positivo n, num plano consideram-se 2n pontos alinhados A1 , A2 ,..., A2n . Cada ponto pintado de azul ou vermelho mediante o seguinte procedimento: No plano dado so traadas n circunferncias com dimetros de extremos Ai e

Aj , disjuntas duas a duas. Cada Ak ,1 k 2n, pertence exatamente a uma


circunferncia. Pintam-se os pontos de modo que os dois pontos de uma mesma circunferncia levem a mesma cor. Determine quantas coloraes distintas dos 2n pontos podem-se obter ao variar as n circunferncias e a distribuio das duas cores.

EUREKA! N25, 2007

19

Sociedade Brasileira de Matemtica

XXI OLIMPADA IBEROAMERICANA DE MATEMTICA


Enunciados e Resultado Brasileiro
A XXI Olimpada Iberoamericana de Matemtica foi realizada na cidade de Guayaquil Equador no perodo de 22 de setembro a 1 de outubro de 2006. A equipe brasileira foi liderada pelos professores Paulo Czar Pinto Carvalho (Rio de Janeiro RJ) e Ccero Thiago Magalhes (Fortaleza CE).
RESULTADOS DA EQUIPE BRASILEIRA BRA1 BRA2 BRA3 BRA4
Andr Linhares Rodrigues Guilherme Rodrigues Nogueira de Souza Leandro Farias Maia Leonardo Ribeiro de Castro Carvalho Medalha de Ouro Medalha de Ouro Medalha de Prata Medalha de Prata

PRIMEIRO DIA

No tringulo escaleno ABC, com BAC = 90 , consideram-se as circunferncias inscrita e circunscrita. A reta tangente em A circunferncia circunscrita corta a reta BC em M. Sejam S e R os pontos de tangncia da circunferncia inscrita com os catetos AC e AB, respectivamente. A reta RS corta a reta BC em N. As retas AM e SR cortam-se em U. Demonstre que o tringulo UMN issceles.
PROBLEMA 2

PROBLEMA 1

Consideram-se n nmeros reais a1, a2,..., an no necessariamente distintos. Seja d a diferena entre o maior e o menor deles e seja

s = (ai a j )
i< j

Demonstre que

(n 1)d s

n2d 4

e determine as condies que devem satisfazer estes n nmeros para que se verifique cada uma das igualdades.
PROBLEMA 3

Colocam-se os nmeros 1,2,3,...,n2 nas casas de um tabuleiro n n, em alguma ordem, um nmero por casa. Uma ficha encontra-se inicialmente na casa com o nmero n2. Em cada passo, a ficha pode mover-se para qualquer das casas que tm
EUREKA! N25, 2007

20

Sociedade Brasileira de Matemtica

um lado em comum com a casa onde se encontra. Primeiro, a ficha desloca-se para a casa com o nmero 1, e para isso toma um dos caminhos mais curtos (com menos passos) entre o n2 e o 1. Da casa com o nmero 1 desloca-se para a casa com o nmero 2, a partir da para a casa com o nmero 3, e assim sucessivamente, at regressar casa inicial, tomando em cada um desses deslocamentos o caminho mais curto. A ficha d N passos no percurso completo. Determine o menor valor e o maior valor possveis de N.
SEGUNDO DIA PROBLEMA 4

Determine todos os pares (a, b) de inteiros positivos tais que 2a + 1 e 2b 1 sejam primos entre si e a + b divida 4ab + 1 .
PROBLEMA 5

Dada uma circunferncia C , considere um quadriltero ABCD com os seus quatro lados tangentes a C, com AD tangente a C em P e CD tangente a C em Q. Sejam X e Y os pontos em que BD corta C, e M o ponto mdio de XY. Demonstre que AMP = CMQ .
PROBLEMA 6

Seja n > 1 um inteiro mpar. Sejam P0 e P1 dois vrtices consecutivos de um polgono regular de n lados. Para cada k _  /01 30
80 Pk como o vrtice do polgono dado que se encontra na mediatriz de Pk-1 e Pk-2. Determine para que valores de n a sucesso P0, P1, P2, percorre todos os vrtices do polgono

'4.H 8,- ,

":0 232582657-1 F 57 24   0 902  /J 948 0 F 4 2, 47 57 24 .43 0. /4 34 2420394 4 /08.4-0794 02  /0 80902-74 /0   547 :79 8 44507 0 $90;03 4430 /4 8 5,79 . 5,3908 /4 !$ 6:0 E 9 3 ,2 /08.4-0794 4 57 24 !$ F :2 2 -1, 4 80 :3/4 2, 47 57 24 .43 0. /4. O 574 094 .44507,9 ;4 3, 3907309 6:0 E 03.43974:  57 248 /0 0780330 '0 , www.mersenne.org 5,7, 2, 8 31472, 08 3. :8 ;0 .424 , :/,7 , ,. ,7 4:9748 57 248 /0 0780330
30402457

EUREKA! N25, 2007

21

Sociedade Brasileira de Matemtica

A FRMULA DE HERO
Fabiano Alberton de Alencar Nogueira
Nvel Intermedirio Uma frmula que sempre exerceu sobre mim um grande fascnio a frmula de Hero para o clculo da rea S de um tringulo qualquer de lados a, b e c:

S=
onde p =

p ( p a )( p b )( p c ) ,

a+b+c o semi-permetro do tringulo. Sua deduo, no entanto, 2

apresenta na maioria dos livros uma certa dose de artificialidade. O objetivo deste artigo sugerir uma deduo que me ocorreu como sendo mais natural, alm de consideravelmente curta. A inspirao veio quando estava revirando papis velhos, alimentando minhas saudades dos tempos em que competia nas Olimpadas de Matemtica. Numa dessas sesses de nostalgia, deparei-me com uma questo da Olimpada Estadual de Matemtica do Rio de Janeiro que propunha que se provasse uma desigualdade envolvendo as medidas perifricas de um tringulo qualquer e o raio r do seu crculo inscrito:

( p a)

( p b)

( p c)

1 r2

Refazendo sua soluo, vislumbrei a possibilidade de fazer as pazes com a frmula de Hero, atravs do que passo a expor. Primeiramente, listemos os pr-requisitos necessrios argumentao: A rea S de qualquer tringulo metade do produto envolvendo um par de seus lados e o seno do ngulo interno formado por eles.

1 S = ab sin C 2
b
C

EUREKA! N25, 2007

22

Sociedade Brasileira de Matemtica

Os segmentos tangentes a um mesmo crculo, traados pelo mesmo ponto, so congruentes. A figura a seguir aplica este princpio aos trs vrtices de um tringulo no qual foi construdo o crculo inscrito, cujo raio r.
z A
r

r r

Na figura acima, as letras x, y e z denotam as medidas dos pares de segmentos que so congruentes por serem tangentes ao crculo inscrito, traados respectivamente pelos vrtices C, B e A. Da mesma figura, retemos os fatos de que CB = x + y e CA = x + z . Tambm imediato que, sendo o permetro

2 p = AB + CA + CB = ( y + z ) + ( x + y ) + ( z + y ) , temos que p = x + y + z .

A rea S de qualquer tringulo igual ao produto do semi-permetro pelo raio do crculo inscrito, ou seja, S = pr .
z A z

I r

De fato, olhando para as bissetrizes do tringulo ABC, que concorrem no incentro I, vemos que AI, BI e CI dividem o interior de ABC em trs tringulos, sendo um deles CBI. O raio r a altura IT de CBI porque o crculo inscrito tangncia o lado CB no ponto T. Portanto, ao considerarmos que a rea de CBI, de base

CB = x + y e altura r, dada por


EUREKA! N25, 2007

(x + y )r
2
23

que um raciocnio anlogo permite

Sociedade Brasileira de Matemtica

concluir que as reas de ABI e ACI valem, respectivamente

( y + z)r
2 =

(x + z )r
2

e que a rea de ABC a soma das reas de CBI, ABI e ACI, temos:

S=

( x + y ) r ( y + z ) r ( x + z ) r (2 x + 2 y + 2z ) r
2 + 2 + 2 = 2

2 pr = pr 2

O Teorema de Pitgoras e um pouco de Trigonometria, em particular a frmula de

. duplicao de arcos sin 2 = 2sin cos , que ser usada com 2 = C


Estes so os ingredientes necessrios deduo da frmula de Hero, numa verso extremamente simples de se memorizar: S =

pxyz !

E aqui vamos ns! Sem precisar dar novamente nome aos bois, temos

1 = 1 ( x + y )( x + z ) sin 2 , S = ab sin C 2 2 = onde = BCI C . Ocorre que, no tringulo retngulo CTI, de catetos x e r, 2 C em funo desses parmetros, podemos expressar o seno e o coseno de = 2
lanando mo do Teorema de Pitgoras:

CI 2 = CT 2 + IT 2 = x 2 + r 2 CI = x 2 + r 2 .
I
x2 + r2

r
C 2

EUREKA! N25, 2007

24

Sociedade Brasileira de Matemtica

Logo sin

IT C = = 2 CI

r x +r
2 2

e cos

CT C = = 2 CI

x x + r2
2

. Por outro lado,


2 xr = 2 2 . +r x

x r = 2sin C cos C , temos sin C = 2 sabendo que sin C 2 2 2 2 2 2 x + r x + r

Lembrando que S = pr r =

S = , substitumos sin C p

2x
2

S p
2

S x + p

na frmula

1 . A manipulao abaixo encerra a deduo, ( x + y )( x + z ) sin C 2 uma vez que se p = x + y + z , ento x = p c , y = p b e z = p a : S 2 x x ( x + z )( x + y ) 1 p 1= S = ( x + z )( x + y ) 2 2 2 S S 2 2 p 2 + x +x p p


da rea S =

S2 p p 2 + x 2 = x x 2 + xy + xz + zy S 2 + p 2 x 2 = p 2 x 2 + pxyz x x + y + z = xp p ) (

S=

pxyz =

p ( p a )( p b )( p c )

Observaes: bem possvel e provvel que algum autor j tenha feito essa deduo em essncia, porm no a encontrei na minha (pobre) bibliografia. A questo da OEM/RJ (creio ser do ano de 1988) que transcrevi pode ser resolvida usando como lema uma desigualdade bem manjada para os alunos olmpicos: a 2 + b2 + c 2 ab + ac + bc , vlida para quaisquer reais a, b e c. O fato S = pr pode e deve ser generalizado para todos os polgonos convexos circunscritveis. Curiosamente, o caso particular dos polgonos regulares, onde o raio r o aptema a p , bem mais popular do que o caso geral. Mesmo que se enfatize o caso particular S = pa p , vale a pena intuir a rea do crculo a partir das substituies a p = r e p = r .
EUREKA! N25, 2007

25

Sociedade Brasileira de Matemtica

REAS PARA ACHAR RAZES DE SEGMENTOS


Ccero Thiago e Marcelo Mendes - Grupo Teorema de Matemtica
Nvel Avanado Apresentaremos aqui uma simples, poderosa e til ferramenta geomtrica para problemas envolvendo razes de segmentos. Como conveno, denotemos por [Q] a rea do polgono Q. Seja ABC um tringulo e P, um ponto em seu interior. Sejam S = [ABC], = [ SA PBC], SB = [PAC] e SC = [PAB] (veja a figura abaixo, esquerda). Temos S = SA + SB + SC.

SC P SA B

SB

C P C
B
A

Agora, prolongue AP at A sobre BC, e defina B e C analogamente (veja figura acima, direita). Como tringulos com mesma altura tm reas proporcionais a suas bases, temos: AP [ PAB ] [ PAC ] [ PAB ] + [ PAC ] SC + S B . = = = = PA ' [ PBA '] [ PCA '] [ PBA '] + [ PCA '] SA Analogamente, BP S A + SC CP S A + S B e . Por outro lado, tambm temos = = PB ' SB PC ' Sc BA ' [ PBA '] [ ABA '] [ PAB ] SC . = = = = A ' C [ PCA '] [ ACA '] [ PAC ] S B CB ' S A AC ' S B Da mesma forma, e . = = B ' A SC C ' B SA

EUREKA! N25, 2007

26

Sociedade Brasileira de Matemtica

Vejamos alguns exemplos.


Exemplo 1: Prove o teorema de Ceva: AX, BY, CZ so cevianas concorrentes de um

tringulo ABC

AZ BX CY =1. ZB XC YA

Soluo: Primeiro, suponha que AX, BY, CZ sejam concorrentes. Pela teoria acima,

BX SC CY S A AZ BX CY , e, diretamente, = = =1. XC S B YA SC ZB XC YA AZ BX CY Reciprocamente, se = 1 e CZ no passasse pela interseo P de AX ZB XC YA e BY, ento, sendo Z a interseo de CP e AB, teramos pela primeira parte que AZ ' BX CY AZ AZ ' , um absurdo. Logo, CZ passa por P e = 1 . Portanto = Z ' B XC YA ZB Z ' B AX, BY e CZ so concorrentes. temos AZ S B , = ZB S A

Z Z P B X

Y C

Exemplo 2: (HUNGRIA/1936) S um ponto no interior do ABC tal que as reas dos tringulos ABS, BCS, CAS so todas iguais. Prove que S o baricentro de ABC. Soluo: Seja T a rea dos tringulos ABS, BCS, CAS. Da, sendo M, N e P as BM CN AP T = = = =1 , intersees de AS, BS e CS com os lados opostos, temos MC NA PB T

isto , M, N e P so os pontos mdios dos lados BC, CA e AB e, portanto, S o baricentro de ABC.

EUREKA! N25, 2007

27

Sociedade Brasileira de Matemtica

N S T T M C

Exemplo 3: (BANCO IMO/1996) Seja ABC um tringulo acutngulo com circuncentro O

e raio R. Seja A1 O o ponto de interseo de AO com a circunferncia circunscrita ao tringulo BOC e defina analogamente B1 e C1. Mostre que OA1 OB1 OC1 8R3. Quando ocorre a igualdade?
Soluo:

Sejam D, E e F as intersees de AO, BO e CO com BC, CA e AB, respectivamente. fcil ver que AO = BO = CO = R. Usando as relaes provadas acima temos que:
AO [ AOB ] + [ AOC ] BO [ AOB ] + [ BOC ] , = = [ BOC ] [ AOC ] OD OE

CO [ AOC ] + [ BOC ] = . [ AOB] OF

Faa [ AOB ] = x,[ AOC ] = y ,[ BOC ] = z . fcil perceber que DCO = C BO = C AO


 e que COD comum a OA 1C e DCO , logo OA 1C OCD. Com isso,

R OA R2 = 1 OA1 = e analogamente, OD R OD OB1 = R2 R2 e OC1 = . Ento, OE OF

O A1 .O B1 .O C1 =

R6 R R R OA BO C O = R3 = R3 = O D .O E .O F OD OE OF OD OE OF

( x + y )( x + z )( y + z ) 3 R xyz

EUREKA! N25, 2007

28

Sociedade Brasileira de Matemtica

8 xyz 3 R3 = R = 8R3 . A igualdade ocorre quando x = y = z . xyz xyz Pelo exemplo 2, O tem que ser baricentro para acontecer a igualdade.

2 xy 2 yz 2 zx

A1

PROBLEMAS PROPOSTOS 1. (IME/1990;AIME/1985) Seja P um ponto no interior de um tringulo ABC,

dividindo-o em seis tringulos, quatro dos quais tm reas 40, 30, 35 e 84, como mostra a figura. Calcule a rea do tringulo ABC.

84

P
40 30

35

EUREKA! N25, 2007

29

Sociedade Brasileira de Matemtica

2. (IMO/1961) Considere tringulo P1P2P3 e um ponto P no interior do tringulo. As

retas P1P, P2P, P3P intersectam os lados opostos nos pontos Q1, Q2, Q3, PP P P P P respectivamente. Prove que dos nmeros 1 , 2 , 3 , ao menos um 2 e PQ1 PQ2 PQ3 ao menos um 2.
3. (AIME/1992) No tringulo ABC, A, B, C esto sobre os lados BC, AC e AB,

respectivamente. Dado que AA, BB, CC so concorrentes no ponto O e que AO BO CO AO BO CO . + + = 92 , encontre o valor de OA ' OB ' OC ' OA ' OB ' OC '
4. Seja P um ponto no interior do ABC. Sejam D, E, F as intersees de AP, BP,

com CP BC, CA, AB, PA PB PB PC PC PA + + 12 . PD PE PE PF PF PD

respectivamente.

Prove

que

5. No tringulo ABC, os pontos L, M, N esto sobre BC, AC, AB, respectivamente, e AL, BM, CN so concorrentes. PL PM PN . Encontre o valor numrico de + + AL BM CN AP BP CP Encontre o valor numrico de . + + AL BM CN 6. (IBERO/1985) Se AD, BE, CF so cevianas concorrentes no circuncentro O do

1 1 1 2 + + = . AD BE CF R Sugesto: Usar problema 5 ABC, demonstre que


7. Em um ABC, AD, BE, CF so concorrentes no ponto P tal que AP = PD = 6,

EP = 3, PB = 9 e CF = 20. Qual a rea do ABC?

8. Em um tringulo ABC, seja S o ponto mdio da mediana correspondente ao

vrtice A e Q, o ponto de interseo de BS com o lado AC. Mostre que BS = 3QS.


9. Seja ABC um tringulo e P um ponto em seu interior tal que AP, BP e CP

intersectam os lados BC, CA e AB nos pontos D, E e F, respectivamente. Se AP = a, BP = b, CP = c, PD = PE = PF = 3 e a + b + c = 43. Determine abc.

EUREKA! N25, 2007

30

Sociedade Brasileira de Matemtica

PROBLEMAS SOBRE PONTOS


Davi Mximo (UFC) e Samuel Feitosa (UFC) Nvel Avanado Distribuir pontos num plano ou num espao uma tarefa que pode ser realizada de forma muito arbitrria. Por isso, problemas sobre pontos podem ser de diversas naturezas. Nesse artigo, trataremos as principais tcnicas para resolver esses tipos de problemas,
1. Fecho Convexo

Pense no seguinte: dados n pontos num plano, podemos escolher alguns deles formando o nico polgono convexo que contm, junto com seu bordo e seu interior, todos os n pontos. Tal afirmao pode ser provada por induo (que alias, uma ferramenta que sempre deve ser lembrada em problemas de matemtica discreta em geral). Tal polgono chamado o fecho convexo desses n pontos. Vamos ver que to pouco j nos ajuda bastante em alguns problemas sobre pontos.
PROBLEMA 1

Seja S um conjunto finito de pontos, no havendo trs colineares, tal que dados quaisquer 4 pontos de S eles formam um quadriltero convexo. Mostre que S um conjunto de vrtices de um polgono convexo.
SOLUO:

Seja H o fecho convexo de S. Suponha um ponto P de S no interior de H. Escolha uma triangulao de H (assim como o fecho convexo, simples provar que todo polgono convexo pode ser dividido por tringulos tendo como lados diagonais ou lados do polgono, tente induo). Assim, P fica no interior de algum . tringulo ABC. Logo, o quadriltero P ABCP no convexo, absurdo! H Portanto, S no pode ter pontos no B interior do seu fecho convexo, donde S convexo, j que S no contm trs A pontos colineares. Os prximos problemas so resolvidos similarmente.
C
EUREKA! N25, 2007

31

Sociedade Brasileira de Matemtica

PROBLEMA 2

Mostre que dados 5 pontos, no trs colineares, existe um quadriltero convexo com vrtices nesses pontos.
PROBLEMA 3

Mostre que dado qualquer conjunto finito de pontos no plano existe uma reta por dois destes pontos que divide o plano em dois semi-planos de modo que um desses semi-planos no contm nenhum ponto do conjunto.
PROBLEMA 4

(Lista Cone Sul 2001) possvel que a reunio de um nmero finito de quadrilteros no convexos seja um polgono convexo? Podemos definir fecho convexo para um conjunto X qualquer do plano. Ele o menor conjunto convexo que contm X.
Definio: O fecho convexo H de X a interseo de todos os conjuntos convexos

do plano que contm X. Deixamos para o leitor a verificao dos seguintes fatos: i-) H convexo ii-) No caso de um conjunto com um nmero finito de pontos esta definio implica que H um polgono convexo cujos vrtices pertencem a este conjunto.
PROBLEMA 5

Dado um conjunto de N discos de raios unitrios. Esses crculos podem se intersectar (mas no coincidir). Mostre que existe um arco de comprimento maior ou igual a

2 pertencendo circunferncia de um desses discos que no N


(IDIA DA SOLUO) Consideremos o fecho

coberto por nenhum outro disco. convexo H desse conjunto de discos. Um arco que esteja na borda do fecho convexo no pode ser coberto por outro disco. Mostre que a juno de todos os arco no bordo de H um crculo de raio unitrio. Como este crculo tem permetro 2 e no mximo juntamos N arcos, pelo menos um dos arcos da juno maior ou igual a

2 . N

EUREKA! N25, 2007

32

Sociedade Brasileira de Matemtica

PROBLEMA 6

(OBM 96) Existe um conjunto A de n pontos ( n 3 ) em um plano tal que: i) A no contm trs pontos colineares; ii) dados quaisquer trs pontos pertencentes a A, o centro da circunferncia que contm estes pontos tambm pertence a A? Os prximos dois problemas so de IMO e podem ser resolvidos usando s fecho convexo (na realidade, muita raa tambm, que algo imprescindvel em qualquer problema, principalmente de IMO).
PROBLEMA 7

(IMO 99/1) Determine todos os conjuntos finitos S de pontos do plano com pelo menos trs elementos que satisfazem a seguinte condio: Para quaisquer dois pontos distintos A e B de S, a mediatriz do segmento AB um eixo de simetria de S. (Veja a soluo desse problema por Fabrcio Siqueira Benevides na Eureka! N.6)
PROBLEMA 8

(IMO 95/3) Determine todos os inteiros n > 3 para os quais existem n pontos A1 , A2 , , An no plano, e nmeros reais r1 , r2 , rn satisfazendo as condies:

no h trs pontos Ai , A j , Ak colineares; para cada tripla i ,j, k (1 i < j < k n ) o tringulo Ai A j Ak tem rea ri + rj + rk .
SOLUO:

Vamos fazer o caso n 5 . Considere, dentre os n pontos, cinco pontos A1 , A2 , A3 , A4 , A5 e seu fecho convexo . Temos trs casos: 1 Caso: O Fecho Convexo um tringulo. Podemos assumir que tal fecho o tringulo A1 A2 A3 , A4 e A5 esto no interior de A1 A2 A3 , com A5 fora de A1 A2 A4 e A4 fora de A1 A2 A4 (faa uma figura). Podemos supor que os tringulos A1 A2 A4 e A2 A3 A5 tm interiores disjuntos. Seguindo nossa notao para reas, temos: r1 + r2 + r3 = [ A1 A2 A3 ] > [ A1 A2 A4 ] + [ A2 A3 A5 ] = r1 + r2 + r4 + r2 + r3 + r5 , donde 0 > r4 + r2 + r5 = [ A2 A4 A5 ], absurdo! 2 Caso: O Fecho Convexo um quadriltero Suponha A5 no interior do fecho convexo A1 A2 A3 A4 . Note que

[ A1 A2 A3 A4 ] = [ A1 A2 A3 ] + [ A1 A3 A4 ] = [ A1 A2 A4 ] + [ A2 A3 A4 ]
EUREKA! N25, 2007

33

Sociedade Brasileira de Matemtica

e portanto,

(r1 + r2 + r3 ) + (r3 + r4 + r1 ) = (r1 + r2 + r4 ) + (r2 + r4 + r3 ) r1 + r3 = r2 + r4 .


Logo, 2[ A1 A2 A3 A4 ] = 3(r1 + r2 + r3 + r4 ) . Tambm,

[ A1 A2 A3 A4 ] = [ A1 A2 A5 ] + [ A2 A3 A5 ] + [ A3 A4 A5 ] + [ A4 A1 A5 ]
Logo, temos r5 = (r1 + r2 + r3 + r4 ) 8 = [ A1 A2 A3 A4 ] 12 < 0 . Agora, observe que como A1 , A3 , A5 no so colineares, podemos supor um dos lados de

A1 A3 A5 < 180 . Ento, um dos quadrilteros A1 A5 A3 A4 , A1 A5 A3 A2 convexo. Digamos, A1 A5 A3 A4 convexo. Ento, temos r1 + r3 = r4 + r5 e portanto, ficamos com r2 = r5 . Analogamente, usando que A2 , A4 , A5 no so
colineares, temos r5 = r1 ou r3 . Assim, trs dos nmeros r1 , r2 , r3, r4 , r5 so negativos, obtendo uma rea negativa. Absurdo! 3 Caso: O Fecho convexo um pentgono Suponha que r1 seja o menor deles. Traando uma paralela l por A1 reta A3 A4 . Como [ A1 A3 A4 ] = r1 + r3 + r4 r2 + r3 + r4 = [ A2 A3 A4 ] , A2 pertence a l ou ao semiplano definido por l oposto ao A3 A4 e, analogamente A5 . Como A1 , A2 , A5 no podem estar todos em l , temos A2 A1 A5 > 180 , absurdo! . Logo, n 4 . Um exemplo para n = 4 um quadrado A1 A2 A3 A4 de lado 1 com

r1 = r2 = r3 = r4 = 1 6 .
Finalizamos essa parte com dois problemas bonitinhos.
PROBLEMA 9

(USAMO 2005) Seja n um inteiro positivo maior que 1. Suponha que so dados 2n pontos no plano, no havendo trs colineares. Suponha que n dos 2n so pintados de azul e os outros n de vermelho. Uma reta no plano dita balanceada se passa por um ponto azul e um ponto vermelho, e o nmero de pontos azuis em cada um de seus lados igual ao nmero de pontos vermelhos. Prove que existem pelo menos duas retas balanceadas. DICA: Prove que cada ponto do fecho convexo dos pontos est em pelo menos uma reta balanceada.
PROBLEMA 10

(Kmal 2002) Dado um conjunto qualquer de pontos no plano, no contendo trs colineares, prove que possvel colorir os pontos com duas cores (azul e
EUREKA! N25, 2007

34

Sociedade Brasileira de Matemtica

vermelho) tal que todo semiplano contendo pelo menos trs pontos do conjunto contenha pelo menos um ponto de cada cor.
2. Princpio das Casas dos Pombos

O princpio da casas dos Pombos, PCP, importante e deve ser lembrado sempre. Ele usado para provar existncias (...se n + 1 pombos esto em n casas, existe pelo menos uma casa contendo pelo menos dois pombos...). Nossos dois primeiros problemas dessa sesso so apenas verses dificultadas daquele exerccio clssico do PCP. : dados cincos pontos num quadrado unitrio, existem dois cuja distncia entre eles menor que
PROBLEMA 11

(Japo 97) Prove que entre quaisquer dez pontos no interior de um crculo de dimetro 5, existem dois cuja distncia entre eles menor que 2.
PROBLEMA 12

(Coria 97) Prove que entre quaisquer quatro pontos no interior de um crculo unitrio, existem dois deles cuja distncia menor que
PROBLEMA 13

2.

(Rioplatense 2002) Daniel escolhe um inteiro positivo n e diz a Ana. Com esta informao, Ana escolhe um inteiro k e diz a Daniel. Daniel traa ento n circunferncias em um papel e escolhe k pontos distintos com a condio de que cada um deles pertena a alguma das circunferncias que traou. Em seguida, apaga as circunferncias que traou, sobrando visveis apenas os k pontos que marcou. A partir desses pontos, Ana deve reconstruir pelo menos uma das circunferncias que Daniel traou. Determinar qual o menor valor de k que permite Ana alcanar seu objetivo independente de como Daniel escolha as n circunferncias e os k pontos.
SOLUO:

O valor mnimo k = 2n 2 + 1 . 1 Passo: k = 2n 2 + 1 suficiente. Se so dados 2n 2 + 1 pontos marcados por Daniel, como estes pontos so distribudos em n circunferncias, pelo Princpio das Casas dos Pombos, pelo menos 2n+1 deles esto em uma mesma circunferncia traada por Daniel. Ento, se Ana traa todas as circunferncias determinadas por estes 2n 2 + 1 , haver uma
EUREKA! N25, 2007

35

Sociedade Brasileira de Matemtica

delas, digamos , com pelo menos 2n+1 dos pontos. Como estes 2n+1 pontos provm das n circunferncias de Daniel, trs deles esto numa mesma circunferncia traada por ele, digamos . Logo, e tm pelo menos trs pontos em comum, e portanto, so a mesma circunferncia (abusando da notao: = ). Assim, Ana consegue determinar umas das circunferncias traadas por Daniel. 2 Passo: Se k < 2n 2 + 1 , Daniel pode traar circunferncias e escolher k pontos de modo a tornar impossvel para Ana determinar tais circunferncias. Basta considerar k = 2n 2 :

Traamos n circunferncias concntricas 1 , 2 , , n e outras n circunferncias

1 , 2

modo que i corta j em dois pontos


n
2

 , ,

duas a duas disjuntas, de

distintos, para i, j = 1,2,...,n. H exatamente 2n 2 pontos de interseco. Se Daniel marca estes pontos e apagas suas circunferncias, Ana no conseguir reconstruir com certeza nenhuma das circunferncias, pois Daniel pode ter traado inicialmente tanto 1 , 2 , , n quanto 1 , 2 ,

PROBLEMA 14

,

(Rioplatense 1999) Dois jogadores A e B disputam o seguinte jogo: A escolhe um ponto de coordenadas inteiras do plano e o pinta de verde; em seguida B escolhe 10 pontos de coordenadas inteiras, ainda no coloridos e os pinta de amarelo. O jogo continua assim com as mesmas regras: A e B escolhem um e dez pontos ainda no coloridos e os pintam de verde e amarelo, respectivamente. (a) O objetivo de A obter 1112 pontos verdes que sejam as intersees de 111 retas horizontais e 111 retas verticais (i.e., paralelas aos eixos de coordenadas). O objetivo de B impedir-lhe. Determine qual dos jogadores tem uma estratgia vencedora que lhe assegura seu objetivo. (b) O objetivo de A obter quatro pontos verdes que sejam vrtices de um quadrado de lados paralelos aos eixos coordenados. O objetivo de B impedir-lhe. Determine qual dos jogadores tem uma estratgia vencedora que lhe assegura seu objetivo.
EUREKA! N25, 2007

36

Sociedade Brasileira de Matemtica

3. Idias Extremais

Na matemtica em geral, problemas de existncia so muito comuns e importantes. So aqueles problemas que nos pedem para provar que a existncia de alguma coisa. Na seo anterior, no explicitamente, nos deparamos com problemas desse tipo. E no foi para vender o artigo que iniciamos ambas as sees falando da importncia dessas idias, mas pelo o fato de que o PCP e o Princpio Extremal juntos so as ferramentas mais indispensveis para o ataque desses problemas. Mas afinal, que Princpio Extremal esse? Digamos que temos um problema onde nos pedido para provar a existncia de um elemento satisfazendo uma certa propriedade P. Ento, ns escolhemos um elemento que satisfaz maximalmente ou minimalmente, ou seja, extremalmente (ser que acabamos de inventar essas palavras?) uma outra propriedade Q, no acidentalmente ligada com a desejada propriedade P. O que ser que isso nos d? Vejamos alguns problemas. (Austrlia 91) So dados n 3 pontos no plano tais que a rea de um tringulo formado por quaisquer trs deles no mximo 1. Prove que os n pontos esto em um tringulo de rea no mximo 4.
SOLUO: PROBLEMA 15

Sejam P1 , P2 , , Pn os n pontos. Dentre os tringulos considerados, seja ABC o de maior rea (o cara com a propriedade Q). Considere por A uma reta a paralela a BC. Sendo assim, qualquer outro ponto Pi deve estar no mesmo semiplano de B e C definido por A, pois do contrrio teramos Z um absurdo [ PBC ] > [ ABC ] (aqui [X], denota a rea de X) . Analogamente, considerando as retas b e c por B e C B A paralelas a AC e AB, respectivamente, conclumos que todos os pontos P devem b estar no tringulo XYZ (acompanhe a figura ao lado). Como, c [ XYZ ] = 4[ ABC ] < 4 1 = 4 , o resultado C Y X segue. (isto , XYZ satisfaz a propriedade P).

EUREKA! N25, 2007

37

Sociedade Brasileira de Matemtica

PROBLEMA 16

(Putnam 1979) Sejam 2n pontos no plano escolhidos de modo que quaisquer 3 no so colineares, n deles so pintados de vermelho e n deles so pintados de azul. Prove que possvel parear os pontos usando segmentos ligando cada ponto vermelho a exatamente um ponto azul de modo que esses segmentos no se cortem.
SOLUO:

Existem n 2 maneiras de parear esses pontos. claro que alguns desses pareamentos no cumprem a condio do enunciado. Olhemos em cada pareamento a soma dos seus segmentos. Escolha o pareamento que tem soma mnima. Suponha que nele existem dois segmentos AB e CD que se cortam (com A e C vermelhos) Pela desigualdade triangular temos:

C O

AO + OD > CD AB + CD > AD + CB OB + OC > CB Logo se trocarmos AB e CD por AD e CB diminuiremos nossa soma. Assim neste pareamento no temos dois segmentos que se cortam.

D B
PROBLEMA 17

(Teorema de Sylvester) Um conjunto S de pontos no plano tem a seguinte propriedade: qualquer reta passando por 2 pontos passa tambm por um terceiro. Mostre que todos os pontos esto sobre uma reta.
SOLUO:

Considere o conjunto L das retas que passam por dois pontos de S. Cada ponto de S tem uma distncia associada a cada reta de L. Como L e S so conjuntos finitos ento temos um nmero finito distncias. Considere o par (l , s ) do ponto s S e l L com a menor distncia no nula associada. Como l passa por dois pontos de S ento dever passar por um terceiro. Pelo menos dois pontos de S , digamos A e B, devero estar em um mesmo lado de l determinado por P (p da perpendicular de s at l ).

EUREKA! N25, 2007

38

Sociedade Brasileira de Matemtica

Suponhamos que A esteja entre B e P. Seja m a reta que passa por B e s ento: distncia(A,m) distncia(P,m) < < distncia(s,l) Absurdo!

Assim todas as distncias associadas tm que ser zero! Todos os pontos so colineares! A seguir, veja como usar o Teorema de Sylvester. So dados ( N 3) pontos no plano, nem todos colineares. Mostre que so necessrios pelo menos n retas para unir todos os possveis pares de pontos. Vamos tentar usar induo. Se N = 3 os trs pontos formaro um tringulo. As retas suportes dos trs lados desse tringulo satisfazem nossa afirmao. Suponha que a afirmao seja vlida para N = k . Considere um conjunto T de N = k + 1 pontos. Como nem todos esses pontos esto sobre uma mesma reta decorre do teorema de Sylvester que existe uma reta que passa por apenas dois pontos (A e B) do conjunto. Pelo menos um dos conjuntos T \ { A} ou T \ {B} no poder ter todos os seus k pontos colineares. Ento pela hiptese teremos pelo menos k retas, mas a reta AB no foi contada, assim a afirmao tambm verdadeira para N = k + 1 .
PROBLEMA 19 SOLUO: PROBLEMA 18

Dado um conjunto finito S de pontos no plano onde no existem quatro sobre um mesmo crculo e nem todos esto sobre uma mesma reta. Mostre que existe um crculo que passa por trs desses pontos e no contm nenhum ponto de S em seu interior.
PROBLEMA 20

(Ibero 93) Prove que para qualquer polgono convexo de rea 1, existe um paralelogramo de rea dois que o contm.

EUREKA! N25, 2007

39

Sociedade Brasileira de Matemtica

PROBLEMA 21

(OBM 94) Considere todos os crculos cujas circunferncias passam por trs vrtices consecutivos de um polgono convexo. Prove que um destes crculos contm todo o polgono.
PROBLEMA 22

(Rioplatense 97) Agustina e Santiago jogam o seguinte jogo sobre uma folha retangular: Agustina diz um nmero n. Santiago, ento marca n pontos sobre a folha . Em seguida, Agustina escolhe alguns dos pontos marcados por Santiago. Santiago ganha o jogo se consegue desenhar um retngulo com lados paralelos aos da folha, que contenha todos os escolhidos por Agustina e nenhum dos restantes. Do contrrio, Agustina ganha. Qual o menor nmero que deve escolher Agustina para assegurar-se da vitria, independente como jogue Santiago?
PROBLEMA 23

(Rssia 2000) So dados 2n+1 segmentos em uma linha reta. Cada segmento intersecta pelo menos n outros. Prove que um desses segmentos intersecta todos os outros.
PROBLEMA 24

(Japo 2002) dado um conjunto S de 2002 pontos no plano xy, no havendo dois deles com a mesma abscissa x ou ordenada y. Para quaisquer dois desses pontos P e Q , considere o retngulo cuja diagonal PQ e cujos lados so paralelos aos eixos. Denotemos por WPQ o nmero de pontos de S no interior desse retngulo, sem contar com P e Q. Determine o maior valor N possvel que satisfaz: no importa como os pontos de S esto arranjados, existe pelo menos um par P e Q deles com WPQ N .
PROBLEMA 25

Dados 2n + 2 pontos no plano, no havendo trs colineares, prove que existem dois deles que determinam uma reta que, dos 2n pontos restantes, separa n em um semi-plano e os outros n no outro semi-plano.
PROBLEMA 26

(Banco IMO 93) Dados 2n+3 pontos num plano, no havendo trs colineares nem quatro concclicos, prove que podemos escolher trs deles de modo o crculo passando por estes tem n dos pontos restantes no seu interior e n no exterior.

EUREKA! N25, 2007

40

Sociedade Brasileira de Matemtica

SOLUO: Basta considerar a figura abaixo. Deixamos os detalhes para o leitor.

A2 n +1
A2 n+ 2 = P A2 n +3 = Q

A1

An +1

PROBLEMA 27

So dados n pontos num plano. Em cada ponto mdio de um segmento ligando dois desses pontos, colocamos um marcador. Prove que pelo menos 2n 3 marcadores so utilizados.
4. Problemas de Coberturas

Nos problemas sobre pontos at agora, ficou claro que um pouco de geometria (sinttica, analtica, trigonomtrica ou utilizando o plano complexo) pode ser til. Finalizaremos esse artigo com uma seo falando um pouco disso, em particular, fazendo coberturas com crculos.
PROBLEMA 28

Seja C um crculo de raio 16 e A um anel tendo raio interno 2 e raio externo 3. Agora suponha que um conjunto S de 650 pontos selecionado dentro de C. Prove que, no importa como os pontos de S so selecionados dentro de C, o anel A pode ser colocado de modo a cobrir pelo menos 10 pontos de S.
SOLUO:

Queremos mostrar que existe um ponto X no plano que possui uma distncia maior que 2 e menor que 3 pelo menos 10 pontos de S. Sobre cada ponto de S coloque um anel A. Basta mostrarmos que existe um ponto X que est no interior de pelo menos 10 desses anis. As intersees desses anis produzem pequenas regies (veja a figura).

EUREKA! N25, 2007

41

Sociedade Brasileira de Matemtica

Veja que existem trs pequenas regies que esto em dois anis e uma que est em trs. Somando as reas dos trs anis contaremos trs regies duas vezes e uma trs vezes. Somando a rea de cada anel temos 650.(9 4 ) = 3250 .Aumentando o raio do crculo C para 19 poderemos cobrir todos esses anis. Se cada pequena regio foi contada no mximo 9 vezes contaremos no mximo 9 vezes a rea desse novo crculo , ou seja, 9.19 2 = 3249 < 3250 . Assim existir uma pequena regio contida em pelo menos 10 anis. Basta escolhermos um ponto X dessa regio.
PROBLEMA 29

(Teste de Seleo da Romnia para a IMO 1978) M um conjunto de 3n pontos no plano tal que a maior distncia entre quaisquer dois desses pontos 1 unidade. Prove que: a. Para quaisquer 4 pontos de M, a distncia entre algum par de pontos pelo menos

1 2

b. Algum crculo de raio

3 cobre todo o conjunto M. 2 c. Existe algum par entre os 3n pontos de M cuja distncia entre eles no mximo 4 (3 n 3 )

SOLUO:

a. Vamos tentar arranjar um tringulo no acutngulo em M. Considere o fecho convexo de quatro pontos de M. Podemos ter um quadriltero (degenerado quando trs pontos forem colineares) ou um tringulo com um ponto de M em seu interior. No caso do quadriltero como pelo menos um dos quatro ngulos internos 90 , basta escolhermos o vrtice com este ngulo e os adjacentes a ele. No caso
EUREKA! N25, 2007

42

Sociedade Brasileira de Matemtica

do tringulo vamos olhar para o ponto no interior. Esse ponto olha para os trs lados do tringulo com ngulos que somados resultam em 360 , . Pelo menos um deles 120 , . Seja XYZ um tringulo com XYZ 90 , . Pela lei dos cossenos temos: y 2 = x 2 + z 2 2 xz cos XYZ x 2 + z 2 . Como y 1 x ou z

1 2

b. Seja r = AB a maior distncia entre dois pontos de M. Tracemos crculos de raio r centrados em A e B. M dever estar contido em cada um desse crculos. Ento M dever estar contido na regio de interseo entre eles. Tracemos um 3r 3 crculo de raio centrado no ponto mdio C de AB. Veja que este novo 2 2 crculo cobre a regio de interseo. c. Vamos usar a mesma idia do problema dos anis. Se dois ponto de M esto em um crculo Y de raio r ento a distncia entre eles no pode ser maior ou igual a 2r . Ento nosso objetivo mostrar que existem dois pontos de M

A C

dentro de um crculo de raio

2 (3 n 3 )

M X
3 que cobre M. Centrado em cada ponto de M 2 tracemos um crculo de raio r . Suponha que Z um ponto na interseo de dois desses crculos. Ento o crculo de centro Z e raio r cobre dois pontos de M(os centros dos crculos que cobriam Z). Como mostrar pelo menos dois desses 2 crculos que traamos iro se intersectar para r = ? Vamos aumentar (3 n 3 ) Seja C um crculo de raio o raio de C e obter um crculo D de raio 3 + r com mesmo centro. Todos esses 2 crculos estaro contidos em D. Se a rea de D for menor que a soma das reas de cada crculo com certeza pelo menos dois deles tero interseo. Mas isso
3 acontece se 3n r 2 < + r . Agora basta fazermos o estudo do sinal. Como 2
EUREKA! N25, 2007
2

43

Sociedade Brasileira de Matemtica

a
r>

maior

raiz

3+3 n e 3n 1 > 0 2(3n 1)

se

3 +3 n 3 . (Veja que podemos melhorar um pouco OK! r > 2(3n 1) 2(3 n 3) a cota do problema, pois 2 > 3 ). 2
PROBLEMA 30

(Ibero 97) Seja P = {P 1 , P2 ,..., P 1997 } um conjunto de 1997 pontos no interior de um crculo de raio 1, com P1 sendo o centro do crculo. Para k = 1,2,...,1997 , seja x k a distncia de Pk ao ponto de P mais prximo de Pk . Mostre que:

x1 + x 2 + ... + x1997 9 .
2 2 2

SOLUO:

Note que x k 1 , para todo k. Para cada ponto

Pk ,

considere

uma

circunferncia k de centro Pk e raio

xk . 2

P1 Pn

Sendo assim, todos essas circunferncias se tocam em no mximo 1 ponto e esto no interior de uma circunferncia de centro P1 e raio 3/2. Logo, [1 ] + [2 ] +

+[ ] (32)
k

x x 1 2 + 2 2 +
PROBLEMA 31

 + x

donde:
2 n

3 2 x2 + x2 + 1 2 2 2

( )

+ x

2 n

9.

(IMO 89) So dados n e k inteiros positivos e um conjunto S de n pontos no plano tais que (i) no h trs pontos em S colineares, (ii) Para qualquer ponto P de S existem pelo menos k pontos de S eqidistantes de P. Prove que k <

1 + 2n . 2

EUREKA! N25, 2007

44

Sociedade Brasileira de Matemtica

PROBLEMA 32

(Ibero 98) Encontre o maior inteiro n para o qual existem pontos P1 , P2 ,..., Pn no plano e nmeros reais r1 , r2 ,..., rn tais que a distncia entre Pi e Pj ri + r j .

Referncias Bibliogrficas: [1] Revista Eureka! N6. [2] Ross Honsberger, Mathematical Gems Vol.I, The Dolciani Mathematical Expositions, MAA. [3] Andreescu, Feng, Mathematical Olympiads: Olympiad problems from around the world, 1999-2000, MAA 2000. [4] Marcin Kuczma , International Mathematical Olympiads, 1986-1999, MAA 2003. [5] www.mathlinks.ro

EUREKA! N25, 2007

45

Sociedade Brasileira de Matemtica

POLINMIOS SIMTRICOS
Carlos A. Gomes, UFRN, Natal RN.
Nvel Avanado

Uma ferramenta bastante til na resoluo de problemas algbricos de fatorao, na resoluo de sistemas de equaes no lineares, na resoluo de algumas equaes irracionais so as funes polinomiais simtricas, que apesar de seu grande poder algbrico so pouco divulgadas entre os nossos alunos. A finalidade deste breve artigo exibir de modo sucinto como estas ferramentas podem ser teis na resoluo de alguns problemas olmpicos.
I. Polinmios Simtricos

Um polinmio f, a duas variveis x, y, dito simtrico quando f(x, y) = f (y, x) para todos os valores x, y. a) 1 = x + y e 2 = x y, so evidentemente polinmios simtricos (chamados polinmios simtricos elementares). b) Os polinmios da forma Sn = xn + yn, com n  tambm so simtricos. Um fato importante a ser observado que um polinmio simtrico f(x, y) pode ser representado como um polinmio em funo de 1 e 2. Vejamos: Se Sn = xn + yn, n , (n 2), ento: Sn = xn + yn = (x + y) (xn1 + yn1) xy (xn 2 + yn 2) = 1 Sn 1 2 Sn 2 (n 2) Mas, S0 = x0 + y0 = 1 + 1 = 2 S1 = x1 + y1 = x + y = 1 Assim temos que: S0 = 2 S1 = 1 S2 = 1 S1 2 S0 = 1 1 2 2 = 12 22 S3 = 1 S2 2 S1 = 1 (12 22) 2 1 = 13 31 2 E da usando a lei de recorrncia Sn = 1 Sn 1 2 Sn 2 (n 2) podemos determinar Sn em funo de 1 e 2 para qualquer nmero natural n.
EUREKA! N25, 2007

Exemplos:

46

Sociedade Brasileira de Matemtica

Agora para garantirmos a afirmao anterior que todo polinmio simtrico f(x, y) pode ser representado como um polinmio em 1 e 2 observemos o seguinte fato: Num polinmio simtrico f(x, y) para os termos da forma a . xK . yK no temos nenhum problema pois a xK yK = a(x y)K = a 2K. Agora com os termos da forma b xi yK, com i < k devemos observar o seguinte fato: Como, por hiptese, f(x, y) simtrico se b xi yk, com i < k estiver presente em f(x, y) temos que b xk yi tambm deve estar presente em f(x, y), visto que deve ser satisfeita a condio f(x, y) = f(y, x). Assim se agruparmos os termos b xi yk + b xk yi (i < k) temos que: b xi yk + b xk yi = b xi yi (xk - i + yk - i) = b 2i Sk i , mas como j mostramos anteriormente Sk i pode ser escrito como um polinmio em 1 e 2, pois k i , visto que i < k.
II. Exemplos Resolvidos 01. (Funes simtricas elementares a 3 variveis)

Definido: 1 = x + y + z 2 = xy + xz + yz 3 = x y z Sn = xn + yn + zn, com n

 (n

2)

Mostre que: a) Sn = 1 Sn 1 2 Sn 2 + 3 Sn 3 (n 3 , com n b) S3 = 13 312 + 33

)

Resoluo: Observe inicialmente que: xn + yn + zn = (x + y + z) (xn 1 + yn 1 + zn 1) (xy + xz + yz) (xn 2 + yn 2 + zn 2) + xyz (xn 3 + yn 3 + zn 3) e da temos que: Sn = 1 Sn 1 2 Sn 2 + 3 Sn 3 (n 3, com n Agora temos que:
EUREKA! N25, 2007

)

47

Sociedade Brasileira de Matemtica

S0 = x0 + y0 + z0 = 1 + 1 + 1 = 3 S1 = x + y + z = 1 S2 = x2 + y2 + z2 = (x + y + z)2 2 (xy + xz + yz) = 12 22 Agora fazendo n = 3 temos na lei de recorrncia Sn = 1 Sn 1 2 Sn 2 + 3 Sn 3 temos que: S3 = 1 S2 2 S1 + 3 S0 = 1 (12 22) 2 1 + 3 3 S3 = 13 31 2 + 33
02. a) Fatore x3 + y3 + z3 3xyz Resoluo:

Essa velha e manjada questo continua ainda hoje pegando alguns bons professores e alunos. A sua soluo pelos mtodos tradicionais envolve uma boa dose de ateno e de pacincia para aplicar velhos truques de fatorao, por outro lado ela imediata usando os polinmios simtricos. Vejamos: x3 + y3 + z3 3xyz = S3 3 3 Mas de acordo com a questo anterior S3 = 13 312 + 33 e da temos que S3 33 = 13 312. Assim: x3 + y3 +z3 3xyz = S3 33 = =13 312 = =1 (12 32) = = [(x + y + z)2 3 (xy + xz + yz)] = (x + y + z) (x2 + y2 + z2 xy xz yz)
Obs. (para os mais curiosos): Na RPM 41, pg.38 existe uma bela resoluo desse problema usando um determinante.

b) Usando a fatorao obtida em (a), verifique a famosa desigualdade das mdias a+b+c e a igualdade aritmtica e geomtrica. Se a, b, c #+ ento 3 abc 3 ocorre, se, e somente se, a = b = c. De fato, em (a) verificamos que x3 + y3 + z3 3xyz = (x + y + z) (x2 + y2 + z2 xy xz yz). Vamos mostrar inicialmente que se x, y, z so nmeros reais positivos ento:

EUREKA! N25, 2007

48

Sociedade Brasileira de Matemtica

(x + y + z) (x2 + y2 + z2 xy xy yz) 0 De fato, x2 + y2 + z2 xy xz + yz = 1 (2x2 + 2y2 + 2z2 2xy 2xz 2zy) 2 1 = (x2 2xy + y2 + x2 2xz + z2 + y2 2yz + z2) 2 1 = [(x y)2 + (x z)2 + (y z)2] 0 (Soma de 2

quadrados) Ora, como estamos supondo x, y, z reais positivos temos que x + y + z 0 e da (x + y + z) (x2 + y2 + z2 xy xz yz) 0 (pois o produto de fatores 0). Assim temos que: x3 + y3 + z3 3xyz = (x + y + z) (x2 + y2 + z2 xy xz yz) 0 e da 3xyz x3 + y3 + z3 xyz fazendo x3 = a, y3 = b e z3 = c temos que:
3

x3 + y 3 + z 3 3

a .3 b .3 c abc

a+b+c 3

e da
3

a+b+c 3

Com a igualdade ocorrendo se e somente se a = b = c, pois em (x + y + z) (x2 + y2 + z2 xy xz yz) 0 a igualdade ocorre apenas quando x = y = z, visto que x + y + z > 0, uma vez que x, y, z so nmeros reais positivos e alm disso, 1 (x2 + y2 + z2 xy xz yz) = [(x y)2 + (x z)2 + (y z)2] = 0 x = y = z. 2
03. Fatore (x + y + z)3 (x3 + y3 + z3) Resoluo:

(x + y + z)3 (x3 + y3 + z3) = 13 S3

EUREKA! N25, 2007

49

Sociedade Brasileira de Matemtica

Mas, no exemplo anterior vimos que S3 = 13 312 + 33 e da (x + y + z)3 (x3 + y3 + z3) = 13 (13 312 + 33) = 3 (12 3) = 3 [(x + y + z) (xy + xz + yz) xyz] = 3 (x2y + x2z + xyz + xy2 + xyz + y2z + xyz + xz2 + yz2 xyz) = 3 [xy(x + y) + xz(x + y) + yz(y + z) + xz(y + z)] = 3 [(x + y)(xy + xz) + (y + z)(yz + xz)] = 3 [(x + y) . x(y + z) + (y + z) . z(x + y)] = 3 (x + y)(y + z)(x + z)
04. Se x1 e x2 so as razes da equao x2 6x + 1 = 0 determine o valor de x15 +

x25.

Fazendo Sn = x1n + x2n, n  , queremos determinar S5 = x15 + x25 Temos que: 1 = x1 + x2 = 6 2 = x1 x2 = 1 S0 = x10 + x20 = 1 + 1 = 2 S1 = x1 + x2 = 6 Sn = 1 Sn 1 2 Sn 2 = 6 Sn 1 Sn 2 e da S2 = 6 S1 S0 = 6 6 2 = 34 S3 = 6 S2 S1 = 6 34 6 = 198 S4 = 6 S3 S2 = 6 198 34 = 1154 S5 = 6 S4 S3 = 6 1154 198 = 6726 Assim x15 + x25 = 6726
05. Determine todas as solues reais do sistema

Resoluo:

x + y + z =1 3 3 3 4 4 4 x + y + z + xyz = x + y + z + 1 De acordo com o sistema acima temos que:

EUREKA! N25, 2007

50

Sociedade Brasileira de Matemtica

1 = 1 , onde S n = x n + y n + z n , n < 1 + = + S S 3 3 4 Mas, S3 = 13 321 + 33 e S4 = 14 4122 + 222 + 41. 3 (verifique isto!) e da S3 + 3 = S4 + 1 13 312 + 33 + 3 = 14 412 2 + 222 + 413 + 1 Como 1 = 1 temos que: 1 3 2 + 43 = 1 42 + 222 + 43 + 1 222 2 + 1 = 0 Como, = (1)2 4 2 1 = 7 < 0, conclumos que existem razes reais. Uma outra aplicao interessante dos polinmios simtricos pode ser encontrada na resoluo de algumas equaes irracionais. Vejamos:
06. Determine todas as razes reais da equao abaixo:
4

272 x + 4 x = 6

Resoluo:

Fazendo

x=y e

272 x = z temos que

y+z=6 x = y4 e 272 x = z4 4 4 y + z = 272 e agora lembrando que: 1 = y + z e 2 = y z e Sn = yn + zn, com n 1 = 6 1 = 6 4 2 2 S 4 = 272 1 41 .2 + 2 2 = 272

Logo, 64 4 62 2 + 222 = 272 22 722 + 512 = 0 2 = 64 ou 2 = 8 y + z = 6 Assim, se 2 = 64 No existem solues reais. y z = 64 y + z = 6 Por outro lado, se 2 = 8 y = 2 e z = 4 ou z = 2 e y = 4 y z = 8 Assim conclumos que: y = 2 x = 16 y = 4 x = 256 Logo as razes reais da equao so 16 e 256.
EUREKA! N25, 2007

51

Sociedade Brasileira de Matemtica

III. Problemas: 01. Se , e so as razes da equao x3 + 3x2 7x + 1 = 0. Determine o valor

de 3 + 3 + 3 + 4 + 4 + 4 x+ y =a 02. Mostre que se o sistema x 2 + y 2 = b tem soluo, ento a3 3ab + 2c = 0 x3 + y 3 = c 03. x, y, z so nmeros reais tais que x + y + z = 5 e yz + zx + xy = 3. Verifique que 13 1 z . 3
04. Se x + y + z = 0, verifique que, para n = 0, 1, 2, ... vale a relao:

xn + 3 + yn + 3 + zn + 3 = xyz(xn + yn + zn) +
05. Determine as razes reais da equao 06. Verifique que:
5

1 2 (x + y2 + z2)(xn + 1 + yn + 1 + zn + 1) 2

33 x + 5 x = 3 .

(x + y + z)3 (y + z x)3 (x + z y)3 (x + y z)3 = 24xyz.


07. Dados a, b e c nmeros reais positivos tais que

log a b + log b c + log c a = 0 ,determine o valor de (log a b ) + (log b c ) + (log c a ) .


3 3 3

08. Se , e so nmeros complexos tais que + + = 1, 2 + 2 + 2 = 3 e

3 + 3 + 3 = 7 , determine o valor de 21 + 21 + 21 .

Referncias: [1] Barbeau, E. J., Polynomials Problems books in Mathematics Springer Verlag. [2] Engel, Arthur , Problem-Solving Strategies Springer Verlag. [3] www.obm.org.br [4] Mathematical Excalibur.
EUREKA! N25, 2007

52

Sociedade Brasileira de Matemtica

OLIMPADAS AO REDOR DO MUNDO


A partir desse nmero a EUREKA! volta a apresentar problemas de olimpadas de vrios pases do mundo. Como antes, esperamos contar com a colaborao dos leitores para a apresentao das solues dos problemas propostos. Aos leitores que se interessarem pela soluo de algum problema particular, pedimos contatar OBM, atravs de carta ou e-mail. Repassada a ns a mensagem, teremos o maior prazer de apresentar as solues solicitadas no nmero subsequente da EUREKA! Bom divertimento! Antonio Caminha Antonio Luiz Santos Bruno Holanda Samuel Barbosa
211. (Baltic Way 2004) Uma seqncia a1, a2, ... de nmeros reais no-negativos satisfaz, para n = 1, 2, ..., as seguintes condies:

(a) an + a2n 3n. (b) a n +1 + n 2 a n (n + 1) . (i) Prove que an n para todo n = 1, 2, ... (ii) D exemplo de uma tal seqncia.
212. (Baltic Way 2004) Seja P um polinmio com coeficientes no-negativos. Prove

que se P(1/x)P(x) 1 para x = 1, ento tal desigualdade se verifica para todo real positivo x.
213. (Baltic Way 2004) Ache todos os conjuntos X, consistindo de ao menos dois

inteiros positivos, tais que para todos m, n X, com n > m, exista um elemento k de X tal que n = mk2.
214. (Rssia 2004) So dados um natural n > 3 e reais positivos x1, x2, ..., xn cujo produto 1. Prove a desigualdade

1 1 1 + + ... + > 1. 1 + x1 + x1 x 2 1 + x 2 + x 2 x3 1 + x n + x n x1
EUREKA! N25, 2007

53

Sociedade Brasileira de Matemtica

215. (Rssia 2004) Uma seqncia a1, a2, ... de nmeros racionais no-negativos tal que am + an = amn para todos m, n naturais. Prove que os termos da seqncia no podem ser todos distintos. 216. (Rssia 2004) Sejam IA e IB os centros das circunferncias ex-inscritas a um tringulo ABC que tangenciam os lados BC e AC, respectivamente. Seja ainda P um ponto sobre a circunferncia circunscrita de ABC. Prove que o ponto mdio do segmento que une os circuncentros dos tringulos IACP e IBCP coincide com o circuncentro de ABC. 217. (Putnam 2005) Calcule o valor da integral

1 ln( x + 1)

x2 +1 a x

dx .

218. (Putnam 2005) Encontre todas as funes diferenciveis f: (0, +) (0, +)

para as quais exista um real positivo a tal que f ' = positivo x.

x para todo real f ( x)

219. (Putnam 2005) Seja Sn o conjunto de todas as permutaes de 1, 2, ..., n. Para

Sn, seja () = 1 se for uma permutao par e () = 1 se for uma permutao mpar. Denote ainda por v() o nmero de pontos fixos de . Prove que

S n

n ( ) = (1)n +1 n + 1 . v ( ) + 1

220. (Moldvia 2006) Seja a e b os catetos de um tringulo retngulo, c sua hipotenusa e h a altura relativa mesma. Encontre o maior valor possvel de

c+h . a+b

inteiro no-nulo, ns definimos a funo fa: M M tal que fa(x) o resto da diviso de ax por n. Encontre uma condio necessria e suficiente para que fa seja uma bijeo. Se fa for uma bijeo e n for um nmero primo, prove que an(n 1) 1 divisvel por n2.

221. (Moldvia 2006) Seja n > 1 um inteiro positivo e M = {0, 1, 2, ..., n 1}. Para a

EUREKA! N25, 2007

54

Sociedade Brasileira de Matemtica

222. (Moldvia 2006) O quadriltero convexo ABCD inscritvel. As tangentes a sua circunferncia circunscrita em A e C se intersectam em P, tal que P no est sobre a reta BD e PA2 = PBPD. Prove que a reta BD passa pelo ponto mdio do segmento AC. 223. (Bielorssia 2005) Seja H o ponto de interseo das alturas BB1 e CC1 do tringulo acutngulo ABC. Seja uma reta passando por A, tal que AC. Prove que as retas BC, B1C1 e possuem um ponto em comum se e somente se H for o ponto mdio de BB1.

224. (Bielorssia 2005) Ache todas as funes f :   satisfazendo

para todos m, n .

f (m n + f (n)) = f (m) + f (n) ,

225. (Bielorssia 2005) Prove que

2 3 3 1 1 a + b + . b 2 + a + 2a + . 2b + , 4 4 2 2
Para quaisquer reais positivos a e b.
226. (Bulgria 2005) Ache todos os naturais de quatro algarismos m, menores que 2005 e para os quais existe um natural n < m tal que m n possui no mximo 3 divisores e mn seja um quadrado perfeito. 227. (Bulgria 2005) Ivo escreve todos os inteiros de 1 a 100 (inclusive) em cartas e d algumas delas para Iana. Sabe-se que para quaisquer duas destas cartas, uma de Ivo e outra de Iana, a soma dos nmeros no est com Ivo e o produto no est com Iana. Determine o nmero de cartas de Iana sabendo que a carta 13 est com Ivo. 228. (Bulgria 2005) Ache todas as triplas de inteiros positivos (x, y, z) tais que

2005 2005 2005 , + + x+ y y+z z+ x

tambm seja um inteiro.

EUREKA! N25, 2007

55

Sociedade Brasileira de Matemtica

229. (Eslovnia 2005) Encontre todos os nmeros primos p para os quais o nmero

p2 + 11 tem menos que 11 divisores.


230. (Eslovnia 2005) Denote por I o incentro do tringulo ABC. Sabe-se que AC + AI = BC. Encontre a razo entre as medidas do ngulos BAC e CBA.

'4.H 8,- ,

":0           0        8 4 57 248 8808 147,2 4 4 9,;4 4 3434 0 4 /F. 24 57 248 /08.4-07948 50 4 574 094 80;039003 47 -:89 803/4 80:8 /08.4-7 /4708 # . ,7/ ,88 07 070 47/43 0 4389,39 3  ,1434; 70850.9 ;,20390 884 24897, 6:0     0   3 4 8 4 3 20748 /0 $ 075 38 0 J25,708 9, 8 6:0   F .4254894 5,7, 94/4 3 
3
;0 , , :70 ,  5E  6:0 84-70  48 70/: 3/4 5,7, 4 3 2074 /0 3,9:7, 8 2034708 .43 0. /4 3 20748  /0 0 6:0 F 4 20347 3 2074 /0 $ 075 38 6:, 8 3 4 80 8,-0  80 8 4 4: 3 4

$ 075 38   31472,

  '0 , 08

 

 



www.seventeenorbust.com

5,7,

2, 8

3. :8 ;0 84-70 .424 5,79 . 5,7 /4 574 094

EUREKA! N25, 2007

56

Sociedade Brasileira de Matemtica


108)

SOLUES DE PROBLEMAS PROPOSTOS


Publicamos aqui algumas das respostas enviadas por nossos leitores.

Sejam
1 i1 < i2 <...< ik n

A1 , A2 ,..., An

conjuntos

finitos.

Para

1 k n,

seja

Sk =

Ai1 Ai2 ... Aik , a soma dos nmeros de elementos das

intersees de k dos conjuntos Ai . Prove que: a) O nmero de elementos que pertencem a exatamente r dos conjuntos Ai

( 1)
k =r

kr

k S k , para 1 r n. r k 1 S k , para 1 r n. r 1

b) O nmero de elementos que pertencem a pelo menos r dos conjuntos Ai

( 1)
k =r

k r

SOLUO ADAPTADA DA SOLUO DE ANDERSON TORRES (SO PAULO - SP)

a) Sejam I = {1, 2,..., n} e P I um conjunto fixo com r elementos. O total de elementos que pertencem a todos os Ai com com i P e que no pertencem a nenhum outro Aj com j I \ P exatamente o total de elementos do conjunto

( A ) que no pertencem a ( ( A ( A ))).


i P i j I \ P j i P i i P

Podemos usar Incluso-Excluso:

Ai Ai

j I P

( A ( A )) =
j i P i KP

i P

KP

i K

Ai +

i k

Ai ... =

K = r +1

K =r+ 2

KP

( 1) K P Ai
i K

Agora, variando o conjunto P:

( 1)
P =r K P

K P

i K

Ai =

K I P K K r P =r

( 1)
57

K P

i K

Ai

EUREKA! N25, 2007

Sociedade Brasileira de Matemtica

O conjunto de ndices P K com P = r (onde K = k ) pode ser escolhido de

k Ai no depende de P, a soma acima maneiras para cada K, e j que i K r


vale

rk n

( 1)

k r

k Ai = i r K I K K =k
k =m

( 1)
k =r k m

k r

k Sk . r

b) Somando a expresso identidade

( 1)
j

k S k de m = r at n, e usando a m k 1 k r = ( 1) k r k r , r 1

( 1)
m =r

k m

k = m

( 1)
j=0

k r

k k r = ( 1) j

que pode ser facilmente provada por induo (ver nota), o resultado segue. Nota: Para provar por induo a identidade

1 k r k = ( 1) , 0 r k , basta ver que isto vale para r = 0 e, j=0 j r r +1 r k k k se r < k, ( 1) j = ( 1) r +1 + ( 1) j = j=0 j r + 1 j=0 j

( 1)

k r k 1 r +1 k k 1 r +1 k 1 = (1) r +1 + (1) = (1) = (1) . r + 1 r r +1 r + 1 r


109) Na figura abaixo, AB = AC , B AC = 100 e AD = BC . Mostre que

x = BCD racional quando expresso em graus.


A

B x D C

EUREKA! N25, 2007

58

Sociedade Brasileira de Matemtica

SOLUO DE GIBRAN MEDEIROS


A 100 40 + x 60

40 x

40

60 C

Construimos o tringulo eqiltero ACE. Os tringulos BCE e ACD so congruentes, pois CE = AC , BCE = DAC = 100 e BC = AD. Logo Como o tringulo ABE issceles, temos BCE = 40 + x. AEB = ABE = 10 e, portanto 60 ( 40 + x ) = 10, donde x = 10. Continuamos aguardando as solues dos seguintes problemas propostos: 110) Um conjunto finito de inteiros positivos chamado de Conjunto DS se cada elemento divide a soma dos elementos do conjunto. Prove que todo conjunto finito de inteiros positivos subconjunto de algum conjunto DS. 111) Prove que existem infinitos mltiplos de 7 na seqncia (an ) abaixo:

a1 = 1999, an = an 1 + p (n), n 2 , onde p(n) o menor primo que divide n.


112) a) Determine todos os inteiros positivos n tais que existe uma matriz n n com todas as entradas pertencentes a { 1, 0, 1} tal que os 2n nmeros obtidos como somas dos elementos de suas linhas e de suas colunas so todos distintos. Nota: Como explicado na pgina 72 da Eureka! No. 24, o item b) do problema 112 foi anulado.
Agradecemos tambm o envio das solues e a colaborao de:
Andr Arajo Carlos Alberto da Silva Victor Diego Andrs de Barros Dymitri Cardoso Leo Fbio Soares Piau Janderson Alencar Mardnio Luz do Amaral Michel Angelucci Renan e Gabriel Lima Novais Samuel Lil Abdalla
EUREKA! N25, 2007

Fortaleza CE Nilpolis RJ Recife PE Recife PE Recife PE Recife PE Recife PE Ibitinga SP Niteri RJ Sorocaba SP

59

Sociedade Brasileira de Matemtica

PROBLEMAS PROPOSTOS
Convidamos o leitor a enviar solues dos problemas propostos e sugestes de novos problemas para prximos nmeros.

113) a1 , a2 , a3 ,... formam uma seqncia de inteiros positivos menores que 2007 tais que

am + an inteiro, para quaisquer inteiros positivos m, n. am + n

Prove que a seqncia (an) peridica a partir de um certo ponto. 114) Sabendo que sen x + sen y + sen z + sen w = 0 e cos x + cos y + cos z + cos w = 0, mostre que

sen 2003 x + sen 2003 y + sen 2003 z + sen 2003w = 0.


115) Suponha que ABC um tringulo com lados inteiros a, b e c com

BCA = 60 e mdc(a, b) = mdc( a, c) = mdc(b, c) = 1. Prove que c 1(mod 6) .


116) Seja ABC um tringulo e sejam X, Y e Z as reflexes de A, B e C em relao s retas BC, CA e AB, respectivamente. Prove que x, y e z so colineares se e somente se cos A cos B cos C = 3 8. 117) Sejam r e s duas retas reversas (i.e., no contidas num mesmo plano) e A, B,

E,C E, D E C, D, E A, B

ED E. Prove que os tetraedros ABCD e E E ED E tm o mesmo volume. CD = C ABC

EE e E, B E r , C , D, C E, D E s, AB = AB pontos tais que A, B , A


2 + 9an , n 1. 3 + 9an

118) Considere a seqncia (an )n 1 dada por a1 = 1 e an +1 = Prove que (an ) converge e calcule a seu limite.

Problema 113 proposto por Anderson Torres (So Carlos SP); Problema 114 proposto por Carlos A. Gomes (Natal RN); Problema 115 proposto por Gabriel Ponce (enviado por email); Problema 116 e 117 propostos por Wilson Carlos da Silva Ramos (Belm PA); Problema 118 proposto por Sidnei Belcides Avelar (Macap AP)

EUREKA! N25, 2007

60

Sociedade Brasileira de Matemtica

AGENDA OLMPICA
XXIX OLIMPADA BRASILEIRA DE MATEMTICA NVEIS 1, 2 e 3 Primeira Fase Sbado, 16 de junho de 2007 Segunda Fase Sbado, 15 de setembro de 2007 Terceira Fase Sbado, 27 de outubro de 2007 (nveis 1, 2 e 3) Domingo, 28 de outubro de 2007 (nveis 2 e 3 - segundo dia de prova). NVEL UNIVERSITRIO Primeira Fase Sbado, 15 de setembro de 2007 Segunda Fase Sbado, 27 e Domingo, 28 de outubro de 2007

XIII OLIMPADA DE MAIO 12 de maio de 2007

XVIII OLIMPADA DE MATEMTICA DO CONE SUL Uruguai 12 a 17 de junho de 2007

XLVIII OLIMPADA INTERNACIONAL DE MATEMTICA 19 a 31 de julho de 2007 Vietn

XIV OLIMPADA INTERNACIONAL DE MATEMTICA UNIVERSITRIA 3 a 9 de agosto de 2007 Blagoevgrad, Bulgria

XXII OLIMPADA IBEROAMERICANA DE MATEMTICA 6 a 16 de setembro de 2007 Portugal

X OLIMPADA IBEROAMERICANA DE MATEMTICA UNIVERSITRIA

EUREKA! N25, 2007

61

Sociedade Brasileira de Matemtica

COORDENADORES REGIONAIS
Alberto Hassen Raad Amrico Lpez Glvez Amarsio da Silva Arajo Andreia Goldani Antonio Carlos Nogueira Ali Tahzibi Benedito Tadeu Vasconcelos Freire Carlos Alexandre Ribeiro Martins Carmen Vieira Mathias Claus Haetinger Cleonor Crescncio das Neves Cludio de Lima Vidal Edson Roberto Abe lio Mega Eudes Antonio da Costa Fbio Brochero Martnez Florncio Ferreira Guimares Filho Genildo Alves Marinho Ivanilde Fernandes Saad Jacqueline Rojas Arancibia Janice T. Reichert Joo Bencio de Melo Neto Joo Francisco Melo Libonati Jos Cloves Saraiva Jos Luiz Rosas Pinho Jos Vieira Alves Jos William Costa Krerley Oliveira Licio Hernandes Bezerra Luzinalva Miranda de Amorim Mrio Rocha Retamoso Marcelo Rufino de Oliveira Marcelo Mendes Newman Simes Nivaldo Costa Muniz Ral Cintra de Negreiros Ribeiro Ronaldo Alves Garcia Rogrio da Silva Igncio Reginaldo de Lima Pereira Reinaldo Gen Ichiro Arakaki Ricardo Amorim Srgio Cludio Ramos Seme Gebara Neto Tadeu Ferreira Gomes Toms Menndez Rodrigues Valdenberg Arajo da Silva Valdeni Soliani Franco Vnia Cristina Silva Rodrigues Wagner Pereira Lopes William Beline (UFJF) (USP) (UFV) FACOS (UFU) (USP) (UFRN) (Univ. Tec. Fed. de Paran) (UNIFRA) (UNIVATES) (UTAM) (UNESP) (Colgio Objetivo de Campinas) (Colgio Etapa) (Univ. Federal do Tocantins) (UFMG) (UFES) (Centro Educacional Leonardo Da Vinci) (UC. Dom Bosco) (UFPB)) (UNOCHAPEC) (UFPI) (Grupo Educacional Ideal) (UFMA) (UFSC) (UFPB) (Instituto Pueri Domus) (UFAL) (UFSC) (UFBA) (UFRG) (Grupo Educacional Ideal) (Colgio Farias Brito, Pr-vestibular) (Cursinho CLQ Objetivo) (UFMA) (Colgio Anglo) (UFGO) (Col. Aplic. da UFPE) (Escola Tcnica Federal de Roraima) (UNIFESP) (Centro Educacional Logos) (IM-UFRGS) (UFMG) (UEBA) (U. Federal de Rondnia) (U. Federal de Sergipe) (U. Estadual de Maring) (U. Metodista de SP) (CEFET GO) (UNESPAR/FECILCAM) Juiz de Fora MG Ribeiro Preto SP Viosa MG Osrio RS Uberlndia MG So Carlos SP Natal RN Pato Branco - PR Santa Mara RS Lajeado RS Manaus AM S.J. do Rio Preto SP Campinas SP So Paulo SP Arraias TO Belo Horizonte MG Vitria ES Taguatingua DF Campo Grande MS Joo Pessoa PB Chapec SC Teresina PI Belm PA So Luis MA Florianpolis SC Campina Grande PB Santo Andr SP Macei AL Florianpolis SC Salvador BA Rio Grande RS Belm PA Fortaleza CE Piracicaba SP So Luis MA Atibaia SP Goinia GO Recife PE Boa Vista RR SJ dos Campos SP Nova Iguau RJ Porto Alegre RS Belo Horizonte MG Juazeiro BA Porto Velho RO So Cristovo SE Maring PR S.B. do Campo SP Jata GO Campo Mouro PR

EUREKA! N25, 2007

62

CONTEDO

XXVIII OLIMPADA BRASILEIRA DE MATEMTICA Problemas e Solues da Primeira Fase XXVIII OLIMPADA BRASILEIRA DE MATEMTICA Problemas e Solues da Segunda Fase XXVIII OLIMPADA BRASILEIRA DE MATEMTICA Problemas e Solues da Terceira Fase

15

32

XXVIII OLIMPADA BRASILEIRA DE MATEMTICA Problemas e Solues da Primeira Fase Nvel Universitrio

55

XXVIII OLIMPADA BRASILEIRA DE MATEMTICA Problemas e Solues da Segunda Fase Nvel Universitrio

59

XXVIII OLIMPADA BRASILEIRA DE MATEMTICA Premiados AGENDA OLMPICA COORDENADORES REGIONAIS

70

74 75

Sociedade Brasileira de Matemtica

XXVIII OLIMPADA BRASILEIRA DE MATEMTICA


Problemas e solues da Primeira Fase PROBLEMAS NVEL 1
1. Em um tanque h 4000 bolinhas de pingue-pongue. Um menino comeou a retirar as bolinhas, uma por uma, com velocidade constante, quando eram 10h. Aps 6 horas, havia no tanque 3520 bolinhas. Se o menino continuasse no mesmo ritmo, quando o tanque ficaria com 2000 bolinhas? A) s 11h do dia seguinte B) s 23h do mesmo dia C) s 4h do dia seguinte D) s 7h do dia seguinte E) s 9h do dia seguinte 2. O grfico a seguir apresenta informaes sobre o impacto causado por 4 tipos de monocultura ao solo. Para cada tipo de monocultura, o grfico mostra a quantidade de gua, em litros, e a de nutrientes (nitrognio, fsforo e potssio), em quilogramas, consumidos por hectare para a produo de 1kg de gros de soja ou 1kg de milho ou 1kg de acar ou 1kg de madeira de eucalipto. Sobre essas monoculturas, pode-se afirmar que:
2000 1500 1000 500 0 cana-deaucar soja gua milho

eucalipto

nutrientes

A) O eucalipto precisa de cerca de 1/3 da massa de nutrientes necessrios de que a cana-de-acar precisa para se desenvolver. B) O eucalipto a que mais seca e empobrece o solo, causando desequilbrio ambiental. C) A soja cultura que mais precisa de nutrientes. D) O milho precisa do dobro do volume de gua de que precisa a soja. E) A cana-de-acar a que necessita do ambiente mais mido para crescer.

EUREKA! N26, 2007

Sociedade Brasileira de Matemtica

3. Um time de futebol ganhou 8 jogos mais do que perdeu e empatou 3 jogos menos do que ganhou, em 31 partidas jogadas. Quantas partidas o time venceu? A) 11 B) 14 C) 15 D) 17 E) 23 4. Efetuando as operaes indicadas na expresso

22007 + 22005 2006 2006 2004 2 +2


obtemos um nmero de quatro algarismos. Qual a soma dos algarismos desse nmero? A) 4 B) 5 C) 6 D) 7 E) 8 5. Quantos nmeros de trs algarismos mpares distintos so divisveis por 3? A) 18 B) 24 C) 28 D) 36 E) 48 6. Uma empresa de telefonia celular oferece planos mensais de 60 minutos a um custo mensal de R$ 52,00, ou seja, voc pode falar durante 60 minutos no seu telefone celular e paga por isso exatamente R$ 52,00. Para o excedente, cobrada uma tarifa de R$ 1,20 cada minuto. A mesma tarifa por minuto excedente cobrada no plano de 100 minutos, oferecido a um custo mensal de R$ 87,00. Um usurio optou pelo plano de 60 minutos e no primeiro ms ele falou durante 140 minutos. Se ele tivesse optado pelo plano de 100 minutos, quantos reais ele teria economizado? A) 10 B) 11 C) 12 D) 13 E) 14 7. Quantos tringulos issceles tm como vrtices os vrtices do pentgono regular desenhado ao lado? A) 5 B) 10 C) 15 D) 20 E) 25 8. Dos nmeros a seguir, qual o nico que pode ser escrito como produto de quatro naturais consecutivos? A) 712 B) 548 C) 1026 D) 1456 E) 1680 9. Ao redor de um grande lago existe uma ciclovia de 45 quilmetros de comprimento, na qual sempre se retorna ao ponto de partida se for percorrida num nico sentido. Dois amigos partem de um mesmo ponto com velocidades constantes de 20 km por hora e 25 km por hora, respectivamente, em sentidos opostos. Quando se encontram pela primeira vez, o que estava correndo a 20 km
EUREKA! N26, 2007

Sociedade Brasileira de Matemtica

por hora aumenta para 25 km por hora e o que estava a 25 km por hora diminui para 20 km por hora. Quanto tempo o amigo que chegar primeiro ao ponto de partida dever esperar pelo outro? A) nada B) 10 min C) 12 min D) 15 min E) 18 min 10. Num relgio digital, as horas so exibidas por meio de quatro algarismos. Por exemplo, ao mostrar 00:00 sabemos que meia-noite e ao mostrar 23:59 sabemos que falta um minuto para meia-noite. Quantas vezes por dia os quatro algarismos mostrados so todos pares? A) 60 B) 90 C) 105 D) 180 E) 240 11. So dadas duas tiras retangulares de papel com 20 cm de comprimento, uma com 5 cm de largura e outra com 11 cm de largura. Uma delas foi colada sobre a outra, perpendicularmente, de modo a formar a figura ilustrada ao lado. Qual o permetro dessa figura, em centmetros? A) 50 E) 120 B) 60 C) 80 D) 1

90

12. Seis amigos planejam viajar e decidem faz-lo em duplas, cada uma utilizando um meio de transporte diferente, dentre os seguintes: avio, trem e carro. Alexandre acompanha Bento. Andr viaja de avio. Carlos no acompanha Drio nem faz uso do avio. Toms no anda de trem. Qual das afirmaes a seguir correta? A) Bento vai de carro e Carlos vai de avio. B) Drio vai de trem e Andr vai de carro. C) Toms vai de trem e Bento vai de avio. D) Alexandre vai de trem e Toms vai de carro. E) Andr vai de trem e Alexandre vai de carro. 13. Usando pastilhas de cermica preta na forma de quadradinhos foi composta uma decorao numa parede, mostrada parcialmente abaixo:

EUREKA! N26, 2007

Sociedade Brasileira de Matemtica

Quantas pastilhas foram empregadas em toda a decorao considerando-se que na ltima pea montada foram utilizadas 40 pastilhas? A) 60 B) 68 C) 81 D) 100 E) 121 14. Sara foi escrevendo nas casas de um tabuleiro 95 por 95 os mltiplos positivos de 4, em ordem crescente, conforme a figura a seguir. 4 760 764 8 756 12 752 16 748 20 744 376 388 380 384

U O nmero que Sara escreveu onde se encontra a letra U : A) 35192 B) 35196 C) 36100 D) 36104 E) 36108

15. O desenho direita representa dois quadrados menores congruentes de lado 20 e um quadrado maior. O vrtice O o nico ponto comum aos dois quadrados menores e o centro do quadrado O A B maior. Os vrtices A, O e B esto alinhados e a rea da regio do quadrado maior no pintada igual a 36% da rea de toda a regio pintada. Qual a rea do quadrado maior? A) 420 B) 496 C) 576 D) 640 E) 900 16. Um certo nmero inteiro positivo, quando dividido por 15 d resto 7. Qual a soma dos restos das divises desse nmero por 3 e por 5? A) 2 B) 3 C) 4 D) 5 E) 6 17. No fim de 1994, Neto tinha a metade da idade de sua av. A soma dos anos de nascimento dos dois 3844. Quantos anos Neto completa em 2006? A) 55 B) 56 C) 60 D) 62 E) 108 18. A figura a seguir representa um Tangram, quebra-cabeas chins formado por 5 tringulos, 1 paralelogramo e 1 quadrado. Sabendo que a rea do Tangram a seguir 64 cm2, qual a rea, em cm2, da regio sombreada?
EUREKA! N26, 2007

Sociedade Brasileira de Matemtica

A) 7,6

B) 8

C) 10,6

D) 12

E) 21,3

19. As permutaes da palavra BRASIL foram listadas em ordem alfabtica, como se fossem palavras de seis letras em um dicionrio. A 361 palavra nessa lista : A) BRISAL B) SIRBAL C) RASBIL D) SABRIL E) LABIRS 20. No planeta POT o nmero de horas por dia igual a nmero de dias por semana, que igual ao nmero de semanas por ms, que igual ao nmero de meses por ano. Sabendo que em POT h 4096 horas por ano, quantas semanas h num ms? A) 8 B) 12 C) 64 D) 128 E) 256

PROBLEMAS NVEL 2
1. Veja o problema No. 4 do nvel 1. 2. Veja o problema No. 11 do nvel 1. 3. Se um nmero de dois dgitos 5 vezes a soma de seus dgitos, ento o nmero formado pela troca dos dgitos a soma dos dgitos multiplicada por: A) 3 B) 5 C) 6 D) 4 E) 7 4. Veja o problema No. 9 do nvel 1. 5. Na figura, AB = AC, AE = AD e o ngulo BAD mede 30o. Ento o ngulo x mede:
A 30 E x B D C

A)10o

B) 20o

C) 15o

D) 30o

E) 5o

EUREKA! N26, 2007

Sociedade Brasileira de Matemtica

6. A soma de trs nmeros naturais consecutivos igual ao produto desses trs nmeros. A soma dos quadrados desses nmeros : A) 14 B) 15 C) 18 D) 24 E) 36 7. Veja o problema No. 17 do nvel 1. 8. Trs quadrados so colados pelos seus vrtices entre si e a dois bastes verticais, como mostra a figura.

75 30 126

A medida do ngulo x : A) 39 B) 41

C) 43

D) 44

E) 46

9. Sejam a, b e c inteiros e positivos. Entre as opes abaixo, a expresso que no pode representar o nmero 24 : A) ab3 B) a2b3 C) a c b c D) ab2c3 E) a b bc c a

10. O nmero de quadrados que podem ser construdos com vrtices nos pontos da figura abaixo :

A) 18

B) 14

C) 9

D) 20

E) 10

11. Veja o problema No. 12 do nvel 1. 13. O mximo divisor comum de todos os termos da seqncia a n = n 3 n , n = 1, 2, 3, ... : A) 2 B) 3 C) 4 D) 5 E) 6
EUREKA! N26, 2007

Sociedade Brasileira de Matemtica

14. Samuel possui trs irmos a mais do que irms. O nmero de irmos de Samila, irm de Samuel, igual ao dobro do nmero de suas irms. O nmero de filhos (homens e mulheres) que possui o pai de Samuel e Samila : A) 10 B) 13 C) 16 D) 17 E) 20 15. Veja o problema No. 18 do nvel 1. 16. Joo escreveu todos os nmeros com menos de 4 dgitos usando apenas os algarismos 1 e 2 numa folha de papel e depois somou todos eles. O valor obtido foi: A) 2314 B) 3000 C) 1401 D) 2316 E) 1716 17. Sejam a, b e c nmeros reais positivos cuja soma 1. Se a, b e c so as medidas dos lados de um tringulo, podemos concluir que 1 1 1 A) 0 < a b < e 0 < b c < e 0 < c a < 2 2 2 1 1 1 B) a < e b < e c < 2 2 2 1 1 1 C) a + b < e b + c < e c + a < 2 2 2 1 1 1 D) a e b e c 3 3 3 1 1 1 E) a e b e c 3 3 3 18. O nmero de solues inteiras e positivas do sistema abaixo : a + b = c 2 a + b + c = 30 A) 45 B) 23 C) 24 D) 25 E) 72 19. Um nmero com dois dgitos distintos e no nulos chamado de bonito se o dgito das dezenas maior do que o dgito das unidades. A quantidade de nmeros bonitos : A) 72 B) 36 C) 35 D) 64 E) 56 20. O professor Piraldo aplicou uma prova para seus cinco alunos e, aps corrigilas, digitou as notas em uma planilha eletrnica que calcula automaticamente a mdia das notas medida que elas so digitadas. Piraldo notou que aps digitar cada nota a mdia calculada pela planilha era um nmero inteiro. Se as notas dos
EUREKA! N26, 2007

Sociedade Brasileira de Matemtica

cinco estudantes so, em ordem crescente, 71, 76, 80, 82 e 91, a ltima nota que Piraldo digitou foi:

A) 71

B) 76

C) 80

D) 82

E) 91

21. Simplificando a expresso:

2+ 3 . 2+ 2+ 3 . 2+ 2+ 2+ 3 . 2 2+ 2+ 3
obtemos:

A)

B)

C) 1

D) 2 + 2

E) 2 + 3

22. Ludmilson percebeu que para numerar as pginas de um livro, consecutivamente, a partir da pgina 2, foram usados 2006 algarismos. O nmero de pginas do livro de Ludmilson : A)701 B) 702 C) 703 D) 704 E) 705 23. Sejam x, y, z nmeros reais no nulos tais que x + y + z = 0 . O valor de

(x

1 1 1 y 2 z 2 ) 3 3 + 3 3 + 3 3 : x z y z x y
B) 1 C) 2 D) 3 E) 4

A) 0

24. Veja o problema No. 10 do nvel 1. 25. Na figura a seguir, ABC um tringulo qualquer e ACD e AEB so tringulos eqilteros. Se F e G so os pontos mdios de EA e AC, respectivamente, a razo BD : FG
D A

A)
G

1 2

B) 1 D) 2

C)
B

3 2

E) Depende das medidas dos lados de ABC.

EUREKA! N26, 2007

Sociedade Brasileira de Matemtica

PROBLEMAS NVEL 3
1. Veja o problema No. 17 do nvel 1. 2. Quantos resultados diferentes podemos obter somando pares de nmeros distintos do conjunto {1, 2,, 2006} ? A) 2006 B) 2007 C) 4009 D) 4011 E) 4012 3. Uma colnia de amebas tem inicialmente uma ameba amarela e uma ameba vermelha. Todo dia, uma nica ameba se divide em duas amebas idnticas. Cada ameba na colnia tem a mesma probabilidade de se dividir, no importando sua idade ou cor. Qual a probabilidade de que, aps 2006 dias, a colnia tenha exatamente uma ameba amarela? 1 2006 1 1 1 A) 2006 B) C) D) E) 2006 2007 2007 2 2006 2007 4. Veja o problema No. 8 do nvel 2. 5. Os dois nmeros reais a e b so no nulos e satisfazem ab = a b. Assinale a
alternativa que exibe um dos possveis valores de

a b + ab . b a
1 2

A) 2

B)

1 2

C)

1 3

D)

E) 2

6. De quantas maneiras podemos colocar, em cada espao abaixo, um entre os algarismos 4, 5, 6, 7, 8, 9, de modo que todos os seis algarismos apaream e formem, em cada membro, nmeros de dois algarismos que satisfazem a dupla desigualdade? __>__>__ A) 100 B) 120 C) 240 D) 480 E) 720

7. Que expresso no pode representar o nmero 24 para valores inteiros positivos convenientes de a, b e c? A) ab3 B) a2b3 C) acbc D) ab2c3 E) abbcca 8. Qual dos valores abaixo de x tal que 2 x 2 + 2 x + 19 no um nmero primo? A) 50 B) 37 C) 9 D) 5 E) 1 9. Veja o problema No. 17 do nvel 2.
EUREKA! N26, 2007

10

Sociedade Brasileira de Matemtica

10. Uma seqncia tem 9 nmeros reais, sendo o primeiro 20 e o ltimo 6. Cada termo da seqncia, a partir do terceiro, a mdia aritmtica de todos os anteriores. Qual o segundo termo da seqncia? A) 8 B) 0 C) 4 D) 14 E) 2006 11. Quantos ternos de nmeros reais x, y, z satisfazem o sistema abaixo? x( x + y + z ) = 2005

y ( x + y + z ) = 2006 z ( x + y + z ) = 2007
A) Nenhum B) 1 C) 2 D) 3 E) 2006

12. Arnaldo tem vrios quadrados azuis 1 1 , vrios quadrados amarelos 2 2 e vrios quadrados verdes 3 3 e quer montar um quadrado maior no qual apaream as trs cores. Qual a menor quantidade de quadrados que ele poder utilizar ao todo? A) 3 B) 6 C) 7 D) 8 E) 9 x 5 2006 13. Sejam x e y nmeros racionais. Sabendo que tambm um 4 y 2006 nmero racional, quanto vale o produto xy? A) 20 B) Pode ser igual a 20, mas tambm pode assumir outros valores. C) 1 D) 6 E) No se pode determinar. 14. Veja o problema No. 20 do nvel 2. 15. Veja o problema No. 25 do nvel 2. 16. O inteiro positivo x mltiplo de 2006 e nmero de possveis valores de x? A) 1 B) 2 C) 3
x est entre 2005 e 2007. Qual o

D) 4

E) 5

17. Na figura temos dois semicrculos de dimetros PS, de medida 4, e QR, paralelo a PS. Alm disso, o semicrculo menor tangente a PS em O. Qual a rea destacada?

EUREKA! N26, 2007

11

Sociedade Brasileira de Matemtica

A) B) C) D) E)

2 2 3 4 2 4

18. Iniciando com o par (2048, 1024), podemos aplicar quantas vezes quisermos a 3a + b a + 3b operao que transforma o par (a, b) no par , , ento, dentre os 4 4 seguintes pares: (1664, 1408) 1) (1540, 1532) 2) (1792, 1282) 3) (1537, 1535) 4) (1546, 1526) 5) A) Todos podem ser obtidos. B) Apenas o par 4 no pode ser obtido. C) Apenas o par 3 no pode ser obtido. D) Existem exatamente dois pares que no podem ser obtidos. E) Existem mais de dois pares que no podem ser obtidos. 19. Num tabuleiro retangular de 13 linhas e 17 colunas colocamos nmeros em cada casinha da seguinte maneira: primeiro, numeramos as casinhas da primeira linha, da esquerda para a direita, com os nmeros 1, 2, 3, , 17, nessa ordem; depois numeramos a segunda linha, tambm da esquerda para a direita, com os nmeros de 18 a 34, e assim por diante. Aps preenchermos todo o tabuleiro, colocamos em cada casinha um segundo nmero, numerando as casinhas da primeira coluna, de cima para baixo, com os nmeros 1, 2, 3, , 13, nessa ordem, depois numeramos a segunda coluna, tambm de cima da baixo, com os nmeros de 14 a 26, e assim por diante. Deste modo, cada casinha tem dois nmeros. Quantas casinhas tm dois nmeros iguais? A) 2 B) 3 C) 4 D) 5 E) 6 20. Altino est encostado num muro bem alto, durante a noite. A rua onde Altino est iluminada por uma lmpada no topo de um poste de 4 metros de altura, a 10 metros de distncia do muro. Altino, um rapaz de 2 metros de altura, anda em direo ao muro. Seja f(x) a altura, em metros, da sombra de Altino produzida
EUREKA! N26, 2007

12

Sociedade Brasileira de Matemtica

pela lmpada no muro quando Altino est a uma distncia de x metros do muro. Qual alternativa representa melhor o grfico de f(x)?
f(x)

f(x)

f(x)

A)
f(x)

B)
f(x)

C)

D)

E)

21. O piso de um quarto tem forma de um quadrado de lado 4 m. De quantas maneiras podemos cobrir totalmente o quarto com oito tapetes iguais de dimenses 1 m e 2 m? Mostramos abaixo trs maneiras de faz-lo:

A) 27

B) 30

C) 34

D) 36

E) 52

22. Dois pontos A e B de um plano esto a 8 unidades de distncia. Quantas retas do plano esto a 2 unidades de A e 3 unidades de B? A) 1 B) 2 C) 3 D) 4 E) 5 23. Considere os 2161 produtos 0 2160 , 1 2159 , 2 2158 , ..., 2160 0 . Quantos deles so mltiplos de 2160? A) 2 B) 3 C) 12 D) 13 E) 2161 24. Qual o menor valor que a expresso

x2 + 1 + ( y x)2 + 4 + ( z y)2 + 1 + (10 z )2 + 9 pode assumir, sendo x, y e z reais? A) 7 B) 13 C) 4 + 109 D) 3 + 2 + 90 E) 149


EUREKA! N26, 2007

13

Sociedade Brasileira de Matemtica

25. Um cubo de aresta 1 cortado em quatro regies por dois planos: um deles contm as arestas AB e CD e o outro contm as arestas AE e DF. Qual o volume da(s) maior(es) das quatro regies?
C E D

A) D)

1 4

B) E)

1 3

C)

2 4

F A B

3 8

1 2

GABARITO
NVEL 1 (5. e 6. Sries) 1) A 6) D 2) A 7) B 3) B 8) E 4) D 9) A 5) B 10) C NVEL 2 (7. e 8. Sries) 1) D 6) A 2) C 7) C 3) C 8) A 4) A 9) B 5) C 10) D NVEL 3 (Ensino Mdio) 1) C 6) B 2) C 7) B 3) C 8) B 4) A 9) B 5) E 10) A 11) C 12) D 13) E 14) C 15) C 11) D 12) C 13) E 14) C 15) D 11) C 12) D 13) A 14) C 15) D 16) B 17) C 18) D 19) E 20) A 16) C 17) B 18) C 19) B 20) C 16) D 17) A 18) D 19) D 20) A 21) C 22) E 23) D 24) C 25) D 21) D 22) D 23) D 24) E 25) B

EUREKA! N26, 2007

14

Sociedade Brasileira de Matemtica

XXVIII OLIMPADA BRASILEIRA DE MATEMTICA


Problemas e Solues da Segunda Fase PROBLEMAS Nvel 1 PARTE A (Cada problema vale 5 pontos)
01. Qual a soma dos algarismos do nmero

2 2 23 2 4 + + + 2 2 2 23

2 2005 2 2006 + ? 2 2004 2 2005

02. A massa de gordura de uma certa pessoa corresponde a 20% de sua massa total. Essa pessoa, pesando 100 kg, fez um regime e perdeu 40% de sua gordura, mantendo os demais ndices. Quantos quilogramas ela pesava ao final do regime? 03. Quantos os nmeros de dois algarismos tm a soma desses algarismos igual a um quadrado perfeito? Lembre-se que, por exemplo, 09 um nmero de um algarismo. 04. Os nmeros de 1 a 99 so escritos lado a lado: 123456789101112...9899.

Ento aplicamos a seguinte operao: apagamos os algarismos que aparecem nas posies pares, obtendo 13579012...89. Repetindo essa operao mais 4 vezes, quantos algarismos iro sobrar?
05. Com a parte destacada da folha retangular ao

lado, pode-se montar um cubo. Se a rea da folha 300cm2, qual o volume desse cubo, em cm3?

06. Na tabela a seguir, escreva os nmeros de 1 a 9 em cada coluna, de modo que

a soma dos nmeros escritos nas 9 linhas seja a mesma, igual a Y. Seja X a soma dos nmeros de cada coluna. Calcule X + Y.
Y Y Y Y Y Y Y Y Y X X X

EUREKA! N26, 2007

15

Sociedade Brasileira de Matemtica

PROBLEMAS Nvel 1 PARTE B


(Cada problema vale 10 pontos) PROBLEMA 1

Jade escreveu todos os nmeros de 3 algarismos em cartes amarelos, um por carto e escreveu todos os nmeros de 4 algarismos em cartes azuis, um por carto. Os cartes so todos do mesmo tamanho. a) Ao todo, quantos cartes foram utilizados? Lembre-se que, por exemplo, 037 um nmero de dois algarismos, bem como 0853 um nmero de trs algarismos. b) Todos os cartes so ento colocados numa mesma urna e embaralhados. Depois Jade retira os cartes, um a um, sem olhar o que est pegando. Quantos cartes Jade dever retirar para ter certeza de que h dois cartes azuis entre os retirados?
PROBLEMA 2

No quadriculado a seguir, cada quadradinho tem 1 cm2 de rea.

a) Qual a rea e o permetro da figura formada pelos quadradinhos pintados de cinza? b) Pintando outros quadradinhos, podemos aumentar a rea dessa figura, sem mudar o seu permetro. Qual o valor mximo da rea que podemos obter dessa maneira?
PROBLEMA 3

Esmeralda inventou uma brincadeira. Digitou alguns algarismos na primeira linha de uma folha. Depois, na segunda linha, fez a descrio dos algarismos digitados da seguinte maneira: ela apresentou as quantidades de cada um dos que apareceram, em ordem crescente de algarismo. Por exemplo, aps digitar 21035662112, ela digitou 103132131526, pois em 21035662112 existe um algarismo 0, trs algarismos 1, trs algarismos 2, um algarismo 3, um algarismo 5 e dois algarismos 6. a) Ela comeou uma nova folha com 1. Fez, ento, sua descrio, ou seja, digitou 11 na segunda linha. Depois, descreveu 11, ou seja, digitou 21 na terceira linha, e assim continuou. O que ela digitou na 10a linha da folha?
EUREKA! N26, 2007

16

Sociedade Brasileira de Matemtica

b) Esmeralda gostou tanto de fazer isso que decidiu preencher vrias folhas com essa brincadeira, comeando com 01 na primeira linha da primeira folha. Quais so os dois primeiros algarismos da esquerda do que ela digitou na 2006a linha?

PROBLEMAS Nvel 2 PARTE A (Cada problema vale 4 pontos)


01. Esmeralda posicionou todos os nmeros naturais de 1 a 2006 no seguinte

arranjo em forma de pirmide:


19 11 5 20 12 6 2 21 13 7 3 1 22 14 8 4 23 15 24 9 16

17

18 10

25

Em qual andar se encontrar o nmero 2006? (Por exemplo: o nmero 1 est no primeiro andar, o 6 no segundo andar e o 23 no terceiro). 02. A soma dos quadrados de trs inteiros consecutivos igual a 302. Qual a soma desses nmeros? 03. Seja ABC um tringulo retngulo em A. Considere M e N pontos sobre a hipotenusa BC tais que CN = NM = MB. Os pontos X e Y so tais que XA = AM e YA = AN. Determine a rea do quadriltero XYBC, sabendo que o tringulo ABC tem rea 12 cm2.
C

04. Um tabuleiro de xadrez 8 8 ser decomposto em retngulos que satisfazem

simultaneamente as seguintes propriedades: (i) cada retngulo possui um nmero inteiro de casas; (ii) os diversos retngulos possuem nmeros de casas distintos entre si; (iii) cada retngulo possui a mesma quantidade de casas brancas e pretas.
EUREKA! N26, 2007

17

Sociedade Brasileira de Matemtica

Qual o maior nmero de retngulos que pode ter a decomposio do tabuleiro?


05. A partir de uma terna ordenada (a, b, c), obtemos uma seqncia de ternas

atravs de sucessivas transformaes do tipo: (a, b, c) (a2 b, a b + c, b c). Por exemplo, a partir da terna (1, 2, 3), obtemos a seguinte seqncia: (1, 2, 3) (2, 2, 1) (8, 1, 3) (64, 12, 4) ... Se comearmos com (1, 1, 1) como a primeira terna ordenada de uma seqncia, qual ser a soma dos trs termos da terna que ocupar a 2006a posio nesta seqncia?

PROBLEMAS Nvel 2 PARTE B (Cada problema vale 10 pontos)


Na Rua do Gengibre, existem n casas numeradas de 1 a n ( n ). As casas de numerao par ficam todas de um mesmo lado da rua, com as casas de numerao mpar do lado oposto. O prefeito Ludmilson Amottarim resolveu derrubar alguma(s) casa(s) a fim de que as somas dos nmeros das casas fossem iguais dos dois lados da rua. Para atingir o seu objetivo, qual o nmero mnimo de casas que o prefeito deve derrubar se: a) a rua tem n = 15 casas? b) a rua tem n = 16 casas? c) a rua tem n = 2006 casas?
PROBLEMA 2 No tringulo ABC issceles abaixo, I o encontro das bissetrizes e PROBLEMA 1

H o encontro das alturas. Sabe-se que HAI = HBC = . ngulo .


B

Determine o

I H A C

EUREKA! N26, 2007

18

Sociedade Brasileira de Matemtica

Sejam a e b nmeros reais distintos tais que a2 = 6b + 5ab e b2 = 6a + 5ab. a) Determine o valor de a + b. b) Determine o valor de ab.
PROBLEMA 4

PROBLEMA 3

Todos os inteiros de 1 a 2006 so escritos num quadro. Ento, cada um destes nmeros substitudo pela soma de seus algarismos. Estas substituies so realizadas repetidas vezes at que tenhamos 2006 nmeros com 1 algarismo cada. Dos nmeros que restaram no quadro, qual aparece mais vezes: o 1 ou o 2?

PROBLEMAS Nvel 3 PARTE A (Cada problema vale 4 pontos)


01. O par ordenado (83; 89) chamado de par centenrio porque 83 + 8 + 9 = 89 + 8 + 3 = 100, isto , a soma de cada nmero com os dgitos do outro nmero 100. Quantos so os pares centenrios? 02. Na figura a seguir, o pentgono regular ABCDE e o tringulo EFG esto inscritos na circunferncia Co, e M ponto mdio de BC. Para qual valor de , em graus, os tringulos EFG e HIG so semelhantes?
A G

I E

M F Co

03. Esmeralda e Jade correm em sentidos opostos em uma pista circular, comeando em pontos diametralmente opostos. O primeiro cruzamento entre elas ocorre depois de Esmeralda ter percorrido 200 metros. O segundo cruzamento ocorre aps Jade ter percorrido 350 metros entre o primeiro e o segundo ponto de encontro. As velocidades das moas so constantes. Qual o tamanho da pista, em metros? 04. Qual a maior quantidade de lados que pode ter uma seco determinada por um plano em um octaedro regular?
EUREKA! N26, 2007

19

Sociedade Brasileira de Matemtica

05. Ao jogarmos uma certa quantidade de dados cbicos com faces numeradas de

1 a 6, a probabilidade de obtermos soma dos pontos 2006 igual probabilidade de obtermos soma dos pontos S. Qual o menor valor possvel de S?

PROBLEMAS Nvel 3 PARTE B (Cada problema vale 10 pontos)


PROBLEMA 1

Seja n inteiro positivo. De quantas maneiras podemos distribuir n + 1 brinquedos distintos para n crianas de modo que toda criana receba pelo menos um brinquedo?
PROBLEMA 2

Encontre todos os pares de inteiros positivos (a; b) tais que (a + 1)(b + 1) mltiplo de ab + 1. No tringulo ABC tem-se AB = 4, AC = 3 e o ngulo BC mede 60o. Seja D o ponto de interseco entre a reta perpendicular a AB passando por B e a reta perpendicular a AC passando por C. Determine a distncia entre os ortocentros dos tringulos ABC e BCD. A seqncia Fn definida por F1 = F2 = 1 e Fn = Fn 1 + Fn 2 para n 3. Encontre todos os pares de inteiros positivos (m, n) tais que Fm Fn = mn.
PROBLEMA 4 PROBLEMA 3

Solues Nvel 1 Segunda Fase Parte A


Problema Resposta
01.

01 5

02 92

03 17

04 6

05 125

06 60

2 2 23 2 4 + + + 2 2 2 23

22005 22006 + = 2 + 2 + 2 + + 2 + 2 = 2005 2 = 4010 . 22004 22005 2005 parcelas iguais

A soma dos algarismos desse nmero 4 + 0 + 1 + 0 = 5 . tem 20% 100 = 20 kg de gordura. Ela perdeu 40% da sua gordura, ou seja, perdeu 40% 20 = 8 kg de gordura, e como manteve os demais ndices, ela pesava ao final do regime 100 8 = 92 kg.
02. Como 20% da massa total dessa pessoa correspondem massa de gordura, ela

EUREKA! N26, 2007

20

Sociedade Brasileira de Matemtica

03. A soma dos algarismos dos nmeros de dois algarismos varia de 1 a 18.

Dessas somas, as que so quadrados perfeitos so 1, 4, 9 e 16. Temos ento Soma 1: nmero 10 Soma 4: nmeros 13, 22, 31 e 40 Soma 9: nmeros 18, 27, 36, 45, 54, 63, 72, 81 e 90 Soma 16: nmeros 79, 88 e 97 Portanto, nas condies propostas, h 17 nmeros.
04. A quantidade inicial de algarismos 9 + 2 90 = 189 , dos quais 94 aparecem nas posies pares e 95 nas posies mpares. Apagados os algarismos que aparecem nas posies pares, sobram 95 algarismos; desses, 47 esto nas posies pares e 48 nas posies mpares. Repetindo a operao, restam 48 algarismos, sendo 24 algarismos em posies pares e 24 em posies mpares. Na terceira aplicao da operao restam 12 algarismos e, na quarta, sobram 6 algarismos. 05. Como a rea da folha 300cm2, cada quadrado destacado tem rea

300 = 25 cm2 e, portanto, lado medindo 5cm. Logo o volume desse cubo 12 5 3 = 125 cm3.
06. A soma dos 27 nmeros escritos na tabela igual a 3 vezes X e a 9 vezes o Y. Como X a soma dos nmeros de cada coluna, temos X = 1 + 2 + 3 + + 9 = 45 . Portanto 3 (1 + 2 + 3 + + 9) = 9 Y 3 45 = 9 Y Y = 15 Logo X + Y = 45 + 15 = 60 . O desenho ao lado mostra uma forma de escrever os nmeros na tabela.

1 2 3 4 5 6 7 8 9 X

5 6 4 9 7 8 2 3 1 X

9 7 8 2 3 1 6 4 5 X

Y Y Y Y Y Y Y Y Y

Solues Nvel 1 Segunda Fase Parte B


a) H 999 100 + 1 = 900 nmeros de trs algarismos, escritos em cartes amarelos, e 9999 1000 + 1 = 9000 nmeros de quatro algarismos, escritos em cartes azuis. Ao todo, foram utilizados 900 + 9000 = 9900 cartes. b) Como existe a possibilidade de serem retirados todos os cartes amarelos antes de aparecer algum azul, para Jade ter certeza de que h dois cartes azuis entre os
EUREKA! N26, 2007

SOLUO DO PROBLEMA 1:

21

Sociedade Brasileira de Matemtica

retirados ela dever retirar 900 + 2 = 902 cartes.


SOLUO DO PROBLEMA 2:

Como cada quadradinho tem 1 cm2 de rea, o lado de cada um mede 1 cm.

a) H 20 quadradinhos pintados de cinza. Logo a rea da figura formada 20 1 cm 2 = 20 cm 2 e como h 8 segmentos verticais esquerda e 8 direita alm de 9 segmentos horizontais pela parte de cima e 9 pela debaixo, o permetro, que a soma das medidas de todos os lados, 2 8 + 2 9 = 16 + 18 = 34 cm . b) O quadriculado inteiro um retngulo de lados 8 cm e 9 cm, e portanto de permetro 2 8 + 2 9 = 16 + 18 = 34 cm . Deste modo, o valor mximo da rea que podemos obter quando a figura for igual a todo o quadriculado e, assim, a rea ser 8 9 = 72 cm 2 .
SOLUO DO PROBLEMA 3:

a) Ela escreveu em cada uma das 9 primeiras linhas, na seguinte ordem, 1, 11, 21, 1112, 3112, 211213, 312213, 212223 e 114213. Logo na 10. linha ela escreveu 31121314. b) Esmeralda escreveu em cada uma das primeiras linhas, na seguinte ordem, 01, 1011, 1031, 102113, 10311213, 10411223, 1031221314, 1041222314, 1031321324, 1031223314, 1031223314,..., e percebeu que, a partir da 10. linha, o nmero 1031223314 comea a repetir. Portanto os dois primeiros algarismos da esquerda do nmero que ela digitou na 2006a. linha sero 1e 0.

Solues Nvel 2 Segunda Fase Parte A


Problema Resposta 01 20 02 30 ou 30 ou 30 03 32 04 07 05 00

EUREKA! N26, 2007

22

Sociedade Brasileira de Matemtica

01. Os nmeros da coluna do meio podem ser dados por: 1 + 2 + 4 + 6 + 8 +...+

2n = n2 + n + 1. Dessa forma o nmero do topo : 442 + 44 + 1 = 1981. Como 1981 est no 45 andar, e 2006 1981 = 25, 2006 deve estar no 20 andar.

02. Podemos representar os trs inteiros consecutivos por

n 1, n e n + 1 .

Temos

( n 1)

+ n2 + ( n + 1) = 302 n2 2n + 1 + n2 + n2 + 2n + 1 = 302 3n2 + 2 = 302


2

3n2 = 300 n2 = 100 n = 10 ou n = 10


Portanto, os trs inteiros consecutivos so 11, 10 e 9 ou 9,10 e 11 . Se admitirmos que estamos falando de inteiros positivos, a resposta 9 + 10 + 11 = 30 . Rigorosamente falando a resposta deveria ser: se os inteiros so positivos, ento a sua soma 30 e se os inteiros so negativos, ento sua soma 30.

Pontuao: 4 pontos para 30 ou para 30 ou para 30.


03.

Observe que os tringulos AXY e ANM so congruentes, e <YXA = <AMN. Assim, XY || MN e como XY = MN = MC = NB, segue que os quadrilteros XYCM e XYNB so paralelogramos, como A ponto mdio de XM e NY temos que [AYC] = [BAX] = (2/3).12 = 8. Logo, [XYCB] = (8/3).12 = 32.
04. Cada retngulo da decomposio possui um nmero par de casas, pois possui a mesma quantidade de casas brancas e pretas. Veja que a maior quantidade de nmeros pares distintos tais que a soma no supera 64 2 + 4 + 6 + 8 + 10 + 12 +
EUREKA! N26, 2007

23

Sociedade Brasileira de Matemtica

14 = 56, pois 2 + 4 + 6 + 8 + 10 + 12 + 14 + 16 = 72, ou seja, a soma de 8 nmeros pares distintos sempre maior que 64. Portanto, a decomposio pode ter no mximo 7 retngulos. Abaixo uma decomposio com 7 retngulos.

05. Fazendo as primeiras transformaes, obtemos a seguinte seqncia: (1, 1, 1) (1, 1, 0) (1, 0, 1) (0, 2, 1) (0, 3, 3) (0, 6, 6) ... Primeiramente, vemos que a partir da quarta terna, o primeiro vai ser sempre igual a 0 (zero). Ento, a partir desta terna, as transformaes so do tipo: (0, b, c) (0, b + c, b c). Logo, a partir da quarta terna ordenada da seqncia, a soma dos termos de todas as ternas ser igual a 0 b + c + b c = 0. Logo, a soma dos trs termos da terna que ocupar a 2006 posio nesta seqncia igual a 0 (zero).

Solues Nvel 2 Segunda Fase Parte B


SOLUO DO PROBLEMA 1:

Vamos usar a notao: S_par = soma de todas as casas de numerao par; S_mpar = soma de todas as casas de numerao mpar. a) Para este caso, temos: S_par = 2 + 4 + 6 + 8 + 10 + 12 + 14 = 56 e S_mpar = 1 + 3 + 5 + 7 + 9 + 11 + 13 + 15 = 64. Como a diferena entre as somas par e S_mpar > S_par, h a necessidade de retirar pelo menos duas casas do lado mpar como, por exemplo, as casas de numerao 7 e 1. A, teremos S_par = S_mpar = 56. Assim, o prefeito deve derrubar pelo menos 2 casas. b) Para este caso, temos: S_par = 2 + 4 + 6 + 8 + 10 + 12 + 14 +16 = 72 e S_mpar = 1 + 3 + 5 + 7 + 9 + 11 + 13 +15 = 64. Como a diferena entre as somas par e S_par > S_mpar, pode-se retirar apenas uma casa do lado par: a casa de numerao 8. A, teremos S_par = S_mpar = 64. Assim, o prefeito deve derrubar 1 casa. c) Para este caso, temos: S_par = 2 + 4 + 6 + 8 + 10 + ... + 2006 e S_mpar = 1 + 3 + 5 + ... + 2005. Assim, temos S_par S_mpar = (2 1) + (4 3) + ... + (2006
EUREKA! N26, 2007

24

Sociedade Brasileira de Matemtica

2005) = 1003. Como 1003 mpar, uma nica casa no suficiente, mas retirar as casas de numerao 1006 e 3 basta para que S_par = S_mpar. Assim, o nmero mnimo de casas que o prefeito deve derrubar 2 casas.
SOLUO DO PROBLEMA 2:

I A H

Q C

Como o tringulo issceles conclumos que, CBM = ABM e ACB = 90o , com isso, CAQ = , pois AQ uma altura. Como AI bissetriz, ento CAI = IAB = 2. Finalmente no AMB: + + 2 + = 90o = 18o.
SOLUO DO PROBLEMA 3:

a) Subtraindo as duas equaes dadas temos a 2 b 2 = 6(b a ) ou seja (a b)(a + b + 6) = 0 . Como a b , temos a + b = 6 . b) Da parte a), elevando ao quadrado, a 2 + b 2 + 2ab = 36 . Mas, somando as a 2 + b 2 = 6(a + b) + 10ab = 36 + 10ab . Portanto, equaes dadas, temos

36 + 2ab + 10 ab = 36 o que d ab = 6.
SOLUO DO PROBLEMA 4:

Quando trocamos um inteiro positivo pela soma de seus algarismos, no alteramos o resto da diviso por 9. Isto explicado pela decomposio do inteiro na forma: abcd = 1000a + 100b + 10c + d = 999a + 99b + 9c + a + b + c + d Da, temos que: abcd ( a + b + c + d ) = 999a + 99b + 9c = 9(111a + 11b + c) Logo, abcd e a + b + c + d deixam o mesmo resto na diviso por 9. Como todos os nmeros que restaram no quadro esto entre 0 e 9, inclusive, todos os nmeros 1 restantes no quadro so originados a partir de nmeros que deixam resto 1 na diviso por 9 (1, 10, 19, 28, 37, ..., 1999). Da mesma forma,
EUREKA! N26, 2007

25

Sociedade Brasileira de Matemtica

todos os nmeros 2 restantes no quadro so originados a partir de nmeros que deixam resto 2 na diviso por 9 (2, 11, 20, 29, 38, ..., 2000). Comparando, vemos que cada um dos nmeros 1 e 2 aparece 223 vezes no quadro. Portanto, ambos os nmeros 1 e 2 aparecem o mesmo nmero de vezes.

Solues Nvel 3 Segunda Fase Parte A


Problema Resposta 01 9 02 36 03 750 04 6 05 339

(10a + b) + (c + d ) = (10c + d ) + ( a + b) = 100 como b + c + d 27 , 10a 73 , e assim a 8 e, de modo anlogo, c 8 . Ainda mais, (10a + b) + (c + d ) = (10c + d ) + (a + b) 9a = 9c a = c . Temos ento 2 casos: I) a = c = 8 80 + b + 8 + d = 100 b + d = 12 , sendo esta uma condio necessria e suficiente para o par em questo ser centenrio. Obtemos assim os 7 seguintes pares: (83;89), (84;88), (85;87), (86;86), (87;85), (88;84) e (89;83). II) a = c = 9 90 + b + 9 + d = 100 b + d = 1 , obtendo outros 2 pares centenrios: (90;91) e (91;90). H, assim, 9 pares centenrios. 02. Seja J a interseo dos segmentos BC e FG. Como M ponto mdio do segmento BC, oposto ao vrtice E, conclui-se que EF dimetro, e FGE = BMF = 90 . Sendo ABCDE um pentgono regular, ABC = 108 . No GHI : GHI = GIH = 90 . No BJH : BHJ = BJH = 72 . No FJM : FJM = 72 JFM = 18 + . Para que os tringulos EFG e HIG sejam semelhantes, como 18 + , a nica possibilidade termos 90 = 18 + = 36 .
03. No momento do primeiro cruzamento, Esmeralda e Jade percorreram a

01. Sejam a, b, c e d algarismos tais que o par (ab, cd) centenrio. Ento,

distncia total igual metade da extenso da pista. Entre o primeiro e o segundo cruzamento, as moas percorreram uma distncia total igual extenso da pista. Portanto Esmeralda correu o dobro da distncia que correu at o primeiro cruzamento, ou seja, 2 200 = 400 metros e, deste modo, a extenso da pista 400 + 350 = 750 metros.
EUREKA! N26, 2007

26

Sociedade Brasileira de Matemtica

At o primeiro encontro

Entre o primeiro e o segundo encontro

04. Considere os trs planos que passam pelo centro do octaedro e contm 4 das

12 arestas do octaedro, formando trs quadrados. A seco corta no mximo dois lados de cada quadrado. Portanto corta no mximo 6 arestas do octaedro. Assim, a maior quantidade de lados que uma seco pode determinar no octaedro regular 6. Um exemplo de seco hexagonal um plano paralelo a duas faces opostas.

05. Seja n > 1 a quantidade de dados. Podemos representar um lanamento dos n dados com a n-upla (a1, a2, , an), sendo ai o resultado do dado i. Como ai um inteiro entre 1 e 6, existe uma bijeo entre os pares (a1, a2, , an) e (7 a1, 7 a2, , 7 an), de modo que a probabilidade de obter soma S = a1 + a2 + + an a mesma de obter soma (7 a1) + (7 a2) + + (7 an) = 7n S . Alm disso, para n + 1 S 7 n / 2 , a probabilidade de obter soma S 1 menor do que a probabilidade de obter soma S. Portanto as somas distintas S e T tm a mesma probabilidade de ocorrer se, e somente se, T = 7n S. Em particular, a nica soma com a mesma probabilidade de ocorrer que a soma 2006 7n 2006. Como 2006 = 334 6 + 2 , precisamos jogar, no mnimo, 335 dados, ou seja, n 335 . Pelo fato acima, o valor procurado 7 335 2006 = 339 .

Solues Nvel 3 Segunda Fase Parte B


SOLUO DO PROBLEMA 1:

Uma soluo: Observe que teremos 1 criana com 2 brinquedos, enquanto cada uma das n 1 crianas restantes ter apenas 1 brinquedo. Assim, temos n possibilidades para a
EUREKA! N26, 2007

27

Sociedade Brasileira de Matemtica

n + 1 escolha da felizarda criana, e 2 possibilidades para escolher os 2 brinquedos desta criana. Restando n 1 brinquedos e n 1 crianas, temos (n 1)! modos de distribuir estes brinquedos entre estas crianas. Assim, temos um n + 1 n + 1 total de n 2 ( n 1)! = 2 n! modos de distribuir os n + 1 brinquedos entre as n crianas.
Outra soluo: Observe que teremos 1 criana com 2 brinquedos, enquanto cada uma das n 1 crianas restantes ter apenas 1 brinquedo. Temos n escolhas para a criana que ter dois brinquedos. Escolhida tal criana, o nmero de maneiras de distribuir os n + 1 brinquedos igual ao nmero de anagramas da palavra A1A1A2A3An, que (n + 1)! (n + 1)! = . Assim, o total de maneiras de distribuir os n + 1 brinquedos 2! 2 (n + 1)! entre as n crianas n . 2
SOLUO DO PROBLEMA 2:

Uma soluo: ab + 1 (a + 1)(b + 1) ab + 1 ab + a + b + 1 ab + 1 (ab + a + b + 1) (ab + 1)


ab + 1 a + b ab + 1 a + b a (b 1) b 1 . Desta ltima desigualdade, observamos que, se b > 1 , ento a 1 a = 1 , ou seja, um dentre os inteiros a e b vale 1. Suponha, ento, sem perda de generalidade, que a = 1. Substituindo, obtemos a = 1 b + 1 2(b + 1) , o que vlido para todo inteiro positivo b. As solues so, ento, (1, b) e (a, 1).

Outra soluo: Como (a + 1)(b + 1) mltiplo de ab + 1, existe um inteiro positivo k tal que (a + 1)(b + 1) = k (ab + 1) a( kb b 1) = b k + 1 . Se kb b 1 = 0, ento 1 k = 1 + , que inteiro se, e somente se, b = 1. Se kb b 1 0 ento b b k +1 a= 1 b k + 1 kb b 1 k 2 . Se k = 1, obtemos a = (b + 1) kb b 1 < 0. Logo k = 2 e a = 1. Verifica-se que (1, b) e (a, 1) so realmente as solues.

EUREKA! N26, 2007

28

Sociedade Brasileira de Matemtica

SOLUO DO PROBLEMA 3:

Uma soluo: Sejam A o ortocentro do tringulo BCD e D o ortocentro do tringulo ABC.


C A

60o A B

Como as retas CD e BD so ambas perpendiculares a AB, so paralelas. Analogamente, as retas BD e CD so paralelas. Logo o quadriltero BDCD um paralelogramo e, portanto, os tringulos BCD e BDC so congruentes. Da mesma maneira, as retas AB e CA so paralelas, pois so perpendiculares a BD. Analogamente, as retas AC e BA so paralelas. Logo o quadriltero CABA um paralelogramo e, assim, os tringulos ABC e ACB so congruentes. Conseqentemente, os quadrilteros ABDC e ACDB so congruentes, de modo que a distncia entre os ortocentros AD igual a AD. D so ambos retos, D e AC Devemos, ento, calcular AD. Como os ngulos AB o somam 180 e, portanto, o quadriltero ABCD inscritvel, sendo AD dimetro de seu circuncrculo.
C

60o A 4 B

Pela lei dos co-senos,

1 BC 2 = AB 2 + AC 2 2 AB AC cos 60 BC 2 = 4 2 + 32 2 4 3 BC = 13 2 Enfim, pela lei dos senos, 13 2 39 BC = = AD = 2 R = 3 3 sen 60


2

EUREKA! N26, 2007

29

Sociedade Brasileira de Matemtica

e, portanto, a distncia entre os ortocentros

2 39 . 3

Outra soluo: Sejam A o ortocentro do tringulo BCD e D o ortocentro do tringulo ABC.


y C A

60o A B

Sejam A = (0;0) e B = (4;0). Sendo AC = 3 e m(BC) = 60o, podemos supor que C 3 3 3 . Como a reta CD perpendicular ao eixo x, = (3 cos 60 ;3 sen 60 ) = ; 2 2 3 admite equao x = . Alm disso, sendo a reta BD perpendicular reta AC, de 2 1 coeficiente angular tg 60 = 3 , seu coeficiente angular . Logo, sendo 3

1 5 3 3 a0 . D' = ; a , 3 = a= 6 2 2 4 3 Calculemos agora A. Como A pertence perpendicular a BD por C, ento 3 3 . A reta CD perpendicular a AC e, portanto, tem coeficiente A' = b; 2 1 angular . Enfim, sendo AB perpendicular a CD, tem coeficiente angular 3 1
1 3

= 3 . Deste modo,

3 3 2

b4
2

= 3b=

11 . 2

Logo a distncia entre os ortocentros A e D


2 3 3 5 3 11 3 = 2 39 . + 2 2 2 6 3

EUREKA! N26, 2007

30

Sociedade Brasileira de Matemtica

SOLUO DO PROBLEMA 4:

Os primeiros valores da seqncia so: F1 = 1, F2 = 1, F3 = 2, F 4 = 3, F 5 = 5, F 6 = 8, F 7 = 13, F 8 = 21, F 9 = 34 Nota-se que, para n > 7, Fn > 2n. De fato, indutivamente, se Fn > 2n e Fn + 1 > 2(n + 1) ento Fn + 2 = Fn + 1 + Fn > 2(n + 1) + 2n > 2(n + 2). 3n 4n > > n para n > 7, de modo que para resolver as Portanto Fn > 2n > 2 3 3n 4n , Fn = , basta testar os valores de n menores ou equaes Fn = n, Fn = 2 3 iguais a 7. Se n > 5, de Fm Fn = mn devemos ter Fm < m, donde m < 5. Logo pelo menos um dos nmeros m e n deve ser no mximo 5. Suponha, sem perda de generalidade, n 5. Observando os possveis valores de n: n = 1 Fm = m, cujas solues so m = 1 e m = 5. n = 2 Fm = 2m, que no possui soluo. n = 3 2Fm = 3m, que no possui soluo. n = 4 3Fm = 4m, que possui a nica soluo m = 6. n = 5 Fm = m, cujas solues so m = 1 e m = 5. Os pares (m, n) que satisfazem a relao pedida so: (1, 1), (1, 5), (4, 6), (5, 1), (5, 5) e (6, 4).

EUREKA! N26, 2007

31

Sociedade Brasileira de Matemtica

XXVIII OLIMPADA BRASILEIRA DE MATEMTICA


Problemas e Solues da Terceira Fase PROBLEMAS NVEL 1
PROBLEMA 1

Considere as seguintes seqncias:

S1: 12345678, 81234567, 78123456, ..., na qual o ltimo algarismo do termo anterior (algarismo das unidades) torna-se o primeiro algarismo esquerda do prximo termo. S2: 1234567898765, 5612345678987, 7856123456789, ..., na qual o algarismo das unidades torna-se o primeiro algarismo esquerda do prximo termo, e o das dezenas torna-se o segundo algarismo esquerda. a) Apresente o quinto termo da seqncia S1 e o quarto termo da seqncia S2. b) A seqncia S1 tem 2006 termos. Qual o seu ltimo termo? c) A seqncia S2 termina quando o primeiro termo se repete. Quantos termos tem essa seqncia?
PROBLEMA 2

Na adio abaixo, cada smbolo representa um nico algarismo e smbolos diferentes representam algarismos diferentes.

Determine o valor de cada smbolo, ou seja, descubra tais valores e mostre que no existem outras possibilidades.
PROBLEMA 3

Um atirador lana flechas no alvo representado ao lado. Os nmeros indicam a pontuao obtida em cada regio atingida pela flecha (se a flecha acertar exatamente uma linha, a pontuao a menor das duas regies). Note que a regio fora do retngulo no rende pontos.

EUREKA! N26, 2007

32

Sociedade Brasileira de Matemtica

a) Se numa competio, cada participante atira 2 flechas, quantas pontuaes diferentes podem ser obtidas? b) Numa outra competio, cada participante atirou 3 flechas. Curiosamente, no houve empates e todas as pontuaes possveis foram atingidas. Quantos participantes havia nesta competio?
PROBLEMA 4

Dentre os polgonos de 5 lados, o maior nmero possvel de vrtices alinhados, isto , pertencentes a uma nica reta, trs, como mostrado a seguir.

Qual a maior quantidade de vrtices alinhados que um polgono de 12 lados pode ter?

Ateno: alm de desenhar um polgono de 12 lados com o nmero mximo de vrtices alinhados, lembre-se de mostrar que no existe um outro polgono de 12 lados com mais vrtices alinhados do que este.
PROBLEMA 5

A partir do tabuleiro mostrado nas figuras abaixo e quatro peas, duas circulares cinzas e duas quadradas pretas, Esmeraldinho inventou o seguinte jogo: Inicialmente, as peas so colocadas no tabuleiro como mostra a figura 1.

Figura 1

EUREKA! N26, 2007

33

Sociedade Brasileira de Matemtica

A meta do jogo , aps um certo nmero de movimentos, trocar as peas de posio, chegando na situao mostrada na figura 2.

Figura 2

Cada movimento consiste em mover uma das quatro peas uma ou mais casas acima, abaixo, esquerda ou direita; todavia, tal pea no pode pular nenhuma pea que, eventualmente, esteja no caminho, ou ocupar uma casa onde j existe uma pea. Por exemplo, a pea marcada com A s pode se mover para alguma das casas destacadas em cinza.

Os movimentos dos crculos e dos quadrados so alternados. O jogo comea com um movimento de um dos quadrados.

Determine a menor quantidade total de movimentos necessrios para terminar o jogo. Mostre, passo-a-passo, atravs de desenhos, como movimentar as peas com esta quantidade de movimentos e prove que no possvel terminar o jogo com menos movimentos.

PROBLEMAS NVEL 2
PROBLEMA 1

Escrevemos, em fila, os nmeros 1, 2, 3, , n. A cada passo, tomamos os dois ltimos nmeros da fila anterior, escrevemos primeiramente o ltimo, depois o penltimo e, enfim, os outros n 2, na ordem em que aparecem. Por exemplo, para n = 12 obtemos 1, 2, 3, 4, 5, 6, 7, 8, 9, 10, 11, 12 12, 11, 1, 2, 3, 4, 5, 6, 7, 8, 9, 10 10, 9, 12, 11, 1, 2, 3, 4, 5, 6, 7, 8 ... Qual a menor quantidade de passos necessrios para escrevermos novamente os nmeros 1, 2, 3, , n, nessa ordem, quando (a) n = 2006? (b) n = 2005?
EUREKA! N26, 2007

34

Sociedade Brasileira de Matemtica

PROBLEMA 2

Veja o problema No. 4 do Nvel 1


PROBLEMA 3 PROBLEMA 4

Encontre todos os pares ordenados (x; y) de inteiros tais que x3 y3 = 3(x2 y2). Quantos subconjuntos {a, b, c} de trs elementos distintos de {1, 2, 3, , 100} so tais que b a mdia aritmtica de a e c (a < b < c)?
PROBLEMA 5

Seja ABC um tringulo acutngulo e H o seu ortocentro. Sejam M, N e R os pontos mdios de AB, BC e AH, respectivamente. Determine a medida do ngulo R se o ngulo AB C mede 70o. MN
PROBLEMA 6

Em um torneio de tnis de mesa (no qual nenhum jogo termina empatado), cada um dos n participantes jogou uma nica vez contra cada um dos outros. Sabe-se que, para todo k > 2, no existem k jogadores J1, J2, , Jk tais que J1 ganhou de J2, J2 ganhou de J3, J3 ganhou de J4, , Jk 1 ganhou de Jk, Jk ganhou de J1. Prove que existe um jogador que ganhou de todos os outros e existe um jogador que perdeu de todos os outros.

PROBLEMAS NVEL 3
PROBLEMA 1

Seja ABC um tringulo, P o p da bissetriz interna relativa ao lado AC e I seu incentro. Se AP + AB = CB, prove que API um tringulo issceles. Seja n um inteiro, n 3 . Definimos f(n) como a maior quantidade possvel de tringulos issceles cujos vrtices pertencem a algum conjunto de n pontos do plano sem trs pontos colineares. Prove que existem constantes positivas a e b tais que an2 < f(n) < bn2, para todo n inteiro, n 3 . Determine todas as funes f : R R tais que f ( xf ( y ) + f ( x) ) = 2 f ( x) + xy para todos x, y reais.
EUREKA! N26, 2007

PROBLEMA 2

PROBLEMA 3

35

Sociedade Brasileira de Matemtica

PROBLEMA 4

Um nmero inteiro positivo arrojado quando tem 8 divisores positivos cuja soma 3240. Por exemplo, o nmero 2006 arrojado porque seus 8 divisores positivos, 1, 2, 17, 34, 59, 118, 1003 e 2006, somam 3240. Encontre o menor nmero inteiro positivo arrojado.
PROBLEMA 5

Seja P um polgono convexo de 2006 lados. As 1003 diagonais ligando vrtices opostos e os 1003 segmentos que ligam os pontos mdios dos lados opostos so concorrentes, ou seja, todos os 2006 segmentos possuem um ponto em comum. Prove que os lados opostos de P so paralelos e congruentes.
PROBLEMA 6

O professor Piraldo participa de jogos de futebol em que saem muitos gols e tem uma maneira peculiar de julgar um jogo. Um jogo com placar de m gols a n gols, m n , dito equilibrado quando m f (n) , sendo f(n) definido por f(0) = 0 e, para n 1 , f (n) = 2n f ( r ) + r , onde r o maior inteiro tal que r < n e f (r ) n .

1+ 5 , prove que um jogo com placar de m gols a n, m n , est 2 equilibrado se m n e no est equilibrado se m n + 1 .
Sendo =

SOLUES NVEL 1
SOLUO DO PROBLEMA 1: BRUNO SILVA MUCCIACCIA (VITRIA ES)

S1: 12345678, 81234567, 78123456, 67812345, 56781234


a) 5 termo da seqncia S1: 56781234 4 termo da seqncia S2: 9878561234567

S2: 1234567898765, 5612345678987, 7856123456789, 9878561234567.


b) O ltimo termo da seqncia S1 : 45678123 Pois quando se aumenta de 8 em 8, a seqncia se repete, ento o 2001 nmero igual ao 1. E o 2006 nmero igual ao 6. c) Essa seqncia tem 43 termos, pois se percebe que quando as posies aumentam de 7 em 7, os nmeros mpares permanecem no lugar e os pares andam 2 casas para a direita, s quando o nmero par estiver na penltima
EUREKA! N26, 2007

36

Sociedade Brasileira de Matemtica

posio que ele anda trs casas; assim, como h 13 algarismos andando 5 vezes duas casas, os algarismos pares andam 10 casas para a direita, mas, no final da seqncia, andam trs casas, ento andando 6 7 vezes, eu repito a sequncia. Como comea no No.1, na seqncia h 6 7 + 1 termos igual a 43. (Pois a seqncia acaba quando o nmero se repete).
SOLUO DO PROBLEMA 2: SOLUO DA BANCA

ab bc + ca abc
Como abc < 3 99 = 297 a = 0, a = 1 ou a = 2. Como a + b + c c (mod 10) a + b 0 (mod 10) b + 1 0 (mod 10) ou b + 2 0 (mod 10), temos b = 8 ou b = 9 (no podemos ter a = b = 0). Como a + b + c + 1 = 10 a + b, temos c + 1 = 9a. Se a = 1, ento c = 8 Se a = 2, c = 17 (no possvel!). Logo, a nica soluo a = 1, b = 9 e c = 8.
SOLUO DO PROBLEMA 3: COLABORAO DE RGIS P. BARBOSA (FORTALEZA CE)

a) Vejamos se possvel obter uma mesma pontuao de dois jeitos diferentes: (a, b) e (c, d), com a b, c d e a + b = c + d . Se tivermos a = c, a + b = a + d b = d , e logo ( a , b ) (c , d ). Agora suponhamos sem perda de generalidade a > c. Teremos a a + b = c + d 2c, mas, se olharmos as pontuaes nas regies, podemos observar que a menor pontuao possvel maior que c 3c ou 3c + 1. No caso c = 0, 1 a 2c = 0. Absurdo! E se c > 0, 3c a 2c c 0, absurdo! Assim s tem um jeito para cada pontuao possvel (a, b) com a b, dada a soma S = a + b. Agora vamos cont-las. Se a = b, pode ser: ( 0, 0 ) ; (1,1) ; ( 3, 3 ) ; ( 9, 9 ) ; ( 27, 27 ) ; ( 81,81) . Se a > b, basta escolhermos um par de pontuaes distintas pois a ordem est definida. Temos 6 possibilidades para o primeiro nmero e 5 para o segundo (que no pode ser igual ao primeiro) e dividimos por 2 j que os pares so contados duas vezes (aparecem
EUREKA! N26, 2007

37

Sociedade Brasileira de Matemtica

tanto {0, 1} quanto {1, 0}), obtendo:

65 = 15 possibilidades. Assim so 6 + 2

15 = 21 pontuaes possveis. b) Usaremos um raciocnio parecido com o do item (a). Veja quando a mesma pontuao pode ser obtida de dois modos: ( a , b, c ) e ( d , e, f ), com a b c, d e f e a + b + c = d + e + f Se a = d recai no item (a): (b ; c); (e, f) com b c, e f e b + c = e + f j vimos que nesse caso b = e e c = f. Assim suponhamos a > d. Temos: a a + b + c = d + e + f 3d . Se d = 0,1 a 0. Absurdo! Assim tomemos d > 0. Se a > d j vimos que a 3d ; como a a + b + c = d + e + f 3d a ocorrero todas as igualdades: b = c = 0 e e = f = d, e conclumos que as nicas pontuaes obtidas de dois modos so: ( 3 x, 0, 0 ) e ( x, x, x ) com x > 0 e 3 x 81 x 27 j que a uma pontuao de uma flecha. Temos agora trs casos: (i) a > b > c : Temos que tomar 3 nmeros distintos, seguindo o raciocnio do item (a), teremos: 6 5 4 mas dessa vez cada trio aparecer 6 vezes: { x, y , z};{ x, z , y};{ y , x, z};{ y , z , x};{ z , x, y};{ z , y , x} que so tudo a mesma

6 54 = 20 possibilidades. 6 (ii) a > b = c ou a = b > c : Basta contar os pares e multiplicar por dois pois o par (a, b) gera (a, a, b) e (a, b, b). J vimos no item (a) que so 15 pares, assim temos aqui 30 possibilidades. (iii) a = b = c: Como deduzimos as somas obtidas por (x, x, x) com 0 < x 27 , que j foram contadas ento devemos contar apenas: x = 0 e x = 81, isto , somente mais 2 possveis somas. Assim, como o nmero de participantes igual ao nmero de somas possveis distintas, so: 20 + 30 + 2 = 52 participantes.
coisa, assim temos
SOLUO DO PROBLEMA 4

Ver soluo do problema 2 do nvel 2.

EUREKA! N26, 2007

38

Sociedade Brasileira de Matemtica

SOLUO DO PROBLEMA 5: LEONARDO BURATO FOUREAUX (LINHARES - ES)

Com 8 movimentos.

Resultado final

No existe outra maneira de mover as peas com menos movimentos, pois o mnimo de cada pea para chegar ao lugar da outra de 2 movimentos e sendo 4 peas, so no mnimo 4 2 = 8 movimentos.

SOLUES NVEL 2
SOLUO DO PROBLEMA 1: THIAGO S. WARWAR TEIXEIRA (RIO DE JANEIRO RJ)

a) n = 2006 Com n sendo par, a cada


n passos, partindo do incio, nota-se que todos os 2

nmeros mpares trocaro de posio com o nmero par a sua frente, depois todos os pares trocaro de posio com o nmero mpar que estiver a sua frente, e assim sucessivamente. Logo, com n = 2006, teremos, aps 1003 passos: 2, 1, 4, 3, 6, 5..., 2004, 2003, 2006, 2005
EUREKA! N26, 2007

39

Sociedade Brasileira de Matemtica

E, depois de mais 1003 passos, teremos: 1, 2, 3, 4, 5, 6, ..., 2003, 2004, 2005, 2006 (ordem inicial) Ou seja, para n = 2006, a menor quantidade de passos necessrios para reescrever: 1, 2, 3,..., 2005, 2006, de 2006 passos. b) n = 2005 Para resolver esta questo, recorreremos a exemplos menores: 1, 2, 3 3, 2, 1 1, 2, 3 2 passos 1, 2, 3, 4, 5 5, 4, 1, 2, 3 3, 2, 5, 4, 1 1, 4, 3, 2, 5 5, 2, 1, 4, 3 3, 4, 5, 2, 1 1, 2, 3, 4, 5 6 passos Com base nos exemplos, percebe-se que os nmeros mpares: 1, 3, 5... estaro sempre nas posies mpares: 1., 3., 5., ..., no necessariamente nesta ordem, conseqentemente, os pares estaro nas posies pares. Nota-se tambm que para os x nmeros mpares reestabelecerem sua ordem original, devero ser feitos x passos, e para os (x 1) nmeros pares, sero tomados (x 1) passos. Logo para achar o nmero mnimo de passos necessrios, devemos calcular o mnimo mltiplo comum entre x e (x 1), que independente do valor de x, ser sempre x ( x 1). Ento, para calcular o valor mnimo de passos para n= 2005, devemos multiplicar 1003 (quantidade de mpares) por 1002 (quantidade de pares), o que resulta em 1005006.
SOLUO DO PROBLEMA 2: ILLAN FEIMAN HALPERN (ITATIAIA RJ)

A maior quantidade de vrtices alinhados que um polgono de 12 lados pode ter 8. Um exemplo de polgono assim :

Mostrarei agora que no existe polgono de 12 lados com 9 vrtices colineares. Em um polgono 3 vrtices consecutivos no podem ser colineares, escolhendo 9 pontos em 12, pelo menos haver um trio de pontos consecutivos. Demostrao: Nomeie os vrtices em ordem de V1 a V12 , sendo que V1 consecutivo de V12 e

V2 ; V2 consecutivo de V1 e V3 e assim por diante. Sejam: G1 = {V1 ;V2 ;V3 }


EUREKA! N26, 2007

40

Sociedade Brasileira de Matemtica

G2 = {V4 ;V5 ; V6 } G3 = {V7 ; V8 ;V9 } G4 = {V10 ;V11 ;V12 } Pelo princpio da casa dos pombos, escolhendo-se 9 vrtices, haver pelo menos um conjunto com 3 vrtices escolhidos.
SOLUO DO PROBLEMA 3: COLABORAO DE RGIS P. BARBOSA (FORTALEZA CE)
3 3 2 2 2 2 Temos: x y = 3( x y ) ( x y )( x + xy + y ) = 3( x y )( x + y ).

x y = 0 x 3 y 3 = 0 e 3( x 2 y 2 ) = 0. Temos assim as solues ( x, y ) = ( a , a ), a Z. Agora suponhamos x y 0; assim podemos cortar logo: x 2 + xy + y 2 = 3 x + 3 y x 2 + ( y 3) x + ( y 2 3 y ) = 0, e logo = ( y 3) 2 4( y 2 3 y ) = y 2 6 y + 9 4 y 2 + 12 y = 3 y 2 + 6 y + 9 =
= 3( y + 1)( y 3)

Note que x Z 0 3( y + 1)( y 3) 0 ( y + 1)( y 3) 0. Analisando a inequao, se y < 1 ( y + 1) < 0, ( y 3) < 0 ( y +1)( y 3) > 0 e

y > 3 ( y 3) > 0 e ( y + 1) > 0 ( y + 1)( y 3) > 0. Assim os ys que buscamos satisfazem: 1 y 3, y Z y = 1, 0,1, 2, 3. Vamos verificar para cada um os possveis valores de x. 2 2 (I) y = 1 = 0 e a equao : x 4 x + 4 = 0 ( x 2) = 0, donde x = 2 ( x, y ) = (2, 1).
2 (II) y = 0 = 9 e a equao : x 3 x = 0 x ( x 3) = 0 , donde x = 0 ou x = 3 ( x, y ) = (0, 0), (3, 0). (III) y = 1 = 12 e a equao :

b 2 12 = x = 1 3. Assim, para y = 1, 2a 2 no existe x Z que satisfaz a equao do segundo grau. (IV) y = 2 = 9 e a equao : x2 2x 2 = 0 x =

b 1 3 = x = 2 ou x = 1 ( x , y ) = (2, 2), ( 1, 2). 2a 2 2 (V) y = 3 = 0 e a equao : x = 0 x = 0 ( x, y ) = (0, 3). x2 x 2 = 0 x =


EUREKA! N26, 2007

41

Sociedade Brasileira de Matemtica

Assim os pares ordenados so: ( x , y ) = (2, 1), ( 1, 2), (3, 0), (0, 3) e ( a , a ),

a Z.
SOLUO DO PROBLEMA 4: RAFAEL HORIMOTO DE FREITAS (SO PAULO SP)

A soma de A e C deve ser par pois a mdia entre A e C ,

A+C , resulta em um 2 inteiro B, para isso A e C devem ser nmeros pares ao mesmo tempo ou devem ser nmeros mpares ao mesmo tempo. No comeo podemos escolher 100 As diferentes; para cada A mpar, restam 49 mpares, e, se A for par, restam 49 pares para escolher no lugar de C, e por ltimo s h um nmero B para escolher pois s h uma mdia aritmtica entre A e C. Em metade dos casos ocorrer A > C, pois para cada casa em que A < C podemos trocar os valores de A e C, metade dos casos so invlidos. 100 49 1 = 2450 subconjuntos. No final temos 2
SOLUO DO PROBLEMA 5: RAFAEL GRIBEL DE PAULA NEVES (RIO DE JANEIRO RJ)

A
20

R M
20 +

20 70 70

H J w

w N

MR base mdia do ABH AM R ABH

MJ mediana relativa a AB (e AJB retngulo) B AJ AJ M MN base mdia do ABC BM N B AC


Seja K a interseo de BH e MN . No BKM , + 20 + = 90 NMR = 90
EUREKA! N26, 2007

42

Sociedade Brasileira de Matemtica

Os tringulos JNR e MNR so retngulos e dividem a mesma base NR o quadriltero MJNR inscritvel M J R M N R , donde M N R = 20 .
SOLUO DO PROBLEMA 6 PARTE A: RENAN HENRIQUE FINDER (JOINVILLE SC)

Inicialmente, vamos provar o teorema para n = 3. Chamemos de vitorioso o jogador que venceu os demais. Usemos a notao (XY) para (X venceu Y). Sejam A, B e C os trs jogadores. Suponhamos (sem perda de generalidade) (AB). Se (BC), ento, como no podemos ter (CA), (teramos J1 = A, J 2 = B e J 3 = C violando o enunciado), temos A vitorioso. Seno, temos que (CB), ou seja, o vencedor de A contra C vitorioso. Suponhamos agora que o enunciado valha para n jogadores. Em um torneio com os jogadores A1 , A2 , A3 ... e An +1 , haver um subtorneio entre os jogadores A1 , A2 , A3 ,... e An . Suponhamos sem perda de generalidade que seja A1 o vitorioso do subtorneio. Se A1 An +1 , A1 o vitorioso do torneio. Se An +1 A1 , podemos analisar ternas de jogadores em subtorneios para chegarmos a uma concluso.

Terna A1 , A2 , An +1

Concluso ( An +1 A1 A1 A2 ) An +1 A2
( An +1 A1 A1 A3 ) An +1 A3 ( An +1 A1 A1 An ) An +1 An

A1 , A3 , An +1 A1 , An , An +1

Conclumos que An +1 vitorioso. De qualquer modo, h um vitorioso. Assim, indutivamente, confirma-se o enunciado. Analogamente, conclui-se que h um jogador que tenha perdido todas as partidas. Obs. O teorema provado mais geral. Poderia ser enunciado como se um jogador A vencer B e B vencer C, impossvel C vencer A. Ento, h um jogador que vena todos os demais. Ele se torna at intuitivo se o enunciarmos assim: quando um jogador vence outro, que vence um terceiro, o primeiro vencer o terceiro. Para indicar o vencedor de uma disputa, vamos utilizar uma seta: J x J y (Isso significa que J x perdeu para J y ). A seta aponta para o vencedor.
EUREKA! N26, 2007

PARTE B: EDSON RYOKEI ONAGA (SO PAULO SP)

43

Sociedade Brasileira de Matemtica

Vamos supor que nenhum jogador perdeu todas as partidas. Assim, J1 ganhou, pelo menos 1 partida. O esquema dele ser assim: J1 J 2 Como J 2 tambm no perdeu todas, o esquema ficar assim: J1 J 2 J 3 ... Observe que nenhum jogador pode aparecer 2 vezes nessa seqncia. Vejamos o porqu: Supondo que J 2 aparea de novo no esquema: J1 J 2 J 3 J 4 J 2 Observe que ocorre a seguinte situao: J 2 ganhou de J 3 , J 3 ganhou de J 4 e J 4 ganhou de J 2 . Como o enunciado da questo no permite essa situao, no podemos repetir nenhum jogador na seqncia. Como n no infinito, essa seqncia finita. O nico modo de terminar o esquema se algum jogador no perder nenhuma partida, pois, aps um invicto, no poderemos colocar nenhum outro jogador. Logo, h um jogador que ganhou de todos os outros. Da mesma forma que a seqncia tem um fim direita, ela deve ter um fim esquerda. Pelas mesmas condies citadas acima, o ltimo esquerda do esquema ser o jogador que perder de todos os outros. H um jogador que perdeu de todos os outros.

SOLUES NVEL 3
SOLUO DO PROBLEMA 1: RENAN LIMA NOVAIS (RIO DE JANEIRO - RJ)

I) Desenhando a figura da questo, temos:


B

I C P II) Por ser o ponto I incentro, sabemos que este ponto eqidista dos trs lados do ABC , podendo-se inscrever um crculo no ABC . Alm disso, por ser o
EUREKA! N26, 2007

44

Sociedade Brasileira de Matemtica

ponto I o incentro do tringulo ABC , temos que AI bissetriz do ngulo

B AC . III) Observemos agora as alteraes da figura:


B

I A C

IV) Aplicando o teorema das bissetrizes internas no tringulo ABP , de bissetriz AI , temos:

BI

AB AP PI BI BI PI + BI Mas como dito no enunciado da questo que AP + AB = BC , logo temos: AB BI PB BC PB V) Assim, podemos notar que os tringulos ABI e CBP so semelhantes pelo
caso LAL de semelhana (pois

PI

AP

AB

AB

AP + AB

BC

AB

BI

AB BC

BI PB

e ABP P BC ).

Logo BCP B AI . VI) Observemos agora a figura novamente alterada:

I A
EUREKA! N26, 2007

P
45

Sociedade Brasileira de Matemtica

VII) Como B I A B PC , AI P suplemento de B I A e, conseqentemente, suplemento de B PC e B PA suplemento de B PC , logo AI P B PA. Assim, o tringulo AIP issceles.
SOLUO DO PROBLEMA 2: LCIO ASSAOKA HOSSAKA (CURITIBA PR)

Primero vamos provar que existe b. Podemos escolher dois pontos de n ( n 1) formas, e traar um segmento entre eles. Suponha agora que sobre cada 2 um deles haja dois tringulos issceles com base no segmento em questo. Mais de dois tringulos com base no mesmo segmento implicaria em 3 vrtices colineares, contidos na mediatriz do segmento. (Observe que a inteno no a de obter um nmero exato, e sim uma cota razovel). Assim, certamente n ( n 1) f (n) 2 = n ( n 1) = n 2 n, donde se ve que b = 1 suficiente, ou 2 seja, existe. No caso de a, vamos dividir em dois casos: n par e n mpar.

n mpar: Se arranjarmos os n pontos como vrtices de um polgono regular de n vrtices, podemos criar um valor mnimo que sabemos que no necessariamente ( n 1) 1 n . f(n), mas que menor ou igual a ele. Veremos que esse valor 2 3 n 1 Veja por que: para cada vrtice h pares de outros vrtices equidistantes, 2 que formaro a base de um tringulo issceles. So n vrtices, e multiplicamos 1 por para evitar a possvel contagem de tringulos mais de uma vez (como no 3 caso do enegono regular, por ex.). n2 n > an 2 , queremos que a seja tal que a inequao seja Resolvendo 6 1 1 n 3. Isso equivale a Como verdadeira para 1 > a. 6 n

1 1 1 2 1 1 n 3, 1 = , e logo qualquer a < serve. Ou seja a existe para 6 n 6 3 9 9 n mpar, pelo menos.

EUREKA! N26, 2007

46

Sociedade Brasileira de Matemtica

n par: anlogo ao caso anterior, com a diferena de que colocamos um ponto no centro do polgono regular, que agora tem n 1 vrtices. O nmero de possveis ( n 1)( n 2) ( n 1)( n 2) + > an 2 (a ltima parcela tringulos issceles de 6 2 corresponde aos que tem vrtice no ponto central). Isso equivale a 2 2 1 2 ( n 1)( n 2) > an 2 1 1 > a. 3 3 n n

2 1 2 2 3 1 1 1 Como n 4, 1 1 = , e logo qualquer a < serve. 3 n n 3 4 2 4 4 Existe a constante nesse caso tambm, finalizando a demonstrao.
SOLUO DO PROBLEMA 3: RAFAEL MENDES DE OLIVEIRA (RIO DE JANEIRO RJ)

Fazendo x = 1 na equao original, temos que: f ( f ( y ) + f (1)) = 2 f (1) + y f sobrejetora, pois repare que, fazendo y = a 2f(1), temos que f ( f ( a 2 f (1)) + f (1)) = a , a . Como f sobrejetora, R tal que f ( ) = 0. Se = 0, temos que, fazendo y = 0 na equao original, obteremos

f ( f ( x )) = 2 f ( x ). Como f sobrejetora, para todo a R xa tal que f ( xa ) = a. De f ( f ( x )) = 2 f ( x ), obtemos: f ( f ( xa )) = 2 f ( xa ) f ( a ) = 2 a , a R. Testando f ( x ) = 2 x na equao original, vemos que esta funo obviamente no soluo. Logo, podemos concluir que 0. Como 0, fazendo x = na equao original, obtemos: f ( f ( y )) = y. Logo, conclumos que f tambm injetora, pois f ( x) = f ( y) f ( f ( x)) = f ( f ( y)) y = x e como

0, y = x x = y (logo f injetora). Fazendo x = na equao original, obtemos:

f ( f ( y )) = y. Fazendo y = 0 em f ( x f ( y ) ) = y , temos que:

f ( f (0)) = 0. Como f ( ) = 0, temos que = f (0), pois f injetora. Logo, como 0, temos que f (0) = 1. Fazendo x = y = 0 na equao original, obtemos: f(1) = 2. Fazendo x = y = 1 na equao original, temos que f(1) = 0. Fazendo x = 1 na equao original, temos que:
EUREKA! N26, 2007

47

Sociedade Brasileira de Matemtica

f ( f ( y )) = y. Fazendo y = 0 na equao original temos: f ( x + f ( x )) = 2 f ( x ). Fazendo x := f (u ) e y := f (v ) na equao original, temos que: f ( f (u )v u ) = 2u + f (u ) f (v ). Fazendo u = 1 nesta ltima, temos que: f (2 y 1) = 2 f ( y ) 2 (*). Fazendo x = 1 na equao original, temos que f ( f ( y ) + 2) = y + 4.
De (*) temos: f (2 y 1) + 2 = 2 f ( y ). Aplicando f dos dois lados desta ltima igualdade, temos que f ( f (2 y 1) + 2) = f (2 f ( y )). Como f ( f ( x ) + 2) = x + 4 , temos que, fazendo x = 2y 1 nesta ltima, temos: f ( f (2 y 1) + 2) = 2 y + 3. Fazendo y = 0 na equao original, temos f ( f ( x ) + x ) = 2 f ( x ) (**). Logo, fazendo x = y nesta ltima e aplicando f dos dois lados, temos que f ( f ( f ( y ) + y )) = f (2 f ( y )). Fazendo y = 1 na equao original, temos que f ( f ( x )) = 2 f ( x ) x fazendo x = f ( y ) + y nesta ltima, temos:

f ( f ( f ( y ) + y )) = 2 f ( f ( y ) + y ) ( f ( y ) + y ). Como, por (**), f ( y + f ( y )) = 2 f ( y ), temos que a ltima igualdade fica: f ( f ( f ( y ) + y )) = 4 f ( y ) f ( y ) y = 3 f ( y ) y.


Como 2 y + 3 = f ( f (2 y 1) + 2) = f (2 f ( y )) = f ( f ( f ( y ) + y )) = 3 f ( y ) y , Temos que 3 f ( y ) y = 2 y + 3 f ( y ) = y + 1, y R. Testando na equao original, vemos que os dois lados ficam xy + 2x + 2 temos que f(x) = x + 1 a nica funo que satisfaz o problema. Resposta: f(x) = x + 1.
SOLUO DO PROBLEMA 4: BRENO VIEIRA DE AGUIAR (FORTALEZA CE) (interpretando que um nmero arrojado deve ter exatamente 8 divisores)

i) A quantidade de divisores positivos de um nmero, calculada pelo produto de cada expoente dos seus fatores primos mais um. Da, como o nmero tem exatamente 8 divisores, ele pode ser das formas: Seja N inteiro positivo arrojado que procuramos: I) N = p7; p primo 3 II) N = p q ; p , q primos distintos III) N = p q t ; p , q , t primos distintos
EUREKA! N26, 2007

48

Sociedade Brasileira de Matemtica

ii) Analisemos cada caso:


7 I) Se N = p , ento:

D( N ) = 1, p, p 2 , p 3 , p 4 , p 5 , p 6 , p 7 1 + p + p 2 + p 3 + p 4 + p 5 + p 6 + p 7 = 3240.
Note que:

1 + p + p 2 + p3 + p 4 + p5 + p6 + p7 =

p8 1 p 1

que

para

p8 1 3280 , j que 1 + p + p 2 + p 3 + p 4 + p 5 + p 6 + p 7 p 1 crescente. Da, p 3 no serve. Para p = 2, temos: p 3 1 + p + p 2 + p 3 + p 4 + p 5 + p 6 + p 7 = 255 p = 2 no serve!


7 Logo N = p no serve.

II) N = p q 3 D(n) = 1, p, q, pq, q 2 , p q 2 , q 3 , p q 3 1 + p + q + p q + q 2 + pq 2 +

+ q 3 + pq 3 = 3240 ( p + 1)(1 + q + q 2 + q 3 ) = 3240. Perceba que estamos atrs do menor inteiro arrojado > 0 e j temos que 2006 um inteiro arrojado. Da, N 2006 .

p q 3 2006 q 3 1003 q 3 1003 < 11 q = 2 ou 3 ou 5 ou 7.


Para q = 2 ( p +1) (1+ 2 + 22 + 23 ) = 3240 ( p +1) 15 = 3240 p + 1 = 816 p = 215 no serve. Para q = 3 ( p +1) (1+ 3 + 32 + 33 ) = 3240 ( p +1) 40 = 3240 p +1 = 81 p = 80 no serve.
2 3 Para q = 5 ( p +1) (1+ 5 + 5 + 5 ) = 3240 ( p + 1) 156 = 3240 p Z

p2

no serve.
2 3 Para q = 7 ( p +1) (1+ 7 + 7 + 7 ) = 3240 ( p +1) 400 = 3240 p Z

no serve.
3 Logo Para N = p q no serve.

III) N = p q t D( N ) = 1, p, q, t, pq, qt, tp, pqt 1+ p + q + t + pq + qt + tp + pqt = 3240

( p +1)(q +1)(t +1) = 3240. Ento temos que achar trs nmeros posteriores a trs
EUREKA! N26, 2007

49

Sociedade Brasileira de Matemtica

primos, tal que o produto desses nmeros 3240. Ento esses primos so divisores positivos de 3240 menos um. Ento vamos ver quais so os D(3240)1 que so primos:
3240 1620 810 405 135 45 15 5 1 2 2 2 3 3 3 3 5 1 2 4 8 3,6,12,24 9,18,36,72 27,54,108,216 81,162,324,648 5,10,20,40,15,30,60,120, 45, 90, 180, 360, 135, 270, 540, 1080, 405, 810, 1620, 3240.

D(3240) 1 = 0,1,2,3,4,5,7,8,9,11,14,17,19,23,26,29,35,39,44,53,59,71,80,89,107, 119,134,161,179,215,269,... Note que no mnimo (p + 1)(q + 1) 12 (quando p = 2 e q = 3), ento no mximo t + 1 270 tmx = 269. Os primos possveis so: 2, 3, 5, 7, 11, 17, 19, 23, 29, 53, 59, 71, 89, 107, 179, 269. Para t = 269 (p + 1)(q +1) = 12 p = 2 e q = 3 N = 1614 Para t = 179 (p + 1)(q +1) = 18 p = 2 e q = 5 N = 1790 Para t = 107 (p + 1)(q +1) = 30 p, q primos Para t = 89 (p + 1)(q +1) = 36 p = 2 e q = 11 N = 1958 Para t = 71 (p + 1)(q +1) = 45 p, q primos Para t = 59 (p + 1)(q +1) = 54 p = 2 e q = 17 N = 2006 Para t = 53 (p + 1)(q +1) = 60 p = 2 e q = 19 N = 2014 Para t = 29 (p + 1)(q +1) = 108 p = 5 e q = 17 N = 2465 Para t = 23 (p + 1)(q +1) = 135 p, q primos Para t = 19,17, 11, 7, 5, 3 ou 2 anlogo aos anteriores. Da, o menor N 1614. iii) Ento o menor inteiro positivo arrojado 1614.
SOLUO DO PROBLEMA 4: PEDRO PAULO GONDIM CARDOSO (SALVADOR BA) (Interpretao que um nmero arrojado pode ter mais que 8 divisores)

Inicialmente observa-se que 1260 um nmero arrojado, pois 1260 + 630 + 420 + 315 + 252 + 210 + 90 + 63 = 3240. Agora deve-se provar que no h nenhum nmero arrojado menor que 1260. Um nmero arrojado tem como divisores a, b, c, d, e, f, g, h tais que 1 1 1 1 1 1 1 1 + + + + + + + K = 3240, onde K o valor do nmero a b c d e f g h
EUREKA! N26, 2007

50

Sociedade Brasileira de Matemtica

arrojado. Se existir um nmero arrojado menor que 1260, devem existir naturais no nulas a, b, c, d, e, f, g, h tais que 1 1 1 1 1 1 1 1 3240 18 + + + + + + + > = = 2, 571428. a b c d e f g h 1260 7 Se existir um nmero arrojado menor que K, ele no poderia ser mpar, seno os menores valores para a, b, c, d, e, f, g, h seriam 1, 3, 5, 7, 11, 13, 15 e 17 e 1 1 1 1 1 1 1 3240 1 + + + + + + + < 2,3 < . Se existisse um nmero arrojado 3 5 7 9 11 13 15 1260 menor que K, ele teria que ser mltiplo de 3, seno as menores valores para a, b, c, d, e, f, g, h seriam 1, 2, 4, 5, 7, 8, 10, 11 e 1 1 1 1 1 1 1 3240 1 + + + + + + + < 2, 5 < . Ento, se existisse um nmero 2 4 5 7 8 10 11 1260 arrojado menor que 1260, ele teria que ser mltiplo de 6. O menor valor possvel para K corresponde aos menores valores possveis para a, b, c, d, e, f, g, h, que so 1, 2, 3, 4, 5, 6, 7, 8. Ento: 1 1 1 1 1 1 1 1 + + + + + + + K = 3240 g h a b c d e f
2283 1 1 1 1 1 1 1 1 K = 3240 + + + + + + + K = 3240 840 1 2 3 4 5 6 7 8 3240 840 K = K 1191,1. 2283 Se houver um nmero arrojado menor que 1260, ele deve ser maior que 1191, 1 e mltiplo de 6. Ento as nicas possibilidades para K so 1194, 1200, 1206, 1212, 1218, 1224, 1230, 1236, 1242, 1248 e 1254; Se K = 1194. A soma dos oito maiores (e nicos) divisores de 1194 1194 + 597 + 398 + 199 + 6 + 3 + 2 + 1 = 2400 < 3240. Portanto 1194 no arrojado. Se K = 1200. A soma dos oito maiores divisores de 1200 1200 + 600 + 400 + 300 + 240 + 200 + 150 + 120 = 3210 < 3240. Portanto 1200 no arrojado. Se K = 1206. A soma dos oito maiores divisores de 1206 1206 + 603 + 402 + 201 + 134 + 67 + 18 + 9 = 2640 < 3240. Portanto 1206 no arrojado. Se K = 1212. A soma dos oito maiores divisores de 1212 1212 + 606 + 404 + 303 + 202 + 101 + 12 + 6 = 2846 < 3240. Portanto 1212 no arrojado. Se K = 1218.
EUREKA! N26, 2007

51

Sociedade Brasileira de Matemtica

A soma dos oito maiores divisores de 1218 1218 + 609 +406 + 203 + 174 + 87 + 58 + 42 = 2797 < 3240. Portanto 1218 no arrojado. Se K = 1224. A soma dos oito maiores divisores de 1224 1224 + 612 + 408 + 306 + 204 + 153 + 136 + 102 = 3145 < 3240. Portanto 1224 no arrojado. Se K = 1230. A soma dos oito maiores divisores de 1230 1230 + 615 + 410 + 246 + 205 + 123 + 82 + 41 = 2952 < 3240. Portanto 1230 no arrojado. Se K = 1236. A soma dos oito maiores divisores de 1236 1236 + 618 + 412 + 309 + 206 + 103 + 12 + 6 = 2902 < 3240. Portanto 1236 no arrojado. Se K = 1242. A soma dos oito maiores divisores de 1242 1242 + 621 + 414 + 207 + 138 + 69 + 54 + 46 = 2791 < 3240. Portanto 1242 no arrojado. Se K = 1248. A soma dos oito maiores divisores de 1248 1248 + 624 + 416 + 312 + 208 + 156 + 104 + 96 = 3164 < 3240. Portanto 1248 no arrojado. Se K = 1254. A soma dos oito maiores divisores de 1254 1254 + 627 + 418 + 209 + 114 + 66 + 57 + 38 = 2783 < 3240. Portanto 1254 no arrojado. Como a soma dos oito maiores divisores menor que 3240, em todas as possibilidades, evidente que a soma de quaisquer outros oito divisores tambm ser menor que 3240. Portanto no h nenhum inteiro positivo menor que 1260 que seja arrojado. Logo o menor nmero inteiro positivo arrojado 1260.
SOLUO DO PROBLEMA 5: LEANDRO FARIAS MAIA (FORTALEZA CE)

Vamos dividir em duas partes: Parte 1: Os lados opostos de de P so paralelos. Seja AB um lado de P e ABseu lado oposto. Suponha que AB no seja paralelo a BA. Por B, trace uma paralela ao lado AB, at trocar AA em C. Sendo M e N pontos mdios dos lados AB e AB, respectivamente, temos: B X XT XC AB // BC = = B X = XC BM TM AM

X B N

C A

EUREKA! N26, 2007

52

Sociedade Brasileira de Matemtica

B X = XC NX // CA, absurdo ( NX CA= T ). Mas veja que: BN = NA Portanto, devemos ter: AB // AB. A1 Parte 2: A2006 AB = AB. Numerando os vrtices, temos que: A1003 + i oposto de Ai ,1 i 1003 Temos: A1 A2 // A1004 A1005 , A2 A3 // A1005 A1006 ,..., A1003 A1004 // // A2006 A1 .
Logo:
A1006 A1005

A2 A3

A1003 A1004

AT A T A T A T A1T AT AT = 2 = 3 = ... = 1003 = 1004 1 = 1004 A1T = TA1004 . TA1004 TA1005 TA1006 TA2006 TA1 TA1004 TA1 Portanto teremos: AT A T AT AT 1 = 1 = 2 = 3 = ... = 1003 TA1004 TA1005 TA1006 TA2005
Mas:

Ai T Ai Ai +1 = = 1 Ai Ai +1 = A1003 + i A1004 + i , o que acaba, pois TA1003 + i A1003 + i A1004 + i

Ai Ai +1 e A1004 + i so lados opostos.


SOLUO DO PROBLEMA 6: JOS MARCOS ANDRADE FERRARO (SO PAULO - SP)

Vamos listar os primeiros termos para estabelecer uma base de induo.

n f(n)

0 0

1 2

2 3

3 5

4 7

5 8

6 10

7 11

8 13

9 15

10 16

11 18

Lema: n = f (r ) ou n = f (r ) + 1. Prova: equivalente a f (r + 1) = ( f (r ) + 1 ou f (r ) + 2). Suponha que isto acontea para r < n 1. Ento f ( n) = 2n f ( r) + r, f (n 1) = 2n 2 f (r ) + r f (n) f (n 1) = 2 f (r) + r+ f (r ) r.
EUREKA! N26, 2007

53

Sociedade Brasileira de Matemtica

Temos r= r ou r= r + 1 , n = f (r ) ou n = f (r ) + 1 e ( f (r) = f (r ) + 1 ou f (r) + 2). Se r= r , f (n) f (n 1) = 2. Se r= r + 1, f (n) f (n 1) = 3 + f (r ) f (r) = (2 ou 1), o que termina a prova do Lema. Vamos agora provar por induo que k 1 < f (k ) < k + 1, para todo k . Suponha que isso vale para 0 k n 1. n +1 n 1 Se n = f (r ), r 1 < n < r + 1 >r> f ( n) = 2n f ( r ) + r = 2n n + r = n + r n +1 n 1 n+ n+r n+ 1 1 1 n + > f (n) n como 1 > , n + 1 > f (n) > n 1. n n2 Se n = f (r ) + 1, r 1 < n 1 < r + 1 > r > f ( n ) = 2 n n + 1 + r f ( n) = n + 1 + r n n2 n +1+ > n +1+ r > n +1+ 2 n + 1 > f (n) n + > n 1, cqd. Assim, n + 1 > f (n) > n 1, n 0. Como f (n) , f (n) > n 1 e n 1 Se

, temos f (n) n 1 +1 = n . n .
Assim

m n, m < n

pois

m n

da

f ( n) n m. Logo se m n, f (n) m, e portanto o jogo equilibrado. Por outro lado, se m > n + 1 > f (n), o jogo desequilibrado, cqd.

EUREKA! N26, 2007

54

Sociedade Brasileira de Matemtica

XXVIII OLIMPADA BRASILEIRA DE MATEMTICA Problemas e Solues da Primeira Fase Nvel Universitrio
PROBLEMA 1

Calcule

ex 1 x (e x 1) x dx 1

PROBLEMA 2

Seja N um inteiro positivo. Calcule, em funo de N, o volume do slido definido por: x, y, z [0, +) x + y + z N
PROBLEMA 3

Dada

f:

[ 0, + )

duas

vezes

diferencivel

com f (0) = 0 , f '(0) = 1 ,

1 + f ( x) =

3 1 , x [0,1] , mostre que f (1) < . 2 f "( x)

PROBLEMA 4

Dada uma hiprbole e uma reta no paralela s assntotas, determine o lugar geomtrico dos pontos mdios das cordas da hiprbole paralelas reta dada.

Obs: Uma corda de uma hiprbole um segmento cujos extremos pertencem hiprbole.
PROBLEMA 5

As funes y1 (t ) = (1 + t 2 ) et , y2 (t ) = (t + t 2 ) et e y3 (t ) = (1 t + t 2 ) et so solues da equao diferencial y"(t ) + a(t ) y '(t ) + b(t ) y (t ) = c(t ) , onde a (t ), b(t ), c(t ) so funes duas vezes diferenciveis. Determine uma funo duas vezes diferencivel y (t ) tal que y"(t ) + a(t ) y '(t ) + b(t ) y (t ) = c(t ) , y (0) = 0, y '(0) = 0 .
PROBLEMA 6

Escolha trs pontos x1, x2, x3 aleatoriamente, independentemente e com distribuio uniforme no intervalo [0, 1]. Determine, em funo do nmero positivo m, a probabilidade de que min{| x1 x2 |,| x1 x3 |,| x2 x3 |} > m .
EUREKA! N26, 2007

55

Sociedade Brasileira de Matemtica

SOLUO DO PROBLEMA 1:

Seja

f ( x) =

ex 1 x . Temos (e x 1) x

f ( x) =

e x 1 + x xe x e x + 1 = , logo (e x 1) ( x) (e x 1) x

f ( x) + f ( x) =
Assim,

xe x x = 1. (e x 1) x
1 1 0 0

f ( x)dx = ( f ( x) + f ( x))dx = 1dx = 1 .

O slido a unio dos cubos unitrios [i, i +1) [j, j + 1) [k, k + 1) para i, j, k inteiros no negativos, i + j + k N. Como cada cubinho tem volume 1, o volume do slido igual ao nmero de triplas (i, j, k) como acima. N + 3 O nmero de triplas (e portanto o volume) igual a = (N + 1)(N + 2)(N + 3 3)/6. Isto pode ser demonstrado de vrias formas. Por exemplo, podemos pensar que temos uma fileira de N + 3 quadrados e vamos escolher 3 posies e botar um marcador em cada uma delas: i ser o nmero de quadrados antes do primeiro marcador, j o nmero de quadrados entre o primeiro e o segundo marcadores e k o nmero de quadrados entre o segundo e o terceiro marcadores. Claramente, a cada configurao corresponde uma tripla e vice-versa.
SOLUO DO PROBLEMA 3:

SOLUO DO PROBLEMA 2:

Como f(x) 0 para todo x R, temos 0 <

1 = f (x) , logo f crescente em 1+ f (x)

[0,1], o que implica que f(x) > 1 para x > 0, ou seja f tambm crescente em [0,1]. Assim, para x > 0, temos 0 <

1 1 = f ( x) = < 1 , logo 1 + f ( x) 1 + f ( x)
0 < f ( x) f (0) < x 1 < f ( x) < x + 1

0dt < f (t )dt < 1dt


0 x 0 x 0
x

1dt < f (t )dt < (t + 1)dt x < f ( x) <


0 0 0

x2 3 + x . Em particular, f (1) < . 2 2

EUREKA! N26, 2007

56

Sociedade Brasileira de Matemtica

PRIMEIRA SOLUO DO PROBLEMA 4:

Seja r a reta dada, no paralela s assntotas. Considere o plano euclidiano como subconjunto do plano projetivo, da maneira usual. Seja P o ponto de r sobre a reta do infinito. O feixe de retas paralelas a r corresponde ao feixe de retas do plano projetivo que passam por P. Para cada uma das retas do feixe, que corta a hiprbole em dois pontos A e B, o ponto mdio M do segmento AB corresponde ao conjugado harmnico de P em relao a A e B. Logo M pertence reta polar de P em relao hiprbole. Seja p essa reta polar. Como o plo da reta do infinito o centro O da hiprbole, conclumos que p passa por O. H, portanto, dois casos a considerar: 1) Se existe uma tangente hiprbole paralela reta r, com ponto de tangncia T (e portanto existir uma outra tangente paralela a r no ponto T, simtrico de T em relao a O), o lugar geomtrico a reta OT menos o segmento TT . 2) Caso contrrio, o lugar geomtrico uma reta completa passando por O, que pode ser obtida traando-se uma corda arbitrria paralela a r (neste caso toda reta paralela corta a hiprbole em dois pontos distintos, um em cada ramo da hiprbole) e unindo seu ponto mdio a O.
SEGUNDA SOLUO:

Aps uma mudana de coordenadas afins, podemos considerar que a hiprbole tem equao xy = 1. Sendo m o coeficiente angular da reta r, queremos determinar o lugar geomtrico dos pontos mdios das interseces das retas de equaes y = mx + t ( t R) com a hiprbole. Sejam (x1, y1) e (x2, y2) esses pontos de interseco. Ento x1 e x2 so as razes da equao x(mx + t) = 1 mx2 + tx 1 = 0 (1). Logo a abscissa do ponto mdio igual a
1 1 t + y1 + y2 x1 x2 x1 + x2 t t , e sua ordenada vale = = = 2m = . Logo o ponto 2 2 2 2 x1 x2 4 2m m

mdio pertence reta de equao y =

m x . Reciprocamente, um ponto dessa 2

reta pertence ao lugar geomtrico desde que a equao (1) tenha duas razes reais, ou seja, quando t 2 + 4m > 0 . Assim, se m > 0, o lugar geomtrico toda a reta de equao y =

m x . Quando m < 0, desta reta devem ser retirados os pontos para 2


m m x . m m

os quais 2 m t 2 m , ou seja, para os quais

EUREKA! N26, 2007

57

Sociedade Brasileira de Matemtica

SOLUO DO PROBLEMA 5:

Podemos tomar y1 como soluo particular e y2 y1 e y3 y1 como solues linearmente independentes da equao homognea associada y ''(t ) + a (t ) y '(t ) + b(t ) y (t ) = 0 . Assim a soluo geral da equao y = c1 y1 + c2 y2 + c3 y3, c1 + c2 + c3 = 1 ou, equivalentemente,

y(t) = (c4 + c5 t + t2) e . Temos y(0) = c4 e y(0) = c5, donde as condies do enunciado nos do

t2

y = t 2 et .
SOLUO DO PROBLEMA 6:

Seja X=[0, 1]3. Temos X = A1 A2 A3 A4 A5 A6 , onde

A1 = {( x, y , z ) X | x y z} , A2 = {( x, y, z ) X | x z y} , A3 = {( x, y, z ) X | y x z} , A4 = {( x, y, z ) X | y z x} ,

A5 = {( x, y, z ) X | z x y} e A6 = {( x, y, z ) X | z y x} . Os conjuntos Ak ,1 k 6 tm todos volume 1/6.


Seja agora Y = {( x1 , x 2 , x3 ) X | min{| x1 x 2 |, | x1 x3 |, | x 2 x3 |} > m} . Temos Y = B1 B2 B3 B4 B5 B6 , onde Bk = Y Ak ,1 k 6 . Todos os conjuntos Bk ,1 k 6, tm o mesmo volume. Vamos ento calcular o volume do conjunto B1 . Como X tem volume 1, a probabilidade desejada ser

6 vol ( B1 ) . Temos B1 = {( x, y , z ) X | y > x + m, z > y + m} . Claramente, se m 1 / 2 , B1 vazio, e portanto a probabilidade desejada 0 para todo m 1 / 2 . Suponha agora 0 m 1 / 2 . Considere a translao f : B1 X dada por f ( x, y, z ) = ( x, y m, z 2m) . Temos f ( B1 ) = {( x, y, z ) [0,1 2m]3 | x < y < z} , e portanto f ( B1 ) tem o
mesmo volume de {( x, y, z ) [0,1 2m]3 | x y z} = g ( A1 ) , onde g a g ( p) = (1 2m) p . homotetia dada por Assim, temos

vol ( B1 ) = vol ( f ( B 1 )) = vol ( g ( A1 )) = (1 2m) 3 vol ( A1 ) = (1 2m) 3 / 6 , logo, para 0 m 1 / 2 , a probabilidade desejada igual 6 vol ( B1 ) = (1 2m) 3 .
EUREKA! N26, 2007

e a

58

Sociedade Brasileira de Matemtica

XXVIII OLIMPADA BRASILEIRA DE MATEMTICA Problemas e Solues da Segunda Fase Nvel Universitrio
PROBLEMA 1:

Seja f :

uma funo integrvel e crescente. Prove que

1 xf ( x)dx f ( x)dx 20 0
PROBLEMA 2:

Prove que, para todo inteiro n 2 , o nmero de matrizes quadradas 2 2 com entradas inteiras e pertencentes ao conjunto {0, 1, 2, , n 1} que tm 1 determinante da forma kn + 1 para algum k inteiro dado por n3 (1 2 ) . p p primo
p|n

PROBLEMA 3:

Uma mesa de bilhar tem o formato de elipse e no tem caapas. Quando uma bola bate em um ponto P na borda da mesa, ela segue uma direo simtrica em relao reta normal elipse em P. Prove que se uma bola parte de um ponto A da elipse e, aps bater na mesa nos pontos B e C, retorna a A, ento ela bater novamente em B.
PROBLEMA 4:

Seja p um polinmio irredutvel em Q[x] de coeficientes racionais e grau maior do que 1. Prove que se p admite duas razes r e s cujo produto 1 ento o grau de p par.
PROBLEMA 5:

Seja f :[0, +) [0, +) uma funo crescente e bijetora. Prove que a srie

n =1

1 converge se, e somente se, a srie f ( n)

n =1

( n)
2

converge, sendo f

funo inversa de f.
PROBLEMA 6:

Considere as matrizes

1 2 1 0 e B= A= 0 1 2 1
EUREKA! N26, 2007

59

Sociedade Brasileira de Matemtica

Prove que, para n > 1, no existem inteiros a1 , a2 , a3 ,, an e b1 , b2 ,, bn 1 , bn com a2 , a3 ,..., an e b1 , b2 ,..., bn 1 no nulos tais que

A a1 B b1 A a2 B b2 A an B bn = I ,
onde I a matriz identidade de ordem 2.
SOLUO DO PROBLEMA 1: LEVI MXIMO VIANA (FORTALEZA CE)

Chame

F ( x) = tf (t )dt e
0

G ( x) = f (t )dt , x [0,1].
0

Veja

ento

que

F( x) = xf ( x) = xG( x), mas como f ( x) = G( x) crescente, temos f ( x) f ( x 2 ),


2 j que 0 x 1, logo F( x) xG( x ) =

G( x2 ) ' . Integrando de 0 a 1 temos: 2 2 1 1 G( x ) ' G (12 ) G (02 ) G (1) ( ) (1) (0) F x dx dx F F F (1) 0 0 2 2 2 1 1 1 xf ( x)dx f ( x)dx . cqd. 0 2 0

SOLUO DO PROBLEMA 2: MURILO VASCONCELOS ANDRADE (MACEI AL)

Primeiramente provaremos que o enunciado vale para n primo:

a b uma matriz cujo determinante da forma k n + 1, temos c d ento que a d b c 1(mod n). Seja h(i) o nmero de pares ordenados de inteiros no conjunto
Seja n primo e produto igual a i(mod n). Desta maneira, temos que o nmero de matrizes satisfazendo as condies do enunciado :

{0,1, 2,..., n 1} cujo

h(i) h(i + 1)
i =0

n 1

(Onde cada h(i) representa o nmero de escolhas possveis para b e c, tais que bc i (mod n) e h(i + 1) representa o nmero de escolhas para a e d, tais que ad i + 1(mod n). Naturalmente h(0) = 2n 1 (os possveis pares so (0,0), (0, 1), ...,(0, n 1), (1, 0), (2, 0),..., (n 1, 0).
EUREKA! N26, 2007

60

Sociedade Brasileira de Matemtica

Alem disso, h(i ) = (n), para 1 i n 1 (aqui representa a funo de Euler, que associa a cada inteiro positivo no nmero de inteiros menores que n e que so primos com n. No caso de n primo, (n) = n 1). Vamos provar isto: Seja k primo com n. Vamos provar que existe k tal que k k i (mod n) : a seqncia

(k

mod n ) assume um nmero finito de valores (entre 0 e n 1). k ( ik n2 n1 1 ) = i


e
n2 i =1

n n Existem ento dois nmeros iguais na seqncia digamos k 1 = k 2 com

n2 > n1 ) .

Assim,
n 1

portanto

h ( i ) (n).

Como

( 2n 1) + (n) (n 1) = n2 ,
i. Assim,
r =0

segue que no podemos ter h(i ) > (n) para algum

h(i) h(i + 1) = (2n 1)(n 1) + (n 1) (n 1) + (n 1)(2n 1) =


1 1- 2 p p primo

= (n 1) [ (n 1)(n 2) + 4n 2] = n(n 1)(n + 1) = n3

pn

O resultado ento fica provado para n primo.


k Vamos agora mostrar por induo que o resultado vale para n = p potncia de

k 1 primo. Para k = 1 j foi provado. Suponha que k 2 e que vale para p .

a b pk 1 n1 + a pk 1 n2 + b , com ad bc 1(mod p k ). Seja n = p e = k 1 k 1 c d p n3 + c p n4 + d


k

adbc 1(mod pk 1 ).
2k 2 0(mod pk ), Assim, como p

ad bc pk 1 (n1 d+n4 abn3 n2 c) + adbc pk 1 (n1d+n4 abn3 n2c) +

+ pk 1 +1 1(mod pk ) n1d+n4 abn3 n2c+ 0(mod p)

EUREKA! N26, 2007

61

Sociedade Brasileira de Matemtica


k 1 (aqui tal que p +1 = adbc ).

3(k 1) k 1 Como adbc 1(mod p ) existem (por hiptese de induo), p 1

1 p2

maneiras de escolhermos a, d, b e c. Fixando-se estes valores (e portanto


k tambm) temos que um deles primo com n = p , pois, caso no fosse assim,

adbc 1(mod p), absurdo! Seja (a, n) = 1, por exemplo. Existem ento p valores possveis (mdulo p) para cada um dos n1, n2 , n3 e para cada combinao
destes, apenas um valor para n4 tal que n1d+n4abn3 n2c+ 0(mod p) .

Existem ento ao todo

1 1 p3(k 1) 1 2 p p p = p3k 1 2 maneiras de p p

escolhermos a, b, c, d. Isto termina nossa prova por induo.


a a a a Para n = p11 p22 ... pk k fcil ver que ad bc 1(mod n) ad bc 1(mod pi i ),

i {1,2,..., k} e portanto o nmero de matrizes que satisfazem as condies do


enunciado igual a

p
i= j

3ai i

1 3 1 1 2 = n 1 2 , cqd. p p primo pi
pn

SOLUO DO PROBLEMA 3: EDUARDO POO (SO PAULO SP)

AB e BC tm o mesmo ngulo com a normal em B tm o mesmo ngulo com a tangente elipse em B

B A CB B A CB VT = VT + VT = 0 , sendo VT o vetor B A CB B A CB
tangente elipse em B. Parametrizando a elipse:
P(t ) = ( k cos t ,sen t ) , 0 t 2 , sem perda de
d P (t ) = ( k sen t , cos t ) . dt

generalidade. Vetor tangente no ponto P (t ) : V (t ) =

EUREKA! N26, 2007

62

Sociedade Brasileira de Matemtica

Sejam P (a) = A , P(b) = B , P(c) = C . As hipteses sobre refletir de AB para BC e sobre refletir de BC para CA se tornam:

B A CB + V (b) = 0 B A C B

C B AC + C B AC

V (c ) = 0

Temos que provar o seguinte, que CA reflete em AB:

AC B A + V (a) = 0 AC B A
Distribuindo o produto vetorial (vamos denotar como um escalar, pois todos os vetores em jogo esto no mesmo plano e os produtos vetoriais tero a mesma direo):

P ( x) V ( y ) = (k cos x, sen x ) ( k sen y, cos y ) = k cos( x y ) 1 cos(a b) cos(c b) 1 + =0 BA CB 1 cos(b c) cos(a c) 1 + =0 CB AC

Aps dividir por k, as hipteses se tornam:

E queremos provar:

1 cos(c a) cos(b a) 1 + =0 AC B A
Para provar isso, basta somar as hipteses.
SOLUO DO PROBLEMA 4: THIAGO BARROS RODRIGUES COSTA (FORTALEZA CE)

Seja p um polinmio irredutvel em 1 r = 1s .

[ x] admitindo duas razes r e s cujo produto

Temos p ( x ) = x n + an 1 x n 1 + ... + a1 x + a0 (podemos supor p mnico sem perda de generalidade). Como p irredutvel em [ x] , p deve ser igual ao polinmio minimal de r e s sobre se g ( x) [ x] tal que g ( s ) = 0 ou g ( r ) = 0, ento p( x) g ( x). Seja g ( x) =
EUREKA! N26, 2007

1 n 1 . x p x a0

( )

63

Sociedade Brasileira de Matemtica

fcil ver que todas as potncias de x sero 0 g ( x) um polinmio sobre [ x] . Mas 1 n 1 g (r ) = r p 1 = r n p( s ) = 0 g (r ) = 0 p ( x) g ( x). r a0 a0 Mas g mnico e tem o mesmo grau de p 1 p( x) = g ( x) p ( x) = x n p 1 x a0

( ( ))

( )

Se raiz de p( x) (obviamente 0 pois p irredutvel e tem grau > 1), ento,

p 1

( ) = a1 1 p( ) = 0.
n n

Logo as razes de p sempre aparecem aos pares , 1 . Alm disso,

( x )

1 1 n 1 k 1 p( x) x = ( ) x x p = p( x). a0 x x
k

Veja que = 1 = 1 ou 1. Mas nenhum desses valores pode ser raiz de p tem grau 1, e p tem grau > 1. pois o polinmio minimal deles sobre as razes aparecem aos pares , 1

p tem um nmero par de razes p tem grau par.


Note que f (0) = 0 , e toda funo crescente e bijetora contnua, e logo integrvel em qualquer intervalo finito. Como 1 f ( x) decrescente, pelo critrio da integral somente se
SOLUO DO PROBLEMA 5: EDUARDO POO (SO PAULO SP)

f ( n)
n =1

converge se e lado,

1 dx f ( x)

converge.

Por

outro

1 n +1 f f 1 ( x) ( x) dx = dx, e 2 2 n x x n =1

f 1 (n) 4n 2 f 1 (n) (n + 1) 2
donde
EUREKA! N26, 2007

n +1

f 1 ( x) x 2 dx f 1 (n + 1) n 2 4 f 1 (n + 1) (n + 1) 2 ,

64

Sociedade Brasileira de Matemtica

1 f 1 ( n ) 4 n =1 n 2

f 1 ( x ) dx 4 f 2 x n =1

( n + 1) ( n + 1) 2 = 4 f
n=2

(m ) m 2
converge

4 f
m =1

( m ) m 2 , e,

em

particular,

( x)
2

dx

f
n =1

( n ) ( n ) 2 converge. Basta agora relacionar a convergncia de

1 dx e de f ( x)

f 1 ( x) dx , que so integrveis em intervalos finitos. x2

Um resultado conhecido (que pode ser facilmente verificado derivando) a frmula da primitiva da inversa:

( x)dx = xg 1 ( x) G g 1 ( x) , sendo G ( x) uma primitiva de g ( x)


1 1 , temos ento g ( x) = f 1 , e assim: f ( x) x
1 1 1

Utilizando g 1 ( x) =
b

f ( x) dx = b f (b) 1 f (1) G f (b) + G f (1)


1

f ( x) dx = f (b) f (1)
1

1 f (b )

1 f (1)

1 f 1 dx x

Essa relao tambm pode ser percebida diretamente do grfico de 1 / f ( x) , que uma funo decrescente. Com a transformao x = 1 / y :

b 1 1 dx = + f ( x) f (b) f (1)

f (b )

f (1)

f 1 ( y ) dy (I). y2

EUREKA! N26, 2007

65

Sociedade Brasileira de Matemtica

Suponha agora que

f ( x) dx converge. Logo:
1

x lim = 2 lim x f ( x) x

x/2

1 dt 2 lim x f ( x)

x/2

1 dt 2 lim x f (t )

x/2

1 dt = 0 f (t )

Ento lim

x = 0 . Fazendo b em (I), temos a integral esquerda x f ( x) b indo para zero, o que resulta num valor finito para convergindo e a parcela f (b)

f 1 ( x) dx , donde conclumos que essa integral converge. x2

Da mesma forma, se

f 1 ( x) f 1 ( x) dx converge, prova-se que lim =0,e x x x2

como f crescente e bijetora, podemos ir para o infinito com x = f ( y ) , de onde tiramos lim

f 1 ( f ( y )) y = 0 lim = 0 . Novamente aplicando o limite y y f ( y) f ( y) b convergem para valores b em (I), temos que a integral direita e f (b)

finitos, donde

1 dx converge. f ( x)

Retornando s sries, pelo critrio da integral, a equivalncia na convergncia das integrais, j provada, transmite equivalncia na convergncia das sries.

SOLUO DO PROBLEMA 6: COLABORAO DE DARIO BERALDO (PISA ITLIA)

Indutivamente, para cada inteiro k, 1 2k 1 k Ak = e B = 0 1 2k


EUREKA! N26, 2007

0 . 1

66

Sociedade Brasileira de Matemtica

Sejam A, B, I transformaes na reta projetiva (uma reta do plano projetivo), de modo que, em coordenadas homogneas, 1 1 1 + 2kt 2k + = Ak = t t t 1 e 1 s h s B = , 1 = hs 1 1 2 + h 2 + s
* para cada s, t R . Assim, para s irracional,

1 2k + s 1 2h + . Ak B h = s 1 1

. Suponhamos, por contradio, que existem inteiros a1 , b1 ,..., an , bn tais que

Iterando, eventualmente obtemos a seguinte formula: 1 2a1 + 1 2b1 + s Aa1 B b1 ... Aan Bbn = 1 2an + 1 1 2bn + s 1

M := Aa1 Bb1 ... Aan B bn = I . Dado um nmero transcendente s, a hiptese M = I implica em particular que as linhas determinadas por 1 2a1 + 1 2b1 + s e 1 1 2an + 1 2bn + s 1
EUREKA! N26, 2007

67

Sociedade Brasileira de Matemtica

devem ser as mesmas, i.e.

s = 2a1 +

1 2b1 + 1 2an + 1 2bn +

1 s Agora, basta certificar-se que existe uma relao algbrica no trivial (certamente quadrtica) para s, o que um absurdo.
Usaremos a seguinte notao:

{c0 , c1 ,..., cn } = c0 +

1 c1 + 1 cn 1 + 1 cn

Ento, nossa relao s = {2a1 , 2b1 ,..., 2an , 2bn , s} = {m0 , m1 , m2 ,...mN , s} , em que os m j so inteiros pares. Precisamos uma regra para escrever fraes contnuas como uma simples frao, i.e. Lema: Dado {c0 , c1 ,..., ck } , para cada inteiro 0 j k ,

{c ,..., c } = q
0

pj
j

onde p j = c j p j 1 + p j 2 , q j = c j q j 1 + q j 2 (com p0 = c0 , p1 = c0 c1 + 1, q0 = 1, q1 = c1 ). Esta uma induo fcil: temos {c0 ,..., c j , c j +1} = {c0 ,..., c j 1 , c j + 1 c j +1} =

1 cj + p j 1 + p j 2 c j +1 c j +1 (c j p j 1 + p j 2 ) + p j 1 c j +1 p j + p j 1 p j +1 = = = = , cqd. c j +1 (c j q j 1 + q j 2 ) + q j 1 c j +1q j + q j 1 q j +1 1 c + q + q j 2 j c j 1 j +1 Em particular, aplicando o lema para {m0 , m1 , m2 ,...mN , s} , obtemos, no caso em
que bn 0,

s=
EUREKA! N26, 2007

pN s + pN 1 (*) qN s + qN 1

68

Sociedade Brasileira de Matemtica

onde pN , qN , pN 1 , qN 1 so inteiros (pelo lema e o fato que p0 , p1 , q0 , q1 e os ms so inteiros). Note que qN 0. De fato, q0 = 1, q1 = m1 = 2b1 0 e, para k 1, qk +1 = mk +1qk + qk 1 , o que, como mk +1 um inteiro par no nulo (pelo menos para 1 k < N 1) implica, por induo, que qk +1 > qk . De fato, teremos

qk +1 mk +1qk qk 1 2 qk qk 1 > qk . No caso em que mN = 2bn = 0, teremos

s = {2a1 , 2b1 ,..., 2an ,1/ s} = m0 , m1 ,..., mN 1 , 1

}= p q

/ s + pN 2 pN 1 + pN 2 s = , qN 1 + qN 2 s N 1 / s + qN 2
N 1

e, como N = 2n 1 3, N 2 1 e logo qN 2 0. Isto implica que s uma raiz de uma equao de segundo grau com coeficientes inteiros, e segue a concluso.

Obs. Em vez de tomar s transcendente, poderamos tomar, por exemplo, s = 3 2. p 3 Temos 3 2 irracional, pois 1 < 3 2 < 2 e, se 2 = q com p e q inteiros, q > 1 e

p3 mdc( p, q) = 1, teramos 2 =

q3

, mas q 3 > 1 e mdc ( p3 , q 3 ) = 1, donde

p3

q3

, absurdo. Se

2 fosse raiz de uma equao do segundo grau

ax 2 + bx + c = 0 com a, b, c e a 0, teramos a 3 4 + b 3 2 + c = 0, donde b c 3 4 = 3 2 = u + v 3 2, com u = c a e v = b a racionais. Multiplicando a a


por
3

2 2, teramos 2 = u 3 2 + v 3 4 = u 3 2 + v u + v 3 2 = uv + ( u + v ) 3 2. Se

u + v 2 0,

teramos

2=

2 uv , u + v2

absurdo.

Assim,

u + v2 = 0

uv = 2 v3 = 2 3 2 = v , absurdo.

EUREKA! N26, 2007

69

Sociedade Brasileira de Matemtica

XXVIII OLIMPADA BRASILEIRA DE MATEMTICA


Resultado Nvel 1 (5a. e 6a. Sries)
CIDADE - ESTADO Pilar do Sul - SP Rio de Janeiro - RJ Vitria - ES Fortaleza - CE Vassouras - RJ Fortaleza - CE S. B. do Campo - SP Salvador - BA Jundia - SP Santo Andr - SP Rio de Janeiro - RJ Fortaleza - CE Campinas - SP Tabapu - SP Nova Lima - MG Fortaleza - CE Braslia - DF So Paulo - SP Rio de Janeiro - RJ Fortaleza - CE Araras - SP Recife - PE Porto Alegre - RS Fortaleza - CE Taubat - SP Fortaleza - CE Teresina - PI S. B. do Campo - SP So Paulo - SP Braslia - DF Vitria - ES Rio de Janeiro - RJ Fortaleza - CE Campo Grande - MS Fortaleza - CE Fortaleza CE Curitiba - PR Ponte Nova - MG Rio de Janeiro - RJ Santa Maria - RS Teresina - PI Belo Horizonte - MG So Paulo - SP So Paulo SP Piracicaba - SP Rio de Janeiro - RJ Parnamirim - RN S.J. da Boa Vista - SP Fortaleza - CE Uberaba - MG Juiz de Fora Belo Horizonte - MG Campinas - SP Fortaleza - CE Pirassununga - SP Campinas - SP Belo Horizonte - MG Fortaleza - CE Salgueiro - PE Araraquara - SP NOME Otvio Augusto de Oliveira Mendes Guilherme Cherman Perdigao de Oliveira Bruno Silva Mucciaccia Joo Lucas Camelo S Douglas Souza Alves Junior Kayo de Frana Gurgel Rafael Ferreira Antonioli Nikolas Leonel Carvalho Gabriela de Paula Gonzalvez Rodrigo Nagamine Ana Thais Castro de Santana Ycaro Csar Campello Izaias Dbora Jun Portugheis Gustavo Lopes Perosini Felipe Figueiredo Souza e Silva Marla Rochana Braga Monteiro Alexandre Crepory Abbott de Oliveira Rafael Kazuhiro Miyazaki Tiago Leandro Estevam Dias Daniel Silva Luiz Crispin Ana Lvia Ruegger Saldanha Wladimir Jos Lopes Martins Daniel dos Santos Bossle Erica Saldanha Freire Simes Lucas Almeida Rocha Hugo Rodrigues Martins Dantas Daniel Cardoso de Sousa Nicolas Iso Magosso Grigolli Gibin Marcos Massayuki Kawakami Paula Dias Garcia Renner Tetzner Ramos Bernardo de Andrade Macdo Marina Pessoa Mota Luis Henrique Kobaya Shi Higa Gabriel Milito Vinhas Lopes Filipe Jos Oliveira Sabia Vitor Silveira da Costa Tiago da vila Palhares Gustavo Pereira de Castro Douglas Michael da Costa Cezar Bruna Rufino Leo Thomas Rincon Reis Ramon Silva de Lima Eric Tada de Souza Rodrigo Gabriel Caetano Renato Soares Nunes Rafael Alves Pinheiro Leticia da Silva Incio Jos Elton Albuquerque Filho Luiz Fernando Cirigliano Villela Gabriel Ilharco Magalhes Fernanda Bahia de Carvalho Coutinho Victor Venturi Lara Timb Arajo Julia Langraf Scatolin Francisco Carvalho Osrio de Souza Luiza Christfaro Bragana de Matos Francisco Jairo Rodrigues Lima Otavia Ruanna Cordeiro de Oliveira Carolina Yumi Vezato PRMIO Ouro Ouro Ouro Ouro Ouro Prata Prata Prata Prata Prata Prata Prata Prata Prata Prata Prata Bronze Bronze Bronze Bronze Bronze Bronze Bronze Bronze Bronze Bronze Bronze Bronze Bronze Bronze Bronze Bronze Meno Honrosa Meno Honrosa Meno Honrosa Meno Honrosa Meno Honrosa Meno Honrosa Meno Honrosa Meno Honrosa Meno Honrosa Meno Honrosa Meno Honrosa Meno Honrosa Meno Honrosa Meno Honrosa Meno Honrosa Meno Honrosa Meno Honrosa Meno Honrosa Meno Honrosa Meno Honrosa Meno Honrosa Meno Honrosa Meno Honrosa Meno Honrosa Meno Honrosa Meno Honrosa Meno Honrosa Meno Honrosa

EUREKA! N26, 2007

70

Sociedade Brasileira de Matemtica

Nvel 2 (7a. e 8a. Sries)


NOME Renan Henrique Finder Hugo Fonseca Arajo Thiago Ribeiro Ramos Matheus Barros de Paula Rafael Alves da Silva Robrio Soares Nunes Leonardo Pereira Stedile Gustavo Lisba Empinotti Victor Reis de Abreu Cavalcanti Thiago Augusto da Silva Baleixo Heverton Carlos Bezerra de Azevedo James Jun Hong Leonardo Caruso de Oliveira Pedro Caetano Cardoso Illan Feiman Halpern Davi de Melo Pontes Mendes Matheus Arajo Marins Marcelo Tadeu de S Oliveira Sales Dan Zylberglejd Joo Mendes Vasconcelos Jos Ailton Azevedo Arajo Filho Leonardo Shimizu Yojo Gelly Whesley Silva Neves Frederico Gaia Costa da Silva Ana Lusa de Almeida Losnak Saulo Moraes de Faria Guilherme Salvador Vieira Fernando Fonseca Andrade Oliveira Rafael Farias Cao Edson Ryokei Onoga Rafael Gribel de Paula Neves Luiz Castelo Branco Cavalcante Filipe Gabriel Soares Rodrigues Rafael Horimoto de Freitas Germano Luis Lopes de Mello Silvio Tacla Alves Barbosa Pedro Pacheco Louzada Andr Saraiva Nobre dos Santos Mateus Bezerra Alves da Costa Leandro Lyra Braga Dognini Gabriel de Andrade Issisaki Isabella Amorim Gonzalez Andr Bina Possatto Obed Leite Vieira Eduardo Kaiser Ururahy Nunes Paulo Ricardo de Souza Costa Guilherme Vieira Melo Stephane Hilda Barbosa Lima Camila Miraglia Ribeiro Patrcia Fernanda Hongo Marcel Ichiro Bastos Kamiyama Camilla Kikuchi Yuri Santana do Carmo Juliana Rangel Cenzi Rafael Eiki Takemura CIDADE ESTADO Joinville - SC Juiz de Fora - MG Varginha - MG Taubat - SP Teresina - PI Ribeiro Preto - SP So Paulo - SP Florianpolis - SC Macei - AL Rio de Janeiro - RJ Rio de Janeiro - RJ So Paulo - SP Rio de Janeiro - RJ Rio de Janeiro - RJ Itatiaia - RJ Fortaleza - CE So Gonalo - RJ Barreiras - BA Rio de Janeiro RJ Fortaleza - CE Fortaleza - CE So Paulo SP Fortaleza - CE Teresina - PI So Paulo - SP Niteri - RJ Rio Claro - SP Belo Horizonte - MG Campo Grande - MS So Paulo - SP Rio de Janeiro - RJ Teresina - PI Teresina - PI So Paulo - SP Rio de Janeiro - RJ So Paulo SP Rio de Janeiro RJ Fortaleza - CE Fortaleza - CE Barcarena - PA Guara - SP Macei - AL So Caetano - SP Fortaleza CE Itatiaia - RJ Rio de Janeiro - RJ Fortaleza - CE Fortaleza - CE Curitiba - PR Bragana Paulista - SP So Paulo - SP So Paulo - SP Belm - PA Jacare - SP So Paulo SP PRMIO Ouro Ouro Ouro Ouro Ouro Prata Prata Prata Prata Prata Prata Prata Prata Prata Prata Bronze Bronze Bronze Bronze Bronze Bronze Bronze Bronze Bronze Bronze Bronze Bronze Bronze Bronze Bronze Bronze Bronze Meno Honrosa Meno Honrosa Meno Honrosa Meno Honrosa Meno Honrosa Meno Honrosa Meno Honrosa Meno Honrosa Meno Honrosa Meno Honrosa Meno Honrosa Meno Honrosa Meno Honrosa Meno Honrosa Meno Honrosa Meno Honrosa Meno Honrosa Meno Honrosa Meno Honrosa Meno Honrosa Meno Honrosa Meno Honrosa Meno Honrosa

EUREKA! N26, 2007

71

Sociedade Brasileira de Matemtica

Nvel 3 (Ensino Mdio)


NOME Jose Marcos Andrade Ferraro Henrique Pond de Oliveira Pinto Guilherme Rodrigues Nogueira de Souza Leandro Farias Maia Ramon Moreira Nunes Rafael Mendes de Oliveira Regis Prado Barbosa Edson Augusto Bezerra Lopes Andr Linhares Rodrigues Willy George do Amaral Petrenko Leonardo Ribeiro de Castro Carvalho Rodrigo Viana Soares Artur de Almeida Losnak Paulo Andr Carvalho de Melo Renato Rebouas de Medeiros Rafael Sampaio de Rezende Rafael Tupynamb Dutra Alfredo Roque de Oliveira Freire Filho Rafael Morioka Oda Adenilson Arcanjo de Moura Jnior Giuliano Pezzolo Giacaglia Wilson Camara Marriel Rafael Sabino Lima Marlen Lincoln da Silva Csar Ryudi Kawakami Hugo Musso Gualandi Raphael Rodrigues Mata Alexandre Hideki Deguchi Martani Ricardo Turolla Bortolotti Felipe Gonalves Assis Max Douglas Peixoto da Silva urope Moraes Gorito Fernando Nascimento Coelho Guilherme Philippe Figueiredo Marcelo Matheus Gauy Luiz Carlos da Silva Sobral Iuri Lima Ribeiro Alex Atsushi Takeda Gabriel Caser Brito Jose Armando Barbosa Filho Henrique Hiroshi Motoyama Watanabe Joo Luiz de Oliveira Madeira Lcio Eiji Assaoka Hossaka Pedro Pinheiro de Negreiros Bessa Davi Lopes Alves de Medeiros Paulo Srgio de Castro Moreira Roberto Akiba de Oliveira Enzo Haruo Hiraoka Moriyama Alexandre Azevedo Cezar Jos Airton Coelho Lima Filho Pedro Paulo Gondim Cardoso Thiago da Silva Pinheiro CIDADE ESTADO So Paulo SP Salvador - BA So Paulo - SP Fortaleza CE Fortaleza CE Rio de Janeiro RJ Fortaleza - CE Fortaleza - CE Fortaleza CE Rio de Janeiro - RJ So Paulo - SP Fortaleza CE So Paulo - SP Rio de Janeiro - RJ Fortaleza - CE Fortaleza - CE Belo Horizonte - MG Salvador - BA So Paulo - SP Fortaleza - CE So Paulo - SP Rio de Janeiro - RJ Rio de Janeiro - RJ Fortaleza CE So Paulo SP Vitria - ES Salvador - BA So Paulo SP Rio Claro SP Campina Grande - PB Fortaleza - CE Rio de Janeiro - RJ Fortaleza CE Fortaleza - CE S.J. do Rio Preto - SP Aracaju - SE Fortaleza - CE Londrina - PR Rio de Janeiro - RJ Fortaleza - CE So Paulo - SP So Paulo - SP Curitiba - PR Fortaleza - CE Fortaleza - CE Fortaleza - CE Sorocaba - SP So Paulo - SP Fortaleza CE Fortaleza CE Salvador - BA So Paulo - SP PRMIO Ouro Ouro Ouro Ouro Ouro Ouro Ouro Prata Prata Prata Prata Prata Prata Prata Prata Prata Prata Prata Bronze Bronze Bronze Bronze Bronze Bronze Bronze Bronze Bronze Bronze Bronze Bronze Bronze Meno Honrosa Meno Honrosa Meno Honrosa Meno Honrosa Meno Honrosa Meno Honrosa Meno Honrosa Meno Honrosa Meno Honrosa Meno Honrosa Meno Honrosa Meno Honrosa Meno Honrosa Meno Honrosa Meno Honrosa Meno Honrosa Meno Honrosa Meno Honrosa Meno Honrosa Meno Honrosa Meno Honrosa

EUREKA! N26, 2007

72

Sociedade Brasileira de Matemtica

Nvel Universitrio
NOME Fbio Dias Moreira Alex Corr Abreu Humberto Silva Naves Samuel Barbosa Feitosa Levi Mximo Viana Rafael Daigo Hirama Rafael Marini Silva Thiago Barros Rodrigues Costa Henry Wei Cheng Hsu Murilo Vasconcelos Andrade Luiz Felipe Marini Silva Thiago da Silva Sobral Felipe Rodrigues Nogueira de Souza Thiago Costa Leite Santos Raphael Constant da Costa Eduardo de Moraes Rodrigues Poo Elton Gomes Coriolano Elder Rodrigo Barbosa Coelho Ronaldo Rodrigues Pel Estillac Lins Maciel Borges Filho Thoms Yoiti Sasaki Hoshina Lus Daniel Barbosa Coelho Renato Francisco Lopes Mello Pedro Henrique Milet Pinheiro Pereira Pedro Henrique Silva Belisrio Luty Rodrigues Ribeiro Jos Mrio da Silva Filho Kellem Corra Santos Marcos Francisco Ferreira Martinelli Evandro Makiyama Nilson Maciel de Paiva Jnior Igor de Castro Lima Eric Campos Bastos Guedes Marcelo de Arajo Barbosa Pedro Meira de Vasconcelos Bezerra Moyses Afonso Assad Cohen Davi de Melo Jorge Barbosa Rodrigo Pereira Maranho CIDADE ESTADO Rio de Janeiro - RJ Niteri - RJ S.J. dos Campos - SP Fortaleza - CE Rio Janeiro - RJ S.J. dos Campos - SP Vila Velha - ES Fortaleza - CE So Paulo - SP Rio de Janeiro - RJ S.J. dos Campos - SP S.J. dos Campos - SP So Paulo - SP So Paula - SP Rio de Janeiro - RJ So Paulo - SP Fortaleza - CE Rio de Janeiro - RJ S.J. dos Campos - SP Belm - PA Rio de Janeiro - RJ Rio de Janeiro - RJ J. dos Guararapes - PE Rio de Janeiro RJ Rio de Janeiro RJ S.J. dos Campos - SP S.J. dos Campos SP Rio de Janeiro RJ Rio de Janeiro - RJ So Paulo SP Rio de Janeiro - RJ Rio de Janeiro RJ Niteri - RJ S.J. dos Campos - SP Recife - PE Rio de Janeiro - RJ Fortaleza -CE Rio de Janeiro - RJ PRMIO Ouro Ouro Ouro Ouro Ouro Prata Prata Prata Prata Prata Prata Prata Prata Prata Prata Bronze Bronze Bronze Bronze Bronze Bronze Bronze Bronze Bronze Bronze Bronze Bronze Bronze Bronze Meno Honrosa Meno Honrosa Meno Honrosa Meno Honrosa Meno Honrosa Meno Honrosa Meno Honrosa Meno Honrosa Meno Honrosa

EUREKA! N26, 2007

73

Sociedade Brasileira de Matemtica

AGENDA OLMPICA
XXIX OLIMPADA BRASILEIRA DE MATEMTICA NVEIS 1, 2 e 3 Primeira Fase Sbado, 16 de junho de 2007 Segunda Fase Sbado, 15 de setembro de 2007 Terceira Fase Sbado, 27 de outubro de 2007 (nveis 1, 2 e 3) Domingo, 28 de outubro de 2007 (nveis 2 e 3 - segundo dia de prova). NVEL UNIVERSITRIO Primeira Fase Sbado, 15 de setembro de 2007 Segunda Fase Sbado, 27 e Domingo, 28 de outubro de 2007

XIII OLIMPADA DE MAIO 12 de maio de 2007

XVIII OLIMPADA DE MATEMTICA DO CONE SUL Uruguai 12 a 17 de junho de 2007

XLVIII OLIMPADA INTERNACIONAL DE MATEMTICA 19 a 31 de julho de 2007 Vietn

XIV OLIMPADA INTERNACIONAL DE MATEMTICA UNIVERSITRIA 3 a 9 de agosto de 2007 Blagoevgrad, Bulgria

XXII OLIMPADA IBEROAMERICANA DE MATEMTICA 6 a 16 de setembro de 2007 Coimbra, Portugal

X OLIMPADA IBEROAMERICANA DE MATEMTICA UNIVERSITRIA 5 de novembro de 2007

EUREKA! N26, 2007

74

Sociedade Brasileira de Matemtica

COORDENADORES REGIONAIS
Alberto Hassen Raad Amrico Lpez Glvez Amarsio da Silva Arajo Andreia Goldani Antonio Carlos Nogueira Ali Tahzibi Benedito Tadeu Vasconcelos Freire Carlos Alexandre Ribeiro Martins Carmen Vieira Mathias Claus Haetinger Cleonor Crescncio das Neves Cludio de Lima Vidal Denice Fontana Nisxota Menegais Edson Roberto Abe lio Mega Eudes Antonio da Costa Fbio Brochero Martnez Florncio Ferreira Guimares Filho Francinildo Nobre Ferreira Genildo Alves Marinho Ivanilde Fernandes Saad Jacqueline Rojas Arancibia Janice T. Reichert Joo Bencio de Melo Neto Joo Francisco Melo Libonati Jos Luiz Rosas Pinho Jos Vieira Alves Jos William Costa Krerley Oliveira Licio Hernandes Bezerra Luciano G. Monteiro de Castro Luzinalva Miranda de Amorim Mrio Rocha Retamoso Marcelo Rufino de Oliveira Marcelo Mendes Newman Simes Nivaldo Costa Muniz Osvaldo Germano do Rocio Ral Cintra de Negreiros Ribeiro Ronaldo Alves Garcia Rogrio da Silva Igncio Reginaldo de Lima Pereira Reinaldo Gen Ichiro Arakaki Ricardo Amorim Srgio Cludio Ramos Seme Gebara Neto Tadeu Ferreira Gomes Toms Menndez Rodrigues Valdenberg Arajo da Silva Vnia Cristina Silva Rodrigues Wagner Pereira Lopes (UFJF) (USP) (UFV) FACOS (UFU) (USP) (UFRN) (Univ. Tec. Fed. de Paran) (UNIFRA) (UNIVATES) (UTAM) (UNESP) (UNIPAMPA) (Colgio Objetivo de Campinas) (Faculdade Etapa) (Univ. Federal do Tocantins) (UFMG) (UFES) (UFSJ) (Centro Educacional Leonardo Da Vinci) (UC. Dom Bosco) (UFPB)) (UNOCHAPEC) (UFPI) (Grupo Educacional Ideal) (UFSC) (UFPB) (Instituto Pueri Domus) (UFAL) (UFSC) (Sistema Elite de Ensino) (UFBA) (UFRG) (Grupo Educacional Ideal) (Colgio Farias Brito, Pr-vestibular) (Cursinho CLQ Objetivo) (UFMA) (U. Estadual de Maring) (Colgio Anglo) (UFGO) (Col. Aplic. da UFPE) (Escola Tcnica Federal de Roraima) (UNIFESP) (Centro Educacional Logos) (IM-UFRGS) (UFMG) (UEBA) (U. Federal de Rondnia) (U. Federal de Sergipe) (U. Metodista de SP) (CEFET GO) Juiz de Fora MG Ribeiro Preto SP Viosa MG Osrio RS Uberlndia MG So Carlos SP Natal RN Pato Branco - PR Santa Mara RS Lajeado RS Manaus AM S.J. do Rio Preto SP Bag RS Campinas SP So Paulo SP Arraias TO Belo Horizonte MG Vitria ES So Joo del Rei MG Taguatingua DF Campo Grande MS Joo Pessoa PB Chapec SC Teresina PI Belm PA Florianpolis SC Campina Grande PB Santo Andr SP Macei AL Florianpolis SC Rio de Janeiro RJ Salvador BA Rio Grande RS Belm PA Fortaleza CE Piracicaba SP So Luis MA Maring PR Atibaia SP Goinia GO Recife PE Boa Vista RR SJ dos Campos SP Nova Iguau RJ Porto Alegre RS Belo Horizonte MG Juazeiro BA Porto Velho RO So Cristovo SE S.B. do Campo SP Jata GO

EUREKA! N26, 2007

75

CONTEDO
AOS LEITORES XVIII OLIMPADA DE MATEMTICA DO CONE SUL Enunciados e resultado brasileiro XIX OLIMPADA DE MATEMTICA DO CONE SUL Enunciados e resultado brasileiro 2 3 11

ARTIGOS
JOGOS E FEIJOADA NO SO PAULOS Emanuel Carneiro SUBSTITUIES ENVOLVENDO NMEROS COMPLEXOS Diego Veloso Ucha INTEGRAIS DISCRETAS Eduardo Poo PRODUTOS NOTVEIS Onofre Campos OLIMPADAS AO REDOR DO MUNDO COMO QUE FAZ SOLUES DE PROBLEMAS PROPOSTOS PROBLEMAS PROPOSTOS AGENDA OLMPICA COORDENADORES REGIONAIS 13 17

25

32 38 48 50 58 61 62

Sociedade Brasileira de Matemtica

AOS LEITORES

com grande alegria que comemoramos em 2008 os 10 anos da Revista EUREKA! e transmitimos aos leitores a nossa satisfao pela acolhida recebida neste perodo. Durante estes 10 anos de existncia temos procurado atender ao leitor mais exigente, apresentando uma publicao especfica que alm de fornecer material atualizado e de alto nvel acadmico, tem tornado o estudo da matemtica olmpica muito mais interessante e acessvel a professores e jovens olmpicos de todo o Brasil. Neste nmero especial da revista apresentamos quatro artigos, cujos autores so todos ex-olmpicos de grande destaque, alm de um bom nmero de novos problemas propostos por nossos leitores, que esto cada vez mais inspirados. Agradecemos tambm a valiosa ajuda dos alunos que trabalharam na reviso deste nmero da Eureka!: lvaro Lopes Pedroso, Ana Lusa de Almeida Losnak, Custdio Moreira Brasileiro Silva, Elder Massahiro Yoshida, Guilherme Phillippe Figueiredo Hanon Guy Lima Rossi, Henrique Pond de Oliveira Pinto, Illan Feiman Halpern, Marco Antonio Lopes Pedroso, Rafael Horimoto de Freitas, Renan Henrique Finder, Talita Alessandra da Silva, Thiago Saksanian Hallak e Thiago da Silva Pinheiro, e particularmente ao Prof. Carlos Yuzo Shine, que coordenou a reviso e que foi responsvel pela seo Como que faz deste nmero. Continuaremos contando com o entusiasmo e a colaborao dos nossos leitores para que a EUREKA! continue sendo um instrumento til formao matemtica e preparao olmpica do nosso pblico. Esperamos que gostem deste nmero. Divirtam-se!

Os editores

EUREKA! N27, 2008

Sociedade Brasileira de Matemtica

XVIII OLIMPADA DE MATEMTICA DO CONE SUL


Enunciados e Resultado Brasileiro
A XVIII Olimpada de Matemtica do Cone Sul foi realizada na cidade de Atlntida, Uruguai no ms de junho de 2007. A equipe brasileira foi liderada pelos professores Yuri Gomes Lima e Samuel Barbosa Feitosa, ambos da cidade de Fortaleza CE.
RESULTADOS DA EQUIPE BRASILEIRA BRA1 BRA2 BRA3 BRA4 Renan Henrique Finder Marcelo Tadeu de S Oliveira Sales Grazielly Muniz da Cunha Thiago Ribeiro Ramos Medalha de Ouro Medalha de Prata Medalha de Prata Medalha de Prata

PRIMEIRO DIA PROBLEMA 1

Achar todos os pares de inteiros (x, y) que satisfazem


x 3 y + x + y = xy + 2 xy 2 .
SOLUO DE MARCELO TADEU DE S OLIVEIRA SALES (SALVADOR BA)

De x 3 y + x + y = xy + 2 xy 2 temos:
x 3 y + x xy 2 xy 2 = y x( x 2 y + 1 y 2 y 2 ) = y x | y xy + 2 xy 2 y x 3 y = x y ( x + 2 xy 1 x3 ) = x y | x Ento x | y e y | x com exceo de x = 0 ou y = 0. Nesses dois casos temos que ambos tm que ser 0. Assim (x, y) = (0, 0) a nossa primeira soluo. Se x | y ento x y (eu j desconsiderei x = 0 e y = 0) e se y | x ento y x , da

x = y . Assim temos dois casos: Primeiro caso: x = y Substituindo temos x 4 + 2 x = x 2 + 2 x 3 . Como x 0 e y 0 ento podemos
simplificar. Assim, x 3 + 2 = x + 2 x 2 e da x | 2 , ento x {2, 1,1, 2}. Desses valores, o nico que no admite soluo x = 2 ento para esse caso ( x, y ) = (1, 1);(1,1);(2, 2).
EUREKA! N27, 2008

Sociedade Brasileira de Matemtica

Segundo caso: x = y. Substituindo temos x 4 = x 2 + 2 x 3 . Como x 0 temos x 4 = x 2 + 2 x 3 x 2 = 1 + 2 x e da x = inteiro, ento no h soluo para esse caso. Assim as solues so ( x, y ) = (1, 1);(0,0);(1,1) e (2, 2).
PROBLEMA 2

2 8 que no 2

Considere 100 inteiros positivos tais que sua soma igual ao seu produto. Determinar a quantidade mnima de nmeros 1 que podem existir entre os 100 inteiros.
SOLUO DE RENAN HENRIQUE FINDER (JOINVILLE SC)

Seja K o nmero de 1s que aparecem. Sejam a1 , a2 ...a100 os nmeros, com a1 = a2 = ... = aK = 1.

a1 + a2 + ... + a100 = a1a2 ...a100 K + aK +1 + ... + a100 = aK +1aK + 2 ...a100


Vamos minimizar aK +1aK + 2 ...a100 aK +1 aK + 2 ... a100 . Para isso, suponha a j 2. Note que aK +1 ...a j ...a100 aK +1 ... a j ... a100 2aK ...a j 1a j +1 ...a100 2 aK +1 ...

a j 1 a j +1 ... a100 aK +1 ...a j ...a100 a j 2aK +1 ...a j 1a j +1 ...a100 2


aK +1...a j 1a j +1...a100 (a j 2) a j 2, o que ocorre de fato. Ento, a diferena
mnima quando aK +1 = aK + 2 = ... = a100 = 2. Logo, K = aK +1aK + 2 ...a100 aK +1 ... a100 2100 K 2 (100 K ) 0 2100 K 200 + K
2100 K 200 K . Se K 93 200 K 107 e 2100 K 27 = 128, o que obriga K 94. Note que h um exemplo para K = 95: a1 = a2 = ... = a45 = 1
a96 = a95 = 2 a98 = a99 = a100 = 3

A soma 195 + 2 2 + 3 3 = 95 + 4 + 9 = 108 e o produto 22 33 = 108. Resta o caso K = 94, isto , o caso a + b + c + d + e + f + 94 = abcdef . Supondo a, b 3, minimizemos abcdef a b c d e f . Temos
EUREKA! N27, 2008

Sociedade Brasileira de Matemtica

abcdef a b c d e f 3bcdef 3 b c d e f ( a 3)bcdef a 3, o que ocorre. Alm disso, abcdef a b c d e f 2abcde a b c d e 2 abcde( f 2) f 2, o que ocorre, pois f 1, logo f 2. Conclumos que a expresso mnima se a = 3 e f = 2. Analogamente, ela mnima quando b = 3 e c = d = e = 2. Ento, abcdef a b c d e f 32 24 4 2 3 2 = 144 14 = 130 > 94. Ento, impossvel abcdef a b c d e f = 94 se duas das variveis forem 3 . J se s uma for 3 (digamos que a, teremos b = c = d = e = f = 2, logo | 104. Se todas as variveis 25 a 10 a = 94 31a = 104, absurdo, pois 31 / forem iguais a 2, obtemos tambm 31a = 104 31 2 = 104, absurdo. Ento K = 95 o mximo que podemos obter.
PROBLEMA 3

Seja ABC um tringulo com todos os seus ngulos agudos, de alturas AD, BE e CF (com D em BC, E em AC e F em AB). Seja M o ponto mdio do segmento BC. A circunferncia circunscrita ao tringulo AEF corta a reta AM em A e X. A reta AM corta a reta CF em Y. Seja Z o ponto de encontro entre as retas AD e BX. Demonstrar que as retas YZ e BC so paralelas.
SOLUO DA BANCA

Observemos que AFHE inscritvel, pois AEH = AFH = 90. Da que AXH = 90. Seja A um ponto sobre a semireta AM tal que AM = MA. O quadriltero ABAC um paralelogramo, onde

ABH = ABC + CBH = ACB + (90 ACB) = 90 = A XH , Ou seja que o quadriltero BHXA inscritvel. Alem disso BHCA tambm inscritvel j que
BHC + BAC = (180 BAC ) + BAC = 180.
Desta forma os pontos B, H, X, C so concclicos, onde
EUREKA! N27, 2008

Sociedade Brasileira de Matemtica

XBM = XBC = XAC = BAM (1)


Seja T = AB XH . Notemos que H tambm o ortocentro do tringulo ATY, uma vez que TY AY e YF AT . Da que AH TY , ento TY // BC. Se provamos que Z TY , o problema estar terminado. Seja ento Z = AD TY . Vamos mostrar que Z = Z . Agora, HZ Y = HXY = 90 ento HXYZ inscritvel, e portanto, XZ Y = XHY = FAX BAM (2) Das relaes (1) e (2) segue-se que XZ Y = XBM , ou seja, que os pontos B, Z e X so colineares. Porm ento Z AD BX = Z e assim Z = Z , como queramos.

A E F H Z B
PROBLEMA 4

X Y

Considere um tabuleiro 2007 2007. So pintadas algumas casas do tabuleiro. Dizemos que o tabuleiro charrua se nenhuma linha est totalmente pintada e nenhuma coluna est totalmente pintada. a) Qual o nmero mximo k de casas pintadas que um tabuleiro charrua pode ter? b) Para tal nmero k, calcular o nmero de tabuleiros charruas distintos que existem.

EUREKA! N27, 2008

Sociedade Brasileira de Matemtica

SOLUO DE GRAZIELLY MUNIZ DA CUNHA (FORTALEZA CE)

a) Note que todas as colunas tm que ter no mximo 2006 casas pintadas. Como so 2007 colunas ento o nmero de casas pintadas no mximo 2007 2006, nmero que atingido pintando todas as casas, exceto uma diagonal, como na figura abaixo

2007

2007
b) como para k no mximo iremos pintar 2006 casas em cada coluna, ento temos que escolher qual casa ficar sem ser pintada. E note que no podemos ter duas casas sem serem pintadas em uma mesma linha, pois se no ter uma linha que ficar toda preenchida. Logo para a primeira coluna poderemos escolher qualquer uma das 2007 casas para no ser pintada, na segunda coluna podemos escolher qualquer uma de 2006 casas, pois no podemos escolher uma casa que esteja na mesma linha qua a que foi escolhida na primeira coluna, na terceira coluna temos 2005 escolhas, na quarta 2004 escolhas e assim sucessivamente, logo so 2007 2006 2005... = 2007! maneiras de escolher, logo so 2007! tabuleiros charruas distintos, com o nmero k.
PROBLEMA 5

Seja ABCDE um pentgono convexo que satisfaz as seguintes condies: Existe uma circunferncia tangente a cada um de seus lados. As medidas de todos os seus lados so nmeros inteiros. Ao menos um dos lados do pentgono mede 1. O lado AB mede 2. Seja P o ponto de tangncia de com o lado AB. a) Determinar as medidas dos segmentos AP e BP.
EUREKA! N27, 2008

Sociedade Brasileira de Matemtica

b) Dar um exemplo de um pentgono que satisfaz as condies estabelecidas.


SOLUO DE RENAN HENRIQUE FINDER (JOINVILLE SC)

a) Sejam Q, R, S e T os pontos de tangncia de em BC, CD, DE e EA, respectivamente, como a seguir:


E S D R P T A

Seja AP = x. Temos: AB = 2 BP = 2 x QB = BP QB = 2 x QC = BC QB = BC 2 + x CR = QC CR = BC 2 + x DR = CD CR = CD BC + 2 x DS = DR DS = CD BC + 2 x ES = DE DS = DE C D + BC 2 + x ET = ES ET = DE CD + BC 2 + x AE = AT + TE = x + DE CD + BC 2 + x 2x = AE DE + CD BC +2 * + Como x < AB = 2 2 x < 4, pode-se ter 2 x = 3, 2 x = 2 ou 2 x = 1. O item b) 3 1 e o caso x = obviamente anlogo mostra uma configurao para x = 2 2 (troque A por B e C por E). Resta ver o que acontece se x = 1. Temos AT = 1 AE > 1. Ento AE 2, porque AE . Como ET = AE 1, vale que ET e ET 1. Assim, SE 1, pois SE = TE. Desse modo, DE > SE DE 2. Como DS = DE SE tem-se DS 1, mas ento DR 1 DC > 1. Logo DC 2, uma vez que DC . Mas ento CR 1, j que CR = DC DR = DC DS . E tambm CQ 1 e CQ , j que RC = QC. Deste modo, BC > CQ BC > 1. E AB > AP = 1. Ento todos os lados so maiores que 1: absurdo.
EUREKA! N27, 2008

Sociedade Brasileira de Matemtica

3 1 3 w Obs. No desenho do item b), de fato AP = , pois AP = x + = 1 + = (o 2 2 2 2 ponto P bissecta o segmento de medida w pois esse segmento o lado de um hexgono regular). Por outro lado, na verdade j provamos que s podamos ter 3 1 3 1 AP = ou AP = . Isso mostra que AP = e BP = uma possibilidade (e 2 2 2 2 3 1 que, analogamente, BP = e AP = tambm ). 2 2

b) Tome um hexgono regular de lado 1, seu incrculo , dois de seus lados no opostos e no adjacentes e os prolongue. Seja A a interseco obtida. O pentgono ABCDE o fecho convexo da unio dos pontos do hexgono e do ponto A, como a seguir:
C D

B P w 120 120 z

x A

Note que o tringulo de lados x e y, com vrtice em A, tem dois ngulos de 180 120 = 60, logo eqiltero, e de lado 1 (seu terceiro lado lado do hexgono L )! Como w e z so lados do hexgono, w = x = y = z = 1. Ento: ABCDE circunscrvel BC = CD = DE = 1 AB = EA = w + x = y + z = 1 + 1 = 2 Todos os lados so inteiros, como queramos.
PROBLEMA 6

Demonstrar que, para cada inteiro positivo n, existe um inteiro positivo k tal que a representao decimal de cada um dos nmeros k, 2k,..., nk contm todos os dgitos 0, 1, 2, 3, 4, 5, 6, 7, 8, 9.

EUREKA! N27, 2008

Sociedade Brasileira de Matemtica

SOLUO DE MARCELO TADEU DE S OLIVEIRA SALES (SALVADOR BA)

Lema: Para todo n, existe k onde n


* +

* +

tal que nk contm todos os dgitos 0, 1, 2, ..., 9,

Demonstrao: Ao fatorarmos n temos que n da forma 2a 5b q onde q o produto dos outros fatores primos de n. Seja c = mx (a, b). Vou mostrar que para n = 10c q o lema vlido. Temos que 2a 5b q divisor de 10c q, e por Bzout existe um x tal que 10c qx 10c (mod 10c +1 ) , pois mdc(q,10) = 1 Assim, se multiplicarmos 10c qx por 2, 3, ..., 9 ele dar restos 2 10c ,...,9 10c , ou seja, aparecerem os dgitos que ns quisermos na base decimal. Considere 10c (qx + 2 10 p qx + 3qx 102 p + ... + 9qx 108 p ) onde x o nmero tal que
10c qx 10c (mod 10c +1 ) e p um inteiro tal que 10 p > 9 10c qx.

Ento k = 10( x + 2 10 p x + 3 102 p x + ... + 9 108 p x) satisfaz as nossas condies pois ao multiplicarmos k por 10c q temos o seguinte: Em 10c q x vai aparecer um dgito 1 porque 10c q x 10c (mod10c +1 ). Em
10c + p q 2 x vai aparecer um dgito 2.

Em 10c +8 p q 9 x vai aparecer um dgito 9 e eu multipliquei tudo por 10 para aparecer o 0. Assim k o que queramos. Agora vou terminar o problema por induo Casos iniciais n = 1 e k = 1234567890 k = 1234567890617283945 n=2e ou 6172839450 Passo indutivo suponha que at n verdadeira ento existe um k tal que k, 2k, 3k,...,nk tm todos os dgitos 0,1,...,9. Observe que se multiplicarmos k por 10 para + * continua sendo verdade para k , 2k , caso, nk . Pelo lema, temos que existe um r tal que (n + 1)r tem todos os dgitos 0,1,...,9. Assim, fazendo s = 10 k + r , onde um inteiro tal que 10 > (n + 1)r , esse s satisfaz as condies do enunciado para 1 at (n + 1) pois de 1 at n teremos que s, 2 s,..., ns tero todos os dgitos porque 10 k ,...,10 nk tm (e porque grande o suficiente para que nr < 10 ). E para n + 1 temos que (n + 1)r < 10 ter todos os dgitos. Assim s satisfaz as condies. Portanto para todo n inteiro positivo existe tal inteiro k.

EUREKA! N27, 2008

10

Sociedade Brasileira de Matemtica

XIX OLIMPADA DE MATEMTICA DO CONE SUL


Enunciados e Resultado Brasileiro
A XIX Olimpada de Matemtica do Cone Sul foi realizada na cidade de Temuco, Chile no ms de maio de 2008. A equipe brasileira foi liderada pelos professores Ccero Thiago Magalhes e Bruno Holanda, ambos da cidade de Fortaleza CE.
RESULTADOS DA EQUIPE BRASILEIRA BRA1 BRA2 BRA3 BRA4 Gustavo Lisba Empinotti Marcelo Tadeu de S Oliveira Sales Matheus Arajo Marins Matheus Secco Torres da Silva Medalha de Bronze Medalha de Prata Medalha de Bronze Medalha de Prata

PRIMEIRO DIA PROBLEMA 1

Defina I(n) como o resultado de inverter os nmeros de um algarismo. Por exemplo, I (123) = 321, etc. Calcule todos os inteiros 1 n 10000 tais que

n I (n) = . 2
PROBLEMA 2

Seja ABC um tringulo, P um ponto em seu interior e X, Y e Z pontos em BC, AC e AB respectivamente tais que PZB = PXC = PYA. Considere os pontos U, W e V sobre BC, AC e AB (ou seus prolongamentos, se necessrio) tais que PV = 2PY; PU = 2PX e PW = 2PZ. Sabendo que a rea de XYZ 1, calcule a rea de UVW.
PROBLEMA 3

Dois amigos A e B devem resolver a seguinte adivinha: cada um deles recebe um nmero do conjunto {1, 2, ..., 250} mas no v o nmero que o outro recebeu. O objetivo que cada amigo descubra o nmero do outro. O procedimento que devem seguir anunciar, por turnos, nmeros inteiros positivos no necessariamente distintos: primeiro A diz um nmero, em seguida B diz um nmero, depois novamente A, etc., de modo que a soma de todos os nmeros
EUREKA! N27, 2008

11

Sociedade Brasileira de Matemtica

anunciados seja 20. Demonstrar que existe uma estratgia de modo que, atravs de um acordo prvio A e B possam atingir o objetivo, sem importar quais nmeros cada um receba no comeo da adivinha.
SEGUNDO DIA PROBLEMA 4

Qual o maior nmero de casas que se pode colorir num tabuleiro 7 7 de maneira que todo subtabuleiro 2 2 tenha no mximo 2 casas coloridas?

PROBLEMA 5

Seja ABC um tringulo issceles de base AB. Uma semicircunferncia C com centro no segmento AB e tangente aos lados iguais AC e BC. Considera-se uma reta tangente a C que corta os segmentos AC e BC em D e E, respectivamente. Suponha que as retas perpendiculares a AC e BC, traadas respectivamente por D e E, se cortam em P interior ao tringulo ABC. Seja Q o p da perpendicular PQ 1 AB reta AB que passa por P. Demonstrar que = . CP 2 AC
PROBLEMA 6

Dizemos que um nmero capicua se ao inverter a ordem de seus algarismos obtivermos o mesmo nmero. Achar todos os nmeros que tem pelo menos um mltiplo no-nulo que seja capicua.

EUREKA! N27, 2008

12

Sociedade Brasileira de Matemtica

JOGOS E FEIJOADA NO SO PAULOS


Emanuel A. S. Carneiro Nvel Iniciante
Bem prximo ao Robert Lee Moore hall, sede do Departamento de Matemtica da Universidade do Texas em Austin, fica o celebrado restaurante brasileiro So Paulos. Comida muito boa (definitivamente a melhor feijoada da cidade), alm do velho e bom guaran Antrtica so apenas alguns dos fatores que nos levam (a comunidade brasileira aqui em Austin) a almoar regularmente no So Paulos. Certo dia eu estava a almoar com dois professores do departamento e algo me fez lembrar dos meus tempos de olimpada de Matemtica. Fernando Rodriguez-Villegas, argentino, professor na rea de teoria dos nmeros e Tams Hausel, hngaro, que trabalha nas reas de geometria algbrica e topologia. Enquanto saborevamos as nossas feijoadas (eu e o Tams, o Fernando no bob de camaro), conversvamos sobre jogos matemticos. Dr. Rodriguez-Villegas, um matemtico extraordinrio e super simptico, que alm de fazer pesquisa do mais alto nvel em teoria dos nmeros bastante interessado em jogos e puzzles matemticos, nos explicava tpicos do curso que estava a ensinar nesse semestre (Math, Puzzles and Computers) alm de outras idias de jogos que ele prprio havia inventado. Entre as torres de Hani, Nim, Resta um, e coisas do tipo escritas em guardanapos do So Paulos, Tams lembrou-se de algo e me perguntou: Emanuel, voc conhece o jogo do 15 out of 3 (15 de 3)? Respondi que no. Ele ento me deu a formulao do jogo: (O jogo 15 out of 3) os nmeros de 1 a 9 esto sobre a mesa. Dois jogadores alternadamente escolhem nmeros para si (sem repetio) e ganha quem primeiro completar 15 somando trs de seus nmeros. (sugiro agora que os leitores joguem um pouquinho antes de prosseguir e desvendar o mistrio). Pensei comigo mesmo: Hummm... isso no me parece estranho.... Eu disse: Bem, minha intuio me leva a crer que o primeiro jogador est em melhor situao para negociar do que o segundo, pois vai receber mais nmeros ao final... Toms foi adiante e disse:
EUREKA! N27, 2008

13

Sociedade Brasileira de Matemtica

Sim, voc est correto. Vamos jogar! Voc comea, escolha o seu primeiro nmero. Est bem. Eu escolho o 5. Nesse momento ele parou e me olhou curioso. Por que voc escolheu o 5? perguntou. Eu disse: No sei exatamente o que , mas algo me faz lembrar um quadrado mgico, e como o 5 sempre est no meio, achei que tinha mais chances de ganhar... Sua intuio mais uma vez est correta. Muito bem. Naturalmente, voc deve saber qua a menos de rotaes e reflexes a configurao do quadrado mgico nica. Por que a gente no desenha um quadrado mgico 3 3 aqui e tenta jogar olhando para ele? Aps um minuto tentando lembrar como se faz um quadrado mgico, desenhamos no guardanapo:
6 1 8 7 5 3 2 9 4

Quando vi os nmeros na mesa matei a charada. Um quadrado mgico de 3 3 como visto acima uma disposio dos nmeros 1, 2, 3, ..., 9, sem repetio, de modo que cada linha, coluna ou diagonal some a mesma quantidade. Nesse caso a soma comum ser 15 e o que vemos acima so todas as maneiras possveis de se escrever 15 como soma de 3 nmeros: 15 = 9 + 1 + 5 = 9 + 2 + 4 = 8 + 1 + 6 = 8 + 2 + 5 = = 8 + 3 + 4 = 7 + 2 + 6 = 7 + 3 + 5 = 6 + 5 + 4. Se pensarmos ento que o primeiro jogador marca X sobre os nmeros do quadrado mgico e o segundo marca O, o objetivo do jogo passa a ser completar uma linha, coluna ou diagonal com seus smbolos. O jogo 15 out of 3 que ele me props nada mais do que uma formulao equivalente, belssima e engenhosa,
EUREKA! N27, 2008

14

Sociedade Brasileira de Matemtica

do milenar jogo da velha (em ingls tic-tac-toe). Fiquei pasmo, havia ganho o meu dia. Por alguns momentos no consegui parar de pensar na beleza e no poder da matemtica, presente at nos mnimos detalhes da nossa vida. Senti-me orgulhoso de poder ser um pesquisador que tenta compreender essa cincia e pequenos fatos como esse me fazem, a cada dia, ter mais conscincia de que ela muito maior do que ns.

Observamos: A histria acima se passou no dia 02 de maio de 2007. Fiquei com o guardanapo como recordao. O jogo 15 out of 3 um belo exemplo para se mostrar como uma pessoa que sabe matemtica realmente pode levar vantagem sobre uma pessoa menos interessada pelo assunto. Todos sabemos que o jogo da velha no admite estratgia vencedora, mas mesmo assim o professor Tams Hausel jogava o 15 out of 3 com seus alunos e ganhava na maioria das vezes. Naturalmente, ele sempre tinha seu quadrado mgico para consultas. At que um dia ele esqueceu-se do quadrado mgico em casa e foi derrotado por uma aluna.

Guardanapo da discusso no So Paulos


No verso do guardanapo acima, h outras discusses tambm belssimas sobre as torres de Hani e verses relacionadas (de formas mais engenhosas do que a
EUREKA! N27, 2008

15

Sociedade Brasileira de Matemtica

analogia acima) inventadas pelo Dr. Rodrguez-Villegas. Ele ainda est buscando a melhor formulao para seu jogo para poder patente-lo e disponibiliz-lo ao pblico em geral. Isso ento vai ficar para uma outra histria.
PROBLEMA 1: Prove que em um quadrado mgico 3 3, como foi descrito acima:

(a) a soma comum deve ser 15. (b) o nmero do centro deve ser 5.
PROBLEMA 2: Usando o problema anterior, prove que s existe um quadrado mgico 3 3 (a menos de rotaes e reflexes). Verifique tambm que no h estratgia vencedora para o jogo da velha, em outras palavras, se os dois jogadores jogam certo, sempre d empate.

REFERNCIAS: [1] Para ver outras discusses sobre jogos e invariantes, h outras listas em minha pgina pessoal: http://www.math.utexas.edu/users/ecarneiro na seo math olympiads. [2] Para mais informaes sobre os trabalhos e o curso (Math, Puzzles and Computers) do Dr. Rodriguez-Villegas sua pgina pessoal www.math.utexas.edu/users/villegas/S07. Um dos jogos que ele criou est descrito no paper: Rodriguez Villegas, F.; Sadun, L.: Voloch, J.F. Blet: a mathematical puzzle. Amer. Math. Monthly 109 (2002), no. 8, 729740. [3] URL: pgina pessoal do Dr. Hausel http://www.math.utexas.edu/users/hausel

EUREKA! N27, 2008

16

Sociedade Brasileira de Matemtica

SUBSTITUIES ENVOLVENDO NMEROS COMPLEXOS


Diego Veloso Ucha Nvel Avanado
bastante til em problemas de olimpada onde temos igualdades ou queremos encontrar um valor de um somatrio fazermos substituies por nmeros complexos aliada a outras ferramentas. Para alguns problemas que possuam equaes com funes seno e co-seno importante saber a frmula de Euler que escreve um nmero complexo na forma polar o que simplifica quando fazemos multiplicaes ou somatrios. Um nmero complexo pode se escrever na sua forma trigonomtrica

(cos + isen )

ou na sua forma polar

e i de onde temos que

cos + isen = ei (Frmula de Euler). Segundo essa equao podemos fazer = ou = , de onde temos: ei = cos + isen (I) e i = cos isen (II) Somando I com II, temos:

cos =
Subtraindo I de II, temos:

e i + e i 2

ei e i 2i Segundo a frmula de Euler podemos verificar imediatamente a frmula de De Moivre: sen =


Para todo n natural temos que ( cos + isen ) = cos(n ) + isen(n ) . No seguinte problema da OIMU, quais idias imediatas poderamos ter sem conhecer a frmula de Euler?
n

PROBLEMA 1: (OIMU 2001)

Calcule:

cos cos n
n =1

2 n

(n 1) ... cos n

n cos n

EUREKA! N27, 2008

17

Sociedade Brasileira de Matemtica

2 (n 1) n Pn = cos cos ... cos cos , observe n n n n k = 0 . Portanto de imediato que se n = 2k ento P2 k = 0 pois cos 2k
SOLUO: Seja

considere n = 2k + 1 ento

2 2k (2k + 1) P2 k +1 = cos cos ... cos cos 2 k + 1 2k + 1 2k + 1 2k + 1


i 2k + 1 i Observe que cos = cos 2k + 1 2k + 1

P2 k +1

2 k k +1 = cos cos ... cos (1) , 2k + 1 2k + 1 2k + 1

Considere
~ 2 (k 1) k P 2 k +1 = cos cos ... cos cos 2 k + 1 2k + 1 2 k + 1 2k + 1 j j w2 j + 1 i w +w j = = portanto Fazendo w = e 2 k +1 ento cos 2 2w j 2k + 1

~ w2 + 1 w4 + 1 w2 k + 1 P 2 k +1 = ... . Faa ento a seguinte multiplicao no 2 k 2 w 2 w 2 w numerador e no denominador ~ w2 + 1 w4 + 1 w2 k + 1 ( w + 1)( w3 + 1)...( w2 k 1 + 1) P 2 k +1 = ... , 2 k 3 2 k 1 + 1) 2 w 2 w 2w ( w + 1)( w + 1)...( w agrupando no numerador os termos tais que

( w 2 j + 1)( w 2 k +12 j + 1) = ( w 2 k +1 + w 2 k +1 2 j + w 2 j + 1) = w 2 j + w 2 k +1 2 j , j que w 2 k +1 = 1 , com j variando de 1 at k. Agrupando agora os termos do


denominador podemos ver que

EUREKA! N27, 2008

18

Sociedade Brasileira de Matemtica

1 ( w2 + w2 k 1 )( w4 + w2 k 3 )...( w2 k + w) , 2k ( w2 k + w)( w2 + w2 k 1 )...( wk +1 + wk ) podemos simplificar a expresso para P 2 k +1 =


~

e usando que

w 2 k +1 = 1

P 2 k +1 =

1 ( w2 w2 )( w4 w4 )...( w2 k w2 k ) . 2k ( w1 + w)( w2 w2 )...( w k + wk )

Agora,

olhando

para

2j 2 j numerador, podemos escolher os termos ( w w ) tais que 2 j > k que so

m termos (para algum m) e substitu-los por (1)( w2 k +1+ 2 j w (2 k +1 2 j ) ) de forma que o numerador e o denominador sero iguais a menos de um sinal (e do fator 2 k ), i.e,

P 2 k +1

~ 1 ( w 1 + w)( w 2 w 2 )...( w k + w k ) m m 1 = k ( 1 ) P 2 k +1 = ( 1) 1 2 2 k k 2 ( w + w)( w w )...( w + w ) 2k

Portanto temos que P2 k +1 = (1) como Pn = 0 para todo n par,

k +1

1 k +1 1 = (1) k +1 k , e ento P 2 k +1 = (1) 2k 2 4


~

n =1 n =1

Pn =
n

k =0

P2 k +1 =

1 1 1 = 1 + + ... (soma de P.G infinita), e assim 4 4 16 k =1

1 4 = . 1 5 1+ 4

Observao: Esse problema pede para demonstrarmos um resultado relacionado aos polinmios de Chebyshev do segundo tipo.

PROBLEMA 2: (OBM U 2001)


x Seja f ( x) = e senx . Calcule f (2001) (0) . (Denotamos derivada de ordem n no ponto x; assim, f (2) ( x) = f '' ( x) ).

por

f ( n ) ( x) a

SOLUO:

f ( x) = e x senx = e x

e ix e ix e (i 1) x e ( i +1) x = aps n 2i 2i 2i

derivaes teremos
EUREKA! N27, 2008

19

Sociedade Brasileira de Matemtica

1 (i 1) n e x (i 1) ( 1 i ) n e ( i +1) x ; para n = 2001 e x = 0, temos 2i 1 4 4 que f (2001) (0) = (i 1) 2001 (1 i ) 2001 sendo que (i 1) = ((i + 1)) = 4. 2i f ( n ) ( x) = Assim
500 500 1 4500 (i 1) 4 (i 1) ((i + 1)) 4 ( 1 i ) = (2i ) = 21000. Ou 2i 2i tro caminho possvel para a soluo desse problema seria: aps 4 derivaes de f perceber um ciclo e assim calcular f (2001) (0) , mtodo esse mais trabalhoso do que o apresentado.

f (2001) (0) =

PROBLEMA 3: (IMO 1963)

Prove que cos

3 1 2 + cos = cos 7 2 7 7

SOLUO: Fazendo

w = e 7 o problema se torna equivalente a demonstrar que:

w + w 1 w 2 + w 2 w 3 + w 3 1 w2 + 1 w4 + 1 w6 + 1 + = + =1 w 2 2 2 2 w2 w3

( w 4 + w 2 ) ( w 5 + w) + ( w 6 + 1) = w 3 w 6 w 5 + w 4 w 3 + w 2 w + 1 = 0
. Veja que isso a soma dos termos de uma P.G cujo primeiro termo 1 e a razo w. Somando a P.G:

S PG =

i ( w) 7 1 11 = = 0 . Lembre que w = e 7 w7 = 1 w 1 w 1

e portanto a igualdade realmente verdadeira. O seguinte resultado (muito conhecido) tem por objetivo mostrar a importncia dos nmeros complexos em problemas de alto grau de dificuldade e que aparentemente no tm nenhuma conexo com nmeros complexos.

EUREKA! N27, 2008

20

Sociedade Brasileira de Matemtica

1 2 PROBLEMA 4: Prove que 2 = 6 n =1 n

SOLUO: Sabemos que para 0 < x <

a desigualdade senx < x < tan x

2 verdadeira. De onde segue que cot x <

x=

k com k = 1,2,..., m e somando de k = 1 at k = m ns obtemos 2m + 1


2 (i) cot
k =1 m

1 < 1 + cot 2 x . Agora fazendo x2

(2m + 1) 2 k 2m + 1 2

m 1 k + m cot 2 2 2m + 1 k =1 k k =1 m

Observe que essa inequao est prxima da desejada, a idia agora tentar mostrar que quando m o termo central fica imprensado entre dois limites que convergem para um mesmo valor. Para isso vamos usar um truque que usa nmeros complexos. Pela lei de De Moivre e usando binmio de Newton temos :

cos(nt ) + isen(nt ) = (cos t + sent )n = sennt (cot(t ) + i)n = sennt

Fazendo n = 2m + 1 e igualando as partes imaginrias, temos:

i k
k =0

cot nk (t )

2m + 1 sen((2m + 1)t ) 2m + 1 2 m 2 m 1 m = (cot t ) (cot t ) + ... + (1) . (*) 2 m +1 1 3 sen t


Agora podemos tratar essa igualdade por meio de um polinmio

2m + 1 m 2m + 1 m 1 m Pm ( x) = x x + ... + (1) 1 3
Substituindo t = pois

sen (2m + 1) =0 2m + 1

k k 2 )=0, em (*) para 1 k m nos d Pm (cot 2m + 1 2m + 1 k k


e

sen 0. 2m + 1

Ento,

EUREKA! N27, 2008

21

Sociedade Brasileira de Matemtica

k , k = 1,..., m so as m razes de Pm cuja soma 2m + 1 2m + 1 3 m(2m 1) m k = 2 = cot (ii) 3 2m + 1 2m + 1 k =1 1 x k = cot 2


De (i) e (ii) segue

m(2m 1) (2m + 1) 2 3 2
Multiplicando essas desigualdades por ao resultado desejado. Exerccios para treinamento:

k
k =1

1
2

m+

m(2m 1) 3

2
(2m + 1) 2

e fazendo m chegamos

(2n + 1) sen 1 2 Prove que + cos( ) + cos(2 ) + ... + cos(n ) = 2 2sen 2 PROBLEMA 6: (IME-2000/2001) Dois nmeros complexos so ortogonais se suas representaes grficas forem perpendiculares entre si. Prove que dois nmeros complexos Z1 e Z 2 so ortogonais se e somente se:

PROBLEMA 5: (IME 1990/1991)

Z1 Z 2 + Z 1 Z 2 = 0
PROBLEMA 7: Prove que

sen
k =1

k = cot n 2n

PROBLEMA 8: Prove a identidade trigonomtrica:

cos n ( ) =
PROBLEMA 9: (IME 2005/2006)

1 2n

cos ( (n 2k ) ) . k
k =1

Sejam as somas S 0 e S1 definidas por


EUREKA! N27, 2008

22

Sociedade Brasileira de Matemtica


0 3 6 9 3[ n / 3] S 0 = Cn + Cn + Cn + Cn + ... + C n 1 4 7 10 3[( n 1) / 3]+1 S1 = C n + Cn + Cn + Cn + ... + C n

Calcule os valores de S 0 e S1 em funo de n, sabendo que [r] representa o maior inteiro menor ou igual ao nmero r.
PROBLEMA 10: (Putnam 1970)

Prove que a srie de potncias de e cos(bx) (com a e b positivos) ou no tem nenhum coeficiente zero ou possui infinitos zeros.
ax

PROBLEMA 11: Ache uma frmula geral para:

(k + 1) cos
k =0

n 1

2 k n

PROBLEMA 12: (OBM Nvel U 2004)

Calcule o valor de

(3k + 1)(3k + 2)(3k + 3) .


k =0

PROBLEMA 13: (IMO 1974) Prove que o nmero

por 5 para qualquer inteiro n 0 .


PROBLEMA 14: Calcule o valor de

2 2k + 1
K =1

2n + 1

3k

no divisvel

n . k 2(mod3) k

PROBLEMA 15: (IMC 99) Atiramos um dado (com faces de nmero 1, 2,..., 6) n

vezes. Qual a probabilidades de que a soma dos valores obtidos seja mltiplo de 5? Admita que as faces sejam igualmente provveis. Dica: Use a funo

x + x 2 + x3 + x 4 + x5 + x 6 f ( x) = . 6 PROBLEMA 16: Mostre que dados n pontos no crculo unitrio sempre existe um outro ponto no crculo unitrio tal que o produto de suas distncias aos n pontos dados maior ou igual a 2.
PROBLEMA 17: (OBM Nvel U 2007)

EUREKA! N27, 2008

23

Sociedade Brasileira de Matemtica

Dados nmeros reais a1 , a2 ,..., an no todos nulos, encontre o (menor) perodo da funo

f ( x) = ak cos(kx).
k =1

PROBLEMA 18: (Mikls Schweitzer-1956)

Ache o mnimo de mx 1 + z , 1 + z 2 complexos.


PROBLEMA 19: (IMO 1995)

} se z percorre todos os nmeros

Seja p um primo mpar. Ache o nmero de subconjuntos A de { 1,2,...,2 p} tais que a) A tem exatamente p elementos b) A soma de todos elementos de A divisvel por p Dica: Use o polinmio f ( x, y ) = (1 + xy )(1 + x 2 y )...(1 + x 2 p y ).

BIBLIOGRAFIA [1] E. Lozansky. C. Rousseau, Wining Solutions, Springer Velrlag, New York, 1996. [2] Contests in Higher Mathematics, Hungary 19491961: in memoriam Mikls Schweitzer, eds.: G. Szsz, L. Gehr, I. Kovcs and L. Pintr, Akadmiai Kiad, Budapest, 1968. [3] URL: http://www.ime.eb.br (Site do Instituto Militar de Engenharia)

[4] URL : http://www.obm.org.br (Site da Olimpada Brasileira de Matemtica)

EUREKA! N27, 2008

24

Sociedade Brasileira de Matemtica

INTEGRAIS DISCRETAS
Nvel Avanado
Eduardo Poo

Integral discreta: dizemos que F (n) integral discreta de f (n) se e somente se:

F ( n + 1) F ( n) = f (n) , para n inteiro (a princpio).


Da mesma forma, dizemos que f (n) a derivada discreta de F (n) .
Notao:

f ( n) = F ( n)

Utilidade: conhecida a integral discreta F (n) da funo f (n) , temos condies de

fazer o somatrio:

f (k ) = F (b + 1) F (a) , a e b inteiros
k =a

A integral discreta transforma uma soma em soma telescpica. Sabendo de algumas propriedades, possvel trabalhar dinamicamente com integrais discretas para obter frmulas novas a partir de outras conhecidas. Aqui, no queremos provar que uma funo dada integral discreta de outra, pois essa verificao simples. Queremos obter ferramentas que nos possibilitem ACHAR integrais discretas de forma rpida, para no final poder calcular o valor de um somatrio que tenha surgido de algum problema. Em alguns casos, suficiente saber a cara da integral discreta (ou seja, se um polinmio, exponencial etc). Algumas integrais discretas (o exerccio de verificao simples):

c = cn

n.n! = n!

qn q 1

log

n = log a ( n 1)!

EUREKA! N27, 2008

25

Sociedade Brasileira de Matemtica

k cos kn 2 sen kn = k 2sen 2 n n sen ( 2n 1) sen 2 n = 2 4sen1 n n sen ( 2n 1) cos2 n = 2 + 4sen1 n n n k = k + 1 n n + k n +k n = n 1


n

k sen kn 2 cos kn = k 2sen 2


n

Propriedades

1) Assim como integrais contnuas (as primitivas), existem vrias integrais discretas para uma dada funo, e todas elas diferem por uma constante.
Exemplo: 2n e 2n + 1 so integrais discretas de

f (n) = 2n . Verifique pela

definio! 2) Integrao discreta uma transformao linear:

[ a. f (n) + b.g (n)] = a f (n) + b g (n) , para constantes a e b.


A igualdade nos fornece uma integral discreta para a funo do lado esquerdo, lembre-se que podemos somar constantes do lado direito e continuar com uma integral discreta. 3) Integral discreta do produto (por partes): sendo

f ( n) = F ( n) e

g (n) = G(n) , ento:


EUREKA! N27, 2008

26

Sociedade Brasileira de Matemtica

F (n) g (n) = F (n)G(n) f (n)G(n + 1)


Exemplo: Calcule

n sen n e n
2

sen n , comparando com o clculo de

x sen xdx e x

sen xdx

4) Sendo f ( x, n) uma funo das variveis x e n, derivvel na varivel x, ento:

n f ( x , n ) = x f ( x, n ) x
n

Podemos usar a prpria varivel n, se a funo tiver derivada nessa varivel:

dn f (n) = dn f (n)
Exemplo: Calcule

nx

, com x uma constante em relao a n.

5) Seguindo um caminho anlogo, temos que:

dx + Cn ( f ( x, n)dx ) = f ( x, n )
n

Para alguma constante C. Essa constante encontrada atravs de valores iniciais conhecidos das funes.
Exemplo: Prove que

(1) k +1 xn ln 2 = + x 1 dx k k =1 1
n 0

Aplicao: Soma de potncias consecutivas.

Seja a seguinte funo:

EUREKA! N27, 2008

27

Sociedade Brasileira de Matemtica

S m (n) = k m = 1m + 2m + ... + n m
k =1

H uma frmula recursiva em que podemos calcular S m (n) a partir de valores anteriores (tente prov-la como exerccio):
m 1 m + 1 (m + 1) S m (n) = (n + 1) m +1 1 S k ( n) k =0 k

O problema dessa frmula a praticidade: precisamos de todas as funes anteriores, e ainda assim faremos um trabalho algbrico grande. Com integrais discretas, conseguimos obter S m (n) a partir de S m 1 ( n) apenas com um trabalho aritmtico. Inicialmente, se queremos S m ( n) , queremos sua integral discreta

. Usando

a propriedade que nos permite trocar a integral discreta com a contnua (escolhendo a prpria varivel n como varivel de integrao contnua):

( n
n

m 1

n dn = n m 1 dn + Cn

A integral contnua pode ser realizada sem problemas:

nm n m 1 dn + Cn m = n
n

Renomeando a constante a ser encontrada:

n = m n m 1 dn + Cn

Essa constante pode ser encontrada pela diferena entre integrais discretas quando n = 0 , fornecendo o oposto da soma dos outros coeficientes j obtidos pela integrao contnua. Resumindo:

EUREKA! N27, 2008

28

Sociedade Brasileira de Matemtica

Se
n

n
m

m 1

= am n m + am 1n m 1 + ... + a2 n 2 + a1n , ento:

= bm +1n m +1 + bm n m + ... + b2 n 2 + b1n


m +1 m ak 1 , para k = 1, 2,..., m + 1 , e b0 = bk . k k =1

Com bk =

Alguns valores:

1 = n
n2 n 2 2 n n3 n 2 n 2 n = + 3 2 6 4 n n n3 n 2 3 n = + 4 2 4 5 4 n n n n3 n n4 = 5 2 + 3 30

n =

Aplicao: Soma de potncias multiplicadas por progresso geomtrica

Agora procuraremos

n
n

x n , com x uma constante em relao a n. Observe:

n n d n m n n m 1 n m n m 1 n n x mn x n x x m n x x = + ln = + ln ( ) nm xn dn

Das formas iniciais de

x n , encontramos uma funo da forma:

m 1 n

x = x n ( am 1n m 1 + am 2 n m 2 + ... + a2 n 2 + a1n )

com as constantes ak sendo funes de x, mas no dependendo de n. natural procurar uma integral discreta com a seguinte forma:

EUREKA! N27, 2008

29

Sociedade Brasileira de Matemtica

x n = x n ( bm n m + bm 1n m 1 + ... + b2 n 2 + b1n )
m ak 1 , k

Essa forma pode ser encontrada, e os coeficientes satisfazem bk =

i = 1, 2,..., m e b0 =

x m bk . 1 x k =1

Alguns valores: n n 1) x n +1 nx n ( xn n = x 1) n x nx = 2 2 ( , x 1 ( x 1) ( x 1)

n2 n
n n n

= 2n ( n 2 )

2 n = 2 n ( n 2 4n + 6 )
n

n 2
3

= 2n ( n3 6n 2 + 18n 26 )

2n = 2n ( n 4 8n3 + 36n 2 104n + 150 )

Problemas

1- Calcule as seguintes integrais discretas: a)

n 3 n 2
1

e)

2n (n 1) (n + 1)! 1 +n 1 n +1 + n 1

b)

3
n

f)

n n

c)

(n + 2)n!
n

g)

n d) 2

h)

cos n cos(n + 1)

EUREKA! N27, 2008

30

Sociedade Brasileira de Matemtica

2- Calcule: lim
n

sen 2 k n n k =1
n

3- Calcule lim

k =1 n n

k
k =1

.2k
n

k .2
3 n m

4- Calcule lim

k
k =1

n m +1 m +1

nm

, com m inteiro positivo.


n 1 1 , sendo hn = . 3 k =1 k n =1 n

5- Prove que

n =1

2n + 1

(n

+ n)

hn =

6- Ache a derivada (contnua) da funo gama ( n) para n inteiro positivo, sabendo que '(1) = , a constante de Euler (um valor conhecido) e (1) = 1 . A funo gama satisfaz ( x + 1) = x ( x) , para todo x real, assim (n) = ( n 1)! para n inteiro positivo. 7- (OBM2002) O dimetro de um conjunto S R definido como sendo D( S ) = max( S ) min( S ) . O conjunto vazio, por definio, tem dimetro igual a zero. Calcule a soma dos dimetros de todos os subconjuntos de A = {1, 2,3,..., n} , em funo de n.

REFERNCIAS:

[1] Uma referncia sobre somatrios e algumas consideraes histricas sobre o raciocnio humano e implementao de algoritmos em computadores: A = B, Marko Petkovsek, Herbert S. Wilf, Doron Zeilberger.

EUREKA! N27, 2008

31

Sociedade Brasileira de Matemtica

PRODUTOS NOTVEIS
Nvel Iniciante
Uma lista de problemas Onofre Campos

1 1 = 5, determine o valor de x 2 + 2 . x x 1 Soluo: Elevando ambos os membros da equao x + = 5 ao quadrado, x obtemos: 1 1 x 2 + 2 x + 2 = 25, x x 1 e da, x 2 + 2 = 23. x
1. Se x um nmero real tal que tal que x + 2. Fatore a expresso E = x3 5 x 2 x + 5. Soluo: Temos

E = x3 5 x 2 x + 5 = x 2 ( x 5) ( x 5)
= ( x 5)( x 2 1) = ( x 5)( x 1)( x + 1).
3. Simplifique a expresso

A=

x2 y2 z2 + + . ( x y )( x z ) ( y z )( y x) ( z x)( z y )

Soluo: Note que podemos escrever a expresso acima da seguinte forma:

x2 y2 z2 + . ( x y )( x z ) ( x y )( y z ) ( x z )( y z ) Assim, reduzindo a expresso ao mesmo denominador comum vem: x2 ( y z) y 2 ( x z) + z 2 ( x y) A= . ( x y )( y z )( x z ) Por outro lado, desenvolvendo o denominador, obtemos: ( x y )( y z )( x z ) = ( xy xz y 2 + yz )( x z ) A=
= x 2 y xyz x 2 z + xz 2 xy 2 + y 2 z + xyz yz 2 = x 2 ( y z ) y 2 ( x z ) + z 2 ( x y ).
EUREKA! N27, 2008

32

Sociedade Brasileira de Matemtica

Portanto: x2 ( y z) y 2 ( x z) + z 2 ( x y) A= 2 = 1. x ( y z) y 2 ( x z) + z 2 ( x y)
4. Se x + y + z = 0, mostre que x 3 + y 3 + z 3 = 3 xyz. Soluo: Observe que

0 = ( x + y + z )3 = x3 + y 3 + z 3 + 3( x + y )( y + z )( x + z ). Como x + y = z , y + z = x e x + z = y, ento: x 3 + y 3 + z 3 + 3( y )( x)( y ) = 0 x 3 + y 3 + z 3 = 3 xyz.


5. Calcule o valor da expresso

(2004)3 (1003)3 (1001)3 S = . 2004 1003 1001


Soluo: Vamos tomar x = 1003 e y = 1001. Dessa forma, a expresso S se reduz

a:

S=

( x + y )3 x 3 y 3 . xy ( x + y )

Mas, como sabemos, ( x + y )3 = x 3 + 3 x 2 y + 3xy 2 + y 3 . Dessa forma, obtemos: 3 x 2 y + 3 xy 2 3xy ( x + y ) S= = = 3. xy ( x + y ) xy ( x + y )


6. Sabendo que x, y e z so reais satisfazendo xyz = 1, calcule o valor da

expresso:

1 1 1 . + + 1 + x + xy 1 + y + yz 1 + z + xz Soluo: Como xyz = 1, ento x 0, y 0 e z 0. Assim, z x 1 A= + + z (1 + x + xy ) x(1 + y + yz ) 1 + z + xz z x 1 = + + z + xz + xyz x + xy + xyz 1 + z + xz A=


EUREKA! N27, 2008

33

Sociedade Brasileira de Matemtica

z xz 1 z x 1 + + = + + 1 + z + xz 1 + x + xy 1 + z + xz 1 + z + xz 1 + z + xz 1 + z + xz 1 + z + xz = 1. = 1 + z + xz a2 b2 7. Se ab = 1 e a 2 + b 2 = 3, determine 2 + 2 + 2. b a Soluo: Temos: a2 b2 a 4 + 2a 2 b 2 + b 4 (a 2 + b 2 ) 2 + + 2 = = = 9. b2 a 2 a 2b2 ( ab) 2

x y z x2 y 2 z 2 a b c + + =1 e + + = 0, ento 2 + 2 + 2 = 1. a b c x y z a b c x y z Soluo: Elevando a equao + + = 1 ao quadrado, obtemos: a b c 2 2 x y z2 x y y z x z + 2 + 2 + 2 + + = 1, 2 a b c a b b c a c ou seja, x2 y 2 z 2 xyc + xzb + yza + + + 2 = 1. a2 b2 c2 abc a b c Por outro lado, da equao + + = 0, temos xyc + xzb + yza = 0. Logo, x y z
8. Prove que se

x2 y 2 z 2 + + = 1. a 2 b2 c2 9. Se a, b e c so trs nmeros distintos e satisfazem as equaes: a 3 + pa + q = 0 3 b + pb + q = 0 3 c + pc + q = 0,


calcule a + b + c.
Soluo: Multiplicando a segunda equao por 1 e somando com a primeira,

obtemos: a 3 b3 + p (a b) = 0, ou ainda,

EUREKA! N27, 2008

34

Sociedade Brasileira de Matemtica

(a b)(a 2 + ab + b 2 ) + p (a b) = 0, (a b)(a 2 + ab + b 2 + p) = 0. Como a b 0, pois os nmeros so distintos, obtemos: a 2 + ab + b 2 + p = 0. (*) Analogamente, multiplicando a terceira equao por 1 e somando com a primeira equao, obtemos: a 2 + ac + c 2 + p = 0. (**) Agora, multiplicando (**) por 1 e somando com (*), obtemos: ab ac + b 2 c 2 = 0, a (b c) + (b c)(b + c) = 0, (b c)(a + b + c) = 0. Da, como b c 0, segue que a + b + c = 0.
10. Sejam a, b e c nmeros reais distintos e no nulos. Se a + b + c = 0, mostre que a b a b b c c a c + + + + = 9. a b a b b c c a c a b bc ca Soluo: Faamos x = , y= e z= . c a b Assim, devemos provar que 1 1 1 ( x + y + z ) + + = 9, x y z ou seja, x+ y+z x+ y+z x+ y+ z + + = 9, x y z ou ainda, y+z x+z x+ y y+z x+z x+ y 1+ +1+ +1+ =9 + + = 6. x y z x y z Mas, x + y a b b c b a 2 ab + bc c 2 b = + = ac ca z a c a c

(a c)(a + c) b( a c) b ( a 2 c 2 ) b( a c ) b = ac ca ac ca

EUREKA! N27, 2008

35

Sociedade Brasileira de Matemtica

(a c)(a + c b) b (b b)b 2b 2 = = . ac ca ac ac y + z 2c 2 x + z 2a 2 Analogamente, conclumos que = . Logo, pelo exerccio e = y bc x ab 4, segue que a 3 + b3 + c 3 y + z x + z x + y 2a 2 2b 2 2c 2 3abc + + = + + = 2 = 2 = 6, x y z bc ac ab abc abc = como queramos provar.
Exerccios Propostos

1. Fatore a expresso S = x 4 + x 2 + 1. 2. Determine a expresso que deve ser multiplicada por x 3 2 + 2 3 x para obtermos 2 x( x 2 + 4). 3. Calcule o valor da expresso

( x + 1) 2 ( x 2 x + 1) 2 ( x + 1) 2 ( x 2 + x + 1) 2 S = . ( x 3 1) 2 ( x 3 + 1) 2 x y 4. Se x 2 + y 2 = 3 xy, calcule 1 + 1 + . y x 5. Simplifique

(x

+ y 2 + z 2 + xy + yz + xz ) ( x + y + z ) ( x 2 + y 2 + z 2 ) .
2 2

6. Fatore as seguintes expresses: (a) x 3 + 5 x 2 + 3 x 9; (b) ( x y ) z 3 ( x z ) y 3 + ( y z ) x 3 ; (c) ( x 2 + x + 3)( x 2 + x + 4) 12; (d) x 4 + 4 y 4 ; (e) ( x y )3 + ( y z )3 + ( z x)3 ; (f) ( x + y + z )3 x3 y 3 z 3 ; (g) (a + 2b 3c)3 + (b + 2c 3a )3 + (c + 2a 3b)3 . 7. Simplifique as expresses: 1 1 1 1 1 (a) ; 2 4 1 x 1 + x 1 x 1 x 1 x8
EUREKA! N27, 2008

36

Sociedade Brasileira de Matemtica

(b) (c)

1 1 1 ; + + ( x y )( x z ) ( y x)( y z ) ( z x)( z y )

( x 2 y 2 )3 + ( y 2 z 2 )3 + ( z 2 x 2 ) 3 . ( x y )3 + ( y z )3 + ( z x )3 x y z + + = 0, ento 8. Prove que se yz zx x y x y z + + = 0. ( y z ) 2 ( z x)2 ( x y )2


9. Para que os valores de a a expresso a 4 + 4 um nmero primo? 10. Prove que se a + b + c = 0 ento a 5 + b5 + c 5 a 3 + b3 + c3 a 2 + b 2 + c 2 = . 5 3 2 11. Mostre que (a + b)7 a 7 b7 = 7 ab( a + b)( a 2 + ab + b 2 ) 2 . 12. Prove que se a + b + c = 0, ento a 7 + b 7 + c 7 a 5 + b5 + c 5 a 2 + b 2 + c 2 = . 7 5 2 13. Se a, b e c so reais no nulos que satisfazem a + b + c = 0, calcule (a 3 + b3 + c3 ) 2 (a 4 + b 4 + c 4 ) . (a 5 + b5 + c5 ) 2 14. Prove que se x, y e z so racionais distintos ento a expresso 1 1 1 + + 2 2 ( y z) ( z x) ( x y )2 um quadrado perfeito. 15. Fatore 8( x + y + z )3 ( x + y )3 ( y + z )3 ( x + z )3 .

EUREKA! N27, 2008

37

Sociedade Brasileira de Matemtica

OLIMPADAS AO REDOR DO MUNDO


Apresentamos, como sempre, questes que no so encontradas facilmente na Internet. Divirtam-se e enviem as suas solues. Continuamos disposio na OBM para aqueles que estiverem interessados na soluo de algum problema particular. Para tanto, basta contactar a OBM, atravs de carta ou e-mail.

Bruno Holanda Carlos Augusto David Ribeiro

Primeiramente vamos aos problemas propostos deste nmero


224.(Balcnica Junior - 2007) Seja a um real positivo tal que a = 6( a + 1). Prove
3

que a equao x 2 + ax + a 2 6 = 0 no possui soluo real.


225.(Bulgria - 2007) Ache todos os inteiros positivos x, y tais que o nmero

( x 2 + y )( y 2 + x) a quinta potncia de um primo.


BAC = 60. Seja O o circuncentro e H o ortocentro. A reta OH encontra AB em P e AC em Q. Prove que PO = HQ.
227.(Austria 226.(Inglaterra 2007) Seja ABC um tringulo acutngulo com AB > AC e

2007) Sejam 0 < x0 , x1 ,..., x669 < 1 reais distintos. Mostre que

existem i, j {0,1,..., 669} para os quais

0 < xi x j ( x j xi ) <
228.(Bulgria -

1 . 2007

2008) Para cada inteiro positivo n, seja

(n) a quantidade de

divisores de n maiores que 2008. Defina an = 0 se (n) par e an = 1 caso contrrio. O nmero = 0, a1a2 ...an ... racional?
229.(Bielorrssia - 2001) No losango ABCD, A = 60. Os pontos F, H, e G esto sobre os segmentos AD, DC e AC de modo que DFGH um paralelogramo. Prove que FBH um tringulo eqiltero.
EUREKA! N27, 2008

38

Sociedade Brasileira de Matemtica

230.(Rssia - 2007) Sejam a, b, c nmeros reais. Prove que pelo menos uma das trs

equaes

x 2 + (a b) x + (b c) = 0, x 2 + (b c) x + (c a) = 0, x 2 + (c a) x + (a b) = 0,
possui soluo real.

ABC , com ACB = 60, sejam AA1 e BB1 ( A1 BC , B1 AC ) as bissetrizes de BAC e ABC. A reta A1 B1 encontra o circuncrculo do tringulo ABC nos pontos A2 e B2 . (a) Sejam O e I o circuncentro e o incentro do ABC , respectivamente, prove que OI paralelo a A1 B1 .
231.(Bulgria - 2007) No tringulo

(b) Se R o ponto mdio do arco AB, no contendo o ponto C, P e Q so os pontos mdios de A1 B1 e A2 B2 , respectivamente, prove que RP = RQ.
232.(Romnia - 2007) Encontre todos os conjuntos A de pelo menos dois inteiros

positivos, tais que para quaisquer dois elementos distintos x, y A tenhamos

x+ y A. Aqui mdc(x, y) denota o mximo divisor comum de x e y. mdc( x, y )


233.(Romnia - 2007) Determine todas as progresses aritmticas infinitas de inteiros positivos, com a seguinte propriedade: existe N , tal que para qualquer primo p, p > N, o p-simo termo da seqncia tambm primo. 234.(Bulgria - 2007) O incrculo do tringulo acutngulo ABC toca os lados AB, BC e CA nos pontos P, Q e R respectivamente. O ortocentro H do tringulo ABC est sobre o segmento QR. (a) Prove que PH QR. (b) Sejam I e O o incentro e o circuncentro do ABC , respectivamente, e N o ponto comum entre o lado AB e ex-incrculo relativo a este lado. Prove que os pontos I, O e N so colineares. 235.(Olimpada Checa e Eslovaca 2007) Se x, y, z so nmeros reais no intervalo (1, 1) satisfazendo xy + yz + zx = 1, mostrre que:

6 3 (1 x 2 )(1 y 2 )(1 z 2 ) 1 + ( x + y + z ) 2 .
EUREKA! N27, 2008

39

Sociedade Brasileira de Matemtica

236.(Romnia 2007) Um conjunto de pontos no plano livre se no existe

tringulo eqiltero cujos vrtices esto entre os pontos do conjunto. Mostre que qualquer conjunto de n pontos no plano contm um subconjunto livre com pelo menos n pontos.
SOLUO DE PROBLEMAS PROPOSTOS NOS NMEROS ANTERIORES: 211. (Baltic Way 2004) Uma seqncia a1, a2, ... de nmeros reais no-negativos

satisfaz, para n = 1, 2, ..., as seguintes condies: ( a ) an + a2 n 3 n .

(b) a n +1 + n 2 a n (n + 1) . (i) Prove que an n para todo n = 1, 2, ... (ii) D exemplo de uma tal seqncia.
SOLUO DE ESTILLAC LINS MACIEL BORGES FILHO (BELM PA)

(i) Utilizando a desigualdade das mdias, temos:

an + n + 1 a n +1 + n a n + n + 1 a n +1 a n + 1 2 Supondo vlido que a n + k a n + k , temos que: a n + k +1 a n + k + 1 a n + k + 1 E como a desigualdade vale para k = 1 , fica provado por induo que a n + k a n + k , k . Em particular, para k = n , temos: a 2 n a n + n a n + a 2 n 2a n + n Finalmente, como a n + a 2 n 3n , temos: 3n a n + a 2 n 2a n + n 2a n 2n a n n (ii) Uma seqncia que satisfaz as condies a n = n + 1 . De fato, temos: (1) a n + a 2 n = n + 1 + 2n + 1 = 3n + 2 3n a n +1 + n 2 a n (n + 1) 2
(2) an+1 + n = 2n + 2 = 2(n + 1) = 2 (n + 1)(n + 1) = 2 an (n + 1) 2 an (n + 1)
212. (Baltic Way 2004) Seja P um polinmio com coeficientes no-negativos. Prove

que se P(1/x)P(x) 1 para x = 1, ento tal desigualdade se verifica para todo real positivo x.
EUREKA! N27, 2008

40

Sociedade Brasileira de Matemtica

SOLUO DE GELLY WHESLEY (FORTALEZA CE)

Para x > 0 temos P ( x) > 0.

Da condio dada, temos ( P (1)) 2 1. Agora, denote P ( x) = a0 x n + a1 x n 1 + ... + an . Ento:

1 1 1 P ( x) P = ( a0 x n + a1 x n 1 + ... + an ) a0 + a1 x x x (Por Cauchy-Schwarz)


n

n 1

+ ... + an

a0 a1 n n 1 ... a x a x a a + + + 0 1 n n x = x 1 n n 2 2 (a0 + a1 + ... + an ) = ( P(1)) 1.


213. (Baltic Way 2004) Ache todos os conjuntos X, consistindo de ao menos dois

inteiros positivos, tais que para todos m, n X, com n > m, exista um elemento k de X tal que n = mk2.
SOLUO DE ESTILLAC LINS MACIEL BORGES FILHO (BELM PA)

Suponha que o nmero 1 X . Logo, seja p X , com p > 1 , temos que deve existir k X , tal que p = k 2 , isto , p quadrado perfeito. Obviamente, k > 1 e com raciocnio anlogo, conclumos que k tambm deve ser quadrado perfeito e sua raiz quadrada deve pertencer a X . Ou seja, estendendo o raciocnio, todas as potncias de p da forma p
1
1

2n

, com n

tambm devem

pertencer a X , o que impossvel, dado que p 2 n para algum valor de n . Logo, 1 X . Vamos ento tentar montar o conjunto X . Para tal, vamos supor, inicialmente que X possua somente dois elementos p 1 e p 2 , com p 2 > p1 > 1 . Desta forma, temos que deve existir k X , tal que p 2 = p1 k 2 . Obviamente, k < p 2 e, portanto, a nica alternativa k = p1 e, portanto, p 2 = p1 . De fato, todo conjunto X = n, n 3 , com n N e n > 1 , satisfaz as condies do problema e somente tais conjuntos de dois elementos satisfazem, conforme verificado. Vamos ento supor que o conjunto X tenha mais do que dois elementos, isto , X = {p1 , p 2 , p3 , p 4 ,..., p r } , com p1 ,..., p r N . Suponha, sem perda de
EUREKA! N27, 2008

41

Sociedade Brasileira de Matemtica

generalidade, que

p r > p r 1 > ... > p 3 > p 2 > p1 > 1 . Com um raciocnio


3

anlogo ao pargrafo anterior, conclumos que p 2 = p1 . Seguindo o raciocnio, temos que: (1) p3 = p1 k1
3 2 2

(2) p3 = p1 k 2 onde k1 , k 2 X .

Obviamente, temos que k1 < p 3 , k 2 < p 3 e k1 k 2 . Portanto, como p3 o terceiro menor elemento do conjunto, s restam as possibilidades

k1 = p1 k 2 = p1
3

ou

k1 = p13 3 . Porm, no primeiro caso, temos que p3 = p1 = p 2 : absurdo! J no k 2 = p1


segundo caso, temos que: (1) p3 = p1 ( p1 ) 2 = p1 (2) p3 = p1 p1 = p1
3 2 5 5 7 3 7

Portanto, temos que p1 = p1 p1 = 0 ou p1 = 1 . Absurdo! Logo, no possvel que o conjunto X possua mais de 2 elementos. E assim, todos os conjuntos que satisfazem o enunciado so: X = n, n 3 , com n e n > 1 .

223. (Bielorssia 2005) Seja H o ponto de interseo das alturas BB1 e CC1 do uma reta passando por A, tal que AC. tringulo acutngulo ABC. Seja Prove que as retas BC, B1C1 e possuem um ponto em comum se e somente se H for o ponto mdio de BB1.

EUREKA! N27, 2008

42

Sociedade Brasileira de Matemtica

SOLUO DE DAVI LOPES ALVES DE MEDEIROS (FORTALEZA CE)


A

B1 M C B P C1

Seja P o ponto de interseo de BC e so colineares BM = MB1. i) ii)

. suficiente mostrarmos que B1 , C1 , P

C , M , C1 colineares por Menelaus no BAB1 :


e da

CB1 AC1 BM = 1, (I) AC C1 B MB1


donde

BB1 AC e AC // BB1 PB AB1 PC CB = (II) e = (III) BC CB1 AC CB1

ACP BCB1 ,

iii) P, B1 e C1 so colineares

PC AB1 C1 B = 1 (IV) PB CB1 AC1

Multiplicando (I) e (IV) membro a membro, temos que P, B1 e C1 so colineares

CB1 AC1 BM PC AB1 C1 B BM PC AB1 =1 = 1 (V) AC C1 B MB1 PB CB1 AC1 MB1 AC PB

Mas de (II) :

AB1 CB1 = e multiplicando este resultado por (III) PB BC PC AB1 CB CB1 = = 1 (*) AC PB CB1 CB

EUREKA! N27, 2008

43

Sociedade Brasileira de Matemtica

Substituindo

(*)

em

(V),

temos

que

P, B1 , C1

so

colineares

BM = 1 BM = MB1. c.q.d. MB1

224. (Bielorssia 2005) Ache todas as funes f : N N satisfazendo

f (m n + f (n)) = f (m) + f (n) ,


para todos m, n N.
SOLUO DE ESTILLAC LINS MACIEL BORGES FILHO (BELM PA)

Primeiramente, seja k

. Fazendo m = n = k , temos: f (k k + f (k )) = f (k ) + f (k ) f ( f (k )) = 2 f (k ) , k Em seguida, faamos m = 0 e n = f (k ) , temos: f ( f (k ) + f ( f (k ))) = f (0) + f ( f (k )) f ( f (k ) + 2 f (k )) = f (0) + 2 f (k ) f ( f (k )) = f (0) + 2 f (k ) f (0) = 0 Agora, seja x , tal que x = f (1) . Tomando m = k + 1 e n = 1 , temos: f (k + x) = x + f (k + 1) , k Seja t , com t 1 . Substituindo sucessivamente na equao anterior k por (t j ) x + ( j 1) , com 1 j t , temos: f (tx) = x + f ((t 1) x + 1) f ((t 1) x + 1) = x + f ((t 2) x + 2) f ((t 2) x + 2) = x + f ((t 3) x + 3)

f ((t j + 1) x + j 1) = x + f ((t j ) x + j )

f (2 x + t 2) = x + f ( x + t 1) f ( x + t 1) = x + f (t )
Somando as equaes anteriores, temos:

f (tx) = tx + f (t ) , t Com este resultado em mos, vamos provar, por induo, que f (k ) = kx , k . De fato, temos: (1) Se k = 0 , temos f (k ) = f (0) = 0 = kx (2) Se k = 1 , temos f (k ) = f (1) = x = kx
EUREKA! N27, 2008

44

Sociedade Brasileira de Matemtica

(3) Se f (t ) = tx , para t , temos: f (t 1 + x) = x + f (t ) = x + tx = (t + 1) x f ( f (t i + x)) = f ((t + i) x) = (t + 1) x + f (t + 1) 2 f (t 1 + x) = (t + 1) x + f (t + 1)

(t + 1) x + f (t + 1) = 2(t + 1) x f (t + 1) = (t + 1) x
Portanto, nos resta descobrir quais os valores possveis para x = f (1) . Para isso, vamos utilizar o ltimo resultado encontrado na relao inicial proposta no problema, para n 0 :

f (m n + f (n)) = (m n + nx) x = mx nx + nx 2 f (m) + f (n) = mx + nx


x = 0 mx nx + nx = mx + nx nx = 2nx x = 2 x ou x = 2
2 2 2

Logo, as funes possveis so: f ( y ) = 0 e f ( y ) = 2 y . fcil ver que ambas satisfazem as condies e, portanto, so todas as funes procuradas. 227. (Bulgria 2005) Ivo escreve todos os inteiros de 1 a 100 (inclusive) em cartas e d algumas delas para Iana. Sabe-se que para quaisquer duas destas cartas, uma de Ivo e outra de Iana, a soma dos nmeros no est com Ivo e o produto no est com Iana. Determine o nmero de cartas de Iana sabendo que a carta 13 est com Ivo.
SOLUO DE ESTILLAC LINS MACIEL BORGES FILHO (BELM PA)

Primeiramente, notamos que a carta de nmero 1 deve estar com Iana. De fato, se a carta 1 estivesse com Ivo, para qualquer carta y de Iana, o produto dos nmeros tambm estaria com Iana, contradizendo a hiptese. Isso implica que ou a carta 1 pertence a Iana ou Iana no possui cartas, o que no verdade por hiptese. Sendo assim, Iana possui a carta 1 e, dada qualquer carta x de Ivo, temos que Iana dever possuir a carta x + 1 . Continuando o raciocnio, temos que Iana tambm dever possui as cartas 2 x + 1 , 3 x + 1 , ..., kx + 1 , com kx + 1 100 e k . Alm disso, Iana tambm deve possuir a carta x 1 , pois caso contrrio, Iana deveria possui a carta x 1 + 1 = x , o que no verdade. E logo, Iana tambm deve possuir as cartas 2 x 1 , ..., kx 1 , com kx 1 100 e k . Assim, sabendo que Ivo possui a carta de nmero 13 , j sabemos que Iana possui as cartas:

1, 12, 14, 25, 27, 38, 40, 51, 53, 64, 66, 77, 79, 90, 92
EUREKA! N27, 2008

45

Sociedade Brasileira de Matemtica

Notemos agora que a carta nmero 2 deve pertencer Iana. De fato, se a carta pertencesse a Ivo, teramos pelo raciocnio anterior que Iana deveria possuir todas as cartas mpares, o que no possvel, j que Ivo possui a carta nmero 13 . Como a carta 2 pertence a Iana e a carta 13 pertence a Ivo, temos que a carta 26 deve pertencer a Ivo e, conseqentemente, a carta 52 tambm. Portanto, at o momento, temos: Ivo: 13, 26, 52 Iana: 1, 2, 12, 14, 25, 27, 38, 40, 51, 53, 64, 66, 77, 79, 90, 92 Novamente, temos que a carta 3 deve pertencer a Iana, pois caso pertencesse a Ivo, a carta 6 = 3 2 tambm pertenceria a Ivo, assim como a carta 12 = 6 2 , o que no verdade. Analogamente, a carta 4 tambm pertence a Iana, pois se pertencesse a Ivo, tambm pertenceria a Ivo a carta 12 = 4 3 . Isso implica que as cartas 39 e 78 pertencem a Ivo. Portanto, at o momento: Ivo: 13, 26, 39, 52, 78 Iana: 1, 2, 3, 4, 12, 14, 25, 27, 38, 40, 51, 53, 64, 66, 77, 79, 90, 92 Se Ivo possusse o nmero 5 , tambm deveria possuir o nmero 10 = 5 2 e assim, deveria possuir o nmero 40 = 10 4 , o que no verdade. Logo Iana possui o nmero 5. J os nmeros 6 e 7 tambm no podem estar com Ivo, pois neste caso, os nmeros 12 = 6 2 e 14 = 7 2 no poderiam estar com Iana, o que no acontece. Logo, Iana tambm possui os nmeros 6 e 7 . Assim, at o momento: Ivo: 13, 26, 39, 52, 78 Iana: 1, 2, 3, 4, 5, 6, 7, 12, 14, 25, 27, 38, 40, 51, 53, 64, 66, 77, 79, 90, 92 Analogamente, temos que se 8, 9, 10 e 11 pertencessem a Ivo, 40 = 8 5 , 27 = 9 3 , 40 = 10 4 e 77 = 11 7 tambm deveriam pertencer a Ivo, o que no verdade. Logo, 8, 9, 10 e 11 tambm pertencem a Iana. Neste momento, temos que: Ivo: 13, 26, 39, 52, 78 Iana: 1, 2, 3, 4, 5, 6, 7, 8, 9, 10, 11, 12, 14, 25, 27, 38, 40, 51, 53, 64, 66, 77, 79, 90, 92 Agora notamos que Ivo possui todos os mltiplos de 13 menores que 100 . Todos os demais nmeros menores que 100 devem ento ter a forma 13k + t , com k , t e 0 < t < 13 . Como Iana possui todos os nmeros t com 0 < t < 13 , temos que todos os nmero menores que 100 que no so mltiplos de 13 devem pertencer a Iana, pois 13k pertence a Ivo e a soma no pode pertencer a Ivo.
EUREKA! N27, 2008

46

Sociedade Brasileira de Matemtica

Logo, Ivo possui apenas 7 nmeros e Iana possui os 93 restantes.


230. (Eslovnia 2005) Denote por I o incentro do tringulo ABC. Sabe-se que AC + AI = BC. Encontre a razo entre as medidas do ngulos BAC e CBA. SOLUO DE RAFAEL ALVES DA PONTE (FORTALEZA CE)

Denote por I o incentro do tringulo ABC. Sabe-se que AC + AI = BC. Encontre a razo entre as medidas dos ngulos BAC e CBA.
I

Construa AI em AC de modo que AI = AI , conforme a figura acima. Sejam BAI = IAC = e IBA = IBC = . Note que BI C issceles, e sendo I BA = , BI A mede + 2 e, pelo teorema do ngulo externo, BAC = 2 + 2 , donde vem = + [*]. Veja que I AI = 180 , e visto que I BI = (por [*]), I BIA inscritvel. Como as cordas AI e AI so congruentes, = , da

= 2 2 = 2 2 BAC = 2CBA

BAC =2 CBA

EUREKA! N27, 2008

47

Sociedade Brasileira de Matemtica

COMO QUE FAZ?


PROBLEMA PROPOSTO POR WILSON CARLOS DA SILVA RAMOS (BELM PA)

Dado um tringulo ABC com incentro I, considere uma reta varivel l passando por I que intersecta o lado AB em P, o lado AC em N e a reta suporte do lado BC em M. Prove que o valor de
SOLUO:
AB AC BC independe da escolha de l. + PA PB NA NC MB MC

Suponha, sem perda de generalidade, que B est entre M e C.


A

/2 /2

/2 /2
B I

/ 2 / 2
C

P M

AB PA + PB 1 1 = , Primeiro, note que = + PA PB PA PB PA PB AC NA + NC 1 1 = + = NA NC NA NC NA NC modo que queremos provar que e BC MC MB 1 1 = , de = MB MC MB MC MB MC

1 1 1 1 1 1 + + + + PA PB NA NC MB MC no depende da reta l. Isso um trabalho para a lei dos senos! De fato, nos tringulos AIP e AIN, e r , lembrando que, sendo r o inraio de ABC, AI = sen 2
EUREKA! N27, 2008

48

Sociedade Brasileira de Matemtica

)e sen sen ( + AP AI 1 2 2 = = PA r sen sen sen ( + 2 ) ) sen sen ( NA AI 1 2 2 = = NA r sen sen sen ( 2 )
1 1 e , obtemos PA NA (sen ( + 2 ) + sen ( 2 )) sen 1 1 2 + = PA NA r sen y cos Utilizando a frmula de Prostafrese sen x + sen y = 2 sen x + 2

Somando

( + ) + ( ) ( + ) ( ) 2 2 2 2 2sen sen cos 2 sen (2sen cos ) 2 2 1 1 sen 2 2 + = = = PA NA r sen r sen r Sorte grande! Esse valor no depende da escolha de l, j que r e s dependem do tringulo ABC. Podemos concluir, analogamente, que 1 1 sen + = NC MC r tambm no depende de l. 1 1 , como era de se esperar, um pouquinho diferente, mas J o caso de PB MB s um pouquinho: pela lei dos senos nos tringulos MIB e PIB, sen sen MB BI 1 2 2 e = = sen sen 2 MB r sen

( ) ( ),
x y 2

sen sen + PB BI 1 2 2 = = sen sen 2 + PB r sen

Note a mudana de sinal de

para

: destrocando o sinal, obtemos

sen sen 1 2 = MB r sen

) e a podemos trabalhar como nos demais casos, obtendo

1 1 sen + = MB PB r sen + sen + sen A soma pedida , ento, igual a , que no depende de l. r
EUREKA! N27, 2008

49

Sociedade Brasileira de Matemtica

SOLUES DE PROBLEMAS PROPOSTOS


Publicamos aqui algumas das respostas enviadas por nossos leitores.

110) Um conjunto finito de inteiros positivos chamado de Conjunto DS se cada elemento divide a soma dos elementos do conjunto. Prove que todo conjunto finito de inteiros positivos subconjunto de algum conjunto DS.
SOLUO DE ZOROASTRO AZAMBUJA NETO (RIO DE JANEIRO RJ)

Basta provar que, para todo n inteiro positivo, existe um conjunto DS que contm {1,2,...,n}. Para n 3 , isso segue do fato de {1,2,3} ser um conjunto DS. Vamos mostrar, por induo em n, que, para todo n 3 , existe um conjunto DS, X n = {a1 , a2 ,..., ak ( n ) }, com a j = j para 1 j n. Note que a soma dos seus elementos S n = Dado
k (n) j =1

a
e

par (pois a2 = 2 X n 2 | S n ). conjunto

n3

um

Xn

como

acima,

podemos

tomar

(n + 1)(n + 2) (n + 1)(n + 2) (n + 1)(n + 2) X n +1 = 1, 2,3,..., n, n + 1, a2 , a3 ,..., ak ( n ) 2 2 2


A soma de seus elementos

Sn+1 = 1+ 2 + ... + n + (n +1) + =

(n +1)(n + 2) k (n) (n +1)(n + 2) (n +1)(n + 2) (Sn a1) = aj = + 2 2 2 j =2

(n +1)(n + 2) Sn , pois a1 = 1. 2 Como S n par, S n +1 mltiplo de n + 1. Como S n mltiplo de a j para todo j,


S n +1 =

(n + 1)(n + 2) (n + 1)(n + 2) a j para todo j, e logo S n mltiplo de 2 2

X n +1 um conjunto DS.
111) Prove que existem infinitos mltiplos de 7 na seqncia (an ) abaixo:

a1 = 1999, an = an 1 + p(n), n 2 , onde p(n) o menor primo que divide n.

EUREKA! N27, 2008

50

Sociedade Brasileira de Matemtica

Seja N um nmero da forma 510510r = 2 3 5 7 11 13 17 r , onde r um inteiro positivo. Temos ento p ( N + 2) = 2, p ( N + 3) = 3, p ( N + 4) = 2,

SOLUO DE JOS DE ALMEIDA PANTERA (RIO DE JANEIRO RJ)

p( N + 5) = 5, p( N + 6) = 2, p( N + 7) = 7, p( N + 8) = 2, p( N + 9) = 3, p( N + 10) = 2, p( N + 11) = 11, p( N + 12) = 2, p( N + 13) = 13, p( N + 14) = 2, p( N + 15) = 3, p( N + 16) = 2, e p( N + 17) = 17. Assim, aN + 2 aN +1 + 2(mod 7), aN +3 aN +1 + 5(mod 7), aN + 4 aN +1 (mod 7), aN +5 aN +1 + 5(mod 7), aN + 6 aN +1 (mod 7), aN + 7 aN +1 (mod 7), aN +8 aN +1 + 2(mod 7), aN +9 aN +1 + 5(mod 7), aN +10 aN +1 (mod 7), aN +11 aN +1 + 4(mod 7), aN +12 aN +1 + 6(mod 7), aN +13 aN +1 + 5(mod 7),
aN +14 aN +1 (mod 7), aN +15 aN +1 + 3(mod 7), aN +16 aN +1 + 5(mod 7) e aN +17 aN +1 + 1(mod 7), e portanto sempre h um mltiplo de 7 em
{aN +1 , aN + 2 , aN +3 , aN +11 , aN +12 , aN +15 , aN +17 }, pois aN +1 , aN + 2 , aN +3 , aN +11 , aN +12 ,

aN +15 , aN +17 percorrem todas as classes de congruncia mdulo 7.


112) Determine todos os inteiros positivos n tais que existe uma matriz n n com todas as entradas pertencentes a { 1, 0, 1} tal que os 2n nmeros obtidos como somas dos elementos de suas linhas e de suas colunas so todos distintos.
SOLUO DE ASDRBAL PAFNCIO SANTOS (BOTUCATU SP)

Vamos mostrar que existe uma matriz como no enunciado se e somente se n par. Se n par, digamos n = 2k, podemos construir uma matriz A = ( aij )1i , j 2 k com aij = 1 se 1 i, j k , aij = 1 se k + 1 i, j 2k , aij = 1 se i k e

j k + i, aij = 0 se i k e k + 1 j < k + i, aij = 1 se i k + 1, j k


como somas dos elementos das linhas e das colunas de A

j + k > i, e aij = 0 se i k + 1 e j + k i. fcil ver que os 2n nmeros obtidos

{n, n 1, n 2,...,1, 0, 1,..., (n 2), (n 1)}. Suponha agora que exista uma matriz A = (aij )1i , j n como no enunciado, com
aij {1, 0,1}, i, j n. Claramente permutar linhas ou colunas no altera as 2n + 1 elementos em propriedades do enunciado. Como h { n, (n 1),..., 1, 0,1,..., n 1, n}, um desses elementos, digamos c, no

valor da soma dos elementos de nenhuma linha ou coluna de A. Podemos supor (trocando o sinal de todos os elementos de A, se necessrio) que esse elemento c
EUREKA! N27, 2008

51

Sociedade Brasileira de Matemtica

que falta menor ou igual a 0. Podemos supor (permutando linhas e colunas, se necessrio) que os valores 1, 2,..., n 1, n so obtidos como somas dos elementos das primeiras k linhas e das primeiras n k colunas de A, para certo k n. Sejam x= aij , y = aij , z= aij e w = aij . Temos ento
1i k 1 j n k

1i k nk < j n

k <i n 1 j n k

k <i n nk < j n

n(n + 1) n(n + 1) = 1 + 2 + ... + n = ( x + y ) + ( x + z ) = 2 x + y + z , e c = 2 2 = ( z + w) + ( y + w) = 2w + y + z. Portanto, 2 x 2 w = n(n + 1) + c n 2 , pois c n, e, como claramente temos x k (n k ) e w k (n k ), 4k (n k )


donde n 2k = 0, e portanto n par. Note que, nesse caso, o elemento c que falta deve ser necessariamente igual a n (ou n, se for positivo).

2 x 2 w n 2 0 n 2 4kn + 4k 2 = (n 2k ) 2 ,

114) Sabendo que sen x + sen y + sen z + sen w = 0 e cos x + cos y + cos z + cos w = 0, mostre que

sen 2003 x + sen 2003 y + sen 2003 z + sen 2003 w = 0.


SOLUO BASEADA NAS SOLUES ENVIADAS POR SAMUEL LIL ABDALLA E DOUGLAS RIBEIRO SILVA

De cos x + cos y + cos z + cos w = 0 obtemos

sen x + sen y + sen z + sen w = 0,

eix + eiy + eiz + eiw = (cos x + isenx) + (cos y + iseny) + (cos z + isenz) + (cos w+ isenw) = 0. Vamos supor sem perda de generalidade 0 x y z w < 2 . Temos y x e w z , seno os quatro nmeros complexos eix , eiy , eiz e eiw
pertenceriam a um mesmo semicrculo do crculo unitrio, e sua soma no poderia ser 0. Temos eix + eiy = (cos x + cos y ) + i (senx + seny ) =

y x x+ y x + y y x i = 2 cos sen + i cos = 2 cos e 2 2 2 2

x+ y 2

w z i Analogamente, e + e = 2 cos e 2 Como eix + eiy = ( eiz + eiw ) , obtemos


iz iw

w+ z 2

EUREKA! N27, 2008

52

Sociedade Brasileira de Matemtica


yx y x i w z i 2 2 e 2 cos . Temos ainda 0 = 2 cos e e 2 2 2 2 w z yx w z 0 , donde cos 0 e cos 0. Temos agora dois 2 2 2 2 x+ y w+ z

casos: i) cos

yx w z = 0. Nesse caso, devemos ter tambm cos = 0, e, 2 2 portanto y x = w z = , seny = senx e senw = senz , donde segue
yx w z yx > 0. Nesse caso, devemos ter cos = cos >0e 2 2 2

imediatamente o resultado. ii) cos

w z y x w+ z x+ y = = + e . Somando, 2 2 2 2 obtemos w = y + e, substraindo, obtemos z = x + . Assim, senw = seny e senz = senx , donde segue o resultado, como antes.

x+ y i 2

= e

w+ z 2

, donde

116) Seja ABC um tringulo e sejam X, Y e Z as reflexes de A, B e C em relao s retas BC, CA e AB, respectivamente. Prove que x, y e z so colineares se e somente se cos A cos B cos C = 3 8.
SOLUO ENVIADA POR DOUGLAS RIBEIRO SILVA COM CONTRIBUIES DE CARLOS EDDY ESAGUY NEHAB E MARCIO COHEN

Construam o tringulo ABC e as reflexes X, Y e Z de seus vrtices A, B e C. Se, dados U, V, W no plano, (UVW) denota a rea (orientada) do tringulo UVW, temos S(XYZ) = [S(ABC) + S(CBX) + S(ACY) + S(BAZ)] S(AZY) S(BXZ) S(CYX). (*) Ver nota abaixo. Temos S(ABC) = S(CBX) = S(ACY) = S(BAZ), por construo. As reas de AZY, BXZ e CYX podem ser somadas ou subtradas, dependendo de

= 3 A, ZBX = 3C (onde A, B e C denotam = 3B e XCY os ngulos YAZ os ngulos internos do tringulo ABC) serem maiores ou menores que 180 graus. Valer a igualdade se usarmos as expresses S(AZY) = bc sen(3A)/2, S(BXZ) = ac sen(3B)/3 e S(CYX) = ab sen(3C)/2. Ento a relao passa a ser S(XYZ) = 4S(ABC) bc sen(3A)/2 ac sen(3B)/3 ab sen(3C)/2.
EUREKA! N27, 2008

53

Sociedade Brasileira de Matemtica

Agora substitumos sen(3 ) = 4(sen( )) 3 + 3 sen( ) para = A, B, C e substitumos tambm bc/2 ac/2 e ab/2 respectivamente por S(ABC)/senA, S(ABC)/senB e S(ABC)/senC, devido frmula de rea em funo dos lados e do ngulo para o tringulo original. Fazendo as devidas substituies acima, simplificamos os senos e basta trocar (sen( ))2 por 1 (cos( ))2 para = A, B, C para chegar em S(XYZ) = S(ABC) [7 4((cosA)2 + (cosB)2 + (cosC)2)]. Para que os trs pontos estejam alinhados, a rea do tringulo XYZ deve ser igual a zero, donde 7 4((cosA)2 + (cosB)2 + (cosC)2) = 0 Faamos z = (cosA)2 + (cosB)2 + (cosC)2 Dai, como cos2x = 2(cosx)2 1 temos z = (1 + cos2A)/2 + (1+cos2B)/2 + (cosC)2 = 1 + (cos(2A) +cos(2B))/2 + (cosC)2. Mas cos(2A) + cos(2B) = 2cos(A + B)cos(A B) = 2cosC cos(A B). Substituindo em z: z = 1 cosC [cos(A B) cosC] = 1 cosC [cos(A B) + cos(A + B)]. Da temos: z = 1 cosC. [2cosA.cosB] = 1 2cosA.cosB.cosC. Substituindo este z na expresso anterior, chegamos na desejada expresso do enunciado: 0 = 7 4 ((cosA)2 + (cosB)2 + (cosC)2) = 7 4 (1 2cosA.cosB.cosC) = 3+8cosA.cosB.cosC. Logo, chegamos na esperada relao cosAcosBcosC = 3/8. Nota: A igualdade(*) pode ser mostrada por meio de algumas figuras, considerando alguns casos, mas daremos a seguir uma prova algbrica dela. A 2 igual metade do rea (orientada) de um tringulo UVW no plano determinante det (V U , W U) da matriz cujas linhas coincidem com os vetores V U e W U. Se identificarmos cada vetor ( x, y ) 2 com ( x, y, 0) 3 , o produto vetorial (V U) (W U) igual ao vetor (0,0, det (V U, W U)) = (0,0, 2 S (UVW)). Basta provar ento que (Y X) (Z X) = (B A) (C A) + (B C) (X C) + (C A) (Y A) + (A B) (Z B) (Z A) (Y A) (X B) (Z B) (Y C) (X C), mas o lado direito igual a (B A) (C A) + (B Y) (X C) + (C Z) (Y A) + (A X) (Z B), que, desenvolvendo (e usando a desigualdade U V = V U, para quaisquer U, V), igual a B C B A A C + B X B C Y X + Y C + C Y CAZY+ZA+AZABXZ+XB=YXZY XZ = Y Z Y X X Z = (Y X) (Z X).
EUREKA! N27, 2008

54

Sociedade Brasileira de Matemtica

117) Sejam r e s duas retas reversas (i.e., no contidas num mesmo plano) e A,
B, C, D, A, B, C , D pontos tais que A, B, A, B r , C , D, C , D s, AB = AB e

CD = C D. Prove que os tetraedros ABCD e ABC D tm o mesmo volume.


SOLUO DE DOUGLAS RIBEIRO SILVA (RECIFE - PE)

Note que o que o problema pede equivalente demonstrar que o volume do tetraedro s depende da medida de duas arestas reversas e da distncia entre as retas-suporte dessas duas arestas. Na figura que segue, as arestas reversas so AB e CD. A distncia entre as retas suporte EF. O volume do tetraedro ser calculado a partir da rea da base ABC e a altura relativa a D V = (rea (ABC) HD)/3 Note que a rea de ABC pode ser definida como AB EC/2, pois como EC est no mesmo plano de EF e EF perpendicular a AB, EC tambm (Teorema das 3 perpendiculares). V = ((AB EC/2) HD)/3 Conservando a rea do triangulo retangulo EFC temos: EF FC/2 = EC FG/2. Logo FG = EF FC/EC Pela semelhana dos triangulos FGC e DHC tiramos o valor de HD: HD/CD = FG/FC Logo HD = FG CD/FC = (EF FC/EC) (CD/FC) Logo HD = EF CD/EC Finalizando, temos que V = (AB EC/2) (EF CD/EC)/3 Logo V = AB CD EF/6 Assim, provamos que o volume de um tetraedro no depende da posio dos segmentos AB e CD nas suas retas-suporte, mas sim, unicamente dos tamanhos dos segmentos e da distncia entre as retas-suporte dos mesmos.

EUREKA! N27, 2008

55

Sociedade Brasileira de Matemtica

D F

A E B
118) Considere a seqncia (an ) n 1 dada por a1 = 1 e an +1 =
Prove que (an ) converge e calcule o seu limite.
SOLUO DE ESTILLAC LINS MACIEL BORGES FILHO (BELM PA)

C H G

2 + 9an , n 1. 3 + 9an

Primeiramente, iremos provar que a n > induo. De fato:

1+ 3 , para todo n , utilizando 3

1+ 3 3 1+ 3 (2) Supondo a n > , temos 1 + 3a n > 2 + 3 . Portanto: 3 2 + 9a n 1 1 1 1 1+ 3 = 1 ( a n +1 = ) > 1 ( )= 3 2+ 3 3 3 + 9a n 3 1 + 3a n


(1) a1 = 1 > Em seguida, provaremos que, se a n >

1+ 3 , temos que a n +1 < a n . De fato: 3

an >

1+ 3 3a n 1 > 3 (3a n 1) 2 > 3 3


56

EUREKA! N27, 2008

Sociedade Brasileira de Matemtica

Portanto,

2 + 9a n 3 (3a n 1) 2 an = <0 a n +1 a n = 3 + 9a n 3 + 9a n Logo, (a n ) uma seqncia estritamente decrescente limitada inferiormente por

1+ 3 , o que garante sua convergncia. Vamos mostrar agora que 3 1+ 3 x = lim an = lim an +1. . Seja Temos ento lim a n = 3
x = lim an +1 = lim 2 + 9an 2 + 9 x = , donde 9 x 2 6 x 2 = 0. Como x 0, devemos ter 3 + 9an 3 + 9 x

x=

3 + 27 1 + 3 = . 9 3

Obs: a soluo acima bastante artificial. Ela construda j se sabendo de antemo qual o provvel limite da seqncia. Este provvel limite obtido e resolvendo a equao facilmente fazendo a n +1 = a n = x

x=

2 + 9x 1+ 3 x= , pois x 0. 3 + 9x 3

Continuamos aguardando as solues dos problemas a seguir: 113) a1 , a2 , a3 ,... formam uma seqncia de inteiros positivos menores que 2007
tais que

am + an inteiro, para quaisquer inteiros positivos m, n. am + n

Prove que a seqncia (an) peridica a partir de um certo ponto.

115) Suponha que ABC um tringulo com lados inteiros a, b e c com

BCA = 60 e c 1(mod 6) .

mdc(a, b) = mdc(a, c) = mdc(b, c) = 1.

Prove

que

EUREKA! N27, 2008

57

Sociedade Brasileira de Matemtica

PROBLEMAS PROPOSTOS
Convidamos o leitor a enviar solues dos problemas propostos e sugestes de novos problemas para prximos nmeros. 119. Mostre que no existem inteiros positivos a e b tais que tais que (36a + b)(36b + a) seja uma potncia de 2. 120. Sejam a, b, c nmeros reais e soma Sn definida como S n = a n + b n + c n , para

qualquer n inteiro no negativo, Sabe-se que S1 = 2, S2 = 6 e S3 = 14, mostre que


2 Sn S n 1 S n +1 = 8 para todo inteiro n > 1.

121. Na figura abaixo o lado do quadrado vale 4, obter o valor da altura h para que

a rea da regio 1 seja igual a rea da regio 2.

4 1 4 h 2 4

4
122. Dado um tringulo ABC tal que AB = AC = a + b e BC = a , traa-se uma ceviana partindo de B determinando em AC um ponto D tal que DA = a e
DC = b . Sabendo que ABD = 10 , determine os ngulos internos desse tringulo.

123.
2

Determine
2 3 2

todas

as
2

funes

f : * *

tais

que

2 f (m + n ) = f (m) f (n) + f (m) f (n) , para quaisquer m, n * distintos.


Obs.
* = {1, 2,3,...} o conjunto dos inteiros positivos.

EUREKA! N27, 2008

58

Sociedade Brasileira de Matemtica

124.
an =

Considere
2 an 1an 3 + an 2

seqncia

( an ) n 1

definida

por

a1 = a2 = a3 = a4 = 1 e

an 4

, n 5.

Prove que an um inteiro positivo, para todo inteiro positivo n.


125. Considere dois naturais ( a0 , a1 , a2 ,..., amn ), ai {0,1}.
m2

n 2,

as

seqncias

As seqncias de tipo m satisfazem as condies: ak ak + m = 0, para todo k; Se ak ak +1 = 1 ento m divide k As seqncias de tipo n so definidas analogamente. Prove que existem tantas seqncias do tipo m quanto do tipo n.
126. As circunferncias i , 0 i 5, so tangentes a uma circunferncia nos

pontos Ai . Alm disso, i tangente a i +1 para 0 i 5 e 5 tangente a 0 . Prove que A0 A3 , A1 A4 , A2 A5 so concorrentes.


127. Determine todos os inteiros positivos k tais que existem inteiros positivos x,

y, z com

x2 + y2 + z 2 = k. xyz

128. Barango Joe era um sapo de mtiplos talentos que habitava a Terra das

Chances Diminutas, localizada no alto de uma montanha. Aps sua maioridade, Barango Joe decidiu tentar a vida no Reino das Grandes Oportunidades, localizado no cume da montanha vizinha. Para isso, ele atravessaria a extensa ponte de madeira por cima do Desfiladeiro da Morte. Entretanto, a ponte era guardada pela Esfinge Vegas, exmia jogadora que sempre desafiava os viajantes para algum jogo. O viajante vitorioso tinha a passagem franqueada; e o perdedor era lanado ao abismo. Assim chegando cabeceira da ponte, Barango Joe foi desafiado a uma partida de Pachang jogo que lembra o Black Jack ou Vinte e um, mas jogado por 2 oponentes da seguinte maneira: Os jogadores, designados por banca e apostador, utilizam um dado gerador de nmeros aleatrios reais uniformemente distribudos no intervalo [0,1] Inicialmente, a banca sorteia um nmero X. Se no estiver satisfeita com o nmero obtido, pode descart-lo e ento sortear um novo nmero. Este procedimento pode ser executado 2 vezes, Isto , pode haver at 3 sorteios na definio do nmero X da banca.
EUREKA! N27, 2008

59

Sociedade Brasileira de Matemtica

Ento, o apostador sorteia quantos nmeros forem necessrios at que a soma de seus nmeros ultrapasse o nmero X da banca. Neste momento, se esta soma for inferior a 1, o apostador ganha; caso contrrio, perde. Ou seja, para ganhar, o apostador precisa chegar mais prximo de 1 que a banca, sem no entanto estourar o limite de 1. Aps explicar as regras do Pachang, a Esfinge Vegas deu uma opo ao sapo: - Voc prefere ser a banca ou o apostador? O que o Barango Joe deveria responder? Obs. Utilize lpis, papel, e uma calculadora cientfica simples.
129. Um coelho est numa rua infinita dividida em quadrados numerados pelos

inteiros, e comea no quadrado 0. Se num dado momento ele est no quadrado k, ele escolhe, com probabilidade probabilidade
1 , pular para o quadrado k + 2 ou, tambm com 2

1 , pular para o quadrado k 1. Ele continua esse processo 2 indefinidamante. Dado m , determine a probabilidade de, em algum momento,

o coelho pisar no quadrado m.


Problema 119 proposto por Adriano Carneiro, problemas 120 e 121 proposto por Samuel Lil Abdalla, de Sorocaba SP, problema 122 proposto por Renan Lima Novais, do Rio de Janeiro RJ, problema 123 proposto por Wilson Carlos da Silva Ramos, de Belm PA, problemas 124, 125, 126 e 127 propostos por Anderson Torres, de So Paulo SP, problema 128 proposto por Rogrio Ponce da Silva, do Rio de Janeiro RJ, problema 129 proposto por Nicolau Coro Saldanha e Zoroastro Azambuja Neto, do Rio de Janeiro RJ. Agradecemos tambm o envio das solues e a colaborao de:

Gelly Whesley Evandro A. dos Santos Davi Lopes Alves de Medeiros Rafael Alves da Ponte Andr Felipe M da Silva Carlos Alberto da Silva Victor

Fortaleza CE Campinas SP Fortaleza CE Fortaleza CE Rio de Janeiro RJ Nilpolis RJ

EUREKA! N27, 2008

60

Sociedade Brasileira de Matemtica

AGENDA OLMPICA
XXX OLIMPADA BRASILEIRA DE MATEMTICA NVEIS 1, 2 e 3 Primeira Fase Sbado, 14 de junho de 2008 Segunda Fase Sbado, 13 de setembro de 2008 Terceira Fase Sbado, 25 de outubro de 2007 (nveis 1, 2 e 3) Domingo, 26 de outubro de 2008 (nveis 2 e 3 - segundo dia de prova). NVEL UNIVERSITRIO Primeira Fase Sbado, 13 de setembro de 2008 Segunda Fase Sbado, 25 e Domingo, 26 de outubro de 2008

XIV OLIMPADA DE MAIO 10 de maio de 2008

XIX OLIMPADA DE MATEMTICA DO CONE SUL Temuco Chile 18 a 23 de junho de 2008

XLIX OLIMPADA INTERNACIONAL DE MATEMTICA 10 a 22 de julho de 2008 Madri Espanha

XIV OLIMPADA INTERNACIONAL DE MATEMTICA UNIVERSITRIA 25 a 31 de julho de 2008 Blagoevgrad, Bulgria

XXIII OLIMPADA IBEROAMERICANA DE MATEMTICA 18 a 28 de setembro de 2008 Salvador, Bahia Brasil

XI OLIMPADA IBEROAMERICANA DE MATEMTICA UNIVERSITRIA

EUREKA! N27, 2008

61

Sociedade Brasileira de Matemtica

COORDENADORES REGIONAIS
Alberto Hassen Raad Amrico Lpez Glvez Amarsio da Silva Arajo Andreia Goldani Antonio Carlos Nogueira Ali Tahzibi Benedito Tadeu Vasconcelos Freire Carlos Alexandre Ribeiro Martins Carmen Vieira Mathias Claus Haetinger Cleonor Crescncio das Neves Cludio de Lima Vidal Denice Fontana Nisxota Menegais Edson Roberto Abe lio Mega Eudes Antonio da Costa Fbio Brochero Martnez Florncio Ferreira Guimares Filho Francinildo Nobre Ferreira Genildo Alves Marinho Ivanilde Fernandes Saad Jacqueline Rojas Arancibia Janice T. Reichert Joo Bencio de Melo Neto Joo Francisco Melo Libonati Jos Luiz Rosas Pinho Jos Vieira Alves Jos William Costa Krerley Oliveira Licio Hernandes Bezerra Luciano G. Monteiro de Castro Luzinalva Miranda de Amorim Mrio Rocha Retamoso Marcelo Rufino de Oliveira Marcelo Mendes Newman Simes Nivaldo Costa Muniz Osvaldo Germano do Rocio Raul Cintra de Negreiros Ribeiro Ronaldo Alves Garcia Rogrio da Silva Igncio Reginaldo de Lima Pereira Reinaldo Gen Ichiro Arakaki Ricardo Amorim Srgio Cludio Ramos Seme Gebara Neto Tadeu Ferreira Gomes Toms Menndez Rodrigues Valdenberg Arajo da Silva Vnia Cristina Silva Rodrigues Wagner Pereira Lopes (UFJF) (USP) (UFV) FACOS (UFU) (USP) (UFRN) (Univ. Tec. Fed. de Paran) (UNIFRA) (UNIVATES) (Inst. de Tec. e Educ. Galileo da Amaznia) (UNESP) (UNIPAMPA) (Colgio Objetivo de Campinas) (Faculdade Etapa) (Univ. Federal do Tocantins) (UFMG) (UFES) (UFSJ) (Centro Educacional Leonardo Da Vinci) (UC. Dom Bosco) (UFPB)) (UNOCHAPEC) (UFPI) (Grupo Educacional Ideal) (UFSC) (UFPB) (Instituto Pueri Domus) (UFAL) (UFSC) (Sistema Elite de Ensino) (UFBA) (UFRG) (Grupo Educacional Ideal) (Colgio Farias Brito, Pr-vestibular) (Cursinho CLQ Objetivo) (UFMA) (U. Estadual de Maring) (Colgio Anglo) (UFGO) (Col. Aplic. da UFPE) (Escola Tcnica Federal de Roraima) (UNIFESP) (Centro Educacional Logos) (IM-UFRGS) (UFMG) (UEBA) (U. Federal de Rondnia) (U. Federal de Sergipe) (U. Metodista de SP) (CEFET GO) Juiz de Fora MG Ribeiro Preto SP Viosa MG Osrio RS Uberlndia MG So Carlos SP Natal RN Pato Branco - PR Santa Mara RS Lajeado RS Manaus AM S.J. do Rio Preto SP Bag RS Campinas SP So Paulo SP Arraias TO Belo Horizonte MG Vitria ES So Joo del Rei MG Taguatingua DF Campo Grande MS Joo Pessoa PB Chapec SC Teresina PI Belm PA Florianpolis SC Campina Grande PB Santo Andr SP Macei AL Florianpolis SC Rio de Janeiro RJ Salvador BA Rio Grande RS Belm PA Fortaleza CE Piracicaba SP So Luis MA Maring PR Atibaia SP Goinia GO Recife PE Boa Vista RR SJ dos Campos SP Nova Iguau RJ Porto Alegre RS Belo Horizonte MG Juazeiro BA Porto Velho RO So Cristovo SE S.B. do Campo SP Jata GO

EUREKA! N27, 2008

62

CONTEDO

XXIX OLIMPADA BRASILEIRA DE MATEMTICA Problemas e Solues da Primeira Fase XXIX OLIMPADA BRASILEIRA DE MATEMTICA Problemas e Solues da Segunda Fase XXIX OLIMPADA BRASILEIRA DE MATEMTICA Problemas e Solues da Terceira Fase

15

35

XXIX OLIMPADA BRASILEIRA DE MATEMTICA Problemas e Solues da Primeira Fase Nvel Universitrio

57

XXIX OLIMPADA BRASILEIRA DE MATEMTICA Problemas e Solues da Segunda Fase Nvel Universitrio

62

XXIX OLIMPADA BRASILEIRA DE MATEMTICA Premiados AGENDA OLMPICA COORDENADORES REGIONAIS

73

77 78

Sociedade Brasileira de Matemtica

XXIX OLIMPADA BRASILEIRA DE MATEMTICA


Problemas e solues da Primeira Fase PROBLEMAS NVEL 1
01) Observe as multiplicaes a seguir: 101 11 = 1111 101 111 = 11211 101 1111 = 112211 101 11111 = 1122211 K Qual a soma dos algarismos do nmero obtido quando multiplicamos 101 pelo nmero 11111 K 11 ? 1 4 2 4 3
2007 algarismos 1

A) 1001

B) 2007

C) 2009

D) 4008

E) 4014

02) Quantos nmeros inteiros positivos de trs algarismos tm a soma de seus algarismos igual a 4? Observao: lembre-se de que zeros esquerda no devem ser contados como algarismos; por exemplo, o nmero 031 tem dois algarismos. A) 4 B) 6 C) 7 D) 10 E) 12 03) Juntando dois retngulos iguais lado a lado, sem sobreposio, podemos formar dois tipos de figura: um quadrado de rea igual a 144 cm2 ou um retngulo de largura diferente do comprimento. Qual o permetro deste ltimo retngulo, em cm? A) 12 B) 24 C) 48 D) 60 E) 72 04) A figura ao lado formada por dois quadrados de rea 100 cm2 cada um, parcialmente sobrepostos, de modo que o permetro da figura (linha mais grossa) igual 50 cm. Qual a rea da regio comum aos dois quadrados, em cm2 ? A) 20 B) 25 C) 30 D) 40 E) 50

EUREKA! N28, 2008

Sociedade Brasileira de Matemtica

05) A soma de todos os nmeros positivos mpares at 2007 menos a soma de todos os nmeros positivos pares at 2007 igual a: A) 1003 B) 1004 C) 2005 D) 2006 E) 2007 06) Slvia pensou que seu relgio estava atrasado 10 min e o acertou, mas na verdade o relgio estava adiantado 5 min. Cristina pensou que seu relgio estava adiantado 10 min e o acertou, mas na verdade o relgio estava atrasado 5 min. Logo depois, as duas se encontraram, quando o relgio de Slvia marcava 10 horas. Neste momento, que horas o relgio de Cristina indicava? A) 9h 30min B) 9h 50min C) 10h D) 10h 5min E) 10h 15min 07) A frao

a , onde a e b so inteiros b

0
a b

positivos, representa um nmero entre 0 e 1, na posio indicada no desenho ao lado. Qual um possvel valor para a soma a + b ? A) 1 B) 2 C)3 D) 4 E) 5

08) Em uma prova de olimpada, 15% dos estudantes no resolveram nenhum problema, 25% resolveram pelo menos um problema, mas cometeram algum erro, e os restantes, 156 estudantes, resolveram todos os problemas corretamente. O nmero de estudantes que participaram da olimpada foi: A) 200 B) 260 C) 93 D) 223 E) 300 09) Em uma certa cidade, a razo entre o nmero de homens e mulheres 2 : 3 e entre o nmero de mulheres e crianas 8 : 1. A razo entre o nmero de adultos e crianas : A) 5 : 1 B) 16 : 1 C) 12 : 1 D) 40 : 3 E) 13 : 1 10) Na figura, o lado AB do tringulo eqiltero ABC paralelo ao lado DG do quadrado DEFG. Qual o valor do ngulo x? A) 80o B) 90o C) 100o D) 110o E) 120o
B G A x D C E F

EUREKA! N28, 2008

Sociedade Brasileira de Matemtica

11) Uma loja de CD`s realizar uma liquidao e, para isso, o gerente pediu para Anderlaine multiplicar todos os preos dos CD`s por 0,68. Nessa liquidao, a loja est oferecendo um desconto de: A) 68% B) 6,8% C) 0,68% D) 3,2% E) 32% 12) Esmeralda e Prola esto numa fila. Faltam 7 pessoas para serem atendidas antes de Prola e h 6 pessoas depois de Esmeralda. Duas outras pessoas esto entre Esmeralda e Prola. Dos nmeros abaixo, qual pode ser o nmero de pessoas na fila? A) 9 B) 11 C) 13 D) 14 E) 15 13) Preenchemos as casas vazias da tabela ao lado com o produto dos nmeros que esto sombreados na mesma linha e na mesma coluna da casa vazia a ser preenchida. Quantas dessas casas contero nmeros primos? A) 6 B) 7 C) 12 D) 14 E) 26
x 1 2 3 5 7 11 13 1 2 3 5 7 11 13

14) O contedo de uma garrafa de refrigerantes enche trs copos grandes iguais e mais meio copo pequeno ou 5 desses copos pequenos iguais mais a metade de um daqueles grandes. Qual a razo entre o volume de um copo pequeno e o de um grande? A) 2

B) 3

C) 7

10

D) 5

E) 3

15) Um cdigo de barras formado por barras verticais pretas de trs larguras diferentes. Duas barras pretas sempre so separadas por uma barra branca, tambm com trs larguras diferentes. O cdigo comea e termina com uma barra preta, como no exemplo ao lado. Considere um cdigo S, formado por uma barra preta fina, duas mdias e uma grossa, separadas por barras brancas finas. Quantos cdigos S diferentes podem ser assim formados? A) 4 B) 6 C) 12 D) 24 E) 36
EUREKA! N28, 2008

Sociedade Brasileira de Matemtica

16) No quadriculado ao lado, cada quadradinho tem 1 cm2. Os segmentos inclinados ligam pontos mdios dos lados dos quadradinhos ou um vrtice ao centro de um quadradinho. Qual a rea ocupada pela sigla OBM, em cm2? A) 28 B) 32 C) 33 D) 34 E) 35 17) Lina e Lana brincam da seguinte maneira: a primeira a jogar pensa em um nmero de 10 a 99 e diz apenas a soma dos algarismos do nmero; a segunda tem ento que adivinhar esse nmero. Qual o maior nmero de tentativas erradas que a segunda pessoa pode fazer? A) 7 B) 8 C) 9 D) 10 E) 11 18) Anita imaginou que levaria 12 minutos para terminar a sua viagem, enquanto dirigia velocidade constante de 80 km/h, numa certa rodovia. Para sua surpresa, levou 15 minutos. Com qual velocidade constante essa previso teria se realizado? A) 90 km/h B) 95 km/h C) 100 km/h D) 110 km/h E) 120 km/h 19) O grfico ao lado mostra o percentual de acertos numa prova de 60 testes de seis candidatos finalistas de um concurso. Qual foi o nmero mdio de questes erradas por esses candidatos nessa prova? A) 14 B) 24 C) 30 D) 32 E) 40
70% 60% 50% 40% 30% 20% 10% A B C D E F

20) Ao efetuar a soma 131 + 132 + 133 + L + 132006 + 132007 obtemos um nmero inteiro. Qual o algarismo das unidades desse nmero? A) 1 B) 3 C) 5 D) 7 E) 9

EUREKA! N28, 2008

Sociedade Brasileira de Matemtica

PROBLEMAS NVEL 2
01) Veja o problema No. 1 do Nvel 1. 02) Veja o problema No. 7 do Nvel 1. 03) Veja o problema No. 10 do Nvel 1. 04) Em uma certa cidade, a razo entre o nmero de homens e mulheres 2 : 3 e entre o nmero de mulheres e crianas 8 : 1. A razo entre o nmero de adultos e crianas : A) 5 : 1 B) 16 : 1 C) 12 : 1 D) 40 : 3 E) 13 : 1 05) Veja o problema No. 8 do Nvel 1. 06) Se N o quadrado do quadrado de um nmero inteiro e tem 12 como fator, o menor valor para A) 3

N : 12
C) 36 D) 54 E) 108

B) 12

07) O jardim da casa de Maria formado por cinco quadrados de igual rea e tem a forma da figura abaixo. Se AB = 10 m, ento a rea do jardim em metros quadrados :

A) 200

B) 10 5

C) 100

D)

500 3

E)

100 3

EUREKA! N28, 2008

Sociedade Brasileira de Matemtica

08) Sejam a, b, c e k nmeros reais diferentes de zero satisfazendo as relaes

k=

a b c = = . Qual o nmero de possveis valores que k pode b+c c+a a+b


B) 1 C) 2 D) 3 E) 4

assumir? A) 0

09) Doze pontos esto sobre um crculo. Quantos polgonos convexos podemos formar com vrtices nesses 12 pontos? A) 4017 B) 220 C) 4095 D) 66 E) 3572 10) De quantas maneiras diferentes podemos escrever o nmero 2007 como soma de dois ou mais nmeros inteiros positivos e consecutivos? A) 1 B) 2 C) 3 D) 4 E) 5 11) As equaes do 2o grau 2007 x 2 + 2008 x + 1 = 0 e x 2 + 2008 x + 2007 = 0 tm uma raiz comum. Qual o valor do produto das duas razes que no so comuns? A) 0 B) 1 C) 2007 D) 2008 E) 2007 12) Qual o mximo valor que o nmero a (b + c) b( a + c) pode assumir se a, b e c , so inteiros satisfazendo 1 a 10 , 1 b 10 e 1 c 10 ? A) 80 B) 81 C) 84 D) 90 E) 100 13) A quantidade de inteiros x com trs dgitos tais que 6x e 7x possuem a mesma quantidade de dgitos : A) 767 B) 875 C) 876 D) 974 E) 975 14) A figura abaixo formada por trs quadrados de lado 1 e um retngulo que os contorna.

A rea do retngulo : A) 3 2 B) 4 2 C) 6 D) 6 2 E) 8

EUREKA! N28, 2008

Sociedade Brasileira de Matemtica

15) Se x real positivo e 1 + (x2 + x)(x2 + 5x + 6) = 1812, ento o valor de x(x + 3) : A) 180 B) 150 C) 120 D) 182 E) 75 16) A figura abaixo mostra um retngulo, um pentgono, um tringulo e um crculo, com reas respectivamente 121, 81, 49 e 25 centmetros quadrados. A diferena entre a rea preta e a rea cinza, em centmetros quadrados, :

A) 25

B) 36

C) 49

D) 64

E) 81

17) As seguradoras de automveis A e B cobram um valor anual (prmio) mais um valor que o usurio deve pagar em caso de acidente (franquia). Jean quer fazer um seguro para seu automvel e recebeu as seguintes propostas das seguradoras: Seguradora A: Prmio anual de R$ 1500,00 e franquia de R$ 1400,00 Seguradora B: Prmio anual de R$ 1700,00 e franquia de R$ 700,00 Para valer a pena Jean contratar a Seguradora A, ele no deve se acidentar com o carro por pelo menos N anos. O valor de N : A) 2 B) 3 C) 4 D) 5 E) 6 18) O desenho abaixo mostra um dado comum cujas somas das pontuaes em faces opostas sempre igual a 7. Ele colocado em uma mesa horizontal com a face 1 voltada para Leste. O dado , ento, movido quatro vezes.
Norte Leste

EUREKA! N28, 2008

Sociedade Brasileira de Matemtica

Um movimento consiste em uma rotao de 90 em relao a uma aresta. Depois do primeiro movimento a face em contato com a mesa passa a ser a 1, depois a 2, ento a 3 e, finalmente, a face 5. Para que sentido est voltada a face 1 aps esta seqncia de movimentos? A) Oeste B) Leste C) Norte D) Sul E) Cima 19) Uma avenida possui 100 prdios numerados de 1 a 100, onde prdios com numerao par se situam do lado direito da rua e prdios com numerao mpar se situam no lado esquerdo. A quantidade de andares de cada prdio igual soma dos algarismos do nmero correspondente ao prdio. Assim, podemos afirmar que: A) A quantidade de prdios com mais de 10 andares maior do lado direito da rua. B) A quantidade de prdios com menos de 5 andares maior do lado direito da rua. C) Pelo menos metade dos prdios possui 10 ou mais andares. D) Em ambos os lados da rua h a mesma quantidade de prdios com exatos 8 andares. E) Pelo menos 25% dos prdios possui menos de 5 andares. 20) Qual o menor permetro inteiro possvel de um tringulo que possui um dos lados com medida igual a A) 8 B) 9
5 3 ? 2

C) 10

D) 11

E)12

21) Determine em qual dos horrios abaixo o ngulo determinado pelos ponteiros de um relgio o menor. A) 02h30 B) 06h20 C) 05h40 D) 08h50 E) 09h55 22) O mximo divisor comum entre os nmeros 1221, 2332, 3443, 4554,........, 8998 : A) 3 B) 33 C) 37 D) 11 E) 101 23) Uma mesa de bilhar tem dimenses de 3 metros por 6 metros e tem caapas nos seus quatro cantos P, Q, R e S. Quando uma bola bate na borda da mesa, sua trajetria forma um ngulo igual ao que a trajetria anterior formava.

EUREKA! N28, 2008

Sociedade Brasileira de Matemtica

Uma bola, inicialmente a 1 metro da caapa P, batida do lado SP em direo ao lado PQ, como mostra a figura. A quantos metros de P a bola acerta o lado PQ se a bola cai na caapa S aps duas batidas na borda da mesa? 6 3 2 3 A) 1 B) C) D) E) 7 4 3 5 24) Considere todos os nmeros abc de trs algarismos onde b = a2 + c2 e a 0 . A diferena entre o maior e o menor destes nmeros um nmero: A) Mltiplo de 3 B) Primo C) Com ltimo algarismo igual a 7 D) Cuja soma dos algarismos 10 E) Mltiplo de 7 25) Seja {an} uma seqncia na qual cada termo definido como o dobro da soma dos algarismos do termo anterior, mais uma unidade. Por exemplo, se an = 234, ento an +1 = 2(2 + 3 + 4) +1. Se, a1 = 1 o valor de a31 + a32 + a33 + a34 + a35 igual a: A) 44 B) 54 C) 64 D) 77 E) 84

PROBLEMAS NVEL 3
01) A figura mostra dois quadrados sobrepostos. Qual o valor de x + y, em graus?

y
A) 270
EUREKA! N28, 2008

B) 300

C) 330

D) 360

E) 390

10

Sociedade Brasileira de Matemtica

02) Um nmero de quatro dgitos dito peroba se possui pelo menos dois dgitos vizinhos com a mesma paridade. Quantos nmeros perobas existem? A) 8999 B) 8874 C) 7875 D) 8000 E) 7750 03) Veja o problema No. 15 do Nvel 2. 04) Veja o problema No. 18 do Nvel 2. 05) Os nmeros 72, 8, 24, 10, 5, 45, 36, 15 so agrupados em duplas de modo que o produto de cada dupla o mesmo. Qual nmero fica com o 10? A) 36 B) 45 C) 24 D) 15 E) 72 06) Tintas pretas opacas absorvem 97% da luz, refletindo o restante. Cientistas desenvolveram uma nova cobertura superpreta que dez vezes mais preta que tintas pretas opacas, querendo dizer que ela reflete 1/10 da luz refletida pelas tintas pretas opacas. Que porcentagem de luz a nova cobertura absorve? A) 9,7 B) 90,3 C) 99,7 D) 99,9 E) 970 07) Considere a seguinte seqncia: 27 = 3 3 3 , 207 = 3 3 23 , 2007 = 3 3 223 , 20007 = 3 3 2223 , ... Qual dos seguintes inteiros um mltiplo de 81? A) 200.007 B) 20.000.007 C) 2.000.000.007 D) 200.000.000.007 E) 20.000.000.000.007 08) Qual dos inteiros positivos abaixo satisfaz a seguinte equao: 4 5 6 n4 6 n4 5 n4 4 + + + L + + + = 309 ? n4 n4 n4 n4 n4 n4 A) 2007 B) 309 C) 155 D) 25 E) 5 09) O desenho abaixo mostra um semicrculo e um tringulo issceles de mesma rea. Qual o valor de tg x ?

xo

EUREKA! N28, 2008

11

Sociedade Brasileira de Matemtica

A) 1

B)

3 2

C)

D)

E)

10) Um episdio muito conhecido na Matemtica foi quando ao visitar o grande matemtico Ramanujam no hospital, o outro grande matemtico Hardy disse que o nmero do txi que o trouxe, 1729, era um nmero sem graa; Ramanujam respondeu prontamente: No diga isso, Hardy! 1729 o menor nmero inteiro positivo que pode ser escrito como soma de dois cubos perfeitos positivos de duas maneiras diferentes! De fato, 1729 = 103 + 93 = 123 + 13. Um outro episdio no muito conhecido na Matemtica foi quando o pequeno matemtico Muralijam foi visitado pelo outro pequeno matemtico Softy, que disse que o nmero do lotao que o trouxe era um nmero sem graa. Muralijam responde imediatamente: No, Softy, ele o menor inteiro positivo que pode ser escrito como soma de dois quadrados perfeitos positivos de duas maneiras diferentes! A que nmero Muralijam e Softy se referem? A) 18 B) 41 C) 45 D) 50 E) 65 11) Dizemos que uma palavra Q quase-anagrama de outra palavra P quando Q pode ser obtida retirando-se uma letra de P e trocando a ordem das letras restantes, resultando em uma palavra com uma letra a menos do que P. Um quase-anagrama pode ter sentido em algum idioma ou no. Por exemplo, RARO, RACR e ARCO so quase-anagramas de CARRO. Quantos so os quase-anagramas da palavra BACANA que comeam com A? A) 48 B) 60 C) 72 D) 96 E) 120 12) As cidades Apolis, Bepolis e Cepolis so ligadas por estradas retas. Sabese a estrada que liga Apolis e Bepolis perpendicular estrada que liga Apolis e Cepolis. Rubens mora em Bepolis e tem um compromisso em Cepolis. Todavia, a estrada que liga Bepolis a Cepolis est interditada, de modo que Rubens obrigado a fazer o trajeto Bepolis-Apolis-Cepolis. Para chegar ao compromisso na hora certa, Rubens trafega com uma velocidade 24% maior do que trafegaria se utilizasse a estrada interditada. Se o menor ngulo do tringulo determinado pelas trs estradas, ento

1 6 1 1 C) < tg < 5 4
A) 0 < tg <
EUREKA! N28, 2008

1 < tg < 6 1 D) < tg < 4


B)

1 5 1 3

E)

1 < tg < 1 3

12

Sociedade Brasileira de Matemtica

13) Todo nmero real a pode ser escrito de forma nica como a = a + {a} , em que a inteiro e 0 {a} < 1 . Chamamos a parte inteira de a e {a} parte fracionria de a. Se x + y + {z} = 4,2 , y + z + {x} = 3,6 e z + x + {y} = 2 , quanto vale x y + z? A) 1 B) 0,5 C) 0 D) 0,5 E) 1 14) Dizemos que um natural X um repunit quando os seus algarismos so todos iguais a 1, ou seja, quando X da forma 111. Sejam p, q e r inteiros, p > 0 , tais que pX 2 + qX + r um repunit sempre que X um repunit. Qual dos valores a seguir um possvel valor de q? A) 2 B) 1 C) 0 D) 1 E) 2 15) O conjunto dos valores de c para os quais a equao soluo real est contido em: B) ] ;1] C) [3;2] D) [2;3] A) [1; [
x= x + c possui

E) Z

16) No tringulo ABC, AD a altura relativa ao lado BC. Se AB = DC = 1, assinale a alternativa que corresponde rea mxima do tringulo ABC. 1 3 3 3 2 2 A) B) C) D) E) 8 2 3 2 2 17) O nmero de pares (x, y) de inteiros positivos que satisfazem a equao x8 + 3y 4 = 4x 2 y 3 , com 1 y 2007, igual a: A) 40 B) 41 C) 42 D) 43 E) 44 18) Sejam a, b e c nmeros tais que a2 ab = 1 b2 bc = 1 c2 ac = 1 O valor de abc(a + b + c) igual a: A) 0 B) 1 C) 2 19) Veja o problema No. 19 do Nvel 2. 20) Veja o problema No. 20 do Nvel 2. 21) Veja o problema No. 21 do Nvel 2.
EUREKA! N28, 2008

D) 1

E) 3

13

Sociedade Brasileira de Matemtica

22) O mximo divisor comum entre os nmeros 1221, 2332, 3443, 4554,........, 8998 : A) 3 B) 33 C) 37 D) 11 E) 101 23) Veja o problema No. 23 do Nvel 2. 24) Veja o problema No. 24 do Nvel 2. 25) Veja o problema No. 25 do Nvel 2.

GABARITO
NVEL 1 (5. e 6. Sries) 1) E 6) A 2) D 7) E 3) D 8) B 4) E 9) D 5) B 10) E NVEL 2 (7. e 8. Sries) 1) E 6) E 2) E 7) C 3) E 8) C 4) D 9) A 5) B 10) E NVEL 3 (Ensino Mdio) 1) A 6) C 2) C 7) E 3) A 8) E 4) A 9) E 5) A 10) D 11) E 12) B 13) C 14) D 15) C 11) B 12) D 13) C 14) C 15) A 11) B 12) D 13) B 14) E 15) A 16) D 17) B 18) C 19) D 20) E 16) D 17) B 18) A 19) B 20) B 16) A 17) E 18) D 19) B 20) B 21) E 22) D 23) B 24) Anulada 25) Anulada 21) E 22) D 23) B 24) Anulada 25) D

EUREKA! N28, 2008

14

Sociedade Brasileira de Matemtica

XXIX OLIMPADA BRASILEIRA DE MATEMTICA


Problemas e Solues da Segunda Fase PROBLEMAS Nvel 1 PARTE A (Cada problema vale 5 pontos)
01. O nmero N = 1010010100101... contm somente os algarismos 0 e 1, de modo que o nmero de algarismos 0 entre dois algarismos 1 um ou dois, alternadamente. O nmero N tem exatamente 101 algarismos. Qual a soma de todos os algarismos do nmero N? 02. Uma folha de papel tem 20 cm de

comprimento por 15 cm de largura. Dobramos essa folha ao meio, paralelamente sua largura. Em seguida, dobramos a folha retangular dupla, de modo que dois vrtices opostos coincidam. Ao desdobrar a folha, as marcas da segunda dobra dividem a folha em duas partes, conforme mostrado na figura ao lado. Qual a rea da parte escura, em cm2?
03. Observe as igualdades a seguir:

1+ 2 +1 = 4 1+ 2 + 3 + 2 +1 = 9

1 + 2 + 3 + 4 + 3 + 2 + 1 = 16 M 1 + 2 + 3 + L + 2006 + 2007 + 2006 + L 3 + 2 + 1 = A A ? Qual o valor de 2232


04. Uma folha retangular de cartolina foi cortada ao longo de sua diagonal. Num

dos pedaos restantes, na forma de um tringulo retngulo, foram feitos dois cortes, paralelos aos lados menores, pelos meios desses lados. Ao final sobrou um retngulo de permetro 129 cm. O desenho abaixo indica a seqncia de cortes.

EUREKA! N28, 2008

15

Sociedade Brasileira de Matemtica

Em centmetros, qual era o permetro da folha antes do corte?


05. Um reservatrio cbico internamente tem 2 metros de lado e contm gua at

a sua metade. Foram colocados no reservatrio 25 blocos retangulares de madeira, que no absorvem gua, de dimenses 20 30 160 centmetros. Sabendo que 80% do volume de cada bloco permanece submerso na gua, calcule, em centmetros, a altura atingida pela gua, no reservatrio.
06. A adio ao lado est incorreta. Entretanto, se

substituirmos somente um certo algarismo a, toda vez que ele aparece, por um certo algarismo b, a conta fica correta. Qual o valor de a b ? PARTE B (Cada problema vale 10 pontos)
PROBLEMA 1

A rea do quadrado ABCD 300 cm2. Na figura, M ponto mdio de CD e o ponto F pertence reta BC. a) Qual a rea do tringulo ABF ? b) Qual a rea do tringulo ADF ?
M

PROBLEMA 2

Esmeralda comprou seis discos de ferro para usar num aparelho de ginstica. Esses discos tm massas 1, 2, 3, 4, 5 e 6 quilogramas, respectivamente. Esmeralda pode combin-los e obter outras massas, como por exemplo: 1 disco de 2 kg + 1 disco de 6 kg = 8 kg. Qual a maior quantidade de massas diferentes que ela pode obter?

EUREKA! N28, 2008

16

Sociedade Brasileira de Matemtica

PROBLEMA 3

Observe como o quadriculado ao lado preenchido.


3

a) Qual a soma dos elementos da diagonal 9? b) Qual o resto da diviso por 100 da soma dos elementos da diagonal 2007?
3

3 3

PROBLEMAS Nvel 2 PARTE A (Cada problema vale 4 pontos)


01. Ludmilson descobriu que o produto da idade que tinha h 55 anos atrs pela

idade que ter daqui a 55 anos igual ao cubo de um nmero primo. Qual a idade atual de Ludmilson?
02. Sendo f(x) = 100x + 3, calcule o valor de

f (10 8 ) f (103 ) 10 8 103

f (1) .

03. Na figura abaixo temos um pentgono regular, um quadrado e um tringulo

eqiltero, todos com a mesma medida de lado.


Q C R P B E

Determine a medida, em graus, do ngulo QCE.


04. Um inteiro positivo K tem n algarismos e igual a 2608.n. Determine a soma dos algarismos de K 05. Em 1949 o matemtico indiano D. R. Kaprekar, inventou um processo

conhecido como Operao de Kaprekar. Primeiramente escolha um nmero de


EUREKA! N28, 2008

17

Sociedade Brasileira de Matemtica

quatro dgitos (no todos iguais), em seguida escreva a diferena entre o maior e o menor nmero que podem ser formados a partir de uma permutao dos dgitos do nmero inicial. Repetindo o processo com cada nmero assim obtido, obtemos uma seqncia. Por exemplo, se o primeiro nmero for 2007, o segundo ser 7200 0027 = 7173. O terceiro ser 7731 1377 = 6354. Comeando com o nmero 1998, qual ser o 2007-simo termo da seqncia?

PROBLEMAS Nvel 2 PARTE B (Cada problema vale 10 pontos)


PROBLEMA 1

O tringulo ABC retngulo em B. Sejam I o centro da circunferncia inscrita em ABC e O o ponto mdio do lado AC. Se AOI = 45, quanto mede, em graus, o ngulo ACB?
PROBLEMA 2

Sejam e as razes da equao quadrtica (x 2)(x 3) + (x 3)(x + 1) + (x + 1)(x 2) = 0.

Determine o valor de
PROBLEMA 3

1 1 1 . + + ( + 1)( + 1) ( 2)( 2) ( 3)( 3)

a) Determine a quantidade de divisores do nmero N = 235 23. b) Mostre que para todo nmero natural n , n5 n mltiplo de 30.
PROBLEMA 4

Um quadrado 4 4 dividido em 16 quadrados unitrios. Cada um dos 25 vrtices desses quadrados deve ser colorido de vermelho ou azul. Ache o nmero de coloraes diferentes tais que cada quadrado unitrio possua exatamente dois vrtices vermelhos.

PROBLEMAS Nvel 3 PARTE A (Cada problema vale 4 pontos)


01. Quantos divisores positivos do nmero 123456 so menores que 2007?

EUREKA! N28, 2008

18

Sociedade Brasileira de Matemtica

02. Considere o conjunto A dos pares ordenados (x;y) de reais no negativos tais

que x + y = 2. Se a probabilidade de um elemento de A escolhido aleatoriamente estar a uma distncia da origem menor ou igual a 5 3 p, quanto vale 2535p2?

111...1? 03. Qual a soma dos algarismos do inteiro mais prximo de 1 424 3
1000 uns

04. Veja o problema 1 da parte B do nvel 2. 05. Veja o problema 4 da parte B do nvel 2.

PROBLEMAS Nvel 3 PARTE B (Cada problema vale 10 pontos)


PROBLEMA 1

Ache todos os pares (x, y) de inteiros positivos tais que 2(x + y) + xy = x2 + y2.
PROBLEMA 2

Encontre todos os nmeros n de seis algarismos da forma AAABBB, em que A e B so algarismos diferentes e no nulos e n + 1 um quadrado perfeito.
PROBLEMA 3

No quadriltero convexo ABCD, A + B = 120, AD = BC = 5 e AB = 8. Externamente ao lado CD, construmos o tringulo eqiltero CDE. Calcule a rea do tringulo ABE.
PROBLEMA 4

Em um certo pas h 21 cidades e o governo pretende construir n estradas (todas de mo dupla), sendo que cada estrada liga exatamente duas das cidades do pas. Qual o menor valor de n para que, independente de como as estradas sejam construdas, seja possvel viajar entre quaisquer duas cidades (passando, possivelmente, por cidades intermedirias)?

EUREKA! N28, 2008

19

Sociedade Brasileira de Matemtica

Solues Nvel 1 Segunda Fase Parte A


Problema Resposta 01 41 02 150 03 81 04 258 05 148 06 64

1. [41] O nmero formado por blocos iguais, de 5 algarismos na forma 10100. Como o nmero tem 101 algarismos, conclumos que formado por 20 desses blocos inteiros mais o primeiro algarismo de um bloco, que 1. A soma dos algarismos de cada bloco 1 + 0 +1 + 0 + 0 = 2, portanto a soma dos algarismos de N 20 2 + 1 = 41 . 2. [150] O desenho abaixo esquerda mostra como fica a folha aps a primeira dobra. direita, mostra como fica a folha aps as duas dobras.

Observamos que CE = EA e que CF = FA. Por uma propriedade da dobra, sabemos que o segmento FE perpendicular ao segmento AC e esses segmentos se cruzam em seus pontos mdios. Portanto, os quatro tringulos que compem o quadriltero AECF so congruentes; so congruentes tambm os tringulos EBC e FDA. Portanto, a dobra FE divide o retngulo ABCD em dois trapzios, EBCF e AEFD, de mesma rea. Desdobrando inteiramente a folha, obtemos duas metades iguais. Portanto, a rea do pentgono convexo BEFEB igual rea do pentgono no convexo AAEFE, ou seja, a rea da parte escura metade da rea da folha, portanto igual a

15 20 = 150 cm 2 . 2

EUREKA! N28, 2008

20

Sociedade Brasileira de Matemtica

3. [81] Pelo padro observado, as somas so iguais ao quadrado da parcela central (aquela cujo nmero de parcelas esquerda igual ao nmero de parcelas direita). Portanto, A = 2007 2 e, assim,

A 2007 2 2007 2 = = = 9 = 81 . 2 2 223 223 223

4. [258] O retngulo que sobra aps os cortes tem lados iguais s metades dos lados da cartolina original, cujo permetro, ento, o dobro do permetro desse retngulo. Logo, o permetro da cartolina antes do corte 129 2 = 258 cm. 5. [148] O volume de cada bloco de madeira 0, 2 0,3 1, 60 = 0, 096 m 3 ; o volume de cada bloco que fica submerso no lquido 0,80 0, 096 m 3 . O volume de lquido deslocado pelos 25 blocos igual a 25 0,80 0, 096 =1,92 m3 . Como o reservatrio um cubo de 2 m de lado, sua base um quadrado de rea 4 m2. Podemos pensar no lquido deslocado como se fosse um bloco cuja base igual base do reservatrio, de altura h e volume acima. Portanto 4h = 1,92 h =

1,92 = 0, 48 m = 48 cm . Como a altura inicial 4

do lquido era 100 cm, a nova altura ser 148 cm.

EUREKA! N28, 2008

21

Sociedade Brasileira de Matemtica

6. [64] primeira inspeo, podemos admitir que os trs algarismos direita de todos os nmeros esto corretos, isto , esto corretamente escritos os algarismos 0, 1, 3, 4, 5, 6 e 8. Portanto, dentre os algarismos 2, 7 e 9, um deles est escrito incorretamente. O 9 est escrito corretamente, pois se o mudarmos, a soma com 2 no estar certa. Logo ou 2 ou 7 est errado. Se o 7 estiver errado, ento 2 estar correto, mas isso no possvel pois a soma de 2 com 4 mais 1 no estaria certa. Logo, o 2 que deve ser substitudo; olhando novamente a soma de 2 com 4 mais 1 resultando 1 vemos que o resultado s dar certo se no lugar de 2 colocarmos 6. Fazendo a substituio, verificamos que o resto se encaixa. Teremos, ento, a b = 26 = 64 .

Solues Nvel 1 Segunda Fase Parte B


1. Temos

) = m( AMD ) m( FMC (ngulos opostos pelo vrtice), ) (pois ABCD quadrado, logo esses ngulos so ) = m( FCM m( ADM

retos) e MC = MD (pois M ponto mdio de CD). Logo, os tringulos AMD e FMC so congruentes.

EUREKA! N28, 2008

22

Sociedade Brasileira de Matemtica

a) Vemos que a rea ABF = rea FMC + rea ABCM . Como rea FMC = rea AMD , temos:

rea ABF = rea AMD + rea ABCM = rea do quadrado ABCD = 300
cm2. b) rea ADF = rea AMD + rea DMF = = rea FMC + rea DMF = rea FCD

eD so retos, Como AD = FC, CD lado comum e os ngulos C conclumos que os tringulos FCD e ADC so congruentes,
logo rea FCD = rea ADC = ADF igual a

rea ABCD . Portanto, a rea do tringulo 2

300 = 150 cm 2 . 2

2. Dadas as massas de 1 a 6, podemos adicionar 1 a 6, 2 a 6, etc, at obter todos os pesos de 7 a 11; podemos adicionar 1 + 5 a 6, 2 + 5 a 6, etc, at obter todos os pesos de 12 a 15; podemos adicionar 1 + 4 + 5 a 6, etc, obtendo os pesos de 16 a 18; somando 1 + 3 + 4 + 5 a 6 obtemos 19; 2 + 3 + 4 + 5 a 6 obtemos 20 e, finalmente, somando 1 + 2 + 3 + 4 + 5 a 6 obtemos 21. Portanto, a quantidade de massas diferentes que Esmeralda pode obter 21. 3. Pode-se concluir, examinando a tabela, que a soma dos elementos da diagonal n igual a 2n + (n 1)k, onde k o algarismo das unidades do nmero n. Por exemplo, na diagonal de nmero 4 a soma dos nmeros
EUREKA! N28, 2008

23

Sociedade Brasileira de Matemtica

2 4 + ( 4 1) 4 = 20 , na diagonal de nmero 10 a soma dos nmeros

2 10 + (10 1) 0 = 20 , etc.
a) Na diagonal de nmero 9, a soma dos elementos 2 9 + ( 9 1) 9 = 90 . De outra forma, na diagonal 9 h 10 nmeros 9; portanto a soma 10 9 = 90 . b) Na diagonal 2007 a soma ser 2 2007 + ( 2007 1) 7 = 4014 + 14042 = 18056 . O resto da diviso desse nmero por 100 56.

Solues Nvel 2 Segunda Fase Parte A


Problema Resposta
01. Seja

01 66

02 197

03 174

04 8

05 6174

x a idade de Ludmilson. Logo, ( x 55)( x + 55) = p 3 , onde p

primo. Temos ento, duas possibilidades: i)

x 55 = 1 3 x + 55 = p Nesse caso teramos x = 56 e p = 111 , absurdo, pois 111 no primo.


ii)

x 55 = p 2 x + 55 = p
Com isso, 110 = p 2 p = p ( p 1) = 11.10 . E assim teremos p = 11 e x = 66 . Logo, a idade de Ludmilson 66 anos. 02. (100.10-8 + 3 100.10 3 3) / (10 8 10 3) 100.(1) 3 = 100(108 10 3) / (108 103) + 97 = 100 + 97 = 197. 03. Note que os tringulos PTA, ABD, BCE, e PQC so todos issceles. Como STP = 108, PTA = PAT = 72. Assim, temos que TPA = 36 e BAD = BDA = 18. Alm disso, ABD = 144 e CBE = 66. Como QPC = 126, temos que QCP = 27 e ECB = 57. Logo, QCE = 174.
EUREKA! N28, 2008

24

Sociedade Brasileira de Matemtica

04. Tente 1, 2, 3 ... e perceba que, somente com n = 5, K ter 5 algarismos. Assim, K = 2608 . 5 = 13040. Com isso, a soma dos algarismos de K 8. 05. A partir do stimo termo, todos sero iguais a 6174.

Solues Nvel 2 Segunda Fase Parte B


SOLUO DO PROBLEMA 1:

Como ABC um tringulo retngulo, ento AO = BO = CO. Se ABI = AOI = 45 o e BAI = OAI , ento ? ABI = ? AOI (ALA). Com isso, AB = AO = BO, e portanto, tringulo ABO eqiltero. Assim, ACB = 30 o .
SOLUO DO PROBLEMA 2:

fcil ver que ( x 2)( x 3) + ( x 3)( x + 1) + ( x + 1)( x 2) = 3( x )( x ) . Fazendo x = 1, 2 e 3, nesta igualdade, temos que,

( + 1)( + 1) = 4 , ( 2)( 2) = 1 , ( 3)( 3) =


Com isso,

4 . 3

1 1 1 1 3 + + = 1 + = 0. ( + 1)( + 1) ( 2)( 2) ( 3)( 3) 4 4

SOLUO DO PROBLEMA 3:

a) N = 23 (234 1) = 23 (232 + 1)(232 1) = 23 (232 + 1)(23 + 1)(23 1) =


EUREKA! N28, 2008

25

Sociedade Brasileira de Matemtica

23 530 24 22 = 25 3 5 11 23 53 O nmero de divisores (positivos) de N 6 2 2 2 2 2 = 192 . b) N = n5 n = n(n 2 + 1)(n + 1)(n 1).


Necessariamente, n ou n + 1 par. Logo, 2 divide N. Do mesmo modo, um dos nmeros n 1, n ou n + 1 mltiplo de 3. Logo 3 tambm divide N. Finalmente, se nenhum dos 3 nmeros n 1, n ou n + 1 mltiplo de 5, ento n da forma 5k + 2 ou 5k + 3. No primeiro caso, temos n 2 + 1 = 25k 2 + 10k + 5 e, no segundo, n 2 + 1 = 25k 2 + 15k + 10 , ambos mltiplos de 5. Portanto, um dos nmeros n, n 1, n + 1 ou n 2 + 1 mltiplo de 5. Assim N , simultaneamente, mltiplo dos nmeros primos entre si 2, 3 e 5, o que prova que N mltiplo de 30.
SOLUO DO PROBLEMA 4:

Vamos comear colorindo a primeira linha de vrtices. Cada colorao dessa linha uma seqncia de letras A e V, por exemplo, A V V A V. Observe que, uma vez colorida a primeira linha, se aparecerem duas letras consecutivas iguais, o restante dos vrtices do tabuleiro j esto determinados. De fato, ao aparecer dois Vs consecutivos, os dois vrtices imediatamente abaixo deles devero ser coloridos com dois As, os que esto mais abaixo devero ter dois Vs, e assim por diante. Isto completa a colorao dessas duas colunas. Dessa forma, cada coluna vizinha tambm estar determinada, pois em cada retngulo teremos trs vrtices previamente coloridos, o que obriga o quarto vrtice a ter sua cor determinada. Ento, para cada seqncia de As e Vs na primeira linha que contm pelo menos duas letras iguais consecutivas, h exatamente uma maneira de colorir o tabuleiro. Como h 25 2 = 30 de tais seqncias, contamos 30 coloraes possveis. A V A V A V V A V A V A V

Falta-nos analisar um segundo caso, em que no h duas letras consecutivas iguais na primeira linha. H duas possibilidades de seqncias: comeando com A ou comeando com V.

EUREKA! N28, 2008

26

Sociedade Brasileira de Matemtica

A V A V A V Para cada uma dessas seqncias, h duas maneiras de escolhermos a primeira letra da segunda linha. Uma vez escolhida esta letra, a segunda linha inteira tambm estar determinada. Para a primeira letra da terceira linha tambm h 2 possibilidades. Com este raciocnio, cada vez que escolhemos a primeira letra de uma linha, determinamos a colorao desta linha. Logo, como h duas maneiras de escolhermos a primeira letra de cada linha, h 25 = 32 maneiras de colorirmos o tabuleiro, neste segundo caso. Logo, o total de coloraes igual a 30 + 32 = 62. Observao: Veja que, no caso geral, para um quadrado n n, o raciocnio anlogo. No primeiro caso, teremos 2n + 1 2 coloraes; no segundo caso, mais 2n + 1. Logo, teremos 22n+1 2 = 2n + 2 2 coloraes.

Solues Nvel 3 Segunda Fase Parte A


Problema Resposta 01 17 02 3024 03 1500 04 30 05 62

01. Seja a fatorao de 123456 = 2 6 3 643 e seja d um de seus divisores

menores do que 2007. Podemos analisar dois casos: - d no mltiplo de 643: ento d um divisor de 2 6 3 = 192 < 2007 . Portanto podemos contar todos os divisores de 192, que so (6 + 1)(1 + 1) = 14 divisores. - d mltiplo de 643: 1 643 = 643 , 2 643 = 1286 e 3 643 = 1929 so menores que 2007, mas a partir de 4 643 = 2572 , eles so maiores que 2007. Portanto h 3 divisores neste caso. Portanto o total de divisores d de 123456 menores do que 2007 14 + 3 = 17.
02. Seja B o conjunto dos pontos de A cuja distncia origem menor do que 5

3 e seja P = ( x; y ) um ponto de B. Sabe-se que P est sobre o segmento


EUREKA! N28, 2008

27

Sociedade Brasileira de Matemtica

x + y = 2; x, y 0 e que a distncia
5 . Portanto: 3

x 2 + y 2 de P origem menor ou igual a y = 2 x 25 2 11 2x 4x + 0 9 9

x+ y =2

x2 + y2 5

( 3)

y = 2x x2 + 4 4x + x2

11 11 9 = 1 14 , que nos d os As razes de 2 x 2 4 x + = 0 so x0 = 9 4 6 14 14 14 14 pontos extremos P1 = 1 6 ;1 + 6 e P2 = 1 + 6 ;1 6 de B. Pela inequao, temos que os pontos de B esto na reta x + y = 2 , delimitados pelos 4 16 8

pontos P 1 e P2 , logo B o segmento de reta P 1P 2 . Queremos a probabilidade p de escolher um ponto do conjunto A estar contido no segmento P1 P2 , que a razo entre P1 P2 e o comprimento de A. Como A est delimitado pelos pontos (0;2) e (2;0) , seu comprimento vale
(0 2) 2 + (2 0) 2 = 2 2 . O comprimento de B vale
1 + 14 1 + 14 + 1 14 1 14 = 2 14 = 2 7 , 3 6 6 6 6 3
2 2 2

portanto

2 7 14 14 p= 3 = e 2 5 35 p 2 = 2 5 35 = 2 4 33 7 = 3024 . 6 36 2 2 1000 noves 6 7 8 99 K 9 101000 1 03. Inicialmente, temos 11 K 1= = . 1 2 3 9 9 1000 uns

Portanto

11 K 1= 1 4 2 4 3
1000 uns

101000 1 . 9

Com isso, observando que


101000 1 (10 500 1)(10 500 + 1) (10 500 1)(10 500 1) 10 500 1 = > = e 9 9 9 3

EUREKA! N28, 2008

28

Sociedade Brasileira de Matemtica

10 500 1 10 500 101000 1 101000 10500 K . <1 11 1 < < = , temos 4 2 4 3 9 9 3 3 3 1000 uns
10 500 10500 1 10 500 + 2 inteiro e seu consecutivo, , maior do que ,o 3 3 3 500 noves 6 7 8 500 10 1 99K 9 K 3 , cuja soma dos 11 K 1 = = 33 inteiro mais prximo de 1 { 4 2 4 3 3 3 500 trs 1000 uns

Como

dgitos 3 500 = 1500 .


04. Veja a soluo do problema 1 da parte B do nvel 2. 05. Veja a soluo do problema 4 da parte B do nvel 2.

Solues Nvel 3 Segunda Fase Parte B


SOLUO DO PROBLEMA 1:

Uma soluo: Multiplicando a equao dada por 2, obtemos 2x2 + 2y2 2xy 4x 4y = 0, ou ainda, (x2 4x + 4) + (y2 4y + 4) + (x2 2xy + y2) = 8. Da, (x 2)2 + (y 2)2 + (x y)2 = 8. A nica maneira de escrevermos 8 como a soma de trs quadrados 8 = 0 + 4 + 4, em alguma ordem. Logo (x 2, y 2) = (0, 2), (2, 0) ou (2, 2), de onde conclumos que as solues so (x, y) = (2, 4), (4, 2) ou (4, 4). Outra soluo: Escrevendo a equao dada como uma equao do segundo grau em x, temos: x2 (y + 2)x + (y2 2y) = 0. O discriminante desta equao = (y + 2)2 4(y2 2y) = 3y2 + 12y + 4. 4 3 4 3 y2+ . Resolvendo a inequao 0, ainda obtemos 2 3 3 Como y inteiro positivo, as nicas possibilidades so y = 1, 2, 3 ou 4. Se y = 1, ficamos com = 13, que no quadrado perfeito. Logo, este caso no tem soluo.
EUREKA! N28, 2008

29

Sociedade Brasileira de Matemtica

Se y = 2, obtemos = 16 e x =

44 = 0 ou 4. Como x inteiro positivo, a 2 nica soluo neste caso (x, y) = (4, 2). Se y = 3, ficamos com = 13, absurdo! 62 Se y = 4, obtemos = 4. Neste caso, x = = 2 ou 4. Logo, (x, y) = (2, 4) ou 2 (4, 4). Portanto, o conjunto soluo {(2, 4), (4, 2), (4, 4)}.

Mais uma soluo: Observe que 8(x + y) = 4x2 4xy + 4y2 = (x + y)2 + 3(x y)2 (x + y)2, de modo que 8(x + y) (x + y)2, ou seja, x + y 8. Alm disso, note que x2 xy + y2 = 2(x + y) par, e portanto ao menos uma das parcelas do primeiro membro par (se todos forem mpares, x2 xy + y2 mpar), o que implica que x ou y par. Suponha, sem perda de generalidade, que x par. Ento y2 = 2(x + y) + xy x2 par e, assim, y tambm par. Logo, dos dois fatos acima, conclui-se que as nicas possibilidades para os pares (x, y) so (2, 2), (2, 4), (2, 6), (4, 2), (4, 4) e (6, 2). Substituindo os pares, vemos que as nicas solues so (2, 4), (4, 2) e (4, 4).
SOLUO DO PROBLEMA 2:

Seja k inteiro positivo tal que k2 = n + 1. Primeiro, notemos que o algarismo das unidades dos quadrados perfeitos so 0, 1, 4, 5, 6 e 9, de modo que B igual a 9, 3, 4, 5 ou 8. Porm, podemos eliminar alguns casos: Se B = 9, pois nesse caso k2 = AAABBB + 1 terminaria com exatamente trs zeros (note que A no pode ser igual a 9, pois diferente de B); Se B = 3, k2 terminaria com 34, e seria par e no mltiplo de 4, j que os dois ltimos algarismos de todo mltiplo de 4 formam outro mltiplo de 4, um absurdo. Se B = 4, k2 terminaria com 45, e seria mltiplo de 5 mas no de 25, j que os dois ltimos algarismos de um mltiplo de 25 so 25, 50, 75 ou 00. Outro absurdo.

EUREKA! N28, 2008

30

Sociedade Brasileira de Matemtica

Sobram somente os casos B = 5 e B = 8. Observe que n = k2 1 = (k 1)(k + 1) = AAABBB = 111(1000A + B) mltiplo de 111 = 3 37 e, portanto, os primos 3 e 37 dividem k + 1 ou k 1, de modo que k da forma 111x 1 ou 111x 38 . Alm disso, 2 111556 k < 1000000 300 < k < 1000 , de modo que 3 x 9 . k = 111x 1 : Temos AAABBB = k 2 1 = 1112 x 2 222 x 1000 A + B = 111x 2 2 x . O dgito das unidades de 1000A + B B. Note que 111x 2 2 x = 2(55 x 2 x) + x 2 tem a mesma paridade que x. Assim, se B = 5, x mpar, ou seja, 3, 5, 7 ou 9. Se x = 3, 5, 7, 9, o algarismo das unidades de 111x 2 + 2 x 5, 5, 3, 9, respectivamente, de modo que x = 3 ou x = 5, para o qual 1000A + B iguala 111 9 + 6 = 1005 e 111 25 + 10 = 2785 , o que gera a soluo x = 3, A = 1 e n = 111555. Alm disso, se x = 3, 5, 7, 9, o algarismo das unidades de 111x 2 2 x 3, 5, 5, 3, respectivamente, de modo que as nicas possibilidades so x = 5 ou x = 7, para os quais 1000 A + B iguala 2765 e 111 49 14 = 5425 respectivamente, o que tambm no possvel. Se B = 8, x par, ou seja, 4, 6 ou 8. Se x = 4, 6, 8, o algarismo das unidades de 111x 2 + 2 x 4, 8, 0, respectivamente, de modo que obtemos x = 6 e 1000A + B = 111 36 + 12 = 4008 , ou seja, A = 4. Obtemos assim a soluo n = 444888. Alm disso, se x = 4, 6, 8, o algarismo das unidades de 111x 2 2 x 8, 4, 8 respectivamente, de modo que obtemos x = 4 ou x = 8, para os quais 1000A + B igual a 111 16 8 = 1768 e 111 64 16 = 7088 , respectivamente, o que no possvel. k = 111x 38 : Temos AAABBB = k 2 1 = 1112 x 2 2 111 38 x + 38 2 1 = 1112 x 2 111 76 x + 37 39 = 111(111x 2 76 x + 13) 1000 A + B = 111x 2 76 x + 13 . Estudemos, como no caso anterior, o dgito das unidades de 111x 2 76 x + 13 . Se B = 5, x par, ou seja, igual a 4, 6 ou 8. Se x = 4, 6, 8, o algarismo das unidades de 111x 2 + 76 x + 13 3, 5, 5, respectivamente, de modo que x = 6 ou 8, para os quais 1000A + B iguala respectivamente 111 36 + 76 6 + 13 = 4465 e 111 64 + 76 8 + 13 = 7725 , nenhum dos dois gerando soluo. Alm disso, se
EUREKA! N28, 2008

31

Sociedade Brasileira de Matemtica

x = 4, 6, 8, o algarismo das unidades de 111 x 2 76 x + 13 5, 3, 9, respectivamente, de modo que x = 2 e 1000A + B igual a 111 16 76 4 + 13 = 1485 , o que no possvel. Se B = 8, x mpar, ou seja, igual a 3, 5, 7 ou 9. Se x = 3, 5, 7, 9 o algarismo das unidades de 111x 2 + 76 x + 13 0, 8, 4, 8, respectivamente, de modo que x = 5 ou x = 9, para os quais 1000A + B = 111 25 + 76 5 + 13 = 3168 e k = 111 9 + 38 > 1000 , o que no possvel. Alm disso, se x = 3, 5, 7, 9 o algarismo das unidades de 111x 2 76 x + 13 4, 8, 0, 0, respectivamente, de modo que x = 5, para o qual 1000A + B = 111 25 76 5 + 13 = 2408 , o que no possvel. Portanto os nicos nmeros n que satisfazem o enunciado so 111555 e 444888.
SOLUO DO PROBLEMA 3:

Uma soluo:
E F

B A

Prolongue AD e BC at se encontrarem no ponto F. Veja que AFB = 60 = DEC. Com isso, o quadriltero FECD inscritvel. Temos: (i) FDE = FCE = ADE = BCE = 180 . (ii) AD = BC e ED = EC. De (i) e (ii), conclumos que ADE BCE. Portanto, EA = EB.
EUREKA! N28, 2008

32

Sociedade Brasileira de Matemtica

Alm disso, DEA = CEB, de onde conclumos que AEB = DEC = 60. Dessa forma, o tringulo ABE eqiltero de lado 8 e sua rea igual a 82 3 = 16 3 cm2. 4 Outra soluo: Considere os pontos no plano complexo. Representaremos o nmero complexo correspondente ao ponto X com a letra correspondente minscula x. Fixemos o ponto mdio de AB como origem e sejam a = 4 e b = 4. Assim, sendo = BAD e = ABC , ambos no sentido anti-horrio, podemos encontrar as coordenadas de C e D: 5 c b = (a b) cis( ) c = 4 5 cis( ) 8 5 d a = (b a) cis d = 4 + 5 cis 8 Sendo = cis a raiz sexta da unidade e raiz da equao x 2 x + 1 = 0 , 3 e d = (c d ) e = (1 )d + c = c 2 d = 4 5 cis( ) + 4 2 5 2 cis
2 e = 4( + 1) 5 cis 3 + cis 3 + 1+ i 3 2 2 cis + e = 4 2 1 5 + + cis 2 3 3 3 2 2 e = 4 3i 5 cis 3 + + + cis 3 + 2 2 e = 4 3i 5 cis 3 + + cis 3 + = 4 3i

Assim, o tringulo ABE, com pontos de coordenadas A = (4, 0), B = (4, 0) e 84 3 E = (0,4 3 ) , eqiltero e tem rea = 16 3 cm2. 2

EUREKA! N28, 2008

33

Sociedade Brasileira de Matemtica

SOLUO DO PROBLEMA 4:

Escolha 20 das cidades do pas. Ligando duas quaisquer delas por uma estrada, 20 20.19 utilizaremos 2 = 2 = 190 estradas, e a cidade restante no poder ser alcanada de automvel. Logo se deve construir pelo menos 191 estradas. Vamos mostrar que com essa quantidade possvel atingir nosso objetivo. Suponha que n = 191, mas que seja possvel dividir as cidades do pas em dois grupos A e B, digamos com a e b cidades, respectivamente, de tal sorte que nenhuma cidade de A possa ser alcanada de automvel a partir de qualquer a b cidade de B. Ento o nmero de estradas no pas no mximo 2 + , de 2
a b 2 2 modo que 2 + 2 191, ou ainda, (a + b ) (a + b) 2191 = 382.

Como a + b = 21, segue da inequao acima que a2 + b2 282 + 21 = 403. Logo (a + b) 2 (a 2 + b 2 ) 441 403 ab = = 19. 2 2 Mas, como a + b = 21 e a e b so naturais, temos ab 120 = 20, uma contradio. Logo, se n = 191, sempre possvel viajar entre quaisquer duas cidades.

EUREKA! N28, 2008

34

Sociedade Brasileira de Matemtica

XXIX OLIMPADA BRASILEIRA DE MATEMTICA


Problemas e Solues da Terceira Fase PROBLEMAS NVEL 1
PROBLEMA 1

Parte das casas de um quadriculado com o mesmo nmero de linhas (fileiras horizontais) e colunas (fileiras verticais) pintada de preto, obedecendo ao padro apresentado pelo desenho ao lado. a) Quantas casas sero pintadas num quadriculado com 14 linhas e 14 colunas, de acordo com esse padro? b) Quantas linhas tem um quadriculado com 199 casas pintadas?

PROBLEMA 2

Uma sala quadrada com 81 m2 de rea tem o seu piso inteiramente coberto por dois tapetes retangulares A e B, que no se superpem, conforme mostrado na figura (1) abaixo. Em certo momento, o tapete B deslocado, o tapete A girado de 90o e colocado sobre o tapete B, conforme indicado na figura (2).

Sabendo que a rea do tapete B o dobro da rea do tapete A, calcule a rea da parte do piso que ficou descoberta.
PROBLEMA 3

Em uma face de cada um de trs cartes foi escrito um nmero inteiro positivo. Em seguida, os cartes foram colocados lado a lado sobre uma mesa, com a face numerada para baixo.
EUREKA! N28, 2008

35

Sociedade Brasileira de Matemtica

Arnaldo, Bernaldo e Cernaldo sabem que: I. Os nmeros escritos nos cartes so todos diferentes. II. A soma dos trs nmeros 13. III. Os nmeros crescem da esquerda para a direita. a) Considerando as condies I, II e III, escreva todas as possibilidades de numerao dos cartes. b) Agora hora de descobrir os nmeros que foram escritos nos cartes. Primeiramente, Arnaldo olha o nmero do primeiro carto esquerda e diz que no tem informaes suficientes para descobrir os outros dois nmeros sem levantar os outros cartes. Depois, Bernaldo levanta o ltimo carto direita, olha o nmero e diz tambm que no consegue descobrir os dois nmeros esquerda, sem levantar todos os cartes. E o mesmo acontece com Cernaldo, que levanta o carto do meio, olha seu nmero e afirma que no consegue descobrir os nmeros nos outros dois cartes. Sabendo que todos ouvem o que os demais dizem, mas no vem o carto que o outro olhou, qual nmero est escrito no carto do meio?
PROBLEMA 4

Considere a tabela a seguir com quatro linhas (fileiras horizontais) e quatro colunas (fileiras verticais) a qual est preenchida com nmeros naturais, ocorrendo repeties de nmeros: 1 5 1 6 0 1 1 1 0 2 2 4 3 4 3 0

Ao somarmos os nmeros de cada uma de suas linhas (L1, L2, L3 e L4) e colunas (C1, C2, C3 e C4) obtemos 8 nmeros distintos: 3, 4, 7, 8, 10, 11, 12, 13. Veja: C1 C2 C3 C4 L1 L2 L3 L4 Soma da coluna
EUREKA! N28, 2008

1 5 1 6

0 1 1 1

0 2 2 4 8

3 4 3 0 10

Soma da linha 4 12 7 11

13 3

36

Sociedade Brasileira de Matemtica

Apresente, se for possvel: a) uma tabela com 4 linhas e 4 colunas, formada por nmeros naturais, podendo ocorrer repeties de nmeros, na qual apaream como somas de linhas ou colunas os nmeros de 1 a 8. b) uma tabela com 8 linhas e 8 colunas, formada por nmeros naturais, podendo ocorrer repeties de nmeros, na qual apaream como somas de linhas ou colunas os nmeros de 1 a 16. c) uma tabela com 9 linhas e 9 colunas, formada por nmeros naturais, podendo ocorrer repeties de nmeros, na qual apaream como somas de linhas ou colunas os nmeros de 1 a 18. Ateno: caso seja impossvel montar alguma tabela, voc deve explicar porque.
PROBLEMA 5

L 5 222222 L 2 , calcule a soma dos algarismos de Sendo A = 555555 14 24 3 14 24 3


2007 cincos 2007 dois

9 A . No se esquea de justificar a sua resposta.


PROBLEMAS NVEL 2
PROBLEMA 1

Seja ABC um tringulo e O seu circuncentro. Seja ainda P a interseco das retas BO e AC e S a circunferncia circunscrita a AOP. Suponha que BO = AP e que a medida do arco OP em S que no contm A 40. Determine a medida do ngulo OBC. Obs: A circunferncia circunscrita de um tringulo a circunferncia que passa pelos seus vrtices e seu centro chamado de circuncentro.

EUREKA! N28, 2008

37

Sociedade Brasileira de Matemtica

PROBLEMA 2

Considere a tabela a seguir com quatro linhas (fileiras horizontais) e quatro colunas (fileiras verticais) a qual est preenchida com nmeros naturais, ocorrendo repeties de nmeros: 1 5 1 6 0 1 1 1 0 2 2 4 3 4 3 0

Ao somarmos cada uma de suas linhas (L1, L2, L3 e L4) e colunas (C1, C2, C3 e C4) obtemos 8 nmeros distintos: 3, 4, 7, 8, 10, 11, 12, 13. Veja: C1 C2 C3 C4 Soma da Linha L1 1 0 0 3 4 L2 5 1 2 4 12 L3 1 1 2 3 7 L4 6 1 4 0 11 Soma da 13 3 8 10 Coluna Apresente, se for possvel: a) uma tabela com 4 linhas e 4 colunas, formada por nmeros naturais, podendo ocorrer repeties de nmeros, na qual apaream como somas de linhas ou colunas os nmeros de 1 a 8. b) uma tabela com 8 linhas e 8 colunas, formada por nmeros naturais, podendo ocorrer repeties de nmeros, na qual apaream como somas de linhas ou colunas os nmeros de 1 a 16. c) uma tabela com 9 linhas e 9 colunas, formada por nmeros naturais, podendo ocorrer repeties de nmeros, na qual apaream como somas de linhas ou colunas os nmeros de 1 a 18. Ateno: caso seja impossvel montar alguma tabela, voc deve explicar porque.
PROBLEMA 3

Mostre que existe um inteiro positivo a tal que fatores primos distintos.

a 29 1 tem pelo menos 2007 a 1

EUREKA! N28, 2008

38

Sociedade Brasileira de Matemtica

SEGUNDO DIA PROBLEMA 4

Prove que no existem solues inteiras e positivas para a equao

3m + 3n + 1 = t 2 .
PROBLEMA 5

Seja ABC um tringulo retngulo issceles. K e M so pontos sobre hipotenusa AB, com K entre A e M, e o ngulo KCM = 45 o. Prove que AK2 + MB2 = KM2.
PROBLEMA 6

Quadradinhos iguais esto arrumados formando um tabuleiro n n. Ludmilson e Ednalva jogam o seguinte estranho jogo. Cada jogada de Ludmilson consiste em retirar 4 quadradinhos que formem um quadrado 2 2. Cada jogada de Ednalva consiste em retirar apenas 1 quadradinho. Ludmilson e Ednalva jogam alternadamente, sendo Ludmilson o primeiro a jogar. Quando Ludmilson no puder fazer sua jogada, ento Ednalva fica com todas as peas restantes do tabuleiro. Ganha o jogo aquele que possuir mais quadradinhos no final. Diga se possvel que Ednalva ganhe o jogo, no importando como Ludmilson jogue, em cada um dos seguintes casos: a) n = 10. b) Caso geral (n qualquer).

TERCEIRA FASE NVEL 3 (Ensino Mdio)


PRIMEIRO DIA PROBLEMA 1

Seja f(x) = x2 + 2007x + 1. Prove que, para todo n inteiro positivo, a equao f ( f (K ( f ( x)) K)) = 0 tem pelo menos uma soluo real 1 4 24 3
n vezes

PROBLEMA 2

Para quantos nmeros inteiros c, 2007 c 2007 , existe um inteiro x tal que x2 + c mltiplo de 22007

EUREKA! N28, 2008

39

Sociedade Brasileira de Matemtica

PROBLEMA 3

So dados n pontos no plano, os quais so os vrtices de um polgono convexo. Prove que o conjunto das medidas dos lados e das diagonais do polgono tem pelo menos n / 2 elementos distintos. Observao: x denota o maior nmero inteiro que no excede x. Por exemplo, 2,5 = 2 , 3 = 3 e 1,2 = 2 .
SEGUNDO DIA PROBLEMA 4

Arrumam-se 20072 quadradinhos iguais, formando um tabuleiro 2007 2007 . Arnaldo e Bernaldo disputam o seguinte jogo: cada jogada de Arnaldo consiste em retirar 4 quadradinhos que formem um quadrado 2 2 . Cada jogada de Bernaldo consiste em retirar apenas 1 quadradinho. Os jogadores jogam alternadamente, sendo Arnaldo o primeiro a jogar. Quando Arnaldo no puder fazer sua jogada, Bernaldo fica com todas as peas restantes do tabuleiro. Ganha o jogo aquele que possuir mais quadradinhos no final. possvel que Bernaldo ganhe o jogo, no importando como Arnaldo jogue?
PROBLEMA 5

Seja ABCD um quadriltero convexo, P a interseo das retas AB e CD, Q a interseo das retas AD e BC e O a interseo das diagonais AC e BD. Prove que se POQ um ngulo reto ento PO bissetriz de AOD e QO bissetriz de AOB.
PROBLEMA 6

Dados nmeros reais x1 < x2 < < xn, suponha que todo nmero real ocorre no mximo duas vezes entre as diferenas xj xi, com 1 i < j n . Prove que h pelo menos n / 2 nmeros reais que ocorrem exatamente uma vez entre tais diferenas. Observao: caso voc tenha se esquecido da prova de ontem, x denota o maior nmero inteiro que no excede x. Por exemplo, 2,5 = 2 , 3 = 3 e 1,2 = 2 .

EUREKA! N28, 2008

40

Sociedade Brasileira de Matemtica

SOLUES NVEL 1
SOLUO DO PROBLEMA 1: LIARA GUINSBERG (SO PAULO SP)

Considerando a figura, conseguimos ver um padro (de cima para abaixo e da esquerda para a direita). Nmero de quadrados pintados: 22:2 33:7 44:8 5 5 : 17 6 6 : 18 7 7 : 31 8 8 : 32 Podemos perceber que, do 3 3 (7 pintados) para o 4 4 (8 pintados) que o nmero aumentou 1 unidade pintada. O fato se deve seqncia de quadrados pintados, do 2 2 para o 3 3, o nmero de quadrados pretos cresceu em 5 unidades enquanto o branco permaneceu igual, mas do 3 3 para o 4 4, o nmero de brancos aumentou 6, enquanto o preto somente 1. Em geral, se n par, do n n para o (n + 1) (n + 1) o nmero de quadrados pretos cresce em 2n + 1 unidades, mas se n mpar cresce em apenas 1 unidade. Para o caso do quadrado n n, com n par, como a quantidade de casas pretas n2 . Para o igual quantidade de casas brancas, a quantidade de casas pretas ser 2 caso do quadrado n n, com n mpar, percebemos que, a quantidade de casas (n + 1)2 pretas ser 1 (devido s descobertas anteriores). Com efeito, para n par, 2 2 ( n + 2 ) 1, e , para n mpar, (n + 1)2 1 + 1 = n + 1 2 . n2 + 2n + 1 = 2 2 2 2 Usando estes fatos: a) Num quadriculado de 14 14, usamos o padro para pares: casas pretas =
142 = nmero de 2

196 = 98 casas pretas. 2 b) Para descobrirmos quando o quadrado tem 199 casas pintadas, vamos testar os casos:

EUREKA! N28, 2008

41

Sociedade Brasileira de Matemtica

Usando o padro para n par, temos:

n2 = 199 n 2 = 398, mas e equao no tem 2

soluo inteira. Usando o padro para n mpar, vemos que: (n + 1)2 (n + 1)2 1 = 199 = 200, achamos (n + 1) = 20, donde n = 19, portanto 2 2 o nmero de linhas ser igual a 19.
SOLUO DO PROBLEMA 2: CAROLINA RODRIGUES SILVA (FORTALEZA CE)

(2) B A A

(1)

Na figura 1 chamamos a rea de A de x e a de B de 2x. Teremos ento 3x = 81 m2 e x = 27 m2, ento a rea de A = 27 m2 e seus lados so: 3 e 9; rea de B = 54m2 e seus lados 6 e 9. Na figura 2, vemos que se juntarmos as reas descobertas teremos como largura 3 e altura 9 3 = 6. Obtemos assim como rea do piso que ficou descoberta o seguinte valor: 6 3 = 18m2.
SOLUO DO PROBLEMA 3: FELIPE BELLIO DA NBREGA (RIO DE JANEIRO RJ)

x, y , z : nmeros nos cartes vamos supor x< y<z x + y + z = 13

a)

1 + 2 + 10 2+3+8 3+4+6 1+3+9 2+4+7 1+4+8 2+5+6 1+5+7 b) Quando Arnaldo olha, pode-se eliminar o 3 + 4 + 6, pois ele saberia, j que o nico que comea com 3.
EUREKA! N28, 2008

42

Sociedade Brasileira de Matemtica

Quando Bernaldo olha, pode-se eliminar o 1 + 2 + 10, o 1 + 3 + 9 e o 2 + 5 + 6. O primeiro porque o nico que acaba com 10. O segundo com 9. E o ltimo, j que no pode ser o 3 + 4 + 6 graas a Arnaldo o nico que acaba com 6. Quando Cernaldo olha, pode-se eliminar o 1 + 5 + 7 e o 2 + 3 + 8. J que o 2 + 5 + 6 foi eliminado por Bernaldo, o 1 + 5 + 7 o nico com 5 no meio. E j que Bernaldo tambm eliminou o 1 + 3 + 9, o 2 + 3 + 8 o nico com 3 no meio. Resposta: Assim sobraram apenas o 1 + 4 + 8 e o 2 + 4 + 7. Ento o 4 est no carto do meio.
SOLUO DO PROBLEMA 4: RAFAEL KAZUHIRO MIYAZAKI (SO PAULO SP)

a) C1 L1 L2 L3 L4 Soma da coluna 0 5 2 0 7 C2 0 0 4 2 6 C3 0 0 1 2 3 C4 1 0 1 0 2 Soma da linha 1 5 8 4

b) C1 L1 L2 L3 L4 L5 L6 L7 L8 Soma da coluna 0 0 0 3 3 0 1 0 7 C2 0 0 3 0 0 0 0 0 3 C3 0 0 0 2 3 3 2 0 10 C4 1 0 1 0 0 0 0 0 2 C5 0 4 0 0 2 1 3 1 11 C6 0 0 0 0 1 0 3 11 15 C7 0 1 0 3 0 2 0 0 6 C8 0 0 0 0 0 7 7 0 14 Soma da linha 1 5 4 8 9 13 16 12

c) No possvel. Para que seja possvel montar uma tabela, a soma das somas das colunas e das somas das linhas deve ser igual ao dobro da soma dos nmeros internos(nmeros preenchendo a tabela, exceto os de soma). 1 + 2 + 3 +...+ 16 + 17 + 18 = 171
EUREKA! N28, 2008

43

Sociedade Brasileira de Matemtica

171 , onde n a soma dos nmeros internos e estes devem ser 2 171 naturais, mas no natural. Portanto no podemos montar a tabela pedida. 2
2n = 171 n =
SOLUO DO PROBLEMA 5: SOLUO DA BANCA

Observamos inicialmente que 9 5 2 = 9 10 = 90 9 55 22 = 9 1210 = 10890 9 555 222 = 9 123210 = 1108890 9 5555 2222 = 9 12343210 = 111088890 9 55555 22222 = 9 1234543210 = 11110888890 Isso nos leva a conjecturar que 9 A = 9 555...555 1 4 24 3 222...22 1 424 3 = 111...1110888...8890 1 424 3 1 424 3
2007 cincos 2007 dois 2006 uns 2006 oitos

Para mostrar que nossa conjectura verdadeira, devemos garantir que, ao continuar as multiplicaes acima, o padro se repete. Digamos que voc j tenha feito n multiplicaes e tenha obtido Ento 9 555...555 1 4 24 3 222...22 1 424 3 = 111...1110888...8890. 1 424 3 1 424 3
n cincos n dois n -1 uns n -1 oitos

9 555...555 1 4 24 3 222...22 1 424 3 = 9 555...55550 1 4 24 3 + 5 222...220 1 424 3 + 2 = n +1 cincos n +1 dois n cincos n dois 9 555...55550 222...220 + 9 555...55550 2 + 9 5 222...220 1 4 24 3 1 424 3 1 4 24 3 1 424 3 +9 5 2 =
n cincos n dois n cincos n dois

9 555...5555 1 4 24 3 222...2200 1 424 3 + 9 111...111100 1 4 24 3 + 9 111...111100 1 4 24 3 + 90 =


n cincos n dois n uns n uns

111...1110888...889000 + 2 999...9900 1 424 3 1 424 3 1 424 3 + 90 =


n 1 uns n 1 oitos n noves

111...1110888...889000 + 1999...9800 + 90 = 1 424 3 1 424 3 123


n 1 uns n 1 oitos n 1 noves

111...1110888...8890 1 424 3 1 424 3


n uns n oitos

Portanto, nossa conjectura verdadeira. Logo, a soma dos algarismos de 9 A igual a 2006 1 + 2006 8 + 9 = 2006 9 + 9 = 2007 9 = 18063.

EUREKA! N28, 2008

44

Sociedade Brasileira de Matemtica

SOLUES NVEL 2
PROBLEMA 1: SOLUO DE HERMANO HENRIQUE DA SILVA (FORTALEZA CE)

B 30

O 120 80 60 A 20 80 100 P

30 20

Propriedade do circuncentro: Est a igual distncia dos vrtices! Como O o circuncentro, AO = BO = AP, logo APO issceles e como o = 40 OAP = 20, AOP = APO = 80. OP Da, OPC = 100, OCP = 20, POC = 60. BOC isosceles, da OBC = OCB = 30. Logo
BOC = 120,

mas

PROBLEMA 2: Veja a soluo do problema No. 4 do Nvel 1. PROBLEMA 3: SOLUO DA BANCA

Observe a seguinte fatorao


(a 2 )29 1 a 29 + 1 a 29 1 = . a2 1 a +1 a 1

Sabemos que a 29 + 1 = (a + 1)(a 28 a 27 + a 26 ... a + 1) e


a 29 1 = ( a 1)(a 28 + a 27 + ... + a + 1).
29 29

Dessa forma cada uma das fraes

a +1 a 1 e inteira. a +1 a 1 Alm disso, se a for par, pelo lema de Euclides:

mdc(a 29 + 1, a 29 1) = mdc(a 29 + 1, 2) = 1.
EUREKA! N28, 2008

45

Sociedade Brasileira de Matemtica

a 29 + 1 a 29 1 Assim, mdc , = 1. Com isso, podemos concluir que, se a for a + 1 a 1 (a 2 )29 1 maior que 1, possui pelo menos um divisor primo a mais do que a2 1 2007 a 29 1 . Portanto, o nmero a = 32 satisfaz s condies do problema. a 1
PROBLEMA 4: SOLUO DE MATHEUS SECCO TORRES DA SILVA (RIO DE JANEIRO RJ)

3m + 3n + 1 = t 2 .
Sabe-se que todos os nmeros da forma 3k so mpares. Assim, 3m + 3n um nmero par obrigatoriamente. Logo, 3m + 3n + 1 um mpar. Sendo t 2 um nmero mpar, t tambm deve ser mpar, ento podemos escrever t = 2k + 1, onde k inteiro positivo. Voltando equao original, obtemos: 3m + 3n + 1 = 4k 2 + 4k + 1; 3m + 3n = 4k (k + 1). Pelo princpio da Casa dos Pombos, k(k + 1) um nmero par necessariamente, fazendo com que 4k (k + 1) seja mltiplo de 8. Devemos ter ento 3m + 3n 0 (mod8). Porm, 3m 1 ou 3 (mod8) e

3n 1 ou 3 (mod8). Assim, 3m + 3n 0 (mod8) um absurdo!


Por isso, a equao 3m + 3n + 1 = t 2 no tem solues nos inteiros positivos (c.q.d.)
PROBLEMA 5: SOLUO DE DEBORAH BARBOSA ALVES (SO PAULO SP)
A 45 K

M a C 45 a

45

B A

K
EUREKA! N28, 2008

46

Sociedade Brasileira de Matemtica

Girando o AKC em torno de C, at A B, temos o CK A (ou que CAK = CAK = 45, ento MBK reto. Sendo ACK = e MCB = , BCK = ACK = . Como ABC retngulo, e a hipotenusa AB, ACB + + 45 = 90 + = 45. Como KC = K C ; MCK = + = 45 = KCM e como MCK so congruentes (caso LAL) Ento, todos os seus ladas e iguais. Assim, KM = K M . ACK congruente com ACK , por construo. Ento AK = AK = BK . MK B (ou MK A) retngulo. Ento, pelo teorema de Pitgoras, temos: MB 2 + BK 2 = MK 2 e como BK = AK = AK e MK = KM , AK 2 + MB 2 = KM 2 .

CK B ) em

reto. Ento
MCK e ngulos so

PROBLEMA 6: BASEADA NA SOLUO DE JOO MENDES VASCONCELOS (FORTALEZA CE)

a) Se n par, dividimos o tabuleiro em

n2 quadrados 2 2. Em cada jogada, 4 Ludmilson retira um quadrado 2 2 desses em que dividimos o tabuleiro. Nas n2 primeiras jogadas, Ednalva retirou quadrados pertencentes a, no mximo, 8

n2 n2 desses quadrados 2 2. Assim, se k 1 < , no momento de Ludmilson 8 8 n2 n 2 n 2 n 2 fazer a k-sima jogada, foram tocados no mximo k 1 + < + = 8 4 8 8

desses quadrados 2 2, e portanto sobra algum desses quadrados para Ludmilson n2 retirar. Assim, Ludmilson consegue retirar pelo menos desses quadrados, 8
n2 n 2 que contm 4 quadrados 1 1, ficando com pelo menos a metade dos 8 2 quadradinhos do tabuleiro.

EUREKA! N28, 2008

47

Sociedade Brasileira de Matemtica

n2 102 Se n = 10, 4 = 4 13 = 52 > , e Ludmilson de fato ganha o jogo. 2 8

Obs.: x denota o menor inteiro que maior ou igual a x b) Para fazermos o caso geral, dividiremos em casos: Primeiro caso: n par: Como vimos acima, Ludmilson consegue retirar pelo menos metade dos quadradinhos do tabuleiro, e logo Ednalva no consegue ganhar o jogo. Na verdade Ludmilson ganha se n for da forma 4k + 2 e o jogo empata se n for da forma 4k. Segundo caso: n mpar. Ns faremos uma pintura como segue: A cada duas linhas, uma ficar em branco e outra ser pintada em um quadradinho sim e um no. Veja a figura para melhor compreenso:

... ... n

M
... n

Como n mpar, as linhas pintadas tero um quadradinho pintado a menos que os no pintados. Pelo mesmo motivo, o nmero de linhas pintadas ser uma unidade menor que o de no pintadas. Isso garante que o nmero de casas pintadas seja mnimo e ns possamos ter ao mesmo tempo todos os quadrados 2 2 com uma casa pintada. Agora vamos contar o nmero de quadrados pintados: n 1 Em cada linha pintada, ns temos quadrados pintados. 2 n 1 (n 1)2 Como so linhas pintadas, o total de quadradinhos pintados ser . 2 4
EUREKA! N28, 2008

48

Sociedade Brasileira de Matemtica

A estratgia de Ednalva se resume a retirar, a cada jogada, um quadradinho preto at que no reste mais nenhum. Percebemos tambm que a cada jogada de Ludmilson ele tambm retira um quadradinho preto obrigatoriamente, j que todos os quadrados 2 2 do tabuleiro esto pintados em uma casa. (n 1) 2 (n 1) 2 Desse modo, aps jogadas de Ludmilson, e jogadas de 8 8
(n 1)2 (n 1) 2 (n 1)2 Ednalva, so retiradas casas pintadas, ou seja, + = 4 8 8 todas as casas pintadas, e Ludmilson no consegue mais jogar. Como, ao final, ( n 1)2 1 (n 1) 2 ( n 1) 2 n2 quadradinhos Ludmilson tem 4 + = +2< 4 8 2 2 2 8 (pois n 3 nesse caso), Ednalva vence sempre nesse caso.

SOLUES NVEL 3
PROBLEMA 1: BASEADA NA SOLUO DE LEANDRO FARIAS MAIA (FORTALEZA CE)

Sejam f 1 ( x) = f ( x) e para cada n 1, f n +1 ( x) = f ( f n ( x)).


2007 + 1 . 2 Temos f ( x1 ) = 0. Vamos mostrar por induo que existe uma seqncia de reais

Sejam 1 = 2007 2 4, x1 =

positivos ( n ) tal que, definindo xn = todo n , donde f n +1 ( xn +1 ) = f n ( xn ) = 0.

2007 + n , temos f ( xn +1 ) = xn , para 2


2007 + n +1 2

Para isso, note que a maior raiz de x 2 + 2007 x + 1 = xn


n +1 = 2007 2 4 + 4 xn = 2007 2 4018 + 2 n > 0, c.q.d.

, onde

PROBLEMA 2: SOLUO DE RAMON MOREIRA NUNES (FORTALEZA CE)

Vamos provar que todo nmero da forma 8q + 1 resduo quadrtico mdulo 2n (usaremos no que segue a palavra resduo significando resduo quadrtico) e que so os nicos resduos mpares para n maior ou igual a 3. Temos que 1 o nico resduo mpar mdulo 8. De fato, (2k + 1) 2 = 4k (k + 1) + 1 1(mod8), k . Assim, se n 3 ento todo nmero mpar que resduo mdulo 2n congruente a 1 mdulo 8.
EUREKA! N28, 2008

49

Sociedade Brasileira de Matemtica

Mostraremos, por induo que todo nmero da forma 8q + 1 resduo mod 2k, para todo k 3. Caso inicial k = 3: 8q +1 resduo mod 8 porque 1 resduo mod 8. Passo: Todo nmero da forma 8q + 1 resduo mod 2k; tome x dessa forma. Ento, existe y com y 2 x(mod 2k ). Se y 2 x(mod 2k +1 ), acabou. Seno,

y2 x + 2k (mod 2k +1 ), e ( y + 2 k 1 ) = y 2 + 2 k y + 2 2 k 2 y 2 + 2 k x (mod 2 k +1 ),
2

donde x resduo mdulo 2k +1 , e conclumos a demonstrao. Aprendemos a contar os nmeros mpares resduos quadrticos. Como x2 + c 0(mod22007 ) para algum x o mesmo que c x2 (mod22007 ) para algum x, queremos saber o nmero de cs tais que c resduo quadrtico; bem, entre os mpares temos: 2001,..., +2007. Quantos nmeros temos entre eles? Como 2001 = 8(250) 1 e 2007 = 8 251 1, temos 502 mpares. Agora para os pares: claro que c tem que ser mltiplo de 4, pois x2 + c 0(mod22007 ) x2 + c 0(mod4) , que s acontece para c mltiplo de 4. Bem, claro tambm que x deve ser par, ou seja, x = 2y; queremos 4y2 + 4d 0(mod22007 ) y2 + d 0(mod22005 ); novamente, sabemos contar os y mpares. Tnhamos os seguintes mltiplos de 4: 2004, .., 2004; dividindo por 4, ficamos com: 501, 500, ..., 500, 501. Os mpares da forma 8q 1 so 497,..., 495; como 497 = 8 (62) 1 e 495 = 8 (62) 1, temos 125 mpares aqui. Agora, seguindo o algoritmo, pegamos os mltiplos de 4: 500,...,500 e vemos quais deles so simtricos de resduos mod22005. Dividindo por 4, vemos que eles correspondem aos elementos de 125,...,125 que so simtricos de resduos mod 22003. Encontramos agora os nmeros 8q 1 entre esses: Veja que 121 = 8 (15) 1 e 119 = 8(15) 1; so 31 nmeros aqui. Mltiplos de 4: 124, ..., 124, dividindo por 4: 31,..., 31 (mod 22001 agora) 25 = 8 (3) 1 e 31 = 8(4) 1; temos 8 nmeros aqui. Mltiplos de 4: 28, ..., 28, dividindo por 4: 7,..., 7 (mdulo 21999 agora). 1 = 8(0) 1 e 7 = 8(1) 1; temos 2 nmeros aqui. Mltiplos de 4: 4, 0, 4, dividindo: 1, 0, 1 (mod 21997 agora); desses nmeros somente (1) e (0) (1 e 0) so resduos, (1) no , logo temos 2 nmeros aqui. Total: 502 + 125 + 31 + 8 + 2 + 2 = 670 nmeros.

EUREKA! N28, 2008

50

Sociedade Brasileira de Matemtica

PROBLEMA 3: SOLUO DA BANCA

Primeiro considere dois pontos P e Q do polgono cuja distncia mxima. Tome Q de modo que PQ separe o polgono em dois polgonos, um deles com PQ como nica distncia mxima. Em cada um desses dois polgonos vamos aplicar o seguinte Lema: Seja A 1A 2 ...A k um polgono convexo tal que a maior distncia entre dois de seus vrtices, incluindo diagonais, A k . Ento esse polgono tem k 2 1A distncias diferentes; caso A 1A k seja a nica distncia mxima, ento h k 1 distncias diferentes. Demonstrao: Sejam Ap e Aq , 1 < p < q < k dois vrtices do polgono. Vamos provar que, para quaisquer m e n com p < m n < q um dos segmentos Ap An , Aq Am menor do que Ap Aq . Em seguida, conseguiremos uma seqncia de k 2 distncias diferentes. Como conseguir distncias menores? Ou, de modo mais geral, como compara segmentos? Muitas vezes melhor transferir tudo para ngulos, para que possamos fazer...isso mesmo, um arrasto! Sejam = Am A1 Ak ,1 = A1 Am Ak ,2 = Ap Am Aq ,3 = Aq Ap Am , A a interseo de

Ap Aq e A 1A m (note que, como o polgono convexo, A est no interior do


segmento Ap Aq ) e 4 = Am AAq .

An Am Ap 4 A 4 B Aq

A1

Ak

Suponha que Ap Aq Am Aq . Ento, no tringulo Am Ap Aq , 2 3. Alm disso, pelo teorema do ngulo externo no tringulo AAp Am ,3 < 4 . Ademais, 1 < 2 e, sendo
EUREKA! N28, 2008

51

Sociedade Brasileira de Matemtica

A 1A k a maior distncia de todas (e esse o passo decisivo da demonstrao e mostra o poder do princpio do extremo), no tringulo A 1A mA k , < 1. Logo < 1 < 2 3 < 4 < 4 . Definindo os s analogamente e supondo que Ap Aq An Ap , obtemos < 4 .
Porm, observando os quadrilteros A e ABAn Am , temos que 1A kA nA m + +A 1A mA n +A kA nA m = 4 + 4 +AA mA n +BA nA m = 360 + = 4 + 4 . Mas < 4 + < 4 + 4 , < 4 contradio. O caso em que m = n fica a cargo do leitor. Para terminar, basta fazer uma espcie de zigue-zague. Comece com A2 Ak 1, que menor do que A 1A k (por qu?). Pelo que acabamos de provar, A 2A k 2 ou A3 Ak 1 menor do que A2 Ak 1. Suponha, por exemplo, que A3 Ak 1 seja menor. Ento, aplicando o nosso fato de novo, A4 Ak 1 ou A3 Ak 2 menor do que A3 Ak 1. Continuamos assim, at acabar o polgono, e assim conseguimos k 2 distncias diferentes. No caso em que A k a nica distncia mxima, fica para voc provar (use o 1A poder do arrastro novamente!) que, no quadriltero A 1A 2A k 1 A k , uma das diagonais (na verdade as duas) menor do que A 1A k (bem, isso imediato) e maior do que A2 Ak 1 , de modo que ganhamos mais uma distncia, totalizando k 1. Agora, vamos terminar o problema. Lembre que cortamos o polgono original do problema em dois por uma diagonal PQ com medida mxima. Suponha que os polgonos obtidos tenham k + 1 e n k + 1 lados, sendo que o de k + 1 lados tem a distncia mxima nica. Nele, obtemos (k + 1) 1 = k distncias diferentes, e no outro, (n k + 1) 2 = n k 1. Ento conseguimos d = mx{k, n k 1} distncias. Mas d

k + (n k 1) n 1 n = d . 2 2 2

PROBLEMA 4: SOLUO DE HENRIQUE POND DE OLIVEIRA PINTO (SALVADOR BA)

Numeremos as casas do tabuleiro de acordo com o seguinte padro:

EUREKA! N28, 2008

52

Sociedade Brasileira de Matemtica


A C A C A C B D B D B D A C A C A C B D B D B D A C A C A C B D B D B D

fcil ver que cada quadrado 2 2 de Arnaldo ocupa exatamente uma casa de cada tipo (A; B; C e D). Agora uma contagem simples nos mostra a quantidade de casas de cada tipo.
Tipo A B C D Quantidade 10042 1004 1003 1003 1004 10032

Veja que a soma total de fato 20072. Veja que as casas tipo D so as menos numerosas. Agora suponha que Bernaldo s jogue em casas tipo D. Teremos que a cada jogada de cada um dos jogadores exatamente uma casa tipo D ocupada. Assim aps 10032 jogadas Arnaldo no 10032 + 1 poder mais jogar. Como Arnaldo comea, quando ele fizer sua -sima 2 jogada acabaro as casas D ento Bernaldo pode escolher qualquer casa que (10032 + 1) Arnaldo no poder jogar novamente. Ento assim Arnaldo ter 4 2 2 2007 casas. Ele s ganha se pegar mais que ou seja 2 4(10032 + 1) 2007 2 > 20062 + 4 > 2007 2. Absurdo. 2 2 Logo jogando assim Bernaldo ganha independentemente de como Arnaldo jogar.
PROBLEMA 5: SOLUO DE RAFAEL TUPYNAMB DUTRA (BELO HORIZONTE - MG)

Sejam E a interseo de OC e PQ e F a interseo de BD e PQ.


EUREKA! N28, 2008

53

Sociedade Brasileira de Matemtica


C

D O B A F P

Pelo teorema de Ceva aplicado ao tringulo CPQ,

PE CD QB =1. QE PD CB PF CD QB = 1. QF PD CB

Pelo teorema de Menelaus aplicado ao tringulo CPQ, Assim, temos

PE PF e, portanto, P, Q, E, F formam uma qudrupla = QE QF harmnica. Assim sendo, OP, OQ, OE, OF formam um feixe harmnico. Portanto qualquer reta intersecta esse feixe em uma qudrupla harmnica. Vamos criar uma reta r que passa por Q e perpendicular a OQ.
O P
' P

E F Q F
o

Supondo PQ = 90 , provaremos que OQ bissetriz de FE, o que mostra que OQ bissetriz de AB. Analogamente, teremos OP bissetriz de AD (as duas bissetrizes das retas AC e BD so perpendiculares). Como PQ = 90o, temos OP // r. Assim, r intersecta o feixe harmnico na
' ' qudrupla harmnica P , Q, F, E, sendo P o ponto do infinito correspondente ao feixe de retas paralelas a r. Dessa forma, precisamos ter QE = QF, ou seja, Q o ponto mdio de EF. Assim, pelo teorema de Pitgoras,

temos OF = OE = OQ 2 + QE ' 2 e, como o tringulo OEF issceles, a altura OQ tambm bissetriz de EF, de EF e de AB, como queramos demonstrar.
EUREKA! N28, 2008

54

Sociedade Brasileira de Matemtica

Observao: a maioria das solues utilizou trigonometria ou geometria analtica, eventualmente com algumas aplicaes dos teoremas de Menelaus e de Ceva. A demonstrao de Rafael bastante interessante por explorar o potencial da Geometria Projetiva, evitando clculos. Veja a edio 8 da Eureka! para ver a teoria utilizada nesse problema.
PROBLEMA 6: SOLUO DE RGIS PRADO BARBOSA (FORTALEZA - CE)

Seja Ai = {x j xi , i < j n} . Note que, se a b,


x a xb ; assim, Ai = n i .

x a xi xb xi

pois

Considere agora Ak Am com k > m. Se Ak Am 2 , ento existem a, b, c, d distintos tais que x a x k = x b x m x a xb = x k x m e xc x k = x d x m x c x d = x k x m . Assim, x a xb = x c x d = x k x m , ou seja, um real aparece trs vezes como diferena, um absurdo. Logo Ak Am 1 . Vamos contar os reais que aparecem duas vezes do seguinte modo: se ele pertence a Ak e Am , k > m , a contagem registrada na linha de Ak (ou seja, no conjunto de maior ndice). Faamos ento tal contagem, comeando de An e indo at A1 . Isto quer dizer que se o nmero aparece outra vez em outro conjunto, ele o faz em um conjunto de ndice menor. Para n par:
ndice Quantidade de elementos

An = 0

Reais que aparecem duas vezes 0

n 1
M n +1 2 n 2 n 1 2 M
2 1

An1 = 1
M

1
M

n An / 2+1 = 1 2 n An / 2 = 2 n An / 21 = + 1 2

A2 = n 2 A1 = n 1

n 1 2 n 1 2 n 2 2
1
0

EUREKA! N28, 2008

55

Sociedade Brasileira de Matemtica

Justificando a contagem acima: note que h k 1 Ak conjuntos com ndice menor do que k. Como Ak Am 1 para m < k, h no mximo k 1 nmeros que podem se repetidos nos conjuntos de ndice menor; ou seja, a quantidade de novos nmeros de Ak que aparecem duas vezes menor ou igual a k 1; os outros podem aparecer duas vezes, mas eles j foram contados nos conjuntos de ndice maior. Alm disso, a quantidade de nmeros de Ak que aparecem duas vezes menor ou igual quantidade total de elementos de Ak . Logo a quantidade de novos nmeros que aparecem duas vezes no mximo min Ak , k 1 .

Com isso, a quantidade de nmeros que aparecem duas vezes menor ou igual a
n ( 1) n n n 2 2n 2 2 21 + 2 + 3 + L + 1 = 2 = . 2 2 4

A quantidade dos nmeros que aparecem uma vez pode ser obtida tomando o

n(n 1) e subtraindo dele duas vezes a quantidade 2 i =1 de nmeros que aparecem duas vezes. Sendo d1 a quantidade de nmeros que aparecem uma vez e d 2 a quantidade de nmeros que aparecem duas vezes,
total de elementos

Ai

ento d1 =

n(n 1) n(n 1) n 2 2n n n 2d 2 2 = = . 2 2 4 2 2 n 1 n Analogamente, para n mpar, prova-se que d1 = . 2 2

Nota dos editores: Rgis fez o estudo completo do caso n mpar; porm, o procedimento totalmente anlogo e foi decidido no coloc-lo aqui.

EUREKA! N28, 2008

56

Sociedade Brasileira de Matemtica

XXIX OLIMPADA BRASILEIRA DE MATEMTICA


Problemas e Solues da Primeira Fase Nvel Universitrio

PROBLEMA 1:

Joozinho joga repetidamente uma moeda comum e honesta. Quando a moeda d cara ele ganha 1 ponto, quando d coroa ele ganha 2 pontos. Encontre a probabilidade (em funo de n) de que Joozinho em algum momento tenha exatamente n pontos.
PROBLEMA 2:

Dados nmeros reais a1 , a2 ,..., an no todos nulos, encontre o (menor) perodo da funo

f ( x) = ak cos(kx).
k =1

PROBLEMA 3:

Calcule o volume do slido definido pelas desigualdades abaixo:

z 3x2 + 2 y 2 , 3x 2 + 2y 2 + 5 z 2 1
PROBLEMA 4:

Seja a um inteiro no nulo. Prove que se a uma n-sima potncia modulo 4a2, ou seja, existe um inteiro b tal que a bn mltiplo de 4a2, ento a uma n-sima potncia.
PROBLEMA 5:

Calcule os autovalores da matriz (n + 1) (n + 1) abaixo: 0 n 1 0 n 1 M = 2 0 O O 0 1 n 0 Em outras palavras, M i ,i +1 = n + 1 i, M i +1,i = i, M ij = 0 se i j 1. Obs: Os autovalores de M so as razes da seguinte equao em x: det(M xI) = 0.
EUREKA! N28, 2008

57

Sociedade Brasileira de Matemtica

PROBLEMA 6:

Seja y(t) uma funo real de varivel real tal que

y(t ) + et y(t ) + 3ty (t ) = 2sen(t ) + tg(t ), y (0) = 1, y(0) = 0.


2

Calcule o limite:

lim
t 0

ty(t ) . y (t ) 1

Solues Nvel Universitrio


SOLUO DO PROBLEMA 1:

Seja pn a probabilidade pedida. Claramente p0 = 1, p1 =

1 . 2 A probabilidade de que ele nunca tenha n pontos 1 pn . Por outro lado, a

nica forma de nunca ter n pontos completar n 1 pontos e depois tirar coroa. Assim:

1 pn =
donde

pn 1 2

p 2 1 1 2 pn = n 1 = pn 1 e portanto 3 2 3 2 3
n

2 2 1 pn = p0 3 3 2 pn = 2 1 1 + . 3 3 2
n

SOLUO DO PROBLEMA 2:

Seja m = mdc k ak 0 .

2 um perodo de f: afirmamos que este o menor perodo. m n ak k ak k ix f ( x ) = Escreva 2 z + 2 z , z = e k =1


Claramente
EUREKA! N28, 2008

58

Sociedade Brasileira de Matemtica


n a w k k ak w k k f ( x + p) = k z + z , w = eip 2 2 k =1

Duas funes racionais s so iguais (ou iguais para nmeros complexos de mdulo 1) se seus coeficientes forem iguais. Assim, se p um perodo, temos

ak wk = ak para k = 1,..., n. Em outras palavras ou ak = 0 ou


Equivalentemente, p deve ser um mltiplo inteiro de
SOLUO DO PROBLEMA 3:

kp 2

. .

2 . m

Seja A( a ) a rea da elipse 3x + 2 y a.


2 2

Os semieixos da elipse so

a e 2

a a donde A( a ) = . 3 6

O slido do problema pode ser descrito como a unio disjunta de

3 x 2 + 2 y 2 z , 0 z < b, b =
3z 2 + 2 y 2 1 5 z 2 , b z
donde
1 5

21 1 10
1 5
2
b

v = A( z )dz +
b 0
2

6 b 1 1 5 31 2 5 7 21 = b + b3 = + + 300 15 100 6 2 5 3 5 3 6
b

A(1 5 z )dz = 0

dz +

1 5

(1 5z ) dz 6
2

SOLUO DO PROBLEMA 4:
2 Suponha a uma n-sima potncia mod 4a
p Escreva a = 2 2 3 3 ... p ...

Vamos provar que o expoente e p mltiplo de n. Segue da hiptese que a = b p p) = 1.


EUREKA! N28, 2008

ep

n-sima potncia mdulo p

2ep

onde mdc(b,

59

Sociedade Brasileira de Matemtica

Assim existem c, d com mdc(c, p) = 1,

c n p nd b p p (mod p
e

2e p

) donde nd = e p . Assim

uma n-sima

potncia. Falta provar que se n par ento a > 0. Suponha por absurdo o contrrio: n par, a < 0.
nd % % mpar, b % > 0. , b Escreva a = 2 b
nd + 2 : Assim a e a so ambos n-simas potncias mdulo 2

% 2nd (mod 22 nd + 2 ) c n 2nd b

% e b % so b n n nd nd nd + 2 2 nd + 2 % % % % c 2 b 2 (mod 2 c b(mod 2 ) )
quadrados mdulo 4 1 quadrado mdulo 4 Absurdo!
SOLUO DO PROBLEMA 5:

% (mod 2nd + 2 ) c n b

Os autovalores so

n, n 2, n 4,..., n + 2, n, ou seja, 2k n para


t

k = 0,1,..., n.
Vamos exibir os autovetores de M . Interprete o vetor

( a0 , a1 ,..., an ) n +1

como

polinmio

P = a0 x n + a1 x n 1 y + ... + an y n .
t O polinmio correspondente a M ( a0 ,..., an ) y

p p + x . Se expandirmos x y

os polinmios em

u = x+ y

v = x y este operador passa a ser

p p v . u v v ( u k v n k ) = ( 2k n ) u k v n k . Mas u v u u
Assim este o autovetor associado ao autovalor (2k n).
SOLUO DO PROBLEMA 6:
t Expanda as funes y, e e 2 sen(t ) + tg (t ) em series de potncias:
2

y (t ) = a0 + a1t + a2 t 2 + ...

EUREKA! N28, 2008

60

Sociedade Brasileira de Matemtica

et = b0 + b1t + b2 t 2 + ...
2

2 sent + tgt = c0 + c1t + c2 t 2 + ...


Temos lim t 0

a t + 2a2 t 2 + ... + nan t n + ... ty = lim 1 y 1 t 0 a1t + a2 t 2 + ... + an t n + ...

donde este limite igual a N se aN 0 e am = 0 para 0 < m < N . Substituindo as sries de potncias na EDO:

2a2 +6a3t +... +(n+1)(n+2)an+2tn +...


2 b0a2 )t +... +(bna1 +2bn1a2 +... +kb0+1k ak +... +(n +1)b0an+1)tn +3a0t +3at +b0a1 +(ba 1 1 +2 1 +... + 2 n +3an1tn +... = c0 +ct 1 + c2t +... + cnt +... donde

an + 2 =

1 ( cn bn a1 2bn 1a2 ... (n + 1)b0 an +1 3an 1 ) (n + 1)(n + 2)

Segue facilmente que a1 = a2 = a3 = 0. Se

a1 = a2 = ... = an +1 = 0 cn = 0 temos an + 2 = 0 e e se a1 = a2 = ... = an +1 = 0 e cn 0 temos an + 2 0. Devemos portanto procurar tal que c 0 e c = 0 para 1 < m < N . N
N

t t5 + ... Temos sent = t + 6 120 t 3 2t 5 tan t = t + + + ... 3 15 3t 5 2 sent + tan t = 3t + + ... 20 ty = 5 donde lim =7. Assim N t 0 y 1

EUREKA! N28, 2008

61

Sociedade Brasileira de Matemtica

XXIX OLIMPADA BRASILEIRA DE MATEMTICA Problemas e Solues da Segunda Fase Nvel Universitrio PRIMEIRO DIA
Considere a funo de R em R dada por f(x) = ax2 + bx + c, com a, b, c e ac < 0. Prove que, para todo n inteiro positivo, a equao f ( f (K ( f ( x)) K)) = 0 tem 1 4 24 3
n vezes

PROBLEMA 1:

pelo menos uma soluo real.


PROBLEMA 2:

Dado um inteiro positivo n, mostre que existe um inteiro positivo N com a seguinte propriedade: se A um subconjunto de {1,2, K , N } com pelo menos N/2 elementos, ento existe um inteiro positivo m N n tal que k A {m + 1, m + 2, K , m + k} 2 para todo k = 1, 2, , n.
PROBLEMA 3:

Considere o conjunto Pn dos polinmios mnicos de grau n > 0 e coeficientes complexos p( x) = x n + a n1 x n1 + L + a 0 satisfazendo
a0
2

+ a1

+ L + a n 1

=1.

Para p( x) Pn , seja r ( p ( x) ) o mximo entre os mdulos das razes de p(x) e s (n) = sup r ( p ( x) ) .
p ( x )Pn

Determine lim s (n) .


n

SEGUNDO DIA
PROBLEMA 4:

Seja f : uma funo contnua tal que f ( f ( x)) = e x para todo x . Prove que, para todo n inteiro positivo,
EUREKA! N28, 2008

62

Sociedade Brasileira de Matemtica

x +

lim

f ( x) = + . xn

PROBLEMA 5:

Seja A uma matriz real quadrada simtrica de ordem n, e 1 2 n seus autovalores (contados com multiplicidade). Determine, em funo de 1 , 2 , , n : a) O nmero de matrizes reais B simtricas de ordem n tais que B 2 = A . b) O nmero de matrizes reais B de ordem n tais que B 2 = A .
PROBLEMA 6:

Para a, b , definimos o conjunto

S (a , b) = {ax 2 + by 2 | x, y }

dos nmeros racionais que podem ser escritos na forma ax 2 + by 2 com x, y . Dados a, b, c, d racionais no nulos, mostre que S(a,b) = S(c,d) se, e ab o quadrado de um racional e existe um racional no nulo somente se, cd q S ( a , b ) S (c , d ) ,

SOLUES
PROBLEMA 1: SOLUO DE MAURCIO RODRIGUES COLLARES NETO (ARACAJU SE)

1 ) Provemos a afirmao por induo. Para n = 1 o discriminante da equao do segundo grau b 2 4ac positivo (pois ac < 0) e, portanto, a equao possui raiz real. Suponhamos agora a afirmao vlida para n. Para provar a afirmao para n + 1, vejamos que f ( f (...( f ( x))...)) um polinmio cujo coeficiente do termo lder

1 4 24 3
n +1 vezes

uma potncia mpar de a (para n = 1 isto verdade; se isto vlido para n, temos que f (...( f ( x))) = f (a 2r +1 x k + ...) = a(a 2 r +1 x k + ...)2 + b(a 2 r +1 x k + ...) + c

123
n +1 vezes

que possui termo lder igual a x 2 k com coeficiente a 4 r +3 ). Assim, se

a < 0(resp. a > 0), lim f ( f (...( f ( x)) = (resp. + ) (basta colocar o termo 4 24 3 x + 1
n +1vezes

lder em evidncia e verificar que os outros termos vo a zero). Mas


EUREKA! N28, 2008

63

Sociedade Brasileira de Matemtica

f ( f (...( f ( x))...)) 1 4 24 3
n vezes n +1 vezes

tem

uma

raiz

r,

por

hiptese,

f ( f (...( f (r ))...)) = f (0) = c > 0 (resp. < 0). (Pois ac < 0). Assim, pelo 1 4 24 3
Teorema do Valor Intermedirio, como a funo passa de um valor positivo (resp. negativo) para um valor negativo (resp. positivo) e contnua, ela tem raiz real:
PROBLEMA 2: SOLUO ADAPTADA DA SOLUO DE RAFAEL DAIGO HIRAMA (S.J. DOS CAMPOS SP)

Fixe N. Para cada 1 i N , defina ai = A {1,..., i} . Temos

A {m + 1,..., m + k} = am + k am
Isso nos induz a definir bi = ai

k m+k m am + k am 0. 2 2 2

i , e portanto a desigualdade anterior equivale a 2

bm + k bm . Assim, queremos mostrar que, se N suficientemente grande, ento existe m N n tal que bm bm +1 ,..., bm + n . (I)
claro que

1 i +1 i +1 i +1 1 = ai ai +1 (ai + 1) = bi + , ou seja: 2 2 2 2 2 1 1 bi +1 bi , bi , bi + . 2 2 Tome N > n(n + 2) e suponha que (I) no ocorra para cada m N n. Em 1 1 particular, para m = 1, existe i1 {2,..., n + 1} tal que bi1 < b1 bi1 b1 < . 2 2 Por induo, construmos uma seqncia i1 ,..., in +1 de ndices tal que i j +1 {i j + 1,..., i j + n} e bi j +1 < bi j . Podemos fazer isso pois N n > n(n + 1). bi
Da, bin+1 < bin < ... < bi1 , e, como bi j +1 < bi j bi j+1 bij temos bin+1 bi1 (n + 1)

1 , para todo j n, 2

n 1 1 1 (n + 1) = . 2 2 2 2

EUREKA! N28, 2008

64

Sociedade Brasileira de Matemtica

Assim, bin+1 .

n 2

N N , donde bN = aN 0, e a 2 2 desigualdade anterior garante que in +1 N n. Assim, se bm = min{b1 ,..., bN }, n n 1 e b j +1 b j para todo j), o que garante ento bm . (pois bN bm 2 2 2 que m N n e bm bm +1 ,..., bm + n .
Como

A {1,..., N } = A, aN = A

PROBLEMA 3: SOLUO DE FBIO DIAS MOREIRA (RIO DE JANEIRO RJ)

Por Cauchy- Schwarz, temos que

n 1 ai i =0

n 1 2i n 1 i p ai p . i =0 i =0
2

2n 1 Chame = p ; ento 2 1
como

a p
i =0 i

n 1

2 i

. Se p raiz de p(x) e p > 1, ento,

ai pi =pn , temos
i=0

n1

2n 1 2n 2n 1 2n+2 2n 2n (2 2 ) 1, 2 1

logo, como 0, < 2. Por outro lado, se 2 = 1 + (0 < < 1), a desigualdade de Bernoulli diz que

1 , suficiente 1 n 1 que (1 + n )(1 ) 1 1 + ( n 1) n 2 1 . n

2 n 1 + n , e portanto, para que 2 n (2 2 ) 1 2 n

n 1 2 1 Chame = 1 + e k= ; defina p(x) tal que ai = k i , para 2n n 1 0 i n 1.


fcil
n 1 i=0

verificar

que

ai

= 1.
j

Ademais, que

p ( ) = n k 2 i = n k
EUREKA! N28, 2008

2n 1 1 = n 0, 2 1 k
65

Sociedade Brasileira de Matemtica

1 2n 1 2n 0 2 2 n (2 2 ) 1, o que verdadeiro pela 1 k n 1 definio de (temos 2 = 1 + , com = ). Logo p ( x ) possui uma raiz n maior ou igual a (j que lim p ( x ) = +).
n
x +

n 1 n 1 lim inf s (n) lim 1 + = 2. Mas < 2 n n n n implica s (n) 2 lim sup s( n) 2 e, finalmente, lim s (n) = 2.
Assim, s (n) 1 +
n
n +

PROBLEMA 4: SOLUO DE FBIO DIAS MOREIRA (RIO DE JANEIRO RJ)

Note inicialmente que f ( f ( f ( x)) = e f ( x ) = f (e x ). Fazendo as substituies


lim f ( y ) ny f ( x) e f ( y) x = e , y = e , z = e , temos lim n = lim ny = e y . x x y e f ( z ) z lim ( f ( y ) ny ) = lim ( e f ( z ) ne z ) = lim e z n e . y z w

Logo basta provar que lim ( f ( z ) z ) = +.


z

Mas

f ( w) w lim ( f ( z ) z ) = lim e w 1 ou seja, e , z w f ( w) w + c para c > 0 fixo e todo w suficientemente grande.

basta

que

Como f ( x) = f ( y ) f ( f ( x)) = f ( f ( y )) e x = e y x = y, f injetora e, em virtude de ser contnua, montona. Se f fosse decrescente, como ]0, +[ Im( f ), teramos lim f ( x) = + mas,
x

por outro lado, lim e = lim f ( f ( x)) = 0, logo lim f ( x) = 0. Isso implica
x

Im( f ) = ]0, +[ , mas ento


absurdo! Logo f crescente. Se lim f ( x) = ,
x x

f ( f ()) = ]0, +[ = f lim f ( f ( x)) =

x +

( ]0, +[ ) = ]0, f (0)[ ,


x

ento

mas

lim e x = 0, logo

lim f ( x) = A, onde A < 0 uma constante (veja que se A 0 ento

f ( f ()) = f (]A; +[) =] f ( A); +[ f ( A) = 0 f ( A) = 0 < eA = f ( f ( A)) = f (0),


absurdo).
EUREKA! N28, 2008

66

Sociedade Brasileira de Matemtica

Defina

I 0 = ], A]

I n +1 = f ( I n );

assim

I1 = ] A, 0] ,

A A I2 = 0, e , I3 = e ,1 ... . fcil ver que os I i s formam uma partio de , e logo f ( x) > x, x .

Considere g ( x) = f ( x) x. No intervalo ], 2 A[ , g ( x) > A 2 A = A . No intervalo compacto

Com isso provamos que f ( x ) x + k para x I 0 I 1 I 2 I 3 ; vamos provar por induo que isso vale para x I 2 k I 2 k + 1 para todo k 2 : de fato,

g : ],1] assume um mnimo positivo, digamos k.

]2 A,1[ ,

a funo g assume um mnimo positivo. Logo

x = f ( f ( y )) = e y

para

y I 2 k 2 I 2 k 1

portanto

f ( x ) = f ( f ( f ( y ))) = e f ( y ) e y + k = e y ek > e y (1 + k ) > e y + k , j y > 0 e y > 1. que Mas e y = f ( f ( y )) = x e portanto f ( x) > x + k , concluindo a demonstrao.
PROBLEMA 5: SOLUO DA BANCA

V = {v | Av = v} invariante por B. Suponha v V . Afirmamos que Bv V . A( Bv) = B3v = B( Av) = B( v) = Bv.

Primeiro afirmamos que o autoespao V de A associado a um autovalor

Assim devemos em cada tal autoespao definir B. Note que a definio de B em cada autoespao independente. (a) Se existir algum autovalor negativo ento no existe B pois seus autovalores deveriam satisfazer 2 = < 0 , o que contradiz a hiptese de B ser simtrica. Se existir autovalor zero ( = 0) com multiplicidade k ento B restrita a V deve ser igual a 0 pois B diagonalizvel e todos os seus autovalores so iguais a 0. Se existir autovalor positivo ( > 0) com multiplicidade 1 ento h duas possibilidades correspondentes a = e = . Se existir autovalor positivo ( > 0) com multiplicidade maior que 1 ento h infinitas possibilidades pois podemos escolher de infinitas maneiras subespaos complementares para corresponderem a Assim, o nmero pedido :

e .

EUREKA! N28, 2008

67

Sociedade Brasileira de Matemtica

0, se algum for negativo. k 2 , se nenhum for negativo, se todo positivo for simples e se houver k autovalores positivos. , se nenhum for negativo e houver pelo menos um autovalor positivo com multiplicidade maior que 1.

(b) Os casos > 0 so como no item (a). Se existir autovalor zero com multiplicidade 1 ento ainda h apenas uma opo. Por outro lado, se existir autovalor zero com multiplicidade maior do que 1 ento h infinitas possibilidades para B (pois h infinitas matrizes B com B 2 = 0) . Se existir autovalor negativo com multiplicidade par (2k) ento h infinitas solues: basta tomar B = basta definir J em

J , J 2 = I . H infinitas tais matrizes J pois


uma base

w1 , w2 ,..., wk , wk +1 ,..., w2 k

por

Jw j = wk + j , Jwk + j = w j , j = 1,..., k .
Note que neste caso B tem autovalores

i e i com multiplicidades k e k.

Se existir autovalor negativo com multiplicidade mpar ento impossvel pois no h como os autovalores de B virem aos pares conjugados. Assim, o nmero pedido :
0, se existe autovalor negativo com multiplicidade mpar, k 2 , se no existe autovalor negativo, todos os autovalores so simples, e h k autovalores positivos. , se todos os autovalores negativos tm multiplicidade par e existe pelo menos um autovalor com multiplicidade maior do que 1.
PROBLEMA 6: SOLUO DA BANCA

ax 2 + by 2 pode ser visto como x a + y b

)( x

a y b , o que

funciona como uma espcie de "norma" de x a + y b . Vamos usar o fato de que o produto ou a razo de dois nmeros dessa forma so da forma z + w ab , e assim a raiz quadrada que aparece s depende do produto ab.
EUREKA! N28, 2008

68

Sociedade Brasileira de Matemtica

ab % z + dw2 , onde c % = cr 2 , donde Se = r 2 , com r * , cz 2 + dw2 = c cd r % = r 2 cd = ab e S (c % , d ) = S (c , d ). Assim, se ab o quadrado de um cd cd


racional, podemos supor sem perda de generalidade que ab = cd. Se ax 2 + by 2 = tz 2 +

ab ab 2 quadrado, podemos w no nulo (se cd t

juntar esse quadrado com uma varivel e supor ab = cd), e queremos provar que qualquer nmero da forma tu 2 + escrevemos

ab 2 v t

tambm da forma ar 2 + bs 2 ,

2 ab 2 2 ab 2 ab tz + w tu + v ( ax 2 + by 2 ) tu 2 + v 2 ab t t t tu 2 + v 2 = = . 2 ab 2 2 ab 2 t tz + t w tz + t w Abusando de notao, isso


Acontece que

( ax

ab ab + by2 ) tu2 + v2 N x a + y b N u t + v t t = . 2 ab 2 ab N z t + w tz + t w t

ab u t + v uzt + vwab + (vz uw) ab t = t , que, multiplicado por ab 2 2 ab tz + w z t + w t t x a + y b , d um nmero da forma k a + l b , a saber

EUREKA! N28, 2008

69

Sociedade Brasileira de Matemtica

vwab vwab x uzt + t by ( vz uw ) a + y uzt + t + ax ( vz uw ) b , 2 ab 2 tz + t w 2 2 cuja "norma" ak + bl , ou seja,


+b 2 2 2 ab 2 2 ab 2 + + tz w tz w t t ab 2 igual a tu 2 + v , como queramos (isso pode ser verificado diretamente, t a
mas chutar essa ltima expresso seria um pouco de sorte...). (Note que a condio de que h um valor comum no nulo das formas ax 2 + by 2 e cu 2 + dv 2 importante. No o caso, por exemplo, se a = 2, b = 3, c = 6 e d = 1). Vamos agora mostrar a outra implicao. Queremos provar que se

vwab x uzt + t by ( vz uw )

vwab y uzt + t + ax ( vz uw )

quadrado de um racional ento as imagens de ax 2 + by 2 (quando x e y percorrem os racionais) so diferentes. A imagem de ax 2 + by 2 com x e y racionais no muda se multiplicarmos a ou b pelo quadrado de um racional no nulo. Assim, podemos supor que em ax 2 + by 2 e

cd no o ab e de cx 2 + dy 2

cx 2 + dy 2 temos a, b, c, d inteiros livres de quadrados. claro que, se os sinais


de ab e cd forem diferentes, as imagens no podem ser iguais. Assim, nos casos

ab > 0. Basta ver ento que cada primo p divide um nmero par cd ab de nmeros dentre a, b, c, d para concluir que o quadrado de um racional. cd
interessantes, Se p divide exatamente 3 deles, digamos a, b e c, ou seja, a = pk, b = pl, c = pm,
2 2 y temos que ax + by = p(kx + ly ) e cx + dy = pmx + dy = p mx + pd p
2 2 2 2

EUREKA! N28, 2008

70

Sociedade Brasileira de Matemtica

tm a mesma imagem se e s se kx 2 + ly 2 e mz 2 + pdw2 tm a mesma imagem, e agora p divide exatamente um nmero dentre k, l, m e pd. Assim, reduzimos o problema a provar que, se um primo p divide exatamente um dentre os nmeros a, b, c, d, digamos a, ento ax 2 + by 2 e cx 2 + dy 2 no tm a mesma imagem. Suponhamos por absurdo que tenham. Notemos primeiro que para quaisquer u e v inteiros, existiriam x e y racionais com ax 2 + by 2 = cu 2 + dv 2 . Como p | a e a livre de quadrados, a maior potncia de p que divide ax 2 mpar e a maior potncia de p que divide by 2 par, donde x e y no podem ter p no denominador, seno a maior potncia de p que dividiria ax 2 + by 2 seria negativa e logo ax 2 + by 2 = cu 2 + dv 2 no poderia ser inteiro. Assim, x e y podem ser vistos como inteiros mdulo p, e cu 2 + dv 2 by 2 (mod p), donde bcu 2 + bdv 2 (by ) 2 (mod p) quadrado mod p para quaisquer u, v inteiros. Fazendo u = 1, v = 0 temos que bc quadrado mod p. Fazendo v = 1, temos que bcu 2 + bd quadrado mod p para todo u inteiro, donde, como bc quadrado mod p, u2 + d/c quadrado mod p para todo u inteiro (note que b, c e d so invertveis mod p). Ou seja, se r quadrado mod p ento r +

d kd tambm , mas isso implica por induo que r + c c

quadrado mod p para todo k natural, donde todo inteiro quadrado mod p. Isso s possvel se p = 2. Se p = 2 dividir um nmero par de nmeros dentre a, b, c, d teremos que todo primo p divide um nmero par de nmeros dentre a, b, c, d, como queramos. Caso contrrio, teremos ainda algum trabalho extra, que realizaremos a seguir. Podemos supor como antes que 2 divide a mas no divide bcd. Sejam r = mdc(c, d ) e K o produto dos primos mpares que dividem ab mas no

c d e n = . Se n (que, como d, mpar) for r r congruente a 3 ou 3 mdulo 8, tomaremos R = (4rK ) 2 c + d , e se n for


dividem cd. Sejam m = congruente a 1 ou 1 mdulo 8, tomaremos R = (2rK ) 2 c + d . Temos em qualquer caso que R pertence imagem de cx 2 + dy 2 . Alm disso,

R r

congruente a 3 ou 3 mdulo 8 e primo com m, n, r e K, e portanto primo com a, b, c e d.

R tem que ter algum fator primo q congruente a 3 ou 3 mdulo 8 r

que aparece com expoente mpar em sua fatorao (pois um produto de nmeros
EUREKA! N28, 2008

71

Sociedade Brasileira de Matemtica

que so 1 ou 1 mdulo 8 ainda dessa forma). Temos que R um nmero da forma c x 2 + d com x inteiro, e portanto c x 2 + d = R = 0(mod q ), donde cd = (cx) 2 (mod q ). Por outro lado, se as imagens so iguais, existem u e v racionais com au 2 + bv 2 = R. Podemos escrever u =

U V e v = D D

onde D o menor denominador comum de u e v. Temos ento aU 2 + bV 2 = R D 2 = 0(mod q). Se q j a maior potncia de q que divide U e V simultaneamente, escrevemos U = q j T e V = q j S obtendo ento

aT 2 + bS 2 = R q 2 j D 2 , que ainda mltiplo de q. Como q

R , q primo r

com a e b, e logo q no pode dividir T, caso contrrio q dividiria bS2, donde q dividiria tambm S, contradizendo a escolha de j. Assim, 2 2 ab T = (bS ) (mod q ) implica que ab um quadrado mdulo q. Portanto,

ab ( ab) = cd (cd )

tambm um quadrado mdulo q, mas, pelas

consideraes anteriores,

ab = 2 w2 , para algum racional w, e da seguiria cd

que 2 quadrado mdulo q, o que um absurdo, pois q congruente a 3 ou 3 mdulo 8 (veja o artigo Reciprocidade quadrtica, de Carlos Gustavo Moreira e Nicolau Saldanha, na Eureka! No. 15). Errata: Na Eureka! No. 27, no artigo Substituies envolvendo nmeros complexos, de Diego Veloso Uchoa, na pgina 21, o trecho entre as linhas 12 e 15 deveria ser: Fazendo n = 2m + 1 e igualando as partes imaginrias, temos: 2m + 1 sen((2m + 1)t ) 2m + 1 2 m 2 m 1 m = (cos t ) (cos t ) + ... + ( 1) . (*) 2 m +1 sen t 1 3 Agora podemos tratar essa igualdade por meio do polinmio 2m + 1 m 2m + 1 m 1 m Pm ( x) = x x + ... + (1) . 1 3

EUREKA! N28, 2008

72

Sociedade Brasileira de Matemtica

XXIX Olimpada Brasileira de Matemtica


Nvel 1 (5. e 6. Sries)
Nome Rafael Kazuhiro Miyazaki Guilherme Renato Martins Unzer Marina Pessoa Mota Ivan Tadeu Ferreira Antunes Filho Danilo Hikari Motoyama Watanabe Arthur Oenning Fagundes Breno Levi Correa Thoms Rincon Reis Lucas Finger Roman Lucas Nishida Ana Cristina Barreto Sabino de Arajo Gabriel Santa Rosa Cavaresi Victor Kioshi Higa Ana Beatrice Bonganha Zanon Maria Paula Silva Serrasqueiro Dbora Barreto Ornellas Igor Arajo Pedro Ivo Coelho de Arajo Ramon Silva de Lima Dnnis Dantas de Souza Nathlia Roscoe e Firace Renan Fernandes Moreira Nicolas Seoane Miquelin Nicolas Fernandez Leito Tiago Sueda Limone Gabriel Pacianotto Gouveia Jonathan Henrique de Oliveira Murilo Dria Guimares Julio Barros de Paula Cesar Nobuo Moniwa Ishiuchi Danilo Kenji Shido Francisco Markan Nobre de Souza Filho Natlia Rodrigues Parrode Joo Felipe Ribeiro Soares Sofia Sayuri Yamamura Paula Dias Garcia Lara Timb Arajo Nathalia Novello Fernandes Ribeiro Eric Luiz Rodrigues de Frana Pedro Ducci Serafim Lucas Bitran Giestas Wederson Santos Silva Ayrton Barros de Lira Leonardo Kazunori Tsuji Lucas Guedes de Almeida Rocha Liang Wei Dong Rafael Wingester Ribeiro de Oliveira Hugo Diehl de Souza Matheus de Oliveira Leo Rodolfo Vieira Fontenele Henrique Gasparini Fiza do Nascimento Victor Venturi Gabrielle Macanhan Guimares Reinaldo Abad Junior Henrique Vieira G. Vaz Gabriela Loiola Vilar Igor Tetsuo Boninsenha Kunizaki Marcelo Cargnelutti Rossato Arthur Ferreira do Nascimento Liara Guinsberg Filipe Bellio da Nbrega Israel Rodrigues Soares Matheus Carneiro Campagnani Cidade Estado So Paulo SP So Paulo SP Fortaleza CE Lins SP So Paulo SP Palmas TO Campo Belo MG Belo Horizonte MG Florianpolis SC Pedreira SP Itapissuma PE Birigi SP So Paulo SP Santo Andr SP Braslia DF Salvador BA Rio de Janeiro RJ Caucaia CE So Paulo SP Campina Grande PB Belo Horizonte MG Taubat SP Mau SP Florianpolis SC Jundia SP So Paulo SP Cordeirpolis SP So Paulo SP Taubat SP Campinas SP So Paulo SP Fortaleza CE Goinia GO Braslia DF Araatuba SP Braslia DF Fortaleza CE Rio de Janeiro RJ Recife PE Campinas SP Vitria ES Massaranduba PB Recife PE So Paulo SP Macei AL Salvador BA Belo Horizonte MG Cricima SC Teresina PI Cocal dos Alves PI Braslia DF Campinas SP Anpolis GO Guarulhos SP So Paulo SP Fortaleza CE Taubat SP Santa Maria RS So Paulo SP So Paulo SP Rio de Janeiro RJ Goinia GO Niteri RJ Prmio Ouro Ouro Ouro Ouro Ouro Prata Prata Prata Prata Prata Prata Prata Prata Prata Prata Bronze Bronze Bronze Bronze Bronze Bronze Bronze Bronze Bronze Bronze Bronze Bronze Bronze Bronze Bronze Meno Honrosa Meno Honrosa Meno Honrosa Meno Honrosa Meno Honrosa Meno Honrosa Meno Honrosa Meno Honrosa Meno Honrosa Meno Honrosa Meno Honrosa Meno Honrosa Meno Honrosa Meno Honrosa Meno Honrosa Meno Honrosa Meno Honrosa Meno Honrosa Meno Honrosa Meno Honrosa Meno Honrosa Meno Honrosa Meno Honrosa Meno Honrosa Meno Honrosa Meno Honrosa Meno Honrosa Meno Honrosa Meno Honrosa Meno Honrosa Meno Honrosa Meno Honrosa Meno Honrosa

EUREKA! N28, 2008

73

Sociedade Brasileira de Matemtica

Nvel 2 (7a. e 8a. Sries)


Nome Joo Mendes Vasconcelos Matheus Barros de Paula Gabriel Milito Vinhas Lopes Thiago Saksanian Hallak Paulo Henrique Dias Vieira Joo Lucas Camelo S Ana Beatriz Prudncio de Almeida Rebouas Hanon Guy Lima Rossi Danilo Silva de Albuquerque Felipe Vieira de Paula Leonardo Ferreira Patrcio Deborah Barbosa Alves Vinicius Cipriano Klein Fernando Fonseca Andrade Oliveira Maria Clara Mendes Silva Ruan Alves Pires Natan Lima Viana Gleycianne Arruda de Freitas Silva Matheus Secco Torres da Silva Felipe Mostavenco Carmo Jonas Rocha Lima Amaro Gustavo Lisba Empinotti Guilherme da Rocha Dahrug Victorio Takahashi Chu Itamar Sales de Oliveira Filho Francisco Vagner Dantas Leite Filho Kayo de Frana Gurgel Rodrigo Rolim Mendes de Alencar Igor Rosiello Zenker Daniel Lucas Filgueira Mario Valney Pereira de Andrades Rafael Dias da Fonseca Matheus Cordeiro Wilhelm da Costa Elder Massahiro Yoshida Lucas de Freitas Smaira Lo Nunes Benevides Felipe Bento Vargas de Moraes Rubens Cainan Sabia Monteiro Alessandro Macdo de Arajo Sandoel de Brito Vieira Rafael Ferreira Antonioli Leonardo Victor Maciel Pontes Bryan Levy Salinas Carrillo Filipe Jos Oliveira Sabia Dbora Jun Portugheis Kelve Torres Henrique Nicols Francisco E. C. Hespanhol Santos Andr Austregesilo Scussel lvaro Lopes Pedroso Wellington Biing Jung Lee Bruno Csar da Silva Guedes Luiz Filipe Martins Ramos Jssica Kazumi Okuma Pedro Vieira Rodrigues Serradas Leonardo Henrique Caldeira Pires Ferrari Carlos Henrique de Andrade Silva Gregory Cosac Daher Cidade Estado Fortaleza CE Taubat SP Fortaleza CE So Paulo SP Rio de Janeiro RJ Fortaleza CE Fortaleza CE So Paulo SP Fortaleza CE Fortaleza CE Rio de Janeiro RJ So Paulo SP Venda Nova do Imigrante ES Belo Horizonte MG Pirajuba MG Rio de Janeiro RJ Fortaleza CE Fortaleza CE Rio de Janeiro RJ Rio de Janeiro RJ Fortaleza CE Florianpolis SC Santo Andr SP So Paulo SP Cedro CE Fortaleza CE Fortaleza CE Fortaleza CE So Paulo SP Fortaleza CE Fortaleza CE Macei AL Rio de Janeiro RJ So Paulo SP Guaxup MG Fortaleza CE Rio de Janeiro RJ Fortaleza CE Fortaleza CE Cocal dos Alves PI S. B. do Campo SP Fortaleza CE So Paulo SP Fortaleza CE Campinas SP Recife PE Bauru SP Fortaleza CE Santa Isabel SP So Paulo SP Recife PE Niteri RJ So Paulo SP Rio de Janeiro RJ Rio de Janeiro RJ Fortaleza CE Rio de Janeiro RJ Prmio Ouro Ouro Ouro Ouro Ouro Prata Prata Prata Prata Prata Prata Prata Prata Prata Prata Bronze Bronze Bronze Bronze Bronze Bronze Bronze Bronze Bronze Bronze Bronze Bronze Bronze Bronze Bronze Bronze Bronze Meno Honrosa Meno Honrosa Meno Honrosa Meno Honrosa Meno Honrosa Meno Honrosa Meno Honrosa Meno Honrosa Meno Honrosa Meno Honrosa Meno Honrosa Meno Honrosa Meno Honrosa Meno Honrosa Meno Honrosa Meno Honrosa Meno Honrosa Meno Honrosa Meno Honrosa Meno Honrosa Meno Honrosa Meno Honrosa Meno Honrosa Meno Honrosa Meno Honrosa

EUREKA! N28, 2008

74

Sociedade Brasileira de Matemtica

Nvel 3 (Ensino Mdio)


Nome Rafael Tupynamb Dutra Rgis Prado Barbosa Ramon Moreira Nunes Henrique Pond de Oliveira Pinto Henrique Hiroshi Motoyama Watanabe Adenilson Arcanjo de Moura Junior Renan Henrique Finder Guilherme Philippe Figueiredo Marco Antonio Lopes Pedroso Rafael Sampaio de Rezende Giuliano Pezzolo Giacaglia Jorge Henrique Craveiro de Andrade Marcelo Matheus Gauy Mateus Oliveira de Figueiredo Paulo Srgio de Castro Moreira Robrio Soares Nunes Marlen Lincoln da Silva Esdras Muniz Mota Grazielly Muniz da Cunha Davi Lopes Alves de Medeiros Gabriel Lus Mello Dalalio Jos Airton Colho Lima Filho Leandro Farias Maia Marcos Victor Pereira Vieira Alfredo Roque de Oliveira Freire Filho Francisco Osman Pontes Neto Leonel Lopes Lima Neto Renan Braz Parente Alex Atsushi Takeda Marcelo Tadeu de S Oliveira Sales Thiago Ribeiro Ramos Luiz Paulo Freire Moreira Antnio Felipe Cavalcante Carvalho Luca Mattos Mller Hugo Fonseca Arajo Fernando Nascimento Coelho Filipe de Almeida Araujo Vital Illan Feiman Halpern Alexandre Nobuo Kunieda Alysson Espndola de S Silveira Thiago S. Pinheiro Orlando Alencar Lustosa Neto Ricardo Turolla Bortolotti Gustavo Pacianotto Gouveia Felipe Holanda Moreira Artur de Almeida Losnak Rafael Parpinel Cavina Filipe Alves Tom Custodio Moreira Brasileiro Silva Marlia Valeska Costa Medeiros Pollyanna Stfani Borges Freitas Gustavo Sampaio Sousa Joas Elias dos Santos Rocha Raphael Luiz Frana Greco Rafael Morioka Oda Cidade Estado Belo Horizonte MG Fortaleza CE Fortaleza CE Salvador BA So Paulo SP Fortaleza CE So Paulo SP So Paulo SP Santa Isabel SP Fortaleza CE Santo Andr SP Rio de Janeiro RJ S.J. do Rio Preto SP Fortaleza CE Fortaleza CE Ribeiro Preto SP Fortaleza CE Fortaleza CE Fortaleza CE Fortaleza CE S. J. dos Campos SP Fortaleza CE Fortaleza CE Fortaleza CE S. J. dos Campos SP Fortaleza CE Macei AL Fortaleza CE Londrina PR Salvador BA Varginha MG Fortaleza CE Fortaleza CE Niteri RJ Juiz de Fora MG Fortaleza CE Rio de Janeiro RJ Itatiaia RJ So Paulo SP Fortaleza CE So Paulo SP Fortaleza CE Rio Claro SP So Paulo SP Fortaleza CE So Paulo SP So Paulo SP Fortaleza CE Cam BA Fortaleza CE Fortaleza CE Fortaleza CE Muribeca SE Rio de Janeiro RJ So Paulo SP Prmio Ouro Ouro Ouro Ouro Ouro Prata Prata Prata Prata Prata Prata Prata Prata Prata Prata Prata Bronze Bronze Bronze Bronze Bronze Bronze Bronze Bronze Bronze Bronze Bronze Bronze Bronze Bronze Bronze Meno Honrosa Meno Honrosa Meno Honrosa Meno Honrosa Meno Honrosa Meno Honrosa Meno Honrosa Meno Honrosa Meno Honrosa Meno Honrosa Meno Honrosa Meno Honrosa Meno Honrosa Meno Honrosa Meno Honrosa Meno Honrosa Meno Honrosa Meno Honrosa Meno Honrosa Meno Honrosa Meno Honrosa Meno Honrosa Meno Honrosa Meno Honrosa

EUREKA! N28, 2008

75

Sociedade Brasileira de Matemtica

Nvel Universitrio
Nome Fbio Dias Moreira Rafael Marini Silva Guilherme Rodrigues Nogueira de Souza Jos Marcos Andrade Ferraro Rafael Daigo Hirama Eduardo de Moraes Rodrigues Poo Felipe Rodrigues Nogueira de Souza Murilo Vasconcelos Andrade Leonardo Ribeiro de Castro Carvalho Luty Rodrigues Ribeiro Andr Linhares Rodrigues Maurcio de Lemos Rodrigues Collares Neto Henry Wei Cheng Hsu Kellem Correa Santos Levi Maximo Viana Ronaldo Rodrigues Pel Lus Daniel Barbosa Coelho Thiago Costa Leite Santos Helder Toshiro Suzuki Raphael Constant da Costa Rafael Sabino Lima Erick Costa e Silva Talarico Rodrigo Aguiar Pinheiro Renato Rebouas de Medeiros Jos Armando Barbosa Filho Evandro Makiyama de Melo Tiago Barbin Batalho Gabriel Ponce Vitor Gabriel Kleine Alexandre Hideki Deguchi Martani Vitor Humia Fontoura Ana Maria Menezes de Jesus Eduardo Fischer Anderson Hoshiko Aiziro Daniel Lopes Alves de Medeiros Paulo Andr Carvalho de Melo Pedro Meira de Vasconcellos Bezerra Willy George do Amaral Petrenko Gustavo Antnio da Silva Amaro Ricardo Monteiro da Silva Lanna Felipe Gonalves Assis Elder Rodrigo Barbosa Campos Matheus Pimentel Rodrigues Rafael Montezuma Pinheiro Cabral Nivan Roberto Ferreira Jnior Elton Gomes Coriolano Thoms Yoiti Sasaki Hoshina Samir Rodrigues Vieira Frederico de Souza Frydman Jordan Freitas Piva Rodrigo Viana Soares Bruno Euzbio dos Santos Antonia Taline de Souza Mendona Cidade Estado Rio de Janeiro RJ Vila Velha ES So Paulo SP So Paulo SP Campinas SP So Paulo SP So Paulo SP Macei AL So Paulo SP S.J. dos Campos SP Campinas SP Aracaju SE So Paulo SP Rio de Janeiro RJ Rio de Janeiro RJ So Carlos SP Rio de Janeiro RJ So Paulo SP So Paulo SP Rio de Janeiro RJ Rio de Janeiro RJ Rio de Janeiro RJ S.J. dos Campos SP S.J. dos Campos SP S.J. dos Campos SP So Paulo SP So Carlos SP So Carlos SP S.J. dos Campos SP So Paulo SP Rio de Janeiro RJ Itabaiana SE Encantado RS So Paulo SP S.J. dos Campos SP Rio de Janeiro RJ Recife PE Rio de Janeiro RJ So Carlos SP Belo Horizonte MG Campina Grande PB Rio de Janeiro RJ Rio de Janeiro RJ Fortaleza CE Olinda PE Campinas SP Rio de Janeiro RJ Fortaleza CE S.J. dos Campos SP Rio de Janeiro RJ Fortaleza CE Malhados SE Rio de Janeiro RJ Prmio Ouro Ouro Ouro Ouro Ouro Prata Prata Prata Prata Prata Prata Prata Prata Prata Bronze Bronze Bronze Bronze Bronze Bronze Bronze Bronze Bronze Bronze Bronze Bronze Bronze Bronze Bronze Bronze Bronze Bronze Bronze Bronze M. Honrosa M. Honrosa M. Honrosa M. Honrosa M. Honrosa M. Honrosa M. Honrosa M. Honrosa M. Honrosa M. Honrosa M. Honrosa M. Honrosa M. Honrosa M. Honrosa M. Honrosa M. Honrosa M. Honrosa M. Honrosa M. Honrosa

EUREKA! N28, 2008

76

Sociedade Brasileira de Matemtica

AGENDA OLMPICA
XXX OLIMPADA BRASILEIRA DE MATEMTICA NVEIS 1, 2 e 3 Primeira Fase Sbado, 14 de junho de 2008 Segunda Fase Sbado, 13 de setembro de 2008 Terceira Fase Sbado, 25 de outubro de 2007 (nveis 1, 2 e 3) Domingo, 26 de outubro de 2008 (nveis 2 e 3 - segundo dia de prova). NVEL UNIVERSITRIO Primeira Fase Sbado, 13 de setembro de 2008 Segunda Fase Sbado, 25 e Domingo, 26 de outubro de 2008

XIV OLIMPADA DE MAIO 10 de maio de 2008

XIX OLIMPADA DE MATEMTICA DO CONE SUL Temuco Chile 18 a 23 de junho de 2008

XLIX OLIMPADA INTERNACIONAL DE MATEMTICA 10 a 22 de julho de 2008 Madri Espanha

XIV OLIMPADA INTERNACIONAL DE MATEMTICA UNIVERSITRIA 25 a 31 de julho de 2008 Blagoevgrad, Bulgria

XXIII OLIMPADA IBEROAMERICANA DE MATEMTICA 18 a 28 de setembro de 2008 Salvador, Bahia Brasil

XI OLIMPADA IBEROAMERICANA DE MATEMTICA UNIVERSITRIA

EUREKA! N28, 2008

77

Sociedade Brasileira de Matemtica

COORDENADORES REGIONAIS
Alberto Hassen Raad Amrico Lpez Glvez Amarsio da Silva Arajo Andreia Goldani Antonio Carlos Nogueira Ali Tahzibi Benedito Tadeu Vasconcelos Freire Carlos Alexandre Ribeiro Martins Carmen Vieira Mathias Claus Haetinger Cleonor Crescncio das Neves Cludio de Lima Vidal Denice Fontana Nisxota Menegais Edson Roberto Abe lio Mega Eudes Antonio da Costa Fbio Brochero Martnez Florncio Ferreira Guimares Filho Francinildo Nobre Ferreira Genildo Alves Marinho Ivanilde Fernandes Saad Jacqueline Rojas Arancibia Janice T. Reichert Joo Bencio de Melo Neto Joo Francisco Melo Libonati Jose de Arimatia Fernandes Jos Luiz Rosas Pinho Jos Vieira Alves Jos William Costa Krerley Oliveira Licio Hernandes Bezerra Luciano G. Monteiro de Castro Luzinalva Miranda de Amorim Mrio Rocha Retamoso Marcelo Rufino de Oliveira Marcelo Mendes Newman Simes Nivaldo Costa Muniz Osnel Broche Cristo Osvaldo Germano do Rocio Raul Cintra de Negreiros Ribeiro Ronaldo Alves Garcia Rogrio da Silva Igncio Reginaldo de Lima Pereira Reinaldo Gen Ichiro Arakaki Ricardo Amorim Srgio Cludio Ramos Seme Gebara Neto Tadeu Ferreira Gomes Toms Menndez Rodrigues Valdenberg Arajo da Silva Vnia Cristina Silva Rodrigues Wagner Pereira Lopes
EUREKA! N28, 2008

(UFJF) (USP) (UFV) FACOS (UFU) (USP) (UFRN) (Univ. Tec. Fed. de Paran) (UNIFRA) (UNIVATES) (Inst. de Tec. e Educ. Galileo da Amaznia) (UNESP) (UNIPAMPA) (Colgio Objetivo de Campinas) (Faculdade Etapa) (Univ. Federal do Tocantins) (UFMG) (UFES) (UFSJ) (Centro Educacional Leonardo Da Vinci) (UC. Dom Bosco) (UFPB)) (UNOCHAPEC) (UFPI) (Grupo Educacional Ideal) (UFPB) (UFSC) (UFPB) (Instituto Pueri Domus) (UFAL) (UFSC) (Sistema Elite de Ensino) (UFBA) (UFRG) (Grupo Educacional Ideal) (Colgio Farias Brito, Pr-vestibular) (Cursinho CLQ Objetivo) (UFMA) (UFLA) (U. Estadual de Maring) (Colgio Anglo) (UFGO) (Col. Aplic. da UFPE) (Escola Tcnica Federal de Roraima) (UNIFESP) (Centro Educacional Logos) (IM-UFRGS) (UFMG) (UEBA) (U. Federal de Rondnia) (U. Federal de Sergipe) (U. Metodista de SP) (CEFET GO)

Juiz de Fora MG Ribeiro Preto SP Viosa MG Osrio RS Uberlndia MG So Carlos SP Natal RN Pato Branco PR Santa Mara RS Lajeado RS Manaus AM S.J. do Rio Preto SP Bag RS Campinas SP So Paulo SP Arraias TO Belo Horizonte MG Vitria ES So Joo del Rei MG Taguatingua DF Campo Grande MS Joo Pessoa PB Chapec SC Teresina PI Belm PA Campina Grande PB Florianpolis SC Campina Grande PB Santo Andr SP Macei AL Florianpolis SC Rio de Janeiro RJ Salvador BA Rio Grande RS Belm PA Fortaleza CE Piracicaba SP So Luis MA Lavras MG Maring PR Atibaia SP Goinia GO Recife PE Boa Vista RR SJ dos Campos SP Nova Iguau RJ Porto Alegre RS Belo Horizonte MG Juazeiro BA Porto Velho RO So Cristovo SE S.B. do Campo SP Jata GO

78

CONTEDO
XIV OLIMPADA DE MAIO XLVIII OLIMPADA INTERNACIONAL DE MATEMTICA Enunciados e Resultado Brasileiro XLIX OLIMPADA INTERNACIONAL DE MATEMTICA Enunciados e Resultado Brasileiro XXII OLIMPADA IBEROAMERICANA DE MATEMTICA Enunciados e Resultado Brasileiro XXIII OLIMPADA IBEROAMERICANA DE MATEMTICA Enunciados e Resultado Brasileiro 2 5

9 12

ARTIGOS
0,999... OU COMO COLOCAR UM BLOCO QUADRADO EM UM BURACO REDONDO Pablo Emanuel ALGORITMO DE GOSPER E APLICAES Humberto Silva Naves DOMINGO REGADO A REPUNITS Valberto Rmulo Feitosa Pereira HOMOTETIAS, COMPOSIO DE HOMOTETIAS EO PROBLEMA 6 DA IMO 2008 Carlos Yuzo Shine COMO QUE FAZ? OLIMPADAS AO REDOR DO MUNDO SOLUES DE PROBLEMAS PROPOSTOS PROBLEMAS PROPOSTOS AGENDA OLMPICA COORDENADORES REGIONAIS 14 21 27 32 44 48 52 62 63 64

Sociedade Brasileira de Matemtica

XIV OLIMPADA DE MAIO


PRIMEIRO NVEL
PROBLEMA 1

Quantos nmeros distintos de 6 algarismos e mltiplos de 45 podem ser escritos colocando um dgito esquerda e outro direita de 2008?
PROBLEMA 2

No colgio Olmpico as provas so avaliadas com nmeros inteiros, a menor nota possvel 0, e a maior 10. Na aula de Matemtica o professor aplica as provas. Este ano a turma tem 15 alunos. Quando um dos alunos tira na primeira prova uma nota menor que 3 e na segunda prova uma nota maior que 7, o aluno chamado de aluno superado. O professor, ao terminar de corrigir as provas, fez uma mdia com as 30 notas e obteve 8. Qual a maior quantidade de alunos superados que pode ter tido a turma?
PROBLEMA 3

Num quadro negro esto escritos os nmeros inteiros de 1 a 2008 inclusive. Apagam-se dois nmeros e escreve-se a diferena entre eles. Por exemplo, se apagamos o nmero 5 e 241, escrevemos 236. Assim continuamos, apagando os nmeros e escrevendo a diferena, at que fica somente um nmero. Determine se o nmero que ficou por ltimo pode ser 2008. E 2007? Em cada caso, se a resposta afirmativa indique uma seqncia com esse nmero final, e se negativa, explique o porqu.
PROBLEMA 4

Sobre o lado AB de um quadrado ABCD desenhado exteriormente o tringulo retngulo ABF, de hipotenusa AB. Sabe-se que AF = 6, e que BF = 8. Chamamos de E o centro do quadrado. Calcule o comprimento de EF.
PROBLEMA 5

Num tabuleiro de 16 16 colocamos 25 moedas, como na figura abaixo. permitido selecionar 8 linhas e 8 colunas e retirar do tabuleiro todas as moedas que se encontram nessas linhas e colunas. Determine se possvel retirar todas as moedas do tabuleiro. Se a resposta afirmativa, indique as 8 linhas e as 8 colunas selecionadas, e se negativa, explique o porqu.

EUREKA! N29, 2009

Sociedade Brasileira de Matemtica

SEGUNDO NVEL
PROBLEMA 1

Num quadro negro est escrita a seguinte expresso: 1 2 22 23 24 25 26 27 28 29 210. Juan distribui parntesis de distintas maneiras e efetua o clculo que fica. Por exemplo: 1 2 (22 23 ) 24 (25 26 27 ) 28 (29 210 ) = 403 ou
1 (2 22 (23 24 ) (25 26 27 )) (28 29 ) 210 = 933. Quantos resultados diferentes pode obter Juan?
PROBLEMA 2

No retngulo ABCD de lados AB, BC, CD e DA, seja P um ponto do lado AD tal = 90 . A perpendicular a BP traada por A corta BP em M e a que BPC perpendicular a CP traada por D corta CP em N. Demonstre que o centro do retngulo est no segmento MN.
PROBLEMA 3

Nos nmeros 1010...101 esto alternados uns e zeros; se h n uns, h n 1 zeros (n 2). Determine os valores de n para os quais o nmero 1010...101, que tem n uns, primo.
PROBLEMA 4

No plano h 16 retas tais que no h duas paralelas nem trs concorrentes. Sebastin tem que pintar os 120 pontos que so interseo de duas retas de modo que em cada reta todos os pontos pintados sejam de cor diferente. Determine o nmero mnimo de cores que Sebastin precisa para concluir a tarefa. E se as retas so 15 (neste caso, os pontos so 105)?

EUREKA! N29, 2009

Sociedade Brasileira de Matemtica

PROBLEMA 5

Matias cobriu um tabuleiro quadrado de 7 7, dividido em casas de 1 1, com peas dos trs tipos a seguir

Tipo 1

Tipo 2

Tipo 3

sem buracos nem superposies, e sem sair do tabuleiro. Cada pea do tipo 1 cobre exatamente 3 casas e cada pea do tipo 2 ou do tipo 3 cobre exatamente 4 casas. Determine a quantidade de peas do tipo 1 que Matias pode ter utilizado. (As peas podem girar e ser viradas). RESULTADO BRASILEIRO
2008: Nvel 1 (at 13 anos)
Nome Rafael Kazuhiro Miyazaki Dbora Ornellas Nicolas Seoane Miquelin Guilherme Renato Martins Unze Ana Beatrice Bonganha Zanon Paula Dias Garcia Lara Timb Arajo Francisco Markan Nobre de Souza Filho Henrique Gasparini Fiza do Nascimento Henrique Vieira G. Vaz Cidade - Estado So Paulo SP Salvador BA Mau SP So Paulo SP Santo Andr SP Braslia DF Fortaleza CE Fortaleza CE Braslia DF So Paulo SP Pontos 45 36 34 33 26 24 24 24 23 23 Prmio Medalha de Ouro Medalha de Prata Medalha de Prata Medalha de Bronze Medalha de Bronze Medalha de Bronze Medalha de Bronze Meno Honrosa Meno Honrosa Meno Honrosa

2008: Nvel 2 (at 15 anos)


Nome Joo Lucas Camelo S Guilherme da Rocha Dahrug Matheus Barros de Paula Rafael Ferreira Antonioli Nara Gabriela de Mesquita Peixoto Rodrigo Nagamine Henrique Lopes de Mello Victorio Takahashi Chu Jonas Rocha de Lima Amaro Deborah Barbosa Alves Cidade - Estado Fortaleza CE Santo Andr SP Tauba SP S.B. do Campo SP Fortaleza CE Santo Andr SP Rio de Janeiro RJ So Paulo SP Fortaleza CE So Paulo SP Pontos 37 30 29 23 22 22 21 21 17 16 Prmio Medalha de Ouro Medalha de Prata Medalha de Prata Medalha de Bronze Medalha de Bronze Medalha de Bronze Medalha de Bronze Meno Honrosa Meno Honrosa Meno Honrosa

EUREKA! N29, 2009

Sociedade Brasileira de Matemtica

XLVIII OLIMPADA INTERNACIONAL DE MATEMTICA


Enunciados e Resultado Brasileiro
A XLVIII Olimpada Internacional de Matemtica foi realizada na cidade de Hani, Vietn no perodo de 19 a 31 de julho de 2007. A equipe brasileira foi liderada pelos professores Carlos Gustavo Moreira do Rio de Janeiro RJ e Onofre Campos da Silva Farias de Fortaleza CE.

RESULTADOS DA EQUIPE BRASILEIRA BRA1 BRA2 BRA3 BRA4 BRA5 BRA6 Rgis Prado Barbosa Henrique Pond de Oliveira Pinto Ramn Moreira Nunes Rafael Sampaio de Rezende Rafael Tupynamb Dutra Guilherme Phillipe Figueiredo Medalha de Prata Medalha de Prata Medalha de Bronze Medalha de Bronze Medalha de Bronze Meno Honrosa

PRIMEIRO DIA
PROBLEMA 1:

Sejam a1 , a2 ,..., an nmeros reais. Para cada i(1 i n) definimos


d i = max{a j :1 j i} min{a j : i j n}

e
d = max{d i :1 i n}.

(a) Prove que para quaisquer nmeros reais x1 x2 ... xn ,


d . (*) 2 (b) Prove que existem nmeros reais x1 x2 ... xn para os quais vale a igualdade em (*). max{ xi ai :1 i n}
PROBLEMA 2:

So dados cinco pontos A, B, C D e E tais que ABCD um paralelogramo e BCED um quadriltero cclico (e convexo). Seja l uma recta que passa por A.
EUREKA! N29, 2009

Sociedade Brasileira de Matemtica

Suponhamos que l intersecta o segmento DC num ponto interior F e a recta BC em G. Suponhamos tambm que EF = EG = EC. Prove que l a bissectriz do ngulo DAB.
PROBLEMA 3:

Numa competio de Matemtica alguns participantes so amigos. A amizade sempre recproca. Dizemos que um grupo de participantes um clique se dois quaisquer deles so amigos (em particular, qualquer grupo com menos de dois participantes um clique). O tamanho de um clique o nmero de seus elementos. Sabe-se que nesta competio o tamanho mximo dos cliques par. Prove que os participantes podem ser distribudos em duas salas, de modo que o tamanho mximo dos cliques contidos numa sala igual ao tamanho mximo dos cliques contidos na outra sala.

SEGUNDO DIA
PROBLEMA 4:

No tringulo ABC a bissectriz do ngulo BCA intersecta a circunferncia circunscrita em R ( R C ), a mediatriz de BC em P e a mediatriz de AC em Q. O ponto mdio de BC K e o ponto mdio de AC L. Mostre que os tringulos RPK e RQL tm reas iguais.
PROBLEMA 5:

Sejam a e b inteiros positivos tais que 4ab 1 divide (4a 2 1) 2 . Prove que a = b.
PROBLEMA 6:

Seja n um nmero inteiro positivo. Considere o conjunto S = {( x, y , z ) : x, y , z {0,1,..., n}, x + y + z > 0} de (n + 1)3 1 pontos no espao tridimencional. Determine o menor nmero possvel de planos cuja unio contm todos os pontos de S mas no contm (0, 0, 0).

EUREKA! N29, 2009

Sociedade Brasileira de Matemtica

XLIX OLIMPADA INTERNACIONAL DE MATEMTICA


Enunciados e Resultado Brasileiro
A XLIX Olimpada Internacional de Matemtica foi realizada na cidade de Madri, Espanha no perodo de 10 a 22 de julho de 2008. A equipe brasileira foi liderada pelos professores Luciano Monteiro de Castro do Rio de Janeiro RJ e Carlos Yuzo Shine de So Paulo SP. RESULTADOS DA EQUIPE BRASILEIRA BRA1 BRA2 BRA3 BRA4 BRA5 BRA6 Davi Lopes Alves de Medeiros Henrique Pond de Oliveira Pinto Marcelo Matheus Gauy Rafael Tupynamb Dutra Rgis Prado Barbosa Renan Henrique Finder Medalha de Prata Medalha de Prata Medalha de Bronze Medalha de Prata Medalha de Prata Medalha de Prata

PRIMEIRO DIA
PROBLEMA 1:

Seja ABC um tringulo acutngulo e seja H o seu ortocentro. A circunferncia de centro no ponto mdio de BC e que passa por H intersecta a reta BC nos pontos A1 e A2 . Analogamente, a circunferncia de centro no ponto mdio de CA e que passa por H intersecta a reta CA nos pontos B1 e B2 , e a circunferncia de centro no ponto mdio de AB e que passa por H intersecta a reta AB nos pontos C1 e C2 . Mostre que A1 , A2 , B1, B 2 ,C 1, C 2 esto sobre uma mesma circunferncia.
PROBLEMA 2:

(a) Prove que


x2 y2 z2 + + 1 2 2 ( x 1) ( y 1) ( z 1)2 para todos os nmeros reais x, y, z, diferentes de 1, com xyz = 1.

(*)

(b) Prove que existe uma infinidade de ternos de nmeros racionais x, y, z, diferentes de 1, com xyz = 1, para os quais ocorre a igualdade em (*).
EUREKA! N29, 2009

Sociedade Brasileira de Matemtica

PROBLEMA 3:

Prove que existe um nmero infinito de inteiros positivos n tais que n 2 + 1 tem um divisor primo maior que 2n + 2n .
PROBLEMA 4:

Determine todas as funes f ]0, [ ]0, [ (ou seja, f uma funo dos reais positivos para os reais positivos) tais que

( f ( w) )

+ ( f ( x) )

f ( y2 ) + f (z2 )

w2 + x 2 y2 + z2

para todos os nmeros reais positivos w, x, y com wx = yz.

Sejam n e k nmeros inteiros positivos tais que k n e k n um nmero par. So dadas 2n lmpadas numeradas de 1 a 2n, cada uma das quais pode estar acesa ou apagada. Inicialmente todas as lmpadas esto apagadas. Uma operao consiste em alterar o estado de exatamente uma das lmpadas (de acessa para apagada ou de apagada para acesa). Consideremos seqncias de operaes. Seja n o nmero de seqncias com k operaes aps as quais as lmpadas de 1 a n esto todas acessas e as lmpadas de n + 1 a 2n esto todas apagadas. Seja M o nmero de seqncias com k operaes aps as quais as lmpadas de 1 a n esto todas acesas e as lmpadas de n + 1 e 2n esto todas apagadas, e durante as quais todas as lmpadas de n + 1 a 2n permanecem sempre apagadas. N . Determine a razo M
PROBLEMA 6:

PROBLEMA 5:

Seja ABCD um quadriltero convexo cujos lados BA e BC tam comprimentos diferentes. Sejam w1 e w2 as circunferncias inscritas nos tringulos ABC e ADC, respectivamente. Suponhamos que existe um circunferncia w tangente reta BA de forma que A est entre B e o ponto de tangncia, tangente reta BC de forma que C est entre B e o ponto de tangncia, e que tambm seja tangente s retas AD e CD. Prove que as tangentes comuns exteriores a w1 e w2 se intersectam sobre w.
EUREKA! N29, 2009

Sociedade Brasileira de Matemtica

XXII OLIMPADA IBEROAMERICANA DE MATEMTICA


Enunciados e Resultado Brasileiro
A XXII Olimpada Iberoamericana de Matemtica foi realizada na cidade de Coimbra, Portugal no perodo de 6 a 16 de setembro de 2007. A equipe brasileira foi liderada pelos professores Eduardo Wagner e Edmilson Motta da cidade do Rio de Janeiro RJ e So Paulo SP respectivamente. RESULTADOS DA EQUIPE BRASILEIRA BRA1 BRA2 BRA3 BRA4 Guilherme Rodrigues Nogueira de Souza Henrique Pond de Oliveira Pinto Ramon Moreira Nunes Rgis Prado Barbosa Medalha de Ouro Medalha de Prata Medalha de Ouro Medalha de Ouro

PRIMEIRO DIA
PROBLEMA 1:

Dado um inteiro positivo m, define-se a sucesso {an } da seguinte maneira: m a1 = , an +1 = an an , si n 1. 2 Determinar todos os valores de m para os quais a2007 o primeiro inteiro que aparece na sucesso. Nota: Para um nmero real x se define x como o menor inteiro que maior ou igual a x. Por exemplo, = 4, 2007 = 2007. Sejam ABC um tringulo com incentro I e uma circunferncia de centro I, de raio maior que o da circunferncia inscrita e que no passa por nenhum dos vrtices. Sejam X 1 o ponto de interseco de com a reta AB mais perto de B, X 2 e X 3 os pontos de interseco de com a recta BC sendo X 2 o mais perto de B e X 4 o ponto de interseco de com a recta CA mais perto de C. Seja K o ponto de interseco das rectas X 1 X 2 e X 3 X 4 . Demonstre que AK corta o segmento X 2 X 3 no seu ponto mdio.
PROBLEMA 2:

EUREKA! N29, 2009

Sociedade Brasileira de Matemtica

PROBLEMA 3:

Duas equipes, A e B disputam o territrio limitado por uma circunferncia. A tem n bandeiras azuis e B tem n bandeiras brancas ( n 2, fixo). Jogam alternadamente e A comea o jogo. Cada equipe, na sua vez, coloca uma das suas bandeiras num ponto da circunferncia que no se tenha usado numa jogada anterior. Cada bandeira, uma vez colocada, no se pode mudar de lugar. Uma vez colocadas as 2n bandeiras reparte-se o territrio entre as duas equipes. Um ponto do territrio da equipe A se a bandeira mais prxima dele azul, e da equipe B se a bandeira mais prxima dele branca. Se a bandeira azul mais prxima de um ponto est mesma distncia que a bandeiras branca mais prxima deste ponto, ento o ponto neutro (no de A nem de B). Uma equipe ganha o jogo se seus pontos cobrem uma rea maior que a rea coberta pelos pontos da outra equipe. H empate se ambos cobrem reas iguais. Demonstre que, para todo n a equipe B tem estratgia para ganhar o jogo.

SEGUNDO DIA
PROBLEMA 4:

Em um tabuleiro quadriculado de tamanho 19 19, uma ficha chamada drago d saltos da seguinte maneira: desloca-se 4 casas numa direco paralela a um dos lados do tabuleiro e 1 casa em direo perpendicular anterior.

X D

A partir de D, o drago pode saltar para uma das quatro posies X. Sabe-se que, com este tipo de saltos, o drago pode mover-se de qualquer casa a qualquer outra. A distancia dragoniana entre duas casas o menor nmero de saltos que o drago deve dar para mover-se de uma casa a outra. Seja C uma casa situada num canto do tabuleiro e seja V a casa vizinha a C que a toca num nico ponto.
EUREKA! N29, 2009

10

Sociedade Brasileira de Matemtica

Demonstre que existe alguma casa X do tabuleiro tal que a distncia dragoniaana de C a X maior que a distncia dragoniana de C a V.
PROBLEMA 5:

Um nmero natural n atrevido se o conjunto dos seus divisores, incluindo 1 e n, pode ser dividido em trs subconjuntos tais que a soma dos elementos de cada subconjunto a mesma nos trs. Qual a menor quantidade de divisores que pode ter um nmero atrevido?
PROBLEMA 6:

Seja F a famlia de todos os hexgonos convexos H que satisfazem as seguintes condies: a) os lados opostos de H so paralelos; b) quaisquer trs vrtices de H podem ser cobertos por uma faixa de largura 1. Determine o menor nmero real l tal que cada um dos hexgonos da famlia F pode ser coberto com uma faixa de largura l . Nota: Uma faixa de largura l a regio do plano compreendida entre duas rectas paralelas que esto distancia l (includas ambas as rectas paralelas).

EUREKA! N29, 2009

11

Sociedade Brasileira de Matemtica

XXIII OLIMPADA IBEROAMERICANA DE MATEMTICA


Enunciados e Resultado Brasileiro
A XXIII Olimpada Iberoamericana de Matemtica foi realizada na cidade de Salvador, Bahia no perodo de 20 a 28 de setembro de 2008. A equipe brasileira foi liderada pelos professores Eduardo Wagner e Fbio Dias Moreira, ambos da cidade de Rio de Janeiro RJ. Com este resultado a equipe brasileira obteve tambm a maior pontuao total da competio ficando em primeiro lugar com 155 pontos. RESULTADOS DA EQUIPE BRASILEIRA BRA1 BRA2 BRA3 BRA4 Henrique Ponde de Oliveira Pinto Renan Henrique Finder Ramon Moreira Nunes Rgis Prado Barbosa Medalha de Ouro Medalha de Prata Medalha de Ouro Medalha de Prata

PRIMEIRO DIA
PROBLEMA 1:

Os nmeros 1, 2, 3,..., 20082 so distribudos num tabuleiro 2008 2008, de modo que em cada casa haja um nmero distinto. Para cada linha e cada coluna do tabuleiro calcula-se a diferena entre o maior e o menor dos seus elementos. Seja S a soma dos 4016 nmeros obtidos. Determine o maior valor possvel para S. Sejam ABC um tringulo escaleno e r a bissectriz externa do ngulo ABC. Sejam P e Q os ps das perpendiculares recta r que passam por A e C, respectivamente. As rectas CP e AB intersectam-se em M e as rectas AQ e BC intersectam-se em N. Demonstre que as rectas AC, MN e r tm um ponto em comum.
PROBLEMA 3: PROBLEMA 2:

Sejam m e n inteiros tais que o polinmio P ( x) = x 3 + mx + n tem a seguinte propriedade: se x e y so inteiros e 107 divide P ( x) P ( y ), ento 107 divide x y. Demosntre que 107 divide m.

EUREKA! N29, 2009

12

Sociedade Brasileira de Matemtica

SEGUNDO DIA
PROBLEMA 4:

Demonstre que no existem inteiros positivos x e y tais que x 2008 + 2008! = 21y.
PROBLEMA 5:

Seja ABC um tringulo e X, Y, Z pontos interiores dos lados BC, AC, AB respectivamente. Sejam A, B, C os circuncentros dos tringulos AZY, BXZ, CYX, respectivamente. Demonstre que ( ABC ) ( ABC) 4 e que a igualdade ocorre se, o somente se, as rectas AA, BB, CC tm um ponto em comum. Observao: Para um tringulo qualquer RST, denotamos a sua rea por (RST).
PROBLEMA 6:

Numa partida de biribol enfrentam-se duas equipes de quatro jogadores cada uma. Organiza-se um torneio de biribol em que participam n pessoas, que formam equipes para cada partida (as equipes no so fixas). No final do torneio observouse que cada duas pessoas disputaram exactamente uma partida em equipes rivais. Para que valores de n possvel organizar um torneio com tais caractersticas?

EUREKA! N29, 2009

13

Sociedade Brasileira de Matemtica

0,999... OU COMO COLOCAR UM BLOCO QUADRADO EM UM BURACO REDONDO


Pablo Emanuel Nvel Intermedirio

Quando um jovem estudante de matemtica comea a estudar os nmeros reais, difcil no sentir certo desconforto e estranhamento. De repente, algumas coisas que faziam sentido param de funcionar to bem assim. Com certeza, uma das mais estranhas pode ser resumida na igualdade 0,9999..... = 1,0000... Como assim? No mundo dos nmeros inteiros (com a exceo muito razovel de 0 = 0), qualquer nmero tem uma nica representao decimal (os zeros esquerda no so um problema srio, ou definimos que nunca podemos comear com 0 o que deixa o prprio zero em uma situao meio desconfortvel de ter uma representao decimal vazia ou definimos que sempre vamos completar com infinitos zeros esquerda), e esta se comporta bem i.e. qualquer nmero da forma 8xxxxxxx menor que qualquer nmero da forma 9yyyyyyy com o mesmo nmero de dgitos. Como pode haver um nmero comeado por 0,9 que no seja menor que um nmero comeado por 1,0 e, pior, que seja igual!? A resposta explica, mas no convence: se definirmos as seqncias: an = 0.999999...9 (n dgitos 9) e bn = 1.000000...0 (n dgitos 0) temos que an < 0.999... 1.000... < bn. No entanto, como bn an = 1/10n, que converge para 0, a diferena entre 1.000... e 0.999..., que menor que qualquer das diferenas bn an s pode ser 0, logo os nmeros so iguais. OK, entendido, mas ainda tem caroo neste angu. Por que a representao decimal, que se comporta to bem para nmeros inteiros tem este tipo de esquisitice para nmeros reais? A verdade que usar a representao decimal para nmeros reais enfiar um bloco quadrado em um buraco redondo. Em primeiro lugar, at agora estamos usando a representao decimal (base 10) apenas porque a representao com que estamos mais acostumados. Ser que o
EUREKA! N29, 2009

14

Sociedade Brasileira de Matemtica

problema est com o nmero 10? Infelizmente no. Mesmo que usemos outras bases, o problema continua: 0,1111.... = 1,000.... em base 2 0,2222... = 1,000... em base 3 e o mesmo problema acontece em qualquer base que escolhamos (e a demonstrao exatamente a mesma, mutatis mutandis, que fizemos l em cima). Antes de entrarmos a fundo em por que a representao decimal (ou em qualquer base N) tem estes problemas, bom ver que outras opes ns temos para representar os nmeros reais. Em primeiro lugar, nossas notaes foram feitas para trabalhar com nmeros inteiros, que o que nos familiar (a ponto de Pitgoras afirmar que os nmeros inteiros so a fundao do universo). Dos nmeros inteiros conseguimos de forma natural derivar os nmeros racionais (os nmeros da forma a/b, onde a e b so inteiros). O pulo para os nmeros reais irracionais, no entanto, menos natural, e as nicas maneiras que temos para tentar nomear os nmeros irracionais so atravs das aproximaes por nmeros racionais. A prpria representao decimal a aproximao por nmeros racionais da forma A/10n (ou A/Nn, em base N). Duas outras formas de representar os nmeros reais so as fraes contnuas e os cortes de Dedekind. A representao por fraes contnuas tem a forma N1 + 1 _________________ N2 + 1 ___________ N3 + ...

O algoritmo para construir a representao por fraes contnuas de um nmero x bastante simples. Em primeiro lugar, N1 a parte inteira de x (i.e. o maior nmero inteiro que menor ou igual a x). Se x N1 = 0, acabou, seno, x N1 maior que 0 e menor que 1, portanto y = 1/(x N1) um nmero maior que 1. Seja N2 ento a parte inteira deste nmero, o que equivale a dizer que 1/(N2 + 1) < x N1 1/N2. Agora repita o processo com y no lugar de x (i.e. N3 o nmero tal que 1/(N3 + 1) < y N2 1/N3), e assim por diante at chegar a um nmero inteiro ou continuando para sempre.

EUREKA! N29, 2009

15

Sociedade Brasileira de Matemtica

A representao por fraes contnuas no tem o mesmo problema da representao decimal (cada nmero real tem uma e somente uma representao por fraes contnuas), mas tem outros inconvenientes. O primeiro, que no to srio assim, que os nmeros Ni, que seriam equivalentes aos dgitos, podem ser arbitrariamente grandes, ao contrrio dos dgitos decimais que s podem ser de 0 a 9. O segundo, muito mais srio, que terrivelmente difcil fazer contas com fraes contnuas. Este foi o principal motivo de a representao decimal ter substitudo a numerao romana na Europa no sculo XV era imensamente mais simples fazer contas com a representao decimal do que com a representao romana. (E este tambm o principal motivo de a representao binria ter tomado espao da representao decimal no sculo XX, ainda mais fcil fazer contas em binrio). Outra representao dos nmeros reais atravs dos cortes de Dedekind. A diferena do corte de Dedekind para a representao decimal ou de fraes contnuas que, enquanto as ltimas usam uma seqncia convergente de nmeros racionais, os cortes usam conjuntos sem uma seqncia definida. Um corte de Dedekind um conjunto de nmeros racionais limitado, que no possui um maior elemento e fechado inferiormente. Ou seja, um conjunto de racionais A tal que: 1) existe um racional X tal que X > a para todo a pertencente a A 2) para todo a pertencente a A, existe um b pertencente a A tal que b > a. 3) Se a pertence a A, e um racional c < a, ento c pertence a A. Em outras palavras, cada um destes conjuntos o conjunto de todos os nmeros racionais menores que um nmero real determinado. Esta representao til teoricamente para provar algumas propriedades dos nmeros reais, mas praticamente impossvel de ser utilizada na prtica para as tarefas corriqueiras do dia-a-dia. A concluso que, apesar dos seus defeitos, a representao de base N a representao mais prtica que temos para os nmeros reais. Ento vamos entendla mais a fundo. Em primeiro lugar, a parte antes da vrgula (ou ponto decimal, dependendo de em qual lugar do mundo voc vive) apenas a parte inteira do nmero, com a nossa velha e bem comportada representao decimal de nmeros inteiros. Vamos nos concentrar ento na parte direita da vrgula, os nmeros entre 0 e 1.
EUREKA! N29, 2009

16

Sociedade Brasileira de Matemtica

0,0

0,1

0,2

0,3

0,4

0,5

0,6

0,7

0,8

0,9

O primeiro dgito decimal diz em qual dos dez intervalos acima o nmero est. Para descobrir o segundo dgito, basta subdividir cada intervalo novamente em 10 pedaos, ou, equivalentemente, multiplicar o nmero por 10 e ver em qual dos intervalos a parte fracionria deste novo nmero (10x) cai, de acordo com o grfico abaixo:
0,9 0,8 0,7 0,6 0,5 0,4 0,3 0,2 0,1 0,0 0,0 0,1 0,2 0,3 0,4 0,5 0,6 0,7 0,8 0,9

Os pontos onde nasce o nosso problema so justamente estes pontos marcados na nossa linha (0; 0,1; 0,2; ...; 1). Repare que o grfico acima descontnuo nestes pontos por exemplo, as imagens dos pontos se aproximando de 0,2 pela esquerda tendem a 1, enquanto pela direita tendem a 0. Dito de outra forma, os nmeros se aproximando pela esquerda de 0,2 vo ter uma seqncia longa de 9s depois do 0,1 e pela direita uma seqncia longa de 0s depois do 0,2. Antes de prosseguir com a anlise, j vimos que no existe nada de especial no nmero 10, portanto, por pura preguia, a partir de agora vou mudar para base 2, mas deixo ao leitor desconfiado a tarefa de reescrever tudo o que se segue em base 10. Apenas para comear do ponto onde paramos, o grfico acima para base 2 fica assim:

EUREKA! N29, 2009

17

Sociedade Brasileira de Matemtica

0,1

0,0 0,0 0,1

Relembrando, o problema est no ponto da descontinuidade, neste caso o 0,1 (tambm conhecido como ). A origem do problema o fato de haver nmeros comeados por 0,0 arbitrariamente prximos de 0,1, quando, naturalmente, nmeros comeados por 0,0 deveriam ser distintamente menores do que 0,1. Vamos tentar corrigir este problema dando um pequeno espao entre os intervalos no nosso desenho.

0,1

0,0 0,0 0,1

Vamos passar agora para o segundo dgito.

0,1

0,0 0,00
EUREKA! N29, 2009

0,01

0,10

0,11

18

Sociedade Brasileira de Matemtica

O espao que demos entre os intervalos do primeiro dgito naturalmente se propaga para o segundo dgito (seno teramos o mesmo problema nos nmeros 0,01 - e 0,11 ). [Em base 10 seriam 9 buracos na primeira rodada e 90 na segunda, entenderam a minha preguia?] Neste ponto, j no difcil imaginar o processo para o terceiro dgito criar mais um buraco no meio de cada um dos 4 intervalos restantes, e assim por diante. O conjunto dos nmeros que sobram depois de fazermos este processo infinitas vezes um conjunto em que podemos usar a nossa representao de base 2 (ou base 10, se fizermos o processo no desenho original l de cima) sem medo cada nmero tem apenas uma representao, e elas respeitam a ordem e as distncias que esperaramos: por exemplo, se um nmero comea por 0,11010 ele estritamente menor que qualquer nmero que comece por 0,11011. O nico problema que o conjunto que temos no final deste processo to esfarelado que ele no contm nem um intervalinho que seja. Se escolhssemos um nmero entre 0 e 1 aleatoriamente, ele teria 0% de chance de estar no nosso conjunto repare que eu disse 0% no uma em um quaquilho elevado a um googleplex, no a chance de ganhar na megasena todas as semanas pelo resto da vida, mas zero! Este conjunto to importante que tem um nome conjunto de Cantor, em homenagem ao matemtico Georg Cantor que, entre outras coisas criou a teoria dos conjuntos (todos os conjuntos ento so um pouquinho conjuntos de Cantor) e provou que existiam mais nmeros irracionais do que nmeros racionais. Dado que acabamos de ver que o prprio sistema de representao decimal que usamos todos os dias na verdade um conjunto de Cantor, de se admirar que apenas no final do sculo XIX tenhamos passado a conhec-lo. Pelo que acabamos de ver, o nosso sistema de numerao um mapeamento entre um conjunto de Cantor e o intervalo [0,1]. O cerne da nossa questo que esta funo no injetiva, ou seja, existem elementos distintos a e b no conjunto de Cantor que so mapeados para o mesmo nmero real (a diferente de b, f(a) = f(b)), por exemplo, 0,19999... e 0,2. [Parnteses: outra forma de ver isto afirmar que o intervalo [0,1] o espao quociente do conjunto de Cantor pela relao de equivalncia entre os nmeros terminados por 999... e os seus correspondentes terminados por 000..., ver o artigo Egalit] no endereo: http://www.impa.br/~gugu/pablo-egalite.doc Vamos tentar desenhar o grfico desta funo:

EUREKA! N29, 2009

19

Sociedade Brasileira de Matemtica

Os dois extremos do buraco do meio so os pontos 0,01111.... e 0,10000... que representam o mesmo nmero , portanto a funo constante igual a neste intervalo. Da mesma forma cada um dos buracos de geraes maiores corresponde a diferentes plats no grfico. O mais surpreendente que, depois de criarmos os infinitos plats, o grfico final um grfico contnuo, que chamado de funo de Cantor, ou de escada do diabo (porque sempre existem infinitos degraus entre quaisquer dois degraus da escada, voc nunca conseguiria sair do lugar assim como Aquiles correndo atrs da tartaruga). Infelizmente, o que se tem quando se tenta botar um bloco quadrado em um buraco redondo, ou neste caso, botar um intervalo dentro de um conjunto de Cantor.

EUREKA! N29, 2009

20

Sociedade Brasileira de Matemtica

ALGORITMO DE GOSPER E APLICAES


Humberto Silva Naves Nvel Avanado

Continuando com as idias do artigo Integrais discretas (de Eduardo Poo na Eureka nmero 27), vamos tentar descobrir frmulas fechadas para alguns somatrios da forma:
(*) zk
k =0 n 1

Algumas consideraes devem ser feitas antes de continuarmos: 1- Vamos assumir que zk uma seqncia hipergeomtrica, isto , a razo r (k ) zk +1 / zk uma funo racional de k. 2- Por frmula fechada, entendemos que existe uma seqncia n 1 hipergeomtrica sn tal que sn = k (essa definio no to =0 zk restritiva assim, pois veremos que a classe das seqncias hipergeomtricas bastante ampla). Por exemplo, vamos tentar achar uma frmula fechada para:
2k n 1 k sn = zk , onde zk = 2 k (k + 1)4 k =0
2

Claramente zk hipergeomtrica, uma vez que r (k ) = zk +1 / zk = uma funo racional de k. Por (*), vale:
zn = sn +1 sn zn sn +1 = 1 sn sn

(2k + 1) 2 4(k + 1)(k + 2)

s A nossa frmula fechada sn hipergeomtrica, logo n +1 1 sn racional y(n), portanto:


sn = y (n) zn y (n + 1) zn +1 y ( n) zn = zn
EUREKA! N29, 2009

uma funo

21

Sociedade Brasileira de Matemtica

r ( n ) y (n + 1) y ( n) = 1

onde: r (n) =

(2n + 1)2 4(n + 1)(n + 2)

Como y(n) racional, existem polinmios P (n), Q( n) com mdc{P (n), Q (n)} = 1 P (n) tais que y (n) = e substituindo na equao anterior, vale: Q ( n)
(2n + 1)2 P (n + 1)Q(n) 4(n + 1)(n + 2)Q( n + 1) P (n) =
= 4(n + 1)( n + 2)Q (n + 1)Q( n), n 0

Disto conclu-se que: 1- Q(n) | 4(n + 1)( n + 2)Q (n + 1) 2- Q(n + 1) (2n + 1) 2 Q (n) Q (n) (2n 1)2 Q (n 1) (Observao: todas as relaes de divisibilidade e os mdcs referencem-se aos polinmios em si e no aos seus valores em um dado ponto) Por (1), temos que se uma raiz de Q ento: = 1 ou = 2 ou + 1 raiz de Q. E por (2) se raiz de Q ento: = 1 ou 1 tambm raiz de Q. Portanto, 2 como Q no pode ter infinitas razes, Q no possui raiz alguma, isto , Q(n) constante (sem perda de generalidade, Q(n) = 1). Disto conclu-se que:
(2n + 1)2 P (n + 1) 4(n + 1)(n + 2) P(n) = 4(n + 1)(n + 2)

Logo: (n + 1)(n + 2) P(n + 1) n(n + 1) P (n) , da P (n) = n( n + 1) P (n) , ento vale:


(2n + 1)2 P (n + 1) 4n(n + 1) P (n) = 4, onde P (n) um polinmio

Seja d = deg P, P (n) = ad n d + ... + a0 ; substituindo na equao anterior, vale (para o caso d > 0):
(4n 2 + 4 n + 1)(ad (n + 1) d + ... + a0 ) (4n 2 + 4n)(ad n d + ... + a0 ) = 4 (4n 2 + 4 n + 1)(ad n d + (ad 1 + dad )n d 1 + ...) (4n 2 + 4n)(ad n d + ad 1n d 1 + ...) = 4
EUREKA! N29, 2009

22

Sociedade Brasileira de Matemtica

4dad n d +1 + ... = 4

Mas, se d > 0, 4dad n d +1 + ... = 4 um polinmio de grau d + 1. Portanto


d = 0 P(n) = 4 (que de fato soluo da equao anterior), donde conclumos que: y (n) = 4n(n + 1), logo:
2k 2n 2n n n 1 k = 4n(n + 1) n = n 2k (n + 1)42 n 42 n 1 k = 0 ( k + 1)4
2 2 2

( fcil ver que a frmula vale para n = 0 e n = 1, e, pela identidade que provamos, vale sempre). Vamos agora generalizar essa idia para qualquer seqncia hipergeomtrica zk . O leitor atento deve ter notado que na concluso de que Q(n) = 1, implicitamente usamos o fato que: mdc{A(n), B(n + h)} = 1, para qualquer inteiro h 0 , onde A(n) = (2n + 1) 2 , e B (n) = 4(n + 1)(n + 2) , so o numerador e o denominador de r(n) respectivamente. Mas nem sempre possvel escrever r (n) = polinmios satisfazendo as condies:
mdc{ A(n), B (n + h)} = 1, h + .

zn +1

zn

como a razo de dois

O leitor pergunta: Ento essa tcnica no se aplica para todos os casos?!?!?. O autor responde: No se desesperem!, uma vez que possvel escrever r(n) da seguinte forma:
(**)r (n ) = A(n) C (n + 1) B ( n) C ( n)

onde A(n), B(n), C(n) so polinmios tais que: mdc{ A(n), B (n + h)} = 1, h + . No vamos demonstrar esse fato aqui (pois um dos exerccios deste artigo), mas (n + 4)(n + 3) vamos exibir um exemplo bem ilustrativo! Como escrever r (n) = da n 2 (n + 1) forma (**)? Note que:

EUREKA! N29, 2009

23

Sociedade Brasileira de Matemtica

r (n) =

1 n + 4 n + 3 1 n + 4 n + 3 n + 2 n + 1 n + 3 n + 2 A(n) C (n + 1) = = , n n n + 1 n n + 3 n + 2 n + 1 n n + 2 n + 1 B ( n) C ( n)

onde A(n) = 1, B(n) = n e C (n) = n( n + 1) 2 (n + 2) 2 (n + 3). De forma geral se uma raiz do denominador de r(n) e h uma raiz do numerador de r(n), onde h + , ento vale:
n ( h) n + h n + h 1 n + 1 T (n + 1) = K = n n + h 1 n + h 2 n T ( n)

Onde T (n) = (n + h 1)(n + h 2)L (n ) Vamos tentar usar a frmula (**) em r(n) y(n + 1) y(n) = 1
y ( n + 1) A(n) C ( n + 1) y (n) = 1 B (n) C (n) B (n 1) y (n), C ( n)

Como y(n) racional em n, podemos fazer a substituio y (n) = portanto:


A(n ) y (n + 1) B (n 1) y( n) = C (n) (***)

Agora o milagre acontece! Se y (n) uma funo racional que satisfaz (***), ento
y (n) um polinmio! Se y (n) =
P ( n) , com mdc{P (n), Q (n)} = 1, ento: Q( n)

A( n) P(n + 1)Q(n) B (n 1) P (n )Q(n + 1) = C (n )Q (n )Q( n + 1)

Logo: 1- Q(n) B (n 1)Q (n + 1) 2- Q(n + 1) A( n)Q( n) Q (n) A(n 1)Q( n + 1) Logo se raiz de Q , ento: 1- raiz de B(n 1) ou raiz de Q(n + 1)

EUREKA! N29, 2009

24

Sociedade Brasileira de Matemtica

2- raiz de A(n 1) ou raiz de Q(n 1) Como mdc{ A(n), B (n + h)} = 1, h + , Q (n) = 1. Vamos agora resolver a seguinte equao polinomial:
A(n ) y (n + 1) B( n 1) y( n) = C (n), onde y (n) = ad n d + ... + a0

Temos casos a considerar 1- deg A deg B 2- deg A = deg B e A B ( A o coeficiente lder de A): Nesses casos, pela equao (***), vale: deg C = deg y + max{deg a,deg b} d = deg C max{deg A,deg B}. 3- deg A = deg B = m e A = B = k Se d > 0, ento: (kn m + an m 1 + ...)(ad (n + 1) d + K + a0 )
(kn m + an m 1 + ...)( ad n d + (ad 1 + dad )n d 1 + K) (kn m + bn m 1 + ...)(ad n d + K + a0 ) = C (n)

[(a ad + k (ad 1 + dad )) (b ad + kad 1 )]n m + d 1 + K = C (n) ba Se (a b ) ad + kdad = 0 d = , mas se k (a b ) ad + kdad 0 deg C = m + d 1 d = deg C deg A + 1 Em todos os casos, possvel calcular o valor de d e uma vez calculado o valor de d, a equao polinomial se transforma em um sistema linear com d variveis que pode ser resolvido (quando possvel) usando tcnicas bsicas de lgebra linear.

(kn m + bn m 1 + ...)(ad n d + ad 1n d 1 + K) = C (n)

Exerccios: 1) Calcule os seguintes somatrios 1 1 a) n k =0 2 k + 5k 1 4 k 1 k 4 b) n k =0 2k k

EUREKA! N29, 2009

25

Sociedade Brasileira de Matemtica

c)

n 1 k =0

k 2ak

2) Prove que qualquer funo racional r(n), pode ser escrita como: A(n)C (n + 1) com mdc{ A(n), B (n + h)} = 1, h + e r ( n) = B (n )C (n) mdc{ A(n), C (n)} = mdc{B (n), C (n + 1)} = 1 (o leitor atento novamente notar que adicionamos duas novas restries). 3) Prove que a menos de multiplicaes por constantes, A(n), B(n) e C(n) so nicos no exerccio anterior (a forma acima chama-se forma cannica). 4) Dizemos que duas seqncias hipergeomtricas so similares, quando a razo dos duas uma funo racional e neste caso, escrevemos sn : tn . Prove: a) Se sn no constante, ento sn : sn , onde sn = sn +1 sn . b) Se sn e tn so hipergeomtricas, e sn + tn 0 ento sn + tn hipergeomtrica se e somente se sn : tn . (Esse resultado pode levar o leitor a indagar sobre a nossa suposio inicial do que seria uma frmula fechada ser algo bem restritivo).
(1) (2) (m) c) Se tn , tn ,..., tn so hipergeomtricas e vale

algum i, j com 1 i < j m. d) Se vale

m (i ) i =1 n

(i ) ( j) = 0, ento tn : tn para

n 1 k =0

(1) (2) ( m) zk = tn +tn + ... + tn , onde

(1) (2) (m) tn , tn ,..., tn

zn

so

hipergeomtricas, ento

n 1 k =0

zk hipergeomtrica! (Ufa! De fato nossa definio

de frmula fechada no to restritiva assim).

EUREKA! N29, 2009

26

Sociedade Brasileira de Matemtica

DOMINGO REGADO A REPUNITS


Valberto Rmulo Feitosa Pereira Cefetce Uned Cedro Nvel Iniciante

No final do ano de 2007 fui convidade pelo professor e amigo Onofre Campos, por quem tenho admirao, para ministrar aulas para um jovem que no podia se locomover, pelo valor que eu iria receber pelas aulas, pensei que o rapaz pertencia a uma famlia muito rica; grande foi a minha surpresa ao perceber justamente o contrrio: em segunda conversa com o supra-citado professor, soube da frgil situao financeira do jovem aluno, mas tambm, por outro lado, do seu incrvel potencial e de sua fora de vontade, fatos que me entusiasmaram em conhec-lo. Este aluno era Ricardo Oliveira, o qual havia conquistado duas medalhas de Ouro na OBMEP. No ltimo encontro que tive com Ricardo, em sua residncia, ainda promovido pelo projeto de iniciao cientfica, deparamo-nos com o seguinte problema: O inteiro A formado por 666 algarismos iguais a 3, e o nmero B por 666 algarismos iguais a 6. Que algarismos apareceram no produto AB? Enquanto Ricardo fazia uma atividade, eu folhava uma apostila que continha as colunas semanais Olimpada de Matemtica do jornal O Povo em parceria com o Departamento de Matemtica da UFC. Neste momento, vi o problema acima e falei: - Olha Ricardo que belo problema! Nesse instante Ricardo para sua atividade, l o problema e passa a resolvlo. Eu tambm caio na tentativa de resolv-lo, lembrei que:
111111...11 14243 =
n algarismos 1

10n 1 . 9

Minha soluo com o uso desta informao saiu; Ricardo sem esta informao errou por um algarismo. Expliquei a Ricardo minha soluo, percebemos que a informao que eu havia usado era importante. A aula continuou, mas ainda fiquei pensando como esta igualdade daria para resolver belos problemas. No dia seguinte tive uma conversa com meu amigo Secco, olmpico do Rio de Janeiro. Perguntei-lhe se conhecia problemas que em sua soluo usava esta
EUREKA! N29, 2009

27

Sociedade Brasileira de Matemtica

igualdade; Secco falou que conhecia e mais ainda: estes nmeros eram chamados de Repunits e indicou [4]. Com a dica de Secco e o entusiasmo de Ricardo, cataloguei cinco problemas da antiga coluna, os quais passaremos a resolver. Tambm apresentei aos meus alunos do projeto OBMEP 2008, realizado no Cefet, Uned de Cedro-Ce. 1. REPUNITS Os Repunits so nmeros que s tm algarismos 1, por exemplo: 11, 111, 1111, 11111, ... Estes nmeros podem ser escritos de outra forma, vejamos:
k k 678 678 999...9 1000...0 1 10k 1 111...1 = . 1 2 3= 9 = 9 9 k

A beleza destas informaes poder resolver problemas interessantes sem usar tcnicas sofisticadas. 2. EXEMPLOS Exemplo 1: O inteiro positivo n formado de k algarismos 9. Mostre que a soma de todos os algarismos de n2 igual a 9k. Demonstrao: Pelas hipteses temos
N = 999...9 123 = 9(111...1) = 9
k

10k 1 = 10k 1. 9

Calculemos N2 da seguinte forma: N 2 = N .N


N 2 = (999...9).(10k 1) N 2 = 999...9000...0 123 123 999...9 123
k k k

N = 999...98000...01 123 123


2 k 1 k 1

A soma dos algarismos : 9(k 1) + 8 + 1 = 9k .

EUREKA! N29, 2009

28

Sociedade Brasileira de Matemtica

Exemplo 2: Mostre que os nmeros 49, 4489, 444889, ..., obtidos colocando o nmero 48 no meio do nmero anterior, so quadrados de nmeros inteiros. Demonstrao: Vejamos as igualdades: 49 = 4.1.101 + 8.1 + 1
4489 = 4.11.102 + 8.11 + 1 444889 = 4.111.103 + 8.111 + 1 n temos: + 8.111...1 N = 444...488...89 123 { = 4.111...1.10 1 2 3 1 2 3 + 1.
n n 1 n n
n

De

modo

geral

10 1 Substituindo 11...11 na expresso acima ficamos: 1 2 3= 9 n


4 8 N = (10n 1).10n + (10n 1) + 1 9 9 4 4 8 8 N = 102 n 10n + 10n + 1 9 9 9 9 2.10n + 1 N = 3 n O nmero 2.10 + 1 mltiplo de 3, portanto N um quadrado perfeito.
2

Exemplo 3: Para cada inteiro positivo n, sejam A(n) e B(n) dois nmeros inteiros formados por 2n algarismos iguais a 1 e n algarismos iguais a 2 respectivamente. Mostre que A(n) B(n) um quadrado perfeito. Demonstrao. Pelas hipteses temos: A(n) B (n) = 111...1 1 2 3 222...2 123
2n n

10n 1 102 n 1 Como 222...2 = 2 e 111...1 = , substituindo teremos: 123 1 2 3 9 9 n 2n 102 n 1 10n 1 2 9 9 (10n 1)(10n + 1) 10n 1 A(n) B (n) = 2 9 9 n n (10 1)[(10 + 1) 2] A(n) B (n) = 9 (10n 1) 2 A(n) B (n) = 9 A(n) B (n) =

EUREKA! N29, 2009

29

Sociedade Brasileira de Matemtica

(10n 1)2 . 32 Assim A(n) B (n) quadrado perfeito. A(n) B (n) =

Exemplo 4: Sem efetuar a multiplicao, calcule o valor de (999.999.999) 2 . Soluo: Vamos escrever a expresso (999.999.999) 2 da seguinte maneira:
109 1 18 9 (999.999.999) = 9. = 10 2.10 + 1 = 1000...0 123 2 000...0 123 + 1 9 18 zeros 9 zeros fazendo as contas ficamos: (999.999.999) 2 = 999...98000...01 123 123
2 2

8 noves

8 zeros

Finalmente o problema motivdor do nosso trabalho. Exemplo 5: O inteiro A formado por 666 algarismos iguais a 3, e o nmero B por 666 algarismos iguais a 6. Que algarismos apareceram no produto AB? 10666 1 = 6.111...1 Soluo: Como A = 666...6 e B = 333...3 = 3.111...1 = 3. , vamos 1 2 3 1 2 3 1 2 3 1 2 3 9 666 666 666 666 calcular AB mas usando alguns artifcios, como segue abaixo: 10666 1 AB = 3.6.(111...1). 9 666 AB = 2.(111...1)(10 1)
AB = (222...2)(10666 1) AB = (222...2).10666 222...2 AB = 222...2 123 000...0 123 222...2 123
666 666 666

AB = 222...21777...78 123 123


665 665

Logo apareceram no produto AB: - Um algarismo 1; - Um algarismo 8; - 665 algarismos 2; - 665 algarismos 7.

EUREKA! N29, 2009

30

Sociedade Brasileira de Matemtica

3. PROBLEMAS PROPOSTOS 1. Achar a soma: 2 + 22 + ... + 222...2 se a ltima parcela tem n algarismos iguais a 2.
2 Prove que: 111...1 1 2 3 = 222...2 123 + (333...3) 1 2 3 . 2n n n

2. 3. 4. 5.

Prove que se 111...1 1 2 3 divisvel por 41 se e somente se n divisvel por 5.


n

Mostre que nenhum inteiro da seqncia: 11,111,1111,11111,... um quadrado perfeito. Mostrar que os inteiros: 1111,111111,..., cada um dos quais formado por um nmero par de algarismos 1, so compostos.

Nota dos editores: No difcil mostrar que se 111...1 1 2 3 primo ento n primo
n

(exerccio!) . Os nicos valores de n para os quais se sabe provar atualmente que 111...1 1 2 3 primo so 2, 19, 23, 317 e 1031. Recentemente (entre 1999 e 2007) foram
n

descobertos os seguintes valores de n tais que 111...1 1 2 3 provavelmente primo (i.e.,


n

passa por diversos testes probabilsticos de primalidade): 49081, 86453, 109297 e 270343. De acordo com os testes j realizados, qualquer outro repunit primo deve ter mais de 400.000 algarismos.

REFERNCIAS
[1] [2] [3] [4] Emanuel Carneiro, Francisco Antonio M. de Paiva, Onofre Campos, Olimpadas Cearenses de Matemtica do Ensino Fundamental, Edies Realce Editora e Indstria Grfica, Fortaleza, 2006. Alencar Filho, Edgar de, Teoria Elementar dos Nmeros, Nobel, So Paulo, 1988. Coluna Semanal Olimpadas de Matemtica, Jornal O Povo em parceria com o Departamento de Matemtica da UFC, No. 01, ao No. 200. Titu Andreescu, Razvan Gelca, Mathematical Olympiad Challenges, 2000.

EUREKA! N29, 2009

31

Sociedade Brasileira de Matemtica

HOMOTETIAS, COMPOSIO DE HOMOTETIAS E O PROBLEMA 6 DA IMO 2008


Carlos Yuzo Shine Nvel Avanado

Antes de comear a discusso, vamos enunciar o problema 6 da IMO 2008, que a motivao principal desse artigo. Problema 6, IMO 2008. Seja ABCD um quadriltero convexo cujos lados BA e BC tm comprimentos diferentes. Sejam w1 e w2 as circunferncias inscritas nos tringulos ABC e ADC, respectivamente. Suponhamos que existe uma circunferncia w tangente reta BA de forma que A est entre B e o ponto de tangncia, tangente reta BC de forma que C est entre B e o ponto de tangncia, e que tambm seja tangente s retas AD e CD. Prove que as tangentes comuns exteriores a w1 e w2 se intersectam sobre w. claro que um problema de geometria no pode ficar sem um bom desenho. razoavelmente difcil desenhar a figura do problema e sugerimos que o leitor tente faz-lo por conta prpria (dica: comece com o crculo w ). No se perca: queremos provar que o ponto Z est sobre a circunferncia w.

w O Z
2

C D w O2 K L A O1 B w1

Quem j estudou homotetia j deve ter enxergado diversas homotetias entre as circunferncias, mas muitos dos mais poderosos olmpicos do mundo foram derrotados por esse problema. De fato, dos 535 estudantes que participaram da IMO 2008, somente 13 resolveram (um deles fez 6 pontos) e 53 conseguiram pelo menos um ponto. Isto quer dizer que mais de 90% dos estudantes zeraram o problema!
EUREKA! N29, 2009

32

Sociedade Brasileira de Matemtica

Isso sinal de que esse problema deve ter algo novo para ser explorado. De fato, uma transformao geomtrica que esteve em voga nos anos 80 e desapareceu nos anos 90 foi a homotetia. E ela voltou, discretamente em 2007 e com tudo em 2008! Vamos definir homotetia, ver algumas de suas propriedades e expandir as adias envolvidas nessa transformao. 1. Homotetia: definio Voc vai ver que homotetia nada mais do que fazer sombrinha. Aparecem muitos paralelismos, mas o mais interessante so as colinearidades que iro aparecer. No incio parece mgica; mas um bom matemtico sempre revela seus truques! Vamos comear com a definio de homotetia com razo positiva ou homotetia direta: Definio 1.1. Homotetia de uma figura F com centro O e razo k, um nmero real positivo, uma transformao geomtrica que associa a cada ponto P de F o ponto P sobre a semi-reta OP, de origem O, tal que OP= k OP.

A F B B
Talvez com vetores seja mais interessante: sendo O o centro da homotetia, o ponto uuur uuu r P transformado no ponto P de modo que OP = k OP. Note que a homotetia uma funo que leva pontos do plano (ou do espao, se voc estiver trabalhando em dimenses maiores) a pontos do plano (espao). De fato, podemos fazer P= ( P ), tal que ( P) O = k ( P O) ( P) = O + k ( P O). Com isso, podemos definir homotetias para k negativo tambm, obtendo as chamadas homotetias de razo negativa ou homotetias inversas:
EUREKA! N29, 2009

33

Sociedade Brasileira de Matemtica

Definio 1.2. Homotetia de uma figura F com centro O e razo k, sendo k um nmero real negativo, uma transformao geomtrica que associa a cada ponto uuur uuu r P de F o ponto Psobre a reta OP, de origem O, tal que OP = k OP.

A C

B C A

2. Propriedades da homotetia As principais propriedades de homotetias tm a ver com colinearidade e concorrncia. Algumas tm a ver com paralelismo. 2.1. Colinearidade O centro de homotetia, o ponto e seu transformado so colineares. Em outras palavras, O, P e P= ( P ) so colineares. Isso decorre diretamente da definio, mas homotetias no vm de graa! Normalmente as encontramos nos problemas e, com essa propriedade, obtemos pontos colineares. 2.2. Concorrncia O centro de homotetia pertence a todas as retas que ligam pontos a seus transformados. Em outras palavras, O pertence a toda reta do tipo PP= P ( P). Novamente, uma propriedade que decorre diretamente da definio (na verdade, a mesma da colinearidade!), mas que aparece quando descobrimos alguma homotetia. 2.3. Paralelismo A reta que liga dois pontos paralela reta que liga os seus tranformados. Em outras palavras, PQ e PQ= ( P ) (Q ) so paralelas. A demonstrao desse fato vem da semelhana entre OPQ e OPQ (pelo caso LAL) .
EUREKA! N29, 2009

34

Sociedade Brasileira de Matemtica

Dois tringulos com lados respectivamente paralelos so homotticos. Para provar isso, sendo ABC e DEF os trigulos com AB, DE, AC, DF e BC, EF respectivamente paralelos, use o teorema de Desargues para provar que esses tringulos so perspectivos.

Em particular, algumas figuras so sempre semelhantes: os crculos! Com isso, temos a seguinte propriedade: 2.4. Crculos Dois crculos so sempre homotticos. Na maioria dos casos, eles admitem duas homotetias, uma direta e uma inversa. No caso de crculos disjuntos, os centros de homotetias so fceis de encontrar: so as intersees das tangentes comuns internas (inversa) e das tangentes comuns externas (direta).

O+

Com isso, podemos resolver alguns problemas. Homotetia esteve bastante na moda na IMO durante o incio dos anos 80, como voc vai ver nos exemplos e nos exerccios. Exemplo 2.1. Problema 5, IMO 1981. Trs crculos congruentes tm um ponto comum O e esto no interior de um tringulo. Cada crculo tangente a dois lados do tringulo. Prove que o incentro e o circuncentro do tringulo e o ponto O so colineares. Resoluo: O nome do ponto dado no O por acaso: sejam A, B e C os centros dos trs crculos congruentes e ABC o tringulo cujos lados tangenciam esses trs
EUREKA! N29, 2009

35

Sociedade Brasileira de Matemtica

crculos. Note que os raios dos crculos congruentes so OA = OB = OC , isto , O circuncentro de ABC. Alm disso, das tangncias dos crculos com os lados temos que AA, BB e CCso as bissetrizes do tringulo ABC e se interceptam no incentro I do tringulo.

C I A A O O B B

As distncias de A e B a AB so iguais aos raios dos crculos congruentes a so, portanto, iguais. Ento AB e AB so paralelos. Analogamente, AC paralelo a AC e BC paralelo a BC, de modo que os tringulos ABC e ABC so homotticos. O centro de homotetia I. Essa homotetia leva O ao circuncentro O de ABC. Assim, I, O e O so colineares. Note que a dificuldade foi achar a homotetia; depois bastou aplicar a propriedade de colinearidade. Exerccios: 01. (Problema 2, IMO 1982) Seja A1 A2 A3 um tringulo escaleno com lados a1 , a2 e a3 ( ai o lado oposto a Ai ). Seja M i o ponto mdio do lado ai e Ti o ponto onde o incrculo do tringulo toca o lado ai , para i = 1, 2, 3. Seja Si o simtrico de Ti em relao bissetriz interna do ngulo Ai . Prove que as retas M 1S1 , M 2 S2 e M 3 S3 so concorrentes.

EUREKA! N29, 2009

36

Sociedade Brasileira de Matemtica

02. (Problema 2, IMO 1983) Seja A um dos dois pontos de interseo dos crculos C1 e C2 , de centros O1 e O2 , respectivamente. Uma das tangentes comuns aos crculos toca C1 em P 1 e C2 em P 2 , e a outra toca C1 em Q1 e C2 em Q2 . Seja M 1 o ponto mdio de PQ o ponto mdio de P2Q2 . Prove que 1 1 e M2 O1 AO2 = M 1 AM 2 . 03. (Prova de seleo 2008, Banco da IMO 2007) As diagonais do trapzio ABCD cortam-se no ponto P. O ponto Q est na regio determinada pelas retas paralelas BC e AD tal que AQD = CQB e a reta CD corta o segmento PQ. Prove que BQP = DAQ. 3. O Fenmeno Homottico Circular Algumas aplicaes de certos teoremas so to conhecidos quanto os prprios. Para homotetias, o caso com o fenmeno homottico circular, que mostra uma colinearidade bastante interessante envolvendo incrculo e ex-incrculo. Fenmeno Homottico Circular. Seja ABC um tringulo e sejam K e L os pontos de tangncia do incrculo e ex-incrculo relativo a A em BC. Ento A, L e o ponto K diametralmente oposto a K no incrculo so colineares. Demonstrao:
A B K I B K L C C

IA

Basta traar a reta BC paralela a BC que tangencia o incrculo de ABC em K. ABC e ABC so homotticos com centro em A. Para terminar, o incculo de ABC ex-incrculo de ABC, de modo que os pontos Ke L so correspondentes na homotetia e esto, portanto, alinhados com A.
EUREKA! N29, 2009

37

Sociedade Brasileira de Matemtica

Vale a pena lembrar tambm que, na figura acima, BK = LC. Exerccios: 04 (Problema 4, IMO 1992) No plano, considere uma circunferncia C, uma reta L tangente circunferncia e M um ponto da reta L. Encontre o lugar geomtrico dos pontos P com a seguinte propriedade: existem dois pontos Q, R da reta L tais que M o ponto mdio de QR e C a circunferncia inscrita no tringulo PQR. 4. Composio de Homotetias A principal inovao na IMO 2008 no problema 6 foi explorar o seguinte fato: Composio de Homotetias. Se 1 uma homotetia de centro O1 e 2 uma homotetia de centro O2 ento a composio de homotetias = 2 o 1 uma homotetia de centro O, e O1 , O2 e O esto alinhados. A nica exceo quando a composio uma translao. Demonstrao Utilizaremos vetores para provar esse fato Seja P um ponto qualquer e sejam k1 e k2 as razes de homotetia de 1 e 2 , respectivamente. Ento 1 ( P ) = O1 + k1 ( P O1 ) e, portanto,
( P ) = 2 o 1 ( P ) = 2 ( 1 ( P )) = O2 + k 2 ( 1 ( P) O2 ) = O2 + k 2 (O1 + k1 ( P O1 ) O2 ) = k2 (1 k1 ) O1 + (1 k2 ) O2 + k1k2 P

(*)

Primeiro, se uma homotetia, ento sua razo k1k2 (as figuras so multiplicadas por k1 e depois por k2 ; ou seja, so multiplicadas por k1k2 ). Assim, para provarmos que uma homotetia, temos que provar que existe um ponto O tal que ( P ) = O + k1k2 ( P O ) = (1 k1k2 ) O + k1k2 P (**) Comparando os coeficientes em (*) e (**) conclumos que (1 k1k2 )O = k2 (1 k1 ) O1 + (1 k2 ) O2 . Se k1k2 = 1, uma translao k (1 k1 ) O1 + (1 k2 ) O2 (verifique!). Caso contrrio, e, como O= 2 1 k1k2 k2 (1 k1 ) + (1 k2 ) = k2 k1k2 + 1 k2 = 1 k1k2 , O uma mdia ponderada de O1 e O2 . Em outras palavras, O pertence reta O1O2 .
EUREKA! N29, 2009

38

Sociedade Brasileira de Matemtica

Os partidrios da geometria sinttica devem estar sentindo falta de uma demonstrao sinttica. Vamos provar a parte da colinearidade sinteticamente. Demonstrao sinttica da colinearidade Considere os pontos P e Q e seus P = ( P ), Q = ( Q ), P = ( P ) = ( P ) e Q = ( Q ) = (Q ). 2 2 1 1 1 1 1 2 2 1
O2 P1 P2

transformados

P O O1 Q Q1

Q2

Note que, das homotetias, PQ PQ 1 1 e P 2 Q2 so paralelos. Em termos projetivos, eles so concorrentes em um ponto do infinito. Isto quer dizer que os tringulos PPP 1 2 e QQ1Q2 so perspectivos e podemos aplicar o teorema de Desargues: as intersees entre lados correspondentes, PP 1 QQ1 = {O1}, P 1P 2 Q1Q2 = {O2 } e PP2 QQ2 = {O} so colineares. 4.1 Detalhe tcnico Geralmente, trabalhamos com homotetias sinteticamente, e aparecem homotetias diretas e inversas. Homotetias inversas multiplicam figuras por fatores negativos, de modo que a composio de duas homotetias do mesmo tipo direta e a composio de duas homotetias de tipos diferentes inversa. Para facilitar, a homotetia inversa faz o papel do sinal de menos e a homotetia direta, do sinal de mais. Na composio de homotetias, seguimos a regra dos sinais da multiplicao. Agora estamos prontos para resolver o problema 6 da IMO 2008. Vamos reenunciar o problema e resolv-lo. Exemplo 4.1 Problema 6, IMO 2008. Seja ABCD um quadriltero convexo cujos lados BA e BC tm comprimentos diferentes. Sejam w1 e w2 as circunferncias inscritas nos tringulos ABC e ADC, respectivamente. Suponhamos que existe um
EUREKA! N29, 2009

39

Sociedade Brasileira de Matemtica

circunferncia w tangente reta BA de forma que A est entre B e o ponto de tangncia, tangente reta BC de forma que C est entre B e o ponto de tangncia, e que tambm seja tangente s retas AD e CD. Prove que as tangentes comuns exteriores a w1 e w2 se intersectam sobre w. Resoluo Vamos comear trabalhando com segmentos tangentes.

G Z D

C O2 K w2 O1 w1 B

O H F

L A

Temos BE = BF, AF = AG, CE = CH e DG = DH. Ento AB = BF AF = BE AG = BC + CE (AD + DG) = BC AD + (CH DH) = BC AD + CD AB + AD = BC + CD. Note que esse fato depende somente de w ser tangente aos prolongamentos dos lados do quadriltero ABCD (guarde esse fato, ele pode ser til em outros problemas!). Isso implica
AC + CD AD AC + AB BC = CK = AL. 2 2

Essa igualdade simples, mas abre muitas portas para ns! De fato, ela quer dizer que os ex-incrculos w3 e w4 relativos a AC dos tringulos ABC e ADC tocam AC em K e L, respectivamente. Isso nos d muitas, mas muitas homotetias, e pelo menos duas oportunidades de utilizar o fenmeno homettico circular! Desenhemos as circunferncias:

EUREKA! N29, 2009

40

Sociedade Brasileira de Matemtica

G Z

C w4

w3 B

D O2 K O1 w2 L w1 A

H F

Vamos compor homotetias para descobrir colinearidades, utilizando w3 e w4 como intermedirios!


24 41 21 w2 w4 w1 e w2 w1. O centro K da homotetia (direta) 24 , o centro B da homotetia (direta) 41 e o centro Z da homotetia (direta) 21 esto alinhados. Isso quer dizer que Z pertence reta BK. 21 23 31 w2 w3 w1 e w2 1. O centro D da homotetia (direta) 23 , o centro L da homotetia (direta) 31 e o centro Z da homotetia (direta) 21 esto alinhados. Isso quer dizer que Z pertence reta DL.

Com isso, conclumos que Z a interseo de BK e DL. Note que at agora no envolvemos o crculo w nas homotetias. Agora hora, mas vamos provar colinearidades de outra forma. Seja W a interseo de BK e w. Provaremos que W = Z, resolvendo o problema.
r w E G Z O T O D w2 2 w4 K O1 L w1 A H F s C w3 B

EUREKA! N29, 2009

41

Sociedade Brasileira de Matemtica

Primeiro, note que a homotetia direta 4 que leva w4 a w tem centro B e, portanto, leva K a W. Mais ainda: como AC tangente a w4 em K, a reta r paralela a AC que passa por W tangente a w, pois a reta AC levada a r por 4 . Agora, considere a homotetia inversa 2 que leva w a w2 . Essa homotetia tem centro em D, leva W a T e r a s, que paralela a r e AC e tangente a w2 . Assim, D, W e T esto alinhados, ou seja, W pertence reta DT. Falta ainda identificar melhor o ponto T. Na verdade, ele bem conhecido: como s e AC so paralelos, T e K so diametralmente opostos. Podemos, assim, aplicar o fenmeno homottico circular: D, T e L so colineares e L tambm pertence reta DT. Portanto D, L e W so colineares, de modo que W pertence a DL. Como W pertence, por definio, reta BK, W a interseo de BK e DL, e s pode ser igual a Z. Observao: Note que a condio AB AC importante para que as retas BK e DL no coincidam. Exerccios 05. (Banco da IMO 2007) O ponto P pertence ao lado AB do quadriltero convexo ABCD. Seja w o incrculo do tringulo DPD e I o seu incentro. Suponha que w tangente aos incrculos dos tringulos APD e BPC em K e L, respectivamente. As retas AC e BD se encontram em E e as retas AK e BL se encontram em F. Prove que os pontos E, I e F so colineares. 06. (Romnia) Seja ABC um trigulo e wa , wb , wc crculos dentro de ABC tangentes exteriormente dois a dois, tais que wa tangente a AB e AC, wb tangente a AB e BC e wc tangente a AC e BC. Sejam D o ponto de tangncia entre wb e wc , E o ponto de tangncia entre wa e wc e F o ponto de tangncia entre wa e wb . Prove que as retas AD, BE e CF tm um ponto em comum. 07. (Ir) Sejam w e o incrculo e o circuncrculo do tringulo ABC. w toca BC, CA e AB em D, E e F respectivamente. Os trs crculos wa , wb e wc tangenciam w em D, E e F, respectivamente, e em K, L e M, respectivamente. (a) Prove que DK, EL e FM tm um ponto P em comum. (b) Prove que o ortocentro do tringulo DEF pertence reta OP.
EUREKA! N29, 2009

42

Sociedade Brasileira de Matemtica

08. Seja uma circunferncia e A, B e C pontos em seu interior. Construa as seguintes trs circunferncias: 1 tangente a , AB e AC; 2 tangente a , AB e BC; 3 tangente a , AC e BC. Sendo C1 , C2 e C3 os respectivos pontos de tangncia de 1 , 2 , 3 com , prove que AC1 , BC2 e CC3 passam por um mesmo ponto.

Que 3366127031232 + 1 primo? Esse foi o dcimo primeiro primo descoberto pelo projeto "seventeen or bust" e foi encontrado por Sturle Sunde em 17 de outubro de 2007. Isso mostra que 33661 no um nmero de Sierpinski (nmeros de Sierpinski so naturais mpares k tais que k 2n + 1 composto para todo n N; veja a Eureka! 18, pg. 61 e a Eureka! 25 pgina 56), reduzindo para 6 o nmero de naturais menores que 78557 (que o menor nmero de Sierpinski conhecido), sobre os quais no se sabe se so ou no nmeros de Sierpinski: 10223, 21181, 22699, 24737, 55459 e 67607. Veja www.seventeenorbust.com para mais informaes (inclusive sobre como participar do projeto).

Voc sabia

EUREKA! N29, 2009

43

Sociedade Brasileira de Matemtica

COMO QUE FAZ?


PROBLEMA PROPOSTO POR MARCEL MENEZES DE ANDRADE PRADO

Seja ABC um tringulo e P um ponto em seu interior tal que AP, BP e CP intersectam os lados BC, CA e AB nos pontos D, E e F, respectivamente. Se AP = a, BP = b, CP = c, PD = PE = PF = 3 e a + b + c = 43, determine abc.
SOLUO:

Escrevemos P em coordenadas baricntricas:

F=

t1 A + t2 B t1 + t2

E=

t1 A + t3C t1 + t3

t B + t3C D= 2 t 2 + t3

P = t , A + t2 B + t3C , com t1 , t 2 , t3 (0,1) tais que t1 + t 2 + t3 = 1. Devemos ter ento D=

t B + t3C t 2 B + t3 C t1 A + t3C (note que P = t1 A + (1 t1 ) 2 = t1 A + (1 t1 ) D ) , E = t2 + t3 t1 + t3 1 t1

t1 A + t2C . t1 + t2 Temos

e F=

1 t B + t3C 3 = PD = D P = 1 ( t2 B + t3C ) t1 A = t1 2 A = t1 D A = t 2 + t3 t2 + t3

EUREKA! N29, 2009

44

Sociedade Brasileira de Matemtica

3 3 . Analogamente, t2 = e a+3 b+3 3 3 3 3 t3 = . Como t1 + t 2 + t3 = 1, temos + + = 1, donde c+3 a+3 b+3 c+3 3 ( ( b + 3 )( c + 3) + ( a + 3)( c + 3) + ( a + 3)( b + 3) ) = ( a + 3)( b + 3 )( c + 3) , e logo
= t1 AD = t1 AP + PD = t1 (a + 3), donde t1 =

3 ( 6 a + 6 b + 6 c + 27 ) = abc + 9 a + 9 b + 9 c + 27, e finalmente abc = 9 ( a + b + c ) + 54 = 9 43 + 54 = 441.

PROBLEMA PROPOSTO POR WILSON CARLOS DA SILVA RAMOS

Determine todas as funes F : ( 0, + ) que sejam derivveis em x = 1 e que satisfaam: F ( xy ) = xF ( y ) + yF ( x), x, y (0, +).
SOLUO:

Fazendo y = x, obtemos F ( x 2 ) = 2 xF ( x), x + (0, +). Com x = 1 temos F (1) = 2 F (1), donde F (1) = 0. Vamos mostrar por induo que
F ( x ) = 2 x F ( x), para todo inteiro positivo k e todo x (0, +). Para k = 1 isso a afirmao anterior e, supondo que isso vale para um certo k,
2k k 2k 1 2

F ( x 2 ) = F (( x 2 )2 ) = 2 x 2 F ( x 2 ) = 2 x 2 como queramos mostrar.


k k k

k +1

2k x
k

2k 1 2

F ( x) = 2k +1 x

2k +1 1 2

F ( x),

Fazendo x = e
k k +

h 2k

, obtemos F (e ) = 2 e
h k

2k 1 2k +1

F (eh 2 ), k 1.
h h2 k 2k

Como lim 2 e

2k 1 h 2k +1

F(e

h 2k

) = e lim2 F (e
h2 k k

h 2k

) = e lim2 (e 1)lim
k k

F(eh 2 ) F(1)
k

eh 2 1
k

= he h 2 F (1), temos F (e h ) = che h 2 , onde c = F (1), para todo h > 0.

Fazendo x = e h , obtemos F ( x) = c(log x) x , x (0, +). Todas as funes desse tipo satisfazem a equao funcional. De fato, F ( xy ) = c ( log x + log y ) xy = xF ( y ) + yF ( x), x, y (0, +). Resolvemos a seguir, a pedido de Mauro Felix de Sousa, trs problemas da seo Olimpadas ao redor do mundo, propostos nas Eureka! No. 9 e 10.

EUREKA! N29, 2009

45

Sociedade Brasileira de Matemtica

47 (Ir 1999)

Determine todas as funes f : que satisfazem


SOLUO:

f ( f ( x) + y ) = f ( x 2 y ) + 4 f ( x) y para todos os nmeros reais x e y.

Fazendo x = y = 0, obtemos f ( f (0)) = f (0). Fazendo x = 0, y = f (0), obtemos f (0) = f ( f (0)) 4 f (0)2 , donde 4 f (0) 2 = 0, e logo f (0) = 0. Fazendo agora y = 0, obtemos f ( f ( x )) = f ( x 2 ), x . Se f ( a ) = f (b ) com a b, teremos donde, se t = b 2 a 2 , f ( z + t ) = f ( z ), z . Note agora que, se t 0, fazendo y = t obtemos f ( f ( x )) = f ( f ( x ) + t ) =

f (b 2 y ) = f ( f (b ) + y ) 4 f (b ) y = f ( f ( a ) + y ) 4 f ( a ) y = f ( a 2 y ), y ,

donde 4 f ( x)t = 0, x , e logo f ( x) = 0, x , o que uma soluo (claramente). Por outro lado, fazendo x = 0 na equao funcional, obtemos f ( y ) = f ( y ), y . Assim, se f no identicamente nula,
f (a ) = f (b) a = b ou a = b. Em particular, como f ( f ( x)) = f ( x 2 ), x ,
x

= f ( x 2 t ) + 4 f ( x )t = f ( x 2 ) + 4 f ( x )t = f ( f ( x )) + 4 f ( x )t , x ,

para todo

devemos ter

f ( x) = x 2

ou

f (x) =x2. Se para algum

donde 4x2 y = 4 f (x) y = 0,y , absurdo. Assim, devemos ter f ( x ) = x2 , x . Isso outra soluo, pois, de fato, (x2 + y)2 = (x2 y)2 + 4x2 y,x, y .
71 (Belarus 2000)

x 0, f ( x) = x2 , temos f (x2 y) = f ( y x2 ) = f ( f (x) + y) = f (x2 y) + 4 f (x) y,y ,

Determine todos os pares de inteiros positivos (m, n) que satisfazem a equao (m n)2 (n 2 m) = 4m 2 n.
SOLUO:

Seja d = mdc(m, n). Temos m = da, n = db, com mdc(a , b) = 1, e a equao equivale a (a b) 2 (db 2 a ) = 4a 2b. Como mdc(b, a b) = 1, segue que b | db 2 a, donde b | a, e logo b = 1. Temos ento (a 1) 2 (d a) = 4 a 2 . Como mdc(a, a 1) = 1 e (a 1) 2 | 4a 2 , segue que (a 1) 2 | 4, donde a 1 = 1 ou a 1 = 2. No primeiro caso, temos a = 2, d a = 16, donde d = 18, e no segundo a = 3, d a = 9, donde d = 12. Assim, as duas solues so (m, n) = (36, 18) e (m, n) = (36, 12).
EUREKA! N29, 2009

46

Sociedade Brasileira de Matemtica

89 (Balcnica 2000)

Determinar todas as funes


2

f ( x f ( x) + f ( y )) = f ( x) + y, para todos os nmeros reais x e y.

f :

que possuem a propriedade:

Fazendo x = 0, obtemos f ( f ( y )) = f (0) 2 + y, y , donde f uma bijeo, e logo existe a tal que f(a) = 0. Fazendo x = a, obtemos f ( f ( y )) = y, y , e logo f (0) = 0. Fazendo y = 0 obtemos ento f ( xf ( x)) = f ( x)2 , x , donde
xf ( x) = f ( f ( x) 2 ), ou seja, f ( z 2 ) = zf ( z ), z (basta fazer x = f(z), donde f ( x) = z ). Aplicando f dos dois lados da equao obtemos f ( f ( x) 2 + y ) = f ( f ( xf ( x) + f ( y ))) = xf ( x) + f ( y ) = f ( f ( x)2 ) + f ( y ),

SOLUO:

donde,

fazendo

f ( x) = t 0, obtemos
2

que f sobrejetiva, donde t = f ( x) pode assumir qualquer valor nonegativo).


2

f ( y + t ) = f ( y ) + f (t ), y , t 0 (lembramos

Assim, 2 f ( x) + f (1) = f (2 x + 1) = f (( x + 1)2 x2 ) = f (( x + 1)2 ) f ( x2 ) = = ( x + 1) f ( x + 1) xf ( x) = xf (1) + f ( x) + f (1), x , donde f ( x) = xf (1), x . .

Como f (1)2 = f ( f (1)) = 1, isso implica que as nicas solues so f ( x) = x, x e f ( x) = x, x ( fcil ver que essas funes satisfazem o enunciado).

Voc sabia
Que 243112609-1 e 237156667-1 so primos? Eles tm 12978189 e 11185272 dgitos respectivamente e so os dois maiores primos conhecidos no momento. Foram descobertos em 23 de agosto de 2008 e 6 de setembro de 2008 por Edson Smith e Hans Michael Elvenich respectivamente, dois participantes do GIMPS. O GIMPS um projeto cooperativo na internet que j encontrou 12 primos de Mersenne. Veja www.mersenne.org para mais informaes, inclusive como ajudar a achar outros primos de Mersenne.

EUREKA! N29, 2009

47

Sociedade Brasileira de Matemtica

OLIMPADAS AO REDOR DO MUNDO


Neste nmero apresentamos algumas solues enviadas pelos leitores da nossa seo. Bruno Holanda Carlos Augusto David Ribeiro Obs. Nas Eurekas 25 e 27 apareceram, por descuido na edio, problemas na seo Olimpadas ao redor do mundo com numerao repetida. Nesses casos, procuraremos mencionar o exemplar em que o problema foi publicado, ao nos referirmos a um desses problemas. 224. (Balcnica Jnior 2007) Eureka! No. 27 Seja a um real positivo tal que a 3 = 6 ( a + 1) . Prove que a equao x 2 + ax + a 2 6 = 0 no possui soluo real.
SOLUO DE JOS DO NASCIMENTO PANTOJA JNIOR (FORTALEZA CE)

6 + 6, e como a > 0 conclumos que a 2 > 6, da: a a ( a 2 6 ) = 6 < a 2 a 2 a 6 < 0, ou seja, o valor de a tal que: a 3 = 6a + 6 a( a 2 6) = 6 a 2 =
2 = 1 12 4 1 ( 6 ) 2 1 a > 6 > 2, fica: <a< 1 + 12 4 1 ( 6 ) 2 1 = 3, e como j sabamos que

6 < a < 3.

Substituindo na equao em x,
a a 2 4 1

(a

6 ) por

(6 / a ),

teramos como razes

6 24 a x= , mas estas razes no so reais, pois a 2 < 0, devido ao 2 a fato de a 3 = 6 ( a + 1) < 6 ( 3 + 1) = 24.

226. (Inglaterra 2007) Eureka! No. 27 Seja ABC um tringulo acutngulo com AB > AC e BAC = 60. Seja O o circuncentro e H o ortocentro. A reta OH encontra AB em P e AC em Q. Prove que PO = HQ.
EUREKA! N29, 2009

48

Sociedade Brasileira de Matemtica

SOLUO DE EMERSON RAMOS BARROSO (FORTALEZA CE)

Q C

O P

Seja M o ponto mdio de BC. Sabemos inicialmente que AH = 2OM e que


QAH = OAP

(1)

(2)

Temos que BOC = 2BAC = 120 BOM = MOC = 60 OM = OB cos 60 = AO cos 60 2OM = AO, e por (1): AO = AH (3) AHO = AOH e como AHO e AOH so ngulos externos dos tringulos AQH e AOP, temos HQA + QAH = OPA + PAO e por (2): AQH = APO Logo o trigulo AQP issceles, onde AP = AQ. Tambm por (2) e (3), temos que os tringulos APO e AQH so congruentes (A.L.A.) e assim: PO = HQ, como queramos demonstrar. 229. (Bielorssia 2001) Eureka! No. 27 No losango ABCD, A = 60 . Os pontos F, H e G esto sobre os segmentos AD, CD e AC de modo que DFGH um paralelogramo. Prove que FBH um tringulo equiltero.

EUREKA! N29, 2009

49

Sociedade Brasileira de Matemtica

SOLUO DE ANTONIO MARCOS S. ALMEIDA (MANAUS AM)

A 30 l

l F b

B ? l

? G

60 a b l H

30 30 b C

DBF . Portanto BH BF e CBH CBH + H BD H BD + DBF H BF = 60. Desta forma Observe que CBD conclui-se que o tringulo BFH equiltero, pois issceles e tem o ngulo do vrtice de 60.

H CG e o tringulo Sendo AD // GH , pois DFGH paralelogramo, temos CGH HCG issceles, logo HC = GH = b. Mas FD = b, logo os tringulos BCH e = 60, j que ABCD losango. BDF so congruentes, pois BC = BD = l e F DB

230. (Rssia 2007) Eureka! No. 27 Sejam a, b, c nmeros reais. Prove que pelo menos uma das trs equaes x 2 + (a b) x + (b c) = 0,
x 2 + (b c ) x + ( c a ) = 0 x 2 + (c a ) x + ( a b ) = 0

possui soluo real.


SOLUO DE JEAN PIERRE YOUYOUTE (RIO DE JANEIRO - RJ)

Analisaremos o caso em que as duas primeiras equaes no tem soluo real e provaremos que a terceira possui soluo real. Os demais casos so anlogos. 1 = (a b) 2 4(b c) < 0 (a b) 2 + (b c) 2 < 4(b a ). 2 2 = (b c) 4(c a ) < 0
EUREKA! N29, 2009

50

Sociedade Brasileira de Matemtica

Como a, b, c so reais, ento: 0 < (a b)2 + (b c)2 < 4(b a ) 4(b a) > 0 4(a b) < 0
3 = (c a) 2 4(a b) 3 > 0 (c a )2 > 4(a b). Mas (c a )2 0 > 4(a b) (c a ) 2 > 4(a b). Portanto, a terceira equao possui soluo real.

235. (Olimpada Checa e Eslovaca 2007) Eureka! No. 27 Se x, y, z so nmeros reais no intervalo (1, 1) satisfazendo xy + yz + zx = 1, mostre que
6 3 (1 x 2 )(1 y 2 )(1 z 2 ) 1 + ( x + y + z ) 2
SOLUO DE JEAN PIERRE YOUYOUTE (RIO DE JANEIRO - RJ)

1 + ( x + y + z ) 2 6 3 (1 x 2 )(1 y 2 )(1 z 2 ) 1 + x 2 + y 2 + z 2 + 2( xy + yz + zx) 6 3 (1 x 2 )(1 y 2 )(1 z 2 ) 3 + x 2 + y 2 + z 2 6 3 (1 x 2 )(1 y 2 )(1 z 2 ). Aplicando a desigualdade das mdias obtemos: (1 + x)(1 y ) + (1 + y )(1 z ) + (1 + z )(1 x) 3 3 (1 x 2 )(1 y 2 )(1 z 2 ) 1 + x x xy + 1 + y z yz + 1 + z x zx 3 3 (1 x 2 )(1 y 2 )(1 z 2 ) 3 ( xy + yz + zx) 3 3 (1 x 2 )(1 y 2 )(1 z 2 ) 3 + 1 6 3 (1 x 2 )(1 y 2 )(1 z 2 ).

Mas x 2 + y 2 + z 2 xy + yz + zx x 2 + y 2 + z 2 1. Logo,
3 + x 2 + y 2 + z 2 3 + 1 6 3 (1 x 2 )(1 y 2 )(1 z 2 ) 3 + x 2 + y 2 + z 2 6 3 (1 x 2 )(1 y 2 )(1 z 2 ).

EUREKA! N29, 2009

51

Sociedade Brasileira de Matemtica

SOLUES DE PROBLEMAS PROPOSTOS !


113. Publicamos aqui algumas das respostas enviadas por nossos leitores.

a1 , a2 , a3 ,... formam uma seqncia de inteiros positivos menores que 2007 a + an inteiro, para quaisquer inteiros positivos m, n. tais que m am + n
Prove que a seqncia (an) peridica a partir de um certo ponto.
SOLUO DE ZOROASTRO AZAMBUJA NETO (RIO DE JANEIRO RJ)

Seja c < 2007 o maior inteiro positivo tal que a seqncia (an )r tem infinitos termos iguais a c. Existe n0 tal que an c, n > n0 . Note que, se N , n so tais que n0 < N n < N e aN = an = c, ento, como c = aN = a( N n)+n divide aN n + an = aN n + c, segue que c | aN n e, como aN n c, devemos ter aN n = c. Precisaremos agora do seguinte Lema: Para quaisquer inteiros positivos b1 , b2 ,..., bk , existe m0 tal que qualquer mltiplo de d = mdc(b1 , b2 ,..., bk ) que seja maior que m0 pode ser escrito como r1b1 + r2b2 + ... + rk bk com r1 , r2 ,..., rk . Prova: Vamos usar induo em k. Para k = 1 o resultado bvio. 0 tal que, se d = mdc(b , b ,...b ), ento todo mltiplo de Suponhamos que exista m 1 2 k d maior que m 0 se escreva como r b + ... + r b , com r , i k . Sejam agora
1 1 k k i

donde, pela hiptese de induo, existem r1 , r2 ,..., rk com (tx + q b ) = r b + r b + ... + r b , d e, definindo rk +1 = s , teremos k +1 1 1 2 2 k k t d = r1b1 + r2b2 + ... + rk bk + rk +1bk +1 , o que termina a prova do Lema. Sejam agora X = {n | an = c}, e d o mximo divisor comum dos elementos de X. Existem b1 , b2 ,..., bk X com mdc(b1 , b2 ,..., bk ) = d , e existe m0 tal que todo mltiplo de d maior que m0 se escreve como r1b1 + r2b2 + ... + rk bk com
EUREKA! N29, 2009

+s e 0 s<d . q, s com ty = q d (tx + q b ) + b (ty q d ) = d (tx + q b ) + b s. t d = d Temos ento k +1 k +1 k +1 k +1 (tx + q b ) = td b s > m b s > m b d =m 0, Temos s , e d
k +1 k +1 0 k +1 0 k +1

,b ) e m = m 0 + b d d = mdc(b1 , b2 ,..., bk , bk +1 ) = mdc( d k +1 k +1 . Sabemos que d pode ser 0 + b y, com x, y . Se t d = d (tx) + b (ty ) > m , tomamos escrito como dx
k +1 k +1 0

52

Sociedade Brasileira de Matemtica

> n + m tal que a = c, temos N mltiplo de d, donde N n >m Tomando N 0 0 N

r1 , r2 ,..., rk , pelo Lema. Afirmamos que, se se n > n0 mltiplo de d ento an = c. De fato, como vimos anteriormente, se N > N bi > n0 e aN = c ento aN bi = c, para todo i k .

tambm mltiplo de d, e logo se escreve como r1b1 + r2 b2 + ... + rk bk , com r1 , r2 ,..., rk . Aplicando a observao acima vrias vezes, conclumos que an = aN r1b1 r2b2 ,...,rk bk igual a c, o que mostra nossa afirmao. Seja agora n > m0 . Seja t inteiro positivo tal que n + t mltiplo de d. Temos ento que c = an + t divide an + at , e, analogamente, c = an + d + t divide
an + d + at , donde c divide ( an + d + at ) ( an + at ) = an + d an . Como an e an + d so

inteiros positivos menores ou iguais a c, devemos ter an + d = an . Assim, an + d = an para todo n > n0 , o que prova o resultado.
115. Suponha que ABC um tringulo com lados inteiros a, b e c com

e mdc(a, b) = mdc(a , c) = mdc(b, c) = 1.

= 60 BCA

Prove que c 1(mod 6) .


SOLUO DE EDEL PREZ CASTILLO (PINAR DEL RIO CUBA)

c 2 = a 2 + b 2 2ab cos(60) = a 2 + b 2 ab Demonstremos que c mpar. Como mdc(a , b) = 1 ento a mpar ou b mpar. Suponhamos que b mpar. Se a par ou mpar ento c mpar. Demonstremos que c no divisvel por 3. Se 3 | c 9 | c 2 ento c 2 = a 2 + b 2 ab = (a + b) 2 3ab
3 | ( a + b ) 3 | ( a + b ) 9 | ( a + b ) 9 | 3ab 3 | ab
2 2

Ento 3 | a ou 3 | b, o que mdc(a , b) = mdc(a , c) = mdc(b, c) = 1. Logo c no divisvel por 3. Demonstremos que c 1(mod 3).

em

qualquer

caso

contradiz

que

Os nmeros ( 2c 2a + b ) e ( 2c + 2a b ) so mpares porque b mpar.


EUREKA! N29, 2009

c2 = a 2 + b2 ab 4c2 = 4a2 4ab + 4b2 = ( 2a b) + 3b2 ( 2c 2a + b)( 2c + 2a b) = 3b2 .


2

53

Sociedade Brasileira de Matemtica

Seja d um divisor comum de ( 2c 2a + b ) e ( 2c + 2a b ) . Ento d | 4c e d | 2(a b) porm como d mpar temos que d | c e d | (2a b). J demonstramos que mcd ( c,3) = 1 ento mcd (d ,3) = 1. Da equao 4c 2 = ( 2a b ) + 3b 2 deduzimos que d 2 | 3b 2 d 2 | b 2 d | b d = 1
2

(porque d | c e mcd ( b, c ) = 1 ).
2c 2 a + b = 3 x 2

Como mcd ( 2c 2a + b , 2c + 2a b ) = 1 e seu produto 3b 2 temos que:


2c + 2 a b = y 2

Ou tambm poderia ser:


2c 2 a + b = y 2
2c + 2 a b = 3 x 2 Os valores de x, y so mpares porque b mpar.

Em qualquer caso temos que c = ( 3 x 2 + y 2 ) 4


c y =
2

c y 2 0(mod 3) c y 2 (mod 3) 4 Logo c 1(mod3), porque c no divisvel por 3.

3( x2 y2 )

Como c 1(mod3) ento c 1(mod 6) ou c 4(mod 6), porm c mpar, logo c 1(mod 6).
119.

Mostre que no existem inteiros 36 a + b )( 36b + a ) seja uma potncia de 2. (

positivos

tais

que

SOLUO DE MARCILIO MIRANDA DE CARVALHO (TERESINA PI)

( 36a + b ) e ( 36b + a ) so potncias de 2 maiores que 36. Da ( 36a + b ) e so ( 36b + a ) so pares, logo a e b so pares. Agora seja S o conjunto de pares (a, b) tais que ( 36a + b )( 36b + a ) seja
Se uma potncia de 2, ento uma potncia de 2. Se S no vazio ento S possui um par (a, b) tal que a mnimo. Como a e b so pares ento a = 2c e b = 2d, logo 4 ( 36c + d )( 36d + c ) uma potncia de 2, da ( 36c + d )( 36d + c ) uma potncia de 2, onde c < a, absurdo!
EUREKA! N29, 2009

( 36a + b )( 36b + a )

54

Sociedade Brasileira de Matemtica

120. Sejam a, b, c nmeros reais e soma Sn definida como Sn = a n + bn + cn , para

qualquer n inteiro no negativo. Sabe-se que S1 = 2, S2 = 6 e S3 = 14. Mostre que


2 Sn 1 Sn +1 = 8 para todo inteiro n > 1. Sn

SOLUO DE MARIA CLARA MENDES SILVA (PIRAJUBA MG)

Vamos definir como:


1 = a + b + c 2 = ab + ac + bc 3 = abc

Veja que: an + bn + c n = ( a + b + c ) ( an1 + bn1 + c n1 ) ( ab + bc + ac ) ( an 2 + bn 2 + cn2 )


+ abc ( a n 3 + bn 3 + c n 3 ) .

Isso pode ser verificado diretamente abrindo os produtos. Substituindo com a nossa notao: Sn = 1 Sn 1 2 S n 2 + 3 Sn 3 . Assim podemos definir Sn por recorrncia. Falta descobrir 1 , 2 e 3 . Inicialmente note que S1 = a + b + c = 2. Assim 1 = 2. Elevando a + b + c ao quadrado:
4 = ( a + b + c ) = a 2 + b2 + c 2 + 2 ( ab + bc + ac ) = S 2 + 2 2 = 6 + 22 .
2

Resolvendo achamos 2 = 1. Finalmente: 14 = a 3 + b3 + c 3 = ( a + b + c ) ( a 2 + b2 + c 2 ab ac bc ) + 3abc =


= 2 (6 + 1) + 33 = 14 + 3 3 3 = 0. Na equao de recorrncia que tnhamos obtido anteriormente: Sn = 2 S n 1 + S n 2 . Agora usaremos induo em n: Para n = 2, o que queremos demonstrar claramente verdadeiro, uma vez que S2 2 S1S3 = 62 28 = 8. Suponhamos que essa propriedade seja verdadeira para todo natural menor que um certo n, queremos provar que tambm vale para n. 2 2 2 2 Sn S n 1 S n +1 = Sn S n 1 ( 2 Sn + Sn 1 ) = Sn 2S n 1 Sn S n 1 =
EUREKA! N29, 2009

55

Sociedade Brasileira de Matemtica


2 = Sn 1 S n ( 2 S n 1 S n ) .

Como 2 S n 1 + S n 2 = S n , 2 S n 1 Sn = Sn 2 . Substituindo: 2 2 Sn S n 1 S n +1 = S n 1 + S n S n 2 . Essa expresso tem o mesmo mdulo do que a obtida se trocarmos todos seus 2 sinais, ou seja Sn 1 S n 2 S n . Mas pela hiptese de induo o mdulo desta 8.
2 Assim, o mdulo de Sn S n 1S n +1 8. Por induo provamos que essa sentena vlida para todo n inteiro positivo.

121. Na figura abaixo o lado do quadrado vale 4, obter o valor da altura h para que a

rea da regio 1 seja igual a rea da regio 2.


4 1 4 3 h 2 4

4
SOLUO DE BRUNO SALGUEIRO FANEGO (VIVEIRO ESPANHA)

A unio das regies 1 e 3 um quadrante de um semicrculo de raio 4 (de rea


42 ) e a unio das regies 2 e 3, um retngulo de lados 4 e h (de rea 4 h), 4 Ai a rea da regio i, 1 i 3, teremos donde, sendo A1 = A2 A1 + A3 = A2 + A3 42 = 4h h = . 4

122. Dado um tringulo ABC tal que AB = AC = a + b e BC = a , traa-se uma

ceviana partindo de B determinando em AC um ponto D tal que DA = a e DC = b . = 10 , determine os ngulos internos desse tringulo. Sabendo que ABD

EUREKA! N29, 2009

56

Sociedade Brasileira de Matemtica

SOLUO DE CARLOS ALBERTO DA SILVA VICTOR (NILPOLIS RJ)


A ? D

AD = BC = a AB = AC

10 B C

Tomando BAC = e, usando que AD = BC e a lei dos senos nos tringulos ABD e BDC, encontramos: sen10 = 2 sen sen ( + 10 ) . 2 Observe que = 20 soluo. Vamos mostrar que ela nica para o tringulo em questo. No difcil de verificar que < 80, pois no poderamos ter AD = BC para
80 (BC ser o maior lado); conseqentemente, + 10 < 90, e sen e 2 sen( + 10) sero crescentes no primeiro quadrante.

I) Suponha que > 20 sen10 = 2sen sen ( + 10 ) > 2sen10 sen30 = sen10 2 (absurdo). II) Suponha que < 20 sen10 = 2 sen sen ( + 10 ) < 2 sen10 sen30 = sen10 2 (absurdo). Concluso: = 20 a nica soluo.
127. Determine todos os inteiros positivos k tais que existem inteiros positivos x, y,

z com

x2 + y 2 + z 2 = k. xyz

SOLUO DE EDEL PREZ CASTILLO (PINAR DEL RIO CUBA)

Se x = y = z = 1 obtemos k = 3; se x = y = z = 3 obtemos k = 1. Falta provar que estes so os nicos valores que pode tomar k. Provemos que k deve ser mpar.

EUREKA! N29, 2009

57

Sociedade Brasileira de Matemtica

Seja S = {( x, y , z ) (*)3 :

x2 + y 2 + z 2 = k onde k par} xyz

Provemos que este conjunto vazio. Se no vazio existem x0 , y0 , z0 S tais que x0 + y0 + z0 mnimo.
2 2 2 Temos x0 + y0 + z0 = kx0 y0 z0 . Como o membro direito par , h dois casos: (1) Um dos nmeros par e os outros dois nmeros so mpares. (2) Os trs so pares. 2 2 2 No caso (1), se x0 par, y0 , z0 mpares, ento x0 + y0 + z0 2(mod 4) e no outro membro temos que kx0 y0 z0 0(mod 4) porque k e x0 so pares. Contradio. No caso (2) x0 = 2 x1 , y0 = 2 y1 , z0 = 2 z1 facilmente comprovamos que:
2 2 2 x1 + y1 + z1 = 2k , logo ( x1 , y1 , z1 ) S . x1 y1 z1

x1 + y1 + z1 = ( x0 + y0 + z0 ) 2 < x0 + y0 + z0 .

Isto absurdo porque x0 + y0 + z0 mnimo. Logo o conjunto S vazio. Provemos que k 3, o que resolve o problema. Seja C = {( x, y , z ) (*)3 :
x2 + y 2 + z 2 = k 4} xyz

Provemos que este conjunto em C vazio. Se no vazio existem x0 , y0 , z0 tais que x0 + y0 + z0 mnimo. Podemos supor que x0 y0 z0 . Ento:
x0 y z + 0 + 0 4 y0 z0 x0 z0 x0 y0

Mas

x0 y0 z 1 e 1, donde temos que 1 + 1 + 0 4, o que implica que y0 z0 x0 z0 x0 y0

z0 2 x0 y0 .
2 2 Consideremos a equao t 2 kx0 y0 t + x0 + y0 = 0 que tem a z0 como uma das suas solues. Seja z1 a outra soluo da equao; ento temos: (1) z0 + z1 = kx0 y0 2 2 (2) z0 z1 = x0 + y0 De (1) deduzimos que z1 inteiro, de (2) deduzimos que z1 positivo. Logo ( x0 , y0 , z1 ) C mas x0 + y0 + z0 x0 + y0 + z1 porque x0 + y0 + z0 mnimo.

EUREKA! N29, 2009

58

Sociedade Brasileira de Matemtica


2 2 2 2 2 Ento 2 x0 y0 z0 z1 , e substituindo em (2) obtemos 4 z0 y0 x0 + y0 2 y0 donde se 2 obtm que 4 x0 2 , o que absurdo. Portanto o conjunto C vazio.

128. Barango Joe era um sapo de mtiplos talentos que habitava a Terra das

Chances Diminutas, localizada no alto de uma montanha. Aps sua maioridade, Barango Joe decidiu tentar a vida no Reino das Grandes Oportunidades, localizado no cume da montanha vizinha. Para isso, ele atravessaria a extensa ponte de madeira por cima do Desfiladeiro da Morte. Entretanto, a ponte era guardada pela Esfinge Vegas, exmia jogadora que sempre desafiava os viajantes para algum jogo. O viajante vitorioso tinha a passagem franqueada; e o perdedor era lanado ao abismo. Assim chegando cabeceira da ponte, Barango Joe foi desafiado a uma partida de Pachang jogo que lembra o Black Jack ou Vinte e um, mas jogado por 2 oponentes da seguinte maneira: Os jogadores, designados por banca e apostador, utilizam um dado gerador de nmeros aleatrios reais uniformemente distribudos no intervalo [0,1]. Inicialmente, a banca sorteia um nmero X. Se no estiver satisfeita com o nmero obtido, pode descart-lo e ento sortear um novo nmero. Este procedimento pode ser executado 2 vezes, Isto , pode haver at 3 sorteios na definio do nmero X da banca. Ento, o apostador sorteia quantos nmeros forem necessrios at que a soma de seus nmeros ultrapasse o nmero X da banca. Neste momento, se esta soma for inferior a 1, o apostador ganha; caso contrrio, perde. Ou seja, para ganhar, o apostador precisa chegar mais prximo de 1 que a banca, sem no entanto estourar o limite de 1. Aps explicar as regras do Pachang, a Esfinge Vegas deu uma opo ao sapo: - Voc prefere ser a banca ou o apostador? O que o Barango Joe deveria responder? Obs. Utilize lpis, papel, e uma calculadora cientfica simples.
SOLUO DE RAFAEL TUPYNAMB DUTRA (BELO HORIZONTE MG)

Primeiramente, supondo que X j tenha sido escolhido, vamos determinar a funo f ( X ) que d a probabilidade de o apostador ganhar. Seja pn ( x) a funo densidade de probabilidade para a soma dos nmeros do apostador (ou seja, aps n nmeros sorteados pelo apostador, a probabilidade de a soma dos n 1 ser menor ou igual a X e a soma desses nmeros estar entre a e b dada por

p ( x)dx ). Definimos
n a
EUREKA! N29, 2009

qn ( x) igual a pn ( x) para x [0, X ] e qn ( x) igual a 0

59

Sociedade Brasileira de Matemtica

caso contrrio. Seja tambm g n =

p ( x)dx
n X

a probabilidade de o apostador

ganhar aps n nmeros sorteados. Temos p1 ( x) = 1, x [0,1] . Lema:

qn ( x) = pn ( x) =

x n 1 para x [0, X ] e (n 1)!

pn ( x) =

X n 1 para (n 1)!

x ( X ,1] .
Prova por induo: o caso inicial n = 1 trivial. Hiptese de induo:

x n 1 para x [0, X ] . Caso o jogo continue, a densidade de (n 1)! probabilidade da soma dos n primeiros nmeros dada por qn ( x) (afinal, para o jogo continuar, essa soma deve estar no intervalo [0, X ] ). E a densidade de probabilidade do (n + 1) -simo nmero igual a p1 ( x) = 1x [0,1] . Assim, qn ( x) =
somando, obtemos pn +1 ( x) =
x 1

q (t )dt .
n

Dessa forma, para x [0, X ] , temos E para

pn +1 ( x) = pn +1 ( x) =

x 1 x

qn (t )dt =
X

t n 1 xn dt = . ( n 1 )! n ! 0
x

x ( X ,1] ,

x 1

qn (t )dt =

t n 1 Xn dt = , c.q.d. (n 1)! n! 0

Pelo lema, temos g n =

pn ( x)dx = (1 X )
X

X n 1 e a probabilidade de o (n 1)!

apostador ganhar f ( X ) =

g n = (1 X )
n =1

X n 1 = (1 X )e X . n =1 ( n 1)!

Agora imagine que a banca tenha k sorteios disponveis para determinar X . Seja Pk a probabilidade de o apostador ganhar nesse caso. Queremos calcular P3 . Vamos calcular Pk recursivamente. Primeira situao: imagine que a banca tem apenas 1 sorteio disponvel para determinar X . Sabendo que, dado X , a probabilidade de o apostador ganhar

f ( X ) = (1 X )e X , descobrimos que P 1 = f ( X ) dX = e 2 0,71828 .


0

EUREKA! N29, 2009

60

Sociedade Brasileira de Matemtica

Segunda situao: imagine agora que a banca tem at 2 sorteios para determinar X . Se, no primeiro sorteio, ela obteve um nmero x , ela pode ficar com esse nmero ou descart-lo. A probabilidade de o apostador ganhar ser f ( x) = (1 x)e x no 1 caso e ser P 1 0,71828 no 2 caso. Assim, a banca deve descartar x somente se (1 x)e x > P 1 . Seja a a raiz positiva da equao transcendente (1 a )e a = P 1 . Temos a 0,60954 . Dessa forma, a banca deve descartar o nmero x se x < a e mant-lo se x > a . Se a banca usar essa estratgia, a probabilidade de o apostador vencer ser
a P2 = P 1a + f ( X ) dX =P 1a + e ( 2 a )e 0,59823 . a 1

Terceira situao: agora considere o problema original (a banca tem at 3 sorteios para determinar X ). Se, no primeiro sorteio, ela obteve o nmero x , ela pode mant-lo ou descart-lo. A probabilidade de o apostador ganhar ser f ( x) = (1 x)e x no 1 caso e ser P2 0,59823 no 2 caso. Assim, a banca deve descartar x somente se (1 x)e x > P2 . Seja b a raiz positiva da equao transcendente (1 b)eb = P2 . Temos b 0,70416 . Dessa forma, a banca deve descartar o nmero x se x < b e mant-lo se x > b . Se a banca usar essa estratgia, a probabilidade de o apostador vencer ser

P3 = P2b + f ( X )dX =P2b + e (2 b)eb 0,51915 >


b

1 . Assim, o apostador tem 2

probabilidade maior que 1 de ganhar, mesmo quando a banca usa a melhor 2 estratgia possvel. Barango Joe deve responder que prefere ser o apostador.
Agradecemos o envio de solues e a colaborao de:
Alexandre Salim Saud de Oliveira Alixanzito R. S. Costa Andr Felipe M. da Silva Doraci Gabriel da Rosa Evandro Makiyama de Melo Flvio Antonio Alves Glauber Moreno Barbosa Kellem Corra Santos Larissa Brito Sousa Marcel Menzes de Andrade Prado Oswaldo Mello Sponquiado Renato Carneiro de Souza Samuel Lil Abdalla Wallace Alves Martins Niteri RJ Fortaleza CE Rio de Janeiro RJ Fartura SP So Paulo SP Amparo SP Rio de Janeiro RJ Rio de Janeiro RJ Fortaleza CE Braslia DF So Paulo SP Belo Horizonte MG Sorocaba SP Rio de Janeiro RJ

Continuamos aguardando solues para os problemas 123, 124, 125, 126 e 129.
EUREKA! N29, 2009

61

Sociedade Brasileira de Matemtica

PROBLEMAS PROPOSTOS *
Convidamos o leitor a enviar solues dos problemas propostos e sugestes de novos problemas para prximos nmeros.

130) Suponha que a, b, c e a equao x 2 ( a + b + c ) x + ( ab + ac + bc) = 0 no tem razes reais. Prove que a, b e c tm todos o mesmo sinal e existe um tringulo de lados
a, b e c.

131) a) Considere o seguinte jogo: no incio um jogador A entrega um nmero k 2 ao jogador B . Quando A entrega um nmero m 2 a B, B pode devolver m 1 ou m + 1 a A. Quando A recebe um nmero n 2 deve, se n for mpar n devolver 3n a B; se n for par mas no mltiplo de 4, pode devolver ou 3n a B, e, 2 n n se n for mltiplo de 4, pode devolver , ou 3n a B. Qualquer jogador ganha o 4 2 jogo se devolver 1 ao adversrio. Caso algum jogador devolva ao adversrio um nmero maior que 1000k, o jogo empata. Determine, para cada valor de k 2 , se algum dos jogadores tem estratgia vencedora, e, nesses casos, qual deles. b) Resolva o item anterior supondo que A, ao receber um nmero n 2, deve n devolver 3n a B se n for mpar, deve devolver a B se n for par mas no mltiplo 2 n de 4 e deve devolver a B se n for mltiplo de 4. 4 132) a) Considere uma famlia de 2000 crculos de raio 1 no plano tal que dois crculos de nunca so tangentes e cada crculo de intersecta pelo menos dois outros crculos de . Determine o nmero mnimo possvel de pontos do plano que pertencem a pelo menos dois crculos de .
Problema 130 proposto por Wlson Carlos da Silva Ramos (Belm PA), problema 131 proposto por Benedito Tadeu Vasconcelos Freire (Natal RN), problema 132 proposto por Juan Manuel Conde Calero (Alicante Espanha).

EUREKA! N29, 2009

62

Sociedade Brasileira de Matemtica

AGENDA OLMPICA
XXXI OLIMPADA BRASILEIRA DE MATEMTICA NVEIS 1, 2 e 3 Primeira Fase Sbado, 06 de junho de 2009 Segunda Fase Sbado, 12 de setembro de 2009 Terceira Fase Sbado, 17 de outubro de 2009 (nveis 1, 2 e 3) Domingo, 18 de outubro de 2009 (nveis 2 e 3 - segundo dia de prova). NVEL UNIVERSITRIO Primeira Fase Sbado, 12 de setembro de 2009 Segunda Fase Sbado, 17 e Domingo, 18 de outubro de 2008

XV OLIMPADA DE MAIO 09 de maio de 2009

XX OLIMPADA DE MATEMTICA DO CONE SUL 14 a 20 de abril de 2009 Mar del Plata Argentina

L OLIMPADA INTERNACIONAL DE MATEMTICA 10 a 22 de julho de 2009 Bremen Alemanha

XVI OLIMPADA INTERNACIONAL DE MATEMTICA UNIVERSITRIA 25 a 30 de julho de 2009 Budapeste, Hungria

XXIV OLIMPADA IBEROAMERICANA DE MATEMTICA Setembro de 2009 Mrida Mxico

XII OLIMPADA IBEROAMERICANA DE MATEMTICA UNIVERSITRIA

EUREKA! N29, 2009

63

Sociedade Brasileira de Matemtica

COORDENADORES REGIONAIS
Alberto Hassen Raad Amrico Lpez Glvez Amarsio da Silva Arajo Andreia Goldani Antonio Carlos Nogueira Benedito Tadeu Vasconcelos Freire Carlos Alexandre Ribeiro Martins Carmen Vieira Mathias Claus Haetinger Cleonor Crescncio das Neves Cludio de Lima Vidal Denice Fontana Nisxota Menegais Edson Roberto Abe Eduardo Tengan lio Mega Eudes Antonio da Costa Fbio Brochero Martnez Florncio Ferreira Guimares Filho Francinildo Nobre Ferreira Genildo Alves Marinho Ivanilde Fernandes Saad Jacqueline Rojas Arancibia Janice T. Reichert Joo Bencio de Melo Neto Joo Francisco Melo Libonati Jose de Arimatia Fernandes Jos Luiz Rosas Pinho Jos Vieira Alves Jos William Costa Krerley Oliveira Licio Hernandes Bezerra Luciano G. Monteiro de Castro Luzinalva Miranda de Amorim Mrio Rocha Retamoso Marcelo Rufino de Oliveira Marcelo Mendes Newman Simes Nivaldo Costa Muniz Osnel Broche Cristo Osvaldo Germano do Rocio Raul Cintra de Negreiros Ribeiro Ronaldo Alves Garcia Rogrio da Silva Igncio Reginaldo de Lima Pereira Reinaldo Gen Ichiro Arakaki Ricardo Amorim Srgio Cludio Ramos Seme Gebara Neto Tadeu Ferreira Gomes Toms Menndez Rodrigues Valdenberg Arajo da Silva Vnia Cristina Silva Rodrigues Wagner Pereira Lopes
EUREKA! N29, 2009

(UFJF) (USP) (UFV) FACOS (UFU) (UFRN) (Univ. Tec. Fed. de Paran) (UNIFRA) (UNIVATES) (EDETEC) (UNESP) (UNIPAMPA) (Colgio Objetivo de Campinas) (USP) (Faculdade Etapa) (Univ. Federal do Tocantins) (UFMG) (UFES) (UFSJ) (Centro Educacional Leonardo Da Vinci) (UC. Dom Bosco) (UFPB)) (UNOCHAPEC) (UFPI) (Grupo Educacional Ideal) (UFPB) (UFSC) (UFPB) (Instituto Pueri Domus) (UFAL) (UFSC) (Sistema Elite de Ensino) (UFBA) (UFRG) (Grupo Educacional Ideal) (Colgio Farias Brito, Pr-vestibular) (Cursinho CLQ Objetivo) (UFMA) (UFLA) (U. Estadual de Maring) (Colgio Anglo) (UFGO) (Col. Aplic. da UFPE) (Escola Tcnica Federal de Roraima) (UNIFESP) (Centro Educacional Logos) (IM-UFRGS) (UFMG) (UEBA) (U. Federal de Rondnia) (U. Federal de Sergipe) (U. Metodista de SP) (CEFET GO)

Juiz de Fora MG Ribeiro Preto SP Viosa MG Osrio RS Uberlndia MG Natal RN Pato Branco PR Santa Mara RS Lajeado RS Manaus AM S.J. do Rio Preto SP Bag RS Campinas SP So Carlos SP So Paulo SP Arraias TO Belo Horizonte MG Vitria ES So Joo del Rei MG Taguatingua DF Campo Grande MS Joo Pessoa PB Chapec SC Teresina PI Belm PA Campina Grande PB Florianpolis SC Campina Grande PB Santo Andr SP Macei AL Florianpolis SC Rio de Janeiro RJ Salvador BA Rio Grande RS Belm PA Fortaleza CE Piracicaba SP So Luis MA Lavras MG Maring PR Atibaia SP Goinia GO Recife PE Boa Vista RR SJ dos Campos SP Nova Iguau RJ Porto Alegre RS Belo Horizonte MG Juazeiro BA Porto Velho RO So Cristovo SE S.B. do Campo SP Jata GO

64

CONTEDO
XXX OLIMPADA BRASILEIRA DE MATEMTICA Problemas e Solues da Primeira Fase XXX OLIMPADA BRASILEIRA DE MATEMTICA Problemas e Solues da Segunda Fase XXX OLIMPADA BRASILEIRA DE MATEMTICA Problemas e Solues da Terceira Fase XXX OLIMPADA BRASILEIRA DE MATEMTICA Problemas e Solues da Primeira Fase Nvel Universitrio XXX OLIMPADA BRASILEIRA DE MATEMTICA Problemas e Solues da Segunda Fase Nvel Universitrio XXX OLIMPADA BRASILEIRA DE MATEMTICA Premiados AGENDA OLMPICA COORDENADORES REGIONAIS 2 16

33

51

57 66 70 71

Sociedade Brasileira de Matemtica

XXX OLIMPADA BRASILEIRA DE MATEMTICA


Problemas e Solues da Primeira Fase
PROBLEMAS NVEL 1 01) Com segmentos de 1 cm de comprimento podemos formar tringulos. Por exemplo, com nove desses segmentos podemos formar um tringulo eqiltero de lado 3 cm. Com qual nmero de segmentos a seguir impossvel formar um tringulo? A) 4 B) 5 C) 6 D) 7 E) 8 02) Esmeralda compra cinco latas de azeite a quatro reais e setenta centavos a lata, cinco latas de leite em p a trs reais e doze centavos cada e trs caixas de iogurte com seis iogurtes cada caixa ao preo de oitenta centavos por iogurte. Paga com uma nota de cinqenta reais e quer saber quanto ir receber de troco. Qual das expresses aritmticas a seguir representa a soluo para este problema? A) 50 5 (4,70 + 3,12) + 18 0,80 B) 5 4,70 + 5 3,12 + 3 6 0,80 50 C) [5 (4,70 + 3,12) + 3 6 0,80] + 50 D) 50 [5 (4,70 + 3,12) + 3 6 + 0,80] E) 50 [5 (4,70 + 3,12) + 6 0,80] 03) Uma pesquisa foi feita entre pessoas de ambos os sexos, em igual nmero, com a seguinte pergunta: Entre as cores azul, vermelho e amarelo, qual a cor que voc prefere? Cada pessoa apresentou a sua preferncia por uma, e s uma, dessas cores. E o resultado da pesquisa aparece nos grficos abaixo:

EUREKA! N30, 2009

Sociedade Brasileira de Matemtica

Podemos concluir que, em relao ao total de pessoas pesquisadas, a ordem de preferncia das cores : A) I, II, III B) I, III, II C) II, I, III D) II, III, I E) III, II, I

04) O quociente e o resto na diviso de 26097 por 25 so, respectivamente: A) 1043 e 22 B) 1044 e 3 C) 143 e 22 D) 1044 e 22 E) 144 e 3 05) Numa reunio da comunidade do bairro, cada uma das 125 pessoas presentes recebeu um nmero diferente, a partir do nmero 1 at o 125. Em dado momento, foi feita uma lista das pessoas com nmero par e das pessoas com nmero mltiplo de 3, que deveriam participar de um projeto. Algumas pessoas reclamaram, dizendo que o seu nome aparecia duas vezes na lista. Quantas pessoas apareceram duas vezes na lista? A) 2 B) 6 C) 20 D) 41 E) 62 06) Sobre uma mesa retangular de uma sala foram colocados quatro slidos, mostrados no desenho. Uma cmera no teto da sala, bem acima da mesa, fotografou o conjunto. Qual dos esboos a seguir representa melhor essa fotografia?

EUREKA! N30, 2009

Sociedade Brasileira de Matemtica

07) Uma classe tem 22 alunos e 18 alunas. Durante as frias, 60% de todos os alunos dessa classe foram prestar trabalho comunitrio. No mnimo, quantas alunas participaram desse trabalho? A) 1 B) 2 C) 4 D) 6 E) 8 08) Uma urna contm 2008 cartes. Cada carto recebeu um nmero diferente, a partir do nmero 1 at o 2008. Retiram-se dois cartes ao acaso e somam-se os nmeros dos cartes. Quantos nmeros mpares diferentes podem ser obtidos dessa maneira? A) 1004 B) 1005 C) 2007 D) 2008 E) 4016 09) Juntando quatro trapzios iguais de bases 30 cm e 50 cm, como o da figura ao lado, podemos formar um quadrado de rea 2500 cm2, com um buraco quadrado no meio. Qual a rea de cada trapzio, em cm2? A) 200 B) 250 C) 300 D) 350
30cm 45o 50 cm 45o

E) 400

10) Quantos nmeros pares de trs algarismos tm dois algarismos mpares? A) 20 B) 48 C) 100 D) 125 E) 225 11) Sabe-se que

5 2 do contedo de uma garrafa enchem de um copo. Para 9 6

encher 15 copos iguais a esse, quantas garrafas devero ser usadas? A) 2 B) 3 C) 4 D) 5 E) 6

12) Quantos quadrados tm como vrtices os pontos do reticulado ao lado? A) 6 E) 10 B) 7 C) 8 D) 9

EUREKA! N30, 2009

Sociedade Brasileira de Matemtica

13) A primeira fase da OBM se realiza no dia 14 de junho, um sbado do ano bissexto 2008. Daqui a quantos anos o dia 14 de junho ser novamente no sbado? A) 4 B) 5 C) 6 D) 7 E) 8 14) No desenho temos AE = BE = CE = CD. Alm disso, e so medidas de ngulos. Qual o valor da razo ? A)

3 5 5 4

B)

4 5 5 3

C) 1

D)

E)

15) Na multiplicao ao lado, alguns algarismos, no necessariamente iguais, foram substitudos pelo sinal *. Qual a soma dos valores desses algarismos? A) 17 E) 57 B) 27 C) 37 D) 47

16) Trs amigos moram na mesma rua: um mdico, um engenheiro e um professor. Seus nomes so: Arnaldo (A), Bernaldo (B) e Cernaldo (C). O mdico filho nico e o mais novo dos trs amigos. Cernaldo mais velho que o engenheiro e casado com a irm de Arnaldo. Os nomes do mdico, do engenheiro e do professor, nessa ordem, so: A) A, B, C B) C, A, B C) B, A, C D) B, C, A E) A, C, B

EUREKA! N30, 2009

Sociedade Brasileira de Matemtica

17) Dois cartes iguais tm a forma de um tringulo retngulo de lados 5 cm, 12 cm e 13 cm. Esmeralda juntou os dois cartes sobre uma folha de papel e, contornando as beiradas com um lpis, obteve uma figura como a ao lado, que est fora de escala. Qual o permetro dessa figura? A) 28 cm B) 35 cm C) 42 cm D) 43 cm E) 60 cm

18) Qual o maior nmero de algarismos que devem ser apagados do nmero de 1000 algarismos 200820082008, de modo que a soma dos algarismos restantes seja 2008? A) 130 B) 260 C) 510 D) 746 E) 1020 19) Soninha tem muitos cartes, todos com o mesmo desenho em uma das faces. Ela vai usar cinco cores diferentes (verde, amarelo, azul, vermelho e laranja) para pintar cada uma das cinco partes do desenho, cada parte com uma cor diferente, de modo que no haja dois cartes pintados da mesma forma. Na figura abaixo, por exemplo, os cartes so iguais, pois um deles pode ser girado para se obter o outro. Quantos cartes diferentes Soninha conseguir produzir?

A) 16

B) 25

C) 30

D) 60

E) 120

EUREKA! N30, 2009

Sociedade Brasileira de Matemtica

20) Trs carros com velocidades constantes cada um, na mesma estrada, passam no mesmo momento por Brasilpolis. Ao viajar 100 quilmetros, o carro A passa por Americanpolis, 20 quilmetros frente do carro B e 50 quilmetros frente do carro C. Quando o carro B passar por Americanpolis, quantos quilmetros estar frente do carro C? A) 20 B) 25,5 C) 30 D) 35 E) 37,5 PROBLEMAS NVEL 2 01) Veja o problema No. 14 do Nvel 1. 02) Quantos dos nmeros abaixo so maiores que 10?

3 11 , 4 7 , 5 5 , 6 3 , 7 2
A) 1 B) 2 C) 3 D) 4 E) 5

03) 1212 igual a: A) 6 6 B) 12 2


3

C) 212.3 6

D) 612

E) 12

12

04) Uma grande empresa possui 84 funcionrios e sabe-se que cada funcionrio fala pelo menos uma das lnguas entre Portugus e Ingls. Alm disso, 20% dos que falam Portugus tambm falam Ingls e 80% dos que falam Ingls tambm falam Portugus. Quantos funcionrios falam as duas lnguas? A) 12 B) 14 C) 15 D) 16 E) 18 05) Edmilson, Carlos e Eduardo ganharam um total de R$150,00 lavando carros. Eles ganharam quantidades diferentes de dinheiro. Como eles so muito amigos decidiram dividir o dinheiro ganho em partes iguais. Para isto, Edmilson deu metade do que ganhou para dividir em partes iguais entre Carlos e Eduardo, porm, Carlos tinha muito dinheiro e, portanto, deu R$ 10,00 a cada um dos outros dois. Finalmente, para que cada um tivesse a mesma quantidade de dinheiro, Eduardo deu R$ 2,00 a Edmilson. Quanto Eduardo ganhou antes da diviso? A) R$ 76,00 B) R$ 51,00 C) R$ 23,00 D) R$ 50,00 E) R$ 100,00 06) Nove nmeros so escritos em ordem crescente. O nmero do meio a mdia aritmtica dos nove nmeros. A mdia aritmtica dos 5 maiores 68 e a mdia aritmtica dos 5 menores 44. A soma de todos os nmeros : A) 560 B) 504 C) 112 D) 56 E) 70
EUREKA! N30, 2009

Sociedade Brasileira de Matemtica

07) Veja o problema No. 12 do Nvel 1. 08) Veja o problema No. 13 do Nvel 1. 09) Os algarismos a , b e c so tais que os nmeros de dois algarismos aa , bc e

cb so nmeros primos e aa + bc + cb = aa . Se b < c , ento bc igual a:


A) 19 B) 17 C) 37 D) 29 E) 59 10) Cinco inteiros positivos a, b, c, d , e maiores que um satisfazem as seguintes condies:
a (b + c + d + e) = 128 b(a + c + d + e) = 155 c(a + b + d + e) = 203 d (a + b + c + e) = 243 e(a + b + c + d ) = 275

Quanto vale a soma a + b + c + d + e ? A) 9 B) 16 C) 25

D) 36

E) 49

11) Em um tringulo ABC foi traada a altura AH. Sejam M e N pontos sobre os lados AB e AC, respectivamente, tais que HM perpendicular a AB e HN perpendicular a AC. Achar MN, sabendo que o permetro do tringulo rtico do tringulo ABC igual a 10. Observao: o tringulo rtico de um tringulo aquele cujos vrtices so as intersees das alturas do tringulo com os respectivos lados. Pode-se demonstrar que o incentro (encontro das bissetrizes) do tringulo rtico sempre igual ao ortocentro (encontro das alturas) do tringulo original. A) 5 B) 6 C) 7 D) 8 E) 9 12) Quantos nmeros inteiros positivos menores que 500 tm exatamente 15 divisores inteiros positivos? A) 0 B) 1 C) 2 D) 3 E) 4 13) Seja P (n) a soma dos algarismos pares do nmero n . Por exemplo, P (1234) = 2 + 4 = 6. Qual o valor de P (1) + P (2) + P (3) + ... + P(100) ? A) 200 B) 360 C) 400 D) 900 E) 2250

EUREKA! N30, 2009

Sociedade Brasileira de Matemtica

14) De quantas maneiras podemos dividir R$ 10,00 em moedas de 10 centavos e de 25 centavos, se pelo menos uma moeda de cada valor tem que ser usada? A) 15 B) 16 C) 17 D) 18 E) 19 15) Sejam a, b, c, d nmeros inteiros tais que a < 2b , b < 3c , c < 4d . Se d < 40 , o maior valor possvel de a ser: A) 960 B) 959 C) 951 D) 934 E) 927 16) A figura abaixo um exemplo de um quadrado mgico de ordem 4. A soma dos 4 nmeros em cada linha, coluna e diagonal 34. Ento dizemos que a soma mgica deste quadrado mgico 34. Suponha que exista um quadrado mgico de ordem 7, formado pelos nmeros inteiros de 1 a 49. Determine sua soma mgica.
16 5 9 4 3 10 6 15 2 11 7 14 13 8 12 1

A) 175

B) 2450

C) 1225

D) 190

E) 100

17) Observe que:

32 + 4 2 = 5 2 , 3 2 + 4 2 + 12 2 = 13 2 , 3 2 + 4 2 + 12 2 + 84 2 = 85 2. Qual o menor valor possvel da soma x + y com x, y inteiros positivos tais que
A) 289 B) 250

3 2 + 4 2 + 12 2 + 84 2 + x 2 = y 2 ? C) 425 D) 795

E) 103

18) Um nmero de trs algarismos 629 vezes menor que a soma de todos os outros nmeros de trs algarismos. Este nmero : A) 450 B) 785 C) 630 D) 471 E) 525 19) Veja o problema No. 19 do Nvel 1.
EUREKA! N30, 2009

Sociedade Brasileira de Matemtica

20) Em um tringulo ABC, A = 20o e B = 110o. Se I o incentro (centro da circunferncia inscrita) e O o circuncentro (centro da circunferncia circunscrita) do tringulo ABC, qual a medida do ngulo IAO ? A) 20o B) 25o C) 30o D) 40o E) 35o 21) Veja o problema No. 7 do Nvel 1. 22) Na figura abaixo os pontos A, B, C so colineares, assim como os pontos D, E, F. As duas retas ABC e DEF so paralelas.
A B C

A1

A3

Sendo A1, A2 e A3 as reas das regies destacadas na figura, podemos afirmar que: A) A2 = 2A1 = 2A3 B) A2 = A1 + A3 C) A2 > A1 + A3 D) A2 < A1 + A3 E) A22 = A1.A3

23) O grupo A da ltima Copa do Mundo de futebol terminou com os seguintes resultados:
Equipe ustria Brasil Camares Dinamarca Nmero de Pontos 7 5 4 0

Sabe-se que ustria e Camares levaram apenas 1 gol, cada um. Alm disso, Brasil e Dinamarca marcaram apenas 1 gol, cada um, enquanto que ustria marcou 3 gols. Qual o resultado da partida ustria Dinamarca? Observao: no grupo, cada seleo joga com as demais exatamente uma vez e, em cada partida, o time vencedor ganha 3 pontos, o perdedor no ganha nem perde pontos e, em caso de empate, cada time ganha 1 ponto. A) 1 0 B) 2 1 C) 2 0 D) 0 0 E) Nada se pode afirmar.

EUREKA! N30, 2009

10

Sociedade Brasileira de Matemtica

24) Abaixo temos um quadrado mgico multiplicativo, onde o produto dos nmeros em cada linha, coluna e diagonal o mesmo e igual ao nmero de quatro dgitos ABCD, onde cada letra representa um dgito e cada casa contm um nmero inteiro. Se AC representa o nmero de dois dgitos no centro do quadrado, a soma A + B + C + D vale:
4

AC C A) 17 B) 18 C) 19
24

D) 20

E) 21

25) Tenho um cubo de madeira, com trs faces vermelhas e trs faces azuis. O cubo cortado em 333 = 27 cubos menores. Quantos destes cubos menores tm, pelo menos, uma face vermelha e outra azul? A) 6 B) 12 C) 14 D) 16 E) depende de quais faces do cubo so vermelhas e quais so azuis. PROBLEMAS NVEL 3 01) Veja o problema No. 14 do Nvel 1. 02) Sendo x = 102008, assinale a alternativa que apresenta o maior valor. 1 1 1 x A) B) C) D) x E) 1 1 x( x + 1) x x+ 1+
1+ 1 x
x

03) O nmero inteiro positivo a e o nmero


maneira:

1 localizam-se na reta da seguinte a

Qual a soma desses dois nmeros?

A)

9 81

B)

9 80

C)

81 9

D)

82 9

E) 9

EUREKA! N30, 2009

11

Sociedade Brasileira de Matemtica

04) Veja o problema No. 4 do Nvel 2 05) Rafael tem 10 cartes. Cada um tem escrito um dos nmeros 3, 8, 13, 18, 23, 28, 33, 48, 53, 68, e todos os dez nmeros aparecem. Qual o menor nmero de cartes que Rafael pode escolher de modo que a soma dos nmeros nos cartes escolhidos seja exatamente 100? A) 2 B) 3 C) 4 D) 5 E) no possvel obter soma 100 com esses cartes. 06) Em uma pista de corrida, cujo formato de um polgono regular de n vrtices, numerados de 1 at n no sentido anti-horrio, existem trs pessoas: Nelly, Snia e Penha, estando inicialmente todas em um mesmo vrtice. Em um dado momento elas comeam a caminhar pelos lados do polgono. Nelly caminha no sentido antihorrio, enquanto que Snia e Penha caminham no sentido contrrio. Nelly cruza com Snia pela primeira vez em um vrtice e com Penha dois vrtices frente. A velocidade de Nelly o dobro da velocidade de Snia e a velocidade de Snia o dobro da velocidade de Penha. Quantos vrtices tem o polgono? A) 30 B) 60 C) 15 D) 10 E) 6 07) Veja o problema No. 6 do Nvel 2. 08) A primeira fase da OBM se realiza no dia 14 de junho, um sbado do ano bissexto 2008. Daqui a quantos anos o dia 14 de junho ser novamente no sbado? A) 4 B) 5 C) 6 D) 7 E) 8 09) Veja o problema No. 14 do Nvel 2. 10) O inteiro n tal que n2n possui 2008 divisores a mais que n. A soma dos algarismos de n igual a: A) 5 B) 7 C) 9 D) 11 E) 12 11) Quantos dos nmeros 2, 3, 5, 7, 11 so divisores de 3714 414? A) um B) dois C) trs D) quatro E) cinco 12) Veja o Problema No. 25 do Nvel 2.

EUREKA! N30, 2009

12

Sociedade Brasileira de Matemtica

13) O nmero de solues reais do sistema

a 2 = b + 2 2 b = c + 2 c 2 = a + 2
igual a: A) 0

B) 1

C) 2

D) 4

E) 8

14) Arnaldo, Bernaldo, Cernaldo e Dernaldo baralharam as 52 cartas de um baralho e distriburam 13 cartas para cada um. Arnaldo ficou surpreso: Que estranho, no tenho nenhuma carta de espadas. Qual a probabilidade de Bernardo tambm no ter cartas de espadas? A)

39! 26!52!

B)

26! 13!39!

C)

39!39! 26!52!

D)

26!26! 13!39!

E)

39!13! 52!

15) Veja o problema No. 19 do Nvel 2. 16) Dado o quadriltero ABCD tal que CAD = 25, ACD = 45 e BAC = BCA = 20, qual o valor do ngulo DBC? A) 40 B) 45 C) 50 D) 55 E) 60 17) No tringulo PQR issceles, com PQ = PR = 3 e QR = 2, a tangente sua circunferncia circunscrita no ponto Q encontra o prolongamento do lado PR em X. O valor de RX : A)

16 5

B)

12 5

C)

8 3

D)

9 2

E)

9 4

18) Dado um tringulo ABC de lados AB = 3, BC = 4 e AC = 5. Sejam R1 e R2, respectivamente, os raios da circunferncia inscrita e da circunferncia com centro
sobre o lado BC que passa por B e tangente ao lado AC. A razo

A)

3 4

B)

2 3

C)

3 2

D)

8 9

R1 vale: R2 4 E) 5

19) Qual o nmero de solues reais do sistema e x | x | + y | y |= 1


EUREKA! N30, 2009

x + y = 1 ,

13

Sociedade Brasileira de Matemtica

onde x representa a parte inteira de x?

A) 0

B) 1

C) 2

D) 4

E) infinitas

20) Um nmero de quatro dgitos dito paladino se mltiplo de 9 e nenhum de seus dgitos nulo. Quantos nmeros paladinos existem? A) 1284 B) 1024 C) 849 D) 1109 E) 729 21) Considere a funo f, definida no conjunto dos nmeros reais e satisfazendo

f ( x) =

cx , para todo x 3/2. Determine o nmero de tais funes f para as 2x + 3


E) infinitas.

quais f (f (x)) = x, para todo x tal que f (f (x)) est bem definida. A) 0 B) 1 C) 2 D) 4

22) O brinquedo favorito de Ccero um cone reto de vidro com 5 cm de altura. Ccero encheu o cone com areia at a altura de 3 cm, como mostrado na figura 1. Em seguida, Ccero fechou a base do cone e virou-o de cabea para baixo, como indicado na figura 2. A que altura da base do cone, em cm, ficou a marca de areia?

3 cm

Figura 1

Figura 2

A) 1 E) 1
3

B) 2

C) 5 3 98

D)

98

98 5

23) Veja o problema No. 24 do Nvel 2. 24) Considere 10 pessoas, todas de alturas diferentes, as quais devem ficar em fila de tal modo que, a partir da pessoa mais alta, as alturas devem decrescer para ambos os lados da fila (se a pessoa mais alta for a primeira ou a ltima da fila,
EUREKA! N30, 2009

14

Sociedade Brasileira de Matemtica

todas as pessoas a partir dela devem estar em ordem decrescente de altura). Obedecendo essas condies, de quantos modos essas pessoas podem ficar em fila? A) 256 B) 768 C) 1260 D) 512 E) 2560

25. Veja o problema No. 10 no Nvel 2.

GABARITO
NVEL 1 (6. ou 7. Anos) 1) A 6) E 2) C 7) B 3) B ou D 8) C 4) A 9) E 5) C 10) D NVEL 2 (8. ou 9. Anos) 1) D 6) B 2) C 7) E 3) C 8) C 4) D 9) C 5) C 10) D NVEL 3 (Ensino Mdio) 1) D 6) C 2) A 7) B 3) D 8) C 4) D 9) E 5) D 10) A ou B

11) C 12) E 13) C 14) D 15) C

16) C 17) C 18) D 19) C ou D 20) E

11) A 12) D 13) C 14) E 15) E

16) A 17) A 18) B 19) C ou D 20) C

21) B 22) B 23) B 24) B 25) E

11) D 12) B 13) E 14) D 15) C ou D

16) C 17) B 18) B 19) C 20) E

21) B 22) C 23) B 24) D 25) D

EUREKA! N30, 2009

15

Sociedade Brasileira de Matemtica

XXX OLIMPADA BRASILEIRA DE MATEMTICA


Problemas e Solues da Segunda Fase
PROBLEMAS NVEL 1 PARTE A

(Cada problema vale 5 pontos)


01. Nicanor quer completar o Sudoku ao lado,

de modo que em cada linha (fileira horizontal) e cada coluna (fileira vertical) apaream todos os nmeros de 1 a 6. Qual a soma de todos os nmeros que faltam para completar o Sudoku?

2 4 6 6 4 3

1 5 2

02. A partir das igualdades

32 12 = 8 = 8 1, 5 2 32 = 16 = 8 2, 7 2 5 2 = 24 = 8 3, e 2009 2 2007 2 = 8 N,
podemos escrever 2009 2 1 = 4 N ( N + 1) . Qual o valor de N?
03. Certo banco brasileiro obteve um lucro de R$ 4,1082 bilhes ao final do primeiro semestre de 2008. Esse valor representa um aumento de 2,5% em relao ao resultado obtido no mesmo perodo do ano passado. Qual a soma dos dgitos do nmero inteiro que representa, em reais, o lucro desse banco no primeiro semestre de 2007? 04. A piscina do clube que Esmeralda freqenta tem a forma

de um hexgono (polgono com seis lados), com um ngulo interno de 270, os demais ngulos de 90 e os quatro lados menores com 12 metros cada. Esmeralda costuma nadar pelo meio da piscina, a partir do ponto A, descrevendo o trajeto representado, na figura, pelo ngulo reto ABC, em que AB = BC.
EUREKA! N30, 2009

16

Sociedade Brasileira de Matemtica

Certo dia, ela nadou por esse trajeto 4 vezes, isto , foi e voltou 2 vezes. Quantos metros ela percorreu?
05. Com o dinheiro que Carlinhos tinha, poderia ter comprado 600 gramas de queijo

ou 400 gramas de presunto. Usando esse dinheiro, ele resolveu comprar quantidades iguais de presunto e queijo. Quantos gramas de cada item ele comprou?
06. Quantos nmeros inteiros maiores que zero e menores que 100 possuem algum

divisor cuja soma dos dgitos seja 5?

PROBLEMAS NVEL 1 PARTE B

(Cada problema vale 10 pontos)


PROBLEMA 1

Zezinho tem 37 cartes quadrados de lado 6 cm e 21 cartes quadrados de lado 9 cm. Ele quer colar esses cartes lado a lado, sem sobrep-los nem deixar buracos, formando quadrados maiores. a) Apresente, atravs de desenhos, duas maneiras diferentes de Zezinho construir um quadrado de lado 27 cm. b) Quantos cartes so necessrios para construir o quadrado com a maior rea possvel?
PROBLEMA 2

Para construir o arranjo triangular de letras ao lado, que tem 2008 linhas, obedeceu-se a uma certa regra. a) Quantas vezes a palavra OBM aparece completamente na maior coluna desse arranjo? b) Quantas vezes a letra O aparece no arranjo?

PROBLEMA 3

Em Ferius, os pontos do domin vo de 0 a 7, ao contrrio de um domin comum, em que os pontos vo de 0 a 6. Uma pea do domin de Ferius chamada importante se a soma de seus pontos par. Por exemplo, os seguintes domins so importantes:

EUREKA! N30, 2009

17

Sociedade Brasileira de Matemtica

a) Quantas peas diferentes possui o domin jogado em Ferius? b) Quantas dessas peas so importantes? c) Qual a soma dos pontos de todas as peas importantes?

PROBLEMAS NVEL 2 PARTE A

(Cada problema vale 5 pontos)


01. Sejam x e y nmeros reais positivos satisfazendo as equaes x 2 + y 2 = 1 e

x4 + y 4 =

17 1 . Calcule o valor de . xy 18

02. Um viajante, que se encontrava perdido na floresta, andou 1 metro para o Leste,

2 metros para o Norte, 3 para o Oeste, 4 para o Sul, 5 para o Leste, 6 para o Norte,..., 2006 metros para o Norte, 2007 para o Oeste e 2008 para o Sul. Calcule, em metros, o valor inteiro mais prximo da distncia entre as posies inicial e final do viajante.
03. Os nmeros

so as razes da equao x 2 x 1 = 0. Calcule

13 5 + 5 7 .
04. Em um tringulo ABC, seja D um ponto sobre o lado BC tal que DB = 14, DA = 13 e DC = 4. Sabendo que o crculo circunscrito ao tringulo ADB tem raio igual ao do crculo circunscrito ao tringulo ADC, calcule a rea do tringulo ABC. 05. Dado um nmero natural N, multiplicamos todos os seus algarismos. Repetimos

o processo com o nmero obtido at obtermos um nmero com um algarismo. Este nmero ser chamado de primitivo de N. Por exemplo, como 3 2 7 = 42 e 4 2 = 8, conclumos que o primitivo de 327 8. Calcule a soma dos algarismos do maior nmero natural com todos os algarismos diferentes cujo primitivo mpar.

EUREKA! N30, 2009

18

Sociedade Brasileira de Matemtica

PROBLEMAS NVEL 2 PARTE B (Cada problema vale 10 pontos)


PROBLEMA 1

Encontre todos os tringulos retngulos, de lados com medidas inteiras, nos quais a rea tem valor numrico igual ao do permetro.
PROBLEMA 2

No quadro negro so escritos os nmeros 12 , 22 ,32 , 42 ,..., 20082. Pedro e Igor jogam um jogo onde eles apagam alternadamente um nmero por vez at sobrarem apenas dois nmeros. Se a diferena entre estes dois nmeros for mltiplo de 2009, Igor vence. Caso contrrio, quem vence Pedro. Sabendo que Pedro o primeiro a jogar, diga quem possui a estratgia vencedora. Justifique sua resposta.
PROBLEMA 3

Seja ABC um tringulo acutngulo com BC = 5. Seja E o p da altura relativa ao lado AC e F o ponto mdio do lado AB. Se BE = CF = 4, calcule a rea do tringulo ABC.
PROBLEMA 4

Um pas tem 8 cidades, A1, A2, ..., A6, B, C, ligadas por rodovias de mo dupla satisfazendo as seguintes condies: B e C so ambas ligadas s cidades A1, A2, ..., A6, mas no so ligadas uma outra; A1, A2, ..., A6 so ligadas duas a duas. Calcule o nmero de maneiras distintas de viajar de carro de B a C, sem passar duas vezes por uma mesma cidade.

PROBLEMAS NVEL 3 PARTE A

(Cada problema vale 5 pontos)


01. Um trapzio issceles ABCD, com lados paralelos AB e CD, tal que a diagonal

BD mede 100 m e o ngulo BDC mede 30. Seja S a rea do trapzio em m2. Determine S 3.
02. Se x um nmero real, denotamos por x o maior inteiro que menor ou

igual a x. Por exemplo, 2 =3 e 2,1 = 3. Calcule o valor da soma = 2, 4 1 + 4 2 + 4 3 + 4 4 + ... + 4 2008 . 03. Um inteiro positivo n chamado de auto-replicante se os ltimos dgitos de n 2 formam o nmero n. Por exemplo, 25 auto-replicante pois 252 = 625. Determine
EUREKA! N30, 2009

19

Sociedade Brasileira de Matemtica

a soma de todos os nmeros auto-replicantes com exatamente 4 dgitos (isto , nmeros auto-replicantes n com 1000 n 9999 ).

1 i < 9, os nmeros que aparecem entre i e i + 1 (onde i pode aparecer tanto antes como depois de i + 1) so todos menores do que i? Por exemplo, 976412358 uma permutao com esta propriedade.
05. Suponha que raiz de algum polinmio no-nulo com coeficientes racionais. O polinmio minimal de o polinmio de menor grau m(x) tal que: m( ) = 0;

04. Quantas permutaes de 1, 2, 3, ..., 9 h com a propriedade de que, para todo

m( x) Mnico (isto , o seu coeficiente lder 1) e todos os seus coeficientes so racionais.


2 x 2 2. Determine o produto dos
3

Por exemplo, o polinmio minimal de

coeficientes no nulos do polinmio minimal de

27 + 5 33 3 27 + 5 33.

PROBLEMAS NVEL 3 PARTE B (Cada problema vale 10 pontos)


PROBLEMA 1

Determine todos os inteiros positivos m e n tais que m 2 + 161 = 3n


PROBLEMA 2

Determine a quantidade de funes f :{1, 2,3, 4,5} {1, 2,3, 4,5} f ( f ( x)) = f ( x) para todo x {1, 2,3, 4,5}.
PROBLEMA 3

tais que

Um trapzio ABCD, com lados paralelos AB e CD, est inscrito em uma circunferncia de raio 25. Sabe-se que CD um dimetro e a altura desse trapzio 24. Seja E um ponto no arco menor determinado por A e B e sejam F e G os pontos AF BG . de interseo de ED e EC com AB, respectivamente. Calcule FG
Em uma matriz 2008 2008 o elemento na linha i e coluna j o nmero i + j (as
EUREKA! N30, 2009

PROBLEMA 4

20

Sociedade Brasileira de Matemtica

linhas e colunas so numeradas de 1 a 2008). Escolhem-se 2008 elementos desta matriz de modo que no haja dois elementos escolhidos numa mesma linha ou coluna. Os elementos so multiplicados. Qual o menor produto que se pode obter desta forma?

Solues Nvel 1 Segunda Fase Parte A


Problema Resposta 01 91 02 1004 03 12 04 144 05 240 06 34

dos nmeros em cada linha, ou seja, 6 (1 + 2 + + 6 ) = 6 21 = 126 . A soma dos nmeros que j esto escritos no Sudoku 35. Logo a soma dos nmeros que faltam para completar o Sudoku 126 35 = 91.
02. [1004] Temos:

01.[91] A soma de todos os nmeros do Sudoku completo igual a 6 vezes a soma

20092 12 = 4 N ( N +1) ( 2009 1)( 2009 +1) = 4N( N +1) 2008 2010 = 4N( N +1) 2008 2010 4N( N +1) = 1004 1005 = N( N +1) N = 1004 2 2 2 2
Solues alternativas: 1a soluo Cada linha pode ser associada a um nmero mpar e a um mltiplo de 8 da seguinte forma: na linha 1 temos o quadrado de 1 = 2 1 1 (no lado esquerdo da igualdade) e 8 vezes 1 (no lado direito da igualdade), na linha 2 temos o quadrado de 3 = 2 2 1 e 8 vezes 2, na linha 3 temos o quadrado de 5 = 2 3 1 e 8 vezes 3 e assim sucessivamente, at chegarmos linha N onde temos o quadrado de 2007 = 2N 1 e 8 vezes N. Assim, 2N 1 = 2007 2N = 2008 N = 1004 . 2a soluo Cada linha pode ser associada um mltiplo de 8 da seguinte forma: na linha 1 temos 8 vezes 1 (no lado direito da igualdade), na linha 2 temos 8 vezes 2, na linha 3 temos 8 vezes 3 e assim sucessivamente, at chegarmos a ltima linha, onde 2009 1 temos 2009 2 2007 2 = 8 N , que a linha = 1004 , ou seja, N = 1004. 2
EUREKA! N30, 2009

21

Sociedade Brasileira de Matemtica

3a soluo Temos: 20092 20072 = 8 N (2009 2007 )(2009+ 2007 ) = 8 N 2 4016= 8 N N = 1004

03. [12] Seja x o lucro desse banco no primeiro semestre de 2007, em bilhes de reais. Logo x + 2 ,5% x = 4 ,1082 x + 0 ,025 x = 4 ,1082 1,025 x = 4 ,1082 x = 4 ,008 bilhes de reais, ou seja, o lucro foi de R$ 4008000000,00, cuja soma dos dgitos 12. 04. [144] A partir das informaes dadas, conclumos que na figura ID = DE = EF = FG = 12 metros e que A o ponto mdio de ID , ou seja, AD = 6 metros e, da mesma forma, FC = 6 metros. Logo AB = BC = 12 + 6 = 18 metros e, portanto, Esmeralda nadou 4 (18 + 18) = 4 36 = 144 metros.
I D

05. [240] Supondo que Carlinhos tem Q reais, o preo do grama de queijo
o preo do grama de presunto

Q e 600

Q . Seja m a quantidade, em gramas, de queijo e 400 de presunto que Carlinhos comprou. Dessa forma: Q Q 1 1 400 600 240000 1 m + m = Q m + = = = 240 =1 m = 1 1 600 400 400 + 600 1000 600 400 + 600 400 Portanto ele comprou 240 gramas de cada item.

06. [34] So os mltiplos de 5, que nesse intervalo so 19; os mltiplos de 14, que so 6 (pois o 70 j foi contado); os mltiplos de 23, que so 4; os mltiplos de 32, que so 3 e, finalmente, os mltiplos de 41, que so 2. Note que o nico mltiplo de 50 no intervalo, que o prprio 50, j foi contato nos mltiplos de 5. Portanto ao todo so 19 + 6 + 4 + 3 + 2 = 34 nmeros.

EUREKA! N30, 2009

22

Sociedade Brasileira de Matemtica

Solues Nvel 1 Segunda Fase Parte B


PROBLEMA 1 a) Os desenhos mostram as duas formas de construo dos quadrados. Elas so as nicas possveis. De fato, sendo x o nmero de quadrados de lado 6 cm e y o nmero de quadrados de lado 9 cm usados para construir um lado de 27 cm, temos: 9 2x 6x + 9y = 27 2x + 3y = 9 y = Como x e y so inteiros no negativos, 3 podemos substituir x apenas por 0, 1, 2, 3 ou 4. As nicas solues para essa situao so x = 0 e y = 3 ou x = 3 e y = 1, representadas nos desenhos.
b) Repetindo mais 3 vezes a segunda construo acima, obtm-se um quadrado de lado 54 cm, com a utilizao de 36 cartes de lado 6 cm e 20 cartes de lado 9 cm, sobrando apenas 1 carto de lado 6 cm e 1 carto de lado 9 cm. Esse quadrado o maior que se pode construir, usando-se o maior nmero de cartes, 56 cartes. De fato, como os quadrados construdos com os cartes devem ter lados com medidas inteiras, conclumos que o quadrado maior do que o construdo deveria ter lado de 60 cm, pelo menos, j que o carto menor tem lado 6 cm. Como 602 542 = 684 cm 2 maior do que 6 2 + 9 2 = 117 cm 2 , que a soma das reas dos quadrados que sobraram, conclumos que realmente o quadrado de lado 54 cm o maior que se pode construir usando o maior nmero de cartes.

PROBLEMA 2 a) A maior coluna tem 2008 letras e OBM um bloco de 3 letras. Como 2008 = 669 3 + 1 , o nmero de vezes em que a palavra OBM aparece completamente na maior coluna 669.
b) Da esquerda para a direita, fazendo a contagem ao longo das flechas, a primeira passa por 2008 letras O. Como a segunda inicia 3 linhas abaixo, ela passa por 2008 3 = 2005 letras O. Nesse padro, a prxima passar por 2002 letras O, a seguinte, por 1999, e assim at a ltima flecha, que passar por 1. Portanto o nmero de vezes que a letra O aparece no arranjo
EUREKA! N30, 2009

23

Sociedade Brasileira de Matemtica

2008 + 2005 + 2002 + 1999 +

+1 =

(2008 + 1) 670 = 673015 . 2

PROBLEMA 3 87 a) H = 28 peas com quantidades diferentes de pontos em cada lado e 8 com 2 quantidades iguais, ou seja, o domin de Ferius tem 28 + 8 = 36 peas diferentes.
Outra soluo: O domin comum possui 28 peas. Como existem mais 8 novas peas que possuem alguma casa marcando 7 pontos, o domin de Ferius tem 28 + 8 = 36 peas diferentes. b) Como a soma de um par e um mpar mpar e h 4 quantidades mpares de pontos (1, 3, 5, 7) e 4 quantidades pares de pontos (0, 2, 4, 6), h 4 4 = 16 peas que no so importantes. Logo existem 36 16 = 20 peas importantes. c) Cada quantidade de pontos aparece exatamente 9 vezes. Assim a soma dos pontos de todas as peas 9 (1 + 2 + 3 + + 7) = 252 . A soma dos pontos de todas as peas que no so importantes 4 (1 + 2 + 3 + + 7) = 112 , pois cada quantidade de pontos aparece exatamente 4 vezes em peas que no so importantes. Assim, a soma pedida 252 112 = 140 .

Solues Nvel 2 Segunda Fase Parte A


Problema Resposta
01. De
2 17 = x 4 + y 4 = ( x 2 + y 2 ) 2( xy ) 2 = 1 2( xy ) 2 , 18 1 1 2 = 6. obtemos ( xy ) = , e da xy 36

01 6

02 1420

03 144

04 108

05 22

02. O deslocamento lquido do viajante na direo Leste-Oeste foi de


502 vezes

(1 3) + (5 7) + ... + (2005 2007) = (2) + (2) + ... + (2) = 1004.

EUREKA! N30, 2009

24

Sociedade Brasileira de Matemtica

Analogamente, o deslocamento lquido na direo Norte-Sul foi de 1004. Portanto, pelo teorema de Pitgoras a distncia entre as posies inicial e final do viajante 1004 2. Observe agora que, como 2 1, 414, temos 1004 2 1419,656. Para ter certeza se estamos usando uma aproximao boa o suficiente, basta checar se 1419,5 < 1004 2 < 1420, quer dizer, se (1419,5) 2 < 10042 2 < 14202. Mas fcil efetuar os clculos e verificar que essas desigualdades realmente se verificam. Logo, a melhor aproximao pedida 1420 metros.
03. Veja que + = 1 e

3 = 2 = ( + 1) = 2 + = 2 + 1, 4 = 3 = (2 + 1) = 2 2 + = 3 + 2,
5 = 4 = (3 + 2) = 3 2 + 2 = 5 + 3.
Analogamente,

7 = 4 3 = (5 + 3)( + 1) = 5 2 + 8 + 3 = 13 + 8.
Portanto, 13 5 + 5 7 = 13(5 + 3) + 5(13 + 8) = 65( + ) + 79 = 65 + 79 = 144.
04. Como os dois crculos circunscritos so iguais, segue do teorema do ngulo

inscrito que ACB = ABC e, com isso, AB = AC.

B M D C
Seja AM a altura relativa ao lado BC. Como ABC issceles de base BC, segue que AM tambm mediana, e da MC = 9. Portanto, MD = 5 e, pelo teorema de
EUREKA! N30, 2009

25

Sociedade Brasileira de Matemtica

Pitgoras, AM = 12. Finalmente, 1 1 ( AM )( BC ) = (12 )(18) = 108. 2 2

rea

do

tringulo

ABC

05. Para que o primitivo de um nmero seja mpar, todos os seus algarismos

precisam ser mpares, pois o produto de um nmero par por um nmero qualquer sempre um nmero par. Assim, s nos restam os algarismos 1, 3, 5, 7 e 9 para construir o nmero pretendido. Por outro lado, como os algarismos precisam ser todos diferentes, o nmero ter, no mximo, 5 algarismos. Contudo, qualquer nmero com 5 algarismos mpares e todos distintos tem primitivo 0. De fato, o produto dos nmeros 1, 3, 5, 7 e 9 945 e seu primitivo 0. O maior nmero com 4 algarismos mpares e todos diferentes 9753, mas esse nmero tem primitivo 0. O nmero que o antecede e tem seus 4 algarismos mpares e distintos 9751, e seu primitivo 5. Portanto, a soma de seus algarismos 9 + 7 + 5 + 1 = 22.

Solues Nvel 2 Segunda Fase Parte B


SOLUO DO PROBLEMA 1:

Os catetos do tringulo medem a e b, e a hipotenusa mede c. Como a rea e o 1 1 permetro so iguais, temos ab = a + b + c, e da c = ab a b. Usando o 2 2 teorema de Pitgoras, segue que 1 1 a 2 + b 2 = ( ab a b) 2 = a 2 + b 2 + 2ab a 2 b b 2 a + a 2b 2 , 2 4 ou ainda 8ab 4a 2 b 4b 2 a + a 2 b 2 = 0. . Dividindo por ab, obtemos ( a 4 )( b 4 ) = 8, de maneira que a 4 divide 8. Portanto, os possveis valores de a so 2, 3, 5, 6, 8 e 12. Determinando os valores de b e c, encontramos os tringulos de lados 5, 12, 13 ou 6, 8, 10.
SOLUO DO PROBLEMA 2:

Note que ( 2009 x ) x 2 = 2009 ( 2009 2 x ) , um mltiplo de 2009. Assim, sempre que Pedro apagar um nmero, x2 digamos, basta Igor apagar o nmero (2009 x)2. Desse modo, no final restaro dois nmeros cuja diferena um mltiplo de 2009.
2

EUREKA! N30, 2009

26

Sociedade Brasileira de Matemtica

SOLUO DO PROBLEMA 3:
A D F E

Seja D o p da perpendicular baixada de F a AC. Pelo teorema de Pitgoras, segue que EC = BC 2 BE 2 = 52 42 = 3. Por outro lado, por semelhana de 1 tringulos temos FD = BE = 2 e AE = 2 DE. Portanto, 2

DC = CF 2 FD 2 = 42 22 = 2 3,
e da DE = 2 3 3, de maneira que AE = 4 3 6. Finalmente, 1 1 [ ABC ] = ( AE + EC ) BE = 4 3 6 + 3 4 = 8 3 6. 2 2

SOLUO DO PROBLEMA 4:

H duas escolhas envolvidas e que determinam a maneira de viajar de B a C: por quais dentre as cidades A1 ,..., A6 devemos passar, e em que ordem. Digamos que escolhamos passar por exatamente k dentre as cidades A1 ,..., A6 , com 1 k 6; o

6 nmero de modos de escolher as k cidades . Por outro lado, aps k escolhermos as k cidades, devemos escolher em que ordem vamos visit-las, o que corresponde a k! possibilidades. Logo, o nmero de modos de viajar de B a C 6 6 6 6! 6! 6! 6! = = + + ... + = 1956. ! k 5! 4! 0! k =1 k k =1 ( 6 k ) !

Solues Nvel 3 Segunda Fase Parte A


Problema Resposta 01 7500 02 9779 03 9376 04 256 05 18

EUREKA! N30, 2009

27

Sociedade Brasileira de Matemtica

01. Seja P a projeo ortogonal de B sobre CD.


A B

100

30

Temos que CP =

AB + CD CD AB logo PD = CP + AB = . Assim, a rea do 2 2

trapzio : AB + CD S = BP = BP PD = (100 sen30) (100 cos30) = 2500 3 m 2 e portanto 2 S 3 = 7500.


02. Observe que para i 1 temos

4 n = i i 4 n < i + 1 i 4 n < (i + 1) 4 e assim h ( i + 1)4 i 4 nmeros n tais 4 que n = i. Portanto a soma pedida : 1 ( 24 14 ) + 2 ( 34 24 ) + 3 ( 44 34 ) + 4 ( 54 44 ) + 5 ( 64 54 ) + 6 ( 2008 64 + 1) = 9779.
03. Seja n um inteiro de 4 dgitos. Temos que n auto-replicante se e somente se

n 2 n divisvel por 10000, isto , 24 | n( n 1) e 54 | n(n 1) . Como n e n 1 so primos entre si, temos 4 possibilidades:

2 4 | n e 54 | n

24 | ( n 1) e 54 | ( n 1)
24 | n e 54 | ( n 1) 24 | ( n 1) e 54 | n.

A primeira possibilidade implica que 104 | n, o que impossvel pois 1000 n 9999. Da mesma forma, a segunda no ocorre.
EUREKA! N30, 2009

28

Sociedade Brasileira de Matemtica

Na terceira possibilidade, de 54 | ( n 1) temos que n = 625k + 1 para algum k inteiro e que 625k + 1 0(mod16) k + 1 0(mod16) k 15(mod16) Assim, k = 15 + 16 para algum inteiro e n = 625 (15 + 16 ) + 1 = 9376 + 10000 E como 1000 n 9999, a nica possibilidade n = 9376. Finalmente, para a quarta possibilidade, temos que n = 625k, k inteiro, e que n 1 0(mod16) k 1(mod16) . inteiro, e n = 625(1 + 16 ) = 625 + 10000 . Como Assim, k = 1 + 16 , 1000 n 9999, no h solues neste caso. Logo o nico nmero auto-replicante de 4 dgitos 9376.
04. Da propriedade, decorre que 9 s pode aparecer ou como primeiro ou como

ltimo elemento da permutao e que os elementos de 1 a 8 formam uma permutao com a mesma propriedade. Assim, o nmero pedido o dobro do nmero de permutaes de 1, 2,...,8 com a mesma propriedade. Da mesma forma, o nmero de permutaes de 1, 2,.., 8 com a propriedade o dobro do nmero de permutaes de 1, 2,.., 7 com a propriedade. Repetindo o raciocnio, conclumos que o nmero pedido portanto 28 = 256.
05. Seja = 3 27 + 5 33 3 27 + 5 33. Temos

3 = 27 + 5 33 27 + 5 33 3 3 27 + 5 33 3 27 + 5 33
3 = 54 3 3 96 ( 3 + 54 ) = 25 34 3
3

27 + 5 33 3 27 + 5 33

Agora faa 18 y = 3 . Temos

(18 y + 54 ) = 26 36 y 3 ( y + 3) = 23 y y 3 + 9 y 2 + 35 y + 27 = 0 ( y + 1)( y 2 + 8 y + 27) = 0


3

Como , e portanto y, so reais e y 2 + 8 y + 27 = 0 no tem razes reais, conclumos que y = 1 e portanto = 3 18 (pasmem!). Assim, raiz do polinmio x 3 + 18 = 0, que o polinmio minimal de j que x 3 + 18 = 0 no possui razes racionais.

EUREKA! N30, 2009

29

Sociedade Brasileira de Matemtica

Solues Nvel 3 Segunda Fase Parte B


SOLUO DO PROBLEMA 1:

Olhando a equao mdulo 7, temos: m 2 3n , porm m 2 s poder ser congruente a 0,1,2,4 enquanto que se n for mpar 3n s poder ser congruente a 3, 5, 6, ento n dever ser par. Logo existe n0 tal que n = 2n0 . Voltando equao original temos: m 2 + 161 = 32 n0 32 n0 m 2 = 161 3n0 m 3n0 + m = 161. Como m e n so

inteiro positivos, logo o mdulo de

(3

n0

m positivo e 161 = 7 23, ento temos as opes:


e

(3

)(

n0

m menor que

(3

n0

+ m , e como

e e m = 80 n0 = 4 3n0 m = 1 3n0 + m = 161 3n0 = 81 m = 80 n = 8 e m = 80 3n0 m = 7 e 3n0 + m = 23 3n0 = 15 e m = 8. No h soluo inteira. Logo m = 80 e n= 8 a nica soluo.
SOLUO DO PROBLEMA 2:

Para que f ( f ( x ) ) = f ( x ) ento a imagem de f dever s conter pontos fixos. Utilizando esse fato temos: Com 5 pontos fixos na imagem teremos 1 funo possvel. 5 Com 4 pontos fixos na imagem teremos 4 = 20 funes 1

5 Com 3 pontos fixos na imagem teremos 32 = 90 funes 2 5 Com 2 pontos fixos na imagem teremos 23 = 80 funes 3

5 Com 1 ponto fixo na imagem teremos 14 = 5 funes 4 logo o total de funes f satisfazendo f ( f ( x)) = f ( x) igual a 196.

EUREKA! N30, 2009

30

Sociedade Brasileira de Matemtica

SOLUO DO PROBLEMA 3:
A E F G B

Como ABE ADE (ambos enxergam o arco AE ) temos que FBE FDA e portanto FB BE = (1) FD DA AEG CBG e Analogamente, das semelhanas EBG ACG , AEF DBF obtemos respectivamente BG EB = (2) CG AC

AE AG = (3) CB CG AE AF (4) = DB DF Assim, utilizando o fato que ABCD issceles (de modo que AD = BC e BD = AC) temos AF BG (2) e (4) 1 AE DF CG EB = FG FG DB AC 1 ( AE CG )( DF EB ) (1) e (3) AD 2 AG BF = = AC 2 FG AC 2 FG 2 AD ( AF + FG )( BG + FG ) = FG AC

EUREKA! N30, 2009

31

Sociedade Brasileira de Matemtica

AD FG ( AF + FG + BG ) + AF BG = FG AC
AF BG AD = AB + FG AC Em suma, temos AF BG AD AF BG = AB + FG FG AC AF BG AD 2 AB = FG AC 2 AD 2
Utilizando o fato de que ABCD issceles com base CD = 50 e altura 24, aplicando Pitgoras vrias vezes fcil calcular AB = 14, AD = 30, AC = 40. AF BG Assim, = 18 . FG
SOLUO DO PROBLEMA 4:
2 2

Vamos mostrar que o menor produto obtido quando tomamos os elementos da diagonal principal. Neste caso, o produto dado por 2008 (1 + 1)(2 + 2)(3 + 3)...(2008 + 2008) = 2 2008! Suponha que todos os elementos (1, 1), (2, 2),..., (i 1, i 1) tenham sido escolhidos mas que os elementos nas i simas linha e colunas sejam (i, j) e (k, i) com j e k maiores ou iguais a i + 1. Vamos mostrar que trocando estes dois elementos por (i, i) e (k, j) obtemos um produto menor. De fato, para isto devemos mostrar que

(i + i )( j + k ) < (i + j )(i + k )
2i ( j + k ) < i 2 + ( j + k )i + jk i 2 ( j + k )i + jk > 0 (i j )(i k ) > 0 O que verdade, j que i j < 0 e i k < 0.

EUREKA! N30, 2009

32

Sociedade Brasileira de Matemtica

XXX OLIMPADA BRASILEIRA DE MATEMTICA


Problemas e Solues da Terceira Fase
NVEL 1 (6o. e 7o. Anos)
PROBLEMA 1

Um quadrado de lado 12 foi dividido em sete regies retangulares que no se sobrepem, conforme a figura. Uma delas um quadrado de vrtice C, cuja rea metade da rea de cada um dos dois retngulos vizinhos; outra um quadrado de vrtice A, cuja rea metade da rea de cada um dos dois retngulos vizinhos.

D
a) Mostre que o quadriltero destacado um quadrado. b) Calcule a rea do quadrado destacado.
PROBLEMA 2

Esmeralda escolhe um nmero inteiro positivo qualquer e realiza a seguinte operao com ele: cada um de seus algarismos trocado pelo seu sucessor, com exceo do 9, que trocado por 0. Em seguida, os eventuais zeros que aparecem esquerda so eliminados. Por exemplo, ao se realizar a operao no nmero 990003953 obtm-se 1114064 (note que os dois zeros esquerda gerados pelos dois primeiros algarismos 9 foram eliminados). A operao repetida at que se obtenha 0. Por exemplo, comeando com 889, obtemos a seqncia de nmeros 889, 990, 1, 2, 3, 4, 5, 6, 7, 8, 9, 0 a) Apresente a sequncia de nmeros quando o primeiro nmero 2008.
EUREKA! N30, 2009

33

Sociedade Brasileira de Matemtica

b) Mostre que, independente do nmero inicial, aps uma quantidade finita de operaes Esmeralda obtm 0. Jade tem n peas iguais 3 1 e quer utiliz-las para cobrir um tabuleiro 3 n , sendo n um inteiro positivo. Por exemplo, para n = 4 ela pode cobrir o tabuleiro da seguinte maneira:
PROBLEMA 3

a) Determine de quantas maneiras Jade pode fazer a cobertura para n = 1, 2, 3, 4, 5, 6, 7. b) De quantas maneiras Jade pode cobrir o tabuleiro para n = 15?
PROBLEMA 4

Considere o seguinte hexgono:

Com cpias desse polgono podemos cobrir todo o plano, sem sobreposies, como mostra a figura a seguir.

a) possvel cobrir o plano com cpias de um pentgono regular?


EUREKA! N30, 2009

34

Sociedade Brasileira de Matemtica

Observao: um polgono regular quando todos os seus lados so de mesma medida e todos os seus ngulos internos so iguais. b) Seja ABCDE um pentgono com todos os lados iguais e tal que a medida do ) = 100 e m( B ) = 80 . Mostre como ngulo interno nos vrtices A e B so m( A possvel cobrir todo o plano com cpias desse pentgono, sem sobreposies.
PROBLEMA 5

Vamos chamar de garboso o nmero que possui um mltiplo cujas quatro primeiras casas de sua representao decimal so 2008. Por exemplo, 7 garboso pois 200858 mltiplo de 7 e comea com 2008. Observe que 200858 = 28694 7 . a) Mostre que 17 garboso. b) Mostre que todos os inteiros positivos so garbosos.

TERCEIRA FASE NVEL 2 (8o. e 9o. Anos)


PRIMEIRO DIA PROBLEMA 1

Em cada casa de um tabuleiro n n , colocamos um dos nmeros 1,2,3,4, de modo que cada casa tem exatamente uma casa vizinha com o mesmo nmero. possvel fazer isso quando a) b)

n = 2007 ? n = 2008 ?

Observao. Duas casas so vizinhas se possuem um lado em comum. PROBLEMA 2

Seja P um pentgono convexo com todos os lados iguais. Prove que se dois dos ngulos de P somam 180 graus, ento possvel cobrir o plano com P, sem sobreposies.
PROBLEMA 3

Prove que existem infinitos inteiros positivos n tais que 5 n2 1 n um inteiro.


EUREKA! N30, 2009

35

Sociedade Brasileira de Matemtica

TERCEIRA FASE NVEL 2 (8o. e 9o. Anos)


SEGUNDO DIA PROBLEMA 4

Mostre que se p,q so inteiros positivos primos tais que r = ento r primo.
PROBLEMA 5

p2 + q2 inteiro, p+q

Seja ABC um tringulo acutngulo e O, H seu circuncentro e ortocentro, respectivamente. Sabendo que

AB 2
calcule os ngulos do tringulo ABC.
PROBLEMA 6

= BH = OB,

Sendo A um conjunto de nmeros inteiros, definimos S(A) como o conjunto formado pelas somas de dois elementos, no necessariamente distintos e D(A) como o conjunto formado pelas diferenas de dois elementos, no necessariamente distintos. Por exemplo, se A = {1, 2, 3, 10} ento S(A) = {2, 3, 4, 5, 6, 11, 12, 13, 20} e D(A) = {9, 8, 7, 2, 1, 0, 1, 2, 7, 8, 9}. Mostre que existe um conjunto finito A tal que S(A) tem no mximo 1097 elementos e D(A) tem no mnimo 10100 elementos.

TERCEIRA FASE NVEL 3 (Ensino Mdio)


PRIMEIRO DIA PROBLEMA 1

Vamos chamar de garboso o nmero que possui um mltiplo cujas quatro primeiras casas de sua representao decimal so 2008. Por exemplo, 7 garboso pois 200858 mltiplo de 7 e comea com 2008. Observe que 200858 = 28694 7 . Mostre que todos os inteiros positivos so garbosos.
PROBLEMA 2

Sobre uma reta h um conjunto S de 6n pontos. Destes, 4n so escolhidos ao acaso e pintados de azul; os 2n demais so pintados de verde. Prove que existe um segmento que contm exatamente 3n pontos de S, sendo 2n pintados de azul e n pintados de verde.
EUREKA! N30, 2009

36

Sociedade Brasileira de Matemtica

PROBLEMA 3

Sejam x, y, z reais quaisquer tais que x + y + z = xy + yz + zx. Encontre o valor mnimo de y x z + 2 + 2 2 x +1 y +1 z +1

TERCEIRA FASE NVEL 3 (Ensino Mdio)


SEGUNDO DIA PROBLEMA 4

Seja ABCD um quadriltero cclico e r e s as retas simtricas reta AB em relao s bissetrizes internas dos ngulos CAD e CBD , respectivamente. Sendo P a interseo de r e s e O o centro do crculo circunscrito a ABCD, prove que OP perpendicular a CD.
PROBLEMA 5

Prove que para quaisquer inteiros a > 1 e b > 1 existe uma funo f dos inteiros positivos nos inteiros positivos tal que f (a f ( n)) = b n para todo n inteiro positivo.
PROBLEMA 6

O profeta venusiano Zabruberson enviou a seus discpulos uma palavra de 10000 letras, sendo cada uma delas A ou E: a Palavra Zabrbica. Seus seguidores passaram a considerar, para 1 k 10000 , cada palavra formada por k letras consecutivas da Palavra Zabrbica uma palavra proftica de tamanho k. Sabe-se que h no mximo 7 palavras profticas de tamanho 3. Determine o nmero mximo de palavras profticas de tamanho 10.

SOLUES TERCEIRA FASE NVEL 1 (6o. e 7o. Anos)


PROBLEMA 1 SOLUO DE LUCAS CAWAI JULIO (CAUCAIA CE)

a) Vamos chamar o lado do quadrado de vrtice C de x, e o lado do quadrado de vrtice A de y. Como os retngulos que esto vizinhos a esses quadrados tm o dobro da rea deles, ento eles iro ter a largura com a mesma medida dos quadrados e comprimento, igual ao dobro do lado do quadrado. Veja a figura:
EUREKA! N30, 2009

37

Sociedade Brasileira de Matemtica

A y

2y

2x 2y 2x y

x D

2x 2x

y x

x C

Podemos ver que um lado do quadrado maior mede 3x. Para calcularmos o lado do quadriltero central, basta retirarmos o que no pertence a ele. Logo, retiraremos x + y . Mas isso ocorrer dos dois lados, ento os dois lados do quadriltero destacado so iguais a 2x y. Assim temos que ele um quadrado. b) Como um lado do quadrado maior 12, e j havamos falado que tambm igual a 3x. Logo x = 4. Mas tambm podemos perceber que a medida 2x 8 equivalente a 3y. Como x = 4, ento y = . 3 Agora, como o lado do quadrado destacado 2x y, ento sua rea

( 2x y )
256 . 9

. Substituindo x e y, e resolvendo temos que a rea do quadrado destacado

PROBLEMA 2 SOLUO DE LARA VIANA DE PAULA CABRAL e RAFAEL RODRIGUES ROCHA DE MELO (FORTALEZA CE)

a) A sequencia 2008, 3119, 4220, 5331, 6442, 7553, 8664, 9775, 886, 997, 8, 9, 0 b) Independente do dgito que ocupa a 1 posio do nmero, aps uma certa quantidade de operaes, ele chegar a 9 e, basta mais uma operao para ele chegar a 0, que desaparecer, e o nmero ficar assim com um dgito a menos. Em seguida, independente do dgito que agora ocupa a 2 posio, aps uma certa quantidade de operaes ele tambm chegar a 9 e, logo depois, a 0, que tambm desaparecer, e o nmero ter assim outro dgito a menos. Continuando esse processo at o nmero ter um nico dgito, esse dgito tambm chegar a 9 e, depois, a 0, encerrando o processo.
EUREKA! N30, 2009

38

Sociedade Brasileira de Matemtica

PROBLEMA 3 SOLUO OFICIAL DA BANCA

Seja fn o nmero de maneiras possveis de cobrir o tabuleiro 3 n . Se a primeira coluna coberta por uma pea vertical, falta cobrir um tabuleiro 3 ( n 1) . Seno, comeamos com trs peas na horizontal, e falta cobrir um tabuleiro 3 ( n 3) . Assim, temos f n = f n-1 + f n- 3 , para todo n 4. Como claramente temos f1 = 1, f 2 = 1 e f 3 = 2 , temos f4 = f3 + f1 = 3, f5 = f4 + f2 = 4, f6 = f5 + f3 = 6, f7 = f6 + f4 = 9,
f8 = f7 + f5 = 13, f9 = f8 + f6 = 19, f10 = f9 + f7 = 28, f11 = f10 + f8 = 41, f12 = f11 + f9 = 60, f13 = f12 + f10 = 88, f14 = f13 + f11 = 129 e, finalmente, f15 = f14 + f12 = 189.

Assim, as respostas so: a) 1, 1, 2, 3, 4, 6, 9, respectivamente. b) De 189 maneiras.


PROBLEMA 4 SOLUO OFICIAL DA BANCA

a) No possvel. Para que seja possvel cobrir o plano com uma figura, em cada vrtice determinado pelas figuras que a cobrem a soma dos ngulos internos deve ser 180 ou 360:

Soma 180

Soma 360

Todo pentgono pode ser cortado em trs tringulos, de modo que a soma de seus ngulos internos 3 180 = 540 . Assim, cada ngulo interno de um pentgono regular

540 = 108 . Como 108 < 180 < 2 108 e 3 108 < 360 < 4 108 , 5

no possvel cobrir o plano com cpias de um pentgono regular.

) + m( B ) = 180 , EA e BC so paralelos, de modo que b) Note que, como m( A EABC um losango. Assim CE = DE = CD e CDE um tringulo equiltero.
EUREKA! N30, 2009

39

Sociedade Brasileira de Matemtica

Assim possvel cobrir o plano com o pentgono ABCDE, como mostra a figura a seguir:
A B

PROBLEMA 5 SOLUO ADAPTADA DA SOLUO DE GABRIEL YASHIMI BARRN TOYAMA (SO PAULO SP)

a) Observe que 200800 dividido por 17 tem resto 13. Assim, 200804 mltiplo de 17 e, portanto, 17 garboso. Na verdade, 17 tem infinitos mltiplos comeados por 2008. b) Seja x a quantidade de algarismos de um nmero inteiro positivo y qualquer. Considere o resto m da diviso de 2008 10 x por y. Temos 0 m y 1, e portanto

1 y m y. Como y tem x algarismos, y < 10 x , e logo 1 y m y < 10 x . Assim,


y m tem no mximo x algarismos, e portanto 2008 10 x + ( y m ) comea sua
representao decimal por 2008. Como 2008 10 x = y z + m, para algum inteiro z,

2008 10 x + ( y m ) = y ( z + 1) mltiplo de y, e portanto y garboso.


SOLUES TERCEIRA FASE NVEL 2 (8o. e 9o. Anos)
PROBLEMA 1 SOLUO DE DANIEL DOS SANTOS BOSSLE (PORTO ALEGRE RS)

Perceba que a distribuio dos nmeros no tabuleiro forma domins 2 1, pois a cada casa est associada exatamente uma casa vizinha com o mesmo nmero. Logo, para que todos os domins se encaixem, deve haver um nmero par de casas no tabuleiro. Assim, impossvel cobrir um tabuleiro 2007 2007.
EUREKA! N30, 2009

40

Sociedade Brasileira de Matemtica

Por outro lado, possvel cobrir um 2008 2008. Uma soluo a seguinte, bastando repetir o padro at o fim:

1 2 3 4
Assim, as respostas so: a) No b) Sim

1 2 3 4

2 3 4 1

2 3 4 1

3 4 1 2

3 4 1 2

4 1 2 3

4 1 2 3

PROBLEMA 2 SOLUO DE JOO LUCAS CAMELO S (FORTALEZA CE)

Suponha que os ngulos suplementares sejam adjacentes. Vamos cham-los de A e B e os outros de C , D e E . Observe a montagem a seguir:
r

A E

A E
D

C
B

C
D

D
C E

D C
E B
s

E A

Como

A + B + C + D + E = 180(5 2) = 540

A + B = 180,

temos

que

C + D + E = 360. Logo, possvel encaixar os pentgonos desta maneira, em faixas. Ao encaixarmos faixa sobre a outra pelas retas r e s da figura, poderemos cobrir o plano inteiro.
Temos agora que analisar o caso quando os suplementares (dessa vez A e C ) no so adjacentes. Sendo B o ngulo do vrtice entre A e C , e D e E os outros ngulos, temos a seguinte configurao: (Lembrando que B + D + E = 360 )

EUREKA! N30, 2009

41

Sociedade Brasileira de Matemtica

B A
C D
E B

D
D

B E

D C A

Hexgono

Vamos mostrar que podemos agrupar vrios hexgonos de modo a cobrir o plano. Basta seguir as faixas abaixo: Faixas:
B D E D

E
B D E

B E
D

B
E

B
D E

D
D

Como os ngulos de fora valem 360 B E = D e os da ponta tambm, possvel encaixar, cobrindo todo o plano.

EUREKA! N30, 2009

42

Sociedade Brasileira de Matemtica

PROBLEMA 3 SOLUO DE JOO LUCAS CAMELO S (FORTALEZA CE)

Seja p um primo 3 e diferente de 5. Temos 52 p 2 1 52( p 1) 1 (5( p 1) 1) (5( p 1) + 1) = = . Analisando mdulo p, pelo pequeno 2p 2p 2 p Teorema 5
p 1

de
p 1

Fermat,

5 p 1 1(mod p ) 5 p 1 1 0(mod p )

1(mod 2) 5

+ 1 0(mod 2).

5 p 1 + 1 5n 2 1 inteiro inteiro quando n = 2p. 2 p n Como existem infinitos primos p, existem infinitos n que satisfazem a condio do enunciado.
Assim,

p 1

inteiro e

PROBLEMA 4 SOLUO DE JOO LUCAS CAMELO S (FORTALEZA CE)

Suponha p = q Caso

p 2 + q 2 2q 2 = = q r = q r primo. p+q 2q p 2 + q 2 p 2 q 2 + 2q 2 2q 2 = = pq+ p + q | 2q 2 . p+q p+q p+q


p q,( p, q) = 1 (2 p 2 , 2q 2 ) = 2( p 2 , q 2 ) = 2.

contrrio,

Analogamente, p + q | 2 p 2 . Como

Logo p + q | (2 p 2 , 2q 2 ) p + q | 2 p + q 2. Mas p = q , e portanto r primo. Obs.: Joo Lucas utilizou a notao (a, b) = mdc(a, b).
PROBLEMA 5 SOLUO DE MARIA CLARA MENDES SILVA (PIRAJUBA MG)
A 30 45

p, q 2, absurdo. Logo,

M 120 O 150 C 30 = 15

45 15 =

EUREKA! N30, 2009

43

Sociedade Brasileira de Matemtica

O circuncentro equidistante dos 3 vrtices. AB BH = OC = OA = OB = . 2

AB = 2OB. A AB 2 = 2OB 2 = OB 2 + OA2 OAB retngulo em O pela recproca do Teorema de Pitgoras. Como OA = OB, ele tambm issceles e
O AB = OBA = 45 . Seja M o ponto mdio de AC. OM perpendicular a AC, e BH OA temos que OM = . A AO o dobro de OM, logo sen M AO = 0,5 e = 2 2 como M AO 0, , M AO = 30. Logo OCM = 30, j que COA issceles. 2

Assim COA = 180 60 = 120 e COB = 360 90 120 = 150. 180 150 Finalmente = = 15 . Os ngulos so: 2 30 + 45 = 75,30 + 15 = 45 e 45 + 15 = 60.
PROBLEMA 6 SOLUO OFICIAL DA BANCA

Considere o conjunto C = {0,1,3}. Temos S (C ) = {0,1, 2,3, 4,6} e D(C ) = {3, 2, 1,0,1, 2,3}. Assim, S(C) tem 6 elementos, enquanto D(C) tem 7. Vamos agora, para cada inteiro positivo n, considerar o conjunto An dos naturais com no mximo n algarismos na base 7, todos pertencentes a C, isto , n 1 n 1 n 1 A( n ) = a j 7 j ; a j C ,0 j n 1 . Dados a = a j 7 j e b = b j 7 j em j =0 j =0 j =0

An , com

a j , bj C,

para

0 j n 1,

temos

a + b = ( a j + bj ) 7 j
j =0

n 1

n 1 n 1 a b = ( a j b j ) 7 j. Assim, temos S ( An ) = u j 7 j , u j S (C ),0 j n 1 j =0 j =0 n 1 e D ( An ) = v j 7 j , v j D (C ),0 j n 1 . j =0 Como S(C) tem 6 elementos entre 0 e 6, e a representao em base 7 nica, S( An )

tem exatamente 6n elementos. Por outro lado, como


EUREKA! N30, 2009

7 n 1 n 1 = 3 7 j , temos 2 j =0

44

Sociedade Brasileira de Matemtica

7n 1 7n 1 + D ( An ) : = + m, m D ( An ) = 2 2 n1 n1 = ( 3 + v j ) 7 j , v j D(C),0 j n 1 = rj 7 j , rj {0,1,2,3,4,5,6},0 j n 1 = j =0 j =0 n n pois todo inteiro entre 0 e pode ser representado na base 7, 7 1 = {0,1, 2,...,7 1} ,
usando os algarismos 0, 1, 2, 3, 4, 5 e 6. Assim, D( An ) tem 7n elementos. Como
105 10100 < 105 < 76 ,6120 < 20 < 1097 < 10100 < 7120 , e portanto o conjunto 2 2 A = A120 , que tem 3120 elementos, certamente satisfaz as condies do enunciado. 66 <

SOLUES TERCEIRA FASE NVEL 3 (ENSINO MDIO)


PROBLEMA 1 SOLUO DE CUSTDIO M. B. SILVA

Seja n um inteiro positivo. Como n inteiro finito, n < 10k , para algum k. Seja p = 10k 2008 + n q , onde q < n o resto da diviso de 10k 2008 por n. Assim, n q < 10k e portanto p comea com 2008 e mltiplo de n.
PROBLEMA 2 SOLUO DE RAFAEL SUSSUMU YAMAGUTI MIADA (SO PAULO SP)

Considere que os pontos so numerados de 1 a 6n. Sabe-se que, para 1 b 3n + 1, um segmento de b at 3n + b 1 contm exatamente 3n pontos e ser representado como b 3n + b 1. Como os pontos devem ser consecutivos, pode-se formar 3n + 1 segmentos (1 3n;2 3n + 1;3 3n + 2,...,3n + 1 6n) . Vamos analisar a variao do nmero de pontos verdes de b 3n + b 1 at b + 1 3n + b. Considere que em b 3n + b 1 h z pontos verdes. Pode acontecer: z pontos z 1 pontos: z pontos z pontos: z pontos z pontos:
EUREKA! N30, 2009

Sai um ponto verde e no entra outro ponto verde no segmento. Sai um ponto verde e entra outro ponto verde no segmento. No sai um ponto verde e no entra outro ponto

45

Sociedade Brasileira de Matemtica

verde no segmento. z pontos z + 1 pontos:


(quantidade de pontos verdes)

No sai um ponto verde e entra outro ponto verde no segmento.


(em relao ao segmento anterior)

Maior variao: 1 ponto para mais ou para menos. Considere ento os pontos de 1 3n e 3n + 1 6n (usando o fato de que h 2n pontos verdes e 4n pontos azuis). Se em 1 3n h n + k pontos verdes, em 3n + 1 > 6n haver n k pontos verdes. Alm disso, em 1 3n haver 2n k pontos azuis e em 3n 1 6n haver 2n+ k pontos azuis. Temos os seguintes casos: a) para k = 0: verdadeiro na primeira e ltima sequncia (1 3n e 3n + 1 6n) (verdadeiro!). b) para k < 0: deve aumentar o nmero de pontos verdes de 1 3n a 3n + 1 6n, porm com a mxima variao entre cada sequncia 1 ponto e n + k < n < n k , conclui-se que existe a a + 3n 1 talque o nmero de pontos verdes igual a n (verdadeiro!). O caso k > 0 anlogo. Como h 3n pontos na sequncia a a + 3n 1 e os pontos so verdes ou azuis, pode-se qualificar o fato de que existe uma sequncia a a + 3n 1 tal que h n pontos verdes e 2n pontos azuis como verdadeiro.
PROBLEMA 3 SOLUO DE RGIS PRADO BARBOSA (FORTALEZA CE)

Para (x, y, z) = (1, 1, 1), temos x + y + z = 1 1 + 1 = 1 xy + yz + zx = (1)(1) + (1)1 + (1)1 = 1 e x y z 1 1 1 1 + 2 + 2 = + = . 2 x +1 y +1 z +1 2 2 2 2

1 o mnimo, ou seja, sendo x + y + z = xy + yz + zx 2 x y z 1 mostraremos que 2 + 2 + 2 . x +1 y +1 z +1 2


Provaremos que A desigualdade equivalente a
EUREKA! N30, 2009

46

Sociedade Brasileira de Matemtica

2 x( y 2 + 1 )( z 2 + 1 ) ( x2 + 1 )( y 2 + 1 )( z 2 + 1 ) 2( xy 2 z 2 + xy 2 + xz 2 + x ) ( x2 y 2 z 2 + x2 y 2 + y 2 z 2 + z 2 x2 + x2 + y 2 + z 2 + 1 ) ( x2 y 2 + x2 + 2 xy 2 z 2 + 2 x ) + 2 x2 y + x2 y 2 z 2 + 1 0
cic cic sim cic

(usamos as anotaes

cic

sim

para denotar soma cclica e soma simtrica

respectivamente). Mas xy 2 z 2 = xyz( xy + yz + zx ) = xyz( x + y + z ) = x 2 yz e


cic cic

x
sim

y = x y + xy + x z + x z + y z + yz + 3 xyz 3 xyz
2 2 2 2 2 2

= xy ( x + y + z ) + yz ( x + y + z ) + zx ( x + y + z ) 3 xyz = ( xy + yz + zx )( x + y + z ) 3xyz = ( x + y + z ) 3 xyz


2

Assim, a desigualdade equivalente a

( x
cic

y 2 + x 2 + 2 x 2 yz ) + 2 x + 2 ( x + y + z ) 6 xyz + x 2 y 2 z 2 + 1 0
2

Agora montemos quadrados:

x 2 2 xyz + y 2 z 2 = ( x yz ) 2
y 2 2 xyz + x 2 z 2 = ( y zx) 2 z 2 2 xyz + x 2 y 2 = ( z xy ) 2 ( x + y + z ) 2 + 2( x + y + z ) + 1 = ( x + y + z + 1) 2 ( x + y + z ) 2 + 2 xyz ( x + y + z ) + x 2 y 2 z 2 = ( x + y + z + xyz ) 2
Observando que

x
cic

yz = xyz ( x + y + z ) , a desigualdade equivalente a

( x yz ) 2 + ( y zx) 2 + ( z xy ) 2 + ( x + y + z + 1) 2 + ( x + y + z + xyz ) 2 0 ,
que verdadeira pois A 2 0 para todo A real. Logo o mnimo da soma dada

1 . 2

PROBLEMA 4 SOLUO DE MARCO ANTONIO LOPES PEDROSO (SANTA ISABEL SP)

Para termos as bissetriz de C AD e C BD determinados no precisa saber as posies dos pontos C e D, basta a posio do ponto mdio do arco DC , que
EUREKA! N30, 2009

47

Sociedade Brasileira de Matemtica

vamos chamar de L. Perceba tambm que a mediatriz de CD perpendicular a CD e passa por O; desse modo o nosso problema passa a ser provar que P tambm est na mediatriz de CD. Mas j sabemos que L est na mediatriz de CD, ento na realidade queremos provar que O, P, L so colineares (agora podemos esquecer o C e o D e pensar s no L). Ento nosso problema passa a ser:
L

Provar que O,P,L so colineares.

Perceba que LA a bissetriz externa do PAB relativa ao vrtice A; e LB a bissetriz externa do PAB relativa ao vrtice B. Logo L o ex-incentro do PAB relativo a P; desse modo PL bissetriz do ngulo

APB.
natural pensar no incentro I do PAB; como PL bissetriz de APB ento P, I, L so colineares.
L

90 90

90 90 I

P
EUREKA! N30, 2009

48

Sociedade Brasileira de Matemtica

Como bissetriz interna e externa de um ngulo so perpendiculares ento IA AL; assim como LB BI , desse modo temos L AI = 90 = LBI o quadriltero LAIB inscritvel, e seu centro est no centro da hipotenusa do LAI . Logo O est no ponto mdio de LI (pois o centro da circunferncia que passa por L, A, B). Ento L, O, I so colineares, e como j provamos que P, I, L so colineares ento P, O, L so colineares, como queramos demonstrar.
PROBLEMA 5 SOLUO DE GABRIEL LUIS MELLO DALALIO (S.J. DOS CAMPOS SP)

Seja S n a sequncia crescente dos inteiros positivos no mltiplos de a e Rn a sequncia crescente dos inteiros positivos no mltiplos de b. Definindo f : * *:

se a |n, f (n)=Rk , onde k tal que S k = n f (n)= se n = ak, com k *, b|k, f (n) = b Si onde i tal que Ri = k, se n = abj, com j *, f ( n) = ab f ( j ) Devemos provar que a recurso acaba, mas de fato, como ao passar pelo terceiro caso precisamos do valor da funo em um nmero j < n , j que n = abj, e portanto alguma hora a recurso cai em algum dos dois primeiros casos. Vamos provar que f (af (n)) = bn para todo n inteiro positivo. Temos os seguintes casos: 1) a | n f ( af (n)) = f ( a Rk ), onde S k = n, e temos f (a Rk ) = b S k =bn , donde f (a f (n)) = bn quando a | n .
2) a |n . Temos dois subcasos: 2.i) n = ak, b | k f (a f (n)) = f ( af (ak)) = f (ab . Si), onde Ri = k, e temos

f (ab Si) = abf(Si) = abRi = bak = bn f(af(n)) = bn quando n = ak e b | k .


2.ii) n = abj, j

* f (a f (n)) = f (af (abj)) = f (ab a f( j)) = ab f(af (j)).

Se ab | j, f (af ( j )) = bj f (af (n)) = ab bj = bn. Se ab | j, j=abi,i *, i < j. Podemos supor, por induo, que f(a f(j)) = bj, donde f ( af (n)) = ab f (af ( j )) = ab bj = bn, c.q.d.
PROBLEMA 6 SOLUO OFICIAL DA BANCA

Seja f(n) o nmero de palavras profticas de tamanho n.


EUREKA! N30, 2009

49

Sociedade Brasileira de Matemtica

Temos f (1) 2, f (2) 4 e f (3) 7. Assim, h uma palavra XYZ de trs letras X,Y,Z pertencentes a {A,E} que no proftica. Para n 1, uma palavra profticas de tamanho n + 3 pode ser de trs tipos (no mximo): - uma palavra proftica de tamanho n + 2 seguida da letra U {A, E} distinta de Z, - uma palavra proftica de tamanho n + 1 seguida de TZ, onde T { A,E} a letra distinta de Y ou uma palavra proftica de tamanho n seguida de SYZ, onde S { A,E} a letra distinta de X. f ( n + 3) f ( n + 2) + f ( n + 1) + f ( n ) para todo n 1; logo, Assim, f (4) 13, f (5) 24, f (6) 44, f (7) 81, f (8) 149, f (9) 274 e f (10) 504. Vamos agora ver que possvel que haja 504 palavras profticas de tamanho 10. Para isso observamos inicialmente que h 504 palavras de tamanho 10 que no tm trs letras E consecutivas. Para n 1, uma palavra de tamanho n + 3 sem 3 Es seguidos pode ser de trs tipos, todos distintos: uma palavra sem 3 Es seguidos de tamanho n + 2 seguida da letra A, uma palavra sem 3 Es seguidos de tamanho n + 1 seguida de AE ou uma palavra sem 3 Es seguidos de tamanho n seguida de AEE. Isso mostra que, se g(n) o nmero de palavras de n letras, todas A ou E, sem 3 Es consecutivos, ento g(n + 3) = g(n + 2) + g(n + 1) + g(n) para todo n 1. Como g(1) = 2, g(2) = 4 e g(3) = 7, segue que g(10) = 504. Agora; como 11 504 < 10000, basta listar todas essas palavras, colocar uma letra A no final de cada uma delas e concaten-las, completando com letras A at obtermos uma palavra de10.000 letras para concluir.

EUREKA! N30, 2009

50

Sociedade Brasileira de Matemtica

XXX OLIMPADA BRASILEIRA DE MATEMTICA


Problemas e Solues da Primeira Fase Nvel Universitrio
PROBLEMA 1

Determine todos os valores inteiros de n para os quais a equao x 3 13 x + n = 0 possua trs razes inteiras.
PROBLEMA 2

Considere as retas de equaes paramtricas

( x, y, z ) = (0,0,1) t ( x, y, z ) = (1, 2,0) + (1,0,0) t ( x, y, z ) = (1,1,1) + (0,1,0) t ( x, y, z ) = (1,0,0) + (1,1,1) t Quantas retas intersectam simultaneamente as 4 retas acima?
PROBLEMA 3

Esmeralda passeia pelos pontos de coordenadas inteiras do plano. Se, num dado momento, ela est no ponto (a, b), com um passo ela pode ir para um dos seguintes pontos: (a +1, b), (a 1, b), (a,b + 1) ou (a, b 1). De quantas maneiras Esmeralda pode sair do (0, 0) e andar 2008 passos terminando no (0,0)?
PROBLEMA 4

Suponha que existem duas matrizes inversveis n n , A e B, diferentes da matriz identidade I e satisfazendo as relaes 7 A = I 2 1 ABA = B Mostre que existe um inteiro k > 0 tal que B k = I e determine o menor k com esta propriedade.
PROBLEMA 5

Dizemos que uma hiprbole cobre um ponto se este pertence a uma das duas regies infinitas por ela determinada que contm os focos. Qual o menor nmero de hiprboles necessrias para cobrir todos os pontos do plano?
EUREKA! N30, 2009

51

Sociedade Brasileira de Matemtica

PROBLEMA 6

Seja Pn =

0 k n

sen

k . n

Calcule lim

Pn Pn +1 . n

SOLUES PRIMEIRA FASE NVEL UNIVERSITRIO


PROBLEMA 1

Sejam , , as trs razes do polinmio. As relaes de Girard implicam que

s = + + = 0 e p = + + = 13, logo 2 + 2 + 2 = s 2 2 p = 26. As nicas possibilidades para { , , }:{+4, 3, 1} ou {4, +3, +1}. Logo n = = 12.
PROBLEMA 2 PRIMEIRA SOLUO

Sejam A = (0,0, a), B = (1 + b,2,0), C = (1,1 + c,1) e D = (1 + d , d , d ) pontos genricos, um sobre cada uma das 4 retas dadas. Esses pontos so colineares se, e somente se, a matriz

0 0 1 + b 2 M = 1 1+ c d 1 + d tem posto 2. Subtraindo a primeira linha de equivalente

a 1 0 1 1 1 d 1 M das demais obtemos a matriz

a 0 1 0 1 + b 2 a 0 M = 1 1 + c 1 a 0 d d a 0 1 + d que tem posto 2 se, e somente se 2 a d d a = e 1+ d = = . 1+ b = 1+ c 1 a 1+ c 1 a


Trs dessas quatro igualdades nos permitem expressar b, c e d em funo de a:
EUREKA! N30, 2009

52

Sociedade Brasileira de Matemtica

1 a2 1 ,c = , d = ; a quarta, ento, equivale a 2a 2 a 2 = 0, equao que a 1 a a possui duas solues reais. Logo h duas retas que intersectam simultaneamente as 4 retas dadas. b=
SEGUNDA SOLUO

As coordenadas de Plcker das quatro retas so:


r1 : 0,0,1 0,0,0 r2 : 1,0,0 0,0, 2

r3 : 0,1,0 1,0,1
r4 : 1,1,1 0, 1,1 Qualquer soluo r : d x , d y , d z px , p y , pz como rd rp = 0, temos que ter tem que ser ortogonal s quatro retas. Resolvendo o sistema linear, temos que r : 2 , ,0 , 2 + , ; finalmente,

2 + 3 2 2 = 0

3 17 = 2

logo existem duas retas que intersectam as quatro retas dadas.


PROBLEMA 3

Cada movimento de subida

descida ( ) , e cada movimento para a esquerda ( ) deve ser compensado por um movimento para a direita ( ) . Assim, se fizermos k movimentos , temos que fazer tambm k movimentos , 1004 k movimentos e 1004 k movimentos . Para cada k, o nmero de caminhos , portanto, igual ao nmero de anagramas com 4 letras distintas, duas aparecendo k vezes e as outras duas, 1004 k vezes cada. Logo a resposta R=
1004

()

deva ser compensado por um movimento de

2008! ! !(1004 k )!(1004 k )! k k k =0

EUREKA! N30, 2009

53

Sociedade Brasileira de Matemtica

1004

2008! 1004!1004! k = 0 1004!1004! k !k !(1004 k )!(1004 k )!


2

2008 1004 1004 = . 1004 k = 0 k


Considere agora um conjunto de n meninos e n meninas. De quantas maneiras 2n podemos escolher um grupo de n crianas? Por um lado, a resposta . n

n n n Por outro lado, se escolhermos k meninos = maneiras de formar k n k k um grupo. Logo n 2n = k =0 k n


n
2

e portanto

2008 R= . 1004
SEGUNDA SOLUO

2008 Esmeralda tem maneiras de escolher dois conjuntos de 1004 passos dentre 1004 os 2008 passos que andar: o conjunto X dos passos para cima ou para a direita ( ou ) e o conjunto Y dos passos para baixo ou para a direita ( ou ) .
Essas escolhas determinam unicamente todos os passos: O conjunto dos passos para a direita ser X Y , para a esquerda ser X c Y c , para cima X Y c e para baixo X c Y (onde X c e Y c denotam os complementares de X e Y, respectivamente). Se X Y = k , teremos X c Y = 1004 k , X Y c = 1004 k

2008 e X c Y c = k . Assim, a resposta . 1004


PROBLEMA 4

Note que B 4 = ( B 2 ) = ( ABA1 ) = AB 2 A1 = A ( ABA1 ) A1 = A2 BA2 .


2 2

De forma anloga,

EUREKA! N30, 2009

54

Sociedade Brasileira de Matemtica

B 8 = A3 BA3 , B16 = A4 BA4 , B 32 = A5 BA5 , B 64 = A6 BA6 , B128 = A7 BA7 = B,


127

logo

B = I. Suponha agora que existe 0 < k < 127 tal que B k = I ; como 127 primo, o m.d.c. entre 127 e k vale 1. Pelo Teorema de Bzout, existem a, b inteiros tais que 127 a + kb = 1; ento
B = B1 = B127 a + kb = ( B127 ) ( B k ) = I .
a b

Isso uma contradio, pois B I . Logo o menor valor de k 127. Nota: No necessrio exibir exemplos de tais matrizes A e B, mas tais exemplos existem. Podemos fazer n = 127, enumerar uma base de 127 como {e0 , e1 ,..., e126 } e definir A e B por Ae j = e2 j (mod127) e Be j = e2 j +1(mod127) ,0 j 126.
PROBLEMA 5

Como toda hiprbole tem duas assntotas no paralelas, dadas duas hiprboles, sempre existe pelo menos um ponto comum a uma assntota de cada uma delas. Esse ponto no coberto por qualquer uma das duas hiprboles, logo impossvel cobrir todo o plano com apenas duas hiprboles. As seguintes trs hiprboles cobrem todo o plano: x2 y 2 = 1 ( y 2) 2 x 2 = 1 ( y + 2) 2 x 2 = 1 De fato, para qualquer ( x, y ) x > y + 1,
2 2 2

, vale pelo menos das seguintes desigualdades: ou


2

max

{( y 2)

x < ( y 1) 1
2 2

1, ( y + 2 ) 1 = ( y + 2 ) 1 = y 2 + 4 y + 3 > y 2 + 1.
2

x 2 < ( y + 2) 2 1.

Com

efeito,

Assim o nmero mnimo de hiprboles necessrias para cobrir todos os pontos do plano 3.
PROBLEMA 6

Observe inicialmente que

( Pn 1) < 0 senn xdx < ( Pn + 1) .


n

Defina I n = sen n xdx. Integrando por partes, temos que, para n > 2,
0

I n = sen n 1 xsenxdx =
0

EUREKA! N30, 2009

55

Sociedade Brasileira de Matemtica


n 1 n2 = sen x cos x (n 1) sen x cos x( cos x)dx = 0 0

= ( n 1) sen
0

n2

x(1 sen 2 x) dx =

= ( n 1) I n 2 (n 1) I n
e portanto I n = I n 1 I n 2 ; da segue que lim n = 1. Como, para todo n, n I n n2 In I n 1 I n , temos lim = 1. n I n 1

I n2

Como I1 = 2 e I 2 =

, temos que para todo k 0,

(2k )!! (2k + 1)!! 2, I 2 k + 2 = , (2k + 1)!! (2k + 2)!! 2 (2k + 1) onde n !! = k > 0 ( n 2k ). Assim, lim(2k + 1) I 2 k +1 I 2 k + 2 = lim = 2 , e k k 2k + 2 2 (2k + 2) = 2 , ou seja, lim k nI n I n +1 = 2 , donde lim(2k + 2) I 2 k + 2 I 2 k + 3 = lim k k 2k + 3 PP nI I 2 lim n n +1 = lim n 2n +1 = . k k n I 2 k +1 =
Obs.: Alternativamente, pela aproximao de Stirling, (2k 1)!! (2k )! = = (2k )!! (2k )!!2 (2k )! = ~ 2 k k 2 ! 2 k 2 k (2k ) e 4 k (1 + O(k 1 )) ~ k 2 k 2 k ~ 4 k e 2 k (1 + O( k 1 ))

~
e portanto I n ~

3 + Ok 2 k
1 2n

3 2 + O n 2 . Mas isso implica ~ n

+ O(1) e portanto

lim n

Pn Pn +1 2 = . n

EUREKA! N30, 2009

56

Sociedade Brasileira de Matemtica

XXX OLIMPADA BRASILEIRA DE MATEMTICA


Problemas e Solues da Segunda Fase Nvel Universitrio
PRIMEIRO DIA PROBLEMA 1

Seja f n :

x 2008 + x 2 nx, para cada n , e seja mn o 2008 m valor mnimo assumido por f n . Determine tal que o limite lim n existe e n n no-nulo, e calcule esse limite (para esse valor de ).

dada por f ( x) =

PROBLEMA 2

No
2

,
2

considere
2 2

elipse

definida

pelas

equaes

x=0

41 y + 41z 80 yz + 36 y + 36 z 81 = 0, e a elipse superfcie cnica de revoluo no


3

definida pelas equaes

y = 0 e 71x + 41z 40 xz + 18 x + 36 z 81 = 0. Prove que existe uma nica y = 0 em 2, e determine a interseo dessa superfcie com o plano z = 0.
PROBLEMA 3

que intersecta o plano x = 0 em 1 e o plano

Mostre que existem a1 , a2 ,...,

tais que a srie

a x
n =1 n

converge para todo

e, definindo f ( x) = an x n , temos:
n =1

em que satisfaz f ( x) > 0, x . i) f uma bijeo de ii) f ( ) = A , onde A = { | p( x) polinmio com coeficientes inteiros tal que p( ) = 0} o conjunto dos algbricos reais.
SEGUNDO DIA PROBLEMA 4

Seja Q = [0,1] [0,1] 2 um quadrado de lado 1 e f : Q uma funo contnua e positiva. Prove que possvel dividir Q em duas regies R1 e R2 de
EUREKA! N30, 2009

57

Sociedade Brasileira de Matemtica

mesma rea, separadas por R1 f ( x, y)dxdy = R2 f ( x, y)dxdy.


PROBLEMA 5

um

segmento

de

reta,

tais

que

Prove que no existe uma matriz 7 7, A = (aij )1i , j 7 , com aij 0,1 i, j 7 cujos autovalores (contados com multiplicidade) so: 6, 5, 5, 1, 1, 1, 1.

PROBLEMA 6

Prove que

( + n
n =1

2 2

<

1 1 , 0. 2 n =1 + n 2

SOLUES SEGUNDA FASE NVEL UNIVERSITRIO


PRIMEIRO DIA PROBLEMA 1 SOLUO DE EDUARDO POO (SO PAULO SP)

Seja xn = 2007 n. Como mn o mnimo de f n , ento:

1 mn f ( xn ) = 1 n 2007 n + 2007 n 2 , n . 2008 2008 x Seja agora g n ( x) = nx. 2008 Temos f n ( x ) g n ( x ) ,x , e sendo kn o valor mnimo de g n :
f n ( x ) g n ( x ) kn ,x mn kn ,n Calculando kn : g n ( x ) mnimo gn ( x ) = 0

1 x 2007 n = 0 x = 2007 n kn = 1 n 2007 n 2008 Assim:


1 1 2 1 1+ 2007 1 1+ 2007 2007 n m n n + 1 1 n 2008 2008 2 1 1 2008 Como <1+ , devemos ter = 1 + = , e o limite 2007 2007 2007 2007

EUREKA! N30, 2009

58

Sociedade Brasileira de Matemtica

1 2007 1 = . 2008 2008


PROBLEMA 2 SOLUO OFICIAL DA BANCA

Sejam ( x0 , y0 ,z0 ) o vrtice do cone e ( a,b,c ) um vetor no nulo na direo do eixo do cone. Se ( x, y,z ) um ponto do cone, existe um ngulo tal que o ngulo entre os vetores particular
2 2 2

( x x0 , y y0 ,z z0 )
de
2

( a,b,c )

sempre ou , e em igual a

mdulo
2

seu

cosseno
2

cos .

Assim,

a( x x0 ) + b( y y0 ) + c( z z0 ) a +b +c

( x x0 ) + ( y y0 ) + ( z z0 )
escrever
2 0

( a,b,c ) ( x x0 , y y0 ,z z0 ) ( a,b,c ) ( x x0 , y y0 ,z z0 )
equao
0

= cos ,
como onde

( a ( x x ) + b( y y ) + c ( z z ) )
0 0

podemos

(( x x ) + ( y y ) + ( z z ) ) = 0,
2 2 2 0 0

do

cone

d = cos . a2 + b2 + c2 ,
2 2 2

qual

pode

ser

escrita

na

forma

Ax + By + Cz + Dxy + Exz + Fyz + Gx + Hy + Iz + J = 0. Como, fazendo x = 0, obtemos

J = 81, e como, fazendo y = 0, obtemos uma equao da elipse 2, teremos A = 71, E = 40 e G = 18 (note que os coeficientes de z2, z e o coeficiente constante so

uma equao da elipse 1, podemos supor (ajustando o mdulo de (a, b, c), e trocando os sinais, se necessrio) que B = 41, C = 41, F = 80, H = 36, I = 36 e

os mesmos nas equaes dadas de 1 e 2). Assim, basta determinar D. Comparando as equaes, devemos ter a2 d 2 = 71, b2 d 2 = c2 d 2 = 41,2ac = 40 e 2bc = 80 ou a2 d 2 = 71, b2 d 2 = c2 d 2 = 41,2ac = 40 e 2bc = 80. Nos dois casos, b = 2a, donde b2 d 2 a2 d 2 , o que no acontece no primeiro caso. Assim, ocorre o segundo caso, e portanto D = 2ab = b2 . Como b2 d 2 = c2 d 2 , b = c , e portanto
80 = 40. Em particular, a interseo do cone com o 2 2 plano z = 0 dada pela equao 71x2 + 41y2 40xy +18x + 36y 81 = 0, e logo uma elipse. Obs.: Fazendo ( x0 ,y0 ,z0 ) = (1,2,2) ,( a,b,c) = 10, 2 10, 2 10 e d = 9 obtemos a D = b2 = b = bc =
2

2bc

equao desejada (aps trocar os sinais).


EUREKA! N30, 2009

59

Sociedade Brasileira de Matemtica

PROBLEMA 3 SOLUO OFICIAL DA BANCA

Construiremos uma funo que satisfaz as condies do enunciado da forma

f (x) = x + cn g n ( x ) , onde as funes g n tm derivada limitada na reta real e as


n =1

constantes

convergem a 0 muito rpido, de tal forma que 1 1 3 gn sup gn ( x ) ,x < n+1 ,n 1, o que garantir que < f ( x ) < ,x , 2 2 2

cn

e portanto f ser uma bijeo crescente de Os conjuntos enumer-los como um conjunto Bn


bBn

em

. . Podemos

e A so subconjuntos enumerveis densos de = { xn ,n

} e A = { yn ,n } . Para cada

n 1 construiremos . Tomamos

g n ( x ) = sen ( x b ) , que limitada e tem derivada limitada em

com n elementos, com Bn Bn +1 ,n 1 e tomaremos

B1 = {0} e c1 = 0.

1 1 1 1 Como sen u cos v = sen( u + v) + sen( u v) , sen u sen v = cos ( u v) cos ( u + v) , 2 2 2 2 sen ( x + a ) = cos a sen x + sen a cos x, cos ( x + a ) = cos a cos x sen a sen x,
sen ( u ) = sen u
n n n k k

cos ( u ) = cos u,

podemos escrever

gn ( x )
k

como
2 k +1

( r ( ) sen ( kx ) + s( ) cos ( kx ) ) .
k =0
k

Como, para todo x , sen x =

( 1) x k = 0 ( 2k + 1)!

( 1) x2k cos x = , temos que ( 2k ) ! k =0


uma constante kn > 0 tal que a Como,
(n)

( n) m ( n) g n ( x ) = am x , onde os am so tais que existe


m=0

( n) m

kn nm m!,m 0.
2m 3

para

todo

m m 1, m ! k > m 3 k>
3

temos
m

que

am m
m

m 2

kn n m
m

m 2

m! < k n n m
m

m 2

m 3

2m 3

n 32 3 = kn 1 6 , m

donde

( n) ( n) lim am mm 2 = 0. Em particular, existe jn 1 tal que am < 1 mm 2 ,m jn .

EUREKA! N30, 2009

60

Sociedade Brasileira de Matemtica

1 1 ( n) , n . Definimos tn = max am + 1 e wn = min n +1 jn 2 0 m< jn 2 t j 2 sup gn ( x ) ,x n n 1 ( n) < n m 2 ,m 1,n 0. Escolheremos as constantes cn Temos ento wn am 2 m satisfazendo sempre cn < wn (sempre escolhemos cn depois de j ter escolhido o

conjunto Bn , e logo j tendo determinado a funo gn ).

Teremos ento satisfaz

f ( x ) = x + cn gn ( x ) = am x m ,
n =1

onde

a1 < 1 + wn a1( n ) < 1 +


n =1

1 =2 e n n =1 2

m =1

a1 = 1 + cn a1( n)
n =1

( n) ( n) am cn am < wn am <
n=1 n =1

1 1 = m 2 ,m 2. Em particular m2 m n=1 2 m
n

a
m =1

xm

Para n par, tomamos o menor k tal que xk Bn1 , e definimos Bn = Bn1 { xk } . Como irracional, g n1 ( xk ) 0. Assim, como A denso em escolher cn 1 ( wn 1 ,wn 1 ) tal que xk + que, se y = a + 2k , com a , k \ {0} , ento y +
1 m< n 1

convergir para todo x . Escolheremos agora os conjuntos Bn e as constantes cn 1 ,n 2, recursivamente.

, podemos

cm g m ( xk ) + cn 1 g n1 ( xk ) A, e tal
1 m n 1

cm g m ( y ) A (de fato

o conjunto dos cn 1 tais que a ltima condio falha enumervel usamos aqui o fato de ser transcendente). Seja agora

n3

mpar. Seja

f n2 ( x ) = x + cm g m ( x ) .
m =1

n2

Temos que
em .

1 2 < fn1 ( x ) < 3 2 ,x , e logo f n 2 uma bijeo crescente de


Considere agora o menor r construo, que f
n 1

tal que yr { f n 2 ( b ) ,b Bn 1} . Temos, por

1 gn 1 ( f n 2 ( yr ) ) 0, e portanto podemos escolher cn 1 ( wn 1 ,wn 1 ) tal que, se 1 f n1 ( x ) = x + cm gm ( x ) , f n , 1 ( yr )

1 n2

( yr )

no da forma a + 2k , com a Bn 1 ,k . Assim,

tal

que,

se

y = a + 2k ,
Tomamos

com ento

a ,k \ {0} ,
EUREKA! N30, 2009

m =1

ento

f n1 ( y ) = y +

1 m n 1

cm gm ( y ) A.

61

Sociedade Brasileira de Matemtica


1 Bn = Bn 1 { f n 1 ( yr )} . Ao final dessa construo claro que f ( x ) A para todo

x e f 1 ( y ) para todo y A. Obs.: Se no quisermos usar o fato de ser transcendente podemos trocar as funes sen ( x c ) por sen ( ( x c ) ) , com transcendente.
SEGUNDO DIA PROBLEMA 4 SOLUO DE RENATO REBOUAS DE MEDEIROS (S.J. DOS CAMPOS SP)

Lema: Se um segmento de reta divide o retngulo em duas regies R1 e R2 da mesma rea, ento o segmento passa pelo centro do retngulo, que (0,5, 0,5), no caso. Prova: De fato, se o segmento dividisse igualmente a rea do retngulo e no passasse pelo seu centro, aconteceria:
A A

M (0,5, 0,5)

Segmento que passa pelo centro ( AB )

B Segmento que no passa pelo centro ( AB )

Nessa situao cada metade limitada por AB teria rea

1 , enquanto cada metade 2 limitada por A B tambm teria essa mesma rea (por semelhana de tringulos, A B divide igualmente) e o tringulo AAB teria rea nula, absurdo. Assim, tais segmentos passam pelo centro do retngulo.
y 1 M

EUREKA! N30, 2009

62

Sociedade Brasileira de Matemtica

direita da seta ( R1 ) e por A2 a integral sobre a regio esquerda da seta, que ser

Girando um segmento (de comprimento varivel) em torno do centro do retngulo (M), graas ao lema, obtm-se todas as possibilidades de dividir a rea em duas regies de mesma rea. Orientando esse segmento, possvel denominar por A1 a integral sobre a regio

R2 . Alm disso, A1 = A1 ( ) e A2 = A2 ( ) , sendo o ngulo no sentido antihorrio entre o segmento e um eixo horizontal por M. Resumindo, A1 ( ) = f ( x, y ) dxdy e A1 ( ) = f ( x, y ) dxdy.
Outra constatao que A1 ( ) = A2 ( 0 ) e A2 ( ) = A1 ( 0 ) , pois as regies R1 e
R1 R2

R2 de

=0
Q

so

trocadas

para

=.

Ainda,

tem-se

que

A1 ( ) + A2 ( ) = f ( x, y ) dxdy.
Como f :Q

A2 ( ) so funes contnuas em em todo o intervalo [ 0 , ] em que as funes A1 e A2 merecem anlise. Provaremos este fato no final da soluo.

* +

contnua, h indcios de que A1 ( ) e, conseqentemente,

Como as funes A1 ( ) e A2 ( ) tm soma constante e trocam de valor entre = 0 e = , alm de serem contnuas, ento as duas assumem o mesmo valor para algum , com 0 < . De fato, basta ver que, se A1 ( 0 ) A2 ( 0 ) , ento a funo G ( ) = A1 ( ) A2 ( ) no mesmo domnio de A1 e A2 tem um sinal em = 0 e o sinal contrrio em = , pois G ( 0 ) = A1 ( 0 ) A2 ( 0 ) e G ( ) = A1 ( ) A2 ( ) = A1 ( 0 ) A2 ( 0 ) = G ( 0 ) . Ento G tem uma raiz entre 0 e , por ser contnua e pelo Teorema do Valor Intermedirio. Nessa raiz, digamos , tm-se as condies desejadas. Se A1 ( 0 ) = A2 ( 0 ) , as condies j so satisfeitas para o segmento inicial. Resta provar que A1 ( ) contnuo em .

Para isso, basta ver que, por f ser contnua em [ 0,1] [ 0 ,1] , e ainda positiva, assume um valor mximo M e um valor mnimo m. Sendo assim,

2 A1 ( + ) A1 ( ) ( M m ) 2 (pois

na figura so

2 ). 2

EUREKA! N30, 2009

63

Sociedade Brasileira de Matemtica

2 > 0. Temos 0 < A1 ( ) A1 ( 0 ) < M m +1 e assim A1 contnua, permitindo aplicar o Teorema do Valor Intermedirio para G como feito acima. Assim, > 0 , para =
PROBLEMA 5 SOLUO DE FABIO DIAS MOREIRA (RIO DE JANEIRO RJ)

Se os autovalores de A so (6, 5, 5, 1, 1, 1, 1), os autovalores de B = A3 so (216, 125, 125, 1, 1, 1, 1). Por outro lado, se X = ( xij ) e Y = ( yij ) so matrizes com xij 0, yij 0, ento a mesma propriedade vale para Z = XY : de fato,

zij = xij ykj 0. Logo bij 0 1 i, j 7 e da Tr B = b11 + ... + b77 0.


k =1

Mas Tr B = 216 125 125 + 1 + 1 + 1 + 1 = 30, contradio. Portanto no existe nenhuma matriz A com a propriedade pedida.
PROBLEMA 6 SOLUO OFICIAL DA BANCA

Comeamos com a expresso de 2 x senx = x 1 2 2 . n n =1 Sabendo que essa igualdade vale para todo x funes analticas em

senx como

produto

infinito:

todo x . Fazendo x = iy obtemos

, como os dois lados definem eix e ix (lembramos que senx = ), a igualdade vale para 2i

EUREKA! N30, 2009

64

Sociedade Brasileira de Matemtica


e y e y x2 y = senx = x 1 2 2 = iy 1 + donde i n n 2i n =1 n =1 y2 senh ( y ) = y 1 + 2 , para todo y , e, em particular, para todo y > 0. n n =1

senh ( y )

y2 Aplicando logaritmos, obtemos log senh ( y ) = log + log y + log 1 + 2 , y > 0. n=1 n Derivando, temos: cos h ( y ) 1 2y , y > 0. = + 2 (*) senh ( y ) y n =1 y + n 2

Derivando novamente, obtemos, para todo y > 0,

( senh ( y ) )

2 ( ( senh( y) ) ) ( cos h( y) )
2

( senh ( y ) )

) = 1 +
y2

2 4 y2 2 + n 2 ( y 2 + n2 ) 2 n =1

1 y2 2 = 1 Portanto, 2 2 2 , y > 0. 2 2 2 n =1 y + n 2 y 2 ( senh ( y ) )2 n =1 ( y + n ) O lado direito dessa igualdade positivo para todo y > 0, de fato, isso equivale a senh ( y ) > y , y > 0; os dois lados dessa ltima desigualdade coincidem para
e y + e y y = 0, e a derivada do lado esquerdo, que cosh ( y ) = maior 2 que , que a derivada do lado direito, para todo y > 0. 1 y2 Portanto temos, para todo y > 0, 2 2 . > 2 2 2 2 n =1 y + n n =1 ( y + n ) Essa desigualdade obviamente tambm vale para y = 0. Finalmente, tomando y = , conclumos que a desigualdade do enunciado vale para todo 0.

EUREKA! N30, 2009

65

Sociedade Brasileira de Matemtica

XXX OLIMPADA BRASILEIRA DE MATEMTICA


Resultado Nvel 1 (6o. e 7o. Anos)
CIDADE ESTADO So Paulo SP Fortaleza CE Fortaleza CE So Paulo SP Belo Horizonte MG Bragana Paulista SP Salvador BA So Paulo SP Belo Horizonte MG Caucaia CE Petrpolis RJ So Paulo SP Braslia DF Belo Horizonte MG Cascavel PR Rio de Janeiro RJ Fortaleza CE Jundia SP Fortaleza CE Florianpolis SC Recife PE So Paulo SP So Paulo SP So Carlos SP Araatuba SP Rio de Janeiro RJ Mogi das Cruzes SP Porto Alegre RS Braslia DF Salvador BA Vassouras RJ Santos Dumont MG Recife PE So Paulo SP Santo Andr SP Teresina PI Anpolis GO Rio de Janeiro RJ Salvador BA Campo Grande MS Fortaleza CE Jaboato dos Guararapes PE Fortaleza CE So Paulo SP Salvador BA So Paulo SP Curitiba PR Rio de Janeiro RJ Rio de Janeiro RJ Jacare SP Rio de Janeiro RJ Cachoeira Paulista SP So Paulo SP Amparo SP Serra Negra SP So Paulo SP Joo Pessoa PB Rio de Janeiro RJ So Paulo SP Rio de Janeiro RJ Pinhais PR Salvador BA NOME Guilherme Renato Martins Unzer Francisco Markan Nobre de Souza Filho Rafael Rodrigues Rocha de Melo Liara Guinsberg Pedro Augusto de Paula Barbosa Matheus Silva Lima Vincius Canto Costa Michel Rozenberg Zelazny Vinicius Luiz Ferreira Lucas Cawai Julio Pereira Mrio de Mello Figueiredo Neto Henrique Vieira G. Vaz Elias Brito Oliveira Igor Albuquerque Araujo Luis Fernando Veronese Trivelatto Daniel Lima Santanelli Glauber Lima da Cunha Jnior Tiago Sueda Limone Leyberson Pereira Assuno Joo Marcos Carnieleto Nicolodi Fellipe Sebastiam da Silva P. Pereira Lucas Cardoso Zuccolo Maria Clara Cardoso Daniel Vincent Cacsire Garibay Rafael Tedeschi Eugnio Pontes Barone Gabriel Nogueira Coelho de Togni de Souza Thomas Akio Ikeda Valvassori William Cechin Guarienti Gabriel Yoshimi Barrn Toyama Daniel Behrens Cardoso Ricardo Vidal Mota Peixoto Jardel da Silva Pires Felipe Mendes de Holanda Lins Daniel Shinji Hoshi Jlio Csar de Barros Gabriel Queiroz Moura Lucas Carvalho Daher Daiana Luna Filipe Santana do Vale Rosane Thiemi Toma Gundim Guilherme de Oliveira Rodrigues Breno Soares da Costa Vieira Lara Viana de Paula Cabral Nicolas Chiu Ogassavara Juliana Amoedo Amoedo Plcido Bruno Cordeiro de Macedo Arthur Schott Lopes Bruno Costa Silva Alessandro Augusto Pinto de Oliveira Pacanowski Vitria Carolina Rondon Pereira Jorge Luiz Soares Pereira Pedro Carvalho da Fonseca Guimares Guilherme Ryu Odaguiri Kobori Lorena Marroni Carvalho Luiz Akyhito Miyazaki Marcos Felipe Nunes Lino Ribeiro Gustavo Souto Henriques Campelo Joo Vitor Fernandes Paiva Fbio Kenji Arai Roberto Tadeu Abrantes de Arajo Lucas Butschkau Vida Joo Ribeiro Pacheco PRMIO Ouro Ouro Ouro Ouro Ouro Ouro Prata Prata Prata Prata Prata Prata Prata Prata Prata Prata Bronze Bronze Bronze Bronze Bronze Bronze Bronze Bronze Bronze Bronze Bronze Bronze Bronze Bronze Meno Honrosa Meno Honrosa Meno Honrosa Meno Honrosa Meno Honrosa Meno Honrosa Meno Honrosa Meno Honrosa Meno Honrosa Meno Honrosa Meno Honrosa Meno Honrosa Meno Honrosa Meno Honrosa Meno Honrosa Meno Honrosa Meno Honrosa Meno Honrosa Meno Honrosa Meno Honrosa Meno Honrosa Meno Honrosa Meno Honrosa Meno Honrosa Meno Honrosa Meno Honrosa Meno Honrosa Meno Honrosa Meno Honrosa Meno Honrosa Meno Honrosa Meno Honrosa

EUREKA! N30, 2009

66

Sociedade Brasileira de Matemtica

Nvel 2 (8o. e 9o. Anos)


Nome Joo Lucas Camelo S Gustavo Haddad F. e Sampaio Braga Gabriel Milito Vinhas Lopes Rubens Cainan Sabia Monteiro Maria Clara Mendes Silva Marla Rochana Braga Monteiro Otvio Arajo de Aguiar Caque Porto Lira Tuane Viana Pinheiro Carlos Henrique de Andrade Silva Otvio Augusto de Oliveira Mendes Rafael Kazuhiro Miyazaki Vinicius Cipriano Klein Daniel dos Santos Bossle Mateus Braga de Carvalho Luiz Henrique Vieira Leo Kayo de Frana Gurgel Davi Coelho Amorim Rodolfo Rodrigues da Costa Rafael Henrique dos Santos Ivan Tadeu Ferreira Antunes Filho Lucas Cordeiro Herculano Breno Lev Corra Tbata Cludia Amaral de Pontes Marina Pessoa Mota Marcos Massayuki Kawakami Bruno Ferri de Moraes Tiago Leandro Estevam Dias Gabriel Pacianotto Gouveia Filipe Jos Oliveira Sabia Daniel Prince Carneiro Bruno Moraes Moreno Renan Fernandes Moreira Murilo Dria Guimares Lucas Nishida Leonardo Ferreira Patrcio Victor Kioshi Higa Marina de Moura Faleo Lucas Almeida Pereira de Lima Arthur Ribeiro Notaro Pedro Mendona de Lima Rafael Ferreira Antonioli Lucas Okumura Ono Vinicius Affonso de Carvalho Julio Barros de Paula Joo Francisco Goes Braga Takayanagi Dbora Barreto Ornellas Luis Henrique Kobayashi Higa Vtor Gabriel Barra Souza Pedro Ivo Colho de Arajo Guilherme Cherman Perdigo de Oliveira Cesar Nobuo Moniwa Ishiuchi Letcia Dias Mattos Raul Arago Rocha Tiago de vila Palhares Gabriel Leal Teixeira de Souza Cidade - Estado Fortaleza CE S. J. dos Campos SP Fortaleza CE Fortaleza CE Pirajuba MG Fortaleza CE Fortaleza CE Fortaleza CE Rio de Janeiro RJ Fortaleza CE Pilar do Sul SP So Paulo SP Venda do Imigrante ES Porto Alegre RS Teresina PI Rio de Janeiro RJ Fortaleza CE Fortaleza CE Fortaleza CE Santa Cruz do Rio Pardo SP Lins SP Fortaleza CE Campo Belo MG So Paulo SP Fortaleza CE So Paulo SP So Paulo SP Rio de Janeiro RJ So Paulo SP Fortaleza CE So Loureno MG Porto Nacional TO Taubat SP So Paulo SP Pedreira SP Rio de Janeiro RJ So Paulo SP Recife PE Recife PE Recife PE Goinia GO S.B. do Campo SP So Paulo SP So Paulo SP Taubat SP So Paulo SP Salvador BA Campo Grande MS Juiz de Fora MG Caucaia CE Rio de Janeiro RJ Campinas SP Contagem MG Recife PE Braslia DF Rio de Janeiro RJ Prmio Ouro Ouro Ouro Ouro Ouro Ouro Prata Prata Prata Prata Prata Prata Prata Prata Prata Prata Prata Bronze Bronze Bronze Bronze Bronze Bronze Bronze Bronze Bronze Bronze Bronze Bronze Bronze Bronze Bronze Meno Honrosa Meno Honrosa Meno Honrosa Meno Honrosa Meno Honrosa Meno Honrosa Meno Honrosa Meno Honrosa Meno Honrosa Meno Honrosa Meno Honrosa Meno Honrosa Meno Honrosa Meno Honrosa Meno Honrosa Meno Honrosa Meno Honrosa Meno Honrosa Meno Honrosa Meno Honrosa Meno Honrosa Meno Honrosa Meno Honrosa Meno Honrosa

EUREKA! N30, 2009

67

Sociedade Brasileira de Matemtica

Nvel 3 (Ensino Mdio)


Nome Henrique Pond de Oliveira Pinto Rgis Prado Barbosa Marcelo Tadeu de S Oliveira Sales Guilherme Philippe Figueiredo Marcelo Matheus Gauy Gabriel Lus Mello Dalalio Renan Henrique Finder Alfredo Roque de Oliveira Freire Filho Rafael Tupynamb Dutra Marco Antonio Lopes Pedroso Thiago da Silva Pinheiro Eduardo Queiroz Peres Davi Lopes Alves de Medeiros Ricardo Turolla Bortolotti Alex Atsushi Takeda Deborah Barbosa Alves Marlen Lincoln da Silva Hugo Fonseca Arajo Rafael Parpinel Cavina Thiago Ribeiro Ramos Rafael Horimoto de Freitas Rafael Alves da Ponte Robrio Soares Nunes Gustavo Lisba Empinotti Henrique Hiroshi Motoyama Watanabe Ivan Guilwon Mitoso Rocha Jos Airton Colho Lima Filho Hudson do Nascimento Lima Hanon Guy Lima Rossi Ricardo Bioni Liberalquino Luiz Filipe Martins Ramos Illan Feiman Halpern Matheus Secco Torres da Silva Matheus Arajo Marins Paulo Cesar Neves da Costa Leonardo Pereira Stedile Jonas Rocha Lima Amaro Matheus Barros de Paula Rafael Sussumu Yamaguti Miada Rafael Alves da Silva James Jun Hong Joas Elias dos Santos Rocha Gelly Whesley Silva Neves Joo Mendes Vasconcelos Marilia Valeska Costa Medeiros Jlio Czar Batista de Souza Jos Cabadas Duran Neto Thiago Saksanian Hallak Luiz Eduardo Schiller Esdras Muniz Mota Fbio Lus de Mello Ana Beatriz Prudncio de Almeida Rebouas Victorio Takahashi Chu Isabella Amorim Gonalez Grazielly Muniz da Cunha Jos Leandro Pinheiro Antnio Deromir Neves Silva Jnior Cidade - Estado So Paulo SP Fortaleza CE Salvador BA So Paulo SP S.J. do Rio Preto SP S.J. dos Campos SP So Paulo SP Salvador BA Belo Horizonte MG Santa Isabel SP So Paulo SP Jundia SP Fortaleza CE Rio de Janeiro RJ Londrina PR So Paulo SP Fortaleza CE Rio de Janeiro RJ So Paulo SP Varginha MG So Paulo SP Fortaleza CE Ribeiro Preto SP Florianpolis SC So Paulo SP Fortaleza CE Fortaleza CE Fortaleza CE So Paulo SP Macei AL Rio de Janeiro RJ So Paulo SP Rio de Janeiro RJ Rio de Janeiro RJ Braslia DF So Paulo SP Fortaleza CE Taubat SP Valinhos SP Teresina PI So Paulo SP Muribeca SE Fortaleza CE Fortaleza CE Fortaleza CE Salvador BA Salvador BA So Paulo SP Rio de Janeiro RJ Fortaleza CE So Paulo SP Fortaleza CE So Paulo SP Macei AL Fortaleza CE Deputado Irapuan Pinheiro CE Fortaleza CE Prmio Ouro Ouro Ouro Ouro Ouro Ouro Prata Prata Prata Prata Prata Prata Prata Prata Prata Prata Bronze Bronze Bronze Bronze Bronze Bronze Bronze Bronze Bronze Bronze Bronze Bronze Bronze Bronze Bronze Bronze Bronze Meno Honrosa Meno Honrosa Meno Honrosa Meno Honrosa Meno Honrosa Meno Honrosa Meno Honrosa Meno Honrosa Meno Honrosa Meno Honrosa Meno Honrosa Meno Honrosa Meno Honrosa Meno Honrosa Meno Honrosa Meno Honrosa Meno Honrosa Meno Honrosa Meno Honrosa Meno Honrosa Meno Honrosa Meno Honrosa Meno Honrosa Meno Honrosa

EUREKA! N30, 2009

68

Sociedade Brasileira de Matemtica

Nvel Universitrio
Nome Fbio Dias Moreira Rafael Daigo Hirama Guilherme Rodrigues Nogueira de Souza Eduardo Poo Ramn Moreira Nunes Renato Rebouas de Medeiros Thiago Costa Leite Santos Raphael Constant da Costa Andr Linhares Rodrigues Edson Augusto Bezerra Lopes Marcelo de Arajo Barbosa Levi Mximo Viana Thoms Yoiti Sasaki Hoshina Felipe Gonalves Assis Fernando Nascimetno Coelho Marcos Victor Pereira Vieira Reinan Ribeiro Souza Santos Mateus Oliveira de Figueiredo Caio Ishizaka Costa Paulo Srgio de Castro Moreira Pedro henrique Milet Pinheiro Pereira Willy George do Amaral Petrenko Jorge Henrique Craveiro de Andrade Leandro Farias Maia Alysson Espndola de S Silveira Leonardo Ribeiro de Castro Carvalho Rafael Montezuma Pinheiro Cabral Rafael Sampaio de Rezende Luty Rodrigues Ribeiro Leandro Augusto Lichtenfelz Sidney Cerqueira Bispo dos Santos Filho Andr Jorge Carvalho Jordan Freitas Piva Juan Raphael Diaz Simes Eduardo Fischer Adenilson Arcanjo de Moura Jnior Kellem Correa Santos Roberto Akiba de Oliveira Jos Armando Barbosa Filho Daniel Lopes Alves de Medeiros Vitor Humia Fontoura Bruno da Silva Santos Luca Mattos Mller Marcelo Salhab Brogliato Jos Marcos Andrade Ferraro Evandro Makiyama Antonia Taline de Souza Mendona Diego Andrs de Barros Lima Barbosa Gabriel Ponce Luiz Paulo Freire Moreira Felipe Rodrigues Nogueira de Souza Igor Magalhes Oliveira Rafael Ghussn Cano Cidade - Estado Rio de Janeiro RJ S.J. dos Campos SP So Paulo SP So Paulo SP Fortaleza CE S.J. dos Campos SP So Paulo SP Rio de Janeiro RJ Campinas SP Fortaleza CE S.J. dos Campos SP Rio de Janeiro RJ Rio de Janeiro RJ Campina Grande PB S.J. dos Campos SP S.J. dos Campos SP Aracaj SE S.J. dos Campos SP S.J. dos Campos SP S.J. dos Campos SP Rio de Janeiro RJ Rio de Janeiro RJ Rio de Janeiro RJ Fortaleza CE S.J. dos Campos SP So Paulo SP Fortaleza CE Fortaleza CE S.J. dos Campos SP Florianpolis SC S.J. dos Campos SP So Paulo SP Rio de Janeiro RJ So Paulo SP Encantado RS Fortaleza CE Rio de Janeiro RJ So Paulo SP S.J. dos Campos SP S.J. dos Campos SP Rio de Janeiro RJ Belford Roxo RJ S.J. dos Campos SP Rio de Janeiro RJ So Paulo SP So Paulo SP Rio de Janeiro RJ Rio de Janeiro RJ So Carlos SP Fortaleza CE So Paulo SP Macei AL Campinas SP Prmio Ouro Ouro Ouro Ouro Ouro Prata Prata Prata Prata Prata Prata Prata Prata Prata Prata Prata Bronze Bronze Bronze Bronze Bronze Bronze Bronze Bronze Bronze Bronze Bronze Bronze Bronze Bronze Bronze Meno Honrosa Meno Honrosa Meno Honrosa Meno Honrosa Meno Honrosa Meno Honrosa Meno Honrosa Meno Honrosa Meno Honrosa Meno Honrosa Meno Honrosa Meno Honrosa Meno Honrosa Meno Honrosa Meno Honrosa Meno Honrosa Meno Honrosa Meno Honrosa Meno Honrosa Meno Honrosa Meno Honrosa Meno Honrosa

EUREKA! N30, 2009

69

Sociedade Brasileira de Matemtica

AGENDA OLMPICA
XXXI OLIMPADA BRASILEIRA DE MATEMTICA NVEIS 1, 2 e 3 Primeira Fase Sbado, 06 de junho de 2009 Segunda Fase Sbado, 12 de setembro de 2009 Terceira Fase Sbado, 17 de outubro de 2009 (nveis 1, 2 e 3) Domingo, 18 de outubro de 2009 (nveis 2 e 3 - segundo dia de prova). NVEL UNIVERSITRIO Primeira Fase Sbado, 12 de setembro de 2009 Segunda Fase Sbado, 17 e Domingo, 18 de outubro de 2008

XV OLIMPADA DE MAIO 09 de maio de 2009

XX OLIMPADA DE MATEMTICA DO CONE SUL 14 a 20 de abril de 2009 Mar del Plata Argentina

L OLIMPADA INTERNACIONAL DE MATEMTICA 10 a 22 de julho de 2009 Bremen Alemanha

XVI OLIMPADA INTERNACIONAL DE MATEMTICA UNIVERSITRIA 25 a 30 de julho de 2009 Budapeste, Hungria

XXIV OLIMPADA IBEROAMERICANA DE MATEMTICA 17 a 27 de setembro de 2009 Quertaro, Mxico

XII OLIMPADA IBEROAMERICANA DE MATEMTICA UNIVERSITRIA

EUREKA! N30, 2009

70

Sociedade Brasileira de Matemtica

COORDENADORES REGIONAIS
Alberto Hassen Raad Amrico Lpez Glvez Amarsio da Silva Arajo Andreia Goldani Antonio Carlos Nogueira Benedito Tadeu Vasconcelos Freire Carlos Alexandre Ribeiro Martins Carmen Vieira Mathias Claus Haetinger Cleonor Crescncio das Neves Cludio de Lima Vidal Denice Fontana Nisxota Menegais Edson Roberto Abe Eduardo Tengan lio Mega Eudes Antonio da Costa Fbio Brochero Martnez Florncio Ferreira Guimares Filho Francinildo Nobre Ferreira Genildo Alves Marinho Ivanilde Fernandes Saad Jacqueline Rojas Arancibia Janice T. Reichert Joo Bencio de Melo Neto Joo Francisco Melo Libonati Jose de Arimatia Fernandes Jos Luiz Rosas Pinho Jos Vieira Alves Jos William Costa Krerley Oliveira Licio Hernandes Bezerra Luciano G. Monteiro de Castro Luzinalva Miranda de Amorim Mrio Rocha Retamoso Marcelo Rufino de Oliveira Marcelo Mendes Newman Simes Nivaldo Costa Muniz Osnel Broche Cristo Osvaldo Germano do Rocio Raul Cintra de Negreiros Ribeiro Ronaldo Alves Garcia Rogrio da Silva Igncio Reginaldo de Lima Pereira Reinaldo Gen Ichiro Arakaki Ricardo Amorim Srgio Cludio Ramos Seme Gebara Neto Tadeu Ferreira Gomes Toms Menndez Rodrigues Valdenberg Arajo da Silva Vnia Cristina Silva Rodrigues Wagner Pereira Lopes
EUREKA! N30, 2009

(UFJF) (USP) (UFV) FACOS (UFU) (UFRN) (Univ. Tec. Fed. de Paran) (UNIFRA) (UNIVATES) (EDETEC) (UNESP) (UNIPAMPA) (Colgio Objetivo de Campinas) (USP) (Grupo Educacional Etapa) (Univ. Federal do Tocantins) (UFMG) (UFES) (UFSJ) (Centro Educacional Leonardo Da Vinci) (UC. Dom Bosco) (UFPB)) (UNOCHAPEC) (UFPI) (Grupo Educacional Ideal) (UFPB) (UFSC) (UFPB) (Instituto Pueri Domus) (UFAL) (UFSC) (Sistema Elite de Ensino) (UFBA) (UFRG) (Grupo Educacional Ideal) (Colgio Farias Brito, Pr-vestibular) (Cursinho CLQ Objetivo) (UFMA) (UFLA) (U. Estadual de Maring) (Colgio Anglo) (UFGO) (Col. Aplic. da UFPE) (Escola Tcnica Federal de Roraima) (UNIFESP) (Centro Educacional Logos) (IM-UFRGS) (UFMG) (UEBA) (U. Federal de Rondnia) (U. Federal de Sergipe) (U. Metodista de SP) (CEFET GO)

Juiz de Fora MG Ribeiro Preto SP Viosa MG Osrio RS Uberlndia MG Natal RN Pato Branco PR Santa Mara RS Lajeado RS Manaus AM S.J. do Rio Preto SP Bag RS Campinas SP So Carlos SP So Paulo SP Arraias TO Belo Horizonte MG Vitria ES So Joo del Rei MG Taguatingua DF Campo Grande MS Joo Pessoa PB Chapec SC Teresina PI Belm PA Campina Grande PB Florianpolis SC Campina Grande PB Santo Andr SP Macei AL Florianpolis SC Rio de Janeiro RJ Salvador BA Rio Grande RS Belm PA Fortaleza CE Piracicaba SP So Luis MA Lavras MG Maring PR Atibaia SP Goinia GO Recife PE Boa Vista RR SJ dos Campos SP Nova Iguau RJ Porto Alegre RS Belo Horizonte MG Juazeiro BA Porto Velho RO So Cristovo SE S.B. do Campo SP Jata GO

71

CONTEDO
AOS LEITORES XV OLIMPADA DE MATEMTICA DE MAIO Problemas e Resultado Brasileiro XX OLIMPADA DE MATEMTICA DO CONE SUL Problemas e Resultado Brasileiro L OLIMPADA INTERNACIONAL DE MATEMTICA (IMO) Problemas e Resultado Brasileiro XXIV OLIMPADA IBEROAMERICANA DE MATEMTICA Problemas e Resultado Brasileiro 2 3 7 9 11

ARTIGOS
PAR OU MPAR? EIS A QUESTO Samuel Barbosa Feitosa e Einstein do Nascimento Jnior GEOMETRIA DO TRINGULO: FATOS E PROBLEMAS Carlos Yuzo Shine SRIE HARMNICA DE NMEROS PRIMOS Lenimar Nunes de Andrade COMO QUE FAZ? SOLUES DE PROBLEMAS PROPOSTOS PROBLEMAS PROPOSTOS AGENDA OLMPICA COORDENADORES REGIONAIS 13 28 45 50 52 59 61 62

Sociedade Brasileira de Matemtica

AOS LEITORES
Por mais um ano consecutivo estamos iniciando a realizao da Olimpada Brasileira de Matemtica. A Olimpada Brasileira de Matemtica OBM tem crescido substancialmente nos ltimos anos, contando, em 2009, com a adeso ao evento de mais de 3.700 escolas, sendo 2.180 da rede pblica e 1.608 da rede privada de ensino, o que implicou em uma participao efetiva de cerca de 180.000 jovens estudantes e seus professores. Alm disso, a iniciativa contou com a colaborao de professores universitrios em 155 instituies de ensino superior: eles participaram de todas as atividades, inclusive aquelas referentes OBM Nvel Universitrio em atividades de coordenao, divulgao, treinamento de alunos, aperfeioamento de professores e aplicao das distintas fases da Olimpada Brasileira de Matemtica. Paralelamente, o projeto apiou a realizao de Olimpadas Regionais de Matemtica, contando com a participao de 165.148 estudantes das escolas pblicas e privadas em todo o Brasil nas competies estaduais. No que se refere participao em competies internacionais, os resultados foram excelentes: Em particular, Henrique Ponde conquistou a oitava medalha de Ouro do Brasil na IMO. Alm disso, nesta IMO, todos os alunos da equipe brasileria ganharam medalhas. Durante 2009 a CAPES e o CNPq lanaram o Programa de Iniciao Cientfica Mestrado (PICME) para medalhistas da OBMEP e OBM, beneficiando 19 estudantes premiados na Olimpada Brasileira de Matemtica OBM, com o objetivo de aumentar o nmero de matemticos no pas, e oferecer uma formao matemtica mais slida a jovens profissionais de outras reas cientficas e tecnolgicas. Todos estes resultados nacionais e internacionais demonstram que, alm de influenciar positivamente o ensino da Matemtica nas instituies de ensino fundamental, mdio e superior, conseguimos detectar jovens muito talentosos que so estimulados a seguir uma carreira cientfica, o que fundamental para o crescimento da Cincia e Tecnologia no pas. A Olimpada Brasileira de Matemtica um projeto conjunto da Sociedade Brasileira de Matemtica, do Instituto Nacional de Matemtica Pura e Aplicada (IMPA) e conta com o apoio do Conselho Nacional de Desenvolvimento Cientfico e Tecnolgico (CNPq) e do Instituto Nacional de Cincia e Tecnologia de Matemtica (INCTMat).

Os editores

EUREKA! N31, 2010

Sociedade Brasileira de Matemtica

XV OLIMPADA DE MAIO
PRIMEIRO NVEL
PROBLEMA 1

A cada nmero natural de dois algarismos associamos um dgito da seguinte forma: Multiplicam-se seus algarismos. Se o resultado um dgito, este o dgito associado. Se o resultado um nmero de dois dgitos, multiplicam-se estes dois algarismos, e se o resultado um dgito, este o dgito associado. Caso contrario, repetimos a operao. Por exemplo, o dgito associado a 32 o 6 pois 3 2 = 6; o dgito associado a 93 o 4 pois 9 3 = 27, 2 7 = 14, 1 4 = 4. Encontre todos os nmeros de dois algarismos aos que se associa o dgito 8.
PROBLEMA 2

Encontre nmeros primos p, q, r para os quais p + q 2 + r 3 = 200 . Diga todas as possibilidades. Obs: Lembre-se que o nmero 1 no primo.
PROBLEMA 3

Temos 26 cartes e cada um tem escrito um nmero. H dois com o nmero 1, dois com o nmero 2, dois com o 3, e assim por diante at dois com o 12 e dois com o 13. Deve-se distribuir os 26 cartes em pilhas de maneira que sejam cumpridas as duas condies a seguir: Se dois cartes tm o mesmo nmero esto na mesma pilha. Nenhuma pilha contm um carto cujo nmero igual soma dos nmeros de dois cartes dessa mesma pilha. Determine qual o nmero mnimo de pilhas que temos que formar. D um exemplo com a distribuio dos cartes para esse nmero de pilhas e justifique por qu impossvel ter menos pilhas.
PROBLEMA 4

Trs circunferncias so tangentes entre si, tal como mostramos na figura. A regio do crculo exterior que no est coberta pelos dois crculos interiores tem rea igual a 2. Determine o comprimento do segmento PQ.

EUREKA! N31, 2010

Sociedade Brasileira de Matemtica

PROBLEMA 5

Pelas linhas de um tabuleiro quadriculado formado por 55 linhas horizontais e 45 linhas verticais caminha uma formiga. Queremos pintar alguns trechos de linhas para que a formiga possa ir de qualquer cruzamento at outro cruzamento qualquer, caminhando exclusivamente pelos trechos pintados. Se a distncia entre linhas consecutivas de 10 cm, qual a menor quantidade possvel de centmetros que devero ser pintados? SEGUNDO NVEL
PROBLEMA 1

Inicialmente no quadro est escrito o nmero 1. Em cada passo, apaga-se o nmero do quadro e se escreve outro, que obtido aplicando alguma das seguintes operaes: Operao A: Multiplicar o nmero escrito no quadro por

1 . 2

Operao B: Trocar o nmero escrito no quadrado pela diferena entre 1 e ele.

Por exemplo, se no quadro est escrito o nmero

3 podemos substitu-lo por 8

1 3 3 3 5 ou por 1 = . = 2 8 16 8 8
Encontre uma sequncia de passos ao fim dos quais o nmero do quadro seja

2009 . 2 2009
PROBLEMA 2

Seja ABCD um quadriltero convexo tal que o tringulo ABD equiltero e o

= 90o . Se E o ponto mdio do lado AD, tringulo BCD issceles, com C . determine a medida do ngulo CED
PROBLEMA 3

Na seguinte soma: 1 + 2 + 3 + 4 + 5 + 6, se suprimirmos os dois primeiros sinais de + obtemos a nova soma 123 + 4 + 5 + 6 = 138. Suprimindo trs sinais de + podemos obter 1 + 23 + 456 = 480. Consideremos agora a soma 1 + 2 + 3 + 4 + 5 + 6 + 7 + 8 + 9 + 10 + 11 + 12 + 13, na qual sero suprimidos alguns sinais de +. Quais so os trs menores mltiplos de 100 que podemos obter desta forma?
EUREKA! N31, 2010

Sociedade Brasileira de Matemtica

PROBLEMA 4

Cada casa de um tabuleiro 5 5 pintada de vermelho ou de azul, de tal forma que seja cumprida a seguinte condio: Para quaisquer duas filas e duas colunas, das 4 casas que esto em suas intersees, h 4, 2 ou 0 pintadas de vermelho. De quantas formas podemos pintar o tabuleiro?
PROBLEMA 5

Um jogo de pacincia se inicia com 25 cartas em fila. Algumas esto viradas para cima, e outras viradas para baixo. Em cada movimento devemos escolher uma carta que esteja virada para cima, retir-la e virar as cartas vizinhas que foi retirada (se houver). Ganha-se o jogo de pacincia quando conseguimos, repetindo este movimento, retirar as 25 cartas da mesa. Se inicialmente h n cartas viradas para cima, encontre todos os valores de n para os quais se pode ganhar o jogo. Explique a estratgia vencedora, independentemente da localizao inicial das cartas viradas para cima, e justifique por qu impossvel ganhar para os outros valores de n. Duas cartas so vizinhas quando uma est imediatamente ao lado de outra, direita ou esquerda. Por exemplo: a carta marcada com A tem duas cartas vizinhas e a marcada com B apenas uma. Depois de retirar uma carta fica um espao, de C B A D modo que a marcada com C tem unicamente uma carta vizinha, e a marcada com D no tem nenhuma.

EUREKA! N31, 2010

Sociedade Brasileira de Matemtica

RESULTADO BRASILEIRO 2009: Nvel 1 (at 13 anos) Nome Luis Fernando Veronese Trivelatto Lucas Carvalho Daher Guilherme Renato Martins Unzer Elias Brito Oliveira Lucas Cardoso Zuccolo Gustavo Lima Lopes Rafael Rodrigues Rocha de Melo Igor Albuquerque Arajo Liara Guinsberg Cidade - Estado Cascavel - PR Anpolis - GO So Paulo - SP Braslia - DF So Paulo - SP Barra de So Fco. - ES Fortaleza - CE Rio de Janeiro - RJ So Paulo - SP Pontos Prmio 30 29 27 26 25 24 24 23 23 23 Medalha de Ouro Medalha de Prata Medalha de Prata Medalha de Bronze Medalha de Bronze Medalha de Bronze Medalha de Bronze Meno Honrosa Certificado Certificado

Fellipe Sebastiam da Silva Paranhos Pereira Rio de Janeiro - RJ

2009: Nvel 2 (at 15 anos) Nome Joo Lucas Camelo S Csar Ilharco Magalhes Bruno Silva Mucciaccia Daniel dos Santos Bossle Otvio Arajo de Aguiar Gabriel Milito Vinhas Lopes Lara Timb Arajo Artur A. Scussel Bruno Ferri de Moraes

Cidade - Estado Fortaleza - CE Barbacena - MG Vitria - ES Porto Alegre - RS Fortaleza - CE Fortaleza - CE Fortaleza - CE Fortaleza - CE So Paulo - SP

Pontos Prmio 50 42 40 40 38 38 35 33 32 31 Medalha de Ouro Medalha de Prata Medalha de Prata Medalha de Bronze Medalha de Bronze Medalha de Bronze Medalha de Bronze Meno Honrosa Meno Honrosa Meno Honrosa

Gustavo Haddad Francisco e Sampaio Braga S.J. dos Campos - SP

EUREKA! N31, 2010

Sociedade Brasileira de Matemtica

XX OLIMPADA DE MATEMTICA DO CONE SUL


Enunciados e resultado brasileiro
A XX Olimpada de Matemtica do Cone Sul foi realizada na cidade de Mar del Plata, Argentina entre os dias 16 e 17 de abril de 2009. A equipe foi liderada pelos professores Pablo Rodrigo Ganassim, de So Paulo SP e Alex Correa Abreu, de Niteri RJ. RESULTADOS DA EQUIPE BRASILEIRA BRA1 BRA2 BRA3 BRA4 Deborah Barbosa Alves Gabriel Milito Vinhas Lopes Matheus Barros de Paula Matheus Secco Torres da Silva Medalha de Prata Medalha de Bronze Medalha de Prata Medalha de Bronze

PROBLEMA 1

Os quatro crculos da figura determinam 10 regies limitadas. Nessas regies so escritos 10 nmeros inteiros positivos distintos cuja soma 100, um nmero em cada regio. A soma dos nmeros contidos em cada crculo igual a S (a mesma para os quatro crculos). Determine o maior e o menor valor possvel de S.

PROBLEMA 2

Um corchete composto por trs segmentos de comprimento 1, que formam dois ngulos retos como mostra a figura.

dado um quadrado de lado n dividido em n2 quadradinhos de lado 1 por meio de retas paralelas aos seus lados. Corchetes so colocados sobre esse quadrado de modo que cada segmento de um corchete cubra um lado de algum quadradinho. Dois segmentos de corchete no podem ficar sobrepostos. Determine todos os valores de n para os quais possvel cobrir os lados dos n2 quadradinhos.
EUREKA! N31, 2010

Sociedade Brasileira de Matemtica

PROBLEMA 3

Sejam A, B e C trs pontos tais que B ponto mdio do segmento AC e seja P um ponto tal que PBC = 60. So construdos o tringulo equiltero PCQ tal que B e Q esto em semiplanos diferentes em relao a PC, e o tringulo equiltero APR tal que B e R esto no mesmo semiplano em relao a AP. Seja X o ponto de interseo das retas BQ e PC; seja Y o ponto de interseo das retas BR e AP. Demonstre que XY e AC so paralelos.
PROBLEMA 4

Ana e Beto jogam em um tabuleiro de 11 linhas e 9 colunas. Primeiro Ana divide o tabuleiro em 33 zonas. Cada zona formada por 3 casas adjacentes alinhadas vertical ou horizontalmente, como mostra a figura.

Depois, Beto escreve em cada casa um dos nmeros 0, 1, 2, 3, 4, 5, de modo que a soma dos nmeros de cada zona seja igual a 5. Beto ganha se a soma dos nmeros escritos em cada uma das 9 colunas do tabuleiro um nmero primo; caso contrrio, Ana ganha. Demonstre que Beto tem uma estratgia vencedora.
PROBLEMA 5

Dada uma sequncia S de 1001 nmeros reais positivos no necessariamente distintos, e dado um conjunto A de nmeros inteiros positivos distintos, a operao permitida : eleger um k A (k = 1001), selecionar k nmeros de S, calcular a mdia dos k nmeros (mdia aritmtica) e substituir cada um dos k nmeros selecionados por essa mdia. Se A um conjunto tal que para cada S pode-se conseguir, mediante uma sucesso de operaes permitidas, que os nmeros sejam todos iguais, determine o menor valor possvel do maior elemento de A.
PROBLEMA 6

Pablo tem uma certa quantidade de retngulos cujas reas somam 3 e cujos lados so todos menores ou iguais a 1. Demonstre que com esses retngulos possvel cobrir um quadrado de lado 1 de modo que os lados dos retngulos sejam paralelos aos lados do quadrado. Nota: Os retngulos podem estar sobrepostos e podem sair parcialmente do quadrado.

EUREKA! N31, 2010

Sociedade Brasileira de Matemtica

L OLIMPADA INTERNACIONAL DE MATEMTICA (IMO)


Enunciados e resultado Brasileiro
A L Olimpada Internacional de Matemtica (IMO) foi realizada na cidade de Bremen, Alemanha entre os dias 14 e 21 de julho de 2009. A equipe foi liderada pelos professores Carlos Yuzo Shine, de So Paulo SP e Ralph Costa Teixeira, de Niteri RJ. RESULTADOS DA EQUIPE BRASILEIRA BRA1 BRA2 BRA3 BRA4 BRA5 BRA6 Henrique Ponde de Oliveira Pinto Renan Henrique Finder Marcelo Tadeu de S Oliveira Sales Matheus Secco Torres da Silva Marco Antonio Lopes Pedroso Davi Lopes Alves de Medeiros Medalha de Ouro Medalha de Prata Medalha de Prata Medalha de Prata Medalha de Bronze Medalha de Bronze

PRIMEIRO DIA
PROBLEMA 1

Seja n um inteiro positivo e sejam a1 ,...,ak ( k 2 ) inteiros distintos do conjunto divide ak ( a1 1) .


PROBLEMA 2

{1,...,n}

tais que n divide ai ( ai +1 1) , para i = 1,...,k 1. Demonstre que n no

Seja ABC um tringulo cujo circuncentro O. Sejam P e Q pontos interiores dos lados CA e AB, respectivamente. Sejam K, L, e M os pontos mdios dos segmentos BP, CQ e PQ, respectivamente, e a circunferncia que passa por K, L, e M. Suponha que a recta PQ tangente circunferncia . Demonstre que OP = OQ.
PROBLEMA 3

Seja s1 ,s2 ,s3 ,... uma sucesso estritamente crescente de inteiros positivos tal que as subsucesses ss1 ,ss2 ,ss3 ,... e ss1 +1 ,ss2 +1 ,ss3 +1 ,... so ambas progresses aritmticas. Demonstre que a sucesso s1 ,s2 ,s3 ,... tambm uma progresso aritmtica.
EUREKA! N31, 2010

Sociedade Brasileira de Matemtica

SEGUNDO DIA
PROBLEMA 4

Seja ABC um tringulo com AB = AC. As bissectrizes dos ngulos CAB e ABC intersectam os lados BC e CA em D e E, respectivamente. Seja K o incentro do tringulo ADC. = 45. Determine todos os possveis valores de C Suponha que BEK AB.
PROBLEMA 5

Determine todas as funes f do conjunto dos inteiros positivos no conjunto dos inteiros positivos tais que, para todos os inteiros positivos a e b, existe um tringulo no degenerado cujos lados medem, a, f (b) e f (b + f(a) 1). (Um tringulo no degenerado se os seus vrtices no so colineares).
PROBLEMA 6

Sejam a1 ,a2 ,...,an inteiros positivos distintos e M um conjunto de n 1 inteiros positivos que no contm o nmero s = a1 + a2 + ... + an . Um gafanhoto pretende saltar ao longo da recta real. Ele comea no ponto 0 e d n saltos para a direita de comprimentos a1 ,a2 ,...,an , em alguma ordem. Prove que essa ordem pode ser escolhida de modo que o gafanhoto nunca caia num ponto de M.

EUREKA! N31, 2010

10

Sociedade Brasileira de Matemtica

XXIV OLIMPADA IBEROAMERICANA DE MATEMTICA


Enunciados e resultado Brasileiro
A XXIV Olimpada Iberoamericana de Matemtica foi realizada na cidade de Santiago de Queretaro, Mxico no perodo de 17 a 27 de setembro de 2009. A equipe brasileria foi liderada pelos professores Onofre Campos, de Fortaleza CE e Luzinalva Miranda de Amorim, de Salvador BA.

RESULTADOS DA EQUIPE BRASILEIRA


BRA1 BRA2 BRA3 BRA4 Renan Henrique Finder Matheus Secco Torres da Silva Marco Antonio Lopes Pedroso Marcelo Tadeu de S Oliveira Sales Medalha de Ouro Medalha de Ouro Medalha de Prata Medalha de Prata

PRIMEIRO DIA PROBLEMA 1

Seja n um natural maior que 2. Suponhamos que n ilhas estejam localizadas ao redor de um crculo e que entre cada duas ilhas vizinhas haja duas pontes, como na figura:

x1

x2

xn

x3

xn 1

xj

Partindo da ilha x1 , de quantas maneiras se podem percorrer as 2n pontes passando por cada ponte exatamente uma vez?
EUREKA! N31, 2010

11

Sociedade Brasileira de Matemtica

PROBLEMA 2

Para cada inteiro positivo n definimos an = n + m, onde m o maior inteiro tal que
22 n 2n . Determinar quais inteiros positivos no aparecem na sequncia an .
m

PROBLEMA 3

Sejam C1 e C2 duas circunferncias de centros O1 e O2 , com o mesmo raio, que se intersectam em A e B. Seja P um ponto sobre o arco AB de C2 que est dentro de C1 . A reta AP intersecta C1 em C, a reta CB intersecta C2 em D e a bissetriz de CAD intersecta C1 em E e C2 em L. Seja F o simtrico do ponto D em relao ao ponto mdio de PE. Demonstrar que existe um ponto X que satisfaz XFL = XDC = 30 e CX = O1O2 .
SEGUNDO DIA PROBLEMA 4

Seja ABC um tringulo com AB AC. Sejam I o incentro de ABC e P o outro ponto de interseo da bissetriz externa do ngulo A com o circuncrculo de ABC. A reta PI intersecta o circuncrculo de ABC no ponto J. Demonstrar que os circuncrculos dos tringulos JIB e JIC so tangentes s retas IC e IB, respectivamente.
PROBLEMA 5

A sequncia an est definida por


1 , para todo inteiro k 1. a2 k Demonstrar que todo nmero racional positivo aparece exatamente uma vez nesta sequncia. a1 = 1, a2 k = 1 + ak e a2 k +1 =
PROBLEMA 6

Ao redor de uma circunferncia marcam-se 6000 pontos, cada um dos quais se pinta com uma de 10 cores dadas, de modo que entre quaisquer 100 pontos consecutivos sempre figuram as 10 cores. Achar o menor inteiro k com a seguinte propriedade: para toda colorao deste tipo existem k pontos consecutivos nos quais se encontram as 10 cores.

EUREKA! N31, 2010

12

Sociedade Brasileira de Matemtica

PAR OU MPAR? EIS A QUESTO


Einstien do Nascimento Jr e Samuel Barbosa Feitosa
Paridade Quando duas pessoas esto indecisas sobre uma escolha, muitas vezes elas utilizam uma brincadeira chamada Par ou mpar para se decidirem. Por trs desse simples critrio, podem se resolver problemas que parecem ser bastante complicados. Dizemos que um nmreo tem paridade par se ele for par, e paridade mpar, se ele for mpar. Observar a paridade de um nmero algo bem simples mas com aplicaes fantsticas em problemas de olimpadas. Vejamos um exemplo: Paridade como invariante Vamos comear com um problema bastante famoso que j foi utilizado at em entrevistas para grandes empresas de computao. Problema 1. 100 pessoas so postas em uma fila e cada uma delas recebe um chapu, que pode ser preto ou branco. Cada pessoa s consegue ver os chapus das pessoas que esto a sua frente. pedido que cada uma delas tente adivinhar a cor do seu chapu. Qual o mximo nmero de acertos que se pode garantir, dado que as pessoas podem combinar uma estratgia antes de receb-los. Soluo: Facilmente consegue-se 50 acertos. Podemos dividir as pessoas em pares: (100,99), (98, 97),...(2, 1) e assim o maior nmero de cada par falar a cor da pessoa da frente. Que apenas precisa repeti-lo, para garantir 1 acerto por par. De uma forma um pouco mais elaborada, se garante 66 acertos. Separando em trios: (100, 99, 98),...(4, 3, 2). O maior nmero de cada trio pode falar BRANCO caso os dois da sua frente tenham a mesma cor e PRETO, caso as cores sejam distintas. Assim, aps o maior nmero falar, o nmero do meio pode acertar sua cor e em seguida, o primeiro do trio pode acertar a dele. Curiosamente esse nmero pode chegar a 99 acertos utilizando esse poderoso argumento que a paridade. Notemos que ningum sabe a cor do ltimo da fila. Ento no importa a estratgia de ordem das pessoas, nenhuma informao pode ser obtida para esse chapu. O que no ocorre com os 99 chapus restantes. Note ainda que a diferena de conhecimentos entre a pessoa e a pessoa que encontra atrs dela apenas o seu chapu. Ento, basta seguir a estratgia: As cores sero faladas das pessoas de trs para as da frente. E a ltima pessoa vai falar BRANCO caso a quantidade de chapus brancos a sua frente seja par e PRETO, caso contrrio. Como a 99. pessoa sabe a paridade da quantidade de chapus brancos estritamente sua frente, e a paridade da
EUREKA! N31, 2010

13

Sociedade Brasileira de Matemtica

quantidade de chapus brancos sua frente, incluindo ela mesma, que foi informada pela 100. pessoa, ela acertar o seu chapu. A 98., computando ambas as informaes pode acertar o dela, e assim sucessivamente. Problema 2. Em cada casa de um tabuleiro de 5 x 5 est escrito 1 ou 1. Em cada passo troca-se o nmero de cada uma das 25 casas pelo resultado da multiplicao dos nmeros de todas as suas casas vizinhas. Inicialmente se tem o tabuleiro da figura. Mostre como fica o tabuleiro ao final de 2004 passos. Observao: Duas casas so vizinhas se tiverem um lado em comum. 1 1 1 1 1 1 1 1 1 1 1 1 1 1 1 1 1 1 1 1 1 1 1 1 1

Dica: Muitas vezes, quando no se tem ideia de como ser a soluo de uma questo, pode-se obter vrias pistas fazendo alguns casos iniciais do enunciado, esperando observar algum padro. Meus nmeros da sorte so 5 e 9. Ao achar um padro repetitivo, basta analisar em que caso cair o nmero 2004. Problema 3. Em cada um dos 10 degraus de uma escada existe uma r. Cada r pode, de um pulo, colocar-se em outro degrau, mas quando uma r faz isso, ao mesmo tempo, uma outra r pular a mesma quantidade de degraus em sentido contrrio: uma sobe e outra desce. Conseguiro as rs colocar-se todas juntas num mesmo degrau? Soluo: Uma maneira muito utilizada para atacar problemas onde dada uma condio inicial e um conjunto de operaes para manipul-la tentar procurar o que no muda, independentemente dos movimentos que utilizamos. Note que se uma r vai de um degrau par para um mpar (muda de paridade), a outra r que se movimenta com ela tambm pular um nmero mpar de degraus, mudando tambm a paridade. Caso a primeira no mude, a sua parceira de movimento tambm permanecer num degrau de mesma paridade. UM INVARIANTE: Paridade da quantidade de rs em degraus de nmero par (comprove testando os movimentos possveis). Como na posio inicial h 5 rs nos degraus de posio par e na posio final h ou dez ou zero rs nos degraus de posio par, a posio final NO pode ser obtida da posio inicial apenas fazendo essas operaes permitidas.
EUREKA! N31, 2010

14

Sociedade Brasileira de Matemtica

Essa estratgia de invariantes utilizada principalmente para provar a impossibilidade de ocorrer algum evento. Definiremos uma pea prncipe (que no existe no jogo de xadrez) como uma que s pode andar na horizontal e vertical, uma casa por vez. Um jeito comum de fazer notaes em um tabuleiro de xadrez nomear as colunas da esquerda para a direita de a a h e as linhas de baixo para cima de 1 a 8 tomando o referencial da pessoa que joga com as casas brancas. Problema 4. Sobre um tabuleiro de xadrez, um prncipe comea do quadrado a1 e retorna aps fazer alguns movimentos. Mostre que o prncipe fez um nmero par de movimentos. Soluo: veja que em cada movimento, o prncipe muda para uma casa de cor oposta. Como a casa a1 preta, aps um nmero mpar de movimentos o prncipe estar numa casa da cor branca. Para ele ter retornado at a casa preta do incio, ele dever ter feito um nmero par de movimentos. Problema 5. Pode um prncipe comear do quadrado a1 de um tabuleiro de xadrez, ir at o quadrado h8, visitando cada um dos quadrados restantes exatamente uma vez? Soluo: A resposta no. Em cada movimento, o prncipe pula para um quadrado da cor oposta. Como o prncipe tem que fazer 63 movimentos, o ltimo movimento ir deix-lo em uma casa da cor oposta a cor de a1. Entretanto, a1 e h8 tm a mesma cor. Isto um absurdo. O ltimo problema nos conduz a um tipo muito importante de demonstrao: prova por absurdo. Suponha que lhe perguntaram se possvel somar cinco nmeros mpares e obter o nmero 100. Aps algumas tentativas voc comea a desconfiar que isto no possvel. Mas como provar que no possvel? Se realmente fosse possvel somar 5 nmeros mpares e obter 100 o que aconteceria? Como a soma de cinco nmeros mpares sempre mpar obteramos que 100 um nmero mpar. Mas 100 no mpar! Logo no possvel existirem tais 5 nmeros. Para provar que algo no possvel, basta supormos que possvel e chegarmos a um absurdo. Problema 6. Uma linha poligonal fechada composta por 11 segmentos. Pode uma reta (no contendo um vrtice da linha poligonal) intersectar cada um desses segmentos?
EUREKA! N31, 2010

15

Sociedade Brasileira de Matemtica

Problema 7. Trs bolas de gude, A, B, e C esto no cho. Um movimento permitido passar uma bola entre as outras duas. possvel, aps 25 movimentos, que todas as bolas estejam nas suas posies originais? Dica: Que horas so? (Sentidos horrio e anti-horrio...) Problema 8. Ktia e seus amigos esto em um crculo. Sabemos qua ambos os vizinhos de cada criana so do mesmo sexo. Determine o nmero de garotas sabendo que existem 5 garotos no crculo. Dica: Comece a analisar por um vizinho da Ktia. Problema 9. (Rssia 1970) O rei Luis estava desconfiado de alguns de seus cortesos. Ele fez uma lista completa de cada um dos seus cortesos e disse a cada um deles para espionar um outro corteso. O primeiro da lista foi espionar o corteso que estava espionando o segundo da lista, o segundo da lista foi espionar o corteso que estava espionando o terceiro da lista, e assim sucessivamente; o penltimo foi espionar o corteso que estava espionando o ltimo e o ltimo foi espionar o corteso que estava espionando o primeiro. Prove que o rei Luis tinha um nmero mpar de cortesos. Soluo. Seja n o nmero de corteso da lista e suponha que n par. Coloque-os sentados ao redor de uma mesa circular de modo que cada um esteja espionando o seu vizinho da direita.
1

Y
n 2

X 2

O corteso 1 espia o corteso X que espia o corteso 2, o corteso 2 espia o n corteso Z que espia o corteso 3, e assim sucessivamente at que o corteso 2 n espia o corteso Y que espia o corteso + 1. Como os nmeros 1, 2, 3,...,n devem 2
EUREKA! N31, 2010

16

Sociedade Brasileira de Matemtica

se alternar sobre o crculo, conclumos que o corteso ou seja, n = 0. Esse absurdo mostra que n mpar.

n + 1 igual ao corteso 1, 2

Problema 10. Um cubo 1 1 1 est posicionado em um plano quadriculado de modo que uma de suas faces coincide com um dos quadradinhos do plano. Em cada movimento podemos tombar o cubo por uma de suas arestas, fazendo coincidir uma face, que tinha essa aresta, com um dos quadradinhos do plano. possvel fazer o cubo voltar a sua posio inicial aps 2005 movimentos? Dica: Algum a joga xadrez? Paridade e Contagens Nesta seo, abordaremos duas ideias muito simples: 1. Se contamos os elementos de um conjunto de duas maneiras diferentes, os valores obtidos devem ter a mesma paridade (Porque so iguais!) 2. Se os elementos de um conjunto podem ser pareados ento o conjunto tem uma quantidade par de elementos. Problema 11. Em Brasilndia existem apenas 9 casas muito distantes entre si. possvel que cada casa esteja ligada a exatamente 7 outras casas atravs de estradas? Soluo: No possvel. Some a quantidade de estradas que saem de cada casa. Bem, facilmente obtemos 9 7 estradas. Como cada estrada liga duas cidades, a contagem que fizemos contou cada estrada duas vezes. Logo o nmero obtido teria que ser par. Voc deve ter ficado com uma pulga atrs da orelha. Ser que cada casa ligada a exatamente 7 outras foi realmente crucial? possvel revolvermos o problema anterior com um eneunciado mais geral: Problema 12. Prove que numa festa com n pessoas, o nmero de pessoas que conhecem um nmero mpar de outras pessoas na festa par. Soluo: Numere as pessoas de 1 at n e denote por d i o nmero de amigos da pessoa i. Imagine que existe um fio entre duas pessoas que se conhecem. Se E denota a quantidade de fios, temos d1 + d2 + ... + d n = 2 E,
EUREKA! N31, 2010

17

Sociedade Brasileira de Matemtica

pois cada fio contado duas vezes, um para cada ponta. Como o lado direito par, no lado esquerdo devemos ter uma quantidade par de nmeros mpares. Problema 13. (Olimpada de Maio 2000) O conjunto {1, 2, 3, 4} pode ser dividido em dois subconjuntos A = {1,4} e B = {3, 2} sem elementos comuns e tais que a soma dos elementos de A seja igual soma dos elementos de B. Essa diviso impossvel para o conjunto {1, 2,3, 4,5} e tambm para o conjunto {1, 2, 3, 4, 5, 6} . Determine todos os valores de n para os quais o conjunto dos primeiros n nmeros naturais pode ser dividido em dois subconjuntos sem elementos comuns tais que a soma dos elementos de cada subconjunto seja a mesma. Soluo. Como a soma dos elementos de A deve ser igual soma dos elementos de B, a soma dos nmeros do conjunto {1, 2,3,...,n} deve ser o dobro da soma dos elementos de A, ou seja, deve ser um nmero par. Voc j deve saber que
. 2 Voc no sabia disso? No fique a parado! Tente descobrir porque isso verdade! n ( n + 1) Veja que par se n ( n + 1) mltiplo de 4. Como estamos interessados no 2 resto na diviso por 4 de algum nmero, talvez seja interessante procurar quais os possveis restos de n na diviso por 4. Podemos escrever n na forma n = 4q + r onde r = 0,1, 2 ou 3. Mos obra! n ( n + 1) = 2q ( 4q + 1) par. 1. Se n = 4q ento 2 n ( n + 1) 2. Se n = 4q + 1 ento = ( 2q + 1)( 4q + 1) mpar. 2 n ( n + 1) 3. Se n = 4q + 2 ento = ( 2q + 1)( 4q + 1) mpar. 2 n ( n + 1) 4. Se n = 4q + 3 ento = ( 2q + 2 )( 4q + 3) par. 2 Podemos concluir que n deve ser da forma 4q ou 4q + 3. Acabou? No! Precisamos construir EXEMPLOS para cada uma dessas possibilidades mostrando que realmente esses valores satisfazem as condies do problema. Para n = 4q, considere os conjuntos 1 + 2 + 3 + ... + n = n ( n + 1)

EUREKA! N31, 2010

18

Sociedade Brasileira de Matemtica

B = {( 2,3) ,( 6,7 ) ,(10 ,11) ,...,( 4q 2 ,4q 1)} .

A = {(1, 4 ) ,( 5,8 ) ,( 9 ,12 ) ,...,( 4q 3, 4q )} .

Para n = 4q + 3, considere os conjuntos A = {( 4,7 ) ,( 8,11) ,(12,15 ) ,...,( 4q, 4q + 3)} {(1, 2 )} .

B = {( 5,6 ) ,( 9 ,10 ) ,(13,14 ) ,...,( 4q + 1, 4q + 2 )} {( 3)} .

Note que os conjuntos foram divididos em parntesis. Cada parntese de A possui correspondente em B com a mesma soma, facilitando a construo de um exemplo generalizado. Problema 14. Podemos desenhar uma linha poligonal fechada feita por 9 segmentos de reta, cada um deles intersectando exatamente outro segmento? Soluo. Se tal construo possvel, ento todos os segmentos podem ser agrupados em pares de segmentos intersectantes. Mas o nmero de segmentos mpar! Absurdo! Os prximos dois problemas tratam de domins. Um domin consiste de um tabuleiro 1 x 2 com pontos em cada casinha. A quantidade de pontos varia de 0 at 6. Ento, o nmero total de domins distintos 28. Problema 15. Todos os domins so arranjados em uma cadeia de duas pontas (a quantidade de pontos na extremidade de dois domins consecutivos a mesma). Se em uma ponta existe o nmero 5, qual o nmero de outra ponta? Problema 16. Em um conjunto de domins, descartamos todos aqueles que possuem pelo menos uma casinha vazia. possvel arranjarmos todos os restantes em uma cadeia? Problema 17. (Eslovnia 1992) Prove que para quaisquer inteiros positivos a1 ,a2 ,...,an o nmero:
a1 a2 + a2 a3 + ... + an a1

par. Observao: x y chamado de valor absoluto da diferena entre x e y e denota o mximo entre x y e y x. Na reta real, ele representa a distncia entre os nmeros x e y.

EUREKA! N31, 2010

19

Sociedade Brasileira de Matemtica

Soluo: Perceba que x y = x y para alguma escolha de sinais. Ento a soma total a1 a2 a2 a3 ... an a1 . Como cada nmero ai aparece duas vezes, basta mostrarmos que cada uma das expresses a1 ai par para qualquer escolha de sinais. Vejamos os casos: 1. 2. 3. 4.
ai ai ai ai ai ai ai ai = + ai + ai = ai + ai = + ai ai = ai ai = 2ai par. = 0 par. = 0 par. = 2ai par.

1.3 Miscelnia Problema 18. Podemos trocar uma nota de 25 reais usando dez notas que podem assumir os valores 1, 3, 5? Soluo. No. Como a soma de um nmero par de nmeros mpares par, a soma dos valores dessas 10 notas s pode ser um nmero par. Mas 25 mpar. Problema 19. Peter comprou um caderno com 96 folhas, e numerou com os nmeros de 1 at 192. Victor rasgou 25 folhas consecutivas do caderno, e adicionou os 50 nmeros. Victor pode ter obtido o nmero 1990 como resultado da soma?
1 1 1 1 1 1 + + + + + = 1 no admite solues a b c d e f com todos os nmeros sendo mpares. Dica: Faa o produto dos denominadores.

Problema 20. Prove que a igualdade

Problema 21. O produto de 21 inteiros igual a 1. Mostre que sua soma no pode ser zero. Dica: compare as quantidades de nmeros positivos e negativos. Problema 22. Trs gafanhotos esto brincando ao longo da uma linha. Na sua vez, cada gafanhoto pode pular sobre um outro gafanhoto, mas no sobre os outros dois. Eles podem retornar para suas posies iniciais aps 1991 movimentos?

EUREKA! N31, 2010

20

Sociedade Brasileira de Matemtica

Soluo. Sejam A, B, C os trs gafanhotos. Estaremos interessados apenas na ordem em que os gafanhotos se dispem ao longo da reta, digamos que inicialmente eles esto na ordem (A, B, C). Podemos fazer os seguintes movimentos:

( A,B,C ) ( B,A,C ) ( B,C,A) ( C,B,A) ...


Em cada passo, disponha as letras A, B e C em um crculo (como mostra a figura) e leia a palavra ABC. Percebeu alguma coisa? Antes de efetuarmos nosso primeiro movimetno, a leitura estava no sentido horrio e logo em seguida passou para o sentido anti-horrio. Como cada movimento alternar os sentidos, aps 1991 movimentos estaremos em um sentido diferente do original. Logo, no possvel retornarmos para a posio original.

Observao: Compare com o problema 7. Problema 23. Os nmeros de 1 at 10 so escritos em uma linha. Podemos colocar os sinais + e entre eles de modo que o resultado da expresso resultante seja 0? Soluo: No possvel. Perceba que quando escolhemos um nmero para trocarmos de sinal, por exemplo, de + para , a soma total varia o dobro do nmero escolhido, ou seja, a paridade da soma no muda. Basta ver agora que 1 + 2 + ... + 10 = 55 no tem a mesma paridade que 0. Um INVARIANTE a paridade da soma. Problema 24. Um gafanhoto pula ao longo de uma linha. No seu primeiro pulo, ele anda 1cm, no segundo 2cm, e assim sucessivamente. Ele pode pular para a esquerda ou para a direita. Mostre que aps 1985 pulos, o gafanhoto no pode retornar ao ponto em que comeou. Dica: Perceba que voc pode associar aos pulos do gafanhoto um nmero com
EUREKA! N31, 2010

21

Sociedade Brasileira de Matemtica

sinal (+ se o pulo para a esquerda e se para a direita). Agora use o problema anterior. Problema 25. Os nmeros 1, 2,...,1984, 1985 so escritos em um tabuleiro. A operao permitida apagar dois nmeros e colocar sua diferena positiva. Aps algumas operaes, resta apenas um nico nmero no tabuleiro. Pode este nmero ser 0? Problema 26. Pode um tabuleiro 8 8 ser coberto com domins 1 2 de modo que somente os quadrados a1 e h8 no sejam cobertos? Soluo. No possvel. Pinte o tabuleiro de preto e branco da maneira usual. Cada domin cobre exatamente um quadrado preto e outro branco (Invariante), portanto, a quantidade de quadrados pretos cobertos igual quantidade de quadrados brancos cobertos. Como a1 e h8 tm a mesma cor, sobrariam 30 quadrados de uma cor e 32 de outra para serem cobertos. Absurdo! Problema 27. 45 pontos so escolhidos sobre a reta AB, todos fora do segmento de reta AB. Prove que a soma das distncias desses pontos ao ponto A no pode ser igual soma das distncias ao ponto B. Soluo. Sejam A e B dispostos, sem perda de generalidade como na figura abaixo. Tomemos um ponto X.
A B X

X pode estar direita de B ou esquerda de A. Ou ocorre: AX + AB = BX ou BX + AB = AX . Assim, se estivssemos somando em x as distncias dos 45 pontos para A e em y para B, estaramos na verdade, s somando uma diferena de AB em x ou em y. Como 45 mpar, no podemos distribuir uma igual quantidade de AB s para o grupo de A e o de B. Assim, segue que no possvel. Problema 28. Um nmero de 17 dgitos somado com o seu reverso (um nmero com os mesmo dgitos mas escritos na ordem inversa). Mostre que sua soma contm pelo menos um dgito par. Problema 29. Existem 100 soldados em um quartel. Toda noite, trs deles ficam de guarda. Aps um certo perodo de tempo, possvel que cada soldado tenha ficado de guarda exatamente uma vez com cada outro soldado?
EUREKA! N31, 2010

22

Sociedade Brasileira de Matemtica

Soluo: Suponha, por absurdo, que seja possvel. Tomemos o Soldado Ryan, ele possui 99 companheiros. Suponha que ele em particular tenha conseguido ficar exatamente uma vez de pernoite com cada um dos outros. A cada dia, Ryan formava 2 duplas diferentes, que no poderiam se repetir nos dias posteriores. Caso Ryan tivesse pernoitado x vezes, a quantidade de duplas que ele teria formado seria 2x, que por hiptese, deve ser igual a 99. Chegando concluso que 99 par. Absurdo! Problema 30. 25 garotos e 25 garotas esto sentados ao redor de uma mesa. Prove que sempre possvel encontrar uma pessoa tal que ambos os seus vizinhos so garotas. Soluo: Suponha, por absurdo, que no necessariamente haja uma pessoa que possua duas garotas como vizinhas. Denotemos h para garoto e m para garota. Cada pessoa ou possui como vizinho 2h ou h+m. Somando todas as 50 possibilidades, devemos estar contando cada pessoa duas vezes (j que essa vizinho de duas pessoas). Assim: x ( 2h ) + y ( h + m ) = 50h + 50m onde x o nmero de pessoas que tm 2 garotos como vizinhos e y o nmero de pessoas que tm um garoto e uma garota. Notemos ainda que x + y = 50. Obtemos xh = ( 50 y ) m assim xh = xm. Mas x garotos s sero iguais a x garotas, se x for nulo. Assim, todas as pessoas tm um garoto e uma garota como vizinhos. Pintemos as 50 posies do crculo apenas de branco e preto. E analisemos apenas as pretas. Todas as pretas tero que ter vizinhos sendo um garoto e uma garota. Logo, as casas brancas sero alternadas: garoto, garota, garoto... Absurdo. Pois com 25 casas brancas, na ltima e na primeira brancas haver 2 garotos. Absurdo! Segue o resultado. Problema 31. (Ucrnia 1997) Um tabuleiro colorido de branco e preto da maneira usual, e cada casa contm um inteiro. Sabemos que a soma dos nmeros em cada coluna e a soma dos nmeros em cada linha par. Mostre que a soma dos nmeros nas casas pretas par. Soluo. Suponha sem perda de generalidade que o quadrado do canto esquerdo superior preto. A partir desse quadrado, numere as colunas da esquerda para a direita e as linhas de cima para baixo. Some os nmeros das colunas em posies mpares e os nmeros das linhas em posies pares. Perceba que cada quadrado preto do tabuleiro contado apenas uma vez nessa soma enquanto que os quadrados brancos das linhas e colunas mencionadas so contados duas vezes. Logo, esse soma tem a mesma paridade que a soma de todos os nmeros escritos
EUREKA! N31, 2010

23

Sociedade Brasileira de Matemtica

nos quadrados pretos. Como a soma de quaisquer linhas e colunas par, a soma dos nmeros nos quadrados pretos par.

Problema 32. Considere um tabuleiro 1998 2002 pintado alternadamente de preto e branco da maneira usual. Em cada casa do tabuleiro, escrevemos 0 ou 1, de modo que a quantidade de 1s em cada linha e em cada coluna do tabuleiro mpar. Prove que a quantidade de 1s escritos nas casa brancas par. Dica: Tente imitar a soluo anterior. Problema 33. (Austrlia 2007) Em cada casa de um tabuleiro 2007 2007 escrevemos um nmero inteiro mpar. Sejam Z i a soma dos nmeros na i-sima linha e S j a soma dos nmeros na j-sima coluna, para 1 i, j 2007. Alm disso, sejam A = Z i Z 2 ...Z 2007 e B = S1 S 2 ...S 2007 . Mostre que A + B no pode ser igual a zero. Problema 34. (China 1986) possvel arranjar os nmeros 1, 1, 2, 2, 3, 3,...,1986, 1986 em fila de modo que entre quaisquer dois is hajam (i 1) nmeros? Soluo: Vamos tentar fazer alguns casos pequenos. fcil ver que no conseguimos fazer o que o enunciado pede com os nmeros 1, 1, 2, 2 mas com os nmeros 1, 1, 2, 2, 3, 3, 4, 4 temos um exemplo: 1. 2. 3. 4. 5. 6. 7. 8. a3 a4 a2 b3 b2 b4 a1 b1 3 4 2 3 2 4 1 1 Contados da squerda para a direita, denotemos por ai e bi as posies do primeiro e segundo nmero i, respectivamente. No nosso exemplo, a2 = 3 e b2 = 5. Como existem i 1 nmeros entre dois nmeros is, devemos ter bi ai = i. Se possvel escrever os nmeros 1, 1, 2, 2, ..., n, n em linha como no enunciado,
EUREKA! N31, 2010

24

Sociedade Brasileira de Matemtica

obtemos:

( a1 + a2 + ...an ) + ( b1 + b2 + ...bn ) = 1 + 2 + ... + 2n = n ( 2n + 1) n ( n + 1) . ( b1 a1 ) + ( b2 a2 ) + ...( bn an ) = 1 + 2 + ...+ n =


2 2 ( b1 + b2 + ...bn ) = n ( 5n + 3 ) 2 n ( 5n + 3) 2

Somando as duas linhas,

Como o lado esquerdo sempre par, a frao

deve ser um inteiro par.

Isso j restringe os possveis valores de n. Para n = 1986, n ( 5n + 3 ) = 9863469 2 mpar e conseqentemente no possvel dispormos esses nmeros em linha. Uma pergunta natural que voc deve tentar responder : para quais n tal distribuio possvel? Problema 35. possvel arranjar os nmeros de 1 at 9 em uma sequncia, de modo que exista uma quantidade mpar de nmeros entre 1 e 2, entre 2 e 3,..., e entre 8 e 9? Problema 36. (Rssia 1984) O nmero de todos os inteiros positivos de 64 dgitos sem zeros em sua representao e que so divisveis por 101 par ou mpar? Soluo: Precisamos bolar alguma maneira de agrupar os nmeros em pares. Seja A = 11 ...3 110 repeties do nmero 1. 1 2
64 vezes

Como 1111 mltiplo de 101 fcil ver que A mltiplo de 101. Para todo nmero de 64 dgitos a = a1a2 ...a63 a64 , sem zeros em sua representao decimal, considere o seu conjugado b = b1b2 ...b63b64 = (10 a1 )(10 a2 ) ...(10 a64 ). Nenhum dgito de a igual a zero, portanto, cada nmero 10 ai pertence ao conjunto

{1,2,..,9} . Da equao a + b = A obtemos que a divisvel por 101 se e somente se


64 vezes

b divisvel por 101 (lembre-se que A mltiplo de 101). Como o nico nmero que igual ao seu conjugado o nmero 55 ...55 (que mltiplo de 101) e os 123 demais nmeros que satisfazem o enunciado podem ser pareados, conclumos que a
EUREKA! N31, 2010

25

Sociedade Brasileira de Matemtica

quantidade procurada mpar. inteiros positivos ( a1 ,a2 ,...,an ) tais que Problema 37. (Putnam 1997) Seja Bn a quantidade de n uplas ordenadas de
1 1 1 + + ... + = 1 a1 a2 an B10 par ou mpar?

Soluo: Uma ideia natural tentar agrupar as solues em pares. Qualquer soluo com a1 a2 pode ser pareada com a outra soluo obtida pela troca de posio entre a1 e a2 . Logo, B10 tem a mesma paridade que o nmero de solues com a1 = a2 . Das solues com a1 = a2 , podemos parear aquelas que tem a3 a4 da mesma maneira. Repetindo esse argumento com ( a5 ,a6 ) ,( a7 ,a8 ) e ( a9 ,a10 ) , conclumos que a paridade de B10 a mesma do nmero de solues com a5 = a6 ,a7 = a8 e a9 = a10 , ou seja, das solues de: 2 2 2 2 2 + + + + = 1. a1 a3 a5 a7 a9 Como anteriormente, podemos nos restrigir quantidade de solues com a1 = a3 e a5 = a7 , que igual ao nmero de solues da equao: 4 4 2 + + = 1. a1 a5 a9 Mais uma vez, podemos nos restringir quantidade de solues com a1 = a5 , que igual ao nmero de solues da equao: 8 2 + = 1. a1 a9 Agora ficou fcil! Basta contar explicitamente o nmero de solues da equao anterior. Como fazer isso? Bem, ela pode ser fatorada como: ( a1 8 )( a9 2 ) = 16 que admite 5 solues correspondendo s fatoraes de 16 como 2i 24i para i = 0,1,2 ,3,4. Ento B10 mpar. Problema 38: Prove que numa festa com 2n pessoas existem duas com um nmero par de amigos em comum.

EUREKA! N31, 2010

26

Sociedade Brasileira de Matemtica

Soluo: Suponha que quaisquer duas pessoas tenham um nmero mpar de amigos em comum e seja A um dos participantes da festa. Seja M = { F1 ,F2 ,...,Fk } o conjunto dos amigos de A. Considere uma nova festa restrita apenas ao conjunto M. Como cada Fi tem um nmero mpar de amigos em comum com A, na nova festa, cada Fi possui um nmero mpar de amigos. Pelo problema 12, k deve ser par. O mesmo argumento vale para qualquer pessoa na festa e conseqentemente todos tm um nmero par de amigos. Pea para cada um dos amigos de A fazerem uma lista de seus amigos diferentes de A. A soma da quantidade de nomes listados par, pois uma soma de uma quantidade par (igual a k) de nmeros mpares (cada Fi possui um nmero mpar de amigos diferentes de A). Agora comparemos o nmero de aparies de cada uma das 2n 1 pessoas diferentes de A nessas listas. Se cada uma delas aparecer em um nmero mpar de listas, a soma total de todos os nomes em todas as listas seria mpar. (Lembre-se que a soma de uma quantidade mpar de nmeros mpares mpar!). Mas isso uma contradio. Logo, existe uma pessoa diferente de A que aparece em um nmero par de listas, e portanto tem um nmero par de amigos em comum com A. Problema 39. Alex desenhou uma coleo de K retas no plano em posio geral (quaisquer duas retas se intersectam em um ponto e quaisquer trs definem um tringulo no degenerado). Para quais valores de K sempre possvel (no importa como as retas so desenhadas) colocar um elemento do conjunto {1, 2 ,...,K 1} em cada ponto de interseo das retas de modo que em toda reta no existam nmeros iguais. Problema 40. (Rssia) Em cada planeta de um sistema solar existe um astrnomo observando o planeta mais prximo. As distncias entre os planetas so distintas duas a duas. Demonstre que se a quantidade de planetas mpar, ento existe pelo menos um planeta que no observado. Dica: Procure as cadeias de planetas que um olha para o outro que olha para o outro com mais de 2 planetas.
REFERNCIAS
[1] D. Fomin, S. Genkin e I. Itenberg, Mathematical Circles, MAS (1996). [2] C. Augusto, S. Feitosa, B. Holanda e Y. Lima, treinamento Cone Sul 2007, Fortaleza, Realce (2007). [3] P. J. Taylor, Tournament of the Towns 1980 to 1984, Australian Mathematical Trust (1993). [4] D. Fomin e A. Kirichenko, Leningrand Mathematical Olympiadas 1987-1991, MathPro Press (1994). [5] E. Wagner, Paridade, Eureka! No. 2, pp. 32-38, (1998).

EUREKA! N31, 2010

27

Sociedade Brasileira de Matemtica

GEOMETRIA DO TRINGULO: FATOS E PROBLEMAS


Carlos Yuzo Shine
1. O Teorema de Miquel Comeamos com o teorema em si, que um dos vrios pequenos milagres dos chamados quadrilteros completos (veja um pouco mais desses milagres nos exerccios!), que so os quadrilteros conhecidos unidos com as retas que contm os lados. Isto , um quadriltero completo a unio de quatro retas em vez de quatro segmentos. Teorema de Miquel. Sejam a, b, c, d quatro retas coplanares, de modo que no h duas paralelas nem trs concorrentes. Os circuncrculos dos quatro tringulos determinados pelas quatro retas passam por um mesmo ponto, denominado ponto de Miquel das quatro retas.

Demonstrao: Seja M a interseco dos circuncrculos de CEF e BDF na figura acima. Ento MEA = MEC = 180 MFC = BFM = BDM = 180 ADM , de modo que MEA + ADM = 180 e, portanto, MDAE inscritvel. Isso quer dizer que M pertence ao circuncrculo de ADE. Analogamente, prova-se que M pertence ao
EUREKA! N31, 2010

28

Sociedade Brasileira de Matemtica

circuncrculo de ABC. Vamos resolver, a ttulo de exemplo, o problema 6 da olimpada norteamericana de 2006. Exemplo 1.1. (USAMO 2006, Problema 6) Seja ABCD um quadriltero e E e F os pontos sobre os lados AD e BC, respectivamente, tal que AE / ED = BF / FC. A semirreta FE corta as semirretas BA e CD em S e T, respectivamente. Prove que os circuncrculos dos tringulos SAE, SBF, TCF e TDE passam por um mesmo ponto. Resoluo: Ao fazer a figura, voc provavelmente vai notar uma certa semelhana com a figura anterior.

Queremos provar que os pontos de Miquel de ADTS e BCTS coincidem! Isso no difcil, na verdade: seja M a interseco dos circuncrculos de SAE e SBF. Mostraremos que os circuncrculos de TED e TFC tambm passam por M. Um arrasto e uma semelhana do conta do recado: primeiro, note que AME = ASE = BSF = BMF e MEA = MSA = MSB = MFB. Ento os tringulos AME e BMF so semelhantes, e da igualdade AE / ED = BF / FC os tringulos MAD e MBC so semelhantes tambm. Uma rpida verificao mostra que MAB e MDC tambm so semelhantes: de fato (pois como MAD e MBC so
EUREKA! N31, 2010

29

Sociedade Brasileira de Matemtica

AM DM = (novamente da semelhana). BM CM Voc pode imaginar que o tringulo MAD gira em torno de M e, aps um acerto de escala, transformado no tringulo MBC. Isso uma transformao geomtrica conhecida como roto-homotetia de centro M. Assim, A levado em B e D elevado em C. Note que a semelhana obtida anteriormente envolve o centro de roto-homotetia M, os pontos e suas imagens na transformao. Isso na verdade sempre acontece ( uma das semelhanas automticas). Agora podemos terminar o problema: da semelhana entre MAB e MDC, os ngulos externos MDT e MAS so congruentes. Como M pertence ao circuncrculo de SAE, MAS = MES = MET , ou seja, MDT = MET , o que significa que MEDT cclico e, portanto, M pertence ao circuncrculo de TED. Utilizando outra semelhana automtica, entre MEF e MDC (pois E levado em F!), prova-se que M pertence tambm ao circuncrculo de TFC. Note que se U a interseo de AB e CD, ento pelo teorema de Miquel M tambm pertence ao circuncrculo de STU. Ento na vaerdade cinco crculos passam pelo ponto M! A seguinte verso do teorema de Miquel tambm til:

semelhantes ento DMA = CMB ) e

Teorema de Miquel para tringulos. Seja ABC em tringulo e D, E, F pontos sobre as retas BC, CA, AB, respectivamente. Ento os circuncrculos de AEF, BFD e CDE tm um ponto em comum. Esse ponto tambm chamado de ponto de Miquel.

Demonstrao: Seja M a segunda interseo dos circuncrculos de AEF e BFD. Ento CDM = BFM = AEM = 180 CEM .
EUREKA! N31, 2010

30

Sociedade Brasileira de Matemtica

Exerccios: 01. Demonstre o teorema de Miquel para quadrilteros utilizando o teorema de Miquel para tringulos. 02. Seja ABCDE um pentgono convexo e F, G, H, I, J as intersees dos prolongamentos de EA, AB, AB, BC, CD, DE e DE, EA respectivamente. Prove que as segundas intersees dos circuncrculos de ABF, BCG, BCG, CDH, DEI, DEI, EAJ, e EAJ, ABF pertencem a uma mesma circunferncia. 03. Considere um quadriltero completo. Seja M o seu ponto de Miquel. Prove que: (a) os circuncentros dos quatro tringulos determinados pelo quadriltero e M esto sobre uma mesma circunferncia. (b) as projees ortogonais de M sobre as quatro retas do quadriltero pertencem a uma mesma reta r; alm disso, M o nico ponto do plano com essa propriedade. (c) os ortocentros dos quatro tringulos pertencem a uma mesma reta s. (d) as retas r e s so paralelas, e a distncia de M e r metade da distncia de M a s. 04. Seja ABCD um quadriltero convexo e X e Y as intersees dos lados opostos AD e BC e AB e CD, respectivamente. Prove que os pontos mdios de AC, BD e XY so colineares. Observao: a reta que passa pelos trs pontos a reta de Gauss do quadriltero completo. 2. Conjugados isogonais A ideia de conjugado fazer uma associao entre objetos. Objetos conjugados supostamente tm propriedades semelhantes. Isso bastante comum em equaes: se um nmero raiz, ento o conjugado tambm raiz. Em geometria, tambm existe a ideia de conjugado. De fato, dado um tringulo, cada ponto tem um conjugado isogonal e um conjugado isotmico. Aqui, trataremos somente de conjugados isogonais. Definio 2.1. Dado um tringulo ABC, o conjugado isogonal em relao a ABC de um ponto T do plano de ABC obtido refletindo as retas TA, TB e TC em relao s bissetrizes internas de ABC que passam por A, B, e C, respectivamente. As retas resultados so concorrentes no isogonal T 1 de T. A seguir, as linhas pontilhadas so as bissetrizes, e as cevianas cinzas so as reflexes das cevianas pretas.

EUREKA! N31, 2010

31

Sociedade Brasileira de Matemtica

O fato de que as retas isogonais so concorrentes extremamente importante, tanto que ser enunciado novamente. Teorema fundamental dos conjugados isogonais. Dados um tringulo e trs retas que passam pelos respectivos vrtices e concorrem em um ponto P, as retas isogonais a elas, obtidas atravs da reflexo em relao bissetriz interna correspondente, so concorrentes no conjugado isogonal P 1 de P. Demonstrao Por que as cevianas cinzas so concorrentes? Isso decorre de duas aplicaes do teorema de Ceva trigonomtrico: primeiro com as cevianas concorrentes em T e depois, com as cevianas concorrentes em T 1 , que formam os mesmos ngulos que as outras cevianas, porm no sentido contrrio. Na verdade, pode ocorrer de as trs cevianas serem paralelas. Isso ocorre se, e somente se, T est sobre o circuncrculo de ABC; nesse caso, pensamos projetivamente, ou seja, o conjugado isogonal um ponto do infinito. 2.1 para que servem isogonais? O que mais til em conjugados isogonais simplesmente que as cevianas so reflexes umas das outras em relao s bissetrizes, e isso costuma levar a algumas igualdades entre ngulos um pouco mais difceis de obter ou mesmo de se imaginar com contas.
EUREKA! N31, 2010

32

Sociedade Brasileira de Matemtica

Exemplos 2.1 No tringulo ABC, P e Q so pontos no interior de ABC tais que CBP = PBQ = QBA = ABC / 3 e BCP = PCQ = QCA = ACB / 3. Sejam D e E as projees ortogonais de P sobre AB e AC, respectivamente. Prove que AQ perpendicular a DE. Resoluo Seja = PAD. Ento APD = 90 e, como ADP e AEP so retos, o quadriltero ADPE inscritvel. Logo AED = APD = 90 .

Olhando a figura, note que basta provarmos que QAC = . A que entram os conjugados isogonais. Como PBC = QBA e BCP = QCA, os pares de retas BP;BQ e CP;CQ so simtricos entre si em relao s bissetrizes de ABC e ACB, respectivamente. Ou seja, P e Q so conjugados isogonais e, portanto, PAB e QAC tambm so iguais. Logo QAC = e o ngulo entre as retas AQ e DE
180 ( 90 ) = 90. Note que para provar o resultado na conta, bastaria repetir a demonstrao do teorema fundamental dos conjugados isogonais. Mas o mais interessante que, sabendo da existncia dos conjugados isogonais, natural pensar nessa soluo. Em contraste, fazer a conta sem pensar em conjugados isogonais no parece ser to natural. Ento d para pensar que os conjugados isogonais nos economizaram no s fazer a conta, mas mostraram onde fazer as contas relevantes.
EUREKA! N31, 2010

33

Sociedade Brasileira de Matemtica

2.2. Conjugados isogonais dos pontos notveis Voc j deve estar familiarizado com os pontos notveis do tringulo: o baricentro (encontro das medianas), o incentro (encontro das bissetrizes internas), o ortocentro (encontro das alturas) e o circuncentro (encontro das mediatrizes). Quais so os conjugados isogonais desses pontos? Vamos aproveitar e conhecer mais um ponto notvel (mas no to conhecido). Vamos fazer isso em ordem de dificuldade. Incentro As reflexes coincidem com as prprias bissetrizes. Logo o conjugado isogonal do incentro, que o encontro das bissetrizes internas, ele mesmo. O mesmo vale para os ex-incentros (encontros de duas bissetrizes externas e uma bissetriz interna e centros dos ex-incrculos, que so tangentes externamente aos lados ou seus prolongamentos). Pense sobre o assunto! Ortocentro e circuncentro A figura a seguir deve convenc-lo de que o ortocentro e o circuncentro so conjugados isogonais.

Baricentro Os isogonais das medianas so as simedianas (SImtrico + MEDIANA). O ponto de encontro das simedianas o ponto de Lemoine, tambm conhecido como ponto simediano. O ponto de Lemoine costumeiramente denotado por K. Primeiro, vamos aprender a tra-las de modo mais prtico.

EUREKA! N31, 2010

34

Sociedade Brasileira de Matemtica

Lema: Seja D a interseo das retas tangentes ao circuncrculo do tringulo ABC por B e C. Ento a reta AD contm a simediana que passa por A. Demonstrao

Construa o paralelogramo ABEC. Ento AD contm a mediana AM. Afirmamos que D e E so conjugados isogonais. De fato, BCE = B e o ngulo entre AC e CD, pela tangncia, igual a B. Assim, as retas CD e CE so conjugadas isogonais. Analogamente, BD e BE tambm so, e o resultado segue do teorema fundamental dos conjugados isogonais. Exerccios 05. Sejam P e Q pontos no interior do ngulo BAC tais que BP = CP,BQ = CQ e ABP + AQC = 180. Prove que BAP = CAQ. 06. As retas obtidas atravs das reflexes da diagonal BD do quadriltero ABCD em relao s bissetrizes de B e D passam pelo ponto mdio de AC. Prove que as reflexes da diagonal AC do quadriltero ABCD em relao s bissetrizes de
EUREKA! N31, 2010

35

Sociedade Brasileira de Matemtica

A e C passam pelo ponto mdio de BD.

07. (Prova de Seleo EUA, 2008) Seja ABC um tringulo e G o seu baricentro. O ponto P varia sobre o segmento BC. Os pontos Q e R pertencem aos lados AC e AB respectivamente, e so tais que PQ paralelo a AB e PR paralelo a AC. Prove que, ao variar P sobre BC, o circuncrculo de AQR passa por um ponto fixado X tal que BAG = CAX . 08. (IMO 2004, Problema 5) Num quadriltero convexo ABCD a diagonal BD no bissetriz do ngulo ABC nem do ngulo CDA. Um ponto P no interior de ABCD satisfaz PBC = DBA e PDC = BDA. Prove que os vrtices do quadriltero ABCD pertencem a uma mesma circunferncia se, e somente se, AP = CP. 3. Tringulo Pedal Definio 3.1. Seja P um ponto no plano do tringulo ABC e D, E e F as projees de P sobre as retas BC, CA e AB. O tringulo DEF o tringulo pedal de P em relao ao tringulo ABC. O que tringulos pedais tm de especial? Primeiro, aparecem muitos ngulos retos, o que propicia o aparecimento de quadrilteros inscritveis. Segundo, eles normalmente minimizam reas. Teorema do mnimo. Dados dois tringulos T e ABC, considere todos os tringulos DEF semelhantes a T, todos na mesma ordem, com D sobre o lado BC, E sobre o lado CA e F sobre o lado AB. Dentre todos esses tringulos, o de menor rea o tringulo pedal de algum ponto P. Demonstrao No provaremos aqui a existncia de um tringulo de rea mnima (caso voc esteja curioso, estude topologia e depois volte!). Seja DEF o tringulo de rea mnima. Seja M o ponto de Miquel de ABC e DEF, e sejam P, Q e R as projees de M sobre os lados.

EUREKA! N31, 2010

36

Sociedade Brasileira de Matemtica

Note que o quadriltero CPMQ inscritvel (pois MPC e MQC so retos), de modo que DME = PMQ = 180 C. Portanto, PMD = QME : imagine o ngulo DME girando em torno de M para coincidir com PMQ;MD vira MP e ME vira MQ. Analogamente, RMF = QME. Portanto os tringulos PMD, QME e RMF so semelhantes e induzem uma rotohomotetia (voc se lembra o que isso?) que leva DEF a PQR. A razo de MP homotetia 1, de modo que a rea de PQR menor ou igual rea de DEF. MD Como DEF tem rea mnima, os tringulos devem ser congruentes e deste modo MP = MD, ou seja, P = D. Analogamente, Q = E e R = F, de modo que DEF o tringulo pedal de P. Exemplo 3.1. (Prova de seleo EUA, 2008) Sejam P, Q, R pontos sobre os lados BC, CA, AB de um tringulo acutngulo ABC tais que PQR equiltero e tem rea mnima entre todos tais tringulos equilteros. Prove que a reta perpendicular a QR que passa por A, a reta perpendicular a RP que passa por B e a reta perpendicular a PQ que passa por C tm um ponto comum. Resoluo Pelo teorema do mnimo, PQR tringulo pedal de algum ponto T.

EUREKA! N31, 2010

37

Sociedade Brasileira de Matemtica

Como os ngulos TQA e TRA so ambos retos, o quadriltero AQTR inscritvel, e o seu circuncentro o ponto mdio A de AT . Assim, a reta perpendicular a QR e que passa por A, que contm a altura relativa a QR, isogonal a AT, que contm o circuncentro, em relao ao tringulo AQR. Como os ngulos BAC e QAR so iguais, a perpendicular e AT so isogonais em relao ao tringulo ABC tambm. O anlogo para as perpendiculares a PR por B e a PQ por C. Como AT, BT e CT so concorrentes em T, seus isogonais so concorrentes no conjugado isogonal de T. A ttulo de curiosidade, o ponto T o primeiro ponto isodinmico. Os dois pontos isodinmicos (adivinhe o nome do outro ponto!) so os pontos de interseo dos crculos de Apolnio de A, B e C (que passam pelos vrtices, o p da bissetriz interna e tm centro sobre o lado oposto). Os seus conjugados isogonais so os pontos de Fermat. O primeiro ponto de Fermat o ponto cuja soma das distncias aos vrtices mnima (supondo que os ngulos internos do tringulo so todos menores do que 120 ). Veja [5] para aprender isso e muito, muito mais. 3.1. Voltando s simedianas Uma aplicao interessante da ideia de tringulo pedal est relacionada s simedianas. Uma outra maneira de construir as simedianas a seguinte: Lema. Construa quadrados ABBc Ac , BCCa Ba e CAAb Cb externamente sobre os lados do tringulo ABC. Prolongue Ac Bc , Ba Ca e Cb Ab para obter o tringulo A BC. Ento as retas AA,BB e CC concorrem no ponto simediano K de ABC.
EUREKA! N31, 2010

38

Sociedade Brasileira de Matemtica

Demonstrao Por simplicidade, sejam BC = a, CA = b e AB = c e L o encontro de AA,BB,CC. Queremos provar que L = K. Primeiro, como os pares de retas AB; A B,BC;BC e CA;C A so paralelos, os tringulos ABC e A BC so semelhantes. Seja k a razo de semelhana. Sejam ka ,kb e kc as distncias de L a BC, CA e AB, respectivamente. Das semelhanas entre LAB; LAB, LBC; LBC e LCA; LCA, todas de razo k,
ka k k k k k k = b = c =k a = b = c = ka + a kb + b kc + c a b c 1 k

Isto quer dizer que as distncias de L a cada um dos lados proporcional aos seus comprimentos. Alm disso, considerando uma semelhana prova-se que um ponto X pertence a, digamos, AL se, e somente, as distncias de X aos lados AB e AC so proporcionais a seus comprimentos. Basta provar que a simediana por A tem a mesma propriedade. Para isso, considere a construo anterior, sendo D o mesmo ponto definido anteriormente.

EUREKA! N31, 2010

39

Sociedade Brasileira de Matemtica

Sendo x e y as distncias de D a AB e AC, respectivamente, considerando que o ngulo entre AB e BD FBD = ACB = C e o ngulo entre AC e CD DCE = ABC = B (no se preocupe com tringulos obtusngulos; nesse caso, troque o ngulo obtuso por seu suplementar), nos tringulos retngulos BDF e CDE,x = BDsenC e y = DCsenB. Observando ainda que, sendo DB e DC x senC AB tangentes, DB = DC, temos = = . Logo D pertence a AL e, y senB AC consequentemente, K tambm. Da mesma forma provamos que K pertence a BL e CL, de modo que L = K. Assim como no teorema das bissetrizes, as simedianas dividem os lados opostos em razes interessantes.
BN AB Lema. Seja ABC um tringulo e NA uma simediana. Ento = . CN AC Demonstrao
2

EUREKA! N31, 2010

40

Sociedade Brasileira de Matemtica

J provamos anteriormente que as distncias do ponto simediano K aos lados so proporcionais a seus comprimentos. Ento existe t real tal que ka = ta, kb = tb e kc = tc c tc 2 tb b tb 2 tc. Assim, as reas de KAB, KAC e KBC so = , = e 2 2 2 2 ta a ta 2 = , respectivamente. Logo 2 2 2 BN rea ABN rea KBN rea ABN rea KBN rea KAB c 2 AB = = = = = 2 = CN rea ACN rea KCN rea ACN rea KCN rea KAC b AC Lema. Sejam d a ,d b e d c as distncias de um ponto P aos lados BC, CA, e AB do BN c d c tringulo ABC. Se AP corta BC em N , ento = . CN b db Demonstrao Fica a cargo do leitor. 3.2 A desigualdade de Erds-Mordell Um dos principais teoremas sobre tringulos pedais a desigualdade de ErdsMordell: Desigualdade de Erds-Mordell. Seja P um ponto no plano do tringulo ABC e d a ,d b ,d c as distncias de P s retas BC, CA, AB respectivamente. Ento
PA + PB + PC 2 ( d a + d b + d c )

Demonstrao Seja PA = d. Multiplique a figura original por d e construa tringulos semelhantes aos tringulos obtidos por PA e as projees de P sobre AB e AC:

EUREKA! N31, 2010

41

Sociedade Brasileira de Matemtica

Note que GDE = 90 e HDF = 90 , de modo que G, D e H so colineares. Alm disso, EGD e FHD so ambos retos, de modo que as retas EG e FH so paralelas. A distncia entre essas duas retas GH = AB d b + AC d c , que menor ou igual a EF = d BC. Lembrando que d = PA, temos AB AC AB db + AC d c PA BC PA db + d c . Analogamente, BC BC AB BC PB da + dc AC AC AC BC PC da + db AB AB 1 Somando as trs desigualdades e lembrando que t + 2 para todo t real positivo, t
AB AC AB BC AC BC PA + PB + PC + + + da + db + dc 2 ( d a + d b + d c ) AC AB BC AB BC AC

Exemplo 3.2. (IMO 1991, Problema 4) Sejam ABC um tringulo e M um ponto interior. Mostre que pelo menos um dos ngulos MAB,MBC e MCA menor ou igual a 30. Resoluo: Sejam P, Q e R as projees de M sobre BC, CA, e AB, respectivamente.

Pela desigualdade de Erds-Mordell, MA + MB + MC 2 ( MP + MQ + MR ) . Se


EUREKA! N31, 2010

42

Sociedade Brasileira de Matemtica

todas

1 MR MP MQ so maiores do que , ento , , 2 MA MB MC MA < 2 MR,MB < 2 MP e MC < 2 MQ, e MA + MB + MC < 2 ( MP + MQ + MR ) ,

as

razes

contradio. Ento uma das razes, digamos,


MR = senMAB, de modo que MAB 30. MA

MR 1 , menor ou igual a . Todavia, MA 2

Exerccios 09. Dado um tringulo com permetro L, seja P o permetro de um tringulo pedal. Prove que L 2 P. Quando ocorre a igualdade? 10. Seja P um ponto interior ao tringulo ABC e T o seu tringulo pedal. Prove que R 2 OP 2 vezes a rea de ABC. a rea de T igual a 4R2 11. Seja G o baricentro do tringulo ABC e D, E, F as projees ortogonais de G sobre os lados BC, CA e AB, respectivamente. Prove que 4 rea DEF 1 < 27 rea ABC 4 12. Seja P um ponto qualquer no plano do tringulo ABC. As projees de P sobre BC, CA, AB so D, E e F respectivamente. (a) Prove que as perpendiculares a EF, FD, DE por A, B, C respectivamente tm um ponto Pem comum. (b) Sejam Q e Q as segundas intersees de AP e AP com o circuncrculo de ABC, respectivamente. Prove que as retas QQ e BC so paralelas. 13. Os pontos X, Y e Z esto sobre BC, CA, e AB, respectivamente, e so tais que XYZ e ABC so semelhantes, nessa ordem. Prove que o circuncentro de XYZ equidistante aos ortocentros de ABC e XYZ. 14. (Ibero 2008, Problema 5) Seja ABC um tringulo e X, Y, Z pontos interiores dos lados BC, AC, AB, respectivamente. Sejam A, B, C os circuncentros dos tringulos AZY, BXZ, CYX, respectivamente. Demonstre que
EUREKA! N31, 2010

43

Sociedade Brasileira de Matemtica

4 e que a igualdade ocorre se, e somente se, as retas AA,BB,CC tm um ponto em comum. Observao: Para um tringulo qualquer RST, denotamos a sua rea por (RST).

( A BC )

( ABC )

15. (OPM, 2001)

uuur uuu r (a) Na figura acima, considere pontos B1 e C1 sobre as semirretas AB e AC, respectivamente.

(i) Mostre que a soma das reas dos paralelogramos com lados AB1 e AM e com lados AC1 e AM igual rea do paralelogramo tal que um de seus lados B1C1 e o outro paralelo e igual a AM. (ii) Tomando AB1 = AC e AC1 = AB, conclua que AB v + AC w BC x (b) Prove a Desigualdade de Erds-Mordell: 2 ( u + v + w ) x + y + z Referncias Bibliogrficas
[1] Uma tima fonte de problemas o Mathlinks: http://www.mathlinks.ro/ (em ingls). [2] Para quem gosta de Geometria, o Forum Geometricorum um prato cheio! Tudo sobre quadrilteros completos foi retirado do artigo Steiners Theorems on the Complete Quadrilateral, de Jean Pierre Ehrmann, Volume 4 (2004), pp 35-52. [3] Para quem quer saber mais sobre o teorema de Erds-Mordell, na Eureka! 18. [4] O livro Modern Geometry of the Triangle, de William Gallatly, contm muita informao interessante, incluindo a maior parte dos fatos sobre simedianas e o ponto simediano. [5] Mais conjugados isogonais? Isso e muito mais no livro Geometry of Conics (o livro do bode Veja a Capa!), de A. V. Akoplyan e A. A. Zaslavsky.
EUREKA! N31, 2010

44

Sociedade Brasileira de Matemtica

SRIE HARMNICA DE NMEROS PRIMOS


Lenimar Nunes de Andrade UFPB Joo Pessoa, PB
1. Srie harmnica H sculos que se sabe que a soma dos recprocos dos nmeros inteiros positivos 1 1 1 1 H n = 1 + + + + ... + 2 3 4 n pode ultrapassar o valor de qualquer constante positiva pr-estabelecida, bastando, para isso, somar determinada quantidade de parcelas, considerando o valor de n suficientemente grande. Quando consideramos uma infinidade de parcelas desse tipo, temos uma srie infinita conhecida pelo nome de srie harmnica, e, como a soma H n vai aumentando medida que n aumenta e ultrapassa qualquer valor prestabelecido, temos que se trata de uma srie divergente. Existem pelo menos 20 demonstraes diferentes desse fato (veja, por exemplo, a referncia [3]) e algumas demonstraes simples podem ser encontradas em [1] ou [2].* Sabe-se que o crescimento das somas parciais H n da srie harmnica bastante lento. Se somarmos 1000 termos da srie, obtemos 7, 4855 como resultado. Se somarmos 1000000 de termos, obtemos 14,3927. Para a soma ultrapassar 100, estima-se que seja necessrio somar-se aproximadamente 1,5 1043 parcelas um nmero de parcelas to grande que nem os computadores mais modernos de hoje em dia, trabalhando ininterruptamente ao longo de vrios milnios, conseguiriam efetuar todos os clculos. Se for escolhido um determinado algarismo, e retirados da srie harmnica todos os termos que contenham esse algarismo, ento surpreendentemente, obtem-se uma srie infinita na qual a soma dos n primeiros termos sempre inferior a 80, no

Nota do editor: Uma demonstrao particularmente simples deste fato a seguinte:

1 1 1 1 1 1 1 1 1 + + + + + + + ... + k 1 + ... + k 2 3 4 5 6 7 8 2 2 +1 1 1 1 1 1 1 1 1 1 1 1 1 1 1 1 + + + + + + + + ... + k + k + ... + k = 1 + + + + ... + 2 4 4 8 8 8 8 2 2 2 2 2 2 2 H 2k = 1 +

=1+

k , que pode ultrapassar o valor de qualquer constante positiva pr-estabelecida. 2

EUREKA! N31, 2010

45

Sociedade Brasileira de Matemtica

importando qual seja o valor de n**. Obtemos, portanto, o que chamamos de srie convergente. 2. Srie dos recprocos de nmeros primos A sequncia de nmeros primos 2, 3, 5, 7, 11, 13,... infinita. Apesar de estarem bem prximos uns dos outros, para valores pequenos, medida que aumentamos o valor de n, torna-se difcil encontrar nmeros primos maiores do que n, e eles vo ficando cada vez mais distantes uns dos outros em mdia. Se retirarmos da srie hermnica todos os termos cujos denominadores no sejam primos, obtemos ainda assim uma srie infinita: 1 1 1 1 1 1 1 + + + + + + + ... 2 3 5 7 11 13 17 vamos denominar a srie assim obtida de srie harmnica de nmeros primos. O principal objetivo deste artigo mostrar que essa srie harmnica de nmeros primos divergente, ou seja, a soma dos seus n primeiros termos pode ultrapassar qualquer valor pr-estabelecido. Esse fato foi observado pela primeira vez por Leonhard Euler (17071783). O crescimento do valor das somas dos n primeiros termos da srie harmnica de nmeros primos exatamente lento. Muito mais lento do que o da srie harmnica. Com os recursos computacionais atuais, impossvel realizar a tarefa de somar uma certa quantidade de termos dessa srie e obtermos o resultado igual ou superior a 5,0. 3. Demonstrao da divergncia Inicialmente, vamos mostrar que dado um n inteiro positivo temos que
k k2 1 1 + < 1 + + 2 n n n para todo inteiro positivo k tal que k n. Em particular, fazendo k = n, obtemos que n n2 1 (1) 1 + < 1 + + 2 = 3. n n n Para isso, vamos usar o mtodo da induo matemtica. Para k = 1, a desigualdade 1 1 1 reduz-se a 1 + < 1 + + 2 o que verdadeiro. Suponho a desigualdade vlida n n n para k, vamos verificar que vale para k + 1 tambm:
n k

**

Veja o problema proposto no. 141, na pgina 60.

EUREKA! N31, 2010

46

Sociedade Brasileira de Matemtica

1 1 + n

k +1

2 k k2 1 1 1 k k 1 = 1 + 1 + < 1 + + 2 1 + = 1 + + 2 + n n n n n n n n k 2 2

k 2 n ( k + 1) k k2 k + 1 ( k + 1) k + 1 ( k + 1) + 2 + 3 =1+ + + <1+ + , 2 3 n n n n n n n2 14 4244 3


<0

onde utilizamos que k < n ( k + 1) porque k n. Desse modo, a desigualdade fica demonstrada. Escolhido n, um inteiro positivo qualquer, consideremos r o inteiro tal que 2r n < 2r +1 , ou seja, r o expoente da maior potncia de 2 que no ultrapassa n. Sejam p1 = 2 , p2 = 3 , p3 = 5 ,..., ps os primos positivos menores ou iguais a n. Se m for um inteiro tal que 1 m n, ento o Teorema Fundamental da Aritmtica nos garante que m pode ser escrito de modo nico como um produto de potncias dos primos pk com expoentes inteiros no negativos t1 ,t2 ,...,ts :
2

t 2 t3 m = p1t1 p2 p3 ...psts

Note que nenhum dos expoentes tk pode ser maior do que r , pois, se assim fosse,
tk teramos m pk 2tk 2r +1 > n, o que seria um absurdo. Assim, para todo k, temos 0 tk r.

Temos, ento, a seguinte desigualdade:


1+ 1 1 1 1 1 1 1 + + + ... + < 1 + + + ... + r n 2 4 2 3 4 2 1 1 1 1 + + + ... + r 3 3 9

1 1 1 1 1 1 + ... + r ... 1 + + 2 + ... + r 1 + + 5 ps ps ps 5 25

(2)

Sua demonstrao consiste na observao de que cada parcela 1 m que aparece do lado esquerdo da desigualdade pode ser escrita de modo nico na forma
1 1 1 1 = t1 t2 ... ts m p1 p2 ps

e que cada frao da forma

1 ocorre uma nica vez como uma das parcelas do tk pk

1 1 1 fator 1 + + 2 + ... + r que aparece no segundo membro da desigualdade. pk pk pk


EUREKA! N31, 2010

47

Sociedade Brasileira de Matemtica

1 2 e q 1 q calculando a seguinte soma de uma progresso geomtrica de razo 1 q , obtemos: 1 1 r +1 ( q 1) + 1 = 1 + 1 1 1 1 1 q q 1 + + 2 + ... + r = < = = 1 1 q q q q 1 q 1 q 1 1 1 q q Usando agora a desigualdade (1) obtida no incio desta seo, obtemos

Supondo q 2 , temos 2q q 2 , ou seja, 2 ( q 1) q que equivale a

1 1 < 3 q 1 . 1 + < 3 que equivalente a 1 + q 1 q 1 Aplicando-se logaritmos, obtemos: q1 1 log 1 + < log 3 1 , q 1 ou seja, 1 log 3 2 log 3 log 1 + < q q 1 q 1 de onde finalmente obtemos 1 1 1 1 2log3 log 1 + + 2 + ... + r < log 1 + < q q q q q 1
1

q 1

(3)

Aplicando-se logaritmos aos dois membros da desigualdade (2), e usando-se propriedade log ( ab ) = log a + log b, obtemos:
1 1 1 1 1 1 1 log 1 + + + + ... + < log 1 + + + ... + r + n 2 2 3 4 2 4 1 1 1 1 1 1 log 1 + + + ... + r + ... + log 1 + + 2 + ... + r (4) 3 ps ps ps 3 9 Usando-se vrias vezes o resultado (3) na desigualdade (4), obtemos: 1 2 log 3 2 log 3 2 log 3 2 log 3 1 1 1 log 1 + + + + ... + < + + + ... + n 2 3 5 ps 2 3 4

1 1 1 1 = 2 log 3 + + + ... + . ps 2 3 5

EUREKA! N31, 2010

48

Sociedade Brasileira de Matemtica

Se existisse uma constante L tal que

1 1 1 + + ... + < L para todo inteiro positivo 2 3 ps

1 1 1 1 s, ento teramos log 1 + + + + ... + < 2 log 3 L = log ( 32 L ) (5) n 2 3 4 1 1 1 1 (6) o que implicaria 1 + + + + ... + < 32 L , para todo n natural, 2 3 4 n o que seria um absurdo, pois a srie harmnica no limitada e, para algum n, a 1 1 1 soma 1 + + + ... + ultrapassaria a constante 32 L . n 2 3

4. Algumas somas parciais Sabe-se que quando maior o valor de n, mais prximo de ln ( ln ( n ) ) + B1 ser a soma de todos os recprocos de primo inferiores a n. A constante B1 conhecida como constante de Mertens e tem valor igual a 0,2614972128.... Com a ajuda de um computador, se somarmos os recprocos dos nmeros primos inferiores a 1000, obtemos 2,1990 como resultado. Observe que esse valor prximo de ln ( ln (1000 ) ) + B1 2 ,1941. Somando-se todos os recprocos de nmreos primos inferiores a 107 (dez milhes), obtemos uma soma total igual a 3,041449. Calculando-se
ln ln (107 ) + B1 , obtemos 3,041440 que muito prximo da soma obtida.

A aproximao ln ( ln ( n ) ) + B1 S para a soma dos recprocos de primos inferiores a n pode ser escrita na forma
n ee
S B1

.
5 B1 4 ,7385

Por exemplo, para chegar a 5,0, a soma necessitaria de n = ee = ee 4, 2 1049 parcelas, o que um nmero realmente assustador: nem o computador mais rpido de hoje em dia, trabalhando incessantemente por milnios a fio, conseguiria somar tal quantidade de termos. Referncias Bibliogrficas
[1] G. vila, As sries infinitas, RPM 30, 1996. [2] G. Garbi, A surprendente srie harmnica, RPM 42, 2000. [3] S.J. Kifowit, T. A. Stamps, The harmonic series diverges again and again, The AMATYC Review, Vol. 27, No. 2, 2006. [4] D. O. Shkiyarsky, N. N. Chentsov, I. M. Yaglom, Selected problemas and theorems in elementary Mathematics Arithmetic and Algebra, Mir Publishers, Moscow, 1979.
EUREKA! N31, 2010

49

Sociedade Brasileira de Matemtica

COMO QUE FAZ?


PROBLEMA PROPOSTO POR WILSON CARLOS DA SILVA RAMOS (BELM PA)

3 85 2 2 = 2 4 xy + 4 ( x + y ) + ( x + y) 3 1) Resolva o sistema 2 x + 1 = 13 x+ y 3

Soluo: Da segunda equao, obtemos y =

1 x. Substituindo esse valor de 13 2x 3


2

y na primeira equao, obtemos

4x 13 85 + 4 y 2 + 3 2 x = . Fazendo 13 3 3 2x 3 13 1 13 1 1 u 13 u = 2 x, temos x = u , e y = x = + , u u 2 6 3 2 3
2

26 13 85 26 68 26 4 2 2 + + u + 3u 2 = , ou seja, 4u 2 u + = 0. 3u 3 3 3 9 3u u 2 u 1 1 26 140 Fazendo w = u + , temos u 2 + 2 = w2 2, donde 4 w2 w = 0, e logo u u 3 9 10 7 1 10 1 7 w= ou w = . Assim, u + = ou u + = . A primeira dessas 3 6 u 3 u 6 1 equaes tem solues u = 3 e u = , que nos do as duas solues 3 1 13 1 1 1 13 1 2 reais x = u = , y = x = e x = u = 2, y = x = 1 . A 2 3 u 3 2 3 u 3 segunda no tem solues reais, mas tem as solues complexas conjugadas 7 i 95 , que nos do as solues complexas correspondentes u= 12 12 59 + i 95 73 i 95 59 i 95 73 + i 95 , e x , y = , ( x, y ) = ( ) . 24 24 24 24

donde

EUREKA! N31, 2010

50

Sociedade Brasileira de Matemtica

PROBLEMA PROPOSTO POR MARCLIO MIRANDA DE CARVALHO (TERESINA PI) (teste de seleo da Romnia para IMO de 1978)

2) Para cada n natural, resolva a equao: sen x sen 2 x...sen nx + cos x cos 2 x...cos nx = 1
SOLUO DE RENAN HENRIQUE FINDER (JOINVILLE SC)

Se n 2, usando a desigualdade triangular e o fato de que max { sen , cos } 1 , temos 1 = sen x sen 2x...sen nx + cos x cos2 x...cos nx sen x sen 2x...sen nx + cos x cos2 x...cos nx
sen x sen 2 x + cos x cos 2 x

Pela desigualdade de Cauchy-Schwarz, 1 sen 2 x + cos 2 x sen 2 2 x + cos 2 2 x = 1. Para que ocorra a igualdade, devemos ter sen x cos 2 x = cos x sen 2 x, logo sen x = sen ( 2 x x ) = 0, e portanto x = m , m senx = 0. Ento cos x cos 2 x...cos nx = 1. Se m par, isso sempre ocorre. Se m mpar,
2 . Portanto, cos x = 1, cos 2 x = 1, cos3 x = 1,..., logo cos x...cos nx = ( 1) n

n Se n 2 e par (ou seja, se n da forma 4j 1 ou 4j), as solues so 2 x = m , m . n Se n 2 e mpar (ou seja, se n da forma 4j + 1 ou 4j + 2), as 2 solues so x = 2m , m .

Se n = 1 o problema equivale a
x = 2m ou x = 2m +

, m . 2

2 2 2 sen x + cos x sen = sen x + = 2 2 2 4 4

EUREKA! N31, 2010

51

Sociedade Brasileira de Matemtica

!
123.
2 2 3

SOLUES DE PROBLEMAS PROPOSTOS


Publicamos aqui algumas das respostas enviadas por nossos leitores.

Determine
2

todas

as
2

funes

f : * *

tais

que

2 f (m + n ) = f (m) f (n) + f (m) f (n) , para quaisquer m, n * distintos. Obs: * = {1, 2,3,...} o conjunto dos inteiros positivos.
SOLUO DE TALO DOWELL LIRA MELO (TERESINA PI) Primeiro note que se f uma funo constante ento f soluo. Agora suponhamos que exista uma funo no constante que seja soluo. Assim existem naturais a e b com f ( a ) < f ( b ) . Da temos que 2 f ( a ) = f ( a ) + f ( a ) < f ( a ) f ( b ) + f ( a ) f ( b ) < 2 f ( b ) .
3 3 3 2 2 3

Como 2 f ( a2 + b2 ) = f ( a) f ( b) + f ( a) f ( b) , segue que 2 f ( a) < 2 f ( a2 + b2 ) < 2 f ( b) .


3 2 2 3 3 3 3 3 3

Se dividirmos por 2 encontramos que f ( a ) < f ( a 2 + b2 ) < f ( b )

f ( a ) < f (a 2 + b 2 ) < f ( b ) . Isto nos diz que entre quaisquer dois valores distintos de f podemos encontrar um outro valor de f mas isto no pode ocorer sempre uma vez que f assume valores em *. Esta contradio mostra que tal funo no existe. Assim as funes constantes so as nicas solues.
124.

Considere a seqncia
2 an 1an 3 + an 2

(an )n 1

definida por

a1 = a2 = a3 = a4 = 1

an =

an 4

, n 5.

Prove que an um inteiro positivo, para todo inteiro positivo n. SOLUO DE ZOROASTRO AZAMBUJA NETO (RIO DE JANEIRO RJ) Vamos provar por induo que, para todo n 5, valem as seguintes afirmaes:
2 ak 4 ak 1ak 3 + ak 2 , k ,5 k n (e logo ak , k n ). 3 2 ak 3 ak 2 + ak 1ak 4 , k ,5 k n . 2 3 ak 2 ak 1ak 4 + ak 3 , k ,5 k n .

mdc ( ak 1 , ak ) = 1, k ,1 k 1 < k n.

mdc ( ak 2 , ak ) = 1, k ,1 k 2 < k n.

EUREKA! N31, 2010

52

Sociedade Brasileira de Matemtica

Note que, como a1 = a2 = a3 = a4 = 1, temos a5 = 2 e a6 = 3 , e portanto os itens acima se verificam para todo n 6 . Vamos agora verific-los para k= n + 1: queremos mostrar que 2 ( an1an3 + an22 ) an2 + an21an4 . an 1an 3 + an 2 2 2 an 3 an an 2 + an = a + a = n2 1 n 1 an 4 an 4 Como o lado direito inteiro e mdc ( an 3 , an 4 ) = 1, isso equivale a mostrar que
2 2 3 2 an 3 ( an 1an 3 + an 2 ) an 2 + an 1an 4 , o que segue de an 3 an 2 + an 1an 4 .

mdc ( ak 3 , ak ) = 1, k ,1 k 3 < k n.

Tambm queremos mostrar que


an 2 a
3 n 1

+a a

2 n n 3

=a

3 n 1

3 2 2 2 an a a + an 1an 4 + ( an 1an 3 + an 2 ) an 3 2 + n 1 n 3 = a . n 3 2 an 4 an 4 2 2

Como o lado direito inteiro e mdc ( an 2 , an 4 ) = 1, isso equivale a mostrar que


3 2 2 an 2 an 1an 4 + ( an 1an 3 + an 2 ) an 3 , o que equivale a 2

3 2 2 3 2 2 3 2 3 an 2 an 1an 4 + an 1an 3 = an 1 ( an 1an 4 + an 3 ) , e isso segue de a n 2 a n 1 a n 4 + a n 3 .

E tambm queremos mostrar que


2 n n 3 3 n2

2 2 3 2 an 1an 3 + an ( an 1an 3 + an 2 2 ) an 3 + an 2 an 4 3 2 an 1 a a + a = . an 3 + an 2 = an 4 an 4 Como o lado direito inteiro e mdc ( an 1 , an 4 ) = 1, isso equivale a mostrar que

2 2 3 an 1 ( an 1an 3 + an 2 ) an 3 + an 2 an 4 , o que equivale a

2 2 3 2 2 2 an 1 an an 1 an 2 an 3 + an 2 an 4 = an 2 ( an 3 + an 2 an 4 ) , que segue de 3 + an 2 an 4 ,

2 que por sua vez segue da igualdade an 1an 5 = an 3 + an 2 an 4 , que vem da definio de an 1. 2 2 Finalmente, de an +1an 3 = an an 2 + an 1 , segue que mdc ( an +1 , an ) mdc ( an , an 1 ) = 1,
2 mdc ( an +1 , an 1 ) mdc ( an 1 , an an 2 ) = 1 e mdc ( an +1 , an 2 ) mdc ( an 2 , an 1 ) = 1.

125. Considere dois naturais (a0 , a1 , a2 ,..., amn ), ai {0,1}.

m2

n 2,

as

seqncias

As seqncias de tipo m satisfazem as condies:


EUREKA! N31, 2010

53

Sociedade Brasileira de Matemtica

ak ak + m = 0, para todo k; Se ak ak +1 = 1 ento m divide k As seqncias de tipo n so definidas analogamente. Prove que existem tantas seqncias do tipo m quanto do tipo n. SOLUO DE JOS DE ALMEIDA PANTERA (RIO DE JANEIRO RJ) Considere a matriz m n, B = ( bij )1i m dada por bij = am (i 1) + j ,1 i m,1 j n.
1 j n

Temos que ( a0 ,..., amn ) uma sequncia de tipo m se e somente se B uma matriz m n cujas entradas pertencem a {0, 1} sem dois termos vizinhos iguais a 1 numa mesma linha ou numa mesma coluna, e tal que b1m = 1 a0 = 0. Considere agora a funo f que leva uma matriz B m n na matriz f ( B ) n m dada por

( f ( B ))

i, j

= B( m +1 j ),( n +1i ) . Temos que f uma bijeo entre as matrizes

m n cujas entradas pertencem a {0, 1} sem dois termos vizinhos iguais a 1 numa mesma linha ou numa mesma coluna e as matrizes n m cujas entradas pertencem a {0, 1} sem dois termos vizinhos iguais a 1 numa mesma linha ou numa mesma coluna, tal que ( f ( B ) )1,m = B1,n . Isso mostra que o nmero de seqncias do tipo m igual ao nmero de seqncias do tipo n.
126. As circunferncias i , 0 i 5, so tangentes a uma circunferncia nos pontos Ai . Alm disso, i tangente a i +1 para 0 i 5 e 5 tangente a 0 . Prove que A0 A3 , A1 A4 , A2 A5 so concorrentes.

SOLUO DE MATHEUS SECCO TORRES DA SILVA (RIO DE JANEIRO RJ) Sejam Oi os centros de i e ri os raios de i . Alm disso, seja r o raio de , que tem centro O. Vamos supor inicialmente que as circunferncias i so exteriores a . Usando Ceva trigonomtrico no A2 A4 A0 , devemos provar que
sen ( < A1 A4 A2 ) sen ( < A3 A0 A4 ) sen ( < A5 A2 A0 ) =1 sen ( < A1 A4 A0 ) sen ( < A3 A0 A2 ) sen ( < A5 A2 A4 )

A OA A OA AOA sen < 1 2 sen < 3 4 sen < 5 0 2 2 2 =1 A OA A OA A OA sen < 2 3 sen < 4 5 sen < 0 1 2 2 2
EUREKA! N31, 2010

54

Sociedade Brasileira de Matemtica

A0 A5 A4 A3 A1

A2

Denotamos < AOA i i +1 = i ,0 i 5, ndices mdulo 6. Mas no OOi Oi +1 , temos:

( ri + ri +1 )

= ( r + ri ) + ( r + ri +1 ) 2 ( r + ri )( r + ri +1 ) cos i
2 2

cos i = 1 1 2 sen 2 sen 2

2ri ri +1 ( r + ri )( r + ri +1 )

i 2ri ri +1 =1 2 ( r + ri )( r + ri +1 )

ri ri +1 i = , donde 2 ( r + ri )( r + ri +1 )

sen 2 1 sen 2 3 sen 2 5 2 2 2 =1 sen 2 2 sen 2 4 sen 2 6 2 2 2 sen 1 sen 3 sen 5 2 2 2 = 1, pois sen i > 0, i 5, 2 sen 2 sen 4 sen 6 2 2 2 donde obtemos o desejado. Isso conclui a prova!. Obs.: Supusemos que as circunferncias i estavam no exterior de 1 , mas se
EUREKA! N31, 2010

55

Sociedade Brasileira de Matemtica

fossem interiores, obteramos sen2

i ri ri +1 = , o que tambm nos daria o 2 ( r ri )( r ri +1 )

desejado, com um argumento anlogo.


129. Um coelho est numa rua infinita dividida em quadrados numerados pelos

inteiros, e comea no quadrado 0. Se num dado momento ele est no quadrado k, ele escolhe, com probabilidade probabilidade
1 , pular para o quadrado k + 2 ou, tambm com 2

o coelho pisar no quadrado m.

1 , pular para o quadrado k 1. Ele continua esse processo 2 indefinidamante. Dado m , determine a probabilidade de, em algum momento,

SOLUO DE ASDRUBAL PAFNCIO SANTOS (BOTUCATU SP) Denotemos por am a probabilidade de, em algum momento, o coelho pisar no quadrado m. Seja n 0 um inteiro. Aps o primeiro passo do coelho, ele pode estar no quadrado 1 1 1, com probabilidade , ou no quadrado 2, com probabilidade , ficando a 2 2 distncias respectivamente n + 1 e n 2 do quadrado n. Assim, para todo 1 1 n 0, an = an 2 + an +1 ( *) . 2 2
2 n 5 n 1 tende a 0 exponencialmente rpido, com probabilidade total n 2 k =0 k temos que, para um certo n0 , e para todo n n0 , pelo menos 40% dos n primeiros passos do coelho so para frente, o que faz com que, aps n passos, ele 2n 3n n esteja num quadrado de nmero maior ou igual a 2 = , para todo 5 5 5 n n0 . Em particular, a probabilidade de o coelho pisar no quadrado m tende a 0 quando m tende a .

Como

De (*), temos am + 3 = 2am + 2 am , m 0, donde existem constantes A, B, C com


1+ 5 1 5 am = A + B + C 2 2 , m 0, pois o polinmio caracterstico da
EUREKA! N31, 2010
m m

56

Sociedade Brasileira de Matemtica

1 + 5 1 5 x recorrncia acima x3 2 x 2 + 1 = ( x 1) x . 2 2 Como ( am ) limitada, devemos ter B = 0. Como a0 = 1, devemos ter A + C = 1.

De (*), tambm temos a m 3 = 2a m a m +1 , m 1, ou seja, fazendo bk = a k , temos bk + 3 = 2bk +1 bk , k 0, donde, como


1 + 5 1 5 ,C com x3 2 x + 1 = ( x 1) x , existem A, B x 2 2
1 + 5 1 5 bk = A+ B +C , k 0. Como bk no s limitado como 2 2 = 0, e como b = a = 1, tende a 0 quando k tende a + , devemos ter A=C
= 1. devemos ter B
0 0

1+ 5 1+ 5 5 , m 0 e am = bm = 2 = 2 , m 0. 1 1 Fazendo n = 1 em (*), obtemos a1 = a1 + a2 , donde 2 2 1 5 1 1 + 5 1 3 5 1 C + C = + 1 C + C , e portanto 2 2 2 2 2 73 5 C= . 2 3 5 5 7 3 5 1 5 m + , m 0 2 2 2 Assim, temos am = . m 1 + 5 2 , m 0


130. Suponha que a, b, c e a equao x 2 ( a + b + c ) x + ( ab + ac + bc) = 0 no

1 Assim, am = 1 C + C 2

tem razes reais. Prove que a, b e c tm todos o mesmo sinal e existe um tringulo de lados
a, b e c.

EUREKA! N31, 2010

57

Sociedade Brasileira de Matemtica

SOLUO DE DANIEL EITI NISHIDA KAWAI (TAUBAT SP) Se a equao x 2 ( a + b + c ) x + ( ab + ac + bc ) = 0 no tem razes reais, temos
< 0 ( a + b + c ) 4 1 ( ab + ac + bc ) < 0
2

a 2 + b2 + c 2 + 2ab + 2 ac + 2bc 4ab 4ac 4bc < 0 a 2 + b2 + c 2 2ab 2 ac 2bc < 0


2 2 2

a 2 + b2 + c 2 + 2ab 2 ac 2bc 4ab < 0 ( a + b c ) 4ab < 0 ( a + b c ) < 4ab 0 ( a + b c ) < 4ab 4ab > 0 ab > 0 a e b tm o mesmo sinal. De maneira anloga, b e c tm o mesmo sinal. Assim, a, b e c tm todos o mesmo sinal, e logo ab = a b , ac = a c e bc = b c .

Assim, a 2 + b2 + c 2 2 ab 2 ac 2 bc < 0, donde


a 2 + b2 + c 2 + 2 ab 2 ac 2 bc < 4 ab

a + b + c + 2 a b 2 a c 2 b c <4 a b
2 2 2

( a + b c) < 2 a b
2

c < a +2 a

( c ) <(
2

a + c c.

) 2 a b < a + b c b + b ( c ) <( a ) + 2 a b +( b ) b ) c < a + b.


2 2 2 2

De maneira anloga,
b < a +

a <

b +

c . Assim, existe um tringulo de lados

a, b e

Agradecemos o envio de solues e a colaborao de:


Carlos Alberto da Silva Victor (Nilpolis RJ) Rodrigo dos Anjos Azevedo (Trs Rios RJ) Vinicius dos Nascimento S. Mano (Petrpolis RJ) Marcelo Robeiro de Souza (Rio de Janeiro RJ) Jheimyson Rego Barnab (Imperatriz MA) Flvio Antonio Alves (Amparo SP) talo Dowell Lira Melo (Teresina PI) Curro Fernndez Lpez (Lugo, Espanha) Miguel Amengual Covas (Mallorca, Espanha) Bruno Salgueiro Fanego (Galicia, Espanha) Prob. 123, 130 Prob. 130 Prob. 130 Prob. 130 Prob. 130 Prob. 130 Prob. 130 Prob. 42 Prob. 110 Prob. 116, 117, 118

Continuamos aguardando solues para os problemas 131 e 132.


EUREKA! N31, 2010

58

Sociedade Brasileira de Matemtica

PROBLEMAS PROPOSTOS *
Convidamos o leitor a enviar solues dos problemas propostos e sugestes de novos problemas para prximos nmeros.

133) Considere um ngono regular inscrito em um crculo unitrio, fixe um vrtice i e denote por dj a distncia entre este vrtice i e o vrtice j. Prove que

(5 d ) = F
j i j =0 2 j

n 1

2 n

onde F1 = 0, F1 = 1 e Fn = Fn 1 , Fn 2 se n 2.
3

134) Considere a operao entre dois vetores do ( x, y , z ) ( u, v, w ) = ( xu + yw + zv, xw + zu + yv, xv + yu + zw )


k

definida por:

Prove que, para todo k 1, se ( x, y , z ) = ( 0,0,0 ) ento x = y = z = 0. Obs.: Para qualquer

( x, y , z ) 3 , ( x, y , z ) = ( x, y , z ) k k 1 ( x, y , z ) = ( x , y , z ) ( x, y , z ) .
1

e, para todo

k > 1,

hemisfrio paralelo a ( C1 ) , de forma que existem n crculos do hemisfrio, congruentes, tangentes entre si, a ( C1 ) e a ( C2 ) . Mostre que a razo K(n) entre os raios de ( C2 ) e ( C1 ) igual a: K ( n ) =
cos 2 n . 1 + sen 2 n

135) Considere um hemisfrio cuja base um crculo ( C1 ) . Um crculo ( C2 ) do

136) Sejam R, r1 , r2 e r3 os raios dos crculos de centro O, O1 , O2 e O3 , respectivamente, conforme a figura abaixo. Prove que: R = r1r2 + r1r3 + r2 r3 .

O1 R O R2 O2 O3

137) Sendo A um conjunto de quinze pontos de 2 tal que a distncia de cada ponto origem positiva e menor do que 1 e que quaisquer dois deles nunca sejam
EUREKA! N31, 2010

59

Sociedade Brasileira de Matemtica

colineares com a origem. Mostre que existe um tringulo com dois vrtices em A e 1 um na origem cuja rea menor que . 4 138) Calcule o mximo divisor comum entre todos os nmeros da forma x y z , onde ( x, y , z ) percorre todas as solues inteiras da equao x 2 + y2 = z 2 com x y z 0. 139) Determine todos os inteiros positivos x, y, z satisfazendo x3 y3 = z 2 , onde y primo, z no divisvel por 3 e z no divisvel por y. 140) Mostre que 2903n 803n 464n + 261n divisvel por 1897, para todo n . 141) Dado a {0,1, 2,3, 4,5,6,7,8,9} , seja X um conjunto finito de inteiros positivos, tal que nenhum dos seus elementos possui o algarismo a em sua 1 representao decimal. Prove que < 80. n X n

Problema 133 e 134 proposto por Evandro Makiyama de Melo (So Paulo SP) (foram propostos originalmente na IX e na II Olimpada Iberoamericana de Matemtica Universitria, respectivamente); 135 e 136 propostos por Ramilson Medeiros Pitombeira (Rio de Janeiro RJ); 137 proposto por talo Dowell Lira Melo (Teresina PI); 138 proposto por Luiz Felipe Silva; 139 proposto por Adriano Carneiro (Caucaia CE); 140 proposto por Wilson Carlos da Silva Ramos (Belm PA).

EUREKA! N31, 2010

60

Sociedade Brasileira de Matemtica

AGENDA OLMPICA
XXXII OLIMPADA BRASILEIRA DE MATEMTICA
NVEIS 1, 2 e 3 Primeira Fase Sbado, 12 de junho de 2010 Segunda Fase Sbado, 18 de setembro de 2010 Terceira Fase Sbado, 16 de outubro de 2010 (nveis 1, 2 e 3) Domingo, 17 de outubro de 2010 (nveis 2 e 3 - segundo dia de prova). NVEL UNIVERSITRIO Primeira Fase Sbado, 18 de setembro de 2010 Segunda Fase Sbado, 16 e Domingo, 17 de outubro de 2010

ASIAN PACIFIC MATH OLYMPIAD (APMO)


06 de maro de 2010

XVI OLIMPADA DE MAIO


08 de maio de 2010

XXI OLIMPADA DE MATEMTICA DO CONE SUL


13 a 19 de junho de 2010 guas de So Pedro, SP Brasil

LI OLIMPADA INTERNACIONAL DE MATEMTICA


02 a 14 de julho de 2010 Astana, Cazaquisto

XVII OLIMPADA INTERNACIONAL DE MATEMTICA UNIVERSITRIA


24 a 30 de julho de 2010 Blagoevgrad, Bulgria

XXIV OLIMPADA IBEROAMERICANA DE MATEMTICA


17 a 27 de setembro de 2010 Paraguai

II COMPETIO IBEROAMERICANA INTERUNIVERSITRIA DE MATEMTICA


3 a 9 de outubro de 2010 Rio de Janeiro, Brasil

XIII OLIMPADA IBEROAMERICANA DE MATEMTICA UNIVERSITRIA

EUREKA! N31, 2010

61

Sociedade Brasileira de Matemtica

COORDENADORES REGIONAIS
Alberto Hassen Raad Amrico Lpez Glvez Andreia Goldani Antonio Carlos Nogueira Benedito Tadeu Vasconcelos Freire Carmen Vieira Mathias Claus Haetinger Cludio de Lima Vidal Denice Fontana Nisxota Menegais Disney Douglas Lima de Oliveira Edson Roberto Abe Edney Aparecido Santulo Jr. lio Mega Eudes Antonio da Costa Fbio Brochero Martnez Florncio Ferreira Guimares Filho Francinildo Nobre Ferreira Genildo Alves Marinho Graziela de Souza Sombrio Gilson Tumelero Ivanilde Fernandes Saad Joo Bencio de Melo Neto Joo Francisco Melo Libonati Jose de Arimatia Fernandes Jos Luiz Rosas Pinho Jos Vieira Alves Jos William Costa Krerley Oliveira Licio Hernandes Bezerra Luciano G. Monteiro de Castro Luzinalva Miranda de Amorim Marcelo Rufino de Oliveira Marcelo Mendes Newman Simes Nivaldo Costa Muniz Nivaldo de Ges Grulha Jr. Osnel Broche Cristo Uberlndio Batista Severo Raul Cintra de Negreiros Ribeiro Ronaldo Alves Garcia Rogrio da Silva Igncio Reginaldo de Lima Pereira Reinaldo Gen Ichiro Arakaki Ricardo Amorim Srgio Cludio Ramos Seme Gebara Neto Tadeu Ferreira Gomes Toms Menndez Rodrigues Valdenberg Arajo da Silva Vnia Cristina Silva Rodrigues Wagner Pereira Lopes
EUREKA! N31, 2010

(UFJF) (USP) FACOS (UFU) (UFRN) (UNIFRA) (UNIVATES) (UNESP) (UNIPAMPA) (UFAM) (Colgio Objetivo de Campinas) (UEM) (Grupo Educacional Etapa) (Univ. Federal do Tocantins) (UFMG) (UFES) (UFSJ) (Centro Educacional Leonardo Da Vinci) (UNOCHAPEC) (UTFPR) (UC. Dom Bosco) (UFPI) (Grupo Educacional Ideal) (UFPB) (UFSC) (UFPB) (Instituto Pueri Domus) (UFAL) (UFSC) (Sistema Elite de Ensino) (UFBA) (Grupo Educacional Ideal) (Colgio Farias Brito, Pr-vestibular) (Cursinho CLQ Objetivo) (UFMA) (USP So Carlos) (UFLA) (UFPB)) (Colgio Anglo) (UFGO) (Col. Aplic. da UFPE) (Escola Tcnica Federal de Roraima) (UNIFESP) (Centro Educacional Logos) (IM-UFRGS) (UFMG) (UEBA) (U. Federal de Rondnia) (U. Federal de Sergipe) (U. Metodista de SP) (CEFET GO)

Juiz de Fora MG Ribeiro Preto SP Osrio RS Uberlndia MG Natal RN Santa Mara RS Lajeado RS S.J. do Rio Preto SP Bag RS Manaus AM Campinas SP Maring PR So Paulo SP Arraias TO Belo Horizonte MG Vitria ES So Joo del Rei MG Taguatingua DF Chapec SC Pato Branco PR Campo Grande MS Teresina PI Belm PA Campina Grande PB Florianpolis SC Campina Grande PB Santo Andr SP Macei AL Florianpolis SC Rio de Janeiro RJ Salvador BA Belm PA Fortaleza CE Piracicaba SP So Luis MA So Carlos SP Lavras MG Joo Pessoa PB Atibaia SP Goinia GO Recife PE Boa Vista RR SJ dos Campos SP Nova Iguau RJ Porto Alegre RS Belo Horizonte MG Juazeiro BA Porto Velho RO So Cristovo SE S.B. do Campo SP Jata GO

62

CONTEDO
XXXI OLIMPADA BRASILEIRA DE MATEMTICA Problemas e solues da Primeira Fase XXXI OLIMPADA BRASILEIRA DE MATEMTICA Problemas e solues da Segunda Fase XXXI OLIMPADA BRASILEIRA DE MATEMTICA Problemas e solues da Terceira Fase XXXI OLIMPADA BRASILEIRA DE MATEMTICA Problemas e solues da Primeira Fase Nvel Universitrio XXXI OLIMPADA BRASILEIRA DE MATEMTICA Problemas e solues da Segunda Fase Nvel Universitrio XXXI OLIMPADA BRASILEIRA DE MATEMTICA Premiados AGENDA OLMPICA COORDENADORES REGIONAIS

2 14 34 59 65 75 81 82

Sociedade Brasileira de Matemtica

XXXI OLIMPADA BRASILEIRA DE MATEMTICA


Problemas e solues da Primeira Fase
PROBLEMAS NVEL 1

1 1 5 de um nmero , quanto vale desse nmero? 8 5 8 1 1 8 B) C) 1 D) A) 8 5 5

1. Se

E) 2

2. Na figura, C um ponto do segmento BD tal que ACDE um retngulo e ABCE um paralelogramo de rea 22 cm2. Qual a rea de ABDE, em cm2? A) 28 B) 33 C) 36 D) 42 E) 44

A E C D B

3. Numa festa, o nmero de pessoas que danam igual a 25% do nmero de pessoas que no danam. Qual a porcentagem do total de pessoas na festa que no danam? A) 50% B) 60% C) 75% D) 80% E) 84% 4. De quantas maneiras dois casais podem sentar-se em quatro cadeiras em fila se marido e mulher devem sentar-se em cadeiras vizinhas? A) 2 B) 4 C) 8 D) 12 E) 24 5. Eliana tem 27 cubos iguais em tamanho, mas 4 so brancos e os demais, pretos. Com esses 27 cubos, ela monta um cubo maior. No mximo, quantas faces inteiramente pretas ela poder obter? A) 1 B) 2 C) 3 D) 4 E) 5 6. A figura abaixo o mapa de um bairro: os pontos A, B, C e D so as casas e os segmentos so as ruas. De quantas casas possvel fazer um caminho que passa exatamente uma vez por cada uma das ruas? permitido passar mais de uma vez por uma mesma casa.

EUREKA! N32, 2010

Sociedade Brasileira de Matemtica

A) 0

B) 1

C) 2

D) 3

E) 4

7. Se a = 240, b = 320 e c = 710, ento: A) c < b < a B) a < c < b C) b < a < c

D) b < c < a

E) c < a < b

8. Esmeralda lanou um dado dez vezes e obteve 57 como soma de todos os pontos obtidos nesses lanamentos. No mnimo, quantas vezes saram 6 pontos? A) 5 B) 6 C) 7 D) 8 E) 9 9. Usando palitos de fsforos, podemos construir um hexgono regular, formado por seis tringulos equilteros unitrios, como mostra a figura. Juntando mais palitos a esse hexgono, queremos obter outro hexgono regular com o qudruplo da rea, tambm formado por tringulos equilteros unitrios. Quantos palitos devero ser acrescentados? A) 12 B) 24 C) 30 D) 36 E) 48 10. Cinco cartas iguais tm um lado branco e um lado preto. Elas se encontram em fila com a face branca para cima. Um movimento consiste em escolher um nico par de cartas vizinhas e vir-las. No mnimo, quantos movimentos so necessrios para que as cartas fiquem como na figura ao lado? B) 3 C) 4 A) 2 obter a configurao acima.

D) 5

E) No possvel

EUREKA! N32, 2010

Sociedade Brasileira de Matemtica

11. Uma barra de chocolate dividida entre Nelly, Penha e Snia. Sabendo que 2 1 da barra, Penha ganha e Snia ganha 70 gramas, o peso da Nelly ganha 5 4 barra, em gramas, : A) 160 B) 200 C) 240 D) 280 E) 400 12. Numa fila para compra de ingressos para um jogo da seleo brasileira, havia 49 pessoas: 25 corintianos, 14 flamenguistas e 10 gremistas. Sabendo que cada pessoa da fila torce para um nico time, dois torcedores do mesmo time no esto em posies consecutivas, podemos concluir que: A) tal fila no existe. B) algum dos torcedores das extremidades da fila gremista. C) algum dos torcedores das extremidades da fila flamenguista. D) algum flamenguista vizinho de um gremista. E) algum gremista vizinho de dois corintianos. 13. Na figura, P um ponto da reta CD. A regio cinza comum ao retngulo ABCD e ao tringulo ADP. Se AB = 5 cm, AD = 8 cm e a rea da regio cinza 3 da rea do retngulo, quanto vale a distncia 4 PC? A) 1 cm B) 2 cm C) 3 cm D) 4 cm E) 5 cm
A B

14. Numa pesquisa sobre o grau de escolaridade, obtiveram-se os resultados expressos no grfico abaixo: Que frao do total de entrevistados representa o total de pessoas que terminaram pelo menos o Ensino Fundamental?

EUREKA! N32, 2010

Sociedade Brasileira de Matemtica

A)

1 17

B)

3 13

C)

5 16

D)

11 13

E)

16 17

15. Um nmero natural A de trs algarismos detona um nmero natural B de trs algarismos se cada algarismo de A maior do que o algarismo correspondente de B. Por exemplo, 876 detona 345; porm, 651 no detona 542 pois 1 < 2. Quantos nmeros de trs algarismos detonam 314? A) 120 B) 240 C) 360 D) 480 E) 600 16. O relgio de parede indica inicialmente meio-dia. Os ponteiros das horas e dos minutos iro formar um ngulo de 90 graus pela primeira vez: A) entre 12h e 12h10min. B) entre 12h10min e 12h15min. C) entre 12h15min e 12h20min. D) entre 12h20min e 12h25min. E) aps as 12h25min.

12 9 6 3

17. Eduardo escreveu todos os nmeros de 1 a 2009 numa folha de papel. Com os amigos, combinou o seguinte: cada um deles poderia apagar quantos nmeros quisesse e escrever, no fim da lista, o algarismo das unidades da soma dos nmeros apagados. Por exemplo, se algum apagasse os nmeros 28, 3, 6, deveria escrever no fim da lista o nmero 7, pois 28 + 3 + 6 = 37. Aps algum tempo, sobraram somente dois nmeros. Se um deles era 2000, qual dos nmeros a seguir poderia ser o outro? A) 0 B) 1 C) 3 D) 5 E) 6 18. Uma folha de caderno de Carlos um retngulo com dois lados (bordas) amarelos de 24 cm e dois lados (bordas) vermelhos de 36 cm. Carlos pinta cada ponto do retngulo na mesma cor do lado mais prximo desse ponto. Qual a rea da regio pintada de amarelo? A) 144 cm2 B) 288 cm2 C) 364 cm2 D) 442 cm2 E) 524 cm2 19. O professor Piraldo aplicou uma prova de 6 questes para 18 estudantes. Cada questo vale 0 ou 1 ponto; no h pontuaes parciais. Aps a prova, Piraldo elaborou uma tabela como a seguinte para organizar as notas, em que cada linha representa um estudante e cada coluna representa uma questo.

EUREKA! N32, 2010

Sociedade Brasileira de Matemtica

Questes Estudantes Arnaldo Bernaldo Cernaldo

0 1 0

1 1 1

1 1 1

1 0 1

1 0 1

0 1 0

Piraldo constatou que cada estudante acertou exatamente 4 questes e que cada questo teve a mesma quantidade m de acertos. Qual o valor de m? A) 8 B) 9 C) 10 D) 12 E) 14

20. Alguns cubos foram empilhados formando um bloco. As figuras ao lado representam a vista da esquerda e da frente desse bloco. Olhando o bloco de cima, qual das figuras a seguir no pode ser vista?
esquerda esquerda esquerda

vista da esquerda

vista da frente

esquerda

A)

B) frente

C) frente

D) frente

E) frente

esquerda

frente

PROBLEMAS NVEL 2

1. Veja o Problema No. 1 do Nvel 1. 2. Veja o Problema No. 9 do Nvel 1. 3. Veja o problema No. 4 do Nvel 1. 1 1 4. Se : = 4, o valor de x+6 x+5 A)

1 5

B)

1 4

C)

2 3

D)

4 5

E) 1

5. Veja o Problema No. 6 do Nvel 1. 6. Os inteiros positivos m e n satisfazem 15m = 20n. Ento possvel afirmar, com certeza, que mn mltiplo de:
EUREKA! N32, 2010

Sociedade Brasileira de Matemtica

A) 5 B) 10 C) 12 7. Veja o problema No. 15 do Nvel 1. 8. Veja o Problema No. 11 do Nvel 1. 9. Veja o Problema No. 8 do Nvel 1.

D) 15

E) 20

10. Na figura abaixo, = 18 e AB = AC = AD = AE. O valor do ngulo :

D
D) 20o

E
E) 30o

A) 18o

B) 36o

C) 15o

11. Veja o Problema No. 10 do Nvel 1. 12. Na figura abaixo, ABCDE um pentgono regular, CDFG um quadrado e DFH um tringulo equiltero. O valor do ngulo :
H D E C A B F G

A) 30o

B) 36o

C) 39o

D) 45o

E) 60o

EUREKA! N32, 2010

Sociedade Brasileira de Matemtica

13. Veja o problema No. 12 do Nvel 1. 14. Veja o Problema No. 13 do Nvel 1. 15. A famosa Conjectura de Goldbach diz que todo nmero inteiro par maior que 2 pode ser escrito como a soma de dois nmeros primos. Por exemplo, 18 pode ser representado por 5 + 13 ou, ainda, por 7 + 11. Considerando todas as possveis representaes de 126, qual a maior diferena entre os dois primos que a formam? B) 100 C) 92 D) 88 E) 80 A) 112 16. Na figura ao lado, E o ponto mdio de AB, F o ponto mdio de AC e BR = RS = SC. Se a rea do tringulo ABC 252, qual a rea do pentgono AERSF? A) 168 B) 189 C) 200 D) 210 E) 220

17. Quantos pares ordenados (x, y) de nmeros reais satisfazem a equao

( x y ) + ( x y 2)
2 2

= 0?

A) 0

B) 1

C) 2

D) 3

E) infinitos

18. Veja o Problema No. 19 do Nvel 1. 19. Entre os inteiros positivos n + 4018, n = 1, 2,..., 20092 , quantos so quadrados perfeitos? A) 1945 B) 1946 C) 1947 D) 1948 E) 1949 20. Para cada nmero natural n, seja S n a soma dos dez primeiros mltiplos positivos de n. Por exemplo, S 2 = 2 + 4 + 6 + 8 + 10 + 12 + 14 + 16 + 18 + 20. Quanto S1 + S 2 + S 3 + + S10 ? A) 2925 B) 3025 C) 3125 D) 3225 E) 3325

EUREKA! N32, 2010

Sociedade Brasileira de Matemtica

21. Em uma folha quadriculada em que cada quadrado tem lado 2cm, so desenhados dois crculos como na figura ao lado. A distncia mnima entre os dois crculos mede: A) 3cm B) 10 cm

( D) ( E) (
C)

) 10 2 ) cm 10 3) cm
10 + 3 cm

22. Quantos nmeros naturais de 1 a 100, inclusive, podem ser escritos na forma de potncia a b , com a, b e a, b > 1? A) 10 B) 12 C) 14 D) 16 E) 18 23. Veja o Problema No. 18 do Nvel 1. 24. Os inteiros 0 < x < y < z < w < t so tais que w = z(x + y) e t = w(y + z). Sendo w = 9, ento t igual a A) 45 B) 54 C) 63 D) 72 E) 81 25. Veja o Problema No. 20 do Nvel 1.
PROBLEMAS NVEL 3

1. Veja o problema No. 15 do Nvel 1. 2. Veja o problema No. 6 do Nvel 2. 3. Se x2 = x + 3 ento x3 igual a: A) x2 + 3 B) x + 4 C) 2x + 2 D) 4x + 3 E) x2 2

4. Na figura, o quadrado ABCD foi obtido a partir de uma rotao no sentido horrio do quadrado ABCD de 25 graus em torno do ponto mdio de AB. Qual o ngulo agudo, em graus, entre as retas AC e BD?

EUREKA! N32, 2010

Sociedade Brasileira de Matemtica

C C

A A
A) 5 B) 25 C) 45

B B
D) 65 E) 85

5. Um dos cinco nmeros a seguir divisor da soma dos outros quatro. Qual esse nmero? A) 20 B) 24 C) 28 D) 38 E) 42 6. Sempre que Agilulfo volta para casa depois da escola com uma advertncia, se sua me est em casa, ela o coloca de castigo. Sabendo-se que ontem tarde Agilulfo no foi colocado de castigo, qual das seguintes afirmaes certamente verdadeira? A) Agilulfo recebeu advertncia ontem. B) Agilulfo no recebeu advertncia ontem. C) Ontem tarde a sua me estava em casa. D) Ontem tarde a sua me no estava em casa. E) Nenhuma das afirmaes acima certamente verdadeira. 7. Qual o menor valor de n > 1 para o qual possvel colocar n peas sobre um tabuleiro n n de modo que no haja duas peas sobre a mesma linha, mesma coluna ou mesma diagonal? As figuras a seguir mostram pares de peas na mesma linha, na mesma coluna e na mesma diagonal em diversos tabuleiros.

A) 3
EUREKA! N32, 2010

B) 4

C) 5

D) 6

E) 7

10

Sociedade Brasileira de Matemtica

8. Na figura a seguir, ABCD um quadrado de lado 4, K pertence ao lado AD, L pertence ao lado AB, M pertence ao lado BC e KLM um tringulo retngulo issceles, sendo L o ngulo reto. Ento a rea do quadriltero CDKM igual a A) 6 B) 8 C) 10 D) 12 E) 14 9. Veja o Problema No. 6 do Nvel 1. 10. Veja o Problema No. 16 do Nvel 1.

A K

M D C

11. Considere o nmero inteiro positivo n tal que o nmero de divisores positivos do dobro de n igual ao dobro do nmero de divisores positivos de n. Podemos concluir que n A) um nmero primo B) um nmero par C) um nmero mpar D) um quadrado perfeito E) potncia inteira de 2 12. Esmeralda tem cinco livros sobre herldica em uma estante. No final de semana, ela limpou a estante e, ao recolocar os livros, colocou dois deles no lugar onde estavam antes e os demais em lugares diferentes de onde estavam. De quantas maneiras ela pode ter feito isso? A) 20 B) 25 C) 30 D) 34 E) 45 13. Veja o Problema No. 19 do Nvel 1. 14. Seja f : uma funo tal que f(0) = 0, f(1) = 1, f(2) = 2 e f(x + 12) = f(x + 21) = f(x) para todo x . Ento f(2009) : A) 0 B) 1 C) 2 D) 3 E) 2009

EUREKA! N32, 2010

11

Sociedade Brasileira de Matemtica

15. Na figura, CD = BC, BAD = 72 , AB o dimetro e O o centro do semicrculo. Determine a medida do ngulo DEC. A) 36o B) 42o C) 54o D) 63o E) 18o

C E

16. Sabe-se que 2x2 12xy + ky2 0 para todos x, y reais. O menor valor real de k A) 9 B) 16 C) 18 D) 27 E) 36 17. Veja o problema No. 15 do Nvel 2. 18. Um subconjunto de {1,2,3,,20} superpar quando quaisquer dois de seus elementos tm produto par. A maior quantidade de elementos de um subconjunto superpar : A) 3 B) 4 C) 6 D) 7 E) 11 19. Veja o problema No. 20 do Nvel 2. 20. Os crculos C1 e C2, de raios 3 e 4, respectivamente, so tangentes externamente em T. As tangentes externas comuns tocam C1 em P e Q e C2 em R e S. A tangente interna comum em T corta as tangentes externas nos pontos M e N, como mostra a figura. A razo entre as reas dos quadrilteros MNPQ e MNRS : A)

P C1 Q

N T M
3 2

C2 S
E)

1 7

B)

9 16

C)

3 4

D)

13 15

EUREKA! N32, 2010

12

Sociedade Brasileira de Matemtica

21. Dois carros deixam simultaneamente as cidades A e B indo de uma cidade em direo outra, com velocidades constantes, e em sentidos opostos. As duas cidades so ligadas por uma estrada reta. Quando o carro mais rpido chega ao ponto mdio M de AB, a distncia entre os dois carros de 96 km. Quando o carro mais lento chega ao ponto M, os carros esto a 160 km um do outro. Qual a distncia, em km, entre as duas cidades? A) 320 B) 420 C) 480 D) 520 E) 560 22. Seja N = 8 8 , em que aparecem 2009 nmeros 8. Agilulfo ficou de castigo: ele deve escrever a soma dos dgitos de N, obtendo um nmero M; em seguida, deve calcular a soma dos dgitos de M; e deve repetir o procedimento at obter um nmero de um nico dgito. Vamos ajudar Agilulfo: esse dgito A) 1 B) 2 C) 3 D) 7 E) 8 23. Veja o Problema No. 20 do Nvel 1. 24. Veja o Problema No. 18 do Nvel 1. 25. Os lados de um tringulo formam uma progresso aritmtica de razo t. Ento a distncia entre o incentro e o baricentro deste tringulo : A) t B)
8

GABARITO
NVEL 1 (6. ou 7. Anos) 1) C 6) C 2) B 7) A 3) D 8) C 4) C 9) C 5) D 10) B NVEL 2 (8. ou 9. Anos) 1) C 6) C 2) C 7) B 3) C 8) B 4) D 9) C 5) C 10) A NVEL 3 (Ensino Mdio) 1) B 6) E 2) C 7) B 3) D 8) B 4) D 9) C 5) D 10) E

t 2

C)

t 3

D)

2t 3

E) faltam dados

11) B 12) E 13) E 14) E 15) B 11) B 12) C 13) E 14) E 15) B 11) C 12) A 13) D 14) C 15) C

16) C 17) D 18) B 19) D 20) C 16) A 17) C 18) D 19) B 20) B 16) C 17) B 18) E 19) B 20) E 21) E 22) B 23) B 24) A 25) C 21) C 22) A 23) C 24) B 25) C

EUREKA! N32, 2010

13

Sociedade Brasileira de Matemtica

XXXI OLIMPADA BRASILEIRA DE MATEMTICA


Problemas e solues da Segunda Fase
PROBLEMAS NVEL 1 PARTE A (Cada problema vale 5 pontos) 01. A figura ao lado mostra

castelos de cartas de 1, 2 e 3 andares. Para montar esses castelos, foram usadas 2, 7 e 15 cartas, respectivamente. Quantas cartas sero necessrias para montar um castelo de 5 andares?
02. Numa classe do 6 ano, de cada 11 estudantes, 4 so meninas. Se h 15 meninos

a mais que meninas, quantos alunos h na classe?


03. Num curso com durao de cinco dias, a frequncia dos alunos foi registrada na tabela abaixo: Dia de aula Quantidade de alunos presentes 1 dia 271 2 dia 296 3 dia 325 4 dia 380 5 dia 168

Cada aluno faltou exatamente dois dias. No dia de menor frequncia, de quantos por cento foi o total de faltas?
04. Mariazinha deseja cobrir o tampo de uma mesa retangular de 88 cm por 95 cm colando quadrados de cartolina de lado 10 cm, a partir de um canto, como mostrado na figura. Ela cola os quadrados sem buracos nem superposies, at chegar s bordas opostas. A, em vez de cortar as folhas para no ultrapassar as bordas, ela as sobrepe, formando regies retangulares com duas folhas de espessura (regio cinza) e uma pequena regio retangular com quatro folhas de espessura (regio preta). Qual a rea da regio coberta por quatro folhas?
EUREKA! N32, 2010

14

Sociedade Brasileira de Matemtica

05. O nmero 200920092009... 2009 tem 2008 algarismos. Qual a menor

quantidade de algarismos que devem ser apagados, de modo que a soma dos algarismos que restarem seja 2008?
06. Dizemos que dois ou mais nmeros, com a mesma quantidade de algarismos, so membros da mesma famlia, quando todos possuem pelo menos um algarismo comum. Por exemplo, os nmeros 72, 32, 25 e 22 pertencem mesma famlia, pois todos possuem o algarismo 2, enquanto que os nmeros 123, 245 e 568 no pertencem mesma famlia, pois no h um algarismo que aparea nesses trs nmeros. Qual a maior quantidade de membros de uma famlia, cujos elementos tm trs algarismos?

PROBLEMAS NVEL 1 PARTE B (Cada problema vale 10 pontos) PROBLEMA 1

Carlinhos tem folhas iguais na forma de tringulos retngulos de lados 6 cm, 8 cm e 10 cm. Em cada tringulo, o ngulo assinalado ope-se ao menor lado. Fazendo coincidir lados iguais desses tringulos sobre uma mesa, sem superpor as folhas, ele desenha o contorno de cada figura obtida (linha grossa), como nos exemplos ao lado. O permetro de uma figura o comprimento do seu contorno. a) Qual a diferena entre os permetros das figuras 1 e 2 do exemplo? b) Com figuras de trs tringulos, qual o maior permetro que pode ser obtido?
PROBLEMA 2

Esmeralda ia multiplicar um nmero A de trs algarismos por outro nmero B de dois algarismos, mas na hora de multiplicar inverteu a ordem dos dgitos de B e obteve um resultado 2034 unidades maior. a) Qual era o nmero A, se os dgitos de B eram consecutivos? b) Qual seria o nmero A, se os dgitos de B no fossem consecutivos?

EUREKA! N32, 2010

15

Sociedade Brasileira de Matemtica

PROBLEMA 3

Um campeonato de xadrez de 7 rodadas, com 4 jogos por rodada, tem 8 participantes, cujas pontuaes por jogo so as usuais: um ponto por vitria, meio ponto por empate e nenhum ponto por derrota. Cada par de jogadores se enfrenta exatamente uma vez. a) Ao trmino da terceira rodada, possvel que um grupo de jogadores esteja em primeiro lugar e o restante dos jogadores esteja em segundo lugar? Explique por meio de um exemplo. b) Ao trmino da terceira rodada, possvel que todos os jogadores tenham pontuaes diferentes? Explique.
PROBLEMAS NVEL 2 PARTE A (Cada problema vale 5 pontos) 01. Esmeralda tem uma garrafa com 9 litros de uma mistura que tem 50% de lcool e 50% de gua. Ela quer colocar gua na garrafa de tal forma que apenas 30% da mistura seja de lcool. Quantos litros de gua ela ir colocar? 02. Se a, b, c e d so, em alguma ordem, 1, 2, 3 e 4. Qual o maior valor possvel

de ab + bc + cd + da?
03. Dizemos que dois ou mais nmeros, com a mesma quantidade de algarismos, so membros da mesma famlia, quando todos possuem pelo menos um algarismo em comum. Por exemplo, os nmeros 32, 25 e 22 pertencem mesma famlia, enquanto que 123, 245 e 568 no pertencem mesma famlia, pois 123 e 568 no pertencem mesma famlia. Qual a maior quantidade de membros de uma famlia, cujos elementos tm trs algarismos? 04. Determine a quantidade de inteiros de dois algarismos que so divisveis pelos seus algarismos. 05. Na figura abaixo, ABCD e EFGH so quadrados de lado 48 cm. Sabendo que A

o ponto mdio de EF e G o ponto mdio de DC, determine a rea destacada em cm2.

EUREKA! N32, 2010

16

Sociedade Brasileira de Matemtica

E A F K D G C L B H

PROBLEMAS NVEL 2 PARTE B (Cada problema vale 10 pontos) PROBLEMA 1

Sejam m e n dois inteiros positivos primos entre si. O Teorema Chins dos Restos afirma que, dados inteiros i e j com 0 i < m e 0 j < n, existe exatamente um inteiro a, com 0 a < mn, tal que o resto da diviso de a por m igual a i e o resto da diviso de a por n igual a j. Por exemplo, para m = 3 e n = 7, temos que 19 o nico nmero que deixa restos 1 e 5 quando dividido por 3 e 7, respectivamente. Assim, na tabela a seguir, cada nmero de 0 a 20 aparecer exatamente uma vez.
Restos por 7
Restos por 3

0 1 2 19

Qual a soma dos nmeros das casas destacadas?


PROBLEMA 2

Observe:

(x r)(x s) = x2 (r + s)x + rs

Assim, substituindo x por r e por s, obtemos

r 2 (r + s ) r + rs = 0 s 2 (r + s ) s + rs = 0
EUREKA! N32, 2010

a ( r n + 2 (r + s ) r n +1 + rs r n ) = 0 b( s n + 2 ( r + s ) s n +1 + rs s n ) = 0

17

Sociedade Brasileira de Matemtica

Somando as duas equaes e sendo S n = a r n + b s n , verifica-se que

S n + 2 = (r + s ) S n +1 rsS n
Dados
4

S1 = ar + bs = 1 ,
4

S 2 = ar 2 + bs 2 = 2 ,
5 5

S 3 = ar 3 + bs 3 = 5

S 4 = ar + bs = 6 , determine S 5 = ar + bs .
Seja N o ponto do lado AC do tringulo ABC tal que AN = 2 NC e M o ponto do lado AB tal que MN perpendicular a AB . Sabendo que AC = 12 cm e que o baricentro G do tringulo ABC pertence ao segmento MN, determine o comprimento do segmento BG. OBS: Baricentro o ponto de interseo das medianas do tringulo.
PROBLEMA 4 PROBLEMA 3

Um campeonato de xadrez de 7 rodadas, com 4 jogos por rodada, tem 8 participantes, cujas pontuaes por jogo so as usuais: um ponto por vitria, meio ponto por empate e nenhum ponto por derrota. Cada par de jogadores se enfrenta exatamente uma vez. a) Ao trmino da terceira rodada, possvel que todos os jogadores tenham pontuaes distintas? b) Se no final do campeonato todos os jogadores tm pontuaes distintas qual o menor nmero possvel de pontos obtidos pelo primeiro colocado?
PROBLEMAS NVEL 3 PARTE A (Cada problema vale 5 pontos) 01. Veja o problema No. 1 do Nvel 2. 02. No tringulo retngulo ABC, A = 90, AB = 5cm e BC = 9cm. Se I o incentro de ABC, determine o comprimento do segmento CI. 03. Seja c a maior constante real para a qual

x2 + 3y2 c(x2 + xy + 4y2). para todos x, y reais. Determine o inteiro mais prximo de 2009c.
EUREKA! N32, 2010

18

Sociedade Brasileira de Matemtica

04. No programa de auditrio Toto Bola, o apresentador Cio Magallanes dispe de

duas caixas idnticas. Um voluntrio da platia chamado a participar da seguinte brincadeira: ele recebe dez bolas verdes e dez bolas vermelhas e as distribui nas duas caixas, sem que o apresentador veja, e de modo que em cada caixa haja pelo menos uma bola. Em seguida, o apresentador escolhe uma das caixas e retira uma bola. Se a bola for VERDE, o voluntrio ganha um carro. Se for VERMELHA, ele ganha uma banana. A mxima probabilidade que o voluntrio tem de ganhar um m , em que m e n so inteiros positivos primos entre si. Determine o carro igual a n valor de m + n.
05. Determine o maior inteiro n menor que 10000 tal que 2n + n seja divisvel por 5. PROBLEMAS NVEL 3 PARTE B (Cada problema vale 10 pontos) PROBLEMA 1

Determine a quantidade de nmeros n = a1a2a3a4a5a6, de seis algarismos distintos, que podemos formar utilizando os algarismos 1, 2, 3, 4, 5, 6, 7, 8, 9 de modo que as seguintes condies sejam satisfeitas simultaneamente: i) a1 + a6 = a2 + a5 = a3 + a4; ii) n divisvel por 9.
PROBLEMA 2

Encontre todos os inteiros a > 0 e b > 0 tais que 4 3 a = 11 + 5 b Para cada inteiro positivo n, seja An = {x R+ ; x x = n} , em que R+ o conjunto dos reais positivos e x o maior inteiro menor ou igual a x. Determine a quantidade de elementos do conjunto A1 A2 A3 ... A2009. No tringulo ABC, temos A = 120 e BC = 12 cm. A circunferncia inscrita em ABC tangencia os lados AB e AC, respectivamente, nos pontos D e E. Sejam K e L os pontos onde a reta DE intersecta a circunferncia de dimetro BC. Determine a distncia entre os pontos mdios dos segmentos BC e KL.
PROBLEMA 4 PROBLEMA 3

EUREKA! N32, 2010

19

Sociedade Brasileira de Matemtica

SOLUES NVEL 1 SEGUNDA FASE PARTE A

Problema Resposta

01 40

02 55

03 65

04 10

05 392

06 252

01. Para fazer um novo andar num castelo j construdo, precisamos de trs cartas para cada andar anterior mais duas para o topo. Assim, a partir do castelo de 3 andares, para fazer o de 4 andares, precisamos de mais 3 3 + 2 = 11 cartas, num total de 15 + 11 = 26 cartas. Portanto, para fazer o castelo de 5 andares, precisamos de 26 + 4 3 + 2 = 40 cartas.

Soluo alternativa: Para acrescentarmos um quarto andar a um castelo de 3 andares, precisamos de 3 cartas para separar a base dos demais andares e 4 pares de cartas para a base, totalizando 3 + 2.4 = 11 cartas a mais. Veja a figura a seguir:

Analogamente, para acrescentarmos um quinto andar a um castelo de 4 andares, precisamos de 4 cartas para separar a base dos demais andares e 5 pares de cartas para a base, totalizando 4 + 2.5 = 14 cartas a mais. Assim, para montar um castelo de 5 andares, precisamos de 15 + 11 + 14 = 40 cartas. Observao: De fato, o acrscimo de um n-simo andar necessita de n 1 cartas para apoiar a base anterior, e n pares de cartas para a nova base. Portanto, so acrescentadas n 1 + 2 n = 3n 1 cartas por andar.
02. Seja x a quantidade de meninas. Assim, a quantidade de meninos x + 15 e a quantidade total de alunos ser 2 x + 15 . Fazendo a proporo, temos:

x 4 = 2 x + 15 11 Resolvendo a equao, obtemos x = 20 .


EUREKA! N32, 2010

20

Sociedade Brasileira de Matemtica

03. Se cada aluno compareceu exatamente trs dias, o nmero total de alunos do

curso

271 + 296 + 325 + 380 + 168 1440 = = 480 . A menor frequncia foi de 3 3
312 = 0, 65 = 65% . 480

168 alunos, num total de 480 168 = 312 faltas. Portanto, o percentual de faltas nesse dia foi

04. Na direo da medida 88 cm, Mariazinha ir usar 9 folhas e na direo da medida 95 cm, ir usar 10 folhas. Mariazinha comea colando as folhas sem sobreposio da esquerda para a direita e de cima para baixo (como na figura) e ao chegar s bordas direita e inferior, desloca, respectivamente, 2 cm esquerda e 5 cm para cima (as regies em cinza representam as sobreposies de 2 folhas). A regio retangular preta a interseco dessas duas faixas de sobreposio, logo coberta por 4 folhas. Sua rea de 10 cm2.

05. No nmero existem 502 algarismos 2 e 502 algarismos 9. Para retirar a menor

quantidade possvel de algarismos, devemos tentar deixar a maior quantidade possvel de algarismos 2. Porm, a soma de todos os algarismos 2 1004. Ainda falta 1004 para completar a soma 2008. Como 1004 = 9 111 + 5 devemos deixar pelo menos 111 algarismos 9. Porm, impossvel deixar exatamente 111 algarismos 9. Se deixarmos 112 algarismos 9, devemos deixar 500 algarismos 2. Portanto, deve-se retirar no mnimo 2 + 390 = 392 algarismos.
06. Como todos os membros de uma famlia devem possuir pelo menos um

algarismo comum, a maior quantidade de membros de uma famlia cujos elementos tm trs algarismos igual ao nmero de elementos de qualquer conjunto formado
EUREKA! N32, 2010

21

Sociedade Brasileira de Matemtica

por todos os nmeros de trs algarismos que possuem um determinado algarismo em sua representao decimal. O algarismo das centenas no pode ser zero. Vamos contar ento todos os nmeros que tm um determinado algarismo a, no nulo, pois h mais deles. H 9 9 = 81 nmeros em que a aparece uma nica vez, como algarismo das centenas. H 8 9 = 72 nmeros em que a aparece uma nica vez, como algarismo das dezenas (lembre-se que o das centenas no pode ser 0) e h 72 nmeros em que o a aparece uma nica vez, como algarismo das unidades. H 9 nmeros com a na centena e na dezena, menos na unidade, 9 nmeros com a na centena e na unidade, menos na dezena e 8 nmeros com a na dezena e na unidade, menos na centena e um nico nmero formado inteiramente de a. A quantidade total de nmeros em que figura o algarismo no nulo a 81 + 72 + 72 + 9 + 9 + 8 + 1 = 252.

Soluo alternativa: Para simplificar o raciocnio, vamos contar quantos nmeros de trs algarismos no contm um algarismo a, no nulo, fixado. Assim, nessa situao, existem 8 escolhas para o algarismo das centenas (no pode ser 0 ou a), 9 escolhas para o algarismo das dezenas (no pode ser a), e 9 escolhas para os algarismos das unidades (no pode ser a). Logo, pelo Princpio Fundamental da Contagem, h 8.9.9 = 648 nmeros que no possuem o algarismo a. Assim, como existem 900 nmeros de 3 algarismos, h 900 648 = 252 nmeros que possuem o algarismo a ( a 0 ). Essa a maior quantidade de membros que uma famlia pode ter.
Observao: Podemos verificar que a famlia formada por todos os nmeros de trs algarismos que possuem o zero tem 900 9 9 9 = 171 membros.
SOLUES NVEL 1 SEGUNDA FASE PARTE B PROBLEMA 1

EUREKA! N32, 2010

22

Sociedade Brasileira de Matemtica

a) O permetro da primeira figura 8 + 6 + 6 + 10 + 6 = 36 e da segunda figura 10 + 8 + 6 + 8 + 8 = 40 . Portanto a diferena 40 36 = 4 . b) A figura de maior permetro obtida quando fazemos coincidir os dois menores lados de cada um dos tringulos. Isso mostrado na figura ao lado cujo permetro 10 + 10 + 10 + 8 + 6 = 44 (h outras com o mesmo permetro). Seja A o nmero de trs dgitos e B = 10 x + y o nmero de dois dgitos. Portanto, ao trocar a ordem dos dgitos de B , obtemos o nmero 10 y + x . Montando a equao segundo as condies do problema, temos:
PROBLEMA 2

A(10 x + y ) A(10 y + x) = 9 A( x y ) = 2034

Com isso,

A( x y ) = 226 = 2 113 Da, se x, y so consecutivos, A = 226 , caso contrrio A = 113 .


PROBLEMA 3

a) Sim, possvel. Por exemplo (h outros), podem existir quatro jogadores com pontuao 2 e outros quatro com pontuao 1. Fazendo A, B, C, D o primeiro grupo e E, F, G, H o segundo grupo, temos:

1 Rodada A vence E B vence F C vence G D vence H 2 Rodada A empata com B E empata com F C empata com D G empata com H 3 Rodada A empata com F B empata com E C empata com H D empata com G
EUREKA! N32, 2010

23

Sociedade Brasileira de Matemtica

b) Aps trs rodadas, um jogador pode acumular no mximo 3 pontos. Como as pontuaes so mltiplos inteiros de

1 , os possveis valores de pontuao aps a 2

terceira rodada so: 0, ,1, , 2, ,3 (7 resultados possveis) Como existem 8 jogadores e apenas 7 possibilidades, dois jogadores tero pontuaes iguais.
SOLUES NVEL 2 SEGUNDA FASE PARTE A

1 2

3 2

5 2

Problema Resposta

01 06

02 25

03 252

04 14

05 1704

01. Inicialmente temos 4,5 litros de gua e 4,5 litros de lcool. Colocados x litros de

gua, para termos 30% de lcool na mistura, basta que

x = 6.
02.

30 (9 + x) = 4,5 , ento 100

fcil ver que

ab + bc + cd + da = b ( a + c ) + b ( c + a ) = ( a + c )( b + d ) .

Suponha sem perda de generalidade que a = 1. . Com isso, {a,c} = {1, 2} ,{1,3} ou

{1,4} e conseqentemente {b,d } = {3,4} ,{2,4} ou {2,3} , respectivamente. Assim os possveis valores do produto so 21, 24 e 25 e o mximo 25.
03. O algarismo das centenas no pode ser zero. Vamos contar ento todos os

nmeros que tm um determinado algarismo x, no nulo, pois h mais deles. H 9 9 = 81 nmeros em que x aparece uma nica vez, como algarismo das centenas. H 8 9 = 72 nmeros em que x aparece uma nica vez, como algarismo das dezenas (lembre-se que o das centenas no pode ser 0) e h 72 nmeros em que o x aparece uma nica vez, como algarismo das unidades. H 9 nmeros com x na centena e na dezena, menos na unidade, 9 nmeros com x na centena e na unidade, menos na dezena e 8 nmeros com x na dezena e na unidade, menos na centena e um nico nmero formado inteiramente de x. A quantidade total de nmeros em que figura o algarismo no nulo x 81 + 72 + 72 + 9 + 9 + 8 + 1 = 252

EUREKA! N32, 2010

24

Sociedade Brasileira de Matemtica

04. Seja n = 10 A + B o nmero de dois dgitos. Se A divide n , ento

B . Se A > 5 , ento B = A , pois B no pode ser 0 e B < 10 < 2 A .

A divide

Listemos as possibilidades: Se A = 1 ento AB pode ser 11, 12, 15. Se A = 2 , ento AB pode ser 22, 24. Se A = 3 , ento AB pode ser 33, 36. Se A = 4 , ento AB pode ser 44, 48. Se A = 5 , ento AB pode ser 55. Se A = 6 , ento AB pode ser 66. Se A = 7 , ento AB pode ser 77. Se A = 8 , ento AB pode ser 88. Se A = 9 , ento AB pode ser 99. Logo, o total de nmeros 3 + 2 + 2 + 2 + 5 = 14.

EH . Por simetria, veja que KD = KF e AK = KG . Considere FK = x . Dessa forma, AK = 48 x . Usando teorema de Pitgoras no tringulo AFK , temos:

05. Sejam K a interseo dos lados AD e FG , e L a interseo dos lados AB e

242 + x 2 = ( 48 x ) .
2

Que nos d x = 18 . Agora, veja que os tringulos AFK e ALE so semelhantes. Portanto, AE EL . = FK AF Assim, EL = 32 . Para achar a rea procurada, basta subtrair a rea do quadrado EFGH das reas dos tringulos AFK e AEL . Portanto a rea ser 1704.
E L B H K D
EUREKA! N32, 2010

A F

25

Sociedade Brasileira de Matemtica

SOLUES NVEL 2 SEGUNDA FASE PARTE B PROBLEMA 1:

0 0 1 2
0 7 14

1
15 1 8

2
9 16 2

3
3 10 17

4
18 4 11

5
12 19 5

6
6 13 20

A resposta 15 + 8 + 10 + 11 + 12 + 13 = 69.
PROBLEMA 2:

S 4 = (r + s ) S 3 rsS 2 = (r + s ).5 rs.2 = 5r + 5s 2rs = 6 S 3 = (r + s ) S 2 rsS1 = (r + s ).2 rs.1 = 2r + 2 s rs = 5 Com isso, encontramos que r + s = 4 e rs = 13 . Da, S5 = (r + s)S 4 rsS3 = 24 + 65 = 41.
PROBLEMA 3:

Se BP uma mediana do tringulo ento AP = CP = 6 e PN = 2. Como G o PG 1 PN 1 baricentro do tringulo ento e = = , assim, pela recproca do GB 2 NC 2 teorema de Tales, GN paralelo a BC e B = 90o . Como o tringulo ABC retngulo ento AP = CP = BP = 6. Com isso, BG = 4 e GP = 2.

EUREKA! N32, 2010

26

Sociedade Brasileira de Matemtica

PROBLEMA 4:

a) Aps trs rodadas, um jogador pode acumular no mximo 3 pontos. Como as pontuaes so mltiplos inteiros de , os possveis valores de pontuao aps a terceira rodada so: 0,1/2, 1, 3/2, 2, 5/2, 3 Como existem 8 jogadores e apenas 7 possibilidades, dois jogadores tero pontuaes iguais. b) Se k a pontuao do primeiro colocado e todas as pontuaes so distintas, a soma das pontuaes dos oito jogadores ser no mximo:

1 3 5 7 k + k + ( k 1) + k + ( k 2 ) + k + ( k 3) + k = 8k 14 2 2 2 2 Como foram disputados exatamente 4 7 = 28 pontos, temos

8k 14 28
Logo, k 5 + 1 1 pois as pontuaes so mltiplos inteiros de . Basta mostrarmos 2 2 um exemplo onde este valor atingido. Na tabela abaixo, marcamos na interseo da linha Ai com a coluna A j o nmero de pontos que Ai ganhou na partida disputada contra Aj.

A1 A2 A3 A4 A5 A6 A7 A8 Total

A1 X A2 0 A3 0 A4 0 A5 0 A6 0 A7 A8 1

1 1

1 1 1

1 1 1 1

1 1 1 1 0

1 1 1 0

0 0 1 0 1 1

5+ 5 4+ 4 0 2+ 3 3+

x
0 0 0 0 0 1

x
0 0 0 0

X
0 0 0 0

X
1 1 1

X
0

x
0

EUREKA! N32, 2010

27

Sociedade Brasileira de Matemtica

SOLUES NVEL 3 SEGUNDA FASE PARTE A

Problema Resposta

01 0069

02 0006

03 1339

04 0033

05 9993

01. [RESPOSTA: 0069] SOLUO:

0 0 1 2
0 7 14

1
15 1 8

2
9 16 2

3
3 10 17

4
18 4 11

5
12 19 5

6
6 13 20

A resposta 15 + 8 + 10 + 11 + 12 + 13 = 69.
02. [RESPOSTA: 0006] SOLUO: Pelo teorema de Pitgoras, imediato que

AC 2 = 9 2 5 2 = 56 AC = 2 14 . Seja r o raio do crculo inscrito, como mostrado na figura abaixo.


B 5r 5r 5 r A r I r
2 14 r 2 14 r

Como os comprimentos das tangentes ao crculo inscrito partindo de cada vrtice so iguais, ficamos com a equao (5 r) + (2 14 r ) = 9, de onde obtemos r = 14 2 . Novamente pelo teorema de Pitgoras, obtemos:

EUREKA! N32, 2010

28

Sociedade Brasileira de Matemtica

CI 2 = r 2 + ( 2 14 r ) 2 = ( 14 2) 2 + ( 14 + 2) 2 = 36 CI = 6 .
03. [RESPOSTA: 1339] SOLUO: Fazendo x = ty, a equao inicial reduz-se a

t2 + 3 c(t2 + t + 4). Logo, devemos ter (c 1)t2 + ct + (4c 3) 0, para todo t real. Para isto, devemos ter c 1 < 0 e o discriminante = c2 4(c 1)(4c 3) 0. Da ltima inequao, obtemos 15c2 + 28c 12 0, cuja soluo c

2 ou 3

6 . Como c < 1, o maior valor possvel de c 2/3. Da, 2009c = 1339,333... . 5

04. [RESPOSTA: 0033] SOLUO: Seja P(a, b) a probabilidade de o voluntrio ganhar o carro no caso em

que ele tenha colocado a bolas VERDES e b bolas VERMELHAS na caixa 1. Ento, necessariamente haver (10 a) bolas VERDES e (10 b) bolas VERMELHAS na caixa 2. Segue que P ( a, b ) = a 1 1 10 a + . 2 a + b 2 20 a b

Podemos supor, sem perda de generalidade, que a + b 10, j que as caixas so idnticas. Suponha, ainda, que haja alguma bola VERMELHA na caixa 1. Vejamos o que acontece com essa probabilidade se transferirmos uma bola VERDE da caixa 2 para a caixa 1 e uma bola VERMELHA da caixa 1 para a caixa 2. Ficamos com P( a + 1, b 1) = Dessa forma, 1 a +1 1 9a . + 2 a + b 2 20 a b

P( a + 1, b 1) P( a, b) =
pois a + b 10.

1 1 1 0, 2 a + b 20 a b

EUREKA! N32, 2010

29

Sociedade Brasileira de Matemtica

Assim, o voluntrio sabe que, enquanto houver bola VERMELHA na caixa que contm menos bolas, a probabilidade pode ser aumentada, bastando, para isto, que ele troque uma das bolas VERMELHAS desta caixa com uma VERDE da outra. Por isso, para maximizarmos a probabilidade, basta considerarmos o caso em que a caixa 1 contm apenas bolas VERDES e a caixa 2 contm o restante das bolas. Teremos 1 1 10 a P( a,0) = + 2 2 20 a . 1 10 a 1 10 5 = 1 + =1 = 2 2 20 a 2 20 a 20 a Logo, a probabilidade ser mxima quando a for mnimo. Como em cada caixa deve haver pelo menos uma bola, devemos ter a = 1. Neste caso, a probabilidade : 5 14 P(1,0) = 1 = . 19 19 Segue que m = 14, n = 19 e m + n = 33.
05. [RESPOSTA: 9993] SOLUO: Vamos analisar os restos das divises de 2n e n por 5.
n 2n 2n +n 1 1 2 3 2 2 4 1 3 3 3 1 4 4 1 0 5 0 2 2 6 1 4 0 7 2 3 0 8 3 1 4 9 4 2 1 10 0 4 4 11 1 3 4 12 2 1 3 13 3 2 0 14 4 4 3 15 0 3 3 16 1 1 2 17 2 2 4 18 3 4 2 19 4 3 2 20 0 1 1

Veja que os restos das divises de 2n por 5 formam uma seqncia de perodo 4, enquanto que os restos das divises de n por 5 formam uma seqncia de perodo 5. Logo, os restos das divises de 2n + n formam uma seqncia de perodo 20, dada pela ltima linha da tabela acima. Dessa forma, tomando os nmeros de 1 a 10000 em intervalos de tamanho 20, o maior n tal que 2n + n deixa resto zero na diviso por 5 o 13o termo do ultimo intervalo, ou seja, o nmero 9980 + 13 = 9993.
SOLUES NVEL 3 SEGUNDA FASE PARTE B PROBLEMA 1:

Seja k = a1 + a6 = a2 + a5 = a3 + a4. Temos 3k = a1 + a2 + ... + a6 mltiplo de 9, uma vez que n mltiplo de 9. Da, segue que k mltiplo de 3. Mas, como os algarismos so distintos, perceba que
EUREKA! N32, 2010

30

Sociedade Brasileira de Matemtica

1 + 2 + ... + 6 a1 + a2 + ... + a6 4 + 5 + ... + 9 21 3k 39 7 k 13. Como k mltiplo de 3, temos dois casos: k = 9 e k = 12.

1o caso: k = 9. Veja que suficiente escolhermos a1, a2 e a3, pois a4 = 9 a3, a5 = 9 a2 e a6 = 9 a1. Como os dgitos devem ser distintos, devemos escolher a1, a2 e a3 de modo que haja no mximo um dgito em cada um dos conjuntos {1, 8}, {2, 7}, {3, 6} e {4, 5}. Esta escolha pode ser feita da seguinte forma:
Escolhemos trs dos quatro conjuntos: 4 maneiras; Em cada um dos trs conjuntos acima, escolhemos um dos dois dgitos: 23 = 8 maneiras; Permutamos os dgitos escolhidos: 3! = 6 maneiras. Logo, o total de nmeros, neste caso, igual a 486 = 192.

2o caso: k = 12. Neste caso, os dgitos a1, a2 e a3 devem ser escolhidos do conjunto {3, 4, 5, 7, 8, 9} de modo que haja no mximo um dgito em cada um dos conjuntos {3, 9}, {4, 8} e {5, 7}. Esta escolha pode ser feita da seguinte maneira:
Em cada um dos trs conjuntos acima, escolhemos um dos dois dgitos: 23 = 8 maneiras; Permutamos os dgitos escolhidos: 3! = 6 maneiras. Logo, o total de nmeros, neste caso, igual a 8 6 = 48. O total de nmeros , portanto, 192 + 48 = 240.
PROBLEMA 2:

Analisando a equao mdulo 5, obtemos 4 3a 1( mod 5 ) 3a 4 ( mod 5 ) . Mas os valores de 3a mod 5 so peridicos de perodo 4: 0 1 2 3 4 5 6 7 a 3 mod 5 1 3 4 2 1 3 4 2 Assim, conclumos que 3a 4 ( mod 5 ) a = 2 + 4t para t . Agora, analisando a equao mdulo
b b

3,

obtemos

11 + 5 0 ( mod 3) ( 1) 1( mod 3) o que ocorre se, e s se, b par. Portanto a


EUREKA! N32, 2010

31

Sociedade Brasileira de Matemtica

e b so ambos pares, digamos a= 2c e b = 2d para dois inteiros positivos c, d. Assim, 4 3a = 11 + 5b ( 2 3c ) 52 d = 11


2

( 2 3c 5d )( 2 3c + 5d ) = 11

2 3c 5d = 1 2 3c + 5d = 11
3c = 3 5d = 5 a = 2c = 2 b = 2d = 2

Assim, a nica soluo : ( A,B ) = ( 2 , 2 )


PROBLEMA 3:

Vamos

f ( x) = x x . Para cada k natural, se

fazer

grfico

da

funo

k x k + 1 , temos

x = k .

Logo, o grfico
6

de f formado por segmentos de reta y = kx, como mostra a figura ao lado: Assim, para um n fixo, a equao f(x) = n tem no mximo uma soluo. Portanto, a quantidade de elementos de A1 A2 A3 ... A2009 igual quantidade de inteiros n, tais que 1 n 2009, para os quais f (x) = n admite soluo, isto , os n tais que f (k) = k2 n < k(k + 1) = k2 + k, para algum k N.
PROBLEMA 4

4 2 1 1 2 3

Vamos mostrar inicialmente que BL e CK so as bissetrizes dos ngulos B e C do

ABC. Para isto, sejam K e L as interseces das bissetrizes de C e B com a circunferncia de dimetro BC , como na figura. Seja ainda I o incentro de ABC
e e as medidas de B e C , respectivamente, de modo que + = 60.

EUREKA! N32, 2010

32

Sociedade Brasileira de Matemtica

L 120 K D F /2 /2 I A

/2
E G

/2 /2

Sejam D e E as interseces de K L com os lados AB e AC do tringulo. Para mostrar que K L = KL, basta mostrar que E e D so as projees ortogonais de I aos lados AC e AB . Como BC dimetro, temos que BLC reto, assim se mostrarmos que o quadriltero IE LC cclico, provaremos que IEC reto, e analogamente para D. Denote por F e G os encontros das bissetrizes de C e B com os lados opostos. + Temos m GIC = m F IB = m AFC m F BI = + = = 30. da 2 2 2 + mesma forma, temos m GEL = m BGA m ( GL E ) = + = = 30 2 2 2

) ( ) ( ) ( ) ( ) pois m ( GL E ) = m ( BLK ) = m ( BCK ) j que ambos os ngulos subtendem o mesmo arco BK. Assim, m ( GEL ) = m ( GIC ) , provando que IELC cclico. ( ) (
Sendo

) m ( LOK ) Assim a distncia pedida LO cos


2

m K OL = 180 m LOC m K OB = 180 = 120


=
BC cos 60 = 3cm. 2

) (

ponto

mdio

de

BC,

temos

EUREKA! N32, 2010

33

Sociedade Brasileira de Matemtica

XXXI OLIMPADA BRASILEIRA DE MATEMTICA


Problemas e solues da Terceira Fase
TERCEIRA FASE NVEL 1
PROBLEMA 1

A sequncia 121, 1221, 12221, ... contm todos os nmeros da forma 122 21 . A
n dgitos 2

quantidade de dgitos 2 indica a posio do nmero na sequncia. Por exemplo, o nmero 122222221 o stimo termo da sequncia. a) Dentre os 2009 primeiros termos da sequncia, quantos so divisveis por 3? b) Qual o menor nmero mltiplo de 1001 da sequncia?
PROBLEMA 2

O hexgono regular ABCDEF tem rea de 12 cm2. a) Traando segmentos a partir de um vrtice, o hexgono ABCDEF foi repartido em 4 tringulos, conforme figura. Calcule as reas desses tringulos.

b) Usando os quatro tringulos em que foi dividido o hexgono, podemos montar o retngulo PQRS, na figura. Qual a rea desse retngulo?

EUREKA! N32, 2010

34

Sociedade Brasileira de Matemtica

PROBLEMA 3

As casas de um tabuleiro 4 4 devem ser numeradas de 1 a 16, como mostrado parcialmente no desenho, formando um Quadrado Mgico, ou seja, as somas dos nmeros de cada linha, de cada coluna e de cada uma das duas diagonais so iguais.

a) Que nmeros devem ser escritos no lugar de X e de Y? b) Apresente o Quadrado Mgico completo na sua folha de respostas.
PROBLEMA 4

Carlinhos tem vrias peas formadas por quatro quadradinhos de lado unitrio, na forma de L:

Ele forma figuras maiores com essas peas, fazendo coincidir um ou mais lados dos quadradinhos, como no exemplo, em que foram usadas duas dessas peas, fazendo coincidir um lado unitrio. No permitido formar buracos nas figuras.

Permitido No permitido a) Desenhe uma figura cujo permetro 14. b) Descreva como formar uma figura de permetro 2010. c) possvel formar uma figura de permetro mpar? Justifique sua resposta.
PROBLEMA 5

Um domin formado por 28 peas diferentes. Cada pea tem duas metades, sendo que cada metade tem de zero a seis pontos:
EUREKA! N32, 2010

35

Sociedade Brasileira de Matemtica

Esmeralda coloca 4 peas de domin dentro de um estojo, respeitando as regras do jogo, isto , peas vizinhas se tocam em metades com as mesmas quantidades de pontos. Caso seja possvel guardar as quatro peas no estojo, dizemos que o conjunto de quatro peas precioso.

Por exemplo, a figura acima mostra as maneiras de guardar o conjunto precioso formado pelas peas , , , . a) Mostre que um conjunto precioso no pode conter duas peas duplas. A figura abaixo mostra as peas duplas.

b) Quantos conjuntos preciosos contm uma pea dupla? c) Determine a quantidade total de conjuntos preciosos.

EUREKA! N32, 2010

36

Sociedade Brasileira de Matemtica

TERCEIRA FASE NVEL 2 PRIMEIRO DIA PROBLEMA 1

Veja o problema No. 5 do Nvel 1.


PROBLEMA 2

Seja A um dos pontos de interseo de dois crculos com centros X e Y. As tangentes aos crculos em A intersectam novamente os crculos em B e C. Seja P o ponto de plano tal que PXAY um paralelogramo. Prove que P o circuncentro do tringulo ABC.
PROBLEMA 3

Prove que no existem inteiros positivos x e y tais que x3 + y3 = 22009.


SEGUNDO DIA PROBLEMA 4

Resolva, em nmeros reais, o sistema

x+

1 1 1 =y+ =z+ y z x xyz = 1.

PROBLEMA 5

Uma formiga caminha no plano da seguinte maneira: inicialmente, ela anda 1cm em qualquer direo. Aps, em cada passo, ela muda a direo da trajetria em 60o para a esquerda ou direita e anda 1cm nessa direo. possvel que ela retorne ao ponto de onde partiu em (a) 2008 passos? (b) 2009 passos?
1 cm 1 cm 60 60 1 cm

EUREKA! N32, 2010

37

Sociedade Brasileira de Matemtica

PROBLEMA 6

Seja ABC um tringulo e O seu circuncentro. As retas AB e AC cortam o circuncrculo de OBC novamente em B1 B e C1 C , respectivamente, as retas BA e BC cortam o circuncrculo de OAC em A2 A e C 2 C , respectivamente, e as retas CA e CB cortam o circuncrculo de OAB em A3 A e B3 B , respectivamente. Prove que as retas A2A3, B1B3 e C1C2 passam por um mesmo ponto.
TERCEIRA FASE NVEL 3 PRIMEIRO DIA

Esmeralda escreve 20092 nmeros inteiros em uma tabela com 2009 linhas e 2009 colunas, colocando um nmero em cada casa da tabela. Ela soma corretamente os nmeros em cada linha e em cada coluna, obtendo 4018 resultados. Ela percebeu que os resultados so todos distintos. possvel que esses resultados sejam todos quadrados perfeitos?
PROBLEMA 2

PROBLEMA 1

Considere um primo q da forma 2p + 1, sendo p > 0 um primo. Prove que existe um mltiplo de q cuja soma dos algarismos na base decimal menor ou igual a 3.
PROBLEMA 3

So colocadas 2009 pedras em alguns pontos (x, y) de coordenadas inteiras do plano cartesiano. Uma operao consiste em escolher um ponto (a, b) que tenha quatro ou mais pedras, retirar quatro pedras de (a, b) e colocar uma pedra em cada um dos pontos (a, b 1), (a, b + 1), (a 1, b), (a + 1, b). Mostre que, aps um nmero finito de operaes, cada ponto ter no mximo trs pedras. Alm disso, prove que a configurao final no depende da ordem das operaes.

EUREKA! N32, 2010

38

Sociedade Brasileira de Matemtica

SEGUNDO DIA PROBLEMA 4

Mostre que existe um inteiro positivo n0 com a seguinte propriedade: para qualquer inteiro n n0 possvel particionar um cubo em n cubos menores.
PROBLEMA 5

Seja ABC um tringulo e O seu circuncentro. As retas AB e AC cortam o circuncrculo de OBC novamente em B1 B e C1 C , respectivamente, as retas BA e BC cortam o circuncrculo de OAC em A2 A e C 2 C , respectivamente, e as retas CA e CB cortam o circuncrculo de OAB em A3 A e B3 B , respectivamente. Prove que as retas A2A3, B1B3 e C1C2 passam por um mesmo ponto.
PROBLEMA 6

Seja n > 3 um inteiro fixado e x1, x2, , xn reais positivos. Encontre, em funo de n, todos os possveis valores reais de

x3 x1 x2 + + + x n + x1 + x 2 x1 + x 2 + x3 x 2 + x3 + x 4
SOLUES DA TERCEIRA FASE NVEL 1

xn2

x n 1 xn + + x n 1 + x n x n 1 + x n + x1

PROBLEMA 1: SOLUO DE PEDRO HENRIQUE ALENCAR COSTA (FORTALEZA CE)

(a) Um nmero divisvel por 3 tem a soma de seus algarismos como mltiplo de 3. Assim, o primeiro termo mltiplo de 3 1221, pois 1 + 2 + 2 + 1 = 6, que mltiplo de 3. O prximo o mesmo com 3 algarismos 2 a mais. Ento, para saber quantos n2 mltiplos de 3 escritos dessa forma existem at n, fazemos: + 1. Sendo n= 3 2009 2 2007 2009, fica: +1 = + 1 = 669 + 1 = 670. 3 3 (b) Vejamos inicialmente um exemplo de como multiplicar por 1001. Temos 1001 vezes 80 = 80080, pois:

EUREKA! N32, 2010

39

Sociedade Brasileira de Matemtica

0080 0080 + 0080080


7 a lg arismos

O primeiro termo da sequencia que mltiplo de 1001 possui 7 algarismos, sendo ele desta forma 1222221, que igual a 1221 1001, pois:

1221 + 1221 1222221

fcil verificar que os termos anteriores no so mltiplos de 1001.


PROBLEMA 2: SOLUO DE ANA BEATRIZ MOTTA ARAGO CORTEZ (CAMPINAS SP) a)

A G

B C

Seja G o centro do hexgono. A rea GDE e GCD igual a de AFE. Tomando a figura como desenho representativo, podemos dividir o hexgono em seis figuras de reas iguais: AFE; AGE; GDE; GCD; AGC; ABC. Sabendo que sua rea de 12 cm2, dividimos-na por 6 (nmero de partes em que o hexgono foi fracionado; assim, cada frao tem 2 cm2 de rea (12 cm2 : 6). Para calcularmos a rea dos tringulos pedidos, s fazer:
AFE 2cm 2 AED AGE+ EGD 2cm 2 + 2cm 2 4cm 2 ADC AGC+ GCD 2cm 2 + 2cm 2 4cm 2 ABC 2cm 2

EUREKA! N32, 2010

40

Sociedade Brasileira de Matemtica

Temos ento a rea dos dois tringulos iguais AFE e ABC como 2cm 2 (cada um) e a rea dos outros dois tringulos iguais AED e ADC como 4cm 2 (cada um), totalizando 12cm 2 . Obs. H outras formas de resolver o problema com este mesmo raciocnio. Poderamos dividi-lo em 3 losangos, ou 12 pequenos tringulos por exemplo. b) Dividimos a figura, com um raciocnio parecido com o da letra a).
P W U Q S V T R

Cada tringulo acima possui a mesma rea. Utilizando a informao de que o tringulo em questo (SVR ou PQU) possui rea de 2cm2, calculamos a rea do quadriltero multiplicando 2cm 2 pelo nmero em que foi fracionada a figura, o que d 2cm 2 8 = 16cm 2 , que a rea do retngulo PQRS.
PROBLEMA 3: SOLUO DE DIMAS MACEDO DE ALBUQUERQUE (FORTALEZA CE)

a) Veja os quadrados mgicos:

a1 a5 a9 a13

a2 a6 a10 a14

a3 a7 a11 a15

a4 a8 a12 a16 =

14

11 8

12

3 Y

Vendo-os, posso afirmar que a soma total do quadrado a1 + a2 + + a16 o que

equivale a 1 + 2 + + 16 que igual a (16 17 ) 2 = 136. Sabendo que em cada


EUREKA! N32, 2010

41

Sociedade Brasileira de Matemtica

linha a soma a mesma, a soma de cada uma delas ser 136 4 = 34 . Como em cada linha, coluna e diagonal a soma ser 34 os valores de X e Y sero: X = 34 (14 + 11 + 5 ) = 34 30 = 4
Y = 34 (14 + 8 + 3) = 34 25 = 9.

b) Vamos denominar os espaos vazios do quadrado de: b1 ,b2 , b3 ,b4 ,b5 ,b6 , b7 e b8 como mostra a figura:

D1 C1 L1 L2 L3 L4 D2 14 b1 12 b6

C2 11 8 b4 b7

C3 5 b2 3 b8

C4 4 b3 b5 9

Sabendo que em cada linha, coluna ou diagonal a soma 34, temos as seguintes equaes: b3 +b5 = 22 (as razes s podem ser 15 e 6, pois alguns dos nmeros dos outros pares j aparecem). b2 +b8 = 26 (as razes s podem ser 16 e 10, pois alguns dos nmeros dos outros pares j aparecem). b4 +b7 = 15 (as razes s podem ser 13 e 2, pois alguns dos nmeros dos outros pares j aparecem). b1+b6 = 8 (as razes s podem ser 7 e 1, pois se fossem 6 e 2 no daria certo, pois o 2 j aparece em b4 ou b7 ). Sendo assim, na linha 3 a nica combinao qua d certo b4 = 13 e b5 = 6, caso fossem valores diferentes a soma da linha no daria 34. Tendo descoberto esses dois valores eu posso descobrir os outros: Se b3 no 6, s pode ser 15. Se b7 no 13, s pode ser 2. Na linha 2 a nica combinao que d certo b1 = 1 e b2 = 10, pois caso fossem outros valores a soma no daria 34.
EUREKA! N32, 2010

42

Sociedade Brasileira de Matemtica

Tendo descoberto esses outros dois valores posso descobrir mais outros: Se b1 = 1 , b6 s pode ser 7. Logo se b6 7 e b7 2, b8 s pode ser 16. Sabendo todos os valores desconhecidos, o quadrado mgico completo assim:
14 1 12 7 11 8 13 2 5 10 3 16 4 15 6 9

PROBLEMA 4: SOLUO DE ISABELLA AYRES PINHEIRO DE LIMA (GOINIA GO)

a)

P =14

b) Primeiro, vamos utilizar figuras de permetro 12, nas pontas da figura:

Esses dois lados estaro no meio da figura, e por isso, no sero contados, ou seja, o permetro que essa figura vai ocupar na grande figura ser de apenas 10. Como so duas desses figuras (nas pontas), j conseguimos 20 de permetro dos 2010 que precisamos. Agora colocamos figuras de permetro 14 no meio

Como 4 lados de cada figura estaro no meio da grande figura, cada uma delas ocupar 10, no permetro 2010. Teremos que usar 199 destas figuras de permetro 14, no meio; e 2 figuras de permetro 12, nas pontas. Ao todo: 2 10 + 199 10 = 2010 .
EUREKA! N32, 2010

43

Sociedade Brasileira de Matemtica

c) No possvel formar uma figura de permetro mpar, porque uma simples pea tem permetro par e, toda vez que adicionamos outra pea, o permetro aumentou em 10 2. (nmero de lados usados na colagem), que sempre par.
PROBLEMA 5

Veja a soluo do problema 1 do nvel 2.


SOLUES DA TERCEIRA FASE NVEL 2 PROBLEMA 1: SOLUO DE VINCIUS CANTO COSTA (SALVADOR BA)

a) Supondo o contrrio, isto , que seja possvel um conjunto precioso com 2 peas duplas, elas estariam intercaladas por uma pea, pois caso contrrio, elas se encaixariam e isto no possvel pois no tem nmeros em comum e isto no seria de acordo com a regra. Assim, as peas estariam arrumadas dessa forma:

X X ? ?

? Y Y

e as outras duas como iriam se encaixar com as peas duplas de X e Y, seriam da forma
X Y

Mas isto um absurdo, pois no existem peas iguais no jogo (c.q.d). b) Se formarmos um conjunto precioso com uma pea dupla, ele seria organizado dessa forma, seguindo as regras do jogo:

EUREKA! N32, 2010

44

Sociedade Brasileira de Matemtica

X X X

Y Y

Logo, se ns escolhermos as peas duplas e as que no tm contato com ela, ns formamos o conjunto: Apenas pegamos a pea com o nmero da pea dupla e um dos nmeros da que no dupla e a outra com o nmero da pea dupla e o outro nmero da que no dupla e organizamos da maneira certa, que nica, como 6 podemos observar. Logo, a quantidade ser 7 = 105, pois so 7 peas duplas 2 e a outra pea deve ter nmeros diferentes entre si e da pea dupla tambm, logo, so 2 nmeros para escolher em 6, j que uma no pode ser usada. c) A quantidade total de conjuntos preciosos ser a quantidade que inclui uma pea dupla mais a que no tem esse tipo de pea. J temos pelo item b) que com pea dupla 105. Basta contar os conjuntos sem pea dupla. Esses conjuntos sero da forma

X W W

Y Y

com todos os nmeros diferentes dois a dois. Repare que para cada conjunto de 4 nmeros de 0 a 6 temos 3 conjuntos preciosos que seriam:

X W W

Y ,

X W W

Z ,

X Y Y

1. Y Z Z

2. Z Y Y

3. Z W W

EUREKA! N32, 2010

45

Sociedade Brasileira de Matemtica

7 Logo os conjuntos preciosos sem pea dupla totalizam 3, que so as maneiras 4 de escolher 4 nmeros dentre 7 vezes 3. Assim, 7 7 6 5 4 3 2 1 a 105 + 3 = 105 + 3 = 105 + 7 5 3 = 105 + 105 = 210 4 3 2 1 3 2 1 4 quantidade total de conjuntos preciosos.

PROBLEMA 2: SOLUO DE FRANCISCO MARKAN NOBRE DE SOUZA FILHO (FORTALEZA CE)

X A X B
90 180 2

X1

Se P circuncentro de ABC , ento ele deve ser a interseo entre as mediatrizes dos segmentos AB e AC. Como AB e AC so tangentes s circunferncias,

B AY = C AX = 90. Esses dois ngulos tm B AC

em comum e portanto

B AX = C AY , que chamarei de :
B AX = C AY = B AC = 90 X AY = 90 +

EUREKA! N32, 2010

46

Sociedade Brasileira de Matemtica

Da ltima igualdade, como PXAY paralelogramo, temos P X A = 90 . Por outro lado, como o tringulo BXA issceles ( XA = XB ) , temos

AX B = 180 2 , ou seja, PX bissetriz do ngulo AX B. Usando mais uma vez que BXA issceles, PX tambm a mediana e altura relativa ao lado AB. Assim, PX a mediatriz do segmento AB. Pela mesma rzo, PY a mediatriz do segmento AC, o que conclui a prova.
PROBLEMA 3: SOLUO DE LUCAS CAWAI JULIO PEREIRA (CAUCAIA CE)

Para provarmos o que o enunciado quer, basta analisar a equao mdulo 7. Queremos descobrir, ento, quais os restos que um cubo qualquer i3 deixa na diviso por 7. Conseguimos isso elevando ao cubo os possveis restos que um nmero qualquer deixa por 7, que so 0, 1, 2, 3, 4, 5, e 6. Conclumos que os possveis restos que um cubo pode deixar so 0, 1 e 6. Agora analisemos as potncias de 2 mdulo 7.
21 2 ( mod 7 ) 22 4 ( mod 7 ) 23 1( mod 7 ) 24 2 ( mod 7 )

Encontramos o perodo, ento dividimos 2009 por 3. Como o resto dessa diviso 2, logo 22009 4 ( mod 7 ) . Da encontramos um absurdo j que qualquer soma dos possveis restos de dois cubos jamais ser 4. Logo x 3 + y 3 = 22009 no possui soluo nos inteiros.
PROBLEMA 4: SOLUO DE ANDRE MACIEIRA BRAGA COSTA (BELO HORIZONTE MG)

Olhemos para a primeira equao: 1 1 1 x + = y + Vamos substituir o termo em termos das variveis x e y. y z z Da segunda equao, temos: xyz = 1 xy =

1 (substitumos na primeira equao) z

x+

1 = y + xy (multiplicamos tudo por y) y

xy + 1 = y 2 + xy 2 (reordenando)

y 2 ( x + 1) xy 1 = 0 (resolvemos pela forma de Bhskara)


EUREKA! N32, 2010

47

Sociedade Brasileira de Matemtica

1 . x +1 Separemos em dois casos: 1) Caso y = 1.

y1 = 1 e y2 =

Substitumos na segunda igualdade: 1 1 1 1 y + = z + 1+ = z + z x z x xyz = 1 x 1 z = 1 1 x z = 1 Como xyz = 1, temos z = e da x 1 z = x 1 1 + = z + z (multiplicando por z. z 0 ) z 2 z 2 z 1 = 0 (Resolvendo pela frmula de Bhskara) 1 z1 = 1 e z2 = . 2 1 Da equao z = ,x1 = 1 e x2 = 2. x 1 Nesse caso, temos as solues (1, 1, 1) e 2 ,1, . 2 1 2) Caso y = ( x 1) x +1 Substitumos na segunda equao: 1 x z =1 x +1 x 1 = x +1 z 1 1 x x +1 1 1 x +1 = 1 = + =z+ , e ; note que, nesse caso, y = + = z= z x +1 x +1 x x x x a segunda igualdade tambm satisfeita.

EUREKA! N32, 2010

48

Sociedade Brasileira de Matemtica

Resposta: 1 (1,1,1) ; 2 ,1, e todas as triplas da forma 2 a {0, 1} .

1 a +1 , a, , com a +1 a

1 Obs. 2 ,1, esta soluo da forma 2

1 a +1 , a, . a + a 1

PROBLEMA 5: SOLUO DE DANIEL EITI NISHIDA KAWAI (SO PAULO SP)

120 120

60 60

120 120

120

Temos o diagrama infinito de possveis posies em que a formiga pode chegar.

a) Resposta: Sim. Para voltar posio inicial em 2008 passos, basta seguir as instrues abaixo:

EUREKA! N32, 2010

49

Sociedade Brasileira de Matemtica

Posio inicial

Posio inicial

D 333 voltas no hexgono (isso dar 1998 passos e depois siga o trajeto abaixo, em que so usados 10 passos e volta-se posio inicial. No total, s formiga dar 2008 passos e voltar posio inicial b) Resposta: No. Pinte as posies da figura inicial de preto e branco alternadamente. A formiga comea em uma bolinha preta e toda bolinha preta est cercada de bolinhas brancas e toda bolinha branca est cercada de bolinhas pretas. Assim, quando a formiga anda um nmro par de passos, ela sempre termina em uma bolinha preta e quando anda um nmero mpar de passos, ela sempre terminar em uma bolinha branca. Como 2009 mpar, a formiga, se comear em uma bolinha preta, sempre terminar em uma bolinha branca; logo, ser impossvel voltar posio inicial depois de 2009 passos.
PROBLEMA 6: SOLUO DE DANIEL EITI NISHIDA KAWAI (SO PAULO SP)

B O1 O A
EUREKA! N32, 2010

X M C1 C

50

Sociedade Brasileira de Matemtica

Temos AO BO so ngulos

CO e BO1 CO1 inscritos do

C1O1 OO1 . Alm disso, C1 BO e C1 CO mesmo arco de circunferncia

C1 MO . Como ACO issceles (j que AO CO ), 2 Como ABO issceles, C1 AO = C1 CO C1 AO = C1 BO. C1 MO C1 BO = C1 CO =

B AO = ABO B AO + C1 AO = ABO + C1 BO B AC1 = ABC1 ABC1 issceles C1 mAB . De maneira anloga, A2 mBC , A3 mBC , B1 mAC , B3 mAC e C2 mAB A2 A3 mBC , B1 B3 mAC e C1C2 mAB . Como mAB ,mAC e mBC se
encontram em 0, as retas A2 A3 ,B1 B3 e C1C2 passam por um mesmo ponto. Obs. mXY mediatriz do segmento XY .
SOLUES DA TERCEIRA FASE NVEL 3 PROBLEMA 1: SOLUO DE HUGO FONSECA ARAJO (RIO DE JANEIRO RJ)

Sim, possvel. Considerando a tabela como uma matriz aij tome aij = 0 , para

1 i, j 2008, e ai ,2009 = ( 2i 1) ,a2009 ,i = ( 2i ) , para 1 i 2008.


2 2

12 32

22 42

...

Ento j temos 4016 fileiras cujas somas so quadrados perfeitos e distintos. As duas que faltam so a ltima linha e ltima coluna. Seja a2009 ,2009 = d . Queremos que 12 + 32 + ... + 40152 + d = b 2 , 22 + 42 + ... + 40162 + d = c 2 onde b, c so distintos e maiores que 4016. Subtraindo as equaes, temos:
EUREKA! N32, 2010

51

Sociedade Brasileira de Matemtica

c 2 b 2 = 1 + 2 + ... + 4016 = 2008 4017 ( c b )( c + b ) = 2008 4017. Tomando c = 502 4017 + 2 e b = 502 4017 2 , a igualdade acima satisfeita. Para concluir, tome

d = b 2 (12 + 32 + ... + 40152 ) = ( 502 4017 2 ) (12 + 32 + ... + 40152 ) .


2
2

Desse modo, 12 + 32 + ... + 40152 + d = ( 502 4017 2 )

22 + 42 + ... + 40162 + d = ( 502 4017 + 2 ) .


2

PROBLEMA 2: SOLUO DE MATHEUS SECCO TORRES DA SILVA (RIO DE JANEIRO RJ)

Vamos organizar as idias. Se p = 2,q = 5 e 10 satisfaz. Se p > 2, o mltiplo de q s poder ter soma 2 ou 3, pois se tivesse soma 1, seria uma potncia de 10, e como q primo > 5, q no divide 10n ,n Ento, devemos conseguir um mltiplo com soma 2 ou 3. Mltiplos com soma 2: 10a + 1 Mltiplos com soma 3: 10a + 10b + 1 ( a b ) . Pelo Pequeno Teorema de Fermat, 10q 1 1( q ) 10
q 1 2

1( q ) ou 10

q 1 2

1( q )

Se 10 1( q ) ,10 dgitos 2).

q 1 2

q 1 2

+ 1 satisfaz as condies do problema (tem soma dos 1( q ) 10 p 1( q ) ord q 10 = 1 ou p.

Suponha ento que 10

q 1 2

Se ord q 10 = 1,q 9 q = 3 p = 1, absurdo. Logo, ord q 10 = p. Nesse caso, vamos tentar um mltiplo com soma 3, isto , vamos procurar inteiros positivos a e b tais que 10a + 10b + 1 0 (q). 101 ,102 ,...,10 p so p resduos distintos mdulo q = 2 p + 1. De fato, se

10 x 10 y ( q ) com y < x p ,10 ( x y ) 1 ( q ) , 0 < x y < p , contradio,


ord q 10 = p .

pois

Se x tal que 10 x p ( q ) , tomemos a = b = x 2 10 x + 1 0 ( q ) e o problema acaba. Suponha ento que x tal que 10 x p ( q ) . Temos ento p resduos para 101 ,102 ,...,10 p dentre 0 ,1, 2 ,3,..., p 1, p + 1,..., 2 p. Vamos considerar a lista formada por esses p resduos.
EUREKA! N32, 2010

52

Sociedade Brasileira de Matemtica

0 no est na lista, pois q = 2 p + 1 > 5. Se y tal que 10 y 1( q ) , teramos 102 y 1( q ) e

ord q 10 = p p 2 y 2 y = kp 2 kp 2 k k = 2k0 y = k0 p
10 y = 10k0 p (10 p ) 1( q ) , absurdo, pois estamos supondo 10 y 1( q ) .
k0

Logo, 0 e 2p no entram na lista! Considere os pares (1, 2 p 1) ; ( 2 , 2 p 2 ) ; ( 3, 2 p 3) ;...( p 1, p + 1) . Eles incluem todos os resduos que 101 ,102 ,...,10 p podem assumir. Temos p 1 pares e p resduos a escolher. Pelo Princpio da Casa dos Pombos, escolheremos dois nmeros do mesmo par. Mas a soma de dois nmeros do mesmo par 2p(mod q). Logo, x, y com 10 x + 10 y 2 p ( q ) A = 10 x + 10 y + 1 0 ( q ) A mltiplo de q e tem soma dos dgitos 3.
PROBLEMA 3: SOLUO DE RENAN HENRIQUE FINDER (JOINVILLE SC) Vamos chamar operao o ato de tirar 4 pedras de (a, b) e colocar uma pedra em cada um dos pontos ( a + 1,b ) ,( a 1,b ) ,( a,b 1) e ( a,b + 1) . No faremos distino de operaes no mesmo ponto que usam pedras diferentes; assim, atentaremos para quantas pedras h em cada ponto, e no quais. Provemos por induo o seguinte resultado, trivial para n 4. Para qualquer n, existe A ( n ) tal que, para quaisquer pedras p1 , p2 ,..., pn , no

possvel realizar mais de A ( n ) operaes. Suponhamos que isso valha para todo k < n para fazer o caso em que temos n pedras. importante observar que A ( n ) depende apenas de n e no da distribuio das pedras. Claramente o centro de massa das pedras invariante. Logo, podemos fix-lo como origem (desconsiderando a hiptese de as pedras terem coordenadas inteiras). Dada uma sequncia de operaes, chamaremos pi ( t ) a posio de pi aps t operaes , de modo que pi ( 0 ) a posio inicial de pi . Note que x ,( x 1) + ( x + 1) = 2 x 2 + 2
2 2

2 2 2 2 a,b ,( a + 1) + b2 + ( a 1) + b2 + a2 + ( b 1) + a2 + ( b + 1) = 4a2 + 4b2 + 4 Assim, se uma operao em (a, b) move para p1 , p2 , p3 , p4 , com

p1 ( t + 1) = ( a + 1,b) , p2 ( t + 1) = ( a 1,b) , p3 ( t + 1) = ( a,b + 1) e p4 ( t + 1) = ( a,b 1) ,

EUREKA! N32, 2010

53

Sociedade Brasileira de Matemtica

pi ( t ) = pi ( t 1) + 4.
2 2 i =1 i =1

Por induo, t, pi ( t ) 4t i : pi ( t )
2 2 i =1

4t t pi ( t ) 2 n n
2

Vamos escolher t grande (veremos que t 9n3 A ( n 1) basta).

onde C ( p,r ) = { x

t Definimos p = pi ( t ) . Da invarincia do centro de massa, j , p j ( t ) C p, , n


2

: x p r} o crculo de centro p e raio r. Isso ocorre

t porque, se todas as pedras estivessem em C p, , que convexo, seu centro de n t t massa tambm estaria, o que significa ( 0 ,0 ) C p, n p n , absurdo. Agora vemos as regies R1 = C ( p,3 A ( n 1) )
R2 = C ( p,6 A ( n 1) ) R3 = C ( p,9 A ( n 1) ) Rn = C ( p,3nA ( n 1) ) .
Uma das n + 1 regies R1 ,R2 \ R1 ,R3 \ R2 ,...Rn \ Rn 1 e pedra. Como t 9n3 A ( n 1)
2 2

\ Rn no contm nenhuma

t t 3nA ( n 1) , teremos p j C p, e n n

t 2 Rn C p, p j Rn \ Rn no est vazia. R1 no est vazia porque n p R1 . Logo, k ,1 k n tal que Rk \ Rk 1 no tem pedras.
Assim temos at n 1 pedras em Rk 1 e at n 1 pedras em
2

Rk .

A distncia entre uma pedra de Rk 1 e uma fora de Rk sempre pelo menos

3 A ( n 1) (vide definio). As pedras em Rk 1 e as fora de Rk se movero independentemente at que duas delas ocupem a mesma posio. Para que isso
EUREKA! N32, 2010

54

Sociedade Brasileira de Matemtica

ocorra, pela hiptese de induo, as pedras fora de Rk no realizaro mais de

A ( n 1) movimentos, bem como as de dentro de Rk 1 . Portanto, depois de

2 A ( n 1) rodadas, cada pedra se deslocar no mximo A ( n 1) unidades, logo uma pedra fora de Rk no poder ficar no mesmo ponto que uma pedra que estava dentro de Rk 1 , o que torna os dois conjuntos necessariamente independentes. Assim, basta tomar A ( n ) = 9n3 A ( n 1) + 2 A ( n 1) .
2

Isso resolve a primeira parte. Para a segunda parte, comecemos lembrando que, se chegarmos mesma configurao de duas maneiras diferentes, a igualdade

p (t )
i =1 i

= 4t + pi ( 0)
i =1

diz que o nmero de operaes, t, igual nas duas maneiras. Para a prova, suponhamos que na configurao inicial, os pontos com 4 pedras ou mais sejam X1 ,X 2 ,... e X e . Considere tambm uma sequncia de operaes que leva o plano a um estado em que no possvel fazer mais operaes. Certamente, ocorreram operaes com centro em X1 ,X 2 ,... e X e .
que Considerando duas sequncias de operaes O1 ,O2 ,...,O e O1 ,O2 ,...,O

terminam em uma configurao na qual no possvel fazer mais operaes, provaremos que uma permutao da outra via induo em min{,} , o que resolve o problema. Seja X um ponto em que h mais de quatro pedras no princpio. Seja O a primeira operao em {O1 ,O2 ,...,O } com centro em X. Vamos provar que a sequncia de operaes O ,O1 ,O2 ,...,O1 ,O+1 ,...O leva ao mesmo resultado que O1 ,O2 ,...,O . Basta provar que O ,O1 ,...,O1 ,O+1 ,...O uma sequncia de operaes vlidas, j que cada operao tira o mesmo nmero de pedras de cada ponto e coloca o mesmo nmero em cada ponto, independentemente de quando foi realizada, de forma que as operaes so comutativas. Pelo mesmo argumento, basta ver que O ,O1 ,...,O1 uma sequncia possvel. Mas comear com O claramente possvel e, da minimalidade de , as operaes

O1 ,... e O1 tm centro em um ponto diferente de X. Assim, O s pode ter


EUREKA! N32, 2010

55

Sociedade Brasileira de Matemtica

aumentado o nmero de pedras nos centros de O1 ,... e O1 , e no diminudo, o que faz com que toda essa sequncia seja possvel. Em outras palavras, sem perda de generalidade, O1 = O O1 tem centro em X.
' Analogamente, podemos supor que, O1 tem centro em X. Agora, aps a realizao

da operao com centro em X,


min{,} diminui, e vemos que as seqncias

O2 ,...,O e O2,...,O so iguais pela hiptese de induo.


PROBLEMA 4: SOLUO DE MARLEN LINCOLN DA SILVA (FORTALEZA CE)

Seja S = {n

: possvel particionar um cubo em n cubos menores}.

Lema: Se x, y S , ento x + y 1 S. Prova: Particione o cubo inicial em x cubos menores. Escolha um desses cubos e o particione em y cubos. Da, o cubo inicial estar particionando em x + y 1 cubos menores. Claramente, n3 S , para n 2 inteiro. Assim 23 S; portanto, x S ,x + 8 1 = x + 7 S. Para termos o resultado desejado, basta provarmos que existem a1 ,a2 ,...,a7 S , tais que ai a j ( mod 7 ) , para 1 i < j 7 (basta escolhermos n0 = max {ai ,1 i 7} ). De fato, se x n0 = max {ai ,1 i 7} e x a j ( mod 7 ) ,x = a j + 7 k S. Seja S = { x ( mod 7 ) ,x S } . De forma anloga, se x, y S ento

( x + y 1) ( mod 7 ) S.
Claramente 1,6 ,0 S , j que 23 ,33 e 73 S. Logo ( 6 + 6 1) mod 7 = 4 S e

( 0 + 6 1) mod 7 = 5 S. Ento ( 4 + 0 1) mod 7 = 3 S' e ( 3 + 0 1) mod 7 = 2 S' S = {0,1,2,3,4,5,6} e o problema est acabado.
PROBLEMA 5: SOLUO DE GUSTAVO LISBOA EMPINOTTI (FLORIANPOLIS SC)

Considere uma inverso com respeito ao circuncrculo do ABC. Como o circuncrculo do AOC passa pelo centro de inverso (O), seu inverso uma reta pelos inversos de A e C. Mas A e C pertencem ao crculo de inverso, de modo que so seus prprios inversos. Ou seja, a reta AC o inverso do circuncnculo de AOC . O inverso do ponto A2 a interseo do inverso do circuncrculo de

AOC que AC com o inverso de AB que o circuncrculo do AOB isto


EUREKA! N32, 2010

56

Sociedade Brasileira de Matemtica

, o ponto A3 . Ento A2 e A3 so inversos, logo A2 A3 passa pelo centro de inverso, O. Analogamente, B1 B3 e C1C2 passam por O, como queramos.
PROBLEMA 6: SOLUO DE RENAN HENRIQUE FINDER (JOINVILLE SC)

Sendo rn ( a ) o resto da diviso do inteiro a por n (i.e., o nico nmero r em ]0,n ] tal que a r mod ( n ) , definamos xa = xrn ( a ) . Claramente, a funo S : ( dada por:
* n +

S ( x1 ,...xn ) =
i =1

n xi xi =1 satisfaz S ( x1 ,...xn ) > xi 1 + xi + xi +1 i =1 x1 + x2 + ... + xn

Alm disso, f ( t ) = S (1,t,t 2 ,t 3 ,...,t n 1 ) = contnua e tal que f (1) =

( n 2) t + 1 t n 1 + 1 + t + t n 1 t 2 + t + 1 1 + t n 2 + t n 1

n e lim f ( t ) = 0 + ( n 2 ) 0 + 1 = 1, de modo que t 3 n S ( x1 ,...xn ) pode assumir qualquer valor no intervalo 1, . Por outro lado, se n 3 n 1 t n par, g (1,t,1,t,...,1,t ) = + contnua e tal que g (1) = , enquanto 2 2t + 1 t + 2 3 n n n lim g ( t ) = . Portanto, S ( x1 ,...,xn ) assume todos os valores do intervalo , . t 2 3 2 Se n for mpar, definamos a funo 2 n 1 t n3 1 n h ( t ) = S (1,t,1,t,...,1,t,1) = + + . Temos h (1) = e t+2 2 t+2 2 2t + 1 3 n 1 h (t ) quando t . Segue disso que n pode tomar qualquer valor em 2 n n 1 3 , 2 . No caso em que n par, no difcil resolver o problema se notarmos x x que < x, y,z *+ . x+ y+z x+ y De fato: 2k k xi x2i 1 x2i = + x2i 1 + x2i + x2i +1 i =1 xi 1 + xi + xi +1 i =1 x2 i 2 + x2 i 1 + x2 i k x2i 1 x2i < + = k. x2i 1 + x2i i =1 x2 i 1 + x2 i
EUREKA! N32, 2010

57

Sociedade Brasileira de Matemtica

Assim, sob a hiptese de n ser par, os valores possveis de S ( x1 ,...xn ) so os

n elementos de 1, . 2 n 1 Para o caso n mpar, queremos mostrar que a imagem de S 1, . 2 Adaptaremos a idia usada anteriormente. Veja que essa idia prova que x5 xn 1 xn x4 n3 + + ... + + < 2 x3 + x4 + x5 x4 + x5 + x6 xn 2 + xn 1 + xn xn 1 + xn + xn +1 n 1 , suficiente termos Logo, para que se garanta que S ( x1 ,x2 ,...xn ) < 2 x3 x + x + x3 x1 x2 + + 1 2 x0 + x1 + x2 x1 + x2 + x3 x2 + x3 + x4 x1 + x2 + x3 as condies x0 x3 e x4 x1 implicam essa desigualdade. Se supusermos por n 1 absurdo S ( x1 ,...xn ) , conclumos que x0 x3 x1 > x4 e x1 x4 x0 < x3 . 2 Analogamente, supondo x0 x3 , x1 > x4 x2 > x5
x2 > x5 x3 > x6 x3 n 4 > x3 n 1 x3 n 3 > x3n
O absurdo que x0 x3 > x6 > x9 > ... > x3n = x0 . A suposio x1 x4 pode ser tratada similarmente. Obs: Outra maneira de se fazer o caso x0 x3 definir yn = x3 n , o que d y3 y0 . Alem disso, n 1 n 1 S ( x1 ,...,xn ) < S ( y1 ,..., yn ) < , o que j sabemos provar. 2 2

EUREKA! N32, 2010

58

Sociedade Brasileira de Matemtica

XXXI OLIMPADA BRASILEIRA DE MATEMTICA


Problemas e solues da Primeira Fase Nvel Universitrio
PROBLEMA 1

(a) Encontre o valor mnimo da funo f : (aqui e = 2,71828... a base do logaritmo natural). (b) Qual destes nmeros maior: e ou e ?
PROBLEMA 2

dada por f ( x ) = e( x / e ) x

Seja
3

uma raiz de x 7 1, com 1. Existe um polinmio Mnico p de grau 2 z1 = + 2 + 4 e


5 6

com coeficientes inteiros cujas razes so os nmeros

z2 = + + . Calcule p ( 3) .
PROBLEMA 3

A r D descansa no vrtice A de um tringulo equiltero ABC. A cada minuto a r salta do vrtice em que est para um vrtice adjacente, com probabilidade p de o salto ser no sentido horrio e 1 p de ser no sentido anti-horrio, onde p ( 0,1) uma constante. Seja Pn a probabilidade de, aps n saltos, D estar novamente no vrtice A. (a) Prova que, qualquer que seja p ( 0 ,1) ,lim Pn = 1 / 3. (b) Prove que existe p ( 0 ,1 / 100 ) tal que, para algum n ,Pn = 1 / .
PROBLEMA 4
n

Determine a quantidade de nmeros inteiros positivos n menores ou iguais a 31! Tais que 3n + n divisvel por 31.
PROBLEMA 5

Dados os nmeros reais a, b, c, d, considere a matriz

Se f ( x ) = a + bx + cx 2 + dx3 , prove que

a d A= c b

b a d c

c b a d

d c . d a

EUREKA! N32, 2010

59

Sociedade Brasileira de Matemtica

det A = f (1) f ( i ) f ( 1) f ( i ) . (Aqui i representa a unidade imaginria.)


PROBLEMA 6

Considere a sequncia a0 ,a1 ,a2 ,... definida por a0 = 0 ,a1 = / 3 e, para n 1,

an +1 =
Calcule

( a0 an + a1an 1 + a2 an 2 + ... + an a0 )

3 ( n + 1)

2
k =0

ak
k

= a0 +

a1 a2 a3 + + + ... . 2 4 8

SOLUES NVEL UNIVERSITRIO PRIMEIRA FASE PROBLEMA 1

1 a) A derivada da funo f f ( x ) = e( x e) 1, que se anula apenas para x = e, e sendo negativa para x < e e positiva para x > e. Assim, o valor mnimo de f f (e) = 0. b) Pelo resultado do item anterior, como e temos que f ( ) > 0 , logo e( e) > , ou seja, e( ) > e .

PROBLEMA 2

Um polinmio que satisfaz 2 p ( x ) = x ( z1 + z2 ) x + z1 z2 .

as

condies

do

enunciado

Logo p ( x ) = x 2 + x + 2 e p ( 3) = 14.
PROBLEMA 3

7 1 = 1. 1 z1 z2 = 4 + 6 + 7 + 5 + 7 + 8 + 7 + 9 + 10 = 3 + + 2 + 3 + 4 + 5 + 6 = 2. z1 + z2 = + 2 + 3 + 4 + 5 + 6 = 1 +

a) Sejam A, B, e C os vrtices do tringulo no sentido anti-horrio. Seja Qn (resp. Rn ) a probabilidade de, aps n saltos, D estar no vrtice B (resp. C). Temos P0 = 1,Q0 = R0 = 0 e, para todo n 0 ,

EUREKA! N32, 2010

60

Sociedade Brasileira de Matemtica

Pn +1 = (1 p ) Rn + pQn (*) Qn +1 = (1 p ) Pn + pRn Rn +1 = (1 p ) Qn + pPn


Dado n natural, seja

Dn = max { Pn Qn , Qn Rn , Rn Pn } . Vamos provar que,

para todo n, Dn +1 max { p,1 p} . Dn . Dado n, h 6 possibilidades para a ordem dos nmeros Pn ,Qn ,Rn . Vamos analisar o caso Pn Qn Rn (os outros 5 casos so anlogos). Nesse caso, a maior distncia Dn entre dois dos nmeros Pn ,Qn e Rn Rn Pn . De (*), obtemos: Pn +1 Qn +1 = (1 p )( Rn Pn ) + p ( Qn Rn ) max {(1 p )( Rn Pn ) , p ( Rn Qn )} ,

max {1 p, p} . Dn , pois Rn Pn e Qn Rn tm sinais contrrios. Qn +1 Rn +1 = (1 p )( Pn Qn ) + p ( Rn Pn ) max {(1 p )( Qn Pn ) , p ( Rn Pn )}


max { p,1 p} Dn , pois Pn Qn e Rn Pn tm sinais contrrios. Rn +1 Pn +1 = (1 p )( Qn Rn ) + p ( Pn Qn ) = (1 p )( Rn Qn ) + p ( Qn Pn )

max { p,1 p} ( Rn Qn + Qn Pn ) = max { p,1 p} ( Rn Pn ) = max { p,1 p} Dn .


Assim, Dn +1 = max { Pn +1 Qn +1 , Qn +1 Rn +1 , Rn +1 Pn +1 } max { p,1 p} Dn , para todo n 0 , donde Dn ( max { p,1 p} ) ,n 0.
n

Como Pn + Qn + Rn = 1,n ,

Pn

n P + Qn + Rn ( Pn Qn ) + ( Pn Rn ) 2Dn 2 1 = Pn n = ( max { p,1 p} ) ,n 0. 3 3 3 3 3

Como 0 < max { p,1 p} < 1, segue imediatamente que lim Pn


n n

1 = 0 , e que 3

1 lim Pn = . 3

b) Para p = 0 teramos Pn = 1 quando n mltiplo de 3 e Pn = 0 caso contrrio. Por outro lado, tomando p = 1 100 , temos lim P3 k +1 = 1 3 . Em particular, existe
k

1 1 1 tal que P3r +1 > , pois < . 3


61

EUREKA! N32, 2010

Sociedade Brasileira de Matemtica

Considerando P3r +1 = P3r +1 ( p ) como funo de p, temos que P3r +1 ( p ) um polinmio (de grau no mximo 3r + 1) em p, e portanto depende continuamente de 1 1 p. Como P3r +1 ( 0 ) = 0 e P3r +1 > , existe, pelo teorema do valor 100 1 1 tal que P3r +1 ( p ) = . intermedirio, p com 0 < p < 100 Soluo alternativa para o item a): Podemos (Como no incio da soluo anterior), escrever p 1 p Pn Pn +1 0 0 p Qn ,n 0. Qn +1 = 1 p R p 1 p 0 n +1 Rn 1 2 p 1 Os autovalores dessa matriz 3 3 so 1 e i 3 . 2 2 As normas dos autovalores distintos de 1 so iguais a 1 3 p (1 p ) < 1, donde

Pn ,Qn e Rn convergem a certos nmeros, que denotaremos por x, y, z, respectivamente. Devemos ento ter: x = (1 p ) z + py, y = (1 p ) x + pz,z = (1 p ) y + px, donde x = (1 p ) z + p ( (1 p ) x + pz ) (1 p + p 2 ) x = (1 p + p 2 ) z x = z y = (1 p ) x + px = x, e logo x = y = z = 1 3 (pois x + y + z = 1).
PROBLEMA 4

Pelo pequeno teorema de Fermat, 330 1( mod 31) , e logo

( 3 ( mod 31) )
n

peridico com perodo divisor de 30. Por outro lado, obviamente ( n ( mod 31) ) peridico com perodo 31.

Portanto, ( 3n + n ( mod 31) ) peridico com perodo divisor de 31 30 = 930.

Pelo teorema chins dos restos, para cada a com 0 a 29 e b com 0 b 30 , existe um nico c ( a,b ) com 0 c ( a,b ) 929 tal que c ( a,b ) a ( mod 30 ) e

c ( a,b ) b ( mod 31) .


Temos 3c ( a ,b ) + c ( a,b ) 3a + b ( mod 31) .
EUREKA! N32, 2010

62

Sociedade Brasileira de Matemtica

Fixando a e fazendo b variar, 3a + b percorre todas as 31 classes (mod 31). Assim, 3m + m , 0 m 929 passa 30 vezes por cada classe (mod 31). Como 930 = 31 30 31 !, 3n + n 0 ( mod 31) para 31!/31 = 30! inteiros positivos menores ou iguais a 31!.
PROBLEMA 5 1. Soluo

Se x uma raiz quarta da unidade, temos xf ( x ) = d + ax + bx 2 + cx 3 ,x 2 f ( x ) = c + dx + ax 2 + bx3 e x 3 f ( x ) = b + cx + dx 2 + ax 3 , de modo que

Assim, o vetor (1,x,x 2 ,x3 ) autovetor de A, com autovalor f(x), para x = 1, i, 1, i. deduzimos que A possui 4 autovetores independentes e, portanto, det A o produto dos respectivos autovalores, ou seja, det A = f (1) f ( i ) f ( 1) f ( i ) .
2. Soluo

1 f ( x) 1 xf x x x ( ) A 2 = 2 = f ( x) 2 . x x f ( x) x 3 3 3 x x f ( x) x

Observamos que det A um polinmio do quarto grau nas variveis a, b, c, d, enquanto f(1), f(i), f ( 1), f ( i) so polinmios irredutveis distintos do primeiro grau nessas mesmas variveis. Podemos realizar operaes lineares nas linhas de A para provar que o polinmio det A divisvel por f(1), f(i), f (1), f (i). Isto fica mais rpido utilizando a mesma ideia da primeira Soluo: se x raiz quarta da unidade, multiplicando a segunda coluna por x, a terceira por x2 e a quarta por x3 e somando tudo isso primeira coluna, obtemos ( f ( x ) ,xf ( x ) ,x 2 f ( x ) ,x3 f ( x ) ) . Assim, temos det A = kf (1) f ( i ) f ( 1) f ( i ) , onde k uma constante a ser determinada. Fazendo a = 1 e b = c = d = 0, obtemos det A = 1 e f (1) f ( i ) f ( 1) f ( i ) = 1, logo k = 1, como queramos demonstrar.
PROBLEMA 6

Defina

f ( x ) = an x n = a0 + a1 x + a2 x 2 + ... + an x n + ...
n =0

Ento
EUREKA! N32, 2010

63

Sociedade Brasileira de Matemtica

n x n =0 l =0 = ( a0 a0 ) + ( a0 a1 + a1a0 ) x + ...
2 n l n l

( f ( x )) = a a

+ ( a0 an + a1an 1 + a2 an 2 + ... + an a0 ) x n + ... e

f ( x ) = ( n + 1) an +1 x n = a1 + 2a2 x + ... + ( n + 1) an +1 x n + ...


2 f ( x ) ) coincidem, ( 3 2 exceto pelo coeficiente constante. Temos portanto f ( x ) ( f ( x ) ) = . Logo 3 3 temos n=0

Pela condio do enunciado, os coeficientes de f ( x ) e de

df 2 df = ( f + 1) 2 = dx dx 3 f +1 3
( x + C ) f ( x ) = tg ( x + C) 3 3 Como f ( 0 ) = a0 = 0 , conclumos que C = 0, e portanto arctg f =

2
k =0

ak
k

1 1 . = f = tg = 3 2 6

EUREKA! N32, 2010

64

Sociedade Brasileira de Matemtica

XXXI OLIMPADA BRASILEIRA DE MATEMTICA


Problemas e solues da Segunda Fase Nvel Universitrio
PRIMEIRO DIA PROBLEMA 1

Seja f :[0,1] [0,1] crescente, derivvel e inversvel. Se

f ( x)dx = f 1 ( x)dx , prove que existem dois reais diferentes a e b,


0

0 a < b 1 , tais que f '(a ) = f '(b) = 1 .


Obs.: f 1 denota a inversa da funo f .
PROBLEMA 2

Seja N = {0,1,2,3,...}. Dados conjuntos A, B N , para cada inteiro positivo n denote por r(A, B, n) o nmero de solues da equao a + b = n, a A, b B . Prove que existe n0
PROBLEMA 3

tal que r(A, B, n + 1)> r(A, B, n) para todo n > n0 se e

somente se N \ A e N \ B so finitos. Dados

n, a1 , a2 ,..., an

inteiros

positivos,

definimos

q0 = 1, q1 = a1

qk +1 = ak +1qk + qk 1 , para 1 k n 1 .
Prove que, dado c > 1, existe K > 0 tal que, para todo M > K, existem n inteiro positivo e a1 , a2 ,..., an pertencentes a {1,2} tais que M qn < c M .
SEGUNDO DIA PROBLEMA 4

Seja H o hiperboloide de equao 3x 2 + 3y 2 z 2 1 = 0 . i) Prove que todo ponto ( x, y, z ) H pertence a exatamente duas retas contidas em H. ii) Prove que todas as retas contidas em H formam o mesmo ngulo com o plano de equao z = 0, e determine esse ngulo.

EUREKA! N32, 2010

65

Sociedade Brasileira de Matemtica

Ache todas as funes f : Z Z que satisfazem: i) f (f (n)) = f (n + 1), para todo n Z . ii) f(2009n + 2008) = 2009.f(n) para todo n Z .
PROBLEMA 6

PROBLEMA 5

Para n inteiro positivo seja f(n) o nmero de produtos de inteiros maiores que 1 cujo resultado no mximo n, isto , f(n) o nmero de k-uplas ( a1 , a2 ,..., ak ) onde k algum natural, ai 2 inteiro para todo i e a1 a2 ... ak n (contando a 0-upla vazia ( ), cujo produto dos termos 1). Assim, por exemplo, f(1) = 1, por causa da 0-upla ( ) e f(6) = 9, por causa da 0-upla ( ), das 1-uplas (2), (3), (4), (5) e (6) e das 2-uplas (2, 2), (2, 3) e (3, 2). Seja > 1 tal que

m
m =1

= 2.

a) Prove que existe uma constante K > 0 tal que f ( n) K n para todo inteiro positivo n. b) Prove que existe uma constante c > 0 tal que f ( n) c n para todo inteiro positivo n.
PROBLEMA 1: SOLUO DE MARLON DE OLIVEIRA GOMES (FORTALEZA CE)

portanto sobrejetora, logo, existem a e b [ 0,1] tais que f ( a ) = 0 e f ( b ) = 1. Se a 0, b 1,

(I) Notemos primeiramente que f (0) = 0 e f (1) = 1. De fato, f inversvel e

f ( t ) < 0, t [ 0, a )
f ( t ) > 1, t ( b,1]

Um absurdo. (II) Afirmao: f possui um ponto fixo em (0, 1). Isto , c ( 0,1) tal que

f ( c ) = c.
Prova: Suponha que seja f ( x ) > x, x ( 0,1) . Ento, f 1 f ( x ) = x > f 1 ( x ) x, pois f crescente f 1 crescente. Logo, seria f ( x ) > x > f 1 ( x ) , x ( 0,1)

EUREKA! N32, 2010

66

Sociedade Brasileira de Matemtica

Se supusermos f ( x ) < x, x ( 0,1) temos um resultado anlogo. Sendo f diferencivel, tambm contnua, donde g : [ 0,1] Suponha agora que existam x1 e x2 ( 0,1) tais que f ( x1 ) > x1 e f ( x2 ) < x2 .

f ( x ) dx =
0 1 0

f ( x ) dx > xdx > f 1 ( x ) dx, um absurdo, pois


0 1 0 0

f 1 ( x ) dx.

, g (t ) = f (t ) t

contnua. Note que g ( x1 ) > 0 e g ( x2 ) < 0, logo, pelo teorema do valor intermedirio, existe c ( x1 , x2 ) tal que g ( x ) = 0 f ( c ) = c, o que prova o resultado. (III) Pelo teorema do valor mdio, existem a ( 0, c ) e b ( c,1)

f (c) =1 f (c) f ( 0 ) = f ( a ) ( c 0 ) f ( a ) = c . tais que f (1) f c = f b 1 c f b = 1 f ( c ) = 1 ( ) ( )( ) ( ) 1 c


PROBLEMA 2: SOLUO DE RAMON MOREIRA NUNES (FORTALEZA CE)

Para cada n , seja q ( A, B, n ) o nmero de pares ( x, y ) tais que x + y = n e, alm disso, x A ou y B . Assim, q ( A, B, n ) o nmero de pares (x, y) com x + y = n que r ( A, B, n ) no conta. Portanto, q ( A, B, n ) = n + 1 r ( A, B, n ) . Veja que:

r ( A, B, n + 1) > r ( A, B, n )

r ( A, B, n + 1) r ( A, B, n ) + 1 n + 1 q ( A, B, n + 1) n q ( A, B, n ) + 1
q ( A, B, n + 1) q ( A, B, n )

n n0 ; da, q ( A, B, n ) q ( A, B, n0 ) , n n0 .
Como s existem finitos n menores que n0 , isso nos diz que q ( A, B, n ) limitada como funo de n. Agora, se (x, y) tal que x + y = n e x A, ento ( x, y ) contado por

q ( A, B, n ) . Dessa forma, q ( A, B, n ) # ({1, 2,..., n} \ A) .

EUREKA! N32, 2010

67

Sociedade Brasileira de Matemtica

Se

\ A fosse infinito, poderamos tomar # ({1, 2,..., n} \ A ) to grande quanto

quisssemos, mas como q ( A, B, n ) limitada isso no pode ocorrer, logo, finito. Anlogo \ B tambm finito. Agora, suponha \ A e \ B finitos.

\A

Logo (x, y) no contado por r ( A, B, n ) apenas quando x

\ A = {a1 , a2 ..., ak } ; a1 < ... < ak \ B = {b1 , b2 ..., bm } ; b1 < ... < bm tome n0 = ak + bm . Se n > n0 , tem-se que (x, y) com x + y = n ento x > ak ou y > bm pois x ak e y bm n = x + y ak + bm = n0 . Absurdo. \ A ou y

\B

(ambos no ocorrem simultaneamente); o primeiro caso ocorre n vezes e o segundo m vezes. Assim, r ( A, B, n ) = n + 1 k m , e logo r satisfaz a condio do enunciado.
PROBLEMA 3: SOLUO DA BANCA

Vamos escolher dois inteiros positivos grandes r, s e tomar m = r + s, a j = 1 para

1 j r e a j = 2 para r + 1 j r + s = m. Seja qk = qk ( a1 , a2 ,..., ak ) , para


1 k m. Temos qk +1 = qk + qk 1 , para 1 k r 1, e portanto q j = Fj +1 , para

0 j r , onde, para j 1
j j j 1 + 5 1 5 1 + o (1) 1 + 5 1 Fj = 2 = 2 5 5 2

j-simo

termo
j

da

sequncia

de

Fibonacci.

Assim,

1+ 5 1+ 5 q j = (1 + o (1) ) c para j grande, onde c = . Por outro lado, temos 2 2 5 qk +1 = 2qk + qk 1 para r k m 1, e logo qr + j = u j +1qr + u j qr 1 , onde ( u j )
j 0

a sequncia dada por u0 = 0, u1 = 1 e uk + 2 = 2uk +1 + uk , para k 0. Como

uk =
obtemos

1 1+ 2 2 2

) (1 2 )
k

= 1 + o (1) 1 + 2 2 2

, para k 0,

EUREKA! N32, 2010

68

Sociedade Brasileira de Matemtica

qr + j =

1 + o (1) 2 2

((1 + 2 ) q + q ) (1 + 2 )
r r 1

j 1+ 5 1 + o (1) 5 1 = c 1+ 2 1+ 2 + 2 2 2 2

j 1+ 5 4 + 10 + 2 c 1+ 2 = (1 + o (1) ) 2 , 8 para j e r grandes.

Como log 1 + 2

1+ 5 log 2
m

irracional (pois no possvel termos

1+ 5 com m e n inteiros positivos), a concluso do problema 1+ 2 = 2 , segue (tirando logaritmos) do seguinte fato, que provado a seguir:

Dados , > 0 tais que irracional, > 0 e r > 0, existe x0 > 0 tal que, para qualquer

x , x x0 ,

existem

inteiros

m, n r

tais

que

m + n x < .
Para provar este fato, notemos que, se

(p

qn

n0

a sequncia de reduzidas da

frao contnua de , podemos escolher k natural com q 2 k +1 > . Teremos ento (ver o artigo sobre Fraes Contnuas na Revista Eureka! No. 3)

< 1 q 2 k + 2 < q 2 k +1 p 2 k +1 < 0 < q 2 k p 2 k < 1 q 2 k +1 <


e portanto < q 2 k +1 p 2 k +1 < 0 < q 2 k p 2 k < . Seja X o conjunto de todos os nmeros tomando da forma

a + b

com e

a, b 0

inteiros.

Se

y = a + b q 2 k +1 + p 2 k pertence a X, temos a q 2 k +1 ou b p 2 k , e
portanto,

repetidamente, segue que, para todo z y, [ z , z + ) X . Como, para

0 < 1 , 2 <

1 = p 2 k +1 q 2 k +1 e teremos y + 1 X ou

2 = q 2 k p 2 k , temos y + 2 X . Usando este fato

q + p2k k0 := 2 k +1 , temos k0 X e k0 q 2 k +1 + p 2 k , portanto, para


EUREKA! N32, 2010

69

Sociedade Brasileira de Matemtica

todo z k0 , [ z , z + ) X . Assim, tomando x0 := k0 + r ( + ) , temos que, dado

x r ( + ) , x r ( + ) + ) X . Assim, existem a, b 0 inteiros a + b e portanto com x r ( + ) , x r ( + ) + ) ,


x ( r + a ) + ( r + b ) < x + , o que prova o fato acima.

x x0 ,

temos

x r ( + ) k0 ,

portanto

PROBLEMA 4: SOLUO DE GABRIEL LUIS MELLO DALALIO (S.J. DOS CAMPOS SP)

Tomando um ponto esse ponto como:

( x, y , z ) H ,

podemos representar uma reta que passa por

r : {( x + ka, y + kb, z + kc ) , k
2 2

}. Para ( a, b, c ) que faam r estar contida em


2

H teremos:

3 ( x + ka ) + 3 ( y + kb ) ( z + kc ) 1 = 0, k

3 x 2 + 6 xka + 3k 2 a 2 + 3 y 2 + 6 ykb + 3k 2b 2 z 2 2 zkc k 2 c 2 1 = 0, k Como 3 x 2 + 3 y 2 z 2 1 = 0, pois ( x, y, z ) H tem-se:


3 xa + 3 yb zc = 0 (I) 2 2 2 3a + 3b c = 0 (II)
(I) (3x, 3y, z) vetor normal
Cone (II) 30o 60o

Plano

Como o menor ngulo que a direo (a, b, c) forma com o plano z = 0 igual a

c arctg , por (II) tem-se: 2 2 a +b

EUREKA! N32, 2010

70

Sociedade Brasileira de Matemtica

c = arctg 2 = arctg 3 = 60. c 3 Assim est provado ii), qualquer reta contida em H ter ngulo igual a 60 com o plano z = 0. A soluo do sistema a interseo do plano com o cone, que pode ser apenas a origem, o que indicaria que no haveria r possvel, ou pode ser igual a duas retas, indicando duas direes possveis para (a, b, c), ou seja, duas retas r possveis, ou ainda uma reta apenas de interseo, que o caso de ( 3 x,3 y , z ) ter ngulo c = arctg 2 2 a +b

( )

menor de 30 com o plano z = 0. Para que haja interseo de duas retas, o vetor normal do plano ( 3 x,3 y, z ) deve ter um ngulo formado com o plano z = 0 menor que o ngulo do cone, que de 30 . Esse ngulo dado por z z 1 < arctg arctg = arctg = 30 2 2 2 9x + 9 y 3 3z (pois 9 x 2 + 9 y 2 = 3 z 2 + 1 > 3 z 2 ).

z Ento, como arctg 9x2 + 9 y2

< 30, fica provado o item i).

( f (n) ) = f ( n + 1) e alm disso a funo f (n) = n + 1 satisfaz (ii) f (2009n + 2008) = 2009 f ( n ) . Suponhamos 2 2 ento que f (n1 ) = f ( n2 ) com n1 < n2 f ( n1 ) = f ( n2 ) f ( n1 +1) = f ( n2 +1) ; indutivamente teremos f ( n1 + k ) = f ( n2 + k ) para todo k 0 , e assim f ser
Se f injetiva, teremos f (n) = n + 1, pois f peridica a partir de n1 com periodo n2 n1. Da f

PROBLEMA 5: SOLUO DE JOSIAS ELIAS DOS SANTOS ROCHA (MURIBECA - SE)

[ n1 , + ) ) finito, f

[ n1 , + ) ) = {a1 ,..., ak } .

Suponhamos sem perda de generalidade que a1 max ai : i = 1,...k mas 2009m + 2008
EUREKA! N32, 2010

e que

f ( m ) = a1 com m > 0 e m > n1 (f peridica) f ( 2009m + 2008 ) = 2009a1 , [ n1 + ) f ( 2009m + 2008 ) f

[ n1 , + ) )

71

Sociedade Brasileira de Matemtica

2009 a1 a1 a1 = 0 a1 = 0 f
Note-se ainda que

[ n1 , + ) ) = {0}.
basta fazer

f ( 1) = 0

pois

n = 1

em

(ii) com

2009 f ( 1) = f ( 1) f ( 1) = 0.
peridica a partir de 1 e

Assim

f ( 1) = f ( n1 ) = 0 f

f ( n ) = r f ( 2009n + 2008) = 2009r f ( 2009 ( n + 1) ) = f ( 2009r ) .


ou

f ( n ) = 0, n 1. Seja
peridica a

n < 1,
partir

Assim, de

r = n +1

2009 ( n + 1) f

2009 ( n + 1) , + ) = {0}. Fazendo m1 = 2009 ( n + 1) e

ou

mk = 2009mk 1 + 2008, teremos 1 > m1 > m2 > ..., e alm disso temos

f ( mk ) = 0,k pois f ( m1 ) = 0 e f ( mk +1 ) = f ( 2009mk + 2008) = 2009 f ( mk ) = 0

se f ( mk ) = 0. Mas para todo n < 1 podemos achar mi e mi +1 tais que

mi +1 n < mi f ( n ) = 0, logo f ( n ) = 0, n. Assim temos apenas as seguintes


solues: (1) f ( n ) = n + 1, n . (3) f ( n ) = (2) f ( n ) = 0, n .

n + 1, se n 1 . 0, se n 1

PROBLEMA 6: SOLUO DA BANCA

Vamos inicialmente estender a funo f para [1,+) definindo f ( x) = f ( x ) , x [1,+) . Podemos agora mostrar que a funo f satisfaz a recorrncia seguinte:
x f ( x) = 1 + f ( ), x [1, +) . De fato, temos o vetor ( ) (correspondente a m m=2 k = 0 ), e, se k 1 e (a1 , a2 ,..., ak ) tal que a j inteiro, a j 2, j e

a1a2 ...ak x , ento 2 a1 x e, se a1 = m , a2 ...ak


possveis escolhas para (a2 ,..., ak ) .
EUREKA! N32, 2010

x x e h f m m

72

Sociedade Brasileira de Matemtica

Note tambm que < 2 , pois a funo ( ) =

m
m =1

decrescente e

m
m =1

1
2

= 1+

1 1 1 1 1 1 1 1 1 1 1 1 + + + + + +...<1+ + + + + + +... 4 9 16 25 36 49 4 4 16 16 16 16

=1+

Vamos resolver o item a): Mostraremos que f ( x ) x , x 1, por induo em

1 1 + +...= 2. 2 4

x x , m 2, x . Temos f ( x ) = 1 x , x (1, 2] . Para x 2 temos < m


e portanto, por hiptese de induo,
x x 1 x x f ( x ) = 1 + f 1 + = 1 + x 1 m m m=2 m=2 m x m >

(pois

m=2

1 = m =1 m

1 = 1).

1 x x x x x Como x > + + + + x +1 x + 2 x + 3 x + 4 x + 5 m> x m


2 2 2 2 2 2 2 2 2 2 + + + + > + + + + > 1, para 3 4 5 6 7 3 4 5 6 7 todo x 2, segue que f ( x ) < x , o que prova o resultado.
Vamos agora resolver o item b). Mostraremos inicialmente que 2 > 3. De fato,

2 2 2 2 2

1 1 1 1 1 1 1 2 4 1 1 1 1 + + + ... > + + + ... = + + + ... = 1 + 1 + 1 + ... 2 3 4 2 4 4 2 4 8 2 2 4 8 1 1 1 2 2 = 1 , donde 2 2 > 1 e logo 2 > 3. Vamos mostrar que, se = 1 1 1 2 2 2 3k 1 k 6 e x < 2k , ento f ( x ) cx k cx , onde c = o que k 3 92 20480 claramente implica o resultado. Note que essa desigualdade vale para k = 6 e x < 26 , pois nesse caso 1=
EUREKA! N32, 2010

73

Sociedade Brasileira de Matemtica

3k x x2 212 81 cx k = cx < 5c x = < < = 1 f ( x). 3 9 2k 17 4096 4096 4096

Vamos agora mostrar essa desigualdade por induo em k. Suponhamos que ela vale para um certo k 6.

3k +1 para todo x com 2k x < 2k +1. Para 3k +1 9 2k +1 x um tal valor de x, temos 1 < 2k para 2 m x , e logo m k x 2k 3k 3k 1 x 2 x x f ( x) =1+ f > f c k c = x 1 . k k k 3 9 2 m=2 m 3 9 2 m=2 m m=2 m m>2k m 1 1 1 1 1 1 1 Temos < = + +... + + ( k+1) +... + +... < k k k k + 1 + 2 m>2k m m2k m ( 2 ) ( 2 +1) ( 2 1) 2 ( 2 1)
Mostraremos que f ( x ) cx

<

2k 2 k +1 1 1 2 2 + + ... = k ( 1) + ( k +1)( 1) + ... < + k k +1) ( 2 2 2 2 3 3


k k +1

k +1

+ ...
k

2 3 2 2 2 1 pois 2 = > , e, como + + ... = 3 , temos 2 2 3 3 3 k 3k 3 k +1 2 pois > x , 1 3 f (x) > c k x c k +1 k +1 3 9 2k 3 3 9 2 k k k 3k 3k +1 2 3 3 2 (de fato, essa ltima = > 1 3 k k k +1 k +1 3k 9 2k 3 3 9 2 3 9 2 6 2k 9 2 k +1 > 1 + k +1 , que desigualdade pode ser escrita como 1 + k 3 9 2k 3 9 2 k +1 equivale a 6 3k +1 9 2k +1 > 18 3k 9 2k , que por sua vez equivale a

162 2k > 108 2k , que obviamente vale para todo k).


Obs: aproximadamente igual a 1,7286472389.... Klmar provou que lim
x

f ( x) x

1 = 0,3181736521... ( )

EUREKA! N32, 2010

74

Sociedade Brasileira de Matemtica

XXXI OLIMPADA BRASILEIRA DE MATEMTICA PREMIADOS


Nvel 1 (6. e 7. Anos)
NOME Alexandre Mendona Cardoso Daniel de Almeida Souza Pedro Henrique Alencar Costa Ana Beatriz Motta Arago Cortez Cristhian Mafalda rika Rizzo Aquino Bianca Lima Barretto Adriana de Sousa Figueiredo Ricardo Ken Wang Tsuzuki Joo Pedro Sedeu Godoi Leonardo Gomes Gonalves Leonardo Gushiken Yoshitake Paulo Henrique Omena de Freitas Edgar Kenji Ishida Rodrigo Pommot Berto Kiane Sassaki Menezes Dimas Macedo de Albuquerque Mauricio Najjar da Silveira Murilo Corato Zanarella Victor Almeida Costa Elcio Koodiro Yoshida Carolina Lima Guimares Bruno da Silveira Dias Emilly Guaris Costa Bruno Almeida Costa Gabriel Averbug Zukin Marcelo Ericsson de Carvalho Sarah Barreto Ornellas Isabella Ayres Pinheiro de Lima Shadi Bavar Viviane Silva Souza Freitas Matheus Jos Arajo Oliveira Beatriz Miranda Macedo Matheus Ucha Constante Julio S. Akiyoshi Antonio Wesley de Brito Vieira Vinicius Jras Padro Mateus Guimares Lima de Freitas Vitor Dias Gomes Barrios Marin Mariana Teatini Ribeiro Rodrigo Silva Ferreira Artur Souto Martins Tiago Martins Npoli
EUREKA! N32, 2010

CIDADE ESTADO Salvador BA Braslia DF Fortaleza CE Campinas SP Leme SP Goinia GO Salvador BA Porto Alegre RS So Paulo SP Rio de Janeiro RJ Braslia DF So Paulo SP So Paulo SP So Paulo SP Braslia DF Rio de Janeiro RJ Fortaleza CE So Paulo SP Amparo SP Fortaleza CE So Paulo SP Vitria ES Florianpolis SC Macei AL Fortaleza CE Rio de Janeiro RJ So Paulo SP Salvador BA Goinia GO Blumenau SC Salvador BA Recife PE Niteri RJ Goinia GO So Paulo SP Cocal de Alves PI Rio de Janeiro RJ Fortaleza CE Presidente Prudente SP Belo Horizonte MG Salvador BA Fortaleza CE It SP

PRMIO Ouro Ouro Ouro Ouro Ouro Ouro Prata Prata Prata Prata Prata Prata Prata Prata Prata Prata Prata Bronze Bronze Bronze Bronze Bronze Bronze Bronze Bronze Bronze Bronze Bronze Bronze Bronze Bronze Bronze Bronze Bronze Meno Honrosa Meno Honrosa Meno Honrosa Meno Honrosa Meno Honrosa Meno Honrosa Meno Honrosa Meno Honrosa Meno Honrosa

75

Sociedade Brasileira de Matemtica


Las Monteiro Pinto Guilherme Anitele Silva Pedro Papa Paniago Gabriel Yudi Hirata Iago Carvalho de Moraes Adam Yuuki Oyama Luze Mello Durso Vianna Enrico Pascucci Loffel Daniel Charles M. Gomes Ellen Tamie Ikefuti Morishigue Ana Emlia Hernandes Dib Marcelo Liu Guo Gabriel Queiroz Moura Gabriel Branco Frizzo Ana Jssyca Mendes Belarmino Mariana Bonfim Moraes Morant de Holanda Ricardo Vidal Mota Peixoto Bruno de Marchi Andrade Juliana Amoedo Amoedo Plcido Rio de Janeiro RJ Presidente Prudente SP Belo Horizonte MG So Paulo SP Recife PE Curitiba PR Rio de Janeiro RJ S.B.do Campo SP Mogi das Cruzes SP Bastos SP S.J. do Rio Preto SP So Paulo SP Teresina PI Curitiba PR Fortaleza CE Rio de Janeiro RJ Vassouras RJ Valinhos SP Salvador BA Meno Honrosa Meno Honrosa Meno Honrosa Meno Honrosa Meno Honrosa Meno Honrosa Meno Honrosa Meno Honrosa Meno Honrosa Meno Honrosa Meno Honrosa Meno Honrosa Meno Honrosa Meno Honrosa Meno Honrosa Meno Honrosa Meno Honrosa Meno Honrosa Meno Honrosa

Nvel 2 (8. e 9. Anos)


NOME Andr Macieira Braga Costa Gabriel Ilharco Magalhes Daniel Eiti Nishida Kawai Henrique Gasparini Fiuza do Nascimento Marina Pessoa Mota Fellipe Sebastiam S. P. Pereira Liara Guinsberg Lara Timb Arajo Victor Kioshi Higa Fernando Lima Saraiva Filho Lucas Cawai Julio Pereira Mateus Henrique Ramos de Souza Henrique Vieira Gonalves Vaz Lucas Nishida Pedro Vctor Falci de Rezende Rafael Kazuhiro Miyazaki Francisco Markan Nobre de Souza Filho Vincent Cherng Hsi Lee Vincius Canto Costa Thiago Poeiras Silva Victor de Oliveira Bitaraes Victor Hugo Corra Rodrigues Luciano Drozda Dantas Martins Breno Lev Corra CIDADE ESTADO Belo Horizonte MG Juiz de Fora MG Atibaia SP Braslia DF Fortaleza CE Rio de Janeiro RJ So Paulo SP Fortaleza CE So Paulo SP Eusbio CE Caucaia CE Pirapora MG So Paulo SP Pedreira SP Juiz de Fora MG So Paulo SP Fortaleza CE So Paulo SP Salvador BA Belo Horizonte MG Betim MG Rio de Janeiro RJ Fortaleza CE Campo Belo MG PRMIO Ouro Ouro Ouro Ouro Ouro Ouro Prata Prata Prata Prata Prata Prata Prata Prata Prata Prata Bronze Bronze Bronze Bronze Bronze Bronze Bronze Bronze

EUREKA! N32, 2010

76

Sociedade Brasileira de Matemtica


Wilson Aparecido Sedano Filho Victor Venturi Daniel Lima Santanelli Felipe Penha Alves Lucas Grimauth Evangelista Gabriel Nogueira Coelho de Togni de Souza Ana Thais Castro de Santana Caio Cesar do Prado Dorea Reis Rafael Rodrigues Rocha de Melo Murilo Freitas Yonashiro Coelho Gabriel Jos Moreira da Costa Silva Nathalia Novello Fernandes Ribeiro Gabriel Pacianotto Gouveia Pedro Ivo Colho de Arajo Elias Brito Oliveira Fernando Tomimura Miyashiro Igor Augusto Marques do Carmo Juliane Trianon Fraga Aim Parente de Sousa Tadeu Pires de Matos Belfort Neto Yuri Zeniti Sinzato Gabriel Sena Galvo Filipe Mouro Leite Gabriela Loiola Vilar Marcelo Cargnelutti Rossato Pedro Henrique Botolozo Maria Jair Gomes Soares Jnior Maria Clara Cardoso Jlio Csar Prado Soares Fbio Kenji Arai Julio Barros de Paula Francisco Matheus Gonalves de Souza Daniel Kantorowitz Ivan Tadeu Ferreira Antunes Filho Vitor Ramos de Paula Victor Santos de Andrade Paulnia SP Campinas SP Rio de Janeiro RJ So Lus MA So Paulo SP Rio de Janeiro RJ Rio de Janeiro RJ Nova Iguau RJ Caucaia CE So Paulo SP Macei AL Rio de Janeiro RJ So Paulo SP Fortaleza CE Braslia DF So Paulo SP Juiz de Fora MG So Paulo SP Fortaleza CE Fortaleza CE Braslia DF Braslia DF Teresina PI Fortaleza CE Santa Maria RS Curitiba PR Montes Claros MG So Paulo SP Braslia DF So Paulo SP Taubat SP Joo Pessoa PB Bragana Paulista SP Lins SP Belo Horizonte MG Teresina PI Bronze Bronze Bronze Bronze Bronze Bronze Bronze Bronze Meno Honrosa Meno Honrosa Meno Honrosa Meno Honrosa Meno Honrosa Meno Honrosa Meno Honrosa Meno Honrosa Meno Honrosa Meno Honrosa Meno Honrosa Meno Honrosa Meno Honrosa Meno Honrosa Meno Honrosa Meno Honrosa Meno Honrosa Meno Honrosa Meno Honrosa Meno Honrosa Meno Honrosa Meno Honrosa Meno Honrosa Meno Honrosa Meno Honrosa Meno Honrosa Meno Honrosa Meno Honrosa

Nvel 3 (Ensino Mdio)


NOME Renan Henrique Finder Marcelo Tadeu de S Oliveira Sales Davi Lopes Alves de Medeiros Matheus Secco Torres da Silva Hugo Fonseca Arajo Marco Antonio Lopes Pedroso Gustavo Lisba Empinotti CIDADE ESTADO So Paulo SP Salvador BA Fortaleza CE Rio de Janeiro RJ Rio de Janeiro RJ Santa Isabel SP Florianpolis SC PRMIO Ouro Ouro Ouro Ouro Ouro Ouro Prata

EUREKA! N32, 2010

77

Sociedade Brasileira de Matemtica


Marlen Lincoln da Silva Deborah Barbosa Alves Illan Feiman Halpern Thiago Ribeiro Ramos Hanon Guy Lima Rossi Joo Lucas Camelo S Carlos Henrique de Andrade Silva Custdio Moreira Brasileiro Silva Rafael Alves da Ponte Matheus Barros de Paula Bruno Silva Mucciaccia Matheus Araujo Marins Guilherme da Rocha Dahrug Victorio Takahashi Chu Voltaire Laplace dos Reis Jardiel Freitas Cunha Rafael Horimoto de Freitas Lucas de Freitas Smaira Robrio Soares Nunes Gabriel Milito Vinhas Lopes Alvaro Lopes Pedroso Alan Anderson da Silva Pereira Maria Clara Mendes Silva Nara Gabriela de Mesquita Peixoto Rodrigo de Sousa Serafim da Silva Rodrigo Rolim Mendes de Alencar Joo Mendes Vasconcelos Otvio Augusto de Oliveira Mendes Renan Roveri do Amaral Gurgel Fernando Fonseca Andrade Oliveira Caque Porto Lira Gustavo Haddad Francisco e S. Braga Gustavo Cellet Marques Wagner Rosales Chaves Wagner Carlos Morto Loyola Filho James Jun Hong Kayo de Frana Gurgel Nathana Alcntara Lima Gabriel Lima Guimares Ivan Guilhon Mitoso Rocha Elder Massahiro Yoshida Ruan Alves Pires Andr Saraiva Nobre dos Santos Luiz Filipe Martins Ramos Felipe Abella C. Mendona de Souza Eduardo Machado Capaverde Thales Sinelli Lima Tuane Viana Pinheiro Vincius Cipriano Klein Fortaleza CE So Paulo SP So Paulo SP Varginha MG So Paulo SP Fortaleza CE Fortaleza CE Embu SP Fortaleza CE Taubat SP Vitria ES So Gonalo RJ Santo Andr SP So Paulo SP Manhuau MG Recife PE So Paulo SP Guaxup MG Riberio Preto SP Fortaleza CE Santa Isabel SP Unio dos Palmares AL Pirajuba MG Fortaleza CE Itatiba SP Fortaleza CE Fortaleza CE Pilar do Sul SP Jundia SP Belo Horizonte MG Fortaleza CE S.J.dos Campos SP So Paulo SP Jundia SP Vitria ES So Paulo SP Fortaleza CE Fortaleza CE Vitria ES Fortaleza CE So Paulo SP Rio de Janeiro RJ Fortaleza CE Niteri RJ Joo Pessoa PB Florianpolis SC So Paulo SP Rio de Janeiro RJ Viosa MG Prata Prata Prata Prata Prata Prata Prata Prata Prata Bronze Bronze Bronze Bronze Bronze Bronze Bronze Bronze Bronze Bronze Bronze Bronze Bronze Bronze Bronze Bronze Meno Honrosa Meno Honrosa Meno Honrosa Meno Honrosa Meno Honrosa Meno Honrosa Meno Honrosa Meno Honrosa Meno Honrosa Meno Honrosa Meno Honrosa Meno Honrosa Meno Honrosa Meno Honrosa Meno Honrosa Meno Honrosa Meno Honrosa Meno Honrosa Meno Honrosa Meno Honrosa Meno Honrosa Meno Honrosa Meno Honrosa Meno Honrosa

EUREKA! N32, 2010

78

Sociedade Brasileira de Matemtica


Andr Austregsilo Scussel Thiago Augusto da Silva Baleixo Marcos Massayuki Kawakami Ana Lusa de Almeida Losnak Fortaleza CE Rio de Janeiro RJ So Paulo SP So Paulo SP Meno Honrosa Meno Honrosa Meno Honrosa Meno Honrosa

Nvel Universitrio
NOME Leonardo Ribeiro de Castro Carvalho Rgis Prado Barbosa Rafael Assato Ando Rafael Daigo Hirama Guilherme Rodrigues Nogueira de Souza Adenilson Arcanjo de Moura Junior Felipe Rodrigues Nogueira de Souza Ramon Moreira Nunes Thoms Yoiti Sasaki Hoshina Rafael Tupynamb Dutra Lus Fernando Schultz Xavier da Silveira Thiago Costa Leite Santos Gabriel Ponce Carlos Henrique Melo Souza Marcelo Matheus Gauy Antnio Felipe Cavalcante Carvalho Kellem Corra Santos Rafael Montezuma Pinheiro Cabral Ricardo Turolla Bortolotti Alexandre Hideki Deguchi Martani Rafael Sampaio de Rezende Maurcio de Lemos Rodrigues Collares Neto Joas Elias dos Santos Rocha Gabriel Lus Mello Dalalio Carlos Coelho Lechner Enzo Haruo Hiraoka Moriyama Paulo Srgio de Castro Moreira Helder Toshiro Susuki Mateus Oliveira de Figueiredo Gabriel Caser Brito Paulo Andr Carvalho de Melo Caio Ishizara Costa Willy George do Amaral Petrenko Sidney Cerqueira Bispo dos Santos Filho Rafael Endlich Pimentel Guilherme Philippe Figueiredo Bruno da Silva Santos Francisco Osman Pontes Neto Luty Rodrigues Ribeiro CIDADE ESTADO So Paulo SP Fortaleza CE Campinas SP Campinas SP So Paulo SP Fortaleza CE So Paulo SP Fortaleza CE Rio de Janeiro RJ Belo Horizonte MG Florianpolis SC So Paulo SP So Carlos SP S.J. dos Campos SP So Paulo SP Fortaleza CE Rio de Janeiro RJ Fortaleza CE Rio de Janeiro RJ So Paulo SP Fortaleza CE Aracaj SE Muribeca SE S.J. dos Campos SP Rio de Janeiro RJ So Paulo SP Fortaleza CE So Paulo SP Fortaleza CE Rio de Janeiro RJ S.J. dos Campos SP S.J. dos Campos SP Rio de Janeiro RJ S.J. dos Campos SP Vitria ES So Paulo SP Belford Roxo RJ Fortaleza CE S.J. dos Campos SP PRMIO Ouro Ouro Ouro Ouro Ouro Ouro Ouro Ouro Prata Prata Prata Prata Prata Prata Prata Prata Prata Prata Bronze Bronze Bronze Bronze Bronze Bronze Bronze Bronze Bronze Bronze Bronze Bronze Bronze Bronze Bronze Bronze Bronze Bronze Bronze Bronze Bronze

EUREKA! N32, 2010

79

Sociedade Brasileira de Matemtica


Renato Rebouas de Medeiros Jos Olegrio de Oliveira Neto Eduardo Fischer Guilherme Loureno Mejia Jorge Henrique Craveiro de Andrade Edson Augusto Bezerra Lopes Eric Campos Bastos Guedes Thiago da Silva Pinheiro Leandro Farias Maia Pedro Paulo Albuquerque Goes lvaro Krger Ramos Alysson Espndola de S Silveira Alfredo Roque de Oliveira Freire Filho Rafael Ghussn Cano Antnio Deromir Neves da Silva Jnior Rafael Sabino Lima Hudson do Nascimetno Lima Diego Andrs de Barros Lima Barbosa Reinan Ribeiro Souza Santos Marcos Victor Pereira Vieira Daniel Ungaretti Borges Fortaleza CE S.J. dos Campos SP Encantado RS S.J. dos Campos SP Rio de Janeiro RJ Fortaleza CE Niteri RJ So Paulo SP Fortaleza CE Fortaleza CE Porto Alegre RS Fortaleza CE S.J. dos Campos SP Campinas SP Fortaleza CE Rio de Janeiro RJ Fortaleza CE Rio de Janeiro RJ Aracaju SE Fortaleza CE Belo Horizonte MG Meno Honrosa Meno Honrosa Meno Honrosa Meno Honrosa Meno Honrosa Meno Honrosa Meno Honrosa Meno Honrosa Meno Honrosa Meno Honrosa Meno Honrosa Meno Honrosa Meno Honrosa Meno Honrosa Meno Honrosa Meno Honrosa Meno Honrosa Meno Honrosa Meno Honrosa Meno Honrosa Meno Honrosa

EUREKA! N32, 2010

80

Sociedade Brasileira de Matemtica

AGENDA OLMPICA
XXXII OLIMPADA BRASILEIRA DE MATEMTICA
NVEIS 1, 2 e 3 Primeira Fase Sbado, 12 de junho de 2010 Segunda Fase Sbado, 18 de setembro de 2010 Terceira Fase Sbado, 16 de outubro de 2010 (nveis 1, 2 e 3) Domingo, 17 de outubro de 2010 (nveis 2 e 3 - segundo dia de prova). NVEL UNIVERSITRIO Primeira Fase Sbado, 18 de setembro de 2010 Segunda Fase Sbado, 16 e Domingo, 17 de outubro de 2010

ASIAN PACIFIC MATH OLYMPIAD (APMO)


06 de maro de 2010

XVI OLIMPADA DE MAIO


08 de maio de 2010

XXI OLIMPADA DE MATEMTICA DO CONE SUL


13 a 19 de junho de 2010 guas de So Pedro, SP Brasil

LI OLIMPADA INTERNACIONAL DE MATEMTICA


02 a 14 de julho de 2010 Astana, Cazaquisto

XVII OLIMPADA INTERNACIONAL DE MATEMTICA UNIVERSITRIA


24 a 30 de julho de 2010 Blagoevgrad, Bulgria

XXIV OLIMPADA IBEROAMERICANA DE MATEMTICA


17 a 27 de setembro de 2010 Paraguai

II COMPETIO IBEROAMERICANA INTERUNIVERSITRIA DE MATEMTICA


3 a 9 de outubro de 2010 Rio de Janeiro, Brasil

XIII OLIMPADA IBEROAMERICANA DE MATEMTICA UNIVERSITRIA

EUREKA! N32, 2010

81

Sociedade Brasileira de Matemtica

COORDENADORES REGIONAIS
Alberto Hassen Raad Amrico Lpez Glvez Andreia Goldani Antonio Carlos Nogueira Benedito Tadeu Vasconcelos Freire Carmen Vieira Mathias Claus Haetinger Cludio de Lima Vidal Denice Fontana Nisxota Menegais Disney Douglas Lima de Oliveira Edson Roberto Abe Edney Aparecido Santulo Jr. lio Mega Eudes Antonio da Costa Fbio Brochero Martnez Florncio Ferreira Guimares Filho Francinildo Nobre Ferreira Genildo Alves Marinho Graziela de Souza Sombrio Gilson Tumelero Ivanilde Fernandes Saad Joo Bencio de Melo Neto Joo Francisco Melo Libonati Jose de Arimatia Fernandes Jos Luiz Rosas Pinho Jos Vieira Alves Jos William Costa Krerley Oliveira Licio Hernandes Bezerra Luciano G. Monteiro de Castro Luzinalva Miranda de Amorim Marcelo Rufino de Oliveira Marcelo Mendes Newman Simes Nivaldo Costa Muniz Nivaldo de Ges Grulha Jr. Osnel Broche Cristo Uberlndio Batista Severo Raul Cintra de Negreiros Ribeiro Ronaldo Alves Garcia Rogrio da Silva Igncio Reginaldo de Lima Pereira Reinaldo Gen Ichiro Arakaki Ricardo Amorim Srgio Cludio Ramos Seme Gebara Neto Tadeu Ferreira Gomes Toms Menndez Rodrigues Valdenberg Arajo da Silva Vnia Cristina Silva Rodrigues Wagner Pereira Lopes (UFJF) (USP) FACOS (UFU) (UFRN) (UNIFRA) (UNIVATES) (UNESP) (UNIPAMPA) (UFAM) (Colgio Objetivo de Campinas) (UEM) (Grupo Educacional Etapa) (Univ. Federal do Tocantins) (UFMG) (UFES) (UFSJ) (Centro Educacional Leonardo Da Vinci) (UNOCHAPEC) (UTFPR) (UC. Dom Bosco) (UFPI) (Grupo Educacional Ideal) (UFPB) (UFSC) (UFPB) (Instituto Pueri Domus) (UFAL) (UFSC) (Sistema Elite de Ensino) (UFBA) (Grupo Educacional Ideal) (Colgio Farias Brito, Pr-vestibular) (Cursinho CLQ Objetivo) (UFMA) (USP So Carlos) (UFLA) (UFPB)) (Colgio Anglo) (UFGO) (Col. Aplic. da UFPE) (Escola Tcnica Federal de Roraima) (UNIFESP) (Centro Educacional Logos) (IM-UFRGS) (UFMG) (UEBA) (U. Federal de Rondnia) (U. Federal de Sergipe) (U. Metodista de SP) (CEFET GO) Juiz de Fora MG Ribeiro Preto SP Osrio RS Uberlndia MG Natal RN Santa Mara RS Lajeado RS S.J. do Rio Preto SP Bag RS Manaus AM Campinas SP Maring PR So Paulo SP Arraias TO Belo Horizonte MG Vitria ES So Joo del Rei MG Taguatingua DF Chapec SC Pato Branco PR Campo Grande MS Teresina PI Belm PA Campina Grande PB Florianpolis SC Campina Grande PB Santo Andr SP Macei AL Florianpolis SC Rio de Janeiro RJ Salvador BA Belm PA Fortaleza CE Piracicaba SP So Luis MA So Carlos SP Lavras MG Joo Pessoa PB Atibaia SP Goinia GO Recife PE Boa Vista RR SJ dos Campos SP Nova Iguau RJ Porto Alegre RS Belo Horizonte MG Juazeiro BA Porto Velho RO So Cristovo SE S.B. do Campo SP Jata GO

EUREKA! N32, 2010

82

CONTEDO
XVI OLIMPADA DE MAIO Enunciados e resultado brasileiro XXI OLIMPADA DE MATEMTICA DO CONE SUL Enunciados e resultado brasileiro LI OLIMPADA INTERNACIONAL DE MATEMTICA (IMO) Enunciados e resultado brasileiro XXV OLIMPADA IBEROAMERICANA DE MATEMTICA Enunciados e resultado brasileiro ARTIGOS ASSOCIANDO UM TRIGONOMTRICAS Marclio Miranda POLINMIO A EXPRESSES ALGBRICAS E 11 18 31 33 42 45 59 61 62 2 5 7 9

SOMAS TRIGONOMTRICAS: DE PROSTAFRESE A FRMULA DE EULER Rogrio Possi Junior UMA INTERESSANTE DEDUO PARA A FRMULA DE HERO Flvio Antonio Alves RAZES DA UNIDADE Anderson Torres & Eduardo Tengan COMO QUE FAZ SOLUES DE PROBLEMAS PROPOSTOS PROBLEMAS PROPOSTOS AGENDA OLMPICA COORDENADORES REGIONAIS

Sociedade Brasileira de Matemtica

XVI OLIMPADA DE MAIO


PRIMEIRO NVEL
PROBLEMA 1

Um recipiente fechado com formato de paraleleppedo retangular contm 1 litro de gua. Se o recipiente se apoia horizontalmente sobre trs faces distintas, o nvel da gua de 2cm, 4cm e 5cm. Calcule o volume do paraleleppedo.
PROBLEMA 2

Na etapa 0 escrevem-se os nmeros 1, 1. Na etapa 1 intercala-se a soma dos nmeros 1, 2, 1. Na etapa 2 entre cada par de nmeros da etapa anterior intercala-se a soma deles: 1, 3, 2, 3, 1. Uma etapa mais: 1, 4, 3, 5, 2, 5, 3, 4, 1. Quantos nmeros h na etapa 10? Qual a soma de todos os nmeros que h na etapa 10?
PROBLEMA 3

possvel pintar os inteiros positivos com trs cores de modo que, sempre que se somam dois nmeros de cores distintas, o resultado da soma seja da terceira cor? (H que usar as trs cores.) Se a resposta afirmativa, indique um possvel modo de pintar; se no possvel, explique o porqu.
PROBLEMA 4

Encontre todos os nmeros naturais de 90 dgitos que so mltiplos de 13 e tm os primeiros 43 dgitos iguais entre si e distintos de zero, os ltimos 43 dgitos iguais entre si, e os 4 dgitos do meio so 2, 0, 1, 0, nessa ordem.
PROBLEMA 5

Num tabuleiro de 2 7 quadriculado em casas de 1 1 se consideram os 24 pontos que so vrtices das casas. Joo e Matias jogam sobre este tabuleiro. Joo pinta de vermelho uma quantidade igual de pontos em cada uma das trs linhas horizontais. Se Matias pode escolher trs pontos vermelhos que sejam vrtices de um tringulo acutngulo, Matias vence o jogo. Qual a mxima quantidade de pontos que Joo pode pintar para ter certeza de que Matias no vencer? (Para o nmero encontrado, d um exemplo de pintura que impea que Matias vena e justifique por qu Matias vence sempre se o nmero maior.)
EUREKA! N33, 2011

Sociedade Brasileira de Matemtica

SEGUNDO NVEL
PROBLEMA 1

Determine o menor inteiro positivo que tenha todos seus dgitos iguais a 4, e que seja mltiplo de 169.
PROBLEMA 2

Consideramos o retngulo ABCD e a circunferncia de centro D e raio DA, que corta o prolongamento do lado AD no ponto P. A reta PC corta a circunferncia no ponto Q e o prolongamento do lado AB no ponto R. Demonstre que QB = BR.
PROBLEMA 3

Encontre o menor k > 2 para o qual existem k nmeros inteiros consecutivos, tais que a soma dos seus quadrados um quadrado.
PROBLEMA 4

Seja n um inteiro tal que 1 < n < 2010. Dado um polgono regular de 2010 lados e n moedas, devemos pintar os vrtices do polgono utilizando n cores dadas, e logo colocar as n moedas em n vrtices do polgono. Em seguida, a cada segundo, todas as moedas se deslocam para o vrtice vizinho, girando no sentido dos ponteiros do relgio. Determine os valores de n para os quais possvel pintar e escolher as posies iniciais das moedas, de forma que em todo momento as n moedas estejam todas em vrtices de cores distintas.
PROBLEMA 5

Temos as seguintes peas: um retngulo de 4 1, dois retngulos de 3 1, trs retngulos de 2 1 e quatro quadrados de 1 1. Ariel e Bernardo jogam o seguinte jogo num tabuleiro de n n , onde n um nmero escolhido por Ariel. A cada movimento, Bernardo recebe de Ariel uma pea R. Em seguida Bernardo analisa se poder colocar R no tabuleiro de modo que no tenha pontos em comum com nenhuma das peas colocadas anteriormente (nem sequer um vrtice em comum). Se existe uma tal colocao para R, Bernardo deve escolher uma delas e colocar R.
EUREKA! N33, 2011

Sociedade Brasileira de Matemtica

O jogo para se impossvel colocar R da forma explicada, e Bernardo vence. Ariel vence somente se esto colocadas as 10 peas no tabuleiro. a) Suponhamos que Ariel d as peas a Bernardo em ordem decrescente de tamanho. Qual o menor n que garante a vitria do Ariel? b) Para o n encontrado em a), se Bernardo recebe as peas em ordem crescente de tamanho. Ariel tem garantida a vitria?
ESCLARECIMENTO: cada pea deve cobrir exatamente um nmero de quadrados

unitrios do tabuleiro igual ao seu prprio tamanho. Os lados das peas podem coincidir com partes da borda do tabuleiro.
RESULTADO BRASILEIRO 2010: Nvel 1 (at 13 anos)
Nome Murilo Corato Zanarella Daniel de Almeida Souza Viviane Silva Souza Freitas Carolina Lima Guimares Pedro Henrique Alencar Costa Samuel Brasil de Albuquerque Juliana Amoedo Plcido Lucca Morais de Arruda Siaudjionis Antonio Wesley de Brito Vieira Cidade Estado Amparo SP Braslia DF Salvador BA Vitria ES Fortaleza CE Fortaleza CE Salvador BA Fortaleza CE Cocal dos Alves PI Prmio Medalha de Ouro Medalha de Prata Medalha de Prata Medalha de Bronze Medalha de Bronze Medalha de Bronze Medalha de Bronze Meno Honrosa Meno Honrosa

2010: Nvel 2 (at 15 anos)


Nome Rafael Kazuhiro Miyazaki Lucas Cauai Julio Pereira Pedro Ivo Colho de Arajo Francisco Markan Nobre de Souza Filho Fellipe Sebastiam da Silva Paranhos Pereira Tadeu Pires de Matos Belfort Neto Henrique Gasparini Fiuza do Nascimento Rafael Rodrigues Rocha de Melo Mateus Henrique Ramos de Souza Cidade Estado So Paulo SP Caucaia CE Caucaia CE Fortaleza CE Rio de Janeiro RJ Fortaleza CE Braslia DF Caucaia CE Pirapora MG Prmio Medalha de Ouro Medalha de Prata Medalha de Prata Medalha de Bronze Medalha de Bronze Medalha de Bronze Medalha de Bronze Meno Honrosa Meno Honrosa

EUREKA! N33, 2011

Sociedade Brasileira de Matemtica

XXI OLIMPADA DE MATEMTICA DO CONE SUL


Enunciados e resultado brasileiro
O Brasil, e particularmente o Estado de So Paulo teve a honra de sediar a 21 Olimpada de Matemtica do Cone Sul, que aconteceu at o dia 19 de junho na cidade de guas de So Pedro, SP. A equipe foi liderada pelos professores Francisco Bruno Holanda, de Fortaleza CE e Tertuliano Franco Santos Franco, de Rio de Janeiro RJ.
RESULTADOS DA EQUIPE BRASILEIRA BRA1 BRA2 BRA3 BRA4 Joo Lucas Camelo S Gabriel Milito Vinhas Lopes Maria Clara Mendes Silva Caque Porto Lira Medalha de Ouro Medalha de Prata Medalha de Prata Medalha de Bronze

PRIMEIRO DIA
PROBLEMA 1

Pedro tem que escolher duas fraes irrredutveis, cada uma com numerador e denominador positivos, tais que: A soma das duas fraes seja igual a 2. A soma dos numeradores das duas fraes seja igual a 1000.

De quantas maneiras Pedro pode fazer isso?


PROBLEMA 2

Marcam-se em uma reta 44 pontos, numerados 1, 2, 3, ..., 44 da esquerda para a direita. Vrios grilos saltam na reta. Cada grilo parte do ponto 1, salta por pontos marcados e termina no ponto 44. Alm disso, cada grilo sempre salta de um ponto marcado a outro marcado com um nmero maior. Quando todos os grilos terminaram da saltar, notou-se que para cada par i, j, com 1 i j 44, h um grilo que saltou diretamente do ponto i para o ponto j, sem pousar em nenhum dos pontos entre eles. Determine a menor quantidade de grilos para que isso seja possvel.
EUREKA! N33, 2011

Sociedade Brasileira de Matemtica

PROBLEMA 3

Recortar um polgono convexo de n lados significa escolher um par de lados consecutivos AB, BC do polgono e substitu-los por trs segmentos AM, MN, e NC, sendo M o ponto mdio de AB e N o ponto mdio de BC. Em outras palavras, corta-se o tringulo MBN e obtem-se um polgono convexo de n + 1 lados. Seja P6 um hexgono regular de rea 1. Recorta-se P6 e obtm-se o polgono P7. Ento recorta-se P7, de uma das sete maneiras possveis, e obtm-se o polgono P8, e assim sucessivamente. Prove que, independentemente de como sejam feitos os recortes, a rea de Pn sempre maior do que 2 3. SEGUNDO DIA
PROBLEMA 4

Pablo e Slvia jogam em um tabuleiro 2010 2010. Primeiro Pablo escreve um nmero inteiro em cada casa. Feito isso, Slvia repete tantas vezes quanto quiser a seguinte operao: escolher trs casas que formem um L, como uma figura, e somar 1 a cada nmero dessas trs casas. Slvia ganha se fizer com que todos os nmeros do tabuleiro sejam mltiplos de 10. Demonstre que Slvia sempre pode escolher uma sequncia de operaes com as quais ela ganha o jogo.

PROBLEMA 5

O incrculo do tringulo ABC toca os lados BC, CA, e AB em D, E e F, respectivamente. Sejam a , b e c os circuncrculos dos tringulos EAF, DBF e DCE, respectivamente. As retas DE e DF cortam a em Ea E e Fa F , respectivamente. Seja rA a reta Ea Fa . Defina rB e rC de modo anlogo. Prove que as retas rA , rB e rC determinam um tringulo cujos vrtices pertencem aos lados do tringulo ABC.
PROBLEMA 6

Determine se existe uma sequncia infinita a0 , a1 , a2 , a3 ,... de inteiros no negativos que satisfaz as seguintes condiciones: (i) Todos os nmeros inteiros no negativos aparecem na sequncia uma nica vez; (ii) A sequncia bn = an + n, n 0, formada por todos os nmeros primos, cada um aparecendo uma nica vez.
EUREKA! N33, 2011

Sociedade Brasileira de Matemtica

LI OLIMPADA INTERNACIONAL DE MATEMTICA (IMO)


Enunciados e resultado Brasileiro
A LI Olimpada Internacional de Matemtica (IMO) foi realizada na cidade de Astana, Cazaquisto entre os dias 2 e 14 de julho de 2010. A equipe foi liderada pelos professores Edmilson Luis Rodrigues Motta, de So Paulo SP e Marcelo Mendes de Oliveira, de Fortaleza CE.
RESULTADOS DA EQUIPE BRASILEIRA BRA1 BRA2 BRA3 BRA4 BRA5 BRA6 Marcelo Tadeu de S Oliveira Sales Matheus Secco Torres da Silva Gustavo Lisba Empinotti Deborah Barbosa Alves Hanon Lima Rossi Joo Lucas Camelo S Medalha de Prata Medalha de Prata Medalha de Bronze Meno Honrosa Meno Honrosa Meno Honrosa

PRIMEIRO DIA
PROBLEMA 1

Determine todas as funes f :

tais que

f ( y) = f ( x) f ( y) x para os nmeros x, y . ( z designa o maior inteiro que menor ou igual a z).


PROBLEMA 2

Seja ABC um tringulo, I o seu incentro e a sua circunferncia circunscrita. A recta AI intersecta novamente no ponto D. Sejam E um ponto do arco BDC e F um ponto do lado BC tais que 1 B AF = C AE < B AC. 2 Seja G o ponto mdio do segmento IF. Mostre que as rectas DG e EI se intersectam sobre .
PROBLEMA 3

Seja * o conjunto dos inteiros positivos. Determine todas as funes g : * * tais que

EUREKA! N33, 2011

Sociedade Brasileira de Matemtica

( g ( m) + n ) ( m + g ( n))
um quadrado perfeito para todos m, n *. SEGUNDO DIA Seja a circunferncia circunscrita ao tringulo ABC e P um ponto no interior do tringulo. As rectas AP, BP e CP intersectam novamente nos pontos K, L, e M, respectivamente. A recta tangente a em C intersecta a recta AB em S. Supondo que SC = SP, mostre que MK = ML.
PROBLEMA 5 PROBLEMA 4

Em cada uma das seis caixas B1 , B2 , B3 , B4 , B5 , B6 h inicialmente s uma moeda. Dois tipos de operaes so possveis:
Tipo 1: Escolher uma caixa no vazia B j , com 1 j 5. Retirar uma moeda da B j e

adicionar duas moedas a B j +1.


Tipo 2: Escolher uma caixa no vazia Bk , com 1 k 4. Retirar uma moeda da Bk e

trocar os contedos das caixas (possivelmente vazias) Bk +1 e Bk + 2 . Determine se existe uma sucesso finita destas operaes que deixa as caixas

B1 , B2 , B3 , B4 , B5 vazias e a caixa B6 com exactamente 20102010


que a b = a
c

2010

moedas. (Observe

(b ) . )
c

PROBLEMA 6

Seja a1 , a2 , a3 ,... uma sucesso de nmeros reais positivos. Sabe-se que para algum inteiro positivo s, an = max {ak + an k tal que 1 k n 1} para todo n > s. Mostre que existem inteiros positivos e N, com s, tais que an = a + an para todo n N .

EUREKA! N33, 2011

Sociedade Brasileira de Matemtica

XXV OLIMPADA IBEROAMERICANA DE MATEMTICA


Enunciados e resultado Brasileiro
A XXV Olimpada Iberoamericana de Matemtica foi realizada na cidade de Assuno, Paraguai no perodo de 20 a 30 de setembro de 2010. A equipe brasileria foi liderada pelos professores Onofre Campos, de Fortaleza CE e Luzinalva Miranda de Amorim, de Salvador BA. A equipe brasileira ficou em primeiro lugar na soma dos pontos dos participantes.

RESULTADOS DA EQUIPE BRASILEIRA BRA1 BRA2 BRA3 BRA4 Marcelo Tadeu de S Oliveira Sales Deborah Barbosa Alves Matheus Secco Torres da Silva Gustavo Lisboa Empinotti Medalha de Ouro Medalha de Ouro Medalha de Ouro Medalha de Prata

PRIMEIRO DIA
PROBLEMA 1

Numa fila de dez moedas indistinguveis h duas delas que so falsas, ocupando posies consecutivas. Para cada conjunto de posies, pode-se perguntar quantas moedas falsas ele contm. possvel determinar quais so as moedas falsas fazendo apenas duas destas perguntas? No se sabe a resposta da primeira pergunta antes de se formular a segunda.
PROBLEMA 2

Determinar se existem nmeros inteiros positivos a e b tais que todos os termos da suceso definida por x1 = 2010, x2 = 2011,

xn + 2 = xn + xn +1 + a xn xn +1 + b , n 1,
sejam inteiros.
PROBLEMA 2

A circunferncia inscrita ao tringulo escaleno ABC tangente aos lados BC, CA e AB nos pontos D, E e F respectivamente. A recta EF corta a recta BC em G. A circunferncia de dimetro GD corta em R ( R D ) . Sejam P e Q

( P R, Q R )
EUREKA! N33, 2011

as interseces de BR e CR com , respectivamente. As rectas BQ

Sociedade Brasileira de Matemtica

e CP cortam-se em X. A circunferncia circunscrita a CDE corta o segmento QR em M e a circunferncia circunscrita a BDF corta o segmento PR em N. Demonstrar que as rectas PM, QN e RX so concorrentes.

SEGUNDO DIA
PROBLEMA 4

As mdias aritmtica, geomtrica e harmnica de dois nmeros inteiros positivos distintos so nmeros inteiros. Encontrar o menor valor possvel para a mdia aritmtica. Nota: Se a e b so nmeros positivos, suas mdias aritmticas, geomtrica e a+b 2 harmnica so respectivamente: , a b e . 1 1 2 + a b
PROBLEMA 5

Seja ABCD um quadriltero cclico sujas diagonais AC e BD so perpendiculares. Sejam O o circuncentro de ABDC, K a interseco das diagonais, L O a interseco das circunferncias circunscritas a OAC e OBD, e G a interseco das diagonais do quadriltero cujos vrtices so os pontos mdios dos lados de ABCD. Provar que O, K, L e G so colineares.

PROBLEMA 6

Ao redor de uma mesa circular sentam-se 12 pessoas e sobre a mesa h 28 vasos de flores. Duas pessoas podem ver-se uma outra se, e somente se, no h nenhum vaso alinhado com elas. Provar que existem pelo menos duas pessoas que podem ver-se.

EUREKA! N33, 2011

10

Sociedade Brasileira de Matemtica

ASSOCIANDO UM POLINMIO A EXPRESSES ALGBRICAS E TRIGONOMTRICAS


Marclio Miranda, IFRN (Caic RN)
Nvel Intermedirio O objetivo deste artigo mostrar uma tcnica que pode ser bastante til na hora de resolver problemas de olimpadas de Matemtica.Tal tcnica consiste em voc associar um polinmio a uma determina expresso. Com isso voc pode calcular o valor de expresses trigonomtricas, expresses algbricas e mostrar que um determinado nmero irracional. Vejamos alguns exemplos disso:
I) EXPRESSES TRIGONOMTRICAS

Esse problema deixa bem clara a idia de associarmos um polinmio a uma expresso trigonomtrica:
EXERCCIO RESOLVIDO 1 (BLGICA 2006):

a) Encontre todos os nmeros reais tais que cos ( 4 ) = cos ( 3 ) b) Determine inteiros a, b, c, d tais que cos equao ax 3 + bx 2 + cx + d = 0.
SOLUO:

2 4 6 , cos , cos , so solues da 7 7 7

a)

cos ( 4 ) = cos ( 3 ) 4 = 3 + 2k

ou

4 = 3 + 2k = 2k

ou

2k 2 4 6 so as razes dessa equao. , logo 1,cos ,cos , cos 7 7 7 7 Por outro lado temos que cos ( 4 ) = 8 cos 4 8 cos 2 + 1 e

cos ( 3 ) = 4 cos3 3 cos . Faa cos = t. Da temos que


Assim, solues cos a equao

cos ( 4 ) = cos ( 3 ) ( t 1) ( 8t 3 + 4t 2 4t 1) = 8 t 4 4 t 3 8 t 2 + 3 t + 1 = 0.

(8t

+ 4t 2 4t 1) = 0 tem

como

2 4 6 ,cos ,cos . 7 7 7

EUREKA! N33, 2011

11

Sociedade Brasileira de Matemtica

sec 400 + sec 800 + sec 1600 = 6.


SOLUO:

EXERCCIO RESOLVIDO 2 (MOCP, JULHO DE 2003): Prove que

satisfazem a equao 1 cos3 = 8cos3 6cos + 1 = 0, logo cos 400, cos 800, cos 1600 so as 2 razes do polinmio 8cos3 6 cos +1, e assim temos que: 6 cos 40 cos80 + cos160 cos80 + cos 40 160 = 8 1 cos 40 cos80 cos160 = 8 1 1 1 sec 40 + sec80 + sec160 = + + = cos 40 cos80 cos160 cos 40 cos80 + cos 40 cos80 + cos 40 cos80 = 6. cos160 cos80 cos 40
EXERCCIO RESOLVIDO 3 (IMO 1963): Prove que cos SOLUO: Note que

Note

que

400,

800

1600

cos

2 3 1 + cos = . 7 7 2

3 3 5 5 3 5 so + 4 = 3 e 3 + 4 = 5 , logo , , 7 7 7 7 7 7 7 7 7 solues da equao cos 4 x = cos3x. 7x x 7x Essa equao equivale a cos 4 x + cos3 x = 0 2 cos cos = 0 cos = 0 ou 2 2 2 x cos = 0. 2 7x PARTE 1: Resolver a equao cos =0 2 7x 2 k 3 5 9 11 13 = + k x = + x = , , , , , , , mas 2 2 7 7 7 7 7 7 7 7 13 3 11 5 9 cos = cos ,cos = cos ,cos = cos , logo h 4 solues distintas 7 7 7 7 7 7 3 5 entre 0 e 2 : , , , . 7 7 7 x PARTE 2: Resolver a equao cos = 0 2 3 + 4 = ,3
EUREKA! N33, 2011

12

Sociedade Brasileira de Matemtica

x = + k x = + 2k , logo x = a nica soluo entre 0 e 2 . 2 2 Por outro lado temos que cos 4 x = 8cos 4 x 8cos 2 x + 1 e cos3 x = 4cos3 x 3cos x. cos4 x = cos3x 8cos4 x + 4cos3 x 8cos2 x 3cos x + 1 = 0 8t 4 + 4t 3 8t 2 3t + 1 = 0, onde t = cos x. Claramente 1 raiz desse polinmio, e temos 8t4 +4t3 8t2 3t + 1 = (t +1) (8t3 4t2 4t + 1), donde o polinmio 8t3 4t2 4t + 1 tem como razes 3 5 cos ,cos ,cos . Logo temos pelas relaes de Girard que: 7 7 7 3 5 4 1 2 3 cos + cos + cos = = = cos cos cos . 7 7 7 8 2 7 7 7
II) CALCULANDO O VALOR DE UMA EXPRESSO ALGBRICA: EXERCCIO RESOLVIDO 4: Prove que
3

20 + 14 2 2 + 3 20 14 2 2 = 4.

SOLUO: Seja x = 3 20 + 14 2 2 + 3 20 14 2 2 . Temos x3 = 40 + 6x x3 6x 40

= 0. fcil ver que 4 raiz desse polinmio e x3 6x 40 = (x 4).(x2 + 4x + 10). Note que as razes de x2 + 4x + 10 no so reais e real, logo
3

20 + 14 2 2 + 3 20 14 2 2

20 + 14 2 2 + 3 20 14 2 2 = 4.

EXERCCIO RESOLVIDO 5 (CROCIA 2001): Se a + b + c = 0, calcule o valor da

expresso

a 7 + b7 + c 7 . abc ( a 4 + b 4 + c 4 )

SOLUO: Seja x3 + mx2 + px + q = 0. um polinmio de terceiro grau tal que suas

razes so a, b, c. Da temos que a + b + c = m = 0, ab + ac + bc = p e abc = q. Assim temos que: (a + b + c)2 = a2 + b2 + c2 + 2(ab + ac + bc) a2 + b2 + c2 = 2p Por outro lado temos que: a3 + pa + q = 0 a3 = pa q (i) 3 3 b + pb + q = 0 b = pb q (ii) c3 + pc + q = 0 a3 = pc q (iii) somando (i) + (ii) + (iii), temos que a3 + b3 + c3 = p.(a + b + c) 3q = 3q Da mesma forma temos que: a4 + pa2 + qa = 0 a4 = pa2 qa
EUREKA! N33, 2011

(iv)

13

Sociedade Brasileira de Matemtica

b4 + pb2 + qb = 0 b4 = pb2 qb (v) 4 2 4 2 c + pc + qc = 0 c = pc qc (vi) somando (iv) + (v) + (vi), temos que a4 + b4 + c4 = p.(a2 + b2 + c2) q.(a + b + c) = 2 p2 .
Analogamente temos que: a5 + pa3 + qa2 = 0 a5 = pa3 qa2 (vii) b5 + pb3 + qb2 = 0 b5 = pb3 qb2 (viii) c5+ pc3 + qc2 = 0 c5 = pc3 qc2 (ix) somando (vii) + (viii) + (ix), temos que a5 + b5 + c5 = p (a3 + b3 + c3) q (a2 + b2 + c2) = 5pq. Proseguindo do mesmo modo, temos que: a7 + pa5 + qa4 = 0 a7 = pa5 qa4 (x) b7 + pb5 + qb4 = 0 b7 = pb5 qb4 (xi) c7 + pc5 + qc4 = 0 c7 = pc5 qc4 (xii) somando (x) + (xi) + (xii): a7 + b7 + c7 = p (a5 + b5 + c5) q (a4 + b4 +c4) = 7 p 2q . a 7 + b7 + c7 7 p 2 q 7 = = . Com isso temos que 4 4 4 2 abc ( a + b + c ) q ( 2 p ) 2
III) PROVANDO A IRRACIONALIDADE DE UM NMERO:

Antes do prximo problema vamos provar o seguinte teorema: p TEOREMA (TESTE DA RAIZ RACIONAL): Se o nmero , onde p e q so inteiros e q mdc(p, q) = 1, uma raiz do polinmio com coeficientes inteiros an x n + an 1 x n 1 + ... + a1 x + a0 , ento p um divisor de a0 e q um divisor de an.
PROVA: Como
n

p raiz do polinmio temos que q


n1

p p p an + an1 + ... + a1 + a0 = 0 an pn + an1 pn1 q + ... + a1 p qn1 + a0 qn = 0, q q q logo temos que p um divisor de a0 e q um divisor de an.
EXERCCIO RESOLVIDO 6: Prove que 2 + 3 irracional.
EUREKA! N33, 2011

14

Sociedade Brasileira de Matemtica

2 + 3 x 2 = 3 x2 1 = 2 2 x x4 2x2 + 1 = 8x2 x4 10x2 + 1 = 0. Logo pelo teorema acima as razes racionais da equao s podem ser 1 ou 1, que claramente no so solues (em ambos os casos o valor numrico do polinmio 8). Logo esse polinmio s possui razes irracionais, portanto 2 + 3 irracional.
Soluo: Seja x = EXERCCIOS PROPOSTOS:

2 3 7 sen = . 7 7 7 8 2) (Vietn 1982) Ache a, b, c inteiros tais que as razes da equao ax2 + bx + c = 0 so cos 720 e cos 1440.
1) (EUA) Prove que sen

sen

3) (Prova de Seleo da Romnia para a IMO 1970): Prove que para todo inteiro positivo n: ( n 1) tg n = 2n + 1. 2 3 tg tg tg ...tg 2n + 1 2n + 1 2n + 1 2n + 1 2n + 1 4) (Prova de Seleo da Sua para a IMO 2004): Sejam a, b, c, d nmeros reais

distintos satisfazendo as equaes:


a = 45 21 a , b = 45 21 b , c = 45 21 c , d = 45 21 d Prove que abcd = 2004.

(a Calcule os possveis valores de


2 + 3 raiz.

5) (OBM 2003): Sejam a, b, c nmeros reais no-nulos tais que a + b + c = 0.


3

+ b3 + c 3 ) ( a 4 + b 4 + c 4 )
2

(a

+ b5 + c5 )

6) (Blgica 1978): Encontre um polinmio com coeficientes inteiros tal que

7) (Moldvia 2000): Os nmeros a, b, c satisfazem a relao a + b + c = 0. Mostre que o nmero 2a4 + 2b4 +2c4 um quadrado perfeito. 8) Prove que 2 + 3 3 irracional. 9) Prove que x = 2cos

satisfaz a equao: x3 + x2 2x + 1 = 0.

Use este fato para provar que cos

irracional. 7 10) Prove que tg2 10 + tg2 30 +....+ tg2 870 + tg2 890 = 4005.
EUREKA! N33, 2011

15

Sociedade Brasileira de Matemtica

11) Prove que cos 200. cos 400.cos 800 = 12) Prove que:

1 . 8

2 3 tg = 7. 7 7 7 2 3 4 5 6 b) tg tg tg tg tg tg = 13 . 13 13 13 13 13 13
a) tg

tg

13) Prove que cossec 6 + cossec 78 cossec 42 cossec 66 = 8. 14) Calcule as expresses:

2 3 tg 2 . 7 7 7 2 3 b) tg 2 + tg 2 . + tg 2 7 7 7 2 3 2 3 c) tg 2 tg 2 + tg 2 tg 2 + tg 2 tg 2 . 7 7 7 7 7 7 2 4 1 15) Prove que cos cos cos = . 7 7 7 8 a) tg 2


tg 2
16) Ache uma equao do terceiro grau cujas razes so cos 17) Calcule as expresses:

,cos

3 5 ,cos . 7 7

3 5 cos . 7 7 7 3 5 3 5 b) cos cos + cos cos . cos + cos 7 7 7 7 7 7 3 5 c) cos + cos + cos . 7 7 7 3 5 + cos 2 d) cos 2 + cos 2 . 7 7 7 1 1 1 e) . + + 3 5 cos cos cos 7 7 7 18) Prove que tg 810 tg 630 + tg 90 tg 270 = 4.
a) cos

cos

EUREKA! N33, 2011

16

Sociedade Brasileira de Matemtica

19) Sejam u, v, w as razes do polinmio x3 10x + 11. Determine o valor de arctg u

+ arctg v + arctg w.

5 13 + cossec + cossec = 6. 18 18 18 21) Prove que tg 200. tg400. tg 600. tg 800 = 3.


20) Prove que cossec 22) Sejam a, b, c nmeros reais tais que a + b + c = 0, prove que:

a) a3 + b3 + c3 = 3abc. a 2 + b 2 + c 2 a 5 + b5 + c 5 a 7 + b 7 + c 7 = . b) 2 5 5

3 . 8 2 3 24) Prove que cot g 2 + cot g 2 + cot g 2 = 5. 7 7 7 4 7 25) Calcule o valor da expresso tg + tg + tg . 9 9 9
23) Prove que sen20 sen40 sen80 = REFERNCIAS [1] MIRANDA, Marclio. Problemas Selecionados de Matemtica ITA-IME Olimpadas, Volume 1, Fortaleza (CE), Editora Vestseller, 2010. [2] ANDREESCU, Titu; FENG , Zuming. 103 Trigonometry Problems from the Training of the USA IMO Team, Birkhauser, 2004. [3] ANDREESCU, Titu; GELCA, Razvan. Putnam and Beyond. New York: SpringerVerlag, 2006. [4] DOMINGUES, Hygino. Fundamentos de Aritmtica, So Paulo, Atual Editora, 1991. SITES ACESSADOS [1] The IMO Compendium, Disponvel em <http://www.imomath.com/index.php?options=oth|other&p=0>, Acesso em: 10/08/2009. [2] Treinamento do Cone Sul. Disponvel em: < http://treinamentoconesul.blogspot.com/>, Acesso em: 12/08/2009. [3]Notas de Aula de Kin Yin Li. Disponvel em: <http://www.math.ust.hk/~makyli/190_2003Fa/lect-notes_03fa.pdf>, Acesso em: 15/08/2009. [4] Pgina de Olimpada da Sociedade Canadense de Matemtica. Disponvel em: < http://www.cms.math.ca/Olympiads/ >, Acesso em: 20/07/2009. [5] Matemtica Nick Puzzles. Disponvel em: < http://www.qbyte.org/puzzles/>, Acesso em : 15/11/2009. [6] Olimpada Brasileira de Matemtica. Disponvel em: <http://www.obm.org.br >, Acesso em: 20 /11/2009.
EUREKA! N33, 2011

17

Sociedade Brasileira de Matemtica

SOMAS TRIGONOMTRICAS: DE PROSTAFRESE FRMULA DE EULER


Rogrio Possi Junior
Nvel Intermedirio
INTRODUO

So apresentados fundamentos bsicos da matemtica elementar, cujos conceitos somados podem auxiliar na resoluo de problemas mais elaborados, como os que podem aparecer quando se depara com o incio do estudo das Variveis Complexas e o uso dos teoremas de De Moivre. Seja atravs das frmulas de Transformao de Soma em Produto, conhecidas como Frmulas de Prostafrese, ou atravs da Relao de Euler, so calculados alguns exemplos de somas de funes trigonomtricas aparentemente complexas.
AS FRMULAS DE TRANSFORMAO TRIGONOMTRICAS.

Admitamos conhecidas as frmulas da soma e diferena de arcos para as funes seno e cosseno, isto

sen(a + b) = sen a cos b + sen b cos a sen(a b) = sen a cos b sen b cos a cos(a + b) = cos a cos b sen a sen b cos(a b) = cos a cos b + sen a sen b
Somando-se (a) e (b) tem-se

(a) (b) (c) (d)

sen(a + b) + sen(a b) = 2 sen a cos b sen(a + b) sen(a b) = 2 sen b cos a cos(a + b) + cos(a b) = 2 cos a cos b

(e)

Subtraindo-se (a) de (b) tem-se

(f)

Somando-se (c) e (d) teremos

(g)

E por fim, subtraindo-se (c) de (d)


EUREKA! N33, 2011

18

Sociedade Brasileira de Matemtica

cos(a + b) cos(a b) = 2 sen a sen b


Fazendo a + b = e a b = teremos que a =

(h)

+
2

e b=

, cujos

valores substitudos nas relaes (e), (f), (g) e (h) fornecero as seguintes relaes

+ sen + sen = 2 sen cos 2 2 + sen sen = 2 sen cos 2 2 + cos + cos = 2 cos cos 2 2 + cos cos = 2 sen sen , que so as conhecidas 2 2
Frmulas de Transformao de soma em produto ou Frmulas de Prostafrese.
A FRMULA DE EULER

Segundo GUIDORIZZI (1987), seja f ( x) uma funo derivvel at a ordem n em um intervalo aberto I e seja x0 I . Define-se o polinmio P( x ) a seguir como o polinmio de Taylor, de ordem n , de f ( x) em torno do ponto x0 , isto

P ( x) = f ( x0 ) + f ( x0 )( x x0 ) + + f (

n)

( x0 )

( x x0 )
n!

=f (
k =1

k)

( x0 )

( x x0 )
k!

(i)

que, se fixado em torno de x0 = 0 , tambm pode ser chamado de polinmio de Mac-Laurin. Tomando-se (i) f ( x) = ex e x0 = 0, pode-se demonstrar que

x 2 x3 xn x 2 x3 e x = lim x x = + + + + 1 1 + + + + + n n! 2 3! 2 3!
A expresso da direita pode ser usada para definir ex para x para x complexo. Analogamente demonstra-se que

(j)

EUREKA! N33, 2011

19

Sociedade Brasileira de Matemtica


n 1 x3 x5 xn x3 x5 2 = x + + x sen x = lim ( 1 ) + n n! 3! 5! 3! 5!

(k)

e que
2n x2 x4 x2 x4 n x cos x = lim1 + + + (1) = 1 + n 2 4! (2n)! 2! 4!

(l)

Para x = Z = iY , Y R e observando-se (j), (k) e (l) teremos que

Y2 Y4 Y3 Y5 e iY = 1 + + i Y + = cos Y + i sen Y 2 4! 3! 5!
que a conhecida frmula de Euler. No obstante, tambm se demonstra que se e Z = e X +iY , onde X 0 , ento

(m)

e Z = e X (cos Y + i sen Y )

(n)
Z

Se, alternativamente, adotssemos a expresso de (n) como definio de e , no de fato, se Z1 = X 1 + iY1 e difcil mostrar que e Z +W = e Z eW ,Z ,W

( cosY cosY
1

Z 2 = X 2 + iY2 , eZ1 +Z2 = ex1 + x2 ( cos (Y1 + Y2 ) + isen (Y1 + Y2 ) ) = ex1 + x2


2

senY1senY2 + i ( senY1cosY2 + senY2 cosY1 ) =


x2 Z1

=e

x1

( cosY1 + isen Y1 ) e ( cosY2 + isen Y2 ) = e

e Z2 .

PROBLEMAS DE APLICAO PROBLEMA 1: Comearemos com um exemplo de problema anlogo ao proposto em um exame de admisso ao Instituto Militar de Engenharia (IME). O problema pede que se calcule as somas a seguir.

S1 = sen x + sen 2 x + sen 3 x + + sen nx


S 2 = cos x + cos 2 x + ... + cos nx

(1) (2)

Utilizaremos a transformao de somas de funes trigonomtricas em produto, conhecidas como Frmulas de Prostafrese. Observamos que

EUREKA! N33, 2011

20

Sociedade Brasileira de Matemtica

sen

3x x x sen = 2 sen cos x 2 2 2 5x 3x x sen sen = 2 sen cos 2 x 2 2 2 7x 5x x sen = 2 sen cos 3 x sen 2 2 2 sen (2n 1) x (2n 3) x x sen = 2 sen cos(n 1) x 2 2 2 (2n + 1) x x (2n 1) x sen = 2 sen cos nx sen 2 2 2

(3)

Somando-se as linhas acima encontraremos uma Soma Telescpica, cujo valor ser dado por

x x n (2n + 1) x sen = 2 sen cos jx 2 2 2 j =1 (n + 1) x nx sen cos n 2 2 S 2 = cos jx = x j =1 sen 2 sen


Analogamente, para a soma das funes seno S1 pode-se escrever que: 3x x x cos cos = 2 sen sen x 2 2 2 5x 3x x cos cos = 2 sen sen 2 x 2 2 2 7x 5x x cos cos = 2 sen sen 3 x 2 2 2
(2n 1) x (2n 3) x x cos = 2 sen sen(n 1) x 2 2 2 x (2n + 1) x (2n 1) x cos cos = 2 sen sen nx 2 2 2 cos

(4)

(5)

Somando-se as linhas acima encontraremos outra Soma Telescpica, cujo valor

EUREKA! N33, 2011

21

Sociedade Brasileira de Matemtica

cos

(2n + 1) x x x n cos = 2 sen sen jx 2 2 2 j =1


n

S1 = sen jx =
j =1

sen

(n + 1) x nx sen 2 2 x sen 2

(6),

que a soma procurada. No obstante, este problema tambm pode ser resolvido utilizando-se a conhecida Relao de Euler. Seja e 2 = C = cos
2
ix

x x + i sen , onde i 2 = 1 ; assim tem-se que 2 2

x x C = cos + i sen = cos x + i sen x 2 2


2

x x C 4 = cos + i sen = cos 2 x + i sen 2 x 2 2 x x = cos + i sen 2 2


2 4 2n 2n

(7)

2n

= cos nx + i sen nx

n C 2 (C 2 n 1) C n C n C n (cos jx + i sen jx ) C + C ++ C = = C = (C 2 1) C C 1 j =1 nx sen n x x nx nx 2 = S + iS (cos jx + i sen jx ) = cos + i sen cos + i sen 2 1 2 2 2 2 sen x j =1 2 nx (n + 1) x (n + 1) x nx sen cos sen sen 2 2 +i 2 2 (8) S 2 + iS1 = x x sen sen 2 2

( (

de onde tiramos os valores de interesse S1 e S 2 igualando-se as partes reais e imaginrias da igualdade acima respectivamente.
EUREKA! N33, 2011

22

Sociedade Brasileira de Matemtica

PROBLEMA 2: Considere agora o problema de se determinar as somas dadas por

S1 = sen 2 jx e S 2 = cos 2 jx . Para tal, observa-se inicialmente, da


j =1 j =1

Trigonometria que sen 2 x =

1 1 1 1 cos 2 x e cos 2 x = + cos 2 x , assim pode2 2 2 2

se reescrever S1 e S 2 como sendo


n 1 1 1 1 S1 = sen 2 jx = + + + (cos 2 x + cos 4 x + + cos 2nx ) " n " 2 2 2 2 j =1

(9)

onde S= cos 2 x + cos 4 x + + cos 2nx . Com o auxlio da Relao de Euler, seja C = cos x + i sen x , assim

C 2 + C 4 + + C 2 n = C.
n

n C n C n C n (cos 2 jx + i sen 2 jx ) = C C 1 j =1

( (

cos(n +1)x.sennx sen(n +1)x.sennx +i = S+iS senx sen x j =1 n n 1 onde S= sen 2 jx . Sendo S1 = S 2 2 j =1 1 cos(n + 1) x. sen nx S1 = n 2 sen x (cos2 jx + i sen2 jx) =

(10)

(11)

Outra soluo para o clculo da soma S consiste em transform-la segundo as frmulas de Prostafrese. Para este caso tem-se que,

sen 3x sen x = 2 sen x cos 2 x sen 5 x sen 3 x = 2 sen x cos 4 x


(12)

sen(2n 1) x sen(2n 3) x = 2 sen x cos(2n 2) x sen(2n + 1) x sen(2n 1) x = 2 sen x cos 2nx

EUREKA! N33, 2011

23

Sociedade Brasileira de Matemtica

Somando-se todas as linhas acima tem-se que sen(2n + 1) x sen x = 2 sen x S

S=

sen nx. cos(n + 1) x , que exatamente o valor encontrado da parte real do sen x

somatrio dado por (10). Observando-se (9) e que cos 2 x =

1 1 + cos 2 x 2 2 n 1 1 cos(n + 1) x. sen nx S 2 = cos 2 jx = [n + S] = n + 2 2 sen x j =1

(13)

que resolve o problema do clculo de S 2 .


PROBLEMA 3: Considere a seguir o problema do clculo das somas dadas por
n n

S1 = sen 3 kx e S 2 = cos 3 kx .
k =1 k =1

Seja C = cosx + isenx. Sendo C K = cos kx i sen kx pode-se escrever que

C k + C k 2 k C C k sen kx = 2i cos kx =
Elevando-se
k

(14) (15) relao


k
3

C C C C 3 C C sen 3 kx = = 2i 8i 3 sen kx sen 3kx (16) sen 3 kx = 4 n n 1 n (17) S1 = sen 3 kx = 3 sen kx sen 3kx 4 k =1 k =1 k =1 nx (n + 1) x sen sen n 2 2 e observando-se que se Por (6) tem-se que sen kx = x k =1 sen 2 D = cos 3 x + i sen 3 x teremos que:
3k

(15)
3 k

) (

ao
k
k

cubo

tem-se

que

EUREKA! N33, 2011

24

Sociedade Brasileira de Matemtica

D 2 = cos 6 x + i sen 6 x D 3 = cos 9 x + i sen 9 x D n = cos 3nx + i sen 3nx


1 n n n 2 2 D 2 .D 2 D D S D = D + D 2 + D3 + + D n = 1 1 D2 D 2 (3n + 1) x (3n + 1) x + i sen cos 2 2 sen 3nx SD = 3x 2 sen 2

(18)

Tomando-se a parte imaginria da relao (18) tem-se que

(3n + 1) x 3nx sen sen 2 2 sen 3kx = 3x k =1 sen 2


n

(19)

Logo, por (6) e (19) teremos que

(n + 1) x (3n + 1) x 3nx nx 3 sen sen sen sen 1 2 2 2 2 S1 = sen 3 kx = 3x x 4 k =1 sen sen 2 2


n

(20)

Vale lembrar que a soma S =

sen 3kx
k =1

tambm poder ser calculada

observando-se as igualdades a seguir, isto

EUREKA! N33, 2011

25

Sociedade Brasileira de Matemtica

cos

9x 3x 3x cos = 2 sen sen 3x 2 2 2 15 x 9x 3x cos cos = 2 sen sen 6 x 2 2 2 cos

3(2n 1) x 3(2n 3) x 3x cos = 2 sen sen(3n 1) x 2 2 2 3(2n + 1) x 3(2n 1) x 3x cos cos = 2 sen sen 3nx 2 2 2
cuja soma resultar em

3(2n + 1) x 3x 3x n cos = 2 sen sen 3kx . 2 2 2 k =1 3nx 3(n + 1) x sen sen n 2 2 sen 3kx = , que exatamente a expresso (19). x 3 k =1 sen 2 cos
Para o clculo de S 2 =
3

cos
k =1

kx elevando-se a expresso (14) ao cubo teremos

C k + C k C 3 k + C 3 k + 3 C k + C k = que cos kx = 2 8 3 cos kx + cos 3kx cos 3 kx = 4 n n 1 n S 2 = cos 3 kx = 3 cos kx + cos 3kx 4 k =1 k =1 k =1

) (

)
(21) (22)

Utilizando-se a relao (4) e a parte real da relao (18) e substituindo-as em (22) tem-se que

(n + 1) x (3n + 1) x 3nx nx 3 sen cos cos sen 1 2 2 2 2 S 2 = cos kx = + 3x x 4 k =1 sen sen 2 2


n

(23)

EUREKA! N33, 2011

26

Sociedade Brasileira de Matemtica

Ressaltamos que a soma

cos 3kx
k =1

tambm pode ser calculada atravs das

frmulas de Prostafrese, ou seja, fazendo

3x 3x 9x sen = 2 sen cos 3x 2 2 2 15 x 9x 3x sen sen = 2 sen cos 6 x 2 2 2 sen 3(2n 1) x 3(2n 3) x 3x sen = 2 sen cos(3n 1) x 2 2 2 3(2n + 1) x 3(2n 1) x 3x sen sen = 2 sen cos 3nx 2 2 2 sen
e somando-se as linhas teremos uma Soma Telescpica, cujo valor ser

3x n 3x 3(2n + 1) x sen = 2 sen cos 3kx 2 k =1 2 2 3nx 3(n + 1) x sen cos n 2 2 , que exatamente a parte real da cos 3kx = 3x k =1 sen 2 sen
expresso (18).
PROBLEMA 4 (IMO-62): Aqui proposto resolvermos a equao a seguir (observamos

que o segundo problema resolvido trata desta questo de forma generalizada).

cos 2 x + cos 2 2 x + cos 2 3x = 1 1 1 Notando que cos 2 x = + cos 2 x segue que 2 2 3 1 cos 2 x + cos 2 2 x + cos 2 3x = + (cos 2 x + cos 4 x + cos 6 x ) 2 2 Sendo Z = cos x + i sen x Z 2 = cos 2 x + i sen 2 x
Z 4 = cos 4 x + i sen 4 x Z 6 = cos 6 x + i sen 6 x
EUREKA! N33, 2011

(A)

27

Sociedade Brasileira de Matemtica

Z 2 ( Z 6 1) sen 3 x Z +Z +Z = = (cos 4 x + i sen 4 x ) 2 ( Z 1) sen x


2 4 6

(B)

Tomando-se a parte real de (B) tem-se que

(cos 2 x + cos 4 x + cos 6 x ) = cos 4 x sen 3x


sen x

(C)

1 (sen 7 x sen x) = sen 3x. cos 4 x , ento teremos que a equao (A) 2 reduz-se a sen 7 x + sen x = 0 .
Como

2 sen 4 x. cos 3 x = 0 sen 4 x = 0 cos 3 x = 0


Logo, a soluo da equao proposta ser dada pelo conjunto

S = x

x=

k ( 2k + 1 ) ( 2k + 1 ) x= x= ,k 2 4 6

PROBLEMA 5: Determinaremos agora o valor das somas

a) cos x + 2 cos 2 x + 3 cos 3 x + + n cos nx e b) senx + 2sen2 x + 3sen3 x + + nsennx Sejam S1 = cos x + 2 cos 2 x + 3 cos 3 x + + n cos nx e S 2 = senx + 2sen2 x + 3sen3 x + + nsennx

S1 + iS 2 = ( cosx + isenx ) + 2 ( cos2 x + isen2 x ) + + n ( cosnx + isennx )

Sendo Z = cosx + isenx S1 + iS 2 = Z + 2 Z 2 + 3Z 3 + + nZ n . Multiplicandose ambos os termos por (1 Z ) teremos

Z + Z + Z + Z nZ S1 + iS 2 = (1 Z )
2 3 n

n +1

nZ
1 2

2 n +1 2 1 2

( Z n 1) (Z Z
1 2 1 2 2

(Z Z

2n + 1 2n + 1 n cos x + isen x 2 2 ( cosnx + isennx 1) S1 + iS 2 = x x 2isen 4i 2 sen 2 2 2


EUREKA! N33, 2011

(1)

28

Sociedade Brasileira de Matemtica

Observando-se que a parte real de (1) nos dar o valor de S1 e que a parte imaginria nos dar o valor de S 2 tem-se, aps alguma manipulao algbrica que

S1 = jcosjx =
j =1

( n + 1) cosnx ncos ( n + 1) x 1
4sen 2

S2 =
j =1

x 2 ( n + 1) sennx nsen ( n + 1) x jsenjx = x 4sen 2 2

,e

PROBLEMAS PROPOSTOS: 1) (URSS) Calcule o valor das somas


1 n b) senx + Cn sen2 x + + Cn sen ( n + 1) x

1 n cos 2 x + + Cn cos(n + 1) x a) cos x + Cn

n k = denota o binomial n escolhe k. Obs: Cn k 2) (URSS) Mostre que

cos

2 4 6 2 n 1 + cos + cos + + cos = . 2n + 1 2n + 1 2n + 1 2n + 1 2

3) (URSS) Prove que

a) sen + sen ( + ) + sen ( + 2) + + sen ( + n) =

sen

( n + 1) sen + n
2 sen

b) cos + cos ( + ) + cos ( + 2) + + cos ( + n) =

sen

( n + 1) cos + n
2 sen 2 2

4) Calcule o valor da soma S = 5) Mostre que

cos 2

4+

cos

n 2 cos 4 . 4 ++ 2 n 2 2

EUREKA! N33, 2011

29

Sociedade Brasileira de Matemtica

ak +2 cosk ak +1 cos(k +1) a cos +1 a) 1+ a cos + a cos2 + + a cosk = a2 2a cos +1 sen + asen ( + h) + + ak sen ( + kh) =
2 k

b) ak +2sen ( + kh) ak +1sen + ( k + 1) h asen ( h) + sen

a2 2acos h+ 1 0 6) Mostre que 72 o menor ngulo positivo que satisfaz simultaneamente s equaes: 1 + cosx + cos2 x + cos3 x + cos4 x = 0 senx + sen2 x + sen3 x + sen4 x = 0 ( 2n + 1) x sen 1 2 7) (IME-92) Mostre que + cosx + cos2 x + + cosnx = . x 2 2sen 2

REFERNCIAS [1] FADDEEV, D.; SOMINSKY, I. Problems in Higher Algebra, Moscou: Ed. MIR, 1968. [2] GREITZER, S.L. International Mathematical Olympiads 1959-1977, Fifth Printing, Washington D.C.: The Mathematical Association of America, 1978. [3] GUIDORIZZI, H.L. Um curso de clculo Vol. 1, 2a Edio, So Paulo: Ed. Livros Tcnicos e Cientficos, 1987. [4] IEZZI, G. Fundamentos de Matemtica Elementar, Vol. 3 (Trigonometria), 6a Edio, So Paulo: Editora Moderna, 1985. [5] IEZZI, G. Fundamentos de Matemtica Elementar, Vol. 6 (Complexos Polinmios Equaes), 4a Edio, So Paulo: Editora Moderna, 1983. [6] LIDSKI, V. B.; OVSIANIKOV, L. V.; TULAIKOV, A. N.; SHABUNIN M. I. Problemas de Matematicas Elementales, Moscou: Ed. MIR, 1972. [7] MORGADO, A. C; WAGNER, E.; DO CARMO, M. P., Trigonometria e Nmeros Complexos, 4a Edio, Rio de Janeiro: Publicao da Sociedade Brasileira de Matemtica, 2001. [8] SHKLARSKY, D.O., CHENTZOV, N.N., YAGLOM, I.M. The USSR Olympiad Problem Book, New York, Dover Publications, Inc., 1994.

EUREKA! N33, 2011

30

Sociedade Brasileira de Matemtica

UMA INTERESSANTE DEDUO PARA A FRMULA DE HERO


Flvio Antonio Alves, Amparo SP
Nvel Intermedirio
Nesta nota sugerimos uma deduo para a fascinante frmula de Hero por meio de aplicaes dos nmeros complexos geometria. Sejam z1 = a + bi e z 2 = c + di dois nmeros complexos no nulos e distintos. Vamos considerar o tringulo de vrtices o, z1 e z2 (veja a figura abaixo).
Im

z2

2 1

z1
Re

A rea S do tringulo acima dada por: S= 1 1 z1 z2 sen ( 2 1 ) = Im { z2 z1} . 2 2

Vamos multiplicar essa expresso, membro a membro, por 2 e elevar ao quadrado ambos os termos da igualdade. Assim,

4S 2 = Im { z2 z1} =
2

( z2 z1 z1 z2 ) 2 ( 2i )

1 2 z2 4

2 z1 z22 z12 z12 z2

EUREKA! N33, 2011

31

Sociedade Brasileira de Matemtica

1 1 2 2 2 4 z2 z1 (z2 z1 + z1 z2 ) = [2 z2 z1 (z2 z1 + z1 z2 )] [2 z2 z1 + (z2 z1 + z1 z2 )] 4 4

1 2 z1 z 2 ( z1 z 2 4
2

) ] [( z
2
1

+ z2

z1 z 2

].

Notemos que: i) z1 z 2

( z1 z 2

) = (z
2

z 2 + z 2 z1 )( z1 z 2 + z1 z 2 ),

E, do mesmo modo, temos que: ii) z1 + z 2

z1 z 2

= ( z1 + z 2 z1 z 2

)( z

+ z 2 + z1 z 2 ) .

Substituindo (i) e (ii) na expresso acima, vem:

1 ( z1 z2 + z2 z1 )( z1 z2 + z1 z2 )( z1 + z2 z1 z2 )( z1 + z2 + z1 z2 ) 4 ( z1 + z 2 + z1 z 2 ) Nesse caso, pondo-se p = , onde p o semi-permetro, 2 =


conclumos que:

4S 2 =

1 (2 p 2 z1 )(2 p 2 z 2 4

)(2 p 2 z
1

z2

) (2 p )

4 S 2 = 4( p z1 )( p z 2
S 2 = ( p z1 )( p z 2
S=

)( p z
1

z2

) ( p)

)( p z

z2

) ( p)

( p ) ( p z1 )( p z 2 )( p z1 z 2 ) , que a frmula de Hero.

EUREKA! N33, 2011

32

Sociedade Brasileira de Matemtica

Anderson Torres & Eduardo Tengan


Nvel Intermedirio Para a Frmula de Euler nos permite escrever ei = cos + i sen. Ela nos fornece uma maneira prtica de multiplicar nmeros complexos. Por exemplo, o Teorema de De Moivre, normalmente escrito

RAZES DA UNIDADE

( cos + i sen )
conciso: ( e
i n

= cosn + i senn , na notao exponencial fica bem mais

= ei ( n ) . Mas, e as razes da unidade? Elas so os complexos que

zeram o polinmio P ( z ) = z n 1. Por De Moivre, sabemos que k = e 2 k i n so razes deste polinmio (com 0 k < n ), e, como so n no total, elas so todas as razes. E assim temos o primeiro resultado do artigo:

zn 1 =
em que = e 2 k i n .

0 k < n

(z ),
k

Razes da unidade tm um monte de aplicaes. Uma das mais imediatas simplificar contas com funes trigonomtricas, usando estas frmulas aqui:

cos =

e i + e i e i e i ; sen = 2 2i

PROBLEMA 1: calcule a soma tenebrosa


0 k < n

sen

k n

SOLUO: Usando a nossa recente descoberta, esta soma se transforma numa progresso geomtrica! Sendo = ei n , temos

0 k < n

sen

k k k k 1 = = k ( 1 ) n 0 k < n 2i 2i 0 k < n 0 k < n

EUREKA! N33, 2011

33

Sociedade Brasileira de Matemtica


1 n k 1 n 1 ( ) 1 sen = 1 n 2i 1 1 0 k < n

Talvez voc deva estar pensando: uma diferena de complexos dando um real? Mas como?? Simples: 1 = , logo a soma acima uma diferena de conjugados dividida por 2i. por isso que o resultado real...

0 k < n

sen

k 1 2 2 1 2 + 1 2 = + 1 = i 1 2 = cotg 1 2 n 2i 1 1 + 2n

Agora, uma aplicao da fatorao de z n 1 :


PROBLEMA 2: Prove que, para todo inteiro positivo n existem polinmios

f n ,g n

[ x ] tais que

f n ( x )( x + 1) + g n ( x ) x 2 + 1 = 2
2r
n

SOLUO: Primeiro, testar alguns casos pequenos: n = 1

f1 ( x )( x + 1) + g1 ( x ) ( x 2 + 1) = 2
2

Para eliminar g1 , podemos aplicar x = i, o que nos d

f1 ( i )( i + 1) = 2 f1 ( i ) =
2

2
2

Podemos tomar f1 ( x ) = x. Mas e quanto a funcionar! Veja que


2

(1 + i ) g1 ( x ) ? Calma, coisas so feitas para


2

= i

2 f1 ( x )( x + 1) = 2 + x ( x + 1)

tem i com zero, e automaticamente i (conjugados, a-h!). Portanto o polinmio acima mltiplo de x 2 + 1 e basta efetuar a diviso com Briot-Ruffini para achar g1 .
EUREKA! N33, 2011

34

Sociedade Brasileira de Matemtica

Para o caso geral, vamos considerar os zeros de x 2 + 1. Mas os zeros de

x +1 =

2n

x2 1 x2 1
n

n +1

so justamente as razes 2n +1 -simas da unidade que no so


n +1

razes 2n -simas da unidade. Logo, se escolhermos = e 2 i 2 uma raiz 2n +1 -sima primitiva da unidade (isto , que no raiz t-sima da unidade para nenhum t menor que 2n +1 ), temos

x2 + 1 =
Escrevendo x = 1,

1 k 2n +1 k 1 ( mod 2 )

(x )
k

( 1)

2n

+1 =

1 k 2n +1 k 1 ( mod 2 )

( 1 ) 2 = ( 1 )
k k 1 k 2n +1 k 1 ( mod 2 )

Basta demonstrar que cada 1 + k mltiplo de 1 + . Moleza:

1 + k = (1 + ) (1 + 2 3 + ... k 2 + k 1 )
Portanto, podemos escolher f n tal que 2 f n ( x )(1 + x ) admite razes k ,k
2n

mpar. Portanto, divisvel por x 2 + 1, o que acaba a demonstrao. Agora, um problema de Geometria:
PROBLEMA 3: ABCDE um pentgono cclico de circuncentro O. Os ngulos

internos do pentgono so A = 70,B = 120,C = 120,D = 130 ,E = 100. Demonstre que as diagonais BD e CE encontram-se em um ponto pertencente reta AO.

SOLUO: Como em qualquer problema de geometria, um bom arrasto para

comear. Inicialmente, vamos ligar o centro aos vrtices do pentgono. Esta a melhor maneira de aproveitar a conciclicidade dos pontos. Assim sendo, AOB = 80,BOC = 40,COD = 80,DOE = 20,EOA = 140. Mas MDC ( 80,40,20,140) = 20 e portanto os vrtices do pentgono esto entre os vrtices de um 18-gono regular (afinal, 360 = 18 )! Agora, vamos colocar as 20 coisas nos eixos: inicialmente, O = 0 , A = 1 (podemos fazer isto por homotetia: se

EUREKA! N33, 2011

35

Sociedade Brasileira de Matemtica

OA 1, aplicamos uma homotetia de centro O e razo 1 OA ). Seja = ei 2 18 uma raiz 18-sima (primitiva, por sinal) da unidade. Com isto, os vrtices esto determinados. Vamos usar minsculas para os nmeros complexos associados aos pontos.

a = 1,b = 4 ,c = 6 ,d = 10 ,e = 11
Temos que provar que AO,BD,CE so concorrentes. Dada a escolha esperta que fizemos, basta demonstrar que as retas BD e CE se intersectam em um ponto real puro. Ou, em outras palavras, que se z o complexo comum a BD e CE ento z = z . Bem, para calcular equaes de retas, vamos a uma tcnica, ou melhor, um teorema, bastante til (e que fica como exerccio para o leitor, haha!): Dados os complexos p, q do crculo unitrio, a reta pq tem equao dada por

z + pqz = p + q
Temos ento:

AO : z

=z

BD : z + bd z = b + d CE : z + cez = c + e
que equivale a

AO : z

=z

BD : z + 14 z = 4 + 110 CE : z + 17 z = 6 + 11
Basta provar que AO BD : z =

4 + 10 6 + 11 ; AO CE : z = 1 + 14 1 + 17

Antes de comear a calculeira, vamos estudar algumas propriedades interessantes de . Bem, sabemos que ele zero do polinmio x18 1, e 18 = 2 32. A ideia ser fatorar este polinmio at a exausto... x18 1 = x9 1 x9 + 1 . Como raiz 18sima primitiva da unidade, o primeiro fator no contm como raiz. Assim sendo, vamos pensar no outro fator: x9 + 1 = x3
EUREKA! N33, 2011

)(

( )

+ 1 = ( x3 + 1)( x6 x3 + 1) . Pode-

36

Sociedade Brasileira de Matemtica

se demonstrar (mas no ser necessrio) que este ltimo fator irredutvel. Ento 6 3 + 1 = 0, e de quebra 9 = 1. Depois dessa volta toda, vamos ao que interessa: comparar as duas expresses de z: 4 + 10 6 + 11 = 1 + 14 1 + 17

( + )(1 + ) = ( ( )(1 ) = (
4 10 17 4 1 8

6 6

+ 11 )(1 + 14 ) 2 )(1 5 )

4 1 12 + 9 = 6 2 11 + 7 4 1 + 3 1 = 6 2 + 2 + 7 4 1 + 3 1 = 3 1 + 7 4 1 = 7 3 1 = 6 0=0
E fim! Outra aplicao interessante das razes da unidade como marcadores. Veja este problema:
PROBLEMA 4: Determine uma frmula fechada para

k
3k

SOLUO: Bem, algum a conhece algo parecido? Que tal o Binmio de Newton?

k z = (1 + z ) n
k 3k

Agora, j tem alguma ideia do que se pode fazer? Temos que filtrar os mltiplos de 3 desta expanso, e nada melhor que usar uma raiz cbica da unidade = e2 i 3 . Substituindo z por 1, e 2 , temos

EUREKA! N33, 2011

37

Sociedade Brasileira de Matemtica

n n k = (1 + 1) k n k n n n 2k 2 n k n k = (1 + ) k (1 + + ) = 2 + (1 + ) + (1 + ) k k n n 2 k = (1 + 2 ) k k
Agora, se k mltiplo de 3,1 + k + 2 k = 3; caso contrrio, temos uma progresso 3k 1 = 0. geomtrica de razo k 1, e portanto 1 + k + 2 k = k 1 Ou seja, matamos todos os no mltiplos de 3!
n n n n n n n n + 3 = 2n + (1 + ) + (1 + 2 ) = 2n + ( 2 ) + ( ) = 2n + 2 ( 1) 2 3k k n 2 n 2n + 2 ( 1) cos n 3 = 3 3k k

Esta ltima tcnica tem um nome chique: multiseco. Vamos us-la em um problema de, adivinha s, Combinatria Enumerativa!
PROBLEMA 5: (IMO 1995, Canad) Seja p um primo mpar, e seja S = {1,2,3...2 p} .

Determine o total de subconjuntos A S que satisfazem as condies a seguir:


A = p;
p xA x.

SOLUO: Este foi o problema 6 da Olimpada Internacional de 1995, em Montreal, Canad. Ela foi tida como uma das mais interessantes pela riqueza de problemas legais e divertidos daquele ano, algo comparvel apenas IMO da Argentina, que aconteceria dois anos depois.

A soluo aqui apresentada uma pequena modificao daquela dada por Nikolai Nikolov, ganhador de um Special Prize (prmio especial, dado pela originalidade).
EUREKA! N33, 2011

38

Sociedade Brasileira de Matemtica

Vamos pensar em uma raiz p-sima da unidade, primitiva por sinal: = e2 i p . Veja que = k tambm uma raiz p-sima da unidade, para k {1,2,3,..., p 1} . Exclumos o 1 propositalmente, pois ele no ter propriedades to interessantes quanto as outras razes (logo vers o porqu). Os complexos

{ , , ,..., } = {1, ,
0 1 2 p 1 k

2k

,...,k ( p 1) so razes p-simas da


ik jk

unidade. Elas so distintas: de fato, se = para 0 i j < p, temos

e j i k = e0 = 1 p ( j i ) k e, como 0 < k < p, p ( j i ) j i = 0.

z j . Agora vamos ao bom e velho polinmio f ( z) = z p 1= 0 jp1 z j = 1 j p


Pensando em Sries Formais, conseguimos trabalhar com este polinmio os elementos de 1 a p. Como podemos alcanar 2p? Oras, eleva ao quadrado!

( f ( z )) = ( z
2

1) =
2
j

1 j 2 p

(z )
2

0 j p 1

(z ) (z ) = (z ) (z ) =
j j j j 0 j p 1 1 j p p +1 j 2 p
2

Vamos abrir ( f ( z ) ) : ( f ( z ) ) = a0 + a1 z + a2 z 2 + ... + ap z p + ... + a2 p1 z 2 p1 + a2 p z 2 p Agora, vamos observar como o ap produzido de uma maneira combinatria. Primeiramente, escolhemos arbitrariamente p fatores, e coletamos o termo z deles; isto nos dar o expoente 2p. J dos outros p fatores, escolhemos o termo j . O

resultado ser ento

ap =

1 j1 j2 <...< j p 2 p

( )( ) ...( ) =
j1 j2 jp

0r p 1

cr r

em que cr o total de p-tuplas j1 < j2 < ... < j p tais que

j1 + j2 + ... + j p r ( mod p ) . A nossa tarefa achar c0 !


Mas ( f ( z ) ) = z 2 p 2 z p + 1 a p = 2. Assim,
2

c0 + c1 + c2 + ... + c p 1 p 1 = 2
Em outras palavras, zero do polinmio

EUREKA! N33, 2011

39

Sociedade Brasileira de Matemtica

g ( z ) = ( c0 2 ) + c1 z + c2 z 2 + ... + c p 1 p 1
Lembre-se que todo o raciocnio usado at aqui foi puramente combinatrio, e vlido para qualquer que seja raiz p-sima da unidade (exceto o 1). Logo, todas as razes p-simas primitivas da unidade so razes de g. Mas g tem grau p 1, portanto: f ( z) = c p 1 (1 + z + z 2 + ... + z p 1 ) g ( z ) = c p 1 z 1 Igualando os coeficientes, c0 2 = c1 = c2 = ... = c p 1 .

2p Mas c0 + c1 + c2 + ... + c p 1 = . Contagem dupla: cada p-subconjunto de S p contado em exatamente um dos ci , justamente aquele correspondente soma de seus elementos mdulo p. Resolvendo as equaes acima, conclumos que
c0 = 2 +
E fim! Bem, que tal uns exerccios?
EXERCCIOS PROPOSTOS: 1) Determine o valor numrico da srie

1 2p 2 p p

j n n 1 1 j n A+ B +C 2) Sejam x, y, z, A, B, C reais tais que inteiro. Defina K r = x r sen ( rA ) + y r sen ( rB ) + z r sen ( rC ) .

cos

Prove que se K1 = K 2 = 0 ento K n = 0 para todo n > 0.


3) Fixe um dos vrtices de um n-gono regular inscrito numa circunferncia de raio 1, e considere os segmentos que ligam este vrtice a todos os outros. Prove que o produto das medidas de todos estes n 1 segmentos n.

EUREKA! N33, 2011

40

Sociedade Brasileira de Matemtica

4) Calcule sen

2 4 8 + sen + sen . 7 7 7
2 i

Dica: sejam = e 7 , p = + 2 + 4 ,q = 3 + 5 + 6 . O que queremos calcular a parte real de p. Calcule p + q e p q e seja feliz!
5) Se P ,Q , R , S so polinmios tais que

P ( x5 ) + xQ ( x 5 ) + x 2 R ( x 5 ) = ( x 4 + x 3 + x 2 + x + 1) S ( x ) , prove que P (1) = 0.

6) Frmula de Multiseco: Sendo p ( x ) = a0 + a1 x + a2 x 2 + ... + an x n , e l,m ,

com 0 l m, temos

k l ( mod m )

ak

0 k m

lk p ( k )

m
2

em que = e

2 i

7) Mostre que

n n cos ( k ) = 2 cos cos 2 2 0 k n k

8) (Irlanda) Sabe-se que a, b, c so complexos tais que as razes da equao

x 3 + ax 2 + bx + c = 0 tm mdulo 1. Prove que as razes de x 3 + a x 2 + b x + c = 0 tambm tm mdulo 1.


9) Seja 1 + x + x 2 + x 3 + x 4

496

= a0 + a1 x + a2 x 2 + ... + a1984 x1984 .

Determine MDC ( a3 ,a8 ,...,a1983 ) Prove que 10340 < a992 < 10347

10) Determine todos os polinmios P tais que P x 2 = P ( x ) P ( x 1) . 11) Determine o nmero de polinmios de grau 5 com coeficientes entre 1 e 9

( )

inclusive e que sejam divisveis por x 2 x + 1.


12) Prove que o nmero

0 k n

n 0.

2n + 1 3 k 2 no mltiplo de 5 para qualquer 2k + 1

EUREKA! N33, 2011

41

Sociedade Brasileira de Matemtica

COMO QUE FAZ?


mundo.

Resolvermos aqui, a pedidos, trs problemas propostos na seo Olimpadas ao redor do

1) (Problema 109 Sua, 2000, proposto na Eureka! 11) Seja q ( n ) a soma dos
algarismos de n. Calcule q q q ( 20002000 ) de Reriutaba CE). Como 20002000 = 22000 106000 , sua representao decimal a representao decimal de 22000 seguida de 6000 zeros, e logo
SOLUO:

((

)) (Proposto por Ccero Soares Furtado,

q ( 20002000 ) = q ( 22000 ) . Como 23 < 10, 22000 < 22001 = ( 23 )


cada

667

< 10667 , donde 22000


no mximo 9,

tem no mximo 667 dgitos. Como q ( 20002000 ) = q ( 22000 ) 9 667 = 6003. Portanto, q q 20002000

dgito

((

) ) 6 + 9 + 9 + 9 = 33, e logo q ( q ( q ( 2000 ) ) ) 3 + 9 = 12.


2000

Por outro lado, como n e q(n) sempre deixam o mesmo resto na diviso por 9, o resto da diviso de q q q ( 20002000 ) = q q q ( 20002000 )
22000 = 26333+ 2 = ( 26 )
333

((

)) ( (

))

por 9 igual ao resto


por 9. Como

da diviso de 22000 por 9. Mas, como 26 = 64 deixa resto 1 quando dividido por 9, 22 = 4
quando dividido

)) (( q ( q ( q ( 2000 ) ) ) = 4.
2000

q q q ( 20002000 ) 12 e 4 + 9 = 13 > 12, conclumos que necessariamente

2) (Problema 110 Grcia, 2000, proposto na Eureka! 11) Determine os nmeros primo p para os quais o nmero 1 + p + p 2 + p 3 + p 4 um quadrado perfeito. (proposto por Ccero Soares Furtado, de Rariutaba CE).
Vamos encontrar todos os naturais n tais que 1 + n + n 2 + n3 + n 4 quadrado

n +1 5n 2 n 1 4 3 perfeito. Note que n 2 + + + > n 4 + n3 + n 2 + n + 1 para =n +n + 2 4 2 4


EUREKA! N33, 2011

42

Sociedade Brasileira de Matemtica

todo n > 3 (pois

n2 n 3 > 0 para todo n > 3). Por outro lado, para todo 4 2 4 2 n n2 n , n 2 + = n 4 + n3 + < n 4 + n3 + n 2 + n + 1. 4 4

n +1 n , se n > 3 temos ou k n 2 + 2 2 k 2 < n 4 + n3 + n 2 + n + 1 ou k 2 > n 4 + n 3 + n 2 + n + 1 Assim, basta olhar os casos n {0,1, 2, 3} . Para n = 0, n 4 + n3 + n 2 + n + 1 = 1 = 12. Para n = 1,
Como, para todo k , temos k n 2 +

n = 2, n 4 + n3 + n 2 + n + 1 = 5, que no quadrado perfeito. Para 4 3 2 para n = 3, n + n + n + n + 1 = 31, que no quadrado perfeito, e, 4 3 2 2 Assim, o nico primo p tal que n + n + n + n + 1 = 121 = 11 . 1 + p + p 2 + p3 + p4 quadrado perfeito p = 3.

3) (Problema 188 Rssia, 2002, proposto na Eureka! 15) No intervalo ( 22 n ,32 n )


so escolhidos 22 n 1 + 1 nmeros mpares. Mostre que podemos encontrar entre estes nmeros dois nmeros tais que o quadrado de cada um deles no divisvel pelo outro. (Proposto por Anderson Torres, de Santana de Parnaba SP).
SOLUO: Se x < y so mpares e y divide x 2 , ento y

yx . Em particular, 2

yx y , donde y x 2 y , e logo 2
temos

y 1 = y 2 y + 1 > x, donde

y > x + 1. Assim, se 22 n < x0 < x1 < ... < x22 n1 < 32 n so os nmeros em questo, x j +1 > x j + 1, para todo j 0, e logo x22 n1 > 2n + 22 n 1 , donde x j > 2n + j , j 0.
32 n > x22 n1 > ( 2n + 22 n 1 ) ,
2

Em particular,

e logo

3n > 2n + 22 n 1 , mas isso falso para todo n 1 (para n = 1, 3 < 4, para n = 2, 9 < 4 + 8 e, para n 3,3n < 22 n 1 : com efeito, 27 = 33 < 25 = 32 e, se 3n < 22 n 1 ,3n +1 = 3 3n < 3 22 n 1 < 4 22 n 1 = 22 n +1 = 22( n +1)1 ), absurdo.
4) (Problema 113 Polnia, 2000, proposto na Eureka! 11) Uma sequncia p1 , p2,...
de nmeros primos satisfaz seguinte condio: para n 3, pn o maior divisor
EUREKA! N33, 2011

43

Sociedade Brasileira de Matemtica

primo de pn 1 + pn 2 + 2000. Mostre que a sequncia por Anderson Torres, de Santana de Parnaba SP). Para todo

( pn )

limitada. (Proposto

k 0 existe j com 1 j 40 tal que 2 max ( pk + j , pk + j +1 ) max ( pk , pk +1 ) + 40000 (de fato, a afirmao implica que 3 pn 160000, para todo n n0 ; note que existe n0 tal que pn + 2 max ( pn , pn +1 ) + 1000, n 1).

SOLUO: Vamos mostrar a seguinte afirmao, que implica o resultado:

Suponhamos inicialmente que, para todo r, com 0 r 35, pk + r um primo mpar. p + pk + r 2 + 2000 Ento, para todo r, com 2 r 36, pk + r k + r 1 . Definindo 2 q j 1 + q j 2 + 2000 q0 = pk , q1 = pk +1 e q j = para 2 j 36, temos pk + r qr , para 2 0 r 36. Se, para algum r 36, pk + r qr , tomando um tal r mnimo temos p + pk + r 2 + 2000 , e a afirmao vale para j = r. Temos ainda que q j pk + r k + r 1 6 j p + 2pk+1 4000 2pk 2pk+1 4000 1 2000 j , para dado pela expresso qj = k + + + 3 9 3 9 3 2 0 j 42. Assim, 1 9q6 j = ( 3 pk + 6 pk +1 4000) + ( 6 pk 6 pk +1 + 4000) + 6000 j 9 pk + 6000 j ( mod7) . 2 Portanto, existe s com 0 s 6 tal que 9q6 s (e logo q6 s ) mltiplo de 7. Se s = 0, a afirmao j vale para j = 1. Se tivssemos pk + r = qr para 0 r 36, tomamos s q com 1 s 6 tal que q6 s mltiplo de 7, e teramos pk + 6 j 6 s , absurdo. 7 Se pk = 2, a afirmao j vale para j = 1. Finalmente, se pk + r = 2 para algum r com 1 r 35, teremos pk + r +1 pk + r 1 + 2002 e pk + 2 + 2 pk +11 + 4004, mas um
6j

desses nmeros ( pk + r 1 , pk + r 1 +2002 e pk + r 1 + 4004 ) mltiplo de 3, logo a afirmao vale para j = r + 2.

EUREKA! N33, 2011

44

Sociedade Brasileira de Matemtica

SOLUES DE PROBLEMAS PROPOSTOS


Publicamos aqui algumas das respostas enviadas por nossos leitores.

131) a) Considere o seguinte jogo: no incio um jogador A entrega um nmero k 2 ao jogador B . Quando A entrega um nmero m 2 a B, B pode devolver m 1 ou m + 1 a A. Quando A recebe um nmero n 2 deve, se n for mpar n devolver 3n a B; se n for par mas no mltiplo de 4, pode devolver ou 3n a B, e, 2 n n se n for mltiplo de 4, pode devolver , ou 3n a B. Qualquer jogador ganha o 4 2 jogo se devolver 1 ao adversrio. Caso algum jogador devolva ao adversrio um nmero maior que 1000k, o jogo empata. Determine, para cada valor de k 2 , se algum dos jogadores tem estratgia vencedora, e, nesses casos, qual deles.
b) Resolva o item anterior supondo que A, ao receber um nmero n 2, deve n a B se n for par mas no mltiplo devolver 3n a B se n for mpar, deve devolver 2 n de 4 e deve devolver a B se n for mltiplo de 4. 4

SOLUO DE JOS DE ALMEIDA PANTERA (RIO DE JANEIRO RJ) a) Naturalmente A perde se entregar o nmero k = 2 ao jogador B, pois B poder devolver 1 imediatamente a A.
Vamos mostrar que A ganha se entregar a B um nmero de k da forma da forma

2 4n + 1 ou 3

4n 1 , para algum n 1 , e se k > 2 no for de nenhuma dessas formas 3 nenhum dos jogadores tem estratgia vencedora.
Para isso, note que, se A entrega 3 =

2 41 + 1 a B, B pode devolver 2 ou 4 a A, e, 3 41+1 1 em qualquer caso, A pode devolver 1 a B e ganhar o jogo. Se A entrega 5 = 3 a B, B pode devolver 4 ou 6 a A. Se devolve 4, A pode devolver 1 a B e ganhar. Se devolve 6, A pode devolver 3 a B, e ganhar a seguir, como vimos antes.
EUREKA! N33, 2011

45

Sociedade Brasileira de Matemtica

2 4n +1 + 1 a B, com 3 2 4n +1 2 4n +1 1 a A, caso em que A pode devolver a B, n 1, B pode devolver 3 3 2 4n +1 + 4 a A, caso em que A pode devolver ganhando o jogo, ou B pode devolver 3 2 4n + 1 4n + 2 1 a B, ganhando o jogo. De modo similar, se A entrega a B, com 3 3 4n + 2 4 4n +1 1 a A, caso em que A pode devolver a B, n 1, B pode devolver 3 3 4n + 2 + 2 a A , caso em que A pode devolver ganhando o jogo, ou B pode devolver 3 2 4n +1 + 1 a B, ganahndo o jogo. 3 Notemos que A sempre pode no mnimo empatar o jogo se k 3. De fato, se em algum momento do jogo A entrega m 3 a B, B devolve no mnimo m 1, e A pode devolver o triplo, que no mnimo 3 ( m 1) > m.
Em geral, podemos argumentar por induo: se A entrega Assim, A pode devolver nmeros cada vez maiores, que em algum momento ultrapassaro 1000k, empatando o jogo. Veremos agora que B pode garantir o empate se A entrega um nmero que no das formas descritas anteriormente. Mais precisamente, veremos que, se A envia um nmero que no pertence ao conjunto n n +1 2 4 +1 4 1 , n 0 , n 1 , ento B pode devolver um nmero a X := 3 3 partir do qual A no pode devolver nenhum nmero pertencente a X (note que 1 X ). Temos X = {1,3,5,11, 21,...} . Se A envia a B um nmero par m, B pode devolver m 1 ou m + 1, que so mpares, a A, que deve devolver o triplo a B. Como no possvel que 3 ( m 1) e 3 ( m + 1) pertenam ambos a X, isso mostra nossa afirmao no caso m par. Se A envia a B um nmero mpar m, que no pertence a X, no difcil ver que m 1 {2k , k X } {4k , k X } ou

m + 1 {2k , k X } {4k , k X } . Isso implica a afirmao no caso m par.


b) No meio do jogo, B s recebe um nmero par se A tiver acabado de dividir um nmero (necessariamente mltiplo de 8) por 4. E, se B recebe um mpar, devolver um par, o que forar A a dividi-lo por 2 ou por 4. Assim, os nmeros tendem a
EUREKA! N33, 2011

46

Sociedade Brasileira de Matemtica

decrescer, e , nos casos que empatavam, B ganha o jogo. Por outro lado, A ganha o jogo, com a mesma estratgia, nos mesmos casos que no item A, pois nesses casos sempre devolve nmeros mpares.

132) a) Considere uma famlia de 2000 crculos de raio 1 no plano tal que dois crculos de nunca so tangentes e cada crculo de intersecta pelo menos dois outros crculos de . Determine o nmero mnimo possvel de pontos do plano que pertencem a pelo menos dois crculos de . SOLUO DE ZOROASTRO AZAMBUJA NETO (RIO DE JANEIRO RJ) Mostraremos que esse nmero mnimo igual a 2000.
Para isso, consideramos um tringulo equiltero de lado 3 e os seguintes quatro crculos: o crculo circunscrito ao tringulo e os trs crculos que contm o circuncentro do tringulo e dois de seus vrtices. Esses quatro crculos tm raio 1, e cada um deles intersecta os outros trs. Considerando 500 cpias disjuntas dessa configurao de crculos, obtemos 2000 crculos como no enunciado tais que h 2000 pontos que pertencem a pelo menos dois deles.

Para concluir, vamos mostrar que, numa configurao de n crculos de raio 1 no plano ( n 2 ) em que cada crculo intersecta pelo menos outro crculo e no h dois crculos tangentes, h sempre pelo menos n pontos que pertencem a pelo menos dois dos crculos. Vamos mostrar, por induo em n, que, na situao acima, no apenas h pelo menos n pontos que pertencem a pelo menos dois dos crculos, mas tambm que existe uma funo injetiva do conjunto dos n crculos no conjunto dos pontos que pertencem a pelo menos dois dos crculos tal que a imagem de cada crculo pertence a ele.
EUREKA! N33, 2011

47

Sociedade Brasileira de Matemtica

outros crculos, digamos C2 , seja C1 C2 = { p, q}.

Para isso, note que se n = 2 isso claramente verdadeiro. Suponha agora que m 2 e que isso vale para todo n com 2 n m, e considere uma configurao de m + 1 crculos como antes. Suponha inicialmente que algum desses crculos, digamos C1 , intersecta s um dos

Temos dois casos: no primeiro, C2 s intersecta C1. Ento associamos p a C1 , q a C2 e usamos a hiptese de induo para os m 1 crculos restantes. No segundo caso, C2 intersecta algum dos outros crculos. Ento associamos P a C1 e usamos a hiptese de induo para os m crculos C2 , C3 ,..., Cm +1. Se, por outro lado, cada um desses m + 1 crculos intersecta pelo menos dois dos outros, temos de novo dois casos: Se h no total pelo menos m + 1 pontos que pertencem a pelo menos dois dos crculos, podemos separar um dos crculos, digamos Cm +1 , e fixar uma injeco de

{C1 , C2 ,..., Cm } no
Ci .

conjunto dos pontos que pertencem a pelo menos dois dos

crculos C j , j m tal que a imagem de cada Ci , que chamaremos de Pi , pertence a Para cada X {C1 ,..., Cm } , o conjunto dos pontos que pertencem a pelo menos dois crculos de X {Cm +1} tem pelo menos X + 1 elementos. Se algum ponto

P {P 1 ,..., P m } pertence a Cm +1 e a algum dos outros crculos, simplesmente


conjuntos A1 , A2 ,... do seguinte modo: A1 = {i m Pi Cm +1}. Se A1 , A2 ,..., Ar j menos dois crculos de definimos estendemos a injeo associando P a Cm +1 . Seno, construmos uma sequncia de

esto definidos, se o conjunto (de pelo menos Ar + 1 ) pontos que pertencem a pelo

Ar +1 = Ar j m Pj

{Cm +1} {C j , j Ar }}. Note que Ar +1 > Ar . Em algum momento, haver ponto P fora de { P 1 ,..., P m } que pertence a pelo menos dois crculos

{Cm +1} {C j , j Ar }

est contido em

{P1 ,..., Pm } ,
um de

pertence a pelo menos dois crculos de

Cm +1 {C j , j Ar } , e logo a dois crculos C j e C j , com j , j Ar , j Ar 1 . jr 1 Ar 1 \ Ar 2 tal que Pj pertence a C jr 1 , associamos C jr 1 a Pj , e, em geral,

Podemos ento alterar a injeo associando C j a P; como existe algum para cada s com 1 < s r 1, se j definimos js As \ As 1 , existe js 1 As 1 \ As 2
EUREKA! N33, 2011

48

Sociedade Brasileira de Matemtica

tal que Pjs pertence a C js1 ; associamos ento C js1 a Pjs . Fazemos isso at associar o que prova nossa afirmao. Finalmente, suponhamos que h apenas m pontos que pertencem a pelo menos dois dos crculos. Observamos que, como os crculos tm raio 1, se um par de pontos est contido em dois crculos de famlia, no estar contido em nenhum outro crculo de famlia, e portanto, se um ponto pertence a r 2 crculos da famlia, cada um desses r crculos intersecta os outros r 1 em outros r 1 pontos distintos, e distintos do ponto comum aos r crculos. Assim, cada um desses r crculos contm pelo menos r pontos que pertecem a pelo menos dois crculos da famlia. Podemos considerar uma injeo que leva {C1 , C2 ,..., Cm } no conjunto desses matriz ( aij ) ,1 i m + 1,1 j m, onde aij = 1 se Pj pertence a Ci e aij = 0, caso contrrio. Se, para j m, n j = i Pj Ci = i aij = 1 pontos, a qual ser uma bijeo. Sendo Pi a imagem de Ci , podemos considerar a

{C1 , C2 ,..., Cm , Cm+1} ,

C j1 a Pj2 . Como j1 A1 , podemos associar Cm +1 a Pj1 , estendendo nossa injeo a

} {
j

}
ij

e,

para

i m + 1, si := j Pj Ci = j aij = 1 , temos, si ni , i m, donde

} {
m i

pelo que observamos acima,


m

s n ,
i =1 i =1 i

mas

s = n = {( i, j ) a
i =1 i j =1

m +1

= 1 , e sm +1 > 0,

pois Cm +1 intersecta outros crculos, absurdo.

133) Considere um ngono regular inscrito em um crculo unitrio, fixe um vrtice i e denote por dj a distncia entre este vrtice i e o vrtice j. Prove que

(5 d ) = F
j i j =0 2 j

n 1

2 n

onde F1 = 0, F1 = 1 e Fn = Fn 1 , Fn 2 se n 2.

SOLUO DE ASDRUBAL PAFNCIO SANTOS (BOTUCATU SP) Podemos supor sem perda de generalidade que i = 0 e que o vrtice j e 2 j i n , para

0 j n 1. Queremos ento provar que


Temos
2

3 + 2cos
j =1
2 j i n

n 1

2 j n

2 = Fn . ;

e 2 j i n 1 = ( e 2 j i n 1)( e 2 j i n 1) = 2 e

2 j e 2 j i n = 2 2cos n

queremos provar portanto que


EUREKA! N33, 2011

49

Sociedade Brasileira de Matemtica

( f ( x ))
n

3 + 2cos
j =1 n0

n 1

2 j 2 = Fn . Considere agora a sequncia de polinmios n dada por f 0 ( x ) = 2, f1 ( x ) = x e f n +1 ( x ) = xf n ( x ) f n 1 ( x ) , n 1.

Temos, para todo n 0 e todo , f n ( 2cos ) = 2cos ( n ) . De fato isso vale para n = 0 e n = 1 e, por induo, f n+1 ( 2cos ) = 2cos f n ( 2cos ) f n1 ( 2cos ) = 4cos cos ( n ) 2cos ( ( n 1) ) = = 2cos ( ( n + 1) ) . Alm disso, para todo n 1, f n ( x ) um polinmio mnico de grau n. Como as soluo de 2cos ( n ) = 2 so dadas por =

2 k , k , temos n n 1 n 1 2 j f n ( x ) 2 2 k donde . f n ( x ) 2 = x 2cos , n 1, x 2cos = x2 n n j =1 k =0 O que queremos provar equivale a n 1 2 j f n ( 3) 2 1 n 1 = ( f n ( 3) 2 ) , o que ( 1) Fn2 = 3 2cos = n 3 2 5 j =1
equivalente a f n ( 3) = 2 + ( 1) 5 Fn2 . Como
n

n n 1 5 1 1+ 5 Fn = 2 2 , n 0, temos 5

3 + 5 3 5 2 + ( 1) 5F = + . Por outro lado, a sequncia 2 2 xn = f n ( 3) satisfaz x0 = 2, x1 = 3 e xn +1 = 3xn xn 1 , n 1, e logo (usando o


n

2 n

fato de as razes de x 2 + 3 x + 1 = 0 serem


o que prova a igualdade desejada.

3+ 5 3 5 3 5 ), xn = 2 + ,n 0, 2 2

136) Sejam R, r1 , r2 e r3 os raios dos crculos de centro O, O1 , O2 e O3 ,


respectivamente, conforme a figura abaixo. Prove que: R = r1r2 + r1r3 + r2 r3 .

EUREKA! N33, 2011

50

Sociedade Brasileira de Matemtica

O1 R R2 O O2 O3

SOLUO DE ANDERSON TORRES (SANTANA DE PARNABA SP) A boa e velha trigonometria... A


X ra

OA TA

Y r O TB OB B C OC

Como sempre, A = , B = e C = , + + = . Vamos calcular ra , para comear: AOA + OATA + TAO = AO AOA + OATA = AO OTA r r AOA X : AOA = a ; AOY ; AO = . Substituindo: sen sen 2 2

EUREKA! N33, 2011

51

Sociedade Brasileira de Matemtica

1 sen 1 sen 2 ra 2 +r = r ra = r = , ou a r 1 + sen sen sen 1 + sen 2 2 2 2 Como precisaremos de um quadrado, vamos aplicar um truque: a tangente do meio arco.
ra r

1 tan 2 = 4 = 4 4 = 4 . 1 + tan 1 + sen 2 tan 1 + 2 tan + tan 2 2 1+ 4 4 4 4 2 1 + tan 4 Com isto, j podemos substituir na igualdade que queremos demonstrar:
1 sen 1 + tan
2

2 tan

1 2 tan

+ tan 2

1 tan 1 + tan
Para


4 4

1 tan 1 + tan


4 + 4

1 tan 1 + tan


4 4

1 tan 1 + tan


4+ 4

1 tan 1 + tan


4 4

1 tan 1 + tan


4 =1 4

escrever

menos,

seja

a = tan

, b = tan

, c = tan . 4 4

Abrindo

os

denominadores, (1 a )(1 b )(1 + c ) + (1 a )(1 + b )(1 c ) + (1 + a )(1 b )(1 c ) = (1 + a )(1 + b )(1 + c )


3 ( a + b + c ) ( ab + ac + bc ) + 3 ( abc ) = 1 + ( a + b + c ) + ( ab + ac + bc ) + ( abc )

1 ( a + b + c ) ( ab + ac + bc ) + ( abc ) = 0 . Mas isto fcil?


+ tan + 4 4 4 = 1 = tan = tan + + = 4 4 4 4 1 tan tan + 4 4 4 tan

EUREKA! N33, 2011

52

Sociedade Brasileira de Matemtica

tan =

tan

+ tan

1 tan

tan


4 =

tan

+ tan

+ tan

tan tan

tan

tan

tan 4 + tan 4 1 tan 4


1 tan 4 tan 4

1 tan

tan

tan

tan

tan

a + b + c abc = 1 ( ab + ac + bc ) , como esperado.

137) Seja A um conjunto de quinze pontos de 2 tal que a distncia de cada ponto origem positiva e menor do que 1 e que quaisquer dois deles nunca sejam colineares com a origem. Mostre que existe um tringulo com dois vrtices em A e 1 um na origem cuja rea menor que . 4 SOLUO DE ITAMAR SALES DE OLIVEIRA FILHO (CEDRO CE)
1 x y
3 2

15

Distribumos aleatoriamente os 15 pontos. Como a distncia origem sempre menor do que 1, com certeza todos esses pontos so interiores circunferncia de raio 1 e centro na origem.
EUREKA! N33, 2011

53

Sociedade Brasileira de Matemtica

Pelo fato de no existerem dois colineares com o centro, temos os 15 ngulos representados na figura (1 , 2 ,...,15 ) . Obviamente:

1 , 2 ,...,15 = 360. Vamos provar que existe pelo menos um ngulo menor do que ou igual a 24 . Para isso, suponha o contrrio, ou seja, n > 24, para todo n Ento: 1 + 2 + ... + 15 > 15 24 = 360 absurdo. Ento realmente existe pelo menos um ngulo 24. Suponha i = XOY . Olhando para o tringulo XOY: 1 rea XOY = OX OY sen1 . Contudo, OX e OY so menores do que 1 e 2 sen i sen24 < sen30, substituindo: 1 1 1 1 A < 1 1 sen30 A < A < . 2 2 2 4 1 Ento existe um tringulo como no enunciado cuja rea menor do que . 4
138) Calcule o mximo divisor comum entre todos os nmeros da forma x y z ,
onde ( x, y, z ) percorre todas as solues inteiras da equao x 2 + y 2 = z 2 com x y z 0.

SOLUO DE MARCLIO MIRANDA DE CARVALHO (TERESINA PI) Seja d mdc entre todos os inteiros da forma x y z onde (x, y, z) percorre todas as
solues inteiras da equao x 2 + y 2 = z 2 com x y z 0. Note que (3, 4, 5) soluo, logo temos que d 60.
AFIRMAO 1: Se uma tripla

( x, y , z )

soluo da equao x 2 + y 2 = z 2 ento

x y z mltiplo de 3.
PROVA: Suponhamos, por absurdo, que x e y no so mltiplos de 3. Ento

z 2 = x 2 + y 2 2 ( mod 3) , absurdo!. Logo x ou y tem que ser mltiplo de 3. Assim temos que x y z mltiplo de 3.

AFIRMAO 2: Se uma tripla ( x, y, z ) soluo da equao x 2 + y 2 = z 2 ento

x y z mltiplo de 5.
EUREKA! N33, 2011

54

Sociedade Brasileira de Matemtica

PROVA: Suponhamos, por absurdo, que x e y no so mltiplos de 5. Ento

x 2 + y 2 0 ou 2 ou 3 (mod 5). No primeiro caso temos que z mltiplo de 5, portanto x y z mltiplo de 5. No segundo e terceiro casos temos um absurdo, pois um nmero quadrado perfeito s pode deixar restos 0, 1 ou 4 mdulo 5. Assim, temos que se ( x, y, z ) soluo da equao x 2 + y 2 = z 2 ento x y z mltiplo de 5.
AFIRMAO 3: Se uma tripla

( x, y , z )

soluo da equao x 2 + y 2 = z 2 ento

x y z mltiplo de 4.
PROVA:

Suponhamos, por absurdo, que x e y so mpares. Ento x 2 + y 2 2 ( mod 4 ) , absurdo. Portanto x ou y tem que ser par. Se x for par, mas no

for mltiplo de 4, ento x 2 4 ( mod8 ) x 2 + y 2 4 , 5 ou 0 (mod 8). No primeiro e terceiro casos temos que y par, portanto x y z mltiplo de 4. No segundo caso temos um absurdo, pois um nmero quadrado perfeito s pode deixar restos 0, 1 ou 4 mdulo 8. E se y for par anlogo. Assim, temos que x y z mltiplo de 4. Portanto d mltiplo de 3 4 5 = 60, logo d = 60.
139) Determine todos os inteiros positivos x, y, z satisfazendo x 3 y 3 = z 2 , onde y

primo, z no divisvel por 3 e z no divisvel por y.

SOLUO DE ADRIANO CARNEIRO TAVARES (CAUCAIA CE) Suponha que exista uma soluo. (I) Ento z 2 = x 3 y 3 = ( x y ) ( x 2 + xy + y 2 ) = ( x y )(( x y ) 2 + 3 xy ) Como z no divisvel por 3 e nem por y, e y um nmero primo, teremos por (I) mdc( x, y ) = 1 e mdc( x y, 3) = 1.
Ento mdc ( x 2 + xy + y 2 , x y ) = mdc (3 xy, x y ) = 1
2 2 2 2

(II)

Agora (I) e (II) implicam que x y = m , x + xy + y = n e z = mn , para certos inteiros positivos m e n. Temos 4n 2 = 4 x 2 + 4 xy + 4 y 2 = (2 x + y ) 2 + 3 y 2 . Ento 3 y 2 = (2n + 2 x + y )(2n 2 x y ). Sendo y um primo, ento existem trs possibilidades: a) 2n + 2 x + y = 3 y 2 , 2n 2 x y = 1

EUREKA! N33, 2011

55

Sociedade Brasileira de Matemtica

b) 2n + 2 x + y = 3 y, 2n 2 x y = y c) 2n + 2 x + y = y 2 , 2n 2 x y = 3 Em (a), aps a subtrao das equaes temos: 3 y 2 1 = 2(2 x + y ) = 2(2m 2 + 3 y ). Da, m 2 + 1 = 3 y 2 6 y 3m 2 0 (mod 3). Por outro lado, temos sempre m 2 + 1 1 ou 2 (mod 3). Ns chegamos a uma contradio. Em (b), subtraindo as equaes chegamos x = 0, o que absurdo! Subtraindo as equaes em (c), chegamos em y 2 3 = 2(2 x + y ) = 2(2m 2 + 3 y ), que pode ser escrito assim: ( y 3) 2 4m 2 = 12, ou seja, ( y 3 + 2m)( y 3 2m) = 12. Da equao chegamos a y = 7 e m = 1, pois devemos ter y 3 + 2m = 6 e

y 3 2m = 2 . Segue que x = y + m2 = 8 e z = 13 mn = m x + xy + y 2 .
Veja que de fato 83 73 = 132. Esta a nica soluo.

140) Mostre que 2903n 803n 464n + 261n divisvel por 1897, para todo n .

SOLUO DE MARCELO RIBEIRO DE SOUZA (RIO DE JANEIRO RJ) LEMA: Sejam p1 , p2 dois nmeros inteiros primos entre si. Ento, se p1 a e

p2 a , ter-se- p1 p2 a .
DEMONSTRAO: p1 a k

a = kp1. No entanto, deve-se ter p2 a = kp1 , ora,


que

como

( p1 , p2 ) = 1, conclui-se

p2 k k1

= k1 p2 .

Finalmente,

a = k1 p1 p2 p1 p2 a .
Note-se, inicialmente, que 1897 = 7 271. Escreva-se, ento:

2903n 464n 803n + 261n ( 78) ( 78) ( 10) + ( 10) 0 ( mod271) , n


n n n n

(i)

2903 464 803 + 261 5 2 5 + 2 0 ( mod 7 ) , n


n n n n n n n n

(ii) ,

Assim, temos, pelo Lema, que 2903n 464n 803n + 261n 0 ( mod1897 ) , n como se quis demonstrar.
EUREKA! N33, 2011

56

Sociedade Brasileira de Matemtica

141) Dado a {0,1, 2,3, 4,5,6,7,8,9} , seja X um conjunto finito de inteiros positivos, tal que nenhum dos seus elementos possui o algarismo a em sua 1 representao decimal. Prove que < 80. n X n SOLUO DE FABRCIO VASCONCELLOS PUPPI (SO PAULO SP) Por um simples raciocnio combinatrio, nota-se que a quantidade de inteiros positivos com k algarismos que no apresentam algum dgito a em sua representao decimal 8 9k 1 , caso a 0, e 9k , caso a = 0. Seja N = max( X ) e d o nmero de dgitos de N. Seja S(T) a operao definida sobre um subconjunto finito T qualquer de * , tal que:

S (T ) =

1 nT n

Pela definio de S, como cada elemento de T tem contribuio positiva no valor da soma que caracteriza a operao, claramente S montona em relao ao seu argumento, de tal modo que T Q S (T ) S ( Q ) . Assim, para

X = {` 1, 2,3,..., N ,...,10d 1} {n a no dgito de n} , X X . Para


k 1 k 1

todo

inteiro

positivo n de k dgitos, n 10 1 n 1 10 , sendo que a desigualdade estrita vale para todos os n de k dgitos exceto para um deles. Considerando inicialmente o caso em que a 0, tem-se:

n X

(8 9 ) 10 n< 1
11

1 1 1 d 8 9k 1 2 1 d 1 + 8 9 + ... + 8 9 = k 1 ( ) ( ) 11 1021 10d 1 k =1 10 9k 1 k 1 k =1 10


d

S ( X ) S X < 8

( )

Como a somatria direita uma srie geomtrica, trivialmente tem-se que:

S ( X ) < 8

1 ( 9 /10 )

1 ( 9 10 )

d = 80 1 ( 9 10 ) < 80

EUREKA! N33, 2011

57

Sociedade Brasileira de Matemtica

Para o caso a = 0 necessrio um refinamento de anlise, pois o uso de processo idntico ao acima s permitiria afirmar que S ( X ) < 90. Pra este caso, separou-se cada conjunto dos nmeros de k algarismos em dois subconjuntos disjuntos: um dos 4 9k 1 inteiros que satisfazem 10k 1 n < 5 10k 1 e outro dos 5 9k 1 inteiros que satisfazem 5 10k 1 n < 10k . Assim, por um raciocionio anlogo ao do caso anterior: S ( X ) S X < 4

( )

d d 9 k 1 9k 1 9 k 1 5 5 + = k 1 k 1 k 1 k =1 10 k =1 5 10 k =1 10 d

S ( X ) < 5

1 ( 9 10 )

d = 50 1 ( 9 10 ) < 50 < 80 . 1 ( 9 10 )

Agradecemos o envio de solues e a colaborao de:


Adriano Carneiro Tavares (Caucaia CE) Anderson Torres (Santana de Parnaba SP) Douglas Oliveira de Lima (Braslia DF) Fabrcio Vasconcellos Puppy (So Paulo SP) Flvio Antonio Alves (Amparo SP) Itamar Sales de Oliveira Fiolho (Cedro CE) Jean Pierre Youyoute (Rio de Janeiro RJ) Lucas Alves, Douglas Oliveira de Lima, Danillo Leal, Gustavo Campelo, Jlio Castro (Braslia DF) Lucas Colucci Marcelo Ribeiro de Souza (Rio de Janeiro RJ) Marclio Miranda de Carvalho (Teresina PI) Marcos Martinelli (Braslia DF) Matheus Henrique Alves Moura (Fortaleza CE) Renato Carneiro (Belo Horizonte MG) Prob. 140 Prob. 133, 137, 138, 139, 140, 141 Prob. 140 Prob. 137, 138, 140. Prob. 136 Prob. 136 Prob. 138 Prob. 136 Prob. 138 e 140 Prob. 136 Prob. 140 Prob. 133 e 136 Prob. 136 Prob. 140

Continuamos aguardando solues para os problemas 134 e 135.

EUREKA! N33, 2011

58

Sociedade Brasileira de Matemtica

PROBLEMAS PROPOSTOS
Convidamos o leitor a enviar solues dos problemas propostos e sugestes de novos problemas para prximos nmeros.

142) Seja A = {4,8,9,16, 25, 27,36,64,...} o conjunto das potncias no triviais


(nmeros da forma a b , com a 2, b 2 naturais). Prove que, para todo natural n 1, existe um natural k tal que todos os termos da progresso aritmtica k , 2k ,3k ,..., nk pertencem a A.

143)

f ( xy ) = g ( x + y ) + h ( x + y ) , x, y .
3 3

Determime

todas

as

funes

f , g,h :

tais

que

144) Seja x 1 um nmero racional tal que existe uma constante c 0 e uma sequncia ( an )n1 de inteiros tal que lim ( cx n an ) = 0. Prove que x inteiro.
n

145)

todos os nmeros 2 3 p cos + q cos + r cos = 1. 7 7 7

Encontre

racionais

p,

q,

de

modo

que

146) Determine todos os subconjuntos no-vazios A, B, C de

de modo que:

a) A B = B C = C A = . b) A B C = . c) para quaisquer a A, b B e c C , temos: a + c A, b + c B e a + b C.

147) Demonstre que

( 1) ( 1) =n+ 2 ( 2k + 1) k =0 sen
n2 k

, para todo inteiro n 2.

4n 2

148) Sejam m e n inteiros positivos. Calcule

n k . m k =0
n 1

149) a) Deseja-se organizar um torneio de futebol com n times ( n 2 ) em que cada time joga uma vez contra cada um dos outros, dividido em um certo nmero de rodadas. Em cada rodada cada time joga no mximo uma partida.
EUREKA! N33, 2011

59

Sociedade Brasileira de Matemtica

Prove que, se n mpar, possvel organizar um tal torneio com n rodadas e, se n par, possvel organizar um tal torneio com n 1 rodadas. b) Uma matriz n n preenchida com elementos do conjunto S = {1, 2,3,..., 2n 1} . Sabe-se que, para todo i {1, 2,..., n} , a i-sima linha e a isima coluna contm juntas todos os elementos de S. Quais os possveis valores de n?

150) Sejam a, b e c nmeros reais tais que ( a b ) + ( b c ) + ( c a ) = 9.


3 3 3

Prove que

(a b)

( a b)

(c a)

3 3.

Problema 142 adaptado de um problema proposto por Anderson Torres (Santana de Parnaba SP); 143 proposto por Anderson Torres (Santana de Parnaba SP); 144, 145 e 146 propostos por Carlos da Silva Ramos (Belm PA); 147 e 148 propostos por Marcos Martinelli; 149 adapatado de um problema proposto por Anderson Torres (Santana de Parnaba SP); 150 adaptado de um problema proposto por Adriano Carneiro (Caucaia CE).

EUREKA! N33, 2011

60

Sociedade Brasileira de Matemtica

AGENDA OLMPICA
XXXIII OLIMPADA BRASILEIRA DE MATEMTICA NVEIS 1, 2 e 3 Primeira Fase sbado, 18 de junho de 2011 Segunda Fase sbado, 3 de setembro de 2011 Terceira Fase sbado, 15 de outubro de 2011 (nveis 1, 2 e 3) domingo, 16 de outubro de 2011 (nveis 2 e 3 - segundo dia de prova) NVEL UNIVERSITRIO Primeira Fase sbado, 3 de setembro de 2011 Segunda Fase sbado, 15 e domingo, 16 de outubro de 2011 IV ROMANIAN MASTER OF MATHEMATICS (RMM) 23 a 28 de fevereiro de 2011(Bucareste, Romnia) ASIAN PACIFIC MATH OLYMPIAD (APMO) 12 de maro de 2011 XVII OLIMPADA DE MAIO 7 de maio de 2011 XXII OLIMPADA DE MATEMTICA DO CONE SUL 14 a 20 de agosto de 2011(La Paz, Bolvia) LII OLIMPADA INTERNACIONAL DE MATEMTICA 13 a 24 de julho de 2011(Amsterdam, Holanda) I OLIMPADA DE MATEMTICA DA LUSOFONIA 20 a 31 de julho de 2011(Coimbra, Portugal) XVII OLIMPADA INTERNACIONAL DE MATEMTICA UNIVERSITRIA (IMC) 24 a 30 de julho de 2011(Blagoevgrad, Bulgria) XXV OLIMPADA IBEROAMERICANA DE MATEMTICA 23 de setembro a 1 de outubro de 2011(So Jos, Costa Rica) II COMPETIO IBEROAMERICANA INTERUNIVERSITRIA DE MATEMTICA 2 a 8 de outubro de 2011(Quito, Equador) XIII OLIMPADA IBEROAMERICANA DE MATEMTICA UNIVERSITRIA

EUREKA! N33, 2011

61

Sociedade Brasileira de Matemtica

COORDENADORES REGIONAIS
Alberto Hassen Raad Amrico Lpez Glvez Antonio Carlos Nogueira Benedito Tadeu Vasconcelos Freire Bruno Holanda Carmen Vieira Mathias Claus Haetinger Cludio de Lima Vidal Denice Fontana Nisxota Menegais Disney Douglas Lima de Oliveira Edson Roberto Abe Edney Aparecido Santulo Jr. Fbio Brochero Martnez Florncio Ferreira Guimares Filho Francinildo Nobre Ferreira Genildo Alves Marinho Herivelto Martins Gilson Tumelero Ivanilde Fernandes Saad Joo Bencio de Melo Neto Joo Francisco Melo Libonati Diogo Diniz Jos Luiz Rosas Pinho Jos Vieira Alves Jos William Costa Krerley Oliveira Licio Hernandes Bezerra Luciano G. Monteiro de Castro Luzinalva Miranda de Amorim Marcelo Dias Marcelo Antonio dos Santos Marcelo Rufino de Oliveira Newman Simes Nivaldo Costa Muniz Osnel Broche Cristo Uberlndio Batista Severo Raul Cintra de Negreiros Ribeiro Reginaldo de Lima Pereira Reinaldo Gen Ichiro Arakaki Ricardo Amorim Ronaldo Alves Garcia Rogrio da Silva Igncio Rosangela Ramon Srgio Cludio Ramos Seme Gebara Neto Tadeu Ferreira Gomes Toms Menndez Rodrigues Valdenberg Arajo da Silva Vnia Cristina Silva Rodrigues Wagner Pereira Lopes Wanderson Breder William Serafim dos Reis
EUREKA! N33, 2011

(UFJF) (USP) (UFU) (UFRN) (CAEN UFC) (UNIFRA) (UNIVATES) (UNESP) (UNIPAMPA) (UFAM) (Colgio Objetivo de Campinas) (UEM) (UFMG) (UFES) (UFSJ) (Centro Educacional Leonardo Da Vinci) (USP So Carlos) (UTFPR) (UC. Dom Bosco) (UFPI) (Grupo Educacional Ideal) (UFPB) (UFSC) (UFPB) (Instituto Pueri Domus) (UFAL) (UFSC) (Sistema Elite de Ensino) (UFBA) (Grupo Educacional Etapa) FACOS (Grupo Educacional Ideal) (Cursinho CLQ Objetivo) (UFMA) (UFLA) (UFPB) (Colgio Anglo) (Escola Tcnica Federal de Roraima) (UNIFESP) (Centro Educacional Logos) (UFGO) (Col. Aplic. da UFPE) (UNOCHAPEC) (IM-UFRGS) (UFMG) (UEBA) (U. Federal de Rondnia) (U. Federal de Sergipe) (U. Metodista de SP) (CEFET GO) (CEFET RJ) (UFT TO)

Juiz de Fora MG Ribeiro Preto SP Uberlndia MG Natal RN Fortaleza CE Santa Mara RS Lajeado RS S.J. do Rio Preto SP Bag RS Manaus AM Campinas SP Maring PR Belo Horizonte MG Vitria ES So Joo del Rei MG Taguatingua DF So Carlos SP Pato Branco PR Campo Grande MS Teresina PI Belm PA Campina Grande PB Florianpolis SC Campina Grande PB Santo Andr SP Macei AL Florianpolis SC Rio de Janeiro RJ Salvador BA So Paulo SP Osrio RS Belm PA Piracicaba SP So Luis MA Lavras MG Joo Pessoa PB Atibaia SP Boa Vista RR SJ dos Campos SP Nova Iguau RJ Goinia GO Recife PE Chapec SC Porto Alegre RS Belo Horizonte MG Juazeiro BA Porto Velho RO So Cristvo SE S.B. do Campo SP Jata GO Nova Friburgo RJ Arraias TO

62

CONTEDO
XXXII OLIMPADA BRASILEIRA DE MATEMTICA Problemas e solues da Primeira Fase XXXII OLIMPADA BRASILEIRA DE MATEMTICA Problemas e solues da Segunda Fase XXXII OLIMPADA BRASILEIRA DE MATEMTICA Problemas e solues da Terceira Fase XXXII OLIMPADA BRASILEIRA DE MATEMTICA Problemas e solues da Primeira Fase Nvel Universitrio XXXII OLIMPADA BRASILEIRA DE MATEMTICA Problemas e solues da Segunda Fase Nvel Universitrio XXXII OLIMPADA BRASILEIRA DE MATEMTICA Premiados AGENDA OLMPICA COORDENADORES REGIONAIS 85 86 78 67 59 34 15 3

Sociedade Brasileira de Matemtica

Esta edio dedicada memria do professor Sergio Plaza Salinas da Universidad de Santiago de Chile, que colaborou como membro do comit editorial da revista Eureka! desde 1998, e que nos deixou neste ano de 2011.

Os editores

EUREKA! N34, 2011

Sociedade Brasileira de Matemtica

XXXII OLIMPADA BRASILEIRA DE MATEMTICA


Problemas e solues da Primeira Fase
PROBLEMAS NVEL 1

1. Qual dos nmeros a seguir no mltiplo de 15? A) 135 B) 315 C) 555 D) 785

E) 915

2. Ana, Esmeralda e Lcia tm, juntas, 33 reais. Ana e Esmeralda, juntas, tm 19 reais e Esmeralda e Lcia, juntas, tm 21 reais. Quantos reais tem Esmeralda? A) 6 B) 7 C) 10 D) 12 E) 14 3. Aumentando 2% o valor um nmero inteiro positivo, obtemos o seu sucessor. Qual a soma desses dois nmeros? A) 43 B) 53 C) 97 D) 101 E) 115

4. Qual o maior nmero de fichas que podemos colocar em um tabuleiro 5 5 , no mximo uma em cada casa, de modo que o nmero de fichas em cada linha e cada coluna seja mltiplo de 3? A) 6 B) 9 C) 12 D) 15 E) 24 5. Carlos tem 2010 blocos iguais de 10 cm de largura por 20 cm de comprimento e 1,5 cm de espessura e resolveu empilh-los formando uma coluna de 20 cm de largura por 40 cm de comprimento, como na figura. Qual dos valores a seguir, em metros, o mais prximo da altura dessa coluna? A) 7 B) 7,5 C) 8 D) 8,5 E) 9

6. Qual das alternativas apresenta um divisor de 35 44 53 ? A) 42 B) 45 C) 52 D) 85 7. Dividindo-se o nmero 4 A) 2 B) 43

E) 105

(4 )
2

por 44 obtemos o nmero: C) 44 D) 48

E) 412

EUREKA! N34, 2011

Sociedade Brasileira de Matemtica

8. As quatro faces de um dado so tringulos equilteros, numerados de 1 a 4, como no desenho. Colando-se dois dados iguais, fazemos coincidir duas faces, com o mesmo nmero ou no. Qual dos nmeros a seguir no pode ser a soma dos nmeros das faces visveis? A) 12 B) 14 C) 17 D) 18 E) 19 9. Quantos divisores positivos de 120 so mltiplos de 6? A) 4 B) 5 C) 6 D) 8 10. O desenho mostra dois quadrados de papel sobrepostos, um de lado 5 cm e outro de lado 6 cm. Qual o permetro da figura formada (linha grossa no contorno do desenho), em centmetros? A) 31 B) 34 C) 36 D) 38 E) 41 E) 12

11. O horrio indicado pelo relgio ao lado est correto. A partir desse momento, porm, o relgio comea a atrasar exatamente 5 minutos a cada hora real. Depois de quantos dias o relgio voltar a apresentar um horrio correto? A) 1 B) 2 C) 4 D) 6 E) 12

12. No reticulado a seguir, pontos vizinhos na vertical ou na horizontal esto a 1 cm de distncia.


1cm 1cm

Qual a rea da regio sombreada? A) 7 B) 8 C) 8,5


EUREKA! N34, 2011

D) 9
4

E) 9,5

Sociedade Brasileira de Matemtica

13. Um jornal publicou a tabela de um campeonato de futebol formado por quatro times, apresentando os gols marcados e os gols sofridos por cada time. Por uma falha de impresso, a tabela saiu com dois nmeros borrados, conforme reproduo a seguir. Craques do Momento Independentes EC Boleiros Esmeralda FC Gols marcados 8 1 4 5 Gols sofridos 4 6 *** ***

Sabe-se que o time Esmeralda FC sofreu dois gols a mais que o time EC Boleiros. Quantos gols sofreu o time Esmeralda FC? A) 2 B) 3 C) 4 D) 5 E) 6 14. Ana comeou a descer uma escada no mesmo instante em que Beatriz comeou a subi-la. Ana tinha descido

3 da escada quando cruzou com Beatriz. No 4 1 12 5 12 2 3

momento em que Ana terminar de descer, que frao da escada Beatriz ainda ter que subir? A)

1 4

B)

1 3

C)

D)

E)

15. Alguns nmeros inteiros positivos, no necessariamente distintos, esto escritos na lousa. A soma deles 83 e o produto 1024. O menor nmero igual a: A) 1 B) 2 C) 4 D) 8 E) 16 16. Numa sala do 6 ano, todos gostam de pelo menos uma das duas matrias: Matemtica ou Portugus. Sabe-se que

3 5 dos alunos gostam de Matemtica e 4 7

dos alunos gostam de Portugus. A sala tem 56 alunos. Quantos alunos gostam dessas duas matrias ao mesmo tempo? A) 4 B) 8 C) 13 D) 24 E) 26

EUREKA! N34, 2011

Sociedade Brasileira de Matemtica

17. O desenho representa um canto de um tabuleiro retangular convencional, formado por quadradinhos de lado 1 cm. Nesse tabuleiro, 17 quadradinhos so brancos. Qual a rea do tabuleiro, em centmetros quadrados? A) 29 E) 150 B) 34 C) 35 D) 40

18. A figura representa uma barra de chocolate que tem um amendoim apenas num pedao. Elias e Fbio querem repartir o chocolate, mas nenhum deles gosta de amendoim. Ento combinam dividir o chocolate quebrando-o ao longo das linhas verticais ou horizontais da barra, um depois do outro e retirando o pedao escolhido, at que algum tenha que ficar com o pedao do amendoim. Por sorteio, coube a Elias comear a diviso, sendo proibido ficar com mais da metade do chocolate logo no comeo. Qual deve ser a primeira diviso de Elias para garantir que Fbio fique com o amendoim ao final? A) Escolher a primeira coluna esquerda. B) Escolher as duas primeiras colunas esquerda. C) Escolher a terceira linha, de cima para baixo. D) Escolher as duas ltimas linhas, de cima para baixo. E) Qualquer uma, j que Fbio forosamente ficar com o amendoim. 19. Quatro amigos, Arnaldo, Bernaldo, Cernaldo e Dernaldo esto jogando cartas. So 20 cartas diferentes, cada carta tem uma entre 4 cores (azul, amarelo, verde, vermelho) e um nmero de 1 a 5. Cada amigo recebe cinco cartas, de modo que todas as cartas so distribudas. Eles fazem as seguintes afirmaes: Arnaldo: Eu tenho quatro cartas com o mesmo nmero. Bernaldo: Eu tenho as cinco cartas vermelhas. Cernaldo: As minhas cinco cartas so de cores que comeam com a letra V. Dernaldo: Eu tenho trs cartas de um nmero e duas cartas de outro nmero. Sabe-se que somente uma das afirmaes falsa. Quem fez essa afirmao? A) Arnaldo B) Bernaldo C) Cernaldo D) Dernaldo E) No possvel definir.
EUREKA! N34, 2011

Sociedade Brasileira de Matemtica

20. A figura a seguir foi recortada em cartolina e depois dobrada para formar um icosaedro. As faces em branco foram numeradas de modo que ao redor de cada vrtice (pontas do slido) apaream os nmeros de 1 a 5. Qual nmero est na face com a interrogao?

ICOSAEDRO
A) 1 B) 2 C) 3 D) 4 E) 5

PROBLEMAS NVEL 2

1. Veja o problema No. 6 do Nvel 1. 2. Aumentando em 2% o valor do menor de dois nmeros consecutivos, obtm-se o maior deles. Qual a soma desses nmeros? A) 43 B) 53 C) 97 D) 101 E) 115 3. Veja o problema No. 7 do Nvel 1 4. Ceclia pegou uma cartolina e recortou as 8 peas direita, formadas por quadradinhos de mesmo tamanho. De quantas maneiras diferentes ela pode escolher 3 dessas peas para montar o quadrado 3 3 esquerda? A) 3 B) 4 C) 5

D) 6

E) 7

5. Os nmeros x e y so distintos e satisfazem x A) 4 B) 1 E) preciso de mais dados.


EUREKA! N34, 2011

C) 1

1 1 = y . Ento xy igual a x y D) 4

Sociedade Brasileira de Matemtica

6. Snia calculou a mdia aritmtica de dois diferentes nmeros de dois dgitos e obteve 98. Qual a diferena entre esses nmeros? A) 1 B) 2 C) 3 D) 4 E) um nmero maior que 4 7. Veja o problema No. 17 do Nvel 1. 8. Quantos inteiros da lista 100, 101, 102, ..., 999 no possuem algarismos iguais a 2, 5, 7 ou 8? A) 160 B) 170 C) 180 D) 190 E) 200 9. No tringulo ABC, m(BC) = 140o. Sendo M o ponto mdio de BC, N o ponto mdio de AB e P o ponto sobre o lado AC tal que MP perpendicular a AC, qual P ? a medida do ngulo NM o o A) 40 B) 50 C) 70o D) 90o E) 100o 10. Veja o problema No. 4 do Nvel 1 11. Para quantos inteiros n o nmero A) 1 B) 6

n tambm inteiro? 100 n C) 10 D) 18 E) 100

12. Ana comeou a descer uma escada de 24 degraus no mesmo instante em que 3 da escada quando cruzou com Beatriz comeou a subi-la. Ana tinha descido 4 Beatriz. No momento em que Ana terminar de descer, quantos degraus Beatriz ainda ter que subir? A) 2 B) 6 C) 8 D) 10 E) 16 13. Veja o problema 19 do Nvel 1. 14. No desenho, o retngulo cinza tem seus vrtices sobre os lados do tringulo equiltero de rea 40 cm2. O menor lado do retngulo um quarto do lado do tringulo. A rea do retngulo em cm2 :

EUREKA! N34, 2011

Sociedade Brasileira de Matemtica

A) 5

B) 10

C) 15

D) 18

E) 22

15. Veja o problema No. 15 do Nvel 1. 16. De quantas maneiras possvel desenhar a figura a seguir sem tirar o lpis do papel (ou qualquer outro utenslio, se voc preferir!) comeando de P e sem passar sobre o mesmo ponto mais de uma vez, com exceo do ponto comum aos trs tringulos?

A) 48

B) 24

C) 16

D) 108

E) 27

17. Os pontos P, Q, R, S e T so vrtices de um polgono regular. Os lados PQ e TS S mede so prolongados at se encontrarem em X, como mostra a figura, e QX o 140 . Quantos lados o polgono tem?

A) 9

B) 18

C) 24

D) 27

E) 40

18. Veja o Problema No. 20 do Nvel 1. 19. O professor Piraldo tem dois relgios, ambos digitais de 24 horas. Nenhum dos dois funciona: um muda de horrio com o dobro da velocidade normal e o outro vai
EUREKA! N34, 2011

Sociedade Brasileira de Matemtica

de trs para frente, na velocidade normal. Ambos mostram corretamente 13:00. Qual a hora certa na prxima vem em que os dois relgios mostrarem o mesmo horrio? A) 05:00 B) 09:00 C) 13:00 D) 17:00 E) 21:00

20. Uma figura no formato de cruz, formada por quadrados de lado 1, est inscrita em um quadrado maior, cujos lados so paralelos aos lados do quadrado tracejado, cujos vrtices so vrtices da cruz. Qual a rea do quadrado maior?

A) 9

B)

49 5

C) 10

D)

81 8

E)

32 3

21. Quantos so os pares (x, y) de inteiros positivos tais que x2 y2 = 22010? A) 1000 B) 1001 C) 1002 D) 1003 E) 1004 22. Quatro nmeros inteiros positivos a < b < c < d so tais que o mdc entre quaisquer dois deles maior do que 1, mas mdc(a, b, c, d) = 1. Qual o menor valor possvel para d? A) 10 B) 12 C) 15 D) 30 E) 105 23. Veja o problema No. 8 do Nvel 1. 24. Na figura, BC = 2BH.
B
x 80o

H
30o

A) 10o

B) 15o

C) 16o

D) 20o

E) 25o

EUREKA! N34, 2011

10

Sociedade Brasileira de Matemtica

25. Os nmeros a e b so reais no negativos tais que a3 + a < b b3. Ento A) b < a < 1 B) a = b = 1 C) a < 1 < b D) a < b < 1 E) 1 < a < b
PROBLEMAS NVEL 3

1. Dividindo-se o nmero 4 A) 2 B) 43

( 4 ) por 44 obtemos o nmero:


2

C) 44

D) 48

E) 412

2. Qual dos seguintes nmeros um divisor de 35 4 4 53 ? A) 42 B) 45 C) 52 D) 85 3. Veja o Problema No. 8 do Nvel 1. 4. Veja o Problema No. 14 do Nvel 1.

E) 105

5. Um quadrado PQRS tem lados medindo x. T o ponto mdio de QR e U o p da perpendicular a QS que passa por T. Qual a medida de TU?

A)

x 2

B)

x 3

C)

x 2

D)

x 2 2

E)

x 4

6. Os nmeros x e y so distintos e satisfazem x A) 4 B) 1 E) preciso de mais dados C) 1

1 1 = y . Ento xy igual a x y D) 4

7. Considere todos os nmeros de trs algarismos distintos, cada um igual a 0, 1, 2, 3 ou 5. Quantos desses nmeros so mltiplos de 6? A) 4 B) 7 C) 10 D) 15 E) 20
EUREKA! N34, 2011

11

Sociedade Brasileira de Matemtica

8. O mximo divisor comum de todos os nmeros que so o produto de cinco mpares positivos consecutivos A) 1 B) 3 C) 5 D) 15 E) 105 9. Veja o problema 17 do Nvel 2. 10. Veja o problema 19 do Nvel 1. 11. Esmeralda ia desenhar o grfico de y = 2x + 6 mas trocou os eixos de lugar. Como fica o desenho dessa relao com os eixos trocados de lugar?

12. Qual das seguintes fraes mais prxima de 7 ? 3 5 8 13 A) B) C) D) 1 2 3 5

E)

18 7

13. No tringulo ABC, m(BC) = 140. Sendo M o ponto mdio de BC, N o ponto mdio de AB e P o ponto sobre o lado AC tal que MP perpendicular a AC, qual ? a medida do ngulo NMP A) 40 B) 50 C) 70 D) 90 E) 100 14. Veja o problema 16 do Nvel 2. 15. Veja o problema No. 20 do Nvel 1. 16. Os nmeros a e b so reais no negativos tais que a3 + a < b b3. Ento A) b < a < 1 B) a = b = 1 C) a < 1 < b D) a < b < 1 E) 1 < a < b 17. Quantos so os pares (x, y) de inteiros positivos tais que x2 y2 = 22010? A) 1000 B) 1001 C) 1002 D) 1003 E) 1004

EUREKA! N34, 2011

12

Sociedade Brasileira de Matemtica

18. Veja o problema No. 8 do Nvel 1. 19. Seja ABC um tringulo e X, Y e Z pontos sobre os lados BC, CA, AB tais que
CX AY BZ = = =2. XB YC ZA

A Z Y B X C

A razo entre as reas do tringulo XYZ e do tringulo cujos lados so congruentes s medianas de ABC : Obs.: as medianas de um tringulo so os segmentos que ligam os vrtices do tringulo aos pontos mdios dos lados opostos. 2 1 4 1 1 A) B) C) D) E) 3 2 9 3 4

20. Para cada subconjunto A de {1;2;3;4;5;6;7;8;9;10}, seja p(A) o produto de seus elementos. Por exemplo, p({1;2;4;5}) = 40 e p(A) = 10! = 1 2 3 10 . Por conveno, adote p ( ) = 1. A soma de todos os 210 produtos p(A) igual a: A) 211 B) 11! C) 1111 D) 211! E) 112! 21. Sendo n = 20102010 e log n igual ao nmero m tal que 10m = n, ento A) n! < nlog n < (log n)n B) nlog n < n! < (log n)n C) (log n)n < nlog n < n! D) (log n)n < n! < nlog n E) nlog n < (log n)n < n! 22. Quatro nmeros inteiros positivos a < b < c < d so tais que o mdc entre quaisquer dois deles maior do que 1, mas mdc(a, b, c, d) = 1. Qual o menor valor possvel para d? A) 10 B) 12 C) 15 D) 30 E) 105 23. Qual o maior valor de xy2 se x e y so reais positivos cuja soma 3? A) 3 B) 4 C) 5 D) 6 E) 7
EUREKA! N34, 2011

13

Sociedade Brasileira de Matemtica

24. Um ponto P escolhido ao acaso no interior de um quadrado QRST. Qual a ser agudo? probabilidade do ngulo RPQ

A)

3 4

B)

2 1

C)

1 2

D)

E) 1

25. Qual o menor valor positivo de 21m2 n2 para m e n inteiros positivos? A) 1 B) 2 C) 3 D) 5 E) 7

GABARITO
NVEL 1 (6. ou 7. Anos do Ensino Fundamental) 1) D 6) B 11) D 2) B 7) E 12) B 3) D 8) E 13) D 4) D 9) C 14) E 5) B 10) D 15) A NVEL 2 (8. ou 9. Anos do Ensino Fundamental) 1) B 6) B 11) D 2) D 7) C 12) E 3) E 8) C 13) B 4) E 9) D 14) C 5) C 10) D 15) A NVEL 3 (Ensino Mdio) 1) E 6) C 2) B 7) B 3) E 8) D 4) E 9) D 5) D 10) B 11) E 12) C 13) D 14) A 15) D 16) E 17) C 18) A 19) B 20) D 16) A 17) D 18) D 19) E 20) B 16) D 17) E 18) A 19) C 20) B 21) E 22) C 23) E 24) Anulada 25) D 21) E 22) C 23) B 24) E 25) C

EUREKA! N34, 2011

14

Sociedade Brasileira de Matemtica

XXXII OLIMPADA BRASILEIRA DE MATEMTICA


Problemas e solues da Segunda Fase
PROBLEMAS NVEL 1 PARTE A (Cada problema vale 5 pontos) 01. Uma jarra contm

1 de sua capacidade em gua. Despejando um copo cheio de 4


3

gua na jarra, o volume de gua atinge 1 da sua capacidade. Quantos copos cheios mais ainda sero necessrios para acabar de encher a jarra?
02. Joozinho tem que fazer uma multiplicao como lio de casa, mas a chuva

molhou o caderno dele, borrando alguns algarismos, que esto representados por (cada algarismo borrado pode ser diferente dos outros). 1

4 0 1 0

2 4 2 0

3 + 2

Qual a soma dos algarismos que foram borrados?


03. Soninha pintou as seis faces de um cubo da seguinte

maneira: uma face preta e a face oposta vermelha, uma face amarela e a face oposta azul, uma face branca e a oposta verde. Ao olhar para o cubo, de modo a ver trs faces, como na figura, e considerando apenas o conjunto das cores das trs faces visveis, de quantas maneiras diferentes pode ser visto esse cubo?
04. Esmeralda foi escrevendo os quadrados dos nmeros inteiros positivos um em

seguida ao outro formando o nmero 149162536... e parou quando chegou no centsimo algarismo. Qual foi o ltimo algarismo que ela escreveu?

EUREKA! N34, 2011

15

Sociedade Brasileira de Matemtica

05. Carlinhos escreve nmeros inteiros positivos diferentes e menores do que 1000

em vrias bolas e coloca-as numa caixa, de modo que Mariazinha possa pegar ao acaso duas dessas bolas. Quantas bolas no mximo Carlinhos ir colocar na caixa se os nmeros das duas bolas devero ter um divisor comum maior do que 1?
06. Num concurso com 10 questes, cada resposta correta valia 3 pontos, cada

resposta errada valia 1 ponto negativo e cada questo no respondida valia 0 ponto. No houve dois candidatos que apresentassem a mesma nota, feitas as correes. Quantos candidatos no mximo fizeram essa prova?
PROBLEMAS NVEL 1 PARTE B (Cada problema vale 10 pontos) PROBLEMA 1

Com cinco quadrados com lados de 27 cm, formamos uma sequncia de figuras, das quais as quatro primeiras so:

a) Na 4 figura, qual a rea do quadrado cinza? b) Na 5 figura, qual a rea do quadrado cinza?
PROBLEMA 2

Maria tem 90 cartes. Ela numerou os cartes de 10 a 99 numa das faces e, para cada nmero escrito, escreveu a soma dos seus algarismos na outra face. Por exemplo, o carto de nmero 43 tem o nmero 7 escrito no verso. Em quais cartes um nmero de uma face o dobro do nmero escrito na outra face?

EUREKA! N34, 2011

16

Sociedade Brasileira de Matemtica

PROBLEMA 3

Fazendo trs cortes num quadrado 3 3 e juntando as quatro partes resultantes a um quadrado 4 4 , obtemos um quadrado 5 5 , conforme indicado na figura. Os cortes devem ser paralelos aos lados dos quadrados e no pode haver sobreposio de figuras para a realizao dos cortes.
a) Transforme um quadrado de lado 8 cm e um quadrado de lado 15 cm num nico quadrado de lado 17 cm, fazendo quatro cortes apenas no quadrado de 8 cm. b) Qual o menor nmero de cortes para transformar trs quadrados, de reas respectivamente iguais a 4 cm2, 9 cm2 e 36 cm2, num nico quadrado?
PROBLEMAS NVEL 2 PARTE A (Cada problema vale 5 pontos) 01. Seja N o menor nmero inteiro positivo que multiplicado por 33 resulta em um nmero cujos algarismos so todos iguais a 7. Determine a soma dos algarismos de N. 02. Na figura seguinte, os tringulos ABC e ABD so retngulos em A e D,

respectivamente. Sabendo que AC = 15 cm, AD = 16 cm e BD = 12 cm, determine, em cm2, a rea do tringulo ABE. C
D E

A
2

B pq + p + 1 . q

03. Sejam p, q nmeros reais satisfazendo as relaes 2p 3p 1 = 0, q2 + 3q 2 =

0 e pq 1. Ache o valor de

EUREKA! N34, 2011

17

Sociedade Brasileira de Matemtica

04. Em uma cidade arbitrria o prefeito organizou uma rifa com bilhetes numerados

de 100 a 999. O prmio de cada bilhete determinado pela soma dos algarismos do nmero do bilhete. Para que ningum leve trs prmios iguais, estabeleceu-se que quem retirar trs bilhetes com as trs somas iguais tem direito a um superprmio. Qual o menor nmero de bilhetes que um cidado deve comprar para ter a certeza de que vai receber um superprmio?
05. Sejam r e s nmeros inteiros. Sabe-se que a equao do segundo grau

x2 (r + s)x + rs + 2010 = 0 tem as duas solues inteiras. Quantos so os possveis valores de |r s|?
PROBLEMAS NVEL 2 PARTE B (Cada problema vale 10 pontos) PROBLEMA 1

Joozinho deseja colorir um tabuleiro 2 2010 com duas cores A e B. Uma colorao dita legal se no possvel encontrar um L-trimin, como na figura abaixo, com todos os seus quadradinhos de mesma cor. Determine o nmero de coloraes legais.

L Trimin

Veja abaixo duas coloraes que no so legais:

PROBLEMA 2

Determine todos os nmeros primos m e n tais que 0 < m < n e os trs nmeros 2m + n, m + 2n e m + n 18 sejam tambm primos.]
PROBLEMA 3

Chamaremos de imagem de um nmero natural de dois algarismos o nmero que se


EUREKA! N34, 2011

18

Sociedade Brasileira de Matemtica

obtm trocando a ordem de seus algarismos. Por exemplo, a imagem de 34 43. Quais so os nmeros de dois algarismos que somados com sua imagem resultam em um quadrado perfeito?

e C do tringulo ABC cortam-se no ponto I. As bissetrizes internas dos ngulos A A) = 2m( IA C ) . Determine a medida do ngulo Sabe-se que AI = BC e que m( IC

PROBLEMA 4

C . AB
PROBLEMAS NVEL 3 PARTE A (Cada problema vale 4 pontos)

01. Seja N o menor nmero inteiro positivo que multiplicado por 33 resulta em um nmero cujos algarismos so todos iguais a 7. Determine a soma dos algarismos de N. 02. Sejam r e s nmeros inteiros. Sabe-se que a equao do segundo grau

x2 (r + s)x + rs + 2010 = 0 tem as duas solues inteiras. Quantos so os possveis valores de |r s|?
03. Na figura a seguir, as trs circunferncias em trao contnuo so tangentes s

retas r e s e a circunferncia tracejada passa pelos pontos A, B, C e D. Alm disso, a circunferncia menor tangente tambm a AD e a circunferncia maior tambm tangente a BC. Se os raios das circunferncias externas ao quadriltero ABCD so 8 e 18, calcule o raio R da circunferncia inscrita em ABCD. r A
8

B R C

18

D s

EUREKA! N34, 2011

19

Sociedade Brasileira de Matemtica

04. Cada uma das oito casas de um retngulo de duas linhas e quatro colunas

pintada de uma entre trs cores. Uma coluna chamada de corte se as suas duas casas so da mesma cor. De quantas maneiras possvel pintar o retngulo de modo que haja exatamente um corte?
05. Calcule

(2 (1

+ 2 2 + 1 4 4 + 4 2 + 1 6 4 + 6 2 + 1 32 4 + 32 2 + 1 4 + 12 + 1 3 4 + 3 2 + 1 5 4 + 5 2 + 1 314 + 312 + 1

)( )(

)( )(

) ( ) (

PROBLEMAS NVEL 3 PARTE B (Cada problema vale 10 pontos)

e C do tringulo ABC cortam-se no ponto I. As bissetrizes internas dos ngulos A A) = 2m( IA C ) . Determine a medida do ngulo Sabe-se que AI = BC e que m( IC C . AB
PROBLEMA 2

PROBLEMA 1

Diamantino gosta de jogar futebol, mas se jogar dois dias seguidos ele fica com dores musculares. De quantas maneiras Diamantino pode escolher em quais de dez dias seguidos ele vai jogar bola sem ter dores musculares? Uma maneira no jogar futebol em nenhum dos dias.
PROBLEMA 3

Resolva o sistema

x + y + z = 77 xy + yz + zx + xyz = 946
sendo x y z inteiros no negativos.
PROBLEMA 4

Uma mesa de bilhar tem o formato de um quadrado ABCD. SuperPablo tem uma misso especial: ele deve dar uma tacada em uma bola de bilhar, inicialmente colocada no vrtice A, de modo que, aps bater exatamente 2010 vezes nos lados do quadrado, a bola chegue, pela primeira vez, a um vrtice do quadrado.

EUREKA! N34, 2011

20

Sociedade Brasileira de Matemtica

Quantos so os possveis valores do ngulo formado pelo lado AB com a trajetria inicial da bola?

Observao: ao bater nos lados do quadrado, a bola sofre reflexo perfeita, ou seja, o ngulo de incidncia igual ao ngulo de reflexo. Suponha tambm que a bola seja um ponto.

SOLUES NVEL 1 SEGUNDA FASE PARTE A

Problema Resposta

01 8

02 60

03 8

04 9

05 499

06 38

01. Volume de um copo de gua igual a

1 1 43 1 = = do volume da jarra. 3 4 12 12 2 2 1 2 Falta encher 1 = da jarra. Para isso so necessrios 3 = 12 = 8 copos. 1 3 3 3 12

02. Como o algarismo das unidades do produto 2, o algarismo das unidades do multiplicando 4. Assim, obtemos o algarismo da direita da 3 linha do algoritmo e 1 4 1 4 tambm os dois ltimos 3 2 2 9 3 algarismos da 5 linha, conforme 4 2 4 2 figura direita. Como o algarismo 4 2 + 4 2 6 + das dezenas do produto 0, o algarismo da direita na 4 linha do 0 2 8 0 2 8 0 0 2 1 0 0 2 algoritmo deve ser 6. Logo o 1 algarismo das dezenas do multiplicador 9, conforme figura esquerda. Como o 2 algarismo direita 5 linha 0, o algarismo das centenas
EUREKA! N34, 2011

21

Sociedade Brasileira de Matemtica

do multiplicando 5. A partir do algoritmo completo, mostrado direita, conclumos que a soma dos algarismos que foram borrados

5 + 4 + 9 + 1 + 5 + 2 + 6 + 6 + 1 + 2 + 8 + 5 + 6 = 60
03. Cada 3 faces que podem ser vistas ao mesmo tempo

1 1

4 0 5

1 6 2 0

5 2 5 2 8 6

1 9 4 6 0

4 3 2 + 2

compartilham um vrtice. Como o cubo tem 8 vrtices, o nmero de composies de cores percebidas visualmente 8.
04. Os nmeros 12, 22, 32 possuem um algarismo. Os nmeros 42, 52, ..., 92 possuem

dois algarismos. Os nmeros 102, 112, ..., 312 possuem trs algarismos. Assim, ao escrever o quadrado do nmero 31, o nmero de algarismos escritos 1 3 + 2 6 + 3 22 = 81 , faltando escrever 19 algarismos. Com os quadrados de 32, 33, 34 e 35, temos mais 4 4 = 16 algarismos, faltando ainda escrever apenas trs algarismos. Como o quadrado de 36 1296, conclumos que o ltimo algarismo escrito foi o 9, o centsimo algarismo escrito por Esmeralda.

05. No podemos colocar o nmero 1 em nenhuma bola, pois o MDC entre 1 e qualquer outro nmero 1, assim temos 998 nmeros disponveis. Alm disso, se forem usadas 500 bolas ou mais, haver duas com nmeros consecutivos, sempre primos entre si, ento no podemos colocar mais que 499 bolas. Mas existe uma forma de colocar 499 bolas, usando os nmeros pares de 2 a 998. 06. Quem acerta a questes e erra b obtm 3a b pontos, com a + b 10 . Obtemos

os nmeros de 0 a 10 com a = 0, ao todo 11 inteiros. Obtemos os nmeros de 1 a 30 usando os valores 0, 1 ou 2 para b, no obtendo apenas 3.9 2 = 25, 3.10 1 = 29 e 3.10 2 = 28, pois nesses casos ficamos com a + b > 10, ao todo 30 3 = 27 inteiros. Logo, o nmero mximo de candidatos nas condies apresentadas 11 + 27 = 38.
SOLUES NVEL 1 SEGUNDA FASE PARTE B SOLUO DO PROBLEMA 1:

Em cada figura, a rea do quadrado cinza uma frao da rea do quadrado original. Nas figuras apresentadas, a partir da segunda, as reas so iguais,

EUREKA! N34, 2011

22

Sociedade Brasileira de Matemtica

4 27 27 9 4 4 respectivamente, a 27 27 9 9 4 4 4 27 27 = 64 9 9 9 a) Na 4 figura, a rea do quadrado cinza igual a 64, segundo os produtos acima. b) Na 5 figura, admitindo que a obteno do quadrado cinza seja feita da mesma 4 maneira, a sua rea igual a da rea do quadrado cinza da 4 figura, ou seja, 9 4 256 2 igual a 64 = cm . 9 9
SOLUO DO PROBLEMA 2:

Para um nmero cujo algarismo das dezenas a e cujo algarismo das unidades b, temos 10a + b = 2 ( a + b ) ou a + b = 2 (10a + b ) . A segunda equao no tem solues positivas, e na primeira equao temos 10a + b = 2 ( a + b ) 10a + b = 2a + 2b 8a = b . Necessariamente temos a = 1 e b = 8. De fato, no carto de nmero 18 a soma dos algarismos 9.
SOLUO DO PROBLEMA 3:

a) Bastam 4 cortes no quadrado de lado 8 cm, conforme ilustrado nos desenhos direita.

1, 2 e 3 cortes

Ou ainda, como a figura a seguir.


EUREKA! N34, 2011

4corte

23

Sociedade Brasileira de Matemtica

4corte 1, 2 e 3 cortes

b) Uma possibilidade (exemplo 1) juntar ao quadrado maior pedaos dos quadrados menores, obtendo-se um quadrado de rea 4 + 9 + 36 = 49 cm2. Para isso, dividimos o quadrado de lado 3 em trs tiras 3 1 com dois cortes e o quadrado de lado 2 em duas tiras 2 1 com um corte, num total de 3 cortes, conforme desenho esquerda. Menos que 3 cortes no formam peas que se encaixam na regio sombreada.

(exemplo 1)
Outras maneiras (exemplos 2 e 3) demontar o quadrado tambm com trs cortes so apresentadas ao lado.

(exemplo 2)

(exemplo 3)

EUREKA! N34, 2011

24

Sociedade Brasileira de Matemtica

SOLUES NVEL 2 SEGUNDA FASE PARTE A

Problema Resposta

01 25

02 75

03 1

04 53

05 8

01. O critrio de divisibilidade por 11 nos diz que se o nmero 33N possui todos os

seus algarismos iguais e divisvel por 11, ento ele deve possuir um nmero par da algarismos. O critrio de divisibilidade por 3 tambm nos diz que a soma dos algarismos deve ser mltipla de 3 e isso obriga que a quantidade de algarismos 7 seja divisvel por 3. O menor nmero que cumpre essas condies 777777, ou seja, N = 777777/33 = 23569.
02. Pelo teorema de Pitgoras, temos que

AB = AD 2 + BD 2 = 20 e que
temos

CB = AC 2 + AB 2 = 25 . Os tringulos ABC e ADB so semelhantes pois os seus EAB = EBA e ACB = 90 EBA = 90 EAB = CAE. . Conclumos assim que E o ponto mdio de 15 20 CB e a rea procurada metade da rea do tringulo CAB, ou seja, = 75. 4
lados so proporcionais e consequentemente
03. Como p no pode ser zero, podemos dividir a primeira equao por p 2 e

obter

1 1 + 3 2. Isto nos diz que as razes da primeira equao so os inversos 2 p p 1 pois a soma das razes 3 q

das razes da segunda equao. Como pq 1. , p igual ao inverso da outra raz da segunda equao que diferente de q, ou seja, p =

da segunda equao igual a 3. Substituindo na expresso procurada: 2 2 pq + p + 1 q 1 + 3 + q = = = =1 2 2 q 3q + q 3q + q 2


04. A soma dos dgitos dos bilhetes no mnimo 1 e no mximo 27. Para as somas 1 e 27 existem apenas dois bilhetes, enquanto que para qualquer outro valor existem pelo menos trs bilhetes. Ento retirando 1 + 1 + 2 ( 27 2 ) + 1 = 53 iremos

escolher pelo menos trs nmeros com mesma soma.

EUREKA! N34, 2011

25

Sociedade Brasileira de Matemtica

05. Para que as solues sejam inteiras, o discriminante da equao do segundo

grau deve ser o quadrado de um inteiro positivo, digamos t 2 . Assim

(r + s)

2
2

4rs 4 2010 = t 2

( r s ) t 2 = 4 2010 (( r s ) + t ) (( r s ) t )
2 2 Como os nmeros

= 2010

( ( r s ) + t ) e ( ( r + s ) + t ) possuem a mesma paridade e 2010

inteiro, conclumos que os termos no produto anterior so inteiros. A cada para de

2010 divisores do tipo d , do nmero 2010, temos uma soluo para t e r s d na ltima equao. Como 2010 possui 16 divisores, o nmero de solues 8.
SOLUES NVEL 2 SEGUNDA FASE PARTE B SOLUO DO PROBLEMA 1:

A pintura da primeira coluna 2 x 1 do tabuleiro limita o nmero de maneiras de pintarmos o restante do tabuleiro. Temos dois casos a considerar:

Primeiro caso: As casas desta coluna so pintadas com a mesma cor. Necessariamente a prxima coluna ter ambas casa da cor oposta aquela da primeira coluna e. Pela mesma razo, teremos que as cores das colunas do tabuleiro devem ser alternadas. Assim, neste caso, temos apenas 2 pinturas diferentes.
Figura 1

Segundo caso: As casas desta coluna so pintadas com cores diferentes. Necessariamente a prxima coluna igual primeira ou tem as cores opostas. O mesmo se passar com as prximas colunas. Como para cada coluna sempre 2010 pinturas teremos duas escolhas a fazer, incluindo a coluna inicial, temos 2
diferentes.

EUREKA! N34, 2011

26

Sociedade Brasileira de Matemtica

Figura 2

O total de pinturas : 2 + 22010


SOLUO DO PROBLEMA 2:

Como os primos 2m + n e , m + 2n so maiores que dois, temos que ambos so mpares e consequentemente 2m + n + m + 2n = 3m + 3n um nmero par. Assim

m + n par e m + n 18 um primo par, ou seja, dois. O nico par de primos (m, n) que cumpre m + n = 20 e satisfaz o enunciado (m, n) = (3,17)
SOLUO DO PROBLEMA 3:

A soma de um nmero de dois algarismos com a sua imagem da forma (10a + b) + (10b + a) = 11( a + b ) , onde a e b so seus algarismos. Se 11( a + b ) um quadrado perfeito, devemos ter outro fator primo 11 na soma a + b. Alm disso, como a e b so menores que 10, conclumos que a + b um mltiplo de 11 menor que 20 e maior que 0, ou seja, igual 11. Os pares de dgitos ( a, b ) que verificam a + b = 11 so: ( 2,9 ) , ( 3,8 ) , ( 4, 7 ) , ( 5, 6 ) , ( 6,5 ) , ( 7, 4 ) , ( 8,3) e ( 9, 2 ) . Portanto, existem 8 nmeros de dois algarismos que cumprem o enunciado.
PRIMEIRA SOLUO DO PROBLEMA 4:

Seja N o ponto de encontro da bissetriz do ngulo ACD com o lado AB . Pelo caso A.L. A , os tringulos NCA e ADC so congruentes. Consequentemente NC = AD = BC .

BNC = NAC + ACN = NCB . Pelo teorema do ngulo externo, portanto BN = BC = NC e BNC equiltero. Da ABC = 60, BCA = 80 e BAC = 40

SEGUNDA SOLUO DO PROBLEMA 4:

Lema 1: Se

ABC um tringulo com ACB = 2BAC ento a (a + b) = c 2 .


27

EUREKA! N34, 2011

Sociedade Brasileira de Matemtica

Lema 2: A bissetriz do vrtice C do tringulo

ABC tem comprimento

2ab cos

ACB 2 a+b

lado AB . Pelo segundo lema temos

cb 2ab cos 2 = e da c = 2a cos 2 . a+b a+b 2 2 2 2 Pelo lema 1 temos a (a + b) = c = 4a cos 2 e da b = a (4 cos 2 1) . p a = AD cos = a cos AD = BC temos que Como e da a + b + c = 2a(cos + 1) . Substituindo os valores encontrados anteriormente
temos a + a (4 cos 2 2 1) + 2a cos 2 = 2a (cos + 1)

Sejam = BAD e P o ponto de encontro da bissetriz do ngulo C com o

1 + 2 cos 4 + 1 + 2 cos 2 = 2 cos + 2 cos 4 + cos 2 = cos cos (1 2 cos 3 ) = 0 cos 3 = 1 2

E consequentemente ABC = 3 = 60 .

SOLUES NVEL 3 SEGUNDA FASE PARTE A

Problema Resposta

01 25

02 8

03 12

04 2592

05 1057

01. Queremos o menor mltiplo de 33 formado apenas por algarismos 7. Teremos

33N = 7777...77, com k algarismos 7. Para ser mltiplo de 33, deve ser mltiplo de 11 e de 3. Assim, k deve ser par (pelo critrio de divisibilidade por 11) e, tambm, k deve ser mltiplo de 3, pois a soma dos algarismos de 33N 7k. Logo, o menor N procurado satisfaz 33N = 777.777, o que nos d N = 23.569. A soma dos algarismos de N 2 + 3 + 5 + 6 + 9 = 25.
02. A expresso x2 (r + s)x + rs pode ser escrita como (x r)(x s). Logo,

devemos ter (r x)(x s) = 2010.


EUREKA! N34, 2011

28

Sociedade Brasileira de Matemtica

Fazendo r x = a e x s = b, a e b com o mesmo sinal, devemos encontrar a + b = r s sabendo que a e b so inteiros positivos tais que ab = 2010. O nmero de pares {a, b} que satisfazem esta equao igual a oito, sendo {a, b} = {1, 2010}, {2, 1005}, {3, 670}, {5, 402}, {6, 335}, {10, 201}, {15, 134}, {30, 67}.
03. Seja O o ponto de interseo entre as retas AB e CD. Veja que os tringulos ODA e OBC so semelhantes, pois OAD = 180 DAB = BCA. Logo, podemos igualar a razo de semelhana razo entre os raios das circunferncias inscritas, bem como das ex-inscritas, obtendo: 8 R = R 2 = 144 R = 12. R 18 04. Em primeiro lugar, escolhemos a coluna que conter o corte. Isso pode ser feito de 4 modos. Em seguida, escolhemos a cor das casas do corte, o que pode ser feito de 3 modos. Ficamos, ento, com trs colunas restantes para preencher. Preencheremos primeiramente as casas da primeira linha. Temos 3 modos de colorirmos cada casa da primeira linha, ou seja, 33 modos. Finalmente, resta-nos colorir as casas da segunda linha, o que pode ser feito de 23 modos, j que as cores das casas dessas colunas devem ser diferentes das cores das casas imediatamente superiores. O total de coloraes 433323 = 2592.

Em primeiro lugar, veja que cada termo do produto do tipo (k + 1) 4 + (k + 1) 2 + 1 . Alm disso, podemos escrever k 4 + k 2 +1 x 4 + x 2 + 1 = ( x 4 + 2 x 2 + 1) x 2 = ( x 2 + 1) 2 x 2 = ( x 2 x + 1)( x 2 + x + 1). Assim, ficamos com (k + 1) 4 + ( k + 1) 2 + 1 [(k + 1) 2 (k + 1) + 1].[(k + 1) 2 + ( k + 1) + 1] . Agora, veja que = ( k 2 k + 1)(k 2 + k + 1) k 4 + k 2 +1 (k + 1)2 (k + 1) + 1 = k2 + k + 1 e k2 k + 1 = (k 1)2 + (k 1) + 1. Logo, a ltima expresso fica (k + 1) 4 + (k + 1) 2 + 1 (k + 1) 2 + ( k + 1) + 1 . = ( k 1) 2 + ( k 1) + 1 k 4 + k 2 +1 Logo, o produto pedido igual a 22 + 2 + 1 42 + 4 + 1 62 + 6 + 1 32 2 + 32 + 1 = 32 2 + 32 + 1 = 1057 . 02 + 0 + 1 22 + 2 + 1 42 + 4 + 1 30 2 + 30 + 1
05.

EUREKA! N34, 2011

29

Sociedade Brasileira de Matemtica

SOLUES NVEL 3 SEGUNDA FASE PARTE B PROBLEMA 1:

A) = 2 . Prolongue a reta CI at encontrar o lado AB Seja = m(IC). Ento m( IC em D. Como m(CD) = 2m(IC) = 2, o tringulo ACD issceles e, portanto, suas bissetrizes AI e CE so congruentes.
C

E D

B ) e CE = AI = BC, o tringulo BCE Logo, sendo m(CB) = + 2 = 3 = m( EC C ) = 60 . equiltero. Assim, m( AB


Outra soluo: Considere a mesma figura acima. Aplicando a lei dos senos nos

tringulos ACI e ABC, obtemos

AC AI AC sen 3 = = sen 2 AI sen 2 sen(180 2 ) AC BC AC sen 6 = = BC sen 2 sen(180 2 4 ) sen 2

Como AI = BC e 0 < 3 < 6 < 180,

C ) = 180 6 = 60 . sen 3 = sen 6 3 + 6 = 180 = 20 . Logo m( AB


EUREKA! N34, 2011

30

Sociedade Brasileira de Matemtica

PROBLEMA 2:

Note que Diamantino pode jogar futebol no mximo 5 vezes; caso contrrio ele necessariamente joga dois dias seguidos. Suponha que ele joga k dias. Ento os k dias em que ele joga devem ser imediatamente seguidos por dias em que ele no joga. Assim, acrescentando um dia ao perodo, podemos dividir os 11 dias em k blocos de dois dias e 11 2k blocos de um dia. Podemos permutar os k + 11 2k = 11 k (11 k )! = 11 k blocos de maneiras. k!(11 2k )! k Assim, o total de maneiras de Diamantino escolher os dias em que vai jogar 11 0 11 1 11 2 11 3 11 4 11 5 = 1 + 10 + 36 + 56 + 35 + 6 = 144 + + + + + 0 1 2 3 4 5 .
Outra soluo:

Seja an o nmero de maneiras de Diamantino escolher os dias em que vai jogar entre n dias. Se ele jogar no dia n ele no pode ter jogado no dia n 1, mas no h restries aos demais n 2 dias; assim, nesse caso h an2 maneiras de escolher os dias em que vai jogar; se ele no jogar no dia n no h restries aos demais n 1 dias, ento nesse caso h an1 maneiras de escolher os dias. Assim, an = an1 + an2, com a0 = 1 (a nica opo no jogar) e a1 = 2 (ele joga ou no no nico dia). Dessa forma, podemos encontrar os valores de an a partir dos anteriores: n an 0 1 1 2 2 3 3 5 4 8 5 13 6 21 7 34 8 55 9 89 10 144

Logo Diamantino pode escolher os dias de 144 maneiras.


Comentrios:

Temos que an = Fn+2, em que Fn a famosa sequncia de Fibonacci (clique no link para saber algumas de suas muitas propriedades!) Comparando e generalizando as duas solues voc pode obter a identidade n n 1 n 2 n 3 Fn +1 = + + + 0 + 1 2 3 que soma as outras diagonais do tringulo de Pascal.

EUREKA! N34, 2011

31

Sociedade Brasileira de Matemtica

PROBLEMA 3:

Observando que (1 + x)(1 + y)(1 + z) = 1 + x + y + z + xy + yz + zx + xyz,

x + y + z = 77 x + y + z = 77 xy + yz + zx + xyz = 946 1 + x + y + z + xy + yz + zx + xyz = 1024 (1 + x) + (1 + y ) + (1 + z ) = 80 10 (1 + x)(1 + y )(1 + z ) = 1024 = 2 Como x, y e z so inteiros no negativos, 1 + x, 1 + y e 1 + z so potncias de 2.
Considerando que 80 = 26 + 24 > 324, 80 < 27 e x y z, temos 24 < 1 + z < 27, ou seja, 1 + z = 25 = 32 ou 1 + z = 26 = 64. Se 1 + z = 32, temos 1 + x + 1 + y = 48 e (1 + x)(1 + y) = 25 = 32. Mas, sendo 1 + x e 1 + y potncias de 2 com soma par, temos 1 + x 2 e, portanto, 1 + y 16. Ento 1 + x 16 e 1 + x + 1 + y 32 < 48, e no h solues nesse caso. Se 1 + z = 64, temos 1 + x + 1 + y = 16 e (1 + x)(1 + y) = 24 = 16. Desse modo, 1 + x e 1 + y so solues da equao do segundo grau t2 16t + 16 = 0, que no tem solues inteiras. Logo no h solues.
PROBLEMA 4:

Como a bola sofre reflexo perfeita, ao refletir a mesa em relao a cada lado em que a bola bate obtm-se uma linha reta. Repetindo as reflexes obtemos a seguinte figura, em que a trajetria da bola reta:
A

Assim, o problema equivalente a encontrar uma trajetria em um retngulo de dimenses inteiras m e n, dividido em mn quadradinhos unitrios, que comea em um vrtice, termina no vrtice oposto e corte os lados dos quadradinhos unitrios 2010 vezes, sem passar por nenhum dos vrtices internos dos quadrados unitrios (pois se passasse, a bola chegaria a um vrtice do quadrado antes de 2010 rebatidas nos lados).
EUREKA! N34, 2011

32

Sociedade Brasileira de Matemtica

Como a bola deve atravessar m 1 quadrados em um sentido e n 1 no outro, m 1 + n 1 = 2010 m + n = 2012; como a bola no passa por vrtices do quadrado unitrio, mdc(m, n) = 1 mdc(m, m + n) = 1 mdc(m, 2012) = 1. Assim, o nmero pedido a quantidade de nmeros coprimos com 2012 menores do que 1 1 2012, que (2012) = (22 503) = 20121 1 = 1004 . 2 503

EUREKA! N34, 2011

33

Sociedade Brasileira de Matemtica

XXXII OLIMPADA BRASILEIRA DE MATEMTICA


Problemas e solues da Terceira Fase
TERCEIRA FASE NVEL 1

PROBLEMA 1

Esmeralda tem muitos tringulos retngulos iguais aos da figura.

3 cm

5 cm 4 cm

a) Fazendo coincidir partes dos lados, sem sobrepor tringulos, Esmeralda montou a figura a seguir. Qual a rea e qual o permetro dessa figura?

b) Usando o mesmo processo, Esmeralda montou o menor quadrado possvel com lado de medida inteira. Mostre, atravs de uma figura, como Esmeralda pode fazer isso.
PROBLEMA 2

As casas de um tabuleiro 3 3 so numeradas de 1 a 9, cada nmero sendo utilizado exatamente uma vez. Em cada linha horizontal, pintamos de vermelho a casa com o maior nmero e, de verde, a casa com o menor nmero. Seja A o menor dos nmeros das casas vermelhas e B o maior dos nmeros das casas verdes.

EUREKA! N34, 2011

34

Sociedade Brasileira de Matemtica

a) Mostre uma maneira de preencher o tabuleiro de forma que A B = 4. b) Mostre uma maneira de preencher o tabuleiro de forma que A B = 3. c) possvel obter A = 4 e B = 3? No se esquea de justificar a sua resposta.
PROBLEMA 3

Dado um slido formado por cubos de 1 cm de aresta, como mostra a figura 1, podemos indicar a quantidade de cubos em cada direo, como mostra a figura 2.
3 3 2 2 0 3 1 3 3 2 3 0 1 2 0 1 1 0 1 3 1 0 1 2 2 1 1

Esmeraldino montou um slido com cubos de 1 cm de aresta e fez uma figura similar figura 2.
1 3 a 2 2 2 x 1 2 1 m 2 3 2 2 1 d 1 3 2 b e 3 1 c f 2

Encontre os valores de a, b, c, d, e, f, x e m.
PROBLEMA 4

Dizemos que um nmero inteiro positivo n abestado se ao lermos da direita para esquerda obtivermos um inteiro maior que n. Por exemplo, 2009 abestado porque 9002 maior que 2009, por outro lado, 2010 no abestado pois 0102, que o
EUREKA! N34, 2011

35

Sociedade Brasileira de Matemtica

nmero 102, menor que 2010 e 3443 no abestado pois quando lido da direita para esquerda exatamente igual ao original. Quantos inteiros positivos de quatro algarismos so abestados?
PROBLEMA 5

a) Exiba um nmero inteiro positivo menor ou igual a 1000 com pelo menos 20 divisores positivos. b) Existe um nmero inteiro positivo menor ou igual a 11000 com pelo menos 200 divisores positivos? No se esquea de justificar a sua resposta.
TERCEIRA FASE NVEL 2 PRIMEIRO DIA PROBLEMA 1

Dizemos que um nmero inteiro positivo n abestado se ao lermos da direita para esquerda obtivermos um inteiro maior que n. Por exemplo, 2009 abestado porque 9002 maior que 2009, por outro lado, 2010 no abestado pois 0102, que o nmero 102, menor que 2010 e 3443 no abestado pois quando lido da direita para esquerda exatamente igual ao original. Quantos inteiros positivos de quatro algarismos so abestados? Seja ABCD um paralelogramo e a circunferncia circunscrita ao tringulo ABD. Se E e F so as intersees de com as retas BC e CD respectivamente, prove que o circuncentro do tringulo CEF est sobre .
PROBLEMA 3 PROBLEMA 2

Arnaldo e Bernaldo participam do seguinte jogo em um tabuleiro m n, m, n 2. Arnaldo comea escolhendo uma casinha e colocando um cavalo na casinha escolhida; em seguida, Bernaldo e Arnaldo movem alternadamente o cavalo, comeando por Bernaldo, com a restrio de que o cavalo no pode cair em casinhas que j foram visitadas. Perde quem no poder mover o cavalo. Determinar, em funo de m e n, qual jogador tem uma estratgia para ganhar o jogo, no importando os movimentos do outro jogador e mostrar como ele deve jogar para ganhar. Observao: Cada movimento de um cavalo consiste em ir duas casas na vertical ou na horizontal e, em seguida, uma casa na direo perpendicular.

EUREKA! N34, 2011

36

Sociedade Brasileira de Matemtica

SEGUNDO DIA PROBLEMA 4

Sejam a, b e c reais tais que a b e a2(b + c) = b2(c + a) = 2010. Calcule c2(a + b).
PROBLEMA 5

As diagonais de um quadriltero inscritvel ABCD se intersectam em O. Os crculos circunscritos aos tringulos AOB e COD intersectam as retas BC e AD, pela segunda vez, nos pontos M, N, P e Q. Prove que o quadriltero MNPQ est inscrito em um crculo de centro O.
PROBLEMA 6

Os trs lados e a rea de um tringulo so nmeros inteiros. Qual o menor valor da rea desse tringulo?
TERCEIRA FASE NVEL 3 PRIMEIRO DIA PROBLEMA 1

Encontre todas as funes f do conjunto dos reais nos conjuntos dos reais tais que f(a + b) = f(ab) para todos a, b irracionais.
PROBLEMA 2

Seja P(x) um polinmio com coeficientes reais. Prove que existem inteiros positivos n e k tais que k tem n dgitos e mais de P(n) divisores positivos.
PROBLEMA 3

Qual a maior sombra que um cubo slido de aresta 1 pode ter, no sol a pino? Observao: Entende-se maior sombra de uma figura no sol a pino como a maior rea possvel para a projeo ortogonal da figura sobre um plano.
SEGUNDO DIA PROBLEMA 4

Seja ABCD um quadriltero convexo e M e N os pontos mdios dos lados CD e AD, respectivamente. As retas perpendiculares a AB passando por M e a BC
EUREKA! N34, 2011

37

Sociedade Brasileira de Matemtica

passando por N cortam-se no ponto P. Prove que P pertence diagonal BD se, e somente se, as diagonais AC e BD so perpendiculares.
PROBLEMA 5

Determine todos os valores de n para os quais existe um conjunto S de n pontos, sem que haja trs deles colineares, com a seguinte propriedade: possvel pintar todos os pontos de S de modo que todos os ngulos determinados por trs pontos de S, todos da mesma cor ou de trs cores diferentes, no sejam obtusos. A quantidade de cores disponveis ilimitada.
PROBLEMA 6

Encontre todos os pares (a, b) de inteiros positivos tais que 3a = 2b2 + 1.


SOLUES DA TERCEIRA FASE NVEL 1 PROBLEMA 1: SOLUO DE HELENA VERONIQUE RIOS (SO CARLOS SP)

a) Permetro 3 + 3 + 4 + 4 + 5 + 5 + ( 4 3) + ( 4 3) = 26. O permetro da figura 26cm. rea de um tringulo: bh 4 3 12 = = 6cm 2 2 2 2 Cada tringulo tem 6cm2 de rea. Se na figura temos 4 desses tringulos, a rea da figura 4 6, ou seja, 24cm2. b) 6cm2 rea de cada tringulo Qual o menor mltiplo de 6 que um quadrado perfeito? 36 ( 6 6 ) O quadrado dever ter 36cm 2 e 6cm de lado, se possvel. Este quadrado, porm, impossvel de ser formado por causa da forma do tringulo. Teriam de ter dois lados medindo 3cm em cada lado do quadrado, o que seria impossvel j que precisariam de 8 lados de 3cm sendo que s tem 6. ( 4 + 3 6; 4 + 5 6; 5 + 3 6 ) O prximo menor quadrado possvel de ser feito com formas de 6cm 2 o de lado 12, cuja rea 12 12 (144cm 2 ) .

36cm 2 lado 6 ,
EUREKA! N34, 2011

38

Sociedade Brasileira de Matemtica

49cm 2 lado 7 , 64cm 2 lado 8 , 81cm 2 lado 9 , 100cm 2 lado 10 , 121cm 2 lado 11 , 144cm 2 lado 12 ; 49, 64, 81, 100 e 121 no so divisveis por 6 (rea do tringulo). 144 divisvel por 6 ( 6 24 = 144 ) .

Quadrado de lado 12cm, rea 144cm2 com 24 tringulos retngulos de lados 3, 4 e 5 cm.

PROBLEMA 2: SOLUO DE LUCCA MORAIS DE ARRUDA SIAUDZIONIS (FORTALEZA CE)

a)
Verde Vermelho

A=7 B=3 AB=4

Verde

Vermelho

Verde

Vermelho

EUREKA! N34, 2011

39

Sociedade Brasileira de Matemtica

b)
Verde Vermelho

A=3 B=6 A B = 3

Verde

Vermelho

Verde

Vermelho

c) Para a ser igual a 4 os dois nmeros que estaro juntos com ele na fileira devem ser (1, 2 ) ; ( 2 ,3) ou (1,3) . Porm o 3 no pode estar junto com ele na fileira, seno ele no seria pintado de verde. Ento uma fileira horizontal : 1, 2, 4. Porm, para que o 3 seja o B, as outras duas casas verdes teriam que ser 1 e 2. Porm, 1 e 2 esto na mesma fileira, ento casas verdes so (1,3) , j que o 2 no verde, a terceira casa verde um nmero 4, ocasionando o fato de B 3. Portanto, no possvel.
PROBLEMA 3: SOLUO DE VICTRIA MOREIRA REIS COGO (TERESINA PI)

Nesse cubo, podemos formar expresses a partir das placas 3 3 1 e a partir delas, encontramos o resultado. Veja:
3 1

a 2 2 1

a+3+1=2+2+1 a=54 a=1

1 2 x 2

1+b+c =2+x+2 1+b+c =2+3+2 b + c = 7 1 b+c=6

EUREKA! N34, 2011

40

Sociedade Brasileira de Matemtica

2 x 2 3 3

2+2+x=3+3+1 x=74 x=3

Como os resultados s podem ser de 0 a 3, e a nica soma que d 6 3 + 3, ento: b= 3 e c = 3

1 2 1 m 2

1+2+1=m+2+2 m=44 m=0

e 3 3 2

3+b+e =3+3+2 3+3+e =3+3+2 e =86 e=2

d 1 3 m

a+1+d=1+3+m 1+1+d=1+3+0 d=42 d=2

f 2 1 2

1+c+f =2+1+2 1+3+f =5 f =54 f =1

PROBLEMA 4: SOLUO DE LUCCA MORAIS DE ARRUDA SIAUDZIONIS (FORTALEZA CE)

Chamamos o primeiro algarismo de A, o segundo de B o terceiro de C e o quarto de D. Testamos os casos: 1. Caso: o ltimo algarismo maior que o primeiro. Se A = 1, temos: 1 10 10 8 = 800 Se A = 2, temos: 1 10 10 7 = 700 Se A = 3, temos: 1 10 10 6 = 600 Se A = 4, temos: 1 10 10 5 = 500 Se A = 5, temos: 1 10 10 4 = 400 Se A = 6, temos: 1 10 10 3 = 300 Se A = 7, temos: 1 10 10 2 = 200 Se A = 8, temos: 1 10 10 1 = 100 Total de 3600 casos.
EUREKA! N34, 2011

41

Sociedade Brasileira de Matemtica

2. Caso: A = D, Se B = 0, Temos: Se B = 1, Temos: Se B = 2, Temos: Se B = 3, Temos: Se B = 4, Temos: Se B = 5, Temos: Se B = 6, Temos: Se B = 7, Temos: Se B = 8, Temos:

C > B. 9 1 9 1 = 81 9 1 8 1 = 72 9 1 7 1 = 63 9 1 6 1 = 54 9 1 5 1 = 45 9 1 4 1 = 36 9 1 3 1 = 27 9 1 2 1 = 18 9 1 1 1 = 9

Total de 405 casos. Resposta final: 4005 nmeros.


PROBLEMA 5: SOLUO DA BANCA

a) Por exemplo, 900 = 22 32 52 , que tem positivos.

( 2 + 1) ( 2 + 1) ( 2 + 1) = 27

divisores

b) No, no existe. Seja n um nmero com pelo menos 200 divisores. Se o i-simo n menor divisor d, ento o i-simo maior divisor . Seja m o centsimo menor d n > m, donde n > m 2 10000. Chegamos perto, mas divisor. Temos m 100 e m isso ainda no resolve o problema. Consideremos o 98., o 99. e o 100. menores divisores de n, que chamaremos de k, l, e m. Note que, se m 105, teremos como n antes > m, donde n > m 2 1052 = 11025 > 11000. m Podemos supor ento que 98 k < l < m 104. Como para quaisquer inteiros positivos distintos a, b temos e mdc ( a,b ) b a ,

mmc ( a,b ) = = =

a b , conclumos que n mmc ( k ,l,m ) = mmc ( k ,mmc ( l,m ) ) = mdc ( a,b ) mdc ( k ,l ) mdc ( k ,m ) k mmc ( l,m ) =

mdc ( k ,mmc ( l,m ) )

k mmc ( l,m )

klm klm 98 99 100 . mdc ( l,m ) mdc ( k ,l ) mdc ( k ,m ) ( m l )( l k )( m k ) ( m l )( l k )( m k )

EUREKA! N34, 2011

42

Sociedade Brasileira de Matemtica

Como

( m l ) + ( l k ) = m k 104 98 = 6,
n

temos

( m l )( l k ) 3 3 = 9 e

( m l )( l k )( m k ) 9 6 = 54, donde
SOLUES DA TERCEIRA FASE NVEL 2

98 99 100 > 11000. 54

PROBLEMA 1: SOLUO DE LIARA GUINSBERG (SO PAULO SP)

Considere o nmero da forma ABCD. Temos 3 possibilidades: A > D o nmero no abestado. A = D o nmero abestado somente se C > B : calculando o nmero de casos temos:

A = D : 9 possibilidades, pois A = D 0 , seno o nmero teria somente 3 algarismos. C > D : 45 possibilidades, pois o resultado do somatrio 9 + 8 + 7 + ... + 1, j que C pode assumir o valor zero.
Totalizando 9 45 = 405 nmeros abestados. A < D o nmero abestado, independentemente dos valores B e C. Novamente calculando o nmero de casos: A < D : 36 possibilidades, pois o somatrio de 8 + 7 + 6 + ... + 1, j que A no pode assumir o valor zero. B e C : 100 possibilidades, j que B pode assumir 10 valores diferentes, assim como C. Total: 36 100 = 3600 nmeros abestados. Finalizando, teremos 405 + 3600 = 4005 nmeros abestados de quatro algarismos.
PROBLEMA 2: SOLUO DE PEDRO MORAIS DE ARRUDA SIAUDZIONIS (FORTALEZA CE)

Seja B AD = , logo BCD = , pois em um paralelogramo os ngulos opostos so iguais. Seja BDA = . Assim ABD = 180 . Veja que o arco AB = 2 BDA = 2 e

AD = 2 ABD AD = 360 2 2.

EUREKA! N34, 2011

43

Sociedade Brasileira de Matemtica

180

360 2 2 E D F 2

BAD EF 2 = 360 2 EF EF = 360 4 2 Com isso EBAF = 4 .


Note que BCD = Seja O o circuncentro do FCE. Sabemos que FOE = 2 FCE FOE = 2 . Como

EBAF EBAF = 2 FOE = O , pois FOE ngulo inscrito. 2 2

PROBLEMA 3: SOLUO DA BANCA

Chamaremos Arnaldo de A e Bernaldo de B. Suponha, sem perda de generalidade, m n. Vamos mostrar que para m = 2, A tem estratgia vencedora se e somente se n no mltiplo de 4; para m 3, A tem estratgia vencedora se e somente se m e n so mpares. Estudemos o caso m = 2. Se 4 no divide n, A pode vencer colocando o cavalo na primeira coluna se n = 4k + 1 e na segunda coluna se n = 4k + r, para r = 2 ou r = 3. O cavalo deve ser movido sempre duas colunas direita em cada jogada, permitindo 2k jogadas a mais. Considere agora o caso n = 4k. Divida o tabuleiro em tabuleiros 2 4, e forme pares de casas de modo que possvel mover o cavalo entre casas do mesmo par:
EUREKA! N34, 2011

44

Sociedade Brasileira de Matemtica

1 3

2 4

3 1

4 2

Como todo o tabuleiro est dividido em pares, B consegue jogar, no importando onde A coloque o cavalo: basta mover o cavalo para a outra casa do par. Quando A jogar, colocar o cavalo numa casa de outro par, e B repete a estratgia. Deste modo, se n = 4k o jogador B tem estratgia ganadora. Isto termina o caso m = 2. O caso m 3 segue de modo semelhante, dividindo o tabuleiro em vrios tabuleiros menores. Considere as seguintes maneiras de se formar pares, alm da descrita anteriormente: 1 3 2 2 6 5 3 1 4 4 5 6 1 3 2 2 4 7 3 1 8 4 7 9 5 8 6 6 9 5 1 4 2 2 A 3 3 1 4 1 3 2 2 4 1 3 6 7 4 A 5 5 7 6

Juntando esses tabuleiros se prova que B tem estratgia vencedora para tabuleiros 3 n, n par e A tem estratgia vencedora para tabuleiros 3 n, n mpar: divida o tabuleiro num 3 3 ou 3 5 e tabuleiros 3 4; basta colocar o cavalo na casa marcada com A e seguir a mesma estratgia de B. Para verificar o caso m = 4, basta juntar tabuleiros 4 2 se n par e um tabuleiro 4 3 e vrios tabuleiros 4 2 se n mpar. Isso tambm prova que se A ou B tem estratgia vencedora para um tabuleiro m n ento tem estratgia vencedora tambm para tabuleiros (m + 4) n, m 3. Ento basta resolver o problema para m {3,4,5,6}. Esses tabuleiros resolvem o caso m = 5: 1 3 2 2 4 3 1 4 7 9 5 8 6 7 6 9 5 8 1 3 2 11 2 4 1 A 3 5 10 12 4 6 8 7 9 5 7 9 6 8

10 11

12 13 14 15

10 11 12 13 14 15

12 10 11

EUREKA! N34, 2011

45

Sociedade Brasileira de Matemtica

Note que B tem estratgia vencedora para o tabuleiro 5 4 e A tem estratgia vencedora para o tabuleiro 5 3. Ento, para o caso 5 n, n par, juntamos tabuleiros 5 4 se n mltiplo de 4 e um tabuleiro 5 6 e tabuleiros 5 4 se n na forma 4k + 2; se n mpar, juntamos vrios tabuleiros 5 4 ao tabuleiro 5 3 ou 5 5, se n = 4k + 3 ou n = 4k + 1, respectivamente. O caso 6 n segue diretamente do caso 3 n se n par (basta juntar dois tabuleiros 3 n) e juntando tabuleiros 6 4 a um tabuleiro 6 3 ou 6 5, se n = 4k + 3 ou n = 4k + 1, respectivamente. Com isso, todos os casos esto cobertos.
PROBLEMA 4: SOLUO DE PAULO HENRIQUE OMENA DE FREITAS (SO PAULO SP)

Vamos trabalhar com a equao: a 2 ( b + c ) = 2010 ab b 2 ( c + a ) = 2010 Usando o mtodo da subtrao: a 2 ( b + c ) b2 ( c + a ) = 0

a 2b + a 2 c b 2 c b 2 a = 0 ab ( a b ) + c ( a 2 b 2 ) = 0
ab ( a b ) + c ( a + b )( a b ) = 0 ( a b )( ab + ac + bc ) = 0 a = b ou ab + ac + bc = 0. J que, do enunciado, a b, ab + ac + bc = 0. Colocando c em evidncia: c ( a + b ) = ab c 2 ( a + b ) = abc . Colocando a em evidncia: a ( b + c ) = bc a 2 ( b + c ) = abc . Assim, temos a igualdade: c 2 ( a + b ) = abc = a 2 ( b + c ) = 2010 .
Finalmente, c 2 ( a + b ) = 2010.

EUREKA! N34, 2011

46

Sociedade Brasileira de Matemtica

PROBLEMA 5: SOLUO DE FELLIPE SEBASTIAM S. P. PEREIRA (RIO DE JANEIRO RJ)


A

N M
P

B D
N M

O Q

D B

Observe que como ABCD um quadriltero inscritvel temos que os ngulos ACB e e BDA so iguais. Temos tambm que OQCDP inscritvel, logo, como ACB

BDA so iguais, segue que os arcos OQ e OP so iguais. Podemos concluir que


os segmentos OQ e OP so iguais tambm. Analogamente, fazendo tudo com o quadriltero ANOMB, chegamos concluso que OM e ON so iguais. Agora para provar que O o centro da circunferncia que passa pelos vrtices do quadriltero MNQP (no sabemos ainda se ela existe), basta provarmos que OP = ON , pois assim teremos OM = ON = OP = OQ, concluindo assim que existe uma circunferncia de centro O que passa pelos vrtices do quadriltero MNQP. Para provarmos isto faamos o seguinte: chamemos o ngulo ACD de . Como o quadriltero ABCD inscritvel segue que DBA = . Temos tambm que

OPN = , pois OPD = 180 (pois o quadriltero OPDC inscritvel).


Temos tambm que PNO = , pois NOBA inscritvel. Logo os ngulos O NP e OPN so iguais, donde segue que OM = OP , c.q.d.
PROBLEMA 6: SOLUO DE TADEU PIRES DE MATOS BELFORT NETO (FORTALEZA CE)

Sabemos pela frmula de Heron que a rea de um tringulo :

S=

p ( p a )( p b )( p c )

EUREKA! N34, 2011

47

Sociedade Brasileira de Matemtica

a+b+c 2 a + b + c a + b + c a + b + c a + b + c S= a b c 2 2 2 2 p=
a + b + c b + c a a + c b a + b c S= 2 2 2 2 16 S 2 = ( a + b + c )( b + c a )( a + c b )( a + b c ) sabemos que a,b,c , S + e a,b e c so lados de um tringulo. Podemos ver que a+b+c par. Caso contrrio a + b + c,( a + b + c ) 2a,( a + b + c ) 2c e ( a + b + c ) 2b seriam mpares e teriam o produto par, o que claramente um absurdo. O valor mnimo para essa soma 4, mas no caso a nica tripla de interiros positivos que tm essa soma (1,1, 2 ) . Mas desobedecem a desigualdade triangular: 1 + 1 no maior que 2. Essa soma tambm no pode ser 6. Porque nesse caso o produto teria um fator 3, mas como um quadrado perfeito teria que ter dois ou mais fatores 3, Assim, ( b + c a ) ,( a + c b ) ou ( a + b c ) teriam esse fator 3. Mas eles so pares e menores que 6, logo no h como isso acontecer (absurdo!). Se a + b + c = 8, nenhum dos outros fatores poderia ser 6, caso contrrio teria que haver dois fatores iguais a 6. Sem perda de generalidade supomos que b+ca=6 a+cb = 6 2c = 12 c = 6 a + b = 2 c > a + b. Absurdo. Assim, temos alguns casos a analisar, pois cada fator s pode ser 2 ou 4. a+bc = 4 I) a + b + c = 12 . Absurdo! a+cb = 4

b+ca =4
a+bc = 4
II)

a +cb = 4 b+ca =2

a + b + c = 10 . Absurdo!

EUREKA! N34, 2011

48

Sociedade Brasileira de Matemtica

III)

a+b+c =8 a+bc = 4 a +cb = 2 b+ca =2

( a + b + c )( a + b c )( a + c b )( b + c a ) = 128 .
+

128 no da forma 16S2 com S

, Absurdo!

a+bc = 2
IV)

a +cb = 2 b+ca =2

a + b + c = 6 . Absurdo!

Podemos ver que a soma no pode ser 10, por um argumento anlogo ao do 6. Pois teria que haver outro fator 5, o que faria com que um dos fatores fosse 10, o que um absurdo! J que para a + b + c = 12 possvel, basta tornar a= 5, b= 4, c = 3. Vale a desigualdade triangular e a rea dada por

S=

p ( p a )( p b )( p c )

p = 6 , a = 5, b = 4 , c = 3
S = 6 1 2 3

S = 62 = 6. Vamos provar que o valor mnimo de

( p a )( p b )( p c )

6, caso o produto

p ( p a )( p b )( p c ) fosse menor que 36. Supondo que no fosse, teramos as seguintes possibilidades: b+ca a+bc a+cb = 5, = 1, =1 ( p a )( p b )( p c ) = 5 2 2 2 p = 5 + 1 + 1 = 7 e p ( p a )( p b )( p c ) = 7 5 = 35, que no quadrado perfeito. Absurdo! ( p a )( p b )( p c ) = 4 . Para o produto disso com p ser menor que 36 e quadrado perfeito, p seria 4 e j vimos anteriormente que isso um absurdo. ( p a )( p b )( p c ) = 3 . Nesse caso p seria 3, para o produto ser menor que 36 e quadrado perfeito, mas nesse caso a + b + c = 6 e j analisamos esse caso. a+bc a+cb b+ca =1 =1 =2 ( p a )( p b )( p c ) = 2 2 2 2 p = 1 + 1 + 2 = 4 , S = 8 . Absurdo!

EUREKA! N34, 2011

49

Sociedade Brasileira de Matemtica

Logo os valores de ( p a )( p b )( p c ) e p so mnimos e portanto o produto mnimo. Assim, a rea mnima 6.


SOLUES DA TERCEIRA FASE NVEL 3 PROBLEMA 1: SOLUO DE DAVI COELHO AMORIM (FORTALEZA CE)

Temos que f : Logo, temos:

e f ( a + b) = f ( a b)

(1)

,a,b irracionais.

f (a + b) = f (a b) = f

( ( a ) ( b ) ) = f ( a b ) f ( a + b ) = f ( a b ) .

Lema: Todo nmero real pode ser representado como a soma de dois nmeros irracionais. 1. Caso: Nmero irracional. Seja x um racional e um irracional. Logo, sendo = x , um irracional, pois se for racional, = x e seria racional. Absurdo! Logo, como x = + ( x ) = + , todo racional pode ser escrito como a soma de dois irracionais. 2. Caso: Nmero irracional Seja x esse irracional. Vamos supor que para todo 0 < < 1 irracional, x = , onde racional. Logo temos:

p r x = , onde = e = , p,q,r,s q s x (1 ) = p r ps + rq + sq x racional. Absurdo! Somando obtemos 2 x 1 = + x = q s 2qs


Logo, todo irracional pode ser escrito como a soma de dois irracionais. Com o lema provado, temos que f ( x ) = f ( x ) ,x (2) Em (1), fazendo a = b, temos:

f ( 0 ) = f ( b 2 ) = f ( b 2 ) . Seja f ( 0 ) = k f ( b 2 ) = k ,b irracional.
Logo, provamos que x que os nmeros y
+

( 2)

tal que

x , f ( x ) = k. Basta provarmos agora


tambm satisfazem f ( y ) = k. Porm,

tais que

isso no difcil de provar: seja y tal que tal que y = ( y ) + f ( y ) = f


EUREKA! N34, 2011

. Temos um 0 < < 1 irracional

( ( y ) ) e tambm
50

Sociedade Brasileira de Matemtica

y = ( y + ) + ( ) f ( y ) = f

( ( y + )( ) ) .

Como, para todo x irracional,

f ( x ) = r, vamos provar que um dos nmeros ( y ) e ( y + ) ( ) irracional, fazendo assim com que todo nmero tenha imagem r. Vamos supor o contrrio, ou seja, que os dois so racionais: p p y 2 = ( y ) = q q ( y + )( ) = r y 2 = r , onde p,q,r,s s s p r 2 y = q s
Como

t , onde t,u u 2t ps rq psu rqu = = racional. Absurdo! u qs 2qst y


y y=

Com isso, provamos que f ( x ) = k ,x , onde k uma constante qualquer. Para qualquer k, essa funo serve, de acordo com a questo, pois f ( a + b ) = f ( a b ) k = k . OK!
PROBLEMA 2: SOLUO DE CARLOS HENRIQUE DE ANDRADE SILVA (FORTALEZA CE)

Seja d = grau do polinmio p. Agora vamos numerar os primos em ordem crescente, Logo P Ento seja b inteiro positivo tal que 1 = 2, P 2 = 3, P 3 = 5, P 4 = 7 , ...
a a a b 10b 1 < P2 d + 2 < 10b . Vamos definir k como sendo k = P 1 P 2 ...P 2 d + 2 < (10 )

2d +2 a

).

Logo n ab ( 2d + 2 ) . Podemos supor que para x suficientemente grande p ( x + 1) > p ( x ) . Se isso no ocorre ento p constante ou lim p ( n ) = , e ento claramente p ( n ) < d ( k ) se a suficientemente grande, onde d ( k ) = nmero de divisores positivos de k. Ento basta provarmos que d ( k ) > p ( ab ( 2d + 2 ) ) para a suficientemente grande j que teremos p ( ab ( 2d + 2 ) ) p ( n ) . Ento vamos s contas: d ( k ) = ( a + 1)
2d +2

; e como o polinmio tem grau d ento

p ( x) < x

d +1

para x suficientemente grande. Como no nosso problema

EUREKA! N34, 2011

51

Sociedade Brasileira de Matemtica

x = ab ( 2d + 2 ) com a varivel ento nosso x pode ser to grande quanto quisermos. Ento basta provar que:

( a + 1)

2d +2

> ( ab ( 2d + 2 ) )

d +1

> p ( n ) ( a + 1) > ab ( 2d + 2 ) . Como b, d so


2

constantes ento a nica varivel a e como no lado direito a tem menor grau, ento para a suficientemente grande a desigualdade vlida, mostrando que existem k, n que satisfazem a condio do enunciado.
PROBLEMA 3: SOLUO DA BANCA

Sejam ABCD e EFGH duas faces opostas, AE, BF, CG e DH sendo lados do cubo. Denotaremos por X a projeo ortogonal do ponto X no plano. Note que { A,G} ,{B,H } ,{C ,E} e {D,F } so pares de vrtices opostos. Suponha, sem perda de generalidade, que A pertence fronteira da projeo do cubo. Ento, considerando a simetria do cubo em relao ao seu centro, o simtrico G de A tambm pertence fronteira. Dois dos trs vrtices vizinhos de A sero projetados em vrtices vizinhos de A na fronteira (a menos que, digamos, a face AEHD seja projetada em um segmento, mas nesse caso podemos considerar um vrtice degenerado nesse segmento). Suponha, sem perda de generalidade que esses vizinhos so B e D . Ento E interior projeo. Novamente pela simetria, H e F pertencem fronteira da projeo e C pertence ao interior da projeo. Finalmente, como AE = BF = CG = DH , a projeo do cubo A D HG F B.

As faces ABCD, BCGF e CDHG so projetadas sobre paralelogramos (ou segmentos) e Trace diagonais A BC D, BCG F C D HG. B D,BG e DG. A rea da projeo portanto o dobro da rea do tringulo BDG. Esse tringulo equiltero de lado
EUREKA! N34, 2011

2 , e logo o mximo desejado

52

Sociedade Brasileira de Matemtica

= 3 . Uma projeo ortogonal num plano paralelo ao plano BDG 4 realiza a igualdade.

( 2) 2

PROBLEMA 4: SOLUO DE GUSTAVO LISBA EMPINOTTI (FLORIANPOLIS SC)

Na verdade a recproca verdadeira, mas a implicao direta nem sempre vale, como veremos a seguir.

( ) Suponha

AC BD.

Ento podemos tornar um sistema de coordenadas em que AC o eixo x e BD o eixo y. Sejam A = (a, 0), B = (0, b), C = (c, 0) e D = (0, d). Claramente abcd 0. O coeficiente angular de PM da reta PM

a c d (pois AC BD). Como M = , , a equao b 2 2 d a c y = x . Analogamente, a equao de PN 2 b 2

d c a = x . Como P ( x p , y p ) = PM PN , temos 2 b 2

c a a xp = c xp xp (c a ) = 0 e a c 2 2 x p = 0 P BD (eixo y).

()

Suponha P BD. em que

Ento podemos tomar um sistema de coordenadas P = ( 0 ,0 ) , B = ( 0 ,b ) , D = ( 0,d ) . Sejam A = ( xa , ya ) e C = ( xc , yc )

x y +d x y +d M = c , c , N = a , a . 2 2 2 2 y +d y b (claramente xc 0 ) e o de AB, a O coeficiente angular de PM c xc xa (tambm temos xa 0 ).


y + d ya b PM AB c = 1 ( yc + d )( ya b ) = xa xc (*) xc xa

Analogamente, PN BC ( ya + d )( yc b ) = xa xc
EUREKA! N34, 2011

53

Sociedade Brasileira de Matemtica

( ya + d )( yc b) = ( yc + d )( ya b) ya yc b ya + d yc bd = ya yc byc + d ya bd ya ( b + d ) = yc ( b + d ) ya = yc ou b + d = 0.
Se ya = yc , AC paralelo ao eixo x e, portanto, perpendicular a BD, que o eixo y.
y b ya b yc b Se b + d = 0, (*) vira c = 1, o que implica BC AB (pois xc xc xa ya b so, respectivamente, os coeficientes angulares de BC e AB), ou seja, e xa ABC = 90. Isso caracteriza todos os contra exemplos:

A partir de um tringulo PNM retngulo em P, tome um ponto D dentro do ngulo NPM mas fora do tringulo NPM, e tal que ( DP,NM ) 90. Aplique uma homotetia de centro em D e razo 2. Fazemos N = A, M = C e P = B. Temos um quadriltero ABCD convexo no qual M e o ponto mdio de CD; N , o de AD; e P, o de BD. Como PN // AB e PM PN, temos PM AB e da mesma forma PN BC. ABC = NPM = 90. Esse o contraexemplo, pois pela escolha de D, P BD mas BD no perpendicular a AC.
P= B

N= A

M= C

PROBLEMA 5: SOLUO DE ANDR SARAIVA NOBRE DOS SANTOS (FORTALEZA CE)

Se tivermos 4 pontos, todos da mesma cor ou todos de cores diferentes, teremos o seguinte: 1. Caso: Eles formam um quadriltero convexo:

EUREKA! N34, 2011

54

Sociedade Brasileira de Matemtica

a + b+ c + d = 360 logo, se eles forem diferentes, haver um deles maior que 90 (pois se todos forem menores, a soma no daria 360), logo, todos tm que ser de 90, ou seja, eles tm que formar um retngulo.

2. Caso: Eles formam um quadriltero cncavo: Como a + b > 180, garanto que um deles maior que 90, logo, isso no pode acontecer.

a b

Ento, como 4 pontos de cores diferentes tm que formar um retngulo, no temos mais como ter 5 pontos de 5 cores diferentes nesse conjunto. Sendo assim, podemos ter 1, 2, 3 ou 4 cores: 1 cor: s podemos ter at 4 pontos, pois 3 deles s definem a posio do prximo, s que os 4 tm que formar um retngulo: ex:
A A

2 cores: podemos ter at 8 pontos, de cada cor 4: ex:


A A V V

A V V

EUREKA! N34, 2011

55

Sociedade Brasileira de Matemtica

3 cores: conseguimos uma configurao com at 12 pontos, basta eles ficarem muito afastados, cada retngulo acutngulo, pois assim, ao escolhermos 3 cores diferentes, os ngulos do tringulo vo ser aproximadamente os do tringulo acutngulo: ex:
A A A A

60

V V

V V

60

60

L L

L L

4 cores: j mostramos que s conseguimos 4 pontos: de fato, 3 pontos de cores diferentes determinam uma nica posio possvel para os pontos da outra cor, e logo h apenas um ponto de cada cor. Logo, como checamos todos os casos, vimos que no conseguimos mais de 12 pontos e achamos um exemplo com 12, qualquer n menor tambm satisfaz, pois tirar pontos de uma configurao faz com que a restante tambm satisfaa. Logo, n de 0 at 12 soluo, no existindo mais nenhuma.
PROBLEMA 6: SOLUO DA BANCA

As solues so (1,1), (2,2) e (5,11). Se a par e maior que 2, a equao equivale a (3a/2 1) (3a/2 + 1) = 2b2. Porm mcd(3a/2 1, 3a/2 + 1) = mcd(3a/2 1, 2) = 2 e se conclui que 3a/2 + 1 = 4u2 e 3a/2 1 = 2v2 ou 3a/2 + 1 = 2u2 e 3a/2 1 = 4v2. No primeiro caso, 3a/2 = (2v 1)(2v + 1), e como mcd(2v 1, 2v + 1) = mcd(2v 1, 2) = 1, 2v 1 = 1 v = 1 e a/2 = 1 a = 2 e portanto b = 2.

EUREKA! N34, 2011

56

Sociedade Brasileira de Matemtica

No segundo caso, 3a/2 = 4v2 + 1 0 v2 + 1 (mod 3) v2 1 (mod 3), o que impossvel. Se a mpar, a equao equivalente a 3 (3(a1)/2)2 2b2 = 1. Seja c = 3(a1)/2. Encontremos as solues de 3c2 2b2 = 1 (*). Como e

3+ 2

)(

3 2 =1

3+ 2

) (
2 k +1

3 2

2 k +1

=1

3+ 2

2 k +1

= ck 3 + bk 2 y

3 2

2 k +1

= ck 3 bk 2 (**)

(ck, bk) so solues de (*), para k 0 inteiro. Suponha que existe uma soluo (, ) distinta com , . Ento existe k tal que

3+ 2

2 k 1

< 3 + 2 <

3+ 2

2 k +1

3+ 2< 3+

(5 + 2 6 ) 2 < ( 3 + 2 )(5 2 6 )
k 1

3+ 2

< 9 3 + 11 2
k 1

< 9 3 + 11 2

possvel provar por induo que 3 + 2 5 2 6

)(

k 1

= 3 + 2 ,
Alm disso,

ambos

inteiros

satisfazendo

3 2 2 2 = 1 . e portanto

3 + 2 >1>

Portanto ( , ) soluo de (*), com 1 < < 9. Porm possvel verificar (testando) que no h solues com 1 < < 9, contradio. Para k = 0 temos

3 + 2

= 3 2 > 0,

3 > 2 > 0.

3+ 2

2 k +1

= 3 + 2 , o que nos d a soluo ( c,b ) = (1,1) de


2

(*), e, para k = 1, temos (c, b) = (9, 11) de (*).

3+

2 k +1

= 9 3 + 11 2 , o que nos d a soluo

Suponha que c > 9, ou seja, k > 1. Por (**) e pelo teorema do binmio temos k 2k + 1 m k m 2k + 1 k 1 = ( 2k + 1) 2 k + (***) c= 2m + 1 3 2 3 3 2 + m=0

EUREKA! N34, 2011

57

Sociedade Brasileira de Matemtica

Logo 3 divide 2k + 1. Seja 3t a maior potncia de 3 que divide 2k + 1. Como 2k + 1 k 1 k 1 3 2 = ( 2k + 1) k ( 2k 1) 2 , a maior potncia de 3 que divide o 3 segundo termo de (***) tambm 3t . Para m > 1, sendo 3s a maior potncia de 3 2k + 1 m k m 2k + 1 2k m k m = que divide 2m + 1, o m-simo termo, 2m + 1 3 2 3 2 2m + 1 2m tem pelo menos t + m s fatores 3 (t de 2k + 1, m de 3m, subtraindo s de 2m + 1). Temos m s 2 para todo m > 1. De fato, para m = 2 e m = 3 temos s = 0 e, para (isso segue da desigualdade m 4 ,m s m log 3 ( 2m + 1) 2

3r 1 r + 2 ,r 2 , que pode ser facilmente provada por induo). 2


Ento, como k > 1, todos os termos a partir do terceiro tem pelo menos t + 2 fatores 3, e c = (2k + 1) 2 k + (2k + 1) k (2k 1) 2 k 1 + 3t + 2 N

3( a 1) / 2 = (2k + 1) 2 k 1 [(2k + 1)(k 1) + 3] + 3t + 2 N


Note que como 3 divide 2k + 1 ento tambm divide (2k + 1)(k 1) + 3; alm disso, 9 divide (2k + 1)(k 1), e portanto a maior potncia de 3 que divide (2k + 1)(k 1) + 3 3. Portanto a maior potncia de 3 que divide ( 2 k + 1) 2 k 1 [( 2 k + 1)( k 1) + 3 ] 3t+1 (t fatores de 2k + 1 e 1 de (2k + 1)(k 1) + 3). Finalmente, 3(a1)/2 = c = 3t+1 (1 + 3N), que no possvel pois N > 0. Dessa forma, no h mais solues.

EUREKA! N34, 2011

58

Sociedade Brasileira de Matemtica

XXXII OLIMPADA BRASILEIRA DE MATEMTICA


Problemas e solues da Primeira Fase Nvel Universitrio

PROBLEMA 1:

H muito tempo em uma galxia muito distante, utilizavam-se como referncia para viagens espaciais os pontos A, B, C, D, E, F, G, H, vrtices de um cubo de ares igual a um ano-luz tendo os quadrados ABCD e EFGH como faces e tendo os segmentos AE, BF, CG e DH como arestas. Uma nave espacial viaja com velocidade constante em trajetria retilnea de B para C. Outra nave viaja com velocidade constante igual ao triplo da velocidade da primeira, em trajetria retilnea de A para G. Sabendo que a primeira atinge o ponto C no mesmo instante em que a segunda atinge o ponto G, determine a menor distncia entre as naves durante esse deslocamento.
PROBLEMA 2:

Quantos so os pares ordenados (x, y) com x, y {0 ,1,2 ,...,142} tais que 5 x 2 + 7 y 2 1 mltiplo de 143?
PROBLEMA 3:

prove que existe um polinmio g ( x ) tal que f ( x ) = g ( h ( x ) ) se, e somente se, existe um polinmio com coeficientes complexos em duas variveis q ( x, y ) tal que f ( x ) f ( y ) = q ( x, y ) ( h ( x ) h ( y ) ) .
PROBLEMA 4:

Dados os polinmios com coeficientes complexos em uma varivel f ( x ) e h ( x ) ,

polgono convexo de vrtices (1,0) = (1,ln (1) ) ,( 2,ln ( 2) ) ,( 3,ln ( 3) ) ,...,( n,ln ( n) ) ,( n,0) . Seja Cn = An Bn , o complemento de Bn em relao a An . a) Calcule as eas de An ,Bn e Cn . Simplifique sua resposta. b) Mostre que a rea de Cn menor que 1, para qualquer inteiro positivo n. Obs: ln representa o logaritmo na base e.
EUREKA! N34, 2011

Seja n um inteiro positivo. Seja An o subconjunto do plano definido por 1 x n,0 y ln ( x ) . Seja Bn o

59

Sociedade Brasileira de Matemtica

PROBLEMA 5:

Suponha que temos um grafo com n + 1 4 vrtices e queremos pintar suas arestas com duas cores de forma que no haja duas arestas disjuntas da mesma cor. Mostre que h no mximo 2n tais coloraes. Observaes: Um grafo formado por um conjunto de vrtices e um conjunto de arestas, cada aresta unindo dois vrtices distintos e cada para de vrtices sendo unido por no mximo uma aresta. Arestas disjuntas so arestas que no tm vrtices em comum.
PROBLEMA 6:

Cada um dos itens a seguir apresenta um valor diferente para a matriz B. Para cada um desses valores, determine quantas matrizes reais A existem tais que A3 3 A = B. 0 1 a) B = 1 0

4 b) B = 0 4 c) B = 0

0 4 1 4

SOLUES NVEL UNIVERSITRIO PRIMEIRA FASE PROBLEMA 1

Dando coordenadas, suponha sem perda de generalidade que A = ( 0 ,0 ,0 ) , B = (1,0 ,0 ) , C = (1,1,0 ) , D = ( 0 ,1,0 ) ,

E = ( 0,0 ,1) , F = (1,0,1) , G = (1,1,1) , H = ( 0 ,1,1) .


Se as posies (em funo do tempo) das duas naves so o instante em que ( t ) = B temos

(t )

( t ) , respectivamente, se t = 0 o instante em que ( t ) = C e ( t ) = G e t = 1

( t ) = (1,1,0 ) + t ( 0,1,0 ) , ( t ) = (1,1,1) + 3 t (1,1,1) . Assim o quadrado da distncia em funo do tempo


h ( t ) = (1) 1 + 3t
2

)) + ((1 + t ) (1 + 3t )) + ( 0 (1 + 3t )) ( = 3t + ( 3 1) t + 1 + 2 3t + 3t = (10 2 3 ) t + 2 3t + 1. (
2 2 2 2 2 2 2

EUREKA! N34, 2011

60

Sociedade Brasileira de Matemtica

Temos Para

h ( t ) = 20 4 3 t + 2 3 .
t0 = 2 3 20 4 3 = 3 + 5 3 44 0 , 265 temos h ( t0 ) = 0; para

) (

t < t0 temos h ( t ) < 0 e para t > t0 temos h ( t ) > 0. Assim o mnimo do quadrado da distncia h ( t0 ) = 29 3 3

44 0 ,541

e a distncia mnima

( 29 3 3 )

44 0,7355

Note que 143 = 11 13. Se N p o nmero de pares ordenados (x, y) com

PROBLEMA 2

x, y {0 ,1,..., p 1} tais que 5 x 2 + 7 y 2 1 mltiplo de p, ento a resposta do


problema ser N11 N13 . de fato, 5 x 2 + 7 y 2 1 mltiplo de 143 se, e somente se, mltiplo de 11 e 13. Por outro lado, pelo teorema chins dos restos, dados pares com com ordenados x, y {` 0,1,...,10} e ( x, y ) ( x, y )

x, y {0,1,...,12} , existe um nico par ordenado ( x, y ) com x, y {` 0,1,...,142} tal


que x x ( mod 11) ,x x" ( mod 13) , y y ( mod 11) e y y" ( mod 13) . Vamos agora calcular N11 e N13 . Os possveis valores de x 2 ( mod 11) so 0, 1, 4, 9, 5, 3, sendo cada valor no nulo atingido para duas classes de congruncia mdulo 11. Assim, 5 quadrado mdulo 11 mas 7 no , e portanto 5 x 2 ( mod 11) assume os valores 0, 1, 3, 4, 5, 9, enquanto 7 x 2 ( mod 11) assume os valores 0, 2,6,7,8,10 (nos dois casos os valores no nulos so assumidos duas vezes). Temos que 1 o resultado mdulo 11 da soma de nmeros dessas duas listas nas formas 1 + 0,4 + 8 e 5 + 7, o que d 2 + 4 + 4 = 10 solues mdulo 11 de 5 x 2 + 7 y 2 = 1, e portanto N11 = 10. Analogamente, os possveis valores de x 2 ( mod 13) so 0, 1, 4, 9, 3, 12, 10, sendo cada valor no nulo atingido para duas classes de congruncia mdulo 13. Assim, 5 e 7 no so quadrados mdulo 13, e portanto 5 x 2 ( mod 13) e 7 x 2 ( mod 13) assumen os valores 0, 2, 5, 6, 7, 8, 11 (os valores no nulos so assumidos duas vezes). Temos que 1 o resultado mdulo 13 da soma de dois nmeros dessa lista nas formas 6 + 8,8 + 6 e 7 + 7, o que d 4 + 4 + 4 = 12 solues mdulo 13 de 5 x 2 + 7 y 2 = 1, e portanto N13 = 12.
EUREKA! N34, 2011

61

Sociedade Brasileira de Matemtica

Assim, a resposta do problema N11 N13 = 10 12 = 120. Note que, se g ( x ) = an x n + ... + a1x + a0 , ento
PROBLEMA 3

g ( u ) g ( v ) = an ( u n v n ) + ... + a1 ( u v ) = ( u v ) ( an u n 1 + u n 2 v + ... + v n 1 ) + ... + a1 )


f ( x ) = g ( h ( x ) ) , ento

= R ( u,v ) ( u v ) , para um certo polinmio em duas variveis R ( x, y ) , e logo, se


f ( x ) f ( y ) = g ( h ( x ) ) g ( h ( y ) ) = R ( h ( x ) ,h ( y ) ) h ( x ) h ( y ) = q ( x, y ) ( h ( x ) h ( y ) ) ,

com q ( x, y ) : = R ( h ( x ) ,h ( y ) ) , o que mostra a primeira implicao. Vamos provar a volta por induo no grau de f. Se o grau de f for 0, as duas afirmaes so verdadeiras. Suponha agora que f no constante e que f ( x ) f ( y ) = q ( x, y ) ( h ( x ) h ( y ) ) (da segue que h no constante). Fazendo y = 0, obtemos f ( x ) f ( 0 ) = q ( x,0 ) ( h ( x ) h ( 0 ) ) , para todo x, e portanto, para todo y. Substraindo, obtemos f ( y ) f ( 0 ) = q ( y,0 ) ( h ( y ) h ( 0 ) ) ,

q ( x, y ) ( h ( x ) h ( y ) ) = f ( x ) f ( y ) = q ( x,0 ) ( h ( x ) h ( 0 ) ) q ( y,0 ) ( h ( y ) h ( 0 ) ) =
Assim, h ( x ) h ( y ) divide o polinmio ( q ( x,0 ) q ( y,0 ) ) ( h ( y ) h ( 0 ) ) . Como h no constante e h ( y ) h ( 0 ) um polinmio s na varivel y, segue que h ( x ) h ( y ) no tem nenhum fator comum (no constante) com h ( y ) h ( 0 ) , e portanto
q ( x ) : = q ( x ) q ( 0) . Temos f ( x ) f ( 0) = q ( x,0) ( h ( x ) h ( 0) ) = q ( x ) ( h ( x ) h ( 0) ) ,

h( x) h( y)

divide

polinmio

q ( x,0 ) q ( y,0 ) .

Seja

donde o grau de q menor que o grau de f. Por outro lado, h ( x ) h ( y ) divide o polinmio q ( x,0 ) q ( y,0 ) = q ( x ) q ( y ) , e portanto, por hiptese de induo, existe um polinmio

g ( x)

tal

que

q ( x ) = g ( h ( x )) ,

donde

f ( x ) = f ( 0) + q ( x ) ( h ( x ) h ( 0)) = f ( 0) + g ( h ( x )) ( h ( x ) h ( 0)) = g ( h ( x )) ,
onde g ( x ) : = f ( 0 ) + g ( x ) ( x h ( 0 ) ) , o que completa a demonstrao.

EUREKA! N34, 2011

62

Sociedade Brasileira de Matemtica

PROBLEMA 4

A figura mostra as regies A5 (abaixo da curva vermelha), B5 (abaixo da poligonal azul), e C5 (entre a poligonal azul e a curva vermelha). Temos rea ( An ) = ln ( t ) dt = nln ( n ) n + 1 ;
1 n

1 1 rea ( Bn ) = ln ( 2 ) + ln ( 3) + ... + ln ( n 1) + ln ( n ) = ln ( n! ) ln ( n ) ; 2 2 1 rea ( Cn ) = nln ( n ) n + ln ( n ) + 1 ln ( n! ) . 2 Para estimar rea rea ( Cn ) escreva

ak =

k +1

ln ( t ) ln ( k ) ( t k ) ( ln ( k + 1) ln ( k ) ) dt ;

note que ak a rea da k-sima bochechinha entre a poligonal azul e a curva vermelha. Assim, rea ( Cn ) = a1 + a2 + ... + an 1 ; Queremos estimar ak para mostrar que a srie abaixo converge para S < 1: 0 < S = a1 + a2 + ... + ck + ... < 1.

Seja uk ( t ) = ln ( t ) ln ( k ) ( t k ) ( ln ( k + 1) ln ( k ) ) ; temos uk ( k ) = uk ( k + 1) = 0.
2 Note que u" k ( t ) = t . Integrando por partes temos

ak =
=
k

k +1

uk ( t ) dt

k +1

1 ' t k uk ( t ) dt 2

EUREKA! N34, 2011

63

Sociedade Brasileira de Matemtica


2 2 1 k +1 1 1 1 " t k = t k u'k ( t ) + uk ( t ) dt k 2 2 2 2 k k +1

k +1

Para k t k + 1 temos ( u" k ( t ) ) dt donde


2 1 1 1 1 t k dt = . 2 k 8 2 k 2 12 Temos portanto 1 S (1 + 1 22 + ... + 1 k 2 + ...) 12 + 1 1 1 + t 2 dt ; 1 12 6 Completando a demonstrao.

2 1 1 1 t k ( u" k ( t ) ) dt 8 2 2

ak k 2

k +1

Observao: Este problema mostra como obter estimativas como a de Stirling: temos 0 rea ( Cn ) 1 6 donde

0 nln ( n ) n +

1 1 ln ( n ) + 1 ln ( n! ) 12 6 1 5 1 nln ( n ) n + ln ( n ) + ln ( n! ) nln ( n ) n + ln ( n ) + 1 12 6 2

n n e n e5 3 n n! n n e n e 2 n Sabemos por Stirling que a melhor aproximao n! n n e n 2n ;


note que e5 3 < 2 < e 2 .
PROBLEMA 5

Suponha que algum vrtice do grafo esteja contido em todas as arestas do grafo. Ento o grafo uma estrela com n pontas, e o resultado segue (h exatamente 2n coloraes para este exemplo). Suponha que o grafo tenha um vrtice x de grau 3 (i.e., que pertena a pelo menos 3 arestas) e que exista uma aresta disjunta de x, digamos e. Devido hiptese sobre o grau de x, para aqualquer aresta e, h uma aresta f = f ( e ) que incide em x que disjunta de e. Ento, em qualquer colorao das arestas que
EUREKA! N34, 2011

64

Sociedade Brasileira de Matemtica

incidem em x, a cor de f define a cor de e (a cor de f a oposta de e). Assim, h no grau x mximo 2 ( ) 2n coloraes. Se o grafo tem um vrtice x de grau 2, ligado a dois outros vrtices y e z, ento para toda aresta e disjunta de x que no seja a (possvel) aresta yz, h uma aresta f = f ( e ) que incide em x que disjunta de e, cuja cor determina a cor de e. Assim, as cores de xy, xz e de yz (se existir) determinam todas as outras. Assim, h no mximo 23 2n coloraes. Finalmente, se todo vrtice tem grau no mximo 1, todas as arestas so disjuntas, e nesse caso, pelas hiptese do problema, o grafo pode mximo duas arestas e h no mximo 22 < 2 n coloraes. Antes de mais nada vamos esboar o grfico de f ( x ) = x 3 3 x.
PROBLEMA 6

Vemos que para 2 < y < 2 a equao f ( x ) = y admite trs solues reais enquanto para y < 2 ou y > 2 ela admite uma soluo real e duas complexas conjugadas. a) Os autovalores de B so 1 e 1 donde podemos escrever B = XDX 1 para X inversvel e

EUREKA! N34, 2011

65

Sociedade Brasileira de Matemtica

1 0 D= . 0 1 Sejam c1 ,c2 ,c3 (resp. d1 ,d 2 ,d3 ) as solues reais de f ( x ) = 1( resp. f ( x ) = 1) . Se f ( A) = B temos f ( X 1 AX ) = D donde X 1 AX da forma

c1 0 X 1 AX = 0 dj Para i, j escolhidos independentemente. H portanto 9 matrizes reais A que satisfazem f ( A ) = B.


b) Sejam z,z as razes complexas de f ( x ) = 4. Seja v = ( w1 ,w2 )
2

um vetor

linearmente independente com v = w1 ,w2 e considere A unicamente definida por Av = zv, Av = zv. Em outras palavras,

w w1 z 0 w1 w1 A= 1 . w w2 2 0 z w2 w2 Para qualquer tal A temos f ( A ) = 4 I = B. Temos alm disso A real: h portanto


infinitas matrizes reais A que satisfazem f ( A ) = B. c) Se M diagonalmente ento f ( M ) tambm o . Como B no diagonalmente ento A tambm no o . Assim A deve ter autovalor com multiplicidade algbrica igual a 2 logo o nico autovalor de A o nico real c com f ( c ) = 4. Alm disso qualquer autovetor de A autovetor de B; como e1 (a menos de mltiplo escalar) o nico autovalor de B ento e1 deve ser autovetor de A. J e2 deve ser autovetor generalizado, isto , devemos ter Ae2 = ce2 + ye1 (para algum real y). Assim

e devemos ter y = 1 ( 3c 2 3) . H portanto uma nica soluo.

c y A= 0 c 4 ( 3c 2 3) y f ( A ) = A3 3 A = 0 4

EUREKA! N34, 2011

66

Sociedade Brasileira de Matemtica

XXXII OLIMPADA BRASILEIRA DE MATEMTICA


Problemas e solues da Segunda Fase Nvel Universitrio
PRIMEIRO DIA PROBLEMA 1:
/ 4

Calcule

( sen x + cos x ) cos x dx.


0

PROBLEMA 2:

Qual a maior rea possvel para a sombra de um cubo de aresta 1? (Obs.: supomos que o sol est a pino, isto , a sombra uma projeo ortogonal; o cubo pode estar em qualquer posio).
PROBLEMA 3:

Sejam n1 e n2 inteiros positivos e n = n1n2 .

Considere a matriz real simtrica n n, A = ai , j

( )

1 i , j n

, tal que para todo i,

ai ,i = 4,

ai ,i +1 = ai +1,i = 1

para 1 i n 1 tal que ( i + 1) no mltiplo de n1 ,

ai ,i + n1 = ai + n1 ,i = 1,
e as demais entradas ai , j so iguais a 0. Prove que A invertvel e todas as entradas de A1 so positivas.

EUREKA! N34, 2011

67

Sociedade Brasileira de Matemtica

SEGUNDO DIA PROBLEMA 4:

x x ( x 1)( x 2 ) ...( x j + 1) Definimos os polinmios = para todo j natural, j! j x com = 1. 0


a) Prove que todo polinmio no identicamente nulo pode ser escrito como uma x combinao linear desses de forma nica; j b) Seja

k Calcule n n + k k +1 . n +1
k +1

x o coeficiente de no desenvolvimento de x n (como no item a)). k

Se F um subconjunto finito de 3 , denotamos por Vr ( F ) a vizinhana de raio r de F (i.e., a unio das bolas abertas de raio r com centros pertencentes a F). Prove que, se 0 < r < R, vol (VR ( F ) ) ( R / r ) . vol (Vr ( F ) ) .
3

PROBLEMA 5:

PROBLEMA 6:

Prove que se 102 n + 8.10n + 1 tem um fator primo da forma 60k + 7 ento n e k so pares.

EUREKA! N34, 2011

68

Sociedade Brasileira de Matemtica

PROBLEMA 1: SOLUO DE BRUNO DA SILVA SANTOS (BELFORD ROXO RJ)

I =

4 x sec 2 x dx = x dx. 0 tgx + 1 ( senx + cosx ) cosx

Fazendo: x = u;dv =

sec 2 x dx du = dx e v = ln (1 + tgx ) 1 + tgx

como u dv = u v v du :

I = x ln (1 + tgx )
Seja I 2 =
4

ln (1 + tgx ) dx =

4 ln2 ln (1 + tgx ) dx 0 4

ln (1 + tgx ) dx. Fazendo u = 4 x e du = dx :


4

I2 =

0
4

ln (1 + tg ( 4 u ) ) du =

1 tgu ln 1 + du 1 + tgu
4

I2 = I2 =

ln2 ( ln2 ln (1 + tgu ) ) du = 4

ln (1 + tgu ) du

ln2 I 2 I 2 = ln2. 4 8 4 Portanto I = ln2 ln (1 + tgx ) dx = ln2 ln2 = ln2. 0 4 4 8 8 I = ln2. 4


PROBLEMA 2: SOLUO DE RAFAELTUPYNAMB DUTRA (BELO HORIZONTE MG)

Sendo A o vrtice do cubo que est mais em baixo (um vrtice com altura mnima), as trs faces que contm A esto no escuro. Colocamos os eixos de forma que os vetores unitrios normais a essas trs faces sejam i = (1,0,0);

j = (0,1,0);
com

k = (0,0,1),
2 x 2 y 2 z

consideramos

vetor

unitrio

N = ( N x ,N y ,N z ) ,

N + N + N = 1, paralelo direo dos raios solares.


Projetando as faces escuras do cubo sobre o plano horizontal, vemos que a rea da sombra do cubo igual soma das reas das projees das trs faces escuras. Mas a rea da projeo de uma face igual rea da face original (que 1) multiplicada pelo mdulo do produto escalar entre o vetor normal face e o vetor normal ao
EUREKA! N34, 2011

69

Sociedade Brasileira de Matemtica

plano

de

projeo.

Assim,

rea

da

sombra

N i + N j + N k = Nx + Ny + Nz .
Como a mdia aritmtica menor ou igual mdia quadrtica, temos

Nx + Ny + Nz 3

2 N x2 + N y + N z2

1 3

Nx + N y + Nz 3 .
Logo, a maior rea possvel 3 , que ocorre quando a direo N dos raios solares paralela reta HA que liga dois vrtices opostos do cubo.
PROBLEMA 3: SOLUO DA BANCA

Precisamos supor que n2 > 1. De fato, se n1 = 2 e n2 = 1, nem todas as entradas de A1 so positivas. Pedimos desculpas... Vamos encontrar o inverso de A = I X , onde todas as entradas de X so 0 ou 4 1 1 . Vamos usar a srie ( I X ) = I + X + X 2 + X 3 + ... (note que 4

( I X ) ( I + X + X 2 + ... + X n ) = I X n+1 ).

Vamos mostrar inicialmente que essa

srie converge. Para isso, vamos mostrar (ao final da soluo) que o menor M R tal que Xv M v para todo vetor v em R n (aqui ||.|| denota a norma euclidiana usual) menor que 1. Da segue que X k v M k v para todo vetor v em R n e todo inteiro positivo k, e logo a srie claramente converge. Considere o grafo cujos vrtices so v1 ,v2 ,...,vn e conectamos vi e v j se e somente se a entrada X ij em X
1 4

Pela definio da matriz A, esse grafo pode ser ( v1 ,v2 , ...,vn1 1 ),

decomposto em diversos caminhos, como os seguintes:

( vkn1 ,vkn1 +1, ...,v( k +1) n1 1 ),1 k n2 1 e ( vr ,vr + n1 ,vr + 2 n1 ,...vr + n n1 ),1 r n1 .
Claramente esse grafo conexo. Como a entrada mij em X k no nula se e somente se existe um caminho de i a j com k lados, para quaisquer i, j existe k tal que a entrada correspondente mij em X k no nula (e logo positiva). Isso prova que todas as entradas de A1 so positivas. Finalmente, para mostrar que o menor M R tal que Xv M v para todo vetor v em R n menor que 1, consideremos um vetor v = ( a1 ,a2 ,...,an ) tal que
EUREKA! N34, 2011

70

Sociedade Brasileira de Matemtica

Xv = M v . Note que, como X simtrica e no-negativa, podemos tomar um tal v com Xv=Mv. Temos que, para cada j n , a j-sima coordenada de Xv da ar + arj 2 + ... + arjs( j ) , onde s( j ) 4 , cujo quadrado menor ou igual a forma j1 4 arj1 2 + arj 2 2 + ... + arjs( j ) 2 arj1 2 + arj 2 2 + ... + arjs( j ) 2 s( j) , e, se vale a igualdade, todos 16 4 arj1 2 + arj 2 2 + ... + arjs ( j ) 2 os arji devem ser iguais. Somando os termos para todos os 4 j n , o resultado menor ou igual a a12 + a2 2 + ... + an 2 , pois em cada coluna de X
h no mximo 4 elementos no nulos. Da segue que Xv v e portanto M 1 . Suponha por absurdo que M = 1, isto , que valha a igualdade. Para cada j com a j 0 , todos os arji devem ser iguais a a j (e portanto no nulos). Pela conexidade do grafo definido acima , deveramos ter ento todos os a j iguais, mas nesse caso no vale a igualdade, pois nem todas as colunas tm 4 entradas no nulas.
PROBLEMA 4: SOLUO DE MATEUS OLIVEIRA DE FIGUEIREDO (FORTALEZA CE)
2 2

a)

x i) O polinmio possui grau j j que o produto de j polinmios de grau 1. j ii) Dado n + provemos que todo polinmio de grau n pode ser escrito de x maneira nica como combinao linear dos . j Seja Pn o espao vetorial de todos os polinmios de grau n. Uma base trivial para esse espao : {x0 ,x1 ,x 2 ,...,x n } , j que a0 x0 + a1 x1 + ... + an x n 0 a0 = a1 = a2 = ... = an = 0 e
qualquer polinmio de grau menor ou igual a n pode ser escrito como combinao linear deles. Assim, a dimenso de Pn n + 1.

x iii) Se escrevermos os nessa base temos: j

EUREKA! N34, 2011

71

Sociedade Brasileira de Matemtica

x 1 j j 1 j 2 0 = x + b j 1 x + b j 2 x + ... + b0 x j j! x Assim, note que para escrever s precisamos dos vetores { x 0 ,x1 ,x 2 ,...,x j } j 1 . pois por i) ele possui grau j e o coeficiente de x j j!
Escrevendo matricialmente

1 n! 0 0

b j 1
1

( n 1)!

b j 2 c j 2
1 n 2!

0 0
M

c j 3 d j 3 1

x n xn x n 1 n 1 x x n 2 = x n 2 x0 x 0

Como abaixo da diagonal principal s temos zeros, utilizando Laplace fcil ver que o determinante da matriz : det M =

1 1 1 1 ... 0 n! ( n 1)! ( n 2 )! 1!

Como Det M 0 a matriz possui inversa, ou seja, os x j podem ser escritos como x x x x combinao linear dos . Logo o conjunto S = , ,..., gera Pn e j n 0 1 como possui n + 1 elementos base. Portanto, todo polinmio de Pn pode ser escrito como combinao linear dos elementos de S.

x Podemos estender a propriedade para S' = j 0 j que, para j

j > n,

x deg > n e portanto no aparecer na combinao linear para escrever um j polinmio de Pn .


EUREKA! N34, 2011

72

Sociedade Brasileira de Matemtica

Mas como n foi pego genrico, qualquer polinmio pode ser escrito de manera x nica como combinao dos . j b) x i) Vamos escrever x na base S. i

(ix ) x ( x 1)( x 2 )( x 3) ...( x i + 1)( x i ) x ( x 1)( x 2 ) ...( x i + 1) = +i =


=

x x x ( x 1)( x 2 ) ...( x i 1) ( x i + i ) x ( x 1)( x 2 ) ...( x i + 1) = x = = i! i! i

i! x ( x 1)( x 2 ) ...( x i + 1)( i + 1)

i!

( i + 1)!

x x x + i = ( i + 1) + i i i + 1 i

(i +x1)
x x x x = ( i + 1) + i . i i + 1 i
ii) Sabemos que: n n x xn = . Multiplicando por x temos: i =0 i i

n x n n x x x = i + 1) ( + i (I) i =0 i i i =0 i + 1 i i n +1 n + 1 x Por outro lado, x n +1 = (II) i i =0 i Como todo polinmio no nulo pode ser escrito de maneira nica por S, (I) e (II) devem ter os mesmos coeficientes. n n x Olhando para o coeficiente de + . em (I) temos ( k + 1) k +i k + 1 k n +1 E em II o coeficiente . k +1 x n +1 =
n

EUREKA! N34, 2011

73

Sociedade Brasileira de Matemtica

n n n +1 Logo ( k + 1) + = k +1 k +1 k

n n + k k +1 1 = . n +1 k +1 k +1

PROBLEMA 5: SOLUO DE RAMON MOREIRA NUNES (FORTALEZA CE)

Vamos proceder por induo em #F. No caso inicial F tem um elemento. Trivial:

9 Vol (Vr ( F ) ) = r 3 ,r 3

R Vol (Vr ( F ) ) = Vol (Vr ( F ) ) ,0 < r < R. r

Agora, suponha que se F possui k pontos ento vale o resultado. Se F um conjunto de ( k + 1) pontos, escreva F = { x1 ,...,xk ,xk +1} . Se F = { x1 ,...,xk } , vale o resultado para F i.e.

Vol Vr F r3

( ( )) funo decrescente de r.
Vol (Vr ( F ) ) r
3

Como

Vol Vr F
decrescente

( ( )) + Vol ( B ( x
r

k +1

))

constante

Vol (Vr F Br ( xk +1 ) r3
4 , basta provar 3

( )

E a primeira funo decrescente, a segunda constante igual a

Vol V F B ( x ) r r k +1 . Para fazer isso suponha sem que a ltima crescente 3 r perda de generalidade que xk +1 a origem de 3 i.e. xk +1 = ( 0,0 ,0 ) = 0.
Considere a homotetia

( ( )

Tr

de

centro

x onde Tr ( Br ( x ) ) = B1 ,Tr Vr F = V1 F r , r 1 Vol (Tr A ) = 3 Vol ( A ) para qualquer A pois r


EUREKA! N34, 2011

( ( )) ( )

1 . r x x F r = 1 ,..., k . r r
e razo

Temos Como

74

Sociedade Brasileira de Matemtica

Vol (Tr ( A ) ) = dx = det Tr dx = Vol Vr F Br ( 0 )


3

) = Vol V F B ( 0) . Portanto, ( ( ) ) r r Vol (V ( F ) B ( 0 ) ) crescente. Basta para


1

( ( )
1

Tr A

1 1 dx = 3 Vol ( A ) , temos 3 r r
basta isso mostrar mostrar que que

V1 F r B1 ( 0 ) V1 F R B1 ( 0 ) se r < R. Para isso, usaremos o seguinte lema:


Lema: Dado y
1 2
3

( )

( )
3

fixado, a funo x x y convexa, isto ,


2

( tx + (1 t ) x ) y t x y + (1 t ) x
Prova: Dados x1 ,x2
1 2

y ,t [ 0,1] .

, temos
2

( tx + (1 t ) x ) y = tx + (1 t ) x
1 1

ty (1 t ) y = t ( x1 y ) + (1 t )( x2 y ) . y + (1 t ) x2 y .

Pela desigualdade triangular, isso menor ou igual a t ( x1 y ) + (1 t )( x2 y ) . Como a norma homognea,

( tx + (1 + t ) x ) y t x
2

Agora voltemos prova de que V1 F r B1 ( 0 ) V1 F R B1 ( 0 ) . Tome a V1 Fr B1 ( 0 ) . Ento, i k tal que a Como R > r,

( )

( )

( )

xi < 1 e a < 1. r

xi x est no intervalo 0 , i . Vamos usar a convexidade de R r x x r x x a ; escolha t = [ 0 ,1] tal que i = t i + (1 t ) 0. Temos R R r xi xi x a t a + (1 t ) a 0 < t 1 + (1 t ) 1 = 1. Ou seja, a B1 i V1 F R . R r R

( )

Como a B1 ( 0 ) ,a V1 F R B1 ( 0 ) . Ou seja, acabamos de provar que V1 F r B1 ( 0 ) V1 FR B1 ( 0 ) . Como j


r r k +1

( ) Vol (V ( F ) B ( x ) ) vimos, isso implica que r


j vimos isso implica que r

( )

( )

r Vol (Vr ( F ) )
r3

crescente, e como tambm

decrescente. Conclumos.

EUREKA! N34, 2011

75

Sociedade Brasileira de Matemtica

PROBLEMA 6: SOLUO DE RGIS PRADO BARBOSA (FORTALEZA CE)

Temos n,k

e p primo com p = 60k + 7 tal que p 102 n + 8 10n + 1 . Queremos

provar que n,k so pares. Temos

10 2 n + 8 10 n + 1 = (10 2 n + 2 10 n + 1) + 6 10 n = (10 n + 1) + 6 10 n (10 n + 1)


2 2

6 10n 6 10n ( mod p ) = 1 onde p 0 , se p a a = 1, se a resduo quadrtico mdulo p. p 1, se a no resduo quadrtico mdulo p. / 6 10n = 2n +1 3 5n pois p = 60k + 7 2 60 ,3 60 ,5 60 Veja que claramente p \ / 7 ,3 \ / 7 ,5 \ / 7 p 2,3 e 5. e 2\ Sabemos que o smbolo de Legendre possui a seguinte propriedade: a b a b = . p p p

6 10n 1 2 3 5 Logo: = = 1 (simplesmente separei os fatores). p p p p p Calculemos cada um deles: p 1 60 k + 7 1 1 1 1 30 k + 3 = ( 1) 2 = ( 1) 2 = ( 1) = ( 1) = ( 1) (I) p p p


p 1 2 = ( 1) 8 . Temos p = 60k + 7 p 2 = 3600k 2 + 2 60k 7 + 49 p 2 p 1 3600k 2 + 840k + 48 . = 8 8 p2 1 = 450k 2 + 105k + 6 = 450k 2 + 104k + 6 + k. Assim, 8 par p 2 1 p2 1 2 2 2 k 450 k +104 k + 6 ) + k 450 k +104 k + 6) k ( 1) 8 = ( 1)( ( 1) = ( 1) (II) ( 1) 8 = ( 1)( p
2

n +1

EUREKA! N34, 2011

76

Sociedade Brasileira de Matemtica

3 5 Para calcular e precisaremos da lei da Reciprocidade Quadrtica: dados p p p 1 q 1 p q p, q primos mpares: = ( 1) 2 2 q p


3 1 p 1 3 p 3 p 1( 30 k + 3) Assim: com q = 3, = ( 1) 2 2 = ( 1) = ( 1) . p 3 p 3 p 60k + 7 1 p Assim precisamos de calcular = = = 1 = 1 (usando que 3 3 3 3 2 60k + 7 1 1 ( mod 3) ).

3 3 Substituindo acima tem-se: 1 = ( 1) = ( 1) (III) p p 5 1 p 1 5 p 5 p 2( 30 k + 3) Com q = 5, = ( 1) 2 2 = ( 1) = 1. p 5 p 5

Assim

p 60k + 7 2 p precisamos calcular o valor de = = = ( 1) = ( 1) (pois 5 5 5 5 60k + 7 2 ( mod 5 ) e os resduos quadrticos mdulo 5 so 0, 1, 4 j que

x 0,1,2,3,4 ( mod 5 ) x 2 0 ,1,4 ,4,1( mod 5 ) ). 5 5 p 5 Substituindo acima = 1 ( 1) = 1 = ( 1) (IV) p p 5 p Juntam-se (I), (II), (III) e (IV) em (*): 1 2 3 5 k ( n +1) n k ( n +1) + n ( 1) ( 1) = 1 ( 1) =1 = 1 ( 1) ( 1) p p p p k ( n + 1) + n par.
se n mpar ( n + 1) par k ( n + 1) par k ( n + 1) + n mpar. Absurdo! Logo n par ( n + 1) mpar. se k mpar k ( n + 1) mpar k ( n + 1) + n mpar. Absurdo! Logo k par . Assim, dados n,k
+ n +1 n

com p = 60k + 7 primo tal que p 102 n + 8 10n + 1 ento n e

k so pares.

EUREKA! N34, 2011

77

Sociedade Brasileira de Matemtica

XXXII OLIMPADA BRASILEIRA DE MATEMTICA


Premiados
NVEL 1 (6. e 7. Anos)
NOME Ana Emlia Hernandes Dib Pedro Henrique Alencar Costa Ryunosuke Watanabe Tagami Helena Veronique Rios Italo Lesione de Paiva Rocha Jos Henrique Carvalho Juliana Bacelar de Freitas Daniel Lima Braga Hermes Lins e Nascimento Las Monteiro Pinto Lucca Morais de Arruda Siaudzionis Leandro Alves Cordeiro Henrique Gontijo Chiari Andr Akinaga Benites Gabriel Diniz Vieira e Sousa Rafael Seiji Uezu Higa Adriana de Sousa Figueiredo Gustavo Figueiredo Serra Matheus Ucha Constante Kristian Holanda Nogueira Fbio Itikama Loic Dominguez Jiang Zhi Ricardo Ken Wang Tsuzuki Ana Caroline Obana da Cruz Ana Paula Lopes Schuch Jos Marcio Machado de Brito Lucas Bastos Germano Victria Moreira Reis Cogo Thiago Araujo Oliveira Gabriel Toneatti Vercelli Nathan Bonetti Teodoro Jefferson Daxian Hong Cristbal Sciutto Rodriguez Aruana Almeida Correa Cynthia Lacroix Herkenhoff Kaque Maestrini Sacchi Igor de Lacerda
EUREKA! N34, 2011

CIDADE ESTADO S.J. do Rio Preto - SP Fortaleza - CE Rio Claro - SP So Carlos - SP Fortaleza - CE Curitiba - PR Braslia - DF Eusbio - CE Fortaleza - CE Rio de Janeiro - RJ Fortaleza - CE Ribeiro Pires - SP Belo Horizonte - MG So Paulo - SP Fortaleza - CE So Paulo - SP Porto Alegre - RS So Paulo - SP Goinia - GO Manaus - AM So Paulo - SP Fortaleza - CE So Paulo - SP So Paulo - SP Curitiba - PR Porto Alegre - RS Cocal dos Alves - PI Fortaleza - CE Teresina - PI Jaboato dos Guararapes - PE Osasco - SP Curitiba - PR So Paulo - SP So Paulo - SP Porto Alegre - RS Vitria - ES So Paulo - SP Curitiba - PR

PRMIO Ouro Ouro Ouro Ouro Ouro Ouro Prata Prata Prata Prata Prata Prata Prata Prata Prata Prata Prata Prata Bronze Bronze Bronze Bronze Bronze Bronze Bronze Bronze Bronze Bronze Bronze Bronze Bronze Bronze Bronze Bronze Bronze Bronze Meno Honrosa Meno Honrosa

78

Sociedade Brasileira de Matemtica


Rafael Reple Geromee Leonardo de Matos Felippetti Mariano Gabriel Passamani Correa Daniel de Almeida Souza Diego Teixeira Nogueira Fidalgo Natan Novellu Tu Ricardo Borsari Brinati Rafael Neves Vieira Juliano Pecica Negri Gustavo Rodrigues Machado Zoltan Flamarion Glueck Carvalho Gabriel Ribeiro Barbosa Pedro Henrique Rocha de Freitas Pedro Henrique Sacramento de Oliveira Guilherme Goulart Kowalczuk Pedro de Vasconcellos Oporto Aryssa Victoria Shitara Ives Vaz Caldeira Lopes Marcos Vincius de Oliveira Soares Jssica Carolina Zilio Joo Pedro Graa Melo Vieira Henrique Medici Pontieri Gabriel Caino Castilho Rodrigues Tamara P. de A. Moraes Karine Quaresma Lima Natlia Brasileiro Lins Barbosa Lucki Li Helosa Antunes de Medeiros Iuri Grangeiro Carvalho Lara Sampaio Pinheiro de Freitas Maria Jlia Costa Medeiros Kevin Korpasch Sofa Leite Correia Lima Joo Baptista de Paula e Silva Bernardo Puetter Schaeffer Jlia Bertelli Rafael Purim de Azevedo Pedro Henrique da Silva Dias Marcelo Bandeira de Melo Boavista Gabriel Branco Frizzo Maria Eduarda Mller Eyng Henrique Martnez Rocamora Felipe Roz Barscevicius Joo Vitor Vaz Oliveira Mateus Siqueira Thimoteo
EUREKA! N34, 2011

So Paulo - SP Curitiba - PR Vitria - ES Braslia - DF Salvador - BA So Paulo - SP So Paulo - SP Braslia - DF Piracicaba - SP Sorocaba - SP Belo Horizonte - MG Fortaleza - CE Braslia - DF Loureira - SP Porto Alegre - RS Nova Lima - MG So Paulo - SP So Paulo - SP Rio de Janeiro - RJ Piracicaba - SP Rio de Janeiro - RJ Campo Grande - MS Salvador - BA Feira de Santana - BA Taguatinga - DF Jaboato dos Guararapes - PE So Paulo - SP Itamogi - MG Fortaleza - CE Olinda - PE Fortaleza - CE Guarapuana - PR Fortaleza - CE Belo Horizonte - MG Rio de Janeiro - RJ Joinville - SC Pirassununga - SP Porto Alegre - RS Teresina - PI Curitiba - PR Porto Alegre - RS So Bernardo do Campo - SP Sorocaba - SP Recife - PE Mogi das Cruzes - SP

Meno Honrosa Meno Honrosa Meno Honrosa Meno Honrosa Meno Honrosa Meno Honrosa Meno Honrosa Meno Honrosa Meno Honrosa Meno Honrosa Meno Honrosa Meno Honrosa Meno Honrosa Meno Honrosa Meno Honrosa Meno Honrosa Meno Honrosa Meno Honrosa Meno Honrosa Meno Honrosa Meno Honrosa Meno Honrosa Meno Honrosa Meno Honrosa Meno Honrosa Meno Honrosa Meno Honrosa Meno Honrosa Meno Honrosa Meno Honrosa Meno Honrosa Meno Honrosa Meno Honrosa Meno Honrosa Meno Honrosa Meno Honrosa Meno Honrosa Meno Honrosa Meno Honrosa Meno Honrosa Meno Honrosa Meno Honrosa Meno Honrosa Meno Honrosa Meno Honrosa

79

Sociedade Brasileira de Matemtica


Ebenezeer Pinto Banoeira Neto Maria Clara Vasconcelos Andrade Rafael Beck Arthur Monteiro Dos Santos Jlia Wotzasek Pereira Gabriel Oliveira Rigo Leonardo Galante Barco Bruno Scatolini Lucas Pereira Galvo de Barros Vtor Ossamu Rodrigues Okamura Fortaleza - CE Braslia - DF Salvador - BA Salvador - BA So Paulo - SP Cotia - SP So Paulo - SP So Paulo - SP So Paulo - SP Braslia DF Meno Honrosa Meno Honrosa Meno Honrosa Meno Honrosa Meno Honrosa Meno Honrosa Meno Honrosa Meno Honrosa Meno Honrosa Meno Honrosa

NVEL 2 (8. e 9. Anos)


NOME Rafael Rodrigues Rocha de Melo Vincius Canto Costa Henrique Vieira G. Vaz Fellipe Sebastiam da Silva P. Pereira Roberto Tadeu Abrantes de Arajo Pedro Victor Falci de Rezende Alessandro A. de Oliveira Pacanowski Lincoln de Queiroz Vieira Tadeu Pires de Matos Belford Neto Vitor Ramos de Paula Francisco Markan N. de Souza Filho Jair Gomes Soares Jnior Breno Soares da Costa Vieira Gabriel Jos Moreira da Costa Silva Pedro Morais de Arruda Siaudzionis Gabriel Sena Galvo Fabio da Silva Soares Michel Rozenberg Zelazny Bruno Eidi Nishimoto Franco Matheus de Alencar Severo Aim Parente de Sousa Marcos Paulo Nunes de Lima Silva Gabriel N. Coelho de Togni de Souza Rafael T. Eugnio Pontes Barone Murilo Corato Zanarella Rodrigo Sanches Angelo Alexandre Perozim de Faveri Luze Mello D'urso Vianna Maria Clara Cardoso Liara Guinsberg Lucas Cawai Julio Pereira
EUREKA! N34, 2011

CIDADE ESTADO Caucaia - CE Rio de Janeiro - RJ So Paulo - SP Rio de Janeiro - RJ Rio de Janeiro - RJ Santo Antonio - MG Rio de Janeiro - RJ Fortaleza - CE Fortaleza - CE Belo Horizonte - MG Fortaleza - CE Montes Claros - MG J. dos Guararapes - PE Macei - AL Fortaleza - CE Guar - DF Planaltina - DF So Paulo - SP Jales - SP Rio de Janeiro - RJ Fortaleza - CE Rio de Janeiro - RJ Rio de Janeiro - RJ Aracatuba - SP Amparo - SP So Paulo - SP Neves Paulista - SP Rio de Janeiro - RJ So Paulo - SP So Paulo - SP Caucaia - CE

PRMIO Ouro Ouro Ouro Ouro Ouro Ouro Prata Prata Prata Prata Prata Prata Prata Prata Prata Prata Prata Prata Prata Prata Prata Bronze Bronze Bronze Bronze Bronze Bronze Bronze Bronze Bronze Bronze

80

Sociedade Brasileira de Matemtica


Luis Guilherme Gomes Aguiar Carlos Adriano Vieira Daniel Santana Rocha Raphael Mendes de Oliveira Samuel Brasil de Albuquerque Gustavo Souto Henriques Campelo Lucas de Moura Herlin Vitor Dias Gomes Barrios Marin Joo Pedro Sedeu Godoi Suzane Eberhart Ribeiro da Silva caro Sampaio Viana Pedro Henrique Bortolozo Maria Fbio Kenji Arai Guilherme de Oliveira Rodrigues Alexandre Mendona Cardoso Leyberson Pereira Assuno Rubens Martins Bezerra Farias Joo Vtor Fernandes Paiva Bruno Almeida Costa Daniel Lima Santanelli Marlia Nascimento Monteiro Igor Albuquerque Araujo Josu Knorst Ricardo Vieira Marques Jlio Csar de Barros Thomas Akio Ikeda Valvassori Gabriel Fazoli Domingos Henrique Luan Gomes Pereira Braga Beatriz Yumi Ota Kiane Sassaki Menezes Eric Gripa Marques Samuel Kuo Chen Shao Pedro Henrique Jagosenit Vilaa Caio de Souza Cmara Lucas David Noveline Lucas Rebelo Vieira da Silva Elias Brito Oliveira Guilherme Ryu Odaguiri Kobori Mariana Souza de Arajo Francisco Cludio Coelho Murilo Leo Pereira Jadi Diniz Guimares de Queiroz Caio Lima Albuquerque Carolina Lima Guimares Rio de Janeiro - RJ Igarap - MG Rio de Janeiro - RJ Rio de Janeiro - RJ Fortaleza - CE Joo Pessoa - PB Rio de Janeiro - RJ Presidente Prudente - SP Rio de Janeiro - RJ Campo Grande - MS Fortaleza - CE Colombo - PR So Paulo - SP Fortaleza - CE Salvador - BA Fortaleza - CE Sobral - CE Rio de Janeiro - RJ Fortaleza - CE Rio de Janeiro - RJ Recife - PE Rio de Janeiro - RJ Picada Caf - RS Braslia - DF Santo Andr - SP Mogi das Cruzes - SP Urups - SP Belem - PA So Paulo - SP Rio de Janeiro - RJ Rio de Janeiro - RJ So Paulo - SP Santa Branca - SP Manaus - AM Belem - PA Recife - PE Braslia - DF So Paulo - SP Recife - PE Rio de Janeiro - RJ Belem - PA Recife - PE So Paulo - SP Vitria - ES Bronze Bronze Bronze Bronze Bronze Bronze Meno Honrosa Meno Honrosa Meno Honrosa Meno Honrosa Meno Honrosa Meno Honrosa Meno Honrosa Meno Honrosa Meno Honrosa Meno Honrosa Meno Honrosa Meno Honrosa Meno Honrosa Meno Honrosa Meno Honrosa Meno Honrosa Meno Honrosa Meno Honrosa Meno Honrosa Meno Honrosa Meno Honrosa Meno Honrosa Meno Honrosa Meno Honrosa Meno Honrosa Meno Honrosa Meno Honrosa Meno Honrosa Meno Honrosa Meno Honrosa Meno Honrosa Meno Honrosa Meno Honrosa Meno Honrosa Meno Honrosa Meno Honrosa Meno Honrosa Meno Honrosa

EUREKA! N34, 2011

81

Sociedade Brasileira de Matemtica

Nvel 3 (Ensino Mdio)


NOME Gustavo Lisba Empinotti Marcelo Tadeu de S Oliveira Sales Joo Lucas Camelo S Hanon Guy Lima Rossi Maria Clara Mendes Silva Matheus Secco Torres da Silva Lucas Loureno Hernandes Deborah Barbosa Alves Henrique G. Fiuza do Nascimento Luiz Filipe Martins Ramos Andr Macieira Braga Costa Thiago Saksanian Hallak Victor Juca Martins Caque Porto Lira Gustavo H. F. e Sampaio Braga Alvaro Lopes Pedroso Andr Amaral de Sousa Marcos Massayuki Kawakami Carlos Henrique de Andrade Silva Rafael Kazuhiro Miyazaki Andr Saraiva Nobre dos Santos Daniel Eiti Nishida Kawai Lucas de Freitas Smaira Cssio dos Santos Sousa Alessandro Macdo de Arajo Breno Vieira da Silva Passos Iago Dalmaso Brasil Dias Isabella Amorim Gonalez Daniel dos Santos Bossle Davi Coelho Amorim Lucas Mestres Mendes Vincius Gomes Pereira Renan Pablo da Cruz Jonas Rocha Lima Amaro Iuri Rezende Souza Matheus Arajo Marins Felipe Vieira de Paula Rafael Farias Marinheiro Elvis Falcao de Araujo Pablo Almeida Gomes Paulo Gabriel Ramos Monteiro Victor de Oliveira Bitares CIDADE ESTADO Florianpolis - SC Salvador - BA Fortaleza - CE So Paulo - SP Pirajuba - MG Rio de Janeiro - RJ So Paulo - SP So Paulo - SP Braslia - DF Niteri - RJ Belo Horizonte - MG So Paulo - SP Fortaleza - CE Fortaleza - CE So Jos dos Campos-SP Santa Isabel - SP Diadema - SP So Paulo - SP Fortaleza - CE So Paulo - SP Fortaleza - CE Atibaia - SP Guaxup - MG Osasco - SP Fortaleza - CE Aracaju - SE Rio de Janeiro - RJ Marlia - SP Porto Alegre - SP Fortaleza - CE Fortaleza - CE Rio de Janeiro - RJ Fortaleza - CE Fortaleza - CE Mineiros - GO So Gonalo - RJ Fortaleza - CE Recife - PE Fortaleza - CE Santana de Pirapama - MG Rio de Janeiro - RJ Betim - MG PRMIO Ouro Ouro Ouro Ouro Ouro Prata Prata Prata Prata Prata Prata Prata Prata Prata Prata Prata Prata Bronze Bronze Bronze Bronze Bronze Bronze Bronze Bronze Bronze Bronze Bronze Bronze Bronze Bronze Bronze Bronze Bronze Bronze Meno Honrosa Meno Honrosa Meno Honrosa Meno Honrosa Meno Honrosa Meno Honrosa Meno Honrosa

EUREKA! N34, 2011

82

Sociedade Brasileira de Matemtica


Daniel Caueh Dunaiski Figueira Leal Raphael Julio Barcelos Fernando Fonseca Andrade Oliveira Felipe Mendes dos Santos Felipe Abella C. Mendona de Souza Francisco Raul Lobo Rodrigues Gabriel Leite de Carvalho Andr Austregesilo Scussel Victorio Takahashi Chu Victor Jos Tiburtius Franco Matheus Cavalcante Lima Cleberton de Santana Oliveira Mauro Brito Jnior Gabriel Jos Guimares Barbosa Lucas Colucci Cavalcante de Souza Sarah Villanova Borges Ivan Tadeu Ferreira Antunes Filho Dalton Felipe de Menezes Thiago de Paula Vasconcelos Jardiel Freitas Cunha Ana Beatriz Prudncio de A. Rebouas Rafael Sussumu Yamaguti Miada Davi Sampaio de Alencar Bruno Ferri de Moraes Curitiba - PR Taguatinga - DF Belo Horizonte - MG Gama - DF Joo Pessoa - PB Fortaleza - CE Fortaleza - CE Fortaleza - CE So Paulo - SP Recife - PE Fortaleza - CE So Miguel do Aleixo - SE Fortaleza - CE Pequi - MG So Paulo - SP Juiz de Fora - MG Lins - SP So Jos dos Campos-SP Fortaleza - CE Recife - PE Fortaleza - CE Valinhos - SP Fortaleza - CE So Paulo - SP Meno Honrosa Meno Honrosa Meno Honrosa Meno Honrosa Meno Honrosa Meno Honrosa Meno Honrosa Meno Honrosa Meno Honrosa Meno Honrosa Meno Honrosa Meno Honrosa Meno Honrosa Meno Honrosa Meno Honrosa Meno Honrosa Meno Honrosa Meno Honrosa Meno Honrosa Meno Honrosa Meno Honrosa Meno Honrosa Meno Honrosa Meno Honrosa

Nvel Universitrio
NOME Rafael Tupynamb Dutra Renan Henrique Finder Regis Prado Barbosa Ramon Moreira Nunes Thoms Yoiti Sasaki Hoshina Guilherme Rodrigues N. de Souza Jorge Henrique Craveiro de Andrade Rafael Assato Ando Gabriel Lus Mello Dalalio Charles Barbosa de Macedo Brito Leonardo Ribeiro de Castro Carvalho Marcelo Matheus Gauy Leandro Farias Maia Adenilson Arcajo de Moura Jnior Paulo Andr Carvalho de Melo Joas Elias dos Santos Rocha CIDADE ESTADO Belo Horizonte - MG Rio de Janeiro - RJ Fortaleza - CE Fortaleza - CE Rio de Janeiro - RJ S.J. dos Campos - SP Rio de Janeiro - RJ Campinas - SP S.J. dos Campos - SP Rio de Janeiro - RJ S.J. dos Campos - SP So Jos do Rio Preto-SP Rio de Janeiro - RJ Fortaleza - CE Piedade - RJ Muribeca - SE PREMIO Ouro Ouro Ouro Ouro Ouro Prata Prata Prata Prata Prata Prata Prata Prata Bronze Bronze Bronze

EUREKA! N34, 2011

83

Sociedade Brasileira de Matemtica


Guilherme Loureno Mejia Reinan Ribeiro Souza Santos Rafael Alves da Ponte Davi Lopes Alves de Medeiros Luca Mattos Moller Renato Rebouas de Medeiros Danilo Furlan Kaio Rafael Endlich Pimentel Paulo Srgio de Castro Moreira Carlos Coelho Lechner Thiago Ribeiro Ramos Hugo Fonseca Arajo Alysson Espndola de S Silveira Jordan Freitas Piva rik Fernando de Amorim Daniel Ungaretti Borges Antnio Deromir Neves Silva Jnior Rafael Parpinel Cavina Isaque Santa Brigida Pimentel Mateus Oliveira de Figueiredo Davi Dos Santos Lima Bruno da Silva Santos Francisco Osman Pontes Neto Breno Vieira de Aguiar Ricardo Turolla Bortolotti Guilherme Philippe Figueiredo Daniel de Barros Soares Hudson do Nascimento Lima Eduardo Fischer Luty Rodrigues Ribeiro Jos Leandro Pinheiro Caio Ishizaka Costa Gabriel Caser Brito Leonardo Donisete da Silva Alan Anderson da Silva Pereira Diego Andrs de Barros Lima Barbosa Renato Dias Costa Ivan Guilhon Mitoso Rocha Willy George do Amaral Petrenko Leonardo Borges Avelino Jose Armando Barbosa Filho S.J. dos Campos - SP Lagarto - SE Fortaleza - CE Fortaleza - CE Nova Friburgo - RJ Fortaleza - CE So Paulo - SP Vitria - ES Fortaleza - CE Rio de Janeiro - RJ Varginha - MG Rio de Janeiro - RJ Fortaleza - CE Rio de Janeiro - RJ Araraquara - SP Belo Horizonte - MG Fortaleza - CE So Paulo - SP Barcarena - PA Fortaleza - CE Macei - AL Belford Roxo - RJ Fortaleza - CE Rio de Janeiro - RJ Rio de Janeiro - RJ So Paulo - SP Rio de Janeiro - RJ Fortaleza - CE Encantado - RS Fortaleza - CE Fortaleza - CE S.J. dos Campos - SP Rio de Janeiro - RJ Campinas - SP Unio dos Palmares - AL Rio de Janeiro - RJ Rio de Janeiro - RJ Fortaleza - CE Rio de Janeiro - RJ Rio de Janeiro - RJ Fortaleza - CE Bronze Bronze Bronze Bronze Bronze Bronze Bronze Bronze Bronze Meno Honrosa Meno Honrosa Meno Honrosa Meno Honrosa Meno Honrosa Meno Honrosa Meno Honrosa Meno Honrosa Meno Honrosa Meno Honrosa Meno Honrosa Meno Honrosa Meno Honrosa Meno Honrosa Meno Honrosa Meno Honrosa Meno Honrosa Meno Honrosa Meno Honrosa Meno Honrosa Meno Honrosa Meno Honrosa Meno Honrosa Meno Honrosa Meno Honrosa Meno Honrosa Meno Honrosa Meno Honrosa Meno Honrosa Meno Honrosa Meno Honrosa Meno Honrosa

EUREKA! N34, 2011

84

Sociedade Brasileira de Matemtica

AGENDA OLMPICA
XXXIII OLIMPADA BRASILEIRA DE MATEMTICA NVEIS 1, 2 e 3 Primeira Fase sbado, 18 de junho de 2011 Segunda Fase sbado, 3 de setembro de 2011 Terceira Fase sbado, 15 de outubro de 2011 (nveis 1, 2 e 3) domingo, 16 de outubro de 2011 (nveis 2 e 3 - segundo dia de prova) NVEL UNIVERSITRIO Primeira Fase sbado, 3 de setembro de 2011 Segunda Fase sbado, 15 e domingo, 16 de outubro de 2011 IV ROMANIAN MASTER OF MATHEMATICS (RMM) 23 a 28 de fevereiro de 2011(Bucareste, Romnia) ASIAN PACIFIC MATH OLYMPIAD (APMO) 12 de maro de 2011 XVII OLIMPADA DE MAIO 7 de maio de 2011 XXII OLIMPADA DE MATEMTICA DO CONE SUL 14 a 20 de agosto de 2011(La Paz, Bolvia) LII OLIMPADA INTERNACIONAL DE MATEMTICA 13 a 24 de julho de 2011(Amsterdam, Holanda) I OLIMPADA DE MATEMTICA DA LUSOFONIA 20 a 31 de julho de 2011(Coimbra, Portugal) XVII OLIMPADA INTERNACIONAL DE MATEMTICA UNIVERSITRIA (IMC) 24 a 30 de julho de 2011(Blagoevgrad, Bulgria) XXV OLIMPADA IBEROAMERICANA DE MATEMTICA 23 de setembro a 1 de outubro de 2011(So Jos, Costa Rica) III COMPETIO IBEROAMERICANA INTERUNIVERSITRIA DE MATEMTICA 2 a 8 de outubro de 2011(Quito, Equador) XIII OLIMPADA IBEROAMERICANA DE MATEMTICA UNIVERSITRIA 26 de novembro de 2011

EUREKA! N34, 2011

85

Sociedade Brasileira de Matemtica

COORDENADORES REGIONAIS
Alberto Hassen Raad Amrico Lpez Glvez Antonio Carlos Nogueira Benedito Tadeu Vasconcelos Freire Bruno Holanda Carmen Vieira Mathias Claus Haetinger Cludio de Lima Vidal Denice Fontana Nisxota Menegais Disney Douglas Lima de Oliveira Edson Roberto Abe Edney Aparecido Santulo Jr. Fbio Brochero Martnez Florncio Ferreira Guimares Filho Francinildo Nobre Ferreira Genildo Alves Marinho Herivelto Martins Gilson Tumelero Ivanilde Fernandes Saad Joo Bencio de Melo Neto Joo Francisco Melo Libonati Diogo Diniz Jos Luiz Rosas Pinho Jos Vieira Alves Jos William Costa Krerley Oliveira Licio Hernandes Bezerra Luciano G. Monteiro de Castro Luzinalva Miranda de Amorim Marcelo Dias Marcelo Antonio dos Santos Marcelo Rufino de Oliveira Newman Simes Nivaldo Costa Muniz Osnel Broche Cristo Uberlndio Batista Severo Raul Cintra de Negreiros Ribeiro Reginaldo de Lima Pereira Reinaldo Gen Ichiro Arakaki Ricardo Amorim Ronaldo Alves Garcia Rogrio da Silva Igncio Rosangela Ramon Srgio Cludio Ramos Seme Gebara Neto Tadeu Ferreira Gomes Toms Menndez Rodrigues Valdenberg Arajo da Silva Wagner Pereira Lopes Wanderson Breder William Serafim dos Reis
EUREKA! N34, 2011

(UFJF) (USP) (UFU) (UFRN) (CAEN UFC) (UNIFRA) (UNIVATES) (UNESP) (UNIPAMPA) (UFAM) (Colgio Objetivo de Campinas) (UEM) (UFMG) (UFES) (UFSJ) (Centro Educacional Leonardo Da Vinci) (USP So Carlos) (UTFPR) (UC. Dom Bosco) (UFPI) (Grupo Educacional Ideal) (UFPB) (UFSC) (UFPB) (Instituto Pueri Domus) (UFAL) (UFSC) (Sistema Elite de Ensino) (UFBA) (Grupo Educacional Etapa) FACOS (Grupo Educacional Ideal) (Cursinho CLQ Objetivo) (UFMA) (UFLA) (UFPB) (Colgio Anglo) (Escola Tcnica Federal de Roraima) (UNIFESP) (Centro Educacional Logos) (UFGO) (Col. Aplic. da UFPE) (UNOCHAPEC) (IM-UFRGS) (UFMG) (UEBA) (U. Federal de Rondnia) (U. Federal de Sergipe) (CEFET GO) (CEFET RJ) (UFT TO)

Juiz de Fora MG Ribeiro Preto SP Uberlndia MG Natal RN Fortaleza CE Santa Mara RS Lajeado RS S.J. do Rio Preto SP Bag RS Manaus AM Campinas SP Maring PR Belo Horizonte MG Vitria ES So Joo del Rei MG Taguatingua DF So Carlos SP Pato Branco PR Campo Grande MS Teresina PI Belm PA Campina Grande PB Florianpolis SC Campina Grande PB Santo Andr SP Macei AL Florianpolis SC Rio de Janeiro RJ Salvador BA So Paulo SP Osrio RS Belm PA Piracicaba SP So Luis MA Lavras MG Joo Pessoa PB Atibaia SP Boa Vista RR SJ dos Campos SP Nova Iguau RJ Goinia GO Recife PE Chapec SC Porto Alegre RS Belo Horizonte MG Juazeiro BA Porto Velho RO So Cristvo SE Jata GO Nova Friburgo RJ Arraias TO

86

CONTEDO
XVII OLIMPADA DE MAIO Enunciados e resultado brasileiro XXII OLIMPADA DE MATEMTICA DO CONE SUL Enunciados e resultado brasileiro LII OLIMPADA INTERNACIONAL DE MATEMTICA (IMO) Enunciados e resultado brasileiro LIII OLIMPADA INTERNACIONAL DE MATEMTICA (IMO) Enunciados e resultado brasileiro II OLIMPADA DE MATEMTICA DA COMUNIDADE DOS PASES DE LNGUA PORTUGUESA Enunciados e resultado brasileiro XXVI OLIMPADA IBEROAMERICANA DE MATEMTICA Enunciados e resultado brasileiro 16 ARTIGOS PARIDADE DO NMERO BINOMIAL MDIO J.C.S. de Miranda (USP) e Iesus C. Diniz (FRN) PONTO MDIO LEMBRA? OUTRO PONTO MDIO! DOIS PONTOS MDIOS LEMBRAM? BASE MDIA! Ccero Thiago Magalhes CAMPEONATOS Jos Armando Barbosa Filho SOLUES DE PROBLEMAS PROPOSTOS PROBLEMAS PROPOSTOS AGENDA OLMPICA COORDENADORES REGIONAIS 03

06

08 10

13

18 29 36

47 59 61 62

Sociedade Brasileira de Matemtica

Esta edio da Revista Eureka! dedicada memoria do professor Srgio Cludio Ramos da Universidade Federal do Rio Grande do Sul (UFRGS), que colaborou como coordenador regional da Olimpada Brasileira de Matemtica (OBM) desde 1998, e que nos deixou neste ano de 2012. A comunidade da Olimpada Brasileira de Matemtica (OBM)

EUREKA! N35, 2012

Sociedade Brasileira de Matemtica

XVII OLIMPADA DE MAIO


PRIMEIRO NVEL
PROBLEMA 1

As quatro palavras codificadas

*
so em alguma ordem AMO Decifrar

#
SUR

*
REO

MAS

*# .

PROBLEMA 2

Utilizando apenas uma vez cada um dos dgitos 1, 2, 3, 4, 5, 6, 7 e 8 se escrevem o quadrado e o cubo de um nmero inteiro positivo. Determine quanto pode valer este nmero.
PROBLEMA 3

1 CD e F o ponto onde se cortam AE e BD. 3 O tringulo DFE tem rea 12 e o tringulo ABF tem rea 27. Encontre a rea do quadriltero BCEF.

No retngulo ABCD, BC = 5, EC =

PROBLEMA 4

Utilizando vrios cubinhos brancos de aresta 1, Guille monta um cubo grande. Em seguida, escolhe quatro faces do cubo grande e as pinta de vermelho. Finalmente desmonta o cubo grande e observa que os cubinhos com pelo menos uma face pintada de vermelho so 431. Encontre a quantidade de cubinhos que Guille utilizou para montar o cubo grande. Analise todas as possibilidades.
EUREKA! N35, 2012

Sociedade Brasileira de Matemtica

SEGUNDO NVEL
PROBLEMA 1

Encontre um nmero inteiro positivo x tal que a soma dos dgitos de x seja maior que 2011 vezes a soma dos dgitos do nmero 3x (3 vezes x).

PROBLEMA 2

Dizemos que um nmero de quatro dgitos abcd (a 0) por se valem as seguintes condies: a b; ab cd = cd ba. por exemplo, 2011 por porque 20 11 = 11 02. Encontre todos os nmeros por.

PROBLEMA 3

Num tringulo retngulo ABC tal que AB = AC, M o ponto mdio de BC. Seja P um ponto da mediatriz de AC que pertence ao semiplano determinado por BC que no contm A. As retas CP e AM se cortam em Q. Calcule o ngulo que formam AP e BQ.
PROBLEMA 4

Dados n pontos em uma circunferncia se escreve ao lado de um deles um 1 e ao lado de cada um dos outros um 0. A operao permitida consiste em escolher um ponto que tenha um 1 e trocar o nmero desse ponto e tambm os nmeros dos seus dois vizinhos, o da esquerda e o da direita (onde h 1 se escreve 0 e onde h 0 se escreve 1). a) Se n = 101, mostre que se pode conseguir, mediante uma sucesso de operaes permitidas, que cada um dos n pontos tenha escrito 0. b) Se n = 102, mostre que impossvel obter todos 0.
PROBLEMA 5

Determine para quais nmeros naturais n possvel cobrir completamente um tabuleiro de n n dividido em casas de 1 1 com peas como a da figura, sem
EUREKA! N35, 2012

Sociedade Brasileira de Matemtica

buracos nem superposies e sem sair do tabuleiro. Cada uma das peas cobre exatamente seis casas. Nota: As peas podem girar.

RESULTADO BRASILEIRO 2011: Nvel 1 (at 13 anos)


Nome Nathan Bonetti Teodoro Jlia Bertelli Lucas Pereira Galvo de Barros Pedro Henrique Sacramento de Oliveira Pedro Henrique da Silva Dias Jiang Zhi Lucki Li Lucca Morais de Arruda Siaudzionis Helena Veronique Rios Cristbal Sciutto Cidade Estado Curitiba - PR Joinville - SC So Paulo - SP Vinhedo - SP Porto Alegre - RS So Paulo - SP So Paulo - SP Fortaleza - CE So Carlos - SP So Paulo - SP Prmio Medalha de Ouro Medalha de Prata Medalha de Prata Medalha de Bronze Medalha de Bronze Medalha de Bronze Medalha de Bronze Meno Honrosa Meno Honrosa Meno Honrosa

2011: Nvel 2 (at 15 anos)


Nome Murilo Corato Zanarella Dimas Macedo de Albuquerque Rodrigo Sanches Angelo Rafael Rodrigues Rocha de Melo Liara Guinsberg Luize Mello D'Urso Viana Lincoln de Queiroz Vieira Tadeu Pires de Matos Belfort Neto Pedro Henrique Alencar Costa Michel Rozenberg Zelazny Cidade Estado Amparo - SP Fortaleza - CE So Paulo - SP Fortaleza - CE So Paulo - SP Rio de Janeiro - RJ Fortaleza - CE Caucaia - CE Fortaleza - CE So Paulo - SP Prmio Medalha de Ouro Medalha de Prata Medalha de Prata Medalha de Bronze Medalha de Bronze Medalha de Bronze Medalha de Bronze Meno Honrosa Meno Honrosa Meno Honrosa

EUREKA! N35, 2012

Sociedade Brasileira de Matemtica

XXII OLIMPADA DE MATEMTICA DO CONE SUL


Enunciados e resultado brasileiro
Os estudantes brasileiros tiveram uma participao destacada na Olimpada de Matemtica do Cone Sul que foi realizada entre os dias 14 e 20 de agosto na cidade de La Paz, Bolvia. A equipe foi liderada pelos professores Francisco Bruno Holanda e Rgis Barbosa Feitosa, ambos de Fortaleza (CE).
RESULTADOS DA EQUIPE BRASILEIRA BRA1 BRA2 BRA3 BRA4 Rafael Kazuhiro Miyazaki Henrique Gasparini Fiuza do Nascimento Rafael Rodrigues Rocha de Melo Vincius Canto Costa Medalha de Prata Medalha de Prata Medalha de Prata Medalha de Prata

PRIMEIRO DIA
PROBLEMA 1

Encontrar todas as ternas de inteiros positivos (x, y, z) tais que

x 2 + y 2 + z 2 = 2011
PROBLEMA 2

Em uma lousa esto escritos os nmeros inteiros positivos de 1 at 4n inclusive. Em cada momento, Pedro apaga dois nmeros da lousa, a e b, e escreve o nmero

ab 2a 2 + 2b 2

. Pedro repete o procedimento at que sobre apenas um nmero.

Demostrar que este nmero ser menor que escolha em cada momento.
PROBLEMA 3

1 , sem importar quais nmeros Pedro n

Seja ABC um tringulo equiltero. P um ponto interior tal que a raiz quadrada da distancia de P at um dos lados seja igual soma das razes quadradas das distancias de P at os outros dois lados. Encontre o lugar geomtrico do ponto P.

EUREKA! N35, 2012

Sociedade Brasileira de Matemtica

SEGUNDO DIA PROBLEMA 4


Dizemos que um nmero de 4 dgitos abcd equilibrado se a + b = c + d. Encontre todos os nmeros equilibrados de quatro dgitos que podem ser expresos como a soma de dois nmeros palndromos.
PROBLEMA 5

Seja ABC um tringulo e D um ponto sobre o lado AC. Se CBD ABD = 60 , BDC = 30 e, alm disso AB BC = BD 2 , encontre as medidas dos ngulos do tringulo ABC.
PROBLEMA 6

Algumas casas de um tabuleiro Q (2n + 1) (2n + 1) so pintadas de preto, de modo que qualquer quadrado 2 2 de Q contenha, no mximo, 2 casas pretas. Achar o nmero mximo de casas pretas que o tabuleiro pode ter.

EUREKA! N35, 2012

Sociedade Brasileira de Matemtica

LII OLIMPADA INTERNACIONAL DE MATEMTICA (IMO)


Enunciados e resultado brasileiro
O Brasil obteve um timo resultado na 52a Olimpada Internacional de Matemtica (IMO), que aconteceu at o dia 24 de julho na cidade de Amsterd na Holanda, conquistando trs medalhas de prata e trs de bronze. A equipe foi liderada pelos professores Nicolau Coro Saldanha do Rio de Janeiro (RJ) e Eduardo Tengan de So Carlos (SP).
RESULTADOS DA EQUIPE BRASILEIRA BRA1 BRA2 BRA3 BRA4 BRA5 BRA6 Joo Lucas Camelo S Andr Macieira Braga Costa Henrique Gasparini Fiza do Nascimento Deborah Barbosa Alves Gustavo Lisba Empinotti Maria Clara Mendes Silva Medalha de Prata Medalha de Prata Medalha de Prata Medalha de Bronze Medalha de Bronze Medalha de Bronze

PRIMEIRO DIA
PROBLEMA 1

Para qualquer conjunto A = {a1 , a2 , a3 , a4 } de quatro inteiros positivos distintos, a


soma a1 + a2 + a3 + a4 denotada por s A . Seja nA o nmero de pares de ndices

( i, j ) ,

com 1 i < j 4, para os quais ai + a j divide s A .

Encontre todos os conjuntos A de quatro inteiros positivos para os quais nA alcana o seu valor mximo.
PROBLEMA 2

Seja S um conjunto finito de dois ou mais pontos do plano. Em S no h trs pontos colineares. Um moinho de vento um processo que comea com uma reta que passa por um nico ponto P S . Roda-se no sentido dos ponteiros do relgio ao redor do pivot P at que a reta encontre pela primeira vez um outro ponto de S, que denotaremos por Q. Com Q como novo pivot, a reta continua sem parar, sendo sempre o pivot algum ponto de S. Demonstre que se pode escolher um ponto P S e uma reta que passa por P tais que o moinho de vento resultante usa cada ponto de S como pivot infinitas vezes.
EUREKA! N35, 2012

Sociedade Brasileira de Matemtica

PROBLEMA 3

Seja f : satisfaz

uma funo real definida no conjunto dos nmeros reais que

f ( x + y ) yf ( x ) + f ( f ( x ) )
para quaisquer nmeros reais x e y. Demonstre que f ( x ) = 0 para todo x 0.

SEGUNDO DIA
PROBLEMA 4

Seja n um inteiro positivo. Temos uma balana de dois pratos e n pesos cujas massas so 20 , 21 ,..., 2n 1. Devemos colocar os pesos na balana, um por um, de tal forma que o prato direito nunca seja mais pesado do que o prato esquerdo. A cada passo, devemos escolher um dos pesos que ainda no estejam na balana e coloclo sobre o prato esquerdo ou sobre o prato direito, procedendo assim at que todos os pesos tenham sido colocados nela. Determine o nmero de maneiras em que isso pode ser feito.

PROBLEMA 5

Seja f :

uma funo do conjunto dos inteiros para o conjunto dos inteiros

positivos. Supomos que para quaisquer inteiros m e n, a diferena f ( m ) f ( n ) divisvel por f ( m n ) . Demonstre que, para todos os inteiros m e n com f ( m ) f ( n ) , o nmero f ( n ) divisvel por f ( m ) .
PROBLEMA 6

Seja ABC um tringulo acutngulo cuja circunferncia circunscrita . Seja uma reta tangente a e sejam a , b e c as retas obtidas ao refletir em relao s retas BC, CA e AB, respectivamente. Demonstre que a circunferncia circunscrita ao tringulo determinado pelas retas a , b e c tangente circunferncia .

EUREKA! N35, 2012

Sociedade Brasileira de Matemtica

LIII OLIMPADA INTERNACIONAL DE MATEMTICA (IMO)


Enunciados e resultado brasileiro
O estudante Rodrigo Sanches ngelo (SP) conquistou medalha de ouro na 53 Olimpada Internacional de Matemtica (IMO). O evento foi realizado entre os dias 4 e 16 de julho na cidade de Mar del Plata, na Argentina reunindo 551 estudantes de 100 pases. Alm da medalha de ouro o Brasil conquistou uma medalha de prata e trs de bronze. Com este resultado o Brasil se classificou em 19 lugar entre os pases participantes. A equipe foi liderada pelos professores Luciano Guimares Castro, do Rio de Janeiro (RJ) e Carlos Yuzo Shine, de So Paulo (SP).
RESULTADOS DA EQUIPE BRASILEIRA BRA1 BRA2 BRA3 BRA4 BRA5 BRA6 Rodrigo Sanches Angelo Joo Lucas Camelo S Franco Matheus de Alencar Severo Henrique Gasparini Fiza do Nascimento Rafael Kazuhiro Miyazaki Maria Clara Mendes Silva Medalha de Ouro Medalha de Prata Medalha de Bronze Medalha de Bronze Medalha de Bronze Meno Honrosa

PRIMEIRO DIA
PROBLEMA 1

Dado um tringulo ABC, o ponto J o centro da circunferncia ex-inscrita oposta ao vrtice A. Esta circunferncia ex-inscrita tangente ao lado BC em M, e s retas AB e AC em K e L, respectivamente. As retas LM e BJ intersectam-se em F, e as retas KM e CJ intersectam-se em G. Seja S o ponto de interseco das retas AF e BC, e seja T o ponto de interseco das retas AG e BC. Prove que M o ponto mdio de ST. (A circunsferncia ex-inscrita de ABC oposta ao vrtice A a circunferncia tangente ao segmento BC, ao prolongamento do segmento AB no sentido de A para B e ao prolongamento do segmento AC no sentido de A para C.)
PROBLEMA 2

Seja n 3 um inteiro e sejam a2 , a3 ,..., an nmeros reais positivos tais que a2 a3 ...an = 1. Prove que
EUREKA! N35, 2012

10

Sociedade Brasileira de Matemtica

(1 + a2 ) (1 + a3 ) ...(1 + an )
PROBLEMA 3

> nn .

O desafio do mentiroso um jogo para dois jogadores A e B. As regras do jogo dependem de dois inteiros positivos k e n conhecidos por ambos os jogadores. No incio do jogo, o jogador A escolhe inteiros x e N com 1 x N . O jogador A mantm x em segredo, e diz a B o verdadeiro valor de N. Em seguida, o jogador B tenta obter informao sobre x fazendo perguntas a A da seguinte maneira: em cada pergunta, B especifica um conjunto arbitrrio S de inteiros positivos (que pode ser um dos especificados em alguma pergunta anterior), e pergunta a A se x pertence a S. O jogador B pode fazer tantas perguntas desse tipo como deseje. Depois de cada pergunta, o jogador A deve responder imediatamente com sim ou no, mas pode mentir tantas vezes como queira. A nica restrio que dadas quaisquer k + 1 respostas consecutivas, pelo menos uma deve ser verdadeira. Quando B tenha feito tantas perguntas como pretenda, deve especificar um conjunto X com no mximo n inteiros positivos. Se x pertencer a X ento ganha B; caso contrrio, B perde. Prove que:
1. 2. Se n 2 k , ento B pode garantir a sua vitria.

Para todo k suficientemente grande, existe um inteiro n 1,99k tal que B no pode garantir a sua vitria.

SEGUNDO DIA
PROBLEMA 4

Determine todas funes f : tais que, para todos os inteiros a, b, c que satisfazem a + b + c = 0, a seguinte igualdade verdadeira:
f ( a ) + f (b ) + f (c ) = 2 f ( a ) f (b ) + 2 f (b ) f ( c ) + 2 f (c ) f ( a ). ( designa o conjunto dos nmeros inteiros.)
2 2 2

PROBLEMA 5

Seja ABC um tringulo tal que BCA = 90 , e seja D o p da altura relativa a C. Seja X um ponto no interior do segmento CD. Seja K o ponto do segmento AX tal
EUREKA! N35, 2012

11

Sociedade Brasileira de Matemtica

que BK = BC. Analogamente, seja L o ponto do segmento BX tal que AL = AC. Seja M o ponto de interseco de AL com BK. Prove que MK = ML.
PROBLEMA 6

Determine todos os inteiros positivos n para os quais existem inteiros no negativos a1 , a2 ,..., an tais que 1 1 1 1 2 n + a2 + ... + an = a1 + a2 + ... + an = 1. a1 2 2 2 3 3 2

EUREKA! N35, 2012

12

Sociedade Brasileira de Matemtica

II OLIMPADA DE MATEMTICA DA COMUNIDADE DOS PASES DE LNGUA PORTUGUESA


Enunciados e resultado brasileiro
O Brasil conquistou duas medalhas de ouro e duas de prata na 2 Olimpada de Matemtica da Comunidade dos Pases de Lngua Portuguesa, realizada de 20 a 28 de julho, na cidade de Salvador (BA). Com este resultado o pas ficou pelo segundo ano consecutivo com a primeira posio na classificao geral, com 160 pontos, seguido pela equipe de Portugal, que obteve 149 pontos. A equipe foi liderada pelos professores Marcelo Mendes de Oliveira, de Fortaleza (CE) e Guilherme Philippe Figueiredo, de So Paulo (SP).
RESULTADOS DA EQUIPE BRASILEIRA BRA1 BRA2 BRA3 BRA4 Daniel Santana Rocha Murilo Corato Zanarella Daniel Lima Braga Victor Oliveira Reis Medalha de Ouro Medalha de Ouro Medalha de Prata Medalha de Prata

PRIMEIRO DIA
PROBLEMA 1

Arnaldo e Bernaldo treinam para uma maratona ao longo de uma pista circular, a qual possui em seu centro um mastro com uma bandeira hasteada. Arnaldo corre mais rpido que Bernaldo, de modo que a cada 30 minutos de corrida, enquanto Arnaldo d 15 voltas na pista, Bernaldo s consegue dar 10 voltas completas. Arnaldo e Bernaldo partiram no mesmo instante da linha e correram com velocidades constantes, ambos no mesmo sentido. Entre o minuto 1 o o minuto 61 da corrida, quantas vezes Arnaldo, Bernaldo e o mastro ficaram colineares?
PROBLEMA 2

Maria dispe de um tabuleiro de tamanho n n , inicialmente com todas as casas pintadas de cor branca. Maria decide pintar algumas casas do tabuleiro de preto, formando um mosaico, como mostra a figura abaixo, da seguinte maneira: ela pinta todas as casas do bordo do tabuleiro de preto, e em seguida deixa pintadas de branco as casas do bordo do tabuleiro que ainda no foi pintado. Ento, pinta novamente de preto as casas do bordo do prximo tabuleiro restante, e assim sucessivamente.
EUREKA! N35, 2012

13

Sociedade Brasileira de Matemtica

a) Determine um valor de n para que o nmero de casas pretas seja igual a 200. b) Determine o menor valor de n para que o nmero de casas pretas seja maior que
2012. PROBLEMA 3

Seja n um inteiro positivo. Abigail e Berenice disputam o seguinte jogo, que utiliza n bolas numeradas de 1 at n. Elas dispem de duas caixas, rotuladas com os smbolos e , respectivamente. Na sua vez, cada jogador escolhe uma bola e a coloca em uma das caixas. Ao final, os nmeros das bolas que esto na caixa so multiplicados, obtendo-se um valor P e os nmeros das bolas que esto na caixa

so somados, obtendo-se um

valor S (se a caixa

estiver vazia,

ento adotamos P = 1; se a caixa

estiver vazia, adotamos

S = 0). Elas jogam alternadamente, iniciando por

Abigail, at que no haja mais bolas fora das caixas. Se o valor de P + S for par, Abigail ganha. Caso contrrio, Berenice ganha. a) Qual jogador possui estratgia vencedora para n = 6? b) Qual jogador possui estratgia vencedora para n = 2012?

SEGUNDO DIA
PROBLEMA 4

Uma formiga decide passear sobre o permetro de um tringulo ABC. A formiga pode comear em qualquer vrtice. Sempre que a formiga est num vertice, ela escolhe um dos vrtices adjacentes e caminha diretamente (em linha reta) at o vrtice escolhido. a) De quantos modos a formiga pode passear visitando cada vrtice exatamente duas vezes?
EUREKA! N35, 2012

14

Sociedade Brasileira de Matemtica

b) De quantos modos a formiga pode passear visitando cada vrice exatamente trs vezes? Observao: Em cada item, considere que o vrtice inicial visitado.
PROBLEMA 5

Arnaldo e Bernaldo esto brincando no quadro da sala de aula da seguinte maneira: eles escrevem inicialmente no quadro um nmero inteiro positivo n. Ento, alternadamente, comeando com Arnaldo, apagam o nmero que est no quadro e escrevem um novo nmero que pode ser:

o que acabou de ser apagado menos a maior potncia de 2 (com expoente inteiro no-negativo) menor do que ou igual ao nmero apagado; o que acabou de ser apagado dividido por 2, caso o nmero apagado seja par.
Vence a brincadeira quem obtiver primeiro o nmero zero. a) Determine qual dos jogadores possui uma estratgia vencedora para n = 40 e descreva-a. b) Determine qual dos jogadores possui uma estratgia vencedora para n = 2012 e descreva-a.
PROBLEMA 6

Um quadriltero ABCD est inscrito numa circunferncia de centro O. Sabe-se que as diagonais AC e BD so perpendiculares. Sobre cada um dos lados construmos semicrculos, externamente, como mostra a figura. a) Mostre que os tringulos AOB e COD tm a mesma rea. b) Se AC = 8 cm e BD = 6 cm, determine a rea da regio sombreada.

EUREKA! N35, 2012

15

Sociedade Brasileira de Matemtica

XXVI OLIMPADA IBEROAMERICANA DE MATEMTICA


Enunciados e resultado brasileiro
A equipe brasileira formada por quatro estudantes do ensino mdio conquistou trs medalhas de prata e uma de bronze na 26 Olimpada Iberoamericana de Matemtica (OIM), realizada entre os dias 23 e 30 de setembro na cidade de San Jos, Costa Rica. A equipe foi liderada pelos professores Onofre Campos, de Fortaleza (CE) e Carlos Moreira, do Rio de Janeiro (RJ).
RESULTADOS DA EQUIPE BRASILEIRA BRA1 BRA2 BRA3 BRA4 Henrique Gasparini Fiza do Nascimento Maria Clara Mendes Silva Joo Lucas Camelo S Andr Macieira Braga Costa Medalha de Prata Medalha de Prata Medalha de Prata Medalha de Bronze

PRIMEIRO DIA
PROBLEMA 1

No quadro est escrito o nmero 2. Ana e Bruno jogam alternadamente, comeando por Ana, da seguinte maneira: cada um na sua vez substitui o nmero escrito pelo que se obtm multiplicando-o por 2, ou por 3, ou somando-lhe 1. O primeiro que obtenha um resultado maior ou igual a 2011 ganha. Mostre que um dos dois tem uma estratgia vencedora e descreva-a.
PROBLEMA 2

Encontrar todos os inteiros positivos n para os quais existem trs nmeros inteiros no nulos x, y, z tais que 1 1 1 1 x+ y+z =0 e + + = . x y z n

PROBLEMA 3

Seja ABC um tringulo acutngulo e X, Y, Z os pontos da tangncia de sua circunferncia inscrita com os lados BC, CA, AB, respectivamente. Sejam C1 , C2 , C3 circunferncias com cordas YZ, ZX, XY, respectivamente, de maneira que C1 e C2 se intersectem sobre a reta CZ e que C1 e C3 se intersectem sobre a reta BY. Suponha que C1 intersecta XY em J e intersecta ZX em M; que C2 intersecta YZ
EUREKA! N35, 2012

16

Sociedade Brasileira de Matemtica

em L e intersecta XY em I; e que C3 intersecta YZ em K e intersecta ZX em N. Demonstrar que I, J, K, L, M, N esto sobre uma mesma circunferncia.

SEGUNDO DIA
PROBLEMA 4

Seja ABC um tringulo acutngulo e O o seu circuncentro. Sejam P e Q pontos tais que BOAP e COPQ so paralelogramos. Demonstrar que Q o ortocentro de ABC.

PROBLEMA 5

Sejam x1 ,..., xn nmeros reais positivos. Demonstrar que existem a1 ,..., an {1,1} tais que
2 a1 x12 + ... + an xn ( a1 x1 + ... + an xn ) . 2

PROBLEMA 6

Sejam k 2 e n inteiros positivos. Temos kn caixas em linha reta e em cada caixa coloca-se uma pedra de uma de k cores diferentes de tal forma que haja n pedras de cada cor. Uma troca consiste em trocar de caixa duas pedras que se encontrem em caixas adjacentes. Encontrar o menor inteiro positivo m para o qual sempre possvel conseguir mediante m trocas que as n pedras de cada cor fiquem em caixas consecutivas se: a) n par. b) n mpar e k = 3.

EUREKA! N35, 2012

17

Sociedade Brasileira de Matemtica

PARIDADE DO NMERO BINOMIAL MDIO


J.C.S. de Miranda, USP/SP Iesus C. Diniz, UFRN
Nvel Avanado

1. INTRODUO

Apresentaremos duas solues ao problema da paridade do nmero binomial 2n mdio, isto , mostrar que 2 para todo n * . Uma delas baseada em um n argumento combinatrio, enquanto que a outra ser obtida de maneira aritmtica. Outras solues deste problema so dadas em [6] e [8]. Ademais, como consequncia dos mtodos de soluo dados nas sees 2 e 3, respectivamente, resolveremos os problemas olmpicos internacionais dados em [5] e [4]. O nmero binomial mdio aparece em vrias reas da matemtica, por exemplo: teoria dos nmeros [1], anlise [2] e combinatria [7]. A segunda soluo mostra quo interessantes podem ser as resolues de determinados problemas quando utilizamos argumentos combinatrios para estabelec-las. A primeira obtida a partir de uma conveniente fatorao do nmero binomial mdio observando-se o expoente do fator 2. Por fim, para exemplificar o uso de argumentos combinatrios na soluo de problemas apresentamos uma srie de exerccios na seo 5; ver tambm em [3] e [8]. Alm disso, calculamos de maneira aproximada a srie cujos termos so os recprocos dos nmeros binomiais mdios.
2. SOLUO ARITMTICA

Indicaremos por f ( n ) , para todo n

, o expoente do fator 2 na decomposio

2n de em fatores primos, isto , f ( n ) tal que: n

2n 2n 2 f ( n ) e 2 f ( n ) +1 | (1) n n
Note que
EUREKA! N35, 2012

18

Sociedade Brasileira de Matemtica

2n 2 n n = ( 2i 1) . (2) n n! i =1 Pois,

( 2n )( 2n 2 ) ...2 2n ( 2n )( 2n 1)( 2n 2 ) ...2.1 ( 2n 1)( 2n 3) ...1 = = = n !n ! n !n ! n n 2 ( n ) 2 ( n 1) ...2 (1) ( 2n 1)( 2n 3) ...1 = 2 n ! ( 2n 1)( 2n 3) ...1 = n !n ! n !n ! 2n 2n n ( 2n 1)( 2n 3) ...1 = ( 2i 1) . n! n! i =1
Seja k
Assim, o mximo expoente de 2 tal que 2k n, isto , 2k n < 2k +1.

k log 2 n < k + 1, isto , k = log 2 n .


n Note que em n! h k = j fatores que so mltiplos de 2 j , para todo 2 j {1,..., k } . Logo o expoente do fator 2 em n! dado por,
n n k n 2 n + ... + = = 2 2k 2 j 2 j . (3) j =1 j =1 Portanto, da definio dada em (1) e das equaes (2) e (3) temos: log 2 n n f ( n ) = n j (4) j =1 2 e podemos escrever 2n f (n) = 2 c, em que 2 | c. (5) n

log n

Agora,
log 2 n

j =1

2 n n n < = n 2j 2j j n 1, j =1 j =1 2

log n

donde, da equao (4), resulta que f ( n ) n ( n 1) = 1.


2n Logo, da equao (5), 2 pois f ( n ) 1. n
EUREKA! N35, 2012

19

Sociedade Brasileira de Matemtica

3. SOLUO POR ARGUMENTO COMBINATRIO

Teorema 1 Para todo n Prova: Para todo n


*

2n ,2 . n

, o nmero de parties no-ordenadas de um conjunto de 2n 2n n n n elementos em dois subconjuntos de n elementos cada, dado por . 2! Claramente, o referido nmero de parties um inteiro positivo, donde segue que
2n n n n 2!
4. COMENTRIOS

2n , e portanto, 2 . n

A soluo por argumento combinatrio claramente mais objetiva; por outro lado, da soluo aritmtica podem ser obtidos os seguintes resultados: O expoente da maior potncia de um primo p que divide n!
log p n

j =1

n n p j = p j , (6) j =1

pois o argumento do qual se obtm a equao (3) pode ser repetido para um primo p qualquer. Este resultado foi provado por Legendre em 1808. O expoente r da maior potncia, de um nmero no nulo m = pii que divide n!,
i =1 k

isto , r tal que m r n ! e mr +1 / n !, o maior valor de r que satisfaz s desigualdades r i ri para todo i {1,..., k} , nas quais ri o expoente da potncia mxima de pi em n!. Assim,
r n r = min i = min j / i . (7) 1 i k 1 i k j =1 pi i

EUREKA! N35, 2012

20

Sociedade Brasileira de Matemtica

Na prova da paridade do nmero binomial mdio dada em [8], exerccio 39 pgina 235, usado um argumento combinatrio juntamente com propriedades aritmticas da fatorao do nmero binomial mdio. Ei-la: 2n ( 2n )! 2n ( 2n 1)! ( 2n 1)! = 2 2n 1 par. = =2 = n ( n 1)!n! n n!n! n ( n 1)!n!
5. EXERCCIOS

Prove via argumentos combinatrios que: n + m n m n m n m a) = + + ... + r 0 r 1 r 1 r 0


Soluo: Consideremos um grupo de n + m pessoas em que h n homens e m

mulheres. n + m O nmero de subconjuntos de tamanho r do grupo das n + m pessoas . r Por outro lado, particionando os subconjuntos em relao quantidade de homens n m e mulheres neles presentes, temos um total de deles em que no h 0 r n m nenhum homem presente e r mulheres, em que h 1 homem presente e 1 r 1 n m r 1 mulheres ... em que h r homens presentes e nenhuma mulher. r 0
n n b) = p n p

2n n n c) = (Frmula de Lagrange) n k =0 k
Soluo: Do item a com m e r iguais a n e do uso da identidade dada em b, segue-se o resultado, pois:

EUREKA! N35, 2012

21

Sociedade Brasileira de Matemtica

n + n n n n n n n = + + ... + n 0 n 1 n 1 n 0 n n n n n n n n = + + ... + = 0 0 1 1 n n k =0 k
n n n 1 d) k = ( n k + 1) = n k k 1 k 1
Soluo: Considere um grupo de n pessoas pertencentes a uma empresa, dentre as quais k constituem uma diretoria em que h um nico diretor-presidente e todos os demais membros tm iguais poderes. Podemos configurar a diretoria das trs seguintes formas, em que o total de possibilidades de cada uma delas representa, respectivamente, cada um dos termos das igualdades dadas no item d.
2

1. Escolher a diretoria e depois o diretor-presidente; 2. Escolher os k 1 membros de iguais poderes pertencentes a diretoria e depois o diretor-presidente. 3. Escolher o diretor-presidente entre as n pessoas pertencentes a empresa e depois os k 1 indivduos que completaro a diretoria. n n i 1 e) = (Identidade Combinatria de Fermat) k i = k k 1
Soluo: O lado esquerdo da igualdade o nmero de amostras no ordenadas de

tamanho k, extradas sem reposio de uma populao de tamanho n, que sem perda de generalidade suporemos numerada de 1 a n. Por outro lado, se considerarmos a partio de todas essas amostras com relao presena de seu k 1 k n 1 maior elemento, temos que existem , ,..., de tais amostras k 1 k 1 k 1 cujo maior elemento respectivamente k, k + 1,...,n. f)

k k = n2
k =1

n 1

EUREKA! N35, 2012

22

Sociedade Brasileira de Matemtica

Soluo: Considere uma empresa formada por n funcionrios dentre os quais ser

formada uma diretoria de no mximo n pessoas, a qual constituda de um nico diretor-presidente escolhido entre os membros pertencentes diretoria. O lado esquerdo da igualdade representa o total de configuraes das diretorias possveis, obtidas primeiramente a partir da determinao dos k membros pertencentes diretoria, seguindo-se da determinao de seu diretor-presidente entre os k membros da diretoria. O lado direito da igualdade representa o total de configuraes das diretorias possveis, obtidas primeiramente escolhendo-se o diretor-presidente e em seguida determinando-se para cada um dos demais n 1 membros se pertencero ou no diretoria. n n 2 = 2 n ( n + 1) k k =1 n n h) k 3 = 2n 3 n 2 ( n + 3) k =1 k g)

i)

j i = i 2
j =i

n j n

n i

j) (British Mathematical Olympiad [4]) Para todo ( m, n )

( m!) ( n!)

n +1

m +1

{( mn )!}

Soluo: Sejam P 1 e P 2 , respectivamente, o nmero de maneiras de dividir mn

pessoas em n grupos de m pessoas e m grupos de n pessoas. Tem-se que: P 1 =

( mn )! + , P = ( mn )! + e, portanto, 2 n m ( m!) n! ( n!) m! 2 ( mn )!} { 2 n +1 m +1 P + , isto , ( m!) ( n !) {( mn )!} 1 P 2 = n +1 m +1 ( m!) ( n!)

k) (XIV Olimpada Internacional de Matemtica [5]) Quaisquer que sejam os inteiros naturais m e n, temos: ( 2m )!( 2n )! . m!n!( m + n )!

EUREKA! N35, 2012

23

Sociedade Brasileira de Matemtica

Soluo: Seja p um primo arbitrrio. A maior potncia de p que divide o numerador da frao acima p a onde
2m 2n a = i + i i =1 p p ao passo que a maior potncia de p que divide seu denominador p b com m n m + n b = i + i + i . i =1 p p p

Basta mostrar que a b. Para qualquer k inteiro maior ou igual a 1 podemos escrever m = m1k + r e n = n1k + s em que 0 r k 1,0 s k 1, m1 e n1 so inteiros. Assim, 2m 2 n 2r 2 s k + k = 2m1 + 2n1 + k + k (8) e m n m + n r s r + s k + k + k = m1 + n1 + ( m1 + n1 ) + k + k + k (9) r + s = 2m1 + 2n1 + . k Como r + s 2 max {r , s} 2r 2s + , k k k k por (8) e (9) temos 2m 2 n m n m + n k + k k + k + k para todo k 1 e, em particualr, para k da forma imediatamente que a b.

k = pi , donde segue

l) Dada a srie S =

1 : k =1 2k k l.1) mostre que a srie convergente, isto , S < ;

EUREKA! N35, 2012

24

Sociedade Brasileira de Matemtica

Soluo: A partir da Frmula de Lagrange, apresentada no item c) desta seo, tem-

se que 2n n n n 2 = > = n , (10) n j 1 j =0 e portanto, segue de (10) que


2 2

n Ou ainda, da equao (2) tem-se que, para n > 1,

2n < n
n =1 n =1

1
2

2
6

2n 1 1 1 1 n > 2 , donde segue que 2n < n , e se n = 1, resulta que 2n = n . 2 2 n n n Portanto 1 1 < = 1. n n =1 2n n =1 2 n l.2) prove tambm que 2 8 <S< ; 3 9 Soluo: A partir da equao (2) tem-se que n 2n ( 2i 1) n i =1 = n , logo, para n > 1, n! 2 2n 1.4.6... 2n n 1 1.2.4... ( 2n 2 ) n ( ) n 1 (11) 2 = < n< =2 . n 1.2.3...n 2 1.2.3...n De (11) resulta que 1 2 e portanto,
2 n 1

<

1 n < 2 n 1 , 2n 2 n

EUREKA! N35, 2012

25

Sociedade Brasileira de Matemtica


1 n < . (12) 2 n 1 2 n 1 n =1 n =1 2n n =1 2 n Os limitantes so obtidos a partir do clculo das somas dadas em (12).

<

2
n =1

1
2 n 1

= 2

1 2 1 4 = 2 = e 2n 1 n =1 2 1 4 3

2
n =1

n
2 n 1

= 2

n 1 1 1 1 1 1 = 2 2 + 4 + 4 + 6 + 6 + 6 + ... = 2n 2 2 2 2 n =1 2 2 2

1 1 1 1 1 1 1 2 2 + 4 + 6 + ... + 4 + 6 + ... + 6 + 8 + ... = 2 2 2 2 2 2 2


1 1 12 8 1 4 6 1 1 2 2 1 + 2 1 + 2 1 + ... = 2 + 4 + 6 + ... = 1 1 1 3 2 2 2 22 22 22 1 8 22 8 = . 1 3 1 22 9

l.3) Finalmente, prove que 0,73639 < S < 0,73641.


Soluo: Das desigualdades estabelecidas em (12), resulta que k
*

2n +
n =1

1 2
2 n 1

<

n = k +1

k 1 1 n < + 2 n 1 . (13) n =1 2n n =1 2n n = k +1 2 n n

Analogamente ao que foi feito no item 12, temos que:

n = k +1

1 2
2 n 1

1 2
2 k +1

1 2
2 k +3

1 1 + ... = 2 k 1 (14) 3 2

e tambm
n k 4 1 = + 2 k 1 . (15) 2 n 1 2 3 9 2 n = k +1

Pois,

EUREKA! N35, 2012

26

Sociedade Brasileira de Matemtica

n = k +1

n 2
2 n 1

k +1 k + 2 k + 3 + + ... = 22 k +1 22 k + 3 22 k + 5

( k + 1)

1
2 k +1

1 1 1 1 + ... + 2 k + 3 + 2 k + 5 + ... + 2 k + 5 + 2 k + 7 + ... + ... = 2 2 2 2 2 1 1 1 2 k +1 2 k +3 2 k +5 ( k + 1) 2 1 + 2 1 + 2 1 + ... = 1 1 1 4 4 4


2 k +3

4 1 4 1 1 + 2 k +3 + 2 k +5 + ... = ( k + 1) 2 k +1 3 2 2 3 2 1 1 1 4 22 k + 3 ( k + 1) 2 k 1 + 1 = 3 2 3 1 4 1 1 1 1 + 2 k 1 = ( k + 1) 2 k 1 3 2 9 2 k 4 1 + 2 k 1 . 3 9 2

De (14) e (15) resulta que k k 1 1 1 1 1 k 4 1 + 2 k 1 < < + + 2 k 1 . (16) 2 n 2 n 2 n 3 2 n =1 3 9 2 n =1 n =1 n n n Para estabelecermos o resultado no item l.3, suficiente tomarmos k = 10 em (16).
Observao: A srie

n k 1 3k + 7 1 3k + 1 1 com um erro de no mximo + 2 k . 2k 9 2 9 2 n =1 2n n Assim, se quisermos um erro no superior a e em nossa estimativa da srie, 3m + 1 1 suficiente escolhermos k = min m : 2 m . 9 2
EUREKA! N35, 2012

2n
n =1

pode ser estimada por

27

Sociedade Brasileira de Matemtica

Terminamos com um problema proposto.

Problema (Harvey Abbott e Murray Klamkin)


Sabe-se que ( 3m )!( 3n )!

( 4m )!( 4n )! m!n!( m + n )!( n + m )! m!n!( 2m + n )!( 2n + m )!


,

( 5m )!( 5n )! m!n!( 3m + n )!( 3n + m )! ( 6m )!( 6n )! m!n!( 4m + n )!( 4n + m )!

so todos inteiros para quaisquer inteiros positivos m, n. a) Encontre inteiros positivos m, n tais que I ( m, n ) = seja inteiro. b) Seja A o conjunto dos pares (m, n) com m n, para os quais I ( m, n ) no inteiro, e seja A( x) o nmero de pares em A para os quais 1 m n x. Prove que A contm uma proporo positiva dos pares de naturais, no sentido de que existem uma constante c > 0 e um inteiro positivo n0 tal que, para todo no

x n0 , A ( x ) c x 2 .

Obs.: Para o item b) pode-se usar o chamado Postulado de Bertrand, um teorema segundo o qual, para todo n 2, existe um primo p com n < p < 2n.
Referncias
[1] Berend, D.; Harmse, J.E. On some arithmetical properties of middle binomial coefficients. Acta arith. 84 (1998), no. 1, 31-41. [2] Bloom, D. M.; Swanson, C.N; Schilling, K. A. Convolution of Middle Binomial Coefficients; 10921 The American Mathematical Monthly Vol 110 (Dec., 2003), no. 10, 958-959. [3] Ross, S. M. A first course in probability.8th. ed. Upper Saddle River, N.J.: Prentice Hall, 2008. [4] British Mathematical Olympiad -16/02/2005 Exercice 4 item bhttp://www.bmoc.maths.org/home/bmolot.pdf [5] XIV Olimpada Internacional de Matemtica. Cracvia, Polnia, 1972 Exercice 3 http://imo.math.ca/Exams/1972imo.html [6] Gardiner, A. Four Problems on Prime Power Divisibility. The American Mathematical Monthly Vol 95 (Dec., 1998), no. 10, 926-931. [7] Knuth D. E.; Vardi I.; Richberg R. The asymptotic expansion of the middle binomial coefficient. The Amer. Math. Monthly Vol 97 (1990), 626-630. [8] Morgado, A. C. O.; Carvalho, J. B. P.; Carvalho, P. C. P.; Fernandez, P. Anlise combinatria e probabilidade. 9. ed. Rio de Janeiro: Sociedade Brasileira de Matemtica, 2006. (Coleo do Professor de Matemtica).
EUREKA! N35, 2012

28

Sociedade Brasileira de Matemtica

PONTO MDIO LEMBRA? OUTRO PONTO MDIO! DOIS PONTOS MDIOS LEMBRAM? BASE MDIA!
Ccero Thiago Magalhes

Nvel Iniciante
Propriedade 1 Num tringulo retngulo ABC, a mediana BM relativa hipotenusa mede metade da hipotenusa AC.
A D

Prova

Seja D o ponto sobre o prolongamento da mediana BM tal que BM = MD. Os tringulos AMB e CMD so congruentes, pelo caso LAL. Da, AB = CD e BAM = DCM , ou seja, AB e CD so segmentos iguais e paralelos e portanto
ABC = DCB = 90.

Assim, os tringulos ABC e DCB so congruentes, pelo caso LAL, e portanto AC BD = AC 2 BM = AC BM = . 2 Definio 1 Uma base mdia de um tringulo um segmento que une os pontos mdios de dois de seus lados.

Assim, todo tringulo possui exatamente trs bases mdias.


Propriedade 2 Sejam ABC um tringulo e M, N os pontos mdios dos lados AB, AC,

respectivamente. Ento MN BC e MN = BC . 2

EUREKA! N35, 2012

29

Sociedade Brasileira de Matemtica


A

Prova

Inicialmente, prolonguemos a base mdia MN at um ponto P tal que MN = NP. Em seguida, construmos o tringulo CNP. Note que os tringulos ANM e CNP so congruentes, pelo caso LAL. Da, CP = AM e MAN = PCN e portanto CP AM CP BM . Assim, MBCP um paralelogramo, pois CP e BM so segmentos paralelos e iguais. Mas ento MP BC e BC . 2 Definio 2 A base mdia de um trapzio o segmento que une os pontos mdios de seus lados no paralelos. MP = BC 2 MN = BC MN =
Propriedade 3 Sejam ABCD um trapzio de bases AB e CD, e sejam M e N os pontos mdios dos lados BC e AD, respectivamente. Ento,

MN AB, MN CD e MN =

AB + CD . 2

Prova

Inicialmente, prolonguemos AM at encontrar DC no ponto E. fcil ver que ABM CME ( ALA) AM = ME. Portanto, MN base mdia do tringulo ADE. Assim,
EUREKA! N35, 2012

30

Sociedade Brasileira de Matemtica

MN DE MN DC , e MN = Finalmente, MN =
Problema 1 (OBM)

DE . 2

DC + CE DC + AB = . 2 2

Considere um tringulo acutngulo ABC com BAC = 30. Sejam B1 , C1 os ps das alturas relativas aos lados AC, AB, respectivamente, e B2 , C2 os pontos mdios dos lados AC, AB, respectivamente. Mostre que os segmentos B1C2 e B2 C1 so perpendiculares.

Soluo

Seja O a interseo entre B1C2 e B2 C1 . O segmento B1C2 uma mediana do tringulo retngulo AB1 B e portanto
AC2 = B1C2 e C2 B1 A = BAB1 = 30. Analogamente, AC1 B2 = 30. Da, BC2 B1 = C2 B1 A + BAB1 = 60

e portanto C1OC2 = 180 BC2 B1 AC1 B2 = 90.


Problema 2 Sejam ABC um tringulo e M o ponto mdio do lado BC. Se D, E so os

ps das alturas relativas aos lados AC, AB, respectivamente, prove que ME = MD.

EUREKA! N35, 2012

31

Sociedade Brasileira de Matemtica

Soluo

Note que ME mediana relativa hipotenusa do tringulo BEC. Da,


ME = BM = CM

e, analogamente,
MD = BM = CM .

Assim, ME = MD.
Comentrios M o centro da circunferncia circunscrita ao quadriltero inscritvel

BCDE.
Problema 3 Dado um quadriltero ABCD, prove que os pontos mdios M, N, P, Q dos lados AB, BC, CD, DA formam um paralelogramo. Soluo

Temos Tringulo ABC : MN AC e MN = AC 2. Tringulo DAC : PQ AC e PQ = AC 2. Assim, MN PQ e MN = PQ, isto , MNPQ paralelogramo.
Problema 4 Sejam ABC um tringulo e M o ponto mdio de BC. Se AM = BM = CM, prove que BAC = 90.
EUREKA! N35, 2012

32

Sociedade Brasileira de Matemtica

Problema 5 (Torneio das Cidades) Sejam ABCD um paralelogramo, M o ponto

mdio de CD e H o p da perpendicular baixada de B a AM. Prove que BCH um tringulo issceles.


Problema 6 Em um tringulo ABC, retngulo em A e issceles, sejam D um ponto no

lado AC ( A D C ) e E o ponto no prolongamento de BA tal que o tringulo ADE issceles. Se P o ponto mdio de BD, R o ponto mdio de CE e Q a interseco entre ED e BC, prove que o quadriltero ARQP um quadrado.
Problema 7 No tringulo acutngulo ABC, CF altura e BM mediana. Sabendo que

BM = CF e MBC = FCA , prove que o tringulo ABC equiltero.


Problema 8 Seja ABCD um quadriltero convexo tal que ABC = CDA = 90 e

BCD > BAD. Prove que AC > BD.


Problema 9 Seja ABC um tringulo acutngulo tal que B = 2C , AD

perpendicular a BC, com D sobre BC, e E o ponto mdio de BC. Prove que AB = 2 DE.
Problema 10 Seja ABC um tringulo e D um ponto sobre o lado AC tal que

AB = CD. Sejam E e F os pontos mdios de AD e BC, respectivamente. Se a reta BA intersecta a reta FE em M, prove que AM = ME.
Problema 11 Uma reta r passa pelo baricentro de um tringulo ABC. As projees de

A, B e C sobre a reta r so M, N e P, respectivamente. Prove que AM = BN + CP.


Problema 12 (OBM) Seja ABCD um quadriltero convexo, onde N o ponto mdio de DC, M o ponto mdio de BC e O a interseo entre as diagonais AC e BD. Mostre que O o baricentro do tringulo AMN se, e somente se, ABCD um paralelogramo. Problema 13 (China) Seja ABCD um trapzio, AB // BC , B = 30, B = 60, E, M, F, N os pontos mdios de AB, BC, CD, DA respectivamente. Se BC = 7, MN = 3, determine a medida de EF. Problema 14 (China) Seja ABCD um trapzio, AB // CD, DAB = ADC = 90, e o

3 tringulo ABC equiltero. Se a base mdia do trapzio EF = a, determine o 4 comprimento da menor base AB, em funo de a.
EUREKA! N35, 2012

33

Sociedade Brasileira de Matemtica

Problema 15 (Moscou) Seja ABCD um quadriltero convexo, O um ponto em seu

interior tal que AOB = COD = 120, AO = OB, CO = OD. Sejam K, L, M os pontos mdios de AB, BC, CD respectivamente, prove que KLM equiltero.
Problema 16 (China) Seja ABCD um quadriltero tal que AD // BC. Se a bissetriz do

ngulo DAB intersecta CD em E, e BE bissecta o ngulo ABC , prove que AB = AD + BC.


Problema 17 (China) Seja ABCD um quadriltero tal que AD > BC . Sejam E e F os

pontos mdios de AB e CD respectivamente. Se as retas AD e BC intersectam FE em H e G respectivamente, prove que AHE < BGE.
Problema 18 Seja ABC um tringulo e sejam D e E pontos sobre os lados AB e AC ,

respectivamente, tais que AD = DB , AE = 2 EC e BE intersecta CD em F. Prove que 4 EF = BE.


Problema 19 (OBM) Num quadriltero convexo, a reta que passa pelos pontos mdios de dois lados opostos forma ngulos iguais com ambas as diagonais. Mostre que as duas diagonais tm o mesmo comprimento. Problema 20 Se um segmento paralelo a um lado de um tringulo tem uma

extremidade no ponto mdio de um lado e a outra extremidade no terceiro lado, prove que esta extremidade ponto mdio do terceiro lado.
Problema 21 (OBM) No tringulo ABC, D ponto mdio de AB e E um ponto sobre o lado BC tal que BE = 2 EC . Sabendo que ADC = BAE , calcule o valor de BAC . Problema 22 (Austrlia) Sejam ABC um tringulo e P um ponto em seu interior de modo que PAC = PBC . Se L, M so os ps das perpendiculares por P aos lados BC, AC, respectivamente, e D o ponto mdio de AB, prove que DL = DM. Problema 23 (Romnia) Sejam ABC um tringulo issceles com AB = AC, D o ponto

mdio de BC, M o ponto mdio de AD e N a projeo de D sobre BM. Prove que ANC = 90.
Problema 24 (Eslovnia) Seja ABCD um trapzio, com AB paralelo a CD. Sabendo que a distncia entre os pontos mdios das bases igual distncia entre os pontos mdios das diagonais, prove que DAC e DBC so ngulos obtusos.

EUREKA! N35, 2012

34

Sociedade Brasileira de Matemtica

Problema 25 Em um tringulo issceles ABC, com AB = BC, sejam K, L pontos sobre AB, BC, respectivamente, tais que AK + LC = KL. A reta paralela a BC passando pelo ponto mdio M de KL intersecta AC em N. Ache a medida de KNL. Problema 26 Sejam ABC um tringulo e D, E, F os pontos mdios de BC, CA, AB, respectivamente. Prove que DAC = ABE AFC = ADB. Problema 27 Seja ABCD um trapzio com bases AB = a e CD = b. Sejam tambm

M, N os pontos mdios dos lados AB, CD, respectivamente. Sabendo que DAB + ABC = 90, determine o comprimento de MN.
Problema 28 (OBM) Sejam ABCD um quadriltero convexo, N o ponto mdio de

DC, M o ponto mdio de BC e O a interseo entre as diagonais AC e BD. Mostre que O o baricentro do tringulo AMN se e somente se ABCD um paralelogramo.
Problema 29 (Cone Sul) Seja ABC um tringulo acutngulo e sejam NA, BM e CP as alturas relativas aos lados BC, CA e AB, respectivamente. Sejam R, S as projees de N sobre os lados AB, CA, respectivamente, e Q, W as projees de N sobre as alturas BM, CP, respectivamente.

a) Mostre que R, Q, W e S so colineares. b) Mostre que MP = RS QW.


Problema 30 (TST Brasil) Sejam Q o ponto mdio de lado AB de um quadriltero inscritvel ABCD e S a interseo das diagonais AC e BD. Sejam P, R as projees ortogonais de S sobre AD, BC, respectivamente. Prove que PQ = QR.

Bibliografia
[1]. Lecture Notes on Mathematical Olympiad Courses, for section vol. 1. Xu Jiagu, World Scientific. [2]. Problems and solutions in euclidean geometry. M.N. Aref e William Wernick, Dover. [3]. Challenging problems in geometry. Alfred Pasamentier e Charles Salkind, Dover.

EUREKA! N35, 2012

35

Sociedade Brasileira de Matemtica

CAMPEONATOS
Jos Armando Barbosa Filho
Nvel Iniciante
H uma grande variedade de problemas de olimpadas que envolvem campeonatos. A principio, para simplificar o problema, vamos analisar casos onde cada partida acontece entre somente 2 entidades (pessoas, times,...). Por exemplo, h situaes em que: 1. O vencedor ganha 3 pontos e, em caso de empate, cada time ganha 1 ponto. 2. O vencedor ganha 1 ponto e, em caso de empate, cada time ganha 0,5 ponto. 3. No h empates. Nesse caso, o vencedor pode ganhar x pontos, onde x uma varivel a ser definida pelo problema. Em cada caso, expresses diferentes podem ser obtidas. Por exemplo, no caso 2, pode-se deduzir que, em qualquer momento: Nmero de jogos = soma dos pontos de todos os jogadores. No entanto, essa relao no vlida para o caso 1. Porm, nem tudo to especfico. Existem concluses que podem ser facilmente deduzidas. Por exemplo, em qualquer instante: Nmero de vitrias = nmero de derrotas (pois, sempre que um time ganha, um time perde) (1) Nmero de empates sempre par (afinal cada empate contado duas vezes: uma para cada time envolvido) (2) Nmero de vitrias + nmero de empates + nmeros de derrotas = 2 (nmero de jogos) (pois para cada partida em que houve vencedor, o nmero de vitrias e derrotas aumenta um e para cada partida que terminou empate, o nmero de empates aumenta dois) (3) Em situaes que envolvam futebol, de forma anloga, podemos concluir as mesmas equaes acima para nmeros de gols, sendo que em vez de nmeros de vitrias seriam nmeros de gols a favor e, em vez de nmeros de derrotas, seriam nmeros de gols contra. No caso da contagem de gols, falar em empates fica sem sentido. Portanto, poderamos ter as equaes abaixo:

EUREKA! N35, 2012

36

Sociedade Brasileira de Matemtica

Nmero de gols a favor = nmero de gols contra Nmero de gols a favor + nmero de gols contra = 2 (nmero total de gols)

(4) (5)

Muito mais pode ser comentado sobre o assunto. No entanto, a abordagem por meio de exemplos pode ser bem mais interessante. A primeira questo foi adaptada para adequar-se melhor realidade.
Problema 1: (Rioplatense Nvel A - 2003)

Os cinco melhores times de futebol da Amrica participaram de um torneio, onde todos jogam contra todos, uma nica vez. De forma usual, em caso de vitria, o time vencedor ganha 3 pontos, em caso de empate, cada time ganha 1 ponto e, em caso de derrota, o time perdedor ganha 0 pontos. No final do torneio, faz-se uma tabela que mostra o total de pontos obtidos (Ptos), o nmero de vitrias (V), a quantidade de gols a favor (GF) e a quantidade de gols contra (GC). Porm, faltam alguns dados: Time Cear Palmeiras Boca Juniors Amrica Alianza Lima Ptos V 10 8 5 3 ? 3 2 1 1 ? GF GC 4 7 2 1 0 0 1 ? 4 6

a) Indique o total de pontos, o nmero de vitrias do time Alianza Lima e a quantidade de gols recebidos pelo Boca Juniors. Justifique. b) Determine o resultado de cada partida indicando o nmero de gols de cada equipe. Justifique.
Soluo:

a) Para saber o nmero de pontos do time Alianza Lima, precisamos saber quantas vitrias e quantos empates ele teve. Desse modo, o nosso ponto de partida ser encontrar quantas vitrias e quantos empates ele teve. Se soubermos quantos jogos aconteceram, poderemos aplicar as equaes (1), (2) e
EUREKA! N35, 2012

37

Sociedade Brasileira de Matemtica

(3) do comeo. Ento, tentemos, inicialmente, descobrir quantos jogos aconteceram. Sendo 5 times, jogando todos contra todos, uma nica vez, teremos que cada time joga 4 vezes, pois um time no joga contra si mesmo. No entanto, nessa contagem, cada jogo contado duas vezes: uma para cada time. Por isso, temos que dividir por 2. Logo, a quantidade de jogos igual a: nmero de jogos = 5 4 = 10. 2

Agora, busquemos tentar descobrir quantos empates os outros times tiveram. Como cada vitria vale 3 pontos e cada empate vale 1 ponto, descobrir a quantidade de empates fica fcil. Veja por que: 3 (vitrias) + empates = pontos Empates = pontos 3 (vitrias) Da, aplicando a equao (6) para os times, exceto o Alianza Lima, temos que: Cear: 10 3 3 = 1 empate Palmeiras: 8 3 2 = 2 empates Boca Juniors: 5 3 1 = 2 empates America = 3 3 1 = 0 empates. Os times, exceto o Alianza Lima, possuem: 3 + 2 + 1 + 1 = 7 vitrias. Da, sendo x e y, respectivamente, a quantidade de vitrias e empates do Alianza Lima, temos, pelas equaes anteriores, que: Pela equao (1), Pela equao (2), Pela equao (3), derrotas = vitrias = x + 7 empates = y + 1 + 2 + 2 = y + 5 par, y mpar (x + 7) + (y + 5) + (x + 7) = 2 10 = 20 2x + y + 19 = 20 2x + y = 1 (6)

Sabemos que x e y so nmeros positivos. Logo, x = 0 e y = 1. Em outras palavras, o Alianza Lima venceu 0 jogos e empatou 1 jogo. Logo, ele fez 1 ponto. Para saber quantos gols o Boca Juniors fez, basta aplicar a equao (4) do comeo.
EUREKA! N35, 2012

38

Sociedade Brasileira de Matemtica

Sendo z a quantidade de gols que o Boca Juniors fez, temos que: Pela equao (4), 4 + 7 + 2 + 1 + 0 = 0 + 1 + z + 4 + 6 14 = z + 11 z = 3 b) Para comear, calculemos o nmero de derrotas de cada time, sabendo que cada um jogou 4 jogos. Para facilitar, faamos a tabela abaixo, sabendo que j calculamos o nmero de empates de cada time no item anterior: Time Cear Palmeiras Boca Juniors Amrica Alianza Lima Ptos Jogos V 10 8 5 3 1 4 4 4 4 4 3 2 1 1 0 E 1 2 2 0 1 D 4 3 -1 = 0 422=0 412=1 410=3 401=3 GF 4 7 2 1 0 GC 0 1 3 4 6

Da, como Cear e Palmeiras no perderam e, alm disso, o Cear no tomou gol, logo teremos que o jogo entre Cear e Palmeiras terminou empate e, alm disso: Cear 0 0 Palmeiras Alm do empate com o Cear, o Palmeiras empatou outra partida. Alm desse empate, houve mais 2 empates em jogos do Boca Juniors e outro empate do Alianza Lima. Se Palmeiras e Alianza Lima empataram, o Boca Juniors ter empatado 2 vezes contra no se sabe quem, o que absurdo. Logo, houve empates entre Palmeiras e Boca Juniors e entre Boca Juniors e Alianza Lima. Como o Alianza Lima no fez gol, logo temos que: Boca Juniors 0 0 Alianza Lima Alm do empate com o Boca Juniors, o Alianza Lima perdeu todas as outras partidas. Inclusive o jogo contra o Amrica. O Amrica s fez um gol, logo s h um resultado possvel para o jogo entre Amrica e Alianza Lima: Amrica 1 0 Alianza Lima
EUREKA! N35, 2012

39

Sociedade Brasileira de Matemtica

Logo, nos jogos entre Alianza Lima ou Amrica contra Cear, Palmeiras ou Boca Juniros, os times Alianza Lima e Amrica no fizeram gol. Portanto, o gol que o Palmeiras levou foi contra o Boca Juniors em jogo que terminou empate. Logo, temos mais um resultado: Palmeiras 1 1 Boca Juniors O Boca Juniors fez 2 gols e levou 3. Considerando que o Boca Juniors teve 1 gol a favor e 1 gol contra, no jogo contra o Palmeiras e um empate de 0 a 0 contra o Alianza Lima, falta considerar a derrota contra o Cear e a vitria contra o Amrica. A nica forma disso acontecer : Cear 2 0 Boca Juniors Boca Juniors 1 0 Amrica O Cear fez 4 gols e levou 0. Sendo que ele venceu Alianza Lima e Amrica e fez 2 gols contra o Boca Juniors, logo s h uma possibilidade: Cear 1 0 Alianza Lima Cear 1 0 Amrica Faltam apenas os jogos do Palmeiras contra o Alianza Lima e o Amrica. Como sabemos os placares de Alianza Lima e de Amrica contra dos demais e o resultado do jogo entre eles, logo temos que: Palmeiras 4 0 Alianza Lima Palmeiras 2 0 Amrica Portanto, os 10 jogos esto com os resultados especificados. A questo 1 um exemplo de uma idia importante: dividir o problema em casos e resolv-los. Algumas outras idias podem ser aplicadas para resolver propriamente o problema ou apenas para conseguir novas equaes para resolver o problema. Outras idias importantes podem ser necessrias para resoluo dos problemas. Vejamos mais 2 problemas cujas solues dependiam um pouco de algum conhecimento sobre campeonatos e mais algumas outras idias.

EUREKA! N35, 2012

40

Sociedade Brasileira de Matemtica

Problema 2: (OBM Nvel 1 2 fase 2009) Um campeonato de xadrez de 7 rodadas, com 4 jogos por rodada, tem 8 participantes, cujas pontuaes por jogo so as usuais: um ponto por vitria, meio ponto por empate e nenhum ponto por derrota. Cada par de jogadores se enfrenta exatamente uma vez.

a) Ao trmino da terceira rodada, possvel que um grupo de jogadores esteja em primeiro lugar e o restante dos jogadores esteja em segundo lugar? Explique por meio de um exemplo. b) Ao trmino da terceira rodada, possvel que todos os jogadores tenham pontuaes diferentes? Explique.
Soluo:

a) Sim, basta na primeira rodada 4 jogadores vencerem e nas seguintes rodadas ser tudo empate. b) Cada jogador fez no mximo 3 1 = 3 pontos (vencendo todas) e no mnimo 0 pontos. As possibilidades de pontuao de cada jogador so: 0; 0,5; 1; 1,5; 2; 2,5; 3. So 7 possibilidades para 8 jogadores, logo haver, pelo menos, 2 jogadores com a mesma quantidade de pontos, pelo Princpio das Casas dos Pombos. Na questo acima, vemos que uma forma interessante pensar nos casos de todos empatarem todas as vezes ou, pelo menos, em uma determinada rodada. Isso pode nos ajudar a entender ou resolver uma questo. Na prxima questo, note que no importa a pontuao atribuda a uma vitria. Importa apenas que no h empates, ou seja, h apenas vitrias e derrotas.
Problema 3: (OBM Nvel 2 3 fase 2006) Em um torneio de tnis de mesa (no qual nenhum jogo termina empatado), cada um dos n participantes jogou uma nica vez contra cada um dos outros. Sabe-se que, para todo k > 2, no existem k jogadores J1, J2, , Jk tais que J1 ganhou de J2, J2 ganhou de J3, J3 ganhou de J4, , J(k 1) ganhou de Jk, Jk ganhou de J1.

Prove que existe um jogador que ganhou de todos os outros e existe um jogador que perdeu de todos os outros.

EUREKA! N35, 2012

41

Sociedade Brasileira de Matemtica

Soluo: Seja Jv o jogador que mais venceu. Suponhamos que o jogador Jv no

venceu todas. Ento, existe um jogador Jx que o venceu. Note que se existe um jogador Ji que perdeu para Jv e venceu Jx, logo teremos que Jv ganhou de Ji que ganhou de Jx que ganha de Jv, fato que no permitido pelo enunciado. Logo todos os jogadores que Jv venceu, Jx venceu tambm. Mas, Jx venceu todos os jogadores vencidos por Jv e o Jv, ou seja, ele venceu mais jogadores do que o Jv. Absurdo! Portanto, se Jv o jogador que mais venceu, ele venceu todas. A idia para mostrar que o jogador que mais perdeu, perdeu todas totalmente anloga e fica como exerccio para o leitor. Na questo acima, a ideia crucial para resoluo do problema olhar para os casos de mximo e mnimo. Em questes alm desse assunto, essa idia, tambm, pode fazer sentido e, por isso, ela de extrema importncia. Em outras questes parecidas, podemos ter situaes onde pessoas conseguem pontuaes em determinadas avaliaes. Geralmente, nesses casos, deve-se tentar achar relaes, por meio de equaes. Nem sempre necessrio achar o valor das variveis. Para exemplificar, veja o exemplo abaixo:
Problema 4: (Rioplatense Nvel A 2000) As possveis pontuaes para um exame so 0, 1, 2, 3, 4. Depois da correo, observou-se que o nmero de estudantes que obtiveram 3 pontos foi igual ao nmero de estudantes que obtiveram 2 pontos. Alm disso, todos conseguiram, pelo menos, 1 ponto. A soma de todos os pontos obtidos por todos os estudantes no exame foi igual ao nmero de estudantes aumentado em 30. Encontre o nmero de estudantes que conseguiram, pelo menos, 3 pontos. Soluo:

Lembre-se que todo estudante fez, ao menos, 1 ponto. Seja x o nmero de estudantes que conseguiram 1 ponto. Seja y o nmero de estudantes que conseguiram 2 pontos. Seja z o nmero de estudantes que conseguiram 3 pontos. Seja w o nmero de estudantes que conseguiram 4 pontos. Pelo enunciado, temos que:

y=z x + 2y + 3z + 4w = x + y + z + w + 30
Substituindo a primeira equao acima na segunda acima, temos que:
EUREKA! N35, 2012

42

Sociedade Brasileira de Matemtica

x + 2z + 3z + 4w = x + z + z + w + 30 5z + 4w = 2z + w + 30 3z + 3w = 30 z + w = 10
Logo, foram, pelo menos, 10 estudantes que conseguiram, pelo menos, 3 pontos. Por ltimo, faamos mais uma questo que envolve achar relaes e equaes. Note que na questo abaixo o resultado em gols de cada equipe no importa.
Problema 5: (Rioplatense Nvel A 2000) Quatro equipes A, B, C e D disputam um torneio de futebol. Todos jogam contra todos, uma nica vez. Em caso de vitria, a equipe vencedora obtm 3 pontos e o perdedor 0 pontos. Em caso de empate, cada equipe obtm 1 ponto. Ao final do torneio, as pontuaes das equipes A, B, C e D foram a, b, c e d. Observou-se que a b c d e que a b = b c = c d

Encontre todas as possveis pontuaes finais das 4 equipes. Para cada uma dessas possibilidades, construa uma tabela com os resultados dos jogos e justifique porque estas so as nicas possveis pontuaes. Por exemplo, uma pontuao final possvel : a = 3, b = 3, c = 3 e d = 3 e a tabela : A empata com B B empata com C A empata com C B empata com D A empata com D C empata com D
Soluo: Note que temos exatamente 6 partidas. Seja S = a + b + c + d; em outras

palavras, S igual soma total de pontos. Note que cada empate diminui o valor de S, pois ao invs de somar 3 pontos, pontuao em caso de vitria, soma-se 2 pontos, 1 para cada time que empatou. Da, temos que S mximo acontece quando no h empates e S mnimo acontece quando h somente empates. Ajeitando tudo numa equao, temos que: 6 2 = 12 a + b + c + d 18 = 6 3 Alm disso, temos que: (7)

a b = b c a = 2b c b c = c d d = 2c b
EUREKA! N35, 2012

43

Sociedade Brasileira de Matemtica

Substituindo as equaes acima na equao (7) e fazendo as contas, tem-se que: 12 < 2 (b + c) < 18 6<b+c<9 Agora, temos 4 casos: 1) b + c = 6. o caso onde tudo empate, pois corresponde minimalidade na equao (7). O caso que d certo aqui do enunciado. 2) b + c = 7. Como b > c, temos s 1 possibilidade: 2.1) b = 4, c = 3. Nesse caso, como a b = b c = c d = 1, logo temos que a = 5 e d = 2. Como h, no mximo, 3 empates para cada equipe, logo, nesse caso, podemos perceber que A tem 2 empates, B tem 1 empate, D tem 2 empates e h uma quantidade par de empates (equao (2) l no comeo), logo C tem 3 empates. Portanto, A tem 1 vitria, B tem 1 vitria e 1 derrota e D tem 1 derrota. O caso A vence D absurdo, pois B no pode vencer a si mesmo. Logo A vence B e B vence D. O resto das partidas termina empatado. 2.2) b=5, c=2. Nesse caso, d= -1. Absurdo. Logo, no temos mais casos. 3) b + c = 8. Como b > c, temos 2 possibilidades. 3.1) b = 4, c = 4. Nesse caso, como a b = b c = c d = 0, logo temos a = 4 e d = 4. Nesse caso, h 1 vitria, 1 empate e 1 derrota para cada equipe. Nesse caso, podemos fazer, por exemplo, A vence B, que vence C, que vence D, que vence A e empates entre A e C e entre B e D. 3.2) b = 5, c = 3. Nesse caso, como a b = b c = c d = 2, logo temos a = 7 e d = 1. Nesse caso, A tem 1 empate, B tem 2 empates e D tem 1 empate. C ter 3 empates geraria uma quantidade mpar de empates, o que absurdo com a paridade de empates (equao (2) do comeo). Logo, C no tem empates. Se A empata com D, B fica sem ter com quem empatar, o que geraria um absurdo. Logo, A empata com B e vence C e D, pois A faz 7 pontos. Alm disso, B vence C e empata com A e D, pois B faz 5 pontos. Por ltimo, C vence D. 3.3) b = 6, c = 2. Nesse caso, d = 2. Absurdo. Logo, no temos mais casos.
EUREKA! N35, 2012

44

Sociedade Brasileira de Matemtica

4) b + c = 9. o caso onde tudo vitria e derrota, pois corresponde a maximalidade da equao (7). Sabendo que a > b > c > d, logo temos que A vence todas, B vence C e D e, por ltimo, C vence D.
Mais problemas: Problema 6: Na questo anterior, seria possvel A fazer 8 pontos? Justifique sua

resposta.
Problema 7: Considere um campeonato brasileiro de futebol com 20 times. Pelas

regras de futebol, sabe-se que, em caso de vitria, o time vencedor soma 3 pontos e o time perdedor soma 0 pontos. Em caso de empate, os times que empataram somam 1 ponto cada um. No comeo do campeonato, todos comeam com nenhum ponto ganho. Cada rodada composta por 10 jogos, com cada time jogando uma nica vez por rodada. Sabe-se que ao final de 7 rodadas, nenhum time venceu todas e nenhum time perdeu todas. Prove que, aps 7 rodadas, haver, ao menos, dois times com a mesma pontuao.
Problema 8: Um campeonato de basquete acontece no estilo todos jogam contra

todos, uma nica vez. Prove que acontece, pelo menos, uma das situaes abaixo: 1. Um time vence todas. 2. Um time perde todas. 3. Dois times vencem uma quantidade igual de partidas.
Problema 9: (Olmpiada de Maio Nvel 1 1999) Ana, Beatriz, Carlos, Diego e Emilia participam de um torneio de xadrez. Cada jogador enfrenta s uma vez cada um dos outros quatro jogadores. Cada jogador soma 2 pontos se vence a partida, 1 ponto se empata e 0 ponto se perde. No final do torneio, nota-se que as pontuaes dos 5 jogadores so todas distintas. Ache o nmero mximo de empates que pode haver acontecido no torneio e justifique por que no pode haver um nmero maior de empates. Problema 10: (Olmpiada de Maio Nvel 1 2008)

No colgio Olmpico, as notas dos exames so, sempre, nmeros inteiros, sendo a menor nota igual a 0 e a maior nota igual a 10. Na classe de aritmtica, o professor aplica dois exames na sua turma de 15 alunos. Quando um de seus alunos obtm menos que 3 no primeiro exame e mais que 7 no segundo, ele chamado de aluno superado. O professor ao terminar de corrigir os exames, fez a mdia das 30 notas e
EUREKA! N35, 2012

45

Sociedade Brasileira de Matemtica

obteve a nota 8 de mdia. Qual a maior quantidade de alunos superados que pode ter havido nessa classe?
Problema 11: (OBM Nivel 1 2 fase 2002)

No jogo pega-varetas, as varetas verdes valem 5 pontos cada uma, as azuis valem 10 pontos, as amarelas valem 15, as vermelhas, 20 e a preta, 50. Existem 5 varetas verdes, 5 azuis, 10 amarelas, 10 vermelhas e 1 preta. Carlinhos conseguiu fazer 40 pontos numa jogada. Levando em conta apenas a quantidade de varetas e suas cores, de quantas maneiras diferentes ele poderia ter conseguido essa pontuao, supondo que em cada caso fosse possvel pegar as varetas necessrias?
Problema 12: (Seletiva de Fortaleza para a Olmpiada Rioplatense de Matemtica

2001) Quatro problemas so propostos em uma Olmpiada de Matemtica. Cada problema vale 4 pontos. Aps todas as provas terem sido corrigidas, notou-se que quaisquer dois participantes no obtiveram mesma pontuao em mais de um problema. (Cada estudante pode obter 0, 1, 2, 3 ou 4 pontos em cada problema). a) Mostre que o nmero de participantes no pode ser igual a 26. b) D um exemplo mostrando que podemos ter 25 estudantes.

EUREKA! N35, 2012

46

Sociedade Brasileira de Matemtica

SOLUES DE PROBLEMAS PROPOSTOS


Publicamos aqui algumas das respostas enviadas por nossos leitores.

134) Considere a operao entre dois vetores do ( x, y, z ) ( u, v, w) = ( xu + yw + zv, xw + zu + yv, xv + yu + zw) .


k
3 1

definida por:

Prove que, para todo k 1, se ( x, y, z ) = ( 0,0,0 ) ento x = y = z = 0.

Obs.: Para qualquer

( x, y , z ) k k 1 ( x, y , z ) = ( x, y , z ) ( x, y , z ) .

, ( x, y , z ) = ( x, y , z )

e, para todo

k > 1,

SOLUO DE ZOROASTRO AZAMBUJA NETO (RIO DE JANEIRO RJ) Se s ( x, y, z ) = x + y + z, temos

s ( ( x, y, z ) ( u, v, w) ) = xu + yw + zv + xw + zu + yv + xv + yu + zw = ( x + y + z )( u + v + w) = = s ( x, y, z ) s ( u, v, w ) . Assim, se ( x, y, z ) = ( 0,0,0 ) , temos


0 = s ( 0,0,0 ) = s ( x, y, z ) , donde s ( x, y, z ) = 0. Se X =
k k

{( x, y, z )

s ( x, y, z ) = 0 = {( x, y, x y ) , x, y

}, temos

( x , y , z ) , ( u , v , w ) X ( x , y , z ) ( u , v, w ) X . De fato, ( x, y, x y ) ( u, v, u v ) = ( a, b, a b ) , onde a = xu y ( u + v ) v ( x + y ) e b = x ( u + v ) u ( x + y ) + yv. Considere a


transformao linear T :
2

dada por

T ( u, v ) = ( xu y ( u + v ) v ( x + y ) , x ( u + v ) u ( x + y ) + yv ) .

( u, v ) , as duas primeiras coordenadas de ( x, y, x y ) so dadas por T ( u, v ) . Assim, para k k 1, temos ( x, y, z ) = ( 0,0,0 ) se e somente se z = x y e T k 1 ( x, y ) = ( 0,0 ) . x y ( x + 2y) A matriz de T , cujo determinante yx ( 2x + y )
Se as duas primeiras coordenadas de um elemento X so
2 y 3 y2 ( x y ) ( 2 x + y )( x + 2 y ) = 3 ( x + xy + y ) = 3 x + + , que s se 2 4 anula quando x = y = 0. Assim, se ( x, y ) ( 0,0 ) , temos det T 0 e logo 2 2 2

EUREKA! N35, 2012

47

Sociedade Brasileira de Matemtica

det ( T k 1 ) = ( det T )

k 1

0,

para todo
k

k 1,

donde T k 1 ( x, y ) ( 0,0 )

(pois

( x, y ) ( 0,0 ) ), e logo ( x, y, x y )

( 0,0,0 ) , o que resolve o problema.

135) Considere um hemisfrio cuja base um crculo ( C1 ) . Um crculo ( C2 ) do


hemisfrio paralelo a ( C1 ) , de forma que existem n crculos do hemisfrio, congruentes, tangentes entre si, a ( C1 ) e a ( C2 ) . Mostre que a razo K(n) entre os raios de ( C2 ) e ( C1 ) igual a: K ( n ) =
cos 2 n . 1 + sen 2 n

SOLUO DE JOS DE ALMEIDA PANTERA (RIO DE JANEIRO RJ) Sejam C j ,0 j n 1, os n crculos tangentes a C1 e a C2 . Podemos supor sem
perda de
3

generalidade
2 2 2

que

hemisfrio

H=

{( x, y, z )

z 0 e x + y + z = 1 , C1 = {( cos , sen ,0 ) , [ 0,2 ]}

C2 = {( cos cos ,cos sen , sen ) , [ 0, 2 ]} , para um certo 0, (e logo 2 o raio de C2 cos ). Podemos tambm supor que C j tangente a C1 em 2 j 2 j 2 j 2 j , sen ,0 , e (logo) a C2 em cos cos ,cos sen , sen . cos n n n n 1 + cos 2 j 1 + cos 2 j sen Assim, o centro de C j , , cos sen . 2 n 2 n 2

Considere os crculos C 0 e C 1 .
1cos sen 1cos Seus raios so iguais a ,0, = 2 2 2
2

1 cos sen + = sen = 2 2 2

(que a distncia entre o centro de C 0 e o ponto de tangncia entre C 0 e C1 ), e as distncias entre o centro 0 = ( 0,0,0 ) da esfera e os centros de C 0 e C1 so iguais a 1 + cos 1 + cos sen = cos . + = 2 2 2 2 Considerando a reta r, interseo dos planos que contm os crculos C 0 e C1 , temos que r tangente esfera e aos crculos C 0 e C1 . O plano perpendicular a r passando por P contm os centros Q1 e Q2 de C 0 e de C 1 e o centro O da esfera.
EUREKA! N35, 2012
2 2

48

Sociedade Brasileira de Matemtica

Os tringulos OQ1P e OQ2 P so retngulos com hipotenusa OP , e logo Q1Q2 o


dobro da altura do tringulo retngulo OQ1 P , que, como OP = 1, OQ1 = cos e 2 Q1 P = sen , igual a cos sen 1 . Assim, Q1Q2 = 2cos sen , 2 2 2 2 2 sen 1 + cos mas Q1Q2 a distncia entre os pontos ,0, e 2 2 1 + cos 2 1 + cos 2 sen a qual igual a , , cos sen , 2 n 2 n 2
2 1 + cos 1 + cos 2 2 2 2 = 1 cos + sen 2 2cos = cos 2sen , n n 2 2 n 2 n sen 2 = sen . e logo 2cos sen = 2cos2 sen , donde tan = 2 2 2 n 2 cos n 2 1 sen 2 cos 2 1 tan 2 ( 2 ) n= n , cqd. = Assim, o raio de C2 cos = 1 + tan 2 ( 2 ) 1 + sen 2 1 + sen 2 n n
2

142) Seja A = {4,8,9,16, 25, 27,32,36, 49,64,...} o conjunto das potncias no


triviais (nmeros da forma a b , com a 2, b 2 naturais). Prove que, para todo natural n 1, existe um natural k tal que todos os termos da progresso aritmtica k , 2k ,3k ,..., nk pertencem a A.

SOLUO DE MARCELO RIBEIRO (RIO DE JANEIRO RJ) Lema: ( b


ab 1( mod n ) ) mdc ( a, n ) = 1.

Demonstrao: Artigo Divisibilidade, Congruncias e Aritmtica Mdulo n, Eureka N2, ou qualquer texto sobre Teoria dos Nmeros. Mostraremos que, dado n 2, n , podemos escolher (n 1) nmeros naturais, digamos 1 ,..., n 1 , de forma a termos k = 21 3 2 ... n n1 satisfazendo o enunciado.
EUREKA! N35, 2012

49

Sociedade Brasileira de Matemtica

Tomemos, para isso, p1 , p2 ,..., pn nmeros primos, distintos. Basta, agora, mostrar que o sistema de equaes 1 2 ... n 1 0 ( mod p1 ) 1 + 1 2 ... n 1 0 ( mod p2 ) ... + 1 0 ( mod p ) 2 n 1 n 1

Apresenta soluo em inteiros. De fato, i {1, 2,..., n 1} , tomemos p j k i , ki + . j =1, j i +1 Este valor satisfar a todas as equaes, a menos talvez de i + 1 0 ( mod pi +1 ) .
n

i =

Contudo, pelo lema, podemos escolher ki , de forma a termos

i =
j que mdc

p j ki 1( mod pi +1 ) , j =1, j i +1
n

((

n j =1, j i

p j , pi +1 = 1. (Podemos supor k i naturais seno somamos

) )

um mltiplo conveniente de pi +1 ).

145)

os nmeros 2 3 p cos + q cos + r cos = 1. 7 7 7

Encontre

todos

racionais

p,

q,

de

modo

que

SOLUO DE MICHEL FALEIROS MARTINS (CAMPINAS SP) Considere os polinmios pn tais que Pn ( 2cos ) = 2cos ( n ) , n 0.
Como cos ( n + 1) + cos ( n 1) = 2cos cos ( n ) , temos que Pn +1 ( x ) = x Pn ( x ) Pn 1 ( x ) , onde x = 2 cos e n 1. Agora, P0 ( x ) = 2 e P 1 ( x) = x P2 ( x ) = x 2 2, P3 ( x ) = x 3 3x, P4 ( x ) = x 4 4 x 2 + 2,... , 7 (I) raiz da equao x3 x 2 2 x + 1 = 0;
EUREKA! N35, 2012

Afirmamos que, sendo = 2cos

50

Sociedade Brasileira de Matemtica

(II) irracional; (III) 2cos 2 3 2cos + 2cos = 1. 7 7 7 Prova das afirmaes:

(I)

3 4 3 4 = cos + cos = 0 P3 ( ) + P4 ( ) = 0 7 7 7 7 ( 3 3 ) + ( 4 4 2 + 2 ) = 0 4 + 3 4 2 3 + 2 = 0
( + 2 ) ( 3 2 2 + 1) = 0 3 2 2 + 1 = 0.

(II) Pelo Teorema da Raiz Racional, as possveis razes racionais no nulas de x3 x 2 2 x + 1 = 0 so 1. No entanto, nenhum desses dois valores , de fato, raiz. (III) Como 2sen cos = sen ( + ) + sen ( ) , temos que
2sen

2 3 cos cos + cos 7 7 7 7

2 3 4 2 = sen + sen 0 sen + sen + sen + sen = sen 7 7 7 7 7 7 2 3 2cos 2cos + 2cos = 1. 7 7 7 Lema: (0, 0, 0) o nico terno de racionais ( p, q, r) que satisfaz 2 pcos

2qcos

2 3 + 2rcos =0 7 7

(1).

Prova: 2cos

2 = P2 ( ) = 2 2 7 3 2cos = P3 ( ) = 3 3 = 2 + 2 1 3 = 2 1 7 (1) p ( 2 2 ) q+ ( 2 1) r= 0

2 ( r q) + ( p r) + ( 2q r) = 0
Temos os dois casos: (i ) r q= 0.

Se p r 0, ento =

r2q , absurdo. pr

EUREKA! N35, 2012

51

Sociedade Brasileira de Matemtica

Se p r= 0, ento 2q r= 0 p= q= r= 0 e obtemos a soluo trivial de (1). (ii ) r q 0.

Resolvendo a equao de segundo grau temos que s + t , s t com s, t .

Se t < 0, complexo, absurdo. Se t = 0 ou t > 0 e t , racional, absurdo. t . Mas

Portanto, t > 0 e

(s t ) (s t )
(s
3

2 s t +1=

s 2 + 3st 2s t + 1) t ( 3s 2 2s 2 + t )

Como 3 2 2 + 1 = 0 e t * , 3s 2 2s 2 + t = 0 (2) e s 3 s 2 + 3st 2s t + 1 = 0 (3). ( 2 ) t = 3s 2 + 2s + 2. Substituindo em (3), s 3 s 2 + 3s ( 3s 2 + 2 s + 2 ) 2s ( 3s 2 + 2 s + 2 ) + 1 = 0

( 2 s ) 2 ( 2 s ) ( 2s ) + 1 = 0. Pelo Teorema da Raiz Racional, 2s = 1 so as nicas possibilidades de raiz racional no nula, mas nenhum desses dois valores , de fato, raiz. Portanto, p= q= r= 0 a nica soluo racional de (1). 2 3 Voltando ao problema original, se p cos + q cos + r cos = 1 ento, de (III), 7 7 7 obtemos 2 3 ( p 2 ) cos + ( q + 2 ) cos + ( r 2 ) cos = 0. 7 7 7 Pelo Lema, p 2 = q + 2 = r 2 = 0 p = 2, q = 2 e r = 2.
Portanto, a equao dada tem uma nica soluo racional: ( p, q, r ) = ( 2, 2, 2 ) .

146) Determine todos os subconjuntos no-vazios A, B, C de

de modo que:

a) A B = B C = C A = . b) A B C = . c) para quaisquer a A, b B e c C , temos: a + c A, b + c B e a + b C.

EUREKA! N35, 2012

52

Sociedade Brasileira de Matemtica

SOLUO DE MICHEL FALEIROS MARTINS (CAMPINAS SP) A = {1, 4,7,...} , B = {2,5,8,...} e C = {0,3,6,...} ou A = {2,5,8,...} , B = {1, 4,7,...} e C = {0,3,6,...} . Lema: Se S e T so dois subconjuntos no vazios dos naturais com a propriedade que s + t S se s S e t T , ento s + k t S para todo k = {0,1, 2,...} , se s S e t T . Prova: Por induo sobre k : para k = 0, s S verdadeiro. Supondo que s + k t S para algum k , ento s + ( k + 1) t = ( s + k t ) + t S , o que completa a induo. Considere a A, b B e c C elementos quaisquer. Sejam , e os menores elementos de A, B e C, respectivamente. Se = 0,0 + b C b C , absurdo. Analogamente se = 0. Logo, = 0. Como + C e + 3 ento C admite um elemento mnimo m 1. Se m = 1, pelo lema, + k m = + k A e + l B para quaisquer k , l + A e + B, absurdo. Pela simetria do problema em relao a A e B, suponha = 1. Se m = 2, pelo lema, A o conjunto dos naturais mpares e ento par. Mas a + C e como + = 1 + mpar + A, absurdo. Se m = 3, temos dois casos: (i) 2 A 3k + 1,3k + 2 A para todo k , pelo lema. Ento, = 3l para algum l 1 + = 1 + 3l A, mas + C , absurdo. (ii) 2 B 3k + 2 B para todo k , pelo lema; e 1 A 3l + 1 A para todo l , pelo lema. Ento, a + b = ( 3k + 2 ) + ( 3l + 1) = 3 ( k + l + 1) C para quaisquer
k , l . Assim, A = {1, 4,7,...} , B = {2,5,8...} e C = {0,3,6,...} . Esses conjuntos

satisfazem as condies dadas e, pela simetria, A = {2,5,8,...} , B = {1, 4,7,...} e C = {0,3,6,...} tambm cumprem as condies do problema. Se m 4, + m m 1 3 {1, 2,3,...m 2} A. Temos dois casos: (i) m 1 A, pelo lema, todo natural n = r + k m com 1 r m 1 e k pertence a A. Assim, = l m para algum l 2 + = 1 + m ( k + l ) A, mas + C , absurdo. (ii)
m 1 B.

Ento

= m 1 3

2 A 2 + ( m 1) C ,

mas

2 + ( m 1) = 1 + m A, absurdo.
EUREKA! N35, 2012

53

Sociedade Brasileira de Matemtica

147) Demonstre que

( 1) ( 1) =n+ 2 ( 2k + 1) k =0 sen
n2

, para todo inteiro n 2.

4n 2

SOLUO DE MICHEL FALEIROS MARTINS (CAMPINAS SP)


Seja

( 1) Sn = , n 2. Ento, ( 2k + 1) k =0 sen
n2

4n 2

2 n 2

k =0

0 n3 ( 1) ( 1) ( 1) = + ... + ( 2k + 1) ( 2n 5 ) sen sen sen 4n 2 4n 2 4n 2


k n 1 n +1

( 1) + + ( 2n 3 ) sen 4n 2
n2
2 n2

n ( 1) ( 1) + + ( 2n 1) ( 2n + 1) sen sen 4n 2 4n 2
2n2

( 1) ( 1) + + ... + ( 2n + 3) ( 4n 3) sen sen 4n 2 4n 2

( 1) n 1 = 2 Sn + ( 1) . ( 2k + 1) k =0 sen
4n 2
pois

( 2n + 1) + ( 2n 3) = , ( 2n + 3) + ( 2n 5) = ,..., ( 4n 3)
4n 2
n+a

4n 2

4n 2

4n 2

4n 2

4n 2

= , ( 1)

= ( 1)

n2a

para 0 a n 2, e sen ( ) = sen ( ) .

Faamos m = 4n 2. Ento, 1 1 3 sen sen m m

+ ...

1 ( m 3) sen m

1 ( m + 1) sen m

1 ( m + 3) sen m

... +

1 ( m 1) sen m 1 1 ( 2m 3 ) ( 2m 1) sen sen m m

EUREKA! N35, 2012

54

Sociedade Brasileira de Matemtica


2 n2 ( 1) ( 1) n 1 = 2 = 4 Sn + 2 ( 1) . ( 2k + 1) ( 2k + 1) k =0 k =0 sen sen

m 1

4n 2

J que

( m +1) ( m 1)
+

m m m sen ( ) = sen ( ) e m par.


k i

= 2 ,

( m + 3) ( m 3)
+ m

= 2 ,...,

( 2m 1) + = 2 , sen ( 2 ) =
m m

Seja k = e m com k 1. Como sen ( ) =


m1 k k

ei ei , temos que 2i
k k

m1 1 ( 1) ( ) 2k +1 = i m1 ( 1) + ( 1) . = 2 i 2 ( 2k + 1) k =0 k =0 2k +1 1 k =0 2k +1 1 2k +1 + 1 sen

Mas, 2k +1+m = e

( 2k +1+m) i m

=e

( 2k +1) i m + i

= 2k +1 2k +1 =

m 2 +k +1 2

20+1 =
Assim,

m 2 +0 +1 2

21+1 =

m 2 +1+1 2

m 2 1+1 2

m m 2 + 1+1 2 2

k =0

m 1 2

( 1)

2k +1 1

=
m k= 2

m1

( 1)

k +1

2k +1 1

m k= 2

m1

( 1)
2k +1

=
k =0

m 1 2

( 1)

k +1

2k +1 1

Como

m = 2n 1 mpar, o expoente k teve de ser trocado por k + 1 em ambas as 2 igualdades, pois os ndices dos somatrios comeam em paridades diferentes.

Logo,
EUREKA! N35, 2012

55

Sociedade Brasileira de Matemtica

k =0

m1

( 1)
2k +1

1
k

=
k =0

m1

( 1)

2k +1 + 1

4Sn + 2( 1) e
i m i m

n1

= 2i
k =0

m1

( 1)
k +1

2k +1 1

k =0

m1

( 1)
2k +1

=
k =0

m1

( 1)
e
m 1

2k i m

=e

i m1 m
k =0

( 1)
e
i m
m 1

2k i m
k +1
2 k i m

( 1) , 1 Sejam S + ( x ) = e S ( x ) = k = 0 x k k = 0 x k 0 k m 1 so as msimas razes da unidade.


j j

onde

k = e

com

Observe que, para um polinmio P ( x ) = ( x r0 )( x r1 ) ... ( x rj ) , 1 x r = ( ln x r )= ( ln x r ) = ( ln P ( x ) )= P ( x ) .


j k k =0 k k =0

P( x )

k =0

m m m Agora, sendo P ( x ) = x m 1 = x 2 1 x 2 + 1 = x 2 1 Q ( x ) , as razes de m P x ( ) m so as razes de x 2 1 = 2 k ,0 k 1, e Q ( x ) so Q ( x) 2

2 k +1 ,0 k

m 1, pois 2
m

(2k +1 )

m 2

2( 2 k +1) 2 i m m = e = ei ( 2 k +1) = 1,0 k 1. 2


m 1 2 k =0

Ento, S + ( x ) + S ( x ) = 2

1 x 2 k +1
2 m 2 1 m 1 x m x m 1 m x 2 2 m = m x 1 1 xm 2 x +1
m

S ( x ) = 2

Q( x) P( x) = Q ( x ) P ( x)

4 Sn + 2 ( 1)

n 1

= 2i e

i m

i i S e m = 2i e m

i 2 me m = m = 4n 2. i m m 1 e

EUREKA! N35, 2012

56

Sociedade Brasileira de Matemtica

Portanto, S n = n +

( 1)

para todo n 2.
n k . m k =0
n 1

148) Sejam m e n inteiros positivos. Calcule

SOLUO DE CARLOS ALBERTO DA SILVA VICTOR (NILPOLIS RJ) n 1 n k s = onde m e n so inteiros positivos: k =0 m
i) Seja n m
n n 1 1 s= + + ... m m m

Tomando: = menor inteiro maior do que ou igual a x, teremos: x s = 1 + 1 + ... + 1 = n 1 = n. ii) Seja n > m, logo n = q m + r com r {0,1, 2,..., m 1} Logo
n n 1 m m 1 1 s= + + ... + + + ... + m m m m m s = r ( q + 1) + m ( q + q 1 + ... + 2 + 1)

2 mq s = ( q + 1) + r . 2 Se quisermos escrever esta expresso em termos apenas de m e n, obtemos n m n m n n n s = + 1 + n m = + 1 n . Note que essa m 2 m m m 2 m frmula tambm vale se n m.

s = r ( q + 1) + m q

( q + 1)

150) Sejam a, b e c nmeros reais tais que ( a b ) + ( b c ) + ( c a ) = 9.


EUREKA! N35, 2012

57

Sociedade Brasileira de Matemtica

Prove que

(a b)

(a b)

(c a)

3 3.

SOLUO DE RENATO CARNEIRO (BELO HORIZONTE MG) Fazendo a b = x, b c = y e z = c a, ento teremos que provar que 1 1 1 + 2 + 2 3 3. 2 x y z
x3 + y 3 + z 3 = 9 , ento vamos l: Para isso, temos que x + y + z = 0 Fazendo z = ( x + y ) , teremos
x3 + y 3 + z 3 = x3 + y 3 ( x + y ) = 3x 2 y 3xy 2 = 3xy ( x + y ) = 3 xyz = 9 xyz = 3.
3

Agora basta usar a famosa desigualdade das mdias: 1 1 1 + + 2 1 1 1 1 1 1 1 x2 y 2 z 2 3 2 2 2 2 + 2 + 2 3 3 x y z x y z 3 xyz

1 1 1 1 1 1 1 + + 3 3 2 + 2 + 2 3 3. x2 y 2 z 2 3 x y z

Agradecemos o envio de solues e a colaborao de:


Marcos Ferreira (Eunpolis BA) Besaleel Jnior (Teresina PI) Renato Carneiro (Belo Horizonte MG) Marcelo Ribeiro (Rio de Janeiro RJ) Jean-Pierre Youyoute (Rio de Janeiro RJ) Itamar Sales (Fortaleza CE) Samuel Abdalla (Sorocaba SP) Douglas Oliveira de Lima (Braslia DF) Lucas Justo de Freitas Neto (Mossor RN) Jos Armando Barbosa Filho (Fortaleza CE) Carlos Alberto da Silva Victor (Nilpolis RJ) Prob. 134 Prob. 144 Prob. 145, 146 Prob. 150 Prob. 150 Prob. 150 Prob. 150 Prob. 150 Prob. 149 Prob. 134 Prob. 146, 150

Continuamos aguardando solues para os problemas 143, 144, 149.

EUREKA! N35, 2012

58

Sociedade Brasileira de Matemtica

PROBLEMAS PROPOSTOS
Convidamos o leitor a enviar solues dos problemas propostos e sugestes de novos problemas para prximos nmeros.
12 1 151) Encontre todas as solues reais positivas de x x = . 2 152) Sejam a, b, c nmeros reais positivos tais que a + b + c = 1. a bc b ca c ab 3 Prove que + + . a + bc b + ca c + ab 2 153) Quatro pontos P, Q, R, S pertencem a um crculo de tal forma que o ngulo

PSR reto. Sejam H e K as projees de Q nos segmentos PR e PS, respectivamente. Prove que a reta HK divide o segmento QS ao meio.

154)

Determine

todas

as

funes

f:

[ 0, + )

tais

que

f ( xy ) = f ( x ) f ( y ) e f ( x + y ) f ( x ) + f ( y ) , x, y .

155) Sejam a, b e c inteiros positivos, tais que existe um tringulo T de lados


a , b e c . Prove que so equivalentes: i) Existe um tringulo congruente a T cujos vrtices tm coordenadas inteiras em 2 . ii) T tem rea racional e existem x, y inteiros com a = x 2 + y 2 .

iii) T tem rea racional e existem u, v inteiros com mdc ( a, b, c ) = u 2 + v 2 .


156) Denominamos Mquina de Conway o par C = ( e; S ) formado por uma entrada
e = 21 , a1 , e uma sequncia finita S = ( si )1i n de racionais no nulos, com sn . Dada uma mquina de Conway C, construmos a sequncia pi definida por: (1) p1 = e; (2) pk = pk 1 si , onde i o menor nmero inteiro positivo tal que pk 1 si . Dizemos que a uma sada de C se, e somente se, existe um inteiro positivo k tal que pk = 2a. O conjunto de todas as sadas denominado conjunto gerado por C. Exemplo: A mquina de Conway formada pela entrada 22 e pela sequncia

EUREKA! N35, 2012

59

Sociedade Brasileira de Matemtica

17 78 19 23 29 77 95 77 1 11 13 15 15 , , , , , , , , , , , , ,55 91 85 51 38 33 29 23 19 17 13 11 14 2 gera os nmeros primos. Mostre que possvel construir uma Mquina de Conway que gere o conjunto dos quadrados perfeitos.

Problema 151 proposto por Douglas Oliveira de Lima, de Braslia (DF). Problemas 152 e 153 propostos por Adriano Carneiro Tavares, de Caucaia (CE). Problema 156 proposto em uma prova de seleo para a Olimpada Internacional de Matemtica (IMO) de 1996.

EUREKA! N35, 2012

60

Sociedade Brasileira de Matemtica

AGENDA OLMPICA
XXXIV OLIMPADA BRASILEIRA DE MATEMTICA NVEIS 1, 2 e 3 Primeira Fase sbado, 16 de junho de 2012 Segunda Fase sbado, 22 de setembro de 2012 Terceira Fase sbado, 27 de outubro de 2011 (nveis 1, 2 e 3) domingo, 28 de outubro de 2012 (nveis 2 e 3 - segundo dia de prova) NVEL UNIVERSITRIO Primeira Fase sbado, 22 de setembro de 2012 Segunda Fase sbado, 27 e domingo, 28 de outubro de 2012 V ROMANIAN MASTER OF MATHEMATICS (RMM) 29 de fevereiro a 4 de maro de 2012 (Bucareste, Romnia) ASIAN PACIFIC MATH OLYMPIAD (APMO) 12 de maro de 2012 XVIII OLIMPADA DE MAIO 12 de maio de 2012 XXIII OLIMPADA DE MATEMTICA DO CONE SUL 27 de outubro a 03 de novembro (Lima, Peru) LIII OLIMPADA INTERNACIONAL DE MATEMTICA 4 a 16 de julho de 2012 (Mar del Plata, Argentina) II OLIMPADA DE MATEMTICA DA LUSOFONIA 20 a 28 de julho de 2012 (Salvador, Bahia) XVIII OLIMPADA INTERNACIONAL DE MATEMTICA UNIVERSITRIA (IMC) 26 de julho a 1 de agosto de 2012 (Blagoevgrad, Bulgria) XXVI OLIMPADA IBERO-AMERICANA DE MATEMTICA 29 de setembro a 6 de outubro de 2012 (Cochabamba, Bolvia) IV COMPETIO IBERO-AMERICANA INTERUNIVERSITRIA DE MATEMTICA 1 a 5 de outubro de 2012 (Guanajuato, Mxico) XIV OLIMPADA IBEROAMERICANA DE MATEMTICA UNIVERSITRIA A confirmar

EUREKA! N35, 2012

61

Sociedade Brasileira de Matemtica

COORDENADORES REGIONAIS
Alberto Hassen Raad Amrico Lpez Glvez Antonio Carlos Nogueira Benedito Tadeu Vasconcelos Freire Bruno Holanda Carmen Vieira Mathias Claus Haetinger Cludio de Lima Vidal Denice Fontana Nisxota Menegais Disney Douglas Lima de Oliveira Edson Roberto Abe Edney Aparecido Santulo Jr. Fbio Brochero Martnez Florncio Ferreira Guimares Filho Francinildo Nobre Ferreira Genildo Alves Marinho Herivelto Martins Gilson Tumelero Ivanilde Fernandes Saad Joo Bencio de Melo Neto Joo Francisco Melo Libonati Diogo Diniz Jos Luiz Rosas Pinho Jos Vieira Alves Jos William Costa Krerley Oliveira Licio Hernandes Bezerra Luciano G. Monteiro de Castro Luzinalva Miranda de Amorim Marcelo Dias Marcelo Antonio dos Santos Marcelo Rufino de Oliveira Newman Simes Nivaldo Costa Muniz Osnel Broche Cristo Uberlndio Batista Severo Raul Cintra de Negreiros Ribeiro Reginaldo de Lima Pereira Reinaldo Gen Ichiro Arakaki Ricardo Amorim Ronaldo Alves Garcia Rogrio da Silva Igncio Rosangela Ramon Seme Gebara Neto Tadeu Ferreira Gomes Toms Menndez Rodrigues Valdenberg Arajo da Silva Wagner Pereira Lopes Wanderson Breder William Serafim dos Reis
EUREKA! N35, 2012

(UFJF) (USP) (UFU) (UFRN) (CAEN UFC) (UNIFRA) (UNIVATES) (UNESP) (UNIPAMPA) (UFAM) (Colgio Objetivo de Campinas) (UEM) (UFMG) (UFES) (UFSJ) (Centro Educacional Leonardo Da Vinci) (USP So Carlos) (UTFPR) (UC. Dom Bosco) (UFPI) (Grupo Educacional Ideal) (UFPB) (UFSC) (UFPB) (Instituto Pueri Domus) (UFAL) (UFSC) (Sistema Elite de Ensino) (UFBA) (Grupo Educacional Etapa) FACOS (Grupo Educacional Ideal) (Cursinho CLQ Objetivo) (UFMA) (UFLA) (UFPB) (Colgio Anglo) (Escola Tcnica Federal de Roraima) (UNIFESP) (Centro Educacional Logos) (UFGO) (Col. Aplic. da UFPE) (UNOCHAPEC) (UFMG) (UEBA) (U. Federal de Rondnia) (U. Federal de Sergipe) (CEFET GO) (CEFET RJ) (UFT TO)

Juiz de Fora MG Ribeiro Preto SP Uberlndia MG Natal RN Fortaleza CE Santa Mara RS Lajeado RS S.J. do Rio Preto SP Bag RS Manaus AM Campinas SP Maring PR Belo Horizonte MG Vitria ES So Joo del Rei MG Taguatingua DF So Carlos SP Pato Branco PR Campo Grande MS Teresina PI Belm PA Campina Grande PB Florianpolis SC Campina Grande PB Santo Andr SP Macei AL Florianpolis SC Rio de Janeiro RJ Salvador BA So Paulo SP Osrio RS Belm PA Piracicaba SP So Luis MA Lavras MG Joo Pessoa PB Atibaia SP Boa Vista RR SJ dos Campos SP Nova Iguau RJ Goinia GO Recife PE Chapec SC Belo Horizonte MG Juazeiro BA Porto Velho RO So Cristvo SE Jata GO Nova Friburgo RJ Arraias TO

62

CONTEDO
XXXIII OLIMPADA BRASILEIRA DE MATEMTICA Problemas e solues da Primeira Fase XXXIII OLIMPADA BRASILEIRA DE MATEMTICA Problemas e solues da Segunda Fase XXXIII OLIMPADA BRASILEIRA DE MATEMTICA Problemas e solues da Terceira Fase XXXIII OLIMPADA BRASILEIRA DE MATEMTICA Problemas e solues da Primeira Fase Nvel Universitrio XXXIII OLIMPADA BRASILEIRA DE MATEMTICA Problemas e solues da Segunda Fase Nvel Universitrio XXXIII OLIMPADA BRASILEIRA DE MATEMTICA Premiados COORDENADORES REGIONAIS 87 79 63 55 32 15 2

Sociedade Brasileira de Matemtica

XXXIII OLIMPADA BRASILEIRA DE MATEMTICA


Problemas e solues da Primeira Fase
PROBLEMAS NVEL 1

1. Em maio, o valor total da conta de telefone celular de Esmeralda foi R$119,76, sem os impostos. Esse valor corresponde aos itens: chamadas, acesso internet, envio de mensagens. Se ela gastou R$29,90 com acesso Internet e R$15,50 com o servio de envio de mensagens, quanto foi que ela gastou com chamadas? A) R$74,36 B) R$74,46 C) R$84,36 D) R$89,86 E) R$104,26 2. Numa padaria, uma lata de 200g de achocolatado em p CHOCOBM custa R$3,00, uma lata de 400g custa R$5,00 e a de 800g custa R$9,00. Lara precisa de 1,2kg de CHOCOBM para fazer um enorme bolo. Qual das opes a seguir a maneira mais econmica de comprar 1,2kg de CHOCOBM nessa padaria? A) 6 latas de 200g B) 1 lata de 400g e 1 lata de 800g C) 4 latas de 200g e 1 lata de 400g D) 2 latas de 200g e 1 lata de 800g E) 2 latas de 200g e 2 latas de 400g 3. O grfico mostra a variao dos preos de alguns produtos alimentcios no primeiro semestre em uma certa regio. Com base no grfico possvel afirmar com certeza que

A) o milho sempre foi mais barato que o arroz e o feijo B) o preo do arroz foi o mais estvel no perodo C) o feijo sempre custou mais caro que o milho D) nunca houve dois produtos com o mesmo preo E) o produto com menor variao de preos foi o feijo
EUREKA! N36, 2012

Sociedade Brasileira de Matemtica

4. Uma data curiosa neste ano o dia 11/11/11, pois o dia, ms e dois ltimos dgitos do ano so iguais. No ano passado, esse padro aconteceu em 10/10/10. Quantos dias h desde 10/10/10 at 11/11/11, incluindo o dia 10 e o dia 11? A) 396 B) 398 C) 400 D) 402 E) 404 5. Luana colou com fita adesiva 6 tringulos equilteros nos lados de um hexgono, conforme a figura, obtendo um polgono de 12 lados.

Se ela trocar 3 tringulos por 2 quadrados e 1 pentgono regular, todos com lado de mesmo tamanho do lado do hexgono, ela vai obter um polgono com quantos lados? A) 14 B) 16 C) 17 D) 18 E) 25 6. Por conta de uma erupo de um vulco, 10% dos voos de um aeroporto foram cancelados. Dos voos restantes, 20% foram cancelados pela chuva. Que porcentagem do total de voos deste aeroporto foram cancelados? A) 28% B) 30% C) 35% D) 38% E) 70% 7. O produto de trs nmeros naturais 105 e a sua soma a maior possvel. Qual essa soma? A) 15 B) 23 C) 27 D) 39 E) 107 8. O retngulo da figura abaixo est dividido em 10 quadrados. As medidas dos lados de todos os quadrados so nmeros inteiros positivos e so os menores valores possveis.

A rea desse retngulo : A) 180 B) 240


EUREKA! N36, 2012

C) 300

D) 360

E) 450

Sociedade Brasileira de Matemtica

9. Numa classe de 36 alunos, todos tm alturas diferentes. O mais baixo dos meninos mais alto do que cinco meninas, o segundo menino mais baixo mais alto do que seis meninas, o terceiro menino mais baixo mais alto do que sete meninas e assim por diante, observando-se que o mais alto dos meninos mais alto do que todas as meninas. Quantas meninas h nessa classe? A) 12 B) 14 C) 16 D) 18 E) 20

10. Esmeralda escolheu quatro nmeros e, ao somar cada um deles mdia aritmtica dos outros trs, achou os nmeros 60, 64, 68 e 72. Qual a mdia aritmtica dos quatro nmeros que ela escolheu no incio? A) 30 B) 31 C) 32 D) 33 E) 66

11. Luca comprou uma revista por R$9,63 e deu uma nota de R$10,00 para pagar. De quantas maneiras ele pode receber o troco de 37 centavos em moedas, se as moedas disponveis no caixa so as de 1, 5, 10 e 25 centavos? Suponha que h muitas moedas de cada tipo.
A) 10 B) 12 C) 15 D) 24 E) 30

12. Dois tringulos equilteros de permetro 36 cm cada um so sobrepostos de modo que sua interseo forme um hexgono com pares de lados paralelos, conforme ilustrado no desenho. Qual o permetro desse hexgono? A) 12 cm B) 16 cm C) 18 cm D) 24 cm E) 36 cm 13. Numa corrida com 2011 participantes, Dido chegou frente do qudruplo do nmero de pessoas que chegaram sua frente. Em que lugar chegou o Dido? A) 20 B) 42 C) 105 D) 403 E) 1005

14. Quantos nmeros inteiros positivos menores do que 30 tm exatamente quatro divisores positivos? A) 9 B) 10 C) 11 D) 12 E) 13

EUREKA! N36, 2012

Sociedade Brasileira de Matemtica

15. Um cubo de madeira, pintado de vermelho, foi serrado em 27 cubos menores iguais e as faces desses cubos ainda no pintadas o foram de branco. Qual a razo entre a rea da superfcie total pintada em vermelho e a rea da superfcie total pintada de branco?
A) 1:2 B) 1:1 C) 2:1 D) 1:3 E) 2:3

16. Esmeralda rasgou uma folha de papel em n pedaos e, em seguida, pegou uma dessas partes e rasgou-a tambm em n pedaos. No satisfeita, pegou uma destas ltimas partes e tambm a rasgou em n partes. Qual dos nmeros a seguir poderia ser a quantidade total de pedaos obtida por Esmeralda? A) 15 B) 18 C) 24 D) 26 E) 28 17. O nmero n 999999 tem 2011 algarismos e todos iguais a 9. Quantos algarismos 9 tem o nmero n 2 ? A) nenhum B) 11 C) 2010 D) 2011 E) 4022 18. No desenho, trs cubos iguais apoiados sobre uma mesa tm suas faces pintadas com os nmeros 0, 1, 3, 4, 5 e 9. Qual a soma dos nmeros de todas as faces em contacto com a mesa? A) 6 B) 8 C) 9 D) 10 E) 12

19. Representamos por n! o produto de todos os inteiros positivos de 1 a n. Por !2011 !, exemplo, 5! 1 2 3 4 5 . Calculando a soma 1!2!3!4! 2010 qual o algarismo das unidades do resultado obtido? A) 1 B) 3 C) 4 D) 7 E) 9 20. Esmeralda tem 11 notas de dois reais, Rosa tem 7 notas de cinco reais e Nelly tem 3 notas de dez reais. Qual o menor nmero possvel do total de notas que devem mudar de mos de forma que todas as moas fiquem com a mesma quantia? A) 5 B) 6 C) 7 D) 8 E) 9

EUREKA! N36, 2012

Sociedade Brasileira de Matemtica

PROBLEMAS NVEL 2

1. A figura ao lado representa um mapa de estradas. Os nmeros escritos nas setas indicam quanto de pedgio um viajante deve pagar ao passar pela estrada. Todas as estradas so de mo nica, como indicam as setas. Qual o valor mnimo de pedgio pago por um viajante que sai da cidade A e chega na cidade B?

7 4 9 8

6 B

5 4

A) 11

B) 14

C) 12

D) 10

E) 15

2. O pai de Esmeralda comprou um terreno retangular de 120 metros de comprimento por 80 metros de largura. Devido a leis ambientais, ele deve plantar rvores em 20% do terreno. Ele faz isso plantando-as em duas faixas de mesma largura nas laterais do terreno, conforme mostra a figura. Qual essa largura? A) 6m B) 8m C) 10m D) 16m E) 24m

3. Veja o problema N 18 do Nvel 1


2 2 4. Qual o valor da expresso 20112011 20112003 16 20112007? 2 2 A) 2 20112007 B) 2 20112003 C) 2 20112007 2 D) 2 20112003 E) 2 20112011

5. Quantos tringulos no congruentes de permetro 7 tm todos os lados com comprimentos inteiros? A) 1 B) 2 C) 3 D) 4 E) 5 6) Veja o problema N 12 do Nvel 1

EUREKA! N36, 2012

Sociedade Brasileira de Matemtica

7. Qual o produto da quantidade de vogais pela quantidade de consoantes na alternativa correta? (No considere as letras A, B, C, D, E das alternativas na contagem.) A) Vinte e quatro. B) Trinta e seis. C) Quarenta e dois. D) Quarenta e oito. E) Cinquenta e seis. 8. Luca comprou um gibi por R$4,63 e pagou com uma nota de R$5,00. De quantas maneiras ele pode receber seu troco de 37 centavos, com moedas de 1, 5, 10 e 25 centavos? Suponha que h muitas moedas de cada tipo. A) 10 B) 12 C) 15 D) 24 E) 25 9. Quantos nmeros inteiros positivos menores que 30 tm exatamente quatro divisores positivos? A) 6 B) 7 C) 8 D) 9 E) 10
C ) 50 , a bissetriz do ngulo C ) m( BA 10. Em um tringulo ABC com m( AB
B intersecta o lado AB em D. Seja E o ponto do lado AC tal que AC E ) 90 . A medida do ngulo AD E : m(CD

A) 25o

B) 30o

C) 40o

D) 45o

E) 50o
13 , 14

11. Subtraindo um mesmo nmero do numerador e do denominador da frao obtemos a frao A) 1


14 . A soma dos algarismos desse nmero : 13 B) 3 C) 5 D) 7

E) 9

12. Se multiplicarmos todos os inteiros positivos menores que 2011 que no so mltiplos de 5, qual ser o algarismo das unidades do nmero obtido? A) 2 B) 4 C) 6 D) 7 E) 8
Y 90 . Sejam M e N os pontos 13. Seja XOY um tringulo retngulo com XO mdios de OX e OY, respectivamente. Dado que XN = 19 e YM = 22, determine a medida do segmento XY. A) 24 B) 26 C) 28 D) 30 E) 32

EUREKA! N36, 2012

Sociedade Brasileira de Matemtica

14. Safira rasgou uma folha de papel em n pedaos e, em seguida, pegou um desses pedaos e rasgou tambm em n pedaos. No satisfeita, ela pegou um desses ltimos pedaos e tambm rasgou em n pedaos. Qual dos nmeros a seguir pode representar a quantidade final de pedaos em que Safira rasgou a folha? A) 15 B) 26 C) 28 D) 33 E) 36 15. Qual a maior quantidade de nmeros do conjunto 1,2,3,,20 que podemos escolher de modo que nenhum deles seja o dobro do outro? A) 10 B) 11 C) 12 D) 13 E) 14 16. Topzio desenhou cada figura a seguir, exceto uma, tirando o lpis do papel exatamente uma vez e nunca passando pela mesma linha duas vezes. Qual das figuras abaixo ela no desenhou?

A)

B)

C)

D)

E)

17. No tringulo ABC, os pontos D e E pertencem ao lado BC e so tais que BD = E ) 40 , BA e CE = CA. Dado que m( DA C ? quanto mede, em graus, o ngulo BA A) 80 B) 90 C) 100 D) 110 E) 120

18. Em um teste de mltipla escolha com 24 problemas, cada um pode receber uma das seguintes pontuaes: 4 pontos se a resposta correta, menos 1 ponto se a resposta incorreta e 0 ponto se a resposta est em branco. Sabendo que um estudante recebeu exatamente 52 pontos, qual o nmero mximo de respostas corretas que ele pode ter obtido? A) 14 B) 15 C) 16 D) 17 E) 18 19. A calculadora de Esmeralda est quebrada: quando ela aperta o boto ,a calculadora faz, ao acaso, uma das duas seguintes operaes: tirar a raiz quadrada (como deveria fazer) ou dividir o nmero por 100 (como no deveria fazer). Esmeralda digitou o nmero 201120112011 na calculadora e comeou a apertar o
EUREKA! N36, 2012

Sociedade Brasileira de Matemtica

boto repetidamente. Quantas vezes, no mximo, Esmeralda aperta o boto at aparecer pela primeira vez um nmero menor que 2? A) 2 B) 4 C) 5 D) 8 E) 9 20. Renan quer pintar os quadradinhos da figura ao lado, usando at trs cores diferentes, de modo que quadradinhos que compartilham um lado em comum possuam cores diferentes. Quantas pinturas distintas Renan poder fazer? A) 39 B) 246 C) 178 D) 150 E) 120 21. No Planeta Nrdia, existem trs espcies de nerds: ET-nerds, UFO-nerds e OVNI-nerds. A primeira mente quando chove e diz a verdade quando no chove; a segunda sempre mente; a terceira sempre diz a verdade. Certo dia Bruberson, um nerd muito camarada, se encontra com quatro nerds. E eles falam: X: "Hoje est chovendo." Y: "O nerd que acabou de falar est mentindo." Z: "Hoje no est chovendo." W: "O primeiro nerd mentiu ou eu sou um ET-nerds." Com quantos ET-nerds Bruberson falou no mximo? A) 0 B) 1 C) 2 D) 3 E) 4 22. Qual o primeiro dgito no nulo aps a vrgula na representao decimal da 1 frao 12 ? 5 A) 1 B) 2 C) 4 D) 5 E) 7 23. Esmeralda tem 2011 balas e quer coloc-las em fileiras, cada fileira com a mesma quantidade de balas. Ela estabelece que tanto a quantidade de fileiras como a quantidade de balas em cada fileira devem ser maiores do que 32. Ela sabe que no consegue fazer isso com 2011 balas, pois 2011 primo, ento faz isso com a maior quantidade de balas que puder usar e d as balas que sobrarem para Jade. Quantas balas Jade ganhou? A) 1 B) 2 C) 5 D) 7 E) 11

EUREKA! N36, 2012

Sociedade Brasileira de Matemtica

24. Uma circunferncia passando pelos vrtices B, A, D do paralelogramo ABCD D) 60 e AD 10, o encontra o segmento CD em Q. Sabendo que m( BA tamanho do segmento CQ : 5 3 A) 10 B) 20 C) D) 10 3 E) 15 2 25. Rosa escreveu os nmeros de 1 a 6 nos vrtices do hexgono ao lado. Em seguida, para cada lado do hexgono, ela multiplicou os nmeros escritos nas suas extremidades, obtendo seis nmeros. Qual o valor mnimo da soma dos seis nmeros obtidos? A) 69 B) 58 C) 59 D) 61 E) 57
PROBLEMAS NVEL 3

1. Veja o problema N 1 do Nvel 2 2. Um cubo de madeira, pintado de vermelho, foi serrado em 27 cubos menores iguais e as faces desses cubos ainda no pintadas o foram de branco. Qual a razo entre a rea da superfcie total pintada em vermelho e a rea da superfcie total pintada de branco? A) 1:2 B) 1:1 C) 2:1 D) 1:3 E) 2:3 3. Numa padaria, uma lata de 200g de achocolatado em p CHOCOBM custa R$3,00, uma lata de 400g custa R$5,00 e a de 800g custa R$9,00. Lara precisa de 1,2kg de CHOCOBM para fazer um enorme bolo. Qual das opes a seguir a maneira mais econmica de comprar 1,2kg de CHOCOBM nessa padaria? A) 6 latas de 200g B) 1 lata de 400g e 1 lata de 800g C) 4 latas de 200g e 1 lata de 400g D) 2 latas de 200g e 1 lata de 800g E) 2 latas de 200g e 2 latas de 400g 4. Os inteiros positivos 30, 72 e N possuem a propriedade de que o produto de quaisquer dois divisvel pelo terceiro. Qual o menor valor possvel de N? A) 60 B) 30 C) 30 72 D) 360 E) 6
EUREKA! N36, 2012

10

Sociedade Brasileira de Matemtica

5. Numa classe de 36 alunos, todos tm alturas diferentes. O mais baixo dos meninos mais alto do que cinco meninas, o segundo menino mais baixo mais alto do que seis meninas, o terceiro menino mais baixo mais alto do que sete meninas e assim por diante, observando-se que o mais alto dos meninos mais alto do que todas as meninas. Quantas meninas h nessa classe? A) 12 B) 14 C) 16 D) 18 E) 20 6. Qual o produto da quantidade de vogais pela quantidade de consoantes na alternativa correta? (No considere as letras A, B, C, D, E das alternativas na contagem.) A) Vinte e quatro. B) Trinta e seis. C) Quarenta e dois. D) Quarenta e oito. E) Cinquenta e seis. 7. Sendo a e b reais tais que 0 < a 1 e 0 < b 1, o maior valor que assumir : A) 0 B)
1 4 1 3 1 2

ab pode ab

C)

D)

E) 1

8. Por conta de uma erupo de um vulco, 10% dos voos de um aeroporto foram cancelados. Dos voos restantes, 20% foram cancelados pela chuva. Que porcentagem do total de voos deste aeroporto foram cancelados? A) 28% B) 30% C) 35% D) 38% E) 70%
2 2 9. Qual o valor da expresso 20112011 20112003 16 20112007? 2 A) 2 20112007 2 B) 2 20112003

C) 2 20112007

D) 2 20112003

E) 2 20112011
2

10. Luca comprou uma revista por R$9,63 e deu uma nota de R$10,00 para pagar. De quantas maneiras ele pode receber o troco de 37 centavos em moedas, se as moedas disponveis no caixa so as de 1, 5, 10 e 25 centavos? Suponha que h muitas moedas de cada tipo. A) 10 B) 12 C) 15 D) 24 E) 30 11. Quantos nmeros inteiros positivos menores que 30 tm exatamente quatro divisores positivos? A) 6 B) 7 C) 8 D) 9 E) 10

EUREKA! N36, 2012

11

Sociedade Brasileira de Matemtica

C ) 50 , a bissetriz do ngulo C ) m( BA 12. Em um tringulo ABC com m( AB


B intersecta o lado AB em D. Seja E o ponto do lado AC tal que AC E ) 90 . A medida do ngulo AD E : m(CD

A) 25o

B) 30o

C) 40o

D) 45o

E) 50o

13. Esmeralda tem 11 notas de dois reais, Rosa tem 7 notas de cinco reais e Nelly tem 3 notas de dez reais. Qual o menor nmero possvel do total de notas que devem mudar de mos de forma que todas as moas fiquem com a mesma quantia? A) 5 B) 6 C) 7 D) 8 E) 9 14. Qual o primeiro dgito no nulo aps a vrgula na representao decimal da 1 frao 12 ? 5 A) 1 B) 2 C) 4 D) 5 E) 7 15. No Planeta Nrdia, existem trs espcies de nerds: ET-nerds, UFO-nerds e OVNI-nerds. A primeira mente quando chove e diz a verdade quando no chove; a segunda sempre mente; a terceira sempre diz a verdade. Certo dia Bruberson, um nerd muito camarada, se encontra com quatro nerds. E eles falam: X: "Hoje est chovendo." Y: "O nerd que acabou de falar est mentindo." Z: "Hoje no est chovendo." W: "O primeiro nerd mentiu ou eu sou um ET-nerds." Com quantos ET-nerds Bruberson falou no mximo? A) 0 B) 1 C) 2 D) 3 E) 4 16. Um peso de papel tem a forma de um A tringulo de lados BC = 6 cm e AB = AC = 5 cm e est parcialmente preenchido com gua. 6 5 5 h Quando o peso de papel se apoia sobre o lado 3 BC, a gua tem uma altura de 3 cm. Qual a 6 5 B C B altura da gua, em cm, quando o peso de papel se apoia sobre o lado AB? 4 3 8 18 24 A) B) C) D) E) 3 2 5 5 5 17. O maior inteiro positivo n tal que (2011!)! divisvel por ((n!)!)! : A) 3 B) 4 C) 5 D) 6 E) 7
EUREKA! N36, 2012

C 5 A

12

Sociedade Brasileira de Matemtica

18. A calculadora de Esmeralda est quebrada: quando ela aperta o boto ,a calculadora faz, ao acaso, uma das duas seguintes operaes: tirar a raiz quadrada (como deveria fazer) ou dividir o nmero por 100 (como no deveria fazer). Esmeralda digitou o nmero 201120112011 na calculadora e comeou a apertar o boto repetidamente. Quantas vezes, no mximo, Esmeralda aperta o boto at aparecer pela primeira vez um nmero menor que 2? A) 2 B) 4 C) 5 D) 8 E) 9 19. Existem 3 valores inteiros positivos de n > 1 tais que 10 pode ser escrito como soma de n inteiros positivos e distintos: n = 2: 10 = 3 + 7 n = 3: 10 = 2 + 3 + 5 n = 4: 10 = 1 + 2 + 3 + 4 Quantos valores inteiros e positivos de n > 1 existem para os quais possvel expressar 2011 como soma de n inteiros positivos e distintos? A) 59 B) 60 C) 61 D) 62 E) 63 20. Qual a maior quantidade de nmeros do conjunto 1,2,3,,20 que podemos escolher de modo que nenhum deles seja o dobro do outro? A) 10 B) 11 C) 12 D) 13 E) 14 21. Seja ABC um tringulo retngulo em A. O ponto D pertence ao lado AC e tal que BD = CD. Sejam M o ponto mdio de BC e N a interseo de AM e BD. Sendo N o ponto mdio de AM, qual a medida, em graus, do A ? ngulo BC A) 15 B) 22,5 C) 30 D) 37,5 E) 45 22. Sendo a e b inteiros tais que 1 2 A) a 2b (a b) 2 C) a 2b (b a) 2 E) a 2b (a b) 2
EUREKA! N36, 2012

B M N A D C

2011

a b 2 , 1 2

2010

igual a

B) a 2b (a b) 2 D) 2b a (b a) 2

13

Sociedade Brasileira de Matemtica

23. Se a, b e c so inteiros positivos tais que a b c e o menor valor possvel de a? A) 2011 B) 2012 C) 2013 D) 2014

1 1 1 1 , qual a b c 2011

E) 2011 2012

24. Trs polgonos regulares, de 8, 12 e 18 lados respectivamente, esto inscritos em uma mesma circunferncia e tm um vrtice em comum. Os vrtices dos trs polgonos so marcados na circunferncia. Quantos vrtices distintos foram marcados? A) 20 B) 24 C) 26 D) 28 E) 30 25. Seja ABCD um quadriltero inscritvel (ou seja, cujos vrtices pertencem a uma circunferncia) com AB = 4, BC = 8 3 , AC = 4 13 e AD = 2 13 . Sendo E a interseo das diagonais AC e BD, o comprimento do segmento BE : 13 3 15 3 12 3 16 3 A) B) C) 2 3 D) E) 7 7 7 7

GABARITO
NVEL 1 (6 ou 7 anos do Ensino Fundamental)

1) A 2) B 3) E 4) B 5) B 1) B 2) E 3) B 4) B 5) C 1) E 2) C 3) B 4) D 5) D

6) A 7) E 8) C 9) E 10) D 6) D 7) E 8) B 9) E 10) D 6) C 7) D 8) B 9) D 10) D

11) D 12) D 13) D 14) A 15) A 11) B 12) B 13) D 14) D 15) C 11) E 12) C 13) B 14) A 15) A

16) E 17) C 18) D 19) B 20) C 16) C 17) D 18) C 19) C 20) C 16) B 17) A 18) A 19) B 20) C 21) A 22) D 23) D 24) C 25) B 21) D 22) A 23) B 24) E 25) B

NVEL 2 (8 ou 9 anos do Ensino Fundamental)

NVEL 3 (Ensino Mdio)

EUREKA! N36, 2012

14

Sociedade Brasileira de Matemtica

XXXIII OLIMPADA BRASILEIRA DE MATEMTICA


Problemas e solues da Segunda Fase
PROBLEMAS NVEL 1 PARTE A (Cada problema vale 5 pontos) 01. Na figura, os vrtices do retngulo PQRS

pertencem aos lados do retngulo ABCD. Sendo AP = 3cm, AS = 4 cm, SC = 6 cm e CR = 8 cm, qual a rea do retngulo PQRS, em cm2?

02. Em cada vrtice de um cubo foi escrito um nmero. Esmeralda calcula a soma

dos nmeros escritos nos vrtices de cada face e encontra os nmeros 8, 10, 11, 12, 13 e x. Se a face de soma 8 oposta face de soma x, qual o valor de x?
03. Duas tribos vivem numa ilha. Os da tribo azul s dizem a verdade e os da

vermelha, s mentira. Um dia, 100 pessoas da ilha se reuniram num crculo e um reprter se dirigiu a cada uma delas, com a pergunta: O seu vizinho direita um mentiroso?. Terminada a pesquisa, verificou-se que 48 pessoas responderam sim. No mximo, quantas pessoas da tribo vermelha poderiam estar no crculo?
04. Com cubinhos de mesmo tamanho construiu-se um cubo 4 4 4 . Os cubinhos

so feitos de madeiras diferentes e foram colados assim: cubinhos com trs cubos vizinhos (cubos com faces comuns) pesam 10 gramas, com quatro vizinhos pesam 8 gramas, com cinco vizinhos pesam 6 gramas e com seis vizinhos pesam 4 gramas. Qual a massa do cubo, em gramas?
05. Quantos nmeros de trs algarismos diferentes de zero tm pelo menos dois

algarismos iguais?
06. Dizemos que dois ou mais nmeros so irmos quando tm exatamente os

mesmos fatores primos. Por exemplo, os nmeros 10 2 5 e 20 22 5 so irmos, pois tm 2 e 5 como seus nicos fatores primos. O nmero 60 tem quantos irmos menores do que 1000?

EUREKA! N36, 2012

15

Sociedade Brasileira de Matemtica

PROBLEMAS NVEL 1 PARTE B (Cada problema vale 10 pontos) PROBLEMA 1

A sequncia 1, 5, 4, 0, 5, ... formada pelos algarismos das unidades das somas a seguir
12 1 12 22 5 12 22 32 14 12 22 32 42 30 12 22 32 42 52 45 12 22 32 42 52 ??

a) Escreva a sequncia formada pelos algarismos das unidades das dez primeiras somas obtidas da forma descrita acima. b) Qual o algarismo das unidades da soma 12 22
PROBLEMA 2

20112 ?

Vamos chamar de selo de um nmero inteiro positivo o par x; y no qual x o nmero de divisores positivos desse nmero menores do que ele e y a soma desses divisores. Por exemplo, o selo do nmero 10 3;8 pois o nmero 10 tem como divisores menores do que ele os nmeros 1, 2 e 5, cuja soma 8. J o selo do nmero primo 13 1;1 . a) Qual o selo do nmero 9? b) Qual nmero tem o selo 2;3 ? c) H nmeros cujo selo 6; m . Qual o menor valor possvel para m?
PROBLEMA 3

Amarrando um pedao de barbante em um dos pregos do seu geoplano, Diamantino consegue formar quadrados, sem passar o barbante duas vezes pelo mesmo lado desses quadrados. A figura ao lado mostra um quadrado obtido desta maneira.
EUREKA! N36, 2012

16

Sociedade Brasileira de Matemtica

A figura abaixo representa de forma simplificada uma parte do geoplano em que foram obtidos dois quadrados da maneira descrita acima, partindo-se de qualquer um dos pregos.

a) Desenhe, na parte do geoplano representada ao lado, a maior quantidade de quadrados iguais que Diamantino pode obter com um nico pedao de barbante. Coloque as flechinhas como no exemplo para indicar como foi colocado o barbante.

b) Diamantino garante que pode obter 11 quadrados no seu geoplano. Mostre que voc tambm pode obter a mesma quantidade na figura abaixo. No se esquea das flechinhas no desenho.

PROBLEMAS NVEL 2 PARTE A (Cada problema vale 5 pontos) 01. Sejam a e b nmeros reais no nulos tais que a equao x + ax + b = 0 possui
2

solues a e b. Determine a b.
02. Quantos nmeros compostos de dois algarismos distintos podem ser formados

usando os algarismos 2, 3, 4, 5 e 6?
EUREKA! N36, 2012

17

Sociedade Brasileira de Matemtica

03. O tringulo ABC retngulo em B. As bissetrizes interna e externa do ngulo

BC cortam a reta BC em D e E, respectivamente. Dado que AD = 360 e AE = 480, determine a medida do lado AB.
04. O nmero 7, quando elevado quarta potncia, termina com 01: 7 = 2401.
4

Quantos so os nmeros de dois algarismos que, quando elevados quarta potncia, terminam com 01?
C mede 30 e o 05. Na figura a seguir, o tringulo ABC equiltero, o ngulo BD

D mede 70. Determine, em graus, a medida do ngulo BD. ngulo AC


A

30 B 70 C

PROBLEMAS NVEL 2 PARTE B (Cada problema vale 10 pontos) PROBLEMA 1

Inicialmente o nmero 5 est escrito na tela de um computador. Em qualquer momento, o nmero n escrito na tela do computador pode ser trocado por qualquer nmero da forma a b sendo a e b inteiros positivos tais que a + b = n. a) Mostre como obter o nmero 19 realizando tais operaes. b) possvel obter o nmero 2011? No se esquea de justificar sua resposta.
PROBLEMA 2

Sejam a, b e c nmeros reais positivos tais que a(b + c) = 152, b(c + a) = 162 e c(a + b) = 170. Determine o valor de abc.

EUREKA! N36, 2012

18

Sociedade Brasileira de Matemtica

PROBLEMA 3

Quantos so os pares ordenados (a,b), com a, b inteiros positivos, tais que


a b mdc(a, b) 33 ?
PROBLEMA 4

No quadriltero convexo ABCD, os pontos X e Y dividem o lado AB em trs segmentos iguais enquanto que os pontos Z e T dividem o lado DC em trs segmentos iguais (veja a figura abaixo). Se a rea do quadriltero ABCD 60, mostre que a rea do quadriltero XYZT no depende do formato do quadriltero ABCD e calcule tal rea.
B Y X A

PROBLEMAS NVEL 3 PARTE A (Cada problema vale 4 pontos) 01. A equao do segundo grau x 5x + m = 2011 tem pelo menos uma soluo
2

inteira. Qual o menor valor inteiro positivo possvel de m?


02. Uma sequncia de letras, com ou sem sentido, dita alternada quando

formada alternadamente por consoantes e vogais. Por exemplo, EZEQAF, MATEMTICA, LEGAL e ANIMADA so palavras alternadas, mas DSOIUF, DINHEIRO e ORDINRIO no so. Quantos anagramas da palavra FELICIDADE (incluindo a palavra FELICIDADE) so sequncias alternadas?
03. O ngulo interno do vrtice A de um tringulo acutngulo ABC mede 75 graus.

A altura relativa ao vrtice A toca o lado BC no ponto D. As distncias de D ao vrtice B e ao ortocentro do tringulo so ambas iguais a 10 cm. Qual a rea do tringulo ABC, aproximada para o inteiro mais prximo? Se necessrio, use 3 1,732 .

EUREKA! N36, 2012

19

Sociedade Brasileira de Matemtica

04. Qual o maior valor possvel do mdc de dois nmeros distintos pertencentes ao

conjunto {1,2,3,,2011}?
05. Seja f uma funo dos reais no nulos nos reais no nulos tal que

( f ( x) f ( y) f ( z)) 2 ( f ( x)) 2 ( f ( y)) 2 ( f ( z)) 2 para todos x, y, z reais no nulos tais que x + y + z = 0; f ( x) f ( x) para todo x real no nulo; f(2011) = 1. Encontre o inteiro mais prximo de f(33).
PROBLEMAS NVEL 3 PARTE B (Cada problema vale 10 pontos) PROBLEMA 1

No tringulo ABC, o ngulo BC mede 45. O crculo de dimetro BC corta os lados AB e AC em D e E, respectivamente. Dado que DE = 10, encontre a distncia do ponto mdio M de BC reta DE.
PROBLEMA 2

Encontre todas as solues reais (x, y, z) do sistema


1 x 1 2z y y 1 2x z z 2y x
PROBLEMA 3

Seja P(x) um polinmio de coeficientes inteiros. Sabe-se que P(x) = 2011 tem pelo menos duas razes inteiras distintas iguais a 1 e t, e que P(x) = 0 tem pelo menos uma raiz inteira. Determine todos os possveis valores de t.
PROBLEMA 4

Esmeralda tem um crculo de cartolina dividido em n setores circulares, numerados de 1 a n, no sentido horrio. De quantas maneiras Esmeralda pode pintar a cartolina, pintando cada setor com uma cor, tendo disponveis k cores e de modo que quaisquer dois setores circulares vizinhos (isto , que tm um segmento em
EUREKA! N36, 2012

20

Sociedade Brasileira de Matemtica

comum como fronteira) tenham cores diferentes? Note que isso implica que os setores de nmeros 1 e n devem ter cores diferentes.
SOLUES NVEL 1 SEGUNDA FASE PARTE A Problema Resposta 01 0050 02 0015 03 0076 04 0448 05 0225 06 0018

01. Por simetria, os tringulos APS e DRQ so congruentes, assim como os

tringulos SCR e QBP. Assim, os lados do retngulo ABDC so AS + SC = 4 + 6 = 10 cm e CR + RD = CR + AP = 8 + 3 = 11 cm. Deste modo, a rea do retngulo PQRS obtida subtraindo as reas dos tringulos APS, DRQ, SCR e QBP da rea 3 4 68 do retngulo ABDC, ou seja, 8 11 2 2 50 cm2. 2 2 02. Se em cada face estiver escrita a soma dos nmeros dos vrtices correspondentes a face, ento a soma dos nmeros em duas faces opostas igual a soma dos nmeros de todos os vrtices do cubo. Logo se 8 e x um par de faces opostas, ento outro par de faces opostas 10 e 13 e o terceiro par 11 e 12, para que 10 + 13 = 11 + 12 = soma dos nmeros em todos os vrtices. Portanto 8 x 23 x 15 .
03. Observe que se uma pessoa responde sim, ento esta pessoa e a da direita no

so da mesma tribo, mas se responder no, ento ela e a pessoa sua direita so da mesma tribo. Assim, se 48 pessoas responderam sim, ento ao percorrer o crculo no sentido horrio, observaremos 48 trocas de cor da tribo. Para que haja 48 trocas, devem haver pelo menos 24 pessoas da tribo azul e 24 da tribo vermelha dispostas alternadamente. Como queremos o mximo de pessoas da tribo vermelha, ento podemos colocar as 100 24 24 = 52 pessoas restantes juntas num mesmo bloco vermelho, como indicado a seguir: AVAVA VAV VV VV
24 AV 's 52V 's

Logo h no mximo 100 24 = 76 pessoas da tribo vermelha.


04. No cubo 4 4 4 , h 8 cubinhos nos vrtices (que tem 3 vizinhos), 2 12 24

cubinhos nas arestas (que tem 4 vizinhos), 4 6 24 cubinhos nas faces (que tem 5 vizinhos) e 8 cubinhos no interior do cubo maior (que tem 6 vizinhos). Assim, o cubo maior pesa 8 10 24 8 24 6 8 4 448g.

EUREKA! N36, 2012

21

Sociedade Brasileira de Matemtica

H 9 9 9 729 nmeros de trs algarismos no nulos. Destes, 9 8 7 504 tem os trs algarismos distintos. Portanto, h 729 504 225 nmeros com pelo menos dois algarismos iguais.
05. 06. 60 2 2 3 5 tem os fatores 2, 3 e 5, logo os irmos de 60 so mltiplos de

1000 2 3 5 30 . Como h 33 mltiplos de 30 menores que 1000, ento 60 30 tem no mximo 32 irmos. Destes mltiplos, os que tem outros fatores alm de 2, 3 e 5 so 7 30 , 11 30 , 13 30 , 14 30 , 17 30 , 19 30 , 21 30 , 22 30 , 23 30 , 26 30 , 28 30 , 29 30 , 31 30 e 33 30 . Logo, 60 tem 32 14 18 irmos.

SOLUES NVEL 1 SEGUNDA FASE PARTE B PROBLEMA 1

a) Para calcular os termos, basta considerar os dgitos das unidades na soma e no resultado. Assim, como os dgitos das unidades de 12 ,2 2 ,,102 so 1, 4, 9, 6, 5, 6, 9, 4, 1 e 0, ento comeando por 1, temos: 1, 1 + 4 = 5, 5 + 9 = 14, 4 + 6 = 10, 0 + 5 = 5, 5 + 6 = 11, 1 + 9 = 0, 0 + 4 = 4, 4 + 1 = 5 e 5 + 0 = 5, logo os 10 primeiros termos da seqncia so 1, 5, 4, 0, 5, 1, 0, 4, 5 e 5. b) Observe que a partir do 11 termo, vamos comear a somar novamente os dgitos 1, 4, 9, 6, 5, 6, 9, 4, 1 e 0, j que os dgitos das unidades de 112 ,122 ,,202 , so os mesmos dgitos das unidades de

12 ,22 , ,102 .

Assim,

na

soma

12 2 2 20112 , faremos as somas dos dgitos das unidades de 12 a 102 2011 2 10 201vezes e adicionaremos 1 de 2011 . Assim, o algarismo das unidades da soma 12 2 2 20112 o mesmo algarismo das unidades de 201 1 4 9 6 5 6 9 4 1 0 1 201 25 1 5026, que 6.
PROBLEMA 2

a) Os divisores positivos de 9 menores que 9 so 1 e 3, logo o selo do nmero 9 o par (2, 4). b) Observe que todo nmero inteiro positivo tem 1 como divisor. Como o nmero que estamos procurando tem apenas dois divisores menores que ele, 1 ter que ser um desses divisores e como a soma dos dois divisores 3, ento o outro divisor deve ser 2. Como no h outros divisores, ento o nmero que procuramos uma
EUREKA! N36, 2012

22

Sociedade Brasileira de Matemtica

potncia de 2, e para ter apenas dois divisores menores que ele prprio, ento o nmero deve ser 4. c) Seja n um nmero com selo (6, m). n possui 7 divisores contando com ele prprio, logo a nica possibilidade que ele seja da forma p6, com p primo, e m igual a 1 p p 2 p 5 . Para que m seja mnimo, p ter que ser mnimo, logo p = 2 e m 1 2 2 2 25 63 .
PROBLEMA 3

a) Observe que para cada prego do geoplano deve entrar e sair o mesmo nmero de flechas (o barbante ao passar por um prego deve entrar em uma direo e sair em outra), com exceo de onde comea e termina o barbante. Logo nos pregos onde no comea ou termina o barbante temos um nmero par de flechas, metade entrando e metade saindo. Mas no geoplano 4 4 , h 8 pregos com 3 arestas cada (os da borda do geoplano), logo em 6 deles haver pelo menos uma aresta por onde o barbante no pode passar. No melhor caso, conseguimos fazer com que um quadrado contenha 2 dessas arestas, assim no poderemos completar 3 quadrados. Na figura abaixo temos um exemplo onde 9 3 = 6 quadrados so formados, em que o barbante comea no vrtice S e termina no vrtice T:

b) Uma maneira de construir 11 quadrados com o barbante est descrita abaixo:

EUREKA! N36, 2012

23

Sociedade Brasileira de Matemtica

SOLUES NVEL 2 SEGUNDA FASE PARTE A

Problema Resposta

01 3

02 17 ou 20

03 288

04 6

05 0225

06 100

01. Utilizando o produto das razes, obtemos:

ab b a 1 pois b 0. Utilizando a soma das razes, obtemos: a b a 1 b 1 a b 3


02. Todos os nmeros terminados em 2, 4, 5 e 6 so compostos. Existem 4 4 16

tais nmeros. Dos nmeros terminados em 3, apenas 63 composto. O enunciado apresenta uma ambiguidade. Outra interpretao seria considerar os nmeros de dois algarismos constitudos por dois nmeros distintos do conjunto {2,3,4,5,6}. Nesse caso, a resposta correta 5 4 = 20. Ambas as respostas devem ser consideradas corretas.
03.
A 90 x 90 E B y D C

Da semelhana dos tringulos ABD e EAD obtermos:


360 y 3 y x 480 x 4

Pelo Teorema de Pitgoras no tringulo ABD , temos:


x2 y 2 3602
9 2 2 x 360 16 x 288 x2

EUREKA! N36, 2012

24

Sociedade Brasileira de Matemtica

04. Um nmero de dois dgitos (ab) elevado quarta potncia possui a seguinte

forma:

10a b

104 a 4 4 103 a3b 6 102 a 2b2 4 10ab3 b4

Assim, os ltimos dois dgitos so determinados por 4 10ab3 b4 . Qualquer nmero mpar diferente de 5 elevado quarta potncia termina em 1. Logo, temos quatro possibilidades para b: Se b 1 , para o nmero 40a 1 terminar em 01 devemos ter a 5. Se b 3 , para o nmero 1080a 81 terminar em 01devemos ter a 4 ou a 9 . Se b 7 , para o nmero 13720a 2401 terminar em 01 devemos ter a 5 . Se b 9 , para o nmero 29160a 6561 terminar em 01 devemos ter a 4 ou a 9.
05. Como BDC mede 30 e B AC 60, o ponto D est no crculo de centro A e

raio AB. Como o tringulo ACD issceles com ngulo da base igual a 70, temos

C AD 40 e BAD 100
SOLUES NVEL 2 SEGUNDA FASE PARTE B SOLUO DO PROBLEMA 1

a) Uma possvel sequncia de operaes :


5 6 9 20 19
23 3 3 4 5 19 1

b) Perceba que se possvel obtermos o nmero


n 1 1

tambm possvel obtermos o

nmero n 1 com a operao n n 1 e consequentemente poderemos obter todos os inteiros positivos menores que n 1 repetindo essa operao. Ento suficiente obtermos um inteiro maior que 2011 comeando em 5. Uma possvel sequncia de operaes para isso seria:
5 6 8 16 64 1024 2044.
SOLUO DO PROBLEMA 2
23 4 2 4 4 8 8 32 32 1022 2

Somando as trs equaes obtemos: 2ab 2bc 2ca 484 Da, ab 242 c a b 72,
bc 242 a b c 90, ca 242 b a c 80.
EUREKA! N36, 2012

25

Sociedade Brasileira de Matemtica

Logo, abc 72 90 80 e abc 720.


2

SOLUO DO PROBLEMA 3

Seja d mdc a, b . Podemos reescrever a equao como:


a b 33 1 . d d d

O lado esquerdo uma soma de nmeros inteiros logo, divide 33. Agora temos que mdc a / d , b / d mdc a / d ,33 / d 1 mdc b / d ,33 / d 1 1. Fixado d , suficiente encontrarmos os pares de inteiros positivos x, y com mdc x,33 / d 1 1 tais que x y 33 / d 1 pois da obteremos tambm que mdc y,33 / d 1 1 e que a, b dx, dy soluo. Vejamos ento as possibilidades para d : Para d 1 e
a b 33 , temos 16 solues, pois basta escolhermos x 1 d d d

mpar. Para d 3 e x y 10, temos 4 solues pois x no pode ser par nem mltiplo de 5. Para d 11 e x y 2, temos 1 soluo apenas. No podemos ter d 33 , pois a e b so positivos. Logo, existem 21 pares de solues.
SOLUO DO PROBLEMA 4
B Y X A

D
EUREKA! N36, 2012

26

Sociedade Brasileira de Matemtica

Temos as seguintes relaes de reas: 1 1 ADT BYC ADC ABC ABCD 20. 3 3 Portanto a rea do quadriltero ATCY igual 40. Alm disso,

Consequentemente, XTZY ATCY ATX YZC 20.

ATX YZC ATY YTC ATCY 20. 2 2


1 1

Segunda Soluo Sejam P a interseo de AB e DC, a PA, b PD, x AX e y DT . Temos:


2 ABCD / senAPD a 3x b 3 y ab 9 xy 3bx 3ay 2 XYZT / senAPD a 2 x b 2 y a x b y bx ay 3xy

Logo a rea do quadriltero XYZT um tero da rea do quadriltero ABCD.


SOLUES NVEL 3 SEGUNDA FASE PARTE A

Problema Resposta

01 0037

02 3600

03 0237
2

04 1005

05 0061
2

01. Como m inteiro positivo, temos x 5x < 2011. Sendo x inteiro e 47 5 47 <

2011 < 482 5 48, devemos ter x2 5x < 2011 x 47. Assim, o menor valor de m 2011 (472 5 47) = 37.
02. As consoantes de FELICIDADE so F, L, C, D, D e as vogais so E, I, I, A, E.

As posies das vogais so as pares ou as mpares, as consoantes podem se 5! permutar entre si de 60 maneiras e as vogais podem se permutar de 1!1!1!2! 5! 30 maneiras. Assim, o total de anagramas alternados de FELICIDADE 2!1!2! 2 60 30 3600 .
03. Seja H o ortocentro do tringulo ABC. Ento BD = HD = 10 cm. Ento o

tringulo retngulo BDH issceles e, portanto,


EUREKA! N36, 2012

D) 45 . Logo, m( HB

27

Sociedade Brasileira de Matemtica

considerando o tringulo retngulo m(C ) 90 m( HBD) 90 45 45 .


D) m( A ) m( DA C ) 75 45 30 . m( BA D) 90 30 60 . ) 90 m( BA Enfim, m( B

de

hipotenusa

BC,

temos

C ) 90 m(C ) 90 45 45 e Assim, m( DA

H B D C

Assim, AD BD tg60 10 3 cm e CD = AD = 10 3 cm. Portanto a rea de ABC


BC AD (10 10 3 )10 3 150 50 3 150 50 1,732 236,6 cm2, 2 2 valor inteiro mais prximo 237 cm2.

cujo

04. O mdc de dois nmeros divisor de cada um dos dois nmeros, ou seja, cada

um dos dois nmeros mltiplo de seu mdc. Logo queremos o maior valor de d que tem dois mltiplos positivos menores ou iguais a 2011. O maior dos dois mltiplos de d maior ou igual a 2d, logo 2d 2011 d 1005. Como 1005 e 2 1005 = 2010 so ambos menores do que 2011, o valor procurado 1005.
05. A condio ( f ( x) f ( y) f ( z )) 2 ( f ( x)) 2 ( f ( y)) 2 ( f ( z )) 2 para x + y +

z = 0 equivalente a

1 1 1 0 . Como f(x) = f(x), sendo f ( x) f ( y ) f ( x y ) 1 1 1 1 temos g ( x) 0 g ( x y ) g ( x) g ( y ) . f ( x) f ( x) f ( y ) f ( x y )

Fazendo y = x, obtemos g(2x) = 2g(x). Fazendo y = 2x, obtemos g(3x) = g(2x) + g(x) = 2g(x) + g(x) = 3g(x) e, indutivamente, prova-se que g(nx) = ng(x) para n inteiro positivo. Fazendo x = 1 e n = 2011, obtemos g(2011) = 2011g(1). Como g (2011 ) 1 1 . Enfim, fazendo x = 1 e g (2011 ) 1 , temos g (1) 2011 2011 f (2011 )
EUREKA! N36, 2012

28

Sociedade Brasileira de Matemtica

1 33 1 2011 . Logo f (33) . 2011 2011 g (33) 33 Como 2011 = 3360 + 31, o inteiro mais prximo de f(33) 61.

n = 33, temos g (33) 33 g (1) 33

SOLUES NVEL 3 SEGUNDA FASE PARTE B PROBLEMA 1

A 45 E D d B M C

C ) m(CB ) m( DE ) 2 45 180 m( DM E ) m( DM E ) 90 . Temos m( BA 2 Logo o tringulo DME retngulo e, sendo M o centro do crculo, issceles. Ento, sendo a projeo de M sobre DE o ponto mdio de DE e, portanto, circuncentro de DME. Logo DE 2d 10 2d d 5 .
PROBLEMA 2

Somando as trs equaes, obtemos 1 1 1 1 1 1 2x 2 y 2z x y z x y z . x y z x y z


1 1 Note que x, y e z tm o mesmo sinal. De fato, se x > 0 ento y x 0 e 2 x 1 1 z y 0 . Analogamente, se x < 0 ento y < 0 e z < 0. 2 y
EUREKA! N36, 2012

29

Sociedade Brasileira de Matemtica

1 1 Agora, veja que, pela desigualdade das mdias, y x 2 x 1 analogamente, x 1 e z 1 . Mas isso implicaria 1, x

x 1 1 e, x 1 1 1 e 1, y z

x y z x y z 3 e
Mas x y z

1 1 1 1 1 1 3. x y z x y z

1 1 1 , logo todas as desigualdades anteriores so igualdades, x y z

ou seja, x y z 1 . Lembrando que x, y e z tm o mesmo sinal, as nicas possibilidades so x = y = z = 1 e x = y = z = 1. Verifica-se facilmente que as duas possibilidades so realmente solues.

PROBLEMA 3

Seja Q(x) = P(x) 2011. Ento Q(x) = 0 tem coeficientes inteiros e duas de suas razes so 1 e r. Logo Q(x) = (x 1)(x r)R(x), sendo R(x) um polinmio de coeficientes inteiros e, portanto, P(x) = (x 1)(x r)R(x) + 2011. Como P(x) = 0 (x 1)(x r)R(x) = 2011 tem solues inteiras, e R(x) inteiro para x inteiro, x 1 e x r so dois divisores distintos (no necessariamente positivos) de 2011. Sendo 2011 primo, cada um desses dois fatores pode ser 2011, 1, 1 ou 2011, com a nica restrio sendo que eles no podem ser 2011 e 2011 simultaneamente. Assim, (x 1) (x r) = r 1 pode ser igual a 2010, 2010, 2012, 2012, 2 ou 2, ou seja, r pode ser igual a 2011, 2009, 2013, 2011, 3 ou 1.
PROBLEMA 4

Para n = 1, temos k possibilidades (basta escolher a cor da regio 1); para n = 2, h k(k 1) possibilidades (k escolhas para a regio 1 e k 1 para a regio 2, que deve ter cor diferente da regio 1). Suponha n 3 e seja an a quantidade desejada de maneiras de pintar um crculo dividido em n setores, sem que haja setores vizinhos de mesma cor. Pintemos a regio de qualquer uma das k cores e cada uma das regies 2, 3, , n de qualquer uma das k 1 cores diferentes da cor da regio anterior. Observe a cor da regio n: se a cor diferente da cor da regio 1, obtemos uma pintura vlida com n setores; se a cor igual cor da regio 1, se juntarmos a regio 1 e a regio n obtemos uma pintura vlida com n 1 setores. Note que qualquer pintura com n
EUREKA! N36, 2012

30

Sociedade Brasileira de Matemtica

setores e qualquer pintura com n 1 setores obtida de maneira nica com esse procedimento. Assim, an + an1 = k(k 1)n1 para n 3. Agora aplique a igualdade repetidas vezes: an + an1 = k(k 1)n1 an1 an2 = k(k 1)n2 an2 + an3 = k(k 1)n3 an3 an4 = k(k 1)n4 n+1 n+1 (1) a3 + (1) a2 = (1)n+1k(k 1)2 Somando as n 2 igualdades, obtemos an + (1)n+1a2 = k(k 1)2((k 1)n3 (k 1)n4 + + (1)n+1) = (1) n 1 ((k 1) n 2 1) k (k 1) 2 k 1 1 an (1) n k (k 1) (k 1) 2 (1) n ((1) n (k 1) n2 1) (k 1) n (k 1)(1) n Logo, considerando que a frmula obtida vale para n 2, temos k , se n 1 an n n (k 1) (k 1)(1) se n 3

EUREKA! N36, 2012

31

Sociedade Brasileira de Matemtica

XXXIII OLIMPADA BRASILEIRA DE MATEMTICA Problemas e solues da Terceira Fase


TERCEIRA FASE NVEL 1 PROBLEMA 1

Esmeralda escreveu no quadro negro a sequncia de todos os nmeros inteiros de 1 a 2011. Em seguida, apagou todos os nmeros pares da lista. a) Quantos nmeros restaram? b) Dos nmeros restantes, quantos foram escritos apenas com os algarismos 0 e 1?
PROBLEMA 2

Temos um cubo vermelho de aresta 2 cm. Qual o nmero mnimo de cubinhos


12 iguais que devemos juntar ao vermelho para obter um cubo de volume cm3? 5
PROBLEMA 3
3

Dizemos que um nmero inteiro positivo chapa quando ele formado apenas por algarismos no nulos e a soma dos quadrados de todos os seus algarismos tambm um quadrado perfeito. Por exemplo: - o nmero 2115522 chapa, pois 22 12 12 52 52 22 22 82 e todos os seus algarismos so no nulos (diferentes de zero); - o nmero 403 no chapa, pois, apesar de 42 02 32 52 , um dos algarismos de 403 nulo (igual a zero); - o nmero 12 no chapa, pois 12 22 5 e 5 no um quadrado perfeito. a) Qual o maior inteiro positivo com dois algarismos que chapa? b) Existe um inteiro positivo com 2011 algarismos que chapa? Justifique sua resposta.

EUREKA! N36, 2012

32

Sociedade Brasileira de Matemtica

PROBLEMA 4

Na

figura,

centro

do

quadrado,

OA OC 2, AB CD 4, CD perpendicular a OC
que perpendicular a OA , que perpendicular a AB . A rea do quadrado 64 cm2. a) Calcule a rea do trapzio ABCO. b) Calcule a rea do quadriltero BCDE.
PROBLEMA 5

Num tabuleiro 3 3 escrevemos os nmeros de 1 a 9, um em cada casa. Em seguida, achamos a soma dos nmeros de cada linha, de cada coluna e de cada diagonal e contamos o nmero de somas que so mltiplos de trs. Por exemplo, no tabuleiro ao lado as 8 somas (as trs linhas, as trs colunas e as duas diagonais) so nmeros mltiplos de 3.

a) Copie o tabuleiro ao lado na sua folha de respostas e o preencha com os nmeros de 1 a 9 de modo existam exatamente 3 somas que so nmeros mltiplos de 3.

b) possvel que nenhuma das 8 somas seja um mltiplo de 3? Lembre-se de que voc deve justificar sua resposta.
TERCEIRA FASE NVEL 2 PRIMEIRO DIA PROBLEMA 1

Num tabuleiro 3 3 escrevemos os nmeros de 1 a 9, um em cada casa. Em seguida, achamos a soma dos nmeros de cada linha, de cada coluna e de cada diagonal e contamos o nmero de somas que so mltiplos de trs. Por exemplo, no tabuleiro ao lado as 8 somas (as trs linhas, as trs colunas e as duas diagonais) so nmeros mltiplos de 3.

EUREKA! N36, 2012

33

Sociedade Brasileira de Matemtica

possvel que nenhuma das 8 somas seja um mltiplo de 3? Lembre-se de que voc deve justificar sua resposta.
PROBLEMA 2

Seja ABCD um quadriltero convexo tal que AD = DC, AC = AB e ADC CAB. Se M e N so os pontos mdios dos lados AD e AB, prove que o tringulo MNC issceles.
PROBLEMA 3

Esmeralda e Jade participam de um jogo: Esmeralda faz uma lista de 2011 nmeros inteiros positivos, mas no mostra para Jade. Jade deve descobrir o produto dos nmeros. Para isso, ela pode perguntar qual o mdc ou o mmc dos nmeros de qualquer subconjunto com mais de um elemento dos 2011 nmeros (por exemplo, qual o mdc do 1, 2, 10 e 2000 nmeros da sua lista? ou qual o mmc de todos os nmeros da lista?). Jade pode fazer quantas perguntas quiser, mas s obtm as respostas (corretas) de Esmeralda aps fazer todas as suas perguntas (Esmeralda generosa e tambm diz qual a resposta de cada pergunta). Jade ento pode fazer qualquer uma das quatro operaes fundamentais (soma, subtrao, multiplicao e diviso) com os nmeros que obtiver de Esmeralda. Jade consegue uma estratgia para obter o produto dos 2011 nmeros de Esmeralda? Justifique sua resposta.
SEGUNDO DIA PROBLEMA 4

Esmeralda escreveu uma lista de nmeros inteiros positivos em uma folha de papel. Renan percebeu que todos os nmeros da lista e todas as somas de qualquer quantidade de nmeros distintos da lista no eram divisveis por nenhum quadrado perfeito diferente de 1. Qual a quantidade mxima de nmeros na lista de Esmeralda?
PROBLEMA 5

No interior de um quadrado de lado 16 so colocados 1000 pontos. Mostre que possvel colocar um tringulo equiltero de lado 2 3 no plano de modo que ele cubra pelo menos 16 destes pontos.

EUREKA! N36, 2012

34

Sociedade Brasileira de Matemtica

PROBLEMA 6

Para qualquer nmero natural N de 2k dgitos, seja I(N) o nmero de k dgitos obtido escrevendo os algarismos de ordem mpar de N da esquerda para a direita e P(N) como o nmero de k dgitos obtido escrevendo os algarismos de ordem par de N da esquerda para a direita. Por exemplo, I(249035) = 405 e P(249035) = 293. Provar que no possvel encontrar um natural N de 2k algarismos tal que

N I ( N ) P( N ).

TERCEIRA FASE NVEL 3 PRIMEIRO DIA PROBLEMA 1

Dizemos que um nmero inteiro positivo chapa quando ele formado apenas por algarismos no nulos e a soma dos quadrados de todos os seus algarismos tambm um quadrado perfeito. Por exemplo, 221 chapa pois 22 + 22 + 12 = 9 e todos os seus algarismos so no nulos, 403 no chapa, pois, apesar de 42 + 02 + 32 = 52, um de seus algarismos de 403 nulo e 12 no chapa pois 12 + 22 = 5 no quadrado perfeito. Prove que, para todo inteiro positivo n, existe um nmero chapa com exatamente n algarismos.
PROBLEMA 2

Um lbum, composto por 2011 figurinhas, est sendo colecionado por 33 amigos. Uma distribuio de figurinhas entre os 33 amigos incompleta quando existe pelo menos uma figurinha que nenhum dos 33 amigos tem. Determinar o menor valor de m com a seguinte propriedade: toda distribuio de figurinhas entre os 33 amigos tal que, para quaisquer dois dos amigos, faltam, para ambos, pelo menos m figurinhas em comum, incompleta.
PROBLEMA 3

Mostre que, para todo pentgono convexo P1P2P3P4P5 de rea 1, existem dois tringulos PiPi+1Pi+2 e PjPj+1Pj+2 (em que P6 = P1 e P7 = P2), formados por trs vrtices consecutivos do pentgono, tais que 5 5 rea Pi Pi 1 Pi 2 rea Pj Pj 1 Pj 2 10

EUREKA! N36, 2012

35

Sociedade Brasileira de Matemtica

TERCEIRA FASE NVEL 3 SEGUNDO DIA PROBLEMA 4

Existem 2011 inteiros positivos a1 < a2 < < a2011 tais que, para todo 1 i < j 2011, mdc(ai, aj) = aj ai?
PROBLEMA 5

Seja ABC um tringulo acutngulo e H seu ortocentro. As retas BH e CH cortam AC e AB em D e E, respectivamente. O circuncrculo de ADE corta o circuncrculo de ABC em F A. Provar que as bissetrizes internas de BFC e BHC se cortam em um ponto sobre o segmento BC.
PROBLEMA 6

Sejam x1, x2, x3, , x2011 reais no negativos cuja soma

2011 . Prove que 2

(x x
i cc

i 1 )

( x1 x2 )(x2 x3 )(x3 x 4 )(x4 x5 ) ( x2009 x 2010)(x2010 x2011)(x2011 x1 )

3 3 . 16

SOLUES NVEL 1 PROBLEMA 1: SOLUO ADAPTADA DE MATEUS SIQUEIRA TIMTEO (MOGI DAS CRUZES SP)

a) Podemos notar que restaro os nmeros mpares de 1 a 2011. Comeando a partir do 1, os nmeros inteiros, tomados dois a dois, so compostos de um par e um mpar. Logo, a quantidade de nmeros restantes de

2010 1, cujo resultado 1006 2

nmeros. Vamos supor inicialmente que os algarismos 0 e 1 devem de fato aparecer. b) Podemos perceber que todos os nmeros com algarismos 0 e 1 apenas devero ter ao menos trs algarismos, j que devem comear a terminar com 1, pois, alm de terem restado apenas algarismos mpares, nenhum deles pode comear com zero, e devem ter ao menos um de seus algarismos zero. Nesse caso temos: Nmeros com trs algarismos (1): 101
EUREKA! N36, 2012

36

Sociedade Brasileira de Matemtica

Nmeros com quatro algarismos (3): 1001, 1101, 1011. Quatro desses nmeros foram escritos apenas com alagarismos 0 e 1. Se incluirmos os nmeros que tm apenas o algarismo 1, temos mais quatro nmeros: 1, 11, 111 e 1111.
PROBLEMA 2: SOLUO DE WILLIAM HIDEKI KONDO (SO PAULO SP)

O lado do cubo vermelho de 2cm. O lado de um cubo de volume igual a

12 3 cm igual a 5

12 12 2, 4cm. Ao juntar-se cubinhos iguais com o 5 5


12 3 cm , a aresta do cubo 5
3

cubo vermelho, formando um cubo de volume

vermelho aumenta 2, 4 2 0, 4cm. Dessa forma, formou-se trs tipos de blocos. Os blocos de dimenses 2 0, 4 2cm , cujos volumes so iguais a

2 0, 4 2 1,6cm3 e h 3 destes blocos, sobrepostos a trs faces do cubo


vermelho com um vrtice em comum. Os blocos de dimenses 0, 4 0, 4 2cm, cujos volumes so iguais a

2 0, 4 0, 4 0,32cm3 e h trs desses blocos, entre os blocos de dimenses 2 2 0, 4cm.


E um bloco cbico de 0,4cm de lado para preencher o espao que falta. Os cubinhos preenchero esses blocos. O menor lado de todos os blocos 0,4cm. Para se ter o menor nmero de cubinhos usados, os lados dos cubinhos tm que ser os maiores possveis, ou seja, 0,4cm. Dessa forma, o volume dos cubinhos seria 0, 4 0, 4 0, 4 0,064cm3.

1, 6 25 cubinhos destes em um bloco de dimenses 2 0, 4 2cm. 0, 064 Como so trs blocos, so necessrios 25 3 75 cubinhos. 0,32 Cabem 3 5 3 15 cubinhos em trs blocos de dimenses 0, 064 0, 064 1 cubinho no bloco de dimenses 0, 4 0, 4 2cm. E cabe 0, 064 0, 4 0, 4 0, 4cm. Logo, o nmero mnimo de cubinhos a serem usados 75 15 1 91 cubinhos.
Cabem

EUREKA! N36, 2012

37

Sociedade Brasileira de Matemtica

PROBLEMA 3: SOLUO DE WILLIAM HIDEKI KONDO (SO PAULO SP)

a) Nmeros de 2 algarismos: 10 a 99

9 92 81 81 162 92 82 81 64 145 92 72 81 49 130 92 62 81 36 117 92 52 81 25 106 92 42 81 16 97 92 32 81 9 90 92 22 81 4 85 92 12 81 1 82


2

82 92 145 82 72 64 49 113 82 62 64 36 100


86 : 82 62 102

86 o maior inteiro positivo com dois algarismos que chapa. b) Sim, possvel. O nmero seria 111...111322
2008 "uns"

Esse nmero seria chapa, pois 12 ...12 32 22 22


2008 vezes

2008 9 4 4 2025 452 .


PROBLEMA 4: SOLUO DE LAURA MELLO DURSO VIANNA (RIO DE JANEIRO RJ)

4 OABC 4 DCBE 64cm2 OABC DCBE 16cm2

4 4 2 2 O 2 2 2 2 4 C A

D
EUREKA! N36, 2012

38

Sociedade Brasileira de Matemtica

A princpio traamos quatro figuras iguais a AOCB e encontramos o quadrado inteiro. Ento ABCO BCDE 16cm2
P A 2 O 2 2 C 2
2cm2 2cm2 2cm
2

2 4 2

Traamos uma reta paralela a reta OA que a reta s. Ligamos o ponto C ao ponto A e dividimos a figura ABCO em trs tringulos retngulos issceles iguais, cada um com catetos medindo 2cm, e logo com rea igual a 2cm2 . a) Ento a rea de ABCO 2cm2 3 6cm2 . b) Ento a rea da figura BCDE 16cm2 6cm2 10cm2 , pois,

ABCO BCDE 16cm2 BCDE 16cm2 ABCO BCDE 16cm2 6cm2 BCDE 10cm2 .
PROBLEMA 5: a) SOLUO DE PEDRO HENRIQUE SACRAMENTO DE OLIVEIRA (VINHEDO SP)
5 7 3 8 9 6 2 4 1

b) Veja a soluo do problema 1 do nvel 2.


EUREKA! N36, 2012

39

Sociedade Brasileira de Matemtica

SOLUES NVEL 2 PROBLEMA 1: SOLUO DE ANA EMLIA HERNANDES DIB (SO JOS DO RIO PRETO SP)

No possvel. Basta analisarmos a tabela em mdulo 3:

1 1 mod 3

4 1 mod 3

7 1 mod 3
8 2 mod 3 9 0 mod 3

2 2 mod 3 3 0 mod 3

5 2 mod 3 6 0 mod 3

Para que a soma no seja um mltiplo de 3, no podem ocorrer os seguintes casos nas linhas, colunas ou diagonais:

0, 0, 0 1,1,1 2, 2, 2 0,1, 2 0, 2,1

1, 0, 2 1, 2, 0 2, 0,1 2,1, 0

Assim, conclumos que no poderamos colocar mais que dois nmeros congruentes (mod 3) na mesma linha, coluna ou diagonal, ou seja, cada linha, coluna e diagonal deveria ter dois nmeros congruentes e um diferente. Os possveis casos seriam (sendo x, y e z, em qualquer ordem, 0, 1 e 2), e levando em conta as simetrias que preservam as diagonais, como trocar as posies da primeira e da terceira linhas:
X X Y Z Y Z

Que no possvel, j que a soma da diagonal seria

0 mod 3 .

X Y

X Y Z Z

Que tambm no possvel, j que, como faltam 1X ,1Y e 1Z , a soma da diagonal seria 0 mod 3 .

EUREKA! N36, 2012

40

Sociedade Brasileira de Matemtica

X Y Z Z

X Y

Os outros casos incluem uma mesma linha ou coluna com a soma x y z 0 mod 3 .

Ou seja, impossvel que nenhuma das oito somas seja mltiplo de 3. Obs. Um mltiplo de 3 congruente a 0 mod 3 .
PROBLEMA 2: SOLUO ADAPTADA DA SOLUO DE ISABELLE FERREIRA DE OLIVEIRA (FORTALEZA CE)

Seja CDA CAN e P o ponto mdio de CD. Logo,

CA AB AN 2 2 Alem disso, como CDA issceles, CP MA. PM


B

N C
90 2

90 2 A M

Veja tambm que CPM 180 DPM 90 E que NAM MAC CAN 90

2 2 Portanto, CPM MAN LAL CM MN CMN issceles.

90

EUREKA! N36, 2012

41

Sociedade Brasileira de Matemtica

PROBLEMA 3: SOLUO DE DANIEL SANTANA ROCHA (RIO DE JANEIRO RJ)

Sim. Obs: a1 ,..., an denotar o mmc dos nmeros a1 ,..., an e a1 ,..., an o mdc dos nmeros a1 ,..., an . Lema 1: abc

a, b, c a, b a, c b, c a, b, c

para todos a, b, c inteiros positivos.

Prova: Seja p primo. Usaremos a notao Se p k || n significando que p k | n e

p k 1 | n. p1 || a , p2 || b e p3 || c

min1 , 2 min1 ,3 min 2 ,3 max1 , 2 ,3 min1 , 2 ,3

||

a, b, c a, b a, c b, c . a, b, c

Provaremos

que o expoente de p 1 2 3 . Supondo sem perda de generalidade

1 2 3 , o expoente do p 1 1 2 3 1 1 2 3. Logo, pelo


teorema fundamental da aritmtica vale a igualdade. Sejam a1 , a2 ,..., a2011 os nmeros de Esmeralda. Basta Jade perguntar o mdc ai , ai 1 e o mmc ai , ai 1 i 1,3,5,..., 2007 e perguntar tambm o

mdc a2009 , a2010 , a2011 ,

mmc a2009 , a2010 , a2011 ,

mdc a2009 , a2010 , mdc a2009 , a2011 , mdc a2010 , a2011 .


Multiplicando os primeiros 2 Jade obter

2007 1 2006 valores, pelo Lema para c 1, 2

mmc a1 , a2 mdc a1 , a2 mmc a3 , a4 mdc a3 , a4 ...mmc a2007 , a2008 mdc a2007 , a2008
a1a2 ...a2008 .

Multiplicando os 4 ltimos e dividindo pelo quinto de trs para frente Jade encontra

a2009 , a2010 , a2011 a2009 , a2010 a2009 , a2010 a2010 , a2011 a2009 , a2010 , a2011

a2009 a2010 a2011 , agora basta Jade multiplicar os dois valores encontrados e obter a1a2 ...a2011 , c.q.d..

EUREKA! N36, 2012

42

Sociedade Brasileira de Matemtica

PROBLEMA 4: SOLUO DE DANIEL SANTANA ROCHA (RIO DE JANEIRO RJ)

A quantidade mxima 3. Primeiro vamos escrever uma lista com 3 nmeros: 13, 17, 21. Como

13 17 21 30 21 51 3 17 13 17 30 2 3 5 13 21 34 2 17 17 21 38 2 19 13 13 17 17 21 3 7
Todos os nmeros so livres de quadrados (i.e. no possuem quadrados perfeitos diferentes de 1 como divisor), pois seno teriam um expoente > 1 na sua fatorao em primos. Agora suponha por absurdo que a lista de Esmeralda tenha mais que 3 nmeros. Ento existem pelo menos 4 nmeros digamos a, b, c, d . Note que 4 22 1 quadrado perfeito 4 | a, 4 | b, 4 | c, 4 | d . Logo, os a, b, c, d s podem ter resto

1, 2,3 na diviso por 4. Pelo princpio da casa dos pombos existem dois que
deixam o mesmo resto. Suponha, sem perda de generalidade, que sejam

a e b a b mod 4 a b 2a mod 4. Como 4 | a b 4 | 2a a mpar

2a 2 mod 4 se a 2k 1, 2a 4k 2 . Logo

a b 2mod 4.

Como

4| a b c c (a b) 2mod 4 e 4| a b d d (a b) 2mod 4. Como


c, d 0 mod 4, os restos da diviso por 4 de c e d so 1 ou 3. Se

c d mod 4 c 1 mod 4 e d 3mod 4 ou c 3mod 4 e d 1mod 4 . De qualquer forma, absurdo. Logo c d d c 0 mod 4 , 4 | 2d d e de mpar c d mod 4 c d 2d mod 4 c d 2d 2mod 4 a b c d 2 2 0 mod 4. Absurdo!
PROBLEMA 5: SOLUO DE FABIO DA SILVA SOARES (PLANALTINA DF)

Primeiro vamos encontrar a quantidade de tringulos necessrios para colorir o quadrado. Note que a altura do tringulo 3, pelo Teorema de Pitgoras, e sabemos que o lado do tringulo 2 3. Usando o seguinte esquema:
EUREKA! N36, 2012

43

Sociedade Brasileira de Matemtica

2 3

2 3

3
2 3
Usaremos 6 fileiras com 11 tringulos, 6 virados para cima e o resto para baixo. Obs: usamos 6 fileiras pois cada fileira tem altura 3 e com 5 no cobriremos o quadrado.

10 3 17,3 16

2 3

2 3

2 3

2 3

2 3

3 1
6 fileiras

10

11

Dessa forma, temos que a figura que fizemos capaz de cobrir o quadrado. Sabemos ainda que a figura tem 11 6 tringulos, 66. Ns usaremos essa figura pois o enunciado pede que o tringulo esteja no plano, assim, se metade do tringulo estiver dentro do quadrado e metade fora, o enunciado ainda vale. Cubramos ento o quadrado de lado 16 com a figura. Note que existem 66 tringulos que cobrem toda a sua superfcie. Pelo Princpio da Casa dos Pombos, temos que uma casa, ou tringulo, ter mais que 16 pontos, pois suponha que no acontecesse isso, todo tringulo teria no mximo 15 pontos nele, totalizando 15 66 990. Entretanto faltariam 10 pontos, pois o total de pontos mil. Logo, um tringulo ter no mnimo 16 pontos nele. c.q.d..

EUREKA! N36, 2012

44

Sociedade Brasileira de Matemtica

PROBLEMA 6: SOLUO DE ALESSANDRO PACANOWSKI (RIO DE JANEIRO RJ)

Vamos fazer por induo em k.

Se k = 1, vamos provar que sempre N I N P N .

N ab. Mas N 10a b ab a b P N I N a 10 b b 0. Mas


a 0,10 b 0; b 0 a 10 b b 0; s ocorrendo igualdade se b = 0 e

a 10 b 0. Como b 0 10 b 0 a 0 N 00, impossvel (N tem 2


dgitos). Olhemos agora para o caso com N com 2k dgitos. (Suponha, pela induo que, para todo e qualquer N, com 2 k 1 dgitos, N P N I N . ) Seja N xyN , onde x e y so algarismos e N um nmero com 2k 2 dgitos. Seja P N e I N P N x e I N y . Vamos provar que:

N P( N )I ( N ) N 102k 1 x 102k 2 y N P( N ) I ( N ) ( x 10k 1 )( y 10k 1 )


(N tem 2k 2 dgitos, N 102k 1 x 102k 2 y N; e tm, ambos, k 1

dgitos) 102k 1 x 102k 2 y N 102k 2 xy 10k 1 x y . Pela hiptese de


induo, N P N I N s precisamos provar que:

102k 1 x 102k 2 y 102k 2 xy 10k 1 x y . Como e tem ambos k 1 dgitos,


k 1 2k 2 k 1 2k 2 k 1 10k 1 1 x y temos que: , 10 1 10 xy 10 x y 10 xy 10

102k 2 xy 102k 2 x 102k 2 y 10k 1 x y . Ento, s precisamos provar que:


102k 1 x 102k 2 y 102k 2 xy 102k 2 x 102 k 2 y 10k 1 x y 102 k 1 x 10k 1 x y

102 k 2 x y 1 .
Agora, observamos que 102k 1 x 102 k 2 x y 1 10 y 1 , o que verdade, j que y 9. Assim, como x y 0 10k 1 x y 0 102k 1 x 10k 1 x y 102k 2 x y 1 .
SOLUES NVEL 3 PROBLEMA 1: SOLUO DE RAFAEL KAZUHIRO MIYAZAKI (SO PAULO SP)

Se n quadrado perfeito, n k 2 , o nmero : 555...55, cujo somatrio dos


k2

EUREKA! N36, 2012

45

Sociedade Brasileira de Matemtica

quadrados dos dgitos 25k 2 5k , logo existe um nmero chapa com essa
2

quantidade de algarismos. Agora mostraremos como partir deste nmero obter nmeros chapas de at

2k 2 k 1 k 2 2k 1, conseguimos um algoritmo para obter nmeros


2

chapas a partir de ento.

Para k 2 x 0 x k 2 , a configurao :

343434...34 555...5 . O qual tem soma dos quadrados igual a 25k 2 5k .


2

x vezes 34

k 2 x vezes 5

Vamos ver a partir de qual nmero esta configurao cobre os inteiros:

2k 2 k 2 2k 1 k 1 2 e k 1 2 k 3. Provamos que a partir de n = 9 construmos um nmero chapa de tamanho n. Faltam os casos: n = 1 : 1 chapa n = 2 : 34 chapa n = 3 : 221 chapa n = 4 : 5555 chapa n = 5 : 34555 chapa n = 6 : 343455 chapa n = 7 : 3434345 chapa n = 8 : 34343434 chapa n = 9 : 555555555 chapa
n 10 :
utilize
2

o
2

algaritmo

acima.

(s

fazer

k n

x n n n , obedecendo as condies do nmero construdo).


PROBLEMA 2: SOLUO DE ALEXANDRE PEROZIM DE FAVERI (SO PAULO SP)

Inicialmente, m = 1889 no possvel, pois dando a 32 dos amigos, 61 figurinhas diferentes para cada, e 59 figurinhas (diferentes das distribudas anteriormente) a um dos amigos, cada figurinha possuda por exatamente um amigo. Assim, h uma distribuio completa. Veja que ela possvel, pois, analisando primeiro o amigo com 59 figurinhas, ele no possui 1957 figurinhas, 61 das quais cada amigo possui. Assim, comparando-o com cada amigo, eles no possuem 1891 figurinhas em comum. Alm disso, comparando cada amigo que possui 61 figurinhas com outro que possui a mesma quantidade, pelo mesmo argumento, como para qualquer par de amigos essas figurinhas so todas diferentes, eles no possuem
EUREKA! N36, 2012

46

Sociedade Brasileira de Matemtica

2011 61 61 1889. Assim, existe uma distribuio completa (no incompleta) com m = 1889. evidente que se m 1889 , tambm existiro distribuies completas, pois cada
amigo ter a mesma quantidade ou mais figurinhas, no caso de uma distribuio como exemplificamos (e ela j completa). Assim, m 1889. Agora provaremos que o mnimo m 1890. Primeiro, escolha um amigo ao acaso e d-lhe x figurinhas. Considere dois outros amigos. Agora, denote por y e z o nmero de figurinhas exclusivas diferentes que esses amigos tm. Observe que caso contrrio, x y 121,

2011 x y 1890, e no se cumpre m 1890. Assim, analogamente, x z 121 e y z 121.


Somando 2 x y z 363 x y z

363 x y z 181, pois a soma 2

inteira. Assim, dados quaisquer 3 amigos, eles tm no mximo 181 figurinhas diferentes, o que nos d, para os 33 amigos, 11181 1991 figurinhas diferentes. Como 1991 2011, h figurinhas que ningum possui, e toda distribuio incompleta. Assim, a resposta : m = 1890.
PROBLEMA 3: SOLUO DA BANCA

Vamos provar que existe um tringulo Pj Pj 1Pj 2 com rea maior ou igual a

5 5 . Suponha, por absurdo, que as reas de todos os tringulos Pj Pj 1Pj 2 10 so todas menores de que .

P4 Q P3

P5

P1

P2

Seja Q a interseo entre PP 1 4 e P 3P 5.


EUREKA! N36, 2012

47

Sociedade Brasileira de Matemtica

Note

que

rea

PP 1 2Q max rea PP 1 2P 5 , rea PP 1 2P 3

e,

portanto,

rea PP 1 2P 4 1 rea PP 1 4P 5 rea P 2P 3P 4 1 2 . Assim, PQ rea PP 1 1 2Q PP rea PP 1 2 1 4 1 2P 4

rea PP PQ 1 3P 5 1 . Como rea P 3P 4P 5 e P4Q rea P3 P4 P5 (analogamente a PP 1 3P 5 1 2 , 1 2P 4 ) rea PP PQ PQ 1 2 1 2 1 1 . P4Q PP 1 1 4


Mas tambm verdade que Logo

1 2 PQ 5 5 5 5 1 5 2 5 1 0 , 1 PP 1 2 10 10 1 4
absurdo. Analogamente o outro lado da desigualdade pode ser provado, bastando inverter as desigualdades.
PROBLEMA 4: SOLUO DE TADEU PIRES DE MATOS BELFORT NETO (FORTALEZA CE)

Sim, basta ver a seguinte sequncia: onde b1 mmc b2 b3 ... b2011 , b2 b3 ... b2010 ,..., b2 b3 , b2

b1 , b1 b2 , b1 b2 b3 ..., b1 b2 b3 ... b2011 ,

b2 mmc b3 b4 ... b2011 , b3 ... b2010 ..., b3 b4 , b3 b3 mmc b4 b5 ... b2011 , b4 ... b2010 ..., b4 b5 , b4 b4 mmc b5 ... b2011 , b5 ... b2010 ,..., b5 b6 , b5
b2010 b2011k , k 1, k
*

e b2011 , um inteiro positivo qualquer. Agora vejamos que essa sequncia estritamente crescente, pois sempre estamos somando novos inteiros positivos. E que: a j ai b1 b2 ... b j b1 b2 ... bi bi 1 bi 2 ... b j . Mas como pela construo da sequncia bi 1 bi 2 ... b j bi e bi bi 1 bi 2 ... b1 ; pois cada bi mltiplo do bi 1. Assim
EUREKA! N36, 2012

48

Sociedade Brasileira de Matemtica

bi 1 bi 2 ... b j b1 b2 ... bi ai e

bi 1 bi 2 ... b j b1 b2 ... bi bi 1 ... b j a j

a j ai ai a j ai ai a j ai a j a j ai mdc ai , a j .
Mas mdc ai , a j ai e mdc ai , a j a j

mdc ai , a j a j ai a j ai mdc ai , a j . Logo tal sequncia existe.


PROBLEMA 5: SOLUO DE KAYO DE FRANA GURGEL (FORTALEZA CE)

Note que o crculo circunscrito ao ADE tambm passa por H porque

AEH ADH 180 o que torna o quadriltero AEHD inscritvel.


A C2 F E H B P C D C1

Note que

F BA FCA porque ambos "olham" para o arco FA no crculo C1 (ngulos inscritos) F EA F DA porque ambos "olham" para o arco FA no crculo C2 (ngulos inscritos)
Com isso, FEB

FDC (dois ngulos comuns: e 180 ) e

FB BE FC DC
EUREKA! N36, 2012

49

Sociedade Brasileira de Matemtica

Veja tambm que BEH

BEH F DH 90 CDH pois

EHB DHC O.P.V

, ento

BE BH DC HC
Sendo HP a bissetriz interna do ngulo BHC , o teorema da bissetriz interna garante que

BH BP HC PC FB BP Essas trs igualdades mostram que , e isso nos diz que FP a bissetriz FC PC interna do ngulo BFC , conforme a recproca do teorema da bissetriz interna no BFC . Desse modo fica provado que as duas bissetrizes se encontram em um ponto comum P sobre BC .
PROBLEMA 6: SOLUO DA BANCA

No que se segue, tomaremos os ndices mod 2011, ou seja, xi 2011 xi para todo i inteiro. Lema 1. No caso em que a expresso dada mxima, no existe i tal que xi 1 , xi , xi 1 so todos no nulos. Suponha, por absurdo, que a expresso assume seu valor mximo e que exista xi tal que xi 1 , xi , xi 1 so todos no nulos (ou, o que o mesmo, xi 1 xi xi 1 0 ). Defina os conjuntos A xi xi 0

e B xi xi 1 xi xi 1 0 . Ento B A e

B 0. Seja xk o menor elemento de B e considere xk 1 e xk 1. Temos os


seguintes casos: xk xk 1 e xk xk 1. Nesse caso, sejam

se xi 0 ou i k 0, xi' x x k , se xi 0 e i k i A 1 Essencialmente, zeramos xk e distribumos o seu valor uniformemente entre os


demais termos no nulos. Ento
EUREKA! N36, 2012

xi xi 1

no muda se

xi , xi 1 A e

50

Sociedade Brasileira de Matemtica

k i, i 1 , ou
max xi , xi 1
xk A 1

xi , xi 1 A; aumenta de

xi xi 1 max xi , xi 1 para

se xi A ou xi 1 A, nas no ambos; aumenta de

xk 1 xk xk 1 xk para xk 1

xk se k i, i 1 . A 1

xk 1 xk xk 1. Isso significa que xk 1 B, e sendo xk B, xk 1 0, ou

seja, xk 1 A \ B, ou seja, xk 2 0. Nesse caso, trocamos

'
k 1

' , xk xk 1 xk , 0 .

xk 1 , xk
muda

por para para

Nesse

caso,

xi xi 1
de

no

i k 2, k 1, k ;

para

i k 2

aumenta

xk 2 xk 1 xk 1 xk xk 1 xk 1 xk

' xk 1 xk xk xk 1 para xk 2 xk 1 xk 1 xk ; para i k 1 aumenta de ' xk' 1 xk xk 1 xk ; para i k

aumenta de

para

' xk xk 1 xk 1.

xk 1 xk xk 1. Anlogo ao anterior. xk xk 1 e xk xk 1. Isso quer dizer que xk 1 , xk 1 A \ B, ou seja,


Nesse caso, troque
' ' , xk , xk 1 xk 1 xk 2,0, xk 1 xk 2 . Todas as diferenas xi xi 1 no

xk 2 xk 2 0.
' k 1

xk 1 , xk , xk 1

por

mudam exceto para i k 2, k 1, k , k 1. O que muda no produto final

xk 2 xk 1 xk 1 xk xk xk 1 xk 1 xk 2 xk 1 xk xk 1 xk xk 1 xk 1
que muda para

k 2

' ' ' ' ' ' xk 1 xk 1 xk xk xk 1 xk 1 xk 2 xk 1 x2 2 xk 1 xk 2 . 2 2

Mas

xk 1 xk 2 xk 1 xk 2
2

2 xk 1 xk 1 xk xk / 4 xk 1 xk 1 xk xk2 4

xk 1 xk xk 1 xk xk 1 x k 1
xk 1 xk xk 1 xk xk 1 x k 1.
Como todos os casos esto cobertos, e em todos eles obtivemos um produto maior ou igual ao original, o lema est provado. Agora s temos grupos de uma ou duas variveis consecutivas no nulas. Se temos
EUREKA! N36, 2012

51

Sociedade Brasileira de Matemtica

um grupo 0, xk , 0 , obtemos o produto um grupo 0, xk , xk 1 ,0 , obtemos

xk 1 xk xk xk 1 xk2 ;

se temos

xk 1 xk xk xk 1 xk 1 xk 2 xk xk 1 xk 1 xk .

Note que os grupos agora

podem ser permutados, de modo que podemos deixar todos os grupos com duas variveis vizinhas. Com isso, temos agora o seguinte lema:
Lema 2: No caso em que a expresso dada mxima h exatamente um grupo de

duas variveis no nulas. Suponha, por absurdo, que h pelo menos dois grupos de variveis no nulas 0, a, b,0 e 0, a, b,0 . Pela observao acima, podemos supor sem perda de generalidade que os grupos so consecutivos, ou seja, so, na ordem, 0, a, b,0, c, d ,0 . Troque essas variveis por 0, a b 2,0, b c 2,0, d c 2,0 . Trocamos o produto abcd a b c d por
2

a b 2 b c 2 d c 2
2 2

Mas j vimos que a b 2 a a b , d c d c d e, pela desigualdade


2

das mdias,

b c 2

bc. Multiplicando tudo, prova-se que obtivemos um

produto maior com a troca, o que prova o lema. Com isso, podemos supor, sem perda de generalidade, que as variveis no nulas so as de ndice mpar, ou seja, x1 , x3 ,..., x2011. Nesse caso, obtemos o produto
2 2 2 x1 x2011 x1 x2011 x3 x5 ...x2009 , e podemos otimizar localmente.

Primeiro, seja x1 x2011 s e suponha, sem perda de generalidade, x1 x2011.

Sejam , reais positivos a serem determinados. Ento, pela desigualdade das mdias,

x1 x2011 x1 x2011

x1 x2011 x1 x2011
3

1 x1 x2011 x1 x2011 3

Para que tudo d certo, escolhamos e tais que


EUREKA! N36, 2012

1 1 x1 1 x2011 3

52

Sociedade Brasileira de Matemtica

Obtenhamos s no final, ou seja, 1 1 2; A igualdade possa ocorrer, ou seja, e

x1 x2011 x1 x2011 x2011 1 x1


1 1 1 2.

x1 1 x2011 ,

ou

seja,

Assim, e so as razes da equao t 2 2t 2 0. Assim 3 1 e

1 3, e
1 1 x1 1 x2011 1 3 x1 x2011 3 3 x1 x2011 x1 x2011 s 3 3 18 2
3 3

Agora,

vamos

otimizar

resto,

que

simples.

Sendo

x3 x5 ... x2009
2 2 3 5

2011 s, temos 2
2 2009

x x ...x

Com isso, podemos terminar o problema. Seja real positivo a ser escolhido. Ento a nossa expresso

x x ... x2009 3 5 1004

2008

2011 s 2 1004

2008

x1 x2011 x1 x2011 x x ...x


2 2 3 5

2 2009

s 3 3 2011 s 2 18 1004

2008

2011 2 s s 3 18 1004 3
3
2011

2008

3 3 s 2008 18 3 2011

2011 s 2

1004

3 2011 3 2 s 18 3 2011
Escolhemos 2 3, e enfim temos
2 2 2 x1 x2011 x1 x2011 x3 x5 ...x2009

2011

3 3 . 16

EUREKA! N36, 2012

53

Sociedade Brasileira de Matemtica

Observaes: claro que essa parte final poderia ter sido feita com clculo, mas evitei faz-lo para manter a soluo elementar. A igualdade pode ocorrer. Basta refazer as contas: obtemos

x3 x5 ... x2009 1, x1 34 3 e x2011 34 3 .

EUREKA! N36, 2012

54

Sociedade Brasileira de Matemtica

XXXIII OLIMPADA BRASILEIRA DE MATEMTICA


Problemas e solues da Primeira Fase Nvel Universitrio
PROBLEMA 1

Calcule o valor de

min m, n . 3m n m0 n 0

PROBLEMA 2

Encontre o volume da regio definida por

x 2 y 2 1, 0 z 2 x 4 6 x 2 y 2 y 4

2011

PROBLEMA 3

Z Roberto precisa sortear alguns nmeros primos para elaborar uma questo de teoria dos nmeros para a Olimpada de Matemtica. Ele resolve jogar um dado comum e ir somando os pontos at alcanar um primo. Ele pede para o seu filho mais velho Umberto ir anotando as respostas. Da primeira vez que ele joga o dado sai o nmero 2. Umberto anota que o primeiro primo ser p1 2. No segundo lanamento sai 1. Como 1 no primo, Z Roberto volta a lanar o dado e desta vez sai 4. Umberto anota que o segundo primo ser p2 5. Z Roberto lana o dado novamente e obtm 6. Neste momento seu segundo filho Doisberto, que assistia ao sorteio, declara: Tenho a intuio de que o prximo primo ser p3 11.". Z Roberto fica um pouco surpreso mas decide continuar a lanar o dado normalmente. Qual a probabilidade de que o palpite de Doisberto venha a se confirmar?
PROBLEMA 4

Para n natural, seja f(n) o nmero de pares ordenados (x, y) com x, y inteiros tais que

3x2 2 xy 3 y 2 n.
Calcule o valor mdio de f(n), ou seja, calcule

lim
n

f 0 f 1 f 2 ... f n 1 . n
55

EUREKA! N36, 2012

Sociedade Brasileira de Matemtica

PROBLEMA 5

A funo f : 0, satisfaz

contnua em

0, , derivvel

em

0,

f x 1 cos f x

para todo x 0, . Sabemos que f 0 0 e f 2 1. Mostre que existe um nico nmero real d tal que o limite abaixo exista e pertena a 0, :

a lim
x 0

f x . xd

Determine os valores de d e de a.

PROBLEMA 6

Seja X uma matriz real quadrada n n. Suponha que existe um inteiro positivo m com X 2 I

0.

(a) Mostre que n 2011. (b) Se n = 2010, possvel concluir que X 2 I 0?

SOLUO DO PROBLEMA 1

Para n 0 fixo, seja

Sn
Assim,

min m, n m n m n m m m m 3 2 3n 1 m0 0 m n 3 m n 3 0 m n 3
m n m 1 n m 1 n2 m 23 2 3n 2 0 m n 3 0 m n 1 3

(*)

3Sn

(**)

Substraindo (*) de (**), obtemos

2Sn

1 n n 1 n 1 n1 m. m n2 n 1 23 3 23 0 m n 1 3 0 m n 1 3

Somando a PG, obtemos

3 1 Sn 1 n . 4 3
Assim,
EUREKA! N36, 2012

56

Sociedade Brasileira de Matemtica

min m, n Sn n m n 3 m0 n 0 n0 3

1 1 1 n 1 2 n 1 4 n1 3 n 1 3 1 3 3 4 2 8 9 . 32
SOLUO DO PROBLEMA 2

Em primeiro lugar, notemos que que, em coordenadas polares,


2 2

x 4 6 x 2 y 2 y 4 x 2 y 2 8x 2 y 2
2

r 4 8 r cos r sin r 4 1 8cos2 sin 2 r 4 1 2sin 2 2 r 4 cos 4


0 z 2 r 4 cos 4

Assim, em coordenadas cilndricas, a regio fica definida por

0 r 1,

2011

Onde vale a pena notar que, como r 1, tem-se 2 r 4 cos 4

2011

2 1 1, isto

, a tampa superior do tronco de cilindro no toca a base z = 0. Para encontrar o volume, basta colocar tudo em coordenadas cilndricas

2 r
1 0

8044

cos

2011

4 rdrd

2 r 8046 cos2011 4 d r 8046 r 0

r 1

8 2011 d 2 C 0 cos u du Mas, fazendo a substituio sin u

2 cos 2011 4 1 0 8046

Portanto, o volume pedido 2 .

cos2011 u du 1 2
0 0

1005

d 0.

EUREKA! N36, 2012

57

Sociedade Brasileira de Matemtica

SOLUO DO PROBLEMA 3

Para que o palpite de Doisberto venha a se confirmar, o prximo resultado do dado no pode ser 1, seno a soma at a seria 1 + 6 = 7, que um primo menor que 11. Doisberto estar certo se o prximo resultado do dado aps o 6 inicial for maior que 1 e alguma sequencia de resultados do dado aps o 6 inicial tiver a soma igual a 5 (pois 11 = 6 + 5). As sequncias de resultados com essa propriedade so: (5), (4, 1), (3, 2), (3, 1, 1), (2, 3) (2, 2, 1), (2, 1, 2) e (2, 1, 1, 1). A probabilidade de que o palpite de Doisberto venha a se confirmar , portanto igual probabilidade de que, aps o 6 inicial, saia uma dessas sequncias de resultados, a qual

1 1 1 1 1 1 1 1 1 3 3 1 343 2 2 3 2 3 3 4 . 6 6 6 6 6 6 6 6 6 36 216 1296 1296


SOLUO DO PROBLEMA 4

Aplicando uma rotao de 45 curva determinada pela equao acima, obtemos

x y x y x y x y 3 2 3 n 2 2 2 2

x2 n2

y2 n4

1.

Ou seja, a curva uma elipse com semi-eixos

n 2 e

n 4 e rea

An

n n n . 2 4 2 2

Por outro lado, f 0 f 1 f 2 ... f n 1 igual ao nmero de pontos

x, y com coordenadas inteiras tais que 3x2 2xy 3 y 2 n 1, ou seja, igual ao


nmero de pontos inteiros no interior da elipse de equao 3x2 2 xy 3 y 2 n, que essencialmente a rea An desta elipse. Mais precisamente, para cada ponto inteiro (x, y) no interior da elipse de equao 3x2 2 xy 3 y 2 n, considere o quadrado de vrtices (x, y), (x + 1, y), (x, y + 1), (x + 1, y + 1). Afirmamos que este quadrado est contido na elipse

3x2 2 xy 3 y 2 n 10 n. Como x , y n , temos


EUREKA! N36, 2012

58

Sociedade Brasileira de Matemtica

3 x 1 2 x 1 y 1 3 y 1 3x2 2 xy 3 y 2 4 x y 4 n 10 n
2 2

e analogamente para os demais vrtices do quadrado. Assim, f 0 f 1 f 2 ... f n 1 An10 n Da mesma forma, prova-se que f 0 f 1 f 2 ... f n 1 An10

Assim, pelo sanduche, temos A A f 0 f 1 f 2 ... f n 1 lim n10 n lim lim n10 n n n n n n ou seja, f 0 f 1 f 2 ... f n 1 lim . n n 2 2
SOLUO DO PROBLEMA 5

Estritamente falando, o enunciado deste problema est incorreto: possvel que exista 0 tal que f x 0 para todo x 0, . O enunciado ficaria correto com qualquer uma das seguintes alteraes: (a) Tomar f : 0, 0, ; (b) Considerar, no enunciado, a lim
x 0

f ( x) . xd

(c) Pedir que a

\ 0.

Vamos resolver o problema com esta ltima variao no enunciado, talvez a que menos altera o problema. De f x 1 cos f x , obtemos f 1 cos f 0 cos 0 1. Derivando f x 1 cos f x , obtemos Fazendo x = 1, obtemos 1 f 2 sen f 1 f 1 sen 1 f 1 , donde

f x 1 sen f x f x , x 0.

f 1

1 . sen1

Assim, f 1 h 1
EUREKA! N36, 2012

1 r h h , onde lim
sen1
59

h 0

r h 0.

Sociedade Brasileira de Matemtica

1 s x x Por outro lado, cos x 1


2
2

, onde lim x0 s x 0, e logo


2

1 s f h f h f 1 h cos f h 1
1 r h h f 1
sen1 2

Assim,
2

para

h 0,

temos

1 s f h f h 1 h 1

, donde
1

2 1 r h h 2 , f h 1 s f h sen1

E portanto

f h 2 lim 1 2 , h 0 h sen1
12

Pois como f contnua em 0, limh

constante em um intervalo da forma 0, , 0 pequeno. Assim

s f h limh

r h 0. Assim f tem sinal

f h 2 lim 1 2 . h 0 h sen1
12

Se d 1 2,

lim
h 0

f h f ( h) lim 1 2 lim h1 2d 0, d h 0 h h h 0

e, se d 1 2,

lim
h 0

f h f ( h) lim 1 2 lim h1 2d d h 0 h h h 0

Donde a no est definido. Assim, temos necessariamente d 1 2 e a 2 sen1.


SOLUO DO PROBLEMA 6

(a) Se n = 2011, ento o polinmio caracterstico de X tem grau 2011, e portanto deve ter uma raiz real. Em suma, X teria um autovetor real w associado a um autovalor real . Mas ento:
EUREKA! N36, 2012

60

Sociedade Brasileira de Matemtica

I w 2 1 w
m m

I w 2 1 w 0.

Como w no nulo, devemos ter 2 1

0, isto , 2 1 0, absurdo.

(b ) No. Em primeiro lugar, mostremos que existe uma matriz B44 que satisfaz

I 0 mas que no satisfaz B2 I 0. Por exemplo:


2

0 1 1 0 1 0 0 1 0 1 0 0 I N B2 0 0 1 0 0 1 1 0 0 0 0 1 onde N uma matriz no-nula, triangular superior e, portanto, nilpotente (alis, N 2 0 ). Assim

0 1 1 0 B 0 0 0 0

B2 I N 0 mas B 2 I N 2 0
2

Enfim, considere

B X

A A
0 1 2 (note que A I ; todas 1 0

Onde h um bloco B44 e 1003 blocos A22 as entradas em branco so nulas). Ento

B2 X2
donde
EUREKA! N36, 2012

A2

I N 2 A

I I

61

Sociedade Brasileira de Matemtica

Ento X 2 I 0 mas X 2 I

N 2

0 (e X 2 I 0 para m = 2, 3, 4...).
m

0. 0 0

EUREKA! N36, 2012

62

Sociedade Brasileira de Matemtica

XXXIII OLIMPADA BRASILEIRA DE MATEMTICA


Problemas e solues da Segunda Fase Nvel Universitrio
PRIMEIRO DIA PROBLEMA 1

3 Para cada t reta, seja P t x x 12 x t , e seja

t max c

Pt c 0 min c

Pt c 0

A diferena entre a maior raiz real e a menor raiz real de P t x . Determine o conjunto de valores que t pode assumir quando t varia.
PROBLEMA 2

Considere um polgono regular de n lados inscrito em um crculo unitrio. Determine a soma das reas de todos os tringulos cujos vrtices so vrtices do polgono.
PROBLEMA 3

Para n inteiro positivo e A um subconjunto do conjunto n, definimos f A min t n A A t , onde Definimos g n max f A ; A

n dos inteiros mdulo A t x t , x A n .

n , A n 2 .

(a) Prove que g n n 4 1, n 1.

(b) Prove que g n n 4 1 para infinitos valores de n 1.


SEGUNDO DIA PROBLEMA 4

Considere o polinmio

f x x3 x 2 4 x 1.

(a) Mostre que se r raiz de f x , ento r 2 r 3 tambm uma raiz de

f x.
(b) Sejam , e as trs razes de f x , em alguma ordem. Determine todos os possveis valores de
EUREKA! N36, 2012

63

Sociedade Brasileira de Matemtica

.
PROBLEMA 5

Se u1 ,..., uk

, denote por C u1 ,..., uk o cone gerado por u1 ,..., uk :

C u1 ,..., uk a1u1 ... ak uk ; a1 ,..., ak 0, .


Sejam v1 , v2 , v3 , v4 pontos sorteados independentemente e uniformemente na esfera unitria x 2 y 2 z 2 1. a) Qual a probabilidade de que C v1 , v2 , v3 , v4
3

b) Qual a probabilidade de que cada um dos quatro vertores seja necessrio para gerar C v1 , v2 , v3 , v4 , isto , que C v1 , v2 , v3 C v1 , v2 , v3 , v4 ,

C v1 , v2 , v4 C v1 , v2 , v3 , v4 , C v1 , v3 , v4 C v1 , v2 , v3 , v4 e
C v2 , v3 , v4 C v1 , v2 , v3 , v4 ?
PROBLEMA 6

Seja

xn n0

uma sequncia de nmeros inteiros que satisfaz uma recorrncia

linear de ordem k para um certo inteiro positivo k fixado, i.e., existem constantes reais c1 , c2 ,..., ck tais que xn k

r 1 r n k r

cx

, n 0. Suponha que k o menor

inteiro positivo com essa propriedade. Prove que c j , para todo j com

1 j k.
SOLUES DA SEGUNDA FASE PROBLEMA 1: SOLUO DE LUCAS COLUCCI CAVALCANTE DE SOUZA (SO PAULO SP)

Vamos esboar o grfico de f x x3 12 x.

EUREKA! N36, 2012

64

Sociedade Brasileira de Matemtica

16

2 3

2 16

2 3

3 Seja P t x x 12 x t. Como sua derivada tem razes 2, P t s pode ter mais

de uma raiz real se houver raiz tal que 2 2 , pois em

, 2 a derivada 3x2 12 positiva.

2,

Assim, vamos separar em casos: Pt do terceiro grau, logo tem: ou trs razes reais ou s uma raiz real. i) Pt s possui uma raiz real: Nesse caso, do grfico, t 16 ou t 16, j que Pt obtido transladando o grfico de f t unidades para cima. Nesse caso,

t 0 (pois a nica raiz real max e min).

ii) Pt possui trs razes reais. (obrig. uma , 2 , outra em 2, 2 e outra em 2, sejam 1 , , 3 essas razes (com 1 3 . Na realidade no podemos ter 1 3 , j que a derivada de Pt tem duas razes simples). Das relaes de Girard,

1 3 * 1 3 0 1 3 1 3 12 t 1 3
Pondo
2

(*)

na

segunda
2

equao

do

sistema

obtemos

13 12 13 12.
EUREKA! N36, 2012

65

Sociedade Brasileira de Matemtica

Por fim, note que 3 1 3 1 431 48 3 2 , por (*) e (**).


2 2

Como 2, 2 , temos o grfico de t em funo de : Assim,


2

36 t 48 6 t 4 3.
2

Por

fim

temos

que

Im t 0 6, 4 3 .
t
2

PROBLEMA 2: SOLUO DE GABRIEL LUS MELLO DALALIO (SO PAULO SP)

O primeiro fato til para o problema que:

B
rea ABC 1 2 sen

1 A

1 C

O segundo fato til que tomando A, B, C pontos de um polgono regular inscrito em crculo unitrio centrado na origem, se A, B, C estiverem no sentido antihorrio, tem-se:

rea ABC

onde P o ngulo que o ponto P forma com a origem e o eixo x medido no sentido anti-horrio (como no crculo trigonomtrico) Isso ocorre porque:

1 sen B A sen C B sen A C 2

EUREKA! N36, 2012

66

Sociedade Brasileira de Matemtica

Se a origem estiver dentro de ABC

C 1 1 A 1 B

1 rea ABC sen sen sen 2

Se a origem estiver fora de ABC

1 1 A
rea ABC

1 C

1 sen sen sen 2

Com isso, podemos calcular a rea do tringulo com parcelas relativas aos lados. Numerando os vrtices do polgono regular como P 0, P 1 ,..., P n 1 no sentido antihorrio, temos de contar quantas vezes cada lado aparece em algum tringulo orientado no sentido anti-horrio. Um lado P0 Pk aparece em n k 1 tringulos orientados no sentido anti-horrio

P0 Pk Pk 1 , P0 Pk Pk 2 ..., P0 Pk Pn1 . Isso tambm vai ocorrer com mais n lados que tem o mesmo tamanho que P 0P k PP 1 k 1 , P 2P k 2 ... .
A parcela de rea de P0 Pk dada por:

1 1 2 sen Pk P0 sen k Com isso, j possvel desenvolver um 2 2 n


somatrio que resulta na resposta:
EUREKA! N36, 2012

67

Sociedade Brasileira de Matemtica


n2 1 2 A n n k 1 sen 2 n k 1

Agora iremos resolver o somatrio utilizando sen progresso geomtrica. Para facilitar faamos

ei ei e soma de 2i

2 . n

n n2 n n2 nk 1 n k 1 sen k sen k 2 k 1 2 k 1 j 1
n n 2 n j 1 ik e eik 4i j 1 k 1

Trocando a ordem das somatrias e substituindo sen k tem-se:

Com soma de progresso geomtrica, tem-se:

Como ein

i i n j 1 1 ei e i n j 1 1 n n2 e e A 4i j 1 ei 1 ei 1 in e 1: n n 2 eij ei eij ei A i i 4i j 1 e 1 e 1

Utilizando mais uma vez progresso geomtrica e soma de termo constante tem-se:

i e 1 ei e 1 n 2 ei n 2 ei n e A 4i ei 1 ei 1 ei 1 ei 1 ei 1 e i 1 i n 1 i n 1 ei e ei n 2 ei n 2 ei n e A i 4i 1 ei ei 1 e 1 e i 1 i i i i i i n e e e e n 2 n 2 e n 2 n 2 e A 4i 1 ei ei 1 i i n ne e , Voltando para sen : A 4i 2 2 cos


i n2 i n2

EUREKA! N36, 2012

68

Sociedade Brasileira de Matemtica

2 n 2sen 2 n n sen n sen n n cot . A A A 2 2 2cos 4 1 cos 2 4 n 4 1 cos n


2

PROBLEMA 3: SOLUO ADAPTADA DA SOLUO DE RGIS PRADO BARBOSA (FORTALEZA CE)

a) Dado A

(n) com A m, digamos A x1 , x2 ,..., xm ,

Construmos uma matriz M nm mij Como dado


j

1i n 1 j m

dada por mij

1 se x j i A 0 caso contrrio

j m,

1 i n x i A m, j m, e, como x n x A, 1 i n 1 x i A m 1, j m. Assim, o nmero de m m 1 , e logo 1 j m 1 x i A m m 1 ,


j j

i,1 i n percorre todo o

(n),

temos

1s em M donde existe

n 1 i 1

i n 1 tal que

m m 1 A A i 1 j m xj i A . Para n 1 nn 1 m m 1 n n m m 1 2 2 n 1. Assim, m , , donde 4 n 1 n 1 n 1 4 2

g n n 1. 4
b) Vamos mostrar que se n 3 mod 4 primo ento

n 1 A x 2 mod n ,1 x um conjunto com 2 n n f A 1, donde g n 1. Isso resolve 4 4 infinitos primos n 3 mod 4 .


EUREKA! N36, 2012

n elementos tal que 2


o problema, pois existem

69

Sociedade Brasileira de Matemtica

Dado

x, x k , 1 k n 1, x, x k n \ 0 tais que 2 Isso equivale a x 2 2kx k 2 x k x 2 t mod n . 2kx t k 2 mod n , que tem uma nica soluo em n , que distinta de 0 desde que k 2 t mod n . Note que se em xk 0 n , ento x k , e 2k 2 2kx t k 2 mod n t k 2 mod n , e se em x0 n ,0 2kx t k 2 mod n , donde t k 2 mod n . Note que para um mesmo t no podem existir 1 r , k n 1 com t r 2 mod n e t k 2 mod n , seno, pelo pequeno teorema de Fermat teramos, em
n ,1 r n1 r 2
(note que
n 1 2

com

1 t n 1,

estimemos

nmero

de

pares

k 2

n 1 2

n 1 2

k 2

n 1 2

k n1 1,

absurdo

n 1 mpar, pois n 3 mod 4 ). 2 Agora, dados a, b n \ 0 , existem no mximo 4 pares


2

x, x k com

x, x k n \ 0 tais que x 2 a e x k b. Como dado t com 1 t n 1, existem no mximo dois valores de r com 1 r n 1 tais que r 2 t mod n , e no mximo dois valores de k com 1 k n 1 tais que k 2 t mod n , x, x k
h pelo
2

menos

n 3

pares

x, x k

com

n \ 0 e x k

x 2 t. Portanto,

n 1 n3 A A t u, v 1 u, v , u v, v 2 u 2 t mod n 2 4 n 1, t n \ 0 . 4

EUREKA! N36, 2012

70

Sociedade Brasileira de Matemtica

PROBLEMA 4: SOLUO DE LEANDRO FARIAS MAIA (RIO DE JANEIRO RJ)

a) fcil perceber que 0 no raiz do polinmio. Seja r uma raiz de f. Queremos mostrar que p raiz de f, onde p r 2 r 3 Note que:

r 3 r 2 4r 1 0 r r 2 r 4r 1 r 2 r 4
Logo,

1 r

1 1 p r 2 r 3 4 3 p 1 (*) r r
Temos,

4 3 3 1 2 1 p 3 p 2 4 p 1 1 2 3 1 2 4 1 r r r r r r 2 3 1 4 1 1 4r r r 0 3 2 1 3 0. 3 r r r r r
Logo, provamos que p raiz de f. b) Vamos mostrar que p 2 p 3 outra raiz de f, diferente de r e p. Temos: i)

p r , pois caso p r r 2 r 3 r r 3. Porm 3 nem

3 so razes de f.
ii)

p 2 p 3 p, pois caso p 2 p 3 p p 2 3 p 3 e, da

mesma forma, como p tambm uma raiz, nem 3 nem 3 raiz de f. iii) p 2 p 3 r. Esta parte apenas mais trabalhosa. iv) Sabemos que p 1 , assim.

1 r

1 1 2 1 1 1 1 3 r 1 2 1 3 r r r r r r 1 3 2 1 r 0 1 3r r 2 r 3 0 r r 2 1 3r 4r 1 0 2 7r, porm, r no raiz de f. 7


Vamos agora achar os possveis valores para a expresso do problema. Vamos dividir em casos.
EUREKA! N36, 2012

71

Sociedade Brasileira de Matemtica

Caso 1: Se 2 3 Perceba que

2 3 passo iii e 2 3 passo 2 , assim

2 3. Do mesmo raciocnio, tiramos que 2 3 1 Relaes de Girard 4 1


Logo, T

2 3 2 3 2 3
1 (de *), r

Sabemos que se r raiz, ento: r 2 r 3 1 Assim,

1 1 1 1 1 1 1 1 1 1 1 1 T 4 1 3. 1 1
Caso 2: Se 2 3 Do mesmo modo no caso 2, conclumos: 2 3 e 2 3 . Portanto

2 3 2 3 2 3 T 1 1 1 3 3 1 3 3
1 3 3

4 1 3 12 10. 1

Resposta: 10 e +3.
PROBLEMA 5: SOLUO DE RAFAEL TUPYNAMB DUTRA (BELO HORIZONTE MG)

Com 3 vetores, impossvel gerar

C v1 , v2 , v3

(atravs do cone). Ou seja,

EUREKA! N36, 2012

72

Sociedade Brasileira de Matemtica

Prova: se v1 C v1 , v2 , v3 , ento v1 a1v1 a2v2 a3v3 v1 (ou seja, precisamos ter

v1 C v2 , v3 ), Assim,
3

v1 , v2 , v3

a a2 v2 3 v3 1 a1 1 a1
so L.D.

obviamente no geram
ter

.
3

Usando o mesmo argumento, se 4 vetores geram

(atravs do cone), precisamos Ou

v1 a1v1 a2v2 a3v3 a4v4 v1

a a2 a v2 3 v3 4 v4 . 1 a1 1 a1 1 a1
j i

seja, v1 C v2 , v3 , v4 condio necessria vi C

v .
j

Se tivermos ter

v1 C v2 , v3 ,

ento

v1 , v2 , v3

sero

L.D..

Mas

precisamos

v4 C v1 , v2 , v3 . Mas isso implicaria que v4 est no plano gerado por

v1 , v2 , v3 os
obviamente eles

quatro vetores v1 , v2 , v3 , v4 esto no mesmo plano e, assim, no geram


3

v1 C v2 , v3 . Ou analogamente, no podemos ter vi C v j , vk .

Conclumos

que

no

podemos

ter

Mas isso implica que v1 a2v2 a3v3 a4v4 com coeficientes a2 , a3 , a4 todos no nulos esses coeficientes a2 , a3 , a4 so todos positivos temos

v1 a2v2 a3v3 a4v4 0.

Assim,

para

termos

C v1 , v2 , v3 , v4

necessrio que exista combinao linear de v1 , v2 , v3 , v4 , com todos os coeficientes positivos que igual a 0. Suponha que existem 1 , 2 , 3 , 4 positivos tais que A partir disso, evidente que vi C

v
i 1
i

i i
4

0.

j i

v . Temos v
j
j i j 1

j
i

vj.

Conjectura: Se os vetores v1 , v2 , v3 , v4 forem trs a trs L.I. (no h 3 coplanares) e existem 1 , 2 , 3 , 4 positivos tais que Prova da conjectura: Como

v
i 1

i i

0, esto C v1 , v2 , v3 , v4

Note que a recproca da conjectura j foi provada.

v1 , v2 , v3 so

L.I., qualquer vetor v

pode ser

escrito como 1v1 2v2 3v3 . Se algum dos i for negativo, trocamos i vi por
EUREKA! N36, 2012

73

Sociedade Brasileira de Matemtica

i
j i

j v j . Fazendo esse processo, obteremos v como combinao linear de j 1 i


4

v1 , v2 , v3 , v4 com coeficientes todos no negativos.


Sorteamos v1 na esfera S 2 . Depois sorteamos v2 . Temos probabilidade 1 de que

v2 v1 , v1.

Ento, sorteamos v3 e v4 . H probabilidade 1 de que no haver trs vetores coplanares em v1 , v2 , v3 , v4 . Ento, vamos supor que esse o caso. Sendo assim, teremos C v1 , v2 , v3 , v4
3

se e s se existem 1 , 2 , 3 , 4 positivos tais que

v
i 1

i i

0, o que acontece se e s se v4 C v1 , v2 , v3 .

Ento, queremos calcular a probabilidade de v4 C v1 , v2 , v3 . Isso depende apenas do ngulo esfrico (rea sobre a esfera) do cone C v1 , v2 , v3 . A esfera S 2 fica dividida em 8 regies, cada uma coberta por um cone da forma

C v1 , v2 , v3 .

Assim, fcil ver que o valor esperado para o ngulo esfrico do cone

1 C v1 , v2 , v3 um oitavo do ngulo total E 4 . 8 2

Sejam

C v1 , v2 , v3 , C v1 , v2 , v3 , C v1 , v2 , v3 , C v1 , v2 , v3 , C v1 v2 , v3 ,...
Temos sempre

0 , 1 , 2 ,..., 7
7

os

oito

ngulos

esfricos

dos

cones

k 0

4 , pois as regies da esfera delimitadas por esses

ngulos tm interseo de rea nula e a unio delas toda a esfera (*). Mas, por simetria, obvio que os valores esperados de i , j so iguais. Assim,

E 4 8 E 4 E 2 .
7 k 0 k 0 0

O valor esperado do ngulo esfrico do cone C v1 , v2 , v3


EUREKA! N36, 2012

. 2

74

Sociedade Brasileira de Matemtica

Depois de sortear v1 , v2 , v3 , quando sorteamos v4 , a probabilidade de termos

0 . Assim, a probabilidade procurada (de que 4 1 C v1 , v2 , v3 , v4 3 ) E 0 2 . 4 4 8

v4 C v1 , v2 , v3

(*)

Estamos

supondo

v1 , v2 , v3

L.I .

Todo

vS2

escrito

como

1v1 2v2 3v3 . Ento, se 1 23 0 (o que acontece quase certamente)


temos

v C sign 1 v1 , sign 2 v2 , sign 3 v3 ,

sendo

sign x

x . x

Assim, v est em exatamente 1 dos 8 cones. b)Temos C v1 , v2 , v3 C v1 , v2 , v3 , v4 v4 C v1 , v2 , v3 . No item a), calculamos a probabilidade de termos obtivemos

v4 C v1 , v2 , v3 e

1 . 8

Suponha que v1 , v2 , v3 j foram escolhidos e eles so L.I. Novamente a esfera est dividida em 8 regies (quadrantes) gerados pelas cores C v1 , v2 , v3 . Vamos analisar em quais desses quadrantes o vetor v4 pode ser colocado de forma a satisfazer o enunciado. Se v4 C v1 , v2 , v3 , ento obviamente o enunciado no satisfeito. Se v4 C v1 , v2 , v3 , ento temos

v4 a1v1 a2v2 a3v3 v3


enunciado no satisfeito. Analogamente, se

a1 a 1 v2 2 v2 v4 C v1 , v2 , v4 a3 a3 a3
temos

v4 C v1 , v2 , v3 ,

v2 C v1 , v3 , v4

se

v4 C v1 , v2 , v3 , temos v1 C v2 , v3 , v4 .
Afirmo que, em qualquer outro caso o enunciado satisfeito. De fato, podemos supor que v1 , v2 , v3 , v4 so vetores trs a trs L.I. (no coplanares), pois isso ocorre com probabilidade 1.
EUREKA! N36, 2012

75

Sociedade Brasileira de Matemtica

Se v4 C v2 , v2 , v3 , temos v4 a1v1 a2v2 a3v3 0 para certos a1 , a2 , a3 positivos. Ento, escrevemos v1 como combinao linear (com coeficientes negativos) de

v2 , v3 , v4 . Mas essa escrita nica, pois v2 , v3 , v4 L.I.. Ento v1 no pode ser

expresso como combinao linear de v2 , v3 , v4 com coeficientes positivos

v1 C v2 , v3 , v4 . Com argumento anlogo para v2 e v3 , conclumos que o


*

enunciado satisfeito. J se v4 C v1 , v2 , v3 , temos v4 a1v1 a2v2 a3v3 0, a1 , a2 , a3 Assim, v1

a a a a 1 1 v4 2 v2 3 v3 , v2 v4 2 v2 3 v3 , a1 a1 a1 a2 a2 a2

v3

a a 1 v4 2 v2 3 v2 essas combinaes lineares no tm todos os a3 a3 a3

coeficientes positivos. Logo, o enunciado satisfeito. Analogamente, os casos v4 C v1 , v2 , v3 e v4 C v1 , v2 , v3 tambm satisfazem o enunciado. Ento 4 dos 8 quadrantes possveis para v4 satisfazem e os outros 4 no satisfazem. E mais. Por simetria, o ngulo esfrico de dois quadrantes opostos igual. Por exemplo, C v2 , v2 , v3 e C v2 , v2 , v3 tm mesmo gulo esfrico. E os quadrantes que no satisfazem so sempre opostos aos quadrantes que satisfazem. Assim, a soma dos gulos esfricos dos quadrantes que satisfazem obviamente

1 2 1 4 2 e a probabilidade procurada . 2 4 2
PROBLEMA 6: SOLUO DA BANCA

Devemos supor que a sequncia xn no identicamente nula. Notemos inicialmente que os vetores

x0 , x1 ,..., xk 1 , x1, x2 ,..., xk ,..., xk 1 , xk ,..., x2k 2


r

so linearmente

independentes. De fato, se para algum inteiro r com 1 r k 1 tivssemos

x j r di x j r i , para 0 j k 1, por induo em n k teramos


i 1

EUREKA! N36, 2012

76

Sociedade Brasileira de Matemtica

xn r cs xn r s cs di xn s r i di cs xn r i s di xn r i , ou seja,
s 1 s 1 i 1 i 1 s 1 i 1

xn

satisfaz uma recorrncia linear de ordem r k , xn r

d x
i 1

i n r i

, n 0,

absurdo. Seja agora

a transformao linear dada por A: k k k A y1 , y2 ,..., yk y2 ,..., yk , cs yk 1s . Se v j : x j , x j 1 ,..., x j k 1 , para s 1 j 0, temos A v j v j 1 , para todo j 0, donde Ar v j v j r , j, r 0.
k

Como vo , v1 ,..., vk 1 tm coordenadas inteiras e formam uma base de

, e, para

todo r 1, Ar v0 vr , Ar v1 vr 1 ,..., Ar vk 1 vr k 1 , a matriz de Ar na base

v0 , v1 ,..., vk 1 tem entradas racionais com denominadores uniformemente limitados


(independentemente de r). r Se 1 , 2 ,..., k so os autovalores de A, os autovalores de Ar so 1r , 2 ,..., kr e, pelo

x x ... x ,
r 1 r 2 r k

pargrafo

anterior,

polinmio

caracterstico

de

Ar ,

que

tem coeficientes racionais com denominadores

uniformemente limitados (por um inteiro positivo independente de r), e portanto as r r funes simtricas j : j 1r , 2 ,..., kr ir ir2 ... irj ,1 j k 1

1i1 i2 ...i j k

so racionais com denominadores uniformente limitados. Veremos agora, por induo em m + k, que qualquer polinmio simtrico de grau m em 1 , 2 ,..., k com coeficientes inteiros pode ser escrito como um polinmio de
r coeficientes inteiros e grau menor ou igual a k nas funes simtricas j , j, r 1

(de modo que em cada monmio do tipo c

(
i 1

ri i
ji

temos

i 1

k e

r j
i 1

i i i

n ) mais especificamente, o polinmio se escreve como um


1

polinmio em k cujos coeficientes so polinmios de graus menores que k nos

jr ,1 j k 1, r 1. Em particular, no nosso caso, os polinmios simtricos com coeficientes inteiros em 1 , 2 ,..., k tero valores racionais com
EUREKA! N36, 2012

77

Sociedade Brasileira de Matemtica

denominadores uniformemente limitados (independente do polinmio simtrico). Para provar nossa afirmao, abusando de notao para supor agora que 1 , 2 ,..., k so variveis, se f 1 ,..., k o nosso polinmio,

f 1 ,..., k 1 ,0 um polinmio simtrico em k 1 variveis de grau m, logo


pode ser escrito como um polinmio g de coeficientes inteiros e grau menor ou igual a

k 1 nas funes simtricas j

1 ,..., k 1 .

Subtraindo de

f 1 ,..., k esse polinmio g nas funes simtricas jr jr 1 ,..., k ,


cujo grau nos i menor ou igual a m, obtemos um polinmio que se anula se

k 0, e logo mltiplo de 1 2 ... k . Dividindo esse polinmio por 1 2 ... k , obtemos um polinmio de grau menor
que m, que, por hiptese de induo, se escreve como um polinmio em k cujos
1
r coeficientes so polinmios de graus menores que k nos j ,1 j k 1, r 1.

Portanto, podemos escrever nosso polinmio f 1 ,..., k como um polinmio


(1) em k 1... k cujos coeficientes so polinmios simtricos em 1 ,..., k que podem ser escritos como polinmios de coeficientes inteiros de graus menores ou

r iguais a k 1 nas funes simtricas j 1 ,..., k , j, r 1, o que implica nossa


(s) afirmao, pois, para todo s 1 , 1 2 ... k k . s

Aplicando

1 j

1 ,..., k

esse
n

resultado

aos

polinmios

simtricos

, j k , n , e voltando a tomar 1 ,..., k como sendo os

autovalores de A, vamos que esses nmeros tm valores racionais com denominadores uniformemente limitados (independentemente de n), e logo, para

1 j k , c j j 1 ,..., k um inteiro, c.q.d..


1

EUREKA! N36, 2012

78

Sociedade Brasileira de Matemtica

XXXIII OLIMPADA BRASILEIRA DE MATEMTICA


Premiados
NVEL 1 (6 e 7 anos)
Nome Pedro Henrique Sacramento de Oliveira Rogerio Aristida Guimares Junior Mateus Siqueira Thimteo William Hideki Kondo Bruna Malvar Castello Branco Nathan Bonetti Teodoro Mariana Miwa Okuma Miyashiro Lucas dos Anjos Dantas Teixeira Maria Jlia Costa Medeiros Mateus Pereira Carolina Carvalho Silva Laura Mello DUrso Vianna Henrique Gontijo Chiari Nicolas Wolaniuk do Amaral Carvalho Lucas Diniz Gonalves Villas Boas Leonardo de Matos Fellippetti Mariano Lcia Vernica Copque Aguiar de Souza Adriana Mayumi Shiguihara Daniel Akira Hasimoto Rodrigo Gonalves Correa Csar Ricardo Silva Filippi Marina Maciel Ansanelli Henrique Bittencourt Netto Monteiro Julia Perdigo Saltiel Jonathan Pereira Maria Lucas Tokio Kawahara Sandra Ayumi Nihama Joo Guilherme Madeira Arajo Andrey Jhen Shan Chen Bruno Brasil Meinhart EUREKA! N36, 2012 Cidade Estado Louveira SP Teresina PI Mogi das Cruzes SP So Paulo SP Rio de Janeiro RJ Curitiba PR So Paulo SP So Paulo SP Fortaleza CE Belo Horizonte MG Rio de Janeiro RJ Rio de Janeiro RJ Belo Horizonte MG Curitiba PR Salvador BA Curitiba PR Rio de Janeiro RJ So Paulo SP Salto SP Rio de Janeiro RJ Jundia SP So Paulo SP So Paulo SP Rio de Janeiro RJ Ribeiro Pires SP So Paulo SP So Paulo SP Sobral CE Campinas SP Fortaleza CE Total 297 267 266 253 251 248 240 233 232 232 231 227 222 219 216 213 213 212 211 210 209 209 207 207 202 201 201 200 199 199 Prmio Ouro Ouro Ouro Ouro Ouro Ouro Prata Prata Prata Prata Prata Prata Prata Prata Prata Bronze Bronze Bronze Bronze Bronze Bronze Bronze Bronze Bronze Bronze Bronze Bronze Bronze Bronze Bronze

79

Sociedade Brasileira de Matemtica


Daniel Quinto de Moraes Diene Xie Felipe Reyel Feitosa Henrique Corato Zanarella Alcia Fortes Machado Andr Yuji Hisatsuga Bernardo Puetter Schaeffer Bruno Teixeira Gomes Eduardo Reis Cavalcante de Farias Bruno Vinicius da Silva Alves Adriano Henrique de C. A. e Silva Fernando Seiji B. dos Santos Alba Clara Vasconcelos Leopoldo Feitosa Bruno Kenzo Ozaki Eduardo Lennert Ramm Iara Rohn Kombrink Davies Victor Alves Benevides Samuel Sena Galvo Vitor Thomaz da Cruz Francisco Bruno Dias Ribeira da Silva Bryan Diniz Borck Jonathan Aires Pinheiro Nicolas Meira Sinott Lopes Rafael Tchen Yin Hang Wei Joo Alberto Moreira Serdio Loc Dominguez Vincius Soares de Abreu Silva Breno Maia Baptista Lusa Hller Lee Brendon Diniz Borck Eduardo Emilio Costa Trunci Bernardo Gabriele Collao Lucas Hideki Takeuchi Okamura Plinio Melo Guimares Valrio Rodrigo Vieira Casanova Monteiro Victor M.K. Tsuda Arthur Henrique Craveiro Costa Pedro Orii Antoncio EUREKA! N36, 2012 Rio de Janeiro RJ Curitiba PR Teresina PI Amparo SP Teresina PI So Paulo SP Rio de Janeiro RJ Fortaleza CE Teresina PI Curitiba PR Rio de Janeiro RJ So Paulo SP Teresina PI So Paulo SP Joinville SC Rio Claro SP Fortaleza CE Braslia DF So Paulo SP Teresina PI Porto Alegre RS Fortaleza CE Salvador BA Rio de Janeiro RJ Braslia DF Fortaleza CE Rio de Janeiro RJ Fortaleza CE Curitiba PR Porto Alegre RS Curitiba PR Fortaleza CE Suzano SP Belo Horizonte MG Rio de Janeiro RJ So Paulo SP Natal RN So Paulo SP 198 198 196 195 194 194 193 192 191 190 184 184 183 182 181 179 179 178 178 177 176 176 176 174 173 173 173 172 172 171 170 168 168 166 166 165 163 163 Bronze Bronze Meno Honrosa Meno Honrosa Meno Honrosa Meno Honrosa Meno Honrosa Meno Honrosa Meno Honrosa Meno Honrosa Meno Honrosa Meno Honrosa Meno Honrosa Meno Honrosa Meno Honrosa Meno Honrosa Meno Honrosa Meno Honrosa Meno Honrosa Meno Honrosa Meno Honrosa Meno Honrosa Meno Honrosa Meno Honrosa Meno Honrosa Meno Honrosa Meno Honrosa Meno Honrosa Meno Honrosa Meno Honrosa Meno Honrosa Meno Honrosa Meno Honrosa Meno Honrosa Meno Honrosa Meno Honrosa Meno Honrosa Meno Honrosa

80

Sociedade Brasileira de Matemtica


Gabriel Moura Brana Victria Santos Duarte Ramos talo Rennan Lima Amanda Barbosa Schirmbeck Thiago Ferreira Teixeira Gabriel Dante Cawamura Seppelfelt Lucas Siqueira Arago Milena Delarete Drummond Marques Rodrigo Moutinho Faria Daniel Lopes de Castro Joo Vitor Vaz Oliveira Matheus Bevilacqua Fortaleza CE Rio de Janeiro RJ Fortaleza CE Braslia DF Vinhedo SP So Caetano do Sul SP Vitria ES Belo Horizonte MG Belm PA Braslia DF Recife PE Campinas SP 162 162 161 160 160 159 159 159 159 158 158 158 Meno Honrosa Meno Honrosa Meno Honrosa Meno Honrosa Meno Honrosa Meno Honrosa Meno Honrosa Meno Honrosa Meno Honrosa Meno Honrosa Meno Honrosa Meno Honrosa

Nvel 2 (8 e 9 anos do Ensino Fundamental)


Nome Alessandro A.P. de Oliveira Pacanowski Gabriel Fazoli Domingos Daniel Santana Rocha Vitor Dias Gomes Barrios Marin Luze Mello DUrso Vianna Daniel Lima Braga Fbio da Silva Soares Joo Pedro Sedeu Godoi Murilo Corato Zanarella Bruno Eidi Nishimoto Mariana Teatini Ribeiro Samuel Brasil de Albuquerque Lucas Mioranci Mateus Bezrutchka Ana Karoline Borges Carneiro Ana Emlia Hernandes Dib Pedro Henrique Alencar Costa Pedro Augusto Brasileiro Lins Barbosa Gabriel Mayrink Verdun Leonardo Santos Matiello Matheus Caris Castro Lucca Morais de Arruda Siaudzionis Luiz Cludio Sampaio Ramos EUREKA! N36, 2012 Cidade - Estado Rio de Janeiro RJ Urups SP Rio de Janeiro RJ Presidente Prudente SP Rio de Janeiro RJ Eusbio CE Planaltina DF Rio de Janeiro RJ Amparo SO Sales SP Belo Horizonte MG Fortaleza CE S. J. do Rio Preto SP Taboo da Serra SP Fortaleza CE S. J. do Rio Preto SP Fortaleza CE J. dos Guararapes PE Rio de Janeiro RJ Campo Grande MS Fortaleza CE Fortaleza CE Rio de Janeiro RJ Total 364 357 337 315 307 279 278 271 259 252 246 243 225 223 221 217 211 207 203 201 198 195 188 Prmio Ouro Ouro Ouro Ouro Ouro Prata Prata Prata Prata Prata Prata Prata Prata Prata Prata Prata Bronze Bronze Bronze Bronze Bronze Bronze Bronze

81

Sociedade Brasileira de Matemtica


Matheus Carioca Sampaio Jos Wanderclesson Nobre Damasceno Filho Suzane Eberhart Ribeiro da Silva Estevo Waldow Erika Rizzo Aquino Pedro Jorge Luz Alves Cronemberger Alexandre Mendona Cardoso Ricardo Ken Wang Tsuzuki Leonardo Alves Ramalho Ana Paula Lopes Schuch Flvia Nakazato Hokama Lucas Bastos Germano Helena Veronique Rios Isabelle Ferreira de Oliveira Rafael Wilton Barboza Coracini Eduardo Serpa Giovana Sachett Maia Paulo Henrique Omena de Freitas Amanda Vidotto Cerqueira Bruno Cicone de Almeida Gabriel Picano Costa Guilherme Anitele Silva Mateus Arraes Feitosa Borges Rodrigo Zanette de Magalhes Luis Eduardo de Sousa Lima Gabriel Vidigal de Paula Santos Bruno Almeida Costa Joo Baptista de Paula e Silva Gabriel Ribeiro Barbosa Kevin Eiji Inashita Dimas Macedo de Albuquerque Mauricio Najjar da Silveira Juliano Petry Pesarico Bruna Caroline Pimentel Gonalves Gustavo Torres da Silva Artur Corassa Martins Italo Lesione de Paiva Rocha Nathan Antonio de Azevedo Milagres EUREKA! N36, 2012 Fortaleza CE Fortaleza CE Campo Grande MS Piraquara PR Goinia GO Teresina PI Salvador BA So Paulo SP Curitiba PR Porto Alegre RS So Paulo SP Fortaleza CE So Carlos SP Fortaleza CE Rinpolis SP Caucaia CE Belm PA So Paulo SP So Paulo SP S. J. dos Campos SP Fortaleza CE Presidente Prudente SP Fortaleza CE So Paulo SP Fortaleza CE Rio de Janeiro RJ Fortaleza CE Belo Horizonte MG Fortaleza CE So Paulo SP Fortaleza CE So Paulo SP Laguna Carap MS Fortaleza CE So Paulo SP Braslia DF Fortaleza CE Belo Horizonte MG 187 181 179 178 177 175 173 172 170 168 168 168 166 164 164 161 161 159 158 158 156 152 152 152 150 148 147 144 143 142 140 139 137 134 134 132 132 131 Bronze Bronze Bronze Bronze Bronze Bronze Bronze Bronze Meno Honrosa Meno Honrosa Meno Honrosa Meno Honrosa Meno Honrosa Meno Honrosa Meno Honrosa Meno Honrosa Meno Honrosa Meno Honrosa Meno Honrosa Meno Honrosa Meno Honrosa Meno Honrosa Meno Honrosa Meno Honrosa Meno Honrosa Meno Honrosa Meno Honrosa Meno Honrosa Meno Honrosa Meno Honrosa Meno Honrosa Meno Honrosa Meno Honrosa Meno Honrosa Meno Honrosa Meno Honrosa Meno Honrosa Meno Honrosa

82

Sociedade Brasileira de Matemtica


Juliana Amoedo Amoedo Plcido Victria Moreira Reis Cogo Leandro Alves Cordeiro Rmulo Gabriel Lima da Costa Bruno Vasconcelos Silva Alexandro Vtor Serafim de Carvalho Cristhian Mafalda Douglas Matos Gomes Gabriel Diniz Vieira e Sousa Enrico Pascucci Lffel Ricardo Vidal Mota Peixoto Salvador BA Teresina PI Ribeiro Pires SP Fortaleza CE Fortaleza CE Macei AL Leme SP So Paulo SP Fortaleza CE S. B. do Campo SP Vassouras RJ 128 128 126 125 121 120 120 120 118 116 116 Meno Honrosa Meno Honrosa Meno Honrosa Meno Honrosa Meno Honrosa Meno Honrosa Meno Honrosa Meno Honrosa Meno Honrosa Meno Honrosa Meno Honrosa

Nvel 3 (Ensino Mdio)


Nome Joo Lucas Camelo S Henrique Gasparini Fiuza do Nascimento Rafael Kazuhiro Miyazaki Andr Macieira Braga Costa Rodrigo Sanches Angelo Maria Clara Mendes Silva Victor de Oliveira Bitaraes Tadeu Pires de Matos Belfort Neto Rafael Rodrigues Rocha de Melo Gustavo Haddad Francisco e Sampaio Braga Daniel Eiti Nishida Kawai Henrique Vieira G. Vaz Carlos Henrique de Andrade Silva Victor Hugo Corra Rodrigues Franco Matheus de Alencar Severo Gabriel Ilharco Magalhes Lucas Loureno Hernandes Ivan Tadeu Ferreira Antunes Filho Kayo de Frana Gurgel Michel Rozenberg Zelazny Alexandre Perozim de Faveri Davi Coelho Amorim Marcos Massayuki Kawakami Daniel dos Santos Bossle EUREKA! N36, 2012 Cidade Estado Fortaleza CE Braslia DF So Paulo SP Belo Horizonte MG So Paulo SP Pirajuba MG Betim MG Fortaleza CE Fortaleza CE S. J. dos Campos SP Itibaia SP So Paulo SP Fortaleza CE Rio de Janeiro RJ Rio de Janeiro RJ Rio de Janeiro RJ So Paulo SP Lins SP Fortaleza CE So Paulo SP Neves Paulista SP Fortaleza CE So Paulo SP Porto Alegre RS Total 295 286 285 273 270 263 245 245 237 234 234 218 196 189 184 183 177 174 173 173 171 169 169 167 Prmio Ouro Ouro Ouro Ouro Ouro Ouro Prata Prata Prata Prata Prata Prata Prata Prata Prata Prata Bronze Bronze Bronze Bronze Bronze Bronze Bronze Bronze

83

Sociedade Brasileira de Matemtica


Gabriel Milito Vinhas Lopes Mateus Henrique Ramos de Souza Victor Venturi Ramon Silva de Lima Gabriel Jos Moreira da Costa Silva Otvio Augusto de Oliveira Mendes Marcelo Luiz Gonalves Artur Dubeux Duarte Natan Lima Viana Bruno Silva Mucciaccia Juliana Lemes Arai Matheus Henrique Alves Moura Felipe Sampaio Lima Davi Sampaio de Alencar Pedro Morais de Arruda Siaudzionis Luiz Castelo Branco Cavalcante Glauber de Lima Guarinello Victor Oliveira Reis Jos Ney Alves Feitosa Neto Andre Bandeira Pinheiro Fernando Lima Saraiva Filho Rafael Tedeschi Eugnio Pontes Barone Vincius Canto Costa Lincoln de Queiroz Vieira Thiago Poeiras Silva Andr Amaral de Souza Carlos Alexandre Silva dos Santos Felipe Viana Sousa Liara Guinsberg Otavio Arajo de Aguiar Rodolfo Rodrigues da Costa Caque Porto Lira Kelvin Azevedo Santos Eric Tada de Souza Marcelo Cargnelutti Rossato Marina Pessoa Mota Fortaleza CE Pirapora MG Campinas SP So Paulo SP Macei AL Pilar do Sul SP Franca SP Recife PE Fortaleza CE Vitria ES S. J. dos Campos SP Itapipoca CE Fortaleza CE Fortaleza CE Fortaleza CE Fortaleza CE So Paulo SP Recife PE Fortaleza CE Fortaleza CE Eusbio CE Araatuba SP Rio de Janeiro RJ Fortaleza CE Belo Horizonte MG Diadema SP Fortaleza CE Fortaleza CE So Paulo SP Fortaleza CE Fortaleza CE Fortaleza CE S. J. dos Campos SP So Paulo SP Santa Maria RS Fortaleza CE 167 162 157 156 152 152 150 145 142 141 138 138 137 135 135 134 134 133 132 131 130 130 130 128 128 124 124 124 122 121 121 117 117 116 115 115 Bronze Bronze Bronze Bronze Bronze Bronze Bronze Meno Honrosa Meno Honrosa Meno Honrosa Meno Honrosa Meno Honrosa Meno Honrosa Meno Honrosa Meno Honrosa Meno Honrosa Meno Honrosa Meno Honrosa Meno Honrosa Meno Honrosa Meno Honrosa Meno Honrosa Meno Honrosa Meno Honrosa Meno Honrosa Meno Honrosa Meno Honrosa Meno Honrosa Meno Honrosa Meno Honrosa Meno Honrosa Meno Honrosa Meno Honrosa Meno Honrosa Meno Honrosa Meno Honrosa

EUREKA! N36, 2012

84

Sociedade Brasileira de Matemtica

Nvel Universitrio
Nome Renan Henrique Finder Rafael Tupynamb Dutra Rgis Prado Barbosa Guilherme Rodrigues Nogueira de Souza Davi Lopes Alves de Medeiros Gabriel Luis Mello Dalalio Felipe Gonalves Assis Darcy Gabriel Augusto de Camargo Cunha Hugo Fonseca Arajo Matheus Secco Torres da Silva Erik Fernando de Amorim Lucas Colucci Cavalcante de Souza Jos Leandro Pinheiro Reinan Ribeiro Souza Santos Daniel de Barros Soares Rafael Endlich Pimentel Carlos Henrique Melo Souza Ivan Guilhon Mitoso Rocha Thiago Ribeiro Ramos Lucas de Freitas Smaira Paulo Srgio de Castro Moreira Alexandre Azevedo Csar Ricardo Turolla Bortolotti Robrio Soares Nunes Marcelo Matheus Gauy Charles Barbosa de Macedo Brito Renato Dias Costa Felipe Vincent Yannik Romero Pereira Rafael Alves da Ponte Carlos Coelho Lechner Joo Fernando Doriguello Diniz Luiz Filipe Martins Ramos Guilherme de Sena Brandine Bruno de Nadai Sarnaglia EUREKA! N36, 2012 Cidade Estado Rio de Janeiro RJ Belo Horizonte MG Fortaleza CE So Paulo SP Fortaleza CE S. J. dos Campos SP Campina Grande PB Campinas SP Rio de Janeiro RJ Rio de Janeiro RJ Araraquara SP So Paulo SP Fortaleza CE Lagarto SE So Gonalo RJ Vitria ES Ap. de Goinia GO Fortaleza CE Varginha MG Guaxup MG S. J. dos Campos SP Rio de Janeiro RJ Rio de Janeiro RJ Ribeiro Preto SP S. J. do Rio Preto SP Natal RN Rio de Janeiro RJ Braslia DF Fortaleza CE Rio de Janeiro RJ Santo Andr SP Niteri RJ Fortaleza CE Vila Velha ES Total 278 271 269 252 231 207 205 202 202 192 180 175 165 162 158 157 156 155 153 151 149 148 147 145 144 143 139 139 136 135 134 134 133 132 Prmio Ouro Ouro Ouro Ouro Ouro Prata Prata Prata Prata Prata Prata Prata Bronze Bronze Bronze Bronze Bronze Bronze Bronze Bronze Bronze Bronze Bronze Bronze Bronze Bronze Meno Honrosa Meno Honrosa Meno Honrosa Meno Honrosa Meno Honrosa Meno Honrosa Meno Honrosa Meno Honrosa

85

Sociedade Brasileira de Matemtica


Iuri Rezende Souza Pedro Veras Bezerra da Silva Douglas Machado dos Santos Leandro Farias Maia Willy George do Amaral Petrenko Cassio Henrique Vieira Morais Michel Faleiros Martins Jos Armando Barbosa Filho Alysson Espndola de S Silveira Fernando Nascimento Coelho Gabriel Caser Brito Fernando Fonseca Andrade Oliveira Breno Vieira de Aguiar Thales Graa Athansio Tiago Fonseca Gabriel Queiroz de Brito Melo Rafael Pereira de Paula de Lucas Simon Mineiros GO Rio de Janeiro RJ Eldorado do Sul RS Rio de Janeiro RJ Rio de Janeiro RJ Belo Horizonte MG Campinas SP Fortaleza CE Fortaleza CE Fortaleza CE Rio de Janeiro RJ Belo Horizonte MG Fortaleza CE So Paulo SP So Carlos SP Recife PE Recife PE 132 131 130 128 127 124 124 123 115 114 113 113 110 108 107 107 105 Meno Honrosa Meno Honrosa Meno Honrosa Meno Honrosa Meno Honrosa Meno Honrosa Meno Honrosa Meno Honrosa Meno Honrosa Meno Honrosa Meno Honrosa Meno Honrosa Meno Honrosa Meno Honrosa Meno Honrosa Meno Honrosa Meno Honrosa

EUREKA! N36, 2012

86

Sociedade Brasileira de Matemtica

COORDENADORES REGIONAIS
Alberto Hassen Raad Amrico Lpez Glvez Antonio Carlos Nogueira Benedito Tadeu Vasconcelos Freire Bruno Holanda Carmen Vieira Mathias Claus Haetinger Cludio de Lima Vidal Denice Fontana Nisxota Menegais Disney Douglas Lima de Oliveira Edson Roberto Abe Edney Aparecido Santulo Jr. Fbio Brochero Martnez Florncio Ferreira Guimares Filho Francinildo Nobre Ferreira Genildo Alves Marinho Herivelto Martins Gilson Tumelero Ivanilde Fernandes Saad Joo Bencio de Melo Neto Joo Francisco Melo Libonati Diogo Diniz Jos Luiz Rosas Pinho Jos Vieira Alves Jos William Costa Krerley Oliveira Licio Hernandes Bezerra Luciano G. Monteiro de Castro Luzinalva Miranda de Amorim Marcelo Dias Marcelo Antonio dos Santos Marcelo Rufino de Oliveira Newman Simes Nivaldo Costa Muniz Osnel Broche Cristo Uberlndio Batista Severo Raul Cintra de Negreiros Ribeiro Reginaldo de Lima Pereira Reinaldo Gen Ichiro Arakaki Ricardo Amorim Ronaldo Alves Garcia Rogrio da Silva Igncio Rosangela Ramon Seme Gebara Neto Tadeu Ferreira Gomes Toms Menndez Rodrigues Valdenberg Arajo da Silva Wagner Pereira Lopes Wanderson Breder William Serafim dos Reis
EUREKA! N36, 2012

(UFJF) (USP) (UFU) (UFRN) (CAEN UFC) (UNIFRA) (UNIVATES) (UNESP) (UNIPAMPA) (UFAM) (Colgio Objetivo de Campinas) (UEM) (UFMG) (UFES) (UFSJ) (Centro Educacional Leonardo Da Vinci) (USP So Carlos) (UTFPR) (UC. Dom Bosco) (UFPI) (Grupo Educacional Ideal) (UFPB) (UFSC) (UFPB) (Instituto Pueri Domus) (UFAL) (UFSC) (Sistema Elite de Ensino) (UFBA) (Grupo Educacional Etapa) FACOS (Grupo Educacional Ideal) (Cursinho CLQ Objetivo) (UFMA) (UFLA) (UFPB) (Colgio Anglo) (Escola Tcnica Federal de Roraima) (UNIFESP) (Centro Educacional Logos) (UFGO) (Col. Aplic. da UFPE) (UNOCHAPEC) (UFMG) (UEBA) (U. Federal de Rondnia) (U. Federal de Sergipe) (CEFET GO) (CEFET RJ) (UFT TO)

Juiz de Fora MG Ribeiro Preto SP Uberlndia MG Natal RN Fortaleza CE Santa Mara RS Lajeado RS S.J. do Rio Preto SP Bag RS Manaus AM Campinas SP Maring PR Belo Horizonte MG Vitria ES So Joo del Rei MG Taguatingua DF So Carlos SP Pato Branco PR Campo Grande MS Teresina PI Belm PA Campina Grande PB Florianpolis SC Campina Grande PB Santo Andr SP Macei AL Florianpolis SC Rio de Janeiro RJ Salvador BA So Paulo SP Osrio RS Belm PA Piracicaba SP So Luis MA Lavras MG Joo Pessoa PB Atibaia SP Boa Vista RR SJ dos Campos SP Nova Iguau RJ Goinia GO Recife PE Chapec SC Belo Horizonte MG Juazeiro BA Porto Velho RO So Cristvo SE Jata GO Nova Friburgo RJ Arraias TO

87

CONTEDO
17 OLIMPADA DE MAIO Enunciados e resultado brasileiro 18 OLIMPADA DE MAIO Enunciados e resultado brasileiro 23 OLIMPADA DE MATEMTICA DO CONE SUL Enunciados e resultado brasileiro 24 OLIMPADA DE MATEMTICA DO CONE SUL Enunciados e resultado brasileiro 54 OLIMPADA INTERNACIONAL DE MATEMTICA (IMO) Enunciados e resultado brasileiro 3 OLIMPADA DE MATEMTICA DA COMUNIDADE DOS PASES DE LNGUA PORTUGUESA Enunciados e resultado brasileiro 27 OLIMPADA IBERO-AMERICANA DE MATEMTICA Enunciados e resultado brasileiro 28 OLIMPADA IBERO-AMERICANA DE MATEMTICA Enunciados e resultado brasileiro ARTIGOS TEOREMA DE MORLEY: O QUE OS TRINGULOS AINDA PODEM NOS REVELAR Daniel Cordeiro de Morais Filho (UFCG), Arthur Cavalcante Cunha (UFCG) e Amauri da Silva Barros (UFAL) JOGOS Bruno Holanda (IMPA) EQUAES FUNCIONAIS PARA OS MAIS JOVENS Ricardo Csar da Silva Gomes (IFCE - Jaguaribe CE) 03

06

09

11

13

16

18

20

22

28

34

Sociedade Brasileira de Matemtica

COMO QUE FAZ? SOLUES DE PROBLEMAS PROPOSTOS PROBLEMAS PROPOSTOS AGENDA OLMPICA COORDENADORES REGIONAIS

46 47 60 61 62

Esta edio da Revista Eureka! dedicada memoria do professor Angelo Barone Netto, que nos deixou neste ano de 2013. Barone foi uma das personalidades mais relevantes da Olimpada Brasileira de Matemtica (OBM), com a qual colaborou ativamente desde seu incio em 1979. Seu trabalho foi fundamental para o desenvolvimento da matemtica olmpica no Brasil. Barone foi homenageado pela OBM durante a realizao da Semana Olmpica em 2009. A comunidade da Olimpada Brasileira de Matemtica (OBM)
EUREKA! N37, 2013

Sociedade Brasileira de Matemtica

18 OLIMPADA DE MAIO
PRIMEIRO NVEL
PROBLEMA 1

Pablo disse: Somo 2 ao dia do meu aniversrio e multiplico o resultado por 2. Somo 4 ao nmero obtido e multiplico o resultado por 5. Ao novo nmero obtido somo o nmero do ms do meu aniversrio (por exemplo, se junho, somo 6) e obtenho 342. Qual a data do aniversrio de Pablo? Encontre todas as possibilidades.

PROBLEMA 2

Chamamos S (n) soma dos algarismos do inteiro n. Por exemplo, S (327) = 3 + 2 + 7 = 12 . Encontre o valor de A = S (1) S (2) + S (3) S (4) + ... + S (2011) S (2012) . (A tem 2012 termos).

PROBLEMA 3

A partir de um quadriltero de papel, como o da figura ao lado, deve-se recortar um novo quadriltero cuja rea seja igual metade da rea do quadriltero original. Somente podemos dobrar o papel uma ou mais vezes e cortar por algumas das linhas das dobras. Descreva as dobras e os cortes e justifique por que a rea obtida a metade.

PROBLEMA 4

Pedro tem 111 fichas azuis e 88 fichas brancas. Existe uma mquina que faz dois tipos de operaes: uma trocar 14 fichas azuis por 11 fichas brancas e outra trocar 7 fichas brancas por 13 azuis. Determine se Pedro pode conseguir, mediante sucessivas operaes com a mquina, aumentar em 33 o nmero total de fichas, de modo que a quantidade de fichas azuis seja igual a

brancas. Se isto for possvel, indique como faz-lo. Se no possvel, explique por qu.
EUREKA! N37, 2013

5 da quantidade de fichas 3

Sociedade Brasileira de Matemtica

PROBLEMA 5

Em uma reunio h 12 pessoas. Sabemos que para cada duas pessoas A e B da reunio h (pelo menos) uma outra pessoa C da reunio que amiga de A e de B. Determine o nmero mnimo de pares de amigos que h na reunio. Obs: Cada pessoa pode integrar vrios pares de amigos. Se X amigo de Y ento Y amigo de X.

SEGUNDO NVEL
PROBLEMA 1

Um nmero de quatro algarismos gago se tem os dois primeiros algarismos iguais entre si e os dois ltimos algarismos iguais entre si. Por exemplo, 3311 e 2222 so nmeros gagos. Encontre todos os nmeros gagos de quatro algarismos que so quadrados perfeitos.
PROBLEMA 2

Temos dois octgonos regulares de papelo. Os vrtices de cada octgono so numerados de 1 a 8, em qualquer ordem (a ordem para um octgono pode ser diferente da ordem do outro). Em seguida os octgonos so sobrepostos, de modo que cada vrtice de um deles fique em contato com um vrtice do outro. Os nmeros dos vrtices em contato so multiplicados, e soma-se os 8 produtos obtidos. Demonstre que, qualquer que seja a ordem em que tenham sido numerados os vrtices, sempre possvel sobrepor os octgonos de maneira que a soma obtida seja maior ou igual a 162.
PROBLEMA 3

No tringulo ABC , verificamos que B = 2C e A > 90o . Seja M o ponto mdio de BC . A perpendicular por C ao lado AC corta a reta AB no ponto D. Demonstre que AM B = DMC .
PROBLEMA 4

Temos seis pontos de maneira que no haja trs pontos colineares e que os comprimentos dos segmentos determinados por estes pontos sejam todos distintos. Consideramos todos os tringulos que tm seus vrtices nesses pontos. Demonstre que um dos segmentos , ao mesmo tempo, o menor lado de um desses tringulos e o maior lado de outro.

EUREKA! N37, 2013

Sociedade Brasileira de Matemtica

PROBLEMA 5

Temos 27 caixas em fila; cada uma delas contm pelo menos 12 bolinhas. A operao permitida transferir uma bolinha de uma caixa para sua vizinha da direita, se essa vizinha da direita tem mais bolinhas. Dizemos que uma distribuio inicial das bolinhas feliz se possvel, mediante uma sucesso de operaes permitidas, fazer com que todas as bolinhas fiquem numa mesma caixa. Determine o menor nmero total de bolinhas de uma distribuio inicial feliz.
RESULTADO BRASILEIRO 2012: Nvel 1 (at 13 anos)
Nome Pedro Henrique Sacramento de Oliveira Bryan Diniz Borck Julia Perdigo Saltiel Lucas Iokio Kawahara Bruno Kenzo Ozaki Brendon Diniz Borck Joo Guilherme Madeira Arajo Lucas Diniz Gonalves Villas Boas Laura Mello DUrso Viana Italo Rennan Lima Silva Cidade Estado Vinhedo SP Porto Alegre RS Rio de Janeiro RJ So Paulo SP So Paulo SP Porto Alegre RS Sobral CE Salvador BA Rio de Janeiro RJ Fortaleza - CE Prmio Ouro Prata Prata Bronze Bronze Bronze Bronze M. Honrosa M. Honrosa M. Honrosa

2012: Nvel 2 (at 15 anos)


Nome Murilo Corato Zanarella Daniel Lima Braga Pedro Jorge Luz Alves Gronemberger Pedro Henrique Alencar Costa Felipe Brando Forte Lucca Morais de Arruda Siaudizionis Eduarda Ramos Bezerra de Alencar Ocimar Mota dos Santos Filho Daniel Santana Rocha Estevo Waldow Cidade Estado Amparo SP Eusbio CE Teresina PI Fortaleza CE Fortaleza CE Fortaleza CE Teresina PI Teresina PI Rio de Janeiro RJ Piraquara - PR Prmio Ouro Prata Prata Bronze Bronze Bronze Bronze M. Honrosa M. Honrosa M. Honrosa

EUREKA! N37, 2013

Sociedade Brasileira de Matemtica

19 OLIMPADA DE MAIO
PRIMEIRO NVEL
PROBLEMA 1

Encontre a quantidade de formas de escrever o nmero 2013 como soma de dois inteiros maiores ou iguais a zero, de modo que ao somar no exista nenhum vaium. OBSERVAO: Na soma 2008 + 5 = 2013 , por exemplo, existe vai-um das unidades s dezenas.

PROBLEMA 2

Elisa soma os dgitos do seu ano de nascimento e observa que o resultado coincide com os dois ltimos dgitos do ano em que nasceu o seu av. Alm disso, os dois ltimos dgitos do ano em que ela nasceu so precisamente a idade atual do seu av. Encontre o ano em que nasceu Elisa e o ano em que nasceu o av dela.

PROBLEMA 3

Seja ABCD um quadrado de papel de lado 10 e P um ponto no lado BC. Ao dobrar o papel no comprimento da reta AP, o ponto B determina o ponto Q, como vemos na figura ao lado. A reta PQ corta o lado CD em R. Calcule o permetro do tringulo PCR.

B P Q

C R

PROBLEMA 4

Pablo escreveu 5 nmeros numa folha e logo aps escreveu os nmeros 6,7,8,8,9,9,10,10,11 e 12 em outra folha que este deu a Sofa, dizendo que esses nmeros so as somas possveis de dois dos nmeros que ele tem escondidos. Decida se com esta informao Sofa pode determinar os cinco nmeros que Pablo escreveu.

PROBLEMA 5

Num quadro est desenhado um quadrado de 88 dividido em 64 quadradinhos de 11 mediante linhas paralelas aos lados. Gustavo apaga alguns segmentos de comprimento 1 de modo que de cada quadradinho de 11 apaga 0, 1 ou 2 lados.
EUREKA! N37, 2013

Sociedade Brasileira de Matemtica

Gustavo afirma que apagou 6 segmentos de longitude 1 da borda do quadrado de 88 e que a quantidade de quadradinhos de 11 que tm exatamente 1 lado apagado igual a 5. Decida se o que afirma Gustavo pode ser verdadeiro. SEGUNDO NVEL
PROBLEMA 1

Sofa somou os nmeros das pginas de um livro comeando pelo nmero 1 na primeira pgina e obteve 2013. Pablo viu como Sofia fez a soma e percebeu que ela pulou uma pgina. Quantas pginas tem o livro e qual o nmero da pgina que Sofia pulou?

PROBLEMA 2

Temos uma rgua sem nmeros e um trissector que marca em qualquer segmento os dois pontos que o dividem em trs partes iguais. Construa o ponto mdio de um segmento dado utilizando exclusivamente estas duas ferramentas.

PROBLEMA 3

Marcamos vrios pontos distintos no plano, e traamos todos os segmentos determinados por esses pontos. Uma reta r no passa por nenhum dos pontos marcados e corta exatamente 60 dos segmentos que foram traados. Quantos segmentos no foram cortados por r? Encontre todas as possibilidades.

PROBLEMA 4

possvel escrever 100 nmeros mpares numa fila de tal forma que a soma de cada 5 nmeros adjacentes seja um quadrado perfeito e que a soma de cada 9 nmeros adjacentes tambm seja um quadrado perfeito?

PROBLEMA 5

Temos 600 cartes. 200 deles tm escrito o nmero 5, 200 tm escrito o nmero 2 e os outros 200 tm escrito o nmero 1. Usando estes cartes queremos formar grupos de tal forma que em cada grupo a soma dos nmeros seja 9. Qual a maior quantidade de grupos que podemos formar?

EUREKA! N37, 2013

Sociedade Brasileira de Matemtica

RESULTADO BRASILEIRO 2013: Nvel 1 (at 13 anos)


Nome Bryan Diniz Borck Lucas dos Anjos Dantas Teixeira Bruno Brasil Meinhart Brendon Diniz Borck Andrey Jhen Shen Chen Guilherme Goulart Kowalczuk Davi Cavalcanti Sena David Felipe Brochero Giraldo Wanderson Ferreira de Almeida Fernando Ribeiro de Senna Cidade Estado Porto Alegre RS So Paulo SP Fortaleza CE Porto Alegre RS So Paulo SP Porto Alegre RS Caruaru PE Belo Horizonte MG Paulista PB Jundia SP Prmio Ouro Prata Prata Bronze Bronze Bronze Bronze M. Honrosa M. Honrosa M. Honrosa

2013: Nvel 2 (at 15 anos)


Nome Joo Csar Campos Vargas Pedro Henrique Sacramento de Oliveira Lucca Morais de Arruda Siaudzionis Lucas Pereira Galvo de Barros Daniel Quinto de Morais Pedro Henrique da Silva Dias Ana Paula Lopes Schuch Gabriel Toneatti Vercelli Marcantnio Soares Figueiredo Andr Yuji Hisatsuga Cidade Estado Passa Tempo MG Vinhedo SP Fortaleza CE Pinheiros SP Rio de Janeiro RJ Porto Alegre RS Porto Alegre RS Osasco SP Juazeirinho PB So Paulo SP Prmio Ouro Prata Prata Bronze Bronze Bronze Bronze M. Honrosa M. Honrosa M. Honrosa

EUREKA! N37, 2013

Sociedade Brasileira de Matemtica

23 OLIMPADA DE MATEMTICA DO CONE SUL


Enunciados e resultado brasileiro
Os estudantes brasileiros tiveram uma participao destacada na Olimpada de Matemtica do Cone Sul que foi realizada entre os dias 27 de outubro e 3 de novembro de 2012 na cidade de Quito, Peru. A equipe foi liderada pelos professores Francisco Bruno Holanda, do Rio de Janeiro (RJ) e Marcelo Tadeu de S Oliveira Sales, de So Paulo (SP).
RESULTADOS DA EQUIPE BRASILEIRA BRA1 BRA2 BRA3 BRA4 Rodrigo Sanches ngelo Henrique Gasparini Fiuza do Nascimento Rafael Rodrigues Rocha de Melo Tadeu Pieres de Matos Belfort Neto Medalha de Ouro Medalha de Prata Medalha de Prata Medalha de Bronze

PRIMEIRO DIA
PROBLEMA 1

Ao redor de uma circunferencia esto escritos 2012 nmeros, cada um deles e igual a 1 ou a 1. Se no h 10 nmeros consecutivos cuja soma seja 0, ache todos os valores possveis da soma dos 2012 nmeros.
PROBLEMA 2

Em um quadrado ABCD, seja P um posto sobre o lado CD, distinto de C e D. No tringulo ABP traa-se as alturas AQ e BR, e seja S o ponto de interseco das retas CQ e DR. Demosntre que ASB = 90 .
PROBLEMA 3

Demonstre que no existem inteiros positivos a, b, c, d, primos entre si dois a dosi, tais que ab + cd , ac + bd , e ad + bc sejam divisores mpares de

( a + b c d )( a b + c d )( a b c + d ) .
SEGUNDO DIA
PROBLEMA 4

Encontre o maior inteiro positivo n, menor que 2012, que cumpra a seguinte propriedade:
EUREKA! N37, 2013

Sociedade Brasileira de Matemtica

Se p um divisor primo de n, ento p 2 1 um divisor de n.


PROBLEMA 5

A e B jogam alternadamente sobre um tabuleiro 2012 2013 com peas suficientes dos tipos:

tipo 1

tipo 2

tipo 3

Em seu turno, A debe colocar uma pea do tipo 1 sobre casas vazias de tabuleiro B em seu turno, debe colocar exatamente uma pea de cada tipo sobre casas vazias do tabuleiro. Perde o jogadore que no puder realizar sua jogada. Se A o primeiro a jogar, determine quem possui uma estratgia vencedora. Observao: As peas podem ser rotacionadas, mas no podem se sobrepor, nem sair do tabuleiro. As peas do tipo 1, 2 e 3 cobrem exatamente 3, 2 e 1 casas do tabuleiro, respectivamente.
PROBLEMA 6

Demonstre que a circunferncia circunscrita do tringulo AMN passa por um ponto fixo distinto de A.

AB 3 < . Sejam M e N respectivamente AC 2 MB NC pontos variveis sobre os lados AB e AC, distintos de A, tais que = 1. AC AB
Considere um tringulo ABC com 1 <

EUREKA! N37, 2013

10

Sociedade Brasileira de Matemtica

24 OLIMPADA DE MATEMTICA DO CONE SUL


Enunciados e resultado brasileiro
Os estudantes brasileiros tiveram uma excelente participao na Olimpada de Matemtica do Cone Sul que foi realizada entre os dias 2 e 7 de junho de 2013, na cidade de Assuno, Paraguai. A equipe foi liderada pelos professores Fabio Brochero Martnez de Belo Horizente (MG) e Jos Armando Barbosa de Fortaleza (CE).
RESULTADOS DA EQUIPE BRASILEIRA BRA1 BRA2 BRA3 BRA4 Victor Oliveira Reis Murilo Corato Zanarella Pedro Henrique Sacramento de Oliveira Daniel Santana Rocha Medalha de Ouro Medalha de Ouro Medalha de Prata Medalha de Prata

PRIMEIRO DIA
PROBLEMA 1

Sobre uma reta marcamos quatro pontos distintos. Para cada ponto marcado calculada a soma das distncias deste ponto aos outros trs, obtendo assim quatro valores. Decidir se possvel que os quatro valores sejam, em alguma ordem: a) 29, 29, 35, 37 ; b) 28, 29, 35, 37 ; c) 28, 34, 34, 37.
PROBLEMA 2

Em um tringulo ABC, denotamos por M o ponto mdio do lado BC e por I o ponto de interseo de suas bissetrizes. Se IM = IA, determinar o menor valor possvel para a medida do ngulo AIM .
PROBLEMA 3

Semciclolndia um pas com 500 cidades e 2013 estradas de mo dupla, cada uma conectando diretamente duas cidades. Duas cidades A e B so chamadas de vizinhas se existe uma estrada que as conecta e duas cidades A e B so chamadas de quase-vizinhas se existe uma cidade C tal que A vizinha de C e C vizinha de B. Sabemos que em Semciclolndia no existem duas cidades conectadas diretamente por mais de uma estrada e no existem quatro cidades A, B, C e D tais que simultaneamente A vizinha de B, B vizinha de C, C vizinha de D e D vizinha de A. Demonstrar que existe uma cidade que quase-vizinha de pelo menos 57 cidades.
EUREKA! N37, 2013

11

Sociedade Brasileira de Matemtica

SEGUNDO DIA
PROBLEMA 4

Seja M o conjunto dos nmeros inteiros de 1 at 2013 inclusive. A cada um dos subconjuntos de M atribumos uma das k cores disponveis, com a condio de que, se dois conjuntos distintos A e B cumprem que A B = M , ento aos conjuntos A e B so atribudas cores distintas. Qual o menor valor possvel que pode ter k?

PROBLEMA 5

Seja d ( k ) o nmero de divisores positivos do inteiro k. Um nmero n chamado equilibrado se:

d ( n 1) d ( n ) d ( n + 1) ou d ( n 1) d ( n ) d ( n + 1) .
Demonstrar que existem infinitos nmeros equilibrados.

PROBLEMA 6

Seja ABCD um quadriltero convexo. Seja n 2 um nmero inteiro. Demonstrar que existem n tringulos da mesma rea cumprindo todas as seguintes propriedades: Seus interiores so disjuntos, isto , os tringulos no se sobrepem; Cada tringulo est contido em ABCD ou em seu interior; A soma das reas dos tringulos pelo menos quadriltero ABCD.

4n 4n + 1

da rea do

EUREKA! N37, 2013

12

Sociedade Brasileira de Matemtica

54 OLIMPADA INTERNACIONAL DE MATEMTICA (IMO)


Enunciados e resultado brasileiro
O Brasil obteve um timo resultado na 54a Olimpada Internacional de Matemtica (IMO), que aconteceu entre os dias 20 e 28 de julho na cidade de Santa Marta na Colmbia, conquistando trs medalhas de prata, uma de bronze e duas menes honrosas. A equipe foi liderada pelos professores Edmilson Motta de So Paulo (SP) e Onofre Campos da Silva Farias de Fortaleza (CE).
RESULTADOS DA EQUIPE BRASILEIRA BRA1 BRA2 BRA3 BRA4 BRA5 BRA6 Rodrigo Sanches ngelo Rafael Kazuhiro Miyazaki Victor Oliveira Reis Franco Matheus de Alencar Severo Alessandro de Oliveira Pacanowski Victor de Oliveira Bitares Medalha de Prata Medalha de Prata Medalha de Prata Medalha de Bronze Meno Honrosa Meno Honrosa

PRIMEIRO DIA
PROBLEMA 1

Demonstrar que, para qualquer par de inteiros positivos k e n, existem k inteiros positivos m1 , m2 ,..., mk (no necessariamente distintos) tais que:

1+
PROBLEMA 2

2k 1 1 1 1 = 1 + 1 + ... 1 + n m1 m2 mk

Uma configurao de 4027 pontos do plano dos quais 2013 so vermelhos e 2014 azuis, e no h trs pontos colineares, diz-se colombiana. Traando algumas retas, o plano fica dividido em vrias regies. Um conjunto de retas bom para uma configurao colombiana se satisfaz as duas seguintes condies: Nenhuma reta passa por algum ponto da configurao; Nenhuma regio contem pontos de ambas as cores.

Encontrar o menor valor de k tal que, para qualquer configurao colombiana de 4027 pontos, h um conjunto bom de k retas.

EUREKA! N37, 2013

13

Sociedade Brasileira de Matemtica

PROBLEMA 3

Seja A1 o ponto de tangncia do excrculo do tringulo ABC oposto ao vrtice A com o lado BC. Definem-se os pontos B1 em CA e C1 em AB , de modo anlogo, uitilizando os excrculos opostos a B e a C, respectivamente. Suponha que o circuncentro do tringulo A1 B1C1 pertence circunferncia circunscrita ao tringulo ABC. Demonstrar que o tringulo ABC e retngulo. O excrculo de ABC oposto ao A e a circunferncia que tangente ao segmento BC, ao prolongamento do lado AB no sentido de A para B e ao prolongamento do lado AC no sentido de A para C. Os excrculos opostos a B e a C definem-se de modo semelhante.

SEGUNDO DIA
PROBLEMA 4

Seja ABC um tringulo acutngulo com ortocentro H e seja W um ponto do lado BC, estritamente entre B e C. Os pontos M e N so os pes das alturas traadas desde B e C, respectivamente. Designa-se por w1 a circunferncia circunscrita ao tringulo BWN; seja X o ponto de w1 tal que WX um dimetro de w1 . Analogamente, designa-se por w2 a circunferncia circunscrita ao tringulo CWM; seja Y o ponto de w2 tal que WY um dimetro de w2 . Demonstrar que os pontos X, Y e H so colineares.
PROBLEMA 5

Seja

>0 >0

o conjunto dos nmeros racionais maiores do que zero. Seja uma funo que satisfaz as trs seguintes condies:
>0

f:

(i)

f ( x ) f ( y ) f ( xy ) para quaisquer x, y

;
>0

(ii) f ( x + y ) f ( x ) + f ( y ) para quaisquer x, y Demonstrar que f ( x ) = x para qualquer x

(iii) Existe um nmero racional a > 1 tal que f ( a ) = a.


>0

EUREKA! N37, 2013

14

Sociedade Brasileira de Matemtica

PROBLEMA 6

Seja n 3 um nmero inteiro. Considera-se uma circunferncia na qual esto marcados n + 1 pontos igualmente espaados. A cada ponto atribui-se um dos nmeros 0, 1,..., n de modo que cada nmero e usado exatamente uma vez; duas atribuies de nmeros consideram-se a mesma se uma pode ser obtida da outra por uma rotao da circunferncia. Uma atribuio de nmeros chama-se bonita se, para quaisquer quatro nmeros a < b < c < d com a + b = b + c, a corda que une os pontos correspondentes a a e a d no intersecta a corda que une os pontos correspondentes a b e a c. Sejam M o nmero de atribuies bonitas o N o nmero de pares ordenados ( x, y ) de inteiros positivos que x + y n e mdc ( x, y ) = 1. Demonstrar que

M = N + 1.

EUREKA! N37, 2013

15

Sociedade Brasileira de Matemtica

3 OLIMPADA DE MATEMTICA DA COMUNIDADE DOS PASES DE LNGUA PORTUGUESA


Enunciados e resultado brasileiro
O Brasil conquistou uma medalha de ouro e trs de prata na 3 Olimpada de Matemtica da Comunidade dos Pases de Lngua Portuguesa, realizada de 5 a 10 de agosto, na cidade de Maputo, Moambique. Com este resultado o pas ficou pelo terceiro ano consecutivo com a primeira posio na classificao geral, seguido pela equipe de Portugal. A equipe foi liderada pelos professores Carlos Bahiano, de Salvador (BA) e Marcelo Xavier de Mendona, do Rio de Janeiro (RJ).
RESULTADOS DA EQUIPE BRASILEIRA BRA1 BRA2 BRA3 BRA4 Gabriel Toneatti Vercelli Lucca Morais de Arruda Siaudzionis Lucas Pereira Galvo de Barros Joo Csar Campos Vargas Medalha de Ouro Medalha de Prata Medalha de Prata Medalha de Prata

PRIMEIRO DIA
PROBLEMA 1

Quantos cestos e quantas laranjas tem a Xiluva sabendo que se ela arruma duas laranjas em cada cesto ficam quatro laranjas de sobra e se ela arruma cinco laranjas em cada cesto fica um cesto vazio?
PROBLEMA 2

Seja ABC um tringulo acutngulo. A circunferncia de dimetro AB intercepta os lados AC e BC nos pontos E e F respectivamente. As tangentes ao crculo nos pontos E e F se cruzam em P. Mostre que o ponto P pertence altura do tringulo ABC a partir do vrtice C.
PROBLEMA 3

Um evento ocorre h muitos anos, realizando-se regularmente em x anos consecutivos seguidos de uma pausa de y anos consecutivos. Sabe-se que o evento se realizou em 1964, 1986, 1996 e 2008 e no se realizou em 1976, 1993, 2006 e 2013. Qual o primeiro ano em que o evento se vai realizar novamente?

EUREKA! N37, 2013

16

Sociedade Brasileira de Matemtica

SEGUNDO DIA
PROBLEMA 4

Determine todos os pares ( x, y ) de inteiros positivos que satisfazem a equao

x 2 xy + 2 x 3 y = 2013.
PROBLEMA 5

Determine todos os nmeros de 5 algarismos no nulos tais que apagando consecutivamente o algarismo da esquerda, em cada etapa, se obtm um divisor do nmero anterior.
PROBLEMA 6

Considere um tringulo ABC. Seja S a circunferncia que tangencia internamente os lados BC, CA e AB do tringulo nos pontos D, E e F, respectivamente. Externamente ao tringulo constroem-se trs circunferncias S A , S B e SC . A circunferncia S A tangente a BC no ponto L e aos prolongamentos das retas AB e BC, nos pontos M e N, respectivamente. A circunferncia S A tangente a BC no ponto L e aos prolongamentos das retas AB e AC nos pontos M e N, respectivamente. A circunferncia S B tangente a AC no ponto E e ao prolongamento da reta BC no ponto P. Prove que as retas EP, FQ e AL so concorrentes em um nico ponto sobre S.

EUREKA! N37, 2013

17

Sociedade Brasileira de Matemtica

27 OLIMPADA IBERO-AMERICANA DE MATEMTICA


Enunciados e resultado brasileiro
A equipe brasileira formada por quatro estudantes do ensino mdio conquistou duas medalhas de ouro e duas de prata na 27 Olimpada IberoAmericana de Matemtica (OIM), realizada entre os dias 29 de setembro e 6 de outubro na cidade de Cochabamba, Bolvia. A equipe foi liderada pelos professores Matheus Secco e Hugo Fonseca, ambos do Rio de Janeiro (RJ).
RESULTADOS DA EQUIPE BRASILEIRA BRA1 BRA2 BRA3 BRA4 Rafael Kazuhiro Miyazaki Rodrigo Sanches ngelo Franco Matheus de Alencar Severo Andr Macieira Braga Costa Medalha de Ouro Medalha de Ouro Medalha de Prata Medalha de Prata

PRIMEIRO DIA
PROBLEMA 1

Sobre o retngulo ABCD constroem-se os tringulos equilteros BCX e DCY, de modo que cada um compartilhe pontos com o interior do retngulo. A reta AX intersecta a reta DC em P. A reta AY intersecta a reta BC em Q. Demonstrar que o tringulo APQ equiltero.
PROBLEMA 2

Um inteiro positivo diz-se bissomado se puder ser escrito como soma de dois inteiros positivos que tenham a mesma soma dos algarismos. Por exemplo, 2012 bissomado, pois 2012 = 2005 + 7 e tanto 2005 como 7 tm soma dos algarismos igual a 7. Encontrar todos os inteiros positivos que no so bissomados.
PROBLEMA 3

Seja n um inteiro positivo. Dado um conjunto {a1 , a2 ,..., an } de inteiros entre 0 e

2n 1 inclusive, associados a cada um dos seus 2n subconjuntos a soma dos seus elementos; em particular, o subconjunto vazio tem soma 0. Se estas 2n somas deixam todas restos distintos na diviso por 2n , dizemos que o conjunto {a1 , a2 ,..., an } n-completo. Determinar, para cada n, a quantidade de conjuntos
n-completos.
EUREKA! N37, 2013

18

Sociedade Brasileira de Matemtica

SEGUNDO DIA
PROBLEMA 4

Sejam a, b, c, d nmeros inteiros tais que a b + c d mpar e divide

a 2 b 2 + c 2 d 2 . Demonstrar que, para qualquer inteiro positivo n, a b + c d divide a n b n + c n d n .


PROBLEMA 5

Seja ABC um tringulo e sejam P e Q os pontos de interseo da paralela a BC por A com as bissetrizes exteriores dos ngulos B e C, respectivamente. A perpendicular a BP por P e a perpendicular a CQ por Q intersectam-se em R. Seja I o incentro de ABC. Demonstrar que AI = AR.
PROBLEMA 6

Demonstrar que, para qualquer inteiro positivo n, existem n inteiros positivos consecutivos tais que nenhum deles divisvel pela soma dos seus respectivos algarismos.

EUREKA! N37, 2013

19

Sociedade Brasileira de Matemtica

28 OLIMPADA IBERO-AMERICANA DE MATEMTICA


Enunciados e resultado brasileiro
A equipe brasileira formada por quatro estudantes do ensino mdio conquistou, por segundo ano consecutivo, o primeiro lugar geral na 28 Olimpada Ibero-Americana de Matemtica (OIM), realizada entre os dias 20 e 28 de setembro na cidade do Panam, Panam. O evento contou com a participao de 78 jovens de 20 pases da Amrica Latina, Portugal e Espanha. A equipe foi liderada pelos professores Eduardo Wagner do Rio de Janeiro (RJ) e Pablo Rodrigo Ganassim de So Paulo (SP).

RESULTADOS DA EQUIPE BRASILEIRA BRA1 BRA2 BRA3 BRA4 Rodrigo Sanches ngelo Rafael Kazuhiro Miyazaki Franco Matheus de Alencar Severo Victor Oliveira Reis Medalha de Ouro Medalha de Prata Medalha de Prata Medalha de Prata

PRIMEIRO DIA
PROBLEMA 1

Um conjunto S de inteiros positivos distintos chama-se canaleiro se para quaisquer trs nmeros a, b, c S , todos diferentes, se tem que a divide bc, b divide ca e c divide ab. a) Demonstrar que para qualquer conjunto finito de inteiros positivos {c1 , c2 ,..., cn } existem infinitos inteiros positivos k, tais que o conjunto

{kc1 , kc2 ,..., kcn } canaleiro.


b) Demonstrar que para qualquer inteiro n 3 existe um conjunto canaleiro que tem exatamente n elementos e tal que nenhum inteiro maior que 1 divide todos os seus elementos.
PROBLEMA 2

Sejam X e Y os extremos de um dimetro de uma circunferncia e N o ponto mdio de um dos arcos XY de . Sejam A e B dois pontos do segmento XY. As retas NA e NB cortam novamente nos pontos C e D, respectivamente. As tangentes a em C e D encontram-se em P. Seja M o ponto de interseco do
EUREKA! N37, 2013

20

Sociedade Brasileira de Matemtica

segmento XY com o segmento NP. Demonstrar que M o ponto mdio do segmento AB.
PROBLEMA 3

Seja A = {1, 2,3,..., n} com n > 5. Demosntrar que existe um conjunto finito B de inteiros positivos distintos tal que A B e possui a propriedade:

x =x ,
2
xB xB

Ou seja, o produto dos elementos de B igual soma dos quadrados dos elementos de B.

SEGUNDO DIA
PROBLEMA 4

Sejam uma circunferncia de centro O, AE um dimetro de e B o ponto mdio de um dos arcos de AE de . O ponto D E est sobre o segmento OE. O ponto C tal que o quadriltero ABCD um paralelogramo com AB paralelo a CD e BC paralelo a AD. As retas EB e CD cortam-se no ponto F. A reta OF corta o menor arco EB de no ponto I. Demonstrar que a reta EI a bissetriz do ngulo BEC.
PROBLEMA 5

Sejam A e B dois conjuntos tais que: i) A B o conjunto dos inteiros positivos. ii) A B vazio. iii) Se dois inteiros positivos tm como diferena um primo maior que 2013, ento um deles est em A e o outro em B. Encontrar todas as possibilidades para os conjuntos A e B.
PROBLEMA 6

Uma configurao um conjunto finito S de pontos do plano entre os quais no h trs colineares e tal que a cada ponto se atribui alguma cor, de modo que se um tringulo cujos vrtices esto em S tem um ngulo maior ou igual a 120, ento exatamente dois de seus vrtices so de uma mesma cor. Encontrar o nmero mximo de pontos que pode ter uma configurao.
EUREKA! N37, 2013

21

Sociedade Brasileira de Matemtica

TEOREMA DE MORLEY: O QUE OS TRINGULOS AINDA PODEM NOS REVELAR


Daniel Cordeiro de Morais Filho, UFCG Arthur Cavalcante Cunha, UFCG Amauri da Silva Barros, UFAL
Nvel Intermedirio
INTRODUO

Problemas de Geometria sempre encantaram os admiradores da Matemtica. Como a Geometria uma das mais antigas manifestaes matemticas da humanidade, era de se esperar, h alguns sculos atrs, que os mais importantes resultados sobre tringulos j tivessem sido descobertos e estabelecidos. Mas isso no verdade. No comeo do sculo passado ainda se estava descobrindo resultados envolvendo tringulos, e sobre um desses belos resultados que falaremos neste artigo.
O PROBLEMA

Como sabemos, as bissetrizes de um tringulo qualquer, semirretas que dividem os ngulos internos em dois ngulos congruentes, se intersectam em um ponto chamado incentro. Esse ponto tem a interessante propriedade de ser o centro de uma circunferncia inscrita no tringulo. O famoso matemtico grego Euclides (c.325 a.C-c.365 a.C.) provou esse resultado no Livro IV dos seus Elementos (Proposio 4), h pelo menos 24 sculos! Nessa direo, a histria guardou uma pergunta bem natural que parece no ter sido feita ao longo dos sculos: e se dividirmos os ngulos internos de um tringulo em trs partes iguais? Encontraremos algum resultado tambm interessante? Vejamos. As duas semirretas que dividem um ngulo interno de um tringulo em trs ngulos congruentes chamam-se trissetrizes. As trissetrizes de um tringulo geram seis pontos de interseo. H alguma propriedade interessante que envolve esses pontos? Para ver o que ocorre, preciso distinguir um pouco mais essas trissetrizes. Diremos que duas trissetrizes so adjacentes quando partem de vrtices opostos pertencentes a um mesmo lado do tringulo e formam o menor ngulo que uma trissetriz pode formar com esse lado. Una os pontos de interseo das trissetrizes adjacentes de um tringulo qualquer. Voc obter um tringulo. Observe melhor, mais do que isso, voc descobrir um resultado surpreendente: o tringulo obtido sempre um tringulo eqiltero! Vide figura a seguir.
EUREKA! N37, 2013

22

Sociedade Brasileira de Matemtica

Figura 1: O Tringulo equiltero DEF chamado Tringulo de Morley.

Esse o Teorema de Frank Morley (1860-1937), matemtico anglo-americano, presidente da American Mathematical Society no binio 1919-1920, que provou esse teorema no final do Sculo XIX (vide uma pequena bibliografia de Morley em [1]). Na verdade, a demonstrao do teorema como conhecido hoje s foi publicada em 1929 ([2]). O enunciado do teorema o seguinte:
Teorema de Morley: Em um tringulo qualquer, a unio dos pontos de interseo das trissetrizes adjacentes forma um tringulo equiltero. Uma animao do teorema pode ser vista em [3]. Porque esse problema importante

Bem, alm de ser um belo resultado, esse problema envolve um dos antigos problemas famosos da Matemtica, que a trisseco de um ngulo em trs ngulos congruentes. Saber se possvel fazer isso para qualquer ngulo usando apenas rgua e compasso um problema famoso, que levou 20 sculos para ser
EUREKA! N37, 2013

23

Sociedade Brasileira de Matemtica

resolvido, e a soluo negativa (vide [4]): no possvel, em geral, trissectar ngulos com rgua e compasso. Por essa razo, talvez os resultados envolvendo trisseco de ngulos tenham sido considerados desinteressantes, e velado o teorema at ser descoberto e provado apenas no comeo do sculo XX. Mas, convenhamos, na prtica, ningum precisa trissectar um ngulo para descobrir ou provar o Teorema de Morley! Em segundo lugar, das demonstraes possveis desse teorema (e h vrias, dos mais variados gostos, e algumas at bem modernas. Vide [5].) optamos pela que usasse certos resultados vistos na Trigonometria, que s vezes so apenas memorizados pelos estudantes do Ensino Mdio. Na maioria das vezes, esses resultados so apenas apresentados sem qualquer aplicao de destaque que merecesse estud-los. Nossa demonstrao tambm fornece a oportunidade de usar algumas identidades trigonomtricas como as que transformam produtos em soma, bem como uma aplicao das Leis dos Senos e dos Cossenos em um nico problema.
Demonstrao do Teorema de Morley

Consideremos o tringulo ABC, da Figura 1, com ngulos internos a = B AC ,

b = ABC e c = ACB. Consideremos tambm o tringulo DEF , formado pela


interseo das trissetrizes adjacentes relativas a esses ngulos. A ideia dessa demonstrao calcular os comprimentos dos lados do tringulo DEF , mostrando que so iguais. Como usaremos a Lei dos Senos, seja R o raio da circunferncia inscrita ao tringulo ABC. Vamos mostrar que

a b c DE = EF = FD = 8 R sen sen sen . 3 3 3


Iremos usar as seguintes identidades trigonomtricas: 1.

x x+ x + 2 sen x = 4sen sen sen 3 3 3 1 2. sen 2 x = (1 cos 2 x ) 2 1 3. sen x seny = cos ( x y ) cos ( x + y ) 2 a + b + 2 2 + c c c 4. cos = cos = cos = cos 3 3 3 3

EUREKA! N37, 2013

24

Sociedade Brasileira de Matemtica

5.

1 x+ y x y cos ( cos x + cos y ) = cos 2 2 2

Pela Lei dos Senos aplicada ao tringulo ABC, e pela identidade (1) temos

6.

BC a a+ a + 2 = 2 R BC = 8R sen sen sen . sen a 3 3 3

Considere agora o tringulo BDC. As medidas de seus ngulos internos so:

b c b c 3 b c ( b c ) + 2 a + 2 C BD = , BCD = e BDC = = = = . 3 3 3 3 3 3 3
Agora, aplicando a Lei dos Senos ao tringulo BDC e usando a igualdade (6) obtemos:

BC sen b BC CD 3 = CD = a+2 sen a +32 sen b sen 3 3


CD =
a + a+2 b 8R sen a 3 sen 3 sen 3 sen 3 sen a +32

7.

a b a+ CD = 8R sen sen sen . 3 3 3

Fazendo o mesmo para o tringulo ACE, resulta em

8.

a b b+ CE = 8 R sen sen sen . 3 3 3

Para encontrar o comprimento do lado DE, usaremos a Lei dos Cossenos ao tringulo CDE, e por (7) e (8):

c DE 2 = CD 2 + CE 2 2CD CE cos 3
EUREKA! N37, 2013

25

Sociedade Brasileira de Matemtica

( 9.)

a b a+ b+ a+ b+ c DE 2 = 64R2sen 2 sen 2 sen 2 + sen 2 2sen sen cos . 3 3 3 3 3 3 3

Para simplificar a escrita, chamemos

a b M = 8Rsen sen , donde 3 3 M 2 = 64R 2sen 2 a b sen 2 . 3 3

De (2) e (3), segue da expresso (9) que

1 2a + 2 1 2b + 2 1 a b a + b + 2 c DE2 =M 2 1 cos cos + 1 cos 2 cos cos . 3 2 3 2 3 3 3 2


De (4) e (5), obtemos

1 2a + 2 2b + 2 a b c c c DE 2 =M 2 1 + 1 cos cos + cos cos = cos cos 3 3 3 3 3 3 2


c ab a b c c c c = M 2 1 cos cos cos cos 2 = M 2 1 cos 2 = M 2 sen 2 . cos 3 3 3 3 3 3 3
Logo,

DE = Msen

c a b c =8R sen sen sen . 3 3 3 3

Se fizermos o mesmo procedimento para os tringulos BDF e AFE, encontraremos que o comprimento dos lados DF e EF do tringulo DEF tambm dado por

a b c DF =EF = 8R sen sen sen . 3 3 3


Portanto, como DE, DF e EF so iguais, o tringulo DEF equiltero. Como queramos demonstrar.
EUREKA! N37, 2013

26

Sociedade Brasileira de Matemtica

Concluso

Em [3] pode-se encontrar 23 demonstraes desse teorema. Nossa demonstrao baseia-se no artigo [5], onde pode ser encontrado um detalhado histrico desse teorema, que parece ter cado no gosto de muitos. Enfim, ficar famoso por provar, em pleno Sculo XX, um teorema da Geometria Elementar um fato que muitos gostariam de ter tido o privilgio de que ocorresse com eles. Coisas da Histria, coisas da Matemtica...

BIBLIOGRAFIA [1] http://faculty.evansville.edu/ck6/bstud/morley.html [2] American Journal of Mathematics, F. Morley, 51 (1929), pp. 465-472. [3] http://www.cut-the-knot.org/triangle/Morley/index.shtml [4] Gonalves, Adilson; Introduo lgebra, IMPA (1999). [5] Cletus O. Oakley e Justine Davis, The Morley Trisector Theorem, Amer. Math Monthly (1978), p. 737-745.O artigo pode ser encontrado na pgina http://www.haverford.edu/math/cgreene/399/morley/morley.pdf.
EUREKA! N37, 2013

27

Sociedade Brasileira de Matemtica

JOGOS
Bruno Holanda, Fortaleza CE

Nvel Iniciante
Problemas sobre jogos esto entre os mais atrativos para a maioria dos alunos que esto iniciando o seu gosto pela matemtica e, por isso, vm ganhando grande importncia nas provas da OBM. Neste artigo vamos mostrar as duas idias que mais aparecem nas provas: a simetria e o uso das posies vencedoras.
1. Simetria

Uma das estratgias mais simples o uso de alguma simetria que pode ocorrer durante o jogo em vantagem de um dos jogadores, forando sempre uma nova rodada para o jogador destinado derrota. Para entender melhor, veja o seguinte exemplo:
Exemplo 1.1: Pedro e Mnica jogam em um tabuleiro 1 11 . Cada um, em sua vez, pode pintar um dos quadrados (que no foram pintados anteriormente), ou dois quadrados consecutivos (se ambos estiverem brancos). Quem no puder mais jogar perde. Sabe-se que Pedro ser o primeiro a jogar. Quem pode sempre garantir a vitria? Soluo. Pedro sempre poder ganhar se seguir a seguinte estratgia: i. Inicialmente, Pedro deve pintar o quadrado do meio.

ii. Agora, depois que Mnica fizer sua jogada, Pedro deve jogar sempre simetricamente em relao ao centro do tabuleiro (i.e. sempre deixando o tabuleiro simtrico). Por exemplo, se Mnica jogar nas casas 9 e 10, Pedro deve jogar nas casas 2 e 3.

iii. Assim, Mnica nunca poder ganhar, pois, na sua jogada, ela recebe uma posio simtrica e quebra a simetria, e na configurao final do jogo, todas as casas estaro pintadas, ou seja, a configurao simtrica.

EUREKA! N37, 2013

28

Sociedade Brasileira de Matemtica

O prximo exemplo um dos problemas que apareceu na prova da OBM de 2004. Vamos apresentar uma soluo diferente da soluo proposta na Eureka! 22, usando simetria:
Exemplo 1.2: Arnaldo e Bernaldo disputam um jogo em um tabuleiro 2 n :

As peas do jogo so domins 2 1 . Inicialmente Arnaldo coloca um domin cobrindo exatamente duas casas do tabuleiro, na horizontal ou na vertical. Os jogadores se revezam colocando uma pea no tabuleiro, na horizontal ou na vertical, sempre cobrindo exatamente duas casas do tabuleiro. No permitido colocar uma pea sobre outra j colocada anteriormente. Quem no conseguir colocar uma pea no tabuleiro perde. Qual dos dois jogadores tem uma estratgia vencedora, ou seja, uma estratgia que o leva vitria quaisquer que sejam as jogadas de seu adversrio, para: (a) n = 2004 ? (b) n = 2005 ?
Soluo. Quando n = 2005 o primeiro jogador garante a vitria. Ele pode fazer isto colocando um domin na vertical no meio do tabuleiro e, em seguida, jogar simetricamente ao segundo jogador. Quando n = 2004 o tabuleiro possui um nmero par de colunas. Desse modo, o segundo ganha jogando simetricamente ao primeiro jogador. PS: Quando falamos jogar simetricamente significa fazer um movimento para deixar o tabuleiro simtrico em relao ao centro. 2. Posies vencedoras

Alguns tipos de jogos possuem certas configuraes que sempre levam um jogador vitria. Essas configuraes so chamadas de posies vencedoras. O prximo exemplo um jogo bastante simples em que essa estratgia aparece facilmente.

EUREKA! N37, 2013

29

Sociedade Brasileira de Matemtica

Exemplo 2.1: Na primeira casa de um tabuleiro 1 13 est um pino. Tiago e Maria

movem o pino alternadamente. Em cada turno permitido avanar 1,2,3,4 ou 5 casas. Quem colocar o pino na ltima casa o vencedor. Se Maria comear jogando, ela pode ter certeza da vitria?
Soluo. Como em muitos problemas de olimpada, vamos analisar alguns casos pequenos. Vamos supor que em vez de 13 casas o tabuleiro tivesse apenas quatro. Neste caso, fica fcil ver que quem comea ganha: basta avanar trs casas. O mesmo iria ocorrer se o tabuleiro tivesse 2,3,4,5 ou 6 casas. Porm, veja que em um tabuleiro 7 1 o primeiro jogador perde. Isso acontece pois, aps sua jogada, o pino ficar em uma das casas 2,3,4,5 ou 6. E j sabemos que essas casas levam o jogador vitria.

Desse modo, vamos dizer que 7 uma posio perdedora e 6,5,4,3 e 2 so posies vencedoras. Agora, se um o jogador que estiver em uma das casas 8,9,10,11 ou 12 pode garantir a vitria movendo o pino para a casa 7. Ou seja, deixando o seu adversrio em uma posio perdedora. Assim, podemos afirmar que as posies 8,9,10,11 e 12 tambm so vencedoras. Resta analisar a 13 casa. Observe que a partir desta casa podemos mover o pino apenas para uma das casas 8,9,10,11 ou 12, que so vencedoras. Da, quem comear perde pois, inicia em uma posio perdedora. A grande dificuldade para a maioria dos alunos descobrir quais so as posies vencedoras de um jogo. Para evitar esse tipo de problema, tenha sempre em mente as seguintes definies:
Posio vencedora: A partir dela, podemos escolher um movimento e repassar uma

posio perdedora para o adversrio.


Posio perdedora: A partir dela, impossvel escolher um movimento e repassar

uma posio perdedora para o adversrio. Ou seja, no importa o movimento escolhido, o adversrio ir receber uma posio vencedora.

EUREKA! N37, 2013

30

Sociedade Brasileira de Matemtica

E como fazer para descobrir quais so as posies vencedoras e perdedoras? A melhor maneira de se fazer isto analisando o final do jogo e aplicar as definies acima. Vamos praticar um pouco resolvendo o seguinte problema:
Exemplo 2.2: Em um tabuleiro 8 8 , uma torre est na casa a1. Dois jogadores

movem a torre com objetivo de colocar a torre na casa h8. Sabendo que a torre pode mover-se apenas para cima ou para direita (quantas casas o jogador desejar) e que no se pode passar a vez, determine qual jogador tem a estratgia vencedora.
Soluo. Primeiramente note que todas as casas da ltima linha e da ltima coluna

(exceto a h8) so vencedoras pois, a partir delas podemos escolher um movimento que nos leve vitria. Com isso, a casa g7 se torna perdedora pois, a partir dela qualquer movimento leva o outro jogador a uma posio vencedora (veja a figura 1).

Agora, como g7 perdedora, as demais casas da stima linha e da stima coluna so vencedoras. Mais ainda, a casa f6 tambm deve ser perdedora (figura 2). Continuando de maneira anloga, obtemos que a casa a1 perdedora (figura 3). Logo, quem comear, perde.

EUREKA! N37, 2013

31

Sociedade Brasileira de Matemtica

3. Problemas

1. Sobre uma mesa existem duas pilhas (uma com 15 e outra com 16 pedras). Em um jogo cada jogador pode, em sua vez, retirar qualquer quantidade de pedras de apenas uma pilha. Quem no puder mais jogar perde. Quem possui a estratgia vencedora? 2. Dois jogadores colocam alternadamente reis (de cores todas distintas) em um tabuleiro 9 9 , de forma que nenhum rei ataque outro. Quem no puder mais jogar perde. Quem possui a estratgia vencedora? 3. (OBM 2002) So dados um tabuleiro de xadrez ( 8 8 ) e palitinhos do tamanho dos lados das casas do tabuleiro. Dois jogadores jogam alternadamente e, em cada rodada, um dos jogadores coloca um palitinho sobre um lado de uma das casas do tabuleiro, sendo proibido sobrepor os palitinhos. Vence o jogador que conseguir completar primeiro um quadrado 1 1 de palitinhos. Supondo que nenhum dos jogadores cometa erros, qual dos dois tem a estratgia vencedora? 4. (Leningrado 1987) Dois jogadores colocam alternadamente 's e 's em um tabuleiro 9 9 . O primeiro escreve 's e o segundo 's. Quando o tabuleiro for completamente preenchido o jogo termina e os pontos so contados. Um ponto dado ao jogador para cada linha ou coluna em que ele possuir mais casas dos que o adversrio. O jogador que possuir mais pontos vence. Quem pode sempre ganhar? 5. (Leningrado 1989) Um pino est no centro de um tabuleiro 11 11 . Dois jogadores fazem alternadamente o pino saltar para qualquer outra casa do tabuleiro, mas a cada movimento (a partir do segundo) o tamanho do salto deve ser maior que o anterior. O jogador que no puder mais jogar perde. Ache a estratgia vencedora. 6. Sobre uma mesa existem 2006 pedras. Em um jogo, cada jogador pode, em sua vez, retirar de 1 a 10 pedras (mas sempre retirando pelo menos uma pedra). Ganha o jogador que retirar a ltima pedra. Qual dos jogadores possui a estratgia vencedora? 7. (Leningrado 1990) Tom e Jerry jogam um jogo e Tom faz a primeiro passo. Em cada turno o jogador pode diminuir de um dado natural N um
EUREKA! N37, 2013

32

Sociedade Brasileira de Matemtica

dos seus dgitos no-nulos. Inicialmente o nmero N 1234. O jogador que obtiver zero ganha. Quem pode garantir a vitria? 8. (Leningrado 1988) Uma pilha de 500 pedras dada. Dois jogadores jogam o seguinte jogo: Em cada turno, o jogador pode retirar 1,2,4,8,... (qualquer potncia de 2) pedras da pilha. O jogador que no puder mais jogar perde. Quem possui estratgia vencedora? 9. Em uma caixa existem 300 bolinhas. Cada jogador pode retirar no mais do que a metade das bolinhas que esto na caixa. O jogador que no puder mais jogar perde. Quem possui estratgia vencedora?

Referncias: [1] Dmitri Fomin, Alexey Kirichenkko, Leningrad Mathematical Olympiads 1987-1991, 1994 [2] Dmitri Fomin, Sergey Genkin e Ilia Itenberg, Mathematical Circles (russian experience), 1996

EUREKA! N37, 2013

33

Sociedade Brasileira de Matemtica

EQUAES FUNCIONAIS PARA OS MAIS JOVENS


Ricardo Csar da Silva Gomes, IFCE, Jaguaribe CE

Nvel Intermedirio
Um dos temas mais desafiadores para um olmpico so os problemas sobre equaes funcionais. Porm, existem poucos artigos escritos para iniciantes, o que contribui para dificultar mais ainda a implantao de um treinamento para a OBM numa escola que deseja se aventurar no encantador universo das Olimpadas de Matemtica. Nosso objetivo neste artigo suprir um pouco essa carncia. Inicialmente, vamos fazer um aquecimento com os exemplos abaixo.
1. Um bom comeo... Exemplo 01. (Austrlia) Para todo inteiro positivo n, temos:

f ( n) =

1
3

n 2 + 2n + 1 + 3 n 2 1 + 3 n 2 2n + 1

Determine o valor da soma

f (1) + f ( 3) + f ( 5 ) + ... + f ( 999997 ) + f ( 999999 ) .


Soluo:
3 3

Observando
2

com
2

bastante

cuidado

vemos

que

a b = ( a b ) ( a + ab + b

) (*) nos ajudar a sair dessa aparente dificuldade.

Basta fazermos a = 3 n + 1 e b = 3 n 1 e a funo dada pode ser reescrita como

f ( n) =

1 a b a b = 3 3= . 2 a + ab + b a b 2
2

Escrevendo f desta forma fica bastante claro porque usaremos (*), em seguida voltaremos varivel n. Veja que legal:

2 f ( n ) = a b = 3 n + 1 3 n 1.
Agora vamos tacada final,

2 f (1) = 3 2 2 f ( 3) = 3 4 3 2 2 f ( 5) = 3 6 3 4 2 f ( 999997 ) = 3 999998 3 999996


EUREKA! N37, 2013

34

Sociedade Brasileira de Matemtica

2 f ( 999999 ) = 3 1000000 3 999998


Adicionando, membro a membro, as equaes acima temos:

f (1) + f ( 3) + f ( 5 ) + ... + f ( 999997 ) + f ( 999999 ) =


Exemplo 02. Determine todas as funes f :

13 1000000 = 50. 2
(*)

tais que

f
2

(( x y ) ) = ( f ( x ))
2

2 xf ( y ) + y 2 .

Soluo. Fazendo x = y = 0 em (*), obtemos:

f ( 0 ) f ( 0 ) = 0 f ( 0 ) ( f ( 0 ) 1) = 0
H ento dois casos: 1 Caso: f ( 0 ) = 0. Escolhendo y = x em (*) teremos:

( f ( x ) x ) = ( f ( x ))
2 Caso:
2

2 xf ( x ) + x 2 = f ( 0 ) = 0

da temos f ( x ) = x, onde x um nmero real qualquer.

f ( 0 ) = 1. Novamente escolhendo y = x em (*), teremos:

( f ( x) x) = ( f ( x))
da temos que, para todo x ,

2 xf ( x ) + x 2 = f ( 0 ) = 1 f ( x ) = x + 1 ou f ( x ) = x 1. Assim,
Temos ento, de (*),

f ( x) = x + x ,

onde
2

x {1,1} , x .
2

( x y ) + ( x y)
2

= ( x + x ) 2 x ( y + y ) + y 2 , donde

( x y ) = 2 x ( x y ) + x2 , x, y . Como f ( 0 ) = 1, 0 = 1.
Se existe z com z 1, existe y
(1 y )2

com y 1 . Fazendo x = 1 na

igualdade acima, temos

2 = 2 (1 y ) + x , donde

2 2 2 ( 1 y ) = 1 y 2 x , absurdo, pois 2 ( 1 y ) = 4 e 1 y 2 x {0, 2} . ( )

Assim, devemos ter x = 1, x , donde f ( x ) = x + 1, x . fcil ver que f ( x ) = x ou f ( x ) = x + 1 so solues de (*). Nos prximos exemplos precisaremos de um pouco mais de criatividade!

EUREKA! N37, 2013

35

Sociedade Brasileira de Matemtica

Exemplo 03. (Balcnica) Encontre todas as funes

f:

tais que (*)

f ( xf ( x ) + f ( y ) ) = ( f ( x ) ) + y
para quaisquer nmeros reais x e y.
Soluo: Fazendo x = 0 na equao acima obtemos:

f ( f ( y )) = ( f (0)) + y

(1)

para todo nmero real y. De (1) podemos ver que f sobrejetiva, portanto existe um nmero real k tal que f ( k ) = 0. Fazendo x = k em (*), teremos f f ( y ) = y ,

para todo nmero real y. Por outro lado, vamos trocar x por f ( x ) em (*) e ver o que acontece:

f f ( x ) f ( f ( x )) + f ( y ) = x2 + y x
Note que o lado esquerdo da ltima equao idntico ao lado esquerdo de (*), portanto
2

( f ( x ))

+ y = x 2 + y f ( x ) = x 2 , x .

Em particular, f ( 0 ) = 0. Alm disso, se x 0, f ( x ) = x ou f ( x ) = x, isto , f ( x ) x {1,1} . Temos ento, para

x, y 0, f ( xf ( x ) + f ( y ) ) = f ( x ) + y = x 2 + y. Elevando ao
2 2 2

quadrado, obtemos

x 4 + 2 x 2 y + y 2 = ( x 2 + y ) = f ( xf ( x ) + f ( y ) ) = ( xf ( x ) + f ( y ) ) =
= x 2 f ( x ) + 2 xf ( x ) f
2

(y)+ f (y)
e logo

= x 4 + 2 xf ( x ) f

(y)+

y 2 , donde
ou seja,

2 xf ( x ) f ( y ) = 2 x 2 y ,
1

f ( x ) f ( y ) = xy ,

f ( x ) x = ( f ( y ) y ) . Assim, f ( x ) x = 1, x
donde f ( x ) = x, x ou f ( x ) = x, x .

ou f ( x ) x = 1, x ,

fcil ver que essas so as solues. necessrio notar que de (1) concluimos que f devia ser sobrejetiva. Sugerimos ao leitor no familiarizado com este fato que consulte [2], l ele encontrar a demonstrao deste e de outros fatos interessantes sobre vrios teoremas envolvendo composio de funes.
EUREKA! N37, 2013

36

Sociedade Brasileira de Matemtica

Exemplo 04. (RPM 77) Encontre todas as funes

f:

tais que (*)

f ( f ( x ) y ) + f ( y ) = f ( x + f ( y )) y
Para quaisquer nmeros reais x e y.
Soluo: Faamos

x = y = 0 na equao dada e facilmente concluiremos

que f ( 0 ) = 0. Agora faamos somente y = 0, obtendo assim:

f ( f ( x)) = f ( x)
Em seguida faremos x = 0 em (*), veja:

(1)

f ( y ) + f ( y ) = f ( f ( y ) ) y = f ( y ) y,
donde f ( y ) = y, y . Assim, f ( t ) = t , onde t um nmero real qualquer. fcil ver que esta a soluo da equao. Antes de irmos para a prxima etapa exercite as idias anteriores nos seguintes problemas:

4n + 4 n 2 1 2n + 1 + 2n 1 Calcule f (1) + f ( 2 ) + f ( 3) + ... + f ( 39 ) + ... + f ( 40 ) .


01. (Romnia) Para todo inteiro positivo n, seja

f ( n) =

2 para todo x real. Determine: 4 +2 1 2 2000 f + f + ... + f . 2001 2001 2001 x2 04. (Brasil) Seja f ( x ) = . Calcule 1 + x2 1 2 3 n f + f + f + ... + f 1 1 1 1 1 2 3 n + f + f + f + ... + f 2 2 2 2 + ...
03. (Coria) Seja

f ( x) =

1 + f + n

2 f + n

3 f + ... + n

n f . n

EUREKA! N37, 2013

37

Sociedade Brasileira de Matemtica

ax 05. (Crocia) Seja f uma funo definida por f ( x ) = x , com a a + a


Determine:

* +

3 2000 f + ... + . 2001 2001 1 06. (Holanda) Seja f uma funo definida por f ( x ) = . Determine o x + x2 1
valor de

1 S= f + 2001

2 f + 2001

f (1) + f ( 2 ) + f ( 3) + ... + f (1921) .


07. Considerando a funo

f : ]0, ]

tal que f ( a ) = 1, sendo a um nmero

real positivo. Prove que, se

a f ( x) f ( y) + f x

a f = 2 f ( xy ) y 1 , para todo natural n. n + n2 + 1


4

Para x e y ( 0, ) , ento f uma funo constante.


08. Seja f uma funo definida por

f ( n) =

Mostre que:

1 f (1) + f ( 2 ) + f ( 3) + f ( 4 ) + ... + f (100 ) < . 2


2. Caracterizando a funo afim

Apresentaremos a partir de agora uma ferramenta muito simples e bastante poderosa que ser til na resoluo de equaes funcionais cujas solues sejam funes afins. Suponha que em determinadas equaes funcionais cheguemos seguinte expresso: f ( x + h ) f ( x ) = . O Teorema abaixo, garante que se depender apenas de h, mas no de x, ou seja, = ( h ) , ento f uma funo afim.
Teorema: Seja

f:

uma funo montona injetiva. Se o acrscimo

f ( x + h ) f ( x ) = ( h ) depender apenas de h, mas no de x, ento f uma


funo afim.
EUREKA! N37, 2013

38

Sociedade Brasileira de Matemtica

Prova: A demonstrao deste Teorema ir recorrer ao Teorema fundamental da

proporcionalidade (fica como exerccio para o leitor procurar sua demonstrao em [1]). Suporemos que a funo f seja crescente, caso f seja decrescente o resultado segue por analogia. fcil ver que : tambm uma funo crescente e que ( 0 ) = 0. Alm disso, para quaisquer h, k temos:

(h + k ) = f ( x + h + k ) f ( x)
= f

(( x + k ) + h) f ( x + k ) + f ( x + k ) f ( x )

= ( h) + (k )
Logo, pelo Teorema fundamental da proporcionalidade, ( h ) = a h, onde

a = (1) para todo h . Da temos f ( x + h) f ( x) = (h) = a h


Fazendo x = 0 na equao acima obtemos:

f ( h) f ( 0) = a h ,
o que nos mostra que

f ( h ) = a h + b, onde b = f ( 0 ) para todo h . A

recproca tambm verdadeira e fica como exerccio para o leitor a sua demonstrao. Vejamos agora alguns exemplos:
Exemplo 01 (Adaptado de um problema da Estnia) Encontre todas as funes

montonas f : para x, y .

que satisfazem a equao

f ( x + f ( y ) ) = y + f ( x + 1) ,

Soluo: Fazendo x = 0 na equao dada temos

EUREKA! N37, 2013

39

Sociedade Brasileira de Matemtica

f ( f ( y ) ) = y + f (1) ,

(*)

o que nos mostra a bijetividade da funo f; portanto existe um nmero real k tal que f ( k ) = 0. Voltando equao funcional dada e fazendo y = k , temos

f ( x + 1) f ( x ) = k .
Fazendo z = x + 1 e h = f ( y ) 1, temos

f ( z + h) = f ( x + f ( y) ) = y + f ( x +1) = f ( z) + ( h) , onde ( h) = f ( h +1) f (1) .


Pelo teorema anterior f do tipo f ( x ) = kx + b, e fcil ver que b = k 2 (pois

f ( k ) = 0 ). Agora vamos fazer y = 0 em (*): f ( f ( 0 ) ) = f (1)


Sendo k 0 (seno f seria identicamente nula, o que no possvel) temos

f ( k 2 ) = f (1) k = 1.
Para concluirmos temos que as solues procuradas so satisfazem perfeitamente as condies do problema. Infelizmente nem sempre h no problema uma hiptese de monotonicidade, nem possvel provar de modo fcil que a funo procurada montona, o que muitas vezes torna a soluo do problema mais complicada tecnicamente.
Exemplo 02. (Equao de Cauchy) Encontre todas as funes

f ( x) = x +1 e

f:

que

satisfazem as seguintes condies:

(i ) f ( x + y ) = f ( x ) + f ( y ) ,
( ii ) f

para quaisquer que sejam os nmeros reais x e y.

1 f ( x) = 2 para x 0. x x
Como e logo

Soluo:

( a b ) + b = a, f ( a b ) + f ( b ) = f ( a ) , f ( a b ) = f ( a ) f ( b ) , a, b .
40

EUREKA! N37, 2013

Sociedade Brasileira de Matemtica

Temos

1 1 para t {0,1 1 = } . Portanto, 1 1 t 1 t 1 2 f ( t ) f 1 t 2 f (1) t f (1 t ) t f (1) = 2 = 2 2 (1t ) 1 (1t ) 1 t


2 2 2

f (1) f ( t ) (1 t ) f (1) = f (1 t ) (1 t ) f (1) = f ( t ) t 2 f (1) 2 f ( t ) = t 2 f (1) + f (1) (1 t ) f (1) = 2 f (1) t.


(*) Assim, f ( t ) = f (1) t , t {0,1} ; naturalmente, essa igualdade tambm vale para

t = 1. Como f ( 0 ) = f ( 0 0 ) = f ( 0 ) f ( 0 ) = 0, a igualdade tambm vale para t = 0. Assim, f ( t ) = ct , t , onde c = f (1) . fcil ver que todas essas
funes so solues.
Exemplo 03. (Austrlia) A funo f satisfaz as seguintes condies:

(i) para todo nmero racional x, f ( x ) assume um valor real; (ii) f (1988 ) f (1987 ) ; (iii) f ( x + y ) = f ( x ) f ( y ) f ( xy ) + 1, onde x e y so nmeros racionais quaisquer. Mostre que f
Soluo: Fazendo

1 1987 . = 1988 1988

x = y = 0 em (iii), teremos

( f ( 0 ) 1)

= 0 f ( 0 ) = 1.

Em seguida faremos a seguinte escolha: x = 1 e y = 1 e voltaremos condio (iii),

f ( 0 ) = f (1) f ( 1) f ( 1) + 1 f ( 1) ( f (1) 1) = 0
Da temos duas possibilidades: f ( 1) = 0 ou f (1) = 1.
1 Caso:

f (1) = 1. Fazendo y = 1, em (iii):


41

EUREKA! N37, 2013

Sociedade Brasileira de Matemtica

f ( x + 1) = f ( x ) f (1) f ( x ) + 1 = 1
para todo x . Porm esta equao contradiz (ii), portanto f ( 1) = 0 .
2 Caso:

f ( 1) = 0 . Fazendo x = y = 1, em (iii), obtemos: f ( 2 ) = f ( 1) f ( 1) f (1) + 1 f ( 2 ) = 1 f (1)


(*)

Fazendo x = 2, y = 1, , em (iii) temos:

f ( 1) = f ( 2 ) f (1) f ( 2 ) + 1
+ f (1)

f (1) f ( 2 ) + 1 =0
Agora temos mais duas possibilidades. Caso A: f (1) = 0. Voltando a (iii) faamos y = 1:

f ( x + 1) = f ( x ) f (1) f ( x ) + 1 f ( x + 1) = 1 f ( x )
trocando x por x + 1, obtemos

f ( x + 2) = f ( x )
Assim, f ( 2 ) = f ( 0 ) = 1, e tomando x = contradio, veja:

(**)

1 e y = 2 em (iii) encontraremos uma 2

1 5 1 1 f = f = f f ( 2 ) f (1) + 1 = f + 1, 2 2 2 2
absurdo.
Caso B:

f ( 2 ) = 1. Usando este fato em (*) temos: f (1) = 2

Por fim fazendo y = 1 em (iii) chegaremos seguinte concluso:

f ( x + 1) = f ( x ) + 1 (oba!).
Da segue que f ( x + k ) = f ( x ) + k para todo k inteiro; como f ( 0 ) = 1, temos

f ( k ) = k + 1 para todo k inteiro.


EUREKA! N37, 2013

42

Sociedade Brasileira de Matemtica

Se x = p q , com p, q inteiros, q 0, fazendo y = q em (iii), obtemos

f ( x) + q = f ( x + q) = ( q +1) f ( x) f ( qx) +1 = ( q +1) f ( x) f ( p) +1 = ( q +1) f ( x) p,


donde qf ( x ) = p + q, e logo f ( x ) = Em particular, f

p+q p = + 1 = x + 1, para todo x . q q

1987 1 1987 +1 = . = 1988 1988 1988 f:


*

Exemplo 04. (Seletiva Blgara para a IMO) Encontre todas as funes

, tais

que

f ( x2 + y ) = ( f ( x )) +
para quaisquer x, y
*

f ( xy ) f ( x)
*

, com y x 2 .

Fazendo y = 1, obtemos f x 2 + 1 = f ( x ) + 1, x
2

.
*

Fazendo x = 1, obtemos f ( y + 1) = f (1) + f ( y ) f (1) , y


2

\ {1} .
2

Assim, f ( 2 ) = f (1) + 1 e f ( 5 ) = f 22 + 1 = f ( 2 ) + 1 = f (1) + 1 + 1. Por outro lado, f ( 3) = f ( 2 + 1) = f (1) + f ( 2 )


2 2 2

f (1) + 1 f (1) = f (1) + , f (1)


2

f ( 4 ) = f ( 3 + 1) = f (1) + f ( 3) f (1) = f (1) + f (1) +


2 2

f (1) + 1 f (1)
2

f ( 5 ) = f ( 4 + 1) = f (1) + f ( 4 ) f (1) = f (1) + f (1) + 1 +


4 2

f (1) + 1 f (1)
3

, e logo
2

f (1) + 2 f (1) + 2 = f (1) + 1 + 1 = f ( 5 ) = f (1) + f (1) + 1 +


donde f (1) + f (1) +1 = portanto f (1) = 1.
EUREKA! N37, 2013

f (1) + 1 f (1)
3

f (1) + f (1) +1 f (1)


3

, e, como f (1) + f (1) +1 > 0, f (1) = 1, e

43

Sociedade Brasileira de Matemtica

Assim, temos

f ( y + 1) = f (1) + f ( y ) f (1) = f ( y ) + 1, y

* 2

y 1.

Assim, f x 2 + 1 = f x 2 + 1, x

( )
*

, donde f ( x 2 ) + 1 = f ( x ) + 1, e logo
2

f ( x 2 ) = f ( x ) , x
*

. Em particular, f ( x ) > 0, x > 0.

Fazendo y = 1, obtemos f x2 1 = f ( x) + f ( x) f ( x) = f x2 + f ( x) f ( x) ,

\ {1,1} . Como
*

( ) f ( x ) = f ( x 1 + 1) = f ( x 1) + 1,
2 2 2

segue que
* 2

f ( x ) f ( x ) = 1, x

\ {1,1} , i.e.,

f ( x ) = f ( x ) , x

\ {1,1} .

Como f ( 1) = f ( 2 + 1) = f ( 2 ) + 1, e f ( 2) = f ( 2) = f (1) + 1 = 2, temos tambm f ( 1) = 1 = f (1) . Assim, f ( x ) = f ( x ) , x mostrar que f ( a ) = a, a > 0, donde f ( x ) = x, x
* *

. Vamos

Se x > 0, y < x2 , f x2 + y = f x2 + f ( xy) f ( x) e f x2 y = f x2 + f ( xy) f ( x) = f x 2 f ( xy ) f ( x ) , donde

) ( )

( )

) ( ) f ( x + y) + f ( x y) = 2 f ( x ),
2 2 2

ou seja, se uma

u, u h, u + h > 0, f ( u + h ) + f ( u h ) = 2 f ( u ) . Assim, se
progresso aritmtica de termos positivos,

( an )n0
*

( f ( a ))
n

n 0

tambm uma progresso


*

aritmtica de termos positivos. Em particular, q progresso aritmtica de termos positivos. Como

, ( f ( p q ) ) p
*

uma

f ( q q ) = f (1) = 1 e .

f ( 2q q ) = f ( 2 ) = 2, segue que f ( p q ) = p q , p, q
caso

Dado a > 0, suponha por absurdo que f ( a ) a. Vamos supor que f ( a ) > a (o

f ( a ) < a anlogo). Sejam p, q

com a < p q < f ( a ) . Seja

x = a > 0, e seja y = p q a > 0.


Temos p q = f ( p q ) = f x 2 + y = f x 2 + f ( xy ) f ( x ) > f x 2 = f ( a ) , absurdo. Assim, f ( a ) = a, a > 0, c.q.d..
Exerccios 01. Determine todas as funes

( )

( )

f:

que satisfazem as condies abaixo:


+

(i) f ( x + 1) = f ( x ) + 1, para quaisquer que sejam os nmeros x e y


EUREKA! N37, 2013

44

Sociedade Brasileira de Matemtica

(ii) f x 3 = f ( x ) , para x

( ) (

. f:
*

02. (Austrlia) Prove que existe apenas uma funo

que satisfaz as

seguintes condies: (i) f ( x ) = xf para todo nmero real x no-nulo;

1 x (i) f ( x ) + f ( y ) = 1 + f ( x + y ) , para todos os pares ( x, y ) de reais no-nulos tais que x y.


03. (Austrlia) So dados m e n inteiros maiores do que 1 com m par e uma funo f

que assume apenas valores reais, e que est definida nos reais no negativos, satisfazendo as condies abaixo: (i) para quaisquer x1 , x2 , x3 , x4 ,..., xn ,
m m m x m + x2 + x3 + ... + xn f ( x1 ) + f ( x2 ) + f ( x3 ) + ... + f ( xn ) ; f 1 = n n (ii) f (1986 ) 1986; m m m m

(iii) f (1988 ) 0. Prove que

f (1987 ) = 1.

Bibliografia: [1] Lima, Elon Lages et al. A Matemtica do Ensino Mdio. Rio de Janeiro. SBM, Coleo Professor de Matemtica-2000. [2] Marcelo Rufino e Mrcio Rodrigo. Coleo Elementos da Matemtica, vol. 01. Editora VestSeller-2010. [3] www.obm.org.br

EUREKA! N37, 2013

45

Sociedade Brasileira de Matemtica

COMO QUE FAZ?


Descubra um quadrado n n, com n > 1 natural, dividido em n 2 quadrados unitrios, preenchidos com quadrados perfeitos distintos e no nulos, de modo que a soma dos valores de cada coluna e de cada linha tambm sejam quadrados perfeitos. (Proposto por Marta de Ftima Severiano).
SOLUO

Por exemplo o quadrado correspondente matriz abaixo:

42 2 32 47 2

232 442 282

522 16 529 2704 2 17 = 1024 1936 289 para a qual as somas dos elementos 162 2209 784 256

de todas as linhas e de todas as colunas so iguais a 3249 = 572. H infinitas matrizes com essas propriedades. Por exemplo, para qualquer inteiro k, a matriz

22 2 2 (11 + 10k + 3k ) 2 (10 + 10k + 4k 2 )


2

(14 + 10k )

2 2 2

( 5 + 14k + 5k )
(10 + 8k ) (10 + 6k )
2 2

2 2

( 2 + 10k + 4k ) ( 5 + 10k + 3k )

2 2

tal que as somas dos elementos de todas as linhas e de todas as colunas so iguais a 15 + 14k + 5k 2

EUREKA! N37, 2013

46

Sociedade Brasileira de Matemtica

SOLUES DE PROBLEMAS PROPOSTOS


Publicamos aqui algumas das respostas enviadas por nossos leitores.

143)

Determime
3

todas
3

as

funes

f , g,h :

tais

que

f ( xy ) = g ( x + y ) + h ( x + y ) , x, y .
SOLUO DE ZOROASTRO AZAMBUJA NETO (RIO DE JANEIRO RJ)

Chamemos de (*) a equao funcional f ( xy ) = g ( x3 + y ) + h ( x + y 3 ) . Fazendo x = 0 em (*), obtemos f ( 0 ) = g ( y ) + h ( y 3 ) , y , e, fazendo y = 0,

temos f ( 0 ) = g ( x 3 ) + h ( x ) , x . Assim, h ( x ) = f ( 0 ) g ( x3 ) , e (*) se escreve como f ( xy ) = f ( 0 ) + g ( x3 + y ) g ( x + y 3 ) , x, y . Fazendo x = 0, obtemos


f ( 0 ) = f ( 0 ) g ( y ) + g ( y 9 ) , donde g ( y 9 ) = g ( y ) , y . Segue facilmente por

induo que g y 9

( ) = g ( y ) , y
k

,k .

Fazendo x = y 3 em (*), temos f ( y 4 ) = g ( y y 9 ) + h ( 0 ) , e trocando y por y, temos f ( y4 ) = g ( y + y9 ) + h ( 0) , donde g ( y y9 ) = g ( y + y9 ) = g ( y y9 ) , y .

Como a funo t : g ( z ) = g ( z ) , z .

dada por t ( y ) = y y

sobrejetiva, segue que

Dado a 0, 3 3 , vamos mostrar que existem x, y


3

com xy = a e

x3 + y = ( x + y 3 ) . Assim, teremos f ( a) = f ( xy) = f ( 0) + g ( x3 + y) g ( x + y3 ) = f ( 0) .

Como

(x + y )

3 3

= x 3 + 3x 2 y 3 + 3xy 6 + y 9 , a igualdade x3 + y = ( x + y 3 ) pode ser


y 9 + 3xy 6 + 3 x 2 y 3 y = 0, e, de
xy = a,

escrita como

temos

xy 6 = ay 5 e

x 2 y 3 = a 2 y, logo a igualdade se torna y 9 + 3ay 5 + ( 3a 2 1) y = 0. Dividindo por y temos y8 + 3ay 4 + ( 3a2 1) = 0. Para y = 0 temos y 8 + 3ay 4 + ( 3a 2 1) = 3a 2 1 < 0, e para y = 1 temos y 8 + 3ay 4 + ( 3a 2 1) = 3a + 3a 2 > 0. Assim, existe y com
0 < y <1

tal

que

y 8 + 3ay 4 + ( 3a 2 1) = 0 ,

e,

tomando

x = a y,

temos

EUREKA! N37, 2013

47

Sociedade Brasileira de Matemtica

y 9 + 3xy 6 + 3x 2 y 3 y = y ( y 8 + 3xy 5 + 3 x 2 y 2 1) = y y 8 + 3ay 4 + ( 3a 2 1) = 0. Isto prova que f ( a ) = f ( 0 ) para todo a com 0 a < 3 3.
Assim, se c > 0, x, y 0, x + y 3 = c e xy < 3 3, temos, por (*), g ( c y 3 ) + y = g ( x3 + y ) = f ( xy ) h ( x + y 3 ) = f ( 0 ) h ( c ) .
3 = c = 24 25 e 0 y 3 10, temos xy c 10 24 25 3 10 < 3 3 3 3

Escolhendo

, sempre que x, y 0 e
3

x + y 3 = c.

Para
3

y = 0,

(c y )

+ y = c3 = ( 24 25 ) < 9 10,
3

e,

para

y = 3 10, ( c y 3 ) + y = 24 25 ( 3 10 )
9 10 z 11 10,

+ 3 10 > 11 10. Assim, dado z com com

existe
3

y [ 0,3 10]

(c y )

3 3

+ y = z, Ou

e seja,

logo se

g ( z ) = g ( c y 3 ) + y = f ( 0 ) h ( c ) = f ( 0 ) h ( 24 25 ) . d = f ( 0 ) h ( 24 25 ) , temos g ( z ) = d , z [9 10,11 10]. Dado


w > 0,

existe

k
k

com

z [9 10,11 10] com z 9 = w. Assim,

( 9 10 ) < w < (11 10 ) , e logo existe g ( w ) = g ( z 9 ) = g ( z ) = d . Tambm temos


k

9k

9k

g ( w ) = g ( w ) = d . Assim g ( w ) = d , w
~

. Analogamente, existe d

tal

que h ( w ) = d w * (os papis de g e h em (*) so anlogos). Assim,


f ( 16 ) = f ( 23 2 ) = g ( 2 29 ) + h ( 0 ) = d + h ( 0 ) , e, por outro lado,
~

f ( 16 ) = f ( 4 4 ) = g ( 4 43 ) + h ( 43 4 ) = d + d , donde
~
~

d + h ( 0) = d + d ,

logo h ( 0 ) = d . Analogamente, g ( 0 ) = d . Assim, g e h so constantes, donde


f ( x ) = f ( x 1) = g ( x3 + 1) + h ( x + 1) = d + d , x , ou seja, f constante. Como

dadas quaisquer constantes d , d , se


~

g ( x ) = d , x , h ( x ) = d , x

f ( x ) = d + d , x , a condio (*) do enunciado satisfeita, essas so todas as solues da equao funcional (*).

EUREKA! N37, 2013

48

Sociedade Brasileira de Matemtica

144) Seja x 1 um nmero racional tal que existe uma constante c 0 e uma sequncia ( an ) n1 de inteiros tal que lim ( cx n an ) = 0. Prove que x inteiro.
n

SOLUO DE ASDRBAL PAFNCIO SANTOS (BOTUCATU SP)

Seja x =
n

p , com p, q q

, p q e mdc ( p, q ) = 1.
n n

Se lim ( cx n an ) = 0, temos lim ( c p xn p an ) = 0, e lim ( c q x n +1 q an +1 ) = 0, e, como qx = p, c q x n +1 = c p x n , sempre inteiro, Assim, donde segue
lim ( p an q an +1 ) = 0.
n

Como que donde

p an q an +1

que

existe

n0

tal

p an q an +1 = 0, n n0 .

p an +1 = an , n n0 , q

p an = an0 q

n n0

, n n0 . Como mdc ( p, q ) = 1, q n n0 an0 , n n0 , e logo q = 1 ou


n n0

p an0 = 0. Caso an0 = 0, teramos an = an0 q n n pois lim ( cx an ) = 0 e cx c > 0.


n

= 0, n n0 , o que um absurdo,

Assim, q = 1 e x = p

inteiro. q Obs: importante a hiptese de x ser racional. fato, an = 1 + 2

De

) + (1 2 )
n

satisfaz

a1 = 2, a2 = 6

an + 2 = 2an +1 + an , n 1, e logo inteiro, para todo n 1.

Por outro lado, como 1 2 < 1, lim 1 + 2

((

an = 0.

149) a) Deseja-se organizar um torneio de futebol com n times ( n 2 ) em que cada time joga uma vez contra cada um dos outros, dividido em um certo nmero de rodadas. Em cada rodada cada time joga no mximo uma partida. Prove que, se n mpar, possvel organizar um tal torneio com n rodadas e, se n par, possvel organizar um tal torneio com n 1 rodadas.
b) Uma matriz n n preenchida com elementos do conjunto S = {1, 2,3,..., 2n 1}. Sabe-se que, para todo i {1, 2,..., n} , a i-sima linha e a i-sima coluna contm juntas todos os elementos de S.
EUREKA! N37, 2013

49

Sociedade Brasileira de Matemtica

Quais os possveis valores de n?


SOLUO DE JOS DE ALMEIDA PANTERA (RIO DE JANEIRO RJ)

a) Se n mpar, podemos organizar o torneio da seguinte forma: para 1 k n, na rodada k so realizadas todas as partidas entre jogadores i e j (os jogadores so numerados de 1 a n) tais que i + j k ( mod n ) . Nessa rodada exatamente um dos jogadores no joga (fica bye): o jogador ak , com 2ak k ( mod n ) . Para cada par de jogadores

i + j k ( mod

{i, j} , existe exatamente um k, n ) : i joga contra j na k-sima rodada.

com 1 k n,

tal que

Se n par, n 1 mpar, e construmos um torneio de n 1 rodadas como acima, em que os jogadores 1, 2,..., n 1 jogam entre si. Para cada k com 1 k n 1, o jogador ak , com 1 ak n 1 tal que 2ak k ( mod n 1) , no joga contra nenhum dos jogadores 1, 2,..., n 1. Completamos o torneio fazendo com que na rodada k, para 1 k n 1, o jogador ak jogue contra o jogador n. Como

{ak ,1 k n 1} = {k ;1 k n 1} ,

o torneio satisfaz as condies do enunciado.

b) A resposta : os inteiros positivos pares. De fato, se n par, existe um torneio de n 1 rodadas com n jogadores em que todos jogam contra todos. Construmos uma matriz ( aij ) da seguinte forma:
1 i , j n

Se i > j , tomamos aij {1, 2,..., n 1} tal que os jogadores i e j se enfrentam na rodada aij , e definimos a ji = aij + n. Finalmente, definimos aii = n para 1 i n. fcil ver que esta matriz satisfaz as condies do enunciado. Se n 3 mpar, suponha por absurdo que existe uma tal matriz. Como 2n 1 > n, existe k 2n 1 tal que a jj k , j n. Temos ento
n = 1 i n a ji = k + 1 i n aij = k
j =1 n

({

} {

})=

= 2 {( i, j ) ;1 i, j n, aij = k} ,

absurdo, pois n mpar.

EUREKA! N37, 2013

50

Sociedade Brasileira de Matemtica


12 1 151) Encontre todas as solues reais positivas de x x = . 2

SOLUO DE BRUNO PEDRA (RIO DE JANEIRO RJ)

Seja f : Temos

dada por f ( x ) = x x .

12

ln[ f ( x)] = x 2 ln x.
Derivando, obtemos
1 f '( x) 1 21 1 = x ln x + x 2 f ( x) 2 x 1 1 f '( x) = f ( x) x 2 (ln x + 2) 2 0 < x < e 2 f '( x) < 0 f decrescente em (0, e2 ) x > e2 f '( x) > 0 f crescente em (e2 , )
2

x = e 2 ponto de mnimo, f (e2 ) = e e . 1 1 2 < e < 4 4 < e2 < 16 < e 2 < . 16 4 1 1 1 f = f = . 4 16 2 1 1 (e2 , ) e (0, e2 ). 4 16
Em (0, e 2 ) temos apenas x = decrescente.
1 como raiz, uma vez que f estritamente 16

Em (e 2 , ) temos apenas x = 1 como raiz, uma vez que f estritamente 4 crescente. 1 1 Assim, o conjunto das solues reais positivas da equao S = , . 4 16

152) Sejam a, b, c nmeros reais positivos tais que a + b + c = 1. a bc b ca c ab 3 Prove que + + . a + bc b + ca c + ab 2


EUREKA! N37, 2013

51

Sociedade Brasileira de Matemtica

SOLUO DE CARLOS ALBERTO DA SILVA VICTOR (NILPOLIS RJ)

a bc b ca c ab 3 equivalente + + a + bc b + ca c + ab 2 2bc 2ac 2ab 3 1 +1 +1 que equivalente a a + bc b + ac c + ab 2 ab bc ac 3 I) + + , e j que c + ab a + bc b + ac 4 c + ab = (1 a )(1 b ) a + b + c = 1, teremos: a + bc = (1 b )(1 c ) b + ac = (1 a )(1 c ) Da (I) ficar escrita da seguinte forma ab bc ac 3 + + que equivalente a: (1 a )(1 b ) (1 b )(1 c ) (1 a )(1 c ) 4 Provar que

4 ( ab (1 c ) + bc (1 a ) + ac (1 b ) ) 3 (1 a )(1 b )(1 c ) ou 4 ( ab + bc + ac 3abc ) 3 ( ab + ac + bc abc ) ,


ab + ac + bc 9abc, que equivalente a provar que

pois

a + b + c =1

ou

1 1 1 + + 9. a b c Das desigualdades das mdias aritmtica e harmnica temos: 1 a+b+c 3 1 1 1 = , ou seja + + 9. 1 1 1 3 3 a b c + + a b c

153) Quatro pontos P, Q, R, S pertencem a um crculo de tal forma que o ngulo


PSR reto. Sejam H e K as projees de Q nos segmentos PR e PS, respectivamente. Prove que a reta HK divide o segmento QS ao meio.
SOLUO DE MARCELO RIBEIRO DE SOUZA (RIO DE JANEIRO RJ)

Considere a semireta de origem em H passando por Q e tomemos um ponto T qualquer, situado acima de Q. Tracemos tambm o segmento PQ. Chamemos M o ponto de interseo entre o segmento QS e HK . Agora vamos marcar os ngulos e nome-los: KQS = , KQT = , conforme o desenho a seguir:
EUREKA! N37, 2013

52

Sociedade Brasileira de Matemtica

Note que a reta que contm o segmento KQ paralela reta que contm o segmento SR, assim sendo, so alternos internos os ngulos KQS = QSR = . Alm disso, QPR olha para o arco QR, portanto, temos QPR = . Como o quadriltero KQHP circunscrvel, TQK = KPH = , portanto, podemos escrever KPQ = . Ainda pelo fato de KQHP ser circunscrvel, devemos ter KPQ = KHQ = . Finalmente, sabemos que QHK + QKH = , ou seja, retngulo, devemos ter QKM = . Como o tringulo QKS QM = KM = MS , assim, temos M ponto mdio de QS .

154)

Determine

todas

as

funes

f:

[ 0, + )

tais

que

f ( xy ) = f ( x ) f ( y ) e f ( x + y ) f ( x ) + f ( y ) , x, y .
SOLUO DE BRUNO SALGUEIRO FANEGO (VIVEIRO ESPANHA)

Seja f : [ 0, + ) uma funo verificando as duas propriedades do enunciado, que denominaremos multiplicatividade e desigualdade triangular, respectivamente.
EUREKA! N37, 2013

53

Sociedade Brasileira de Matemtica

Tomando

x = y =1

na

propriedade

de

multiplicatividade,

resulta

f (1) = f (1) f (1) . Assim, f (1) ( f (1) 1) = 0 f (1) = 0 ou f (1) 1 = 0.


n vezes n vezes Se f (1) = 0, ento 0 f ( n ) = f 1 + ... + 1 = f (1) + ... + f (1) = nf (1) = n 0 = 0

a 1 1 f = f a = f ( a ) f = 0, a , b , b 0, quer b b b dizer, f a funo nula, que multiplicativa e verifica a desigualdade triangular, logo uma soluo vlida.
f ( n ) = 0 n

Supomos ento, no que segue, que f (1) = 1. Ento


1 = f (1) = f ( 1 ( 1) ) = f ( 1) f ( 1) = f ( 1) f ( 1) {1,1} f ( 1) = 1.
2

Da segue que f ( x ) = f ( 1) f ( x ) = f ( x ) , x .
1 Alm disso, 1 = f (1) = f b = f ( b ) f b 1 f ( b ) 0, b 1 1 f = , b 0 b f (b )

a f = f (a) f b
n

1 f (a) , a, b , b 0 e = b f (b )
*

n vezes n vezes n f ( x ) = f x ... x = f ( x ) ... f ( x ) = f ( x ) x , n

Tambm temos f ( 0 ) = f ( 0 ) f ( 0 ) , donde f ( 0 ) = 0 ou f ( 0 ) = 1. Se f ( 0 ) = 1, 1 = f ( 0 ) = f ( 0; x ) = f ( 0 ) f ( x ) = 1 f ( x ) = f ( x ) , x . nmero primo p e um 0 < < 1 e escrevamos x


*

Fixemos agora um

a como x = p n , sendo n b n a e a, b coprimos com p e definamos f ( x ) = f p = n . Ento f satisfaz a b a e propriedade multiplicativa e a desigualdade triangular, pois se x = p m b c ac m+n y = p n , ento f ( xy ) = f p m + n = m n = f ( x ) f ( y ) e = d bd
EUREKA! N37, 2013

54

Sociedade Brasileira de Matemtica

* * f ( x + y ) = f pmin{m,n} = f pmin{m,n} f min{m,n} = mx{ f ( x) , f ( y )} f ( x) + f ( y ) . bd bd Portanto, um nmero racional pequeno em relao a f se e s se divisvel por uma potncia grande de p. Dizemos agora que duas funes f , g : [ 0, + ) satisfazendo a

multiplicatividade e a desigualdade triangular so equivalentes se f = g c para 1 alguma contante c > 0. A funo para a qual = ser denotada por f p . p Provaremos a seguir que qualquer funo no nula no constante igual a 1 f : [ 0, + ) verificando a multiplicatividade e a desigualdade triangular e distinta da funo [ 0, + ) dada por

0, x = 0, que denominaremos funo trivial, equivalente funo valor x 1, x 0


absoluto ou a uma funo f p para algum p, ficando assim resolvido o problema. Sejam a, b, n , a 2, b 2 e escrevemos b n na base a : b n = c0 + ci a i , com
i =1 m

0 ci a 1. Seja M = max { f ( a 1) ,..., f (1)}; ento

f ( b ) f ( b n ) f ( c0 ) + f ( ci ) f ( a ) ( m + 1) M max 1, f ( a ) , f ( n ) ,..., f ( a )
n i 2 i =1

( n log a b + 1) M max 1, f ( a ) f ( b ) max 1, f ( a ) 1) Se

}.
para
log a b

Tomando razes de ndice n e fazendo tender n a infinito, resulta

log a b

}.
algum
b , b 2,

f (b ) > 1

obtemos

f ( b ) max 1, f ( a )

log a b

} = f (a)

, donde f ( a ) > 1, a , a 2.
logb a

Intercambiando a e b em (*), resulta f ( a ) f ( b )

, logo f ( b ) log b = f ( a ) log a

ou equivalentemente, f ( a ) = a para todos os a 1 e algum independente de a, sendo ento f equivalente funo valor absoluto. Para que a desigualdade triangular seja verificada, necessrio e suficiente que 0 < 1. Temos ento
f ( x ) = x , x .

EUREKA! N37, 2013

55

Sociedade Brasileira de Matemtica

2) Caso contrrio, quer dizer, se f ( b ) 1 para todos os b , b 1, existe um


b 2 tal que f ( b ) < 1, pois em caso contrrio f a funo trivial ou a funo
nk constante igual a 1. Tomemos um desses bs e escrevamos b = p1n1 ... pk . Ento

1 > f ( b ) = f ( p1 ) 1 ... f ( pk ) k , donde existe p primo igual a um dos p j tal que


f ( p ) < 1. Basta provar que f ( q ) = 1 para todos os primos q p, de onde se

deduzir que f equivalente a f p . Suponhamos que f ( p ) < 1 e f ( q ) < 1 para primos p q. Tomemos m, n 1 tais que f ( p m ) <
qn

so

co-primos,
m

ento
n

1 f ( x) f ( p) + f ( y) f (q)

xp m + yq n = 1 1 1 < + , o que uma contradio. 2 2

1 1 e f ( q n ) < . Como p m e 2 2 para alguns x, y , donde

155) Sejam a, b e c inteiros positivos, tais que existe um tringulo T de lados


a , b e c . Prove que so equivalentes: i) Existe um tringulo congruente a T cujos vrtices tm coordenadas inteiras em 2 . ii) T tem rea racional e existem x, y inteiros com a = x 2 + y 2 .

iii) T tem rea racional e existem u, v inteiros com mdc ( a, b, c ) = u 2 + v 2 .


SOLUO DE JOS DE ALMEIDA PANTERA (RIO DE JANEIRO RJ)

Vamos usar mais de uma vez a existncia e unicidade de fatorao para elementos de [i ] (Vejam o artigo de Guilherme Fujiwara na Eureka! N 14).
Lema 1: Seja D = { x 2 + y 2 , x, y
b D ab D. Prova: Se a D \ {0} ,

}.

Dado a D \ {0} , temos, para todo inteiro para certos inteiros


u, v x, y. Se

temos para
2

a = x2 + y 2

b D,b = u2 + v2
ab = ( x + y
2 2

certos
2

inteiros

portanto

)( u

+ v ) = ( xu yv ) + ( xv + yu ) D.
2

Vamos provar que ab D b D por induo em a. Se a = 1 a implicao obvia. Suponha agora que > 1, D e a implicao vale para todo a < . Temos = x 2 + y 2 = ( x + iy )( x iy ) . Seja um fator irredutvel de x + iy. Suponha que b = u 2 + v 2 = ( u + iv )( u iv ) .
EUREKA! N37, 2013

56

Sociedade Brasileira de Matemtica

Ento ( u + iv )( u iv ) , donde u + iv ou u iv. Vamos supor sem perda de generalidade que u + iv , ou seja, u + iv = , com

[i ]. Assim, temos b = ( u + iv ) ( u + iv ) = = . Como x + iy , xiy e logo ( x + iy )( x iy ) = . Se = r + si, = ( r + si )( r si ) = r 2 + s2 > 1.

Assim,
b = < , e cb = = D (de 2 r +s = m + ni, = m 2 + n 2 D). Assim, por hiptese de induo, b D. c=
2

fato,

se

Lema 2: Seja T um tringulo cujos vrtices tm coordenadas inteiras em

lados medam inteiras em


2

a, b e c

e cujos (com a, b, c inteiros positivos). Ento, se


a d, b d e
2 2

d = mdc ( a, b, c ) , existe um tringulo T cujos vrtices tambm tm coordenadas

e cujos lados medem

c d . Em particular, os

inteiros d , a d , b d e c d pertencem a D = { x + y , x, y

}.

Prova: Vamos provar a afirmao por induo em d = mdc ( a, b, c ) .

Para d = 1 no h o que provar. Se d > 1, podemos supor sem perda de generalidade que um dos vrtices de T ( 0,0 ) (seno fazemos uma translao). Sejam em perda
2

( u, v )

( r, s )

os outros vrtices de T. Temos ento (digamos)


2 2

u 2 + v 2 = a , r 2 + s 2 = b e ( u r ) + ( v s ) = c. Seja um fator irredutvel de d

[i ].

Temos ento que a = u 2 + v 2 = ( u + iv )( u iv ) . Podemos supor sem de


2

generalidade
2

que
2

u + iv .

Temos

tambm

que

b = r + s = ( r + is )( r is ) , donde r + is ou r is.
Alm disso,

c = ( u r ) + ( v s ) = ( ( u + iv ) ( r + is ) ) ( ( u iv ) ( r is ) ) ,

donde ( u + iv ) ( r + is ) ou ( u iv ) ( r is ) . Temos dois casos: i)

r + is

ii) / | r + is. Nesse caso, r is, donde r + is, e no podemos ter

( u + iv ) ( r + is ) , pois, como u + iv, teramos r + is , absurdo.

EUREKA! N37, 2013

57

Sociedade Brasileira de Matemtica

Portanto,

( u iv ) ( r is ) ,

logo

( u + iv ) ( r + is ) ,

donde,

como

r + is , u + iv .
Assim, no caso i), u + iv e r + is , e, no caso ii), u + iv e r + is . Dividindo os vrtices de T (identificados com os nmeros complexos 0, u + iv e r + is, respectivamente) por , no caso i), ou por , no caso ii) obtemos um tringulo T (cujos vrtices tm coordenadas inteiras) semelhante a T. Se = m + in, e k = m 2 + n 2 > 1, T tem lados a k , b k e c k . Temos
mdc ( a k , b k , c k ) = d k < d , e a afirmao segue por induo.

Se os vrtices de T so

( 0,0 ) ,

(u, v )

( r, s ) ,

temos u + v = a d . Assim,

a = u 2 + v 2 e a d pertencem a D, e, pelo lema 1, d = a ( a d ) tambm pertence a D. Analogamente, b d e c d tambm pertencem a D. Isto termina a prova do lema 2. Vamos agora provar a equivalncia dos itens do enunciado:

1 us vr , e 2 2 2 seus lados so a , b e c , onde a = u 2 + v 2 , b = r 2 + s 2 e c = ( u r ) + ( v s ) so somas de dois quadrados. Para as prximas implicaes, notemos que, se um tringulo tem lados a , b e c , com a, b e c inteiros positivos, e rea racional, ento existe um
i ) ii ) : Se os vrtices de T so ( 0,0 ) , ( u , v ) e ( r , s ) , sua rea

tringulo semelhante a ele, com lados 2a, 2 ab e 2 ac , cujos vrtices tm coordenadas inteiras: de fato, podemos colocar seus vrtices em
C = ( 0,0 ) , B = ( 2a,0 )

A = ( m, h ) ,

com
m=

m 2 + h 2 = 2 ab

= 4ab

4ab 4ac + 4a 2 e =bc+a 4a h 2 = 4ab m 2 so inteiros, mas, como a rea do tringulo, que racional, igual a 2ah = ah, h racional, e logo inteiro. Assim, segue do lema 2 que existe um 2 tringulo T cujos vrtices tm coordenadas inteiras e cujos lados so

( m 2a )

+ h 2 = 2 ac

= 4ac,

donde

4a 2 d, 4ab d

4ac d , onde

d = mdc ( 4a 2 , 4ab, 4ac ) = 4ad ,

onde

d = mdc ( a, b, c ) . Assim, os lados de T so a d , b d e c d pertencem a D.


EUREKA! N37, 2013

a d , b d e c d . Em particular,

58

Sociedade Brasileira de Matemtica

Agora, para ver que

( ii ) ( iii ) basta usar o lema 1, pois, se a D, como a d D, segue que d = mdc ( a, b, c ) D. E, finalmente, para ver que ( iii ) ( i ) , vemos da discusso anterior que existe um

tringulo T cujos vrtices tm coordenadas inteiras e cujos lados medem a d , b d e c d . Se d = x 2 + y 2 , com x, y , multiplicando os vrtices de
T por x + iy , obtemos um tringulo T cujos vrtices tm coordenadas inteiras e

cujos lados medem

a , b e c , como queramos.

Agradecemos o envio de solues e a colaborao de: Bruno Salgueiro Fanego (Viveiro, Espanha) Daniel Vacaru (Pitesti, Romnia) Marcelo Ribeiro Lucas Justo de Freitas Neto (Mossor, RN) Geraldo Perlino Jr. (Cotia, SP) Ricardo Klein Hoffman (Rio de Janeiro, RJ) Carlos Alberto da Silva Victor (Nilpolis, RJ) Osvaldo Mello Sponquiado (Olmpia, SP) Continuamos aguardando a soluo do problema 156. Prob. 151, 152, 153 Prob. 151 Prob. 151, 152 Prob. 149, 151 Prob. 151, 153 Prob. 151 Prob. 153 Prob. 152

EUREKA! N37, 2013

59

Sociedade Brasileira de Matemtica

PROBLEMAS PROPOSTOS
Convidamos o leitor a enviar solues dos problemas propostos e sugestes de novos problemas para prximos nmeros.

157) Sejam x e y inteiros positivos tais que x 2 1 divisvel por 2n y + 1 para todo inteiro positivo n. Prove que x = 1. 158) Ache todas as funes f :

tais que

f ( xy + 1) = f ( x + y ) + f ( x ) f ( y ) , x, y .

159) Dizemos que um conjunto A progressista se, sempre que x, y A com temos 2 y x A. Prove que se A progressista e x y, com e ento x, x + a1 , x + a2 ,..., x + ak A k2 0 < a1 < a2 < ... < ak
x + a1 + ak 3d , x + a1 + ak 2d A, onde d = mdc ( a1 , a2 ,..., ak ) .

160) Considere a sequncia definida por an = n 2003 para n 1. Prove que, para quaisquer inteiros positivos m e p, a sequncia contm m elementos em uma progresso geomtrica de razo maior que p.

161) Um sapo faz um caminho infinito no plano euclidiano da seguinte forma: no incio ele est no ponto (0, 0), e, se num dado momento est no ponto (x, y), no segundo seguinte salta para o ponto (x + 1, y) ou para o ponto (x, y +1). Prova que, para todo inteiro positivo n, existe uma reta l tal que o sapo passa por pelo menos n pontos de l em seu caminho. 162) Uma prova da IMO tem 6 problemas, e cada problema de cada participante recebe uma nota inteira n com 0 n 7. Dizemos que duas provas de dois participantes so comparveis se uma delas, digamos a do participante A menor ou igual prova do participante B, no seguinte sentido: em cada um dos 6 problemas a nota do participante A menor ou igual nota do participante B. Determine o menor inteiro positivo m tal que, se houver m participantes numa IMO, necessariamente haver duas provas comparveis.
O problema 157 foi proposto na shortlist da IMO de 2012, e o problema 158 uma generalizao de um problema proposto na shortlist da IMO de 2012. Ambos foram sugeridos para publicao nesta seo por Luis Farias Maia. O problema 159 uma generalizao de um problema proposto na IV Competio Ibero-Americana Interuniversitria de Matemtica (CIIM), realizada em 2012 em Guanajuato no Mxico. O problema 160 foi proposto no teste de seleo da IMO 2003. O problema 161 foi proposto originalmente por T. C. Brown para o American Mathematical Monthly, e foi sugerido para esta edio por Fabio Brochero.
EUREKA! N37, 2013

60

Sociedade Brasileira de Matemtica

AGENDA OLMPICA
35 OLIMPADA BRASILEIRA DE MATEMTICA NVEIS 1, 2 e 3 Primeira Fase sbado, 15 de junho de 2013 Segunda Fase sbado, 21 de setembro de 2013 Terceira Fase sbado, 19 de outubro de 2013 (nveis 1, 2 e 3) domingo, 20 de outubro de 2013 (nveis 2 e 3 - segundo dia de prova) NVEL UNIVERSITRIO Primeira Fase sbado, 21 de setembro de 2013 Segunda Fase sbado, 19 e domingo, 20 de outubro de 2013 6 ROMANIAN MASTER OF MATHEMATICS (RMM) 27 de fevereiro a 3 de maro de 2013 (Bucareste, Romnia) ASIAN PACIFIC MATH OLYMPIAD (APMO) 11 de maro de 2013 19 OLIMPADA DE MAIO 11 de maio de 2013 24 OLIMPADA DE MATEMTICA DO CONE SUL 1 a 7 de junho de 2013 (Assuno, Paraguai) 54 OLIMPADA INTERNACIONAL DE MATEMTICA 18 a 28 de julho de 2013 (Santa Marta, Colmbia) 3 OLIMPADA DE MATEMTICA DA COMUNIDADE DOS PASES DE LNGUA PORTUGUESA 5 a 10 de agosto de 2013 (Maputo, Moambique) 19 COMPETIO INTERNACIONAL DE MATEMTICA UNIVERSITRIA (IMC) 6 a 12 de agosto de 2013 (Blagoevgrad, Bulgria) 27 OLIMPADA IBERO-AMERICANA DE MATEMTICA 20 a 28 de setembro de 2013 (Cidade do Panam, Panam) 5 COMPETIO IBERO-AMERICANA INTERUNIVERSITRIA DE MATEMTICA 15 a 20 de outubro de 2013, (Armenia, Colmbia) 15 OLIMPADA IBERO-AMERICANA DE MATEMTICA UNIVERSITRIA 23 de novembro de 2013

EUREKA! N37, 2013

61

Sociedade Brasileira de Matemtica

COORDENADORES REGIONAIS
Alberto Hassen Raad Amrico Lpez Glvez Antonio Carlos Nogueira Carlos Alexandre Gomes da Silva Fabrcio Siqueira Benevides Carmen Vieira Mathias Claus Haetinger Cludio de Lima Vidal Denice Fontana Nisxota Menegais Dbora de Jess Bezerra Disney Douglas Lima de Oliveira Edson Roberto Abe Edney Aparecido Santulo Jr. Eduardo Leandro Emiliano Chagas Fabio Brochero Martnez Florncio Ferreira Guimares Filho Francinildo Nobre Ferreira Diego Marques Herivelto Martins Gisele Detomazi Almeida Gilson Tumelero Ivanilde Fernandes Saad Joo Bencio de Melo Neto Joo Francisco Melo Libonati Diogo Diniz Jos Luiz Rosas Pinho Jos Vieira Alves Jos William Costa Krerley Oliveira Licio Hernandes Bezerra Luciano G. Monteiro de Castro Luzinalva Miranda de Amorim Marcelo Antonio dos Santos Marcelo Rufino de Oliveira Mrcio Fialho Chaves Newman Simes Nivaldo Costa Muniz Uberlndio Batista Severo Raul Cintra de Negreiros Ribeiro Reinaldo Gen Ichiro Arakaki Ricardo Amorim Ronaldo Alves Garcia Rosangela Ramon Seme Gebara Neto Tadeu Ferreira Gomes Toms Menndez Rodrigues Valdenberg Arajo da Silva Wanderson Breder
EUREKA! N37, 2013

(UFJF) (USP) (UFU) (UFRN) (UFC) (UNIFRA) (UNIVATES) (UNESP) (UNIPAMPA) (Universidade Metodista de SP) (UFAM) (Colgio Objetivo de Campinas) (UEM) (UFPE) (Grupo Educacional Etapa) (UFMG) (UFES) (UFSJ) (UnB) (USP So Carlos) (UFT) (UTFPR) (UC. Dom Bosco) (UFPI) (Grupo Educacional Ideal) (UFPB) (UFSC) (UFPB) (Instituto Pueri Domus) (UFAL) (UFSC) (Sistema Elite de Ensino) (UFBA) FACOS (Grupo Educacional Ideal) (UFLA) (Cursinho CLQ Objetivo) (UFMA) (UFPB) (Colgio Anglo) (UNIFESP) (Centro Educacional Logos) (UFGO) (UNOCHAPEC) (UFMG) (UEBA) (U. Federal de Rondnia) (U. Federal de Sergipe) (CEFET RJ)

Juiz de Fora MG Ribeiro Preto SP Uberlndia MG Natal RN Fortaleza CE Santa Mara RS Lajeado RS S.J. do Rio Preto SP Bag RS S.B. do Campo SP Manaus AM Campinas SP Maring PR Recife PE So Paulo SP Belo Horizonte MG Vitria ES So Joo del Rei MG Braslia DF So Carlos SP Arraias TO Pato Branco PR Campo Grande MS Teresina PI Belm PA Campina Grande PB Florianpolis SC Campina Grande PB Santo Andr SP Macei AL Florianpolis SC Rio de Janeiro RJ Salvador BA Osrio RS Belm PA Lavras MG Piracicaba SP So Luis MA Joo Pessoa PB Atibaia SP SJ dos Campos SP Nova Iguau RJ Goinia GO Chapec SC Belo Horizonte MG Juazeiro BA Porto Velho RO So Cristvo SE Nova Friburgo RJ

62

You might also like